Download as pdf or txt
Download as pdf or txt
You are on page 1of 1411

SEPERAC RELEASED MEE ESSAY COMPILATION

MEE QUESTIONS AND ANSWERS FROM FEB 1995 TO JULY 2019


50 MEE EXAMS - 333 MEE QUESTIONS
DATE RELEASED: DECEMBER 2, 2019
CIVIL PROCEDURE: 46 OF 49 MEE EXAMS: (94%)
#001-JUL 2019–MEE Q06: QUESTION SIX (CIVIL PROCEDURE/CONLAW)

Trident Healthcare Inc., incorporated in State X, owns and operates hospitals and clinics in States X, Y,
and Z. Medical information for all of Trident’s current and former patients is stored on computer
equipment housed at Trident’s corporate headquarters in State X.

Last December, unknown persons hacked into Trident’s computer system and obtained the personal
medical information of at least 30,000 Trident patients, including 5,000 patients living in State X, 10,000
patients living in State Y, and 15,000 patients living in State Z. However, there is no evidence that the
thieves have used any of this medical information.

The State X Privacy Protection Act imposes an absolute duty on health-care providers, including
companies like Trident, to keep patient medical information private. The legislature concluded that the
“invasion of privacy” resulting from data breaches causes significant harm to the individuals involved.
Thus, the law allows any person whose private medical information is obtained by an unauthorized third
party in any manner to recover actual damages from the health-care provider. Further, because such
damages are sometimes difficult to quantify, the state law provides that an individual is entitled to a
minimum statutory (nominal) damages award of $500 to compensate for this “invasion of privacy.” This
state law is not preempted by any federal law.

A man, who is a citizen of State X and whose medical records were stored in the Trident computers, has
brought a class action in the federal district court of State X against Trident on behalf of himself and all
the persons whose health-care information was taken during the hacking of Trident’s computer system.
The man is represented by counsel with extensive experience in class actions of this type. The complaint
is limited to claims arising out of the hacking of medical information. It seeks no actual damages but does
seek statutory damages on behalf of all members of the class pursuant to the State X statute. The
complaint alleges the facts detailed above and alleges that the court has jurisdiction based on diversity,
pursuant to 28 U.S.C. § 1332. The complaint also alleges that most if not all of Trident’s patients are U.S.
citizens who are domiciled in the states where they receive their health care.

State X’s legislatively adopted Civil Practice Rules provide that “if any statute or law of this state allows
for an award of statutory or nominal damages, recovery of such damages may be sought in an individual
action but not in a class action.”

Trident has moved to dismiss the man’s class action brought in federal district court, arguing that (i) the
court lacks subject-matter jurisdiction over the state-law claim raised by the class action, (ii) the action
fails to allege a claim upon which relief can be granted because of the state law barring class actions to
recover statutory damages, and (iii) the man does not have standing to bring a statutory damages claim in
federal court.

With respect to each of these arguments, how should the court rule? Explain.

Seperac-J19 Exam-Released MEE Essay Compilation © 2016-2020 1


#001: J19-6 MEE: ANSWER: NCBE (CIVIL PROCEDURE/CONLAW)

POINT (1) [35%] ISSUE: Does a federal district court have diversity jurisdiction over a class action
when (a) the amount in controversy is $15,000,000 and (b) there is no diversity between the
defendant and the class representative, though there is diversity between the defendant and other
class members? ANSWER: Yes. Because the amount in controversy in this class action exceeds
$5,000,000 and there is minimal diversity, the federal district court has subject-matter jurisdiction
to hear the case under its diversity jurisdiction.

POINT (2) [35%] ISSUE: Does a state-law rule prohibiting class actions for recovery of statutory
damages apply to bar a class action brought in federal court when the state rule is contained in a
state statute governing court procedure and is not tied to any particular state-law substantive
right? ANSWER: No. When a claim based on state-law rights is brought in federal court, it is
governed by federal procedure. So federal procedural rules for class actions govern, and the federal
court is not bound by State X’s procedural law barring class actions to recover statutory damages.
Thus, the action should not be dismissed because of State X’s law.

POINT (3) [30%] ISSUE: Do the members of a plaintiff class have standing to bring a claim in
federal court when they cannot prove tangible damage to themselves but do have a claim for
statutory damages for violations of privacy rights protected by state law? ANSWER: Yes. The
court is likely to conclude that there is a “concrete” injury arising from the hacking of the medical
information, as required for there to be standing under Article III. Thus the court should probably
not grant the motion to dismiss for lack of the plaintiff’s standing.

ANSWER DISCUSSION:

Trident’s motion to dismiss for lack of subject-matter jurisdiction should be denied. A federal district
court can exercise diversity jurisdiction over a class action if the amount in controversy exceeds
$5,000,000 and any member of the class is diverse from any defendant (i.e., there is minimal diversity).
Here, those requirements are met. Each member of the class is alleged to be entitled to statutory damages
of $500 and there are at least 30,000 class members. If the class claims are upheld, total damages will be
at least $15,000,000, far in excess of the $5,000,000 required to be in controversy for this kind of action.
There is minimal diversity because the defendant is a citizen of State X, while some members of the class
live in States Y and Z. When a claim based on state-law rights is brought in federal court, it is governed
by federal procedure law. So federal procedural rules for class actions govern, not State X procedural
rules. The federal court is not bound by State X’s law barring class actions to recover statutory damages.
Thus, the action should not be dismissed on this basis. The plaintiff probably has standing to bring the
statutory (nominal) damages claim in federal court. The issue is whether the alleged “invasion of privacy”
suffered by the plaintiff is an “injury in fact” even though the plaintiff cannot prove any actual damages.
Given the state’s determination that data breaches involving private medical information cause harm to
individual privacy even when actual damages cannot be shown, it is likely that a federal court would
conclude that the invasion of privacy resulting from disclosure of private medical information is
sufficiently concrete to warrant a finding of standing.

ANSWER EXPLANATION:

Explanation to Point One (35%):

Seperac-J19 Exam-Released MEE Essay Compilation © 2016-2020 2


Because the amount in controversy in this class action exceeds $5,000,000 and there is minimal diversity,
the federal district court has subject-matter jurisdiction to hear the case under its diversity jurisdiction.

In 2005, Congress amended the federal diversity-jurisdiction statute, 28 U.S.C. § 1332, to allow large
class actions to be brought in federal court, even if there is not complete diversity between the defendants
and the plaintiffs. Under § 1332, a federal district court can exercise diversity jurisdiction over a class
action if: (1) “the matter in controversy exceeds the sum or value of $5,000,000;” and (2) “any member of
a class of plaintiffs is a citizen of a State different from any defendant” (i.e. “minimal diversity”).

Here, those requirements are met. First, the amount in controversy exceeds $5,000,000. The plaintiff is
suing on behalf of a class of at least 30,000 plaintiffs, each of whom has a statutory damages claim of
$500. These claims can be aggregated in determining jurisdiction. Thus, if the plaintiff’s class claim is
upheld, the amount in controversy exceeds $15,000,000.

Second, there is minimal diversity between some members of the plaintiff class and the defendant. Trident
is a citizen of State X, where it is incorporated and where its corporate headquarters are located. A
corporation is a citizen of its state of incorporation and the state where it has its principal place of
business. A corporation’s principal place of business is the state where the corporation’s high level
officers direct, control, and coordinate the corporation’s activities. All that is required, therefore, is that at
least one member of the plaintiff class be a citizen of a state other than State X.

Here, 10,000 members of the plaintiff class live in State Y, and 15,000 members live in State Z. The
complaint further alleges that most, if not all, of these class members are U.S. citizens. Assuming that
Trident does not contest these factual claims, there is almost certainly at least one member of the class
who is a citizen of either State Y or State Z and therefore diverse from Trident, a citizen of State X. A
U.S. citizen is a citizen of the state in which she has her domicile. Domicile is defined as “residence in
fact, combined with the intention of making the place of residence one’s home for an indefinite period.”
Among the 25,000 members of the class actually living in States Y or Z, there will doubtless be thousands
who meet the requirements of U.S. citizenship and domicile in those states, so that the minimal diversity
requirements are met. It does not matter that there are also members of the plaintiff class (i.e., those living
in State X) who are not diverse from the defendant.

[NOTE: According to the class-action provisions of the diversity-jurisdiction statute, a court may decline
(and sometimes must decline) to exercise jurisdiction when a class involves a significant proportion of
nondiverse class members. A court “may” decline when nondiverse members constitute greater than one-
third and less than two-thirds of the total class; and court “shall” decline when nondiverse members
constitute greater than two-thirds of the total class. Here, the number of State X class members (5,000) is
less than one-third of the total class (at least 30,000), and thus these special provisions do not apply. In
addition, the further exceptions to the class-action diversity rules (for class actions against state officials,
class actions with fewer than 100 members, and class actions involving securities or corporate fiduciary
claims) are not raised by the facts of this question.]

Explanation to Point Two (35%):

When a claim based on state-law rights is brought in federal court, it is governed by federal procedure. So
federal procedural rules for class actions govern, and the federal court is not bound by State X’s
procedural law barring class actions to recover statutory damages. Thus, the action should not be
dismissed because of State X’s law.

Seperac-J19 Exam-Released MEE Essay Compilation © 2016-2020 3


This action is appropriate if it is authorized by Federal Rule of Civil Procedure 23 regardless of what the
state procedural law says. In Erie Railroad Co. v. Tompkins, the Supreme Court held that federal courts
sitting in diversity must look to state law for the substantive rules of decisions governing such cases.
However, in a series of subsequent cases, the Court made clear that federal law would continue to govern
procedural matters in federal court, even if the federal procedure affected the outcome of the litigation.
For example, a federal rule providing for jury determination of a particular issue controls in federal court
despite a contrary state rule requiring the issue to be decided by a judge. If there is a federal procedural
rule that governs an issue, it applies “unless it exceeds statutory authorization or Congress’s rulemaking
power.”

In the Shady Grove case, the Court considered a New York law barring class actions to enforce statutory
damages claims. The Court concluded that such actions can be maintained in federal court, despite the
state law, if the action is authorized by Rule 23. According to the Court, Rule 23 is both a procedural rule
within the scope of the Rules Enabling Act and a rule that entitles “a plaintiff whose suit meets the
specified criteria to pursue his claim as a class action.” Thus, a class action may be maintained in federal
court even though there is a contrary state-law provision forbidding class actions.

[NOTE: To bring a class action under Rule 23, a plaintiff must first satisfy the requirements of Rule
23(a). Here, the requirements of Rule 23(a) (numerosity, commonality, typicality, and adequate
representation) are satisfied.]

[NOTE: The facts of this problem fall squarely within the holding of Shady Grove, so there should be no
question that Rule 23 should be applied, not the contrary law of State X. However, the Court was
fractured as to the rationale for its decision in Shady Grove, and Justice Stevens (a critical fifth vote)
suggested that Rule 23 would not apply if a state law limiting class actions is “so intertwined with a state
right or remedy that it functions to define the scope of the state-created right.” Thus, some examinees
might emphasize that the state law in this problem is a general law that does not directly modify rights
granted by any particular substantive law, including the State X Privacy Protection Act at issue in this
question.]

Explanation to Point Three (30%):

The court is likely to conclude that there is a “concrete” injury arising from the hacking of the medical
information, as required for there to be standing under Article III. Thus the court should probably not
grant the motion to dismiss for lack of the plaintiff’s standing.

Federal court jurisdiction is limited by Article III of the Constitution to “Cases” and “Controversies.” To
establish standing, a plaintiff must show (1) an injury in fact, (2) fairly traceable to the challenged conduct
of the defendant, and (3) likely to be redressed by a favorable judicial decision.

Here, the plaintiff’s claimed injury is the invasion of his privacy resulting from unauthorized access by
third-party hackers to his personal medical information. The facts indicate that his injury was a result of
the defendant’s failure to protect that information (as it was required to do by the State X Privacy
Protection Act). A favorable decision would redress the injury through an award of damages, but that
award would be based on the statute’s provision for statutory damages, not on proof of the actual extent of
harm to the plaintiff. Thus, the issue is whether the asserted “invasion of privacy” counts as an “injury in
fact” when the plaintiff does not claim that he, or any member of the class, suffered any actual damage
and seeks to recover only statutory damages.

Seperac-J19 Exam-Released MEE Essay Compilation © 2016-2020 4


In Spokeo, the Supreme Court concluded that “Article III standing requires a concrete injury even in the
context of a statutory violation.” In Spokeo, inaccurate information about a plaintiff was included in a
credit-reporting database, but there was no evidence that the plaintiff had been harmed by that fact. The
Supreme Court held that there was no “injury” for standing purposes, despite the fact that a state law
would have awarded the plaintiff statutory damages based on the presence of the inaccurate information
alone. The Court emphasized that the “injury in fact” element of the Court’s standing test required a
showing that the plaintiff suffered “an invasion of a legally protected interest” that is “concrete” as well as
“particularized.” The mere fact that a defendant has acted improperly toward a particular plaintiff (e.g., by
including inaccurate information about the plaintiff in a credit-reporting database) is not enough; nor is it
enough that the defendant’s conduct violated a state statute. For the plaintiff to have standing to bring a
claim in federal court, the defendant’s statutory violation must have “concretely” injured that plaintiff.

The Court in Spokeo also said that actual monetary harm to a plaintiff isn’t required for standing: an
“intangible” injury can nonetheless be a “concrete injury.” Moreover, the Court indicated that a legislative
judgment that certain intangible harms constitute injury in fact would be “instructive.” But the Court
made clear that a plaintiff does not satisfy the injury-in-fact requirement merely because a statute has
granted the plaintiff a statutory right and the defendant has allegedly violated that statute.

After Spokeo, the question is whether an invasion of a plaintiff’s privacy, without more, is a sufficiently
“concrete” injury to satisfy standing requirements. Although it can be argued that no harm is present on
our facts here because there is no evidence that the plaintiff’s improperly disclosed medical information
has been improperly used to the plaintiff’s detriment, the invasion of privacy alone may constitute
concrete harm, albeit ‘intangible’ harm, sufficient for standing. Knowledge that one’s private medical
information is in the hands of a third party may cause emotional distress or worry, even if the information
is never used. Support for a conclusion that such harms are concrete harms is also found in State X’s
legislative judgment that the loss of privacy that occurs when medical records are disclosed constitutes
injury warranting a recovery of damages even when no actual damages can be established.

In cases decided since Spokeo, several federal courts have held that an invasion of privacy is, in and of
itself, a concrete injury, even when extensive damages cannot be proved. For example, disclosure to a
third party of a user’s internet viewing history or private financial information is a violation of personal
privacy sufficient to constitute a “concrete injury”. Courts have also found it is an invasion of privacy
when a person receives autodialed and prerecorded messages, and even a solitary text message is
sufficient injury to establish standing.

Seperac-J19 Exam-Released MEE Essay Compilation © 2016-2020 5


#002-FEB 2019–MEE Q04: QUESTION FOUR (CIVIL PROCEDURE)

An airline is incorporated in State A, where its corporate headquarters are located. The facility where it
receives and processes online and telephone reservation requests is located in State B. It employs 150
people at that facility. The airline's base of physical operations, including its transport hub and major
maintenance facility, is in State C, where more than 12,000 of its 15,000 employees are located. The
airline serves States A and C but not State B.

In August, a woman who lived in State C called the reservation center in State B to obtain a round-trip
ticket for the woman to fly between State C and State A in early September.

In early September, the woman used the ticket to fly to State A. The purpose of her trip was to hunt for an
apartment in State A, where she was planning to start working at a new job that was set to begin in
December. The woman found an apartment and signed an agreement to rent the apartment for one year,
starting on December 1.

On the woman's return flight from State A to State C, a mechanical failure forced the plane to make an
emergency landing in State A. The woman suffered serious and permanent injuries during the emergency
landing and was hospitalized for three weeks in State A. Upon leaving the hospital, she returned to her
home in State C. Because of the injuries she suffered, the woman has been unable to work, and she has
received an indefinite deferral of the starting date for her job in State A. She continues to live in State C,
where she has lived her entire life, although she hopes one day soon to move to the apartment in State A
and begin working at her new job.

The woman has retained an attorney, who recommended filing a personal injury claim against the airline
in State B because of the larger awards that State B juries tend to give in such cases. Accordingly, the
woman sued the airline in federal court in State B, making a state-law tort claim for damages in excess of
$1 million for the injuries she suffered during the plane's emergency landing.

The airline promptly filed a motion to dismiss for lack of subject-matter and personal jurisdiction.

State B's long-arm statute allows its courts to exercise personal jurisdiction to "the maximum extent
allowed by the Fourteenth Amendment of the United States Constitution."

How should the federal district court rule on the motion to dismiss? Explain.

Seperac-J19 Exam-Released MEE Essay Compilation © 2016-2020 6


#002: F19-4 MEE: ANSWER: NCBE (CIVIL PROCEDURE)

POINT (1)(a) [10%] ISSUE: Does the woman's claim for $1 million in damages satisfy the amount-
in-controversy requirement for diversity jurisdiction? ANSWER: Yes. The court has diversity
jurisdiction only if the amount in controversy exceeds $75,000 and the parties are citizens of
different states.

POINT (1)(b) [20%] ISSUE: Is the woman a citizen of State C or State A for the purpose of
determining federal diversity jurisdiction? ANSWER: Here, the woman is a citizen of State C
because, despite her desire to move to State A, she has never established a residence in State A.

POINT (1)(c) [20%] ISSUE: Is the airline a citizen of State A, State B, or State C for the purpose of
determining federal diversity jurisdiction? ANSWER: The airline is a citizen of State A, where it is
incorporated and where, under the "nerve center" test of Hertz Corp. v. Friend, the airline has its
principal place of business.

POINT (2)(a) [25%] ISSUE: Are there sufficient contacts between the airline and State B for a
State B court to exercise specific personal jurisdiction over the airline in a personal injury action
brought by a State C resident, arising out of an accident in State A, and involving travel between
States A and C? ANSWER: No. The fact that the airline maintains an online reservation facility in
State B is not a sufficient basis for State B to exercise specific personal jurisdiction over a personal
injury action that arose out of an incident in another state, involved airline transportation between
two different states, and was brought by a plaintiff who is a resident of another state.

POINT (2)(b) [25%] ISSUE: Are there sufficient contacts between the airline and State B for a
State B court to exercise general personal jurisdiction over the airline in a personal injury action
brought by a State C resident, arising out of an accident in State A, and involving travel between
States A and C? ANSWER: No. The airline's operation of a reservation center in State B is not a
sufficient basis for a State B court to exercise general jurisdiction over the airline when the airline is
incorporated elsewhere, its principal place of business is elsewhere, and its operations in other
states are far more extensive than in State B.

ANSWER DISCUSSION:

The court should grant the motion to dismiss. Although the court would have diversity jurisdiction, the
court lacks personal jurisdiction over the airline. The woman's claim against the airline is based on state
law, so diversity is the only basis upon which a federal district court could exercise subject-matter
jurisdiction over this case. Here, the amount in controversy is more than $1 million and clearly sufficient
for diversity jurisdiction. Further, the parties are diverse. The woman's citizenship, determined by her
domicile, is State C, where she has lived her entire life. The airline is a citizen of State A, where it is
incorporated and where its corporate headquarters are located. The federal court in this case can exercise
personal jurisdiction over the airline only if the airline would be subject to the jurisdiction of the State B
state courts. Because the State B courts exercise jurisdiction to the limits of the Constitution, the question
is whether the airline has sufficient "minimum contacts" with State B "such that the maintenance of the
suit [in State B] does not offend traditional notions of fair play and substantial justice." Here, there are
insufficient contacts with State B for either general or specific jurisdiction. The only contact between the
airline and State B that is mentioned in the facts is that State B is the location of the airline's online and
telephone reservation center. The woman's personal injury action arises out of an accident that occurred in

Seperac-J19 Exam-Released MEE Essay Compilation © 2016-2020 7


State A and had no connection at all to State B. Although the woman booked her ticket through the
reservation center in State B, the woman's claim for bodily injury has no relation to the manner in which
she purchased her ticket or to the fact that the reservation center is in State B. On these facts, it does not
seem that the airline could reasonably anticipate being haled into a State B court for this particular claim.
Moreover, if this claim is allowed, then any personal injury claim against the airline could be litigated in
State B, regardless of where and how the injury occurred, if the related ticket was booked online or by
telephone. This seems unreasonable, without some additional evidence concerning the nature and extent
of the airline's State B contacts.

ANSWER EXPLANATION:

Explanation to Point One(a) (10%):

The court has diversity jurisdiction only if the amount in controversy exceeds $75,000 and the parties are
citizens of different states.

The diversity jurisdiction statute, 28 U.S.C. § 1332, grants original federal jurisdiction to the United
States District Courts in cases where the amount in controversy exceeds $75,000 and is between citizens
of different States. Diversity of citizenship is determined as of the time the suit is filed.

Here the woman's claim for relief seeks in excess of $1 million, a potentially credible amount since the
facts note that the woman sustained serious and permanent physical injuries that interfered with her ability
to act upon a job offer. In deciding a motion to dismiss on jurisdictional grounds, the court will take the
woman's claim for relief at face value with respect to the amount in controversy unless it appears to a
legal certainty that the claim is really for less than the jurisdictional amount.

Explanation to Point One(b) (20%):

Here, the woman is a citizen of State C because, despite her desire to move to State A, she has never
established a residence in State A.

The citizenship of individuals for diversity purposes is determined by domicile at the time of the lawsuit.
Domicile continues until changed, and in this case, the facts demonstrate that when the events began, the
woman, a lifelong resident of State C, was a State C domiciliary and a State C citizen for diversity
purposes. A person can change her domicile by: (1) taking up residence in a new jurisdiction with; (2) the
intent to remain there indefinitely. Here, the woman appears to have had an intention to move to State A
and make it her home indefinitely. She accepted a job in State A, and she signed a lease on an apartment
there.

However, the woman never took up residence in State A and therefore did not change her domicile. The
mere expression of an intention to move one's residence, without actually moving is not sufficient to
establish a domicile. The woman was temporarily present in State A and signed a one-year lease for an
apartment. She was also briefly hospitalized in State A. But her physical presence in State A never
coincided with a present intent to remain in State A indefinitely. At the time of her trip to State A, she
always intended to return to State C and to make her permanent move to State A at a later date. Thus, that
trip alone would be insufficient to change the woman's diversity citizenship from State C to State A.

[NOTE: An examinee who erroneously concludes that the September events did change the woman's
domicile to State A should consider whether subsequent events changed her domicile back to State C.
Thus, after her hospitalization, she returned to State C to live, with a desire to move to State A in the

Seperac-J19 Exam-Released MEE Essay Compilation © 2016-2020 8


future, but with no clear time at which that would happen. Thus, it would be reasonable to conclude that
the woman re-acquired her State C domicile because she is now present there with no immediate plans to
leave (i.e., she intends to remain there for an indefinite period of time).]

Explanation to Point One(c) (20%):

The airline is a citizen of State A, where it is incorporated and where, under the "nerve center" test of
Hertz Corp. v. Friend, the airline has its principal place of business.

The airline's citizenship for diversity purposes is determined by both the place of its incorporation and the
place where it has its principal place of business. Here, its place of incorporation is State A. Its principal
place of business is "the place where the corporation's high-level officers direct, control, and coordinate
the corporation's activities." The facts state that the airline's corporate headquarters are in State A, and
there is no suggestion that the officers run the corporation from any other location. The airline is,
therefore, a citizen of State A under the principal-place-of-business rule as well as the place-of-
incorporation rule.

Because the airline is a citizen of State A and the woman is a citizen of State C, the parties are diverse. As
noted earlier, the amount in controversy exceeds $75,000. Thus, the court has diversity jurisdiction.

Explanation to Point Two(a) (25%):

The fact that the airline maintains an online reservation facility in State B is not a sufficient basis for State
B to exercise specific personal jurisdiction over a personal injury action that arose out of an incident in
another state, involved airline transportation between two different states, and was brought by a plaintiff
who is a resident of another state.

A federal court can exercise personal jurisdiction over a defendant who is subject to the jurisdiction of the
courts of the state in which the federal court sits. In this case, State B's courts exercise jurisdiction to the
limits of the Constitution. The federal district court can therefore take personal jurisdiction over the airline
if the airline has sufficient "minimum contacts" with State B "such that the maintenance of the suit in
State B does not offend traditional notions of fair play and substantial justice." In this inquiry, two critical
questions are whether the quality and nature of the airline's contacts with the jurisdiction are such that it
can reasonably anticipate being haled into court there and whether those contacts are such that it is
reasonable to expect the airline to defend the action in State B. In Helicopteros Nacionales de Colombia,
the Supreme Court recognized a distinction between "general jurisdiction" (also called "all-purpose"
jurisdiction), which permits a court to exercise jurisdiction over any claim against a defendant if the
defendant has extensive connections with the forum, and "specific jurisdiction" (also called "case-linked"
jurisdiction), which permits a forum to hear a case only if the suit arises out of or relates to the defendant's
forum contacts.

In this case, there does not seem to be a basis for State B to exercise specific jurisdiction. For specific
jurisdiction to be proper, there must be "'an affiliation between the forum and the underlying controversy,'
principally, activity or an occurrence that takes place in the forum State and is therefore subject to the
State's regulation."

Here, the only connection between the airline and State B is that State B is the location of the airline's
reservation center. The woman's suit, however, is not directly related to the presence of the airline
reservation center in State B. Rather, the woman's claim against the airline arises from the accident in
State A and her use of the airline to fly between States A and C. The airline's affiliation with State B is not

Seperac-J19 Exam-Released MEE Essay Compilation © 2016-2020 9


sufficiently related to the woman's claim to require the airline to defend the action in State B on the basis
of specific jurisdiction.

Explanation to Point Two(b) (25%):

The airline's operation of a reservation center in State B is not a sufficient basis for a State B court to
exercise general jurisdiction over the airline when the airline is incorporated elsewhere, its principal place
of business is elsewhere, and its operations in other states are far more extensive than in State B.

General (all-purpose) jurisdiction is proper over a corporation only when the continuous corporate
operations within a state are so substantial and of such a nature as to justify suit on causes of action
arising from dealings entirely distinct from those activities. Although courts and commentators often have
described the standard for general jurisdiction as whether a corporation has engaged in a "substantial,
continuous and systematic course of business" in a state, the Supreme Court has recently described that
standard as "unacceptably grasping." The proper test is not whether the activities are "continuous and
systematic," it is whether the business's affiliations with the State are so continuous and systematic as to
render it essentially at home in the forum state. There are only a limited set of affiliations with a forum
that will justify concluding that the corporation is "essentially at home" there. The paradigmatic
affiliations are place of incorporation and principal place of business, each of which has the virtue of
being unique as well as easily ascertainable.

In this case, the maintenance of a reservation center in State B that employs 150 people is unlikely to be
enough to justify a conclusion that the airline is "essentially at home" there. The airline is incorporated
elsewhere, its headquarters are elsewhere, and the bulk of its physical operations (employing thousands of
people) occur elsewhere. The fact that it engages in substantial business activities in State B (as well as in
other states where it conducts part of its operations) is, by itself, not sufficient to satisfy the court's
requirement that it be "essentially at home" there, especially given the Court's admonition that only a
"limited set of affiliations" will suffice for a forum to assert all-purpose jurisdiction over a defendant. For
example, one court found that a drug company was not "at home" in California, even though it sold large
volumes of its products in California, maintained several research facilities in California, and employed
hundreds of people in California, because the company's forum operations were much less extensive than
its activities elsewhere in the United States, and the company was incorporated and had its principal place
of business elsewhere.

Because there is no personal jurisdiction, the court should grant the motion to dismiss.

[NOTE: Pursuant to 28 U.S.C. § 1631, the court has the authority to transfer the case to another court in
order to avoid the jurisdictional problem. Some examinees may mention this as an alternative to
dismissal.]

Seperac-J19 Exam-Released MEE Essay Compilation © 2016-2020 10


#003-FEB 2018–MEE Q05: QUESTION FIVE (CIVIL PROCEDURE)

While speeding down a rural highway in State A, the driver of a moving van lost control of the van and
struck a car. A passenger in the car was seriously injured.

The passenger filed suit in the federal district court for the district in State A where the accident had taken
place. She sought damages for her injuries from the driver of the van and the moving company that
employed him. Among other allegations, the complaint alleged that

• the driver and the moving company are citizens of State A;

• the driver resides in the federal judicial district where the suit was brought;

• the accident occurred in the federal judicial district where the suit was brought;

• the passenger is a citizen of State B;

• the amount in controversy exceeds $75,000;

• venue is proper in the federal judicial district where the suit was brought;

• the driver was employed by the moving company and was acting in the course of his employment
at the time of the accident;

• the driver of the moving van was negligent; and

• the passenger suffered serious injuries as a result of that negligence.

The defendant driver and the defendant moving company were both represented by an attorney who was a
partner in a 30-lawyer law firm. The attorney was retained and received a copy of the complaint only four
days before an answer was due. The attorney was conducting another trial at the time. Rather than ask
another lawyer in the firm to answer the complaint, the attorney personally prepared and filed a timely
answer to the complaint on behalf of the defendants.

The answer to the complaint, which was signed by the attorney, read simply: "General Denial: Defendants
Hereby Deny Each and Every Allegation in the Complaint."

Two months later, the plaintiff (the passenger) properly served Requests for Admission on the defendants,
requesting admission of each allegation in the complaint. Responding to the Requests for Admission, the
defendants (still represented by the attorney) denied the allegations concerning the driver's negligence and
the plaintiffs injuries, but admitted all other alleged facts.

The plaintiff then served on the defendants' attorney a motion for sanctions on the ground that the general
denial in the answer was inappropriate. The plaintiff requested that the defendants withdraw their original
answer and file an amended answer admitting the allegations that the defendants had admitted in their
response to the Requests for Admission.

One month later, after the defendants had failed to withdraw or amend their answer, the plaintiff filed the
motion for sanctions in court. The plaintiff's lawyer submitted evidence that his customary billing rate is

Seperac-J19 Exam-Released MEE Essay Compilation © 2016-2020 11


$300 per hour and that he had spent seven hours preparing the motion and corresponding with the
defendants' attorney about the answer, for a total of $2,100.

1. May the court properly grant the plaintiff's motion for sanctions? Explain.

2. If the court grants the plaintiff's motion for sanctions, (a) what sanctions are appropriate and (b)
against whom should the sanctions be ordered? Explain.

Seperac-J19 Exam-Released MEE Essay Compilation © 2016-2020 12


#003: F18-5 MEE: ANSWER: NCBE (CIVIL PROCEDURE)

POINT (1)(a) [35%] ISSUE: Does a party violate Rule 11 by filing an answer containing a general
denial, when there is no evidentiary support for the denial of some of the complaint’s allegations
and the attorney did not make any inquiry into the facts? ANSWER: Yes. The court should grant
the motion for sanctions because the defendants’ attorney violated Rule 11 by filing an answer that
contained improper denials of factual and legal contentions.

POINT (1)(b) [15%] ISSUE: What requirements must a party satisfy before filing a motion for
sanctions under Rule 11? ANSWER: The plaintiff’s lawyer properly gave the defendants’ attorney
an opportunity to correct the pleading before the plaintiff's lawyer sought Rule 11 sanctions.

POINT (2)(a) [30%] ISSUE: When an attorney files an answer on behalf of a client that contains a
general denial that violates Rule 11, what sanctions should be imposed? ANSWER: The court has
considerable discretion in imposing sanctions. The purpose of any sanctions award should be to
deter similar future conduct by this lawyer or others. The sanctions may include an award of
reasonable attorney’s fees that were incurred because of the violation.

POINT (2)(b) [20%] ISSUE: When an attorney files an answer on behalf of a client that contains a
general denial that violates Rule 11, on whom should the sanctions be imposed? ANSWER:
Sanctions may be imposed upon the defendants’ attorney, his law firm, and the defendants.
However, monetary sanctions for asserting an unwarranted legal contention may not be imposed
upon the defendants, but only upon the attorney or the law firm.

ANSWER DISCUSSION:

The filing of a general denial by the defendants’ attorney violated Federal Rule of Civil Procedure 11.
Rule 11 requires an attorney to conduct a reasonable inquiry before filing an answer that denies all of the
complaint’s allegations. Here it appears that the attorney conducted no inquiry into the facts and that
many of the factual allegations that were denied concerned facts that could easily have been checked by
the attorney before the answer was filed. In addition, the attorney violated Rule 11 by denying that venue
was proper because that denial was a “legal contention” that was not warranted by existing law. Finally,
the attorney failed to withdraw or correct the answer, even though the plaintiff properly gave the attorney
a 21-day “safe harbor” in which to correct the pleading. Thus, the court should find that the defendants’
answer violated Rule 11. The court has discretion not to impose any sanctions even if it finds a violation
of Rule 11. If the court does impose sanctions, it has discretion to choose among a range of monetary and
nonmonetary sanctions. If warranted for effective deterrence, the court could order payment to the
plaintiff of the reasonable attorney’s fees directly resulting from the Rule 11 violation. The sanctions
could be imposed on the defendants’ attorney, the law firm of which he is a partner, and the defendants
themselves. However, if the court finds that the defendants’ attorney violated Rule 11 by asserting a legal
contention unwarranted by existing law, any monetary sanctions attributable to that violation could be
imposed only on the attorney and not on the defendants. The facts do not indicate that the driver and the
moving company are responsible for, or even knew of, the attorney’s decision to file a general denial.
Thus, the sanctions, if any, likely would be imposed on the defendants’ attorney and his law firm, and not
on the defendants.

ANSWER EXPLANATION:

Seperac-J19 Exam-Released MEE Essay Compilation © 2016-2020 13


Explanation to Point-One(a) (35%):

The court should grant the motion for sanctions because the defendants’ attorney violated Rule 11 by
filing an answer that contained improper denials of factual and legal contentions.

Every pleading, written motion, or other paper (except for discovery requests and responses) must be
signed by an attorney, or by a party if the party is unrepresented. An attorney’s signature on a pleading
certifies that to the best of the person’s knowledge, information, and belief, formed after an inquiry
reasonable under the circumstances: (1) the pleading is not being presented for an improper purpose; (2)
the legal contentions are warranted by existing law or by a nonfrivolous argument for changing existing
law; (3) the factual contentions have evidentiary support or, if specifically so identified, will likely have
evidentiary support after a reasonable opportunity for further investigation or discovery; and (4) the
denials of factual contentions are warranted on the evidence or, if specifically so identified, are reasonably
based on belief or a lack of information.

Here, the driver and the moving company filed an answer, which is a “pleading” subject to Rule 11. The
facts state that the defendants’ attorney signed the answer, as required.

The defendants’ answer was a general denial, which is technically allowed under the Federal Rules of
Civil Procedure, but only if the party intends in good faith to deny all the allegations of a complaint—
including the jurisdictional grounds. A defendant’s answer must respond to legal conclusions stated in the
complaint (e.g. an assertion that venue is proper) as well as to factual allegations. Because there is almost
always something in the complaint that, in good faith, should be admitted, a general denial is likely to be
improper in most cases.

In this problem, the effect of the general denial was to deny all of the complaint’s allegations, including
accurate factual contentions that the driver and the moving company would be unable to deny in good
faith, such as their state of citizenship and the driver’s employment by the moving company at the time of
the accident. Because these denials were not “specifically identified” as being “reasonably based on belief
or a lack of information,” they were proper only if they were “warranted on the evidence.” However,
these denials weren’t warranted on the evidence, as the defendants’ later admissions made clear. In
addition, the denial of the propriety of venue was not warranted by the facts or by existing law. Venue is
appropriate in the federal district court in which a “substantial part of the events or omissions giving rise
to the claim occurred.” The accident occurred in the district in which the plaintiff filed suit. Even if the
defendants’ attorney did not know the venue rule, Rule 11 required him to investigate the statutory
provision before filing his answer. If he had done so, he would have seen that denying the propriety of
venue was unwarranted under the law.

A lawyer who files an inaccurate pleading is not subject to sanctions if the lawyer acted in good faith after
making a pre-filing inquiry that was reasonable under the totality of the circumstances. Reasonableness is
judged as of the time the pleading was submitted, and depends on “such factors as how much time for
investigation was available to the signer; whether he had to rely on a client for information as to the facts
underlying the pleading, motion, or other paper; whether the pleading, motion, or other paper was based
on a plausible view of the law; or whether he depended on forwarding counsel or another member of the
bar.”

Here, the defendants’ attorney had only a few days to investigate, and the attorney was in the midst of a
trial at the time. Nonetheless, the attorney or another member of the firm should have known, or could
have easily discovered, simple facts such as the state of citizenship of his clients, that the driver was
acting in the course of employment at the time of the accident, and that the driver resided in the district

Seperac-J19 Exam-Released MEE Essay Compilation © 2016-2020 14


where the action was filed. Even a brief conversation with the clients would have permitted verification of
those facts. The inference that the earlier denial of these factual contentions was not warranted on the
evidence is further supported by the defendants’ response to Requests for Admission, in which the
defendants admitted these same facts. On the basis of the facts given in this problem, it seems likely that
several of the original factual denials were unwarranted.

Explanation to Point-One(b) (15%):

The plaintiff’s lawyer properly gave the defendants’ attorney an opportunity to correct the pleading before
the plaintiff's lawyer sought Rule 11 sanctions.

Before a party may seek sanctions under Rule 11, the party must serve on the opposing party a motion
that describes the specific conduct that allegedly violated the rule. The opposing party must be given 21
days to withdraw or correct the challenged pleading. If the 21-day period passes, and the pleading is not
corrected, the motion for sanctions may be filed with the court.

Here, the plaintiff's lawyer followed the proper procedure for presenting a Rule 11 motion. First, the
motion requesting that the answer be withdrawn and amended to reflect the admissions made in the
response to the Requests for Admission was first served on the defendants’ attorney. The plaintiff's
lawyer then waited at least 21 days, the rule’s “safe harbor” period, to allow the defendants’ attorney to
withdraw or appropriately correct the answer. When the attorney failed to do so, the Rule 11 motion was
then filed in court. The motion set forth the specific conduct that allegedly violated Rule 11, insofar as the
plaintiff requested the defendants’ attorney to correct the answer by admitting the allegations that had
been admitted in the responses to the Requests for Admission.

Explanation to Point-Two(a) (30%):

The court has considerable discretion in imposing sanctions. The purpose of any sanctions award should
be to deter similar future conduct by this lawyer or others. The sanctions may include an award of
reasonable attorney’s fees that were incurred because of the violation.

The court will probably determine that the answer violated Rule 11. Nonetheless, the court is not
obligated to impose any sanctions. Whether a violation has occurred and what sanctions, if any, to impose
for a violation are matters committed to the discretion of the trial court.

Sanctions should be limited to what suffices to deter repetition of the conduct or comparable conduct by
others similarly situated. Sanctions can be monetary or nonmonetary. If the sanctions are monetary, they
can include an order to pay a penalty into court; or, if imposed on motion and warranted for effective
deterrence, an order directing payment to the movant of part or all of the reasonable attorney’s fees and
other expenses directly resulting from the violation. Examples of nonmonetary sanctions would be
striking the offending paper; issuing an admonition, reprimand, or censure; requiring participation in
seminars or other educational programs; or referring the matter to disciplinary authorities.

A non-exhaustive list of factors the court may consider in deciding what sanctions, if any, should be
imposed for a violation includes whether the improper conduct was willful, or negligent; whether it was
part of a pattern of activity, or an isolated event; whether it infected the entire pleading, or only one
particular count or defense; whether the person has engaged in similar conduct in other litigation; whether
it was intended to injure; what effect it had on the litigation process in time or expense; whether the
responsible person is trained in the law; what amount, given the financial resources of the responsible

Seperac-J19 Exam-Released MEE Essay Compilation © 2016-2020 15


person, is needed to deter that person from repetition in the same case; and what amount is needed to deter
similar activity by other litigants.

Here, the defendants admitted, in their response to the Requests for Admission, the very facts they had
earlier denied in their answer. This suggests that they were not willfully seeking to impose any
unnecessary burden on the plaintiff or to deny uncontested facts. Moreover, because only a few months
had gone by, the effect of the offending answer on the litigation process in time or expense was likely
minimal. On the other hand, if this particular lawyer or law firm had a history of failing to follow the
Federal Rules of Civil Procedure, or if improper general denials were a particular problem in this district,
then the court here would be justified in ordering sanctions consisting of the plaintiff’s reasonable”
attorney’s fees directly resulting from the violation. The court would evaluate the time records submitted
to ensure that the seven hours of work claimed directly resulted from the violation, but seven hours does
not, on its face, seem unreasonable. Depending on the geographic region of the country and the plaintiff’s
attorney’s seniority and level of skill, $300 per hour is also probably a reasonable amount. A sanctions
award that fully compensates the plaintiff for the costs of challenging the defendants’ attorney’s
misconduct can promote deterrence by helping ensure that misconduct does not go unchallenged in the
future.

Explanation to Point-Two(b) (20%):

Sanctions may be imposed upon the defendants’ attorney, his law firm, and the defendants. However,
monetary sanctions for asserting an unwarranted legal contention may not be imposed upon the
defendants, but only upon the attorney or the law firm.

The court may impose sanctions on any attorney, law firm, or party that violated the rule or is responsible
for the violation. Here, this would certainly include the driver’s and moving company’s attorney. In
addition, the attorney’s law firm should probably also be held jointly responsible, because there do not
seem to be any “exceptional circumstances” that would justify a departure from the general rule that a law
firm must be held jointly responsible for a violation committed by its partner.

There is authority for imposing sanctions on a party when the party authorizes its attorney to pursue a
claim that it knew, or should have known was legally and factually baseless. But there are no facts here
that indicate that the driver and the moving company knew the content of the answer or were responsible
for the attorney’s decision to make an unwarranted general denial.

If the plaintiff were to argue that the defendants should also be sanctioned because they contributed to the
attorney’s decision to file a general denial by failing to give the attorney a copy of the complaint until
very close to the date an answer was due, a court would almost certainly reject that argument.

Even if the defendants were negligent in failing to get the complaint to their attorney at an earlier time,
instead of filing a general denial the attorney could have requested an extension of time in which to
answer and used that time to conduct a reasonable inquiry into the facts and law. Furthermore, the
attorney could have avoided sanctions by filing an amended answer when requested by the plaintiff. There
are no facts indicating that the defendants were responsible for these failures by the attorney.

Finally, insofar as the general denial included an unwarranted legal contention (e.g. the denial of proper
venue), any monetary sanctions resulting from that particular violation could not be imposed upon the
driver and the moving company.

Seperac-J19 Exam-Released MEE Essay Compilation © 2016-2020 16


#004-JUL 2017–MEE Q06: QUESTION SIX (CIVIL PROCEDURE/CONFLICTS)

Taxes Inc. (“Taxes”) is a tax preparation business incorporated in State A, where it has its corporate
headquarters. Taxes operates five tax preparation offices in the “Two Towns” metropolitan area, which
straddles the border between State A and State B. Three of the Taxes tax preparation offices are located in
Salem, State A; the other two are in Plymouth, State B.

A woman, a recent college graduate, was hired by Taxes and trained to work as a tax preparer in one of its
offices in Salem, State A. The woman and Taxes entered into a written employment contract in State A
that included a noncompete covenant prohibiting her from working as a tax preparer in the Two Towns
metropolitan area for a period of 24 months after leaving Taxes’s employ. The employment contract also
provided that it was “governed by State A law.”

After working for Taxes for three years, the woman quit her job with Taxes, moved out of her parents’
home in State A (where she had been living since her college graduation), and moved into an apartment
she had rented in Plymouth, State B. Two weeks later, she opened a tax preparation business in Plymouth.

Taxes promptly filed suit against the woman in the federal district court for State A, properly invoking the
court’s diversity jurisdiction. The complaint alleged all the facts stated above, claimed that the woman
was preparing taxes in violation of the noncompete covenant in her employment contract, and sought an
injunction of 22 months’ duration against her continued preparation of tax returns for any paying
customers in the Two Towns metropolitan area.

Taxes delivered a copy of the summons and complaint to the home of the woman’s parents in State A (the
address that she had listed as her home address when she was employed by Taxes). The process server left
the materials with the woman’s father.

Each state has service-of-process rules identical to those in the Federal Rules of Civil Procedure.

Under State A law, covenants not to compete are valid so long as they are reasonable in terms of
geographic scope and duration. The State A Supreme Court has previously upheld noncompete covenants
identical to the covenant at issue in this case. When determining whether to give effect to a contractual
choice-of-law clause, State A follows the Restatement (Second) of Conflict of Laws.

Under State B law, covenants not to compete are also valid if they are reasonable in scope and duration.
However, the State B Supreme Court has held that noncompete covenants are unreasonable and
unenforceable as a matter of law if they exceed 18 months in duration. While State B generally gives
effect to choice-of-law clauses in contracts, it has a statute that provides that choice-of-law clauses in
employment contracts are unenforceable. When there is no effective choice-of-law clause, State B follows
the lex loci contractus approach to choice of law in contract matters.

Rather than file an answer to Taxes’s complaint, the woman filed a motion pursuant to Rule 12(b)(6) to
dismiss the action for failure to state a claim upon which relief can be granted. The woman’s motion
argued that the noncompete covenant is invalid and unenforceable as a matter of law. Two days after
filing the motion to dismiss, and before Taxes had responded to the motion, the woman filed an “amended
motion to dismiss.” The amended motion sought dismissal on the same basis as the original motion
(failure to state a claim), but also asked the court to dismiss the action for insufficient service of process.

Seperac-J19 Exam-Released MEE Essay Compilation © 2016-2020 17


1. Should the court consider the woman’s motion to dismiss for insufficient service of process?
Explain.

2. If the court considers the woman’s motion to dismiss for insufficient service of process, should it
grant that motion? Explain.

3. In ruling on the woman’s motion to dismiss for failure to state a claim, which state’s choice- of-
law approach should the court follow? Explain.

4. Which state law should the court apply to determine the enforceability of the noncompete
covenant? Explain.

Seperac-J19 Exam-Released MEE Essay Compilation © 2016-2020 18


#004: J17-6 MEE: ANSWER: NCBE (CIVIL PROCEDURE/CONFLICTS)

POINT (1) [20%] ISSUE: Can a motion to dismiss be amended, prior to a responsive filing being
made, to add a ground for dismissal that would otherwise be waived because it was not raised in the
initial motion to dismiss? ANSWER: Yes. The court should consider the woman’s insufficiency-of-
service motion. Although she did not raise her defense of insufficient service of process in her
original motion to dismiss, she avoided waiving the defense by promptly amending her motion to
dismiss.

POINT (2) [30%] ISSUE: Is service of a summons and complaint sufficient when the documents are
not served personally on the defendant, but instead are given to the defendant’s parent at the
defendant’s parents’ home where the defendant previously lived? ANSWER: No. The court should
grant the woman’s motion and dismiss the action for insufficient service of process, because she did
not live at her parents’ house at the time the summons and complaint were delivered to her father.

POINT (3) [20%] ISSUE: What choice-of-law approach should be followed by a federal court
exercising its diversity jurisdiction? ANSWER: A federal court sitting in diversity must apply the
choice-of-law rules of the state where the federal court sits. Thus, the court here should apply State
A’s choice-of-law rules.

POINT (4) [30%] ISSUE: Under the Restatement (Second) approach to choice of law, should a
court honor a contractual choice-of-law clause when the parties have chosen the law of a state that
is connected to their transaction but that law is different from the law of another state with an
interest in the matter? ANSWER: Yes. Under the Restatement (Second), the court should honor
the parties’ choice of State A law because (a) State A has a close connection to the parties and the
transaction, and (b) State A’s law upholding the noncompete covenant is not contrary to the
fundamental policy of State B.

ANSWER DISCUSSION:

Although normally the motion to dismiss for insufficient service of process would be waived because it
was not included in the original motion, the court should consider the woman’s amended motion in this
case. Where, as here, the opposing party has not briefed the motion and the case will not be unduly
delayed by an amendment, courts generally allow an amendment of a motion to dismiss, even when the
added grounds would be waived if not raised in a defendant’s initial motion to dismiss. The court should
dismiss the action for insufficient service of process. Serving process at a defendant’s parents’ home is
not sufficient service when the defendant no longer lives there. Whether the court should dismiss Taxes’s
claim that the woman violated the noncompete covenant of her employment contract is a matter of law,
which depends on whether State A or State B law applies to the claim. If State A law applies, the
covenant is valid and the motion should be denied; if State B law applies, the covenant is invalid and the
motion should be granted. Which law applies depends, in turn, on whether the court will honor the
choice-of-law clause in the parties’ contract. Choice-of-law rules are substantive rules for diversity
purposes. A federal court sitting in diversity must apply the choice-of-law rules of the state where it sits.
Here, the federal court should apply the Restatement (Second) of Conflict of Laws approach, which State
A follows, in determining whether to enforce the contractual choice-of-law clause. Applying the
Restatement (Second), the court should honor the parties’ contractual choice-of-law clause, which said
that their agreement would be governed by State A law. State A has a close connection to both the
transaction and the parties, and its rules on noncompete covenants are not contrary to a fundamental

Seperac-J19 Exam-Released MEE Essay Compilation © 2016-2020 19


public policy of State B. Thus, because the parties’ noncompete covenant is valid under State A law, the
court should deny the woman’s motion to dismiss for failure to state a claim.

ANSWER EXPLANATION:

Explanation to Point-One (20%):

The court should consider the woman’s insufficiency-of-service motion. Although she did not raise her
defense of insufficient service of process in her original motion to dismiss, she avoided waiving the
defense by promptly amending her motion to dismiss.

Generally, when a party makes a pre-answer motion under Federal Rule of Civil Procedure 12, the party
must raise any claim of insufficiency of service of process that the party has at the time of the motion;
otherwise, the defense is waived. Rule 12 states that “a party that makes a motion under this rule must not
make another motion under this rule raising a defense or objection that was available to the party but
omitted from its earlier motion.” Further, “a party waives any defense listed in rule 12(b)(2)–(5) by
omitting it from a motion in the circumstances described in Rule 12(g)(2).” Taken together, these rules
would normally mean that the woman waived her Rule 12(b)(5) defense of insufficiency of service of
process when she failed to include it in her original motion to dismiss.

Nonetheless, courts have allowed a motion to dismiss to be amended before the motion is heard, so long
as the adverse party is not prejudiced by the amendment and no delay results in the prosecution and
determination of the case. Thus, although not expressly provided for in Federal Rule 12(g), a preliminary
motion may be amended to include a defense or objection inadvertently omitted by the movant.

Here, the woman amended her motion to dismiss only two days after making the motion. Taxes had not
yet responded to the motion and is not likely to have been prejudiced by the woman’s failure to raise her
insufficiency-of-service defense two days earlier. There is no indication that the proceedings will be
delayed if her amended motion is treated as adequate to raise the defense. Accordingly, the woman’s
insufficiency-of-service defense was not waived by her failure to include it in the original motion.

Explanation to Point-Two (30%):

The court should grant the woman’s motion and dismiss the action for insufficient service of process,
because she did not live at her parents’ house at the time the summons and complaint were delivered to
her father.

Federal Rule of Civil Procedure 4(e) allows service by delivering a summons and complaint to an
individual personally or by “leaving a copy of each at the individual’s dwelling or usual place of abode
with someone of suitable age and discretion who resides there.” Here, Taxes sought to serve process on
the woman by leaving a copy of the summons and complaint at her parents’ home with her father.
Although the woman had once lived with her parents, she had moved and was living in her own apartment
in another state when process was served. Thus, service was not made at the woman’s current dwelling or
usual place of abode, and leaving the summons and complaint at her parental home was not sufficient.
Service is not proper where the defendant no longer resides at the address to which the summons and
complaint are delivered. For example, leaving a summons and complaint with parents is not sufficient
service when the defendant “has left home and established residence elsewhere. Thus, the court should
grant the woman’s motion to dismiss for insufficient service of process.

Explanation to Point-Three (20%):

Seperac-J19 Exam-Released MEE Essay Compilation © 2016-2020 20


A federal court sitting in diversity must apply the choice-of-law rules of the state where the federal court
sits. Thus, the court here should apply State A’s choice-of-law rules.

A federal court sitting in diversity must apply the substantive law of the state in which it sits, including
that state’s choice-of-law rules. In this case, the court’s determination whether to apply State A or State B
law to the parties’ noncompete covenant must be made by applying the choice-of-law rules of State A, the
state where the court is sitting. Thus, the court must apply the standards set out in the Restatement
(Second) of Conflict of Laws because these are the standards that would be applied by a State A court.

Explanation to Point-Four (30%):

Under the Restatement (Second), the court should honor the parties’ choice of State A law because (a)
State A has a close connection to the parties and the transaction, and (b) State A’s law upholding the
noncompete covenant is not contrary to the fundamental policy of State B.

The parties’ employment contract specified that it would be governed by State A law. The Restatement
(Second) of Conflict of Laws generally favors the enforcement of such choice-of- law clauses. Even
when, as here, the legal issue is one that the parties could not have resolved on their own (the validity of a
clause of their contract), the Restatement provides that the law chosen by the parties will be applied unless

(a) the chosen state has no substantial relationship to the parties or the transaction and there is no
other reasonable basis for the parties’ choice, or

(b) application of the law of the chosen state would be contrary to a fundamental policy of a state
which has a materially greater interest than the chosen state in the determination of the particular
issue and which, under the rule of § 188 a “most significant relationship” test, would be the state of
the applicable law in the absence of an effective choice of law by the parties.

Here, the parties chose the law of State A, which has a substantial relationship to the parties and the
transaction: both Taxes and the woman were located in State A when they signed their contract, and the
contract concerned the terms of the woman’s employment in State A. Accordingly, the issue is whether
the parties’ chosen law should be disregarded pursuant to Restatement (Second) § 187.

Although State B’s law would not enforce this particular noncompete covenant because its duration is
more than 18 months, State B’s policy does not appear to reflect a fundamental public policy against
noncompete clauses in general. In fact, State B, like State A, generally enforces reasonable noncompete
covenants, and this covenant would be considered unreasonable under State B law only because it is six
months longer than State B courts allow. There are no facts to suggest that enforcing a noncompete clause
longer than 18 months would violate a fundamental policy of State B. Accordingly, the parties’ choice of
State A law should be honored, and the noncompete covenant should be enforced. Thus, if the court rules
on the woman’s motion to dismiss for failure to state a claim, the court should deny the motion.

[NOTE: If an examinee erroneously concludes that a federal court should follow State B’s choice-of-law
rules, then the examinee should conclude that the choice-of-law clause in the contract is unenforceable
because expressly forbidden by a State B statute. As to the validity of the noncompete clause, State A law
would apply under State B’s lex loci contractus rule because State A was the place where the employment
contract was made and was to be performed. If the court concluded that the noncompete covenant
violated a fundamental public policy of State B, it might conclude that State B would refuse to enforce the
covenant despite the fact that it was made in State A.]

Seperac-J19 Exam-Released MEE Essay Compilation © 2016-2020 21


#005-JUL 2016–MEE Q06: QUESTION SIX (CIVIL PROCEDURE)

A woman and a man have both lived their entire lives in State A. The man once went to a gun show in
State B where he bought a gun. Otherwise, neither the woman nor the man had ever left State A until the
following events occurred.

The woman and the man went hunting for wild turkey at a State A game preserve. The man was carrying
the gun he had purchased in State B. The man had permanently disabled the gun’s safety features to be
able to react more quickly to a turkey sighting. The man dropped the gun and it accidentally fired,
inflicting a serious chest wound on the woman. The woman was immediately flown to a hospital in
neighboring State C, where she underwent surgery.

One week after the shooting accident, the man traveled to State C for business and took the opportunity to
visit the woman in the hospital. During the visit, the woman’s attorney handed the man the summons and
complaint in a suit the woman had initiated against the man in the United States District Court for the
District of State C. Two days later, the woman was released from the hospital and returned home to State
A where she spent weeks recovering.

The woman’s complaint alleges separate claims against the man: 1) a state-law negligence claim and 2) a
federal claim under the Federal Gun Safety Act (Safety Act). The Safety Act provides a cause of action
for individuals harmed by gun owners who alter the safety features of a gun that has traveled in interstate
commerce. The Safety Act caps damages at $100,000 per incident, but does not preempt state causes of
action. The woman’s complaint seeks damages of $100,000 on the Safety Act claim and $120,000 on the
state-law negligence claim. Both sets of damages are sought as compensation for the physical suffering
the woman experienced and the medical costs the woman incurred as a result of the shooting.

The man has moved to dismiss the complaint, asserting (a) lack of personal jurisdiction, (b) lack of
subject-matter jurisdiction, and (c) improper venue. State C’s jurisdictional statutes provide that state
courts may exercise personal jurisdiction “to the limits allowed by the United States Constitution.”

With respect to each asserted basis for dismissal, should the man’s motion to dismiss be granted? Explain.

Seperac-J19 Exam-Released MEE Essay Compilation © 2016-2020 22


#005: J16-6 MEE: ANSWER: NCBE (CIVIL PROCEDURE)

POINT (1) [25%] ISSUE: May the U.S. District Court for the District of State C exercise personal
jurisdiction based on personal service on the man while he was temporarily in State C? ANSWER:
Yes. Personal service of process on the man while he was voluntarily present in State C is sufficient
to warrant the U.S. District Court’s exercise of personal jurisdiction over the man.

POINT (2)(a) [15%] ISSUE: Does the U.S. District Court for the District of State C have subject-
matter jurisdiction over the woman’s Safety Act claim? ANSWER: Yes. The U.S. District Court
has federal-question jurisdiction over the woman’s Safety Act claim.

POINT (2)(b) [35%] ISSUE: Does the U.S. District Court for the District of State C have subject-
matter jurisdiction over the woman’s state-law negligence claim? ANSWER: No. The U.S. District
Court does not have diversity jurisdiction over the state-law negligence claim, but it may
nonetheless hear the claim pursuant to its supplemental jurisdiction.

POINT (3) [25%] ISSUE: Is venue proper in the U.S. District Court for the District of State C?
ANSWER: No. Venue is not appropriate in the U.S. District Court for the District of State C
because the man does not reside in State C, the events giving rise to the woman’s claims did not
occur there, and there is another district (in State A) where venue would have been proper.

ANSWER DISCUSSION:

Personal service of process on an individual who is voluntarily physically present in a state is sufficient to
give the courts of the state (including federal courts located in the state) jurisdiction over that individual.
In this case, there are no facts that would warrant a departure from the ordinary rule allowing a court to
exercise such “transient jurisdiction.” Accordingly, the court has personal jurisdiction over the man. The
U.S. District Court for the District of State C possesses federal-question jurisdiction over the woman’s
Safety Act claim and supplemental jurisdiction over the woman’s negligence claim. However, the District
of State C is not an appropriate venue for the woman’s suit. The man does not reside there, and the events
giving rise to the woman’s claims occurred in State A. The fact that the woman received medical
treatment and may have experienced pain and suffering in State C is not a sufficient basis for venue when
the acts causing injury occurred elsewhere. The man’s motion to dismiss for improper venue should be
granted.

ANSWER EXPLANATION:

Explanation to Point-One (25%):

Personal service of process on the man while he was voluntarily present in State C is sufficient to warrant
the U.S. District Court’s exercise of personal jurisdiction over the man.

FRCP Rule 4 provides that a federal court can take personal jurisdiction over a defendant “who is subject
to the jurisdiction of a court of general jurisdiction in the state where the district court is located.” Thus,
the U.S. District Court’s jurisdiction over the man in this action depends on whether the state courts of
State C would take jurisdiction over the man under the circumstances of the case. The facts of the
problem indicate that State C courts generally will take jurisdiction over a defendant whenever the U.S.
Constitution permits them to do so.

Seperac-J19 Exam-Released MEE Essay Compilation © 2016-2020 23


Here, the man was personally served with process while he was voluntarily present in State C. In
Burnham v. Superior Court of California, a unanimous Supreme Court held that a court’s exercise of such
“transient jurisdiction,” i.e., jurisdiction based on physical presence alone, is generally consistent with due
process. A plurality concluded that transient jurisdiction is constitutional simply “because it is one of the
continuing traditions of our legal system that define the due process standard.” Although five members of
the Court believed that tradition alone did not warrant upholding the constitutionality of transient
jurisdiction, they did all agree at least that “the Due Process Clause of the Fourteenth Amendment
generally permits a state court to exercise jurisdiction over a defendant if he is served with process while
voluntarily present in the forum State.”

Here, as in Burnham, the man was physically present in State C when he was served with process, and his
presence in State C was voluntary. Furthermore, given that transient jurisdiction has long been the rule in
American courts, the man was on notice that his presence within State C could, upon service, subject him
to personal jurisdiction. Moreover, by traveling to State C, the man availed himself of various benefits
provided by State C, such as the benefits of “police, fire, and emergency medical” protection; access to
the state’s transportation system; and “fruits of the State’s economy.” At the same time, the burden on the
man of defending the suit in neighboring State C is insubstantial in light of modern communication and
transportation and the man’s demonstrated ability to travel to State C.

Thus, the man’s motion to dismiss for lack of personal jurisdiction should be denied.

[NOTE: Although no facts suggest that the Federal Gun Safety Act has special rules related to service of
process and personal jurisdiction, some examinees may mention that possibility.]

Explanation to Point-Two(a) (15%):

The U.S. District Court has federal-question jurisdiction over the woman’s Safety Act claim.

Under 28 U.S.C. § 1331, district courts may exercise subject-matter jurisdiction over “all civil actions
arising under the Constitution, laws, or treaties of the United States.” As a general rule, causes of action
that are created by federal law qualify for federal-question jurisdiction while those created by state law do
not. In this case, the allegations of the woman’s well-pleaded complaint state a Safety Act claim that is
created by federal law. As a result, federal-question jurisdiction exists over the woman’s Safety Act claim.

Explanation to Point-Two(b) (35%):

The U.S. District Court does not have diversity jurisdiction over the state-law negligence claim, but it
may nonetheless hear the claim pursuant to its supplemental jurisdiction.

As noted above, the U.S. District Court for the District of State C does not have federal-question
jurisdiction over the woman’s state-law negligence claim. The court also lacks diversity jurisdiction over
that claim because the woman and the man are not citizens of different states. State citizenship for
individual U.S. citizens is determined by their domicile: the true, fixed, permanent home to which the
individual intends to return when absent. The facts make clear that the woman and the man are both
domiciled in (and therefore citizens of ) State A, where they have lived their entire lives, and where they
both currently live. While the woman was convalescing in a hospital in State C when the suit was
brought, the woman exhibited no intent to change domicile, and she returned to State A upon her release
from the hospital. Because both parties are State A citizens, the fundamental requirement for diversity
jurisdiction is not met.

Seperac-J19 Exam-Released MEE Essay Compilation © 2016-2020 24


However, under 28 U.S.C. § 1367, a district court may exercise supplemental jurisdiction over claims that
form part of the same “case or controversy under Article III” as claims over which the district court has
original jurisdiction. It is generally understood that claims are part of the same case or controversy if they
arise from a common nucleus of operative facts.

Here, the woman’s Safety Act and negligence claims arise from a common nucleus of operative facts: the
man’s disabling of his gun’s safety features and the resulting accidental shooting of the woman. As a
result, the U.S. District Court possesses supplemental jurisdiction over the woman’s state-law negligence
claim.

Although a court has power to exercise supplemental jurisdiction, it need not do so in all cases. Under §
1367(c), a district court may decline to exercise supplemental jurisdiction under certain conditions,
including when “the claim over which the court has only supplemental jurisdiction substantially
predominates over the claim over which the district court has original jurisdiction.

Here, one might argue that the woman’s negligence claim substantially predominates over the federal-
question claim because the damages sought on the woman’s negligence claim are larger than the damages
sought on the woman’s federal claim. Nonetheless, the U.S. District Court is unlikely to dismiss the
negligence claim given the factual and evidentiary overlap in the two claims and the resulting efficiency
of resolving them together.

In short, the U.S. District Court has federal-question jurisdiction over the woman’s Safety Act claim and
possesses supplemental jurisdiction over the woman’s state-law negligence claim. As a result, the man’s
motion to dismiss for lack of subject-matter jurisdiction should be denied.

Explanation to Point-Three (25%):

Venue is not appropriate in the U.S. District Court for the District of State C because the man does not
reside in State C, the events giving rise to the woman’s claims did not occur there, and there is another
district (in State A) where venue would have been proper.

28 U.S.C. § 1391 governs venue in this action. Under § 1391(b), venue is appropriate “in (1) a judicial
district in which any defendant resides, if all defendants are residents of the State in which the district is
located, (2) a judicial district in which a substantial part of the events or omissions giving rise to the claim
occurred, or (3) if there is no district in which an action may otherwise be brought , any judicial district in
which any defendant is subject to the court’s personal jurisdiction with respect to such action.”

The man does not reside in the district of State C, so venue cannot be based on § 1391(b)(1).

State C also does not qualify as an appropriate venue pursuant to § 1391(b)(2) because it is not “a judicial
district in which a substantial part of the events or omissions giving rise to the claim occurred.” Here, the
events giving rise to the woman’s claims were the disabling of the gun’s safety features and the accidental
shooting – both of which occurred in State A. One might argue that a substantial part of the events giving
rise to the claim occurred in State C because the woman received medical treatment there and, as a result,
it was in State C that the woman experienced much of the harm (physical suffering and medical expenses)
for which she seeks to recover damages. However, most courts that have considered this statutory
language in the context of personal injury torts have concluded that venue is proper in the district where
the defendant’s tortious acts occurred, but not in a district that is connected to the incident only because
the plaintiff received medical treatment there.

Seperac-J19 Exam-Released MEE Essay Compilation © 2016-2020 25


Nor would venue be appropriate in State C on the § 1391(b)(3) grounds that the man was found there at
the time of service. Although the man was in State C at the time of service, § 1391(b)(3) is a backup
provision that applies only when no other district qualifies as an appropriate venue. Here, the man resides
in State A and the acts giving rise to the cause of action occurred there. Accordingly, a relevant district in
that state would qualify as an appropriate venue under either § 1391(b)(1) or § 1391(b)(2). So §
1391(b)(3) is inapplicable.

Thus, the man’s motion to dismiss for improper venue should be granted.

Seperac-J19 Exam-Released MEE Essay Compilation © 2016-2020 26


#006-JUL 2015–MEE Q02: QUESTION TWO (CIVIL PROCEDURE)

A woman attended a corporation’s sales presentation in State A. At this presentation, the corporation’s
salespeople spoke to prospective buyers about purchasing so-called “super solar panels,” rooftop solar
panels that the corporation’s salespeople said were 100 times as efficient as traditional solar panels. The
salespeople distributed brochures that purported to show that the solar panels had performed successfully
in multiple rigorous tests. The brochures had been prepared by an independent engineer pursuant to a
consulting contract with the corporation.

Based on what she was told at this presentation and the brochure she received, the woman decided to
purchase solar panels from the corporation for $20,000. The corporation shipped the panels to the woman
from its manufacturing facility in State B. The woman had the panels installed on the roof of her house in
State A. The panels failed to work as promised, even though they were properly installed.

A federal statute prohibits “material misstatements or omissions of fact in connection with the sale or
purchase of solar panels” and provides an exclusive civil remedy for individuals harmed by such
statements. This remedy preempts all state-law claims that would otherwise apply to this purchase.

Relying on this federal statute, the woman has sued the corporation and the independent engineer in the
U.S. District Court for the district of State A. She alleges that the statements made by the engineer in the
brochure and the statements made by the corporation’s salespeople at the presentation were false and
misleading with respect to the solar panels’ performance and value. She seeks damages of $30,000 (the
cost of the solar panels plus the expense of installing them).

The woman is a State A resident. The corporation is incorporated in State B and has its principal place of
business in State B. The engineer, who has never been in State A, is a State B resident with his principal
place of business in State B. He prepared the brochures in State B and delivered them to the corporation
there. He knew that the brochures would be distributed to prospective buyers at sales presentations around
the country.

The federal statute has no provision on personal jurisdiction. State A’s long-arm statute has been
interpreted to extend personal jurisdiction as far as the U.S. Constitution allows.

The engineer has timely moved to dismiss the complaint against him for lack of subject-matter and
personal jurisdiction. The engineer has also filed an answer (subject to his motion to dismiss) denying the
claims against him and asserting a cross-claim against the corporation. The engineer’s cross-claim alleges
that the corporation must indemnify the engineer for any damages he may have to pay the woman. The
indemnity claim is based on the terms of the consulting contract between the corporation and the
engineer.

The corporation has filed timely motions to dismiss the woman’s complaint for lack of subject-matter and
personal jurisdiction and to dismiss the engineer’s cross-claim for lack of subject-matter jurisdiction.

1(a). Does the State A federal district court have personal jurisdiction over the corporation?
Explain.

1(b). Does the State A federal district court have personal jurisdiction over the engineer? Explain.

Seperac-J19 Exam-Released MEE Essay Compilation © 2016-2020 27


2(a). Assuming that there is personal jurisdiction over both defendants, does the State A federal
district court have subject-matter jurisdiction over the woman’s claim against the corporation and
the engineer? Explain.

2(b). Assuming that there is personal jurisdiction over both defendants, does the State A federal
district court have subject-matter jurisdiction over the engineer’s cross-claim against the
corporation? Explain.

Seperac-J19 Exam-Released MEE Essay Compilation © 2016-2020 28


#006: J15-2 MEE: ANSWER: NCBE (CIVIL PROCEDURE)

POINT (1)(a) [25%] ISSUE: Does a court have personal jurisdiction to hear a claim against a
nonresident corporation when the corporation engaged in significant activity in the state related to
the transaction that is the subject matter of the plaintiff’s cause of action? ANSWER: Yes. The
corporation has sufficient connection with State A to permit a court to exercise jurisdiction over it
with respect to a claim arising out of those contacts.

POINT (1)(b) [15%] ISSUE: Does a court have personal jurisdiction to hear a claim against a
nonresident individual when the individual produced a brochure for a business, where the brochure
is related to the transaction that is the subject matter of the plaintiff’s cause of action, and the
individual had knowledge that the business would circulate the brochure to consumers in other
states in an effort to influence their purchasing decisions? ANSWER: No. The engineer is probably
not subject to jurisdiction in State A, given that his activities were confined to State B and he did
not purposefully avail himself of the benefits or protections of State A law.

POINT (2)(a) [20%] ISSUE: Does a federal district court have subject-matter jurisdiction over a
complaint that seeks damages from a defendant for making false or misleading statements
committed in violation of a federal statute? ANSWER: Yes. The federal district court has subject-
matter jurisdiction over the woman’s claim because it arises under a federal statute.

POINT (2)(b) [40%] ISSUE: Can a federal district court assert supplemental jurisdiction over a
cross-claim asserted by one defendant against another defendant when the claim is part of the same
case or controversy as the plaintiff’s claim but the cross-claim is based on state law and the
defendants are not diverse from each other? ANSWER: Yes. The federal court has supplemental
jurisdiction over the engineer’s cross-claim, and there is no reason for it to decline to exercise that
jurisdiction.

ANSWER DISCUSSION:

The court has personal jurisdiction over the corporation. The corporation sent representatives to State A
where those representatives made a presentation on the corporation’s behalf. That presentation, which the
woman alleges contained statements that were false, is the basis of the woman’s claim. By performing
activities in State A, the corporation purposely availed itself of the benefit and protection of State A law,
and opened itself to being sued in State A on any claim that might arise from its activities. Whether the
court can assert personal jurisdiction over the consulting engineer is a much closer question. On the one
hand, the engineer provided materials to the corporation that he knew would be distributed to prospective
buyers across the country. He could reasonably foresee that buyers in other states (including State A)
might rely on the brochures in making their purchasing decisions. On the other hand, the engineer’s
activities (preparing brochures for the corporation) occurred entirely within State B. The engineer was
never in State A and did not take any action of his own directed at that state in particular. In the absence
of any action by the engineer that was purposefully directed at State A, a court is likely to deny personal
jurisdiction over the engineer. The woman’s cause of action is based on federal law. The court therefore
has federal-question jurisdiction over the claim. The court also has subject-matter jurisdiction over the
engineer’s cross-claim. The cross-claim is part of the same case or controversy as the woman’s claim, and
the court can therefore exercise supplemental jurisdiction over it. In some cases a court can decline to take

Seperac-J19 Exam-Released MEE Essay Compilation © 2016-2020 29


supplemental jurisdiction over a claim over which it would not otherwise have jurisdiction, but no
grounds for declining jurisdiction are present here.

ANSWER EXPLANATION:

Explanation to Point-One(a) (25%):

The corporation has sufficient connection with State A to permit a court to exercise jurisdiction over it
with respect to a claim arising out of those contacts.

Federal district courts may exercise personal jurisdiction to the same extent as the courts of general
jurisdiction of the state in which the district court sits. Here, State A’s long-arm statute authorizes its
courts to exercise jurisdiction as far as the U.S. Constitution allows. So the question is whether the
defendants have sufficient contacts with State A such that a State A court could constitutionally exercise
jurisdiction over them.

The due process clause of the Fourteenth Amendment permits states to assert personal jurisdiction over
nonresident defendants who have established minimum contacts with the state such that the exercise of
personal jurisdiction would not offend traditional notions of fair play and substantial justice. This test may
be met even if a nonresident defendant has only a few contacts with the state, so long as the contacts
relate directly to the causes of action asserted. This is referred to as specific jurisdiction. Under the test for
specific jurisdiction, a nonresident defendant is subject to specific jurisdiction when its contacts with the
forum state demonstrate purposeful availment of the benefits of the forum state, and the cause of action is
related to those contacts.

In this case, the corporation sent salespeople to State A to solicit the woman to purchase its product.
During this visit, the corporation presented information aimed at inducing the woman to buy the super
solar panels, and its sales presentation succeeded. It also shipped its product to the woman in State A. By
these actions, the corporation purposefully availed itself of the privilege of doing business in State A and
received the benefits that State A generally provides to businesses operating in the state – for example,
roads, transportation facilities, and police protection. The woman’s claim arises directly out of the
corporation’s contacts with the state, and the corporation could certainly foresee that it might be haled
into a State A court if its conduct during its sales presentations in State A was wrongful. Here, the
corporation had sufficient contacts with State A to warrant a court in exercising jurisdiction over the
corporation.

Even when a nonresident defendant has the necessary minimum contacts with the forum state, the
exercise of personal jurisdiction over the defendant may offend due process if it is inconsistent with
traditional notions of fair play and substantial justice. The burden is on the defendant to make a
compelling case that the fairness considerations outweigh the existence of minimum contacts. Here, there
are good reasons for the litigation to be in State A rather than anywhere else (e.g., the woman lives there,
the defendant engaged in the tortious activity there, and the panels were installed there), and there is
nothing to suggest that litigating in State A would impose a particularly onerous burden on the
corporation. The court has jurisdiction over the corporation.

Explanation to Point-One(b) (15%):

The engineer is probably not subject to jurisdiction in State A, given that his activities were confined to
State B and he did not purposefully avail himself of the benefits or protections of State A law.

Seperac-J19 Exam-Released MEE Essay Compilation © 2016-2020 30


The engineer has never been physically present in State A, and there is no evidence that the engineer does
business in State A. There is also no evidence that the engineer’s activities specifically targeted State A in
any way. On the other hand, the engineer did prepare material designed to induce people to purchase
super solar panels, with the knowledge that the material would be distributed to potential buyers at sales
presentations throughout the country. Furthermore, it was reasonably foreseeable to the engineer that a
prospective buyer in another state, including State A, might rely on the accuracy of the information in the
brochure when making a purchase decision. The engineer has, in effect, placed the brochure in the stream
of commerce and thus reached out across state lines to communicate with and affect the decisions of State
A purchasers.

In J. McIntyre Machinery v. Nicastro, the Supreme Court rejected the so-called “stream of commerce”
theory of personal jurisdiction and held that merely placing a product in the stream of commerce with
awareness that it might reach a particular state was not a sufficient basis to exercise jurisdiction over the
manufacturer of the product. “The exercise of judicial power is not lawful,” said the four-justice plurality,
“unless the defendant ‘purposefully avails itself of the privilege of conducting activities within the forum
State.” In particular, according to the plurality, the transmission of goods (e.g., the brochure) to the forum
is a sufficient basis for jurisdiction “only where the defendant can be said to have targeted the forum; as a
general rule, it is not enough that the defendant might have predicted that its goods will reach the forum
State.” Two concurring justices agreed with the rejection of the “stream of commerce” theory, although
they felt that a flexible approach needed to be taken with regard to what additional facts might warrant
jurisdiction in a particular case.

Here, the only connection between the engineer and State A is that the engineer put the brochure into the
stream of commerce with awareness that it might be relied upon in other states (including, perhaps, State
A). Under the decision in Nicastro, that connection is simply not enough. In the absence of any facts to
suggest that the engineer targeted State A when he prepared the brochure and supplied it to the
corporation for distribution, or that the engineer has other connections with State A, the State A court
should dismiss the action against the engineer for lack of personal jurisdiction.

Explanation to Point-Two(a) (20%):

The federal district court has subject-matter jurisdiction over the woman’s claim because it arises under a
federal statute.

Pursuant to 28 U.S.C. § 1331, federal courts have “original jurisdiction of all civil actions arising under
the laws of the United States.” This provision gives a federal court subject-matter jurisdiction over any
case in which a plaintiff’s well-pleaded complaint alleges a cause of action based on federal law.

In this case, the woman’s claim is based on allegations that the defendants made false representations in
violation of the federal statute. Whether the woman has a claim on these facts, and whether the
defendants’ alleged actions did, indeed, violate the federal statute, are questions of federal law central to
her claim. Because the claim the woman is asserting is created by federal statutory law, it lies squarely
within the district court’s original federal-question jurisdiction under § 1331.

[NOTE: The court does not have diversity jurisdiction over the case because the woman is seeking only
$30,000, well below the $75,000 amount-in-controversy requirement that applies to diversity actions.]

Explanation to Point-Two(b) (40%):

Seperac-J19 Exam-Released MEE Essay Compilation © 2016-2020 31


The federal court has supplemental jurisdiction over the engineer’s cross-claim, and there is no reason for
it to decline to exercise that jurisdiction.

There is no basis on which a federal court could hear the engineer’s cross-claim if it had been brought as
an independent action. The claim is an ordinary contract claim based on state law, so there is no federal-
question jurisdiction. Furthermore, because the engineer is a resident of State B and the corporation is
incorporated and has its principal place of business in State B, they are both considered citizens of that
state, so the court would have no diversity jurisdiction. Even if they were diverse, the amount in
controversy (a $30,000 claim for indemnity) is inadequate for diversity jurisdiction.

However, once a court has original jurisdiction over some claims in the action, it may also exercise
supplemental jurisdiction over additional claims that are part of the same case or controversy. As noted, in
the present problem the federal court could exercise federal-question jurisdiction over the woman’s claims
against the two defendants. The question, therefore, is whether the engineer’s cross-claim is sufficiently
related to the woman’s claim to be part of the same case or controversy. There are significantly
overlapping facts indicating that the two claims really are part of a single case. The engineer alleges that
the corporation owes him indemnity based on the terms of contract between the corporation and the
engineer pursuant to which the engineer prepared the very material that the woman alleges was false. The
material in question, and its accuracy or inaccuracy, is central to the woman’s claim, and the contract
between the engineer and the corporation for the production of that material will probably be relevant to
an assessment of her allegations against each party. Second, the two claims are legally linked. For the
corporation to be liable to the engineer under the contract, the engineer must first be found liable to (or
must settle with) the plaintiff.

Assuming that the court finds that the § 1367(a) standard is met, then the court must consider whether
there are any discretionary reasons to decline to exercise jurisdiction, as set forth in § 1367(c). The
discretionary grounds for declining jurisdiction include the presence of novel or complex issues of state
law in the case, state-law claims that predominate over the federal claim in the case, or the district court’s
dismissal of all the claims over which it had original jurisdiction. In the present case, none of the
identified grounds for refusing jurisdiction are applicable, and there are no exceptional circumstances that
provide a compelling reason for declining jurisdiction.

Seperac-J19 Exam-Released MEE Essay Compilation © 2016-2020 32


#007-FEB 2015–MEE Q05: QUESTION FIVE (CIVIL PROCEDURE)

MedForms Inc. processes claims for medical insurers. Last year, MedForms contracted with a data entry
company (“the company”) to enter information from claims into MedForms’s database. MedForms hired
a woman to manage the contract with the company.

A few months after entering into the contract with the company, MedForms began receiving complaints
from insurers regarding data-entry errors. On behalf of MedForms, the woman conducted a limited audit
of the company’s work and discovered that its employees had been making errors in transferring data
from insurance claims forms to the MedForms database.

The woman immediately reported her findings to her MedForms supervisor and told him that fixing the
problems caused by the company’s errors would require a review of millions of forms and would cost
millions of dollars. In response to her report, the supervisor said, “I knew we never should have hired a
woman to oversee this contract,” and he fired her on the spot.

The woman properly initiated suit against MedForms in the United States District Court for the District of
State A. Her complaint alleged that she had been subjected to repeated sexual harassment by her
supervisor throughout her employment at MedForms and that he had fired her because of his bias against
women. Her complaint sought $100,000 in damages from MedForms for sexual harassment and sex
discrimination in violation of federal civil rights law.

After receiving the summons and complaint in the action, MedForms filed a third-party complaint against
the company, seeking to join it as a third-party defendant in the action. MedForms alleged that the
company’s data-entry errors constituted a breach of contract. MedForms sought $500,000 in damages
from the company. MedForms served the company with process by hiring a process server who
personally delivered a copy of the summons and complaint to the company’s chief executive officer at its
headquarters.

MedForms is incorporated in State A, where it also has its headquarters and document processing
facilities. The woman is a citizen of State A. The company’s only document processing facility is located
in State A, but its headquarters are located in State B, where it is incorporated and where its chief
executive officer was served with process.

State A and State B each authorize service of process on corporations only by personal delivery of a
summons and complaint to the corporation’s secretary.

1(a). The company has moved to dismiss MedForms’s third-party complaint for insufficient service
of process. How should the District Court rule on this ground asserted in the company’s motion to
dismiss? Explain.

1(b). The company has moved to dismiss MedForms’s third-party complaint for lack of subject-
matter jurisdiction. How should the District Court rule on this ground asserted in the company’s
motion to dismiss? Explain.

Seperac-J19 Exam-Released MEE Essay Compilation © 2016-2020 33


1(c). The company has moved to dismiss MedForms’s third-party complaint for improper joinder.
How should the District Court rule on this ground asserted in the company’s motion to dismiss?
Explain.

Seperac-J19 Exam-Released MEE Essay Compilation © 2016-2020 34


#007: F15-5 MEE: ANSWER: NCBE (CIVIL PROCEDURE)

POINT (1) [30%] ISSUE: Do the Federal Rules of Civil Procedure permit service of process on a
corporation to be made by delivery of the summons and complaint to a chief executive officer of the
corporation at corporate headquarters when the relevant state law requires service on the
corporation’s secretary? ANSWER: Yes. MedForms’s delivery of the summons and complaint to
the company’s CEO satisfies the requirements of Rule 4 of the Federal Rules of Civil Procedure
and of the Constitution.

POINT (2) [35%] ISSUE: Are two corporations diverse for purposes of federal jurisdiction when
they are incorporated and headquartered in different states but their main facilities are located in
the same state, which is also the state of incorporation of one of the businesses? ANSWER: Yes.
The District Court has diversity jurisdiction over MedForms’s breach of contract claim because the
amount in controversy exceeds $75,000 and MedForms and the company are citizens of different
states.

POINT (3) [35%] ISSUE: Can a third-party defendant be joined to a case when the claim against
that third-party defendant is factually related to the plaintiff’s original cause of action but the
claimant does not allege that the third-party defendant is liable to the original plaintiff or
responsible for the original plaintiff’s damages? ANSWER: No. MedForms’s joinder of the
company as a third-party defendant is improper because a third-party defendant may be joined by
the original defendant only when the original defendant claims that the third party is liable for all
or part of the plaintiff’s original claim.

ANSWER DISCUSSION:

The company was properly served. Rule 4 of the Federal Rules of Civil Procedure provides that service of
process may be made on a corporation by delivering a copy of the summons and complaint to a corporate
officer or to a general or managing agent. Because the Federal Rules explicitly authorized this manner of
service, it does not matter that it did not comply with State A or State B law. The District Court has
diversity jurisdiction over MedForms’s state-law claim against the company. The amount at stake
($500,000) satisfies the amount-in-controversy requirement, and the parties are diverse. MedForms is a
citizen of State A, where it is incorporated and has its head-quarters. The company is a citizen of State B,
where it is incorporated and has its headquarters. The fact that the document processing facilities for both
companies are in State A does not prevent them from being diverse. However, the third-party complaint
should be stricken. Rule 14 of the Federal Rules of Civil Procedure authorizes a defendant to bring a
complaint against a nonparty (a “third-party complaint”) only in situations in which the original defendant
asserts that the third-party defendant is somehow liable for whatever the original defendant might owe the
original plaintiff. Here, MedForms is not asserting that the company is liable to MedForms for any
damages owed to the woman. Rather, MedForms seeks to obtain relief for separate injuries it suffered as a
result of the company’s breach of contract. It cannot bring such a claim as a third-party complaint in this
action.

ANSWER EXPLANATION:

Explanation to Point-One (30%):

Seperac-J19 Exam-Released MEE Essay Compilation © 2016-2020 35


MedForms’s delivery of the summons and complaint to the company’s CEO satisfies the requirements of
Rule 4 of the Federal Rules of Civil Procedure and of the Constitution.

Under Rule 4 of the Federal Rules of Civil Procedure, service upon a corporation may be effected within
a United States judicial district by “following state law for serving a summons in an action brought in
courts of general jurisdiction in the state where the district court is located or where service is made,” or
“by delivering a copy of the summons and of the complaint to an officer, a managing or general agent.”
Here, MedForms did not serve the secretary of the company and therefore did not comply with the service
laws of either State A, where the District Court sits, or State B, where service was made. However,
MedForms nonetheless properly served the company under Rule 4 because it delivered a copy of the
summons and complaint to the company’s CEO. This service was explicitly authorized by Rule 4(h). Rule
4 does not specify which corporate representatives count as “officers” for the purpose of receiving service
of process, but a court would almost certainly conclude that service on a CEO satisfies the rule because
such an official would be “‘so integrated with the corporation sued as to make it a priori supposable that
he will realize his responsibilities and know what he should do with any legal papers served on him.’”

It is irrelevant that the manner of service chosen by MedForms was in violation of state law. Rule 4 of the
Federal Rules of Civil Procedure, which governs the service of process, regulates procedural matters and
thus is controlling law in federal diversity suits, notwithstanding any conflict with state service law.
Finally, this method of service (personal delivery of the summons and complaint to the CEO) was
reasonably calculated to provide the company with actual notice of the case and therefore satisfies
constitutional requirements.

Explanation to Point-Two (35%):

The District Court has diversity jurisdiction over MedForms’s breach of contract claim because the
amount in controversy exceeds $75,000 and MedForms and the company are citizens of different states.

MedForms’s breach of contract claim against the company is created by state law and therefore cannot
support federal-question jurisdiction. Original jurisdiction will exist, if at all, under 28 U.S.C. § 1332
(diversity jurisdiction).

Federal district courts can exercise jurisdiction over claims between citizens of different states where the
amount in controversy exceeds $75,000. Here, MedForms has claimed damages of $500,000, and the
court will accept this as the amount in controversy unless the allegation was made in bad faith or it
appears to a legal certainty that the plaintiff cannot recover that amount. There is nothing in the facts of
this problem to suggest that MedForms’s claim was made in bad faith or that its damages were less than
$75,000 to a legal certainty. Hence, the amount-in-controversy requirement is satisfied.

The question is therefore whether MedForms (the third-party plaintiff) and the company (the third-party
defendant) are citizens of different states. For diversity purposes, corporations have dual citizenship. A
corporation is a citizen both of the state where it is incorporated and also of the state where it has its
principal place of business. A corporation’s principal place of business is the corporation’s “nerve center”
(i.e., “the place where a corporation’s officers direct, control, and coordinate the corporation’s activities”).
Ordinarily, this will be “the place where the corporation maintains its headquarters,” unless the facts
indicate that the corporation’s designated “headquarters” is not really its “nerve center” but is simply an
office for occasional meetings.

Seperac-J19 Exam-Released MEE Essay Compilation © 2016-2020 36


In this case, MedForms is incorporated in State A. It also has its principal place of business in State A, as
that is where its headquarters is located. There is no evidence that its headquarters is not its nerve center.
Therefore, MedForms is a citizen of State A.

On the other hand, the company is incorporated in State B. Its principal place of business is also in State
B, as that is where its headquarters is located. There is no evidence that its headquarters is not its nerve
center. The fact that the company’s only document processing facility is in State A does not matter. In
Hertz Corp., the Supreme Court determined that a corporation’s principal place of business (and
citizenship) for diversity purposes would be determined by the location of the corporation’s headquarters
or “nerve center,” rather than by the location of its business activities. Thus, the company is a citizen of
State B.

Hence, MedForms and the company are citizens of different states and diversity exists.

[NOTE: An examinee who incorrectly concludes that there is no diversity jurisdiction should go on to
analyze whether the district court could exercise supplemental jurisdiction over MedForms’s claim. 28
U.S.C. § 1367 would authorize the district court to exercise supplemental jurisdiction over MedForms’s
claim against the company if it is part of the same constitutional “case or controversy” as the woman’s
discrimination claim against MedForms. In this case, there would be no supplemental jurisdiction
because the two claims are not part of the same case or controversy.

Claims form part of the same case or controversy if they arise out of a common nucleus of operative facts.
Here, the woman’s and MedForms’s claims arise from related facts, but they do not arise from a common
nucleus of operative facts. The operative facts underlying the woman’s claim are based on how her
MedForms supervisor treated her and his allegedly gender-based firing of her. MedForms’s claim rests
on the company’s unsatisfactory performance of its data-entry obligations under the contract. The main
operative facts that form the basis of the two claims are completely separate – the supervisor’s alleged
harassment and biased action on the one hand, and the company’s alleged inadequate contract
performance on the other.]>

Explanation to Point-Three (35%):

MedForms’s joinder of the company as a third-party defendant is improper because a third-party


defendant may be joined by the original defendant only when the original defendant claims that the third
party is liable for all or part of the plaintiff’s original claim.

Rule 14 of the Federal Rules of Civil Procedure authorizes a defendant to bring a nonparty into an action
only in very limited circumstances. If the defendant claims that the nonparty “is or may be liable to the
defendant for all or part of the claim against it,” then the defendant may bring a third-party complaint
against the nonparty and the nonparty may be joined as a third-party defendant.

Here, MedForms has attempted to bring the company into the action by alleging that the company
breached its contract with MedForms. MedForms does not claim that the company is or might be liable
for any damages that MedForms might be ordered to pay to the woman. Moreover, there are no facts that
suggest that the company played any role in, or had any obligation to indemnify MedForms for, the
behavior of MedForms’s supervisor that is the basis for the woman’s claim against MedForms. In short,
any liability that the company might have to MedForms is entirely independent of any liability MedForms
may incur to the woman. Thus, MedForms’s attempt to bring a third-party complaint against the company
exceeds the bounds of proper joinder, and MedForms’s complaint should be stricken.

Seperac-J19 Exam-Released MEE Essay Compilation © 2016-2020 37


[NOTE: In the facts, the company moves to “dismiss MedForms’s third-party complaint for improper
joinder.” FRCP Rule 14 allows a “motion to strike” a third-party claim for improper impleader, but
makes no mention of a motion to dismiss a third-party complaint. Litigants, however, commonly move to
dismiss, and courts regularly grant such motions, without regard to terminology. Similarly, although Rule
14 does not use the word “joinder,” litigants, courts, and scholars commonly refer to third-party
complaints as a form of joinder. It is important to distinguish impleader from the other claim-joinder
devices that are available.

Many examinees may discuss this issue in terms of the joinder standards of Rule 18 (Joinder of Claims)
or Rule 19 (Required Joinder of Parties). These examinees fail to recognize that the standards for Rule 14
impleader are different. Those who take this approach might be given some credit if they recognize that it
is proper to strike MedForms’s claim against the company because that claim is only tangentially related
to the claim of the original plaintiff (the woman) against MedForms. However, full credit is appropriate
only if the examinee also recognizes that a third-party complaint is not appropriate merely because the
claims are related or arise out of the same transaction. A third-party claim must be based on some legal
theory of derivative liability (e.g., that the third-party defendant has a legal obligation to indemnify the
original defendant in the event of the original defendant’s liability).]>

Seperac-J19 Exam-Released MEE Essay Compilation © 2016-2020 38


#008-JUL 2014–MEE Q04: QUESTION FOUR (CIVIL PROCEDURE)

The United States Forest Service (USFS) manages public lands in national forests, including the Scenic
National Forest. Without conducting an environmental evaluation or preparing an environmental impact
statement, the USFS approved a development project in the Scenic National Forest that required the
clearing of 5,000 acres of old-growth forest. The trees in the forest are hundreds of years old, and the
forest is home to a higher concentration of wildlife than can be found anywhere else in the western United
States.

The USFS solicited bids from logging companies to harvest the trees on the 5,000 acres of forest targeted
for clearing, and it ultimately awarded the logging contract to the company that had submitted the highest
bid for the trees. However, the USFS has not yet issued the company a logging permit. Once it does so,
the company intends to begin cutting down trees immediately.

A nonprofit organization whose mission is the preservation of natural resources has filed suit in federal
district court against the USFS. The nonprofit alleges that the USFS violated the National Environmental
Policy Act (NEPA) by failing to prepare an environmental impact statement for the proposed logging
project. Among other remedies, the nonprofit seeks a permanent injunction barring the USFS from issuing
a logging permit to the logging company until an adequate environmental impact statement is completed.
The nonprofit believes that the logging project would destroy important wildlife habitat and thereby cause
serious harm to wildlife in the Scenic National Forest, including some endangered species.

Assume that federal subject-matter jurisdiction is available, that the nonprofit has standing to bring this
action, and that venue is proper.

1. If the logging company seeks to join the litigation as a party, must the federal district court allow
it to do so as a matter of right? Explain.

2. What types of relief could the nonprofit seek to stop the USFS from issuing a logging permit
during the pendency of the action, what must the nonprofit demonstrate to obtain that relief, and is
the federal district court likely to grant that relief? Explain.

Seperac-J19 Exam-Released MEE Essay Compilation © 2016-2020 39


#008: J14-4 MEE: ANSWER: NCBE (CIVIL PROCEDURE)

POINT (1) [50%] ISSUE: Is the logging company entitled to join this action as a matter of right?
ANSWER: Yes. Rule 24(a) of the Federal Rules of Civil Procedure requires federal courts to allow
a person to intervene in an action as a matter of right if the person a) is interested in the property
or transaction that is the subject of the action, b) is so situated that its interest may be impaired or
impeded if the litigation goes forward without it, and c) is not adequately represented by existing
parties. Here, the logging company likely meets all three requirements and should be allowed to
intervene as a matter of right.

POINT (2)(a) [25%] ISSUE: May the nonprofit organization obtain a temporary restraining order
to stop the USFS from issuing a logging permit? ANSWER: Yes. The nonprofit organization could
seek and would likely obtain a temporary restraining order to stop the USFS from issuing a logging
permit, pending a hearing on an application for a preliminary injunction.

POINT (2)(b) [25%] ISSUE: May the nonprofit organization obtain a preliminary injunction to
stop the USFS from issuing a logging permit during the pendency of the action? ANSWER: Yes.
The nonprofit could also seek, and would likely obtain, a preliminary injunction to stop the USFS,
which is likely to be granted if the nonprofit’s claim that the USFS violated NEPA has a strong
basis in fact and law.

ANSWER DISCUSSION:

The logging company is entitled to intervene in this action as a matter of right because it has an interest in
the property or transaction that is the subject of the action and is so situated that its interest may be
impaired or impeded as a practical matter if the action goes forward without it. The logging company’s
interest is not adequately represented by the USFS’s presence in the lawsuit. The nonprofit organization
may seek a temporary restraining order (TRO), followed by a preliminary injunction, to prevent the USFS
from issuing a logging permit pending the outcome of the action. The nonprofit is likely to obtain a TRO
if it can demonstrate a risk of immediate and irreparable injury. The nonprofit is also likely to obtain a
preliminary injunction if it can demonstrate a significant threat of irreparable harm and a likelihood of
success on the merits of its National Environmental Policy Act (NEPA) claim.

ANSWER EXPLANATION:

Explanation to Point-One (50%):

Rule 24(a) of the Federal Rules of Civil Procedure requires federal courts to allow a person to intervene in
an action as a matter of right if the person a) is interested in the property or transaction that is the subject
of the action, b) is so situated that its interest may be impaired or impeded if the litigation goes forward
without it, and c) is not adequately represented by existing parties. Here, the logging company likely
meets all three requirements and should be allowed to intervene as a matter of right.

Rule 24 of the Federal Rules of Civil Procedure governs intervention, the process by which a non-party to
an action may join the litigation. Under Rule 24(a) (intervention of right), a person must be permitted to
intervene if three conditions are met: (1) the movant “claims an interest relating to the property or
transaction that is the subject of the action,” (2) the movant “is so situated that disposition of the action
may as a practical matter impair or impede the movant’s ability to protect its interest,” and (3) “existing

Seperac-J19 Exam-Released MEE Essay Compilation © 2016-2020 40


parties” do not “adequately represent the movant’s interest.” The three requirements for intervention of
right are often “very interrelated.”

Here, the court should find that the logging company meets this test. First, the logging company has a
strong interest in the property or transaction that is the subject of this action. The USFS has accepted the
logging company’s bid, and the logging company is merely awaiting issuance of a logging permit to begin
logging. The nonprofit organization is seeking to prevent this logging. The logging company therefore has
a strong, direct, and substantial interest in the subject matter of the lawsuit and in having its winning bid
honored and a logging permit issued. Timber companies have direct and substantial interests in a lawsuit
aimed at halting logging. If there is a direct substantial legally protectable interest in the proceedings, it is
clear that this requirement of the rule is satisfied.

Second, the logging company’s interest in receiving a logging permit may well be impaired, as a practical
matter, by the outcome of the lawsuit. If the USFS loses the lawsuit, it will have to prepare an
environmental impact statement before issuing the logging company’s permit. This will, at a minimum,
delay the logging company’s ability to exercise its rights and may, in the long run, mean that no logging
permit is ever issued. Intervention of right is not limited to those that would be legally bound as a matter
of preclusion doctrine. Rather, “the rule is satisfied whenever disposition of the present action would put
the movant at a practical disadvantage in protecting its interest.” Here, that condition is easily satisfied
because timber companies have direct and substantial interests in a lawsuit aimed at halting logging.

Given that the logging company has an interest that may be impaired by disposition of the action, it
should be allowed to intervene unless the court is persuaded that the USFS adequately represents the
logging company’s interest. Here, it could be argued that the USFS adequately represents the logging
company’s interest because the USFS presumably wants the court to uphold its development plan and
allow it to proceed with issuance of the logging permit, which is the same relief that the logging company
would seek. However, whether representation is truly adequate depends upon “a discriminating appraisal
of the circumstances.” Although both the government and the logging company wish to avoid the
preparation of an environmental impact statement, their interests are distinct. The USFS’s interest is
proper management of the national forest system, while the logging company’s interest is making a profit
from logging the 5,000-acre tract. The USFS’s handling of the litigation is likely to be affected by a
variety of policy concerns and political considerations that have nothing to do with the logging company’s
purely economic interest in securing the right to cut trees in the Scenic National Forest. The government
represents numerous complex and conflicting interests in matters of this nature. The straightforward
business interests asserted by intervenors here may become lost in the thicket of sometimes inconsistent
governmental policies.

[NOTES: (1) EXAMINEES WHO MISTAKENLY ANALYZE THE LOGGING


COMPANY’S CASE FOR JOINDER UNDER THE related but incorrect Rule 19 “Required
Joinder of Parties” may receive credit. Rule 19 allows existing parties to demand joinder of non-parties
(or seek dismissal of the case if they can’t get it). There is a close relationship between Rule 24 and Rule
19 and both contain a similar standard for determining when “interested” third parties are “entitled” or
“required” to be in the lawsuit. Indeed, the two prongs of the Rule 24 intervention test that are discussed
above are nearly identical to the two prongs of the Rule 19(a) required joinder test. Examinees who
discuss and apply the test should receive credit even if they cite Rule 19 rather than Rule 24.

(2) Examinees may discuss permissive joinder. Although permissive joinder is a possibility here, the
question asks only whether the logging company can join the action as a matter of right, and a permissive
joinder analysis is not responsive to the question. To the extent an examinee discusses permissive joinder,

Seperac-J19 Exam-Released MEE Essay Compilation © 2016-2020 41


the analysis will focus on whether the logging company “has a claim or defense that shares with the main
action a common question of law or fact.” The district court also “must consider whether the intervention
will unduly delay or prejudice the adjudication of the original parties’ rights.” On our facts, the logging
company’s claim for the issuance of a logging permit would certainly share common questions of law and
fact with the USFS’s defense against the nonprofit’s claim. There are no facts suggesting that the logging
company’s presence would unduly delay or otherwise prejudice adjudication of the original action. Thus,
the district court would have discretion to permit the logging company to intervene even if it denied
intervention of right.]>

Explanation to Point-Two(a) (25%):

The nonprofit organization could seek and would likely obtain a temporary restraining order to stop the
USFS from issuing a logging permit, pending a hearing on an application for a preliminary injunction.

The first type of interim relief the nonprofit could seek to stop the USFS from issuing a logging permit to
the logging company is a temporary restraining order (TRO) prohibiting the USFS from issuing the
logging permit. A TRO can be issued without notice to the adverse party, but only in limited
circumstances and only for a limited time. To secure a TRO without notice, the nonprofit would need to
submit an affidavit containing specific facts that demonstrate a risk of “immediate and irreparable injury”
if a permit is issued. In deciding whether to grant a TRO, courts will also consider the same factors that
are relevant in deciding whether to grant a preliminary injunction (e.g. the moving party’s likelihood of
success on the merits, the balance of hardships, and the public interest). The TRO would last only long
enough for the court to consider and resolve a request by the nonprofit for a preliminary injunction, but no
longer than 14 days (unless the court extends it for good cause or the adverse party consents to an
extension). In addition, bond is required.

Here, the court is likely to grant the nonprofit’s request. The nonprofit could plausibly claim that cutting
down 5,000 acres of old-growth forest in an area that is home to the highest concentration of wildlife in
the western United States would have “an immediate and irreparable” adverse impact on the environment
and cause irreparable harm to the nonprofit’s interest in preserving and protecting natural resources,
including wildlife habitat.

Explanation to Point-Two(b) (25%):

The nonprofit could also seek, and would likely obtain, a preliminary injunction to stop the USFS, which
is likely to be granted if the nonprofit’s claim that the USFS violated NEPA has a strong basis in fact and
law.

Because the TRO would be temporary, the nonprofit would need to move for a preliminary injunction to
prevent the USFS from issuing a logging permit throughout the pendency of the litigation. Preliminary
injunctions are injunctions that seek to “protect the plaintiff from irreparable injury and to preserve the
court’s power to render a meaningful decision after a trial on the merits.” Rule 65 of the Federal Rules of
Civil Procedure sets out the procedural requirements for preliminary injunctions. Preliminary injunctions
may be granted only upon notice to the adverse party, and only if the movant “gives security in an amount
that the court considers proper to pay the costs and damages sustained by any party found to have been
wrongfully enjoined or restrained.”

While Rule 65 sets out the procedural requirements for preliminary injunctive relief, it does not specify
the substantive grounds upon which it may be granted. The court’s discretion in ruling upon a motion for

Seperac-J19 Exam-Released MEE Essay Compilation © 2016-2020 42


a preliminary injunction “is exercised in conformity with historic federal equity practice.” The court
typically considers four factors:

(1) the significance of the threat of irreparable harm to the plaintiff if the injunction is not granted,

(2) the balance between this harm and the injury that granting the injunction would inflict on the
defendant,

(3) the probability that the plaintiff will succeed on the merits, and

(4) the public interest.

The most important of these factors is the risk of irreparable harm to the plaintiff. If the plaintiff has an
adequate remedy at law (e.g., if money damages can compensate the plaintiff for its loss), then a
preliminary injunction will be denied.

Here, a court would likely conclude that the potential for environmental damage to the forest creates a
significant threat of irreparable harm. “Environmental injury is often irreparable. Courts have recognized
that logging such as would occur here can have long-term environmental consequences and thus satisfy
the irreparable injury criterion.” Courts have noted that the imminent and continuing logging activities
presented evidence of environmental harm sufficient to tip the balance in favor of injunctive relief. The
old growth forests that the plaintiffs seek to protect would, if cut, take hundreds of years to reproduce.

The second factor, the balance between the harm to the plaintiff and the harm the defendant will suffer if
the injunction is issued, also appears to support issuance of a preliminary injunction here. The USFS will
have to wait before it can develop the Scenic National Forest, and the logging company may lose money
if the delay is prolonged. These economic harms could be compensated monetarily if an injunction is
issued inappropriately. Where “an injunction bond can compensate the defendant for any harm the
injunction is likely to inflict, the balance should be struck in favor of the plaintiff.” The relative absence
of harmful effects on the Forest Service weighs in favor of granting the injunction.

The third factor is the likelihood that the plaintiff will prevail on the merits. Although there is limited
information concerning the merits of the action, the nonprofit alleges that the federal statute (NEPA)
requires an environmental impact statement and further states that the USFS created no environmental
impact analysis or statement at all. Assuming that those allegations are correct, it seems plausible to
conclude that the nonprofit will be able to show a likelihood of success on the merits.

Finally, courts deciding whether or not to issue preliminary injunctive relief are to consider the public
interest. “Focusing on this factor is another way of inquiring whether there are policy considerations that
bear on whether the order should issue.” If the court concludes that the nonprofit is likely to succeed on its
NEPA claim because the USFS wrongfully failed to conduct an environmental impact assessment, it is
likely to find that the public interest would be served by restraining the USFS from proceeding with
logging in a national forest. Violations by federal agencies of NEPA’s provisions as established by
Congress harm the public as well as the environment.

Thus, a court is very likely to grant a preliminary injunction if it concludes that the nonprofit has a
significant likelihood of success on the merits.

Seperac-J19 Exam-Released MEE Essay Compilation © 2016-2020 43


#009-FEB 2014–MEE Q04: QUESTION FOUR (CIVIL PROCEDURE)

A builder constructed a vacation house for an out-of-state customer on the customer’s land. The house
was completed on June 1, at which point the customer still owed $200,000 of the $800,000 contract price,
which was payable in full five days later.

On June 14, the basement of the house was flooded with two inches of water during a heavy rainfall.
When the customer complained, the builder told the customer, “The flooding was caused by poorly
designed landscaping. Our work is fine and fully up to code. Have an engineer look at the foundation. If
there’s a problem, we’ll fix it.”

The customer, pleased by the builder’s cooperative attitude, immediately hired a structural engineer to
examine the foundation of the house. On June 30, the engineer provided the customer with a written
report on the condition of the foundation, which stated that the foundation was properly constructed.

Unhappy with the conclusions in the engineer’s report, the customer then hired a home inspector to
evaluate the house. The home inspector’s report concluded that the foundation of the house had been
poorly constructed and was inadequately waterproofed.

On July 10, the customer sent the builder the home inspector’s report with a note that said, “Until you fix
this problem, you won’t get another penny from me.” The builder immediately contacted an attorney and
directed the attorney to prepare a draft complaint against the customer for nonpayment. Hoping to avoid
litigation, the builder sent several more requests for payment to the customer. The customer ignored all
these requests.

On September 10, the builder filed suit in federal district court, properly invoking the court’s diversity
jurisdiction and seeking $200,000 in damages for breach of contract. The customer’s answer denied
liability on the basis of alleged defective construction of the house’s foundation.

Several months later, the case is nearly ready for trial. However, two discovery disputes have not yet been
resolved.

First, despite a request from the builder, the customer has refused to provide a copy of the report prepared
by the structural engineer who examined the foundation of the house. The customer claims that the report
is “work product” and not discoverable because the customer does not intend to ask the engineer to testify
at trial. The builder has asked the court to order the customer to turn over the engineer’s report.

Second, the customer has asked the court to impose sanctions for the builder’s failure to comply with the
customer’s demand for copies of all emails concerning construction of the foundation of the house. The
builder has truthfully informed the customer that all such emails were destroyed on August 2. This
destruction was pursuant to the builder’s standard practice of permanently deleting all project-related
emails from company records 60 days after construction of a project is complete. There is no relevant
state records-retention law.

1. Should the court order the customer to turn over the engineer’s report? Explain.

2. Should the court sanction the builder for the destruction of emails related to the case, and if so,
what factors should the court consider in determining those sanctions? Explain.

Seperac-J19 Exam-Released MEE Essay Compilation © 2016-2020 44


#009: F14-4 MEE: ANSWER: NCBE (CIVIL PROCEDURE)

POINT (1) [40%] ISSUE: Is a document prepared in the course of a contract dispute protected
from discovery as “work product” when there is no evidence that the document was prepared in
anticipation of litigation? ANSWER: Yes. The customer must turn over the engineer’s report
because it was not prepared in anticipation of litigation.

POINT (2)(a) [30%] ISSUE: Is a party’s failure to provide relevant electronically stored
information excused when the information was destroyed pursuant to a routine document retention
scheme at a time when litigation was contemplated by the destroying party? ANSWER: No.
Because the builder anticipated that it might be involved in litigation concerning its contract with
the customer, the builder acted wrongfully in destroying emails that were relevant to the house’s
construction, even though the emails were destroyed pursuant to a routine document retention
plan.

POINT (2)(b) [30%] ISSUE: What sanctions should be imposed on a party for allowing the
destruction of evidence that is relevant to potential future litigation? ANSWER: In determining
appropriate sanctions for spoliation, courts consider both the level of culpability of the spoliating
party and the degree of prejudice the loss of evidence has caused the other party. Here, the
builder’s destruction of evidence does not appear to have been willful, nor is it likely to pose a
significant obstacle to the customer’s defense. Any sanctions imposed by the court should be
modest.

ANSWER DISCUSSION:

The report prepared by the structural engineer is probably not work product and is thus discoverable. The
engineer examined the foundation of the house at the customer’s request, and the engineer’s findings are
potentially relevant to the customer’s claim that the foundation is defective. The report was not prepared
in anticipation of litigation. The customer appears to have sought the engineer’s opinion in response to the
builder’s offer to fix any problems with the foundation that an engineer might identify. Because the report
was not prepared in anticipation of litigation, it is not protected by the work-product doctrine. The builder
should have taken appropriate steps to preserve evidence, including suspending its document retention
program, as soon as it began planning for litigation – i.e., on July 10. Its destruction of potentially relevant
material after that date was wrongful. However, a court is unlikely to impose severe sanctions on the
builder because there are no facts indicating that the builder acted in bad faith and the customer can prove
that the foundation is defective without the destroyed emails.

ANSWER EXPLANATION:

Explanation to Point-One (40%):

The customer must turn over the engineer’s report because it was not prepared in anticipation of litigation.

In general, a party to a lawsuit in federal court “may obtain discovery regarding any nonprivileged matter
that is relevant to any party’s claim or defense.” This includes the right to inspect and copy documents in
the other party’s possession. Here, the customer hired a structural engineer to examine the foundation of
the house. The engineer’s report on the foundation is likely to include information that would be relevant
to the customer’s claim that the foundation was defectively constructed.

Seperac-J19 Exam-Released MEE Essay Compilation © 2016-2020 45


The so-called “work product” rule allows a party to refuse to turn over “documents that are prepared in
anticipation of litigation or for trial” by that party’s representative, including a consultant. Thus, if the
customer had hired the structural engineer to prepare a report “in anticipation of litigation,” that report
might not be discoverable.

In this case, however, the customer hired the engineer to evaluate the foundation of the house as part of
the customer’s negotiation with the builder concerning the house’s flooding problem. The builder told the
customer that the house’s landscaping was the reason for the flooding, and the builder told the customer,
“Have an engineer look at the foundation. If there’s a problem, we’ll fix it.” The customer appears to have
acted in response to that statement. There is no indication that the customer anticipated any kind of legal
action at the time that the structural engineer was hired. Accordingly, the structural engineer’s report is
discoverable and the court should order the customer to turn it over.

[NOTE: If an examinee concludes that the structural engineer’s report was prepared in anticipation of
litigation, then the examinee should also conclude that the report is not discoverable. Documents
prepared in anticipation of litigation do not need to be disclosed to an adverse party unless that party can
demonstrate a “substantial need” for the documents and an inability to obtain substantially equivalent
information without “undue hardship.” Furthermore, a report prepared by an expert who is not expected
to testify is not discoverable in the absence of “exceptional circumstances” making it “impracticable” to
obtain the information in another way. The builder probably cannot make these showings here, unless the
engineer’s report deals with circumstances that have since changed. There is no evidence that the
structural engineer would have had access to any information or facts that the builder would not already
know as a result of its construction and subsequent inspection of the house. In addition, if necessary, the
builder could ask the court for permission to arrange for a further inspection of the house by a structural
engineer hired by the builder. Accordingly, if an examinee concludes that the report was prepared in
anticipation of litigation, the examinee should also conclude that the builder is not entitled to see the
report.]

Explanation to Point-Two(a) (30%):

Because the builder anticipated that it might be involved in litigation concerning its contract with the
customer, the builder acted wrongfully in destroying emails that were relevant to the house’s construction,
even though the emails were destroyed pursuant to a routine document retention plan.

As noted above, a party to a lawsuit in federal court “may obtain discovery regarding any nonprivileged
matter that is relevant to any party’s claim or defense.” This includes emails and other electronically
stored information. Here, the customer has requested all the builder’s emails pertaining to work done on
the foundation of the house. Ordinarily, the builder would be obliged to turn over this information, which
is relevant to the customer’s defense that the house’s foundation was poorly constructed.

Unfortunately, the emails in question no longer exist because the builder destroyed them on August 2.

In general, spoliation of evidence (destruction or alteration of evidence) is improper if the party who
destroyed or altered the evidence “has notice that the evidence is relevant to litigation or should have
known that the evidence may be relevant to future litigation.” It is improper for a party to destroy
electronic information relevant to pending litigation, even if the destruction occurs before there is any
request or order seeking the information. For example, a plaintiff’s intentional destruction of computer
files warranted dismissal even though spoliation occurred before the order compelling discovery.
Similarly, the duty to preserve evidence applies to a party who anticipates litigation, even if litigation has
not yet been commenced.

Seperac-J19 Exam-Released MEE Essay Compilation © 2016-2020 46


The builder destroyed the emails on August 2. At that time, the builder knew that litigation was a
possibility because the builder had already directed its attorney to prepare a draft complaint for possible
filing. Knowing that litigation was a possibility, the builder had a duty to take steps to preserve evidence,
including the emails in question.

In this case, the builder’s destruction of the emails was pursuant to a routine document retention plan. The
Federal Rules provide expressly that, in the absence of “exceptional circumstances,” parties should not be
sanctioned for the loss of electronically stored information when the loss occurs pursuant to “routine,
good-faith operation of an electronic information system.” However, when a party anticipates litigation,
“it must suspend its routine document retention/destruction policy and put in place a ‘litigation hold’ to
ensure the preservation of relevant documents.”

Thus, the builder’s destruction of potentially relevant emails at a time when it knew that litigation was a
possibility was improper. It had a duty to preserve evidence, and it breached that duty.

[NOTE: Because courts have used different words to describe the test for when evidence must be
preserved, an examinee’s precise formulation of the test is not critical.]

Explanation to Point-Two(b) (30%):

In determining appropriate sanctions for spoliation, courts consider both the level of culpability of the
spoliating party and the degree of prejudice the loss of evidence has caused the other party. Here, the
builder’s destruction of evidence does not appear to have been willful, nor is it likely to pose a significant
obstacle to the customer’s defense. Any sanctions imposed by the court should be modest.

Federal courts have inherent power to control the litigation process and can sanction misbehavior,
including spoliation, even when there has been no specific violation of the Federal Rules of Civil
Procedure. The range of available sanctions is broad. It can include such sanctions as the payment of
expenses incurred by the other party as a result of the destruction of the evidence, an instruction to the
jury authorizing it to draw an adverse inference from the destruction of the evidence, a shifting of the
burden of proof on the relevant issue, or even judgment against the responsible party.

In determining appropriate sanctions for spoliation, courts consider both the level of culpability of the
spoliating party and the degree of prejudice the loss of evidence has caused the other party. Many courts
impose severe sanctions (such as an adverse-inference instruction or the entry of judgment against the
spoliating party) only when there is evidence of bad faith in the form of an intentional effort to hide
information. A sanction for spoliation requires intentional destruction based on a desire “to suppress the
truth”. However, other courts have said that negligence in preserving evidence can support an adverse-
inference instruction.

Although a court might well order an evidentiary hearing on the issue of sanctions, the facts presented do
not seem appropriate for severe sanctions. First, the evidence was destroyed pursuant to the builder’s
standard document retention plan, and there is no evidence that the builder deliberately failed to suspend
its usual procedures with the purpose of allowing the destruction of evidence. Second, the loss of this
evidence will not severely hinder the customer’s presentation of his case. The central issue is whether the
foundation of the house was properly constructed. If the construction job was poorly done, the customer
can present evidence, derived from inspection of the premises, to prove that point. The customer can also
depose witnesses about any issues that arose during construction.

Seperac-J19 Exam-Released MEE Essay Compilation © 2016-2020 47


Under the circumstances, a court is not likely to impose particularly severe sanctions, although it might
shift the burden to the builder to show that the foundation was properly constructed, or it might require
the builder to reimburse any expenses the customer incurs to discover and prove the facts about issues or
disputes that arose during construction of the foundation.

[NOTE: The result reached by the examinee is less important than the examinee’s recognition that (a) a
range of sanctions is available to the court and (b) the appropriate sanction depends both on the
culpability of the builder and the prejudice suffered by the customer.]

Seperac-J19 Exam-Released MEE Essay Compilation © 2016-2020 48


#010-JUL 2013–MEE Q01: QUESTION ONE (CIVIL PROCEDURE)

A woman was born and raised in the largest city (“the city”) of State A, where she also attended college.

Three years ago, the woman purchased a 300-acre farm and a farmhouse in neighboring State B, 50 miles
from the city. She moved many of her personal belongings to the State B farmhouse, registered her car in
State B, and acquired a State B driver’s license. She now spends seven months of the year in State B,
working her farm and living in the farmhouse. She pays income taxes in State B, but not in State A, and
lists State B as her residence on her federal income tax returns.

However, the woman has not completely cut her ties with State A. She still lives in the city for five
months each year in a condominium that she owns. She still refers to the city as “home” and maintains an
active social life there. When she is living on the farm, she receives frequent weekend visits from her city
friends and occasionally spends the weekend in the city at her condominium. She is a member of a health
club and a church in the city and obtains all her medical and dental care there. She is also registered to
vote and votes in State A.

A food product distributor sells food items to grocery stores throughout a five-state region that includes
States A and B. The distributor is a State C corporation. Its corporate headquarters are in State B, where
its top corporate officers, including its chief executive officer (CEO), have their offices and staff. The
distributor’s food processing, warehousing, and distribution facilities are all located in State A.

Three years ago, the woman and the distributor entered into a 10-year written contract providing that the
woman would sell all the produce grown on her farm each year to the distributor. The contract was
negotiated and signed by the parties at the distributor’s corporate headquarters in State B.

The woman and the distributor performed the contract for two years, earning her $80,000 per year.
Recently, the distributor decided that the woman’s prices were too high. At a meeting at its corporate
headquarters, the distributor’s CEO asked the woman to drop her prices. When she refused, the CEO
informed her that the distributor would no longer buy produce from her and that it was terminating the
contract.

The woman has sued the distributor for anticipatory breach of contract. She seeks $400,000 in damages.
She has filed suit in the United States District Court for the District of State A, invoking the court’s
diversity jurisdiction.

State A’s long-arm statute provides that “a court of this State may exercise personal jurisdiction over
parties to the fullest extent permitted by the due process clause of the Fourteenth Amendment to the
United States Constitution.”

The distributor has moved to dismiss the woman’s action for lack of subject-matter jurisdiction and for
improper venue.

1. Should the court grant the motion to dismiss for lack of subject-matter jurisdiction? Explain.

2. Should the court grant the motion to dismiss for improper venue? Explain.

Seperac-J19 Exam-Released MEE Essay Compilation © 2016-2020 49


#010: J13-1 MEE: ANSWER: NCBE (CIVIL PROCEDURE)

POINT (1)(a) [10%] ISSUE: What requirements must be met for a federal court to have
jurisdiction in a case based on diversity? ANSWER: A federal court has jurisdiction of a complaint
based on state law if the amount in controversy exceeds $75,000 and there is complete diversity
between the parties. Here, the amount-in-controversy requirement is satisfied.

POINT (1)(b) [35%] ISSUE: For purposes of diversity jurisdiction, is the woman a citizen of State
B, where she maintains a home and conducts her farming business, or of State A, where she has
lived for most of her life and to which she returns for several months each year, and which she calls
“home”? ANSWER: The woman is probably a citizen of State A. But an argument can be made
that she has acquired a new domicile in State B and therefore has become a citizen of State B.

POINT (1)(c) [25%] ISSUE: For purposes of diversity jurisdiction, is the distributor a citizen of
State A (where it has its food processing, warehousing, and distribution facilities), State B (where it
has its corporate headquarters), and/or State C (where it is incorporated)? ANSWER: The
distributor is a citizen of both State C, where it is incorporated, and State B, where it has its
principal place of business (i.e., where its corporate headquarters are located).

POINT (2) [30%] ISSUE: Is venue proper in State A, even though the acts and omissions giving rise
to the woman’s claim occurred in State B and the distributor’s corporate headquarters and state of
incorporation are outside State A? ANSWER: Yes. Because the distributor has sufficient contacts
with State A to be subject to personal jurisdiction there, the distributor resides in State A for
purposes of venue.

ANSWER DISCUSSION:

Because the woman’s claim is based on state law and the amount in controversy exceeds the jurisdictional
amount ($75,000), the key issue in determining whether subject-matter jurisdiction exists is whether the
plaintiff, the woman, and the defendant, the distributor, are citizens of different states. The woman is most
likely a domiciliary and therefore a citizen of State A, where she has lived continuously all her life and
which she still calls “home.” While an argument can be made that she has become a citizen of State B, the
fact that she spends the majority of her time each year in State B and treats it as her residence for certain
purposes probably does not amount to an abandonment of her State A domicile under the circumstances.
For diversity purposes, corporations may have dual citizenship. The distributor is a citizen of State C,
where it is incorporated. The distributor is also a citizen of State B, the state where it has its principal
place of business. Because the distributor is a citizen of States B and C, and the woman is probably a
citizen of State A, complete diversity exists and the court should not dismiss the action for lack of
jurisdiction. Assuming that diversity jurisdiction exists, venue is proper in State A, and the court should
not dismiss for improper venue. In an action founded solely on diversity, venue is proper in a district in
which the defendant resides. Corporate defendants are deemed to “reside” in any state where they are
subject to personal jurisdiction. Although the distributor is incorporated in State C and its headquarters
are in State B, it conducts sufficient business activities to subject it to personal jurisdiction in State A, and
thus is deemed to reside there for venue purposes.

ANSWER EXPLANATION:

Explanation to Point-One(a) (10%):

Seperac-J19 Exam-Released MEE Essay Compilation © 2016-2020 50


A federal court has jurisdiction of a complaint based on state law if the amount in controversy exceeds
$75,000 and there is complete diversity between the parties. Here, the amount-in-controversy requirement
is satisfied.

Federal district courts have original jurisdiction over state-law-based suits involving citizens of different
states where the amount in controversy exceeds $75,000. In determining the amount in controversy, a
court will accept the plaintiff’s good faith allegations as to damages unless it appears to a legal certainty
that the plaintiff cannot recover that amount. Here, the woman’s claim for $400,000 in damages appears
to have been made in good faith, given that the woman had earned $80,000 a year in the first two years of
the contract and that the distributor breached the contract with eight years remaining. Certainly, there is
nothing to suggest that the woman will recover less than $75,000 to a legal certainty. Accordingly, the
key issue is whether the woman and the distributor are citizens of different states.

Explanation to Point-One(b) (35%):

The woman is probably a citizen of State A. But an argument can be made that she has acquired a new
domicile in State B and therefore has become a citizen of State B.

To determine an individual U.S. citizen’s state citizenship for diversity purposes, courts look to the state
of the individual’s domicile when the complaint was filed. A person’s domicile is generally determined
by two elements: (1) residence in a state, and (2) an intent to remain in that state. In cases like this one,
where an individual resides for extended periods in more than one state, the determination of domicile
often turns on “a complex inquiry into an individual’s intent.” Courts look at a variety of facts, none of
which is dispositive on its own (and many of which can be manipulated by the individual concerned) in
an attempt to determine the individual’s true, fixed, permanent home and the place to which the individual
intends to return when absent. Relevant facts include the individual’s residence, the location of the
person’s property, the person’s voting behavior, the location of bank accounts, the individual’s
memberships and personal associations, automobile registration, place of employment, and other matters.

It is unclear whether the woman is domiciled in State A or State B. The woman was clearly a domiciliary
of State A, the only state in which she had ever lived, until she bought the farm and farmhouse in State B.
Despite her purchase of the State B farm, she maintains many social connections with State A, continues
to secure medical and dental treatment in State A, returns to State A to live every winter, and continues to
vote and attend church there. One could argue that these facts show that the woman never intended to
abandon her State A domicile, but always retained the intent to return there.

On the other hand, the woman did acquire property in State B, she moved many of her personal
belongings there, and she lives there for most of the year. Moreover, she took several steps that suggest
her intent to establish a State B domicile, including registering her car there, obtaining a State B driver’s
license, filing state income taxes only in State B, and listing State B as her residence on her federal tax
returns.

Although the matter is debatable, the facts suggest that the woman did not move to State B with the
requisite intent to remain there. Instead, it appears that she always intended that State A would remain her
home, although her livelihood requires her presence in State B for much of the year. In cases of this sort,
some courts seek to determine the “center of gravity” of an individual’s life. Despite the fact that the
woman lives in State B for two months more of the year than she lives in State A, it appears that the
“center of gravity” of her life is State A, and that a court would conclude that she is a domiciliary and
therefore a citizen of State A.

Seperac-J19 Exam-Released MEE Essay Compilation © 2016-2020 51


[NOTE: Examinees who demonstrate an understanding of the complexity of this issue, and who use the
key facts to develop a cogent argument, should receive full credit regardless of their conclusion on this
point.]

Explanation to Point-One(c) (25%):

The distributor is a citizen of both State C, where it is incorporated, and State B, where it has its principal
place of business (i.e., where its corporate headquarters are located).

By statute, a corporation is a citizen both where it is incorporated and where it has its principal place of
business. The distributor is incorporated in and therefore is a citizen of State C. So the question is where
the distributor has its principal place of business.

Until the Supreme Court’s decision in Hertz Corp. v. Friend, the federal courts were divided about how to
identify a corporation’s principal place of business. Some courts used a nerve-center test that focused on
where the corporation’s decision-making authority was located. Other courts applied a corporate-
activities/operating-assets test that focused on where a corporation’s business activities occurred.

In Hertz Corp., the Supreme Court decided that a corporation’s principal place of business for diversity
purposes would be deemed to be “the place where a corporation’s officers direct, control, and coordinate
the corporation’s activities,” its so-called “nerve center.” In this case, the distributor’s corporate
headquarters are in State B, its top officers have their offices and staff in State B, and its decisions
regarding the contract with the woman were made in State B. All of these facts suggest that State B is the
place where the corporate officers “direct, control, and coordinate” the distributor’s activities, and that
State B is therefore the distributor’s principal place of business.

The distributor, therefore, is a citizen of State B (as well as of State C). If the woman is a domiciliary of
State A, there is complete diversity of citizenship and the motion to dismiss for lack of jurisdiction should
be denied.

[NOTE: If an examinee previously concluded that the woman is a domiciliary of State B, then that
examinee should also conclude that the action must be dismissed for lack of subject-matter jurisdiction
because there would not be diversity of citizenship between the woman and the distributor.]

Explanation to Point-Two (30%):

Because the distributor has sufficient contacts with State A to be subject to personal jurisdiction there, the
distributor resides in State A for purposes of venue.

The woman brought this action in the United States District Court of State A. The issue, therefore, is
whether a State A venue can be justified under the general federal venue statute. When, as here, a federal
court’s jurisdiction is based on diversity, 28 U.S.C. § 1391 provides that venue is proper (1) in any district
where a defendant resides if all defendants reside in the same state, or (2) in any district where a
substantial part of the acts or omissions giving rise to the action occurred, or (3) in any district where any
defendant is subject to personal jurisdiction, if there is no other district in which the action might be
brought.

A State A venue is appropriate if the distributor “resides” in the District of State A for purposes of the
venue statute. According to 28 U.S.C. § 1391, a corporation resides where it is subject to personal

Seperac-J19 Exam-Released MEE Essay Compilation © 2016-2020 52


jurisdiction at the time the action is commenced. So the question is whether the distributor is subject to
personal jurisdiction in State A.

Rule 4(k) indicates that a federal district court may exercise personal jurisdiction to the same extent as a
state court of general jurisdiction in the state where the district court sits. State A’s long-arm statute
extends jurisdiction as far as the Due Process Clause of the 14th Amendment allows. The Due Process
Clause permits the exercise of personal jurisdiction over out-of-state defendants like the distributor if they
have minimum contacts with the forum state such that the exercise of jurisdiction would not offend
traditional notions of fair play and substantial justice. The nature of the contacts required depends on
whether the suit arises out of or is related to the defendant’s contacts with the forum state.

Here, the distributor maintains its business facilities in State A, including food processing, warehousing,
and distribution facilities. Regardless of whether the woman’s cause of action is deemed to be related to
those contacts, these continuous and substantial business connections with State A would warrant a State
A court to exercise jurisdiction over the distributor. Moreover, requiring the distributor to defend this
action in State A is not likely to be viewed by the court as unfair or unreasonable. Whether the exercise of
personal jurisdiction is fair and reasonable depends on various considerations such as the burden on the
defendant of having to litigate in the forum state, the plaintiff’s interest in a convenient forum for
obtaining relief, the state’s interest in providing a forum, and the interests of the interstate judicial system
in efficient resolution of disputes. Given modern transportation and communication, the distributor’s
burden of defending in a state where it does continuous and substantial business is minimal even if the
suit is unrelated to the distributor’s business in that forum.

While the woman lives in State B for most of the year, the State A federal court is located close to her
home in the city, where she lives for part of the year. Furthermore, because the woman is a part-year
resident of State A, State A has some interest in providing a forum where she can obtain redress for legal
harms. On the other hand, adjudication in State A is not particularly efficient. Aside from the woman,
who resides in State A for part of the year, it appears that the bulk of the evidence and witnesses relevant
to her case will be located in State B. Nonetheless, that fact alone would not be enough to defeat State A’s
jurisdiction over the distributor in this case.

Because State A could assert personal jurisdiction over the distributor at the commencement of the action
(had the issue been raised), the distributor will be deemed to “reside” in State A, rendering the District of
State A an appropriate venue under 28 U.S.C. § 1391. The motion to dismiss for improper venue should
be denied.

[NOTE #1: By failing to raise the defense of personal jurisdiction in its motion, the distributor waived the
defense. However, the venue statute asks whether the distributor was subject to personal jurisdiction at
the commencement of the suit, so the distributor’s waiver of possible personal jurisdiction defense does
not affect the venue inquiry. An examinee should receive credit for a personal jurisdiction analysis ONLY
if it is offered in the context of determining where the distributor “resides” for venue purposes.]

[NOTE #2: Here, neither the second nor the third basis for venue is applicable. As to the second basis for
venue (a district where a “substantial part” of the relevant acts occurred), the woman’s claim is based on
an alleged breach of contract. The facts state that this contract was negotiated in State B, that
performance occurred in State B, and that the decision to stop performing on the contract was made in
State B. It does not appear that any of the acts or omissions giving rise to the woman’s claim occurred in
the District of State A.

Seperac-J19 Exam-Released MEE Essay Compilation © 2016-2020 53


The third basis for venue is applicable only if there is no other district where the case could be brought.
Here, as just noted, the actions giving rise to the woman’s claim took place in State B, and there were no
impediments to her bringing the claim there. Because there is another district where suit could have been
brought, a State A venue cannot be based on the absence of such a district.]>

Seperac-J19 Exam-Released MEE Essay Compilation © 2016-2020 54


#011-FEB 2013–MEE Q05: QUESTION FIVE (CIVIL PROCEDURE)

Mother and Son, who are both adults, are citizens and residents of State A. Mother owned an expensive
luxury car valued in excess of $100,000. Son borrowed Mother’s car to drive to a store in State A. As Son
approached a traffic light that had just turned yellow, he carefully braked and brought the car to a
complete stop. Driver, who was following immediately behind him, failed to stop and rear-ended
Mother’s car, which was damaged beyond repair. Son was seriously injured. Driver is a citizen of State B.

Son sued Driver in the United States District Court for the District of State A, alleging that she was
negligent in the operation of her vehicle. Son sought damages in excess of $75,000 for his personal
injuries, exclusive of costs and interest. In her answer, Driver alleged that Son was contributorily
negligent in the operation of Mother’s car. She further alleged that the brake lights on Mother’s car were
burned out and that Mother’s negligent failure to properly maintain the car was a contributing cause of the
accident.

Following a trial on the merits in Son’s case against Driver, the jury answered the following special
interrogatories:

Do you find that Driver was negligent in the operation of her vehicle? Yes.

Do you find that Son was negligent in the operation of Mother’s car? No.

Do you find that Mother negligently failed to ensure that the brake lights on her car were in proper
working order? Yes.

The judge then entered a judgment in favor of Son against Driver. Driver did not appeal.

Two months later, Mother sued Driver in the United States District Court for the District of State A,
alleging that Driver’s negligence in the operation of her vehicle destroyed Mother’s luxury car. Mother
sought damages in excess of $75,000, exclusive of costs and interest.

State A follows the same preclusion principles that federal courts follow in federal-question cases.

1. Is Mother’s claim against Driver barred by the judgment in Son v. Driver? Explain.

2. Does the jury’s conclusion in Son v. Driver that Mother had negligently failed to maintain the
brake lights on her car preclude Mother from litigating that issue in her subsequent suit against
Driver? Explain.

3. Does the jury’s conclusion in Son v. Driver that Driver was negligent preclude Driver from
litigating that issue in the Mother v. Driver lawsuit? Explain.

Seperac-J19 Exam-Released MEE Essay Compilation © 2016-2020 55


#011: F13-5 MEE: ANSWER: NCBE (CIVIL PROCEDURE)

POINT (1) [35%] ISSUE: Does a judgment in a prior action preclude a nonparty from suing the
same defendant on a closely related claim when the nonparty and the original plaintiff are in a
family relationship? ANSWER: No. Under the doctrine of claim preclusion, the judgment rendered
in the first action does not preclude Mother, a nonparty, from suing Driver for the damage to her
car because the judgment binds only parties or those in privity with them, and Mother and Son are
not in privity.

POINT (2) [35%] ISSUE: Does a judgment rendered in an earlier action preclude a nonparty from
litigating an issue that was actually decided in the first suit? ANSWER: No. Under the doctrine of
issue preclusion, the judgment rendered in the first action does not preclude Mother, a nonparty,
from litigating the issue of her negligence in maintaining her car’s brake lights because the
judgment binds only parties or those in privity with them, and Mother and Son are not in privity.

POINT (3) [30%] ISSUE: May a nonparty to an earlier action invoke the judgment in that action to
preclude a party to the prior action from relitigating an issue that the party had a full and fair
opportunity to litigate in the earlier action? ANSWER: Under the doctrine of non-mutual issue
preclusion, the judgment rendered in the first action might preclude Driver from relitigating the
issue of her negligence. However, Driver has a strong argument that such a result would be
inconsistent with the policy against offensive use of non-mutual estoppel when the non-party
plaintiff easily could have joined as a plaintiff in the first action.

ANSWER DISCUSSION:

Pursuant to the doctrines of claim preclusion (res judicata) and issue preclusion (collateral estoppel), a
judgment is binding on the parties thereto. In the absence of privity, nonparties to a prior suit cannot be
bound by a judgment rendered in their absence. Thus, in the absence of privity, a nonparty to the first suit
is not precluded from presenting her claim in a second suit even if it is factually related to the claims and
defenses presented in the first suit; nor is she bound by determinations of issues made in the first suit. A
family relationship, without more, does not support a finding of privity. For this reason, Mother, as a
nonparty, is not bound by the judgment in the Son-Driver action. She may bring her separate claim for
damage to her car, and she is not precluded from litigating the question of whether she was negligent in
the maintenance of her car. Driver, on the other hand, could be precluded from relitigating the issue of her
negligence pursuant to the doctrine of non-mutual issue preclusion (also called non-mutual offensive
collateral estoppel), which allows a nonparty to a prior action to invoke issue preclusion to prevent a party
to that prior action from relitigating determinations of issues made therein. However, Mother may be
prevented from invoking non-mutual collateral estoppel in this case because she could easily have joined
her claim in the prior action but did not do so. [NOTE: Under Semtek, federal common law governs the
preclusive effect of a judgment rendered by a federal court sitting in diversity. But the Semtek Court
concluded that federal common law, in this context, incorporates the preclusion law of the state in which
the rendering federal court sits (unless the state law is incompatible with federal interests). Thus, State A’s
preclusion law determines the preclusive effect of the judgment rendered in Son’s suit against Driver. The
problem says that State A preclusion law is identical to federal preclusion law, so the following analysis
utilizes general principles of preclusion drawn from Supreme Court case law (announcing federal
preclusion rules) and the Second Restatement of Judgments.]

ANSWER EXPLANATION:

Seperac-J19 Exam-Released MEE Essay Compilation © 2016-2020 56


Explanation to Point-One (35%):

Under the doctrine of claim preclusion, the judgment rendered in the first action does not preclude
Mother, a nonparty, from suing Driver for the damage to her car because the judgment binds only parties
or those in privity with them, and Mother and Son are not in privity.

Driver may contend that the doctrine of claim preclusion (res judicata) precludes Mother from presenting
a claim arising from the same nucleus of facts that was presented in the first action brought by Son.
According to the doctrine of claim preclusion, “when a court of competent jurisdiction has entered a final
judgment on the merits of a cause of action, the parties to the suit and their privies are thereafter bound
‘not only as to every matter which was offered and received to sustain or defeat the claim or demand, but
as to any other admissible matter which might have been offered for that purpose.’”

However, the doctrine of claim preclusion does not apply to Mother on the facts of this problem. First,
Mother was not a party to the earlier case. “It is a principle of general application in Anglo-American
jurisprudence that one is not bound by a judgment in personam in a litigation in which he is not
designated as a party or to which he has not been made a party by service of process.” This rule reflects
our “deep-rooted historic tradition that everyone should have his own day in court.” Since Mother was not
a party to the first suit, she is not bound by the judgment unless an exception to the general rule applies.

Mother might be bound by the prior judgment if she were considered to have been sufficiently in privity
with Son that Son represented her interests in that action. “A person who is not a party to an action but
who is represented by a party is bound by and entitled to the benefits of a judgment as though he were a
party.” But there is no suggestion in the facts of the problem that Son, who is an adult, purported to
represent Mother’s interests in the first suit. “Close family relationships are not sufficient by themselves
to establish privity with the original suit’s party, or to bind a nonparty to that suit by the judgment entered
therein

In Taylor v. Sturgell, the Supreme Court identified other special circumstances in which nonparties may
be bound by a prior judgment – when a nonparty consents to be bound; when a nonparty is in a pre-
existing substantive legal relationship with a party (such as preceding and succeeding property owners);
when a nonparty assumed control of the prior litigation; when a party seeks to relitigate through a proxy;
or where a special statutory scheme seeks to foreclose successive litigation by nonparties. None of these
circumstances exists here.

Because Mother was not a party to the first suit and is not in privity with Son, who is an adult, the
judgment in the first action does not preclude her from bringing her own claim against Driver.

Explanation to Point-Two (35%):

Under the doctrine of issue preclusion, the judgment rendered in the first action does not preclude Mother,
a nonparty, from litigating the issue of her negligence in maintaining her car’s brake lights because the
judgment binds only parties or those in privity with them, and Mother and Son are not in privity.

By its affirmative response to a special interrogatory, the jury in the first action expressly concluded that
“Mother negligently failed to ensure that the brake lights on her car were in proper working order.” Driver
may attempt to invoke the doctrine of issue preclusion to preclude Mother from relitigating this issue in
the second action.

Seperac-J19 Exam-Released MEE Essay Compilation © 2016-2020 57


Issue preclusion arises in a second action on the basis of a prior decision when the same ‘issue’ is
involved in both actions; the issue was ‘actually litigated’ in the first action, after a full and fair
opportunity for litigation; the issue was ‘actually decided’ in the first action, by a disposition that is
sufficiently ‘final,’ ‘on the merits,’ and ‘valid’; it was necessary to decide the issue in disposing of the
first action, and the later litigation is between the same parties or involves nonparties that are subject to
the binding effect or benefit of the first action Once these requirements are met, issue preclusion is
available not only to defend against a demand for relief, but also as offensive support for a demand for
relief. Issue preclusion, moreover, is available whether or not the second action involves a new claim or
cause of action.

Here, several of the elements necessary for issue preclusion are present. The same issue is involved in
both actions – the issue of Mother’s negligence in failing to maintain the brake lights on her car. That
issue was actually litigated in the first action and decided by the jury. There is nothing to suggest anything
less than a full and fair opportunity to litigate. The judgment disposing of the issue was final.

Nevertheless, the judgment will not preclude Mother from relitigating the issue for two reasons. First,
Mother was not a party to the first action and, as explained above, Mother and Son are not in privity.
Therefore, she cannot be denied an opportunity to litigate the issue of her negligence. Second, it does not
appear that the jury’s decision as to Mother’s negligence was necessary to the prior judgment against
Driver. Nothing suggests that the finding on Mother’s negligence had any bearing on the outcome of the
first action.

Explanation to Point-Three (30%):

Under the doctrine of non-mutual issue preclusion, the judgment rendered in the first action might
preclude Driver from relitigating the issue of her negligence. However, Driver has a strong argument that
such a result would be inconsistent with the policy against offensive use of non-mutual estoppel when the
non-party plaintiff easily could have joined as a plaintiff in the first action.

Because Son already convinced the jury in the first action that “Driver was negligent in the operation of
her vehicle,” Mother may wish to invoke the doctrine of non-mutual issue preclusion to prevent Driver
from relitigating the question of her negligence. As noted above, “issue preclusion arises in a second
action on the basis of a prior decision when the same ‘issue’ is involved in both actions; the issue was
‘actually litigated’ in the first action, after a full and fair opportunity for litigation; the issue was ‘actually
decided’ in the first action, by a disposition that is sufficiently ‘final,’ ‘on the merits,’ and ‘valid’; it was
necessary to decide the issue in disposing of the first action.”

Here, these basic requirements for issue preclusion are met. First, the same issue is involved in both suits:
whether Driver was negligent in the operation of her car. Second, this issue was actually litigated and
decided in the first action; the jury answered a special interrogatory raising this very question. There is
nothing to suggest that Driver lacked a full and fair opportunity to litigate the issue. Since a judgment was
rendered against Driver for the injuries Son sustained as a result of Driver’s negligence, resolution of the
issue was necessary to dispose of the first action. Driver was a party to the first action, so she may be
bound by the judgment.

[NOTE: Traditionally, issue preclusion required mutuality – both the party asserting issue preclusion and
the party against whom issue preclusion was asserted were bound by the prior judgment. Under the
traditional mutuality rule, Mother could not assert issue preclusion against Driver because Mother would
not be bound by the judgment if Driver sought to rely on it. There is no mutuality between Mother and
Driver with respect to the prior judgment.

Seperac-J19 Exam-Released MEE Essay Compilation © 2016-2020 58


This traditional mutuality requirement has been abandoned in most jurisdictions. The Supreme Court
rejected a strict mutuality requirement in Blonder-Tongue Laboratories v. University of Illinois
Foundation Non-mutual defensive collateral estoppel has been used by defendants to preclude a plaintiff
from relitigating a claim the plaintiff previously litigated. Likewise, non-mutual offensive collateral
estoppel used by a plaintiff to preclude a defendant from relitigating a claim the defendant previously
litigated. In Parklane Hosiery, the Court concluded (as a matter of federal preclusion law) that trial
courts should have “broad discretion” to determine whether or not to permit a plaintiff to invoke non-
mutual issue preclusion. “The general rule should be that in cases where a plaintiff could easily have
joined in the earlier action or where the application of offensive estoppel would be unfair to a defendant,
a trial judge should not allow the use of offensive collateral estoppel.”

The Parklane Hosiery decision identified a number of circumstances that might make it unfair to allow a
plaintiff to invoke non-mutual issue preclusion (non-mutual offensive collateral estoppel in the traditional
terminology) against a defendant. In particular, the Parklane Hosiery court suggested that issue
preclusion may not be appropriate if the plaintiff in the second action “could easily have joined in the
earlier action.” Prohibiting plaintiffs from using non-mutual estoppel under such circumstances would
promote judicial efficiency by encouraging plaintiffs to join the prior action. It would also discourage
plaintiffs from staying out of prior litigation in order to secure, in effect, two bites at the apple: using the
prior litigation offensively if the defendant loses and forcing the defendant to litigate a second time if the
defendant wins the prior action.

An exceptional exam answer might therefore argue that non-mutual issue preclusion should be denied on
these facts. Son and Mother both reside in State A; since they are related, they know each other well, and
Son was driving Mother’s car when the accident occurred. They could have sued together, and Rule 20 of
the Federal Rules of Civil Procedure would have authorized joinder of their claims because those claims
arose from the same transaction or occurrence and raised a common question of law or fact. The facts do
not suggest that Mother had any reason not to join Son’s suit other than a desire to see how Son’s action
concluded before bringing her own claim. Because it appears that Mother may be a “wait-and-see”
plaintiff who could easily have joined the original action, a trial court might disallow, as a matter of
discretion, her use of non-mutual issue preclusion.]>

Seperac-J19 Exam-Released MEE Essay Compilation © 2016-2020 59


#012-JUL 2012–MEE Q07: QUESTION SEVEN (CIVIL PROCEDURE)

Plaintiff, a female employee of Defendant, a large manufacturing firm, sued Defendant in federal district
court for violating a federal statute that creates a right to be free of sex discrimination in the workplace.

Plaintiff alleged the following: (1) Plaintiff worked for Defendant in a position for which females had
seldom been hired in the past. (2) Shortly after Plaintiff was hired, male coworkers began to make
sexually charged remarks to Plaintiff. (3) Plaintiff’s male supervisor asked her out on dates and became
angry each time she refused. (4) There were occasional incidents in which the supervisor or another male
worker “accidentally” made contact with various parts of Plaintiff’s body. (5) No one from company
management ever took steps to monitor or limit behavior of this sort. (6) As a result of this behavior,
Plaintiff began to suffer from various physical ailments that were related to stress. (7) Plaintiff made no
complaint to management about the situation because the job paid very well and there were, to her
knowledge, no comparable opportunities that would be available to her if she lost this particular job.

Defendant’s answer to the complaint admitted that Plaintiff was an employee and that the individual
named as her supervisor was her supervisor. Defendant denied all allegations relating to the alleged sex
discrimination.

A well-established affirmative defense is available in cases of this sort if the defendant employer proves
that (a) the plaintiff employee was not subject to any adverse job action (firing, demotion, loss of
promotion opportunity, etc.), (b) the employer exercised reasonable care to prevent and promptly correct
any sexually harassing behavior, and (c) the plaintiff employee unreasonably failed to take advantage of
any preventive or corrective opportunities provided by the employer.

In a pretrial deposition, Plaintiff admitted that she had suffered no loss of pay or promotion opportunity.
Plaintiff also admitted that she was aware of company policies forbidding sex discrimination and sexual
harassment, as well as the procedures that employees could use to complain about perceived
discrimination. Plaintiff stated that although she was aware of those policies and procedures, she had not
seen any effort on the part of Defendant to enforce the policies and was afraid that she would suffer
retaliation if she made use of the procedures available to complain of sex discrimination.

After the close of discovery, Defendant moved to amend its answer to add the affirmative defense set
forth above. It also moved for summary judgment, claiming that Plaintiff’s deposition testimony
sufficiently established the elements of the affirmative defense to warrant a judgment in Defendant’s
favor.

Plaintiff opposed both motions. The trial judge ruled in Defendant’s favor, allowing the amendment and
granting summary judgment.

Did the judge err? Explain.

Seperac-J19 Exam-Released MEE Essay Compilation © 2016-2020 60


#012: J12-7 MEE: ANSWER: NCBE (CIVIL PROCEDURE)

POINT (1) [50%] ISSUE: Do the Federal Rules of Civil Procedure permit a defendant to amend its
answer after discovery has closed in order to add an affirmative defense that has been suggested by
facts revealed in discovery when that defense could have been raised at the outset of the action?
ANSWER: Yes. The trial court properly granted Defendant leave to amend its answer to the
complaint, given that justice favors allowing Defendant to raise a potentially valid affirmative
defense and no facts suggest that Plaintiff was unfairly prejudiced by the amendment.

POINT (2) [50%] ISSUE: Should summary judgment be granted when facts revealed in discovery
support a defendant’s affirmative defense but when there are other facts at issue that would permit
a jury to reject the defense? ANSWER: No. The trial court erred in granting summary judgment.
Plaintiff’s deposition answers, which were the basis of Defendant’s summary judgment motion,
raise genuine issues of material fact sufficient to preclude entry of judgment as a matter of law.

ANSWER DISCUSSION:

Under FRCP Rule 15, leave to amend should be freely given when justice requires. Here, the merits of the
case are advanced by allowing the amendment adding the affirmative defense, and there are no facts to
suggest that Plaintiff would be prejudiced in the preparation and presentation of her case by allowing the
amendment. Although Plaintiff will be harmed if Defendant is allowed to prove a valid defense, the harm
of being subject to a valid defense is not the kind of “prejudice” that would warrant denying an
amendment. On the applicable “abuse of discretion” standard, it is unlikely that the appellate court would
reverse the trial court’s decision on this issue. The trial court erred in granting Defendant’s motion for
summary judgment. Under Rule 56, the standard for granting such a motion is that there is no genuine
issue of material fact raised. Here, Plaintiff’s deposition testimony suggests that there are genuine issues
of material fact concerning whether Defendant exercised reasonable care to prevent and correct
harassment and whether Plaintiff’s failure to complain about the harassment was reasonable.

ANSWER EXPLANATION:

Explanation to Point-One (50%):

The trial court properly granted Defendant leave to amend its answer to the complaint, given that justice
favors allowing Defendant to raise a potentially valid affirmative defense and no facts suggest that
Plaintiff was unfairly prejudiced by the amendment.

A defendant has the burden of pleading all affirmative defenses. Federal Rule of Civil Procedure 8(c) is
very clear when it states that a party defending a claim “must affirmatively state any avoidance or
affirmative defense.” Defendant had failed to raise a potentially valid affirmative defense in its answer
and later sought to amend.

Under Rule 15, a district court “should freely give leave to amend when justice so requires.” The Supreme
Court has defined this standard “negatively” by indicating that amendments should be allowed unless they
result in a form of injustice. The categories of injustice the Court describes are undue delay, bad faith or
dilatory motive, repeated failure to cure defects by amendment, undue prejudice to the party opposing the
amendment, or futility of the amendment. Here, the only possible ground that Plaintiff might use to resist
the amendment is “undue prejudice.”

Seperac-J19 Exam-Released MEE Essay Compilation © 2016-2020 61


Defendant has a strong argument that it would be unjust to deny the amendment because doing so would
deny Defendant an opportunity to have the case decided on its legal merits. Indeed, having cases decided
on their merits is one of the key goals of the Federal Rules of Civil Procedure and one reason that the
Rules direct courts to “freely give leave” to amend. Moreover, allowing a potentially valid defense does
not cause “undue prejudice” to Plaintiff. Plaintiff’s legal claim is only as good as its ability to overcome
defenses, and it is not unjust to require that she respond to those defenses.

Plaintiff might argue that she is prejudiced by injecting the defense after the close of discovery. But no
facts suggest prejudice, and it is difficult to know what the prejudice might be. The defense is supported
by testimony of Plaintiff and is based upon matters known to her all along. Moreover, Plaintiff’s counsel
knew or should have known about this well-established affirmative defense. Finally, if Plaintiff requires
more time for discovery, that is a matter that the court could address by granting more time (if the issue
were raised) and is not itself a basis for denying leave to amend. In short, justice seems better served by
allowing the amendment to ensure that the case is decided on its merits and not on the basis of
Defendant’s failure to plead an affirmative defense in its original answer.

Explanation to Point-Two (50%):

The trial court erred in granting summary judgment. Plaintiff’s deposition answers, which were the basis
of Defendant’s summary judgment motion, raise genuine issues of material fact sufficient to preclude
entry of judgment as a matter of law.

FRCP Rule 56 allows a summary judgment motion to be granted only if “there is no genuine issue as to
any material fact and the movant is entitled to judgment as a matter of law (JML).” An issue of fact is
“genuine” if, based on the evidence presented by the nonmoving party, a reasonable jury could return a
verdict for that party. A fact is “material” if it is relevant to an element of a claim or defense and its
existence would affect the outcome of the case under the governing law. In determining whether there is a
genuine dispute as to material fact, the court should consider the “depositions, documents, electronically
stored information, affidavits or declarations, stipulations (including those made for purposes of the
motion only), admissions, interrogatory answers, or other materials.”

Here, Defendant has an affirmative defense to a claim of sex discrimination if Defendant can prove that
no adverse job action was taken against Plaintiff, that Defendant exercised reasonable care to prevent and
correct harassing behavior, and that the employee unreasonably failed to take advantage of protective or
corrective opportunities. When, as here, a party moves for summary judgment on an issue on which it has
the burden of persuasion at trial, it “must support its motion with credible evidence that would entitle it to
a directed verdict if not controverted at trial.” Defendant sought to establish facts supporting the elements
of its affirmative defense by relying on Plaintiff’s deposition testimony. The issue is whether that
testimony establishes that there is no “genuine issue as to any material fact” relating to Defendant’s
affirmative defense and that, accordingly, Defendant should be granted judgment as a matter of law.

Plaintiff admitted that she suffered no adverse job action, that Defendant had policies against sexual
harassment, and that procedures were available to her to complain about harassment. However, by itself
this is not enough to warrant summary judgment, because Plaintiff’s testimony also raised genuine issues
with respect to two facts that are essential to Defendant’s affirmative defense (as described in the facts of
this question): whether Defendant exercised reasonable care to prevent and correct any sexually harassing
behavior and whether Plaintiff unreasonably failed to take advantage of corrective opportunities provided
by Defendant.

Seperac-J19 Exam-Released MEE Essay Compilation © 2016-2020 62


Although part of Plaintiff’s deposition testimony suggests that Defendant did make reasonable efforts to
address harassment by providing procedures that Plaintiff did not use, Plaintiff also testified that
Defendant made no independent effort to enforce its policies (relying on Plaintiff to report or on
supervisors to monitor the situation). In considering this evidence, the court must draw those inferences
“most favorable to the party opposing the motion” for summary judgment. A jury could reasonably find
that this was not reasonable enforcement action for a workplace that had previously not had any women.
Furthermore, Plaintiff testified that she was afraid to report the incidents for fear of retaliation. A jury
might find her fear of retaliation to be a reasonable basis for her failure to use available procedures, given
that the supervisor was himself one of the alleged harassers.

In short, it appears that there are genuine issues about the reasonableness of the employer’s actions to
prevent harassment and the reasonableness of Plaintiff’s failure to report the behavior. Without a
resolution of those factual issues, it is not clear whether Defendant’s affirmative defense is valid, and
summary judgment should not have been granted.

Seperac-J19 Exam-Released MEE Essay Compilation © 2016-2020 63


#013-FEB 2012–MEE Q07: QUESTION SEVEN (CIVIL PROCEDURE/CONFLICTS)

The owner of a rare antique tapestry worth more than $1 million is a citizen of State A. The owner
contacted a restorer, a citizen of State B, to restore the tapestry for $100,000. The owner and the restorer
met in State A and negotiated a contract, but the final documents, prepared by the parties’ respective
attorneys, were drafted and signed in State B. The contract has a forum-selection clause that specifies that
any litigation arising out of or relating to the contract must be commenced in State B.

The restorer repaired the tapestry in State B and then informed the owner that the restoration was
complete. The owner picked up the tapestry and paid the restorer $100,000. Subsequently, the owner
discovered that the restorer had done hardly any work on the tapestry.

Despite the forum-selection clause in the contract, the owner filed suit against the restorer in a state court
in State A, claiming breach of contract. The owner’s suit sought rescission of the contract and a return of
the full contract price – $100,000.

The laws of State A and State B are different on two relevant points. First, State A courts do not enforce
forum-selection clauses that would oust the jurisdiction of State A courts, regarding such clauses as
against public policy; State B courts always enforce forum-selection clauses. Second, State A would
allow contract rescission on these facts; State B would not allow rescission but would allow recovery of
damages.

Under the conflict-of-laws rules of both State A and State B, a state court would apply its own law to
resolve both the forum-selection clause issue and the rescission issue.

After the owner filed suit in State A court, the restorer removed the case to the United States District
Court for the District of State A and then moved for a change of venue to the United States District Court
for the District of State B, citing the contractual forum-selection clause in support of the motion. (There is
only one United States District Court in each state.) The owner moved for remand on the ground that the
federal court did not have removal jurisdiction over the action. Alternatively, the owner argued against the
motion to transfer on the basis that the forum-selection clause was invalid under State A law.

1. Does the federal court in State A have removal jurisdiction over the case? Explain.

2. Should the change-of-venue motion, seeking transfer of the case to the federal court in State B,
be granted? Explain.

3. Would a change of venue affect the law to be applied in resolving the rescission issue? Explain.

Seperac-J19 Exam-Released MEE Essay Compilation © 2016-2020 64


#013: F12-7 MEE: ANSWER: NCBE (CIVIL PROCEDURE/CONFLICTS)

POINT (1) [30%] ISSUE: Does a federal court have removal jurisdiction when the parties to the
state action are diverse, the amount in controversy exceeds $75,000, and the defendant is not from
the state where the action was originally filed? ANSWER: Yes. It was proper to remove the action
from a state court in State A to the federal court in State A because that federal court would have
had diversity jurisdiction over the case had it been filed there originally.

POINT (2) [40%] ISSUE: Should a federal court grant a change of venue pursuant to a forum-
selection clause when the chosen venue is reasonable but the forum-selection clause is invalid under
the law of the state where the case was originally filed? ANSWER: Yes. The change-of-venue
motion is very likely to be granted because State B is a reasonably convenient forum in which to
hear the action, the parties’ contractual agreement to litigate in State B provides a strong reason for
granting a transfer, and the State B federal court would have been an appropriate place in which to
bring the action in the first place.

POINT (3) [30%] ISSUE: When federal jurisdiction is based on diversity, what effect does a change
of venue have on the substantive law that applies? ANSWER: The United States District Court in
State B must apply the law of State A to the transferred action, including State A’s choice-of-law
rules. Because State A would have applied its own rescission law, the federal court in State B must
also apply State A rescission law.

ANSWER DISCUSSION:

Pursuant to 28 U.S.C. § 1441, a case filed in state court is removable by a defendant to federal court if the
case falls within the original jurisdiction of the federal court. Here, the case was properly removed
because it falls within the original diversity jurisdiction of the federal court: the two parties are of diverse
citizenship, and the amount in controversy is in excess of $75,000. Venue changes are proper under 28
U.S.C. § 1404 when they serve “the convenience of parties and witnesses” and are “in the interest of
justice.” Here, State B is arguably a more convenient forum for the litigation. Moreover, the Supreme
Court has ruled that the “interest of justice” language in the statute provides a basis for a federal policy of
enforcement of forum-selection clauses through venue transfers. Finally, venue in State B is proper under
28 U.S.C. § 1391, because State B is a place where all defendants reside, and also under § 1391(a)(2),
because a “substantial part” of the events relating to the dispute occurred in State B. When a case is
transferred under § 1404, the transferee court must apply the law of the original transferor forum,
including that state’s choice-of-law rules. Here, because the facts say that State A courts would apply
State A law to the case, the federal court in State A is required to do so, and that law also must be applied
in the transferee State B federal court. Therefore, the change of venue would not affect what law applies,
and therefore contract rescission will be allowed.

ANSWER EXPLANATION:

Explanation to Point-One (30%):

It was proper to remove the action from a state court in State A to the federal court in State A because that
federal court would have had diversity jurisdiction over the case had it been filed there originally.

Seperac-J19 Exam-Released MEE Essay Compilation © 2016-2020 65


A defendant may remove a civil action from state court to federal court if the action is a type over “which
the district courts of the United States have original jurisdiction.” In this case, if the action had been filed
originally in a federal court, it would have been within the diversity jurisdiction of the federal court.

Federal diversity jurisdiction extends to “all civil actions where the matter in controversy exceeds
$75,000, and which are between citizens of different States.” The restorer and owner are citizens of
different states, so the diversity requirement is satisfied. Furthermore, the remedy sought by the owner – a
return of the $100,000 contract price – exceeds the jurisdictional amount of $75,000. It does not matter
that the owner may recover less than this: a plaintiff’s claim of damages in excess of the jurisdictional
amount will be accepted unless it “appears to a legal certainty that the claim is really for less than the
jurisdictional amount.” Here the facts relating to performance of the restoration contract, the amount paid
on that contract, and the amount sought in rescission are adequate to avoid any suggestion that the
recoverable damages are less than the jurisdictional amount “to a legal certainty.”

Accordingly, because the case falls within the federal court’s diversity jurisdiction, the federal court had
removal jurisdiction over the action.

[Note: (1) Removal itself does not raise any personal jurisdiction issues, and discussions of personal
jurisdiction are not relevant to the question asked, which is whether the federal court has removal
jurisdiction. (2) In actions where federal jurisdiction is based on diversity, a defendant cannot remove the
case to federal court if the case was filed in a state court in the defendant’s home state. That bar to
removal is inapplicable on our facts because State A is not the defendant’s (the restorer’s) home state.]

Explanation to Point-Two (40%):

The change-of-venue motion is very likely to be granted because State B is a reasonably convenient
forum in which to hear the action, the parties’ contractual agreement to litigate in State B provides a
strong reason for granting a transfer, and the State B federal court would have been an appropriate place
in which to bring the action in the first place.

The motion for a change of venue is governed by 28 U.S.C. § 1404, which provides in its entirety, “For
the convenience of parties and witnesses, in the interest of justice, a district court may transfer any civil
action to any other district or division where it might have been brought.” Transfer pursuant to the
“convenience and interest of justice” standard of § 1404 is discretionary (“may”), but transfer under §
1404 may be made only to a district in which the action might originally “have been brought.”

In this case, the action originally could have been brought in a federal district court in State B. First, a
State B federal court would have had diversity jurisdiction over the action. Second, a State B federal court
would have been an appropriate venue for the action. Under 28 U.S.C. § 1391(a)(1), venue is proper in a
district where “any defendant resides, if all defendants reside in the same State.” On that basis, State B is
a proper original venue simply because the restorer resides there. In addition, venue is proper in State B
under § 1391(a)(2), which permits venue in any judicial district “in which a substantial part of the events
or omissions giving rise to the claim occurred.” The contract was signed in State B, and the restorer’s
work on the tapestry was performed there. Finally, because the restorer is a resident citizen of State B, the
State B federal court would have had jurisdiction over him. In short, the action could originally have been
brought in State B.

The question, therefore, is whether convenience and the “interest of justice” favor a transfer from State A
to State B. Absent the forum-selection clause, the issue could be argued either way. Arguably, most of the
evidence and witnesses in the case would be found in State B, which is where the work on the tapestry

Seperac-J19 Exam-Released MEE Essay Compilation © 2016-2020 66


was performed and where the contract was drafted and signed. On the other hand, State A is the owner’s
home (and a convenient place for the owner to litigate), and it is also connected to the transaction – it was
the situs of the negotiations. Judges have a good deal of discretion under § 1404, but normally a judge
would allow a plaintiff’s original venue choice to prevail where the facts are closely divided on the issues
of convenience and interest of justice.

However, the presence of the forum-selection clause alters this analysis. In Stewart Organization, Inc. v.
Ricoh Corp., the Supreme Court ruled that a reasonable forum-selection clause is a relevant factor
favoring a § 1404 change-of-venue motion, even if the forum-selection clause is unenforceable under the
applicable state law. According to the Supreme Court, the presence of a reasonable forum-selection clause
is an important consideration relevant to the “interest of justice” determination under § 1404. In this case,
there are several connections between the transaction, the parties, and State B, and the parties’ selection of
a State B forum was certainly reasonable.

In sum, State B is an appropriate venue under the federal venue statute, there are several important
connections between the case and State B, and the forum-selection clause provides a strong reason to
grant the transfer. Therefore, the State A federal district court should grant the motion for change of
venue.

Explanation to Point-Three (30%):

The United States District Court in State B must apply the law of State A to the transferred action,
including State A’s choice-of-law rules. Because State A would have applied its own rescission law, the
federal court in State B must also apply State A rescission law.

In this diversity case based on a state law breach-of-contract claim, the Erie rule would have required the
State A federal court to apply state substantive law. Moreover, for Erie purposes, the state substantive law
includes any choice-of-law rules that a state court would apply as part of state law. In this case, the facts
state that State A choice-of-law rules would lead a State A court to apply its own rescission law.

Following a § 1404 transfer, the transferee court must apply the same law to the case as would have been
applied by the transferor court. Thus, the State B federal court must apply whatever law would have been
applied by the State A federal court. In short, the change of venue does not affect the law that will be
applied to the substantive issues in the case.

Therefore, whether the case is heard in federal court in State A or federal court in State B, State A
rescission law will be applied.

Seperac-J19 Exam-Released MEE Essay Compilation © 2016-2020 67


#014-JUL 2011–MEE Q05: QUESTION FIVE (CIVIL PROCEDURE)

OfficeEquip is a U.S. distributor of office machines. It is incorporated in State A, where it has its
principal place of business. BritCo is a manufacturer of copiers. It is incorporated in Scotland and has its
principal place of business in London, England. OfficeEquip sued BritCo, alleging that BritCo had
breached a long-term contract to supply copiers to OfficeEquip.

The suit was filed in the United States District Court for State A, and OfficeEquip properly invoked the
court’s diversity (alienage) jurisdiction.

BritCo made a timely motion to dismiss the complaint on the ground that it was filed in violation of a
forum-selection clause in the supply contract that required all contract disputes to be adjudicated in
London. While its motion to dismiss was pending, BritCo filed an answer to the complaint.

In its answer, BritCo denied breaching the supply contract. BritCo also made a counterclaim seeking
damages for OfficeEquip’s alleged breach of a contractual covenant not to compete with BritCo.

OfficeEquip filed a motion for judgment on the pleadings on BritCo’s counterclaim, arguing that the
covenant not to compete was unenforceable as a matter of law.

After a short period of discovery, the district judge issued the following two orders:

OfficeEquip’s motion for judgment on the pleadings is granted. The contractual covenant not to compete
is void as a matter of public policy and is therefore unenforceable. Given that this is strictly a legal issue
and entirely severable from OfficeEquip’s breach of contract claim, there is no just reason for delay, and I
accordingly direct that judgment should be entered in favor of OfficeEquip on BritCo’s counterclaim.

BritCo’s motion to dismiss is denied. Enforcement of the forum-selection clause would be unreasonable
in this case. OfficeEquip has never done business in London, and it would be extremely inconvenient for
it to litigate there.

Trial on the breach of contract claim is scheduled in three months.

1. Can BritCo immediately appeal the district court’s order granting OfficeEquip’s motion for
judgment on the pleadings with respect to BritCo’s counterclaim? Explain.

2. Can BritCo immediately appeal the district court’s order denying its motion to dismiss? Explain.

Seperac-J19 Exam-Released MEE Essay Compilation © 2016-2020 68


#014: J11-5 MEE: ANSWER: NCBE (CIVIL PROCEDURE)

POINT (1) [45%] ISSUE: Is a district court’s grant of judgment on the pleadings with respect to a
counterclaim a “final decision” subject to immediate appeal when the district court also orders
judgment to be entered on the counterclaim? ANSWER: Yes. The district court’s grant of
judgment on the pleadings with respect to BritCo’s counterclaim is immediately appealable because
the district court judge stated that there was no just reason for delay and ordered entry of
judgment on the counterclaim.

POINT (2) [40%] ISSUE: Is a district court’s refusal to enforce a contractual forum-selection
clause a “final decision” subject to immediate appeal? ANSWER: No. The district court’s refusal to
enforce the forum-selection clause is not an appealable final judgment.

POINT (3) [10%] ISSUE: Does a district court’s refusal to enforce a contractual forum-selection
clause fall within any of the exceptions to the final-judgment rule? ANSWER: No. A district court’s
order refusing to dismiss a case pursuant to a contractual forum-selection clause is not immediately
appealable under the judge-made collateral-order doctrine or pursuant to any other exception to
the final-judgment rule.

POINT (4) [5%] ISSUE: If a district court makes several orders in a case and one of those orders is
appealable, may an appellate court review the non-appealable orders on grounds of efficiency?
ANSWER: No. The doctrine of pendent appellate jurisdiction would not authorize the appellate
court to hear an appeal from the forum-selection decision, even though the court would have
jurisdiction over another decision in the same case.

ANSWER DISCUSSION:

BritCo can appeal immediately from the district court’s decision on BritCo’s counterclaim. Where there
are multiple claims or counterclaims in a case, a final disposition of one of those claims will be appealable
when, as here, the trial court orders entry of judgment on that claim and certifies that there is no just
reason for delay. BritCo cannot appeal immediately from the court’s refusal to enforce the forum-
selection clause. The court’s decision on this issue did not end litigation on the merits of the case and was
therefore not an appealable final judgment. In addition, the court’s forum-selection order is not appealable
under the collateral-order doctrine. Assuming the lower court’s decision on the forum-selection clause
was incorrect, immediate appeal is not necessary to vindicate BritCo’s rights under the contractual forum-
selection clause. Those rights can be protected after final judgment, by striking down any judgment issued
in violation of the forum-selection clause. Nor do the facts of this problem present the kind of
extraordinary abuse of power that would prompt an appellate court to review a lower court’s action by
writ of mandamus. Even though the appellate court would have jurisdiction over another decision in the
same case, considerations of efficiency do not give the court the power to hear an appeal from the lower
court’s decision on the forum-selection issue.

ANSWER EXPLANATION:

Explanation to Point-One (45%):

Seperac-J19 Exam-Released MEE Essay Compilation © 2016-2020 69


The district court’s grant of judgment on the pleadings with respect to BritCo’s counterclaim is
immediately appealable because the district court judge stated that there was no just reason for delay and
ordered entry of judgment on the counterclaim.

In the federal courts, litigants ordinarily may appeal only from final judgments of the district courts. A
final judgment is one that “ends the litigation on the merits and leaves nothing for the court to do but
execute the judgment.”

When, as in this problem, more than one claim for relief is presented in an action, Federal Rule of Civil
Procedure 54 provides that a decision that “adjudicates fewer than all the claims” in the action “does not
end the action as to any of the claims or parties.” Under this default rule, a grant of judgment on one of
several claims in an action is not a final judgment and is not subject to immediate appeal.

However, Rule 54 permits district courts to reverse the default rule and to designate their resolutions of
particular claims as immediately appealable “final judgments” even when there are other claims in the
action that have not yet been decided. A district court may “direct entry of a final judgment” as to a claim
if the district court “expressly determines that there is no just reason for delay.” This procedure “does not
relax the finality required of each decision, as an individual claim, to render it appealable, but it does
provide a practical means of permitting an appeal to be taken from one or more final decisions on
individual claims, in multiple claims actions, without waiting for final decisions to be rendered on all the
claims in the case.”

In this problem, multiple claims for relief were presented by the parties – OfficeEquip sued for breach of
contract and BritCo counterclaimed for breach of the non-competition clause in the contract. When the
district court granted judgment on the pleadings for OfficeEquip on BritCo’s counterclaim, the court also
expressly directed entry of judgment and expressly determined that there was no reason for delay. Thus,
under Rule 54, the district court’s decision constituted a “final judgment” on the counterclaim and is
immediately appealable.

Explanation to Point-Two (40%):

The district court’s refusal to enforce the forum-selection clause is not an appealable final judgment.

The district court’s refusal to enforce the forum-selection clause did not resolve any of the claims in the
action. Far from “ending the litigation on the merits,” the court’s decision allowed the litigation to go
forward on the merits. Under the final-judgment rule, this is a non-appealable interlocutory order.

Although there are certain exceptions to the final-judgment rule that BritCo could invoke in an attempt to
secure immediate review of the court’s decision, none of the exceptions would be applicable on the facts
of this problem.

Explanation to Point-Three (10%):

A district court’s order refusing to dismiss a case pursuant to a contractual forum-selection clause is not
immediately appealable under the judge-made collateral-order doctrine or pursuant to any other exception
to the final-judgment rule.

a. Collateral-order doctrine. In Cohen v. Beneficial Industrial Loan Corp., the U.S. Supreme Court created
the so-called “collateral order” doctrine as an exception to the final-judgment rule. Under the collateral-
order doctrine, an immediate appeal may be taken from an interlocutory order that conclusively

Seperac-J19 Exam-Released MEE Essay Compilation © 2016-2020 70


determines an important issue in a case if that issue is independent of (collateral to) the merits of the
action and if a delay in appellate review would effectively preclude the losing party from an opportunity
to vindicate its rights on appeal.

The collateral-order doctrine does not apply in this case because an immediate appeal is not necessary to
preserve BritCo’s contractual right to have contract disputes adjudicated in London. If the lower court’s
ruling on the forum-selection clause is wrong, BritCo’s contractual rights can be vindicated on an appeal
from a final judgment in the case. An appellate court could order dismissal of the case at that time, thus
forcing OfficeEquip to pursue its claim in the appropriate court. The collateral-order doctrine does not
permit immediate appeal from an order denying enforcement of a contractual forum-selection clause
because the right to have “binding adjudication” only in a particular forum can be vindicated on appeal
from final judgment by reversing a judgment entered in violation of that right.

b. Mandamus or prohibition. Another means of obtaining immediate review of interlocutory orders is to


seek an extraordinary writ (mandamus or prohibition) from the appellate court reversing the trial court’s
ruling. However, an appellate court is highly unlikely to grant an extraordinary writ in a case of this sort.

Mandamus and prohibition are available to allow appellate courts “to confine trial judges to the lawful
exercise of their jurisdiction or to compel them to act if they have abdicated their jurisdictional
obligations.” Although courts often favor the enforcement of forum-selection clauses, their enforceability
is not guaranteed and ultimately depends on the terms of the clause, the reasons for its adoption, and its
impact on the parties to the litigation. For example, a court enforced a forum-selection clause in a cruise
line ticket on the basis that the clause was an efficient tool for determining the location of litigation in the
event of an accident, use of clause avoids litigation in multiple forums, and plaintiffs did not satisfy their
burden of establishing serious inconvenience if the case were heard in the chosen forum. Given that the
writs of mandamus and prohibition are extraordinary writs to be used only in exceptional circumstances, it
is highly unlikely that an appellate court would grant a writ to exercise immediate review over a district
court’s judgment about such matters.

[NOTES: (1) TECHNICALLY, THE WRIT THAT WOULD BE APPROPRIATELY


SOUGHT IN THIS CASE IS A WRIT OF PROHIBITION. But examinees who mention
either writ should receive credit. (2) There are other statutory exceptions to the final-judgment rule, but
none of them apply to this problem and you should not expect examinees to discuss them. For example,
the immediate appealability of certain orders relating to injunctions, receiverships, and admiralty and the
appealability of interlocutory orders when a district judge certifies certain grounds for immediate
appeal.]

Explanation to Point-Four (5%):

The doctrine of pendent appellate jurisdiction would not authorize the appellate court to hear an appeal
from the forum-selection decision, even though the court would have jurisdiction over another decision in
the same case.

Otherwise non-appealable rulings are sometimes reviewed by appellate courts under the doctrine of
“pendent appellate jurisdiction.” The argument for pendent appellate jurisdiction is that the final-
judgment rule “is designed to prevent parties from interrupting litigation by pursuing piecemeal appeals.
Once litigation has already been interrupted by an authorized pretrial appeal, there is no cause to resist the
economy that pendent appellate jurisdiction promotes,” and a party should be able immediately to appeal
any decisions in the case that can conveniently be reviewed along with the appealable order. BritCo might

Seperac-J19 Exam-Released MEE Essay Compilation © 2016-2020 71


argue persuasively that judicial economy would be served if the appellate court would hear its appeal
from the order denying enforcement of the forum-selection clause at the same time the court hears its
appeal from the judgment on the pleadings order.

However, pendent appellate jurisdiction would not be available in this case. Although the U.S. Supreme
Court has implied that the exercise of pendent appellate jurisdiction might sometimes be appropriate, it
has also suggested that the scope of such jurisdiction should be narrowly confined. The appellate courts
are bound by 28 U.S.C. § 1291, which limits their basic appellate jurisdiction to “final decisions,” and
reasons of judicial economy alone would not justify a court in expanding its jurisdiction to cover non-
final orders. Rather, a party seeking review of a non-final order on the ground that it is pendent to an
appealable order would need to show that the two rulings were “inextricably intertwined” or that
“meaningful review” of the ruling over which the court has jurisdiction would not be possible unless the
court also reviewed the otherwise non-appealable ruling.

The forum-selection ruling and the ruling denying BritCo’s counterclaim are not inextricably intertwined.
Moreover, meaningful review of the counterclaim issue is possible without any consideration of the
forum-selection question. Consequently, pendent appellate jurisdiction is not available in this case.

[NOTE: Even well-prepared examinees may be unfamiliar with the concept of pendent appellate
jurisdiction. While the Supreme Court has mentioned the concept and it may be a useful label for some
decisions, the Court has not expressly developed a doctrine of “pendent appellate jurisdiction.”
Nonetheless, examinees should be expected to spot the issue – raised by these facts – of how an appellate
court ought to respond when a litigant seeks review of multiple orders of a lower court, some of which are
appealable orders and some of which are not. What is important is that examinees recognize that there is
a judicial economy argument in favor of reviewing all the orders simultaneously, but that the judicial
economy argument fails because of the final-judgment rule, unless some special reason can be shown why
simultaneous review will contribute to the court’s ability to review the appealable order. Examinees who
spot the problem and understand that judicial economy alone is not grounds for review of the forum-
selection clause decision should receive full credit for this portion of the analysis, even if they make no
mention of pendent appellate jurisdiction.]

Seperac-J19 Exam-Released MEE Essay Compilation © 2016-2020 72


#015-FEB 2011–MEE Q08: QUESTION EIGHT (CIVIL PROCEDURE)

Plaintiff, a citizen of State B, was vacationing in State A, where he visited the O.K. Bar. While he was at
the bar, Plaintiff was attacked and seriously beaten by Dave, a regular bar patron and a citizen of State A.
Bartender, a citizen of State A, attempted to stop the attack and was also injured by Dave.

Plaintiff sued Dave and Bartender in the United States District Court for the District of State A, properly
invoking the court’s diversity jurisdiction. Plaintiff’s complaint states a state law battery claim against
Dave, seeking damages from Dave in excess of $75,000. Plaintiff’s complaint also states a claim against
Bartender based on Bartender’s alleged negligence in serving alcohol to Dave after Dave became visibly
intoxicated and belligerent. Plaintiff’s complaint seeks damages from Bartender in excess of $75,000.
Plaintiff’s damages claims are reasonable in light of the injuries Plaintiff suffered in the attack.

Dave was personally served with the summons and complaint. However, the process server could not find
Bartender. He therefore taped the summons and complaint to the front door of the O.K. Bar, where
Bartender found them the next day.

Bartender made a timely motion to dismiss Plaintiff’s complaint for failure to state a cause of action.
When that motion was denied by the district court judge, Bartender filed a second motion to dismiss for
insufficiency of service of process. The judge also denied that motion.

Bartender then filed an answer to the complaint, denying liability. The answer also stated a state law
claim for battery against Dave, seeking $20,000 damages for the injuries Bartender suffered when he tried
to stop Dave’s attack on Plaintiff.

Dave has moved to dismiss Bartender’s cross-claim on the grounds of improper joinder and lack of
subject-matter jurisdiction.

1. Did the United States District Court for the District of State A properly deny Bartender’s motion
to dismiss for insufficiency of service of process? Explain.

2. Do the Federal Rules of Civil Procedure permit Bartender to join a claim for battery against
Dave in Bartender’s answer to Plaintiff’s complaint? Explain.

3. Assuming that the Federal Rules of Civil Procedure permit Bartender to join his state law claim
against Dave, does the United States District Court for the District of State A have subject-matter
jurisdiction over that claim? Explain.

Seperac-J19 Exam-Released MEE Essay Compilation © 2016-2020 73


#015: F11-8 MEE: ANSWER: NCBE (CIVIL PROCEDURE)

POINT (1) [30%] ISSUE: May a defendant make a motion to dismiss for insufficiency of service of
process when the defendant has already moved to dismiss for failure to state a claim upon which
relief can be granted? ANSWER: Yes. The court correctly denied Bartender’s motion to dismiss
for insufficiency of service of process because that defense was not raised when Bartender made his
motion to dismiss for failure to state a claim.

POINT (2) [30%] ISSUE: Do the Federal Rules of Civil Procedure permit a defendant to join a
cross-claim against a co-defendant if it arises out of the same events as the plaintiff’s claim against
the defendants? ANSWER: Yes. Rule 13(g) of the Federal Rules of Civil Procedure permits a
defendant, in the answer, to state a cross-claim against a co-defendant if it arises out of the same
transaction or occurrence as the original claim.

POINT (3)(a) [20%] ISSUE: Does a federal district court have independent subject-matter
jurisdiction over a state law cross-claim for $20,000 between two defendants who are citizens of the
same state? ANSWER: No. A federal district court would not have independent subject-matter
jurisdiction over a state law cross-claim for $20,000 between two defendants who are citizens of the
same state because the claim does not arise under federal law, the claimants are not diverse, and the
amount-in-controversy requirement for diversity cases is not satisfied.

POINT (3)(b) [20%] ISSUE: Does a federal district court have supplemental subject-matter
jurisdiction over a state law cross-claim for $20,000 between two defendants who are citizens of the
same state? ANSWER: Yes. A federal district court would have supplemental subject-matter
jurisdiction over a state law cross-claim for $20,000 between citizens of the same state as long as it is
so related to the plaintiff’s claim against the defendants, of which the court has original jurisdiction,
that the claims form part of the “same case or controversy” under Article III of the Constitution.

ANSWER DISCUSSION:

Because Bartender made a motion to dismiss the action against him for failure to state a cause of action
without raising his defense of insufficiency of service of process, he was barred under FRCP rules from
raising that defense at a later time. The court therefore was correct in denying his motion, even if
Bartender had a valid claim that the manner of service was improper. SEPERAC NOTE: Recently, courts
have allowed a motion to dismiss to be amended before the motion is heard, so long as the adverse party
is not prejudiced by the amendment and no delay results in the prosecution and determination of the case.
Here, this is not applicable, because the court ruled on the first motion, which implies that the other party
responded to it. However, keep this in mind and mention it should a similar issue come up.Bartender’s
claim against Dave arises out of the same transaction or occurrence as Plaintiff’s claim. The claim
therefore qualifies as a cross-claim under Rule 13(g) of the Federal Rules of Civil Procedure, and
Bartender may state that claim in his answer to the complaint. A federal court would not have subject-
matter jurisdiction over Bartender’s claim against Dave if that claim were filed as an independent action.
However, Plaintiff’s original claim and Bartender’s cross-claim are sufficiently closely related to form
part of the same case or controversy under Article III of the United States Constitution. Accordingly, the
supplemental jurisdiction statute authorizes the federal court to take jurisdiction over Bartender’s cross-
claim.

ANSWER EXPLANATION:

Seperac-J19 Exam-Released MEE Essay Compilation © 2016-2020 74


Explanation to Point-One (30%):

The court correctly denied Bartender’s motion to dismiss for insufficiency of service of process because
that defense was not raised when Bartender made his motion to dismiss for failure to state a claim.

Although it is likely that service of process was improper in this case, the district court nonetheless was
correct to deny Bartender’s motion to dismiss on that basis. Bartender initially responded to Plaintiff’s
complaint by making a motion to dismiss for failure to state a cause of action. When that motion was
denied, Bartender moved to dismiss for insufficiency of service of process.

When a motion pursuant to Rule 12(b) is initially made, a party may join with that motion all other
motions permitted by Rule 12 that are then available to the party. If a party makes a Rule 12(b) motion
and fails to join certain other available defenses or objections, the party may not later raise those
objections and defenses. In particular, a party that makes a Rule 12(b) motion but omits from that motion
a defense of insufficiency of service of process is deemed to have waived that defense.

Because Bartender made a Rule 12(b) motion to dismiss for failure to state a claim but did not join his
available defense of insufficiency of service of process, Bartender waived that defense. Hence, the district
court was correct to deny Bartender’s motion to dismiss on that ground. It does not matter that Bartender
appears to have had a valid claim that service was improper.

NOTE: Generally, when a party makes a pre-answer motion under Federal Rule of Civil Procedure 12, the
party must raise any claim of insufficiency of service of process that the party has at the time of the
motion; otherwise, the defense is waived. Rule 12 states that “a party that makes a motion under this rule
must not make another motion under this rule raising a defense or objection that was available to the party
but omitted from its earlier motion.” Further, “a party waives any defense listed in rule 12(b)(2)–(5) by
omitting it from a motion in the circumstances described in Rule 12(g)(2).” Nonetheless, courts have
allowed a motion to dismiss to be amended before the motion is heard, so long as the adverse party is not
prejudiced by the amendment and no delay results in the prosecution and determination of the case. Thus,
although not expressly provided for in Federal Rule 12(g), a preliminary motion may be amended to
include a defense or objection inadvertently omitted by the movant if there is no indication that the
proceedings will be delayed or the opposing party will be prejudiced. Here, this is not applicable, because
the court ruled on the first motion, which implies that the other party responded to it. However, keep this
in mind and mention it should a similar issue come up.

Explanation to Point-Two (30%):

Rule 13(g) of the Federal Rules of Civil Procedure permits a defendant, in the answer, to state a cross-
claim against a co-defendant if it arises out of the same transaction or occurrence as the original claim.

Rule 13(g) of the Federal Rules of Civil Procedure provides that a defendant’s answer may state as a
cross-claim against a co-defendant any claim that “arises out of the transaction or occurrence that is the
subject matter of the original action” between the plaintiff and the defendants. The policy underlying Rule
13(g) is efficiency; multiple lawsuits are avoided if defendants bring related cross-claims in the context of
the plaintiff’s suit.

Here, Plaintiff’s claims concern Dave’s injury of Plaintiff and Bartender’s service of alcohol to Dave.
Bartender’s cross-claim concerns Dave’s injury of Bartender. The question is whether these various
claims arise out of the same transaction or occurrence.

Seperac-J19 Exam-Released MEE Essay Compilation © 2016-2020 75


To determine whether two different claims arise out of the same “transaction or occurrence” for purposes
of Rule 13, federal courts look at a number of factors: (1) whether the issues of fact and law in the
plaintiff’s claim and the defendant’s cross-claim are essentially the same, (2) whether the same evidence
would support or refute the plaintiff’s claim and the defendant’s cross-claim, (3) whether there is a logical
relationship between the plaintiff’s claim and the defendant’s cross-claim, and (4) whether res judicata
would bar a subsequent suit on the cross-claim. The presence of any of these factors supports a conclusion
that the “transaction or occurrence” requirement is met.

The most frequently considered factor is whether there is a logical relationship between the defendant’s
cross-claim and the plaintiff’s original claim. Here, the alleged batteries of Plaintiff and Bartender
occurred in the same place, within seconds of one another. Bartender’s efforts to intercede in the initial
battery presumably prompted Dave to injure Bartender. There appears to be a causal link, and thus a
logical relationship, between the two claims.

In addition, the two claims involve almost identical factual and legal issues. Both Plaintiff’s claim against
Dave and Bartender’s cross-claim against Dave are claims for battery arising under State A’s law. Both
claims involve factual questions about the cause and course of the fight in which Dave, Bartender, and
Plaintiff were involved. Even though the factual issues may differ – whether Dave battered Plaintiff is a
distinct question from whether Dave battered Bartender – the substantial overlap between the legal and
factual issues supports the conclusion that the claims arise from the same transaction or occurrence. The
identity-of-issues inquiry does not require a complete overlap between the claim and the cross-claim.

The “same evidence” test is also likely to be satisfied. To prove the battery claim against Dave, Plaintiff
will need the testimony of witnesses to the brawl. It is likely that the patrons of the bar who witnessed
Dave’s alleged battery of Plaintiff also witnessed Dave’s alleged battery of Bartender. Thus, a substantial
amount of the evidence (i.e., the testimony of the witnesses to the event) will be the same.

The only factor that militates against allowing Bartender to bring a cross-claim is that a judgment on
Plaintiff’s claim against Dave would not preclude Bartender, who is not in privity with either Dave or
Plaintiff, from suing on his own claim. Denying Bartender the ability to bring the claim would not prevent
him from having his case heard at some future point. However, given that Bartender’s claim against Dave
is logically related to Plaintiff’s claims, that it raises overlapping factual and legal issues, and that the
same evidence is relevant to both claims, a court should allow Bartender’s claim to be included as a cross-
claim in Bartender’s answer.

Explanation to Point-Three(a) (20%):

A federal district court would not have independent subject-matter jurisdiction over a state law cross-
claim for $20,000 between two defendants who are citizens of the same state because the claim does not
arise under federal law, the claimants are not diverse, and the amount-in-controversy requirement for
diversity cases is not satisfied.

The federal question statute grants the federal district courts original jurisdiction “of all civil actions
arising under the Constitution, laws, or treaties of the United States.” A claim arises under federal law if
federal law creates the cause of action. Here, state law creates Bartender’s battery claim against Dave.
Under this test, Bartender’s cross-claim does not arise under federal law, and therefore the court would
not have jurisdiction under the federal question statute,

Seperac-J19 Exam-Released MEE Essay Compilation © 2016-2020 76


Nor would a federal court have diversity jurisdiction over Bartender’s claim. First, there is no diversity.
Bartender and Dave are both citizens of State A. Second, Bartender is seeking damages of only $20,000,
well below the $75,000 jurisdictional minimum in diversity actions.

Explanation to Point-Three(b) (20%):

A federal district court would have supplemental subject-matter jurisdiction over a state law cross-claim
for $20,000 between citizens of the same state as long as it is so related to the plaintiff’s claim against the
defendants, of which the court has original jurisdiction, that the claims form part of the “same case or
controversy” under Article III of the Constitution.

In the instant case, the federal district court has original diversity jurisdiction over Plaintiff’s claims
against Bartender and Dave. Each claim is for an amount in excess of $75,000, and Plaintiff, a citizen of
State B, is diverse from both defendants, who are citizens of State A.

The supplemental jurisdiction statute permits federal district courts that have original jurisdiction of a
civil action to assert “supplemental jurisdiction over all other claims that are so related to claims in the
action within such original jurisdiction that they form part of the same case or controversy under Article
III of the United States Constitution.” Claims form part of the “same case or controversy” under Article
III if they “derive from a common nucleus of operative fact.” Cross-claims that satisfy Rule 13(g)’s
“transaction or occurrence” test presumptively satisfy the Gibbs “common nucleus of operative fact” test
and are within the court’s supplemental jurisdiction. As noted in Point One, Bartender’s claim and
Plaintiff’s claims arise out of the same occurrence, raise many of the same factual and legal issues, and
will be proved, in part, by the same evidence. These claims clearly arise out of a “common nucleus of
operative fact” – the bar fight that gave rise to all the claims in the case.

Because Bartender’s claim is part of the “same case or controversy” as Plaintiff’s original claims, it falls
within the federal court’s supplemental jurisdiction.

Seperac-J19 Exam-Released MEE Essay Compilation © 2016-2020 77


#016-JUL 2010–MEE Q07: QUESTION SEVEN (CIVIL PROCEDURE)

Until recently, Paul had always lived in State A. Last year, he decided he would move to State B for at
least one year and, after a year, decide whether to remain in State B or return to State A. Six months ago,
Paul moved to State B, rented an apartment, and took a job as a temporary employee. Paul has enjoyed
living in State B so much that he recently left his temporary job and accepted a position as a permanent
employee at a law firm in State B.

Shortly after he moved to State B, Paul bought a vacation home in State A, which he visits about once a
month for two or three days. To pay for the vacation home, Paul obtained a loan from Credit Union in
State A. Credit Union is incorporated in and chartered by State A. Its only office, located in State A, is
both its corporate headquarters and the place where it transacts business with its customers. Ninety-five
percent of Credit Union’s customers are State A residents who do business with Credit Union in person at
its State A office.

Paul’s loan agreement with Credit Union provides that he will repay the loan in monthly installments over
a 30-year period. Credit Union has a mortgage on Paul’s vacation home to secure the debt. The loan
paperwork lists Paul’s State B address as his mailing and home address. The loan agreement also contains
a privacy provision whereby Credit Union agrees not to disclose Paul’s personal information to any third
party without Paul’s written permission. Credit Union sends a loan statement and payment coupon to
Paul’s State B address each month, and Paul returns the payment coupon with a check for the payment
amount.

After the loan closed, a Credit Union employee mailed copies of all the loan paperwork to Paul.
Unfortunately, the employee misread Paul’s address in State B and sent the paperwork to an incorrect
address. Several months later, Paul discovered that someone had gotten his loan paperwork and had used
the information (including Paul’s Social Security number and credit card numbers) to steal his identity.
The identity thief had quickly accumulated $150,000 in unpaid bills in Paul’s name. Paul’s credit rating
was ruined, and no one would extend him new credit.

Paul has sued Credit Union in the United States District Court for the District of State B for breach of the
privacy provisions of the loan contract. The parties have stipulated that Paul’s actual loss was $80,000.
Paul’s suit seeks $240,000 in damages, plus attorney’s fees, pursuant to a State A statute that entitles
victims of identity theft to recover treble damages and attorney’s fees from anyone who wrongfully
discloses their personal information. Paul’s complaint also asserts that a federal statute restricting
damages in state-law identity-theft cases to actual damages is unconstitutional and therefore does not
preempt the treble damages provisions of the State A statute. The complaint asserts that the State B
federal court has both diversity and federal-question jurisdiction over the case.

The long-arm statute of State B extends personal jurisdiction as far as the Constitution allows.

1. May the United States District Court for the District of State B exercise personal jurisdiction
over Credit Union? Explain.

2. Does the United States District Court for the District of State B have diversity jurisdiction over
the case? Explain.

Seperac-J19 Exam-Released MEE Essay Compilation © 2016-2020 78


3. Does the United States District Court for the District of State B have federal-question
jurisdiction over the case? Explain.

Seperac-J19 Exam-Released MEE Essay Compilation © 2016-2020 79


#016: J10-7 MEE: ANSWER: NCBE (CIVIL PROCEDURE)

POINT (1) [35%] ISSUE: May a federal district court exercise personal jurisdiction over a
nonresident defendant that contracted outside the forum state to provide a loan to a plaintiff who
lived in the forum state and that mailed loan paperwork to the plaintiff in the forum state,
including the incorrectly mailed paperwork that gave rise to the plaintiff's lawsuit? ANSWER:
Yes. The United States District Court for the District of State B may exercise personal jurisdiction
over Credit Union because Credit Union’s contacts with the forum state gave rise to the suit and
demonstrate purposeful availment and because the exercise of jurisdiction would not be unfair.

POINT (2) [35%] ISSUE: Does a federal district court have diversity jurisdiction when the plaintiff
previously was domiciled in the defendant’s state of citizenship and still returns there about once a
month, but has moved to another state to work and may intend to stay there permanently?
ANSWER: Credit Union is a citizen of State A. As a result, complete diversity will exist if Paul is a
domiciliary of State B. However, it is uncertain whether Paul is a domiciliary of State B or is still a
domiciliary of State A. If Paul is a State B domiciliary, then there is diversity jurisdiction; Paul’s
claim satisfies the amount-in-controversy requirement.

POINT (3) [30%] ISSUE: Does a federal district court have federal-question jurisdiction over a
lawsuit when the plaintiff’s complaint refers to a potentially relevant federal statute but denies the
applicability of the statute? ANSWER: No. Paul’s assertion of a defense to the possible applicability
of a federal statute does not create federal-question jurisdiction.

ANSWER DISCUSSION:

The State B district court may exercise specific personal jurisdiction over nonresident defendant Credit
Union because Credit Union engaged in contacts with the forum state that demonstrated purposeful
availment and that gave rise to Paul’s claims. The exercise of personal jurisdiction would not be unfair.
Diversity of citizenship exists if Paul intends to remain in State B, which could be argued either way.
Defendant Credit Union is a citizen of State A, where it is incorporated and has its principal place of
business. Paul’s complaint seeks sufficient monetary damages to satisfy the amount-in-controversy
requirement. The district court does not have federal-question jurisdiction in this case because Paul’s
complaint does not state a federal claim. It instead seeks to anticipate and rebut a potential federal
defense. Under the well-pleaded complaint rule, federal-question jurisdiction is appropriate only when a
federal claim is raised on the face of the complaint.

ANSWER EXPLANATION:

Explanation to Point-One (35%):

The United States District Court for the District of State B may exercise personal jurisdiction over Credit
Union because Credit Union’s contacts with the forum state gave rise to the suit and demonstrate
purposeful availment and because the exercise of jurisdiction would not be unfair.

The United States District Court for the District of State B has personal jurisdiction over Credit Union.
Federal district courts may exercise personal jurisdiction to the same extent as the courts of general
jurisdiction of the state in which the district court sits. State courts of general jurisdiction may exercise
personal jurisdiction over nonresident defendants to the extent authorized by both the state’s long-arm

Seperac-J19 Exam-Released MEE Essay Compilation © 2016-2020 80


statute and the due process clause of the 14th Amendment of the U.S. Constitution. In this case, State B’s
long-arm statute authorizes jurisdiction as far as the Constitution allows, so the long-arm analysis is the
same as the constitutional analysis.

The due process clause of the 14th Amendment permits states to assert personal jurisdiction over
nonresident defendants who have established minimum contacts with the state such that the exercise of
personal jurisdiction would not offend traditional notions of fair play and substantial justice.

In this case, Paul’s suit arises out of or relates to Credit Union’s contacts with State B. Paul’s claim is
based on a loan agreement which informed Credit Union that Paul was a resident of State B at the time of
the agreement and that loan information should be mailed to Paul in State B. Credit Union in fact sends
information about the loan to Paul in State B, and receives payments from Paul from State B. Moreover,
the lawsuit arises out of one of Credit Union’s mailings to State B, albeit a mailing to an incorrect
address.

Although these contacts probably fall short of the continuous and substantial contacts required for general
jurisdiction, they are probably sufficient for the court to exercise specific jurisdiction over claims against
Credit Union that arise out of its contacts with State B.

A nonresident defendant is subject to specific jurisdiction when its contacts with the forum state
demonstrate purposeful availment of the benefits of the forum state and/or render it foreseeable that the
defendant may be haled into the forum state’s courts. While Credit Union might argue that its contacts
with State B result from Paul’s unilateral act of moving to State B and therefore do not demonstrate
purposeful availment, this argument will fail. Credit Union entered a long-term contract with Paul
knowing that he lived in State B and would be performing his part of the contract (making loan payments)
in State B. Credit Union mailed several payment statements addressed to Paul in State B. As a result,
Credit Union purposefully availed itself of the benefits of State B and could foresee being haled into court
in State B on matters related to the contract with Paul.

Even when a nonresident defendant has established the necessary minimum contacts with the forum state,
the exercise of personal jurisdiction over the defendant may offend due process if inconsistent with
traditional notions of fair play and substantial justice. The defendant must make a compelling case for
fairness considerations to outweigh the existence of minimum contacts. Credit Union will not succeed in
making such a case, as the fairness considerations tend to support the exercise of personal jurisdiction by
the courts of State B. Paul, who now lives in State B, has an interest in convenient and effective relief
close to home. State B has an interest in providing its residents with a forum for redress of injuries caused
by out-of-state defendants. And although litigation in State B will impose a burden on Credit Union,
which maintains no branches in State B, that burden is minimized by modern transportation and
communication. As a result, the exercise of personal jurisdiction by the courts of State B will not offend
notions of fairness.

Explanation to Point-Two (35%):

Credit Union is a citizen of State A. As a result, complete diversity will exist if Paul is a domiciliary of
State B. However, it is uncertain whether Paul is a domiciliary of State B or is still a domiciliary of State
A. If Paul is a State B domiciliary, then there is diversity jurisdiction; Paul’s claim satisfies the amount-
in-controversy requirement.

Seperac-J19 Exam-Released MEE Essay Compilation © 2016-2020 81


28 U.S.C. § 1332 vests the district courts with jurisdiction over cases in which (a) there is complete
diversity of citizenship, meaning no plaintiff shares the state of citizenship of any defendant, and (b) the
amount in controversy exceeds $75,000.

In determining citizenship, § 1332 directs that a corporation is a citizen where it is incorporated and where
it has its principal place of business. Under Hertz, the principal place of business is best read as referring
to the place where a corporation’s officers direct, control, and coordinate the corporation’s activities – the
place where the corporation maintains its headquarters – the ‘nerve center'. Credit Union is incorporated
in State A. Moreover, its principal place of business (indeed, its only place of business) is in State A.
Credit Union is a State A citizen for diversity purposes.

An individual U.S. citizen’s citizenship is determined by domicile, which is defined as the individual’s
“true, fixed, and permanent home to which he has the intention of returning whenever he is absent
therefrom.” Until recently, Paul’s domicile was clearly State A, where he had lived his entire life. Six
months ago, however, Paul moved to State B, where he now resides most of the time, although he
sometimes returns for short visits to his vacation home in State A.

The question is whether Paul has changed his domicile to State B. To change domicile, an individual must
both take up residence in a new state and intend to remain there. While Paul has taken up residence in
State B, it is unclear whether he intends to remain there. He originally planned to stay in State B for a year
and then decide whether he would remain or return to State A. Consistent with that plan, he took a job as
a temporary employee. Recently, however, Paul decided to obtain more permanent employment, as he has
really enjoyed living in State B. This change to more permanent employment suggests that Paul now
intends to remain in State B on a permanent basis, in which case his domicile has changed to State B. On
the other hand, his purchase of a vacation home in State A may suggest that he does not, in fact, intend to
change his State A domicile.

[NOTE: The issue can be fairly argued either way.]

If Paul is a citizen of State B, complete diversity exists. If Paul remains a citizen of State A, complete
diversity does not exist, as Paul and Credit Union would both be citizens of State A.

To determine whether the amount in controversy exceeds $75,000, courts rely on a plaintiff’s good-faith
allegations unless it appears “to a legal certainty” that the plaintiff cannot recover the amount alleged.
Paul’s claim for $240,000 clearly satisfies the amount-in-controversy requirement, and there are no facts
to suggest that it was not made in good faith. Even if the state statute providing for treble damages is
preempted by the federal identity-theft statute, Paul’s $80,000 claim for actual loss still satisfies the
amount-in-controversy requirement.

Explanation to Point-Three (30%):

Paul’s assertion of a defense to the possible applicability of a federal statute does not create federal-
question jurisdiction.

28 U.S.C. § 1331 provides district courts with jurisdiction to hear suits arising under the laws of the
United States. To determine whether such federal-question jurisdiction exists, a court looks at the
allegations necessary for a well-pleaded complaint and asks whether the claims stated by those allegations
are created by federal or state law. Claims created by federal, but not state, law generally fall within the
scope of a district court’s federal-question jurisdiction.

Seperac-J19 Exam-Released MEE Essay Compilation © 2016-2020 82


Paul’s breach-of-contract claim is created by state law and therefore lies outside the district court’s
original federal-question jurisdiction. Although Paul’s complaint mentions a federal statute, it does not
state any claim that is based on the federal law. Instead, in order to rebut an anticipated defense, Paul’s
complaint asserts that certain provisions of federal law are unconstitutional.

Pursuant to the well-pleaded complaint rule, there is no federal-question jurisdiction when federal law is
mentioned in the complaint only to deny the applicability of that law. Federal-question jurisdiction exists
“only when the plaintiff’s statement of his own cause of action shows that it is based upon federal laws or
the Constitution.” Here, Paul is asserting a cause of action based only on state law; the only federal issue
in the case is whether Paul’s cause of action is preempted. Federal-question jurisdiction does not exist
under these circumstances.

Seperac-J19 Exam-Released MEE Essay Compilation © 2016-2020 83


#017-FEB 2010–MEE Q06: QUESTION SIX (CIVIL PROCEDURE)

Husband is an American citizen domiciled in State A. Wife is a citizen of a foreign country who was
admitted to permanent residency in the United States five years ago and has been domiciled in State A
since then.

After struggling with infertility, Husband and Wife consulted with Doctor, who created embryos in a
laboratory using Husband’s sperm and Wife’s ova. Husband and Wife then entered into a surrogacy
contract with Surrogate, a domiciliary of State B. Pursuant to the contract, Surrogate agreed to carry the
couple’s embryo, to relinquish to them any child born as a result of the implantation, and to waive any
and all parental and/or custodial rights to the child. Husband and Wife also agreed, jointly and severally,
to pay all of Surrogate’s expenses and to assume custody and full financial and legal responsibility for any
child born as a result of the implantation.

Doctor implanted one of the embryos in Surrogate. Surrogate gave birth to a baby in State A and listed
Husband and Wife as the parents on the baby’s birth certificate. Husband and Wife obtained a judgment
from a State A court declaring that they were the legal parents of the baby and were entitled to sole
custody.

The baby had serious medical problems at birth and remained in the State A hospital for three months.
When the baby left the hospital, she went home with Husband and Wife. Surrogate returned to her home
in State B.

The hospital sent the bill for the baby’s medical care, which exceeded $500,000, to Surrogate. Surrogate
has medical insurance with Insureco, an insurance company incorporated under the laws of State A with
its principal place of business in State C. Surrogate’s insurance policy covers all reasonable and necessary
medical expenses incurred by Surrogate and her dependent(s), including “any natural child of Surrogate
born after the policy is in force.” However, Surrogate’s policy expressly provides that Insureco will not
cover expenses if a third party is liable for those expenses.

Insureco has refused to pay the baby’s medical bill on the grounds that she is not a “natural child” of
Surrogate within the meaning of the insurance policy and that the baby’s expenses are Husband and
Wife’s responsibility.

Husband and Wife have also refused to pay the bill, claiming that they cannot afford to pay it and that the
surrogacy contract is unenforceable under the applicable state law.

Surrogate has filed suit in the federal district court of State A against Insureco, Husband, and Wife.
Surrogate alleges that Husband and Wife breached the surrogacy contract and that Insureco breached the
terms of the insurance policy. Surrogate seeks to compel any or all of the defendants to pay the $500,000
hospital bill.

The defendants have moved to dismiss the action on the grounds that (i) the federal court lacks
jurisdiction over the case, (ii) the case involves state-law domestic-relations issues (i.e., the biological
parentage of the child and the enforceability of a surrogacy contract) that are inappropriate for resolution
by a federal court, and (iii) Surrogate improperly joined her separate claims against Insureco, on the one
hand, and Husband and Wife, on the other, in a single action.

Seperac-J19 Exam-Released MEE Essay Compilation © 2016-2020 84


1. Does the federal district court of State A have subject-matter jurisdiction over Surrogate’s
claims? Explain.

2. Should the federal district court of State A dismiss the action because it involves domestic-
relations issues? Explain.

3. Did Surrogate properly join Insureco, Husband, and Wife as defendants in a single action?
Explain.

Seperac-J19 Exam-Released MEE Essay Compilation © 2016-2020 85


#017: F10-6 MEE: ANSWER: NCBE (CIVIL PROCEDURE)

POINT (1) [35%] ISSUE: Does a federal district court have diversity jurisdiction over an action
between a plaintiff domiciled in one state and three defendants who are domiciled in another state
when one of the defendants is a permanent resident alien and not a U.S. citizen? ANSWER: Yes.
The federal district court has diversity jurisdiction to hear these breach-of-contract claims because
complete diversity of citizenship exists between the plaintiff and all of the defendants, and the
amount-in-controversy requirement is met.

POINT (2) [30%] ISSUE: Does a federal district court have diversity jurisdiction over a case that
arises out of a surrogacy agreement but does not seek a divorce, alimony, or child custody decree?
ANSWER: Yes. Under the domestic relations exception, federal courts do not have authority to
issue divorce, alimony, and child custody decrees. Here, the district court would retain jurisdiction
to adjudicate the plaintiff’s claims, because they raise contractual questions.

POINT (3) [35%] ISSUE: Do the Federal Rules of Civil Procedure permit a plaintiff to join three
defendants in a single action when her claims against the defendants, although based on different
contracts, all seek recovery for expenses generated by a single occurrence? ANSWER: Yes.
Surrogate properly joined Husband, Wife, and Insureco as defendants because her claims against
each of them arose out of the same transaction, occurrence, or series of transactions or occurrences
and involve questions of law or fact common to all defendants.

ANSWER DISCUSSION:

The federal district court has jurisdiction to adjudicate Surrogate’s claims against the defendants because
there is complete diversity among all of the parties and the amount-in-controversy requirement is met.
The domestic relations exception to diversity jurisdiction is not implicated by these facts because
Surrogate does not seek a divorce, alimony, or child custody decree, and a state court already has
determined legal parentage. Surrogate properly joined all three defendants in a single action because her
claims against them arise out of the same transaction or occurrence (the birth and hospitalization of the
baby) and involve a common question of law or fact.

ANSWER EXPLANATION:

Explanation to Point-One (35%):

The federal district court has diversity jurisdiction to hear these breach-of-contract claims because
complete diversity of citizenship exists between the plaintiff and all of the defendants, and the amount-in-
controversy requirement is met.

Surrogate is suing Insureco for breaching the terms of the insurance policy, in which it agreed to insure
Surrogate and her dependent children. She is suing Husband and Wife for breaching the surrogacy
agreement, in which they agreed to assume full financial and legal responsibility for any child resulting
from the implantation of their embryo. These claims arise under state contract law and do not raise any
federal questions. Therefore, federal court jurisdiction is proper only if the case satisfies the requirements
of 28 U.S.C. § 1332, the diversity statute.

Seperac-J19 Exam-Released MEE Essay Compilation © 2016-2020 86


Section 1332 grants federal district courts original jurisdiction of all civil actions with an amount in
controversy in excess of $75,000 if the plaintiff and defendant are “citizens of different States.” In
addition, the “diversity” between the parties on either side of the case must be “complete.” In other words,
no plaintiff may be a citizen of the same state as any of the defendants.

On these facts, there is complete diversity. Plaintiff Surrogate is deemed to be a citizen of State B because
she is domiciled there. Defendant Husband, similarly, is a citizen of State A, where he is domiciled.
Surrogate and Husband are diverse.

Surrogate is also diverse from Insureco. Under the diversity statute, “a corporation shall be deemed to be
a citizen of any State by which it has been incorporated and of the State where it has its principal place of
business.” Thus, Insureco is a citizen of both State A and State C and is therefore diverse from Surrogate,
a State B citizen.

Even though Wife is not a U.S. citizen, § 1332 provides that “an alien admitted to the United States for
permanent residence shall be deemed a citizen of the State in which such alien is domiciled.” Since Wife
has been admitted for permanent residence, she is deemed to be a citizen of State A, where she is
domiciled. Thus, she is also diverse from Surrogate.

Because the plaintiff, Surrogate, is a citizen of only State B and none of the defendants is a citizen of State
B, complete diversity exists.

Surrogate alleges that each defendant is obliged by contract to pay the $500,000 hospital bill. She seeks a
court order compelling payment of that bill. She thus raises a claim against each defendant that exceeds
the $75,000 amount-in-controversy threshold. In cases where a plaintiff seeks equitable relief, the amount
in controversy is the value to the plaintiff of what is sought to be obtained.

Explanation to Point-Two (30%):

Under the domestic relations exception, federal courts do not have authority to issue divorce, alimony,
and child custody decrees. Here, the district court would retain jurisdiction to adjudicate the plaintiff’s
claims, because they raise contractual questions.

The federal courts have long declined to exercise jurisdiction over domestic relations issues. More than a
century ago, the Supreme Court of the United States announced that “the whole subject of the domestic
relations of husband and wife, parent and child, belongs to the laws of the States and not to the laws of the
United States.” Although the domestic relations exception is not grounded in the Constitution, the
Supreme Court has concluded that such a long-established exception to diversity jurisdiction should be
retained, absent a Congressional decision to repudiate it.

Nonetheless, the mere fact that a case involves domestic relations (as the current problem arguably does)
does not mean that there is no diversity jurisdiction over that case. According to one leading treatise, the
domestic relations exception applies only to cases that are primarily marital disputes. This conclusion was
reinforced in Ankenbrandt, in which the Supreme Court limited the scope of the domestic relations
exception, stating that it “encompasses only cases involving the issuance of a divorce, alimony, or child
custody decree.” There, the Court held that a federal district court had diversity jurisdiction to adjudicate a
tort claim brought by the mother of two children on their behalf against their father and his companion,
alleging sexual and physical abuse, because the suit did not seek a divorce, alimony, or child custody
decree.

Seperac-J19 Exam-Released MEE Essay Compilation © 2016-2020 87


On the facts of this question, the domestic relations exception does not apply. No one is seeking a divorce
or alimony decree. And there is no dispute over custody of the baby. To the contrary, Husband and Wife
already have obtained a state court declaration that they are the baby’s parents and are entitled to sole
custody; Surrogate’s suit does not seek to alter that judicial custody determination. All Surrogate seeks is
specific performance of Husband and Wife’s contractual obligation to assume financial responsibility for
the baby.

Finally, while Surrogate’s claim against Insureco raises the issue of whether the baby is Surrogate’s
“natural child” within the meaning of the insurance policy, any determination of that issue would affect
only Insureco’s financial responsibility to cover the baby’s hospital bills. Resolution of the issue would
not affect the custody of the child.

Therefore, this case does not appear to fall within the domestic relations exception, and the district court
should exercise its diversity jurisdiction and adjudicate Surrogate’s claims. As the Supreme Court held
long ago, a federal court has “no more right to decline the exercise of jurisdiction which is given, than to
usurp that which is not given.” The Supreme Court has rejected a broad interpretation of the probate
exception to federal jurisdiction and required lower courts to exercise jurisdiction.

Explanation to Point-Three (35%):

Surrogate properly joined Husband, Wife, and Insureco as defendants because her claims against each of
them arose out of the same transaction, occurrence, or series of transactions or occurrences and involve
questions of law or fact common to all defendants.

The Federal Rules of Civil Procedure embody a liberal and flexible joinder policy. Rule 20, which
governs permissive joinder of parties, permits the plaintiff to join multiple defendants whenever two
conditions are met: (1) “any right to relief is asserted against the defendants jointly, severally, or in the
alternative, with respect to or arising out of the same transaction, occurrence, or series of transactions or
occurrences” and (2) “any question of law or fact common to all defendants will arise in the action.” Both
of these conditions must be satisfied. Rule 20 is designed to “promote trial convenience and expedite the
final determination of disputes, thereby preventing multiple lawsuits.” The Rule 20 requirements should
be read as broadly as possible whenever doing so is likely to promote judicial economy.

In determining whether claims against the three defendants arise out of the same transaction or
occurrence, courts often employ the logical-relationship test, under which “all logically related events
entitling a person to institute a legal action against another generally are regarded as comprising a
transaction or occurrence.” Emphasizing the liberality and flexibility of this standard, one leading treatise
notes that “courts are inclined to find that claims arise out of the same transaction or occurrence when the
likelihood of overlapping proof and duplication in testimony indicates that separate trials would result in
delay, inconvenience, and added expense to the parties and to the court.” The inclusion of the language
“in the alternative” was specifically intended to permit a plaintiff to join multiple defendants when she or
he “is entitled to relief from someone, but the plaintiff does not know which of two or more defendants is
liable under the circumstances set forth in the complaint.”

Here, Surrogate’s claims against all three defendants arise out of the same event – the baby’s birth and
hospitalization. In addition, certain factual questions will be common to both claims, including the
circumstances of the birth and the terms of the surrogacy agreement between Surrogate and Husband and
Wife, which are facts important both to Surrogate’s claim against Husband and Wife and to Insureco’s
defense that the baby is not Surrogate’s “natural child.” Thus, joinder is appropriate in this case.

Seperac-J19 Exam-Released MEE Essay Compilation © 2016-2020 88


[NOTE: An applicant who demonstrates knowledge of the joinder rule and the ability to apply the rule to
the facts of the problem in a sensible way should receive some credit, even if the applicant concludes that
joinder is improper in this case. However, an applicant who concludes that joinder is improper should
also note that dismissal of the action is not an appropriate remedy for improper joinder. The appropriate
remedy is to sever the claims and allow Surrogate to proceed separately against the defendants.]

Seperac-J19 Exam-Released MEE Essay Compilation © 2016-2020 89


#018-JUL 2009–MEE Q06: QUESTION SIX (CIVIL PROCEDURE)

Ann and Bill, both citizens of State X, were walking by a construction site in State X when an overhead
crane dropped a load of plate glass windows on the sidewalk near them. The windows shattered when
they hit the pavement, and both Ann and Bill were struck by flying glass. Ann’s injuries were severe; she
incurred more than $450,000 in medical expenses. Bill suffered only minor injuries and had medical bills
of $500.

At the time of the accident, the glass windows were being installed in a new skyscraper by GlassCo, Inc.,
a State Y corporation with its principal place of business in State Y. The crane operator was a GlassCo
employee, and the crane was owned by GlassCo. A subsequent investigation by GlassCo’s insurance
company concluded that the accident was due to crane operator error and improper crane maintenance.

Ann and Bill have joined as plaintiffs and filed a suit against GlassCo in a state trial court in State X. Ann
is seeking more than $1,000,000 in damages, and Bill is seeking $5,000. They have both refused
GlassCo’s request to enter into settlement negotiations.

You are an associate in the law firm that has been retained by GlassCo. The partner in charge of the case
wants to remove the lawsuit from state to federal court. She has asked:

1. What must GlassCo do to remove the case from state to federal court? Explain.

2. If the case is removed to federal court, do the Federal Rules of Civil Procedure permit the
separate claims of Ann and Bill to remain joined in a single lawsuit? Explain.

3. If Ann’s and Bill’s claims remain joined, will the federal court have jurisdiction over the case?
Explain.

Seperac-J19 Exam-Released MEE Essay Compilation © 2016-2020 90


#018: J09-6 MEE: ANSWER: NCBE (CIVIL PROCEDURE)

POINT (1) [15%] ISSUE: What steps must be taken to effect removal from state to federal court?
ANSWER: The defendant begins the process of removal by filing a notice of removal in the federal
court “for the district and division within which the state court action is pending.” Removal is then
accomplished by giving notice of the filing to all adverse parties and filing a copy of the notice of
removal with the state court.

POINT (2) [35%] ISSUE: Is the joinder of Ann and Bill as plaintiffs proper under the Federal
Rules of Civil Procedure? ANSWER: Yes. FRCP Rule 20 permits the claims of Ann and Bill to be
joined in a single action because those claims arose out of the “same transaction or occurrence” and
involve “common questions” of law and fact.

POINT (3) [50%] ISSUE: If Ann’s and Bill’s claims remain joined, will the federal court have
jurisdiction over the case? ANSWER: Yes. The federal court will have subject matter jurisdiction
over the removed case because Ann’s claim is within the diversity jurisdiction of the court and
Bill’s claim, although it does not satisfy the amount-in-controversy requirement for diversity
jurisdiction, is within the federal court’s supplemental jurisdiction.

ANSWER DISCUSSION:

To remove a case from state to federal court, the defendant must file a notice of removal with the federal
district court, give written notice of that filing to all adverse parties, and file a copy of the notice of
removal with the state court. The joinder of Ann and Bill as plaintiffs would be proper under Federal Rule
of Civil Procedure 20 because their claims for relief arose out of “the same transaction or occurrence” and
involve a “common question” of law or fact. The case is within the subject matter jurisdiction of a federal
court in State X. Both Ann and Bill, citizens of State X, are diverse from GlassCo, which is a citizen of
State Y (the state of its incorporation and principal place of business). Ann’s claim for $1,000,000 easily
satisfies the required jurisdictional amount of $75,000. Although Bill’s claim does not satisfy the required
jurisdictional amount, the federal court will have “supplemental jurisdiction” over Bill’s claim because it
is sufficiently related to Ann’s claim to be part of the same “case or controversy.”

ANSWER EXPLANATION:

Explanation to Point-One (10-20%):

The defendant begins the process of removal by filing a notice of removal in the federal court “for the
district and division within which the state court action is pending.” Removal is then accomplished by
giving notice of the filing to all adverse parties and filing a copy of the notice of removal with the state
court.

Procedurally, the first step for removing an action from state to federal court is for the defendant to file a
notice of removal in the federal district court for the district and division within which the state action is
pending. The notice must be filed within 30 days of the defendant’s “receipt , through service or
otherwise, of a copy of the initial pleading , or within 30 days after the service of summons if such initial
pleading has then been filed and is not required to be served , whichever period is shorter.” The notice of
removal must be signed and must include a statement of the basis for federal jurisdiction. A copy of the
materials from the state court proceeding must also be filed.

Seperac-J19 Exam-Released MEE Essay Compilation © 2016-2020 91


After filing the notice of removal, the defendant must serve notice of the filing on all adverse parties and
file a copy of the notice of removal with the state court. Removal is “automatic.” Once the state court has
a copy of the notice of removal, the case is removed and the state court can take no further action in the
case.

[NOTE: If the defendant removes the action to federal court, and the plaintiff wishes to contest the
removal, the plaintiff must file a motion to remand.]

Explanation to Point-Two (30-40%):

FRCP Rule 20 permits the claims of Ann and Bill to be joined in a single action because those claims
arose out of the “same transaction or occurrence” and involve “common questions” of law and fact.

FRCP Rule 20 provides that two or more plaintiffs may join their claims in a single lawsuit if the
plaintiffs “assert any right to relief jointly, severally, or in the alternative with respect to or arising out of
the same transaction, occurrence or series of transactions or occurrences; and any question of law or fact
common to all plaintiffs being joined will arise in the action.

Both requirements for joinder are met in this case. First, Ann and Bill are each asserting claims that arose
out of a single occurrence – the accident involving the falling glass. Second, those claims will certainly
raise some common questions of fact (e.g., whether the glass accident was caused by the negligence of
GlassCo employees) and may also raise common questions of law (e.g., the legal standard for measuring
damages). This is precisely the kind of case in which Rule 20 allows joinder of plaintiffs in the interest of
promoting efficiency and consistency of results.

The fact that Ann and Bill are asserting separate claims and separate rights to relief does not matter. Rule
20 allows plaintiffs to join when they assert their rights to relief “severally.”

Explanation to Point-Three (45-55%):

The federal court will have subject matter jurisdiction over the removed case because Ann’s claim is
within the diversity jurisdiction of the court and Bill’s claim, although it does not satisfy the amount-in-
controversy requirement for diversity jurisdiction, is within the federal court’s supplemental jurisdiction.

Removal of a case from state to federal court is proper only when the “civil action” that is removed is
within the “original jurisdiction” of the federal court.

In this case, Ann’s tort claim against GlassCo would be within the original diversity jurisdiction of the
federal district court of State X. Ann and GlassCo are citizens of different states, and Ann’s claim for
$1,000,000 exceeds the $75,000 amount-in-controversy requirement of the diversity statute.

Bill’s tort claim, on the other hand, is not within the diversity jurisdiction of the court. Although Bill and
GlassCo are diverse, Bill is seeking damages of only $5,000, which is substantially less than the $75,000
minimum amount-in-controversy for federal diversity actions.

Nonetheless, the federal court will have jurisdiction over Bill’s claim pursuant to 28 U.S.C. § 1367, the
“supplemental jurisdiction” statute. When a federal court has original jurisdiction over a claim in an
action, § 1367(a) provides that the court “shall have supplemental jurisdiction over all other claims that
are so related to the main claim that they form part of the same case or controversy under Article III of the
United States Constitution.”

Seperac-J19 Exam-Released MEE Essay Compilation © 2016-2020 92


On the facts of this problem, Bill’s claim is sufficiently related to Ann’s claim to be “part of the same case
or controversy.” Both claims arose out of a “common nucleus of operative fact” (the accident involving
the glass) and are so similar in facts and law that it is sensible to expect them to be tried in a single
judicial proceeding. Thus, the court has supplemental jurisdiction over Bill’s claim.

Seperac-J19 Exam-Released MEE Essay Compilation © 2016-2020 93


#019-FEB 2009–MEE Q05: QUESTION FIVE (CIVIL PROCEDURE/CONFLICTS)

Bearco is a corporation incorporated under the laws of State A. Bearco maintains its corporate and
administrative offices in State A; its factories are located in State B. Bearco’s popular stuffed toy bear,
“Griz,” is sold throughout the United States. Bearco has registered the trademark “Griz” with the United
States Patent and Trademark Office.

Copyco is a corporation incorporated under the laws of Country X, a foreign country, where Copyco has
its manufacturing facilities and corporate offices. Copyco sells a line of toy bears called “Griz,” which
look remarkably similar to the Bearco “Griz” bears. Copyco sells its bears to consumers throughout the
United States. However, it sells only on the Internet, using a parcel delivery service to deliver the bears to
consumers. The Copyco website does not list a telephone number, street address, or post office box for
the company. It lists only an e-mail address and an Internet address.

Bearco has filed an action against Copyco in the United States District Court for State A, properly
invoking the court’s federal question and diversity jurisdiction. Bearco alleges both trademark
infringement (a federal law claim) and unfair competition (a tort claim that, in the United States, is based
on state law).

State A and State B have materially different unfair-competition laws. Unfair competition is not
actionable under the law of Country X.

To address choice-of-law problems, State A follows the “most significant relationship” approach of the
Restatement (Second) of Conflict of Laws. State B applies the “vested rights” approach of the
Restatement (First) of Conflict of Laws. Country X’s choice-of-law methodology is unknown.

Bearco has been unable to determine Copyco’s street address or post office box address either in the
United States or in Country X and has filed a motion requesting that the district court authorize service of
the summons and complaint by e-mail. There are no international agreements that affect the court’s
resolution of the issues in this case.

1. If the United States District Court for State A permits service of process on Copyco by e-mail,
would such e-mail service be consistent with the Federal Rules of Civil Procedure and the United
States Constitution? Explain.

2. Which jurisdiction’s law should the United States District Court for State A apply to resolve
Bearco’s unfair-competition claim? Explain.

Seperac-J19 Exam-Released MEE Essay Compilation © 2016-2020 94


#019: F09-5 MEE: ANSWER: NCBE (CIVIL PROCEDURE/CONFLICTS)

POINT (1)(a) [25%] ISSUE: Do the Federal Rules of Civil Procedure permit a federal district court
to authorize service of process on a foreign defendant by e-mail? ANSWER: Yes. Rule 4 of the
Federal Rules of Civil Procedure permits service of process on a foreign corporation outside of the
United States by any means directed by the court as long as it is not prohibited by international
agreement.

POINT (1)(b) [25%] ISSUE: Is e-mail service of process constitutional under the circumstances of
this case? ANSWER: Yes. Service by e-mail is constitutional where it is reasonably calculated to
give notice.

POINT (2)(a) [25%] ISSUE: Which jurisdiction’s choice-of-law rules should be used to select the
applicable unfair-competition law? ANSWER: A federal court exercising diversity jurisdiction over
non-federal claims must apply the choice-of-law rule of the state in which it sits.

POINT (2)(b) [25%] ISSUE: Under the “most significant relationship” test, what factors govern the
choice among State A, Country X, and State B unfair-competition law? ANSWER: Because the
federal court will apply State A’s choice-of-law rule, the Second Restatement of Conflict of Laws
(the methodology employed by State A) applies here. Under the Second Restatement’s approach,
issues in tort are governed by the law of the state that has the most significant relationship to the
occurrence and the parties.

ANSWER DISCUSSION:

Absent a governing international agreement, the district court has authority to order service of process by
e-mail so long as the service is reasonably calculated to give notice to Copyco of the suit. Once it obtains
jurisdiction over Copyco, the court should use State A’s “most significant relationship” approach to
determine the proper law to apply to the unfair-competition claim. The proper choice of law depends on
several factors, including the policies underlying the competing laws. Here, given the many connections
of the case to State A, it is likely that a court sitting in State A would apply State A law.

ANSWER EXPLANATION:

Explanation to Point-One(a) (20-30%):

Rule 4 of the Federal Rules of Civil Procedure permits service of process on a foreign corporation outside
of the United States by any means directed by the court as long as it is not prohibited by international
agreement.

In a proper case, a federal court has the power to permit a plaintiff to serve a foreign corporate defendant
by e-mail. Rule 4 authorizes service of process upon a corporation outside the United States “in any
manner prescribed by Rule 4(f) for serving an individual, except personal delivery.” Subdivision (f) of
Rule 4, in turn, permits service in places outside the United States by a variety of methods, including
means of service allowed by international agreement, by foreign law, and by general international
practice. Importantly, subdivision (f) gives the district court broad authority to direct service by any
“means not prohibited by international agreement.” In other words, the court can authorize any method of
making service of process abroad, including e-mail, in its discretion, if no international agreement

Seperac-J19 Exam-Released MEE Essay Compilation © 2016-2020 95


prohibits it. Moreover, the court’s authority to direct non-traditional means of service “is neither a ‘last
resort’ nor ‘extraordinary relief,’” it is simply one of the tools the court has to ensure that procedural
matters are handled sensibly and fairly.

On the facts of this case, it is likely that e-mail service of process would be approved by the court. First,
the facts state that there are no applicable international treaties, so the court’s power to direct any “means”
of service, including e-mail, is not limited by international agreement. Second, this seems like a perfect
case for e-mail: the defendant’s street address cannot be ascertained, and the defendant’s business conduct
evinces a preference for communication through the Internet. It would seem that service by e-mail would
be convenient for the defendant and reasonably calculated to give the defendant notice of the action. It
would, moreover, prevent the significant hardship to the plaintiff that might result if the plaintiff were
required to find other means to track down and serve this elusive corporate defendant. For these reasons,
the court is likely to authorize e-mail service in this situation.

Explanation to Point-One(b) (20-30%):

Service by e-mail is constitutional where it is reasonably calculated to give notice.

Even if permitted by the Federal Rules of Civil Procedure, a method of service of process cannot be
employed if it would violate the United States Constitution. The due process clause guarantees litigants
“notice reasonably calculated, under all the circumstances, to apprise interested parties of the pendency of
the action and afford them an opportunity to present their objections. The means employed must be such
as one desirous of actually informing the absentee might reasonably adopt to accomplish it.” Service by e-
mail is reasonably calculated to reach Copyco, which lists its e-mail address on its website but does not
list a street address or post office box. In fact, in these circumstances, service by e-mail may be the
method of service most likely to reach Copyco. Since service via e-mail is reasonably calculated to reach
Copyco, it comports with due process.

Explanation to Point-Two(a) (20-30%):

A federal court exercising diversity jurisdiction over non-federal claims must apply the choice-of-law rule
of the state in which it sits.

In Klaxon Co. v. Stentor Electric Manufacturing Co., the Supreme Court of the United States held that a
federal district court sitting in diversity must apply the choice-of-law approach prevailing in the state in
which it sits. The Klaxon rule is designed to ensure that a federal court sitting in diversity and the state
court sitting next door would reach the same result if presented with the same case. If federal courts were
free to craft their own choice-of-law rules, then the federal court might choose a different jurisdiction’s
substantive law to govern the dispute than the state court would, and the goals of uniformity and equal
administration of justice would be frustrated. Thus, the federal district court sitting in State A should
apply the choice-of-law approach followed by the courts of State A. The facts indicate that State A applies
the Second Restatement of Conflict of Laws.

Explanation to Point-Two(b) (20-30%):

Because the federal court will apply State A’s choice-of-law rule, the Second Restatement of Conflict of
Laws (the methodology employed by State A) applies here. Under the Second Restatement’s approach,
issues in tort are governed by the law of the state that has the most significant relationship to the
occurrence and the parties.

Seperac-J19 Exam-Released MEE Essay Compilation © 2016-2020 96


Unfair competition is a tort claim governed by § 145 of the Second Restatement of Conflict of Laws.
Section 145 states that “the rights and liabilities of the parties with respect to an issue in tort are
determined by the local law of the state which, with respect to that issue, has the most significant
relationship to the occurrence and the parties under the principles stated in § 6.” Section 145 lists four
contacts to be considered in performing this analysis: “(a) the place where the injury occurred, (b) the
place where the conduct causing the injury occurred, (c) the place of incorporation, and place of business
of the parties, and (d) the place where the relationship, if any, between the parties is centered. These
contacts are to be evaluated according to their relative importance with respect to the particular issue.”

Here, an evaluation of the § 145 contacts reveals that a number of jurisdictions have important
connections to the parties and the transaction. First, Bearco’s injury – the loss of customers who bought
Copyco’s knockoff “Griz” bears – occurred in all 50 states and would have been felt most acutely in State
A (where Bearco maintains its corporate and administrative offices) and State B (where Bearco’s factories
are located). Copyco’s conduct occurred both in Country X, where it manufactured the knockoff bears,
and also in all 50 states, where it sold bears in competition with Bearco. The two companies are
incorporated in different locations, State A (Bearco) and Country X (Copyco), and their places of business
are likewise in different locales.

Where the contacts are so thoroughly split among so many jurisdictions, courts applying the Second
Restatement will pay special attention to the principles for making choice-of-law decisions as stated in § 6
of the Restatement. In a tort case like this one, a court will ordinarily focus its attention on the policies of
the laws that are in conflict and, in particular, on any interest of its own state in having its policies apply
to the case.

Here it is likely that the purpose of State A’s unfair-competition law is to protect businesses from the
economic harm caused by: the kind of unfair competition involved in this case. Given that Bearco is a
local business, incorporated and headquartered in State A, and that the injury from Copyco’s unfair
competition will be felt in State A, State A would certainly have an interest in seeing its policy applied to
force Copyco to compensate Bearco. On the other hand, Country X would be interested in seeing its law
apply to protect Copyco if the policy behind Country X’s failure to regulate unfair competition is intended
to facilitate competition by its companies and to encourage their manufacturing operations in Country X.

Here, a federal court sitting in State A is likely to conclude that State A has the “most significant
relationship” to the case. In a situation of conflict between local and foreign law, where both the local and
foreign jurisdictions’ policies would be furthered by applying their law, most courts will apply local law
and further local policies unless there is a strong reason not to do so. There is nothing unfair in subjecting
Copyco to State A law where its activities were directed in part at customers in State A, it injured a State
A corporation, and it was foreseeable that it could be subject to State A law on account of its activities.
The federal court will likely apply State A’s unfair-competition law.

[NOTE: An applicant’s conclusion is less important than his or her ability to state and use the relevant
principles of the Second Restatement of Conflict of Laws. Applicants are expected to know that the federal
court will apply State A’s choice-of-law methodology to choose the law governing the supplemental state
claim and that the Second Restatement requires application of the law of the state with the most
significant relationship to the parties and the occurrence. They should also know that the Second
Restatement requires consideration of a variety of personal and territorial factors and an analysis of the
policies underlying the laws competing for application. Applicants are not expected to recall specific
Restatement sections or comments, nor are applicants expected to reach the same results as the model
analysis.]

Seperac-J19 Exam-Released MEE Essay Compilation © 2016-2020 97


#020-JUL 2008–MEE Q05: QUESTION FIVE (CIVIL PROCEDURE)

Guest, a citizen of State A, ate oysters at Ron’s Restaurant in State B. Guest paid for the meal with a $50
check. Ron’s Restaurant is owned and operated by Ron, a citizen of State B.

After eating the oysters at Ron’s Restaurant, Guest ate an ice cream sundae at the ice cream shop next
door, which is owned and operated by CreamCorp, a State B corporation with its principal place of
business in State B.

An hour later, Guest became ill and went to a hospital emergency room. Guest had to be admitted to the
hospital for several days of tests, treatment, and observation. Ultimately, the doctors concluded that Guest
was suffering from a severe case of food poisoning. Guest’s hospital bills exceeded $75,000.

Guest stopped payment on the $50 check to Ron’s Restaurant before the check cleared and has not
otherwise paid for the meal.

Guest sued Ron (doing business as Ron’s Restaurant) in the federal district court for the District of State
B. Guest’s complaint alleged that the oysters she ate at Ron’s Restaurant caused her food poisoning.
Guest further alleged that her damages exceed $75,000, exclusive of costs and interest.

Ron doubts that the oysters were contaminated because no other patrons suffered an adverse reaction to
the oysters served that day. Ron believes that Guest became ill because the ice cream served at
CreamCorp’s shop was made with unpasteurized milk. Thus, Ron has moved to compel the joinder of
CreamCorp as an additional defendant in the lawsuit so that, if the jury concludes Guest became sick from
the ice cream, it can render a verdict against CreamCorp and not Ron. Ron has also added to his answer a
claim against Guest for the unpaid $50.

Guest objects to the joinder of CreamCorp. Guest has also moved to strike Ron’s claim for the unpaid $50
from Ron’s answer.

1. Should the court order the joinder of CreamCorp as an additional defendant? Explain.

2. Do the Federal Rules of Civil Procedure permit Ron to join his claim against Guest for the
unpaid $50 to Guest’s lawsuit against Ron? Explain.

3. If the Federal Rules of Civil Procedure permit Ron to join his claim against Guest for the unpaid
$50, will the court have subject matter jurisdiction to hear that claim? Explain.

Seperac-J19 Exam-Released MEE Essay Compilation © 2016-2020 98


#020: J08-5 MEE: ANSWER: NCBE (CIVIL PROCEDURE)

POINT (1) [45%] ISSUE: Do the Federal Rules of Civil Procedure permit a defendant to compel a
plaintiff to join other parties as additional defendants when the original defendant alleges that those
other parties are responsible for the plaintiff’s injury? ANSWER: No. Rule 19 of the Federal Rules
of Civil Procedure does not afford Ron a means to compel Guest to join CreamCorp as an
additional defendant because CreamCorp does not qualify as a necessary party.

POINT (2) [25%] ISSUE: Do the Federal Rules of Civil Procedure permit a defendant to join a
counterclaim against the plaintiff when the counterclaim is for an unpaid bill relating to the
transaction that gave rise to the plaintiff’s claim? ANSWER: Yes. Rule 13 of the Federal Rules of
Civil Procedure not only permits, but requires, Ron to bring his claim for the unpaid restaurant bill
as a counterclaim against Guest in the context of Guest’s lawsuit against Ron because the
counterclaim arises out of or relates to the same transaction or occurrence.

POINT (3) [30%] ISSUE: Does a federal court have jurisdiction over a defendant’s state-law
counterclaim when that counterclaim is for less than $75,000 but is related to the plaintiff’s original
claim? ANSWER: Yes. The federal district court likely would have jurisdiction to hear the
counterclaim under the supplemental jurisdiction statute, 28 U.S.C. § 1367, as long as the
counterclaim for failure to pay the check and the plaintiff’s claim for negligence form part of the
same case for purposes of the case-or-controversy requirement of Article III.

ANSWER DISCUSSION:

Ron probably cannot force Guest to join CreamCorp as a defendant because CreamCorp is not a necessary
party within the meaning of Rule 19 of the Federal Rules of Civil Procedure. Ron’s claim that CreamCorp
caused Guest’s injury does not provide a sufficient justification for forcing the joinder of CreamCorp over
Guest’s objection. The Federal Rules permit Ron to join any counterclaim he may have against Guest,
regardless of whether the counterclaim arises out of or relates to Guest’s claim against Ron. A federal
court ordinarily would not have jurisdiction over Ron’s $50 state-law claim because it does not satisfy the
amount-in-controversy requirement of diversity jurisdiction. Here, however, Ron’s counterclaim arose out
of the same transaction as Guest’s tort claim. Therefore, it is likely that the federal district court will have
subject matter jurisdiction to hear the counterclaim under the supplemental jurisdiction statute, 28 U.S.C.
§ 1367.

ANSWER EXPLANATION:

Explanation to Point-One (40-50%):

Rule 19 of the Federal Rules of Civil Procedure does not afford Ron a means to compel Guest to join
CreamCorp as an additional defendant because CreamCorp does not qualify as a necessary party.

Ordinarily the plaintiff is the master of her suit and gets to decide whom to sue, with whom to sue, where
to sue, and what claims to bring. Thus, if a plaintiff like Guest sues only one defendant when she has
claims against two defendants, ordinarily there is nothing that a defendant, like Ron, can do to compel the
plaintiff to join another party.

Seperac-J19 Exam-Released MEE Essay Compilation © 2016-2020 99


But there are some checks on plaintiff autonomy. Rule 19, which governs the joinder of persons needed
for just adjudication, authorizes joinder of defendants over the plaintiff’s objection only when:

• “the court cannot accord complete relief among existing parties” if the absent person is not joined, or

• the absent person claims an interest in the action that would “as a practical matter” be impaired or
impeded if that person is not joined, or

• the person’s absence may leave any of the parties subject to a risk of multiple liability or inconsistent
obligations.

Where any of these criteria is met, courts and commentators commonly refer to the absent party as a
necessary party, and a court is empowered to join that party, even over the plaintiff’s objection.

In this case, none of these requirements is satisfied. First, CreamCorp’s presence is not necessary for the
court to afford complete relief as between Guest and Ron. If Guest got sick because the oysters were
contaminated, the court can afford complete relief by holding Ron liable and awarding damages to Guest.
If, on the other hand, Guest became sick because the ice cream was made from unpasteurized milk, then
the court can afford complete relief between Ron and Guest by refusing to hold Ron liable. Either way,
complete and proper relief can be accorded as between Ron and Guest, without CreamCorp’s presence. If
the jury or court finds that the oysters did not cause the food poisoning, Guest will have to initiate a
separate suit to determine if she can recover from CreamCorp. While inefficient, this result does not
render CreamCorp a necessary party under Rule 19. Joint tortfeasors are permissive, not necessary, parties
under Rule 19. In short, Guest’s ability to recover whatever (if anything) she is entitled to recover from
Ron is unaffected by CreamCorp’s absence.

Second, although CreamCorp may have an interest in the subject matter of the action if Ron attempts to
prove that CreamCorp’s product caused Guest’s food poisoning, that interest is not likely to be impaired
or impeded if CreamCorp is not joined. If the jury finds that Guest’s illness was caused by the oysters,
CreamCorp’s interest would not be impaired. If the jury finds that Guest’s illness was caused by
CreamCorp’s ice cream, CreamCorp will not be bound by the judgment since it was not a party, and it
will be free to relitigate the issue of the ice cream’s toxicity if Guest sues it in a separate lawsuit. The
mere fact that Party A, in a suit against Party B, intends to introduce evidence that will indicate that a non-
party, C, behaved improperly does not, by itself, make C a necessary party.

Third, Ron does not face a substantial risk of incurring inconsistent obligations or double liability if
CreamCorp is not joined here. If Ron is held liable to Guest, Ron will pay Guest but face no other liability
or obligation on the same claim. If Ron is not held liable, then the possibility of double liability or
inconsistent obligations does not exist.

Therefore, under Rule 19 Ron cannot compel Guest to join CreamCorp as an additional defendant.

Explanation to Point-Two (20-30%):

Rule 13 of the Federal Rules of Civil Procedure not only permits, but requires, Ron to bring his claim for
the unpaid restaurant bill as a counterclaim against Guest in the context of Guest’s lawsuit against Ron
because the counterclaim arises out of or relates to the same transaction or occurrence.

A claim for relief brought by a defendant against a plaintiff is called a counterclaim. Counterclaims are
governed by Rule 13 of the Federal Rules of Civil Procedure. Rule 13 requires a defendant to bring a

Seperac-J19 Exam-Released MEE Essay Compilation © 2016-2020 100


counterclaim against the plaintiff if the counterclaim arises out of the same transaction or occurrence as
the plaintiff’s claim against the defendant. Here, Ron’s claim to be paid for the restaurant meal arises out
of the same transaction and occurrence as Guest’s claim that she became sick as a result of the meal. Both
claims arose out of the meal Ron served to Guest.

Rule 13 further provides that an answer “may state as a counterclaim” any claim against the plaintiff “that
is not compulsory” Read together, these portions of the rule authorize Ron to bring a counterclaim against
Guest for the unpaid $50 regardless of whether it arises out of the same transaction or occurrence as
Guest’s claim against Ron. Therefore, it is not necessary to determine whether Guest’s claim and Ron’s
claim arose out of the same transaction or occurrence. In either case, Ron may bring the claim.

Explanation to Point-Three (25-35%):

The federal district court likely would have jurisdiction to hear the counterclaim under the supplemental
jurisdiction statute, 28 U.S.C. § 1367, as long as the counterclaim for failure to pay the check and the
plaintiff’s claim for negligence form part of the same case for purposes of the case-or-controversy
requirement of Article III.

Whether the federal court can assert jurisdiction over Ron’s counterclaim is another matter. The
counterclaim against Guest for failure to pay the $50 is a state-law claim that could not be brought on its
own in federal court. While there is complete diversity of citizenship between Guest and Ron, the amount
in controversy on the unpaid restaurant check claim is only $50 and doesn’t come close to satisfying the
$75,000 amount-in-controversy requirement for diversity jurisdiction.

However, where a federal court already has jurisdiction over some claims, the supplemental jurisdiction
statute, 28 U.S.C. § 1367, authorizes district courts to hear claims that could not otherwise be heard in
federal court if those claims “are so related to the primary claims in the action that they form part of the
same case or controversy under Article III of the United States Constitution.” In determining whether or
not the primary claim and the supplemental claim “are so related,” most courts apply the “common
nucleus of operative fact” test announced by the United States Supreme Court in United Mine Workers v.
Gibbs. The state and federal claims must derive from a common nucleus of operative fact” to be part of
the same “case” for constitutional purposes.

Here, it is likely that Guest’s claim against Ron for food poisoning and Ron’s claim against Guest for the
unpaid $50 arise out of a common nucleus of operative fact, namely the alleged food-poisoning incident.
The reason that Guest stopped payment on the check and declined to pay for the food was because the
food allegedly made her ill. For example, in one case, the court concluded that the defendant’s claim that
the plaintiff committed medical malpractice was a compulsory counterclaim to the plaintiff’s claim to
recover for unpaid medical bills; the counterclaim was an offshoot of the same basic controversy as the
plaintiff’s claim and necessarily involved common issues of law and fact that would require duplicative
effort if tried in a separate action.

Section 1367 affords the district court discretion to decline to exercise supplemental jurisdiction in four
circumstances: “(1) the claim raises a novel or complex issue of State law; (2) the claim substantially
predominates over the claim or claims over which the district court has original jurisdiction; (3) the
district court has dismissed all claims over which it has original jurisdiction; or (4) in exceptional
circumstances, there are other compelling reasons for declining jurisdiction.” Here, the counterclaim for
failure to pay the restaurant bill does not raise any novel or complex issues of state law. The restaurant bill
claim does not predominate over the more factually complex claim of food poisoning. The district court
has not dismissed the plaintiff’s claim. Nor are there any compelling reasons to decline to exercise

Seperac-J19 Exam-Released MEE Essay Compilation © 2016-2020 101


jurisdiction. Therefore, it is likely that the district court would have supplemental jurisdiction to hear
Ron’s counterclaim against Guest.

Seperac-J19 Exam-Released MEE Essay Compilation © 2016-2020 102


#021-FEB 2008–MEE Q06: QUESTION SIX (CIVIL PROCEDURE)

Plaintiff worked for Corporation. In her fifth year of employment, Plaintiff complained that she had been
passed over for promotion to a management position in favor of a less experienced male colleague.
Shortly after voicing her discontent, Plaintiff was fired. Plaintiff sued Corporation in federal district court
alleging that she was fired because of sex discrimination in violation of federal law. Corporation filed an
answer denying the material allegations of the complaint and alleging that Plaintiff was fired for
inadequate work performance.

During voir dire, the court asked each prospective juror whether he or she had ever been a party to an
employment discrimination lawsuit. None of the prospective jurors answered in the affirmative.

At trial, Plaintiff offered evidence to support her claim that she had been fired for discriminatory reasons.
Her evidence included testimony by a former co-worker relating a conversation the co-worker had had
with Plaintiff’s supervisor. In the conversation, the supervisor reportedly said that women were ill-suited
for managerial positions and that men didn’t like taking orders from women.

At the close of Plaintiff’s evidence, Corporation moved for judgment as a matter of law (JMOL). The
court denied the motion. Corporation then submitted its own evidence. The evidence consisted primarily
of Plaintiff’s employment records and testimony by Plaintiff’s supervisor, who testified that Plaintiff’s
work had been rated unsatisfactory several times in the past.

At the close of all the evidence, neither party made further motions. After instructions from the court, the
case was submitted to the jury. The jury deliberated for several hours and then returned a verdict for
Plaintiff. The jury specifically found that Plaintiff had been passed over for promotion and fired as the
result of sex discrimination and that she was entitled to backpay and reinstatement.

Two days after the entry of judgment, Corporation learned for the first time that the jury foreperson had
previously filed and lost two employment discrimination lawsuits and that, upon learning she was part of
the jury pool in this case, had said to a friend, “I’m going to get on that jury and stick it to Corporation.
Somebody needs to teach these companies not to discriminate.”

Eight days after the entry of judgment, Corporation moved for a post-judgment JMOL or, in the
alternative, for a new trial. Corporation made two arguments. First, Corporation claimed that the evidence
it presented at trial regarding the reasons for Plaintiff’s termination was more persuasive and more
credible than Plaintiff’s evidence. Second, Corporation claimed that a new trial is required because of the
jury foreperson’s failure to disclose during voir dire her prior involvement in employment discrimination
lawsuits.

The trial judge believes that Corporation’s evidence at trial was more credible and persuasive than
Plaintiff’s. The trial judge also believes that Corporation’s information about the jury foreperson’s prior
experiences and her statements to her friend is credible and suggestive of possible juror bias.

1. Was Corporation’s motion for a post-judgment JMOL procedurally proper? Explain.

2. Without regard to its procedural propriety, should the trial court grant Corporation’s motion for a
post-judgment JMOL? Explain.

Seperac-J19 Exam-Released MEE Essay Compilation © 2016-2020 103


3. Should the trial court grant Corporation’s motion for a new trial on the ground that Corporation’s
evidence was more persuasive and credible than Plaintiff’s evidence? Explain.

4. Should the trial court grant Corporation’s motion for a new trial because of the jury foreperson’s
conduct? Explain.

Seperac-J19 Exam-Released MEE Essay Compilation © 2016-2020 104


#021: F08-6 MEE: ANSWER: NCBE (CIVIL PROCEDURE)

POINT (1) [25%] ISSUE: Under Rule 50 of the Federal Rules of Civil Procedure, can a post-
judgment motion for judgment as a matter of law be granted if the defendant moved for judgment
as a matter of law at the close of the plaintiff’s evidence but did not move for judgment as a matter
of law at the close of all the evidence? ANSWER: No. Because Corporation failed to move for
judgment as a matter of law at the close of all the evidence, Rule 50(b) bars it from moving for
judgment as a matter of law after trial.

POINT (2) [25%] ISSUE: Should a court reject a jury verdict and grant judgment as a matter of
law on the basis that the losing party’s evidence was more persuasive than the evidence of the party
in whose favor the jury ruled? ANSWER: No. Even if Corporation had preserved its right to file
(or “renew”) the motion for JMOL after the verdict, the motion should still be denied because
Corporation has not advanced arguments that justify granting such a motion.

POINT (3) [25%] ISSUE: Should a court grant a motion for a new trial if the movant shows that
the movant’s evidence was more persuasive and more credible than the opponent’s evidence?
ANSWER: No. Corporation’s alternative motion for a new trial should be granted if the court is
convinced that the jury’s verdict was clearly against the weight of the evidence and a miscarriage of
justice, but such a result is unlikely here.

POINT (4) [25%] ISSUE: Should a court grant a motion for a new trial on the ground that a juror
during voir dire withheld information suggesting that she would be biased against one of the parties
in the suit? ANSWER: Yes. Corporation’s motion for a new trial should be granted if a juror
improperly failed to reveal information that would bias that juror against Corporation.

ANSWER DISCUSSION:

In Plaintiff’s employment discrimination suit against Corporation, the post-trial motion by Corporation
for judgment as a matter of law (JMOL) should be denied. First, Corporation did not move for JMOL at
the close of all the evidence as required by Rule 50 of the Federal Rules of Civil Procedure. Corporation’s
motion at the close of Plaintiff’s evidence does not satisfy the Rule’s requirement. Second, Corporation’s
arguments in support of the motion do not state grounds sufficient for entering judgment for Corporation
as a matter of law. On the other hand, Corporation’s motion for a new trial under Rule 59 of the Federal
Rules of Civil Procedure may have merit. Corporation does not face a procedural bar such as that
precluding its post-judgment motion for JMOL. Corporation’s alternative motion for a new trial should be
granted if Corporation can show that the jury’s verdict was clearly against the weight of the evidence and
a miscarriage of justice, or if a juror failed to reveal information indicating a strong bias against
Corporation. It is unlikely that Corporation can show that the verdict is clearly against the weight of the
evidence, but Corporation should be able to show juror bias.

ANSWER EXPLANATION:

Explanation to Point-One (20-30%):

Because Corporation failed to move for judgment as a matter of law at the close of all the evidence, Rule
50(b) bars it from moving for judgment as a matter of law after trial.

Seperac-J19 Exam-Released MEE Essay Compilation © 2016-2020 105


Under Rule 50 of the Federal Rules of Civil Procedure, a party may make a renewed motion for judgment
as a matter of law (“RJMOL”) if the party makes the motion no later than 28 days after entry of judgment
and if the party previously made a motion for JMOL at the close of all the evidence. Here, the defendant
made its post-verdict RJMOL motion in a timely manner, but it did not make an earlier JMOL motion “at
the close of all the evidence.” Even though Corporation moved for JMOL at the close of plaintiff’s
evidence, it must make the motion again at the close of the record or it loses its right to renew the motion
after entry of judgment. Moreover, a post-trial motion for judgment can be granted only on grounds
advanced in such a pre-verdict motion.

The federal courts are divided on how strictly to enforce Rule 50 when a party has moved for judgment at
the close of the plaintiff’s case but not at the close of all the evidence. Some circuits have required strict
compliance and have held that a motion at the close of plaintiff’s case never is sufficient to support a
renewed motion for JMOL. Courts in these circuits would deny Corporation’s renewed motion because no
JMOL motion was made at the close of all evidence.

Other circuit courts have held that, if a JMOL motion was made at the close of plaintiff’s case, the failure
to again move at the close of all the evidence need not be fatal to a post-verdict motion if the evidence
introduced by the defendant was brief and could not have possibly changed the court’s decision on the
earlier motion. This exception would not help Corporation here, however, because the witness testimony
it presented in defense was substantial and could well have changed the trial court’s view on the merits of
the motion if Corporation had made that motion at the close of all evidence after presenting its own case.

Thus, Corporation is now precluded from renewing its JMOL motion.

Explanation to Point-Two (20-30%):

Even if Corporation had preserved its right to file (or “renew”) the motion for JMOL after the verdict, the
motion should still be denied because Corporation has not advanced arguments that justify granting such a
motion.

A motion for JMOL at the close of all the evidence, or a renewed motion after the return of the verdict,
can be granted only if “there is no legally sufficient evidentiary basis for a reasonable jury” to find for the
party opposing the movant. In ruling on such a motion, the trial judge is not authorized to weigh the
evidence, pass on credibility of witnesses, or otherwise substitute the judge’s view of the evidence for the
jury’s view. The standard expressed in Rule 50 makes clear that the grant of a motion for JMOL is “a
performance of the court’s duty to assure enforcement of the controlling law and is not an intrusion on
any responsibility for factual determinations conferred on the jury by the Seventh Amendment.” This
standard is also used as a reference point for entry of summary judgment under Rule 56.

Thus, Corporation’s argument that its evidence was more persuasive and more credible than Plaintiff’s
evidence does not provide the court with a basis for overturning the jury’s verdict. Even if true, the
superior strength of Corporation’s evidence does not mean that the jury acted unreasonably by returning a
verdict for Plaintiff. Similarly, even if the trial judge agrees with Corporation that its witnesses were more
credible than the witnesses for Plaintiff, that fact still would be insufficient to support a motion for JMOL.
Assessments of witness credibility are core jury functions, and Rule 50 does not give the judge license to
usurp that core function.

The verdict may be overturned only if, on the basis of the evidence, no reasonable jury could have found
that Plaintiff had met her burden of proof. Here the jury had an objective, reasonable basis for accepting
Plaintiff’s version of the facts: the suspicious timing of Plaintiff’s firing and the testimony by a witness to

Seperac-J19 Exam-Released MEE Essay Compilation © 2016-2020 106


the discriminatory statements by Plaintiff’s supervisor. Even if the trial court thinks the jury erred, the
trial court should defer to the jury’s choice between competing versions of events when the jury choice is
reasonably based on the evidence presented in the case.

Explanation to Point-Three (20-30%):

Corporation’s alternative motion for a new trial should be granted if the court is convinced that the jury’s
verdict was clearly against the weight of the evidence and a miscarriage of justice, but such a result is
unlikely here.

Corporation’s motion for new trial was filed within 28 days of the entry of judgment, as required by Rule
59. Unlike motions for JMOL under Rule 50, no pre-verdict motions are required in order to make the
post-verdict motion for new trial. Thus, Corporation’s alternative motion for new trial is procedurally
proper and can be considered on the merits.

Rule 59 does not specify any standard for the granting of a new trial, but instead provides that such
motions can be granted “for any of the reasons for which new trials have heretofore been granted in
actions at law in courts of the United States.” In effect, Rule 59 gives the trial judge power and discretion
to prevent a miscarriage of justice. Because an order for a new trial is not a final judgment, such motions
are more freely granted than are motions for JMOL. In general, the grounds for a new trial are that the
verdict is against the weight of the evidence, that the damages are excessive, or that for other reasons –
such as errors of law – the trial was not fair. Where the court is convinced that a mistake has been made,
such as where the prevailing party’s witnesses were wholly lacking in credibility, a new trial may be
ordered. On a motion for new trial, unlike a motion for JMOL, the court is not required to view the
evidence in the light most favorable to the verdict winner.

In this case, the first ground offered in support of the motion is Corporation’s argument that its evidence
was more persuasive and more credible than Plaintiff’s evidence. Because the court is free to weigh the
evidence itself on a motion for new trial, this argument may prevail. Courts have held that an order for a
new trial is appropriate when the trial judge is convinced that the jury has reached a seriously erroneous
result.

However, the trial judge must respect the collective wisdom of the jury. It is not proper for a court to grant
a new trial on the ground that the verdict is against the weight of the evidence unless the record shows that
the jury’s verdict resulted in a miscarriage of justice. “Since the credibility of witnesses is peculiarly for
the jury, it is an invasion of the jury’s province to grant a new trial merely because the evidence was
sharply in conflict.” The issue here is simple and the facts were highly disputed at trial. Notwithstanding
the fact that the court may believe that Corporation’s evidence was “more persuasive,” it would be
improper for the court to grant Corporation’s motion for a new trial on this basis.

Explanation to Point-Four (20-30%):

Corporation’s motion for a new trial should be granted if a juror improperly failed to reveal information
that would bias that juror against Corporation.

Corporation’s second argument in support of a grant of new trial is stronger. By showing that a juror
improperly failed to disclose information that would bias the juror against Corporation, Corporation has
identified an error that could have influenced the verdict. Through questions during voir dire, jurors are
screened for potential bias. Although motions for a new trial based on juror nondisclosures during voir
dire are often unsuccessful, if the withheld information would have justified a disqualification for cause, a

Seperac-J19 Exam-Released MEE Essay Compilation © 2016-2020 107


new trial should be granted. Here, had Corporation known of the juror’s prior experience with
employment discrimination suits and her hostility toward companies that are accused of discrimination,
Corporation surely would have moved that the potential juror be struck for cause.

Of course, the fact that a juror has potential biases is not enough to warrant striking the juror from the jury
for cause. The question is whether those potential biases are also actual biases. Here, there is evidence that
the jury foreperson had actual bias and manifested that bias even before the trial began.

Under these circumstances, the trial court could find that the jury foreperson’s apparent bias against
Corporation fundamentally undermined the fairness of the trial. If the court so concludes, it should grant
Corporation’s motion for a new trial.

Seperac-J19 Exam-Released MEE Essay Compilation © 2016-2020 108


#022-JUL 2007–MEE Q02: QUESTION TWO (CIVIL PROCEDURE)

Al is a citizen and domiciliary of State A. While Al was visiting his parents in State B, he was involved in
an automobile accident. Al’s sports car was demolished in the accident, but he was miraculously unhurt.
He returned to his home in State A shortly after the accident. Bert, a citizen and domiciliary of State B
who was the driver of the other automobile involved in the accident, was not so lucky. Bert was seriously
injured in the accident and was hospitalized for several weeks.

Shortly after Bert’s release from the hospital, Al sued him in the federal district court for State B. Al’s
complaint properly invoked the court’s diversity jurisdiction, alleged that the collision had been caused by
Bert’s negligence, and sought $90,000 in damages (the value of Al’s demolished sports car).

Bert, who was uninsured and unemployed, failed to answer Al’s complaint and did not defend the action,
despite having been properly served and having received notice of the action. The court entered a default
judgment against Bert for $90,000. The judgment was not paid, and Al took no steps to enforce it.

One year after the accident, Al died at his home in State A. His estate is being administered by Executor,
who is a citizen of State B.

Bert recently filed a timely lawsuit against Executor, as administrator of Al’s estate. The lawsuit, filed in
state court in State B, alleges that Al’s reckless driving was the cause of the accident and that Bert is
permanently disabled by the injuries he suffered in the accident. Bert is seeking $3 million in damages
from Al’s estate.

Executor filed a timely notice of removal of the state action with the federal district court in State B. She
then served the notice on Bert and filed a copy with the State B state court. She also arranged for copies of
all records and proceedings in the state court action to be filed with the clerk of the federal district court in
State B. Executor then filed a timely motion with the federal district court to dismiss Bert’s case with
prejudice on the grounds that it was barred by the prior default judgment awarded to Al in his earlier suit
against Bert.

Bert has filed a timely motion with the federal court asking it to remand the action to state court on the
ground that the requirements for removal are not met on these facts. Alternatively, in the event the federal
court retains the action, Bert has asked it to deny Executor’s motion to dismiss.

1. Was removal of Bert’s claim to federal district court appropriate? Explain.

2. If the federal court retains the action, should it grant Executor’s motion to dismiss Bert’s suit?
Explain.

Seperac-J19 Exam-Released MEE Essay Compilation © 2016-2020 109


#022: J07-2 MEE: ANSWER: NCBE (CIVIL PROCEDURE)

POINT (1)(a) [27%] ISSUE: What are the requirements for an action to be removed from a state
court to a federal court? ANSWER: Removal of a case from a state court is appropriate if the
federal court in that state would have original jurisdiction over the action and the defendant is not a
citizen of that state.

POINT (1)(b) [27%] ISSUE: Are the requirements for removal met in a suit against a decedent’s
estate executor who is a citizen of the same state as the plaintiff but the decedent was not?
ANSWER: Yes. Because Executor is “deemed to be a citizen only of the same state as the
decedent,” the parties to the state court action were diverse and the case was properly removable to
the federal court pursuant to the court’s diversity jurisdiction.

POINT (2)(a) [18%] ISSUE: Can the default judgment against Bert have preclusive effect in
subsequent cases even though Bert never appeared to defend himself? ANSWER: Yes. A default
judgment is entitled to preclusive effect, subject only to the requirements that the rendering court
had subject matter jurisdiction and personal jurisdiction over the defendant and that the defendant
had notice of the action and an opportunity to appear and defend himself. Those requirements are
met here.

POINT (2)(b) [27%] ISSUE: Is Bert’s claim against Al so closely related to Al’s original claim that
Bert is barred from raising it in a subsequent action? ANSWER: Yes. The judgment in the prior
suit between Al and Bert precludes Bert from asserting his personal injury claim because that claim
was a compulsory counterclaim that should have been raised in the original action.

ANSWER DISCUSSION:

Bert’s tort action against Al’s estate raises a state law claim and may be removed to federal court only if
the federal court would have had diversity jurisdiction over the action had it originally been filed in that
court. Moreover, the case cannot be removed if any defendant is a citizen of the state in which the case is
pending. In this case, removal was proper. The amount in controversy clearly exceeds the $75,000
threshold. Although Executor is a citizen of State B and not diverse from Bert, she is a party to the action
only in her capacity as a representative of Al’s estate, and for diversity and removal purposes she is
“deemed to be a citizen only of the same state as the decedent.” Thus, Executor is treated as a State A
citizen, the diversity requirements are met, and removal is appropriate. Bert’s action is barred by the
default judgment in the prior suit between Bert and Al and should therefore be dismissed by the federal
court. A default judgment is conclusive and binding on the parties if, as here, the rendering court had
jurisdiction over the subject matter and the parties. Bert’s claim against Al arose out of the same
transaction or occurrence as Al’s original claim, and the same evidence would have been relevant to the
disposition of both claims. Under Federal Rule of Civil Procedure 13, Bert’s claim was a compulsory
counterclaim which Bert was required to raise in the initial action and, under standard rules of preclusion,
Bert is barred from bringing a subsequent action asserting that claim.

ANSWER EXPLANATION:

Explanation to Point-One(a) (25-35%):

Seperac-J19 Exam-Released MEE Essay Compilation © 2016-2020 110


Removal of a case from a state court is appropriate if the federal court in that state would have original
jurisdiction over the action and the defendant is not a citizen of that state.

An action filed in state court may be removed to the federal district court of the district where the state
action is pending if the district court has “original jurisdiction” over the action. The federal court would
have jurisdiction over Bert’s state law tort action against Al only if the requirements of diversity are met.
The amount in controversy must exceed $75,000 and the parties must be diverse (citizens of different
states). In addition, removal by a defendant is prohibited if the defendant is a citizen of the state where the
action is pending.

Explanation to Point-One(b) (25-35%):

Because Executor is “deemed to be a citizen only of the same state as the decedent,” the parties to the
state court action were diverse and the case was properly removable to the federal court pursuant to the
court’s diversity jurisdiction.

The requirements for removal are met here. Bert’s complaint seeks damages of $3 million for personal
injuries leading to permanent disability. This clearly exceeds the required amount in controversy, and
there is nothing to suggest that the damage claim is not made in good faith. The parties are diverse
because diversity depends on the citizenship of Bert and the decedent, Al, not the citizenship of Bert and
Executor. Although Executor is the named defendant, for diversity purposes Executor is “deemed to be a
citizen only of the same state as the decedent,” the person whose interest she represents. The decedent, Al,
was a citizen of State A and therefore was diverse from Bert, a State B citizen. The requirements of
diversity jurisdiction were therefore satisfied and the case is removable.

Moreover, although the removal statute forbids a defendant from removing a case to federal court on the
basis of diversity jurisdiction if the defendant is a citizen of the state where the action is pending, the rule
that an executor or administrator of an estate is deemed to be a citizen only of the state in which the
decedent was a citizen is applicable here as well. Because Executor is treated as a State A citizen, removal
was appropriate.

Explanation to Point-Two(a) (15-25%):

A default judgment is entitled to preclusive effect, subject only to the requirements that the rendering
court had subject matter jurisdiction and personal jurisdiction over the defendant and that the defendant
had notice of the action and an opportunity to appear and defend himself. Those requirements are met
here.

A default judgment rendered by a court is entitled to preclusive effect so long as the court had subject
matter and personal jurisdiction over the action and the parties. Although Bert did not appear and defend
himself in the original action, the federal court clearly had jurisdiction over the action and over Bert. The
parties were diverse (Bert was from State B and Al from State A) and the amount in controversy (the
value of Al’s automobile) exceeded $75,000. Moreover, as Bert was a citizen of the state in which the
action was pending, and was domiciled in the state at the time, the court had personal jurisdiction over
him. The facts also state that Bert was properly served and had notice of the action.

Explanation to Point-Two(b) (25-35%):

Seperac-J19 Exam-Released MEE Essay Compilation © 2016-2020 111


The judgment in the prior suit between Al and Bert precludes Bert from asserting his personal injury
claim because that claim was a compulsory counterclaim that should have been raised in the original
action.

Because the judgment in Al’s suit against Bert was rendered by a federal court sitting in diversity, its
judgment will be given the same preclusive effect as would be given under State B law to a State B state
court judgment (because State B is the state in which the judgment-rendering court was located). The
precise preclusive effect that should be given to a default judgment is subject to some variance among the
states, but on this issue there is substantial uniformity. It is widely accepted that a default judgment bars
the losing party from asserting defenses that could have been raised in the action or claims that were
subject to a compulsory counterclaim statute or rule in the original action.

The federal compulsory counterclaim rule, Federal Rule of Civil Procedure 13, would have required Bert
to bring his claim against Al in the first action, had Bert actually defended that action. Rule 13 requires a
defendant to “state as a counterclaim any claim” that he or she has against the plaintiff if the claim “arises
out of the transaction or occurrence that is the subject matter” of the plaintiff’s claim. Here, Bert’s claim
against Al for damages arises out of the same occurrence (the traffic accident) as Al’s previous claim
against Bert. Moreover, key issues of law and fact raised by both claims would be the same as both claims
involve the basic question of who was at fault in the accident. Much of the evidence that would support or
refute Al’s claim against Bert is likely to be relevant to Bert’s claim against Al. In short, the two claims
are logically, factually, and transactionally related. These are precisely the circumstances under which the
federal rules require Bert to assert his claim as a counterclaim. Because Bert’s claim for damages was a
compulsory counterclaim that should have been raised in the original action, the judgment in that original
action bars the claim. It does not matter that Bert did not actually litigate in the prior case but merely
allowed a default judgment to be entered against him.

[NOTE: Because the question of the preclusive effect of this judgment involves the effect of the federal
counterclaim rule, some applicants might argue that the preclusion issue is governed by federal law and
not by state preclusion rules. State law governs the preclusive effect of a diversity judgment absent
substantial federal concern. But it does not really matter whether an applicant concludes that state or
federal preclusion rules apply. The basic rule – that a default judgment bars the defendant from raising in
future litigation a claim that was subject to a compulsory counterclaim statute or rule in the original
litigation – is well established in state and federal practice.]

Seperac-J19 Exam-Released MEE Essay Compilation © 2016-2020 112


#023-FEB 2007–MEE Q03: QUESTION THREE (CIVIL PROCEDURE)

Transit Authority, Inc. (Transit Authority) operates a bus system in Big City. Last month, a Transit
Authority bus collided with a passenger car driven by Tourist. The accident occurred when Tourist
suddenly veered into the bus operator’s lane at a major intersection. The bus operator was unable to stop
the bus in time to avoid the collision, and Tourist was injured. Immediately after the accident occurred,
the bus operator telephoned his supervisor to report the accident. Then, following Transit Authority’s
standard procedures, the bus operator completed an “Operator’s Report of Accident” form. The completed
form included the date, time, and place of the accident, the road conditions, the names of witnesses, a
brief description of how the accident occurred, and a description of the personal injuries and property
damage caused by the accident.

When a Transit Authority supervisor arrived twenty minutes after the accident occurred, she took a
statement from the bus operator and recorded that statement on a “Supervisor’s Investigative Report”
form. Then she interviewed Tourist and recorded Tourist’s statement on the “Supervisor’s Investigative
Report” form. The supervisor noted all witnesses’ names, addresses, and telephone numbers in her report.
She took photographs of the accident scene, including the position of each vehicle. Finally, she drew a
diagram of the scene on the last page of the “Supervisor’s Investigative Report” form.

Tourist has filed a personal injury action against Transit Authority in federal court, properly invoking the
court’s diversity jurisdiction. Tourist alleges that the bus operator, Transit Authority’s employee, was
driving negligently. She further alleges personal injury and property damage in a total amount exceeding
$200,000. Transit Authority has filed an answer denying the claim of negligence and asserting
contributory negligence.

Tourist served two requests for production of documents on Transit Authority. One request was for “any
and all accident reports, diagrams, photographs, and any other documents which relate in any way to the
collision between the bus and the car.” A second request was for the bus operator’s “entire personnel file
that is maintained by Transit Authority, including disciplinary actions, safety records, and driving
records.” Transit Authority has refused to produce the accident reports that the operator and the supervisor
created on the grounds that the reports were “prepared in anticipation of litigation.” In addition, Transit
Authority refuses to produce the bus operator’s personnel file because the information that it contains “is
not relevant.”

Tourist has made a motion to compel production of the accident reports and the bus operator’s “entire
personnel file.”

Should Tourist’s motion be granted in whole or in part? Explain.

Seperac-J19 Exam-Released MEE Essay Compilation © 2016-2020 113


#023: F07-3 MEE: ANSWER: NCBE (CIVIL PROCEDURE)

POINT (1) [40%] ISSUE: Are accident and investigative reports discoverable when they are
prepared in the ordinary course of business and not in anticipation of any immediately pending
litigation? ANSWER: Yes. The motion to compel production of the accident and investigative
reports should be granted because they were not prepared in anticipation of litigation.

POINT (2) [60%] ISSUE: Is information in a bus operator’s personnel file about the operator’s
driving record, safety record, and disciplinary record discoverable in an action against the bus
operator’s employer that alleges the bus operator was driving negligently? ANSWER: Under FRCP
Rule 26, the bus operator’s driving and safety records are relevant because they address his driving
skills and care in a case that involves his alleged negligent driving, but his disciplinary record may
not be relevant. Thus, the court is likely to order disclosure of the personnel file, but it may restrict
disclosure or use of the information in some fashion.

ANSWER DISCUSSION:

The motion to compel should be granted in part and denied in part. FRCP Rule 37 provides that a party
may be compelled to provide discovery. Here, the accident and investigative reports are not trial
preparation materials that are protected under FRCP Rule 26. They are routine reports that were not
prepared in anticipation of litigation. Records in the operator’s personnel file regarding his driving and
safety record are relevant and discoverable under FRCP Rule 26. That information is related to his driving
skills and his employer’s potential liability. On the other hand, the operator’s disciplinary records may not
be discoverable. While some disciplinary information may relate to the issues in the case, other
disciplinary information may have nothing whatever to do with the bus operator’s driving skills, and its
disclosure would serve simply to harass and embarrass the bus operator. A court may order an in camera
review to excise irrelevant material before allowing release of the personnel file.

ANSWER EXPLANATION:

Explanation to Point-One (35-45%):

The motion to compel production of the accident and investigative reports should be granted because they
were not prepared in anticipation of litigation.

Under FRCP Rule 26, a party “may obtain discovery regarding any matter, not privileged, that is relevant
to a claim or defense” in the action. Relevant information is discoverable even if it would be inadmissible
at trial, so long as “the discovery appears reasonably calculated to lead to the discovery of admissible
evidence.” There is no question that the bus operator’s accident report and his supervisor’s investigative
report satisfy this relevancy requirement.

On the other hand, FRCP Rule 26 protects trial preparation materials from discovery. Materials “prepared
in anticipation of litigation or for trial by a party” are discoverable only if the party seeking discovery can
show a “substantial need” for the materials and an inability “without undue hardship to obtain the
substantial equivalent of the materials by other means.”

Thus, the crucial question is whether the accident reports that the bus operator and the supervisor
completed were “prepared in anticipation of litigation” within the meaning of this rule. On the facts of this

Seperac-J19 Exam-Released MEE Essay Compilation © 2016-2020 114


problem, they were not. Both the bus operator’s report and the supervisor’s report appear to be standard
Transit Authority business records that are routinely prepared in the event of an accident. The facts say
that the bus operator was following standard procedure when he called his supervisor and when he
completed the “Operator’s Report of Accident” form. Similarly, the supervisor was following standard
procedure when she went to the accident scene and completed the “Supervisor’s Investigative Report”
form. While Transit Authority may honestly claim that it has such procedures precisely because it
“anticipates” that there may be litigation whenever there is an accident, routinely prepared records that are
completed in the regular course of business do not become trial preparation materials merely because the
business anticipates that they might be useful in litigation.

The timing for completion of the reports supports the conclusion that they were not prepared in
anticipation of litigation. The bus operator began to complete his report immediately after the accident
occurred. The supervisor completed the investigative report on the same day of the accident and within
one hour of the time that the accident occurred. Accident and investigative reports like the ones described
here are protected from discovery only when they are prepared in response to a threat of imminent
litigation. There was no threat of litigation when these reports were completed. Therefore, these materials
are discoverable.

Explanation to Point-Two (55-65%):

Under FRCP Rule 26, the bus operator’s driving and safety records are relevant because they address his
driving skills and care in a case that involves his alleged negligent driving, but his disciplinary record may
not be relevant. Thus, the court is likely to order disclosure of the personnel file, but it may restrict
disclosure or use of the information in some fashion.

As noted above, parties to an action “may obtain discovery regarding any matter, not privileged, which is
relevant to the claim or defense of any party.” Even if not directly relevant to a claim or defense, a matter
that is relevant to the general “subject matter involved in the action” may be discovered if the court
determines that there is good cause for such discovery. The facts and the circumstances of each particular
case determine the relevancy of information that a party seeks during discovery. Accordingly, in its
discretion, and in light of the facts of each case, the court decides whether the requested information is
relevant.

Personnel records are discoverable, despite frequent claims that they should be protected for privacy
reasons. Where discovery is resisted, the court must balance the interest of the party seeking discovery
against any alleged privacy interest that would be infringed by disclosure.

Here, Tourist has alleged that the bus operator was negligent. To the extent that the driving and safety
records in the personnel file include material about this accident, they are clearly relevant to Tourist’s
claim. The bus operator’s more general driving and safety records are a closer question. If they reveal a
pattern of poor driving, they might be relevant to the claim against Transit Authority.

Even if it cannot be shown that the driving and safety information is directly relevant to the specific claim
in this case, this seems a good case for showing good cause to compel production of the information
because it is relevant to the subject matter of the lawsuit. An investigation of the bus operator’s driving
and safety record might lead to information of direct benefit to Tourist’s case. Therefore, information in
the bus operator’s personnel file about his driving skills, including his driving record and any other
accident reports, is discoverable. The motion to compel production of the personnel file should be
granted, with respect to the driving records.

Seperac-J19 Exam-Released MEE Essay Compilation © 2016-2020 115


Regarding the bus operator’s disciplinary records, the discovery request is much more troubling. The
personnel record may well contain disciplinary information about the bus operator that could be entirely
irrelevant to the action and that would be embarrassing and damaging to the bus operator if disclosed
(e.g., that he has been disciplined for sexual harassment). On the other hand, it is conceivable that the
records might include evidence that could bolster Tourist’s case, such as evidence of disciplinary action
taken against the bus operator for driving-related behavior.

Under these circumstances, a court may limit Tourist’s access to the bus operator’s personnel files by
restricting access to those portions deemed clearly relevant to the action. For example, in an action
alleging driving negligence, defendants are required to make available information in their personnel file
including driving records and disciplinary records related to driving skills, but forbidding disclosure of
other information. Alternatively, a court might order an in camera review of the records to determine
whether they contain anything of “a particularly sensitive or private nature” that warrants protection. Or, a
court may order release of the entire file but enter a confidentiality order restricting access to the file to
attorneys and parties reviewing it for purposes of the litigation.

Seperac-J19 Exam-Released MEE Essay Compilation © 2016-2020 116


#024-JUL 2006–MEE Q06: QUESTION SIX (CIVIL PROCEDURE)

Pat, a State A resident, was driving in State B when her car was struck by a truck driven by Driver. A sign
on the truck’s door read “Smith Brothers Transport Co.” The police accident report correctly listed
Driver’s name and license number, but mistakenly identified the truck’s owner as Smith Brothers
Trucking Company, Inc., instead of the correct owner, Smith Brothers Transport Company, Inc. Smith
Brothers Trucking and Smith Brothers Transport are both owned and operated by Robert and William
Smith. The two companies share office space. The Smith brothers are careful, however, to maintain the
two corporations as separate and distinct legal entities, following all legal requirements to avoid any alter
ego problems. Both companies are incorporated in and have their principal places of business in State B.
Both companies identify Robert Smith as agent for service of process.

Two days before the deadline for filing suit under the applicable statute of limitations, Pat filed a proper
diversity action in federal court in State B, alleging damages exceeding $75,000 against both Driver and
Smith Brothers Trucking (the incorrect defendant). The summons and complaint were promptly served on
Robert Smith, as agent for Smith Brothers Trucking.

Five days after filing her complaint and before any responsive pleading had been served, Pat’s attorney
realized that she had incorrectly named Smith Brothers Trucking as defendant. She promptly amended her
complaint to name Smith Brothers Transport as defendant and served that amended complaint and an
amended summons on Robert Smith, as agent for Smith Brothers Transport.

Two days later, Smith Brothers Transport moved to dismiss the amended complaint on the ground that the
applicable statute of limitations had run before the amended complaint was filed. The court denied this
motion and refused Smith Brothers Transport’s request for certification of the matter as appropriate for an
immediate appeal.

Following the filing of responsive pleadings, the judge directed the attorneys for all parties to appear
before her for a pretrial conference to discuss the possibility of settlement. The attorneys for Pat and
Smith Brothers Transport appeared, but the attorney for defendant Driver did not appear. Instead, Driver’s
attorney left a message for the judge, stating that he would not appear because Driver was not prepared to
engage in settlement discussions.

The judge noted Driver’s attorney’s nonappearance and conducted the pretrial conference between the
attorneys representing Pat and Smith Brothers Transport. The judge also ordered Driver’s attorney to
appear before her to explain why she should not impose sanctions on Driver for his attorney’s non-
attendance at the pretrial conference. Driver’s attorney appeared and argued that he did not attend the
pretrial conference because his client had no intention of negotiating a settlement and had directed him
(the attorney) not to “waste my money” by attending. The judge, angry at this apparent disregard of her
authority, ordered that Driver’s answer be stricken and that a post-answer default judgment be entered
against Driver.

1. Did the trial court err in refusing to dismiss the amended complaint against Smith Brothers
Transport? Explain.

2. May Smith Brothers Transport immediately appeal the denial of its motion to dismiss? Explain.

Seperac-J19 Exam-Released MEE Essay Compilation © 2016-2020 117


3. Did the trial court err in striking Driver’s answer and ordering the entry of a default judgment as
a sanction for the failure of Driver’s attorney to participate in a pretrial conference? Explain.

Seperac-J19 Exam-Released MEE Essay Compilation © 2016-2020 118


#024: J06-6 MEE: ANSWER: NCBE (CIVIL PROCEDURE)

POINT (1) [33%] ISSUE: Can Smith Brothers Transport assert a statute-of-limitations bar to an
action when it was added to the action by an amended complaint filed after the applicable statute of
limitations had run? ANSWER: No. Because Pat’s amended complaint was served on Smith
Brothers Transport within the time for original service of process under FRCP Rule 4, and because
Smith Brothers Transport knew or should have known of the mistaken identity and would not be
prejudiced by the delay, the court properly allowed the amended complaint to relate back to the
date of the original filing.

POINT (2) [33%] ISSUE: May a defendant immediately appeal a judge’s denial of a motion to
dismiss when the defendant claims that an action against it is barred by the statute of limitations?
ANSWER: No. Because defendant Smith Brothers Transport’s appeal does not fall within any of
the exceptions to the final judgment rule, the denial of the motion to dismiss may be appealed only
after the trial is concluded and the judgment is final.

POINT (3) [33%] ISSUE: Does the trial court have the power to dismiss a party’s defense and enter
default judgment against the party because the party’s attorney failed to appear at a pretrial
conference? ANSWER: No. The trial judge may have abused her discretion in entering a default
judgment against Driver for his attorney’s failure to appear at the pretrial conference, at least
absent any other instances of misbehavior by Driver or his attorney or any evidence that lesser
sanctions would not work.

ANSWER DISCUSSION:

Pat’s amendment of her complaint to name the proper defendant, Smith Brothers Transport, meets the
requirements necessary for the amendment to relate back to the date of the original pleading. Thus, Pat’s
action against Smith Brothers Transport should be treated as timely filed, notwithstanding that Smith
Brothers Transport was not added to the action until after the limitations period had run. The trial judge
therefore correctly denied the motion to dismiss. Smith Brothers Transport may not immediately appeal
the denial of its motion to dismiss. The denial of a motion to dismiss is not a final judgment, and the
judge’s order in this case does not fall within any of the exceptions to the final judgment rule. Thus, the
decision may be appealed only after the trial is concluded and the judgment is final. Driver’s attorney
failed to appear as ordered at the pretrial conference because Driver didn’t wish to discuss settlement.
Although a trial judge has the power to sanction such behavior, including by entering a default judgment,
the sanction in this case was probably so harsh as to be an abuse of discretion, at least absent other
misbehavior by Driver or evidence that a lesser sanction would have been ineffective in securing future
adherence to the judge’s orders.

ANSWER EXPLANATION:

Explanation to Point-One (30-40%):

Because Pat’s amended complaint was served on Smith Brothers Transport within the time for original
service of process under FRCP Rule 4, and because Smith Brothers Transport knew or should have
known of the mistaken identity and would not be prejudiced by the delay, the court properly allowed the
amended complaint to relate back to the date of the original filing.

Seperac-J19 Exam-Released MEE Essay Compilation © 2016-2020 119


Federal Rule of Civil Procedure 15 allows a complaint to be amended “once as a matter of course at any
time before a responsive pleading is served.” Thus, Pat’s complaint was properly amended.

Not only can Pat amend her complaint to name the proper defendant, Smith Brothers Transport, but her
amendment will “relate back to the date of the original pleading” if certain requirements are met. An
amendment that “changes the party or the naming of the party against whom a claim is asserted” relates
back to the date of the original pleading when:

• the claim asserted in the amended pleading “arose out of the conduct, transaction, or occurrence set forth
or attempted to be set forth in the original pleading,” and

• within 90 days of the original filing of the action (the normal time limit for service of the summons and
complaint under FRCP Rule 4, the new party “has received such notice of the institution of the action”
that it will “not be prejudiced in maintaining a defense on the merits,” and the new party

• “knew or should have known that, but for a mistake concerning the identity of the proper party, the
action would have been brought against” it.

Each of these requirements for relation back is met here. The amended complaint asserted the same claim,
arising out of the same occurrence, as had been originally asserted against Smith Brothers Trucking, thus
satisfying the first requirement. As to the second requirement, the facts suggest that both the original
summons and complaint and the amended summons and complaint were served within the 90-day
summons period. Given that Smith Brothers Trucking and Smith Brothers Transport have common
owners, common premises, and the same agent for service of process (Robert Smith), either service of the
summons and complaint would have given Smith Brothers Transport sufficient “notice of the institution
of the action” that it would “not be prejudiced in maintaining” its defense.

Robert Smith received notice of the commencement of the original action and at that point he “should
have known” that a lawsuit was being filed for Pat’s accident and that a mistake had been made as to the
proper defendant. Even if he did not know at that point, he surely knew when the amended complaint was
served. Thus, the amendment naming Smith Brothers Transport relates back to the date of the original
filing, and the motion to dismiss on statute of limitations grounds should be denied.

Explanation to Point-Two (30-40%):

Because defendant Smith Brothers Transport’s appeal does not fall within any of the exceptions to the
final judgment rule, the denial of the motion to dismiss may be appealed only after the trial is concluded
and the judgment is final.

In most cases, a party may appeal only from a final judgment. A final judgment is a controlling and
dispositive order that “ends the litigation on the merits and leaves nothing for the court to do but execute
on the judgment.” In addition, FRCP Rule 54(b) allows a trial judge to sever portions of a multi-claim
case for appeal with a finding that “there is no just reason for delay” and with a direction for the entry of
judgment on the decided claim(s) or defense(s). But unless this is done, early decided claims must
ordinarily await final judgment as to all claims before they are eligible for appeal. Under either the
standard final judgment rule or FRCP Rule 54(b), Smith Brothers Transport cannot appeal the denial of its
motion to dismiss. Rather than ending the litigation on the merits or terminating a particular claim, the
trial judge’s decision allowed the litigation to continue. Smith Brothers Transport was joined to the action
and now faces trial.

Seperac-J19 Exam-Released MEE Essay Compilation © 2016-2020 120


This issue also does not qualify for any of the statutory exceptions to the final judgment rule. It is not an
interlocutory appeal from an order granting or dissolving an injunction, and it does not qualify as a
certified appeal because the trial judge refused to certify the question,

The judge-made collateral order exception to the final judgment rule is also inapplicable. The collateral
order rule allows an immediate appeal from an order that (1) “finally determines” a “claim of right,” (2)
that is separable from, and collateral to, the rights asserted in the main action, and (3) that is “too
important to be denied review.” The Court restated the doctrine in Coopers & Lybrand v. Livesay: “To
come within the ‘small class’ of decisions excepted from the final judgment rule by Cohen, the order must
conclusively determine the disputed question, resolve an important issue completely separate from the
merits of the action, and be effectively unreviewable on appeal from a final judgment.” The denial of
Smith Brothers Transport’s motion to dismiss fails on all counts. It does not conclusively determine the
disputed question (whether the statute of limitations has run), the issue is not completely separate from the
merits, but is instead intertwined with the merits, and the question is reviewable on appeal.

[NOTE: A writ of mandamus may sometimes be used to secure review of a trial court’s action. Mandamus
is a drastic remedy, to be used only when justice demands and less extraordinary remedies are
unavailable. Mandamus is generally used by an appellate court “to prevent a district court from acting
beyond its subject matter jurisdiction, or to compel it to take action that it lacks the power to withhold.”
On the facts of this problem, mandamus is inappropriate because the trial court’s decision in no way
involved action beyond the court’s power. To the contrary, the trial court clearly has the power under the
Federal Rules to permit an amendment of a complaint to relate back to the date of the original complaint
for statute-of-limitations purposes. Thus, there was no abuse of power here.]

Explanation to Point-Three (30-40%):

The trial judge may have abused her discretion in entering a default judgment against Driver for his
attorney’s failure to appear at the pretrial conference, at least absent any other instances of misbehavior by
Driver or his attorney or any evidence that lesser sanctions would not work.

Federal Rule of Civil Procedure 16 authorizes a district judge to convene pretrial conferences for the
purpose of ensuring that cases are processed effectively and efficiently. Although courts are not required
to hold a pretrial conference, once a conference is ordered, appearance is compulsory. The scheduling and
running of pretrial conferences is largely within the trial court’s discretion. Moreover, the court has the
power to sanction a party or the party’s attorney if “no appearance is made on behalf of the party at a
pretrial conference.” Possible sanctions include any order authorized under FRCP Rule 37, including “an
order striking out pleadings or dismissing the action or rendering a judgment by default against the
disobedient party.”

Thus, the trial judge had the power to sanction Driver for the nonappearance of Driver’s attorney.
Moreover, the sanction imposed (striking Driver’s pleading and rendering judgment against him) is
authorized by the rules. Nonetheless, Driver may prevail on appeal if he can show that the trial judge
abused her discretion in light of the facts and the severity of the sanction.

Driver’s attorney stated that he did not attend the pretrial conference because Driver was unwilling to
settle the case and, accordingly, did not want the attorney to waste money by attending an unnecessary
hearing. This explanation of non-attendance is plainly inadequate. Pretrial conferences have multiple
purposes. In addition to facilitating settlement, they are used to establish early court control over an action
to ensure good case management and to discourage “wasteful pretrial activities.” Driver’s assessment that

Seperac-J19 Exam-Released MEE Essay Compilation © 2016-2020 121


his attorney’s attendance at the conference would be a “waste” does not justify the disregard of the court’s
order.

Nonetheless, the court’s sanction against Driver was probably so harsh as to be an abuse of discretion.
Dismissal of an action or the entry of a default judgment are the most severe sanctions available to a
district court and are usually appropriate “only when a party’s misconduct is serious, repeated,
contumacious, extreme, or otherwise inexcusable.” Lesser sanctions should be utilized before the more
extreme sanctions, absent reason to believe that lesser sanctions would be ineffective to secure
compliance with court orders. Absent additional facts showing repeated defiance of the court or the likely
ineffectiveness of lesser sanctions, the trial judge’s decision to enter a default judgment in this case was
probably an abuse of her discretion to “make such orders (with regard to the attorney’s failure to attend
the conference) as are just.”

Seperac-J19 Exam-Released MEE Essay Compilation © 2016-2020 122


#025-FEB 2006–MEE Q03: QUESTION THREE (CIVIL PROCEDURE)

Defendant is a political commentator and free-lance journalist who moved from his home state of State A
to State B approximately three years ago. Defendant has told his family and friends that he still considers
State A “home” and intends to return “someday,” but that “I’m happy in State B for now.” Defendant
votes, pays taxes, and owns property only in State B.

Defendant publishes an online newsletter called “Nothing But the Truth” that is accessible over the
Internet from his website, www.NBT.com. Viewers can access www.NBT.com and download articles,
but they cannot post their messages through the site. The central computer that people access when they
view the website is located in State B.

The political commentary and journalism posted on the www.NBT.com website focus almost exclusively
on people and events in State A – Defendant’s old stomping grounds and the region where he is best
known as a political reporter. Indeed, Defendant’s website receives so many hits from people in State A
that most of the advertisers on the website are State A firms seeking to attract customers who live in State
A.

Plaintiff is a resident and domiciliary of State A who previously worked as a high-level State A
government employee. In a recent story posted on his website, Defendant reported on rumors he had
heard concerning Plaintiff’s alleged acceptance of kickbacks for the award of State A contracts.

Shortly after Defendant published this story on his website, Plaintiff was dismissed from employment
with State A. Plaintiff maintains that the allegations contained in Defendant’s article about Plaintiff are
patently false. Plaintiff contacted Defendant shortly after the initial publication, asking Defendant to
retract the story. Defendant refused and insisted that he was protected by the First Amendment of the
United States Constitution, even if the story was false, so long as he had not acted maliciously.

Plaintiff has filed a lawsuit against Defendant in the United States District Court for the District of State
A. Her complaint alleges that the story published on Defendant’s website defamed Plaintiff under
applicable state law. In addition, the complaint asserts that the First Amendment does not shield
Defendant from the defamation claim under these circumstances. The complaint alleges facts establishing
damages for Plaintiff in the amount of $200,000. Convinced that Defendant acted maliciously, Plaintiff
also seeks punitive damages in the amount of $1,000,000. The complaint alleges that jurisdiction is proper
on both federal question and diversity grounds.

The State A long-arm statute provides that the courts of State A may exercise jurisdiction over absent
defendants to the “full extent permitted by the due process clause of the United States Constitution.”

In lieu of an answer to the complaint, Defendant has filed a motion to dismiss for lack of subject matter
and personal jurisdiction.

1. On what basis, if any, would a federal district court have subject matter jurisdiction over the
lawsuit by Plaintiff against Defendant? Explain.

2. Would the United States District Court for the District of State A have personal jurisdiction over
Defendant? Explain.

Seperac-J19 Exam-Released MEE Essay Compilation © 2016-2020 123


#025: F06-3 MEE: ANSWER: NCBE (CIVIL PROCEDURE)

POINT (1)(a) [16%] ISSUE: Does a federal district court have federal question jurisdiction over a
claim based on the state law of defamation when the plaintiff’s complaint alleges that an anticipated
First Amendment defense is invalid? ANSWER: No. The federal district court does not have federal
question jurisdiction because the complaint alleges a state law tort claim and federal issues exist
only as a possible defense.

POINT (1)(b) [37%] ISSUE: Does a federal district court have diversity jurisdiction when the
defendant previously resided in the same state as the plaintiff, still considers that state “home,” and
plans to return someday? ANSWER: The federal district court will have subject-matter
jurisdiction based on diversity of citizenship under 28 U.S.C. § 1332 only if Defendant is considered
a domiciliary of State B.

POINT (2) [47%] ISSUE: Can a federal district court in State A assert personal jurisdiction over a
defendant whose activities occurred outside State A when those activities included posting allegedly
defamatory material concerning a State A plaintiff on a website viewable by State A residents and
when the defendant knows that the website is viewed by State A residents and derives revenue as a
result of that fact? ANSWER: Yes. Defendant may be subject to personal jurisdiction in State A
because he appears intentionally to have targeted State A readers from his State B-based website.

ANSWER DISCUSSION:

Federal question jurisdiction is not available in this case because the complaint asserts a tort claim under
state law. The well-pleaded complaint rule governs here; thus the negation of the federal defense in the
complaint would not create federal question jurisdiction under Whether the court has diversity jurisdiction
over the case depends on whether Defendant is considered to have established a domicile in State B. If he
is domiciled in State B, the parties are diverse and the amount in controversy is sufficient for jurisdiction.
If he is still a State A domiciliary, then diversity is defeated. The matter can be argued either way, but the
facts appear to support a conclusion that Defendant’s current domicile is in State B because his intent is to
remain and live there for the time being. The question of personal jurisdiction over Defendant based on
his publication of an alleged defamation over the Internet asks whether Defendant had sufficient contact
with State A to justify the assertion of personal jurisdiction. Under the approach of most courts, this will
turn on the degree to which Defendant targeted State A readers and the State A-based Plaintiff and
published the story with knowledge that it might cause harm to Plaintiff in State A. A consideration of the
“fairness/reasonableness” factors would also be necessary.

ANSWER EXPLANATION:

Explanation to Point-One(a) (10-20%):

The federal district court does not have federal question jurisdiction because the complaint alleges a state
law tort claim and federal issues exist only as a possible defense.

Plaintiff’s complaint does not raise a federal question but, instead, asserts a tort claim under state law.
According to the well-pleaded complaint rule, a plaintiff’s anticipation of a federal defense in the
complaint does not create federal question jurisdiction. Thus, Plaintiff’s allegation that Defendant has no
defense under the First Amendment does not, in itself, create subject-matter jurisdiction.

Seperac-J19 Exam-Released MEE Essay Compilation © 2016-2020 124


Explanation to Point-One(b) (30-40%):

The federal district court will have subject-matter jurisdiction based on diversity of citizenship under 28
U.S.C. § 1332 only if Defendant is considered a domiciliary of State B.

The complaint states a claim well in excess of the $75,000 jurisdictional threshold required for diversity
jurisdiction. Thus, the question is whether there is complete diversity between Plaintiff and Defendant.
Plaintiff is a citizen of State A by virtue of her residence and domicile in State A. If Defendant is a citizen
of State B, there is diversity. If Defendant is still a citizen of State A (where he lived until three years
ago), then there is no diversity between Defendant and Plaintiff, and the court lacks jurisdiction.

For purposes of the diversity statute, a person is considered a citizen of the state in which he is domiciled.
A person’s domicile is his or her permanent home, the place where a person intends to remain
indefinitely, and the place to which the person intends to return when temporarily absent.

Under the facts, there is a strong argument that State B has become Defendant’s new domicile. He has
lived there for three years; he votes, pays taxes, and owns property there. The center of his life appears to
be State B, and he appears to intend to establish his permanent home there for the time being at least.

One might argue, on the other hand, that Defendant’s expressed intent to return to State A someday and
his statement that he views State A as his home mean that his domicile is still in State A; State B is but a
temporary sojourn. While such an argument is plausible, the facts that Defendant’s residence in State B
has some permanence, that he treats it as his current permanent home for all purposes, and that he has no
intention of returning to State A at any definite future time, make it most likely that a court would
conclude that his domicile is in State B and that he has become a State B citizen for diversity purposes.

Explanation to Point-Two (40-50%):

Defendant may be subject to personal jurisdiction in State A because he appears intentionally to have
targeted State A readers from his State B-based website.

Under Rule 4 of the Federal Rules of Civil Procedure, the federal district court can assert jurisdiction over
a defendant who could be subjected to jurisdiction in the state courts of State A. Because of State A’s
long-arm statute (which is a general long-arm statute that extends to the constitutional limits), the only
inquiry here is the constitutionality of the assertion of personal jurisdiction.

The Due Process Clause of the Fourteenth Amendment permits a court to exercise general personal
jurisdiction over a foreign defendant when that defendant maintains substantial, continuous, and
systematic contacts with the forum state. Where such contacts exist, the state may assert jurisdiction
whether or not the claim is related to the defendant’s contacts. Alternatively, a state may exercise specific
personal jurisdiction where a nonresident defendant has purposefully availed itself of the benefits and
protections of the forum state through minimum contacts, the claim arises out of or relates to those
contacts, and the exercise of jurisdiction does not offend traditional notions of fair play and substantial
justice.

Although the United States Supreme Court has not yet addressed the question of personal jurisdiction for
Internet-based contacts, many lower courts have grappled with the question. Since websites on the
Internet can be viewed everywhere, most courts have been reluctant to hold that the maintenance of a
website alone is sufficient to subject the owner of the site to jurisdiction everywhere. One of the leading
opinions is Zippo Mfg. Co. v. Zippo Dot Com, Inc. Zippo suggested that the more interactive a website,

Seperac-J19 Exam-Released MEE Essay Compilation © 2016-2020 125


the more likely courts should be to uphold personal jurisdiction, and that a purely passive website would
not be sufficient to establish jurisdiction. Here, because Defendant’s website, www.NBT.com, is passive,
that is, it does not enable viewers to interact by posting their own messages, general jurisdiction based
solely on the newsletter’s accessibility across the United States would be unlikely.

However, Defendant did more than simply maintain a website that was accessed by State A residents.
Defendant’s website was largely devoted to commentary and reporting on events in State A, and
Defendant knew (and profited from) the fact that many of his readers came from that area. In Calder v.
Jones, the U.S. Supreme Court held that a California court could exercise jurisdiction over a Florida
writer and a Florida editor of a defamatory article about a California resident’s conduct in California. The
Court reasoned that the defendants knew that their magazine (the National Enquirer) had its largest
circulation in California and that the injury would be felt by the plaintiff in her home state. Under those
circumstances, the defendants must have reasonably anticipated being haled into court in California when
sued for the defamatory nature of their article. Lower courts have applied the logic of Calder to uphold
jurisdiction where defamatory communications over the Internet intentionally targeted readers in the
plaintiff’s home state. For example, one court rejected jurisdiction where the Internet posting did not
manifest an intent to target an audience in the forum state. In contrast, another court upheld jurisdiction
where the Internet communication did target a forum resident.

In this case, the facts likewise suggest a targeting of State A readers and a State A plaintiff in such a way
that Defendant could “reasonably anticipate” being haled into State A courts to answer a complaint by
Plaintiff against him. Not only did Defendant know that State A readers were likely to view his website,
but Defendant could reasonably predict that the impact of the article would be felt by Plaintiff in State A.
In addition, Defendant’s commentary concerned actions taken in part in State A.

Although Defendant has not engaged in any physical conduct in State A, his targeting of a State A
resident, the commercial benefits he derives from serving State A readers, and the large State A
readership he has all add up to a strong case for specific personal jurisdiction over Defendant for purposes
of this action. State A has a strong interest in the case, Plaintiff has a strong interest in having it tried in
State A, and the needs of interstate commerce would be served if defendants who cause harm through
instruments of commerce (e.g., the Internet) were subject to being held to account where foreseeable harm
occurs, at least if the defendant’s primary business activity is directed at the forum state. It is
unreasonable to exercise jurisdiction in the absence of a forum interest, when the defendant has no
commercial or other ties to the forum apart from a product ending up there through the stream of
commerce. On the other hand, the proper reach of state court jurisdiction in Internet cases has not been
definitively decided, and courts may be reluctant to extend jurisdiction to reach a defendant who, like
Defendant, does not actively reach out to State A readers but simply maintains a passive website to which
State A readers can direct their web browsers should they so choose.

Seperac-J19 Exam-Released MEE Essay Compilation © 2016-2020 126


#026-JUL 2005–MEE Q06: QUESTION SIX (CIVIL PROCEDURE)

Drugco, a State B corporation with its principal place of business in State B, employs sales
representatives and assigns them to work in exclusive geographical areas. The Drugco sales
representatives provide pharmacists in their areas with product literature, pricing information, and highly
confidential sheets (known as “chem sheets”) listing the chemical composition of each drug. The chem
sheets ensure that the pharmacists understand how each Drugco drug interacts with other commonly
prescribed drugs.

Drugco hired Clair, a resident of State A, under an employment agreement that contained a “non-
compete” clause barring her, for one year following the termination of her employment with Drugco,
from soliciting any pharmacists in her territory (defined as Elm, Maple, and Cherry counties of State A)
or from engaging in any other activity competitive with Drugco’s business within her territory. The
agreement also included a trade-secrets clause, which required Clair to return to Drugco all pricing
information and chem sheets in her possession upon termination of her employment.

The laws of State A and State B provide that: (1) chemical formulas and drug pricing information are
trade secrets and (2) non-compete clauses in employment agreements are valid so long as they are
reasonable in duration and geographical scope.

Clair was an excellent sales representative for Drugco, and she quickly developed strong working
relationships with the pharmacists in her territory. Eighteen months after she began working for Drugco,
Clair tendered her resignation to Drugco and the next day began working for Medico, a competing
pharmaceuticals firm. Clair, who had retained a number of Drugco chem sheets, was contacting the same
pharmacists in the same geographical area that she had covered for Drugco. Moreover, she was selling
Medico products that were in direct competition with Drugco products.

Drugco has properly invoked diversity jurisdiction and has sued Clair in federal district court in State A
for breach of contract.

Drugco seeks to enforce the non-compete clause in Clair’s employment contract and to bar her from
soliciting the pharmacists with whom she developed relationships while working for Drugco. Drugco
fears that it may take more than a year to litigate the case to final judgment and wants to bar Clair from
soliciting these pharmacists now. Drugco also wants to compel Clair to return immediately all Drugco
chem sheets and pricing information within her possession.

What provisional remedies might Drugco seek to enforce the non-compete and trade-secrets clauses
immediately and during the pendency of the lawsuit, and what is the likelihood that the court will grant
each such remedy? Explain.

Seperac-J19 Exam-Released MEE Essay Compilation © 2016-2020 127


#026: J05-6 MEE: ANSWER: NCBE (CIVIL PROCEDURE)

POINT (1) [14%] ISSUE: Does the federal district court have authority immediately and during the
pendency of the lawsuit to enjoin Clair from violating the non-compete provision in the contract
and/or from retaining the chem sheets and pricing information? ANSWER: Yes. The district court
in State A has authority under Rule 65 of the Federal Rules of Civil Procedure to issue a temporary
restraining order prohibiting Clair from violating the non-compete provision in her contract, but
such an order must expire within a few days and Drugco must seek a preliminary injunction.

POINT (2) [48%] ISSUE: Is the federal district court likely to enjoin Clair from violating the non-
compete clause throughout the pendency of the lawsuit? ANSWER: Yes. The district court may
grant a preliminary injunction to enjoin Clair from violating the non-compete provision in the
contract during the pendency of the lawsuit if it finds that Drugco is likely to succeed on the merits
of the lawsuit, that Drugco is likely to suffer irreparable injury if an injunction is not granted, and
that the potential injury to Drugco is greater than any harm that Clair would suffer as a result of
an injunction.

POINT (3) [38%] ISSUE: Is the federal district court likely to compel Clair to return to Drugco the
chem sheets and pricing information throughout the pendency of the lawsuit? ANSWER: Yes. The
district court in State A is likely to issue a preliminary injunction to compel Clair to return the
chem sheets and pricing information to Drugco during the pendency of the lawsuit.

ANSWER DISCUSSION:

The district court has the power to issue a temporary restraining order, followed by a preliminary
injunction, prohibiting Clair from soliciting customers in violation of her contract with Drugco. In
determining whether to grant an injunction, the court will consider Drugco’s likelihood of winning the
lawsuit, the possibility of irreparable injury if the injunction is not granted, the balance of hardships
between the parties, and any relevant public interest factors. The court will also consider the same factors
in determining whether to issue a preliminary injunction ordering Clair to return Drugco’s chem sheets
and pricing information. On the facts of this problem, the court is likely to grant both preliminary
injunctions.

ANSWER EXPLANATION:

Explanation to Point-One (10-20%):

The district court in State A has authority under Rule 65 of the Federal Rules of Civil Procedure to issue a
temporary restraining order prohibiting Clair from violating the non-compete provision in her contract,
but such an order must expire within a few days and Drugco must seek a preliminary injunction.

A preliminary injunction is an equitable remedy that enjoins a person from engaging in specified
behavior, or requires a party to engage in specified behavior, during the pendency of the action. A
temporary restraining order (TRO) is an equitable remedy that may be issued ex parte (without notice) to
prevent irreparable harm until the court has an opportunity to rule on a motion for a preliminary
injunction. Rule 65 of the Federal Rules of Civil Procedure governs the issuance of both TROs and
preliminary injunctions by federal district courts. Although the Federal Rules permit the issuance of a
TRO without notice and on an ex parte basis, all TROs must expire within 14 days, and courts generally

Seperac-J19 Exam-Released MEE Essay Compilation © 2016-2020 128


set an earlier expiration date for TROs issued ex parte. Moreover, local rules may require that notice be
given prior to the issuance of a TRO. Rule 65(c) requires an applicant seeking either a TRO or a
preliminary injunction to post bond or provide some other security in the event that a party is later “found
to have been wrongfully enjoined or restrained.”

Explanation to Point-Two (45-55%):

The district court may grant a preliminary injunction to enjoin Clair from violating the non-compete
provision in the contract during the pendency of the lawsuit if it finds that Drugco is likely to succeed on
the merits of the lawsuit, that Drugco is likely to suffer irreparable injury if an injunction is not granted,
and that the potential injury to Drugco is greater than any harm that Clair would suffer as a result of an
injunction.

The substantive prerequisites for a preliminary injunction derive from traditional principles of equity
jurisdiction. Courts generally consider four factors in deciding whether or not to grant a preliminary
injunction: (1) the risk of irreparable harm to the plaintiff if the preliminary injunction is not granted; (2)
the likelihood that the plaintiff will succeed on the merits; (3) the likelihood that the harm the plaintiff
will suffer in the absence of preliminary injunctive relief outweighs the harm the defendant will suffer if
the injunction is granted; and (4) the public interest. The first two factors are the most important.

In assessing irreparable harm in the preliminary injunction context, courts ask whether the threatened
harm would impair the court’s ability to grant an effective remedy at the conclusion of the case.
Numerous courts have granted preliminary injunctions to enforce non-compete clauses in employment
agreements, concluding that an employer would suffer irreparable harm if a former employee solicited its
customers. “When the failure to grant preliminary relief creates the possibility of permanent loss of
customers to a competitor or the loss of goodwill, the irreparable injury prong is satisfied.”

In assessing whether the plaintiff is likely to prevail on the merits, courts most commonly ask whether the
plaintiff has demonstrated “a reasonable probability of success. All courts agree that plaintiff must present
a prima facie case but need not show that he is certain to win.” In this case, both States A and B enforce
non-compete clauses as long as they are reasonable in geographical scope and duration. Since Clair’s
contract with Drugco barred her from competing only within a three-county area of State A and only for
one year, and since there is strong evidence that she is breaching the contract, a court is likely to conclude
that Drugco will prevail on the merits.

In balancing the relative hardships, courts not only consider the injuries that the parties are likely to
suffer, but also consider the likelihood of success on the merits. Thus, the stronger the showing on the
merits, the less weight is given to the harm the defendant will suffer if the injunction is issued
erroneously. Here, the harm to the plaintiff – the loss of customers Chas already been discussed. The
potential harm to the defendant – loss of her job or at least the inability to continue to work for Medico in
the same geographical area for the remainder of the year – is likewise significant. But if the plaintiff
demonstrates a substantial likelihood of success on the merits (as here), the preliminary injunction will be
issued even if the harm factor favors the defendant.

Finally, courts consider the public interest or policy considerations that militate in favor of or against
issuance of the preliminary injunction. The decision here is not likely to have a significant impact on
strangers to the litigation, so this factor is not likely to influence the outcome of the motion for
preliminary injunctive relief.

Seperac-J19 Exam-Released MEE Essay Compilation © 2016-2020 129


On balance, it is likely that a court would issue a preliminary injunction to enjoin Clair from violating the
non-compete provision in the contract during the pendency of the lawsuit. It is far less important that
applicants reach this conclusion, however, than that they discuss intelligently Rule 65 and the factors that
would guide the court’s analysis.

Explanation to Point-Three (35-45%):

The district court in State A is likely to issue a preliminary injunction to compel Clair to return the chem
sheets and pricing information to Drugco during the pendency of the lawsuit.

Just as Rule 65 allows a federal district court to issue an injunction restraining Clair’s action, it also
permits the court to issue an injunction compelling Clair to engage in particular acts (a “mandatory
injunction”) if the four-part test for a preliminary injunction is satisfied. On our facts, the four factors
favor an order compelling Clair to return the chem sheets and pricing information.

First, the chem sheets, which reveal the chemical composition of the drugs sold by Drugco, are trade
secrets, and their disclosure would give rise to irreparable harm. Second, pricing information is highly
confidential and Drugco’s competitive position would be compromised if its pricing structure were
available to a competitor. Such harm could not be remedied at the conclusion of the case, so preliminary
relief is warranted.

Given that the chem sheets and pricing information are trade secrets under state law and given that the
employment agreement required Clair to return them to Drugco upon termination of her employment, it is
quite likely that Drugco will succeed on the merits of its claim to obtain the return of this data. Because
neither Medico nor Clair is entitled to retain this information, the balance of hardships tips decidedly in
favor of Drugco, and the public interest (as embodied in the law of States A and B) favors the
preservation of trade secrets. Thus, it is likely that a court would issue a preliminary injunction to require
Clair to return the chem sheets and pricing information during the pendency of the lawsuit.

[NOTE: In addition to its authority to grant a preliminary injunction, the court also has authority to grant
any preliminary remedy for the seizure of property that would be available under state law. In some
states, for example, a pre-judgment writ of replevin to secure the return of property is available to a
movant who demonstrates a superior right of possession to the property and a likelihood of success on the
merits of the underlying claim to possession. Graders should consider awarding extra credit to applicants
who note the possibility of pursuing state law replevin remedies as an alternative approach to recovering
the chem sheets and pricing information.]

Seperac-J19 Exam-Released MEE Essay Compilation © 2016-2020 130


#027-FEB 2005–MEE Q03: QUESTION THREE (CIVIL PROCEDURE)

Buyer and Seller were both citizens of State X, where they attended State University. Just before they
graduated, Buyer purchased Seller’s car for $2,500. At the time, Seller told Buyer that the car was “in
good working order” and that it was a “safe, reliable little car.” Seller knew, however, that a local
mechanic’s inspection of the car had revealed that its brakes were so worn that they presented a
significant safety hazard and required immediate replacement. Instead of informing Buyer of the danger,
Seller showed Buyer the mechanic’s bill and told him that the car “has just been checked out.”

After graduation, Buyer decided to move permanently to Big City in State Y. Buyer decided to use the car
he bought from Seller for the move. He believed that in a few round-trips he could transport all of his
belongings to his new apartment in Big City. On his way back to his State X apartment after his first trip
to his new apartment in Big City, Buyer was seriously injured in a one-car accident on a highway in State
X. Friend, a citizen of State X and a passenger in Buyer’s car, was the only witness to the accident. He
and Buyer will testify that the accident happened when the brakes failed as Buyer attempted to negotiate a
curve in the highway.

After weeks of hospitalization in State X, Buyer settled permanently in his apartment in Big City in State
Y. Having discovered Seller’s deceit, Buyer sued Seller in the U.S. District Court for the District of State
X, seeking to recover $500,000 under State X tort law for his injuries and lost wages. In addition, Buyer
claimed Seller’s misrepresentations constituted a breach of warranty under State X contract law and
sought recovery of the $2,500 paid for the car.

One week after the suit was filed, Seller also decided to relocate from State X to Big City, taking a job
there and assuming State Y citizenship.

Seller has filed a motion to dismiss Buyer’s two claims for lack of subject matter jurisdiction. Seller has
also moved for a change of venue to the U.S. District Court for the District of State Y.

1. Should the court dismiss Buyer’s tort claim, Buyer’s contract claim, or both for lack of subject
matter jurisdiction? Explain.

2. How should the court rule on Seller’s motion for a change of venue? Explain.

Seperac-J19 Exam-Released MEE Essay Compilation © 2016-2020 131


#027: F05-3 MEE: ANSWER: NCBE (CIVIL PROCEDURE)

POINT (1)(a) [33%] ISSUE: Were the requirements for federal diversity jurisdiction satisfied as to
the tort claim, given that Buyer and Seller were citizens of different states when the suit was filed
but subsequently became citizens of the same state? ANSWER: Yes. Buyer properly invoked
federal diversity jurisdiction as to the tort claim because Buyer was a citizen of State Y and Seller
was a citizen of State X when the complaint was filed, and the amount in controversy exceeds
$75,000.

POINT (1)(b) [33%] ISSUE: Does the federal district court have jurisdiction over Buyer’s contract
claim, even though it does not independently satisfy the jurisdictional amount-in-controversy
requirement? ANSWER: Yes. Although the $2,500 contract claim alone would not satisfy the
amount-in-controversy requirement for a diversity action, Buyer may aggregate the contract claim
with his tort claim in order to satisfy the amount-in-controversy requirement.

POINT (2) [33%] ISSUE: May the court transfer a case to another district when venue did not lie
in the other district at the time the suit was filed? ANSWER: No. The State X federal district court
cannot transfer the action to the federal court in State Y because venue would not have been proper
in State Y at the time the action was first filed.

ANSWER DISCUSSION:

Because Buyer and Seller were citizens of different states at the time the suit was filed, the requirements
for diversity jurisdiction were met with respect to the tort claim. Moreover, the contract claim was
properly aggregated with the tort claim to satisfy the amount-in-controversy requirement, and in any event
there was supplemental jurisdiction over the contract claim because both claims derive from a common
nucleus of operative fact. Transfer to the federal court in State Y would be inappropriate because the
claim could not have been brought there in the first instance.

ANSWER EXPLANATION:

Explanation to Point-One(a) (30-40%):

Buyer properly invoked federal diversity jurisdiction as to the tort claim because Buyer was a citizen of
State Y and Seller was a citizen of State X when the complaint was filed, and the amount in controversy
exceeds $75,000.

The court should deny Seller’s motion to dismiss for lack of subject matter jurisdiction over the tort
claim. The requirements for federal diversity jurisdiction have been established in this case.

Buyer’s claims are based on state law. Therefore, to proceed in federal court, he must satisfy 28 U.S.C. §
1332(a), which establishes the requirements for federal subject matter jurisdiction based on diversity of
citizenship. According to the plain terms of the statute, “the matter in controversy must exceed the sum or
value of $75,000,” and be between “citizens of different states.” The courts have interpreted § 1332 as
requiring the parties to be diverse at the time the suit is filed, and not at the time the cause of action arises.
Here the amount-in-controversy requirement has been met with regard to the claim for personal injury and
lost wages ($500,000), and the only question is whether the diversity requirement is met.

Seperac-J19 Exam-Released MEE Essay Compilation © 2016-2020 132


In determining the citizenship of the parties, the courts look to the domicile of each party, examining “the
fact of residency coupled with a finding of intent to remain indefinitely.” A change of domicile is effected
“by a combination of two elements: (a) taking up residence in a different domicile with (b) the intention to
remain there.” The important question is whether Buyer was a citizen of State Y at the time the suit was
filed. Having completed his move to State Y at that time, and having the intent to remain in State Y,
Buyer was then a State Y citizen. Buyer was therefore diverse from Seller, who was still a citizen of State
X when the suit was filed.

Indeed, some applicants may conclude, based on Buyer’s rental of an apartment and his intent to remain
in Big City, that he was already a citizen of State Y at the time of the accident (and hence before the time
the suit was filed).

Because citizenship is determined at the time suit is filed, Seller’s post-filing move to State Y does not
affect the court’s diversity jurisdiction. The parties were diverse at the time the suit was filed and changes
in citizenship after filing are irrelevant, absent bad faith on the part of a party.

Explanation to Point-One(b) (30-40%):

Although the $2,500 contract claim alone would not satisfy the amount-in-controversy requirement for a
diversity action, Buyer may aggregate the contract claim with his tort claim in order to satisfy the amount-
in-controversy requirement.

Buyer’s effort to join the small contract claim to his large tort claim can be analyzed in two ways. First,
under traditional aggregation rules, a single plaintiff with “two entirely unrelated claims against a single
diverse defendant” may bring them in federal court as long as the aggregate of the claims exceeds the
$75,000 amount-in-controversy requirement. The rationale is that “the diversity statute confers federal
jurisdiction over civil actions’ satisfying the required minimum amount in controversy not over counts,
thus permitting the plaintiff to aggregate the stakes in his separate claims or counts to come up to the
minimum.” Thus, because the aggregate of Buyer’s tort and contract claims exceeds $75,000, the court
has jurisdiction to hear them all, including the contract claim for $2,500.

Second, under the federal supplemental jurisdiction statute, 28 U.S.C. § 1367(a), district courts “shall
have supplemental jurisdiction over all claims that are so related to claims in the action within such
original jurisdiction that they form part of the same case or controversy.”

The standard test is whether the claims “derive from a common nucleus of operative fact.” In this
problem, the tort and contract claims, which both derive from the sale of a car with defective brakes, are
“so related” as to arise from the “same case or controversy,” and the district court may, accordingly, take
supplemental jurisdiction over the contract claim. None of the exceptions in § 1367(b) apply.

Explanation to Point-Two (30-40%):

The State X federal district court cannot transfer the action to the federal court in State Y because venue
would not have been proper in State Y at the time the action was first filed.

Seller’s motion for a change of venue should be denied. In a diversity case, venue properly lies, and a
civil action may only be brought, in a judicial district “where any defendant resides.” or “in which a
substantial part of the events or omissions giving rise to the claim occurred.” At the time the case was
commenced, Seller, the defendant, resided in State X. Moreover, all relevant events occurred in State X.
Hence, at the time the suit was filed, venue was proper only in the State X federal district court.

Seperac-J19 Exam-Released MEE Essay Compilation © 2016-2020 133


Seller’s subsequent move to State Y does not warrant a change of venue. The change of venue statute, 28
U.S.C. § 1404, permits a district court to transfer actions “for the convenience of parties and witnesses, in
the interest of justice.” However, a change of venue is only permitted “to any other district or division
where the case might have been brought.”

The relevant time frame under both the basic venue statute and the venue transfer statute is the time that
the suit is filed. As a consequence, the court should deny Seller’s motion for a change of venue. At the
time Buyer’s suit was brought, the only possible venue was State X. As noted above, State X was both
Seller’s residence and the place where a substantial part of the events giving rise to Buyer’s claim – the
sale of the car and the accident – occurred. There was little or no connection between State Y and the
events giving rise to the action. Accordingly, because the case could not have been brought in State Y, the
requirements of § 1404 are not met, and the motion for a change of venue must be denied. It is irrelevant
that Seller now resides in State Y.

In addition, it is possible to argue that discretionary factors also would justify denial of a transfer. Because
the events occurred in State X, and most, if not all, of the witnesses are from State X (the mechanic, those
responding to the accident, the hospital employees treating Buyer, and Friend), it can be argued that “the
convenience of the parties and witnesses, and the interests of justice” are all best served by permitting the
case to go forward in State X.

[NOTE: While an answer that mentioned only the discretionary factors might lead to a correct result, it
would not be an adequate answer. The key point, which controls the result, is that the action could not
have been brought originally in State Y and therefore could not be transferred there.]

Seperac-J19 Exam-Released MEE Essay Compilation © 2016-2020 134


#028-FEB 2004–MEE Q04: QUESTION FOUR (CIVIL PROCEDURE)

Motorist was driving his automobile in the northbound lane on a highway in State X. Exceeding the
posted maximum speed limit by 10 miles per hour, Motorist was gaining on a slow-moving U.S. Army
truck convoy traveling in the same lane. At the same time, a motorcycle, driven by Husband with Wife as
a passenger, was approaching the convoy from the opposite direction. Husband was operating the
motorcycle at the posted speed limit and was traveling in the proper (southbound) lane.

The soldier driving the last truck in the convoy was feeling drowsy because he had not slept the night
before. Suddenly, realizing that he had driven his truck far too close to the Army truck just ahead of him,
he slammed on his brakes. Motorist, who had nearly overtaken the convoy, reacted to the sight of the
truck’s brake lights by frantically swerving his automobile into the southbound lane, where he sideswiped
Husband’s motorcycle. Husband was thrown to the pavement and seriously injured. Wife, however,
miraculously avoided physical injury.

At the time of the accident, Husband and Wife were citizens of State Y, but a few months later they
moved permanently to State X. Motorist was at all times a citizen of State X.

Shortly after moving to State X, Husband and Wife filed an action in the United States District Court for
the District of State X seeking to recover for the injuries they suffered as a result of the accident. In the
action, Husband claimed $50,000 in damages for his injuries, and Wife claimed $5,000 for her loss of
Husband’s consortium. They named as defendants both the United States of America and Motorist. Their
claims against the United States are based on the Federal Tort Claims Act, which provides that federal
district courts have exclusive jurisdiction over tort claims against the United States government. Their
claims against Motorist are based on the tort law of State X, where the accident occurred.

1. Can Husband and Wife, as plaintiffs, join their respective personal injury and loss of consortium
claims in a single action in the U.S. district court? Explain.

2. Can Husband and Wife join their respective claims against the United States and Motorist, as
defendants, in a single action in the U.S. district court? Explain.

3. Does the U.S. district court have subject matter jurisdiction over the state law claims of Husband
and Wife against Motorist? Explain.

Seperac-J19 Exam-Released MEE Essay Compilation © 2016-2020 135


#028: F04-4 MEE: ANSWER: NCBE (CIVIL PROCEDURE)

POINT (1) [21%] ISSUE: Can Husband and Wife join their respective personal injury and loss-of-
consortium claims in a single action in the U.S. district court? ANSWER: Yes. Federal Rule 20
permits Husband and Wife to join their respective personal injury and loss-of-consortium claims in
a single action.

POINT (2) [21%] ISSUE: Can Husband and Wife join their respective claims against the United
States and Motorist in a single action in the U.S. district court? ANSWER: Yes. Federal Rule 20
permits Husband and Wife to join their respective claims against the United States and Motorist in
a single action.

POINT (3) [58%] ISSUE: Does the U.S. district court have subject matter jurisdiction over the state
law claims? ANSWER: Yes. Even though the federal court lacks diversity jurisdiction over
Husband’s and Wife’s claims against Motorist, it has “supplemental jurisdiction” over those claims.

ANSWER DISCUSSION:

Rule 20 of the Federal Rules of Civil Procedure permits Husband and Wife to assert their federal and state
law claims against the United States and Motorist because their claims arose out of the same transaction.
Although the court would not have diversity jurisdiction in an action by Husband and Wife against
Motorist if such an action were brought on its own, the court has supplemental jurisdiction over that claim
because it shares a “common nucleus of operative fact” with Husband and Wife’s federal question claim
against the United States.

ANSWER EXPLANATION:

Explanation to Point-One (15-25%):

Federal Rule 20 permits Husband and Wife to join their respective personal injury and loss-of-consortium
claims in a single action.

FRCP Rule 20 permits plaintiffs to join their claims in a single action when those claims arise out of a
single event and share at least one common issue of law or fact. The rule provides, in relevant part: All
persons may join in one action as plaintiffs if they assert any right to relief jointly, severally, or in the
alternative in respect of or arising out of the same transaction, occurrence, or series of transactions or
occurrences and if any question of law or fact common to all these persons will arise in the action.

Again, the requirements of Rule 20 are clearly met on these facts. The claims of each plaintiff against the
United States and Motorist arise out of the same occurrence and involve at least one common issue of fact
or law (for example, the extent of Husband’s injuries). When this is so, the Federal Rules permit (but do
not require) a party to join two or more defendants in a single action.

[NOTE: Rule 20 does not confer subject matter jurisdiction on the U.S. district court to hear the
respective claims of the plaintiffs. It simply provides the mechanism for joining claims over which the
Constitution and the Congress have given it subject matter jurisdiction. Accordingly, the joinder of
Husband and Wife as party plaintiffs is proper, provided the U.S. district court has subject matter
jurisdiction over the claims by each plaintiff.]

Seperac-J19 Exam-Released MEE Essay Compilation © 2016-2020 136


Explanation to Point-Two (15-25%):

Federal Rule 20 permits Husband and Wife to join their respective claims against the United States and
Motorist in a single action.

Just as Rule 20 permits plaintiffs to join in an action, it permits a plaintiff or plaintiffs to join defendants
where the plaintiff(s)’ claims against each defendant arise from a single transaction and share a common
issue of fact or law. The rule provides, in relevant part:

All persons may be joined in one action as defendants if there is asserted against them jointly, severally,
or in the alternative, any right to relief arising out of the same occurrence and if any question of law or
fact common to all defendants will arise in the action.

Again, the requirements of Rule 20 are clearly met on these facts. The claims of each plaintiff against the
United States and Motorist arise out of the same occurrence and involve at least one common issue of fact
or law (for example, the extent of Husband’s injuries). When this is so, the Federal Rules permit (but do
not require) a party to join two or more defendants in a single action.

[NOTE: As is the case with joinder of plaintiffs, this rule does not confer subject matter jurisdiction on
the federal court to hear a claim against a joined defendant. Thus, joinder may be defeated where the
court lacks subject matter jurisdiction over the joined claim.]

Explanation to Point-Three (50-60%):

Even though the federal court lacks diversity jurisdiction over Husband’s and Wife’s claims against
Motorist, it has “supplemental jurisdiction” over those claims.

Federal courts are tribunals with limited subject matter jurisdiction in civil cases. Basically, they may
decide only “federal question” and “diversity” cases. The claims of Husband and Wife against Motorist
do not involve a federal question; they are state tort law claims. Nor do their claims against Motorist
qualify for diversity jurisdiction. First, diversity is determined on the date the suit is filed. Although
Husband and Wife were citizens of a different state than Motorist on the day of the accident, they were all
citizens of State X on the date that suit was filed. Second, their claims do not satisfy the amount in
controversy requirement. Neither claim is for an amount in excess of $75,000, so an effort to pursue their
claims in a federal court under its diversity jurisdiction would fail.

However, a state cause of action may be heard in a federal court under supplemental jurisdiction
(formerly termed “ancillary” or “pendent” jurisdiction). Where a party has both a federal question claim
and a state law claim and both claims arise out of a common nucleus of operative fact, it has long been
recognized that the federal court may, in the interest of the economical and efficient administration of
justice, decide both the federal and the state claims in a single action.

Subsection (a) of Section 1367 of the Judicial Code authorizes a federal court to exercise “supplemental
jurisdiction” over any state law claim that has a nucleus of operative fact in common with a claim that is
itself within the subject matter jurisdiction of the federal court. The statute not only codifies the “pendent
claim” jurisdiction as formulated in the Gibbs case, but the final sentence of subsection (a) expands
supplemental jurisdiction expressly to include “pendent party” jurisdiction. “Such supplemental
jurisdiction,” the statute reads, “shall include claims that involve the joinder or intervention of additional
parties.”

Seperac-J19 Exam-Released MEE Essay Compilation © 2016-2020 137


Accordingly, since the State X tort claims of Husband and Wife against Motorist arise out of the same
occurrence as their federal question claims against the United States, the federal court in State X has
supplemental jurisdiction over these State X claims. The claims against Motorist share a “common
nucleus of operative fact” with the claims against the United States, and it is reasonable to expect them to
be resolved in one action. Under such circumstances, the courts have routinely held that 21 U.S.C. §
1367(a) confers supplemental jurisdiction over such claims. For example, supplemental jurisdiction
appropriate when disparate tort claims arise from same basic occurrence. Thus, even though the state law
claims, standing alone, would be outside the court’s subject matter jurisdiction, they may be joined to the
case against the United States, over which the court has exclusive jurisdiction pursuant to the Federal Tort
Claims Act, 28 U.S.C. § 1346(b).

[NOTE: The statute gives the court discretion to reject claims on prudential grounds, including when the
pendent claims raise novel issues of state law, when they substantially predominate over the claims over
which the federal court has original jurisdiction, when the claims over which the court had original
jurisdiction have all been dismissed, or in other “exceptional circumstances.” However, none of these
prudential reasons for denying supplemental jurisdiction is present on our facts.]

Seperac-J19 Exam-Released MEE Essay Compilation © 2016-2020 138


#029-JUL 2003–MEE Q04: QUESTION FOUR (CIVIL PROCEDURE)

Farmer brought a class action lawsuit in federal district court in State A, alleging that the defendant,
Truckco, marketed a line of pickup trucks with defective shock absorbers. Farmer’s complaint identified
the members of the class as 100,000 individuals nationwide who had bought the trucks from 1995-2000
and suffered losses as a result of the defective shock absorbers. The alleged losses ranged from the $250
cost of replacing the shock absorbers to serious personal injuries suffered in accidents alleged to have
been caused by the defective shock absorbers. The only claim personal to Farmer was the $250
replacement cost claim. Farmer properly asserted that federal jurisdiction was based on a breach of
warranty claim under a recently enacted federal automobile safety statute.

Farmer moved to certify the class. Opposing this motion, Truckco submitted court papers from lawsuits
brought by individual owners who claimed to have suffered a wide variety of personal injuries as a result
of accidents said to have resulted from the failure of the defective shock absorbers during the years in
question. Truckco also noted that Farmer had previously filed (and still has pending) a class action against
Truckco in a state court in State Z. In the State Z case, which was premised on state-law warranty claims,
Farmer sought relief similar to the relief sought in the federal action and asked to represent the same class
of plaintiffs. Finally, Truckco pointed out that Farmer’s lawyer in both actions was a recent bar admittee
who had not previously handled class action litigation.

In addition to arguing against certification of the class, Truckco asked the federal district court to abstain
from adjudicating the class action in light of the pendency of the state court class action.

The federal district court first denied Truckco’s motion for abstention. The court then denied Farmer’s
motion for class certification, finding that class certification was “inappropriate under the circumstances.”

1. Was the court’s ruling on Truckco’s motion for abstention correct? Explain.

2. Was the court’s ruling on Farmer’s motion for class certification appropriate? Explain.

Seperac-J19 Exam-Released MEE Essay Compilation © 2016-2020 139


#029: J03-4 MEE: ANSWER: NCBE (CIVIL PROCEDURE)

POINT (1) [35%] ISSUE: Does the pendency of a parallel state court proceeding in State Z require
the federal district court to abstain from adjudicating the class action? ANSWER: No. Although
duplicative federal and state court litigation is wasteful, no federal rule requires abstention simply
on the basis of duplication. The court’s denial of abstention was correct because no existing federal
abstention rule would support abstention in this case.

POINT (2) [65%] ISSUE: Is class certification appropriate in this case under the guidelines set
forth in Fed. R. Civ. P. 23? ANSWER: No. The district court was justified in ruling that class
certification was unwarranted because the facts suggest the absence of claim typicality and
adequate representation.

ANSWER DISCUSSION:

The court’s refusal to abstain was correct. The court is not required to abstain merely because the federal
litigation is duplicative of state litigation. The facts do not present any other basis for abstention. The
district court properly denied class certification because Farmer’s breach-of-warranty claim is not typical
of the claims of those class members who have suffered personal injuries. In addition, Farmer is not an
adequate representative of the class because his interests differ significantly from the interests of the
personal injury claimants and Farmer’s attorney is inexperienced with this kind of litigation.

ANSWER EXPLANATION:

Explanation to Point-One (30-40%):

Although duplicative federal and state court litigation is wasteful, no federal rule requires abstention
simply on the basis of duplication. The court’s denial of abstention was correct because no existing
federal abstention rule would support abstention in this case.

In general, parties may proceed to judgment in a federal district court action without regard to the
pendency of state proceedings that seek similar relief. Indeed, a federal district court has a “duty to
adjudicate a controversy properly before it,” and it may abstain “only in the exceptional circumstances
where the order to the parties to repair to the State court would clearly serve an important countervailing
interest.”

The facts of this problem do not support abstention under any established abstention rule. Pullman
abstention operates only when the state court’s resolution of unsettled state law issues may obviate the
necessity of resolving a difficult federal law issue. Burford abstention is appropriate only if federal
adjudication would interfere with the state’s administration of a complex regulatory scheme, and Younger
abstention is limited to cases where the federal court is asked to enjoin the actions of state officials

Nothing in this problem implicates any of these standard abstention doctrines.

There is an argument that the district court should dismiss the action under the U.S. Supreme Court’s
decision in Colorado River. Viewed expansively, the Colorado River doctrine, which is based on
“principles of federalism, comity, and conservation of judicial resources,” could be said to warrant federal
court restraint when parallel state proceedings mean that pursuit of federal proceedings will waste judicial

Seperac-J19 Exam-Released MEE Essay Compilation © 2016-2020 140


resources and potentially involve federal resolution of issues also pending before state courts. Such an
argument could be bolstered by observing that dismissal under Colorado River is not governed by a “hard
and fast rule,” but by the application of an elaborate balancing test.

However, Colorado River does not justify abstention in this case. In Colorado River, the Supreme Court
emphasized that the dismissal of the federal action in that case (in favor of parallel state proceedings) was
an “extraordinary and narrow exception,” to the “virtually unflagging obligation of the federal courts to
exercise the jurisdiction given them.”

The mere fact of duplicative litigation, which is all that appears to justify abstention on our facts, is not
enough to justify abstention under Colorado River. (“Duplicative litigation, wasteful though it may be, is
a necessary cost of our nation’s maintenance of two separate and distinct judicial systems.”) There must
be evidence of other factors that militate against federal litigation, such as the risk of inconsistent rulings
with respect to a particular piece of property or clear evidence of a federal policy favoring unitary
adjudication of the claims at issue. In conducting the necessary balancing inquiry, moreover, “the balance
is heavily weighted in favor of the exercise of jurisdiction.” In our case, there appears to be no special
state interest to protect, no federal policy supporting unified state court adjudication of these product
liability claims, and no apparent risk of inconsistent adjudications. It thus appears that the court’s ruling
was correct, as the class action here falls under the general rule of no abstention, rather than the narrow
exception in Colorado River.

Explanation to Point-Two (60-70%):

The district court was justified in ruling that class certification was unwarranted because the facts suggest
the absence of claim typicality and adequate representation.

Federal Rule of Civil Procedure Rule 23(a) identifies the prerequisites that must be satisfied before any
class may be certified. Rule 23(a) requires evidence that:

(1) the class is so numerous that joinder of all members is impracticable, (2) there are questions of law or
fact common to the class, (3) the claims or defenses of the representative parties are typical of the claims
or defenses of the class, and (4) the representative parties will fairly and adequately protect the interests of
the class.

If the Rule 23(a) prerequisites are satisfied, a class action may be certified if it falls within one of the
categories specified in Rule 23(b). Of particular relevance to this case is Rule 23(b)(3), which also
requires that “questions of law or fact common to the members of the class predominate over any
questions affecting only individual members, and a class action is superior to other available methods for
the fair and efficient adjudication of the controversy.”

For a number of reasons, the district court’s refusal to certify Farmer’s class action was appropriate.
While Farmer has satisfied a few of the prerequisites specified in Rule 23(a), he has not satisfied all of
them. With respect to the few, the 100,000 members of the class easily satisfy the numerosity
requirement, and there are issues of law and fact common to the members of the class. In particular, there
are the questions whether the shock absorbers are defective and, if so, whether relief is appropriate under
federal warranty law.

Farmer has not likely, however, satisfied the typicality and adequacy of representation requirements. As
for typicality, the claims of representatives and class members are typical when they stem from a single
event or are based on common legal theories. Here, Farmer’s individual claim seeks to recover for out-of-

Seperac-J19 Exam-Released MEE Essay Compilation © 2016-2020 141


pocket losses resulting from replacement of the shocks. While Farmer’s claim is “typical” of other class
members’ claims to the extent that all the claims involve the question of the defectiveness of the shock
absorbers, Farmer’s warranty claim otherwise differs significantly from claims for personal injuries,
which some members of the class have here. Personal injury claims are highly individualized, involving
issues of causation, extent of damages, contributory negligence, etc., that are not present with respect to
Farmer’s breach-of-warranty claim. Because of differences of this sort, federal courts have been reluctant
to grant class certification where the claims of injured persons will be represented by class representatives
who have no personal injury or where personal injury claims are lumped with claims that do not involve
personal injury. Moreover, personal injury claimants have a strong interest in individually controlling the
prosecution of their claims and in making individual decisions on whether to settle. A class action is
designed for situations where individual claims are too small to warrant individual prosecution.
Consequently, a typicality problem arises here. For example, in one case, class certification was proper
because the class was limited to economic loss claimants and personal injury claimants were excluded.

The adequacy of representation requirement involves a two-part inquiry. First, the court must ask whether
the representative’s interests are aligned closely enough with other class members to ensure fair
representation of the absentee class members. Second, the court must ensure that class counsel is
experienced and qualified to carry out the litigation in order to fairly and adequately protect the interests
of the class. Both requirements pose problems in this case. The difference between the nature of the
contract and warranty claims of Farmer and those of the personal injury victims presents a potential
conflict of interest that raises doubts about Farmer’s adequacy as a representative of the interests of the
personal injury claimants. Farmer might be more willing to accept a modest settlement than would the
personal injury claimants in the class. This adequacy of representation problem is magnified in view of
the lawyer’s inexperience with class litigation. There is a real question as to whether a new bar admittee is
appropriate counsel for such complex litigation.

Even if the district court were satisfied that the Rule 23(a) prerequisites were satisfied, these same
concerns would warrant denial of class certification pursuant to Rule 23(b). As noted above, the court
may certify the class action only if common questions predominate over questions affecting individual
members of the class. Given the existence of personal injury claimants in the class, some of whom have
already filed independent claims, and given the existence of questions about the adequacy of the
representation being provided by Farmer for the class, it was certainly within the scope of the court’s
discretion to conclude that the efficiency to be had by litigating “common questions of law and fact” in a
class action would be overwhelmed by the problems posed by the presence of litigants with diverse
personal injury claims, raising non-common issues of fact and law. In this regard, the personal injury
claimants probably have a strong interest “in individually controlling the prosecution of their separate
actions.” Moreover, given the potential variability in the nature of the claims asserted by the class
members, and the inexperience of class counsel, the “difficulties likely to be encountered in the
management” of an action involving such diverse claims would probably significantly reduce the
efficiencies stemming from class litigation.

Accordingly, although the class is large and there are some common issues of law and fact, given the wide
discretion residing with the court, it appropriately refused to certify the class.

Seperac-J19 Exam-Released MEE Essay Compilation © 2016-2020 142


#030-FEB 2003–MEE Q05: QUESTION FIVE (CIVIL PROCEDURE)

Acme Corporation, a citizen of State X, manufactures widgets. Acme widgets are distributed to retailers
throughout the United States by Widgets, Inc., a citizen of State Y. Plaintiff, a citizen of State Y,
purchased an Acme widget from a retailer in her hometown. Shortly after purchasing the widget, Plaintiff
was seriously injured when the widget overheated and exploded.

Plaintiff sued Acme in the federal district court located in State Y, properly invoking the court’s diversity
jurisdiction. Plaintiff sought $100,000 in damages on two state-law tort theories: (1) failure to warn, and
(2) sale of a dangerously defective product.

Under the applicable state law, a manufacturer’s duty to warn is fully discharged if a proper warning is
affixed to the product at the point of delivery to its distributor. A distributor’s duty is fully discharged if
the warning is affixed at the point of delivery to the retailer. State law further provides that both
manufacturers and distributors may be held separately and strictly liable for selling a “dangerously
defective” product, even if they have given adequate warning of the risks. Plaintiff’s complaint alleged
both that Acme had failed to affix a warning label to the product and that Acme’s widgets had a
dangerous propensity to overheat.

After extensive discovery, Acme filed a motion for summary judgment on the failure to warn claim. It
attached to its motion the supporting affidavits of employees of both Acme and Widgets attesting that a
proper warning label had been affixed to the widget both at the time of delivery to Widgets and at the time
of distribution to the retailer who sold the widget to Plaintiff. While conceding that the warning label
usually provided with the product did give adequate notice of the danger of overheating and explosion
under certain circumstances, Plaintiff nevertheless contested the motion for summary judgment with her
own affidavit, in which she stated that there had been no warning label affixed to her widget when she
purchased it from her local retailer.

The federal court granted Acme’s motion for summary judgment on the failure to warn claim and entered
judgment on that claim against Plaintiff. No appeal was taken. Soon afterward, Acme and Plaintiff settled
the dangerous defect claim for an undisclosed amount.

Shortly after the conclusion of the federal litigation, Plaintiff filed suit in the state court of State Y,
asserting against Widgets, Inc., the same two claims she had asserted against Acme in federal court:
failure to warn and sale of a dangerously defective product. Widgets answered and then moved to dismiss
on grounds of claim and issue preclusion.

1. In Plaintiff’s suit against Acme, did the federal court properly grant Acme’s motion for summary
judgment on the failure to warn claim? Explain.

2. Should the State Y state court give preclusive effect to the federal court judgment and dismiss
Plaintiff’s claims against Widgets? Explain.

Seperac-J19 Exam-Released MEE Essay Compilation © 2016-2020 143


#030: F03-5 MEE: ANSWER: NCBE (CIVIL PROCEDURE)

POINT (1) [41%] ISSUE: Under Fed. R. Civ. P. 56, should summary judgment be granted to a
defendant who supports his motion with evidence negating the plaintiff’s claim when the plaintiff’s
response fails to directly controvert that evidence? ANSWER: Yes. Because Plaintiff’s response to
Acme’s motion for summary judgment failed to demonstrate a genuine issue of material fact as to
Acme’s supported assertion that a warning label was affixed to the product at the time of delivery
to the distributor, the court properly granted summary judgment to Acme on the failure to warn
claim.

POINT (2)(a) [29%] ISSUE: Does claim preclusion arising from a prior federal suit against a
product’s manufacturer bar a subsequent suit against the product’s distributor? ANSWER: No.
Because Widgets was not a party to the prior action, it probably cannot bar Plaintiff’s action
against it on claim preclusion grounds.

POINT (2)(b) [29%] ISSUE: In this case, may the distributor successfully invoke issue preclusion
against the plaintiff? ANSWER: No. Widgets will not be able to assert issue preclusion against
Plaintiff because the issues in Plaintiff’s suit against Widgets are not the same as the issues that
were actually litigated in Plaintiff’s prior suit against Acme.

ANSWER DISCUSSION:

In the federal court action, defendant Acme’s motion for summary judgment was premised on the claim
that a proper warning label was attached to its widget at the time the widget was delivered to the
distributor. Under the applicable state law, Acme’s duty to warn was fully discharged if that fact were
true. To support its claim, Acme provided affidavits of witnesses who said the label was present at the
relevant time. In the face of this evidence, Plaintiff had a duty to present evidence of “specific facts”
showing that there was a genuine issue for trial. Plaintiff’s affidavit, which asserted only the absence of a
label when she purchased the widget, did not contradict Acme’s evidence that a label was present when it
delivered the widget to the distributor. Hence, summary judgment was appropriate on that issue. Nothing
that happened in the federal court action should preclude Plaintiff from proceeding with the state court
action. Claim preclusion is inappropriate because there is no mutuality of estoppel: the defendant in the
state court action was not the same as, or in privity with, the federal defendant and would not have been
precluded or otherwise affected by the result in the federal action. Issue preclusion is inappropriate
because the only issue actually decided in the federal action (whether a warning label was present when
the manufacturer delivered the widget to the distributor) is not an issue in the state action, where the
question is whether a warning label was present when the widget was delivered to the retailer.

ANSWER EXPLANATION:

Explanation to Point-One (30-40%):

Because Plaintiff’s response to Acme’s motion for summary judgment failed to demonstrate a genuine
issue of material fact as to Acme’s supported assertion that a warning label was affixed to the product at
the time of delivery to the distributor, the court properly granted summary judgment to Acme on the
failure to warn claim.

Seperac-J19 Exam-Released MEE Essay Compilation © 2016-2020 144


A motion for summary judgment should be granted in favor of a party if “there is no genuine issue as to
any material fact” and the party is “entitled to a judgment as a matter of law.” A defendant on a claim may
move for summary judgment by attacking any necessary element of the plaintiff’s case. In determining
whether there are any genuine issues of material fact, the court should construe all factual matters in the
light most favorable to the non-moving party. However, where the moving party presents evidence of
facts that would defeat the non-moving party’s claim, the non-moving party “may not rest upon the mere
allegations” of her pleading. The non-moving party has a responsibility to offer, “by affidavits or as
otherwise provided” in Rule 56, evidence of “specific facts showing that there is a genuine issue for trial.”

On the facts of this problem, the court properly granted Acme’s motion for summary judgment on the
failure to warn claim. The law of State Y (which the federal court is obliged to apply under the Erie
doctrine in this diversity case) states that a manufacturer fully discharges its duty to warn if adequate
warning labels are affixed to the product at the time of delivery to its distributor. Thus, to win on her
failure to warn claim, Plaintiff must prove that no proper warning label was affixed to the widget at the
time of delivery to the distributor.

Defendant Acme’s summary judgment motion alleged that adequate warning labels were, in fact, affixed
to the product at the relevant time, and that allegation was supported by affidavits attesting to the presence
of the warning label at the time of delivery to the distributor. This properly supported motion therefore
negates a key element of Plaintiff’s claim, thereby discharging Acme’s duty to support its summary
judgment motion under

At this point, the burden shifted to Plaintiff to come forward with controverting evidence. Plaintiff
responded to the summary judgment motion with her own affidavit, in which she attested that no warning
label had been present on her widget when she purchased it. This does not directly controvert Acme’s
motion, since it does not contend that the warning label was missing at the point of delivery to the
distributor, the critical moment for manufacturer liability under applicable state law. Consequently,
Plaintiff failed to meet her burden to produce evidence controverting Acme’s version of the facts.

Thus, Acme has demonstrated that no genuine issue of material fact exists on a key element of Plaintiff’s
claim, and that it is, therefore, entitled to judgment as a matter of law.

Explanation to Point-Two(a) (20-30%):

Because Widgets was not a party to the prior action, it probably cannot bar Plaintiff’s action against it on
claim preclusion grounds.

[NOTE: Even though the current suit is in state court, the effect of the Acme judgment is governed by
federal law because the judgment came from a federal court. A state court is required to give to a federal
judgment the same preclusive effect that the judgment would have in federal court. The preclusive effect
of a judgment by a federal court sitting in diversity ordinarily would be determined by the preclusion
rules of the state in which the federal court sits.]

In order to bar a plaintiff’s subsequent lawsuit on claim preclusion grounds, a defendant must show (1)
that the prior judgment is final, valid, and on the merits (which is true here), (2) that the present claims are
within the scope of the prior judgment, as measured by the same transaction test (also true here), and (3)
that there is mutuality of estoppel (i.e., that the defendant urging preclusion is the same as, or in privity
with, the defendant in the prior suit). Although the first two elements of claim preclusion are easily
satisfied on the facts of the problem, Widgets’ claim preclusion argument will probably fail on the final
prong: mutuality of estoppel.

Seperac-J19 Exam-Released MEE Essay Compilation © 2016-2020 145


Here, there are no facts to suggest that Widgets had such a close relationship with Acme that Widgets
would have been bound by the prior judgment had that decision gone against Acme. Widgets apparently
did not control or participate in the Acme litigation. Nor are Widgets and Acme in any agency or
representative relationship that would warrant treating Acme’s actions as binding on Widgets. In short,
there is nothing to suggest that Widgets bore such a close relationship to the first suit or to Acme that it
would be proper to treat the first action as binding on Widgets. Under the circumstances, the usual rule
that a judgment operates only against parties to the first suit seems fully applicable.

Where, as here, a plaintiff brings separate suits against alleged joint tortfeasors whose liability is joint and
several (and not derivative), the courts traditionally hold that the plaintiff is entitled to maintain separate
actions and that a judgment in a case involving one tortfeasor does not preclude the bringing of a claim
for the same harm against another tortfeasor. For example, under Florida law, the manufacturer, the
wholesale distributor, and the retailer of an allegedly defective product are treated as identical parties for
res judicata purposes and an action against a retailer is a bar to a subsequent action against the
manufacturer on product liability grounds.

[NOTE: Although the doctrine of mutuality of estoppel has been widely abandoned in relation to issue
preclusion, it is still largely operative in the context of claim preclusion. The traditional rule requiring
mutuality of estoppel for claim preclusion purposes appears to be undergoing some modification in
federal courts. Wright, Miller and Cooper report that increasing numbers of federal courts now allow
claim preclusion against a party to a prior action by litigants who would not themselves have been bound
by the results of the prior case if there are “good reasons why the new defendant should have been joined
in the first action and the old party cannot show any good reasons to justify a second chance.” In the
present case, however, there is no evidence of a good reason why Plaintiff ought to be required to sue
both Acme and Widgets at the same time.]

Explanation to Point-Two(b) (20-30%):

Widgets will not be able to assert issue preclusion against Plaintiff because the issues in Plaintiff’s suit
against Widgets are not the same as the issues that were actually litigated in Plaintiff’s prior suit against
Acme.

Widgets’ attempt to invoke issue preclusion will also fail. The problem for Widgets is that none of the
issues involved in Plaintiff’s suit against it were actually decided in Plaintiff’s suit against Acme. As to
the failure to warn issue, although evidence was presented in Acme’s suit on whether the required
warning was present at the time Widgets delivered the product to the retailer (in the form of the employee
affidavits), the court did not need to use that evidence to decide the summary judgment motion, since the
key point there was whether the label was affixed at the point of delivery to the distributor, not the
retailer. As to the dangerous defect issue, the settlement in Acme’s case obviated any actual litigation on
the point. The settlement also precluded any actual litigation as to the amount of Plaintiff’s damages. It is
well settled that a judgment entered pursuant to a settlement cannot serve as the basis for issue preclusion
in a later case.

Because issue preclusion essentially exports the factual findings of one case into another, any issue to be
precluded must be precisely the same one that was actually litigated, decided, and necessary to the
judgment in the prior suit. Here, that is not the case.

Seperac-J19 Exam-Released MEE Essay Compilation © 2016-2020 146


#031-JUL 2002–MEE Q03: QUESTION THREE (CIVIL PROCEDURE)

Seller manufactures vending machines at a facility located in State A, where Seller is incorporated and
has its principal place of business. Buyer, a German company with its principal place of business in
Munich, Germany, contracted to purchase 1,000 vending machines from Seller for a total price of
$500,000.

The contract was carefully negotiated during lengthy discussions held in Germany. Early in the
negotiations, each side insisted that the contract should be governed by its own law and that disputes
should be resolved in its own courts. In the end, however, the parties agreed on contract clauses that
provided: (1) “the substantive rights and remedies of the parties to this contract shall be governed by the
Commercial Code of State N”; (2) “any and all litigation brought concerning this contract shall be brought
in the state or federal courts of State N”; and (3) “Seller and Buyer hereby consent to venue, jurisdiction,
and service of process by courts in State N.” Apart from these clauses, there is no connection between
State N and the parties or the transaction. State N is located on the eastern seaboard of the United States.
The parties chose State N because it has convenient air links to Germany and a widely respected judiciary
that is regarded as expert in commercial law matters.

Seller shipped the vending machines to Germany, but Buyer refused the shipment after discovering that
the goods had been seriously damaged during the ocean voyage and arrived in Germany in a worthless
condition. Seller then sued Buyer in federal district court in State A, properly invoking the court’s subject
matter jurisdiction and seeking recovery of the $500,000 contract price on the ground that the risk of
damage to the goods during transport was on Buyer throughout the ocean voyage.

In responding to Seller’s complaint, Buyer moved to transfer the case to the federal district court in State
N, pursuant to 28 U.S.C. § 1404(a) and the forum-selection clause in the contract. Seller resisted Buyer’s
transfer request on the grounds that (a) State A was a more convenient forum for Seller, and (b) the
forum-selection clause was unenforceable under State A law, which declares such clauses to be “void as a
matter of public policy.”

While the transfer motion was pending, Seller delivered a notice of deposition to Buyer, demanding that
Buyer’s chief executive officer appear for a deposition. Buyer responded by asking the court for a
protective order on the ground that its “officers, directors, and managing agents” are all in Germany and
are therefore beyond the subpoena power or other authority of the court.

Assume that there are no applicable international law principles or treaties.

1. Should the federal district court in State A transfer the action to State N? Explain.

2. Irrespective of how the court rules on the transfer motion, should Buyer’s request for a protective
order be granted? Explain.

Seperac-J19 Exam-Released MEE Essay Compilation © 2016-2020 147


#031: J02-3 MEE: ANSWER: NCBE (CIVIL PROCEDURE)

POINT (1)(a) [29%] ISSUE: What effect should the federal district court give to the forum-
selection clause? ANSWER: Because Buyer is seeking to transfer the case from one federal court to
another federal court, the motion to transfer is governed by State A’s policy against forum-selection
clauses is irrelevant, and the forum-selection clause is an important factor the court should consider
in deciding whether to grant the motion to transfer.

POINT (1)(b) [48%] ISSUE: Is the court likely to transfer the case to State N in these
circumstances? ANSWER: Yes. The federal district court in State A should grant the motion to
transfer because the transfer would uphold the parties’ expectations and provide them with the
neutral and experienced forum for which they bargained.

POINT (2) [24%] ISSUE: Can a foreign-based party to U.S. litigation refuse to make its officers
available for depositions on the ground that they are “beyond the subpoena power” of the court?
ANSWER: No. Under Federal Rule of Civil Procedure 30, Seller has the right to compel the
deposition of an officer of Buyer without resorting to the court’s subpoena power.

ANSWER DISCUSSION:

A motion for transfer of a case from one federal venue to another should be granted if a transfer would
serve the convenience of the parties and the witnesses, if it would be in the interests of justice, and if the
transferee court would have been a proper venue in the first instance. All of these requirements are
satisfied here. The existence of a freely and fairly negotiated forum selection clause favoring State N is
strong evidence that State N is a convenient forum, and the facts support the conclusion that State N is
generally convenient for both parties, although State A is more convenient for plaintiff. Justice also favors
upholding the expectations of the parties when they have freely selected State N as an appropriate forum.
The action could also have been brought in State N initially. When a corporate party is properly before a
court (and no objection to personal jurisdiction was made here), the opposing party may depose an
appropriate officer or director to speak for the corporation. The deposing party does not need to rely upon
the court’s subpoena power for the exercise of authority over the officer or director. A protective order is
proper only if there is some other basis to oppose the deposition (e.g., the party that noticed the deposition
did not proceed in accordance with the federal rules governing the timing of discovery).

ANSWER EXPLANATION:

Explanation to Point-One(a) (25-35%):

Because Buyer is seeking to transfer the case from one federal court to another federal court, the motion
to transfer is governed by State A’s policy against forum-selection clauses is irrelevant, and the forum-
selection clause is an important factor the court should consider in deciding whether to grant the motion to
transfer.

Motions to transfer cases from one federal court to another are governed by 28 U.S.C. § 1404, which
provides:

For the convenience of parties and witnesses, in the interest of justice, a district court may transfer any
civil action to any other district or division where it might have been brought.

Seperac-J19 Exam-Released MEE Essay Compilation © 2016-2020 148


Under this statute, motions to transfer are determined by weighing a number of factors, including the
“convenience of witnesses and parties,” and “public-interest factors of systemic integrity and fairness.”

When a venue-transfer motion is predicated on the existence of a forum-selection clause in a contract, the
forum-selection clause “will be a significant factor that figures centrally in the district court’s calculus.”
This is because “the presence of a forum-selection clause,” bears upon the “convenience of the proposed
forum,” given the parties’ expressed preference for that venue. Thus, the court must evaluate the fairness
of the proposed transfer in light of the forum-selection clause. Further, the existence of a state law rule
invalidating the forum-selection clause does not excuse the court from considering the clause as one
factor bearing on the propriety of a venue transfer.

Although the forum-selection clause must be treated as a “significant factor” when the district court
evaluates Buyer’s transfer motion, the Supreme Court has said that the existence of a forum-selection
clause is not “dispositive.” For instance, an inequality of bargaining power between the parties may
warrant giving less effect to the forum-selection clause. Similarly, litigating in the proposed forum may
impose significant and unusual hardships on one of the parties. Absent such special factors, however,
many courts hold that transfer motions under § 1404 should always be granted when doing so will give
effect to a freely negotiated and fair forum-selection clause.

Explanation to Point-One(b) (45-55%):

The federal district court in State A should grant the motion to transfer because the transfer would uphold
the parties’ expectations and provide them with the neutral and experienced forum for which they
bargained.

Applying the factors set out in Point One(a) should lead to transfer of this action. The forum-selection
clause in this case was a bargained clause in a contract between parties of apparently equal bargaining
power. State N appears to be a neutral forum, and State N courts are well known for their expertise in
commercial matters. Although State N does not appear to be an especially convenient forum, the parties’
decision to select that forum in their contract indicates that each party believed State N to be a reasonably
convenient alternative to less desirable options, including the option of litigating in either party’s home
jurisdiction. Absent some evidence to suggest that a transfer to State N would result in special hardship or
a denial of justice to either party, the court is likely to take the view that the parties’ decision to select the
State N forum was ample evidence of the general superiority of that forum to State A.

This conclusion is bolstered by the fact that this is an international case. In Bremen v. Zapata Off-Shore
Co., the Supreme Court adopted the view that a forum-selection clause in an international contract
“should control absent a strong showing that it should be set aside.” According to the Court, the need for a
neutral forum, the desirability of avoiding uncertainty about where litigation might occur, and the need for
a forum with expertise in the subject matter all justify giving deference to the parties’ choice of forum
when that choice is made in “a freely negotiated private international agreement, unaffected by fraud,
undue influence, or overweening bargaining power.”

[NOTE: Although Bremen was an admiralty case, the Supreme Court has described the decision as
“instructive” for other cases, including motions under § 1404 to transfer diversity actions.]

A transfer under § 1404 must be to a “district or division” where the case “might have been brought”
initially. Here, it might be argued that the case could not have been brought in State N because there are
no connections with State N sufficient to give State N courts jurisdiction over the defendant. However,
personal jurisdiction can be based on consent, and both parties clearly consented to State N jurisdiction in

Seperac-J19 Exam-Released MEE Essay Compilation © 2016-2020 149


their contract. Pre-lawsuit contractual consent serves as a basis for State N service of process on the
defendant corporation, and would therefore have been an adequate basis for personal jurisdiction and
venue had the action been brought in State N originally. In addition, because the defendant corporation is
an alien corporation, venue in a suit brought against it would be proper in any district in the U.S.,
including in a federal district court in State N.

Explanation to Point-Two (20-30%):

Under Federal Rule of Civil Procedure 30, Seller has the right to compel the deposition of an officer of
Buyer without resorting to the court’s subpoena power.

A party to an action may secure the deposition of another party simply by providing notice to the other
party. No subpoena of a party deponent is necessary, and a party is subject to sanctions for failing to
provide a deponent, even if the party would be beyond the reach of the court’s subpoena power if the
party were a non-party witness.

As a corporate party, Buyer cannot speak for itself but requires a natural person to speak for it. Seller
could have noticed Buyer’s deposition and required Buyer to nominate a representative to be deposed.
Alternatively, Seller can do what it did here – it can identify an officer or director of Buyer to be deposed
as Buyer’s representative. If the person fails to appear, sanctions are authorized by Rule 37. Hence,
Buyer’s argument that its officers are “beyond the court’s subpoena power” does not present a valid
reason to prevent the deposition of an officer. Buyer, as a party to the litigation, is obliged to produce its
officers for deposition. Accordingly, Buyer’s request for a protective order should be denied, unless
Buyer can offer sound reasons for resisting the deposition other than the simple claim that its CEO is
beyond the reach of the court’s subpoena power.

[NOTE: If the court had no personal jurisdiction over defendant corporation, the claim that the officers
and directors are beyond the court’s subpoena power might be a colorable basis to resist the notice of
deposition. The facts, however, suggest that personal jurisdiction is not a real issue. First, any problem
with the State A federal court’s personal jurisdiction over defendant Buyer was waived when Buyer
responded to Seller’s complaint without raising the personal jurisdiction issue. ) Second, if the case is
transferred, the federal court in State N would have personal jurisdiction over defendant Buyer because
of its contractual consent to the jurisdiction of that court.]

Seperac-J19 Exam-Released MEE Essay Compilation © 2016-2020 150


#032-FEB 2002–MEE Q01: QUESTION ONE (CIVIL PROCEDURE)

Teacher lived in State A, where he taught in the public schools for eight years. Toward the end of his
eighth year, Teacher was offered a lucrative teaching position in State B. Teacher decided to accept the
offer and to move to State B. Teacher notified his school principal that he was leaving and put his State A
home up for sale.

The day he sold his house, Teacher packed his belongings, rented a truck from Rentco, a State C
corporation with its principal place of business in State A, and began the long drive to State B. While
speeding on a State A highway toward State B, Teacher came upon a slow-moving car and applied the
brakes in an effort to avoid an accident. The brakes failed, and Teacher rear-ended the slow-moving car.
Passenger, a citizen of State A who was riding in the slower car, suffered severe back and neck injuries in
the collision. After the State A police prepared a report of the accident, Teacher continued on to State B.
Shortly after he arrived there, he purchased a home and began his new job.

Nine months after the accident, Passenger filed suit against Teacher in the United States District Court for
the District of State A, alleging that Teacher’s negligence caused Passenger serious personal injury.
Passenger’s complaint sought damages in excess of $100,000.

Teacher timely moved to dismiss the complaint on the grounds that the court lacked subject matter
jurisdiction and that venue was improper. While the motion to dismiss was pending, Teacher filed a third-
party complaint against Rentco, seeking indemnification and claiming that the accident occurred because
the brakes on the rental truck were defective. Before Teacher’s motion could be heard, Passenger
amended her complaint to state a claim directly against Rentco for negligence.

1. How should the federal district court rule on Teacher’s motion to dismiss for lack of subject
matter jurisdiction? Explain.

2. How should the federal district court rule on Teacher’s motion to dismiss for improper venue?
Explain.

3. If the district court were to deny Teacher’s motion to dismiss on both grounds, would it have
subject matter jurisdiction over Passenger’s direct claim against Rentco? Explain.

Seperac-J19 Exam-Released MEE Essay Compilation © 2016-2020 151


#032: F02-1 MEE: ANSWER: NCBE (CIVIL PROCEDURE)

POINT (1) [38%] ISSUE: Were the requirements of the diversity jurisdiction statute met here, even
though at the time of the accident Passenger, the plaintiff, and Teacher, the defendant were citizens
of the same state? ANSWER: Yes. Because Passenger and Teacher were citizens of different states
when the complaint was filed and the amount in controversy exceeds $75,000, Passenger properly
invoked federal diversity jurisdiction. It is irrelevant that the plaintiff and the defendant were
citizens of the same state at the time of the accident.

POINT (2) [19%] ISSUE: Were the requirements of the venue statute met here, given that
Passenger filed in her home state, which was where the accident occurred? ANSWER: Yes.
Because venue is proper in a diversity case in the judicial district in which a “substantial part of the
events or omissions giving rise to the claim occurred,” and because the accident occurred in State
A, venue is proper in the federal district court of State A.

POINT (3) [43%] ISSUE: Is supplemental jurisdiction available over Passenger’s claim against
Rentco, the third-party defendant, given that they are citizens of the same state? ANSWER: No.
Although Passenger’s claim against Teacher and her claim against third-party defendant Rentco
derive from a common nucleus of operative fact and hence are part of the same case for purposes of
Article III of the Constitution, supplemental jurisdiction is statutorily unavailable over claims by
plaintiffs against persons made parties under Rule 14.

ANSWER EXPLANATION:

Explanation to Point-One (35-45%):

Because Passenger and Teacher were citizens of different states when the complaint was filed and the
amount in controversy exceeds $75,000, Passenger properly invoked federal diversity jurisdiction. It is
irrelevant that the plaintiff and the defendant were citizens of the same state at the time of the accident.

The court should deny Teacher’s motion to dismiss for lack of subject matter jurisdiction. The
requirements for federal diversity jurisdiction have been established in this case.

Passenger’s complaint states only a state-law negligence claim, so federal question jurisdiction under 28
U.S.C. § 1331 is unavailable. Diversity jurisdiction is available under 28 U.S.C. § 1332(a)(1) only if
Passenger and Teacher are citizens of different states and the amount in controversy exceeds $75,000,
exclusive of costs and interest. An individual is a citizen of the state in which he or she is domiciled. In
order to change one’s domicile, one must be physically present in the new place with the intent to make
that place one’s permanent home.

The problem states that Passenger is a citizen of State A. At the time of the accident, Teacher had sold his
home in State A, quit his job in State A, and accepted a new job in State B. Teacher was, however, still a
domiciliary of, and hence a citizen of, State A because he had not yet reached State B after having formed
the requisite intent to change his domicile. Had the suit been filed the day of the accident, diversity
jurisdiction would have been unavailable because both the physical and mental elements must be present
for a change of domicile.

Seperac-J19 Exam-Released MEE Essay Compilation © 2016-2020 152


It is well settled, however, that the availability of diversity jurisdiction is determined as of the date the suit
is commenced. Having purchased a home in State B and begun his new job shortly after his arrival there,
Teacher had established a new domicile in State B. Hence, at the time suit was filed in this case, nine
months after the accident, Teacher had become a domiciliary and citizen of State B. Thus, diversity of
citizenship existed between Passenger and Teacher.

In addition to diversity of citizenship, the amount in controversy must exceed $75,000, exclusive of costs
and interest. Because Passenger’s complaint seeks damages in excess of $100,000, the amount in
controversy requirement is met.

The motion to dismiss for lack of jurisdiction should be denied: there is diversity of citizenship, and the
amount in controversy requirement is satisfied.

Explanation to Point-Two (15-25%):

Because venue is proper in a diversity case in the judicial district in which a “substantial part of the events
or omissions giving rise to the claim occurred,” and because the accident occurred in State A, venue is
proper in the federal district court of State A.

The court should deny Teacher’s motion to dismiss for improper venue. The requirements of the venue
statute, 28 U.S.C. § 1391, are met.

Venue is a geographical concept that localizes lawsuits in places that are connected either to the parties or
to the events giving rise to the action. Under 28 U.S.C. § 1391(a), which applies to civil actions in which
jurisdiction is founded solely on diversity of citizenship (as is the case here), venue is proper in “(1) a
judicial district where any defendant resides , or (2) a judicial district in which a substantial part of the
events or omissions giving rise to the claim occurred.” However, the accident itself occurred in State A.
Thus, a substantial part of the events giving rise to the claim occurred in State A, and venue in State A is
proper under § 1391(a)(2).

[NOTE: Some examinees might suggest that venue is proper in the federal district court of State A
because the plaintiff, Passenger, resides there. Although under the pre-1990 version of the venue statute,
venue in diversity cases was proper in the district “where all plaintiffs reside,” that language was
eliminated by enactment of the Judicial Improvements Act of 1990. Thus, under current law, the plaintiff’s
residence in State A is irrelevant to venue.]

Explanation to Point-Three (40-50%):

Although Passenger’s claim against Teacher and her claim against third-party defendant Rentco derive
from a common nucleus of operative fact and hence are part of the same case for purposes of Article III of
the Constitution, supplemental jurisdiction is statutorily unavailable over claims by plaintiffs against
persons made parties under Rule 14.

The district court cannot exercise subject matter jurisdiction over Passenger’s direct claim against Rentco.
There is no independent basis for jurisdiction (neither diversity nor federal question), and supplemental
jurisdiction is not available for this type of claim.

Rule 14(a) of the Federal Rules of Civil Procedure permits a defendant to bring a third party into an action
if the defendant believes the third party “may be liable” to defendant for all or part of any judgment
against defendant in the action. Pursuant to this rule, defendant Teacher brought Rentco into the action as

Seperac-J19 Exam-Released MEE Essay Compilation © 2016-2020 153


a third-party defendant. Passenger sought thereafter to amend the original complaint to state a claim
directly against Rentco. The issue is whether the court has jurisdiction over Passenger’s direct claim
against Rentco. For reasons explained below, the court cannot take jurisdiction over Passenger’s claim.

In the first place, there is no independent basis for federal jurisdiction over this claim. Plaintiff Passenger
and third-party defendant Rentco are both citizens of State A, so diversity jurisdiction is not available for
Passenger’s claim against Rentco. Under § 1332(c)(1), a corporation is deemed a citizen of both its state
of incorporation and the state in which it maintains its principal place of business. Hence, Rentco is a
citizen of both State A and State C. Because Passenger is a citizen of State A as well, there is no
independent basis for diversity jurisdiction over Passenger’s direct claim against third-party defendant
Rentco. Further, because Passenger’s claim against Rentco is a state-law negligence claim, there is no
federal question jurisdiction.

Supplemental jurisdiction is also not available. Section 1367(a) provides that in a civil action of which the
district court has original jurisdiction, the court has supplemental jurisdiction over “all other claims that
are so related to claims in the action within such original jurisdiction that they form part of the same case
or controversy under Article III of the United States Constitution.” In United Mine Workers of America v.
Gibbs, the U.S. Supreme Court held that two claims will have that requisite connection if they “derive
from a common nucleus of operative fact.” Here, Passenger’s claim against Teacher and Passenger’s
claim against third-party defendant Rentco derive from a common nucleus of operative fact: the failure of
the brakes, the accident, and Passenger’s resultant injuries. Without regard to their federal or state
character, we would expect plaintiff to bring these claims together in a single action.

However, Congress created statutory exceptions to supplemental jurisdiction. One of those exceptions, 28
U.S.C. §1367(b), applies here to prevent the court from taking jurisdiction. Section 1367(b) provides that
supplemental jurisdiction in diversity cases shall not extend to “claims by plaintiffs against persons made
parties under Rule 14.” Because Passenger’s claim against Rentco is a claim by a plaintiff against a
person made a party under Rule 14 (Rentco was brought into the suit when Teacher filed a third-party
complaint against it under Rule 14), supplemental jurisdiction is unavailable. Passenger could proceed
directly against Rentco only if there were an independent basis for jurisdiction (e.g., diversity), and there
is none.

Because there is no independent basis for jurisdiction over this claim and because supplemental
jurisdiction is unavailable, the district court should dismiss Passenger’s claim against Rentco

Seperac-J19 Exam-Released MEE Essay Compilation © 2016-2020 154


#033-JUL 2001–MEE Q07: QUESTION SEVEN (CIVIL PROCEDURE)

Plaintiff, a resident of State X, is a shareholder in Silver, Inc., a State X corporation. Plaintiff purchased
10,000 shares of Silver stock for $20 a share in Silver’s initial public offering (IPO). Three weeks after
the IPO, the value of Silver stock had fallen to $15 a share, and Plaintiff filed an action for securities fraud
against Silver in federal district court. The complaint alleged that the registration statement for the IPO
filed by Silver with the Securities and Exchange Commission (SEC) was materially false and misleading,
and therefore violated relevant provisions of the Securities Exchange Act of 1934 as well as SEC
regulations. Plaintiff sought damages of $50,000 for this violation.

Shortly after Plaintiff’s initial filing, the SEC filed a public enforcement lawsuit against Silver, SEC v.
Silver, in another federal district court. The claim asserted by the SEC, that the registration statement for
the IPO contained false and misleading representations, was identical to the claim asserted in the Plaintiff
v. Silver lawsuit. The SEC lawsuit went to trial very quickly, and Silver vigorously contested the
misrepresentation issues in the SEC lawsuit. The SEC won its lawsuit. A declaratory judgment was
entered finding that Silver’s registration statement contained false and misleading information and
enjoining Silver from making those misrepresentations in the future. That judgment became final, and
Silver did not appeal.

Plaintiff then moved for partial summary judgment against Silver based on the judgment in SEC v. Silver.
Plaintiff argued that the SEC v. Silver judgment conclusively determined the issue whether the
information contained in the registration statement was false and misleading. Silver opposed the motion
on the ground that it had the right to litigate this issue against Plaintiff because Plaintiff had not been a
party to the SEC v. Silver lawsuit.

The district court judge denied Plaintiff’s motion for partial summary judgment on the ground that
Plaintiff, a non-party to the SEC v. Silver action, could not rely upon the judgment in that case to preclude
Silver from relitigating issues that were central to Plaintiff’s claim. Plaintiff filed an appeal from this
decision, although Plaintiff’s case against Silver is still going forward in the district court.

Silver has moved to dismiss Plaintiff’s appeal on the following grounds: (a) that the district court’s denial
of Plaintiff’s judgment was not a final judgment subject to review; (b) that the collateral order exception
is inapplicable; and (c) mandamus does not lie.

1. Did the district court properly refuse to give preclusive effect to the SEC v. Silver judgment
when it denied Plaintiff’s motion for partial summary judgment? Explain.

2. How should the appellate court rule on each of the grounds asserted in Silver’s motion to dismiss
the appeal? Explain.

Seperac-J19 Exam-Released MEE Essay Compilation © 2016-2020 155


#033: J01-7 MEE: ANSWER: NCBE (CIVIL PROCEDURE)

POINT (1) [50%] ISSUE: May a litigant who was not a party to a prior judgment invoke that
judgment against another litigant who was a party in order to preclude relitigation of the identical
issue previously decided? ANSWER: Yes. Plaintiff, a non-party to the SEC v. Silver action, may
rely on the decision in that action to prevent Silver from relitigating the question of whether
Silver’s registration statement was false and misleading.

POINT (2)(a) [35%] ISSUE: Is a federal district court’s order refusing to grant summary judgment
on collateral estoppel grounds a final judgment? ANSWER: No. The appellate court should dismiss
Plaintiff’s interlocutory appeal because there is no final judgment, and the appeal is premature.

POINT (2)(b) [15%] ISSUE: Are there any exceptions to the final judgment rule that would permit
review of an order refusing to grant summary judgment? ANSWER: There are no exceptions to the
final judgment rule that are likely to apply to these facts.

ANSWER EXPLANATION:

Explanation to Point-One (45-55%):

Plaintiff, a non-party to the SEC v. Silver action, may rely on the decision in that action to prevent Silver
from relitigating the question of whether Silver’s registration statement was false and misleading.

This question involves the scope of the doctrine of collateral estoppel and the right of a non-party to an
action to use collateral estoppel offensively against a losing party in the action. If Silver is precluded by
the SEC v. Silver judgment from relitigating the question of whether its registration statement was false or
misleading, and if Plaintiff is entitled to assert that preclusive effect against Silver, then Plaintiff’s motion
for summary judgment on that issue should have been granted.

The doctrine of collateral estoppel (or issue preclusion) generally prevents a litigant from relitigating
issues that have been previously litigated and determined in a prior action. The traditional requirements
for asserting collateral estoppel are: (1) a valid and final judgment was rendered in a prior action; (2) an
issue of fact was actually litigated, determined, and essential to the judgment in the prior action; (3) the
same issue arises in a subsequent action; and (4) the same parties are litigants in both actions.

The first three elements of this test are clearly satisfied. First, the judgment in the prior case (SEC v.
Silver) was final according to the facts. Second, the issue of fact relating to the misrepresentation was
actually litigated, determined, and essential to the judgment in the prior action. Here, Silver vigorously
contested the misrepresentation claims in the SEC lawsuit. Moreover, the finding of misrepresentation in
that lawsuit was the basis for a grant of both declaratory and injunctive relief. Thus, the factual
misrepresentation issues were actually litigated, determined (e.g., declaratory judgment), and essential to
the judgment enjoining Silver. Third, the misrepresentation issues in the prior action, SEC v. Silver, were
identical to the misrepresentation claims in the subsequent action, Plaintiff v. Silver. Both actions
revolved around whether the registration statement contained false and misleading representations of fact.
However, the parties to the current action are not the same, and that fact alone would prevent collateral
estoppel under traditional doctrine.

Seperac-J19 Exam-Released MEE Essay Compilation © 2016-2020 156


The modern approach to collateral estoppel, however, allows a non-party to the prior litigation (like
Plaintiff) to invoke collateral estoppel against a party to the prior litigation, at least under certain
circumstances. Section 29 of the Second Restatement of Judgments reflects the modern view in stating
that “a party precluded from relitigating an issue with an opposing party, in accordance with Sections 27
and 28, is also precluded from doing so with another person unless the fact that he lacked full and fair
opportunity to litigate the issue in the first action or other circumstances justify affording him an
opportunity to relitigate the issue.” When § 29 applies, there is a new fourth element in place of the “same
parties” requirement. The new fourth element is that the party to be precluded had a full and fair
opportunity to litigate the same issue in the prior action.

In Parklane Hosiery Co. v. Shore, the U.S. Supreme Court adopted this approach, upholding the use of
offensive collateral estoppel against Parklane, which had litigated the relevant misrepresentation issues in
a prior action brought by the SEC. Shore, a non-party to the prior lawsuit, was allowed to use collateral
estoppel against Parklane in a subsequent lawsuit to preclude Parklane from relitigating the same issue a
second time. The Court held that Parklane was not unfairly prejudiced because it had had a full and fair
opportunity to litigate the issues in the earlier lawsuit. Thus, the Parklane case is consistent with the
requirements of § 29, and it adopts offensive collateral estoppel as controlling federal law in federal
courts.

Thus, while Plaintiff was not a party to the SEC lawsuit, and therefore could not have been bound if the
SEC lawsuit judgment had been for Silver, Plaintiff is permitted to use collateral estoppel offensively in
the subsequent lawsuit provided that Silver had a full and fair opportunity to litigate the same issue in the
prior lawsuit. Here, Silver had that opportunity, and it vigorously litigated those issues. Accordingly, the
trial judge erred in denying Plaintiff’s motion for summary judgment.

Explanation to Point-Two(a) (30-40%):

The appellate court should dismiss Plaintiff’s interlocutory appeal because there is no final judgment, and
the appeal is premature.

The basic policy governing appeals from district court decisions is embodied in the final judgment rule,
which prohibits appeals, with some exceptions, prior to entry of a final judgment by the trial court at the
completion of the case in the trial court. This policy is designed to prevent the negative impact on trial and
appellate courts that would occur if there were several appeals from rulings of the trial court during the
course of the trial of a case. Frequent appeals would disrupt the trial court’s ability to move forward with
the case and would strain the judicial resources of the appellate courts. The final judgment policy is
embodied in Title 28 U.S.C. § 1291, which provides in relevant part that “the courts of appeals shall have
jurisdiction of appeals from all final decisions of the district courts of the United States.”

Plaintiff’s appeal of the denial of the motion for summary judgment is clearly interlocutory and not final.
Although the statute does not define “final decision,” the courts hold that a “final decision is one that ends
the litigation on the merits so that the only thing left for the district court to do is to execute the
judgment.” Here, the denial of Plaintiff’s motion does not end the litigation; to the contrary, the trial on
the merits will be more complex because of the decision. There was, accordingly, no final decision, and
Plaintiff’s appeal was premature under § 1291 unless an exception applies.

Explanation to Point-Two(b) (10-20%):

There are no exceptions to the final judgment rule that are likely to apply to these facts.

Seperac-J19 Exam-Released MEE Essay Compilation © 2016-2020 157


The applicants are asked to discuss the following exceptions to the final judgment rule: (1) the collateral
order doctrine, and (2) mandamus review under 28 U.S.C. § 1651;

The collateral order doctrine allows review of final orders of district court judges when those orders are
collateral to (or separate from) the merits of the lawsuit, and the orders are effectively unreviewable on
appeal. Here, however, the order at issue (allowing relitigation of the issue of misrepresentation) is
intimately connected with the merits of the lawsuit and is certainly subject to review on appeal of any
final judgment. The collateral order doctrine is therefore inapplicable.

Plaintiff will not be able to rely on mandamus review because this requires a showing that the trial court
decision is beyond its jurisdiction or violated a mandatory duty owed by the trial court. Although the trial
court should have granted the motion for partial summary judgment, the fact that the trial court made an
error of law does not warrant mandamus where there is otherwise no abuse of judicial authority. A wider
use of mandamus to review legal errors would effectively eviscerate the final judgment rule, and appellate
courts have accordingly been wary of a broad application of mandamus review. “A party may not use a
writ of mandamus to correct ordinary error by the district court. The burden is on the applicant to show
intolerable error or misbehavior.” The error in this case would not cross that high threshold, at least absent
some evidence of extraordinary hardship to Plaintiff as a result of the error.

Seperac-J19 Exam-Released MEE Essay Compilation © 2016-2020 158


#034-FEB 2001–MEE Q05: QUESTION FIVE (CIVIL PROCEDURE)

Victim, a resident of State A, suffered personal injuries in a fall that occurred when she was exiting a tour
bus operated by Bus, Inc., a company incorporated under the laws of State B with its principal place of
business in State B. At the time of the injury, Victim and other tour guests were sightseeing in State A.

Victim filed a complaint against Bus in federal district court in State B. The complaint alleged negligence
and requested damages in the amount of $100,000 for Victim’s medical expenses, lost wages, and pain
and suffering. In its answer, Bus denied that it was negligent.

The federal district court in State B requires the initial disclosure provided for by Federal Rule of Civil
Procedure 26(a)(1).

After the pleadings closed and the deadline for initial disclosure of documents had passed, the parties
engaged in formal discovery for several months. At no time during discovery did either party request or
produce the tour ticket or any copy of it.

The federal district judge called a pretrial conference, ordering both counsel and litigants to appear “to
discuss further scheduling and the possibility of settlement.” No corporate representative of Bus attended
the conference. When the judge raised the question of settlement and asked why no representative of Bus
had appeared as required by the judge’s order, counsel for Bus stated that Bus was unwilling to settle or
even discuss settlement and argued that the court lacked authority to require Bus’s participation. At that
point, the judge held counsel and Bus in contempt for their refusal to participate in settlement discussions.

The final pretrial order, issued after the conference, identified the issues for trial as whether Bus had
negligently maintained and operated the tour bus, whether such negligence caused Victim’s injuries, and
whether Victim’s injuries were compensable in damages.

At trial, after the presentation of Victim’s case in chief, Bus introduced the stub of the ticket used by
Victim, which contained a “Waiver of Liability” clause. When Victim’s counsel objected, the court
sustained the objection, ruling that the waiver was beyond the scope of the final pretrial order and beyond
the scope of the pleadings. Bus’s counsel then moved to amend the answer and the pretrial order to
include the question of waiver. The judge denied the motion. The jury ultimately returned a general
verdict for Victim and awarded Victim full damages. The court entered judgment on the verdict.

Bus has appealed the contempt order on the ground that the trial judge lacked the power to order Bus to
participate in settlement discussions. Bus has also appealed the judgment on the grounds that the trial
judge erred in not allowing Bus to (1) amend the answer, (2) amend the pretrial order, and (3) introduce
the waiver into evidence despite not having produced it earlier.

1. Should the contempt order be set aside on the ground urged by Bus? Explain.

2. Should the judgment be reversed on the grounds urged by Bus? Explain.

Seperac-J19 Exam-Released MEE Essay Compilation © 2016-2020 159


#034: F01-5 MEE: ANSWER: NCBE (CIVIL PROCEDURE)

POINT (1) [40%] ISSUE: Did the federal district court abuse its discretion when it ordered Bus to
participate in settlement discussions and cited it for contempt for violating that order? ANSWER:
No. A federal district judge has explicit authority under FRCP Rule 16 to order litigants to
participate in settlement discussions, although sanctions to compel participation should be used
with caution.

POINT (2)(a) [50%] ISSUE: What standard governs a motion to amend pleadings and to amend a
pretrial order to allow a litigant to introduce a new issue during trial? ANSWER: Under Rule 16, a
final pretrial order may be modified only to prevent manifest injustice. That standard supersedes
the more liberal policy governing amendment of pleadings under Rule 15.

POINT (2)(b) [10%] ISSUE: Should a party be permitted to amend a pretrial order to introduce
evidence that the party did not mention or disclose during discovery or the pretrial conference?
ANSWER: No. If a party fails to produce a document that should have been disclosed under the
mandatory disclosure provisions of the federal discovery rules, the party ordinarily will not be
permitted to use the document as evidence at trial.

ANSWER EXPLANATION:

Explanation to Point-One (35-45%):

A federal district judge has explicit authority under FRCP Rule 16 to order litigants to participate in
settlement discussions, although sanctions to compel participation should be used with caution.

Until the 1993 amendments of Rule 16 of the Federal Rules of Civil Procedure, the question of whether
federal courts could order litigants to participate in settlement talks was in dispute. In the 1993
amendments to Rule 16, the drafters added the following language to Rule 16(c):

If appropriate, the court may require that a party or its representative be present or reasonably available by
telephone in order to consider possible settlement of the dispute.

As explained in the Advisory Committee Notes, the change was intended, in part, to make it clear that
courts may issue orders designed to facilitate settlement. There is, therefore, no question that the court had
authority to require a corporate representative of Bus to be present or available for settlement discussions.

The judge also had authority to hold Bus in contempt for failing to attend the conference. Rule 16(f)
explicitly provides that “if no appearance is made on behalf of a party at a scheduling or pretrial
conference , or if a party or a party’s attorney fails to participate in good faith, the judge, upon motion or
the judge’s own initiative, may make such orders with regard thereto as are just, and among others, any of
the orders provided for in Rule 37.” Rule 37 permits the court to treat “as a contempt of court the failure
to obey any orders.” Because Bus failed to attend the conference and failed to participate in settlement
negotiations in good faith, the court did not abuse its discretion under Rules 16 and 37 by holding Bus in
contempt.

The Advisory Committee Notes to Rule 16 caution that settlement talks will be unproductive if a party is
uncooperative. Bus, then, has a reasonable argument on appeal that the district judge should not have

Seperac-J19 Exam-Released MEE Essay Compilation © 2016-2020 160


imposed the contempt sanction once Bus’s counsel made it clear that Bus was recalcitrant about
settlement and that discussions would be pointless. It is not likely, however, that the district court’s
decision was an abuse of discretion sufficient to warrant reversal. There is no evidence that the order for
party participation imposed great hardship on Bus, and the court may have believed that Bus’s
recalcitrance was merely posturing that could be overcome by serious discussion. Thus, although a case
for reversal can be made, it does not seem likely to succeed on these facts.

Explanation to Point-Two(a) (45-55%):

Under Rule 16, a final pretrial order may be modified only to prevent manifest injustice. That standard
supersedes the more liberal policy governing amendment of pleadings under Rule 15.

In the fact pattern, the defendant has sought to introduce the issue of a “waiver of liability” at trial,
without having raised that issue in the pleadings or having included it in the pretrial order. Ordinarily,
when evidence is objected to at trial on the ground that it is not within those issues raised by the
pleadings, Rule 15(b) authorizes the court to allow an amendment of the pleadings at trial and directs the
court to “do so freely when the presentation of the merits of the action will be subserved thereby and the
objecting party fails to satisfy the court that the admission of such evidence would prejudice the party in
maintaining the party’s action or defense upon the merits.” An amendment allowing Bus to raise the
waiver as a defense would subserve the presentation of the merits. Any prejudice that Victim might suffer
presumably could be minimized by granting an extension to permit Victim to meet the new evidence.
Thus, an examination of Rule 15(b) alone suggests that the court abused its discretion in declining to
permit Bus to amend its answer and to offer the ticket stub at trial.

But Rule 15(b) cannot be read in a vacuum. Under Rule 16(e) of the Federal Rules of Civil Procedure,
“the order following a final pretrial conference shall be modified only to prevent manifest injustice.”
According to the Advisory Committee Notes, pretrial orders should not be changed lightly, but total
inflexibility is undesirable. In determining whether to admit evidence and to allow issues that were not
identified in the pretrial order, the courts have discretion to consider such factors as the prejudice to the
opposing party, the extent to which introduction of the new evidence would disrupt the orderly
presentation of the trial, and the possible bad faith of the party seeking to disregard the pretrial order.
Thus, Rule 16 imposes a heavy burden on a party seeking to amend a pretrial order, but the trial court has
such discretion when the danger of prejudice is slight and the failure to amend might result in an injustice.
On the other hand, if the evidence or issue was within the knowledge of the party seeking modification at
the time of the pretrial conference, the argument for amending the order is considerably weakened.

On the given facts, Bus does not have a strong argument that the trial court abused its discretion in barring
introduction of the evidence of a waiver of liability. It is difficult to imagine that “manifest injustice”
would result from denying Bus the opportunity to present evidence on a defense that the court could
reasonably conclude has been available to it from the beginning. Bus likely knew that the terms of its
ticket included a waiver. It apparently had the ticket stub in its possession throughout the proceedings.
Moreover, introduction of the new issue of waiver would probably prejudice Victim unless a continuance
were granted, and that would result in trial delay. In short, any harm caused to Bus by restricting it to the
evidence and claims allowed by the pretrial order does not appear to be a “manifest injustice.”

How should one resolve the tension between Rule 15(b), on the one hand, and Rule 16(e), on the other? It
appears that the Rule 16(e) amendment standard should apply because, according to the Rule itself, the
pretrial order “shall control the subsequent course of the action.” One should note, however, that “several
courts have permitted pretrial orders to be amended by concluding that Rule 16 must be read in

Seperac-J19 Exam-Released MEE Essay Compilation © 2016-2020 161


conjunction with Rule 15(b).” Thus, while it appears that the district court did not abuse its discretion in
declining to amend the final pretrial order, one could make a reasonable argument that the more liberal
Rule 15(b) standard should have been applied.

However, even if that argument is successful, it appears that the court properly barred Bus from offering
the ticket stub into evidence as a sanction for failing to produce it pursuant to Rule 26(a)(1).

Explanation to Point-Two(b)(05-15%):

If a party fails to produce a document that should have been disclosed under the mandatory disclosure
provisions of the federal discovery rules, the party ordinarily will not be permitted to use the document as
evidence at trial.

Bus’s failure to disclose the ticket stub pursuant to its mandatory disclosure obligation provides an
additional reason for the court to bar introduction of the evidence. Under Rule 26(a), a party must provide
a copy or description of “all documents that are relevant to disputed facts alleged with particularity in the
pleadings.” Here, the waiver of liability contained on the tour ticket is relevant because it would tend to
defeat Victim’s claim for recovery. The trial court’s decision to prevent Bus from using the evidence is an
appropriate sanction for non-disclosure of the ticket. “A party that without substantial justification fails to
disclose information required by Rule 26(a) shall not, unless such failure is harmless, be permitted to use
such information as evidence at a trial.”

Seperac-J19 Exam-Released MEE Essay Compilation © 2016-2020 162


#035-JUL 2000–MEE Q03: QUESTION THREE (CIVIL PROCEDURE)

Claimant properly invoked the diversity jurisdiction of the federal district court to bring suit to recover
damages for the death of Claimant’s spouse, Wife. Claimant alleged that Wife died as a result of exposure
to asbestos at BrakeCo, an employer that regularly used asbestos in its brakelining operation. The
complaint sought damages from all major asbestos manufacturers, including Fiberco, and demanded a
jury trial. Controlling law in all relevant jurisdictions would preclude recovery from Fiberco absent
specific proof by the plaintiff of exposure to Fiberco asbestos products.

At the final pretrial conference, Fiberco argued that Claimant had failed to produce any evidence through
discovery that would tend to establish that Wife had ever been exposed to Fiberco’s product. Although the
district court refused to grant summary judgment on this issue of exposure, the judge’s final pretrial order
did schedule the exposure issue as the first issue for trial to the jury (rather than permitting Claimant to
put on the case in the order he would prefer). In compliance with the pretrial order, Claimant opened the
trial with testimony from a former BrakeCo manager who testified that he often saw Fiberco asbestos
packages in the storeroom. The manager admitted on cross-examination, however, that the first time he
ever saw Fiberco products in the storeroom was after Wife had ceased working at BrakeCo, even though
he was in the storeroom at least once a week throughout the period that Wife worked at BrakeCo.

At the conclusion of this testimony, Claimant’s counsel indicated that Claimant had no other evidence to
offer on the issue. Fiberco thereupon moved for judgment as a matter of law. The court granted the
motion and dismissed the action as to Fiberco.

1. Did the district court have the power to require Claimant to present evidence on the exposure
issue before proceeding with any other portion of Claimant’s case? Explain.

2. Did the court err procedurally in granting the motion for judgment as a matter of law before the
close of evidence? Explain.

3. Did the court err substantively in granting the motion for judgment as a matter of law? Explain.

Seperac-J19 Exam-Released MEE Essay Compilation © 2016-2020 163


#035: J00-3 MEE: ANSWER: NCBE (CIVIL PROCEDURE)

POINT (1) [33%] ISSUE: Does the district court have the power to issue a pretrial order that
dictates the order in which issues will be tried to a jury? ANSWER: Yes. Under Rule 16, the
district court has the power to enter a pretrial order that specifies the order in which issues will be
tried.

POINT (2) [33%] ISSUE: Was the court justified procedurally in granting the motion for judgment
as a matter of law before the close of evidence? ANSWER: Yes. A motion for judgment as a matter
of law may be granted before the close of evidence if the opposing party has been fully heard on the
dispositive issue.

POINT (3) [33%] ISSUE: Was the court justified substantively in granting the motion for judgment
as a matter of law? ANSWER: Yes. A motion for judgment as a matter of law is properly granted
when it is clear that there is no legally sufficient basis for a verdict in favor of the nonmoving party.

ANSWER EXPLANATION:

Explanation to Point-One (25-35%):

Under Rule 16, the district court has the power to enter a pretrial order that specifies the order in which
issues will be tried.

Rule 16 of the Federal Rules of Civil Procedure gives district court judges the power to structure trials in
a manner that facilitates their expedited disposition. In particular, Rule 16(c) empowers courts to take
action to simplify potentially difficult or complex litigation. Rule 16(c)(14) specifically authorizes courts
to direct the parties “to present evidence early in the trial with respect to a manageable issue that could, on
the evidence, be the basis for a judgment as a matter of law under Rule 50(a).” This language offers ample
support for the court’s decision to require Claimant to present evidence of Fiberco exposure early to
facilitate a motion for judgment as a matter of law.

Explanation to Point-Two (25-35%):

A motion for judgment as a matter of law may be granted before the close of evidence if the opposing
party has been fully heard on the dispositive issue.

Rule 50 simplifies and clarifies federal practice on what were once called motions for directed verdicts,
bringing the practice into line with that on motions for summary judgment. In brief, Rule 50 permits a
party like Fiberco to move for judgment as a matter of law at any time during the course of a jury trial if
the opposing party “has been fully heard” on an issue. Thus, Claimant’s indication that the testimony of
BrakeCo’s manager concluded his proof on the issue of exposure to Fiberco asbestos suggests that the
motion for judgment as a matter of law was procedurally proper at the time it was made. Fiberco was not
required to await submission of Claimant’s entire case to make a procedurally proper motion for judgment
as a matter of law, and the trial court is not obliged to wait before ruling on it.

Explanation to Point-Three (25-35%):

Seperac-J19 Exam-Released MEE Essay Compilation © 2016-2020 164


A motion for judgment as a matter of law is properly granted when it is clear that there is no legally
sufficient basis for a verdict in favor of the nonmoving party.

It also appears that the court correctly resolved the motion in Fiberco’s favor. Rule 50 specifies the
standard for the grant of a motion for judgment as a matter of law, indicating that such motions may be
granted whenever “there is no legally sufficient evidentiary basis for a reasonable jury to find for” the
nonmoving party. In this case, the problem specifies that the relevant jurisdiction has rejected the so-
called market share theory of liability and places the burden on the plaintiff to produce specific evidence
establishing exposure to the defendant’s asbestos product. While the manager’s testimony places Fiberco
products in the firm’s storeroom, it does not establish that the products were there at the time Wife
worked at the firm. One can argue that the presence of Fiberco products at one point in time provides
evidentiary support for an inference that they were there on other occasions as well. However, the
manager appears to negate that inference with the admission that he had never seen the Fiberco products
in the storeroom until after Wife’s employment had ended. Claimant might have attempted to bolster his
case with other evidence of Fiberco exposure, either through company records or other testimony. In
failing to do so, Claimant appears to have failed to provide evidence sufficient to support a rational jury
verdict in his favor on the issue of Fiberco exposure.

Seperac-J19 Exam-Released MEE Essay Compilation © 2016-2020 165


#036-FEB 2000–MEE Q03: QUESTION THREE (CIVIL PROCEDURE)

Adam is a citizen of State A and practices law in State A. He represented a citizen of State A in a criminal
prosecution involving drugs and in a related civil forfeiture proceeding, both of which were pending in
federal district court in State A. With his client’s consent, Adam telephoned Barbara, a citizen of State B
practicing law in State B, and reached an agreement that Barbara would become co-counsel in the civil
forfeiture case. The client had agreed to pay Adam a contingent fee equal to 50% of the value of any
property prevented from being forfeited. Barbara and Adam agreed that Adam would keep the first one-
third of any fee resulting from the representation in the civil forfeiture action, and that they would share in
the remaining two-thirds in proportion to their respective time spent on the forfeiture matter.

During the next two months, Barbara and Adam talked by telephone on two occasions and exchanged two
letters in connection with the civil forfeiture matter. Adam’s work on the case was performed entirely in
State A, and Barbara’s work was performed entirely in State B.

At the end of the two-month period, the government entered into an agreement accepting a guilty plea in
the drug action. It also settled the forfeiture claim, agreeing to allow Adam’s client to retain property
valued at $480,000.

Adam refused to pay Barbara any portion of the $240,000 received as Adam’s contingent fee. Barbara
subsequently brought a breach of contract action against Adam in federal district court in State B,
claiming $80,000 as her share of the contingent fee. Adam filed a motion to dismiss the action based on
lack of jurisdiction over the subject matter or the person. Alternatively, Adam requested a transfer of
venue to the federal district court of State A, where he resides and has his place of business. Adam’s
supporting affidavit stated that the settlement was in no way related to Barbara’s efforts, and that
Barbara’s time spent on the forfeiture matter was less than one-third of the time spent by Adam.

A statute of State B provides:

A court may exercise personal jurisdiction over a person, who acts directly or by an agent, as to a cause of
action arising from the person’s transacting any business in this state.

How should the court rule on each issue raised by Adam’s motion to dismiss and request for transfer of
venue? Explain.

Seperac-J19 Exam-Released MEE Essay Compilation © 2016-2020 166


#036: F00-3 MEE: ANSWER: NCBE (CIVIL PROCEDURE)

POINT (1) [16%] ISSUE: Does the federal district court in State B have subject matter
jurisdiction? ANSWER: Yes. The federal district court has subject matter jurisdiction based upon
diversity of citizenship and the required amount in controversy.

POINT (2) [53%] ISSUE: Does the federal district court in State B have personal jurisdiction?
ANSWER: Yes. The federal district court has personal jurisdiction based upon requisite minimum
contacts and the state’s long-arm statute.

POINT (3) [32%] ISSUE: Should venue be transferred to the federal district court of State A?
ANSWER: No. Venue should not be transferred to the federal district court of State A because the
convenience of the parties and the convenient administration of justice would not be better served
there than in the federal district court of State B.

ANSWER EXPLANATION:

Explanation to Point-One (10-20%):

The federal district court has subject matter jurisdiction based upon diversity of citizenship and the
required amount in controversy.

The challenge to jurisdiction raises issues as to personal jurisdiction as well as subject matter jurisdiction.
Subject matter jurisdiction exists under 28 U.S.C. § 1332 if there is diversity of citizenship and the
amount in controversy exceeds $75,000. Diversity of citizenship exists between Barbara, a citizen of State
B, and Adam, a State A citizen.

Barbara’s claim for $80,000 on its face satisfies the statutory amount in controversy. Adam’s supporting
affidavit suggests the possibility that Barbara may ultimately recover less than the statutory minimum, but
that does not defeat the federal court’s jurisdiction. “The sum claimed by the plaintiff controls if the claim
is apparently made in good faith. It must appear to a legal certainty that the claim is really for less than the
jurisdictional amount to justify dismissal.” Barbara’s actual recovery will depend upon factual issues and
cannot be said to a legal certainty to be less than the statutory minimum.

Explanation to Point-Two (45-55%):

The federal district court has personal jurisdiction based upon requisite minimum contacts and the state’s
long-arm statute.

Determining the existence of personal jurisdiction requires analysis both of whether personal jurisdiction
can be constitutionally exercised by the State B federal district court and of whether the appropriate state
statute allows the federal court to assert personal jurisdiction.

The constitutional question is dependent on the classic line of cases beginning with International Shoe
requires the defendant to have sufficient “minimum contacts” with State B such that requiring defense in
State B would not “offend ‘traditional notions of fair play and substantial justice.’”

Seperac-J19 Exam-Released MEE Essay Compilation © 2016-2020 167


The sufficiency of contacts required under International Shoe and subsequent decisions depends on
whether the court is seeking to assert general or specific jurisdiction. General jurisdiction is involved
where the subject matter of the lawsuit is not related to the contacts the defendant has with the forum
state. Specific jurisdiction is involved when the lawsuit arises out of the contacts which form the basis for
determining whether there are minimum contacts. Fewer contacts are required to assert specific personal
jurisdiction than to assert general personal jurisdiction:

Where a forum seeks to assert specific jurisdiction over an out-of-state defendant who has not consented
to suit there, this “fair warning” requirement is satisfied if the defendant has “purposely directed” his
activities at residents of the forum, and the litigation results from alleged injuries that “arise out of or
relate to” those activities.

Were this a general jurisdiction case – for example, if Barbara were suing Adam on an unrelated personal
injury claim – the contacts would clearly be insufficient.

Here, however, Barbara seeks to assert specific jurisdiction, since the cause of action arises out of Adam’s
contacts (the fee-splitting contract) with State B. The contacts appear sufficient to assert personal
jurisdiction. Purposeful activity was directed toward State B. Although there are factual points to the
contrary, including where the contract was written, the contacts with State B are sufficient for an action
arising out of such contacts. Adam initiated the contact, solicited the contract, and had further related
contacts with Barbara.

Recognizing that personal jurisdiction is constitutionally permissible, the question remains whether there
is statutory authority to assert that jurisdiction. Under Federal Rule of Civil Procedure 4, Barbara must
look to the law of State B to determine whether personal jurisdiction may be asserted in a diversity action.
This requires interpreting State B’s long-arm statute. The statute clearly contemplates the exercise of
jurisdiction in an action such as this because contracting with Barbara constitutes transacting business
within the state.

Explanation to Point-Three (25-35%):

Venue should not be transferred to the federal district court of State A because the convenience of the
parties and the convenient administration of justice would not be better served there than in the federal
district court of State B.

Venue transfers are controlled by 28 U.S.C. § 1404, which provides “in the interest of justice, a district
court may transfer any civil action to any other district or division where it might have been brought.” The
action could have been brought in State A. The subject matter jurisdiction is the same. Adam is doing
business within the state sufficient to confer personal and subject matter jurisdiction, and venue is proper
since all the defendants are in State A. The question is whether the interests of justice require the transfer.
It would appear not. The relative difficulty to Barbara and Adam seems the same. The evidence necessary
at trial is not predominately in one jurisdiction rather than the other. There appears to be no hardship on
any third party who might be a witness. The logical conclusion would be that the preference for allowing
Barbara to choose the forum has not been overcome.

Seperac-J19 Exam-Released MEE Essay Compilation © 2016-2020 168


#037-JUL 1999–MEE Q03: QUESTION THREE (CIVIL PROCEDURE/CONFLICTS)

A few years ago, the legislature of State A passed the following statute:

The owners of lots fronting on public sidewalk areas between the property line of the lot and the street
line shall repair and maintain those areas at their own expense; failure to do so will result in an assessment
against the property for the city’s cost in making those repairs.

In the few cases that have arisen under this statute, the trial judges of State A have taken diametrically
opposed positions as to its impact on tort cases arising from dangerous conditions in sidewalk areas. Some
State A courts have concluded that the statute changes the common law by shifting from the city to the
abutting landowner all liability in tort for any injury to a pedestrian caused by a dangerous condition
anywhere in the sidewalk area. Other State A trial judges have interpreted the statute merely to obligate
abutting landowners either to repair any dangerous condition themselves or to reimburse the city for the
cost of doing so, but not to impose on them tort liability for personal injuries. The appellate courts of
other states with similar sidewalk-repair statutes also are divided as to its effect. The Supreme Court of
State A has yet to rule on a case involving the scope of the statute.

Defendant has long owned and occupied a bungalow on Main Street in the State A city of Suburbia. Main
Street is a paved residential street with curbs running the full length of each side. Between Defendant’s
property line and the street is a “sidewalk area” consisting of a sidewalk, a 10-foot-wide grassy area
(called a “parking strip”), and the curbing for the street. This entire area is a “public sidewalk area” within
the meaning of the statute quoted above. Suburbia holds title to the street, curbs, parking strip, and
sidewalks on Main Street.

Late one winter evening, Plaintiff, at all times a resident and citizen of State B, traveled by taxi to attend a
gathering at the home of Defendant’s next-door neighbor, Todd. Because two autos were already parked
in front of Todd’s residence, the taxi driver let Plaintiff out at the curb in front of Defendant’s house.
After taking a half-dozen steps in the direction of Todd’s house, Plaintiff stepped in a leaf-covered six-
inch hole in the parking strip area, which caused her to fall and fracture her hip.

Plaintiff sued Defendant for $250,000 in a federal court in State A. The complaint alleged that the hole
was located in a part of the parking strip that abutted Defendant’s property. At the week-long jury trial of
the case, the testimony was in sharp conflict as to the location of the hole. Some witnesses asserted that it
was in front of Defendant’s property; others insisted that it was actually in front of Todd’s property.
Except for this conflict in the testimony, Defendant did not contest Plaintiffs proof of the elements of her
case. At the conclusion of all the evidence, Defendant moved for judgment as a matter of law. In support
of this motion, Defendant argued that, even if the hole was in an area abutting his lot, the sidewalk-repair
statute did not impose upon owners any tort liability for injuries to others from dangerous conditions in
the publicly owned parking strip in front of their lots.

The federal district court has taken Defendant’s motion under consideration. The court believes that the
sounder interpretation of such a “sidewalk-repair” statute would be that it does impose tort liability on
abutting owners. However, it also has concluded, from a study of decisions by the State A Supreme Court
construing other statutes, that when the issue of the scope of the sidewalk repair statute eventually reaches
that court, it will probably decide that it does not impose tort liability on abutting owners.

Seperac-J19 Exam-Released MEE Essay Compilation © 2016-2020 169


1. Is Defendant entitled to judgment as a matter of law? Explain.

2. Assuming Defendant is entitled to judgment as a matter of law, should the judge defer ruling for
Defendant until after the jury renders its verdict? Explain.

Seperac-J19 Exam-Released MEE Essay Compilation © 2016-2020 170


#037: J99-3 MEE: ANSWER: NCBE (CIVIL PROCEDURE/CONFLICTS)

POINT (1) [55%] ISSUE: In ruling on Defendant’s motion for judgment as a matter of law, should
the federal district court adopt the interpretation it prefers of the State A sidewalk-repair statute or
apply what it believes is the likely interpretation by the State A Supreme Court? ANSWER: Yes.
The federal district court should adopt the interpretation of the statute that it believes would be
adopted by the State A Supreme Court. On that interpretation, Defendant is entitled to judgment as
a matter of law.

POINT (2) [45%] ISSUE: If the trial court concludes that the defendant is entitled to judgment as a
matter of. law, should the trial court defer its decision until after the jury renders a verdict?
ANSWER: Yes. Having concluded that Defendant is entitled to judgment as a matter of law, the
court should nevertheless defer ruling on the motion until after the jury has rendered a verdict in
order to avoid the necessity for a retrial in the event the judge’s legal ruling is overturned on
appeal.

ANSWER EXPLANATION:

Explanation to Point-One (50-60%):

The federal district court should adopt the interpretation of the statute that it believes would be adopted by
the State A Supreme Court. On that interpretation, Defendant is entitled to judgment as a matter of law.

Under the Federal Rules of Civil Procedure, judgment as a matter of law (JMOL) is appropriate when a
“party has been fully heard” and “there is no legally sufficient evidentiary basis for a reasonable jury to
find for that party” Thus, the question is whether, under the applicable law, the jury could find any facts
that would entitle plaintiff to recover from defendant. If not, then defendant’s motion for judgment as a
matter of law may be granted.

This case is a diversity case. When a federal court is hearing a case under its diversity jurisdiction, it must
apply the same substantive law that would have been applied if the suit had been filed in a court of the
state where the federal court is located. The federal court is not free to make its own independent
judgment, from a policy standpoint, of the better construction of a statute worded like State A’s sidewalk-
repair law. Instead, the federal court must try to predict from available State A case law how the State A
Supreme Court would be likely to interpret the statute. “The federal court must keep in mind that its
function is not to choose the rule that it would adopt for itself, if free to do so, but to choose the rule that it
believes the state court, from all that is known about its methods of reaching decisions, is likely in the
future to adopt.” Once the federal court makes this prediction, it must apply that construction of the
statute, whether it agrees with it or not.

In this case, the federal district court has concluded, albeit reluctantly, that the State A Supreme Court
probably would rule that its sidewalk-repair statute was not intended to impose tort liability on abutting
landowners. Therefore, even if the hole was in fact located in an area of the parking strip that abutted
Defendant’s property, he is not liable for Plaintiffs injury and is entitled to prevail in this case as a matter
of law.

Explanation to Point-Two (40-50%):

Seperac-J19 Exam-Released MEE Essay Compilation © 2016-2020 171


Having concluded that Defendant is entitled to judgment as a matter of law, the court should nevertheless
defer ruling on the motion until after the jury has rendered a verdict in order to avoid the necessity for a
retrial in the event the judge’s legal ruling is overturned on appeal.

In view of the answer to the first question, it may seem that the answer to the second one is a foregone
conclusion: the trial court should now grant Defendant’s motion for a directed verdict. In fact, however,
the much preferred course is for the trial court to reserve its ruling on this motion and submit the case to
the jury to resolve the disputed factual question concerning the location of the hole. Rule 50 permits, but
does not require, a trial judge to grant judgment as a matter of law during the trial. Sometimes there are
sound reasons for the judge to reserve decision on such a motion until after a jury verdict.

If the jury concludes that the hole was actually located in front of the adjoining Todd house, the legal
issue raised by Defendant’s motion becomes moot. Without deciding the meaning of the State A statute
one way or the other, the court could then simply enter judgment for Defendant on the basis of the jury’s
verdict in his favor. This result would be preferable to a ruling based on a guess about the statute’s
meaning. On the other hand, if the jury resolves the factual issue concerning the hole’s location in favor of
Plaintiff, the defense simply renews post-trial its motion for judgment as a matter of law. The judge can
then grant the motion. The advantage of this “reserved decision” procedure comes into play if Plaintiff
appeals from the judgment entered for Defendant. If the trial judge’s decision as to the meaning of the
State A statute is overturned, the appellate court can reverse the judgment entered for Defendant and
direct the trial court to enter judgment for Plaintiff on the basis of the jury verdict in her favor.

If the court were to grant the defense motion for judgment made at the conclusion of the evidence and the
plaintiff then succeeded in reversing the trial court, the appellate court would have to remand the case for
a new trial. The time and effort of the trial court, the parties, and the jury at the first trial would have been
unnecessarily wasted.

Seperac-J19 Exam-Released MEE Essay Compilation © 2016-2020 172


#038-FEB 1999–MEE Q04: QUESTION FOUR (CIVIL PROCEDURE)

Laptop, Inc., a State X corporation, manufactures laptop computers at its principal place of business in
State Y. Paul, a resident of State Z, purchased one of Laptop’s computers while attending college in State
Y. During summer break, Paul took his computer back home with him to State Z, where he has his
permanent residence. Shortly after Paul returned home, the computer caught fire while Paul was using it
to access Laptop’s website. Paul was severely burned in the fire.

The fire was caused by a defective memory chip installed by Laptop during manufacture in State Y. The
memory chip was manufactured by Chip, Inc., a company incorporated overseas in Country A.

Laptop sells 50% of its computers in State Y, 25% in State X, and 25% in State Z. It regularly advertises
in the State Z market through computer magazines with national circulations. Laptop also maintains a
website in State Y where customers can contact it to make purchases and obtain technical help when
problems are encountered in the use of its products. The website is accessible to anyone with Internet
access anywhere in the world.

Chip sends to Laptop in State Y all of the memory chips it exports to the United States. Laptop installs
them in all the computers it manufactures. Chip has sold to Laptop $5 million worth of memory chips per
year for the last five years. Chip’s income from memory chips sold to Laptop and installed in computers
sold in State Z is less than 2% of its total annual income. Although Chip has never advertised in States X,
Y, or Z and does not design its products specifically for those markets, Chip knew that its computer chips
would be installed in computers sold in States X, Y, and Z.

Paul filed a lawsuit in federal district court in State Z against Laptop and Chip for personal injury
damages of $250,000 basing subject matter jurisdiction on diversity of citizenship. Paul alleged both
negligence and strict product liability claims against both defendants. Laptop was served with process in
State Y, and Chip was served in Country A. The State Z long-arm statute authorizes personal jurisdiction
over nonresident defendants “on any basis not inconsistent with the United States Constitution.”

Both defendants move to dismiss Paul’s claims for lack of personal jurisdiction. Alternatively, they move
to dismiss and/or transfer venue to the U.S. District Court in State Y, arguing that venue in the district
court in State Z is improper.

1. Should the court grant Laptop’s motion to dismiss Paul’s claim for lack of personal jurisdiction?
Explain.

2. Should the court grant Chip’s motion to dismiss Paul’s claim for lack of personal jurisdiction?
Explain.

3. Should the court grant both defendants’ motions to dismiss and/or transfer venue to the district
court in State Y? Explain.

Seperac-J19 Exam-Released MEE Essay Compilation © 2016-2020 173


#038: F99-4 MEE: ANSWER: NCBE (CIVIL PROCEDURE)

POINT (1) [40%] ISSUE: Can the State Z district court exercise personal jurisdiction over Laptop,
when the defective computer that caused Paul’s injuries was purchased by Paul in State Y and
transported by Paul into State Z? ANSWER: Yes. The district court in State Z can assert personal
jurisdiction over Laptop because Laptop’s activities in the state are extensive enough to warrant
the exercise of jurisdiction, even over a cause of action unrelated to those activities.

POINT (2) [20%] ISSUE: Can the State Z district court exercise personal jurisdiction over Chip,
when Chip’s only connection with State Z is that its computer chips enter that state as part of
computers sold there by Laptop? ANSWER: No. In all probability, Chip should not be subject to
State Z jurisdiction because the defective chip reached State Z through Paul’s unilateral act and
Chip’s connections with State Z are not otherwise substantial enough for it reasonably to expect to
be haled into court there.

POINT (3) [40%] ISSUE: Is venue proper in State Z? ANSWER: Yes. Venue is clearly proper in
State Z. but the action should be transferred to State Y if the district court in State Z is found to
lack jurisdiction over Chip.

ANSWER EXPLANATION:

Explanation to Point-One (35-45%):

The district court in State Z can assert personal jurisdiction over Laptop because Laptop’s activities in the
state are extensive enough to warrant the exercise of jurisdiction, even over a cause of action unrelated to
those activities.

State Z may exercise in personam jurisdiction over Laptop so long as the defendant has “certain minimum
contacts with the forum such that the maintenance of the suit does not offend ‘traditional notions of fair
play and substantial justice.’” The “defendant’s conduct and connection with the forum state” must be
“such that he should reasonably anticipate being haled into court there.” A defendant corporation satisfies
this requirement if it “purposefully avails itself of the privilege of conducting activities within the forum
State.”

Laptop’s connection to State Z easily satisfies the minimum contacts test. The combination of Laptop’s
substantial sales in State Z, its advertising in State Z and its provision of a website (by which customers in
State Z can obtain advice in the use of its products) all show a clear intent to serve the State Z market. As
Laptop’s conduct includes actions “purposefully directed toward the forum State,” those actions should
satisfy the minimum contacts requirement.

The only argument against jurisdiction might be that the particular computer involved in this case was not
sold by Laptop in State Z, but was purchased by Paul in State Y and carried by him to State Z. In
WorldWide Volkswagen Corp., the court held that the unilateral act of a customer in bringing a product
into a state could not be regarded as creating minimum contacts between the state and the defendant, even
if the product’s presence in the state was foreseeable to defendant. That holding, however, has little
relevance to this situation. Here, Laptop’s connection to State Z is not merely a result of Paul’s unilateral
act of bringing the computer to State Z. Laptop has made its own intentional decision to serve the State Z
market. Moreover, its contacts with State Z are “continuous and systematic general business contacts,”

Seperac-J19 Exam-Released MEE Essay Compilation © 2016-2020 174


25% of its sales are there; it advertises there; and it provides technical support to customers who are
located there. This is enough for State Z to assert jurisdiction over Laptop, even as to a cause of action
that is not directly connected to those business contacts.

Explanation to Point-Two (15-25%):

In all probability, Chip should not be subject to State Z jurisdiction because the defective chip reached
State Z through Paul’s unilateral act and Chip’s connections with State Z are not otherwise substantial
enough for it reasonably to expect to be haled into court there.

Chip put its goods into the stream of commerce in the U.S. by selling them to Laptop. It knew its chips
could end up in computers that would be sold in State Z. It also earned revenue from the sale of chips that
were destined to be a part of products sold in State Z. However, the cause of action in this case, although
it occurred in State Z, arose out of an alleged defect in a chip that was sold in another state and brought
into State Z by the purchaser.

On these unique facts, jurisdiction is probably not proper over Chip. First, jurisdiction over Chip would
have to be based solely on a “stream of commerce” theory: by placing its goods in the “stream of
commerce” with knowledge that they would end up in State Z, Chip avails itself of the privilege of doing
business in State Z and subjects itself to suit there. The problem with this theory is that it may no longer
be valid and, even if it is, the injuries in this case were not caused by a chip that entered State Z through
the stream of commerce.

The “stream of commerce” theory of jurisdiction was last endorsed by the Supreme Court in dicta in the
WorldWide Volkswagen case, decided in 1980. In 1987, the Court split 4-4 on the validity of the stream-
of-commerce theory, with four members of the Court writing that placing goods into the stream of
commerce was not a sufficient basis to exercise jurisdiction over a nonresident defendant, absent evidence
of an explicit intention by the defendant to serve the target market. Since that time, the federal courts have
been divided on the viability of the stream-of-commerce test.

In our case, the argument for jurisdiction on a stream-of-commerce rationale is especially weak. Although
Chip sells some chips knowing that they will reach State Z through the stream of commerce, the particular
chip that caused injury did not reach State Z via the stream of commerce. The allegedly defective chip
entered State Z because of Paul’s, unilateral act of bringing his laptop computer into the state. Moreover,
Chip did nothing that indicated any special intention to serve the State Z market. Finally, Chip’s revenue
from State Z sales is relatively small (less than 2% of its annual revenue). Even if the stream-of-
commerce theory is a viable basis for jurisdiction over Chip, that jurisdiction probably would not extend
to acts unrelated to its in-state sales. The fact that Chip’s products enter State Z on a small scale through
the stream of commerce would not seem to be the “continuous and systematic general business contacts”
required for exercise of jurisdiction in connection with claims unrelated to those contacts.

However, a case for jurisdiction can be supported by emphasizing that it was foreseeable to Chip that its
products might cause injury in State Z. given its knowledge that they were incorporated into computers
sold there.

A second reason for not asserting jurisdiction over Chip is the burden that Chip would face in defending
in State Z. In the leading case involving jurisdiction over a foreign component manufacturer, the Supreme
Court noted that “the unique burdens placed upon one who must defend oneself in a foreign legal system
should have significant weight in assessing the reasonableness” of an exercise of jurisdiction. Even if an
assertion of jurisdiction over Chip were otherwise appropriate, it might be unreasonable in light of the

Seperac-J19 Exam-Released MEE Essay Compilation © 2016-2020 175


burden Chip would face in defending itself, especially in light of the small amount of revenue it actually
makes from sales of its product in State Z.

On the other hand, Paul and State Z have a strong interest in having the case go forward in State Z. A
State Z action would be convenient for Paul and would allow State Z courts to vindicate whatever interest
the state may have in Paul’s recovery. These interests, which were not present in Asahi, offset the burden
on Chip and provide strong reasons for hearing the action in State Z. If there were an otherwise sound
basis for jurisdiction over Chip, it would probably be reasonable to assert it, despite the burdens that it
would impose on Chip.

In sum, it would appear that the court should grant Chip’s motion to dismiss because jurisdiction is
inappropriate where (1) Chip’s connections with State Z are slight, (2) Chip takes no actions itself that are
directed at serving the State Z market, (3) the injury in the case was caused by a chip which entered State
Z through the unilateral actions of the consumer, and (4) the burden on Chip of defending the action
would be severe. However, a good argument could be made in favor of jurisdiction, stressing the
foreseeability to Chip of an injury in State Z and the reasonableness of State Z jurisdiction in light of the
interests of the state and the plaintiff.

Explanation to Point-Three (35-45%):

Venue is clearly proper in State Z. but the action should be transferred to State Y if the district court in
State Z is found to lack jurisdiction over Chip.

Under 28 United States Code § 1391(a), venue in a diversity case is proper in “(1) a judicial district where
any defendant resides, if all defendants reside in the same, state, (2) a judicial district in which a
substantial part of the events or omissions giving rise to the claim occurred,.” or “(3) a judicial district in
which the defendants are subject to personal jurisdiction.” Where corporate defendants are involved, they
are deemed to reside “in any judicial district in which they are subject to personal jurisdiction.” If State Z
has jurisdiction over the defendants, it would clearly have venue over the action under any of these
provisions. It would be a “district in which the defendants are subject to personal jurisdiction,” it would
be (by definition) a district in which the defendants “reside,” and “a substantial part of the events giving
rise to the claim” occurred there – the computer fire and Paul’s injuries.

If State Z has jurisdiction, the court is unlikely to grant a motion to transfer venue. Venue transfer is
permitted, “in the interest of justice and for the convenience of the parties,” to any district in which the
action could have been brought originally. There is no reason to believe that any outlier court would be
significantly superior to State Z as a venue for the action. State Y might be a somewhat more appropriate
venue for a product liability action, as it is the state of manufacture. But pertinent evidence (e.g., relating
to Paul’s injuries) is likely to be present in State Z; a State Z trial will be convenient for Paul (as a State Z
resident); and courts generally give deference to plaintiffs choice of forum where the choice is not
unreasonable and convenience factors do not weigh strongly toward another forum.

If State Z does NOT have jurisdiction, however, the court is very likely to grant a motion to transfer
venue to State Y. Both Laptop and Chip would almost certainly be amenable to suit in State Y (Laptop
manufactures its laptops there and Chip ships its parts directly to State Y), State Y is a proper venue for
the action because “a substantial part of the events or omissions giving rise to the claim occurred” there
(manufacture of the computer), and a venue transfer to State Y would serve “the interests of justice” better
than an outright dismissal of the case. Under 28 U.S.C. § 1631, a court may transfer an action to another
court to cure a “want of jurisdiction” if it is “in the interest of justice” to do so.

Seperac-J19 Exam-Released MEE Essay Compilation © 2016-2020 176


#039-JUL 1998–MEE Q03: QUESTION THREE (CIVIL PROCEDURE/CONFLICTS)

Husband and Wife, residents of State Y, are owners in joint tenancy of a large tract of land in State Z,
which they purchased as an investment several years ago. They recently made an oral agreement to sell
the land for $1 million to Developer, a shopping center developer who lives in State Z. The price seemed
low to Husband, but he agreed to the sale because Wife told him she was anxious to raise some cash so
they could buy a vacation home.

All negotiations concerning the transaction were held at Husband and Wife’s home in State Y. Final
agreement was reached over dinner after a day of discussions. No written memorandum was made of the
oral agreement, but Developer immediately made a $150,000 down payment on the agreed purchase
price.

Shortly after the oral agreement was reached, Wife separated from Husband, but she remained domiciled
in State Y. Husband later learned from friends that Wife had secretly intended to leave him as soon as the
land was sold, that she had privately approached Developer about selling the land before Developer ever
contacted Husband and Wife formally, and that Wife had received a separate $250,000 “finder’s fee”
from Developer for persuading Husband to agree to the sale of the land.

Husband has filed a diversity action against Developer in the federal district court of State Z. The
complaint recites the preceding facts and seeks a declaration that the oral agreement for the sale of the
land is unenforceable under the State Z statute of frauds.

Under State Z’s statute of frauds, oral agreements for the sale of land are never enforceable. Under State
Y law, however, such agreements are enforceable when the purchaser has made a down payment.

In lieu of an answer to the complaint, Developer has filed a Rule 12(b) motion. The motion seeks relief
from the complaint on two grounds. First, Developer asks for dismissal of the complaint because of
Husband’s failure to join Wife in the action. Alternatively, Developer seeks dismissal on the ground that
the complaint fails to state a claim on which relief can be granted because the oral contract is enforceable
under the law of State Y, where the agreement was made.

1. What ruling should the court make with respect to the motion to dismiss for failure to join Wife
as a party in the action? Explain.

2. If the case were not dismissed because of Husband’s failure to join Wife, how should the court
rule on the motion to dismiss the complaint for failure to state a claim? Explain.

Seperac-J19 Exam-Released MEE Essay Compilation © 2016-2020 177


#039: J98-3 MEE: ANSWER: NCBE (CIVIL PROCEDURE/CONFLICTS)

POINT (1) [50%] ISSUE: Under Fed. R. Civ. P. 19, is Wife a necessary and indispensable party
who cannot be joined without destroying the court’s diversity jurisdiction? ANSWER: Yes. As a
co-owner of the land and a joint obligor on the contract. Wife is a necessary and indispensable
party to any action that potentially affects title to the land or validity of the contract. Since her
joinder as a defendant would destroy diversity, the court should dismiss the action.

POINT (2) [50%] ISSUE: Which state’s law will be applied to determine whether the oral
agreement for the sale of land is enforceable? ANSWER: State Z’s law should be applied to
determine the enforceability of a contract for conveyance of land located in State Z. Because the
contract is unenforceable under that law, the court should deny Developer’s motion to dismiss the
complaint for failure to state a claim.

ANSWER EXPLANATION:

Explanation to Point-One (45-55%):

As a co-owner of the land and a joint obligor on the contract. Wife is a necessary and indispensable party
to any action that potentially affects title to the land or validity of the contract. Since her joinder as a
defendant would destroy diversity, the court should dismiss the action.

A motion to dismiss an action for “failure to join a party” raises three issues. First, is joinder of the absent
party proper under Rule 19(a) of the Federal Rules of Civil Procedure? Second, if joinder is proper, is it
possible to join the party? Finally, if it is not possible to join the absent party, should the court proceed in
the absence of the party or dismiss the action? In other words, is the absent party an “indispensable” party
– a party in whose absence the action cannot fairly proceed?

1. Joinder is proper.

A federal court should join a person as a party if (1) in the person’s absence complete relief cannot be
accorded among those already parties, or (2) the person claims an interest relating to the subject of the
action and is so situated that the disposition of the action in the person’s absence may (i) as a practical
matter, impair or impede the person’s ability to protect that interest or (ii) leave any of the persons already
parties subject to a substantial risk of incurring double, multiple, or otherwise inconsistent obligations by
reason of the claimed interest.

Wife is a “necessary party” under this rule. Her substantial interest in the contract for the sale of the real
estate will invariably be affected if the court declares the contract unenforceable. Similarly, her interest in
the land is implicated by the action. If Developer establishes that the contract is enforceable and valid,
Developer effectively proves his equitable ownership of the land, and Wife’s land interest will have been
transferred. If Developer loses, then Wife’s effort to transfer her interest in the land to Developer under
the contract will be effectively thwarted. Moreover, Developer faces potentially inconsistent liabilities if
Husband wins this case and Wife later sues to recover a portion of the purchase price.

2. Joinder is not possible.

Seperac-J19 Exam-Released MEE Essay Compilation © 2016-2020 178


When the court joins Wife, it must determine her ultimate interest in the action and align her accordingly.
Wife, of course, shares with Husband the position of owner of the land and obligor under the sales
contract. However, her true position in the action is adverse to Husband’s. She has a strong interest in
seeing the contract enforced (to protect her “finder’s fee”) and she was a party to the alleged fraud against
Husband.

In these circumstances, Wife is properly considered a defendant and her joinder would destroy diversity
jurisdiction because she has the same state citizenship as Husband, the plaintiff. For example, in one case,
the court found that a wife, as co-owner of real estate with her husband, should be joined as a defendant in
action alleging that the wife agreed with others to defraud the husband in the sale of land. Accordingly,
the court cannot join Wife and must therefore decide whether to proceed without her or dismiss the action.

3. The action should be dismissed.

When a party with an interest in the action under Rule 19(b) cannot be joined without destroying the
court’s jurisdiction, the court must determine “whether in equity and good conscience the action should
proceed among the parties before it, or should be dismissed, the absent person being thus regarded as
indispensable.” In making this decision, the court must consider, among other factors, (a) the extent to
which a judgment rendered without the party might prejudice the absentee or the existing parties; (b)
whether the prejudice can be lessened or avoided by appropriately shaping the relief granted; (c) whether
adequate relief can be granted without the absentee; and (d) whether the plaintiff has an adequate remedy
if the action is dismissed for nonjoinder.

These factors compel dismissal in a case of this nature. Wife has interests in both the contract and the land
that will be affected if the contract is rescinded. There would not appear to be any relief that the court
could grant that would be both responsive to Husband’s complaint and protective of Wife’s interest.
Moreover, if Developer wins the action, Developer cannot receive complete relief if Wife is not joined. It
is always possible that Developer might face a later action by Wife to defeat the contract if Wife at some
point changed her mind about the contract’s desirability.

Husband, on the other hand, has an alternative forum in which to try this action: Husband can pursue his
case in state court in State Z. Jurisdiction over Developer can easily be established (he resides in State Z)
and Wife’s ownership of the land in State Z provides an adequate basis for the state courts to exercise
jurisdiction over her in an action involving title to the land and a contract for sale of the land. A suit in
state court would thus provide Husband with an adequate remedy.

In cases of this sort, courts almost invariably dismiss the action rather than proceed in the absence of a
necessary party. Numerous cases hold that “an action seeking rescission of a contract must be dismissed
unless all parties to the contract, and others having a substantial interest in it, can be joined.” Furthermore,
when ownership of land is involved in an action, persons with interest in the land are almost always
considered “indispensable parties” in part because “real property disputes are particularly suited to
resolution by state courts.”

Explanation to Point-Two (45-55%):

State Z’s law should be applied to determine the enforceability of a contract for conveyance of land
located in State Z. Because the contract is unenforceable under that law, the court should deny
Developer’s motion to dismiss the complaint for failure to state a claim.

Seperac-J19 Exam-Released MEE Essay Compilation © 2016-2020 179


Developer’s motion to dismiss the complaint for failure to state a claim raises a choice of law issue. To
determine the applicable substantive law, the federal district court must apply the choice-of-law rules of
State Z (where it is hearing the action). The problem does not state what State Z’s choice of law rules are,
so examinees should focus on providing some sensible analysis of this issue.

The traditional choice of law rule is that all issues affecting title to land are determined by the law of the
situs of the land. Thus, in a case of this nature (a contract is negotiated and made in State Y for the sale of
land in State Z), the law of State Z would determine the enforceability of that contract. Occasionally, a
court will characterize a case of this sort as a contract case and apply the law of the place where the
contract was made. But, generally, situs law governs the validity of contracts to convey land.

Modern approaches to choice of law, however, would permit departure from this traditional rule if a state
other than the situs state had a “more significant relationship” to the facts of the case or a greater interest
in the outcome.” It is possible to argue on these facts, using either a “most significant relationship”
analysis or an interest analysis, that State Y law should apply. State Y is the state where the contract at
issue was negotiated and where the parties entered the contract. Both Husband and Wife are citizens of
State Y and Developer journeyed there to negotiate the contract. Developer also made a substantial down
payment on the contract in State Y. State Y’s law reflects a policy of protecting the expectations of land
purchasers when they have made a down payment on the purchase price, even though the contract is not
written. Arguably State Y has an interest in upholding the contract and protecting Developer, even though
Developer is a State Z resident, because of its general interest in securing the integrity of transactions that
occur within its borders. The policy of upholding the expectations of the parties would also generally
point to an application of the law that would validate the contract. Indeed, were this a contract
unconnected to land, its validity probably would be determined by the law of State Y given that it was
negotiated and made there, Husband and Wife live there, and the contract would be upheld as valid under
that state’s law. State Z arguably has little interest in applying its statute of frauds to protect two non-
residents (Husband and Wife) whose own home jurisdiction does not protect them.

Although attractive, the above analysis is not likely to be adopted by most courts. Examinees should
receive credit for arguments of that type, but it is likely that most courts would adhere to the traditional
rule that State Z’s law determines the validity of contracts to convey land in State Z. The simple
explanation for this is that State Z always has a strong, perhaps dominant, interest in any action that will
affect title to its land. State Z is the only state that can, in the end, award title to anyone. Problems
concerning title must ultimately be resolved in State Z, and State Z has a stronger interest than any other
state in promulgating rules that govern questions concerning how and when and with what degree of
formality legal actions affecting title should be taken. If State Z says that contracts to convey land must be
written to be enforceable, that is not merely a policy statement affecting the private interests of the parties
to that contract; it is a policy about the proper formalities to be used when actions are taken that affect title
to land in State Z. It affects the rights and interests of the state and other parties in the event that title is
ever put in question. In short, State Z is likely to have the most significant relationship to this issue,
despite the strong connections with State Y.

Thus, unless State Z has a special choice of law rule that points to the law of the state of contracting
(which is unlikely), the traditional rule would probably be followed by the court and State Z’s law would
be used to determine the validity of the contract. Under State Z’s law, the contract is unenforceable. The
motion to dismiss the complaint therefore should be denied.

Seperac-J19 Exam-Released MEE Essay Compilation © 2016-2020 180


#040-FEB 1998–MEE Q04: QUESTION FOUR (CIVIL PROCEDURE)

Passenger was riding as a passenger on a motorcycle operated by Cyclist in State A. She sustained a
severe head injury when a westbound car driven by Driver collided with Cyclist’s northbound motorcycle
at an intersection controlled by stop signs for both roads. The accident left Passenger with permanent
amnesia concerning the accident and the events leading to the collision.

Passenger, a citizen of State B, sued Driver, a citizen of State A, in federal district court in State A for $ 1
million in damages for personal injuries. The federal court properly has diversity jurisdiction.

Passenger’s complaint alleged that Driver failed to stop at the stop sign and that Driver’s negligent
driving was the proximate cause of her injuries. Driver’s answer denied that he failed to stop and that
there was any negligence on his part. In response to Driver’s interrogatories, Passenger stated that her
claim of negligence was based on Driver’s failure to bring his automobile to a stop before proceeding
through the intersection, despite the presence of a plainly visible stop sign regulating westbound traffic.
She also stated, on information and belief, that Cyclist’s motorcycle had reached the intersection first and
was proceeding through it only after coming to a complete stop.

Before either side could take his deposition, Cyclist died of causes not connected with the accident. The
only other eyewitness to the collision was Witness, who was traveling in a westbound truck immediately
behind Driver’s auto. When Witness’s deposition was taken, he testified that he saw Driver come to a
complete stop at the intersection, but that Cyclist did not stop. He also stated that the stop sign for
westbound traffic was partly obscured by tall weeds, which made it difficult to see until a motorist was
quite near the intersection. At her deposition, Passenger testified that, because of the accident-induced
amnesia, she had no personal recollection of the events surrounding the collision.

After the close of discovery, Driver moved for summary judgment. Driver supported this motion with: (1)
Passenger’s answer to the interrogatories; (2) the deposition of Witness; (3) Driver’s own affidavit to the
effect that the westbound stop sign was difficult to see because of the growth of tall weeds in front of it,
but that he had come to a complete stop before entering the intersection, whereas Cyclist did not; and (4)
Passenger’s deposition testimony regarding her amnesia and lack of personal recollection of the events
surrounding the collision.

Passenger opposed the motion for summary judgment. Her opposition papers cited the conflicting facts
raised by the allegation in her complaint that Driver failed to stop and Driver’s denial of that allegation in
his answer. In support of her opposition, she offered (1) an affidavit of the police officer who responded
to the accident, stating that the westbound stop sign was not hidden by weeds, but, on the contrary, was
plainly visible; and (2) Passenger’s own affidavit stating, on information and belief, that Cyclist must
have arrived at the intersection first, come to a complete stop, and only then proceeded into the
intersection, and that Driver must have entered the intersection without stopping or even slowing down.

Should the trial court grant or deny Driver’s motion for summary judgment? Explain.

Seperac-J19 Exam-Released MEE Essay Compilation © 2016-2020 181


#040: F98-4 MEE: ANSWER: NCBE (CIVIL PROCEDURE)

POINT (1) [26%] ISSUE: Does the existence of a material issue on the face of the pleadings
preclude adjudication of a case without trial? ANSWER: No. Existence of a material issue on the
face of the pleadings will not preclude adjudication of a case without trial when it is shown on
motion for summary judgment, as here, that plaintiff has no evidence to support her pleading on
the issue of defendant’s negligence.

POINT (2) [26%] ISSUE: Does conflict in affidavits concerning an immaterial fact preclude a
summary judgment? ANSWER: No. The conflict in the affidavits concerning the visibility of the
stop sign will not preclude a summary judgment because that issue will be immaterial in the light of
uncontradicted evidence that Driver did stop.

POINT (3) [47%] ISSUE: Does the plaintiffs affidavit controverting defendant’s version of the
accident preclude a summary judgment, when affidavits and depositions clearly show that plaintiff
cannot present evidence to support her version of the facts? ANSWER: No. Plaintiffs affidavit
controverting defendant’s version of the accident will not preclude a summary judgment because
her affidavit fails to show that she can produce any evidence to support her allegation that
defendant failed to stop.

ANSWER EXPLANATION:

Explanation to Point-One (20-30%):

Existence of a material issue on the face of the pleadings will not preclude adjudication of a case without
trial when it is shown on motion for summary judgment, as here, that plaintiff has no evidence to support
her pleading on the issue of defendant’s negligence.

The face of the pleadings in this case presents a material issue of fact, namely, whether Driver’s
negligence was the proximate cause of the collision between his auto and Cyclist’s motorcycle. A motion
for summary judgment is a procedural device that is capable of going behind the pleadings and
demonstrating that there really is no genuine issue as to any material fact. Absent any genuine and
material factual issue, there is no need for a trial of the action. One side or the other is entitled to win the
case as a matter of law.

Driver has used an interrogatory to force Passenger to state a more specific basis for her conclusory
allegation that Driver was “negligent.” He has then set about showing that when the case comes to trial,
Passenger will have no evidence to support this claim. If this is so, at trial Driver will be entitled to a
directed verdict at the close of the plaintiffs case. His motion for a summary judgment is in essence an
accelerated motion for a directed verdict: it seeks to show the court now what will happen later at trial
after the plaintiff has rested her case at trial. “The standard for granting summary judgment mirrors the
standard for a directed verdict.”

Under Ceotex v. Catrett, Driver must inform the court of the basis for his motion, explaining why there is
no genuine issue of fact. The papers supporting and opposing the motion for summary judgment indicate
that there are three factual issues: (1) Was the stop sign for westbound traffic obscured by weeds? (2) Did
Driver bring his vehicle to a complete stop before proceeding into the intersection? (3) Did Cyclist stop at

Seperac-J19 Exam-Released MEE Essay Compilation © 2016-2020 182


the stop sign? Driver must explain to the court that the first factual issue is immaterial, that he has
affirmatively produced evidence in his favor on the second and third, and that Passenger will be unable to
provide evidence to refute Driver’s evidence regarding the second and third issues.

Explanation to Point-Two (20-30%):

The conflict in the affidavits concerning the visibility of the stop sign will not preclude a summary
judgment because that issue will be immaterial in the light of uncontradicted evidence that Driver did
stop.

Driver, in his affidavit, and Witness, in his deposition, swear that the stop sign was obscured by weeds.
On the other hand, the affidavit of the investigating police officer says that it was plainly visible. This
creates a genuine issue of fact on this matter. Although a motion for summary judgment can dissolve a
“paper” issue, it cannot resolve a genuine one. Therefore, if this issue is also a material one, the motion
for summary judgment must be denied.

This issue would be material if Driver was claiming that he failed to stop because he could not see the
stop sign. However, Driver’s supporting papers show that notwithstanding the obscured sign, he did
manage to see and obey the sign by stopping his auto. Therefore, the visibility of the stop sign is not a
material fact in this case.

Explanation to Point-Three (40-50%):

Plaintiffs affidavit controverting defendant’s version of the accident will not preclude a summary
judgment because her affidavit fails to show that she can produce any evidence to support her allegation
that defendant failed to stop.

This is clearly a material factual issue whether Driver failed to stop at the intersection. However, for it to
be a genuine issue it must appear that at trial Passenger will have enough evidence to take her version of
the accident to the jury. Driver swears that he stopped and that Cyclist did not; Witness corroborates this.
This makes it incumbent on Passenger to show that at trial she will have some evidence to the contrary.

At trial Passenger cannot testify herself about which driver ran the stop sign, because her amnesia
precludes her from having any personal knowledge about this matter. Since she has no admissible
evidence to support her version of the accident, she will be unable, at trial, to carry her burden of proof on
the issue of Driver’s negligence. Passenger cannot make an issue merely by questioning the credibility of
Driver’s or Witness’s affidavits, unless she provides affidavits supporting her case on the merits or
specifically casting doubts on the veracity of their statements. Therefore, there is no genuine issue as to
this material fact. It may be conventional wisdom that summary judgment is generally inappropriate in
negligence cases, because the jury may still infer negligence when underlying facts are undisputed; but
that does not apply here, because plaintiff has no evidence to establish the only particular fact (Driver’s
failure to stop) from which negligence could be inferred.

The outcome of the case is a foregone conclusion: Driver would be entitled to a directed verdict because
of Passenger’s lack of evidence to support her version of the accident. Accordingly, the trial court should
enter a summary judgment in his favor.

Seperac-J19 Exam-Released MEE Essay Compilation © 2016-2020 183


#041-JUL 1997–MEE Q05: QUESTION FIVE (CIVIL PROCEDURE)

State C has recently enacted an “Unfair Trade Practices Act” (UTPA), the key provision of which reads:

No person shall engage in an unfair method of competition or an unfair act or practice in the conduct of
any trade or business.

The Act further provides a private right of action for “any individual who suffers an ascertainable
monetary loss” as a result of conduct that violates the UTPA.

Acme is an auto liability insurance corporation incorporated under the laws of State A, with its principal
place of business in State B. Acme engages in the insurance business in most states in the United States,
including State C. Concerned about the size of jury verdicts in personal injury cases tried in State C
courts, Acme ran full-page ads in all major newspapers in State C criticizing as grossly excessive the
verdicts being returned by State C jurors. In the ads, Acme urged those who were to be jurors in the future
to take into account in their deliberations the impact their verdicts would have on their own liability
insurance premiums.

Parker is the plaintiff in a major personal injury case soon to be tried in a State C court. Parker, who is not
insured by Acme, considers Acme’s ad campaign to be a form of “unfair act or practice” in violation of
the UTPA. Consequently, Parker has filed an action for $500,000 in damages against Acme in the United
States District Court for State C, invoking diversity jurisdiction. At the time this suit was filed, Parker was
a citizen of State C. However, one month after the suit was filed, and one week before Acme was served
with the summons and complaint, Parker moved his permanent residence to State B, where he now
resides.

Acme has simultaneously filed two motions. The first is a motion to dismiss Parker’s suit for lack of
jurisdiction over the subject matter on the grounds that: (1) Parker, a citizen of State C on the date suit
was filed, was not eligible to file a diversity case in a federal court located in the state where Parker was
then domiciled; and (2) even if there were diversity jurisdiction at the time the suit was filed, it has been
divested by Parker’s change of domicile to State B.

Acme’s second motion asks the federal district court to stay further proceedings in the case on the ground
that it has raised the following defenses: (1) the legislative history of the UTPA reveals that the private
right of action it confers is available only to competitors or customers of the one allegedly violating the
Act; (2) Acme’s ad campaign is not an “unfair act or practice” within the meaning of the UTPA; and (3) if
the UTPA is interpreted to prohibit Acme’s ad campaign, the Act violates the free speech provisions of
the United States Constitution. Acme argues that the federal district court should postpone deciding these
issues because a State C trial court has recently dismissed an action brought under the UTPA because the
plaintiff in that action was neither a customer nor a competitor of the defendant in that case. An appeal
from that ruling has been accepted by the State C Supreme Court.

1. How should the federal district court rule on the motion to dismiss for lack of subject matter
jurisdiction? Explain.

2. How should the federal district court rule on the motion to stay the case? Explain.

Seperac-J19 Exam-Released MEE Essay Compilation © 2016-2020 184


#041: J97-5 MEE: ANSWER: NCBE (CIVIL PROCEDURE)

POINT (1) [50%] ISSUE: Were the requirements for federal diversity jurisdiction satisfied in this
case, even though the plaintiff sued in a federal district court in his home state and, subsequent to
filing the complaint, moved so as to become domiciled in the same state as defendant? ANSWER:
Yes. Because plaintiff Parker was a citizen of a different state than defendant Acme when the
complaint was filed and the amount in controversy exceeds $75,000 Parker properly invoked
federal diversity jurisdiction, which is not defeated by the fact that the suit was filed in a federal
district court in his home state, nor by the fact that he later moved to State B where Acme is also
deemed to be a citizen.

POINT (2) [50%] ISSUE: In a diversity case in which both the meaning and the constitutionality of
a new state statute are challenged, should a federal district court stay the case to allow the courts of
the state an opportunity to rule on the statute’s meaning and thereby possibly render the
constitutional issue moot? ANSWER: Yes. Because the state courts of State C may interpret the
Unfair Trade Practices Act (UTPA) either to deny Parker standing to sue for the UTPA violation,
or to exclude Acme’s ad campaign from its coverage, and because either interpretation would moot
the constitutional issue raised by Acme, the federal district court should abstain from deciding
either of these issues until the State C courts have an opportunity to resolve the statute’s meaning.

ANSWER EXPLANATION:

Explanation to Point-One (45-55%):

Because plaintiff Parker was a citizen of a different state than defendant Acme when the complaint was
filed and the amount in controversy exceeds $75,000 Parker properly invoked federal diversity
jurisdiction, which is not defeated by the fact that the suit was filed in a federal district court in his home
state, nor by the fact that he later moved to State B where Acme is also deemed to be a citizen.

The court should deny the defendant’s motion to dismiss for lack of subject matter jurisdiction. The
requirements for federal diversity jurisdiction have been established in this case. Parker’s complaint does
not state a claim arising under the Constitution or laws of the United States, 28 U.S.C. § 1331, so the
federal court’s jurisdiction must be based upon diversity of citizenship. The plaintiff and defendant must
be citizens of different states. For purposes of diversity jurisdiction, Parker was a citizen of the state in
which he was domiciled when the complaint was filed, State C, and Acme is deemed a citizen of the state
in which it is incorporated, State A, and the state in which it has its principal place of business, State B.

For federal diversity jurisdiction, the amount in controversy must exceed $75,000, exclusive of interests
and costs. The amount in controversy is generally measured by the amount claimed in apparent good faith
on the face of the complaint, which in Parker’s case exceeds $75,000.

It is sometimes said that the purpose of diversity jurisdiction is to protect out-of-state litigants from local
prejudice. However, the constitutional language authorizing diversity jurisdiction and the statutory
language implementing it allows a plaintiff to invoke original federal jurisdiction based upon diversity of
citizenship not only in the state where the defendant resides but also where the plaintiff resides.
Accordingly, the first ground of Acme’s attack on subject matter jurisdiction is without merit. Acme’s
argument on this point confuses the requirements for original federal jurisdiction with the requirement for

Seperac-J19 Exam-Released MEE Essay Compilation © 2016-2020 185


removal jurisdiction; for the latter, only a non-resident defendant may remove a state action to a federal
court based upon diversity of citizenship.

If Parker had been domiciled in State B at the time suit was filed against Acme, the requirement for
original federal diversity jurisdiction would not have been satisfied. However, it is well settled that the
determination of whether there is diversity of citizenship is made as of the date the suit is filed. If there is
diversity jurisdiction on that date, it will not be lost or divested by later events, such as the acquisition of a
common domicile by plaintiff and defendant. On the date Parker’s suit was filed, plaintiff Parker was a
citizen of State C, and corporate defendant Acme was a citizen of States A and B. At that time, Parker’s
citizenship was different from either of Acme’s dual citizenships. Therefore, plaintiff’s motion to dismiss
for lack of subject matter jurisdiction should be denied.

Explanation to Point-Two (45-55%):

Because the state courts of State C may interpret the Unfair Trade Practices Act (UTPA) either to deny
Parker standing to sue for the UTPA violation, or to exclude Acme’s ad campaign from its coverage, and
because either interpretation would moot the constitutional issue raised by Acme, the federal district court
should abstain from deciding either of these issues until the State C courts have an opportunity to resolve
the statute’s meaning.

Because this case is before the federal district court for State C by virtue of its diversity jurisdiction, the
court is required by the Erie doctrine to apply the substantive law of State C, whose statutes have created
any cause of action that Parker may have. Although it is clear that State C substantive law governs this
case, it is unclear what that substantive law is. This happens frequently in diversity cases, and the normal
procedure is for the federal district court to go forward, do its best to divine what the state law is, and
decide the case. The leading decision is Meredith v. Winter Haven, in which the court stated:

Congress having adopted the policy of opening the federal courts to suitors in all diversity cases involving
the jurisdictional amount, we can discern in its action no recognition of a policy which would exclude
cases from the jurisdiction merely because they involve state law or because the law is uncertain or
difficult to determine.

Only in rare instances (e.g., eminent domain cases where an important state interest is also involved in the
ultimate decision) should a federal judge abstain from deciding a diversity case simply because the
applicable state law is unclear.

However, in this case there are two reasons why it is appropriate for the court to grant the motion for a
stay and postpone deciding the case. The best argument for a stay is under the well-established Pullman
abstention doctrine. Under that doctrine, “if there are unsettled questions of state law in a case that may
make it unnecessary to decide a federal constitutional question, the federal court should abstain until the
state court has resolved the state questions.” The facts of this question present a pristine case for
application of the Pullman abstention doctrine. If the federal court goes forward with the case, it may
decide that Parker has standing to sue under the state statute and that Acme’s ad campaign violates the
statute. It would then be required to evaluate the constitutionality of the statute. If, on the other hand, it
waits for the State C Supreme Court’s interpretation of the statute, that Court may decide that only
customers or competitors of the defendant have standing to sue under the state UTPA. In that case, Parker
would have no standing and the federal court could dismiss his claim without reaching the question of the
constitutionality of the state statute. Thus, by granting a stay, the court may avoid unnecessarily deciding
a federal constitutional question. A stay should therefore be granted.

Seperac-J19 Exam-Released MEE Essay Compilation © 2016-2020 186


A second reason for granting a stay in this case is that state law is unclear AND there is a state action
pending which may result in a clarification of the law. As noted above, mere difficulty or uncertainty in
interpreting state law is NOT an adequate basis for a federal court to stay its hand in a diversity action.
However, abstention in private litigation to avoid deciding difficult questions of state law IS regarded as
proper when “the federal court merely postpones decision for a time to await an opinion of the state court
in an action already pending.”

In the instant action, the proper course for the federal district court is to stay Parker’s case. The State C
Supreme Court may then decide the case pending before it in a way that would deny Parker’s standing to
sue under the UTPA. This would moot the constitutional issue. In addition, the newly-enacted State C
statute is unclear and an action which will clarify it is currently pending before the State G Supreme
Court. Those facts also would support a postponement of the federal action.

Seperac-J19 Exam-Released MEE Essay Compilation © 2016-2020 187


#042-FEB 1997–MEE Q05: QUESTION FIVE (CIVIL PROCEDURE/CONFLICTS)

Ten years ago, a valuable painting by an old master was stolen from Museum, a nonprofit organization
incorporated and located in State X. At the time of the theft, the painting was worth approximately
$800,000. Today it is worth several million dollars. Police investigated the theft, but no suspects were
identified. Museum made diligent efforts to locate the painting, but it was never recovered.

Two days ago, on Wednesday, Museum Director learned from a friend in the art business that the stolen
painting had surfaced in the hands of Collector, a resident and citizen of State Y. Museum Director’s
friend told him that Collector plans to sell the painting to an art dealer from Germany; that the sale is
scheduled to take place tomorrow, on Saturday, at Collector’s home in State Y; and that the German
dealer has airplane tickets back to Germany shortly after the scheduled sale.

Museum’s lawyer prepared a civil action against Collector to recover possession of the painting. This
morning (Friday), the verified complaint in that civil action was filed in federal district court in State Y,
properly invoking the diversity jurisdiction of the court. Museum simultaneously filed a motion asking the
federal district court to restrain Collector’s sale of the painting.

Both the verified complaint and the motion allege (among other things) that Museum owns the painting;
that it was stolen; that Collector now has possession of it; that the painting is a priceless example of
Renaissance artwork; that it was the centerpiece of Museum’s collection before its theft; that Museum
believes on the basis of reliable information that Collector plans to sell the painting to a foreign purchaser;
and that Museum will suffer irreparable injury if the painting is sold and taken out of the country. The
motion is also supported by an affidavit, based upon Museum Director’s personal knowledge, detailing
the difficulty Museum would have in recovering the painting if it left the country.

A separate affidavit by Museum’s attorney states that several efforts have been made to locate Collector at
his home and office in order to serve process and a copy of the motion, but that those efforts have failed,
and that the location of Collector is unknown. Collector is, however, expected to be at his home to
consummate the sale of the painting on Saturday morning, and Museum plans to have both the summons
and the requested restraining order served on Collector at that time. Museum has filed with its motion a
properly secured bond in an amount adequate to reimburse Collector for costs and damages that might
result if Collector is wrongfully restrained.

The State Y statute of limitations for a civil action to recover possession of personal property has run.
State X has a 15-year statute of limitations for art theft cases, included as part of a law that created a cause
of action specifically for recovery of stolen art objects. This statute of limitations has not run.

1. May the federal district court issue an order restraining the sale of the painting before Collector
has been served with process or given notice of the motion to restrain the sale? Explain.

2. If the court issues a restraining order and it is properly served on Collector (together with copies
of the summons and complaint), will the order remain in effect until there can be a trial on the
merits of the civil action, or must Museum take further steps to continue the restraint on Collector
pending such trial? Explain.

3. Will the federal district court apply State Y’s statute of limitations, thus barring the cause of
action created by the law of State X? Explain.

Seperac-J19 Exam-Released MEE Essay Compilation © 2016-2020 188


#042: F97-5 MEE: ANSWER: NCBE (CIVIL PROCEDURE/CONFLICTS)

POINT (1) [43%] ISSUE: May a federal district court issue an order restraining Collector from
selling the painting even though Collector has not been served with process in the action and does
not have notice of the request for the order? ANSWER: Yes. Under FRCP Rule 65(b) the court
may grant a temporary restraining order against Collector without notice to Collector if
irreparable injury will result without the issuance of the ex parte order and if notice cannot
reasonably be given.

POINT (2) [24%] ISSUE: If the court issues an ex parte order restraining the sale of the painting,
how long will the order be effective and what further steps, if any, must Museum take to preserve
the order in force until the date of trial? ANSWER: Absent Collector’s consent or extraordinary
circumstances, an ex parte restraining order can stay in effect a maximum of 14 days after which
Museum must seek a preliminary injunction (which requires a hearing).

POINT (3) [33%] ISSUE: Should the State Y statute of limitations be applied to bar Museum’s
cause of action to recover possession of the painting? ANSWER: Yes. The federal district court
should probably apply the State Y statute of limitations to this action, although it might apply the
State X statute on the theory that it goes to the right and not the remedy.

ANSWER EXPLANATION:

Explanation to Point-One (40-50%):

Under FRCP Rule 65(b) the court may grant a temporary restraining order against Collector without
notice to Collector if irreparable injury will result without the issuance of the ex parte order and if notice
cannot reasonably be given.

FRCP Rule 65(b) permits federal courts to issue temporary restraining orders on an ex parte basis when
that is necessary to prevent irreparable injury and preserve the status quo pending a hearing on an
application for a preliminary injunction. Because the issuance of a restraining order without notice to a
party is an extraordinary action that implicates the restrained party’s due process rights, Rule 65(b) strictly
circumscribes the circumstances under which ex parte restraining orders may be granted.

To obtain an ex parte temporary restraining order, the moving party must meet three conditions set out in
the federal rules: (1) the moving party must set out “specific facts” in the verified complaint or by
affidavit that show that “immediate and irreparable injury will result to the applicant before the adverse
party can be heard in opposition”; (2) the moving party’s attorney “must certify to the court in writing the
efforts, if any, which have been made to give the notice and the reasons supporting the claim that notice
should not be required,” and (3) the moving party must provide “security.in such sum as the court deems
proper” before the order is issued.

Even if the basic requirements of the rules are met, the issuance of a temporary restraining order is still a
matter for the sound discretion of the trial court. In addition to insisting on a showing of irreparable
injury, the court may demand a demonstration of some likelihood that plaintiff will prevail on the merits.
The court will also consider any harm that granting the order will cause and will weigh that harm against
the hardship caused to the plaintiff if the order is not granted.

Seperac-J19 Exam-Released MEE Essay Compilation © 2016-2020 189


The facts of the problem suggest that it is an appropriate case for entry of an ex parte temporary
restraining order. First, Museum is in danger of suffering irreparable injury if the order is not granted. The
painting is about to be sold and taken out of the country, beyond the reach of the court’s process. If that
happens, Museum will have great difficulty in recovering it. (As required by Rule 65, these facts are set
out in the verified complaint or by affidavit.) Moreover, the loss of the painting, a “centerpiece of
Museum’s collection,” is not the kind of loss that could be adequately compensated by damages through
an action for conversion against Collector.

Second, Museum has made a good showing that the order must be issued before notice is given to
Collector if the requested relief is to be effective. Museum has attempted to find and notify Collector of
the action and of its motion, but has failed to do so. It expects to be able to find Collector at his home at
the time of the sale, but unless it has a restraining order in hand at that time, it will be unable to prevent
the sale.

Other factors support issuance. There is no reason to believe that Collector would be greatly harmed by a
delay in the sale of the painting. Certainly, any damages suffered by Collector as a result of the delay
could be compensated. Moreover, once Collector learns of the restraining order, Collector can seek a
hearing from the court to have it dissolved. (Under FRCP Rule 65(b) such a hearing would have
precedence over all other matters on the court’s docket, and so could occur very quickly. See the next
point in this analysis.)

Because Museum also has provided adequate security pursuant to Rule 65(c), issuance of an ex parte
temporary restraining order would appear to be appropriate.

The fact that no summons has yet been served in this action is irrelevant. A civil action in the federal
courts is commenced by the filing of a complaint. Service of the summons is not necessary for the court to
take steps otherwise authorized by the rules. Given that Rule 65(b) authorizes ex parte TROs in limited
circumstances like these, it is irrelevant whether the summons has yet been served or not, especially when
the same factors that prevent Museum from giving notice of the motion for a TRO would also prevent
service of the summons.

Explanation to Point-Two (20-30%):

Absent Collector’s consent or extraordinary circumstances, an ex parte restraining order can stay in effect
a maximum of 14 days after which Museum must seek a preliminary injunction (which requires a
hearing).

Because an ex parte temporary restraining order is an extraordinary remedy, issued without the usual due
process guarantees of notice and a hearing, federal practice requires that temporary restraining orders be
dissolved as soon as practicable. If Museum wishes to restrain Collector’s sale of the painting until the
date of trial, the appropriate remedy is a preliminary injunction, which will issue only after notice to
Collector and a hearing.

FRCP Rule 65(b) ensures that a party who is restrained by a TRO will have an early opportunity for a
hearing by providing that any TRO “shall expire by its terms within such time after entry, not to exceed
14 days, as the court fixes” unless the restrained party consents to an extension or good cause can be
shown for an extension. Moreover, when a temporary restraining order is entered without notice, a
“motion for a preliminary injunction shall be set down for hearing at the earliest possible time,” and the
party who received the TRO must proceed with an application for a preliminary injunction or the TRO

Seperac-J19 Exam-Released MEE Essay Compilation © 2016-2020 190


will be dissolved. Finally, the party restrained by the TRO may move for its dissolution on two days'
notice to the party who obtained the TRO.

On the facts given, the court is unlikely to extend the TRO beyond 14 days and, in all probability, it will
expire before that date. Museum believes it will be able to locate Collector within 24 hours. This will
permit Collector to have notice of the restraint of the sale of the painting, and the court should allow
Collector to be heard as soon as possible. Certainly, given that Collector is expected to be located shortly,
there is no reason to extend the ex parte order beyond 14 days. Collector is certainly entitled to an
opportunity to be heard. In all likelihood, Collector will move to dissolve the TRO as soon as possible.

Thus, the TRO may not, without further steps, prevent sale of the painting until the civil action to recover
possession of the painting is resolved. Museum must take further steps to restrain Collector: it must seek a
preliminary injunction, which requires notice and a hearing. If Museum does not seek a preliminary
injunction, the restraint on Collector will expire within 14 days, if not earlier.

Explanation to Point-Three (30-40%):

The federal district court should probably apply the State Y statute of limitations to this action, although it
might apply the State X statute on the theory that it goes to the right and not the remedy.

In diversity actions, federal courts use state law to determine the limitation period applicable to state
causes of action. In determining which state’s limitation period to use, federal courts must look to the
choice of law rules of the state in which they sit.

The problem does not specify what choice of law approach is followed by the courts of State Y, where
this action is being heard. However, most states treat statute of limitations as procedural for choice of law
purposes. If State Y treats statutes of limitations as procedural, it would ordinarily apply its own statute of
limitations in the absence of a borrowing statute indicating that some other state’s statute of limitations
should be used. Because the problem does not indicate that there is any applicable borrowing statute, the
usual rule is that State Y should apply its own statute of limitations if that statute bars the action. If that is
done, the action is barred.

State X, however, has a special statute of limitations for this type of claim which is part of a law creating
a special cause of action for the recovery of stolen art work. Where a statute of limitations is specific to a
particular kind of claim and was created by the law that created the cause of action to which it applies,
courts will often characterize the statute of limitations as substantive. In such cases, a court may disregard
its own statute of limitations and apply the foreign “substantive” statute of limitations, at least if the
foreign law creating the cause of action will be used to determine other substantive rights in the case. This
old doctrine (that statutes of limitations are substantive when they go to the right and not just the remedy)
was preserved in the Second Restatement of Conflict of Laws. However, this traditional rule may not lead
to application of State X’s statute of limitations, because the rule was traditionally applied only when the
foreign statute of limitations BARRED an action otherwise timely under forum law, not when the foreign
statute extended the period for bringing the action.

Instead of treating statutes of limitations as procedural or using the traditional right-remedy tort, some
states use interest analysis to resolve choice of law issues, including statute of limitations issues. It is not
clear what result is proper if interest analysis is used to determine the applicable statute of limitations in
this case. On the one hand, State Y has a strong interest in applying its short statute to protect the interest
of its resident (Collector) in repose and of its residents and courts in being free from stale claims. On the
other hand, State X has a strong interest in its resident’s ability to recover property stolen from it and in

Seperac-J19 Exam-Released MEE Essay Compilation © 2016-2020 191


discouraging art theft by allowing the recovery of stolen art even years after the theft. The case is a true
conflict. State Y could either apply its own law to resolve the conflict, or seek somehow to determine
which state’s interest should be honored in the case. In short, the outcome of interest analysis is uncertain.

Seperac-J19 Exam-Released MEE Essay Compilation © 2016-2020 192


#043-JUL 1996–MEE Q04: QUESTION FOUR (CIVIL PROCEDURE)

Artist filed suit in federal district court, properly based on diversity of citizenship, against Gallery for
damages in excess of $50,000. Artist alleged that Gallery had breached a contract with Artist to purchase
an original painting that Artist was to create specially for Gallery for $60,000. In its answer, Gallery
denied the allegations, specifically asserting that Artist failed to deliver the painting when due. Gallery
also made a $100,000 counterclaim against Artist for defamation. Artist filed a timely reply denying the
counterclaim.

After the discovery deadline had passed, Gallery filed two motions: one for summary judgment against
Artist on his claim for breach of contract, and the other for partial summary judgment to establish Artist's
liability on Gallery's defamation counterclaim.

Gallery's motion for summary judgment on Artist's claim for breach of contract was supported by an
affidavit authenticating a copy of the written contract between Artist and Gallery. Paragraph 4 of the
contract provides that Artist shall deliver the painting to Gallery on or before December 31, 1995, and
further provides that timely delivery is "of the essence." Paragraph 5 provides that Gallery will pay Artist
$60,000 upon delivery of the painting. Paragraph 10 provides that no amendment or alteration of the
contract shall be valid unless it is in writing and signed by the parties.

Gallery's motion for summary judgment on Artist's claim for breach of contract was also supported by
documents, affidavits, and depositions of witnesses who were competent to testify to the following facts:
(1) Artist called Gallery's curator on December 29, 1995, asking for a one-month extension of time to
deliver the painting, but Curator denied the request, explaining that Gallery needed the painting by
December 31,1995, to prepare it and the gallery for a major exhibition opening on February 3,1996; (2)
Artist did not deliver the painting on or before December 31, 1995; and (3) Gallery sent and Artist
received a letter dated January 29, 1996, canceling the contract because of Artist's failure to make timely
delivery of the painting.

In opposition to Gallery's motion for summary judgment on the claim for breach of contract, Artist filed
his own affidavit stating that: (1) on December 29,1995, he called Gallery's curator and asked for a one-
month extension of the deadline to deliver the painting, and that Curator said, "Yes, all right"; (2) the
painting was 99% finished when he received the January 29,1996 letter notifying him that Gallery was
canceling the contract; (3) Gallery refused to accept the painting when Artist tried to deliver it on January
31,1996; and (4) the real reason for cancellation was that Gallery had been losing money for several
months before it canceled Artist's contract.

Gallery's motion for partial summary judgment on its counterclaim for defamation was supported by five
affidavits. Curator's affidavit stated that on February 3, 1996, in the presence of a large number of artists
and art patrons, Artist accused Gallery of "stealing" Artist's creative ideas and that after the incident
several popular painters told Curator they would never again sell their works to or through Gallery. Four
other persons submitted affidavits stating that they personally heard Artist's February 3 accusation that
Gallery had "stolen" some of his creative ideas.

In opposition to Gallery's motion for partial summary judgment on its defamation counterclaim, Artist
filed his own affidavit stating that he had never said that Gallery had "stolen" any of his creative ideas.

Seperac-J19 Exam-Released MEE Essay Compilation © 2016-2020 193


Under the applicable substantive law, a party to a contract may cancel the contract if the other party fails
to make timely performance of a material provision; an oral contract, an oral modification, or an oral
waiver of a provision of a written contract is not enforceable, and Artist cannot assert an estoppel under
these circumstances; and a false accusation made in the hearing of others that someone has "stolen"
another's creative ideas renders one liable for the tort of defamation, if proved by clear and convincing
evidence, even if special damages have not been alleged and proved.

1. Should the federal district court grant Gallery's motion for summary judgment on Artist's claim
for breach of contract? Explain.

2. Should the federal district court grant Gallery's motion for partial summary judgment of liability
on its counterclaim for defamation? Explain.

Seperac-J19 Exam-Released MEE Essay Compilation © 2016-2020 194


#043: J96-4 MEE: ANSWER: NCBE (CIVIL PROCEDURE)

POINT (1) [20%] ISSUE: What is the standard for determining whether to grant a motion for
summary judgment? ANSWER: A motion for summary judgment should be granted when there is
no genuine issue of material fact and the moving party is entitled to judgment as a matter of law.

POINT (2) [40%] ISSUE: Should a federal court grant summary judgment for a defendant on
plaintiff's claim for breach of a contract when the plaintiff's failure to timely fulfill a material
obligation of the contract is not disputed, and the only dispute pertains to possible existence of an
unenforceable oral contract? ANSWER: Yes. Defendant's motion for summary judgment on
plaintiff's claim for breach of contract should be granted because there has been adequate time for
discovery and plaintiff has shown no genuine issue of material fact.

POINT (3) [40%] ISSUE: Should a federal court grant partial summary judgment on liability to a
claimant on a legal claim for defamation, when the claimant provides affidavits or depositions of
five apparently credible witnesses who say they heard the allegedly defamatory words, but the
opposing party files an affidavit denying that he ever spoke those words? ANSWER: No. Gallery's
motion for partial summary judgment of liability on the defamation counterclaim should be denied
because there are genuine issues of material fact, even though the weight of the evidence seems
against Artist.

ANSWER EXPLANATION:

Explanation to Point-One (15-25%):

A motion for summary judgment should be granted when there is no genuine issue of material fact and
the moving party is entitled to judgment as a matter of law.

Rule 56 of the Federal Rules of Civil Procedure provides that summary judgment "shall be rendered
forthwith" when the moving party shows "that there is no genuine issue as to any material fact and that
the moving party is entitled to judgment as a matter of law." The party opposing a motion for summary
judgment must do more than merely speculate about theories or evidence it might be able to produce to
support them, nor may it simply rest on the pleadings. To defeat a motion for summary judgment that is
supported by affidavits and other evidence, the opposing party must present evidence showing that there
is a genuine issue of material fact.

Explanation to Point-Two (35-45%):

Defendant's motion for summary judgment on plaintiff's claim for breach of contract should be granted
because there has been adequate time for discovery and plaintiff has shown no genuine issue of material
fact.

Gallery has met the requirements of Rule 56 with respect to Artist's contract claim, and since Artist has
failed to show a genuine issue of material fact, the motion should be granted as to that claim.

First, it is noteworthy that Artist, the party opposing summary judgment, has had “adequate time for
discovery.” This is evident from the fact that the time set by the court for discovery has expired. If Artist
were unable to get needed affidavits to respond to Gallery's motion for summary judgment, he certainly

Seperac-J19 Exam-Released MEE Essay Compilation © 2016-2020 195


could have asked for a continuance of the motion for summary judgment pursuant to Rule 56(f) (“to
permit affidavits to be obtained or depositions to be taken or discovery to be had”), or even asked for an
extension of the time for discovery, pursuant to Rule 16(b) (on showing of “good cause”). But he has
done neither. Thus, the motion for summary judgment is not premature.

Second, paragraph 4 of the contract explicitly requires delivery of the painting by December 31, 1995,
and further provides that time is of the essence. It is undisputed that Artist failed to deliver the painting by
that date. Artist's own deposition so admits; his affidavit does not dispute that fact. The applicable law
provides that a party to a contract may cancel a contract for failure of the other party to make timely
performance of a material provision. There is no dispute about the materiality of timely performance. The
contract itself states that time is of the essence and Curator's affidavit that the gallery needed it then to
prepare it for the forthcoming exhibition confirms the materiality. Thus, the contract, affidavits, and
depositions establish Gallery's legal right to terminate the contract, and Gallery is entitled to judgment on
Artist's claim “as a matter of law.”

Third, it can be argued that there appears to be a “genuine issue” as to whether Curator approved Artist's
request for an extension of the date for delivery of the painting. Artist's affidavit states that Curator told
him it was “all right,” whereas Curator's deposition states that she could not agree to an extension. If that
disputed fact were material to Artist's claim, summary judgment clearly could not be granted. But
paragraph 10 of the contract and the given statement of substantive law clearly preclude any claim based
upon an oral contract, an oral modification, or waiver or estoppel. Thus, the dispute over oral
modification, oral contract, waiver, or estoppel is simply not “material” to Artist's contract. A real dispute
over immaterial facts does not bar entry of summary judgment.

Fourth, Artist's speculations and unsupported allegations concerning possibly improper motives for
Gallery's cancellation of the contract are insufficient to defeat Gallery's motion for summary judgment on
Artist's claim for breach of contract once Gallery proves that an essential element necessary for Artist to
recover is lacking. Furthermore, if Artist's mere speculations and hearsay are inadmissible, it would be
improper to consider them in passing on the motion for summary judgment. Only admissible evidence
(though not necessarily in admissible form) may be considered in passing on a motion for summary
judgment.

Finally, Artist's claim rests entirely on the written contract. Both the substantive law and paragraph 10 of
the contract exclude the possibility of a claim of oral contract or for oral modification of contract, and
there is no dispute about material acts or conduct. In cases in which documentary evidence is dispositive
of the critical issues, courts are less reluctant to enter summary judgment.

Explanation to Point-Three (35-45%):

Gallery's motion for partial summary judgment of liability on the defamation counterclaim should be
denied because there are genuine issues of material fact, even though the weight of the evidence seems
against Artist.

A court may grant partial summary judgment disposing of some but not all issues raised in the case. Thus,
it may grant summary judgment determining the liability portion of a claim even if damages remain to be
ascertained. The legal standard for granting summary judgment is the same, whether the movant is a
claimant or defendant (i.e., “that there is no genuine issue as to any material fact and that the moving
party is entitled to judgment as a matter of law”). It is essentially the same as that for granting a motion
for directed verdict.

Seperac-J19 Exam-Released MEE Essay Compilation © 2016-2020 196


The main obstacle to Gallery's motion for summary judgment on its defamation claim is that there clearly
is a “genuine issue” as to the material fact of whether Artist said that Gallery had “stolen” some of his
creative ideas. Curator and four others have sworn in their affidavits that they heard Artist say exactly
that; Artist's affidavit directly denies it. That creates a “genuine issue,” and makes summary judgment
inappropriate.

The purpose of summary judgment is only to dispose of cases in which there are no genuinely disputed
issues of material facts, not to resolve such issues. Even though the witnesses offering evidence on the
question of whether Artist made the alleged statement are five-to-one against Artist, and even though one
might argue that the witnesses for Gallery appear to have greater credibility than the self-interested party
opposing the motion, the functions of weighing the evidence and judging credibility are beyond the scope
of a motion for summary judgment. Credibility of witnesses is precisely the kind of issue which is rarely
appropriate for summary judgment.

One additional consideration discourages granting the motion for summary judgment on the defamation
claim. In passing on the motion for summary judgment, the court must apply the standard of proof that the
fact-finder would have to apply. In this case, the substantive law provides that Gallery can recover for
defamation only if it is proven by “clear and convincing evidence.” That makes entry of summary
judgment on Gallery's claim even more difficult.

Seperac-J19 Exam-Released MEE Essay Compilation © 2016-2020 197


#044-FEB 1996–MEE Q04: QUESTION FOUR (CIVIL PROCEDURE)

Patient, a resident and domiciliary of State A, traveled to Clinic in State B to receive treatment from
Doctor. Due to the negligence of both Doctor and Clinic, Patient's treatment was unsuccessful and Patient
was left permanently disabled.

Doctor is a resident and domiciliary of State B. Clinic is owned and operated by Hospitals Inc., a
multistate hospital chain which is incorporated in State B and which has its principal place of business in
State B. States A and B are adjoining states, and Patient's home in State A is located just a few miles from
Clinic and in the same metropolitan area.

Patient decided to sue Doctor and Hospitals. Patient's lawyer advised Patient to sue Doctor and Hospitals
in State Z, a state located about 750 miles from States A and B. State Z was the lawyer's preferred forum
because State Z juries are widely known for the generosity of the awards they make in medical
malpractice actions, and because Hospitals has recently received bad publicity in State Z for alleged
incompetence and substandard care at hospitals it owns in State Z.

Patient sued Doctor and Hospitals for $2 million in state court in State Z. Proper service was made and
jurisdiction obtained over Hospitals because Hospitals owns and operates several hospitals in State Z.
Jurisdiction was obtained over Doctor when Doctor was properly served with process in State Z while on
vacation.

Doctor and Hospitals properly removed the action from state court to the federal district court of State Z,
which had jurisdiction over the action because the parties are diverse and the amount in controversy
exceeds $50,000. Once in federal district court, Doctor and Hospitals moved to dismiss the action for
improper venue. The motion was denied.

Doctor and Hospitals then moved for a transfer of venue from the federal district court of State Z to the
federal district court of State B. The federal district court of State Z denied the transfer motion as well,
stating that "Patient is entitled to her choice of forum."

Upon entry of the final order denying their motion to transfer, Doctor and Hospitals immediately filed a
notice of appeal. Patient responded by filing a motion to dismiss the appeal for lack of a final judgment.

1. Did the federal district court of State Z correctly deny the motion to dismiss for lack of venue?
Explain.

2. Assuming venue was proper in the federal district court of State Z, should the court have granted
the motion to transfer to the federal district court of State B? Explain.

3. Should defendants' appeal of the order denying transfer be dismissed? Explain.

Seperac-J19 Exam-Released MEE Essay Compilation © 2016-2020 198


#044: F96-4 MEE: ANSWER: NCBE (CIVIL PROCEDURE)

POINT (1) [24%] ISSUE: Was venue proper in the federal district court of State Z? ANSWER: No.
Although venue would not have been proper in the federal district court of State Z if the action had
been filed there originally, venue is proper in a federal district court when, as here, the action has
been properly removed to federal court from a state court in the district.

POINT (2) [43%] ISSUE: Is this a proper case for transfer of venue? ANSWER: Yes. The district
court probably erred in refusing to transfer the action because the obvious superiority of State B as
a forum for the action would appear to outweigh Patient's interest in having her choice of a location
for the lawsuit preserved.

POINT (3) [33%] ISSUE: Is an order denying transfer of venue an appealable "final judgment"
under either 28 U.S.C.A. §1291 or the "collateral order" doctrine? ANSWER: No. Although the
order denying a transfer was probably in error, it is not a "final judgment" from which defendants
can appeal, and the appeal should be dismissed. Under the circumstances of this case, however,
defendants might attempt to have the order reviewed by a higher court under the "collateral order
doctrine" or by seeking an extraordinary writ.

ANSWER EXPLANATION:

Explanation to Point-One (20-30%):

Although venue would not have been proper in the federal district court of State Z if the action had been
filed there originally, venue is proper in a federal district court when, as here, the action has been properly
removed to federal court from a state court in the district.

a. General Venue Rule

When an action in federal district court is founded solely on diversity jurisdiction, as here, venue is
ordinarily proper only in “(1) a judicial district where any defendant resides, if all defendants reside in the
same state, (2) a judicial district in which a substantial part of the events or omissions giving rise to the
claim occurred or (3) a judicial district in which the defendants are subject to personal jurisdiction at the
time the action is commenced, if there is no district in which the action may otherwise be brought.” Under
this general venue provision, a federal district court in State Z would not ordinarily have venue over
Patient's case. State Z is not a district or state in which defendants reside, nor is it a district in which the
events giving rise to the claim occurred. Venue might be proper under subsection (3) because both
defendants were subject to personal jurisdiction in State Z at the time the action was commenced.
However, subsection (3) is applicable only if there is no other jurisdiction in which the action could have
been brought, and this action could easily have been brought in State B, the state of domicile and
residence of both defendants.

b. Special Venue Rule for Removed Actions

Although venue would appear to be improper in the federal district court of State Z if the ordinary venue
rules of 28 U.S.C.A. § 1391 were applied, the fact that this case was removed from state court leads to a
different result. Under 28 U.S.C.A. § 1441(a), when the district courts of the United States have original
jurisdiction over an action brought in a state court, the action may be removed by the defendants “to the

Seperac-J19 Exam-Released MEE Essay Compilation © 2016-2020 199


district court for the district embracing the place where such action is pending.” In this case, there is
diversity of citizenship between defendants and plaintiff, and the amount in controversy exceeds
$75,000.00 Therefore, the federal district courts would have original jurisdiction over the action. In such a
case, the action may be removed from the state court of State Z to the federal district court for State Z, the
place where the action is pending.

Section 1441(a)'s prescription that a removed action will be brought to “the district court for the district
embracing the place where such action is pending” is treated by the federal courts as a venue provision
which trumps section 1391(a) in removed actions. Thus, even if venue would have been improper had the
action originally been filed in federal district court for State Z, such venue is proper for a removed action
under 28 U.S.C.A. §1441(a). It has also been argued that a defendant, by seeking removal, waives any
objection to the venue of the court to which the action is removed. Thus, venue was proper in State Z, and
the motion to dismiss for lack of venue was properly denied.

Explanation to Point-Two (40-50%):

The district court probably erred in refusing to transfer the action because the obvious superiority of State
B as a forum for the action would appear to outweigh Patient's interest in having her choice of a location
for the lawsuit preserved.

Under 28 U.S.C.A. § 1404, a district court may transfer an action “to any other district or division where
it might have been brought.” The motion for transfer must be made on the ground that it is “for the
convenience of parties and witnesses” and “in the interest of justice.” Whether to grant a transfer is a
matter for the discretion of the trial judge, and the judge's decision is subject to review only for abuse of
that discretion.

Section 1404(a) calls for “a particularized determination, under all the circumstances of an individual
case, on where it may most conveniently be tried.” Under this test, the action probably should have been
transferred from the federal district court of State Z to the federal district court of State B.

First, the action could have been brought originally in the federal district court of State B. Venue would
have been proper in that court under 28 U.S.C. § 1391, because both defendants reside there. Moreover,
because both defendants reside, are domiciled, and do business in State B, Patient would have had no
difficulty obtaining personal jurisdiction over them in that state. Thus, the first requirement of a transfer,
that the transfer be to a district where the action might have been brought, is easily satisfied.

It would also appear that “the convenience of the parties and witnesses” and the “interest of justice” call
for a transfer. Both defendants reside in State B, and plaintiff lives just a short distance away in State A;
for all three parties, a State B forum is likely to be more convenient than a distant State Z forum.
Moreover, the facts that form the basis of the cause of action arose in State B; relevant evidence is likely
to be located there, as are the relevant witnesses. It is therefore likely that a State B district court would
have easier access to evidence, witnesses, and other sources of proof, and that a trial in State B would be
more convenient for the likely witnesses than a trial in State Z.

The “interests of justice” would also seem to favor State B as a forum. Not only will evidence necessary
to the trial of the action be more readily available to a district court in State B, but there is some indication
that a trial in State Z would be prejudiced by the adverse publicity that one of the defendants has been
receiving in State Z on unrelated matters.

Seperac-J19 Exam-Released MEE Essay Compilation © 2016-2020 200


There appear to be only two possible reasons that might justify the State Z district court's refusal to
transfer the action. First, some courts have said that a plaintiff’s choice of forum is entitled to substantial
weight and should rarely be disturbed. However, this consideration ought not to be of controlling weight
where, as here, the chosen forum has no real connection to the case and considerations of convenience
weigh heavily in favor of transfer.

Second, because State Z choice-of-law rules will apply to this action even after transfer, it is arguable that
the case should remain in a court that has familiarity with State Z law. However, given the complete
absence of any real connection with State Z, and the very close connections of the case with States A and
B, it is quite probable that, under State Z's choice-of-law rules, the applicable substantive law ultimately
used to decide this case will be the law of State A or B. Under such circumstances, transfer is likely to
serve the purpose of having the case tried in a court with familiarity with the applicable law, while a
refusal to transfer may result in a federal court in State Z having to apply the medical malpractice law of a
faraway jurisdiction.

On balance, therefore, it would appear that the convenience of the parties and witnesses and the interests
of justice overwhelmingly favor a transfer. The interest of Patient in having her forum choice respected
should not be allowed to trump the congressional policy, embodied in section 1404(a), of encouraging
transfers under circumstances like these. The district court of State Z therefore abused its discretion in
denying the motion to transfer.

Explanation to Point-Three (30-40%):

Although the order denying a transfer was probably in error, it is not a "final judgment" from which
defendants can appeal, and the appeal should be dismissed. Under the circumstances of this case,
however, defendants might attempt to have the order reviewed by a higher court under the "collateral
order doctrine" or by seeking an extraordinary writ.

Under the final judgment rule, an appeal from a federal district court decision can ordinarily be taken only
after the court has reached a “final decision” in the case. A final judgment (or “final decision,” in the
language of the statute) is a decision or order that “ends the litigation on the merits and leaves nothing for
the court to do but execute the judgment.”

It is well settled and fairly obvious that an order denying a motion for transfer of venue is not a final
judgment. It does not “end the litigation,” nor is it a decision “on the merits.” An order granting or
denying a transfer is “interlocutory and not immediately appealable under 28 U.S.C.A. § 1291.”
Accordingly, an appeal from such an order should be dismissed.

In some circumstances, an order granting or denying transfer might be appealable under the collateral
order doctrine. The judge-made “collateral order” doctrine allows immediate appeal of otherwise non-
final orders when the order finally decides “a matter collateral to the rights underlying the action,” the
matter is “too important to be denied review,” and “a denial of immediate review as a practical matter
may preclude any review whatsoever.” Although the first of these requirements might be satisfied, it is
doubtful that decisions on transfer create such potentially adverse consequences that they are “too
important to be denied review” in the ordinary case. More critically, appellate courts can and do review
trial court rulings on transfer after the entry of final judgment in a case. Thus, there is no necessity for
immediate review. Moreover, courts tend to read the collateral order doctrine narrowly and restrictively,
for fear of being overwhelmed by interlocutory appeals.

Seperac-J19 Exam-Released MEE Essay Compilation © 2016-2020 201


The State Z court's transfer ruling might be reviewed other than by appeal. In some cases, parties have
obtained immediate review of transfer rulings by seeking the extraordinary writs of mandamus or
prohibition from courts of appeals to compel district courts to transfer (or retain) cases. Although many
commentators think the use of these writs to review transfer decisions is inappropriate when the challenge
is based on allegations that the judge improperly weighed the various factors relevant to a transfer,
Wright, Miller & Cooper have recently concluded that “the propriety of some form of interlocutory
review seems quite clear if the issue goes to the power of the district court to make the order it did and
only a question of law is presented.”

Seperac-J19 Exam-Released MEE Essay Compilation © 2016-2020 202


#045-JUL 1995–MEE Q06: QUESTION SIX (CIVIL PROCEDURE)

For several years, Employee worked for Boss at Custom Computers in Bordertown, State X. Employee
was Boss's sole employee. Together they assembled and sold customized computer hardware and
software to local businesses in Bordertown and in surrounding communities, including Eastville, which is
located in State Y. Boss is a citizen of State X. Employee, on the other hand, lives in Eastville and is a
citizen of State Y.

On January 1,1994, Boss fired Employee. On February 1,1994, Employee opened a new business,
Employee's Customized Computers, in Eastville, and began soliciting customers from among the
businesses that previously had done business with Boss's firm. Employee won business, in part, by telling
Boss's former customers that "I did all the work when I worked for Boss" and that "Boss is a drunken bum
who cannot be relied upon."

In late 1994, Boss sued Employee in federal district court in Eastville, alleging that Employee had stolen
most of Boss's customers and caused Boss damages of more than $100,000. The complaint alleged that
Employee had "maliciously interfered with Boss's contractual relationships with former customers by
lying about the extent of Employee's role in Boss's business and by lying about Boss's drinking habits."

A summons and complaint in the action were served on Employee in State Y by a process server who
went to Employee's home. Not finding Employee there, the process server slid the summons and
complaint under the front door. Employee found the summons upon returning home that evening. State
Y's local rules for service of process are identical to Rule 4(e) of the Federal Rules of Civil Procedure.

Both State X and State Y recognize the tort of malicious interference with contractual relations. The law
of each state provides that "malice" exists only when one person interferes with another person's
contractual relationships either (a) with improper motive-that is, with intent to injure the plaintiff's
business for a purpose other than competition or (b) by improper means-that is, by means that are civilly
actionable or criminally unlawful.

Employee filed a motion to dismiss the complaint for failure to state a claim upon which relief could be
granted. Employee attached to the motion to dismiss an affidavit in which Employee swore "that
Employee's efforts to take Boss's customers away were motivated not by malice but by an honest desire to
compete and win the business for Employee."

The trial judge accepted Employee's motion and affidavit, treated the motion as a motion for summary
judgment, and gave Boss the opportunity to file opposing affidavits. Boss filed a memorandum in
opposition to Employee's motion but did not attach any documents or affidavits. The trial judge then
denied Employee's motion.

Following denial of the first motion, Employee filed a second motion to dismiss the action, this time on
the ground of insufficiency of service of process. The trial judge granted the second motion and dismissed
the action for insufficiency of service of process.

Did the trial judge rule correctly on Employee's two motions? Explain.

Seperac-J19 Exam-Released MEE Essay Compilation © 2016-2020 203


#045: J95-6 MEE: ANSWER: NCBE (CIVIL PROCEDURE)

POINT (1) [58%] ISSUE: Was it proper for the trial judge to accept evidence on the motion to
dismiss for failure to state a claim, to treat the motion as a motion for summary judgment, and then
to deny that motion? ANSWER: Yes. The trial judge properly treated the first motion to dismiss as
a motion for summary judgment and properly denied summary judgment because there is a
genuine issue of material fact as to whether Employee's interference with Boss's contracts was
accomplished by improper means that were civilly actionable.

POINT (2) [21%] ISSUE: Are a summons and complaint properly served when they are left at the
home of defendant when defendant is absent? ANSWER: No. Service of process was improper
because the summons and complaint were not left with a person of suitable age and discretion.

POINT (3) [21%] ISSUE: Was it proper to grant a motion to dismiss for insufficiency of service of
process when that motion was made after an earlier motion to dismiss for failure to state a claim?
ANSWER: No. Although service was improper, the motion to dismiss for insufficiency of service
should have been denied because Employee waived this defense by not raising it in Employee's
earlier motion to dismiss.

ANSWER EXPLANATION:

Explanation to Point-One (50-60%):

The trial judge properly treated the first motion to dismiss as a motion for summary judgment and
properly denied summary judgment because there is a genuine issue of material fact as to whether
Employee's interference with Boss's contracts was accomplished by improper means that were civilly
actionable.

When, in connection with a motion to dismiss a pleading for failure to state a claim, the court is presented
with “matters outside the pleading,” and the court does not exclude that information, “the motion shall be
treated as one for summary judgment and disposed of as provided in Rule 56” of the Federal Rules of
Civil Procedure (hereinafter “FRCP”). Thus, when Employee attached an affidavit to the motion and the
trial judge considered that affidavit in connection with the motion (rather than excluding the affidavit), the
trial judge was obliged to treat the motion “as one for summary judgment.” On this score, the judge acted
properly.

A motion for summary judgment should be granted only if “the pleadings together with the affidavits
show that there is no genuine issue as to any material fact and that the moving party is entitled to a
judgment as a matter of law.” Summary judgment can be used to obtain a decision in cases where a
pleading sufficiently alleges a claim or defense, but where there is no evidence to support some essential
element of that claim or defense. It can also be used in cases where the parties agree on the basic facts, but
there is some dispositive legal issue that needs to be resolved.

The trial judge correctly denied summary judgment on the tort claim. Employee's affidavit tries to show
that there is no dispute as to Employee's motive in interfering with Boss's contracts – Employee hoped to
get the customers to do business with Employee. Such a motive does not constitute malice and is not itself
actionable. If “improper motive” were the only legal theory on which Boss's claim could be established,
Employee's affidavit might be enough to carry the initial burden of showing the lack of a “genuine

Seperac-J19 Exam-Released MEE Essay Compilation © 2016-2020 204


dispute” as to a material fact. Even though issues as to motive or intent are usually inappropriate for
summary judgment, Employee's affidavit might justify summary judgment for Employee, unless Boss
came forward with opposing affidavits, etc., to show evidentiary support for some motive other than
business competition.

However, under the law given in the problem, malice may be established by showing either (a) “improper
motive,” or (b) “improper means.” The complaint alleges that Employee misrepresented Employee's role
in Boss's company and that Employee lied about Boss's drinking habits. If proved, such conduct,
especially the slanderous allegations that Boss is a drunk, would establish the “civilly action able”
conduct necessary for malice. Because no evidence has been presented by Employee to contradict these
allegations or to establish that they are not “in dispute,” they constitute “material facts” as to which there
currently appears to be a genuine issue. The motion for summary judgment should therefore be denied.

If, on the other hand, Employee were to support the summary judgment motion with evidence suggesting
that Boss really could not prove that the statements attributed to Employee had been made or that they
were false, Boss could not rest on the pleadings but would need “by affidavits or otherwise to set forth
specific facts showing that there is a genuine issue for trial.”

Explanation to Point-Two (15-25%):

Service of process was improper because the summons and complaint were not left with a person of
suitable age and discretion.

Service of the summons and complaint in actions filed in federal district court is governed by Rule 4(e).
Service of the summons and complaint upon an individual may be accomplished by:

(i) delivering a copy to the individual personally,

(ii) leaving a copy at the person's “dwelling house or usual place of abode with some person of suit able
age and discretion then residing therein,” or

(iii) delivering a copy of the summons and complaint to an agent authorized to receive service of process.

Under Rule 4 it is also proper to serve an individual defendant in any manner permitted under the “law of
the State in which the district court is held.” However, the facts of this problem provide that State Y's
service rules parallel the federal rules exactly.

Under these rules, service was improper. Although the summons and complaint were left at Employee's
home, they were simply slid under the door. They were not left “with some person of suitable age and
discretion.” Service was therefore improper.

Explanation to Point-Three (15-25%):

Although service was improper, the motion to dismiss for insufficiency of service should have been
denied because Employee waived this defense by not raising it in Employee's earlier motion to dismiss.

It is a basic rule of federal motion practice under Rule 12 that certain defenses or objections that can be
raised under that rule should be consolidated and raised together, if possible. Thus, when a motion is first
made under Rule 12, the rule provides that certain defenses and objections are waived if they are not

Seperac-J19 Exam-Released MEE Essay Compilation © 2016-2020 205


raised in the first motion, provided that (i) under Rule 12, the defense or objection could have been raised
in the first motion, and (ii) the defense or objection was “available” at the time of the motion.

Employee's original motion to dismiss for failure to state a claim was raised under Rule 12. Under Rule
12(g), several other Rule 12 defenses or objections which were available at that time should have been
consolidated and raised in Employee's motion to dismiss for failure to state a claim. A claim of
“insufficiency of service of process” is one of the “defenses or objections” that can be raised in a Rule 12
motion. Moreover, Employee's claim that service of process was insufficient was available to Employee
at the time of the initial Rule 12 motion because Employee knew of the method of service at that time.
Thus, the “insufficiency of service” claim could have been raised in the initial motion. Employee's failure
to do so means that Employee may no longer make a motion “based on the defense so omitted.” In
addition, the “defense of insufficiency of service of process is waived” when it is omitted from the initial
Rule 12 motion, so that the defense may not be raised in the answer or at any other time.

Lastly, Rule 12(b) applies to “motions made under this Rule 12(b) rule.” Thus, it might also be argued
that, because the trial judge treated Employee's first motion as a motion for summary judgment under
Rule 56, the motion did not really “arise” under Rule 12(b) and that, therefore, the waiver rule is
inapplicable.

NOTE: Generally, when a party makes a pre-answer motion under Federal Rule of Civil Procedure 12, the
party must raise any claim of insufficiency of service of process that the party has at the time of the
motion; otherwise, the defense is waived. Rule 12 states that “a party that makes a motion under this rule
must not make another motion under this rule raising a defense or objection that was available to the party
but omitted from its earlier motion.” Further, “a party waives any defense listed in rule 12(b)(2)–(5) by
omitting it from a motion in the circumstances described in Rule 12(g)(2).” Nonetheless, courts have
allowed a motion to dismiss to be amended before the motion is heard, so long as the adverse party is not
prejudiced by the amendment and no delay results in the prosecution and determination of the case. Thus,
although not expressly provided for in Federal Rule 12(g), a preliminary motion may be amended to
include a defense or objection inadvertently omitted by the movant if there is no indication that the
proceedings will be delayed or the opposing party will be prejudiced.

Seperac-J19 Exam-Released MEE Essay Compilation © 2016-2020 206


#046-FEB 1995–MEE Q03: QUESTION THREE (CIVIL PROCEDURE)

Daniel was driving a Gasco gasoline truck on an interstate freeway at night when Paul, driver of a
minivan, struck the rear end of Daniel's truck, causing a collision and explosion. Paul was severely
burned, and Daniel suffered a total memory loss from the accident. Based on physical evidence at the
scene of the accident, the cause of the accident appeared to be excessive speed on Paul's part. The only
witness to the accident was Wendy, who was driving a Volkswagen behind Paul and Daniel. She stopped
after the accident and called the police and paramedics.

Alex, a claims investigator for Gasco's liability insurer, obtained a written statement from Wendy two
days after the accident. In Wendy's statement she said that Daniel caused the accident by making an
unsafe lane change into Paul's lane. Wendy said that Paul was too close to avoid the accident, although
she could not estimate how fast Paul was driving. Alex wrote two comments on Wendy's statement: 1)
Wendy is a very credible witness; and 2) her statement is very harmful to Daniel's potential defense.
Shortly after making her statement, Wendy disappeared and cannot be found. Alex immediately
forwarded Wendy's statement to Lawyer, Gasco's in-house attorney.

Paul sued Daniel and Gasco in a federal district court for personal injury damages based on Daniel's
negligent driving of the truck. Paul's suit was properly based upon diversity jurisdiction. Defendants filed
an answer specifically denying each allegation of wrongful conduct in Paul's complaint. During
discovery, Paul demanded production of Wendy's statement. Defendants claimed that the statement was
privileged or otherwise protected from discovery. Paul moved for an order compelling production of the
statement. Defendants filed a written response opposing production solely on the ground that the
statement was not reliable because it was not under oath. The written response was prepared and signed
by Lawyer. Lawyer developed the reliability argument without conducting any research or citing any
legal authority.

(a) Should the federal district court judge grant the motion compelling production of Wendy's
statement? Explain.

(b) What sanctions, if any, should the federal district court judge impose on Lawyer for making this
response? Explain.

Seperac-J19 Exam-Released MEE Essay Compilation © 2016-2020 207


#046: F95-3 MEE: ANSWER: NCBE (CIVIL PROCEDURE)

POINT (1)(a) [37%] ISSUE: Does a statement obtained by a claims investigator for defendant's
liability insurer two days after an accident qualify for limited immunity against discovery as trial
preparation material under FRCP 26(b)(3)? ANSWER: Yes. The requested statement may receive
qualified immunity against discovery as trial preparation material if it is determined to have been
prepared in anticipation of litigation, and not in the regular course of business.

POINT (1)(b) [37%] ISSUE: If the statement obtained by a claims investigator qualifies as trial
preparation material, has plaintiff made a sufficient showing of need to overcome the limited
immunity against discovery? ANSWER: The qualified immunity given the witness's statement, if it
was prepared in anticipation of litigation, may be overcome by a demonstration of substantial need,
but the court must still protect against disclosure of the mental impressions, conclusions, opinions,
or legal theories of an attorney or other representative.

POINT (2) [26%] ISSUE: Under FRCP 11 and 26(g), should sanctions be imposed against an
attorney who signs a written response to a discovery motion opposing discovery on grounds that are
not supported factually or legally? ANSWER: Yes. FRCP 26(g) imposes a mandatory duty on
attorneys who sign pleadings, motions, or discovery papers to certify that the signer has read the
paper and that to the best of the signer's knowledge, based on a reasonable inquiry into the facts
and law related to the paper, it is well grounded in fact, warranted by existing law or a good-faith
argument for change in existing law, and not imposed for any improper purpose. Sanctions for
violating the rule include an order to pay reasonable expenses incurred by the opposing party in
responding to the paper, including reasonable attorney's fees.

ANSWER EXPLANATION:

Explanation to Point-One(a) (30-40%):

The requested statement may receive qualified immunity against discovery as trial preparation material if
it is determined to have been prepared in anticipation of litigation, and not in the regular course of
business.

This question must be analyzed under the provisions of Rule 26 of the Federal Rules of Civil Procedure
(hereinafter “FRCP”), which codifies the work product doctrine of Hickman v. Taylor. (The statement is
not exempt from discovery under the attorney-client privilege, because it was given by a witness, not a
client.) The requested statement is clearly relevant and would be subject to discovery under FRCP 26,
unless it qualifies for limited protection against discovery as trial preparation material. The fact that the
statement was obtained by Alex, who is not a lawyer, is not controlling, because he is a claims
investigator for a liability insurer, and is one of the specified party representatives included under the
work product rule.

Documents “prepared in anticipation of litigation or for trial” are given qualified protection against
discovery under FRCP 26. On the other hand, documents prepared for ordinary business purposes, public
regulatory requirements, or other non-litigation purposes are not protected. Claims investigations may be
the ordinary course of business for an insurance company. In determining the applicability of the qualified
immunity under FRCP 26 to a witness statement obtained in the regular course of claims investigation, a

Seperac-J19 Exam-Released MEE Essay Compilation © 2016-2020 208


court should therefore consider all of the circumstances surrounding the preparation of the statement and
determine at what point, if any, the insurance company's activity shifted to the probability of litigation.

In this problem, the insurer's investigator obtained Wendy's statement two days after the accident.
Retention of counsel and counsel's involvement in the investigative efforts for an insurance company
would be factors, not necessarily dispositive, in deciding whether the requested document is protected
from discovery under the rule. Even though a copy of Wendy's statement was immediately sent to
Lawyer, Gasco's in-house attorney who ultimately represented defendants in the litigation, the given facts
do not specifically indicate whether the insurer's investigation was conducted under Lawyer's direction.
The point at which the insurer's investigative activity shifted from the ordinary course of business to
anticipation of litigation is not clearly resolved by the given facts. Therefore, it may be argued that the
statement is not trial preparation material and is not protected from discovery. Some courts have held,
however, that statements taken by a claims agent soon after an accident are taken in anticipation of
litigation and are entitled to qualified protection against discovery.

Explanation to Point-One(b) (30-40%):

The qualified immunity given the witness's statement, if it was prepared in anticipation of litigation, may
be overcome by a demonstration of substantial need, but the court must still protect against disclosure of
the mental impressions, conclusions, opinions, or legal theories of an attorney or other representative.

If Wendy's statement is trial preparation material, FRCP 26 provides a qualified immunity against
discovery. The discovering party can obtain discovery of the statement only if the substantial need and
undue hardship tests are satisfied. Substantial need requires something more than showing that the
statement will be useful. Rather, the discovering party has to show real need for the material. That party
also has to show undue hardship, in that the witness is no longer available, or can no longer remember the
material contained in the statement. In other words, if the witness can be questioned or deposed by the
discovering party's attorney, then the necessity and hardship requirements are not satisfied.

Thus, the next issue is whether the qualified immunity requirements have been met. In this case, Wendy
has disappeared, and she was the only non-party witness to the accident. Paul cannot depose Wendy, and
her statement is the only other direct observable source of information about the accident now that Daniel
has suffered a total memory loss. The statement is also very helpful to Paul's case. Paul has shown both
substantial need (highly relevant evidence and the only eyewitness account of the accident) and undue
hardship (Wendy is unavailable and cannot be deposed or questioned). Thus, the statement is discoverable
even if it is otherwise protected under the work product qualified immunity doctrine.

However, there is one remaining issue: the exclusion of the mental impressions, conclusions, opinions, or
legal theories of an attorney or other representative. Mental impressions must be protected from
disclosure under the provisions of the rule. In this case, that means that the two comments of Alex in the
statement, which contained his opinion of Wendy's credibility and of the effect of her statements on the
defense, must be excluded from the order compelling disclosure. These comments can be excised from
the statement before it is produced by Alex.

[NOTE: In 1993, the Federal Rules of Civil Procedure were amended. New FRCP Rule 26 requires any
party who withholds information on the basis of a claim of “protection as trial preparation material (i.e.,
work product)” to “make the claim expressly.” Because the defendant in this problem did not expressly
raise the work product claim, it might be argued that the work product issue has been waived and need
not be discussed at all. Rule 26, which requires that work product protection be “expressly claimed,”
would strongly support such a waiver analysis. Thus, an analysis that does not discuss the work product

Seperac-J19 Exam-Released MEE Essay Compilation © 2016-2020 209


issue is valid, as long as the analysis rests on the claim that Defendant's failure to expressly claim work
product protection constituted a waiver of such protection.]

Explanation to Point-Two (20-30%):

FRCP 26(g) imposes a mandatory duty on attorneys who sign pleadings, motions, or discovery papers to
certify that the signer has read the paper and that to the best of the signer's knowledge, based on a
reasonable inquiry into the facts and law related to the paper, it is well grounded in fact, warranted by
existing law or a good-faith argument for change in existing law, and not imposed for any improper
purpose. Sanctions for violating the rule include an order to pay reasonable expenses incurred by the
opposing party in responding to the paper, including reasonable attorney's fees.

FRCP Rule 26(g), pertaining specifically to discovery papers, states that the signature of an attorney
constitutes a certificate that the signer has read the discovery paper; that to the best of the signer's
knowledge, information, and belief formed after reasonable inquiry it is well grounded in fact and is
warranted by existing law or a good-faith argument for the extension, modification, or reversal of existing
law; and that it is not interposed for any improper purpose, such as to harass or to cause unnecessary delay
or needless increase in the cost of litigation. If a discovery paper is signed in violation of these rules, the
court should impose upon the person who signed it an appropriate sanction. The sanction may include an
order to pay to the other party or parties the amount of reasonable expenses incurred because of the filing
of the pleading, motion, or other paper, including a reasonable attorney's fee.

Lawyer signed the responsive papers to Paul's motion to compel production of documents. Thus, Lawyer
is subject to the certification requirements of FRCP Rule 26(g). The facts indicate that Lawyer did not
conduct any research, and developed the reliability argument without checking its validity under relevant
law. Had Lawyer done some research, Lawyer would have discovered better arguments relating to the
work product requirements of the federal rules. Lawyer's response then would have been focused on the
issues that the judge had to resolve to decide the motion. As it was, Lawyer's response was not at all
helpful to the judge, because it did not address any of the real issues. Thus, Lawyer violated the
reasonable inquiry test because the argument was not supported by existing law, or a good-faith argument
for change. The most likely sanction would be to require Lawyer to pay the attorney's fees incurred by
Paul in bringing the motion to compel production of Wendy's statement.

Seperac-J19 Exam-Released MEE Essay Compilation © 2016-2020 210


CONSTITUTIONAL LAW: 11 OF 24 MEE EXAMS: (46%)
#047-JUL 2018–MEE Q01: QUESTION ONE (CONSTITUTIONAL LAW)

In Gonzales v. Raich, 545 U.S. 1 (2005), the Supreme Court held that Congress has the power under the
Commerce Clause of Article I, Section 8, of the Constitution “to prohibit the local cultivation and use of
marijuana,” even when applicable state law permits such cultivation and even when the cultivation and
use are entirely within state borders. At the time of that decision, at least nine states authorized the use of
marijuana for medicinal reasons. Since the decision, medicinal use of marijuana has been approved in
numerous other states, and some states have also begun to allow the recreational use of marijuana.

Concerned with the widespread disregard of federal law in states that have “legalized” marijuana use,
Congress recently passed the Federal Drug Abuse Prevention Act. Sections 11 and 15 of that Act provide
as follows:

Section 11. Any state law enforcement officer or agency that takes any individual person into
custody for violation of any state law must make a reasonable investigation within five business
days to ascertain whether the individual in custody was under the influence of marijuana at the time
of the alleged offense. Such officers or agencies must file monthly reports with the federal Drug
Enforcement Agency on the outcome of these required investigations, including the name of any
individual determined to have been under the influence of marijuana at the time of his or her
alleged offense.

Section 15. No state government, state agency, or unit of local government within a state shall be
eligible to receive any funding through the federal Justice Assistance Grant program unless use of
marijuana is a criminal act in that state.

The Justice Assistance Grant program has been in existence for many years. It is the primary program
through which the federal government provides financial assistance for state law enforcement agencies.
Last year, the federal government made approximately $300 million in grants to state and local law
enforcement agencies through this program. Congress has appropriated another $300 million for such
grants in the upcoming fiscal year.

State A has a population of about 4 million people. Its crime rate is below average. Last year, total
spending by law enforcement agencies in State A was $600 million, of which $10 million came from
federal grants under the Justice Assistance Grant program.

State A recently adopted legislation decriminalizing the use of marijuana for all purposes by persons over
the age of 21.

As applied to State A,

1. Is Section 11 of the Federal Drug Abuse Prevention Act a constitutional exercise of federal
power? Explain.

2. Is Section 15 of the Federal Drug Abuse Prevention Act a constitutional exercise of federal
power? Explain.

Seperac-J19 Exam-Released MEE Essay Compilation © 2016-2020 211


#047: J18-1 MEE: ANSWER: NCBE (CONSTITUTIONAL LAW)

POINT (1) [50%] ISSUE: May the federal government force a state law enforcement officer or
agency to assist in the enforcement of federal drug laws by requiring the officer or agency to
conduct investigations of possible drug use by persons they take into custody and to make reports to
the federal government? ANSWER: No. The federal government may not command a law
enforcement officer or agency of State A to investigate and report on potential violations of federal
law.

POINT (2) [50%] ISSUE: May the federal government condition the grant of federal money for
state and local law enforcement activities on a state’s adoption of laws that criminalize use of
federally controlled drugs? ANSWER: Yes. The federal government probably can deny federal law
enforcement funds to State A if it does not criminalize the use of marijuana.

ANSWER DISCUSSION:

In the Federal Drug Abuse Prevention Act, Congress has sought to induce the states to adhere to the
federal policy of criminalizing the use of marijuana. The issue in this problem is whether Congress’s
efforts in this regard unconstitutionally intrude upon the sovereignty that the Constitution reserves to the
states. Section 11 of the Federal Drug Abuse Prevention Act is unconstitutional because it seeks to
commandeer state officers or agencies to provide assistance in the enforcement of federal drug laws.
Federal directives requiring the states to carry out federal regulatory programs are inconsistent with the
system of dual sovereignty created by the federal structure of the Constitution. Section 15 of the Federal
Drug Abuse Prevention Act, on the other hand, is a constitutional exercise of Congress’s spending power.
When Congress provides funds to the states, it may condition those funds on a state’s compliance with
federal directives, provided that the law meets certain requirements. Those requirements are met here:
Congress’s decision to spend money on state and local law enforcement activities is in pursuit of the
general welfare. The condition it is imposing on a state’s receipt of that funding is unambiguous. The
requirement that federally unlawful drug use must be criminalized at the state level bears a close
relationship to the law enforcement objectives of the spending program, and the condition imposed by the
federal government is not barred by any constitutional provision. Finally, Congress’s threat to withhold a
relatively small amount of federal money is not so coercive as to improperly intrude upon state
sovereignty.

ANSWER EXPLANATION:

Explanation to Point One (50%):

The federal government may not command a law enforcement officer or agency of State A to investigate
and report on potential violations of federal law.

The central issue raised by the statute described in this question (the “Federal Drug Abuse Prevention
Act”) is whether its provisions violate fundamental principles of federalism. Under the system of dual
sovereignty established by the Constitution, the States retain a significant measure of sovereign authority.
The Tenth Amendment confirms that the powers of the federal government are subject, in some cases, to
limits necessary to protect “state sovereignty” from federal intrusion. One of those limits is that Congress
may not “require the States to govern according to Congress’ instructions.” For example, a federal law

Seperac-J19 Exam-Released MEE Essay Compilation © 2016-2020 212


that commandeers the legislative processes of the States by directly compelling them to enact and enforce
a federal regulatory program is unconstitutional.

In Printz v. United States, the Supreme Court held that “commandeering” of State officials was also
unconstitutional under the federalism principle emanating from the Tenth Amendment. In Printz,
Congress ordered state law enforcement officials to conduct background checks of persons purchasing
firearms. By legislating to force the law enforcement officers to take certain actions “in their official
capacities as state officers,” the Court said, Congress was acting to control their actions “as agents of the
State.” Such an effort by the federal government “to direct the functioning of the state executive, and
hence to compromise the structural framework of dual sovereignty” is unconstitutional. The Court held
definitively that “the Federal Government may neither issue directives requiring the States to address
particular problems, nor command the States’ officers, or those of their political subdivisions, to
administer or enforce a federal regulatory program.” Section 11 of the Federal Drug Abuse Prevention
Act violates federalism principles. The law requires a State A law enforcement officer or agency to
undertake investigations aimed at detecting violations of federal drug laws and to report to federal
authorities on suspected violations. It seeks to compel state officers to participate in the enforcement of
the federal laws against the use of marijuana and thus unconstitutionally intrudes upon state sovereign
authority.

Explanation to Point Two (50%):

The federal government probably can deny federal law enforcement funds to State A if it does not
criminalize the use of marijuana.

Section 15 of the Federal Drug Abuse Prevention Act seeks to implement the federal anti- marijuana
policy by denying funding from the Justice Assistance Grant program to states that do not criminalize use
of marijuana. Congress may use a threat to withhold federal money to induce a state to exercise its
sovereign authority (e.g., by passing certain laws) to achieve congressional goals. The Supreme Court has
repeatedly held that such threats are constitutional exercises of Congress’s power to spend money for the
“general welfare of the United States” unless they are unduly coercive.

In South Dakota v. Dole, the Court held that Congress may condition the states’ receipt or use of federal
funds on state compliance with “federal statutory and administrative directives.” When using its spending
power in this way, Congress must satisfy certain requirements. First, the spending must be for the general
welfare, although a “court should defer substantially to the judgment of Congress” in this regard. Second,
the condition imposed by Congress must be imposed unambiguously. Third, the condition imposed must
be related “to the federal interest in particular national projects or programs.” Fourth, the condition
imposed must not “be used to induce the States to engage in activities that would themselves be
unconstitutional.” Finally, a condition will be deemed improper if it is “so coercive as to pass the point at
which ‘pressure turns into compulsion.’” For example, conditioning continued receipt of Medicaid funds
on compliance with new requirements is unconstitutional economic dragooning that leaves the States with
no real option but to acquiesce, because the threatened funding constituted over 10% of the State budgets.

In this case, Section 15 of the Act is probably constitutional. First, both the federal spending program and
the imposed condition are in pursuit of the general welfare. The Supreme Court has said that Congress’s
view of “the general welfare” deserves substantial deference, and there is no reason to believe that a court
would second-guess Congress’s judgment that the general welfare is served by assisting with the funding
of state law enforcement agencies in states that criminalize the use of drugs that Congress considers
dangerous.

Seperac-J19 Exam-Released MEE Essay Compilation © 2016-2020 213


The other three basic requirements are also satisfied. The condition being imposed on states that receive
funding from this particular program is unambiguous. The condition also relates generally to the purpose
of the federal funding, which is evidently to support and improve state and local law enforcement. Finally,
a requirement that the states criminalize the use of certain drugs does not induce any state to engage in
unconstitutional activity.

The threat of a loss of Justice Assistance Grant funds is probably not so coercive as to amount to an
unconstitutional intrusion on State A’s sovereignty. The amount of money involved in this case ($10
million) is only a small fraction (less than 2%) of State A’s law enforcement budget and thus likely a far
smaller part of its total state budget. This is utterly unlike the substantial economic loss (typically 10% of
the entire state budget) that faced the states in Sebelius, where the Court concluded that the states had “no
real option” other than to follow federal wishes. Rather, this is much closer to the “relatively mild
encouragement” that was upheld in South Dakota v. Dole (requiring South Dakota to raise the drinking
age to 21 years or lose highway funding amounting to less than half of one percent of the state’s total
budget). In short, although the funding condition acts as an incentive for State A to adhere to federal
policy, it does not “indirectly coerce” the State “to adopt a federal regulatory system as its own.” It
therefore is a proper exercise of Congress’s spending power and does not run afoul of constitutional
principles of federalism.

Seperac-J19 Exam-Released MEE Essay Compilation © 2016-2020 214


#048-JUL 2017–MEE Q02: QUESTION TWO (CONSTITUTIONAL LAW)

Businesses in the United States make billions of dollars in payments each day by electronic funds
transfers (also known as “wire transfers”). Banks allow their business customers to initiate payment
orders for wire transfers by electronic means. To ensure that these electronic payment orders actually
originate from their customers, and not from thieves, banks use a variety of security devices including
passwords and data encryption. Despite these efforts, thieves sometimes circumvent banks’ security
methods and cause banks to make unauthorized transfers from business customers’ bank accounts to the
thieves’ accounts.

To combat this type of fraud, State A recently passed a law requiring all banks that offer funds transfer
services to State A businesses to use biometric identification (e.g., fingerprints or retinal scans) to verify
payment orders above $10,000. Although experts dispute whether biometric identification is significantly
better than other security techniques, the State A legislature decided to require it after heavy lobbying
from a State A–based manufacturer of biometric identification equipment.

A large bank, incorporated and headquartered in State B, provides banking services to businesses in every
U.S. state, including State A. Implementation of biometric identification for this bank’s business
customers in State A would require the bank to reprogram its entire U.S. electronic banking system at a
cost of $50 million. The bank’s own security experts do not believe that biometric identification is a
particularly reliable security system. Thus, instead of complying with State A’s new law, the bank
informed its business customers in State A that it would no longer allow them to make electronically
initiated funds transfers. Many of the bank’s business customers responded by shifting their business to
other banks. The bank estimates that, as a result, it has lost profits in State A of $2 million.

There is no federal statute that governs the terms on which a bank may offer funds transfer services to its
business customers or the security measures that banks must implement in connection with such services.
The matter is governed entirely by state law.

The bank’s lawyers have drafted a complaint against State A and against State A’s Superintendent of
Banking in her official capacity. The complaint alleges all the facts stated above and asserts that the State
A statute requiring biometric identification as applied to the bank violates the U.S. Constitution. The
complaint seeks $2 million in damages from State A as compensation for the bank’s lost profits. The
complaint also seeks an injunction against the Superintendent of Banking to prevent her from taking any
action to enforce the allegedly unconstitutional State A statute.

1. Can the bank maintain a suit in federal court against State A for damages? Explain.

2. Can the bank maintain a suit in federal court against the state Superintendent of Banking to
enjoin her from enforcing the State A statute? Explain.

3. Is the State A statute unconstitutional? Explain.

Seperac-J19 Exam-Released MEE Essay Compilation © 2016-2020 215


#048: J17-2 MEE: ANSWER: NCBE (CONSTITUTIONAL LAW)

POINT (1) [30%] ISSUE: Can a private company maintain a suit against a state in federal court
seeking damages based upon a claim that the state injured its business by enforcing an
unconstitutional law? ANSWER: No. Because states are immune under the Eleventh Amendment
from suits for damages in federal court, a federal court would dismiss the bank’s damages claim
against State A if State A made a claim of sovereign immunity.

POINT (2) [30%] ISSUE: Can a private company maintain a suit against a state official in federal
court to enjoin that official from enforcing an allegedly unconstitutional law? ANSWER: Yes.
Pursuant to the doctrine of Ex parte Young, a suit against State A’s Superintendent of Banking to
enjoin the enforcement of an allegedly unconstitutional statute is not barred by the Eleventh
Amendment.

POINT (3) [40%] ISSUE: Does a state law that requires a multistate business to adopt expensive
security measures as a condition of providing certain services in the state impose an
unconstitutional burden on interstate commerce? ANSWER: Yes. Although the statute does not
discriminate against interstate commerce, it does impose a significant burden on interstate
commerce. A court could conclude that the law unconstitutionally burdens interstate commerce if
the court determines that the burden imposed is clearly excessive in relation to the purported
benefits.

ANSWER DISCUSSION:

The bank cannot maintain a suit against State A for damages in federal court. The Eleventh Amendment
precludes the federal court from exercising jurisdiction over a suit by a private party seeking to recover
damages from a state. The court can hear the bank’s claim against the Superintendent of Banking because
the bank has sued the superintendent in her official capacity and is seeking injunctive relief only.
Although the statute does not discriminate against interstate commerce, it does impose a significant
burden on interstate commerce. A court could conclude that the law unconstitutionally burdens interstate
commerce if the court finds that the burden imposed is clearly excessive in relation to the purported
benefits. A balancing of benefits and burdens would require the court to evaluate the extent of the actual
burden the statute imposes on the bank and whether the statute has substantial fraud-protection benefits.

ANSWER EXPLANATION:

Explanation to Point-One (30%):

Because states are immune under the Eleventh Amendment from suits for damages in federal court, a
federal court would dismiss the bank’s damages claim against State A if State A made a claim of
sovereign immunity.

The Eleventh Amendment provides that “the Judicial power of the United States” does not extend to “any
suit in law or equity, commenced or prosecuted against one of the United States by Citizens of another
State .” As “one of the United States,” State A is immune from suit unless it agrees to be sued. While this
immunity of States from suits has been described as an “anachronistic survival of monarchical privilege,”
it is nonetheless firmly established. While a state may waive its immunity, there is no evidence that State

Seperac-J19 Exam-Released MEE Essay Compilation © 2016-2020 216


A has done so in this case. Here, the bank, a resident of State B, is suing State A for damages in federal
court; this is barred by the Eleventh Amendment.

[NOTE: A state’s Eleventh Amendment immunity may be abrogated in certain circumstances by


congressional action under Congress’s enforcement powers in the Fourteenth Amendment. There is no
such action by Congress in this case, so the exception is not germane.]

Explanation to Point-Two (30%):

Pursuant to the doctrine of Ex parte Young, a suit against State A’s Superintendent of Banking to enjoin
the enforcement of an allegedly unconstitutional statute is not barred by the Eleventh Amendment.

“Official-capacity actions against state officials for prospective relief are not treated as actions against the
State.” Thus, even when a damages claim against the state is barred under the Eleventh Amendment, a
suit against public officials in their official capacity seeking an injunction may be maintained.

Here, the bank could maintain an action in federal court against State A’s Superintendent of Banking in
her official capacity to enjoin enforcement of an allegedly unconstitutional law.

[NOTE: An examinee might also point out that the federal court would have jurisdiction over this suit
because the bank’s claim raises a federal question.]

Explanation to Point-Three (40%):

Although the statute does not discriminate against interstate commerce, it does impose a significant
burden on interstate commerce. A court could conclude that the law unconstitutionally burdens interstate
commerce if the court determines that the burden imposed is clearly excessive in relation to the purported
benefits.

The Supreme Court of the United States has long held that the Constitution’s grant to Congress of the
power to regulate interstate commerce also limits, by implication, the right of state or local governments
to adopt laws that regulate interstate commerce. This is often referred to as “dormant commerce clause”
analysis. A state law that discriminates against interstate commerce in a way “that operates as a tariff or
trade barrier against out-of-state interests” is subject to strict review and is virtually per se
unconstitutional. A nondiscriminatory state law that imposes an “incidental” burden on interstate
commerce will nonetheless be unconstitutional if the burden it imposes is “clearly excessive in relation to
the putative local benefits.”

The State A law in this case is facially nondiscriminatory. It applies equally to local banks and to banks
from other states. There are also no facts to suggest that it operates in a discriminatory fashion or that it
imposes a heavier burden on out-of-state banks offering services to State A businesses than it imposes on
in-state banks.

Because the law “regulates evenhandedly,” the question is whether it “effectuates a legitimate local public
interest” and whether the burden, if any, is “clearly excessive in relation to the putative local benefits.”

State A plainly has a “legitimate local public interest” in protecting local businesses from the significant
losses that can result from electronic funds transfer fraud. State A’s law seeks to reduce such fraud by
requiring banks to adopt certain security measures that the legislature believes will reduce the risk of such
fraud. The legislature’s judgment that biometric identification is superior to other anti-fraud techniques is

Seperac-J19 Exam-Released MEE Essay Compilation © 2016-2020 217


not a judgment that a court will normally second-guess. State A adopted its law in response to lobbying
by a local business that stands to benefit from the law. But that does not mean that the law does not serve
a legitimate state interest. For example, one court accepted the judgment of a state legislature that a ban
on plastic nonreturnable milk containers served environmental goals despite contrary evidence suggesting
that such a ban would cause continued use of ecologically undesirable paperboard milk containers.
However, it is unclear whether the security measures required by State A produce real and substantial
benefits.

The benefits of the law must be weighed against the burden it imposes. The law burdens interstate
commerce by increasing the expenses of out-of-state banks that wish to offer certain electronic banking
services to State A businesses. Compliance with the law would require the bank to make substantial
changes to its entire electronic banking system at a cost of $50 million. This cost is substantial enough to
deter the bank from offering certain services in State A at all. A court could find that this is a real and
substantial burden placed on interstate commerce.

In sum, if the benefits of the security measures required by State A are substantial enough to justify the
burdens, the statute is constitutional. Otherwise, it is not.

Seperac-J19 Exam-Released MEE Essay Compilation © 2016-2020 218


#049-FEB 2016–MEE Q04: QUESTION FOUR (CONSTITUTIONAL LAW)

State A, a leader in wind energy, recently enacted the “Green Energy Act” (“the Act”).

Section 1 of the Act requires that 50% of the electricity sold by utilities in the state come from
“environmentally friendly energy sources.” Wind energy, which is produced in State A, is classified by
the Act as an “environmentally friendly energy source.” Natural gas, which is not produced in State A, is
not classified by the Act as environmentally friendly. The preamble of the Act contains express findings
that the burning of natural gas releases significant quantities of greenhouse gases into the atmosphere and
requires the diversion of scarce water resources for use in gas-burning thermoelectric plants.

Section 2 of the Act prohibits the Public Service Commission of State A from approving any new coal-
burning power plants in the state, unless it finds that “the construction of the plant is necessary to meet
urgent energy needs of this state.” A public utility in neighboring State B has applied for a permit to build
a coal-burning power plant on property it owns across the border in State A. The Commission has denied
the utility’s application based on its finding that there is no evidence of any urgent energy needs in State
A. The State B utility presented undisputed evidence of severe energy shortages in State B, but the
Commission rejected this evidence as irrelevant to the statutory exception.

Section 3 of the Act requires State A, whenever possible, to buy goods and services only from
“environmentally friendly vendors located within the state.” To qualify as an “environmentally friendly
vendor,” a firm must meet specified standards concerning energy efficiency, chemical use, and use of
recycled materials. A vendor located outside of State A meets all the standards to qualify as an
environmentally friendly vendor. The vendor has sought to sell goods and services to State A. The
relevant State A agencies have refused to purchase from this vendor, pointing out that the Act requires
them to purchase, if possible, only from “environmentally friendly vendors located within the state,” of
which there are several.

There is no federal statute or regulation relevant to this problem.

Which provisions, if any, of the Green Energy Act unconstitutionally burden or discriminate against
interstate commerce? Explain.

Seperac-J19 Exam-Released MEE Essay Compilation © 2016-2020 219


#049: F16-4 MEE: ANSWER: NCBE (CONSTITUTIONAL LAW)

POINT (1) [40%] ISSUE: May a state enact a law that has the effect of favoring an in-state industry
at the expense of an out-of-state industry where there are environmental reasons to favor the in-
state industry? ANSWER: Yes. Section 1 of the Act, which requires utilities to use environmentally
friendly energy sources, is probably valid given that it is not facially discriminatory against out-of-
state energy producers and its discriminatory impact is not in the market being regulated
(generation of electricity), but instead affects another market (natural gas production). Further, the
law appears to satisfy the Pike balancing test, given that its burdens on interstate commerce are not
clearly excessive in light of the putative in-state benefits.

POINT (2) [35%] ISSUE: May a state deny an out-of-state utility a permit to construct a coal-
burning power plant because the plant, although it meets urgent out-of-state energy needs, does not
meet urgent energy needs of the permitting state? ANSWER: No. Section 2 of the Act, as applied by
the Public Service Commission, is likely unconstitutional because it discriminates against out-of-
state consumers by preventing the export of electricity from new coal-burning power plants.
Although the environmental purposes of the law are legitimate, the law is not narrowly tailored to
meet them.

POINT (3) [25%] ISSUE: May a state favor in-state vendors when purchasing goods and services?
ANSWER: Yes. Section 3, even though it discriminates against out-of-state vendors by requiring
the state to prefer in-state vendors, is a valid exercise of the state’s role as a “market participant.”

ANSWER DISCUSSION:

Even when Congress has not acted, the Commerce Clause of the United States Constitution imposes by
negative implication a limitation on state laws that discriminate against or unduly burden interstate
commerce. Section 1 of the Act probably passes constitutional muster under the Commerce Clause. It is
not facially discriminatory because it applies equally to in-state and out-of-state utilities; and it does not
impede the import or export of electricity. The law’s incidental effect of favoring an in-state industry
(wind) at the expense of an out-of-state industry (natural gas production) probably does not trigger strict
scrutiny. Moreover, the burden on interstate commerce is not clearly excessive in light of the in-state
benefits. Section 2 of the Act, as applied, probably does not pass constitutional muster. A state law or
administrative decision that explicitly discriminates against nonresidents violates the Commerce Clause
unless it is narrowly tailored to meet a legitimate, nonprotectionist purpose. The legislative ban on new
coal-burning power plants, with its limited exception when State A has urgent energy needs, is
discriminatory because it blocks the export of coal-produced electricity to other states. Even if reducing
pollution from coal-burning power plants is a legitimate, nonprotectionist purpose, the permit denial
likely fails strict scrutiny because there are other, nondiscriminatory means to accomplish the purpose.
Section 3 of the Act should pass constitutional muster. The Commerce Clause does not limit state action
when the state acts as a “market participant.” Thus, State A may favor in-state vendors when purchasing
goods and services.

ANSWER EXPLANATION:

Explanation to Point-One (40%):

Seperac-J19 Exam-Released MEE Essay Compilation © 2016-2020 220


Section 1 of the Act, which requires utilities to use environmentally friendly energy sources, is probably
valid given that it is not facially discriminatory against out-of-state energy producers and its
discriminatory impact is not in the market being regulated (generation of electricity), but instead affects
another market (natural gas production). Further, the law appears to satisfy the Pike balancing test, given
that its burdens on interstate commerce are not clearly excessive in light of the putative in-state benefits.

State laws that discriminate against out-of-state commerce in favor of in-state commerce – either on their
face or in practical effect – are subject to strict scrutiny and thus a nearly per se rule of invalidity. Even if
not discriminatory, state laws that affect interstate commerce can also be invalidated if the burden on
interstate commerce is clearly excessive in relation to the putative in-state benefits.

Section 1 is not facially discriminatory. Utilities may meet the requirement that 50% of their electricity
come from environmentally friendly energy sources by acquiring electricity from out- of-state wind or
other environmentally friendly energy sources; natural gas does not qualify as an environmentally friendly
energy source regardless of where it is produced.

Section 1, however, may be discriminatory in practical effect because it favors an in-state industry (wind)
over an out-of-state industry (natural gas). For example, the Supreme Court has invalidated state statutes
that impose labeling requirements on out-of-state apple producers, effectively advantaging in-state apple
producers. This discriminatory-impact argument, however, likely fails under Exxon Corp. v. Governor of
Maryland. In Exxon, the Court read the Hunt discriminatory-impact test to apply to a direct impact on
out-of-state firms in the primary market (e.g. apples) regulated by the state. In Exxon, the discriminatory
impact was in a market (e.g. refining) different from the one regulated by the state (e.g. service stations),
and so the state law was not found to be discriminatory. Here, the discriminatory impact of Section 1 is
felt in a market (i.e. natural gas production) different from the one being regulated (i.e. generation of
electricity). For example, the Supreme Court upheld a state law requiring milk to be sold in paper cartons,
even though it favored the in-state paper industry over the out-of-state plastics industry. Although
evidence of protectionist motives (such as statements in the legislative history) might be relevant to
whether the law is discriminatory in practical effect, the facts do not suggest any such motive.

Further, Section 1 does not appear to burden interstate commerce in ways that are clearly excessive in
relation to the putative in-state benefits. There is no indication of an especially significant burden on
interstate commerce. Conversely, the findings of the legislature indicate that the law’s goal is to promote
environmentally friendly energy sources, which could reduce air pollution and generate other significant
local benefits (e.g., less use of water in electricity production).

Explanation to Point-Two (35%):

Section 2 of the Act, as applied by the Public Service Commission, is likely unconstitutional because it
discriminates against out-of-state consumers by preventing the export of electricity from new coal-
burning power plants. Although the environmental purposes of the law are legitimate, the law is not
narrowly tailored to meet them.

Section 2 of the Act, and the Public Service Commission’s denial of a permit for an out-of-state utility’s
coal-burning power plant, are discriminatory on their face. While a general ban on the construction of
coal-burning power plants would not be discriminatory because it would treat resident and nonresident
producers and consumers alike, the State A law creates an exception for the urgent energy needs of state
residents only. Thus, the law treats in-state electricity consumers more favorably than out-of-state
consumers and effectively bans the export of electricity from new in-state coal-burning plants.

Seperac-J19 Exam-Released MEE Essay Compilation © 2016-2020 221


The permit denial here discriminates against out-of-state consumers. If the application had been for the
sale of electricity to meet the urgent needs of consumers in State A, the application could have been
approved. Instead, it was denied because the State B utility only identified the urgent needs of consumers
in State B. The case is analogous to City of Philadelphia v. New Jersey, where the Court invalidated a
New Jersey law prohibiting the disposal of out-of-state waste in New Jersey landfills, effectively
precluding the export of waste disposal services and preferring in-state consumers. In City of
Philadelphia, the Court made clear that it does not matter whether the law has a legitimate environmental
purpose – the state may not use discriminatory means to accomplish it.

Insofar as the law is discriminatory, it is invalid unless it is narrowly tailored to meet a legitimate, non-
protectionist purpose. For example, the Supreme Court upheld a ban on importation of live baitfish
because of threat of parasites introduced into in-state waters. In particular, a law is not narrowly tailored if
there are less discriminatory alternative means to accomplish the state’s purpose. Thus, although reducing
air pollution from coal-burning plants (the apparent reason for Section 2) may be a legitimate, non-
protectionist purpose, the law is not narrowly tailored. There are less discriminatory alternatives that
would better accomplish the state’s objectives, such as: (1) strict environmental regulation of all in-state
coal-burning power plants; (2) an across-the-board ban on all in-state coal-burning power plants (without
any exception); and (3) an exception for such plants for urgent energy needs that does not discriminate
against out-of-state consumers.

Explanation to Point-Three (25%):

Section 3, even though it discriminates against out-of-state vendors by requiring the state to prefer in-state
vendors, is a valid exercise of the state’s role as a “market participant.”

The state may discriminate in favor of residents when buying or selling goods and services because the
state is acting as a “market participant” rather than as a regulator of an economic activity. For example, in
one case, a state-owned cement plant could confine sales to state residents during cement shortage. In
another case, the court held that a state bounty for scrap automobiles can favor in-state processors of
junked vehicles. Thus, even though the out- of-state vendor meets all of State A’s requirements for an
“environmentally friendly” vendor, State A is still entitled to favor in-state vendors over the out-of-state
vendor. As such, State A may limit its purchases to vendors in the state.

Seperac-J19 Exam-Released MEE Essay Compilation © 2016-2020 222


#050-FEB 2015–MEE Q02: QUESTION TWO (CONSTITUTIONAL LAW)

State A, suffering from declining tax revenues, sought ways to save money by reducing expenses and
performing services more efficiently. Accordingly, various legislative committees undertook
examinations of the services performed by the state. One service provided by State A is firefighting. The
legislative committee with jurisdiction over firefighting held extensive hearings and determined that older
firefighters, because of seniority, earn substantially more than younger firefighters but are unlikely to
perform as well as their younger colleagues. In particular, exercise physiologists testified at the
committee’s hearings that, in general, a person’s physical conditioning and ability to work safely and
effectively as a firefighter decline with age (with the most rapid declines occurring after age 50) and that,
as a result, firefighting would be safer and more efficient if the age of the workforce was lowered.

State A subsequently enacted the Fire Safety in Employment Act (the Act). The Act provides that no one
may be employed by the state as a firefighter after reaching the age of 50.

A firefighter, age 49, is employed by State A. He is in excellent physical condition and wants to remain a
firefighter. His work history has been exemplary for the last two decades. Nonetheless, he has been told
that, as a result of the Act, his employment as a firefighter will be terminated when he turns 50 next
month.

The firefighter is considering (a) challenging the Act on the basis that it violates his rights under the
Fourteenth Amendment’s Equal Protection Clause, and (b) lobbying for the enactment of a federal statute
barring states from setting mandatory age limitations for firefighters.

1. Does the Act violate the Equal Protection Clause of the Fourteenth Amendment? Explain.

2. Would Congress have authority under Section Five of the Fourteenth Amendment to enact a
statute barring states from establishing a maximum age for firefighters? Explain.

Seperac-J19 Exam-Released MEE Essay Compilation © 2016-2020 223


#050: F15-2 MEE: ANSWER: NCBE (CONSTITUTIONAL LAW)

POINT (1) [50%] ISSUE: Does the Act violate the Equal Protection Clause of the Fourteenth
Amendment? ANSWER: No. A court would assess the constitutionality of the Act under the
“rational basis” test. Here, the state has a “legitimate interest” in promoting safe and efficient
firefighting, and lowering the retirement age is “rationally related” to achieving this interest. Thus,
a court is likely to conclude that State A has not violated the Equal Protection Clause of the
Fourteenth Amendment.

POINT (2) [50%] ISSUE: Would Congress have authority under Section Five of the Fourteenth
Amendment to enact a statute barring states from establishing a maximum age for firefighters?
ANSWER: No. Because age-based discrimination, in the form of a mandatory retirement age, is not
a plausible constitutional injury, Congress does not have the authority under Section Five of the
Fourteenth Amendment to enact legislation to remedy that injury.

ANSWER DISCUSSION:

The lowest level of equal protection scrutiny – so-called “rational basis” scrutiny – applies when a state
government engages in age-based discrimination. The question is whether the distinctions drawn by the
Act on the basis of age are rationally related to a legitimate governmental purpose. The court is likely to
answer that question in the affirmative because safe and efficient firefighting is a legitimate governmental
purpose and, in light of the legislative committee’s findings, the Act is rationally related to that purpose.
With respect to Congress’s power to enact a statute barring states from establishing a maximum age for
firefighters, Congress has the power to enforce the Fourteenth Amendment pursuant to its powers under
Section Five of the Fourteenth Amendment. However, Congress can exercise its authority only if it does
so in a way that is “congruent and proportional” to any constitutional violation that it may be addressing.
A federal statute prohibiting states from having a mandatory retirement age for firefighters would not
meet that standard. The statute would be unconstitutional because it would ban conduct by a state that is
not itself unconstitutional and that is not related to any constitutional violations by the state.

ANSWER EXPLANATION:

Explanation to Point-One (50%):

A court would assess the constitutionality of the Act under the “rational basis” test. Here, the state has a
“legitimate interest” in promoting safe and efficient firefighting, and lowering the retirement age is
“rationally related” to achieving this interest. Thus, a court is likely to conclude that State A has not
violated the Equal Protection Clause of the Fourteenth Amendment.

The applicable constitutional provision is the Equal Protection Clause of the Fourteenth Amendment,
which states: “Nor shall any State deny to any person within its jurisdiction the equal protection of the
laws.” The allegedly unconstitutional discrimination is age-based discrimination because employees like
the firefighter cannot continue as firefighters once they reach 50 years of age.

The Supreme Court has developed three levels of scrutiny for equal protection claims: strict, intermediate,
and the lowest, “rational basis.” The Court has consistently applied rational basis scrutiny to age-based
classifications. For example, the Supreme Court has upheld a 50-year-old retirement age for state police
and applied rational basis review to such age-based classifications.

Seperac-J19 Exam-Released MEE Essay Compilation © 2016-2020 224


Under the rational basis test, the issues are whether State A has a “legitimate interest” that is served by the
discriminatory classification and whether the means used to achieve this legitimate state interest are
“reasonably related” or “rationally related” to that state interest. The Court generally applies this test with
substantial deference to legislative judgment.

Here, the firefighter will likely argue that State A is violating his right to the “equal protection of the
laws” by depriving him, and other firefighters, of employment solely because they have reached the age
of 50. More specifically, he will argue that he and the other firefighters 50 and older are being forced to
retire without regard to whether they are capable firefighters, an action not taken against those under the
age of 50.

State A will likely argue that lowering the retirement age for firefighters will improve workforce quality,
enhance public safety, and reduce expenses. Because these are “legitimate” state interests, this argument
is likely to succeed.

Given the legitimacy of State A’s objectives, the question then becomes whether a mandatory retirement
at age 50 is reasonably related to attaining those objectives. Although the firefighter may be a qualified
firefighter notwithstanding his age, that is not the relevant question. The question is whether State A has
reason to believe that one’s physical fitness and ability to be a firefighter, in general, decline with age.
The question specifies that the legislature heard evidence from relevant professionals in support of that
position. Hence, the conclusion that a mandatory retirement age would, in general, improve the fitness of
the workforce is reasonable. Under the rational basis test, it is not necessary for the fit between ends and
means to be perfect. The fit merely has to be “reasonable” or “rational.”

The fact that State A may have also enacted the statute to save money does not alter this analysis. One
legitimate purpose to which the lines drawn by the statute are rationally related is sufficient to uphold a
statute under the lenient rational basis test. Because State A has a legitimate governmental purpose for
enacting this statute, and because lowering the retirement age is rationally related to the achievement of
this purpose, a court is likely to conclude that the Act does not violate the Equal Protection Clause of the
Fourteenth Amendment.

Explanation to Point-Two (50%):

Because age-based discrimination, in the form of a mandatory retirement age, is not a plausible
constitutional injury, Congress does not have the authority under Section Five of the Fourteenth
Amendment to enact legislation to remedy that injury.

Congress’s powers are limited to those expressed or implied in the Constitution. To enact a law on a
particular topic, Congress must rely on some identified grant of legislative authority in the Constitution.
Section Five of the Fourteenth Amendment is one such grant of authority.

While a mandatory retirement age for firefighters does not violate the Equal Protection Clause of the
Fourteenth Amendment, “legislation which deters or remedies constitutional violations can fall within the
sweep of Congress’s enforcement power even if in the process it prohibits conduct which is not itself
unconstitutional.” Congress’s power, however, is remedial. It has been given the power “to enforce,” not
the power to determine what constitutes a constitutional violation. In drawing “the line between measures
that remedy or prevent unconstitutional actions and measures that make a substantive change in the
governing law,” the Supreme Court stated that the constitutional question is whether there is a
“congruence and proportionality between the constitutional injury to be prevented or remedied and the
means adopted to that end.” Lacking such a connection, legislation may become substantive in operation

Seperac-J19 Exam-Released MEE Essay Compilation © 2016-2020 225


and effect. This proportionality requirement allows Congress to outlaw conduct that courts likely would
hold unconstitutional under existing judicial precedent. Congress may also outlaw a broader range of
conduct to prevent constitutional violations. But Congress cannot rely on its Fourteenth Amendment
enforcement power to prohibit a kind of behavior that is unlikely to involve a constitutional violation at
all.

Because age is not a suspect classification under the Equal Protection Clause, states may discriminate on
the basis of age without offending the Fourteenth Amendment if the age classification in question is
rationally related to a legitimate state interest. The proposed federal statute would prohibit mandatory
retirement requirements that courts likely would find constitutional. In 2000, the Supreme Court held that
a federal statute generally prohibiting age discrimination by employers (including states) exceeded the
power of Congress to legislate pursuant to Section Five of the Fourteenth Amendment. Indeed,
Congress’s primary goal here would be to outlaw a kind of discrimination that does not violate the
Fourteenth Amendment. The Supreme Court clearly held that Congress cannot, under its Fourteenth
Amendment power, legislate to prohibit constitutional behavior where there is no constitutional injury to
be prevented or remedied. Therefore, a court would likely hold that Congress would not have the power
under Section Five of the Fourteenth Amendment to enact a statute barring age requirements for
firefighters.

[NOTE: This question does not raise any questions about sovereign immunity under the Eleventh
Amendment inasmuch as Congress can abrogate that immunity when it acts pursuant to Section Five of
the Fourteenth Amendment. In addition, this question does not ask whether Congress could pass such a
statute under its Commerce Power. ]

Seperac-J19 Exam-Released MEE Essay Compilation © 2016-2020 226


#051-FEB 2014–MEE Q01: QUESTION ONE (CONSTITUTIONAL LAW)

A city ordinance required each downtown business to install high-powered halogen floodlights that would
illuminate the property owned by that business and the adjoining sidewalks. A study commissioned by the
city estimated that installation of the floodlights would cost a typical business about $1,000, but that
increased business traffic due to enhanced public safety, especially after dark, would likely offset this
cost.

A downtown restaurant applied to the city for a building permit to construct an addition that would
increase its seating capacity. In its permit application, the restaurant accurately noted that its current
facility did not have sufficient seating to accommodate all potential customers during peak hours. The city
approved the permit on the condition that the restaurant grant the city an easement over a narrow strip of
the restaurant’s property, to be used by the city to install video surveillance equipment that would cover
nearby public streets and parking lots. The city based its permit decision entirely on findings that the
increased patronage that would result from the increased capacity of the restaurant might also attract
additional crime to the neighborhood, and that installing video surveillance equipment might alleviate that
problem.

The restaurant has challenged both the ordinance requiring it to install floodlights and the easement
condition imposed on approval of the building permit.

1. Under the Fifth Amendment as applied to the states through the Fourteenth Amendment, is the
city ordinance requiring the restaurant to install floodlights an unconstitutional taking? Explain.

2. Under the Fifth Amendment as applied to the states through the Fourteenth Amendment, is the
city’s requirement that the restaurant grant the city an easement as a condition for obtaining the
building permit an unconstitutional taking? Explain.

Seperac-J19 Exam-Released MEE Essay Compilation © 2016-2020 227


#051: F14-1 MEE: ANSWER: NCBE (CONSTITUTIONAL LAW)

POINT (1) [50%] ISSUE: Is the city ordinance requirement that businesses install floodlights a
taking? ANSWER: No. The ordinance requiring that businesses install floodlights is not a per se
taking under Loretto. It is not a regulatory taking under the Penn Central balancing test because
the cost of compliance with the ordinance may be offset by an expected increase in business and
compliance does not interfere with the business’s primary use of its property as a restaurant.

POINT (2) [50%] ISSUE: Is conditioning the approval of a building permit on the grant of an
easement to install surveillance equipment a taking of property? ANSWER: Yes. The permit
condition may be unconstitutional as an uncompensated taking of property because the city has not
made an individualized determination that the easement condition is roughly proportional to the
impact of the restaurant’s proposed addition.

ANSWER DISCUSSION:

The ordinance requiring businesses to install floodlights is not a per se taking under Loretto, because it
does not force a private landowner to allow a third party to enter and place a physical object on the land.
Here, the city ordinance requires the business – not a third party – to install the floodlights. The ordinance
is likely not a regulatory taking under the Penn Central balancing test. While the ordinance will impose a
cost on business owners, that cost may be offset by the expected increase in business due to the ordinance,
and the ordinance does not appear to interfere with the owner’s primary use of the property as a
restaurant. The permit condition, however, is likely an uncompensated taking of property. While the
condition has an essential nexus with the city’s legitimate interest in promoting public safety, the city has
not made an individualized determination that the easement condition is roughly proportional to the
possibility of increased crime due to the restaurant’s proposed addition. Thus, the permit condition likely
violates the Fifth Amendment as applied to the states through the Fourteenth Amendment.

ANSWER EXPLANATION:

Explanation to Point-One (50%):

The ordinance requiring that businesses install floodlights is not a per se taking under Loretto. It is not a
regulatory taking under the Penn Central balancing test because the cost of compliance with the ordinance
may be offset by an expected increase in business and compliance does not interfere with the business’s
primary use of its property as a restaurant.

The city ordinance requiring a business to install floodlights does not effect a per se taking of the sort
described in Loretto v. Teleprompter Manhattan CATV Corp., because no property is physically taken by
the government and the ordinance does not involve a physical invasion of private property by a third
party.

Even though the ordinance does not constitute an occupation of the property by either the government or a
third party, it is still subject to the three-factor balancing test under Penn Central Transportation Co. v.
City of New York, to determine whether it is a “regulatory taking.” Under Penn Central, a court must
balance (1) “the economic impact of the regulation on the claimant,” (2) “the extent to which the
regulation has interfered with distinct investment-backed expectations,” and (3) “the character of the
governmental action.” Here, each factor weighs against finding that the ordinance is a taking.

Seperac-J19 Exam-Released MEE Essay Compilation © 2016-2020 228


First, the ordinance requirement likely has a minimal economic impact on the restaurant. Compliance with
the ordinance is estimated to cost $1,000, and the city has found that businesses will likely recoup that
cost in increased sales. Also, because the ordinance does not interfere with the operation of the restaurant,
the owner may still earn a reasonable return on its investment in the property.

Second, the ordinance does not interfere with the business’s investment-backed expectations. As in Penn
Central, the challenged law does not interfere with the owner’s “primary expectation” for use of the
property – in Penn Central, as a railroad terminal, and here, as a restaurant. Further, the ordinance does
not prevent the restaurant from expanding to meet the changing business environment.

Third, the character of the government action does not weigh in favor of a taking. While Penn Central
does say that a “physical invasion” is more likely to pose a taking, Loretto suggests that the Court’s main
concern is with physical invasions by third parties. Also, like the landmark law challenged in Penn
Central, the ordinance here “adjusts the benefits and burdens of economic life to promote the common
good.” In Penn Central, the landmark law restricted development of the railroad terminal to promote the
common interest in preserving historic landmarks. Here, the ordinance requires the businesses to install
floodlights to promote the common interest in crime prevention and public safety.

Because the ordinance is clearly a valid exercise of the police power, it satisfies the takings clause’s
public-use requirement.

In sum, all three factors weigh against finding a taking under the Penn Central balancing test.

Explanation to Point-Two (50%):

The permit condition may be unconstitutional as an uncompensated taking of property because the city
has not made an individualized determination that the easement condition is roughly proportional to the
impact of the restaurant’s proposed addition.

In Dolan v. City of Tigard, the Supreme Court set forth the test for determining whether an exaction
imposed by a government in exchange for a discretionary benefit conferred by the government, such as a
condition on the approval of a building permit in this case, constitutes an uncompensated taking under the
Fifth Amendment. The exaction is not a taking if (1) there is an “essential nexus” between the “public
need or burden” to which the proposed development contributes and “the permit condition exacted by the
city,” and (2) the government makes “some sort of individualized determination that the required
dedication is roughly proportional both in nature and extent to the impact of the proposed development.”

Here, the city likely can meet the nexus requirement. In Dolan, the landowner sought to double the size of
its business, which would have increased traffic on nearby roadways. In exchange for approving the
development, the city sought an easement for a bike and pedestrian path. The Court found the required
nexus between the easement and the city’s “attempt to reduce traffic congestion by providing for
alternative means of transportation.” Here, a similar nexus likely exists between the requested easement
and the city’s interest in crime prevention and public safety. Increased patronage and economic activity at
the restaurant might attract additional crime to the area, and the requested easement to install surveillance
equipment would attempt to address that increased crime.

The exaction here, however, may fail the second prong of the Dolan test – that the exaction be roughly
proportional to the anticipated impact of the requested development. As noted, the city in Dolan claimed
that a bike and pedestrian path was needed to offset the increase in traffic due to the proposed doubling of
the business. The Court explained that the government must demonstrate that the additional traffic

Seperac-J19 Exam-Released MEE Essay Compilation © 2016-2020 229


reasonably was related to the requested exaction and that the government must “make some effort to
quantify its findings in support of the dedication for the pedestrian/bicycle pathway beyond the
conclusory statement that it could offset some of the traffic demand generated.” The city simply
speculates that increased patronage of the restaurant “might” increase crime, and that the surveillance
equipment “might” alleviate this increased crime. Because the city has not made “some effort to quantify
its findings” in support of the easement, it has not shown that the burden of the easement is roughly
proportional to the benefits thought to flow from it.

Thus, the exaction appears to be an uncompensated taking of property in violation of the Fifth
Amendment as applied to the states through the Fourteenth Amendment.

Seperac-J19 Exam-Released MEE Essay Compilation © 2016-2020 230


#052-FEB 2013–MEE Q03: QUESTION THREE (CONSTITUTIONAL LAW)

AutoCo is a privately owned corporation that manufactures automobiles. Ten years ago, AutoCo
purchased a five-square-mile parcel of unincorporated land in a remote region of the state and built a large
automobile assembly plant on the land. To attract workers to the remote location of the plant, AutoCo
built apartment buildings and houses on the land and leased them to its employees. AutoCo owns and
operates a commercial district with shops and streets open to the general public. AutoCo named the area
Oakwood and provides security, fire protection, and sanitation services for Oakwood’s residents. AutoCo
also built, operates, and fully funds the only school in the region, which it makes available free of charge
to the children of its employees.

A family recently moved to Oakwood. The father and mother work in AutoCo’s plant, rent an apartment
from AutoCo, and have enrolled their 10-year-old son in Oakwood’s school. Every morning, the students
are required to recite the Pledge of Allegiance while standing and saluting an American flag. With the
approval of his parents, the son has politely but insistently refused to recite the Pledge and salute the flag
at the school on the grounds that doing so violates his own political beliefs and the political beliefs of his
family. As a result of his refusal to say the Pledge, the son has been expelled from the school.

To protest the school’s actions, the father walked into the commercial district of Oakwood. While
standing on a street corner, he handed out leaflets that contained a short essay critical of the school’s
Pledge of Allegiance policy. Some of the passersby who took the leaflets dropped them to the ground. An
AutoCo security guard saw the litter, told the father that Oakwood’s anti-litter rule prohibits leaflet
distribution that results in littering, and directed him to cease distribution of the leaflets and leave the
commercial district. When the father did not leave and continued to distribute the leaflets, the security
guard called the state police, which sent officers who arrested the father for trespass.

1. Did the son’s expulsion from the school violate the First Amendment as applied through the
Fourteenth Amendment? Explain.

2. Did the father’s arrest violate the First Amendment as applied through the Fourteenth
Amendment? Explain.

Seperac-J19 Exam-Released MEE Essay Compilation © 2016-2020 231


#052: F13-3 MEE: ANSWER: NCBE (CONSTITUTIONAL LAW)

POINT (1) [30%] ISSUE: Does AutoCo’s operation of a “company town” result in its actions
counting as those of the state for purposes of constitutional analysis? ANSWER: Yes. AutoCo’s
operation of a company town (including a school) makes it a state actor under the public function
strand of the state action doctrine.

POINT (2) [35%] ISSUE: Does the expulsion of a schoolchild for failure to recite the Pledge of
Allegiance violate the First Amendment as applied through the Fourteenth Amendment?
ANSWER: Yes. The son’s expulsion for failure to recite the Pledge of Allegiance violates the First
Amendment as applied through the Fourteenth Amendment as a compelled expression of political
belief.

POINT (3) [35%] ISSUE: Does the arrest of a pamphleteer in connection with violation of an anti-
littering rule, where the littering is done by the recipients of leaflets distributed by the pamphleteer,
violate the First Amendment as applied through the Fourteenth Amendment? ANSWER: Yes.
Because the father was distributing leaflets in a traditional public forum, his trespass arrest
violated the First Amendment as applied through the Fourteenth Amendment.

ANSWER DISCUSSION:

The First Amendment, as applied through the Fourteenth Amendment, applies only to state action. It does
not typically govern private actors. However, courts have found state action where the private actor has
exercised a “public function,” such as running a privately owned “company town,” as AutoCo has done
here. Thus, First Amendment protections apply. By requiring the son to participate in a mandatory Pledge
of Allegiance ceremony, AutoCo has compelled the expression of political belief in violation of the First
Amendment as applied through the Fourteenth Amendment. The father’s arrest in connection with
breaching the anti-litter rule also violated the First Amendment as applied through the Fourteenth
Amendment. Although state actors can regulate the incidental effects of speech on the public streets on a
content-neutral basis, this power is limited and cannot extend to punishing a distributor of literature
because of littering by third parties.

ANSWER EXPLANATION:

Explanation to Point-One (30%):

AutoCo’s operation of a company town (including a school) makes it a state actor under the public
function strand of the state action doctrine.

The individual rights protections of the Constitution apply only where there is “state action” – either
direct action by the government or some action by a private party that is fairly attributable to the
government. As a general rule, the actions of a private company like AutoCo or of a private school like
the school operated by AutoCo would not constitute state action, and the protections of the Constitution
(in this case the First Amendment) would not apply.

However, there are situations in which the actions of a private actor are attributed to the state. One such
situation is when the private actor undertakes a public function. There are not many bright-line rules in the
Supreme Court’s state action doctrine, but one of them is this: Where a private actor undertakes a “public

Seperac-J19 Exam-Released MEE Essay Compilation © 2016-2020 232


function,” the Constitution applies to those actions. Where a corporation operates a privately owned
“company town” that provides essential services typically provided by a state actor, the public function
doctrine applies and the Constitution binds agents of the town as if they were agents of the government.
Here, AutoCo does more than own the town; it provides security services, fire protection, sanitation
services, and a school. Thus the actions of AutoCo constitute state action and are governed by the
Fourteenth Amendment.

Explanation to Point-Two (35%):

The son’s expulsion for failure to recite the Pledge of Allegiance violates the First Amendment as applied
through the Fourteenth Amendment as a compelled expression of political belief.

As explained in Point One, the First Amendment applies to the school as a state actor.

Although children in public schools (and in schools subject to the First Amendment like the Oakwood
school) have some First Amendment rights, Tinker v. Des Moines Independent Community School
District, schools have greater leeway to regulate the speech of students and teachers than the state would
have outside the school context. However, the Supreme Court has long held that public schools may not
force their students to participate in a flag salute ceremony when it offends the political or religious
beliefs of the students or their families. For example, in prior cases, the Supreme Court has invalidated a
mandatory public school flag salute ceremony and invalidated compelled expression of political belief on
state-issued license plates.

In this case, the school requires its students to participate in a flag salute and Pledge of Allegiance
ceremony and punishes them when they refuse to participate. Pursuant to this policy, the school has
expelled the son. This expulsion violates the First Amendment ban on compelled expression.

Explanation to Point-Three (35%):

Because the father was distributing leaflets in a traditional public forum, his trespass arrest violated the
First Amendment as applied through the Fourteenth Amendment.

As explained in Point One, AutoCo is treated as a state actor. Thus, Oakwood’s commercial district is
treated as government-owned property for purposes of the First Amendment. Thus, the leafleting here is
subject to the First Amendment because it is an expressive activity. When expression takes place on
government-owned property, government regulation of the expression is assessed under the public forum
doctrine. Public streets and sidewalks have long been held to be the classic example of a “traditional
public forum” open to the public for expression. Because the father was distributing leaflets while
standing on a street corner in the commercial district, his expressive activity occurred in a traditional
public forum.

When a state tries to regulate expressive activity in a traditional public forum, it is prohibited from doing
so based on the expressive activity’s content unless its regulation is narrowly tailored to achieve a
compelling governmental interest (“strict scrutiny”). In this case, however, AutoCo is regulating the
father’s expressive activity on the ostensibly neutral ground that his expressive activity has produced litter
and made the street unsightly. When a state tries to regulate expressive activity without regard to its
content, intermediate scrutiny applies. Under intermediate scrutiny, the true purpose of the regulation may
not be the suppression of ideas (if so, then strict scrutiny applies), the regulation must be narrowly tailored
to achieve a significant governmental interest, and it must leave open ample alternative channels for
expressive activity.

Seperac-J19 Exam-Released MEE Essay Compilation © 2016-2020 233


Here, the application of the ordinance to the father will fail for two reasons. First, the Supreme Court has
held that the government’s interest in keeping the streets clean is insufficient to ban leafleting in the
public streets, as the government power to regulate with incidental effects on public sidewalk speech is
very limited. Second, the regulation (a blanket ban on distribution that results in littering) is not narrowly
tailored to protect expression. A narrowly tailored alternative would be prosecution only of people who
litter. Moreover, the effect of the littering rule is likely to be a ban on all leafleting, thus eliminating an
entire class of means of expression. This raises the possibility that there are not “ample alternative
channels of communication” open to the father as required under the Court’s standard of review for
content-neutral regulation of speech.

[NOTE: Some examinees might argue that this is a “time, place, and manner” restriction, and that
AutoCo might have greater latitude to regulate the public sidewalks under this theory. This argument is
incorrect for two reasons. First, the Supreme Court has held that the power to regulate speakers through
littering laws is very limited, for the reasons given and in the cases cited above. But more generally, a
“time, place, and manner” restriction involves the shifting of speech from one time and place to another
or to another manner; here, there is no shifting, but a direct punishment for expressive activity (albeit one
couched in content-neutral terms). In addition, some examinees might read the ordinance to be, in effect,
a total ban on leafleting, since most leafleting will produce some litter. Those examinees might note that
the Court has required total bans on an entire mode of expression to satisfy strict scrutiny and analyze the
father’s prosecution here accordingly. The Supreme Court has held that additional restrictions such as an
absolute prohibition on a particular type of expression will be upheld only if narrowly drawn to
accomplish a compelling governmental interest.]

Seperac-J19 Exam-Released MEE Essay Compilation © 2016-2020 234


#053-JUL 2012–MEE Q03: QUESTION THREE (CONSTITUTIONAL LAW)

Congress recently enacted the Violence at Work Act (the Act).

Title I of the Act provides that an employee who has been injured in the workplace by the violent act of a
coworker has a cause of action for damages against that coworker.

Title II of the Act imposes several duties on employers subject to the Act and creates a cause of action
against employers who do not fulfill those duties. Section 201 provides that all employers, “including all
States, their agencies and subdivisions,” who have more than 50 employees are subject to the Act. Section
202 requires employers subject to the Act to (i) train employees on certain methods of preventing and
responding to workplace violence, (ii) conduct criminal background checks on job applicants, and (iii)
establish a hotline to report workplace violence. Section 203 provides that if an employer subject to the
Act does not fulfill the duties imposed by Section 202, an employee who has been injured by the violent
act of a fellow employee may recover damages from the employer for the harm resulting from that violent
act. Section 204 provides that any action brought pursuant to Section 203 may be brought in federal or
state court and that “if brought in federal court against a State, its agencies or subdivisions, any defense of
immunity under the Eleventh Amendment to the United States Constitution is abrogated.”

The House and Senate committee reports on the Act note that Congress passed the Act under its power to
regulate interstate commerce. To support its use of that power, Congress found that acts of workplace
violence directly interfere with economic activity by causing damage to business property, injury to
workers, and lost work time due to the violent acts and their aftermath. The House report estimated that
total interstate economic activity is diminished by $5 to $10 billion per year as a result of losses
associated with workplace violence.

After the Act’s effective date, an employee of a state agency was injured in the workplace by the violent
act of a disgruntled coworker. The state agency, which has over 100 employees, conceded that it had not
implemented the measures required by Section 202 of the Act. Accordingly, the employee has sued the
state agency in United States District Court to recover damages for the harm caused by the act of
workplace violence. The state agency has moved to dismiss the lawsuit on three grounds: (1) Congress
did not have the power to enact the Act, (2) Congress did not have the power to apply the Act to state
agencies, and (3) the Eleventh Amendment bars the employee’s lawsuit.

1. Is the Act a valid exercise of Congress’s power to regulate interstate commerce? Explain.

2. Assuming that the Act is a valid exercise of Congress’s power, may the Act constitutionally be
applied to state agencies as employers? Explain.

3. Does the Eleventh Amendment bar the employee’s lawsuit in federal court against the state
agency? Explain.

Seperac-J19 Exam-Released MEE Essay Compilation © 2016-2020 235


#053: J12-3 MEE: ANSWER: NCBE (CONSTITUTIONAL LAW)

POINT (1) [50%] ISSUE: Does Congress have authority under the Commerce Clause to regulate
employer precautions against workplace violence? ANSWER: Yes. Congress has power under the
Commerce Clause to regulate workplace violence only if the court concludes that the Act regulates
an economic activity with a substantial aggregate effect on interstate commerce.

POINT (2) [20%] ISSUE: Do federalism principles bar Congress from applying the Act to state
agencies as employers? ANSWER: No. The Act does not violate federalism principles because it
regulates both public and private employers on the same terms.

POINT (3) [30%] ISSUE: Does the Eleventh Amendment bar the employee’s suit against the state?
ANSWER: Yes. The Eleventh Amendment bars the employee’s federal court lawsuit against the
state, and the Act does not validly abrogate that immunity.

ANSWER DISCUSSION:

Under the Commerce Clause, Congress has power to regulate economic activities that substantially affect
interstate commerce. Here, the Act regulates a non-economic aspect of an economic activity (i.e., the
employment relationship) that has a substantial effect ($5 to $10 billion per year) on interstate commerce.
This regulation probably falls within the scope of the Commerce Clause, although at least one case can be
read to suggest the opposite possibility. The Act does not violate federalism principles embodied in the
Tenth Amendment to the Constitution by improperly commandeering states or by regulating state
employers differently than private employers. However, the employee’s federal court lawsuit is barred by
the state agency’s Eleventh Amendment immunity, and the Act cannot abrogate that immunity.

ANSWER EXPLANATION:

Explanation to Point-One (50%):

Congress has power under the Commerce Clause to regulate workplace violence only if the court
concludes that the Act regulates an economic activity with a substantial aggregate effect on interstate
commerce.

In United States v. Lopez, the Supreme Court of the United States clarified that Congress may enact three
types of regulations under the Commerce Clause. First, Congress may regulate the channels of interstate
commerce, which are the pathways through which interstate travel and communications pass. Examples
of the channels include interstate highways and phone lines. Second, Congress may regulate the people
and instrumentalities that work and travel in the channels of interstate commerce. Examples include
people such as airline pilots and flight attendants, as well as the airplanes on which they travel. Third,
Congress may regulate activities that substantially affect interstate commerce.

The Act does not fit within either of the first two Lopez categories. First, the statute applies to any
workplace, regardless of its location, and so it does not narrowly regulate the channels of interstate
commerce. Second, the Act applies to all employees and not only those people or instrumentalities in the
channels of interstate commerce. Consequently, the Act will be valid only if it regulates an activity that
substantially affects interstate commerce.

Seperac-J19 Exam-Released MEE Essay Compilation © 2016-2020 236


The key to satisfying the substantial effects requirement is the threshold determination of whether the
regulated activity is economic or commercial in nature. When Congress regulates an economic or
commercial activity, the Court will uphold the regulation if Congress had a rational basis for concluding
that the class of activities subject to regulation, in the aggregate, has a substantial effect on interstate
commerce. Aggregation on a national scale typically makes this an easy standard to meet. On the other
hand, if the regulated activity is not economic or commercial in nature, the Court will not aggregate to
find a substantial effect, and the standard becomes extremely difficult to meet.

Therefore, the key question is whether violence in the workplace is an economic or commercial activity.
In Morrison, the Court held that Congress exceeded its commerce power by enacting a statute giving a
cause of action to the victims of gender-motivated violence. It therefore can be argued here, as Morrison
held, that acts of violence are not economic or commercial in nature, and thus in applying the substantial
effects test, the court may only measure the effect of the particular act of violence at issue in the suit and
not the aggregate effect of all acts of violence in the workplace.

One could argue that Morrison is distinguishable because the statute at issue here is limited to violence in
the workplace. The workplace is an economic environment, and workplace violence directly impedes
productivity of the workplace. The court therefore should conclude that the statute at issue here is an
economic regulation and thus is within the commerce power of Congress because, based on the facts
given in the problem, Congress had a rational basis for concluding that workplace violence, in the
aggregate, has a substantial effect on interstate commerce.

Explanation to Point-Two (20%):

The Act does not violate federalism principles because it regulates both public and private employers on
the same terms.

In Garcia v. San Antonio Metropolitan Transportation Authority, the Supreme Court held that Congress
may regulate the states on the same terms as private actors. For example, Garcia upheld application of the
federal minimum wage and maximum hour law to both public and private employers. In New York v.
United States, however, the Court held that Congress may not “commandeer” the states to regulate private
conduct. In New York v. United States, the Court struck down a federal statute that commandeered states
to regulate private disposal of low-level hazardous waste.

The Act does not commandeer the state to regulate private conduct. Instead, the Act merely requires both
public and private employers to obey the same federal requirement – to address workplace violence under
the threat of civil liability. It is true that the state, as an employer, must adopt policies and regulations to
implement the Act’s mandates. But Reno v. Condon, clarifies that a federal mandate requiring state
personnel to alter their own activities is not unconstitutional commandeering.

Explanation to Point-Three (30%):

The Eleventh Amendment bars the employee’s federal court lawsuit against the state, and the Act does
not validly abrogate that immunity.

The Eleventh Amendment provides: “The Judicial power of the United States shall not be construed to
extend to any suit in law or equity, commenced or prosecuted against one of the United States by Citizens
of another State, or by Citizens or Subjects of any Foreign State.”

Seperac-J19 Exam-Released MEE Essay Compilation © 2016-2020 237


Despite the text of the Eleventh Amendment, the Supreme Court has interpreted it to bar lawsuits between
a state and one of its own citizens, as well as lawsuits that arise under federal law. Further, this immunity
extends to state agencies. Consequently, the Eleventh Amendment would bar the employee’s lawsuit
against the state agency in federal court, unless the Act validly abrogates the state’s immunity.

A federal statute abrogates Eleventh Amendment immunity if, first, the statute unambiguously asserts that
it does so, and second, Congress enacted the statute under a power that may abrogate Eleventh
Amendment state immunity. Here, the Act satisfies the first requirement because Section 204
unequivocally attempts to abrogate state Eleventh Amendment immunity. The Act fails the second
requirement, however, because Congress did not pass the Act under a grant of power that may abrogate
state immunity. In Seminole Tribe, the Court held that Article 1, section 8 of the Constitution does not
grant Congress power to abrogate state sovereign immunity. (The Supreme Court has held that Congress
can abrogate state immunity when it exercises its powers under amendments that postdate the Eleventh
Amendment. By way of contrast, Section 5 of the Fourteenth Amendment does grant Congress that
power. Because the Act does not validly abrogate the state’s Eleventh Amendment immunity, the District
Court should dismiss the employee’s lawsuit.

Seperac-J19 Exam-Released MEE Essay Compilation © 2016-2020 238


#054-JUL 2011–MEE Q08: QUESTION EIGHT (CONSTITUTIONAL LAW)

There are two nursing schools in State A: Public Nursing School (Public) and Private Nursing School
(Private). Public is an agency of the state government, and all its faculty and staff are state employees.
Private is owned by a private corporation and receives no direct funding from the state. The State A Board
of Education regulates the curriculum of each nursing school and certifies all graduates of the two nursing
schools as eligible to become licensed nurses in State A.

Both Public and Private have a long-standing policy of restricting admission to women. Neither school
has ever admitted a male applicant. There has been general discrimination against women in State A in
the health care field. Historically, however, 95 percent of State A nurses have been female.

A male resident of State A wants to be a nurse. The man first applied to Private and was denied
admission. His rejection letter from Private stated that he was “not eligible to enroll because Private was
established as an all-female institution and does not admit or enroll male students.”

The man next applied to Public and was again denied admission. His letter from Public stated that “you
are not eligible to enroll because Public does not enroll male students. Mindful of the historical
discrimination that women have faced in State A, our state has established Public to remedy this
discrimination and provide opportunities for women who want to work in the growing field of health care
as nurses.” The letter continued, “Because your grades and test scores would have been sufficient to admit
you if you were female, we offer you admission to our new Male Nursing Opportunity Program instead.”

The Male Nursing Opportunity Program allows male residents of State A to become nurses by studying at
a nursing school in an adjacent state. Graduates of the program are certified by the State A Board of
Education as eligible to become licensed nurses in State A. However, the Male Nursing Opportunity
Program facilities are not as modern as those at Public, the faculty is not as experienced, and graduates of
the Male Nursing Opportunity Program do not enjoy the same employment opportunities as graduates of
either Public or Private.

1. Has Private violated the man’s rights under the Equal Protection Clause of the Fourteenth
Amendment? Explain.

2. Has Public violated the man’s rights under the Equal Protection Clause of the Fourteenth
Amendment? Explain.

Seperac-J19 Exam-Released MEE Essay Compilation © 2016-2020 239


#054: J11-8 MEE: ANSWER: NCBE (CONSTITUTIONAL LAW)

POINT (1) [30%] ISSUE: Are the actions of a private nursing school subject to the Equal
Protection Clause of the Fourteenth Amendment if the school receives no direct state funds, but the
state regulates the school and certifies its graduates as eligible to become licensed nurses?
ANSWER: No. Private’s actions are not subject to the Equal Protection Clause because it is not a
state actor.

POINT (2)(a) [40%] ISSUE: Does it violate the Equal Protection Clause of the Fourteenth
Amendment for a state-owned nursing school to deny admission to a male applicant because of his
gender? ANSWER: Yes. Public has violated the Equal Protection Clause because a state college
may not deny admission on the basis of gender absent an “exceedingly persuasive justification.”

POINT (2)(b) [30%] ISSUE: Does it violate the Equal Protection Clause of the Fourteenth
Amendment for a state to have an in-state nursing program reserved for female students, and an
out-of-state program for male students, where the facilities of and employment from the all-female
program are superior? ANSWER: Yes. The Male Nursing Opportunity Program is inconsistent
with the guarantees of equal protection.

ANSWER DISCUSSION:

Private has not violated the man’s rights under the Equal Protection Clause of the Fourteenth Amendment
by denying him admission, because the actions of Private are not the actions of the state for purposes of
the Equal Protection Clause. By contrast, as an arm of the state itself, Public’s actions are state action.
When Public denied the man admission based upon his gender, Public denied him equal protection of the
laws. The creation of the Male Nursing Opportunity Program fails to cure this constitutional violation. As
in United States v. Virginia, if a state creates two separate single-gender educational institutions, the
institutions must (at a minimum) provide equivalent educational experiences. Because in this case the
Male Nursing Opportunity Program is clearly inferior as an educational institution, it fails to pass scrutiny
under the Equal Protection Clause.

ANSWER EXPLANATION:

Explanation to Point-One (30%):

Private’s actions are not subject to the Equal Protection Clause because it is not a state actor.

The Fourteenth Amendment provides that “No State shall deny to any person within its jurisdiction the
equal protection of the laws.” The U.S. Supreme Court has interpreted this language as applying not only
to the states themselves but also to private parties whose actions constitute “state action.” Simply put, in
order for there to be a violation of the Fourteenth Amendment, the allegedly unconstitutional action (here,
denial of admission based upon gender) must be attributable to the state. Actions of private parties are not
typically considered state action, but the actions of otherwise “private” parties can constitute state action
in certain exceptional cases. For instance, private parties have been held to be state actors where (1) they
have performed a traditional public function, (2) there is the enforcement of certain private contracts, (3)
there is joint action or “entanglement” between a state and private actor, or (4) there is state
encouragement of private discrimination. A State normally can be held responsible for a private decision

Seperac-J19 Exam-Released MEE Essay Compilation © 2016-2020 240


only when it has exercised coercive power or has provided such significant encouragement, either overt or
covert, that the choice must in law be deemed to be that of the State.

Private is a private entity and none of the factors described above contribute to a conclusion that its
actions constitute state action. Running a private school or college is not state action. First, Private is not
performing a traditional public function. Second, there has been no judicial enforcement of a private
contract here. Third, it is highly unlikely that a court would hold that mere state regulation of the
curriculum and state certification of graduates is sufficient to constitute entanglement. Fourth, there is no
evidence that the state has directly encouraged Private to discriminate on the basis of gender.

Explanation to Point-Two(a) (40%):

Public has violated the Equal Protection Clause because a state college may not deny admission on the
basis of gender absent an “exceedingly persuasive justification.”

Laws that classify on the basis of gender are typically assessed under heightened scrutiny. Although some
early cases suggested that strict scrutiny was the correct standard, modern cases have settled on
intermediate scrutiny. United States v. Virginia, the U.S. Supreme Court held that state laws that make
classifications on the basis of gender are unconstitutional unless the state can establish an “exceedingly
persuasive justification” for the classification. An “exceedingly persuasive justification” is one that serves
“important governmental objectives” and which does not rely upon outdated or overbroad generalizations
and stereotypes about differences between men and women. The burden of justification is demanding and
rests entirely on the state. The state must at least show that the “challenged classification serves important
governmental objectives and that the discriminatory means employed are substantially related to the
achievement of those objectives.” These objectives must be real in the sense that they are real state
purposes rather than hypothetical justifications for the gender classification. Moreover, gender
classifications must be “substantially related” to the achievement of such important governmental
objectives. “A State can evoke a compensatory purpose to justify an otherwise discriminatory
classification only if members of the gender benefited by the classification actually suffer a disadvantage
related to the classification.”

Because the man has been denied admission to Public solely on the basis of his gender, the action of
Public – clearly the action of the state, as Public is the state nursing school – is presumptively
unconstitutional. The state interest in this case is remedying past discrimination against women – as the
letter states, “mindful of the historical discrimination that women have faced in State A, our state has
established Public to remedy this discrimination and provide opportunities for women who want to work
in the growing field of health care as nurses.” Remedying past discrimination is certainly an important
governmental objective; in the contexts of race and gender classifications, the Supreme Court has upheld
this governmental objective in the face of heightened scrutiny. However, it is unclear whether this quote
from Public’s letter sets forth the actual state purpose. Although there has been gender discrimination in
the field of health care in State A in the past, there is no evidence that there has been discrimination
against women in the nursing field. Indeed, the evidence that 95 percent of nurses have historically been
women suggests that State A is more likely reinforcing outdated stereotypes of women as nurses (rather
than, say, doctors) by restricting Public admissions to women. Finally, if the goal of the admissions policy
is to end discrimination against women in the health care field, it is not clear how the admissions policy
advances this objective.

Explanation to Point-Two(b) (30%):

The Male Nursing Opportunity Program is inconsistent with the guarantees of equal protection.

Seperac-J19 Exam-Released MEE Essay Compilation © 2016-2020 241


In certain cases, a state may treat men and women differently consistent with the equal protection
guarantee and provide separate facilities for each gender (for example, male and female sports teams,
dormitories, and bathroom facilities at state universities). In such cases, moreover, the state must bear the
burden of (1) demonstrating the “exceedingly persuasive justification” for the separate treatment, and (2)
demonstrating that the separate facilities are substantially equivalent. Thus, for example, in United States
v. Virginia, the Supreme Court suggested that the Commonwealth of Virginia could offer all-male and all-
female public military-style education consistent with the Equal Protection Clause. However, because the
new all-female “Leadership Program” in that case provided facilities and career opportunities inferior to
those that the Commonwealth was providing in its established all-male program at the Virginia Military
Institute, the Court concluded that the separate program violated the Fourteenth Amendment.

In this case, because the Male Nursing Opportunity Program is markedly inferior to the all-female Public
program, it is insufficient to satisfy the Equal Protection Clause. Although graduates of the all-male
program are still eligible to become nurses in State A, the two programs are not substantially equivalent.
They differ in both overall quality and the employment opportunities they offer to their graduates. The
facilities of the Male Nursing Opportunity Program are not as modern as those at Public, the faculty is not
as experienced, and graduates of the Male Nursing Opportunity Program do not enjoy the same
employment opportunities as graduates of either Public or Private. For these reasons, the Male Nursing
Opportunity Program is inferior to the regular all-female Public program, and it cannot be offered as a
substitute consistent with Equal Protection.

Seperac-J19 Exam-Released MEE Essay Compilation © 2016-2020 242


#055-JUL 2010–MEE Q04: QUESTION FOUR (CONSTITUTIONAL LAW)

The Church of Peace (the Church) is a religious organization that advocates “peace to everyone.”
Recently, a Church chapter (Chapter) was organized in the town of Homestead. Chapter members decided
to spread the Church’s message to the people of Homestead by handing out leaflets that proclaimed in
bold letters, “PEACE TO ALL!” Chapter members who participated in passing out the leaflets stood on a
public sidewalk and distributed the leaflets to pedestrians. The Chapter members did not block traffic or
take any actions except passing out leaflets and remarking, “Peace to all!”

Many people who took the leaflets threw them onto the sidewalk, and Homestead employees spent several
hours cleaning up these discarded leaflets. Chapter was fined $3,000 under a municipal anti-leafleting
ordinance that prohibits any distribution of leaflets “in or on any public space, including roads, streets,
and sidewalks.” No Chapter member threw leaflets or other litter onto the ground.

Chapter members who attend High School, a public school in Homestead, recently formed the “Church of
Peace Club” (Church Club) to pray together and to do good works. High School has a policy that permits
student groups to meet in High School classrooms after scheduled classes. Under this policy, student
groups must first obtain permission from Principal before using a classroom for a meeting. Pursuant to
this policy, the Chess Club, the Drama Club, and the Future Lawyers Club all use classrooms for after-
school meetings. Church Club officers asked Principal if they could meet in a classroom after school.
Principal denied this request and stated that after-school use of a classroom by Church Club would be “a
violation of the separation of church and state.”

Father, a Chapter member and the parent of a Church Club officer, learned about Principal’s decision and
went to High School to see Principal. Outside Principal’s office was a sign reading “No admittance
without an appointment.” Father, who had no appointment, threw open the closed door and marched into
Principal’s office, interrupting a meeting between Principal and another parent, and told Principal, “Your
policy is unwise and unconstitutional. I believe that you are discriminating against members of my faith.”
Principal asked Father to leave the office until the meeting with the other parent was concluded, but
Father refused. Principal called the police, who forcibly removed Father from Principal’s office.

Father was convicted of trespassing on government property.

Does the First Amendment, as applied to state and local governments through the Fourteenth
Amendment,

1. Preclude Homestead’s enforcement of its anti-leafleting ordinance against Chapter? Explain.

2. Preclude Principal’s denial of Church Club’s request to use classroom space for its meetings?
Explain.

3. Provide grounds to vacate Father’s trespass conviction? Explain.

Seperac-J19 Exam-Released MEE Essay Compilation © 2016-2020 243


#055: J10-4 MEE: ANSWER: NCBE (CONSTITUTIONAL LAW)

POINT (1) [40%] ISSUE: May a municipality prohibit leafleting in a public forum based on an
anti-leafleting rationale? ANSWER: No. Homestead cannot enforce its anti-leafleting ordinance
against Chapter because a municipality must allow speech in a traditional public forum even if
doing so imposes costs on the municipality.

POINT (2) [35%] ISSUE: May a public school deny use of classroom space to a student religious
group when, by making that space available to other student groups, it has created a limited public
forum? ANSWER: No. A public school that makes classroom space available to student groups has
created a limited public forum and may not discriminate as to use of that forum based on the
content of speech. Thus High School cannot deny use of a classroom after school hours to Church
Club.

POINT (3) [25%] ISSUE: May a speaker be convicted for trespassing on government property that
is not a public forum when the government has an interest unrelated to speech in preventing the
trespass? ANSWER: Yes. The First Amendment does not shield Father from a trespass conviction.
First, government can regulate conduct that is not imbued with communicative value. Second, even
if the conduct has communicative value, the state can regulate such conduct in a nonpublic forum if
the state’s interest is important or substantial, and is unrelated to the suppression of expression,
and if the regulation is no greater than necessary to serve the state’s interest. Here, Principal’s
office was a nonpublic forum, Father’s act of trespass was not communicative, and High School had
an important and substantial interest in preventing trespassers from entering Principal’s office.

ANSWER DISCUSSION:

For First Amendment purposes, there are three types of forums: the public forum, the limited public
forum, and the nonpublic forum. A public street or sidewalk is a public forum. In a public forum, the state
may not regulate speech based on its content, nor may it penalize speech based on an anti-leafleting
rationale. Thus, Homestead may not fine Chapter under the anti-leafleting ordinance. When a
governmental entity deliberately opens government property to the public for meetings or other forms of
speech, it creates a limited public forum subject to the same First Amendment requirements as a
traditional public forum. Thus, because High School created a limited public forum, Principal may not
deny Church Club the right to use that forum. In a nonpublic forum, governmental actors may restrict
speech as long as their actions are reasonable and content-neutral. Principal’s office was a nonpublic
forum because it was clearly off-limits to the public except by invitation, and Principal’s actions were
reasonable under the circumstances. Thus, Father has no First Amendment grounds to vacate his trespass
conviction.

ANSWER EXPLANATION:

Explanation to Point-One (40%):

Homestead cannot enforce its anti-leafleting ordinance against Chapter because a municipality must allow
speech in a traditional public forum even if doing so imposes costs on the municipality.

Seperac-J19 Exam-Released MEE Essay Compilation © 2016-2020 244


The First Amendment, as applied to the states and local governments through incorporation in the
Fourteenth Amendment, applies to Homestead’s enforcement of its anti-leafleting ordinance, which is a
state action.

Although the United States Supreme Court once granted local governments broad powers to regulate
speech in streets and parks, for the better part of a century, the Court has treated a public street or
sidewalk as a First Amendment “public forum.” In a public forum, government may regulate speech only
if certain conditions are met. If the regulation is not content-neutral, it must meet the requirements of
strict scrutiny. If the regulation is content-neutral, it must meet the requirements of intermediate scrutiny;
to pass muster under this standard, the regulation must impose only reasonable restrictions on the time,
place, or manner of speech that are narrowly tailored to serve a significant governmental interest, and it
must leave open ample alternative opportunities to engage in speech.

Here, Homestead’s regulation of speech under its anti-leafleting ordinance is content-neutral; the
ordinance forbids one means of communication, leafleting, without regard to the content of the speech.
Thus, Homestead must show that its ordinance passes intermediate rather than strict scrutiny.

The Homestead ordinance is unlikely to pass muster under intermediate scrutiny because Homestead has
not left open an alternative channel for the prohibited speech. Instead of regulating the time, place, and
manner of leafleting, Homestead has banned it entirely. Particularly where a form of speech is
inexpensive and available to all, a governmental entity may not prohibit it altogether.

The fact that the prohibited speech imposes costs upon the governmental entity does not alter this result.
In the seminal case of Schneider v. State of New Jersey, Town of Irvington, the Supreme Court struck
down an anti-leafleting ordinance much like the Homestead ordinance. In so doing, the Court held that
“the purpose to keep the streets clean and of good appearance is insufficient to justify an ordinance which
prohibits a person rightfully on a public street from handing literature to one willing to receive it. Any
burden imposed upon the city authorities in cleaning and caring for the streets as an indirect consequence
of such distribution results from the constitutional protection of the freedom of speech and press.”

Thus, Homestead may not fine Chapter for leafleting.

Explanation to Point-Two (35%):

A public school that makes classroom space available to student groups has created a limited public forum
and may not discriminate as to use of that forum based on the content of speech. Thus High School cannot
deny use of a classroom after school hours to Church Club.

A “limited public forum” (also known as a “designated public forum”) is created when a governmental
entity could close a location to speech but instead opens it to speech. All the rules applicable to a
traditional public forum apply to a limited public forum. Thus, under the First Amendment, as applied to
the states and local governments through incorporation in the Fourteenth Amendment, the government
may not discriminate among speakers based upon the content of their speech.

There is no exception to the requirement of content neutrality when religious speech is at issue. The
Supreme Court has repeatedly held that, in a public forum, religious speech stands on an equal footing
with nonreligious speech, and content-neutral access rules do not violate the Establishment Clause. Many
of the cases have involved student or community groups that wanted to use, for religious speech, limited
public forums created by public schools. In Good News, the case most like this one, the Supreme Court
held that, when a school opens its classrooms for use by student groups, it creates a limited public forum

Seperac-J19 Exam-Released MEE Essay Compilation © 2016-2020 245


and may not deny access to a student religious club that intends to use the classroom for prayer based on
the religious content of that speech. The Court also held that a school that opens its facilities to religious
speech under a content-neutral policy does not violate the Establishment Clause under the standard
established in Lemon v. Kurtzman.

Thus, High School may not deny Church Club use of its classrooms for after-school meetings.

Explanation to Point-Three (25%):

The First Amendment does not shield Father from a trespass conviction. First, government can regulate
conduct that is not imbued with communicative value. Second, even if the conduct has communicative
value, the state can regulate such conduct in a nonpublic forum if the state’s interest is important or
substantial, and is unrelated to the suppression of expression, and if the regulation is no greater than
necessary to serve the state’s interest. Here, Principal’s office was a nonpublic forum, Father’s act of
trespass was not communicative, and High School had an important and substantial interest in preventing
trespassers from entering Principal’s office.

Father was convicted for his physical entry into Principal’s office, not for what he said while there. The
act of marching into Principal’s office – which is the act for which Father was convicted – is not conduct
imbued with communicative value and does not implicate the First Amendment. Thus, Father’s trespass
conviction will likely survive a First Amendment challenge.

Even if Father’s conduct is viewed as conduct imbued with communicative value because he entered
Principal’s office to express an opinion, the conviction would be upheld under United States v. O’Brien.
The test in that case is whether a law that regulates speech-related conduct serves an interest within the
government’s regulatory powers that is important or substantial and that is both unrelated to the
suppression of expression and no greater than necessary to serve the state’s interest. Here, Principal’s
office was clearly not a public forum. The door was closed, a sign announced that there was “no
admittance without an appointment,” and Principal was meeting with another parent at the time that
Father marched in. High School clearly has a legitimate and important interest in prohibiting trespassers
from entering Principal’s office where private conversations about individual students or official school
business may be taking place. High School’s interest is not related to suppression of expression.
Moreover, the means of protecting that interest (prohibiting admittance without appointment) are closely
related to advancement of High School’s interest.

Seperac-J19 Exam-Released MEE Essay Compilation © 2016-2020 246


#056-JUL 2009–MEE Q02: QUESTION TWO (CONSTITUTIONAL LAW)

Debate! is a local cable-TV program devoted to public affairs. Each program features a debate about a
controversial public issue.

One year ago, a state official proposed legislation that would substantially increase tax rates. The proposal
was very controversial, and the producers of Debate! invited two guests, Tax and Anti-Tax, to debate the
proposal.

During a live broadcast, the following exchange took place:

Anti-Tax: Taxes are already much too high. If this ridiculous tax increase goes through, you’ll have a
revolution on your hands!

Tax: Don’t be ridiculous. This tax increase is necessary and affordable.

Anti-Tax: You’re a dishonest imbecile. The people have had enough! I call on all viewers to refuse to pay
this proposed tax. And to make it clear that we mean business, I call on viewers to make Tax pay up. He
lives at 224 Oak Street, right here in town. Let’s show him what a taking really means.

At the conclusion of the program, Anti-Tax was arrested and charged with violating two state laws. One
law, the “Sedition Statute,” prohibits “any person from teaching the duty, necessity, or propriety of crime,
violence, or unlawful acts of terrorism as a means of accomplishing political reform.” The Sedition
Statute has been construed to apply only to advocacy of imminent law-breaking.

The other law, the “Abusive Words Statute,” punishes “directing any abusive word or term at another.”
This statute has not been interpreted by the state courts.

Two days after Anti-Tax’s remarks, an unknown arsonist started a fire that destroyed Tax’s home.

The proposed tax increase has not been enacted.

1. Assuming that Anti-Tax’s statements fall within the scope of the Sedition Statute, what
constitutional arguments can be made against convicting him for violating the statute? Explain.

2. Assuming that Anti-Tax’s statements fall within the scope of the Abusive Words Statute, what
constitutional arguments can be made against convicting him for violating the statute? Explain.

Seperac-J19 Exam-Released MEE Essay Compilation © 2016-2020 247


#056: J09-2 MEE: ANSWER: NCBE (CONSTITUTIONAL LAW)

POINT (1) [50%] ISSUE: Does the First Amendment permit Anti-Tax’s conviction under the state
sedition statute based on advocacy of (a) illegal nonpayment of a future tax increase, or (b) “making
Tax pay up . . . and showing him what a taking really means”? ANSWER: No. Anti-Tax cannot be
convicted for advocating nonpayment of the tax increase because the proposed illegal conduct could
not be performed imminently. It is possible, but not certain, that Anti-Tax could be convicted for
urging viewers to “make Tax pay up” and “show him what a taking really means.”

POINT (2) [50%] ISSUE: Does the First Amendment permit Anti-Tax’s conviction under the
state’s broad abusive-words statute? ANSWER: No. The First Amendment “fighting words”
doctrine does not permit Anti-Tax’s conviction under the Abusive Words Statute.

ANSWER DISCUSSION:

Under the Brandenburg test, a conviction for violation of the Sedition Statute based on inciting illegal
conduct is consistent with the First Amendment if there is advocacy of illegal conduct that is imminent
and likely to occur and if the statute is limited to criminalizing advocacy of such conduct. Anti-Tax’s
advocacy of nonpayment of the proposed tax increase does not meet the “imminent and likely” test
because the proposed tax legislation has not been enacted. It is unclear whether Anti-Tax’s advocacy of
“making Tax pay up and showing him what a taking really means” satisfies the Brandenburg test. Anti-
Tax may not be convicted under the Abusive Words Statute because the statute is overbroad and facially
unconstitutional.

ANSWER EXPLANATION:

Explanation to Point-One (45-55%):

Anti-Tax cannot be convicted for advocating nonpayment of the tax increase because the proposed illegal
conduct could not be performed imminently. It is possible, but not certain, that Anti-Tax could be
convicted for urging viewers to “make Tax pay up” and “show him what a taking really means.”

In Brandenburg v. Ohio, the Supreme Court of the United States held that the First Amendment precludes
the conviction of individuals who incite or advocate breaking the law unless (1) there is advocacy of
illegal conduct and not just an abstract expression of ideas, (2) the advocacy calls for imminent
lawbreaking, and (3) the lawbreaking is likely to occur. Moreover, a person cannot be convicted on the
basis of a statute that does not distinguish between abstract expression of ideas and such advocacy.

In this case, Anti-Tax made two statements. He advocated nonpayment of the proposed tax increase and
he urged viewers to “make Tax pay up” and “show him what a taking really means.”

Anti-Tax’s statement urging nonpayment of a future tax increase does not satisfy the Brandenburg
requirements. Although the statement satisfies the illegality requirement, the imminence requirement
cannot be satisfied because the proposed tax legislation had not been enacted. It also seems doubtful that
the state could show likelihood; it is unclear that the tax increase legislation will ever be passed, and there
is no evidence that anyone will take seriously Anti-Tax’s suggestion.

Seperac-J19 Exam-Released MEE Essay Compilation © 2016-2020 248


It is unclear whether Anti-Tax’s statement urging viewers to “make Tax pay up” and “show him what a
taking really means” meets the Brandenburg requirements for advocacy of imminent lawbreaking. The
statement does not expressly advocate immediate, illegal conduct.

On the other hand, the statement does urge a “taking,” which implies theft or property destruction. This
implication, coupled with the fact that Anti-Tax provided Tax’s home address, suggests that Anti-Tax was
calling for immediate action. Although there is no evidence of the likelihood that a viewer would act on
Anti-Tax’s suggestion, the short time lapse between the broadcast and the arson provides circumstantial
evidence that a viewer did act on it. It is thus possible, but not certain, that Anti-Tax’s statement will be
found to satisfy both Brandenburg requirements.

While the Sedition Statute is not limited on its face to punishing advocacy of imminent lawbreaking, it
has been construed to apply only to such advocacy. Accordingly, overbreadth of the statute is not a bar to
conviction under it.

Explanation to Point-Two (45-55%):

The First Amendment “fighting words” doctrine does not permit Anti-Tax’s conviction under the Abusive
Words Statute.

In Chaplinsky v. New Hampshire, the Supreme Court excluded from the protection of the First
Amendment so-called “fighting words,” i.e., words “which by their very utterance inflict injury or tend to
incite an immediate breach of the peace.” Fighting words are unprotected speech because they play little
or no part in the exposition of ideas.

Speech does not come within the fighting-words doctrine unless it is likely to cause a violent reaction
from others. For example, wearing of a jacket reading “F*ck the Draft” in a courthouse does not
constitute fighting words because it is not likely to cause a violent reaction from others under the
circumstances. Here, it is possible, but not certain, that the state can establish that Anti-Tax’s statement
urging viewers to “make Tax pay up,” etc., was likely to cause a violent reaction.

However, even if the state establishes a likelihood of a violent reaction, Anti-Tax may not be convicted
under the Abusive Words Statute because of its overbreadth. In Gooding v. Wilson, the Court held that a
man who told a police officer “I’ll kill you” and “I’ll choke you to death” could not be punished for
uttering what were clearly fighting words because the statute under which conviction was sought was
overbroad and unconstitutional on its face.

Here, the Abusive Words Statute, which punishes “directing any abusive word or term at another,” is
clearly overbroad. Commentary on matters of public concern is afforded the highest level of First
Amendment protection, and this protection extends to “vehement, caustic, and sometimes unpleasantly
sharp attacks on government and public officials.” A statute which punishes language that is merely rude
or abusive will of necessity reach protected speech. Indeed, Anti-Tax’s statement “You’re a dishonest
imbecile” is a textbook example of rude and abusive speech protected by the First Amendment.

Thus, even if Anti-Tax’s statement urging viewers to “make Tax pay up” represented fighting words that
threatened personal violence against Tax, Anti-Tax’s conviction under this overbroad statute would
violate the First Amendment.

Seperac-J19 Exam-Released MEE Essay Compilation © 2016-2020 249


#057-JUL 2008–MEE Q02: QUESTION TWO (CONSTITUTIONAL LAW)

Lex, a nationally prominent criminal defense attorney, has a weekly television show on CNN, a national
cable television station devoted to legal news. Lex’s show deals with a variety of legal topics. Lex often
uses his show as a platform to argue that adultery should be criminalized. Lex and his wife are both 60
years old.

Scoop is a reporter for News, a nationally distributed newspaper. Scoop received an accurate tip that Lex
was engaging in an adulterous affair at a hotel in State X. Scoop then broke into the hotel through a back
door, an act constituting trespass under generally applicable tort law, and attempted to get into Lex’s hotel
room. Before he could get into the room, however, he was discovered by a hotel employee and escorted
out of the hotel. Later that day, while waiting on a public street outside the hotel, Scoop saw Lex get into
a car with a young woman who was clearly not Lex’s wife. Scoop took a photograph of Lex and the
young woman, who were kissing passionately. Scoop mistakenly thought the woman was Star, a world-
famous actress.

Scoop hurried back to the newspaper and wrote a news story that was published with the photograph in
News the next day. The story stated that Lex was having an affair with Star and that Lex’s adultery was
contrary to the beliefs he advocated on his television show. By coincidence, the next page of the same
edition of News featured a separate story about the premiere of Star’s new movie, correctly stating that
Star had been in State Y for the entirety of the previous day. Scoop honestly believed that the woman in
the photograph was Star, even though most people would have been able to tell from the photograph that
this was not the case.

1. Would the First Amendment preclude liability if Star sued News for libel on the ground that the
news story falsely stated that she was having an affair with Lex? Explain.

2. Would the First Amendment preclude liability if the hotel sued Scoop for trespass? Explain.

3. Would the First Amendment preclude liability if Lex sued News for invasion of privacy,
claiming that the publication of the news story and photograph disclosed the truthful but highly
offensive fact that he had engaged in an extramarital affair? Explain.

Seperac-J19 Exam-Released MEE Essay Compilation © 2016-2020 250


#057: J08-2 MEE: ANSWER: NCBE (CONSTITUTIONAL LAW)

POINT (1) [45%] ISSUE: To what extent does the First Amendment shield a newspaper from
liability in a defamation action for publishing a false statement about a public figure? ANSWER:
Star cannot successfully sue News under a libel theory because the facts do not appear to support a
finding that the reporting was done with actual malice.

POINT (2) [20%] ISSUE: Does the First Amendment shield a reporter from liability in a civil
trespass action for trespassing on private property while investigating a news story on a matter of
public concern? ANSWER: No. There is no First Amendment privilege giving the press immunity
from liability arising under generally applicable law, even when in pursuit of a news story on a
matter of public concern.

POINT (3) [35%] ISSUE: Does the First Amendment shield a newspaper from liability in an
invasion-of-privacy action based on disclosure of private facts in a newspaper story about a matter
of public concern if the reporter did not break the law in obtaining the story? ANSWER: Yes.
News is probably immune from liability for invading Lex’s privacy because the published
information was lawfully obtained and involves a matter of public concern.

ANSWER DISCUSSION:

New York Times v. Sullivan requires plaintiffs who are public figures to prove “actual malice” in order to
receive money damages for defamation. If News was merely negligent in identifying the unknown woman
as Star, Star cannot recover damages. But because of the conflicting stories about Star, News may have
acted with reckless disregard of the truth and, therefore, with actual malice. The First Amendment does
not shield Scoop from liability for trespass under generally applicable tort law; here, because Scoop’s acts
constituted trespass under generally applicable tort law, he is liable for damages. Finally, Lex’s action for
invasion of privacy should fail. Truthful disclosure of private facts is protected by the First Amendment
unless the press did not obtain the information lawfully or the disclosure is not on a matter of public
concern. Here, the published facts were on a matter of public concern because Lex is a public figure and
his hypocrisy about adultery is newsworthy. Because neither News nor Scoop broke the law in obtaining
the photograph, News is not liable to Lex under the First Amendment.

ANSWER EXPLANATION:

Explanation to Point-One (40-50%):

Star cannot successfully sue News under a libel theory because the facts do not appear to support a
finding that the reporting was done with actual malice.

In New York Times v. Sullivan, the United States Supreme Court held that public officials seeking to
recover damages in a defamation action (libel or slander) must prove that the defendant reporter acted
with “actual malice,” defined as “knowledge that the published defamation was false” or “reckless
disregard of whether it was false or not.” Proof of negligent falsehood is insufficient to permit liability for
defamation.

The Supreme Court has extended this standard to public figures who assume “roles of especial
prominence in the affairs of society. Some occupy positions of such persuasive power and influence that

Seperac-J19 Exam-Released MEE Essay Compilation © 2016-2020 251


they are deemed public figures for all purposes. More commonly, under the Gertz test, those classed as
public figures have thrust themselves to the forefront of particular public controversies in order to
influence the resolution of the issues involved.”

Here, Star, a world-famous actress, is undeniably a public figure. She can recover only if she proves that
News acted with actual malice, i.e., that News knew that the woman Scoop photographed was not Star, or
that it acted with reckless disregard with regard to that fact. There is no indication that News acted with
knowledge of falsity; indeed, we are told that Scoop “honestly believed” that Star was Lex’s adulterous
partner. But it is a much closer question whether News acted with “reckless disregard” for the truth.
While logical or factual consistency is not demanded by New York Times, the fact that the same edition
of News reported Star to be in another state shows that News very easily could have discovered Scoop’s
report to be untrue. The fact that “most people” could have seen that Star was not the woman in Scoop’s
photograph also provides some support for recklessness in reporting. These factors probably are
inadequate to show that News acted with reckless disregard for the facts, but it is a close question.

Explanation to Point-Two (15-25%):

There is no First Amendment privilege giving the press immunity from liability arising under generally
applicable law, even when in pursuit of a news story on a matter of public concern.

The First Amendment does not shield the press from liability arising under generally applicable law not
aimed at suppression of free speech. In Cohen v. Cowles Media Co., the United States Supreme Court
stated that “generally applicable laws do not offend the First Amendment simply because their
enforcement against the press has incidental effects on its ability to gather and report the news.
Enforcement of such general laws against the press is not subject to stricter scrutiny than would be
applied to enforcement against other persons or organizations.”

Here, Scoop’s actions in breaking into the hotel constituted trespass under generally applicable law that
does not single out the press for special treatment. Scoop, like any other member of the public, is subject
to tort law that applies to all members of society, even when he is engaged in the journalistic activities of
gathering and reporting the news. Thus, the First Amendment is no shield to liability for him, and the
hotel can collect damages for his trespass.

Explanation to Point-Three (30-40%):

News is probably immune from liability for invading Lex’s privacy because the published information
was lawfully obtained and involves a matter of public concern.

In a series of cases, the United States Supreme Court has held that where a media defendant has lawfully
obtained a private fact, such as the identity of a rape victim, the First Amendment shields the media from
liability as long as the news story involves a matter of public concern. In some jurisdictions, the First
Amendment protection is incorporated directly into the tort rule, and the disclosure of private facts is not
tortious if the facts are “of legitimate concern to the public.”

In this case, the First Amendment would shield News from liability. First, neither News nor its employee
Scoop acted unlawfully in procuring the picture. Scoop took the photograph from a public vantage point
and was not breaking the law in doing so. Indeed, publication of the picture was not really publication of a
“private” fact at all. Lex was kissing the young woman on a public street. Although he was in a private
car, passersby (e.g., Scoop) apparently could observe the kiss. A court might well conclude that Lex had
no reasonable expectation of privacy under the circumstances.

Seperac-J19 Exam-Released MEE Essay Compilation © 2016-2020 252


Moreover, the news story about Lex’s adultery addresses a matter of public concern. Lex is undeniably a
public figure under the Gertz test, as he is a nationally famous lawyer and television personality. In
addition, given the strong position he has taken publicly opposing marital infidelity, evidence that his
personal actions belie his public arguments is relevant to the public debate that Lex has voluntarily thrust
himself into. Accordingly, because the photograph was newsworthy and lawfully obtained, News cannot
be held liable for invasion of privacy.

Seperac-J19 Exam-Released MEE Essay Compilation © 2016-2020 253


CONTRACTS: 15 OF 24 MEE EXAMS: (63%)
#058-JUL 2019–MEE Q04: QUESTION FOUR (CONTRACTS)

On March 1, a contractor and an owner of movie theaters signed an agreement providing that, no later
than August 15, the contractor would install seats in the owner’s new movie theater. The agreed-upon
price was $100,000, which was less than the $150,000 that other similar contractors would charge for the
same work. The agreement required that the owner pay the contractor half the price at the time the work
commenced and the other half at completion. The contractor was willing to do the work for less money
than its competitors because the contractor was new to the area and hoped to build up a positive
reputation.

The agreement further provided that the contractor would start work no later than July 1. On July 1,
before beginning the agreed-upon work, the contractor informed the owner that it would not perform its
obligations under the agreement because it had obtained a more lucrative installation contract elsewhere.
At that point, no payments had been made to the contractor.

The installation of the seats was the last step necessary for the theater to open to the public. The owner,
which had anticipated that the contractor would install the seats by the August 15 deadline, had planned
and widely promoted a film festival for September 1–10 to celebrate the opening of the new movie
theater.

Immediately after learning that the contractor would not install the seats, the owner began to look for a
substitute contractor. Despite diligent efforts, the owner could not find a contractor that would agree to
install the seats by August 15. Eventually, after an extensive search, the owner found a substitute
contractor that agreed to install the seats for $150,000 by September 15. No other contractor could be
found who would agree to install the seats at a lower price or before September 15.

Installation of the seats was completed on September 15, the substitute contractor was paid $150,000, and
the theater opened a few days later. Because the theater had no seats at the time of the film festival
scheduled for September 1–10, the owner canceled the festival.

The owner sued the original contractor for breach of contract, and the parties agreed to a non-jury trial.
The judge has concluded that the contractor’s actions with respect to the seat-installation agreement
constituted a breach of contract by the contractor. In addition, the judge has made the following findings
of fact:

• The contractor was unaware that the owner was planning to hold a film festival when it entered
into the contract.

• The owner would have made a profit of $35,000 if the seats had been installed in the new movie
theater and the film festival had been presented there as scheduled on September 1–10.

• The owner could have relocated the film festival to a nearby college auditorium that was available
September 1–10 and, if this had occurred, the owner would have made a profit of $25,000.

Seperac-J19 Exam-Released MEE Essay Compilation © 2016-2020 254


1. Do the damages recoverable by the owner include $50,000 for the amount paid to the substitute
contractor above the $100,000 price to be paid to the original contractor under the contract?
Explain.

2. May the owner recover for lost profits resulting from the cancellation of the film festival?
Explain.

3. Assuming that the owner is entitled to recover for lost profits resulting from the cancellation of
the film festival, how much should the owner recover? Explain.

Seperac-J19 Exam-Released MEE Essay Compilation © 2016-2020 255


#058: J19-4 MEE: ANSWER: NCBE (CONTRACTS)

POINT (1) [20%] ISSUE: Do the damages recoverable by the owner from the original contractor
include $50,000 for the additional amount paid to the substitute contractor beyond the $100,000
price to be paid to the original contractor under the contract? ANSWER: Yes. The owner is
entitled to recover expectation damages equal to the difference between the price that the owner
paid to the substitute contractor ($150,000) and the price that the owner would have paid the
original contractor ($100,000). That is, the owner is entitled to recover $50,000 from the original
contractor.

POINT (2)(a) [40%] ISSUE: Can the owner recover from the original contractor as consequential
damages lost profits from the canceled film festival even though the contractor was unaware of the
owner’s plan to hold the film festival? ANSWER: No. It is unlikely that the owner is entitled to
recover the loss it suffered as a consequence of the cancellation of the film festival (consequential
damages). That loss would be recoverable if the original contractor had reason, at the time the
contract was made, to foresee it as a probable result of the breach. Inasmuch as the court has found
that the contractor was unaware that the owner was planning to hold a film festival when it entered
into the contract, the loss would be recoverable only if the original contractor nonetheless had
reason to know of the film festival.

POINT (2)(b) [40%] ISSUE: Assuming that the owner can recover consequential damages resulting
from the canceled film festival, does the owner’s failure to reduce those damages by relocating the
film festival rather than canceling it result in lower consequential damages? ANSWER: Even if the
cancellation of the film festival was reasonably foreseeable, it is unclear whether the owner would
be entitled to recover the $35,000 in lost profits resulting from the cancellation. If holding the film
festival at the college auditorium was a reasonable substitute for holding it at the new movie
theater, the owner will not be able to recover that portion of the loss that could have been averted
by holding the film festival at the college auditorium. If, on the other hand, holding the film festival
at the college auditorium was not a reasonable substitute because a primary purpose of the film
festival was to showcase the new movie theater, the failure of the owner to relocate the film festival
will not reduce the owner’s recovery for lost profits.

ANSWER DISCUSSION:

Damages for breach of contract are typically based on the aggrieved party’s expectation interest—how
much worse off the aggrieved party is than if the breaching party had fulfilled its obligation under the
contract. In this case, the owner would have had to pay the contractor $100,000 to install the seats but,
because the contractor breached its contract, had to pay the substitute contractor $150,000 for that task.
Thus, the owner is $50,000 worse off than it would have been if the contractor had satisfied its obligation.
As a result, the owner’s “direct damages” are $50,000. Whether the owner is also entitled to consequential
damages arising from the cancellation of the film festival depends on whether the original contractor had
reason to foresee the cancellation as a probable result of its breach. Here, the original contractor did not
know about the scheduled film festival, and there is no indication in the facts that the original contractor
had reason to know of it. Thus, it is unlikely that the owner will be able to recover damages arising from
the cancellation of the film festival. Even if the original contractor had reason to know of the owner’s
plans for the film festival, the owner might not be entitled to recover its $35,000 in lost profits as
consequential damages. If the college auditorium was a reasonable substitute location for the film festival,

Seperac-J19 Exam-Released MEE Essay Compilation © 2016-2020 256


the owner could have avoided $25,000 of the $35,000 in lost profits and thus would be entitled to recover
only the $10,000 that would have been lost had the film festival been held at the college auditorium. But
the college auditorium may not have been a reasonable substitute. An apparent reason for the film festival
was to showcase the new theater, and moving the film festival to the college auditorium would not then
have been a reasonable substitute, so the fact that the owner did not move the film festival there would not
reduce the recovery for lost profits.

ANSWER EXPLANATION:

Explanation to Point One (20%):

The owner is entitled to recover expectation damages equal to the difference between the price that the
owner paid to the substitute contractor ($150,000) and the price that the owner would have paid the
original contractor ($100,000). That is, the owner is entitled to recover $50,000 from the original
contractor.

Generally, the party injured by a breach of contract has a right to damages based on the party’s
expectation interest. Such damages are intended to give him the benefit of his bargain by awarding him a
sum of money that will, to the extent possible, put him in as good a position as he would have been in had
the contract been performed. Here, the contractor promised to install the seats by August 15 for $100,000
and did not do so. No other contractor could be found who would agree to install the seats by August 15;
the substitute contractor, though, agreed to do the installation later for $150,000. Thus, to put the owner,
to the extent possible, in as good a position as it would have been in had the contract been performed (the
owner would have paid $100,000 and the seats would have been installed) requires an award to the owner
of $50,000. This is because use of the substitute contractor resulted in the owner spending $150,000 for
seat installation instead of $100,000. Thus, the owner is $50,000 worse off than if the original contractor
had performed under the contract, so it will take a payment of $50,000 to the owner to put it in “as good a
position as it would have been in had the contract been performed.”

Explanation to Point Two(a) (40%):

It is unlikely that the owner is entitled to recover the loss it suffered as a consequence of the cancellation
of the film festival (consequential damages). That loss would be recoverable if the original contractor had
reason, at the time the contract was made, to foresee it as a probable result of the breach. Inasmuch as the
court has found that the contractor was unaware that the owner was planning to hold a film festival when
it entered into the contract, the loss would be recoverable only if the original contractor nonetheless had
reason to know of the film festival.

In addition to the expectation damages described in Point One, the owner may seek to recover
consequential damages arising from the breach. However, under the rule announced in the landmark case
of Hadley v. Baxendale, as currently applied in almost every state, such damages are not recoverable for
loss that the party in breach did not have reason to foresee as a probable result of the breach when the
contract was made. Loss may be foreseeable as a probable result of a breach if it follows from the breach
either in the ordinary course of events or as a result of special circumstances beyond the ordinary course
of events that the party in breach had reason to know of.

Here, it is unlikely that a court would conclude that cancellation of a film festival would follow in the
ordinary course of events from breach of a contract to install seats in a movie theater by a stated date.
Although the contractor likely would have known that general ticket sales would be lost because of the
delayed opening, there is nothing in the facts to indicate that film festivals or similar events are usually

Seperac-J19 Exam-Released MEE Essay Compilation © 2016-2020 257


scheduled immediately following the construction of a movie theater. It might be argued, however, that
the original contractor would have reason to know that the owner of a new theater would promote its
opening with some sort of special event that could generate significant revenue. Thus, while it might be
argued that the loss from the cancellation of the film festival was foreseeable as following from the breach
in the ordinary course of events, these facts offer little to support that argument.

[NOTE: New York does not follow the rule in Hadley v. Baxendale. Rather, recovery of consequential
damages in New York is limited to liability the defendant fairly may be supposed to have assumed
consciously, or to have warranted the plaintiff reasonably to suppose that it assumed, when the contract
was made. This test is often referred to as the “tacit agreement” test.]

Explanation to Point Two(b) (40%):

Even if the cancellation of the film festival was reasonably foreseeable, it is unclear whether the owner
would be entitled to recover the $35,000 in lost profits resulting from the cancellation. If holding the film
festival at the college auditorium was a reasonable substitute for holding it at the new movie theater, the
owner will not be able to recover that portion of the loss that could have been averted by holding the film
festival at the college auditorium. If, on the other hand, holding the film festival at the college auditorium
was not a reasonable substitute because a primary purpose of the film festival was to showcase the new
movie theater, the failure of the owner to relocate the film festival will not reduce the owner’s recovery
for lost profits.

In addition to foreseeability, consequential damages can be limited for reasons related to “mitigation” or
“avoidability.” As a general rule, a party cannot recover damages for loss that the party could have
avoided by reasonable efforts. Once a party has reason to know that performance by the other party will
not be forthcoming, the party is expected to take such affirmative steps as are appropriate in the
circumstances to avoid loss by making substitute arrangements or otherwise. Thus, the amount of loss that
the aggrieved party could reasonably have avoided by stopping performance, making substitute
arrangements, or otherwise is simply subtracted from the amount that would otherwise have been
recoverable as damages. Affirmative steps to avoid loss are not required, however, if they would involve
undue risk, burden, or humiliation.

Here, the facts indicate that the owner could have reduced its loss by putting on the film festival at a
nearby college auditorium. Because the owner could have avoided some of the loss by moving the
festival, it can be argued that the owner cannot recover for the portion of the loss that could have been
avoided that way. In that case, the owner would be entitled to recover only the $10,000 lost profits that it
would have suffered if it had moved the film festival to the college auditorium. But it might also be
argued that a purpose of the film festival was to showcase the new movie theater, in which case moving
the festival to another location would not serve this purpose. “Whether an available alternative transaction
is a suitable substitute depends on all the circumstances, including the similarity of the performance.” In
that case, holding the film festival at the college auditorium would not have been a reasonable substitute
for a film festival at the new theater, and the consequential damages recoverable by the owner would not
be reduced by the amount that could have been saved by moving the film festival to the college
auditorium.

[NOTE: Sometimes the reduction in consequential damages because the aggrieved party did not take
affirmative steps to avoid contractual losses is described as a breach of the party’s “duty to mitigate
damages.” This terminology is misleading because that party has no duty to avoid loss; rather, contract

Seperac-J19 Exam-Released MEE Essay Compilation © 2016-2020 258


law simply provides that the aggrieved party cannot recover losses that could reasonably have been
avoided.]

Seperac-J19 Exam-Released MEE Essay Compilation © 2016-2020 259


#059-JUL 2018–MEE Q02: QUESTION TWO (CONTRACTS)

A homeowner, who knew that his neighbor wanted to buy a lawn mower, called the neighbor and offered
to sell his lawn mower to her for $350. The neighbor replied, “No way! That price is too high.” The
homeowner responded, “The price is a good one. See if you can find another lawn mower as good as mine
for as little as $350. I’m confident that you’ll come to your senses. In fact, I’m so confident that not only
am I still willing to sell you the lawn mower for $350, but I promise to keep this offer open for a week so
that you have time to do some comparison shopping. If you don’t get back to me within a week, I’ll sell
the lawn mower to someone who knows what a good value it is.”

Four days later, the neighbor concluded that $350 was, indeed, a very good price for the homeowner’s
lawn mower. Accordingly, she decided that she would go see the homeowner the next morning and accept
the offer to buy the lawn mower from him for $350. That evening, the neighbor got a telephone call from
an acquaintance who lived on the same block as the homeowner and the neighbor. The acquaintance said,
“Congratulate me! I just got a great deal on a used lawn mower. [The homeowner] agreed to sell me his
lawn mower for $375. At that price, it’s a steal. I’m picking it up tomorrow afternoon.” The neighbor
replied, “This must be a mistake; he offered to sell that lawn mower to me.” The acquaintance said,
“There’s no mistake; we wrote up the deal and everything. I’ll come by your place right now and show
you the signed contract.” A few minutes later, the acquaintance went to the neighbor’s house and showed
her a signed document pursuant to which the homeowner had agreed to sell his used lawn mower to the
acquaintance for $375.

The neighbor went to the homeowner’s house the first thing the next morning, rang his doorbell, and as
soon as the homeowner came to the door, said, “I accept your offer.” The homeowner replied, “Too late.
I’ve agreed to sell the mower to someone else for $375. Next time, act quickly when you are presented
with such a great bargain.”

The neighbor is furious about the homeowner’s refusal to sell her the lawn mower for $350. In her view,
the homeowner was bound to keep his offer open for a week and, in any event, her statement “I accept
your offer” created a contract that bound the homeowner to the deal.

1. Was the homeowner bound by his promise to keep his offer open for a week? Explain.

2. Assuming that the homeowner was not bound by his promise to keep the offer open, did the
neighbor’s statement “I accept your offer” create a contract with the homeowner for the sale of the
lawn mower? Explain.

Seperac-J19 Exam-Released MEE Essay Compilation © 2016-2020 260


#059: J18-2 MEE: ANSWER: NCBE (CONTRACTS)

POINT (1) [20%] ISSUE: What body of contract law governs this dispute? ANSWER: The lawn
mower constitutes goods. Therefore, the transaction between the homeowner and the neighbor is a
“transaction in goods” and thus governed by Article 2 of the Uniform Commercial Code.

POINT (2) [40%] ISSUE: Is a promise to keep an offer to sell goods open for a week binding when
the offer is made by a nonmerchant and is not supported by consideration? ANSWER: No. The
homeowner’s promise to hold his offer open for one week was not supported by consideration and
thus, under common law contract principles, was not binding. The promise was not otherwise made
enforceable by Article 2 of the UCC.

POINT (3) [40%] ISSUE: Does an expression of acceptance of an offer create a contract when the
offeree is aware that the offeror has taken actions inconsistent with an intention to enter into the
contract? ANSWER: No. A contract would have been formed if the neighbor had accepted the
homeowner’s offer before it was revoked. Here, the offer was probably revoked when the neighbor
learned from the acquaintance that the homeowner had contracted to sell the lawn mower to the
acquaintance.

ANSWER DISCUSSION:

Because the lawn mower constitutes “goods,” the transaction is within the scope of Article 2 of the
Uniform Commercial Code, although common law principles remain applicable to the extent that they are
not displaced by the UCC. The homeowner made an offer to sell the lawn mower to the neighbor and
promised to hold that offer open for a week. That promise was not supported by consideration, and no
exception in UCC Article 2 overrides the requirement of consideration to make that promise binding.
Accordingly, the promise to hold the offer open was not binding. The neighbor’s expression of acceptance
of the homeowner’s offer would have created a contract for the sale of the lawn mower if the offer had not
been revoked. But the offer was probably revoked when the neighbor learned from the acquaintance that
the homeowner had contracted to sell the lawn mower to the acquaintance. Thus, the neighbor’s attempt
to accept the offer was probably too late, and no contract was formed.

ANSWER EXPLANATION:

Explanation to Point One (20%):

The lawn mower constitutes goods. Therefore, the transaction between the homeowner and the neighbor
is a “transaction in goods” and thus governed by Article 2 of the Uniform Commercial Code.

UCC Article 2 governs transactions in goods. Lawn mowers are goods. Thus, the transaction in question
is a transaction in goods that is governed by Article 2 of the UCC. Common law principles remain
applicable, though, to the extent not displaced by the UCC.

Explanation to Point Two (40%):

The homeowner’s promise to hold his offer open for one week was not supported by consideration and
thus, under common law contract principles, was not binding. The promise was not otherwise made
enforceable by Article 2 of the UCC.

Seperac-J19 Exam-Released MEE Essay Compilation © 2016-2020 261


The homeowner’s statement at the beginning of the conversation with the neighbor was an offer to sell
her the lawn mower, and her reply was a rejection of that offer. The homeowner followed up, however, by
renewing the offer and promising to hold it open for a week. Under the common law of contracts, an offer
may be revoked by the offeror at any time before acceptance unless an option contract is created limiting
the power of revocation. Here, however, no option contract was created.

First, there was no consideration for the homeowner’s promise to keep the offer open, and there was no
writing reciting a purported consideration. As a result, the promise to hold the offer open was not
enforceable under the general rule requiring consideration for such a promise.

Second, as the Restatement notes, a promise to hold an offer open may be made binding by statute.
Because the lawn mower constitutes goods, Article 2 of the Uniform Commercial Code is the relevant
statute. In some circumstances, UCC § 2-205 makes a promise to hold open an offer to buy or sell goods
binding, even in the absence of consideration. UCC § 2-205 does not apply in these facts, however. First
of all, it applies only to an offer by a “merchant.” The term “merchant” is defined in the UCC as “a person
who deals in goods of the kind or otherwise by his occupation holds himself out as having knowledge or
skill peculiar to the practices or goods involved in the transaction.” Under these facts, the homeowner is
not a merchant. Second, UCC § 2-205 applies only to an offer made in a signed writing. In this case,
however, the offer by the homeowner was oral. Therefore, the homeowner’s promise to hold the offer
open was not made binding by statute.

In some cases, “an offer which the offeror should reasonably expect to induce action or forbearance of a
substantial character on the part of the offeree before acceptance and which does induce such action or
forbearance is binding as an option contract.” There is nothing in these facts, however, that would justify
application of this rule.

In sum, the homeowner was not bound by his promise to hold the offer open for a week and could revoke
it at any time.

Explanation to Point Three (40%):

A contract would have been formed if the neighbor had accepted the homeowner’s offer before it was
revoked. Here, the offer was probably revoked when the neighbor learned from the acquaintance that the
homeowner had contracted to sell the lawn mower to the acquaintance.

An offeree may accept an offer and thereby create a contract unless the offeree’s power of acceptance has
been terminated. The power of acceptance may be terminated by a rejection or counteroffer by the
offeree, the lapse of time, revocation by the offeror, or death or incapacity of either party. Here, none of
those events is relevant except for revocation by the offeror. (In these circumstances, it does not appear
that four days would constitute a sufficient lapse of time, especially in light of the expressed willingness
of the homeowner to keep the offer open for a week). Therefore, the neighbor’s power of acceptance was
terminated only if the homeowner revoked the offer before the neighbor accepted it.

In most cases, revocation of an offer occurs when the offeree receives from the offeror a manifestation of
intention not to enter into the proposed contract. Here, the neighbor did not receive such a manifestation
of intention directly from the homeowner before she tried to accept his offer. Yet some cases have held
that revocation may also be communicated to the offeree indirectly, when the offeror takes definite action
inconsistent with an intention to enter into the proposed contract and the offeree acquires reliable
information to that effect.

Seperac-J19 Exam-Released MEE Essay Compilation © 2016-2020 262


In this case, the homeowner took definite action inconsistent with an intention to sell the lawn mower to
the neighbor—namely, he entered into a contract to sell it to the acquaintance. Moreover, the neighbor
acquired reliable information that the homeowner did not intend to enter into the proposed contract with
the neighbor. Not only did the acquaintance tell the neighbor about the homeowner’s contract with the
acquaintance, but he also showed her their written agreement. Thus, under this principle of indirect
revocation, the homeowner revoked the offer, terminating the neighbor’s power of acceptance, before her
purported acceptance. As a result, there was no contract.

[NOTE: While the Restatement indicates that indirect revocation is a generally applicable principle (and
provides illustrations to that effect), the Reporter’s Note to Section 43 notes that existing cases all
involved real estate transactions. Accordingly, it could be argued that the homeowner had not revoked his
offer before it was accepted by the neighbor and a contract was therefore created by the acceptance.]

Seperac-J19 Exam-Released MEE Essay Compilation © 2016-2020 263


#060-FEB 2018–MEE Q03: QUESTION THREE (CONTRACTS)

A woman whose hobby was making pottery wanted to improve her pottery skills both for her own
enjoyment and to enable her to create some pottery items that she could sell. Accordingly, she entered
into negotiations with an experienced professional potter about the possibility of an apprenticeship at his
pottery studio.

The negotiations went well, and after some discussion, the woman and the professional potter orally
agreed to the following on May 1:

• The woman would be the potter's apprentice for three months beginning May 15. During the
apprenticeship, the potter would provide education and guidance about the artistry and business of
pottery. The woman would pay the potter $4,000 for the right to serve as the potter's apprentice,
payable on the first day of the apprenticeship.

• The potter would supply the woman with equipment and tools that she would use during the
apprenticeship and would be entitled to take with her at the conclusion of the apprenticeship. On or
before May 8, the woman would pay the potter $5,000 for the equipment and tools.

• The woman would be provided with a private room in the potter's studio in which to stay during
the apprenticeship.

On May 2, the woman and the potter signed a document titled "Memorandum of Agreement." It contained
the terms orally agreed to the day before, except that it did not refer to the woman's living in a private
room in the potter's studio. The last sentence of the document stated, "This is our complete agreement."

On May 8, the woman went to the potter's studio and paid him the $5,000 called for in the agreement for
the equipment and tools. While she was there, the potter said that he had decided that the $4,000 price was
too high for the right to serve as his apprentice and proposed lowering it to $3,500. The woman happily
agreed, and they shook hands on this new arrangement.

On May 15, the woman arrived at the potter's studio to begin the apprenticeship and move into the room
she would occupy during that time. The potter refused to let her move in, however, and said that their deal
did not require him to provide lodging for the woman. When the woman protested that they had agreed to
the lodging arrangement, the potter took the signed Memorandum of Agreement out of his pocket and
pointed out to her that it contained no reference to the woman's living in his studio. He then said, "If it's
not in here, it's not part of the deal."

The woman then said, "At least you were reasonable in agreeing to change the price for the apprenticeship
to $3,500. Saving that extra five hundred dollars means a lot to me."

In response, the potter pointed to the Memorandum of Agreement again and said to the woman, "That's
not what this says. This says that you'll pay me $4,000 today. Even if I agreed to lower the price, I didn't
get anything for that, so why should I be bound by it?"

The woman is quite angry about this turn of events and is considering suing the potter.

Seperac-J19 Exam-Released MEE Essay Compilation © 2016-2020 264


1. If the woman sues the potter about the disputes relating to the apprenticeship, will those disputes
be governed by the common law of contracts or by Article 2 of the Uniform Commercial Code?
Explain.

2. Assuming that the common law of contracts governs, is the oral agreement concerning the
woman's lodging binding on the parties? Explain.

3. Assuming that the common law of contracts governs, is the oral agreement lowering the price for
the apprenticeship binding on the parties? Explain.

Seperac-J19 Exam-Released MEE Essay Compilation © 2016-2020 265


#060: F18-3 MEE: ANSWER: NCBE (CONTRACTS)

POINT (1) [25%] ISSUE: Does Article 2 of the Uniform Commercial Code or the common law of
contracts apply to a dispute under a mixed contract for a transaction in both goods and services?
ANSWER: In this transaction involving both goods and services, the “predominant purpose” of the
contract determines whether Article 2 of the Uniform Commercial Code or the common law of
contracts governs. Here, persuasive arguments can be made that either the goods aspect or the
nongoods aspect predominates.

POINT (2)(a) [30%] ISSUE: When parties have entered into a written agreement which states that
it is their complete agreement, under what circumstances are terms that were orally agreed to
before the written agreement part of the resulting contract? ANSWER: Under the parol evidence
rule, whether an oral agreement relating to lodging is part of the contract will depend on whether
the parties intended the subsequent written agreement to be a complete and exclusive statement of
the terms of the transaction.

POINT (2)(b) [10%] ISSUE: When parties have entered into a written agreement which states that
it is their complete agreement, is a subsequent oral agreement part of the resulting contract?
ANSWER: The parol evidence rule does not prevent a later oral agreement from being part of the
parties’ contract.

POINT (3) [35%] ISSUE: Is an agreement to modify a contract in which one party gives up rights
and gets nothing in return enforceable? ANSWER: No. Under the common law of contracts, the
agreement modifying the parties’ contract would not be binding because it was not supported by
consideration.

ANSWER DISCUSSION:

Contracts for the sale of goods are governed by Article 2 of the Uniform Commercial Code; most other
contracts are governed by the common law of contracts. In a mixed (or hybrid) contract, involving both
goods and nongoods (such as services), courts typically use a “predominant purpose” test to determine
which body of law—the common law of contracts or Article 2— applies to the whole contract. In this
case, it is not clear whether the goods aspect or the nongoods aspect predominates. Good arguments can
be made either way. Whether an oral agreement that predates a written agreement is part of the resulting
contract is governed by the parol evidence rule. Under the common law parol evidence rule, the test is
whether the written agreement is completely integrated (in which case a prior oral agreement that is
within the scope of the written agreement is discharged by it) or only partially integrated (in which case
the prior oral agreement is discharged only if it is inconsistent with the written agreement). Here, a strong
argument can be made that the written agreement is completely integrated in light of the statement in the
agreement that it is the parties’ “complete agreement.” Yet such a statement is not definitive. If the
written agreement is completely integrated, the prior oral agreement about the lodging (which appears to
be within the scope of the written agreement) would be discharged by the written agreement. If the written
agreement is partially integrated, the prior oral agreement about lodging does not appear to be inconsistent
with the written agreement and thus would probably not be discharged. The parol evidence rule does not
apply to the oral agreement lowering the price of the apprenticeship because the rule does not apply to
oral agreements entered into subsequent to a written agreement. There was no consideration supporting
the potter’s agreement to lower the price, so, under the common law, the modification would not be
binding on the potter because of the lack of consideration.

Seperac-J19 Exam-Released MEE Essay Compilation © 2016-2020 266


ANSWER EXPLANATION:

Explanation to Point-One (25%):

In this transaction involving both goods and services, the “predominant purpose” of the contract
determines whether Article 2 of the Uniform Commercial Code or the common law of contracts governs.
Here, persuasive arguments can be made that either the goods aspect or the nongoods aspect
predominates.

Contracts for the sale of goods are governed by Article 2 of the Uniform Commercial Code. Most other
contracts are governed by the common law of contracts. The contract here is mixed (or “hybrid”) in the
sense that it contains both sale-of-goods aspects (the sale of equipment and tools used in the pottery-
making process) and nongoods aspects (the apprenticeship training). In such a hybrid contract, involving
both goods and nongoods, courts typically use a “predominant purpose” test to determine which body of
law applies to the whole contract, rather than dividing the contract into goods and nongoods aspects.

Which aspect predominates here—the sale of the tools and equipment or the apprenticeship training?
Comparing the prices assigned to each aspect of the transaction by the parties, the $5,000 goods portion is
larger than the amount attributed to the services portion (whether the initial $4,000 or the potter’s
agreement to lower it to $3,500), suggesting that the goods aspect predominates and, thus, that Article 2
applies. But it can be argued that, notwithstanding the dollar amounts, the apprenticeship aspect
predominates. After all, the woman’s main interest was to improve her pottery skills through the
apprenticeship, and the purchase of the tools and equipment could seem to be secondary to her main
purpose. If the apprenticeship aspect predominates, the contract is governed by the common law of
contracts.

[NOTE: Good arguments can be made for either conclusion. Credit should be given for understanding of
the criterion (predominant purpose) and thoughtful application of that criterion to the facts, regardless of
which conclusion the examinee reaches. In addition, answers may note that there are some decisions in
which courts have found a contract to be divisible and applied Article 2 to the goods portion and the
common law of contracts to the nongoods portion.]

Explanation to Point-Two(a) (30%):

Under the parol evidence rule, whether an oral agreement relating to lodging is part of the contract will
depend on whether the parties intended the subsequent written agreement to be a complete and exclusive
statement of the terms of the transaction.

Whether the terms of an oral agreement that predates a written agreement are part of the resulting contract
is determined by application of the “parol evidence rule.” The common law parol evidence rule applies if
a contract has been reduced to a writing that is “integrated”; that is, constituting a final expression of one
or more terms of an agreement. This is the case here. Notwithstanding the subsequent modification of the
agreement, it appears that the parties intended the terms stated in the written agreement to be final with
respect to the matters they addressed; thus, the contract was integrated with respect to those terms.

The effect of the integrated writing under the common law parol evidence rule depends on whether the
writing is “completely integrated” or only “partially integrated.” A completely integrated agreement, one
that is adopted by the parties as a complete and exclusive statement of the terms of the agreement,
discharges prior agreements to the extent that they are within its scope. If the writing is only partially

Seperac-J19 Exam-Released MEE Essay Compilation © 2016-2020 267


integrated (i.e., integrated but not completely integrated), it discharges prior agreements only to the extent
that the written agreement is inconsistent with the prior agreement.

Thus, the fate of the oral agreement about lodging depends on whether the writing in this case is a
completely integrated agreement or only partially integrated. If the writing is completely integrated, it will
discharge the oral agreement about lodging so long as that oral agreement is within the scope of the
writing. It is likely that the statement “This is our complete agreement” would lead a court to conclude
that the written agreement is completely integrated “in the absence of credible contrary evidence.” But “a
writing cannot of itself prove its own completeness.” In this regard, the Restatement provides that “wide
latitude must be allowed for inquiry into circumstances bearing on the intention of the parties,” and
approaches to this inquiry vary from state to state.

A court would likely hold that the oral lodging agreement is within the scope of the writing inasmuch as it
deals with arrangements for the apprenticeship. If a court also held that the written agreement is
completely integrated, the written agreement would therefore discharge the oral lodging agreement. If, on
the other hand, the writing is only partially integrated, it discharges prior agreements only to the extent
that it is inconsistent with them. The oral lodging agreement does not appear to be inconsistent with the
written agreement; thus, if the written agreement is held to be only partially integrated, the oral lodging
agreement would not be discharged.

Explanation to Point-Two(b) (10%):

The parol evidence rule does not prevent a later oral agreement from being part of the parties’ contract.

The later oral agreement lowering the price of the apprenticeship is not subject to the parol evidence rule.
Only prior or contemporaneous terms are subject to the parol evidence rule. Thus, the parol evidence rule
does not prevent the later oral agreement from being part of the parties’ contract.

Explanation to Point-Three (35%):

Under the common law of contracts, the agreement modifying the parties’ contract would not be binding
because it was not supported by consideration.

Under the common law, an agreement modifying an existing contract, like other promises, generally must
be supported by consideration. There was no consideration for the potter’s agreement to lower the price of
the apprenticeship. Rather, under the modified arrangement the woman had the same duties to the potter
as before (while getting the benefit of the lower price). Performance of those existing duties (or, as they
are often denominated, “pre-existing duties”) is not consideration supporting the potter’s agreement to
accept a lower price for the apprenticeship than under the original agreement. Nor was anything else
exchanged in return for the potter’s agreement to lower the amount of money to which he was entitled.
While there are exceptions to the consideration requirement for modifications, such as the rule that
consideration is not required if the modification is fair and equitable in view of circumstances not
anticipated by the parties when the contract was made, no exception is relevant here. Thus, under the
common law, the modification would likely not be binding on the potter.

Seperac-J19 Exam-Released MEE Essay Compilation © 2016-2020 268


#061-FEB 2017–MEE Q01: QUESTION ONE (CONTRACTS)

On June 15, a professional cook had a conversation with her neighbor, an amateur gardener with no
business experience who grew tomatoes for home use and to give to relatives. During the conversation,
the cook mentioned that she might be interested in "branching out into making salsa" and that, if she did
branch out, she would need to buy large quantities of tomatoes. Although the gardener had never sold
tomatoes before, he told the cook that, if she wanted to buy tomatoes for salsa, he would be willing to sell
her all the tomatoes he grew in his half-acre home garden that summer for $25 per bushel.

Later on June 15, shortly after this conversation, the cook said to the gardener, "I'm very interested in the
possibility of buying tomatoes from you." She then handed a document to the gardener and asked him to
sign it. The document stated, "I offer to sell to [the cook] all the tomatoes I grow in my home garden this
summer for $25 per bushel. I will hold this offer open for 14 days."

The gardener signed the document and handed it back to the cook.

On June 19, the proprietor of a farmers' market offered to buy all the tomatoes that the gardener grew in
his home garden that summer for $35 per bushel. The gardener, happy about the chance to make more
money, agreed, and the parties entered into a contract for the gardener to sell his tomatoes to the
proprietor.

On June 24, the cook, who had not communicated with the gardener since the June 15 conversation,
called the gardener. As soon as the cook identified herself, the gardener said, "I hope you are not calling
to say that you want my tomatoes. I can't sell them to you because I have sold them to someone else." The
cook replied, "You can't do that. I called to accept your offer to sell me all your tomatoes for $25 per
bushel. You promised to hold that offer open for 14 days. I accept your offer!"

Is the gardener bound to sell the cook all the tomatoes he grows that summer for $25 per bushel? Explain.

Seperac-J19 Exam-Released MEE Essay Compilation © 2016-2020 269


#061: F17-1 MEE: ANSWER: NCBE (CONTRACTS)

POINT (1) [10%] ISSUE: What body of contract law governs the dealings between the gardener
and the cook? ANSWER: Tomatoes are “goods,” and, therefore, the transaction between the cook
and the gardener is governed by Article 2 of the UCC. Further, the document signed by the
gardener was an “offer” for a contract under the UCC.

POINT (2) [60%] ISSUE: Was the gardener entitled to revoke his offer before the cook accepted it?
ANSWER: Yes. Generally, an offer to buy or sell goods may be revoked at any time before
acceptance unless the offeror promised to hold the offer open and that promise is supported by
consideration, or revocation of the offer is barred by UCC § 2-205.

POINT (3) [30%] ISSUE: Did the gardener revoke that offer before the cook accepted it, thereby
preventing the formation of a contract? ANSWER: Yes. The gardener revoked his offer to sell all
his tomatoes to the cook for $25 per bushel when he told her that he could not sell the tomatoes to
her because he had sold them to someone else before the cook told him that she accepted his offer.

ANSWER DISCUSSION:

Because tomatoes are “goods,” the gardener-cook transaction is within the scope of Article 2 of the
Uniform Commercial Code (UCC), although common law principles remain applicable to the extent that
they are not displaced by the UCC. The gardener made an offer to the cook to sell the cook his tomatoes
for $25 per bushel and promised to hold that offer open for 14 days. That promise was not supported by
consideration, however, and it is not binding on the gardener because no exception in the UCC overrides
the requirement of consideration in order to make such a promise binding. The gardener revoked that
offer before the cook accepted it when he informed the cook that he had sold his tomatoes to the
proprietor of the farmers’ market before the cook told the gardener that she accepted the offer. Thus, the
gardener’s revocation is effective and the gardener is not bound to sell the tomatoes to the cook.

ANSWER EXPLANATION:

Explanation to Point-One (10%):

Tomatoes are “goods,” and, therefore, the transaction between the cook and the gardener is governed by
Article 2 of the UCC. Further, the document signed by the gardener was an “offer” for a contract under
the UCC.

UCC Article 2 governs “transactions in goods.” The term “goods” includes, among other things, growing
crops (such as tomatoes). Thus, the transaction in question is a transaction in goods that is governed by
Article 2 of the UCC. Common law principles remain applicable, though, to the extent not displaced by
the UCC.

The document signed by the gardener, which was capable of being accepted, constituted an “offer” for a
contract under UCC § 2-206.

Explanation to Point-Two (60%):

Seperac-J19 Exam-Released MEE Essay Compilation © 2016-2020 270


Generally, an offer to buy or sell goods may be revoked at any time before acceptance unless the offeror
promised to hold the offer open and that promise is supported by consideration, or revocation of the offer
is barred by UCC § 2-205.

A person makes an offer when the person communicates to another a statement of “willingness to enter
into a bargain, so made as to justify” the other person “in understanding that his assent to that bargain is
invited and will conclude it.” Here, the gardener’s statement in the document that he signed was an offer
because it expressed the gardener’s willingness to enter into a bargain in a way that justified the cook in
understanding that assent would create a contract.

Under the common law of contracts, an offer may be revoked by the offeror at any time before acceptance
unless an option contract is created limiting the power of revocation. No option contract is created here
because there was no consideration for the gardener’s promise to keep the offer open, and there was no
writing reciting a purported consideration. As a result, the promise to hold the offer open is not
enforceable under the general rule requiring consideration for such a promise.

As the Restatement notes, a promise to hold an offer open may also be made binding by statute. Because
the tomatoes constitute goods, Article 2 of the Uniform Commercial Code is the relevant statute. In some
circumstances, the “firm offer” rule in UCC § 2-205 makes a promise to hold open an offer to buy or sell
goods binding even in the absence of consideration. UCC § 2-205 does not apply in this situation,
however, because it applies only to an offer by a “merchant.” The term “merchant” is defined in UCC § 2-
104 as “a person who deals in goods of the kind or otherwise by his occupation holds himself out as
having knowledge or skill peculiar to the practices or goods involved in the transaction.” Here, the
gardener is an amateur gardener with no business experience who has never sold tomatoes. Under these
facts, the gardener is not a merchant. As a result, UCC § 2-205 does not make the gardener’s promise to
hold the offer open binding in the absence of consideration.

In some cases, “an offer which the offeror should reasonably expect to induce action or forbearance of a
substantial character on the part of the offeree before acceptance and which does induce such action or
forbearance is binding as an option contract.” There is nothing in these facts, however, that would justify
application of this rule.

In sum, the gardener was not bound by his promise to hold the offer open for 14 days and could revoke it
at any time.

[NOTE: Even if the gardener were a merchant, UCC § 2-205 would not prevent revocation of this offer.
This is because the promise to hold the offer open was on a form supplied by the cook (the offeree) and
UCC § 2-205 requires that, when the term of assurance is on a form supplied by the offeree, that term of
assurance must be “separately signed” by the offeror. There is no indication that the promise to hold the
offer open was separately signed by the gardener (such as by placing his initials next to that sentence).]

[NOTE: Nonetheless, an examinee should receive credit if he or she concludes that the gardener had
become a merchant by virtue of the sale of all of his tomato crop and then goes on to discuss the
gardener’s liability to the cook under a theory of promissory estoppel.]

Explanation to Point-Three (30%):

The gardener revoked his offer to sell all his tomatoes to the cook for $25 per bushel when he told her that
he could not sell the tomatoes to her because he had sold them to someone else before the cook told him
that she accepted his offer.

Seperac-J19 Exam-Released MEE Essay Compilation © 2016-2020 271


An offer is revoked when the offeree “receives from the offeror a manifestation of an intention not to
enter into the proposed contract.” Notice of the offeror’s revocation must be communicated to the offeree
to effectively terminate the offeree’s power to accept the offer. Here, the gardener told the cook that he
could not sell her tomatoes because he had sold them to the proprietor of the farmers’ market. The
gardener’s statement is a clear manifestation of an intention not to enter into the proposed contract and is
a revocation of the gardener’s offer to sell his tomatoes to the cook. The revocation occurred before the
cook accepted the gardener’s offer. Thus, the cook’s power to accept the offer and create the contract was
terminated before her acceptance and there is no contract between them with respect to the tomatoes.

Seperac-J19 Exam-Released MEE Essay Compilation © 2016-2020 272


#062-JUL 2016–MEE Q05: QUESTION FIVE (CONTRACTS)

A homeowner and his neighbor live in houses that were built at the same time. The two houses have
identical exteriors and are next to each other. The homeowner and his neighbor have not painted their
houses in a long time, and the exterior paint on both houses is cracked and peeling. A retiree, who lives
across the street from the homeowner and the neighbor, has complained to both of them that the peeling
paint on their houses reduces property values in the neighborhood.

Last week, the homeowner contacted a professional housepainter. After some discussion, the painter and
the homeowner entered into a written contract, signed by both of them, pursuant to which the painter
agreed to paint the homeowner’s house within 14 days and the homeowner agreed to pay the painter
$6,000 no later than three days after completion of painting. The price was advantageous for the
homeowner because, to paint a house of that size, most professional housepainters would have charged at
least $8,000.

The day after the homeowner entered into the contract with the painter, he told his neighbor about the
great deal he had made. The neighbor then stated that her parents wanted to come to town for a short visit
the following month, but that she was reluctant to invite them. “This would be the first time my parents
would see my house, but I can’t invite them to my house with its peeling paint; I’d be too embarrassed.
I’d paint the house now, but I can’t afford the going rate for a good paint job.”

The homeowner, who was facing cash-flow problems of his own, decided to offer the neighbor a deal that
would help them both. The homeowner said that, for $500, the homeowner would allow the neighbor to
take over the homeowner’s rights under the contract. The homeowner said, “You’ll pay me $500 and take
the contract from me; the painter will paint your house instead of mine, and when he’s done, you’ll pay
him the $6,000.” The neighbor happily agreed to this idea.

The following day, the neighbor paid the homeowner $500 and the homeowner said to her, “The paint
deal is now yours.” The neighbor then invited her parents for the visit that had been discussed. The
neighbor also remembered how annoyed the retiree had been about the condition of her house.
Accordingly, she called the retiree and told him about the plans to have her house painted. The retiree
responded that it was “about time.”

Later that day, the homeowner and the neighbor told the painter about the deal pursuant to which the
neighbor had taken over the contract from the homeowner. The painter was unhappy with the news and
stated, “You can’t change my deal without my consent. I will honor my commitment to paint the house I
promised to paint, but I won’t paint someone else’s house.”

There is no difference in magnitude or difficulty between the work required to paint the homeowner’s
house and the work required to paint the neighbor’s house.

1. If the painter refuses to paint the neighbor’s house, would the neighbor succeed in a breach of
contract action against the painter? Explain.

2. Assuming that the neighbor would succeed in the breach of contract action against the painter,
would the retiree succeed in a breach of contract action? Explain.

Seperac-J19 Exam-Released MEE Essay Compilation © 2016-2020 273


3. If the painter paints the neighbor’s house and the neighbor does not pay the $6,000 contract
price, would the painter succeed in a contract claim against the neighbor? Against the homeowner?
Explain.

Seperac-J19 Exam-Released MEE Essay Compilation © 2016-2020 274


#062: J16-5 MEE: ANSWER: NCBE (CONTRACTS)

POINT (1) [45%] ISSUE: May a contractual right to the performance of services be assigned
without the obligor’s consent, giving the assignee the right to enforce the contract, when the
assignment would not change the obligor’s duty in any material respect? ANSWER: Yes. The
homeowner’s rights against the painter are assignable because the substitution of neighbor for
homeowner does not materially change the painter’s duty or materially increase the burden
imposed on the painter, and would not materially increase the painter’s risk or chance of obtaining
return performance, or materially reduce the value of the contract to him.

POINT (2) [20%] ISSUE: If the obligor does not perform, may a third party who would have
benefitted from performance enforce the contract? ANSWER: No. The retiree is neither an
assignee of the contract nor a third-party beneficiary of the painter’s promise; accordingly, the
retiree cannot enforce the painter’s obligations.

POINT (3) [35%] ISSUE: If the obligor performs the services for the assignee pursuant to the
assigned contract, is the assignee liable for payment? Is the assignor liable for payment? ANSWER:
Yes. The painter’s right to be paid for the completed paint job is enforceable against the delegatee
(the neighbor) and also against the original/delegator (the homeowner).

ANSWER DISCUSSION:

The homeowner’s rights against the painter can be assigned to the neighbor because the substitution of
neighbor for homeowner will not materially change the painter’s duty or increase the painter’s burden.
Because those rights have been assigned, the neighbor can enforce them against the painter. The retiree
cannot enforce those rights because he is not a party to the contract (either initially or by assignment) and
does not qualify as a third-party beneficiary of it. The assignment of rights to the neighbor does not
relieve the homeowner of his payment obligation; if the painter paints the neighbor’s house and the
neighbor does not pay the painter, the painter will have causes of action against both homeowner and
neighbor.

ANSWER EXPLANATION:

Explanation to Point-One (45%):

The homeowner’s rights against the painter are assignable because the substitution of neighbor for
homeowner does not materially change the painter’s duty or materially increase the burden imposed on
the painter, and would not materially increase the painter’s risk or chance of obtaining return
performance, or materially reduce the value of the contract to him.

While contract rights are generally assignable, a contract is not assignable if the assignment (i) would
materially change the duty of the obligor (here, the painter), (ii) would materially increase the burden or
risk imposed on the obligor, (iii) would materially impair the obligor’s chance of obtaining return
performance or materially reduce the value of that return performance to the obligor, (iv) is forbidden by
statute or precluded by public policy, or (v) is validly precluded by contract. In this case, there is no
indication of a statute or public policy that would forbid or preclude the assignment, and no contractual
provision prohibiting assignment. Thus, the only questions are whether the assignment would materially
change the duty of the painter or increase the burden imposed on the painter, and whether the assignment

Seperac-J19 Exam-Released MEE Essay Compilation © 2016-2020 275


would materially impair the painter’s chance of obtaining return performance or materially reduce the
value of that return performance to the painter.

Under these facts, which specify that there is no difference in magnitude or difficulty between the work
required to paint the homeowner’s house and the work required to paint the neighbor’s house, the
assignment from the homeowner to the neighbor does not materially increase the painter’s duty or risk.
The facts specify that the exterior of the neighbor’s house is identical to the exterior of the homeowner’s
house, that both are peeling, and that the labor to paint each house would be comparable in magnitude and
difficulty. Moreover, the neighbor’s house is next door to the homeowner’s house, so no additional travel
burden would be placed on the painter by painting the neighbor’s house rather than the homeowner’s
house. Thus, a court would likely conclude that the assignment from the homeowner to the neighbor
would neither materially change the painter’s duty nor materially increase the burden imposed on the
painter.

There is no indication under these facts that the assignment would materially impair the painter’s chance
of obtaining return performance (the agreed $6,000 payment), or materially reduce the value of the
contract to the painter. None of these factors is present here, particularly in light of the fact that the
assignor (here, the homeowner) remains liable to pay the painter if the painter fulfills his obligation and
the assignee (here, the neighbor) will be liable as well.

All that is generally necessary for an effective assignment is (a) that the assignor manifest his or her intent
to transfer the right to the assignee, without reserving any right to confirm or nullify the transfer, and (b)
that the assignee manifest assent to the assignment. Here, both conditions are satisfied by the conversation
between the homeowner and the neighbor. No action or manifestation of assent is required from the
obligor. No particular form is required for the assignment. With minor exceptions not relevant here, the
relevant manifestations of assent may be made either orally or in writing.

If the painter does not paint the neighbor’s house, the neighbor has a cause of action against the painter.
By virtue of the assignment, the assignor’s right to performance by the obligor is extinguished, and the
assignee acquires a right to such performance.

[NOTE: Some examinees might argue against assignability of the contract because there are inevitable
differences between the paint jobs, such as the relative ease of dealing with the homeowners. If well-
reasoned, such analysis should receive credit.]

Explanation to Point-Two (20%):

The retiree is neither an assignee of the contract nor a third-party beneficiary of the painter’s promise;
accordingly, the retiree cannot enforce the painter’s obligations.

The retiree has no cause of action for the painter’s breach. The homeowner’s rights under the contract
have been assigned to the neighbor, not to the retiree; therefore, the retiree may not enforce the contract as
an assignee.

Moreover, the retiree does not qualify as a third-party beneficiary who can enforce the contract. While the
retiree would benefit from the painter’s performance of his obligations, not all who benefit from
performance of a promise may enforce it. Rather, contract law distinguishes between “incidental”
beneficiaries and “intended” beneficiaries, and only the latter can enforce a promise of which he or she is
not the promisee. The retiree is not an intended beneficiary inasmuch as there is no indication that
benefitting him was in the contemplation of any of the parties when the contract was entered into.

Seperac-J19 Exam-Released MEE Essay Compilation © 2016-2020 276


Explanation to Point-Three (35%):

The painter’s right to be paid for the completed paint job is enforceable against the delegatee (the
neighbor) and also against the original/delegator (the homeowner).

The transaction between the homeowner and the neighbor is not only an assignment to the neighbor of the
homeowner’s rights against the painter, but also a delegation to the neighbor of the homeowner’s
obligation to the painter. This is shown by the fact that the neighbor assented to the homeowner’s idea of
the neighbor paying the painter. An assignment in general terms includes a delegation of the assignor’s
unperformed duties under the contract. As a result, if the painter completes the paint job and is not paid in
accordance with the terms of the contract, then he has a cause of action against the neighbor, even though
the neighbor was not a party to the original contract.

The homeowner’s delegation to the neighbor of the duty to pay the painter does not, however, relieve the
homeowner of that payment responsibility in the absence of the painter’s agreement to the discharge of
the homeowner. As a result, if the painter is not paid in accordance with the terms of the contract, then the
painter retains a cause of action against the homeowner as well.

Seperac-J19 Exam-Released MEE Essay Compilation © 2016-2020 277


#063-JUL 2015–MEE Q03: QUESTION THREE (CONTRACTS)

A seller and a buyer both collect antique dolls as a hobby. Both live in the same small city and are avid
readers of magazines about antique dolls. The seller placed an advertisement in an antique doll magazine
seeking to sell for $12,000 an antique doll manufactured in 1820.

On May 1, the buyer saw the advertisement and telephoned the seller to discuss buying the doll. During
this conversation, the seller and the buyer agreed to a sale of the doll to the buyer for $12,000 and also
agreed that the seller would deliver the doll to the buyer’s house on May 4, at which time the buyer would
pay the purchase price.

The next day, May 2, the buyer changed his mind and decided not to buy the doll. He signed and mailed a
letter to the seller, which stated in relevant part:

I have decided not to buy the 1820 doll that we agreed yesterday you would sell to me.

The seller received the letter on May 3, immediately telephoned the buyer, and said, “I consider your
letter of May 2 to be the final end to our deal. I will sell the doll to someone else and will hold you
responsible for any loss.”

On May 4, the seller received a telephone call from another antique doll collector. The collector had seen
the seller’s advertisement for the doll and expressed interest in buying it. After some discussion, the seller
and the collector agreed to a sale of the doll to the collector for $11,000. Because the collector lived in a
distant part of the state, the agreement provided that the seller, at her expense, would arrange for delivery
of the doll by an express delivery service. The express delivery service that they selected charges $150 for
deliveries of this type. The sale, the method of delivery, and the fee were all commercially reasonable.
The seller acted in good faith in entering into this agreement with the collector.

On May 5, the buyer telephoned the seller and said, “I made a mistake when I sent the letter, and I will
buy the doll from you on the terms we agreed to. Come to my house tomorrow – I’ll have the $12,000 for
you.” The seller replied, “You’re too late. I’ve already sold the doll to someone else.” The seller then took
the doll to the delivery service and paid the $150 delivery fee. The delivery service delivered the doll to
the collector, who immediately wired the $11,000 payment to the seller. Two weeks later, the seller sued
the buyer for breach of contract.

1. Is there a contract for the sale of the doll that is enforceable against the buyer? Explain.

2. Assuming that there is a contract enforceable against the buyer, did the buyer breach that
contract? Explain.

3. Assuming that there is a contract enforceable against the buyer and that the buyer breached that
contract, how much can the seller recover in damages? Explain.

Seperac-J19 Exam-Released MEE Essay Compilation © 2016-2020 278


#063: J15-3 MEE: ANSWER: NCBE (CONTRACTS)

POINT (1) [30%] ISSUE: Is an oral contract for the sale of goods for a price of $500 or more
enforceable against a buyer who later sends a signed letter that indicates that the contract exists?
ANSWER: Yes. The parties entered into a contract for the sale of goods governed by Article 2 of
the Uniform Commercial Code. Because the contract price was $500 or more, the contract would
not be enforceable against the buyer unless the buyer signed a writing sufficient to indicate that a
contract for the sale of goods had been made (or an exception to that rule was applicable). The
buyer’s letter stating his intention not to perform the contract is sufficient to indicate that a
contract was made. Therefore, the contract is enforceable against the buyer.

POINT (2) [40%] ISSUE: If a party sends a letter stating a clear intention not to perform in
accordance with a contract, but later tries to withdraw that statement, may the other party bring
an action for breach of contract? ANSWER: Yes. The buyer repudiated the contract, and his
attempt to retract that repudiation was unsuccessful. Therefore, the seller has a cause of action
against the buyer for breach of the contract.

POINT (3) [30%] ISSUE: What is the measure of damages if a buyer breaches a contract for the
sale of goods and the seller resells the goods? ANSWER: The seller resold the doll and, accordingly,
is entitled to damages equal to the $1,000 difference between the contract price and the resale price
plus incidental damages of $150.

ANSWER DISCUSSION:

The parties’ agreement created a contract for the sale of goods governed by Article 2 of the Uniform
Commercial Code (UCC). Under the statute-of-frauds provision in Article 2 of the UCC, a contract for the
sale of goods for a price of $500 or more is not enforceable unless there is a writing signed by the party
against whom enforcement is sought that is sufficient to indicate that a contract for the sale of goods has
been made between the parties. Although the writing need not state all the terms of the contract, the
quantity must be stated. The buyer’s signed letter to the seller, indicating that he would not buy in
accordance with the agreement between them, is sufficient to indicate that a contract for the sale of goods
was made, and the letter clearly indicates that the agreement was for only one doll, so the quantity
requirement is satisfied. As a result, the contract is enforceable against the buyer. The buyer’s letter was
also a repudiation of the contract. When a party repudiates, the other party may await performance or
immediately resort to any remedy for breach. While a repudiating party may sometimes “retract” its
repudiation, the buyer’s attempt to do so here was unsuccessful because the seller had materially changed
her position in reliance on the repudiation and had indicated that she considered the repudiation to be
final. Because the seller resold the doll to the collector for $1,000 less than the contract price with the
buyer, the seller is entitled to that $1,000 difference plus the additional delivery costs of $150 incurred by
the seller.

ANSWER EXPLANATION:

Explanation to Point-One (30%):

The parties entered into a contract for the sale of goods governed by Article 2 of the Uniform Commercial
Code. Because the contract price was $500 or more, the contract would not be enforceable against the

Seperac-J19 Exam-Released MEE Essay Compilation © 2016-2020 279


buyer unless the buyer signed a writing sufficient to indicate that a contract for the sale of goods had been
made (or an exception to that rule was applicable). The buyer’s letter stating his intention not to perform
the contract is sufficient to indicate that a contract was made. Therefore, the contract is enforceable
against the buyer.

Article 2 of the Uniform Commercial Code applies to transactions in goods. “Goods” are “things
moveable” at the time of identification to the contract. The doll was clearly moveable at the time of
identification, so the transaction is governed by Article 2.

Under UCC Article 2, a contract may be formed in any manner sufficient to show agreement. The
telephone conversation between the seller and the buyer clearly satisfies this requirement and created a
contract to sell the doll to the buyer.

However, a contract for the sale of goods for a price of $500 or more is not enforceable against a party
unless there is a writing signed by that party sufficient to indicate that a contract for sale has been made
(or an exception to this rule applies). Here, the contract price for the doll was $12,000, so this writing
requirement is applicable. The writing need not contain all the terms of the contract, but the contract is not
enforceable against the party beyond the quantity of the goods shown in writing. This rule is generally
interpreted as indicating that a writing does not suffice unless it contains a quantity term.

Although the contract was oral, the letter the buyer sent to the seller on May 2 was signed by the buyer
and is sufficient to indicate that a contract for sale was made between them. After all, it states that the
parties “agreed” to the sale of the doll. As to the quantity requirement, the letter refers to “the” 1820 doll,
clearly indicating that the contract was for the sale of only one doll. Hence, the buyer’s letter to the seller
is sufficient to satisfy the requirements of the Article 2 statute of frauds. Accordingly, the contract is
enforceable by the seller against the buyer.

Explanation to Point-Two (40%):

The buyer repudiated the contract, and his attempt to retract that repudiation was unsuccessful. Therefore,
the seller has a cause of action against the buyer for breach of the contract.

UCC § 2-610 provides that if “either party repudiates the contract with respect to a performance not yet
due the loss of which will substantially impair the value of the contract to the other” the aggrieved party
may for a reasonable time await performance by the repudiating party or resort to any remedy for breach.
Although Article 2 does not define “repudiation,” a comment to UCC § 2-610 states that repudiation
“centers upon an overt communication of intention or an action which renders performance impossible or
demonstrates a clear determination not to continue with performance.”

Here, the buyer’s letter said: “I have decided not to buy the 1820 doll that we agreed yesterday you would
sell to me.” That positive and unequivocal statement that the buyer will not perform “demonstrates a clear
determination not to continue with performance.” In addition, the buyer’s failure to buy the doll would
clearly “substantially impair the value of the contract” to the seller. Thus, the buyer’s action constituted
repudiation and would entitle the seller to pursue remedies for breach.

A repudiating party may, under some circumstances, retract its repudiation. On May 5, the buyer
indicated that he would perform when he said, “I made a mistake when I sent the letter, and I will buy the
doll from you on the terms we agreed to.” But the power to retract a repudiation terminates when the
aggrieved party has done any of the following: (1) cancelled, (2) materially changed his position, or (3)
otherwise indicated that he considers the repudiation to be final.

Seperac-J19 Exam-Released MEE Essay Compilation © 2016-2020 280


Here, the seller materially changed her position in reliance on the buyer’s repudiation when she agreed to
resell the doll to the collector, because the seller committed herself to a substitute transaction inconsistent
with performing the contract with the buyer. Moreover, the seller clearly indicated to the buyer on May 3
that she considered the buyer’s letter to be “the final end” to their deal. Therefore, the buyer lost the
ability to retract the repudiation no later than May 3, two days before the buyer attempted to retract his
repudiation. Hence, the buyer’s attempted retraction of his repudiation was ineffective.

Explanation to Point-Three (30%):

The seller resold the doll and, accordingly, is entitled to damages equal to the $1,000 difference between
the contract price and the resale price plus incidental damages of $150.

The purpose of remedies under the UCC is to put the aggrieved party “in as good a position as if the other
party had fully performed.” This is another way of stating the “expectation” principle of contract
remedies. Here, if the buyer had performed, the seller would have sold the doll to the buyer for $12,000
and would have incurred no additional delivery costs (other than the negligible costs of driving to the
buyer’s home in the same city). After the buyer’s breach, the seller sold the doll for only $11,000 and
incurred an additional cost of $150 to transport the doll to the collector.

Under UCC Article 2, when a buyer breaches or repudiates, the seller has several remedies, including the
remedy of reselling the goods. If the resale is made in good faith and in a commercially reasonable
manner, the seller can recover the difference between the contract price and the resale price plus
incidental and consequential damages. If the resale is by private sale (as opposed to, say, by a public
auction), this remedy is available only if the seller gives the buyer reasonable notification of the seller’s
intention to resell. Here, the facts indicate that when the seller sold the doll to the collector, the seller
acted in good faith and in a commercially reasonable manner. Moreover, the seller told the buyer on May
3 that she intended to resell the doll and hold the buyer responsible for any damages, thus providing the
buyer with notice of the seller’s intention to resell the goods.

Accordingly, the seller is entitled to recover the $1,000 difference between the contract price ($12,000)
and the resale price ($11,000). The seller is also entitled to recover “incidental damages,” which include
“any commercially reasonable charges or expenses incurred in connection with return or resale of the
goods.” Here, the seller incurred $150 in commercially reasonable transportation costs in connection with
the resale of the doll to the collector, and as a result, the seller’s incidental damages would be $150. There
are no facts indicating that the seller suffered any consequential damages. Hence, the seller’s total
damages would be $1,000 + $150, for a total of $1,150.

Seperac-J19 Exam-Released MEE Essay Compilation © 2016-2020 281


#064-JUL 2014–MEE Q02: QUESTION TWO (CONTRACTS)

A music conservatory has two concert halls. One concert hall had a pipe organ that was in poor repair,
and the other had no organ. The conservatory decided to repair the existing organ and buy a new organ for
the other concert hall. After some negotiation, the conservatory entered into two contracts with a business
that both repairs and sells organs. Under one contract, the business agreed to repair the existing pipe organ
for the conservatory for $100,000. The business would usually charge a higher price for a project of this
magnitude, but the business agreed to this price because the conservatory agreed to prepay the entire
amount. Under the other contract, the business agreed to sell a new organ to the conservatory for the other
concert hall for $225,000. As with the repair contract, the business agreed to a low sales price because the
conservatory agreed to prepay the entire amount. Both contracts were signed on January 3, and the
conservatory paid the business a total of $325,000 that day.

Two weeks later, before the business had commenced repair of the existing organ, the business suffered
serious and unanticipated financial reversals. The chief financial officer for the business contacted the
conservatory and said, Bad news. We had an unexpected liability and as a result are in a real cash crunch.
In fact, even though we haven’t acquired the new organ from our supplier or started repair of your
existing organ, we’ve already spent the cash you gave us, and we have no free cash on hand. We’re really
sorry, but we’re in a fix. I think that we can find a way to perform both contracts, but not at the original
prices. If you agree to pay $60,000 more for the repair and $40,000 more for the new organ, we can
probably find financing to finish everything. If you don’t agree to pay us the extra money, I doubt that we
will ever be able to perform either contract, and you’ll be out the money you already paid us.

After receiving this unwelcome news, the conservatory agreed to pay the extra amounts, provided that the
extra amount on each contract would be paid only upon completion of the business’s obligations under
that contract. The business agreed to this arrangement, and the parties quickly signed documents
reflecting these changes to each contract. The business then repaired the existing organ, delivered the new
organ, and demanded payment of the additional $100,000.

The conservatory now has refused to pay the business the additional amounts for the repair and the new
organ.

1. Must the conservatory pay the additional $60,000 for the organ repair? Explain.

2. Must the conservatory pay the additional $40,000 for the new organ? Explain.

Seperac-J19 Exam-Released MEE Essay Compilation © 2016-2020 282


#064: J14-2 MEE: ANSWER: NCBE (CONTRACTS)

POINT (1) [45%] ISSUE: In the case of a service contract (governed by the common law of
contracts), is a modification enforceable when a party agrees to pay more for the same performance
than was originally promised? ANSWER: No. The business probably cannot recover the additional
$60,000 for the organ repair because the conservatory’s promise to pay more money was not
supported by consideration.

POINT (2) [45%] ISSUE: In the case of a contract for the sale of goods (governed by Article 2 of
the UCC), is a modification enforceable when a party agrees to pay more for the same goods than
was originally promised? ANSWER: Yes. The business can recover the additional $40,000 for the
new organ because no consideration is required under Article 2 of the UCC for good-faith contract
modifications.

POINT (3) [10%] ISSUE: May a party avoid an agreement on the basis of economic duress?
ANSWER: No. The conservatory is unlikely to be able to defend against enforcement of its
promises to pay additional money under the theory of economic duress, because the business
probably did not make an improper threat.

ANSWER DISCUSSION:

There are two arguments that the conservatory can make to support the claim that it is not bound to pay
the higher prices: lack of consideration and economic duress. The organ repair contract is governed by the
common law of contracts. Under the common law, the business would have difficulty recovering the
additional $60,000 for the organ repair because, under the “preexisting duty rule,” the agreement of the
conservatory to pay the extra price was not supported by consideration. However, the business might
argue that the modification is enforceable under an exception to the preexisting duty rule for fair and
equitable modifications made in light of unanticipated circumstances. The organ sale contract is governed
by Article 2 of the Uniform Commercial Code. The business would likely recover the additional amount
under that contract because Article 2 provides that consideration is not required for a modification to be
binding. In both cases, the conservatory could seek to avoid its agreement on the grounds of economic
duress, but that argument is not likely to succeed.

ANSWER EXPLANATION:

Explanation to Point-One (45%):

The business probably cannot recover the additional $60,000 for the organ repair because the
conservatory’s promise to pay more money was not supported by consideration.

The general rule is that, to be enforceable, a promise must be supported by consideration. Under the
Second Restatement of Contracts, a promise is supported by consideration if it is bargained for in
exchange for a return promise or performance. However, under the “preexisting duty rule”, promise of
performance of a legal duty already owed to a promisor which is neither doubtful nor the subject of honest
dispute is not consideration.

If the business had promised the conservatory anything new or different in exchange for the agreement to
pay the additional $60,000 (such as, for example, repairing the pipe organ more quickly or using better

Seperac-J19 Exam-Released MEE Essay Compilation © 2016-2020 283


parts), that would constitute consideration, especially in light of the principle that courts do not inquire
into the adequacy of consideration. Here, however, the business already had a legal duty under the
original contract and did not agree to do anything else in exchange for the conservatory’s promise to pay
$60,000 more.

However, an exception to the preexisting duty rule is sometimes applied in situations of unanticipated
changed circumstances. Under the Restatement of Contracts § 89, followed in many jurisdictions, a
promise modifying a duty under a contract not fully performed on either side is binding even if not
supported by consideration, if the modification is fair and equitable in view of circumstances not
anticipated by the parties when the contract was made.

If a court applies the rule in Restatement § 89, the critical issues will be whether the modification was in
fact “fair and equitable” and whether it can be justified in light of unanticipated circumstances. In many
cases in which modifications have been upheld, a party encountered difficulties or burdens in performing
far beyond what was knowingly bargained for in the original contract, with the result bordering on
impracticability, such as having to excavate solid rock instead of soft dirt, or having to remove garbage
far in excess of the amounts contemplated. The conservatory would argue that the business’s performance
difficulties were not of this sort at all – nothing about repairing the pipe organ itself was any different
from or more difficult than originally contemplated, except that the business itself encountered financial
distress unrelated to its burdens in performing its obligations under these contracts.

Even if the business satisfies that element of the rule in Restatement § 89, the business must also
demonstrate that the circumstances that gave rise to the need to modify the contract were “unanticipated”
at the time the original contract was made. Here, the facts suggest that when the business entered into the
original contract, it expected that the price paid by the conservatory would enable it to perform. However,
any evidence that the business knew or had reason to know at the time of execution that it would need
more money from the conservatory to be able to perform would mean that the request to modify was not
“unanticipated.”

[NOTE: Some cases, such as Schwartzreich v. Bauman-Basch, find that if the parties mutually agreed to
rescind the original contract and then, after rescission, entered into an entirely new contract for a higher
price, the new contract is supported by consideration. There is no evidence that such a rescission
followed by a new contract took place here.]

Explanation to Point-Two (45%):

The business can recover the additional $40,000 for the new organ because no consideration is required
under Article 2 of the UCC for good-faith contract modifications.

The contract to buy a new organ is a contract for the sale of goods and therefore is governed by Article 2
of the Uniform Commercial Code. Under Article 2, unlike the common law, an agreement modifying a
contract needs no consideration to be binding. Section 2-209 thus obviates the preexisting duty rule
entirely in contracts for the sale of goods.

Even though consideration is not required, modifications governed by § 2-209 must satisfy the obligation
of good faith imposed by the UCC. Good faith means “honesty in fact and the observance of reasonable
commercial standards of fair dealing.” In this context, the obligation of good faith means that “the
effective use of bad faith to escape performance on the original contract terms is barred, and the extortion
of a ‘modification’ without legitimate commercial reason is ineffective as a violation of the duty of good
faith.” Here, because the business’s financial reversals were serious and apparently unanticipated at the

Seperac-J19 Exam-Released MEE Essay Compilation © 2016-2020 284


time that the business entered into the contract with the conservatory, and commitment of the extra money
was needed to enable the business to perform, a court would likely find that the business acted in good
faith. Thus, a court would likely uphold the enforceability of the conservatory’s promise to pay the
additional $40,000.

Explanation to Point-Three (10%):

The conservatory is unlikely to be able to defend against enforcement of its promises to pay additional
money under the theory of economic duress, because the business probably did not make an improper
threat.

Under the common law of contracts, parties may raise the defense of duress. This common law defense
also applies to contracts governed by UCC Article 2.

A contract is voidable on the ground of economic duress by threat when it is established that a party’s
manifestation of assent is induced by an improper threat that leaves the party no reasonable alternative. A
threat to withhold essential goods can constitute duress. In order to void its agreement to pay the
additional sum because of economic duress, the conservatory must demonstrate that (1) the business made
a threat to the conservatory, (2) the threat was “improper” or “wrongful,” (3) the threat induced the
conservatory’s manifestation of assent to the modification, and (4) the threat was sufficiently grave to
justify the conservatory’s assent.

Here, it appears that three of the four elements are likely satisfied. The business plainly made a threat.
Moreover, the threat induced the conservatory’s assent to the modification, and the threat was sufficiently
grave to justify that assent. If the conservatory had not agreed to pay the business the extra amounts, the
conservatory would have lost its entire $325,000 investment. In light of this potential loss, a court could
easily conclude that the conservatory had no reasonable alternative.

However, the business has a strong argument that its threat (indicating that it would breach the contracts
unless the prices were increased) was not wrongful or improper, but was instead nothing more than a
communication of the reality of its own perilous situation to the conservatory.

A mere threat to breach a contract is not, in and of itself, improper so as to support an action of economic
duress or business compulsion. Something more is required, such as a breach of the duty of good faith and
fair dealing. Because the business could not perform the original contract without the requested
modification, the economic duress claim for the conservatory would likely fail for much the same reason
that the business would be able to enforce the modification. At the time the modification was requested,
the business was not trying to extort a price increase because of the conservatory’s vulnerability, but
instead was simply stating the reality that the business could not perform without more money.

Seperac-J19 Exam-Released MEE Essay Compilation © 2016-2020 285


#065-JUL 2013–MEE Q07: QUESTION SEVEN (CONTRACTS)

On May 1, a manufacturer and a chef met at a restaurant trade show. The manufacturer showed the chef
some carving knives that were on sale for $100 each. After examining the knives, the chef said, “I love
these knives! I’ll take 10 of them. Please send them to my restaurant within the month. As soon as I
receive them, I’ll send you a check for $1,000.” The manufacturer said, “I’ll ship the 10 knives to your
restaurant in a few weeks,” and he took the chef’s address for shipping purposes.

On May 15, the manufacturer sent six knives to the chef. Enclosed in the shipping box was a document on
the manufacturer’s letterhead that stated in its entirety: “It is a pleasure to do business with you. Enclosed,
pursuant to our agreement, are six knives. Please remit $600 at your earliest convenience.”

On May 17, the chef sent the manufacturer a check for $600 and included in the envelope an unsigned
note to the manufacturer, handwritten on plain paper, requesting the remaining four knives. The
manufacturer did not respond to the note.

The knives were particularly well-suited for the chef’s uses, and the $100 price was a bargain, so the chef
was very eager for the manufacturer to deliver the remaining four knives. On June 17, the chef wrote to
the manufacturer claiming that the manufacturer was contractually bound to sell the chef 10 knives and
that the manufacturer had breached that contract by furnishing only 6 knives. The manufacturer did not
reply to the chef’s letter.

Is there an enforceable contract against the manufacturer that binds him to sell 10 knives to the chef?
Explain.

Seperac-J19 Exam-Released MEE Essay Compilation © 2016-2020 286


#065: J13-7 MEE: ANSWER: NCBE (CONTRACTS)

POINT (1) [20%] ISSUE: Is there a contract between the manufacturer and the chef for the
purchase and sale of 10 knives? ANSWER: Yes. The manufacturer and the chef entered into a
contract at the trade show when they agreed that the chef would buy 10 knives from the
manufacturer for $100 each.

POINT (2) [15%] ISSUE: Is enforceability of the contract between the manufacturer and the chef
subject to the statute of frauds? ANSWER: Yes. The contract between the manufacturer and the
chef is a contract for the sale of goods for a price of $500 or more and therefore is subject to the
statute of frauds in Article 2 of the Uniform Commercial Code.

POINT (3) [25%] ISSUE: Does the document sent by the manufacturer to the chef satisfy the
applicable statute of frauds? ANSWER: The document enclosed in the shipping box with the knives
is sufficient to indicate that a contract has been made, but it is not clear that it has been “signed” by
the manufacturer – the party against whom enforcement is sought. If it is a sufficient writing, it
makes the contract enforceable, but only to the quantity of goods stated in the writing: six knives.

POINT (4) [20%] ISSUE: Is the chef’s handwritten note to the manufacturer a “record in
confirmation” of the contract sufficient to satisfy the statute of frauds? ANSWER: The chef’s note
is arguably a letter of confirmation of a contract for the sale of 10 knives. If it is, then it satisfies the
statute of frauds because the manufacturer failed to object to it within 10 days. However, because
the note was unsigned, it is unlikely to be given this effect.

POINT (5) [20%] ISSUE: Does the manufacturer’s shipment of 6 knives and the chef’s acceptance
of those 6 knives establish the existence of a contract sufficiently to allow its enforcement as to all 10
knives? ANSWER: No. The chef’s acceptance and payment for six knives eliminates the statute-of-
frauds problem as to those six knives, but does not make the contract enforceable as to the
remaining four knives.

ANSWER DISCUSSION:

The manufacturer and the chef clearly entered into a contract pursuant to which the manufacturer agreed
to sell 10 knives to the chef for $1,000. Because it is a contract for the sale of goods, the contract is
governed by Article 2 of the Uniform Commercial Code. Even though the contract exists, a contract for
the sale of goods for a price of $500 or more is not enforceable against a party unless there is a writing
sufficient to indicate that a contract has been made that is signed by that party (or unless an exception
applies). Here, the contract price is more than $500, so a writing signed by the manufacturer is required in
order to enforce the contract against it unless an exception applies. The document included with the
knives is a writing sufficient to indicate that a contract has been made, but it is not clear whether that
writing has been “signed” by the manufacturer. If the document constitutes a writing signed by the
manufacturer, the contract is enforceable against the manufacturer for only six knives – the quantity
mentioned in that writing – and not for the remaining four knives. Even if the document does not
constitute a writing signed by the manufacturer, the fact that the manufacturer delivered, and the chef paid
for, six knives would make the contract enforceable for the six knives but not the remaining four knives.

ANSWER EXPLANATION:

Seperac-J19 Exam-Released MEE Essay Compilation © 2016-2020 287


Explanation to Point-One (20%):

The manufacturer and the chef entered into a contract at the trade show when they agreed that the chef
would buy 10 knives from the manufacturer for $100 each.

Because the knives are goods, the situation presented by these facts is governed by Article 2 of the
Uniform Commercial Code. UCC § 2-204 provides that “a contract for sale of goods may be made in any
manner sufficient to show agreement, including conduct by both parties which recognizes the existence of
such a contract.” The conversation between the manufacturer and the chef clearly shows an agreement
pursuant to which the manufacturer would sell, and the chef would buy, 10 knives at $100 each. Thus,
UCC § 2-204 is satisfied and there is a contract for the chef to buy 10 knives from the manufacturer.

Explanation to Point-Two (15%):

The contract between the manufacturer and the chef is a contract for the sale of goods for a price of $500
or more and therefore is subject to the statute of frauds in Article 2 of the Uniform Commercial Code.

UCC § 2-201 provides that “a contract for the sale of goods for the price of $500 or more is not
enforceable by way of action or defense unless there is some writing sufficient to indicate that a contract
for sale has been made between the parties and signed by the party against whom enforcement is sought.”
Here, the contract between the manufacturer and the chef was for the sale of goods (the knives) for a price
of $1,000. Therefore, it is subject to the writing requirement of UCC § 2-201 unless an exception applies.

Explanation to Point-Three (25%):

The document enclosed in the shipping box with the knives is sufficient to indicate that a contract has
been made, but it is not clear that it has been “signed” by the manufacturer – the party against whom
enforcement is sought. If it is a sufficient writing, it makes the contract enforceable, but only to the
quantity of goods stated in the writing: six knives.

UCC § 2-201 requires that, for a contract within its scope to be enforceable, there be a writing sufficient
to indicate that a contract for sale has been made that is signed by the party against whom enforcement is
sought. The facts indicate that the document enclosed with the six knives that were shipped to the chef
stated in its entirety: “It is a pleasure to do business with you. Enclosed, pursuant to our agreement, are
six knives. Please remit the balance due of $600 at your earliest convenience.” This document is a writing.
Further, its contents clearly indicate that a contract for sale has been made: it states that the knives are
enclosed “pursuant to our agreement” and expresses “pleasure to do business with” the chef.

In order for the contract to be enforceable against the manufacturer under UCC § 2-201, the document
must be “signed” by the manufacturer. The facts do not state that the document bore a signature (in the
conventional sense of that term) of the manufacturer, but they do state that the document was on the
manufacturer’s letterhead. As the term “signed” is used in § 2-201(1), the meaning is broader than simply
bearing a conventional signature. “‘Signed’ includes using any symbol executed or adopted with present
intention to adopt or accept a writing.” UCC § 1-201. The Official Comment to UCC § 1-201 states that
“in appropriate cases the symbol may be found in a billhead or letterhead.” Accordingly, some cases have
held that a document on company letterhead can constitute a signed writing as long as the requisite intent
is present. Thus, it is possible, but not certain, that the requirement of a signed writing is satisfied by the
document, depending on whether the letterhead was used “with a present intention to adopt or accept” the
document.

Seperac-J19 Exam-Released MEE Essay Compilation © 2016-2020 288


Although the writing referred to in UCC § 2-201 need not contain detailed information about the contract,
“the contract is not enforceable beyond the quantity of goods shown in such writing.” Here, the document
included by the manufacturer with the knives that were shipped showed a quantity of only 6 knives.
Accordingly, even if the document qualifies as a writing signed by the manufacturer, the document is not
sufficient to make the contract enforceable beyond 6 knives. Thus, this writing alone does not provide a
basis for the chef to enforce the manufacturer’s obligation to ship the remaining 4 knives pursuant to the
oral agreement for a sale of 10 knives.

[NOTE: Some examinees may mention that the manufacturer’s shipment of only 6 knives (when 10 knives
were ordered) violates the perfect tender rule and that the chef could have rejected those knives. That is
true, but irrelevant. The chef was entitled to accept a less than perfect tender, and such acceptance will
not preclude the chef from enforcing the contract for the full amount – if the chef can overcome the statute
of frauds problem.]

Explanation to Point-Four (20%):

The chef’s note is arguably a letter of confirmation of a contract for the sale of 10 knives. If it is, then it
satisfies the statute of frauds because the manufacturer failed to object to it within 10 days. However,
because the note was unsigned, it is unlikely to be given this effect.

The statute of frauds can be satisfied in a contract between merchants if the party seeking enforcement of
the contract has sent the other party a confirmation of the contract that would itself satisfy the statute of
frauds against the enforcing party. In other words, if the chef sends a written contract confirmation to the
manufacturer that would bind the chef, the confirmation will also bind the manufacturer if the
manufacturer does not object to it promptly (i.e. within 10 days).

Here, both the chef and the manufacturer are merchants. The manufacturer deals in goods of the kind at
issue, and the chef’s occupation is such that he would have “knowledge or skills peculiar to the goods
involved in the transaction” (restaurant-quality carving knives).

However, in all likelihood, no confirmation that would satisfy the statute of frauds was sent by the chef on
these facts. Although the chef sent a note to the manufacturer requesting the remaining four knives, that
note must have been signed in order to satisfy the statute of frauds. While the note was handwritten and
accompanied by a signed check, the note itself was not “signed” by the chef as that term is defined in
UCC § 1-201. It did not contain any “symbol executed or adopted with present intention to authenticate”
the writing. A creative examinee might argue that the chef’s decision to handwrite the note indicates an
intent to authenticate by using handwriting alone, but the UCC definition of “signed” seems clearly to
contemplate some authenticating symbol in addition to the content of the writing itself.

Explanation to Point-Five (20%):

The chef’s acceptance and payment for six knives eliminates the statute-of-frauds problem as to those six
knives, but does not make the contract enforceable as to the remaining four knives.

A contract that does not satisfy the statute of frauds is nonetheless enforceable as to “goods for which
payment has been made and accepted or which have been received and accepted.” In this case, however,
payment has been made for only six knives, and only six knives have been received and accepted. Thus,
this exception would not enable enforcement of the contract for the remaining four knives but, rather, only
for the six knives already delivered.

Seperac-J19 Exam-Released MEE Essay Compilation © 2016-2020 289


[NOTE: There are other exceptions to the statute of frauds that are not at issue on these facts, but that
might be discussed by some examinees. One of those exceptions applies if the goods are specially
manufactured for the buyer and are not suitable for sale to others. Here, there is no indication that the
knives are specially manufactured.

Another exception applies if the party against whom enforcement is sought admits in pleading, testimony,
or otherwise in court that a contract was made. Here, there is no indication of such admission by the
manufacturer.

Some examinees may also note that the Uniform Electronic Transactions Act (UETA) and/or the federal
Electronic Signature in Global and National Commerce Act (E-SIGN) allow the writing requirement of
UCC § 2-201 to be satisfied by an electronic communication. This is true, but it does not change the
answer or the analysis, as there is no electronic communication that would satisfy the requirements of
UCC § 2-201.]>

Seperac-J19 Exam-Released MEE Essay Compilation © 2016-2020 290


#066-FEB 2013–MEE Q02: QUESTION TWO (CONTRACTS)

On January 2, a boat builder and a sailor entered into a contract pursuant to which the builder was to sell
to the sailor a boat to be specially manufactured for the sailor by the builder. The contract price was
$100,000. The written contract, signed by both parties, stated that the builder would tender the boat to the
sailor on December 15, at which time payment in full would be due.

On October 15, the builder’s workers went on strike and there were no available replacements.

On October 31, the builder’s workers were still on strike, and no work was being done on the boat. The
sailor read a news report about the strike and immediately sent a letter to the builder stating, “I am very
concerned that my boat will not be completed by December 15. I insist that you provide me with
assurance that you will perform in accordance with the contract.” The builder received the letter on the
next day, November 1.

On November 25, the builder responded to the letter, stating, “I’m sorry about the strike, but it is really
out of my hands. I hope we settle it soon so that we can get back to work.”

Nothing further happened until December 3, when the builder called the sailor and said, “My workers are
back, and I have two crews working overtime to finish your boat. Your boat is task one. Don’t worry;
we’ll deliver your boat by December 15th.” The sailor immediately replied, “I don’t trust you. As far as
I’m concerned, our contract is over. I am going to buy my boat from a shipyard.” Two days later, the
sailor entered into a contract with a competing manufacturer to buy a boat similar to the boat that was the
subject of the contract with the builder.

The builder finished the boat on time and tendered it to the sailor on December 15. The sailor reminded
the builder about the December 3 conversation in which the sailor had announced that “our contract is
over,” and refused to take the boat and pay for it.

The builder has sued the sailor for breach of contract.

1. What was the legal effect of the sailor’s October 31 letter to the builder? Explain.

2. What was the legal effect of the builder’s November 25 response to the sailor’s October 31
letter? Explain.

3. What was the legal effect of the sailor’s refusal to take and pay for the boat on December 15?
Explain.

Seperac-J19 Exam-Released MEE Essay Compilation © 2016-2020 291


#066: F13-2 MEE: ANSWER: NCBE (CONTRACTS)

POINT (1) [35%] ISSUE: What was the legal effect of the sailor’s October 31 letter to the builder?
ANSWER: Because the sailor had reasonable grounds for insecurity with respect to the builder’s
performance, the sailor’s letter to the builder was a justified demand seeking assurance of the
builder’s performance under the contract; failure of the builder to provide such assurance within a
reasonable time constituted repudiation of the contract.

POINT (2)(a) [30%] ISSUE: What was the legal effect of the builder’s November 25 response to the
sailor’s October 31 letter? ANSWER: The builder did not, within a reasonable time, provide the
sailor adequate assurance of due performance; this failure to provide assurance constituted a
repudiation of the contract.

POINT (2)(b) [35%] ISSUE: What was the legal effect of the sailor’s refusal to take and pay for the
boat on December 15? ANSWER: Although the builder repudiated the contract with the sailor, the
builder probably retracted that repudiation on December 3 and the sailor was no longer entitled to
cancel their contract. Thus, the sailor’s failure to perform the sailor’s obligations under the
contract constituted a breach.

ANSWER DISCUSSION:

This is a sale of goods governed by the Uniform Commercial Code. Because the sailor had reasonable
grounds for insecurity about the builder’s ability to deliver the boat in a timely manner when the sailor
learned about the strike on October 31, the sailor was legally justified in sending the letter to the builder
seeking adequate assurance of the builder’s performance pursuant to the contract. The builder’s failure to
provide such assurance within a reasonable time operated as a repudiation of the contract. However, the
builder was free to retract the repudiation before the sailor either cancelled the contract or materially
changed position in reliance on the builder’s repudiation. The builder retracted the repudiation when he
informed the sailor that the workers were back and that the boat would be delivered by the date stipulated
in the parties’ contract. Because the sailor had taken no action in response to the original repudiation, he
no longer had the right to cancel the contract with the builder. The sailor’s subsequent statement that “our
contract is over” may have constituted repudiation by the sailor. In any event, when the sailor failed to
perform on December 15, that constituted breach.

ANSWER EXPLANATION:

Explanation to Point-One (35%):

Because the sailor had reasonable grounds for insecurity with respect to the builder’s performance, the
sailor’s letter to the builder was a justified demand seeking assurance of the builder’s performance under
the contract; failure of the builder to provide such assurance within a reasonable time constituted
repudiation of the contract.

The sailor was legally justified in sending the letter to the builder on October 31. Contract parties are
entitled to expect due performance of contractual obligations and are permitted to take steps to protect that
expectation. UCC § 2-609 states that “when reasonable grounds for insecurity arise with respect to the
performance of either party, the other may in writing demand adequate assurance of due performance.”
Here, the sailor learned on October 31 that the builder’s workers were on strike. This gave the sailor

Seperac-J19 Exam-Released MEE Essay Compilation © 2016-2020 292


reasonable grounds for insecurity about the builder’s ability to complete performance on time and thus
gave the sailor the right to seek adequate assurance from the builder. Because the sailor’s demand for
assurance was justified, the builder was required to provide assurance that was adequate under the
circumstances within a reasonable time (not to exceed 30 days) or be held to have repudiated the contract.

Explanation to Point-Two(a) (30%):

The builder did not, within a reasonable time, provide the sailor adequate assurance of due performance;
this failure to provide assurance constituted a repudiation of the contract.

Because the sailor, with legal justification, demanded from the builder assurance of due performance, the
builder’s failure to provide such assurance within a reasonable time was a repudiation of their contract.
Under UCC § 2-609, after receipt of a justified demand, the failure to provide within a reasonable time
not exceeding thirty days assurance of due performance is a repudiation of the contract. On October 31,
the sailor requested that the builder provide adequate assurance regarding the completion of the boat by
December 15. The builder did not respond to the sailor’s letter until November 25 – nearly a month later.
Even if that response had been given in a reasonable time, it nonetheless did not provide assurance of due
performance. It simply stated, “I’m sorry about the strike, but it is really out of my hands. I hope we settle
it soon so that we can get back to work.” Therefore, the builder’s November 25 response did not provide
adequate assurance in response to the sailor’s justified request. Thus, the builder had repudiated the
contract.

Explanation to Point-Two(b) (35%):

Although the builder repudiated the contract with the sailor, the builder probably retracted that
repudiation on December 3 and the sailor was no longer entitled to cancel their contract. Thus, the sailor’s
failure to perform the sailor’s obligations under the contract constituted a breach.

The builder’s failure to provide adequate assurance of performance constituted a repudiation of their
contract , but the builder was free to retract that repudiation until the sailor cancelled the contract or
materially changed his position or indicated by communication or action that the sailor considered the
repudiation to be final. Pursuant to the UCC, until the repudiating party’s next performance is due, he can
retract his repudiation unless the aggrieved party has since the repudiation cancelled or materially
changed his position or otherwise indicated that he considers the repudiation final.

Here, the facts state that before the builder’s December 3 telephone call to the sailor, the sailor did
nothing in response to the builder’s repudiation, such as contracting with a third party for a boat. The
builder’s December 3 call, informing the sailor that the boat would be timely delivered, probably
constituted a retraction of the repudiation because it clearly indicated to the sailor that the builder would
be able to perform. Thus, after being so informed, the sailor did not have the right to treat their contract as
cancelled. Accordingly, the sailor’s failure to perform the sailor’s obligations under the contract by taking
the boat and paying for it constituted a breach of the contract.

Seperac-J19 Exam-Released MEE Essay Compilation © 2016-2020 293


#067-FEB 2012–MEE Q03: QUESTION THREE (CONTRACTS)

GreenCar owns a fleet of 10 identical energy-efficient, electric “green” cars that it rents out for special
events. GreenCar is the only company that has such specialized cars available for rental. GreenCar needed
all 10 of its cars to fulfill a contract to provide 10 identical green cars to carry dignitaries in the local
Earth Day parade on April 22, but each of the cars needed repair to be operable for the parade.

In order to have all 10 cars repaired in time for the parade, GreenCar entered into a contract with
RepairCo pursuant to which RepairCo promised to “repair all 10 cars and return them to GreenCar no
later than April 21 for $1,000 per car, $10,000 total.”

On April 21, RepairCo had completed repairs on only 6 of the 10 cars and returned those 6 cars to
GreenCar. When RepairCo delivered the 6 cars, it informed GreenCar that the remaining 4 cars were not
ready because RepairCo workers had walked off the job when salary negotiations broke down. RepairCo
also explained to GreenCar that it planned to give its workers the raises they wanted, but it first wanted
“to teach them a lesson.” RepairCo estimated that the remaining 4 GreenCar cars (all still inoperable)
would be repaired by April 30.

GreenCar demanded that RepairCo return the remaining 4 unrepaired cars immediately. RepairCo did so.
GreenCar refused to pay RepairCo for any repairs to the other 6 cars.

RepairCo sued GreenCar, alleging that GreenCar’s refusal to pay anything was a breach of contract.

Is RepairCo entitled to any payment from GreenCar, and if so, under what theory or theories? Explain.

Seperac-J19 Exam-Released MEE Essay Compilation © 2016-2020 294


#067: F12-3 MEE: ANSWER: NCBE (CONTRACTS)

POINT (1) [50%] ISSUE: Did RepairCo substantially perform its contractual obligations by the
contract due date? ANSWER: No. RepairCo did not substantially perform its contractual
obligations when it repaired only 6 of the required 10 cars by the contract due date.

POINT (2) [25%] ISSUE: Can RepairCo recover some portion of the contract price on the theory
that the contract was divisible? ANSWER: Yes. If RepairCo establishes that the contract with
GreenCar is divisible, RepairCo may be entitled to recover an amount less than the full $10,000
contract price.

POINT (3) [25%] ISSUE: Can RepairCo recover some portion of the contract price as restitution
for part performance? ANSWER: Yes. Even if the contract is not divisible, RepairCo may be
entitled to restitution from GreenCar for the benefit it conferred in part performance.

ANSWER DISCUSSION:

RepairCo neither fully nor substantially performed its contractual obligations. Therefore, RepairCo is not
entitled to recover the full contract price. However, RepairCo may be entitled to recover some portion of
the contract price if the contract is divisible. If the contract is not divisible, RepairCo may be entitled to
restitution for part performance.

ANSWER EXPLANATION:

Explanation to Point-One (50%):

RepairCo did not substantially perform its contractual obligations when it repaired only 6 of the required
10 cars by the contract due date.

RepairCo did not fully perform its contractual obligations. However, RepairCo might argue that it
substantially performed its obligations under the contract and therefore is entitled to recover on that
contract. Substantial performance is present when a party completes its contractual obligations with “no
uncured material failure.” Whether a failure to perform is material depends on several factors, including
(1) the extent to which the injured party will be deprived of the benefit he reasonably expected, (2) the
extent to which the injured party can be adequately compensated for the part of that benefit of which he
will be deprived, (3) the extent to which the other party failing to perform or to offer to perform will
suffer forfeiture, (4) the likelihood that the party failing to perform will cure his failure, and (5) the extent
to which the behavior of the party failing to perform comports with standards of good faith and fair
dealing.

Here, because its failure to perform was a material breach, RepairCo did not substantially perform. First,
it repaired only 6 of the 10 vehicles, thereby denying GreenCar a material portion of the benefit GreenCar
reasonably expected. GreenCar could not make other arrangements to obtain that lost material benefit (the
use of 4 “green” cars on April 22) because it was the only company with the specialized cars, and
GreenCar was not aware until April 21 that only 6 of the 10 cars had been repaired. Second, while
GreenCar might be compensated in damages for some of the harm caused by: not having a full fleet of
vehicles for the Earth Day parade, it could not be compensated for such non-economic harms as lost
reputation. Third, RepairCo will not suffer forfeiture – it will not have to undo any work done and might

Seperac-J19 Exam-Released MEE Essay Compilation © 2016-2020 295


recover some amount for the work it completed. Fourth, RepairCo cannot cure its failure because the
contract required it to deliver 10 cars on April 21, which it did not and cannot do. Finally, RepairCo’s
failure to perform likely did not comport with standards of good faith and fair dealing. This is because it
allowed the work stoppage to continue so that it could teach its workers a lesson, rather than accepting
their terms, as it intended to do eventually, and completing the contract.

Because RepairCo failed to substantially perform its contractual obligations, GreenCar is excused from its
obligation to pay on the contract. “Where a contract is made to perform work and no agreement is made
as to payment, the work must be substantially performed before payment can be demanded.” Here, the
facts do not indicate that progress payments or periodic payments were negotiated or agreed to. Thus,
RepairCo is not entitled to recover on the entire contract on the theory of substantial performance.

Explanation to Point-Two (25%):

If RepairCo establishes that the contract with GreenCar is divisible, RepairCo may be entitled to recover
an amount less than the full $10,000 contract price.

If RepairCo’s performance is divisible, RepairCo may be entitled to some payment. A contract is said to
be divisible if the performances to be exchanged can be divided into corresponding pairs of part
performances in such a way that a court will treat the elements of each pair as if the parties had agreed
they were equivalents.

Here, RepairCo could argue that although the contract stated one total contract price ($10,000), the
contract also stated an amount per vehicle. Thus, the total payment could easily be divided among the 10
very similar vehicles, which required similar work, with each car repair corresponding to $1,000 of
GreenCar’s payment. RepairCo could argue, therefore, that it is entitled to 6/10 of the $10,000, or $6,000,
less any additional incidental or consequential losses resulting from its failure to repair the remaining 4
cars on April 21.

GreenCar could argue, however, that the contract is not divisible. GreenCar needed all 10 cars to meet its
contractual obligation to supply 10 cars for the local Earth Day parade. “There is no set formula which
furnishes a foolproof method for determining in a given case just which contracts are severable and which
are entire.” Thus, it is possible, but far from certain, that RepairCo could recover some part of the contract
price from GreenCar on the theory that the contract is divisible.

Explanation to Point-Three (25%):

Even if the contract is not divisible, RepairCo may be entitled to restitution from GreenCar for the benefit
it conferred in part performance.

Even if the contract is not divisible and RepairCo cannot recover on the contract, RepairCo may be
entitled to some recovery from GreenCar as restitution for part performance so that GreenCar is not
unjustly enriched. A party “is entitled to restitution for any benefit that he has conferred by way of part
performance in excess of the loss that he has caused by his own breach.” RepairCo partially performed by
delivering 6 of GreenCar’s vehicles with the work done on the contract due date. Thus RepairCo may be
entitled to restitution for the work done on those cars, less any additional incidental or consequential
losses resulting from its failure to repair the remaining 4 cars on April 21. [NOTE: An examinee might
properly refer to this form of possible recovery as either unjust enrichment or quantum meruit.]

Seperac-J19 Exam-Released MEE Essay Compilation © 2016-2020 296


#068-FEB 2011–MEE Q06: QUESTION SIX (CONTRACTS)

Designer and Retailer entered into a legally binding contract for Designer to maintain Retailer’s website.
Under the terms of the written contract, Retailer was to pay Designer $20,000 per year. Retailer made
timely payments for two years.

Eight months before the third year’s payment was due, Designer learned of an investment opportunity.
Designer called Retailer and said, “I need cash quickly to make an investment that will enable me to make
a $35,000 profit. I know that you owe me $20,000, but if you promise now to pay me $15,000 in cash by
the 25th of this month, I will accept that payment as satisfying your obligation under our contract for this
year.”

Retailer responded, “Thanks. That’s a good deal. I don’t have the cash to pay you now. I’ll do it if I can
get a loan.”

“That will be great,” responded Designer.

Because Designer assumed that Retailer would provide the cash Designer needed, Designer did not try to
raise the cash from another source.

Retailer, however, was busy with other matters. He visited two banks and picked up loan applications, but
he never bothered to submit them. Retailer did not take any other action to obtain a loan before the 25th of
the month had passed.

When the 25th of the month passed without payment from Retailer, Designer telephoned Retailer. The
moment that Retailer heard Designer’s voice saying “Hello,” Retailer quickly said, “Sorry, but I can’t take
you up on your offer to accept early payment.”

Designer was shocked and angered. He had counted on that money. He can prove that he would have
gained $35,000 had he been able to make the planned investment.

Designer has sued Retailer for actual damages plus punitive damages.

1. Is Retailer liable for breach of contract? Explain.

2. Assuming that Retailer is liable, can Designer recover his actual damages from Retailer?
Explain.

3. Assuming that Retailer is liable, can Designer recover punitive damages? Explain.

Seperac-J19 Exam-Released MEE Essay Compilation © 2016-2020 297


#068: F11-6 MEE: ANSWER: NCBE (CONTRACTS)

POINT (1)(a) [40%] ISSUE: Was a contract formed when Designer offered Retailer a discounted
payoff in exchange for early cash payment; Retailer responded, “Thanks. That’s a good deal. I
don’t have the cash to pay you now. I’ll do it if I can get a loan”; and Designer replied, “That will
be great”? ANSWER: Yes. Designer made an offer to Retailer to modify their existing contract.
Retailer did not accept the offer. Instead, Retailer made a counteroffer, which Designer then
accepted, forming a modified contract.

POINT (1)(b) [30%] ISSUE: If a contract was formed, did Retailer breach that contract by making
only minimal efforts to obtain the necessary cash? ANSWER: Yes. Retailer breached the contract
by failing to make good-faith efforts to obtain the necessary loan.

POINT (2) [20%] ISSUE: If Retailer breached the contract, is Retailer liable for Designer’s loss
caused by Designer’s inability to make a planned investment that would have netted a $35,000 gain
for Designer? ANSWER: Yes. Designer is entitled to recover from Retailer his actual damages,
which would include the gain that he would have realized on the investment he intended to make.

POINT (3) [10%] ISSUE: If Retailer breached the contract, is Retailer liable for punitive damages?
ANSWER: No. Designer is not entitled to recover punitive damages because such damages are not
ordinarily recoverable in a contract action.

ANSWER DISCUSSION:

Retailer and Designer entered into a new, modified contract which Retailer breached. Designer’s opening
statement was an offer. However, because Retailer’s response added a condition, his need to take out a
loan to obtain the cash to pay early, the response did not constitute acceptance of Designer’s offer. Rather,
Retailer’s response was a counteroffer, which Designer accepted, thereby creating a contract. That
contract contained Retailer’s implied promise to make a good-faith effort to borrow the needed cash.
Retailer did make some efforts to get the cash, but these efforts were not sufficient to constitute good
faith. Therefore, Retailer breached the contract by not acting in good faith. The ordinary measure of
damages in a contract action is expectation damages, which aim to put the nonbreaching party in the
position he expected to be in following full performance. Here, if Retailer had not breached, Designer
would have had $15,000 to make an investment that would have netted a $35,000 gain. Because these
losses were foreseeable and reasonably certain, Designer will be able to recover his actual damages.
Designer will not be able to collect punitive damages because Retailer’s conduct was not tortious.

ANSWER EXPLANATION:

Explanation to Point-One(a) (40%):

Designer made an offer to Retailer to modify their existing contract. Retailer did not accept the offer.
Instead, Retailer made a counteroffer, which Designer then accepted, forming a modified contract.

Designer’s statement “I need cash quickly to make an investment. If you promise now to pay me $15,000
in cash by the 25th of this month, I will accept that payment as satisfying your obligation under our
contract for this year” was an offer to enter into a new, modified contract. Retailer did not accept this
offer. Instead, Retailer said, “Thanks. That’s a good deal. I don’t have the cash to pay you now. I’ll do it if

Seperac-J19 Exam-Released MEE Essay Compilation © 2016-2020 298


I can get a loan.” This response, impliedly making Retailer’s obligation to perform conditioned upon his
getting a loan, contained an additional term and was, therefore, a counteroffer. Designer’s response, “That
will be great,” indicated assent to the terms of Retailer’s counteroffer even though it did not include
specific words of acceptance. There was consideration for the modified contract under the bargained-for-
exchange test because Designer agreed to accept a reduced payment and Retailer agreed to pay early if he
could get a loan.

It might be argued that Retailer’s response did not constitute a counteroffer because Retailer did not
explicitly state that his performance was conditioned upon getting a loan. However, Retailer’s response
made it clear that Retailer could not complete the contract unless Retailer obtained a loan, and Designer
certainly understood this. Therefore, the parties entered into a contract on the terms offered by Retailer.

[NOTE: An examinee may discuss this issue in terms of an amendment to the existing contract. Such an
analysis should receive full credit.]

Explanation to Point-One(b) (30%):

Retailer breached the contract by failing to make good-faith efforts to obtain the necessary loan.

Retailer might argue that because he did not get the loan, his obligation to pay never arose and he was not
in breach. In fact, the language used only suggested that Retailer’s obligation to perform would be subject
to a condition precedent, that is, his getting the loan.

However, the contract contained an implied obligation to make good-faith efforts to obtain a loan.
Without such an implied obligation, the contract would be illusory because Retailer would have no
obligation.

To satisfy the requirement of making good-faith efforts, Retailer must have taken reasonable steps to
obtain the necessary cash. Here, Retailer did no more than pick up two loan applications which he did not
even submit. His actions were not sufficient to constitute a good-faith effort to obtain the loan. Retailer
therefore breached the contract.

Explanation to Point-Two (20%):

Designer is entitled to recover from Retailer his actual damages, which would include the gain that he
would have realized on the investment he intended to make.

The normal measure of damages for breach of contract is expectation damages, which aim to give the
nonbreaching party the benefit of his bargain. Expectation damages must be foreseeable, and proved with
reasonable certainty.

Here, when Designer made the initial offer, he stated that he needed cash quickly to make a potentially
profitable investment. On these facts, it is foreseeable that Designer would not be able to make the
investment when Retailer failed to make timely payment. It is not necessary that the profitability of the
investment be foreseeable at the time of the breach. It is enough that the fact of the investment is
foreseeable, and here it was, because Designer told Retailer that Designer needed the cash to make an
investment, making the damages foreseeable. The facts state that Designer can prove that he would have
made $35,000 on the investment, making the damages reasonably certain. Therefore, Retailer is liable for
Designer’s actual damages.

Seperac-J19 Exam-Released MEE Essay Compilation © 2016-2020 299


Explanation to Point-Three (10%):

Designer is not entitled to recover punitive damages because such damages are not ordinarily recoverable
in a contract action.

Punitive damages are not generally recoverable as an element of damages in a breach of contract action
“unless the conduct constituting the breach is also a tort for which punitive damages can be recovered.”
The facts do not indicate any conduct on the part of Retailer that would constitute a tort compensable
through punitive damages. Therefore, Designer is not entitled to recover punitive damages.

Seperac-J19 Exam-Released MEE Essay Compilation © 2016-2020 300


#069-JUL 2009–MEE Q05: QUESTION FIVE (CONTRACTS)

Sam was walking down the sidewalk when he heard shouts coming from a burning house. Sam
immediately called 911 on his cell phone and rushed into the house. Inside the house, Sam discovered
Resident trying to coax Resident’s frightened dog from behind a couch. Sam, at great risk to his safety,
crawled behind the couch and pulled the dog from its hiding place. Sam, carrying the dog, and Resident
then safely made their way outside.

Once outside, Resident thanked Sam and asked Sam about his work. Sam told Resident, “I was hoping to
start training as a paramedic in the fall, but I don’t think I’ll be able to afford the cost of the program.”

Resident responded, “We need all the good paramedics that we can get! If you are going to start
paramedic training, I want to help you. Also, my dog means everything to me. I want to compensate you
for your heroism. Give me your address, and I will send you a check for a thousand dollars.”

Sam said, “Thank you so much! Here is my address. I’ll apply to the paramedic program tomorrow.”

Sam applied to the paramedic training program but was denied admission. Sam then applied for and was
accepted into a cosmetology training program and owes that program $1,000. Sam cannot pay the $1,000
he owes because when Resident learned Sam was not attending the paramedic program, he refused to give
Sam the $1,000.

Sam sued Resident to recover the $1,000.

What theories could Sam assert to recover all or some portion of the $1,000, and what is the likelihood of
success on each theory? Explain.

Seperac-J19 Exam-Released MEE Essay Compilation © 2016-2020 301


#069: J09-5 MEE: ANSWER: NCBE (CONTRACTS)

POINT (1) [33%] ISSUE: Was Resident’s promise to pay Sam $1,000 supported by consideration?
ANSWER: No. Sam cannot recover under the theory that Resident’s promise was supported by
consideration because that promise was not part of a bargained-for exchange.

POINT (2) [22%] ISSUE: In the absence of bargained-for consideration, can Sam enforce
Resident’s promise under the material benefit (moral consideration) rule? ANSWER: Yes. Sam
may be able to recover if the material benefit (moral consideration) rule applies, but possibly may
not receive the full $1,000 Resident promised him.

POINT (3) [44%] ISSUE: In the absence of bargained-for consideration, can Sam enforce
Resident’s promise under the theory of promissory estoppel? ANSWER: Yes. Sam may be able to
recover some or all of the $1,000 if the doctrine of promissory estoppel applies. Promissory estoppel
requires that the promisee show that a promise existed, that a detrimental change in position was
made in reasonable reliance on the promise, and that enforcement of the promise is the only way to
avoid injustice.

ANSWER DISCUSSION:

Sam cannot recover the $1,000 under the theory that Resident’s promise was supported by consideration
as there is no evidence of a bargained-for exchange. Sam may be able to recover if the material benefit (or
moral consideration) rule applies because Resident made the promise to pay $1,000 in recognition of a
benefit received. Sam may also be able to recover under the theory of promissory estoppel if he acted in
reasonable reliance on Resident’s promise. However, Sam may recover less than the full $1,000 under
both the material benefit rule and the theory of promissory estoppel.

ANSWER EXPLANATION:

Explanation to Point-One (25-35%):

Sam cannot recover under the theory that Resident’s promise was supported by consideration because that
promise was not part of a bargained-for exchange.

To be legally enforceable, a promise generally must be supported by consideration, which is shown


through bargained-for exchange. This means that the promisor must have sought and received something
of legal value in exchange for the promise. Here, there was no bargained-for exchange.

Sam might argue that his action in rescuing the dog and/or Sam’s promise to apply for paramedic training
constitutes consideration for Resident’s promise to pay $1,000. However, neither of those arguments will
prevail. Resident’s promise to pay the $1,000 was not made in exchange for Sam’s rescue of the dog, but
instead was made in recognition of that prior action. With regard to Sam’s promise to apply for paramedic
training, Resident did not seek that promise in exchange for the promise to pay the $1,000. Resident’s
promise was not part of an exchange and therefore the consideration necessary to make Resident’s
promise legally enforceable is absent.

Explanation to Point-Two (15-25%):

Seperac-J19 Exam-Released MEE Essay Compilation © 2016-2020 302


Sam may be able to recover if the material benefit (moral consideration) rule applies, but possibly may
not receive the full $1,000 Resident promised him.

Some states recognize an exception to the past consideration limitation in cases in which the promise is
made after receipt of a significant benefit. This exception is set out in the Second Restatement of
Contracts § 86 (the material benefit rule) and encapsulates cases in which moral consideration was found
to provide a basis for recovery. The material benefit rule states that a promise not supported by
consideration may be enforceable if it is “made in recognition of a benefit previously received by the
promisor from the promisee.” Here, Resident promised to give Sam $1,000 in recognition of Sam’s act of
saving Resident’s dog. Thus, it could be argued that the material benefit rule applies because Resident
received a benefit from Sam and made the promise to give Sam $1,000 in recognition of that benefit.

However, the material benefit rule does not apply (and the promise is not enforceable) if the promisee
conferred the benefit as a gift, or to the extent that the value of the promise is disproportionate to the
benefit conferred. Here, it is unclear whether Sam intended to confer a gift upon Resident. He may have
been acting out of pure selflessness when he rescued the dog, or he may have believed that his heroic
action would result in a financial reward.

Even if it is determined that Sam did not intend to confer a gift (and that Resident’s promise is therefore
enforceable), a court might limit recovery to something less than the full $1,000 if it finds that $1,000 is
disproportionate to the value of the rescue of the dog.

Explanation to Point-Three (35-45%):

Sam may be able to recover some or all of the $1,000 if the doctrine of promissory estoppel applies.
Promissory estoppel requires that the promisee show that a promise existed, that a detrimental change in
position was made in reasonable reliance on the promise, and that enforcement of the promise is the only
way to avoid injustice.

The doctrine of promissory estoppel allows the enforcement of gratuitous promises to avoid harm to
individuals who have relied on those promises. In order to establish a promissory estoppel claim, all of the
following must be shown: (1) the promisor, when making the promise, should have reasonably expected
that the promisee would change his position in reliance on the promise; (2) the promisee did in fact
change position in reliance on the promise; (3) the change in position was to the promisee’s detriment and
injustice can be avoided only by enforcing the promise.

Here, the facts state that Resident promised to pay $1,000. Sam did change his position in reliance on that
promise by incurring a $1,000 debt. Resident might argue that Sam’s applying to the cosmetology
program was not reasonable reliance because the promise was specific to a different program – Resident
said, “If you are going to start paramedic training, I want to help you.” However, Resident also said, “I
want to compensate you for your heroism,” and Sam could reasonably interpret that remark as indicating
that Resident’s promise to give him $1,000 was a promise to compensate him for rescuing Resident’s dog
and was not conditioned on his career choice. Thus, it is possible that Sam could recover that portion of
the $1,000 promised by Resident that is determined to be the amount required to avoid injustice.

Seperac-J19 Exam-Released MEE Essay Compilation © 2016-2020 303


#070-JUL 2008–MEE Q08: QUESTION EIGHT (CONTRACTS)

Rancher conducts cattle roping clinics in various locations around the country. Rancher thought it would
be more profitable to buy his own land and conduct the clinics there.

In March, Rancher bought the White Oaks Ranch (Ranch) with a large pasture on which Rancher could
hold the roping clinics.

In April, before Rancher had offered any roping clinics on the property, Rancher agreed to allow Gasco,
an oil and natural gas company, to explore for gas reserves on Ranch. Before the parties signed a contract,
Gasco executives drove around Ranch, and Rancher pointed out to them the pasture where he planned to
hold his roping clinics. Rancher told the Gasco executives, “I can’t wait to start holding my clinics here so
that I won’t have to go on the road anymore. Every summer that I travel with my clinics costs me
$50,000. It will cost me only $10,000 to work from Ranch.”

In July, Rancher and Gasco signed a contract in which Gasco agreed to complete its gas exploration and
restore Ranch to its pre-exploration condition by March 31 of the following year. Gasco immediately
began exploring for gas on Ranch.

By March 31 of the following year, Gasco had completed its exploration but chose not to restore Ranch to
its pre-exploration condition. Because of Gasco’s failure to restore Ranch, the pasture was not usable, and
Rancher had to cancel his plans to conduct roping clinics on Ranch that summer.

Rancher sued Gasco for breach of contract. At trial, an expert for Rancher testified that because of
Gasco’s failure to promptly restore Ranch to its pre-exploration condition, it would cost $500,000 and
take three years to restore Ranch. Furthermore, during that time Ranch could not be used for roping
clinics.

An expert for Gasco testified that Ranch was worth only $20,000 less in its unrestored condition than if it
had been restored to its pre-exploration condition. There was no other expert testimony.

Rancher testified that Ranch could not be used for roping clinics for the next three summers. Rancher
estimated that 50 people would have attended the roping clinics each year, and each person would have
paid a fee of $2,000, for a total of $100,000 per year. Therefore, Rancher seeks $300,000 for his losses.

The trial court found that there was an enforceable contract between the parties and that Gasco had
breached the contract by failing to restore Ranch. The court awarded Rancher $500,000 for the cost of
restoring Ranch to its pre-exploration condition and $300,000 for his losses.

1. Did the court err in awarding Rancher the cost of restoring Ranch to its pre-exploration
condition? Explain.

2. Did the court err in awarding Rancher $300,000 for damages resulting from Rancher’s inability
to conduct roping clinics on Ranch for three years? Explain.

Seperac-J19 Exam-Released MEE Essay Compilation © 2016-2020 304


#070: J08-8 MEE: ANSWER: NCBE (CONTRACTS)

POINT (1) [50%] ISSUE: Should Rancher’s damages for harm to Ranch be measured by the cost
of completion ($500,000) or by the difference in value ($20,000)? ANSWER: Rancher is entitled to
cost-of-completion damages because the breach appears to be willful and only cost-of-completion
damages will enable Rancher to use Ranch for its intended purpose.

POINT (2) [8%] ISSUE: Were Rancher’s losses from the roping clinics foreseeable? ANSWER:
Yes. Rancher’s losses based on his failure to conduct roping clinics were foreseeable.

POINT (3) [8%] ISSUE: Were Rancher’s losses from the roping clinics ascertainable with
reasonable certainty? ANSWER: Rancher can recover foreseeable losses from his inability to
conduct the roping clinics only if those losses also were sufficiently certain.

POINT (4) [17%] ISSUE: Should the award of damages for Rancher’s losses from the roping clinics
take into account expenses he avoided? ANSWER: Yes. Rancher’s award should have been
reduced by the expenses Rancher would have incurred in conducting the roping clinics.

POINT (5) [17%] ISSUE: Did the damage award properly account for losses Rancher could
reasonably have avoided by mitigation? ANSWER: It is uncertain whether Rancher’s award
should have been reduced by the amount of loss Rancher could have avoided by going on the road
with his roping clinics.

ANSWER DISCUSSION:

Under basic principles of contract law an injured party is entitled to a damage award that will put him in
the position he would have been in had the contract been performed. Here, with regard to the harm to
Ranch, the question is whether the proper measure of damages is the cost of completion or the difference
in value. Although the answer is not certain, the court probably did not err in awarding the cost of
completion. With regard to the roping clinics, damages for Rancher’s inability to conduct the clinics are
recoverable only if foreseeable and reasonably certain. Here, while the damages are likely foreseeable, it
is less clear that they are sufficiently certain. Even if the loss from the inability to conduct the clinics was
foreseeable and certain, damages should have been reduced by any amount saved due to not conducting
the clinics. Further, damages should be reduced by any amounts that could have been avoided through
mitigation.

ANSWER EXPLANATION:

Explanation to Point-One (40-50%):

Rancher is entitled to cost-of-completion damages because the breach appears to be willful and only cost-
of-completion damages will enable Rancher to use Ranch for its intended purpose.

For breach of contract, the injured party may be entitled to expectation damages. These damages are
intended to put the injured party in the same position as if the contract had been performed. Since Ranch
was not restored to its pre-exploration condition, Rancher is entitled to damages. One measure of damages
is the cost of restoration – here, $500,000. However, where an award might be wasteful, such as when the
cost to restore (here $500,000) would greatly exceed the difference in value (here $20,000), damages may

Seperac-J19 Exam-Released MEE Essay Compilation © 2016-2020 305


be measured by the difference in value. But when the breach appears to be willful, as is the case here, and
only completion of the contract will enable the non-breaching party to use the land for its intended
purposes, the cost of completion is considered the appropriate damage award. Therefore, the court
probably did not err in awarding Rancher the cost of returning Ranch to its pre-exploration condition.

Explanation to Point-Two: (05-10%):

Rancher’s losses based on his failure to conduct roping clinics were foreseeable.

Contract damages must be foreseeable to be recoverable. Damages are foreseeable if a reasonable person
in the position of the breaching party would have known at the time the contract was made that the
damages were likely to occur as a result of the breach. Here, it is likely that Gasco would have known that
if it did not restore Ranch, Rancher would not be able to hold his roping clinics. In fact, Rancher
specifically told Gasco of his plans to hold the clinics on Ranch. Because of this direct communication,
the damages caused by Rancher’s inability to conduct the roping clinics were foreseeable.

Explanation to Point-Three: (05-10%):

Rancher can recover foreseeable losses from his inability to conduct the roping clinics only if those losses
also were sufficiently certain.

Contract damages must be proved with reasonable certainty to be recoverable. Here, Rancher faces the
problem of a “new business.” Although Rancher had conducted clinics on the road in the past, Rancher
had not conducted clinics on Ranch. While most states no longer apply a per se rule denying recovery to
all new businesses, courts still are reluctant to award lost profits to new businesses, because such profits
“are regarded as being too remote, contingent and speculative to meet the standard of reasonable
certainty.”

Here, it is a close call whether there is sufficient certainty to award Rancher his lost profits from the
roping clinics. On one hand, there is no reasonable certainty that the clinics would have been a success, or
that 50 people would have attended each year. On the other hand, Rancher has run roping clinics in the
past and so has some track record on which to base a damage award. The experience of the person
involved is relevant in determining certainty of damages.

Explanation to Point-Four (10-20%):

Rancher’s award should have been reduced by the expenses Rancher would have incurred in conducting
the roping clinics.

Contract damage awards must take into account costs avoided because of the breach. Here, even if
Rancher’s claimed damages of $300,000 were foreseeable and certain, that amount represents the gross
amount that Rancher would have received. In order to receive that amount, Rancher would have incurred
expenses. Gasco’s breach saved him those expenses. Therefore, even if the award of $300,000 was
foreseeable and reasonably certain, it should be reduced to the net amount Rancher would have earned
after expenses.

Explanation to Point-Five (10-20%):

It is uncertain whether Rancher’s award should have been reduced by the amount of loss Rancher could
have avoided by going on the road with his roping clinics.

Seperac-J19 Exam-Released MEE Essay Compilation © 2016-2020 306


The $300,000 award of damages may be reduced by the amount Rancher could have earned by mitigating
his loss. Mitigation requires the injured party to take reasonable steps to reduce the damages. Here,
Rancher offered clinics on the road before he bought Ranch. After the breach Rancher could have
resumed these activities as mitigation, as that alternative may not be viewed as substantially different
from or inferior to the clinics he planned to offer on the ranch. However, the need to travel might make
this alternative inferior. Therefore, it is uncertain whether Rancher’s award should be reduced by the
amount he could have earned by offering the roping clinics on the road.

Seperac-J19 Exam-Released MEE Essay Compilation © 2016-2020 307


#071-JUL 2007–MEE Q01: QUESTION ONE (CONTRACTS)

Baker is a renowned pastry chef. Café, a sole proprietorship, is a well-known restaurant in need of hiring
a pastry chef. Baker and Café’s Owner had extensive conversations regarding Baker coming to work at
Café. On May 1, a week after those conversations occurred, Baker sent Café a signed letter dated May 1
stating: “I will work for Café as head pastry chef for two years for an annual salary of $100,000.”

On the morning of May 7, Café’s Owner telephoned Baker and said: “The $100,000 is pretty stiff. Could
you possibly consider working for less?” Baker replied: “I am a renowned pastry chef. I will not work for
any less!”

Later that morning, Café’s Owner sent Baker a signed letter by regular mail stating: “You obviously think
you are too good for my restaurant. I am no longer interested in hiring you to work at Café.”

Later that afternoon, Café’s Owner had a change of heart and sent Baker a registered, express-mail signed
letter stating: “Okay, if you really won’t work for less, I agree to pay you the $100,000 a year you demand
to work as head pastry chef at Café for two years.”

On May 10, the registered, express-mail letter was delivered to Baker’s office. The regular-mail letter
containing the rejection was still on its way. Baker accepted delivery of the registered, express-mail letter
from the postal carrier and placed it on his desk without opening it.

On May 11, before Baker read the registered, express-mail letter on his desk, he accepted an offer to work
for Restaurant. As a courtesy, Baker called Café’s Owner and said, “Sorry, I just took a job at Restaurant.
Too bad you couldn’t afford me.” Café’s Owner responded, “You can’t work for Restaurant, I already
accepted your offer to work at Café for $100,000 a year.”

Does Café have an enforceable contract with Baker? Explain.

Seperac-J19 Exam-Released MEE Essay Compilation © 2016-2020 308


#071: J07-1 MEE: ANSWER: NCBE (CONTRACTS)

POINT (1)(a) [23%] ISSUE: Did Baker’s letter of May 1 constitute an offer? ANSWER: Yes.
Baker’s signed letter of May 1 to Café agreeing to work as a pastry chef for Café is a valid offer.

POINT (1)(b) [23%] ISSUE: Did Café Owner’s oral response on the morning of May 7 constitute a
counteroffer? ANSWER: No. Café Owner’s phone call to Baker on the morning of May 7 asking if
he would possibly work for less was not a counteroffer but merely a request for changed terms.

POINT (2) [27%] ISSUE: Did Café accept Baker’s offer? ANSWER: Yes. Although Café’s Owner
initially rejected Baker’s offer in writing, he later accepted the offer. Because Baker received the
acceptance before he received the rejection, Baker’s offer is deemed accepted.

POINT (3) [27%] ISSUE: Did the contract between Baker and Café satisfy the Statute of Frauds?
ANSWER: Yes. If a contract cannot be performed within a year, it must meet the requirements of
the Statute of Frauds. Here the contract satisfies that statute and, therefore, is enforceable.

ANSWER DISCUSSION:

A valid contract requires an offer, an acceptance and, when as here, the contract cannot be performed
within one year, a writing that satisfies the Statute of Frauds. Here, Baker made an offer to work for Café
that was accepted when Café’s Owner sent an acceptance by express mail to Baker. It is irrelevant that
Baker did not read the acceptance. The fact that an earlier rejection was mailed is also irrelevant because a
rejection, unlike an acceptance, is effective only upon receipt, and Baker did not receive the rejection
before receiving the acceptance. When both a rejection and an acceptance are sent, whichever is received
first is effective. Lastly, the writings, being signed, satisfy the requirements of the Statute of Frauds.
Therefore, Café has an enforceable contract.

ANSWER EXPLANATION:

Explanation to Point-One(a) (20-30%):

Baker’s signed letter of May 1 to Café agreeing to work as a pastry chef for Café is a valid offer.

A person makes an offer when the person communicates to another a statement of “willingness to enter
into a bargain, so made as to justify” the other person who hears the statement “in understanding that his
assent to that bargain is invited and will conclude it.” Here, Baker’s letter of May 1 to Café was an offer
because an objective recipient of the letter, such as Café, would reasonably conclude that assent would
create a contract.

An offer cannot ripen into a contract by acceptance unless its terms are reasonably certain. Here, the terms
were clear and certain and identified the parties, the subject matter, and the price.

Explanation to Point-One(b) (20-30%):

Café Owner’s phone call to Baker on the morning of May 7 asking if he would possibly work for less was
not a counteroffer but merely a request for changed terms.

Seperac-J19 Exam-Released MEE Essay Compilation © 2016-2020 309


A counteroffer is a statement from the offeree to the offeror, relating to the same subject matter as the
original offer but suggesting a substituted bargain from the original terms. Generally, if an offeree makes
a counteroffer, the offeree can no longer accept the original offer. Here, Café’s Owner said to Baker “The
$100,000 is pretty stiff. Could you possibly consider working for less?” This utterance is not a
counteroffer because it did not offer substitute terms to Baker and did not indicate any unwillingness to
conclude the bargain on Baker’s terms if Baker would not accept an alternative salary. All Café’s Owner
did was ask Baker if he could possibly work for less. Café’s Owner proposed no alternative salary.
Because Café’s Owner’s call to Baker was not a counteroffer, but merely a request for unspecified
changed terms, it did not preclude Café’s later acceptance of Baker’s offer.

Explanation to Point-Two (25-35%):

Although Café’s Owner initially rejected Baker’s offer in writing, he later accepted the offer. Because
Baker received the acceptance before he received the rejection, Baker’s offer is deemed accepted.

A rejection is a manifestation of intent not to accept an offer. A rejection terminates the offeree’s power to
accept an offer. However, a rejection does not extinguish the offeree’s right to accept an offer until the
rejection is received by the offeror. Here, Café’s Owner’s letter stating “I am no longer interested in
hiring you” clearly manifests an intent not to go forward with the bargain and constitutes a rejection of
Baker’s offer.

However, Café’s Owner’s second letter, in which Café agreed to Baker’s terms, was an acceptance
because it was a manifestation of assent to the terms of an offer made in a manner invited by the offer.
The question then becomes which of the two letters sent by Café’s Owner is effective, the rejection or the
acceptance.

An acceptance is effective upon dispatch under the so-called “mailbox rule.” A rejection is effective only
upon receipt. But when an acceptance is sent after a rejection (that is, both the acceptance and the
rejection are sent), whichever gets to the recipient first is effective. Here, Café’s letter of acceptance was
received by Baker first, while the letter containing the rejection was still on the way to Baker. Under the
Second Restatement of Contracts, a communication is received when it comes into the possession of the
person to whom it is addressed. The fact that Baker did not read the letter does not alter this result.
Because the acceptance was the first communication received, it is effective. Therefore, Café accepted
Baker’s offer to work for Café, and a contract was created.

Explanation to Point-Three (25-35%):

If a contract cannot be performed within a year, it must meet the requirements of the Statute of Frauds.
Here the contract satisfies that statute and, therefore, is enforceable.

A contract must satisfy the Statute of Frauds if it cannot be fully performed within one year. Here, the
two-year employment requirement cannot be completed in one year, and therefore the contract is within
the purview of the Statute of Frauds and must satisfy the requirements of the Statute of Frauds to be
enforceable.

A contract within the Statute of Frauds satisfies that statute and is enforceable if it is evidenced by a
writing signed by “the party to be charged,” which (a) reasonably identifies the subject matter of the
contract; (b) is sufficient to indicate that a contract has been made; and (c) “states with reasonable
certainty the essential terms” of the contract.

Seperac-J19 Exam-Released MEE Essay Compilation © 2016-2020 310


Here, each party signed a writing that is sufficient under these criteria as it identifies the position, person,
term, and salary. Therefore, the Statute of Frauds is satisfied and the employment contract is enforceable.
Because this is a personal services contract, if Baker refuses to work for Café, Café can sue Baker for
damages, but cannot get specific performance.

Seperac-J19 Exam-Released MEE Essay Compilation © 2016-2020 311


#072-FEB 1995–MEE Q07: QUESTION SEVEN (CONTRACTS/UCC ART. 9)

On May 1, Bisco entered into a financing arrangement with Lender. As part of the deal, Lender loaned
Bisco $150,000 and Bisco properly executed a security agreement granting Lender a security interest in
all of Bisco's inventory and equipment and after-acquired inventory and equipment. Lender properly
perfected the security interest by filing a financing statement in the appropriate government office on May
3.

On June 1, Bisco and Carton Inc. signed a written agreement for Bisco to purchase 200,000 cardboard
boxes suitable for packing its baked goods. Carton had over one million of these boxes in its inventory.
The price of the boxes, which were to be delivered on June 17, was $5,000. Payment was due seven days
after delivery.

Because of an industry-wide labor dispute on June 5, a general shortage of boxes developed. On June 10,
Carton learned that the market value of 200,000 boxes of the kind ordered by Bisco had risen to $20,000.
Carton also knew that Bisco would be unable to obtain a sufficient number of boxes elsewhere in time for
its busy summer season. Carton's man-ager telephoned the owner of Bisco and told him that Carton would
not deliver the boxes on June 17 unless Bisco agreed to pay $20,000 for them. Bisco protested and asked
Carton to "live up to the contract." When Carton reiterated that Carton would not deliver the boxes unless
Bisco agreed to the price increase, Bisco reluctantly stated, "O.K., I guess I've got no choice." The next
day, Bisco signed a "modification agreement," which was identical to the original contract, except that the
price of the boxes was $20,000 instead of $5,000.

On June 17, Carton delivered the boxes to Bisco from Carton's inventory. On June 20, Carton received a
check from Bisco for $20,000, which Carton's bank returned to Carton five days later marked
"insufficient funds." On the same day, Carton learned that Bisco had, in fact, been insolvent since June
17.

On June 26, Carton demanded that Bisco return the boxes, but Lender had already repossessed them from
Bisco. Bisco had failed to make an installment payment to Lender on June 18, an event of default under
Bisco's security agreement with Lender.

1. Was the modification agreement signed by Bisco enforceable? Explain.

2. If Lender had not already repossessed the boxes, would Carton have had any right to recover
them from Bisco? Explain.

3. What rights, if any, does Carton have to recover the boxes as against Lender? Explain. (Do not
discuss bankruptcy law.)

Seperac-J19 Exam-Released MEE Essay Compilation © 2016-2020 312


#072: F95-7 MEE: ANSWER: NCBE (CONTRACTS/UCC ART. 9)

POINT (1) [47%] ISSUE: Is the contract modification agreement between Carton and Bisco
enforceable? ANSWER: No. The contract modification agreement is probably not enforceable
because Carton acted in bad faith in coercing a price increase from Bisco under circumstances
where Bisco had no choice but to pay an increased price and the original price was not unfair to
Carton.

POINT (2) [21%] ISSUE: Could Carton have reclaimed the boxes from Bisco? ANSWER: Yes.
Carton could have reclaimed the boxes from Bisco under UCC § 2-702 if it had acted before Lender
repossessed. An unpaid seller generally has a right to reclaim from an insolvent buyer if the seller
acts promptly.

POINT (3) [32%] ISSUE: Does Lender have priority over Carton? ANSWER: Yes. Lender has
priority over Carton because, as a perfected secured party with an interest in the boxes, it is treated
as a good-faith purchaser of the boxes with priority over a reclaiming seller.

ANSWER EXPLANATION:

Explanation to Point-One (40-50%):

The contract modification agreement is probably not enforceable because Carton acted in bad faith in
coercing a price increase from Bisco under circumstances where Bisco had no choice but to pay an
increased price and the original price was not unfair to Carton.

The modification agreement, requiring Bisco to pay $20,000 instead of $5,000 for the boxes, is probably
not enforceable but the issue is close. At common law, the issue was whether Bisco's promise to pay
$20,000 was supported by consideration. At first blush, the answer seems to be no. Carton already had the
obligation to deliver the boxes for $5,000 and Bisco's promise to pay more would be unenforceable under
the preexisting-duty doctrine. But Carton could rely on the impracticability doctrine to argue that the
industry- wide labor dispute excused its performance to deliver the boxes for $5,000, and that delivery of
the boxes was fresh consideration for Bisco's promise to pay $20,000. This argument probably would fail,
however, because courts rarely excuse performance, especially when the event making performance
onerous could have been foreseen (here, a labor dispute) and when performance had not become more
costly to Carton (Carton already had the boxes in its inventory; it did not have to pay more for the boxes).

The above analysis has been superseded by UCC § 2-209, although portions are still relevant. Under UCC
§ 2-209, a modification needs no consideration to be binding. But comment 2 to the section makes clear
that an enforceable modification must be in good faith, which under UCC § 1-203 means “honesty in
fact.” The standard under which good faith between merchants is measured is more exacting and requires
“honesty in fact and the observance of reasonable commercial standards of fair dealing in the trade.”
Generally, courts test this standard on subjective and objective bases, and attempt to determine whether
the modification was wrongly coerced. This requires analyzing whether Bisco had any choice and
whether Carton's conduct was proper. It appears that Bisco did not have any commercially practical
choice other than paying the increased price, and courts have not found access to courts a viable choice
for a party backed into a corner. Carton may argue that demanding market value for the boxes after an
unanticipated event that drives up the price 400% is not unreasonable conduct. But most courts probably
would be reluctant to recognize a right in Carton to demand a price increase based on these facts,

Seperac-J19 Exam-Released MEE Essay Compilation © 2016-2020 313


especially because Carton had the boxes in inventory and would have incurred no additional cost in
furnishing them to Bisco. Carton probably acted in bad faith, thereby rendering the modification
agreement un-en force able.

If Carton were able to convince a court that it acted in good faith, the modification would be enforceable
because it satisfied all other requirements, including the requirement of UCC § 2-209 that modifications
be written to be enforce able.

Explanation to Point-Two (15-25%):

Carton could have reclaimed the boxes from Bisco under UCC § 2-702 if it had acted before Lender
repossessed. An unpaid seller generally has a right to reclaim from an insolvent buyer if the seller acts
promptly.

Under UCC § 2-702, when a buyer, while insolvent, receives goods on credit, the seller may reclaim the
goods by making demand within ten days. Thus, Carton could have reclaimed the boxes from Bisco, if
Bisco had possession of them. Carton satisfies all of the elements of the section: Bisco received the boxes
on credit; Carton later “discovered” Bisco to have been insolvent at the time of delivery; Carton made
demand within ten days.

This is true even if the modification agreement is unenforceable and Bisco should have been charged only
$5,000. Carton could have reclaimed as against Bisco because Carton received nothing for the boxes and
thus satisfied UCC § 2-702.

The problem, of course, is that Bisco no longer had possession of the boxes. The issue then becomes one
of priorities as between Lender and Carton.

Explanation to Point-Three (25-35%):

Lender has priority over Carton because, as a perfected secured party with an interest in the boxes, it is
treated as a good-faith purchaser of the boxes with priority over a reclaiming seller.

Lender has a perfected security interest in the boxes in light of the after-acquired property clause in the
security agreement. The boxes are either equipment under UCC § 9-109 because they are used primarily
in business or inventory under UCC § 9-109 because they are materials used or consumed in a business.
The security agreement gave Lender a security interest in both kinds of collateral. Under UCC § 9-503,
Lender has the right to take possession after Bisco's default.

But what of Carton's right to reclaim? Under UCC § 2-702, Carton's right to reclaim is “subject to” the
rights of a good-faith purchaser. Lender is a good-faith purchaser. There is nothing in the facts to suggest
that Lender took the security interest other than in good faith and the creation of a security interest
constitutes a “purchase.” (“since a purchase includes taking by any voluntary transaction creating an
interest in property and a purchaser means a person who takes by purchase. Courts have construed the
“subject to” language of UCC § 2-702 to give priority to the “purchaser,” here Lender.

Seperac-J19 Exam-Released MEE Essay Compilation © 2016-2020 314


CRIMINAL LAW & PROCEDURE: 12 OF 24 MEE EXAMS: (50%)
#073-JUL 2019–MEE Q02: QUESTION TWO (CRIMINAL LAW & PROCEDURE)

On February 1, a woman began serving a 60-day sentence in the county jail for operating a motor vehicle
under the influence of alcohol. On February 4, a detective from the county sheriff’s department took the
woman from her cell to an interrogation room in the jail building. He informed her that she was a suspect
in a homicide investigation and that he wanted to ask her some questions. The detective then read the
woman the state’s standard Miranda warnings:

You have the right to remain silent. Anything you say can be used against you in court. You have
the right to an attorney. If you cannot afford an attorney, one will be appointed for you. If you
decide that you wish to speak with us, you may change your mind and stop the questioning at any
time. You may also ask for a lawyer at any time.

The detective asked the woman if she understood these rights. When she replied, “Yes, and I want a
lawyer,” questioning ceased immediately, and she was returned to her cell.

On March 15, the detective removed the woman from her cell and took her back to the same interrogation
room. The detective told her that he wanted to ask her questions about the homicide because he had new
information about her involvement. The detective read her the same Miranda warnings he had read on
February 4 and asked her whether she understood her rights. She said, “Yes.”

The woman then asked the detective, “If I ask you to get me a lawyer, how long until one gets here?” The
detective replied as follows:

We have no way of getting you a lawyer immediately, but one will be appointed for you, if you
wish, if and when you go to court. We don’t know when that will happen. If you wish to answer
questions now without a lawyer present, you have the right to stop answering questions at any time.
You also have the right to stop answering questions until a lawyer is present.

The detective’s statement accurately characterized the procedure for appointment of counsel. The woman
then said, “I might need a lawyer.” The detective responded, “That’s your call, ma’am.”

After a few minutes of silence, the woman took a Miranda waiver form from the detective and checked
the boxes indicating that the rights had been read to her, that she understood them, and that she wished to
waive her rights and answer questions. She then signed the form. After the detective began to question
her, she confessed to the homicide.

The woman was charged with murder in state court. Her lawyer filed a motion to suppress the woman’s
March 15 statements to the detective, alleging three violations of her Miranda rights by the detective:

(1) Interrogating the woman on March 15 after she had invoked her Miranda right to counsel on
February 4.

(2) Incorrectly conveying to the woman her Miranda right to counsel by the statements he made on
March 15.

Seperac-J19 Exam-Released MEE Essay Compilation © 2016-2020 315


(3) Interrogating the woman on March 15 after she had invoked her Miranda right to counsel on
March 15.

This state affords a criminal defendant no greater rights than those mandated by the U.S. Constitution.

After an evidentiary hearing, the trial court denied the motion to suppress on all three grounds raised by
defense counsel.

Did the court err? Explain.

Seperac-J19 Exam-Released MEE Essay Compilation © 2016-2020 316


#073: J19-2 MEE: ANSWER: NCBE (CRIMINAL LAW & PROCEDURE)

POINT (1) [40%] ISSUE: Did the detective violate the woman’s rights by interrogating the woman
on March 15 after she had invoked her right to counsel under Miranda on February 4? ANSWER:
No. The woman’s invocation of her Miranda right to counsel on February 4 did not prevent the
detective from interrogating her on March 15.

POINT (2) [30%] ISSUE: Did the detective violate the woman’s rights by incorrectly conveying to
the woman her Miranda right to counsel by the statements he made on March 15? ANSWER: No.
On March 15, the detective properly informed the woman of her right to counsel.

POINT (3) [30%] ISSUE: Did the detective violate the woman’s rights by interrogating the woman
on March 15 after she had invoked her right to counsel under Miranda on March 15? ANSWER:
No. On March 15, the woman did not invoke her right to counsel when she said, “I might need a
lawyer” because this was not a “clear and unequivocal request” for counsel.

ANSWER DISCUSSION:

The woman invoked her Miranda right to counsel on February 4 when she said, “I want a lawyer.” At that
point, the detective was required to cease interrogation or provide an opportunity for the woman to obtain
counsel. This restriction on interrogation, however, does not continue in perpetuity. Here, following the
woman's invocation, the detective properly ended the custodial interrogation on February 4. There is no
indication that the detective and the woman had any further contacts until March 15. This was sufficient
time to terminate the detective’s obligation to honor the woman’s February 4 invocation of the right to
counsel. Therefore, it was not a violation of the woman’s Miranda rights for the detective to wait more
than a month, repeat the Miranda warnings, obtain a waiver, and engage in custodial interrogation on
March 15. The fact that the woman remained in jail from February 4 through March 15 does not mean
that she was in “custody” during this time for Miranda purposes. On March 15, the detective properly
read the woman the state’s standard Miranda warnings. The detective’s subsequent statement describing
the procedures for obtaining counsel did not undermine the reasonable conveyance to the woman of her
Miranda rights. On March 15, the woman did not invoke her right to counsel when she said, “I might need
a lawyer.” This was not a “clear and unequivocal request” for counsel. Faced with an ambiguous or
equivocal statement, the detective was not required to (1) cease questioning the woman or (2) take any
further action to determine whether the woman wanted counsel present. Thus, the court did not err in
denying the woman's motion to suppress on any of the grounds raised by the woman’s lawyer.

ANSWER EXPLANATION:

Explanation to Point One (40%):

The woman’s invocation of her Miranda right to counsel on February 4 did not prevent the detective from
interrogating her on March 15.

If a suspect clearly and unequivocally invokes her right to counsel after being informed of her Miranda
rights, the police must cease the custodial interrogation. After invocation, counsel must be provided
before a suspect can be questioned unless the suspect (1) initiates contact with law enforcement, (2) is
given a fresh set of Miranda warnings, and (3) executes a knowing and intelligent waiver. However, in
Maryland v. Shatzer, the Supreme Court held that cessation of custodial interrogation for 14 days

Seperac-J19 Exam-Released MEE Essay Compilation © 2016-2020 317


terminated the Edwards requirements. Thus, after 14 days, law enforcement may approach a suspect who
has previously invoked the right to counsel and (assuming new Miranda warnings and a valid waiver)
reinitiate custodial interrogation even without a lawyer present.

Here, assuming that the woman was undergoing custodial interrogation, she invoked her right to counsel
and was returned to her cell on February 4, more than five weeks before the detective’s March 15
interrogation. Assuming that Miranda was fully complied with on March 15, the woman’s February 4
invocation of the right to counsel had terminated by March 15.

Finally, this analysis is not affected by the fact that the woman remained in jail from February 4 through
March 15. In Maryland v. Shatzer, the Court held that there are “vast differences between Miranda
custody and incarceration pursuant to conviction.” The release of a person from interrogation and back
into his “normal life” in the general prison population ends the “‘inherently compelling pressures’ of
custodial interrogation. Thus, the lengthy break between the woman’s initial invocation of her Miranda
right to counsel and her interrogation on March 15 terminated the Edwards requirement, notwithstanding
that she was in jail during that time.”

The court properly rejected the defense argument.

[NOTE: Custody is an objective, totality-of-the-circumstances test: whether “a reasonable person would


have felt he or she was not at liberty to terminate the interrogation and leave.” The Supreme Court has
held that “not all restraints on freedom of movement amount to custody for purposes of Miranda.” More
specifically, the Court has found that “when a person who is already serving a term of imprisonment is
questioned, the ordinary restrictions of prison life, while no doubt unpleasant, are expected and familiar
and thus do not involve the same ‘inherently compelling pressures’ that are often present when a suspect
is yanked from familiar surroundings in the outside world and subjected to interrogation in a police
station.” Thus, the woman’s ongoing incarceration from February 4 to March 15 should be treated as a
break in her custodial interrogation.]

Explanation to Point Two (30%):

On March 15, the detective properly informed the woman of her right to counsel.

Miranda v. Arizona requires that law enforcement inform a suspect that she has the “right to the presence
of an attorney and that if she cannot afford an attorney one will be appointed for her prior to any
questioning if she so desires.” Miranda does not require perfect adherence to the suggested language in
the decision itself. The Supreme Court has rejected the defense argument that informing a suspect of “the
right to talk to a lawyer before answering any of their questions” and “the right to use any of his rights at
any time he wanted during the interview,” but failing to inform the suspect of the right to counsel during
interrogation violated Miranda. Miranda warnings need not be “the clearest possible formulation,” instead
they must “reasonably convey to a suspect his rights as required by Miranda.”

Here, the state’s standard Miranda warnings read to the woman on both February 4 and March 15
“reasonably conveyed” her Miranda rights.

However, the question remains: did the detective undermine or alter the Miranda warnings on March 15
when he told the woman:

We have no way of getting you a lawyer immediately, but one will be appointed for you, if you wish, if
and when you go to court. We don’t know when that will happen. If you wish to answer questions now

Seperac-J19 Exam-Released MEE Essay Compilation © 2016-2020 318


without a lawyer present, you have the right to stop answering questions at any time. You also have the
right to stop answering questions until a lawyer is present.

Under the relevant constitutional standard, it is clear that the detective did nothing more than inform the
woman of the procedure for appointment of counsel in the state. Taken in conjunction with the detective’s
March 15 reading of the state’s standard Miranda warnings, this statement did not undermine the
reasonable conveyance of Miranda rights. Thus, the court did not err in rejecting the argument that the
woman’s statements should be excluded on this basis.

[NOTE: Some examinees might argue that the woman was not in custody when she was questioned on
March 15 (or on February 4) and therefore was not entitled to any Miranda warnings at all. The Supreme
Court has held that an incarcerated person who was questioned about a crime unrelated to his
incarceration was not “in custody” for Miranda purposes when the circumstances of the questioning
were not coercive. In that case, the prisoner was told twice that he could return to his cell whenever he
wanted; he was not restrained or threatened during interrogation; and the other circumstances were such
that, according to the Court, he would have felt free to leave the interrogation. Hence, the Court
concluded, Miranda warnings were unnecessary.

There are no facts in this problem from which one could determine whether the March 15 interrogation
was custodial and whether Miranda warnings were necessary. Nonetheless, the warnings were given and
the problem asks whether they were correctly conveyed. An examinee who notes the possibility that it was
not a custodial interrogation and that Miranda warnings were not required should nonetheless consider
whether they were given adequately.”]>

Explanation to Point Three (30%):

On March 15, the woman did not invoke her right to counsel when she said, “I might need a lawyer”
because this was not a “clear and unequivocal request” for counsel.

The mere mention of the word “lawyer” does not constitute an invocation of the Miranda right to counsel
requiring cessation of questioning under Edwards. Invocation of the right to counsel “requires, at a
minimum, some statement that can reasonably be construed to be an expression of a desire for the
assistance of an attorney.” To “unambiguously request counsel,” a suspect must “articulate her desire to
have counsel present sufficiently clearly that a reasonable police officer in the circumstances would
understand the statement to be a request for an attorney.” In contrast, where a suspect’s request is
ambiguous or equivocal, questioning need not cease. An ambiguous request is one that “a reasonable
officer in light of the circumstances would have understood only that the suspect might be invoking the
right to counsel.” When a suspect makes an ambiguous or equivocal request for counsel, officers are not
required to attempt to assess “the likelihood that a suspect would wish counsel to be present.”

The woman’s statement, “I might need a lawyer,” on its face, is not an unambiguous invocation of the
right to counsel. In addition, the detective’s response—”That’s your call, ma’am”—and the fact that the
woman remained silent before signing the waiver form suggests that, at the time of the woman’s
statement, she was not asking for a lawyer to be present but was still weighing her options. Under these
circumstances, the woman did not invoke her right to counsel, and the court properly rejected the defense
argument that the woman’s statements to the detective should be excluded.

[NOTE: Discussion of the woman’s Sixth Amendment right to counsel is not relevant to the problem. The
right to counsel under Miranda is a branch of the Fifth Amendment’s "privilege against compelled self-

Seperac-J19 Exam-Released MEE Essay Compilation © 2016-2020 319


incrimination" law. By contrast, the Sixth Amendment right does not attach until adversary judicial
criminal proceedings commence, such as when a suspect is arraigned or indicted. ]

Seperac-J19 Exam-Released MEE Essay Compilation © 2016-2020 320


#074-FEB 2019–MEE Q06: QUESTION SIX (CRIMINAL LAW & PROCEDURE)

One evening, Ben received a visit from his neighbor. Hanging on Ben's living room wall was a painting
by a famous artist. "I love that artist," the neighbor said. "I've collected several of her paintings." Ben
remarked that the famous artist was his ex-wife's mother and that whenever his new girlfriend visited, the
fact that the painting still hung in his house made her jealous. The neighbor said, "I have a solution. Why
don't you give the painting to me for safekeeping? I have an unsigned print by the same artist that you can
hang in its place. The print is not in the artist's usual style, so your girlfriend will not get jealous and your
living room will still have great art."

Ben thought this was a good idea. He and his neighbor carried the painting to the neighbor's house and
hung it in the neighbor's dining room. Ben then took the neighbor's unsigned print home and hung it in his
living room.

The next day, Ben decided that he really didn't like the print, and he took it off the wall. Then, around
10:00 p.m., he decided to retrieve the painting from his neighbor.

Ben went to his neighbor's house and knocked on the door, but there was no answer. Just as he was about
to leave, he noticed that a ground-floor window was ajar. Ben pushed the window fully open and began to
climb into the house to retrieve the painting. The neighbor, who had been asleep upstairs, was awakened
by the noise and ran downstairs to find Ben halfway through the window. The neighbor became enraged.
Ben tried to explain, but the neighbor would not stop yelling. Ben decided that it would be better to return
to his home and retrieve the painting later, after the neighbor had a chance to cool off. But the neighbor
followed him outside and across the lawn, yelling, "How dare you sneak into my house!" The yelling
attracted the attention of a police officer who was passing in her patrol car. The officer stopped to
investigate, and Ben was arrested, questioned, and released.

Two days later, the neighbor returned the painting to Ben, saying "Here's your painting. Give me back the
print that I loaned you and we'll forget the whole thing." However, the previous day Ben had been so
angry with the neighbor about his arrest that he had contacted an art dealer and had sold her the print. Ben
did not tell the art dealer that the unsigned print was by the famous artist. Ben simply offered to sell the
print at a very low price and told the art dealer, "I can sell this print to you at such a good price only
because I shouldn't have it at all." Although the art dealer often investigated the ownership history of her
purchases, she bought the print without further discussion. An hour after the sale, the art dealer contacted
a foreign art collector famously uninterested in exploring the ownership history of his acquisitions, and
sold him the print for 10 times what she had paid for it.

The prosecutor is considering bringing the following charges: (i) a charge of burglary against Ben in
connection with the incident at the neighbor's house, (ii) a charge of larceny or embezzlement against Ben
for his actions involving the unsigned print, and (iii) a charge of receiving stolen property against the art
dealer for her actions involving the print.

The jurisdiction where these events occurred has a criminal code that defines burglary, larceny,
embezzlement, and receiving stolen property in a manner consistent with traditional definitions of these
crimes.

With what crimes listed above, if any, should Ben and the art dealer be charged? Explain.

Seperac-J19 Exam-Released MEE Essay Compilation © 2016-2020 321


#074: F19-6 MEE: ANSWER: NCBE (CRIMINAL LAW & PROCEDURE)

POINT (1) [20%] ISSUE: Does someone who unlawfully enters the dwelling of another commit
burglary when the purpose of the entry was to retrieve property that the person owns? ANSWER:
No. Ben should not be charged with burglary based on his attempt to recover the painting because
he did not act with the necessary mens rea.

POINT (2)(a) [20%] ISSUE: Does someone who sells property that has been temporarily entrusted
to him by its owner commit the crime of larceny when he acts without authorization and with the
intention of depriving the owner of his property? ANSWER: No. Ben should not be charged with
the theft crime of larceny of the print because he did not take possession of the print without the
neighbor's consent with the intent to steal it from the neighbor.

POINT (2)(b) [30%] ISSUE: Does someone who sells property that has been temporarily entrusted
to him by its owner commit the crime of embezzlement when he acts without authorization and
with the intention of depriving the owner of his property? ANSWER: Yes. Ben should be charged
with the theft crime of embezzlement because his sale of the neighbor's print to the art dealer was a
wrongful conversion of that property done with the intent to permanently deprive the neighbor of
his property.

POINT (3) [30%] ISSUE: Does a purchaser of property commit the crime of receiving stolen
property when the surrounding circumstances suggest that a reasonable person should have been
alerted to the fact that the property she received was stolen? ANSWER: Yes. The art dealer
probably should be charged with receiving stolen property because the surrounding circumstances
suggest that she had the requisite knowledge that the print was stolen.

ANSWER DISCUSSION:

Ben, who intended only to retrieve his own painting from the neighbor's house, did not have an intent to
commit a felony therein. Therefore, he should not be charged with burglary. The facts do not support a
charge of larceny against Ben for his acts in connection with the print. Larceny is typically defined under
the common law as the misappropriation of another's property by means of taking it from his possession
without his consent. With respect to the print, Ben did not take it from its owner without his consent. To
the contrary, the neighbor voluntarily loaned the print to Ben. Ben's subsequent sale of the print, while
wrongful, was not larceny. On the other hand, Ben may be charged with the theft crime of embezzlement
for his acts in connection with the print. A person in lawful possession of another's property commits
embezzlement when he wrongfully converts the property with the intent to deprive the owner of it. Ben's
sale of the print to the art dealer constituted an embezzlement because Ben's intent was to permanently
deprive the neighbor of the print. Whether the art dealer should be charged with receiving stolen property
for her acts in connection with the print depends on whether there is sufficient evidence that she knew the
print was stolen. The crime of receiving stolen property typically has two elements: (1) the actus reus of
the receipt of stolen property and (2) the simultaneous mens rea of the defendant's knowledge that the
property was stolen. Here, the art dealer "received" the print when she took physical possession of it
following the sale by Ben. The central question is whether the art dealer satisfies the mens rea
requirement of knowledge that the print was stolen. Proof of the requisite knowledge can be inferred from
the surrounding circumstances, including (1) the low price; (2) Ben's statement to the art dealer, "I can
sell this print to you at such a good price only because I shouldn't have it at all"; (3) the art dealer's failure
to investigate the provenance of the print; (4) the fact that the art dealer contacted, on the same day she

Seperac-J19 Exam-Released MEE Essay Compilation © 2016-2020 322


acquired the print, a foreign art collector famously uninterested in exploring the ownership history of his
acquisitions; and (5) the sale of the print by the art dealer to the collector for 10 times what she had paid
for it. Given these circumstances, the art dealer likely had knowledge that the print was stolen.

ANSWER EXPLANATION:

Explanation to Point One (20%):

Ben should not be charged with burglary based on his attempt to recover the painting because he did not
act with the necessary mens rea.

At common law, burglary was defined as the "breaking and entering of the dwelling house of another in
the night with the intent to commit a felony" therein.

Ben's acts clearly satisfy three of the elements of burglary as defined by the common law and in statutes
based on the common law: (1) Ben illegally entered the neighbor's home by using force to push open the
neighbor's ground-floor window and starting to climb inside; (2) the neighbor's house is a "dwelling," a
"structure," or an "occupied structure" (it would even satisfy the common law requirement of a
"dwelling"); and (3) Ben's acts occurred at night.

Here, the only question is whether Ben satisfies the mens rea requirement. The evidence suggests that Ben
did not intend to commit a felony in the neighbor's home because he intended only to retrieve his own
painting. On these facts, Ben should not be charged with burglary.

Explanation to Point Two(a) (20%):

Ben should not be charged with the theft crime of larceny of the print because he did not take possession
of the print without the neighbor's consent with the intent to steal it from the neighbor.

The crime of "theft" was traditionally three separate crimes: larceny, embezzlement, and false pretenses.
Here, Ben should not be charged with larceny, although he should be charged with embezzlement.

At common law, larceny was defined as the misappropriation of another's personal property by means of
taking it from his possession without his consent. Larceny requires an intent to steal. Intent to steal is not
established by a defendant's honest belief, even if such belief was mistaken, that the property belonged to
him. Moreover, under the common law, the intent to retain property temporarily and then return it to its
rightful owner has long provided a defense to larceny.

Here, Ben should not be charged with larceny because (a) he did not take the print from the neighbor
without the neighbor's consent, and (b) there are no facts showing that he had the requisite intent to steal
when he took the print from the neighbor. It was the neighbor who had proposed swapping the artworks.
Thus, when Ben took possession of the neighbor's print he had the neighbor's permission to take it, and
there are no facts supporting an inference that Ben intended to steal it from the neighbor at that time.

Explanation to Point Two(b) (30%):

Ben should be charged with the theft crime of embezzlement because his sale of the neighbor's print to the
art dealer was a wrongful conversion of that property done with the intent to permanently deprive the
neighbor of his property.

Seperac-J19 Exam-Released MEE Essay Compilation © 2016-2020 323


To cover the case where someone in lawful possession of another person's property wrongfully
misappropriates such property, the English parliament and U.S. legislatures developed the theft crime of
embezzlement. Although precise statutory definitions vary, embezzlement generally occurs when a person
unlawfully converts property owned by another to his own use with the intent to permanently deprive the
lawful owner of the property.

Here, Ben should be charged with embezzlement. The neighbor loaned the print to Ben so that Ben could
hang it in his living room temporarily. When Ben, in a fit of anger following his arrest, sold the print to
the art dealer, he wrongfully converted the neighbor's property. Ben's actions suggest strongly that he
acted with the requisite intent to permanently deprive the neighbor of the print.

Explanation to Point Three (30%):

The art dealer probably should be charged with receiving stolen property because the surrounding
circumstances suggest that she had the requisite knowledge that the print was stolen.

Receiving stolen property typically has two elements: (1) the actus reus of the receipt of stolen property
and (2) the simultaneous mens rea of knowledge that the property was stolen. Most jurisdictions include
the further requirement that the defendant intend to deprive the owner of her property. "Stolen property"
typically includes property unlawfully obtained using larceny, embezzlement, or false pretenses, because
it is inappropriate to make the liability of the receiver turn on the method by which the original thief
acquired the property.

Here, the facts support charging the art dealer with receiving stolen property. The art dealer "received" the
print. Assuming that Ben embezzled the print, it would constitute stolen property.

The central question is therefore whether the facts establish that the art dealer had the requisite mens rea
of knowledge that the print was stolen. Normally, a person is not guilty of receiving stolen property unless
the person knew it was stolen at the moment of receiving it; subsequent discovery that property is stolen
is not sufficient. However, subsequent knowledge is enough when the crime is defined not as "receiving"
stolen property but as "exercising control over" stolen property.

Some jurisdictions require proof of a defendant's actual subjective knowledge that the property was
stolen. In those jurisdictions, evidence that a reasonable person would have known that the property was
stolen will not suffice.

Other jurisdictions follow the modern view in which mens rea can be inferred from all surrounding
circumstances. For example, a low price can support the inference that the purchaser knew the property
was stolen. In such cases, thieves must rid themselves of stolen property as quickly as possible, and a
willingness to sell at a grossly reduced price supports the inference.

The art dealer probably should be charged with receiving stolen property. Under the modern view, or even
in a jurisdiction that requires proof of subjective knowledge, the art dealer's mens rea of knowledge can
be inferred from (1) the low price; (2) Ben's statement to the art dealer: "I can sell this print to you at such
a good price only because I shouldn't have it at all"; (3) the art dealer's failure to investigate the
provenance of the print; (4) the fact that the art dealer contacted, on the same day she acquired the print, a
foreign art collector famously uninterested in exploring the ownership history of his acquisitions; and (5)
the sale of the print by the art dealer to the collector for 10 times what she had paid for it. Given these
circumstances, the art dealer likely had knowledge that the print was stolen.

Seperac-J19 Exam-Released MEE Essay Compilation © 2016-2020 324


#075-FEB 2018–MEE Q02: QUESTION TWO (CRIMINAL LAW)

A defendant, age 25, is charged in State A with armed robbery. According to the indictment, on June 1,
the defendant went into a store, pulled out a gun, and said to a cashier, "Give me all your money or I'll
shoot you!" The cashier gave the defendant $5,000. The police arrived as the defendant was driving away.
The police car followed the defendant, who was driving over 80 mph. The defendant crashed his car into a
tree and suffered a serious head injury, losing consciousness. He was taken by ambulance to a hospital,
where he regained consciousness on June 8. On June 15, he was discharged from the hospital. On July 1,
he was arraigned on the armed robbery charge and released on bail. Over the next few months, the
defendant recovered full physical mobility, but he continued to show symptoms of cognitive impairment
resulting from brain trauma suffered during the car crash.

Police interviews with the defendant's family and friends have revealed that, in the months preceding the
robbery, the defendant had experienced financial and emotional difficulties. According to the defendant's
best friend, the defendant had recently started a new business, which was struggling. A month before the
robbery, the defendant told his best friend, "I cannot attract customers because the United Nations has
organized a secret boycott of my new business." On the day before the robbery, the defendant texted his
best friend: "I've been a victim for too long. I've decided to start making up for my losses. If you read
about me in the papers tomorrow, I'll already be far away, so delete this text and tell the police you never
knew me."

In December, as the state began preparing for trial, two court-appointed psychiatrists evaluated the
defendant and prepared the following joint report to the court:

Before the robbery, the defendant had a slightly above-average IQ. The defendant had completed a
community college program in business administration and had recently opened his own business,
which he owned and managed at the time of the robbery. A few months before the robbery, the
defendant's business was struggling, and he began experiencing some mental health difficulties. His
mental health difficulties apparently did not impair his relationships with his family and friends or
his ability to manage his everyday life and operate his business. The defendant never sought mental
health treatment.

On the day of the robbery, during the crash, the defendant sustained brain trauma that has impaired
his cognitive functioning. The defendant has not returned to work, and there has been no cognitive
improvement to date. When questioned about the pending criminal charge, the defendant typically
responds, "My mother told me I did something bad, but I can't remember what." He is unable to
remember anything about the robbery. When asked about his appointed counsel, the defendant
usually says, "She's nice" or "She comes to see me and helps me." He describes the judge as "the
guy in charge," but when asked to explain what happens in court he responds, "I don't know what
they are talking about." During repeated interviews, we have seen no evidence that the defendant
currently understands abstract language and concepts. We have also seen no evidence that he is
feigning or exaggerating his cognitive impairment.

State A uses the M'Naghten not guilty by reason of insanity (NGRI) test and requires that the affirmative
defense of NGRI be proved by a preponderance of the evidence.

Seperac-J19 Exam-Released MEE Essay Compilation © 2016-2020 325


Defense counsel has requested a hearing to determine whether the defendant is competent to stand trial (in
some jurisdictions, this is called "fitness to stand trial") and has informed the court that, if the trial
proceeds, the defendant will argue that he is NGRI.

Based on all the information presented above, including the information in the psychiatrists' report:

1. Should the prosecution be suspended because the defendant is currently incompetent to stand
trial? Explain.

2. If the defendant is found competent to stand trial and the prosecution proceeds, will the jury
likely find that, with respect to each element of the M'Naghten test, the defendant has met his
burden of proof? Explain.

Seperac-J19 Exam-Released MEE Essay Compilation © 2016-2020 326


#075: F18-2 MEE: ANSWER: NCBE (CRIMINAL LAW)

POINT (1) [40%] ISSUE: Should the prosecution be suspended due to the defendant’s
incompetence to stand trial (in some jurisdictions this is called “lack of fitness to stand trial”) if, due
to a cognitive impairment, he currently cannot consult with his lawyer with a reasonable degree of
rational understanding and he lacks a rational and factual understanding of the proceedings
against him? ANSWER: Yes. The defendant should be found incompetent to stand trial because he
currently (1) cannot consult with his lawyer with a reasonable degree of rational understanding and
(2) lacks a rational and factual understanding of the proceedings against him.

POINT (2) [60%] ISSUE: If the defendant is found competent to stand trial and the prosecution
proceeds, in a jurisdiction that uses the M’Naghten test, should the defendant be found not guilty
by reason of insanity (NGRI)? ANSWER: Yes. If the defendant is ever found competent to stand
trial and the prosecution proceeds, a jury should not find that he is NGRI under the M’Naghten test
because the defense cannot prove by a preponderance of the evidence that, at the time of the armed
robbery, (1) the defendant was suffering from a defect of reason from a disease of the mind; and (2)
as a result of this mental disease or defect, he did not know the nature and quality of the act, or that
the act was wrong.

ANSWER DISCUSSION:

A defendant may not be prosecuted if he is incompetent to stand trial. Here, the defendant should be
found incompetent because the evidence gathered by the police and the court-appointed psychiatrists
demonstrates that he currently (1) lacks a rational and factual understanding of the proceedings against
him and (2) cannot consult with his lawyer with a reasonable degree of rational understanding. The
defendant does not understand the charge against him, the role of appointed counsel or the judge, what
happens in court, or abstract language and concepts. If the defendant is found competent to stand trial and
the prosecution proceeds, a jury should not find that the defendant is NGRI because the defendant cannot
prove by a preponderance of the evidence that the M’Naghten test is satisfied. Under that test, the defense
must prove that, at the time of the offense, (1) the defendant suffered from a defect of reason, from
disease of the mind; and (2) as a result of this mental disease or defect, the defendant at the time of the act
did not know the nature and quality of the act, or that the act was wrong. Under the first prong of the
M’Naghten test, the defendant will probably be unable to prove that he was suffering from a defect of
reason from a disease of the mind. Although at the time of the armed robbery the defendant was
experiencing “mental health difficulties” and some cognitive impairment (which included blaming the
United Nations for a secret boycott of his business), when balanced against the evidence of cognitive
functioning gathered by the police and the court-appointed psychiatrists, the defendant’s evidence is
probably insufficient to meet his burden of proof on this point. Under the bifurcated second prong of the
M’Naghten test, the defense must also prove that, at the time of the robbery, (1) the defendant did not
know the nature and quality of the act or (2) did not know that the act was wrong. Although states’ laws
differ on the definition of “wrongfulness” for NGRI purposes, here the facts suggest that the defendant
knew the nature and quality of his act of armed robbery and that it was legally and morally wrong.

ANSWER EXPLANATION:

Explanation to Point-One (40%):

Seperac-J19 Exam-Released MEE Essay Compilation © 2016-2020 327


The defendant should be found incompetent to stand trial because he currently (1) cannot consult with his
lawyer with a reasonable degree of rational understanding and (2) lacks a rational and factual
understanding of the proceedings against him.

Competence to stand trial is a legal requirement that refers to a defendant’s ability to participate in
criminal proceedings. In some jurisdictions, competence is called “fitness to stand trial.” To be competent,
it is not enough that the defendant be “oriented to time and place and have some recollection of events.”
Dusky v. U.S., 362 U.S. 402 (1960). The Dusky test has two prongs: the defendant must have (1) sufficient
present ability to consult with his lawyer with a reasonable degree of rational understanding; and (2) a
rational as well as factual understanding of the proceedings against him. In Drope v. Missouri, 420 U.S.
162 (1975), the Supreme Court elaborated on the Dusky standard, clarifying that the defendant must be
able to assist in preparing his defense. Assisting counsel also requires that a defendant be able to make
decisions, including whether to go to trial or plead guilty. Whenever the defense can establish a “bona
fide doubt” regarding the defendant’s decision-making abilities, the court must hold a hearing to
determine competence to stand trial. Once the defendant has made this preliminary showing, the burden
of evaluating competency is placed on the courts and court- appointed experts.

Here, the defense has raised bona fide doubt regarding the defendant’s decision-making abilities, and the
court has gathered evidence to determine his competence to stand trial. The evidence obtained by the
police and the findings of the two psychiatrists demonstrate that the defendant cannot currently meet
either of the two prongs of the Dusky competence test.

The defendant does not meet the first prong of Dusky because he lacks the “present ability to consult with
his lawyer with a reasonable degree of rational understanding.” The defendant currently cannot (1)
provide any information about the armed robbery; (2) understand the charge against him (“My mother
told me I did something bad, but I can’t remember what.”); (3) understand the role of his appointed
counsel (“She’s nice” and “She comes to see me and helps me”); or (4) comprehend abstract language and
concepts. There is also no evidence that the defendant is feigning or exaggerating this impairment for the
purpose of avoiding trial. Together, these facts show that the defendant currently lacks the ability to assist
counsel in any fashion. Here, the defendant cannot currently assist counsel because he lacks knowledge of
his case and the myriad consequences of his decisions.

The defendant also does not meet the second prong of Dusky because he does not possess “a rational as
well as factual understanding of the proceedings against him.” As to what is happening in the courtroom,
the defendant admits, “I don’t know what they are talking about,” and he understands only that the judge
is “the guy in charge.” If the defendant cannot understand the crime, charges, role of his appointed
counsel, courtroom proceedings, and the role of the judge, the court should find that he is not currently
competent to stand trial and the case should not proceed.

If the defendant’s mental abilities improve, his competence can be reassessed in the future.

Explanation to Point-Two (60%):

If the defendant is ever found competent to stand trial and the prosecution proceeds, a jury should not find
that he is NGRI under the M’Naghten test because the defense cannot prove by a preponderance of the
evidence that, at the time of the armed robbery, (1) the defendant was suffering from a defect of reason
from a disease of the mind; and (2) as a result of this mental disease or defect, he did not know the nature
and quality of the act, or that the act was wrong.

I. The M’Naghten test (15%)

Seperac-J19 Exam-Released MEE Essay Compilation © 2016-2020 328


Under the M’Naghten test for NGRI used by State A (and a majority of states), the defense must prove
that, at the time of the offense, (1) the defendant suffered from a defect of reason, from disease of the
mind; and (2) as a result of this mental disease or defect, the defendant at the time of the act did not know
the nature and quality of the act, or that the act was wrong. In State A, as in most states, the defense must
prove the affirmative defense of NGRI by a preponderance of the evidence.

II. M’Naghten prong one (15%)

Under the first prong of the M’Naghten test, the defense probably cannot prove by a preponderance of the
evidence that the defendant suffered from a defect of reason caused by a disease of the mind. There has
never been a comprehensive statutory or jurisprudential definition of “defect of reason” or “disease of the
mind.” However, for a court to find a defendant NGRI, the defendant’s mental defect or disease must be
sufficiently severe to cause the consequences described in the second prong of the M’Naghten test.

Here, the evidence indicating that the defendant was suffering from a mental disease or defect is the
psychiatrists’ opinion that the defendant was having “mental health difficulties” and the defendant’s
statement to his best friend: “I cannot attract customers because the United Nations has organized a secret
boycott of my new business.” The court must balance this evidence against the evidence that, at the time
of the armed robbery, the defendant (1) had not been diagnosed with a mental disease or defect, (2) had
not sought mental health treatment, (3) had not displayed any other signs of cognitive impairment to his
family or friends, (4) maintained relationships with family and friends, and (5) managed to run his
everyday life and operate his business. Thus, even if the defendant was suffering from some cognitive
impairment at the time of the armed robbery, there appears to be insufficient evidence of a mental disease
or defect that would satisfy the first prong of the M’Naghten test.

III. M’Naghten prong two (30%)

Under the M’Naghten test, a finding that the defendant was suffering from a “defect of reason” caused by
a “disease of the mind” is necessary—but insufficient—to support a verdict of NGRI. For example, one
court affirmed the defendant’s conviction where his schizophrenic delusions did not prevent him from
knowing right from wrong. The bifurcated second prong of the test requires that the defense also prove
that, as a result of this mental disease or defect, the defendant either did not know the nature and quality
of the act or did not know that the act was wrong.

Here, the evidence gathered by the police and the court-appointed psychiatrists establishes that the
defendant knew the nature and quality of his armed robbery of the store. The day before the robbery, the
defendant texted his best friend: “I’ve been a victim for too long. I’ve decided to start making up for my
losses,” which indicates that the defendant understood the robbery as a solution to his financial woes. The
defendant also described his plan to flee: “If you read about me in the papers tomorrow, I’ll already be far
away.” Finally, the defendant instructed his friend to destroy evidence (“delete this text”) and to lie to the
police (“tell the police you never knew me”). Taken together, these facts establish that the defendant knew
the nature and quality of the act of robbing the store.

On these facts, there is also sufficient evidence that the defendant knew that the act of armed robbery was
wrong. Various state laws (and judicial interpretations of the NGRI defense) differ on the definition of
“wrongfulness.” Some states define “wrongfulness” as legally wrong and provide the NGRI defense if the
defendant’s mental disease or defect prevented him from knowing that his acts were legally wrong (i.e.,
criminal violations). Other states define “wrongfulness” as morally wrong and provide the NGRI defense
only if the defendant did not know that his acts were morally wrong (i.e., the type of acts that society

Seperac-J19 Exam-Released MEE Essay Compilation © 2016-2020 329


would condemn). This means that a “defendant must know that his act was wrong in a moral sense and
not merely know that he has violated a statute.” The remaining states do not define “wrongfulness.”

Regardless of whether the state defines wrongfulness as a legal or moral wrong, none of the evidence
indicates that the defendant believed that his acts were legally or morally justified or acceptable. The
defendant entered a store, demanded “Give me all your money or I’ll shoot you,” stole $5,000, and sped
away from the scene driving over 80 mph. The day before the armed robbery, the defendant texted his
best friend: “I’ve been a victim for too long. I’ve decided to start making up for my losses. If you read
about me in the papers tomorrow, I’ll already be far away, so delete this text and tell the police you never
knew me.” As noted above, taken together these facts establish both that the defendant knew the nature
and quality of his acts and that his armed robbery of the store was legally and morally wrong.

In sum, even if the defense satisfied the first prong of the M’Naghten test by proving that the defendant
suffered from a “defect of reason” caused by a “disease of the mind,” the defense could not satisfy the
second prong because on this record there is insufficient proof to demonstrate, by a preponderance of the
evidence, that the defendant did not know the nature and quality of his armed robbery of the store or that
his act was legally or morally wrong.

[NOTE: The fact that the defendant now suffers from brain damage is obviously not relevant to the
question of the defendant’s mental state at the time of the crime. Nonresponsibility and incompetency to
stand trial are separate inquiries.]

Seperac-J19 Exam-Released MEE Essay Compilation © 2016-2020 330


#076-JUL 2015–MEE Q05: QUESTION FIVE (CRIMINAL LAW & PROCEDURE)

On his way to work one morning, a man stopped his car at a designated street corner where drivers can
pick up passengers in order to drive in the highway’s HOV (high-occupancy vehicle) lanes. When the
man, who was driving alone, opened his car door and announced his destination, a woman (a stranger)
jumped into the front seat.

As soon as the man drove his car onto the busy highway, the woman took a knife from her backpack and
held it against the man’s throat. She said to him, “I am being followed by photographers from another
planet where I am a celebrity. Pictures of me are worth a fortune, so I never give them away for free.
Forget the speed limit and get me out of here fast, or else.”

With the woman holding the knife at his neck, the man sped up to 85 miles per hour (30 mph over the
posted speed limit of 55 mph), weaving in and out of traffic to avoid other cars, while the woman urged
him to drive faster. While attempting to pass a motorcycle at a curve in the highway, the man lost control
of the car, which struck and killed the motorcyclist before crashing into a railing.

A police car arrived at the scene a few minutes later. The man and the woman were treated for minor
injuries at the scene and then arrested and taken to the police station.

While in custody, the woman was examined by two psychiatrists. Both psychiatrists submitted written
reports stating that the woman suffers from schizophrenia and that, at the time of the accident, her
delusions about alien photographers were caused by her schizophrenia.

The State A prosecutor has charged the woman with felony murder for the motorcyclist’s death based on
her kidnapping of the man, but is not sure whether to charge the man with any crime.

In State A, the rules governing crimes and affirmative defenses follow common law principles. However,
in State A the Not Guilty by Reason of Insanity (“NGRI”) defense is defined by statute as follows:

To establish the defense of NGRI, the defendant must show that, at the time of the charged conduct, he or
she suffered from a severe mental disease or defect and, as a result of that mental disease or defect, he or
she did not know that his or her conduct was wrong. The defendant has the burden to prove all elements
of the defense by a preponderance of the evidence.

Assume that the two psychiatric reports will be admitted into evidence.

1. Can the woman establish an NGRI defense? Explain.

2. With what crimes, if any, can the man be charged as a result of the motorcyclist’s death?
Explain.

3. What defenses, if any, will be available to the man if he is charged with a crime related to the
motorcyclist’s death? Explain.

Seperac-J19 Exam-Released MEE Essay Compilation © 2016-2020 331


#076: J15-5 MEE: ANSWER: NCBE (CRIMINAL LAW & PROCEDURE)

POINT (1) [40%] ISSUE: Can the woman establish the affirmative defense of not guilty by reason
of insanity by showing that, under the State A statute, at the time of the charged crime (a) she
suffered from a severe mental disease or defect and (b) as a result of that mental disease or defect,
she did not know that her actions were wrong? ANSWER: No. Even with the admission of the two
psychiatric reports, the evidence does not support the affirmative defense of not guilty by reason of
insanity.

POINT (2) [40%] ISSUE: Do the facts support charging the man with manslaughter based on proof
that (a) he caused the death of another person, (b) he should have been aware of a substantial risk
that another human being could be killed by his conduct, (c) his actions were a gross deviation from
the standard of care that a reasonable person would have exercised in the same situation, and (d)
his actions were in violation of traffic laws? ANSWER: Yes. The facts support charging the man
with manslaughter.

POINT (3) [20%] ISSUE: If the man is charged with manslaughter, do the facts support the
affirmative defense of duress? ANSWER: Yes. The facts support the affirmative defense of duress
because the man’s conduct was caused by his reasonable belief that obeying the woman’s
instructions to drive faster was the only way to avoid imminent death or serious bodily injury.

ANSWER DISCUSSION:

The defense of not guilty by reason of insanity (“NGRI”) is not supported by the evidence and likely
cannot be established by the woman. Under State A’s NGRI statute, the woman must show that (1) she
suffered from a severe mental disease or defect at the time of the charged crime and (2) as a result of that
mental disease or defect, she did not know that her conduct was wrong. The two psychiatric reports
support a finding that the woman suffered from a severe mental disease (schizophrenia) that caused her to
experience delusional beliefs regarding alien photographers. But the evidence does not support a finding
that the woman’s delusions prevented her from knowing that it was wrong to kidnap the man and force
him at knifepoint to drive 85 mph on a busy highway just to avoid having her picture taken without
compensation. The facts support charging the man with some form of involuntary manslaughter (e.g.,
criminal-negligence manslaughter or unlawful-act manslaughter). First, the man’s actions caused the
death of the motorcyclist. Second, anyone who drives 85 mph on a busy highway and tries to pass a
motorcyclist (a relatively exposed and vulnerable highway user) on a curve while weaving in and out of
traffic should be aware of a substantial and unjustifiable risk that another human being could be killed as a
result of his conduct. Third, the man’s actions were a gross deviation from the standard of care that a
reasonable person would have exercised in the same situation. Fourth, the man’s speeding and reckless
driving were serious traffic violations sufficient to make the man criminally accountable for the resulting
death in many jurisdictions. However, if the man is charged with manslaughter, he will almost certainly
raise the affirmative defense of duress. Here, the facts support a legal finding of duress. The man’s
conduct was caused by his reasonable belief that obeying the woman’s demand to drive faster was the
only way for him to avoid imminent death or serious bodily injury. Moreover, because he was already
driving the car on a busy highway when the woman pulled out the knife and placed it against his neck, he
had no opportunity to extricate himself from the threatening situation. Finally, although duress generally
cannot excuse a charge of intentional homicide, here the facts do not support a finding that the man’s
killing of the motorcyclist was intentional.

Seperac-J19 Exam-Released MEE Essay Compilation © 2016-2020 332


ANSWER EXPLANATION:

Explanation to Point-One (40%):

Even with the admission of the two psychiatric reports, the evidence does not support the affirmative
defense of not guilty by reason of insanity.

State A, like most states that provide a not guilty by reason of insanity (“NGRI”) defense, limits the
defense to a defendant who can show that, at the time of the charged crime, she suffered from a “severe
mental disease or defect,” and that, as a result of that mental disease or defect, “she did not know that her
conduct was wrong.” This is essentially the M’Naghten version of the NGRI defense, derived from the
opinion in M’Naghten’s Case.

Although there is no universal definition of mental illness that satisfies the legal requirement of a “severe
mental disease or defect,” a serious mental disease like schizophrenia should qualify, especially when
accompanied by delusions or other significant impairments of a defendant’s capacity to recognize reality.
For example, courts have held that a schizophrenic individual with a delusional thought process had a
severe mental disease. Thus, if the jury finds that the woman was suffering from schizophrenia
accompanied by delusional beliefs at the time of the charged crime, this finding should establish that she
suffered from the type of severe mental disease or defect contemplated by the State A NGRI statute.

However, a finding that the woman was suffering from a “severe mental disease or defect” at the time of
the charged crime is necessary but insufficient to establish her NGRI defense. The jury must also find
that, as a result of that mental disease or defect, the woman did not know that her conduct was “wrong.”
Courts have rejected a defendant’s defense (under a similar NGRI statute) if the defendant had failed to
prove that his mental disease left him unable to distinguish right from wrong and noted that a propensity
to commit wrong acts does not necessarily entail an inability to differentiate right from wrong. For
example, courts have affirmed a defendant’s conviction, despite substantial defense proof that at the time
of the charged crime he suffered from paranoid schizophrenia and delusional beliefs, because his
delusions did not prevent him from knowing right from wrong.

States differ as to the definition of “wrong” for NGRI purposes. Some states provide the defense only if
the defendant’s severe mental disease or defect prevented her from knowing that her acts were criminal.
Other states provide the defense even if the defendant knew that her actions were legally wrong (i.e.,
against the law) but did not know that her actions were morally wrong (i.e., the type of acts that society
would condemn). Finally, other states have not explained whether they define “wrong” as legally or
morally wrong.

Here, there is no evidence that the woman’s schizophrenia prevented her from knowing that it was legally
wrong to kidnap the man and force him at knifepoint to drive well above the speed limit on a busy
highway. As the woman explained, her actions were motivated by her concern that alien photographers
might take her picture for profit without providing her with compensation. Moreover, the fact that she told
the man to “forget the speed limit” provides evidence that she was aware that the speeding was illegal.

There is also no evidence that the woman’s schizophrenia prevented her from knowing that her actions
were morally wrong. The lack of evidence on this point is significant because, in some jurisdictions, a
defendant can be found NGRI – even if she knows that her actions are illegal – if she does not know that
they are morally wrong. For example, North Carolina applies a version of M’Naghten under which a
defendant can be exonerated if he did not think that his acts were morally wrong. Here, there is no
evidence that the woman’s schizophrenic delusion that celebrity-stalking aliens should pay for her

Seperac-J19 Exam-Released MEE Essay Compilation © 2016-2020 333


photograph prevented her from recognizing the immorality of her decision to kidnap the man and force
him at knifepoint to speed on a busy highway. Courts have rejected a defendant’s defense (under a similar
NGRI statute) that a defendant can be adjudged insane who has cognitive ability (the ability to know an
act is wrong, legally and morally), but lacks volitional control (the ability to control one’s behavior
despite cognitive ability).

[NOTE: Some jurisdictions provide that the insanity defense is available to a defendant who, as a result
of a severe mental disease or defect, “lacks substantial capacity” to “appreciate” the wrongfulness of the
behavior. This test is not the rule in State A, and an examinee should receive no credit for analyzing it.
This test is generally understood to broaden the insanity defense to cover cases where a mental disease
results in a significant affective or emotional impairment. In these jurisdictions, unlike in State A, a
defendant who knew that her behavior was wrong might still be found NGRI, if the defense proved that
her mental disease prevented her from emotionally appreciating the wrongfulness of her conduct. In such
a jurisdiction, the woman would have a somewhat higher likelihood of establishing the insanity defense
on the facts of this problem, although she still might be unsuccessful.]

Explanation to Point-Two (40%):

The facts support charging the man with manslaughter.

In most jurisdictions, a person who recklessly causes the death of another can be charged with so-called
“depraved-heart” murder if the person acted with “extreme indifference to the value of human life.”
Generally, however, reckless driving alone would not lead to a charge of depraved-heart murder. Such a
charge would be appropriate only if the reckless driving was combined with intoxication or other
aggravating factors. (e.g. a defendant was intoxicated and driving at an excessive speed of 70 mph on a
40-mph double curve). Here, there are no additional aggravating factors, so it is unlikely that the man
would be charged with depraved-heart murder.

A manslaughter charge, on the other hand, is quite possible. In most jurisdictions, a defendant whose
conduct causes the death of another human being can be charged with manslaughter if the defendant acted
with criminal negligence, which can include the criminally negligent operation of a motor vehicle.

The man likely satisfied the "actus reus" of criminally negligent manslaughter. He engaged in the act of
driving his car in a manner that violated the traffic laws and his high speed and weaving through traffic
created a substantial risk of an accident and serious injury to others. His conduct also was both the actual
(“but for”) and proximate cause of the motorcyclist’s death. The death was the direct result of the man
striking the motorcyclist with his car, and there were no unforeseeable superseding causes.

The only question is whether the man acted with the requisite mens rea required for manslaughter. In
some states, criminal negligence is present whenever a person’s conduct creates a substantial risk of death
and the defendant’s actions were a gross deviation from the standard of care that a reasonable person
would have exercised in the same situation. In other states, it is also necessary to prove that the defendant
was aware of the fact that the conduct created a substantial risk of death.

Here, the man was driving at the excessive speed of 85 mph on a busy highway, was weaving in and out
of traffic, and tried to pass a motorcycle on a curve. Based on these facts, a fact-finder could conclude that
the man had the requisite mens rea for criminal-negligence manslaughter. An ordinary person would have
been aware that this kind of driving created a substantial and unjustifiable risk that a driver would lose
control of the car, hit the motorcyclist or others on the highway, and cause death. The man’s actions were
also a gross deviation from the standard of care that a reasonable person would have exercised in the same

Seperac-J19 Exam-Released MEE Essay Compilation © 2016-2020 334


situation. Courts have upheld jury verdicts finding that defendants acted with criminal negligence under
similar circumstances. (e.g. a court denied a defendant’s motion for a judgment of acquittal when the jury
found him guilty of negligent homicide after he struck and killed the victim while going 20 mph over the
speed limit and not paying attention to the road). If State A requires actual awareness of a substantial risk
of death, a manslaughter charge would still be warranted because the man must have been aware that his
driving could easily cause a fatal accident.

In some states, manslaughter is also committed when a death occurs as a result of a defendant’s
commission of an unlawful act. Here, the man’s excessive speeding and dangerous weaving through
traffic were both serious traffic violations sufficient to serve as predicate offenses for a charge of
unlawful-act manslaughter.

Explanation to Point-Three (20%):

The facts support the affirmative defense of duress because the man’s conduct was caused by his
reasonable belief that obeying the woman’s instructions to drive faster was the only way to avoid
imminent death or serious bodily injury.

The typical affirmative defense of duress excuses defendants from criminal liability if their conduct was
committed “under the pressure of an unlawful threat from another human being to harm” the defendant.
The unlawful threat must cause the defendant to reasonably believe that “the only way to avoid imminent
death or serious bodily injury to himself or to another is to engage in conduct which violates the literal
terms of the criminal law.” The defendant must also prove that he engaged in the criminal behavior
because of the threat and not for some other reason. The rationale for the duress defense is that “even
though he has done the act the crime requires and has the mental state which the crime requires, his
conduct violating the literal language of the criminal law is excused because he ‘lacked a fair opportunity
to avoid acting unlawfully.’”

Here, the man almost certainly will succeed with an affirmative defense of duress. His physical conduct
(driving at an excessive rate of speed and in a reckless manner) was in response to the woman’s direct
threat of death or serious bodily harm. Under the circumstances, it was reasonable for him to fear for his
life because his assailant was holding a knife to his throat in a car, and her behavior indicated that she was
mentally unbalanced, making it more likely that she would carry out her threat. Because the man was
already driving the car on a busy highway when the woman pulled out the knife and placed it against his
neck, he had no opportunity to extricate himself from the threatening situation.

The Model Penal Code defines the affirmative defense of duress as a threat such that a person of
reasonable firmness would be unable to resist it. This standard, which is followed in several jurisdictions,
is also satisfied here. A person of reasonable firmness of character would very likely be unable to resist
the threat posed by the woman under these circumstances. At the common law, and in most jurisdictions
today, duress is not available as a defense to any kind of intentional homicide. In this case, however, the
man had no intention of killing the motorcyclist. Thus, there is no bar to his asserting the defense against
a charge of negligent homicide.

Seperac-J19 Exam-Released MEE Essay Compilation © 2016-2020 335


#077-JUL 2014–MEE Q01: QUESTION ONE (CRIMINAL LAW & PROCEDURE)

While on routine patrol, a police officer observed a suspect driving erratically and pulled the suspect’s car
over to investigate. When he approached the suspect’s car, the officer detected a strong odor of marijuana.
The officer immediately arrested the suspect for driving under the influence of an intoxicant (DUI). While
the officer was standing near the suspect’s car placing handcuffs on the suspect, the officer observed
burglary tools on the backseat.

The officer seized the burglary tools. He then took the suspect to the county jail, booked him for the DUI,
and placed him in a holding cell. Later that day, the officer gave the tools he had found in the suspect’s
car to a detective who was investigating a number of recent burglaries in the neighborhood where the
suspect had been arrested.

At the time of his DUI arrest, the suspect had a six-month-old aggravated assault charge pending against
him and was being represented on the assault charge by a lawyer.

Early the next morning, upon learning of her client’s arrest, the lawyer went to the jail. She arrived at 9:00
a.m., immediately identified herself to the jailer as the suspect’s attorney, and demanded to speak with the
suspect. The lawyer also told the jailer that she did not want the suspect questioned unless she was
present. The jailer told the lawyer that she would need to wait one hour to see the suspect. After speaking
with the lawyer, the jailer did not inform anyone of the lawyer’s presence or her demands.

The detective, who had also arrived at the jail at 9:00 a.m., overheard the lawyer’s conversation with the
jailer. The detective then entered the windowless interview room in the jail where the suspect had been
taken 30 minutes earlier. Without informing the suspect of the lawyer’s presence or her demands, the
detective read to the suspect full and accurate Miranda warnings. The detective then informed the suspect
that he wanted to ask about the burglary tools found in his car and the recent burglaries in the
neighborhood where he had been arrested. The suspect replied, “I think I want my lawyer here before I
talk to you.” The detective responded, “That’s up to you.”

After a few minutes of silence, the suspect said, “Well, unless there is anything else I need to know, let’s
not waste any time waiting for someone to call my attorney and having her drive here. I probably should
keep my mouth shut, but I’m willing to talk to you for a while.” The suspect then signed a Miranda
waiver form and, after interrogation by the detective, made incriminating statements regarding five
burglaries. The interview lasted from 9:15 a.m. to 10:00 a.m.

In addition to the DUI, the suspect has been charged with five counts of burglary.

The lawyer has filed a motion to suppress all statements made by the suspect to the detective in
connection with the five burglaries.

The state supreme court follows federal constitutional principles in all cases interpreting a criminal
defendant’s rights.

1. Did the detective violate the suspect’s Sixth Amendment right to counsel when he questioned the
suspect in the absence of the lawyer? Explain.

2. Under Miranda, did the suspect effectively invoke his right to counsel? Explain.

Seperac-J19 Exam-Released MEE Essay Compilation © 2016-2020 336


3. Was the suspect’s waiver of his Miranda rights valid? Explain.

Seperac-J19 Exam-Released MEE Essay Compilation © 2016-2020 337


#077: J14-1 MEE: ANSWER: NCBE (CRIMINAL LAW & PROCEDURE)

POINT (1) [35%] ISSUE: Did the detective violate the suspect’s Sixth Amendment right to counsel
when he questioned the suspect about the burglaries without the lawyer present, given that the
lawyer represented the suspect in an unrelated criminal matter? ANSWER: No. The suspect’s Sixth
Amendment right to counsel was not violated because the right does not attach on new charges until
formal adversarial judicial proceedings have commenced on those charges.

POINT (2) [30%] ISSUE: Under Miranda, did the suspect effectively invoke his right to counsel
when he said, “I think I want my lawyer here before I talk to you”? ANSWER: No. The suspect did
not effectively invoke his right to counsel under Miranda because his statement was not
unambiguous.

POINT (3) [35%] ISSUE: Was the suspect’s waiver of his right to remain silent under Miranda
valid? ANSWER: Yes. The suspect’s waiver of his Miranda rights was knowing, intelligent, and
voluntary despite the fact that he was never told of the lawyer’s presence in the jail or of the
lawyer’s demands.

ANSWER DISCUSSION:

The Sixth Amendment right to counsel, as applied to states through the Fourteenth Amendment, is
offense-specific. Although the suspect had an attorney representing him on his pending assault charge, he
had no Sixth Amendment right to the assistance of counsel with respect to the five uncharged burglaries
because formal adversarial proceedings had not yet commenced on those charges. The suspect’s Sixth
Amendment right to counsel was not violated by the detective’s failure to inform him that the lawyer was
present or of the lawyer’s demands. However, a person undergoing custodial interrogation also has an
independent constitutional right to counsel during custodial interrogation under Miranda. When a suspect
invokes his right to counsel under Miranda, custodial interrogation must immediately cease for a period of
at least 14 days. However, the invocation of the right to counsel must be unambiguous and clearly convey
that the suspect has requested counsel. Here, because the suspect’s statement, “I think I want my lawyer
here before I talk to you,” was ambiguous, he did not invoke his Miranda right to counsel. A waiver of
rights must be knowing, intelligent, and voluntary. Here, the suspect waived his right to remain silent
under Miranda when he signed the waiver form. The fact that the detective did not correct the suspect’s
assumption that the lawyer would need to drive to the jail – by telling him that the lawyer was in the
waiting room and was demanding to see him – did not affect the validity of the suspect’s waiver.

ANSWER EXPLANATION:

Explanation to Point-One (35%):

The suspect’s Sixth Amendment right to counsel was not violated because the right does not attach on
new charges until formal adversarial judicial proceedings have commenced on those charges.

The Sixth Amendment, as applied to the states through the Fourteenth Amendment, provides that “in all
criminal prosecutions, the accused shall enjoy the right to have the Assistance of Counsel for his defense.”
The right to counsel does not attach with respect to particular charges until formal adversarial judicial
proceedings have commenced (i.e., “at or after the initiation of adversary judicial criminal proceedings –
whether by way of formal charge, preliminary hearing, indictment, information, or arraignment (or in

Seperac-J19 Exam-Released MEE Essay Compilation © 2016-2020 338


some states, arrest warrant),” Once a suspect’s Sixth Amendment right to counsel has attached, any
attempts to “deliberately elicit” statements from him in the absence of his attorney violate the Sixth
Amendment.

The Sixth Amendment right to counsel is charge- or offense-specific. Representation by counsel in one
prosecution does not, in itself, guarantee counsel for uncharged offenses. Here, the suspect’s Sixth
Amendment right to counsel had attached only for the pending aggravated assault charge. The suspect’s
right to counsel for the aggravated assault case did not guarantee counsel for the five unrelated and
uncharged burglaries that were the subject of the detective’s interrogation. Thus, because formal
adversarial judicial proceedings against the suspect for the uncharged burglaries had not begun, he had no
Sixth Amendment right to counsel.

Finally, the detective’s failure to inform the suspect of the lawyer’s presence and demands to speak with
him does not implicate the suspect’s Sixth Amendment right to counsel, which had not yet attached.

Explanation to Point-Two (30%):

The suspect did not effectively invoke his right to counsel under Miranda because his statement was not
unambiguous.

A suspect subject to custodial interrogation has a right to consult with counsel and to have an attorney
present during questioning. When a suspect invokes his right to counsel during an interrogation, law
enforcement must immediately cease all questioning. Custodial interrogation cannot be reinitiated unless
and until the suspect has been re-advised of his Miranda rights, has provided a knowing and voluntary
waiver, and (1) counsel is present and (2) the suspect himself initiated further communication with the
police, or (3) if the suspect was released from custody after the initial interrogation, at least 14 days have
passed.

To invoke the right to counsel, a suspect’s request must be “unambiguous.” This means that the suspect
must articulate the desire for counsel sufficiently clearly that a reasonable officer would understand the
statement to be a request for counsel. If the request is ambiguous, the police are not required to stop the
interrogation.

In this case, the suspect’s statement, “I think I want my lawyer here before I talk to you,” was not an
unambiguous request for counsel. The most reasonable interpretation of this statement is that the suspect
might be invoking his right to counsel. Courts have held that a person stating “maybe I should talk to a
lawyer” or “I think I need a lawyer” are not unambiguous requests for an attorney.

Under these circumstances, the detective was not required to cease the custodial interrogation of the
suspect. Nor was the detective required to clarify or ask follow-up questions to determine whether the
suspect in fact wanted an attorney.

Explanation to Point-Three (35%):

The suspect’s waiver of his Miranda rights was knowing, intelligent, and voluntary despite the fact that he
was never told of the lawyer’s presence in the jail or of the lawyer’s demands.

A valid waiver of Miranda rights must be “voluntary” – i.e., the product of a free or deliberate choice
rather than intimidation, coercion, or deception. In addition, the waiver must be knowing and intelligent.

Seperac-J19 Exam-Released MEE Essay Compilation © 2016-2020 339


That is, it “must have been made with a full awareness of both the nature of the right being abandoned
and the consequences of the decision to abandon it.”

In this case, the suspect signed a Miranda waiver form after receiving proper warnings. There is no
evidence “that the police resorted to physical or psychological pressure to elicit the statements.” The
entire interview lasted only 45 minutes. The only issue is whether the suspect knowingly and intelligently
waived his Miranda rights despite the fact that the detective did not tell the suspect about the lawyer’s
presence and her demands.

The Supreme Court has said that “events occurring outside of the presence of the suspect and entirely
unknown to him surely can have no bearing on the capacity to comprehend and knowingly relinquish a
constitutional right.” If the suspect “knew that he could stand mute and request a lawyer, and was aware
of the State’s intention to use his statements to secure a conviction,” then the waiver is valid regardless of
the information withheld.

Here, the suspect was correctly informed of his rights. His comments demonstrate that he understood that
he could have a lawyer present if he desired (i.e., wondering whether he should call his attorney) and that
he understood that there might be consequences to speaking with the detective (“I probably should keep
my mouth shut, but I’m willing to talk to you for a while.”). His comment, “Let’s not waste any time
waiting for someone to call my attorney and having her drive here,” along with his signature on the
Miranda waiver form, show that his waiver was valid under the constitutional standard.

The fact that the detective did not tell the suspect about the lawyer’s presence and demands has no bearing
on the validity of the suspect’s waiver because “such conduct is only relevant to the constitutional validity
of a waiver if it deprives a defendant of knowledge essential to his ability to understand the nature of his
rights and the consequences of abandoning them.” The Supreme Court has specifically declined to adopt a
rule requiring that law enforcement tell a suspect of an attorney’s efforts to contact him.

[NOTE: An examinee might also recognize that this general rule is further supported by the Supreme
Court’s decision in Florida v. Powell, approving state Miranda warnings that do not explicitly warn
suspects that they have a right to have counsel present during custodial interrogation.]

Seperac-J19 Exam-Released MEE Essay Compilation © 2016-2020 340


#078-FEB 2014–MEE Q05: QUESTION FIVE (CRIMINAL LAW & PROCEDURE)

A defendant was charged under state law with felony theft (Class D) and felony residential burglary
(Class C). The indictment alleged that the defendant entered his neighbors’ home without their consent
and stole a diamond ring worth at least $2,500.

Defense counsel filed a pretrial motion to dismiss the charges on the ground that prosecuting the
defendant for both burglary and theft would constitute double jeopardy. The trial court denied the motion,
and the defendant was prosecuted for both crimes. The only evidence of the ring’s value offered at the
defendant’s jury trial was the owner’s testimony that she had purchased the ring two years earlier for
$3,000.

At trial, the judge issued the following jury instruction on the burglary charge prior to deliberations:

If, after consideration of all the evidence presented by the prosecution and defense, you find beyond a
reasonable doubt that the defendant entered the dwelling without the owners’ consent, you may presume
that the defendant entered with the intent to commit a felony therein.

The jury found the defendant guilty of both offenses.

At the defendant’s sentencing hearing, an expert witness called by the prosecutor testified that the
diamond ring was worth between $7,000 and $8,000. Over defense objection, the judge concluded, by a
preponderance of the evidence, that the value of the stolen ring exceeded $5,000. The judge sentenced the
defendant to four years’ incarceration on the theft conviction. On the burglary conviction, the defendant
received a consecutive sentence of seven years’ incarceration.

In this state, residential burglary is defined as “entry into the dwelling of another, without the consent of
the lawful resident, with the intent to commit a felony therein.” Residential burglary is a Class C felony
for which the minimum sentence is five years and the maximum sentence is ten years of incarceration.

In this state, theft is defined as “taking and carrying away the property of another with the intent to
permanently deprive the owner of possession.” Theft is a Class D felony if the value of the item(s) taken
is between $2,500 and $10,000. The sentence for a Class D felony theft is determined by the value of the
items taken. If the value is between $2,500 and $5,000, the maximum sentence is three years’
incarceration. If the value of the items exceeds $5,000, the maximum sentence is five years’ incarceration.

This state affords a criminal defendant no greater rights than those mandated by the United States
Constitution.

1. Did the trial court err when it denied the defendant’s pretrial motion to dismiss on double
jeopardy grounds? Explain.

2. Did the trial court err in its instruction to the jury on the burglary charge? Explain.

3. Did the trial court err when it sentenced the defendant to an additional year of incarceration on
the theft conviction based on the expert’s testimony? Explain.

Seperac-J19 Exam-Released MEE Essay Compilation © 2016-2020 341


#078: F14-5 MEE: ANSWER: NCBE (CRIMINAL LAW & PROCEDURE)

POINT (1) [30%] ISSUE: Did charging the defendant with both theft and burglary constitute
double jeopardy? ANSWER: No. Charging the defendant with theft and burglary did not constitute
double jeopardy.

POINT (2) [35%] ISSUE: Did the jury instruction violate the due process clause either by relieving
the prosecution of the burden of proving the element of intent or by shifting the burden to the
defendant to disprove that element? ANSWER: Yes. The jury instruction on the burglary charge
violated the Due Process Clause because it created either (1) an irrebuttable conclusive
presumption (which relieved the prosecution of proving the element of intent and removed that
issue from the jury) or (2) a rebuttable mandatory presumption (which unconstitutionally shifted
the burden of proof on an element of a charged offense to the defendant).

POINT (3) [35%] ISSUE: Did the sentence imposed in this case for the theft conviction
unconstitutionally deprive the defendant of his right to a jury trial on the issue of the value of the
stolen item? ANSWER: Yes. The trial court violated the defendant’s Sixth Amendment right to a
jury trial on an essential element of the offense when it found, by a preponderance of the evidence,
that the ring was worth over $5,000 and increased the defendant’s sentence based on this finding.

ANSWER DISCUSSION:

The trial court properly denied the defendant’s pretrial motion to dismiss the charges on double jeopardy
grounds. The defendant may be charged with, and convicted of, both theft and burglary. Each of the
charges has an element that the other does not. Neither charge is a lesser-included offense, nor are they
multiplicitous. Thus, charging both theft and burglary does not violate double jeopardy. The jury
instruction on the burglary charge was constitutionally flawed. It could have been reasonably understood
by the jury as either (1) an irrebuttable conclusive presumption (which relieved the prosecution of proving
the element of intent and removed the issue from the jury) or (2) a rebuttable mandatory presumption
(which unconstitutionally shifted the burden of proof on an element of a charged offense from the
prosecution to the defendant). Because the four-year sentence imposed by the judge was based on the
judge’s finding, by a preponderance of the evidence, that the value of the stolen ring exceeded $5,000, the
sentence violates the defendant’s right to a jury determination, beyond a reasonable doubt, of the value of
the ring.

ANSWER EXPLANATION:

Explanation to Point-One (30%):

Charging the defendant with theft and burglary did not constitute double jeopardy.

The Double Jeopardy Clause of the Fifth Amendment provides that a person shall not be twice put in
jeopardy for the “same offense.” Thus, the question is whether the elements of the theft charge are wholly
contained in the burglary charge or vice versa. If the elements of the lesser charge (theft) are not wholly
contained in the greater charge (burglary) – i.e., if each charge requires proof of a fact that the other does
not – then convicting the defendant of both crimes would not violate double jeopardy even when the two
offenses occurred at the same time and are thus arguably part of the “same transaction.”

Seperac-J19 Exam-Released MEE Essay Compilation © 2016-2020 342


Here, theft and burglary each require proof of an element not required for the other crime. Burglary may
be defined differently in different jurisdictions. However, it almost invariably requires entry into a
building or dwelling of another with the specific intent to commit a felony therein, and the crime of
burglary is complete upon the entry into the building or dwelling with such intent. In contrast, theft,
which also may be defined differently in different states, almost invariably requires the taking and
carrying away of an item of personal property belonging to another with the intent to steal or permanently
deprive the owner of possession.

Here, the “taking” or “stealing” element is not contained in the definition of burglary, and the “entry”
element of burglary is not contained in the definition of theft. Because theft is not a lesser-included
offense of burglary and burglary is not a lesser-included offense of theft, charging the defendant for both
burglary and theft did not violate double jeopardy and the court properly denied the defense motion on
those grounds.

Finally, the defendant’s motion to dismiss all the charges on double jeopardy grounds was improper
because, if both charges were for the same offense, the motion should have requested dismissal of one
charge, not both.

Explanation to Point-Two (35%):

The jury instruction on the burglary charge violated the Due Process Clause because it created either (1)
an irrebuttable conclusive presumption (which relieved the prosecution of proving the element of intent
and removed that issue from the jury) or (2) a rebuttable mandatory presumption (which
unconstitutionally shifted the burden of proof on an element of a charged offense to the defendant).

The Supreme Court has interpreted the Due Process Clause of the U.S. Constitution to require that the
prosecution prove all elements of an offense beyond a reasonable doubt. The burden of proof cannot be
shifted to the defendant by presuming an essential element upon proof of other elements of the offense,
because shifting the burden of persuasion with respect to any element of a criminal offense is contrary to
the Due Process Clause.

The crime of burglary includes entry into a building or dwelling with the specific intent to commit a
felony therein. The requirement that the prosecutor prove, beyond a reasonable doubt, that the defendant
had this specific intent distinguishes burglary from general-intent crimes like trespass.

Here, the jury was instructed that if, “after consideration of all the evidence presented by the prosecution
and defense, you find beyond a reasonable doubt that the defendant entered the dwelling without the
owners’ consent, you may presume that the defendant entered with the intent to commit a felony therein.”
This instruction was unconstitutional because it created either an irrebuttable conclusive presumption or a
rebuttable mandatory presumption.

A conclusive presumption is “an irrebuttable direction by the court to find intent once convinced of the
facts triggering the presumption.” Here, the jurors were instructed that once the prosecutor established
that the defendant entered the neighbors’ house without consent, they “may presume” that he intended to
commit a felony therein. The jurors may have reasonably concluded from this instruction that if they
found that the defendant intended to enter his neighbors’ home without permission, they must further find
that he entered with the specific intent to commit a felony therein. Because this instruction could operate
as a conclusive irrebuttable presumption by eliminating intent “as an ingredient of the offense,” it violated
due process by relieving the prosecution of the burden of proof for this element.

Seperac-J19 Exam-Released MEE Essay Compilation © 2016-2020 343


In the alternative, the jury instruction could have been reasonably understood to create a rebuttable
mandatory presumption, which “tells the jury they must find the elemental fact upon proof of the basic
fact, at least unless the defendant has come forward with some evidence to rebut the presumed connection
between the two facts.” The due process problem created by rebuttable mandatory presumptions is that
“to the extent that the trier of fact is forced to abide by the presumption, and may not reject it based on an
independent evaluation of the particular facts presented by the State, the analysis of the presumption’s
constitutional validity is logically divorced from those facts and based on the presumption’s accuracy in
the run of cases.”

Unlike irrebuttable conclusive presumptions, rebuttable mandatory presumptions are not always per se
violations of the Due Process Clause. However, the Supreme Court of the United States has held that jury
instructions that could reasonably be understood as shifting the burden of proof to the defendant on an
element of the offense are unconstitutional. Here, the argument that the jury instruction operated as a
rebuttable mandatory presumption is supported by the fact that the judge also instructed the jury to
“consider all the evidence presented by the prosecution and defense.” However, even if the instruction
created a rebuttable mandatory presumption, it would be unconstitutional because it shifted the burden to
the defense on an element of the offense.

[NOTE: Whether an examinee identifies the jury instruction as containing a “conclusive” or


“mandatory” presumption is less important than the examinee’s analysis of the constitutional infirmities.]

Explanation to Point-Three (35%):

The trial court violated the defendant’s Sixth Amendment right to a jury trial on an essential element of
the offense when it found, by a preponderance of the evidence, that the ring was worth over $5,000 and
increased the defendant’s sentence based on this finding.

In the statutory scheme under which the defendant was tried and convicted, a Class D felony theft is
defined as theft of item(s) with a value between $2,500 and $10,000. The jury found that the value of the
diamond ring was at least $2,500 and convicted the defendant of felony theft. However, at sentencing, the
trial court made a separate finding, by a preponderance of the evidence, that the value of the ring was
greater than $5,000. Following the statute’s two-tiered sentencing scheme, the judge then imposed on the
defendant a sentence that was one year longer than the maximum that would otherwise have been
allowed.

The judge’s sentence was unconstitutional because it violated the defendant’s Sixth Amendment right to a
jury trial on this question. The Supreme Court held in Apprendi v. New Jersey, that “other than the fact of
a prior conviction, any fact that increases the penalty for a crime beyond the prescribed statutory
maximum must be submitted to a jury, and proved beyond a reasonable doubt” because “it is
unconstitutional for a legislature to remove from the jury the assessment of facts that increase the
prescribed range of penalties to which a criminal defendant is exposed because such facts must be
established by proof beyond a reasonable doubt.” The Court reaffirmed Apprendi in Blakely v.
Washington, holding that the “‘statutory maximum’ for Apprendi purposes is the maximum sentence a
judge may impose solely on the basis of the facts reflected in the jury verdict or admitted by the
defendant.” In United States v. Booker, the Court relied on Blakely and Apprendi to conclude that
protecting a defendant’s Sixth Amendment right to a jury trial required that “any fact which is necessary
to support a sentence exceeding the maximum authorized by the facts established by a plea of guilty or a
jury verdict must be admitted by the defendant or proved to a jury beyond a reasonable doubt.”

Seperac-J19 Exam-Released MEE Essay Compilation © 2016-2020 344


Thus, in order to constitutionally increase a sentence above the statutory maximum of three years, the jury
must have found beyond a reasonable doubt that the value of the ring exceeded $5,000. Here, the court
made the finding based on an appraisal proffered by the prosecutor only at sentencing, and the judge’s
finding was by a preponderance of the evidence rather than beyond a reasonable doubt.

Seperac-J19 Exam-Released MEE Essay Compilation © 2016-2020 345


#079-JUL 2012–MEE Q02: QUESTION TWO (CRIMINAL LAW & PROCEDURE)

At 9:00 p.m. on a Sunday evening, Adam, age 18, proposed to his friend Bob, also age 18, that they dump
Adam’s collection of 2,000 marbles at a nearby intersection. “It’ll be funny,” Adam said. “When cars
come by, they’ll slip on the marbles and they won’t be able to stop at the stop sign. The drivers won’t
know what happened, and they’ll get really mad. We can hide nearby and watch.” “That’s a stupid idea,”
Bob said. “In the first place, this town is deserted on Sunday night. Nobody will even drive through the
intersection. In the second place, I’ll bet the cars just drive right over the marbles without any trouble at
all. It’ll be a total non-event.” “Oh, I’ll bet someone will come,” Adam replied. “And I’ll bet they’ll have
trouble; maybe there will even be a crash. But if you’re not interested, fine. You don’t have to do
anything. Just give me a ride to the intersection – these bags of marbles are heavy.”

At 10:00 p.m. that same night, Bob drove Adam and his bags of marbles to the intersection. Adam
dumped several hundred marbles in front of each of the two stop signs at the intersection. Adam and Bob
stayed for 20 minutes, waiting to see if anything happened. No one drove through the intersection, and
Adam and Bob went home.

At 2:00 a.m., a woman drove through the intersection. Because of the marbles, she was unable to stop at
the stop sign. Coincidentally, a man was driving through the intersection at the same time. The woman
crashed into the side of the man’s car. The man’s eight-year-old child was sitting in the front seat without
a seat belt, in violation of state law. The child was thrown from the car and killed. If the child had been
properly secured with a seat belt, as required by state law, he would likely not have died.

Adam has been charged with involuntary manslaughter as defined at common law, and Bob has been
charged with the same crime as an accomplice. State law does not recognize so-called “unlawful-act”
involuntary manslaughter.

1. Could a jury properly find that Adam is guilty of involuntary manslaughter? Explain.

2. If a jury did find Adam guilty of involuntary manslaughter, could the jury properly find that Bob
is guilty of involuntary manslaughter as an accomplice? Explain.

Seperac-J19 Exam-Released MEE Essay Compilation © 2016-2020 346


#079: J12-2 MEE: ANSWER: NCBE (CRIMINAL LAW & PROCEDURE)

POINT (1)(a) [25%] ISSUE: What are the elements of involuntary manslaughter? ANSWER: To be
guilty of involuntary manslaughter, a person must cause the death of another human being by
conduct that creates an unreasonable (or high and unreasonable) risk of death or serious bodily
injury. The precise mens rea requirement will vary from jurisdiction to jurisdiction.

POINT (1)(b) [25%] ISSUE: Did Adam act with mens rea required to be guilty of involuntary
manslaughter? ANSWER: Yes. A jury could conclude from Adam’s statements and actions that he
acted with a mens rea of recklessness or gross negligence.

POINT (1)(c) [25%] ISSUE: Were Adam’s actions the legal cause of the child’s death? ANSWER:
Yes. Adam’s conduct was the but-for cause of the child’s death. His conduct was also the legal cause
because no intervening event occurred that would be sufficient to break the chain of causation.

POINT (2) [25%] ISSUE: Can Bob be liable as an accomplice for the child’s death? ANSWER: No.
Bob intentionally assisted Adam in his act of dumping the marbles on the road, but Bob may not
have acted with the mens rea required to be held liable as an accomplice to involuntary
manslaughter.

ANSWER DISCUSSION:

A jury could find Adam guilty of involuntary manslaughter both in a jurisdiction that requires
recklessness and in one that requires only gross negligence (sometimes referred to as criminal or culpable
negligence). In a jurisdiction that requires recklessness, Adam’s statements indicate that he was aware that
his conduct created an unreasonable (or high and unreasonable) risk of death or serious bodily injury and
that he consciously disregarded that risk. In a jurisdiction that requires gross negligence, a jury could find
Adam guilty because a reasonable person under similar circumstances would have been aware that his
conduct created an unreasonable (or unreasonable and high) risk of death or serious bodily injury. Adam’s
conduct was the cause of the child’s death. Adam’s conduct was the but-for cause of the child’s death
because if Adam had not placed the marbles on the road, the child would not have died. The fact that the
child was not properly secured in the vehicle does not break the chain of causation because this outcome
was foreseeable. Bob did not commit an act that resulted in the death of the child, nor did he have a duty
to stop Adam from committing that act. However, a jury could find Bob liable as an accomplice to
Adam’s involuntary manslaughter if he assisted in the act with the required mental state. Here, Bob
intentionally assisted Adam in the act by driving him to the scene of the crime. In a jurisdiction that
requires recklessness, Bob’s statements indicate that he was not aware that his assistance created an
unreasonable (or high and unreasonable) risk of death or serious bodily injury. Bob is more likely to be
found guilty as an accomplice in a state that requires gross/criminal/culpable negligence, if a reasonable
person under similar circumstances would have been aware of this risk.

ANSWER EXPLANATION:

Explanation to Point-One(a) (25%):

To be guilty of involuntary manslaughter, a person must cause the death of another human being by
conduct that creates an unreasonable (or high and unreasonable) risk of death or serious bodily injury. The
precise mens rea requirement will vary from jurisdiction to jurisdiction.

Seperac-J19 Exam-Released MEE Essay Compilation © 2016-2020 347


In most jurisdictions, a defendant is guilty of involuntary manslaughter when the defendant causes the
death of another human being by engaging in conduct that creates an unreasonable (or high and
unreasonable) risk of death or serious bodily injury.

The modern and majority view is that the defendant must have acted “recklessly” to be convicted of
involuntary manslaughter (i.e., the defendant must have been aware of the unreasonable (or unreasonable
and high) risk of death or serious bodily injury that his conduct created). Recklessness is typically defined
as conscious disregard of a known risk.

In other jurisdictions it is enough if the defendant acted with greater than ordinary negligence, which
some states call “gross,” “criminal,” or “culpable” negligence. In these states, even if the defendant was
unaware of the risk, the defendant could be found guilty if an ordinary person in the defendant’s situation
would have been aware that her conduct created an unreasonable (or unreasonable and high) risk of death
or serious bodily injury.

Adam will be guilty of involuntary manslaughter only if he acted with the requisite degree of culpability
and his conduct was the legal cause of the child’s death.

Explanation to Point-One(b) (25%):

A jury could conclude from Adam’s statements and actions that he acted with a mens rea of recklessness
or gross negligence.

In order to determine whether Adam has committed involuntary manslaughter, his mens rea with regard to
the result must be assessed.

In most jurisdictions, Adam can only be convicted of involuntary manslaughter if the fact finder
concludes that he acted “recklessly” by consciously disregarding the unreasonable (or high and
unreasonable) risk of death or serious bodily injury that his conduct created. In these jurisdictions, the
prosecution will argue that Adam’s statements indicating that he knew that the marbles could cause an
accident, including his statements that “When cars come by, they’ll slip on the marbles and they won’t be
able to stop,” and that “They’ll have trouble; maybe there will even be a crash,” demonstrate that Adam
was aware of the risk. The prosecution will also argue that Adam (like everyone else) knows that serious
bodily injury and death are frequent outcomes of car crashes.

Adam will likely respond in several ways. First, he will contend that he was not reckless because he did
not consciously disregard an unreasonable (or unreasonable and high) risk of death or serious bodily
injury resulting from the accident. His statements indicated, he will argue, that he believed that drivers
would lose control of their vehicles and would get mad and that there might be a crash, but did not
suggest any awareness of the risk that someone would die or suffer serious bodily injury. He could also
argue that no crash occurred while he watched the intersection, that traffic at the intersection was usually
light, and that he was not aware of the risk that two cars would crash into each other, particularly at 2 a.m.

On the whole, a jury could conclude that Adam’s statements indicate that he was clearly aware that his
actions could cause a car crash and every reasonable person knows that car crashes can cause death or
serious bodily injury. Adam’s statement “I’ll bet someone will come” specifically suggests that he
believed that cars would drive through the intersection that evening.

To prove that Adam had a mens rea of recklessness, the prosecution does not need to prove that Adam
knew the precise time or circumstances of the resulting crash. If, however, the fact finder accepts that

Seperac-J19 Exam-Released MEE Essay Compilation © 2016-2020 348


Adam did not consciously disregard an unreasonable (or unreasonable and high) risk of death or serious
bodily injury from the resulting crash, he would not be guilty of involuntary manslaughter in a jurisdiction
that requires recklessness for the crime.

In some jurisdictions, Adam can be convicted of involuntary manslaughter if the fact finder concludes that
he acted with “gross,” “criminal,” or “culpable” negligence. In these jurisdictions, Adam can be convicted
if a reasonable person in his situation would have been aware that his conduct created an unreasonable (or
unreasonable and high) risk of death or serious bodily injury. It is highly likely that a fact finder would
conclude that a reasonable person would be aware that conduct likely to cause vehicles to slip at an
intersection creates an unreasonable (or unreasonable and high) risk of car crashes and death or serious
bodily injury as a result of the crashes.

Explanation to Point-One(c) (25%):

Adam’s conduct was the but-for cause of the child’s death. His conduct was also the legal cause because
no intervening event occurred that would be sufficient to break the chain of causation.

The state will need to prove that Adam caused the child’s death in order for Adam to be found guilty of
involuntary manslaughter. Causation requires a showing of both causation in fact – otherwise known as
but-for causation – and proximate causation. “A defendant’s conduct is a cause-in-fact of the prohibited
result if the said result would not have occurred ‘but for’ the defendant’s conduct.” Adam was plainly the
but-for cause of the child’s death. If Adam had not dumped the marbles at the intersection, the child
would not have died.

The next question is whether Adam’s conduct was also the “proximate” cause of the child’s death.
Proximate cause under the criminal law is complex to define, but its core is foreseeability. Adam may
argue that the child would not have died but for the fact that the man did not have his child properly
secured in the vehicle. He may argue that this failure was an unforeseeable intervening cause – another
but-for cause of the child’s death – and that in this case it should be treated as a superseding cause that
should cut off his liability for the result. He may bolster this argument by noting that seat belts are
required under state law. This argument should fail. It is true that the man’s failure to properly secure the
child was an intervening cause of the child’s death. However, this intervening cause was not
unforeseeable. Adam should have anticipated that some people, including children, who ride in cars are
not properly secured by seat belts or child restraints.

Adam might also argue that the resulting car crash was such an extraordinary consequence of his action of
dumping the marbles at the intersection that it would be unfair to hold him accountable. This argument
should also fail because in these circumstances a car crash resulting in the death of a child who was not
wearing a seat belt is not such an unusual or extraordinary consequence of Adam’s act as to justify
relieving him of liability. Where a party by his wrongful conduct creates a condition of peril, his action
can properly be found to be the proximate cause of a resulting injury, even though later events which
combined to cause the injury may also be classified as negligent, so long as the later act is something
which can reasonably be expected to follow in the natural sequence of events.

Explanation to Point-Two (25%):

Bob intentionally assisted Adam in his act of dumping the marbles on the road, but Bob may not have
acted with the mens rea required to be held liable as an accomplice to involuntary manslaughter.

Seperac-J19 Exam-Released MEE Essay Compilation © 2016-2020 349


Two elements are necessary for Bob to be found guilty of involuntary manslaughter on a theory of
accomplice liability. First, Bob must have assisted Adam in the commission of the crime. Second, Bob
must have acted with dual intentions: (1) “the intent to assist the primary party” and (2) “the intent that
the primary party commit the offense charged.” In cases where the primary party’s crime is one involving
recklessness or negligence, most jurisdictions hold that the second intent element is satisfied if the
defendant intended to assist the primary party and otherwise acted with the mens rea required for the
underlying offense (i.e., recklessness or negligence, as the jurisdiction requires).

Here, Bob assisted Adam by driving him to the intersection. Even a small amount of assistance can
suffice to create accomplice liability.

[NOTE: An examinee might also argue that Bob assisted Adam by staying with him and watching, on the
theory that he provided some kind of encouragement to Adam. However, mere presence at a crime scene
is not sufficient to constitute assistance, so accompanying Adam is probably not enough.]

Bob also appears to have acted with the intent of assisting Adam. When Bob drove Adam to the
intersection, he knew that it was for the purpose of bringing Adam to that place so that Adam could dump
the marbles in the intersection. Bob had no other purpose.

The next question is whether Bob’s mens rea with regard to the child’s death was sufficient for
involuntary manslaughter. Bob will argue that he had no mens rea at all with regard to the child’s death
and therefore can be guilty of no offense. He will also argue that his statements to Adam suggest that he
believed (1) that no one was likely to drive through the intersection and (2) that the marbles would not
cause any danger to any drivers who would simply drive over them. Accordingly, Bob will argue that he
did not act with the culpability required to be held liable as an accomplice to involuntary manslaughter.

In most jurisdictions, Bob must have acted “recklessly” to be convicted as an accomplice to involuntary
manslaughter (i.e., he must have been aware of the unreasonable (or unreasonable and high) risk of death
or serious bodily injury that his conduct created). Based on Bob’s statements, he does not appear to have
been reckless. The prosecution’s best argument would be that Bob knew about the risk because Adam told
him that cars might crash. In response, Bob can argue that his statements demonstrate that he thought
Adam was wrong about the risks of dumping the marbles.

In some jurisdictions, Bob can be convicted if he demonstrated “gross,” “criminal,” or “culpable”


negligence (i.e., if an ordinary person in his situation would have been aware that his conduct created an
unreasonable (or unreasonable and high) risk of death or serious bodily injury). In these states, Bob could
be found guilty of involuntary manslaughter on a theory of accomplice liability if the fact finder believes
that an ordinary person would have realized that placing 2,000 marbles on a road would interfere with the
ability of cars to stop when they drove over the marbles, thereby creating an unreasonable (or
unreasonable and high) risk of death or serious bodily injury.

In a few jurisdictions, Bob cannot have accomplice liability for involuntary manslaughter as a matter of
law. In these jurisdictions, the courts adhere strictly to the requirement that an “accomplice must want the
crime to be committed by the other party,” and they reason that it is “logically impossible” for a person to
be an accomplice to a crime of recklessness or negligence because one cannot “intend” a negligent or
reckless killing. In most jurisdictions, however, courts hold an accomplice responsible for a crime like
manslaughter if the accomplice intentionally provided assistance to the principal and acted with
recklessness or negligence, as the case may be, with respect to the risk that the principal’s behavior would
cause death.

Seperac-J19 Exam-Released MEE Essay Compilation © 2016-2020 350


[NOTE: With respect to the “dual intent” issue, the key is whether examinees recognize that accomplice
liability requires two levels of culpability. It is less important which approach examinees adopt than that
they see the problem – Bob cannot be guilty as an accomplice unless he was culpable with respect to the
underlying crime, and Bob, in fact, not only did not intend any car crash or resulting death, he actually
did not believe that such an event would occur.]

Seperac-J19 Exam-Released MEE Essay Compilation © 2016-2020 351


#080-JUL 2011–MEE Q02: QUESTION TWO (CRIMINAL LAW & PROCEDURE)

A police officer (Officer) on routine traffic patrol watched Suspect drive by. Suspect was in compliance
with all applicable traffic laws except the state seat belt law. The state motor vehicle code provides that
police officers have discretion to make an arrest for any traffic infraction, including violation of the state
seat belt law. Officer had never stopped a driver merely for violating the seat belt law. However, Officer
knew that Suspect was a reputed drug dealer and stopped Suspect’s vehicle, hoping to uncover evidence
of a more serious crime.

Officer directed Suspect to get out of his vehicle, handcuffed Suspect, and told Suspect that he was under
arrest for violating the seat belt law. Immediately afterward, Officer looked through the driver’s-side car
window and noticed a clear plastic bag containing white powder on the front seat of Suspect’s car. Officer
asked Suspect, “Are those drugs yours?” Suspect responded, “No, that cocaine isn’t mine!” Officer then
opened the car door and removed the bag of white powder.

Officer transported Suspect to the police station for booking. An hour later, Detective visited Suspect in
the police station holding cell to attempt an interview. Detective read Suspect his Miranda rights. Suspect
stated that he understood his Miranda rights but nonetheless would answer Detective’s questions. Suspect
voluntarily answered Detective’s questions for about five minutes and then said, “I’m not sure about this.
Maybe I need a lawyer.” Detective did not seek clarification of Suspect’s statement but continued to
question Suspect, who ultimately confessed to possessing the cocaine found in his car.

The state charged Suspect with misdemeanor violation of the seat belt law and felony drug possession.
Suspect has moved to suppress all the state’s evidence, alleging an unlawful stop, an unlawful arrest, an
unlawful seizure of evidence, and multiple Miranda violations.

1. Did the traffic stop and subsequent arrest violate Suspect’s constitutional rights? Explain.

2. Did Officer’s seizure of evidence from Suspect’s car violate Suspect’s constitutional rights?
Explain.

3. Did Officer’s questioning of Suspect violate Suspect’s Miranda rights? Explain.

4. Should Suspect’s confession to Detective be suppressed? Explain.

Seperac-J19 Exam-Released MEE Essay Compilation © 2016-2020 352


#080: J11-2 MEE: ANSWER: NCBE (CRIMINAL LAW & PROCEDURE)

POINT (1) [25%] ISSUE: Was it constitutional for Officer to stop and arrest Suspect for a petty
traffic offense when Officer’s real motive was to find evidence of other crimes? ANSWER: No. A
traffic stop and arrest with probable cause does not violate the Fourth Amendment even if the
offense was minor and the stop served as a pretext for investigating other criminal wrongdoing.

POINT (2) [25%] ISSUE: Was it constitutional for Officer to seize evidence from Suspect’s car?
ANSWER: Yes. Officer’s seizure of evidence from the front seat of Suspect’s car did not violate the
Fourth Amendment.

POINT (3) [15%] ISSUE: Did Officer’s questioning of Suspect violate Suspect’s rights under
Miranda? ANSWER: Yes. Officer violated Suspect’s Miranda rights when Officer arrested
Suspect and interrogated him without providing Miranda warnings and obtaining a valid waiver.

POINT (4)(a) [25%] ISSUE: Did Officer’s questioning of Suspect make Detective’s subsequent
interrogation of Suspect unconstitutional? ANSWER: No. Officer’s Miranda violation did not taint
Detective’s subsequent interrogation.

POINT (4)(b) [10%] ISSUE: Was Suspect’s ambiguous statement regarding a lawyer sufficient to
invoke Suspect’s right to an attorney and to require Detective to cease interrogation? ANSWER:
No. Suspect’s statement that he might want an attorney was too ambiguous to require Detective to
cease interrogation.

ANSWER DISCUSSION:

Officer had probable cause to believe that Suspect was violating traffic laws based on his own
observations of Suspect. Officer therefore acted constitutionally in stopping and arresting Suspect. The
stop and the subsequent arrest are constitutional despite the fact that the traffic infraction was minor,
Officer had never previously arrested anyone for a seat belt violation, and Officer had another motive for
making the stop. It was also lawful for Officer to engage in a “plain view” search of the front seat of
Suspect’s car by looking into the window after the arrest of Suspect. Officer violated Suspect’s
constitutional rights by interrogating Suspect at the scene after arresting him without first providing
Miranda warnings and obtaining a valid waiver. Thus, Suspect’s responsive statement to Officer at the
scene is inadmissible. However, Officer’s Miranda violation does not make Suspect’s subsequent
interrogation by Detective unconstitutional. A violation of Miranda does not taint derivative evidence
(e.g., Suspect’s statement made during the second interrogation). Moreover, Detective’s second
interrogation was a separate interrogation that involved a change in time, place, and circumstance. Prior to
Detective’s second interrogation, Miranda warnings were given and a knowing and valid waiver was
obtained. Suspect’s equivocal and ambiguous statement about possibly needing a lawyer was not an
invocation by Suspect of his Miranda right to silence, so Detective did not need to cease his interrogation
and provide Suspect with an attorney.

ANSWER EXPLANATION:

Explanation to Point-One (25%):

Seperac-J19 Exam-Released MEE Essay Compilation © 2016-2020 353


A traffic stop and arrest with probable cause does not violate the Fourth Amendment even if the offense
was minor and the stop served as a pretext for investigating other criminal wrongdoing.

Under the Fourth Amendment, the constitutional reasonableness of a traffic stop does not depend on the
motivation of the officer involved or the petty nature of the offense charged, as long as state law permits
an arrest. As a general matter, the decision to stop an automobile is reasonable where the police have
probable cause to believe that a traffic violation has occurred. If the officer has probable cause to believe
that a suspect has committed an offense, then a stop and arrest of that suspect is reasonable if authorized
under the applicable state statute. Under current case law, if an officer has probable cause to believe that
an individual has committed even a very minor criminal offense in his presence, he may, without violating
the Fourth Amendment, arrest the offender.

Here, the arrest was constitutional because Officer had probable cause, based on his personal
observations, to believe that Suspect was committing a traffic violation and the state seat belt statute
permitted arrest. The stop and subsequent arrest were constitutional despite the fact that the traffic
infraction was minor and Officer had never previously arrested anyone for a seat belt violation. In
addition, the stop and arrest were constitutional even if Officer used the seat belt violation as a pretext to
investigate criminal wrongdoing based on Officer’s knowledge that Suspect was a reputed drug dealer.

Explanation to Point-Two (25%):

Officer’s seizure of evidence from the front seat of Suspect’s car did not violate the Fourth Amendment.

Under the Fourth Amendment, a police officer is lawfully permitted to seize evidence without a warrant if
the evidence is found in plain view during lawful observation. It is well established that under certain
circumstances the police may seize evidence in plain view without a warrant. Under the plain view
doctrine, “if police are lawfully in a position from which they view an object, if its incriminating character
is immediately apparent, and if the officers have a lawful right of access to the object, they may seize it
without a warrant.”

In this case, Officer had probable cause to stop and arrest Suspect for a traffic law violation based on his
own observation of Suspect’s violation of the seat belt law. Following the arrest, Officer was lawfully
permitted to look into the car window while the car was stopped on a public road. Officer had probable
cause to believe that white powder contained in a clear plastic bag in the car of a reputed drug dealer is
likely to be an illegal drug, and Officer had additional cause to support that belief after Suspect stated,
“That cocaine isn’t mine,” in response to Officer’s question about the bag. The fact that Suspect’s
statement is inadmissible,, does not prevent Officer from relying on the statement to establish probable
cause. The lawful access requirement “guards against warrantless entry onto premises whenever
contraband is viewed from off the premises but does not bar the seizure of evidence in a parked car” that
is observed by an officer who is viewing contraband from a lawful vantage point. For example, in one
case, once officers saw “what appeared to be bags containing crack cocaine” in a parked car, they had a
right of access to the passenger compartment of the car to seize the likely contraband.

Explanation to Point-Three (15%):

Officer violated Suspect’s Miranda rights when Officer arrested Suspect and interrogated him without
providing Miranda warnings and obtaining a valid waiver.

A person under arrest is in custody and must first receive Miranda warnings before being subjected to
interrogation. Prior to any questioning, a person under arrest “must be warned that he has a right to remain

Seperac-J19 Exam-Released MEE Essay Compilation © 2016-2020 354


silent, that any statement he does make may be used as evidence against him, and that he has a right to the
presence of an attorney, either retained or appointed. Here, Officer arrested Suspect and then interrogated
Suspect, asking him “Are these drugs yours?” without first providing Miranda warnings. This custodial
interrogation involved express questioning of Suspect by Officer that violated Suspect’s Miranda rights.
The Miranda safeguards come into play whenever a person in custody is subjected to either express
questioning or its functional equivalent.

Explanation to Point-Four(a) (25%):

Officer’s Miranda violation did not taint Detective’s subsequent interrogation.

Statements taken during custodial interrogations conducted in violation of Miranda are excluded from the
prosecution’s case-in-chief. However, a violation of Miranda is a violation of a constitutional “rule,” not a
direct violation of the constitution. Unlike direct violations of the constitution, Miranda violations do not
taint derivative evidence and the “fruit of the poisonous tree” doctrine does not apply. Unlike actual
violations of the Self-Incrimination Clause, there is, with respect to mere failures to warn, nothing to
deter, and therefore no reason to apply the ‘fruit of the poisonous tree’ doctrine.

In Oregon v. Elstad, the U.S. Supreme Court held that, if a suspect was subjected to custodial
interrogation without the benefit of Miranda warnings, a subsequent confession by the suspect may
nonetheless be admitted if the totality of circumstances establishes that the second statement was knowing
and voluntary. Unless the police engage in an interrogation process that must “realistically be seen as part
of a single unwarned sequence of questioning,” subsequent custodial interrogation following a break in
the initial proceedings, Miranda warnings, and a waiver of rights is constitutional. Under Elstad, a suspect
who has once responded to unwarned yet uncoercive questioning is not thereby disabled from waiving his
rights and confessing after he has been given the requisite Miranda warnings.

Here, Officer’s initial unwarned custodial interrogation should not prevent the prosecution from using
Suspect’s later confession to Detective. After Officer’s original Miranda violation, Suspect was taken to
the police station where Detective provided proper Miranda warnings and obtained Suspect’s knowing
and voluntary waiver before interrogating Suspect. Detective’s provision of Miranda warnings at a later
time and in a different place rendered Suspect’s later statements to Detective admissible.

Explanation to Point-Four(b) (10%):

Suspect’s statement that he might want an attorney was too ambiguous to require Detective to cease
interrogation.

Pursuant to Davis v. United States, if an arrestee makes an unambiguous and unequivocal request or
demand for an attorney, the police must cease the interrogation and honor the request.

Suspect did not make such a request or demand; the statement “I’m not sure about this. Maybe I need a
lawyer” was ambiguous and equivocal. Likewise, “I think I need a lawyer” is not an unambiguous request
for an attorney. Therefore, Detective was not required to cease the interrogation and provide Suspect with
an attorney. Indeed, Detective was not even obliged to seek clarification of Suspect’s intentions. Courts
have declined to “adopt a rule requiring officers to ask clarifying questions” and concluded that when a
suspect’s statement “is not an unambiguous or unequivocal request for counsel, the officers have no
obligation to stop questioning him”. Detective therefore did not violate Suspect’s rights by ignoring
Suspect’s equivocal statement about an attorney, and the court should not suppress Suspect’s confession.

Seperac-J19 Exam-Released MEE Essay Compilation © 2016-2020 355


#081-JUL 2010–MEE Q08: QUESTION EIGHT (CRIMINAL LAW & PROCEDURE)

Customer went to Star Computer Systems (Star) to buy a refurbished computer. Upon arrival, Customer
was approached by Owner, who identified himself as the owner of Star. Owner directed Customer to a
refurbished desktop computer and told Customer, “We have the best refurbished computers in town. We
send used computers to a computer technician who always installs new hard drives and replaces any
defective parts.” Owner made these claims because Owner believed that they would be effective in
persuading Customer to buy a refurbished computer. In fact, Customer was persuaded by Owner’s claims
and purchased a computer for $250 cash.

At the time of this transaction, Owner did not believe that Star had the best refurbished computers in
town. Owner was aware of at least two other computer stores in town and believed that the refurbished
computers sold by these other stores were better than those sold by Star. Owner also thought it was very
likely that the computer technician used by Star did not actually install new hard drives in the refurbished
computers. Owner had never raised the issue with the technician because the technician offered much
faster service and lower rates than those of any other technician in the area.

After Customer’s purchase, a local news station conducted an investigation into the computer technician
used by Star and reported that the technician did not install new hard drives in any of the computers she
refurbished. After the report aired, the computer technician acknowledged that no new hard drives had
been installed in the computers she had refurbished for Star.

Owner has been charged with larceny by false pretenses in connection with the computer sale to
Customer.

Is Owner guilty of larceny by false pretenses? Explain.

Seperac-J19 Exam-Released MEE Essay Compilation © 2016-2020 356


#081: J10-8 MEE: ANSWER: NCBE (CRIMINAL LAW & PROCEDURE)

POINT (1) [25%] ISSUE: What are the elements of the crime of larceny by false pretenses?
ANSWER: To be guilty of larceny by false pretenses, a defendant must knowingly make a false
representation of material fact to a person, with the intent to defraud, and the false representation
must cause the victim to pass title to something of value to the defendant.

POINT (2) [35%] ISSUE: Did Owner make a false representation of a material present or past fact
that caused Customer to pass title to the property (the $250) to Owner? ANSWER: Yes. The
"actus reus" elements of the crime of false pretenses are met because Owner made a false
representation of a material present or past fact that caused Customer to pass title to something of
value (the $250) to Owner.

POINT (3) [40%] ISSUE: Did Owner act with the mens rea required to be found guilty of false
pretenses? ANSWER: Yes. The mens rea element of the crime of false pretenses is likely met
because Owner made the false representation knowingly and with the intent to defraud Customer.

ANSWER DISCUSSION:

To be guilty of larceny by false pretenses in most jurisdictions, Owner must have made a false
representation of a material fact that caused Customer to pass title to property to Owner. Owner must have
acted with knowledge of the falsity of the fact and with an intent to defraud. While Owner did not believe
that Star had the best refurbished computers in town, his claim to the contrary was a false representation
of opinion, not fact, and could not support a conviction for larceny by false pretenses. However, Owner’s
statement that the computer technician installed a new hard drive in each refurbished computer was a false
statement of a material fact because the technician did not install new hard drives. Owner obtained title to
Customer’s property when Customer paid Owner $250 for the computer. The fact that Customer
purchased the computer after being satisfied by Owner’s claims suggests that Customer relied upon
Owner’s false representation of fact in passing title to property. A court would likely find that Owner
knowingly made the false representation about new hard drives because Owner believed that it was very
likely that the computer technician used by Star did not install new hard drives, and Owner deliberately
decided not to question the technician. Owner also had the requisite intent to defraud because Owner
made the false claim about the new hard drives based upon the belief that it was one of the most important
factors that would lead to a purchase by Customer.

ANSWER EXPLANATION:

Explanation to Point-One (25%):

To be guilty of larceny by false pretenses, a defendant must knowingly make a false representation of
material fact to a person, with the intent to defraud, and the false representation must cause the victim to
pass title to something of value to the defendant.

The crime of “larceny by false pretenses” or, as it is sometimes called, “obtaining property by false
pretenses” or just “false pretenses,” is a type of theft offense. In most jurisdictions, the "actus reus"
elements of false pretenses are (i) a false representation of material fact (ii) that causes another person to
transfer title to property (including money) to the defendant. The mens rea required is knowledge that the
representation of fact is false and an intent to defraud.

Seperac-J19 Exam-Released MEE Essay Compilation © 2016-2020 357


Here, Owner is charged with the crime of false pretenses on the theory that Owner stole Customer’s
money (the $250 that Customer paid for the computer) by lying to Customer in order to induce Customer
to give the money to Owner in exchange for the computer.

Explanation to Point-Two (35%):

The "actus reus" elements of the crime of false pretenses are met because Owner made a false
representation of a material present or past fact that caused Customer to pass title to something of value
(the $250) to Owner.

As noted above, to be guilty of false pretenses in most jurisdictions, Owner must have made a false
representation of a material present or past fact that caused Customer to pass title to property to Owner.
The false representation must be one of fact, not opinion. Commercial puffery is not generally considered
“false pretenses.” While Owner did not believe that Star had the best refurbished computers in town, his
claim to the contrary was a false representation of opinion, not of fact, and could not support a conviction
for larceny by false pretenses.

On the other hand, Owner’s statement that the technician had installed a new hard drive in each
refurbished computer was a false statement of a material fact. It is a misrepresentation of fact where a
defendant accepts payment for repair work that was not performed. Owner claimed that the technician had
installed a new hard drive in every refurbished computer when no such installation had occurred.

Through this false representation, Owner succeeded in obtaining title to Customer’s property when the
purchase of the computer was completed and Customer gave Owner $250.

It does not matter that Customer may have had additional reasons for purchasing the computer – reasons
that had nothing to do with Owner’s false representation. It is enough that Owner’s false representation
contributed to Customer’s parting with property. The fact that Customer purchased the computer after
being satisfied by Owner’s claims, including the claim that the computer technician installed a new hard
drive in every refurbished computer, suggests that the false representation contributed to Customer’s
parting with $250, thus establishing the causal link between Owner’s false representation and the transfer
of title to the property.

Explanation to Point-Three (40%):

The mens rea element of the crime of false pretenses is likely met because Owner made the false
representation knowingly and with the intent to defraud Customer.

To be guilty of false pretenses, Owner must have made the false representation knowingly and with the
intent to defraud Customer. Most courts find that a defendant acts knowingly and has knowledge of a
particular fact when, inter alia, the defendant is aware of a high probability of the fact’s existence and
deliberately avoids learning the truth. A court would likely find that Owner knowingly made the false
representation based upon both Owner’s belief that it was very likely that the computer technician used by
Star did not install new hard drives and Owner’s deliberate decision not to question the technician. The
markedly low rates and fast service provided by the technician would provide additional evidence that
Owner knew or should have known that the technician was not installing new hard drives.

A few states, however, reject this “willful blindness” standard and require actual knowledge. In such
instances, actual knowledge of each element of offense required, but such knowledge may be proved by

Seperac-J19 Exam-Released MEE Essay Compilation © 2016-2020 358


circumstantial evidence. In these states, Owner would lack actual knowledge that the technician failed to
install new hard drives and could not be found guilty of false pretenses.

With respect to the second element, a defendant has the intent to defraud required to establish false
pretenses when the defendant intends that the person to whom the false representation is made will rely
upon it. Because Owner made the claim about the new hard drives based upon the belief that it would help
persuade Customer to buy a computer, Owner also had the requisite intent to defraud.

Seperac-J19 Exam-Released MEE Essay Compilation © 2016-2020 359


#082-JUL 2009–MEE Q07: QUESTION SEVEN (CRIMINAL LAW & PROCEDURE)

John, age 18, and Crystal, age 14, walked into the Minit Mart, a convenience store. They wandered
around the store for a few minutes and then walked up to the counter. John had his hand in a leather bag.
He stared at the store clerk for about 10 seconds and started to sweat. At this point, Crystal began to cry.
She said, “I don’t want to do this,” and ran out of the store. John remained for a few more seconds and
then ran out of the store himself. The store clerk immediately called 911 and nervously said, “Two kids
were about to rob me, but I guess they changed their minds.”

Three days later, the police came to Crystal’s home, where she lives with her parents. The police told
Crystal’s mother that Crystal was a suspect in an attempted robbery and that they wanted to search for
evidence. Crystal’s mother asked the police if they had a warrant. They said, “No, but we can get one.”
Crystal’s mother let the police in.

When the police searched Crystal’s room, they found John’s zipped leather bag in Crystal’s closet.
Without first obtaining a search warrant, or asking Crystal’s mother who owned the bag, the police
opened the bag and found a gun and printouts of e-mails from John to a friend. The police read the e-
mails, which described John’s plans to rob the Minit Mart with the help of his girlfriend, Crystal.

Crystal later confessed that she and John had planned to rob the Minit Mart, but that she got scared and
ran out of the store. She also said that John gave her the leather bag with the gun after they had left the
Minit Mart and told her to “get rid of it.”

John was charged with attempted armed robbery of the Minit Mart store. His lawyer filed a motion to
suppress the leather bag and its contents on the grounds that the bag was seized and searched in violation
of John’s Fourth Amendment rights. The trial court denied the motion, and the issue was properly
preserved for appeal.

At John’s trial for attempted armed robbery, the Minit Mart clerk testified to the facts that he had told the
police earlier. Crystal also testified for the prosecution, repeating what she had previously told the police.
She also said that the leather bag belonged to John, that she and John had planned the robbery together,
that she saw John load the gun and put it in his leather bag before they entered the Minit Mart, and that
John’s hand was in the leather bag with the gun for the entire time that they were in the Minit Mart. She
testified that she “got scared” when she and John were standing at the counter, which is why she ran from
the store. She said that when John got into the car after running out of the store, he said, “Well, that went
bad,” and then drove away from the store.

After Crystal’s testimony, the prosecution introduced the leather bag, the e-mails, and the gun into
evidence. The court admitted all of the evidence over the objection of John’s lawyer.

John presented no evidence.

At the close of the case, the trial court denied both John’s motion for judgment of acquittal and his request
for a jury instruction on the defense of abandonment.

The jury convicted John of attempted armed robbery. Did the trial court err:

1. In denying John’s motion to suppress the leather bag and its contents? Explain.

Seperac-J19 Exam-Released MEE Essay Compilation © 2016-2020 360


2. In denying John’s motion for judgment of acquittal? Explain.

3. In failing to instruct the jury on the defense of abandonment? Explain.

Seperac-J19 Exam-Released MEE Essay Compilation © 2016-2020 361


#082: J09-7 MEE: ANSWER: NCBE (CRIMINAL LAW & PROCEDURE)

POINT (1) [37%] ISSUE: Does John have standing to challenge the search of Crystal’s parents’
home and the seizure and search of the leather bag? ANSWER: No. John does not have standing to
challenge the search of Crystal’s parents’ home because he lacks a reasonable expectation of
privacy in that home. Similarly, he cannot challenge the seizure of his bag and the search of its
contents because he had surrendered control of the bag to Crystal and therefore had no reasonable
expectation of privacy in its contents.

POINT (2) [37%] ISSUE: Did the prosecution produce sufficient evidence to prove all of the
elements of attempted armed robbery? ANSWER: Yes. The prosecution produced sufficient
evidence to prove that John intended to commit an armed robbery and that he committed acts
going “beyond mere preparation” and constituting an attempt to commit the robbery.

POINT (3) [26%] ISSUE: Is “withdrawal,” “abandonment,” or “change of mind” a defense to a


charge of an attempted crime? ANSWER: Although John left the Minit Mart without completing
the robbery, the circumstances of his departure do not support a defense of withdrawal or
abandonment.

ANSWER DISCUSSION:

John does not have a reasonable expectation of privacy in Crystal’s parents’ home and therefore does not
have standing to challenge the legality of the search of that home. Nor can he claim a reasonable
expectation of privacy in the leather bag and its contents since he turned it over to Crystal. The trial court
correctly denied the motion for judgment of acquittal. The prosecution presented sufficient facts to prove
all the elements of attempted armed robbery. Specifically, the state showed that John had the intent to
commit an armed robbery and that he had performed sufficient acts toward the commission of the robbery
to have gone beyond mere preparation. The trial court was probably correct to deny the motion for the
jury instruction on abandonment. In many jurisdictions, abandonment is not a defense in any case in
which an individual has committed sufficient acts to constitute an attempt, as John has in this case. Even
in jurisdictions where abandonment is a defense, it must be a voluntary and complete renunciation by the
defendant of the criminal scheme. John’s decision not to complete the robbery was not due to his
voluntary abandonment of his criminal purpose. It was the result of his accomplice’s flight from the
scene.

ANSWER EXPLANATION:

Explanation to Point-One (30-40%):

John does not have standing to challenge the search of Crystal’s parents’ home because he lacks a
reasonable expectation of privacy in that home. Similarly, he cannot challenge the seizure of his bag and
the search of its contents because he had surrendered control of the bag to Crystal and therefore had no
reasonable expectation of privacy in its contents.

Although the police did not have a warrant, Crystal’s mother consented to the search of her home when
she let them in after they stated that they could get a warrant. Even if her consent to the search of her
home was considered coerced or involuntary and therefore invalid, John could not raise the issue. In
Rakas v. Illinois, the Supreme Court of the United States held that only those who are actual victims of

Seperac-J19 Exam-Released MEE Essay Compilation © 2016-2020 362


the alleged Fourth Amendment violation have “standing” to challenge it. As the Court put it, “a person
who is aggrieved by an illegal search and seizure only through the introduction of damaging evidence
secured by a search of a third person’s premises or property has not had any of his Fourth Amendment
rights infringed.” Here, John can claim no reasonable expectation of privacy in his girlfriend’s family’s
home.

Similarly, a person challenging a seizure and search of an object such as a bag must have a reasonable
expectation of privacy in the object seized and searched. John cannot claim that he had a reasonable
expectation of privacy in his bag found in Crystal’s closet. While he might have a property interest in the
bag, he surrendered his reasonable expectation of privacy in the bag when he turned the bag over to
Crystal and asked her to “get rid of it.” Once John gave the bag to Crystal, he had no way to control who
had access to his bag and how Crystal was to dispose of it.

Accordingly, the trial judge correctly denied John’s motion to suppress the bag and its contents.

Explanation to Point-Two (30-40%):

The prosecution produced sufficient evidence to prove that John intended to commit an armed robbery
and that he committed acts going “beyond mere preparation” and constituting an attempt to commit the
robbery.

A motion for judgment of acquittal should be granted only if the prosecution has failed to present
sufficient evidence for a reasonable jury to find that the defendant committed each element of the charged
offense beyond a reasonable doubt. In this case, there is more than enough evidence to support a
conviction for attempted armed robbery.

In order to prove that John attempted an armed robbery, the prosecution must prove two elements. First,
the prosecution must prove that John intended to commit an armed robbery. Second, the prosecution must
prove that John’s actions went sufficiently beyond “mere preparation” to constitute an actual “attempt” to
commit the crime.

As to John’s intent, the e-mails and the testimony of the clerk and Crystal provide ample evidence that
John planned to either use a weapon or threaten the imminent use of a weapon against the store clerk to
compel the store clerk to turn over money. This evidence shows clearly that John intended to commit the
essential elements of armed robbery – he intended to use force or the threat of force to steal the property
of another. Accordingly, the prosecution has more than met its burden of proving that John intended to
commit armed robbery.

The evidence is also sufficient to show that John’s actions to carry out his plan went beyond “mere
preparation” and constituted an actual “attempt” to commit the crime. States use various tests for
determining when a defendant’s conduct has gone far enough to constitute an attempt to commit a crime.
Some state courts say that the actor’s conduct must be “proximate” to the crime; others say that it must be
“dangerously proximate” to the crime. The Model Penal Code requires only that a defendant’s conduct be
a “substantial step” toward commission of the crime and corroborative of his criminal intent. Some states
say that the defendant’s behavior must “unequivocally” manifest the criminal intent.

In this case, the prosecution’s evidence demonstrates that John’s conduct went beyond mere preparation
and constituted an attempt. John’s behavior – loading a gun, taking it into a store, and approaching the
store clerk – ought to satisfy any test of proximity: John was dangerously close to completing his crime.
Moreover, the conduct on its face bespeaks an intention to rob the store clerk by force or threat of force,

Seperac-J19 Exam-Released MEE Essay Compilation © 2016-2020 363


and the clerk said that he thought John intended to rob him. Indeed, from all the facts, it appears that John
would have robbed the clerk but for the fact that John’s accomplice started to cry and left the store. Most
courts would find that these actions were “near enough to the accomplishment of the substantive offense
to be punishable.”

Explanation to Point-Three (20-30%):

Although John left the Minit Mart without completing the robbery, the circumstances of his departure do
not support a defense of withdrawal or abandonment.

In most jurisdictions, voluntary withdrawal or abandonment is not a defense to the crime of attempt once
the actor’s conduct has gone beyond mere preparation. In those jurisdictions, if John’s conduct had gone
far enough toward completion of a crime to constitute an attempt, he would be guilty of attempt even if he
subsequently decided to renounce his criminal plans completely and voluntarily. In a jurisdiction taking
this view of attempt, the court was correct to refuse a jury instruction on the defense of abandonment
because no such defense exists.

A minority of jurisdictions take the view that the abandonment of an attempt before the crime is
completed is an affirmative defense. However, the abandonment must be utterly voluntary. An
abandonment that is the result of any extrinsic factor is not considered voluntary. For example, passing
out from intoxication is not a voluntary abandonment of an attempted crime.

Here, John abandoned the robbery only after his 14-year-old accomplice got scared and ran out of the
store. Whether his decision to leave the store without completing the robbery was motivated by a concern
for Crystal, a belief that he needed her participation, or some other factor, it was taken in response to
Crystal’s behavior and was not an entirely voluntary decision by John. It does not satisfy the requirements
for the abandonment defense, and the trial judge was correct to deny an instruction on that defense.

Seperac-J19 Exam-Released MEE Essay Compilation © 2016-2020 364


#083-FEB 2008–MEE Q08: QUESTION EIGHT (CRIMINAL LAW & PROCEDURE)

On April 10, a convenience store was robbed by someone carrying a gun. The store’s video camera
caught the robbery on tape. The tape was shown on the evening news.

On April 11, an anonymous caller contacted the police saying, “I saw that tape of the robbery. The robber
kind of looks like Student. He’s an 18-year-old student at the high school.”

On April 12, two police officers took the tape to the high school and showed it to the principal, who said,
“It could be Student. It’s hard to tell because the tape is not clear.” The tape was also shown to Student’s
homeroom teacher, who said, “It might be him, but I couldn’t say for sure.”

Later that day, the police officers went to the store where Student works after school. They asked the
manager if they could talk with Student, who was called to the manager’s office. The police introduced
themselves to Student and said, “We’d like to talk to you.” They walked with Student into the manager’s
office and shut the door. One police officer sat behind the manager’s desk; the other, in full uniform with
his revolver visible, sat near the door. Student sat between them. The manager’s office measures eight feet
by ten feet.

The police officers told Student they wanted to ask him some questions about the convenience store
robbery on April 10. Student said he knew nothing about a robbery. He continued to deny that he had any
knowledge of the robbery for about 20 minutes. Student did not ask to leave, and neither police officer
told Student he was free to leave.

After about 20 minutes, the police officers told Student that they had a videotape of the robbery and that
they had shown it to three people, all of whom positively identified Student as the robber.

Student said nothing for a few minutes. One of the police officers then said, “You know, if we can tell the
prosecutor that you cooperated, she might go a lot easier on you. I’d hate to see you end up doing a long
stretch in prison. Let’s just say it’s not a nice place.” Student then blurted out, “I did the robbery. I used a
little air gun.”

Immediately after Student made that statement, the police officers informed Student that he was under
arrest for the robbery of the convenience store. They read him his Miranda rights. Student stated he
understood his Miranda rights and told the police officers that he was not going to say anything more to
them. The police officers placed Student in handcuffs and took him to the police station where he was
booked for armed robbery.

Student had had two earlier brushes with the law. When he was 16, he had been found delinquent in
juvenile court for auto theft and had been placed on supervision for one year. When he was 17, he had
received a ticket for underage drinking and had paid a fine of $150. He is a “C” student, but his teachers
believe he is an “underachiever.”

Student’s defense attorney has filed a motion to suppress Student’s statements on three grounds:

(1) Student’s statements were obtained in violation of Student’s Fourth Amendment rights.

(2) Student’s statements were obtained in violation of his Miranda rights.

Seperac-J19 Exam-Released MEE Essay Compilation © 2016-2020 365


(3) Student’s confession was not voluntary.

How should the trial court rule on each of the grounds in the motion to suppress? Explain.

Seperac-J19 Exam-Released MEE Essay Compilation © 2016-2020 366


#083: F08-8 MEE: ANSWER: NCBE (CRIMINAL LAW & PROCEDURE)

POINT (1) [32%] ISSUE: Did the police interview of Student violate Student’s Fourth Amendment
right to be free from unreasonable seizure such that Student’s statements (including the confession)
should be suppressed as “fruit of the poisonous tree”? ANSWER: No. Student was seized by the
police within the meaning of the Fourth Amendment, but the seizure was a lawful investigative
detention.

POINT (2) [37%] ISSUE: Should Student’s statements be suppressed because the police failed to
read him his Miranda rights when they questioned him in the manager’s office? ANSWER: No.
The police were probably not required to read Student the Miranda warnings because he was
probably not in custody until they placed him under arrest.

POINT (3) [32%] ISSUE: Should Student’s confession be suppressed because it was involuntary?
ANSWER: No. Based on the totality of the circumstances, Student’s confession was probably
voluntary.

ANSWER DISCUSSION:

Student was “seized” within the meaning of the Fourth Amendment when he was brought into the
manager’s office to be questioned by the police. However, that seizure did not violate Student’s Fourth
Amendment rights because the police had “reasonable articulable suspicion” that Student had been the
perpetrator of an armed robbery. This allowed them to detain him for the purposes of investigation and
will not taint any statements that Student made during the detention. Whether the police were required to
read Student his Miranda warnings depends on whether he was subject to in-custody interrogation. In this
case, the only real question is whether Student was in custody. Applying the objective standard, Student
was probably not in custody when he spoke with the officers in the manager’s office, and Miranda
warnings were not required. The last question is whether Student’s confession to the officers was
voluntary or whether it was the product of police coercion that overcame Student’s will. Here a court must
look at the totality of the circumstances in ruling on the voluntariness of the confession. Most courts
would probably conclude that Student’s confession was voluntary.

ANSWER EXPLANATION:

Explanation to Point-One (25-35%):

Student was seized by the police within the meaning of the Fourth Amendment, but the seizure was a
lawful investigative detention.

Student has a Fourth Amendment right to be free from unreasonable seizure. However, this Fourth
Amendment right was violated in this case only if: (a) Student was in fact “seized” by the police when
they spoke with him in the manager’s office, and (b) that seizure was unreasonable for Fourth
Amendment purposes. If the seizure did violate Student’s Fourth Amendment rights, then Student’s
statements would be suppressible because they were the direct result of that seizure (i.e., “the fruit of the
poisonous tree”).

A person has been seized if, in light of all of the circumstances, “a reasonable person would have believed
that he was not free to leave.” In this case, a court could legitimately find that Student was seized by law

Seperac-J19 Exam-Released MEE Essay Compilation © 2016-2020 367


enforcement. Student was taken into his manager’s office where he remained for approximately 25
minutes with two police officers, one of whom had a visible weapon. One officer sat between Student and
the door, and the door was closed. Student was not told that he was free to leave. Under these
circumstances, a reasonable person in Student’s position would probably have believed that he was
required to stay in the room until the police officers gave him permission to leave.

Assuming that there was a seizure, the next question is whether it was a violation of Student’s Fourth
Amendment rights. Although the police did not have probable cause to arrest Student for the armed
robbery, the Fourth Amendment permits detention of an individual for a relatively brief period of time if
the police have a “reasonable articulable suspicion” that the individual in question has been recently
involved in criminal activity.

In this case, three people said that the person on the tape of the convenience store robbery “could be,”
“might be,” and “looked like” Student. These identifications, although far from certain, gave the law
enforcement officers sufficient specific facts to form the basis of a reasonable suspicion that Student was
the robber. The fact that one of the identifiers was an anonymous caller does not affect the reasonableness
of law enforcement’s actions since the police officers corroborated the possible identification with
Student’s principal and Student’s homeroom teacher.

Thus, Student’s Fourth Amendment rights were not violated, and his statements should not be suppressed.

Explanation to Point-Two (30-40%):

The police were probably not required to read Student the Miranda warnings because he was probably not
in custody until they placed him under arrest.

Law enforcement officers are required to read Miranda warnings to a suspect when the suspect is
subjected to an in-custody interrogation. If the officers were required to read the warnings and failed to do
so, the statements should be suppressed.

Student was clearly subject to interrogation, which has been defined not only as questioning initiated by
law enforcement but as “any words or actions on the part of the police that the police should know are
reasonably likely to elicit an incriminating response” from the suspect. The only question then is whether
Student was “in custody” when he was being questioned.

Custody is a substantial seizure and is defined as either a formal arrest or “restraint on freedom of
movement of the degree associated with a formal arrest.” A suspect need not be in a police station to be in
custody. Whether a suspect is in custody is determined by “how a reasonable person in the suspect’s
situation would perceive his circumstances.” In assessing the question of custody, courts consider the
facts surrounding the interrogation. The test to be applied is purely objective. Therefore, a suspect’s age,
experience, and other personal characteristics are not considered.

In this case, two police officers had Student summoned to the manager’s office, into which they escorted
him and closed the door. Student was outnumbered two to one, and one officer, with a visible firearm, sat
between Student and the door. The police did not tell Student that he was free to leave. Nonetheless,
Student was probably not in custody. The police officers told Student that they would “like to talk” with
him; he was not told that he was under arrest until the end of the interview. During the interview, Student
was not in any restraints, the two officers were seated in chairs, and the interview took place in an office.
While Student may not have felt that he was free to leave, a reasonable person would probably not have
believed that he had been arrested or otherwise taken into formal custody. If Student was not in custody,

Seperac-J19 Exam-Released MEE Essay Compilation © 2016-2020 368


then the police were not required to read him Miranda warnings before talking with him. Therefore, there
should be no suppression for a Miranda violation.

Explanation to Point-Three (25-35%):

Based on the totality of the circumstances, Student’s confession was probably voluntary.

The voluntariness of a confession is based on (1) whether the police subjected the suspect to coercive
conduct, and (2) whether the conduct was sufficient to overcome the will of the suspect,

There is certainly some evidence of coercion. Here, the police interviewed Student in a small, closed
room, two against one, with a firearm displayed. The police did not tell Student that he was free to go if
he chose, did not tell him that he was not obliged to answer their questions, and did not advise him of his
Miranda rights. The police also lied to Student about the strength of their case. By itself, the lie would not
be a ground for suppression. Trickery and deceit do not render a confession inadmissible. “Far from
making the police a fiduciary of the suspect, the law permits the police to pressure and cajole, conceal
material facts and actively mislead.” But in conjunction with the other factors, this deception pushes this
case closer to the line of coercion.

The more difficult question is whether this coercive conduct was sufficient to overcome the will of the
suspect. Here, courts consider the totality of the circumstances and take into account both the conduct of
the police and the suspect’s individual characteristics such as age, level of education, and familiarity with
the criminal justice system.

A court could conclude that the police behavior was insufficiently coercive to have overcome Student’s
will. The facts indicate that Student is an adult (18 years old) of at least average intelligence and that he
has some experience with the juvenile justice system. The interview in his manager’s office lasted less
than 30 minutes. Although the police statements about the identification were not 100 percent true, the
fact that a suspect has been deceived about the strength of evidence does not automatically render a
statement involuntary. The statement by the police officers that the prosecutor might be more lenient
when a defendant is young and cooperates could be construed as an accurate observation of how the
justice system operates rather than as a promise of leniency. Similarly, the police officers’ general
description of prison as “not being a nice place” was not a threat but simply a statement of fact.

On the other hand, a court that focuses on Student’s youth and inexperience might consider the confession
to have been involuntary. Student is a high school senior with minimal experience with the criminal
justice system. He was questioned in his manager’s office by two police officers. His confession was in
direct response to police conduct that could be considered “overreaching.” First, the police deliberately
lied about the strength of the evidence against him. When it appeared that Student was reacting to that
tactic, the police upped the ante by telling him that a confession might result in leniency. Finally, the
police officers made what could be taken as a veiled threat about what would happen to Student in prison
if he did not confess.

Seperac-J19 Exam-Released MEE Essay Compilation © 2016-2020 369


#084-JUL 2007–MEE Q05: QUESTION FIVE (CRIMINAL LAW & PROCEDURE)

Defendant and Friend were on a hunting trip together. One evening, while listening to the radio in their
hunting cabin, Defendant decided to scare Friend, who had fallen asleep in his chair. Defendant loaded his
rifle and aimed it at a lamp that was on a table just behind the chair in which Friend was sleeping. Just as
Defendant pulled the trigger to shoot the lamp, Friend suddenly sat up and moved into the line of fire.
Defendant’s bullet hit Friend in the shoulder and seriously wounded him.

Defendant loaded Friend into his car and sped off toward the nearest hospital, which was 15 minutes
away. En route, Defendant hit a pothole, lost control of his car, and collided with a telephone pole.
Defendant was seriously injured, and Friend suffered further injuries. The accident occurred on a lightly
traveled country road, and no other vehicle passed by for 45 minutes.

By the time help arrived, Friend was dead. An autopsy established that Friend bled to death as a result of
the combined impact of the gunshot wound and the injuries suffered in the car crash. The coroner
concluded that the gunshot wound alone would not have been fatal had Friend received medical treatment
within a half hour of the shooting.

Murder is defined as “a killing with malice aforethought.” In this jurisdiction, second-degree murder is
“all murder that is not deliberate or premeditated.” Defendant has been charged with second-degree
murder on account of his shooting Friend and Friend’s subsequent death.

Is Defendant guilty of second-degree murder? Explain.

Seperac-J19 Exam-Released MEE Essay Compilation © 2016-2020 370


#084: J07-5 MEE: ANSWER: NCBE (CRIMINAL LAW & PROCEDURE)

POINT (1) [50%] ISSUE: Did Defendant act with the mens rea sufficient to be found guilty of
second-degree murder? ANSWER: Yes. A jury could find that Defendant’s act of shooting at a
lamp to scare Friend evidenced an extreme indifference to the value of human life and thus
conclude that Friend acted with “malice aforethought.”

POINT (2) [50%] ISSUE: Was Defendant’s accidental shooting of Friend the legal cause of Friend’s
death? ANSWER: Yes. Defendant’s act of shooting Friend was the legal cause of death because
Friend would not have died “but for” having been shot by Defendant, and the injuries suffered by
Friend in the car accident do not break the chain of proximate causation.

ANSWER DISCUSSION:

Defendant need not have acted with deliberation or premeditation to be guilty of second-degree murder.
However, Defendant must have acted with the mens rea, or mental state, of “malice aforethought.” Malice
aforethought is present where a defendant acts with “depraved indifference to the value of human life.”
On these facts, a jury could conclude that Defendant’s act of shooting at a lamp to “scare Friend”
evidenced a disregard of the risks to Friend and of the value of Friend’s life sufficient to amount to
depraved indifference. Defendant’s act of shooting Friend was the legal cause of Friend’s death. First, it
was the “but for” cause of Friend’s death. Had Friend not been shot, Defendant would not have been
driving him to the hospital, there would have been no accident, and Friend would not have bled to death.
The fact that the accident contributed to Friend’s death does not relieve Defendant of liability. The
accident occurred as a result of Defendant’s response to the shooting and would not be held to break the
chain of causation. Thus, if Defendant were found to have acted with the requisite mens rea, Defendant
would be responsible for Friend’s death and guilty of second-degree murder.

ANSWER EXPLANATION:

Explanation to Point-One (45-55%):

A jury could find that Defendant’s act of shooting at a lamp to scare Friend evidenced an extreme
indifference to the value of human life and thus conclude that Friend acted with “malice aforethought.”

To be guilty of second-degree murder, Defendant must have acted with the requisite mens rea, or mental
state, of malice aforethought. Malice aforethought is a term of art, and it encompasses several different
mental states. In most jurisdictions, the malice aforethought requirement is satisfied if Defendant acted
with intent to kill, with knowledge that his acts would kill, with intent to inflict great bodily harm, or with
reckless disregard of an extreme risk to human life (a depraved heart).

In this case, the evidence does not suggest that Defendant acted with intent or knowledge that Friend
would be harmed. However, the evidence probably does support a conclusion that Defendant acted with
“a depraved heart” and therefore with the malice aforethought necessary for Defendant to be guilty of
murder.

Although the precise terminology used to describe “depraved-heart” murder differs from jurisdiction to
jurisdiction, most states require (i) that the defendant act “recklessly” (i.e., with an awareness of the risk
that his acts could cause death) and (ii) that the defendant’s conduct show a high degree of indifference to

Seperac-J19 Exam-Released MEE Essay Compilation © 2016-2020 371


the value of human life. Thus, convictions for depraved-heart murder have been upheld when a defendant
intentionally shot a gun into a crowded room, or played Russian roulette with a partially loaded revolver

Here, Defendant probably acted recklessly. Defendant shot a gun in the general direction of Friend, while
aiming at a lamp that was behind Friend. This behavior carries an obvious risk to Friend: if Defendant’s
aim was slightly off or if Friend moved (as happened here), Friend could be shot and fatally injured. From
the facts of Defendant’s behavior, and the obvious risks attending that behavior, a jury could infer that
Defendant must have been aware of those risks and thus acted recklessly.

Did Defendant’s recklessness evidence an “extreme indifference to the value of human life?” Courts have
held that behavior that carries a substantial risk of causing death, when it has no legitimate purpose and is
done solely for the amusement of a defendant, is a sufficient predicate for a conclusion that the defendant
acted with the necessary indifference to human life to be liable for depraved-heart murder. Here
Defendant ran a high risk of causing serious harm to Friend for no purpose other than to amuse himself by
scaring Friend. This is the kind of behavior that provides a sufficient predicate for a jury to find “extreme
indifference to the value of human life” and to convict Defendant of second-degree murder.

Explanation to Point-Two (45-55%):

Defendant’s act of shooting Friend was the legal cause of death because Friend would not have died “but
for” having been shot by Defendant, and the injuries suffered by Friend in the car accident do not break
the chain of proximate causation.

Defendant is criminally liable for Friend’s death only if Defendant’s acts were both the actual and
proximate cause of Friend’s death.

Defendant’s act of shooting Friend was the actual cause of Friend’s death. First, Defendant’s act was the
“but for” cause of Friend’s death. Had Defendant not shot Friend, they would not have been driving to the
hospital, the accident would not have occurred, and Friend would not have bled to death. Quite apart from
that, however, it is well established that where a defendant’s wrongful act (the gunshot wound) works in
combination with some other cause (the injuries in the accident) to cause a victim’s death, the defendant’s
act is an actual cause of the death because it “accelerated the death process.” For example, a defendant
who inflicts a second non-mortal wound can be considered the cause of the victim’s death only if his
actions caused the victim to die sooner than he otherwise would. Here, the facts state that Friend’s death
was the result of loss of blood caused by both the gunshot wound and the automobile accident. In that
case, the gunshot wound is an actual cause of death.

Defendant’s acts were also the proximate cause of Friend’s death. The accident was a “dependent
intervening cause” – a consequence of Defendant’s prior wrongful conduct. It is widely held that such an
intervening cause breaks the chain of causation from the original act to the death only if the intervening
force was “so out-of-the-ordinary that it is no longer fair to hold the defendant criminally responsible for
the outcome.” Courts routinely say that the intervening event must have been “abnormal or bizarre” to
prevent a defendant’s original action from being regarded as the proximate cause of death. Here, there is
nothing abnormal about Defendant’s response to his wrongful act (he tried to rush Friend to the hospital).
Nor is there anything abnormal or bizarre about the fact that Defendant was involved in an accident while
rushing to the hospital with a wounded friend. Even if Defendant was not at fault in causing the accident,
the fact that it occurred is not sufficiently “out-of-the-ordinary” to warrant breaking the chain of causation
from the shooting to the death. Therefore, Defendant’s act of shooting Friend was the legal cause of
Friend’s death, and Defendant is guilty of second-degree murder if that act was committed with malice
aforethought.

Seperac-J19 Exam-Released MEE Essay Compilation © 2016-2020 372


EVIDENCE: 12 OF 24 MEE EXAMS: (50%)
#085-JUL 2018–MEE Q05: QUESTION FIVE (EVIDENCE)

A woman has sued a man for injuries she received in an automobile collision at a suburban traffic circle in
State A on January 1. Both drivers were driving alone, there were no other witnesses, and a forensic
accident investigation failed to determine which of the two drivers was at fault.

Among other things, the woman’s complaint alleges the following:

1. The woman was driving her pickup truck in the traffic circle at or below the speed limit when the
man suddenly pulled his car into the traffic circle immediately in front of her.

2. The man’s action left the woman no opportunity to slow down, stop, or avoid colliding with his
car.

3. The woman observed that the man was texting on his phone when he entered the traffic circle
and did not see him look up to check for traffic before entering the circle.

4. The accident caused the onset of significant neck pain for the woman requiring extensive
medical treatment and resulting in lost wages.

The man has denied that he was texting at the time of the accident and alleges that the accident was the
woman’s fault. According to the man, the woman was driving her truck substantially over the speed limit,
her brakes were defective, and despite the fact that the man’s car was far ahead of the woman’s truck
when he entered the traffic circle, the woman failed to slow down to avoid a collision.

A jury trial has been scheduled.

The man’s attorney plans to offer the following evidence:

(a) Testimony by a mechanic to the effect that “I inspected [the woman’s] truck a week before the
accident. The brakes on the truck were worn and in need of repair. I ordered new parts.”

(b) A written invoice signed by the mechanic stating: “New parts for [the woman’s] truck brakes
ordered on December 23 and received on January 2,” found in the mechanic’s file cabinet among
similar invoices for other customers.

(c) Testimony by the woman’s doctor, who treated the woman for neck pain after the accident, that
the woman told the doctor, “I have suffered from painful arthritis in my neck for the past five
years.”

The woman’s attorney plans to call the man’s roommate to testify that “[the man] is addicted to texting
and never puts his phone down. He even texts while driving.”

State A has adopted evidence rules identical to the Federal Rules of Evidence.

1. Is the mechanic’s testimony admissible? Explain.

Seperac-J19 Exam-Released MEE Essay Compilation © 2016-2020 373


2. Is the invoice for the new parts for the woman’s truck brakes admissible? Explain.

3. Is the doctor’s testimony admissible? Explain.

4. Is the roommate’s testimony admissible? Explain.

Seperac-J19 Exam-Released MEE Essay Compilation © 2016-2020 374


#085: J18-5 MEE: ANSWER: NCBE (EVIDENCE)

POINT (1) [20%] ISSUE: Is a mechanic’s testimony describing his opinion of the condition of truck
brakes that he has inspected admissible evidence? ANSWER: Yes. The mechanic’s testimony about
the condition of the brakes is relevant and should be admitted either as lay opinion rationally based
on the mechanic’s personal perception or as expert opinion because the mechanic possesses the
necessary technical knowledge to be qualified as an expert witness.

POINT (2) [20%] ISSUE: Is an invoice for truck brake parts an admissible business record when it
was apparently made by a person with knowledge of its contents and retained in the ordinary
course of business? ANSWER: Yes. The receipt from the mechanic’s shop is a relevant out-of-
court statement that should be admitted because it fits the hearsay exception for business records.

POINT (3)(a) [15%] ISSUE: May a patient assert the physician-patient privilege to bar admission
of a physician’s trial testimony if the patient has placed her medical condition in issue by filing a
personal injury lawsuit? ANSWER: No. The doctor’s proposed testimony is relevant to the cause of
the woman’s neck pain and would not be protected by the physician-patient privilege because the
woman effectively waived the privilege by filing this civil lawsuit, which placed her medical
condition “in issue.”

POINT (3)(b) [20%] ISSUE: Is a patient’s out-of-court statement regarding pre-existing pain
admissible as a statement by an opposing party when the patient is the plaintiff in a personal injury
lawsuit or admissible under the hearsay exception for statements made for the purpose of medical
diagnosis or treatment? ANSWER: Yes. The doctor’s proposed testimony would repeat an out-of-
court statement by the woman, but the woman’s statement is admissible because (1) it is a statement
by an opposing party and is therefore not hearsay, or (2) it fits the hearsay exception for statements
made for medical diagnosis or treatment.

POINT (4) [25%] ISSUE: Is the roommate’s testimony admissible habit evidence? ANSWER: Yes.
The court may admit testimony by the roommate if it decides that the man’s constant texting is a
habit. A court might rule either way.

ANSWER DISCUSSION:

The mechanic’s proposed testimony that he inspected the woman’s truck brakes and that they needed
repair is relevant because it has some tendency to make it more probable that a malfunction of the truck’s
brakes caused the accident. Although the proposed testimony is an opinion, it should be admitted because
it would be rationally based on the mechanic’s personal perception of the condition of the brakes, and the
opinion would be useful to the trier of fact. In the alternative, the mechanic’s opinion could be admitted as
expert testimony based on his technical skill or expert knowledge, if the opinion is the product of reliable
principles and methods and the mechanic reliably applied the principles and methods to the facts of the
case. The invoice from the mechanic’s shop stating, “New parts for [the woman’s] truck brakes ordered
on December 23 and received on January 2,” is relevant because it has some tendency to make it more
probable that the accident was caused by brake malfunction in the woman’s truck. The mechanic’s out-of-
court statement is hearsay, but it fits the hearsay exception for “business records” because the mechanic,
who signed the invoice, was a person with knowledge of the information in the invoice and the facts
suggest strongly that the invoice was made and kept in the ordinary course of the mechanic’s business.
The doctor’s proposed trial testimony that the woman told him “I have suffered from painful arthritis in

Seperac-J19 Exam-Released MEE Essay Compilation © 2016-2020 375


my neck for the past five years” is also admissible. The statement is relevant because it has some
tendency to make it less probable that the woman’s neck pain was caused by the accident, but instead by
arthritis that predated the accident. By filing this civil lawsuit alleging that the accident caused the onset
of neck pain, which placed her medical condition “in issue,” the woman thereby waived the physician-
patient privilege. The doctor’s testimony repeats an out-of-court statement by the woman. However,
because the declarant (the woman) is the plaintiff, it is not hearsay because it is an opposing party’s
statement when offered by the defendant (the man). In the alternative, it fits the hearsay exception for
statements made for the purpose of medical diagnosis or treatment. Finally, the roommate’s testimony
(“[The man] is addicted to texting and never puts his phone down. He even texts while driving.”) may be
admissible as evidence of the man’s “habit” of constant texting. This is a close call. The testimony is
relevant because it has some tendency to make it more probable that the man was texting at the time of the
accident. However, the court must first decide whether constant texting is a habit. Texting is a volitional
act and requires conscious effort. Most courts limit the scope of habit evidence to relevant behaviors that
are consistent and semi-automatic. Texting may be too volitional to be considered semi-automatic by
some courts, but courts that consider the similarity and repetition of behaviors might view evidence of
constant texting as habit evidence.

ANSWER EXPLANATION:

Explanation to Point One (20%):

The mechanic’s testimony about the condition of the brakes is relevant and should be admitted either as
lay opinion rationally based on the mechanic’s personal perception or as expert opinion because the
mechanic possesses the necessary technical knowledge to be qualified as an expert witness.

Evidence is relevant if it has “any tendency to make a fact more probable or less probable than it would be
without the evidence.” The mechanic’s testimony (“I inspected [the woman’s] truck a week before the
accident. The brakes on the truck were worn and in need of repair. I ordered new parts.”) is relevant
because it tends to make it more probable that a brake malfunction may have caused the accident.

The mechanic’s testimony is admissible opinion evidence. Under Federal Rule of Evidence 701, lay
opinion testimony is admissible if it is “(a) rationally based on the witness’s perception; (b) helpful to
clearly understanding the witness’s testimony or to determining a fact in issue; and (c) not based on
scientific, technical, or other specialized knowledge within the scope of Rule 702.” Courts typically hold
that witnesses with “firsthand knowledge” may “offer lay opinion testimony where they have a reasonable
basis—grounded either in experience or specialized knowledge—for arriving at the opinion expressed.”
Here, the mechanic’s opinion is rationally based on his personal perception and would be helpful to the
jury’s determination of whether the woman or the man caused the accident. A court could also find that
because the mechanic’s opinion is based on his personal perception, it is not an expert opinion within the
scope of Rule 702.

In the alternative, the mechanic may be qualified as an expert based on his technical skill or specialized
knowledge. Under Federal Rule of Evidence 702, the mechanic’s expert opinion can be admitted if “(a)
the expert’s scientific, technical, or other specialized knowledge will help the trier of fact to understand
the evidence or to determine a fact in issue; (b) the testimony is based upon sufficient facts or data; (c) the
testimony is the product of reliable principles and methods; and (d) the expert has reliably applied the
principles and methods to the facts of the case.” Here, a court could find that the mechanic used his
technical skills to reliably form the opinion that the woman’s truck brakes required repair, making the
opinion testimony admissible under Rule 702, but not Rule 701.

Seperac-J19 Exam-Released MEE Essay Compilation © 2016-2020 376


Regardless of whether the mechanic’s testimony is viewed by the court as lay or expert opinion, it should
be admitted.

Explanation to Point Two (20%):

The receipt from the mechanic’s shop is a relevant out-of-court statement that should be admitted because
it fits the hearsay exception for business records.

The receipt from the mechanic’s shop stating “New parts for [the woman’s] truck brakes ordered on
December 23 and received on January 2” is relevant because it has some tendency to make it more
probable that the woman’s truck brakes were, in fact, in need of repair and may have malfunctioned,
causing the accident.

The receipt is hearsay under Federal Rule of Evidence 801 because (1) it is a statement (the mechanic’s
“written assertion” that new brake parts were ordered); (2) the mechanic made the statement out of court;
and (3) the statement will be “offered in evidence to prove the truth of the matter asserted in the
statement.” However, the receipt fits the hearsay exception for “business records.” Under Rule 803(6), a
business record may be “a record of an act, event or condition” made “by someone with knowledge, if
kept in the course of a regularly conducted activity of a business, and if making the record was a regular
practice of that activity.” These facts must be shown by the testimony of the custodian of the record or
another qualified witness.

Here, the mechanic was a person with knowledge because he ordered the truck brake parts, as he will
testify. The fact that the receipt was written and signed by the mechanic and kept in the mechanic’s file
cabinet among similar receipts for other customers should establish that the mechanic was the custodian
of the receipt and that it was made and kept by the mechanic in the ordinary course of his repair business.
But the receipt will be admissible only if these facts are established during examination of the mechanic.

Explanation to Point Three(a) (15%):

The doctor’s proposed testimony is relevant to the cause of the woman’s neck pain and would not be
protected by the physician-patient privilege because the woman effectively waived the privilege by filing
this civil lawsuit, which placed her medical condition “in issue.”

The doctor’s proposed testimony that the woman said, “I have suffered from painful arthritis in my neck
for the past five years,” is relevant because it has some tendency to make it less probable that the accident
caused the onset of the woman’s neck pain.

Under Rule 501 of the Federal Rules of Evidence, “state law governs privilege regarding a claim or
defense for which state law supplies the rule of decision.” The physician- patient privilege, which was not
recognized at common law, has been adopted by statute in most jurisdictions. In determining whether to
honor the assertion of a privilege, “courts must balance the public interest in nondisclosure against the
need of the particular litigant for access to the privileged information, keeping in mind that the burden of
persuasion rests on the party seeking to prevent disclosure.”

In most jurisdictions, patient communications or disclosures made for the purpose of medical diagnosis or
treatment are privileged. However, most jurisdictions currently recognize a range of exceptions intended
to prevent the privilege from being used as a shield for fraud. These exceptions apply in any proceeding
where the patient relies on a physical condition as an element of his claim or defense. Thus, in many

Seperac-J19 Exam-Released MEE Essay Compilation © 2016-2020 377


states a patient waives the physician-patient privilege by placing her medical condition “in issue” in a
personal injury lawsuit.

The woman should not be able to invoke the physician-patient privilege to bar admission of the doctor’s
statement. She will be deemed to have waived the privilege by seeking damages for her neck pain, which
put the question of the cause of that pain into issue in the lawsuit.

Explanation to Point Three(b) (20%):

The doctor’s proposed testimony would repeat an out-of-court statement by the woman, but the woman’s
statement is admissible because (1) it is a statement by an opposing party and is therefore not hearsay, or
(2) it fits the hearsay exception for statements made for medical diagnosis or treatment.

The doctor’s statement, which repeats an out-of-court statement by the woman (that she had suffered from
painful arthritis in her neck for the past five years), is admissible for the truth of the matter asserted and is
not hearsay. When offered by the defendant (the man), it is an opposing party’s statement because the
declarant is the plaintiff (the woman).

In the alternative, the woman’s statement, which describes the onset and nature of her neck pain,
describes her “medical history” and provides information about her “past or present symptoms or
sensations; their inception; or their general cause.” Under these circumstances, the woman’s statement is
also admissible under the hearsay exception for statements made for medical diagnosis or treatment.

Explanation to Point Four (25%):

The court may admit testimony by the roommate if it decides that the man’s constant texting is a habit. A
court might rule either way.

The roommate’s testimony (“[The man] is addicted to texting and never puts his phone down. He even
texts while driving.”) is relevant because it has some tendency to make it more probable that the man was
texting at the time of the accident.

Under Federal Rule of Evidence 406, evidence of a person’s habit may be admitted to prove that on a
particular occasion the person acted in accordance with the habit. Typically, under Rule 406, a person’s
“habit” is defined as his or her consistent response to a specific situation. The court may admit habit
evidence “regardless of whether it is corroborated or whether there was an eyewitness.” Testimony
involving habit evidence may be given by the person with the habit or by another person, but the person
testifying must have personal knowledge.

It is sometimes difficult for courts to distinguish habit evidence from character evidence. The problem is
that, depending on the habit described, habit evidence can be similar to evidence of a person’s character
or prior acts. The Federal Rules of Evidence generally prohibit the use of evidence of a person’s character
“to prove that on a particular occasion the person acted in accordance with the character or trait.” The
Rules likewise prohibit evidence of other acts when offered to prove a character trait and action in
conformity with that trait. Because “habit” evidence can run afoul of the bans on character evidence and
prior bad acts evidence, courts generally limit habit evidence to proof of relevant behaviors that are not
just consistent but semi-automatic.

Here, it is a close question whether the roommate’s testimony is admissible habit evidence or prohibited
character evidence. According to the advisory committee notes to Rule 406, a habit “is the person’s

Seperac-J19 Exam-Released MEE Essay Compilation © 2016-2020 378


regular practice of meeting a particular kind of situation with a specific type of conduct.” The roommate
would testify that texting is the man’s constant and consistent practice. But as noted above, many courts
also require that the behavior be semi-automatic in order to be admissible as habit evidence, and texting
may be too volitional to be considered a semi- automatic habit.

Seperac-J19 Exam-Released MEE Essay Compilation © 2016-2020 379


#086-JUL 2017–MEE Q05: QUESTION FIVE (EVIDENCE/CRIM LAW & PROC)

A woman is on trial for the attempted murder of a man whom she shot with a handgun on March 1.
According to a State A police report:

The woman started dating the man in August. A few months later, after the woman broke up with
him, the man began calling the woman’s cell phone and hanging up without saying anything. In
February, the man called and said, “I promise you’ll be happy if you take me back, but very
unhappy if you do not.” The following week, to protect herself against the man, the woman
lawfully bought a handgun.

On March 1, the woman was working late in her office. At 10:00 p.m., the man entered the
woman’s office without knocking. The woman immediately grabbed the gun and shot the man
once, hitting him in the shoulder.

The police arrived at the scene at 10:10 p.m. By this time, a number of people had gathered outside
the doorway of the woman’s office. A police officer entered the office, and his partner blocked the
doorway so that the woman could not leave and no one could enter. The officer immediately seized
the gun from the woman and asked her, without providing Miranda warnings, “Do you have any
other weapons?” She responded, “I have a can of pepper spray in my purse. Is that a weapon?”

At 10:20 p.m., after the woman had been arrested and the man taken to the hospital, a custodian
told the police officer, “I didn’t see the shooting, but I heard some noises in the hall around 10 and
then a loud bang and screaming.”

A few hours later, at the hospital, the man told the police officer that he had entered the woman’s
office just to speak with her and that the woman had shot him without provocation.

The woman will defend against the attempted murder charge on the ground that she acted in self- defense.
In State A, self-defense is defined as “the use of force upon or toward another person when the defendant
reasonably believes that such force is immediately necessary for the purpose of protecting himself against
the use of unlawful force by such other person on the present occasion.”

State A has adopted evidence rules identical to the Federal Rules of Evidence. State A follows the
doctrine of the Supreme Court of the United States when interpreting protections provided to criminal
defendants under the U.S. Constitution.

The prosecution and the defense have fully complied with all pretrial notice requirements, the authenticity
of all the evidence has been established, and the court has rejected defense objections based on the
Confrontation Clause.

The woman, the man, and the police officer will testify at trial. The custodian is unavailable to testify at
trial.

Under the Miranda doctrine and the rules of evidence, explain how the court should rule on the
admissibility of the following evidence:

Seperac-J19 Exam-Released MEE Essay Compilation © 2016-2020 380


1. Testimony from the woman, offered by the defense, repeating the man’s statement, “I promise
you’ll be happy if you take me back, but very unhappy if you do not.”

2. Testimony from the police officer, offered by the prosecution, repeating the woman’s statement,
“I have a can of pepper spray in my purse. Is that a weapon?”

3. Testimony from the police officer, offered by the prosecution, repeating the custodian’s
statement, “I didn’t see the shooting, but I heard some noises in the hall around 10 and then a loud
bang and screaming.”

Seperac-J19 Exam-Released MEE Essay Compilation © 2016-2020 381


#086: J17-5 MEE: ANSWER: NCBE (EVIDENCE/CRIM LAW & PROC)

POINT (1) [25%] ISSUE: Is a victim’s out-of-court statement admissible when offered not to prove
the truth of the matter asserted in the statement, but solely because it is relevant to prove the
criminal defendant’s reasonable fear of the victim? ANSWER: Yes. The court should admit
testimony from the woman repeating the man’s out-of-court statement to the woman (“I promise
you’ll be happy if you take me back, but very unhappy if you do not.”) because it is relevant to
prove the woman’s reasonable fear of the man and is not hearsay.

POINT (2)(a) [30%] ISSUE: Does admission of a criminal defendant’s out-of-court statement to a
police officer, taken during custodial interrogation and without Miranda warnings or a waiver of
Miranda rights by the defendant, fit the well-established Miranda public safety exception when the
police officer asked a single question to secure weapons immediately after a shooting? ANSWER:
Yes. The court should find that the police officer did not violate the woman’s Miranda rights by
asking her “Do you have any other weapons?” before providing Miranda warnings and obtaining a
waiver because this fits the well-established Miranda public safety exception.

POINT (2)(b) [15%] ISSUE: Is a criminal defendant’s out-of-court statement offered by the
prosecution admissible as an opposing party’s statement because it is not hearsay? ANSWER: Yes.
The court should admit testimony from the police officer repeating the woman’s out-of-court
statement (“I have a can of pepper spray in my purse. Is that a weapon?”). The statement is
relevant, and it is not hearsay because it is an opposing party’s statement.

POINT (3) [30%] ISSUE: Under what circumstances is a witness’s out-of-court statement
admissible under the present-sense-impression or excited-utterance exceptions to the hearsay rule?
ANSWER: The court should find the testimony from the police officer repeating the custodian’s
out-of-court statement (“I didn’t see the shooting, but I heard some noises in the hall around 10 and
then a loud bang and screaming.”) relevant. The court might find that the statement meets either
the present-sense-impression or excited-utterance exception to the rule against hearsay.

ANSWER DISCUSSION:

The court should admit testimony from the woman repeating the man’s out-of-court statement to the
woman (“I promise you’ll be happy if you take me back, but very unhappy if you do not.”) The statement
is relevant to her self-defense claim because it helps prove that the woman had a reasonable fear of the
man on the night of the shooting. The statement is not hearsay because it would not be offered by the
defense to prove the truth of the matter asserted in the statement, and the woman’s beliefs are relevant to
assessing the reasonableness of her fear of the man. The court should also admit testimony from the police
officer repeating the woman’s statement to the police officer (“I have a can of pepper spray in my purse.
Is that a weapon?”). Her statement to the police officer is relevant because it could help the prosecution
prove that, when the woman shot the man rather than using her pepper spray, she used greater force than
was necessary under the circumstances. Admission of the statement does not violate Miranda because the
statement fits the well-established public safety exception. Here, the single question was limited to
weapons and the police officer’s objective was to secure the scene and ensure public safety. Finally, the
woman’s statement is not hearsay when offered by the prosecution because it is an opposing party’s
statement. The court might admit testimony from the police officer repeating the custodian’s out-of-court
statement to the police officer (“I didn’t see the shooting, but I heard some noises in the hall around 10
and then a loud bang and screaming.”). The statement is relevant because it helps to establish the time and

Seperac-J19 Exam-Released MEE Essay Compilation © 2016-2020 382


place of the alleged crime. It is hearsay, but it may fit the hearsay exception for present sense impressions.
The statement may also fit the hearsay exception for excited utterances if the court finds that the custodian
was startled by what he heard and that he remained “under the stress of excitement” when he made the
statement.

ANSWER EXPLANATION:

Explanation to Point-One (25%):

The court should admit testimony from the woman repeating the man’s out-of-court statement to the
woman (“I promise you’ll be happy if you take me back, but very unhappy if you do not.”) because it is
relevant to prove the woman’s reasonable fear of the man and is not hearsay.

Here the man’s statement to the woman is relevant because it has a tendency to make a fact (i.e., the
woman’s reasonable fear of the man) “more probable than it would be without the evidence.” The
woman’s fear of the man is essential to her self-defense claim under the State A standard because she
must prove that when she shot the man, she “reasonably believed” that her use of force was “immediately
necessary for the purpose of protecting herself against the use of unlawful force by another person.” Like
all criminal defendants, the woman has a due process right to present a defense, and the defense seeks to
use the man’s statement to substantiate her self-defense claim.

Although the man’s statement was made outside of court, it would not be hearsay if offered to prove the
woman’s reasonable fear. The Federal Rules of Evidence define hearsay as “a statement that (1) the
declarant does not make while testifying at the current trial or hearing; and (2) a party offers in evidence
to prove the truth of the matter asserted in the statement.” Under this definition, an out-of-court statement
is hearsay only when it is offered to prove “the truth of the matter asserted in the statement.” When
conduct or statements are not used to prove the truth of the matter asserted, the statement should generally
not be treated as hearsay because it does not fit the literal definition and because under these
circumstances the danger of insincerity is usually significantly reduced.

Here, the man’s out-of-court statement is relevant to assessing the reasonableness of the woman’s fear
regardless of whether the jury believes that the man’s statement is true. Specifically, the jury does not
need to decide whether the man actually would have made the woman “happy” or “unhappy” to conclude
that her fear of the man was reasonable. Accordingly, when offered by the defense, this statement should
be admitted.

Explanation to Point-Two(a) (30%):

The court should find that the police officer did not violate the woman’s Miranda rights by asking her
“Do you have any other weapons?” before providing Miranda warnings and obtaining a waiver because
this fits the well-established Miranda public safety exception.

Typically, a person who is in police custody must first receive Miranda warnings and waive Miranda
rights before being subjected to interrogation. For Miranda purposes, custody is established if a
reasonable person under similar circumstances would believe she was not free to leave. For Miranda
purposes, interrogation is established by “either express questioning or its functional equivalent” by the
police. If Miranda has been violated, a court should bar the prosecution from introducing the defendant’s
statement during its case-in-chief. However, Miranda warnings are not required for all custodial
interrogations. For over three decades, the Court has consistently upheld a public safety exception to the
Miranda requirements.

Seperac-J19 Exam-Released MEE Essay Compilation © 2016-2020 383


Here, the police officer engaged in custodial interrogation without first giving the woman Miranda
warnings. The woman was in custody because a reasonable person would have not felt free to leave her
office after she had shot someone, two police officers had arrived to investigate and had seized her gun,
and one of the officers had blocked her doorway. The woman was under interrogation because the police
officer asked her a direct question. However, limited interrogation without Miranda warnings, when
intended to protect public safety, fits the Miranda public safety exception. Here, the police officer asked a
single question about additional weapons intended to secure his own safety and the safety of the people
gathered nearby. Thus, the court should find that the police officer did not violate the woman’s Miranda
rights.

Explanation to Point-Two(b) (15%):

The court should admit testimony from the police officer repeating the woman’s out-of-court statement
(“I have a can of pepper spray in my purse. Is that a weapon?”). The statement is relevant, and it is not
hearsay because it is an opposing party’s statement.

The woman’s statement is relevant to the prosecution’s case because it could help the prosecution prove
that, when the woman shot the man rather than using her pepper spray, she used greater force than was
“immediately necessary for the purpose of protecting herself against the use of unlawful force” under
State A self-defense law.

The woman’s statement is not hearsay when offered by the prosecution. Out-of-court statements by a
party are not hearsay if “offered against an opposing party and made by the party in an individual
capacity.”

Explanation to Point-Three (30%):

The court should find the testimony from the police officer repeating the custodian’s out-of-court
statement (“I didn’t see the shooting, but I heard some noises in the hall around 10 and then a loud bang
and screaming.”) relevant. The court might find that the statement meets either the present-sense-
impression or excited-utterance exception to the rule against hearsay.

The custodian’s out-of-court statement to the police officer is relevant because it has a tendency to make a
fact (i.e., the time and place of the alleged crime) more probable. If this statement is offered for these
purposes, it is hearsay; but it should be admitted because it fits within at least one hearsay exception.

The custodian’s statement might fit the hearsay exception for “present sense impressions” because it is “a
statement describing or explaining an event made immediately after the declarant perceived it.” Here, the
custodian’s statement was limited to a description of recent events, and the statement was made to the
police officer 20 minutes after the events occurred. Whether 20 minutes is close enough in time to the
event to qualify under this exception, however, is unclear.

The custodian’s statement also may fit the hearsay exception for “excited utterances,” if the court
concludes that it is “a statement relating to a startling event made while the declarant was under the stress
of excitement that it caused.” Here, the facts support use of this exception if the court finds (1) that the
custodian was startled by hearing the gunshot and scream and (2) that he remained “under the stress of
excitement” 20 minutes later when he was interviewed by the police officer.

Under the Federal Rules of Evidence, the fact that the custodian is not available to testify has no impact
on the application of either hearsay exception (all FRE 803 hearsay exceptions apply “regardless of

Seperac-J19 Exam-Released MEE Essay Compilation © 2016-2020 384


whether the declarant is available as a witness”). In addition, the fact that the custodian heard but did not
see the relevant events has no bearing on the admissibility of his statement.

[NOTE: An examinee’s conclusion is less important than his or her reasoning on whether the statement
comes in under either exception.]

Seperac-J19 Exam-Released MEE Essay Compilation © 2016-2020 385


#087-JUL 2016–MEE Q02: QUESTION TWO (EVIDENCE/CRIM LAW & PROC)

A defendant was tried before a jury for a robbery that had occurred at Jo-Jo’s Bar on November 30. At
trial, the prosecutor called the police officer who had investigated the crime. Over defense counsel’s
objection, the officer testified as follows:

Officer: I arrived at the defendant’s home on the morning of December 1, the day after the robbery. He
invited me inside, and I asked him, “Did you rob Jo-Jo’s Bar last night?” The defendant immediately
started crying. I decided to take him to the station. Before we left for the station, I read him Miranda
warnings, and he said, “Get me a lawyer,” so I stopped talking to him.

Prosecutor: Did the defendant say anything to you at the station?

Officer: I think he did, but I don’t remember exactly what he said.

Immediately after this testimony, the prosecutor showed the officer a handwritten document. The officer
identified the document as notes she had made on December 2 concerning her interaction with the
defendant on December 1. The prosecutor provided a copy of the document to defense counsel. The
document, which was dated December 2, stated in its entirety:

The defendant burst into tears when asked if he had committed the robbery. He then received and invoked
Miranda rights. I stopped the interrogation and didn’t ask him any more questions, but as soon as we
arrived at the station the defendant said, “I want to make a deal; I think I can help you.” I reread Miranda
warnings, and this time the defendant waived his rights and said, “I have some information that can really
help you with this case.” When I asked him how he could help, the defendant said, “Forget it – I want my
lawyer.” When the defendant’s lawyer arrived 30 minutes later, the defendant was released.

The officer then testified as follows:

Prosecutor: After reviewing your notes, do you remember the events of December 1?

Officer: No, but I do remember making these notes the day after I spoke with the defendant. At that time,
I remembered the conversation clearly, and I was careful to write it down accurately.

Over defense counsel’s objection, the officer was permitted to read the document to the jury. The
prosecutor also asked that the notes be received as an exhibit, and the court granted that request, again
over defense counsel’s objection. The testimony then continued:

Prosecutor: Did you speak to the defendant any time after December 1?

Officer: Following my discovery of additional evidence implicating the defendant in the robbery, I
arrested him on December 20. Again, I read the defendant his Miranda rights. The defendant said that he
would waive his Miranda rights. I then asked him if he was involved in the robbery of Jo-Jo’s Bar, and he
said, “I was there on November 30 and saw the robbery, but I had nothing to do with it.”

Defense counsel objected to the admission of this testimony as well. The court overruled the objection.

The defendant’s trial for robbery was held in a jurisdiction that has adopted all of the Federal Rules of
Evidence.

Seperac-J19 Exam-Released MEE Essay Compilation © 2016-2020 386


Were the following decisions by the trial court proper?

1. Admitting the officer’s testimony that the defendant started crying. Explain.

2. Permitting the officer to read her handwritten notes to the jury. Explain.

3. Admitting the officer’s handwritten notes into evidence as an exhibit. Explain.

4. Admitting the officer’s testimony recounting the defendant’s statement, “I have some
information that can really help you with this case.” Explain.

5. Admitting the officer’s testimony recounting the defendant’s statement, “I was there on
November 30 and saw the robbery, but I had nothing to do with it.” Explain.

Seperac-J19 Exam-Released MEE Essay Compilation © 2016-2020 387


#087: J16-2 MEE: ANSWER: NCBE (EVIDENCE/CRIM LAW & PROC)

POINT (1) [25%] ISSUE: Did the admission of the officer’s testimony that the defendant started
crying violate the defendant’s Miranda rights? Was this evidence inadmissible hearsay? ANSWER:
No. The officer’s testimony that the defendant started crying did not violate the defendant’s
Miranda rights because crying is not testimonial/communicative evidence and the defendant was
not in custody when he burst into tears. This testimony is not hearsay because crying is not a
statement.

POINT (2) [25%] ISSUE: Was the officer properly permitted to read her handwritten notes to the
jury? ANSWER: Yes. The notes made by the officer are hearsay, but the officer was properly
permitted to read the notes to the jury as a recorded recollection. Statements by the defendant
contained in the notes are nonhearsay opposing-party’s statements.

POINT (3) [10%] ISSUE: Were the officer’s notes properly received into evidence as an exhibit?
ANSWER: No. The trial court erred by receiving the officer’s notes as an exhibit.

POINT (4) [25%] ISSUE: Did the admission of the officer’s testimony recounting the defendant’s
statement “I have some information that can really help you with this case” violate the defendant’s
Miranda rights? Was this evidence inadmissible hearsay? ANSWER: No. The admission of the
officer’s testimony recounting the defendant’s statement “I have some information that can really
help you with this case” did not violate the defendant’s Miranda rights because the defendant
initiated communication with the officer. This testimony also is not hearsay because it is an
opposing-party statement.

POINT (5) [15%] ISSUE: Did the admission of the officer’s testimony recounting the defendant’s
statement “I was there on November 30 and saw the robbery, but I had nothing to do with it”
violate the defendant’s Miranda rights? Was this evidence inadmissible hearsay? ANSWER: No.
The admission of the officer’s testimony recounting the defendant’s December 20 statement “I was
there on November 30 and saw the robbery, but I had nothing to do with it” did not violate the
defendant’s Miranda rights because, following the defendant’s second invocation of his right to
counsel of December 1, the defendant was released from interrogative custody for 19 days. This
testimony is nonhearsay because it is an opposing-party statement.

ANSWER DISCUSSION:

The trial court properly permitted the officer to testify that the defendant began crying after the officer
asked him, “Did you rob Jo-Jo’s Bar last night?” Crying (that does not include speaking) is not the type of
compelled communication or testimony protected by the privilege against self- incrimination or the
Miranda rule. Furthermore, Miranda warnings are required only when a suspect is both in custody and
under interrogation. The defendant was under interrogation, but he was not in custody when he burst into
tears. Finally, the defendant’s crying did not raise hearsay concerns because it was not a statement. The
trial court properly permitted the officer to read her notes to the jury. Although the notes are hearsay, they
are admissible under the hearsay exception for recorded recollections. They concern a matter about which
the officer once had knowledge but now has inadequate recollection to testify fully and accurately, they
were made by the officer at a time when the events were fresh in her memory, and she has testified that
the notes are accurate. Thus, the notes were properly admitted and read into evidence. However, the trial
court erred by receiving the notes as an exhibit. A written document admitted as “recorded recollection”

Seperac-J19 Exam-Released MEE Essay Compilation © 2016-2020 388


may be read to the jury, but it may not be received as an exhibit unless it is offered as such by the adverse
party. Here, the adverse party (defense counsel) did not offer the document but, in fact, objected to its
admission. The trial court properly permitted the officer to recount the defendant’s statement “I have
some information that can really help you with this case.” There was no Miranda violation because the
defendant initiated communication with the officer. Under the Federal Rules of Evidence, the out-of-court
statement is not hearsay because it is an “opposing party’s statement.” The trial court properly permitted
the officer to recount the defendant’s December 20 statement. Even though the defendant had invoked his
right to counsel on December 1, the officer arrested him on December 20 and interrogated him without
counsel being present. Nonetheless, the defendant’s December 20 statement—“I was there on November
30 and saw the robbery, but I had nothing to do with it.”—was properly admitted. First, at the time he was
arrested on December 20, the defendant had been out of police custody for over two weeks. This was
sufficient time to terminate the officer’s obligation to honor the defendant’s December 1 invocation of the
right to counsel. Second, the officer gave proper Miranda warnings on December 20, prior to reinitiating
interrogation, and the defendant waived his rights. Finally, under the Federal Rules of Evidence, the out-
of-court statement is not hearsay because it is an opposing party’s statement.

ANSWER EXPLANATION:

Explanation to Point-One (25%):

The officer’s testimony that the defendant started crying did not violate the defendant’s Miranda rights
because crying is not testimonial/communicative evidence and the defendant was not in custody when he
burst into tears. This testimony is not hearsay because crying is not a statement.

The trial court properly permitted the officer to testify that the defendant began crying after she asked
him, “Did you rob Jo-Jo’s Bar last night?” The testimony did not violate Miranda, because crying is not
the type of compelled communication or testimony protected by the privilege against self-incrimination.
Miranda has consistently been interpreted to protect only testimonial/communicative evidence. Here, the
defendant’s crying would not be considered a testimonial communication. In order to be testimonial, an
accused’s communication must itself, explicitly or implicitly, relate a factual assertion or disclose
information.

Moreover, Miranda protections apply only when a suspect is both in custody and under interrogation. The
defendant was under interrogation because the officer had asked him a direct question (interrogation is
defined as either express questioning or its functional equivalent). But for Miranda purposes, custody can
only be established if a reasonable person under similar circumstances would believe that she was in
custody. Here, when the defendant burst into tears, he was not entitled to Miranda protections, as he was
not in custody because a reasonable person who had just voluntarily admitted a police officer into his
home would not believe that he was in custody. For example, in one case, a daytime interrogation in the
suspect’s home by several government agents was not viewed as custody without a more “significant”
deprivation of the suspect’s freedom of action.

Finally, crying does not raise hearsay concerns because it is not a statement. Hearsay is an out-of-court
statement “offered in evidence to prove the truth of the matter asserted in the statement.” Here, the
defendant burst into tears, but did not assert or communicate anything while he was crying.

Explanation to Point-Two (25%):

Seperac-J19 Exam-Released MEE Essay Compilation © 2016-2020 389


The notes made by the officer are hearsay, but the officer was properly permitted to read the notes to the
jury as a recorded recollection. Statements by the defendant contained in the notes are nonhearsay
opposing-party’s statements.

The trial court properly permitted the officer to read her notes to the jury. The document containing the
officer’s notes is hearsay because it is an out-of-court statement that is “offered in evidence to prove the
truth of the matter asserted in the statement.” However, the notes are admissible under the hearsay
exception for recorded recollections. A recorded recollection is “a record that is on a matter the witness
once knew about but now cannot recall well enough to testify fully and accurately and was made when the
matter was fresh in the witness’s memory.” The officer’s notes are a recorded recollection because the
officer, who once had knowledge of the contents of those notes, prepared them herself but had insufficient
recollection of the events they described to testify fully and accurately at trial regarding those matters.

The Federal Rules of Evidence also permit the use of a writing, such as the notes, to refresh a witness’s
recollection for the purpose of testifying. Here, the prosecutor’s effort to use the notes to refresh the
officer’s recollection was not successful because even after reading the notes, the officer still had
insufficient recollection to enable her to testify fully and accurately. However, the officer also testified
that she remembered making the notes and that she was careful to write the notes correctly. Thus, the
court properly admitted the notes into evidence and permitted the officer to read them to the jury.

The notes themselves recount additional out-of-court statements made by the defendant (a second level of
hearsay), but these statements are deemed nonhearsay by the hearsay exception for out-of-court
statements by opposing parties.

Explanation to Point-Three (10%):

The trial court erred by receiving the officer’s notes as an exhibit.

Although the officer’s notes fit the hearsay exception for recorded recollections, under this exception “if
admitted, the record may be read into evidence but may be received as an exhibit only if offered by an
adverse party.” Here, the notes were used by the prosecutor and offered as an exhibit by the prosecutor,
not by an adverse party. Therefore, it was error for the court to admit them as an exhibit.

[NOTE: The officer’s notes are not admissible under the hearsay exception for public records under
Federal Rule of Evidence 803 because the rule specifically exempts statements by law enforcement
personnel when offered in a criminal case.]

Explanation to Point-Four (25%):

The admission of the officer’s testimony recounting the defendant’s statement “I have some information
that can really help you with this case” did not violate the defendant’s Miranda rights because the
defendant initiated communication with the officer. This testimony also is not hearsay because it is an
opposing-party statement.

The trial court properly permitted the officer to testify recounting the defendant’s statement “I have some
information that can really help you with this case.” On December 1, the officer provided the defendant
with Miranda warnings and the defendant invoked his right to counsel by stating, “Get me a lawyer.”
After the defendant’s invocation of his right to counsel, the officer was required to cease the interrogation.
Here, the officer immediately stopped the interrogation.

Seperac-J19 Exam-Released MEE Essay Compilation © 2016-2020 390


However, if a custodial suspect who has invoked his right to counsel initiates post-invocation
communication with the police, the suspect’s subsequent statements may be admissible. Although a
suspect’s questions/comments “relating to routine incidents of the custodial relationship” will not be
treated as initiation of communication with the police, statements from a suspect that clearly indicate a
willingness to speak to the police about matters relating to the investigation will be treated as initiation of
communication.

Here, when the defendant said, “I want to make a deal; I think I can help you,” he was clearly initiating
communication with the officer. Following this initiation of communication by the defendant, the officer
properly provided new Miranda warnings and obtained a waiver of rights. Admission of the defendant’s
subsequent statements did not violate his constitutional rights.

Finally, although the defendant’s statement was made out of court because it was made by the defendant
and offered by the prosecutor against the defendant, it is an opposing-party statement and not considered
hearsay.

Explanation to Point-Five (15%):

The admission of the officer’s testimony recounting the defendant’s December 20 statement “I was there
on November 30 and saw the robbery, but I had nothing to do with it” did not violate the defendant’s
Miranda rights because, following the defendant’s second invocation of his right to counsel of December
1, the defendant was released from interrogative custody for 19 days. This testimony is nonhearsay
because it is an opposing-party statement.

As discussed in Point Four, on December 1 the defendant received Miranda warnings from the officer,
invoked his right to counsel by saying “Get me a lawyer,” initiated communication with the officer,
received a fresh set of Miranda warnings, waived his rights, made a statement, and then re-invoked his
right to counsel by saying, “Forget it – I want my lawyer.” Following the defendant’s second invocation,
he was provided with counsel and released. He was not questioned again until more than two weeks later,
when he was arrested and given fresh Miranda warnings.

The Supreme Court has concluded that if a suspect has been released from interrogative custody, the
police obligation to honor an invocation of the Miranda right to counsel terminates after 14 days.
Although the defendant invoked his right to counsel on December 1 by saying “Forget it – I want my
lawyer,” that earlier invocation by the defendant of his right to counsel was no longer binding on the
officer when she re-arrested the defendant on December 20.

On December 20, the officer properly provided the defendant with new Miranda warnings.

The defendant waived his rights and made a voluntary statement to the officer. Admission of the
statement into evidence did not violate the defendant’s constitutional rights. Moreover, the statement is
not hearsay because it is a statement by an opposing party.

Seperac-J19 Exam-Released MEE Essay Compilation © 2016-2020 391


#088-FEB 2016–MEE Q02: QUESTION TWO (EVIDENCE)

A victim had just walked out of a jewelry store carrying a package containing a diamond bracelet when
someone grabbed him from behind, put a gun to his back, and demanded the package. The victim handed
the package over his shoulder to the robber. The robber said, “Close your eyes and count to 20. I’ll be
watching, and if you mess up, I’ll shoot you.” The victim did as he was told, and when he opened his
eyes, the robber was gone. The victim immediately called 911 on his cell phone.

The victim did not see the robber. A witness on the other side of the street saw the entire encounter. While
the victim was speaking to the 911 operator, the witness ran over to the victim and shouted, “Are you all
right? I saw it all!”

A police officer arrived five minutes later and took a statement from the witness, who was wringing her
hands and pacing. The police officer asked the witness, “What did you see?” The witness responded, “The
robber is about six feet tall. He has brownish hair, almost buzzed to the scalp. He was wearing jeans and a
blue jacket.” The police officer called in the description to the police station.

The defendant, who is over six feet tall and has buzzed brown hair, was picked up 30 minutes later. When
the police officer stopped him, he was six blocks from the scene of the robbery. The defendant was
wearing jeans and a blue jacket but did not have a gun or the bracelet in his possession. He was brought to
the police station for questioning and was placed in a lineup.

The police officer brought the witness to the police station to view the lineup. The witness viewed the
lineup and identified the defendant as the robber. The defendant was arrested and charged with robbery.

One week after the robbery, the witness moved overseas. One year later, at the time of the defendant’s
trial, the witness could not be found.

The victim and the police officer both testified at trial for the prosecution. The police officer testified as
follows:

Question: When you arrived at the scene of the robbery, did you obtain a description of the robber?

Answer: Yes. The witness said that the robber was about six feet tall, with very short, brownish hair,
almost buzzed to the scalp, and that he was wearing jeans and a blue jacket.

Question: Did you gather any other evidence indicating that the defendant committed this robbery?

Answer: Yes. When I was walking into the police station with the victim, we overheard the defendant in
an adjoining room. As soon as the victim heard the defendant’s voice, the victim said, “That’s the voice of
the guy who robbed me.”

Question: What do you know about the defendant?

Answer: He’s a known drug dealer who had been hanging around in the area where the jewelry store is
located for six months before the robbery, constantly causing trouble.

The trial was held in a jurisdiction that has rules identical to the Federal Rules of Evidence. Defense
counsel made timely objections to the admission of the following evidence:

Seperac-J19 Exam-Released MEE Essay Compilation © 2016-2020 392


(a) The police officer’s testimony recounting the witness’s statement at the scene.

(b) The police officer’s testimony recounting the victim’s statement while walking into the police station.

(c) The police officer’s testimony that the defendant is a “known drug dealer who had been hanging
around in the area where the jewelry store is located for six months before the robbery, constantly causing
trouble.”

The trial judge overruled all of defense counsel’s objections.

Was this evidence properly admitted? Explain.

Seperac-J19 Exam-Released MEE Essay Compilation © 2016-2020 393


#088: F16-2 MEE: ANSWER: NCBE (EVIDENCE)

POINT (1)(a) [35%] ISSUE: Is a witness’s out-of-court description of a robber provided to the
police just minutes after viewing the robbery inadmissible hearsay? ANSWER: No. The witness’s
description of the robber is hearsay. The police officer’s testimony repeating that out-of-court
statement was properly admitted because the statement was a present sense impression or an
excited utterance.

POINT (1)(b) [20%] ISSUE: Does admission of a non-testifying witness’s out-of-court statement,
provided in response to police questioning just minutes after viewing the crime, violate the
Confrontation Clause of the Sixth Amendment? ANSWER: Whether admission of the police
officer’s testimony repeating the witness’s description of the robber violated the Confrontation
Clause depends on whether the objective primary purpose of the interrogation was to assist the
police to resolve an ongoing emergency or to assist in the investigation and prosecution of the crime.
Here, the facts support reasonable arguments either way.

POINT (2) [20%] ISSUE: Is an out-of-court vocal identification of a defendant inadmissible


hearsay? ANSWER: No. The victim’s statement “That’s the voice of the guy who robbed me” is
admissible as an out-of-court statement of identification.

POINT (3) [25%] ISSUE: Can the prosecutor introduce testimony that a criminal defendant was a
known drug dealer who had been hanging around for six months constantly causing trouble?
ANSWER: No. The police officer’s testimony that the defendant was a “known drug dealer who
had been hanging around in the area where the jewelry store is located for six months before the
robbery, constantly causing trouble” is inadmissible character evidence.

ANSWER DISCUSSION:

The witness’s out-of-court description of the robber is hearsay. However, the statement fits either the
hearsay exception for present sense impressions or the exception for excited utterances. Even if the
statement falls within a hearsay exception, its admission may have violated the Confrontation Clause of
the Sixth Amendment to the U.S. Constitution. The witness was unavailable to testify at trial, and the
defendant had no opportunity to cross-examine the witness. Whether admission of this out-of-court
statement violated the Confrontation Clause depends on whether the statement was testimonial. If the
objective primary purpose of the witness’s interrogation was to help the police to resolve an ongoing
emergency—the need to apprehend an armed and dangerous robber—the witness’s statement was
nontestimonial and the admission of the statement did not violate the Confrontation Clause. However, if
the objective primary purpose of the witness’s interrogation was to help the police investigation or if the
witness reasonably believed that her statement would be used at trial, her statement was testimonial and
its admission violated the Confrontation Clause. A reasonable argument on this point could be made
either way. Because the victim testified at trial and could have been cross-examined, the officer’s
testimony that the victim had stated “That’s the voice of the guy who robbed me” was admissible as a
statement of vocal identification made after perceiving (hearing) the defendant. The officer’s testimony
that the defendant is a “known drug dealer who had been hanging around in the area where the jewelry
store is located for six months before the robbery, constantly causing trouble” was inadmissible character
evidence. This evidence could not be admitted to prove that the defendant acted in conformity with his
character as a drug dealer or criminal. The evidence also could not be admitted to prove other crimes,
wrongs, or acts because it was too vague and served no relevant non-propensity purpose.

Seperac-J19 Exam-Released MEE Essay Compilation © 2016-2020 394


ANSWER EXPLANATION:

Explanation to Point-One(a) (35%):

The witness’s description of the robber is hearsay. The police officer’s testimony repeating that out-of-
court statement was properly admitted because the statement was a present sense impression or an excited
utterance.

The witness’s description of the robber is an out-of-court statement. This statement was offered at trial to
prove the truth of the matter asserted in the statement—that the person who perpetrated the crime matched
the witness’s description. Thus, the witness’s statement is inadmissible hearsay, unless it falls within a
hearsay exception.

The witness’s out-of-court statement falls within two hearsay exceptions. Both of these exceptions apply
regardless of whether the witness (the declarant) is available to testify at trial.

First, the witness’s description is admissible as a “present sense impression.” Rule 803(1) permits the use
of a hearsay statement “describing or explaining an event or condition made while the declarant was
perceiving the event or condition or immediately thereafter.” Here, the witness saw the robbery, ran
across the street to the victim, and provided a description of the robber to the police officer a few minutes
later. A few minutes’ pause is within the period contemplated in Rule 803(1).

Second, the witness’s description of the robber is admissible as an “excited utterance” under Rule 803(2).
The witness’s description is a statement relating to a startling event or condition made while the declarant
was under the stress of excitement that it caused. Here, the witness had just seen an armed robbery, which
is certainly a startling event for victims and spectators. The witness’s description of the robber related to
the startling event of the robbery. Moreover, the fact that the witness was wringing her hands and pacing
up and down while she made the statement indicates that she was still “under the stress of excitement” of
the event. Finally, although the witness’s description was made in response to the police officer’s
question, this normally has no impact on the operation of the excited utterance exception unless the police
questioning was suggestive.

[NOTE: Rule 801(d)(1)(c) does not apply here because under this rule the declarant must testify and be
subject to cross-examination.]

Explanation to Point-One(b) (20%):

Whether admission of the police officer’s testimony repeating the witness’s description of the robber
violated the Confrontation Clause depends on whether the objective primary purpose of the interrogation
was to assist the police to resolve an ongoing emergency or to assist in the investigation and prosecution
of the crime. Here, the facts support reasonable arguments either way.

The Sixth Amendment of the Constitution gives defendants the right to confront witnesses against them.
The use of an out-of-court statement by the prosecutor violates a defendant’s Sixth Amendment rights,
even if the statement falls within a hearsay exception, if (1) the statement was “testimonial,” (2) the
witness who made the statement is unavailable to testify at trial, and (3) the defendant has not had an
opportunity to cross-examine the witness before trial.

Seperac-J19 Exam-Released MEE Essay Compilation © 2016-2020 395


In Crawford, the Supreme Court noted that statements made to police officers in the course of an
interrogation are often testimonial. The Crawford Court also suggested that statements that an objective
witness reasonably believed would be used as part of a criminal prosecution are testimonial.

Two years later, in Davis v. Washington, the Court drew a distinction between (1) statements made to the
police to assist in the investigation and prosecution, which should be considered testimonial; and (2)
statements made to the police to enable them to meet an ongoing emergency, which should be considered
nontestimonial. Thus, after Davis, statements to the police “under circumstances objectively indicating
that the primary purpose of the interrogation is to enable police assistance to meet an ongoing emergency”
are nontestimonial.

In Michigan v. Bryant, the Court elaborated on its objective “primary purpose of the interrogation”
standard, noting that it requires “a combined inquiry that accounts for both the declarant and the
interrogator.” The Bryant Court also set forth a range of factors that might be used to determine the
existence of an “ongoing emergency.” These include: (1) the nature of the dispute; (2) the scope of the
potential harm to the victim; (3) the threat to additional identifiable victims; (4) the existence of a more
generalized threat to the public; (5) the suspect’s choice of weapon; and (6) whether the suspect remained
“at large” or had been located (but not yet apprehended) by the police and/or any other “first responders.”

Finally, in Ohio v. Clark, a 2015 case, the Court reiterated that “the question is whether, in light of all the
circumstances, viewed objectively, the primary purpose of the conversation was to create an out-of-court
substitute for trial testimony.

Here, the witness’s out-of-court statement to the police officer could be viewed as nontestimonial because
it was made under emergency circumstances. A robbery had recently occurred, the dispute was not
private, the suspect was apparently armed, he may have posed a threat to other victims and/or the general
public, and he was still “at large.” Under these circumstances, the “primary purpose of the interrogation,”
from the perspective of the reasonable declarant and the interrogator, could have been to assist the police
to resolve an ongoing emergency. If the statement is deemed nontestimonial, its admission would not
violate the Confrontation Clause.

On the other hand, there are facts that suggest that the witness’s statement was testimonial. The witness
provided the robber’s identity in response to police questioning about a crime, the robber was no longer
present at the scene, and there was no clear ongoing emergency insofar as the witness was concerned.
Based on these facts, the judge could conclude that the objective “primary purpose of the interrogation”
was to assist in the investigation and prosecution of the robbery. Because the witness was not available to
testify and the defendant had no prior opportunity to cross-examine the witness, if the statement were
deemed testimonial, its admission would violate the Confrontation Clause.

The facts support reasonable arguments either way on this issue.

Explanation to Point-Two (20%):

The victim’s statement “That’s the voice of the guy who robbed me” is admissible as an out-of-court
statement of identification.

The victim’s statement, “That’s the voice of the guy who robbed me,” is an out-of-court statement offered
for the truth of the matter asserted. Nonetheless, a statement is not hearsay when the declarant testifies
and is subject to cross-examination about a prior statement, and the statement identifies a person as
someone the declarant perceived earlier. Most courts have found that statements identifying a person after

Seperac-J19 Exam-Released MEE Essay Compilation © 2016-2020 396


hearing the person’s voice qualify as statements of identification under Rule 801(d)(1)(C). Lay witness
voice identification based on prior familiarity with a voice is typically admissible unless it has been
tainted by impermissibly suggestive questioning or investigative procedures. Expert testimony is not
normally required.

Here, because the victim testified at trial and could have been recalled and cross-examined concerning his
out-of-court statement, the statement was correctly admitted under Rule 801(d)(1)(C) as a statement of
identification.

The victim’s statement is not hearsay under the rules, so it need not fit any of the hearsay exceptions to be
admissible. However, credit could be given to examinees who conclude that the victim’s statement falls
within the hearsay exception for an excited utterance or a present sense impression. The victim’s
statement was arguably an excited utterance because it was made in response to an event (the victim
hearing the defendant’s voice) that was evidently “startling” to the victim, it occurred close in time to the
event, and the victim was “under the stress” of the excitement of the event when making the statement.

Similarly, the victim’s statement was a present sense impression: it described an event (hearing “the voice
of the guy who robbed me”) and was made immediately after the event.

Explanation to Point-Three (25%):

The police officer’s testimony that the defendant was a “known drug dealer who had been hanging around
in the area where the jewelry store is located for six months before the robbery, constantly causing
trouble” is inadmissible character evidence.

In relevant part, Federal Rule of Evidence 404(a)(1) provides that evidence of a person’s character or a
character trait is not admissible to prove that on a particular occasion the person acted in accordance with
the character or trait. Here, the only apparent purpose of the police officer’s testimony is to impugn the
defendant’s character and suggest that he is the sort of person who would commit a robbery.

The testimony is not admissible under Federal Rule of Evidence 404(b)(2), which provides that evidence
of crimes, wrongs, or other acts may be admissible for another non-propensity purpose, such as proving
motive, opportunity, intent, preparation, plan, knowledge, identity, absence of mistake, or lack of
accident. It is unlikely that the police officer’s vague reference to the defendant’s “constantly causing
trouble” would be considered evidence of other crimes, wrongs, or acts under Rule 404(b).

Even in the unlikely event that the judge concluded that a vague reference to causing trouble was
evidence of other crimes, wrongs, or acts, this evidence does not support any relevant non-propensity
issue. For example, some courts have allowed prosecutors to admit evidence of a defendant’s drug habit
to establish the motive for a crime (i.e. to prove that a defendant needed to acquire money to purchase
more drugs). Here, there is no indication that the defendant had a drug habit or that his previous behavior
provided a motive for this robbery.

Seperac-J19 Exam-Released MEE Essay Compilation © 2016-2020 397


#089-JUL 2014–MEE Q05: QUESTION FIVE (EVIDENCE)

A prison inmate has filed a civil rights lawsuit against a guard at the prison, alleging that the guard
violated the inmate’s constitutional rights during an altercation. The inmate and the guard are the only
witnesses to this altercation. They have provided contradictory reports about what occurred.

The trial will be before a jury. The inmate plans to testify at trial. The guard’s counsel has moved for
leave to impeach the inmate with the following:

(a) Twelve years ago, the inmate was convicted of felony distribution of marijuana. He served a three-
year prison sentence, which began immediately after he was convicted. He served his full sentence and
was released from prison nine years ago.

(b) Eight years ago, the inmate pleaded guilty to perjury, a misdemeanor punishable by up to one year in
jail. He paid a $5,000 fine.

(c) Seven years ago, the inmate was convicted of felony sexual assault of a child and is currently serving a
10-year prison sentence for the crime. The victim was the inmate’s daughter, who was 13 years old at the
time of the assault.

The inmate’s counsel objects to the admission of any evidence related to these three convictions and to
any cross-examination based on this evidence.

The guard also plans to testify at trial. The inmate’s counsel has moved for leave to impeach the guard
with the following:

Last year, the guard applied for a promotion to prison supervisor. The guard submitted a résumé to the
state that indicated that he had been awarded a B.A. in Criminal Justice from a local college. An official
copy of the guard’s academic transcript from that college indicates that the guard dropped out after his
first semester and did not receive a degree.

The guard’s counsel objects to the admission of this evidence and to any cross-examination based on this
evidence.

The transcript and the résumé have been properly authenticated. The trial will be held in a jurisdiction that
has adopted all of the Federal Rules of Evidence.

1. What evidence, if any, proffered by the guard to impeach the inmate should be admitted?
Explain.

2. What evidence, if any, proffered by the inmate to impeach the guard should be admitted?
Explain.

Seperac-J19 Exam-Released MEE Essay Compilation © 2016-2020 398


#089: J14-5 MEE: ANSWER: NCBE (EVIDENCE)

POINT (1) [10%] ISSUE: Under what circumstances can evidence of prior convictions be used to
impeach a witness’s credibility in a civil case? ANSWER: The Federal Rules of Evidence permit
impeachment of witnesses with evidence of prior convictions.

POINT (1)(a) [25%] ISSUE: May the inmate’s credibility be impeached by evidence of a 12-year-
old felony drug conviction, if he was released from prison 9 years ago? ANSWER: Yes. The court
should admit evidence of the inmate’s 12-year-old felony marijuana distribution conviction.

POINT (1)(b) [15%] ISSUE: May the inmate’s credibility be impeached by evidence of an 8-year-
old misdemeanor perjury conviction that was punishable by 1 year in jail, if he pleaded guilty and
was sentenced only to pay a $5,000 fine? ANSWER: Yes. The court must admit evidence of the
inmate’s eight-year-old misdemeanor conviction because perjury is a crime of dishonesty.

POINT (1)(c) [20%] ISSUE: May the inmate’s credibility be impeached by evidence of a 7-year-old
sexual assault conviction if the inmate is still serving a 10-year prison sentence and the victim was
his 13-year-old daughter? ANSWER: No. The court should exclude evidence of the inmate’s seven-
year-old felony sexual assault conviction because the probative value of this evidence is
substantially outweighed by the danger of unfair prejudice. In the alternative, the details of the
prior conviction could be excluded.

POINT (2)(a) [15%] ISSUE: May the guard’s credibility be impeached by cross-examination
regarding specific instances of misconduct (i.e., lying on his résumé) relevant to credibility?
ANSWER: Yes. The court should permit the inmate’s counsel to cross-examine the guard
regarding the false statement in his résumé because the guard’s misconduct bears on his
truthfulness.

POINT (2)(b) [15%] ISSUE: May the guard’s credibility be impeached by admission of extrinsic
evidence (his résumé and academic transcript) offered to prove specific instances of misconduct
relevant to credibility? ANSWER: No. The court should exclude extrinsic evidence of the guard’s
non-conviction misconduct, even if the guard denies wrongdoing or refuses to answer questions
about the matter.

ANSWER DISCUSSION:

Under the Federal Rules of Evidence, witnesses can be impeached with evidence of prior convictions
and/or specific instances of misconduct. Whether evidence of prior convictions should be admitted to
impeach generally depends on the nature of the crime, the amount of time that has passed, and (only in
criminal cases) whether the “witness” is the defendant. In this civil case, evidence of the inmate’s
conviction for distribution of marijuana should be admitted to impeach the inmate because he was
convicted of a felony and was released from prison fewer than 10 years ago. Credibility is critically
important in this case because the jury will hear conflicting testimony from the two disputing parties and
there were no other eyewitnesses to the altercation. Under Rule 609, the inmate’s conviction should be
admitted because it has some bearing on his credibility and its probative value is not substantially
outweighed by concerns of unfair prejudice, confusion, or delay. Evidence of the inmate’s misdemeanor
conviction for perjury must be admitted because the crime “required proving – or the witness’s admitting
– a dishonest act or false statement” by the inmate. Evidence of the inmate’s felony conviction for sexual

Seperac-J19 Exam-Released MEE Essay Compilation © 2016-2020 399


assault should be excluded because its probative value is substantially outweighed by the danger of unfair
prejudice to the inmate based on the heinous nature of the crime. In the alternative, the judge could limit
the evidence relating to this conviction by excluding details of the inmate’s crime. In all civil (and
criminal) cases, witnesses can also be impeached with evidence of specific instances of prior misconduct
that did not result in a conviction. Pursuant to Rule 608, misconduct probative of untruthfulness can be
inquired into on cross-examination but cannot be proved through extrinsic evidence. Thus, the inmate’s
counsel should be permitted to cross-examine the guard regarding the false statement in the guard’s
résumé. However, extrinsic evidence of the guard’s misconduct (i.e., the guard’s authenticated résumé
and transcript from the local college) should not be admitted, even if the guard denies wrongdoing or
refuses to answer cross-examination questions about these matters.

ANSWER EXPLANATION:

Explanation to Point-One (10%):

The Federal Rules of Evidence permit impeachment of witnesses with evidence of prior convictions.

Whether convictions should be admitted to impeach generally depends on the nature of the crime, the
amount of time that has passed, and (only in criminal cases) whether the “witness” is the defendant.
Under Rule 609, evidence of prior convictions may be admitted for the purpose of “attacking a witness’s
character for truthfulness.”

There are two basic types of convictions that can be admitted for the purpose of impeachment:

(1) convictions for crimes “punishable by death or by imprisonment for more than one year” (which
generally correlates to “felonies”); and

(2) convictions “for any crimes regardless of the punishment if the court can readily determine that
establishing the elements of the crime required proving – or the witness’s admitting – a dishonest act or
false statement.”

Pursuant to Rule 609, in civil cases, the admission of evidence of a felony conviction is “subject to Rule
403 [which says that a court may exclude relevant evidence if its probative value is substantially
outweighed by other factors].” However, Rule 403 does not protect the witness against admission of prior
convictions involving dishonesty – which must be admitted by the court.

Finally, Federal Rule of Evidence 609 contains the presumption that a conviction that is more than 10
years old, or where more than 10 years has passed since the witness’s release from confinement
(whichever is later), should not be admitted unless “its probative value, supported by specific facts and
circumstances, substantially outweighs its prejudicial effect” and the proponent has provided the adverse
party with reasonable written notice.

Explanation to Point-One(a) (25%):

The court should admit evidence of the inmate’s 12-year-old felony marijuana distribution conviction.

The inmate’s conviction for marijuana distribution was for a felony punishable by imprisonment for more
than one year. Moreover, although the conviction was 12 years ago, the 10-year time limit of Rule 609 is
not exceeded because that time limit runs from the date of either “the witness’s conviction or release from

Seperac-J19 Exam-Released MEE Essay Compilation © 2016-2020 400


confinement for it, whichever is later.” Because the inmate served three years in prison, he was released
from confinement nine years ago.

However, pursuant to Rule 609, the admission of felony convictions to impeach a witness in a civil case is
“subject to Rule 403.” Neither Rule 609(a) nor the advisory committee notes specify which factors courts
should consider when balancing the probative value of a conviction against the dangers identified in Rule
403 (which include (1) unfair prejudice, (2) confusion of the issues, (3) misleading the jury, (4) waste of
time or undue delay, and (5) needless presentation of cumulative evidence).

In this case, credibility is very important because the evidence consists primarily of the testimony of the
disputing parties and there were no other eyewitnesses to the altercation. This enhances the probative
value of any evidence bearing on the inmate’s credibility. A court is likely to conclude that the inmate’s
prior felony drug conviction is relevant to his credibility. For example, prior drug-trafficking crimes are
generally viewed as having some bearing on veracity. Although the probative value of any conviction
diminishes with age, the inmate’s ongoing problems with the law suggest that he has continued (and even
escalated) his criminal behavior over the past nine years. The court should admit this evidence because its
probative value is not substantially outweighed by any Rule 403 concerns. Specifically, any prejudice to
the inmate would be slight because the conviction is unrelated to the altercation at issue and the
conviction was not for a heinous crime that might inflame the jury.

[NOTE: Whether an examinee identifies the jury instruction as containing a “conclusive” or


“mandatory” presumption is less important than the examinee’s analysis of the constitutional infirmities.]

Explanation to Point-One(b) (15%):

The court must admit evidence of the inmate’s eight-year-old misdemeanor conviction because perjury is
a crime of dishonesty.

Rule 609(a)(2) provides that evidence of a criminal conviction “must be admitted if the court can readily
determine that establishing the elements of the crime required proving – or the witness’s admitting – a
dishonest act or false statement.” The inmate’s conviction for perjury would have necessarily required
proving that the inmate engaged in an act of dishonesty. This conviction occurred within the past 10
years, so it “must be admitted” because, in contrast to Rule 609(a)(1), admission under Rule 609(a)(2) is
mandatory and not subject to Rule 403.

Explanation to Point-One(c) (20%):

The court should exclude evidence of the inmate’s seven-year-old felony sexual assault conviction
because the probative value of this evidence is substantially outweighed by the danger of unfair prejudice.
In the alternative, the details of the prior conviction could be excluded.

The inmate’s conviction for felony sexual assault was seven years ago, and he has not yet been released
from incarceration, so Rule 609(a) but not 609(b) is applicable here. This conviction is therefore
admissible to impeach the inmate, unless its probative value is substantially outweighed by the danger of
unfair prejudice or any other Rule 403 concern. Sex crimes are generally not considered relevant to
credibility, so the probative value of this conviction is relatively low. Moreover, the heinous nature of the
inmate’s crime (sexual assault on his daughter) makes the danger of unfair prejudice to the inmate very
high. Thus, the court should exclude evidence of the conviction because it was for a heinous offense that
is likely to inflame the jury and it has little bearing on credibility.

Seperac-J19 Exam-Released MEE Essay Compilation © 2016-2020 401


As an alternative to excluding this evidence, the judge could minimize the unfair prejudice to the inmate
by permitting limited cross-examination but refusing to allow specific questions about the nature of the
inmate’s conviction. For example, a court could limit cross-examination to the fact that the inmate was
convicted of a “felony” or perhaps that he was convicted of a “sexual assault” without identifying the
victim. However, because evidence of the inmate’s prior convictions can be admitted solely for the
purpose of enabling the jury to assess his credibility and because his two earlier convictions should have
already been admitted, the court should exclude all evidence of the felony sexual assault conviction.

Explanation to Point-Two(a) (15%):

The court should permit the inmate’s counsel to cross-examine the guard regarding the false statement in
his résumé because the guard’s misconduct bears on his truthfulness.

The inmate wishes to cross-examine the guard about his prior dishonest behavior – lying on his résumé –
that did not involve a criminal conviction. Rule 608 allows witnesses to be cross-examined about specific
instances of prior non-conviction misconduct probative of untruthfulness “in order to attack the witness’s
character for truthfulness.”

The court’s decision to allow cross-examination about the guard’s prior dishonest behavior depends on
the probative value of such evidence balanced against the danger of unfair prejudice to the guard or any
other Rule 403 concern. Here, the guard’s false statement on his résumé that he obtained a degree in
Criminal Justice is highly probative of his untruthfulness because it grossly misrepresents his actual
academic record, was made recently, and was made with the intent to deceive. Because the probative
value of this evidence is very strong and is not substantially outweighed by any Rule 403 concerns, cross-
examination of the guard on this topic should be permitted. The court may also consider it fair to permit
this cross-examination of the guard on these matters, assuming that one or more of the inmate’s prior
convictions have been admitted to impeach his credibility.

Explanation to Point-Two(b) (15%):

The court should exclude extrinsic evidence of the guard’s non-conviction misconduct, even if the guard
denies wrongdoing or refuses to answer questions about the matter.

Although Rule 608 allows cross-examination about specific instances of prior misconduct probative of
untruthfulness, “extrinsic evidence” offered to prove such misconduct is not admissible. The rationale for
this rule is that allowing the introduction of extrinsic evidence of prior misconduct by witnesses, when
these acts are relevant only to the witnesses’ truthfulness and not to the main issues in the case, would
create too great a risk of confusing the jury and unduly delaying the trial. The court does not have
discretion to admit this extrinsic evidence.

Here, the inmate’s counsel may cross-examine the guard about the false statement on his résumé.
However, the inmate’s counsel must accept the guard’s response. Even if the guard denies wrongdoing or
refuses to answer questions about the matter, the inmate’s counsel cannot introduce the guard’s résumé or
the transcript from the local college to prove the guard’s misconduct.

Seperac-J19 Exam-Released MEE Essay Compilation © 2016-2020 402


#090-JUL 2013–MEE Q04: QUESTION FOUR (EVIDENCE)

The city police department received a 911 call regarding a domestic violence incident. The caller said that
she was staying with her sister and her sister’s boyfriend. The caller said that she had called the police
because her sister’s boyfriend was becoming violent. The police department records all 911 calls. The
relevant portions of the 911 recording are as follows:

Caller: My sister’s boyfriend is out of control right now. He just threw a broken beer bottle at my sister. It
hit her on the arm. Now he’s holding a chair like he’s going to throw that at her, too.

Police Dispatcher: Where is your sister?

Caller: She’s running toward the bathroom.

Police Dispatcher: Is she injured?

Caller: I see some blood on her arm.

Police Dispatcher: Does he have a gun?

Caller: I don’t see a gun.

A nearby police officer arrived on the scene five minutes after the caller telephoned 911. The police
officer found the boyfriend pacing in the front yard and ordered him to sit in the rear seat of the patrol car.
The boyfriend sat in the patrol car, and the officer locked the door from the outside so that the boyfriend
would stay in the car while the officer spoke to the sister.

When the sister saw that her boyfriend was locked in the patrol car, she came out on the porch to speak
with the officer. The sister was in a highly agitated and emotional state, and she had several fresh cuts on
her right arm. The officer asked her how she got the cuts. The sister replied, “My boyfriend threw a bottle
at me which cut my arm.” The sister declined the officer’s offer of medical assistance but said that she
wanted to press charges against her boyfriend. The sister was in tears throughout her conversation with
the officer.

The boyfriend was charged in state court with battery and disorderly conduct. The prosecutor made every
effort to secure the appearance of both the sister and the caller at trial, but when the trial began, the sister
and the caller did not appear.

The prosecutor is attempting to convict the boyfriend without trial testimony from the sister or the caller.
The prosecutor plans to introduce the caller’s statements to the police dispatcher and to call the officer to
testify and to repeat the statements the sister made to him at her house to prove that the boyfriend attacked
the sister.

The 911 recording containing the caller’s statements to the police dispatcher has been properly
authenticated. Defense counsel has objected to the admission of (1) the caller’s statements to the police
dispatcher on the 911 recording and (2) the officer’s testimony repeating the sister’s statements to the
officer (at her house). Defense counsel asserts the following:

a) The caller’s statements to the police dispatcher are inadmissible hearsay.

Seperac-J19 Exam-Released MEE Essay Compilation © 2016-2020 403


b) Admission of the caller’s statements to the police dispatcher would violate the boyfriend’s
constitutional rights.

c) The officer’s testimony repeating the sister’s statements is inadmissible hearsay.

d) Admission of the officer’s testimony repeating the sister’s statements would violate the boyfriend’s
constitutional rights.

This jurisdiction has adopted rules of evidence identical to the Federal Rules of Evidence and interprets
the provisions of the Bill of Rights in accordance with relevant United States Supreme Court precedent.

How should the trial court rule on each defense objection? Explain.

Seperac-J19 Exam-Released MEE Essay Compilation © 2016-2020 404


#090: J13-4 MEE: ANSWER: NCBE (EVIDENCE)

POINT (1) [20%] ISSUE: Do the caller’s statements to the police dispatcher fit a hearsay
exception? ANSWER: Yes. If offered to prove the truth of the matter asserted, that the boyfriend
attacked the sister, the caller’s statements to the police dispatcher are hearsay. However, her
statements fit the hearsay exceptions for “present sense impressions” and “excited utterances,” and
probably fit the exception for statements made for purposes of medical diagnosis.

POINT (2) [25%] ISSUE: Would admission of the caller’s statements to the police dispatcher
violate the Confrontation Clause? ANSWER: No. The Confrontation Clause does not bar the
admission of an out-of-court statement when the objective primary purpose of the interrogation
was to address an ongoing emergency.

POINT (3) [25%] ISSUE: Do the sister’s statements to the officer fit a hearsay exception?
ANSWER: Yes. The sister’s statements to the officer fall under the “excited utterance” exception
to the hearsay rule.

POINT (4) [30%] ISSUE: Would admission of the sister’s statements to the officer violate the
Confrontation Clause? ANSWER: Yes. The Confrontation Clause prohibits the use of the sister’s
statements to the officer because they are testimonial. The objective primary purpose of this
interrogation was not to address an ongoing emergency, but to establish past events potentially
relevant to a later criminal prosecution.

ANSWER DISCUSSION:

The court should allow admission of the caller’s out-of-court statements to the police dispatcher on the
authenticated 911 recording because they fit the hearsay exceptions for “present sense impressions” and
“excited utterances.” The caller’s statements may also fit the exception for “statements made for medical
diagnosis or treatment.” Admission of these statements would not violate the Confrontation Clause of the
Sixth Amendment to the United States Constitution. Although the caller is unavailable to testify and the
boyfriend will have had no pretrial opportunity to cross-examine her, the caller’s statements are not
testimonial because the evidence demonstrates that the reasonable and objective primary purpose of both
the caller and the police dispatcher was to address an ongoing emergency. The sister’s statements to the
officer fit the hearsay exception for “excited utterances.” However, admission of the sister’s statements
through the testimony of the officer would violate the Confrontation Clause because the sister is
unavailable to testify, the boyfriend will have had no opportunity to cross-examine her, and her statements
are testimonial. The sister’s statements are testimonial because the reasonable and objective primary
purpose of the sister and the officer was not to address an ongoing emergency, but to establish past events
potentially relevant to a later criminal prosecution of the boyfriend.

ANSWER EXPLANATION:

Explanation to Point-One (20%):

If offered to prove the truth of the matter asserted, that the boyfriend attacked the sister, the caller’s
statements to the police dispatcher are hearsay. However, her statements fit the hearsay exceptions for
“present sense impressions” and “excited utterances,” and probably fit the exception for statements made
for purposes of medical diagnosis.

Seperac-J19 Exam-Released MEE Essay Compilation © 2016-2020 405


As an out-of-court statement offered to prove the truth of what the caller told the police dispatcher (i.e.,
that her sister’s boyfriend was “out of control,” had thrown a broken beer bottle at her sister, and that her
sister was bleeding), the caller’s statements on the authenticated 911 recording are hearsay. However, the
court should overrule defense counsel’s hearsay objection because the caller’s out-of-court statements fit
two hearsay exceptions.

First, the caller’s statements fit the hearsay exceptions for “present sense impressions.” Pursuant to Rule
803(1), a present sense impression is “a statement describing or explaining an event or condition made
while or immediately after the declarant perceived it.” In this case, the caller described the events to the
police dispatcher as they were happening.

Second, the caller’s statements also fit the hearsay exception for “excited utterances,” pursuant to Rule
803(2). An excited utterance is “a statement relating to a startling event or condition, made while the
declarant was under the stress or excitement that it caused.” Here, the caller telephoned 911 while she was
watching her sister’s boyfriend violently assault her sister. The caller’s statements related to the startling
event of the assault on her sister. Moreover, because the caller made the statements as she watched the
assault, she would have been under the stress caused by the assault on her sister at the time her statements
were made.

Third, some of the caller’s statements probably also fit the hearsay exception for “statements made for
medical diagnosis or treatment.” Statements that fit this exception must have been “made for – and
reasonably pertinent to – medical diagnosis or treatment; and describe medical history; past or present
symptoms or sensations; their inception; or their general cause.” Here, the police dispatcher asked specific
questions about the sister’s injuries and the caller stated that her sister had blood on her arm. Thus, this
portion of the caller’s statement was relevant to the sister’s medical condition and the fact that she might
need medical treatment.

[NOTE: The questions from the police dispatcher are not hearsay because they are not assertions.]

Explanation to Point-Two (25%):

The Confrontation Clause does not bar the admission of an out-of-court statement when the objective
primary purpose of the interrogation was to address an ongoing emergency.

Even when, as here, an out-of-court statement falls within a hearsay exception, the Confrontation Clause
of the Sixth Amendment of the United States Constitution will sometimes preclude the admission of the
statement at trial. The Confrontation Clause gives a defendant in a criminal case the right to be confronted
by the witnesses against him. Under the 6th Amendment to the Constitution, in all criminal prosecutions,
the accused shall enjoy the right to be confronted with the witnesses against him. In Crawford v.
Washington, the Supreme Court held that the admission of out-of-court “testimonial” statements violates
a defendant’s right to confrontation if the witness is unavailable to testify at trial and the defense has not
had a prior opportunity to cross-examine the witness. In this case, the caller is unavailable because the
prosecutor’s efforts to secure her appearance at trial have been unsuccessful, and the defense has not had a
prior opportunity to cross-examine the caller.

Thus, the critical constitutional question is whether the caller’s statements to the police dispatcher were
“testimonial.” After Crawford, many statements made to police officers in the course of an interrogation
are testimonial. However, in Davis v. Washington, the Court clarified that when witnesses make
statements to the police “under circumstances objectively indicating that the primary purpose of the
interrogation is to enable police assistance to meet an ongoing emergency,” these statements are not

Seperac-J19 Exam-Released MEE Essay Compilation © 2016-2020 406


testimonial. More recently, in Michigan v. Bryant, the Court elaborated on the “primary purpose of the
interrogation” standard, noting that primary purpose should be determined using “a combined inquiry that
accounts for both the declarant and the interrogator.” The Bryant Court also specified that “an assessment
of whether an emergency that threatens the police and public is ongoing cannot narrowly focus on
whether the threat solely to the first victim has been neutralized because the threat to first responders and
to the public may continue,” and that “the duration and scope of an emergency may depend in part on the
type of weapon employed,” and/or “the medical condition of the victim.”

Here, the caller telephoned the police for assistance while the boyfriend was assaulting the sister. The
sister was injured and bleeding, and the boyfriend could have posed a threat to the caller and/or first
responders. Thus, it is clear that the caller made statements to the police dispatcher while the emergency
was ongoing. This conclusion is reinforced by the fact that the police dispatcher’s questions were aimed at
addressing the ongoing emergency (i.e., the dispatcher attempted to ascertain the extent of the concurrent
violence, the level of danger, the parties involved, and whether anybody was injured). Based on these
facts, the caller and the police dispatcher both had a primary purpose of resolving the ongoing emergency
created by the boyfriend’s violent physical assault of the sister. Thus, the caller’s statements were not
testimonial and admission of these statements does not violate the Confrontation Clause.

Explanation to Point-Three (25%):

The sister’s statements to the officer fall under the “excited utterance” exception to the hearsay rule.

The court should overrule defense counsel’s hearsay objection to admission of the sister’s statements to
the officer. Testimony repeating the sister’s out-of-court statements would be hearsay because the
prosecution is seeking to use the statements to prove the truth of the matter asserted by the sister (that the
boyfriend had just attacked her). However, the sister’s statements would be admissible pursuant to the
“excited utterance” exception to the hearsay rule. An excited utterance is “a statement relating to a
startling event or condition made while the declarant was under the stress or excitement that it caused.”
Certainly the boyfriend’s violent attack was a startling event and the sister was clearly still “under the
stress or excitement that it caused” when the officer arrived at her house five minutes later. The
statements were made within minutes of the boyfriend’s violent conduct, the sister was in a highly
emotional and agitated state (she was in tears when she spoke to the officer), and her statements related
directly to the boyfriend’s attack, which was the event that caused her to be excited and upset.

[NOTE: The sister’s statements here do not fit the hearsay exception for statements made for the purpose
of medical treatment, because she specifically declined the medical assistance offered by the officer.]

Explanation to Point-Four (30%):

The Confrontation Clause prohibits the use of the sister’s statements to the officer because they are
testimonial. The objective primary purpose of this interrogation was not to address an ongoing
emergency, but to establish past events potentially relevant to a later criminal prosecution.

The dispositive question is whether admission of this out-of-court statement would be precluded under the
Confrontation Clause. The Supreme Court defined testimonial out-of-court statements that require
confrontation in Crawford, Davis, and Bryant. Unlike the caller’s statements to the police dispatcher
during the boyfriend’s attack, the sister’s statements to the officer were made when the emergency was
over and described what the boyfriend had done in the past. Although just a short period of time had
passed and the sister was clearly still upset about the boyfriend’s attack, the boyfriend was now locked in
the back of the police car and no longer posed any danger to the sister, the officer, or anyone else.

Seperac-J19 Exam-Released MEE Essay Compilation © 2016-2020 407


In 2006, in Davis v. Washington, the Supreme Court focused on the question of whether the primary
purpose of each interrogation was to enable police assistance to meet an ongoing emergency. In 2011, the
Bryant Court significantly elaborated on the operation of the standard in a range of contexts. The Bryant
Court cautioned that “an assessment of whether an emergency that threatens the police and public is
ongoing cannot narrowly focus on whether the threat solely to the first victim has been neutralized
because the threat to first responders and to the public may continue.”

Here, the facts indicate that the emergency was resolved by the time the sister made her statement. For
example, courts have held that there is no ongoing emergency after an alleged assailant is placed in a
police car. Under these circumstances, the primary purpose of the interrogation was not to resolve an
ongoing emergency but to establish past events potentially relevant to a later criminal prosecution of the
boyfriend. Because the sister’s statements were testimonial, she is unavailable to testify at trial, and the
boyfriend has had no opportunity to cross-examine her, the admission of the sister’s statements through
the trial testimony of the officer would violate the Confrontation Clause.

The facts specify that the prosecutor seeks to admit the caller’s and the sister’s statements to prove the
attack (i.e., as substantive evidence). However, if either statement were admitted for a non-truth purpose
(e.g., to prove the caller’s or the sister’s state of mind), it would not be barred by the Confrontation
Clause.

Seperac-J19 Exam-Released MEE Essay Compilation © 2016-2020 408


#091-FEB 2013–MEE Q07: QUESTION SEVEN (EVIDENCE)

A woman who owns a motorized scooter brought her scooter to a mechanic for routine maintenance
service. As part of the maintenance service, the mechanic inspected the braking system on the scooter. As
soon as the mechanic finished inspecting and servicing the scooter, he sent the woman a text message to
her cell phone that read, “Just finished your service. When you pick up your scooter, you need to schedule
a follow-up brake repair. We’ll order the parts.”

The woman read the mechanic’s text message and returned the next day to pick up her scooter. As the
woman was wheeling her scooter out of the shop, she saw the mechanic working nearby and asked, “Is
my scooter safe to ride for a while?” The mechanic responded by giving her a thumbs-up. The woman
waved and rode away on the scooter.

One week later, while the woman was riding her scooter, a pedestrian stepped off the curb into a
crosswalk and the woman collided with him, causing the pedestrian severe injuries. The woman had not
had the scooter’s brakes repaired before the accident.

The pedestrian has sued the woman for damages for his injuries resulting from the accident. The
pedestrian has alleged that (1) the woman lost control of the scooter due to its defective brakes, (2) the
woman knew that the brakes needed repair, and (3) it was negligent for the woman to ride the scooter
knowing that its brakes needed to be repaired.

The woman claims that the brakes on the scooter worked perfectly and that the accident happened because
the pedestrian stepped into the crosswalk without looking and the woman had no time to stop. The
woman, the pedestrian, and the mechanic will testify at the upcoming trial.

The pedestrian has proffered an authenticated copy of the mechanic’s text message to the woman.

The woman plans to testify that she asked the mechanic, “Is my scooter safe to ride for a while?” and that
he gave her a thumbs-up in response.

The evidence rules in this jurisdiction are identical to the Federal Rules of Evidence.

Analyze whether each of these items of evidence is relevant and admissible at trial:

1. The authenticated copy of the mechanic’s text message;

2. The woman’s testimony that she asked the mechanic, “Is my scooter safe to ride for a while?”;
and

3. The woman’s testimony describing the mechanic’s thumbs-up.

Seperac-J19 Exam-Released MEE Essay Compilation © 2016-2020 409


#091: F13-7 MEE: ANSWER: NCBE (EVIDENCE)

POINT (1)(a) [20%] ISSUE: Is an authenticated copy of the mechanic’s text message relevant?
ANSWER: Yes. The mechanic’s text message to the woman should be admitted because it is
relevant.

POINT (1)(b) [30%] ISSUE: Is an authenticated copy of the mechanic’s text message admissible?
ANSWER: The mechanic’s text message fits either the hearsay exception for present sense
impressions or the exception for business records, or it is admissible non-hearsay.

POINT (2) [10%] ISSUE: Is the woman’s question, “Is my scooter safe to drive for a while?”
relevant and admissible? ANSWER: Yes. The woman’s question to the mechanic should be
admitted because it is not hearsay.

POINT (3)(a) [20%] ISSUE: Is the woman’s testimony describing a mechanic’s thumbs-up
relevant? ANSWER: Yes. The mechanic’s thumbs-up to the woman is a nonverbal assertion that is
relevant, and the woman’s testimony about that response is admissible.

POINT (3)(a) [20%] ISSUE: Is the woman’s testimony describing a mechanic’s thumbs-up
admissible? ANSWER: No. The mechanic’s thumbs-up is relevant to determine whether the
scooter’s brakes malfunctioned, causing the accident, but if offered for this purpose it is also
hearsay.

ANSWER DISCUSSION:

The mechanic’s text message to the woman is relevant to whether (1) the woman lost control of the
scooter due to its defective brakes, (2) the woman knew that the brakes needed repair, and (3) it was
negligent for the woman to drive the scooter knowing that its brakes needed repair. The mechanic’s text
message is hearsay if it is offered by the pedestrian to prove that the scooter’s brakes needed repair.
However, it fits the hearsay exception for present sense impressions and probably also fits the exception
for business records. The mechanic’s text message is not hearsay if it is instead offered by the pedestrian
to prove the woman’s state of mind (i.e., that she had notice that her brakes needed repair). The woman’s
question to the mechanic and his response are also relevant to whether the brakes caused the accident and
whether the woman was negligent. The question is not hearsay because the woman did not make an
assertion. The mechanic’s thumbs-up response is nonverbal conduct intended by the mechanic as an
assertion and is therefore an out-of-court statement. If the woman offers the mechanic’s statement to
prove that the scooter was actually safe to ride, the woman’s testimony about the statement is hearsay.
However, the mechanic’s statement is not hearsay if it is offered by the woman to prove her state of mind.
Therefore, the woman’s question and the mechanic’s response are admissible to prove the woman’s state
of mind.

ANSWER EXPLANATION:

Explanation to Point-One(a) (20%):

The mechanic’s text message to the woman should be admitted because it is relevant.

Seperac-J19 Exam-Released MEE Essay Compilation © 2016-2020 410


Evidence is relevant if it has “any tendency to make a fact more or less probable than it would be without
the evidence.” “Relevant evidence is admissible,” unless it is inadmissible pursuant to some other rule.

The mechanic’s text message to the woman, “When you pick up your scooter, you need to schedule a
follow-up brake repair. We’ll order the parts,” is relevant for two reasons. First, this evidence has some
tendency to make it more probable that the brakes malfunctioned and caused the accident. Second, it has
some tendency to make it more probable that the woman was negligent in riding her scooter after being
told by the mechanic that it required further repair.

Explanation to Point-One(b) (30%):

The mechanic’s text message fits either the hearsay exception for present sense impressions or the
exception for business records, or it is admissible non-hearsay.

The mechanic’s text message is a statement under Rule 801 because it is “a written assertion.” The text
message is hearsay if the pedestrian offers it to prove the “truth of the matter asserted in the statement”
(i.e., that the scooter’s brakes required repair), which resulted in the woman losing control of the scooter
and causing the accident.

However, the mechanic’s text message fits the hearsay exception for “present sense impressions” under
Rule 803(1) because it is “a statement describing or explaining an event or condition made while or
immediately after the declarant perceived it.” Here, the mechanic’s text message described the condition
of the scooter immediately after he perceived it during the maintenance service.

The mechanic is a person with knowledge of the condition of the scooter, so if text messages regarding
repairs were made and kept by the mechanic in the ordinary course of business, this text message also fits
the business records exception. Under Rule 803, a business record is a record of an act “made at or near
the time by someone with knowledge” and “the record was kept in the course of a regularly conducted
activity of a business” and “making the record was a regular practice of that activity.”

However, the text message is not hearsay if it is instead offered to prove that the woman was negligent
because she rode her scooter after the mechanic told her it required repair. If offered for this purpose, it
would not be offered for the truth of the matter asserted in the statement, but to show the woman’s belief
about the condition of the scooter (her state of mind).

Explanation to Point-Two (10%):

The woman’s question to the mechanic should be admitted because it is not hearsay.

The woman’s question to the mechanic is relevant because, along with the mechanic’s thumbs-up
response, it has some tendency to make it more probable that the woman was not negligent and/or that the
scooter brakes did not malfunction and cause the accident. The woman’s question does not raise hearsay
concerns because it is not an assertion.

Hearsay is defined under Rule 801 as “an oral assertion, written assertion, or nonverbal conduct.”
Although “assertion” is not further defined, “a favorite definition of writers in the [evidence] field for at
least a century and a half is that the word simply means to say that something is so, e.g., that an event
happened or a condition existed.” Under this definition, the woman’s question is not hearsay because it is
not an assertion.

Seperac-J19 Exam-Released MEE Essay Compilation © 2016-2020 411


Explanation to Point-Three(a) (20%):

The mechanic’s thumbs-up to the woman is a nonverbal assertion that is relevant, and the woman’s
testimony about that response is admissible.

Hearsay is defined under Rule 801 as a “statement,” that is, “a person’s oral assertion, written assertion,
or nonverbal conduct, if the person intended it as an assertion.” Here, when the mechanic responded to the
woman’s question (“Is my scooter safe to ride for a while?”) with a thumbs-up gesture, the facts suggest
that he intended his nonverbal conduct as an assertion that, in his opinion, the scooter was safe to ride.

The mechanic’s assertion is relevant and admissible to prove that the woman was not negligent because
the evidence makes it more probable that, at the time of the accident, she believed that the scooter was
safe to ride, despite the fact that the brakes required repair. Admission of the woman’s description of the
mechanic’s thumbs-up for this purpose does not raise hearsay concerns because the evidence would not
be offered for the truth of the matter asserted, but to show the woman’s belief about the condition of the
scooter (her state of mind).

Explanation to Point-Three(b) (20%):

The mechanic’s thumbs-up is relevant to determine whether the scooter’s brakes malfunctioned, causing
the accident, but if offered for this purpose it is also hearsay.

The mechanic’s nonverbal assertion is relevant to the determination of whether the scooter’s brakes
malfunctioned, causing the accident. However, if offered to prove the “truth of the matter asserted in the
statement” (i.e., that the scooter was safe to ride for a while), it is hearsay that does not fit any hearsay
exception.

Seperac-J19 Exam-Released MEE Essay Compilation © 2016-2020 412


#092-FEB 2012–MEE Q01: QUESTION ONE (EVIDENCE)

Six months ago, a woman was taken to a hospital following what she alleged was a sexual assault by a
man during a fraternity party. The woman and the man were both seniors attending the college where the
party was held.

At the time of the alleged assault, the hospital’s policy required that “in all cases of alleged or suspected
sexual assault, non-emergency patients must be interviewed by a victim counselor before receiving
medical treatment.” The woman was deemed a non-emergency patient and was told to wait in the waiting
room to see a victim counselor. Three hours later, the victim counselor finally interviewed the woman.
Thereafter, hospital personnel treated the woman for her injuries and sent her home.

There was no contact between the woman and the man until one week later, when the man sent the
woman a text message on her cell phone. The text message said, “If you are upset about what happened, I
can send you a check for $10,000 to help you forget the whole thing. I can also pay any medical
expenses.” The woman did not respond.

Four months after the alleged assault, the woman contacted a lawyer and filed a civil action against the
man and the hospital. She sought damages from the man for physical injuries resulting from the alleged
assault. She also sought damages from the man for psychological injuries. According to the woman, these
injuries were especially traumatic because of her belief in sexual abstinence before marriage and her lack
of prior sexual experience. She sought damages from the hospital for exacerbating her injuries by
negligently delaying her medical treatment.

The man filed an answer admitting that he had had sexual relations with the woman but asserting that they
were consensual. In its answer, the hospital denied that its conduct had exacerbated the woman’s injuries.

Immediately after filing its answer, the hospital contacted the woman and offered to settle the claim for
$5,000. The woman refused the hospital’s offer.

Five weeks after the woman filed her suit, the hospital changed its policy on dealing with sexual assault
victims to provide that “in all cases of alleged or suspected sexual assault, immediate medical care will be
provided to emergency and non-emergency patients.”

The woman’s suit against the man and the hospital is now set for trial. The following properly filed
motions are before the court:

1. The hospital’s motion to exclude evidence of its new policy providing immediate medical treatment to
emergency and non-emergency patients in all cases of alleged or suspected sexual assault.

2. The hospital’s motion to exclude evidence of its offer to settle with the woman.

3. The man’s motion to exclude evidence of

(a) his offer to pay the woman $10,000.

(b) his offer to pay the woman’s medical expenses.

Seperac-J19 Exam-Released MEE Essay Compilation © 2016-2020 413


4. The man’s motion to admit evidence that the woman had sexual relations with another student during
her junior year.

The rules of evidence in this jurisdiction are identical to the Federal Rules of Evidence.

How should the court rule on each of these motions? Explain.

Seperac-J19 Exam-Released MEE Essay Compilation © 2016-2020 414


#092: F12-1 MEE: ANSWER: NCBE (EVIDENCE)

POINT (1) [20%] ISSUE: Should the court admit evidence of the hospital’s subsequent remedial
measures when those measures would have prevented or mitigated the woman’s injuries if they had
been in place at the time of the alleged tortious conduct? ANSWER: Yes. The court should exclude
evidence of the hospital’s subsequent remedial measures.

POINT (2) [20%] ISSUE: Should the court admit evidence of the hospital’s offer to pay the woman
when that offer was made after her lawsuit had been filed? ANSWER: No. The court should
exclude evidence of the hospital’s offer to settle with the woman.

POINT (3)(a) [20%] ISSUE: Should the court admit evidence of the man’s offer to pay the woman
when the offer was made before the woman’s claim was in dispute? ANSWER: Yes. The court
should admit evidence of the man’s offer to pay the woman $10,000.

POINT (3)(b) [20%] ISSUE: Should the court admit evidence of the man’s offer to pay the
woman’s medical expenses? ANSWER: No. The court should exclude evidence of the man’s offer to
pay the woman’s medical expenses.

POINT (4) [20%] ISSUE: Should the court admit evidence of the woman’s past sexual relations in a
civil suit alleging that she was sexually assaulted? ANSWER: Yes. The court should admit evidence
of the woman’s past sexual behavior because the probative value of the evidence substantially
outweighs the danger of harm to her.

ANSWER DISCUSSION:

While evidence of the hospital’s subsequent remedial measures may be relevant to a determination of its
liability for the woman’s physical and psychological injuries, this evidence is inadmissible because
subsequent remedial measures that would have made an injury less likely to occur are not admissible to
prove negligence or other culpable conduct. Although evidence of the hospital’s offer to settle with the
woman may be relevant to a determination of the hospital’s negligence, settlement offers are inadmissible
to prove negligence. Evidence of the man’s offer to pay the woman $10,000 may be relevant to a
determination of his liability. This evidence is admissible because his offer was made before the woman’s
claim was disputed and before she filed her civil suit. Evidence of the man’s offer to pay the woman’s
medical expenses may be relevant to a determination of his liability. However, this evidence is
inadmissible because evidence of an offer to pay medical and other similar expenses caused by an injury
is inadmissible to prove liability for the injury. Evidence that the woman has engaged in sexual relations
in the past with another student should be admitted. This evidence has significant probative value because
the woman has specifically alleged psychological injuries attributable to her belief in abstinence before
marriage and her lack of prior sexual experience. On these facts, the court should find that the probative
value of this evidence “substantially outweighs” the potential harm that could inure to the woman when
the jurors hear evidence that she engaged in sexual relations with another student.

ANSWER EXPLANATION:

Explanation to Point-One (20%):

The court should exclude evidence of the hospital’s subsequent remedial measures.

Seperac-J19 Exam-Released MEE Essay Compilation © 2016-2020 415


Evidence is relevant if it has “any tendency to make a fact” either “more or less probable than it would be
without the evidence.” “Relevant evidence is admissible,” unless it is inadmissible pursuant to some other
rule. The woman has alleged that the hospital caused the exacerbation of her injuries by negligently
delaying her medical treatment for three hours pursuant to its then-existing standard policy. The hospital’s
decision to change its policy and provide immediate treatment to all sexual assault victims suggests that it
has concluded that its old policy was inappropriate. This may be relevant to the woman’s claim that the
hospital’s decision to deny her immediate treatment was negligent.

However, evidence of the policy change is nonetheless inadmissible. Rule 407 of the Federal Rules of
Evidence provides that “when measures are taken that would have made an earlier injury or harm less
likely to occur, evidence of the subsequent measures is not admissible to prove negligence.” Here, the
hospital’s policy change is a subsequent measure that could have made the woman’s alleged injury less
likely had the policy been in place at the time she visited the hospital. The woman wishes to introduce
evidence of the policy change to establish that the hospital’s behavior pursuant to its prior policy was
negligent. There are no facts indicating that this evidence is relevant to prove anything other than the
hospital’s negligence in this case. But such evidence, even when relevant, is excluded based on “a social
policy of encouraging people to take, or at least not discouraging them from taking, steps in furtherance of
added safety.” Rule 407 is intended to guard against the fact that people “would be less likely to take
subsequent remedial measures if their repairs or improvements would be used against them in a lawsuit”.

Explanation to Point-Two (20%):

The court should exclude evidence of the hospital’s offer to settle with the woman.

Evidence of the hospital’s offer to settle with the woman may be relevant to a determination of its
negligence because this evidence has some tendency to make it more probable that the hospital engaged in
negligent conduct. However, the Federal Rules of Evidence bar evidence that a party “furnished, or
promised, or offered a valuable consideration in compromising or attempting to compromise the disputed
claim.” Rule 408 excludes settlement offers and statements made during settlement negotiations.
Evidence of offers to settle “disputed claims” is excluded based on “the promotion of the public policy
favoring the compromise and settlement of disputes.”

Here, the hospital’s offer to settle with the woman falls squarely within this rule and should be excluded
from evidence. Its offer was an effort to settle with the woman. Because the offer was made after the
woman filed her claim, it was made at a time when her “claim was disputed as to validity or amount.” She
seeks to admit the offer to help prove the hospital’s liability for her claim. Rule 408 bars admission of the
evidence under these circumstances.

[NOTE: In their discussion of this issue, some examinees might discuss whether any out-of-court
statements by the man or the hospital would be excluded from evidence by the hearsay rule. In this case,
the hearsay rule would not exclude these statements because they were made by an opposing party. A
statement by the man is “an opposing party’s statement.” A statement by the hospital is either a statement
“made by a person whom the party authorized to make a statement on the subject,” or a statement “made
by the party’s agent or employee on a matter within the scope of that relationship and while it existed.”]

Explanation to Point-Three(a) (20%):

The court should admit evidence of the man’s offer to pay the woman $10,000.

Seperac-J19 Exam-Released MEE Essay Compilation © 2016-2020 416


Evidence of the man’s statement “If you are upset about what happened, I can send you a check for
$10,000 to help you forget the whole thing” may be relevant to a determination of his liability. This
evidence has some tendency to make it more probable that the man believed that the woman might have
some valid basis for filing a complaint against him. The question is whether this offer to the woman, like
the hospital’s offer, is barred from admission by Rule 408.

As noted above, an offer to compromise is barred from admission only if it is made in response to a
“disputed claim.” In this instance, the man made his offer to the woman almost four months before she
filed this suit. Rule 408 excludes settlement offers and statements made during settlement negotiations.
Although Rule 408 does not require that a lawsuit be filed, here the man made his offer before the woman
had made any claim at all against him, much less a “disputed claim,” and therefore his offer is not
excluded under Rule 408.

Explanation to Point-Three(b) (20%):

The court should exclude evidence of the man’s offer to pay the woman’s medical expenses.

Evidence of the man’s offer to pay the woman’s medical expenses is probably relevant, but the court
should nonetheless exclude the evidence. Federal Rule 409 precludes admission of “evidence of
furnishing, promising to pay, or offering to pay medical expenses resulting from an injury to prove
liability for the injury.” Such evidence is excluded because “such payment or offer is usually made from
humane impulses and to hold otherwise would tend to discourage assistance to the injured person.”
Unlike Rule 408, Rule 409 does not require that the statement be made in response to a disputed claim.

Explanation to Point-Four (20%):

The court should admit evidence of the woman’s past sexual behavior because the probative value of the
evidence substantially outweighs the danger of harm to her.

In any “civil or criminal proceeding involving alleged sexual misconduct,” the “Rape Shield” rule of the
Federal Rules of Evidence generally bars the admission of “evidence offered to prove that a victim
engaged in other sexual behavior.” The purpose of the rule is “to safeguard the alleged victim against the
invasion of privacy, potential embarrassment and sexual stereotyping that is associated with public
disclosure of intimate sexual details and the infusion of sexual innuendo into the fact finding process.”

However, in civil cases, Federal Rule of Evidence 412 permits the admission of otherwise inadmissible
evidence of an alleged victim’s sexual behavior “if its probative value substantially outweighs the danger
of harm to any victim.” In this case, evidence of the woman’s past sexual behavior with another student
has significant probative value because the woman has specifically alleged psychological injuries
attributable to her belief in abstinence before marriage and her lack of prior sexual experience. Here, the
court is likely to find that the probative value of this evidence “substantially outweighs” the potential
harm that could inure to the woman when the jurors learn that she previously had sexual relations with
another student.

Seperac-J19 Exam-Released MEE Essay Compilation © 2016-2020 417


#093-FEB 2011–MEE Q02: QUESTION TWO (EVIDENCE)

On May 5, at 2 p.m. in City Park, Victim was hit from behind and temporarily knocked unconscious.
Upon regaining consciousness moments later, Victim discovered that his bag containing valuables had
been stolen.

While investigating the crime later that day, Police Officer interviewed Witness. Witness told Police
Officer that she had seen the robbery of Victim and had recognized Defendant, a resident of the
neighborhood, as the perpetrator. Witness also told Police Officer that Defendant had a reputation in the
neighborhood for violence, that everyone was afraid of him, and that she shouldn’t be talking to the police
at all. Nevertheless, Witness agreed to accompany Police Officer to police headquarters, where she looked
at photographs of suspects and signed a written statement. The statement read, “I was walking in City
Park on May 5, at 2 p.m., when I saw Defendant. I saw Defendant attack Victim and then run away with
Victim’s bag. I know Defendant from the neighborhood and recognized Defendant as suspect number 1
on the 12-person photograph display shown to me today by Police Officer.”

Defendant was subsequently arrested and charged with robbery and assault.

At Defendant’s trial, Prosecutor called Witness to the stand. In response to questions from Prosecutor,
Witness testified that she had no memory of the incident. She stated that she did not remember seeing
anyone in City Park at the time of the alleged robbery. When Prosecutor asked Witness whether her
sudden memory loss was because she was afraid of Defendant, Witness said that she had never seen
Defendant before in her life and was not afraid of him because she did not know him. When Witness was
asked whether she had told Police Officer that Defendant had robbed Victim, Witness denied ever making
that statement.

Immediately after this testimony, Prosecutor offered Witness’s signed statement into evidence to impeach
Witness’s credibility and to prove that Defendant was in City Park and attacked Victim. An authenticated
copy of Witness’s statement was provided to Defense Counsel. Defense Counsel raised no constitutional
challenges to Witness’s identification of Defendant at police headquarters. However, Defense Counsel
objected to Prosecutor questioning Witness about the statement and to admission of the copy of the
statement. The judge sustained both objections.

After the prosecution had rested, Defense Counsel called Buddy to the stand. Buddy testified that he had
never met Defendant. He also testified that some of his friends had recently met Defendant a few times,
and that they think that Defendant is an honest and gentle person who would never hurt anyone.
Prosecutor objected to this testimony. The judge sustained the objection and excluded Buddy’s testimony.

The rules of evidence in this jurisdiction are identical to the Federal Rules of Evidence.

1. Should the judge have permitted Prosecutor to question Witness about Witness’s written
statement and admitted the copy of the statement to impeach Witness’s credibility? Explain.

2. Should the judge have admitted Witness’s written statement to prove that Defendant was in City
Park and attacked Victim? Explain.

3. Should the judge have admitted Buddy’s testimony to prove Defendant’s character for honesty
and gentleness? Explain.

Seperac-J19 Exam-Released MEE Essay Compilation © 2016-2020 418


#093: F11-2 MEE: ANSWER: NCBE (EVIDENCE)

POINT (1) [25%] ISSUE: May a witness who testifies to a lack of memory at trial be impeached
with a prior inconsistent statement, and may extrinsic evidence of the prior inconsistent statement
be admitted? ANSWER: Yes. A prior inconsistent statement is ordinarily admissible for the
purpose of impeaching the credibility of a witness. Extrinsic evidence of the prior statement, such as
the written statement itself, may be admitted when the witness has been given an opportunity to
explain or deny the statement.

POINT (2) [25%] ISSUE: May a prior statement of identification be admitted? ANSWER: Yes.
Part of Witness’s statement – “I was walking in City Park on May 5, at 2 p.m., when I saw
Defendant. I know Defendant from the neighborhood and recognized Defendant as suspect number
1 on the 12-person photograph display shown to me today by Police Officer” – should have been
admitted as a statement of identification, which is not hearsay.

POINT (3) [20%] ISSUE: May a prior inconsistent statement made to the police be admitted to
prove the elements of the charged offenses? ANSWER: No. The remainder of Witness’s prior
inconsistent statement made to the police – “I saw Defendant attack Victim and then run away with
Victim’s bag” – may not be admitted to prove the elements of the charged offenses because it is
hearsay.

POINT (4) [30%] ISSUE: May a defense witness’s testimony about what his friends think of a
criminal defendant whom they met recently just a few times be admitted to prove that the
defendant is honest and gentle? ANSWER: A criminal defendant may introduce evidence of his
good character relating to a relevant character trait. Character may be proved only through
testimony of reputation or opinion.

ANSWER DISCUSSION:

Witness’s statement to Police Officer contradicted her testimony at trial. This is a “prior inconsistent
statement” that can be used at trial to impeach her credibility. Because Witness’s testimony that she did
not remember the incident and did not recognize Defendant was inconsistent with her written statement,
because her lack of memory was likely feigned, and because she was given an opportunity to explain the
statement, the court should have permitted Prosecutor to question Witness about her prior inconsistent
statement and admitted the authenticated copy of Witness’s statement to impeach Witness. Part of
Witness’s statement – “I was walking in City Park on May 5, at 2 p.m., when I saw Defendant. I know
Defendant from the neighborhood and recognized Defendant as suspect number 1 on the 12-person
photograph display shown to me today by Police Officer” – should have been admitted as a statement of
identification, which is not hearsay as defined in the Federal Rules of Evidence. The remainder of
Witness’s statement – “I saw Defendant attack Victim and then run away with Victim’s bag” – was
properly excluded because, although inconsistent, the statement was not made “under oath subject to the
penalty of perjury at a trial, hearing, or other proceeding, or in a deposition.” Buddy’s testimony was
properly excluded because even relevant character evidence must be introduced in the form of proper
opinion or reputation evidence. Buddy did not testify to his own opinion, and Buddy’s knowledge of his
friends’ opinions did not provide evidence of Defendant’s reputation.

ANSWER EXPLANATION:

Seperac-J19 Exam-Released MEE Essay Compilation © 2016-2020 419


Explanation to Point-One (25%):

A prior inconsistent statement is ordinarily admissible for the purpose of impeaching the credibility of a
witness. Extrinsic evidence of the prior statement, such as the written statement itself, may be admitted
when the witness has been given an opportunity to explain or deny the statement.

It is generally permissible for a litigant to impeach the credibility of any witness, including a witness
called by that litigant. Moreover, it is common and proper to impeach a witness’s credibility “by showing
that the witness has made prior statements inconsistent with his or her testimony at trial.”

In order to introduce a prior out-of-court statement to impeach a witness’s credibility, there must be an
inconsistency between the prior out-of-court statement and the witness’s trial testimony. The
inconsistency must involve a relevant issue. If a witness testifies to a lack of memory regarding a relevant
issue contained in a prior out-of-court statement, the judge may find that this testimony is inconsistent
with the out-of-court statement on the same issue. Moreover, if the judge finds that the lack of memory is
feigned, the court is more likely to find that the testimony and statement are inconsistent. Here, there is
more than a claimed lack of present memory concerning the facts. Witness testified that she had never
seen Defendant in her life and denied that she had told Police Officer that she saw Defendant rob Victim.
Thus, her trial testimony is inconsistent with her previous written statement. Her testimony is also
strongly suggestive of feigned memory loss.

Thus, the court should have allowed Prosecutor to question Witness about the entire statement. The
written statement may also be admitted as extrinsic evidence. Witness should also have been given an
opportunity to explain or deny all or part of the written statement, and the statement should have been
admitted to impeach Witness’s credibility by proving that she made prior inconsistent statements.
Admission of Witness’s statement for the limited purpose of impeaching her trial testimony does not raise
hearsay concerns because it is not admitted to prove the truth of the matter asserted in the statement.

[NOTE: At Defense Counsel’s request, the judge should instruct the jury that the prosecution cannot rely
on Witness’s prior written statement, which was admitted solely to impeach Witness’s credibility, to prove
any of the elements of the charged offenses.]

Explanation to Point-Two (25%):

Part of Witness’s statement – “I was walking in City Park on May 5, at 2 p.m., when I saw Defendant. I
know Defendant from the neighborhood and recognized Defendant as suspect number 1 on the 12-person
photograph display shown to me today by Police Officer” – should have been admitted as a statement of
identification, which is not hearsay.

A prior statement of identification of a person made after perceiving the person is not hearsay if the
witness who made the statement testifies at trial and is subject to cross-examination concerning the
statement. The rationale for this rule is that the opportunity to cross-examine the witness concerning the
statement takes it outside the scope of the hearsay rule. For the statement to be admissible to prove the
truth of the matter asserted under this provision, it is not necessary that the speaker be able to confirm the
identification at trial or remember the identity of the person. If the declarant testifies and is subject to
cross-examination concerning the identification, the declarant’s lack of memory at trial does not defeat
admissibility and the requirements of Rule 801 are fulfilled.

In this case, Witness’s prior written statement included an identification of Defendant. That identification
was made by Witness based on Witness’s knowledge of Defendant from the neighborhood, Witness’s

Seperac-J19 Exam-Released MEE Essay Compilation © 2016-2020 420


direct observation of Defendant in City Park on May 5 at 2 p.m., and Witness’s recognition of
Defendant’s photograph. That prior statement of identification of Defendant is excluded from the general
definition of hearsay when, as here, Witness testifies and is subject to cross-examination concerning the
statement.

Explanation to Point-Three (20%):

The remainder of Witness’s prior inconsistent statement made to the police – “I saw Defendant attack
Victim and then run away with Victim’s bag” – may not be admitted to prove the elements of the charged
offenses because it is hearsay.

Under the Federal Rules of Evidence, certain prior inconsistent statements are not hearsay. However, to
be admissible to prove the truth of the matter asserted, the prior inconsistent statement must be made
“under oath, subject to the penalty of perjury at a trial, hearing, or other proceeding, or in a deposition.”
Witness’s statement to the police does not fit within this rule, nor does it fit within any hearsay exception.

Explanation to Point-Four (30%):

A criminal defendant may introduce evidence of his good character relating to a relevant character trait.
Character may be proved only through testimony of reputation or opinion.

Character evidence is generally inadmissible to prove that a person acted in conformity with a particular
character trait. However, a defendant in a criminal case is permitted to offer evidence of a relevant
character trait to prove that the defendant did not commit the charged offense. Because Defendant is
charged with robbery and assault (both crimes of violence), evidence that Defendant is a “gentle person
who would never hurt anyone” would be relevant. Evidence that Defendant is “honest” is a closer call.
Committing a robbery or an assault is not inconsistent with having a reputation for truth-telling, if being
“honest” means being a truth-teller. On the other hand, robbery is a form of theft, and theft is dishonest.
So a court might admit evidence of Defendant’s honesty, as well as evidence of his gentleness.

However, the Federal Rules of Evidence limit the ways in which a defendant’s character traits may be
proven. In particular, “proof may only be made by testimony as to reputation or by testimony in the form
of an opinion.”

Here, Buddy did not testify as to his own opinion or Defendant’s community reputation. Rather, he said
that a few of his friends think that Defendant is honest and gentle. Buddy’s testimony about his friends’
opinions of Defendant does not qualify as evidence of Defendant’s “reputation in the community.”
“Community” includes circles of associates where one lives, works, or regularly socializes. Buddy’s
friends’ opinions do not qualify as reputation testimony because the friends met Defendant recently and
only a few times.

Seperac-J19 Exam-Released MEE Essay Compilation © 2016-2020 421


#094-FEB 2010–MEE Q07: QUESTION SEVEN (EVIDENCE)

Driver was driving an automobile that struck Pedestrian in the crosswalk of a busy street. Pedestrian
suffered painful fractures and a concussion that affected her memory of the accident.

Pedestrian filed a negligence action against Driver, who responded with a general denial and an assertion
that Pedestrian’s negligence caused her injuries. The parties have stipulated to the severity of Pedestrian’s
injuries, to Pedestrian’s pain and suffering, and to the total value of Pedestrian’s damages. The parties are
scheduled for a jury trial on the issues of both Driver’s and Pedestrian’s negligence.

Pedestrian plans to call Witness to testify at trial. Witness did not see the collision occur. However,
Witness will testify that he walked past Pedestrian no more than five seconds before the collision, at
which time Witness saw that Pedestrian was deeply engrossed in a cell phone conversation. Witness will
also testify that he saw Driver’s distinctive sports car as it approached the intersection in which Pedestrian
was hit. Witness, who has no specialized training, experience, or education, will also offer the opinion
that the car was speeding just prior to the collision because it was traveling noticeably faster than the cars
near it, all of which appeared to be traveling at the same slower speed.

Pedestrian plans to call her Spouse to testify that Pedestrian is very cautious and risk-averse.

Pedestrian also plans to testify at trial. She will not deny having been on the cell phone when Witness
walked by, but will claim to have lowered the cell phone and looked for traffic just prior to entering the
intersection. In fact, Pedestrian intends to testify that she has used a cell phone for many years, that she
talks on it while walking almost every day, and that she invariably ends a call or lowers the cell phone
when preparing to cross a street in order to look both ways before entering the intersection.

Driver intends to undermine Pedestrian’s credibility by introducing evidence of her memory loss.
Pedestrian counters that if the jury hears about some of Pedestrian’s injuries, then it must hear about all of
them, and so Pedestrian seeks to introduce evidence on the full nature and extent of her other injuries.

At the final pretrial motion hearing, Driver’s counsel argued that the court should grant these four motions
in limine:

(1) to exclude Witness’s opinion that Driver was speeding;

(2) to exclude Spouse’s testimony;

(3) to exclude evidence of Pedestrian’s cell phone use at any time other than the day of the collision;

(4) to admit evidence of Pedestrian’s memory loss, but to exclude evidence of Pedestrian’s other injuries.

The evidence rules of this jurisdiction are identical to the Federal Rules of Evidence.

How should the court rule on each of these motions? Explain.

Seperac-J19 Exam-Released MEE Essay Compilation © 2016-2020 422


#094: F10-7 MEE: ANSWER: NCBE (EVIDENCE)

POINT (1) [20%] ISSUE: Should the court exclude Witness’s opinion that Driver was speeding?
ANSWER: Yes. The court should allow Witness to offer an opinion on the speed of Driver’s car.

POINT (2) [25%] ISSUE: Should the court exclude Spouse’s testimony as to Pedestrian’s character
for being cautious and risk-averse? ANSWER: Yes. The court should not admit Spouse’s testimony
regarding Pedestrian’s character trait of being cautious and risk-averse.

POINT (3) [25%] ISSUE: Should the court exclude evidence of Pedestrian’s habit of lowering her
cell phone or ending a call when crossing a street? ANSWER: Yes. The court should admit
Pedestrian’s testimony about Pedestrian’s cell phone usage habits.

POINT (4)(a) [15%] ISSUE: Should the court admit evidence of Pedestrian’s memory loss?
ANSWER: Yes. The court should admit the evidence of Pedestrian’s memory loss.

POINT (4)(b) [15%] ISSUE: If the court does admit evidence of Pedestrian’s memory loss, should it
then also admit the evidence of Pedestrian’s other injuries? ANSWER: No. The court should
exclude the evidence of Pedestrian’s other injuries.

ANSWER DISCUSSION:

Witness’s proffered testimony that Driver was speeding is relevant to the issues of Driver’s negligence
and causation. Although it is an opinion, it is admissible because non-experts may offer opinions on
relevant, non-technical issues, rationally based on their personal perceptions. The court should exclude
Spouse’s testimony that Pedestrian is cautious and risk-averse. Evidence of character traits is not
admissible to prove action in conformity with those character traits in civil cases. Pedestrian’s testimony
that she always lowers her cell phone and checks for traffic before entering intersections is admissible
evidence of a habit and may be offered to prove that she acted in conformity with that habit on the day in
question. Proof of Pedestrian’s concussion and memory loss is relevant and admissible to show that
Pedestrian is not a reliable witness and that the facts to which Pedestrian is testifying are therefore “less
probable” than they would otherwise be. However, even if the court admits evidence of Pedestrian’s
concussion, it should not admit evidence of her other injuries. Because the parties have stipulated the
extent of injury and the resulting damages, evidence of those injuries is irrelevant and likely to unfairly
prejudice the jury.

ANSWER EXPLANATION:

Explanation to Point-One (20%):

The court should allow Witness to offer an opinion on the speed of Driver’s car.

To be admissible, evidence must be relevant. Relevant evidence is any evidence that tends “to make the
existence” of a “fact that is of consequence to the determination of the action more probable or less
probable than it would be without the evidence.” Witness’s testimony is relevant to the determination of
the action because the fact that Driver was speeding, if true, would make it more likely that Driver was
acting negligently and was the cause of the accident. Under Rule 403, relevant evidence can be excluded
under certain circumstances, but there are no facts in this question to suggest an appropriate ground for

Seperac-J19 Exam-Released MEE Essay Compilation © 2016-2020 423


exclusion under this rule. Pursuant to Rule 403, relevant evidence may be excluded if it is unduly
prejudicial, confusing, cumulative, a waste of time, or a source of undue delay in the proceedings.

However, Witness’s testimony is in the form of an opinion. Rule 701 places restrictions on non-expert
opinion evidence. A witness who is not testifying as an expert may offer an opinion only if

(a) the opinion is “rationally based on the witness’s perception” of what happened;

(b) the opinion helps determine “a fact in issue”; and

(c) the opinion is “not based on scientific, technical, or other specialized knowledge” that is governed by
Rule 702.

In this case, Witness’s testimony meets all three requirements. First, Witness saw Driver’s car as it
approached the intersection and was able to perceive its speed. In addition, Witness was able to observe
Driver’s speed relative to that of other cars around it, and Witness observed that Driver’s car was moving
noticeably faster than surrounding cars. These perceptions provide a rational and logical basis for
Witness’s opinion that the car was speeding.

Second, there is no question that whether Driver was speeding is a fact relevant to Pedestrian’s claim. If
Driver was speeding, that fact would make it more likely that Driver was driving negligently and that
Driver’s negligence caused the collision.

Finally, Witness is not an “expert in disguise” who is attempting to sneak in an opinion based on scientific
or engineering principles. Witness’s opinion that Driver was speeding is a commonsense conclusion
based on direct observations. No specialized training, experience, or education is necessary to form a
valid opinion on such a basis.

[NOTE: How much weight the jury should give this opinion is an issue for the jury and does not affect
admissibility.]

Explanation to Point-Two (25%):

The court should not admit Spouse’s testimony regarding Pedestrian’s character trait of being cautious
and risk-averse.

Spouse’s proposed testimony that Pedestrian is very cautious and risk-averse is testimony about
Pedestrian’s character. The purpose of offering the testimony is to suggest that Pedestrian would have
acted consistently with that character on the day in question and would have checked before entering the
intersection. If true, that would tend to establish that Pedestrian was not negligent. The evidence is
therefore relevant.

However, Federal Rule of Evidence 404 restricts the use of character evidence, even when such evidence
is relevant. Rule 404 provides that “evidence of a person’s character or a trait of character is not
admissible for the purpose of proving action in conformity therewith on a particular occasion,” except in
three situations that apply exclusively to criminal trials. Here, Spouse’s testimony that Pedestrian
generally is cautious and risk-averse is relevant precisely because it suggests that on the day of the
collision Pedestrian was acting in a cautious and risk-averse manner. This is exactly what Rule 404
forbids, and the testimony is inadmissible.

Seperac-J19 Exam-Released MEE Essay Compilation © 2016-2020 424


Explanation to Point-Three (25%):

The court should admit Pedestrian’s testimony about Pedestrian’s cell phone usage habits.

Habit evidence is admissible to prove that a person acted in conformity with that habit. This is true even if
the only evidence of a habit is Pedestrian’s own testimony. Rule 406 specifically authorizes the use of
habit evidence “whether corroborated or not and regardless of the presence of eyewitnesses.” Pedestrian’s
testimony will provide sufficient background information to establish that she actually has a habit of
lowering her cell phone and looking both ways when crossing a street. Pedestrian claims to have acted in
conformity with this habit when entering the intersection, which is relevant to Pedestrian’s defense
against Driver’s claim that Pedestrian was contributorily negligent. Therefore, this evidence is admissible.

Explanation to Point-Four(a) (15%):

The court should admit the evidence of Pedestrian’s memory loss.

Pedestrian’s concussion is relevant, albeit indirectly, to determining who is liable for the collision.
Pedestrian intends to testify at trial about what happened, but the concussion has erased some of her
memory. Evidence of memory loss is relevant because it has a tendency to suggest to the jury that
Pedestrian’s testimony concerning the events related to the collision is less reliable. The evidence of
memory loss may make the facts to which Pedestrian testifies “less probable.” Rule 403 is not a basis to
deny Driver’s motion because it would not be “unfairly prejudicial” to Pedestrian to allow Driver to
benefit from Pedestrian’s inability to recall what happened. The cross-examiner is permitted to delve into
the witness’s story to test the witness’s perceptions and memory.

Explanation to Point-Four(b) (15%):

The court should exclude the evidence of Pedestrian’s other injuries.

Rules 401-403 also establish that evidence of Pedestrian’s fractures is not relevant and not admissible.
The parties have stipulated to these injuries and have agreed on the value of the compensation due to
Pedestrian if Driver is responsible for these injuries. Therefore, the fact that Pedestrian suffered these
additional injuries does not, by itself, tend to prove any fact of consequence in dispute that will aid in the
determination of who caused the collision. Therefore, such evidence is a “waste of time” under Rule 403.
It is also “unfairly prejudicial in the sense that the evidence is unnecessary and might cause the jury
improperly to sympathize” with Pedestrian without advancing the factual inquiry at all. For example,
courts have held that bifurcating the damages phase of a trial from liability is necessary if the severity of a
plaintiff’s injuries would prejudice the defendants so severely that if the issues of liability and damages
were not bifurcated, the defendants would not receive a fair trial.

[NOTE: To the extent that Driver’s introduction of evidence of Pedestrian’s memory loss includes
evidence of the severity of Pedestrian’s injuries, Pedestrian can argue, and the judge could find, that the
parties’ stipulation has been violated. If the stipulation is no longer in effect, this could open the door to
admission of evidence of Plaintiff’s other injuries, subject to Federal Rules of Evidence 401 and 403.]

Seperac-J19 Exam-Released MEE Essay Compilation © 2016-2020 425


#095-FEB 2009–MEE Q02: QUESTION TWO (EVIDENCE)

Plaintiff, an employee of Contractor, was injured while using a table saw manufactured by Defendant and
owned by Contractor. Plaintiff sued Defendant in federal court to recover damages for his injuries.

At trial, Defendant called Witness, another employee of Contractor. Neither Witness nor Contractor is a
party to Plaintiff’s action against Defendant. On direct examination, Witness testified that he saw Plaintiff
remove a safety guard from the table saw on the morning of the accident.

During cross-examination by Plaintiff’s Counsel, Witness testified as follows:

PLAINTIFF’S COUNSEL: At the time you applied for your job with Contractor, you had three years of
previous construction experience, didn’t you?

WITNESS: Yes.

PLAINTIFF’S COUNSEL: Didn’t you lie about how much construction experience you had when you
applied for the job with Contractor?

DEFENSE COUNSEL: Objection, inadmissible character evidence.

PLAINTIFF’S COUNSEL: We are impeaching this witness with a specific instance of untruthful conduct
under Rule 608(b), Your Honor.

COURT: Overruled.

PLAINTIFF’S COUNSEL: I’ll repeat my question – didn’t you lie about how much construction
experience you had?

WITNESS: No, I did not.

PLAINTIFF’S COUNSEL: Isn’t Plaintiff’s Exhibit 37 a genuine copy of your job application?

WITNESS: Yes, it is.

PLAINTIFF’S COUNSEL: Didn’t you lie on that application?

WITNESS: No.

PLAINTIFF’S COUNSEL: We offer Plaintiff’s Exhibit 37.

DEFENSE COUNSEL: Objection, inadmissible character evidence.

COURT: Approach the bench. (The following occurred outside the hearing of the jury.)

PLAINTIFF’S COUNSEL: Judge, this is a copy of Witness’s job application in which he represented that
he had twelve years of construction experience when he actually had only three.

COURT: Sustained.

Seperac-J19 Exam-Released MEE Essay Compilation © 2016-2020 426


Plaintiff’s Counsel then asked Witness to review Plaintiff’s Exhibit 37 to refresh his recollection about
whether he had lied. Witness did so and then testified: “I didn’t lie.”

Plaintiff’s Counsel thereafter re-offered Exhibit 37, claiming that it was admissible under Rule 612 to
refresh Witness’s recollection. Defense Counsel objected, and the Court sustained the objection.

Later, Plaintiff’s Counsel called Contractor to testify. During direct examination, Plaintiff’s Counsel
asked Contractor, “Did Witness tell you that he had twelve years of construction experience during his job
interview?” Defense Counsel objected that this was inadmissible character evidence, and the Court
sustained the objection.

Did the Court err in:

1. Overruling Defense Counsel’s objection to cross-examination about an alleged lie by Witness?


Explain.

2. Sustaining Defense Counsel’s objection to the introduction of Exhibit 37 as inadmissible


character evidence? Explain.

3. Sustaining Defense Counsel’s objection to the introduction of Exhibit 37 to refresh the


recollection of Witness? Explain.

4. Sustaining Defense Counsel’s objection to Contractor’s testimony? Explain.

Seperac-J19 Exam-Released MEE Essay Compilation © 2016-2020 427


#095: F09-2 MEE: ANSWER: NCBE (EVIDENCE)

POINT (1) [33%] ISSUE: Under Rule 608(b), was it proper to cross-examine Witness about his
alleged lie on his job application? ANSWER: Yes. The trial court acted within its discretion in
permitting cross-examination concerning the alleged lie on Witness’s job application because lying
about one’s job experience is probative of untruthfulness.

POINT (2) [28%] ISSUE: Under Rule 608(b), was Witness’s job application admissible to
contradict Witness’s denial that he had lied, or was it inadmissible extrinsic evidence? ANSWER:
No. The trial court was within its discretion in excluding Exhibit 37 because Rule 608(b) forbids the
use of extrinsic evidence to attack a witness’s character for truthfulness.

POINT (3) [22%] ISSUE: If Witness’s job application was not admissible for impeachment
purposes under Rule 608(b), could the exhibit be admitted into evidence to refresh recollection
under Rule 612? ANSWER: No. Exhibit 37 was not admissible to refresh Witness’s recollection
because Rule 612 permits only counsel for the opposing party to offer such a document into
evidence.

POINT (4) [17%] ISSUE: Under Rule 608(b), was Contractor’s testimony admissible to impeach
Witness, or was it inadmissible extrinsic evidence? ANSWER: The trial court properly excluded the
testimony of Contractor because testimony by another witness represents extrinsic proof of the
prior bad act and therefore is not admissible under Rule 608(b).

ANSWER DISCUSSION:

Under Rule 608(b), a trial court has discretion to permit cross-examination of a witness concerning a
specific instance of untruthful conduct. Lying on a job application represents untruthful conduct and an
important issue in this case – whether Plaintiff removed the safety guard – will probably turn on the
credibility of the witnesses. Thus, the Court was within its discretion in permitting the cross-examination.
With respect to the job application, Rule 608(b) expressly forbids the admission of extrinsic evidence of a
prior bad act. Some federal courts hold that any document is extrinsic evidence; others hold that a
document is not extrinsic evidence if it can be authenticated by the witness. Thus, the Court was within its
discretion in excluding Exhibit 37, the job application. Exhibit 37 was inadmissible even if it had been
used to refresh Witness’s recollection because the use of Exhibit 37 to refresh Witness’s recollection does
not make it automatically admissible. Rule 612 authorizes admission of a recollection-refreshing exhibit
only if offered by the lawyer who has not used the exhibit to refresh the recollection of the witness.
Finally, Contractor’s testimony was not admissible under Rule 608(b) because, although probative of
untruthfulness, it was clearly extrinsic evidence.

ANSWER EXPLANATION:

Explanation to Point-One (25-35%):

The trial court acted within its discretion in permitting cross-examination concerning the alleged lie on
Witness’s job application because lying about one’s job experience is probative of untruthfulness.

Seperac-J19 Exam-Released MEE Essay Compilation © 2016-2020 428


The standard of review regarding an alleged error in an evidentiary ruling is abuse of discretion. A trial
court’s ruling should be affirmed, even if the court of appeals would make a different ruling, if the
challenged ruling lies within the zone of reasonable disagreement.

Character evidence is generally inadmissible to prove action in conformity with the character trait.
However, when a person testifies as a witness, that person’s credibility becomes a material issue; a
witness’s credibility thus may be attacked by showing that the witness has an untruthful character.
Accordingly, a court may, in its discretion, admit evidence relating to a prior bad act if it is offered during
cross-examination of the witness being impeached and is “probative of untruthfulness.” Conduct that
involves falsehood or deception is generally considered probative of untruthfulness. For example,
submitting a false excuse for being absent from work, giving a false name, or giving a false occupation,
name of business, and purpose in an information request to government.

In this case, the trial court acted properly in allowing counsel to ask Witness if he had lied about his job
experience on his application. The facts suggest Witness’s testimony was critical to establish that Plaintiff
had removed a safety guard from the table saw. This testimony could be devastating to Plaintiff’s claim.
Consequently, this line of inquiry was relevant and probative of Witness’s truthfulness and the court was
reasonable in its decision to permit counsel to cross-examine Witness about his job application.

[NOTE: Admission into evidence of a document containing the falsehood might violate the extrinsic
evidence rule, but questioning the witness about such a document does not.]

Explanation to Point-Two (20-30%):

The trial court was within its discretion in excluding Exhibit 37 because Rule 608(b) forbids the use of
extrinsic evidence to attack a witness’s character for truthfulness.

Rule 608(b) expressly prohibits the use of extrinsic evidence to impeach a witness’s character for
truthfulness: “Specific instances of the conduct of a witness, for the purpose of attacking or supporting the
witness’s character for truthfulness may not be proved by extrinsic evidence.” Thus, although a witness
may be cross-examined about a prior alleged lie, if he refuses to admit to lying, he may not be
contradicted with extrinsic evidence. Rule 608(b) does not define “extrinsic evidence,” and there is
disagreement among federal courts and evidence scholars about the term’s meaning. With respect to
documents, one approach holds that a document is extrinsic evidence in all circumstances. The other,
more lenient approach holds that a document is not extrinsic evidence if the witness being impeached can
provide the foundation for admission of the document. For example, a job application containing a lie
would be extrinsic under the former approach, but not under the latter. As such, a court may exclude the
résumé of a witness offered to impeach the witness under Rule 608(b) by showing a misrepresentation of
educational and employment experience.

Thus, whether the Court should have admitted Exhibit 37 depends on which approach prevails in this
jurisdiction.

[NOTE: An applicant should receive full credit if the applicant recognizes the extrinsic evidence issue.]

Explanation to Point-Three (15-25%):

Exhibit 37 was not admissible to refresh Witness’s recollection because Rule 612 permits only counsel for
the opposing party to offer such a document into evidence.

Seperac-J19 Exam-Released MEE Essay Compilation © 2016-2020 429


Since the Court permitted cross-examination concerning Exhibit 37, it was proper to allow the cross-
examiner to try to get Witness to change his denial of lying on the job application. The cross-examiner
need not take the first answer a witness offers. The cross-examiner thus appropriately showed Exhibit 37
to Witness to refresh his recollection before asking him again whether he had lied.

However, when an otherwise inadmissible document is shown to a witness to refresh his recollection, the
witness must read it to himself. It is improper to allow such a document to be read aloud to the jury, and it
may be admitted as an exhibit only if offered by the lawyer who has not used the exhibit to refresh the
recollection of the witness.

[NOTE: It may have been improper for the Court to permit Plaintiff’s Counsel to attempt to “refresh”
Witness’s recollection, given that Witness did not testify to any lack of memory about what was said on
the job application. However, Defense Counsel did not object to the attempt to refresh recollection, but
only to the admission of the job application as an exhibit. Accordingly, only the admission of the exhibit is
at issue.]

Explanation to Point-Four (10-20%):

The trial court properly excluded the testimony of Contractor because testimony by another witness
represents extrinsic proof of the prior bad act and therefore is not admissible under Rule 608(b).

The trial court correctly refused to permit testimony from Contractor that Witness had claimed 12 years of
experience on his job application. This testimony was relevant only to the witness’s credibility and clearly
constituted extrinsic evidence. For example, a witness’s testimony about defendant’s gang membership is
inadmissible extrinsic evidence of the defendant’s veracity. However, evidence of gang membership is
admissible to show the defendant’s bias.

Seperac-J19 Exam-Released MEE Essay Compilation © 2016-2020 430


#096-FEB 2008–MEE Q04: QUESTION FOUR (EVIDENCE)

Victor was taken by ambulance to a hospital. Standard hospital practice requires the admitting nurse in the
emergency room to record all information provided by a patient about the cause of the patient’s illness or
injury. Following that practice, the admitting nurse, Nurse, asked Victor: “What happened?” Victor
responded: “I was stabbed with a big knife. Dan did it.” Nurse immediately wrote Victor’s statement in
the appropriate place in the hospital record.

One week after his hospital admission, Victor unexpectedly died as a result of the stab wound. Dan was
charged with Victor’s murder.

When Victor’s wife, Wife, heard of Dan’s arrest, she was shocked. She told Friend, “When Victor and I
were alone together in the hospital, he told me who stabbed him, and it wasn’t Dan!” But Wife refused to
tell Friend whom Victor had identified as his assailant.

During the trial, in order to prove that Dan stabbed Victor, the prosecutor offered the hospital record made
by Nurse that contained Victor’s statement that Dan stabbed him. The prosecutor cannot locate Nurse to
testify at trial. Defense counsel objected to admission of the hospital record and the statements in it, but
the court overruled the objection.

During the presentation of Dan’s case, defense counsel suggested that Victor had been attacked by
Stepson, Wife’s child by a previous marriage. Defense counsel called Wife as a witness and questioned
her concerning Victor’s statement to her about the identity of his assailant. Wife refused to answer on the
basis of the marital privilege. The prosecutor objected to the questions directed to Wife on the grounds
that they sought to elicit hearsay. The court sustained both Wife’s claim of privilege and the prosecutor’s
hearsay objection.

1. Did the trial court err in admitting into evidence the hospital record containing Victor’s
statement? Explain.

2. Did the trial court err in sustaining Wife’s claim of privilege? Explain.

3. Did the trial court err in sustaining the prosecutor’s hearsay objection to Wife’s testimony?
Explain.

Seperac-J19 Exam-Released MEE Essay Compilation © 2016-2020 431


#096: F08-4 MEE: ANSWER: NCBE (EVIDENCE)

POINT (1)(a) [22%] ISSUE: When a hospital record is offered into evidence to prove the truth of
an out-of-court statement contained therein, must the record itself, the out-of-court statement, or
both be covered by a hearsay exception? ANSWER: Because the hospital record is being offered to
prove both that Victor made certain statements to Nurse and that those statements are true, it is
double hearsay and is admissible only if both the hospital record and the statements made by Victor
fall within hearsay exceptions.

POINT (1)(b) [22%] ISSUE: Is Nurse’s notation of Victor’s statement a business record that can be
admitted to prove the content of Victor’s statement? ANSWER: Yes. The hospital record satisfies
the business-records exception to the hearsay rule.

POINT (1)(c) [22%] ISSUE: Is Victor’s statement, which relates both to the general cause of his
injuries and to the identity of the person causing the injuries, admissible to prove the truth of the
matter stated? ANSWER: Victor’s statement to Nurse that he was stabbed may be admissible for
the truth of the matter stated because it falls within the hearsay exception for statements made for
the purpose of receiving medical diagnosis and treatment; however, Victor’s identification of his
assailant as “Dan” is not admissible because it was not pertinent to diagnosis or treatment.

POINT (2) [22%] ISSUE: May a spouse assert the marital privilege in order not to testify with
respect to communications with a deceased spouse? ANSWER: Yes. The testimonial spousal
privilege may only be asserted by a spouse who is testifying against an accused spouse. The marital-
confidential-communications privilege can be asserted by either spouse. Wife may assert the
privilege and refuse to testify concerning her confidential communications with Victor.

POINT (3) [13%] ISSUE: When a hearsay statement has been admitted, is an inconsistent hearsay
statement admissible to attack the credibility of the hearsay declarant? ANSWER: The
prosecutor’s hearsay objection to testimony about Victor’s statement to Wife should not have been
sustained because the court had previously admitted Victor’s hearsay statement that Dan was his
assailant. Once an out-of-court statement is offered for its truth, the credibility of the hearsay
declarant may be attacked with an inconsistent statement by that declarant, including statements
that would otherwise be hearsay. Thus, once Victor’s identification of Dan as his assailant was
admitted into evidence, his inconsistent statement to Wife could be admitted to attack his
credibility.

ANSWER DISCUSSION:

Both the hospital record and Victor’s statement in it are hearsay, and each must fall within an exception to
the hearsay rule in order for Victor’s statement to be admissible for its truth. The hospital record falls
within the business-records exception to the hearsay rule. Victor’s statement that he was stabbed may fall
within the hearsay exception for statements relating to medical diagnosis and treatment, but Victor’s
identification of Dan as his assailant is not pertinent to his medical treatment and would not fall within the
medical diagnosis or any other hearsay exception. Consequently, that portion of Victor’s statement is
inadmissible to prove that Dan stabbed Victor and should have been excluded from evidence. In most
jurisdictions, Wife will be able to invoke the privilege for confidential marital communications and refuse
to testify to what Victor told her. In some jurisdictions, however, only the “communicating spouse” can
invoke the privilege, so in those jurisdictions, Wife would be required to testify. If Wife cannot invoke the

Seperac-J19 Exam-Released MEE Essay Compilation © 2016-2020 432


marital privilege, her testimony as to Victor’s identification of his assailant is admissible only for the
purposes of impeaching Victor’s hearsay declaration that Dan stabbed him. If that hearsay declaration is
not admitted, Victor’s statement to Wife should also be excluded.

ANSWER EXPLANATION:

Explanation to Point-One(a) (20-30%):

Because the hospital record is being offered to prove both that Victor made certain statements to Nurse
and that those statements are true, it is double hearsay and is admissible only if both the hospital record
and the statements made by Victor fall within hearsay exceptions.

Hearsay is an out-of-court statement “offered in evidence to prove the truth of the matter asserted.”
Hearsay is generally inadmissible.

Here, the hospital record is “double hearsay,” and both levels of hearsay must fall within a hearsay
exception or the record is inadmissible. The record itself is the first level of hearsay. The record is an
“out-of-court statement,” and the prosecutor is offering it to prove the truth of what it says – that Victor
told Nurse that Dan stabbed him. Without Nurse present to prove what she heard Victor say, her “out-of-
court statement” about Victor’s words, in the form of her notation in his hospital record, is admissible
only if the record falls within a hearsay exception.

The prosecutor is also seeking to prove that what Victor said (“Dan stabbed me”) is true. This is also
hearsay because Victor’s statement was made out of court and is offered for its truth. Therefore, the
prosecutor must show that Victor’s statement falls within a hearsay exception.

Explanation to Point-One(b) (20-30%):

The hospital record satisfies the business-records exception to the hearsay rule.

A record of “acts, events, conditions, opinions, or diagnoses” is admissible under the business-records
exception to the hearsay rule if it is “made at or near the time” of the recorded event by “a person with
knowledge” of the event. Additionally, the making of the record must occur in the course of a regularly
conducted business activity, and it must be the regular practice of the business to make the record. This
foundation can be introduced through the testimony of a record custodian or other person with knowledge
of the method of record keeping. It is not necessary that the person with knowledge of the matter entered
testify at trial.

Nurse’s notation in the hospital record concerning Victor’s statements to her falls within the business-
records exception. Nurse had personal knowledge of what Victor said, and she wrote down his statements
“at or near the time” that he made them. The facts state that it is the regular practice of the hospital to
record this sort of information in a patient’s record.

Explanation to Point-One(c) (20-30%):

Victor’s statement to Nurse that he was stabbed may be admissible for the truth of the matter stated
because it falls within the hearsay exception for statements made for the purpose of receiving medical
diagnosis and treatment; however, Victor’s identification of his assailant as “Dan” is not admissible
because it was not pertinent to diagnosis or treatment.

Seperac-J19 Exam-Released MEE Essay Compilation © 2016-2020 433


As noted earlier, Victor’s statement is itself hearsay because the prosecutor is offering it to prove the truth
of the matter asserted – that Dan stabbed Victor. Therefore, even if the hospital record is admissible to
prove what Victor said, the statement should still be excluded unless Victor’s statement itself falls within
a hearsay exception.

Statements made by a person who is seeking medical treatment are exempted from the ban on hearsay if
the statements concern medical history, symptoms, “or the inception or general character of the cause” of
the symptoms. Hence, Victor’s statement to Nurse about the cause of his injuries may qualify for this
hearsay exception.

However, the rationale for this hearsay exception is that statements made by people seeking medical
treatment will generally be reliable because of the declarants’ knowledge that medical diagnosis or
treatment will be facilitated by such statements (or hindered if the statements are false). Thus, this hearsay
exception is limited to statements that are “reasonably pertinent to diagnosis or treatment.”

Statements which relate to the cause of the medical condition being diagnosed or treated are generally
viewed as “pertinent to diagnosis or treatment.” Thus, Victor’s statement that he was stabbed with a knife
would fall within the exception and would be admissible. But statements of fault ordinarily are not
admitted under this exception, because the identity of the person who was at fault in causing the injury is
not relevant to the treatment that is to be given. Thus, the portion of Victor’s statement identifying Dan as
the assailant is inadmissible.

In short, although the hospital record of what Victor said is admissible even though it is hearsay, Victor’s
identification of his assailant is also hearsay and is inadmissible because it does not fall within an
exception. Therefore, the trial court should not admit the record containing that statement.

Explanation to Point-Two (20-30%):

The testimonial spousal privilege may only be asserted by a spouse who is testifying against an accused
spouse. The marital-confidential-communications privilege can be asserted by either spouse. Wife may
assert the privilege and refuse to testify concerning her confidential communications with Victor.

Two evidentiary privileges apply to the marital relationship. The federal courts recognize a testimonial or
witness privilege under which a witness-spouse has the right not to testify against an accused spouse in a
criminal case. This privilege is inapplicable in this case because Wife has not been asked to testify against
her husband, Victor.

The federal courts and most states also recognize a second privilege for confidential communications
between spouses when those communications occur during the marriage. The privilege protects
communications during marriage even if the marriage no longer exists and even if one of the parties to the
marriage is dead.

Under the majority view, both spouses hold the privilege for all communications between them. Here,
Wife could invoke the privilege and be excused from testifying as to Victor’s statements to her because
all the requirements of the privilege are met: Victor’s statements were statements of one spouse to another
in private (it appears that no one overheard them), and neither Wife nor Victor revealed their exact
content to anyone else. These confidential communications between spouses are protected by the
privilege.

Seperac-J19 Exam-Released MEE Essay Compilation © 2016-2020 434


Some courts, however, have taken the position that only the communicating spouse can assert the
privilege. Under that minority view, Wife could not assert the privilege and would have to reveal what
Victor said.

Here, however, it is possible that Wife will not be allowed to invoke the privilege because she broke the
confidentiality of her communications with her husband when she told Friend, a third party, some of what
her husband told her. The marital communications privilege is intended to protect the confidentiality of
spousal discussions. Where one spouse has revealed the content of those communications to a third
person, then confidentiality no longer exists and the privilege probably should not apply. At the very least,
Wife might be required to testify to the fact that she has already revealed to Friend: that Victor told her
that he was attacked by someone other than Dan.

Explanation to Point-Three (10-20%):

The prosecutor’s hearsay objection to testimony about Victor’s statement to Wife should not have been
sustained because the court had previously admitted Victor’s hearsay statement that Dan was his assailant.
Once an out-of-court statement is offered for its truth, the credibility of the hearsay declarant may be
attacked with an inconsistent statement by that declarant, including statements that would otherwise be
hearsay. Thus, once Victor’s identification of Dan as his assailant was admitted into evidence, his
inconsistent statement to Wife could be admitted to attack his credibility.

Victor’s statement to Wife that Dan was not his assailant is hearsay (because it was made outside the
courtroom) and could not have been admitted to prove the truth of the matter stated. Victor’s statement is
nonetheless admissible to impeach his credibility.

On the facts, the court has admitted Victor’s hearsay statement to Nurse that Dan attacked him. Once a
hearsay statement is admitted into evidence, the hearsay declarant’s credibility may be attacked just as
though the hearsay declarant were a witness at trial. In addition, any inconsistent statement made by the
hearsay declarant may be admitted to impeach the declarant’s credibility. Thus, if the hospital record is
admitted to prove that Victor identified Dan as his assailant, then Victor’s statement to Wife (identifying
someone else as his assailant) is admissible to attack Victor’s credibility, despite the existence of the
hearsay rule. (Of course, Wife’s privilege claim may still prevent admission of the statement.)

Seperac-J19 Exam-Released MEE Essay Compilation © 2016-2020 435


REAL PROPERTY: 13 OF 24 MEE EXAMS: (54%)
#097-JUL 2018–MEE Q03: QUESTION THREE (REAL PROPERTY)

In 2015, a man purchased a convenience store that sells gasoline and snack-type grocery items. The man’s
store is located within two miles of three other convenience stores that are larger and contain small dining
areas. When he bought the store, the man planned to expand it as soon as he could in order to offer the
same services and products as the other three stores in the area.

In 2017, the local zoning board passed an ordinance that rezoned the district in which all four stores are
located from “light commercial” to “residential.” Convenience stores are not “residential” uses. The
zoning ordinance contained typical language protecting existing nonconforming uses.

In early 2018, the man decided to expand his store by 1,100 square feet to add a small dining area. To
finance this expansion, he obtained a $200,000 loan commitment from a local bank, with the funds to be
disbursed at such times and in such amounts as the bank determined to be appropriate if, in the bank’s
good-faith judgment, there was “satisfactory progress” being made on the project. Documents reflecting
this commitment were signed by the man and the bank, and a mortgage to secure the repayment of the
loan was promptly and properly filed in the local land records office.

Two weeks after obtaining the loan commitment, the man signed a contract with a general contractor for
construction of the store expansion. In compliance with its loan commitment, the bank disbursed $50,000
to the man, who, in turn, paid that sum to the general contractor. Construction began immediately
thereafter.

Four weeks into the project, a plumbing subcontractor installed all the plumbing fixtures. After the
general contractor failed to pay the $20,000 agreed price to the subcontractor, the subcontractor
immediately filed a mechanic’s lien against the man’s property in the local land records office to secure
its claim for $20,000.

Eight weeks into the project, the bank disbursed an additional $40,000 to the man, who, in turn, paid
$40,000 to the general contractor. The general contractor used these funds to pay various creditors, but
not the plumbing subcontractor.

Two weeks ago, a bank loan officer learned for the first time about the mechanic’s lien. The next day,
when the man approached the bank about making another disbursement, the loan officer refused. The man
asserts that, under the loan agreement, the bank is obligated to disburse further funds.

1. Is the expansion project a nonconforming use? Explain.

2. Assuming that the expansion project does not violate the zoning classification, is the bank
obligated to disburse further funds? Explain.

3. Does the mechanic’s lien have priority, in whole or in part, over the bank’s mortgage? Explain.

Seperac-J19 Exam-Released MEE Essay Compilation © 2016-2020 436


#097: J18-3 MEE: ANSWER: NCBE (REAL PROPERTY)

POINT (1) [35%] ISSUE: Is the expansion project a nonconforming use? ANSWER: No. While
there are good arguments either way, the expansion to the convenience store is most likely not
exempt from the zoning ordinance as a nonconforming use because it would increase the intensity
of use.

POINT (2) [35%] ISSUE: Under the provisions of the loan commitment, is the bank obligated to
disburse further funds? ANSWER: No. The bank’s obligation to make advances to the man is
optional because future advances are conditioned on the bank’s determination that the borrower is
making satisfactory progress on the construction project.

POINT (3) [30%] ISSUE: Does the mechanic’s lien have priority, in whole or in part, over the
bank’s mortgage? ANSWER: With respect to the $50,000 payment, the bank’s mortgage has
priority as it preceded the filing of the mechanic’s lien. With respect to the $40,000 payment, under
the majority rule, the bank’s mortgage has priority over the mechanic’s lien because the bank did
not have actual knowledge that the subcontractor had not been paid when the bank made the
payment. Under the minority rule, the bank’s mortgage would not have priority because the lien
was properly filed before the bank made this payment.

ANSWER DISCUSSION:

The expansion is most likely not a nonconforming use because it results in a substantial change in use that
may increase the volume of users at the store’s location. The bank is not obligated to disburse additional
funds because the bank and the man entered into a future-advances loan/mortgage arrangement under
which future advances depend on the bank’s judgment that satisfactory progress is being made on the
project. The bank’s mortgage has priority over the mechanic’s lien as to the original payment ($50,000)
because the mortgage was recorded and the payment was made before the filing of the mechanic’s lien.
Under the majority rule, the bank has priority with respect to the first future advance ($40,000) because,
when it was made, the bank did not have actual notice that the subcontractor had not been paid; advances
made after the bank acquired such knowledge do not have priority over the mechanic’s lien. Under the
minority rule, the mechanic’s lien is senior to all the future advances but not to the original $50,000
payment because by recording the lien, the subcontractor provided the bank with constructive notice of its
interest. [NOTE: An examinee who wrongly concludes that future advances are obligatory should
conclude that the bank has priority with respect to all disbursements made under the loan commitment
regardless of when made.]

ANSWER EXPLANATION:

Explanation to Point One (35%):

While there are good arguments either way, the expansion to the convenience store is most likely not
exempt from the zoning ordinance as a nonconforming use because it would increase the intensity of use.

This zoning ordinance allows uses that existed prior to the enactment of the ordinance. Prior- enactment
uses are known as “nonconforming uses” and are justified on both fairness and practical monetary
grounds. Here, the convenience store without any expansion is protected by the nonconforming use
provision in the ordinance.

Seperac-J19 Exam-Released MEE Essay Compilation © 2016-2020 437


While a nonconforming use cannot be extended or intensified in ways that constitute a substantial change,
insubstantial changes are permitted, and “owners are entitled to make reasonable alterations to repair their
facilities and render them practicable for their purposes.” Thus, the issue here is whether the proposed
changes are substantial or insubstantial. Because a change would be inconsistent with the zoning law,
“any doubts are resolved against the change.”

Likely, the expansion is not a nonconforming use. The expansion of the convenience store clearly will
increase the intensity of the use in the newly zoned residential neighborhood. In fact, that is precisely
what the man hopes to achieve with the expansion. Increased use will result in more traffic and possibly
impose other externalities on the now residential community. While the goal of the nonconforming-use
doctrine is to protect prior investment, it is not to change an existing use into a different investment.
Furthermore, applying the nonconforming-use doctrine here would only serve to protect a future
investment, not a prior investment. When the man bought the store, he had only a mere expectation of
future investment (i.e., that he would expand the store to offer products and services similar to those
offered by the other convenience stores). A mere expectation is not an investment.

On the other hand, the proposed alteration should arguably be treated as a permissible nonconforming use
because it is necessary for the man’s store to compete effectively with the three other stores in the area.
Furthermore, other local convenience stores that sell gas have small dining areas within them, and thus the
expansion could be viewed as merely a normal expansion of a prior nonconforming use. Additionally, the
point of the nonconforming-use doctrine, in part, is to protect the investment-backed expectations of
persons, such as the man here, who purchased and operated the store in reliance on the law in place when
the property was acquired. This expectation exists here, given that when the man purchased the store there
were three more expansive convenience stores in the area, and he had expected that in the future he would
expand the store to offer the same products and services as his competitors.

However, notwithstanding the argument in favor of applying the nonconforming-use doctrine to the
store’s expansion, given that “any doubts are resolved against the change,” it is more likely that the
expansion is not a nonconforming use.

Explanation to Point Two (35%):

The bank’s obligation to make advances to the man is optional because future advances are conditioned
on the bank’s determination that the borrower is making satisfactory progress on the construction project.

The type of loan/mortgage here is a so-called “future-advances” agreement, and it is very common in the
construction industry. A future-advances loan is one in which the lender provides funds to the borrower
over a period of time, rather than in a lump sum at the signing of the mortgage, in order to finance
ongoing construction when the funds are needed. Future-advances loans are either obligatory or optional.
A future-advances loan is obligatory if the lender has a duty to advance the funds; the loan is optional if
the lender does not have a duty to advance funds but has discretion whether to make future advances.

In this case, the bank’s commitment to make future advances is optional. A commitment is obligatory
only when the lender commits to making future advances without discretionary conditions. With a
satisfactory-progress condition, the bank has no “definite obligation to advance any funds,” making the
future advances optional. The satisfactory-progress condition is designed, among other things, to permit
the bank to withhold funds if any difficulties associated with the project’s completion threaten the bank’s
security by reducing the growth in value anticipated from the project.

Seperac-J19 Exam-Released MEE Essay Compilation © 2016-2020 438


Here, the bank has a right to withhold funds if it concludes in good faith that there is not “satisfactory
progress” on the project. The existence of a mechanic’s lien against the project could undermine the
bank’s security with respect to future amounts it might disburse if the mechanic’s lien has priority. See
Point Three. Thus, the bank officer’s concerns could justify withholding funding.

Explanation to Point Three (30%):

With respect to the $50,000 payment, the bank’s mortgage has priority as it preceded the filing of the
mechanic’s lien. With respect to the $40,000 payment, under the majority rule, the bank’s mortgage has
priority over the mechanic’s lien because the bank did not have actual knowledge that the subcontractor
had not been paid when the bank made the payment. Under the minority rule, the bank’s mortgage would
not have priority because the lien was properly filed before the bank made this payment.

The general rule is that, if a future advance is obligatory, the mortgage securing that advance takes
priority over creditors who file liens after the mortgage is recorded, even if all advances on the loan have
not actually been made. Because the bank must make the future payments, regardless of circumstances,
those payments are deemed to have been made when the mortgage was created. It is as though the loan
had been paid in one lump sum at that time.

[NOTE: If an examinee concludes that future advances are obligatory, that examinee should also
conclude that the mechanic’s lien is junior to all disbursements made by the bank.]

If the future advances are optional, and if the mortgagee has notice when it makes the advance that a
subsequent lienor has acquired an interest in the land, then the advance loses its priority to that creditor.
Here, the bank has priority only to the original $50,000 advance but not to the $40,000 advanced after the
mechanic’s lien was filed, assuming that the bank had notice of it. And, of course, if the bank were to
continue to make advances up to the $200,000 commitment, those additional payments would not have
priority. However, this rule applies only if the bank made the advances with “notice” of the lien.

Most states hold that the mortgagee has notice only when it has actual notice of the lien at the time of the
future advance. This rule is based on the view that the mortgagee should not have the burden of a title
search each time it makes an advance when it is easier for the subcontractor to inform the mortgagee that
its bill has not been paid.

Here, application of this approach would lead to the conclusion that the bank had no notice of the
mechanic’s lien until it learned that the subcontractor had not been paid when the third advance was
sought. Thus, the bank would have priority over the subcontractor with respect to the first two payments
($50,000 and $40,000). Of course, should the bank make additional future advances, as to those it would
not have priority.

Under the minority view, the bank will be charged with constructive notice of the mechanic’s lien if the
lien, as here, has been properly recorded. Under this view, the assessment of the relative burdens of the
mortgagee and the subcontractor are reversed. If the minority view applies, the bank has priority with
respect to the original advance of $50,000 but not to any later advances because it had constructive notice
of the mechanic’s lien as of the time it was filed.

Seperac-J19 Exam-Released MEE Essay Compilation © 2016-2020 439


#098-FEB 2018–MEE Q04: QUESTION FOUR (REAL PROPERTY)

A developer acquired a 30-acre tract of land zoned for residential use. The developer thereafter marked
out 60 building lots. The developer granted various utility providers appropriate easements to install
underground sewer and utility lines. These utility easements were promptly and properly recorded.

Subsequently, the developer contracted with a man to build a home for the man on one of the 60 lots. The
contract provided that, at closing, the developer would convey the home and lot to the man by a warranty
deed excepting all easements and covenants of record. The home was completed nine months later.

At the closing, the developer conveyed the home and lot to the man by a valid warranty deed containing
the six title covenants. Notwithstanding the language in the contract, the deed contained no exceptions to
these six covenants. The deed was promptly and properly recorded.

Two months later, following a heavy storm, the man discovered rainwater in the basement level of his
home. Three bedrooms were located on this level, and the influx of rainwater made all of them unusable.
An expert determined that the cause of the rainwater influx was a defect in the construction of the home's
foundation. The man contacted the developer, who denied any responsibility for the influx. Rather than
argue with the developer, the man contacted a plumber, who concluded that the problem could be solved
by installing a sump pump in the basement. The plumber accurately told the man that the usual cost of
installing a sump pump was $750, but that the location of the sewer lines coming into the home created
more work, raising the installation cost to $1,500. The man told the plumber to install the pump.

Thereafter, the man sued the developer for $5,000 in damages for the cost of the sump pump, its
installation, and damage to the floors and carpeting in the basement. He also sought additional damages
for breach of one or more title covenants.

1. Which present title covenants, if any, did the developer breach with respect to the utility
easements? Explain.

2. Assuming that there was a breach of one or more of the present title covenants, can the man
recover damages from the developer for the breach? Explain.

3. May the man force the utility company that installed the underground sewer lines to remove
them from the land? Explain.

4. May the man recover the $5,000 in damages from the developer? Explain.

Seperac-J19 Exam-Released MEE Essay Compilation © 2016-2020 440


#098: F18-4 MEE: ANSWER: NCBE (REAL PROPERTY)

POINT (1) [35%] ISSUE: Which present title covenants, if any, did the developer breach?
ANSWER: The developer breached the covenant against encumbrances.

POINT (2) [25%] ISSUE: Is the man entitled to damages for breach of the covenant against
encumbrances because of the existence of the utility easement, which was plain and obvious?
ANSWER: No. The law is unclear whether the man can recover damages for breach of the
covenant against encumbrances where, as here, the easement was plain, obvious, or known to the
man.

POINT (3) [15%] ISSUE: Can the man force the utility company to remove the sewer lines from the
land? ANSWER: No. The man cannot compel the utility company to remove the sewer lines from
the land.

POINT (4) [25%] ISSUE: Is the man entitled to recover $5,000 from the developer for water
damage to his home? ANSWER: Yes. The man can recover damages from the developer for breach
of the implied warranty that applies to new home construction.

ANSWER DISCUSSION:

Of the three present title covenants, the developer breached the covenant against encumbrances by giving
the man a warranty deed that contained no exceptions. However, because the utility easements were plain,
obvious, or well known to the man, the man should not be able to obtain damages for breach of that
covenant. Even if the man is not charged with actual notice of the utility easements, because the utility
easements were promptly and properly recorded prior to the man’s purchase of the home, the man had
constructive (record) notice of them; thus, he cannot compel the removal of the sewer lines from the land.
Finally, the man can recover damages from the developer for breach of the implied warranty of
habitability.

ANSWER EXPLANATION:

Explanation to Point-One (35%):

The developer breached the covenant against encumbrances.

A warranty deed contains six title covenants, three present covenants and three future covenants. The
three present covenants are

1. covenant of seisin, a covenant that the grantor owns the land that the deed purports to convey to
the grantee;

2. covenant of right to convey, a covenant that the grantor has a right to convey the land; and

3. covenant against encumbrances, a covenant that there is no outstanding right or interest in a third
party which does not totally negate the title the grantor purports to convey.

[NOTE: A warranty deed also contains three future covenants. These are (1) the covenant of warranty,
(2) the covenant of quiet enjoyment, and (3) the covenant of further assurances. The calls of the question,

Seperac-J19 Exam-Released MEE Essay Compilation © 2016-2020 441


however, only relate to the present covenants, and examinees should receive no credit for addressing the
future covenants.]

These warranties of title apply to all easements on the land except to the extent that they have been
excepted by the terms of the deed. The fact that the contract between the developer and the man provided
for a warranty deed with easements excepted is irrelevant to whether a breach has occurred because, as a
result of the so-called merger doctrine, contractual promises relating to title do not survive the closing and
the delivery of the deed. In other words, the only promises relating to title that survive the closing are
those in the deed.

Here the deed warranted that there were no encumbrances on the land. An encumbrance is “some
outstanding right or interest in a third party which does not totally negate the title which the deed purports
to convey.” An easement is such an encumbrance. Thus, the covenant against encumbrances was
breached by the existence of the utility easements on the land.

The developer may claim that the omission from the deed of the exception in the contract for easements
and covenants of record was a mutual mistake, perhaps due to a scrivener’s error. Modern courts often
apply merger only when use of the doctrine carries out the parties’ probable intent. If a court accepts this
argument, there would be no breach of any title covenant, in effect resulting in a reformation of the deed
to conform to the parties’ intent.

[NOTE: There is plainly no breach of the other two present covenants. The man received title to the land
along with possession, so there is no breach of the covenant of seisin. Nor is there a breach of the
covenant of the right to convey. Nothing indicates that the developer lacked the right to convey title to the
man.]

Explanation to Point-Two (25%):

The law is unclear whether the man can recover damages for breach of the covenant against
encumbrances where, as here, the easement was plain, obvious, or known to the man.

If a court determines that the covenant against encumbrances was breached, it is unclear if the man can
recover damages. Courts are divided on the propriety of damages where the easement is plain or obvious
or is known to the plaintiff.

The man might argue that, because the developer contracted to exclude the easement from the covenant
but then granted the covenant without exclusion in the deed, the developer had knowledge of a change in
circumstances that no longer made it necessary to preserve in the deed the exception stated in the contract
and therefore willingly gave the warranty.

The developer would likely argue that the man had notice of the easements. Although the underground
utilities were not visible, the man had constructive notice of the easements given that they had been
recorded, and furthermore, their presence would also have been established through a home inspection
prior to the closing, and this would constitute actual notice of their existence. The developer might also
argue that, because utilities are essential to use of any house as a typical home, the man must have wanted
the benefit of the easements.

Here, the developer has the stronger arguments; the man did have constructive notice of the easements
(and possibly actual notice). It would also be unreasonable for a home buyer to recover damages for
breach of a warranty for an easement the buyer would have wanted the benefit for the home. By denying

Seperac-J19 Exam-Released MEE Essay Compilation © 2016-2020 442


damages to the man, a court would, effectively, recognize that the failure to exclude the easements from
the warranty in the deed was a mistake and that damages would provide the man with a windfall.

Even if the court were to find that the man is entitled to damages, he could recover them only to the extent
that he could prove the easements reduced the value of the land. This measure is the value of the land not
subject to the easements, minus the value of the land as restricted by the easements. Because utility
easements are so commonly used in residential building subdivisions, it is possible that the man will not
be able to prove any significant decline in value. This may depend on whether the easements are standard
and are located along the perimeter of the lot, so as not to interfere with reasonable present or future
residential uses of the land.

Explanation to Point-Three (15%):

The man cannot compel the utility company to remove the sewer lines from the land.

Resolution of a dispute between the man and the holder of the sewer line easement is not dependent on or
controlled by the fact that the man received a warranty deed from the developer. Rather, the question is
whether the man had knowledge of the easements when he purchased the land. The facts state that the
utility easements had been promptly and properly recorded prior to the delivery of the deed to the man.
Therefore, regardless of the type of notice statute the jurisdiction has (notice, race-notice, or race), the
man, even if he had no actual notice of the easements, was on record (or constructive) notice of the
easements. Thus, he takes subject to the easements, and he cannot force the utility company to remove the
sewer lines from the land.

Explanation to Point-Four (25%):

The man can recover damages from the developer for breach of the implied warranty that applies to new
home construction.

At common law, the rule of caveat emptor applied, and as a result the seller did not make any implied
promises to the buyer relating to the condition of the premises. Today, it is generally true that a builder of
a new home impliedly warrants to the buyer that the home is habitable and fit for its intended purposes.
This implied warranty allows a buyer to recover damages for losses resulting from defective construction
or construction that was not done in a workmanlike manner.

The warranty applies to defects that are discovered within a reasonable period of time, are due to the
builder’s negligence or failure to do the work in a workmanlike manner, and cannot be attributable to later
changes in the structure or to normal deterioration. Courts vary in characterizing the warranty as based in
contract or in tort law.

Here, the defect in the foundation clearly breaches the warranty. Thus, the man should be able to recover
all damages for losses resulting from the developer’s breach of the warranty.

Seperac-J19 Exam-Released MEE Essay Compilation © 2016-2020 443


#099-FEB 2017–MEE Q06: QUESTION SIX (REAL PROPERTY)

On January 1, 2015, a landlord who owned a multi-unit apartment building consisting only of one-
bedroom apartments leased an apartment in the building to a tenant for a two-year term ending on
December 31, 2016, at a monthly rent of $2,000. The tenant immediately took possession of the
apartment.

The lease contained the following provision:

Tenant shall not assign this lease without the Landlord's written consent. An assignment without
such consent shall be void and, at the option of the Landlord, the Landlord may terminate the lease.

On May 1, 2015, the tenant learned that her employer was transferring her to a job overseas to begin on
August 1, 2015. On May 2, the tenant emailed the landlord that she needed to vacate the apartment on
August 1, but that she had found a well-to-do and well-respected lawyer in the community who was
willing to take over the balance of the lease term at the same rent. The landlord immediately emailed the
tenant that he would not consent to the lawyer taking over the lease. He wrote, "I don't rent to lawyers
because I've learned from personal experiences with them as tenants that they argue about everything,
make unreasonable demands, and make my life miserable. Find somebody else."

On July 25, 2015, the tenant vacated the apartment and removed all her personal property from it. She left
the apartment keys in an envelope in the landlord's mail slot. The envelope also contained a note in which
the tenant wrote, "As you know, I am moving overseas and won't be back before my lease ends. So here
are the keys. I won't pay you any rent from August 1 on."

On July 26, 2015, the landlord sent the tenant an email acknowledging that he had found the keys and the
note. In that email, the landlord wrote: "Although this is a problem you created, I want to be a nice guy
and help you out. I feel pretty confident that I can find a suitable tenant who is not a lawyer to rent your
apartment."

As of August 1, 2015, the landlord had four apartments, including the tenant's apartment, for rent in the
building. The landlord put an "Apartments for Rent" sign in front of the apartment building and placed
advertisements in the newspaper and on a website listing all the apartments for rent. However, because of
a recent precipitous decline in the local residential rental property market, the landlord listed the
apartments for a monthly rent of $1,000. The landlord showed all four vacant apartments, including the
tenant's apartment, to each prospective tenant.

By September 1, 2015, the landlord was able to rent only two of the apartments at $1,000. The landlord
was unable to rent the two remaining apartments, including the tenant's, at any price throughout the rest of
2015 and all of 2016, notwithstanding his continued efforts to rent them.

On January 2, 2017, the landlord sued the tenant to recover 17 months of unpaid rent, covering the period
August 1, 2015, through December 31, 2016.

Identify and evaluate the arguments available to the landlord and the tenant regarding the landlord's claim
to 17 months of unpaid rent.

Seperac-J19 Exam-Released MEE Essay Compilation © 2016-2020 444


#099: F17-6 MEE: ANSWER: NCBE (REAL PROPERTY)

POINT (1)(a) [20%] ISSUE: Can the landlord argue that he lawfully withheld his consent to the
assignment of the lease? ANSWER: Yes. The landlord will argue that the lease provision
prohibiting assignment without his written consent was valid and that he could withhold consent
for any reason or for no reason at all. The landlord will also argue that, even if this jurisdiction’s
law requires that his refusal to accept the assignment be reasonable, his decision to withhold
consent here was reasonable, given his past experiences with lawyers as tenants.

POINT (1)(b) [15%] ISSUE: Can the landlord argue that he did not accept the tenant’s surrender
of the apartment? ANSWER: Yes. The landlord will argue that the tenant’s actions (removing her
possessions and leaving the keys in the landlord’s mail slot with a note) constituted an abandonment
of the apartment without cause. Further, the landlord will argue that he did not accept a surrender
of the apartment.

POINT (1)(c) [15%] ISSUE: Can the landlord argue that he had no duty to mitigate, but even if he
did have a duty to mitigate, he had fulfilled that duty? ANSWER: Yes. The landlord will argue
alternatively that he had no duty to mitigate or, if he did, that he discharged it. Either way, he will
claim that he can recover the 17 months of unpaid rent.

POINT (2)(a) [20%] ISSUE: Can the tenant argue that the landlord unreasonably refused to
consent to the assignment of the lease? ANSWER: Yes. The tenant will argue that although the
landlord’s consent to an assignment was required, withholding consent must be reasonable, and
here the landlord did not act reasonably.

POINT (2)(b) [15%] ISSUE: Can the tenant argue that the landlord accepted her surrender of the
apartment and therefore is not entitled to unpaid rent? ANSWER: Yes. The tenant will argue that
the landlord accepted the surrender of the apartment, thus ending her obligation to pay rent.

POINT (2)(c) [15%] ISSUE: Can the tenant argue that even if the surrender was not accepted, the
landlord had a duty to mitigate and failed to do so? ANSWER: Yes. The tenant will argue that
even if the landlord did not accept the surrender, he still had a duty to mitigate and failed to do so.

ANSWER DISCUSSION:

The landlord will argue that the lease provision restricting assignment without his consent was valid and
that he could lawfully refuse to accept the tenant’s proposed assignee for any reason or no reason at all.
The landlord will also argue that, even if this jurisdiction requires that his refusal to consent had to be
reasonable, it was reasonable in light of his prior experiences with lawyers as tenants. He will also argue
that he did not accept the tenant’s surrender of the apartment and that he had no duty to mitigate. The
landlord will argue that even if he did have a duty to mitigate, he fulfilled that duty by trying (albeit
unsuccessfully) to re-let the apartment. The landlord will claim that there was no market for the tenant’s
apartment and that he is thus entitled to 17 months of unpaid rent. The tenant should concede that,
because the restraint on assignment was valid, her purported assignment of the lease required the
landlord’s written consent. However, the tenant will argue that the landlord had to be reasonable if he was
going to withhold his consent to an assignment, and that he was unreasonable in refusing to rent to
lawyers. The tenant will argue that the landlord accepted her surrender and therefore is not entitled to sue
for any unpaid rent. Alternatively, she will argue that even if the landlord did not accept the surrender, he

Seperac-J19 Exam-Released MEE Essay Compilation © 2016-2020 445


had a duty to mitigate and he failed to do so, and thus is not entitled to any unpaid rent. On the whole, the
landlord here has the better arguments and should be entitled to recover 17 months of unpaid rent from the
tenant. [NOTE: The call of this question requires examinees to construct arguments relating to the
following issues: (1) Did the landlord properly withhold consent to an assignment? (2) Did the landlord
accept the tenant’s surrender of the apartment? (3) Did the landlord have a duty to mitigate? and (4) If the
landlord had a duty to mitigate, did he fulfill that duty? The question tests examinees’ ability to identify
the issues regarding consent to the assignment, surrender, and mitigation, and to marshal facts in support
of each party’s arguments.]

ANSWER EXPLANATION:

Explanation to Point-One(a) (20%):

The landlord will argue that the lease provision prohibiting assignment without his written consent was
valid and that he could withhold consent for any reason or for no reason at all. The landlord will also
argue that, even if this jurisdiction’s law requires that his refusal to accept the assignment be reasonable,
his decision to withhold consent here was reasonable, given his past experiences with lawyers as tenants.

Here, the tenant attempted to assign her lease – that is, to transfer the balance of the lease term – to a
lawyer. A restriction on assignment is a valid restraint on alienation. Because a restraint on transfer by
assignment is valid, the lease provision here prohibiting an assignment without the landlord’s consent was
valid.

The landlord will argue that he was free to withhold his consent for any reason or for no reason at all. The
clause in the tenant’s lease is commonly described as a “silent” consent clause because it does not include
an express standard or condition for the giving or withholding of consent. Under the traditional rule – still
the majority rule today – a silent consent clause gives the landlord the right to withhold consent for any
reason or for no reason – even if the withholding of consent is arbitrary and unreasonable. (That rule,
however, is subject to statutory housing discrimination laws that are not at issue here.)

Alternatively, the landlord will argue that, even if this jurisdiction is one of the minority group in which
refusal to consent to an assignment must be reasonable, his refusal was reasonable in light of his personal
experiences renting to lawyers; his refusal represented a legitimate business judgment.

[NOTE: Some examinees may conclude that the lease did not prohibit a sublet. But the tenant informed
the landlord that the lawyer “was willing to take over the balance of the tenant’s lease term.” This
statement evidences that the tenant was purporting to assign the lease and not sublet. This transfer should
be construed as an assignment, the making of which was restricted under the lease. Therefore, the tenant
would not have the right to transfer the lease to the lawyer without the landlord’s consent.]

Explanation to Point-One(b) (15%):

The landlord will argue that the tenant’s actions (removing her possessions and leaving the keys in the
landlord’s mail slot with a note) constituted an abandonment of the apartment without cause. Further, the
landlord will argue that he did not accept a surrender of the apartment.

Abandonment occurs when a tenant vacates the leased premises before the end of the term, has no intent
to return, and defaults in the payment of rent. Under traditional common law principles, a landlord has
three options when a tenant abandons the premises: (1) accept a surrender of the premises, thereby
extinguishing the tenant’s duty to pay rent due after the acceptance of surrender; (2) re-let or attempt to

Seperac-J19 Exam-Released MEE Essay Compilation © 2016-2020 446


re-let the premises on the tenant’s behalf, and recover from the tenant damages based on the difference
between what the tenant owed for rent and what the landlord collected from re-letting; or (3) leave the
premises vacant and sue the tenant for unpaid rent as it accrues.

Here, on July 25, 2015, the tenant vacated the apartment and left the keys in an envelope in the landlord’s
mail slot with a note stating that she was moving abroad, would not return before the end of the lease, and
would not pay any rent from August 1 onward. The landlord emailed the tenant the next day and
acknowledged that he had found the keys and the note. “Although this is a problem you created,” he
wrote, “I want to be a nice guy and help you out. I feel pretty confident that I can find a suitable tenant
who is not a lawyer to rent your apartment.”

The landlord will argue that he did not voluntarily accept the keys; they were merely placed in his mail
slot and the statement in his email was merely an offer to attempt to re-let the premises on the tenant’s
behalf. His email emphasized, “this is a problem you created,” which implied that it was the tenant’s
problem to resolve. This interpretation is bolstered by the landlord’s statement of his willingness to “help
the tenant out.” Nowhere in the email did the landlord state that he was releasing the tenant from her
obligations under the lease.

Furthermore, the facts state that local residential rents had declined precipitously since early July, and it is
unlikely that the landlord would voluntarily suffer any financial loss resulting from a re-let at a reduced
rent.

Explanation to Point-One(c) (15%):

The landlord will argue alternatively that he had no duty to mitigate or, if he did, that he discharged it.
Either way, he will claim that he can recover the 17 months of unpaid rent.

In a jurisdiction following the common-law no-mitigation rule, the landlord will argue that he had no duty
to mitigate and that, because he did not accept the tenant’s surrender, he is entitled to the 17 months of
unpaid rent. In a jurisdiction that has enacted a statute requiring the landlord to mitigate, he will argue that
that duty was satisfied. He advertised available apartments with a sign in front of the building, in a
newspaper, and on a website, and that should be sufficient mitigation. The landlord also showed the
tenant’s apartment along with the other vacant apartments to prospective tenants, thereby providing a fair
chance that the tenant’s apartment would be rented before other apartments in the landlord’s rental pool.
On these facts, a court most likely would find that the landlord made reasonable efforts to mitigate.

Of those states requiring mitigation, none requires that mitigation be successful; in fact, if a court
determined that the landlord made reasonable efforts to mitigate but was unsuccessful, the court would
find that the landlord would be entitled to damages equal to the difference between the tenant’s promised
rent ($2,000/month) and the apartment’s fair rental value. Given that the landlord took reasonable steps to
mitigate and was unable to rent the tenant’s unit “at any price,” it appears that the fair rental value of the
tenant’s unit is $0. Thus, the landlord’s position would be that he is entitled to $34,000 in damages
($2,000/month for 17 months). In other words, if the landlord did not accept the surrender, then regardless
of whether the landlord had a duty to mitigate, the landlord is entitled to the 17 months of unpaid rent.

Explanation to Point-Two(a) (20%):

The tenant will argue that although the landlord’s consent to an assignment was required, withholding
consent must be reasonable, and here the landlord did not act reasonably.

Seperac-J19 Exam-Released MEE Essay Compilation © 2016-2020 447


The tenant should concede that the lease required the landlord’s consent to an assignment. However, she
will argue that the law requires the landlord to act reasonably in rejecting the proposed assignment and
here the landlord was unreasonable. There is an emerging modern trend that a landlord’s consent not be
unreasonably withheld, which is based upon the principle that leases are subject to the good faith
requirements of contracts in general. Thus, a minority of courts require that a landlord have a reasonable
basis for withholding consent to a proposed transfer.

Factors that may be considered under a reasonableness test include the proposed assignee’s financial
ability to pay, the suitability of the premises for the proposed assignee’s use, and the need for alterations
to accommodate the proposed assignee’s use. It is not commercially reasonable to deny consent solely on
the basis of personal taste, convenience, or sensibility. Here, the facts suggest that the lawyer had the
financial ability to pay the rent, and there is no indication that the lawyer intended to use the apartment for
anything other than a residence. The tenant will also argue that the landlord’s stated objection – “I’ve
learned from personal experiences with lawyers as tenants that they argue about everything, make
unreasonable demands, and make my life miserable” – is merely a matter of personal taste or
convenience, and thus the landlord’s refusal to consent was unreasonable.

Explanation to Point-Two(b) (15%):

The tenant will argue that the landlord accepted the surrender of the apartment, thus ending her obligation
to pay rent.

The tenant will argue that the landlord accepted her surrender of the premises when he accepted the keys
and emailed her, stating, “Although this is a problem you created, I want to be a nice guy and help you
out. I feel pretty confident that I can find a suitable tenant who is not a lawyer to rent your apartment.”
Those words could be construed as a willingness to help the tenant by releasing her from her obligations
under the lease. If the lease was terminated by the landlord’s acceptance of the surrender, the landlord had
no right to any rent after he accepted the keys and sent that email on July 26, 2015.

Explanation to Point-Two(c) (15%):

The tenant will argue that even if the landlord did not accept the surrender, he still had a duty to mitigate
and failed to do so.

Alternatively, the tenant will argue that even if the landlord did not accept the surrender, he still had a
duty to mitigate damages and that he failed to fulfill that duty. While most jurisdictions today reject the
common-law rule that a landlord has no duty to mitigate, there is little guidance as to what efforts are
sufficient to satisfy the mitigation requirement. Some statutes have defined reasonable efforts as “steps
which the landlord would have taken to rent the premises if they had been vacated in due course, provided
that those steps are in accordance with local rental practice for similar properties.” The tenant might argue
that the landlord should have shown the tenant’s apartment to the exclusion of other vacant apartments in
the landlord’s rental pool. She might also argue that there was some other deficiency in the landlord’s
efforts. Her arguments on this point are weak.

Lastly, the tenant will claim that because the landlord’s efforts to mitigate were insufficient, his claim that
he could not rent the tenant’s apartment at any price is irrelevant. The tenant will further argue that the
landlord is not entitled to 17 months of unpaid rent ($34,000) under the lease but would only be entitled to
the difference between the rent owed under the lease and the fair rental value (i.e., the amount of rent the
landlord could have recovered if he had made reasonable attempts to secure a substitute tenant). The fair
rental value is presumably $1,000 a month in light of the fact that he was able to rent two of the other

Seperac-J19 Exam-Released MEE Essay Compilation © 2016-2020 448


apartments for that amount. That would reduce the landlord’s damages to $17,000 (unpaid rent of $34,000
for the 17 months less $17,000, the fair rental value of the apartment over the 17-month period).

In some jurisdictions, a landlord’s failure to mitigate would relieve the tenant of any liability for rent or
damages after the date of abandonment because by failing to mitigate, the landlord is deemed to have
accepted the surrender. In those jurisdictions, the landlord would be entitled to no damages.

[NOTE: At common law if a tenant abandoned his or her apartment and the landlord did not accept the
surrender, the landlord had no duty to mitigate damages. However, the landlord could sue the tenant only
for past-due rents and, because rent was not due until the payment date, the landlord could not sue for
future rents. There is an emerging doctrine applying contract principles to a lease that states that even if
the landlord accepts surrender, the landlord is entitled to damages as a result of a tenant’s anticipatory
repudiation of the lease. Under this theory, a tenant would be liable for the difference, if any, between the
rent due under the lease and the fair rental value of the premises plus the landlord’s consequential
damages of advertising the unit for rent and the costs associated with showing it to prospective tenants.
However, that possibility is written out of this question. Of course, if the duty to mitigate applies, contract
damages rules apply.]

Seperac-J19 Exam-Released MEE Essay Compilation © 2016-2020 449


#100-FEB 2015–MEE Q04: QUESTION FOUR (REAL PROPERTY)

Seventeen years ago, a property owner granted a sewer-line easement to a private sewer company. The
easement allowed the company to build, maintain, and use an underground sewer line in a designated
sector of the owner’s three-acre tract. The easement was properly recorded with the local registrar of
deeds.

Fifteen years ago, a man having no title or other interest in the owner’s three-acre tract wrongfully entered
the tract, built a cabin, and planted a vegetable garden. The garden was directly over the sewer line
constructed pursuant to the easement the owner had granted to the sewer company. The cabin and garden
occupied half an acre of the three-acre tract. The man moved into the cabin immediately after its
completion and remained in continuous and exclusive possession of the cabin and garden until his death.
However, he did not use the remaining two and one-half acres of the three-acre tract in any way.

Eight years ago, the man died. Under the man’s duly probated will, he bequeathed to his sister “all real
property in which I have or may have an interest at the time of my death.” The man’s sister took
possession of the cabin and garden immediately after the man’s death and remained in exclusive and
continuous possession of them for one year, but she, too, did not use the remaining two and one-half acres
of the tract.

Seven years ago, the man’s sister executed and delivered to a buyer a general warranty deed stating that it
conveyed the entire three-acre tract to the buyer. The deed contained all six title covenants. Since this
transaction, the buyer has continuously occupied the cabin and garden but has not used the remaining two
and one-half acres.

A state statute provides that “any action to recover the possession of real property must be brought within
10 years after the cause of action accrues.”

Last month, the property owner sued the buyer to recover possession of the three-acre tract.

1. Did the buyer acquire title to the three-acre tract or any portion of it? Explain.

2. Assuming that the buyer did not acquire title to the entire three-acre tract, can the buyer recover
damages from the sister who sold him the three-acre tract? Explain.

3. Assuming that the buyer acquired title to the entire three-acre tract or the portion above the
sewer-line easement, can the buyer compel the sewer company to remove the sewer line under the
garden? Explain.

Seperac-J19 Exam-Released MEE Essay Compilation © 2016-2020 450


#100: F15-4 MEE: ANSWER: NCBE (REAL PROPERTY)

POINT (1)(a) [35%] ISSUE: Were the acts of possession by the man, his sister, and the buyer
sufficient to acquire a title by adverse possession? ANSWER: Yes. The acts of possession of the
man, his sister, and the buyer were sufficient to acquire title by adverse possession to the one-half
acre actually possessed by them.

POINT (1)(b) [20%] ISSUE: Can the periods of possession during which the man, his sister, and the
buyer occupied the cabin and garden be aggregated for the purpose of satisfying the statutory
possession requirement? ANSWER: Yes. Although neither the man, nor his sister, nor the buyer
individually possessed the property for the statutory 10-year period, their periods of possession can
be aggregated because they were all in privity with one another. Thus, the 10-year statute has run,
and the buyer has acquired title to the one-half acre.

POINT (1)(c) [15%] ISSUE: If the acts of possession were sufficient to acquire a title by adverse
possession, did the adverse possession claim extend to the entire three-acre tract or only to the
portion of land on which the cabin and garden were located? ANSWER: The buyer did not acquire
title to the unpossessed two and one-half acres because he did not possess or use that portion of the
tract.

POINT (2) [15%] ISSUE: Can the buyer recover damages from the man’s sister, who gave a
warranty deed for the three-acre tract? ANSWER: Yes. The buyer is entitled to damages from the
sister because the sister did not convey title to the three-acre tract by a general warranty deed.

POINT (3) [15%] ISSUE: Did the buyer take subject to an underground sewer-line easement
created before the adverse possession began? ANSWER: Yes. The buyer cannot compel the
company to remove the sewer line from under the garden because he took subject to the sewer-line
easement and probably did not interfere with that easement.

ANSWER DISCUSSION:

The possessory acts of the man, his sister, and the buyer were sufficient to acquire title by adverse
possession as to the half acre of the three-acre tract on which the cabin and garden are located because
they were (1) actual, (2) open and notorious, (3) exclusive, (4) continuous, and (5) hostile and under claim
of right. Because the man, his sister, and the buyer were in privity with one another, their aggregate time
of possession determines whether the 10-year statute of limitations has run. Here, the aggregate time of
possession exceeds 10 years. Thus, the buyer acquired title to this one-half acre by adverse possession.
However, because there were no possessory acts with respect to the remaining two and one-half acres, the
buyer did not acquire title by adverse possession to the entire three-acre tract. Because the buyer did not
acquire title to the entire three-acre tract, the buyer can successfully sue the sister for damages because the
sister purported to convey the entire three-acre tract by warranty deed and, under the warranty deed, she
covenanted that she had good title to the entire three-acre tract. The adverse possessors took subject to the
sewer-line easement, as their acts of possession probably did not interfere with that easement or give the
sewer company a cause of action against them. Thus, the buyer cannot compel the company to remove the
sewer line.

ANSWER EXPLANATION:

Seperac-J19 Exam-Released MEE Essay Compilation © 2016-2020 451


Explanation to Point-One(a) (35%):

The acts of possession of the man, his sister, and the buyer were sufficient to acquire title by adverse
possession to the one-half acre actually possessed by them.

To acquire title by adverse possession, the possession must be (1) actual, (2) open and notorious, (3)
exclusive, (4) continuous, and (5) hostile and under claim of right. Here, all of these requirements were
satisfied as to the one-half acre on which the cabin was built and the garden planted. The facts state that
the possession of the man, his sister, and the buyer was exclusive and continuous, satisfying these two
requirements of the test.

To be actual, acts of possession must be consistent with how a reasonable owner of land would have used
it if in possession. Here, the acts of possession included building and occupying a cabin as well as
planting, harvesting, and maintaining a garden. These acts are consistent with how a reasonable owner
would have used the one-half acre.

To be open and notorious, the acts of possession must be such that they would have put an owner on
notice of the adverse possession had the owner inspected the land. Here, the cabin and garden occupied a
half acre and were visible. When the owner acquired the land, it was vacant. Had the owner inspected, he
would have determined that someone else was in possession.

Most courts and scholars agree that hostility and claim of right are present when a possessor is on the land
without the owner’s permission. Some courts do hold that to acquire title by adverse possession, the
possessor must have a good-faith belief that she has a good title to the land; others hold that the possessor
must believe that she does not have a good title to the land. But most courts and scholars reject these
contradictory “subjective hostility” tests. Thus, in the vast majority of jurisdictions, the fact that the man,
his sister, and the buyer were on the tract of land without the permission of the owner would suffice to
satisfy the hostility and claim of right requirement.

Thus, the man, his sister, and the buyer satisfied the requirements for acquiring title by adverse possession
as to the one-half-acre portion of the three-acre tract that they actually possessed.

Explanation to Point-One(b) (20%):

Although neither the man, nor his sister, nor the buyer individually possessed the property for the
statutory 10-year period, their periods of possession can be aggregated because they were all in privity
with one another. Thus, the 10-year statute has run, and the buyer has acquired title to the one-half acre.

The period during which possession must endure to create title by adverse possession is determined by
statute. Here, the local statute provides that “any action to recover the possession of real property must be
brought within 10 years after the cause of action accrues.”

A cause of action to recover possession of real property “accrues” when a wrongful act of possession
occurs. Here, the initial cause of action thus accrued when the man wrongfully entered a portion of the
three-acre tract 15 years ago.

Here, the man possessed the property for seven years, the man’s sister possessed it for one year, and the
buyer possessed it for seven years. Although none of these individual periods of possession equals the 10-
year statutory period, when multiple adverse possessors are in “privity” with one another, the period of
their respective possessions can be aggregated for the purpose of determining whether the statutory period

Seperac-J19 Exam-Released MEE Essay Compilation © 2016-2020 452


(here, 10 years) has run against the holder of the cause of action. In this context, privity denotes a
relationship between possessors arising because of a voluntary transfer between them, descent under the
laws of intestacy, or testamentary succession as the result of a bequest. Here, the man and his sister were
in privity as a result of testamentary succession, namely the bequest in the man’s will of all real property
“in which I have or may have an interest” to his sister. The sister and the buyer were also in privity
because of the voluntary transfer between them.

Thus, because the 10-year statutory period has elapsed, the buyer has acquired title by adverse possession
to the one-half-acre portion of the three-acre tract that he, the sister, and the man actually possessed.

Explanation to Point-One(c) (15%):

The buyer did not acquire title to the unpossessed two and one-half acres because he did not possess or
use that portion of the tract.

The buyer acquired title by adverse possession only to the portion of the tract for which he met all
requirements of the five-prong test. Because the man, his sister, and the buyer never possessed (or even
used) any of the two and one-half acres beyond the garden, the buyer cannot claim title by adverse
possession to those acres.

The doctrine of constructive adverse possession does not alter this result. Under this doctrine, if a
possessor enters under color of title (i.e., an instrument creating the possibility of a title in the grantee who
enters under the instrument) and the possessor takes possession of only a portion of the land described in
the instrument, the possessor’s possession is deemed to constructively extend to the portion of the
described land. Here, neither the man nor his sister entered under color of title. Although the buyer did
enter with a deed and, arguably, color of title, his constructive possession endured only seven years, short
of the statutory period in which the legal title holder may regain possession.

Explanation to Point-Two (15%):

The buyer is entitled to damages from the sister because the sister did not convey title to the three-acre
tract by a general warranty deed.

A warranty deed includes numerous covenants. Two of them – the covenant of seisin and the right to
convey – are essentially the same, and they guarantee that the seller owns the conveyed land. Here, the
sister did not own the three-acre tract when she purported to convey it to the buyer by a general warranty
deed. Thus, she was in breach of the covenant of seisin and the right to convey, and the buyer is entitled to
damages for that breach.

[NOTE: Some examinees may confuse the warranty issue with the concept of marketable title. It is true
that the man’s sister did not have a marketable title when she conveyed to the buyer because her adverse
possession claim was clearly subject to the risk of litigation. Nonetheless the buyer agreed to go forward
with the transfer, and the sister gave the buyer a warranty deed. Had she given the buyer a quitclaim
deed, no warranties would have been breached.

While the deed also includes a covenant of quiet enjoyment and a covenant of warranty, no facts suggest
that these have been breached here, as the buyer has not been evicted.]>

Explanation to Point-Three (15%):

Seperac-J19 Exam-Released MEE Essay Compilation © 2016-2020 453


The buyer cannot compel the company to remove the sewer line from under the garden because he took
subject to the sewer-line easement and probably did not interfere with that easement.

Where an adverse possessor acquires title by adverse possession, the nature of the acquired title is no
greater than the title of the holder of the cause of action who was barred by the running of the statute of
limitations. Here, the owner’s title was subject to the properly recorded sewer-line easement at the time
the man wrongfully entered the land.

The man, his sister, and the buyer cannot claim to have adversely possessed the easement unless their
possession interfered with the rights of the sewer company, giving it a cause of action against the man, the
sister, and the buyer while they were in possession. There is nothing in the facts, however, suggesting that
planting and maintaining a garden interfered with the sewer company’s access to the sewer line. In the
absence of such interference, the company has no cause of action against the possessors, in which case the
buyer acquired the owner’s title only – a title subject to the sewer-line easement.

Examinees who make a plausible argument that possession of the garden did interfere with the easement
should conclude that the buyer could compel the sewer company to remove the sewer line. In that case, its
failure to do so within the 10-year statutory period would result in the buyer acquiring a title that is
superior to both the owner and the sewer company.

Seperac-J19 Exam-Released MEE Essay Compilation © 2016-2020 454


#101-JUL 2013–MEE Q08: QUESTION EIGHT (REAL PROPERTY)

Two years ago, a builder constructed a house for a woman and conveyed that house to her for $300,000 at
the closing by a warranty deed, which was promptly recorded. The sale contract contained no express
warranties relating to the condition of the house. To finance the purchase, the woman borrowed $200,000
from a local bank secured by a mortgage on her new house. The mortgage note provided that in the event
of the woman’s failure to make two consecutive monthly mortgage payments, the balance would become
immediately due and payable. The mortgage was promptly recorded.

One year ago, the woman accepted a new job and moved. At that time, her house was worth $360,000 and
there was a balance on the mortgage of $195,000. She sold the house to a man and delivered a quitclaim
deed to him in exchange for $160,000. The quitclaim deed was promptly recorded and made no reference
to the woman’s mortgage obligation. The mortgage obligation was not discharged at the closing.
However, the man immediately began to make the woman’s monthly mortgage payments to the bank after
the closing.

Nine months ago, water seeped into the basement of the house during a major storm, causing substantial
damage. It is undisputed that the seepage was due to defective concrete used by the builder and not to any
negligence on the builder’s part. The man called the builder, told him about the seepage, and demanded
that the builder fix the concrete. The builder responded: “That’s your problem.” The man then repaired
the concrete at a cost of $80,000.

Thereafter, the man sued the builder to recover the $80,000 he had spent to repair the concrete. While the
case was pending, the man stopped making mortgage payments. The bank sued the man to foreclose on
the mortgage and, if necessary, obtain a deficiency judgment against him on the note if the sale proceeds
were insufficient to discharge the mortgage debt. The man has joined the woman as a third-party
defendant in the lawsuit.

1. Is the man likely to prevail against the builder to recover the $80,000 he spent to repair the
concrete? Explain.

2. Is the man personally liable for the outstanding balance on the mortgage note between the
woman and the bank? Explain.

3. If the bank is successful in its foreclosure action, will the man be able to recover damages from
the woman? Explain.

Seperac-J19 Exam-Released MEE Essay Compilation © 2016-2020 455


#101: J13-8 MEE: ANSWER: NCBE (REAL PROPERTY)

POINT (1)(a) [25%] ISSUE: Is there an implied warranty against latent defects in a newly
constructed home? ANSWER: Yes. Because there is an implied warranty against latent defects in
the sale of a new home running from the builder to the buyer, the builder would be liable to the
woman, the original buyer, for damages caused by the defective concrete.

POINT (1)(b) [25%] ISSUE: Does an implied warranty against latent defects protect a remote
grantee? ANSWER: Yes. In many, but not all, jurisdictions the home builder’s warranty against
latent defects extends to a remote grantee like the man.

POINT (2) [35%] ISSUE: Is a buyer who purchases real property from a mortgagor personally
liable on the mortgage debt even if the buyer has not expressly assumed the mortgage obligation?
ANSWER: No. In most, but not all, jurisdictions the man is not liable on the woman’s mortgage
obligation because he did not expressly assume that mortgage obligation. In some jurisdictions, the
man could be held liable on the woman’s mortgage obligation on the theory that he implicitly
assumed this obligation.

POINT (3) [15%] ISSUE: What are the obligations of a grantor who has conveyed title to real
property with a quitclaim deed? ANSWER: If the bank successfully forecloses on the man’s home,
he cannot recover his loss from the woman because she gave the man a quitclaim deed and
quitclaim deeds contain no warranties of title on which a buyer can sue.

ANSWER DISCUSSION:

Although under the common law, there were no implied warranties in the sale of a home, this caveat
emptor approach has been abandoned with respect to builders of new homes who impliedly warrant that
the home is free of latent defects. However, courts are divided as to whether this implied warranty runs to
remote grantees who are not in privity with the home builder. Some courts require privity; others do not,
reasoning that harm to a remote grantee is foreseeable. A purchaser of real estate from a mortgagor has no
liability on the mortgage obligation unless he expressly or impliedly assumes that obligation. Here, the
man did not expressly assume the woman’s mortgage obligation, but it is unclear if he impliedly assumed
it. If the bank successfully forecloses on the man’s house, he has no right to recover his loss from the
woman because he took under a quitclaim, not a warranty, deed.

ANSWER EXPLANATION:

Explanation to Point-One(a) (25%):

Because there is an implied warranty against latent defects in the sale of a new home running from the
builder to the buyer, the builder would be liable to the woman, the original buyer, for damages caused by
the defective concrete.

Under the doctrine of caveat emptor, a home builder makes no implied warranties regarding the condition
of the premises. Today, however, American courts have largely rejected the caveat emptor approach. This
rejection is based on the view that a home buyer does not stand on an “equal footing” with the builder and
relies upon the builder’s skill and integrity, as well as the fact that ordinary home buyers are not in a
position to discover latent defects. Although courts have variously described the home builder’s warranty

Seperac-J19 Exam-Released MEE Essay Compilation © 2016-2020 456


as an implied warranty against latent defects, an implied warranty of fitness, or an implied warranty of
habitability, it is now recognized in almost all states, and it extends to latent defects such as faulty
concrete.

Although the implied warranty is typically called an implied warranty of habitability, in most states it is
not necessary for the man to prove that the house would have been uninhabitable but for his repair. It is
enough that the defect is major, given its nature and the dollar value of the problem. In this respect, the
content of the implied warranty of habitability for the sale of new homes closely resembles the Uniform
Commercial Code implied warranty of merchantability that arises upon the sale of goods by a merchant.
Indeed, some of the courts that first developed the warranty of habitability patterned it after the UCC
warranty.

Thus, under the law of most states today, the builder would be liable for damages to the woman, the
original buyer, for losses caused by the defective concrete.

Explanation to Point-One(b) (25%):

In many, but not all, jurisdictions the home builder’s warranty against latent defects extends to a remote
grantee like the man.

Not all courts have extended the implied warranty of latent defects to remote grantees. Courts that apply a
privity bar have typically done so on the ground that the warranty is contractual in nature and thus should
only run in favor of parties in privity with each other. Other courts, perhaps influenced by the
foreseeability standards of tort law, but also relying on the equal vulnerability of both initial and remote
grantees, have extended the warranty to subsequent purchasers. For example, in Redarowicz v. Ohlendorf,
the Illinois Supreme Court stated:

The warranty of habitability is a creature of public policy. It is a judicial innovation that has evolved to
protect purchasers of new houses upon discovery of latent defects in their homes. While the warranty of
habitability has roots in the execution of the contract for sale, we emphasize that it exists independently.
Privity of contract is not required. Like the initial purchaser, the subsequent purchaser has little
opportunity to inspect the construction methods used in building the home. Like the initial purchaser, the
subsequent purchaser is usually not knowledgeable in construction practices and must, to a substantial
degree, rely upon the expertise of the person who built the home. If construction of a new house is
defective, its repair costs should be borne by the responsible builder-vendor who created the latent defect.
The compelling public policies underlying the implied warranty of habitability should not be frustrated
because of the short intervening ownership of the first purchaser; in these circumstances the implied
warranty of habitability survives a change of hands in the ownership.

Whether the man, as a remote grantee, is protected by the warranty running from the builder to the woman
depends upon which approach governs.

Explanation to Point-Two (35%):

In most, but not all, jurisdictions the man is not liable on the woman’s mortgage obligation because he did
not expressly assume that mortgage obligation. In some jurisdictions, the man could be held liable on the
woman’s mortgage obligation on the theory that he implicitly assumed this obligation.

Because the woman’s mortgage obligation to the bank was recorded, the man took the home subject to the
mortgage, and the bank’s interest had priority over the man’s. Because the bank has priority, it can

Seperac-J19 Exam-Released MEE Essay Compilation © 2016-2020 457


foreclose on the home if the mortgage is in default. However, the recording of a mortgage does not make
the mortgagor’s grantee personally liable for the mortgagor’s obligation.

The predominant rule in the United States is that if a remote grantee takes subject to a mortgage which the
grantee does not assume, the remote grantee is not personally liable on the debt. Of course, that does not
mean that the remote grantee will not want to voluntarily make payments on the mortgage in order to
prevent the mortgagee from foreclosing on the debt. But it does mean that if the mortgagee forecloses on
the property, it cannot hold the remote grantee personally liable on the debt or liable for any deficiency
resulting from a mortgage foreclosure. Following this approach, the man would be personally liable on
the woman’s mortgage obligation if the man assumed (i.e., made a commitment to pay) the mortgage.
There is no evidence of an express assumption agreement here. The deed from the woman to the man
made no mention of the woman’s mortgage, and the facts do not describe any express agreement.

In some jurisdictions, a remote grantee who did not expressly assume a mortgage may be deemed to have
impliedly assumed it where, as here, the remote grantee paid the seller only the difference between what
the house was worth and the outstanding balance on the mortgage obligation. The rationale for this
approach is that “the grantee’s retention of the vendor’s money for the payment of the mortgage imposes
upon him the duty of protecting the vendor against the mortgage debt. This must be so for it would seem
to be almost intolerably unjust to permit him to keep back the (money he would have otherwise paid the
vendor to purchase the property).” Seven states follow this approach.

Here, the man was clearly aware of the mortgage; the mortgage was recorded, and the man paid the
woman only $160,000, considerably less than the house was worth ($360,000). Moreover, the man began
making mortgage payments immediately after buying the house and continued making payments to the
bank for a year.

Thus, in a jurisdiction that follows the implied assumption approach, the man may be liable to the bank.
In a jurisdiction that requires express assumption of a mortgage obligation, the man would not be liable
because there is no evidence that he expressly assumed the mortgage.

[NOTE: If the man is deemed to have impliedly assumed the mortgage, then the woman is merely a surety
and would be liable for any deficiency not paid by the man. ]

Explanation to Point-Three (15%):

If the bank successfully forecloses on the man’s home, he cannot recover his loss from the woman
because she gave the man a quitclaim deed and quitclaim deeds contain no warranties of title on which a
buyer can sue.

If a seller conveys land on which there is an unsatisfied encumbrance such as a mortgage, and that
encumbrance has priority against the buyer, the seller is liable to the buyer for any loss borne by the buyer
only if the seller warranted that there was no such encumbrance against the property. Had the woman
conveyed the land to the man by a warranty deed, that deed would have impliedly included a covenant
against encumbrances, and the woman would have been liable to the man for a breach of this covenant.

Here, however, the woman conveyed the home to the man by a quitclaim deed. A quitclaim deed contains
no warranties of title, and the buyer taking under a quitclaim deed has no claim against the seller for
damages resulting from encumbrances against the property having a priority over the buyer’s interest.
Here the bank has priority over the man’s interest because its mortgage was recorded prior to the
quitclaim deed to the man, and thus the man had notice of the mortgage from the record. The man also

Seperac-J19 Exam-Released MEE Essay Compilation © 2016-2020 458


had actual notice of the mortgage as evidenced by his making of mortgage payments to the bank. Thus,
the man is not entitled to any damages from the woman for the loss he sustained as a result of the
foreclosure.

[NOTE: If the examinee concludes that the man is liable for any deficiency, the man would have a claim
against the woman to recover that deficiency on the ground that he was her surety to guarantee payment
of the deficiency.]

Seperac-J19 Exam-Released MEE Essay Compilation © 2016-2020 459


#102-FEB 2013–MEE Q01: QUESTION ONE (REAL PROPERTY)

In 2008, a landlord and a tenant entered into a 10-year written lease, commencing September 1, 2008, for
the exclusive use of a commercial building at a monthly rent of $2,500. The lease contained a covenant of
quiet enjoyment but no other covenants or promises on the part of the landlord.

When the landlord and tenant negotiated the lease, the tenant asked the landlord if the building had an air-
conditioning system. The landlord answered, “Yes, it does.” The tenant responded, “Great! I will be using
the building to manufacture a product that will be irreparably damaged if the temperature during
manufacture exceeds 81 degrees for more than six consecutive hours.”

On April 15, 2012, the building’s air-conditioning system malfunctioned, causing the building
temperature to rise above 81 degrees for three hours. The tenant immediately telephoned the landlord
about this malfunction. The tenant left a message in which he explained what had happened and asked the
landlord, “What are you going to do about it?” The landlord did not respond to the tenant’s message.

On May 15, 2012, the air-conditioning system again malfunctioned. This time, the malfunction caused the
building temperature to rise above 81 degrees for six hours. The tenant telephoned the landlord and left a
message describing the malfunction. As before, the landlord did not respond.

On August 24, 2012, the air-conditioning system malfunctioned again, causing the temperature to rise
above 81 degrees for 10 hours. Again, the tenant promptly telephoned the landlord. The landlord
answered the phone, and the tenant begged her to fix the system. The landlord refused. The tenant then
attempted to fix the system himself, but he failed. As a result of the air-conditioning malfunction,
products worth $150,000 were destroyed.

The next day, the tenant wrote the following letter to the landlord:

I’ve had enough. I told you about the air-conditioning problem twice before yesterday’s disaster, and you
failed to correct it. I will vacate the building by the end of the month and will bring you the keys when I
leave.

The tenant vacated the building on August 31, 2012, and returned the keys to the landlord that day. At
that time, there were six years remaining on the lease.

On September 1, 2012, the landlord returned the keys to the tenant with a note that said, “I repeat, the air-
conditioning is not my problem. You have leased the building, and you should fix it.” The tenant
promptly sent the keys back to the landlord with a letter that said, “I have terminated the lease, and I will
not be returning to the building or making further rent payments.” After receiving the keys and letter, the
landlord put the keys into her desk. To date, she has neither responded to the tenant’s letter nor taken
steps to lease the building to another tenant.

On November 1, 2012, two months after the tenant vacated the property, the landlord sued the tenant,
claiming that she is entitled to the remaining unpaid rent ($180,000) from September 1 for the balance of
the lease term (reduced to present value) or, if not that, then damages for the tenant’s wrongful
termination.

Is the landlord correct? Explain.

Seperac-J19 Exam-Released MEE Essay Compilation © 2016-2020 460


#102: F13-1 MEE: ANSWER: NCBE (REAL PROPERTY)

POINT (1) [45%] ISSUE: Does the tenant have a defense to the landlord’s action for unpaid rent
based on constructive eviction? ANSWER: No. The tenant was not constructively evicted, because
the landlord had no duty to repair the commercial premises that were the subject of the lease.

POINT (2) [10%] ISSUE: Does the tenant have a defense to the landlord’s action for unpaid rent
based on the tenant’s surrender of the premises? ANSWER: The landlord did not accept the
tenant’s surrender of the lease.

POINT (3) [45%] ISSUE: What, if anything, may the landlord recover from the tenant for the
period after the tenant vacated the building? ANSWER: Under the common law, the landlord had
no duty to mitigate damages. Additionally, a landlord was not entitled to recover unpaid rents due
in the future but was only entitled to recover rents in arrears at the time of the commencement of
the suit. Applying the common law here, the landlord could recover $5,000, the amount of rents due
at the commencement of the suit ($2,500 for September and the same for October). Today, some
courts allow the landlord, under certain circumstances, to sue the tenant for damages (not rent)
equal to the difference, if any, between the unpaid promised rent for the balance of the term (here
$175,000) and the property’s fair rental value for the balance of the term.

ANSWER DISCUSSION:

Under the common law, the tenant does not have a defense to the landlord’s action for unpaid rent based
on constructive eviction. Constructive eviction is based on the tenant proving that (1) the landlord
breached a duty to the tenant, (2) the breach caused a loss by the tenant of the substantial use and
enjoyment of the premises, (3) the tenant gave the landlord adequate notice and opportunity to repair, and
(4) the tenant vacated the leased premises. Here, there was no constructive eviction because, although the
tenant vacated and gave the landlord adequate notice, the landlord breached no express or implied duty to
the tenant to repair the premises. The tenant does not have a defense based on the landlord’s acceptance of
his surrender of the premises; a landlord’s retention of keys does not constitute an acceptance of the
tenant’s surrender unless the landlord so intended, and here, the landlord’s statements to the tenant at the
time of the surrender of the keys do not evidence the intent to accept the tenant’s surrender. Under the
common law, a landlord has no duty to mitigate damages but also cannot sue for rents due in the future.
Under this approach, the landlord can sue only for past-due rents. Using this approach, on November 1,
the landlord could recover all the rent past due (i.e., rent for September and October) but could not
recover for rents due in the future. However, some courts have authorized recovery for future rent minus
the fair market rental value of the premises. It is thus possible that the landlord could recover damages
equal to the amount of rent due from September 1 to the end of the six-year lease term ($180,000) minus
the property’s fair-market rental value over that same period.

ANSWER EXPLANATION:

Explanation to Point-One (45%):

The tenant was not constructively evicted, because the landlord had no duty to repair the commercial
premises that were the subject of the lease.

Seperac-J19 Exam-Released MEE Essay Compilation © 2016-2020 461


The landlord and the tenant entered into a term-of-years lease because the lease specified both a beginning
and an ending date. Although a term-of-years lease normally cannot be terminated by the tenant prior to
the end of the term, a tenant may terminate a term-of-years lease if the tenant is constructively evicted.
Typically, as here, a claim of constructive eviction is made as a defense to a landlord’s action for damages
or unpaid rent.

In order to establish a constructive eviction, the tenant must prove that the landlord breached a duty to the
tenant, such as a duty to repair, and that the landlord’s breach caused a loss of the substantial use and
enjoyment of the premises. The tenant must also show that he gave the landlord notice adequate to permit
the landlord to meet his duty to the tenant and that the tenant vacated the leased premises.

Under the common law, there was no implied duty on the part of a landlord to repair leased premises;
such a duty arose only if expressly set forth in the lease. Here, the written lease contained no term
requiring the landlord to repair the air-conditioning. Even if the conversation created a lease term that the
building had air-conditioning, that itself should not create a duty for the landlord to repair it.

Over the past several decades, courts have generally implied a duty to repair in residential leases either as
part of a revised constructive eviction doctrine or based on an implied warranty of habitability. This shift
has been justified based on the economic disparity between the typical landlord and tenant as well as the
fact that residential tenants generally lack both the authority to authorize repairs to common areas of a
building and the incentive to make repairs that will ultimately benefit the landlord.

However, courts have been more reluctant to imply a duty to repair in commercial leases, a context in
which the tenant is often a valuable business and in a better position to assess and make repairs than is the
landlord. When courts have implied a duty to repair in a commercial lease, it is typically when the repair
has been mandated by public authorities and involves work so substantial that it would not ordinarily fall
within the tenant’s common law repair duty and/or the value of the repair would primarily inure to the
landlord’s reversionary interest. Some courts have also permitted constructive eviction claims by
commercial tenants of office buildings based on repairs required in common areas of the building.

Here, the tenant is the owner of a valuable manufacturing operation and is the exclusive occupant of the
building, the repair has not been mandated by public authorities, and the repair is not structural. To the
contrary, the repair involves a feature of the building of unusual importance in the tenant’s manufacturing
operation, and the tenant is likely far more knowledgeable than the landlord about the air-conditioning
specifications necessary for the manufacture of the tenant’s product.

Based on these facts, it is unlikely that a court will find that the tenant in this case was constructively
evicted. Although the tenant can show that he gave adequate notice to the landlord of the air-conditioning
malfunction and vacated the premises, the lease was commercial, and it did not contain any promises or
covenants by the landlord except a covenant of quiet enjoyment; a covenant of quiet enjoyment does not
entail any repair obligations.

[NOTE: An examinee’s conclusion is less important than his or her demonstrated awareness of the
elements of constructive eviction and the need to imply a repair duty for such a defense to be viable here.
Although the implied warranty of habitability is not available to this tenant, a few states imply a warranty
of suitability in commercial leases in limited circumstances, and an examinee might argue that this
warranty should apply here. If an examinee concludes that this warranty applies, he or she should discuss
the other requirements for constructive eviction.

Seperac-J19 Exam-Released MEE Essay Compilation © 2016-2020 462


If the examinee wrongly concludes that the first element for a constructive eviction has been met, the
examinee will then have to discuss the remaining three elements in order to conclude that the tenant can
claim constructive eviction. The tenant would have a strong argument that the second element –
substantial interference with the use and enjoyment of the premises – also is met. As indicated above, the
landlord was aware that a functioning air-conditioning system was vital to the tenant’s manufacturing
operations. The facts further indicate that the system had failed three times in the past few months. The
landlord may try to argue that the malfunctions did not substantially interfere with the tenant’s use of the
premises because the malfunctions caused the temperature to climb above 81 degrees for only a short
period of time – 3 hours, 6 hours, and 10 hours, respectively – on each occasion. The tenant will argue,
however, that the landlord was aware that the tenant’s manufacturing operations could tolerate
temperatures above 81 degrees for no more than 6 hours. The final malfunction exceeded that limit,
destroying $150,000 worth of the tenant’s products.

The tenant would also have a strong argument that the third element is met: notice and opportunity to
cure. The tenant notified the landlord of the problem immediately upon the system’s first malfunction and
did so again when it malfunctioned a second time and then a third time. The landlord might argue that
there was insufficient time to cure the problem because the system corrected itself within a few hours on
the first and second times. Although the malfunction lasted more than 10 hours the third time, the
landlord might argue that the time period was insufficient to get a repair person on the premises. A court
would be likely to find this argument unpersuasive, however, because the landlord could have attempted
to correct the problem after the first and second malfunctions.

Assuming that the landlord was given sufficient notice and opportunity to cure, a court would be likely to
conclude that the tenant also satisfied the final element of vacating the premises within a reasonable time.
The landlord might argue that the tenant remained in the premises for almost four months after the air
conditioning first failed, which would suggest that the problem was not so severe as to have constructively
evicted the tenant. The tenant will argue, however, that he gave the landlord three months to cure the
problem after the first two malfunctions threatened (but did not actually harm) his operations. The tenant
then moved out shortly after the final malfunction caused temperatures to exceed the tolerance levels of
his manufacturing operations.]>

Explanation to Point-Two (10%):

The landlord did not accept the tenant’s surrender of the lease.

When a tenant wrongfully moves from leased premises with the intent to terminate the lease, the landlord
may either accept the tenant’s surrender of the premises and terminate the lease or hold the tenant to the
terms of the lease. Here, the tenant’s only basis for the claim that the landlord accepted his surrender is
the landlord’s retention of the keys. Many courts have considered whether a landlord’s retention of keys
delivered by a tenant constitutes acceptance of surrender. The weight of the case law holds that retention
of the keys alone does not constitute acceptance of surrender without other evidence showing that the
landlord intended to accept the surrender.

Here, the landlord’s note, saying “I repeat, the air-conditioning is not my problem. You have leased the
building, and you should fix it,” strongly suggests that the landlord did not intend to accept the tenant’s
surrender. The tenant might argue that the landlord’s failure to make a similar statement when the keys
were sent to her a second time and she retained them evidences a change of heart. However, it is likely
that a court would find that the landlord’s retention of the keys represented a decision to safeguard the
keys, not to accept the tenant’s surrender.

Seperac-J19 Exam-Released MEE Essay Compilation © 2016-2020 463


[NOTE: An examinee should receive credit for arguing the other way with a well-reasoned argument.]

Explanation to Point-Three (45%):

Under the common law, the landlord had no duty to mitigate damages. Additionally, a landlord was not
entitled to recover unpaid rents due in the future but was only entitled to recover rents in arrears at the
time of the commencement of the suit. Applying the common law here, the landlord could recover $5,000,
the amount of rents due at the commencement of the suit ($2,500 for September and the same for
October). Today, some courts allow the landlord, under certain circumstances, to sue the tenant for
damages (not rent) equal to the difference, if any, between the unpaid promised rent for the balance of the
term (here $175,000) and the property’s fair rental value for the balance of the term.

Under the common law, because a lease was viewed as a conveyance instead of a contract, a landlord had
no duty to mitigate damages resulting from a tenant’s wrongful termination of a lease. A landlord could
thus recover the full value of rents that were due and unpaid at the time of the suit. However, under the
common law, a landlord could not sue a tenant for rents due in the future because there was always a
possibility that the tenant might pay the rent when it was due. Thus, using the common law approach, on
November 1, the landlord could only recover the full value of the two months’ rent actually due and
unpaid, i.e., $5,000 for September and October.

Some courts have rejected the no-mitigation-of-damages rule based on efficiency concerns and society’s
interest in assuring that resources remain in the stream of commerce rather than lying vacant, and allow
landlords to sue tenants who have wrongfully terminated a lease for damages equal to the difference
between the unpaid rent due under the lease and the property’s fair market rental value. Other courts have
abandoned the no-recovery-for-future-rent rule. These courts, responding to the fact that a tenant may
well disappear or be judgment-proof by the time a lease term is concluded, have allowed a landlord to
collect damages equal to the value of rent over the entire lease term minus the property’s fair rental value
when a tenant has wrongfully terminated a lease and unequivocally shown an intention not to return to the
premises or pay future rent. Under this approach, a landlord receives approximately the same amount he
would have received were there a duty to mitigate damages.

Here, because the tenant returned the keys to the landlord and said, “I will not be returning to the building
or making further rent payments,” the landlord could establish abandonment and an intention not to
return. It is thus possible that the landlord might recover damages in the amount of $5,000 (for the months
of September and October) plus the present value of $175,000 minus the fair market rental value of the
property over the remaining months of the lease.

Seperac-J19 Exam-Released MEE Essay Compilation © 2016-2020 464


#103-FEB 2012–MEE Q08: QUESTION EIGHT (REAL PROPERTY)

Blackacre, which is immediately to the west of Whiteacre, is bounded on its west by a state highway.
Whiteacre is bounded on the east by a county road. Both roads connect to a four-lane highway.

Twenty years ago, Tom, who then owned Blackacre, sold to Sue, who then owned Whiteacre, an
easement over a private gravel road that crossed Blackacre. This easement allowed Sue significantly
better access to the four-lane highway from Whiteacre than she had had using only the county road
adjacent to Whiteacre. The easement was promptly and properly recorded.

After acquiring this easement, Sue discontinued using the county road to the east of Whiteacre and used
the private gravel road crossing Blackacre to travel between Whiteacre and the four-lane highway. Sue
used the private gravel road across Blackacre for that purpose almost every day for the next 18 years.

Fifteen years ago, Sue purchased Blackacre from Tom. The deed from Tom to Sue was promptly and
properly recorded.

Two years ago, Sue sold Whiteacre to Dan. The deed from Sue to Dan, which was promptly and properly
recorded, did not mention the private gravel road crossing Blackacre, although Dan was aware that Sue
had used the road to more easily access the four-lane highway.

Following the purchase of Whiteacre, Dan obtained a construction loan from Bank secured by a mortgage
on Whiteacre. This mortgage was promptly and properly recorded. The loan commitment, in the amount
of $1,500,000, which was reflected in the mortgage, obligated Bank to loan Dan $300,000 immediately. It
further obligated Bank to loan Dan an additional $500,000 in 180 days and $700,000 in 280 days.

After obtaining the second loan installment from Bank, Dan realized that he would need additional funds
and borrowed $400,000 from Finance Company. This loan was also secured by a mortgage on Whiteacre.
Upon Dan’s signing the note and mortgage, Finance Company immediately remitted the $400,000 to Dan
and promptly and properly recorded its mortgage.

Thereafter, Bank advanced the final $700,000 loan installment to Dan.

Recently, Dan defaulted on the loans from both Bank and Finance Company. At the time of these
defaults, Dan owed $1,500,000 to Bank and $400,000 to Finance Company.

At a proper foreclosure sale by Bank, Whiteacre was sold for $1,500,000 net of sale expenses.

1. Immediately before Sue sold Whiteacre to Dan, did Sue have an easement over Blackacre?
Explain.

2. Immediately after Sue sold Whiteacre to Dan, did Dan have an easement over Blackacre?
Explain.

3. How should the proceeds from the sale of Whiteacre be distributed between Bank and Finance
Company? Explain.

Seperac-J19 Exam-Released MEE Essay Compilation © 2016-2020 465


#103: F12-8 MEE: ANSWER: NCBE (REAL PROPERTY)

POINT (1) [25%] ISSUE: Was Sue’s easement over Blackacre that she had acquired from Tom
extinguished when she purchased Blackacre from Tom? ANSWER: Yes. The easement Sue
acquired from Tom was extinguished by merger when Sue bought Blackacre from Tom. Thus,
immediately prior to her sale of Whiteacre to Dan, Sue had no easement over Blackacre.

POINT (2) [40%] ISSUE: Did Dan acquire an easement implied from prior use over Blackacre
when he purchased Whiteacre from Sue? ANSWER: Yes. Upon the conveyance of Whiteacre, Dan
acquired an easement implied from prior use with respect to use of the private gravel road
traversing Blackacre.

POINT (3) [35%] ISSUE: How should the proceeds from the sale of Whiteacre be distributed?
ANSWER: The proceeds from the foreclosure sale should be paid entirely to Bank because Bank
gave Dan a future-advances mortgage.

ANSWER DISCUSSION:

Immediately prior to Sue’s sale of Whiteacre to Dan, Sue had no easement over Blackacre because the
easement she had acquired from Tom was extinguished by merger. Immediately after the sale of
Whiteacre to Dan, Dan had an easement over Blackacre. This easement arose by implication from Sue’s
prior use of the private gravel road over Blackacre for the benefit of Whiteacre. Because Bank was
obligated to advance the $700,000 to Dan under the terms of its loan commitment with Dan, Bank is
entitled to all $1,500,000 from the $1,500,000 sales proceeds, and Finance Company is entitled to
nothing.

ANSWER EXPLANATION:

Explanation to Point-One (25%):

The easement Sue acquired from Tom was extinguished by merger when Sue bought Blackacre from
Tom. Thus, immediately prior to her sale of Whiteacre to Dan, Sue had no easement over Blackacre.

Sue purchased from Tom an easement for a right-of-way over Blackacre to be used in connection with her
ownership of Whiteacre. This easement, an appurtenant easement of which Whiteacre was the dominant
estate and Blackacre was the servient estate, was promptly and properly recorded. When the owner of the
dominant estate (Sue) acquired the servient estate (Blackacre), however, the easement was extinguished
by merger. This result follows from the fact that because of the post-sale unity of ownership of the two
estates, Sue could enforce the easement only against herself, and lawsuits against one’s self are
impermissible.

Explanation to Point-Two (40%):

Upon the conveyance of Whiteacre, Dan acquired an easement implied from prior use with respect to use
of the private gravel road traversing Blackacre.

An easement implied from prior use arises in favor of a grantee when (1) two parcels of land are in
common ownership; (2) one of the parcels is conveyed to a grantee; (3) the parcel conveyed had been

Seperac-J19 Exam-Released MEE Essay Compilation © 2016-2020 466


receiving a benefit from the parcel retained prior to the conveyance to the grantee, i.e., there was a use
over the retained parcel in favor of the conveyed parcel which could have been the subject of an express
easement appurtenant; (4) the usage is reasonably necessary or convenient; and (5) the usage is apparent.

Here, the first three criteria are clearly met: Sue owned both Blackacre and Whiteacre; she conveyed one
parcel (Whiteacre) to Dan; and Whiteacre, the parcel conveyed, had been receiving a benefit from the
parcel retained, i.e., use of the private gravel road traversing Blackacre, which could have been – indeed,
was, before its extinction by merger – the subject of an express easement appurtenant.

Sue’s use of Blackacre also appears to satisfy the “reasonably necessary” and “apparent” requirements.
“The principle underlying the creation of an easement by implication is that it is so evidently necessary to
the reasonable enjoyment of the granted premises, so continuous in its nature, so plain, visible and open,
so manifest from the situation and relation of the two tracts that the law will give effect to the grant
according to the presumed intent of the parties.”

Most courts interpret the “reasonably necessary” requirement “to mean that the easement must be
important to the enjoyment of the” conveyed land or “highly convenient.” Courts are likely to find that an
easement is implied from prior use where access to the transferred land is “extremely difficult by other
routes” and to find that an easement is not implied when the transferred land has “easy access to public
roads in another direction.” Sue’s use of Blackacre for the benefit of Whiteacre meets this test; the facts
specify that the private gravel road provided significantly better access to the four-lane highway than did
the county road abutting Whiteacre.

Most courts have found that the “apparent” requirement is met when the usage is visible. Here, the private
gravel road was open and obvious; indeed, the facts state that Dan was aware that Sue had used that road
to more easily access the four-lane highway.

Thus, upon Sue’s conveyance of Whiteacre to Dan, she conveyed to Dan an easement implied from prior
use.

[NOTE: Dan did not acquire an easement by necessity over Blackacre, because Whiteacre is not
landlocked, and he has access to it from the county road. An easement by necessity arises only when there
has been a conveyance of a portion of the grantor’s land, the grantor retains the remaining portion, and,
after severance of the grantor’s land, it is necessary for the grantee to pass over the grantor’s retained
portion to reach a public street or highway. Landlocked land clearly satisfies the necessity test, and a few
courts have found an easement where there is merely “reasonable” necessity. Additionally, this is not a
case for easement by estoppel. An easement by estoppel arises when A gives B permission to use A’s land
and the licensee, here B, invests substantial funds in that land reasonably relying on the permission. The
facts here do not support the conclusion that there is an easement by estoppel.]

Explanation to Point-Three (35%):

The proceeds from the foreclosure sale should be paid entirely to Bank because Bank gave Dan a future-
advances mortgage.

The typical construction loan provides that the lender will advance funds to the borrower over a fixed
time period. The lender secures a mortgage on the property for the entire amount of the money it has
agreed to lend, including future advances. Such “future advances” mortgages may provide for obligatory
advances, or they may provide for advances that are optional.

Seperac-J19 Exam-Released MEE Essay Compilation © 2016-2020 467


Whether the future-advances mortgage payments are obligatory or optional is critical to the rights of a
junior lender. If payments under a future-advances mortgage are obligatory, then the junior lender’s lien is
junior both to amounts loaned to the debtor before the junior lien was recorded and to amounts loaned
after the junior lien is recorded. If the payments under a future-advances mortgage are optional, the junior
lender has a priority over amounts transferred to the debtor by the senior lender after the junior lender
transfers funds to the debtor and records its mortgage. The rationale for this rule is that in the case of an
obligatory loan, the junior lender can ascertain from public land records the maximum amount of the
senior lender’s claim before loaning money to the creditor, while with an optional loan, the junior lender
cannot know whether subsequent advances will be made and the senior lender can protect itself against
junior lenders by searching the land records before making additional advances.

Here, Bank’s loan commitment to Dan was secured by an obligatory future-advances mortgage, and Bank
both recorded its mortgage and loaned Dan money before Finance Company did so. Thus, Bank is the
senior lender in the amount of $1,500,000, the full value of its loan commitment to Dan, and it is entitled
to all of the foreclosure sales proceeds. Finance Company is entitled to nothing.

Seperac-J19 Exam-Released MEE Essay Compilation © 2016-2020 468


#104-JUL 2011–MEE Q04: QUESTION FOUR (REAL PROPERTY)

In 1980, Oscar sold undeveloped land that he owned in fee simple to Sam, but Sam failed to record the
deed.

In 1985, Sam granted Railroad an easement to operate a rail line across a portion of the land to serve a
grain storage facility located on a neighboring tract of land. Railroad recorded this easement, laid railroad
tracks on the land, and operated trains weekly until the grain storage facility went out of business in 2000.
The tracks are still in place and clearly visible, but no trains have operated over them since 2000.

In 1990, Sam conveyed the land to Daughter as a graduation gift. Daughter promptly recorded the deed
given to her by Sam. Except for the railroad tracks, the land has remained undeveloped.

Oscar died six months ago. Unaware of the prior transactions, the executor of Oscar’s estate sold the land
to Purchaser for its fair market value. Purchaser was also unaware of these prior transactions. The
executor gave Purchaser a quitclaim deed to the land. Purchaser promptly recorded this deed.

The state in which the land is located maintains its records under a grantor-grantee indexing system, and
the state’s recording act provides: “No conveyance or mortgage of real property shall be good against
subsequent purchasers for value and without notice unless the same be recorded according to law.”

What are the rights, if any, of Purchaser, Daughter, and Railroad in the land? Explain.

Seperac-J19 Exam-Released MEE Essay Compilation © 2016-2020 469


#104: J11-4 MEE: ANSWER: NCBE (REAL PROPERTY)

POINT (1) [30%] ISSUE: Has Railroad abandoned its interest? ANSWER: No. Railroad’s
easement was not abandoned unless Railroad had an intent to abandon it.

POINT (2) [10%] ISSUE: In the absence of a state recording act, as between Daughter and
Purchaser, who owns the land? ANSWER: Under the common law first-in-time, first-in-right
principle, Daughter, and not Purchaser, owns the land, unless the state’s recording act mandates a
contrary result.

POINT (3) [25%] ISSUE: Was Purchaser on constructive notice of the interests of Railroad and
Daughter and thus not a bona fide purchaser for value? ANSWER: No. While the state has rejected
the common law first-in-time, first-in-right principle by enacting a notice-type statute with a
grantor-grantee index, Purchaser was not on constructive notice of the interest of either Daughter
or Railroad. Although the deeds to Daughter and Railroad were recorded, they were not properly
indexed and, thus, not discoverable, because the deed to their predecessor in title – Sam – was never
recorded.

POINT (4) [35%] ISSUE: Was Purchaser on inquiry notice of the interests of Railroad and
Daughter and thus not a bona fide purchaser for value? ANSWER: Yes. Purchaser is not a bona
fide purchaser for value under the state’s notice-type recording statute because he had inquiry
notice of Railroad’s interest and, likely, of Daughter’s interest as well.

ANSWER DISCUSSION:

Railroad’s interest would be lost only if it abandoned that interest, and abandonment is not supported by
the facts. Under the common law, in the absence of a recording statute, Daughter would own the land in
fee simple subject to the Railroad’s easement because both of them acquired their interests before
Purchaser bought the land from Oscar’s estate. This result changes under the state’s notice-type recording
statute only if Purchaser is a protected bona fide purchaser without notice of either prior interest. Here, the
most likely conclusion is that Purchaser, although having no actual or constructive notice of the interest of
Railroad or Daughter, had inquiry notice of their interests and, thus, is not protected by the state recording
statute. An argument can be made, however, that Purchaser had inquiry notice only of Railroad’s interest,
in which case Daughter would have no interest in the land, and Purchaser would own it subject to
Railroad’s interest.

ANSWER EXPLANATION:

Explanation to Point-One (30%):

Railroad’s easement was not abandoned unless Railroad had an intent to abandon it.

Easements may be terminated in a variety of ways, such as by their express terms, written release, merger
of the dominant and servient tenements, prescription, estoppel, condemnation, and abandonment. The
only method of termination potentially applicable under these facts is abandonment. If a court were to
find that either Daughter or Purchaser acquired a fee simple title subject to Railroad’s interest, either may
argue that Railroad’s easement was extinguished by abandonment because Railroad has not used the
easement since 2000.

Seperac-J19 Exam-Released MEE Essay Compilation © 2016-2020 470


An abandonment theory is unlikely to succeed, however, because mere non-use of an easement is
insufficient to establish abandonment. Instead, there must be a cessation of use coupled with evidence of
the user’s intent to abandon the easement. Here, there is insufficient evidence of Railroad’s intent to
abandon its easement. Notably, Railroad has not removed its tracks and there are no other facts to suggest
that Railroad has voluntarily relinquished its interest. In the absence of such evidence, a court is likely to
find that Railroad has not abandoned its easement.

Explanation to Point-Two (10%):

Under the common law first-in-time, first-in-right principle, Daughter, and not Purchaser, owns the land,
unless the state’s recording act mandates a contrary result.

Under the common law, a grantor can convey only those rights in land that the grantor had at the time of
the conveyance. Thus, under the common law, priority among successive transfers is dictated by priority
of time – the so-called “first-in-time, first-in-right” principle.

Here, Oscar’s conveyance to Sam in 1980 takes priority over the subsequent transfer by the executor of
Oscar’s estate to Purchaser. Once Oscar conveyed the land to Sam, neither Oscar nor his estate at his later
death had a title left to convey to anyone, including Purchaser. Rather, title was then in Sam. Sam’s
conveyance of legal title to Daughter would prevail over the later conveyance by Oscar’s executor to
Purchaser. However, if the state recording act protects Purchaser, then Purchaser rather than Daughter
would own the land.

Explanation to Point-Three (25%):

While the state has rejected the common law first-in-time, first-in-right principle by enacting a notice-type
statute with a grantor-grantee index, Purchaser was not on constructive notice of the interest of either
Daughter or Railroad. Although the deeds to Daughter and Railroad were recorded, they were not
properly indexed and, thus, not discoverable, because the deed to their predecessor in title – Sam – was
never recorded.

All states have recording statutes that, when applicable, overturn the results of the common law “first-in-
time, first-in-right” principle. The recording statute here is a notice-type statute providing that unrecorded
conveyances are not valid against a “subsequent purchaser for value and without notice” of the
conveyance. Under a notice-type recording statute, Purchaser can prevail against either of or both
Daughter and Railroad only if (1) the prior conveyances were unrecorded, (2) Purchaser paid value for the
land, and (3) Purchaser took without either actual, constructive, or inquiry notice of the prior
conveyances.

[NOTE: Some examinees may incorrectly conclude that the recording act is a race-notice statute because
the last part of the statute states: “unless the same be recorded according to law.” Placed in its proper
grammatical context, however, “the same” relates to the instrument of prior conveyance. There is no
requirement that the subsequent purchaser must record to take priority over an earlier transaction. By
comparison, a typical race-notice statute protects only a subsequent purchaser who records. If he does
not record, he is not protected, even though he is a bona fide purchaser.]

The facts clearly state that Purchaser paid value, so payment of value is not an issue here. Furthermore, no
facts suggest that Purchaser had actual notice of the conveyances of the easement and fee to Railroad and
Daughter, respectively. But there are two other possible ways for a subsequent purchaser to have notice.
One is constructive; the other is inquiry.

Seperac-J19 Exam-Released MEE Essay Compilation © 2016-2020 471


Constructive notice arises through the recording system. A purchaser is placed on constructive notice of
all information that is properly recorded on the public land records whether he sees it or not. While both
Railroad and Daughter recorded their interests, Sam (from whom they both claimed) did not. Because
Sam’s deed from Oscar was not recorded, both the deed from Sam to Railroad conveying an easement and
the later deed from Sam to Daughter conveying to her a fee are so-called “wild deeds,” meaning they
were recorded outside the chain of title. As such, they are undiscoverable by a reasonable search of the
grantor-grantee indexes. A search of the public land records by Purchaser would have shown that as a
matter of public record his seller, Oscar, had not conveyed the property to anyone because the Oscar to
Sam deed was not recorded. As a result, a title searcher could not have discovered Sam’s later
conveyances to Railroad and Daughter even though they were recorded. Courts have uniformly held that a
wild deed is not “properly recorded” and, therefore, imports no constructive notice to a subsequent
purchaser. Accordingly, Purchaser is not on constructive notice of the interests of Daughter and Railroad
and his interest would appear to have priority to theirs.

Explanation to Point-Four (35%):

Purchaser is not a bona fide purchaser for value under the state’s notice-type recording statute because he
had inquiry notice of Railroad’s interest and, likely, of Daughter’s interest as well.

Although Purchaser did not have constructive notice, the law also imputes knowledge to a subsequent
purchaser in another way, namely through an obligation of inquiry. Here there is a strong argument that
Purchaser had inquiry notice of Railroad’s interest and, therefore, also that he had inquiry notice of
Daughter’s interest.

Inquiry notice exists when knowledge is imputed to a buyer from facts and circumstances suggesting the
existence of a prior conveyance. Some courts have held that a purchaser who takes by quitclaim deed, as
Purchaser did, is presumed to take with notice of any interests that could have been discovered by
reasonable diligence. This perspective is based upon the theory that the title is suspicious if a grantor
refuses to give covenants of title. The majority of courts, however, reject this rule because there are many
legitimate reasons for a grantor to convey by quitclaim deed, particularly in an environment where title
insurance is common. Accordingly, in most states, Purchaser would not have inquiry notice from the
quitclaim deed alone.

[NOTE: In the minority of states that would require further inquiry based upon the quitclaim deed alone,
it is unclear what further investigation is required. One could argue that Purchaser could have contacted
Oscar’s heirs or neighboring landowners, but there is no indication from the facts that such an inquiry
would have revealed the prior conveyances. Another possibility would be to search the property tax rolls.
The facts do not indicate who has been paying taxes on the property (or whether such taxes even exist in
this state), but it is possible that such taxes were assessed against Sam (and later Daughter) after they
acquired title to the property.]

The better argument is that inquiry notice arose from the possibility of a visual inspection of the property.
Inquiry notice exists when someone other than the record owner is in possession or use of the property.
Here, a visual inspection would have revealed the existence of railroad tracks on otherwise undeveloped
land. Exercising reasonable diligence, Purchaser would have discovered the tracks, and then should have
engaged in further inquiry to ascertain the interest of whoever put the tracks there. Accordingly, a court is
most likely to find that even though Railroad’s easement was recorded outside the chain of title, Purchaser
had inquiry notice of Railroad’s interest based upon the visible improvements on the land.

Seperac-J19 Exam-Released MEE Essay Compilation © 2016-2020 472


Would this inquiry notice also extend to the interest of Sam and then Daughter? Assuming that Purchaser
was successful in making inquiries to find Railroad, Purchaser could have discovered that Railroad had
acquired its interest from Sam. That discovery would have prompted further inquiry into the nature of
Sam’s interest. In addition, a search of the grantor index for Sam’s name would reveal the deed to
Daughter, which was recorded, albeit outside of the chain of title. Accordingly, if Railroad is still in
existence and easily located, a court is likely to find that Purchaser had inquiry notice not only of
Railroad’s easement but also Daughter’s fee simple interest.

In short, it seems fairly clear that Purchaser would be deemed to have inquiry notice of Railroad’s
easement, and it is likely that Purchaser also would have inquiry notice of Daughter’s interest. [NOTE:
Should an examinee, however, reason that Purchaser’s inquiry obligation did not extend beyond asking
Railroad what its interest was, then Purchaser would prevail as against Daughter, but not Railroad.]

[NOTE: Examinees may observe that Daughter does not qualify as a purchaser for value because
Daughter received the land as a gift. However, Daughter’s status as a donee is irrelevant to the analysis
because she acquired legal title before Purchaser and, therefore, would prevail under the common law
priority rule. Thus, Daughter is protected without having to prove her status as a purchaser under the
recording act.]

Seperac-J19 Exam-Released MEE Essay Compilation © 2016-2020 473


#105-JUL 2010–MEE Q05: QUESTION FIVE (REAL PROPERTY)

Eighty years ago, Owner, the owner of vacant land known as Blackacre, conveyed Blackacre to a local
school district (School) “if School uses Blackacre only to teach children aged 5 to 13.” Shortly after
acquiring title to Blackacre, School erected a classroom building on Blackacre and began teaching
children aged 5 to 13 in that building.

Seventy years ago, Owner died and left his entire estate to Daughter.

School used the classroom building to teach its students aged 5 to 13 until three years ago when, due to
increasing enrollments, School built a new classroom building three miles from Blackacre and converted
the classroom building on Blackacre into administrative offices.

The building on Blackacre is now exclusively occupied by administrative offices, and all School students
aged 5 to 13 are taught in the new classroom building.

Two years ago, Daughter died. Daughter did not object to School’s altered use of Blackacre before her
death. She devised her entire estate to her Husband for life, with the remainder to “my surviving
children.” Daughter was survived by Husband and two children, Ann and Bill.

One year ago, Bill died. Bill’s entire estate passed to his wife, Mary.

One month ago, Husband, the life tenant under Daughter’s will, died. Husband was survived by Ann and
by Bill’s widow, Mary.

State law provides:

I. “Actions to recover the possession of real property shall be brought within 10 years after the cause of
action accrues.”

II. “All future interests are alienable, devisable, and descendible to the extent they do not expire as a result
of the holder’s death.”

III. “Conditions and limitations in a deed shall not be construed as covenants.”

There are no other relevant statutes.

What interests, if any, do School, Ann, and Mary have in Blackacre? Explain.

Seperac-J19 Exam-Released MEE Essay Compilation © 2016-2020 474


#105: J10-5 MEE: ANSWER: NCBE (REAL PROPERTY)

POINT (1) [45%] ISSUE: What interest did School acquire in Blackacre and what interest did
Owner retain? ANSWER: The terms of the conveyance to School were ambiguous. In construing an
ambiguous instrument, courts typically adopt a preference for the fee simple on condition
subsequent. If so construed, it is unclear what interest, if any, Owner retained. It is either no
interest or a right of entry for condition broken (also called a “power of termination”). If, on the
other hand, School acquired a fee simple determinable in Blackacre, then Owner retained a
possibility of reverter.

POINT (2) [10%] ISSUE: Did School breach the condition or limitation in the deed from Owner
when it stopped using the building erected on Blackacre to teach students and began using it for
administrative purposes? ANSWER: Yes. School has likely ceased using Blackacre “only to teach
children aged 5 to 13.” If Owner is treated as having either a possibility of reverter or a right of
entry for condition broken (power of termination), then School’s interest can come to an end and,
Blackacre passes to the successors of Owner’s estate. Otherwise, School continues to own
Blackacre.

POINT (3) [10%] ISSUE: If Owner retained a future interest in Blackacre, did the statute of
limitations run on that interest? ANSWER: No. If Owner’s successors have an interest in
Blackacre, the statute of limitations has not run on that interest.

POINT (4) [35%] ISSUE: What interests, if any, do Ann and Mary have in Blackacre? ANSWER:
Ann has an interest in Blackacre if Owner retained either a possibility of reverter or a power of
termination. In some jurisdictions, Mary (Bill’s widow) would also have such an interest. If Owner
retained only a power of termination, Ann will have to assert that right by exercising the power
because possession will not automatically inure to her as it would have if Owner had retained a
possibility of reverter.

ANSWER DISCUSSION:

Owner conveyed either a fee simple determinable or a fee simple on condition subsequent to School.
Because the Owner-to-School deed used language that did not conform to the classic formulations for
creating either of these interests, it is unclear which interest School acquired. However, in the case of an
ambiguous conveyance, there is a presumption in favor of the fee simple on condition subsequent. If
School has a fee simple on condition subsequent, either (1) Owner retained nothing to pass to Daughter
because the Owner-to-School deed did not expressly specify that Owner retained a power of termination,
or (2) Owner retained an implied power of termination which passed to Daughter and then through
Daughter’s estate to Ann and possibly, upon Bill’s death, to Mary as well. If School has a fee simple
determinable, then Owner’s retained possibility of reverter passed through Daughter’s estate at least to
Ann and possibly to Mary. Any cause of action for breach of the condition or limitation (depending on the
interest created in the School) accrued only three years ago and, thus, the statute of limitations on that
action has not run. Of course, whether School’s interest has been lost depends upon whether using
Blackacre for administrative purposes violates the requirement of using the property to teach children
aged 5 to 13, which it probably does.

ANSWER EXPLANATION:

Seperac-J19 Exam-Released MEE Essay Compilation © 2016-2020 475


Explanation to Point-One (45%):

The terms of the conveyance to School were ambiguous. In construing an ambiguous instrument, courts
typically adopt a preference for the fee simple on condition subsequent. If so construed, it is unclear what
interest, if any, Owner retained. It is either no interest or a right of entry for condition broken (also called
a “power of termination”). If, on the other hand, School acquired a fee simple determinable in Blackacre,
then Owner retained a possibility of reverter.

If the Owner-to-School deed conveyed a fee simple determinable to School, then Owner retained a
possibility of reverter, which became possessory immediately upon the happening of the event designated
in the instrument. If, instead, the Owner-to-School deed conveyed a fee simple on condition subsequent,
Owner may or may not have a power of termination or right of entry for condition broken, depending on
whether the court would be willing to imply a forfeiture provision when none was expressly set forth in
the deed.

Here, Owner conveyed Blackacre to School “if School uses Blackacre only to teach children aged 5 to
13.” The language of the deed does not conform to the paradigms for creating either a fee simple
determinable or a fee simple on condition subsequent. To create a fee simple determinable, the typical
formulation would be “to School, so long as it uses Blackacre only to teach children aged 5 to 13.” To
create a fee simple on condition subsequent, the typical formulation would be “to School, but if School
does not use Blackacre only to teach children 5 to 13, then the grantor may reenter and reclaim
Blackacre.”

In this case, the deed language is ambiguous. The word “if” in the deed expresses the language of
condition and suggests an intent to create a fee simple on condition subsequent, but the deed’s lack of an
express, retained power of termination suggests the intention to instead create a fee simple determinable.
On the other hand, the deed’s failure to use the typical formulation for creating a fee simple determinable,
coupled with the absence of any language suggesting that Owner intended to retain a possibility of
reverter (e.g., “Blackacre shall revert”), suggests that Owner did not intend to convey a fee simple
determinable, but instead a fee simple on condition subsequent.

Where the terms of a conveyance are ambiguous, courts construe the instrument to effectuate the grantor’s
intentions. In construing an ambiguous instrument, courts typically adopt a preference for the fee simple
on condition subsequent.

Here, it doesn’t matter whether the court finds that the Owner-to-School deed created a fee simple
determinable or fee simple on condition subsequent if, in the latter case, it also implies a forfeiture
provision. Either construction would allow Owner to regain possession of Blackacre if School ceased
using Blackacre only to teach children aged 5 to 13. The only practical difference that flows from these
two constructions is that the fee simple on condition subsequent would require Owner (or his successors)
to first make a demand, and the fee simple determinable would not because it becomes possessory
immediately upon the happening of the limitation. In either case, the holder might have to sue the School
to enforce her rights if Blackacre is not vacated voluntarily by School.

If, however, the court construes the deed as creating a fee simple on condition subsequent and does not
imply a power of termination, then School has what amounts to a fee simple absolute because no one has
the power of termination or right of entry for condition broken. In this case, School’s decision to
discontinue use of Blackacre for teaching children aged 5 to 13 has no effect on School’s interest in
Blackacre.

Seperac-J19 Exam-Released MEE Essay Compilation © 2016-2020 476


[NOTE: Applicants who conclude that School has a fee simple on condition subsequent may fail to realize
that a right of entry must be expressly reserved in the deed and, thus, fail to discuss a court’s reluctance
to find a forfeiture.]

[NOTE: An applicant’s conclusion on how the deed should be construed is much less important than
demonstrating an understanding of the consequences that flow from the various constructions.]

Explanation to Point-Two (10%):

School has likely ceased using Blackacre “only to teach children aged 5 to 13.” If Owner is treated as
having either a possibility of reverter or a right of entry for condition broken (power of termination), then
School’s interest can come to an end and, Blackacre passes to the successors of Owner’s estate.
Otherwise, School continues to own Blackacre.

There is a factual question as to whether ceasing to only teach children aged 5 to 13 in the building on
Blackacre and then using that building for administrative purposes is inconsistent with language in the
deed from Owner to School. Here again applicants can make an argument either way, but the stronger
argument appears to be that the terminating event occurred. No teaching appears to be going on in the
building; rather, it is being used for administrative purposes, and that use seems to be beyond the intended
purpose set forth in the Owner-School deed.

Explanation to Point-Three (10%):

If Owner’s successors have an interest in Blackacre, the statute of limitations has not run on that interest.

Whether Owner conveyed to School a fee simple determinable or a fee simple on condition subsequent,
the state statute of limitations applicable to actions for possession permits actions within “10 years after
the cause of action accrues.”

Because School ceased using Blackacre for the permitted purposes only three years ago, the period in
which an action for possession may be brought has not yet expired.

Explanation to Point-Four (35%):

Ann has an interest in Blackacre if Owner retained either a possibility of reverter or a power of
termination. In some jurisdictions, Mary (Bill’s widow) would also have such an interest. If Owner
retained only a power of termination, Ann will have to assert that right by exercising the power because
possession will not automatically inure to her as it would have if Owner had retained a possibility of
reverter.

If Owner retained a future interest in Blackacre, that interest passed to Daughter upon Owner’s death
because state law provides that such an interest is devisable. Upon Daughter’s death, Daughter’s devisable
interest passed to Husband for life, with the remainder to “my surviving children.”

Although Daughter was survived by two children, one of them, Bill, predeceased Husband. According to
the Restatement of Property, which expresses what appears to be the current majority view, a survivorship
contingency like that contained in Daughter’s will applies at the termination of the interests that precede
distribution of the remainder. Thus, under the Restatement approach, Mary has no interest in Blackacre
because Bill, having predeceased Husband, had no interest in Blackacre to devise to Mary. This approach

Seperac-J19 Exam-Released MEE Essay Compilation © 2016-2020 477


is typically justified on the ground that a testator would want an interest in property to pass only to
children who could take possession.

There is another view that interprets a survivorship contingency to require surviving only the testator
(here, Daughter) and not the life tenant (here, Husband). This view is typically justified by a preference
for early vesting of estates. If it applies, then Bill’s interest passed to Mary.

If Owner bequeathed a possibility of reverter in Blackacre, instead of a power of termination, then Ann
(and Mary, in a state that follows the non-Restatement view) is entitled to immediate possession of
Blackacre. If Owner retained and bequeathed only a power of termination, then Ann’s (and, in a non-
Restatement jurisdiction, Mary’s) interest would not become possessory until and unless the power of
termination was exercised.

Seperac-J19 Exam-Released MEE Essay Compilation © 2016-2020 478


#106-FEB 2010–MEE Q02: QUESTION TWO (REAL PROPERTY)

In 1960, Owen, the owner of vacant land, granted a power-line easement over the land to an electric
company by a properly executed written instrument. This easement was never recorded. Consistent with
the easement, the electric company erected power lines over the land. The power lines and supporting
poles remain on the land.

In 1961, Owen granted an underground gas-line easement on the land to a gas company by a properly
executed written instrument. This easement was never recorded. Consistent with the easement, the gas
company dug trenches, laid pipes, and restored the surface of the land to its pre-installation condition.

In 1970, Owen conveyed the land to Abe by a full covenant and warranty deed that made no mention of
the easements. The Owen-to-Abe deed was promptly and properly recorded. Abe paid full value for the
land and had no actual knowledge of the two easements Owen had previously granted.

In 1995, Abe conveyed the land to Bob by a full covenant and warranty deed that made no mention of the
easements. The Abe-to-Bob deed was promptly and properly recorded. Bob, who paid full value for the
land, knew of the underground gas line because he had helped dig the trenches on the land. Bob had not
visited the portion of the land crossed by the power lines and had no actual knowledge of the power-line
easement.

In 2009, Bob decided to build a house on the land and hired an engineer to evaluate the proposed building
site. Following an inspection of the proposed site, the engineer told Bob that each easement precluded
building on the site.

Relevant state statutes provide

(1) “A conveyance of real property is not valid against any subsequent purchaser who, without notice,
purchases said real property in good faith and for valuable consideration,” and

(2) “Easements by prescription are abolished.”

1. Did Bob take the land subject to the power-line easement? Explain.

2. Did Bob take the land subject to the gas-line easement? Explain.

3. Assuming Bob took the land subject to either easement, may Bob obtain damages from Owen
based upon a breach of the covenant against encumbrances? Explain.

Seperac-J19 Exam-Released MEE Essay Compilation © 2016-2020 479


#106: F10-2 MEE: ANSWER: NCBE (REAL PROPERTY)

POINT (1) [40%] ISSUE: Did Bob take the land subject to a power-line easement when the
easement was not recorded but the power lines were visible? ANSWER: Yes. Both Abe and Bob
acquired the land subject to the unrecorded power-line easement. Because the power lines were
visible, Abe and Bob were on inquiry notice of the easement.

POINT (2) [30%] ISSUE: Did Bob take the land subject to a gas-line easement when the easement
was not recorded and the gas line was not visible but Bob had actual knowledge of the gas-line
easement? ANSWER: No. Abe did not take the land subject to the unrecorded and invisible gas-line
easement because he had no actual, constructive, or inquiry notice of it. While Bob had actual
knowledge of the gas-line easement, because of the shelter doctrine, he takes free and clear of that
easement.

POINT (3) [30%] ISSUE: If Bob took the land subject to the power-line easement, may he obtain
damages from Owen based on the Owen-to-Abe warranty deed’s covenant against encumbrances?
ANSWER: Bob cannot obtain damages from Owen for breach of the covenant against
encumbrances because Bob is a remote grantee. However, some jurisdictions do not follow the
common law rule. In those jurisdictions, a remote grantee may sue on the covenant against
encumbrances.

ANSWER DISCUSSION:

Under the state recording statute, a subsequent purchaser for value takes the purchased property free of
prior unrecorded interests unless the purchaser has actual or inquiry notice of those interests. Both Abe
and Bob had inquiry notice of the power-line easement, as the power lines were clearly visible. Had they
looked, they would have discovered the power lines and been aware that they should make further inquiry
regarding the possibility of an existing easement. Thus, Abe and Bob were on notice of any interest the
electric company actually had that they could have ascertained by asking the electric company, and they
take subject to that interest. Abe had no notice of the underground gas-line easement, since it was not
visible and it was not on the record. Thus Abe took the property free and clear of the gas-line easement.
While Bob had actual knowledge of the gas-line easement, because of the shelter doctrine, Bob takes the
land free of that easement. Because Bob is a remote grantee, he cannot recover damages from Owen for
the power-line easement based on the covenant against encumbrances in the Owen-to-Abe deed.

ANSWER EXPLANATION:

Explanation to Point-One (40%):

Both Abe and Bob acquired the land subject to the unrecorded power-line easement. Because the power
lines were visible, Abe and Bob were on inquiry notice of the easement.

In this state, a “conveyance of real property is not valid against any subsequent purchaser who, without
notice, purchases said real property in good faith and for valuable consideration.” Notice may be actual,
constructive, or inquiry. Constructive notice comes from information that is on the public land records;
inquiry notice arises from facts discernible through visual inspection of the premises or the applicable
recorded instruments. Some jurisdictions and authors equate constructive and inquiry notice.

Seperac-J19 Exam-Released MEE Essay Compilation © 2016-2020 480


Here, because Abe paid Owen for the land, he is a subsequent purchaser for value. Abe did not have
actual notice of the power-line easement and, because that easement was never recorded, he did not have
constructive notice of it based on the recorded instruments. However, because the power lines were
discernible from visual inspection, Abe had inquiry notice of the power-line easement. Therefore, under
the relevant statute, Abe took subject to the power-line easement.

The same analysis applies to Bob, who also takes subject to the power-line easement.

Explanation to Point-Two (30%):

Abe did not take the land subject to the unrecorded and invisible gas-line easement because he had no
actual, constructive, or inquiry notice of it. While Bob had actual knowledge of the gas-line easement,
because of the shelter doctrine, he takes free and clear of that easement.

The gas-line easement was never recorded. Thus neither Abe nor Bob acquired constructive notice of this
easement from the land records. Abe had no actual notice of it, either. In contrast to the power-line
easement, the gas-line easement was not discernible through visual inspection; the gas lines were
underground, and the surface of the land had been restored to its pre-installation condition. Therefore,
Abe had no inquiry notice of the gas-line easement. Thus, under the state recording statute, Abe took free
of the gas-line easement.

At first blush, it would appear that Bob took the land subject to the gas-line easement because, in contrast
to Abe, he had actual notice of it. However, under the “shelter doctrine,” when a bona fide purchaser
(here, Abe) acquires title free of a prior encumbrance, he can convey that title to a subsequent purchaser
(here, Bob) free of that encumbrance. In order to ensure that the bona fide purchaser has an unlimited
right to alienate his land in the future, the shelter doctrine applies even when the subsequent purchaser has
actual notice of the prior, unrecorded encumbrance.

Explanation to Point-Three (30%):

Bob cannot obtain damages from Owen for breach of the covenant against encumbrances because Bob is
a remote grantee. However, some jurisdictions do not follow the common law rule. In those jurisdictions,
a remote grantee may sue on the covenant against encumbrances.

Owen conveyed the land to Abe with a full covenant and warranty deed that made no mention of
encumbrances. A full-covenant deed includes a covenant against encumbrances, i.e., a warranty that, at
the time of conveyance, there are no outstanding third-party rights that negate the title the grantor purports
to convey. That covenant is inconsistent with the fact that Abe and later Bob took subject to the power-
line easement. Abe clearly had a cause of action against Owen for breach of the covenant against
encumbrances. The issue is whether Bob may also sue Owen.

Under the common law, the covenant against encumbrances is a “present covenant,” breached, if at all, if
there is an encumbrance at the time of the conveyance to Abe. Furthermore, the covenant does not run
with the land. Therefore, it cannot benefit a remote grantee like Bob.

Some jurisdictions do not follow the common law rule. In those jurisdictions, a remote grantee may sue
on the covenant against encumbrances. Even in those jurisdictions, a remote grantee with notice of the
easement may not sue on the theory that with such notice the grantee (1) never relied on the covenant or
(2) bargained for a reduction in the purchase price to take account of the easement. Courts are divided on
whether the covenant against encumbrances is breached when an unrecorded easement is ascertainable

Seperac-J19 Exam-Released MEE Essay Compilation © 2016-2020 481


through visual inspection. Some courts say yes; others disagree, arguing that the grantee can sue even
though the easement is visible because, if the warranty is in the deed, the grantee can reasonably assume
that an easement is no longer valid when the grantor makes no exception for it when conveying title.

Seperac-J19 Exam-Released MEE Essay Compilation © 2016-2020 482


#107-FEB 2009–MEE Q04: QUESTION FOUR (REAL PROPERTY)

Several years ago, Parent, the record owner of a farm in fee simple absolute, conveyed the farm as a gift
“jointly in fee to my beloved daughters, Jessica and Karen, equally, to share and share alike.” Parent
delivered the deed to Jessica and Karen. The deed was never recorded.

Two years ago, Jessica borrowed $60,000 from Credit Union, securing the loan by granting Credit Union
a mortgage on her interest in the farm. Credit Union properly and promptly recorded the mortgage.

Six months ago, Jessica validly contracted to sell her one-half interest in the farm for $90,000 to Buyer,
who was very anxious to acquire Jessica’s interest. Buyer paid Jessica $40,000 as earnest money and
agreed in the contract to accept a deed with no warranties of any kind and to accept the title regardless of
whether title was marketable. Buyer had no actual notice of the mortgage Jessica had granted to Credit
Union.

Two months ago, before closing the sale with Buyer, Jessica died, survived by Karen. At the time of
Jessica’s death, the loan secured by Credit Union’s mortgage was still outstanding. Jessica’s will
provided: “I give all of my real property to Devisee and all of my personal property to Legatee.” Both
Devisee and Legatee survived Jessica.

Last month, the executor of Jessica’s estate executed a deed purporting to convey a one-half interest in the
farm to Buyer in exchange for the balance of the purchase price.

The jurisdiction has a notice-type recording statute and a grantor-grantee index system.

1. Did Parent convey the farm to Jessica and Karen as “tenants in common” or as “joint tenants
with right of survivorship”? Explain.

2. Assuming Jessica and Karen acquired a joint tenancy with right of survivorship in the farm, what
are the rights, if any, of Karen, Credit Union, and Buyer in the farm? Explain.

3. Assuming Jessica and Karen acquired a joint tenancy with right of survivorship in the farm, who
is entitled to the balance of the purchase price Buyer paid the executor of Jessica’s estate? Explain.

Seperac-J19 Exam-Released MEE Essay Compilation © 2016-2020 483


#107: F09-4 MEE: ANSWER: NCBE (REAL PROPERTY)

POINT (1) [25%] ISSUE: Does a deed conveying property “jointly in fee” to two daughters
“equally, to share and share alike” create a tenancy in common or a joint tenancy with right of
survivorship in the daughters? ANSWER: Parent’s deed of gift to Jessica and Karen created a
tenancy in common unless the language in the deed overcame the statutory presumption that a
conveyance to two or more persons creates a tenancy in common. Whether or not it does here is a
close question. While the word “jointly” itself may not overcome the presumption, when used in
conjunction with the phrase “share and share alike,” it might.

POINT (2) [30%] ISSUE: Assuming a joint tenancy with right of survivorship was created in the
two daughters, was it severed when one of the two joint tenants granted a mortgage on, and entered
into a contract for the sale of, the farm? ANSWER: If the deed of gift created a joint tenancy,
Jessica’s mortgage severed that joint tenancy as to her one-half interest if the state follows the “title
theory” instead of the “lien theory” of mortgages. In any event, Jessica’s contract of sale severed
the joint tenancy. Therefore, Karen had no right of survivorship and the deed from the executor to
Buyer caused Karen and Buyer to acquire a title to the farm as tenants in common.

POINT (3) [25%] ISSUE: Is a buyer subject to a prior recorded mortgage when the buyer has no
actual notice of the mortgage and the previous deed in the chain of title to the seller was
unrecorded? ANSWER: Yes. Buyer cannot qualify as a bona fide purchaser because Buyer had
constructive notice of Credit Union’s recorded mortgage. The failure to record the deed of gift from
Parent to Jessica and Karen should not impair the mortgage because the non-recording of that deed
would not interfere with Buyer’s ability to find the mortgage during a title search. Thus, Buyer
takes the farm subject to Credit Union’s interest.

POINT (4) [20%] ISSUE: Does a deceased seller’s interest in the proceeds of sale due under an
executory real estate contract pass to the beneficiary of the real or personal property under the
deceased seller’s will? ANSWER: When Jessica entered into the contract of sale, the doctrine of
equitable conversion transformed her interest into personalty. As a result, Devisee acquired no
interest in the farm under Jessica’s will. Rather, the sales proceeds pass to Legatee.

ANSWER DISCUSSION:

The language in Parent’s deed to Jessica and Karen probably is sufficient to overcome the statutory
presumption that a conveyance to two or more persons creates a tenancy in common. Thus, Jessica and
Karen probably became joint tenants in the farm. At Jessica’s death her interest would terminate and
Karen would own the farm in fee unless prior to her death Jessica severed the joint tenancy. Whether the
mortgage to Credit Union, executed only by Jessica, severs the joint tenancy depends upon whether the
farm is located in a lien- or title-theory jurisdiction. However, even if the farm is located in a lien-theory
jurisdiction where a mortgage would not sever the joint tenancy, when Jessica entered into a contract to
sell Buyer her interest in the farm, she likely severed the joint tenancy and converted it into a tenancy in
common. Thus, once the executor deeded Jessica’s interest to Buyer, Karen and Buyer owned the farm as
tenants in common. Credit Union also has a mortgage on one-half of the farm. Because Credit Union
recorded the mortgage before Buyer entered into the contract to purchase Jessica’s interest, Buyer had
constructive notice from the record of Credit Union’s interest created by Jessica and takes subject to that
mortgage even though the deed to Jessica was not recorded. Because of the doctrine of equitable

Seperac-J19 Exam-Released MEE Essay Compilation © 2016-2020 484


conversion, Jessica’s interest in the farm at her death is characterized as personalty and passes to Legatee
under Jessica’s will.

ANSWER EXPLANATION:

Explanation to Point-One (20-30%):

Parent’s deed of gift to Jessica and Karen created a tenancy in common unless the language in the deed
overcame the statutory presumption that a conveyance to two or more persons creates a tenancy in
common. Whether or not it does here is a close question. While the word “jointly” itself may not
overcome the presumption, when used in conjunction with the phrase “share and share alike,” it might.

A deed to two or more grantees may create a joint tenancy or a tenancy in common. To create a joint
tenancy, the common law traditionally required the existence of four unities (time, title, interest, and
possession). Under the four-unities test, a joint tenancy presumptively is created by a conveyance to two
or more persons if they acquire their interest at the same time, acquire their interest under the same
instrument, acquire an equal interest in the property, and acquire the right to possession of the property.

The four unities are satisfied by Parent’s deed to the daughters, so at common law they clearly took title
as joint tenants with right of survivorship. With a joint tenancy, each joint tenant has the right of
survivorship.

Today, in most states, there is a statutory presumption that a conveyance to two or more persons creates a
tenancy in common rather than a joint tenancy. Thus, if the presumption is not rebutted, Jessica and Karen
took as tenants in common. Tenants in common have no right of survivorship.

The standard way to overcome the presumption favoring a tenancy in common is for the deed to use the
term “joint tenancy” or “joint tenants,” usually also adding an express reference to “survivorship” or
“survivors,” and it can be argued that where such language is absent the statutory presumption is not
rebutted.

Although survivorship language is missing in Parent’s deed to Jessica and Karen, the language of the deed
could still be construed to overcome the statutory presumption favoring a tenancy in common. For
example, some cases hold that the word “jointly” standing alone rebuts the presumption favoring a
tenancy in common, although there are contrary cases holding that a grantor who uses the word “jointly”
in the deed may intend that the grantees “own together” rather than that they own as joint tenants with the
right of survivorship. Here, however, Parent’s deed not only used the word “jointly” but added the phrase
“equally, to share and share alike.” This phrase, in common with the word “jointly,” may point toward a
joint tenancy as it evidences an intent by Parent to give each daughter an equal interest in the farm,
another hallmark of a joint tenancy but not essential for a tenancy in common. This additional phrase also
reflects the historic conception that joint tenants hold as a unit rather than as separate individuals.

[NOTE: This is a close question, and applicants should receive equal credit for reasoned analyses that
consider the possibilities, whether they conclude that the deed creates a tenancy in common or a joint
tenancy. However, an applicant’s conclusion should not affect the rest of the analysis because the
remaining calls assume Jessica and Karen took from Parent as joint tenants with right of survivorship.]

Lastly, a deed is effective between the parties even if it is not recorded. Thus, the daughters acquired a
tenancy in common or a joint tenancy, notwithstanding their failure to record their deed of gift.

Seperac-J19 Exam-Released MEE Essay Compilation © 2016-2020 485


Explanation to Point-Two (25-35%):

If the deed of gift created a joint tenancy, Jessica’s mortgage severed that joint tenancy as to her one-half
interest if the state follows the “title theory” instead of the “lien theory” of mortgages. In any event,
Jessica’s contract of sale severed the joint tenancy. Therefore, Karen had no right of survivorship and the
deed from the executor to Buyer caused Karen and Buyer to acquire a title to the farm as tenants in
common.

When property is held by two persons in joint tenancy, a conveyance by one joint tenant of her entire
ownership interest severs the joint tenancy as to the conveyed share. This is because the conveyance
severs at least the unities of time and title between the remaining co-tenant and the new co-tenant.

When, as here, a joint tenant transfers a lesser interest, such as a mortgage, a severance of the joint
tenancy may also occur. Courts usually resolve the issue by trying to decide whether the transfer
destroyed any of the four unities.

In the case of mortgages, some states follow the “title theory,” which says that the mortgagee takes title to
the property for the duration of the mortgage. In “title theory” states, a mortgage granted by one joint
tenant severs the joint tenancy as to the conveyed share. This would convert the joint tenancy into a
tenancy in common. Other states follow the “lien theory” of mortgages, which says that the mortgagor
retains title and the mortgagee takes only a lien on the property. This would leave the four unities intact,
and thus Jessica would remain a joint tenant with her sister.

If Jessica’s grant of the mortgage did not sever the joint tenancy, her contract to sell her interest in the
farm to Buyer almost certainly had that effect. Although Jessica retained legal title until the contract
closed, under the four unities analysis she no longer had the same interest as her sister. This follows from
the doctrine of equitable conversion because her interest (but not Karen’s interest) is now subject to
Buyer’s equitable interest or title. Because the joint tenancy was severed, Karen had no right of
survivorship and, therefore, did not become the sole owner of the farm at Jessica’s death.

[NOTE: Many applicants may not address the distinction between the title- and lien-theory jurisdictions
and should not be penalized for failing to do so. What is important is that applicants recognize the legal
issue and come to a resolution. Furthermore, without regard to the mortgage, applicants should address
the effect of the contract with Buyer. Also, if a jurisdiction does not recognize the doctrine of equitable
conversion, the execution of the contract would not sever the joint tenancy.]

Since the joint tenancy was severed, the deed from the executor to Buyer caused Karen and Buyer to hold
the farm as tenants in common.

Explanation to Point-Three (20-30%):

Buyer cannot qualify as a bona fide purchaser because Buyer had constructive notice of Credit Union’s
recorded mortgage. The failure to record the deed of gift from Parent to Jessica and Karen should not
impair the mortgage because the non-recording of that deed would not interfere with Buyer’s ability to
find the mortgage during a title search. Thus, Buyer takes the farm subject to Credit Union’s interest.

Assuming Buyer acquired an interest from Jessica, Buyer may claim to be a bona fide purchaser, who
should take free of Credit Union’s mortgage. To qualify as a bona fide purchaser, a person must (i) pay
value for an interest and (ii) not have actual, inquiry, or constructive notice of the competing prior-in-time
interest.

Seperac-J19 Exam-Released MEE Essay Compilation © 2016-2020 486


Although Buyer paid value, and lacked actual notice of the mortgage, Buyer had constructive notice of the
mortgage because the mortgage was properly recorded before Buyer entered into the contract to buy
Jessica’s interest. Had Buyer made a proper title search by looking in the grantor-grantee index for all of
Jessica’s transactions as a grantor, the mortgage would have been discovered. Therefore, Buyer takes
subject to Credit Union’s interest.

[NOTE: The mortgage is not a “wild deed” that could be deemed unrecorded because it is outside of the
buyer’s chain of title or recorded out of sequence (too early or too late). Since the facts state that the
grantor-grantee index operates in this jurisdiction, if Buyer had searched under Jessica’s name as
grantor, Buyer would have found her mortgage to Credit Union. In the typical wild deed case, the prior
interest outside of the chain of title is created by someone other than the buyer’s grantor.]

[NOTE: As an aside, Buyer’s search would also reveal the fact that Jessica and her sister lacked record
title because they never recorded the deed of gift from Parent. However, here that would not be a title
objection because Buyer agreed to accept a title that was not marketable.]

Explanation to Point-Four (15-25%):

When Jessica entered into the contract of sale, the doctrine of equitable conversion transformed her
interest into personalty. As a result, Devisee acquired no interest in the farm under Jessica’s will. Rather,
the sales proceeds pass to Legatee.

The doctrine of equitable conversion splits title to the property when a real estate contract is signed. Buyer
obtained equitable title, and Jessica as seller retained legal title as trustee to secure payment of the
remainder of the purchase price. Equitable conversion only applies to a contract that is specifically
enforceable; here there are no facts suggesting that Jessica or Buyer would be unable to obtain specific
performance. Buyer cannot reject title because of the outstanding mortgage as Buyer agreed to accept a
title without any warranties and regardless of its marketability. When equitable conversion applies, the
seller’s legal title is considered personal property, and the buyer’s equitable title is considered real
property. When Jessica died, her share passed to Legatee, who took personal property under Jessica’s
will.

[NOTE: The preceding analysis depends upon Jessica’s share prior to her death being classified as a
tenancy in common because (1) the mortgage severed Jessica’s joint tenancy share, or (2) the contract of
sale to Buyer severed Jessica’s joint tenancy share. If, on the other hand, Jessica died still owning a one-
half interest as joint tenant, no part of the farm or its proceeds passed to Legatee or Devisee under
Jessica’s will. Rather, by right of survivorship, Karen owned the entire farm free and clear of the rights of
Credit Union and Buyer because their interests, being wholly derivative from Jessica, would also expire
at Jessica’s death.]

Seperac-J19 Exam-Released MEE Essay Compilation © 2016-2020 487


#108-JUL 2008–MEE Q04: QUESTION FOUR (REAL PROPERTY)

Two years ago, Landlord and Tenant entered into a five-year oral lease of an office at $800 per month,
payable on the first day of each month. The lease was silent regarding Tenant’s right to assign or sublet
the office.

Three weeks ago, Tenant called Landlord, gave two weeks’ notice of lease termination, and said that she
planned to move out of state permanently.

Tenant told Landlord that Friend had agreed to take over the office space for the balance of the five-year
lease. Landlord said that he would not accept Friend as a new tenant even though she was financially
capable of paying the rent and that he would hold Tenant to the lease agreement for the balance of the
five-year term.

Two weeks ago, Tenant vacated the office.

One week ago, on the day the next month’s rent payment was due, Friend sent Landlord a check for $800,
which Landlord refuses to cash. Landlord insists that Tenant is liable for the $800 rent. Tenant has not
paid the $800 monthly rent. Landlord has sued Tenant to recover the $800 rent that she has not paid.

1. What type of tenancy did the oral lease agreement between Landlord and Tenant create? Explain.

2. Did Tenant properly terminate the tenancy? Explain.

3. Is Landlord entitled to collect $800 from Tenant? Explain.

Seperac-J19 Exam-Released MEE Essay Compilation © 2016-2020 488


#108: J08-4 MEE: ANSWER: NCBE (REAL PROPERTY)

POINT (1) [40%] ISSUE: What type of tenancy did the oral lease between Landlord and Tenant
create? ANSWER: The oral five-year lease between Landlord and Tenant violated the Statute of
Frauds. Because Tenant took possession two years ago and Landlord accepted monthly rent
payments from Tenant throughout that time, a periodic month-to-month tenancy was created.

POINT (2) [20%] ISSUE: Did Tenant properly terminate the tenancy? ANSWER: No. A periodic
month-to-month tenancy is terminable by notice given at least one month prior to the termination
date. Here, the tenancy was not properly terminated because Tenant gave Landlord only two
weeks’ notice.

POINT (3) [40%] ISSUE: Is Landlord entitled to collect $800 from Tenant? ANSWER: No.
Because the lease did not prohibit an assignment, Tenant was free to assign the lease to Friend.
Because Friend properly took an assignment of the office lease from Tenant and timely paid the
$800 rent to Landlord, Landlord has no claim against Tenant for unpaid rent.

ANSWER DISCUSSION:

Under the Statute of Frauds, a lease for a term of more than one year (in some states, three years) must be
in writing or its electronic equivalent. When a tenant enters into an oral lease that violates the Statute of
Frauds and the landlord accepts rent from the tenant, a periodic tenancy ensues if, as here, the tenant has
been in possession and paying rent for a substantial period. Either party can terminate a periodic tenancy
by giving proper notice. A proper termination notice provides notice for a period at least equal to the
payment period specified in the lease agreement. Here, the payment period was one month, and Tenant’s
two-week notice was thus improper. As a result, Tenant still owes rent to Landlord. However, because the
lease did not prohibit an assignment, Tenant’s interest was freely assignable; Landlord’s consent was not
required. Because Tenant properly assigned her lease to Friend, who timely paid the $800 rent due to
Landlord, Landlord cannot recover $800 from Tenant.

ANSWER EXPLANATION:

Explanation to Point-One (35-45%):

The oral five-year lease between Landlord and Tenant violated the Statute of Frauds. Because Tenant took
possession two years ago and Landlord accepted monthly rent payments from Tenant throughout that
time, a periodic month-to-month tenancy was created.

Most states have enacted a Statute of Frauds that requires leases of more than one year (in some states,
three years) to be in writing. A lease subject to the Statute of Frauds is voidable until the tenant takes
possession and the landlord accepts rent from the tenant. If the tenant takes possession and the landlord
accepts rent, an at-will or periodic tenancy is created because there has been partial performance.

Here, Tenant took possession of the office and Landlord accepted rent. Thus, a periodic or at-will tenancy
was created.

An at-will tenancy may be terminated without notice; a periodic tenancy requires notice for a period at
least equal to the rent-payment term. Where, as here, rent has been paid for a substantial period of time,

Seperac-J19 Exam-Released MEE Essay Compilation © 2016-2020 489


the tenancy is generally classified as periodic because payment over time evidences a commitment to the
arrangement and creates expectations that justify notice of termination. Assuming that a periodic tenancy
was created, most likely it was a month-to-month tenancy because the lease provided for monthly
payments of rent.

Explanation to Point-Two (15-25%):

A periodic month-to-month tenancy is terminable by notice given at least one month prior to the
termination date. Here, the tenancy was not properly terminated because Tenant gave Landlord only two
weeks’ notice.

A periodic month-to-month tenancy can be terminated by either party with a one-month notice of
termination. At common law, an oral notice was sufficient. Today, many states require written notice of
termination. For example, California requires written notice minimum of 30 days before the date specified
in notice.

Here, Tenant gave Landlord a two-week notice of termination instead of the required one-month notice.
Thus the notice to terminate was ineffective to terminate the month-to-month tenancy as of the beginning
of the next month even if Tenant’s oral notice of termination was acceptable under state law.

Explanation to Point-Three (35-45%):

Because the lease did not prohibit an assignment, Tenant was free to assign the lease to Friend. Because
Friend properly took an assignment of the office lease from Tenant and timely paid the $800 rent to
Landlord, Landlord has no claim against Tenant for unpaid rent.

Where a periodic tenancy arises, the terms of the unwritten lease, except the provision relating to the lease
term, are enforceable. In the absence of a contrary provision in a lease, a tenant’s interest, like other
property interests, is assignable. The assignee of a month-to-month tenant takes a month-to-month
tenancy.

When a tenant assigns his interest in a lease, privity of estate arises between the assignee and the landlord.
Privity of contract continues to exist between the landlord and the tenant. The tenant is still contractually
bound to pay rent, and thus serves as a surety for unpaid rent; if the assignee fails to pay the rent, the
tenant is liable to the landlord. Here, because the assignee, Friend, paid the rent to Landlord when Friend
sent Landlord the $800 check, no additional rent was due and Tenant is not liable to Landlord for $800.

Seperac-J19 Exam-Released MEE Essay Compilation © 2016-2020 490


#109-JUL 2007–MEE Q06: QUESTION SIX (REAL PROPERTY)

Owen owned vacant land (Whiteacre) in State B located 500 yards from a lake and bordered by vacant
land owned by others. Owen, who lived 50 miles from Whiteacre, used Whiteacre for cutting firewood
and for parking his car when he used the lake.

Twenty years ago, Owen delivered to Abe a deed that read in its entirety:

Owen hereby conveys to the grantee by a general warranty deed that parcel of vacant land in State B
known as Whiteacre.

Owen signed the deed immediately below the quoted language and his signature was notarized. The deed
was never recorded.

For the next eleven years, Abe seasonally planted vegetables on Whiteacre, cut timber on it, parked
vehicles there when he and his family used the nearby lake for recreation, and gave permission to friends
to park their cars and recreational vehicles there. He also paid the real property taxes due on the land,
although the tax bills were actually sent to Owen because title had not been registered in Abe’s name on
the assessor’s books. Abe did not build any structure on Whiteacre, fence it, or post no-trespassing signs.

Nine years ago, Abe moved to State C. Since that time, he has neither used Whiteacre nor given others
permission to use Whiteacre, and to all outward appearances the land has appeared unoccupied.

Last year, Owen died intestate leaving his daughter, Doris, as his sole heir. After Owen’s death, Doris
conveyed Whiteacre by a valid deed to Buyer, who paid fair market value for Whiteacre. Neither Doris
nor Buyer knew of the Owen-to-Abe deed. Both Doris and Buyer believed that Owen was the owner of
Whiteacre at the time of his death. Buyer promptly and properly recorded the deed from Doris and
immediately went into possession of Whiteacre.

Last month Abe returned to State B. When he discovered Buyer in possession of Whiteacre, he sued
Buyer for possession.

State B has enacted the following statutes:

1. Actions to recover possession of real property shall be brought within ten years after the cause of
action accrues.

2. No conveyance or mortgage of real property shall be good against subsequent purchasers for value and
without notice unless the same be recorded according to law.

Who is entitled to possession of Whiteacre? Explain.

Seperac-J19 Exam-Released MEE Essay Compilation © 2016-2020 491


#109: J07-6 MEE: ANSWER: NCBE (REAL PROPERTY)

POINT (1) [29%] ISSUE: Did Owen convey a fee simple absolute to Abe by the Owen-to-Abe deed?
ANSWER: No. Abe did not acquire title to Whiteacre by deed from Owen because that deed failed
to specify the name of the grantee.

POINT (2) [38%] ISSUE: Did Abe acquire title to Whiteacre by adverse possession? ANSWER:
Yes. Abe may succeed in establishing that he acquired title to Whiteacre by adverse possession.

POINT (3) [33%] ISSUE: Did Buyer acquire title from Doris that was superior to Abe’s title?
ANSWER: If Abe acquired title to Whiteacre by adverse possession, his claim to possession is good
against a subsequent purchaser for value like Buyer despite his failure to record his interest. If Abe
did not acquire title by adverse possession, Buyer’s deed is controlling because Buyer had neither
actual nor constructive notice of Abe’s interest in Whiteacre when he purchased the property.

ANSWER DISCUSSION:

A valid deed requires that the grantee be named with specificity. Because the Owen-to-Abe deed did not
name a grantee, it was ineffective to convey the land to Abe. Although Abe did not acquire title to
Whiteacre by deed from Owen, he may have acquired title by adverse possession. This would depend on
whether the court finds that Abe’s acts were sufficiently open, notorious, exclusive, continuous, and under
claim of right to warrant the conclusion that he had acquired title by adverse possession. If Abe acquired
title to Whiteacre by adverse possession, his claim to possession is good against a subsequent purchaser
for value like Buyer. If Abe did not acquire title by adverse possession either because an applicant
wrongfully believes the Owen-to-Abe deed was valid or because Abe’s acts are construed as insufficient
to acquire a title by adverse possession, then Buyer’s deed is controlling because he is a bona fide
purchaser for value and had neither constructive nor inquiry notice of Abe’s interest in Whiteacre.

ANSWER EXPLANATION:

Explanation to Point-One (25-35%):

Abe did not acquire title to Whiteacre by deed from Owen because that deed failed to specify the name of
the grantee.

To be valid, a deed must identify the buyer and the seller, describe the land subject to the conveyance,
contain words denoting a present intent to convey, and be signed by the grantor. Here, the deed from
Owen to Abe read in its entirety: “Owen hereby conveys to the grantee by a general warranty deed that
parcel of vacant land in State B known as Whiteacre. Signature of Owen.” The Owen-to-Abe deed names
the seller, describes the land, sets forth a present intent to convey to a grantee, and contains the grantor’s
signature, but it does not contain the name of the grantee. Therefore Owen did not effectively convey the
land to Abe. Although some courts have upheld deeds when the grantee’s name is missing, these cases
involve deeds containing language providing a method for ascertaining the grantee’s identity, such as “to
the grantor’s son” or to the “trustee of a trust.” No such language appears here.

[NOTE: If an applicant does not know the rule that a deed must name the grantee, the applicant should
conclude the Owen-to-Abe deed is valid. If the applicant wrongly concludes that the deed is valid, then

Seperac-J19 Exam-Released MEE Essay Compilation © 2016-2020 492


the applicant might either completely skip Point Two, or, in discussing Point Two, conclude that Abe did
not acquire a title by adverse possession because Abe’s possession was not hostile.]

Explanation to Point-Two (35-45%):

Abe may succeed in establishing that he acquired title to Whiteacre by adverse possession.

An individual may acquire title to land by adverse possession. To do so, the claimant must show that he
entered the land and remained in actual possession that was open, continuous, exclusive, and under claim
of right during the entire period in which the record owner (here, Owen) had a cause of action for
possession. In State B, a cause of action for possession is available for ten years after a possessor’s entry.

Whether the possessor has acquired title by adverse possession can only be conclusively determined by a
judicial proceeding, and the sufficiency of particular acts of possession is not always clear. In most states,
it is not necessary that the possessor have paid taxes. In a few states, a possessor cannot claim title by
adverse possession without paying taxes. The possessor need not possess the land to its highest and best
use; the possession must be consistent with how a true owner would have possessed the land.

Abe has a strong case for adverse possession. His acts of possession were open in the sense that they were
observable by anyone who looked. They continued for eleven years. His grants to others of permission to
use the land suggest exclusivity. He paid taxes, although in most states this is not essential to establish a
title by adverse possession. Lastly, his claim of right or hostility is evidenced by his entry under an invalid
deed. It is irrelevant that he thought he was there rightfully under a valid deed because, in fact, the deed
was invalid, and Owen could have ejected him at any time during the 10-year statutory period. Abe’s acts
of possession endured for eleven years, one year more than required to acquire a title by adverse
possession in State B. His possession was also more active than Owen’s, supporting the claim that it was
consistent with how a true owner would have possessed the land. The fact that Abe did not build
structures on the land, fence it, or post no-trespassing signs should not adversely affect his claim given
that Owen, the previous owner, appears to have acted in the same way.

[NOTE: An applicant’s conclusion is less important than his or her demonstrated mastery of the relevant
legal principles and ability to utilize the facts.]

Explanation to Point-Three (30-40%):

If Abe acquired title to Whiteacre by adverse possession, his claim to possession is good against a
subsequent purchaser for value like Buyer despite his failure to record his interest. If Abe did not acquire
title by adverse possession, Buyer’s deed is controlling because Buyer had neither actual nor constructive
notice of Abe’s interest in Whiteacre when he purchased the property.

If Abe acquired title by adverse possession, that title is as good as a title traceable to a prior record owner;
it is not lost because he allowed the property to go unoccupied. Thus if Abe acquired title by adverse
possession, he is entitled to recover possession from Buyer.

The State B recording statute does not alter this result. Under the statute, a subsequent bona fide purchaser
prevails against a prior donee or purchaser if the purchaser acquired the property without knowledge of
the interest of the prior donee or purchaser. Statutes like this aim to protect subsequent purchasers against
interest holders who could – but have failed to – record the documents describing their interests in the
local land records office. In such a case, equity clearly favors the subsequent purchaser who took without
notice of the prior interest. However, in the case of title acquired by adverse possession, there is no

Seperac-J19 Exam-Released MEE Essay Compilation © 2016-2020 493


document that the interest holder could record. Courts have thus held that title acquired by adverse
possession cannot be defeated by a later conveyance from the prior record title owner even if the land is
vacant at the time the buyer purchases the land.

If Abe did not acquire title by adverse possession, then Buyer acquired title to the property from Doris,
Owen’s successor-in-interest. In that case, Buyer is entitled to possession because, given Abe’s absence at
the time Buyer purchased the property, Buyer had no actual or constructive notice of Abe’s interest in the
land.

Seperac-J19 Exam-Released MEE Essay Compilation © 2016-2020 494


TORTS: 10 OF 24 MEE EXAMS: (42%)
#110-FEB 2019–MEE Q01: QUESTION ONE (TORTS)

One year ago, a man was injured when the car in which he and a woman were traveling slid off an icy
highway during a winter storm and overturned. At the time of the accident, the woman was driving the
car. The man was sitting in the front passenger seat, wearing his seat belt. The woman was driving 40
mph at the time of the accident, although the posted speed limit was 50 mph.

The man and the woman were rushed to a local hospital in its ambulance. There, hospital surgeons
performed emergency surgery on the man. The man remained in the hospital for 10 days following his
admission. Numerous medical instruments were used during his surgery and subsequent hospitalization,
including needles, clamps, and surgical tools. However, he did not receive a blood transfusion or any
blood products.

Three days after the man was released from the hospital, he developed a fever and visited his personal
physician, who is not affiliated with the hospital. The physician ordered routine blood tests. The tests
revealed that the man had a serious infection that is transmitted in nearly all cases through exposure to
either contaminated blood products or improperly sterilized medical instruments (needles, clamps,
surgical tools, etc.) that come into contact with a patient's blood. There are, however, other possible
sources of the infection in a hospital environment, such as a failure of staff to follow proper handwashing
techniques to avoid transmitting infection from one patient to another and staff failure to properly identify
and discard certain used medical instruments that cannot safely be sterilized.

Infections occurring in individuals who have not received a blood product and have not been hospitalized
during the period of likely exposure are possible but rare. The physician told the man that he "must have
contracted this infection at the hospital" because the period between infection and symptom development
is 10 to 13 days and the man was a patient at the hospital during the entire relevant period. The physician
also stated that "at hospitals that have adopted medical-instrument sterilization procedures recommended
by experts, cases of this infection have been almost completely eliminated." The man has no history of
intravenous drug use, and he did not receive any medical treatment for several months before his hospital
stay. All sterilization procedures at the hospital are performed by hospital employees. However, the
particular sterilization procedure used while the man was hospitalized cannot be determined because,
while the hospital now uses the sterilization procedure recommended by experts, there is no record of
when it started using that procedure.

The man has sued the woman and the hospital, alleging negligence. Neither defendant is judgment-proof,
and this jurisdiction has no automobile-guest statute. The parties have stipulated that the man's damages
for the injuries he suffered in the accident are $100,000 and his damages from the infection he contracted
are $250,000.

1. Could a court properly find that the woman was negligent even though she was driving below
the posted speed limit? Explain.

2. Could a court properly find that the woman is liable for the man's damages resulting from the
infection? Explain.

Seperac-J19 Exam-Released MEE Essay Compilation © 2016-2020 495


3. Could a court properly find that the hospital is liable for the man's damages resulting from the
infection? Explain.

4. If a court found that both the woman's negligence and the hospital's negligence caused the man's
infection, could the woman's liability be limited to $100,000 for injuries the man suffered in the
accident? Explain.

Seperac-J19 Exam-Released MEE Essay Compilation © 2016-2020 496


#110: F19-1 MEE: ANSWER: NCBE (TORTS)

POINT (1) [20%] ISSUE: Could a court properly find that the woman was negligent even though
she was driving below the speed limit? ANSWER: Yes. Because compliance with a statutory
standard does not insulate an actor against liability for negligence, the woman could properly be
found liable to the man despite the fact that she was driving below the posted speed limit.

POINT (2) [20%] ISSUE: Could a court properly find that the woman is liable for the man's
damages resulting from the infection? ANSWER: Yes. Because contracting the serious infection
was within the scope of the risk of negligent driving, the court could find that the woman's
negligence was the proximate cause of the man's injuries sustained as a result of contracting the
infection.

POINT (3) [40%] ISSUE: Could a court properly find that the hospital is liable for the man's
damages resulting from the infection? ANSWER: Yes. Although the man cannot directly prove
that he contracted the infection in the hospital or from a specific action by the hospital or its
employees that was negligent, the hospital could be found liable under the doctrine of res ipsa
loquitur because the man can show that (1) contracting the infection does not normally happen
without negligence, and (2) other responsible causes are sufficiently eliminated by the evidence.

POINT (4) [20%] ISSUE: If a court found that both the woman's negligence and the hospital's
negligence caused the man's infection, could the woman's liability be limited to $100,000 for injuries
the man suffered in the accident? ANSWER: No. A finding that the woman’s negligence caused the
car accident would mean that the woman is solely responsible for the $100,000 damages from the
accident and is liable for that amount. She and the hospital together will be jointly and severally
liable for the $250,000 in damages from the man’s infection. Thus, the man can collect any portion,
or all, of the $250,000 damages from the woman. Therefore, the woman’s liability for both injuries
cannot be limited to $100,000.

ANSWER DISCUSSION:

A court could properly find that the woman was negligent despite the fact that she was driving at a speed
lower than the posted limit if it concludes that her conduct was unreasonable under the circumstances.
Given the icy road conditions, a court could find that her conduct was unreasonable. Because exposure to
either negligent or non-negligent medical treatment is a foreseeable risk of negligent driving, the woman
could be found liable for the damages arising from the infection if the court concludes that the man
contracted the infection through the hospital's conduct. Although the man cannot show when or how he
contracted the infection, under the doctrine of res ipsa loquitur the man could recover damages from the
hospital if he can show that the harm he suffered (the infection) does not normally occur without
negligence and that other responsible causes, including his own conduct and that of third persons, are
sufficiently eliminated by the evidence. Here, the evidence shows that the man was in the hospital during
the entire period in which he contracted the infection, that he had no other known means of exposure, and
that the risk of infection can be almost eliminated through the hospital's use of recommended infection-
control procedures. A court thus could properly rely on the res ipsa loquitur doctrine to find the hospital
liable. If the court found that the negligence of both the woman and the hospital caused the infection, the
woman's liability must be greater than $100,000. Because the woman's negligence alone caused the car
accident, she alone would be liable for the $100,000 damages for the injuries the man suffered in the
accident. In addition, in joint and several liability jurisdictions, she and the hospital together would be

Seperac-J19 Exam-Released MEE Essay Compilation © 2016-2020 497


liable for the full amount of damages from the man's infection. Thus, her total damages for both the
accident and the infection would not be limited to $100,000.

ANSWER EXPLANATION:

Explanation to Point One (20%):

Because compliance with a statutory standard does not insulate an actor against liability for negligence,
the woman could properly be found liable to the man despite the fact that she was driving below the
posted speed limit.

Statutory standards typically establish the level of care necessary to avoid a finding of negligence. Thus,
"an actor is negligent if, without excuse, the actor violates a statute that is designed to protect against the
type of accident the actor's conduct causes, and if the accident victim is within the class of persons the
statute is designed to protect." However, an actor is negligent when he or she "does not exercise
reasonable care under all the circumstances." Speed limits are established for normal driving conditions,
not hazardous conditions caused by poor weather. Given that the accident in which the man was injured
occurred on an icy road during a winter storm, a court could find that the woman was negligent even
though she was driving at a speed lower than the posted speed limit. Compliance with a statute does not
establish freedom from fault.

Explanation to Point Two (20%):

Because contracting the serious infection was within the scope of the risk of negligent driving, the court
could find that the woman's negligence was the proximate cause of the man's injuries sustained as a result
of contracting the infection.

An actor is liable for those harms that are a foreseeable consequence of his negligence.

Courts have routinely found that subsequent medical malpractice is within the scope of the risk created by
a tort defendant. If the negligent actor is liable for another's bodily injury, he is also subject to liability for
any additional bodily harm resulting from normal efforts of third persons in rendering aid which the
other's injury reasonably requires, irrespective of whether such acts are done in a proper or a negligent
manner. Liability typically attaches even when the medical services rendered cause harm which is entirely
different from that which the other had previously sustained so long as the mistake or negligence is of the
sort which is recognized as one of the risks which is inherent in the human fallibility of those who render
such services.

Thus, because it is foreseeable that an injured person will require hospitalization and that hospitalization
will expose the injured person to other infections, the woman could be found liable for the man's damages
associated with contracting the infection so long as the trier of fact concludes that the hospital is
responsible, whether negligent or not, for the man's contracting the infection.

Explanation to Point Three (40%):

Although the man cannot directly prove that he contracted the infection in the hospital or from a specific
action by the hospital or its employees that was negligent, the hospital could be found liable under the
doctrine of res ipsa loquitur because the man can show that (1) contracting the infection does not normally
happen without negligence, and (2) other responsible causes are sufficiently eliminated by the evidence.

Seperac-J19 Exam-Released MEE Essay Compilation © 2016-2020 498


Typically, the tort plaintiff bears the burden of proof to establish the specific actions of the defendant or
its employees (acting within the scope of their employment) that were negligent and caused his harm.
Here, the plaintiff has no direct proof of the actions of the hospital or its employees that were negligent
and that caused the infection from which he is suffering.

However, the doctrine of res ipsa loquitur permits the trier of fact to infer that the harm suffered by the
plaintiff was caused by negligence of the defendant when

(a) the event is of a kind which ordinarily does not occur in the absence of negligence;

(b) other responsible causes, including the conduct of the plaintiff and third persons, are sufficiently
eliminated by the evidence; and

(c) the indicated negligence is within the scope of the defendant's duty to the plaintiff.

Res ipsa loquitur is commonly used in actions against medical providers when the patient suffers an
unexplained injury and the evidence establishes that the risk of such an injury can be largely eliminated
when reasonable care is used. If, for example, the evidence shows that a particular adverse result of
surgery is totally preventable when surgeons exercise reasonable and customary care, then res ipsa is
appropriate in the patient's suit against the surgeon.

The man should be able to show that contracting the infection is an event that normally does not occur in
the absence of negligence. A plaintiff need not show that reasonable care would completely eliminate the
risk, only that it ordinarily does not occur in the absence of negligence.

The man should also be able to show that the very likely cause of the infection is one of three
possibilities: (1) improperly sterilized instruments, (2) failure of employees to follow proper handwashing
techniques, or (3) reuse of medical instruments that cannot be properly sterilized. Any of these
possibilities would constitute hospital negligence. Another cause that could suggest either hospital
negligence or negligence by a third-party supplier is the use of contaminated blood, but that cause is
eliminated by the facts. The possible causes that do not suggest hospital negligence are "rare possibilities"
that occur outside the hospital setting. These possible causes are eliminated because the man was
hospitalized during the entire period of potential exposure. Thus, even though the specific cause of the
infection cannot be proven, it appears that there is a very strong inference that the hospital's negligence
caused the infection.

Lastly, here the hospital clearly had a duty to the man to protect him against contracting infections while
hospitalized. Thus, the indicated negligence—failing to protect the man from contracting the infection—
was within the scope of the hospital's duty to the man.

Based on this evidence, the court could use the res ipsa loquitur doctrine to find that the hospital is liable
for the man's infection.

[NOTE: This answer sets out the res ipsa loquitur requirements from the Restatement (Second) of Torts.
Jurisdictions differ as to exactly how they express the requirements of the res ipsa loquitur doctrine. One
traditional variation requires that the plaintiff show three things: (1) the accident which produced a
person's injury was one which ordinarily does not happen unless someone was negligent, (2) the
instrumentality or agent which caused the accident was under the exclusive control of the defendant, and
(3) the circumstances indicated that the untoward event was not caused or contributed to by any act or
neglect on the part of the injured person. The Third Restatement offers another formulation: that

Seperac-J19 Exam-Released MEE Essay Compilation © 2016-2020 499


negligence can be inferred when the accident causing harm is of a type that "ordinarily happens as a
result of the negligence of a class of actors of which the defendant is the relevant member."

Answers relying on any of these variations should be given full credit as long as the examinee recognizes
that courts interpret that variation, regardless of the specific way it sets out its requirements, to limit the
application of the res ipsa loquitur doctrine to those situations in which the defendant's negligence was
more probably than not the cause of the plaintiff's injuries. We should expect variation in local
verbalization of the rules, but always remember that a different verbalization may be intended to express
substantially the same ideas.)]>

[NOTE: An examinee might note that statutes in some jurisdictions restrict the use of res ipsa loquitur in
medical malpractice cases. No such statute appears here, and an examinee should not receive credit for
assuming such and answering accordingly.]

Explanation to Point Four (20%):

A finding that the woman’s negligence caused the car accident would mean that the woman is solely
responsible for the $100,000 damages from the accident and is liable for that amount. She and the hospital
together will be jointly and severally liable for the $250,000 in damages from the man’s infection. Thus,
the man can collect any portion, or all, of the $250,000 damages from the woman. Therefore, the
woman’s liability for both injuries cannot be limited to $100,000.

If the woman negligently caused the auto accident, she would be the sole proximate cause of the accident
and would be liable for the $100,000 stipulated damages. She alone bears responsibility for those
damages.

If the negligence of the woman and the hospital both caused the man's infection, the woman and the
hospital would be jointly and severally liable for the $250,000 stipulated infection damages. Joint and
several liability would be imposed for the infection damages because both the woman and the hospital
have caused an indivisible injury, one of the bases of joint and several liability. Each of them is liable for
the full amount of the man's damages from the infection.

Thus, because the woman is solely liable for the $100,000 of damages just from the accident and is jointly
and severally liable for the foreseeable infection damages, her liability cannot be limited to $100,000.

[NOTE: The man has no obligation or need to ask the court to apportion the infection damages. He can
approach either tortfeasor, or both tortfeasors, and seek total infection damages of $250,000 or a lesser
amount. The man has the choice of how to apportion collection efforts between the two. The fourth call
asks only whether the woman's liability could be limited to $100,000. Clearly the answer is "no" because
she is liable for $250,000 as a joint tortfeasor in addition to liability for $100,000 damages from the
accident. The examinee is not asked to specify how the plaintiff would apportion collection efforts
between the two joint tortfeasors.

The MEE Subject Matter Outline notes that all torts questions occur in a jurisdiction that has joint and
several liability with pure comparative negligence.]>

Seperac-J19 Exam-Released MEE Essay Compilation © 2016-2020 500


#111-JUL 2017–MEE Q01: QUESTION ONE (TORTS)

On the evening of July 4, a woman went to the end of her dock to watch a fireworks display on the lake
where her house was located. The woman’s husband remained inside the house. The fireworks display
was sponsored by the lake homeowners association, which had contracted with a fireworks company to
plan and manage all aspects of the fireworks display.

The fireworks display was set off from a barge in the middle of the lake. During the finale, a mortar flew
out horizontally instead of ascending into the sky. The mortar struck the woman’s dock. She was hit by
flaming debris and severely injured. When the woman’s husband saw what had happened from inside the
house, he rushed to help her. In his hurry, he tripped on a rug and fell down a flight of stairs, sustaining a
serious fracture.

All the fireworks company employees are state-certified fireworks technicians, and the company followed
all governmental fireworks regulations. It is not known why the mortar misfired.

The woman and her husband sued the homeowners association and the fireworks company to recover
damages for their injuries under theories of strict liability and negligence. At trial, they established all of
the above facts. They also established the following:

1) Nationally, accidents involving fireworks cause about 9,000 injuries and 5 deaths each year.
About 15% of these accidents are caused by mortars misfiring in the course of professional
fireworks displays, and some of these accidents occur despite compliance with governmental
fireworks regulations.

2) Even with careful use by experts, fireworks mortars can still misfire.

3) Although a state statute requires a “safety zone” of 500 feet from the launching site of fireworks
when those fireworks are launched on land, the statute does not refer to fireworks launched on
water. Neither the homeowners association nor the fireworks company established such a zone.

4) The average fireworks-to-shore distance for this display was 1,000 feet. The woman’s dock is
450 feet from the location of the fireworks barge; at only three other points on the lake is there land
or a dock within 500 feet of the fireworks barge location.

After the conclusion of the plaintiffs’ case, both the homeowners association and the fireworks company
moved for a directed verdict on the basis that the facts established by the evidence did not support a
verdict for the plaintiffs.

The trial judge granted the motion, based on these findings:

1. Fireworks displays are not an abnormally dangerous activity and thus are not subject to strict liability.

2. Based on the evidence submitted, a reasonable jury could not conclude that the conduct of the
fireworks company was negligent.

3. The misfiring mortar was not the proximate cause of the husband’s injuries.

4. The homeowners association cannot be held liable for the fireworks company’s acts or omissions.

Seperac-J19 Exam-Released MEE Essay Compilation © 2016-2020 501


As to each of the judge’s four findings, was the judge correct? Explain.

Seperac-J19 Exam-Released MEE Essay Compilation © 2016-2020 502


#111: J17-1 MEE: ANSWER: NCBE (TORTS)

POINT (1) [40%] ISSUE: Is a public fireworks display an abnormally dangerous activity subject to
strict liability? ANSWER: The trial court’s finding that a public fireworks display is not an
abnormally dangerous activity subject to strict liability may or may not have been correct. A public
fireworks display is not a matter of common usage and has substantial risks that cannot be
eliminated. But some courts, relying on the Second Restatement of Torts factors, have found that
public fireworks are nonetheless not abnormally dangerous because of their value to the
community.

POINT (2) [25%] ISSUE: Based on the evidence submitted by the woman and her husband, could a
reasonable jury find that the conduct of the fireworks company was negligent? ANSWER: Yes.
The trial judge incorrectly directed a verdict for the fireworks company on the negligence claim.
Adherence to a statutory standard does not insulate a defendant from liability for negligence, and
the evidence showed a foreseeable risk of harm that precautions would have reduced without undue
burden.

POINT (3) [15%] ISSUE: When is an actor’s creation of a dangerous situation the proximate cause
of injuries suffered by a person who was rushing to respond to the danger? ANSWER: The trial
court incorrectly found that the misfiring mortar was not the proximate cause of the husband’s
injuries because danger invites rescue.

POINT (4) [20%] ISSUE: When is a person liable for the acts or omissions of an independent
contractor employed by that person? ANSWER: The trial court incorrectly found that the
homeowners association could not be held liable for the fireworks company’s acts or omissions. One
who employs an independent contractor is subject to liability for the contractor’s failure to take
reasonable precautions as to danger inherent in the work.

ANSWER DISCUSSION:

It is unclear whether the trial judge erred in ruling that a public fireworks display is not an abnormally
dangerous activity subject to strict liability. Such a display is not a matter of common usage, as
traditionally defined, and it presents substantial risks that cannot be eliminated with the exercise of
reasonable care. However, some courts, relying on the Second Restatement of Torts factors, have found
that public fireworks displays are not abnormally dangerous because of their value to the community. The
judge erred in directing a verdict for the defendants on the negligence claim because adherence to a
statutory standard does not insulate a defendant from liability for negligence, and the evidence showed a
foreseeable risk of harm and precautions that would have eliminated the risk without undue burden.
Because danger invites rescue, the trial judge erred in concluding that the misfiring mortar was not the
proximate cause of the husband’s injuries. The judge also erred in concluding that the homeowners
association could not be held liable for the acts of the fireworks company, because one who employs an
independent contractor is subject to liability for the contractor’s failure to take reasonable precautions
when there is a special danger inherent in the work, and the harm that occurred here was the result of a
special danger inherent in fireworks displays.

ANSWER EXPLANATION:

Explanation to Point-One (40%):

Seperac-J19 Exam-Released MEE Essay Compilation © 2016-2020 503


The trial court’s finding that a public fireworks display is not an abnormally dangerous activity subject to
strict liability may or may not have been correct. A public fireworks display is not a matter of common
usage and has substantial risks that cannot be eliminated. But some courts, relying on the Second
Restatement of Torts factors, have found that public fireworks are nonetheless not abnormally dangerous
because of their value to the community.

The modern doctrine of strict liability for abnormally dangerous activities derives from Fletcher v.
Rylands, in which the defendant’s reservoir flooded mine shafts on the plaintiff’s adjoining land. Rylands
has come to stand for the rule that “the defendant will be liable when he damages another by a thing or
activity unduly dangerous and inappropriate to the place where it is maintained, in the light of the
character of that place and its surroundings.”

Today, the determination of whether an activity is unduly dangerous, and thus subject to strict liability, is
generally governed by factors outlined in the Restatement of Torts. Under the First Restatement, strict
liability applied to an “ultra-hazardous” activity. Under the Second and Third Restatements, strict liability
applies to an “abnormally dangerous activity.” Section 520 of the Restatement (Second) lists six factors
that are to be considered in determining whether an activity is abnormally dangerous:

(a) existence of a high degree of risk of some harm to the person, land or chattels of others;

(b) likelihood that the harm that results from it will be great;

(c) inability to eliminate the risk by the exercise of reasonable care;

(d) extent to which the activity is not a matter of common usage;

(e) inappropriateness of the activity to the place where it is carried on; and

(f ) extent to which its value to the community is outweighed by its dangerous attributes.

Comments to the Restatement explain that “several factors are ordinarily required for strict liability but it
is not necessary that each of them be present. The essential question is whether the risk created is so
unusual, either because of its magnitude or because of the circumstances surrounding it, as to justify the
imposition of strict liability for the harm that results from it, even though it is carried on with all
reasonable care.” The Second Restatement continued to define “common usage” narrowly. To be a matter
of common usage, an activity must be carried on “by the great mass of mankind or by many people in the
community.”

As in the First Restatement, under the relatively new Third Restatement, the strict liability determination
is based on only two factors. An activity is abnormally dangerous if (1) the activity creates a foreseeable
and highly significant risk of physical harm even when reasonable care is exercised by all actors; and (2)
the activity is not one of common usage. However, the Third Restatement employs a much broader
definition of “common usage” than that of the First and Second Restatements. Under the Third
Restatement, “activities can be in common use even if they are engaged in by only a limited number of
actors”. Consider the company that transmits electricity through wires. The activity itself is engaged in by
only one party. Even so, electric wires are pervasive within the community. Moreover, most people,
though not themselves engaging in the activity, are connected to the activity. The concept of common
usage can be extended further to activities that, though not pervasive, are nevertheless common and
familiar within the community. If in this sense the activity is normal, it is difficult to regard the activity as
exceptional or abnormally dangerous.

Seperac-J19 Exam-Released MEE Essay Compilation © 2016-2020 504


On the other hand, the Third Restatement specifies that “the value that the defendant or others derive from
the activity is not a direct factor in determining whether the activity is abnormally dangerous.” It is thus
unclear whether, under the Third Restatement, more or fewer activities would be classified as abnormally
dangerous than under the Second Restatement.

The classic example of an abnormally dangerous activity is blasting. Courts in virtually all jurisdictions
have held that this activity is subject to strict liability, citing its potential for extensive harm, the fact that
it is not a matter of common usage, and the actors’ inability to eliminate risk.

Courts have divided as to whether legal fireworks displays should be classified as abnormally dangerous
and thus subject to strict liability. Courts that have classified fireworks displays as abnormally dangerous
have tended to focus on the fact that fireworks are much like blasting in that “anytime a person ignites
aerial shells or rockets with the intention of sending them aloft to explode in the presence of large crowds
of people, a high risk of serious personal injury or property damage is created. Furthermore, no matter
how much care pyro-technicians exercise, they cannot entirely eliminate the high risk inherent in setting
off powerful explosives such as fireworks near crowds.” Courts that have declined to classify legal
fireworks displays as abnormally dangerous have tended to focus on their value to the community.

Under the limited factors of the First and Third Restatements, the case for holding that public fireworks
displays are abnormally dangerous is very strong. This activity is not one carried on by the “mass of men”
given that it is legally performed only by trained, licensed personnel. Its risks, given the large number of
people who watch such displays, cannot be eliminated.

Using the multiple-factor approach of the Second Restatement, one can justify a finding that fireworks
displays are not abnormally dangerous based on their popularity and value to the community and
relatively low risk to any particular individual. Under the Third Restatement, the value of fireworks to the
community would be irrelevant, but it might be characterized as a matter of common usage given that
fireworks, like electrical wires, are pervasive, at least on certain holidays.

Thus, it is unclear whether the trial court erred in finding that public fireworks displays are not an
abnormally dangerous activity. However, if the court relied on the Second Restatement, other courts have
reached the same conclusion. The court’s conclusion could also be justified under the Third Restatement
given its novel definition of common usage.

[NOTE: An examinee’s conclusion is less important than his or her reasoning on whether a public
fireworks display is an abnormally dangerous activity. A good answer need not analyze each theory
discussed above.]

Explanation to Point-Two (25%):

The trial judge incorrectly directed a verdict for the fireworks company on the negligence claim.
Adherence to a statutory standard does not insulate a defendant from liability for negligence, and the
evidence showed a foreseeable risk of harm that precautions would have reduced without undue burden.

Although the unexcused violation of a statutory standard of care is negligence per se, the converse is not
true: an actor who has complied with all statutory standards may still be found negligent if his conduct is
not reasonable under the circumstances. In judging whether an actor’s conduct is reasonable, the trier of
fact will consider the burden of taking precautions as compared to the risks inherent in the actor’s conduct
and the probability that those risks will materialize.

Seperac-J19 Exam-Released MEE Essay Compilation © 2016-2020 505


Here, all the fireworks company’s employees were state-certified fireworks technicians, and the company
followed all governmental fireworks regulations. However, the plaintiffs’ evidence established that there
was a foreseeable risk of a misfiring mortar even when the fireworks display was performed with due
care; the risk of misfire cannot be eliminated. The evidence also established that a misfiring mortar can
cause death or serious injury to a bystander. Nationally, accidents involving fireworks cause about 9,000
injuries and 5 deaths each year. About 15% of fireworks accidents are caused by mortars misfiring in the
course of professional fireworks displays, and some of these accidents occur despite compliance with
governmental fireworks regulations.

In recognition of the risks associated with the discharge of fireworks, a state statute imposes a 500-foot
safety zone for fireworks displays on land. Although the statute does not refer to fireworks displays on
water, it arguably provides a reasonable standard for determining a safety zone for a fireworks display on
water. The evidence also showed that a 500-foot safety zone would not have hindered spectators at the
lake fireworks display to any significant extent, as only four potential viewing spots lie within this area.
Because the fireworks company’s employees were all state-certified fireworks technicians, a reasonable
jury could have concluded that both the risk of injury and the utility of a 500-foot safety zone were known
to the fireworks company. It could also have concluded that the fireworks company or the homeowners
association, with very little cost or inconvenience, could have identified the four viewing spots within the
500-foot safety zone and warned potential spectators of the hazard of watching from those locations.

Thus, the court erred in directing a verdict for the fireworks company on the negligence issue: adherence
to a statutory standard does not insulate an actor from liability, and there was evidence on which a jury
could have based a negligence finding.

Explanation to Point-Three (15%):

The trial court incorrectly found that the misfiring mortar was not the proximate cause of the husband’s
injuries because danger invites rescue.

Liability typically extends only to individuals within the zone of risk. If an actor’s conduct “creates a
recognizable risk of harm only to a particular class of persons, the fact that it causes harm to a person of a
different class, to whom the actor could not reasonably have anticipated injury, does not render the actor
liable to the persons so injured.” Liability also typically extends only to foreseeable hazards. The actor
whose conduct is responsible for an altogether unexpected type of injury usually escapes liability.

Here, because the husband was inside his house, he was probably outside the area in which risks from a
fireworks display were to be anticipated. He also suffered a different injury (fracture) from that which one
would usually anticipate (burns, impact harm) from fireworks exposure.

However, courts have long held that injuries sustained when running from danger are foreseeable. They
have also held that “danger invites rescue.” The wrong that imperils life is a wrong to his rescuer. Thus,
because the husband was a rescuer and his injuries are typical of those an individual rushing from (or to) a
dangerous situation would sustain, the trial court erred in concluding that the acts and omissions of the
homeowners association and the fireworks company were not the proximate cause of his injuries.

Explanation to Point-Four (20%):

The trial court incorrectly found that the homeowners association could not be held liable for the
fireworks company’s acts or omissions. One who employs an independent contractor is subject to liability
for the contractor’s failure to take reasonable precautions as to danger inherent in the work.

Seperac-J19 Exam-Released MEE Essay Compilation © 2016-2020 506


An independent contractor is one who, by virtue of his contract, possesses independence in the manner
and method of performing the work he has contracted to perform for the other party to the contract.
Independent contractors are usually paid by the job instead of receiving ongoing salaries; the individual
who hires an independent contractor typically does not supervise the contractor’s activities or retain a
right to control his activities. The fireworks company is an independent contractor: it was hired by the
homeowners association for a specific job, to plan and manage the fireworks display. The homeowners
association did not supervise the fireworks company’s work or have any control over its operations.

Typically, one who employs an independent contractor is not vicariously liable for the contractor’s acts or
omissions. But when an actor “employs an independent contractor to do work involving a special danger
to others which the employer knows or has reason to know to be inherent in or normal to the work, he is
subject to liability for physical harm caused to such others by the contractor’s failure to take reasonable
precautions against such danger.”

Here, the homeowners association had reason to know that fireworks are inherently risky; thousands of
fireworks injuries occur each year, and the risk cannot be eliminated even when fireworks are used by
experts. Thus, the trial court’s conclusion that the association could not be held liable for the fireworks
company’s acts was erroneous.

Seperac-J19 Exam-Released MEE Essay Compilation © 2016-2020 507


#112-JUL 2016–MEE Q03: QUESTION THREE (TORTS)

Six months ago, a man visited his family physician, a general practitioner, for a routine examination.
Based on blood tests, the physician told the man that his cholesterol level was somewhat elevated. The
physician offered to prescribe a drug that lowers cholesterol, but the man stated that he did not want to
start taking drugs because he preferred to try dietary change and “natural remedies” first. The physician
told the man that natural remedies are not as reliable as prescription drugs and urged the man to come
back in three months for another blood test. The physician also told the man about a recent research report
showing that an herbal tea made from a particular herb can reduce cholesterol levels.

The man purchased the herbal tea at a health-food store and began to drink it. The man also began a
cholesterol-lowering diet.

Three months ago, the man returned to his physician and underwent another blood test; the test showed
that the man’s cholesterol level had declined considerably. However, the test also showed that the man
had an elevated white blood cell count. The man’s test results were consistent with several different
infections and some types of cancer. Over the next two weeks, the physician had the man undergo more
tests. These tests showed that the man’s liver was inflamed but did not reveal the reason. The physician
then referred the man to a medical specialist who had expertise in liver diseases. In the meantime, the man
continued to drink the herbal tea.

Two weeks ago, just before the man’s scheduled consultation with the specialist, the man heard a news
bulletin announcing that government investigators had found that the type of herbal tea that the man had
been drinking was contaminated with a highly toxic pesticide. The investigation took place after liver
specialists at a major medical center realized that several patients with inflamed livers and elevated white
blood cell counts, like the man, were all drinking the same type of herbal tea and the specialists reported
this fact to the local health department.

All commercially grown herbs used for this tea come from Country X, and are tested for pesticide
residues at harvest by exporters that sell the herb in bulk to the five U.S. companies that process, package,
and sell the herbal tea to retailers. U.S. investigators believe that the pesticide contamination occurred in
one or more export warehouses in Country X where bulk herbs are briefly stored before sale by exporters,
but they cannot determine how the contamination occurred or what bulk shipments were sent to the five
U.S. companies. The companies that purchase the bulk herbs do not have any control over these
warehouses, and there have been no prior incidents of pesticide contamination. The investigators have
concluded that the U.S. companies that process, package, and sell the herbal tea were not negligent in
failing to discover the contamination.

Packages of tea sold by different companies varied substantially in pesticide concentration and toxicity,
and some packages had no contaminants. Further investigation has established that the levels of
contamination and toxicity in the herbal tea marketed by the five different U.S. companies were not
consistent.

The man purchased all his herbal tea from the same health-food store. The man is sure that he purchased
several different brands of the herbal tea at the store, but he cannot establish which brands. The store sells
all five brands of the herbal tea currently marketed in the United States.

Seperac-J19 Exam-Released MEE Essay Compilation © 2016-2020 508


The man has suffered permanent liver damage and has sued to recover damages for his injuries. It is
undisputed that the man’s liver damage was caused by his herbal tea consumption. The man’s action is
not preempted by any federal statute or regulation.

1. Is the physician liable to the man under tort law? Explain.

2. Are any or all of the five U.S. companies that processed, packaged, and sold the herbal tea to the
health-food store liable to the man under tort law? Explain.

3. Is the health-food store liable to the man under tort law? Explain.

Seperac-J19 Exam-Released MEE Essay Compilation © 2016-2020 509


#112: J16-3 MEE: ANSWER: NCBE (TORTS)

POINT (1) [25%] ISSUE: Can the man recover damages under tort law from the physician?
ANSWER: No. A doctor is liable to a patient only when the evidence shows that the doctor has
failed to comply with the standard of care for the relevant specialty and medical community and
the failure caused the patient’s injury. Because the facts here do not establish such a failure, the
physician would not be liable for the man’s injury.

POINT (2)(a) [20%] ISSUE: When may a producer of a defective product be found liable for
injuries caused by that product? ANSWER: The producer of goods that cause injury to a person
may be liable to the injured person in tort if the seller was negligent, if the goods were defective, or
if they did not satisfy the implied warranty of merchantability. Here, the herbal tea that the man
purchased was defective because it was contaminated with pesticide.

POINT (2)(b) [35%] ISSUE: Can the man recover from any of the five U.S. companies when he
cannot show what company’s product caused his injuries? ANSWER: No. Because the man
consumed several brands of the herbal tea, he cannot show which producer of the herbal tea
supplied the product that caused his injury, and none of the doctrines that permit a plaintiff to meet
the causation requirement without direct proof of causation are available here.

POINT (3) [20%] ISSUE: Can the man recover damages from the health-food store? ANSWER:
Yes. The health-food store from which the man bought the contaminated herbal tea may be found
strictly liable in tort even though the store did not produce the tea.

ANSWER DISCUSSION:

A doctor is liable to a patient only when the evidence shows that he has failed to comply with the standard
of care for the relevant specialty and medical community, and his failure causes the patient’s injury.
Because liver specialists did not make the link between the herbal tea and symptoms like the man’s until
after being presented with a group of patients, the family physician’s conduct did not fall below the
standard of care either at the initial consultation or thereafter. A producer is liable in tort for a defective
product that is unreasonably dangerous and that causes the plaintiff’s injury. A product that does not meet
the producer’s own specifications is defective; an herbal tea contaminated with toxic pesticide is
unreasonably dangerous. However, the man cannot show which of the five U.S. companies who process,
package, and sell the herbal tea caused his injury because he consumed more than one brand. None of the
doctrines that permit a plaintiff to meet the causation requirement without direct proof of causation are
available here. Thus, because the man cannot show which company caused his injury, he cannot recover
against any of the five herbal tea producers. A retail seller is strictly liable in tort for defective products it
sells even if it had no control over the production process. Thus, the health-food store may be found liable
to the man because the man purchased all of the herbal tea he consumed from that health-food store, and
the evidence shows that the herbal tea caused the man’s injury.

ANSWER EXPLANATION:

Explanation to Point-One (25%):

A doctor is liable to a patient only when the evidence shows that the doctor has failed to comply with the
standard of care for the relevant specialty and medical community and the failure caused the patient’s

Seperac-J19 Exam-Released MEE Essay Compilation © 2016-2020 510


injury. Because the facts here do not establish such a failure, the physician would not be liable for the
man’s injury.

A medical doctor is liable to a patient only when the evidence shows that he has failed to comply with the
standard of care for the relevant specialty and medical community and his failure causes the patient’s
injury. In assessing whether a doctor has met this test, most courts compare the doctor’s conduct to
national standards rather than those that prevail in his or her locality. Because the standard requires
assessment of typical doctor conduct, expert testimony is almost invariably necessary to establish a
doctor’s negligence.

Here, there are two possible negligence claims against the physician. The man might argue that the
physician was negligent in suggesting that drinking the herbal tea might lower his cholesterol. The man
also might argue that, in his follow-up visit, the physician was negligent in failing to determine that his
symptoms were due to the herbal tea if the man could establish that the delay in diagnosis worsened his
medical condition. To succeed with either argument, the man would have to show that, if the physician
had complied with the standard of care for general practitioners, the man would have followed a different
course of action. Additionally, the man would have to show that the physician’s failure to comply with the
standard of care caused him harm. Based on the available evidence, it appears highly unlikely that expert
testimony will be available to make such a showing.

Here, there is no indication that the physician failed to comply with the standard of care by suggesting
that this type of herbal tea lowered cholesterol levels. There is no indication that the physician was aware
that this type of herbal tea would be contaminated with toxic pesticides. Moreover, there is no indication
that complying with the standard of care for general practitioners would have required him to be aware of
such contamination. The facts establish that the physician advised the man that a prescription drug was
the most reliable method of lowering cholesterol levels and told the man to come back for another test in
three months. The physician only mentioned the herbal tea when the man refused a prescription drug in
favor of “natural” methods and dietary change and did so because of a recent research report. Indeed, the
herbal tea may have played a role in lowering the man’s cholesterol.

With respect to the physician’s failure to correctly diagnose the source of the man’s symptoms, the facts
establish that the physician responded to the man’s elevated white blood cell count promptly and ordered
additional tests. After these tests revealed a liver inflammation, the physician promptly referred the man
to a specialist. More importantly, the facts show that even liver specialists were able to determine the link
between the herbal tea and symptoms like the man’s only when they had a cluster of patients with similar
symptoms and discovered that all of the patients were drinking the same herbal tea.

Thus, based on the available evidence, the physician may not be found liable to the man.

Explanation to Point-Two(a) (20%):

The producer of goods that cause injury to a person may be liable to the injured person in tort if the seller
was negligent, if the goods were defective, or if they did not satisfy the implied warranty of
merchantability. Here, the herbal tea that the man purchased was defective because it was contaminated
with pesticide.

“One who sells any product in a defective condition unreasonably dangerous to the user or consumer is
subject to liability for physical harm thereby caused.”

Seperac-J19 Exam-Released MEE Essay Compilation © 2016-2020 511


Products that fail to meet the producer’s own specifications are typically described as having a
“manufacturing” defect. In the case of food products, the presence of a harmful ingredient is generally
considered a manufacturing defect “if a reasonable consumer would not expect the food product to
contain that ingredient.” The herbal tea that the man consumed falls into the manufacturing defect
category even though it is not a manufactured product in the traditional sense because a reasonable
consumer would expect the herbal tea to be free of contamination when processed and packaged. Given
the severe harm caused to consumers by the pesticide residue, its presence clearly rendered the product
unreasonably dangerous. Thus, the processor selling tea with this defect would be liable in tort for
resulting injuries.

In order to recover for injuries sustained because of a manufacturing defect, a plaintiff need not show that
the producer was negligent. A producer is strictly liable whenever “the product departs from its intended
design even though all possible care was exercised in the preparation and marketing of the product.”

The man could also rely on the implied warranty of merchantability to establish the U.S. companies’
liability. Because the sale of the herbal tea by the producers is a sale of goods, it is governed by Article 2
of the Uniform Commercial Code. The producers are “merchants” with respect to those goods, so the
contract of sale included an implied warranty of merchantability. There is no evidence that this warranty
was excluded or modified in any of the contracts under which those companies sold herbal tea. To be
merchantable, goods must, inter alia, be “fit for the ordinary purposes for which such goods are used.”
Clearly, the contaminated herbal tea was not fit for the ordinary purpose for which the herbal tea is used.
Thus, the producers breached the implied warranty of merchantability and are liable for that breach.

Under both warranty theory and strict products liability, the producers may not rely on the fact that the
contamination took place before the herbal tea came into their hands to evade liability. In a warranty
action, the only issue is whether the herbal tea was merchantable. How it came to be unmerchantable is
irrelevant. In a strict products liability action, the issue is whether the product was defective. Thus, the
man could recover against a producer of the herbal tea without proof of negligence if he could show that
any given producer sold the product that caused his injury.

[NOTE #1: In some states, privity requirements may be applied to prevent the man from recovering from
a seller with whom he is not in privity. However, in many states, privity rules have been relaxed or
modified sufficiently to permit a remote purchaser like the man to assert an implied warranty claim
against the seller of a food product.]

[NOTE #2: Some examinees might note that complete analysis of the warranty claim requires knowledge
of whether the relevant contract effectively disclaimed the warranty of merchantability. However, this
would not affect the tort liability claim.]

Explanation to Point-Two(b) (35%):

Because the man consumed several brands of the herbal tea, he cannot show which producer of the herbal
tea supplied the product that caused his injury, and none of the doctrines that permit a plaintiff to meet the
causation requirement without direct proof of causation are available here.

Like all tort plaintiffs, the plaintiff in a products liability action must establish that the defendant caused
his injury. Had the man consumed only one brand of the herbal tea, causation might be established by
showing the link between the tea’s toxicity and the man’s symptoms. However, the man drank the herbal
tea produced by more than one producer, and he cannot link any particular defendant’s product to his
injury.

Seperac-J19 Exam-Released MEE Essay Compilation © 2016-2020 512


There are several doctrines that permit the jury to find a defendant liable when the plaintiff cannot directly
meet the causation requirement. However, none of these doctrines would help the man to establish
causation here.

The “market share” liability doctrine permits the jury to apportion damages based on the market shares of
manufacturers of a defective product. But virtually all courts have held that this doctrine is available only
if the manufacturers’ defective products are fungible in relation to their capacity to cause harm. Here, the
man cannot make such a showing; what caused the man’s injury was pesticide-contaminated herbal tea,
and the facts specify that the contamination was not uniform. Some packages were heavily contaminated
and some not at all. Thus, market-share liability is unavailable.

The “alternative liability” doctrine permits a jury to find two defendants liable when each was negligent
and either could have caused the plaintiff’s injuries. Here, there are five defendants, and there is no
evidence that any producer was negligent.

The “joint venture” or “joint enterprise” doctrine allows the jury to impute one defendant’s tortious
conduct to other defendants who are engaged in a common project or enterprise and who have made an
explicit or implied agreement to engage in tortious conduct. Here, however, there is no evidence that the
producers had any control over the warehouses in which the pesticide contamination originated, let alone
that they collaborated in tortious conduct.

In sum, based on the evidence at hand, none of the producers could be held liable for the man’s injury
because no evidence links a particular producer to that injury and no exception to this requirement
applies.

Explanation to Point-Three (20%):

The health-food store from which the man bought the contaminated herbal tea may be found strictly liable
in tort even though the store did not produce the tea.

Strict products liability applies to all commercial sellers; even a retailer who had no control over the
design and manufacture of a product may be found strictly liable if that retailer sells a defective product.
Because the health-food store is a commercial seller, it may be found liable to the man for the defective
herbal tea that the man purchased there.

The man could also recover on an implied warranty theory against the health-food store in all jurisdictions
because he was in privity with the store.

Just as with the herbal tea producers, the man has the burden of showing causation. But the facts specify
that the man purchased all the herbal tea he consumed from the same health-food store. The identification
problem that makes causation impossible to establish with respect to the producers thus does not arise
with respect to the health-food store.

Thus the health-food store may be found liable for the man’s injury based on the fact that it sold a
defective product that caused the man’s injury. Some states have statutes that exclude strict liability in tort
for retailers who sell products in closed packages.

Seperac-J19 Exam-Released MEE Essay Compilation © 2016-2020 513


#113-JUL 2015–MEE Q01: QUESTION ONE (TORTS)

A boy lives in a northern state where three to four feet of snow typically blankets the ground throughout
the winter, creating excellent conditions for snowmobiling. The boy is an experienced snowmobiler and a
member of a club that maintains local snowmobile trails by clearing them of rocks, stumps, and fallen tree
limbs that could cause an accident when buried under the snow. In January, the boy received a
snowmobile as a present on his 12th birthday. The following Sunday, the boy took his friend, age 10, out
on the boy’s new snowmobile, which was capable of speeds up to 60 miles per hour. The friend had never
been snowmobiling before.

The boy and his friend went snowmobiling on a designated and marked snowmobile trail that follows the
perimeter of a rocky, forested state park near the friend’s home. The trail adjoins forested property owned
by a private landowner. Neither the boy nor his friend had previously used this trail.

The landowner’s property is crossed by a private logging trail that intersects the snowmobile trail. The
logging trail is not marked or maintained for snowmobiling, and access to it is blocked by a chain
approximately 30 inches above ground level on which a “No Trespassing” sign is displayed. However, on
the day in question, both the chain and the sign were covered by snow.

On impulse, the friend, who was driving the snowmobile, turned the snowmobile off the designated
snowmobile trail and onto the logging trail. The snowmobile immediately struck the submerged chain and
crashed. Both the boy and the friend were thrown from the snowmobile and injured. As a result of the
accident, the snowmobile was inoperable.

About an hour after the accident, a woman saw the boy and his friend as she was snowmobiling on the
snowmobile trail. After the woman returned to her car, she called 911, reported the accident and its
location, and then went home. Emergency personnel did not reach the boy and his friend for two hours
after the woman’s departure. No one other than the woman passed the accident site before emergency
personnel arrived.

As a result of the accident, the boy suffered several broken bones and also suffered injuries from frostbite.
These frostbite injuries could have been avoided had the boy been rescued earlier.

The boy has brought a tort action against the friend, the landowner, and the woman.

1. Could a jury properly find the friend liable to the boy for his injuries? Explain.

2. Could a jury properly find the landowner liable to the boy for his injuries? Explain.

3. Could a jury properly find the woman liable to the boy for his injuries? Explain.

Seperac-J19 Exam-Released MEE Essay Compilation © 2016-2020 514


#113: J15-1 MEE: ANSWER: NCBE (TORTS)

POINT (1) [25%] ISSUE: Could a jury properly find that the friend, a 10-year-old child with no
snowmobiling experience, was negligent? ANSWER: Yes. A jury could properly find that the
friend is liable to the boy based on a finding that the friend was engaged in an adult activity and
that he was negligent in turning off a designated snowmobile trail onto an unmarked logging trail.

POINT (2) [35%] ISSUE: Did the landowner have a duty to reveal hidden dangers and, if yes, could
a jury properly find that there was a breach of that duty? ANSWER: A jury could find the
landowner liable to the boy if he is classified as a licensee to whom a limited duty of care is owed. If
the boy is found to be a trespasser rather than a licensee, the jury could find that the landowner is
liable only if the attractive-nuisance doctrine applies.

POINT (3) [25%] ISSUE: Did the woman owe the boy a duty of care? ANSWER: No. A jury could
not properly find that the woman had a duty to the boy because she did not increase the risk of
harm to him beyond that which existed before she called 911, and the boy did not rely on the
woman’s exercise of reasonable care to his detriment.

POINT (4) [15%] ISSUE: Could a jury properly find that the boy was negligent and, if yes, what
impact will his negligence have on his claim? ANSWER: A jury could properly find that the boy
was negligent in entrusting a snowmobile to an inexperienced 10-year-old. A finding of negligence
would reduce the boy’s damages award but, in almost all jurisdictions, would not deprive him of his
claim.

ANSWER DISCUSSION:

Because snowmobiling involves motorized vehicles traveling at high speeds, it is an adult activity. When
a minor engages in an adult activity, he can be found negligent if he fails to exercise the same degree of
care that a prudent adult would exercise under the same circumstances. A jury could thus properly find
that the friend failed to exercise reasonable care by operating a snowmobile on an unmarked trail in a
rocky, forested area where there was a risk of hitting hidden objects. In most jurisdictions, a jury could
properly conclude that the landowner owed the boy a duty to reveal hidden dangers such as a chain
obscured by snow only if the boy is categorized as a licensee or, alternatively, if the boy is categorized as
a trespasser and the landowner is found to have created an attractive nuisance. It is unclear whether the
boy could establish that he was a licensee on these facts. If he is classified as a trespasser, rather than a
licensee, it is unclear whether the landowner owed him a duty of care based on the attractive-nuisance
doctrine because the boy was engaged in an adult activity. The woman owed no duty of care to the boy
because she did not increase the risk of harm by calling 911 and did not induce detrimental reliance. A
jury could properly find that the boy was negligent in entrusting his snowmobile to a 10-year old who had
never been snowmobiling before. In comparative negligence jurisdictions, a finding that the boy was
negligent could reduce his damages; in contributory negligence jurisdictions (which are a minority of
states), a finding that the boy was negligent precludes any recovery.

ANSWER EXPLANATION:

Explanation to Point-One (25%):

Seperac-J19 Exam-Released MEE Essay Compilation © 2016-2020 515


A jury could properly find that the friend is liable to the boy based on a finding that the friend was
engaged in an adult activity and that he was negligent in turning off a designated snowmobile trail onto an
unmarked logging trail.

Unless a child is engaged in an adult activity, his conduct is to be measured against that of a child of
similar age, intelligence, and experience. Under this standard, it is doubtful that a jury could find the
friend negligent: he was only 10 years old, and he had not previously engaged in snowmobiling.

However, when a minor engages in a hazardous activity which is “normally undertaken only by adults,
and for which adult qualifications are required , he may be held to the standard of adult skill, knowledge,
and competence, and no allowance may be made for his immaturity.” Driving a motorized vehicle like a
snowmobile is a classic example of an adult activity. Indeed, the Third Restatement of Torts provides as
examples of dangerous adult activities, “driving a car, a tractor, and a motorcycle, and operating other
motorized vehicles such as minibikes, motorscooters, dirt bikes, and snowmobiles.”

A reasonably prudent adult snowmobiler operating a snowmobile in a rocky, forested area would perceive
that turning off a marked and designated snowmobiling trail onto a trail not designated for snowmobiling
presents a foreseeable risk of injury. An unmarked trail is unlikely to have been prepared for
snowmobiling. As the facts make clear, trails in this area often contain hidden hazards – for example,
rocks, fallen tree limbs, and stumps – that pose the risk of a collision and that may not be readily visible
under the snow on which a snowmobile travels. Thus, a jury could find that the friend was negligent in
turning off a marked and designated snowmobile trail onto a logging trail unmarked for snowmobiling.

[NOTE: An examinee may appropriately note that, if the jury finds that the friend was negligent, it should
also find that his negligence was the cause-in-fact and proximate cause of the boy’s injuries; the accident
would not have happened but for the friend’s action in turning onto the unmarked trail, and hitting a
hidden object is an immediate, foreseeable risk of such an action. These conclusions as to cause-in-fact
and proximate cause are so obvious that an examinee should not be penalized for failing to note them.]

Explanation to Point-Two (35%):

A jury could find the landowner liable to the boy if he is classified as a licensee to whom a limited duty of
care is owed. If the boy is found to be a trespasser rather than a licensee, the jury could find that the
landowner is liable only if the attractive-nuisance doctrine applies.

In most states, the landowner’s duty to the boy depends on whether the boy is classified as a trespasser or
a licensee. A trespasser is one who enters or remains upon land owned or possessed by another without a
privilege to do so. Because the boy and his friend had no privilege to enter the landowner’s property, they
may be classified as trespassers.

However, if “in light of all the surrounding circumstances, a reasonable person would interpret the
possessor’s words or conduct as manifesting that he is in fact willing for another to enter upon his land,”
then the entrant is a licensee, not a trespasser. The presence of a path may manifest consent.

Although posting a “no trespassing” sign would normally negate an implication of consent, the
landowner’s “no trespassing” sign was not visible, and thus the boy could be classified as a licensee if the
trier of fact concludes that it was reasonably foreseeable to the landowner that snow would obscure the
sign and chain during the winter.

Seperac-J19 Exam-Released MEE Essay Compilation © 2016-2020 516


If the boy is classified as a licensee, in most jurisdictions the landowner would owe him a duty to reveal
hidden dangers of which the landowner “knows or has reason to know” and which the boy was not likely
to discover. Here, because the landowner’s property is typically covered with three to four feet of snow
during the snowmobiling season, a jury could properly conclude that the landowner knew or had reason to
know of the hazard posed by a chain and a “no trespassing” sign only 30 inches above the ground and that
he should have posted a higher and more visible warning.

If, however, the boy is classified as a trespasser, in most jurisdictions the landowner would owe him no
duty of care to make the premises reasonably safe or to warn of hidden dangers unless the court finds that
the landowner has created an “attractive nuisance.” Under the attractive-nuisance doctrine, a possessor of
land is subject to liability for physical harm to a trespassing child when that harm is caused by an artificial
condition on the land and

(a) the place where the condition exists is one upon which the possessor knows or has reason to know that
children are likely to trespass, and

(b) the condition is one of which the possessor knows or has reason to know and which he realizes or
should realize will involve an unreasonable risk of death or serious bodily harm to such children, and

(c) the children because of their youth do not discover the condition or realize the risk involved in
intermeddling with it or in coming within the area made dangerous by it, and

(d) the utility to the possessor of maintaining the condition and the burden of eliminating the danger are
slight as compared with the risk to children involved, and

(e) the possessor fails to exercise reasonable care to eliminate the danger or otherwise to protect the
children.

Here, the landowner likely had reason to know that a logging trail intersecting a snowmobile trail would
offer a tempting detour to young snowmobilers and that the landowner should have foreseen that a chain
and sign only 30 inches above the ground could easily become obscured by snow, posing a serious risk of
accident and injury to a snowmobiler who deviated onto the logging trail. There is no question that the
boy and his friend did not discover the dangerous condition (i.e., a hidden chain); the friend, the driver,
clearly did not appreciate the risk and the boy, his passenger, had no opportunity to avoid it. The burden
of eliminating the risk is small; all that would be necessary is posting a sign warning of the hazard at a
sufficient height to ensure that it remains snow-free. Thus, had the boy and his friend not been engaged in
an adult activity, the boy’s claim against the landowner would be strong.

However, the attractive-nuisance doctrine is almost invariably applied when the trespassing child is
engaged in an activity appropriate for children; indeed, most of the cases involve “rather young children,
usually under 12.” The landowner’s duty “is only to exercise reasonable care to keep the part of the land
upon which he should recognize the likelihood of children’s trespassing free from those conditions which
are likely not to be observed by children, or which are beyond the imperfect realization of children.” Here,
the risk inherent in the hidden chain was a risk only to snowmobilers. The landowner could argue that,
because snowmobiling is an adult activity, the attractive-nuisance doctrine would not apply. Under
current case law, courts have refused to apply the attractive-nuisance doctrine to minors engaged in
unforeseeable adult criminal activity.

Because of the difficulties inherent in classifying entrants onto real property, about half of the states have
modified the classifications so as to establish a general duty of reasonable care under the circumstances.

Seperac-J19 Exam-Released MEE Essay Compilation © 2016-2020 517


But only a handful of states have fully abolished the categories; most have retained the trespasser status
and the rules that go with it.

Thus, if the boy is classified as a licensee, a jury may find that the landowner violated his duty of care. If
the boy is classified as a trespasser, a jury may find that the landowner violated his duty of care only if it
applies the attractive-nuisance doctrine, and a fact-finder may choose not to apply that doctrine given that
the boy and his friend were engaged in an adult activity. In a state that has abolished these classifications,
the boy may recover if the fact-finder finds that the chain posed a foreseeable risk to foreseeable entrants.

[NOTE: An examinee’s conclusions on trespasser/licensee categorization and whether the attractive-


nuisance doctrine is available are less important than a demonstrated understanding of the relevant legal
categories. As in Point One, an examinee who concludes that the boy can recover might also
appropriately note that the fact-finder should also find that the landowner’s actions were the cause-in-
fact and proximate cause of the boy’s injuries. Again, however, these conclusions are so obvious that an
examinee should not be penalized for failing to discuss them.]

Explanation to Point-Three (25%):

A jury could not properly find that the woman had a duty to the boy because she did not increase the risk
of harm to him beyond that which existed before she called 911, and the boy did not rely on the woman’s
exercise of reasonable care to his detriment.

Generally, there is no duty to come to the aid of another. An actor who undertakes to render services
designed to reduce the risk of harm to another does acquire a duty of reasonable care toward the other if

a) the failure to exercise such care increases the risk of harm beyond that which existed without the
undertaking, or

b) the person to whom the services are rendered or another relies on the actor’s exercising reasonable care
in the undertaking.

Here, the woman did not increase the risk of harm beyond that which existed without the undertaking to
call for help. Although the time that elapsed between the accident and the boy’s rescue caused frostbite
injuries, the woman caused no delay; indeed, if she had not called 911, aid would likely have arrived even
later than it did. Nor did any individual rely on the woman. She told the 911 dispatcher that she was
leaving the scene, and her call did not prevent the boy or anyone else from reporting the accident or taking
steps to avoid the harm. Under the Second Restatement of Torts, one who takes charge of another who is
helpless is subject to liability to the other for any bodily harm caused to him by the actor’s discontinuing
his aid, if by so doing he leaves the other in a worse position than when the actor took charge of him.

Explanation to Point-Four (15%):

A jury could properly find that the boy was negligent in entrusting a snowmobile to an inexperienced 10-
year-old. A finding of negligence would reduce the boy’s damages award but, in almost all jurisdictions,
would not deprive him of his claim.

Because the boy was an experienced snowmobiler who should have been aware of the risks of driving a
snowmobile without any training or experience, a jury could find that the boy was negligent in allowing
his friend to drive the snowmobile even if it applied the standard of care applicable to children.

Seperac-J19 Exam-Released MEE Essay Compilation © 2016-2020 518


At common law, if the jury found that the plaintiff’s negligence was a cause in fact and proximate cause
of his injuries, the plaintiff was precluded from recovering damages from the defendant. In other words, if
the plaintiff’s negligence contributed to the plaintiff’s injuries, the plaintiff could not recover from the
defendant even though the defendant was also negligent. This contributory-negligence doctrine applies
only in a minority of states. All other states, however, have abandoned the contributory negligence
doctrine in favor of a comparative-negligence doctrine.

Under the modern “comparative negligence” approach, if the jury finds that two or more parties are
negligent, it may apportion fault among them. The amount of damages apportioned to the plaintiff
because of the plaintiff’s negligence is subtracted from the total damages awarded by the jury. Thus, a
finding that the boy was negligent should not deprive him of his cause of action. There are some
comparative-negligence jurisdictions in which apportionment is not available unless the jury concludes
that the plaintiff is less than 50 percent at fault; in those jurisdictions, if the jury concludes that the boy
was 50 percent or more at fault, he cannot recover.

Seperac-J19 Exam-Released MEE Essay Compilation © 2016-2020 519


#114-JUL 2012–MEE Q06: QUESTION SIX (TORTS)

Susan, a student at University, lived in a University dormitory. Access to Susan’s dormitory was
restricted to dormitory residents and guests who entered the dormitory with a resident. Entry to the
dormitory was controlled by key cards. Dormitory key cards opened all doors except for a rear entrance,
used only for deliveries, that was secured with a deadbolt lock.

On November 30, at 2:00 a.m., Ann, a University graduate, entered the dormitory through the rear
entrance. Ann was able to enter because the deadbolt lock had broken during a delivery four days before
Ann’s entry and had not been repaired. Ann attacked Susan, who was studying alone in the dormitory’s
library.

James, another resident of Susan’s dormitory, passed the library shortly after Ann had attacked Susan.
The door was open, and James saw Susan lying on the floor, groaning. James told Susan, “I’ll go for help
right now.” James then closed the library door and went to the University security office. However, the
security office was closed, and James took no other steps to help Susan. About half an hour after James
closed the library door, Susan got up and walked to the University hospital, where she received immediate
treatment for minor physical injuries.

One day after Ann’s attack, Susan began to experience mental and physical symptoms (e.g., insomnia,
anxiety, rapid breathing, nausea, muscle tension, and sweating). Susan’s doctor has concluded that these
symptoms are due to post-traumatic stress disorder (PTSD). According to the doctor, Susan’s PTSD was
caused by trauma she suffered one month before Ann’s attack when Susan was robbed at gunpoint. In the
doctor’s opinion, although Susan had no symptoms of PTSD until after Ann’s attack, Ann’s attack
triggered PTSD symptoms because Susan was suffering from PTSD caused by the earlier robbery. The
symptoms became so severe that Susan had to withdraw from school. She now sees a psychologist
weekly.

Since the attack, Susan has learned that Ann suffers from schizophrenia, a serious mental illness. From
August through November, Ann had been receiving weekly outpatient psychiatric treatment from her
Psychiatrist. Her Psychiatrist’s records show that on November 20, Ann told her Psychiatrist that she
“was going to make sure” that former University classmates who were “cheaters” got “what was coming
to them for getting the good grades I should have received.” Ann’s Psychiatrist did not report these threats
to anyone because Ann had no history of violent behavior. Ann’s Psychiatrist also did not believe that
Ann would take any action based on her statements.

At the time of the attack, Susan knew Ann only slightly because they had been in one class together the
previous semester. Susan received an A in that class.

Susan is seeking damages for the injuries she suffered as a result of Ann’s attack and has sued University,
James, and Ann’s Psychiatrist.

1(a). May Susan recover damages for physical injuries she suffered in Ann’s attack from
University? Explain.

1(b). May Susan recover damages for physical injuries she suffered in Ann’s attack from James?
Explain.

Seperac-J19 Exam-Released MEE Essay Compilation © 2016-2020 520


1(c). May Susan recover damages for physical injuries she suffered in Ann’s attack from Ann’s
Psychiatrist? Explain.

2. Assuming that any party is found liable to Susan, may she also recover damages from that party
for the PTSD symptoms she is experiencing? Explain.

Seperac-J19 Exam-Released MEE Essay Compilation © 2016-2020 521


#114: J12-6 MEE: ANSWER: NCBE (TORTS)

POINT (1)(a) [30%] ISSUE: Could a jury find that University had a duty of care to Susan and that
its breach of that duty was the cause in fact and proximate cause of Susan’s injuries? ANSWER:
Yes. University owed a duty of care to Susan, a resident of a dormitory controlled and maintained
by University, to take reasonable precautions to protect Susan from foreseeable criminal activity.
University’s failure to fix the broken lock or take other precautions against unauthorized entry into
Susan’s dorm violated that duty. It was also the cause in fact and proximate cause of Susan’s
physical injuries.

POINT (1)(b) [25%] ISSUE: Could a jury find that James owed a duty of care to Susan and that his
breach of that duty was the cause in fact and proximate cause of Susan’s injuries? ANSWER: No.
There is no general duty to come to the aid of another. James assumed a duty to Susan only if Susan
relied to her detriment on James’s promise to obtain assistance or if James left Susan in a worse
position. Here, the evidence does not show that Susan relied to her detriment or that Susan was in a
worse position after James took charge.

POINT (1)(c) [30%] ISSUE: Could a jury find that Ann’s Psychiatrist owed a duty of care to Susan
and that Psychiatrist’s breach of that duty was the cause in fact and proximate cause of Susan’s
injuries? ANSWER: No. A psychotherapist has a duty to warn a reasonably identifiable individual
against whom her patient has made credible threats but has no duty to warn any individual who is
a member of an indeterminate class against whom the patient has made threats.

POINT (2) [15%] ISSUE: If any party is liable for Susan’s injuries, may Susan also obtain damages
for PTSD symptoms? ANSWER: Yes. University is responsible for damages related to Susan’s
injuries because a tort defendant takes his victim as he finds him.

ANSWER DISCUSSION:

A jury could find that University is liable to Susan for failing to take reasonable precautions to protect
Susan from foreseeable criminal activity resulting from unauthorized entrance into the dorm. University
owed a duty of care to Susan, a resident of a dormitory controlled and maintained by University.
University’s failure to fix the broken lock or take other steps to prevent entry by an intruder violated that
duty; it was also the cause in fact and proximate cause of Susan’s physical injuries. A jury could not find
that James is liable to Susan unless Susan relied to her detriment on James’s promise to obtain assistance
or unless James increased the risk that Susan would suffer harm. Here, there is no evidence of reliance or
increased risk as a result of James’s failure to obtain assistance. A jury could not find that Ann’s
Psychiatrist is liable to Susan. Although a therapist may be liable for failing to warn a patient’s intended
victim of credible threats of violence when that victim is reasonably identifiable, in virtually all
jurisdictions, a therapist may not be found liable for failing to warn a victim who is a member of an
indeterminate group. If University is found liable to Susan, it will be responsible for damages related to
Susan’s PTSD injuries because a tort defendant takes his victim as he finds him.

ANSWER EXPLANATION:

Explanation to Point-One(a) (30%):

Seperac-J19 Exam-Released MEE Essay Compilation © 2016-2020 522


University owed a duty of care to Susan, a resident of a dormitory controlled and maintained by
University, to take reasonable precautions to protect Susan from foreseeable criminal activity.
University’s failure to fix the broken lock or take other precautions against unauthorized entry into
Susan’s dorm violated that duty. It was also the cause in fact and proximate cause of Susan’s physical
injuries.

A college does not stand in a parens patriae relationship with its students. However, although the common
law imposed almost no duties on landlords to provide safe premises to tenants, modern courts have found
that landlords, including landlords like University, have a duty to take reasonable precautions to protect
tenants against foreseeable attacks.

[NOTE: An examinee might also conclude that University owed a duty of care to Susan because she was
an invitee. A tenant in common areas of a building is an example of an invitee encountering obvious
dangers.]

Criminal activity of the sort that occurred here was foreseeable. Apartment buildings and college
dormitories almost invariably have locked entrances to protect against intrusion by criminals. Indeed,
recognizing this risk, University had taken steps to ensure that nonresidents could not enter the dormitory.

A jury could find that University’s failure to repair the broken lock or to take other steps to secure the
door through which Ann entered the dormitory represented a breach of its duty of care. The lock had
broken four days before Ann’s entry. Therefore, University employees had ample time to discover the
break. Even if repairs were impossible within the relevant time period, University could have taken other
steps to prevent the door from being opened from the outside. Such precautions were warranted given the
foreseeability that unauthorized persons could enter the dormitory to engage in criminal acts. It is this risk
that prompted University to issue key cards to the dormitory residents. For example, a court found a
railroad liable for theft resulting from its failure to guard a plaintiff’s goods when the theft was
foreseeable.

Because Ann’s entrance into Susan’s dorm was made possible by University’s failure to secure the
damaged door, a jury could find that University’s failure to repair the lock was the cause in fact and
proximate cause of Susan’s physical injuries.

Explanation to Point-One(b) (25%):

There is no general duty to come to the aid of another. James assumed a duty to Susan only if Susan relied
to her detriment on James’s promise to obtain assistance or if James left Susan in a worse position. Here,
the evidence does not show that Susan relied to her detriment or that Susan was in a worse position after
James took charge.

Susan and James did not have a special relationship that created a duty to come to the aid of another.
However, when an actor “takes charge of another who is helpless adequately to aid or protect himself,” he
is subject to liability to the other for bodily harm caused by:

a) “the failure of the actor to exercise reasonable care to secure the safety of the other while within the
actor’s charge, or

b) the actor’s discontinuing aid or protection, if by doing so he leaves the other in a worse position than
when the actor took charge of him.”

Seperac-J19 Exam-Released MEE Essay Compilation © 2016-2020 523


One who undertakes, gratuitously or for consideration, to render services to another is liable for physical
harm resulting from his failure to exercise reasonable care if (1) his failure to exercise such care increases
the risk of such harm; or (2) the harm is suffered because of the other’s reliance upon the undertaking.

Here, James’s failure to follow up his unsuccessful attempt to report Susan’s injuries to the University
security office with any other action was arguably negligent. Moreover, James’s closing the library door
made it less likely that anyone else would come to Susan’s aid. However, there is no evidence that Susan
actually suffered any harm as a result of James’s conduct. Susan obtained medical assistance herself only
half an hour after James left her, and there is no evidence that her minor physical injuries or PTSD
symptoms were aggravated by delayed treatment.

Thus, because there is no evidence that James caused or exacerbated Susan’s injuries, Susan cannot
recover damages from James.

Explanation to Point-One(c) (30%):

A psychotherapist has a duty to warn a reasonably identifiable individual against whom her patient has
made credible threats but has no duty to warn any individual who is a member of an indeterminate class
against whom the patient has made threats.

In Tarasoff v. Regents of University of California, the California Supreme Court held that the special
relationship between a psychotherapist and patient justified the imposition of a duty to warn “persons
threatened by the patient.” Both the holding and reasoning of the Tarasoff court have been widely
adopted. Today, in most states, a psychotherapist who fails to warn an intended victim against whom her
patient has made credible threats of physical violence may be found liable for that victim’s injuries. Many
courts have also followed Tarasoff in permitting recovery either (1) when the therapist believed that the
patient posed a real risk to the specified victim or (2) when the therapist negligently failed to take the
threat seriously.

However, California courts have restricted the duty imposed on psychotherapists in Tarasoff to “potential
victims specifically known and designated” by the dangerous patient. For example, in Thompson, the
court refused to extend Tarasoff to a defendant that had released a youth who had a long history of
violence and who had “indicated that he would, if released, take the life of a young child residing in the
neighborhood.” The court held that, in contrast to Tarasoff, “the warnings sought by plaintiffs would of
necessity have to be made to a broad segment of the population and would be only general in nature such
generalized warnings would do little as a practical matter to stimulate increased safety measures and
would be difficult to give.”

Courts and legislatures in other states have also generally confined the Tarasoff duty to intended victims
who are readily ascertainable and subject to a serious threat of physical violence. Courts generally require
a “serious threat of physical violence against a clearly identified or reasonably identifiable victim” or a
“serious threat of substantial damage to real property.” Cases in which courts have imposed a broader
duty have typically involved defendants who directly facilitated the patient’s attack. For example, where a
psychiatrist helped a patient to obtain guns that were confiscated by the police, the court imposed a duty
on the psychiatrist to the patient's victims.

Under Tarasoff and the case law of most states, the fact that Ann’s Psychiatrist did not find Ann’s threats
credible would not be a defense if the evidence showed that a reasonable therapist would have taken
Ann’s threats seriously. Here, Ann’s Psychiatrist had a therapeutic relationship with Ann because Ann
saw Psychiatrist weekly for several months. That relationship imposed on Psychiatrist a duty to warn

Seperac-J19 Exam-Released MEE Essay Compilation © 2016-2020 524


Ann’s ascertainable intended victims if Ann made serious threats of physical violence against them.
Psychiatrist’s decision not to warn anyone was likely reasonable based on Ann’s lack of any history of
violent behavior and the ambiguity of her vague threat to ensure that cheaters “get what is coming to
them,” which is not a clear threat of serious injury. More importantly, Ann’s threats were general; she did
not specify any ascertainable victims. Thus, in the vast majority of states, a jury could not find that
Psychiatrist is liable to Susan.

Explanation to Point-Two (15%):

University is responsible for damages related to Susan’s injuries because a tort defendant takes his victim
as he finds him.

A tort defendant takes his victim as he finds him. The victim with an “eggshell skull” who suffers injuries
greatly in excess of those that a normal victim would suffer is entitled to recover for the full extent of his
injuries.

Here, although Susan sustained only minor physical injuries from Ann’s attack, Susan suffered a
preexisting condition, PTSD. Susan’s PTSD symptoms that emerged after Ann’s attack were attributable
to her preexisting PTSD. Most of the “eggshell skull” cases involve unusual physical consequences of an
underlying precondition. However, there is no reason why, with proper proof, the plaintiff should not also
recover damages for mental symptoms such as anxiety or insomnia. For example, a court permitted
recovery for a plaintiff’s post-accident schizophrenia when evidence showed that the prior concussion
created a predisposition that was exacerbated by the accident for which defendant was liable for. Courts
have sometimes been reluctant to award tort plaintiffs damages for mental distress unaccompanied by any
physical injuries For example, in one case, the court disallowed mental-disorder damages when the
plaintiff suffered no physical harm in an accident caused by the defendant. However, here Susan did
suffer physical injuries during the attack, and the attack also triggered physical symptoms such as nausea,
muscle tension, and sweating.

Thus, if Susan recovers damages from University for her physical injuries, she should also be able to
recover damages for her PTSD symptoms.

Seperac-J19 Exam-Released MEE Essay Compilation © 2016-2020 525


#115-FEB 2012–MEE Q04: QUESTION FOUR (TORTS)

Paul, age eight, and Paul’s mother, Mom, spent the morning at Funworld, an amusement park. Paul
decided to ride the Ferris wheel. Mom, who was pregnant and tired, waited for him about 100 yards away.

After Paul entered a Ferris wheel car, the attendant, Employee, fastened the car’s safety bar. As the Ferris
wheel began to turn, Paul could hear loud screams from a car carrying two boys, both age six. The boys
were rocking their car vigorously. Employee also heard the two boys screaming and saw them rocking
their car, but Employee took no action to stop them.

As Paul’s car began to descend from the top of the wheel, the two boys – whose car was right behind
Paul’s car – shook the safety bar on their car hard enough that it unlatched. Both boys fell to the ground.
One of the boys struck Paul on his way down.

After the two boys fell, Employee stopped the Ferris wheel and sounded an emergency alarm to notify
Funworld security guards of the incident.

Mom did not see the accident, but she heard the alarm and rushed to the Ferris wheel. A crowd had
already gathered, and Mom was unable to see Paul. A bystander told Mom that “a little boy has been
killed.” Mom, panic-stricken, attempted to make her way through the crowd but could not.

Ten minutes later, the two boys who had fallen were taken to the hospital by an ambulance.

Paul and several of the other passengers begged to be taken off the Ferris wheel. Employee, however,
refused without any explanation to restart the Ferris wheel. Thirty minutes later, a manager showed up
and ordered Employee to restart the Ferris wheel and allow the passengers to exit.

Forty minutes after the accident, Mom was finally reunited with Paul. Both Paul and Mom went to the
hospital, where Paul was treated for minor injuries caused by being hit when the two boys fell and where
Mom suffered a miscarriage as a result of accident-related stress.

National accident records show that during the last 40 years, there has been only one other incident in
which injuries have occurred as a result of passengers rocking a Ferris wheel car.

Paul and Mom have sued Funworld. Funworld has conceded that Employee was acting within the scope
of his employment.

1. Based on the facts, could a jury properly find that Funworld falsely imprisoned Paul? Explain.

2. Based on the facts, could a jury properly find that Funworld was negligent because Employee
failed to take action to stop the boys from rocking their car? Explain.

3. Based on the facts, could a jury properly find that Mom is entitled to damages for her emotional
distress and resulting miscarriage? Explain.

Seperac-J19 Exam-Released MEE Essay Compilation © 2016-2020 526


#115: F12-4 MEE: ANSWER: NCBE (TORTS)

POINT (1) [40%] ISSUE: Could a jury properly find that Employee falsely imprisoned Paul by
stopping the Ferris wheel for 30 minutes? ANSWER: Yes. Paul can establish the elements of a
prima facie case of false imprisonment, and there is no defense here to a false imprisonment claim.

POINT (2) [25%] ISSUE: Could a jury properly find that Funworld was negligent because
Employee failed to take action to stop the two boys from rocking their car? ANSWER: Yes. A
reasonable jury could properly conclude that Funworld was negligent for failing to take any action
to stop the two boys from rocking the Ferris wheel car.

POINT (3) [35%] ISSUE: Could a jury properly find that Mom is entitled to recover for her
emotional distress and resulting miscarriage? ANSWER: No. Mom is unlikely to recover for her
emotional distress and resulting miscarriage because she was not in the zone of danger nor did she
contemporaneously observe an injury to Paul.

ANSWER DISCUSSION:

A jury could properly find that Employee falsely imprisoned Paul, because Employee knew with
substantial certainty that Paul would be confined in the Ferris wheel car when he refused to restart the
Ferris wheel, and the facts do not establish any defense to a false imprisonment claim. Because Employee
was acting within the scope of his employment, a jury could impose liability on Funworld for Employee’s
actions. A jury could properly find that Employee’s failure to take action to stop the two boys from
rocking the car was negligent and that it caused Paul’s injuries even though there has been only one
similar accident over the past 40 years. It is unlikely that Mom can recover for her emotional distress and
resulting miscarriage because she was not in the zone of danger and did not contemporaneously observe
any injury to Paul.

ANSWER EXPLANATION:

Explanation to Point-One (40%):

Paul can establish the elements of a prima facie case of false imprisonment, and there is no defense here
to a false imprisonment claim.

An actor is subject to liability to another for false imprisonment if “(a) he acts intending to confine the
other or a third person within boundaries fixed by the actor, and (b) his act directly or indirectly results in
such a confinement of the other, and (c) the other is conscious of the confinement or is harmed by it.”

Confinement need not be physical, and it “need not be stationary.” The individual confined is not required
to endanger his person through escape efforts. Confinement may be by actual or apparent physical
barriers.

The requisite intent to confine can be shown through either motivation or knowledge. An actor may be
found liable for false imprisonment both when “his act was done for the purpose of imposing confinement
upon the other” and when he acted “with knowledge that such confinement would, to a substantial
certainty, result from his actions.”

Seperac-J19 Exam-Released MEE Essay Compilation © 2016-2020 527


Here, Employee confined Paul by stopping the Ferris wheel. Given Paul’s position in midair, he could not
leave the car in which he was riding without endangering his person. Employee knew that Paul would be
unable to leave the car when he acted. Thus, Paul can establish a prima facie case of false imprisonment.
And because Employee was acting within the scope of his employment, Funworld is vicariously liable for
his conduct.

Employee’s actions would have been privileged if stopping the Ferris wheel had been necessary in order
to protect “some interest of the actor or of the public which is of such importance as to justify the harm
caused or threatened by its exercise.” For example, a court held that a school bus driver who drove
straight to police station instead of making regular stops was privileged to confine all students riding the
bus when some students had committed acts of vandalism and continued to be abusive after being warned.
Thus, for example, if Employee had stopped the Ferris wheel briefly to permit rescue workers to remove
the two boys from the scene, Employee’s conduct would likely have been privileged. However,
Employee, for no apparent reason, failed to start the Ferris wheel for 30 minutes after the two boys were
taken to the hospital.

Therefore, a jury could properly find that Employee falsely imprisoned Paul. Because Employee was
acting within the scope of his employment, the jury could also find that Funworld was liable to Paul for
injuries caused by Employee’s actions.

While an argument could be made that Paul consented to being confined on the Ferris wheel, Paul’s
consent to be on the wheel was withdrawn once he begged to be taken off the wheel.

Explanation to Point-Two (25%):

A reasonable jury could properly conclude that Funworld was negligent for failing to take any action to
stop the two boys from rocking the Ferris wheel car.

In any negligence action, a plaintiff must show that the defendant owed the plaintiff a duty to conform his
conduct to a standard necessary to avoid an unreasonable risk of harm to others, that the defendant’s
conduct fell below the applicable standard of care, and that the defendant’s conduct was both the cause in
fact and the proximate cause of the plaintiff’s injuries. Paul would have no difficulty in establishing that
Funworld owed him a duty of care. Paul was a business visitor, and Funworld thus owed Paul reasonable
care to avoid foreseeable risks. Because Employee was acting within the scope of his employment, the
jury could also find that Funworld was liable to Paul for injuries caused by Employee’s negligence.

In determining whether Employee’s conduct was negligent, the jury should measure that conduct against
the conduct of a reasonable, prudent person engaged in a like activity. A reasonable, prudent person takes
precautions to avoid foreseeable risks, even if those risks have rarely materialized. If a Ferris wheel car is
rocking vigorously, there is a small but nonetheless foreseeable risk that persons riding within the car or
objects they are carrying may fall and cause injuries. Given the foreseeability of serious harm, and the
small burden of taking steps to prevent the harm, a jury could properly conclude that Employee was
negligent in failing to take any action to stop the two boys from rocking the car.

Explanation to Point-Three (35%):

Mom is unlikely to recover for her emotional distress and resulting miscarriage because she was not in the
zone of danger nor did she contemporaneously observe an injury to Paul.

Seperac-J19 Exam-Released MEE Essay Compilation © 2016-2020 528


Although early cases typically denied recovery for negligent infliction of emotional distress to a plaintiff
who experienced no physical contact or injury, today virtually all American jurisdictions have abandoned
this approach; a plaintiff who is within the zone of danger created by the defendant and who suffers a
physical manifestation of emotional distress occasioned by a threatened injury may recover in virtually all
states. Here, however, Mom did not apprehend any personal danger. Her emotional harm and resulting
miscarriage were due to her fears for Paul’s safety.

In evaluating claims for emotional distress resulting from accidents involving other individuals, American
jurisdictions have taken two different approaches. Some disallow recovery unless the plaintiff was herself
within the zone of danger. Thirteen jurisdictions follow the "zone-of-danger" approach. A larger group
permits recovery if the plaintiff was closely related to the victim, was located near the scene of the
accident, and suffered shock resulting from “the sensory and contemporaneous observance of the
accident.” Thirty jurisdictions follow some variant of this "Dillon" approach. In virtually all jurisdictions,
emotional distress must result from sensory and contemporaneous observance of the accident itself, not
the receipt of news relating to the accident. “All of us can expect at least once in our lives to be informed
of the serious injury or death of a close family member. Although the shock and grief growing out of such
news is great, it is not compensable emotional distress.” Some jurisdictions that follow the Dillon
approach limit recovery to relatives who were at the scene when the accident happened. Some permit
recovery by a relative who comes upon the scene shortly after the accident happens. Only a handful of
jurisdictions permit recovery based simply on the foreseeability of mental distress. Of these nine
jurisdictions that use a foreseeability approach, they typically require arrival at the scene shortly after the
injury. For example, one court permitted recovery by a wife who saw her husband’s injuries at the
hospital.

Here, Mom was not within the zone of danger; she was some 100 yards away. Moreover, Mom’s fright
was occasioned solely by an erroneous report that “a little boy has been killed.” Although Mom was
located near the scene of the accident, she did not see or hear the accident; she arrived on the scene after
hearing emergency alarms. Indeed, even after she arrived, Mom could neither see nor hear Paul (nor could
she see any other injured person) until several minutes after Paul exited the Ferris wheel, which was 40
minutes after the accident took place.

Thus, in virtually all jurisdictions, Mom will not be able to recover for her distress and resulting
miscarriage even if Funworld is found negligent in failing to prevent Paul’s injury. Mom was not within
the zone of danger, and her emotional harm stemmed from an erroneous report that a boy had been killed,
not from sensory and contemporaneous observance of injuries to Paul.

Seperac-J19 Exam-Released MEE Essay Compilation © 2016-2020 529


#116-FEB 2011–MEE Q04: QUESTION FOUR (TORTS)

After recent terrorist threats, Metro Opera (Metro) decided to place metal detectors in its lobby. Metro
also marked off an area just beyond the metal detectors in which to search patrons who failed the metal-
detector test. Metro posted a sign near the entrance that read: “Warning! No metal objects allowed inside.
All entrants are screened and may be searched.”

Claimant and Friend saw the warning sign as they entered Metro. After entering, they observed several
patrons being frisked. Claimant said to Friend, “I’m certainly not going to allow anyone to touch me!”

Claimant then walked through the metal detector, which buzzed. Without asking Claimant’s permission,
Inspector, a Metro employee, approached Claimant from behind and began to frisk Claimant. Claimant
leaped away from Inspector and snarled, “Leave me alone!” Guard, another Metro employee, then used a
stun device, which administers a painful electric shock, to subdue Claimant.

Unfortunately, the stun device, manufactured by Alertco, malfunctioned and produced a shock
considerably more severe than that described in Alertco’s product specifications. The shock caused minor
physical injuries and triggered a severe depressive reaction that necessitated Claimant’s hospitalization.
Claimant had a history of depression but was in good mental health at the time of the shock. Claimant was
the first person who had ever experienced a depressive reaction to the Alertco device.

The Alertco device malfunctioned because it was incorrectly assembled at the factory and therefore did
not meet Alertco’s specifications. Alertco’s assembly-inspection system exceeds industry standards, and it
is widely recognized as the best in the industry. Nonetheless, it did not detect the assembly mistake in the
device that injured Claimant.

Claimant has filed two tort actions seeking damages for her physical and psychological injuries: (1)
Claimant sued Metro, claiming that both the frisk and the use of the stun device were actionable batteries,
and (2) Claimant brought a strict products liability action against Alertco.

Metro has conceded that the actions of Inspector and Guard were within the scope of their employment.
Metro had instructed its employees to ask permission before frisking patrons, but on the day Claimant was
frisked, a supervisor told employees to frisk without asking permission in order to speed up the entrance
process.

1. Can Claimant establish a prima facie case of battery against Metro for (a) the use of the stun
device and (b) the frisk? Explain.

2. Does Metro have a viable defense to either battery claim? Explain.

3. Can Claimant establish the elements of a strict products liability claim against Alertco based on
the malfunction of the device? Explain.

4. Assuming that Claimant establishes either Metro’s or Alertco’s liability, can Claimant recover
for her depressive reaction to the stun device? Explain.

Seperac-J19 Exam-Released MEE Essay Compilation © 2016-2020 530


#116: F11-4 MEE: ANSWER: NCBE (TORTS)

POINT (1)(a) [20%] ISSUE: Did use of a stun device to shock Claimant establish a prima facie case
of battery? ANSWER: Yes. A prima facie case of battery is established when the plaintiff shows
that she suffered a harmful or offensive contact and that the defendant intended to cause such a
contact. Guard’s use of a stun device against Claimant satisfies this test. Thus, because Guard was
acting within the scope of his employment, Claimant can establish a prima facie case against Metro.

POINT (1)(b) [20%] ISSUE: Did frisking Claimant establish a prima facie case of battery?
ANSWER: No. It is unclear whether Inspector’s frisk of Claimant establishes a prima facie case of
battery. It is possible that Claimant will not be able to show intent to cause a harmful or offensive
contact.

POINT (2) [30%] ISSUE: Did Claimant consent to either the frisk or the stun-device shock?
ANSWER: Because conduct can manifest consent, Claimant may have consented to being frisked.
Claimant did not manifest consent to being shocked with the stun device.

POINT (3) [15%] ISSUE: Is the manufacturing defect in the Alertco device actionable in strict
products liability? ANSWER: Yes. A manufacturer is strictly liable for a defective product.
Alertco is thus liable for the defective stun device even though Claimant cannot show that Alertco
was negligent.

POINT (4) [15%] ISSUE: Can Claimant recover damages for her depressive reaction to the
malfunctioning stun device even though that depressive reaction was unforeseeable? ANSWER:
Yes. Because a tort defendant “takes his victim as he finds him,” Claimant may recover for her
unforeseeable depressive reaction to the stun device.

ANSWER DISCUSSION:

A prima facie case of battery is established when the plaintiff shows that she suffered a harmful or
offensive contact and that the defendant intended to cause such a contact. Guard’s use of a stun device
against Claimant meets this test because Guard intentionally administered a painful and physically
incapacitating shock to Claimant’s body. However, it is unclear whether Inspector’s frisk of Claimant’s
body satisfies this test, as such a frisk may not be sufficiently offensive to constitute battery and Inspector
may have been unaware that Claimant would be offended by the frisk. Moreover, because conduct can
manifest consent, Claimant may have consented to the frisk when she walked through the metal detector.
On the other hand, Claimant did not manifest consent to being shocked with the stun device. A
manufacturer is strictly liable for manufacturing defects. Alertco is thus liable for the defective stun
device because the defect arose during the manufacturing process. Because a tort defendant “takes his
victim as he finds him,” Claimant may recover for her unforeseeable depressive reaction to the stun
device.

ANSWER EXPLANATION:

Explanation to Point-One(a) (20%):

A prima facie case of battery is established when the plaintiff shows that she suffered a harmful or
offensive contact and that the defendant intended to cause such a contact. Guard’s use of a stun device

Seperac-J19 Exam-Released MEE Essay Compilation © 2016-2020 531


against Claimant satisfies this test. Thus, because Guard was acting within the scope of his employment,
Claimant can establish a prima facie case against Metro.

An actor is subject to liability to another for battery if (a) he acts intending to cause a harmful or offensive
contact with the person and (b) a harmful or offensive contact results. Intent, for purposes of establishing
a battery, means either that “the actor desires to cause the consequences of his act” or that he “knows that
the consequences are certain, or substantially certain, to result from his act.”

Guard’s use of a stun device to subdue Claimant meets these standards for an actionable battery. First, the
use of a stun device both incapacitates the target and causes the target to suffer pain. Either effect
constitutes “bodily harm” for purposes of establishing a prima facie case of battery. Bodily harm includes
any physical impairment of the condition of another’s body, or physical pain or illness. Second, Guard
certainly used the stun device for the purpose of physically impairing Claimant. His goal was to make it
easier to subdue Claimant by causing physical impairment. Furthermore, although the facts do not say so
explicitly, it is a reasonable assumption that Guard was aware that stun devices cause pain when they are
used against someone. Thus, Guard’s actions amount to battery: Guard intended to, and did, cause a
harmful contact with Claimant. Because Guard was acting within the scope of his employment, Claimant
can establish a prima facie case against Metro.

Explanation to Point-One(b) (20%):

It is unclear whether Inspector’s frisk of Claimant establishes a prima facie case of battery. It is possible
that Claimant will not be able to show intent to cause a harmful or offensive contact.

Inspector’s frisk of Claimant was not harmful, but a contact can also constitute battery if it is offensive.
“A bodily contact is offensive if it offends a reasonable sense of personal dignity.” The facts make clear
that Claimant found the contact offensive, but this is not determinative. Claimant must additionally show
that being frisked offends a “reasonable” sense of dignity.

The frisk in this case was part of a routine screening process, and screening like this is now frequently
encountered in airports and other public places. The routine nature of such screening may mean that it is
no longer “reasonable” to be offended by it. On the other hand, the fact that Metro generally required
employees to ask permission before frisking opera patrons suggests that frisking is often experienced by
people as offensive, especially if it occurs without permission. It is thus possible, but not certain, that a
routine frisk would be viewed by a court as a contact that “offends a reasonable sense of personal
dignity.”

Even if the frisk were considered offensive, a prima facie case of battery based on the frisk cannot be
established unless Inspector is shown to have acted with intent to cause an offensive contact. To establish
intent, Claimant must show that Inspector had the purpose of causing offense or knew he would cause
offense by frisking Claimant. It is not enough to show that Inspector may have been aware of a risk that
some patrons would be offended by a frisk. Battery’s protection against “offensive contact” is intended to
protect a person’s “dignitary” interest. That interest “is protected only against intentional invasion. The
actor is not liable for an act which involves a risk, no matter how great and unreasonable, that it will cause
an offensive contact.”

Because opera patrons were warned of the possibility of being frisked, Inspector may have assumed that
they would not be offended by such a contact. On the other hand, Inspector did not warn Claimant before
initiating the frisk in question, and it is unclear in what manner Inspector conducted that frisk. Claimant
can establish a prima facie case if she can present some facts suggesting that Inspector knew with

Seperac-J19 Exam-Released MEE Essay Compilation © 2016-2020 532


substantial certainty that Claimant’s “reasonable sense of personal dignity” would be offended by
Inspector’s decision to touch her without warning, or that Inspector knew that the manner in which he
touched Claimant would be offensive to her. But, absent some facts suggesting Inspector’s knowledge
that the frisk would be offensive, the mere fact that Inspector intentionally contacted Claimant’s person
does not satisfy the requirements for an actionable battery.

[NOTE: The examinee’s analysis is more important than his or her conclusion.]

Explanation to Point-Two (30%):

Because conduct can manifest consent, Claimant may have consented to being frisked. Claimant did not
manifest consent to being shocked with the stun device.

Consent – “willingness in fact for conduct to occur” – is a defense to battery. Consent “may be manifested
by action or inaction and need not be communicated to the actor.” Thus, “if words or conduct are
reasonably understood by another to be intended as consent, they constitute apparent consent and are as
effective as consent in fact.”

Claimant did not intend to consent to Inspector’s search of her person. Claimant told Friend, “I’m
certainly not going to allow anyone to touch me,” and told Inspector, “Leave me alone!”

However, a defendant is entitled to rely on objective manifestations of consent. Thus, because Claimant
read the warning sign, saw opera patrons being frisked, and still proceeded through the metal detector,
Metro can argue that its employee, Inspector, was entitled to infer that Claimant had consented to being
frisked herself. For example, in a case regarding vaccinations, the court found that the plaintiff consented
to vaccination when she stood in line with other passengers and there was nothing in her conduct to
indicate that she did not wish to obtain a vaccination card.

On the other hand, the fact that Metro usually required its employees to request permission before frisking
patrons suggests that it did not infer consent to frisking from a patron’s decision to proceed through the
metal detector. A court thus might conclude that it was unreasonable to infer Claimant’s consent to being
frisked.

Metro has no possible argument that Claimant consented to being shocked with a stun device. Claimant
did not see the stun device in use, and there were no signs warning of its use.

There are no other defenses available to Metro. The privilege to exercise force in self-defense or defense
of others is unavailable because Claimant’s statement (“Leave me alone!”) could not reasonably be
interpreted as a threat. (“An actor is privileged to use reasonable force to defend himself against
unprivileged harmful or offensive contact or other bodily harm which he reasonably believes that another
is about to inflict.”).

Explanation to Point-Three (15%):

A manufacturer is strictly liable for a defective product. Alertco is thus liable for the defective stun device
even though Claimant cannot show that Alertco was negligent.

Alertco’s product contained a manufacturing defect; it failed to meet Alertco’s own specifications, and the
defect occurred during the product’s manufacture. In order to recover for injuries sustained because of a
manufacturing defect, a plaintiff need not show that the manufacturer was negligent. The manufacturer is

Seperac-J19 Exam-Released MEE Essay Compilation © 2016-2020 533


strictly liable whenever “the product departs from its intended design even though all possible care was
exercised in the preparation and marketing of the product.” In this case, the evidence establishes that the
defect occurred during the manufacturing process. Thus, despite Alertco’s excellent quality control, it is
strictly liable to Claimant.

Explanation to Point-Four (15%):

Because a tort defendant “takes his victim as he finds him,” Claimant may recover for her unforeseeable
depressive reaction to the stun device.

“When an actor’s tortious conduct causes harm to a person that, because of a preexisting physical or
mental condition or other characteristics of the person, is of a greater magnitude or different type than
might reasonably be expected, the actor is nevertheless subject to liability for all such harm to the
person.” Thus, the plaintiff with an “eggshell skull” who suffers injuries different from or greater than
those that a normal victim would suffer is entitled to recover fully for his injuries.

Claimant’s history of depression is a preexisting condition, like an eggshell skull. Thus, despite the fact
that a depressive reaction to the stun device was unforeseeable, Alertco is liable for both Claimant’s
physical and psychological injuries.

Seperac-J19 Exam-Released MEE Essay Compilation © 2016-2020 534


#117-FEB 2010–MEE Q04: QUESTION FOUR (TORTS)

Penny lives in an apartment on Oak Street across from the Fernbury Baseball Park (“the Park”). The Park
is owned and maintained by the Fernbury Flies, a professional minor league baseball team. As she left her
apartment building one day, Penny was struck in the head by a baseball that had been hit by Dennis, a
Flies player, during a game.

The section of Oak Street that adjoins the Park was once lined with single-family homes. Over the past
two decades, these homes have been replaced by stores and apartment buildings, causing an increase in
both car and pedestrian traffic on Oak Street.

The ball that struck Penny was one of the longest that had been hit at the Park since its construction 40
years ago. During the last 40 years, Flies’ records show that only 30 balls had previously been hit over the
Park fence adjoining Oak Street. Fifteen of the balls hit out of the Park onto Oak Street were hit during
the past decade.

The Park is surrounded by a 10-foot-high fence, which was built during the Park’s construction. All other
ballparks owned by clubs in the Flies’ league are surrounded by fences of similar type and identical
height. These fences are typical of those used by other minor league teams in the United States. However,
in Japan, where ballparks are often located in congested urban neighborhoods, netting is typically attached
to ballpark fences. This netting permits balls to go over a fence but captures balls before they can strike a
bystander or car.

After being struck by the ball, Penny was taken by ambulance to a hospital emergency room. After tests,
the treating physician told Penny that she had suffered a concussion. The physician prescribed pain
medication for Penny. However, because of a preexisting condition, she had an adverse reaction to the
medication and suffered neurological damage resulting in the loss of sensation in her extremities.

Penny has sued Dennis, the player who hit the baseball that struck her, for battery and negligence. Penny
has also sued the Fernbury Flies. She seeks to recover damages for the concussion and the neurological
damage resulting from the medication.

1. Does Penny have a viable tort claim against Dennis? Explain.

2. Does Penny have a viable tort claim against the Fernbury Flies? Explain.

Seperac-J19 Exam-Released MEE Essay Compilation © 2016-2020 535


#117: F10-4 MEE: ANSWER: NCBE (TORTS)

POINT (1) [25%] ISSUE: What must Penny establish in a battery action against Dennis?
ANSWER: Dennis did not commit a battery when the ball he hit struck Penny. Therefore Penny
does not have a viable battery action against Dennis.

POINT (2) [25%] ISSUE: What must Penny establish in a negligence action against Dennis?
ANSWER: Dennis was not negligent in hitting the ball that struck Penny over the Oak Street fence.
Therefore Penny does not have a viable negligence claim against Dennis.

POINT (3) [10%] ISSUE: What must Penny establish in an action against the Flies based on the
team’s employment relationship with Dennis? ANSWER: Penny does not have a viable claim
against the Flies based on the team’s employment of Dennis.

POINT (4) [25%] ISSUE: What must Penny establish in a negligence action against the Flies?
ANSWER: Penny may have a viable negligence action against the Flies based on the team’s failure
to attach netting to the Oak Street fence despite the fact that the fence conforms to customary
standards within professional baseball.

POINT (5) [15%] ISSUE: If Penny succeeds in her action against either Dennis or the Flies, can she
recover for damages for the neurological harm that resulted from a preexisting condition?
ANSWER: Yes. Because a tort defendant “takes his victim as he finds him,” Penny could recover
for harm suffered due to her adverse reaction to medication.

ANSWER DISCUSSION:

Penny does not have a viable battery action against Dennis because Dennis neither intended, nor knew
with substantial certainty, that the ball he hit out of the Park would strike anyone. Penny does not have a
viable negligence action against Dennis because there is no reasonable means by which Dennis could
have avoided hitting Penny. Because Penny does not have a viable claim against Dennis, she has no
viable claim against the Flies based on the team’s employment of Dennis even though Dennis was acting
within the scope of his employment. However, Penny might have a viable negligence action against the
Flies based on the team’s failure to attach netting to the fence adjoining Oak Street. The fact that the Flies
have conformed to customary standards for minor league baseball fence construction is relevant but not
determinative. If the jury finds that the Flies’ failure to install netting along the Oak Street fence was
negligent, Penny could recover for harm suffered as a result of her adverse reaction to medication even
though this harm resulted from a preexisting condition.

ANSWER EXPLANATION:

Explanation to Point-One (25%):

Dennis did not commit a battery when the ball he hit struck Penny. Therefore Penny does not have a
viable battery action against Dennis.

In a battery action, the plaintiff must show that the defendant intentionally caused a harmful or offensive
bodily contact. A defendant intentionally causes such a contact if he “acts with the desire to bring about
that harm” or engages “in action knowing that harm is substantially certain to occur.” In this case, Dennis

Seperac-J19 Exam-Released MEE Essay Compilation © 2016-2020 536


neither desired to bring about a harmful contact between the baseball and Penny nor, in light of his
location inside the Park, could he have known that such a contact would or was substantially certain to
occur. As a result, Penny does not have a viable battery claim against Dennis.

Explanation to Point-Two (25%):

Dennis was not negligent in hitting the ball that struck Penny over the Oak Street fence. Therefore Penny
does not have a viable negligence claim against Dennis.

In a negligence action, the plaintiff must show that the defendant owed the plaintiff a duty to conform his
conduct to a standard necessary to avoid an unreasonable risk of harm to others, that the defendant’s
conduct fell below the applicable standard of care, and that the defendant’s conduct was both the cause in
fact and the proximate cause of the plaintiff’s injuries.

In determining whether Dennis’s conduct fell below the standard of care, the jury would measure that
conduct against the actions of a reasonable, prudent person engaged in a like activity. A reasonable,
prudent person takes appropriate precautions to avoid foreseeable risks; in measuring whether a particular
precaution was warranted, the jury weighs the burden of taking such precautions against the gravity of the
risk and the likelihood that it will eventuate.

Here the burden of taking the precaution against the risk is high. The only meaningful precautions that
Dennis might have taken were not hitting the ball at all or trying to hit it with less than maximum force.
Either precaution would generally be inconsistent with Dennis’s job as a professional baseball player,
which includes the obligation to hit the ball and to hit a home run if possible. For Dennis, the cost of
taking precautions could mean the loss of his career.

The gravity of the risk created by his hitting the ball cannot be determined easily. Being hit with an errant
baseball could cause harm ranging from minimal bruising to far more serious injury. The likelihood of the
harm occurring is low – in 40 years only 30 balls had previously been hit into Oak Street.

Thus Dennis was not negligent in hitting the ball that caused Penny’s injury.

Explanation to Point-Three (10%):

Penny does not have a viable claim against the Flies based on the team’s employment of Dennis.

An employer is vicariously liable for the tortious actions of his employee that are within the scope of the
tortfeasor’s employment. In this case, there is no question that Dennis was acting within the scope of his
employment; he was a baseball player engaged in hitting a baseball. But Penny does not have a viable tort
action against Dennis because Dennis’s conduct was not tortious. Therefore, the Flies are not vicariously
liable for his conduct.

Explanation to Point-Four (25%):

Penny may have a viable negligence action against the Flies based on the team’s failure to attach netting
to the Oak Street fence despite the fact that the fence conforms to customary standards within professional
baseball.

Just as a jury would measure Dennis’s conduct against the actions of a reasonable, prudent person
engaged in a like activity, it would measure the Flies’ conduct in constructing and maintaining the Oak

Seperac-J19 Exam-Released MEE Essay Compilation © 2016-2020 537


Street fence against that of a reasonably prudent ballpark owner. And while Dennis had no means of
avoiding the injury to Penny other than not hitting the ball, the Flies could have added netting to the Oak
Street fence, and that netting would have prevented Penny’s injury.

Custom is relevant in a negligence action, but it is not determinative. Thus, even though the Flies’
conduct conformed to the industry standard, Penny may succeed in her negligence action if she can
establish that the cost of adding netting to the fence was relatively modest in relation to the risk of injuries
from balls exiting the Park onto Oak Street. Given the changed character of the street, the increasing
number of balls hit onto the street in recent years, and the widespread adoption of netting in another
country, it is possible that Penny may succeed in making such a showing.

Explanation to Point-Five (15%):

Because a tort defendant “takes his victim as he finds him,” Penny could recover for harm suffered due to
her adverse reaction to medication.

A tort defendant takes his victim as he finds him. The plaintiff with an “eggshell skull” who suffers
damage greatly in excess of those that a normal victim would suffer thus is entitled to recover fully for his
injuries. Because Penny’s sensitivity to the prescribed medication was a preexisting condition, if Penny
succeeds in her lawsuit against the Flies, she could recover for all injuries suffered as a result of that
sensitivity.

Seperac-J19 Exam-Released MEE Essay Compilation © 2016-2020 538


#118-FEB 2009–MEE Q07: QUESTION SEVEN (TORTS)

Tenant lives in Landlord’s apartment building. The furnace in the building was inoperable during three
periods last winter, causing the loss of heat and hot water. On each of those occasions, Landlord made
temporary repairs.

On March 25, the furnace again broke down. Landlord was promptly notified of the problem and he
ordered the parts needed to fix the furnace on March 26, but they did not arrive until April 6, at which
time Landlord fixed the furnace. Between March 25 and April 5, there was no heat or hot water in the
building.

In order to bathe from March 25 through April 5, Tenant heated a large pot of water on the stove. After
the water boiled, Tenant transferred the water to the bathtub, mixed in cold water, and then used the water
to bathe.

On April 3, Nephew, Tenant’s eight-year-old nephew, arrived for a visit. On April 4, Tenant was carrying
a pot of boiling water down the hall to the bathroom when Nephew, who was chasing a ball out of a
bedroom that opened into the hall, collided with Tenant. As a result of the collision, the hot water spilled
on Nephew, seriously burning him. Nephew did not look or call out before running into the hall.

A state statute provides that “every apartment building . . . and every part thereof shall be kept in good
repair. The owner shall be responsible for compliance . . . . A violation shall be punishable by a fine not
exceeding $500.”

Nephew, by his guardian, sued Tenant and Landlord for damages. At trial, both Tenant and Landlord
argued that Nephew’s negligence was the sole cause of the accident.

Based on these facts, may the jury properly award Nephew damages for his personal injury:

1. From Tenant? Explain.

2. From Landlord? Explain.

Seperac-J19 Exam-Released MEE Essay Compilation © 2016-2020 539


#118: F09-7 MEE: ANSWER: NCBE (TORTS)

POINT (1) [20%] ISSUE: Does Nephew have a cause of action against Tenant for negligence?
ANSWER: Yes. A reasonable jury could conclude that Tenant was negligent and that his
negligence caused Nephew’s injuries.

POINT (2) [20%] ISSUE: What is the standard of care applicable to Nephew? ANSWER: Although
Nephew’s age makes it unlikely, a jury could find that Nephew was negligent and that Nephew’s
negligence was a contributing factor in causing his injuries.

POINT (3) [15%] ISSUE: Will Nephew’s negligence prevent Nephew from recovering damages
from Tenant or Landlord? ANSWER: No. Even if the jury found that Nephew was negligent in
running into the hall, under modern comparative negligence rules, Nephew’s negligence would
reduce his recovery from Tenant but would not eliminate Tenant’s liability.

POINT (4) [20%] ISSUE: Did Landlord’s violation of a state statute requiring that every part of a
building be kept “in good repair” represent negligence per se? ANSWER: The unexcused violation
of a statutory standard is negligence per se if the statute was designed to protect against the type of
accident the actor’s conduct caused and the injured party is within the class of persons the statute
was designed to protect. Under this standard, it is unclear whether Landlord’s conduct constituted
negligence per se.

POINT (5) [25%] ISSUE: Was Landlord’s failure to repair the furnace a proximate cause of
Nephew’s injuries? ANSWER: No. In a negligence action, a defendant is liable only if his conduct
was the proximate cause of the plaintiff’s injury. A reasonable jury could find that the type of harm
Nephew suffered was too remote from Landlord’s negligence in failing to provide adequate heat
and hot water.

ANSWER DISCUSSION:

The jury could award Nephew damages in his action against Tenant. In a negligence action, an adult
defendant’s conduct is measured against that of a reasonable, prudent person engaged in a similar activity;
a minor’s conduct is measured against that of a minor of like age, intelligence, and experience. The
determination of whether a party’s conduct conforms to the applicable standard of care is a question of
fact that is normally left to the jury. Here the jury could conclude that Tenant was negligent and that his
negligence was the cause in fact and proximate cause of Nephew’s injuries. Even if the jury also
concluded that Nephew was negligent, under modern comparative negligence rules Nephew could still
recover from Tenant. It is less clear that the jury could award Nephew damages in his action against
Landlord. Although violation of a state statute is normally considered negligence per se, it is unclear that
the injuries Nephew suffered were within the category of harms the legislature aimed to prevent when
enacting the statute. It is equally unclear whether Landlord’s failure to repair the furnace was a proximate
cause of the injuries.

ANSWER EXPLANATION:

Explanation to Point-One (15-25%):

Seperac-J19 Exam-Released MEE Essay Compilation © 2016-2020 540


A reasonable jury could conclude that Tenant was negligent and that his negligence caused Nephew’s
injuries.

In any negligence action, a plaintiff must show that the defendant owed the plaintiff a duty to conform his
conduct to a standard necessary to avoid an unreasonable risk of harm to others, that the defendant’s
conduct fell below the applicable standard of care, and that the defendant’s conduct was both the cause in
fact and the proximate cause of his injuries.

Nephew would have no difficulty in establishing that Tenant owed him a duty of care. Nephew was
Tenant’s household guest whose presence was known to Tenant. Thus, Tenant owed Nephew an
obligation to exercise reasonable care to avoid foreseeable risks.

In determining whether Tenant’s conduct fell below the standard of care, the jury would measure that
conduct against the conduct of a reasonable, prudent person engaged in a like activity. A reasonable,
prudent person takes precautions to avoid foreseeable risks. A large pot of boiling water, if it spills,
clearly poses a foreseeable risk of serious burns. A jury might also conclude that it was foreseeable that
someone would come out of the bedroom quickly and collide with the boiling water. The burden of taking
precautions to avert this possibility was small; Tenant could have yelled, “I’m coming with a pot of
boiling water,” or closed the bedroom door. Given the foreseeability of serious harm and the small burden
of taking precautions, a jury might conclude that Tenant was negligent.

Tenant’s conduct also was the cause in fact and proximate cause of the injuries. Tenant’s failure to take
precautions appears to have been a substantial factor in producing the accident. Burns were a foreseeable
result of collision with a pot containing boiling water. Thus, based on the facts, a jury could properly
award damages to Nephew.

Explanation to Point-Two (15-25%):

Although Nephew’s age makes it unlikely, a jury could find that Nephew was negligent and that
Nephew’s negligence was a contributing factor in causing his injuries.

Nephew, like Tenant, had a duty to prevent foreseeable risks to himself and others. In judging whether
Nephew exercised reasonable care under the circumstances, the jury would measure Nephew’s conduct
against that of a minor of like age, intelligence, and experience. A few jurisdictions apply the so-called
“tender years doctrine,” under which a minor of less than seven years of age cannot be found negligent,
but Nephew was age eight when the accident occurred.

Nephew chased a ball into the hall without looking or calling out. To an adult, this conduct would create a
foreseeable risk of collision with someone walking in the hall. Age is the key factor here. A normal
toddler does not foresee risks, particularly when engaged in an activity like chasing a ball; a normal
adolescent does foresee such risks. Nephew, age eight, falls between a toddler and an adolescent.
Therefore, it is possible, but not likely, that a jury would find that Nephew’s conduct was negligent.

Explanation to Point-Three (10-20%):

Even if the jury found that Nephew was negligent in running into the hall, under modern comparative
negligence rules, Nephew’s negligence would reduce his recovery from Tenant but would not eliminate
Tenant’s liability.

Seperac-J19 Exam-Released MEE Essay Compilation © 2016-2020 541


At common law, if the jury found the plaintiff’s negligence to be a cause in fact and proximate cause of
his injuries, the plaintiff could not recover from the defendant. This all-or-nothing approach was
frequently criticized; it has now been abandoned by virtually all states.

Under the modern “comparative negligence” approach, if the jury finds that two or more parties are
negligent, it apportions fault between them. The plaintiff’s fault share is subtracted from the total damages
awarded by the jury. Thus, even if the jury found Nephew to be negligent, that finding would reduce his
recovery from Tenant but would not eliminate Tenant’s liability, except in a state that has preserved the
all-or-nothing approach to contributory negligence or in a comparative negligence state where recovery is
barred if the victim was more than 50 percent negligent.

Explanation to Point-Four (15-25%):

The unexcused violation of a statutory standard is negligence per se if the statute was designed to protect
against the type of accident the actor’s conduct caused and the injured party is within the class of persons
the statute was designed to protect. Under this standard, it is unclear whether Landlord’s conduct
constituted negligence per se.

The governing state statute requires that “every apartment building and every part thereof shall be kept in
good repair.” Arguably, Landlord violated the statute, as the furnace was “part” of the “apartment
building” owned by Landlord, and it malfunctioned on three different occasions before Landlord took
steps to correct the problem permanently. “An actor is negligent if, without excuse, the actor violates a
statute that is designed to protect against the type of accident the actor’s conduct causes, and if the
accident victim is within the class of persons the statute is designed to protect.” In such a case, the jury is
required to find that the actor is negligent.

Landlord’s failure to permanently fix the furnace was unexcused. On three earlier occasions, he was given
notice of problems with the furnace but made only temporary repairs. Nephew, as the guest of a tenant, is
within the class of persons the statute is designed to protect. Loss of heat and hot water is certainly among
the types of harms the statute was designed to protect against. However, it is not clear that the accident
which occurred – burns due to a collision – was within the category of harms the statute was designed to
protect against. There is conflicting authority on whether statutes criminalizing the act of leaving a key in
a car ignition are designed to protect against injuries caused by a car thief.

Explanation to Point-Five (20-30%):

In a negligence action, a defendant is liable only if his conduct was the proximate cause of the plaintiff’s
injury. A reasonable jury could find that the type of harm Nephew suffered was too remote from
Landlord’s negligence in failing to provide adequate heat and hot water.

It is unclear whether Landlord’s failure to repair the furnace was a proximate cause of Nephew’s injuries.
(Indeed, the “right type of harm” requirement for use of a statutory standard to establish negligence is
probably just another way of stating the proximate cause requirement.) Even when the defendant is
negligent, his conduct must have “such an effect in producing the harm as to lead reasonable men to
regard it as a cause, using that word in the popular sense, in which there always lurks the idea of
responsibility, rather than in the so-called ‘philosophic sense,’ which includes every one of the great
number of events without which any happening would not have occurred.”

Intervening actors or events that produce harm different in kind from that which one would normally
anticipate from the defendant’s negligence may break the chain of causation and lead a fact-finder to

Seperac-J19 Exam-Released MEE Essay Compilation © 2016-2020 542


conclude that the defendant’s acts are not the proximate cause of the plaintiff’s injury. Thus, a defendant
who negligently exceeds a speed limit and therefore happens to be on the spot where a tree falls during a
violent windstorm is not liable for injuries caused by the tree , and a defendant who negligently fails to
stop a train at a station is not liable for the passenger’s injuries when she elects to walk the mile back to
the station along a dangerous route and is assaulted. These harms are simply too remote from the
defendant’s negligent conduct.

On similar facts, courts have not agreed on whether a landlord’s conduct was the proximate cause of a
plaintiff’s injury. Some courts have found that the landlord’s negligence in failing to timely repair a
furnace is too remote from burn injuries like Nephew’s based on the intervening and arguably
unforeseeable conduct of actors like Nephew and Tenant as well as the nature of the injuries, which are
different in kind from those that would typically be produced by an inoperable heating system. Other
courts have held that a jury might reasonably find heating hot water to be a foreseeable result of not
having hot water and collisions from carrying hot water a foreseeable result of heating it.

[NOTE: The applicant’s conclusion on the proximate cause issue is less important than his or her
analysis. The applicant should receive full credit if he or she recognizes and discusses the applicable
legal principles.]

Seperac-J19 Exam-Released MEE Essay Compilation © 2016-2020 543


#119-FEB 2008–MEE Q02: QUESTION TWO (TORTS)

Last month, Paul attended a fund-raising lunch at Library, where he purchased and ate a chicken salad
sandwich. Later that day, he became severely ill and was diagnosed with food poisoning. As a result of
the food poisoning, Paul developed a permanent digestive disorder.

Several other people also became sick after eating at the lunch, and the Health Department determined
that the chicken salad was contaminated with salmonella bacteria. According to the Health Department,
raw chicken often contains salmonella bacteria. Although the risk of salmonella contamination cannot be
eliminated, proper preparation and cooking can ensure that the chicken is safe for eating. The chicken
must be thoroughly cooked, and all utensils or surfaces that come in contact with raw chicken must be
thoroughly cleaned with hot water and soap before further use.

The Reading Club had initiated and planned the Library’s first and only fund-raising lunch. Ann, Bill, and
Chuck independently volunteered to make the chicken salad. Each made a separate batch of salad, using
their own recipes and working individually at their own homes. Another volunteer combined the three
batches of salad at Library, and a Library employee sold sandwiches at the lunch. All lunch profits went
to Library.

Ann, Bill, and Chuck each purchased their chicken from Supermarket. The chicken was contained in
packages labeled with a prominent warning describing the risk of salmonella contamination and the
precautions necessary to avoid those risks.

A Health Department spokesperson has said that “Someone who made the chicken salad did not take
proper precautions.” Ann, Bill, and Chuck all claim they took the proper precautions.

Paul has consulted an attorney about bringing a tort action against: 1) Library, 2) Supermarket, and 3)
Ann, Bill, and Chuck. If Paul can prove only the facts outlined above:

1. Can Library be found liable to Paul under a strict liability theory? Explain.

2. Can Supermarket be found liable to Paul under a strict liability theory? Explain.

3. Can Ann, Bill, and Chuck be found liable to Paul under either a strict liability or negligence
theory? Explain.

Seperac-J19 Exam-Released MEE Essay Compilation © 2016-2020 544


#119: F08-2 MEE: ANSWER: NCBE (TORTS)

POINT (1) [24%] ISSUE: Is Library, an occasional, noncommercial product seller, strictly liable to
Paul? ANSWER: No. Strict products liability is only available against commercial product sellers.
Thus Library cannot be liable because it is not a commercial product seller.

POINT (2)(a) [18%] ISSUE: Is Supermarket, a commercial product seller, strictly liable for
injuries to Paul when it did not produce or alter the defective product and did not sell the product
to Paul? ANSWER: Yes. A commercial seller may be found strictly liable for a defective product it
sells even if the seller did not produce the product and the injured party did not purchase the
product from the seller.

POINT (2)(b) [24%] ISSUE: Is a product defective when the risk cannot be eliminated and the
product comes with adequate warnings? ANSWER: No. A product seller is liable only for a
defective product. Because the risk of salmonella contamination cannot be eliminated from raw
chicken and the product was accompanied by detailed warnings, the chicken sold by Supermarket
was not defective.

POINT (3) [35%] ISSUE: May Paul recover damages from any of Ann, Bill, or Chuck when he
cannot show who was negligent and caused his injury? ANSWER: Paul could recover on a
negligence theory against Ann, Bill, or Chuck if, and only if, he could show which of them was
negligent in preparing the chicken salad. Furthermore, merely the fact that these batches of chicken
salad were mixed together doesn’t make Ann, Bill, and Chuck joint tortfeasors because there is no
evidence they were engaged in a joint venture or joint enterprise. In fact they acted independently
of one another. Paul cannot recover on a strict liability theory because Ann, Bill, and Chuck are not
product sellers.

ANSWER DISCUSSION:

If Paul proves only the facts given in the problem, he will not be able to recover damages from any of the
parties. Paul cannot recover damages from Library because, as an occasional, noncommercial seller of
food, Library is not strictly liable for injuries caused by its food. Paul cannot recover damages from
Supermarket either. Although Supermarket, as a commercial seller, is strictly liable for defective
products, here the product was not defective as it was prominently labeled as possibly having salmonella
bacteria, and the risk of salmonella contamination cannot be eliminated. There is evidence that at least one
of Ann, Bill, or Chuck was negligent in the preparation of the chicken salad that caused Paul’s injury. If
Paul could establish which party was negligent, he could recover damages from that individual. However,
Paul cannot establish which party was negligent, and the doctrines of res ipsa loquitur, alternative
liability, and joint enterprise liability are all unavailable to assist Paul in his claim.

ANSWER EXPLANATION:

Explanation to Point-One (15-25%):

Strict products liability is only available against commercial product sellers. Thus Library cannot be liable
because it is not a commercial product seller.

Seperac-J19 Exam-Released MEE Essay Compilation © 2016-2020 545


“One who sells any product in a defective condition unreasonably dangerous to the user or consumer is
subject to liability for physical harm thereby caused.” The chicken salad sandwich that Paul consumed
was unreasonably dangerous to the consumer because it contained salmonella bacteria. Thus, if Library
had been a restaurant or other commercial entity, Paul would have a viable products liability action. In
that action, he would not need to establish negligence on the part of Library.

However, a strict products liability action is only available against a “person engaged in the business of
selling products for use or consumption.” Because the facts do not establish that Library is a commercial
seller, Paul’s action against Library will fail on that ground.

Explanation to Point-Two(a) (10-20%):

A commercial seller may be found strictly liable for a defective product it sells even if the seller did not
produce the product and the injured party did not purchase the product from the seller.

Strict products liability applies to all commercial sellers. Even a retailer who has no control over the
design and manufacture of a product may be found strictly liable if that retailer sells a defective product.
Because Supermarket is a commercial seller, it may be found liable if the chicken it sold was defective.

The fact that Paul did not purchase the chicken himself does not alter the result. Modern products liability
law applies to bystanders as well as purchasers. The “privity of contract” approach has everywhere been
abandoned. Strict liability is available for injuries to an ultimate user or consumer.

Explanation to Point-Two(b) (15-25%):

A product seller is liable only for a defective product. Because the risk of salmonella contamination
cannot be eliminated from raw chicken and the product was accompanied by detailed warnings, the
chicken sold by Supermarket was not defective.

A product is defective when it is unreasonably dangerous. A product may be defective because it has a
manufacturing defect, because it is defective in design, or because it provides inadequate instructions or
warnings. The chicken sold by Supermarket carried adequate warnings of the risk of salmonella
contamination and instructions on how to cook the chicken to ensure that it was safe for eating. The
chicken sold by Supermarket did not contain a manufacturing or design defect as the risk of salmonella
contamination is inherent in raw chicken and cannot be eliminated. Thus, the chicken conformed to the
producer’s specifications, and those specifications could not be altered. Because the chicken was not
defective, Paul cannot recover damages from Supermarket.

Explanation to Point-Three (25-35%):

Paul could recover on a negligence theory against Ann, Bill, or Chuck if, and only if, he could show
which of them was negligent in preparing the chicken salad. Furthermore, merely the fact that these
batches of chicken salad were mixed together doesn’t make Ann, Bill, and Chuck joint tortfeasors because
there is no evidence they were engaged in a joint venture or joint enterprise. In fact they acted
independently of one another. Paul cannot recover on a strict liability theory because Ann, Bill, and
Chuck are not product sellers.

The Health Department said that someone who made the chicken salad that Paul ate used improper
precautions. The evidence thus shows that one of Ann, Bill, or Chuck was negligent, that is, failed to
exercise reasonable care under the circumstances.

Seperac-J19 Exam-Released MEE Essay Compilation © 2016-2020 546


Even though Paul cannot show how the negligence occurred, he could still recover if only one individual
had made the salad. In such a situation, Paul could use the doctrine of res ipsa loquitur, which permits the
jury to infer negligence when “the event is of a kind which ordinarily does not occur in the absence of
negligence; other responsible causes are sufficiently eliminated by the evidence; and the indicated
negligence is within the scope of the defendant’s duty to the plaintiff.” Paul can meet the first and third of
these requirements: the Health Department has said that salmonella contamination does not occur when
chicken is cooked and prepared properly. The salad preparers owed a duty to Paul as he was a lunch
patron who would foreseeably have been expected to consume the chicken salad. However, Paul cannot
establish that any one defendant had control of the chicken salad, as there were two other individuals who
made salad independently. Nor can Paul show that Ann, Bill, and Chuck have better information about
who was negligent or are engaged in a conspiracy of silence.

There are other doctrines that permit the jury to find a defendant liable even when there is more than one
defendant and the plaintiff cannot show which defendant’s conduct caused his injury. However, none of
these doctrines would help Paul to establish the liability of Ann, Bill, or Chuck. The “alternative liability”
doctrine, which permits the jury to find two defendants liable when each was negligent and either
individual could have caused the plaintiff’s injuries, is unavailable because Paul cannot show that all
defendants were negligent. The “joint venture” or “joint enterprise” doctrine allows the jury to impute one
defendant’s negligence to other defendants who are engaged in a common project or enterprise and who
have an explicit or implied understanding about how the project is to be carried out. However, Paul
cannot show that Ann, Bill, and Chuck had any common understanding. The facts state that Ann, Bill, and
Chuck acted independently in making their respective batches of chicken salad.

Thus, because Paul cannot show which defendant was negligent, he cannot recover damages from Ann,
Bill, or Chuck.

A strict products liability action would not be available against Ann, Bill, or Chuck because none is a
product seller. Moreover, even if a products liability action were available, it would still be necessary to
identify the source of the defective product, here the contaminated chicken salad.

Seperac-J19 Exam-Released MEE Essay Compilation © 2016-2020 547


AGENCY & PARTNERSHIP: 37 OF 49 MEE EXAMS: (76%)
#120-FEB 2019–MEE Q03: QUESTION THREE (AGENCY & PARTNERSHIP)

Five years ago, three radiologists—Carol, Jean, and Pat—opened a radiology practice together. They
agreed to call their business "Radiology Services," to split the profits equally, and to run the practice
together in a manner that would be competitive. Toward that end, they purchased state-of-the-art
radiology imaging equipment comparable to that of other radiology practices in the community.

Shortly after opening the practice, Carol, Jean, and Pat retained an attorney to organize the practice as a
limited liability company. The attorney prepared all the necessary documents and forwarded the
documents to Carol, Jean, and Pat for signature. However, they were so involved in their radiology
practice that they forgot to sign the documents, and they have never done so.

Four months ago, Carol suggested to Jean and Pat that the practice replace some of the imaging
equipment. Jean was worried about overspending on imaging equipment, but she did not express her
concern to Carol and Pat.

Three months ago, Carol, without discussing the matter further with either Jean or Pat or obtaining their
consent, purchased for the practice a $400,000 state-of-the-art imaging machine like those recently
acquired by other radiology practices in the community.

After the purchase but prior to delivery, Jean learned what Carol had done and was furious. Jean did not
believe the practice could afford such an expensive machine. When Jean confronted Carol, Carol said,
"Too bad, it's a done deal—get over it." At that, Jean responded, "That's it. I've had enough. This machine
was purchased without my consent. It's a terrible idea. I'm out of here and never coming back. Just give
me my share of the value of the practice." Carol responded, "Fine with me." Carol and Pat subsequently
agreed to continue their participation in Radiology Services without Jean.

Radiology Services is in a jurisdiction that has adopted both the Revised Uniform Partnership Act (1997,
as amended) and the Uniform Limited Liability Company Act (2006, as amended).

1. What type of business entity is Radiology Services? Explain.

2. Did Carol have the authority to purchase the imaging machine without the consent of Jean and
Pat? Explain.

3. Did Jean's statements to Carol constitute a withdrawal from Radiology Services? Explain.

4. Were Jean's statements sufficient to entitle her to receive a buyout payment from Radiology
Services for her interest in the practice? Explain.

Seperac-J19 Exam-Released MEE Essay Compilation © 2016-2020 548


#120: F19-3 MEE: ANSWER: NCBE (AGENCY & PARTNERSHIP)

POINT (1) [20%] ISSUE: What type of business entity is Radiology Services? ANSWER: Radiology
Services is a general partnership. Despite their intentions, Carol, Pat, and Jean never formed a
limited liability company.

POINT (2) [30%] ISSUE: Did Carol have the authority to purchase the imaging machine without
the consent of Jean and Pat? ANSWER: Yes. Carol had the authority to purchase the imaging
machine without the consent of Jean and Pat because it appears that Carol purchased the machine
in the ordinary course of business and was unaware of Jean's concerns about purchasing expensive
imaging equipment.

POINT (3) [20%] ISSUE: Did Jean's statements to Carol constitute a withdrawal from Radiology
Services? ANSWER: Yes. Jean's oral statement to Carol, "I'm out of here," resulted in Jean's
dissociation from the partnership.

POINT (4) [30%] ISSUE: Were Jean's statements sufficient to entitle her to receive a buyout
payment from Radiology Services for her interest in the practice? ANSWER: Yes. Because Carol
and Pat agreed to continue the partnership, Jean's dissociation did not result in the dissolution and
winding up of the partnership. Instead, Jean is entitled to receive a buyout price for her
partnership interest 120 days after she makes written demand for payment. Because her demand
was not in writing, the partnership has no specified time in which to make payment.

ANSWER DISCUSSION:

Radiology Services is an "at-will" general partnership because Carol, Jean, and Pat agreed to share profits
and operate the business together without specifying a specific term. They did not form a limited liability
company (LLC), as they intended, because no documentation to create an LLC was signed or filed. As a
partner of the general partnership, Carol had the authority to purchase the imaging machine on behalf of
the partnership without the consent of either Jean or Pat because each partner has the authority to conduct
partnership business. And while a partner may be liable to the other partners for a purchase that exceeds
her authority, here Jean likely has no claim against Carol for purchasing the machine without the consent
of Jean and Pat because the purchase of the imaging machine was most likely in the ordinary course of the
business of the partnership. Jean's oral statements to Carol stating her will to withdraw from Radiology
Services resulted in her dissociation from the partnership, and because the partnership was "at will," her
dissociation was not wrongful. Because Carol and Pat agreed to continue the partnership, Jean's
dissociation did not result in the dissolution and winding up of the partnership. Instead, Jean is entitled to
receive a buyout payment from Radiology Services for her one-third partnership interest 120 days after
making a written demand for payment. Because Jean did not make a written demand, the partnership has
no specified time in which to pay her the buyout price. [NOTE: General partnerships are governed by
either the Revised Uniform Partnership Act (RUPA) (1997, as amended), the Revised Uniform
Partnership Act (1997), or the Uniform Partnership Act (1914) (UPA). The 1997 act, which the Uniform
Law Commission amended in both 2011 and 2013 as part of its harmonization project, applies in all states
(sometimes with the amendments) except for Georgia, Indiana, Louisiana, Massachusetts, Michigan,
Missouri, New Hampshire, New York, North Carolina, Pennsylvania, Rhode Island, South Carolina, and
Wisconsin, which continue to use versions of the UPA (1914). Because the facts state that Radiology
Services is in a jurisdiction that has adopted the RUPA (1997, as amended), citations to the other acts are
omitted, although the results under those acts are similar. The Uniform Limited Liability Company Act

Seperac-J19 Exam-Released MEE Essay Compilation © 2016-2020 549


(2006, as amended) contains principles that are widely adopted in LLC law. In all states, articles of
organization must be signed and filed to create an LLC.]

ANSWER EXPLANATION:

Explanation to Point One (20%):

Radiology Services is a general partnership. Despite their intentions, Carol, Pat, and Jean never formed a
limited liability company.

Under RUPA, as amended, a general partnership is formed whenever two or more persons associate
(whether or not in writing) for the purpose of carrying on a business for profit. The association of two or
more persons to carry on as co-owners a business for profit forms a partnership, whether or not the
persons intend to form a partnership. Here, by agreeing to share profits and operate their radiology
practice together, Carol, Jean, and Pat formed a general partnership, despite their intention to operate their
business as a limited liability company. Furthermore, because the partnership was for neither a fixed term
nor a specific undertaking, it was an "at-will" partnership.

A limited liability company is formed by the filing of a signed certificate of organization with the
Secretary of State. Here, no certificate was signed or filed; thus, Carol, Jean, and Pat did not form a
limited liability company.

[NOTE: Although some jurisdictions have recognized the "de facto" LLC doctrine where there has been a
good-faith but failed attempt to form the LLC, this doctrine insulates members only from liability to third
parties; it does not create an LLC as between the members themselves.]

Explanation to Point Two (30%):

Carol had the authority to purchase the imaging machine without the consent of Jean and Pat because it
appears that Carol purchased the machine in the ordinary course of business and was unaware of Jean's
concerns about purchasing expensive imaging equipment.

Section 401(h) of RUPA, as amended, provides that "each partner has equal rights in the management and
conduct of the partnership's business." This grant of authority to each partner is tempered by subsection
401(k), which provides: "A difference arising as to a matter in the ordinary course of business of a
partnership may be decided by a majority of the partners. An act outside the ordinary course of business
of a partnership and an amendment to the partnership agreement may be undertaken only with the
affirmative vote or consent of all the partners."

As the comments to Section 401(h) note, the scope of a partner's authority is governed by agency law
principles. If the partnership agreement is silent on the scope of the agent-partner's authority, a partner has
actual authority to commit the partnership "to usual and customary matters, unless the partner has reason
to know that (i) other partners might disagree, or (ii) for some other reason consultation with fellow
partners is appropriate." In light of this principle, a partner has authority to bind the partnership as to
"usual and customary" dealings with third parties and need not seek the prior approval of the other
partners unless the partner has reason to believe that the other partners might not approve or might expect
to be consulted. Furthermore, under general agency law principles, a partner does not have actual
authority to take "unusual or non-customary actions that will have a substantial effect on the partnership."

Seperac-J19 Exam-Released MEE Essay Compilation © 2016-2020 550


Here, it appears that purchase of the equipment was in the ordinary course of business. The partners had
purchased state-of-the-art imaging equipment when they started the practice, they had agreed to run the
practice in a manner consistent with other area practices, and other practices had bought equipment like
that which Carol purchased.

Further, although Jean had concerns about the practice purchasing expensive imaging machines, there is
no indication that Carol was aware of Jean's concerns when she made the purchase. The facts clearly state
that Jean did not express her concerns to either Carol or Pat. Furthermore, the facts indicate that the three
partners (including Jean) had agreed that the partnership would have imaging equipment that would allow
it to be competitive with other similar practices in the community.

Explanation to Point Three (20%):

Jean's oral statement to Carol, "I'm out of here," resulted in Jean's dissociation from the partnership.

Under RUPA, as amended, a partner is dissociated from a partnership when the partnership has notice of
the partner's will to withdraw as a partner. A partner can dissociate from the partnership at any time. The
notice of the partner's will to withdraw need not be in writing. The dissociation is rightful—that is, the
dissociating partner has no obligations to the other partners—when the partnership is at will and the
dissociation breaches no express provision in the partnership agreement. Under RUPA, as amended,
"partnership at will" means a partnership in which the partners have not agreed to remain partners until
the expiration of a definite term or the completion of a particular undertaking. A partner's dissociation is
wrongful in an at-will partnership only if it is in breach of an express provision of the partnership
agreement.

Here, Jean's statement to Carol that "I'm out of here and never coming back" constituted a dissociation,
and it was not wrongful because there is no indication in this at-will partnership that her withdrawal
breached any express provision in the partnership agreement.

Explanation to Point Four (30%):

Because Carol and Pat agreed to continue the partnership, Jean's dissociation did not result in the
dissolution and winding up of the partnership. Instead, Jean is entitled to receive a buyout price for her
partnership interest 120 days after she makes written demand for payment. Because her demand was not
in writing, the partnership has no specified time in which to make payment.

Normally, a partner's dissociation in an at-will partnership results in its dissolution, and the business must
be wound up. But such dissolution can be rescinded by the affirmative vote or consent of all remaining
partners. The RUPA permits the partners to "waive the right to have the partnership's business wound up
and the partnership terminated". Under RUPA, as amended, the dissociating partner is no longer
considered a partner and does not participate in this decision to continue the partnership. The term
"partner" does not include a partner who dissociated under Section 601.

When a partnership is continued and not dissolved, the dissociating partner is entitled to have her interest
purchased for a buyout price equal to that partner's interest in the value of the partnership, based on the
greater of its liquidation or going-concern value (plus interest from the date of dissociation). Further, if
the withdrawing partner makes a written demand for payment and no agreement is reached within 120
days after the demand, the partnership must pay in cash the amount it estimates to be the buyout price,
including accrued interest.

Seperac-J19 Exam-Released MEE Essay Compilation © 2016-2020 551


Here, although Jean made clear her will to withdraw, the partnership was not dissolved because Carol and
Pat agreed to continue the partnership. Thus, Jean would be entitled to receive payment of a buyout price.
Although Jean made an oral statement demanding payment for her interest, her demand was not in
writing. Thus, the partnership does not have a specific time in which to pay the buyout price. But if Jean
makes her demand in writing, the partnership would have to reach an agreement with her on the buyout
price within 120 days or, failing this, pay her in cash its estimate of the buyout price plus accrued interest.

Seperac-J19 Exam-Released MEE Essay Compilation © 2016-2020 552


#121-FEB 2018–MEE Q06: QUESTION SIX (AGENCY-PARTNERSHIP)

A man and a woman were equal partners in a neighborhood natural-foods store. The store had been at the
same location for many years and had developed a loyal following. Under their informal arrangement, the
man had managed the business and the woman had supplied capital to the business as needed.

They leased the building in which the store was located and had regularly sought to purchase the building
for the partnership, but the landlord had always refused. Six months ago, however, the landlord called the
man and said, "I thought you would want to know that I'm planning to sell the building." The next day,
the man sent the woman an email: "I am leaving our partnership. I will wind up the business and send you
a check for your half share." Without informing the woman, the man then contacted the landlord and
offered to buy the building. The landlord accepted, and the two entered into a binding purchase
agreement. One month later, the man took title to the building.

Three months ago, the man sent the woman a check for half of the store's inventory and other business
assets. Instead of cashing the check, the woman sent the man an email stating that she regarded the
partnership as still in existence and demanded that the man convey title to the building to the partnership.
The man replied that their partnership was dissolved and that he had moved on. He then began to operate
the store as a natural-foods store with a name different from that of the original store, but with the same
product offerings and the same employees.

The woman has sued the man for withdrawing from the partnership and for breaching his duties by
buying the building from the landlord.

1. Did the man properly withdraw from the partnership? Explain.

2. Assuming that the man's withdrawal was not wrongful, what was the legal effect of the man's
withdrawal from the partnership? Explain.

3. What duties, if any, did the man breach by purchasing the building? Explain.

Seperac-J19 Exam-Released MEE Essay Compilation © 2016-2020 553


#121: F18-6 MEE: ANSWER: NCBE (AGENCY-PARTNERSHIP)

POINT (1) [25%] ISSUE: May a partner properly withdraw from a partnership that has no
definite term or specific undertaking? ANSWER: Yes. The man’s email to the woman constituted a
withdrawal from the partnership and did not violate the partnership agreement.

POINT (2) [25%] ISSUE: What is the legal effect of a partner’s withdrawal from an at-will
partnership— both on the partnership and on the withdrawing partner’s duties? ANSWER: The
man’s withdrawal from the partnership caused the partnership to be dissolved, but did not
terminate the man’s duties to the woman during the winding-up process.

POINT (3) [50%] ISSUE: What are the duties of a withdrawing partner during the winding up of a
dissolved partnership with respect to business opportunities that came to the partner’s attention
during the partnership? ANSWER: During the winding-up process, the man owed the woman a
fiduciary duty to account to the partnership for any benefit derived from the appropriation of a
partnership opportunity, as well as duties of good faith and fair dealing. The man’s purchase of the
building for himself, without telling the woman, breached these duties.

ANSWER DISCUSSION:

The man had the right to withdraw from the partnership, and his email stating that he was “leaving”
expressed his will to withdraw. His withdrawal was not wrongful, given that the partnership had no
definite term or particular undertaking. The man’s withdrawal from the partnership caused the partnership
to be dissolved, but this dissolution did not terminate the man’s duties to the woman during the winding
up of the partnership business. During the winding up of the partnership business, the man continued to
owe fiduciary duties to the partnership and the woman, as well as duties of good faith and fair dealing.
The opportunity to purchase the building was a partnership opportunity because of the partnership’s prior
interest in owning the building and because the opportunity was presented to the man while the
partnership was still in existence and prior to its dissolution. The man’s purchase of the building violated
his duty of loyalty not to appropriate partnership opportunities during the winding-up process, and the
purchase was not in good faith because the man failed to inform the woman of the opportunity when he
learned of it before he withdrew from the partnership. [NOTE: An examinee may analyze this question
under either the Uniform Partnership Act (1914) (UPA) or the Revised Uniform Partnership Act (1997, as
amended in 2013) (RUPA). Although the terminology regarding partnership dissolution and winding up
are somewhat different under the two Acts, the legal ramifications of the partner’s withdrawal in this two-
person, at-will partnership are essentially the same under both Acts. In addition, the revised Act codifies
and clarifies the fiduciary duties of partners, and the analysis on this topic cites both the RUPA provisions
and applicable case law. All states, except Louisiana, have adopted a version of the Uniform Partnership
Act. As of 2017, most states (37) have adopted the revised Act, though only a handful have adopted the
2013 amendments.]

ANSWER EXPLANATION:

Explanation to Point-One (25%):

The man’s email to the woman constituted a withdrawal from the partnership and did not violate the
partnership agreement.

Seperac-J19 Exam-Released MEE Essay Compilation © 2016-2020 554


A partner may withdraw from a partnership by giving notice at any time. A partner is “disassociated”
from a partnership upon the partnership having notice of the partner’s express will to withdraw. A partner
always has the power to withdraw by express will, even if such withdrawal is wrongful or in
contravention of the partnership agreement (such as early withdrawal from a partnership with a definite
term). While a partnership cannot eliminate the power of a partner to dissociate, it can eliminate the right
to dissociate and make such dissociation wrongful.

Dissolution is caused without any violation of the partnership agreement when, among other
circumstances, a partner withdraws from an at-will partnership. In the absence of a definite term or
particular undertaking, a partnership is deemed to be at will. Here, the man’s email to the woman
constituted a withdrawal from the partnership and did not violate the partnership agreement, given that
there is no indication that the partners (who had been carrying on the business for many years) had agreed
on a definite term or particular undertaking for their natural-foods store partnership. Thus, the man’s
withdrawal from their at-will partnership was proper.

Explanation to Point-Two (25%):

The man’s withdrawal from the partnership caused the partnership to be dissolved, but did not terminate
the man’s duties to the woman during the winding-up process.

Dissolution of a partnership results in a change in the legal relation of the partners, but does not
immediately terminate the partnership or the rights and powers of the partners. Upon dissolution, the
partnership continues until the winding up of partnership affairs is completed. Thus, dissolution marks the
point when the partners cease carrying on the partnership business together and begin a process of settling
the partnership affairs. The partners’ rights, powers, and duties continue during the winding-up process
that follows dissolution, during which the partnership liabilities are paid, the business is settled and
closed, and the partnership assets are distributed. The partners’ legal relationship and the partnership
terminate only when all of the partnership affairs are completely wound up.

Under the Uniform Partnership Act (1914), a partnership at will is dissolved by the express will of any
partner, and any partner of the dissolved partnership has the right to have the partnership business wound
up. A similar result follows under the Revised Uniform Partnership Act, which provides that a partner in a
partnership at will can dissociate from the partnership by that partner’s express will, and upon
dissociation the partnership is dissolved and its business must be wound up. Here, although the man’s
withdrawal (or dissociation) caused their at-will partnership to be dissolved, the man continued to have
duties to the woman and the partnership during the winding-up process. Because the man’s withdrawal
(dissociation) from the partnership was not wrongful, he had the power and the right to participate in the
winding up of the partnership business, and his actions during that period bound the partnership.

Explanation to Point-Three (50%):

During the winding-up process, the man owed the woman a fiduciary duty to account to the partnership
for any benefit derived from the appropriation of a partnership opportunity, as well as duties of good faith
and fair dealing. The man’s purchase of the building for himself, without telling the woman, breached
these duties.

During the winding-up process, partners who participate in the winding up of partnership business
continue to have a fiduciary relationship to the partnership and the other partners. Except for the duty not
to compete with the partnership, all of a partner’s fiduciary duties continue to apply during the winding up
of the partnership business.

Seperac-J19 Exam-Released MEE Essay Compilation © 2016-2020 555


Among the partner’s fiduciary duties is a duty to account to the partnership for any benefit derived by the
partner from the appropriation of any partnership opportunities. The duty not to appropriate partnership
opportunities continues during the winding-up process, although the scope of the partnership
opportunities inevitably narrows. For example, a partner’s fiduciary duties in a dissolved partnership no
longer extend to looking to the future of the business.

A partnership opportunity includes one that is closely related to the entity’s existing or prospective line of
business, that would competitively advantage the partnership, and that the partnership has the financial
ability, knowledge and experience to pursue. A partner who learns of a business opportunity during the
term of a partnership may not appropriate that opportunity (without sharing with his co-partners) during
the winding-up process or after the partnership term ends.

In addition, the partner must perform his duties during the winding up of the partnership business
consistently with the contractual obligation of good faith and fair dealing. The obligations of good faith
and fair dealing encompass a disclosure duty. For example, a dissociated partner may not use confidential
partnership information after the dissociation. Here, the opportunity to purchase the building in which the
store was located constituted a partnership opportunity. The partners’ prior interest in purchasing the
building indicates that ownership of the building was an expectancy of the partnership, and there is no
indication that the partnership lacked the financial resources to buy the building. Further, given the
goodwill that the store had gained at its location, ownership of the building would likely be a partnership
opportunity because owning and not having to lease the building would enhance the value of the
partnership’s ongoing business. In fact, the man’s eagerness to purchase the building provides evidence
that ownership of the building created value for the ongoing business of operating the natural-foods store
at that location.

The man breached his fiduciary duty to the partnership and the woman by not informing the woman of the
opportunity to purchase the building so that their partnership could have acquired the goodwill (customer
loyalty) that attached to the building’s location. In addition, the man breached his duty of good faith and
fair dealing by not informing the woman of this business opportunity. The man breached his duties to the
partnership and the woman even though he did not begin negotiations for the building’s purchase and did
not purchase the building until after withdrawing from the partnership. Courts have found a breach of
fiduciary duties even when final negotiations and purchase of the partnership opportunity occur after
withdrawal.

[NOTE: An examinee might conclude that the man’s continuation of the natural-foods store with the same
customers, suppliers, and employees of the partnership also constituted a breach of duties to the
partnership and the woman. These continuing dealings, however, are not raised in the question.

An examinee might also point out that the proper remedy in this case would be for the man to hold the
building in trust as a partnership asset, thus allowing the woman to participate in the building’s special
value to the partnership. But the proper remedy for the man’s breach of his duties is not raised in the
question.]>

Seperac-J19 Exam-Released MEE Essay Compilation © 2016-2020 556


#122-FEB 2017–MEE Q05: QUESTION FIVE (AGENCY-PARTNERSHIP)

An inventor retained a woman to act as his agent to purchase 25 computer chips, 25 blue lenses, and 25
lawn mower shutoff switches. The inventor told her to purchase only:

• Series A computer chips,

• blue lenses that cost no more than $300 each, and

• shutoff switches that could shut down a lawn mower in less than one second after the mower hits
a foreign object.

The woman contacted a chip manufacturer to purchase the Series A computer chips. She told the
manufacturer that she was the inventor's agent and that she wanted to purchase 25 Series A computer
chips on his behalf. The manufacturer told her that the Series A chips cost $800 each but that she could
buy Series B chips, with functionality similar to that of the Series A chips, for only $90 each. Without
discussing this with the inventor, the woman agreed to purchase 25 Series B chips, signing the contract
with the chip manufacturer "as agent" of the inventor. The Series B chips were shipped to her, but when
she then took them to the inventor and explained what a great deal she had gotten, the inventor refused to
accept them. He has also refused to pay the manufacturer for them.

The woman also contacted a lens manufacturer for the purchase of the blue lenses. She signed a contract
in her name alone for the purchase of 25 blue lenses at $295 per lens. She did not tell the lens
manufacturer that she was acting as anyone's agent. The lenses were shipped to her, but when she took
them to the inventor, he refused to accept them because he had decided that it would be better to use red
lenses. The inventor has refused to pay for the blue lenses.

The woman also contacted a switch manufacturer to purchase shutoff switches. She signed a contract in
her name alone for switches that would shut down a lawn mower in less than five seconds, a substantially
slower reaction time than the inventor had specified to her. When she signed the contract, she told the
manufacturer that she was acting as someone's agent but did not disclose the identity of her principal. The
switches were shipped to her. Although the inventor recognized that the switches were not what the
woman had been told to buy, he nonetheless used them to build lawn mowers, but now refuses to pay the
manufacturer for them.

All elements of contract formation and enforceability are satisfied with respect to each contract.

1. Who is liable to the chip manufacturer: the inventor, the woman, or both? Explain.

2. Who is liable to the blue-lens manufacturer: the inventor, the woman, or both? Explain.

3. Who is liable to the shutoff-switch manufacturer: the inventor, the woman, or both? Explain.

Seperac-J19 Exam-Released MEE Essay Compilation © 2016-2020 557


#122: F17-5 MEE: ANSWER: NCBE (AGENCY-PARTNERSHIP)

POINT (1) [30%] ISSUE: When an agent enters into a contract with a third party on behalf of a
disclosed principal on terms that were not authorized by the principal, who is liable to the third
party: the agent, the principal, or both? ANSWER: With respect to the chips, the woman (agent) is
liable on the contract, but the inventor (principal) is not because the woman, notwithstanding her
disclosure that she was acting as his agent, lacked actual or apparent authority to enter into the
contract on behalf of the inventor with the chip manufacturer.

POINT (2) [30%] ISSUE: When an agent enters into a contract with a third party on behalf of an
undisclosed principal on terms authorized by the principal, who is liable to the third party if the
principal later repudiates the contract: the agent, the principal, or both? ANSWER: Both the
inventor and the woman are liable to the blue-lens manufacturer on the contract for blue lenses.
The inventor is liable because the woman acted with actual authority; the woman is liable as a party
to the contract because the principal was undisclosed.

POINT (3) [40%] ISSUE: When an agent enters into a contract with a third party on behalf of a
partially disclosed principal for goods different from those authorized by the principal, who is liable
to the third party if the principal accepts the different goods: the agent, the principal, or both?
ANSWER: Both the inventor and the woman are liable on the contract for the shutoff switches. The
inventor is liable by ratifying the contract; the woman is liable because she acted on behalf of a
partially disclosed principal, and there is no indication that the third party agreed to look solely to
the partially disclosed principal for payment.

ANSWER DISCUSSION:

[NOTE: While the contracts in this question are for the sale of goods and, thus, governed by Article 2 of
the Uniform Commercial Code, the UCC does not contain agency rules resolving the issues in this
question. Thus, common law principles govern.]

As a general matter, an agent binds a principal to a contract, whether or not the principal is disclosed to
the third party, if the agent had either actual or apparent authority to enter into the contract. Without that
authority, the agent alone is liable on the contract unless the principal becomes liable by subsequently
ratifying the contract. An agent acting with authority is not liable on the contract if the principal’s identity
is disclosed to the third party, but is liable if the principal’s identity is not disclosed or only partially
disclosed, unless the contract provides otherwise. Applying these principles here, because the woman
disclosed that she was acting for the inventor on the chip contract, but purchased different chips from
those specified by the inventor, the inventor is not liable because the woman did not have authority to
enter into the contract; the woman is also liable on the chip contract because she impliedly warranted that
she had authority. Both the woman and the inventor are liable on the blue-lens contract. Although the
woman did not disclose that she was acting for the inventor on the blue-lens contract, the inventor is liable
on this contract because he had given the woman actual authority to buy the blue lenses on his behalf; the
woman is also liable because she signed the contract in her own name. Finally, both the inventor and the
woman are liable on the shutoff-switch contract that the woman entered into on behalf of the partially
disclosed inventor, even though the switches were different from those authorized. The inventor became
liable by ratifying the contract when he accepted the different switches, and the woman became liable by
signing a contract on behalf of a partially disclosed principal.

Seperac-J19 Exam-Released MEE Essay Compilation © 2016-2020 558


ANSWER EXPLANATION:

Explanation to Point-One (30%):

With respect to the chips, the woman (agent) is liable on the contract, but the inventor (principal) is not
because the woman, notwithstanding her disclosure that she was acting as his agent, lacked actual or
apparent authority to enter into the contract on behalf of the inventor with the chip manufacturer.

A principal is not liable on a contract entered into by an agent unless the agent had actual or apparent
authority. In the absence of actual or apparent authority, it is irrelevant whether the agent disclosed to the
third party that the agent was acting on behalf of a disclosed principal.

Actual authority exists when the principal by written or spoken words or other conduct “causes the agent
to believe that the principal desires the agent to act on the principal’s account.” While rephrased, the
Third Restatement is similar and provides that “an agent acts with actual authority when, at the time of
taking action that has legal consequences for the principal, the agent reasonably believes, in accordance
with the principal’s manifestations to the agent, that the principal wishes the agent so to act.”

Apparent authority is created with respect to a third person when “by written or spoken words or any
other conduct” the principal causes the third person “to believe that the principal consents to have the act
done on his behalf by the person purporting to act for him.” The Third Restatement is similar: “apparent
authority is created by a person’s manifestation that another has authority to act with legal consequences
for the person who makes the manifestation, when a third person reasonably believes the actor to be
authorized and the belief is traceable to the manifestation.”

Here, the inventor is not liable on the Series B chips contract because the woman had neither actual nor
apparent authority to purchase these chips on the inventor’s behalf. First, there is no basis to conclude that
the woman had actual authority to buy Series B chips. The inventor instructed the woman to buy only
Series A chips; thus, the woman only had actual authority to purchase the Series A chips and no others.
She exceeded her actual authority when she purchased the Series B chips.

Next, there was no apparent authority here. There are no facts to suggest that the inventor (as opposed to
the woman) manifested to the chip manufacturer that the woman had authority to acquire Series B chips.
Without such evidence, there is no apparent authority.

While the inventor (principal) is not liable to the chip manufacturer on the chip contract, the woman
(agent) would be liable to the chip manufacturer for damages because she breached the implied warranty
to the manufacturer that she had authority to act on behalf of the inventor.

A person who “purports to make a contract with a third party on behalf of another person, lacking power
to bind that person, gives an implied warranty of authority to the third party and is subject to liability to
the third party for damages for loss caused by the breach of that warranty, including loss of the benefit
expected from performance by the principal.” Because she breached this warranty, she is liable to the
manufacturer.

Explanation to Point-Two (30%):

Both the inventor and the woman are liable to the blue-lens manufacturer on the contract for blue lenses.
The inventor is liable because the woman acted with actual authority; the woman is liable as a party to the
contract because the principal was undisclosed.

Seperac-J19 Exam-Released MEE Essay Compilation © 2016-2020 559


“An undisclosed principal is bound by contracts made on his account by an agent acting within his
authority.”

An agent who purports to act on his own account, but in fact is making a contract on behalf of an
undisclosed principal, is also a party to the contract. The rationale for this rule is that the third party has
every reason in the case of an undisclosed principal and agency to assume that the person with whom it
contracts expects to be liable on the contract. Additionally, to the extent the third party was relying on the
financial solvency of the person on the other side of the contract, the third party would have no basis to
rely on anyone but the agent who signed the contract.

Here, the woman had actual authority from the inventor to purchase blue lenses at a price not to exceed
$300 per unit. By contracting to purchase blue lenses for $295 per unit, the woman acted with actual
authority in entering into the blue-lens contract. Thus, the inventor became bound to the contract, even
though the third party had no reason to know that the woman was acting on behalf of the inventor. The
inventor had no right to refuse to accept the shipment and refuse to pay for the blue lenses.

The woman is also liable on the contract because she purported to act on her own behalf. The blue-lens
manufacturer relied on the woman being bound by the contract.

[NOTE: Because the inventor (not the woman) was the person who expected to reap the benefit of the
contract with the blue-lens manufacturer, as between them the inventor has the primary responsibility to
carry out the terms of the contract, and the agent acts as a surety for that obligation. Thus, if the
manufacturer were to sue the woman and recover, the woman could seek to be recompensed by the
inventor. If the manufacturer were to sue the inventor and then sue the woman, the woman would be
entitled to raise all defenses available to a surety.

In addition, in some jurisdictions the third party may be required, prior to judgment, to elect whether to
take judgment against the undisclosed principal or the agent. In such an election jurisdiction, either the
inventor (undisclosed principal) or the woman (agent) could escape liability by making a demand on the
third-party manufacturer to elect to take judgment against the other. This “election rule” has been
heavily criticized and has been abandoned in some jurisdictions in favor of a “satisfaction rule” that
allows the third party to take judgment against both the undisclosed principal and the agent, but entitles
the third party to only one satisfaction of the judgment. Thus, in a satisfaction jurisdiction, if both the
inventor and the woman are sued, neither of them can escape judgment by demanding that the third-party
manufacturer make an election.]>

Explanation to Point-Three (40%):

Both the inventor and the woman are liable on the contract for the shutoff switches. The inventor is liable
by ratifying the contract; the woman is liable because she acted on behalf of a partially disclosed
principal, and there is no indication that the third party agreed to look solely to the partially disclosed
principal for payment.

When a third party contracts with a person that the third party knows is acting in an agency capacity for
another but the third party is unaware of the identity of the principal, the principal for whom the agent acts
is called a “partially disclosed principal.” Here, the shutoff-switch manufacturer knew that the woman
was acting as someone’s agent but there is nothing to indicate that the manufacturer was aware of the
principal’s identity. Thus, the inventor is a “partially disclosed principal.” A partially disclosed principal
can be liable on a contract entered into by an agent who had actual or apparent authority. Here, the

Seperac-J19 Exam-Released MEE Essay Compilation © 2016-2020 560


woman acted without actual or apparent authority, so this cannot be the basis to hold the inventor liable
on the contract.

On the other hand, even though the woman acted without actual or apparent authority, the inventor
accepted the shutoff switches and used them in the production of the mowers. This amounts to a
ratification of the contract between the woman and the switch manufacturer. Where the principal ratifies
the act of an agent, the principal is liable on the contract just as if the agent had acted with actual
authority. Ratification occurs if the principal’s conduct “justifies a reasonable assumption that” the
principal consents to the act performed on the principal’s behalf. Here, the inventor’s use of the switches
justifies a reasonable assumption of consent.

As for the woman, she is also liable on the contract. Unless the agent and the third party agree otherwise,
an agent acting on behalf of a partially disclosed principal is a party to the contract if the agent acted with
actual or apparent authority. Here, because of the inventor’s ratification of the contract, the woman is
deemed to have acted with actual authority, and there is no indication that the manufacturer agreed to look
solely to the partially disclosed principal for payment.

[NOTE: In addition, there is no requirement that the manufacturer “elect” either a remedy against the
agent or a remedy against the principal where, as here, there is a partially disclosed principal. Thus,
neither the inventor nor the woman can escape liability by making a demand on the manufacturer to make
an election to take judgment against the other.]

Seperac-J19 Exam-Released MEE Essay Compilation © 2016-2020 561


#123-FEB 2016–MEE Q03: QUESTION THREE (AGENCY-PARTNERSHIP)

Four years ago, a man and a woman properly formed a partnership to own and manage a multi-million-
dollar apartment complex. They qualified the partnership as a limited liability partnership (LLP). The
complex required a good deal of maintenance, and they anticipated regular borrowings of up to $25,000 to
cover maintenance expenses as is customary in this industry.

While the partnership agreement contained no limitations on the authority of the partners to act for LLP,
two months after LLP was formed the man and the woman agreed that neither partner would have
authority to incur indebtedness on behalf of LLP in excess of $10,000 without the consent of the other
partner. They then signed a statement of partnership authority describing this limitation, but this statement
was never filed.

Over the next two years, the man regularly borrowed amounts from LLP’s bank to cover the complex’s
ordinary maintenance expenses. The amounts borrowed ranged from $5,000 to $9,000, and the man did
not ask for the woman’s consent when he entered into these loans on behalf of LLP.

Earlier this year, the man, without the woman’s knowledge, asked the bank to loan $25,000 to LLP. The
man told the bank’s loan officer that the funds would be used for ordinary maintenance of the apartment
complex. This amount, though greater than LLP’s previous borrowings from the bank for maintenance,
was in line with loans made by the bank for maintenance to other similar apartment complexes.

When the loan officer asked the man if he had authority to borrow the money on behalf of LLP, the man
handed the loan officer a copy of the partnership agreement. The man, however, did not give the officer a
copy of the statement of partnership authority, nor did he tell the loan officer that it existed. The bank had
no actual knowledge of the limitation on the man’s authority to obtain the loan on behalf of LLP.

Without contacting the woman, the bank loaned $25,000 to LLP. The loan agreement was signed only by
the man and the bank’s loan officer. The woman, though she had knowledge of the earlier borrowings
from the bank, had no knowledge of this loan.

The man then used the $25,000 to pay his personal gambling debts. LLP has not made any payments to
the bank on the loan.

1. Is LLP liable to the bank on the loan? Explain.

2. Is the woman personally liable to the bank on the loan? Explain.

3. Is the man liable for breaching his fiduciary duties and, if so, to whom is he liable? Explain.

Seperac-J19 Exam-Released MEE Essay Compilation © 2016-2020 562


#123: F16-3 MEE: ANSWER: NCBE (AGENCY-PARTNERSHIP)

POINT (1) [30%] ISSUE: Is LLP liable to the bank on the loan undertaken by a partner acting
beyond his actual authority, but within the partnership’s ordinary course of business? ANSWER:
Yes. LLP is liable to the bank on the loan because the man, as a partner of LLP, had apparent
authority.

POINT (2) [30%] ISSUE: Is the woman personally liable to the bank on the loan it made to LLP?
ANSWER: No. The woman is not personally liable to the bank on its claim under the loan
agreement.

POINT (3)(a) [20%] ISSUE: Did the man breach his fiduciary duties by entering into an
unauthorized transaction and appropriating partnership assets for his own use? ANSWER: Yes.
By improperly obtaining the bank loan and then misappropriating the loan proceeds, the man
breached his fiduciary duty of loyalty and his duty of care.

POINT (3)(b) [20%] ISSUE: If the man breached any fiduciary duties, does the woman and/or LLP
have a claim against the man? ANSWER: Yes. The woman (or the partnership) can bring a direct
action against the man for breaching his duties of loyalty and care. The woman can also bring an
accounting action seeking to have the man pay damages to the partnership for his loyalty breach.

ANSWER DISCUSSION:

Although the man had no actual authority to enter into the loan with the bank, the man was acting within
the partnership’s ordinary course of business and thus had apparent authority to bind LLP on the loan.
Thus, LLP is liable on the loan. The woman is not liable to the bank on the loan because a partner in a
limited liability partnership does not incur personal liability on partnership obligations solely by reason of
being a partner. The woman did not engage in any misconduct that would warrant an exception to this
rule of limited liability. The man breached his fiduciary duties (duty of care and duty of loyalty) when he
wrongfully incurred LLP debt and misappropriated the $25,000 for his personal use. Either the woman or
the partnership can maintain an action against the man to recover damages for losses resulting from the
man’s breach. The woman can also bring an accounting action to have the partnership recover damages
for losses resulting from the man’s breach of his duty of loyalty. [NOTE: Because this is a limited liability
partnership, the question is analyzed under the Revised Uniform Partnership Act, RUPA (1997, as revised
2013), not the prior act, Uniform Partnership Act, UPA (1914). The analysis would be substantially the
same in states that continue to use UPA (1914) and have adopted additional provisions for limited liability
partnerships.]

ANSWER EXPLANATION:

Explanation to Point-One (30%):

LLP is liable to the bank on the loan because the man, as a partner of LLP, had apparent authority.

Even if a partner lacks actual authority, a limited liability partnership can be bound by the acts of a
partner, “including the execution of an instrument in the partnership name,” if the partner was apparently
carrying on in the ordinary course the partnership business or business of the kind carried on by the
partnership. From the bank’s perspective, the man acting as a partner had apparent authority to incur the

Seperac-J19 Exam-Released MEE Essay Compilation © 2016-2020 563


debt because borrowing $25,000 to pay for ordinary maintenance expenses of a multi-million-dollar
apartment complex was entirely consistent with LLP’s ordinary business. This is evident from the facts
that the partners anticipated the need for such loans when they formed the LLP, the LLP had previously
borrowed from the bank for such maintenance expenses, and the bank had previously made similar loans
to other apartment complexes.

Apparent authority did not exist, however, if the bank had actual knowledge that the man lacked
authority. Under the RUPA (1997), a third party is bound by a limitation of authority only if that party
knew or had notice that the partner lacked authority. Here, there are no facts suggesting that the bank had
notice or knowledge of the limitation on authority. Knowledge is generally limited to actual knowledge,
and “notice of a fact” arises from all the facts known to the person at the time in question. The bank asked
the man if he had authority and the man, in response, gave the bank a copy of the partnership agreement
containing no limits on his authority. The man did not give the bank a copy of the statement of
partnership authority evidencing the man’s lack of actual authority. Nor did the bank have knowledge of
this statement, which was never filed.

[NOTE: If an examinee concludes that borrowing $25,000 was not in the ordinary course of business,
then the examinee must conclude that there was no apparent authority either.]

Explanation to Point-Two (30%):

The woman is not personally liable to the bank on its claim under the loan agreement.

As a partner in LLP, the woman has limited liability for any partnership debts and thus is not liable to the
bank. RUPA provides that a partner in a limited liability partnership is not liable for partnership
obligations solely by reason of being or acting as a partner. The loan obligation arises out of contract, and
thus the woman would not be liable for it unless there were some basis for asserting liability against her
other than her being a partner.

Partners can become liable, however, for partnership obligations based on their own personal misconduct
– the principles for piercing a corporate veil apply to limited liability partnerships. However, there is no
indication of fraudulent or inequitable conduct by the woman that would justify liability for personal
misconduct or piercing the entity veil.

[NOTE: Under some statutes, though not RUPA, partners in a limited liability partnership may also
become liable for the negligence, wrongful acts or misconduct of any person under the partner’s direct
supervision and control. Under this type of statute, the bank may argue that the woman failed to supervise
the man when he entered into the loan without authority. There is nothing, however, to suggest that the
man warranted supervision or that the woman was negligent for not supervising the man. Under RUPA,
each partner has equal rights in the management and conduct of the partnership’s business. Thus, in a
partnership, each partner is deemed to be the co-equal of the other partners, and no partner is under the
control of the other partners.]

Explanation to Point-Three(a) (20%):

By improperly obtaining the bank loan and then misappropriating the loan proceeds, the man breached his
fiduciary duty of loyalty and his duty of care.

Under RUPA, a partner owes to the partnership and the other partners the duties of loyalty and care.
Partners are liable for damages to the partnership and co-partners for breach of these duties. Claims for

Seperac-J19 Exam-Released MEE Essay Compilation © 2016-2020 564


breach of duties by partners in a limited liability partnership are not subject to the rule of limited liability
applicable to claims by outside parties. Here, the man’s conduct breached his fiduciary duty of loyalty, as
well as his broader duty of care.

The fiduciary duty of loyalty includes the obligation to refrain from appropriating partnership assets for
personal use. A partner’s duty of loyalty includes the duty to account to the partnership for any property,
profit, or benefit derived by the partner in the conduct of the partnership business. The man breached this
duty by misappropriating the proceeds of the loan from the bank made to the partnership.

The duty of care, which is remediable in damages, includes a duty not to engage in intentional misconduct
and knowing violations of law. The man breached this duty by not mentioning or providing the loan
officer with the statement of partnership authority, which limited his ability to borrow more than $10,000
without the other partner’s consent.

[NOTE: RUPA, as revised in 2013, treats a partner’s duty of loyalty as a fiduciary duty, the breach of
which gives rise to a full range of legal and equitable remedies. On the other hand, the revision does not
refer to a partner’s duty of care as a fiduciary duty, because the duty of care applies in many non-
fiduciary situations. Thus, an examinee who refers to the duty of care as a fiduciary duty in the context of
this question should receive full credit.]

Explanation to Point-Three(b) (20%):

The woman (or the partnership) can bring a direct action against the man for breaching his duties of
loyalty and care. The woman can also bring an accounting action seeking to have the man pay damages to
the partnership for his loyalty breach.

The duties of loyalty and care run to both the partnership and the other partners. Thus, the partnership can
maintain an action against a partner for violating his fiduciary duties to the partnership and thus causing
harm to the partnership. In addition, a partner can maintain an action against another partner, with or
without an accounting, to enforce the partner’s rights under the partnership act, including an action for
violations of duties.

Here, the woman can bring a direct action seeking to have the man make her whole for any losses to her
caused by his misconduct that breached his duties of loyalty and care. She can also bring an accounting
action to have the man account to the partnership for the money he took from the partnership. Although
the partnership could seek damages for these breaches as well, in this two-person partnership it is unlikely
that the man would agree to have the partnership sue him.

[NOTE: Some examinees might conclude that the woman can bring a direct action only for the man’s
care breach, and the woman would have to pursue an accounting action with respect to the loyalty
breach. Although this seems to have been the approach of RUPA (1997), this approach was abandoned in
the 2013 revisions to ensure that partners can bring direct claims to protect their interests. Finally, some
examinees might point out that derivative actions are not permitted on behalf of a partnership under
RUPA or the UPA. This analysis does not affect the conclusion that the woman can bring a direct action
against the man for breaching his duties of loyalty and care, given that both of these duties run to both the
partnership and the other partners.]

Seperac-J19 Exam-Released MEE Essay Compilation © 2016-2020 565


#124-FEB 2015–MEE Q01: QUESTION ONE (AGENCY-PARTNERSHIP/TORTS)

For many years, a furniture store employed drivers to deliver furniture to its customers in vans it owned.

Several months ago, however, the store decided to terminate the employment of all its drivers. At the
same time, the store offered each driver the opportunity to enter into a contract to deliver furniture for the
store as an independent contractor. The proposed contract, labeled “Independent-Contractor Agreement,”
provided that each driver would

(1) provide a van for making deliveries;

(2) use the van only to deliver furniture for the store during normal business hours and according to the
store’s delivery schedule; and

(3) receive a flat hourly payment based upon 40 work hours per week, without employee benefits.

The proposed Independent-Contractor Agreement also specified that the store would not withhold income
taxes or Social Security contributions from payments to the driver.

The store also offered each driver the opportunity to lease a delivery van from the store at a below-market
rate. The proposed lease required the driver to procure vehicle liability insurance. It also specified that the
store would reimburse the driver for fuel and liability insurance and that the lease would terminate
immediately upon termination of the driver’s contract to deliver furniture for the store.

All the drivers who had been employed by the store agreed to continue their relationships with the store
and executed both an Independent-Contractor Agreement and a lease agreement for a van.

Three months ago, a driver delivered furniture to a longtime customer of the store during normal business
hours. The customer asked the driver to take a television to her sister’s home, located six blocks from the
driver’s next delivery, and offered him a $10 tip to do so. The driver agreed, anticipating that this delivery
would add no more than half an hour to his workday.

In violation of a local traffic ordinance, the driver double-parked the delivery van in front of the sister’s
house to unload the television. A few minutes later, while the driver was in the sister’s house, a car
swerved to avoid the delivery van and skidded into oncoming traffic. The car was struck by a garbage
truck, and a passenger in the car was seriously injured.

The passenger has brought a tort action against the store to recover damages for injuries resulting from the
driver’s conduct. Pretrial discovery has revealed that delivery vans routinely double-park; survey
evidence suggests that, in urban areas like this one, 80% of deliveries are made while the delivery van is
double-parked.

In this jurisdiction, there is no law that imposes liability on a vehicle owner for the tortious acts of a driver
of that vehicle solely on the basis of vehicle ownership.

The store argues that it is not liable for the passenger’s injuries because (a) the driver is an independent
contractor; (b) even if the driver is not an independent contractor, the driver was not making a delivery for

Seperac-J19 Exam-Released MEE Essay Compilation © 2016-2020 566


the store when the accident occurred; and (c) the driver himself could not be found liable for the
passenger’s injuries.

1. Evaluate each of the store’s three arguments against liability.

2. Assuming that the store is liable to the passenger for the passenger’s injuries, what rights, if any,
does the store have against the driver? Explain.

Seperac-J19 Exam-Released MEE Essay Compilation © 2016-2020 567


#124: F15-1 MEE: ANSWER: NCBE (AGENCY-PARTNERSHIP/TORTS)

POINT (1) [30%] ISSUE: Is the driver an independent contractor or an employee of the store?
ANSWER: Although the store characterized the driver as an independent contractor, the store had
the right to control his conduct and thus the driver was an employee of the store.

POINT (2) [30%] ISSUE: Did the driver’s violation of the store’s delivery rules place his act outside
the scope of his employment? ANSWER: No. The driver was acting within the scope of his
employment because his conduct was substantially similar to authorized acts, and the driver did not
deviate substantially from his route or likely consume a substantial amount of time by deviating.

POINT (3) [25%] ISSUE: May the driver be found liable to the passenger in the car for personal
injuries resulting from his violation of a double-parking ordinance? ANSWER: Yes. The driver
may be found negligent per se in causing the passenger’s injuries because a purpose of the double-
parking prohibition was almost certainly to prevent traffic accidents. The fact that the driver’s
violation of the statute conformed to custom does not alter this conclusion.

POINT (4) [15%] ISSUE: If the store pays damages to the passenger, is it entitled to
indemnification from the driver? ANSWER: Yes. If the driver’s negligence subjects the store to
liability and the store pays damages to the passenger, under the common law the store is entitled to
indemnification from the driver because there is no evidence that the store itself engaged in tortious
conduct related to the passenger’s injuries.

ANSWER DISCUSSION:

The driver is likely to be characterized as an employee, not as an independent contractor, because his
conduct was subject to the store’s control. Under the respondeat superior principle, an employer is liable
for the tortious acts of its employee if those acts were performed within the scope of the employee’s
employment. Here, the driver’s violation of the store’s delivery rules likely did not place his act outside
the scope of his employment because the driver did not deviate substantially from his route and the risks
caused by his deviation were not different from the risks inherent in his authorized activities. In virtually
all jurisdictions, the driver could be found liable for the passenger’s injuries. The driver was negligent in
violating a traffic ordinance and the type of harm that occurred, a traffic accident, was among those that
the ordinance was aimed to avert. Finally, if the store is found liable for the driver’s negligence and pays
damages to the passenger, under common law principles the store is entitled to indemnification from the
driver because the store was not itself at fault in causing the accident.

ANSWER EXPLANATION:

Explanation to Point-One (30%):

Although the store characterized the driver as an independent contractor, the store had the right to control
his conduct and thus the driver was an employee of the store.

The store can be found liable under these facts if the driver is treated as its employee acting within the
scope of his employment. The store is not liable for torts committed by the driver if the store employed
him as an independent contractor to deliver furniture.

Seperac-J19 Exam-Released MEE Essay Compilation © 2016-2020 568


Under the Restatement of Agency, the test of whether a person is an employee is whether the person’s
“physical conduct in the performance of the services is subject to the employer’s control or right to
control.” Under the Third Restatement of Agency, an employee is an agent whose principal controls or
has the right to control the manner and means of the agent’s performance of work. This is generally a
question of fact. A number of factors are relevant, including the level of skill required to perform the
work, who supplies the instrumentalities used, the duration of the relationship, and whether the work is
part of the principal’s regular business. No single factor is determinative.

Here, the Restatement factors strongly suggest that the store had a right to control the driver’s work. First,
the job of furniture delivery driver does not require a tremendous amount of skill as compared to
occupations such as plumber or electrician that are typically performed by independent contractors.
Second, while the “Independent-Contractor Agreement” between the parties required the driver to supply
the van used in the deliveries, the store was primarily responsible for the van: the store leased the van to
the driver at a below-market rate; the store was responsible for the expenses of operating the van; and the
lease terminated upon the termination of the driver’s relationship with the store. Third, the driver’s
relationship with the store was ongoing, indefinite, and substantial; it was not specific to a single delivery
or series of deliveries. Fourth, the driver was paid by the hour, not by the job, and worked a regular 40-
hour week. Fifth, furniture delivery was a regular part of the store’s business. Finally, although the
“Independent-Contractor Agreement” between the store and the driver provided that the driver was an
independent contractor and had discretion over the means and manner of performing the terms of the
contract, “it is not determinative that the parties believe or disbelieve that the relation of employer and
employee exists, except insofar as such belief indicates an assumption of control by the one and
submission to control by the other.” In sum, the facts strongly suggest that the driver was the store’s
employee.

[NOTE: The Second Restatement of Agency uses the terminology “master” and “servant”; the Third
Restatement of Agency uses the terminology “employer” and “employee.” This shift in terminology does
not affect the analysis. The test is still control of, or the right to control, the performance of the agent’s
work.]

Explanation to Point-Two (30%):

The driver was acting within the scope of his employment because his conduct was substantially similar
to authorized acts, and the driver did not deviate substantially from his route or likely consume a
substantial amount of time by deviating.

If the driver is characterized as an employee instead of an independent contractor, the store will be liable
for the driver’s tortious conduct only if it occurred within the scope of his employment. Whether an
employee was acting within the scope of his employment is generally a question of fact. An employee’s
conduct is within the scope of his employment if (1) it is of the kind that the employee is employed to
perform; (2) it occurs substantially within the authorized time and space limits; and (3) it is motivated, at
least in part, by a purpose to serve the employer. An employee acts within the scope of employment when
performing work assigned by the employer or engaging in a course of conduct subject to the employer’s
control. The fact that the act was not authorized is not determinative.

Here, it is highly likely that the driver will be found to have been acting within the scope of his
employment. First, the conduct that caused the accident, parking, was exactly the kind of conduct that the
driver was employed to perform, and the risks of double-parking were characteristic of those his
employment entailed. Second, the time and space deviations from his authorized activities were not

Seperac-J19 Exam-Released MEE Essay Compilation © 2016-2020 569


substantial: the driver drove a mere six blocks out of the way; he did so during regular business hours; and
as an experienced furniture delivery person, the driver would probably have used no more than half an
hour for the deviation. Finally, the driver may have been motivated at least in part by a desire to serve the
store. The driver may have believed that making the delivery of the television for a longtime customer
would generate goodwill for the store. In sum, the delivery of the television appears to have been a minor
detour that did not take the driver out of the scope of employment. Under the Third Restatement of
Agency, an employee’s act is not within the scope of employment when it occurs within an independent
course of conduct not intended by the employee to serve any purpose of the employer.

[NOTE: While the formulation of the scope of employment doctrine in the Third Restatement of Agency
“is phrased in more general terms” than in the Second Restatement of Agency, the drafters state that the
definition “reflects the definition of scope of employment applied in most cases and in most
jurisdictions.” In sum, the results under both formulations should be similar.]

Explanation to Point-Three (25%):

The driver may be found negligent per se in causing the passenger’s injuries because a purpose of the
double-parking prohibition was almost certainly to prevent traffic accidents. The fact that the driver’s
violation of the statute conformed to custom does not alter this conclusion.

“An actor is negligent if, without excuse, the actor violates a statute that is designed to protect against the
type of accident the actor’s conduct causes, and if the accident victim is within the class of persons the
statute is designed to protect.” Health and safety statutes, like traffic ordinances, are typically deemed to
protect the public at large.

In order to make use of a statutory standard to establish negligence, the plaintiff must show that the type
of injury is one the statute is aimed against. However, “courts are astute at finding multiple and subsidiary
statutory purposes.” Here, the double-parking ordinance was almost certainly aimed at preventing both
traffic congestion and damage resulting from traffic accidents; vehicles parked in the path of moving
traffic are a hazard that forces drivers into lanes of oncoming traffic. The damage resulting from traffic
accidents includes personal injuries, so the passenger’s injuries involve the right type of harm.

The fact that the driver’s actions conformed to custom does not mean that he cannot be found negligent;
“there are precautions so imperative that even their universal disregard will not excuse their omission.”
Courts have sometimes read into statutes customary exceptions to a custom embodied by the statute. For
example, one court interpreted a statute that required pedestrians to walk on the left – the common-law
custom – to include the common-law exception which required pedestrians to walk with the traffic when
traffic coming from behind was much lighter than oncoming traffic. They have done so on the theory that
a legislature would not have intended to decree that individuals should follow a general rule of conduct
“even under circumstances where observance would subject them to unusual risk.” Here, there is no
evidence that double-parking was a safer alternative than parking legally or that the driver was following
any custom dictated by safety concerns. Therefore, a jury could properly conclude that the driver was
negligent per se in violating the double-parking ordinance and that the driver’s negligence was the legal
cause of the passenger’s injuries.

Explanation to Point-Four (15%):

If the driver’s negligence subjects the store to liability and the store pays damages to the passenger, under
the common law the store is entitled to indemnification from the driver because there is no evidence that
the store itself engaged in tortious conduct related to the passenger’s injuries.

Seperac-J19 Exam-Released MEE Essay Compilation © 2016-2020 570


Indemnification (full reimbursement for damages paid to the plaintiff) is available to a tort defendant who
has paid the plaintiff’s damage award when, as between the paying and nonpaying defendants, the paying
defendant was not at fault in causing the plaintiff’s injuries and the non-paying defendant was at fault. By
contrast, a joint tortfeasor who was at fault in causing the plaintiff’s injuries is entitled only to
contribution, i.e., partial reimbursement for damages paid to the plaintiff. A classic case in which
indemnification is appropriate is that of an employer, like the store, who is liable to the plaintiff based
solely on the principle of respondeat superior.

Consequently, if the store is required to pay damages to the passenger, the driver will be liable to the store
for the amount paid.

[NOTE: In some states, this common law principle has been modified by statutes or court decisions that
prevent employers from seeking indemnification from employees in certain circumstances.]

[NOTE: An examinee who concludes that the store’s delivery drivers routinely double-parked, and that
the store had actual or implied notice of that practice, should conclude that the store could be found
negligent and that contribution but not indemnification would be available to the store.]

Seperac-J19 Exam-Released MEE Essay Compilation © 2016-2020 571


#125-FEB 2014–MEE Q06: QUESTION SIX (AGENCY-PARTNERSHIP)

Five years ago, Adam and Ben formed a general partnership, Empire Partnership (Empire), to buy and sell
antique automobiles at a showroom in State A. Adam contributed $800,000 to Empire, and Ben
contributed $200,000. Their written partnership agreement allocated 80% of profits, losses, and control to
Adam and 20% to Ben. No filings of any type were made in connection with the formation of Empire.

Three years ago, a collector purchased one of Empire’s antique cars for $3,400,000. The collector was
willing to pay this price because of Ben’s false representation (repeated in the sales contract) that a
famous movie star had once owned the car. Without the movie-star connection, the car was worth only
$100,000. One month later, when the collector discovered the truth, he sued Adam, Ben, and Empire for
$3,300,000 in damages. The lawsuit is still pending.

Two years ago, Adam and Ben admitted a new partner, Diane, to Empire in return for her contribution of
$250,000. The three agreed to allocate profits, losses, and control 75% to Adam, 10% to Ben, and 15% to
Diane. Before joining the partnership, Diane learned of the collector’s claim and stated her concern to
Adam and Ben that she might become liable if the claim were reduced to a judgment.

Following Diane’s admission to Empire, the three partners sought to convert Empire into a limited
liability partnership (LLP). Adam’s lawyer proposed to file with State A a “statement of qualification”
making an LLP election and declaring the name of the partnership to be “Empire LLP.” Ben’s lawyer
stated that this would not work and that a new LLP had to be formed, with the assets of the old
partnership transferred to the new one. In the end, the conversion was done the way Adam’s lawyer
suggested with the approval of all three partners.

One year ago, a driver purchased a vintage car from Empire LLP, based on the representation that the car
was “fully roadworthy and capable of touring at 70 mph all day.” The driver took the car on the highway
at 50 mph, whereupon the front suspension collapsed, resulting in a crash in which the car was destroyed
and the driver killed. The driver’s estate sued Adam, Ben, Diane, and Empire LLP for $10,000,000. The
lawsuit is still pending.

Although profitable, Empire LLP does not have resources sufficient to pay the collector’s claim or the
claim of the driver’s estate.

Assume that the Uniform Partnership Act (1997) applies.

1(a). Before the filing of the statement of qualification, was Adam personally liable on the
collector’s claim? Explain.

1(b). Before the filing of the statement of qualification, was Diane personally liable on the
collector’s claim? Explain.

2(a). After the filing of the statement of qualification, was Adam, Diane or Ben personally liable as
a partner on the collector’s claim? Explain.

2(b). After the filing of the statement of qualification, was Adam, Diane or Ben personally liable as
a partner on the driver’s estate’s claim? Explain.

Seperac-J19 Exam-Released MEE Essay Compilation © 2016-2020 572


#125: F14-6 MEE: ANSWER: NCBE (AGENCY-PARTNERSHIP)

POINT (1) [30%] ISSUE: Is a partner in a general partnership personally liable on a claim arising
from misrepresentations by another partner made in the course of the partnership business?
ANSWER: Yes. As a general partner of Empire, a general partnership, Adam became personally
liable on the collector’s claim, a valid claim against the partnership that arose because of Ben’s
wrongful act in the ordinary course of the partnership business.

POINT (2) [30%] ISSUE: Does a newly admitted partner in a general partnership become
personally liable on existing claims against the partnership? ANSWER: No. Because the collector’s
claim arose before Diane joined Empire, Diane did not become personally liable on the claim.

POINT (3) [40%] ISSUE: After the filing by a general partnership of a statement of qualification as
a limited liability partnership, are the partners personally liable as partners on (a) an existing claim
against the general partnership and (b) a claim against the partnership that arose after the filing?
ANSWER: Filing the statement of qualification was effective to elect limited liability partnership
status. Despite this new status, Adam and Ben remain personally liable on the collector’s claim,
which arose before the election. But as partners in an LLP, neither Adam, Diane or Ben is
personally liable as a partner on the driver’s estate’s claim, which arose after the election.

ANSWER DISCUSSION:

Adam and Ben formed a general partnership, under which they were jointly and severally liable for
obligations of the partnership. Thus, Adam was personally liable for misrepresentations by Ben made in
the ordinary course of the partnership business. Upon joining the general partnership, Diane became
personally liable for the obligations of the partnership arising after her admission, but not for obligations
pre-existing her admission, such as the collector’s claim. By filing a statement of qualification, the three
partners properly elected limited liability partnership status. As partners in an LLP, none of the three
partners is personally liable as a partner for partnership obligations arising after the election, such as the
claim by the driver’s estate. The election, however, does not change their personal liability on pre-existing
claims that arose before the election, such as the collector’s claim.

ANSWER EXPLANATION:

Explanation to Point-One (30%):

As a general partner of Empire, a general partnership, Adam became personally liable on the collector’s
claim, a valid claim against the partnership that arose because of Ben’s wrongful act in the ordinary
course of the partnership business.

When the collector’s claim arose, Empire was a general partnership composed of Adam and Ben. Under
the UPA, partners of a general partnership are liable jointly and severally for all obligations of the
partnership. Under the UPA, the partnership could become obligated for the loss caused to the collector as
a result of the misrepresentation by Ben, provided he was acting in the ordinary course of the partnership
business. Because there was no statement that limited his partnership authority, Ben as partner was “an
agent of the partnership for the purpose of its business.” Ben’s misrepresentation to the collector, even if
intentional, appears to be in the ordinary course of the partnership’s business of dealing in antique cars.

Seperac-J19 Exam-Released MEE Essay Compilation © 2016-2020 573


Thus, Ben’s wrongful act created a partnership obligation for which Adam was jointly and severally
liable.

[NOTE: Generally, a partnership creditor must “exhaust the partnership’s assets before levying on a
judgment debtor partner’s individual property where the partner is personally liable for the partnership
obligation” as a result of his status as a partner. As the UPA comments explain, this places Adam more in
the position of guarantor than principal debtor on the partnership obligation. Although an examinee
might discuss this point, the call focuses on whether Adam is personally liable, not how the liability might
be enforced.]

Explanation to Point-Two (30%):

Because the collector’s claim arose before Diane joined Empire, Diane did not become personally liable
on the claim.

Diane was admitted to Empire when it was a general partnership and after the collector’s claim arose.
While the general rule under UPA § 306(a) is that the partners of a general partnership are liable jointly
and severally for all obligations of the partnership, there is a special rule for partners who are admitted
during the duration of the partnership. Under UPA § 306(b), a person admitted to an existing partnership
is not personally liable for any partnership obligations incurred before the person’s admission. Because
Diane was admitted to Empire after the collector’s claim arose, Diane is not personally liable on the
claim.

Diane’s knowledge of the pre-existing claim and her stated concern about becoming liable on the
collector’s claim do not change her personal nonliability to the collector. Although partners who have a
liability shield can assume liability to third parties through private contractual guarantees or modifications
to the partnership agreement, Diane’s stated concern constituted neither a guaranty to the collector nor “an
intentional waiver of liability protections.”

At most, Diane will lose her investment in the partnership as a result of the collector’s claim. Although
Diane did not become personally liable on the collector’s claim when she joined the partnership, the
$250,000 she contributed to the partnership is “at risk for the satisfaction of existing partnership debts.”

Explanation to Point-Three (40%):

Filing the statement of qualification was effective to elect limited liability partnership status. Despite this
new status, Adam and Ben remain personally liable on the collector’s claim, which arose before the
election. But as partners in an LLP, neither Adam, Diane or Ben is personally liable as a partner on the
driver’s estate’s claim, which arose after the election.

Under UPA § 1001, a general partnership can make an election and become a limited liability partnership
– if the partners approve the conversion by a vote equivalent to that necessary to amend the partnership
agreement and the partnership then files a statement of qualification that specifies the name of the
partnership, its principal office, and its election to be an LLP. Here the partners agreed unanimously –
sufficient to amend their agreement under UPA § 401 – and the statement of qualification was filed. In
addition, the name of Empire LLP properly included an appropriate ending, “LLP.”

Although another way to effectuate a “conversion” (as suggested by Ben’s lawyer) is to form a new LLP
and transfer the assets of the old general partnership to the new LLP, the method used here (approval by
the partners and the filing of a statement of qualification) is also sufficient to create LLP status.

Seperac-J19 Exam-Released MEE Essay Compilation © 2016-2020 574


Thus, Empire became Empire LLP as of the date of filing of the statement of qualification. What effect
did this have on the collector’s claim, which predated the filing? According to UPA § 306, an obligation
incurred while a partnership is an LLP is solely a partnership obligation. As the collector’s claim predated
the LLP, Adam and Ben remain personally liable on the collector’s claim. Diane, on the other hand, was
not personally liable on the collector’s claim, either before or after the filing of the statement of
qualification.

The driver’s estate’s claim arose after Empire became Empire LLP. Under UPA § 306, an obligation
incurred while a partnership is an LLP is solely a partnership obligation. Thus, Adam, Ben, and Diane as
partners are all protected from personal liability on the driver’s estate’s claim. But there may be personal
liability if any of them was negligent or otherwise acted wrongfully by not informing the buyer of the bad
suspension that caused the accident.

Seperac-J19 Exam-Released MEE Essay Compilation © 2016-2020 575


#126-JUL 2013–MEE Q02: QUESTION TWO (AGENCY-PARTNERSHIP/TORTS)

After a dump truck unloaded gravel at a road construction job site, the trucker negligently drove away
with the truck bed still in a raised position. The raised truck bed hit an overhead cable, causing it to fall
across the highway.

The telephone company that owned the fallen cable sent one of its employees to the scene in a company
vehicle. The employee’s responsibilities were expressly limited to responding to cable-damage calls,
assessing damage, and reporting back to the telephone company so that a repair unit could be dispatched.

The foreman of the road construction job site asked the telephone company employee if the foreman’s
crew could lift the cable off the highway. Fearful that the cable might be damaged by traffic, the
telephone company employee said, “Go ahead, pick it up. Just don’t damage the cable.” The foreman then
directed his crew to stretch the cable over the highway so that traffic could pass underneath.

Shortly thereafter, a bus passing under the telephone cable hit the cable and dislodged it, causing the cable
to strike an oncoming car. The driver lost control of the car and hit a truck carrying asphalt to the road
construction site. As a result of the collision, hot asphalt spilled and severely burned the foreman.

The foreman is now threatening to sue the telephone company on the ground that it is responsible for its
employee’s negligence in authorizing the road construction crew to stretch the cable across the highway.
The telephone company argues that, even assuming that its employee was negligent, the telephone
company is not liable because:

1. the telephone company employee’s acts were outside the scope of his employment and thus
cannot be attributed to the telephone company;

2. there is no other agency theory under which the foreman could hold the telephone company
liable for its employee’s acts; and

3. the telephone company employee’s acts were not the proximate cause of the foreman’s injuries.

Assess each of the telephone company’s responses.

Seperac-J19 Exam-Released MEE Essay Compilation © 2016-2020 576


#126: J13-2 MEE: ANSWER: NCBE (AGENCY-PARTNERSHIP/TORTS)

POINT (1) [35%] ISSUE: Was the employee acting within the course and scope of his employment
when he authorized the foreman to lift the cable? ANSWER: Yes. The telephone company is liable
for torts committed by its employee while acting within the course and scope of his employment
with the telephone company. Because the employee was motivated by a desire to serve the telephone
company, he was acting within the scope of his employment.

POINT (2) [30%] ISSUE: Did the telephone company cloak the employee with apparent authority
that was relied upon by the foreman, resulting in his injury? ANSWER: Yes. The telephone
company could also be liable to the foreman under an apparent authority theory.

POINT (3) [35%] ISSUE: Were the employee’s actions the proximate cause of the foreman’s
injuries? ANSWER: Yes. The employee’s acts were probably the proximate cause of the foreman’s
injuries.

ANSWER DISCUSSION:

The telephone company is liable to the foreman for the telephone company employee’s negligence if the
employee was acting within the course and scope of his employment with the telephone company when
he authorized the foreman to direct the crew to stretch the cable over the highway. Here the employee
probably was. Even if the employee was acting outside the scope of his employment with the telephone
company, the telephone company would be liable to the foreman if the foreman relied on statements or
conduct by the employee that were within the employee’s apparent authority. Although the type of harm
the foreman suffered (burns) was different from the harm risked by the employee’s actions (being struck
by the cable), the employee’s actions are probably the proximate cause of the foreman’s injuries.

ANSWER EXPLANATION:

Explanation to Point-One (35%):

The telephone company is liable for torts committed by its employee while acting within the course and
scope of his employment with the telephone company. Because the employee was motivated by a desire
to serve the telephone company, he was acting within the scope of his employment.

Under the doctrine of respondeat superior, employers are liable for the actions of an employee when the
employee is acting within the scope of his employment.

Here, the employee had no express authority to authorize the raising of the cable. His only duty was to
determine whether a cable was down and, if so, to assess the damage and report to the telephone
company. However, the employee’s unauthorized conduct may fall within the scope of his employment.
Under the Third Restatement of Agency, conduct is not outside the scope of employment merely because
an employee disregards the employer’s instructions. Section 7 of the Third Restatement of Agency states
that “an employee acts within the scope of employment when performing work assigned by the employer
or engaging in a course of conduct subject to the employer’s control. An employee’s act is not within the
scope of employment when it occurs within an independent course of conduct not intended by the
employee to serve any purpose of the employer.”

Seperac-J19 Exam-Released MEE Essay Compilation © 2016-2020 577


Here, the employee’s action (authorizing the raising of the cable) was motivated by a desire to protect the
cable from further damage. Thus, the employee was clearly acting with the intent to serve the telephone
company’s purposes. The cable which ultimately caused the injury to the foreman was the property of the
telephone company. The telephone company should have reasonably expected that when the employee
arrived at the scene he would take necessary action to prevent further damage to its property if he were in
a position to do so. Although it is not certain, the telephone company is probably liable to the foreman
under the doctrine of respondeat superior.

Explanation to Point-Two (30%):

The telephone company could also be liable to the foreman under an apparent authority theory.

A principal will sometimes be liable for torts committed by its agents even if the conditions of respondeat
superior liability (i.e., an employer/employee relationship and conduct within the scope of employment)
are not satisfied.

For example, a principal can be liable if the principal was negligent or reckless in the selection of the
agent. Nothing in the facts suggests that the telephone company was negligent in its hiring of the
employee.

A principal can also be liable for an agent’s torts if the principal has a special relationship with the injured
person that imposed a special duty on the principal to take care to protect against the risk that the agent
would harm the injured person. Nothing on these facts suggests that the telephone company had any such
special relationship with the foreman.

Finally, a principal is liable for a tort committed by an agent when the agent “appears to deal or
communicate on behalf of the principal and the agent’s appearance of authority enables the agent to
commit the tort.” On the facts of this problem, the employee had apparent authority vis-à-vis the foreman.
“Apparent authority is created by a person’s manifestation that another has authority to act with legal
consequences for the person who makes the manifestation, where a third party reasonably believes the
actor to be authorized and the belief is traceable to the manifestation.” An agent has apparent authority
with respect to a third party only “when the third party reasonably believes that the agent has authority to
act on behalf of the principal.” Here, the telephone company made manifestations to the foreman about
the employee’s authority when it sent its employee to the job site in a company vehicle. This would give
the foreman a basis to believe that the employee was acting on the telephone company’s behalf.

The principal is liable under an apparent authority theory only when the injured person’s belief in the
agent’s authority “enables the agent to commit the tort.” As noted, the employee arrived at the job site in
the telephone company’s vehicle. This was sufficient to create a reasonable belief that the employee was
acting on behalf of the company on all matters relating to the fallen cable. As a result of this reasonable
belief, the foreman, acting on the employee’s authorization, raised the cable, creating the circumstances
leading to his injury.

Thus, on an apparent authority theory the company could be liable assuming respondeat superior did not
apply.

Explanation to Point-Three (35%):

The employee’s acts were probably the proximate cause of the foreman’s injuries.

Seperac-J19 Exam-Released MEE Essay Compilation © 2016-2020 578


In a negligence action, a defendant is liable only if his conduct was the proximate cause of the plaintiff’s
injury. Here, it is likely that a court would find that the foreman’s harm was proximately caused by the
employee’s negligence.

Even when a defendant is negligent, his conduct must have “such an effect in producing the harm as to
lead reasonable men to regard it as a cause, using that word in the popular sense, in which there always
lurks the idea of responsibility, rather than in the so-called ‘philosophic sense,’ which includes every one
of the great number of events without which any happening would not have occurred.”

Intervening actors or events that produce harm different in kind from that which one would normally
anticipate may break the chain of causation and lead a court to conclude that the defendant’s acts are not
the proximate cause of the plaintiff’s injury. Thus, a plaintiff who negligently exceeds a speed limit and
therefore happens to be on the spot where a tree falls during a violent windstorm may still recover for
injuries caused by the tree , and a defendant who negligently drops a passenger off at the wrong train
station is not liable for the passenger’s burns caused by a malfunctioning kerosene lamp in the hotel
where she is forced to stay for the night. In evaluating whether intervening acts break the chain of
causation, courts typically analyze both their foreseeability and their degree of dependence on the
defendant’s negligence.

Here, it was entirely foreseeable that a vehicle would strike the raised cable, causing impact damage to
cars and persons within the range of the cable’s fall; that is what made the employee’s conduct negligent.
Given that the cable was stretched over a highway, it was also foreseeable that a car struck by a cable
would go out of control and cause injuries to other vehicles or bystanders, and that is exactly what
happened here: the driver lost control of the car, hitting another vehicle and thereby causing serious
personal injury to a bystander, the foreman. Although the source of the foreman’s injuries (burns from hot
asphalt) would not normally be anticipated from the fall of a cable, the foreman was within the zone of
foreseeable risk and suffered injuries in the course of a foreseeable, reasonably direct chain of
circumstances. A court is thus likely to find that the employee’s negligence was the proximate cause of
the foreman’s injuries.

Seperac-J19 Exam-Released MEE Essay Compilation © 2016-2020 579


#127-FEB 2013–MEE Q06: QUESTION SIX (AGENCY-PARTNERSHIP)

Over 5,000 individuals in the United States operate hot-air balloon businesses. A hot-air balloon has four
key components: the balloon that holds the heated air, the basket that houses the riders, the propane
burner that heats the air in the balloon, and the propane storage tanks.

The owner of a hot-air balloon business recently notified several basket and burner manufacturers that she
or her agent might be contacting them to purchase baskets or burners. The owner did not specifically
name any person as her agent. Basket and burner manufacturers regularly receive such notices from hot-
air balloon operators. Such notices typically include no restrictions on the types of baskets or burners
agents might purchase for their principals.

The owner then retained an agent to acquire baskets, burners, and fuel tanks from various manufacturers.
The owner authorized the agent to buy only (a) baskets made of woven wicker (not aluminum), (b)
burners that use a unique “whisper technology” (so as not to scare livestock when the balloon sails over
farmland), and (c) propane fuel tanks.

The agent then entered into three transactions with manufacturers, all of whom had no prior dealings with
either the owner or the agent.

(1) The agent and a large manufacturer of both wicker and aluminum baskets signed a contract for the
purchase of four aluminum baskets for a total cost of $60,000. The agent never told the manufacturer that
he represented the owner or any other principal. The contract listed the agent as the buyer and listed the
owner’s address as the delivery address but did not indicate that the address was that of the owner rather
than the agent. When the baskets were delivered to the owner, she learned for the first time that the agent
had contracted to buy aluminum, not wicker, baskets. The owner immediately rejected the baskets and
returned them to the manufacturer. Neither the owner nor the agent has paid the basket manufacturer for
them.

(2) The agent contacted a burner manufacturer and told him that the agent represented a well-known hot-
air balloon operator who wanted to purchase burners. The agent did not disclose the owner’s name. The
agent and the burner manufacturer signed a contract for the purchase of four burners that did not have
“whisper technology” for a total price of $70,000. The burner contract, like the basket contract, listed the
owner’s address for delivery but did not disclose whose address it was. The burners were delivered to the
owner’s business, and the owner discovered that the agent had ordered the wrong kind of burners. The
owner rejected the burners and returned them to the manufacturer. Neither the owner nor the agent has
paid the burner manufacturer for the burners.

(3) The agent contracted with a solar cell manufacturer to make three cells advertised as “strong enough
to power all your ballooning needs.” The agent did not tell the manufacturer that he was acting on behalf
of any other person. One week after the cells were delivered to the agent, he took them to the owner, who
installed them and discovered that she could save a lot of money using solar cells instead of propane to
power her balloons. The owner decided to keep the solar cells, but she has not paid the manufacturer for
them.

Assume that the rejection of the baskets and the burners and the failure to pay for the solar cells constitute
breach of the relevant contracts.

Seperac-J19 Exam-Released MEE Essay Compilation © 2016-2020 580


1. Is the owner liable to the basket manufacturer for breach of the contract for the aluminum
baskets? Is the agent liable? Explain.

2. Is the owner liable to the burner manufacturer for breach of the contract for the burners? Is the
agent liable? Explain.

3. Is the owner liable to the solar cell manufacturer for breach of the contract for the solar cells? Is
the agent liable? Explain. (Do not address liability based upon restitution or unjust enrichment.)

Seperac-J19 Exam-Released MEE Essay Compilation © 2016-2020 581


#127: F13-6 MEE: ANSWER: NCBE (AGENCY-PARTNERSHIP)

POINT (1) [35%] ISSUE: Is the principal or the agent, or both, liable on contracts with a third
party when the principal is an “undisclosed principal”? ANSWER: The agent, but not the owner, is
liable to the basket manufacturer. The agent had no actual authority to enter into the contract to
buy aluminum baskets, and because the owner was an undisclosed principal, the manufacturer had
no reason to believe that the agent had apparent authority. Furthermore, the manufacturer had no
reason to believe that the agent was not contracting for his own benefit.

POINT (2) [35%] ISSUE: Is the principal or the agent, or both, liable on contracts with a third
party when the principal is “partially disclosed” or an “unidentified principal”? ANSWER: Yes.
Because the owner is an unidentified (as opposed to undisclosed) principal, both she and the agent
(as a party to the contract) probably are liable on the contract with the burner manufacturer.

POINT (3) [30%] ISSUE: Is the principal or the agent, or both, liable on contracts with a third
party for the purchase of goods when the agent exceeded his authority but the principal nonetheless
accepts the goods? ANSWER: Under the Second Restatement of Agency, the owner is not liable on
the contract for solar cells because the agent did not have actual or apparent authority and the
owner, as an undisclosed principal, cannot ratify the contract. Under the Third Restatement, the
owner could be liable, as she ratified the contract. Under either Restatement, the agent is liable as a
party to the contract.

ANSWER DISCUSSION:

The agent, but not the owner, is liable to the basket manufacturer because the owner is an undisclosed
principal and the agent acted without actual or apparent authority. Both the agent and the owner, however,
are liable on the burner contract because the owner is an unidentified principal and the agent had apparent
authority to enter into that contract. With respect to the solar cells contract, whether the owner is liable
depends upon whether a court would follow the Second or Third Restatement of Agency, which take
different positions on the effect of the ratification of a contract by an undisclosed principal. Under either,
the agent would also be liable on the contract, as he was a party to the contract. [NOTE: The contracts that
are the subject of this question are contracts for the sale of goods and, therefore, are governed by Article 2
of the Uniform Commercial Code. Article 2, however, does not contain agency rules. Accordingly,
common law concepts of agency are applicable.]

ANSWER EXPLANATION:

Explanation to Point-One (35%):

The agent, but not the owner, is liable to the basket manufacturer. The agent had no actual authority to
enter into the contract to buy aluminum baskets, and because the owner was an undisclosed principal, the
manufacturer had no reason to believe that the agent had apparent authority. Furthermore, the
manufacturer had no reason to believe that the agent was not contracting for his own benefit.

An agent acting on behalf of a principal can bind the principal to contracts if the agent has either actual or
apparent authority. An agent has actual authority when contracting on behalf of his principal if he
“reasonably believes, in accordance with the principal’s manifestations to the agent, that the principal
wishes the agent so to act.” Here, the agent was told to buy only wicker baskets, not aluminum baskets.

Seperac-J19 Exam-Released MEE Essay Compilation © 2016-2020 582


Thus, when he contracted with the basket manufacturer to buy aluminum baskets, he had no actual
authority to do so.

An agent acts with apparent authority “when a third party with whom the agent acts reasonably believes
the actor has authority to act on behalf of the principal and that belief is traceable to the principal’s
manifestations.” Here the owner notified basket manufacturers that she or her agent might contact them to
purchase baskets, but that notification did not specifically name the agent or any other person as the
owner’s agent. Furthermore, the basket manufacturer had no prior dealings with the agent or the owner or
any reason to think that the agent was acting for the benefit of anyone but himself. Thus, there is no basis
to conclude that the basket manufacturer thought that the agent had apparent authority to act for the
owner.

Generally, when an agent acts on behalf of an undisclosed principal and the agent lacks authority to enter
into the contract, the agent is liable on the contract as a party to the contract, but the principal is not liable.
This rule is consistent with the third party’s expectations. “The third party expected the agent to be a party
to the contract because the agent presented the deal as if he were acting for himself. Moreover, if the third
party is unaware of the principal’s existence, the third party must be relying on the agent’s solvency and
reliability when entering into the contract.” Furthermore, because the third party has no idea that the agent
is acting or is seemingly acting on behalf of another, there is no reason to believe that the third party
would be expecting an undisclosed principal to be liable on the contract.

Explanation to Point-Two (35%):

Because the owner is an unidentified (as opposed to undisclosed) principal, both she and the agent (as a
party to the contract) probably are liable on the contract with the burner manufacturer.

When the agent contracted with the burner manufacturer, he did not have actual authority to do so, as the
owner had expressly restricted the agent’s authority to purchase only burners with “whisper technology.”
However, the agent may have had apparent authority to buy burners without whisper technology.

An agent acts with apparent authority “when a third party with whom the agent acts reasonably believes
the actor has authority to act on behalf of the principal and that belief is traceable to the principal’s
manifestations.” The owner indicated that an agent might contact the burner manufacturer. The notice
contained no restriction regarding the type of burners that the agent was authorized to purchase. The facts
indicate that burner manufacturers regularly receive such notices.

Although the agent told the burner manufacturer that he represented a well-known hot-air balloon
operator, he did not disclose the owner’s name. Thus, the owner was a partially disclosed or unidentified
principal. An agent for a partially disclosed principal may have apparent authority. Based upon (1) the
notice sent by the owner, (2) the agent’s revelation that he was acting as an agent, and (3) the fact that
burner manufacturers regularly receive such notices and sell to agents, the manufacturer may argue that it
reasonably and actually believed that the agent was authorized to purchase burners without whisper
technology. The manufacturer may also argue that because the agent revealed that he was an agent, his
listing of the owner’s address as the delivery address connects the agent to the notice given by the owner.
Arguably this distinguishes the burner contract from the basket contract. Here, there is a strong case to
support the conclusion that the agent had apparent authority; if he did, then the owner is liable to the
burner manufacturer.

The agent also is liable as a party to the contract because he did not fully disclose his agency relationship.
Although he told the burner manufacturer that he represented a well-known hot-air balloon operator, he

Seperac-J19 Exam-Released MEE Essay Compilation © 2016-2020 583


did not disclose the owner’s name. Generally even an authorized agent of a partially disclosed or
unidentified principal is liable as a party to a contract with a third person, unless the agent and the third
party agree otherwise.

Explanation to Point-Three (30%):

Under the Second Restatement of Agency, the owner is not liable on the contract for solar cells because
the agent did not have actual or apparent authority and the owner, as an undisclosed principal, cannot
ratify the contract. Under the Third Restatement, the owner could be liable, as she ratified the contract.
Under either Restatement, the agent is liable as a party to the contract.

The owner is not liable to the solar cell manufacturer for breach of the contract for the solar cells because
the agent had no actual or apparent authority to purchase solar cells on the owner’s behalf, and the owner,
under the Second Restatement of Agency, did not ratify the contract with knowledge of the material facts.
Thus, she is not liable as a ratifier of the contract.

The facts state that the agent had authority to purchase only propane fuel tanks. In addition, he had no
apparent authority to purchase solar cells. The owner made no manifestations to the solar cell
manufacturer that would lead a reasonable person in the manufacturer’s position to believe that the agent
had the authority to bind the owner to a contract to purchase solar cells. In fact, the agent made no
manifestations at all to the solar cell manufacturer. Unlike with the basket manufacturer and the burner
manufacturer, the owner did not notify the manufacturer of solar cells that an agent might contact it to
purchase solar cells. In addition, the solar cells were delivered to the agent and not to the owner’s address.
In sum, the manufacturer was unaware of any relationship between the owner and the agent. As to the
solar cell manufacturer, the owner is an undisclosed principal. There can be no apparent authority in the
case of an undisclosed principal because there are no manifestations from the principal to the third person.
Pursuant to the Second Restatement of Agency, there can be no apparent authority created by an
undisclosed principal. According to the Second Restatement of Agency, apparent authority is not present
when a third party believes that an interaction is with an actor who is a principal.

The owner also did not ratify the contract. Although the owner used the solar cells, generally a principal
cannot ratify an unauthorized transaction with a third person “unless the one acting purported to be acting
for the ratifier.”

The result differs under the Third Restatement, which expressly rejects the Second Restatement on this
issue. The Third Restatement of Agency § 4.03 states, “A person may ratify an act if the actor acted or
purported to act as an agent on the person’s behalf.” According to comment b, “an undisclosed principal
may ratify an agent’s unauthorized act.” Under the Third Restatement of Agency rule, the owner probably
ratified the transaction. The agent clearly acted on the owner’s behalf, and in addition, the owner’s
conduct in using the solar cells “justifies a reasonable assumption that she is manifesting assent that the
act shall affect her legal relations.”

The agent also is liable to the solar cell manufacturer for breach of the contract for the solar cells because
he is a party to the contract. The facts indicate that the agent never told the solar cell manufacturer that he
represented the owner or any other principal. Consequently, even if the agent were authorized (which, as
discussed above, he is not), he would be liable as a party to the contract. Here, he has no authority or
apparent authority and is liable as a party to the contract.

The agent would also be liable under the Third Restatement. Under the Third Restatement of Agency §
4.02, ratification generally relates back and the transaction is treated as if it were authorized at the time of

Seperac-J19 Exam-Released MEE Essay Compilation © 2016-2020 584


the transaction. However, this does not relieve the agent of an undisclosed principal who ratifies an
unauthorized transaction of liability under the ratified contract. Under the Third Restatement of Agency,
an authorized agent for an undisclosed principal is a party to the contract and an undisclosed principal’s
ratification does not eliminate the agent’s liability to the third party on the transaction.

[NOTE: An examinee may discuss the concept of inherent agency power. This concept is recognized by
the Second Restatement of Agency § 8A, but the concept is not used in the Third Restatement of Agency.
Here, there are no facts to support that the agent had inherent authority. As to contracts with agents for
partially disclosed principals (e.g., the contract for the burners), the basic question is whether the acts
done “usually accompany or are incidental to transactions which the agent is authorized to conduct.” If
so, the principal is bound if the other party “reasonably believes that the agent is authorized to do them
and has no notice that he is not so authorized.” The purchase of burners without whisper technology was
not authorized, nor was it incidental to an authorized transaction. Therefore, there should not be inherent
agency power. As to contracts on behalf of undisclosed principals (e.g., the other two contracts), the basic
question is whether the acts done are usual or necessary in the transactions the agent is authorized to
transact. The other two contracts seem fundamentally different from the authorized transactions.
Therefore, there should not be inherent agency power. Only minimal credit should be given for discussion
of inherent agency power.]

Seperac-J19 Exam-Released MEE Essay Compilation © 2016-2020 585


#128-FEB 2012–MEE Q06: QUESTION SIX (AGENCY-PARTNERSHIP)

A man and a woman validly formed a partnership (“Garden Partnership”) to fix commercial gardening
equipment. Several months after Garden Partnership began operations, it hired an employee who was a
skilled mechanic.

The employee negligently repaired a piece of equipment for a customer. As a result, the customer was
severely injured. The customer successfully sued Garden Partnership and recovered a judgment for
$500,000, which has not been paid.

Shortly after entry of this judgment, the man and woman took the necessary steps to qualify Garden
Partnership as a limited liability partnership, and they renamed it “Garden LLP.”

Shortly thereafter, the man and woman decided to expand the business. Because they needed more capital,
they agreed to admit an investor as a partner. The investor contributed $50,000 and became a partner in
Garden LLP.

1. Is Garden LLP liable for the $500,000 judgment against Garden Partnership? Explain.

2. Are the man and woman personally liable to the customer for the $500,000 judgment against
Garden Partnership? Explain.

3. Is the investor personally liable to the customer for the $500,000 judgment against Garden
Partnership? Explain.

Seperac-J19 Exam-Released MEE Essay Compilation © 2016-2020 586


#128: F12-6 MEE: ANSWER: NCBE (AGENCY-PARTNERSHIP)

POINT (1) [45%] ISSUE: Is Garden LLP liable for the $500,000 judgment owed to the customer
when the injury that led to the judgment occurred prior to the partnership’s qualification as an
LLP? ANSWER: Yes. Garden LLP is liable to the customer for the judgment because the limited
liability partnership is the same entity that existed prior to its qualification as an LLP and assumes
all outstanding liabilities of the prior entity.

POINT (2) [35%] ISSUE: Are the man and woman personally liable for the $500,000 judgment
owed to the customer? ANSWER: Yes. The man and woman are jointly and severally liable to the
customer because the obligation to the customer was incurred prior to Garden Partnership’s
qualification as an LLP.

POINT (3) [20%] ISSUE: Is the investor personally liable for the $500,000 judgment owed to the
customer? ANSWER: No. The investor is not personally liable to the customer because the
customer’s injury occurred before the investor became a partner.

ANSWER DISCUSSION:

Garden LLP is the same entity as Garden Partnership and is liable for the $500,000 judgment owed to the
customer even though the injury that led to that judgment occurred prior to the partnership’s qualification
as an LLP. LLP status does not relieve partners of pre-existing liability for partnership obligations
incurred prior to qualification as an LLP, although it does limit their liability for obligations incurred after
such qualification. Therefore, the man and woman are jointly and severally liable for the $500,000 owed
to the customer because the injury that led to the judgment occurred before Garden Partnership, then a
general partnership in which the man and woman were the general partners, became Garden LLP. A new
partner is not personally liable for LLP obligations incurred before the person’s admission as a partner.
Therefore, the investor is not personally liable for the customer’s injury because the investor was not a
partner in Garden LLP (or Garden Partnership) when that injury occurred.

ANSWER EXPLANATION:

Explanation to Point-One (45%):

Garden LLP is liable to the customer for the judgment because the limited liability partnership is the same
entity that existed prior to its qualification as an LLP and assumes all outstanding liabilities of the prior
entity.

The Uniform Partnership Act (UPA 1997) adopts an entity theory of partnership. In other words, the
partnership itself is a legal entity and not a mere aggregate of individual partners. A partnership is an
entity distinct from its partners.

An LLP continues to be the same entity that existed prior to the filing of a statement of qualification as an
LLP. “The filing of a statement of qualification does not create a ‘new’ partnership. The filing partnership
continues to be the same partnership entity that existed before the filing.”

Here, since Garden LLP is the same entity as Garden Partnership, it has all of Garden Partnership’s
outstanding obligations, and Garden LLP is liable to the customer for the judgment.

Seperac-J19 Exam-Released MEE Essay Compilation © 2016-2020 587


[NOTE: In states that have not adopted the UPA 1997, the same result follows if they have both the UPA
1914 and a separate statute providing for LLPs.]

Explanation to Point-Two (35%):

The man and woman are jointly and severally liable to the customer because the obligation to the
customer was incurred prior to Garden Partnership’s qualification as an LLP.

Under the UPA (1997), the partners in a partnership are jointly and severally liable for all obligations of
the partnership “unless otherwise agreed by the claimant or provided by law.” A similar result follows
from § 15 of the UPA (1914). Because the customer has not otherwise agreed, the man and woman are
jointly and severally liable for the judgment obtained against Garden Partnership unless otherwise
provided by law.

Here, the UPA does not protect the man and woman. The UPA provides that when a partnership has
qualified as an LLP, any obligation “incurred while the partnership is a limited liability partnership,
whether arising in contract, tort, or otherwise, is solely the obligation of the partnership.” Here, however,
the obligation to the customer arose before the partnership qualified as an LLP, and the limited liability
protection does not apply. Garden Partnership became a limited liability partnership only upon the filing
of the statement of qualification with the secretary of state. “The connection between partner status and
personal liability for partnership obligations is severed only with respect to obligations incurred while the
partnership is a limited liability partnership. Partnership obligations incurred before a partnership
becomes a limited liability partnership are treated as obligations of an ordinary partnership.”
Consequently, the man and woman are jointly and severally liable for the judgment owed to the customer
that arose prior to Garden Partnership qualifying as an LLP.

Explanation to Point-Three (20%):

The investor is not personally liable to the customer because the customer’s injury occurred before the
investor became a partner.

A person who is admitted as a partner is not personally liable for any partnership obligation incurred prior
to the admission of the person to the partnership. In this case, the injury to the customer and,
consequently, the partnership obligation arose before the investor was admitted as a partner. Therefore,
the investor has no personal liability to the customer.

[NOTE: While not asked to do so by the question, an examinee may correctly point out that while the
investor has no personal liability to the customer, the $50,000 that the investor contributed to the
partnership is “at risk for the satisfaction of existing partnership debts.”]

Seperac-J19 Exam-Released MEE Essay Compilation © 2016-2020 588


#129-JUL 2011–MEE Q09: QUESTION NINE (AGENCY-PARTNERSHIP)

Portable Shredder Services (PSS) is a partnership that operates a mobile shredding business. When a
client needs paper shredded, PSS sends a truck and a crew to perform the operation.

Adam, Beth, and Chris are partners in PSS. Each of them contributed $50,000 in start-up capital, and each
actively works in the business.

The PSS partnership agreement provides in relevant part that (1) each partner is required to devote
substantially all of the partner’s working efforts to the business and (2) any partner can withdraw from the
partnership upon giving six months’ written notice. The partnership agreement contains no other relevant
provisions modifying any of the statutory default rules.

PSS has not been profitable. Adam is convinced that the assets of PSS are worth more than the value of
the business as a going concern. He believes that the only way he can receive a fair price for his share of
partnership assets is if those assets are sold. Beth and Chris, on the other hand, wish to continue operating
the business, if they can.

Adam would like to withdraw immediately from the partnership in order to force Beth and Chris to cease
the operations of PSS immediately and sell the partnership’s assets.

Adam has asked your law firm to answer the following three questions:

1. If Adam immediately withdraws from the partnership, what will be the consequences (a) to him
and (b) to the partnership? Explain.

2. If Adam gives six months’ written notice before withdrawing from the partnership, what will be
the consequences (a) to him and (b) to the partnership? Explain.

3. If the partnership’s business is wound up after Adam’s withdrawal, will he be liable for
partnership debts incurred during the winding-up process after his withdrawal? Explain.

Seperac-J19 Exam-Released MEE Essay Compilation © 2016-2020 589


#129: J11-9 MEE: ANSWER: NCBE (AGENCY-PARTNERSHIP)

POINT (1)(a) [25%] ISSUE: If Adam immediately withdraws from the partnership, what are the
consequences to him? ANSWER: Yes. Under the Uniform Partnership Act (1997), Adam can
dissociate from PSS without complying with the six months’ written notice requirement under
PSS’s partnership agreement. However, dissociation from the partnership without giving the
required six months’ notice, although effective, would be wrongful and would make Adam liable for
damages. A similar result would follow under the Uniform Partnership Act (1914).

POINT (1)(b) [20%] ISSUE: If Adam immediately withdraws from the partnership, what are the
consequences to the partnership? ANSWER: Under the Uniform Partnership Act (1997), while
Adam may immediately dissociate from the partnership, his wrongful dissociation will not
necessarily lead to a dissolution and winding up of PSS. Beth and Chris may elect not to wind up
the business but instead to purchase Adam’s interest in PSS and continue its operations. A similar
result follows under the Uniform Partnership Act (1914).

POINT (2)(a) [20%] ISSUE: If Adam gives six months’ written notice and then withdraws from the
partnership, what are the consequences to him? ANSWER: Under the Uniform Partnership Act
(1997), if Adam gives the required six months’ notice, his dissociation would not be wrongful; Adam
would be entitled to participate in the dissolution and winding up of PSS.

POINT (2)(b) [20%] ISSUE: If Adam gives six months’ written notice and then withdraws from the
partnership, what are the consequences to the partnership? ANSWER: If Adam gives the required
six months’ notice, the partnership will be dissolved and wound up. Although Adam could
participate in the winding-up process, Beth and Chris would control that process.

POINT (3) [15%] ISSUE: If the partnership’s business is wound up after Adam’s withdrawal, will
he be personally liable for any partnership debts incurred during the winding-up process after his
withdrawal? ANSWER: Yes. Partnership obligations appropriately incurred during the winding
up process are obligations of all partners including a dissociating partner.

ANSWER DISCUSSION:

Adam can withdraw from PSS, without complying with the six months’ written notice requirement in
PSS’s partnership agreement, by giving notice of dissociation. But such a notice, although effective,
would be wrongful because it would violate the notice requirement set forth in the partnership agreement.
Thus, Adam would be liable to the partnership and to Beth and Chris for damages caused by Adam’s
wrongful dissociation. Further, Adam would not be able to force the cessation of the operations of PSS
and a sale of partnership assets. Beth and Chris may vote to buy out Adam’s interest and continue
operations instead. Even if Adam’s dissociation did lead to a sale of partnership assets, Adam would not
be able to participate in the winding-up process. On the other hand, if Adam gives the six months’ notice
required under PSS’s partnership agreement, Adam’s withdrawal would dissolve the partnership and force
the winding up of PSS and the sale of the partnership assets. Adam could participate in that process but
could not dictate the terms of the sale. If the partnership business is wound up, Adam would be liable as a
partner for liabilities appropriately incurred during the winding-up period whether or not he gives notice
properly. [NOTE: Thirty-four jurisdictions have enacted the Uniform Partnership Act (1997). The
remaining states have the UPA (1914).]

Seperac-J19 Exam-Released MEE Essay Compilation © 2016-2020 590


ANSWER EXPLANATION:

Explanation to Point-One(a) (25%):

Under the Uniform Partnership Act (1997), Adam can dissociate from PSS without complying with the
six months’ written notice requirement under PSS’s partnership agreement. However, dissociation from
the partnership without giving the required six months’ notice, although effective, would be wrongful and
would make Adam liable for damages. A similar result would follow under the Uniform Partnership Act
(1914).

Under the Uniform Partnership Act (UPA) § 601 (1997), a partner can dissociate from the partnership at
any time by giving notice to the partnership. Thus, Adam can dissociate from the partnership by giving
the partnership a notice of dissociation. Furthermore, the right to dissociate can be exercised by the
dissociating partner at any time even if the exercise of the power to dissociate was wrongful. Dissociation
is wrongful if it is done in contravention of the terms of the partnership agreement.

Here, Adam’s dissociation would be wrongful because the partnership agreement requires the giving of
six months’ written notice. Because Adam’s dissociation would be wrongful, Adam would be liable to the
partnership and to Beth and Chris for any damages caused by his dissociation. Furthermore, as a
wrongfully dissociating partner, Adam could not participate in the winding up of the business.

Generally, the provisions of the UPA are default law that can be modified by the terms of a partnership
agreement. However, certain rules cannot be modified. For example, a partnership agreement cannot
modify the right to dissociate except to require that a dissociation notice be in writing. Therefore, even
though the PSS partnership agreement requires that a partner give six months’ written notice before the
partnership can be dissolved, Adam can effectively dissociate from PSS. And, because Adam’s
dissociation would be wrongful, Adam would be liable to the partnership and to Beth and Chris for any
damages caused by his dissociation.

The result is similar under the Uniform Partnership Act of 1914. While that act does not recognize a
concept of dissociation as that term is used in UPA (1997), it does recognize the right of a partner to
dissolve a partnership. A dissolution is the “change in the relation of the partners caused by any partner
ceasing to be associated in the carrying on as distinguished from the winding up of the business.”
Dissolution of a partnership is caused by “the express will of any partner when” as here, there is “no
definite term or particular undertaking,” specified in the partnership agreement.

Under the UPA (1914), if Adam notifies the partners that he no longer wishes to be associated with the
partnership, the partnership will be dissolved. As under the UPA (1997), it is irrelevant that Adam’s
notice is in contravention of the terms in the partnership agreement, such as a requirement, as here, that he
gives six months’ notice. Regardless of whether he complies with the partnership agreement, Adam’s
notice would result in the partnership’s dissolution. But, as under the UPA (1997), if Adam wrongfully
caused the partnership to be dissolved in contravention of the terms of the partnership agreement, the
other partners have a right to damages for the breach of the partnership agreement.

Explanation to Point-One(b) (20%):

Under the Uniform Partnership Act (1997), while Adam may immediately dissociate from the partnership,
his wrongful dissociation will not necessarily lead to a dissolution and winding up of PSS. Beth and Chris
may elect not to wind up the business but instead to purchase Adam’s interest in PSS and continue its
operations. A similar result follows under the Uniform Partnership Act (1914).

Seperac-J19 Exam-Released MEE Essay Compilation © 2016-2020 591


Under the UPA (1997), which recognizes the partnership as a separate legal entity from its partners, a
partner’s wrongful dissociation from the partnership does not cause the partnership to be dissolved if the
remaining partners waive the right to have the partnership terminated and its business wound up. Here,
because Adam’s dissociation would be wrongful, the partnership would be terminated and its business
wound up unless the remaining partners, Beth and Chris, waived those rights. If they did, the partnership
would have to purchase Adam’s interest, for a buyout price determined in accordance with § 701(b).

If Beth and Chris did not waive the right to wind up the business and terminate the partnership but
decided to wind up the business and terminate the partnership, Adam could not participate in that
winding-up process. The persons winding up the business, here Beth and Chris, could choose to continue
PSS’s business for a “reasonable time.”

Under the UPA (1914), a rightful or wrongful withdrawal results in the dissolution of the partnership.
However, under § 38 “the partners who have not caused the dissolution wrongfully, if they all desire to
continue the business in the same name, may do so.” A partner who has caused the dissolution wrongfully
is not granted the same right. Thus, Beth and Chris could choose to continue the business. If they did
choose to continue the business, Adam would be entitled to receive the value of his interest less the
damages he caused by wrongfully causing the partnership’s dissolution. If Beth and Chris chose not to
continue the business, Adam would have the same rights to the partnership property as Beth and Chris
would have subject to any damages he would owe them.

Explanation to Point-Two(a) (20%):

Under the Uniform Partnership Act (1997), if Adam gives the required six months’ notice, his
dissociation would not be wrongful; Adam would be entitled to participate in the dissolution and winding
up of PSS.

If Adam gives the six months’ notice required under PSS’s partnership agreement, his dissociation would
not be wrongful. Thus, he would not be liable for damages to either the partnership or Beth or Chris. His
dissociation would result in dissolution of the partnership with the requirement that the partnership’s
business be “wound up.” Because Adam’s dissociation would not be wrongful, he is entitled to participate
in the winding up of the partnership. Thus, by giving notice as required by the partnership agreement,
Adam can force the cessation of operations of PSS and the sale of its assets. Lastly, because his
dissociation would not be wrongful, Beth and Chris could not cause a buyout of his interest, nor would he
be liable to them for damages.

A similar result follows under the Uniform Partnership Act (1914).

Explanation to Point-Two(b) (20%):

If Adam gives the required six months’ notice, the partnership will be dissolved and wound up. Although
Adam could participate in the winding-up process, Beth and Chris would control that process.

If Adam gives the required six months’ notice, PSS will be dissolved and wound up. Furthermore, Adam
can participate in the winding-up process. However, Adam is still only one of three partners, all of whom
may participate in the winding up. Because no facts suggest that Adam has controlling management
authority, Adam may vote to conduct an immediate winding up and sale of assets, but could be
consistently outvoted by Beth and Chris should they decide to continue the partnership for a reasonable
time after its dissolution. Thus, Adam cannot force an immediate cessation of operations and sale of

Seperac-J19 Exam-Released MEE Essay Compilation © 2016-2020 592


partnership assets even if he properly gives notice under the PSS partnership agreement. A similar result
should follow under the Uniform Partnership Act (1914).

Explanation to Point-Three (15%):

Partnership obligations appropriately incurred during the winding up process are obligations of all
partners including a dissociating partner.

Under Section 804 of the Uniform Partnership Act (1997), the partnership is bound by a partner’s act after
dissolution if the act was appropriate to the winding up of the partnership. If the partnership is bound,
then each partner is liable for his proportionate share of the liability. Under the UPA, all partners are
liable jointly and severally for all obligations of the partnership. Thus, a dissociating partner, whether the
dissociation was rightful or wrongful, cannot avoid liability for partnership debts incurred during the
winding up process.

A similar result follows under the UPA § 35 (1914). As to contractual debts, however, the partners’
liability is only joint, not joint and severable.

Seperac-J19 Exam-Released MEE Essay Compilation © 2016-2020 593


#130-JUL 2010–MEE Q01: QUESTION ONE (AGENCY-PARTNERSHIP)

On January 2, Fran opened Petals, a floral shop, and operated it as a sole proprietorship. Petals soon ran
into financial difficulties, and Fran could not pay its bills.

On March 1, Fran asked her friends Gina and Hank for financial help.

On April 1, in response to Fran’s request, Gina delivered a check payable to “Petals” to Fran. In exchange
for this contribution, Fran agreed to pay Gina 25% of the monthly net profits of Petals for as long as
Petals remained in business. Gina also agreed that, if Petals suffered losses, she would share those losses
with Fran.

Gina began working at the shop with Fran and helped Fran with business planning for Petals.

On April 2, also in response to Fran’s request, Hank delivered a check payable to “Petals” to Fran and
noted on the memo line of the check “loan to Petals.” Hank agreed to accept 25% of the monthly net
profits of Petals until his loan (plus interest) was repaid in full.

Fran used the proceeds of the checks from Gina and Hank to purchase equipment, supplies, and a delivery
truck in the name of Petals.

Beginning in April, on the last day of each month, Fran distributed to both Gina and Hank 25% of Petals’
monthly net profits.

On September 1, Gina wrote a letter to her son Ivan stating that she was assigning to Ivan, as a gift, all of
her interest in Petals effective immediately. Gina gave a copy of that letter to Fran. Fran told Gina, “I
don’t want anything to do with Ivan.” Gina continued to be active in the business operations of Petals.

On September 30, Fran distributed the monthly net profits of Petals to Gina and Hank, but distributed
nothing to Ivan.

On October 10, Ivan demanded that Fran distribute Gina’s share of Petals’ net profits to him and that she
also allow him to inspect the books and records of Petals.

On October 15, Gina learned that Fran was using Petals’ delivery truck on Sundays to transport her
children to their soccer games. Gina demanded that Fran stop doing so, but Fran refused, noting that the
truck was not being used for Petals’ business on Sundays.

1. What is the legal relationship among Fran, Gina, and Hank? Explain.

2. Is Ivan entitled to Gina’s share of the monthly net profits of Petals? Explain.

3. Is Ivan entitled to inspect the books and records of Petals? Explain.

4. Is Fran entitled to use the delivery truck on Sundays to take her children to their soccer games?
Explain.

Seperac-J19 Exam-Released MEE Essay Compilation © 2016-2020 594


#130: J10-1 MEE: ANSWER: NCBE (AGENCY-PARTNERSHIP)

POINT (1) [35%] ISSUE: What is the legal relationship among Fran, Gina, and Hank? ANSWER:
Fran and Gina are partners in Petals. Hank is a creditor of Petals.

POINT (2) [25%] ISSUE: Is Ivan entitled to Gina’s share of the monthly net profits of Petals?
ANSWER: Yes. As the transferee of a legally transferable interest in the partnership, Ivan is
legally entitled to receive distributions that Gina would otherwise have received.

POINT (3) [25%] ISSUE: Is Ivan entitled to inspect the books and records of Petals? ANSWER:
No. Ivan is not entitled to inspect the books and records of Petals because he is not a partner in
Petals.

POINT (4) [15%] ISSUE: Is Fran entitled to use the delivery truck on Sundays to take her children
to their soccer games? ANSWER: No. The delivery truck is partnership property, and Fran is not
entitled to use it on Sundays to take her children to their soccer games.

ANSWER DISCUSSION:

Fran and Gina are general partners in the Petals partnership, but Hank is only a creditor of Petals. As a
transferee of Gina’s right to receive net profits in Petals, Ivan is legally entitled to Gina’s share of those
profits. However, that assignment does not make Ivan a partner. Therefore, Ivan does not have a right to
inspect the books and records of Petals. Fran is not entitled to use the delivery truck on Sundays to take
her children to their soccer games.

ANSWER EXPLANATION:

Explanation to Point-One (35%):

Fran and Gina are partners in Petals. Hank is a creditor of Petals.

A partnership is defined as “the association of two or more persons to carry on as co-owners a business
for profit whether or not the persons intend to form a partnership.” Here, a partnership was formed when
Gina joined Fran in running Petals. Because the partnership was formed without the formalities required
of other types of partnerships, it is a general partnership.

Fran and Gina each receive a share of the net profits of Petals, and each is therefore presumed to be a
partner in the business. This presumption is supported by the fact that Gina is to receive her share of the
net profits “for as long as Petals remains in business,” and that Fran and Gina each work in the business
and participate in the business planning for Petals. Further, Gina agreed to share in any losses that Petals
might suffer. Therefore, Fran and Gina are partners in the Petals partnership.

Hank also receives a share of the net profits of Petals and, therefore, also might be presumed to be a
partner in the partnership. However, this presumption does not apply where those profits are “received in
payment of a debt by installment or otherwise.” The statement on the memo line of Hank’s check is
evidence that Hank loaned money to the partnership. Hank is entitled to receive a share of the net profits
of Petals only until his loan (plus interest) to Petals is paid in full, and therefore he is not presumed to be a
partner in the business. In addition, Hank did not agree to share in any losses that Petals might suffer.

Seperac-J19 Exam-Released MEE Essay Compilation © 2016-2020 595


Further, Hank does not play any role in the day-to-day operations of Petals. Hank is a creditor of Petals,
but is not a partner in the partnership.

Explanation to Point-Two (25%):

As the transferee of a legally transferable interest in the partnership, Ivan is legally entitled to receive
distributions that Gina would otherwise have received.

As a partner in the Petals partnership, Gina has a transferable interest in the profits and losses of the
partnership and in her right to receive distributions. The transfer of that interest is permissible and creates
in the transferee a right to receive distributions to which the transferor would otherwise be entitled. Here,
Gina transferred her right to receive a share of the net profits of Petals to Ivan when she assigned all of
her interest in Petals to him. Therefore, Ivan is legally entitled to receive Gina’s share of the net profits of
Petals.

[NOTE: An applicant might add that a partnership need not give effect to a transferee’s rights until it has
notice of the transfer. Here, the partnership had such notice because Gina gave Fran a copy of the letter
effecting the transfer. An applicant might also note that the transfer of a partner’s right to profits and
losses and the right to receive distributions does not effect a termination or dissolution of the
partnership.]

Explanation to Point-Three (25%):

Ivan is not entitled to inspect the books and records of Petals because he is not a partner in Petals.

As a partner in the Petals partnership, Gina has a transferable interest in the partnership. That interest,
however, is limited. For example, a transfer of the partnership interest does not make the transferee a
partner unless the other partner or partners consent to making the transferee a partner. On our facts, such
consent was not given because Fran said, “I don’t want anything to do with Ivan.” Therefore, Ivan did not
become a partner and is not entitled to access information concerning partnership transactions nor to
inspect or copy the partnership books or records.

Explanation to Point-Four (15%):

The delivery truck is partnership property, and Fran is not entitled to use it on Sundays to take her
children to their soccer games.

Property is partnership property if it is acquired in the name of the partnership. Partnership property is
property of the partnership and not of the partners individually. Here, the delivery truck was purchased in
the partnership’s name and, therefore, is partnership property and not Fran’s individual property. The
facts do not suggest that the partners agreed that Fran could use the delivery truck for her own purposes
on Sundays, and, absent such an agreement, Fran is not entitled to use the truck to take her children to
their soccer games even if the partnership is not using the truck on that day.

[NOTE: Under § 27 of the UPA (1914) and § 503 of the UPA (1997), the transfer of all or any part of a
partner’s interest is not a dissolution of the partnership. Thus, Gina’s gift to her son Ivan of her interest
in the partnership does not dissolve the partnership.]

Seperac-J19 Exam-Released MEE Essay Compilation © 2016-2020 596


#131-JUL 2009–MEE Q08: QUESTION EIGHT (AGENCY-PARTNERSHIP)

Six months ago, Andy, Ben, and Carol executed a document entitled “Metropolitan Limited Partnership
Agreement” (the Agreement). Under the terms of the Agreement, Andy, Ben, and Carol were to be
“limited partners” in the “Metropolitan Limited Partnership” (MLP). The Agreement provided that each
would contribute $500,000 to the venture, which they each did.

The Agreement also provided that Warren would be the “general partner” of MLP, but Warren never
signed the Agreement.

The Agreement further provided that the venture would buy undeveloped land and hold it for
development. The land was purchased, and a deed for the land naming “Metropolitan Limited
Partnership” as grantee was executed, delivered, and recorded. In the MLP name, Andy, Ben, and Carol
also hired a marketing company, Marketing, to develop a campaign to resell the land as “Metropolitan
Estates.”

Two months after the Agreement was executed by Andy, Ben, and Carol, Marketing sued MLP, and
Andy, Ben, and Carol individually, for nonpayment of amounts due to Marketing for services it had
provided to MLP. MLP is unable to pay Marketing because the land, its only asset, has substantially
depreciated due to an economic downturn.

Immediately after Marketing filed suit, Andy, Ben, and Carol filed a Certificate of Limited Partnership in
the name of Metropolitan Limited Partnership in the appropriate state office.

Two weeks after the Certificate of Limited Partnership was filed, Zack went onto the land owned in the
name of Metropolitan Limited Partnership and fell down an uncovered well. Zack died as a result of
injuries suffered in the fall. Zack’s estate has filed a wrongful death action against Andy, Ben, and Carol
individually.

1. What type of entity is the Metropolitan Limited Partnership? Explain.

2. Can Marketing recover from Andy, Ben, and Carol personally for the amounts it is owed by
Metropolitan Limited Partnership? What steps must it follow if it tries to do so? Explain.

3. If Zack’s estate is entitled to damages, can it recover from Andy, Ben, and Carol personally for
the wrongful death claim? What steps must it follow if it tries to do so? Explain.

Seperac-J19 Exam-Released MEE Essay Compilation © 2016-2020 597


#131: J09-8 MEE: ANSWER: NCBE (AGENCY-PARTNERSHIP)

POINT (1) [42%] ISSUE: What type of entity is the “Metropolitan Limited Partnership”?
ANSWER: Metropolitan Limited Partnership is not a limited partnership, but is a general
partnership.

POINT (2) [42%] ISSUE: Can Marketing recover from Andy, Ben, and Carol in their individual
capacities for the amounts it is owed by MLP and what steps must it follow if it tries to do so?
ANSWER: Because MLP is a general partnership and Andy, Ben, and Carol are general partners
in that partnership, as such, each is personally liable for partnership debts, including the amounts
owed to Marketing. However, to collect from Andy, Ben, and Carol personally, Marketing must
first obtain a judgment against Andy, Ben, and Carol individually and against MLP and levy
execution against MLP’s assets.

POINT (3) [16%] ISSUE: Can Zack’s estate recover from Andy, Ben, and Carol in their individual
capacities for the wrongful death claim? ANSWER: Yes. Andy, Ben, and Carol are each personally
liable for the wrongful death claim even though that claim arose after they filed the Certificate of
Limited Partnership, because the limited partnership did not have a general partner.

ANSWER DISCUSSION:

Despite being labeled a limited partnership, the Metropolitan Limited Partnership (MLP) is not a limited
partnership because it lacks a general partner. Even though a Certificate of Limited Partnership was
ultimately filed, that filing does not create a limited partnership when no general partner has signed the
partnership agreement. By their actions in buying the land and arranging for the marketing of the
venture’s property, Andy, Ben, and Carol acted as co-owners of a business for profit and thereby formed a
general partnership. As partners in a general partnership, Andy, Ben, and Carol are each personally liable
for partnership debts. Marketing can recover from Andy, Ben, and Carol in their individual capacities for
the amounts it is owed by MLP if Marketing first obtains a judgment against each of Andy, Ben, and
Carol and against MLP and levies execution on MLP’s assets. Zack’s estate can recover for the wrongful
death claim from Andy, Ben, and Carol in their individual capacities if it follows the same procedures.
The fact that the Certificate of Limited Partnership was filed before Zack’s claim arose does not change
this result because that filing did not create a limited partnership.

ANSWER EXPLANATION:

Explanation to Point-One (35-45%):

Metropolitan Limited Partnership is not a limited partnership, but is a general partnership.

Despite being labeled a limited partnership, MLP is not a limited partnership. Although Andy, Ben, and
Carol may have intended to form a limited partnership, they did not succeed in doing so. To create a valid
limited partnership, statutory requirements must be met. A limited partnership must include a general
partner who has signed the initial Certificate of Limited Partnership filed with the Secretary of State.
Here, Warren was designated as the general partner, but never signed the agreement. Thus, MLP never
had a general partner and cannot qualify as a limited partnership.

Seperac-J19 Exam-Released MEE Essay Compilation © 2016-2020 598


Further, to create a limited partnership, a certificate of limited partnership must be filed with the state.
Here, such a certificate was not initially filed. Even though Andy, Ben, and Carol did ultimately file the
Certificate of Limited Partnership, their failure to obtain the signature of a general partner prevents the
formation of a limited partnership.

Instead, the venture is a general partnership. Andy, Ben, and Carol were acting as co-owners of a business
for profit. As such, they formed a partnership whether or not they subjectively intended to. the association
of two or more persons to carry on as co-owners a business for profit forms a partnership, whether or not
the persons intend to form a partnership.”). As is the case here, individuals can inadvertently form a
general partnership notwithstanding their expressed intention to do something else, in this case to form a
limited partnership. Indeed, they may inadvertently create a partnership despite their expressed subjective
intention not to do so.

Had Andy, Ben, and Carol done nothing more than purchase property, they might have avoided a
partnership designation under the Uniform Partnership Act which states that neither “common property”
nor “part ownership” alone is enough to establish a partnership. However, in addition to owning the
property, Andy, Ben, and Carol hired a marketing company to develop a campaign to resell the land as
“Metropolitan Estates.” This exceeds co-ownership and thus Andy, Ben, and Carol are general partners
and MLP is a general partnership.

Explanation to Point-Two (35-45%):

Because MLP is a general partnership and Andy, Ben, and Carol are general partners in that partnership,
as such, each is personally liable for partnership debts, including the amounts owed to Marketing.
However, to collect from Andy, Ben, and Carol personally, Marketing must first obtain a judgment
against Andy, Ben, and Carol individually and against MLP and levy execution against MLP’s assets.

As partners in the general partnership that owns the property, Andy, Ben, and Carol “are liable jointly and
severally for all obligations of the partnership unless otherwise agreed by the claimant or provided by
law.” There being no agreement or statutory provision to the contrary, Andy, Ben, and Carol are jointly
and severally liable for Marketing’s claim.

However, to collect from Andy, Ben, and Carol personally, Marketing must first obtain a judgment
against Andy, Ben, and Carol individually and against MLP and levy execution against MLP’s assets.
Ordinarily, a claimant may not take action against a partner’s separate assets unless the partnership assets
fail to satisfy the claimant’s judgment. Moreover, under the Uniform Partnership Act, a judgment against
a partnership is not, by itself, a judgment against a partner. Consequently, a judgment against a
partnership may not be satisfied out of the partner’s separate assets unless there is also a judgment against
the partner. A judgment against a partner personally may be sought in the same action as a judgment
against the partnership.

Here, if these procedures are followed and MLP’s assets fail to satisfy Marketing’s claim, Marketing may
recover from Andy, Ben, and Carol personally.

Explanation to Point-Three (10-20%):

Andy, Ben, and Carol are each personally liable for the wrongful death claim even though that claim
arose after they filed the Certificate of Limited Partnership, because the limited partnership did not have a
general partner.

Seperac-J19 Exam-Released MEE Essay Compilation © 2016-2020 599


As general partners, Andy, Ben, and Carol are jointly and severally liable for the wrongful death claim.
Although the wrongful death claim arose after the Certificate of Limited Partnership was filed in the name
of Metropolitan, that filing does not protect Andy, Ben, and Carol because it failed to create a limited
partnership due to the lack of a general partner. Therefore, the act of filing is irrelevant, and Andy, Ben,
and Carol are jointly and severally liable for the wrongful death claim. To recover from them, Zack’s
estate must follow appropriate procedures.

Seperac-J19 Exam-Released MEE Essay Compilation © 2016-2020 600


#132-FEB 2009–MEE Q01: QUESTION ONE (AGENCY-PARTNERSHIP)

Hanson’s Fruitcakes (Hanson’s) is the largest producer of fruitcakes in the world. The company was
founded 150 years ago and uses an original secret recipe closely guarded by the company.

Because of a significant drop in the demand for fruitcakes, Hanson’s decided to expand its product line to
include other baked goods that will not compete with its fruitcakes.

Because Hanson’s has limited experience producing baked goods other than fruitcakes, it decided to hire
Taster as a consultant. The contract between Hanson’s and Taster provided as follows:

(1) Taster would travel for six months tasting baked goods at “high-end” bakeries. Taster was expressly
authorized, on behalf of Hanson’s, to buy the recipes of any baked goods he thought Hanson’s could
produce successfully. Taster was expressly prohibited from committing Hanson’s to pay more than
$5,000 cash for any one recipe, because market research confirmed that baked-goods recipes typically
sold for prices between $3,000 and $6,000.

(2) Hanson’s would disclose to Taster the secret recipe for Hanson’s fruitcake so that Taster would not
inadvertently agree to buy any baked-goods recipes that were substantially similar to the fruitcake. Taster
is required to keep Hanson’s recipe secret from everyone.

Hanson’s president announced the company’s plans to expand its product line at the annual baking
industry trade show attended by everyone in the baking industry. At the trade show, Hanson’s president
stated: “Hanson’s is breaking new ground in the baking industry by hiring a consultant, Taster. Taster will
be entering into contracts to buy recipes from other bakers on Hanson’s behalf.” Hanson’s president did
not disclose the precise terms of the Hanson’s-Taster contract.

Purporting to act on behalf of Hanson’s, over the next four months Taster entered into contracts to buy the
following recipes:

(1) Boysenberry-granola muffins for $4,000 from Monumental Muffins,

(2) Almond-pecan tarts for $6,000 from Bakers Bonanza, and

(3) Chocolate truffle cake from Parisian Delights in exchange for a copy of Hanson’s secret fruitcake
recipe.

Purporting to act on behalf of Hanson’s, Taster also entered into a contract with Ironcast Enterprises to
buy a sophisticated baking oven for $5,000.

Is Hanson’s legally bound to any of the four contracts made by Taster? Explain.

Seperac-J19 Exam-Released MEE Essay Compilation © 2016-2020 601


#132: F09-1 MEE: ANSWER: NCBE (AGENCY-PARTNERSHIP)

POINT (1) [17%] ISSUE: Did Taster have actual authority to buy Monumental Muffins’ muffin
recipe for $4,000? ANSWER: Yes. Taster had actual authority to enter into the Monumental
Muffins muffin recipe contract for $4,000 on Hanson’s behalf. Therefore, Hanson’s is legally bound
to that contract.

POINT (2) [39%] ISSUE: Did Taster have apparent authority to buy Bakers Bonanza’s tart recipe
for $6,000? ANSWER: Yes. Taster had apparent authority to enter into the contract with Bakers
Bonanza to buy the almond-pecan tart recipe for $6,000 on Hanson’s behalf. Therefore, Hanson’s is
legally bound to that contract.

POINT (3) [28%] ISSUE: Did Taster have any authority to buy Parisian Delights’ cake recipe in
exchange for a copy of Hanson’s fruitcake recipe? ANSWER: No. Taster did not have authority to
buy Parisian Delights’ chocolate truffle cake recipe in exchange for Hanson’s fruitcake recipe.
Therefore, Hanson’s is not legally bound to that contract.

POINT (4) [17%] ISSUE: Did Taster have any authority to buy the baking oven from Ironcast
Enterprises for $5,000? ANSWER: No. Hanson’s is not liable to Ironcast Enterprises on the
contract for the oven as Taster had no authority to enter into that contract on Hanson’s behalf.

ANSWER DISCUSSION:

Taster purported to act as Hanson’s agent. An agent acts with actual or apparent authority. Taster had
actual authority to buy recipes for $5,000 cash or less. Hanson’s is clearly bound to the contract with
Monumental Muffins because Taster had actual authority to enter into that contract. Taster had apparent
authority to enter into the contract with Bakers Bonanza and Hanson’s is legally bound to that contract.
Taster had no authority to reveal Hanson’s secret recipe, and it was unreasonable for Parisian Delights to
believe that Taster had such authority, so Hanson’s is not bound to that contract. Taster also had no
authority to enter into the contract with Ironcast, and Hanson’s is not legally bound to that contract.

ANSWER EXPLANATION:

Explanation to Point-One (10-20%):

Taster had actual authority to enter into the Monumental Muffins muffin recipe contract for $4,000 on
Hanson’s behalf. Therefore, Hanson’s is legally bound to that contract.

When Hanson’s hired Taster to act on Hanson’s behalf, an agency relationship was created. Under the
Third Restatement of Agency, agency is the fiduciary relationship that arises when one person (a
“principal”) manifests assent to another person (an “agent”) that the agent shall act on the principal’s
behalf. As Hanson’s agent, Taster had actual authority to “take action designated or implied in Hanson’s
manifestations to [Taster] and acts necessary or incidental to achieving Hanson’s objectives.”

The contract between Hanson’s and Taster empowered Taster to enter into contracts on Hanson’s behalf
to buy baked-goods recipes for a price not to exceed $5,000. Thus, Taster had actual authority to enter
into the contract to buy Monumental Muffins’ muffin recipe for $4,000, and Hanson’s is legally bound to
that contract.

Seperac-J19 Exam-Released MEE Essay Compilation © 2016-2020 602


Explanation to Point-Two (30-40%):

Taster had apparent authority to enter into the contract with Bakers Bonanza to buy the almond-pecan tart
recipe for $6,000 on Hanson’s behalf. Therefore, Hanson’s is legally bound to that contract.

An agent has apparent authority to act on behalf of a principal when “a third party reasonably believes the
actor has authority to act on behalf of the principal and that belief is traceable to the principal’s
manifestations.” Apparent authority can co-exist with actual authority; it can also exist in the absence of
any actual authority.

Here, Taster did not have actual authority to enter into contracts exceeding $5,000. Thus, Taster did not
have actual authority to contract with Bakers Bonanza for $6,000. However, Hanson’s president stated at
the annual baking industry trade show that Taster would be buying recipes on Hanson’s behalf. These
manifestations by Hanson’s would reasonably cause Bakers Bonanza to believe that Taster was acting
with authority when contracting to buy recipes. The manifestations were made by Hanson’s president, a
person entitled to speak on behalf of Hanson’s. The amount involved ($6,000) was within the price range
typically paid for baked-goods recipes. Therefore, Taster had apparent authority to enter into the contract
to buy Bakers Bonanza’s tart recipe and Hanson’s is legally bound to that contract.

Explanation to Point-Three (20-30%):

Taster did not have authority to buy Parisian Delights’ chocolate truffle cake recipe in exchange for
Hanson’s fruitcake recipe. Therefore, Hanson’s is not legally bound to that contract.

Taster did not have actual authority to buy Parisian Delights’ cake recipe in exchange for Hanson’s
fruitcake recipe – in fact, Taster’s contract expressly forbids such an action. Nor did Taster have apparent
authority to enter into the contract with Parisian Delights. Here, the facts state that baked-goods recipes
typically sold for $3,000 to $6,000. There is no suggestion that the use of non-cash consideration is
typical. Further, given Hanson’s long history in the baking industry and its close guarding of the fruitcake
recipe, Parisian Delights may well have known that Hanson’s fruitcake recipe was secret and that its
disclosure was not authorized. When facts suggest it may be unreasonable for a third party to believe that
a purported agent has authority, the third party has the duty to make further inquiry. Had Parisian Delights
asked Hanson’s if Taster had authority to obtain recipes for non-cash consideration, Taster’s lack of any
authority would have been readily determinable.

[NOTE: Applicants should not receive any credit for discussing inherent authority as that doctrine is not
applicable generally nor on the facts.]

Explanation to Point-Four (10-20%):

Hanson’s is not liable to Ironcast Enterprises on the contract for the oven as Taster had no authority to
enter into that contract on Hanson’s behalf.

Taster had no actual or apparent authority to enter into the contract with Ironcast Enterprises to buy the
baking oven on Hanson’s behalf. Actual authority does not exist because the contract limited Taster’s
authority to acquiring recipes – contracting to buy an oven was not within the express grant of authority
nor was it an act “necessary or incidental to achieving the principal’s objectives.” Apparent authority does
not exist because no acts of Hanson’s reasonably interpreted could have caused Ironcast to believe that
Taster had authority to buy baking equipment. Therefore, Hanson’s is not legally bound on the contract to
buy the oven from Ironcast for $5,000.

Seperac-J19 Exam-Released MEE Essay Compilation © 2016-2020 603


#133-JUL 2008–MEE Q03: QUESTION THREE (AGENCY-PARTNERSHIP)

Fifteen years ago, Amy, Beck, and Curt formed a partnership, “Amy, Beck, and Curt Co.,” to engage in a
retail shoe store business. The partnership is for a 25-year term. All real estate owned by the partnership is
titled to “Amy, Beck, and Curt Co., a partnership.”

While the partnership has done well, the three partners have not managed their respective personal
finances successfully. In order to deal with personal financial problems, Amy borrowed $25,000 from
Green, Beck borrowed $50,000 from Red, and Curt borrowed $75,000 from White. The three partners
each have defaulted on their respective loans.

Green, Red, and White are pursuing various avenues to recover what is owed to each of them.

Green has obtained a judgment against Amy, who is judgment-proof, and is considering attaching the
partnership real estate.

Red has taken no legal action against Beck but wants to collect the amount of his loan to Beck from
Beck’s interest in the partnership.

Curt has assigned all of his interest in the partnership to White. After the assignment, White asked to
inspect the partnership books and records and demanded the right to participate in the management and
affairs of the partnership. White is also considering the possibility of collecting Curt’s debt by forcing a
dissolution and winding up of the partnership.

1. Can Green, as a judgment creditor of Amy, attach and execute upon the partnership real estate?
Explain.

2. What steps should Red take to collect the amount of his loan to Beck from Beck’s interest in the
partnership? Explain.

3. Does White have a right to inspect partnership books and records and to participate in the
management of the partnership? Explain.

4. Can White force a dissolution and winding up of the partnership? Explain.

Seperac-J19 Exam-Released MEE Essay Compilation © 2016-2020 604


#133: J08-3 MEE: ANSWER: NCBE (AGENCY-PARTNERSHIP)

POINT (1) [25%] ISSUE: Can Green, as a judgment creditor of Amy, attach and execute upon the
partnership real estate? ANSWER: No. Green, a judgment creditor of Amy, an individual partner,
cannot attach and execute upon the partnership real estate.

POINT (2) [25%] ISSUE: What steps should Red take to collect the amount of his loan to Beck
from Beck’s interest in the partnership? ANSWER: Red should reduce his claim against Beck to a
judgment and should then seek a charging order against Beck’s financial interest in the
partnership.

POINT (3) [25%] ISSUE: Does White have a right to inspect partnership books and records and to
participate in the management of the partnership? ANSWER: No. White, an assignee of Curt’s
partnership interest, has received only Curt’s financial interest in the partnership; White has no
right to inspect partnership books and records or to participate in the management of the
partnership.

POINT (4) [25%] ISSUE: Can White force a dissolution and winding up of the partnership?
ANSWER: No. White may not force a dissolution and winding up of the partnership because it is a
partnership for a term and the term has not yet been completed.

ANSWER DISCUSSION:

Green cannot attach and execute upon the partnership property because that property is owned by the
partnership and is not subject to attachment and execution by a creditor of an individual partner. To
collect the amount of his loan to Beck, Red must first reduce his claim to a judgment and seek a charging
order against the partner’s financial interest in the partnership. White does not have a right to inspect
partnership books and records because non-financial partnership rights, such as the right to inspect
partnership books and records, to demand information from other partners, or to participate in the
management or affairs of the partnership, cannot be exercised by a creditor of an individual partner, nor
can they be assigned by a partner. White cannot force a dissolution and winding up of the partnership. A
transferee of a partner’s financial interest in the partnership may only seek dissolution and winding up of
the partnership if the partnership is at will or, if the partnership is for a term or a particular undertaking,
after the term or undertaking has been completed. Here, the partnership is for a specific term, and that
term has not expired.

ANSWER EXPLANATION:

Explanation to Point-One (20-30%):

Green, a judgment creditor of Amy, an individual partner, cannot attach and execute upon the partnership
real estate.

A judgment creditor of an individual partner may not attach and execute upon partnership real estate to
satisfy his claim against that partner.

Under the Uniform Partnership Act (UPA), partners are tenants in partnership in specific items of
partnership property. The incidents of this tenancy include an equal right to use the partnership property

Seperac-J19 Exam-Released MEE Essay Compilation © 2016-2020 605


for partnership purposes. However, a partner’s interest in specific items of partnership property is not
subject to attachment or execution except upon a claim against the partnership.

Under the Uniform Partnership Act (1997), the partnership is a legal entity distinct from its partners.
Partnership property is owned by the partnership and a partner is not a co-owner of such property.
Therefore, partnership property is not subject to execution by an individual partner’s creditors.

Thus, Amy has no interest in partnership real estate that could be subject to attachment and execution by a
judgment creditor such as Green.

Explanation to Point-Two (20-30%):

Red should reduce his claim against Beck to a judgment and should then seek a charging order against
Beck’s financial interest in the partnership.

A creditor of an individual partner who wants to pursue a partner’s financial interest in the partnership
must first reduce his claim to a judgment. A creditor of an individual partner who has reduced his claim to
a judgment may seek a charging order against that partner’s financial interest in the partnership. The
partner’s financial interest (called the “transferable interest in the partnership” under the 1997 Act) is the
partner’s share of profits and losses and the right to receive distributions. This financial interest is the only
interest of a partner in the partnership that can be pursued by a creditor of an individual partner.

Thus, Red should reduce his claim against Beck to a judgment and should then seek a charging order
against Beck’s financial interest in the partnership.

Another way Red might collect from Beck’s interest in the partnership is to convince Beck to assign him
Beck’s interest in the partnership in which case Red would succeed to Beck’s financial interest in the
partnership.

Explanation to Point-Three (20-30%):

White, an assignee of Curt’s partnership interest, has received only Curt’s financial interest in the
partnership; White has no right to inspect partnership books and records or to participate in the
management of the partnership.

Generally a person may become a partner only with the consent of all the partners. Thus, a partner does
not have the unilateral power to make someone a partner by transferring his or her partnership interest to
them.

The assignee of a partnership interest receives only the financial interest of the partner. The 1997 Act
refers to this financial interest as the partner’s transferable interest in the partnership. “Section 502
continues the UPA Section 26 concept that a partner’s only transferable interest in the partnership is the
partner’s share of profits and losses and right to receive distributions, that is, the partner’s financial
rights.” An assignee does not acquire the right to inspect partnership books and records, or to participate
(or interfere) in the management or conduct of the business.

In sum, because White is a mere assignee of Curt’s partnership interest, White has received only Curt’s
financial interest in the partnership; White is not a partner and has no right to inspect partnership books
and records or to participate in the management of the partnership.

Seperac-J19 Exam-Released MEE Essay Compilation © 2016-2020 606


Explanation to Point-Four (20-30%):

White may not force a dissolution and winding up of the partnership because it is a partnership for a term
and the term has not yet been completed.

White is a transferee of Curt’s financial interest in the partnership. A transferee of a partner’s financial
interest in a partnership may seek dissolution of the partnership if the partnership is a partnership at will
or, if a partnership is for a term or a particular undertaking, the term or the undertaking has been
completed. Under the 1997 Act, the court will dissolve a partnership for a term and order a winding up at
the request of a transferee of a partner’s financial interest after the expiration of the term and only if it is
equitable to do so.

In this case the partnership is a partnership for a term of 25 years, not a partnership at will, and the term
has not yet expired. Therefore, a transferee of a partner’s financial interest may not seek dissolution and
winding up of the partnership. Consequently, White cannot force a dissolution and winding up of the
partnership.

Seperac-J19 Exam-Released MEE Essay Compilation © 2016-2020 607


#134-JUL 2007–MEE Q04: QUESTION FOUR (AGENCY-PARTNERSHIP)

Talker is a persuasive salesperson, Fixer is a talented carpenter, and Manager is an experienced small
business manager. The three are friends and often discussed going into business together. Two months
ago they orally agreed to form a cabinet restoration business and to share profits equally.

The three friends began doing business under the name TFM Restored Cabinets (TFM). Manager
collected accounts receivable, paid all bills, and distributed the profits equally among the three. Fixer
restored the cabinets, and Talker marketed them. The three friends operated independently in performing
their respective duties, but they met periodically to discuss and decide other business matters.

Everything went smoothly until Fixer announced that he intended to hire Crafty to help with the
restoration work because there was more work than one carpenter could handle. Talker and Manager
opposed hiring Crafty. They believed that most of TFM’s customers were attracted by Fixer’s reputation
and that the business would be harmed if Crafty performed restoration work. Despite their objections,
Fixer hired Crafty.

One month later, Crafty quit after Manager refused to pay for the work that Crafty had performed.

1. What is the legal relationship of Talker, Fixer, and Manager? Explain.

2. Did Fixer have authority to hire Crafty on behalf of TFM? Explain.

3. Under what circumstances, if any, could Crafty collect from Talker the wages TFM owes
Crafty? Explain.

Seperac-J19 Exam-Released MEE Essay Compilation © 2016-2020 608


#134: J07-4 MEE: ANSWER: NCBE (AGENCY-PARTNERSHIP)

POINT (1) [31%] ISSUE: What is the legal relationship of Talker, Fixer, and Manager? ANSWER:
Talker, Fixer, and Manager are general partners in TFM.

POINT (2)(a) [13%] ISSUE: Did Fixer have actual authority to hire Crafty on behalf of TFM?
ANSWER: No. Fixer had no actual authority to bind the partnership to a contract of employment
with Crafty.

POINT (2)(b) [31%] ISSUE: Did Fixer have apparent authority to hire Crafty on behalf of TFM?
ANSWER: Yes. Fixer had apparent authority as a partner to bind the partnership when he hired
Crafty.

POINT (3) [25%] ISSUE: Can Crafty hold Talker personally liable for unpaid wages? ANSWER:
Yes. Talker is personally liable for Crafty’s unpaid wages. However, to collect against Talker,
Crafty must first obtain judgments against Talker and the partnership and exhaust the assets of the
partnership.

ANSWER DISCUSSION:

Talker, Fixer, and Manager are doing business as a general partnership. As partners in a general
partnership, they must ordinarily act by majority rule; thus Fixer had no actual authority to hire Crafty.
However, Fixer had apparent authority to hire Crafty on behalf of the partnership. Therefore, TFM is
liable for Crafty’s unpaid wages. As a general partner, Talker is personally liable for the partnership’s
debts. However, Crafty can collect those wages from Talker only if Crafty obtains a judgment against
Talker and first exhausts the partnership’s assets.

ANSWER EXPLANATION:

Explanation to Point-One (20-30%):

Talker, Fixer, and Manager are general partners in TFM.

A partnership is the association of two or more persons to carry on as co-owners a business for profit
whether or not the persons intend to form a partnership. “A business is a series of acts directed toward an
end.” As co-owners of a business, each partner “has the power of ultimate control.”

In order to create a partnership, the parties must intend to take actions that create a partnership, but it is
not necessary that they realize they are forming a partnership or describe their business as a partnership.
“Where intent cannot be directly ascertained, it must be established from all of the facts, circumstances,
actions, and conduct of the parties.” A written agreement is generally not required.

A person who receives a share of the profits of the business is presumed to be a partner in the business.
Under the UPA, receipt of a share of profits of a business is prima facie evidence that one is a partner in
the business. The presumption does not apply in specified situations, such as where the profits are
received in payment of a debt, as interest on a loan, as wages, as rent, or for the sale of the goodwill of a
business.

Seperac-J19 Exam-Released MEE Essay Compilation © 2016-2020 609


Here, there is a for-profit business and the three friends share equally in the profits of the business. There
are no facts suggesting that the profits are received in payment of a debt, as interest on a loan, as wages,
as rent, or for the sale of the goodwill of a business. Furthermore, the conduct of Talker, Fixer, and
Manager supports the partnership presumption: each of them participates in the business on a regular
basis; they meet regularly to discuss the business; and the name of the business includes initials from each
of their names.

In sum, although there is no written partnership agreement, the conduct of Talker, Fixer, and Manager
establishes that they have associated as co-owners of a business for profit. This association constitutes a
partnership relationship. Because they have no agreement to the contrary, they have created a general
partnership.

Explanation to Point-Two(a): (05-15%):

Fixer had no actual authority to bind the partnership to a contract of employment with Crafty.

Absent an agreement to the contrary, “a difference arising as to a matter in the ordinary course of business
of a partnership may be decided by a majority of the partners.” An act outside of the ordinary course of
business requires the consent of all partners.

Hiring an employee to perform services in the partnership business is clearly a matter within the ordinary
course of the business of the TFM partnership. No major change in or deviation from the cabinet
restoration business is involved. Fixer was merely hiring additional help to deal with an overflow of
business. The general rule is that each partner has authority to bind the firm by employment of persons
whose services are reasonably necessary for carrying on business. A partner can bind the firm by
engaging employees in the ordinary course of business.

The facts of this problem make clear that a majority of the partners (two of the three) opposed the hiring
of Crafty. Consequently, Fixer did not have actual authority to hire Crafty.

Explanation to Point-Two(b) (20-30%):

Fixer had apparent authority as a partner to bind the partnership when he hired Crafty.

Each partner has authority to bind the partnership by any act “for apparently carrying on in the ordinary
course the partnership business or business of the kind carried on by the partnership unless the partner had
no authority to act for the partnership in the particular matter and the person with whom the partner was
dealing knew or had received a notification that the partner lacked authority.” The effect of UPA § 301
(1997) is to “characterize a partner as a general managerial agent having both actual and apparent
authority co-extensive with the firm’s ordinary business, at least in the absence of a contrary partnership
agreement.”

Under UPA (1997), a two-part analysis is required. First, it must be determined whether the particular act
was “for apparently carrying on in the ordinary course the partnership business or business of the kind
carried on by the partnership.” If it was, then the partnership is liable unless it is proven that the person
dealing with the partner had actual knowledge or notification of the partner’s lack of authority.

Here, hiring Crafty to help in cabinet restoration was in the ordinary course of a cabinet restoration
business.

Seperac-J19 Exam-Released MEE Essay Compilation © 2016-2020 610


Therefore, the partnership will be bound unless it is proved that Crafty had known that Fixer lacked
authority to hire Crafty, in which case Crafty would have no claim. The facts of the problem do not
suggest that Crafty had knowledge of Fixer’s lack of authority; therefore, Fixer had apparent authority as
a partner to bind the partnership by hiring Crafty because the act appeared to be in the ordinary course of
the partnership business.

[NOTE: The result under the 1914 version of the UPA would be the same.]

Explanation to Point-Three (15-25%):

Talker is personally liable for Crafty’s unpaid wages. However, to collect against Talker, Crafty must first
obtain judgments against Talker and the partnership and exhaust the assets of the partnership.

A partner is jointly and severally liable for all obligations of the partnership. As discussed above, Fixer
had apparent authority as a partner to bind the partnership to an employment contract with Crafty.
Therefore, the unpaid wages are a partnership obligation for which Talker is jointly and severally liable.

However, to collect from Talker personally, Crafty must first obtain a judgment against Talker and
against the partnership and levy execution against the partnership’s assets. Ordinarily, a claimant may not
take action against a partner’s personal assets unless the partnership assets fail to satisfy the claimant’s
judgment. Moreover, under the UPA (1997), a judgment against a partnership is not, by itself, a judgment
against a partner. Consequently, a judgment against a partnership may not be satisfied out of the partner’s
separate assets unless there is also a separate judgment against the partner. However, a judgment against a
partner personally may be sought in the same action as a judgment against the partnership.

In sum, the unpaid wages are a partnership obligation. Therefore, Crafty can execute against the separate
assets of Talker to satisfy the claim for unpaid wages only if Crafty obtains a judgment against Talker as
well as obtaining a judgment against the partnership and exhausting its assets.

[NOTE: Under the UPA (1914), a partner is jointly, as opposed to jointly and severally, liable for
obligations of the partnership. Consequently, all partners must be joined as defendants in an action based
upon a partnership contract unless a statute permits otherwise. Moreover, a partnership is not a legal
entity under the UPA (1914). Therefore, a third party cannot sue the partnership and is required to sue
all the individual partners unless a statute permits a suit against the partnership as an entity. While the
UPA (1914) does not require exhaustion of partnership assets prior to execution upon the separate assets
of a partner, case law in a number of UPA jurisdictions imposes such a requirement.]

Seperac-J19 Exam-Released MEE Essay Compilation © 2016-2020 611


#135-FEB 2007–MEE Q06: QUESTION SIX (AGENCY-PARTNERSHIP)

Astoria Limited Partnership (Astoria) is a properly formed limited partnership. The general partner of
Astoria is Baker, an individual. The limited partners are Tim, Uma, and Vivian, who are also individuals.
Under the limited partnership agreement, distributions and voting rights in the limited partnership are
allocated to Baker (10%), Tim (30%), Uma (30%), and Vivian (30%).

The limited partnership agreement of Astoria gives the limited partners the right to remove Baker as the
general partner – with or without cause – if the holders of 60% of the voting rights concur.

Three weeks ago, articles appeared in the business press raising questions about Baker’s management of
Caldonia Limited Partnership (Caldonia). While the articles did not specifically mention Astoria, the
misconduct documented in the articles – contracts between Caldonia and a janitorial company owned by
Baker that charged greatly inflated fees for its services – raised concerns on the part of Astoria’s limited
partners. Tim wrote Baker requesting that Baker, in his capacity as the general partner of Astoria, provide:
(1) copies of any contracts between Astoria and any entities related to or controlled by Baker, (2) copies
of any contracts for janitorial services between Astoria and any entity, (3) copies of Astoria’s federal and
state tax returns for the past three years, and (4) copies of all correspondence between Astoria and any
other parties for the past five years. Alternatively, Tim requested access to Astoria’s records in order to
secure copies of the documents himself.

Baker responded by refusing all four of Tim’s requests. Thereupon, Tim contacted Uma and Vivian to
request their votes that Baker be removed as the general partner. Tim also proposed that the three limited
partners, Tim, Uma, and Vivian, agree in writing to jointly run Astoria until a suitable replacement for
Baker could be found.

1. Did Baker act wrongfully in refusing to provide Tim the information and documents he
requested? Explain.

2. If the limited partners do no more than remove Baker as general partner of Astoria, will they be
liable to persons who transact business with Astoria? Explain.

3. If the limited partners remove Baker and run Astoria until a replacement general partner can be
found, under what circumstances will they be liable to persons who transact business with Astoria?
Explain.

Seperac-J19 Exam-Released MEE Essay Compilation © 2016-2020 612


#135: F07-6 MEE: ANSWER: NCBE (AGENCY-PARTNERSHIP)

POINT (1) [47%] ISSUE: Is a limited partner entitled to (a) copies of contracts between the limited
partnership and entities related to or controlled by the general partner, (b) copies of contracts
generally between the limited partnership and other entities, (c) copies of the limited partnership’s
federal and state tax returns, and (d) copies of all correspondence between the limited partnership
and all other parties for a five-year period? ANSWER: Yes. A limited partner is absolutely entitled
to copies of tax returns and is entitled to such copies of contracts and correspondence as is just and
reasonable.

POINT (2) [26%] ISSUE: If the limited partners remove the general partner, are they by that act
alone liable to persons who transact business with the partnership? ANSWER: No. Limited
partners are not liable as general partners simply for removing a general partner.

POINT (3) [26%] ISSUE: If the limited partners, after removing the general partner, run the
limited partnership until a replacement general partner can be found, are they liable to persons
who transact business with the partnership? ANSWER: Limited partners who participate in the
control of the business can be held liable for the obligations owed by the limited partnership, but
only to those persons who transact business with the limited partnership in the reasonable belief
that the limited partner is a general partner.

ANSWER DISCUSSION:

Limited partners have a right to obtain from the general partner “upon reasonable demand” information
regarding the state of the business and financial condition of the limited partnership, copies of tax returns,
and such other information regarding the partnership’s affairs as is just and reasonable. In this case, the
tax returns are clearly required to be disclosed, the contracts are required to be disclosed under both the
financial condition requirement and the other information requirement, and the correspondence is
probably required to be disclosed under the other information requirement (although a court could
conclude that the request is unreasonably broad). Limited partners have the right to remove a general
partner without that act being treated as an exercise of “control” of the limited partnership and without
becoming liable for the limited partnership’s obligations. If, following removal of the general partner, the
limited partners choose to run the partnership’s business, then they will be exercising control over the
limited partnership and each limited partner will be liable to persons who reasonably believe, based upon
the limited partner’s conduct, that the limited partner is acting as a general partner. Aggressive disclosure
of the limited partners’ status as limited, but not general partners, could limit such liability.

ANSWER EXPLANATION:

Explanation to Point-One (40-50%):

A limited partner is absolutely entitled to copies of tax returns and is entitled to such copies of contracts
and correspondence as is just and reasonable.

Under the Revised Uniform Limited Partnership Act (RULPA), a limited partner has a right to “inspect
and copy” partnership records that are required to be kept pursuant to § 105 of the Act, and, in addition, to
obtain from the general partner “upon reasonable demand”:

Seperac-J19 Exam-Released MEE Essay Compilation © 2016-2020 613


“true and full information regarding the state of the business and financial condition of the limited
partnership,”

“promptly after becoming available, a copy of the limited partnership’s federal, state, and local income
tax returns for each year,”

“other information regarding the affairs of the limited partnership as is just and reasonable.”

In some jurisdictions, the reasonableness of a demand will depend, in part, on the purpose for which the
limited partner is seeking disclosure. Where, as here, the limited partner has substantial reason to suspect
mismanagement of the limited partnership and wishes to obtain partnership documents to investigate the
matter, the limited partner has a right to access those documents.

Baker probably acted improperly in refusing to provide information in response to Tim’s request for
“copies of contracts between the limited partnership and entities related to or controlled by the general
partner.” This request should be covered under the first subsection of the Act because such information
relates to the “business and financial condition of the limited partnership.” In addition, where there is a
suggestion that a general partner is engaged in self-dealing, a limited partner’s request for specific
information that would confirm or disprove that suggestion is surely “just and reasonable.”

Tim’s demands for “copies of contracts generally between the limited partnership and other entities” are
also probably covered under the first and third subsections for the same reasons. Such information is
potentially relevant both to the general “business and financial condition” of the partnership and to the
possibility that Baker has been self-dealing.

The request for copies of the limited partnership’s federal and state tax returns is specifically covered
under the second subsection of the statute. In addition, tax returns are documents that the limited
partnership is required to keep and that a limited partner may inspect and copy at any time.

Tim’s request for “copies of all correspondence between the limited partnership and any other parties for
the past five years” may go too far. This request must be honored, if at all, only if the request is “just and
reasonable” under the third subsection of the statute. Whether a demand for copies of all correspondence
is “just and reasonable” is certainly debatable. Under the circumstances, Tim would claim that it is a
reasonable demand that would provide information useful in determining whether Baker was properly
managing the business. Baker would claim that the request goes far beyond what is reasonable, and is
essentially an effort to delve into all the files of the business on a fishing expedition for evidence of
wrongdoing.

Explanation to Point-Two (20-30%):

Limited partners are not liable as general partners simply for removing a general partner.

A limited partner is generally not liable for the obligations of a limited partnership. However, a limited
partner can become so liable if “in addition to the exercise of his or her rights and powers as a limited
partner, he or she participates in the control of the business.” Thus, the question as to whether the limited
partners would be liable for partnership obligations were they to remove Baker as general partner would
be resolved by determining whether the act of removing the general partner would be “participation in the
control of the business.”

Seperac-J19 Exam-Released MEE Essay Compilation © 2016-2020 614


Because of the inherent “difficulty of determining when the ‘control’ line has been overstepped,” RULPA
(and the parallel laws of most states) specifically enumerates certain activities in which a limited partner
may engage “without being deemed to have taken part in control of the business.” This so-called “safe
harbor” list of activities that do not constitute the exercise of control provides specifically that a “limited
partner does not participate in the control of the business” for purposes of incurring liability under § 303
“solely by proposing or voting [on the] removal of a general partner.” Thus, the limited partners in
Astoria may remove Baker as general partner without incurring liability for the limited partnership’s
obligations.

In states that have adopted the 2001 Uniform Limited Partnership Act, the situation is even clearer.
Limited partners are “not personally liable, directly or indirectly, by way of contribution or otherwise, for
an obligation of the limited partnership solely by reason of being a limited partner, even if the limited
partner participates in the management and control of the limited partnership.” The new Act provides the
limited partners a “full, status-based liability shield” akin to the protection given corporate shareholders.

Explanation to Point-Three (20-30%):

Limited partners who participate in the control of the business can be held liable for the obligations owed
by the limited partnership, but only to those persons who transact business with the limited partnership in
the reasonable belief that the limited partner is a general partner.

Limited partners can become liable for the obligations of the limited partnership if they participate in the
control of the business in ways not within the safe harbor exceptions in RULPA. In particular, Tim’s
suggestion that Tim, Uma, and Vivian jointly “run the limited partnership” after removing Baker from the
role of general partner appears to contemplate a degree of involvement in the business of the partnership
that exceeds what is permitted by any of the safe harbor exceptions. Moreover, the facts suggest that this
management and control of the business would necessarily constitute “participation in the control of the
business,” as the removal of Baker would leave no one but the limited partners in control.

Participation in the control of the business can make a limited partner liable for the obligations of the
limited partnership, but that liability extends only to “persons who transact business with the limited
partnership reasonably believing, based upon the limited partner’s conduct, that the limited partner is a
general partner.” Thus the limited partners could attempt to limit their liability when running the business
by giving all the parties with which the limited partnership does business notice that they are limited, not
general, partners.

In some jurisdictions, however, liability could be imposed, even without third-party reliance, if “a limited
partner’s control activities are so extensive as to be ‘substantially the same as’ those of a general partner.”
In these jurisdictions, Tim, Uma, and Vivian would be well-advised to make their limited role clear and to
search diligently for a replacement general partner to make clear that their temporary control did not
amount to taking on a role “substantially the same” as that of a general partner.

As noted earlier, the 2001 Uniform Limited Partnership Act “renders the control rule extinct.” Under this
Act, Tim, Uma, and Vivian would have no liability for partnership obligations even if they participated
directly in the management and control of the limited partnership.

Seperac-J19 Exam-Released MEE Essay Compilation © 2016-2020 615


#136-JUL 2006–MEE Q02: QUESTION TWO (AGENCY-PARTNERSHIP)

Two years ago Nephew asked Uncle for a loan to purchase a tire dealership. Uncle agreed. Nephew
decided to name the new business “Monster Tires” and to focus on selling oversized tires made by Big
Rubber, Inc. Uncle agreed to accept 50% of the profits generated by Monster Tires, instead of a fixed
payment, until the loan was fully repaid.

One year after Monster Tires was established, Nephew approached Friend, who had a knack for sales and
marketing and was well-connected in the trucking business. Friend agreed to join Monster Tires in
exchange for a one-third share of all of its profits. Friend and Nephew approached Uncle to renegotiate
the repayment of the loan. Uncle was excited by Friend’s ideas to expand the business and agreed to
forgive the loan and instead accept a one-third share of all of the profits of Monster Tires. Uncle also
offered to let Monster Tires use one of his properties rent-free so that Monster Tires could open a second
location.

Nephew and Friend both worked full time at Monster Tires. Friend began an aggressive sales campaign
and spent a significant amount of time entertaining potential customers. Uncle was not involved in the
day-to-day operations of Monster Tires.

Nephew, Uncle, and Friend were all present when Monster Tires opened its second store. The store was
packed with potential customers, but Uncle was dismayed by the sparse inventory. When a representative
of TireCo asked Uncle about placing its line of tires in the stores, Uncle agreed and signed a large
purchase order for TireCo tires without consulting either Nephew or Friend.

After the store opening, Uncle, Nephew, and Friend went out to celebrate. Friend suggested that Monster
Tires should buy a yacht for entertaining. Nephew agreed because he wanted to expand to a third location
near the beach. Uncle vehemently protested and said: “We can’t afford a yacht. I forbid you to purchase
one.” Two weeks later, Friend signed a contract with Custom Yachts to purchase a yacht for Monster
Tires.

Monster Tires is now refusing to pay both TireCo and Custom Yachts.

1. Is Monster Tires liable to TireCo for the tires Uncle ordered? Explain.

2. Is Monster Tires liable to Custom Yachts for the yacht Friend ordered? Explain.

3. Are Uncle, Nephew, and Friend personally liable on either of the contracts? Explain.

Seperac-J19 Exam-Released MEE Essay Compilation © 2016-2020 616


#136: J06-2 MEE: ANSWER: NCBE (AGENCY-PARTNERSHIP)

POINT (1)(a) [29%] ISSUE: Is Uncle a co-owner of a business for profit and therefore a partner of
Monster Tires? ANSWER: No. Uncle did not become a partner at the time he originally lent
Nephew money, but he later became a partner when he agreed to accept a one-third share of all the
profits.

POINT (1)(b) [29%] ISSUE: Did Uncle have authority to bind Monster Tires when he ordered tires
from TireCo? ANSWER: Yes. As a partner, Uncle is an agent of the partnership. He can bind the
partnership to the contract to buy tires from TireCo because this is the kind of business carried on
by the partnership.

POINT (2) [29%] ISSUE: Did Friend have authority to bind Monster Tires when he purchased the
yacht? ANSWER: No. Monster Tires is not liable to Custom Yachts because Friend had no
authority to purchase the yacht.

POINT (3) [14%] ISSUE: Are the partners personally liable for the obligations of the partnership?
ANSWER: Uncle, Nephew, and Friend are jointly and severally liable on the contract with TireCo
because that contract is an obligation of the partnership, but only Friend is liable on the Custom
Yachts contract.

ANSWER DISCUSSION:

Monster Tires is liable to TireCo for the tires Uncle ordered. Uncle is a partner of Monster Tires because
he is a co-owner of a business for profit. As a partner, Uncle had at least apparent authority to bind the
partnership to transactions in the ordinary course of business. Monster Tires is not liable to Custom
Yachts for the yacht Friend ordered. Because the purchase of the yacht was not in the ordinary course of
business of the partnership, all partners must authorize the purchase for it to become an obligation of the
partnership. The partners are personally liable on the TireCo contract because partners in a general
partnership are jointly and severally liable for all partnership obligations. Only Friend is personally liable
on the Custom Yachts contract.

ANSWER EXPLANATION:

Explanation to Point-One(a) (20-30%):

Uncle did not become a partner at the time he originally lent Nephew money, but he later became a
partner when he agreed to accept a one-third share of all the profits.

A partnership is defined as an association of two or more persons to carry on as co-owners of a business


for profit whether or not the persons intend to form a partnership. Uncle originally loaned money to
Nephew so Nephew could buy the tire dealership. Loaning money to Nephew does not make Uncle a
partner, nor does the fact that Nephew repaid the loan on the basis of 50% of the profits of the business.
Under RUPA, a person who shares profits is presumed to be a partner unless the profits were received in
payment of a debt or of interest or other charge on a loan.

Uncle’s status changed, however, when Uncle agreed to forgive the loan in return for a one-third share of
the profits. Now Uncle falls under the partnership presumption and Uncle is a partner. This conclusion is

Seperac-J19 Exam-Released MEE Essay Compilation © 2016-2020 617


strengthened by the fact that Uncle at the same time also agreed to provide rent-free space to the business,
something a co-owner might do for a business.

[NOTE: Better applicants should recognize that Uncle and Nephew did not initially create a partnership
because Uncle did not agree to participate in a business for profit.]

Explanation to Point-One(b) (20-30%):

As a partner, Uncle is an agent of the partnership. He can bind the partnership to the contract to buy tires
from TireCo because this is the kind of business carried on by the partnership.

Every partner is an agent of the partnership for the purposes of its business. If Uncle was a partner at the
time of the second store opening, his act of ordering the tires bound the partnership because it was an act
apparently for carrying on the ordinary course of partnership business. As a partner, Uncle had at least
apparent authority to place orders with Big Rubber, Inc., TireCo, or another manufacturer. This would be
so even if Uncle did not usually order tires and, indeed, even if he was not ever involved in the day-to-day
operations of the business. While it does not appear that Uncle had actual authority from his other two
partners to order the tires, he could still bind the partnership unless the TireCo representative knew or had
received notification prior to the time Uncle placed the order that Uncle lacked authority to do so.
Therefore, Monster Tires is liable to TireCo.

Explanation to Point-Two (20-30%):

Monster Tires is not liable to Custom Yachts because Friend had no authority to purchase the yacht.

Although Friend was a partner at the time he ordered the yacht, this purchase is not apparently for
carrying on the ordinary course of the tire dealership business. This is so, even though Friend regularly
entertains in the course of business, because the purchase of a yacht is an extraordinary transaction. Friend
therefore binds the partnership only if his act was authorized by all the other partners. The facts indicate
that Nephew and Friend agreed that Friend should purchase the yacht, but that Uncle was opposed to
buying it. Because an act outside the ordinary course of business of a partnership may be undertaken only
with the consent of all of the partners, Friend had no actual authority to buy the yacht.

Friend also did not have apparent authority to buy the yacht because Custom Yachts could not reasonably
believe that buying a yacht was in the ordinary course of business of a tire dealership. Therefore, Monster
Tires does not have to pay Custom Yachts.

[NOTE: Applicants who fail to see that the purpose of a yacht is not in the ordinary course of the
partnership’s business could conclude that Friend had actual, or apparent, authority to purchase the
yacht and bind the partnership.]

Explanation to Point-Three (10-15%):

Uncle, Nephew, and Friend are jointly and severally liable on the contract with TireCo because that
contract is an obligation of the partnership, but only Friend is liable on the Custom Yachts contract.

All partners are jointly and severally liable for all obligations of the partnership. The association of Uncle,
Nephew, and Friend is a general partnership because the partners took no steps to create a limited
partnership. The obligation to TireCo is one that occurred in the ordinary course of business. Therefore,
Uncle, Nephew, and Friend are each personally liable to TireCo. However, in order to recover from them

Seperac-J19 Exam-Released MEE Essay Compilation © 2016-2020 618


individually, TireCo must secure a judgment against both the partnership and the named partners.
Furthermore, TireCo must first seek recovery from the assets of the partnership before it can attempt to
levy execution against individual partners.

Only Friend is liable on the Custom Yachts contract because Friend had no authority to purchase the
yacht for the partnership. Under agency law, when an agent acts without authority, a third party (here,
Custom Yachts) can recover from the agent.

Seperac-J19 Exam-Released MEE Essay Compilation © 2016-2020 619


#137-FEB 2006–MEE Q02: QUESTION TWO (AGENCY-PARTNERSHIP)

Rich, who lives in Smalltown, recently bought a sports utility vehicle (SUV) from Dealer in Capital City,
which is located approximately 60 miles from Smalltown. Rich asked if Dealer would have the SUV
delivered to him in Smalltown. Rich offered to pay the driver $75. Dealer does not ordinarily deliver
SUVs to its customers. Nonetheless, Dealer agreed to have Sales, a full-time salesperson employed by
Dealer, deliver the SUV to Rich’s house because Dealer was eager to please Rich. At the end of the
normal workday, Dealer gave Sales directions to Rich’s house in Smalltown. As Sales prepared to drive
off in the SUV, Dealer told him, “Drive straight to Rich’s house, no detours. Drive carefully and no
speeding. I don’t want any dents in that SUV.”

On his way to Rich’s house, Sales stopped to visit Friend, who was in the process of moving. Friend was
distraught because the movers had not shown up. Friend asked Sales if he would like to earn $200 by
transporting “a few loads” in the SUV. Sales agreed and transported many of Friend’s belongings in the
SUV. Friend paid Sales $200.

Helping Friend took several hours, so Sales was running late. In his rush to Rich’s house, Sales
negligently ran over Ped, who was trying to cross the street. Ped was seriously injured.

Dealer learned that Sales had used the SUV to help Friend move and demanded the $200 Friend had paid
Sales.

1. Can Ped recover from Dealer for Sales’s negligence? Explain.

2. Must Sales give Dealer the $200 he earned helping Friend move? Explain.

Seperac-J19 Exam-Released MEE Essay Compilation © 2016-2020 620


#137: F06-2 MEE: ANSWER: NCBE (AGENCY-PARTNERSHIP)

POINT (1) [11%] ISSUE: Is Dealer vicariously liable for Sales’s negligence? ANSWER: Yes.
Dealer is vicariously liable for Sales’s negligence if there is a relationship between Dealer and Sales
that qualifies as master and servant, and Sales was acting within the scope of his employment when
he negligently injured Ped.

POINT (1)(a) [32%] ISSUE: Do Dealer and Sales have a master-servant relationship? ANSWER:
Yes. Dealer and Sales created a master-servant relationship.

POINT (1)(b) [32%] ISSUE: Was Sales acting within the scope of his employment? ANSWER:
Yes. Sales was acting in the scope of his employment when he negligently injured Ped.

POINT (2) [26%] ISSUE: Has Sales violated his fiduciary duty to Dealer such that he must give
Dealer the $200 that Friend paid him? ANSWER: Yes. Sales violated his fiduciary duty to Dealer
and therefore must give the $200 to Dealer.

ANSWER DISCUSSION:

Dealer is vicariously liable for Sales’s negligence. Dealer and Sales have a master-servant relationship
because Dealer employed Sales to perform services on behalf of Dealer, and Dealer had the right to
control the physical conduct of Sales’s performance. When Sales negligently injured Ped, Sales was
acting within the scope of his employment as he had been assigned the task of delivering the SUV to Rich
in Smalltown. Sales, as an agent, is a fiduciary. Sales violated his fiduciary duty to Dealer by failing to
obey instructions and failing to act solely for the benefit of Dealer. Sales must give all profits arising from
the agency relationship to the principal. Therefore, Sales must give the $200 to Dealer.

ANSWER EXPLANATION:

Explanation to Point-One: (05-15%):

Dealer is vicariously liable for Sales’s negligence if there is a relationship between Dealer and Sales that
qualifies as master and servant, and Sales was acting within the scope of his employment when he
negligently injured Ped.

Masters are liable for the torts of their servants committed while acting in the scope of their employment.
Therefore, Dealer is vicariously liable for Sales’s negligence if (a) Dealer and Sales created an agency
relationship that rises to the level of master-servant, and (b) Sales was acting within the scope of his
employment when he negligently injured Ped.

Explanation to Point-One(a) (25-35%):

Dealer and Sales created a master-servant relationship.

Agency is a fiduciary relationship “which results from the manifestation of consent by one person to
another that the other shall act on his behalf and subject to his control, and the consent by the other so to
act.”

Seperac-J19 Exam-Released MEE Essay Compilation © 2016-2020 621


Sales is a full-time employee of Dealer. Although (1) Dealer did not generally deliver SUVs, (2) Sales
was employed as a salesperson, not a driver, (3) delivery was made after the normal workday, and (4)
Rich agreed to pay Sales $75 for delivery, Sales was still an agent, acting on behalf of Dealer and subject
to Dealer’s control, when he delivered the SUV to Rich. Sales delivered the SUV at the request of Dealer.
The delivery was for the benefit of Dealer. Dealer agreed to make the delivery because Dealer “was eager
to please Rich.” Sales made the delivery subject to Dealer’s control. Dealer provided directions to Rich’s
house and instructions regarding the drive (“Drive straight to his house, no detours. Drive carefully and
no speeding. I don’t want any dents in that SUV.”). Overall, the facts support a conclusion that an agency
relationship existed between Dealer and Sales.

This agency relationship between Dealer and Sales rises to the level of master-servant. “A master is a
principal who employs an agent to perform services and who controls or has a right to control the physical
conduct of the agent’s performance.” This is a fact-based analysis. The Second Restatement of Agency
specifies factors to be considered in determining whether an agent is a servant or, instead, an independent
contractor. The most important factor is the extent of control that Dealer may exercise over the details of
the work. Dealer provided Sales with instructions regarding speed and route, which indicates the power to
control the manner in which Sales performed the work in question.

Other factors that support a master-servant relationship include the following: Delivering SUVs is not a
distinct occupation for Sales. Delivering SUVs is not work done by a specialist without supervision.
Delivering SUVs does not require special skills or equipment. Further, Sales is a full-time employee of
Dealer, and Dealer did not pay Sales any extra money for this work.

[NOTE: There are some factors that support an argument that Sales was acting as an independent
contractor rather than servant. Delivering SUVs was not a regular part of Dealer’s business. This was an
ad hoc assignment, not a regular part of Sales’s duties. Sales was paid separately for this work ($75 by
Rich). Further, Sales delivered the SUV after his regular hours of employment. While overall, it is likely
that a court would conclude that the agency relationship between Dealer and Sales rises to the level of a
master-servant relationship, an applicant should receive credit for a cogent argument either way.]

Explanation to Point-One(b) (25-35%):

Sales was acting in the scope of his employment when he negligently injured Ped.

For Dealer to be held vicariously liable for Sales’s negligence, the tort must have been committed within
the scope of Sales’s employment. Conduct is “within the scope of employment if, but only if” it is
conduct that the person was employed to perform, it occurs substantially within the authorized time and
space limits, and it is done to serve the master. Additionally, conduct is “within the scope of employment”
if it is “of the same general nature as that authorized, or incidental to the conduct authorized.”

An applicant might argue that delivering the SUV was not “within the scope of employment.” Although
Sales delivered the SUV to serve Dealer, Sales was employed as a salesperson. Additionally, the delivery
was performed after the normal workday and in Smalltown, not Capital City, where the dealership was
located.

However, scope of employment is determined by the manifestations of the master. It “includes only acts
of the kind authorized, done within limits of time and space which approximate those created by the
authorization.” Dealer specifically authorized Sales to deliver the SUV to Smalltown after work.
Therefore, this act was authorized and within the scope of employment.

Seperac-J19 Exam-Released MEE Essay Compilation © 2016-2020 622


While the escapade with Friend might be a “frolic” that exceeded the scope of Sales’s employment, the
accident occurred after any “frolic” had ended and while Sales was completing the delivery of the SUV
pursuant to Dealer’s instructions.

Explanation to Point-Two (20-30%):

Sales violated his fiduciary duty to Dealer and therefore must give the $200 to Dealer.

As an agent, Sales owes a fiduciary duty to Dealer, his principal, with respect to matters within the scope
of the agency. This includes the duty to obey reasonable instructions and the duty to act solely for the
benefit of the principal with respect to matters within the scope of his agency. When Sales made a detour
to help Friend move, Sales violated his duty to obey the reasonable instruction “Drive straight to his
house, no detours. Drive carefully and no speeding.” By using the SUV for his own benefit, Sales also
violated his duty to act solely for the benefit of Dealer as he used the SUV to make a personal profit. As a
result, Sales is liable to Dealer for the $200. Under the Second Restatement of Agency, an agent who
makes a profit in violation of his duty of loyalty is liable to principal for that profit, and an agent who, in
violation of his duty, uses principal’s assets for his own gain is subject to liability for that profit.

Further, even if an applicant were to argue that Sales had not violated his duty to Dealer, Sales would still
be obligated to give Dealer the $200. An “agent’s primary function is to make profits for the principal,
and his duty to account includes accounting for any unexpected and incidental accretions whether or not
received in violation of duty.”

Seperac-J19 Exam-Released MEE Essay Compilation © 2016-2020 623


#138-FEB 2005–MEE Q07: QUESTION SEVEN (AGENCY-PARTNERSHIP)

For many years, Ruth owned and operated a restaurant as a sole proprietorship doing business as (d/b/a)
Ruth’s Family Restaurant. In 2001, Ruth sold the assets of the restaurant to Scott. Ruth and Scott agreed
that: (1) the restaurant would operate under the name “Ruth’s Family Restaurant”; (2) Ruth would
manage the restaurant for Scott but would have no ownership interest in the restaurant; (3) all necessary
licenses would remain in Ruth’s name; and (4) Ruth would hire all employees, but only on an at-will basis
(as is customary in the restaurant business). No one other than Ruth and Scott was aware that Scott had
bought the restaurant.

Prior to Scott’s purchase of the restaurant, Ruth had purchased supplies from Wholesale Restaurant
Supply Co. (Wholesale), always signing the contracts as “Ruth, d/b/a Ruth’s Family Restaurant.”
Following Scott’s purchase of the restaurant, Scott instructed Ruth in very clear terms not to make any
purchases of restaurant supplies from Wholesale in the future. Ruth complied with this instruction for the
next several months.

In 2003, Ruth hired Nora, her niece, as assistant manager of the restaurant under a written employment
contract for a 20-year term. Ruth signed the contract as “Ruth, d/b/a Ruth’s Family Restaurant.”

Soon after Nora was hired, she pointed out to Ruth that Wholesale’s prices were generally less than those
of the other local supply company. Despite Scott’s clear prohibition, Ruth resumed buying supplies from
Wholesale, again signing all contracts as “Ruth, d/b/a Ruth’s Family Restaurant.”

When Scott discovered what Ruth had done, Scott took over management of the restaurant, discharged
Nora and Ruth, and refused to pay thousands of dollars of invoices from Wholesale for restaurant supplies
delivered to the restaurant.

Wholesale has sued Scott to recover on the outstanding invoices. Nora has sued Scott for breach of the
employment contract.

Under agency law:

1. Is Scott liable to Wholesale? Explain.

2. Is Scott liable to Nora? Explain.

Seperac-J19 Exam-Released MEE Essay Compilation © 2016-2020 624


#138: F05-7 MEE: ANSWER: NCBE (AGENCY-PARTNERSHIP)

POINT (1) [16%] ISSUE: What is the nature of the legal relationship between Scott and Ruth?
ANSWER: Ruth was Scott’s general agent for the operation of the restaurant. Because the agency
was not disclosed to persons dealing with Ruth’s Family Restaurant, Scott was an “undisclosed
principal.”

POINT (2) [26%] ISSUE: Does an agent have actual authority to bind an undisclosed principal to
contracts that are made in violation of the principal’s instructions to the agent? ANSWER: No.
Because her actions violated Scott’s express instructions, Ruth had no actual authority to purchase
restaurant supplies from Wholesale or to hire Nora for a 20-year term.

POINT (3) [26%] ISSUE: Does an agent have apparent authority to bind an undisclosed principal
to contracts that are made in violation of the principal’s instructions to the agent? ANSWER: No.
Ruth had no apparent authority to bind Scott because Scott was an undisclosed principal, and
neither Nora nor Wholesale was aware that Ruth was Scott’s agent.

POINT (4) [32%] ISSUE: Does an agent have inherent agency power to bind an undisclosed
principal to contracts that are made in violation of the principal’s instructions to the agent?
ANSWER: Scott might be liable to Wholesale on the theory that Ruth had inherent agency power
to bind Scott to contracts for the purchase of restaurant supplies, but a court is unlikely to conclude
that Ruth had inherent agency power to bind Scott to an extraordinary employment contract.

ANSWER DISCUSSION:

Ruth was Scott’s agent for purposes of operating the restaurant. However, Ruth did not have actual
authority to purchase supplies from Wholesale or to employ Nora for a 20-year term because these actions
violated Scott’s express instructions to Ruth. Ruth also lacked apparent authority to bind Scott because
Scott was an undisclosed principal, and neither Wholesale nor Nora was aware that Ruth was acting in an
agency capacity. Scott did nothing to manifest to Wholesale or Nora that Ruth had authority to act on his
behalf. Nevertheless, Ruth probably had inherent agency power to bind Scott to pay for the supplies
purchased from Wholesale. The purchase of restaurant supplies is the kind of transaction usually
performed by a restaurant manager, and the principal is appropriately held responsible for that transaction,
even if Ruth violated Scott’s orders in the way in which she carried it out. By contrast, hiring an assistant
manager for a 20-year term is not the type of transaction one would ordinarily expect to be performed by
a restaurant manager, and Ruth’s inherent agency power probably did not extend to binding Scott to such
a contract.

ANSWER EXPLANATION:

Explanation to Point-One (10-20%):

Ruth was Scott’s general agent for the operation of the restaurant. Because the agency was not disclosed
to persons dealing with Ruth’s Family Restaurant, Scott was an “undisclosed principal.”

Ruth and Scott agreed that Ruth would act as Scott’s agent. Moreover, Ruth was a “general agent” for
Scott, having been authorized, as manager of the restaurant, to conduct a series of transactions to ensure

Seperac-J19 Exam-Released MEE Essay Compilation © 2016-2020 625


continuity of service for Scott, as owner of Ruth’s Family Restaurant. However, neither party disclosed
the agency relationship to outsiders. As a result, Scott was Ruth’s “undisclosed principal.”

As Scott’s agent, Ruth could bind Scott to contracts with Wholesale or Nora if Ruth had actual authority,
apparent authority, or inherent agency power to enter such contracts. The issue in this problem is whether
an agent has the power to bind an undisclosed principal to contracts that the agent enters in violation of
instructions from the principal.

Explanation to Point-Two (20-30%):

Because her actions violated Scott’s express instructions, Ruth had no actual authority to purchase
restaurant supplies from Wholesale or to hire Nora for a 20-year term.

Actual authority “to affect the legal relations of the principal” exists only if the agent’s acts are “done in
accordance with the principal’s manifestations of consent.” Thus, Ruth had actual authority to bind Scott
only insofar as Ruth acted consistently with Scott’s “manifestations of consent” to her.

Here, Ruth’s acts were completely contrary to Scott’s instructions. Ruth was told not to contract with
Wholesale, yet she did so. Ruth was instructed to hire employees only on an at-will basis, yet she hired
Nora for a 20-year term. Because these acts were done contrary to Scott’s “manifestations of consent” to
Ruth, they were not done with actual authority.

Explanation to Point-Three (20-30%):

Ruth had no apparent authority to bind Scott because Scott was an undisclosed principal, and neither Nora
nor Wholesale was aware that Ruth was Scott’s agent.

Apparent authority is created when a purported principal causes third persons to believe that the purported
agent has the power to act on behalf of the principal. In other words, apparent authority depends on a
person’s manifestations to a third person “that another is his agent.”

Ruth could not have apparent authority to bind Scott because Scott made no manifestations to anyone that
Ruth was his agent. In fact, no one other than Ruth or Scott was aware of their agency relationship.
Furthermore, Scott and Ruth took pains to avoid disclosing the agency, even to the extent of Ruth’s
signing all contracts as “Ruth, d/b/a Ruth’s Family Restaurant.” “Apparent authority exists only with
regard to those who believe and have reason to believe that there is authority; there can be no apparent
authority created by an undisclosed principal.” Neither Nora nor Wholesale was aware that Scott was
Ruth’s principal.

Explanation to Point-Four (25-35%):

Scott might be liable to Wholesale on the theory that Ruth had inherent agency power to bind Scott to
contracts for the purchase of restaurant supplies, but a court is unlikely to conclude that Ruth had inherent
agency power to bind Scott to an extraordinary employment contract.

Even if Ruth lacked actual or apparent authority to bind Scott, as Scott’s agent, Ruth might have had
“inherent agency power” to bind Scott to transactions undertaken to benefit Scott that were similar to the
types of activities authorized, even if those transactions were in violation of orders. The concept of
“inherent agency power” provides a means for courts to protect persons dealing with agents even when
the agent violates orders. The theory is that an agent is generally acting “in the principal’s interests” and is

Seperac-J19 Exam-Released MEE Essay Compilation © 2016-2020 626


“trusted and controlled” by the principal. When such an agent’s disobedience causes a loss, it is “fairer”
that the loss fall upon the principal than on a third party, at least if the agent is engaged in acts generally
of a kind that would fall within his or her actual authority, but for the violation of instructions.

Inherent agency power has generally been found in precisely the kind of situation presented on these facts
– an agent has been given general power to manage a business and has done acts for the principal’s
account, albeit in violation of instructions.

Under the facts of this problem, Ruth probably did have inherent agency power to contract with
Wholesale for the delivery of restaurant supplies. Ruth was a general agent and was authorized to
purchase restaurant supplies. Purchasing restaurant supplies from the local supply company that offered
the best prices would be a “usual” or ordinary transaction for an agent managing a restaurant. If Ruth
somehow violated Scott’s special instructions, Scott should nonetheless be liable to Wholesale, especially
since Scott’s business certainly benefitted by purchasing cheaper supplies from Wholesale.

On the other hand, Scott is not likely to be held liable to Nora. Ruth’s entry into a contract to employ
Nora as an assistant manager for a 20-year term was not a transaction that would be “usual” in the
restaurant business. Although restaurant managers regularly hire and fire employees, at-will employment
contracts are the norm. An employment contract specifying a 20-year term would be extraordinary and
unusual in any business; certainly a 20-year term of employment is neither usual nor necessary in
employing an assistant restaurant manager. A restaurant’s business manager would not ordinarily have the
power to enter such a contract. Therefore, Ruth had no inherent agency power to bind Scott to a 20-year
employment contract with Nora.

Another fact to consider is that Nora was Ruth’s niece, which suggests that the fairness concern that
undergirds inherent agency analysis may, in fact, cut in Scott’s favor here. It would seem unfair to hold
Scott responsible for Ruth’s violation of instructions when the violation was done to benefit a member of
Ruth’s family.

[NOTE: The draft Third Restatement of Agency eliminates the concept of inherent agency power, but
would reach the same results in the case of an undisclosed agency relationship under a theory that it calls
“estoppel of undisclosed principal.”]

Seperac-J19 Exam-Released MEE Essay Compilation © 2016-2020 627


#139-JUL 2004–MEE Q05: QUESTION FIVE (AGENCY-PARTNERSHIP)

Best Care Hospital, one of five hospitals in City, operates the largest emergency room in City. Best Care
advertises extensively about the quality of care provided in its emergency room. It has billboards
strategically placed throughout City urging local citizens to come to Best Care “because Best Care’s
emergency room doctors are the absolute best and will really care for you.” In fact, Best Care employs no
doctors; instead it contracts with seven doctors in City to staff the emergency room on a 7-day, 24-hour
basis. These contracts provide:

1. Each doctor is an “independent contractor,” not an “agent/employee,” and may conduct a private
practice but may not work in any other emergency room;

2. Each doctor is responsible for the manner in which he or she provides medical care and for the
purchase of malpractice insurance;

3. Each doctor is authorized to purchase supplies and equipment for Best Care’s emergency room from a
list of approved vendors located in City and within Best Care’s price guidelines;

4. Each doctor is periodically reviewed by Best Care’s governing board to assure that each doctor
provides quality care;

5. Each doctor independently bills patients for services provided; and

6. All emergency services are performed in the Best Care emergency room using supplies and equipment
provided by Best Care.

Three months ago, Owen, a local orthopedist and one of the doctors with whom Best Care contracts,
ordered a portable X-ray machine costing $25,000 from Vision, a company located in a town 450 miles
from City. Vision is not on Best Care’s approved vendor list and Owen did not consult with anyone at
Best Care before he placed the order. When Owen ordered the machine, which was to be custom-designed
for Best Care, he truthfully told Vision that he was one of the seven emergency room doctors at Best Care
and needed the machine for the emergency room. Owen also stated that he was acting on behalf of Best
Care. Vision had had no previous dealings with Owen or Best Care and agreed to make the machine
according to the custom specifications provided by Owen. When Vision shipped the X-ray machine, Best
Care refused to accept delivery, even though the price for the machine was within its price guidelines.
Best Care claimed that Owen had no authority to purchase the machine on its behalf. Vision filed an
action for breach of contract against Best Care.

Last month, Anita was hit by a bus. When the ambulance arrived, Anita asked the ambulance driver to
take her to Best Care, quoting the billboard claim that “Best Care’s emergency room doctors are the
absolute best.” When Anita arrived at the emergency room, she was treated by Owen. Owen correctly told
Anita that she needed immediate surgery. During the operation, Owen negligently severed one of Anita’s
arteries, and she bled to death. Anita’s estate has filed a wrongful death action against Owen and Best
Care for damages resulting from Owen’s negligence.

1. Is Owen an independent contractor or servant (employee) of Best Care? Explain.

2. Is Best Care liable to Vision for breach of contract? Explain.

Seperac-J19 Exam-Released MEE Essay Compilation © 2016-2020 628


3. Assuming Owen is an independent contractor, is Best Care liable to Anita’s estate for Owen’s
negligence? Explain.

Seperac-J19 Exam-Released MEE Essay Compilation © 2016-2020 629


#139: J04-5 MEE: ANSWER: NCBE (AGENCY-PARTNERSHIP)

POINT (1) [38%] ISSUE: Is Owen an independent contractor or a servant (employee) of Best
Care? ANSWER: Owen was probably an independent contractor and not an employee of Best
Care, but the facts could be argued differently.

POINT (2) [33%] ISSUE: Did Owen have actual or apparent authority to bind Best Care to the
contract to purchase the X-ray machine? ANSWER: No. Owen had neither actual nor apparent
authority to bind Best Care to the contract to purchase the X-ray machine. Thus, Best Care is not
liable to Vision for breach of contract.

POINT (3) [29%] ISSUE: Is Best Care liable to Anita’s estate for Owen’s negligence on an
apparent (ostensible) agency theory? ANSWER: Yes. Even if Owen is an independent contractor,
Best Care is liable for Owen’s negligence if Anita reasonably believed that Owen was acting as an
agent of Best Care.

ANSWER DISCUSSION:

Whether Owen is an independent contractor or a servant (employee) is a question of fact. The degree of
control is generally the distinguishing factor between an independent contractor and a servant (employee).
Traditionally, Owen would have been an independent contractor because Best Care did not retain
substantial control over how Owen was to practice medicine; however, recent opinions have moderated
this position and look to the right to control rather than actual exercise of control. Best Care is not liable
on the Vision-Owen contract. Owen lacked actual authority to enter into the contract with Vision on
behalf of Best Care because Vision was not a vendor in City. Further, Best Care had made no
manifestations to provide Vision with a reasonable basis to believe that Owen was acting as an agent of
Best Care. Even if Owen is an independent contractor, he appears to the outside world – and to Anita in
particular – to be an employee of Best Care as a result of Best Care’s representations made on the
billboards. Because of this apparent (ostensible) agency, Best Care would be estopped from denying
liability for Owen’s negligence and Anita’s resulting death.

ANSWER EXPLANATION:

Explanation to Point-One (35-45%):

Owen was probably an independent contractor and not an employee of Best Care, but the facts could be
argued differently.

Owen is either an independent contractor or a servant (employee). “An independent contractor is a person
who contracts with another to do something for him but who is not controlled by the other nor subject to
the other’s right to control with respect to his physical conduct in the performance of the undertaking. He
may or may not be an agent.”. “A servant employee is an agent employed by a master employer to
perform service in his affairs whose physical conduct in the performance of the service is controlled or is
subject to the right to control by the master.” A number of factors are relevant in distinguishing an
independent contractor from a servant. The crucial factor is the extent of control that the employer may
exercise over the details of the work. Traditionally, doctors were considered independent contractors
because of their high level of skill and the use of their independent judgment. However, in recent years,
courts have placed greater emphasis on the employer’s “right to interfere or control rather than the

Seperac-J19 Exam-Released MEE Essay Compilation © 2016-2020 630


employer’s actual interference or exercise of control.” Other factors used to distinguish between a servant
and an independent contractor are: (1) whether the one employed is engaged in a distinct occupation or
business, (2) the skill required in the particular occupation, (3) who supplies the materials to perform and
the place to perform the service, (4) method of payment, and (5) how the parties characterize the
transaction.

These facts could be argued either way. Here, most of the factors suggest that Owen is an independent
contractor and not an employee or servant of Best Care. In particular items 1, 2, and 5 of the contract
favor characterizing Owen as an independent contractor, while items 4 and 6 of the contract indicate some
level of oversight and control on the part of Best Care. The self-labeling in item 1 is evidence, but is not
decisive.

Explanation to Point-Two (30-40%):

Owen had neither actual nor apparent authority to bind Best Care to the contract to purchase the X-ray
machine. Thus, Best Care is not liable to Vision for breach of contract.

Agency arises where one person consents that another shall act on his behalf and, at least as respects
matters within the agency, subject to his control.

Actual authority arises where the principal communicates the authority to act to the agent. A principal is
bound by contracts entered into between an agent with actual authority and a third party.

Item 3 of the contract provides that “each doctor is authorized to purchase supplies and equipment for
Best Care’s emergency room from a list of approved vendors located in City and within Best Care’s price
guidelines.” This contract provision gave Owen actual authority to buy equipment for Best Care so long
as it was within specified price guidelines from suppliers located in City for Best Care’s account. If Owen
had acted within this authority, Best Care would be bound even if Vision was unaware of the
authorization. However, Owen exceeded his authority by purchasing equipment from Vision, which was
not an approved vendor and was located 450 miles away from City. Therefore, Best Care is not liable
under the contract because Owen did not have actual authority.

Further, Best Care is not liable under the theory of apparent authority. Apparent authority is created as to
a “third person by written or spoken words or any other conduct of the principal which, reasonably
interpreted, causes the third person to believe that the principal consents to have the act done on his behalf
by the person purporting to act for him.” There are no facts to suggest that Best Care made any
representations that would create a reasonable basis for Vision to believe that Owen was authorized to
purchase the X-ray machine.

Explanation to Point-Three (25-35%):

Even if Owen is an independent contractor, Best Care is liable for Owen’s negligence if Anita reasonably
believed that Owen was acting as an agent of Best Care.

While a master is liable for the torts of its servant (employee) conducted within the scope of employment,
a principal is not necessarily liable for the torts of its independent contractor agent. A principal, however,
may be estopped from denying liability for the torts of its independent contractor if the principal has
created indicia of apparent authority. This is sometimes referred to as the “holding out” theory or
ostensible agency. For example, in one case, a hospital was found liable under the “holding out” theory

Seperac-J19 Exam-Released MEE Essay Compilation © 2016-2020 631


because the hospital acted in some way that led the patient to a reasonable belief that he was being treated
by a hospital employee.

The Second Restatement of Agency § 267 provides that “one who represents that another is his servant or
other agent and thereby causes a third person justifiably to rely upon the care or skill of such apparent
agent is subject to liability to the third person for harm caused by: the lack of care or skill of the one
appearing to be a servant or other agent as if he were such.” Here, Best Care publicly advertised, on
billboards strategically placed throughout City, that people should come to its emergency room because
“Best Care’s emergency room doctors are the absolute best and will really care for you.” Anita apparently
saw and believed the slogan on the billboards because she expressly requested the ambulance driver to
take her to Best Care because “Best Care’s emergency room doctors are the absolute best.” It would be
reasonable for a prospective patient to conclude, based on the billboards, that all doctors in Best Care’s
emergency room are employees of Best Care. As it was reasonable for Anita to believe that Owen was an
agent of Best Care, under the general principle that a principal is liable for the torts committed by its
agents Best Care would be liable to Anita’s estate for Owen’s actions (negligently severing an artery so
that she bled to death).

[NOTE: Most jurisdictions provide that a principal who hires an independent contractor to perform an
“ultrahazardous” or “inherently dangerous” activity has a nondelegable duty to exercise due care to
provide for the safety of persons who may be harmed by that activity. However, the usual tort and agency
law definitions of ultrahazardous activity would not cover surgery, and there is no line of authority
holding that a doctor’s performance of surgery falls within this rule.]

Seperac-J19 Exam-Released MEE Essay Compilation © 2016-2020 632


#140-FEB 2004–MEE Q06: QUESTION SIX (AGENCY-PARTNERSHIP)

One year ago, Randy and Sandy formed a partnership to manufacture widgets. They did not enter into a
written partnership agreement. Randy contributed $10,000, and Sandy contributed $5,000 to the
partnership. They participated equally in the management of the partnership and shared the profits
equally.

The widget market has since declined, and business has been slow. Last Monday, Randy told Sandy, “I
don’t want to do this anymore. I am quitting this partnership.” Randy then left town on an extended
vacation. At that time, the partnership still had some short-term widget contracts to fulfill and owed trade
creditors $30,000.

The next day, Barney, a widget broker, met with Sandy at the widget factory. Barney knew about Randy
and Sandy’s partnership but had not previously done business with the partnership. For over a month,
Sandy had been soliciting a long-term contract to produce widgets for Barney. Without telling Barney
about Randy’s statement the previous day, Sandy, on behalf of the partnership, entered into a contract
with Barney to produce 500,000 widgets a year for three years.

After Randy’s departure, Sandy, using her own funds, paid the $30,000 owed to trade creditors.

1. What is the effect of Randy’s statement, “I don’t want to do this anymore. I am quitting this
partnership”? Explain.

2. Is the partnership bound by the contract with Barney? Explain.

3. What contribution, if any, can Sandy claim from Randy for Sandy’s $30,000 payment to trade
creditors? Explain.

Seperac-J19 Exam-Released MEE Essay Compilation © 2016-2020 633


#140: F04-6 MEE: ANSWER: NCBE (AGENCY-PARTNERSHIP)

POINT (1) [39%] ISSUE: Does the statement “I don’t want to do this anymore. I am quitting this
partnership,” made by one partner, dissolve an “at will” partnership? ANSWER: Yes. Randy’s
statement “I don’t want to do this anymore. I am quitting this partnership,” dissolves the “at will”
partnership. This begins a period of winding up (liquidation) during which time the partnership
affairs are settled. Once winding up is complete, the partnership terminates.

POINT (2) [39%] ISSUE: May a partner bind a partnership to a contract made after dissolution?
ANSWER: Yes. The partnership is probably bound by Sandy’s contract with Barney, even though
it occurred after dissolution, because Sandy had apparent authority to enter into the contract.

POINT (3) [21%] ISSUE: What is a partner’s liability for partnership obligations? ANSWER:
Because there is no agreement to the contrary, when Sandy paid $30,000 to the trade creditors,
Sandy became entitled to a contribution of $15, 000 from Randy.

ANSWER DISCUSSION:

The effect of Randy’s “quitting” is to dissolve the partnership and to put it in a “winding up” period, at
the conclusion of which it terminates. As the partnership had already dissolved, Sandy lacked actual
authority to enter into the contract with Barney on behalf of the partnership. All the same, the partnership
is bound by the contract with Barney because Sandy still had apparent authority to bind the partnership.
Because they did not agree otherwise, Randy and Sandy share losses equally. Because Sandy paid the
trade creditors, she is entitled to a $15,000 contribution from Randy.

ANSWER EXPLANATION:

Explanation to Point-One (35-40%):

Randy’s statement “I don’t want to do this anymore. I am quitting this partnership,” dissolves the “at will”
partnership. This begins a period of winding up (liquidation) during which time the partnership affairs are
settled. Once winding up is complete, the partnership terminates.

The facts do not indicate that the partnership had a definite term or limited undertaking. Therefore, this
was an “at will” partnership. Any partner may dissolve an “at will” partnership by his or her express will.

Under the Uniform Partnership Act (UPA), dissolution occurs when the parties cease to associate in
carrying on the business together. The dissolution of a partnership is the change in the relation of the
partners caused by any partner ceasing to be associated in the carrying on as distinguished from the
winding up of the business. “On dissolution the partnership is not terminated but continues until the
winding up of partnership affairs is completed.” Winding up is “the process of settling partnership affairs
after dissolution.” Once all partnership affairs have been settled, the partnership terminates.

When Randy told Sandy “I don’t want to do this anymore. I am quitting this partnership,” Randy
dissolved the partnership. The partnership must now undergo a period of winding up (i.e., completing the
remaining business and settling debts). Once this is done, the partnership is terminated.

Seperac-J19 Exam-Released MEE Essay Compilation © 2016-2020 634


Under the Revised Uniform Partnership Act (RUPA), Randy’s statement is an event of dissociation.
Sandy has “notice of the partner’s express will to withdraw as a partner.” Therefore, under RUPA § 801
Randy’s statement dissolves the partnership and the business must be wound up. Pursuant to RUPA §
802, “a partnership continues after dissolution only for the purpose of winding up its business. The
partnership is terminated when the winding up of its business is completed.”

Explanation to Point-Two (35-40%):

The partnership is probably bound by Sandy’s contract with Barney, even though it occurred after
dissolution, because Sandy had apparent authority to enter into the contract.

Under the UPA, upon dissolution, a partner’s actual authority to bind the partnership terminates except as
is necessary to wind up the business. The contract with Barney was for new business. When Randy
dissolved the partnership, Sandy’s actual authority to enter into a contract for new business ended.

Under the RUPA, upon dissolution, a “partnership is bound by a partner’s act after dissolution that is
appropriate for winding up the partnership business.” As indicated above, the contract with Barney was
for new business. There is an exception under RUPA § 803 that permits a partner who has not wrongfully
dissociated to preserve the partnership business as a going concern for a reasonable period of time. These
facts do not come within this exception. To date, the business has existed for only a year and it only had
short-term contracts remaining. The contract with Barney was a long-term contract (three years).

While Sandy lacked actual authority to contract with Barney, under both the UPA and the RUPA, Sandy
likely had apparent authority to bind the partnership after dissolution, so long as Barney was not aware of
the dissolution and reasonably believed that Sandy was authorized to act. Under the UPA, after
dissolution, a partner can bind the partnership by any transaction which would bind the partnership if
dissolution had not taken place, provided the other party to the transaction, though he had not so extended
credit, had nevertheless known of the partnership prior to dissolution, and had no knowledge or notice of
dissolution. Under RUPA, a partnership is bound by a partner’s act after dissolution that would have
bound the partnership before dissolution, if the other party to the transaction did not have notice of the
dissolution. RUPA § 301 provides a partner with apparent authority to carry on in the ordinary course the
partnership’s business unless the other party to the transaction knows that there is no authority.

The facts support the conclusion that Sandy had apparent authority to enter into the contract. Even though
Barney had not previously done business with the partnership, the facts state that he was familiar with the
partnership. Sandy, on behalf of the partnership, had been soliciting a long-term contract from him for
over a month. Sandy was acting in the ordinary course of the partnership’s business when soliciting a
contract for widgets. Further, Barney was not aware of the dissolution. Therefore, the partnership is
probably bound by Sandy’s contract with Barney.

Explanation to Point-Three (15-25%):

Because there is no agreement to the contrary, when Sandy paid $30,000 to the trade creditors, Sandy
became entitled to a contribution of $15, 000 from Randy.

When Sandy paid the entire $30,000 debt to the trade creditors, she became entitled to contribution of
$15,000 from Randy. Among partners, unless the parties agree otherwise, profits are shared equally and
losses are shared in the same ratio as profits. Under the UPA, each partner shall share equally in the
profits and must contribute towards the losses according to his share in the profits. Under RUPA, each
partner is entitled to an equal share of the partnership profits and is chargeable with a share of the

Seperac-J19 Exam-Released MEE Essay Compilation © 2016-2020 635


partnership losses in proportion to the partner’s share of the profits. Regardless of the fact that Randy
contributed twice as much capital ($10,000) as Sandy ($5,000) to the partnership, the facts indicate that
they shared profits equally. Because they did not agree otherwise, Randy and Sandy would share losses in
the same ratio as they shared profits.

Seperac-J19 Exam-Released MEE Essay Compilation © 2016-2020 636


#141-JUL 2003–MEE Q05: QUESTION FIVE (AGENCY-PARTNERSHIP)

Adam, Barbara, and Carl are partners in a beverage distribution business. They have no written
partnership agreement. The partnership distributes alcoholic and nonalcoholic beverages to bars and
restaurants throughout State A. Jane owns one of these bars. She also owns a number of pinball and video
game machines, which are located in many of the same bars and restaurants served by the partnership. In
past dealings with the partnership, Jane has dealt exclusively with Carl.

A number of beverage distribution businesses have expanded into the pinball and video game machine
business. To capitalize on this trend, Jane decided to sell her pinball and video game machines. At about
the same time, Adam, Barbara, and Carl decided that their partnership should consider expanding into the
pinball and video game machine business. The partners agreed that Carl would approach Jane to obtain
information about the number of machines she had for sale, their locations and condition, and the
associated revenues and expenses. However, because the opportunity represented a new line of business,
Adam and Barbara instructed Carl not to finalize a deal with Jane without first discussing the terms with
them.

When Carl met with Jane, she said that she owned 127 machines located in 72 bars and restaurants. After
they visited 50 bars and saw 98 machines, Carl decided he had seen enough. Carl told Jane he would go
back to the office to “run some numbers” and would call her soon.

Based on Jane’s representations and his observations of 98 machines, Carl decided to offer $225,000 for
all of Jane’s 127 machines. Jane accepted the offer and signed the contract Carl had drafted. Carl signed
the contract on behalf of the partnership. At no time, however, did Carl consult with Adam or Barbara
before finalizing the deal. Adam and Barbara later discovered that the revenue generated by the machines
was insufficient to justify the contract price.

1. Is the partnership bound by the contract that Carl signed? Explain.

2. Can the partnership recover from Carl any loss on the contract with Jane? Explain.

Seperac-J19 Exam-Released MEE Essay Compilation © 2016-2020 637


#141: J03-5 MEE: ANSWER: NCBE (AGENCY-PARTNERSHIP)

POINT (1) [57%] ISSUE: Under the theory of apparent authority, is a partnership bound by a
contract entered into by a partner when that partner lacked actual authority? ANSWER: Yes.
Although Carl lacked actual authority to enter into the contract, the partnership is bound by the
contract if Carl had apparent authority to act on the partnership’s behalf on this matter.

POINT (2) [43%] ISSUE: Is a partner liable to the partnership for breach of fiduciary duties when
the partner enters into a contract against the other partners’ instructions and without providing
complete information to his partners? ANSWER: Yes. Carl is liable to the partnership for any loss
arising from the contract with Jane because Carl breached his fiduciary duty to the partnership by
signing this contract in violation of the instructions from Adam and Barbara, and without
providing full disclosure to Adam and Barbara.

ANSWER DISCUSSION:

Carl lacked actual authority to bind the partnership by signing the contract. In this three-person
partnership, Adam and Barbara had expressly limited his authority when they told him “not to finalize a
deal with Jane without first discussing the terms with them.” The partnership, however, may be bound by
the contract if Carl had apparent authority. Whether Carl had apparent authority depends on (a) whether
expanding into the pinball and video game machine business could be considered “carrying on in the
ordinary course” of this partnership’s business, and (b) whether Jane viewed Carl as authorized to
negotiate and sign the contract on behalf of the partnership. These issues can be resolved by a fact-based
analysis. Carl is liable to the partnership for any loss arising from the contract because he is an agent of
the partnership and violated his fiduciary duties to the partnership: Carl violated express instructions
when he entered into the contract without first consulting with Adam and Barbara; he also failed to make
adequate disclosure to Adam and Barbara.

ANSWER EXPLANATION:

Explanation to Point-One (50-70%):

Although Carl lacked actual authority to enter into the contract, the partnership is bound by the contract if
Carl had apparent authority to act on the partnership’s behalf on this matter.

Carl’s Actual Authority (10-20%): Each partner is entitled to participate equally “in the management and
conduct of the partnership business.” The rules of partnership management provide for majority rule on
matters arising in the ordinary course and unanimous rule on matters arising out of the ordinary course of
business.

Carl lacked actual authority to sign the contract binding the partnership because Adam and Barbara
expressly instructed Carl “not to finalize a deal with Jane without first discussing the terms with them.”
The express instruction from Adam and Barbara “not to finalize” restricted Carl’s actual authority. In this
three-partner partnership, Carl lacked unanimous or majority approval to proceed beyond investigating
the opportunity. Therefore, Carl lacked actual authority and the partnership is not bound on an actual
authority basis.

Seperac-J19 Exam-Released MEE Essay Compilation © 2016-2020 638


Carl’s Apparent Authority (40-50%): As a partner, Carl is an agent of the partnership. “An act of a partner
for apparently carrying on in the ordinary course the partnership business or business of the kind carried
on by the partnership binds the partnership, unless the partner had no authority to act for the partnership in
the particular matter and the person with whom the partner is dealing knew the partner lacked authority.”
There is no suggestion that Jane was aware of any limitation on Carl’s authority. Thus, two questions
arise: (a) whether Carl had apparent authority to enter into purchase agreements on behalf of the
partnership, and (b) whether the purchase of pinball and video machines was within the ordinary course of
the partnership’s business. These questions are appropriately viewed from Jane’s perspective. A court is
obligated to look at custom and past dealings between the parties or with the particular partnership.

With respect to the first question, it would be perfectly reasonable for Jane to believe that Carl had the
authority to enter into a purchase agreement without consulting his partners and there is no evidence that
she knew that he lacked authority in this instance. The facts state that Carl represented the partnership in
past dealings with Jane, negotiating and signing contracts without input from Adam or Barbara. Thus,
from Jane’s perspective, when Carl signed the contract with Jane, he was carrying on business “in the
usual way” in which it was always carried on for the partnership.

With respect to the second question, the issue is whether the purchase of pinball and video game machines
would appear to be a carrying on of “the partnership business or business of the kind carried on by the
partnership.” The partner’s act must not only be an act carrying out business “in the usual way,” but it
must also be an act the public would “reasonably conclude is directly and necessarily embraced within the
partnership business as being incident or appropriate to such business according to the course and usage
of conducting it.” The act must be “apparently within the nature of the business” of the partnership. Put
simply, the act must be “within the ordinary and apparent scope of the partnership business”.

In this case, the partnership was engaged in the beverage distribution business. An argument could be
made that Carl’s purchase of pinball and video game machines was neither “partnership business” nor
“business of the kind carried on by the partnership.” Ownership of pinball and video games is not
“directly and necessarily embraced” within the scope of a beverage business, nor is it obviously “incident
or appropriate to such business.”

On the other hand, other beverage distribution businesses in the same geographic area as this partnership
have been expanding into the pinball and video game machine business. Thus, even if the business of this
partnership had been limited to beverage distribution, it is arguable that the supply of other goods and
services to bars and restaurants, including game machines, is a “business of the kind carried on by the
partnership” and is therefore within the “nature of the business” of this partnership. Moreover, Jane, who
was aware of the trend of beverage distributors expanding into the pinball and video game business could
view this partnership’s expansion as being “incident or appropriate to its business according to the course
and usage of conducting it in her area.” For example, under the Florida Revised Uniform Partnership Act,
the scope of partnership business is determined with reference to acts of such businesses in the geographic
area in which the partnership operates.

The apparent authority arguments could go either way. The important task for applicants is to recognize
the issue and apply the test appropriately. There is also an argument that even if Carl lacked authority to
enter the contract, the partnership ratified the partnership contract when it presumably accepted the
machines and began operating them.

Explanation to Point-Two (40-50%):

Seperac-J19 Exam-Released MEE Essay Compilation © 2016-2020 639


Carl is liable to the partnership for any loss arising from the contract with Jane because Carl breached his
fiduciary duty to the partnership by signing this contract in violation of the instructions from Adam and
Barbara, and without providing full disclosure to Adam and Barbara.

A partner is an agent of the partnership and owes duties to the partnership and the other partners. When a
partner violates a duty, the partner is liable to the partnership and other partners for the resulting loss. Carl
violated his duties to the partnership and the other partners when he entered into the contract in violation
of his instructions and without discussing the terms with Adam and Barbara. Although the rationale
differs under the UPA and the RUPA, under either statute, Carl is liable to the partnership for the loss
resulting from his breach of duty.

Carl breached several duties to the partnership. First, under the common law and the UPA, Carl is an
agent of the partnership and owes a duty of obedience to his other partners, including a duty to follow
reasonable instructions. Carl breached this duty by failing to follow Adam and Barbara’s instruction that
he not enter into any contract with Jane unless he first discussed the terms with them. (In a jurisdiction
where the RUPA is applicable, one could also argue that this failure to follow instructions was
“intentional misconduct” and a breach of Carl’s duty of care.

Second, Carl had an obligation to provide full information to Adam and Barbara. That obligation was
imposed by their express instruction to him “not to finalize a deal with Jane without first discussing the
terms” with them. Under the UPA, partners must render on demand true and full information of all things
affecting the partnership to any partner. In addition, courts have interpreted the UPA to require full
disclosure of information material to partnership affairs even in the absence of an explicit demand, and
the RUPA specifically imposes such an obligation.

Finally, it is arguable that Carl breached his duty of care by failing to thoroughly investigate the facts
before entering into the contract with Jane. The amount of money involved was substantial, and Carl
arguably should have visited all 72 bars and checked all 127 machines, not just 50 bars and 98 machines,
before entering the contract. The common law, the UPA, and the RUPA all impose on partners a general
duty of care, including a duty to act in good faith when carrying out partnership business.

However, the duty of care is generally violated only by “grossly negligent or reckless conduct, intentional
misconduct, or a knowing violation of the law.” Although Carl may have been negligent in failing to
inspect all machines and was clearly wrong in failing to consult with Barbara and Adam before finalizing
the deal, these omissions may not rise to the level of gross neglect (or worse) necessary to violate the duty
of care. From all that appears, Carl acted in the good faith belief that he was making a good deal for the
partnership, he undertook an extensive (if incomplete) examination of the relevant information before
agreeing to purchase the pinball machines, and his failure to consult with Adam and Barbara, while a
violation of instructions, was consistent with the way the partnership had done business in the past. Under
the circumstances, a breach of the duty of care may be difficult to establish.

There is no evidence of any self-interest in the transaction, and thus there is no duty of loyalty problem.

In summary, Carl likely did not breach his duty of care. He did, however, breach his duty of obedience
and his duty to provide full information. He is therefore liable to the partnership for the losses resulting
from either breach.

Seperac-J19 Exam-Released MEE Essay Compilation © 2016-2020 640


#142-FEB 2003–MEE Q04: QUESTION FOUR (AGENCY-PARTNERSHIP)

Lessor owns and manages apartment buildings in the town of Utopia.

Handy is the sole proprietor and only employee of a small business called “Rapid Repairs.” Most of the
income from this business is generated by making small household repairs for homeowners and apartment
dwellers in Utopia. Handy has a good reputation for performing quality work and charging reasonable
rates.

One year ago, Lessor contracted for an indefinite period with Handy to perform repair work at several
apartment units that Lessor owns in Utopia. The tenants of these units are told to make requests for repairs
by calling a telephone number listed as “Lessor’s Repair Line.”

Under the Lessor/Handy contract, any call to “Lessor’s Repair Line” actually rings directly through to
Rapid Repairs. Handy is obligated to investigate any tenant’s request for repair within 24 hours. Before
actually making any repair, however, Handy is required to contact Lessor, describe the nature of the
repair, and seek authorization to proceed. Once authorized to make the repair, Handy must make it within
24 hours. Lessor is obligated to pay Handy $50 per hour for any work done pursuant to the contract
(including investigating repair requests) and, in addition, to reimburse Handy’s out-of-pocket expenses.

The Lessor/Handy contract further provides that Handy:

• may perform similar work on other apartment buildings, but may not perform work “on the side” for
Lessor’s tenants in Lessor’s buildings;

• must provide his own tools;

• may not perform any electrical work, but must subcontract it to a licensed electrician, approved by
Lessor, who will work under Handy’s supervision.

Last month, Tenant called “Lessor’s Repair Line.” Handy answered. Tenant said he had a cracked sink
drainpipe. Handy immediately investigated. After obtaining permission from Lessor to repair the sink,
Handy returned to make the repair. While Handy was making the repair, Tenant asked Handy to install an
electrical outlet in the apartment for Tenant’s computer. Despite his contract with Lessor, Handy agreed
to do so, but told Tenant there would be a charge of $200 as it was an improvement to the apartment, not a
repair covered under the lease. Tenant agreed to pay $200 for a new outlet. Tenant assumed that the
money would go to Lessor, but Handy intended to keep it for himself.

Handy was negligent in installing the electrical outlet. The outlet caused a fire, which destroyed Tenant’s
personal property. Tenant never paid for the installation work.

On what alternative theories should Tenant argue that Lessor is liable for Handy’s negligence, and what is
the likely outcome on each theory? Explain.

Seperac-J19 Exam-Released MEE Essay Compilation © 2016-2020 641


#142: F03-4 MEE: ANSWER: NCBE (AGENCY-PARTNERSHIP)

POINT (1)(a) [32%] ISSUE: Does Lessor exercise sufficient control over Handy to establish Handy
as Lessor’s “servant” so that Lessor is liable for Handy’s torts? ANSWER: Yes. The relationship
between Handy and Lessor may be a master-servant relationship such that Lessor is liable for torts
committed by Handy while acting within the scope of his employment.

POINT (1)(b) [18%] ISSUE: If Handy is Lessor’s servant, is Handy’s conduct within the scope of
employment? ANSWER: Yes. If Handy is a servant, his actions were within the scope of
employment because, although not authorized, adding an electrical outlet was incidental to
authorized conduct.

POINT (2) [50%] ISSUE: If Handy is not a servant, did Tenant reasonably rely upon Handy’s
apparent authority to install an electrical outlet so that Lessor is nonetheless liable for Handy’s
tort? ANSWER: Yes. Even if Handy is not a servant, or is a servant not acting within the scope of
employment, Lessor is liable if Tenant reasonably believes Handy was Lessor’s agent and relied on
what appeared to be authority for Handy to perform the work.

ANSWER DISCUSSION:

Lessor may be held liable for Handy’s tortious conduct under either of two theories. First, if the Lessor-
Handy relationship is one of master and servant, Lessor is liable for Handy’s torts if Handy was acting
within the scope of his employment when he committed them. Second, even if the relationship between
Lessor and Handy does not rise to the level of a master-servant relationship, Lessor will be liable for
Handy’s tort if Tenant relied on statements or conduct by Handy that were within Handy’s apparent
authority.

ANSWER EXPLANATION:

Explanation to Point-One(a) (30-40%):

The relationship between Handy and Lessor may be a master-servant relationship such that Lessor is
liable for torts committed by Handy while acting within the scope of his employment.

A master is liable for the torts of the master’s servants committed while acting in the scope of their
employment. Handy is clearly an agent as he agreed to work for, and subject to the control of, Lessor. The
more difficult question is whether Handy would be considered to be Lessor’s servant.

[NOTE: The Third Restatement of Agency has adopted “Employer-Employee” language in place of the
traditional “Master-Servant” terminology and applicants may of course use this newer terminology.]

Whether Handy performed the work for Tenant as Lessor’s “servant” or as an independent contractor
agent depends on the degree of control exercised by Lessor over Handy’s activities. The Second
Restatement of Agency § 2 provides that a “servant is an agent employed by a master to perform service
in his affairs whose physical conduct in the performance of the service is controlled or is subject to the
right to control by the master.” Conversely, the Second Restatement of Agency § 2 provides that “an
independent contractor is a person who contracts with another to do something for him but who is not
controlled by the other nor subject to the other’s right to control with respect to his physical conduct in the

Seperac-J19 Exam-Released MEE Essay Compilation © 2016-2020 642


performance of the undertaking.” The Second Restatement of Agency § 220 lists factors used to
determine whether an agent is a servant. These factors are:

(a) the extent of control which, by the agreement, the master may exercise over the details of the work;

(b) whether or not the one employed is engaged in a distinct occupation or business;

(c) the kind of occupation, with reference to whether, in the locality, the work is usually done under the
direction of the employer or by a specialist without supervision;

(d) the skill required in the particular occupation;

(e) whether the employer or the workman supplies the instrumentalities, tools, and the place of work for
the person doing the work;

(f) the length of time for which the person is employed;

(g) the method of payment, whether by the time or by the job;

(h) whether or not the work is a part of the regular business of the employer;

(i) whether or not the parties believe they are creating the relation of master and servant; and

(j) whether the principal is or is not in business.

In Handy’s case, the facts permit a persuasive argument on either side. Several factors suggest that Handy
should be considered a servant of Lessor. First, Lessor exercises significant control over Handy, in
insisting that any repair be approved by Lessor. Second, the relationship appears to be long term, not a
temporary arrangement for a small number of repair jobs; it has already lasted for a year. Third, payment
is by time worked, not by the particular job. Finally, the work performed is a necessary part of Lessor’s
business of owning and operating apartment buildings.

On the other hand, several factors support an argument for classifying Handy as an independent
contractor. First, Handy, as a repairman, is engaged in a distinct occupation and business that requires
some degree of special skill. Second, Handy provides his own tools. Third, Handy has his own separate
repair business and appears to have taken on Lessor’s work in the course of operating that separate
business. Fourth, Lessor approves repairs to his properties but does not specify the methods that Handy
must use.

[NOTE: Either analysis of the servant issue is plausible. The point is that Lessor is only liable under the
theory of respondeat superior if Handy is a servant.]

Explanation to Point-One(b) (15-25%):

If Handy is a servant, his actions were within the scope of employment because, although not authorized,
adding an electrical outlet was incidental to authorized conduct.

Conduct is within the scope of employment if the conduct is of the “same general nature authorized or
incidental to the conduct authorized.”

Seperac-J19 Exam-Released MEE Essay Compilation © 2016-2020 643


Factors examined in determining whether conduct is within the scope of employment include whether the
conduct is the kind the servant is employed to perform; whether it occurs substantially within the
authorized time and space; and whether it was performed, at least in part, to serve the master.

In this case, Handy was not authorized to add the electrical outlet as Handy’s contract with Lessor
expressly prohibited him from doing electrical work. It also prohibited him from doing work “on the side”
for a tenant. However, even forbidden activity can be within the scope of employment.

The Second Restatement of Agency § 229 sets forth factors to consider in determining whether an action
is incidental to the authorized conduct so that it comes within the scope of employment. These factors
include:

(a) whether or not the act is one commonly done by such servants;

(b) the time, place and purpose of the act;

(c) the previous relations between the master and the servant;

(d) the extent to which the business of the master is apportioned between different servants;

(e) whether or not the act is outside the enterprise of the master or, if within the enterprise, has not been
entrusted to any servant;

(f) whether or not the master has reason to expect that such an act will be done;

(g) the similarity in quality of the act done to the act authorized;

(h) whether or not the instrumentality by which the harm is done has been furnished by the master to the
servant;

(i) the extent of departure from the normal method of accomplishing an authorized result; and

(j) whether or not the act is seriously criminal.

A key fact that points to the conclusion that adding the electrical outlet was incidental to the scope of
employment is the fact that Handy did the electrical work while providing Tenant with authorized
services.

Explanation to Point-Two (50-60%):

Even if Handy is not a servant, or is a servant not acting within the scope of employment, Lessor is liable
if Tenant reasonably believes Handy was Lessor’s agent and relied on what appeared to be authority for
Handy to perform the work.

The Second Restatement of Agency § 265 sets out the general rule: “(1) A master or other principal is
subject to liability for torts which result from reliance upon, or belief in, statements or other conduct
within an agent’s apparent authority.” Section 265 conditions this liability upon reliance.

An agent is clothed with apparent authority when a principal “by written or spoken words or any other
conduct of the principal which, reasonably interpreted, causes the third party to believe that the principal
consents to have the act done on his behalf by the person purporting to act for him.” The facts indicate

Seperac-J19 Exam-Released MEE Essay Compilation © 2016-2020 644


that Lessor clothed Handy with apparent authority by advising tenants to call the “Lessor’s Repair Line”
when they needed repairs, and then using Handy’s business phone number as the “Lessor’s Repair Line.”
The act of using Handy’s number reasonably caused Tenant to believe that Handy was authorized by
Lessor to act on Lessor’s behalf. There are no facts to suggest that Tenant knew or could expect to know
that a person authorized to make general repairs was prohibited by contract from adding an electrical
outlet.

With respect to reliance, the facts permit a persuasive argument on either side. Arguably, Tenant did not
reasonably rely on Handy’s apparent authority because Tenant was told that adding an electrical outlet
was an improvement, not a repair, and Handy made repairs. Further, Handy indicated that there was a
charge for the electrical work.

However, the alternative argument could also be made. Lessor arranged to have “Lessor’s Repair Line”
ring directly to Handy’s office. Tenant knew that Handy was working for Lessor; in fact Handy was
present doing work on behalf of Lessor. It is not unreasonable to believe that the same person designated
to make repairs would also make improvements. Further, the facts indicate that Tenant believed the
payment for the work ($200) was to go to Lessor.

Seperac-J19 Exam-Released MEE Essay Compilation © 2016-2020 645


#143-JUL 2002–MEE Q06: QUESTION SIX (AGENCY-PARTNERSHIP)

Sunrise Lodge is a corporation that develops and operates luxury resort hotels. Sunrise recently began
constructing a hotel in East Beach, a beach town on the Atlantic coast of the United States. Sunrise hoped
to give the East Beach hotel a local flavor by using local sources for materials.

Sunrise hired Adam to be its interior design agent on the East Beach project. The contract between them,
which was for a one-year term, included the following language:

Adam has the discretion to make selections for interior floor and wall coverings, works of art, furniture,
plumbing fixtures, and lighting fixtures for East Beach hotel, provided that (a) the cost of such purchases
does not exceed the budgeted amounts listed in Exhibit A, (b) all purchases will be made from local
vendors, and (c) the items selected are within the quantity and style guidelines described in Exhibit B.
Adam shall inform vendors that purchases are for Sunrise East Beach and should arrange for Sunrise to be
billed on a 30-day net basis.

The style guidelines in Exhibit B include a comprehensive list of themes and styles typical of an Atlantic
fishing village like East Beach, including lighthouses, whitewashed wood, lobster traps, wicker furniture,
and sailboats.

After hiring Adam, Sunrise sent a letter to prospective local suppliers on Sunrise stationery signed by the
Sunrise president, announcing Adam's appointment as follows:

Sunrise Lodge is delighted to announce the appointment of Adam, a well-known local interior designer, to
act on its behalf in the selection of interior floor and wall coverings, works of art, furniture, and plumbing
and lighting fixtures for the Sunrise East Beach hotel. We are confident that – working only with local
suppliers – Adam will exercise a wonderful creative flair in coming up with just the right look for this
exciting project. Know that you deal with Sunrise when you deal with Adam on this project.

During the first months of Adam's one-year term, Adam entered into the following transactions with
suppliers who had received Sunrise's letter.

First, Adam contracted with Tahini for the main lobby area of the hotel to be decorated entirely in a
Tahitian theme. The items for the Tahitian decor are within the budget and are from a local supplier.
However, they are not within the Exhibit B style guidelines.

Second, Adam contracted with Moby for the guest rooms to be decorated using authentic themes from the
Atlantic seaboard region as required by Exhibit B. The decor selections are within budget and are from
local suppliers, but the Sunrise officials do not like the design.

Sunrise has refused to pay either vendor and has terminated Adam's contract.

1. On what agency principles, if any, is Sunrise liable to Tahini and Moby on their respective
contracts? Explain.

2. Is Adam liable to Sunrise as a result of the contracts with Tahini and Moby? Explain.

Seperac-J19 Exam-Released MEE Essay Compilation © 2016-2020 646


3. May Sunrise terminate Adam's agency before the end of their one-year contract term without
incurring liability to Adam? Explain.

Seperac-J19 Exam-Released MEE Essay Compilation © 2016-2020 647


#143: J02-6 MEE: ANSWER: NCBE (AGENCY-PARTNERSHIP)

POINT (1)(a) [31%] ISSUE: Did Adam act with either actual or apparent authority in contracting
for the Tahitian lobby decorations? ANSWER: Yes. Sunrise is liable on the contract for the
purchase of the Tahitian decorations because Adam acted with apparent authority.

POINT (1)(b) [19%] ISSUE: Did Adam act with either actual or apparent authority in contracting
with Moby for the guest room furnishings? ANSWER: Yes. Adam acted with actual authority in
contracting with Moby for the guest room furnishings and therefore Sunrise is liable on the
contract.

POINT (2) [19%] ISSUE: Does Sunrise have a claim for indemnification from Adam for either of
the contracts for which Sunrise is liable? ANSWER: Sunrise has a claim against Adam for the
amount of Sunrise’s liability to Tahini but does not have a claim against Adam for its liability to
Moby for the guest room furnishings.

POINT (3) [31%] ISSUE: Can Sunrise terminate Adam’s agency prior to the expiration of the one-
year term, and if so, is Sunrise liable to Adam for damages based on the early termination?
ANSWER: Yes. Sunrise can terminate Adam’s agency at any time and end Adam’s authority to
bind Sunrise. Early termination of Adam’s agency may expose Sunrise to liability for damages to
Adam, but in this case Sunrise has a defense to such a claim.

ANSWER DISCUSSION:

Adam acted with apparent but not actual authority with respect to the contract with Tahini. Adam acted
with apparent authority because Sunrise advised Tahini of the agency agreement but not of the limitations
on Adam’s authority. On the other hand, Adam acted with actual authority with respect to the Moby
contract which was consistent with all of the terms of the agency contract between Sunrise and Adam.
Thus, Sunrise is liable to both Tahini and Moby, albeit on different theories. Sunrise can seek
indemnification from Adam for its liability under the Tahini contract but not under the Moby contract.
Because Adam materially breached the agency contract, Sunrise can terminate that contract prior to the
end of the fixed term in the contract without any liability to Adam.

ANSWER EXPLANATION:

Explanation to Point-One(a) (20-30%):

Sunrise is liable on the contract for the purchase of the Tahitian decorations because Adam acted with
apparent authority.

Adam had no actual authority to purchase the Tahitian lobby items on Sunrise’s behalf. Under the Second
Restatement of Agency, actual “authority to do an act can be created by written or spoken words or other
conduct of the principal which, reasonably interpreted, causes the agent to believe that the principal
desires him so to act on the principal’s account.” Sunrise made it clear to Adam that the items had to be
within the style guidelines described in Exhibit B. The Tahitian items were inconsistent with the
guidelines, and thus Adam had no actual authority to purchase them.

Seperac-J19 Exam-Released MEE Essay Compilation © 2016-2020 648


On the other hand, Adam did have apparent authority to purchase the Tahitian lobby items on Sunrise’s
behalf. Under the Restatement, “apparent authority to do an act is created as to a third person by written
or spoken words or any other conduct of the principal which, reasonably interpreted, causes the third
person to believe that the principal consents to have the act done on his behalf by the person purporting to
act for him.” Sunrise had sent a letter to Tahini appointing Adam “to act on its behalf in the selection of
interior floor and wall coverings, works of art, furniture, and plumbing and lighting fixtures for the
Sunrise East Beach hotel.” The Sunrise letter emphasized Adam’s authority by stating: “Know that you
deal with Sunrise when you deal with Adam on this project.” Sunrise did not indicate to Tahini that there
were any restrictions on Adam’s scope of discretion or authority, and the facts do not suggest that Tahini
had knowledge of any limitations through other means. As a result, it was reasonable for Tahini to
conclude that Adam acted with authority when he ordered the decorations. Since Adam acted with
apparent authority, Sunrise is liable to Tahini.

Explanation to Point-One(b) (10-20%):

Adam acted with actual authority in contracting with Moby for the guest room furnishings and therefore
Sunrise is liable on the contract.

When Adam contracted with Moby for the guest room furnishings, he was within the budget and style
limitations required by Sunrise and Moby was a local supplier. Therefore, he acted within the scope of the
actual authority granted by Sunrise. Because Adam acted within the scope of the grant of actual authority,
Sunrise is liable on the contract to Moby for the guest room furnishings. In addition, Adam also acted
with apparent authority because Moby received the letter from Sunrise.

Explanation to Point-Two (10-20%):

Sunrise has a claim against Adam for the amount of Sunrise’s liability to Tahini but does not have a claim
against Adam for its liability to Moby for the guest room furnishings.

Sunrise has a claim against Adam for the amount of its liability to Tahini because Adam did not act within
the scope of his actual authority and, therefore, breached his duty to follow directions.

Sunrise has no claim against Adam for reimbursement of its liability to Moby for guest room furnishings
because Adam acted within the scope of his grant of authority. He followed directions and fulfilled his
fiduciary obligations.

Explanation to Point-Three (20-30%):

Sunrise can terminate Adam’s agency at any time and end Adam’s authority to bind Sunrise. Early
termination of Adam’s agency may expose Sunrise to liability for damages to Adam, but in this case
Sunrise has a defense to such a claim.

Although the general rule is that authority conferred for a specific time terminates at the expiration of the
period, the Second Restatement of Agency provides that: “authority terminates if the principal manifests
to the agent dissent to its continuance.” Therefore, Sunrise may terminate Adam as its agent at any time,
including before the end of the one-year term.

The principal has the power to terminate the agency even in violation of the agency contract: “The
principal has power to revoke although doing so is in violation of a contract between the parties.”

Seperac-J19 Exam-Released MEE Essay Compilation © 2016-2020 649


However, the principal “has a duty not to repudiate or terminate the employment in violation of the
contract of employment.” In such a situation, the principal may be liable to the agent for damages.

Given Adam’s deviation from the terms of his contract with Sunrise, Adam is not likely to recover from
Sunrise for early termination. First, if a termination is based upon the agent’s breach of contract, then the
principal has an offset against the agent’s damages. Second, “a principal is privileged to discharge before
the time fixed by the contract of employment an agent who has committed such a violation of duty that
his conduct constitutes a material breach of contract.” That appears to be the case in this situation.

Seperac-J19 Exam-Released MEE Essay Compilation © 2016-2020 650


#144-FEB 2002–MEE Q03: QUESTION THREE (AGENCY-PARTNERSHIP)

Several years ago, Bill, Carl, Donna, and several other participants formed a limited partnership named
Transitions L.P., which operated a career counseling business. The limited partnership was properly
formed, a written limited partnership agreement was properly executed, and all required filings were made
in the proper state offices. The Certificate of Limited Partnership and the Limited Partnership Agreement
do not modify any default provisions of the Revised Uniform Limited Partnership Act.

Bill is the sole general partner in Transitions. Carl, Donna, and the remaining partners are limited
partners. Although the career counseling industry has experienced a rapid boom in the past few years,
Transitions’ business has not been financially successful and no distributions of profits have been made to
the limited partners. As a result, several of the limited partners, including Carl and Donna, are unhappy
with Bill’s management of Transitions.

Carl decided that the limited partners should reach a consensus on how Transitions’ day-to-day business
should be conducted and then force Bill to follow their instructions. Carl went to Bill asking questions
about the business operations of the limited partnership and asking to see Transitions’ limited partnership
records, including a list of the names and addresses of the other partners. Bill refused to answer any of
Carl’s questions, to allow Carl to examine any limited partnership records, and to provide him with
information about the other partners. Bill claims that even if the limited partners all agree to change the
way Transitions’ business is run, Bill has no obligation to listen to them.

Donna does not believe that Transitions will be able to succeed in business if Bill remains in control.
When Carl tells Donna that Bill does not intend to give up management of Transitions, Donna sells her
interest in the limited partnership to Edward. Donna informs Bill that she has sold her interest to Edward
and that Edward is now a limited partner in Transitions. Bill tells Donna that he doesn’t approve of the
sale and that Edward has no rights in the limited partnership and is not a partner.

1. Can Carl and the other limited partners decide how the day-to-day business of Transitions is to
be conducted? Explain.

2. What rights, if any, does Carl have to obtain information from Bill about the business operations
and records of Transitions? Explain.

3. What rights, if any, did Edward acquire as a purchaser of Donna’s interest in Transitions?
Explain.

Seperac-J19 Exam-Released MEE Essay Compilation © 2016-2020 651


#144: F02-3 MEE: ANSWER: NCBE (AGENCY-PARTNERSHIP)

POINT (1) [44%] ISSUE: Can the limited partners decide how the day-to-day business of a limited
partnership is to be conducted? ANSWER: No. Carl and the other limited partners have no right to
determine how the day-to-day business of Transitions is to be conducted. Management of the
business is vested exclusively in Bill, the general partner.

POINT (2) [17%] ISSUE: What rights does a limited partner have to obtain information from the
general partner about the limited partnership’s affairs? ANSWER: Carl, as a limited partner, has
the right to inspect various records of Transitions and to obtain information about the affairs of
Transitions from Bill, the general partner.

POINT (3) [39%] ISSUE: What rights does a purchaser of a limited partnership interest have in
the limited partnership if such purchase is not approved by the partnership? ANSWER: Edward’s
purchase of Donna’s limited partnership interest in Transitions did not result in Edward’s
becoming a limited partner, although Edward did obtain Donna’s financial rights in Transitions.

ANSWER EXPLANATION:

Explanation to Point-One (35-45%):

Carl and the other limited partners have no right to determine how the day-to-day business of Transitions
is to be conducted. Management of the business is vested exclusively in Bill, the general partner.

A limited partnership is a partnership which has one or more general partners and one or more limited
partners. A general partner of a limited partnership generally has the powers and the liabilities of a partner
in a general partnership. Management of a limited partnership is entrusted to the general partners, who are
personally liable for the obligations of the business. A general partner controls the business of the limited
partnership to the exclusion of the limited partners.

By comparison, limited partners generally have no control over how the day-to-day business of the
limited partnership is conducted. For example, one court held that a limited partner was not entitled to
have a provision included in limited partnership agreement giving him a voice in the selection of a
manager, because limited partners do not have a statutory right to exercise control of the partnership’s
decisions. The role of limited partners in a limited partnership is generally passive, with only limited
rights to vote on extraordinary events of the partnership, such as dissolution or sale of substantially all of
the partnership’s assets.

Therefore, Bill, as the sole general partner, has exclusive control over the daily management of
Transitions, and Carl and the other limited partners cannot decide how the day-to-day business of
Transitions is to be conducted.

Explanation to Point-Two (10-20%):

Carl, as a limited partner, has the right to inspect various records of Transitions and to obtain information
about the affairs of Transitions from Bill, the general partner.

Seperac-J19 Exam-Released MEE Essay Compilation © 2016-2020 652


A limited partnership is required to keep specified records at its office, including the names and addresses
of each partner, any income tax returns or reports for the three most recent years, and any financial
statements for the three most recent years. Upon reasonable request, any limited partner may inspect and
copy during ordinary business hours any of the records required to be kept by the limited partnership. In
addition, any limited partner has the right to obtain from the general partners upon reasonable demand
“true and full information regarding the state of the business and financial condition of the limited
partnership” and “other information regarding the affairs of the limited partnership as is just and
reasonable.”

Therefore, Carl has the right to inspect the records that Transitions is required to keep and the right to
obtain from Bill, the general partner, true and full information regarding the business and financial
condition of Transitions.

Explanation to Point-Three (30-40%):

Edward’s purchase of Donna’s limited partnership interest in Transitions did not result in Edward’s
becoming a limited partner, although Edward did obtain Donna’s financial rights in Transitions.

A limited partnership interest is personal property and, except as provided in the partnership agreement, is
freely assignable in whole or in part. However, assignment of a limited partnership interest does not
entitle the assignee to become a limited partner or to exercise any of the rights of a limited partner.
Instead, assignment only entitles the assignee to receive (to the extent assigned) any distributions to which
the assignor would be entitled. In other words, a limited partner may only assign his or her economic
rights in the limited partnership.

An assignee of a limited partnership interest may be admitted as a limited partner to the partnership only
if all partners have consented to such admission, unless the partnership agreement expressly gives the
transferring partner the right to admit a new partner without approval of the other partners. The facts
indicate that Bill disapproves of the sale and there is no indication that the other partners approved the
admission of Edward as a limited partner. Edward therefore did not become a limited partner in
Transitions upon Donna’s sale of her limited partnership interest to Edward. As a result, Edward cannot
exercise any of the rights of a limited partner, such as the right to inspect records, the right to obtain
information regarding the limited partnership the right to seek a judicial dissolution or the right to bring a
derivative action

On the other hand, the assignment of Donna’s limited partnership interest to Edward did transfer Donna’s
economic rights in Transitions to Edward. Consequently, Edward is entitled to any distributions from
Transitions to which Donna would have been entitled.

Seperac-J19 Exam-Released MEE Essay Compilation © 2016-2020 653


#145-JUL 2001–MEE Q01: QUESTION ONE (AGENCY-PARTNERSHIP)

Principal is an antiques dealer. As is common in the antiques business, Principal acquires inventory by
using a group of buyers to purchase antiques on his behalf. Principal pays the buyers a percentage
commission on the items they buy for Principal. Principal trains the buyers to be able to evaluate potential
purchases and sends them into the field with specific instructions as to the items Principal wants them to
buy. The buyers are given credentials identifying them as buyers for Principal, and they use these
credentials to introduce themselves to potential sellers. The buyers, using Principal’s contract forms, enter
into contracts with sellers.

Agent, one of Principal’s buyers, was sent out to purchase antiques for Principal. During the next several
months, the following three transactions took place:

Using Principal’s credentials, Agent bought an antique church bell for $3,500 from Bellseller, who
believed that Agent was acting on behalf of Principal. The church bell was on Principal’s acquisition list
and the price was within the range authorized by Principal. However, Agent did not intend to purchase the
bell for Principal. Instead, Agent bought the bell for Greta, a competing antiques dealer, who had agreed
to pay Agent $250 to find such a bell. Agent’s intention was to use Greta’s money to pay for the bell.
Greta rejected the bell. Principal, who has received a demand from Bellseller, has decided he does not
want the bell and has refused to pay Bellseller.

Agent had a written authorization from Principal to buy several books, including Looking Backward by
Edward Bellamy. Agent showed this authorization to Tomeseller, who had a first edition of Looking
Backward. However, Agent did not tell Tomeseller that Principal had given Agent an oral instruction, not
to be disclosed to anyone, that Agent should not pay more than $8,000 for the book. Agent bought the
book on Principal’s behalf for $12,500. Principal now refuses to pay Tomeseller.

When Principal learned of Agent’s transactions with Bellseller and Tomeseller, he decided to stop using
Agent as his buyer. Principal sent Agent a letter terminating his agency and asking Agent to return the
credentials that Principal had provided to Agent.

Several days after Agent received the letter from Principal, Agent purchased for his own account a whale
oil lamp for $5,000 from Lampseller. Agent told Lampseller that the purchase was for Principal’s account,
showed Lampseller his credentials from Principal, and purchased the lamp using the contract forms
provided to him by Principal. Agent did not disclose to Lampseller that Principal had terminated Agent as
his buyer. Principal now refuses to pay Lampseller because, at the time of the purchase, Principal had
terminated Agent and Agent was no longer Principal’s buyer.

Is Principal liable to:

1. Bellseller for the purchase of the church bell? Explain.

2. Tomeseller for the purchase of the book? Explain.

3. Lampseller for the purchase of the whale oil lamp? Explain.

Seperac-J19 Exam-Released MEE Essay Compilation © 2016-2020 654


#145: J01-1 MEE: ANSWER: NCBE (AGENCY-PARTNERSHIP)

POINT (1) [34%] ISSUE: Is a principal bound when an agent who has actual authority enters into
an obligation apparently for the benefit of the principal but in fact for the benefit of a third party?
ANSWER: Yes. Principal is liable to Bellseller for the church bell. Principal is bound by Agent’s
purchase of the church bell apparently on Principal’s behalf, even though Agent was in fact
operating on Greta’s behalf, since Bellseller was unaware of that fact.

POINT (2) [34%] ISSUE: Is a principal bound when an agent who has actual authority enters into
an obligation in violation of an undisclosed limitation placed on the agent’s authority by the
principal? ANSWER: Yes. Agent acted with actual authority to purchase the book, and with
apparent authority because Tomeseller was unaware of Principal’s price limitation. Therefore,
Principal is liable even though Agent exceeded the terms of his authorization.

POINT (3) [31%] ISSUE: Is a principal bound when an agent whose actual authority has been
terminated but who continues to have apparent authority enters into an obligation on behalf of the
principal? ANSWER: Yes. Because Agent was acting with apparent authority, even though
Principal had terminated all actual authority, Principal is liable to Lampseller for the purchase of
the lamp.

ANSWER EXPLANATION:

Explanation to Point-One (25-35%):

Principal is liable to Bellseller for the church bell. Principal is bound by Agent’s purchase of the church
bell apparently on Principal’s behalf, even though Agent was in fact operating on Greta’s behalf, since
Bellseller was unaware of that fact.

There is no doubt that Principal is liable. However, different jurisdictions are likely to reach this result
based on different theories.

First, some jurisdictions would conclude that Agent had actual authority to bind Principal to the contract
with Bellseller because Agent was expressly authorized by Principal to purchase the bell and the price that
Agent agreed to pay was within the range authorized by Principal. Moreover, because the contract entered
into by Agent disclosed that Principal was Agent’s principal, Bellseller knew that Agent was acting as
agent for a third party. This makes Principal a disclosed principal who is “subject to liability upon a
contract purported to be made on his account by an agent authorized to make it for the principal’s benefit,
although the agent acts for his own or another’s improper purposes, unless the third party has notice that
the agent is not acting for the principal’s benefit.” Here, Bellseller had no knowledge that Agent was not
acting for Principal’s benefit, and as a result, Principal is liable to Bellseller for the price of the church
bell. (In some jurisdictions, this same reasoning would be used to hold Principal liable on an “inherent
authority” rationale. Agent, on the other hand, could be liable to Principal for breach of fiduciary duty.

Other jurisdictions would conclude that Agent did not have actual authority. Agent did not make the
purchase for Principal, he made the purchase for Greta and his act of purchasing the bell for Greta was
NOT within the actual authority given by Principal: Principal only authorized agent to make the purchase
for the benefit of Principal. Under the Third Restatement of Agency, an act that would be within the
agent’s actual authority had the agent acted to serve the interests of the principal falls outside actual

Seperac-J19 Exam-Released MEE Essay Compilation © 2016-2020 655


authority if the agent acted to serve the agent’s own purposes or other improper purposes. Thus, although
he purported to be acting for Principal, Agent probably lacked actual authority to make this purchase for
Greta, though he would have had authority to purchase for Principal. Consequently, Principal would
probably not be bound simply on the basis of actual authority.

Nevertheless, jurisdictions that conclude that Agent lacked actual authority would probably also conclude
that Agent had apparent authority to make this purchase. Apparent authority is created “by written or
spoken words or any other conduct of the principal which, reasonably interpreted, causes the third person
to believe that the principal consents to have the act done on his behalf by the person purporting to act for
him.” The principal must “be responsible for the information which comes to the mind of the third
person,” but this can occur if the principal provides “documents or other indicia of authority” to the agent
that are subsequently shown to the third party.

In this case, Principal provided Agent with written credentials which, when they were shown to Bellseller,
caused Bellseller to believe that Agent was authorized to purchase the bell on behalf of Principal. Agent
then purported to act within the scope of this apparent authority, and Bellseller believed him to be so
acting. Consequently, Principal is liable for the contract made by Agent within the scope of his apparent
authority.

In any event, the basic justification for imposing liability on Principal is simply that Principal created a
situation where Bellseller reasonably relied on Agent’s claims to be acting for Principal. Agency law
typically protects third parties who act in reliance on appearances created by a purported principal, and a
court is likely to protect Bellseller for this reason, whether the result is explained on grounds of apparent
authority, inherent authority, or estoppel.

Explanation to Point-Two (25-35%):

Agent acted with actual authority to purchase the book, and with apparent authority because Tomeseller
was unaware of Principal’s price limitation. Therefore, Principal is liable even though Agent exceeded the
terms of his authorization.

Principal specifically authorized Agent to buy the book, so Agent had actual authority to purchase the
book on Principal’s behalf. However, Agent exceeded the limitation on price imposed by Principal, and
therefore exceeded the scope of his actual authority. Although Agent is liable to Principal for breach of
duty to obey instructions, Principal will nevertheless be liable to Tomeseller.

Agent showed the written authorization to Tomeseller, and the purchase contract identified Principal as
the principal, so Principal was a disclosed principal, and Agent had apparent authority. Apparent authority
is created “by written or spoken words or any other conduct of the principal which, reasonably
interpreted, causes the third person to believe that the principal consents to have the act done on his behalf
by the person purporting to act for him.” When Agent presented Tomeseller with the written
authorization, which did not include the limitation, Tomeseller reasonably understood that Agent was
authorized to purchase the book on Principal’s behalf, creating apparent authority on behalf of Agent.

Because the price limitation was not included on the written authorization shown to Tomeseller, she was
unaware of the limitation. “A disclosed or partially disclosed principal authorizing an agent to make a
contract, but imposing upon him limitations as to incidental terms intended not to be revealed is subject to
liability upon a contract made in violation of such limitations with a third person who has no notice of
them.” Secret limitations on an agent’s authority do not bind third parties. Therefore, Principal remains
liable to Tomeseller for payment of the book.

Seperac-J19 Exam-Released MEE Essay Compilation © 2016-2020 656


Explanation to Point-Three (25-30%):

Because Agent was acting with apparent authority, even though Principal had terminated all actual
authority, Principal is liable to Lampseller for the purchase of the lamp.

Principal had terminated the agency relationship by delivery of the letter to Agent, so Agent did not have
actual authority to buy the lamp. Agent did, however, continue to have apparent authority. Once apparent
authority is created, it can be terminated only when the third party has notice of termination or when the
authority is terminated due to impossibility or lack of capacity.

When Principal engaged Agent, Principal gave Agent credentials and form contracts used to buy antiques
on Principal’s behalf. Agent showed Lampseller the credentials and used Principal’s form to purchase the
lamp, so Lampseller could reasonably have understood that Agent had authority from Principal to buy
antiques, including the lamp. When Principal entrusted Agent with “indicia of authority,” Principal
became obliged to give notice of any termination of that authority to third persons who relied upon
Agent’s possession of the credentials. Since Lampseller had no notice that Agent’s credentials were no
longer valid, the apparent authority continued as to Lampseller and, as a result, Principal is liable for
payment to Lampseller for the $5,000 purchase price.

Seperac-J19 Exam-Released MEE Essay Compilation © 2016-2020 657


#146-FEB 2001–MEE Q02: QUESTION TWO (AGENCY-PARTNERSHIP)

Three siblings, Andrew, Brenda, and Charles, are equal partners in ABC Partnership, a general
partnership, which owns and operates a 2,000-acre farm. ABC does not have a written partnership
agreement. The three partners meet periodically to discuss ABC’s business but do not hold formal
partnership meetings.

Andrew lives on the farm and manages its day-to-day operations. Neither Brenda nor Charles lives on the
farm. Brenda owns an accounting business in town and helps keep ABC’s books and records. Charles has
an irrigation business in town and helps maintain ABC’s irrigation system.

Andrew spent $10,000 to purchase a disease-resistant hybrid seed for the farm. Ordinary seed would have
cost $6,000. Andrew purchased the seed in ABC’s name, but the $7,000 down payment for the seed was
made using his own funds. Charles believes Andrew wasted money on this expensive seed because
disease has never been a problem for ABC’s farm. Charles is particularly concerned because the balance
of the purchase price ($3,000) is due in a month, and ABC does not have sufficient funds to pay the bill.
Brenda and Charles never authorized Andrew to buy the more expensive seed and did not ask him to
advance his own money for the down payment.

Andrew spends about twice as much time as his siblings conducting ABC business. Andrew has
demanded that ABC pay him for the value of his services, although there is no express agreement that any
of the partners should be compensated for their services.

Andrew entered into a written agreement with XYZ Farms to swap 500 acres of ABC cropland for 1,000
acres of woodland owned by XYZ that Andrew thinks ABC could divide and develop for a residential
subdivision. Charles disagrees with Andrew’s plan and is upset that any land would be sold since, in the
50 years that the farm has been operated by the partners’ family, no land has ever been transferred.
Andrew defends the swap saying, truthfully, that he and Brenda had agreed to the transaction after all
three partners had discussed it.

1. Is Charles liable for any part of the unpaid balance on the seed? Explain.

2. Is Andrew entitled to reimbursement from the partnership or the partners for the down payment
he made on the seed? Explain.

3. Is Andrew entitled to be paid for the value of all or part of his services to ABC? Explain.

4. Is the land swap agreement with XYZ Farms binding on ABC? Explain.

Seperac-J19 Exam-Released MEE Essay Compilation © 2016-2020 658


#146: F01-2 MEE: ANSWER: NCBE (AGENCY-PARTNERSHIP)

POINT (1) [25%] ISSUE: Is a general partner responsible for the debts of a general partnership if
the partnership has insufficient funds to meet the debt, when the partners did not expressly
authorize the debt? ANSWER: Yes. Partners are agents of a general partnership for the purpose of
binding the partnership and are responsible for the obligations of the partnership.

POINT (2) [25%] ISSUE: Is a general partner who advances his own funds to purchase goods for
the partnership entitled to reimbursement from the partnership? ANSWER: Yes. A partner who
uses his own funds to purchase goods for the partnership is entitled to reimbursement.

POINT (3) [20%] ISSUE: Is a general partner who contributes more services to the general
partnership than the other partners entitled to remuneration from the partnership? ANSWER:
Partners are not entitled to be paid for their services to a general partnership, unless the partners
have an express agreement to provide such payments, but some courts will allow remuneration
based on an implied agreement.

POINT (4) [30%] ISSUE: May a majority, but less than all, of the partners bind the general
partnership to a transaction outside the ordinary course of business without approval of all of the
partners? ANSWER: No. All partners must consent to the land swap by ABC Partnership in order
for it to be binding on ABC since the action is outside the ordinary course of ABC’s business.

ANSWER EXPLANATION:

Explanation to Point-One (20-30%):

Partners are agents of a general partnership for the purpose of binding the partnership and are responsible
for the obligations of the partnership.

Andrew purchased the special hybrid seed without obtaining the consent of the other partners. As a
general rule, partners are agents of the partnership, and, as such, their acts (including entering into
contracts) bind the partnership. “An act of a partner, including the execution of an instrument in the
partnership name, for apparently carrying on in the ordinary course the partnership business or business of
the kind carried on by the partnership, binds the partnership, unless the partner had no authority to act for
the partnership in the particular matter and the person with whom the partner was dealing knew or had
received a notification that the partner lacked authority.”

In this case, the special hybrid seed was purchased in the name of ABC Partnership. Purchasing seed for a
farm, including expensive disease-resistant seed, constitutes carrying on in the ordinary course of ABC’s
partnership business. Because Andrew has operated the farm on a day-to-day basis, the seed vendor could
properly assume that the purchase of the seed was for carrying on the ordinary business of the partnership
and that Andrew had authority to make the purchase. As a result, ABC is liable for payment of the seed.

Partners are jointly responsible for the debts of a general partnership. Under RUPA, partners are jointly
and severally liable for general partnership obligations.

Because the contract to purchase seed is an enforceable obligation of ABC, Charles is jointly (and, under
RUPA, severally) liable with Andrew and Brenda for the obligation.

Seperac-J19 Exam-Released MEE Essay Compilation © 2016-2020 659


Explanation to Point-Two (20-30%):

A partner who uses his own funds to purchase goods for the partnership is entitled to reimbursement.

A partner is entitled to be repaid by the partnership for contributions to partnership property made by the
partner individually. Since Andrew made the $7,000 down payment from his own funds as an advance to
the partnership, ABC is obligated to repay him that amount. Under RUPA, a partnership must reimburse a
partner for an advance to the partnership beyond the amount of capital the partner agreed to contribute.
Although the facts imply that ABC does not have sufficient funds, Andrew still has the right to be
reimbursed by the partnership. Since, as previously stated, each partner is liable for the debts of the
partnership, Charles and Brenda will also be personally liable for a portion of the down payment if the
partnership is unable to reimburse Andrew.

Explanation to Point-Three (15-25%):

Partners are not entitled to be paid for their services to a general partnership, unless the partners have an
express agreement to provide such payments, but some courts will allow remuneration based on an
implied agreement.

Andrew claims that he is entitled to be paid by ABC for his services to the partnership because he
contributed many more services than Brenda and Charles. As a general rule, a partner is not entitled to
separate remuneration for services on the theory that a partner’s compensation for his or her services is his
or her share of profits.

There are two exceptions to this rule. First, in the case of a winding up of the partnership, a surviving
partner is entitled to reasonable compensation for services rendered in connection with winding up the
business of the partnership. Since ABC is not winding up operations, this would not apply to Andrew’s
request.

The second exception is where the partners expressly agree to pay a partner for his or her efforts. The
UPA is explicit that the rights of a partner as stated in the Act, including the right to receive remuneration,
may be changed by an agreement among the partners. If partners want to pay salaries, they must agree to
do so. Here the partners had no express agreement to pay any remuneration to Andrew. On that basis
many courts would find that Andrew is not entitled to any compensation for his services to ABC.
However, other courts have permitted remuneration based on an implied agreement to compensate a
partner but there are no facts here to suggest an implied agreement.

Explanation to Point-Four (25-35%):

All partners must consent to the land swap by ABC Partnership in order for it to be binding on ABC since
the action is outside the ordinary course of ABC’s business.

As a general rule, matters outside the ordinary course of a partnership’s business must be unanimously
approved by the partners. Although the UPA and the RUPA do not define the types of acts that are
sufficiently outside the ordinary course of business to require the consent of all partners, the facts make
clear that ABC’s sale of 500 acres of farmland and purchase of land to be developed for a subdivision are
not within the scope of the ordinary course of business of its farm operations. While a conveyance of land
might be in the ordinary course of business when one partner had been given authority in the past to
convey pieces of land, this was not the case here.

Seperac-J19 Exam-Released MEE Essay Compilation © 2016-2020 660


Because the transaction was not in the ordinary course of ABC’s business and because it was not
unanimously approved by the partners, Andrew’s actions did not bind ABC. Should XYZ claim Andrew
had apparent authority to sell the land, that claim would fail because the transaction was not in the
ordinary course of ABC’s business. The nature of the transaction (i.e., not in the ordinary course of the
partnership’s business) put XYZ constructively on notice that Andrew alone might not have the authority
to engage in the transaction.

Seperac-J19 Exam-Released MEE Essay Compilation © 2016-2020 661


#147-FEB 2000–MEE Q02: QUESTION TWO (AGENCY-PARTNERSHIP)

Dwight L.P. is a properly formed limited partnership created to purchase, renovate, and sell the historic
Dwight Building in the downtown preservation district. The sole general partner of Dwight L.P. is Able,
Inc. The Dwight L.P. partnership agreement provides that it will continue its operations until such time as
the Dwight Building is renovated and sold.

Initially, Able estimated that it would cost $10 million to purchase and renovate the Dwight Building in a
way that would maintain its unique architectural elements. Able’s financial plan for the limited
partnership was to admit limited partners who would contribute a total of $1 million to the partnership and
then to obtain an additional $9 million in financing from a bank. Able found 10 local investors who
contributed $100,000 each to join the partnership as limited partners. Bank made a loan to Dwight L.P. of
$9 million, and Dwight L.P. bought the historic building and began work to renovate it.

Now, one year after formation of Dwight L.P., the Dwight Building renovation project has run into
financial difficulty. Able failed to take into account the special requirements that the city imposed on
materials and workmanship in the historic district and therefore seriously, but not fraudulently,
underestimated the renovation costs of the Dwight Building. As a result, the $10 million financing has
been spent, and the renovation is only three-fourths complete. The limited partners have retained a
reputable independent architectural consultant who has informed them that the project can be completed
for an additional $2.5 million and, upon completion, the building will probably be worth $13.5 million.

In the two weeks since the limited partners and Bank were informed of the financial situation, Bank, Able,
and the limited partners have had a series of meetings. Able wants to complete the project but has no
capital to contribute. Bank is unwilling to grant another loan to the partnership. Able is pressuring the
limited partners to make additional capital contributions to cover the costs of completion. None of the
limited partners wants to contribute any additional funds, and they are not obligated to do so under the
limited partnership agreement. Able has threatened to dissolve the limited partnership unless the limited
partners can come up with the additional funds required within the next 10 days.

Last week, Historical Society contacted Able and offered S10 million to Dwight L.P. to purchase the
Dwight Building in its present uncompleted state. Historical Society has full information about the
project, and its offer is firm. Because Able does not want to sell the building until it has been fully
restored as originally intended, Able has rejected the offer without notifying the limited partners either of
the offer or of Able’s rejection.

Recently, the limited partners found out about Historical Society’s offer and Able’s failure to inform them
of the offer. The limited partners believe they can no longer trust Able to run the partnership. They are
now united in their desire to take one or more of the following actions: (1) dissolve the limited
partnership; (2) push for the sale of the Dwight Building to Historical Society; (3) remove Able as general
partner and elect a new general partner; or (4) allow Able to continue as general partner but require
approval by the limited partners for every action Able takes.

1. Did Able have an obligation to inform the limited partners of Historical Society’s offer? Explain.

2. Can Able carry out its threat to dissolve the limited partnership, and, if so, what options, if any,
do the limited partners have as a result? Explain.

Seperac-J19 Exam-Released MEE Essay Compilation © 2016-2020 662


3. Can the limited partners take each of the four actions listed above without exposing themselves
to unlimited liability? Explain.

Seperac-J19 Exam-Released MEE Essay Compilation © 2016-2020 663


#147: F00-2 MEE: ANSWER: NCBE (AGENCY-PARTNERSHIP)

POINT (1) [33%] ISSUE: Does a general partner of a limited partnership have an obligation to
inform the limited partners of an offer to purchase the limited partnership’s sole asset? ANSWER:
Yes. As general partner. Able had an obligation to inform the limited partners of the offer from
Historical Society.

POINT (2) [33%] ISSUE: Can a general partner of a limited partnership dissolve the limited
partnership without obtaining the approval of the limited partners? If so, what are the limited
partners’ rights? ANSWER: Able can dissolve the limited partnership, but such a dissolution would
be wrongful. In a wrongful dissolution, the limited partners would wind up Dwight L.P. and could
sell the Dwight Building to Historical Society.

POINT (3) [33%] ISSUE: Can the limited partners take each of the desired actions without losing
their status as limited partners and subjecting themselves to unlimited liability? ANSWER: The
limited partners may dissolve the limited partnership, push for a sale to Historical Society, and
remove Able as general partner without the possibility of unlimited liability. The limited partners
may require their approval for all actions taken by the general partner, but, in doing so, they
increase their exposure to the possibility for unlimited liability.

ANSWER EXPLANATION:

Explanation to Point-One (33%):

As general partner. Able had an obligation to inform the limited partners of the offer from Historical
Society.

A general partner of a limited partnership has fiduciary duties to the limited partners of the partnership
similar to that which a partner of a general partnership has to the other general partners. These duties are
the duty of care and the duty of loyalty. Able’s failure to inform the limited partners of the offer from
Historical Society was a violation of its fiduciary obligations as general partner. First, if the limited
partners had a right under the partnership agreement to approve or disapprove the sale, the failure violated
the implied right to information under the Uniform Partnership Act. Second, to the extent the offer was
partnership property and Able would benefit from thwarting the sale, the withholding of information by
Able violated its fiduciary duties under In addition, Able’s failure to inform the limited partners of the
offer to purchase the partnership’s major asset was a violation of Able’s general common-law fiduciary
duty. Case law holds that partners owe one another the duty of the finest loyalty.

Explanation to Point-Two (33%):

Able can dissolve the limited partnership, but such a dissolution would be wrongful. In a wrongful
dissolution, the limited partners would wind up Dwight L.P. and could sell the Dwight Building to
Historical Society.

A nonjudicial dissolution of a limited partnership can occur at a time specified in the limited partnership
agreement, upon the happening of an event specified in the limited partnership agreement, or upon the
written consent of all partners. In this case, the facts do not specify that the Dwight L P. partnership
agreement provides for a time for dissolution. It does provide that on the sale of the Dwight Building, the

Seperac-J19 Exam-Released MEE Essay Compilation © 2016-2020 664


partnership would terminate; however, this event has not occurred nor has the unanimous consent of the
limited partners to the dissolution of the Dwight L.P. been obtained. Therefore, Able cannot properly
dissolve the limited partnership. The general partner can withdraw, but the limited partners have 90 days
in which to decide to continue the partnership. Those are the only means under the statute by which a
limited partnership can be dissolved as a matter of right.

In a general partnership, each partner has the ability to dissolve the partnership even if the dissolution is
not as a matter of right. Although there is no parallel provision under RULPA, that act does contemplate
an effective but wrongful dissolution by a general partner. Thus it appears that Able could dissolve the
limited partnership but that such a dissolution would be wrongful.

If Able wrongfully dissolves the limited partnership, the limited partners would have the right to wind up
the limited partnership’s affairs. “The general partners who have not wrongfully dissolved a limited
partnership or, if none, the limited partners, may wind up the limited partnership’s affairs. The winding up
of a limited partnership is no different than the process of winding up a general partnership. Winding up is
generally the process of completing old business of the partnership and distributing the partnership’s
assets to the partners. Part of the winding up process is the conversion of the partnership’s assets into cash
for distribution to the partners. As part of the winding up of the partnership, the limited partners would
have the right to sell the Dwight Building to Historical Society.

Explanation to Point-Three (33%):

The limited partners may dissolve the limited partnership, push for a sale to Historical Society, and
remove Able as general partner without the possibility of unlimited liability. The limited partners may
require their approval for all actions taken by the general partner, but, in doing so, they increase their
exposure to the possibility for unlimited liability.

A limited partner who is not also a general partner can be held liable for the obligations of a limited
partnership if the limited partner “participates in the control of the business.” However, the Act provides a
safe harbor for certain types of activities that are deemed not to constitute participation in the control of
the business. Action by the limited partners to dissolve the limited partnership, to sell all or substantially
all of the limited partnership’s assets, or to remove the general partner are all within the safe harbor and
therefore those actions by themselves will not expose the limited partners to any unlimited general
partner-type liability.

A decision by the limited partners to keep Able as general partner but to require their approval for every
action Able takes is not within the safe harbor provisions of the statute. It is likely that such oversight of
all of the day-to-day decision-making of a general partner in its operation of the limited partnership would
constitute participation in the control of the business.

However, even if such oversight was deemed to constitute participation in control of the partnership, the
limited partners would not be automatically liable as general partners. Reliance by a third party is required
in order to impose greater liability upon a limited partner. “If the limited partner participates in the control
of the business, he is liable only to persons who transact business with the limited partnership reasonably
believing, based upon the limited partner’s conduct, that the limited partner is a general partner.” Here,
Bank knew of the limited partners’ status; thus, there was no reliance by Bank. If the limited partnership
were to continue its operations, it would be necessary to contract with various third parties to complete the
restoration work. It would be likely that if Able was required, in each instance, to defer to the approval of
the limited partners in the partnership’s dealings with such third parties, they could have a reasonable
belief that the limited partners were controlling the business and acting as a general partner.

Seperac-J19 Exam-Released MEE Essay Compilation © 2016-2020 665


#148-JUL 1999–MEE Q02: QUESTION TWO (AGENCY-PARTNERSHIP)

Susan is a talented furniture restorer. Benjamin is a wealthy furniture collector who enjoys buying and
selling furniture. Benjamin and Susan agreed to form what they called a “limited partnership.” Benjamin
would be the limited partner, would contribute $50,000 for working capital, and would contribute an
additional $100,000 if needed. Susan would be the general partner, with full authority to buy, restore, and
sell furniture, using her own well-equipped workshop and custom tools. They would split the profits as
follows: 70% to Benjamin and 30% to Susan until Benjamin received an amount equal to his $50,000
capital contribution plus interest at 11%. Thereafter, the profits would be split 50%-50%.

Benjamin and Susan did not put their agreement in writing or take any other action to formalize it.
Benjamin gave Susan $50,000 for the business, and Susan purchased, restored, and sold furniture. After
six months of allocating distributions under the 70%-30% formula, Benjamin was repaid his total $50,000
capital contribution and accrued interest. Thereafter, profits were split 50%-50%.

The business has now become so successful that Susan will not need any additional capital from
Benjamin. The business has $8,500 in its bank account and has no outstanding debts.

Susan has told Benjamin that she wants to end their business relationship. Benjamin claims that Susan has
no right to dissolve the business just as it begins to produce profits. He further claims that if Susan does
end the relationship, she must pay him 50% of the cash in the bank account, his share of the goodwill, and
an amount equal to 50% of the value of her tools. The tools, which were Susan’s before she entered into
the agreement with Benjamin, are worth $50,000, and Benjamin asserts that they are assets of the
business. Susan denies that Benjamin is entitled to a share of the goodwill, if any, or to 50% of the value
of the tools.

1. Was Benjamin and Susan’s business relationship a general partnership, a limited partnership, or
neither? Explain.

2. Does Susan have the right to end her business relationship with Benjamin? Explain.

3. Assuming Susan has the right to end the business relationship, how should Benjamin’s claims be
resolved? Explain.

Seperac-J19 Exam-Released MEE Essay Compilation © 2016-2020 666


#148: J99-2 MEE: ANSWER: NCBE (AGENCY-PARTNERSHIP)

POINT (1) [46%] ISSUE: Was either a limited partnership or a general partnership formed by the
oral agreement between Benjamin and Susan? ANSWER: A general partnership was created by
Benjamin and Susan’s oral agreement.

POINT (2) [8%] ISSUE: Does Susan have the legal right to terminate the relationship with
Benjamin? ANSWER: Yes. Susan has the legal right to cause the dissolution of the partnership.

POINT (3) [46%] ISSUE: What elements of the partnership is Benjamin entitled to a share in if
Susan terminates the relationship? ANSWER: After dissolution, Benjamin is entitled to a
distribution of partnership assets, which do not include Susan’s tools but probably do include his
share of the goodwill.

ANSWER EXPLANATION:

Explanation to Point-One (40-50%):

A general partnership was created by Benjamin and Susan’s oral agreement.

In order to form a limited partnership, a written certificate of limited partnership must be executed and
filed with the Secretary of State. Since the relationship between Benjamin and Susan was not
memorialized in writing, no limited partnership was formed. The subjective belief of the parties that they
had formed a limited partnership is not relevant.

The relationship that Benjamin and Susan formed was a general partnership. Unlike a limited partnership,
there is no requirement that a general partnership agreement be in writing. Rather, the creation of a
partnership de jure is a function of the intent of the putative partners to establish “an association of two or
more persons to carry on as co-owners a business for profit.” Benjamin and Susan intended to operate
their furniture restoration business as co-owners for profit, even though they were legally mistaken in
calling the venture a limited partnership. Moreover, under UPA § 7 “the receipt by a person of a share of
the profits of a business is prima facie evidence that he is a partner in the business.” Benjamin and Susan
divided the profits 70% to Benjamin - 30% to Susan until Benjamin received his capital contribution plus
interest and then 50%-50% thereafter.

Explanation to Point-Two (05-10%):

Susan has the legal right to cause the dissolution of the partnership.

Susan has the right to dissolve the partnership because a partner always has such a right. However, a
partner’s act of dissolution may be either a wrongful dissolution or a permitted action. Where the
partnership agreement does not specify a definite term or a specified objective for the partnership to
achieve during its operations, then dissolution does not contravene the partnership agreement. A
partnership has been established for a particular undertaking if it has a stated purpose that can be achieved
at a recognizable time, even if that time could not have been predicted when the partnership was
established.

Seperac-J19 Exam-Released MEE Essay Compilation © 2016-2020 667


The agreement between Benjamin and Susan does not specify any term. Although the purpose of the
partnership was to restore and sell furniture, this is a continuous activity that does not constitute a
particular undertaking. As a result, the partnership was a partnership at will. A partner is not required to
have a reason to dissolve a partnership at will. Therefore, Susan has not only the power but also the legal
right to dissolve the partnership.

Explanation to Point-Three (40-50%):

After dissolution, Benjamin is entitled to a distribution of partnership assets, which do not include Susan’s
tools but probably do include his share of the goodwill.

When dissolution of a partnership is not wrongful or in contravention of the partnership agreement, the
partnership property is applied to discharge the partnership’s liabilities, and the surplus is paid to the
partners. Here there are no partnership liabilities, so the issue is post-dissolution distributions under UPA
§ 40 or RUPA § 807. There being no contrary agreement of the parties, the partnership assets are to be
distributed under UPA § 40. There are no outside or inside creditors, and Benjamin has already been
reimbursed for his $50,000 capital contribution plus interest, so the partnership assets should be
distributed as profits 50% to Benjamin and 50% to Susan.

The issue is what assets of the business constitute partnership assets available for distribution to the
partners. Under the UPA “all property originally brought into the partnership stock or subsequently
acquired on account of the partnership, is partnership property.” The cash on hand, earned by the
partnership from its operations, is clearly a partnership asset.

The facts state that Susan would use her own well-equipped workshop and custom tools for the
partnership’s operations. This indicates that Susan did not contribute ownership of Susan’s tools to the
partnership, but instead only made them available for use by the partnership. An argument that the tools
were a contribution analogous to Benjamin’s $50,000 capital contribution finds some support in the
equivalent amounts involved. However, Benjamin’s capital contribution was repaid with interest, and
there was no similar provision with respect to the tools. Nor is there any evidence that the parties
themselves treated the tools as partnership property. On balance, the tools are not partnership property.

Even if the tools are considered to be partnership property, their contribution to the partnership was a
“capital contribution” by Susan. Under UPA § 40, Susan is entitled to a return of Susan’s “capital
contribution” before either partner is entitled to a distribution of profits under UPA § 40.

Benjamin has argued that, in addition to the tangible assets of the partnership, he is entitled also to an
accounting for the intangible goodwill value of the business. “Goodwill” is the value of the business as a
going concern over and above the aggregate value of its tangible property. Benjamin will argue that a
business that generates such substantial profits on relatively small tangible assets has developed
substantial “goodwill” or “going concern” value. Susan apparently intends to continue the business
without Benjamin, thereby appropriating the value of the goodwill for herself, without compensation to
Benjamin.

When a partnership is wrongfully terminated, the partner who has caused the wrongful dissolution is not
entitled to an accounting for the value of the goodwill of the business if the business is continued. By
implication, where the partner seeking such an accounting has done nothing wrong, he is entitled to
compensation for the value of the goodwill of the business where it is to be continued by the other
partner. The RUPA does not address accounting for goodwill of the business and there have been no
reported cases interpreting the RUPA’s approach to allocating the issue of goodwill among partners.

Seperac-J19 Exam-Released MEE Essay Compilation © 2016-2020 668


Susan will argue, on the other hand, that because she is a professional, there is no goodwill attributable to
the business itself, but rather any “going concern” value is attributable to her own reputation and
individual skill. The reputation of a professional partnership, such as partnerships of attorneys, physicians,
or dentists, depends upon the individual skills of the members, and thus, it could be argued that there is no
goodwill to be distributed as an asset upon the dissolution of such a partnership. For example, where
goodwill is intrinsically tied to the attributes and/or skills of certain individuals, it does not survive the
disassociation of those individuals from the business. Susan will also argue that in continuing the business
she is simply exercising her right to pursue the same business after dissolution; partnership law does not
imply any covenant to compete or to engage in the same business after dissolution of the partnership, and
here there is no explicit agreement to the contrary. Had Benjamin desired to protect himself from the
possibility that Susan would terminate the partnership once the business became profitable, insufficiently
recognizing the value of Benjamin’s start-up capital, Benjamin should have insisted that the partnership
agreement be in writing and for a specific term of years.

Further, Susan will argue that under the terms of the oral partnership agreement, which Susan complied
with, Benjamin’s capital investment has been repaid in full, with interest, and Benjamin has no further
legitimate claims upon the future earnings of Susan’s personal efforts.

The likely outcome is that Benjamin is entitled to receive one-half of the $8,500 in the partnership bank
account, and depending on the conclusion reached from the foregoing discussion, one-half of any
goodwill of the partnership.

Seperac-J19 Exam-Released MEE Essay Compilation © 2016-2020 669


#149-FEB 1999–MEE Q02: QUESTION TWO (AGENCY-PARTNERSHIP)

In 1992, XYZ limited partnership was formed in Central State by the proper filing of all necessary
documents. XYZ’s limited partnership agreement provided that it would engage in the purchase and sale
of rare uncut blue diamonds. The partnership agreement further provided that the agreement could be
amended only by the affirmative vote of 60% of the limited partnership interests.

XYZ has one general partner, GemCorp (a properly organized corporation), and three limited partners,
individuals named Bill, David, and Emily. The three limited partners hold equal percentages of XYZ
limited partnership interests. GemCorp is wholly owned by Bill, who is GemCorp’s only officer and
director. Bill’s ownership of GemCorp has been disclosed to David and Emily. All corporate formalities
have been complied with in the operation of GemCorp.

In addition to being the general partner of XYZ, GemCorp operates a number of other businesses
including one that purchases and sells rare gemstones. XYZ’s limited partnership agreement includes a
provision permitting GemCorp to engage in its other businesses without offering any business
opportunities to XYZ. However, it is silent about whether GemCorp can enter into business transactions
with XYZ.

From 1992 until January 1996, XYZ built a successful diamond business and was a profitable and
harmonious enterprise for all of the partners.

In 1992, GemCorp purchased three virtually identical lots of uncut rare blue diamonds for $ 1 million per
lot from a well-known international dealer. In 1993, GemCorp sold one of the lots to an unrelated third
party for $2 million. In 1994, GemCorp sold the second lot for $3 million to another unrelated third party.
During 1995, vast quantities of uncut blue diamonds began to flood the market. By January 1996, the
diamond market was in chaos and the price of uncut blue diamonds had dropped by 90%.

Shortly thereafter, GemCorp sold the third lot of uncut blue diamonds to XYZ for $5 million.

Central State has adopted the Revised Uniform Limited Partnership Act (1976) with the 1985
amendments (the RULPA) and the Uniform Partnership Act (UP A).

1. On what basis, if any, could the XYZ limited partners challenge the actions of GemCorp in
selling the blue diamonds to XYZ? Explain.

2. Are the claims of the XYZ limited partners against GemCorp derivative or direct, and what
procedural steps, if any, must they take prior to bringing a claim? Explain.

3. What actions, if any, could GemCorp have taken before the sale to eliminate any potential
liability it might have to the XYZ limited partners? Explain.

4. On what basis, if any, could Bill be found individually liable to the other limited partners for the
sale of the blue diamonds to the partnership? Explain.

Seperac-J19 Exam-Released MEE Essay Compilation © 2016-2020 670


#149: F99-2 MEE: ANSWER: NCBE (AGENCY-PARTNERSHIP)

POINT (1) [39%] ISSUE: Has a general partner of a limited partnership violated its duty of loyalty
to the limited partners by entering into an interested transaction between itself and the limited
partnership on terms less advantageous to the partnership than an arm’s-length transaction would
have been? ANSWER: Yes. GemCorp’s sale of the third lot of uncut blue diamonds was an
interested transaction and a breach of GemCorp’s duty of loyalty to the limited partners of XYZ
because the price paid to GemCorp by XYZ was unfair.

POINT (2) [17%] ISSUE: Is the limited partners’ claim of a general partner’s breach of loyalty a
derivative or a direct claim and, if derivative, what procedures must they follow to pursue that
claim? ANSWER: The limited partners’ potential claim against GemCorp for breach of fiduciary
duty is a derivative claim and subject to the procedural requirements for derivative claims, such as
a demand on the general partner.

POINT (3) [28%] ISSUE: Before the interested transaction occurs, can a general partner shield
itself from liability by convincing the limited partners to amend the limited partnership agreement
to expressly permit it to engage in interested transactions with XYZ by making full disclosure about
the potential sale and obtaining the consent of the disinterested limited partners? ANSWER: Yes.
If GemCorp had made full disclosure to the disinterested limited partners of all the relevant details
concerning its sale to XYZ of the third lot of diamonds before the sale, then GemCorp would not
have been liable for the transaction with XYZ.

POINT (4) [17%] ISSUE: Is an individual who is the sole officer, director, and shareholder of a
corporate general partner also deemed to be a general partner of XYZ and therefore liable for any
actions taken by the corporate general partner? ANSWER: Bill would not be liable merely due to
his position as sole shareholder, director, and officer of GemCorp, but may be liable because he
caused GemCorp to breach its fiduciary duties to XYZ.

ANSWER EXPLANATION:

Explanation to Point-One (30-40%):

GemCorp’s sale of the third lot of uncut blue diamonds was an interested transaction and a breach of
GemCorp’s duty of loyalty to the limited partners of XYZ because the price paid to GemCorp by XYZ
was unfair.

A general partner of a limited partnership has a fiduciary duty to the limited partners of the partnership
similar to that which a partner of a general partnership has to the other general partners. This fiduciary
duty includes a duty of loyalty that historically is measured at a high standard. Under the UPA, a partner
is accountable as a fiduciary to other partners. Generally, where a partner has a conflict of interest, he is
obligated to resolve the conflict in favor of the partnership or to fully disclose the conflict and get the
consent of the other partners.

In this case, all partners in XYZ knew when the limited partnership was formed that GemCorp was in the
business of buying and selling rare gemstones. The partners agreed in the partnership agreement that
GemCorp would have the right to continue its gem business without offering XYZ any business

Seperac-J19 Exam-Released MEE Essay Compilation © 2016-2020 671


opportunities. Therefore, GemCorp’s purchase of the three lots of diamonds and its sale of the first two
lots to unrelated third parties was not a breach of its fiduciary duties to XYZ.

However, since the XYZ limited partnership agreement did not expressly permit GemCorp to engage in
business transactions with XYZ, GemCorp had a conflict of interest between its duties to XYZ and the
limited partners and its own self-interest when it sold the third lot of diamonds to XYZ.

David and Emily, as limited partners, could challenge the diamond sale to XYZ on the ground that it was
a breach of GemCorp’s duty of loyalty. They would argue that it was an interested transaction and the
terms of the sale were unfair. Since Bill controls GemCorp, which is accused of the wrongdoing, Bill
could not join with the other limited partners in the challenge. GemCorp will have the burden of
demonstrating that the terms of the transaction are fair, such as would be negotiated by the parties to an
arm’s-length transaction, unless it has obtained prior consent of the other limited partners. In this
situation, no grounds are provided which would support a defense by GemCorp that the terms of the sale
were fair to XYZ.

Explanation to Point-Two (10-20%):

The limited partners’ potential claim against GemCorp for breach of fiduciary duty is a derivative claim
and subject to the procedural requirements for derivative claims, such as a demand on the general partner.

The limited partners’ claim is a derivative claim because the loss they suffer is derived from any loss
suffered by XYZ. If GemCorp has overcharged XYZ for the third lot of diamonds, this will reduce the
value of the assets held by XYZ. Injury to the limited partners would be due to the resulting decrease in
the value of their interests in XYZ.

A limited partner has the right to bring a derivative claim. However, the limited partner must be a partner
both at the time of the conduct that is being challenged as well as at the time it files the lawsuit. Further,
the limited partner must plead with particularity that it has made demand on the general partner to bring
the action or that the “effort to cause the general partners to bring the action is not likely to succeed.”
Here, the limited partners are partners at the time the suit is brought and were partners when XYZ
purchased the lot of blue diamonds from GemCorp. In addition, the limited partners can probably satisfy
this procedural requirement if they plead demand futility, because the claim arises from illegal actions by
the general partner.

Explanation to Point-Three (20-30%):

If GemCorp had made full disclosure to the disinterested limited partners of all the relevant details
concerning its sale to XYZ of the third lot of diamonds before the sale, then GemCorp would not have
been liable for the transaction with XYZ.

GemCorp’s sale of the blue diamonds to XYZ was not per se improper. Unless limited by the partnership
agreement, a partner has the same rights to transact business with the limited partnership as any person
who is not a partner. The XYZ limited partnership agreement is silent on whether GemCorp may engage
in business transactions with XYZ. The absence of an express provision would permit GemCorp to
engage in transactions with XYZ. However, GemCorp’s fiduciary duties to XYZ require that those
transactions be on arm’s-length terms. Since the terms of GemCorp’s sale to XYZ were much more
favorable to GemCorp than GemCorp would have received in an arm’s-length transaction, the transaction
was improper.

Seperac-J19 Exam-Released MEE Essay Compilation © 2016-2020 672


However, GemCorp could have eliminated its liability from the sale to XYZ of the blue diamonds for
such a high price if, prior to the sale, it had disclosed the terms of the sale to the limited partners and
obtained their consent either (1) to amend the limited partnership agreement to allow GemCorp to engage
in transactions with XYZ on terms more favorable to GemCorp than it could have obtained in an arm’s-
length transaction; or (2) to consent to this particular transaction between GemCorp and XYZ on more
favorable terms. In either case, all of the partners would have been put on notice of GemCorp’s right and
sale by GemCorp to XYZ would have been proper. (Note that a vote by the limited partners on a
transaction between the limited partnership and its general partner does not expose the limited partners to
any risk that they are controlling the business of the partnership.

The limited partnership agreement provides that it can be amended by a 60% vote of the limited partners.
Bill is interested in the contemplated transaction and therefore should abstain from voting to amend the
limited partnership agreement. The other limited partners have the power to approve the proposed change.
However, if GemCorp wants to avoid later charges by the limited partners that they were deceived into
voting in favor of the amendment, it would need to make full disclosure to the limited partners of the
terms of the sale by GemCorp to XYZ of the third lot of uncut diamonds before the transaction was
entered into.

Explanation to Point-Four (10-20%):

Bill would not be liable merely due to his position as sole shareholder, director, and officer of GemCorp,
but may be liable because he caused GemCorp to breach its fiduciary duties to XYZ.

Bill is the sole owner of, and controls, GemCorp, the general partner of XYZ. As such, Bill effectively
controls the business of XYZ in his capacity as corporate officer and shareholder of the general partner.

In cases relating to claims of third parties, in contrast to claims of the limited partners, courts have split on
whether such control over the general partner will transform the liability of a limited partner into the
unlimited liability of a general partner. However, RULPA expressly states that, with respect to claims by
third parties, a limited partner will not be subject to the unlimited liability of a general partner solely by
being an officer, director, or shareholder of a corporate general partner of the limited partnership. The
statute does not address whether this result would change if the claim were made by a limited partner
rather than a third party. However, unless the limited partners were unaware of the ownership and control
by a limited partner over the general partner, the better reading would follow the same rule as for third
party claims under RULPA § 303, and the related cases. Since the other XYZ limited partners knew of the
relationship between Bill and GemCorp, Bill should not have any personal liability as a general partner of
XYZ by reason of his ownership and control of GemCorp.

It is possible to argue that Bill would be personally liable on the theory that an officer of a corporation
who commits a tort is personally liable to injured third parties. In this case, Bill was the corporate officer
who caused GemCorp to breach its fiduciary duties to XYZ limited partnership.

Seperac-J19 Exam-Released MEE Essay Compilation © 2016-2020 673


#150-JUL 1998–MEE Q02: QUESTION TWO (AGENCY-PARTNERSHIP/PROPERTY)

Sam, Jake, and Bill are partners in a dry cleaning business. At the time the partnership was created, the
three executed a written partnership agreement. The agreement recites that the partnership is being formed
for the purpose of operating a dry cleaning business. The partnership agreement also states that each
partner has an equal voice in the partnership’s operations, will receive an equal share of any partnership
profits, and will bear an equal share of any partnership losses. The agreement does not contain any
provisions relating to the dissolution of the partnership and is silent on the duration of the partnership.

Lately the partnership has been losing money, and relations among Sam, Jake, and Bill have deteriorated.
Bill believes that the business judgment of Sam and Jake is poor.

On April 15, the partnership contracted with Hotel to dry-clean drapes from Hotel’s guest rooms under a
phased delivery schedule that would last until June 30. The partnership expects to make a profit on this
contract, payment for which is to be made by Hotel on July 15. In the meantime, the partnership has to
pay the cost of performing the contract out of current operating capital.

On May 1, Bill sent separate letters to Sam and Jake stating that Bill was “ending” the partnership. The
letters stated that Bill would no longer be bound by any actions of Sam and Jake, demanded that Sam and
Jake immediately cease transacting any business whatsoever on behalf of the partnership, and stated that
Bill would not be responsible for any expenses incurred or losses suffered by the business after May 5.

1. What is the effect of Bill’s May 1 letters on the ability of the partnership to continue to conduct
business, on the allocation of its future profits, and on Bill’s continuing liability for expenses or
losses? Explain.

2. Is Bill liable to Sam and Jake for wrongful dissolution of the partnership? Explain.

Seperac-J19 Exam-Released MEE Essay Compilation © 2016-2020 674


#150: J98-2 MEE: ANSWER: NCBE (AGENCY-PARTNERSHIP/PROPERTY)

POINT (1) [60%] ISSUE: What effect do Bill’s letters have, if any, on continuation of the
partnership business and allocation of expenses, losses and profits? ANSWER: Bill’s May 1 letters
did not terminate the partnership, although the letters dissolved the partnership of Sam, Jake, and
Bill. The partnership will not terminate until the winding-up of the partnership is completed. Old
business must be completed, with expenses, losses and profits shared as per the partnership
agreement. New business may not be undertaken by the partnership.

POINT (2) [40%] ISSUE: Is Bill liable in damages to Sam and Jake for sending the May 1 letters?
ANSWER: No. Bill is not liable to Sam and Jake for dissolving the partnership because the
partnership was a partnership at will. Dissolution of a partnership at will is not wrongful.

ANSWER EXPLANATION:

Explanation to Point-One (55-65%):

Bill’s May 1 letters did not terminate the partnership, although the letters dissolved the partnership of
Sam, Jake, and Bill. The partnership will not terminate until the winding-up of the partnership is
completed. Old business must be completed, with expenses, losses and profits shared as per the
partnership agreement. New business may not be undertaken by the partnership.

Under the Uniform Partnership Act (UPA), a partner has the power to dissolve a partnership at any time.
Dissolution of a partnership is defined by the UPA as “the change in the relation of the partners caused by
any partner ceasing to be associated in the carrying on as distinguished from the winding-up of the
business.” The partnership is not terminated by the dissolution, but rather “continues until the winding-up
of partnership affairs is completed.” Thus, the terms “dissolution” and “termination,” as used in the UPA,
are not synonyms. Under the UPA, winding-up (sometimes referred to as liquidation of the partnership) is
the process that normally occurs between the dissolution and the termination of a partnership.

Under the above principles, Bill’s May 1 letters did not terminate the partnership, even though they
purported to end it. Instead, the letters dissolved the partnership of Sam, Jake and Bill. As a result, Bill
was no longer associated with Sam and Jake in the carrying on of the partnership’s dry cleaning business.
The letters did not terminate the partnership because after the dissolution it was still necessary to wind up
the partnership affairs. The partnership will terminate at the completion of the winding-up process.
Moreover, Bill will receive his share of the profits from the Hotel contract, and he will also remain liable
for any expenses or losses incurred in the winding-up process, including the expenses of carrying out the
Hotel contract (as opposed to losses resulting from new business), even if the losses are incurred after
May 5.

Sam and Jake have no authority to enter into new business on behalf of the partnership after dissolution of
the partnership by Bill. However, Sam and Jake have authority to wind up the partnership by finishing
uncompleted business, including completion of the contract with Hotel. In addition, they have authority to
do any acts necessary to wind up the partnership business. Acts that may be necessary for the winding-up
of the partnership business include collecting debts owed to the partnership, paying obligations owed by
the partnership, liquidating partnership assets, and settling accounts between the partners.

Seperac-J19 Exam-Released MEE Essay Compilation © 2016-2020 675


The right of Sam and Jake to wind up the partnership is not affected by Bill’s May 1 letters demanding
that Sam and Jake cease transacting any business on behalf of the partnership. Bill does not have the
unilateral power to prevent Sam and Jake from completing unfinished business and winding up
partnership affairs. Unless there is a different agreement between the parties, Sam and Jake have a right to
wind up partnership affairs.

Explanation to Point-Two (35-45%):

Bill is not liable to Sam and Jake for dissolving the partnership because the partnership was a partnership
at will. Dissolution of a partnership at will is not wrongful.

A partner always has the power to dissolve the partnership. However, under the UPA a partner is liable
for damages if he or she dissolves a partnership in contravention of the partnership agreement. For
example, if the partnership agreement specifies a term of duration or a particular undertaking for the
partnership, a dissolution before the expiration of the specified term or the accomplishment of the
particular undertaking contravenes the partnership agreement. In those situations the partner has the
power but not the legal right to dissolve the partnership.

A dissolution of a partnership does not contravene the agreement between the partners where the
agreement does not specify a definite term or a particular undertaking for the partnership. Such a
partnership is often referred to as a partnership at will. The law is clear that a partner is not required to
have cause to dissolve a partnership at will. A partner in a partnership at will has both the power and the
legal right to dissolve the partnership.

The partnership agreement between Sam, Jake, and Bill is a partnership at will. The partnership
agreement does not specify a term for the partnership and the partnership is not for a particular
undertaking. While the partnership agreement states that it is for the purpose of operating a dry cleaning
business, this is insufficient to make the partnership one for a particular undertaking. A partnership for a
particular undertaking is one which has a purpose which is capable of accomplishment at some time, even
if the exact time is unknown or is not ascertainable at the time of the agreement.

Since the partnership is at will, Bill has not only the power to dissolve the partnership but also the legal
right to do so. Bill does not need a justification for dissolving the partnership. Because the dissolution is
not wrongful, i.e., in contravention of the partnership agreement, Bill is not liable in damages for
dissolving the partnership.

Seperac-J19 Exam-Released MEE Essay Compilation © 2016-2020 676


#151-FEB 1998–MEE Q01: QUESTION ONE (AGENCY-PARTNERSHIP)

Ashley, Bill, and Diane are equal partners in ABD Partnership, which manufactures computer
components, including keyboards. Ashley supervises the factory, Bill supervises the sales force, and
Diane is responsible for the financial management of the business.

Ashley and Diane are concerned about Bill’s dealings with XYZ Computer Components, the
partnership’s most direct competitor. Two years ago, ABD Partnership wanted to sell some used
manufacturing equipment valued at $250,000. ABD had an established policy of not selling used
equipment to its competitors. Without the knowledge or consent of Ashley and Diane, Bill sold the
equipment to XYZ for $275,000. Acquisition of the ABD equipment enabled XYZ to bid successfully
against ABD for a profitable government contract. When Ashley and Diane learned of the sale, they
became very angry and confronted Bill. Bill claimed he believed that the premium price paid by XYZ
justified the sale.

Several months later, Bill developed an improved keyboard configuration for disabled computer users
who suffer from limited hand and arm mobility. Bill had developed the configuration to accommodate his
disabled child. He worked on the keyboard at home in his spare time, without using any of ABD’s
facilities. Bill obtained a patent on the special keyboard in his own name, and then he entered into a
partnership with XYZ, called B-XYZ Partnership, to manufacture it.

Ashley and Diane recently learned of Bill’s patent for the special keyboard. They criticize him for
obtaining the patent in his own name and for forming the partnership with XYZ. Bill correctly points out
that he developed the keyboard on his own time and that XYZ is in the business of manufacturing
computers to accommodate people with disabilities, while ABD is not in that line of business.

1. Did Bill violate any duties to ABD Partnership by selling the surplus equipment to XYZ?
Explain.

2. What rights, if any, does ABD Partnership have in the patent for the special keyboard obtained
by Bill? Explain.

3. Did Bill breach any duties to ABD Partnership by entering into B-XYZ Partnership, and, if so,
must he account to ABD Partnership for any portion of the profits received by B-XYZ Partnership
derived from the sale of the computer keyboards designed to accommodate the disabled? Explain.

Seperac-J19 Exam-Released MEE Essay Compilation © 2016-2020 677


#151: F98-1 MEE: ANSWER: NCBE (AGENCY-PARTNERSHIP)

POINT (1) [39%] ISSUE: Did Bill breach his duty of care or fiduciary obligations to his partners in
ABD Partnership by selling the used manufacturing equipment to a competitor without the
partners’ knowledge or consent and in contravention of the partnership’s established policy?
ANSWER: Yes. Bill owes ABD Partnership a duty of care and a duty of loyalty under the RUPA
and has fiduciary obligations to ABD Partnership under the UPA. Bill violated his duty of care and
fiduciary obligations by selling the used assets of ABD Partnership to a competitor, in violation of
partnership policy.

POINT (2) [17%] ISSUE: Did Bill breach his duty of loyalty or fiduciary obligations to ABD
Partnership by patenting the invention in his own name? ANSWER: No. Bill did not breach his
duty of loyalty to ABD Partnership by patenting the invention in his own name.

POINT (3) [44%] ISSUE: Did Bill breach his duty of loyalty or fiduciary obligations to ABD
Partnership by entering into a partnership with a competitor of ABD Partnership? ANSWER: Yes.
Bill did breach his duty of loyalty by competing with ABD Partnership through his participation in
B-XYZ Partnership, and he must account to ABD for the profits derived from the sale of the
keyboards for the disabled.

ANSWER EXPLANATION:

Explanation to Point-One (35-45%):

Bill owes ABD Partnership a duty of care and a duty of loyalty under the RUPA and has fiduciary
obligations to ABD Partnership under the UPA. Bill violated his duty of care and fiduciary obligations by
selling the used assets of ABD Partnership to a competitor, in violation of partnership policy.

At common law, partners’ duties, inter se, are quite strict. According to Justice Cardozo in the frequently
quoted case of Meinhard v. Salmon, “Copartners owe to one another the duty of finest loyalty.” “Not
honesty alone, but the punctilio of an honor the most sensitive, is then the standard of behavior.”

The duties of partners to each other have been recognized by statute as well, although slightly different
standards are applied under the two most important statutes, the Uniform Partnership Act (UPA) and the
Revised Uniform Partnership Act (RUPA). The RUPA expressly recognizes that partners have duties to
each other, stating that the “fiduciary duties a partner owes to the partnership and the other partners are
the duty of loyalty and the duty of care.” The duty of care is somewhat limited, however: A partner owes
a duty of care to the partnership and the other partners to act in the conduct of the business of the
partnership in a manner that does not constitute gross negligence or willful misconduct. An error in
judgment or a failure to use ordinary skill and care is not gross negligence.

Unlike the RUPA, the UPA does not explicitly set out the fiduciary duties of partners toward one another.
However, the fiduciary duties of care and loyalty are implied under the UPA because partners are agents
of the partnership. According to the UPA, “Each partner is an agent of the partnership for the purpose of
its business.” Agents owe a duty of care and a duty of loyalty to their principals. Under § 104 of the
RUPA, principles of the law of agency apply to partnerships.

Seperac-J19 Exam-Released MEE Essay Compilation © 2016-2020 678


In this case, Bill has violated the duty of care for two reasons. First, Bill sold the equipment without the
consent of the other partners. The RUPA provides that “each partner has equal rights to management.”
Further, if there is a difference between partners regarding an act that is within the ordinary course of
business, the difference must be decided by a majority of the partners, and any disagreement regarding
any act outside the ordinary course of business must be resolved by agreement among all of the partners.
Regardless of whether the sale of the equipment to XYZ is considered to have been in the ordinary course
of business, failure to consult the other partners is likely to be considered willful misconduct. Second, Bill
violated the established policy of the partnership against selling used equipment to a close competitor.
Agents (and, as agents, partners) have a duty to act only as authorized.

Bill may claim that his actions amount to nothing more than “an error in judgment or a failure to use
ordinary skill and care,” and accordingly do not violate the duty of care. But Bill’s flagrant violation of
partnership policy and failure to consult with the other partners is the sort of behavior that a court is likely
to regard as willful misconduct and a violation of the duty of care.

The UPA does not expressly impose a duty of care on a partner. However, the UPA does state that “no act
in contravention of any agreement between the partners may be done rightfully without the consent of all
the partners.” Clearly, Bill sold the equipment in contravention of partnership policy and without
obtaining the consent of the other partners, and therefore the action was misconduct under the UPA as
well.

Explanation to Point-Two (10-25%):

Bill did not breach his duty of loyalty to ABD Partnership by patenting the invention in his own name.

The issue in respect of the patent is whether Bill, as an agent and partner, must hold the patent for the
benefit of the partnership. As a general rule, an agent or partner assigned “to do noninventive work is
entitled to patents which are the result of his invention, although the invention is due to the work for
which he is employed.” Because Bill is in charge of sales, he is not employed in an inventive capacity and
may hold the patent in his own name. In addition, Bill did not use any ABD facilities or any of his
business time to develop the keyboard. If, unlike the facts given here, Bill had used ABD’s facilities to
develop the invention, Bill may have owed the partnership a nonexclusive license to use the invention.

Explanation to Point-Three (40-50%):

Bill did breach his duty of loyalty by competing with ABD Partnership through his participation in B-
XYZ Partnership, and he must account to ABD for the profits derived from the sale of the keyboards for
the disabled.

Under the RUPA and under the common law, Bill owes ABD Partnership a duty of loyalty. The duty of
loyalty partners owe to the partnership imposed by the RUPA is as follows

(1) to account to the partnership and hold as trustee for it any property, profit, or benefit derived by the
partner, without the informed consent of the other partners, from a transaction connected with the
formation, conduct, or liquidation of the partnership or from a use by the partner of partnership property;

(2) to refrain from dealing with the partnership as, or on behalf of, an adverse party without the informed
consent of the other partners, and

(3) to refrain from competing with the partnership without the informed consent of the other partners.

Seperac-J19 Exam-Released MEE Essay Compilation © 2016-2020 679


The duty to refrain from competition is also found in agency law: “An agent is subject to a duty not to
compete with the principal concerning the subject matter of the agency.”

The UPA does not expressly impose on a partner a duty of loyalty. However, as noted earlier, a duty of
loyalty is implied because of the agency relationship of the partners. Moreover, the UPA requires that
“every partner must account to the partnership for any benefit derived by him without the consent of the
other partners” in the conduct of the partnership. Bill’s competition with the partnership through his
participation in B-XYZ Partnership without obtaining the consent of the other partners is a breach of his
obligations.

The issue that Bill is likely to raise is that ABD Partnership does not manufacture computer components
to accommodate the disabled. ABD will argue, however, that it manufactures computer keyboards. Given
the broad duties found in Meinhard v. Salmon, it is likely courts would find Bill’s action of competing in
the same general line of business to be a violation of his obligations to ABD Partnership.

It is also possible to argue that Bill should have offered the business opportunity of manufacturing
keyboards accommodating the disabled to ABD Partnership before entering into a new partnership with a
competitor. Cases such as Guth v. Loft, Inc. recognize that part of the duty of loyalty requires that
business opportunities must be made available to the principal before the agent individually profits from
the opportunity. A good argument can be made that as a part of Bill’s duty of loyalty, he should have first
offered to enter into a partnership with ABD to manufacture the specialized keyboard.

Accordingly, the partnership may maintain an action against Bill for his breach of the duty of loyalty and
Bill must account to the partnership for the profits he derived from that breach.

Seperac-J19 Exam-Released MEE Essay Compilation © 2016-2020 680


#152-JUL 1997–MEE Q01: QUESTION ONE (AGENCY-PARTNERSHIP)

Laura, Tammy, and Sam agreed to open Central Subs, a fast food delicatessen. They each invested $
10,000 in Central Subs for its initial expenses, and they agreed that Sam would manage the business and
purchase supplies, although the approval of Laura and Tammy would be required for all expenditures over
$500. Finally, they agreed that Central Subs’ profits and losses would be shared equally among them.

As part of Central Subs’ grand opening, promotional flyers were distributed that pictured Sam, Laura, and
Tammy and identified them as the owners of the business.

Sam worked hard managing the store. However, one day when Sam was cleaning the bathroom, he
negligently left water on the floor, and Karly, a customer, slipped and fell, severely injuring herself. Karly
sued Central Subs and the three investors individually, claiming they were responsible for the injuries she
had incurred. She demanded $100,000 in damages.

Several weeks later, the store ran out of luncheon meats. Without consulting anyone, Sam walked into
Meatco, a nearby butcher shop, and ordered $600 worth of meat. Although Meatco was not the shop
where Central Subs usually did business, Meatco agreed to sell Sam the meat on credit when Meatco’s
owner recognized Sam from the Central Subs promotional flyer.

After several months of operation, Central Subs’ business was not profitable, and Laura and Tammy
blamed it on Sam’s poor management of the store. At the next meeting of the investors, Laura and
Tammy told Sam that they were closing down the business and that his job at the store was terminated.
They also told him to return to the store after the meeting to drop off all of Central Subs’ property. They
also refused to pay Meatco’s bill, claiming that Sam had exceeded his authority by making the purchase
without their approval.

Immediately after the meeting, while he was driving back to the store to drop off Central Subs’ property,
Sam negligently ran over Pat, a pedestrian, causing Pat serious physical harm. Pat filed suit against
Central Subs and the three investors individually for tort damages. Meatco also filed suit against Central
Subs and the three investors individually for the amount of its unpaid bill.

Can Karly, Meatco, and Pat each recover from Central Subs? Explain. From the investors individually?
Explain.

Seperac-J19 Exam-Released MEE Essay Compilation © 2016-2020 681


#152: J97-1 MEE: ANSWER: NCBE (AGENCY-PARTNERSHIP)

POINT (1) [22%] ISSUE: Was a general partnership created by the investors when they formed
Central Subs? ANSWER: Yes. The three investors created a general partnership by agreeing to
share control and the profits of their business.

POINT (2) [22%] ISSUE: What principles of agency and partnership law support Karly’s claim for
damages against Central Subs and the investors for Sam’s negligence? ANSWER: Karly has a valid
claim against (a) Central Subs, since Sam was a partner acting in the ordinary course of the
partnership’s business and (b) each of the investors, since a partner in a general partnership is
jointly and severally liable for any wrongful act of a partner in connection with the partnership.

POINT (3) [28%] ISSUE: Under what principles of agency and partnership law did Sam’s order
from Meatco bind Central Subs and the investors? ANSWER: Sam’s order from Meatco binds
Central Subs both under the UPA and under the common law because he had either apparent
authority or incidental authority.

POINT (4) [28%] ISSUE: Does the dissolution of the partnership affect Pat’s ability to hold Central
Subs and the investors liable for Sam’s negligence? ANSWER: No. Laura and Tammy have
dissolved the partnership, which leads to a winding up of the partnership business. The partners
have authority to conduct old business as part of the winding up process. Sam is completing old
business by acting on the instructions of his former partners to drop off Central Subs’ property,
even though he is no longer working in the store. Therefore, the partnership is liable to Pat for
Sam’s negligence.

ANSWER EXPLANATION:

Explanation to Point-One (15-25%):

The three investors created a general partnership by agreeing to share control and the profits of their
business.

The facts do not indicate whether Laura, Tammy, and Sam intended to form their venture as a general
partnership. However, no intent to form a partnership need be expressed for a partnership to be created. In
most jurisdictions, the governing statute is the Uniform Partnership Act (UPA), which defines a
partnership as “an association of two or more persons to carry on as co-owners a business for profit.” The
key elements are the parties’ agreement to share profits and control. Here both elements are present.
Sharing of control is evidenced by Sam being charged with managing the business, while Laura and
Tammy have the power to veto expenditures over $500. The facts specify that profits and losses are to be
shared equally. UPA § 7 makes the sharing of profits prima facie evidence of the existence of a
partnership. Therefore, the arrangement among Laura, Tammy, and Sam was a general partnership.

Explanation to Point-Two (15-25%):

Karly has a valid claim against (a) Central Subs, since Sam was a partner acting in the ordinary course of
the partnership’s business and (b) each of the investors, since a partner in a general partnership is jointly
and severally liable for any wrongful act of a partner in connection with the partnership.

Seperac-J19 Exam-Released MEE Essay Compilation © 2016-2020 682


Karly was injured as a result of Sam’s negligence in cleaning the bathroom, which was part of his
managerial duties for Central Subs. A partnership is liable for the wrongful acts or omissions of any
partner acting in the ordinary course of the partnership’s business or with the authority of the other
partners to the same extent as the acting partner is liable. Sam was clearly acting in the ordinary course of
business and with the authority of Laura and Tammy when cleaning the bathroom. Since the facts state
that Sam acted negligently and therefore would be personally liable to Karly, the principles of partnership
law allow Karly to recover damages from Central Subs as well.

In addition, UPA § 15 makes the individual partners jointly and severally liable for all obligations of the
partnership arising under UPA § 13. The failure to disavow an agent’s acts by the principal is considered
an affirmance of such acts under the common law. Consequently, Laura, Tammy, and Sam are each
personally liable to Karly.

The UPA does not require that a plaintiff exhaust the partnership assets before making a claim against one
or more partners individually. However, the Revised Uniform Partnership Act (RUPA) adopted in a few
jurisdictions, specifically states that a judgment against a partnership can be satisfied from the personal
assets of a partner only if the partnership has failed to satisfy the judgment or is in bankruptcy or if the
liability would have been imposed on the partner regardless of the existence of the partnership. Under the
RUPA, Karly would have to make a claim and obtain a judgment against Central Subs which was unpaid
before she could look to the personal assets of Laura or Tammy. Since Sam would have been liable to
Karly for his own negligence even if no partnership existed, Karly could immediately seek to recover her
damages from Sam’s personal assets as well as the partnership’s.

Under principles of contribution, if Karly recovered the full amount of her damages from either Laura or
Tammy individually, either under the UPA, the RUPA, or the common law, the paying partner would be
entitled to payment from the other two partners for their proportional share of the damages.

Explanation to Point-Three (20-30%):

Sam’s order from Meatco binds Central Subs both under the UPA and under the common law because he
had either apparent authority or incidental authority.

Under the UPA, the act of every partner apparently carrying on the ordinary business of the partnership
binds the partnership unless (A) the partner has no such authority and (B) the person doing business with
the partnership knows that the partner has no such authority. Sam purchased the meat in the ordinary
course of the partnership’s business. However, Sam violated the investors’ agreement by ordering more
than $500 worth of meat without getting the approval of Laura and Tammy. Since Sam ordered the meat
from a store that had not done business with Central Subs before, Meatco had no way of knowing of any
limitations on his authority. The partnership is therefore bound and Meatco can recover the amount of its
bill from Central Subs.

Under the common law, there are two possible alternative theories under which Central Subs would be
liable for Sam’s meat order: apparent authority and incidental authority. Apparent authority is authority of
an agent not actually possessed by that agent but of the type that the principal holds the agent out as
having. Sam arguably had apparent authority to order the meat because Meatco knew from seeing the
promotional flyers that Sam was an owner of Central Subs, and the purchase of $600 worth of meat would
seem to be an action that an owner generally would have the authority to take.

Incidental authority could also apply to bind Central Subs for the meat purchase because under this
doctrine Sam had authority to take action that a reasonable person in the principal’s position should have

Seperac-J19 Exam-Released MEE Essay Compilation © 2016-2020 683


foreseen the agent would be likely to take, even though the action would be contrary to the agent’s
instructions. Here Sam responded to an emergency at the store by ordering more meat, an act which
should have been foreseeable. The only issue is the foreseeability of ordering more than $500 worth of
meat. Despite the issue of foreseeability of the amount of meat ordered, a court would likely hold Central
Subs liable for Meatco’s bill under this agency theory.

To the extent that Central Subs is liable to Meatco under either the UPA or common-law agency analysis
discussed above, each of the partners will be jointly and severally liable for this debt.

Explanation to Point-Four (20-30%):

Laura and Tammy have dissolved the partnership, which leads to a winding up of the partnership
business. The partners have authority to conduct old business as part of the winding up process. Sam is
completing old business by acting on the instructions of his former partners to drop off Central Subs’
property, even though he is no longer working in the store. Therefore, the partnership is liable to Pat for
Sam’s negligence.

Laura and Tammy’s actions to close down Central Subs effectively dissolved the partnership. However,
dissolution does not terminate the partnership, which continues until the winding up of the partnership’s
affairs is finished. Winding up generally includes completion of old business of the partnership, collection
of monies owed to the partnership, payment of partnership debts, and distribution of assets to the partners.
One item of old business is that Sam return all of Central Subs’ property to the store as instructed by
Laura and Tammy, his former partners.

The general rule is that dissolution of a partnership terminates all authority of a partner, vis-a-vis the other
partners, to act for the partnership. However, there is an exception where a partner takes action in
connection with the winding up of the partnership. In addition, after dissolution a partner can bind the
partnership to a third party in connection with such partner’s activities to wind up the partnership. Since
Sam’s trip back to the store was part of the winding up of the partnership as expressly authorized by the
other investors, he was acting in his capacity as a partner when he injured Pat. As a result, Central Subs
will be liable for Sam’s wrongful act or omission even though a dissolution has occurred. To the extent
that Central Subs is liable tinder UPA §13, each of the investors will also be individually liable (jointly
and severally) to Pat under Furthermore, the fact that the partnership has been dissolved does not affect
the existing liabilities of the investors.

Seperac-J19 Exam-Released MEE Essay Compilation © 2016-2020 684


#153-FEB 1997–MEE Q04: QUESTION FOUR (AGENCY-PARTNERSHIP/CORPS-LLCS)

Wareco is a corporation that operates a small commercial storage warehouse. Pres is the president of
Wareco and Sec is the secretary of Wareco. Both Pres and Sec regularly attend board meetings, although
neither of them is a director or shareholder of Wareco. At a January 1 board meeting, the Wareco board of
directors passed a resolution stating that the president of the corporation may not enter into any contract
for over $ 1 million without the express prior approval of the board of directors.

On February 1, Pres signed a contract as president of Wareco providing that Wareco would purchase a
fleet of 50 trucks from Able, Wareco’s usual supplier of trucks. The purchase price specified in the
contract was $20 million. Able was unaware that the board of directors of Wareco had never approved
this transaction, but he did know that Wareco had never owned more than five trucks at any one time. On
February 2, the board learned of the contract with Able and immediately repudiated it.

On March 1, Pres proposed to the board of directors of Wareco that the corporation purchase a forklift
from Beta. Forklifts are essential to Wareco’s business and it owns several of them. Because the board did
not believe Wareco needed an additional forklift, it passed a resolution disapproving the proposed
purchase. Nevertheless, without the board’s knowledge, on March 5, Pres signed a contract as president of
Wareco to purchase a forklift from Beta for $2,000. Beta had no knowledge of the board’s disapproval.
On March 6, the board learned of the contract with Beta and immediately repudiated it.

On April 1, Pres signed a contract as president of Wareco providing that Wareco would purchase a herd
of cattle from Rancher for $2 million. Sec, as secretary of Wareco, signed and delivered a document
certifying to Rancher that the Wareco board of directors had earlier approved the execution of this
contract by Pres in a resolution validly passed at a duly called meeting of the board. Sec attached a copy
of the text of the resolution to the certificate. Sec frequently signed such certificates as part of Sec’s duties
as secretary. In fact, the execution of this contract had not been approved at a board meeting, but instead
was previously approved without a meeting by a consent resolution signed by four of the five members of
the Wareco board of directors. Rancher was unaware that Sec’s certificate was incorrect. On April 2, the
board learned of the contract with Rancher and immediately repudiated it.

1. Can Able recover damages from Wareco? From Pres? Explain.

2. Can Beta recover damages from Wareco, and, if so, does Wareco have a cause of action against
Pres? Explain.

3. Can Rancher recover damages from Wareco? Explain.

Seperac-J19 Exam-Released MEE Essay Compilation © 2016-2020 685


#153: F97-4 MEE: ANSWER: NCBE (AGENCY-PARTNERSHIP/CORPS-LLCS)

POINT (1) [33%] ISSUE: Did Pres have the real or apparent authority to bind Wareco and, if not,
is he personally liable to Able? ANSWER: The January board resolution denied Pres any real
authority to enter into the contract with Able. Apparent authority would exist only if a reasonable
person in Able’s position would believe authority existed. Without real or apparent authority Pres
did not bind Wareco, but did bind himself by breaching his implied warranty of authority.

POINT (2) [33%] ISSUE: Did Pres have the real or apparent authority to bind Wareco to Beta and,
if so, is he liable to Wareco? ANSWER: Yes. The March board resolution denied Pres real
authority to enter into the contract with Beta. However, apparent authority existed given the
ordinary nature of the transaction and Wareco is bound. Because Pres bound Wareco when he was
directed not to, he is liable to Wareco.

POINT (3) [33%] ISSUE: Is Wareco liable on the contract with Rancher either because Pres had
real or apparent authority or as a result of Sec’s erroneous certificate? ANSWER: Yes. Although
Pres had no real authority because the consent resolution was not unanimous and absent Sec’s
certificate, had no apparent authority because the transaction was extraordinary, Wareco is bound
because Sec had the authority to give the certificate which created apparent authority in Pres.

ANSWER DISCUSSION:

The facts are clear that Wareco acted promptly to repudiate each of the unauthorized contracts. This
eliminates any argument that Wareco had accepted any benefits from the contracts or that Able, Beta or
Rancher had detrimentally relied upon Wareco’s execution of the contracts, and that, therefore, the
contracts would be binding on Wareco regardless of their unauthorized nature.

ANSWER EXPLANATION:

Explanation to Point-One (25-35%):

The January board resolution denied Pres any real authority to enter into the contract with Able. Apparent
authority would exist only if a reasonable person in Able’s position would believe authority existed.
Without real or apparent authority Pres did not bind Wareco, but did bind himself by breaching his
implied warranty of authority.

An agent may bind a principal to a contract if the agent is acting within his real or apparent authority.
Real authority may be expressed or implied to the agent by the principal. In this case no real authority
existed, because Wareco expressly stated in the board resolution that Pres had no authority to enter into
contracts for this amount without prior board approval. Apparent authority exists if the conduct of the
principal leads the third party reasonably to believe that the agent has the authority. By electing Pres
president, Wareco may have led reasonable third parties to believe that Pres had authority to enter into
transactions in the ordinary course of Wareco’s business. Even though the nature of the transaction
suggests that it was in the ordinary course of business, the number of trucks compared to the size of
Wareco’s current fleet made the transaction extraordinary. This should have put Able on notice that no
authority existed. Thus, Pres had neither real nor apparent authority, and Wareco is not liable to Able.

Seperac-J19 Exam-Released MEE Essay Compilation © 2016-2020 686


An agent purporting to act on behalf of a principal, but lacking authority, becomes liable on the contract
for breaching his implied warranty of authority. Because Pres had no real or apparent authority, he is
personally liable to Able for breach of his implied warranty of authority.

Explanation to Point-Two (25-35%):

The March board resolution denied Pres real authority to enter into the contract with Beta. However,
apparent authority existed given the ordinary nature of the transaction and Wareco is bound. Because Pres
bound Wareco when he was directed not to, he is liable to Wareco.

As stated in Point One, an agent can bind a principal if operating within his real or apparent authority.
Pres lacked real authority to purchase the forklift because the March board resolution instructed him not
to enter into this transaction. Apparent authority existed, however, because a reasonable person in Beta’s
position would believe that the president of Wareco had authority to enter into this ordinary transaction.
Thus, Wareco is liable to Beta on the contract.

An agent who injures his principal by acting beyond the principal’s manifestation of consent is liable to
the principal. In this case, Pres entered into the contract with Beta despite the disapproval of the board of
directors of Wareco. Since Wareco became bound on this contract by the act of Pres beyond his real
authority, Pres is liable to Wareco.

Explanation to Point-Three (25-35%):

Although Pres had no real authority because the consent resolution was not unanimous and absent Sec’s
certificate, had no apparent authority because the transaction was extraordinary, Wareco is bound because
Sec had the authority to give the certificate which created apparent authority in Pres.

To be effective, a written consent in lieu of a directors’ meeting typically requires signatures of all of the
directors. Because the consent resolution in this situation was not signed by all of the directors, it was
ineffective. Pres, therefore, had no real authority because the purchase price exceeded the $ 1 million
limitation contained in the January resolution.

In the absence of Sec’s certificate, Pres had no apparent authority because the purchase of a $2 million
herd of cattle was not ordinary for this corporation. Rancher could not reasonably believe that the
president of a commercial warehouse corporation would have the authority to make such a purchase on
behalf of a corporation.

However, delivery of Sec’s certificate created apparent authority. The secretary of a corporation has the
authority to authenticate records of the corporation. Sec erroneously authenticated a resolution approving
the execution by Pres of the contract with Rancher. Although this authentication was not accurate, it binds
Wareco, and Rancher may rely on the authority of Pres indicated in the authenticated resolution.
Therefore, Wareco is liable on the contract with Rancher.

Seperac-J19 Exam-Released MEE Essay Compilation © 2016-2020 687


#154-JUL 1996–MEE Q02: QUESTION TWO (AGENCY-PARTNERSHIP)

Painter, an internationally renowned artist, lived as a recluse in his secluded villa. His paintings were sold
only by a select group of dealers. Agent was Painter's authorized dealer in Lawburg, a large U.S. city. On
August 1, when Collector decided to purchase an original work by Painter, she went to Agent's gallery.
Prominently displayed on the gallery wall was an elegant certificate, signed by Painter, designating Agent
as his exclusive agent in Lawburg, with full authority to enter into any contracts on Painter's behalf for the
sale of his paintings.

Painter insisted on retaining possession of all his paintings until they were sold. Therefore, he prepared
for use by his dealers an elaborate catalog containing pricing information and photographic reproductions
of the paintings. In keeping with standard practice in the art world, on the front page of the catalog was a
statement signed by Painter certifying the catalog's authenticity and declaring that its use was restricted to
his authorized agents. Agent allowed Collector to take the catalog home to make her selection.

Unknown to Collector, a bitter dispute over unpaid commissions had been brewing between Agent and
Painter, who had grown quite ill. On August 15, Agent received a notice from Painter revoking Agent's
authority to act on Painter's behalf. That week, Painter took out a quarter-page advertisement in the
Lawburg Times, stating that Agent was no longer his authorized agent. Collector never saw, or knew of,
the ad. Throughout these developments, Painter's certificate remained displayed on Agent's gallery wall.

On September 1, when Collector returned to Agent's gallery, Painter's certificate was still on display.
Collector signed a contract with Agent for the purchase of one of the Painter paintings displayed in the
catalog.

1. Is Collector's September 1 contract binding against Painter? Explain.

2. If Painter had died two days before the contract was signed, but neither of the parties knew of his
death and no public announcement was made of it until September 5, would the contract be binding
against Painter's estate? Explain.

Seperac-J19 Exam-Released MEE Essay Compilation © 2016-2020 688


#154: J96-2 MEE: ANSWER: NCBE (AGENCY-PARTNERSHIP)

POINT (1)(a) [25%] ISSUE: What effect did Painter's August 15 notice have on Agent's actual
authority to enter into a binding contract for the sale of Painter's paintings? ANSWER: Painter's
August 15 notice was effective to terminate Agent's actual authority irrespective of any contractual
arrangements between them.

POINT (1)(b) [25%] ISSUE: What effect did Painter's August 15 notice have on Agent's apparent
authority to enter into a binding contract for the sale of Painter's paintings? ANSWER: Agent had
apparent authority to sell paintings on Painter's behalf by virtue of the certificate and note in the
catalog; Painter's August 15 notice to Agent was ineffective to terminate this apparent authority as
to Collector.

POINT (1)(c) [25%] ISSUE: What effect did the newspaper advertisement have on Agent's
apparent authority? ANSWER: Because Painter failed to recover the indicia of authority
previously entrusted to Agent, the newspaper advertisement announcing the termination of Agent's
authority was ineffective as to Collector who was unaware of the advertisement and continued to
rely on those indicia of authority.

POINT (2) [25%] ISSUE: What effect did Painter's death have on Agent's actual or apparent
authority? ANSWER: Under the common law, Painter's death operated to terminate both the
actual and apparent authority of his agents, even though they lacked notice; but many jurisdictions
have reversed this rule by statute.

ANSWER EXPLANATION:

Explanation to Point-One(a) (20-30%):

Painter's August 15 notice was effective to terminate Agent's actual authority irrespective of any
contractual arrangements between them.

Actual authority is "the power of the agent to affect the legal relations of the principal by acts done in
accordance with the principal's manifestations of consent to him." Thus, based on Painter's earlier
designation of Agent as one of his select group of dealers, Agent had actual authority to sell Painter's
paintings when Collector first visited his gallery on August 1. But because of the close personal
association embodied in the agency relationship, the law recognizes the principal's power to revoke the
agent's authority at any time, even if that revocation entails a breach of the contract between the principal
and the agent. All that is required is communication to the agent that the principal no longer desires the
agent to act on his or her behalf. Thus, Painter's August 15 notice operated to terminate Agent's actual
authority to enter into contracts such as the one with Collector.

Explanation to Point-One(b) (20-30%):

Agent had apparent authority to sell paintings on Painter's behalf by virtue of the certificate and note in
the catalog; Painter's August 15 notice to Agent was ineffective to terminate this apparent authority as to
Collector.

Seperac-J19 Exam-Released MEE Essay Compilation © 2016-2020 689


Apparent authority is "the power to affect the legal relations of another person by transactions with third
persons, professedly as agent for the other, arising from and in accordance with the other's manifestations
to such third persons." The certificate on Agent's wall and the signed note in the catalog were
communications from Painter that led Collector reasonably to believe that Agent had the authority to sell
Painter's paintings. Thus, they gave Agent apparent authority to enter into contracts for the sale of
Painter's paintings. Because apparent authority is created by communications from the principal to the
third party and protects that third party, it is not terminated by the principal's communication to the agent
if the third party is unaware of that communication. Thus, even though the August 15 notice operated to
terminate Agent's actual authority it did not terminate his apparent authority as to the Collector.

Explanation to Point-One(c) (20-30%):

Because Painter failed to recover the indicia of authority previously entrusted to Agent, the newspaper
advertisement announcing the termination of Agent's authority was ineffective as to Collector who was
unaware of the advertisement and continued to rely on those indicia of authority.

The rules governing termination of apparent authority reflect an effort to strike a reasonable balance
between (1) the burden of requiring the principal to communicate directly with every person aware of the
agent's former authority and (2) the burden of requiring third parties constantly to verify the ongoing
existence of the agent's authority. Thus, the principal is ordinarily required to give direct notice of the
termination only to those third parties he or she can reasonably identify. But the facts supply no reason to
infer that Painter is aware that Agent has begun to deal with Collector. Thus, a public notice such as that
placed by Painter in the Lawburg Times might in some circumstances be effective to terminate Agent's
apparent authority as to persons like Collector even though they were unaware of it.

But the Restatement recognizes a specific exception for persons relying on indicia of authority entrusted
to the agent. As to them, constructive notice through newspaper advertisements and the like is ineffective
to terminate apparent authority.

Although the principal is entitled to have indicia of authority returned to him upon termination of the
relation, if he is unsuccessful in accomplishing this the risk of the deception of third persons who have
otherwise no notice of the termination rests upon the principal.

While there is a tension between Painter's newspaper notice and Agent's continuing display of the
certificate regarding the termination of agency authority, it is more reasonable to expect Painter to recover
the certificate and any other indicia of authority than to expect Collector to learn of the advertisement,
although a plausible argument might be made either way.

Explanation to Point-Two (20-30%):

Under the common law, Painter's death operated to terminate both the actual and apparent authority of his
agents, even though they lacked notice; but many jurisdictions have reversed this rule by statute.

The common law rule was that the death of the principal terminated any existing actual or apparent
authority to act on the principal's behalf. Thus, under the common law rule, Agent's apparent authority to
act for Painter terminated on his death and, therefore, the September 1 contract with Collector is not
binding on his estate, even though neither Agent nor Collector was aware of it.

Many jurisdictions have enacted statutes reversing the common law rule. Under these statutes, actual and
apparent authority persist until the agent and the third party have notice of the principal's death. In these

Seperac-J19 Exam-Released MEE Essay Compilation © 2016-2020 690


jurisdictions, Agent's apparent authority would continue until September 5, and the September 1 contract
would be binding.

Seperac-J19 Exam-Released MEE Essay Compilation © 2016-2020 691


#155-FEB 1996–MEE Q05: QUESTION FIVE (AGENCY-PARTNERSHIP)

Able is in the business of buying and selling rare coins. She buys coins for her inventory mostly at sales
conducted by auction houses. Able uses the services of "purchasers," who attend the sales and bid for
coins to be added to her inventory. Each of the purchasers signs an agreement, the form of which is
reproduced below.

Purchaser Agreement:

The undersigned ("Purchaser") agrees to act on behalf of Able ("Able") as an independent contractor
purchaser of rare coins. Purchaser shall attend sales specified by Able and bid on coins from a
confidential listing supplied by Able (the "Buy List"), at a price not to exceed the amount shown on the
Buy List. Purchaser shall not submit any bid until Able has given telephonic approval for the specific bid.

Purchaser shall contract in the name of Purchaser for such coins, without disclosing the identity of Able.
Funds for authorized purchases shall be supplied by wire transfer upon Purchaser's request.

Purchaser shall be compensated for travel expenses at the lesser of (i) Purchaser's actual costs in attending
such sales or (ii) a per diem of $150. Purchaser shall also receive a quarterly bonus equal to 25% of the
savings effected by Purchaser on coins purchased during each quarter for less than the authorized prices
set forth on the Buy List.

This arrangement may be terminated upon notice by either Purchaser or Able.

Dated

Signed

Purchaser

Baker signed a purchaser agreement. Baker thereafter attended several sales on Able's behalf. At the first
sale, Baker located coins on the Buy List. After calling Able for authorization, Baker contracted to buy
the coins in his name at prices less than the Buy List prices. Able wired funds allowing Baker to
consummate that transaction.

Baker subsequently learned from the other purchasers that, although they have all been informed of the
standard policy requiring them to get prior approval, they never call Able for authorization. If a coin is on
the Buy List, they buy it if they can get it at or under the Buy List price. None of the other purchasers has
ever had a problem getting the funds from Able to complete a purchase. Able has refused to forward the
money only when the sale price has exceeded the Buy List price. After learning this information, Baker
began purchasing coins at prices below the Buy List prices without Able's prior authorization. Able
always supplied Baker with funds to cover these purchases, despite the lack of Able's prior approval.

Last Saturday, Baker attended an auction and found a U.S. 1913 Leaping Liberty quarter in mint,
uncirculated condition. The price shown on the current Buy List for a 1913 Leaping Liberty quarter is
$50,000. Without calling Able, Baker contracted to buy the coin for $30,000, and now claims a bonus of
$5,000 (25% of the difference between the $50,000 Buy List price and the $30,000 contract price).

Seperac-J19 Exam-Released MEE Essay Compilation © 2016-2020 692


It turns out that the Buy List was in error. The entry should have read "1913 Leaping Liberty Quarter,
mint, uncirculated condition: $20,000."

Able refused to wire the funds to close the transaction.

What is the legal relationship between Able and Baker, and, in light of that relationship, what are the
liabilities of Able and Baker to the third-party seller and to one another? Explain.

Seperac-J19 Exam-Released MEE Essay Compilation © 2016-2020 693


#155: F96-5 MEE: ANSWER: NCBE (AGENCY-PARTNERSHIP)

POINT (1) [26%] ISSUE: Is Baker an agent of Able for purposes of the purchase contract?
ANSWER: Yes. Notwithstanding the contract characterization of Baker as an "independent
contractor," he is an agent of Able.

POINT (2) [58%] ISSUE: Did Baker have authority to bind Able to the purchase contract?
ANSWER: Yes. Baker probably possessed the implied actual authority to bind Able to the
purchase contract, and, because he acted within the scope of his authority, he is entitled to the
$5,000 bonus.

POINT (3) [16%] ISSUE: Who is ultimately liable for performance of the contract, Able or Baker?
ANSWER: Baker is liable to the seller, but Able must reimburse Baker for any amount Baker pays
to the seller.

ANSWER EXPLANATION:

Explanation to Point-One (20-30%):

Notwithstanding the contract characterization of Baker as an "independent contractor," he is an agent of


Able.

First, it is the substance of the relationship and not the characterization placed upon it by the parties that
controls the legal consequences of their relationship. Whether Baker is an independent contractor, an
employee, or something else is irrelevant. In any of those capacities, he can still be an agent, if the
requisites for an agency relationship are present.

An “agent” is a “person authorized by another to act on his account and under his control.” An agent can
be either a “servant” or an “independent contractor.” A “servant” is an agent whose physical conduct in
the performance of his agency is subject to the principal's right of control. Independent contractors are
persons who have contracted to do work for another, but “who are not servants in doing the work
undertaken.” Not all independent contractors are agents. However, an independent contractor is an agent
if he contracts to act for the account of another, subject to the other's direction and control (other than
control of the physical conduct of the services or work being performed). “In fact, most of the persons
known as agents, that is, brokers, factors, attorneys, collection agencies, and selling agencies are
independent contractors as the term is used in the Restatement.”

So what is the importance of distinguishing between agents who are servants and agents who are
independent contractors? If the agent tortiously injures someone, the principal's liability probably turns on
whether the agent is a servant, and, thus, subject to the principal's control as to physical conduct of the
agency, or, instead, an independent contractor. Responsibility follows the right to control.

However, in this question there is no tort. Thus, the distinction between servant and independent
contractor is irrelevant. The only issue is whether Baker is an agent. The proper reference would be to the
Second Restatement § 14, which discusses the distinction between agency and other relations.

The most relevant possibility to discuss is whether Baker is an agent who buys for Able's account, or a
supplier who acquires property for his own account and resells to Able. The three most important factors

Seperac-J19 Exam-Released MEE Essay Compilation © 2016-2020 694


indicating that one is not an agent are outlined in § 14K. The most important factor is whether the
purchaser receives a fixed price for property, irrespective of the price paid. If, as here, the price received
by Baker depends on the price paid to the third-party seller, agency is indicated. However, if the
purchaser, as did Baker in this case, takes title to purchased property in his own name, this is a factor
indicating that he is a supplier. Finally, if the purchaser has an independent business in buying and selling
similar property, such fact would suggest that he is a supplier instead of an agent. That does not appear to
be the case here. It seems that Baker is acting on behalf of Able and not carrying on a separate business.
The balance of factors here is in favor of an agency relationship.

Explanation to Point-Two (50-60%):

Baker probably possessed the implied actual authority to bind Able to the purchase contract, and, because
he acted within the scope of his authority, he is entitled to the $5,000 bonus.

Once the agency relationship is established, it is necessary to determine the limits of the agent's authority
to act. The principal can be liable to third parties based either on the agent's actual, apparent, or inherent
authority to act as of the time of the agent's actions, or on the basis of a principal's subsequent ratification
of the agent's actions.

In this case there is no indication that Able has taken any actions subsequent to the bid by Baker
inconsistent with her position that Baker had no authorization. There is no argument for ratification.

The argument that Baker had authority at the time of the action is more complex. Actual authority is
based on words or conduct of the principal that manifest the consent of the principal that the agent act on
behalf of the principal. Such authority can be either express or implied. The purchaser agreement
specifically requires Baker to get Able's telephonic consent to bid. Able did not affirmatively amend those
instructions. Therefore, Baker had no express actual authority to bid without prior approval.

Implied actual authority is also based on the words or conduct of the principal, here involving words or
conduct that would lead a reasonable person in an agent's position to conclude that the agent had authority
to act, although not constituting a direct and specific authorization. Implied actual authority “can be
created by written or spoken words or other conduct of the principal which, reasonably interpreted, causes
the agent to believe that the principal desires him to act on the principal's account.” The silence of the
principal can manifest authority to act, (“a secretary who, without previous authorization, purchases office
supplies which are paid for without objection by the principal, can reasonably conclude that the principal
wishes a continuance of this practice and hence would be authorized to continue to purchase similar
supplies.”), and “acquiescence by the principal in a series of acts by the agent indicates authorization to
perform similar acts in the future.”

There is a strong argument for implied actual authority in this case because Able's consistent practice was
to forward the money for the coins if the purchase was within the guidelines of the Buy List, even if no
prior authorization had been secured. Although Able's practice had not initially been directed at Baker,
Baker learned of the practice, tried it, and relied upon it when Able did not object.

Apparent authority is also based on the principal's words or conduct, the test being whether the words and
conduct of the principal would lead a reasonable person in the third party's position to conclude the agent
had authority to act. In this situation, there is no indication that the seller knew of Able, because Baker
was specifically forbidden from disclosing Able's involvement. Absent reliance by the seller on Able's
actions, there can be no apparent authority.

Seperac-J19 Exam-Released MEE Essay Compilation © 2016-2020 695


The final possibility is inherent authority. Inherent authority is authority derived purely from the agency
relation. The Second Restatement of Agency identifies three classes of inherent authority. The first is
“that in which a general agent does something similar to what he is authorized to do, but in violation of
orders.” Using inherent authority, an agent for an undisclosed principal “subjects his principal to liability
for acts done on his account, if usual or necessary in such transactions, although forbidden by the
principal to do them.” The second deals with disclosed or partially disclosed principals and is inapplicable
to the facts presented. The third basis for this liability is public policy. “Commercial convenience requires
that the principal should not escape liability where there have been deviations from the usually granted
authority by persons who are such essential parts of his business enterprise.” It could be argued that the
authority to buy coins described in the Buy List is “usual or necessary” within the scope of Able's
operation, resulting in inherent authority. The better argument, however, is that Baker has implied actual
authority.

Because Baker possessed implied actual authority, Able is bound by the contract and liable to the third-
party seller. And, because Baker acted within the scope of his authority, he is entitled to the full $5,000
bonus.

Explanation to Point-Three (10-20%):

Baker is liable to the seller, but Able must reimburse Baker for any amount Baker pays to the seller.

The liability of Baker to the third-party seller turns on whether Able was a fully disclosed, partially
disclosed, or undisclosed principal. If Able were a fully disclosed principal, Baker would not be liable to
the third-party seller at all. If the principal were only partially disclosed (the fact of the agency being
disclosed, but not the identity of the principal), or is undisclosed, the agent remains liable to the third
party on the contract. The theory is that the other party is unable to investigate the bonafides of the
principal and is acting in reliance on the identity of the agent. Here, because Able's involvement was
presumably undisclosed, Baker is liable to the third-party seller.

But, because Baker had actual authority, albeit implied actual authority, Able is liable to Baker for
indemnity on any payments made by Baker to the third party seller. Thus, if Baker pays the third party,
Able must reimburse Baker.

Able's liability to Baker can also be argued on the basis that Able was responsible for the error on the Buy
List. Even this basis for liability, however, must be premised on the agency relationship and rationalized
on the ground that Baker possessed implied actual authority to act because Able had waived the prior
“telephonic approval” condition stated in the Purchaser Agreement.

Seperac-J19 Exam-Released MEE Essay Compilation © 2016-2020 696


#156-FEB 1995–MEE Q04: QUESTION FOUR (AGENCY-PARTNERSHIP)

Smith, Jones, and Baker were partners in a consulting business. Approximately 60% of the partnership's
work was performed for two major corporations, ABC Inc. and XYZ Inc. The partners did not have a
written partnership agreement. They never decided how the management responsibility for the affairs of
the partnership was to be allocated or how long the partnership would continue to exist. They always
shared the profits and losses of the business equally and did not receive salaries for working in the
partnership business.

Although the partnership was very profitable, Smith often disagreed with the other two partners regarding
partnership business. When the partnership received a proposal from Newco Inc. to enter into a
substantial consulting contract on terms that appeared to be favorable to the partnership, Jones and Baker
favored entering into the proposed contract. Smith was opposed and wrote a letter to the other two
partners stating that Smith would not agree to any contract with Newco. In spite of Smith's objections,
Jones and Baker entered into the contract, purporting to act on behalf of the partnership.

Smith was very upset that Jones and Baker had entered into the contract with Newco. Smith contacted
ABC and XYZ and entered into contracts in his own name to do consulting work for them. The work to
be done under the contracts was of the same kind the partnership had done for these corporations in the
past.

The contract with Newco turned out to be very unprofitable. In contrast, Smith's contracts with ABC and
XYZ were very profitable. Smith refused to share the losses under the Newco contract, arguing that those
losses were not partnership obligations. In addition, Smith refused to share the profits from the ABC and
XYZ contracts with Jones and Baker, arguing that the profits were not partnership profits.

Jones and Baker now plan to enter into a second contract with Newco, on behalf of the partnership, to do
additional consulting work. The rate of compensation to the partnership would be greater than in the
original contract. Smith wants to prevent them from taking on any new business on behalf of the
partnership, and he tells them that, as far as he is concerned, the partnership is at an end.

(a) Did Jones and Baker have the right to enter into the original contract with Newco on behalf of
the partnership? Explain.

(b) May the partnership recover the profits earned by Smith in performing the contracts with ABC
and XYZ? Explain.

(c) Did Smith terminate the right of Jones and Baker to enter into new contracts with Newco on
behalf of the partnership? Explain.

Seperac-J19 Exam-Released MEE Essay Compilation © 2016-2020 697


#156: F95-4 MEE: ANSWER: NCBE (AGENCY-PARTNERSHIP)

POINT (1) [33%] ISSUE: Did Jones and Baker have the right to enter into the original contract
with Newco on behalf of the partnership? ANSWER: Yes. Jones and Baker had the right to enter
into the original contract with Newco on behalf of the partnership because they constitute a
majority of the partners and the contract involved an ordinary matter of partnership business.

POINT (2) [35%] ISSUE: May the partnership recover the profits earned by Smith in performing
the contracts with XYZ and ABC? ANSWER: Yes. Smith violated his fiduciary duties as a partner
in undertaking the ABC and XYZ contracts in his own name and may be forced to account to the
partnership for the profits earned under those contracts.

POINT (3) [33%] ISSUE: Can Smith terminate the right of Jones and Baker to enter into new
contracts on behalf of the partnership? ANSWER: Yes. Because the partnership is at will, Smith
may terminate the right of Jones and Baker to enter into new contracts on behalf of the partnership
by dissolving the partnership.

ANSWER EXPLANATION:

Explanation to Point-One (25-40%):

Jones and Baker had the right to enter into the original contract with Newco on behalf of the partnership
because they constitute a majority of the partners and the contract involved an ordinary matter of
partnership business.

Subject to any agreement between the partners, the Uniform Partnership Act (hereinafter “UPA”)
provides that all partners have equal rights in the management and conduct of the partnership business and
that disagreements relating to ordinary matters connected with the partnership business may be decided by
a majority of the partners. However, no act in contravention of the partnership agreement may rightfully
be done without the consent of all the partners.

Since there is no agreement between Smith, Jones, and Baker regarding management, each of the partners
has an equal voice in the management of the partnership's business, and the will of the majority controls
as to ordinary matters connected with that business. Consequently, Jones and Baker had the right to enter
into the original contract with Newco on behalf of the partnership despite Smith's objections because (1)
they constitute a majority of the three partners and (2) the contract with Newco is a matter in the ordinary
course of the consulting business. On the facts of the question, the action does not contravene the
partnership agreement.

Explanation to Point-Two (30-40%):

Smith violated his fiduciary duties as a partner in undertaking the ABC and XYZ contracts in his own
name and may be forced to account to the partnership for the profits earned under those contracts.

The partners are in a fiduciary relationship to each other and to the partnership as a whole. An important
fiduciary duty owed by a partner to the partnership is the duty of loyalty. The duty of loyalty is a very
demanding duty. In general, the duty of loyalty requires a partner to act in good faith and to act fairly
toward the other partners. Specifically, a number of activities can violate a partner's duty of loyalty,

Seperac-J19 Exam-Released MEE Essay Compilation © 2016-2020 698


including competing with the partnership within the scope of its business, and usurping a business
opportunity that belongs to the partnership. Strained relations between the partners do not excuse a
violation of fiduciary duties.

The facts support a conclusion that Smith has violated the fiduciary duty of loyalty by competing with the
partnership for the consulting business of ABC and XYZ. The consulting business of these two
corporations constitutes a large portion of the partnership's business, the work to be done under the
contracts is similar to the work the partnership has done for the corporations in the past, and the other
partners have not consented to Smith's actions. The fact that the partners disagree regarding partnership
business is irrelevant.

Smith also violated the duty of loyalty by usurping the partnership's opportunity to do additional business
with the two corporations. Smith undoubtedly learned about the two consulting opportunities through the
partnership, the consulting business of the corporations was very important to the partnership, the
consulting work was similar to the work the partnership had done for the corporations in the past, and the
partnership probably expected to continue to perform consulting work for the two corporations. An
analogy might be made to cases in which a partner purchases land that the partnership is interested in
purchasing or renews a lease held by the partnership. In sum, Smith took for himself a business
opportunity which, in fairness, belonged to the partnership.

The partnership may require a partner who breaches his or her fiduciary duty of loyalty to account to the
partnership for any profits earned as a result of the breach.

Explanation to Point-Three (25-40%):

Because the partnership is at will, Smith may terminate the right of Jones and Baker to enter into new
contracts on behalf of the partnership by dissolving the partnership.

Where the partners do not expressly or implicitly agree that the partnership shall continue for a specified
term or until the completion of a particular undertaking, the partnership is a partnership at will. Any
partner may dissolve a partnership at will at any time without breaching the partnership agreement.

Smith, Jones, and Baker did not agree that the partnership would continue for a specified term or until the
completion of any specific undertaking such as completion of a particular contract. Consequently, the
partnership is at will and Smith may dissolve the partnership at any time by notifying James and Baker
without violating the partnership agreement.

As between the partners, the authority of a partner to act for the partnership terminates when the partner
has knowledge of the dissolution of the partnership by another partner except for actions that are
necessary to wind up the partnership or to complete transactions which have begun but are not yet
completed. Consequently, by dissolving the partnership Smith can prevent Jones and Baker from having
the right to take on any new business on behalf of the partnership. However, it is acknowledged that a
person winding up a partnership's business may continue its business in order to enhance its liquidation
value as a going concern. Consequently, Jones and Baker may be able to justify the new contract with
Newco if they can establish that it is a good-faith interim measure and part of the winding up of the
partnership.

Seperac-J19 Exam-Released MEE Essay Compilation © 2016-2020 699


CONFLICT OF LAWS: 5 OF 49 MEE EXAMS: (10%)
#157-JUL 2004–MEE Q06: QUESTION SIX (CONFLICTS)

Plaintiff, a domiciliary of State X, was severely injured in a car accident in State X. Tortfeasor, the
uninsured owner and driver of the other vehicle involved in the accident, was also a citizen of State X.
Before any litigation regarding the accident began, Tortfeasor died of a heart attack.

Following Tortfeasor’s death, Plaintiff commenced an action against Executor, the legal representative of
Tortfeasor’s estate. Executor is a citizen of State Y. Plaintiff sued Executor in the federal district court of
State Y. The complaint, which alleged that Tortfeasor’s negligence caused the accident and Plaintiff’s
injuries, sought damages in excess of $500,000.

Executor answered the complaint, denying the allegations of negligence but admitting the court’s subject
matter jurisdiction. A year later, however, after extensive discovery, Executor moved to dismiss the
complaint for lack of subject matter jurisdiction. After the submission of briefs and oral argument on the
jurisdictional issue, the federal court denied Executor’s motion to dismiss, ruling that “jurisdiction exists
and, in any event, the motion was untimely.” A trial was held and the jury rendered a verdict in Plaintiff’s
favor for $80,000.

The federal court entered judgment on the verdict. No appeal was taken.

When Executor declined to pay the judgment, Plaintiff commenced suit in a state court in State X to
enforce the federal judgment. In the state suit, Executor challenged the validity of the judgment, claiming
that the federal district court of State Y lacked subject matter jurisdiction.

1. Did the federal district court of State Y err in denying Executor’s motion to dismiss for lack of
subject matter jurisdiction? Explain.

2. Should the state court in State X enforce the federal judgment? Explain.

Seperac-J19 Exam-Released MEE Essay Compilation © 2016-2020 700


#157: J04-6 MEE: ANSWER: NCBE (CONFLICTS)

POINT (1)(a) [25%] ISSUE: Were the requirements for federal diversity jurisdiction satisfied, even
though Plaintiff and decedent Tortfeasor were citizens of the same state? ANSWER: No. Even
though Plaintiff and Executor were from different states, diversity jurisdiction was unavailable
because, under 28 U.S.C. § 1332, “the legal representative of the estate of a decedent shall be
deemed to be a citizen only of the same State as the decedent.” Plaintiff and Executor’s decedent,
Tortfeasor, were both citizens of State X and consequently the requirements for diversity
jurisdiction were not met.

POINT (1)(b) [20%] ISSUE: Was Executor precluded from raising the issue of subject matter
jurisdiction because it failed to raise the issue earlier in the case and, in fact, conceded jurisdiction?
ANSWER: No. Executor’s motion to dismiss for lack of subject matter jurisdiction must be
considered on the merits, even if it was raised a year after Executor answered and admitted
jurisdiction.

POINT (2) [55%] ISSUE: When suit is filed to enforce a federal judgment in a state court, may the
defendant collaterally attack the federal judgment for lack of subject matter jurisdiction?
ANSWER: The state court is required to give the federal court judgment the same credit it would
receive in federal court. Because Executor litigated the issue of subject matter jurisdiction before
the federal court, lost, and did not appeal, Executor is precluded from relitigating the issue of
jurisdiction in state court.

ANSWER DISCUSSION:

The federal district court of State Y should have dismissed the action for lack of subject matter
jurisdiction. The federal court lacked jurisdiction because there was no diversity. Defendant Executor’s
citizenship is deemed to be the same as decedent Tortfeasor’s. Tortfeasor and Plaintiff were both citizens
of State X. The lack of diversity defeats diversity subject matter jurisdiction and is not waivable.
Defendant may raise subject matter jurisdiction at any time, and the federal court erred in denying the
motion. But the issue of subject matter jurisdiction was fully litigated in federal court, so Defendant is
precluded from raising it in a collateral attack on the judgment. The state court in State X must therefore
enforce the federal judgment.

ANSWER EXPLANATION:

Explanation to Point-One(a) (20-30%):

Even though Plaintiff and Executor were from different states, diversity jurisdiction was unavailable
because, under 28 U.S.C. § 1332, “the legal representative of the estate of a decedent shall be deemed to
be a citizen only of the same State as the decedent.” Plaintiff and Executor’s decedent, Tortfeasor, were
both citizens of State X and consequently the requirements for diversity jurisdiction were not met.

Plaintiff’s complaint included only a state-law negligence claim. The court, therefore, had subject matter
jurisdiction only if diversity jurisdiction existed under 28 U.S.C. § 1332. To satisfy the diversity statute,
the amount in controversy in the action must exceed $75,000 and the parties must be diverse.

Seperac-J19 Exam-Released MEE Essay Compilation © 2016-2020 701


Plaintiff’s complaint alleged damages in excess of $500,000. The facts state that Plaintiff was severely
injured and there is nothing to suggest that Plaintiff’s damages claim was made in bad faith.
Consequently, Plaintiff will be deemed to have satisfied the amount-in-controversy requirement of
$75,000.

The diversity of citizenship requirement, however, is not satisfied in this case. For purposes of
determining citizenship, an individual is a citizen of the state in which he or she is domiciled. Plaintiff and
decedent, Tortfeasor, were both citizens of State X. Defendant, Executor, was a citizen of State Y. On the
face of it, the plaintiff and defendant were citizens of different states, satisfying the diversity requirement.
However, diversity jurisdiction should have been unavailable because Executor stands in the shoes of
Tortfeasor and takes Tortfeasor’s citizenship for purposes of diversity. Under 28 U.S.C. § 1332, “the legal
representative of the estate of a decedent shall be deemed to be a citizen only of the same State as the
decedent.” Thus, because Executor was sued as a representative of Tortfeasor’s estate, Executor was
deemed to be a citizen of the same state as Tortfeasor, State X. Plaintiff, of course, is also a citizen of
State X, so diversity jurisdiction should have been unavailable.

Accordingly, the district court erred in denying the motion to dismiss for lack of jurisdiction. The motion
was proper when made, and it should have been granted because the parties were not diverse.

Explanation to Point-One(b) (15-25%):

Executor’s motion to dismiss for lack of subject matter jurisdiction must be considered on the merits,
even if it was raised a year after Executor answered and admitted jurisdiction.

Although the motion to dismiss for lack of subject matter jurisdiction was filed a year after Executor’s
answer to the complaint admitted the existence of jurisdiction, the court was nevertheless obliged to
consider the motion on its merits. The defense of lack of subject matter jurisdiction is not waivable and
may be raised “whenever it appears by suggestion of the parties that the court lacks jurisdiction of the
subject matter.” Accordingly, the court was obliged to consider the merits of Executor’s jurisdictional
challenge.

Explanation to Point-Two (50-60%):

The state court is required to give the federal court judgment the same credit it would receive in federal
court. Because Executor litigated the issue of subject matter jurisdiction before the federal court, lost, and
did not appeal, Executor is precluded from relitigating the issue of jurisdiction in state court.

State courts are required to give full faith and credit to the judgments of other states. Likewise, state,
federal, and territorial courts are required to give state court judgments the same credit that they would
receive in the courts of the rendering state. Although neither Article IV nor the full faith and credit statute
explicitly requires state courts to honor federal judgments, “it is well recognized that the same compulsion
controls and thus state courts must treat federal judgments as those judgments would be treated by the
federal courts themselves.”

In Durfee v. Duke, the United States Supreme Court announced the “general rule that a judgment is
entitled to full faith and credit even as to questions of jurisdiction when the second court’s inquiry
discloses that those questions have been fully and fairly litigated and finally decided in the court which
rendered the original judgment.” Here, Executor raised the lack of subject matter jurisdiction in the
federal court action. The issue was briefed, argued, and decided. Although the court’s decision on the
issue was mistaken, no appeal was taken and the court’s judgment became final. If the second suit had

Seperac-J19 Exam-Released MEE Essay Compilation © 2016-2020 702


been filed in federal court, the doctrine of issue preclusion or collateral estoppel would bar the defendant
from relitigating the issue of subject matter jurisdiction since the identical issue was fully and fairly
litigated in the first suit. Full faith and credit requires that the federal judgment have the same effect in the
state court. Thus, the state court should enforce the federal judgment notwithstanding the challenge to the
federal court’s subject matter jurisdiction. If Executor had wished to continue the jurisdictional challenge,
Executor should have filed an appeal from the decision of the federal district court.

[NOTE: The U.S. Supreme Court has ruled that the preclusive effect of the judgment of a federal court
sitting in diversity should be determined by the law of the state in which the federal court sits. Thus, the
preclusion rules of State Y would determine whether the State X court should enforce the federal
judgment. It is likely, however, that State Y would follow a rule of jurisdictional finality similar to the rule
in Durfee v. Duke: if the question of subject matter jurisdiction was fully and fairly litigated, the judgment
is not subject to collateral attack.]

Seperac-J19 Exam-Released MEE Essay Compilation © 2016-2020 703


#158-FEB 2003–MEE Q03: QUESTION THREE (CONFLICTS)

Husband and Wife married in State X in 1992 and resided there for the following 10 years. They have
three children, ages 6, 8, and 9.

In early February 2002, Husband told Wife that he wanted a divorce and was moving to State Z for a new
job. He asked Wife to let him take the children. Wife refused. The next day, Husband moved to State Z
without the children.

In March 2002, six weeks after arriving in State Z (the minimum period of residence for divorce in State
Z), Husband filed an action for divorce. In this action, he also sought primary custody of the three
children, asserting that Wife had been violent and abusive toward him throughout their marriage.

Wife was served with the summons and complaint in Husband’s action at the family home in State X.
Wife, who has never been to State Z, did not answer or appear in Husband’s divorce proceeding.

In July 2002, the State Z court granted Husband a default judgment granting Husband a divorce, dividing
their property, awarding him primary custody of the three children, and giving Wife “reasonable
visitation.” Husband served Wife with the judgment.

In August 2002, Wife allowed the children to go to State Z for a two-week visit with Husband, whom
they had not seen for nearly five months. This was the children’s first time in State Z. At the end of the
two-week period, Husband notified Wife that he would not return the children to Wife.

Wife immediately filed an action in State X for divorce, property division, and custody.

Husband appeared in Wife’s State X action and contested her claims. He sought enforcement of the State
Z judgment awarding him custody. He also argued that Wife’s other claims were precluded by Husband’s
State Z judgment of divorce.

1. Is the State X court required to enforce the State Z custody decree? Explain.

2. Are Wife’s claims for divorce and property division precluded by Husband’s State Z judgment?
Explain.

Seperac-J19 Exam-Released MEE Essay Compilation © 2016-2020 704


#158: F03-3 MEE: ANSWER: NCBE (CONFLICTS)

POINT (1) [50%] ISSUE: Under the Uniform Child Custody Jurisdiction Act (UCCJA), the
Uniform Child Custody Jurisdiction and Enforcement Act (UCCJEA), or the Parental Kidnapping
Prevention Act (PKPA), must the State X court enforce the State Z decree awarding custody to
Husband? ANSWER: No. The State X court is not required under the UCCJA, the UCCJEA, or
the PKPA to enforce the State Z decree awarding custody to Husband.

POINT (2) [50%] ISSUE: Should the State X court give preclusive effect to Husband’s State Z
judgment and therefore bar Wife’s claims for divorce and property division? ANSWER: Yes. The
State X court must give preclusive effect to Husband’s State Z judgment of divorce, but not to the
State Z determination respecting property division.

ANSWER DISCUSSION:

The State X court is not required to enforce the State Z decree awarding custody to Husband. Although
the custody provision entered by the State Z court was the first custody order entered, the State Z court
lacked jurisdiction under both the Uniform Child Custody Jurisdiction Act (UCCJA) and Uniform Child
Custody Jurisdiction and Enforcement Act (UCCJEA). Consequently, a court in State X applying the
UCCJA or UCCJEA need not recognize or enforce the custody provision in the State Z decree. Also, the
Parental Kidnapping Prevention Act (PKPA) would not require that the State X court give the State Z
decree full faith and credit. The assertion of jurisdiction by State Z was not consistent with the
requirements of the PKPA. The State X court should give preclusive effect to Husband’s State Z
judgment of divorce. The State Z court had jurisdiction to adjudicate the divorce, given Husband’s
domicile. However, the State X court should adjudicate Wife’s claim for property, since the State Z court
lacked in personam jurisdiction over Wife to adjudicate that claim, and since the State X court has such
jurisdiction over Husband.

ANSWER EXPLANATION:

Explanation to Point-One (45-55%):

The State X court is not required under the UCCJA, the UCCJEA, or the PKPA to enforce the State Z
decree awarding custody to Husband.

Every state has adopted either the UCCJA or the UCCJEA. These acts require that states recognize and
not modify sister state custody judgments that were based upon jurisdictional requirements similar to
those found in the UCCJA or UCCJEA.

The UCCJA and UCCJEA both set forth four bases upon which a state court may assert jurisdiction to
make an initial child custody determination. A court has jurisdiction to make an order if (1) the state is the
home state of the child at the time of the commencement of the proceeding (i.e., home state jurisdiction);
(2) the child and at least one contestant have a “significant connection” with the state (i.e., significant
connection jurisdiction); (3) the child is physically present in the state and has been abandoned or subject
to abuse (i.e., emergency jurisdiction); or (4) no other state appears to have jurisdiction under the Act, or
the state with jurisdiction has declined to exercise it, and it is in the best interest of the child for the court
to assert jurisdiction (i.e., default jurisdiction).

Seperac-J19 Exam-Released MEE Essay Compilation © 2016-2020 705


State Z did not have jurisdiction on any of these bases. The children had resided in State X since they
were born, so State Z was not the children’s home state. The children had never been to State Z, so there
was no basis for “significant connection” jurisdiction. State Z also lacked “default” jurisdiction because
State X could have exercised “home state” or “significant connection” jurisdiction, and State X had not
declined to exercise jurisdiction. Finally, the children were not present in State Z when the court issued its
custody decree, nor was there any evidence that they were in danger, so the State Z court lacked
emergency jurisdiction.

[NOTE: Although § 204 of the UCCJEA deals more directly than does the UCCJA with issues of
emergency jurisdiction in situations of spousal abuse, the UCCJEA still requires that the child be present
in the state issuing the emergency order.]

Therefore, under both the UCCJA and UCCJEA, the State Z court lacked jurisdiction to adjudicate
custody and the State X court need not enforce this aspect of the State Z decree.

Nor does federal law require that the State X court enforce the State Z decree. The federal Parental
Kidnapping Prevention Act dictates the circumstances in which the courts of one state must give full faith
and credit to an earlier custody decree from a sister state. The PKPA contains provisions that are
substantially similar to those of the UCCJA and UCCJEA. First, under the PKPA, the issuing court must
have had jurisdiction under its own state laws. Here, State Z surely did not. Second, under the PKPA,
even if State Z had jurisdiction under its own law, the PKPA requires enforcement only if the initial
assertion of jurisdiction was consistent with the jurisdictional standards set out in the PKPA, which are
very similar to the standards required by the uniform acts.

[NOTE: The principal difference between the jurisdictional requirements of the PKPA and the UCCJA is
that the PKPA permits jurisdiction to be based upon a “significant connection” only if the child does not
have a “home state.” For the reasons noted above, State Z’s assertion of jurisdiction was not consistent
with those standards. Since neither prong is satisfied here, the decree need not be enforced.

Accordingly, under all three statutes, State Z lacked jurisdiction to award Husband custody, and
therefore, State X need not enforce the custody decree.]>

Explanation to Point-Two (45-55%):

The State X court must give preclusive effect to Husband’s State Z judgment of divorce, but not to the
State Z determination respecting property division.

The principle of “divisible divorce” provides that the jurisdictional basis for termination of marriage is
different from the jurisdictional basis for deciding the incidents of marriage such as alimony, property
division, and child support. Divorce jurisdiction is based on “domicile” or some equivalent long-term
connection between at least one of the parties to the marriage and the forum state.

In this case, it appears that Husband had changed his domicile from State X to State Z. He had moved to
State Z and his new job there demonstrated his intent to remain there for the foreseeable future. Thus,
State Z was his new domicile at the time he filed for and obtained his default divorce decree.

Because Wife neither answered nor otherwise appeared in Husband’s State Z divorce action, she now may
collaterally attack that judgment in State X. However, based on the Williams II case, Wife may challenge
only the jurisdictional basis for the State Z divorce decree, not the merits. Unlike the situation in Williams
II, where the deserting spouses stayed only temporarily in Nevada to get divorced and then immediately

Seperac-J19 Exam-Released MEE Essay Compilation © 2016-2020 706


returned to their previous home state, Husband has remained in State Z and has accepted a new job there.
Because there is substantial evidence that State Z became the bona fide domicile of Husband when he
moved there, Wife’s collateral attack on the validity of the State Z divorce decree obtained by Husband
will fail. Thus, the State Z dissolution of the marriage must be recognized and will preclude Wife’s action
for divorce.

On the other hand, Husband’s claim that the State Z judgment precludes Wife’s claim in State X for
property division should be rejected. The jurisdictional basis for such claims is in personam. No factual
basis for in personam jurisdiction over Wife by the State Z court has been shown. As Wife had never been
in State Z, and did not answer the divorce petition, there were no “minimum contacts” sufficient to
support the assertion of in personam jurisdiction in State Z over Wife with respect to property division.
Thus, the State Z judgment would have no preclusive effect on Wife’s property division claim in State X.

Seperac-J19 Exam-Released MEE Essay Compilation © 2016-2020 707


#159-FEB 2000–MEE Q05: QUESTION FIVE (CONFLICTS/FAMILY LAW)

Husband and Wife married in State X. Shortly thereafter, they moved to State Y, where they were
domiciled for the next 10 years of their marriage. Wife never returned to State X. Recently, however,
Husband left Wife and moved to State X, which has a six-week residency requirement for a divorce
action. After living continuously for six weeks in State X, Husband filed for divorce in the appropriate
state court in State X. Wife was served by registered mail at the couple’s home in State Y. Although she
received the process, Wife did nothing in response.

The law of State X provides that a plaintiff is presumed to be a domiciliary of State X for divorce
purposes upon proof of six weeks’ continuous residency in State X at the time of the filing of the divorce
action. The long-arm statute of State X also provides for service of process on nonresident defendants by
registered mail at their last known address. It provides that jurisdiction may be exercised over nonresident
defendants in actions for marriage dissolution and for alimony “to the full extent permitted by the due
process clause of the United States Constitution.”

The State X court issued a decree granting Husband a divorce and ruling that neither Husband nor Wife
was entitled to spousal support. As soon as Wife learned of the decree, she filed her own action for
divorce and spousal support in the appropriate state court in State Y, where she was domiciled. Husband
was properly served with process in State Y, to which he had returned after receiving the State X divorce.

Consistent with the due process and full faith and credit clauses, may the court in State Y grant the
divorce and award spousal support to Wife? Explain.

Seperac-J19 Exam-Released MEE Essay Compilation © 2016-2020 708


#159: F00-5 MEE: ANSWER: NCBE (CONFLICTS/FAMILY LAW)

POINT (1) [33%] ISSUE: Did the court in State X have jurisdiction to dissolve the marriage such
that State Y is precluded from granting a divorce? ANSWER: Yes. A divorce decree dissolving a
marriage is entitled to full faith and credit if the court in the rendering state had jurisdiction to
enter it. A court has jurisdiction if at least one of the spouses is domiciled in the rendering state.

POINT (2) [33%] ISSUE: Did the court in State X violate due process by denying Wife spousal
support? ANSWER: Yes. A court that does not have in personam jurisdiction over a person may
not constitutionally assert jurisdiction to deny that person spousal support.

POINT (3) [33%] ISSUE: Does the obligation of the court in State Y to give full faith and credit to
the State X divorce decree preclude it from awarding spousal support to Wife? ANSWER: No.
State Y can probably award spousal support because State X granted an ex parte divorce in which
it did not have jurisdiction to deny Wife support.

ANSWER DISCUSSION:

The power of State Y to grant a divorce decree and award spousal support depends on the validity and
effect of the divorce and support decree issued by State X. Under the doctrine of divisible divorce, the
State X decree may be valid and effective as to the divorce, but not as to spousal support. Points One and
Two concern the validity of the State X decree in these respects. Point Three addresses State Y’s power to
award spousal support, in the event that State X’s decree is valid as to the divorce but not as to the spousal
support.

ANSWER EXPLANATION:

Explanation to Point-One (30-40%):

A divorce decree dissolving a marriage is entitled to full faith and credit if the court in the rendering state
had jurisdiction to enter it. A court has jurisdiction if at least one of the spouses is domiciled in the
rendering state.

The in-state domicile of at least one of the parties is sufficient for a state court’s jurisdiction in a divorce
action. A change of domicile is effected by moving to the new location with intent to remain there,
regarding it as one’s home. Whether Husband intended to regard State X as his home is, of course,
problematic, as he left State X immediately upon receiving his divorce. Arguably, at least, Husband
lacked the intent required for domicile and hence for State X to exercise jurisdiction.

As the burden of proof is on the litigant who is contesting the jurisdiction, jurisdictional challenges based
on lack of domicile when the durational residency requirement has been complied with rarely succeed.
Nonetheless, State Y can independently examine whether State X had jurisdiction. Since Husband does
appear to have satisfied the durational residency requirement, most courts would hold that the court in
State X did have jurisdiction to dissolve Husband and Wife’s marriage and, therefore, State Y cannot
issue a divorce decree.

Explanation to Point-Two (30-40%):

Seperac-J19 Exam-Released MEE Essay Compilation © 2016-2020 709


A court that does not have in personam jurisdiction over a person may not constitutionally assert
jurisdiction to deny that person spousal support.

In personam jurisdiction over a nonresident defendant is not needed for a court to take jurisdiction to
grant a divorce terminating the marital status of a party properly before it. But personal jurisdiction is
required before a court may assert jurisdiction to determine the defendant’s support rights and duties. This
is the doctrine of “divisible divorce.” Thus, a court’s decree can be entitled to full faith and credit with
respect to its marriage dissolution order, but not with respect to its order granting or foreclosing alimony.
Whether a court can constitutionally assert in personam jurisdiction over a nonresident defendant depends
on whether the nonresident defendant has or has had minimum contacts with the rendering state such that
the maintenance of the support action against him or her does not offend traditional notions of fair play
and substantial justice.

The fact that the parties were married in State X might seem to give State X a very strong interest in all
aspects of the marriage’s dissolution, including the financial aspects. Since both Husband and Wife
voluntarily chose to be married in State X and under the laws of State X, it might seem fair to conclude
that they purposefully availed themselves of the benefits and protection of the laws of State X. But the
U.S. Supreme Court in Kulko showed a marked reluctance to apply its minimum contacts theory to
personal relations matters as broadly as it has in ordinary commercial litigation. The parties in Kulko were
married in the rendering state, but the Court held that this contact alone was not sufficient in the context
of a claim for child support. It is also unlikely that it would be sufficient in the context of a claim for
spousal support or separate maintenance. That is especially the case where, as here, the marriage took
place many years ago and the wife has had no significant contact with the rendering state during the
intervening period. Following Kulko, a court would probably conclude that Wife does not have sufficient
contact with State X for courts in that state to have asserted in personam jurisdiction over her. Therefore,
the State X order cutting off her right to spousal support is not entitled to full faith and credit in State Y.

Explanation to Point-Three (30-40%):

State Y can probably award spousal support because State X granted an ex parte divorce in which it did
not have jurisdiction to deny Wife support.

While a valid divorce decree issued by the state in which the plaintiff is domiciled can terminate the
marital status of the parties, the state must have personal jurisdiction over the defendant spouse to
determine spousal support. Thus, even if State X’s divorce decree is accorded full faith and credit, the
effect of the decree is limited to a determination of marital status unless the court had personal jurisdiction
over the defendant.

Although courts typically do not have subject matter jurisdiction to award spousal support if it is not
awarded at the time of the divorce, most states have statutes or case law that allows the spouse who was
not subject to the in personam jurisdiction of the court granting the divorce to seek support
notwithstanding the valid divorce. Some states do not provide an exception, finding that once a divorce
decree entitled to full faith and credit with respect to marital status is issued, then spousal support cannot
be awarded. Nonetheless, regardless of the validity of the State X divorce decree, under the laws of most
states, State Y could determine spousal support.

Seperac-J19 Exam-Released MEE Essay Compilation © 2016-2020 710


#160-JUL 1995–MEE Q03: QUESTION THREE (CONFLICTS/CIVIL PROCEDURE)

Petrol, a company incorporated in State X, purchased an insurance policy by mail from Insurer Inc.
Insurer is incorporated and headquartered in State Y. The policy protected oil fields located in States X
and Y against out-of-control oil wells and other risks. In early 1991, two wells, one in State X and one in
State Y, went out of control, causing more than $75,000 damage to Petrol's fields. Petrol did not notify
Insurer about these losses until 1993.

Insurer promptly rejected Petrol's two claims. Insurer relied on a clause in its policy with Petrol that
denied coverage of any loss unless Petrol gave "written notice to Insurer in State Y of the loss as soon as
practicable." This denial of coverage is valid under the law of State X. The denial of coverage, however,
is not valid under the law of State Y, because Insurer cannot show prejudice caused by Petrol's delay in
notifying it of a loss. There is no federal law on the subject and no choice of law clause in the insurance
contract. State X courts apply First Restatement rules to resolve choice of law issues. State Y courts
proceed first under governmental interest analysis; in the event of a true conflict, they also apply the First
Restatement.

Petrol properly filed a diversity action against Insurer in the federal district court in State Y. After the
court had determined the appropriate choice of law, Insurer successfully moved for a change of venue to
the federal district court for State X. The case therefore is pending before the federal district court of State
X.

Should the law of State X or the law of State Y be applied to determine the validity of the denial-of-
coverage clause in the insurance contract? Explain.

Seperac-J19 Exam-Released MEE Essay Compilation © 2016-2020 711


#160: J95-3 MEE: ANSWER: NCBE (CONFLICTS/CIVIL PROCEDURE)

POINT (1) [20%] ISSUE: In the initial phase of the federal diversity action, as filed in the federal
district court of State Y, which choice-of-law rules apply? ANSWER: In the initial phase of the
federal diversity action filed in State Y, the choice–of–law rules of State Y would apply under the
Erie doctrine.

POINT (2) [20%] ISSUE: After the change of venue to the federal district court for State X, which
law applies? ANSWER: Following the transfer or change of venue from the federal district court of
State Y to that of State X, the State X court must follow State Y choice–of–law rules.

POINT (3) [60%] ISSUE: Do the appropriate choice-of-law rules of State Y call for application of
the substantive law of State Y, which would allow Petrol to claim the proceeds from Insurer under
its insurance contract, or the substantive law of State X, which would give effect to the denial-of-
coverage clause? ANSWER: Under governmental interest analysis, this case presents a situation
where neither state has a significant interest in applying its law. Under such circumstances, a court
would probably apply local law. Thus, the law of State Y would apply to the losses in both State Y
and State X. Therefore, Insurer's denial-of-coverage clause would not bar recovery by Petrol
against Insurer.

ANSWER EXPLANATION:

Explanation to Point-One (15-25%):

In the initial phase of the federal diversity action filed in State Y, the choice–of–law rules of State Y
would apply under the Erie doctrine.

The Erie doctrine, set forth in Erie Railroad v. Tompkins, applies to choice-of-law rules. Under Klaxon,
the choice-of-law rules of the state in which the federal court sits – that is, of State Y in the initial phase –
would be used to determine which state's law governs the validity of the denial-of-coverage clause in the
contract. The principal reason for this application of Erie is to deter forum- shopping as between federal
and state courts in the same state and thereby to protect the integrity and uniformity of state law within its
territory.

Explanation to Point-Two (15-25%):

Following the transfer or change of venue from the federal district court of State Y to that of State X, the
State X court must follow State Y choice–of–law rules.

Van Dusen v. Barrack, established that a transfer of a civil action initiated by a defendant in a federal
case, under 28 U.S. Code § 1404, does not affect the choice-of-law rule that was or would have been
adopted by the transferor court. The case of Ferens v. John Deere Company, carries the Erie-based, anti-
forum-shopping rule of Van Dusen one step further. Ferens held that even where the plaintiff institutes the
transfer, the transferor court's rules generally would govern in the transferee court. Thus, State Y choice-
of- law rules would govern after the transfer of the action to the federal court in State X.

Explanation to Point-Three (55-65%):

Seperac-J19 Exam-Released MEE Essay Compilation © 2016-2020 712


Under governmental interest analysis, this case presents a situation where neither state has a significant
interest in applying its law. Under such circumstances, a court would probably apply local law. Thus, the
law of State Y would apply to the losses in both State Y and State X. Therefore, Insurer's denial-of-
coverage clause would not bar recovery by Petrol against Insurer.

Because the federal district court in State X will use State Y choice-of-law rules, it will decide whose law
to apply by following the governmental interest analysis approach favored by State Y courts.
Governmental interest analysis requires a step-by-step series of considerations. Although many states
have adopted variations of this approach, the basic scheme is as follows:

1. When a court is asked to apply the law of a foreign state different from the law of the forum, it should
inquire into the policies expressed in the respective laws and into the circumstances in which it is
reasonable for the respective states to assert an interest in the application of those policies. In making
these determinations the court should employ the ordinary processes of construction and interpretation.

2. If the court finds that one state has an interest in the application of its policy in the circumstances of the
case and the other has none, it should apply the law of the only interested state.

3. If the court finds an apparent conflict between the interests of the two states, it should reconsider. A
more moderate and restrained interpretation of the policy or interest of one state or the other may avoid
conflict.

4. If, upon reconsideration, the court finds that a conflict between the legitimate interests of the two states
is unavoidable, it should apply the law of the forum.

It should be noted that under the facts State Y has substituted the First Restatement for the law of the
forum as a “tie-breaker” in the event of a true conflict.

State X law requires prompt notice to the insurer and thereby validates the denial-of-coverage clause in
the Insurer contract. Such a law is designed to protect State X insurers, purely and simply. The contrary
policy of State Y, on the other hand, is to protect local (State Y) insured parties. Neither state presumably
has an altruistic interest in protecting the interests of the other state's natural or corporate citizens; the
laws are designed to protect local persons. As a result, neither state has a real interest in application of its
law to this case. State X has no interest in seeing its insurer – protective policy applied to protect a State
Y insurer. State Y has no interest in applying its pro- coverage policy to protect an insured party from
State X. Thus, this a classic “unprovided- for case,” with neither a false conflict (only one state has an
interest, as in step 2, above) nor a true conflict (both states have an interest, as in steps 3 and 4, above).

There is no uniform method by which state courts resolve this kind of case. It is possible that State Y
courts might apply the First Restatement because that is the technique by which they resolve a true
conflict. Under the First Restatement, the question of the validity of a clause in the contract would be
governed by the place of contracting. For insurance contracts delivered by mail, that is the place of
posting, presumably State Y. If the issue is viewed as a performance issue, the law of the place where
notification of a loss would be completed (Petrol's performance, in this case) would apply. In this case,
the law of the place of performance would be the law of State Y, which invalidated the notice clause in
the insurance contract. Finally, the law of State Y might also be applied to invalidate the clause on another
theory: that courts will apply their own law, especially in border line cases such as this one, unless there
are compelling reasons for choosing foreign law. In any event, it would appear that State Y law would
apply to re solve this “unprovided- for case.”

Seperac-J19 Exam-Released MEE Essay Compilation © 2016-2020 713


It might be argued that State Y does have an interest in applying its law to protect Petrol with respect to
the State Y well. The argument would be that State Y has an interest in ensuring insurance coverage of all
local losses, regardless of the domicile of the insured party (in this case, State X). The general ration ale
would be that protection of out- of- - state insurers would promote external insurance coverage of local
losses and relieve the state of the potential economic burden that might result if insurance was unavailable
to cover a particular loss within the state.

Thus, State Y law would apply, either by determining that this is an unprovided-for case to be resolved in
favor of the local law (for any of the reasons mentioned above) or by finding a false conflict (with respect
to the State Y well) because only State Y would have an interest in applying its pro-coverage law in order
to confirm insurability and thereby encourage external insurance cover age of losses in State Y.

Seperac-J19 Exam-Released MEE Essay Compilation © 2016-2020 714


#161-FEB 1995–MEE Q06: QUESTION SIX (CONFLICTS)

Foodco, a food processing company, has its place of incorporation and its principal place of business in
State A. Foodco relies extensively on an independent distributor, located in State A, to sell its products to
restaurants in State A and State B. In addition, the distributor arranges a monthly shipment of Foodco's
sausage to a customer in State C. This sale amounts to less than 5% of Foodco's monthly sales.

Although it obtains most of its supplies from wholesalers in State A, Foodco frequently orders spices
from Spiceco, located in adjoining State D, by telephoning Spiceco from State A. For several years,
Foodco paid its account by check mailed at the end of each month to Spiceco's office in State D.

For the last three months, Foodco has failed to make payments to suppliers. Its balance with Spiceco has
grown to $6,000. After making an unsuccessful demand for payment, Spiceco filed suit against Foodco in
a State D trial court. Spiceco attached $5,000 worth of Foodco sausage that had been stored in a
warehouse in State D by Foodco's distributor, pending shipment to the customer in State C. Spiceco then
served process on Foodco in State A, informing it of the attachment and of the commencement of the
action in State D.

State D statutes permit creditors like Spiceco to institute proceedings by attachment. The statutes permit
State D courts 1) to adjudicate claims against nonresidents to the extent of any property located in the
state and 2) to assert long arm jurisdiction over nonresident corporations that "transact any business" in
the state or fail to perform any contractual obligations with substantial ties to the state. State D courts
construe these statutes as authorizing jurisdiction to the full extent permitted by the Due Process Clause of
the United States Constitution.

Foodco moved to dismiss the State D action for lack of jurisdiction. In addition to reciting the facts,
Foodco supported its motion with an uncontradicted affidavit that no employee of Foodco has ever met
with Spiceco in State D.

Should the court grant or deny Foodco's motion to dismiss? Explain.

Seperac-J19 Exam-Released MEE Essay Compilation © 2016-2020 715


#161: F95-6 MEE: ANSWER: NCBE (CONFLICTS)

POINT (1) [16%] ISSUE: Does the seizure of Foodco's sausage in State D provide a constitutional
basis for the assertion of jurisdiction over Spiceco's claim against Foodco? ANSWER: No. Seizure
of property within a state, standing alone, no longer supplies a constitutional basis for the assertion
of jurisdiction over claims against nonresidents.

POINT (2) [47%] ISSUE: Does the contractual relationship between Foodco and Spiceco create
sufficient contacts with State D to justify the assertion of long-arm jurisdiction over Foodco?
ANSWER: No. Foodco's contractual relationship with Spiceco may not be a sufficient contact with
State D to justify the assertion of jurisdiction over Foodco.

POINT (2)(a) [26%] ISSUE: What significance, if any, should the State D court ascribe to the
seizure of Foodco's property in its analysis of minimum contacts? ANSWER: The contacts
implicated by the presence of Foodco's sausage in State D lend some additional support to the
assertion of jurisdiction.

POINT (2)(b) [11%] ISSUE: Assuming that Foodco has the requisite contacts with State D, is the
assertion of jurisdiction reasonable? ANSWER: Yes. Assuming that Foodco has established the
requisite minimum contacts with State D, the assertion of jurisdiction satisfies the test of
reasonableness.

ANSWER EXPLANATION:

Explanation to Point-One (10-20%):

Seizure of property within a state, standing alone, no longer supplies a constitutional basis for the
assertion of jurisdiction over claims against nonresidents.

In Shaffer v. Heitner, the Supreme Court held that all assertions of judicial jurisdiction over nonresidents,
including those that begin with the attachment of property, must comport with the minimum contacts
standard of International Shoe. The court thus abandoned the suggestion in Pennoyer v. Neff, that a state
can adjudicate claims against the owner of property found within the state merely by attaching or
otherwise asserting control over the property. Presence of the property may still be relevant in assessing
minimum contacts. As applied to Spiceco's seizure of Foodco's sausage, Shaffer teaches that the mere fact
of the seizure of property does not suffice to empower the court of State D to adjudicate the claim. Rather,
the State D court must find that Foodco has minimum contacts with the state such that the assertion of
jurisdiction does not offend "traditional notions of fair play and substantial justice."

Explanation to Point-Two (40-50%):

Foodco's contractual relationship with Spiceco may not be a sufficient contact with State D to justify the
assertion of jurisdiction over Foodco.

State D's long-arm statute, allowing creditors to proceed by attachment and to assert claims against non-
resident corporations that “transact any business” in the state, provides the necessary authorization for its
courts to exercise the specific type of jurisdiction invoked here. The statute will be construed to reach the

Seperac-J19 Exam-Released MEE Essay Compilation © 2016-2020 716


full extent of jurisdiction permitted by the due process clause, so this analysis proceeds to a consideration
of “minimum contacts.”

The leading minimum contacts case in the field of contract, Burger King v. Rudzewicz, provides the
framework for analyzing the assertion of jurisdiction here. Burger King held that Florida could
constitutionally assert jurisdiction over the defendant Rudzewicz in an action seeking to recover unpaid
installments due under a franchise contract. Consistent with such earlier cases as WorldWide Volkswagen
v. Woodson, and Hanson v. Denckla, the court required evidence that the defendant has engaged in some
form of conduct sufficient to support a finding that the defendant had purposefully availed himself of the
benefits and protections of the forum.

In Burger King, the court found that Rudzewicz, a Michigan resident with no other ties to Florida, had
purposefully availed himself of the benefits of Florida law by entering into a franchise relationship with
Burger King, a large restaurant company with its headquarters in that state. The court emphasized both the
commercial significance of, and the tight controls imposed by, the franchise contract – pointing out that
the contract regulated all aspects of the enterprise, resulted in the purchase by Rudzewicz of $165,000
worth of equipment, and called for payment of $1 million over the 20-year life of the relationship. It also
emphasized a provision in the contract that made Florida law applicable to all disputes. The fact that
Rudzewicz had not set foot in Florida was not fatal to the assertion of jurisdiction, in as much as parties
often conduct business by mail or telephone. The court was nonetheless quick to point out that each
jurisdictional inquiry would turn on its particular facts, that not every breach of contract would justify the
assertion of jurisdiction over nonresident contract debtors – a dictum apparently aimed at protecting
consumers and small debtors from default judgments obtained in inconvenient fora.

Burger King provides some support for the assertion of jurisdiction over the claim against Foodco.
Foodco, after all, did reach out repeatedly to Spiceco in State D as a source of supply and can be said to
have purposefully availed itself of the benefits and protections of State D law on that basis. Moreover, the
relationship had been established for some years; it was not a one- shot order. On this basis, it seems
unlikely that the fact that Foodco's employees had never met personally with Spiceco in State D will carry
much weight. Foodco can reach out as purposefully to State D by placing telephone orders as Rudzewicz
did to Florida.

On the other hand, the relationship between Spiceco and Foodco was not as closely regulated as that in
Burger King. Nothing suggests that the parties had negotiated any written contract to govern their
relationship (aside perhaps from the writings contained in order forms and bills), let alone a detailed
contract with a State D choice-of- law provision. Moreover, Foodco retains complete control over the
management of its affairs – a fact that undercuts the claim that Foodco has purposefully affiliated itself
with an out-of-state enterprise. (But while the Burger King court made much of the extent of contract
negotiations and the scope of the contract's controls, one might at least question whether such factors
enjoy dispositive weight. After all, the Uniform Commercial Code or the otherwise applicable common
law of the two states supplies much of the detail that Foodco and Spiceco omitted when they entered into
a more informal relationship.) In any case, the contract here does not involve the same degree of
commercial significance as that in Burger King. While it does not involve a consumer purchase of the
kind that had troubled the lower court and led to the limiting dictum in the court's opinion in Burger King,
a few thousand dollars’ worth of spice does not match the amount owing in Burger King.

Explanation to Point-Two(a) (20-30%):

Seperac-J19 Exam-Released MEE Essay Compilation © 2016-2020 717


The contacts implicated by the presence of Foodco's sausage in State D lend some additional support to
the assertion of jurisdiction.

Given that the spice orders and the failure to pay, standing alone, may not suffice to establish Foodco's
minimum contacts with State D, the property seizure deserves attention. Shaffer suggests that the
assertion of power over property unrelated to the claim adds little to the jurisdictional inquiry. But here,
one can argue that the presence of property counts. To begin with, one might argue that the sausage
contains Spiceco spices, and thus bears some relationship to the claim. But such an argument seems
speculative – we do not know whether Spiceco spices were used in making the specific sausage that was
seized in State D. In any case, a focus on the claimed relationship tends to make the inquiry more
formalistic than the Shaffer court intended. Rather, the analysis should properly focus on the extent to
which use of State D facilities to ship sausage to State C establishes additional ties that support the
assertion of jurisdiction.

Consideration of this question involves the role of the independent distributor. The problem states that
Foodco relies on the distributor to sell its product and, in particular, that the distributor arranges the
shipments to State C. The presence of the property in a warehouse in State D thus appears to owe more to
the convenience of the distributor than to any purposeful involvement by Foodco, and could be
characterized as the kind of “unilateral conduct” that Hanson v. Denckla considered insufficient for
minimum contact purposes. It is nonetheless undeniable that Foodco derives indirect benefits in the form
of revenues derived from the distributor's use of State D facilities to make the shipment to State C. While
these benefits have little to do with the relationship between Spiceco and Foodco, they do establish ties to
State D. A court concerned about the absence of sufficient contractual ties between Spiceco and Foodco
might well ascribe dispositive significance to such additional contacts.

Explanation to Point-Two(b) (05-15%):

Assuming that Foodco has established the requisite minimum contacts with State D, the assertion of
jurisdiction satisfies the test of reasonableness.

Assuming State D concludes that Foodco has the requisite minimum contacts, it should have little
difficulty in regarding the assertion of jurisdiction as reasonable – the second aspect of the International
Shoe test. State D appears to have an interest in making a forum available to its domestic spice firm, and a
good many of the witnesses (those employed by Spiceco) will find the State D forum more convenient. In
recent years, the court has declared unreasonable only the California court's assertion of jurisdiction over
a claim for indemnity involving companies whose relationship bore no connection to the United States.
State D's more obvious stake in providing a convenient forum for Spiceco probably establishes a
reasonable basis for the assertion of jurisdiction that overcomes any claim of inconvenience by Foodco.

Seperac-J19 Exam-Released MEE Essay Compilation © 2016-2020 718


CORPORATIONS & LLCS: 33 OF 49 MEE EXAMS: (67%)
#162-JUL 2019–MEE Q03: QUESTION THREE (CORPS-LLCS)

Parent Inc., a company in the renewable energy business, has several subsidiaries. In all cases, Parent
maintains control of its subsidiaries by selecting the members of each subsidiary’s board of directors,
most of whom also serve as officers and employees of Parent.

One of the subsidiaries, HomeSolar Inc. (incorporated in a jurisdiction that has adopted a version of the
Model Business Corporation Act), was acquired three years ago by Parent. Parent owns 80% of
HomeSolar’s voting shares, with the remaining shares publicly traded on a national stock exchange.
HomeSolar manufactures and sells products exclusively for the residential solar power market.

Another subsidiary, IndustrialSolar Inc., is wholly owned by Parent and manufactures products
exclusively for the industrial solar power market.

A shareholder of HomeSolar, after making a proper demand on the board to which the board failed to
timely respond, brought a derivative suit against Parent, as the controlling shareholder of HomeSolar,
making the following allegations:

(1) HomeSolar has not paid dividends since being acquired by Parent three years ago. In SEC
filings, HomeSolar has explained that its no-dividend policy provides funds for its research and
development budget as it seeks to develop new products for the residential solar power market in
which it operates. Nonetheless, HomeSolar has more than adequate earnings and was obligated to
pay dividends to its shareholders.

(2) Since acquiring HomeSolar, Parent has caused HomeSolar to purchase the “rare earth” minerals
necessary for the manufacture of its residential solar panels from SolarMaterials Corp., a wholly
owned subsidiary of Parent. SolarMaterials was created for the purpose of acquiring such minerals
and reselling them to the various renewable energy subsidiaries of the Parent group. The long-term
contract under which HomeSolar purchases rare earth minerals from SolarMaterials, however, sets
prices significantly higher than the current market prices under similar long-term contracts for such
minerals.

(3) After Parent learned about a large government grant to develop industrial-scale solar projects,
Parent caused IndustrialSolar to apply for and secure this grant, denying HomeSolar the
opportunity to obtain this grant.

1. Did Parent breach any duties to HomeSolar with respect to HomeSolar’s no-dividend policy?
Explain.

2. Did Parent breach any duties to HomeSolar with respect to HomeSolar’s contract with
SolarMaterials for the purchase of rare earth minerals? Explain.

3. Did Parent breach any duties to HomeSolar by denying HomeSolar the opportunity to apply for
the government grant? Explain.

Seperac-J19 Exam-Released MEE Essay Compilation © 2016-2020 719


#162: J19-3 MEE: ANSWER: NCBE (CORPS-LLCS)

POINT (1) [35%] ISSUE: Did Parent breach any duties to HomeSolar with respect to HomeSolar’s
no-dividend policy? ANSWER: No. Parent likely did not violate its fiduciary duties of loyalty and
care by causing HomeSolar to institute a no-dividend policy, given that the policy applied equally to
all shareholders of HomeSolar and had the purpose of ensuring funding for the company’s research
and development budget.

POINT (2) [40%] ISSUE: Did Parent breach any duties to HomeSolar with respect to HomeSolar’s
contract with SolarMaterials for the purchase of rare earth minerals? ANSWER: Yes. Given that
Parent caused HomeSolar to enter into a contract for the purchase of minerals with Parent’s wholly
owned subsidiary SolarMaterials at significantly higher prices than current market prices under
similar long-term contracts for the same minerals, Parent engaged in unfair self-dealing and thus
breached its duty of loyalty to HomeSolar. The business judgment rule does not apply to this
conflict-of interest transaction.

POINT (3) [25%] ISSUE: Did Parent breach any duties to HomeSolar by denying HomeSolar the
opportunity to apply for the government grant? ANSWER: No. Parent likely did not violate its
fiduciary duties by allocating the opportunity to apply for the government grant to its wholly owned
subsidiary IndustrialSolar rather than its partially owned subsidiary HomeSolar, given that the
grant was for development of industrial solar projects and thus beyond the residential business of
HomeSolar.

ANSWER DISCUSSION:

[NOTE: The question involves a corporation incorporated in an MBCA jurisdiction, thus requiring
examinees to consider the relevant provisions of the MBCA. The MBCA, however, does not specify the
fiduciary duties of controlling shareholders in a publicly traded corporation. Thus, in determining whether
Parent breached its duties to HomeSolar and its minority shareholders, the law of the state of
incorporation would likely look to general principles of corporate law in a matter of first impression.
Given the lack of indication of case law on the point in the jurisdiction of incorporation, an examinee
should look to general principles of corporate law, specifically those related to the fiduciary duties of
controlling shareholders. General principles of corporate law, including the case law of Delaware, are thus
relevant to the analysis of the question.]

Controlling shareholders (including parent corporations) generally owe fiduciary duties to their partially
owned subsidiaries and may not use their power to benefit themselves at the expense of the subsidiary and
its minority shareholders. Here, Parent likely did not violate its fiduciary duties of loyalty and care by
causing its subsidiary, HomeSolar, to adopt a no-dividend policy, given that the policy applied equally to
all shareholders of HomeSolar and had the purpose of ensuring funding for HomeSolar’s research and
development budget. Business dealings by parent corporations with partially owned subsidiaries can be
set aside as self-dealing transactions unless approved by a majority of disinterested directors or
shareholders or unless, in the absence of such approval, they are judicially determined to be fair. Here,
Parent breached its fiduciary duty of loyalty by causing HomeSolar to enter into a long-term contract with
SolarMaterials, a wholly owed subsidiary of Parent, at prices significantly higher than current market
prices under similar long-term contracts for the minerals. Because Parent selected the entire board of
HomeSolar, there are no disinterested directors to approve the transaction. Furthermore, there are no facts
to suggest that the minority shareholders of HomeSolar approved the transaction. And, because

Seperac-J19 Exam-Released MEE Essay Compilation © 2016-2020 720


HomeSolar paid higher than market prices, the transaction cannot satisfy the fairness test. The
presumption of the business judgment rule does not apply to this conflict-of-interest transaction. There are
no facts indicating that Parent or the directors of HomeSolar breached a duty of care. Parent did not
violate its fiduciary duties by allocating the opportunity to apply for the government grant to its wholly
owned subsidiary IndustrialSolar and not to its partially owned subsidiary HomeSolar, given that the grant
was for development of industrial solar projects and thus beyond the existing residential business of
HomeSolar.

ANSWER EXPLANATION:

Explanation to Point One (35%):

Parent likely did not violate its fiduciary duties of loyalty and care by causing HomeSolar to institute a
no-dividend policy, given that the policy applied equally to all shareholders of HomeSolar and had the
purpose of ensuring funding for the company’s research and development budget.

The MBCA does not specify the duties of controlling shareholders to a controlled corporation or its
minority shareholders. Instead, the duties of controlling shareholders generally arise as a matter of the
court’s “inherent equity power” to fashion fiduciary duties owed by majority shareholders to minority
shareholders. Actions by majority shareholders “may be reviewable by a court under its inherent equity
power to review transactions for good faith and fair dealing to minority shareholders.

Generally, courts have examined business dealings between a controlling shareholder (such as a parent
corporation) and the controlled corporation using a fairness test. But when the transaction does not
involve self-dealing (as is the case with respect to dividends payable to all shareholders of the controlled
corporation), then the “business judgment” standard applies. In a suit against a parent of a subsidiary,
overreaching must be shown. That is, the fairness test applies to parent-subsidiary dealings only where the
“parent causes the subsidiary to act in such a way that the parent receives something from the subsidiary
to the exclusion of, and detriment to, the minority stockholders.”

Here, the shareholder challenges the no-dividend policy of HomeSolar instituted by Parent. The no-
dividend policy affects the shareholder and all other HomeSolar shareholders (including Parent) equally
and thus is not a self-dealing transaction in which Parent receives something to the exclusion of other
shareholders.

Parent did not breach any duty of care. Under a “business judgment” standard, it is sufficient if Parent can
offer a rational business justification for the no-dividend policy, which it has by explaining that the no-
dividend policy is meant to provide funds for HomeSolar’s research and development budget as the
subsidiary seeks to develop new products for the residential solar power market. Decisions subject to the
business judgment standard “will not be disturbed (by a court substituting its own notions of what is or is
not sound business judgment) if they can be attributed to any rational business purpose. The business
judgment rule “bars judicial inquiry into actions of corporate directors taken in good faith and in the
exercise of honest judgment in the lawful and legitimate furtherance of corporate purposes”.

[NOTE: An examinee who seeks to analyze the question of Parent’s duties to the complaining shareholder
under the “oppression” standard created by the involuntary dissolution provisions of the MBCA should
not receive credit. For one thing, the question asks whether Parent breached any duties to HomeSolar,
not whether the court should order involuntary dissolution. For another, an “oppression” claim is
appropriate only for shareholders of corporations that are not widely held. For example, shareholders in

Seperac-J19 Exam-Released MEE Essay Compilation © 2016-2020 721


a publicly traded corporation, such as one whose shares are listed on the New York Stock Exchange, will
normally have the ability to sell their shares if they are dissatisfied with current management.]

Explanation to Point Two (40%):

Given that Parent caused HomeSolar to enter into a contract for the purchase of minerals with Parent’s
wholly owned subsidiary SolarMaterials at significantly higher prices than current market prices under
similar long-term contracts for the same minerals, Parent engaged in unfair self-dealing and thus breached
its duty of loyalty to HomeSolar. The business judgment rule does not apply to this conflict-of interest
transaction.

As described above, business dealings between a parent corporation (or its wholly owned subsidiary) and
a partially owned subsidiary can involve self-dealing and a breach of the duty of loyalty when the parent
causes the partially owned subsidiary to enter into a transaction that prefers the parent (or its wholly
owned subsidiary) at the expense of the partially owned subsidiary and its minority shareholders. This
breach cannot be cured here because there are no disinterested directors to approve the transaction, as
Parent appointed all the directors of HomeSolar, and there is no evidence that the minority shareholders of
the partially owned subsidiary approved the transaction. This means that liability can be avoided only if
the transaction is judicially determined to be fair.

Generally, the person seeking to justify a self-dealing transaction has the burden of proving its fairness to
the corporation. The MBCA has described “fairness,” in connection with directors’ conflicting-interest
transactions, to include not only “the market fairness of the terms of the deal—whether it is comparable to
what might have been obtainable in an arm’s-length transaction – but also whether the transaction was
one that was reasonably likely to yield favorable results (or reduce detrimental results)” for the
corporation. Thus, contractual dealings by a partially owned subsidiary (here HomeSolar) with a wholly
owned subsidiary (here SolarMaterials) in a corporate group must not only be shown to reflect the terms
obtainable in comparable market transactions but must also be shown to be beneficial to the partially
owned subsidiary. Here, given that the prices set in the contract between HomeSolar and SolarMaterials
were significantly higher than current market prices under similar long-term contracts for the same
minerals, Parent cannot establish the fairness of the contract. Thus, Parent breached its fiduciary duty of
loyalty to HomeSolar.

[NOTE: The business judgment rule does not apply to conflict-of-interest transactions and thus is no
defense to a claim that the defendant breached its duty of loyalty, and no credit should be given to an
examinee who seeks to apply the business judgment rule in the context of a discussion of the duty of
loyalty.]

[NOTE: Some examinees may conclude that the self-dealing transaction in this case is subject to “entire
fairness” review, which requires a showing of both “fair price” and “fair dealing.” The “entire fairness”
review applies to parent-subsidiary merger transactions in which minority shareholders of the subsidiary
are forced out of the corporation. However, it is unclear whether the MBCA would apply this heightened
standard to typical intra-group corporate dealings not involving a change in control. For example, courts
have rejected the “entire fairness” test of Kahn when an interested transaction is approved by both a
majority of informed, independent, disinterested directors (or a special board committee) and an
informed majority of disinterested shareholders. Thus, an examinee should not receive additional credit
for discussing the absence of “fair dealing” in this case where the challenged contract was approved by
neither disinterested directors nor disinterested shareholders. Here, the absence of “fair price” is critical
to the analysis of this intra-group self-dealing transaction.]

Seperac-J19 Exam-Released MEE Essay Compilation © 2016-2020 722


Lastly, there are no facts to indicate that Parent or the directors of HomeSolar breached a duty of care.
First, there are no facts indicating that Parent caused the HomeSolar directors to act with a faulty
decision-making process. Second, there are no facts indicating that the above-market transaction was
“wasteful,” meaning that no reasonable person would have paid those higher prices. There could be any
number of reasons why HomeSolar agreed to pay higher prices, and there are no facts indicating that the
deliberation processes were faulty.

Explanation to Point Three (25%):

Parent likely did not violate its fiduciary duties by allocating the opportunity to apply for the government
grant to its wholly owned subsidiary IndustrialSolar rather than its partially owned subsidiary HomeSolar,
given that the grant was for development of industrial solar projects and thus beyond the residential
business of HomeSolar.

The MBCA does not address the duties of controlling shareholders to not usurp corporate opportunities of
partially owned corporations. Nonetheless, an official comment to the MBCA explains the corporate
opportunity doctrine in connection with the duties of directors:

The corporate opportunity doctrine (applicable to directors) is anchored in a significant body of case law
clustering around the core question whether the corporation has a legitimate interest in a business
opportunity, either because of the nature of the opportunity or the way in which the opportunity came to
the director, of such a nature that the corporation should be afforded prior access to the opportunity before
it is pursued (or, to use the case law’s phrase, “usurped”) by a director. In a similar vein, the American
Law Institute defines a corporate opportunity, for purposes of directors and senior executives, generally as
a business opportunity where either “the person offering the opportunity expects it to be offered to the
corporation,” the opportunity “would be of interest to the corporation,” or the opportunity is “closely
related to a business in which the corporation is engaged or expects to engage.”

For business opportunities allocated within a corporate group, courts have accepted that the parent should
have some leeway in allocating business opportunities within the group. The duty to put the best interest
of a partially owned subsidiary above any interest does not mean that the controller has to subrogate his
own interests so that the minority stockholders can get the deal that they want.

Here, although HomeSolar might have wanted to receive the government grant, there is no indication that
HomeSolar was capable of using the grant to pursue industrial solar projects; instead, the facts indicate
that HomeSolar’s line of business is developing residential solar projects. Finally, it is unclear that the
government would have given the grant to HomeSolar, as opposed to the wholly owned subsidiary
IndustrialSolar.

Seperac-J19 Exam-Released MEE Essay Compilation © 2016-2020 723


#163-JUL 2018–MEE Q06: QUESTION SIX (CORPS-LLCS)

A woman and a man decided to start a solar-panel installation business in State X. They agreed to
incorporate the business and to be equal shareholders. They also agreed that the woman would be solely
responsible for managing the business.

On November 10, the woman mailed to the Secretary of State of State X a document titled “Articles of
Incorporation.” The document included the name of the corporation (Solar Inc.), the name and address of
the corporation’s registered agent, and the woman’s name and address (as incorporator). The woman,
however, inadvertently failed to include in the document the number of authorized shares, as required by
the business corporation act of State X, which in all respects comports with the Model Business
Corporation Act (1984, as revised). The woman signed the document and included a check to cover the
filing fee.

On November 20, the woman, assuming that the articles of incorporation had been filed and purporting to
act on behalf of the corporation, entered into a one-year employment contract with a solar-panel installer.
The woman signed the employment contract as “President, Solar Inc.” and the installer signed
immediately below.

On November 30, the woman received a letter from the Secretary of State’s office returning the articles of
incorporation and her check. The letter stated that the articles, although received on November 15, had not
been filed because they failed to include the number of authorized shares, as required by state law.

On receiving this letter, the woman immediately revised the articles by adding the number of authorized
shares. On December 5, the woman mailed back the revised articles to the Secretary of State’s office,
along with another check to cover the filing fee. The revised articles of incorporation were received and
filed by the Secretary of State’s office on December 10.

Six months later, Solar Inc. went out of business and the installer’s employment was terminated.

1. When did Solar Inc. come into existence? Explain.

2. Is the woman personally liable to the installer on the employment contract that she signed?
Explain.

3. Is the man personally liable to the installer on the employment contract? Explain.

Seperac-J19 Exam-Released MEE Essay Compilation © 2016-2020 724


#163: J18-6 MEE: ANSWER: NCBE (CORPS-LLCS)

POINT (1) [10%] ISSUE: When did Solar Inc. come into existence? ANSWER: The corporation
was formed on December 10, when its articles of incorporation were filed by the Secretary of
State’s office.

POINT (2)(a) [25%] ISSUE: Under the state’s business corporation statute, could the woman be
personally liable on the contract with the installer that she signed as “President, Solar Inc.”?
ANSWER: No. The woman would not be personally liable under MBCA § 2.04 because when she
entered into the contract on behalf of the corporation, she did not know that the business had not
been properly incorporated.

POINT (2)(b) [40%] ISSUE: Under general partnership law, could the woman be personally liable
on the contract with the installer? ANSWER: No. The woman is unlikely to be held personally
liable as a partner in an unincorporated business because a court is likely to infer corporate limited
liability under the “de facto corporation” or the “corporation by estoppel” doctrine.

POINT (3) [25%] ISSUE: Is the man personally liable to the installer on the employment contract?
ANSWER: No. The man, as an inactive investor in the business, is not personally liable on the
employment contract with the installer either under MBCA § 2.04 or as a partner in the
unincorporated business.

ANSWER DISCUSSION:

Normally, a corporation comes into existence with the filing of articles of incorporation. Here, the
corporation came into existence on December 10 when the corrected, resubmitted articles were filed by
the Secretary of State’s office. The liability of participants in a business that has not been properly
incorporated can arise under statute or as a matter of partnership law. Under the MBCA, persons who
purport to act on behalf of a corporation prior to incorporation, knowing that there is no incorporation,
become personally liable. Here, the woman would not be liable under the MBCA because she did not
have actual knowledge, at the time of the dealings with the installer, that the corporation had not been
incorporated. Under partnership law, persons who carry on as co-owners a business for profit become
liable as partners. Although the woman would appear to be a partner in an unincorporated business, a
court applying common law doctrines in cases of defective incorporation is likely to infer corporate
limited liability with respect to the contract claim by the installer against the woman. However, under the
“de facto corporation” doctrine, a court is likely to find that the woman is not personally liable to the
installer because the woman made a good-faith attempt to incorporate and then entered into the contract in
the corporate name. A court could also apply the “corporation by estoppel” doctrine to find that the
woman is not personally liable to the installer because the installer dealt solely with the corporation,
without relying on the personal assets of the woman. As for the installer’s claim against the man, the man
(as an inactive investor in the business) is even less likely to be personally liable. The man did not purport
to act on behalf of the corporation and thus cannot be liable under the MBCA. In addition, even though
the man may have been a partner in the unincorporated business, a court would likely apply the two
defective- incorporation doctrines to infer limited liability for the man.

ANSWER EXPLANATION:

Explanation to Point One (10%):

Seperac-J19 Exam-Released MEE Essay Compilation © 2016-2020 725


The corporation was formed on December 10, when its articles of incorporation were filed by the
Secretary of State’s office.

Normally, the existence of a corporation begins with the filing of the articles of incorporation. Under the
MBCA, unless a delayed effective date is specified, the corporate existence begins when the articles of
incorporation are filed. The articles were not filed on November 15 when they were received by the
Secretary of State because they failed to include the number of authorized shares as required by the
statute. Here, the corporation came into existence on December 10 when the corrected, resubmitted
articles were filed by the Secretary of State’s office.

[NOTE: An examinee might point out that sometimes filings can be corrected, with the corrections
effective as of the date of the original filing. Under the MBCA, a corporation is permitted to file articles
of correction, which become effective as of the date of the document they correct, provided that the
corrections are limited to inaccuracies, defective signatures, and defects in electronic transmission. Here,
however, the Secretary of State’s office did not accept the original articles for filing, and there was no
“filed document” to correct.]

Explanation to Point Two(a) (25%):

The woman would not be personally liable under MBCA § 2.04 because when she entered into the
contract on behalf of the corporation, she did not know that the business had not been properly
incorporated.

Under the MBCA, persons purporting to act as or on behalf of a corporation, knowing there was no
incorporation, are jointly and severally liable for all liabilities created while so acting. The Official
Comment clarifies that an “erroneous but in good faith” belief that incorporation has happened does not
constitute knowledge under the section.

Here, the woman is likely not liable for purporting to act on behalf” of a non-existent corporation under
the MBCA. There is no indication that she actually knew that the original articles had not been filed by
the Secretary of State when she entered into the employment contract with the installer in the corporate
name. Although the woman could have checked with the Secretary of State’s office to ascertain whether
the business had been incorporated or possibly should have known that the articles were incomplete, her
lack of actual knowledge of the incorporation defect precludes her liability under the MBCA.

Explanation to Point Two(b) (40%):

The woman is unlikely to be held personally liable as a partner in an unincorporated business because a
court is likely to infer corporate limited liability under the “de facto corporation” or the “corporation by
estoppel” doctrine.

Generally, co-owners of a for-profit business that has not been properly incorporated are treated as
partners in a partnership and are jointly and severally responsible for all business obligations. Here,
however, although the woman was a co-owner in a business for profit and thus a partner, a court would
likely limit her liability because of the “de facto corporation” and “corporation by estoppel” doctrines.

Under the common law, courts have inferred corporate limited liability in cases of defective incorporation
in two situations. First, under the “de facto corporation” doctrine, courts recognize corporate limited
liability when there was (1) a colorable, good-faith attempt to incorporate and (2) actual use of the
corporate form, such as by carrying on the business as a corporation or contracting in the corporate name.

Seperac-J19 Exam-Released MEE Essay Compilation © 2016-2020 726


Second, under the “incorporation by estoppel” doctrine, most jurisdictions recognize corporate limited
liability when a third party deals solely with the “corporation” and has not relied on the personal assets of
the promoter. In these cases, courts have rejected the imposition of personal liability on equity grounds
because the third party would otherwise receive more than originally bargained for.

Here, a court is likely to apply one or both of the common law doctrines to infer corporate limited liability
in the installer’s contract claim against the woman. Under the “de facto corporation” doctrine, the
woman’s mailing of the articles of incorporation to the Secretary of State constituted a colorable, good-
faith attempt to incorporate. If the submitted articles had included the number of authorized shares, this
would have led to a validly formed corporation. In addition, the woman used the corporate form by
entering into the employment contract in the corporation’s name. She did this in good faith, apparently
believing that the incorporation steps had been completed. There is no indication that the woman was
aware that the articles of incorporation had been rejected.

In addition, under the “corporation by estoppel” doctrine, the installer’s entry into the employment
contract, which the woman signed as president of Solar Inc., is evidence that the installer understood that
he was dealing solely with the “corporation” and was not relying on the personal assets of the woman. To
allow the installer to hold the woman personally liable would create a liability that had not been bargained
for. That is, the installer assumed the risk that the corporation (as happened) would not be able to fulfill
the contract.

[NOTE: Some examinees might seek to analyze the woman’s liability under the “promoter liability” rules
that apply to contracts with a corporation that has not yet come into existence. Unless otherwise agreed,
a person who, in dealing with another, purports to act as agent for a principal whom both know to be
nonexistent or wholly incompetent, becomes a party to such contract. These rules, however, arise when
both the promoter and the third party are aware that no incorporation has happened, though it may
happen later. Thus, the “promoter liability” rules are inapplicable to the situation presented in this
question given that there is nothing to suggest that the woman and the installer understood that the
corporation had not yet been formed.]

Explanation to Point Three (25%):

The man, as an inactive investor in the business, is not personally liable on the employment contract with
the installer either under MBCA § 2.04 or as a partner in the unincorporated business.

Applying the defective-incorporation principles to the contract claim against the man, the man would not
be liable under the MBCA as he did not sign the contract with the installer and thus did not purport to act
on behalf of the non-existent corporation. In addition, even though the man as a co-owner in a business
for profit may have been a partner in the unincorporated business, a court would likely infer corporate
limited liability for the man under the “de facto corporation” and “corporation by estoppel” doctrines for
the same reasons that limited liability is available to the woman. Generally, the “de facto corporation” and
“corporation by estoppel” doctrines are available to all participants of the business, including those who
are inactive. Thus, both active and inactive participants of a “de facto corporation” have the same rights
and are entitled to the same protections. Similarly, the MBCA, in reviewing the approach by courts in
defective-incorporation cases, points out that inactive participants are generally not liable on pre-
incorporation contracts. Therefore, the man here would not be personally liable on the installer’s contract
claim.

Seperac-J19 Exam-Released MEE Essay Compilation © 2016-2020 727


#164-FEB 2017–MEE Q04: QUESTION FOUR (CORPS-LLCS)

A shareholder owns 100 shares of MEGA Inc., a publicly traded corporation. MEGA is incorporated in
State A, which has adopted the Model Business Corporation Act (MBCA).

The shareholder read a news story in a leading financial newspaper reporting that MEGA had entered into
agreements to open new factories in Country X. According to the story, MEGA had paid large bribes to
Country X government officials to seal the deals. If made, these bribes would be illegal under U.S. law,
exposing MEGA to significant civil and criminal penalties.

On May 1, the shareholder sent a letter to MEGA asking to inspect the minutes of meetings of MEGA's
board of directors relating to the Country X factories mentioned in the news story, along with any
accounting records not publicly available relevant to the alleged foreign bribes. The shareholder explained
that she was seeking the information to decide whether to sue MEGA's directors for permitting such
possible illegal conduct.

In her letter, the shareholder also demanded that the MEGA board investigate the possible illegal bribes
described in the news story and take corrective measures if any illegality had occurred.

On June 1, MEGA responded to the shareholder in a letter, which stated in relevant part:

The corporation will not give you access to any corporate documents or take any action regarding
the matters raised in your letter. We cannot satisfy the whim of every MEGA shareholder based on
unsubstantiated news stories. Furthermore, given our continuing operations in Country X, the board
of directors will not investigate or take any other action regarding the matters raised in your letter
because doing so would not be in the best interest of the corporation.

On October 1, the shareholder filed a lawsuit in a State A court. Her petition includes (1) a claim against
MEGA seeking inspection of the documents previously requested and (2) a derivative claim against all of
the MEGA directors alleging a breach of their fiduciary duties for failing to investigate and take action
concerning the alleged foreign bribes.

MEGA's board has asked the corporation's general counsel the following questions:

(1) Is the shareholder entitled to inspect the documents she requested?

(2) May the board obtain dismissal of the shareholder's derivative claim if the board concludes that
it is not in the corporation's best interest to continue the lawsuit, even though the board has not
investigated the allegations of illegal foreign bribes?

(3) Is the board's decision not to investigate or take further action with respect to alleged illegal
foreign bribes consistent with the directors' duty to act in good faith, and is that decision protected
by the business judgment rule?

How should the general counsel answer these questions? Explain.

Seperac-J19 Exam-Released MEE Essay Compilation © 2016-2020 728


#164: F17-4 MEE: ANSWER: NCBE (CORPS-LLCS)

POINT (1) [30%] ISSUE: Does the shareholder have a “proper purpose” in seeking to inspect
MEGA’s board minutes and accounting records to determine whether to bring (and maintain) a
suit concerning the alleged foreign bribes? ANSWER: Yes. The shareholder has a right to inspect
board minutes and accounting records if she has a “proper purpose.” Gathering information to
decide whether to bring a lawsuit alleging a breach of fiduciary duties arising from illegal corporate
behavior is a proper purpose.

POINT (2) [30%] ISSUE: May the board obtain dismissal of the shareholder’s derivative claim if
the board concludes that it is not in the corporation’s best interest to continue the lawsuit, even
though the board has not investigated the allegations of illegal foreign bribes? ANSWER: No. The
board may not obtain dismissal of the shareholder’s derivative claim, even if the board concludes
that it is not in the corporation’s best interest to continue the lawsuit, because the board’s
conclusion was not based on a reasonable investigation into the shareholder’s allegations.

POINT (3) [40%] ISSUE: Is the decision of the MEGA directors not to investigate or take action on
the shareholder’s allegations of illegal corporate behavior a breach of the directors’ duty to act in
good faith, and is that decision protected by the business judgment rule? ANSWER: The MEGA
directors’ disregard of the legality of the corporation’s conduct is a failure to act in good faith and
is not protected by the business judgment rule, even if the directors believed the conduct to be in the
corporation’s best interests.

ANSWER DISCUSSION:

The shareholder had a “proper purpose” when she sought to inspect board minutes and accounting records
relevant to alleged foreign bribes to determine whether to bring a lawsuit concerning possible illegal
corporate conduct. Therefore, the shareholder is entitled to inspect the requested minutes and accounting
records. Because the MEGA board did not conduct a reasonable inquiry before concluding that it was not
in the corporation’s best interest to investigate and remediate alleged illegal activities, the court is not
required to dismiss the shareholder’s suit, and her claim can proceed. The MEGA directors have a
fiduciary duty to act in “good faith,” which encompasses an obligation not to condone illegal corporate
conduct and to investigate credible evidence of such illegality. The business judgment rule does not
protect decisions of directors to condone or disregard illegal corporate conduct, even if such conduct may
be profitable for the corporation.

ANSWER EXPLANATION:

Explanation to Point-One (30%):

The shareholder has a right to inspect board minutes and accounting records if she has a “proper purpose.”
Gathering information to decide whether to bring a lawsuit alleging a breach of fiduciary duties arising
from illegal corporate behavior is a proper purpose.

Under the MBCA, a shareholder, whether of record or who beneficially owns her shares, has a right to
inspect minutes of board meetings and “accounting records” for a proper purpose. A proper purpose is a
purpose reasonably related to a person’s interest as a shareholder, “such as a desire to determine whether
improper transactions have occurred.”

Seperac-J19 Exam-Released MEE Essay Compilation © 2016-2020 729


Here, the shareholder has an interest in determining whether the directors permitted the corporation to
engage in illegal conduct that could result in the corporation paying civil and criminal penalties. An
inspection request must be granted whenever the shareholder articulates a purpose to address “economic
risks” to the corporation. For example, in one case, the court permitted inspection of the shareholders’ list
by a shareholder seeking to propose that the corporation stop doing business in communist Angola, based
on alleged economic risks to the corporation.

A shareholder seeking inspection of corporate documents must offer credible evidence that there was
mismanagement or other improper conduct.

Here, the news story in a leading financial newspaper, coupled with the board’s failure to investigate or to
deny the truth of the allegations in the news story, appears to constitute a credible basis for concluding
that MEGA may have engaged in illegal conduct. Moreover, although MEGA’s response to the
shareholder indicated that the news story about the illegal foreign bribes was “unsubstantiated,” there is
no indication in the facts, nor did MEGA make the assertion, that the news story was untrue.

Under the MBCA, the shareholder’s right to inspect corporate documents relevant to the alleged bribery is
subject to certain limitations. The MBCA allows a shareholder to inspect “only relevant excerpts of board
minutes directly connected with the shareholder’s purpose.” The MBCA also allows inspection of
“accounting records,” although this category is not as broad as the “books and records” category found in
other corporate statutes. According to the Official Comment, accounting records are “records that permit
financial statements to be prepared which fairly present the financial position and transactions of the
corporation.” Many documents related to the payment of foreign bribes could be seen to be “accounting
records,” to the extent that they relate to how such transactions would be presented in the corporation’s
financial statements. Under the MBCA, the corporation could refuse to allow inspection of non- financial
documents related to the alleged foreign bribery.

In sum, the shareholder has a right to inspect MEGA’s board minutes and accounting records related to
the alleged bribery of Country X officials because her purpose is connected to bringing a lawsuit to
protect her economic interest in MEGA.

[NOTE: Under the MBCA, a “proper purpose” need not be shown for the inspection of minutes of
shareholders’ meetings. But here the shareholder asked for board minutes, for which a “proper purpose”
must be shown.

Under the MBCA, the shareholder’s inspection rights were not terminated when she filed her derivative
claim against the MEGA directors. The MBCA instead specifies that shareholders who have been denied
inspection may apply for a court order permitting inspection, which the court is to decide “on an
expedited basis.” Courts have encouraged shareholders to first seek inspection of corporate documents
before bringing suits alleging improper corporate transactions and breaches of fiduciary duty. But the
right to inspection has been recognized even while a lawsuit is proceeding.]>

Explanation to Point-Two (30%):

The board may not obtain dismissal of the shareholder’s derivative claim, even if the board concludes that
it is not in the corporation’s best interest to continue the lawsuit, because the board’s conclusion was not
based on a reasonable investigation into the shareholder’s allegations.

Under the MBCA, the board can seek dismissal of the shareholder’s derivative action if a majority of the
board’s “qualified directors” – those directors who do not have a material interest in the derivative action,

Seperac-J19 Exam-Released MEE Essay Compilation © 2016-2020 730


determine in good faith, after conducting a reasonable inquiry upon which its conclusions are based, that
continuance would be contrary to the corporation’s best interests. Although the Official Comments to the
MBCA suggests that a full-blown board investigation is not necessary, the board’s request for dismissal
must have “some support in the findings of the inquiry.” Failure to investigate credible allegations of
corporate illegality constitutes lack of “good faith.” Directors breach the duty of good faith if they “knew
or should have known” of violations of law and failed to act.

Here, given the apparent lack of any inquiry or corrective action by the board, as evidenced by the letter
of June 1 to the shareholder, the court should not grant the motion to dismiss the derivative action or
accept the board’s conclusion that proceeding with the lawsuit would be contrary to the corporation’s best
interests.

[NOTE: Directors breach their fiduciary duties by failing to act upon “red flags” of corporate illegality.
Directors can breach their duties by ignoring “obvious danger signs” of illegal corporate conduct. Here,
the MEGA board’s failure to investigate the shareholder’s allegations based on a report in a leading
financial newspaper, which the board did not deny, suggests bad faith on the part of the directors, thus
excusing demand. For example, in one case, the court excused demand when the shareholders alleged
that the directors had known of illegal activity and allowed it to continue.]

[NOTE: The question does not call for a discussion of the MBCA’s demand requirement (i.e. a 90-day
waiting period before bringing a derivative claim). Thus, discussion of the demand requirement is
irrelevant. For example, courts have excused demand when the current board was “interested” in the
challenged transaction, the board decision was not appropriately informed, or the challenged transaction
was so egregious that it could not be the product of sound business judgment.]

Explanation to Point-Three (40%):

The MEGA directors’ disregard of the legality of the corporation’s conduct is a failure to act in good faith
and is not protected by the business judgment rule, even if the directors believed the conduct to be in the
corporation’s best interests.

A director is liable to the corporation for the director’s decisions or failures to take action that were “not
in good faith.” The Official Comment to this section of the MBCA states: Conduct involving knowingly
illegal conduct that exposes the corporation to harm will constitute action not in good faith, and belief that
decisions made (in connection with such conduct) were in the best interests of the corporation will be
subject to challenge as well.

Courts applying the duty of good faith have made clear that corporate directors cannot consciously violate
– or permit the corporation to violate – legal norms, even when such violations may be profitable to the
corporation. The Official Comments to the MBCA state that “a lack of good faith is presented where a
transaction is known to constitute a violation of applicable positive law.”

In addition, the duty to act in good faith requires corporate directors to establish procedures to ensure the
corporation’s compliance with legal norms. Thus, courts have required corporate directors to establish
“corporate information and reporting systems” that provide “timely, accurate information concerning both
the corporation’s compliance with law and its business performance.” The board of directors must ensure
that the corporation has information and reporting systems in place to provide directors with appropriate
information to permit them to discharge their responsibilities.

Seperac-J19 Exam-Released MEE Essay Compilation © 2016-2020 731


The “good faith” standard requires that directors, among other things, not approve (or condone) wrongful
or illegal activity. An attitude of “we don’t care about risks” breaches duty of good faith. In particular,
directors violate their fiduciary duties by approving bribery of foreign government officials, even when
the practice is widespread.

Even when disregard of legal norms might result in a net financial benefit to the corporation, directors are
required to comply with the law. A fiduciary cannot manage an entity in an illegal fashion, even if the
fiduciary believes the illegal activity will result in profits for the entity. Here, the failure of MEGA’s
directors to investigate or take corrective action regarding alleged illegal foreign bribes exposes the
corporation to significant penalties and constitutes action not in good faith.

The business judgment rule does not protect decisions by directors not acting in good faith.

As the Official Comment to the MBCA summarizing the business judgment rule states: In basic principle,
a board of directors enjoys a presumption of sound business judgment that, in making a business decision,
directors act in good faith, on an informed basis, and in the honest belief that the action taken is in the best
interests of the corporation.

Specifically, the business judgment rule, while normally protecting the honest business judgment of
directors, does not apply upon a showing of “illegality.” Put simply, the business judgment rule applies
unless there is a showing of fraud, illegality, or conflict of interest.

Directors breach their fiduciary duties – and the business judgment rule provides no protection – when
they approve illegal business operations (or refuse to investigate alleged illegal business activities), even
though the illegal business may be profitable to the corporation.

Thus, the MEGA directors would not be protected by the business judgment rule.

Seperac-J19 Exam-Released MEE Essay Compilation © 2016-2020 732


#165-JUL 2016–MEE Q01: QUESTION ONE (CORPS-LLCS)

Two siblings, a brother and a sister, decided to start a bike shop with their cousin. They filed a certificate
of organization to form a limited liability company. The brother and the sister paid for their LLC member
interests by each contributing $100,000 in cash to the LLC. Their cousin paid for his LLC member
interest by conveying to the LLC five acres of farmland valued at $100,000; the LLC then recorded the
deed.

Neither the certificate of organization nor the members’ operating agreement specifies whether the LLC is
member-managed or manager-managed. However, the operating agreement provides that the LLC’s
farmland may not be sold without the approval of all three members.

Following formation of the LLC, the company rented a storefront commercial space for the bike shop and
opened for business.

Three months ago, purporting to act on behalf of the LLC, the brother entered into a written and signed
contract to purchase 100 bike tires for $6,000 from a tire manufacturer. When the tires were delivered, the
sister said that they were too expensive and told her brother to return the tires. The brother was surprised
by his sister’s objection because twice before he had purchased tires for the LLC at the same price from
this manufacturer, and neither his sister nor their cousin had objected. The brother refused to return the
tires, pointing out that the tires “are perfect for the bikes we sell.” The sister responded, “Well, pay the
bill with your own money; you bought them without my permission.” The brother responded, “No way. I
bought these for the store, I didn’t need your permission, and the company will pay for them.” To date,
however, the $6,000 has not been paid.

One month ago, purporting to act on behalf of the LLC, the cousin sold the LLC’s farmland to a third-
party buyer. The buyer paid $120,000, which was well above the land’s fair market value. Only after the
cousin deposited the sale proceeds into the LLC bank account did the brother and sister learn of the sale.
Both of them objected.

One week ago, the brother wrote in an email to his sister, “I want out of our business. I don’t want to have
anything to do with the bike shop anymore. Please send me a check for my share.”

1. What type of LLC was created—member-managed or manager-managed? Explain.

2. Is the LLC bound under the tire contract? Explain.

3. Is the LLC bound by the sale of the farmland? Explain.

4. What is the legal effect of the brother’s email? Explain.

Seperac-J19 Exam-Released MEE Essay Compilation © 2016-2020 733


#165: J16-1 MEE: ANSWER: NCBE (CORPS-LLCS)

POINT (1) [20%] ISSUE: What type of LLC is created when neither its certificate of organization
nor its operating agreement specifies whether it is member-managed or manager-managed?
ANSWER: The LLC is member-managed, given that neither its certificate of organization nor its
operating agreement specifies otherwise.

POINT (2) [20%] ISSUE: In a member-managed LLC, does a member who purchases goods for the
business have authority (actual or apparent) to bind the LLC when the member acted without the
express approval of the other members? ANSWER: Yes. The LLC is bound by the tire-purchase
contract, given that the brother had both actual and apparent authority to act on behalf of the
LLC.

POINT (3) [30%] ISSUE: In a member-managed LLC, does a member who sells land on behalf of
an LLC that operates a bike shop bind the LLC when the member acted without the approval of
the other members as required in their operating agreement? ANSWER: No. The LLC is likely not
bound by the farmland sales contract, given that the cousin lacked actual authority under the
operating agreement and lacked apparent authority because the sale was outside the ordinary
course of the LLC’s activities.

POINT (4) [30%] ISSUE: In a member-managed LLC, does a member who withdraws unilaterally
dissolve the LLC, thus forcing a winding up of the LLC’s business and distribution of its net assets?
ANSWER: No. The brother’s withdrawal constitutes a “dissociation” from the LLC, but does not
cause a “dissolution” and winding up of the business. Upon dissociation, he becomes entitled to
distributions from the LLC only if and when made by the continuing members.

ANSWER DISCUSSION:

This LLC is member-managed. The default rule is that an LLC is member-managed unless the certificate
of organization or the operating agreement specifies otherwise. Thus, the brother (as a member of the
LLC) had the actual and apparent authority to bind the LLC to contracts involving the carrying out of the
LLC’s ordinary business, such as the tire-purchase contract. On the other hand, the LLC is not bound by
the farmland sale. The cousin lacked actual authority, given the specific limitation in the LLC’s operating
agreement concerning sale of the company’s farmland. In addition, the cousin lacked apparent authority to
sell the farmland; the sale was outside the scope of the company’s ordinary business and there was no
other manifestation by the company that the cousin had authority to sell the farmland. The withdrawal of
the brother resulted in his dissociation from the LLC, but he has no right to payment for his member
interest or to have the LLC dissolved and wound up. Instead, the brother is entitled to distributions only if
and when made by the continuing members.

ANSWER EXPLANATION:

Explanation to Point-One (20%):

The LLC is member-managed, given that neither its certificate of organization nor its operating agreement
specifies otherwise.

Seperac-J19 Exam-Released MEE Essay Compilation © 2016-2020 734


When the certificate of organization fails to specify whether the LLC is member-managed or manager-
managed, the LLC is presumed to be member-managed, unless the members’ operating agreement
specifies how the LLC is to be managed. Here, given the absence of an express election to be manager-
managed in either the certificate or the operating agreement, the LLC is member-managed.

[NOTE: Some examinees may discuss whether an LLC was properly formed. This question is not raised in
the call and does not warrant additional credit. Generally, an LLC is formed when the certificate of
organization (a.k.a. articles of organization) is filed with the Secretary of State and the LLC has at least
one member. Here the certificate was filed and, after the three acquired their membership interests, the
LLC had at least one member.]

Explanation to Point-Two (20%):

The LLC is bound by the tire-purchase contract, given that the brother had both actual and apparent
authority to act on behalf of the LLC.

Under RULLCA, “each member in a member-managed LLC has equal rights in the management and
conduct of the company’s activities.” Thus, consistent with general agency law principles and with the
approach of other acts governing LLCs, each member of a member-managed LLC can bind the company
to contracts for apparently carrying on the ordinary business of the company unless the member lacks
authority to do so and the other party to the contract has notice that the member lacks such authority.
Thus, a member of a member-managed LLC has the authority – both actual and apparent – to bind the
LLC, much as a partner in a general partnership.

Here, the brother had actual authority to bind the LLC, given that he was a member of the LLC and was
carrying out the company’s ordinary business by purchasing tires for the bike shop, even though the other
members had not expressly approved these purchases. He also had apparent authority, given that the tire
manufacturer had previously sold tires to the bike shop and could rely on the appearance that the brother
was again properly acting for the LLC.

Explanation to Point-Three (30%):

The LLC is likely not bound by the farmland sales contract, given that the cousin lacked actual authority
under the operating agreement and lacked apparent authority because the sale was outside the ordinary
course of the LLC’s activities.

Whether there is actual authority for a non-ordinary transaction depends on the operating agreement of the
LLC, which governs “relations between the members and the limited liability company” and “the
activities of the company.” Here, the members’ operating agreement specified that sale of the company’s
land required consent of all members. Thus, the cousin did not have actual authority to transfer the land
without the consent of the other two members. That the cousin sold the land above fair market value is not
relevant to the question of authority.

The 2006 RULLCA does not provide for “statutory” apparent authority, but instead leaves questions of a
member’s authority to agency law principles. Under RULLCA § 301, a person’s status as a member does
not prevent or restrict other laws from imposing liability on a limited liability company because of the
person’s conduct. Under agency law principles, the cousin lacked actual authority to sell the farmland
because the LLC’s operating agreement required consent by all the members for the sale of the company’s
farmland.

Seperac-J19 Exam-Released MEE Essay Compilation © 2016-2020 735


In addition, the cousin lacked apparent authority to sell the land because the buyer could not reasonably
rely on the cousin’s sole signature on behalf of the LLC. There is no indication that the LLC made any
manifestations to the third-party buyer that the cousin was authorized to enter into the sale of the farmland
or that the sale of farmland by a bike shop was in the ordinary course of the LLC’s business. In a member-
managed LLC, matters “outside the ordinary course of the activities of the company” require the consent
of all members.

Nonetheless, some earlier LLC acts provide that, absent a contrary provision in the certificate of
organization, an LLC member has authority to sign and deliver a deed of the company’s interest in real
property and “the instrument is conclusive” in favor of a bona fide purchaser for value without notice.
Under these earlier statutes, acts of members not “in the ordinary course of the company’s business” bind
the company only if authorized by the other members. Here, given that the ordinary business of the LLC
was operating a bike shop, the buyer would not have been a bona fide purchaser because he should have
had doubts that one LLC member alone could bind the company in the sale of land. Moreover, there is no
indication that the buyer had reason to believe that the other LLC members had authorized the farmland
sale.

[NOTE: Examinees might address whether the LLC could set aside the sale or seek a remedy against the
cousin for his violation of fiduciary duties. This should not receive any credit, as the question asks only
whether the LLC is bound by the sale of the farmland.]

Explanation to Point-Four (30%):

The brother’s withdrawal constitutes a “dissociation” from the LLC, but does not cause a “dissolution”
and winding up of the business. Upon dissociation, he becomes entitled to distributions from the LLC
only if and when made by the continuing members.

Under RULLCA, the express will of a member to withdraw results in “dissociation.” Dissociation does
not result in dissolution of the LLC. Dissolution under RULLCA requires the consent of all the members.
Here, the brother’s email reflects an express will to withdraw, thus causing him to be dissociated from the
LLC. The brother’s dissociation results in (1) loss of his rights to participate in the LLC and (2) rights to
distributions (payments by the LLC) only if and when made by the continuing members. He has no right
to payment for his LLC interest, unless the operating agreement specifies that a withdrawing member has
a right to payment upon dissociation, that the remaining members are to agree to have the LLC buy his
interest, or that the other members are to consent to dissolution (and winding up of the business).

The result under the 2006 RULLCA (and other more modern LLC acts) is different from the result under
some older LLC acts, including the 1996 ULLCA, which generally treat the withdrawal of a member of
an at-will LLC (no term) in much the same way as the withdrawal of a partner in an at-will general
partnership.

[NOTE: Examinees might notice that the rights of a withdrawing LLC member are not like those of a
partner in an at-will partnership, but rather more like those of a minority shareholder in a closely held
corporation. Partner withdrawal in an at-will partnership, unless agreed otherwise, causes the
dissolution of the partnership and a right to cash payment for the pro rata share of the withdrawing
partner’s interest, after satisfying any creditor claims. The withdrawal of a minority shareholder in a
closely held corporation results neither in dissolution of the corporation nor in any right to pro rata
payment of the corporation’s net assets. Instead, the minority shareholder remains entitled to dividends
and other distributions only if and when the board of directors (majority shareholders) chooses to
authorize such payments.]

Seperac-J19 Exam-Released MEE Essay Compilation © 2016-2020 736


#166-JUL 2015–MEE Q04: QUESTION FOUR (CORPS-LLCS)

The board of directors of a commercial real estate development corporation consists of the corporation’s
chief executive officer (CEO) and three other directors, who are executives at various other firms.

The corporation owns a commercial office tower, the value of which is approximately 10 percent of the
corporation’s total holdings. The corporation uses one floor of the tower as its corporate headquarters, but
it wants to vacate that floor as soon as it locates suitable replacement space.

Two years ago, the board obtained an independent appraisal of the tower, which indicated a fair market
value of between $12 and $15 million. After considering that appraisal, the board authorized the
corporation’s CEO to seek a purchaser for the tower.

The CEO immediately showed the tower to several sophisticated real estate investors and received offers
ranging from $8 million to $13 million. The CEO decided that these offers were insufficient, and after he
reported back to the board, no further action to sell the tower was taken.

Two months ago, the CEO and the other three directors of the corporation formed a limited liability
company (LLC) in which each holds a 25 percent ownership interest.

One month ago, the corporation’s board unanimously authorized the corporation’s sale of the tower to
LLC for $12 million. The minutes of the board’s meeting at which the tower sale was authorized reflect
that the meeting lasted for 10 minutes and that the only document reviewed by the corporation’s directors
was the two-year-old appraisal of the tower.

The minutes of the board’s meeting further state that the transaction was to be carried out with “a friendly
company so that the corporation will have time to relocate to a new headquarters” and that the board
“authorized the transaction because the $12 million price is toward the high end of the range of offers
received in the past from sophisticated real estate investors and is within the range of fair market values
listed in the appraisal.”

After the board’s authorization of the tower sale, the corporation entered into a contract to sell the tower
to LLC. The board did not seek shareholder approval of the transaction.

A non-director shareholder of the corporation is upset with the board’s decision authorizing the sale of the
tower to LLC. The shareholder believes that the corporation could have obtained a higher price for the
tower.

1. Does the business judgment rule apply to the board’s decision to have the corporation sell the
tower to LLC? Explain.

2. Did the directors breach their fiduciary duties by authorizing the tower sale? Explain.

Seperac-J19 Exam-Released MEE Essay Compilation © 2016-2020 737


#166: J15-4 MEE: ANSWER: NCBE (CORPS-LLCS)

POINT (1) [20%] ISSUE: Was the corporation’s sale of the tower a “director’s conflicting interest
transaction”? ANSWER: Yes. The sale of the tower was a “director’s conflicting interest
transaction” (or director self-dealing) because the directors of the corporation had adverse
financial interests as owners of LLC.

POINT (2) [30%] ISSUE: Was the board’s authorization of the sale of the tower protected by the
business judgment rule? ANSWER: No. The board’s authorization of the sale of the tower is not
protected by the business judgment rule because the transaction was not authorized by a majority
of informed, disinterested directors.

POINT (3) [30%] ISSUE: Did the corporation’s directors breach their fiduciary duty of loyalty by
authorizing the sale? ANSWER: Yes. The directors likely breached their duty of loyalty because it
is unlikely they could show that the transaction was substantively and procedurally fair to the
corporation.

POINT (4) [20%] ISSUE: Did the corporation’s directors breach their fiduciary duty of care in the
way they authorized the sale? ANSWER: Yes. The directors likely breached their fiduciary duty of
care in authorizing the sale of the tower because they did not become adequately informed prior to
their decision.

ANSWER DISCUSSION:

The sale of the tower is subject to judicial scrutiny as a “director’s conflicting interest transaction”
because it was a transaction between the corporation and an entity in which the corporation’s directors
had a material financial interest. The business judgment rule does not protect the board’s authorization of
the sale because all the directors who authorized the transaction had a material financial interest in the
sale, by virtue of their interest in LLC. Nor was there any shareholder approval. The directors likely
breached their duty of loyalty. Because this was a self-dealing transaction, the directors bear the burden of
demonstrating that the sale was substantively and procedurally fair to the corporation. It is unlikely that
they can demonstrate substantive fairness because nothing in the facts presented would support a finding
that the price of the tower at the time of the sale was fair. It is also unlikely that the directors can
demonstrate procedural fairness given their failure to obtain an up-to-date independent appraisal or to
undertake another “market test” of the property’s value. The directors also likely breached their duty of
care by failing to diligently and adequately consider the transaction’s fairness to the corporation. The
directors exhibited gross negligence in not inquiring about the continuing validity of the dated appraisal
and by not considering other means to ensure the fairness of the transaction to the corporation. [NOTE:
The analysis of this question tracks the analysis specified by Subchapter F of the Model Business
Corporation Act (MBCA). Similar analysis would apply in a jurisdiction that has not adopted Subchapter
F, but either has an interested-director provision modeled on former MBCA § 8.31 or relies on modern
common law principles applicable to director self-dealing transactions.]

ANSWER EXPLANATION:

Explanation to Point-One (20%):

Seperac-J19 Exam-Released MEE Essay Compilation © 2016-2020 738


The sale of the tower was a “director’s conflicting interest transaction” (or director self-dealing) because
the directors of the corporation had adverse financial interests as owners of LLC.

The corporation’s directors were on both sides of the transaction, both authorizing it for the corporation
and standing to gain from it as owners of LLC. The MBCA defines a “director’s conflicting interest
transaction” as one effected by the corporation “respecting which the director had knowledge and a
material financial interest.” The sale of the tower meets this definition of director self-dealing and is
subject to special scrutiny.

The directors’ financial interest in the transaction is clearly “material.” Each director’s one-fourth
ownership of LLC is likely to have impaired his or her objectivity in considering the transaction. Under
the MBCA, the materiality of the financial interest is measured under an objective standard. In addition,
all the directors were aware that the transaction was with their LLC.

In a jurisdiction with an “interested director” statute, the sale of the tower would qualify as a “director’s
conflicting interest transaction” given that it was a transaction with the corporation and one in which the
directors had a direct or indirect interest. The conflict of interest is indirect because the directors of the
corporation have a “material financial interest” in the other party, here the new LLC.

[NOTE: In jurisdictions without an “interested director” statute, the common law would treat the
transaction as one involving classic director self-dealing. The duty of loyalty derives from the prohibition
against self-dealing that inheres in the fiduciary relationship.]

Explanation to Point-Two (30%):

The board’s authorization of the sale of the tower is not protected by the business judgment rule because
the transaction was not authorized by a majority of informed, disinterested directors.

A “director’s conflicting interest transaction” (that is, a director self-dealing transaction) is not absolutely
prohibited. Instead, modern corporate law permits such transactions – with the consequence that the
business judgment rule applies if, after full disclosure of all relevant facts, qualified directors authorized
the transaction. The MBCA provides for safe harbor for interested director transactions, resulting in the
application of the business judgment rule, provided the material facts are disclosed or known to the board,
and the majority of disinterested directors authorize the transaction in good faith. If, however, the self-
dealing transaction is not shown to have been properly authorized, the business judgment rule does not
apply and the transaction must be shown to have been fair to the corporation.

Here, the MBCA safe harbor for proper board authorization does not apply. There was no authorization of
the sale of the tower by qualified directors – that is, by directors who did not have an interest in the
transaction. The MBCA defines a "qualified director,” for purposes of director-authorization safe harbor,
as not including a director as to whom the transaction is a director’s conflicting interest transaction. As
elaborated in Point One, the only directors who authorized the transaction were those who had a conflict
of interest. Thus, the presumption of the business judgment rule that the directors were informed and
acted in good faith is not available because each of the directors had a material conflicting interest in the
sale of the tower. Here, the presumption that the directors acted on an informed basis, in good faith, and in
the best interests of the corporation is not available to the interested directors.

[NOTE: Approval or ratification of the transaction by qualified shareholders, after full disclosure of all
relevant facts, would in some jurisdictions remove the transaction from further judicial review and in
others subject it only to review for waste. But here there was no such approval or ratification.]

Seperac-J19 Exam-Released MEE Essay Compilation © 2016-2020 739


Explanation to Point-Three (30%):

The directors likely breached their duty of loyalty because it is unlikely they could show that the
transaction was substantively and procedurally fair to the corporation.

Even if the sale of the tower was not properly authorized, the directors can satisfy their fiduciary duty of
loyalty by showing that “the transaction, judged according to the circumstances at the relevant time, was
fair to the corporation.”

The directors have the burden to show that the transaction as a whole was fair in terms of “fair price” and
“fair dealing.” This means courts will inquire into (1) whether the transaction price was comparable to
what might have been obtained in an arm’s-length transaction, given the consideration received by the
corporation, and (2) whether the process followed by the directors in reaching their decision was
appropriate. A breach of the directors’ duty to deal fairly with the corporation can result in personal
liability.

Here, the directors may argue that the $12 million price paid by LLC for the tower made the transaction
substantively fair because the price was within the range of offers ($8 to $13 million) received by the
corporation when the tower was offered for sale two years before. The directors may point out that the
sales price was also within the range of values of the appraisal ($12 to $15 million). Additionally, the
directors may argue that even if the sale was not at the highest valuation received by the corporation for
the tower, the sale gave the corporation non-economic advantages by enabling it to “have time to relocate
to a new headquarters.” Finally, the directors may argue that the transaction was procedurally fair because
information about the value of the tower was already known to them and there was no reason to make
further inquiries.

It is unlikely, however, that the directors’ arguments would meet their burden to show the transaction’s
fairness. As to substantive fairness, the offers and appraisal relied on by the directors occurred two years
before the sale to LLC, and the $12 million sales price is less than the $13 million offer that the CEO had
rejected as insufficient. As to procedural fairness, the directors did not conduct a new market test or seek
other purchasers. The process of the board’s decision was flawed: the meeting was conducted in 10
minutes, there was no discussion of whether market circumstances had changed since the two-year-old
appraisal or whether to seek another appraisal, and there was no consideration of looking for other
purchasers or conducting another market test. Finally, the directors did not consider whether another
buyer other than LLC would be willing to accommodate the corporation’s need for time to locate and
move to new headquarters.

Explanation to Point-Four (20%):

The directors likely breached their fiduciary duty of care in authorizing the sale of the tower because they
did not become adequately informed prior to their decision.

Under the MBCA, a director is called on to exercise “the care that a person in a like position would
reasonably believe appropriate under similar circumstances” in “becoming informed in connection with
their decision-making function.” A director may be liable for harm to the corporation from a board
decision where it is proven that the director was “not informed to an extent the director reasonably
believed appropriate in the circumstances.” A director breaches his duty of care when he is grossly
negligent in informing himself about board decisions.

Seperac-J19 Exam-Released MEE Essay Compilation © 2016-2020 740


Normally, “the party attacking a board decision as uninformed must rebut the presumption that its
business judgment was an informed one.” But here the presumption of the business judgment rule does
not apply, given that the directors were each financially interested in the transaction.

A strong case can be made that the directors were grossly negligent in informing themselves about the
sale of the tower to LLC, and thus that they breached their fiduciary duty of care. As discussed in Point
Three, the directors decided to sell the tower at a board meeting that lasted 10 minutes; the only document
they reviewed was a two-year-old appraisal; they did not discuss whether market circumstances had
changed since the appraisal; they did not seek another appraisal; they did not consider looking for other
purchasers, including an accommodating purchaser, or conducting another market test; and they did not
discuss how the CEO had conducted the earlier market test. For example, directors have been found
grossly negligent for authorizing a cash-out merger in reliance on a 20-minute oral presentation by the
corporation’s chief executive officer without a written summary of the merger terms or documentation to
support the adequacy of the price.

[NOTE: For the shareholder to recover money damages based on a claim that the directors breached
their duty of care, the shareholder would have to prove that the corporation was harmed by a sales price
that was below market and that the sales price was proximately caused by the directors’ failure to become
adequately informed. These showings, however, are not elements of whether the directors breached their
duty of care.]

Seperac-J19 Exam-Released MEE Essay Compilation © 2016-2020 741


#167-JUL 2014–MEE Q06: QUESTION SIX (CORPS-LLCS)

Mega Inc. is a publicly traded corporation incorporated in a state whose corporate statute is modeled on
the Model Business Corporation Act (MBCA). Mega’s articles of incorporation do not address the
election of directors or amendment of the bylaws by shareholders.

Well within the deadline for the submission of shareholder proposals for the upcoming annual
shareholders’ meeting, an investor, who was a large and long-standing shareholder of Mega, submitted a
proposed amendment to Mega’s bylaws. The proposal, which the investor asked to be included in the
corporation’s proxy materials and voted on at the upcoming shareholders’ meeting, read as follows:

Section 20: The Corporation shall include in its proxy materials (including the proxy ballot) for a
shareholders’ meeting at which directors are to be elected the name of a person nominated for election to
the Board of Directors by a shareholder or group of shareholders that beneficially have owned 3% or
more of the Corporation’s outstanding common stock for at least one year.

This Section shall supersede any inconsistent provision in these Bylaws and may not be amended or
repealed by the Board of Directors without shareholder approval.

Mega’s management decided to exclude the investor’s proposal from the corporation’s proxy materials
and explained its reasons in a letter to the investor:

The investor’s proposed bylaw provision would be inconsistent with relevant state law because the Board
of Directors has the authority to manage the business and affairs of the Corporation. Generally,
shareholders lack the authority to interfere with corporate management by seeking to create a method for
the nomination and election of directors inconsistent with the method chosen by the Board of Directors.

Furthermore, at its most recent meeting, the Board of Directors unanimously approved an amendment to
the Corporation’s bylaws that provides for proxy access for director nominations by a shareholder or a
group of shareholders holding at least 10% of the Corporation’s voting shares for at least three years. This
procedure takes precedence over any nomination methods that might be sought or approved by
shareholders.

The investor is considering bringing a suit challenging management’s refusal to include the investor’s
proposed bylaw provision and challenging the board’s amendment of the bylaws at its recent meeting.

1. Is the investor’s proposed bylaw provision inconsistent with state law? Explain.

2. If the investor’s proposed bylaw provision were approved by the shareholders, would the bylaw
amendment previously approved by the board take precedence over the investor’s proposed bylaw
provision? Explain.

3. Must the investor make a demand on Mega’s board of directors before bringing suit? Explain.

Seperac-J19 Exam-Released MEE Essay Compilation © 2016-2020 742


#167: J14-6 MEE: ANSWER: NCBE (CORPS-LLCS)

POINT (1) [30%] ISSUE: Do shareholders have the authority to amend a corporation’s bylaws
with respect to director nominations? ANSWER: Yes. Shareholders may amend the corporation’s
bylaws where the proposed bylaw provision relates to procedural matters typically included in the
bylaws, such as the nomination of directors.

POINT (2) [30%] ISSUE: Do board-approved bylaws on a particular subject, here nomination of
directors, preempt subsequent conflicting bylaw amendments by shareholders? ANSWER: No.
Shareholders can amend (or repeal) board-approved bylaws. Further, shareholders can limit the
board’s power to later amend and repeal a shareholder-approved bylaw.

POINT (3) [40%] ISSUE: Is a suit challenging both management’s refusal to include the proposed
bylaw amendment in Mega’s proxy statement and the board’s amendment of the bylaws dealing
with nomination of directors a direct or derivative suit? ANSWER: The investor need not make a
demand on the board if the investor states a direct claim, such as an allegation that the board
interfered with the investor’s right to amend the bylaws. But the investor must make a demand on
the board if the investor states a derivative claim (on behalf of the corporation), such as an
allegation that the directors sought to entrench themselves by interfering with the proposed proxy
access.

ANSWER DISCUSSION:

The voting and litigation rights of the shareholders of Mega are subject to the provisions of the Model
Business Corporations Act (MBCA). The investor’s proposed bylaw provision is not inconsistent with
state law. Under the MBCA, shareholders may amend the bylaws when the amendment deals with a
proper matter for the corporation’s bylaws, such as procedures for nominating directors. The Mega
board’s bylaw amendment does not preempt the investor’s proposed bylaw provision or the Mega
shareholders’ power to approve it. While shareholders can limit the board’s power to amend or repeal the
bylaws, the board cannot limit the shareholders’ power. Whether the investor must make a demand on
Mega’s board depends on how the investor frames its claim. If the investor claims a violation of
shareholder voting rights, the claim is direct and pre-suit demand on the board is not required. If, on the
other hand, the investor claims that the directors violated their fiduciary duties by amending the bylaws to
entrench themselves, the claim is derivative and a pre-suit demand is required.

ANSWER EXPLANATION:

Explanation to Point-One (30%):

Shareholders may amend the corporation’s bylaws where the proposed bylaw provision relates to
procedural matters typically included in the bylaws, such as the nomination of directors.

Internal affairs of the corporation, such as the conduct of shareholder meetings and election of directors,
are subject to the corporate law of the state of incorporation. For example, in cases involving voting
rights, Delaware applies the law of the jurisdiction where the corporation was incorporated. This state’s
corporate statute is modeled on the MBCA.

Seperac-J19 Exam-Released MEE Essay Compilation © 2016-2020 743


Under the MBCA, “shareholders may amend the corporation’s bylaws.” Thus, the only question is
whether the bylaws can specify the procedures for shareholder nomination of directors.

The MBCA states that the bylaws “may contain any provision that is not inconsistent with law or the
articles of incorporation.” In addition, the MBCA was revised in 2009 to address shareholder nomination
of directors in public corporations (known as “proxy access”) and specifies that the bylaws “may contain
a requirement that the corporation include in its proxy materials one or more individuals nominated by a
shareholder.”

The inclusion of director-nomination procedures in the bylaws is consistent with practice and is
recognized by the Delaware courts, whose views on corporate law carry significant weight. Typically, the
procedures for nomination of directors are found in the bylaws. The Delaware Supreme Court has
confirmed that the bylaws may “define the process and procedures” for director elections. For example, a
bylaw amendment requiring reimbursement of election expenses to certain successful shareholder
nominators is a “proper subject” under Delaware law.

[NOTE: The question of the proper scope of the bylaws can be answered using the more general MBCA §
2.06(b) or the 2009 MBCA revision adding § 2.06(c)(1). In addition, some examinees might raise the
point that shareholder proposals may not compel the board to take action, such as by including
shareholder nominations in the company’s proxy materials, on the theory that the “business and affairs”
of the corporation are to be managed by the board. Although shareholders are generally limited to
adopting precatory resolutions that recommend or encourage board action, this limitation does not apply
when shareholders have specific authority to take binding action on their own – such as to amend the
bylaws.]

Explanation to Point-Two (30%):

Shareholders can amend (or repeal) board-approved bylaws. Further, shareholders can limit the board’s
power to later amend and repeal a shareholder-approved bylaw.

Under the MBCA, shareholders have the power to amend the bylaws. The board shares this power with
the shareholders, unless (1) the corporation’s articles “reserve that power exclusively to the shareholders”
or (2) “the shareholders in amending, repealing, or adopting a bylaw expressly provide that the board of
directors may not amend, repeal, or reinstate that bylaw.”

Shareholder-approved bylaw provisions can amend or repeal existing bylaw provisions, whether
originally approved by the board or by shareholders. Thus, the Mega board’s bylaw amendment – which
set more demanding thresholds for shareholder nomination of directors than the investor’s proposed
bylaw provision – would be superseded (repealed) if Mega’s shareholders were to approve the investor’s
proposal.

Further, a shareholder-approved bylaw generally can limit the power of the board to later amend or repeal
it. Thus, if Mega’s shareholders approved the bylaw provision proposed by the investor, Mega’s board
could not repeal the provision because it includes a “no board repeal” clause.

The revision to the MBCA in 2009 dealing with shareholder proxy access does not change this
conclusion. That revision specifies that a shareholder-approved bylaw dealing with director nominations
may not limit the board’s power to amend, add, or repeal “any procedure or condition to such a bylaw in
order to provide for a reasonable, practicable and orderly process.” Thus, according to the revision, if
shareholders approve a bylaw amendment that limits further board changes, the board would nonetheless

Seperac-J19 Exam-Released MEE Essay Compilation © 2016-2020 744


retain the power to “tinker” with the bylaw to safeguard the voting process, but could not repeal the
shareholder-approved bylaw. The revision is “not intended to allow the board of directors to frustrate the
purpose of the shareholder-adopted proxy access provision.” Thus, if Mega’s shareholders were to
approve the bylaw provision proposed by the investor, Mega’s board could only amend the provision
regarding its procedures or conditions in a manner consistent with its purpose of permitting proxy access
for Mega’s shareholders.

[NOTE: The board’s attempted interference with a shareholder voting initiative may also have been a
violation of the directors’ fiduciary duties. For example, directors breach their fiduciary duties if they
amend bylaws and expand the size of board to thwart an insurgent’s plan to amend the bylaws and seat a
majority of new directors. The call, however, asks examinees to consider whether shareholders or the
board have “precedence” over amending the corporate bylaws. Thus, an examinee’s answer should be
framed in terms of “power” and not “duty.”]

Explanation to Point-Three (40%):

The investor need not make a demand on the board if the investor states a direct claim, such as an
allegation that the board interfered with the investor’s right to amend the bylaws. But the investor must
make a demand on the board if the investor states a derivative claim (on behalf of the corporation), such
as an allegation that the directors sought to entrench themselves by interfering with the proposed proxy
access.

The MBCA generally requires that shareholders make a demand on the board of directors before initiation
of a derivative suit. Under the MBCA, a shareholder may not bring derivative proceeding until written
demand has been made on corporation and 90 days have expired. A derivative suit is essentially two suits
in one, where the plaintiff-shareholder seeks to bring on behalf of the corporation a claim that vindicates
corporate rights, usually based on violation of fiduciary duties. The demand permits the board to
investigate the situation identified by the shareholder and take suitable action. No demand on the board is
required, however, if the shareholder brings a direct suit to vindicate the shareholder’s own rights, not
those of the corporation.

Is the suit brought by the investor derivative or direct? The MBCA defines a “derivative proceeding” as
one brought “in the right of a domestic corporation.” Thus, the answer to how the investor’s suit should
be characterized turns on what rights the investor seeks to vindicate. If the investor frames its claim as one
of fiduciary breach by directors – for example, for failing to become adequately informed about voting
procedures or for seeking to entrench themselves in office by manipulating the voting structure to avoid a
shareholder insurgency – then the suit is “derivative” and the investor must make a demand on the board.
The derivative suit has historically been the principal method of challenging allegedly illegal action by
management.

If, however, the investor frames its claim as one to vindicate shareholder rights, the suit is direct and no
demand is required. For many courts, the direct-derivative question turns on who is injured and who is to
receive the relief sought by the plaintiff-shareholders. For example, a merger-delay claim is characterized
as direct because a delay of merger only harms shareholders, and not the corporation. Thus, if the investor
claims that management’s refusal to include its proposed bylaw amendment in the corporation’s proxy
materials violates its shareholder rights to initiate corporate governance reforms, the suit will be direct.
Courts have not questioned the ability of shareholders to bring direct suits challenging board action to
exclude their proposed bylaw amendments from the corporation’s proxy materials. For example, a
Delaware court upheld a shareholder’s direct challenge to the board’s interpretation of an advance-notice

Seperac-J19 Exam-Released MEE Essay Compilation © 2016-2020 745


bylaw. Another Delaware court upheld a shareholder’s direct challenge to actions by the board that
effectively prevented it from proposing bylaw amendments in a contest for control.

Is the way that the investor frames its claim conclusive? Courts have permitted shareholder-plaintiffs to
challenge a transaction in a direct suit, even though the same transaction could also be challenged as a
fiduciary breach. For example, courts have permitted direct suits challenging corporate reorganizations as
a dilution of shareholder voting power, even though the reorganization may have involved conflicts of
interest and thus constituted a fiduciary breach. Thus, the investor’s choice to pursue a claim challenging
the legality of management’s decision to exclude the investor’s proposal from the corporation’s proxy
materials – rather than a possible breach of fiduciary duty – is likely to be respected.

[NOTE: Some issues under Delaware corporate law regarding pre-suit demand are not relevant here.
For example, whether the Mega directors are independent and disinterested is not relevant to the MBCA
requirement of a pre-suit demand. The MBCA’s requirement of “universal demand” gives the board “the
opportunity to reexamine the act complained of in the light of a potential lawsuit and take corrective
action,” even when the directors might be non-independent or have conflicts of interest.

Nor is it relevant to the MBCA pre-suit demand requirement that the statutory 90-day waiting period may
be onerous. The first paragraph of MBCA § 7.42 requires a pre-suit demand without exception; the
second paragraph of the section imposes a 90-day waiting period before a derivative suit may be brought,
which can be shortened if the board rejects the demand or “irreparable injury to the corporation would
result by waiting for the expiration of the 90-day period.” The call, as written, asks only whether a pre-
suit demand should be made and does not ask examinees to address whether the post-demand waiting
period should be shortened under the “irreparable injury” standard.]>

Seperac-J19 Exam-Released MEE Essay Compilation © 2016-2020 746


#168-JUL 2013–MEE Q06: QUESTION SIX (CORPS-LLCS)

On February 2, Alice, Bob, and Carla formed ABC Hospitality, LLC (ABC), a member-managed limited
liability company, for the purpose of building, owning, and running a 100-room luxury hotel in their
hometown. ABC soon began to experience unexpected financial problems, prompting Bob to look for
other investment opportunities.

On March 10, Bob told Alice and Carla that, although he would remain as a member of ABC, he would
no longer contribute any capital to ABC, and he was also becoming a co-owner of the Metro Inn, an
existing 200-room hotel in the same town near the ABC hotel project. Alice and Carla objected to Bob’s
plan, fearing that he might put the interests of the Metro Inn ahead of his existing obligations to ABC. In
response, Bob cited § 5.1 of ABC’s Operating Agreement, which states as follows:

Members of ABC shall not in any way be prohibited from or restricted in managing, owning, or otherwise
having an interest in any other business venture that may be competitive with the business of ABC.

Shortly after Bob became a co-owner of the Metro Inn, ABC’s financial situation worsened. Alice and
Carla worried that ABC would not be able to pay a bill it owed to its concrete supplier. Alice proposed to
pay the concrete supplier’s bill from her own personal funds and then obtain reimbursement from ABC
once the hotel project was completed. Alice wanted to do this so that she could file a personal financial
statement which underreported her assets and so enable her son to qualify for student financial aid. Carla
agreed to this proposal. Alice and Carla also agreed to alter ABC’s financial records so that it would
appear as if ABC had paid the concrete supplier’s bill out of its own accounts, without showing the
obligation to reimburse Alice for that amount.

In the weeks following Alice’s payment to the concrete supplier, several other of ABC’s bills became due.
Alice tried to pay as many of these bills as she could using her personal funds, but despite her best efforts,
it soon became clear that ABC was rapidly approaching insolvency. On August 15, the hotel’s designer
left a message for Carla seeking payment of an overdue bill.

Alice and Carla were concerned about the solvency of the company. Without responding to the designer,
Alice and Carla, acting with Bob’s consent, sold all of ABC’s property and remaining assets. Alice and
Carla each kept one-third of the sale proceeds and gave the remaining one-third to Bob. They did not file
articles of dissolution with the state. When the designer later called Carla again about the bill, she
responded that ABC had been “dissolved” and that no payment would be forthcoming.

1. Did Alice and Carla have any legal basis to object to Bob’s co-ownership of the Metro Inn?
Explain.

2. Under what theory or theories could Alice, Bob, or Carla be personally liable to the designer?
Explain.

Seperac-J19 Exam-Released MEE Essay Compilation © 2016-2020 747


#168: J13-6 MEE: ANSWER: NCBE (CORPS-LLCS)

POINT (1) [30%] ISSUE: Did Alice and Carla have any legal basis to object to Bob’s co-ownership
of the Metro Inn? ANSWER: No. Alice and Carla most likely do not have any legal basis to object
to Bob’s decision to own an interest in a competing business in light of the express terms of the ABC
operating agreement.

POINT (2)(a) [35%] ISSUE: Is the designer entitled to hold Alice, Bob, and/or Carla personally
liable to the designer because the winding up was improper? ANSWER: Yes. The designer may be
able to hold Alice, Bob, and Carla proportionately liable, up to the amount of the proceeds each
received in dissolution, for ABC’s debt to the designer because the winding up of ABC following
dissolution was improper.

POINT (2)(b) [35%] ISSUE: Is the designer entitled to hold Alice, Bob, and/or Carla personally
liable to the designer because of the piercing-the-veil theory? ANSWER: Yes. The designer may be
able to hold Alice, Bob, and Carla jointly and severally liable for the debt it is owed from ABC on a
“piercing the LLC veil” theory.

ANSWER DISCUSSION:

ABC is a limited liability company (LLC) governed by state LLC law and the terms of its operating
agreement. Alice and Carla most likely do not have any legal basis to object to Bob’s ownership interest
in a competing business because of the express terms of ABC’s operating agreement. Alice, Bob, and
Carla likely are personally liable to the designer for ABC’s debt up to the amount each member received
from the sale of ABC’s assets because the attempted dissolution and winding up of ABC was improper.
Alternatively, they may be personally liable to the designer on a piercing-the-veil theory because their
siphoning of the proceeds from the sale of ABC’s assets was inequitable, though not necessarily because
the company’s books failed to reflect Alice’s payment of company debts.

ANSWER EXPLANATION:

Explanation to Point-One (30%):

Alice and Carla most likely do not have any legal basis to object to Bob’s decision to own an interest in a
competing business in light of the express terms of the ABC operating agreement.

ABC is a limited liability company (LLC). An LLC is a form of business association that combines many
of the most attractive features of corporations and partnerships. Like corporations, an LLC generally
provides its investors – called “members” – with limited liability for firm debts. Like general partnerships,
LLCs provide members with considerable flexibility in developing rules for decision making and control.
Because LLCs are a relatively new form of business association bearing many similarities to corporations
and partnerships, courts often turn to either corporate or partnership law to analyze issues in the LLC
context.

Here, the issue of whether Alice and Carla have any legal basis to object to Bob’s ownership of an interest
in a competing firm will turn on the duties and obligations that Bob owes to his fellow LLC members.
Courts generally hold that members of an LLC, like partners in a general partnership, are in a “fiduciary
relationship.” This fiduciary relationship is defined as a relationship in which members owe one another

Seperac-J19 Exam-Released MEE Essay Compilation © 2016-2020 748


the duty of utmost trust and loyalty. As a result, in an LLC, as in a general partnership, direct competition
by members would ordinarily be precluded as a violation of the duty of loyalty. Under the ULLCA,
members have a duty to refrain from competing with the company in the conduct of the company’s
activities before the dissolution of the company.

Although the question does not state that ABC’s 100-room luxury hotel project and the 200-room Metro
Inn would compete, their similarity and proximity in the same town suggest that they would likely
compete. Further, the fact that Alice and Carla are bothered by Bob’s interest in Metro Inn suggests that
competition is a strong possibility.

However, here the LLC operating agreement expressly allows members to have an interest in a business
that competes with the firm. Under most LLC statutes, members of an LLC can agree to restrict or limit
the duty of loyalty, provided the opt-out is specified in the operating agreement. Thus, the operating
agreement controls over the provisions of the statute. In this case, when they became members of ABC,
Alice and Carla agreed to abide by the terms of the operating agreement, which expressly allow any
member (i.e., Bob) to manage, own, and otherwise have an interest in a business that competes with the
firm.

States differ slightly on how the opt-out must be structured in order to be effective. Some states, including
Delaware, provide for total freedom of contract. In these states, the terms of the operating agreement
govern, unless they conflict with a mandatory statutory provision designed to protect third parties. Since
Bob’s interest in Metro Inn involves only the relationship between the contracting members – Alice, Bob,
and Carla – this exception is not implicated.

However, most states that permit opt-outs of the duty of loyalty follow the general approach set forth in
the Uniform Limited Liability Company Act. According to the ULLCA, “if not manifestly unreasonable,
the operating agreement may restrict or eliminate the duty to refrain from competing with the company in
the conduct of the company’s business before the dissolution of the company.” In addition, so long as it is
not “manifestly unreasonable,” the operating agreement may also “identify specific types or categories of
activities that do not violate the duty of loyalty.”

In this case, the operating agreement is not a general waiver of the duty of loyalty, but specifies the
activities that do not violate the duty, namely “managing, owning, or otherwise having an interest” in any
competing business. Further, most courts would likely find this provision not “manifestly unreasonable.”

Whether the provision is “manifestly unreasonable” is a question of fact. Nonetheless, assuming that
Alice and Carla each had notice of the provision when they signed the operating agreement, it will be
difficult for them to claim surprise. The provision clearly allowed each member to manage, own, or
otherwise have an interest in a competing business. This type of provision, which is typical in commercial
real estate deals, appears reasonable. Many investors will likely want to be free to invest and exercise
ownership rights in other similar real estate ventures in the same area or town.

Explanation to Point-Two(a) (35%):

The designer may be able to hold Alice, Bob, and Carla proportionately liable, up to the amount of the
proceeds each received in dissolution, for ABC’s debt to the designer because the winding up of ABC
following dissolution was improper.

Like a corporation, the LLC provides limited liability for its members. This means that members of an
LLC may not generally be held personally liable for the debts of the firm. The general rule, however, does

Seperac-J19 Exam-Released MEE Essay Compilation © 2016-2020 749


not apply in a few situations, including when (1) the proper procedures for dissolution and winding up
have not been followed, and (2) a court decides to “pierce the LLC veil.”

Here, the designer may be able to hold Alice, Bob, and Carla liable for ABC’s debts on the ground that
the winding up of ABC was improper. In the absence of a contrary agreement or court order, dissolution
of an LLC requires consent of all the members. In this case, the requirement of unanimous consent has
been met since Bob gave his consent to Carla and Alice selling all of ABC’s assets and property.
Although ABC was properly dissolved by the unanimous consent of the three members, the problem was
that the winding up process was flawed.

After dissolution, as part of the winding up process, the LLC must provide notice of the dissolution to
creditors so that they can make claims against the dissolving entity. The notice sent to creditors must
outline the steps that are necessary for enforcing their claims. Here, it is clear from Carla’s statement that
the designer’s claim was known at the time of dissolution and that ABC did not provide sufficient notice
to the designer of the dissolution.

When such procedures have not been followed, and if the LLC’s assets have been liquidated and
distributed to the members, then a creditor’s claim against the LLC may be enforced against each of the
LLC members to the extent of the member’s proportionate share of the claim or to the extent of the assets
of the LLC distributed to the member in liquidation, whichever is less. However, a member’s total
liability for creditor claims may not exceed the total value of assets distributed to the member in
dissolution.

While the facts state that no articles of dissolution were filed with the state, the filing of such notice is not
required for a proper dissolution of an LLC. As a result, the designer can recover proportionately from
Alice, Bob, and Carla personally for ABC’s debt up to the amount that each member received in the
improper winding up after dissolution.

Explanation to Point-Two(b) (35%):

The designer may be able to hold Alice, Bob, and Carla jointly and severally liable for the debt it is owed
from ABC on a “piercing the LLC veil” theory.

As an alternative basis for holding Alice, Bob, and Carla personally liable for ABC’s debts, the designer
could pursue a claim for piercing the LLC veil. Piercing the veil in the LLC context is a common-law
equitable doctrine that prevents members from hiding behind the veil of limited liability in situations
where they have improperly used the LLC form. To pierce the LLC veil, courts generally apply the same
analysis and factors as in cases where a third party attempts to pierce the corporate veil and hold
shareholders of a corporation personally liable for firm debts. This requires analyzing whether members
have treated the LLC as a separate entity or whether it has instead become the “alter ego” of the members.
If the latter is true, members will not be permitted to “enjoy immunity from individual liability for the
LLC’s acts that cause damage to third parties.” Each member for whom the veil is pierced becomes
subject to joint and several liability to the creditor bringing the claim.

Among the factors that courts have used under the alter ego doctrine in the corporate context is whether
the “dominant shareholder siphoned corporate funds,” a factor also used in the LLC context. In arguing
for piercing the LLC veil, the designer will likely assert that the three members of ABC improperly
distributed to themselves the proceeds of the asset sale, thus disregarding the separateness of firm and
personal finances. The use of business funds for personal use is a frequent factor for piercing in both the
corporate and LLC contexts.

Seperac-J19 Exam-Released MEE Essay Compilation © 2016-2020 750


Alice, Bob, and Carla, all of whom received the proceeds of the asset sale when ABC was liquidated, will
have a difficult time arguing that this distribution was not inequitable. Although they did not misrepresent
ABC’s weak financial situation to the designer, their siphoning of assets would seem to be conclusive.
Under the ULLCA, the “key piercing factor” is the disregard of LLC’s “economic separateness” from its
owners, such as when owner “writes checks on the company’s account for personal expenses”. In fact, in
the corporate context when courts determine that business assets have been intermingled and used for
personal use, piercing follows in about 85 percent of the cases.

In addition, courts have identified the intermingling of personal and business funds as a factor for piercing
in the corporate context, as well as in the LLC context. The designer might point to Alice’s use of
personal funds to pay the concrete supplier and other creditors of ABC, expecting that ABC would later
reimburse her. The designer might further assert that Carla and Alice engaged in “fraud” by altering
ABC’s financial statements to cover up Alice’s personal payments, as further justification for piercing the
LLC veil on equitable grounds.

Nonetheless, Alice, Bob, and Carla may be able to argue that the failure to follow corporate formalities in
the accounting of Alice’s “advances of funds” to the LLC (that is, her undisclosed payment of firm debts,
subject to later reimbursement) is not enough to pierce the veil. The LLC statutes in most states, including
those that follow the Uniform Limited Liability Company Act, provide that “the failure of a limited
liability company to observe any particular formalities relating to management of its activities” is not a
proper ground for imposing personal liability on members for debts of the firm. Moreover, because Alice
and Carla were trying to keep the business afloat during difficult financial times, Alice’s undisclosed
advances of funds did not harm the firm’s creditors, but actually worked to their advantage. In this sense,
Alice’s payments could be equated to additional capital contributions to the firm, not the improper
siphoning of assets. Thus, taken alone, the off-the-books advances by Alice to ABC might not be
sufficient to justify piercing, but this does not “cleanse” the inequitable nature of the distribution of
proceeds to the three members when ABC was dissolved while the firm’s debts remained unpaid.

[NOTE: The question specifically asks whether Alice, Bob, and Carla are “personally” liable to the
designer and not whether the three might be liable to the LLC as members who consented to an improper
distribution by the LLC – that is, a distribution that rendered the company unable to pay its debts as they
became due. The ULLCA imposes liability on members in a member-managed LLC who consent to an
improper distribution.]

[NOTE: Six states have enacted the 2006 act.]

Seperac-J19 Exam-Released MEE Essay Compilation © 2016-2020 751


#169-JUL 2012–MEE Q08: QUESTION EIGHT (CORPS-LLCS)

Acme Inc. manufactures building materials, including concrete, for sale to construction companies. To
create a market for its building materials, Acme enters into agreements with construction companies
under which Acme and the construction company agree to form a member-managed limited liability
company (LLC). The LLC builds the project, purchasing building materials from Acme and contracting
for construction services with the construction company.

The operating agreements for these LLCs always provide that Acme has a 55% voting interest, that Acme
and the construction company contribute equally to the capital of the venture, and that the parties share in
profits at a negotiated rate. The agreements are silent as to the allocation of losses.

Acme entered into such a relationship with Brown Construction Co. LLC (Brown), forming Acme-Brown
LLC (A-B LLC) to build 50 homes. The operating agreement for A-B LLC gives Acme a 55% voting
interest and provides for a 20%/80% division of profits in favor of Brown.

A-B LLC built all 50 homes and sold them to homeowners. The members received a distribution of
profits from the sales, split between them according to their agreement on the division of profits.
However, all the concrete manufactured by Acme and sold to A-B LLC for the foundations of the homes
proved to be defective. After a year, the concrete dissolved, collapsing the homes and rendering them
worthless. In a class action by the homeowners against A-B LLC, the plaintiffs were awarded a $15
million judgment. The LLC has no assets with which to pay the judgment.

Although Acme would be liable to A-B LLC for the loss caused by the defective concrete, A-B LLC has
not brought a claim against Acme. Acme has the financial resources to pay damages equal to the amount
of the $15 million judgment in the homeowners’ lawsuit and to fully cover A-B LLC’s liability.

Brown has sent a letter to A-B LLC demanding that A-B LLC bring a claim against Acme to recover
those damages and pay the judgment to the plaintiffs, after which A-B LLC would be dissolved. But
Acme, as the manager of A-B LLC, has refused to do so.

Acme’s lawyer has sent a letter to Brown stating the following:

(1) Acme has no fiduciary obligations to either A-B LLC or Brown that require it to have A-B LLC bring
the concrete claim against Acme.

(2) Brown cannot bring a claim against Acme.

(3) Brown does not have sufficient grounds to seek the judicial dissolution of A-B LLC.

(4) Because the A-B LLC agreement provides for a 20%/80% division of profits, the losses arising from
the judgment obtained by the plaintiffs against the LLC should also be allocated 20% to Acme and 80%
to Brown.

Is Acme’s lawyer correct? Explain.

Seperac-J19 Exam-Released MEE Essay Compilation © 2016-2020 752


#169: J12-8 MEE: ANSWER: NCBE (CORPS-LLCS)

POINT (1) [25%] ISSUE: Does Acme have fiduciary duties as majority voting member that would
require it to have A-B LLC bring the concrete claim against Acme? ANSWER: Yes. Acme owes A-
B LLC and Brown fiduciary duties of loyalty and care. Refusing to have the LLC bring the
concrete claim violates Acme’s fiduciary duties because the refusal is self-interested and not in the
LLC’s best interests.

POINT (2) [25%] ISSUE: Can Brown bring the concrete claim against Acme? ANSWER: Yes. The
claim based on the defective concrete sold by Acme to A-B LLC is a claim of the LLC as an entity,
not of the LLC’s members. Thus, Brown can maintain a derivative action on behalf of A-B LLC, in
which the LLC brings its concrete claim against Acme. But Brown cannot maintain a claim on
Brown’s own behalf directly against Acme.

POINT (3) [25%] ISSUE: Does Brown have sufficient grounds to seek the dissolution of A-B LLC?
ANSWER: Yes. Based on Acme’s unjustified refusal to advance the concrete claim, Brown may
have grounds to seek judicial dissolution of A-B LLC.

POINT (4) [25%] ISSUE: Should A-B LLC’s losses be borne by Acme and Brown and allocated
between them in the same way as its profits? ANSWER: No. Under the rule of limited liability, LLC
members and managers are not liable beyond their investment in the LLC for losses incurred by
the LLC. There are no facts to support piercing the LLC veil, nor did the homeowners bring direct
claims against Acme or Brown.

ANSWER DISCUSSION:

Acme’s lawyer is wrong in almost every respect. As the member of A-B LLC with management control,
Acme has breached its fiduciary duties of loyalty and care by refusing to have the LLC bring the concrete
claim. Brown can bring the concrete claim on behalf of the LLC against Acme as a derivative action,
though not as a direct action against Acme. Brown may have sufficient grounds to seek judicial
dissolution of A-B LLC, because Acme’s failure to have the LLC bring the concrete claim likely
constitutes oppression. Finally, under the rule of limited liability, the members and managers of an LLC
generally are not liable for LLC losses. Thus, the A-B LLC losses arising from the judgment against the
LLC are not borne by Acme or Brown, nor are they allocated between them. [NOTE: The resolution of
the issues in this problem would generally be the same in all jurisdictions. The Uniform Limited Liability
Company Act (ULLCA) has been adopted in 9 jurisdictions.]

ANSWER EXPLANATION:

Explanation to Point-One (25%):

Acme owes A-B LLC and Brown fiduciary duties of loyalty and care. Refusing to have the LLC bring the
concrete claim violates Acme’s fiduciary duties because the refusal is self-interested and not in the LLC’s
best interests.

A-B LLC is a member-managed limited liability company (LLC). The operating agreement varies the
statutory default to give Acme a 55% voting interest, which is permissible under ULLCA § 110. In
exercising its management powers, Acme has fiduciary duties under ULLCA § 409 as a member in a

Seperac-J19 Exam-Released MEE Essay Compilation © 2016-2020 753


member-managed LLC to both A-B LLC and Brown. These fiduciary duties include both a duty of
loyalty and a duty of care.

The ULLCA does not specifically address whether the duty of loyalty precludes a majority member from
refusing to have the LLC bring a surefire claim against the member. But the ULLCA provides that
members have a duty of loyalty to “account to the company and to hold as trustee for the company any
benefit derived by the member in the conduct of the company’s activities.” The benefit that Acme derives
by virtue of A-B LLC not bringing a claim against Acme is an improper benefit, and thus keeping the
benefit is a breach of Acme’s duty of loyalty.

The ULLCA provides that, subject to the business-judgment rule, the duty of care requires the member to
“act with the care that a person in a like position would reasonably exercise under similar circumstances
and in a manner the member reasonably believes to be in the best interests of the company.” Because
refusing to have the LLC bring the concrete claim against Acme is self-interested, the business-judgment
rule is inapplicable. Under the facts given, Acme cannot reasonably believe that it is in A-B LLC’s best
interest to not bring the surefire concrete claim against Acme.

Explanation to Point-Two (25%):

The claim based on the defective concrete sold by Acme to A-B LLC is a claim of the LLC as an entity,
not of the LLC’s members. Thus, Brown can maintain a derivative action on behalf of A-B LLC, in which
the LLC brings its concrete claim against Acme. But Brown cannot maintain a claim on Brown’s own
behalf directly against Acme.

Brown, as a member of the LLC, may maintain a derivative action under ULLCA § 902 to enforce A-B
LLC’s concrete claim against Acme. A derivative action in a member-managed LLC may be brought only
if (1) a demand is made on the other member to bring an action and the member fails to do so, or (2) such
demand would be futile. Here, Brown has made a demand on Acme and Acme has failed to have the LLC
bring an action against Acme. Therefore, Brown may bring a derivative action against Acme.

Under ULLCA § 104, an LLC is an entity distinct from its members. The claim based on the faulty
concrete is a claim of A-B LLC as an entity, not a claim of its constituent members. For this reason,
Brown cannot advance the concrete claim on its own behalf. This is consistent with the direct action
provision, ULLCA § 901, under which Brown would have to “plead and prove an actual or threatened
injury that is not solely the result of an injury suffered or threatened to be suffered by the limited liability
company.”

Explanation to Point-Three (25%):

Based on Acme’s unjustified refusal to advance the concrete claim, Brown may have grounds to seek
judicial dissolution of A-B LLC.

Brown could apply to the court for an order of involuntary dissolution under ULLCA § 701, alleging that
Acme, as the member in control of the company, is acting in a manner that is oppressive and directly
harmful to Brown.

The comment to ULLCA § 701 states that courts have begun to apply close corporation “oppression”
doctrine to LLCs. Courts in the close corporation context have found oppression when actions by
controlling shareholders violate the reasonable expectations of non-controlling shareholders. Thus, the
question becomes whether Acme’s refusal to have the LLC bring a surefire claim against Acme violates

Seperac-J19 Exam-Released MEE Essay Compilation © 2016-2020 754


Brown’s reasonable expectations. At a minimum, Brown had reasonable expectations that Acme would
not violate its fiduciary duties. In addition, Acme’s refusal to have the LLC bring the concrete claim
violates Brown’s reasonable expectation that Acme would manage the LLC in the best interests of the
LLC. Finally, Acme’s refusal directly harms Brown, because unless Acme makes good on the LLC
losses, Brown will suffer reputational harm for being involved in a construction project where the
customers lost all their money.

Explanation to Point-Four (25%):

Under the rule of limited liability, LLC members and managers are not liable beyond their investment in
the LLC for losses incurred by the LLC. There are no facts to support piercing the LLC veil, nor did the
homeowners bring direct claims against Acme or Brown.

Although allocation of losses among partners in a partnership follows the allocation of profits, no
allocation rule applies in an LLC because of limited liability. Under the rule of limited liability, if the
LLC becomes indebted, obligated, or otherwise liable to an outside party, no member or manager
becomes liable on that debt, obligation, or liability solely by reason of acting as a member or manager.
Thus, if A-B LLC lacks the assets to satisfy the judgment awarded to the homeowners, there can be no
liability of either Acme or Brown on this judgment – unless a court pierces the LLC veil or finds that one
of the members had acted to create direct liability.

Under the facts, there is no indication of fraud or other inequitable conduct in the formation or operation
of A-B LLC that would support a piercing claim against Acme or Brown. Although state courts have
recognized piercing in the LLC context, there must exist some circumstances that would justify piercing
on equitable grounds, such as undercapitalization of the business, failure to follow formalities,
commingling of assets, confusion of business affairs, or deception of creditors. None of these factors
exists here.

At most, the homeowners may have had a direct claim against Acme for supplying defective concrete, but
the homeowners did not bring this claim against Acme. Nor is there any indication that Brown was
responsible for losses caused by the defective concrete. Therefore, because no claims were brought
against either Acme or Brown, there are no losses for which either of them is liable directly, and there is
no basis for allocating losses between them. If A-B LLC lacks the assets to satisfy the judgment awarded
against it, any shortfall will be a loss borne by the homeowners.

Seperac-J19 Exam-Released MEE Essay Compilation © 2016-2020 755


#170-FEB 2012–MEE Q09: QUESTION NINE (CORPS-LLCS)

A corporation’s articles of incorporation state that the corporation shall have a seven-member board of
directors. Neither the articles of incorporation nor the corporation’s bylaws contain any special provisions
regarding the board of directors.

On March 1, the corporation’s president told its secretary to convene a special meeting of the board of
directors. Accordingly, the secretary prepared a Notice of Special Meeting (Notice) and sent it by
overnight mail to six of the seven directors. The secretary did not send the Notice to the seventh director –
Claire – because Claire had recently moved and the corporation did not have a current mailing address for
her.

The Notice stated only that a special meeting of the corporation’s board of directors would be held on
March 31 at 10 a.m., at the corporate headquarters.

On March 2, each member of the board of directors except Claire received the Notice. Directors Alan and
Barb, both of whom had vacation plans for March 31, made arrangements with the secretary to participate
in the special meeting by telephone.

On March 30, Alan called Claire and informed her that a special meeting of the board of directors was
going to be held on March 31.

On March 31, five members of the board of directors (including Claire but neither Alan nor Barb)
gathered in the corporation’s conference room. Alan and Barb called in from their vacation homes. The
five directors present in the conference room could hear both Alan and Barb. Alan and Barb could each
hear the five directors in the conference room but could not hear each other.

After a lengthy discussion, the board of directors voted 4-3 to approve the corporation’s purchase of a
major asset. Alan and Barb both voted to approve the purchase.

Claire, who voted against the purchase, is very upset and has brought an action seeking an injunction to
prevent the purchase of the asset. Claire asserts that the board of directors did not properly approve the
purchase of the asset.

Did the board of directors properly approve the purchase of the asset? Explain.

Seperac-J19 Exam-Released MEE Essay Compilation © 2016-2020 756


#170: F12-9 MEE: ANSWER: NCBE (CORPS-LLCS)

POINT (1)(a) [25%] ISSUE: Did the directors other than Claire receive proper notice of the special
meeting of the board of directors? ANSWER: Yes. The directors other than Claire received proper
notice of the special meeting of the board of directors.

POINT (1)(b) [25%] ISSUE: Did Claire receive proper notice of the special meeting of the board of
directors and, if not, did she waive notice? ANSWER: No. Claire did not receive proper notice of
the special meeting of the board of directors; however, she waived such notice.

POINT (2) [25%] ISSUE: Was a quorum present at the special meeting of the board of directors?
ANSWER: Yes. A quorum was present at the special meeting of the board of directors.

POINT (3) [25%] ISSUE: Did the purchase of the asset receive the required number of votes in
favor to be approved? ANSWER: No. The purchase of the asset did not receive the required
number of votes and was therefore not properly approved by the board of directors.

ANSWER DISCUSSION:

Proper notice of the special meeting of the board of directors was given to all directors other than Claire.
Claire did not receive proper notice of the special meeting, but she waived the right to object to actions
taken at the special meeting when she attended the special meeting and voted. A quorum (a majority of
the directors) is required for action taken at a meeting to be proper. Here, a quorum was present at the
special meeting. However, Alan and Barb were not able to hear each other and therefore cannot be
counted as participating in the meeting. Therefore, in determining whether the purchase of the asset
received the required number of votes to be proper, the votes of Alan and Barb may not be counted. Thus,
the purchase of the asset was not properly approved by the board because it did not receive the required
number of votes.

ANSWER EXPLANATION:

Explanation to Point-One(a) (25%):

The directors other than Claire received proper notice of the special meeting of the board of directors.

Unless the articles of incorporation or bylaws provide otherwise, notice of a special meeting of a
corporation’s board of directors must be given at least two days prior to the date of the meeting. The
notice must include information regarding the time, location, and date of the meeting but does not need to
include information regarding the purpose of the meeting.

Here, the facts state that the corporation’s articles of incorporation and bylaws do not contain any special
provisions regarding the board of directors. Notice of the special meeting was sent to six of the seven
directors 29 days before the meeting was to be held, and it specified a time, location, and date. Therefore,
with respect to those six directors, the notice of the special meeting was proper.

[NOTE: Although the references in the analysis are to the MBCA, the result throughout is the same under
the Delaware General Corporation Law.]

Seperac-J19 Exam-Released MEE Essay Compilation © 2016-2020 757


Explanation to Point-One(b) (25%):

Claire did not receive proper notice of the special meeting of the board of directors; however, she waived
such notice.

A director who does not receive notice of a special meeting of the board of directors at least two days
prior to the meeting does not receive proper notice of that meeting. However, a director who attends a
special meeting of the board of directors despite not receiving proper notice waives such notice unless the
director objects to the holding of the meeting and thereafter does not vote at the meeting. Here, there is no
doubt that Claire did not receive proper notice of the special meeting. No notice was sent to Claire, who
learned of the meeting through a phone call with another director. Even if that phone call constitutes
notice, it was not received at least two days prior to the special meeting and therefore is not proper.

However, Claire attended and voted at the special meeting. By these actions, Claire waived the notice
requirement and therefore cannot claim that proper notice of the special meeting was not given.

Explanation to Point-Two (25%):

A quorum was present at the special meeting of the board of directors.

In order for action taken at a special meeting of directors to be proper, a quorum must be present at the
meeting. Unless the articles of incorporation or bylaws provide otherwise, when a corporation has a fixed
number of directors, a quorum consists of a majority of that fixed number.

Here, the facts state that the corporation’s articles of incorporation mandate a seven-member board of
directors and its articles of incorporation and bylaws contain no special provisions regarding the board of
directors. Therefore, a quorum for the corporation’s purposes consists of four directors. Here, while not
all seven members of the board of directors were legally present at the special meeting, five of them (all
but Alan and Barb) were. Therefore, a quorum was present, and the meeting was legally held.

Explanation to Point-Three (25%):

The purchase of the asset did not receive the required number of votes and was therefore not properly
approved by the board of directors.

Unless the articles of incorporation or bylaws provide otherwise, if a quorum is present when a vote is
taken, a majority vote of directors present is the act of the board of directors. Here, a quorum was present
when the vote was taken. However, there were only five members of the board of directors present at the
special meeting. Alan and Barb were not legally present.

While directors generally are entitled to participate in special meetings over the telephone, as Alan and
Barb attempted to do, such participation is valid only if all directors participating “may simultaneously
hear each other during the meeting.” Only directors who satisfy this requirement are deemed to be present
at the meeting. Here, the facts state that Alan and Barb could hear and be heard by the five directors
present in the conference room but that Alan and Barb could not hear each other. Because Alan and Barb
could not hear each other, neither was legally present at the meeting.

Because Alan and Barb were not legally present at the meeting, their votes on the purchase of the asset
may not be counted. Therefore, the vote on the purchase of the asset was two votes in favor and three
votes against, and the purchase of the asset was not approved by the board of directors.

Seperac-J19 Exam-Released MEE Essay Compilation © 2016-2020 758


#171-FEB 2011–MEE Q07: QUESTION SEVEN (CORPS-LLCS)

On September 1, Adam, Baker, and Clark formed a shoe manufacturing business called Delta Corporation
(Delta). Each was to be a shareholder. Adam was named president of Delta.

Adam agreed to prepare and file articles of incorporation and bylaws for Delta, in accordance with the
state’s corporation statute, which is identical to the Model Business Corporation Act (1984, with 2000
amendments). Adam, Baker, and Clark agreed to include a provision in Delta’s articles of incorporation
stating that the corporation’s existence would begin on September 1.

On October 1, Adam, acting on behalf of Delta, entered into a contract with Mega Stores Corporation
(Mega) pursuant to which Mega was to purchase shoes from Delta for $3,000. Following delivery of the
shoes and after Mega had paid in full, Mega discovered that the shoes did not conform to the contract
specifications and returned the shoes to Delta. It is undisputed that Delta owes Mega the $3,000 purchase
price.

On October 15, Baker learned that Delta’s articles of incorporation had not been filed.

On November 1, Adam, acting on behalf of Delta, entered into a contract with Sole Source, Inc. (Sole), a
supplier of shoe soles, pursuant to which Delta purchased shoe soles from Sole for $100,000. The soles
were delivered to Delta, and it is uncontested that Delta owes Sole the $100,000 purchase price. Adam
learned of the opportunity to contract with Sole from Baker, who had worked with Sole in the past. Baker
helped Adam negotiate the contract with Sole.

On November 15, Adam filed Delta’s articles of incorporation with the appropriate state official.

When Delta did not pay either Mega or Sole the amounts it owed them, each company sued Delta, Adam,
Baker, and Clark for the amounts owed.

At all times, Clark believed that Delta’s articles of incorporation had been filed.

1. When did Delta’s corporate existence begin? Explain.

2. Is Adam, Baker, or Clark personally liable on the Mega contract? Explain as to each.

3. Is Adam, Baker, or Clark personally liable on the Sole contract? Explain as to each.

Seperac-J19 Exam-Released MEE Essay Compilation © 2016-2020 759


#171: F11-7 MEE: ANSWER: NCBE (CORPS-LLCS)

POINT (1) [20%] ISSUE: When did Delta’s corporate existence begin? ANSWER: Delta achieved
corporate status upon the filing of its articles of incorporation on November 15. The filing of Delta’s
articles of incorporation does not relate back to the earlier date of September 1 specified in the
articles.

POINT (2) [40%] ISSUE: Is Adam, Baker, or Clark personally liable on the Mega contract?
ANSWER: Yes. When a corporation’s articles of incorporation have not been filed, persons
purporting to act for the corporation are personally liable for corporate debts if they knew when
the debt was incurred that there was no incorporation.

POINT (3) [40%] ISSUE: Is Adam, Baker, or Clark personally liable on the Sole contract?
ANSWER: Yes. Persons purporting to act for a corporation are personally liable for a corporate
contract if they entered into the contract with knowledge that there was no incorporation.

ANSWER DISCUSSION:

Delta’s corporate existence began only upon Adam’s filing of Delta’s articles of incorporation on
November 15. The recitation in the articles of a stated earlier effective date (September 1) is of no legal
consequence. Only Adam is personally liable on the Mega contract because Adam alone participated in
entering into the contract with knowledge that the articles had not been filed. Adam and Baker, but not
Clark, are liable on the Sole contract. That contract was executed by Delta before its articles were filed.
Adam and Baker, but not Clark, participated in entering into the contract with knowledge that Delta’s
articles had not been filed. Thus, Adam and Baker are personally liable for the Sole contract, and Clark is
not personally liable.

ANSWER EXPLANATION:

Explanation to Point-One (20%):

Delta achieved corporate status upon the filing of its articles of incorporation on November 15. The filing
of Delta’s articles of incorporation does not relate back to the earlier date of September 1 specified in the
articles.

Under the Model Business Corporation Act (MBCA) § 2.03, “corporate existence begins when the
articles of incorporation are filed.” Because “incorporation under modern statutes is so simple and
inexpensive nothing short of filing articles of incorporation should create the privilege of limited
liability.”

This date-of-filing rule applies even if the filed articles recite an effective date prior to the date of filing.
The MBCA allows the parties to specify “a delayed effective date” but not an earlier effective date. Thus,
the recitation in Delta’s articles of an earlier effective date is ineffective; Delta’s corporate existence
began on the date that Adam filed the articles.

Explanation to Point-Two (40%):

Seperac-J19 Exam-Released MEE Essay Compilation © 2016-2020 760


When a corporation’s articles of incorporation have not been filed, persons purporting to act for the
corporation are personally liable for corporate debts if they knew when the debt was incurred that there
was no incorporation.

The fact that Delta’s articles of incorporation were not filed when Delta entered into the contract with
Mega does not automatically make Adam, Baker, and Clark liable to Mega for amounts due on that
contract. Under the MBCA, “all persons purporting to act as or on behalf of a corporation, knowing there
was no incorporation under this Act, are jointly and severally liable for all liabilities created while so
acting.” Thus, under § 2.04 both the participation and knowledge requirements must be met to hold any of
Adam, Baker, and Clark liable.

With regard to the Mega contract, only Adam purported to act on Delta’s behalf and only Adam knew that
Delta’s articles had not been filed. Therefore, Adam is personally liable on the Mega contract. On the
other hand, neither Baker nor Clark satisfies either the participation or knowledge requirement. Thus,
neither is liable on the Mega contract.

[NOTE: The facts clearly state when Baker learned that the articles had not been filed: October 15.
Clark, however, at all times believed that Delta’s articles of incorporation had been filed.]

[NOTE: Examinees may discuss the de facto incorporation doctrine under which a shareholder is not
liable for the obligations of a defectively incorporated corporation when there was “a bona fide attempt
to organize and colorable or apparent compliance with the requirements of the law.” Such analysis is
inapplicable here. First, the MBCA (which the facts state governs this problem) abolished the doctrine.
Even in a non-MBCA jurisdiction, the doctrine would not apply here because Adam did nothing to
attempt compliance with the statutory requirements. Without allegations that a certificate of formation
was ever prepared or filed, there is no basis to claim a good-faith attempt to comply with statutory
requirements.]

Explanation to Point-Three (40%):

Persons purporting to act for a corporation are personally liable for a corporate contract if they entered
into the contract with knowledge that there was no incorporation.

When a corporation’s articles of incorporation have not been filed, a person is liable for pre-incorporation
transactions only when the person purporting to act on behalf of a corporation not yet formed possesses
actual knowledge that the corporation’s charter has not yet been issued. It is not enough to establish
liability that a person should have inquired about the entity’s status and should have known that the
corporation was not formed, as those facts do not rise to the level of actual knowledge required.

Here, with regard to the Sole contract, Adam and Baker knew that Delta’s articles of incorporation had
not been filed at the time Delta entered into that contract, while Clark did not know. Baker might argue
that, as a mere prospective shareholder, he was not a “person purporting to act as or on behalf of a
corporation,” and thus has no liability. This argument is likely to fail. With regard to the Sole contract,
Baker was more than a prospective shareholder – he participated in the negotiation of the contract and
therefore purported to act on behalf of a corporation. Further, the official commentary to § 2.04 suggests
that its protection does not extend to individuals in Baker’s position. (“Section 2.04 does not foreclose the
possibility that persons who urge defendants to execute contracts in the corporate name knowing that no
steps to incorporate have been taken may be estopped to impose personal liability on individual
defendants.”) Rather, the provision is intended to shield those who erroneously but in good faith believe
that the necessary documents have been filed. Here, at the time of the Sole contract, Baker knew that

Seperac-J19 Exam-Released MEE Essay Compilation © 2016-2020 761


Delta’s articles had not been filed and therefore did not have the necessary good-faith belief to the
contrary. Thus, Baker, together with Adam, will be personally liable on the Sole contract. Clark, however,
is not liable because he does not meet the participation requirement nor does he satisfy the knowledge
requirement; he believed at all relevant times that the articles had in fact been filed. However, even if
Clark had knowledge that the articles had not been filed, he would not be liable on the Sole contract
because he does not meet the participation requirement.

Seperac-J19 Exam-Released MEE Essay Compilation © 2016-2020 762


#172-JUL 2010–MEE Q09: QUESTION NINE (CORPS-LLCS)

On December 30, X Corporation’s legal record date, X Corporation had 100 shares of issued and
outstanding common stock. Fifty shares were owned by Amy, 25 shares were owned by Brian, and 25
shares were owned by Carter. X Corporation also had 50 shares of stock that it previously had issued to,
but later repurchased from, Amy.

On January 30, X Corporation’s annual shareholders’ meeting was validly held. Before the meeting, X
Corporation’s staff prepared a list of shareholders entitled to vote at the meeting and mailed proper notice
to them. That notice stated that a proposal requiring shareholder approval would be voted on at the annual
shareholders’ meeting.

Before the annual shareholders’ meeting and in a timely manner, Amy mailed in her duly executed proxy,
directing the secretary of X Corporation to vote her 50 shares in favor of the proposal. However, before
the annual shareholders’ meeting date, Zach called the secretary of X Corporation and truthfully told the
secretary that Amy’s shares belonged to Zach because he had bought the shares from Amy on December
31. Zach then mailed the secretary a duly executed proxy directing the secretary of X Corporation to vote
his 50 shares against the proposal.

Prior to the annual shareholders’ meeting, Brian duly executed a proxy in favor of Dell. The proxy stated
in its entirety, “I, Brian, hereby grant Dell full authority to vote my 25 shares of X Corporation at the
January 30th annual shareholders’ meeting.” Dell timely mailed a duly executed proxy directing the
secretary of X Corporation to vote Brian’s 25 shares against the proposal. Dell also sent the secretary a
copy of the proxy given to Dell by Brian. Brian, however, attended the annual meeting and voted his 25
shares in favor of the proposal.

Carter personally appeared at the annual shareholders’ meeting and voted his 25 shares against the
proposal.

X Corporation’s president attended the annual meeting and, on behalf of X Corporation, voted the 50
shares that X Corporation had repurchased from Amy against the proposal.

X Corporation’s Articles of Incorporation require an affirmative vote by the holders of two-thirds of the
shares entitled to be voted to approve any proposal at a shareholders’ meeting. The bylaws, on the other
hand, require a unanimous vote of such shares to approve any proposal.

Your law firm represents X Corporation. You have been asked to advise the firm’s senior partner on
whether the proposal received sufficient votes to be approved. Explain your conclusion.

Seperac-J19 Exam-Released MEE Essay Compilation © 2016-2020 763


#172: J10-9 MEE: ANSWER: NCBE (CORPS-LLCS)

POINT (1) [25%] ISSUE: Were the shares owned by Amy on the record date but then sold to Zach
properly voted by Amy or Zach? ANSWER: No. Only shareholders of record on the record date
are entitled to vote at an annual shareholders’ meeting. Here, Amy was the owner of the 50 shares
on the record date, and Zach is not entitled to vote them. Therefore, Amy’s vote of 50 shares in
favor of the shareholder proposal is counted, and Zach’s vote of 50 shares against the proposal is
not.

POINT (2) [25%] ISSUE: Were the shares subject to the proxy given to Dell by Brian properly
voted by Dell, or by Brian, who attended the meeting? ANSWER: Generally, a shareholder proxy is
revocable. Any action taken inconsistent with a proxy revokes that proxy. Here, Brian attended the
meeting and voted, which was inconsistent with Brian’s prior grant of a proxy to Dell. Therefore,
Brian effectively revoked the proxy, and his vote of 25 shares in favor of the shareholder proposal
should be counted.

POINT (3) [10%] ISSUE: Were the shares held by Carter properly voted by Carter? ANSWER:
Yes. A shareholder of record may attend and vote at an annual shareholders’ meeting. Thus
Carter’s vote of 25 shares will be counted against the proposal.

POINT (4) [20%] ISSUE: Were the shares voted by X Corporation’s president properly voted?
ANSWER: No. Shares that are repurchased by a corporation are not outstanding shares and
therefore may not be voted. Therefore, the vote of 50 shares by X Corporation’s president against
the proposal cannot be counted.

POINT (5) [20%] ISSUE: When the articles of incorporation of a corporation conflict with the
bylaws of a corporation regarding how many shares must vote in favor of a shareholder proposal,
which takes precedence? ANSWER: When the articles of incorporation conflict with a
corporation’s bylaws regarding how many shares must vote in favor of a shareholder proposal in
order for that proposal to be approved, the articles of incorporation prevail over the bylaws. Here,
the shareholder proposal received sufficient votes in favor and was approved.

ANSWER DISCUSSION:

Amy’s shares were properly voted by Amy in favor of the proposal even though she no longer owned the
shares when the meeting was held. Zach did not acquire ownership of those shares until after the record
date, and Amy did not grant Zach a proxy to vote those shares at any time. Brian’s shares were properly
voted in favor of the proposal because his earlier proxy to Dell was revoked when Brian attended the
meeting and voted his shares. Carter’s shares were properly voted against the proposal because he
properly attended the meeting and voted them. X Corporation’s president was not permitted to vote the
shares reacquired from Amy. When the articles of incorporation of a corporation conflict with its bylaws
regarding how many shares must be voted in favor of a shareholder proposal in order for that proposal to
be approved, the articles of incorporation preempt the bylaws. Therefore, the proposal must receive a two-
thirds vote of the outstanding shares entitled to vote on the matter to be approved. Here, the shareholder
proposal received sufficient votes in favor and was approved.

ANSWER EXPLANATION:

Seperac-J19 Exam-Released MEE Essay Compilation © 2016-2020 764


Explanation to Point-One (25%):

Only shareholders of record on the record date are entitled to vote at an annual shareholders’ meeting.
Here, Amy was the owner of the 50 shares on the record date, and Zach is not entitled to vote them.
Therefore, Amy’s vote of 50 shares in favor of the shareholder proposal is counted, and Zach’s vote of 50
shares against the proposal is not.

A record date determines who is entitled to vote at a particular shareholder meeting, “namely, those
persons who were registered as shareholders ‘of record’ on that date.” Here, on the record date of
December 30th, Amy was the owner of the 50 shares. Therefore, absent her proxy to another, Amy is
entitled to vote the 50 shares. The fact that Zach bought the shares does not give Zach the right to vote the
50 shares because he did so after the record date.

[NOTE: If Amy had given Zach a legally binding proxy together with the shares, Zach might have had the
right to vote them even though Zach was not the record owner on the record date. The facts do not
suggest that any proxy was given, however, and therefore Amy’s vote of 50 shares in favor of the proposal
should be counted, and Zach’s vote of 50 shares against the proposal should not be counted.]

Explanation to Point-Two (25%):

Generally, a shareholder proxy is revocable. Any action taken inconsistent with a proxy revokes that
proxy. Here, Brian attended the meeting and voted, which was inconsistent with Brian’s prior grant of a
proxy to Dell. Therefore, Brian effectively revoked the proxy, and his vote of 25 shares in favor of the
shareholder proposal should be counted.

Brian gave Dell a proxy to vote Brian’s shares. Shareholder proxies generally are revocable, and any
action inconsistent with the grant of a proxy works as a revocation of that proxy. Proxies must be
revocable unless coupled with an interest. A proxy may be made irrevocable only if the proxy form
explicitly so states and the proxy is “coupled with an interest.” An appointment of a proxy is revocable
unless the appointment form or electronic transmission states that it is irrevocable and the appointment is
coupled with an interest.

Here, the proxy given by Brian to Dell did not state that it was irrevocable, nor was it coupled with an
interest, and therefore it was revocable. Brian’s attendance at the shareholders’ meeting and his vote of
the 25 shares were actions inconsistent with the grant of a proxy covering those shares to Dell. Therefore,
Brian effectively revoked the proxy, and Brian’s vote of 25 shares in favor of the proposal should be
counted while Dell’s vote of 25 shares against the proposal should not be counted.

Explanation to Point-Three (10%):

A shareholder of record may attend and vote at an annual shareholders’ meeting. Thus Carter’s vote of 25
shares will be counted against the proposal.

A shareholder of record is entitled to attend and vote at an annual shareholders’ meeting unless he or she
has executed a valid, irrevocable proxy covering his shares. Here, Carter attended the meeting and
properly voted his 25 shares against the proposal.

Explanation to Point-Four (20%):

Seperac-J19 Exam-Released MEE Essay Compilation © 2016-2020 765


Shares that are repurchased by a corporation are not outstanding shares and therefore may not be voted.
Therefore, the vote of 50 shares by X Corporation’s president against the proposal cannot be counted.

Shares that are reacquired by a corporation are considered authorized but not outstanding. Some
jurisdictions refer to these as “treasury shares.”

In counting shareholder votes, “each outstanding share is entitled to one vote on each matter voted on at a
shareholders’ meeting.” The facts indicate that X Corporation’s president voted 50 shares that the
corporation reacquired from Amy. Because reacquired shares are not outstanding, they cannot be voted.
Therefore the 50 shares voted by the president do not count.

Explanation to Point-Five (20%):

When the articles of incorporation conflict with a corporation’s bylaws regarding how many shares must
vote in favor of a shareholder proposal in order for that proposal to be approved, the articles of
incorporation prevail over the bylaws. Here, the shareholder proposal received sufficient votes in favor
and was approved.

When a corporation’s articles of incorporation conflict with its bylaws regarding how many shares must
vote in favor of a shareholder proposal in order for that proposal to be approved, the articles of
incorporation preempt the bylaws. Any bylaw provision that conflicts with the articles of incorporation is
void. Here, X Corporation’s Articles of Incorporation requiring approval of two-thirds of the shares
outstanding and entitled to vote on the matter to approve any shareholder action will preempt the bylaws’
requirement of unanimity.

Here, Amy’s 50 votes and Brian’s 25 votes in favor of the proposal are counted, while only Carter’s 25
votes against the proposal are counted. Therefore, the proposal received the necessary approval by two-
thirds of the shares outstanding and entitled to vote on the matter and should pass.

Seperac-J19 Exam-Released MEE Essay Compilation © 2016-2020 766


#173-FEB 2010–MEE Q05: QUESTION FIVE (CORPS-LLCS)

Smith owns 10% of the common shares of Omega, Inc., a closely held corporation. Baker and Jones each
own 45% of Omega’s common shares. Baker and Jones also serve on Omega’s board of directors and are
paid corporate officers.

Omega has not paid a dividend on its common shares for several years. Smith, who is not an officer of the
corporation and has never received a salary from the corporation, is very unhappy that no dividends are
being paid.

When Smith complained to Baker and Jones about nonpayment of dividends, they said that while Omega
could legally pay dividends, it has not done so in order to retain the corporation’s earnings for expansion
of the business. They also pointed to data showing that Omega’s business has expanded considerably in
the past several years, financed entirely through undistributed earnings, and told Smith that he should “go
away and let us run the show.” Smith complained that “only you are enjoying the fruits of Omega’s
success.” In response to an inquiry from Smith, Baker and Jones refused to reveal the amounts of their
salaries, even though those salaries are within industry range.

Baker and Jones each offered to purchase all of Smith’s shares for $35 per share. Smith suspects that the
shares are worth more than $35 per share. Smith has asked to inspect Omega’s corporate books and
records in order to determine the value of his shares, but Jones and Baker have refused to give Smith
access to any corporate records.

Smith has asked your law firm the following questions:

1. Does Smith have a right to inspect Omega’s corporate books and records to determine whether
$35 per share is a fair price for his shares? Explain.

2. If Smith brings a suit to compel the payment of a dividend, must Smith first make a demand on
the corporation? Explain.

3. If Smith brings a suit to compel the payment of a dividend, is that suit likely to be successful?
Explain.

Seperac-J19 Exam-Released MEE Essay Compilation © 2016-2020 767


#173: F10-5 MEE: ANSWER: NCBE (CORPS-LLCS)

POINT (1) [25%] ISSUE: Does Smith have a right to inspect Omega’s corporate books and records
to determine whether $35 per share is a fair price for his shares? ANSWER: Yes. As a shareholder,
Smith has a right to inspect corporate books and records for a proper purpose. Determining the
value of Smith’s shares for purpose of sale is a proper purpose.

POINT (2) [30%] ISSUE: Is Smith required to make a demand on the corporation prior to bringing
a suit to compel a dividend? ANSWER: No. A suit to compel the payment of a dividend is a suit
seeking to enforce an individual right of the shareholder and not a suit in a representative capacity
seeking to enforce a right of the corporation. Therefore, no demand on the corporation is required
prior to bringing suit.

POINT (3) [45%] ISSUE: Is a suit to compel the payment of a dividend likely to be successful?
ANSWER: No. The decision as to whether to distribute corporate earnings as dividends or to retain
those earnings in order to expand the business is a matter that is generally within the business
judgment of the corporate directors. Smith’s suit to compel the payment of a dividend is unlikely to
be successful because, in order to succeed, Smith would be required to prove that dividends were
withheld in bad faith.

ANSWER DISCUSSION:

Smith has a right to inspect corporate books and records to determine the value of his shares because this
purpose is reasonably related to Smith’s interest as a shareholder. Smith is not required to make a demand
upon the corporation prior to bringing a suit to compel the payment of a dividend because such a suit
attempts to vindicate an individual right of Smith as a shareholder. It is not a suit by Smith as a
representative of Omega to vindicate a corporate right. However, a suit to compel the payment of a
dividend is unlikely to be successful because, even if funds are legally available to pay dividends, Smith
can compel the payment of a dividend only by showing that the decision to withhold dividends was made
in bad faith.

ANSWER EXPLANATION:

Explanation to Point-One (25%):

As a shareholder, Smith has a right to inspect corporate books and records for a proper purpose.
Determining the value of Smith’s shares for purpose of sale is a proper purpose.

A shareholder has a right to inspect corporate books and records for a proper purpose. A proper purpose is
a purpose reasonably related to a person’s interest as a shareholder. The determination of the value of
one’s own shares for purposes of sale is a proper purpose. The fact that Smith has not definitely decided
to sell does not prevent the valuation from being a proper purpose. For example, in one case involving
shares in a closely held corporation, the court held that no present intention to sell or concrete steps to sell
shares was required in order for the valuation of shares to be a proper purpose.

In contrast to a publicly traded corporation, a closely held corporation has no market that continuously
values its shares. The fact that Omega is a closely held corporation reinforces the conclusion that valuing
the shares is a proper purpose for inspection. Thus, “when a minority shareholder in a closely held

Seperac-J19 Exam-Released MEE Essay Compilation © 2016-2020 768


corporation whose stock is not publicly traded needs to value his or her shares in order to decide whether
to sell them, normally the only way to accomplish that is by examining the appropriate corporate books
and records.”

In sum, Smith has a right to inspect Omega’s corporate books and records because this purpose is
reasonably related to Smith’s interest as a shareholder.

[NOTE: An applicant might receive extra credit for recognizing that in order to exercise his inspection
rights, Smith must comply with procedural requirements, which generally include making a written
demand for inspection and allowing the corporation a certain length of time (typically five days) to
respond.]

Explanation to Point-Two (30%):

A suit to compel the payment of a dividend is a suit seeking to enforce an individual right of the
shareholder and not a suit in a representative capacity seeking to enforce a right of the corporation.
Therefore, no demand on the corporation is required prior to bringing suit.

A shareholder is generally required to make a demand on the corporation to take remedial action prior to
bringing a derivative suit. The MBCA requires a demand in all cases while Delaware requires a complaint
to allege a demand or a reason for failure to make a demand. However, a shareholder is not required to
make a demand on the corporation prior to bringing suit where the shareholder brings a suit in his or her
individual capacity to enforce an individual right of the shareholder. The important question, then, is
whether a suit to compel the payment of a dividend is a suit to enforce a right of the corporation or,
conversely, a suit to enforce an individual right of the shareholder.

A suit to compel the payment of a dividend is not a suit to enforce a right of the corporation. It is a suit to
enforce a right of the individual shareholder, that is, the shareholder’s right to share in the net profits of
the corporation. The right to dividends is an incident of the ownership of stock. Consequently, there is no
requirement that Smith make a demand on the corporation prior to bringing suit to compel the payment of
a dividend.

[NOTE: In some jurisdictions, a derivative suit may be appropriate and graders are encouraged to
ascertain local law on this issue. If an applicant approaches this issue solely from the perspective of the
shareholder bringing a derivative suit, then the grader should also expect some analysis of demand
futility. Presumably, Baker and Jones, having made the decision not to pay a dividend, are unlikely to
agree to have the corporation sue the directors for failure to declare a dividend.]

Explanation to Point-Three (45%):

The decision as to whether to distribute corporate earnings as dividends or to retain those earnings in
order to expand the business is a matter that is generally within the business judgment of the corporate
directors. Smith’s suit to compel the payment of a dividend is unlikely to be successful because, in order
to succeed, Smith would be required to prove that dividends were withheld in bad faith.

While directors have fiduciary duties, the business judgment rule protects them from liability for breach
of their fiduciary duties. The business judgment rule is a presumption that in making business decisions,
the directors act on an informed basis, in good faith, and in the honest belief that the action being taken is
in the best interests of the corporation. The decisions regarding whether to declare a dividend and the
amount of any dividend declared are generally matters within the business judgment of the directors.

Seperac-J19 Exam-Released MEE Essay Compilation © 2016-2020 769


To prevail in a suit to compel the payment of a dividend, a shareholder must prove that there are funds
legally available for the payment of a dividend and that the directors acted in bad faith in their refusal to
pay. The plaintiff must show bad faith, fraud, breach of fiduciary duty, or abuse of discretion on the part
of the directors. “The essential test of bad faith is to determine whether the policy of the directors is
dictated by their personal interests rather than the corporate welfare.”

Usually the plaintiff-shareholder will attempt to prove bad faith by establishing various “earmarks of bad
faith.” These earmarks include “intense hostility of the controlling faction against the minority; exclusion
of the minority from employment by the corporation; high salaries, or bonuses or corporate loans made to
the officers in control; the fact that the majority group may be subject to high personal income taxes if
substantial dividends are paid; and the existence of a desire by the controlling directors to acquire the
minority stock interests as cheaply as possible.” Here, there may be several earmarks of bad faith. Baker
and Jones seemed hostile toward Smith, telling him to “go away and let us run the show.” They refused to
disclose their salaries. Further, it could be argued that Baker and Jones were trying to acquire Smith’s
shares as cheaply as possible. The facts do not establish whether $35 per share is a fair price.

The earmarks “are not, however, invariably signs of improper behavior by the majority.” If these
earmarks are not the “motivating causes” of the board’s dividend decision, they do not constitute bad
faith. In a number of cases, courts have recognized that a good-faith decision to retain corporate earnings
for business expansion is an appropriate exercise of business judgment. In judging the decision of the
board of a closely held corporation, a court may also consider that “directors of a closely held, small
corporation must bear in mind the relatively limited access of such an enterprise to capital markets.”

In sum, a suit to compel the payment of a dividend is unlikely to be successful because, even if funds are
legally available to pay dividends, Smith could succeed only by showing that the decision to withhold
dividends was made in bad faith or, in other words, was dictated by the personal interests of the directors
rather than the corporate welfare. It is unlikely that he can do this when a proper motive for withholding
the payment of dividends exists, as it does here.

Seperac-J19 Exam-Released MEE Essay Compilation © 2016-2020 770


#174-FEB 2009–MEE Q09: QUESTION NINE (CORPS-LLCS)

Corporation has 20 shareholders. Its largest shareholder, Major, owns just over 30 percent (30%) of
Corporation’s shares. No other shareholder owns more than five percent (5%) of the shares.

Major is also the president and one of the five directors of Corporation. The other four directors are also
shareholders of Corporation.

Over the past two years Major, acting in his capacity as president of Corporation, has persuaded
Corporation’s board of directors (Board) to approve the purchase of a number of valuable items of
Major’s personal property appropriate for Corporation’s business. Corporation paid vastly inflated prices
for Major’s property.

Major always informed Board of each proposed purchase before it was made. In each case, Minor,
another member of Board, asked Major whether the purchase price was “fair.” Major always replied: “I
have investigated the value of my property to be purchased by Corporation and I assure you that the
purchase price represents its fair market value.” Board, relying on this statement and undertaking no
further inquiry, always approved the purchases, with Major abstaining from voting.

Corporation’s articles of incorporation contain a provision that exculpates the directors of the corporation
for liability to the corporation for money damages “to the fullest extent permitted” by the applicable
corporation-law statute.

A shareholder derivative suit has been properly brought against Corporation’s directors seeking money
damages for breach of their fiduciary duties as directors with regard to the transactions between
Corporation and Major.

1. Will the directors (other than Major) be protected from liability by the business judgment rule?
Explain.

2. Will Major be protected from liability by the business judgment rule? Explain.

3. Will Major be protected from liability by Board’s approval of the transactions? Explain.

4. Will the directors (other than Major) be protected from liability by the exculpatory provision in
the articles of incorporation? Explain.

5. Will Major be protected from liability by the exculpatory provision in the articles of
incorporation? Explain.

Seperac-J19 Exam-Released MEE Essay Compilation © 2016-2020 771


#174: F09-9 MEE: ANSWER: NCBE (CORPS-LLCS)

POINT (1) [41%] ISSUE: Does the business judgment rule protect from liability a director who
authorizes a transaction in which another director has a financial interest without obtaining full
disclosure? ANSWER: No. The directors (other than Major) will not be protected from liability by
the business judgment rule because they did not exercise adequate care when they failed to seek and
receive sufficient information about the transactions.

POINT (2) [18%] ISSUE: Does the business judgment rule protect from liability a director who is
interested in a transaction and who does not make full disclosure about that transaction?
ANSWER: No. Major will not be protected from liability by the business judgment rule because
Major has an interest in the transactions in question.

POINT (3) [18%] ISSUE: Does the approval of the transaction by the disinterested members of the
board of directors protect from liability a director who is interested in a transaction and who does
not make full disclosure about that transaction? ANSWER: No. Major will not be protected from
liability by the approval of the transactions by a disinterested board of directors because Major
failed to make full disclosure regarding the transactions to the directors who approved the
transactions.

POINT (4) [12%] ISSUE: Does an exculpatory provision in the articles of incorporation protect
from liability a director who authorizes a transaction in which another director has an interest
without obtaining full disclosure? ANSWER: The directors (other than Major) will be protected
from liability by the exculpatory provision in the articles of incorporation.

POINT (5) [12%] ISSUE: Does an exculpatory provision in the articles of incorporation protect
from liability a director who is interested in a transaction and who does not make full disclosure
about that transaction? ANSWER: No. Major will not be protected from liability by the
exculpatory provision in the articles of incorporation because Major breached his duty of loyalty
and received an improper personal benefit from Corporation.

ANSWER DISCUSSION:

The directors (other than Major) will not be protected from liability by the business judgment rule because
they did not exercise adequate care in authorizing the transactions with Major. Major will not be protected
from liability by the business judgment rule because he is interested in the transactions and did not
disclose all relevant facts about the transactions to the other directors. Nor will Major be protected from
liability by the approval of the transactions by a disinterested board of directors because he failed to make
full disclosure regarding the transactions to the directors who approved the transactions. The directors
(other than Major) will be protected from liability by the exculpatory provision in the articles of
incorporation because their breach was of the duty of care, not the duty of loyalty, and the exculpatory
provision protects against breaches of the duty of care. Major, however, will not be protected from
liability by the exculpatory provision because he breached his duty of loyalty when he received an
improper financial benefit through the transactions with Corporation.

ANSWER EXPLANATION:

Explanation to Point-One (30-40%):

Seperac-J19 Exam-Released MEE Essay Compilation © 2016-2020 772


The directors (other than Major) will not be protected from liability by the business judgment rule because
they did not exercise adequate care when they failed to seek and receive sufficient information about the
transactions.

Directors owe a duty of loyalty and a duty of care. The directors other than Major received no personal
benefit from the transactions that they approved and did not stand on both sides of the transactions being
approved. Thus the directors, other than Major, did not violate the duty of loyalty.

Consequently, the claims against them are for failure to exercise the level of care required of directors. In
this case, the claim is likely to be that the directors failed to exercise care in informing themselves
regarding the transactions that they approved. Directors have a duty to exercise an informed business
judgment. Under the Model Business Corporation Act, directors must exercise care in becoming
informed.

The exercise of managerial powers by a director is generally subject to the business judgment rule. The
business judgment rule “is a presumption that in making a business decision, the directors of a corporation
acted on an informed basis, in good faith and in the honest belief that the action taken was in the best
interests of the company.” Consequently, “the party attacking a board decision as uninformed must rebut
the presumption that its business judgment was an informed one.”

Under the facts, the directors (other than Major) did not exercise adequate care in becoming informed
about the transactions in question: they relied entirely on Major’s assurances that the prices paid by
Corporation represented the fair market value of the properties purchased, they did not ask how he
determined the fair market value, and they never employed any independent party to evaluate the
properties being purchased, despite their awareness of Major’s interest in the transactions. In sum, the
directors exercised scant, if any, care in becoming informed.

In performing their duty to become informed, directors are generally entitled to rely upon information,
opinions, reports, or statements of corporate officers. In this case, however, the directors had knowledge
that made such reliance unwarranted. They knew that Major was interested in the transactions being
presented to the board for approval. Under the Model Business Corporation Act, a director is not
protected from liability in reliance where he or she has knowledge that makes reliance unwarranted. In
addition, Major’s statement that “I have investigated the value of my property to be purchased by
Corporation and I assure you that the purchase price to be paid by Corporation represents its fair market
value” probably lacked enough substance to be relied upon.

In sum, the directors (other than Major) are not entitled to the protection of the business judgment rule
because their decisions were uninformed.

Explanation to Point-Two (10-20%):

Major will not be protected from liability by the business judgment rule because Major has an interest in
the transactions in question.

Where a director stands on both sides of a transaction, the business judgment rule does not apply. A
director fulfills his duty as a director using good faith business judgment where the director is, among
other things, not interested in the subject of the transaction. Under the Model Business Corporation Act, a
director is not liable to a corporation unless the party asserting liability establishes one of a number of
elements, including receipt of financial benefit by the director to which the director is not entitled, or
other actionable breach of duty by the director to deal fairly with the corporation. Instead, the director has

Seperac-J19 Exam-Released MEE Essay Compilation © 2016-2020 773


the burden of proving the fairness of the transaction. Directors who are on both sides of a transaction in
question have the burden of establishing the entire fairness of the transaction.

Major will not be protected from liability by the business judgment rule because Major was interested in
the transactions and, consequently, Major will be required to establish the fairness of the transactions in
question in order to avoid a claim that he breached a fiduciary duty of loyalty.

Explanation to Point-Three (10-20%):

Major will not be protected from liability by the approval of the transactions by a disinterested board of
directors because Major failed to make full disclosure regarding the transactions to the directors who
approved the transactions.

Major may argue that since the transactions in question were approved by disinterested directors, the
transactions should be subject to the business judgment rule and the party challenging the transactions
should be required to prove that the transactions were unfair to the corporation or, perhaps, even to prove
that the transactions constituted waste. Approval by fully informed disinterested directors permits
invocation of the business judgment rule and limits judicial review to issues of gift or waste, with the
burden of proof upon the party attacking the transaction. In a Model Act jurisdiction, Major may also
argue that the transaction should be within the statutory “safe harbor” for transactions approved by
disinterested directors and, therefore, immune from attack.

None of these arguments is available to Major because approval by disinterested directors does not trigger
the business judgment rule where the director has not disclosed all material facts about the director’s
interest in the transaction. Required disclosure includes, among other things, all facts known by the
director respecting the subject matter of the transaction that an ordinarily prudent person would
reasonably believe to be material to a judgment about whether or not to proceed with the transaction.
Clearly Major did not make adequate disclosure to the board of directors. In fact, Major misrepresented
the facts by assuring the board that the price being paid for his properties was the fair market value.
Certainly an ordinarily prudent board member would find the fact that the price is vastly more than
market price is material to a judgment about whether to proceed with a transaction.

In sum, Major is not protected from liability by the approval of the transactions by a disinterested board of
directors because Major failed to make full disclosure to the directors.

Explanation to Point-Four: (05-15%):

The directors (other than Major) will be protected from liability by the exculpatory provision in the
articles of incorporation.

A corporation’s articles of incorporation may include a provision shielding its directors from liability for
money damages for the failure to exercise adequate care in the performance of their duties as directors.
Delaware, for example, permits provisions that protect directors from liability for breaches of the duty of
care, but does not permit provisions that protect directors from liability for any breach of the duty of
loyalty, for acts or omissions that are not in good faith, or for any transactions from which the director
received an improper personal benefit. Similarly, the Model Business Corporation Act permits provisions
that protect directors from liability for breaches of the duty of care, but does not permit provisions that
limit or eliminate liability “for the amount of a financial benefit received by a director to which he is not
entitled.”

Seperac-J19 Exam-Released MEE Essay Compilation © 2016-2020 774


Here, there is no suggestion that the directors (other than Major) received an improper personal benefit.
The directors (other than Major) do not appear to have breached the duty of loyalty. They have not
engaged in self-dealing and the facts do not suggest they usurped any corporate opportunity or received
excessive compensation. Although their actions in taking Major’s assertions at face value appear very
foolish, negligence does not establish bad faith.

In sum, the directors (other than Major) are protected from liability for money damages for breach of the
duty of care by the exculpatory provision in the articles of incorporation.

[NOTE: An applicant may argue that the directors (other than Major) acted in bad faith because they
failed to take any meaningful action to determine the facts underlying the transactions. If that inaction is
found to constitute gross negligence, it may constitute an action “not in good faith” and so subject the
directors (other than Major) to liability. Applicants who raise this argument should receive extra credit.]

Explanation to Point-Five (05-15%):

Major will not be protected from liability by the exculpatory provision in the articles of incorporation
because Major breached his duty of loyalty and received an improper personal benefit from Corporation.

Major will not be protected from liability by the exculpatory provision in the articles because that
provision does not protect directors for actions undertaken in bad faith or for a breach of the duty of
loyalty, such as the receipt of an improper personal benefit. Major acted in bad faith by intentionally
misrepresenting to the board that the purchase prices represented fair market value when they in fact
reflected vastly inflated prices. Major’s conduct probably violated the duty of loyalty since he had a
personal interest in the transactions, they were not properly approved, and they probably were not fair to
the corporation. Major received an improper personal benefit by getting proceeds for his property at
inflated prices. For these reasons, the exculpatory provision will not protect Major from liability.

Seperac-J19 Exam-Released MEE Essay Compilation © 2016-2020 775


#175-FEB 2008–MEE Q05: QUESTION FIVE (CORPS-LLCS)

Cal is the CEO and chairman of the 12-member board of directors of Prime, Inc. (Prime). Three other
members of Prime’s board of directors (the Board) are also senior officers of Prime. The remaining eight
members of the Board are wholly independent directors.

Recently, the Board decided to hire a consulting firm to help Prime market a new product. The Board met
to consider whether to hire Wiseman Consulting (Wiseman) or Smart Group (Smart). The Board first
heard from a representative of Wiseman. The Wiseman representative described some of the projects
Wiseman had completed for other clients and outlined the work it proposed to do for Prime for $500,000.
The Board then heard from a representative of Smart, another consulting firm. The Smart representative
described a similar work plan and stated that Smart’s proposed fee was $650,000. Either of these amounts
would be a significant outlay for Prime.

After the Board heard both presentations, Cal disclosed to the Board that he had a 25% partnership
interest in Smart. Cal stated that he would not be involved in any work to be performed by Smart for
Prime. He knew but did not disclose to the Board that Smart’s proposed fee for this consulting assignment
was substantially higher than it normally charged for comparable work. The Board did not ask about the
basis for Smart’s proposed fee.

After receiving all of this information, and no other information, the Board discussed the relative merits of
the two proposals for 10 minutes. The Board then voted unanimously (Cal abstaining) to hire Smart, even
though hiring Smart would cost Prime approximately 30% more than hiring Wiseman. Cal was present
throughout the meeting but did not participate except to the extent indicated above.

1. Did Cal violate his duty of loyalty to Prime? Explain.

2. Assuming Cal breached his duty of loyalty to Prime, does he have any defense to liability?
Explain.

3. Did the directors of Prime, other than Cal, violate their duty of care? Explain.

Seperac-J19 Exam-Released MEE Essay Compilation © 2016-2020 776


#175: F08-5 MEE: ANSWER: NCBE (CORPS-LLCS)

POINT (1) [11%] ISSUE: Did Cal violate his duty of loyalty to Prime by failing to make full
disclosure of all relevant information to the Board about a partnership in which he has an interest
and with which Prime is considering entering into a transaction? ANSWER: Yes. Cal violated the
fiduciary duty of loyalty by allowing Prime to contract with a partnership in which Cal is a partner
without fully disclosing all relevant information.

POINT (2) [47%] ISSUE: Assuming that Cal violated the duty of loyalty, does he have any defenses
to avoid liability? ANSWER: Yes. The RMBCA provides three safe harbors for a director who
breaches his duty of loyalty: approval by disinterested directors, approval by disinterested
shareholders, or fairness. It is unlikely that any of these safe harbors will protect Cal.

POINT (3) [42%] ISSUE: Did the members of Prime’s board of directors, other than Cal, violate
their duty of care by approving a more expensive contract without full information? ANSWER:
Yes. Prime’s disinterested directors likely breached their duty of care when they approved, with
minimal discussion and without full information, a contract in which a director had an interest.

ANSWER DISCUSSION:

Cal, the CEO and chairman of the board of Prime, owes a fiduciary duty of loyalty to Prime. Even though
Cal did not participate in the Board’s discussion of the consulting project and even though the contract
with Smart was approved by a majority of disinterested directors, Cal breached his duty of loyalty by
failing to disclose the substantially higher fee charged to Prime by Smart, a partnership in which Cal has a
financial interest. Here, a court would be unlikely to find that the contract was objectively fair to Prime,
given Smart’s substantially higher fee. Therefore, Cal has no defense to a claim of breach of his duty of
loyalty. The Board may have violated its fiduciary duty of care when it approved the contract with Smart
after only a 10-minute discussion and without seeking or receiving full information about the fees to be
charged.

ANSWER EXPLANATION:

Explanation to Point-One: (05-15%):

Cal violated the fiduciary duty of loyalty by allowing Prime to contract with a partnership in which Cal is
a partner without fully disclosing all relevant information.

Cal is the CEO and chairman of the board of Prime and therefore has a fiduciary duty of loyalty to Prime.
Here, he may have breached that duty because he stands to benefit personally if Smart is hired, since he is
a partner in Smart. As a partner, he is presumably entitled to a share of any profits derived from Smart’s
work for Prime, even if he himself performs no services.

Explanation to Point-Two (40-50%):

The RMBCA provides three safe harbors for a director who breaches his duty of loyalty: approval by
disinterested directors, approval by disinterested shareholders, or fairness. It is unlikely that any of these
safe harbors will protect Cal.

Seperac-J19 Exam-Released MEE Essay Compilation © 2016-2020 777


The Revised Model Business Corporation Act (RMBCA) includes three safe harbors that may protect a
director who breaches his duty of loyalty. However, it is unlikely that any of these safe harbors will
protect Cal under the facts of this case.

Under the first RMBCA safe harbor, approval by disinterested directors insulates a director’s conflicting
interest transaction from judicial scrutiny if the interested director disclosed his or her interest and “played
no part, directly or indirectly, in the disinterested directors’ deliberations or vote.” The statute also
requires disclosure of any information in Cal’s possession that “an ordinarily prudent person would
believe to be material to a judgment about whether or not to proceed with the transaction.”

Applying the first prong of this test, it appears that the Prime board, except for Cal, was disinterested
when it approved the Smart consulting contract. As far as disinterested approval is concerned, the entire
Prime board, with Cal abstaining, approved the Smart consulting contract. The facts do not suggest that
any of the directors besides Cal had a personal interest in this matter. Nothing indicates that the other
directors are subject to Cal’s control or serve at his pleasure. Three board members are also senior officers
of Prime and therefore arguably dependent on Cal’s continued goodwill, but the remaining eight are
outside directors with no other connections to the corporation. Accordingly, even if there is some possible
doubt as to the independence of the three inside or management directors, there is no doubt as to the
independence of the other eight, all of whom voted in favor of the contract. Thus, the disinterested
directors approved the transaction and there is no evidence that Cal played any part in the deliberations or
vote.

However, applying the second prong of this test, Cal did not make adequate disclosure. Cal informed the
Board of his status as a partner in Smart before the Board’s approval and therefore disclosed the fact of
his conflict. However, he did not inform them that the fee to be charged by Smart was substantially higher
than would ordinarily be expected for work like this. This is material information a prudent director
would want to know. Therefore, because of the nondisclosure, it is unlikely that the contract with Smart
can be defended on the basis of the disinterested directors’ approval.

Under the second RMBCA safe harbor, approval by disinterested shareholders would insulate a director’s
conflicting interest transaction from judicial scrutiny. No facts support that defense.

Under the third RMBCA safe harbor, a duty of loyalty claim could be rebutted by proof that “the
transaction, judged according to the circumstances at the time of commitment, is established to have been
fair to the corporation.” Although there is some room to argue this point, the fact that Smart’s proposed
fee is substantially higher than what it normally charges for comparable work suggests strongly that a
finding of objective fairness could not be supported.

Explanation to Point-Three (35-45%):

Prime’s disinterested directors likely breached their duty of care when they approved, with minimal
discussion and without full information, a contract in which a director had an interest.

If the directors do not qualify for the protection of the business judgment rule, they likely will be found to
have breached their duty of care in approving the contract with Smart. The business judgment rule creates
a presumption that “in making a business decision, the directors acted on an informed basis, in good faith
and in the honest belief that the action taken was in the best interests of the company.”

Seperac-J19 Exam-Released MEE Essay Compilation © 2016-2020 778


In this case, there is no evidence of bad faith. At least eight directors were entirely disinterested. Nor is
there any evidence that the directors did not have an honest belief that they were acting in the best
interests of Prime.

However, a court could find that the directors did not act on an informed basis in reviewing the two
alternatives. Because the business judgment rule presumes that the Board was adequately informed, a
party claiming that the directors breached their duty of care has the burden of showing that the Board did
not have sufficient information to justify going with the higher-priced firm. The fact that Smart was going
to charge Prime more than its customary rate suggests that the rate was not justified by added value.
Arguably, the $150,000 price difference required the Board to at least ask more about Smart’s rates.
Moreover, if the Board had discussed the contract for more than 10 minutes, it might have realized that it
lacked full information.

Therefore, it is likely that a court would find that the Board violated its fiduciary duty of care by
approving the contract with Smart.

Seperac-J19 Exam-Released MEE Essay Compilation © 2016-2020 779


#176-JUL 2007–MEE Q09: QUESTION NINE (CORPS-LLCS)

Last July Art, Brett, and Chad formed LeaseAll, Limited Liability Company (LLC), to lease personal
property to individuals and businesses. Art, Brett, and Chad had equal ownership interests in LLC and
entered into a written operating agreement (OA). Under the OA, only Art had authority to manage the
business, to hire and fire employees, and to buy and sell real and personal property. Art contributed a
business plan and his expertise to the leasing business, and Brett and Chad each contributed $50,000 to
the capital of LLC.

Over the next year, Brett and Chad did not participate in the business. No meetings were held, and Art did
not provide Brett or Chad with any information about LLC. In accordance with the business plan, Art
purchased, in the name of LLC, a building and inventory for the leasing business.

Things have not gone well for LLC. Its initial capital is exhausted, and the cash generated by operations is
inadequate to allow it to pay its debts as they come due. Additionally, one of LLC’s customers, Peter, was
badly injured when a chainsaw he rented from LLC malfunctioned. Peter sued LLC and obtained a
judgment of $500,000. LLC does not have liability insurance because Art forgot to sign the check when
he sent the premium payment to the insurance company and, as a result, the company did not issue the
policy. LLC cannot pay Peter’s judgment from its current capital.

1. If Brett and Chad bring an action against Art to recover damages claiming that Art mismanaged
LLC, should that action be direct or derivative and what corporate law requirements must they
meet before bringing an action? Explain.

2. Did Art breach his fiduciary duty in managing LLC? Explain.

3. Can Peter recover the $500,000 judgment against LLC from Art, Brett, and/or Chad personally?
Explain.

Seperac-J19 Exam-Released MEE Essay Compilation © 2016-2020 780


#176: J07-9 MEE: ANSWER: NCBE (CORPS-LLCS)

POINT (1)(a) [20%] ISSUE: Can the members of an LLC maintain a direct action against the
manager of the LLC for mismanagement of the LLC? ANSWER: No. Members of an LLC cannot
maintain a direct action against the manager of the LLC for mismanagement of the LLC.

POINT (1)(b) [20%] ISSUE: Can the members of an LLC maintain a derivative action against the
manager of the LLC for mismanagement of the LLC? ANSWER: Yes. Members of an LLC can
maintain a derivative action against the manager of the LLC for mismanagement of the LLC.
Normally, members must make a demand on the manager to bring the action. Here, however,
demand will likely be excused.

POINT (2) [25%] ISSUE: Were Art’s actions a breach of his fiduciary duty of care as the manager
of LLC? ANSWER: No. Simple negligence does not constitute a violation of a manager’s fiduciary
duty of care to the LLC.

POINT (3) [35%] ISSUE: Can Peter pierce the veil of limited liability of LLC and hold any or all of
its members liable for LLC’s debt? ANSWER: No. To pierce the veil of an LLC and hold the
members personally liable, a claimant must demonstrate either that the LLC was a mere
instrumentality of the individual defendants or that there was a unity of interest and ownership
between the LLC and the individual defendants. Here, it is unlikely that Peter can pierce the veil of
LLC.

ANSWER DISCUSSION:

If Brett and Chad bring an action against Art for mismanagement of LLC, that action must be a derivative
(not direct) action because the harm Brett and Chad suffered was derivative of the injury Art caused LLC.
To bring a derivative action, Brett and Chad must comply with the procedural requirements set forth in
the Uniform Limited Liability Company Act (ULLCA). These requirements typically mandate a demand
upon the managing partner before initiating an action. However, here demand is likely to be excused
because Art is unlikely to bring an action against himself. If Brett and Chad bring a derivative action, they
are likely to lose. Under the ULLCA, Art would be liable for a breach of the duty of care only if he acted
in a grossly negligent or reckless manner. In jurisdictions that have not adopted the ULLCA, Art would be
protected by the business judgment rule. Most LLC statutes provide that neither members nor managers
are personally liable for the debts of an LLC. Courts generally recognize the equitable concept of piercing
the veil of limited liability of an LLC. However, the facts in this case do not seem to provide a basis for
piercing, and Peter probably cannot recover from Art, Brett, or Chad.

ANSWER EXPLANATION:

Explanation to Point-One(a) (15-25%):

Members of an LLC cannot maintain a direct action against the manager of the LLC for mismanagement
of the LLC.

Members of a manager-managed LLC do not have the right to maintain a direct action against the
manager of the LLC when the alleged misconduct caused harm only to the LLC. The overwhelming
majority rule that a claim for an injury to a corporate entity against a manager or officer must be brought

Seperac-J19 Exam-Released MEE Essay Compilation © 2016-2020 781


derivatively. The operating agreement provides that Art manages LLC. Therefore, this is a manager-
managed LLC. It does not matter that Art is also a member of LLC.

Here, the harms that Art caused – failing to manage the business profitably and failing to insure the
business properly – were harms to the business and reputation of LLC, not harms directly to Brett and
Chad. Because the harms were derivative, not direct, Brett and Chad may not bring a direct action against
Art.

[NOTE: If Art breached a contractual obligation to Brett and Chad, they could maintain a direct action
against Art. ULLCA § 410 permits a member to sue the LLC or another member to enforce the member’s
rights under the operating agreement. The facts do not suggest that any violation of the operating
agreement occurred.]

Explanation to Point-One(b) (15-25%):

Members of an LLC can maintain a derivative action against the manager of the LLC for mismanagement
of the LLC. Normally, members must make a demand on the manager to bring the action. Here, however,
demand will likely be excused.

Members of an LLC can maintain a derivative action against the manager of the LLC for mismanagement
of the LLC. In order to maintain such an action members must normally first make a demand that the
manager bring the action and allege in their complaint the efforts that they made to “secure initiation of
the action by the manager or the reasons for not making the effort.” Here, however, it is likely that
demand would be excused, as Art would have to agree to bring an action against himself. Obviously, this
demand would be futile. Therefore, Brett and Chad probably could bring a derivative action without first
making a demand on Art, although that action would likely fail.

Explanation to Point-Two (20-30%):

Simple negligence does not constitute a violation of a manager’s fiduciary duty of care to the LLC.

Under the ULLCA, managers (or members in a member-managed LLC) owe a duty of care to the LLC,
but they are not liable for simple negligence. The ULLCA provides that the duty of care consists of
“refraining from engaging in grossly negligent conduct or reckless conduct, intentional misconduct, or a
knowing violation of law.” Under this standard, the question would be whether Art’s failure to manage
LLC profitably and to insure the business rose to the level of gross negligence or reckless conduct.

Some state statutes reject the gross negligence standard of the ULLCA and impose an ordinary negligence
standard for evaluating breaches of an LLC manager’s duty of care. Where that is the case, however, the
business judgment rule may be applied to protect LLC managers from liability for business decisions
made in good faith. The Revised Uniform Limited Liability Company Act adopts an ordinary negligence
standard but makes an LLC manager’s liability for breach of the duty of care subject to the business
judgment rule.

Although the precise operation and content of the business judgment rule will vary from state to state, its
impact is to shield managers from liability for business decisions. Under this rule, a court will not second-
guess a manager’s decision, even if the decision was a bad one, if the manager acted “on an informed
basis, in good faith, and in the honest belief that the action taken was in the best interests of the
company.”

Seperac-J19 Exam-Released MEE Essay Compilation © 2016-2020 782


Under any of these tests, Art is unlikely to be held liable for the fact that LLC’s business failed. The mere
fact that LLC was unprofitable does not indicate that Art violated his duty of care. Many businesses
operate unprofitably, and no facts suggest that Art was either grossly negligent (as required by the
ULLCA) or even negligent in his general operation of LLC’s business. In addition, in those jurisdictions
that apply a negligence standard, Art would also have the protection of the business judgment rule, and
there are no facts to suggest that Art behaved in a way that would take his decisions outside the scope of
that rule.

On the other hand, Art is arguably liable for his failure to insure LLC. His failure to sign the insurance
premium payment check was clearly negligent and could be characterized as grossly negligent. What’s
more, in a jurisdiction that imposed an ordinary negligence standard, the business judgment rule might not
protect Art from liability for this sort of breach of the duty of care. Generally, the business judgment rule
protects business managers from liability for their good faith business decisions; it does not protect them
from liability for negligent failures to perform ministerial acts such as signing corporate checks.

[NOTE: An applicant may note that Art is subject to a duty to act in good faith. The ULLCA includes an
obligation of good faith and fair dealing among the statutory duties of a manager. The facts, however,
suggest that Art did act in good faith; thus, Brett and Chad could not maintain a claim for a breach of this
duty.]

Explanation to Point-Three (30-40%):

To pierce the veil of an LLC and hold the members personally liable, a claimant must demonstrate either
that the LLC was a mere instrumentality of the individual defendants or that there was a unity of interest
and ownership between the LLC and the individual defendants. Here, it is unlikely that Peter can pierce
the veil of LLC.

Ordinarily, members of an LLC are not liable for the debts, obligations, or liabilities of an LLC “solely by
reason of being or acting as a member.” However, either by statute, or by judicial decision, most states to
consider the issue have recognized the possibility of piercing the veil of limited liability of an LLC, under
appropriate circumstances.

Courts that pierce the veil rely on various theories to do so. Two of the most prominent are the “mere
instrumentality” and “unity of interest and ownership” theories.

Under the “mere instrumentality” test, Peter would have to show that (1) the members dominated the
entity in such a way that LLC had no will of its own, (2) the members used that domination to commit a
fraud or wrong, and (3) the control and wrongful action proximately caused the injury complained of.

Under the “unity of interest and ownership” test, Peter must demonstrate that there was such a unity of
interest and ownership between the entity and the members that, in fact, LLC did not have an existence
independent of the members and that failure to pierce through to the members would be unjust or
inequitable.

Whereas a failure to follow formalities in the operation of the corporation is a factor in corporate veil
piercing cases, it is less frequently relevant in LLC cases. This is because many LLC statutes, including
the Uniform Limited Liability Company Act, provide explicitly that “the failure of a limited liability
company to observe the usual company formalities is not a ground for imposing personal liability on the
members or managers.” However, in the absence of a statutory bar, some courts will cite a failure to
follow formalities in the LLC context as part of the rationale for piercing the LLC veil.

Seperac-J19 Exam-Released MEE Essay Compilation © 2016-2020 783


The decision to pierce the LLC veil is highly fact dependent. The factors that are most likely to lead to an
affirmative decision to pierce the veil are facts suggesting that the LLC was simply the “alter ego” of the
owners, evidence of serious undercapitalization of the company, or a lack of substantive separation
between the business of the company and the personal business of the owners.

Under any of these tests, it seems probable that neither Brett nor Chad could be held liable for the
liabilities of LLC as they had no authority to act on behalf of LLC. As to the “mere instrumentality” test,
neither Brett nor Chad dominated LLC in a way that it became a “mere instrumentality.” As to the “unity
of interest” test, there was not, in any sense, a unity of interest and ownership between Brett and Chad and
LLC.

The case against Art may appear stronger, because Art may have dominated LLC, but it is still not likely
that Peter could prevail. Under the mere instrumentality test, even if Peter could show that Art dominated
LLC, it is not likely that Peter would be able to prove that Art used his domination to commit a fraud or
wrong. Although Art sent in the insurance premium without signing the check, this negligence is not
sufficiently wrongful to justify piercing the limited liability shield. Under the unity test, Peter could not
show that Art had a “unity of interest in ownership with LLC” because Chad and Brett had substantial
interests in LLC. Although it might be argued that LLC was undercapitalized because it could not meet its
obligations, its initial capitalization was substantial ($100,000), and there are no facts to suggest that
amount would not have been sufficient capitalization for a business of this sort had it been run more
effectively (and had Art purchased insurance as intended).

Seperac-J19 Exam-Released MEE Essay Compilation © 2016-2020 784


#177-JUL 2006–MEE Q05: QUESTION FIVE (CORPS-LLCS)

Until early this year, Parensco, Inc. (Parensco) owned 75% of the outstanding shares of Subco Corp.
(Subco). The remaining 25% of the outstanding shares of Subco were held by numerous other
shareholders. The president of Parensco is Carr, who is also a director of Parensco.

Several months ago, the president of Aster, Inc. (Aster) approached Carr, expressed an interest in
acquiring control of Subco, and stated that Aster would consider a purchase price in the range of $200 per
Subco share. After Carr’s conversation with Aster, the board of directors of Parensco decided to merge
Subco into Parensco. The purchase price for the 25% of the outstanding shares of Subco not owned by
Parensco was set at $120 per share.

After its board duly authorized this transaction, Parensco issued the following press release:

The board of Parensco announced today that it seeks to acquire the 25% of Subco that it does not own.
Subject to the approval of the Subco board, Parensco will pay $120 for each share of Subco in a cash-out
merger, for a total cost to Parensco of $200 million.

Shortly after this press release was issued, the board of directors of Subco (Subco board), consisting
solely of officers and directors of Parensco, but not including Carr, met to consider the merger offer. At
the meeting, the Subco board heard a brief presentation from Carr and reviewed a report from Banker, an
investment banker. Banker’s report advised that, after undertaking a review of Subco, a fair valuation of
Subco was $800 million and the price of $120 per share was generous for a minority interest in the
corporation. Neither the Subco board nor Banker was aware of the discussion between Carr and Aster.
After hearing Carr’s presentation and reviewing Banker’s report, the Subco board voted to approve the
merger after a brief discussion and without any further investigation.

Parensco and Subco then issued a proxy statement to the Subco shareholders, which was complete and
accurate except for its failure to mention the Aster proposal.

The merger was approved by a vote of 90% of the outstanding Subco shares, consisting of the 75% of the
outstanding shares held by Parensco and 15% of the outstanding shares held by minority shareholders.
The merger was then consummated and Parensco sent checks to the Subco shareholders in payment for
their stock. A few months later, Parensco announced the sale for $3 billion of the division of its business
consisting almost exclusively of assets acquired in the Subco merger.

1. Did Parensco breach any of its fiduciary duties by failing to disclose the Aster proposal to the
minority shareholders of Subco? Explain.

2. Did the Subco board breach its duty of care in approving the merger? Explain.

3. Did Parensco breach any duty to the minority shareholders of Subco by offering them only $120
per share? Explain.

Seperac-J19 Exam-Released MEE Essay Compilation © 2016-2020 785


#177: J06-5 MEE: ANSWER: NCBE (CORPS-LLCS)

POINT (1) [51%] ISSUE: Did Parensco breach any of its fiduciary duties by failing to disclose the
Aster proposal to the minority shareholders of Subco? ANSWER: Yes. Parensco probably
breached its fiduciary duties to the minority shareholders of Subco by failing to disclose the Aster
proposal.

POINT (2) [28%] ISSUE: Did the Subco board breach its duty of care to the Subco shareholders in
approving the merger? ANSWER: Yes. The Subco board appears to have breached its duty of care
to the Subco shareholders in approving the merger.

POINT (3) [21%] ISSUE: Did Parensco breach any duty owed to the minority shareholders of
Subco by offering to pay them only $120 per share? ANSWER: Yes. Parensco probably breached
its fiduciary duty of fair dealing when it offered to pay the minority shareholders of Subco only
$120 per share.

ANSWER DISCUSSION:

Parensco probably did breach its fiduciary duties by failing to disclose the Aster proposal to the minority
shareholders of Subco. A majority shareholder has a duty to disclose information if it is material and if
non-disclosure will cause a loss to minority shareholders. The Subco board probably did breach its duty of
care by approving the merger after only brief discussion. Parensco probably did breach its fiduciary duty
of fair dealing because it did not offer a fair price and did not engage in a fair process when it failed to
make full disclosure of the Aster proposal.

ANSWER EXPLANATION:

Explanation to Point-One (45-55%):

Parensco probably breached its fiduciary duties to the minority shareholders of Subco by failing to
disclose the Aster proposal.

As majority shareholder, Parensco has a duty to disclose any information that it knew or should have
known if that information was material. Here, Parensco will be found to have breached this duty to
disclose if the information regarding the Aster proposal was material, and the nondisclosure caused a loss
to the minority shareholders.

As to the materiality of the nondisclosure, the minority shareholders would argue that the information
regarding Aster went to the fairness of the price that Parensco was offering and, therefore, a reasonable
shareholder would consider it important in deciding how to vote on the transaction. Although the $200 per
share price arose in the context of a preliminary conversation, and may not have been a firm offer, the
shareholders would argue that they are capable of assessing the importance of the information, and they
can take into account that any further discussions regarding the proposal might have broken down or the
deal might not have been consummated.

Parensco will counter that the price offered by Aster was immaterial for two reasons. First, the price
offered by Aster was preliminary in nature and there is no obligation to disclose a preliminary
conversation. Second, the price that Aster might have offered for control of Subco bears little relationship

Seperac-J19 Exam-Released MEE Essay Compilation © 2016-2020 786


to a fair price for a minority interest, which is what Parensco was offering to acquire in the merger. Aster
was negotiating to buy control of Subco, something that the minority shareholders could not sell. The
question of materiality is a close one, but on balance it appears that the information was material.

Assuming that the information was material, the next question is whether its nondisclosure caused any
loss to the minority shareholders. Parensco would argue that the nondisclosure caused no loss; it had
sufficient votes to approve the transaction without any of the minority shareholders’ votes. The minority
will counter that, although the merger could have been approved without their vote, had they had the
disclosure, the minority shareholders could have exercised their rights under the state appraisal statute.
Minority shareholders need only demonstrate that nondisclosure deprived them of a state remedy that
would otherwise have been available. As the merger has now been consummated, this remedy is
foreclosed. Thus, they would argue that the nondisclosure caused them loss.

Explanation to Point-Two (25-30%):

The Subco board appears to have breached its duty of care to the Subco shareholders in approving the
merger.

Ordinarily the Subco board’s action would be protected by the business judgment rule. Here, however,
even though the facts are not extensive on this point, they do suggest that the Subco board was only
minimally involved in the approval of the merger, and failed to exercise due care. The Subco board relied
solely on a report of an investment banker and a presentation from Carr, the president of Parensco, and
voted to approve the deal after a “brief discussion.” The nature of the transaction, resulting in a cash-out
of the minority shareholders, called for high degree of care by the directors. They apparently failed to
undertake much of an investigation. While their reliance on Banker’s report appears to be justified, it
might not be enough to discharge their duty of care.

The Subco board might argue that the approval by the minority shareholders “cured” any breach of the
duty of care and that since this approval was by a majority of the minority shareholders, the burden of
proof shifts to the complaining minority shareholders to prove that the merger was unfair. In fact,
however, the approval by the minority shareholders is without legal effect because there was not full
disclosure, as noted above.

Explanation to Point-Three (15-25%):

Parensco probably breached its fiduciary duty of fair dealing when it offered to pay the minority
shareholders of Subco only $120 per share.

When a majority shareholder purchases the interest of the minority, it has a duty of fair dealing. The
transaction is reviewable to determine whether the majority shareholder discharged this duty. Under the
facts of this case, it is doubtful that Parensco did. Parensco bears the burden of demonstrating that the
process it employed was fair and that the price that it selected was fair. As to the former, the discussion
above in Points One and Two demonstrates that Parensco failed to engage in a fair process. As to the
latter, the negotiations with Aster and the subsequent sale of Subco assets for $3 billion, not $800 million,
so soon after the acquisition of the minority shares for only $120 each, suggest that the minority shares of
Subco were worth more than Parensco was offering in the merger. As a result, Parensco probably
breached its fiduciary duty of fair dealing.

Seperac-J19 Exam-Released MEE Essay Compilation © 2016-2020 787


#178-FEB 2006–MEE Q05: QUESTION FIVE (CORPS-LLCS)

Green Corporation (Green) was properly incorporated in State A. Green’s articles of incorporation
authorize 100 shares of common stock (Common) and 300 shares of Class A non-voting cumulative
preferred stock (Class A Preferred). Each Class A Preferred share is entitled to a quarterly dividend of $1.
Green’s articles provide that, if Green fails to pay this dividend for four consecutive quarters, each Class
A Preferred share becomes entitled to one vote on all matters voted on at shareholders’ meetings until all
arrearages have been paid.

Deb owns all 100 shares of the Common and 100 shares of the Class A Preferred. Ed owns the remaining
200 shares of the Class A Preferred. Neither Deb nor Ed is a director or officer of Green.

Green has been experiencing financial difficulties for some time and has not paid the Class A Preferred
dividends for more than four consecutive quarters. On September 2, Green’s board unanimously adopted
a proposal to dissolve the corporation. The board then called a special meeting of the shareholders to vote
on this proposal. The notice of the special meeting indicated that a special meeting would be held at the
corporation’s principal office on October 15. The notice did not state the purpose of the meeting. The
notice was sent to Deb. She received it on September 10.

On October 13, two days before the special meeting, Deb and Ed met for dinner. At dinner, Deb asked Ed
to serve as her proxy at the special meeting because she could not attend. Ed was surprised because he had
not received any notice of the special meeting from Green. Nonetheless, Ed told Deb that he would attend
the meeting and vote her shares.

On October 15, at breakfast before the meeting began, Ed first learned of the proposed dissolution of
Green. Ed immediately called Deb to inform her of the proposal. Deb directed Ed to vote all of her shares
in favor of the proposal. Ed went to the meeting, voted Deb’s shares in favor and his own shares against
the proposal to dissolve Green.

1. Was the special shareholder meeting called to dissolve Green properly held? Explain.

2. Was the proposal to dissolve Green properly adopted by its shareholders? Explain.

Seperac-J19 Exam-Released MEE Essay Compilation © 2016-2020 788


#178: F06-5 MEE: ANSWER: NCBE (CORPS-LLCS)

POINT (1) [36%] ISSUE: Was the special shareholder meeting to vote on the proposal to dissolve
Green properly called? ANSWER: No. While the board of directors may call a special meeting to
vote on the proposal to dissolve Green, this special meeting was not properly called for two reasons.
First, the notice for the special meeting did not specify the purpose of the meeting. Second, all
shareholders entitled to vote at a special shareholder meeting are entitled to notice of the meeting,
and Ed was not given notice.

POINT (2) [9%] ISSUE: Could the Class A Preferred shares vote at the special shareholder
meeting called to vote on the proposal to dissolve Green? ANSWER: Yes. Ed was entitled to vote
his 200 Class A Preferred shares because under Green’s articles of incorporation these shares were
entitled to voting rights as Green had missed more than four successive dividend payments and
there were arrearages outstanding.

POINT (3) [23%] ISSUE: Did Deb properly appoint Ed as her proxy? ANSWER: No. Ed could not
vote Deb’s shares because Deb did not properly appoint Ed as her proxy.

POINT (4) [32%] ISSUE: If Ed was permitted to represent both his own and Deb’s shares, was
there a quorum present at the special shareholder meeting so that a vote could be taken, and, if so,
what vote was necessary to approve the proposal to dissolve Green? ANSWER: The proposal to
dissolve Green was not properly approved because the meeting was not properly held. Even if the
meeting had been properly held, the votes cast in favor of dissolution were insufficient to adopt the
dissolution proposal because Ed did not have a valid proxy and no majority was cast in favor of the
proposal.

ANSWER DISCUSSION:

Green’s board of directors has the authority to call a special shareholder meeting to vote on the proposal
to dissolve the corporation. The corporation must provide proper notice of the meeting. The notice for this
special meeting was defective for two reasons. First, the notice of the meeting failed to specify the
purpose of the meeting. Second, Ed, a shareholder entitled to vote on the proposal, was not given notice.
Although Ed waived his objection to failure to receive notice when he attended the meeting without
objection, this objection was not waived by Deb because neither she nor her valid proxy attended the
meeting. Thus, the meeting was not properly held. Even if the meeting were deemed to have been
properly held, the proposal failed. At the meeting, Ed properly voted his 200 shares of Class A Preferred
because the articles of incorporation of Green provided that these shares were entitled to voting rights if
the corporation failed to make dividend payments for four successive quarters and had outstanding
arrearages. Ed, however, could not vote Deb’s shares as Deb did not properly appoint Ed as her proxy. A
valid proxy must be evidenced by a signed writing or verifiable electronic transmission. If Ed had been
permitted to represent both his own and Deb’s shares at the meeting, a quorum was present and the vote
could occur. However, the proposal could not be adopted. The vote would be a tie and therefore there
would be no majority (the minimum requirement to approve the action).

ANSWER EXPLANATION:

Explanation to Point-One (35-45%):

Seperac-J19 Exam-Released MEE Essay Compilation © 2016-2020 789


While the board of directors may call a special meeting to vote on the proposal to dissolve Green, this
special meeting was not properly called for two reasons. First, the notice for the special meeting did not
specify the purpose of the meeting. Second, all shareholders entitled to vote at a special shareholder
meeting are entitled to notice of the meeting, and Ed was not given notice.

A proposal to dissolve the corporation adopted by the board of directors must be submitted to a
shareholder vote. For this purpose, the board of directors has the authority to call a special meeting. After
the Green board unanimously agreed to recommend dissolution of the corporation, they sought to call a
special meeting to submit the proposal to a shareholder vote. However, as discussed below, the meeting
was not properly called.

The corporation must provide proper notice of the special meeting. Furthermore, notice of the special
meeting must include a full description of its purpose. The notice providing for this meeting did not
include a statement of purpose. While Deb discovered the purpose of the meeting during Ed’s phone call
to her, she did not waive the notice impropriety because neither she nor her duly appointed proxy attended
the meeting.

In order to properly call a meeting, the corporation also is obligated to notify all shareholders entitled to
vote at the special meeting in a timely manner. Under the RMBCA, the notification must be more than 10
and less than 60 days before the meeting, unless there is a written waiver of notice. Here, notice for this
special meeting was deficient because, although Deb was given timely notice, Ed was not. Ed learned of
the meeting accidentally two days prior to the meeting during his dinner conversation with Deb. However,
Ed, unlike Deb, waived his objection to lack of notice by attending the meeting without objection.

[NOTE: An exceptional answer might add that under the RMBCA, even if the Class A Preferred shares
were not entitled to voting rights at this time, they would still be entitled to notice of this special meeting
because all shareholders are entitled to notice of a meeting to consider fundamental corporate change,
including dissolution.]

Explanation to Point-Two: (05-15%):

Ed was entitled to vote his 200 Class A Preferred shares because under Green’s articles of incorporation
these shares were entitled to voting rights as Green had missed more than four successive dividend
payments and there were arrearages outstanding.

Unless the articles of incorporation provide otherwise, each outstanding share, regardless of class, is
entitled to one vote on each matter voted on at a shareholders’ meeting. Green’s articles of incorporation
provided that the Class A Preferred shares were non-voting shares unless certain conditions were fulfilled.
Therefore, absent the occurrence of those certain conditions, the Class A Preferred shares were not
entitled to vote on a proposed dissolution, though in an RMBCA jurisdiction they were entitled to notice
of the vote.

Under these facts, Ed was entitled to vote his 200 Class A Preferred shares because the conditions
specified in the articles of incorporation had occurred. Green had missed four successive dividends and
there were arrearages outstanding.

Explanation to Point-Three (20-30%):

Ed could not vote Deb’s shares because Deb did not properly appoint Ed as her proxy.

Seperac-J19 Exam-Released MEE Essay Compilation © 2016-2020 790


A shareholder may vote in person or by proxy. Therefore, Deb was entitled to appoint Ed as her proxy to
vote her shares at the special meeting when she was unable to attend. However, an oral proxy
appointment is not sufficient. To appoint a valid proxy, Deb must sign an appointment form or make a
verifiable electronic transmission. In this case, Deb orally asked Ed to serve as her proxy. This
appointment was insufficient.

Explanation to Point-Four (30-40%):

The proposal to dissolve Green was not properly approved because the meeting was not properly held.
Even if the meeting had been properly held, the votes cast in favor of dissolution were insufficient to
adopt the dissolution proposal because Ed did not have a valid proxy and no majority was cast in favor of
the proposal.

First, as noted above, the special meeting was not properly held as it was not called in the proper manner,
and the impropriety was not effectively waived. Furthermore, even if the meeting had been properly
called, there was no quorum. Unless the articles of incorporation provide otherwise, a quorum is usually a
majority of votes entitled to be cast on a matter. As no dividend had been paid in more than four
consecutive quarters, the Class A Preferred shares were entitled to vote. Therefore, there were 400 shares
entitled to vote on the proposal to dissolve Green. Thus, at least 201 votes must be represented in order to
establish a quorum. If Ed had been permitted to represent all of Deb’s and his own shares at the special
meeting, all 400 shares would have been present and the quorum satisfied. However, Ed was not entitled
to vote Deb’s shares. Because only 200 shares were represented at the meeting, no quorum was present,
and the dissolution proposal failed.

Even if the special meeting had been properly held, the votes cast in favor of dissolution were insufficient
to adopt the dissolution proposal because Ed did not have a valid proxy and no majority was cast in favor
of the proposal. The vote required to approve the proposal to dissolve depends on the jurisdiction. Under
the RMBCA, the proposal will be approved if, of the votes represented at a properly held meeting, more
votes are cast in favor of the proposal than against. Under the older Model Business Corporation Act
standard, the proposal to dissolve must be approved by a majority of shareholders entitled to vote. Any
other applicable statute would require an even greater vote in favor of the proposal.

Thus, regardless of the standard applied, the proposal to dissolve Green, even assuming the meeting was
properly held and Ed had properly cast Deb’s votes, would fail because, if Ed had voted Deb’s 200 shares
(100 Common, 100 Class A Preferred) in favor of the proposal and his 200 Class A Preferred shares
against, there would have been a tie vote, and neither a super-majority or even a majority in favor of the
proposal.

Seperac-J19 Exam-Released MEE Essay Compilation © 2016-2020 791


#179-JUL 2005–MEE Q02: QUESTION TWO (CORPS-LLCS)

Peg, an entrepreneur, decided to develop a widget manufacturing business. Widget production requires
the use of a chemical called chromite. Because the market price for chromite has fluctuated between $150
and $200 per ton over the past year, the first thing Peg did was contact numerous chemical companies to
find a long-term supplier for chromite.

On May 20, Peg received a quote from Chem Corp. stating that it could supply Peg’s corporation with all
of its chromite at $145 per ton for the next three years. Chem Corp. indicated that the quote would expire
at the end of the week. Peg informed Chem Corp. that her corporation, to be called “Acme, Inc.,” would
not be formed by that time. Peg further indicated that, “The deal should be between Acme, Inc. and Chem
Corp. If Chem Corp. does not care if I sign for a non-existent corporation, you have a deal.” Chem Corp.
responded: “No problem. We know with whom we are dealing and what the chromite’s for. Do not waste
our time. If you are serious, lock in the deal.”

On May 26, Peg and Chem Corp. signed a three-year requirements contract for Chem Corp. to supply her
soon-to-be-formed corporation with all of its chromite at a price of $145 per ton. Peg signed the contract
“Peg, as agent for Acme, Inc., a corporation to be formed.” The requirements contract was valid under
state law. It contained a valid and substantial liquidated damages clause in the event Acme purchased
chromite from any other supplier.

On August 1, Peg formed Acme, Inc. Peg was a shareholder, but not an officer or a director. Neither the
board of directors nor the officers of Acme, Inc. formally reviewed or approved the contract with Chem
Corp.

On August 15, Chem Corp. made its first delivery of chromite to Acme, Inc. at the contract price of $145
per ton. Vic, an Acme employee, acting pursuant to the direction of Acme’s board of directors, accepted
delivery and approved payment of the invoice after checking the contract and determining that the invoice
agreed with the contract price. Acme, Inc. immediately began widget production.

In September and October, Chem Corp. made deliveries of chromite to Acme and charged Acme
according to the contract. Acme promptly paid each invoice.

During the fall, the market price for chromite fell. By November, the market price for chromite was $100
per ton. The Acme board of directors, seeking a cheaper supply of chromite, entered into an agreement
with Supply, Inc. to provide Acme with its monthly requirements of chromite at $95 per ton.

Acme immediately contacted Chem Corp. and instructed it not to ship Acme any more chromite. When
Chem Corp. demanded payment under the liquidated damages clause, Acme refused.

1. Is Peg personally liable to Chem Corp. under the contract? Explain.

2. Is Acme, Inc. liable to Chem Corp. under the contract? Explain.

Seperac-J19 Exam-Released MEE Essay Compilation © 2016-2020 792


#179: J05-2 MEE: ANSWER: NCBE (CORPS-LLCS)

POINT (1) [50%] ISSUE: Is Peg, as promoter of a corporation to be formed, personally liable on
the contract with Chem Corp.? ANSWER: Yes. Under the contract, Peg, as a promoter, is
personally liable unless the parties, as evidenced by their words and actions, did not intend for Peg
to be liable.

POINT (2) [50%] ISSUE: Is Acme, Inc., liable on the contract with Chem Corp.? ANSWER: Yes.
Acme, Inc., is liable under the contract if it expressly or impliedly adopted it. While there was no
express adoption, by receiving and paying for chromite for three months, Acme, Inc., implicitly
adopted the contract.

ANSWER DISCUSSION:

Peg, as a promoter, is personally liable on the contract unless the parties did not intend for her to be
personally liable. This is a question of fact, which an applicant could argue either way. Acme, Inc., is
liable on the contract with Chem Corp. if it adopted the contract. Although Acme did not expressly adopt
the contract, by accepting delivery of and paying for the chromite, it impliedly adopted the contract with
Chem Corp.

ANSWER EXPLANATION:

Explanation to Point-One (45-55%):

Under the contract, Peg, as a promoter, is personally liable unless the parties, as evidenced by their words
and actions, did not intend for Peg to be liable.

Peg is a “promoter.” A promoter is someone who “forms a corporation and procures for it the rights,
instrumentalities and capital to enable it to conduct its business.”

Generally, a promoter who signs a contract in the name of a proposed corporation is personally liable on
that contract unless the parties agree otherwise. This is true even if the contract does not mention the
promoter’s liability. Subsequent adoption by the corporation does not relieve the promoter of liability
unless the third party and the corporation expressly or impliedly enter into a novation, by which they
agree to relieve the promoter from personal liability on the contract. Therefore, absent a novation, Peg
ordinarily would be personally liable to Chem Corp. for the liquidated damages.

However, if a party who contracts with a promoter knows that the corporation has not yet been formed
and agrees to look only to the corporation for performance, the promoter is not liable. The facts clearly
state that Chem Corp. knew that Acme, Inc. had not yet been formed. The facts, however, do not indicate
that Chem Corp. agreed to look only to the future corporation or that the contract expressly relieved Peg
of liability. Peg signed the contract “as agent for Acme, Inc., a corporation to be formed.” However, Peg
cannot act on behalf of a nonexistent principal. Under the Second Restatement of Agency, one who acts
for a nonexistent principal is himself liable on the contract. Therefore, Peg would be personally liable for
the liquidated damages unless she can show that Chem Corp. did not intend to hold Peg liable. This is a
fact-based analysis, which an applicant could argue either way.

Seperac-J19 Exam-Released MEE Essay Compilation © 2016-2020 793


Chem Corp.’s statement, “No problem. We know with whom we are dealing and what the chromite’s for.
Do not waste our time. If you are serious, lock in the deal,” could be interpreted to support either
argument. It could be interpreted as an indication that Chem Corp. planned only to look to the corporation
for payment, in which case Peg bears no personal liability for the contract. Alternatively, the statement “if
you are serious” could be interpreted to mean “if you, Peg, are willing to be personally liable.”

The fact that Peg signed the contract as an agent for the corporation to be formed is ineffective to
eliminate personal liability, as a matter of agency law. However, it does evidence her belief that she was
not signing in her personal capacity and that Chem Corp. would look to the corporation, not to her, for
performance on the contract.

The facts do not suggest an express novation of the contract. However, some jurisdictions imply novation
upon formation and adoption by the corporation. In such a jurisdiction, Peg might argue that she is not
liable because Acme adopted the contract.

Explanation to Point-Two (45-55%):

Acme, Inc., is liable under the contract if it expressly or impliedly adopted it. While there was no express
adoption, by receiving and paying for chromite for three months, Acme, Inc., implicitly adopted the
contract.

A contract with a promoter is not a contract with the subsequently formed corporation. The newly formed
corporation is not automatically bound by the contracts executed by its promoter prior to formation. The
newly formed corporation is only liable on such a contract if: (1) a statute mandates liability, (2) charter
documents mandate liability, or (3) the contract is adopted or ratified by the corporation.

[NOTE: Although there are technical differences between the terms “adoption” and “ratification,” many
courts use them interchangeably.]

Adoption of a contract can be either express or implied. Express adoption requires the unequivocal action
by someone, with knowledge of all material facts, who would have had the authority to bind the
corporation to this type of contract. The fact that “neither the board of directors nor the officers of Acme,
Inc., formally reviewed or approved the contract with Chem Corp.” indicates that Acme never expressly
adopted the contract.

A corporation can impliedly adopt a contract so long as it is done with knowledge of all material facts by
someone who would have had the authority to bind the corporation to this type of contract. Implied
adoption requires that the corporation receive some benefit or accept some service. This test is clearly
satisfied. Acme received three shipments of chromite at the contract price and paid each invoice promptly.
Because Vic acted pursuant to the direction of Acme’s board of directors, he had the authority to bind the
corporation to this type of contract (raw materials needed for production). Vic approved the invoice after
checking that the invoice price agreed with the contract price. Because Acme received a benefit after Vic
checked the material facts, Acme, Inc., adopted the contract and is liable to pay the liquidated damages.

Seperac-J19 Exam-Released MEE Essay Compilation © 2016-2020 794


#180-FEB 2005–MEE Q01: QUESTION ONE (CORPS-LLCS)

Corp, a corporation validly incorporated in State A, manufactures computer desks and sells them to
furniture stores in State A and several nearby states. The board of directors of Corp consists of three
members. The board of directors appointed Presley as the president of Corp. The bylaws of Corp provide
that “the president, as the chief executive officer of the corporation, shall manage the business of the
corporation and perform such other duties as the board of directors may from time to time direct.”

Furniture Store (FS) owes Corp $11,000 for computer desks it purchased last year. Presley learned that FS
was on the verge of bankruptcy and retained an attorney to file suit against FS to collect payment. The
attorney filed suit as directed.

Presley believes that Corp should pay a dividend to its shareholders. Consequently, Presley sent a letter to
all shareholders declaring a dividend of 10 cents per share to be paid at the end of the month.

Large Corp (Large), a large national furniture chain, contacted Presley and offered to sell its local
manufacturing plant to Corp for what Presley considers a very reasonable price. Further, Large is willing
to accept payment of the purchase price over 10 years. Acquiring this plant would triple Corp’s
manufacturing capacity. Presley signed a purchase agreement for the plant on behalf of Corp without
consulting the board.

1. Did Presley, as president, have the authority to retain an attorney to file suit against FS on behalf
of Corp? Explain.

2. Did Presley, as president, have the authority to declare a dividend payable to Corp’s
shareholders? Explain.

3. Did Presley, as president, have the authority to enter into a purchase agreement with Large to
acquire its local manufacturing plant on behalf of Corp? Explain.

Seperac-J19 Exam-Released MEE Essay Compilation © 2016-2020 795


#180: F05-1 MEE: ANSWER: NCBE (CORPS-LLCS)

POINT (1) [35%] ISSUE: Did Presley, as president of Corp, have authority to retain an attorney to
file suit against FS for payment of $11,000 on behalf of Corp? ANSWER: Yes. As president of
Corp, Presley had authority to retain an attorney to file suit against FS to collect the $11,000 FS
owed Corp because retaining an attorney to collect an account receivable was in the ordinary
course of Corp’s business. Further, as FS was on the verge of bankruptcy, Presley was attempting
to preserve one of Corp’s assets.

POINT (2) [30%] ISSUE: Did Presley, as president of Corp, have authority to declare a dividend
payable to Corp’s shareholders? ANSWER: No. Presley did not have authority to declare a
dividend payable to Corp’s shareholders. Corporate law reserves the authority to declare dividends
to the board of directors.

POINT (3) [35%] ISSUE: Did Presley, as president of Corp, have the authority to enter into a
purchase agreement with Large to acquire its local manufacturing plant for Corp? ANSWER: No.
Presley has no power, as president, to enter into a purchase agreement to acquire Large’s local
manufacturing plant on behalf of Corp because the transaction is extraordinary in nature.

ANSWER DISCUSSION:

According to the bylaws of Corp, Presley, as president, is the CEO and has authority to manage the
business of the corporation. Therefore, Presley had authority to retain an attorney to file suit against FS to
collect the $11,000 FS owed Corp so long as collecting on this type of obligation (an account receivable)
was in the ordinary course of business and the lawsuit was filed to preserve corporate assets. Presley did
not have authority to declare a dividend to Corp’s shareholders. Dividends can be authorized only by the
board of directors. Finally, Presley did not have authority to enter into a purchase agreement with Large to
acquire its local manufacturing plant for Corp. The purchase of an additional manufacturing plant would
be outside the ordinary course of business.

ANSWER EXPLANATION:

Explanation to Point-One (30-40%):

As president of Corp, Presley had authority to retain an attorney to file suit against FS to collect the
$11,000 FS owed Corp because retaining an attorney to collect an account receivable was in the ordinary
course of Corp’s business. Further, as FS was on the verge of bankruptcy, Presley was attempting to
preserve one of Corp’s assets.

Corporations are managed by or under the direction of their boards of directors. The board of directors
generally delegates day-to-day management of the corporation’s business to officers. Traditionally,
officers had only those powers conferred on them by the bylaws or resolutions of the board of directors.
Today, most states presume that the president has the inherent power to act for the corporation so long as
the matter is within the scope of its ordinary business.

Presley was appointed president of Corp by its board of directors. The bylaws provide that “the president,
as the chief executive officer of the corporation, shall manage the business of the corporation and perform
such other duties as the board of directors may from time to time direct.” Pursuant to the bylaws, Presley,

Seperac-J19 Exam-Released MEE Essay Compilation © 2016-2020 796


as chief executive officer responsible for managing the business, had the authority to retain an attorney to
file suit against FS for the following reasons: Corp manufactures computer desks and sells them to
furniture stores like FS. FS purchased computer desks but has not paid for them. Presley was attempting
to collect on an account receivable. Hiring an attorney to bring suit against a retailer for payment is part of
Corp’s ordinary course of business.

Absent a prohibition contained in bylaws or imposed by the board of directors, Presley had authority to
bring suit on behalf of Corp, so long as it was in the ordinary course of business. Absent direct
prohibition, a president has presumptive authority to prosecute suits in the name of the corporation. There
are no facts indicating that the board of directors has withheld authority from Presley to commence
litigation on behalf of Corp.

Some case law indicates that the president may only commence litigation to preserve corporate assets or
prevent dissipation. Even under this test, Presley had authority to sue FS. As FS was on the verge of
bankruptcy, by retaining an attorney to file suit, Presley was attempting to preserve a corporate asset.

Explanation to Point-Two (25-35%):

Presley did not have authority to declare a dividend payable to Corp’s shareholders. Corporate law
reserves the authority to declare dividends to the board of directors.

Although the president of a corporation has the authority to bind the corporation by acts that are in the
ordinary course of business, the president does not have the authority to declare or pay dividends to
shareholders. Corporate law confers this authority only on the directors of the corporation. Consequently,
only the board of directors has the power to declare a dividend. Therefore, Presley, as an officer, did not
have the authority to declare a dividend payable to Corp’s shareholders.

Explanation to Point-Three (30-40%):

Presley has no power, as president, to enter into a purchase agreement to acquire Large’s local
manufacturing plant on behalf of Corp because the transaction is extraordinary in nature.

The president of a corporation has the authority to bind the corporation by acts that are in the ordinary
course of business, but does not have the authority to bind the corporation by extraordinary acts. While
some acts are clearly ordinary or extraordinary, in many cases there is no bright line demarcation. “Any
attempt at precision in drawing this line would almost certainly be futile, because the issue is highly
dependent on the context in which it arises, and the types of business transactions that may arise are
endlessly variable.” Factors that a court might consider in determining whether a transaction is
extraordinary include “the economic magnitude of the action in relation to corporate earnings and assets,
the extent of the risk involved, the time span of the action’s effect, and the cost of reversing the action.”
Other factors include the character of the goods ordered, the amount thereof in relation to the size and
condition of the company, the nature of the company, its purposes and aims.

The facts surrounding Presley’s planned purchase of Large’s local manufacturing plant do not provide
numbers to calculate the economic impact of the purchase on the corporation, but they do suggest that the
impact would be significant. Additionally, while Presley believes the price is “very reasonable,” the
payment process could span 10 years. Finally, it seems likely that it would be costly to reverse the
transaction if it turned out not to be beneficial to Corp.

Seperac-J19 Exam-Released MEE Essay Compilation © 2016-2020 797


Consequently, it is very likely that entering into a purchase agreement to acquire Large’s local
manufacturing plant would be viewed as an extraordinary act. As such, Presley, as president, would not
have actual authority to sign the purchase agreement without the approval of the board of directors.

Further, there are no facts that gave Large a reasonable basis to believe that Presley was authorized to
enter into this purchase agreement. Therefore, Presley also lacked apparent authority to enter into the
purchase agreement.

Seperac-J19 Exam-Released MEE Essay Compilation © 2016-2020 798


#181-JUL 2004–MEE Q02: QUESTION TWO (CORPS-LLCS)

Zeta, Inc., is a corporation with 80,000 shares outstanding. Its articles of incorporation provide for a nine-
person board of directors, with staggered three-year terms. Three directors are elected each year. Zeta’s
articles require cumulative voting. Therefore, when electing the three directors, each share is entitled to
three votes, meaning that there are 240,000 votes eligible to be cast. Accordingly, a person who receives
at least 60,001 votes would be elected a director. Neither the articles nor Zeta’s bylaws contain any other
provisions concerning elections, voting, or removal of directors.

Diane is one of Zeta’s directors. Although Diane’s term does not expire for another two years, at the
request of a group of shareholders, Zeta has scheduled a special shareholders’ meeting on September 1 to
consider removing Diane from office. Proper notice of that meeting has been given to all of Zeta’s
shareholders.

Sam owns 16,000 of the 80,000 outstanding Zeta shares. On August 1, Sam gave Arnie a proxy to vote
Sam’s shares at the special meeting. The proxy signed by Sam stated that it was “irrevocable.”

On August 15, Betty, another Zeta shareholder, convinced Sam that giving a proxy to Arnie was a
mistake. Sam then signed another proxy, dated August 15, which revoked the proxy to Arnie and gave
Betty the right to vote Sam’s 16,000 shares.

At the special shareholders’ meeting on September 1, the shareholders in favor of Diane’s removal argued
that Diane was too critical of the company’s management at board meetings and was “rocking the boat.”
No one alleged any breach of duty or other wrongdoing by Diane. Both Arnie and Betty attempted to vote
Sam’s shares by proxy. Counting Arnie’s vote of Sam’s shares, the result was 117,000 votes to remove
Diane and 123,000 votes not to remove her. Counting Betty’s vote of Sam’s shares, the result was
165,000 votes to remove Diane and 75,000 votes not to remove her.

Zeta’s corporate secretary ruled that Diane was not removed as a director because: (1) no valid cause was
shown for removing Diane and showing cause was required to remove her; (2) Arnie was entitled to vote
Sam’s shares and removal therefore failed by a vote of 117,000 votes to remove Diane and 123,000 votes
not to remove her; and (3) even if Betty had the right to vote Sam’s shares, the resulting vote of 165,000
votes to remove Diane and 75,000 votes not to remove her was insufficient to remove Diane.

Was each of these three rulings correct? Explain.

Seperac-J19 Exam-Released MEE Essay Compilation © 2016-2020 799


#181: J04-2 MEE: ANSWER: NCBE (CORPS-LLCS)

POINT (1) [42%] ISSUE: Must shareholders show cause to remove a director? ANSWER: No.
Shareholders need not show cause to remove a director. Under the common law, shareholders had
the inherent power to remove a director, but only for cause. Cause is defined as substantial
grounds, such as directorial breach of duty or malfeasance in office. “Rocking the boat” and being
critical of management, as alleged here, do not constitute cause for removal.

POINT (2) [42%] ISSUE: May a shareholder revoke a proxy that states it is irrevocable by issuing
a subsequent proxy? ANSWER: Yes. By issuing a subsequent proxy to Betty, Sam revoked the
proxy he issued to Arnie, even though the proxy stated it was irrevocable.

POINT (3) [16%] ISSUE: What vote is required to remove a director when cumulative voting is
authorized by the corporation? ANSWER: Under cumulative voting, a director is not removed if
the number of votes cast against removal would be sufficient to elect the director at an annual
meeting. Here, the number of votes cast against Diane’s removal would have been sufficient to
assure her election. Diane would not be removed under either vote.

ANSWER DISCUSSION:

Zeta’s shareholders may remove Diane without cause so long as they get the appropriate vote. Under
cumulative voting, at least 60,001 votes would be sufficient to elect Diane as a director. Diane cannot be
removed if the votes cast against removal would be sufficient to elect her as a director at an annual
meeting to elect directors. Under either vote, there were sufficient votes cast against removing Diane and
thus, she should not be removed. Sam’s proxy to Arnie was revoked when he issued a subsequent proxy
to Betty. Even though the proxy stated it was irrevocable, words alone are not sufficient to create an
irrevocable proxy. A proxy is only irrevocable if coupled with an interest, and there are no facts to
indicate that this proxy was coupled with an interest.

ANSWER EXPLANATION:

Explanation to Point-One (35-45%):

Shareholders need not show cause to remove a director. Under the common law, shareholders had the
inherent power to remove a director, but only for cause. Cause is defined as substantial grounds, such as
directorial breach of duty or malfeasance in office. “Rocking the boat” and being critical of management,
as alleged here, do not constitute cause for removal.

However, most modern statutes, including the Revised Model Business Corporation Act (RMBCA),
permit shareholders to remove a director without cause. Under the RMBCA, the shareholders may remove
one or more directors with or without cause unless the articles of incorporation provide that directors may
be removed only for cause. As Zeta’s articles of incorporation contain no special provisions concerning
removal of directors, the shareholders may remove Diane without showing cause. Therefore, the first
ruling was incorrect.

Explanation to Point-Two (35-45%):

Seperac-J19 Exam-Released MEE Essay Compilation © 2016-2020 800


By issuing a subsequent proxy to Betty, Sam revoked the proxy he issued to Arnie, even though the proxy
stated it was irrevocable.

Under the common law and state corporation statutes, a proxy is revocable unless it expressly provides
that it is irrevocable and the appointment of the proxy is “coupled with an interest.” Thus, unless it was
coupled with an interest, Sam could revoke the proxy he gave Arnie even though the proxy states it is
irrevocable.

The concept of “coupled with an interest” is not completely clear, but, in general, the proxy holder must
have “either (1) a charge, lien, or some property right in the shares themselves; or (2) a security interest
given to protect the proxy holder for money advanced or obligations incurred.” The RMBCA specifies
various categories of people who qualify: (1) a pledgee; (2) a person who purchased or agreed to purchase
the shares; (3) a creditor of the corporation who extended it credit under terms requiring the appointment;
(4) an employee of the corporation whose employment contract requires the appointment; or (5) a party to
a voting agreement created under the RMBCA.

Sam’s proxy to Arnie was not coupled with an interest. The facts indicate only that Arnie was Sam’s
friend. Because the proxy to Arnie was not coupled with an interest, it was revocable in spite of the
language to the contrary.

Where two or more revocable proxies have been given, the last given revokes all previous proxies. When
Sam gave Betty a proxy on August 15, he revoked the proxy he had given to Arnie on August 1.
Therefore, the second ruling was incorrect. Betty was entitled to vote Sam’s shares.

Explanation to Point-Three (10-20%):

Under cumulative voting, a director is not removed if the number of votes cast against removal would be
sufficient to elect the director at an annual meeting. Here, the number of votes cast against Diane’s
removal would have been sufficient to assure her election. Diane would not be removed under either vote.

In a corporation with straight voting, a shareholder vote removes a director if the number of votes for
removal exceeds the number of votes against removal. However, this corporation’s articles require
cumulative voting. “If cumulative voting is authorized, a director may not be removed if the number of
votes sufficient to elect him under cumulative voting is voted against his removal.” Diane could have
been elected a director with 60,001 votes, and more votes than that were voted against her removal
(123,000 or 75,000, depending on how the proxies were counted). Therefore, the third ruling was correct.
Diane could not have been removed.

[NOTE: Directors are elected by a plurality of the votes cast, so the directors with the greatest number of
votes are elected. According to Zeta’s articles, the nine directors have staggered terms, and three
directors are elected at each annual meeting. Therefore, under cumulative voting, each share is entitled
to three votes (number of shares x number of directors up for election).

As stated in the facts, a director must receive at least 60,001 votes to assure election under the following
formula: 1+(number of votes eligible to be cast/number of directors to be elected+1)]>

Seperac-J19 Exam-Released MEE Essay Compilation © 2016-2020 801


#182-JUL 2003–MEE Q01: QUESTION ONE (CORPS-LLCS)

Corp owns 95% of the outstanding stock of Sub. Pat and Dale own the remaining 5% of Sub stock, and
they are also two of the three directors of Sub. Both Corp and Sub are properly incorporated and have
only one class of stock outstanding. Corp’s board of directors is composed of five directors; none of them
is Pat or Dale.

Corp’s board of directors voted unanimously, for valid business reasons, to merge Sub into Corp and to
adopt a plan for a cash-out merger. Pursuant to the plan of merger, each Sub shareholder would receive
$20 per share, which is $4 per share above the current book value. No changes to Corp’s articles of
incorporation are required as a result of this merger, and Corp shareholders will not experience a change
in their shareholder rights. Corp did not submit the plan of merger to its shareholders for a vote.

At a regularly scheduled meeting of Sub’s board of directors, Corp announced its intent to merge with
Sub. Sub’s board voted against the merger by a 2 to 1 vote, with Pat and Dale voting against it. Corp
decided to proceed with the merger in spite of the vote against it by Sub’s board and subsequently filed
articles of merger with the Secretary of State. Shortly thereafter, Pat and Dale each received a check for
$20 per share, a copy of the plan of merger, and a notice including all other statutorily required
information.

Pat and Dale sued to unwind the merger. They claim the merger was improper because (a) Sub’s board of
directors did not approve the merger, (b) Corp’s shareholders did not vote on the merger, and (c) Sub’s
shareholders did not vote on the merger.

Further, Pat and Dale claim that, even if the merger stands, they are entitled to receive more than the $20
per share paid by Corp. They claim the true value of their shares is $25 per share. Pat and Dale base this
valuation on their knowledge of the financial condition of Sub, which they acquired while serving as
directors of Sub, and the advice they received from an independent financial advisor. The independent
financial advisor is willing to testify that the value of Sub is between $21 and $26 per share.

1. Should the court unwind the merger for any of the reasons asserted by Pat and Dale? Explain.

2. Assuming that the merger is allowed to stand, do Pat and Dale have any legal basis for asserting
the right to receive more than the $20 per share offered by Corp? Explain.

Seperac-J19 Exam-Released MEE Essay Compilation © 2016-2020 802


#182: J03-1 MEE: ANSWER: NCBE (CORPS-LLCS)

POINT (1)(a) [23%] ISSUE: In a parent-subsidiary merger, does the board of directors of the
subsidiary corporation have to approve the merger? ANSWER: No. The approval of Sub’s board of
directors is not required for this parent-subsidiary merger.

POINT (1)(b) [23%] ISSUE: In a parent-subsidiary merger, do the shareholders of the parent
corporation have to approve the merger? ANSWER: No. Approval by the shareholders of Corp is
not required for this parent-subsidiary merger.

POINT (1)(c) [23%] ISSUE: In a parent-subsidiary merger, do the shareholders of the subsidiary
corporation have to approve the merger? ANSWER: No. The shareholders of Sub are also not
required to approve this parent-subsidiary merger.

POINT (2) [32%] ISSUE: In a parent-subsidiary merger, if the subsidiary’s minority shareholders
believe that the parent corporation is paying less than fair value, can they force the parent
corporation to pay more money? ANSWER: Yes. Pat and Dale are entitled to exercise appraisal
rights and seek the “fair value” of their shares.

ANSWER DISCUSSION:

None of the objections presented by Pat and Dale will suffice for unwinding this merger. Corp’s
ownership of 95 percent of Sub’s shares gives it the power to merge Sub into itself without securing the
approval of Sub’s board of directors and without holding any vote of Corp’s or Sub’s shareholders.
However, Pat and Dale are entitled to receive the fair value of their shares, provided they properly invoke
their appraisal rights.

ANSWER EXPLANATION:

Explanation to Point-One(a) (20-30%):

The approval of Sub’s board of directors is not required for this parent-subsidiary merger.

A merger between two corporations is normally effectuated by adoption of a plan of merger by the board
of directors of both corporations. However, when a parent owns at least 90 percent of the outstanding
shares of each class of stock of a subsidiary, the parent may merge the subsidiary into itself without the
approval of the subsidiary’s board of directors. The rationale for this rule is that the parent owns sufficient
shares of the subsidiary to remove any members of the board of directors voting against the merger and
replace them with members who would vote in favor of the transaction.

The facts of this problem present such a parent-subsidiary situation. Because Corp owns 95 percent of
Sub, approval of the merger by the Sub board of directors is not necessary. Thus, Corp could proceed with
the merger despite the Sub board’s negative vote on the plan of merger, and the negative vote provides no
basis for unwinding the merger.

[NOTE: Some state’s parent-subsidiary merger statutes require that the parent corporation own at least
95 percent (e.g., Arkansas), while other states require only 80 percent (e.g., Alabama).]

Seperac-J19 Exam-Released MEE Essay Compilation © 2016-2020 803


Explanation to Point-One(b) (20-30%):

Approval by the shareholders of Corp is not required for this parent-subsidiary merger.

A plan of merger usually must be submitted to shareholders for their approval. However, the RMBCA
provides that approval by shareholders of the surviving corporation is not required when the merger will
not result in a fundamental change in the corporation or the ownership rights of its shareholders. In
particular, no shareholder approval is required if

(1) the corporation will survive the merger ; (2) its articles of incorporation will not be changed; (3) each
shareholder of the corporation whose shares were outstanding immediately before the effective date of the
merger will hold the same number of shares, with identical preferences, limitations, and relative rights,
immediately after the effective date of change; and (4) the issuance in the merger of shares or other
securities does not require a vote under the RMBCA.

The facts presented satisfy the requirements for this exception insofar as Corp’s shareholders are
concerned. First, Corp will survive the merger because Sub is being merged into it. Second, Corp’s
articles of incorporation will not change as a result of the merger. Finally, no vote to issue shares is
required because this is a cash-out merger (the Sub shareholders are receiving cash, not securities). The
cash-out nature of the merger also means that no Corp shareholder will experience a change in the number
of shares owned or the rights and preferences of those shares. Therefore, the fact that the Corp
shareholders did not vote does not constitute a basis to unwind the merger.

[NOTE: Prior to the RMBCA 1999 revision, approval by Corp’s shareholders would not have been
required simply because this was a short-form merger (parent’s subsidiary). Under the revised act, the
analysis is more elaborate but the result is the same.]

Explanation to Point-One(c) (20-30%):

The shareholders of Sub are also not required to approve this parent-subsidiary merger.

As noted above, shareholder approval of a plan of merger is generally required. However, parent-
subsidiary mergers are an exception to this rule when the parent owns at least 90 percent of the
outstanding shares of each class of the subsidiary.

Shareholder approval is not required in these circumstances because the parent corporation owns a
sufficient number of shares to ensure that the plan of the merger will be adopted. In our problem, because
Corp owns 95 percent of Sub, it would easily prevail in any shareholder vote and, accordingly, the law
does not require a vote by the shareholders of Sub for the merger to proceed. Therefore, the failure of the
Sub shareholders to approve the transaction does not constitute a basis to unwind the merger.

Explanation to Point-Two (30-40%):

Pat and Dale are entitled to exercise appraisal rights and seek the “fair value” of their shares.

Appraisal rights (also referred to as dissenter’s rights) allow shareholders to force the corporation to pay
fair value for their shares in the event of certain fundamental changes, including a parent-subsidiary
merger for which shareholder approval is unnecessary. Thus, Pat and Dale, as minority shareholders of
Sub, are entitled to exercise appraisal rights in connection with this merger. As Pat and Dale believe that
each share is worth $5 more than Corp paid them in the merger, they will exercise their appraisal rights.

Seperac-J19 Exam-Released MEE Essay Compilation © 2016-2020 804


Assuming that they follow the procedure established by the applicable statute (including notifying the
corporation of their assertion of appraisal rights and demanding payment), they will be entitled to receive
the fair value of their shares.

If Corp and Pat and Dale cannot agree on fair value, a court will decide the matter. In the court
proceeding, Pat and Dale would offer the testimony of the independent financial advisor, stating the fair
value of Sub shares to be between $21 and $26 per share, to support their contention. They might not get
$25 per share, but based on this independent evaluation, they would probably get more than $20 per share.

Seperac-J19 Exam-Released MEE Essay Compilation © 2016-2020 805


#183-FEB 2003–MEE Q07: QUESTION SEVEN (CORPS-LLCS)

Corn Corp was properly incorporated in 1980 with $500,000 of initial capital. Its shares are owned
equally by Alan, Bruce, and Kathy, who are also the sole directors and officers. Corn Corp produces and
sells corn under the brand name “Super Corn.” Until 2000, it was a profitable corporation.

In the late 1990s, Kathy, who has a Ph.D. in genetics and is in charge of product development, was
experimenting with genetically engineered corn. She created a new strain of corn that was more resistant
to disease than regular corn. Corn Corp patented the new strain of corn. In order to be the first to bring
genetically engineered corn to market, Kathy, without the knowledge of Alan and Bruce, negligently
omitted several tests that would normally be applied to a new strain of corn, including tests to establish
whether it would cause allergic reactions.

Alan, president of Corn Corp, was concerned that some food processors might not want genetically
engineered corn. Therefore, on December 31, 1999, Corn Corp properly incorporated Gen-Corn, Inc.
(GCI), a wholly owned subsidiary, to produce and sell the new strain of corn. In exchange for all of GCI’s
stock, Corn Corp contributed to GCI the patent to the new strain of corn and $6,000, just enough to
produce the seed for the first crop. Alan, Bruce, and Kathy were the sole directors and officers of GCI.

In 2000, GCI, operating out of the offices of Corn Corp, started marketing the genetically engineered corn
under the name “Super Corn Plus.” While Kathy scrupulously maintained minute books for Corn Corp,
she kept no minute books for GCI. Corn Corp kept meticulous records of all of its business transactions,
but GCI did not keep separate records. Although each corporation had its own bank account, Corn Corp
often made informal “emergency loans” to GCI to help with cash flow problems. Unfortunately, GCI
never became profitable.

Shortly after the first harvest of “Super Corn Plus,” Stuart, a young business executive, died from an
allergic reaction after eating cornbread made from “Super Corn Plus.” GCI immediately pulled “Super
Corn Plus” from the market and soon after filed for bankruptcy. Because of the similar product names,
Corn Corp’s sales of “Super Corn” plummeted, and Corn Corp can no longer pay its bills.

Assuming Stuart’s estate obtained a wrongful death judgment against GCI, can it recover the judgment
from Corn Corp or any of Corn Corp’s shareholders? Explain.

Seperac-J19 Exam-Released MEE Essay Compilation © 2016-2020 806


#183: F03-7 MEE: ANSWER: NCBE (CORPS-LLCS)

POINT (1) [55%] ISSUE: Can a judgment creditor “pierce the corporate veil” of a wholly owned
subsidiary corporation to recover the creditor’s judgment from the parent corporation
shareholder? ANSWER: Yes. Stuart’s estate will likely be able to pierce GCI’s corporate veil and
recover its judgment from Corn Corp, the “parent” corporation, because GCI failed to maintain a
formal separate existence as evidenced by its inadequate capitalization and failed to maintain
corporate formalities, and because justice requires piercing to prevent Corn Corp from insulating
itself from its wrongdoing.

POINT (2) [45%] ISSUE: Assuming that a judgment creditor can pierce the corporate veil of a
wholly owned subsidiary corporation, can it further pierce the parent corporation’s corporate veil
to recover its judgment from the parent corporation’s shareholders? ANSWER: No. The facts do
not support piercing Corn Corp’s corporate veil to hold Alan, Bruce, and Kathy, as shareholders,
liable for Stuart’s estate’s judgment. Kathy, however, might be liable for her negligent acts.

ANSWER DISCUSSION:

Stuart’s estate will probably be able to pierce the corporate veil to hold Corn Corp liable for the estate’s
judgment against GCI. Whether a court will pierce the corporate veil depends heavily on the facts of a
particular case, and here the facts support holding Corn Corp liable. In the first place, the usual facts
supporting veil-piercing are present here: the business records of Corn Corp and GCI were completely
intermingled, GCI did not observe the formalities of a separate corporate existence, GCI was inadequately
capitalized, and GCI’s business (selling Super Corn Plus) was held out to the public as a mere extension
of Corn Corp’s business. In addition, the activities that led to the judgment against GCI were the result of
decisions made and directed by an officer of Corn Corp at a time when Super Corn Plus was still a Corn
Corp product. It is less likely that a court would pierce the corporate veil of Corn Corp in order to hold its
shareholders liable to pay Stuart’s estate’s claim. There is nothing to suggest that the shareholders used
Corn Corp as a “mere instrumentality” or their “alter ego.” They respected corporate formalities, and
Corn Corp appears to have been adequately capitalized.

ANSWER EXPLANATION:

Explanation to Point-One (50-60%):

Stuart’s estate will likely be able to pierce GCI’s corporate veil and recover its judgment from Corn Corp,
the “parent” corporation, because GCI failed to maintain a formal separate existence as evidenced by its
inadequate capitalization and failed to maintain corporate formalities, and because justice requires
piercing to prevent Corn Corp from insulating itself from its wrongdoing.

Shareholders generally are not liable for the corporation’s debts. A court, however, will “pierce the
corporate veil” when circumstances indicate that the privilege has been misused or when necessary to do
justice. The decision to pierce the corporate veil will be based on a determination of the facts.

While a parent corporation is expected to exert some measure of control over a subsidiary corporation, the
subsidiary corporation is expected to be separate, at least in form. When a parent corporation so
dominates a subsidiary corporation that the subsidiary has no real separate existence, and justice so

Seperac-J19 Exam-Released MEE Essay Compilation © 2016-2020 807


requires, a court will disregard the corporate form and hold the parent corporation personally liable for the
subsidiary’s debts.

One basis for piercing the corporate veil is fraud or illegitimate purpose. In the absence of fraud or
illegitimate purpose, a court may still pierce the corporate veil when justice so requires if (a) the parent
and subsidiary intermingle their respective business transactions, accounts, and records; (b) the subsidiary
fails to observe the formalities of separate corporate procedures; (c) the subsidiary is not adequately
financed in light of its foreseeable normal obligations; or (d) the parent and subsidiary do not hold
themselves out to the public as separate enterprises.

Based on the facts, GCI is a wholly owned subsidiary of Corn Corp. Corn Corp had a legitimate purpose
for organizing GCI. Creating a separate entity for marketing reasons (because some food processors might
not have wanted genetically engineered corn) is an acceptable business reason. Despite this legitimate
purpose, a court might still pierce the corporate veil because GCI did not formally maintain a separate
existence from Corn Corp (internally or externally). GCI did not formally maintain corporate formalities
and was not initially adequately capitalized.

Corn Corp and GCI intermingled their assets and business transactions. GCI operated out of the offices of
Corn Corp and had exactly the same directors and officers. GCI did not keep separate records of its
business transactions. Additionally, while the two corporations each had separate bank accounts, the fact
that GCI received “informal” loans from Corn Corp is further evidence of the blurred boundaries between
the parent and subsidiary.

GCI did not observe corporate formalities. It did not keep separate minute books. Additionally, Corn Corp
did not adequately finance GCI in light of its foreseeable normal obligations. Corn Corp funded GCI with
only the patent, which was new and of speculative value, and $6,000, which was only enough to produce
the seed for the first crop. The inadequate initial capitalization was further evidenced by GCI’s need for
“emergency loans” from Corn Corp to meet day-to-day cash flow needs.

Finally, Corn Corp and GCI marketed products with very similar names (Super Corn and Super Corn
Plus). These similar product names could be reasonably interpreted by the public to be from the same
entity. The decline in sales experienced by Corn Corp after Stuart’s death seems to indicate that the public
viewed Corn Corp as the same entity as GCI. Further, as discussed above, both Corn Corp and GCI
operated from the same location, which could lead the public to conclude that the businesses were not
distinct.

Justice requires piercing because Super Corn Plus was developed by Corn Corp, and an officer and
director of Corn Corp negligently rushed it to market without the proper testing. Corn Corp should not
now be able to use a wholly owned subsidiary that did not maintain a separate existence to insulate itself
from liability.

Explanation to Point-Two (40-50%):

The facts do not support piercing Corn Corp’s corporate veil to hold Alan, Bruce, and Kathy, as
shareholders, liable for Stuart’s estate’s judgment. Kathy, however, might be liable for her negligent acts.

While the facts support piercing the corporate veil to hold Corn Corp liable for the judgment against GCI,
this will not help Stuart’s estate because the facts indicate that Corn Corp cannot pay its bills (so it will be
unable to pay the judgment). Therefore, Stuart’s estate would like to recover the judgment against GCI

Seperac-J19 Exam-Released MEE Essay Compilation © 2016-2020 808


from the shareholders of Corn Corp (Alan, Bruce, and Kathy). However, the facts do not support a claim
to recover from Alan, Bruce, or Kathy in their capacity as shareholders of Corn Corp.

Shareholders generally are not liable for the corporation’s debts. Courts generally respect the corporate
form and shield shareholders from liability unless the shareholders use the corporation as a “mere
instrumentality” or “alter ego” to carry out an improper or illegal purpose (e.g., to perpetrate fraud, to
evade the law, to escape obligations). “Where corporate formalities are substantially observed, initial
financing reasonably adequate, and the corporation not formed to evade an existing obligation or a statute
or to cheat or to defraud, even a controlling shareholder enjoys limited liability.”

There are no facts to support a conclusion of an illegal or improper purpose when Corn Corp was
incorporated. There is also no suggestion that Corn Corp was inadequately capitalized when it was
incorporated. Corn Corp had $500,000 of initial capitalization and had been profitable for 20 years. The
fact that Corn Corp cannot currently pay its bills is irrelevant to this determination.

Further, there is no evidence that the shareholders did not respect the corporate formalities. For example,
Kathy scrupulously kept Corn Corp’s minute books. Nor is there any evidence that Alan, Bruce, or Kathy
ignored the “separateness” of Corn Corp. In summary, there are no facts to indicate that any of the
shareholders of Corn Corp treated Corn Corp as their “alter ego” or a “mere instrumentality,” and the
interests of justice do not require piercing the corporate veil in this case.

Although the facts do not support a holding that Kathy is liable in her capacity as a shareholder of Corn
Corp, Stuart’s estate could bring a separate action against Kathy for her negligent acts as the head of Corn
Corp’s product development.

Seperac-J19 Exam-Released MEE Essay Compilation © 2016-2020 809


#184-FEB 2002–MEE Q07: QUESTION SEVEN (CORPS-LLCS)

The articles of incorporation of Ergo, Inc. authorize the issuance of 400,000 Class A Common Shares and
1,000,000 Class B Common Shares, all of which are issued and outstanding. Dart owns all of the Class A
shares and none of the Class B shares. Ergo’s Articles provide that Ergo has seven directors elected by
straight voting, with Class A shares to elect four directors and Class B shares to elect three directors.

Several months ago, Ergo’s board of directors properly approved an expansion plan for the business that
would require $5 million of additional capital. At their regular February 1 meeting, the directors discussed
possible sources to fund the expansion plan. One Class B director suggested that Ergo borrow the funds
from a bank.

Dart, who had elected herself as one of the Class A directors, suggested that Ergo issue a new class of
shares that Dart would purchase for $5 million. The new class of shares (Class C Preferred) would be
entitled to a cumulative preferred dividend. In support of this alternative, Dart presented an opinion from
an independent investment bank that stated:

(1) $5 million would be a fair value for the Class C Preferred, and

(2) in the long run, payment of the proposed preferred dividend would be less costly to Ergo than interest
payments on a loan.

After one hour of spirited discussion of these alternatives, all seven directors voted to recommend to the
shareholders that Ergo’s Articles be amended to authorize the issuance of the Class C Preferred as
proposed by Dart. A special meeting of the shareholders was properly called for the purpose of voting on
the proposed amendment to the Articles.

Prior to that meeting, a proxy statement was issued to all shareholders disclosing all relevant information
about the plan to issue the Class C Preferred to Dart. However, the proxy statement did not disclose the
alternative funding method the Class B director initially proposed. At the shareholders meeting, a quorum
was present, and the amendment to the Articles was adopted by the following vote:

Class A Shares: 400,000 in favor and 0 opposed

Class B Shares: 720,000 in favor and 100,000 opposed

Following shareholder approval, the Ergo board of directors met to consider the issuance of the newly
authorized Class C Preferred. All seven directors voted to issue the Class C Preferred to Dart for $5
million in cash.

A Class B shareholder filed a derivative action against the directors to enjoin the issuance of the Class C
Preferred to Dart. The Class B shareholder alleged (a) that the directors erred in deciding to issue the
Class C Preferred rather than borrow the money from the bank; (b) that the directors had breached their
duty of care to Ergo; and (c) that Dart had breached her duty of loyalty to Ergo.

Considering the Class B shareholder’s allegations and all possible defenses, who is likely to prevail?
Explain.

Seperac-J19 Exam-Released MEE Essay Compilation © 2016-2020 810


#184: F02-7 MEE: ANSWER: NCBE (CORPS-LLCS)

POINT (1) [8%] ISSUE: What is the applicable legal standard when a shareholder brings a
derivative action based on breach of a director’s fiduciary duties? ANSWER: In a shareholder
derivative action, decisions of the board on business issues are presumptively correct under the
Business Judgment Rule (BJR).

POINT (2)(a) [32%] ISSUE: Do directors violate their duty of care to the corporation when they
vote to recommend action after only one hour of discussion? ANSWER: No. Although the directors
discussed the matter for only one hour, they did not violate their duty of care to Ergo when they
voted to recommend to shareholders the amendment to the Articles.

POINT (2)(b) [16%] ISSUE: Does a director violate her duty of loyalty to the corporation when she
votes to issue herself shares of the corporation? ANSWER: Yes. Dart violated her duty of loyalty to
Ergo when she voted to issue the Class C Preferred to herself.

POINT (3) [43%] ISSUE: If the court finds sufficient evidence of a breach of a fiduciary duty to
overcome the Business Judgment Rule (BJR), can the director prevail with a defense of (i)
disinterested director approval, (ii) shareholder approval, or (iii) fairness? ANSWER: Yes.
Although the directors did not violate their duty of care to Ergo, Dart violated her duty of loyalty.
Regardless, the court would not enjoin the issuance of the Class C Preferred because Dart could
show the transaction was (1) approved by a majority of disinterested directors after full disclosure,
(2) approved by a majority of shareholders after full disclosure, or (3) fair to Ergo.

ANSWER DISCUSSION:

The Class B Shareholder is unlikely to prevail in this derivative action. Ordinarily, the Business Judgment
Rule (BJR) would preclude the court from second-guessing the board of directors on an issue of this sort
(i.e., how best to finance a business expansion). In this case, however, even though the directors did not
breach their duty of care to the corporation, Dart clearly did breach the duty of loyalty by voting to cause
the corporation to enter into a transaction that provided her with a financial benefit. Because of the breach
of the duty of loyalty, the BJR does not immunize the transaction from scrutiny. Nevertheless, the court
would not enjoin the transaction because it was carried out in a manner that removes the taint of the
breach of loyalty.

ANSWER EXPLANATION:

Explanation to Point-One: (05-10%):

In a shareholder derivative action, decisions of the board on business issues are presumptively correct
under the Business Judgment Rule (BJR).

In order to prevail in this derivative action to enjoin the issuance of Class C Preferred, the Class B
shareholder must overcome the BJR. The BJR is a legal presumption. Absent a showing that the directors
have violated a fiduciary duty to the corporation or committed fraudulent or illegal acts, the court will not
second-guess their judgments. For example, one court refused to enjoin the directors from amending the
certificate of incorporation, stating it is not the court’s function to resolve questions of policy and business

Seperac-J19 Exam-Released MEE Essay Compilation © 2016-2020 811


management – the directors are chosen to pass upon such questions and their judgment, unless shown to
be tainted with fraud, is accepted as final.

If the Class B shareholder puts forth sufficient evidence to demonstrate that the directors breached a
fiduciary duty to Ergo, the BJR will no longer apply.

Explanation to Point-Two(a) (25-35%):

Although the directors discussed the matter for only one hour, they did not violate their duty of care to
Ergo when they voted to recommend to shareholders the amendment to the Articles.

Directors owe a duty of care to Ergo; they must act on an informed basis, in good faith in the honest belief
that the action taken is in the best interest of the corporation. Under the Revised Model Business
Corporation Act (RMBCA), directors are required to discharge their duty “in good faith and in a manner
the director reasonably believes to be in the best interests of the corporation.” When becoming informed
to make a decision, directors must discharge their duty “with the care that a person in a like position
would reasonably believe appropriate under similar circumstances.” The duty of care is procedural
(process oriented), not substantive.

The facts do not support an argument for lack of good faith or failure to act “with the care that a person in
a like position would reasonably believe appropriate under similar circumstances.” Even if the other
funding alternatives might have been better for Ergo, directors are not liable for bad decisions so long as
they follow the appropriate procedures, which they did here.

The directors spent only one hour considering and deciding the issue. In one case, two hours was held
insufficient for a major corporate decision. One court found that the duty of care was violated when
directors approved a merger after a 20-minute oral presentation without reviewing the documents or
assessing the valuation at a meeting that was called without much notice. However, courts are generally
concerned with major corporate decisions. In Ergo’s case, the issue was only the source of funding, as the
directors had already properly approved the expansion plan.

Explanation to Point-Two(b) (10-20%):

Dart violated her duty of loyalty to Ergo when she voted to issue the Class C Preferred to herself.

The duty of loyalty requires a director to put the interests of the corporation before the director’s own
interests. If a director enters into a transaction with the corporation that provides the director with
financial benefit, it is a violation of the duty of loyalty. The purchase of the Class C Preferred by Dart is a
transaction with Ergo that provides Dart with a financial benefit. Therefore, Dart violated her duty of
loyalty to Ergo when she voted to issue the Class C Preferred to herself. This was also a conflicting
interest transaction under the RMBCA because Dart knew she could benefit.

Dart has a conflicting interest transaction because, at the time she voted to issue the Class C Preferred to
herself, she knew she had a significant financial interest.

Explanation to Point-Three (35-45%):

Although the directors did not violate their duty of care to Ergo, Dart violated her duty of loyalty.
Regardless, the court would not enjoin the issuance of the Class C Preferred because Dart could show the

Seperac-J19 Exam-Released MEE Essay Compilation © 2016-2020 812


transaction was (1) approved by a majority of disinterested directors after full disclosure, (2) approved by
a majority of shareholders after full disclosure, or (3) fair to Ergo.

Even though the directors did not violate their duty of care, Dart violated her duty of loyalty to Ergo, and
that fact prevents the application of the BJR. Nonetheless, the court would NOT enjoin the issuance of the
Class C Preferred if Dart could demonstrate that the sale of Class C Preferred to her was (1) approved by
a majority of disinterested directors after full disclosure, (2) approved by a majority of shareholders after
full disclosure, or (3) fair to Ergo. In this case, Dart can demonstrate all three.

Disinterested Director Approval: All seven directors voted to offer the Class C Preferred to Dart after full
disclosure of all material facts relating to the transaction. Disclosure of all material facts constitutes full
disclosure. Approval must also be by a majority of disinterested directors. Dart is interested because she is
a party to the transaction. The three remaining Class A directors are arguably interested because their
positions as Ergo directors are dependent upon election by Dart, the sole Class A shareholder. If they
failed to support Dart, Dart could remove them from their positions as directors. The three Class B
directors, however, are disinterested directors. They all voted in favor of issuing the shares to Dart.
Therefore, a majority (indeed all three) of the disinterested directors approved the offering of the Class C
Preferred to Dart. Under the RMBCA (and most state statutes), the three disinterested directors who voted
are sufficient to constitute a quorum when voting on a conflicting interest transaction.

Shareholder Approval: According to the facts, the proxy solicitation provided full disclosure of all
relevant information regarding the plan to issue the Class C Preferred to Dart for $5 million. The fact that
the directors did not disclose that they had considered and discarded other financing options is
unimportant because information on other options was properly considered and discarded. All Class A
shares voted in favor of the transaction, as did a majority of Class B shares (720,000 of 820,000 shares).
Therefore, a majority of shareholders approved the transaction after full disclosure. Note that there is no
requirement that the shareholders be disinterested.

Fair: Alternatively, Dart can show that the transaction was fair to Ergo at the time the Class C Preferred
shares were issued. In assessing fairness, directors may rely on expert opinions. To demonstrate that the
price was fair to Ergo, Dart would offer the opinion of the independent investment bank stating that (1) $5
million would be fair value for the Class C Preferred, and (2) in the long run, payment of the Class C
Preferred dividend would be less costly to Ergo than interest payments on a loan.

Seperac-J19 Exam-Released MEE Essay Compilation © 2016-2020 813


#185-FEB 2001–MEE Q07: QUESTION SEVEN (CORPS-LLCS)

Acme Corp. has 100 shares of common stock authorized, issued, and outstanding. Each share is entitled to
one vote. The corporate records show that these shares are owned equally by five shareholders: Brenda,
Candace, David, Eric, and Fran.

Acme held its annual meeting of shareholders on March 25. The record date for the meeting was 20 days
before the meeting, as permitted by applicable law. Acme gave proper notice of the meeting to all five
shareholders.

Of the five shareholders of record, only Candace and Fran were present at the meeting. However, two
other individuals, George and Henry, were also present. George handed to the corporate secretary a proxy
signed by Brenda and dated the day before the meeting, authorizing George to vote Brenda’s shares at the
meeting. Henry had purchased Eric’s shares the day before the meeting and produced evidence of a sale.

The Chair of the meeting declared that a quorum was present and that Candace, Fran, and George were
entitled to vote at the meeting. However, the Chair declared that Henry was not entitled to vote at the
meeting.

One of the items on the agenda was the election of directors. According to Acme’s bylaws, three directors
have to be elected at Acme’s annual shareholders’ meeting.

At the meeting, the votes for director were cast as follows:

Candace cast 20 votes for Candace, 20 votes for Fran, and 20 votes for David.

Fran cast 20 votes for Candace, 20 votes for Fran, and 20 votes for David.

George cast 20 votes for George, 20 votes for Brenda, and 20 votes for Henry.

Henry attempted to cast a ballot of 20 votes for George, 20 votes for Brenda, and 20 votes for Henry.
However, the Chair refused to accept Henry’s ballot. The Chair then declared that Candace, Fran, and
David were elected as directors.

There are no provisions in the articles of incorporation or the bylaws of Acme that are relevant to the
following questions.

1. Was Henry entitled to vote at the annual meeting and, if not, what could he have done to acquire
the right to vote? Explain.

2. Was there a quorum present for shareholder action at the annual meeting? Explain.

3. Did Candace, Fran, and David receive sufficient votes to be elected as directors of Acme?
Explain.

Seperac-J19 Exam-Released MEE Essay Compilation © 2016-2020 814


#185: F01-7 MEE: ANSWER: NCBE (CORPS-LLCS)

POINT (1) [33%] ISSUE: Under what circumstances would Henry, as beneficial owner but not
record owner of 20 shares, be entitled to vote at the Acme annual meeting? ANSWER: No. Without
having obtained a proxy, Henry was not entitled to vote at the annual meeting because only
shareholders of record are entitled to vote.

POINT (2) [33%] ISSUE: Was there a quorum present for shareholder action at the Acme annual
meeting? ANSWER: Yes. A quorum was present for shareholder action at the Acme annual
meeting.

POINT (3) [33%] ISSUE: Did Candace, Fran, and David receive sufficient votes to be elected as
directors of Acme meeting? ANSWER: Yes. Candace, Fran, and David each received sufficient
votes to be elected as directors at the Acme annual meeting.

ANSWER EXPLANATION:

Explanation to Point-One (25-35%):

Without having obtained a proxy, Henry was not entitled to vote at the annual meeting because only
shareholders of record are entitled to vote.

Generally only shareholders of record are entitled to vote at a meeting of shareholders. Under Delaware
law, the share ledger is the only evidence as to who are the shareholders entitled to vote, in person or by
proxy, at any meeting of shareholders. A mere beneficial owner of shares is not entitled to vote at a
meeting of shareholders. Beneficial ownership of shares does not carry with it the right to vote without
having the shares transferred on the corporate books.

Where corporate shares are transferred between the record date for a meeting of shareholders and the date
of the meeting, the transferee is not entitled to vote the shares at the meeting. The corporation is not
required to determine the “true” owners of corporate shares on the day of the meeting. Rather, the
corporation is entitled to rely on its share ledger as of the record date for the meeting in determining who
is entitled to vote.

The establishment of a record date is for the convenience of the corporation. A transferee of shares after
the record date who wants to vote at a scheduled shareholder meeting should obtain a proxy from his or
her transferor to vote the shares.

Explanation to Point-Two (25-35%):

A quorum was present for shareholder action at the Acme annual meeting.

Neither the articles of incorporation nor the bylaws of Acme contain a provision relating to the quorum
for a meeting of shareholders. Consequently, a quorum is a majority of the votes entitled to be cast by the
voting group. The voting group in this case consisted of all the common shareholders of Acme.

A shareholder may vote his or her shares either in person or by proxy. Candace and Fran were present at
the meeting in person. Brenda granted a proxy appointment to George to vote her shares, as is the right of

Seperac-J19 Exam-Released MEE Essay Compilation © 2016-2020 815


a shareholder, and, thus, was present by proxy. A proxy should be accepted as valid if on its face it
appears to be “free from all reasonable grounds of suspicion of its genuineness and authenticity.” The
facts do not suggest that the proxy from Brenda was suspicious on its face. Consequently, 60 shares
entitled to vote were present at the annual meeting of Acme. Since, as discussed above, Henry (as a mere
beneficial owner) was not entitled to vote, his shares are not counted as present for quorum purposes.

There were 100 shares entitled to vote at the Acme annual meeting. A majority of 100 is 51. Since, as
discussed above, there were 60 shares entitled to vote present at the Acme annual meeting, a quorum was
present for shareholder action even though only two of the five shareholders owning only 40 of the 100
shares were physically present at the meeting.

Explanation to Point-Three (25-35%):

Candace, Fran, and David each received sufficient votes to be elected as directors at the Acme annual
meeting.

In most states the election of directors requires only a plurality vote of the shares entitled to vote at a
meeting at which a quorum is present. As discussed above, a quorum was present at the Acme annual
meeting.

The plurality vote requirement for directors simply means “that the individuals with the largest number of
votes are elected as directors up to the maximum number of directors to be chosen at the election.” Three
directors were to be elected at the Acme annual meeting of shareholders. Candace, Fran, and David each
received 40 votes, which was more than any other candidates received. George, Brenda, and Henry only
received 20 votes each. Consequently, Candace, Fran, and David were elected as directors.

In a minority of states, directors must be elected by a majority of the shares entitled to vote at a meeting of
shareholders at which a quorum is present. A majority of the 60 shares present and entitled to vote was
31. (As discussed above, Henry was not entitled to vote at the meeting.) Since 40 shares voted for the
election of Candace, Fran, and David, they received enough votes to be elected as directors even in those
few states requiring a majority vote.

In voting his or her shares, each shareholder can cast no more than 20 votes for each directorship to be
filled unless cumulative voting is authorized. If cumulative voting is authorized (which it is not here),
then each shareholder has a number of votes equal to the number of shares owned times the number of
directorships to be filled and the shareholder can allocate those votes among any of the persons running
for director.

Seperac-J19 Exam-Released MEE Essay Compilation © 2016-2020 816


#186-JUL 2000–MEE Q04: QUESTION FOUR (CORPS-LLCS)

In 1950, John Smith founded Store, Inc., and he was its sole shareholder. Since 1970, Store has been a
profitable business with eight large stores. In 1975, Smith took several steps to try to ensure that control
of the business would stay in his family. One such step was properly to amend Store’s Articles of
Incorporation to provide that no Store shareholder could transfer any Store stock unless the shareholders
unanimously consented to the transfer and approved the transferee.

In 1990, observing the formalities required by the amended Articles, Smith gave all of his Store stock in
equal shares to his four children: Anne, Bob, Clyde, and Dave. Smith died in 1992, and his four children
continued Store’s business. After Smith’s death, Anne, Bob, and Clyde served as the officers and
directors of Store, and each was also employed by the corporation. Dave was a full-time dentist and was
not interested in being an officer or a director of Store.

During the years following Smith’s death, Anne, Bob, and Clyde followed a policy of reinvesting Store’s
earnings in order to expand the number of stores. Dave consistently opposed this policy and believed that
the majority of the earnings should be distributed as dividends to Store’s shareholders.

After Smith’s death, Store’s board of directors also substantially increased the compensation paid to
Anne, Bob, and Clyde as directors and doubled their salaries as employees, increasing them to levels well
above market salaries. During the same period, however, the board voted to eliminate the quarterly
dividends that Store had historically paid out, even though Store had sufficient profits to pay those
dividends. At every shareholders’ meeting, Dave objected to the way Store was being run, but he was
ignored each time. Eventually, Dave advised the three other shareholders, Anne, Bob, and Clyde, that he
wanted to become a member of the board of directors or, in the alternative, that he wanted them to buy his
shares. They refused both requests.

Last year, Dave pledged one-half of his Store stock to Bank as collateral for a personal loan, and Dave
and Bank executed a pledge agreement covering those shares. Recently, the loan came due, and Dave was
unable to repay it. Consequently, Bank properly foreclosed and became the owner of the shares under the
pledge agreement.

Bank has told Anne, Bob, and Clyde that it is now the owner of one-half of Dave’s shares of Store stock,
and Bank requests that the shares be transferred on Store’s books. Anne, Bob, and Clyde have told Bank
that it is not the owner of any shares of Store stock because they have not approved the transfer as
required by Store’s Articles of Incorporation. Therefore, Store will not record the transfer of the shares to
Bank.

Dave believes he is being “frozen out” of the corporation and wants to have Store dissolved.

1. Is the stock transfer restriction enforceable against Dave? Explain.

2. Can Bank compel Store to record the stock transfer? Explain.

3. Can Dave force a dissolution of Store? Explain.

Seperac-J19 Exam-Released MEE Essay Compilation © 2016-2020 817


#186: J00-4 MEE: ANSWER: NCBE (CORPS-LLCS)

POINT (1) [39%] ISSUE: Are restrictions on a shareholder’s ability to transfer stock of a close
corporation that require the consent of all other shareholders to the transfer and the transferee
enforceable against a transferring shareholder? ANSWER: Yes. Restrictions on the transfer of
stock that require approval of the transfer and transferee by all other shareholders are generally
enforceable as long as the restrictions are created for reasonable purposes.

POINT (2) [12%] ISSUE: Are restrictions on a shareholder’s ability to transfer stock of a close
corporation that require the consent of all other shareholders to the transfer and the transferee
enforceable against a third party that lacks notice? ANSWER: Restrictions on the transfer of stock
would be enforceable against Bank if it had actual knowledge of the restrictions and would not be
enforceable against Bank if it did not.

POINT (3) [49%] ISSUE: Will a court order the dissolution of Store to protect the minority
shareholders? ANSWER: Yes. A court is likely to order the dissolution of Store in order to protect
the minority shareholders from oppressive conduct by the majority.

ANSWER EXPLANATION:

Explanation to Point-One (35-45%):

Restrictions on the transfer of stock that require approval of the transfer and transferee by all other
shareholders are generally enforceable as long as the restrictions are created for reasonable purposes.

As a general rule, share transfer restrictions are invalid if they are unreasonable restraints on alienation of
stock. Thus, an absolute restraint on alienation of stock is per se unreasonable and void. Usually,
however, courts will permit share transfer restrictions if they are adopted for a lawful purpose. Lawful
purposes include keeping a corporation closely held or keeping a corporation in the family.

The share transfer restriction described in the facts requires the consent of all other shareholders and the
approval by such shareholders of the proposed transferee. Consent restraints, such as this one, are among
the most restrictive stock transfer restraints because they make stock unassignable if one shareholder
refuses to vote to approve the transfer. The courts are split on the enforceability of consent restrictions.
Some jurisdictions have considered such a restriction to be tantamount to an absolute restraint on
alienation, manifestly unreasonable and void. Other jurisdictions, however, have enforced transfer
restrictions of this type. Modern statutory law frequently permits many types of restrictions on the transfer
of stock. The Revised Model Business Corporation Act (RMBCA) permits restrictions established in a
corporation’s “articles of incorporation, bylaws, an agreement among shareholders, or an agreement
between shareholders and the corporation.” “A restriction on the transfer or registration of transfer of
shares is authorized for any reasonable purpose.” The RMBCA also provides that a stock transfer
restriction may “require the corporation, the holders of any class of its shares, or another person to
approve the transfer of the restricted shares, if the requirement is not manifestly unreasonable.”

An initial determination must be made as to whether the restriction on transfer is reasonable and therefore
enforceable against Dave, a shareholder. Arguably, it is not manifestly unreasonable to allow family
members to approve new shareholders so they can keep control in the family or in people acceptable to
the family. Therefore, the restriction would be enforceable against Dave. Alternatively, it could be argued

Seperac-J19 Exam-Released MEE Essay Compilation © 2016-2020 818


that the restriction would not be enforceable because the restriction might unreasonably result in the
current shareholders never having the right to transfer their shares in Store.

Explanation to Point-Two (10-15%):

Restrictions on the transfer of stock would be enforceable against Bank if it had actual knowledge of the
restrictions and would not be enforceable against Bank if it did not.

Assuming that the share transfer restriction is enforceable against Dave, it must be determined whether
the restriction is binding on Bank. Both the RMBCA and the Uniform Commercial Code (UCC) address
the issue of whether the share transfer restrictions are binding on third parties to whom restricted stock is
conveyed. The RMBCA states:

A restriction on the transfer or registration of shares is valid and enforceable against the holder or a
transferee of the holder if the restriction is authorized by this section and its existence is noted
conspicuously on the front or back of the certificate. Unless so noted, a restriction is not enforceable
against a person without knowledge of the restriction.

Section 8-204 of the UCC similarly provides that a stock transfer restriction is invalid unless the
restriction is noted conspicuously on the stock certificate.

The facts state that Store’s Articles of Incorporation provide for the share transfer restrictions. Therefore,
the restriction is “authorized” as required by the RMBCA. However, the facts do not indicate whether the
transfer restriction is conspicuously noted on the stock certificates. Therefore, it is not clear whether Bank
knew of the share transfer restriction on the Store stock that Dave had pledged to the Bank. If the
restriction is conspicuously noted on the certificates, the restriction is valid and enforceable against Bank.
If it is not conspicuously noted, the restriction is enforceable only if Bank had actual notice of it through
other means. Because the facts are silent as to whether Bank had knowledge of the restriction, it cannot be
determined whether the restrictions are enforceable against Bank.

Explanation to Point-Three (45-55%):

A court is likely to order the dissolution of Store in order to protect the minority shareholders from
oppressive conduct by the majority.

In this case, the directors and controlling shareholders may have embarked upon a plan to freeze out
Dave, the minority shareholder. The plan appears to have two elements: to deprive Dave of a voice in
management and to eliminate shareholder dividends while increasing salaries to the majority shareholders
and the directors.

The RMBCA permits courts to intervene in the affairs of closely held corporations under limited
circumstances. A court may dissolve a corporation “in a proceeding by a shareholder if it is established
that the directors or those in control of the corporation have acted, are acting, or will act in a manner that
is illegal, oppressive or fraudulent.”

The issue here is whether the conduct of Anne, Bob, and Clyde, the majority shareholders, is “illegal,
oppressive or fraudulent.” Shareholder dividends were eliminated while directors’ and employees’
compensation was dramatically increased, and the minority shareholder was given no voice in
management. It appears that the directors and controlling shareholders are attempting to pay to themselves
what had formerly been regarded as corporate profits.

Seperac-J19 Exam-Released MEE Essay Compilation © 2016-2020 819


The majority shareholders will argue that they have a right to accumulate profits, that dividends are
usually discretionary and within the business judgment of the board of directors, and that their actions had
a legitimate business purpose. Nonetheless, the combination of eliminating dividends while substantially
increasing the compensation of majority shareholders in their capacities as directors and employees of the
corporation raises the issue of whether the minority shareholders have been oppressed.

The standard for oppressive conduct is not clearly established. Some courts define oppression as violating
the reasonable expectations of shareholders, “One of the most significant trends in the law of close
corporations in recent years is the increasing willingness of courts to look to the reasonable expectations
of shareholders to determine whether ‘oppression’ or similar grounds exist as a justification for
involuntary dissolution or another remedy.”

Many courts have taken an alternative approach, finding that in closely held corporations, controlling
shareholders have a higher fiduciary duty to minority shareholders than in publicly held corporations.
Under such a view, majority shareholders “are required to deal fairly, honestly and openly with minority
shareholders and may not use their corporate control to prevent the minority from having an equal
opportunity in the corporation.” In addition, some courts have held directors in a closely held corporation
to higher fiduciary duties than those of directors in public corporations.

Whether the standard is reasonable expectations of shareholders or the duty of good faith and loyalty from
majority to minority shareholders, it is clear that attempts to freeze out minority shareholders are
oppressive if the majority shareholders profit from the corporation to the detriment of minority
shareholders. If what is properly a dividend is converted to salary for majority shareholders, the
expectations of the minority shareholders have been frustrated and the obligations of the majority
shareholders have been breached, thereby justifying judicial dissolution.

The RMBCA provides that if a proceeding is started to dissolve a corporation, the corporation may
terminate the action if it, or a shareholder, purchases the complaining shareholder’s shares for fair value.

Thus, Dave, the minority shareholder, is likely to get a court-ordered dissolution of Store. The corporation
or any of its shareholders may choose to purchase Dave’s stock to stop the proceeding.

Seperac-J19 Exam-Released MEE Essay Compilation © 2016-2020 820


#187-FEB 2000–MEE Q07: QUESTION SEVEN (CORPS-LLCS)

PrattCo is a corporation. It has been developing a new textile inspection system that promises to
revolutionize the market if it can be made to work. PrattCo has one class of stock, Class A Common, of
which three million shares are authorized. All three million shares of the Class A Common are currently
issued and outstanding. The Class A Common shares are currently trading at $25 a share on a recognized
market.

PrattCo has two immediate problems. First, it needs a new chief executive officer with the expertise and
drive to get the new system working and to expand PrattCo’s operations. Second, PrattCo needs at least
$75 million to finance the development of the new system and the expansion of its operations.

The PrattCo board has offered Leslie the job of PrattCo’s president and chief executive officer. Leslie’s
salary requirements are modest, but as part of her compensation Leslie wants to receive options for
100,000 shares of Class A Common at $25 a share, with the right to exercise the options phased over a
three-year period.

Brenda has offered to invest $75 million in PrattCo in return for three million shares of Class A Common
at $25 per share, but she wants her Class A Common shares to be different from the existing Class A
Common. Brenda wants her shares to have a preference such that any dividends would be allocated first
to Brenda’s Class A Common shares up to $2 per share per annum, then to the other Class A Common
shares up to $2 per share per annum, and then to all Class A Common shares equally.

William currently owns 30% of the Class A Common stock. William was an inside candidate for the
position of president and chief executive officer and is bitter that Leslie was offered the job. William also
disagrees with the $75 million capital infusion from Brenda. William wants to license the new system to
third-party manufacturers rather than have PrattCo expand its own operations.

William objects that: (1) the grant of the proposed stock options to Leslie is illegal; (2) the sale to Brenda
of the Class A Common shares with preferential rights is illegal; (3) if the sale to Brenda can be legally
accomplished, it requires William’s consent; and (4) even if the grant to Leslie and the sale to Brenda can
be made, William would have the right to purchase shares equal to 30% of the stock issued to both Leslie
and Brenda.

PrattCo’s articles of incorporation state that “the corporation elects to have preemptive rights.” Assume
the board and the stockholders of the remaining 70% of Class A Common will vote to hire Leslie, to grant
her the options requested, and to accept Brenda’s financing proposal. Also assume William has sufficient
financial resources to exercise any stock purchase rights he may have.

Are each of William’s four objections to PrattCo’s plans valid and, if so, what steps could PrattCo take to
avoid the objections? Explain.

Seperac-J19 Exam-Released MEE Essay Compilation © 2016-2020 821


#187: F00-7 MEE: ANSWER: NCBE (CORPS-LLCS)

POINT (1)(a) [17%] ISSUE: Can a corporation award an executive stock options in that
corporation as part of the executive’s compensation? ANSWER: Yes. Subject to Point One(b),
PrattCo has the right to award options to its officers for purchase of shares as part of the officers’
compensation.

POINT (1)(b) [17%] ISSUE: Can a corporation issue stock options when all of the authorized
shares of such corporation are issued and outstanding? ANSWER: No. PrattCo has issued all of its
authorized shares: therefore, the stock options cannot be issued unless the board and the
shareholders vote to increase the number of common shares authorized.

POINT (2) [30%] ISSUE: Can a corporation issue additional shares of an existing class of common
stock that have greater rights and preferences than the outstanding shares of that class? ANSWER:
No. Brenda’s proposal to purchase Class A Common shares cannot be implemented because it
provides for preferential dividend rights within an undifferentiated class of stock.

POINT (3) [22%] ISSUE: Is the consent of all common shareholders required for the issuance of
preferred shares by a corporation? ANSWER: No. PrattCo could create a new class of shares
(Preferred Stock and sell Brenda shares of Preferred Stock with preferential rights if the board and
the shareholders approved amendments to the articles to provide for such Preferred Stock.
Amendment of the articles of incorporation to create Preferred Stock requires board approval and
a majority vote of the Class A Common shareholders, which, under the facts, can be obtained.

POINT (4) [13%] ISSUE: Are a common shareholder’s preemptive rights triggered by an award of
common stock options to an executive, by the issuance of additional common stock, or by the
issuance of preferred stock? ANSWER: William has preemptive rights under the articles and
generally would be entitled to his proportional share of any Class A Common stock issued.
However, no preemptive rights exist with respect to shares issued as compensation to officers, such
as those to be issued to Leslie, and no preemptive rights exist for common shareholders with respect
to preferred shares that are not convertible into common, such as the Preferred Stock that could be
issued to Brenda.

ANSWER EXPLANATION:

Explanation to Point-One(a) (15-25%):

Subject to Point One(b), PrattCo has the right to award options to its officers for purchase of shares as
part of the officers’ compensation.

A corporation may issue options for the purchase of its shares on terms and conditions determined by the
board of directors. Shares may be issued, and options extended, for different types of consideration,
including both “services performed” and “contracts for services to be performed.” Thus, the board could
agree to give Leslie the requested options either on an “as earned” basis as Leslie completes periods of
service with the company, or “up front” in consideration for a contract for future services. Therefore, the
board would normally be entitled to issue the options to Leslie subject to all necessary actions to authorize
sufficient shares as discussed below.

Seperac-J19 Exam-Released MEE Essay Compilation © 2016-2020 822


Explanation to Point-One(b) (15-25%):

PrattCo has issued all of its authorized shares: therefore, the stock options cannot be issued unless the
board and the shareholders vote to increase the number of common shares authorized.

A corporation must have a sufficient number of authorized shares to cover all issued and outstanding
shares, as well as all shares subject to options that have been issued. Articles of incorporation are required
to state the number of authorized shares of the corporation. PrattCo’s articles of incorporation authorize
three million shares of Class A Common stock, of which all three million shares have been issued and are
outstanding. This means that PrattCo cannot issue the stock options to Leslie unless its articles are
amended to provide for additional authorized shares of Class A Common stock in a number at least
sufficient to meet Leslie’s options.

An increase in the number of Class A Common shares authorized requires approval of both the board of
directors and the Class A Common shareholders. Although a supermajority vote for certain changes to a
corporation’s articles may be specified in the articles of incorporation, no supermajority requirement is
found in PrattCo’s articles, so a majority of the shares voted will be sufficient to approve the change.
Since all Class A shareholders (other than William) will vote to authorize the additional shares, PrattCo
will be able to carry out its plan to issue the options and hire Leslie.

Explanation to Point-Two (30-40%):

Brenda’s proposal to purchase Class A Common shares cannot be implemented because it provides for
preferential dividend rights within an undifferentiated class of stock.

Brenda’s proposal cannot be implemented because it gives Brenda’s Class A Common shares preferential
dividend rights beyond that given to the other Class A Common shares. All shares within a class of stock
must have identical rights and preferences unless the shares within a class are divided into separate series.
In addition, shares that have a preferred right to receive dividends or distributions on liquidation should
not be called “common” shares. Since Brenda’s shares would have preferred rights to receive dividends,
they cannot be issued as Class A Common shares.

Explanation to Point-Three (20-30%):

PrattCo could create a new class of shares (Preferred Stock and sell Brenda shares of Preferred Stock with
preferential rights if the board and the shareholders approved amendments to the articles to provide for
such Preferred Stock. Amendment of the articles of incorporation to create Preferred Stock requires board
approval and a majority vote of the Class A Common shareholders, which, under the facts, can be
obtained.

PrattCo could accomplish Brenda’s objective by issuing shares of Preferred Stock instead of Class A
Common shares. To do so, PrattCo would have to amend the articles of incorporation to create the
Preferred Stock and designate the shares to be sold to Brenda as Preferred Stock with the preferential
rights that Brenda has requested. Amendment of the articles of incorporation to create Preferred Stock
requires board approval and a majority vote of the Class A Common shareholders. Since William holds
only 30% of the Class A Common shares and all shareholders other than William will vote in favor of
creating the Preferred Stock, William will not be able to block the creation and issuance of the Preferred
Stock.

Explanation to Point-Four (10-20%):

Seperac-J19 Exam-Released MEE Essay Compilation © 2016-2020 823


William has preemptive rights under the articles and generally would be entitled to his proportional share
of any Class A Common stock issued. However, no preemptive rights exist with respect to shares issued
as compensation to officers, such as those to be issued to Leslie, and no preemptive rights exist for
common shareholders with respect to preferred shares that are not convertible into common, such as the
Preferred Stock that could be issued to Brenda.

The PrattCo articles of incorporation include language that “the corporation elects to have preemptive
rights.” This triggers the preemptive rights provisions of the RMBCA. Under the RMBCA and many state
statutes, a corporation must “opt in” to create preemptive rights by expressly including the right in its
articles of incorporation. In other states, preemptive rights are presumed to exist unless the corporation
“opts out” and expressly excludes such rights in its articles. Regardless of the type of law in this
jurisdiction, the statement in the PrattCo articles establishes preemptive rights for its shareholders.

However, there are several widely recognized exceptions to preemptive rights. Under the RMBCA and
many state statutes, no preemptive rights exist as to “shares issued to satisfy option rights created to
provide compensation to officers of the corporation.” Therefore, William has no preemptive right to buy
any PrattCo shares as a result of the issuance of Leslie’s options.

If Brenda were to purchase Class A Common shares, the sale would not come within any of the statutory
exclusions of preemptive rights, and William would have the right to purchase 30% of the Class A
Common shares sold to Brenda. However, since Brenda’s shares must be issued as Preferred Stock (not
Class A Common), as discussed in Point Three above, William’s preemptive rights claim can be defeated.
Holders of common shares have no preemptive rights to preferential shares unless those shares are
convertible to common. So long as the Preferred Stock, which is entitled to preferential rights to
dividends, is not convertible into Class A Common and carries no rights to acquire Class A Common,
William would have no preemptive rights in the Preferred Stock sold to Brenda.

Seperac-J19 Exam-Released MEE Essay Compilation © 2016-2020 824


#188-JUL 1999–MEE Q07: QUESTION SEVEN (CORPS-LLCS)

Carl decided to form a corporation to provide various support services to small businesses on a contract
basis. After several months of searching, Carl found suitable office space in an office park that he thought
was ideal for his future corporation. Leaseco was willing to rent the office space at a very favorable rate
but only if the lease could be signed immediately. Although Carl had not taken any-steps to incorporate,
he began negotiations with Leaseco for the lease. Carl told Leaseco that the lease was on behalf of a
corporation that would be formed, to be called Small Service Corporation (SSC), and that Carl would be
president of SSC. Carl signed a two-year lease with Leaseco for the office space at the favorable rate. The
parties named in the lease were Leaseco and SSC. Carl signed the lease as “Small Service Corporation, a
corporation to be formed, by Carl, President.” The following week, Carl moved his equipment and
supplies into the office space and began business operations.

Two weeks after the lease was signed, Carl properly incorporated SSC. The articles of incorporation
named Carl and two other individuals, David and Ellen, as the initial directors of the corporation, and the
directors properly elected Carl as president.

At the first meeting of the board of directors, Carl provided David and Ellen with copies of the Leaseco
office lease. SSC printed corporate letterhead with the address of the leased office space and began
making monthly lease payments to Leaseco. SSC’s business was an immediate success and after nine
months of operations it began to show a profit. Because of the initial success of the business, Carl, David,
and Ellen needed to hire additional personnel. However, the office space leased from Leaseco was too
small for any expansion. The directors unanimously voted to terminate the Leaseco lease before the end
of its term. Then they entered into a lease with Rental, Inc. for a larger space, vacated the Leaseco
premises, and refused to pay any further rent to Leaseco.

Several months after SSC began operations, at a regularly scheduled meeting of the board of directors for
which all proper notice was given, Carl proposed that SSC contribute 4% of SSC’s profits earned since
incorporation to a local charity that operates a nonprofit job training program. Carl and David voted to
approve the resolution; Ellen voted against it. The SSC articles of incorporation have no provisions
dealing with the power of the corporation to make charitable contributions.

1. Is Carl personally liable on the Leaseco office lease? Explain.

2. Is SSC liable on the Leaseco office lease? Explain.

3. Does SSC have the power to make the charitable contribution? Explain.

Seperac-J19 Exam-Released MEE Essay Compilation © 2016-2020 825


#188: J99-7 MEE: ANSWER: NCBE (CORPS-LLCS)

POINT (1) [41%] ISSUE: Is a promoter personally liable on a preincorporation contract with a
third party? ANSWER: Yes. As a promoter, Carl is probably personally liable on the contract with
Leaseco since the facts do not show that Leaseco agreed to look solely to SSC once formed for
performance.

POINT (2) [41%] ISSUE: Is a corporation, once incorporated, liable on a preincorporation contract
with a third party? ANSWER: Yes. Although not originally a party to the lease. SSC is probably
liable on the lease with Leaseco since it appears that SSC adopted the lease as its own by accenting
the benefits of the lease with full knowledge of the contract.

POINT (3) [18%] ISSUE: Does a corporation have the corporate power to use a portion of its
profits to make a charitable contribution? ANSWER: Yes. SSC has statutory power to make the
charitable contribution because it is reasonable in amount and purpose.

ANSWER EXPLANATION:

Explanation to Point-One (30-40%):

As a promoter, Carl is probably personally liable on the contract with Leaseco since the facts do not show
that Leaseco agreed to look solely to SSC once formed for performance.

Carl was acting as a promoter when he entered into the lease with Leaseco for the office space. A
promoter is the party who assists in the planning and formation of a new business. A promoter who enters
into a contract with a third party on behalf of a corporation that both parties to the transaction know is not
yet formed is personally liable on the contract unless the other party to the transaction agrees to look
solely to the corporation for performance. Where a person knowingly enters into an agreement with a
promoter on behalf of a nonexistent corporation, there is an inference that the person intends to make a
present contract with an existing person. As a result, there is a rebuttable inference that the parties intend
the promoter to be a party to the contract.

The form of the contract entered into is an important factor in determining how preincorporation contracts
are treated. The type of preincorporation contract at issue in this question is a contract by a promoter with
a third party in the name of a corporation to be formed. In this circumstance, it is clear that both parties
were aware at the time the contract was entered into that a corporation had not yet come into existence.
Under such circumstances, the promoter is nearly always liable for the contract until the corporation is
formed. The only exception is where it is clear that the third party agreed to look solely to the to-be-
formed corporation for payment.

The facts do not rebut the inference that the parties intended Carl, the promoter, to be liable on the lease.
Carl moved his operations into the office space and began business immediately after execution of the
contract and prior to incorporation of SSC. This suggests that the parties intended someone to be
immediately bound. Since the corporation was not yet in existence, the only person that the parties could
have intended to be bound was Carl. The facts do not indicate that Leaseco looked solely to the
corporation to be formed for performance under the lease.

Seperac-J19 Exam-Released MEE Essay Compilation © 2016-2020 826


Sometimes a transaction between a promoter and a third person is interpreted as an offer by the third
person to be conveyed to the corporation by the promoter when the corporation is formed. However, in
this situation, where the parties desired to sign the lease immediately and where Carl immediately moved
into the leased space and began business operations, it appears that the parties did not intend only an offer
to the future corporation, but instead a present contract between Carl and Leaseco.

Subsequent adoption of a preincorporation contract by the corporation after it is formed generally would
not release the promoter from liability unless there is a novation by which Leaseco agrees to release Carl
from liability and look solely to SSC. Whether or not SSC adopted the lease, the facts presented here do
not suggest that a novation with Leaseco occurred. Therefore, Carl’s liability on the lease would continue
even if SSC was deemed to have adopted the contract.

A promoter who has not violated any fiduciary duties to the corporation but is held liable on a
preincorporation contract will often be able to obtain indemnity from the corporation. In addition,
indemnification of the promoter may be provided for in the organizational documents signed by the initial
board of directors in connection with incorporation of the corporation.

Explanation to Point-Two (30-40%):

Although not originally a party to the lease. SSC is probably liable on the lease with Leaseco since it
appears that SSC adopted the lease as its own by accenting the benefits of the lease with full knowledge
of the contract.

SSC was not originally a party to the lease. Because SSC did not exist when the lease was entered into,
Carl could not have been acting as the agent of SSC in entering into the lease. Upon incorporation, a
corporation is not automatically bound by the contracts previously entered into by its promoters.
However, after the corporation is formed, a corporation may adopt a preincorporation contract and
become liable under that contract. A corporation may adopt a contract in the same manner as it may make
an original contract. Thus, in an appropriate case the adoption need not be express, but rather may be
inferred from the actions of the corporation or its agents.

On the current facts, the SSC board was given a copy of the Leaseco lease and was aware of its terms,
SSC occupied the leased office space, SSC paid the favorable rent for many months, and SSC generally
accepted the benefits of the lease. A court would likely conclude that SSC had adopted the lease made on
its behalf prior to incorporation.

Explanation to Point-Three (10-20%):

SSC has statutory power to make the charitable contribution because it is reasonable in amount and
purpose.

Modern corporation statutes provide a corporation with statutory powers that need not be stated in the
articles of incorporation. One of these powers is the power to make donations for charitable, scientific, or
educational purposes.

Courts have found that a reasonable charitable contribution is within the statutory power of the
corporation.

In determining whether a contribution is reasonable, a court will focus heavily on the amount of the
contribution. For example, one court concluded that the Internal Revenue Code provisions pertaining to

Seperac-J19 Exam-Released MEE Essay Compilation © 2016-2020 827


corporate charitable contributions provide a “helpful guide” on the issue of reasonableness. In this case a
court is likely to find that the amount of the charitable donation authorized by the resolution of the board
of directors (i.e., 4% of profits) is reasonable in amount.

A court may also look at the purpose of a charitable contribution to determine whether it is reasonable.
However, as long as the donation is not made indiscriminately, the judgment of the corporate officials that
the contribution furthers corporate interests is likely to be accepted. A court is likely to determine that the
donation by SSC is a reasonable means of “increasing good will and promoting patronage.” Since SSC is
in the business of providing support services to small businesses, a contribution to a job training program
has a reasonable relation to its business.

Absent statutory authority to make reasonable charitable contributions or specific authority to do so in the
articles of incorporation, a corporation’s charitable contributions might be void under the ultra vires
doctrine. This doctrine invalidates corporate acts inconsistent with the corporation’s purposes as stated in
its articles. Today, most courts would not find the making of charitable contributions ultra vires and even
in the absence of express statutory authority, find corporate contribution by board action consistent with
the board’s authority to conduct corporate affairs.

Seperac-J19 Exam-Released MEE Essay Compilation © 2016-2020 828


#189-FEB 1999–MEE Q06: QUESTION SIX (CORPS-LLCS)

FurnitureCo has manufactured quality furniture for 20 years. It was incorporated under the Revised Model
Business Corporation Act and its articles of incorporation and bylaws contain no provisions overriding
the statutory provisions. FurnitureCo’s articles of incorporation provide that the corporation can engage in
any lawful business.

Adam owns 70% of the stock of FurnitureCo, and Diane and Edward each own 15%. The board of
directors has five members: Adam; Adam’s loyal children, Beth and Charles; and the two outsiders, Diane
and Edward. Adam is the president of FurnitureCo, Beth is vice president, and Charles is secretary. Their
duties are not specified in the bylaws. Neither Diane nor Edward is an officer.

Over the past few years, FurnitureCo’s business has been flat, but marginally profitable. At its last regular
board meeting, attended by Adam, Beth, Charles, and Edward, Adam presented a plan to diversify
FurnitureCo’s business, which included entering the tennis club business. Diane had been called out of
town suddenly and was unable to attend the board meeting. After hearing Adam’s presentation on the
expanding opportunities in the sports field, the board voted 3 to 1 to direct Adam as president to “study
and implement at his discretion a program for restructuring and/or reorienting the business operations of
the company.” It was Edward who voted against authorizing Adam to restructure the business.

After studying the matter, Adam decided that FurnitureCo’s profits would be maximized by getting out of
the furniture business altogether, selling all of its current assets, and using the sale proceeds and the cash
on hand to buy a tennis club. On his own initiative and without anyone else’s approval, he then scheduled
an auction for next month to sell all of FurnitureCo’s existing assets.

Diane and Edward were sure that Adam’s plan would ruin the business, so they visited Adam in his office
to tell him that they disagreed with his plan and to demand that he cancel the auction. Adam told them
that he did not need their approval to implement his plan and ordered them to leave his office.

1. Does Adam have sufficient authority as president to change FurnitureCo’s business and to sell its
existing assets? Explain.

2. Could Adam, Beth, and Charles lawfully act together to approve and implement the change in
the business and the sale of assets without the participation of Diane and Edward? Explain.

3. Assuming that Diane and Edward cannot block the sale, do they have any other remedies?
Explain.

Seperac-J19 Exam-Released MEE Essay Compilation © 2016-2020 829


#189: F99-6 MEE: ANSWER: NCBE (CORPS-LLCS)

POINT (1) [50%] ISSUE: Is the decision of a president of a corporation to change the business, as
authorized by the board, and his unilateral decision to sell the assets, within his power as an officer?
ANSWER: No. The board of directors authorized Adam, as president, to change the corporation’s
business from furniture to tennis clubs. However, shareholder approval, in addition to board
approval, is required to authorize Adam to sell all of the assets of FurnitureCo.

POINT (2) [25%] ISSUE: Do minority directors have the ability to block a change in the
corporation’s business and the sale of all of its assets? ANSWER: In their capacity as both directors
and shareholders, Diane and Edward must be notified of and given the opportunity to vote on the
sale of FurnitureCo’s assets, although they don’t have enough votes to prevail.

POINT (3) [25%] ISSUE: Can minority shareholders force the corporation to buy their shares
when they object to the corporation’s change of business and sale of its assets? ANSWER: Yes.
Diane and Edward can exercise their dissenters’ rights and force FurnitureCo to buy their shares.

ANSWER EXPLANATION:

Explanation to Point-One (40-60%):

The board of directors authorized Adam, as president, to change the corporation’s business from furniture
to tennis clubs. However, shareholder approval, in addition to board approval, is required to authorize
Adam to sell all of the assets of FurnitureCo.

Officers are authorized to take actions as authorized in the bylaws, as directed by the board of directors
consistent with the bylaws, or as directed by other officers acting under the authority of directors. In this
case, Adam was authorized by the board of directors to “study and implement at his discretion a program
for restructuring and/or reorienting the business operations of the company.” There is nothing in the
RMBCA that would prevent the board from making this decision. Since a quorum was present at the
board meeting, the majority vote was the authorized action of the board. Unless the general direction to
Adam is a violation of the board members’ duties of good faith and care, which the facts do not suggest
given the general briefing on opportunities in the sports field and Adam’s stock holdings, the delegation
to Adam as president is within the board’s power. The bylaws are silent as to whether an officer can
exercise such authority upon the delegation of the board, and therefore such a delegation would not be in
violation of RMBCA § 8.

Unlike the decision to redirect the business, the decision to sell FurnitureCo’s assets is not within Adam’s
authority as president, even as directed by the board of directors. Adam’s intended sale of FurnitureCo’s
current assets and use of the sale proceeds to buy a tennis club appears to be a sale of all, or substantially
all, of the corporation’s assets within the meaning of RMBCA § 12. Since the company has been in the
furniture business for 20 years, the sale is clearly not in the regular course of business. Therefore, the
board must vote to recommend the proposed sale to the shareholders and the shareholders must approve
the sale in order for the sale to be properly authorized. Since the board itself cannot alone authorize the
sale, the board cannot grant the president the authority to implement the sale without shareholder
approval.

Explanation to Point-Two (20-30%):

Seperac-J19 Exam-Released MEE Essay Compilation © 2016-2020 830


In their capacity as both directors and shareholders, Diane and Edward must be notified of and given the
opportunity to vote on the sale of FurnitureCo’s assets, although they don’t have enough votes to prevail.

The board of directors must vote to recommend the sale of all or substantially all of the assets of
FurnitureCo. No resolution authorizing the sale was passed at a regular meeting of the board, since the
general authorization by the board for Adam to study and implement a restructuring plan was insufficient
authorization. Board approval could be obtained either at the next regularly scheduled board meeting or, if
action was needed prior to the scheduled date of the regular meeting, at a special meeting of the board. It
is necessary to hold a board meeting even where, as on these facts, the board has the required number of
votes to approve recommending the sale to the shareholders.

In addition to board of director approval, the shareholders also must vote to approve the sale of all or
substantially all of the assets. Even though Adam owns 70% of the stock and will get the requisite
majority, the action cannot be taken without giving all of the shareholders the opportunity to vote. All
shareholders, including Diane and Edward, must be notified of a special meeting of the shareholders. The
notice must also state the purposes of the special meeting. The only exceptions to the notice requirement
are where a shareholder waives notice or joins in a unanimous written consent.

The sale of assets has not been properly structured to date, but ultimately Adam has the power to prevail
with his plan. First, Adam must get the board to recommend the proposed sale. The facts indicate that
Beth and Charles will vote with Adam to approve the sale. Furthermore, if they don’t vote with him,
Adam can vote as the 70% shareholder to remove them and replace them with directors who will. Second,
Adam will cause the corporation to give notice to the shareholders of a special meeting to vote on the sale
proposal. Since Adam holds 70% of the shares and will vote, the sale will be approved.

Explanation to Point-Three (20-30%):

Diane and Edward can exercise their dissenters’ rights and force FurnitureCo to buy their shares.

The sale of assets is “a sale or exchange of all, or substantially all, of the property of the corporation other
than in the usual and regular course of business,” which triggers dissenters’ rights. Dissenters’ rights are
the right of dissenting shareholders to force the corporation to purchase their shares at an appraised
market value. To exercise their rights, Diane and Edward must deliver to FurnitureCo notice of their
intent to dissent prior to the shareholder vote and must not vote in favor of the sale. Once the shareholder
vote authorizing the sale has occurred, FurnitureCo must deliver a dissenters’ notice to Diane and Edward
identifying the mechanics of the purchase of their shares. Diane and Edward must then demand payment
and deposit the shares within the statutory time period (generally within 30 to 60 days) after the vote.

Seperac-J19 Exam-Released MEE Essay Compilation © 2016-2020 831


#190-JUL 1998–MEE Q06: QUESTION SIX (CORPS-LLCS)

Acquiror, Inc., a publicly held corporation, entered into negotiations to buy the stock of XYZ Corporation
through a merger. Although the companies were close to a final agreement on September 1, Acquiror
insisted on delaying consummation and public announcement of the deal until September 15. XYZ agreed
to the delay.

Usually, a corporation such as Acquiror that makes a merger offer for another corporation can expect that
its stock price will decline after the announcement of the merger.

During the first week of September, while the merger was being delayed, XYZ’s stock price increased
and Acquiror’s stock price remained unchanged at $10 per share. Several press reports suggested that
Acquiror might be planning a merger with XYZ. On September 5, in response to specific inquiries from
financial reporters and from the stock exchange where Acquiror’s stock was listed for trading, Acquiror
denied that it had “any agreement to enter into a merger agreement” with XYZ. Investors believed the
statement, and the price of Acquiror’s stock did not decline but remained at $ 10 per share.

On September 15, Acquiror announced its merger with XYZ. As expected, Acquiror’s stock declined to
$8 per share.

1. Do investors who purchased Acquiror’s stock after Acquiror’s denial on September 5 and before
the disclosure of the merger have a claim for damages under federal law? Explain.

2. Do continuing shareholders in Acquiror have any basis under federal law to make a claim
against Acquiror or its officers and directors if, because of Acquiror’s statements, Acquiror is liable
to other investors of Acquiror’s shares? Explain.

Seperac-J19 Exam-Released MEE Essay Compilation © 2016-2020 832


#190: J98-6 MEE: ANSWER: NCBE (CORPS-LLCS)

POINT (1)(a) [33%] ISSUE: Does a corporation have an obligation to investors in general to give
full and truthful answers to questions about its plans with respect to potential mergers? ANSWER:
Yes. A corporation has an obligation to investors in general to give full and truthful answers to
questions about its plans with respect to potential mergers.

POINT (1)(b) [33%] ISSUE: Do investors who buy in reliance on incomplete and misleading
statements have a cause of action against the corporation making such statements? ANSWER: Yes.
Investors who buy or sell a corporation’s securities in reliance on incomplete and misleading
statements by the corporation have a cause of action against the corporation for making such
statements.

POINT (2) [33%] ISSUE: Do shareholders who were not traders in a corporation’s stock have a
cause of action, either direct or derivative, against the corporation or its officers and directors for
false and misleading statements that result in liability of the corporation to other shareholders?
ANSWER: Yes. A continuing shareholder who did not trade in a corporation’s stock has a
derivative cause of action against the corporation’s officers and directors for false and misleading
statements that result in economic loss to the corporation.

ANSWER EXPLANATION:

Explanation to Point-One(a) (30-40%):

A corporation has an obligation to investors in general to give full and truthful answers to questions about
its plans with respect to potential mergers.

Under Rule 10b-5 of the 1934 Securities Exchange Act, a public or private corporation must disclose the
material facts necessary in order to make its statements not misleading in light of the circumstances in
which they were made. Here, the merger would be a material fact and Acquiror’s statement would violate
this law. Acquiror’s statement that it did not have an agreement to acquire XYZ was at best misleading
because it omitted the fact that there was a clear tentative merger agreement with XYZ and a further
understanding that the final commitment to that agreement would come at the end of the waiting period.

The issue is whether Acquiror could do less than tell the entire truth once it undertook to respond to a
question. Acquiror’s statement was clearly incomplete and misleading because it omitted material
information about the existence as well as the status of the negotiations between Acquiror and XYZ.

A fact is considered to be material if a reasonable person would think such information was important in
determining a course of action, such as a purchase or sale of the corporation’s securities. Information
about a company’s preliminary negotiations for a merger may or may not be material depending on the
relative likelihood that the merger will be completed and on the size of the transaction compared to the
size of the company. Here, it is clear that the negotiations were nearly final, and there is nothing in the
facts to indicate that the size of the merger would be immaterial.

Information about the true status of a major purchase by Acquiror is material information. Moreover, in
light of Basic v. Levinson, the fact that full disclosure of the status of the negotiations might jeopardize
the ultimate outcome of the pending transaction would not permit a misleading statement. In the Basic

Seperac-J19 Exam-Released MEE Essay Compilation © 2016-2020 833


case, the Court suggested that a consistent policy of responding “no comment” to questions would protect
the parties. However, in this case Acquiror chose to respond to the questions, and its answers implied that
it had no discussions, and certainly had no nearly final merger agreement, with XYZ. The statement was
highly misleading even if it was literally correct in a very narrow sense.

Explanation to Point-One(b) (30-40%):

Investors who buy or sell a corporation’s securities in reliance on incomplete and misleading statements
by the corporation have a cause of action against the corporation for making such statements.

Investors who purchased Acquiror’s stock after the denial and before the announcement of the merger
have a claim against Acquiror for their losses under Rule 10b-5. The investors would claim that their
losses are the difference between the $10 price that the investors paid due to Acquiror’s denial of the
merger and the $8 price at which the market valued the shares in light of the merger. Investors must show
reliance of some type on the statements to establish a valid claim. Since the statement about Acquiror’s
plans for XYZ was made publicly, investors have a claim whether or not they heard the statement or
relied on it in deciding to buy Acquiror’s stock. Direct reliance is not essential; under the Basic decision, a
public misstatement by a company that has publicly traded securities which alters the market price of
those securities constitutes a “fraud on the market,” and all investors who purchased or sold stock are
presumed to have relied on the misstatement. Therefore, the investors can claim damages even if they
have not personally relied on the misstatement. However, if Acquiror can show that in fact the market
price was unaffected by the misstatement or that the particular investor’s purchase or sale would have
occurred regardless of market price, the presumption would be rebutted, and the investor would have no
claim because the deception would not have caused any loss.

Explanation to Point-Two (30-40%):

A continuing shareholder who did not trade in a corporation’s stock has a derivative cause of action
against the corporation’s officers and directors for false and misleading statements that result in economic
loss to the corporation.

Continuing shareholders ordinarily have no direct claim against the corporation, since they have neither
bought nor sold securities at the affected market price. However, it is also clear that the corporation has
suffered a loss as a result of the liability that it will incur to those investors who did buy or sell its stock
based on misleading information. Acquiror, as a corporation, has a claim against its officers and directors
who engaged in or approved of this misleading conduct as their conduct constitutes a breach of the duty of
care under state law. Shareholders of Acquiror can bring a shareholder’s derivative action, either
individually or as a class action, against Acquiror to hold the officers or directors of Acquiror responsible
for the misleading statements. Any recovery from the responsible officers or directors would be paid to
Acquiror for the benefit of all shareholders.

Seperac-J19 Exam-Released MEE Essay Compilation © 2016-2020 834


#191-FEB 1998–MEE Q06: QUESTION SIX (CORPS-LLCS)

Several years ago, Able, Baker, and Campbell properly incorporated Transport, Inc., a highway freight
hauling business. Able and Baker each own 45% of Transport stock, and Campbell owns 10%. Since its
incorporation, Transport has been quite profitable. However, most of its earnings have been retained to
help the business grow, and only small dividends have been paid to the three shareholders.

Able, the president of Transport, is in charge of finance and sales for the business. Baker, the vice
president, is in charge of operations. Able and Baker make all major decisions by consensus. Campbell is
an artist and does not participate in the business operations. No shareholder or director meetings have ever
been called or held. Able, who has expensive tastes and lives beyond his means, often uses corporate
funds of Transport to pay his personal bills, telling Baker that once he gets his personal finances in order
he will repay the company.

Last year, Able and Baker decided to expand Transport’s hauling business to start hauling hazardous
waste from local factories to a newly constructed hazardous waste disposal facility. Recognizing that
hauling hazardous waste could be a risky business, Able and Baker wanted to keep the hazardous waste
hauling activities separate from the rest of Transport’s business. They formed a new corporation called
HotTrucks, Inc., which was properly incorporated as a wholly owned subsidiary of Transport. Transport
contributed the use (but not ownership) of a fleet of ten trucks to HotTrucks. HotTrucks’ only asset was
its right to use the trucks. In order to save money, Able and Baker did not obtain general business liability
insurance for HotTrucks.

Able and Baker thought it wise not to be directors or officers of HotTrucks, so they asked Campbell to
serve as the sole director and officer of HotTrucks. Campbell did not want to spend time on business
matters when he could be working on his paintings. However, he agreed to serve as director and officer on
the understanding that Able and Baker would handle all day-to-day management and operation of the
business and that Campbell would not have to attend any directors’ meetings or make business decisions.

A month ago, one of the trucks operated by HotTrucks crashed through the front of a video store. Baker,
who was the driver of the truck, had negligently fallen asleep at the wheel. The regularly scheduled driver
had called in sick, and Baker had taken his place, not having slept for 20 hours. The accident seriously
injured five people.

1. On what basis, if any, can the injured persons hold Transport liable for tort claims resulting from
the HotTrucks accident? Explain.

2. On what basis, if any, can the injured persons hold Able, Baker, or Campbell personally liable
for such claims? Explain.

Seperac-J19 Exam-Released MEE Essay Compilation © 2016-2020 835


#191: F98-6 MEE: ANSWER: NCBE (CORPS-LLCS)

POINT (1) [42%] ISSUE: Do the facts justify piercing the corporate veil of HotTrucks in order to
hold its parent liable for tort claims of the subsidiary? ANSWER: Yes. A court is likely to pierce
the corporate veil of HotTrucks because certain formalities of corporate procedure have not been
followed, it is undercapitalized, and it is an alter ego or agent of Transport.

POINT (2)(a) [32%] ISSUE: Do the facts justify piercing the corporate veil of Transport?
ANSWER: Yes. A court may pierce the corporate veil of Transport because Transport failed to
follow certain corporate formalities in its operation of the business.

POINT (2)(b) [16%] ISSUE: If the court pierces the corporate veil of Transport, is each of Able,
Baker, or Campbell personally liable to the injured persons? ANSWER: If the corporate veil of
Transport is pierced. Able and Baker are likely to be the shareholders personally liable, but
Campbell, a passive shareholder, may not be held personally liable.

POINT (2)(c) [11%] ISSUE: Is Baker, as the negligent driver of the truck, protected from liability
because he was acting on behalf of the corporation? ANSWER: No. Baker, as the negligent driver
of the truck, is not protected from liability by virtue of his corporate position.

ANSWER EXPLANATION:

Explanation to Point-One (35-45%):

A court is likely to pierce the corporate veil of HotTrucks because certain formalities of corporate
procedure have not been followed, it is undercapitalized, and it is an alter ego or agent of Transport.

As a general rule, shareholders of a corporation are not personally liable for the debts of the corporation.
One exception to this general rule is the doctrine of piercing the corporate veil. Courts generally look to
see whether the corporation has been so dominated by an individual or another corporation that it may be
considered as the other’s “alter ego.”

When analyzing whether a parent corporation is liable for the obligations of its subsidiary, courts have
looked for the existence of the following circumstances:

• “The formalities of separate corporate procedures for each corporation are not observed.”

• “The corporation is inadequately financed as a separate unit “The courts analyze whether the corporation
is capitalized to meet “its normal obligations foreseeable in a business of its size and character.”

• “The policies of the corporation are not directed to its own interests primarily but rather to those of the
other corporation.”

Each of the three elements is present in the case of HotTrucks. HotTrucks did not observe required
corporate formalities such as conducting regular directors’ meetings. In addition, Campbell, who is listed
as the corporation’s officer and director, did not manage the business and in fact had nothing to do with it.
The facts indicate that Able and Baker, who actually managed Transport, the parent corporation, also
managed HotTrucks, the subsidiary corporation, even though they were not officers of HotTrucks.

Seperac-J19 Exam-Released MEE Essay Compilation © 2016-2020 836


Furthermore, HotTrucks was inadequately capitalized since its sole asset was the use of ten trucks. The
business of hauling hazardous waste is risky, and HotTrucks does not have assets sufficient to meet the
obligations and risks associated with the type of business in which it is engaged.

Finally, the policies of HotTrucks were directed at the interests of Transport not at its own independent
business interests. This is shown by the fact that no business insurance was obtained to cover its
operations and from the fact that the nature of HotTrucks’ business was to service certain high risk
activities in which Transport was unwilling to engage. The subsidiary corporation was, in substance, an
alter ego of Transport.

As a result, a court is likely to pierce the corporate veil and hold Transport liable for the obligations of
HotTrucks in connection with the truck accident.

Explanation to Point-Two(a) (25-35%):

A court may pierce the corporate veil of Transport because Transport failed to follow certain corporate
formalities in its operation of the business.

The standards for piercing the corporate veil are the same whether the shareholders are corporations or
individuals. As a general rule, “where corporate formalities are substantially observed, initial financing
reasonably adequate, and the corporation not formed to evade an existing obligation or a statute or to
cheat or to defraud, even a controlling shareholder enjoys limited liability.”

There is no evidence that Transport was inadequately capitalized since the business has been profitable
and most of its earnings were retained for use in the business, rather than distributed to the shareholders as
dividends. Transport was not set up for any fraudulent or illegal purpose. A desire by owners of a
business to benefit from the limited liability that a corporate form provides does not constitute an
improper purpose.

However, the shareholders of Transport failed to follow corporate formalities: shareholders’ and
directors’ meetings were not held; Able and Baker made all decisions by consensus without involving
Campbell; and Able did not treat his personal finances and those of Transport separately.

Courts are inconsistent in their approach to piercing the corporate veil when corporate formalities are
disregarded but the other elements of proper corporate existence are found. If the corporation is
adequately capitalized and there is no other evidence of inequity or fraud, some courts refuse to pierce the
corporate veil. Other courts pierce the corporate veil in cases like this because shareholders should not be
permitted to ignore the rules of corporate behavior and claim the advantage of the corporate shield.

Therefore, it is arguable but not certain that a court would pierce the corporate veil of Transport.

Explanation to Point-Two(b) (10-20%):

If the corporate veil of Transport is pierced. Able and Baker are likely to be the shareholders personally
liable, but Campbell, a passive shareholder, may not be held personally liable.

As a general rule, when courts pierce the corporate veil, all of the shareholders of that corporation are
liable. Some courts, however, have not extended liability to passive investors. There “is some authority,
for example, for piercing the corporate veil to hold shareholders active in the business personally liable,
but recognizing the same corporation’s separate existence to protect passive investors from the same

Seperac-J19 Exam-Released MEE Essay Compilation © 2016-2020 837


liability.” Protecting passive investors seems particularly appropriate if the corporate shield is being
pierced because the corporation is the alter ego or agent of dominant shareholders.

Able and Baker were active investors in Transport; they made all major corporate decisions by consensus.
Able and Baker each owned 45% of the shares of Transport and Able did not treat the finances of
Transport separately from his personal finances. In addition, the facts indicate that Transport was operated
as the agent of Able and Baker who were the dominant shareholders. This is shown by the fact that
shareholders’ and directors’ meetings were not called or held and that the decisions were made by Able
and Baker alone without any input from Campbell. This makes it even more likely that a court would
pierce the corporate veil of Transport and hold Able and Baker personally liable.

By contrast, Campbell was only a 10% shareholder and did not participate in the business in any way. As
a result of Campbell’s role as a passive investor, some courts would not hold Campbell personally liable
even if the corporate veil of Transport was pierced and Able and Baker were held personally liable.

Explanation to Point-Two(c) (05-15%):

Baker, as the negligent driver of the truck, is not protected from liability by virtue of his corporate
position.

The general rule is that if “the shareholder personally commits a tort while acting as an agent for his or
her corporation, he or she is personally liable for the tort because of the general rule that tortfeasors are
personally liable even though they are acting as agents.” Likewise, an “agent who does an act otherwise a
tort is not relieved from liability by the fact that he acted at the command of the principal or on account of
the principal.”

Baker was negligent when he fell asleep while driving the truck for HotTrucks that was involved in the
accident. He cannot argue that he is not personally liable because he was acting as an employee of
HotTrucks. Because Baker was negligent, he is liable for the accident whether or not the corporate veil of
Transport is pierced.

Seperac-J19 Exam-Released MEE Essay Compilation © 2016-2020 838


#192-JUL 1997–MEE Q06: QUESTION SIX (CORPS-LLCS)

Chempro is a closely-held chemical company incorporated in State F. In addition to a number of smaller


shareholders, Chempro has five principal shareholders each of whom owns 15% of the shares: Paula, who
is Chempro’s Chief Executive Officer and Chairwoman of the Board; and Lawrence, Alan, Bruce, and
Charles, who are also officers and directors of Chempro. Although each is an officer of Chempro, none of
the five principal shareholders has an employment contract with the company.

The five shareholders signed a valid, enforceable shareholders’ agreement requiring all of them to vote to
elect each other as directors of Chempro.

For many years, Chempro’s business prospered, and its value increased substantially. However, Lawrence
recently became aware that Paula was diverting corporate funds into her personal bank account. Shortly
after his discovery, Lawrence took Paula aside, confronted her with evidence of her wrongdoing from the
company’s files, and demanded that she repay the funds to Chempro. Paula responded by calling a board
of directors’ meeting to discuss dismissing Lawrence from the board of directors and firing Lawrence
from his position as an officer of Chempro. Paula also immediately removed all of the incriminating files
from Lawrence’s office.

At the board meeting, Paula claimed that Lawrence had been taking corporate funds and using them for
personal reasons. Lawrence responded that Paula was lying and that, in fact, Paula was the responsible
party. Lawrence also told the board that he had documents to prove his story but that they had recently
been stolen from his office. Without further investigation, Paula and the remaining board members, Alan,
Bruce, and Charles, all of whom never really liked Lawrence, voted to remove Lawrence as both an
officer and a director of Chempro. Lawrence dissented.

The State F corporate code and Chempro’s articles of incorporation and by-laws permit directors and
officers to be removed by the board without cause.

1(a). What claims, if any, does Lawrence have against Chempro, Paula, Alan, Bruce, and/or
Charles in Lawrence’s capacity as a director? Explain.

1(b). What claims, if any, does Lawrence have against Chempro, Paula, Alan, Bruce, and/or
Charles in Lawrence’s capacity as an officer? Explain.

1(c). What claims, if any, does Lawrence have against Chempro, Paula, Alan, Bruce, and/or
Charles in Lawrence’s capacity a shareholder? Explain.

2. Are Lawrence’s claims derivative or direct, and, as to any derivative claims, are there any
procedural prerequisites he must satisfy before filing the derivative claims in court? Explain.

Seperac-J19 Exam-Released MEE Essay Compilation © 2016-2020 839


#192: J97-6 MEE: ANSWER: NCBE (CORPS-LLCS)

POINT (1) [48%] ISSUE: Does Lawrence have a claim against Chempro, Paula, Alan, Bruce, or
Charles as a result of (a) his dismissal as a director, (b) his dismissal as an officer, or (c) the board’s
failure to investigate the claims against Paula? ANSWER: Yes. (a) Lawrence has a valid claim for
unlawful dismissal as a director: (b) Lawrence does not have a valid cause of action for unlawful
dismissal as an officer: and (c) Lawrence does have a valid claim for failure to investigate his claims
against Paula.

POINT (2) [29%] ISSUE: Are the following claims by Lawrence derivative or direct claims: (a)
unlawful dismissal as a director; and (b) failure to investigate the claim that Paula had stolen
company property? ANSWER: Lawrence’s claim for removal as a director results from fee breach
of the shareholders’ agreement and is a direct claim against the shareholders who are parties to the
shareholders’ agreement. Lawrence’s claim that Paula diverted corporate funds for her own
personal use is a derivative claim because the claim alleges harm to the corporation.

POINT (3) [24%] ISSUE: Is Lawrence required to make a demand on the board prior to bringing a
derivative action or will he be excused from this requirement because the Chempro board of
directors either is interested in the transaction or has breached its duty of care? ANSWER: Under
the RMBCA, demand would be required prior to commencing any derivative action. Under
common law, demand is not required in this case because Paula is interested in the transaction and
the other members of the board breached their duty of care by failing to investigate Lawrence’s
allegations against Paula. This creates a reasonable doubt that the board can fairly determine
whether the corporation should pursue the case.

ANSWER EXPLANATION:

Explanation to Point-One (45-55%):

(a) Lawrence has a valid claim for unlawful dismissal as a director: (b) Lawrence does not have a valid
cause of action for unlawful dismissal as an officer: and (c) Lawrence does have a valid claim for failure
to investigate his claims against Paula.

The facts state that the State F corporations law and the Chempro articles and by-laws permit directors to
be removed by the board without cause. Therefore, without other special circumstances, Lawrence would
have no cause of action against either Chempro or the other shareholders resulting from his dismissal as
director. However, Lawrence, Paula, Alan, Bruce, and Charles had entered into a shareholders’ agreement
which required each of the other shareholders to vote for Lawrence as director. A shareholders’ agreement
requiring the parties to vote for specified directors generally is enforceable and read to bar a vote to
remove such directors, except for cause. The shareholders’ agreement created a binding contractual
obligation on the other shareholders to vote for Lawrence as a director of Chempro. The board of directors
vote by Paula, Alan, Bruce, and Charles to dismiss Lawrence as director was a breach of that
shareholders’ agreement. Since the board had not determined whether Paula’s claims against Lawrence
were true, they would not be entitled to terminate Lawrence for cause. Therefore, Lawrence has a claim
against each of the other principal shareholders individually for breach of contract. The remedy may be
either damages or specific performance of the shareholders’ agreement.

Seperac-J19 Exam-Released MEE Essay Compilation © 2016-2020 840


Lawrence does not have a claim against either Chempro or the other principal shareholders as a result of
his dismissal as an officer of Chempro. Generally, an officer is selected by the board of directors and may
be removed by the board for any reason or for no reason. The primary exception, where the officer has
been selected by the shareholders, does not apply here. Even where an officer has an employment contract
with the company, the board retains the right to remove the officer. However, in that circumstance, the
officer may have a breach of contract claim against the company. The facts state that Lawrence did not
have an employment agreement with Chempro. Chempro’s articles and by-laws permit officers to be
removed by the board without cause. The board acted within its rights when it dismissed Lawrence as an
officer of Chempro.

Lawrence does have a valid claim arising from the board of directors’ failure to investigate the claims
against Paula. A director owes the corporation a fiduciary duty. This duty has been interpreted to mean
that he must discharge his duties “in good faith, with the care an ordinarily prudent person in a like
position would exercise under similar circumstances and in a manner he reasonably believes to be in the
best interests of the corporation.” A director is entitled to rely on information from officers or employees
whom he believes to be reliable. However, a director is not acting in good faith if he has knowledge of the
matter at issue which makes his reliance unjustified. In general, the duty of care requires that a director
take steps to become informed about the facts before taking any action.

Here, the directors of Chempro have a fiduciary duty to investigate the claims made by both Paula and
Lawrence since they each allege harm to the corporation. In this case, Paula clearly breached her fiduciary
duties since she knew that the allegations against Lawrence were false. Alan, Bruce, and Charles also
breached their fiduciary duties because they believed Paula’s allegations and ignored Lawrence’s
conflicting claims without any supporting information. Failure to conduct an investigation into the truth of
the matter is a breach of the directors’ fiduciary duties. As a result, Lawrence has a claim against the
directors for breach of their fiduciary duties.

Explanation to Point-Two (25-35%):

Lawrence’s claim for removal as a director results from fee breach of the shareholders’ agreement and is a
direct claim against the shareholders who are parties to the shareholders’ agreement. Lawrence’s claim
that Paula diverted corporate funds for her own personal use is a derivative claim because the claim
alleges harm to the corporation.

When harm is done to the corporation, a shareholder may bring a derivative action on behalf of the
corporation, with recovery going to the corporation. A breach of the directors’ fiduciary duties that
depletes or diverts corporate assets and injures the corporation, including all of the shareholders, gives
rise to a derivative claim. Lawrence’s claim that Paula diverted corporate assets for her own personal use
would be a derivative claim because the harm from Paula’s actions is damage to the corporation as a
whole rather than to Lawrence individually.

By comparison, a wrongful act that does not deplete the corporate treasury or divert corporate assets but
instead deprives a shareholder of a right to which his shares entitle him, or which is personal to the
shareholder and not shared by other shareholders, gives rise to a direct claim. Lawrence’s claim that he
lost his job as a director is a claim for a violation of the shareholders’ agreement. Lawrence’s rights under
the shareholder agreement are individual. Therefore, this claim should be brought against the other
shareholders in their capacities as shareholders, that is, as a direct claim.

Explanation to Point-Three (20-30%):

Seperac-J19 Exam-Released MEE Essay Compilation © 2016-2020 841


Under the RMBCA, demand would be required prior to commencing any derivative action. Under
common law, demand is not required in this case because Paula is interested in the transaction and the
other members of the board breached their duty of care by failing to investigate Lawrence’s allegations
against Paula. This creates a reasonable doubt that the board can fairly determine whether the corporation
should pursue the case.

Shareholders who bring a derivative action are suing to enforce the corporation’s rights when the
corporation has failed to do so itself. Therefore, the shareholders must allege that they have made a
demand on the corporation to file the suit.

The RMBCA requires that a shareholder make a written demand on the board in all circumstances before
commencing a derivative action. The shareholder must then wait 90 days after making the demand to file
the derivative action, unless the board rejects the demand during the 90-day period.

However, under the common law, demand need not be made when it would be futile to do so. To
establish demand futility, it is insufficient for the shareholder to allege in a conclusory manner that the
suit names all of the directors as defendants and that, therefore, they are incapable of considering the
demand in a disinterested manner. However, if the shareholder can allege facts in its complaint that
demonstrate a reasonable basis for the belief that the members of the board are interested in the
transaction being challenged, or that they failed to exercise their duty of care in approving the matter at
issue, that will be enough to show demand futility.

Here Lawrence’s claim is that the other board members did not investigate his charges that Paula was
stealing corporate funds and instead accepted Paula’s claim that Lawrence was responsible for the thefts
without any investigation of the matter. At common law, this would probably be sufficient to excuse the
making of a demand and Lawrence could immediately file his derivative action.

Seperac-J19 Exam-Released MEE Essay Compilation © 2016-2020 842


#193-JUL 1996–MEE Q07: QUESTION SEVEN (CORPS-LLCS)

Triton Steel is a State X corporation that manufactures high quality steel at its steel mill located in State
X. Five years ago, Triton was the subject of an employee buyout. As a result, the corporation has two
classes of stock: Class A and Class B. Triton Class A shares are publicly owned, although ownership is
concentrated in fewer than 100 sophisticated investors and the stock is not traded on any established
market. There are 7 million shares of Class A authorized, issued, and outstanding. Triton Class B shares
are held by an employee stock ownership plan of the Triton employees, which votes all of the Class B
shares as a block. There are 3 million shares of Class B authorized, issued, and outstanding.

Triton's articles of incorporation contain two special provisions negotiated as part of the employee buyout.
The first requires an affirmative vote of each class of stock before Triton can invest in steel production
facilities located outside of State X. The second special provision adopts class voting for directors, with
Class A shareholders electing seven directors and the Class B shareholder electing three directors.

The investors and the employees disagree over the future of Triton. Triton has posted modest profits over
the past two years. However, the Class A investors believe that for Triton to be profitable in the long
term, production costs have to be cut substantially. They want Triton to construct a steel mill in State Z
where costs are lower and, over time, to shift all production to the new facility. However, the employees
believe that Triton can increase its profitability through other measures that do not require relocating its
steel production facilities.

The board of directors recently voted 7 to 3 to approve the proposal to construct a new steel mill in State
Z, with all seven Class A directors voting in favor and all three Class B directors voting against. However,
when the proposal was brought to the Triton shareholders for a vote, the Class B shareholder voted all of
the Class B shares against it and the proposal was defeated. The Class A shareholders are frustrated
because, although they have a 7 to 3 majority on the board of directors and a 70% to 30% majority of the
total outstanding stock, they have been unable to cut Triton's production costs by moving the steel mill to
State Z.

The Class A shareholders are considering the following three alternatives:

First, attempt to change Triton's articles of incorporation to allow the out-of-state production facility.
Have the Class A directors, acting either alone or with the 70% Class A shareholder majority, amend the
articles of incorporation to delete the provision that requires special voting in order to move the steel mill.

Second, if the suggested change to the articles of incorporation does not work, realize Triton's value as a
going concern. Have the corporation sell all of its assets to an independent third party for a mixture of
cash and stock. Even if the Class B shareholder dissents, it will have no option but to go along with the
Class A vote.

Third, if a going-concern sale cannot be accomplished, have the Class A shareholders dissolve the
corporation.

Discuss whether each of the three alternatives can be successfully implemented. For each alternative that
can be implemented, what rights will the Class B shareholder have? Explain.

Seperac-J19 Exam-Released MEE Essay Compilation © 2016-2020 843


#193: J96-7 MEE: ANSWER: NCBE (CORPS-LLCS)

POINT (1) [32%] ISSUE: Can a majority of the board of directors, acting either alone or with a
majority of all the shareholders voting as a single class, amend the articles of incorporation to the
disadvantage of the Class B shareholder and over its objection? ANSWER: The proposed
amendment to the articles of incorporation is beyond the power of the board of directors to be
done. However, it could be implemented by a majority of the board and a majority of each class of
the shareholders.

POINT (2) [42%] ISSUE: Can a majority of the board of directors and a majority of all the
shareholders voting as a single class sell a corporation's assets to a third party? If so, what recourse
would the minority shareholders have? ANSWER: A majority of the board of directors and a
majority of all the shareholders voting together could authorize the sale of Triton's assets to a third
party. However, such a sale would trigger dissenters' rights for the minority.

POINT (3) [26%] ISSUE: Can a majority of the board of directors and a majority of all the
shareholders voting as a single class dissolve a corporation? ANSWER: Yes. A majority of the
board of directors and a majority of all the shareholders voting together could authorize a
dissolution of the corporation.

ANSWER EXPLANATION:

Explanation to Point-One (25-35%):

The proposed amendment to the articles of incorporation is beyond the power of the board of directors to
be done. However, it could be implemented by a majority of the board and a majority of each class of the
shareholders.

A corporation may change its articles of incorporation at any time to delete a provision not required to be
included in the articles. Because the class vote for out-of-state production facilities is not required to be in
the articles of incorporation, it could be deleted. The Class B shareholder has no vested right in that
provision of the articles.

The directors do have the right to make some changes in the articles of incorporation without the approval
of the shareholders, but the proposed change is not on that short list. However, the contemplated change
could be made by the directors acting in concert with the shareholders. Such an amendment requires the
approval of a majority of the board of directors and a majority of the shareholders. Class voting of the
shareholders is not required under the amendment section unless required by the articles, provided by the
board of directors, or otherwise required under the statute. The articles themselves in this case do not
require class voting on the amendment, and the directors would not require class voting.

Notwithstanding the above, another section of the statute requires class voting in this situation: for class
voting when a proposed amendment would “change the rights of all or part of the shares of the class “
Since the proposed amendment would remove class voting on questions of out-of-state production
facilities it would clearly diminish the rights of the Class B shareholder. As a result, in order to be
adopted, the amendment would require an affirmative vote of Class A and Class B shareholders voting as
separate classes. Since the facts indicate that all of the Class B shares will be voted to oppose the

Seperac-J19 Exam-Released MEE Essay Compilation © 2016-2020 844


amendment, the amendment will not pass, and the restriction on out-of-state production facilities cannot
be eliminated from the articles of incorporation.

Explanation to Point-Two (35-45%):

A majority of the board of directors and a majority of all the shareholders voting together could authorize
the sale of Triton's assets to a third party. However, such a sale would trigger dissenters' rights for the
minority.

A corporation can sell all or substantially all of its assets. The RMBCA differentiates between sales of all
or substantially all of the assets of a corporation made in the usual and regular course of business and such
sales not in the usual and regular course of business. The former may be done without the approval of the
shareholders, unless otherwise required by the articles of incorporation. The latter requires the approval of
the shareholders. Since Triton is a steel manufacturer, it is clear that a sale of all or substantially all of its
assets is not within its usual and regular course of business, and therefore shareholder approval is
required. Class voting, however, is not required. The statute requires class voting only where it is
provided in the articles of incorporation, by the board of directors, or where the statute otherwise requires
it. Because none of these circumstances exist in this situation, the sale of all or substantially all of the
assets of Triton must be approved only by a majority of all the shares voting together. Here, this
requirement will be achieved easily, since the 7 million Class A shares held by the investors will outvote
the 3 million Class B shares. Therefore, the investors could force a sale of the assets to a third party over
the objection of the Class B shareholder.

However, such a sale would trigger the dissenters' rights of the minority. The statute provides “a
shareholder is entitled to dissent from, and obtain payment of the fair value of his shares in the event of
consummation of a sale of all, or substantially all, of the property of the corporation other than in the
usual and regular course of business.” There are exceptions where the sale is pursuant to a court order and
where the sale is for cash proceeds to be distributed within one year, but neither is applicable here, since
there is no court order and the sale is for cash and stock.

The availability of dissenters' rights complicates matters for both sides. On the investors' side, dissenters'
rights raise problems of liquidity and price. Since the dissent process ends with a cash payment to the
dissenters, the investors would have to consider whether the third-party transaction would leave the
corporation liquid enough to pay the dissenters. There are also price concerns: The price paid to the
dissenters is “fair value,” defined as “the value of the shares immediately before the effectuation of the
corporate action to which the dissenter objects, excluding any appreciation or depreciation in anticipation
of the corporate action unless exclusion would be inequitable.” The corporation makes a tender of the
amount it calculates to be the fair value, but if the dissenter is dissatisfied the dissenter may counter, and
ultimately if no agreement is reached the matter will end up before the court for a judicial appraisal. The
process can obviously generate a price in excess of that generated by the third-party agreement, which as
a realistic matter probably sets a floor price, and the loss is suffered by the non-dissenting shareholders.

There is also uncertainty on the minority side. Statutory dissenters' rights are an exclusive remedy: “A
shareholder entitled to dissent and obtain payment for his shares under this chapter may not challenge the
corporate action creating his entitlement unless the action is unlawful or fraudulent with respect to the
shareholder or the corporation.” Thus the Class B shareholder could not sue to block the sale, and
preserve the employees’ jobs, on contract or breach of fiduciary duty grounds while pursuing its
dissenters' remedies.

Explanation to Point-Three (20-30%):

Seperac-J19 Exam-Released MEE Essay Compilation © 2016-2020 845


A majority of the board of directors and a majority of all the shareholders voting together could authorize
a dissolution of the corporation.

The voluntary dissolution of a corporation generally must be recommended by the board of directors and
approved by the shareholders. Shareholder voting on a proposed dissolution is not by class, unless class
voting is required by the articles of incorporation or provided by the board of directors. In this case,
neither the articles nor the directors require class voting so the dissolution of Triton must only be
approved by a majority of all the shares voting together. Therefore, the Class A directors would have
sufficient votes to recommend dissolution and the Class A shareholders would have enough votes to
approve the dissolution. Dissolution does not trigger dissenters' rights, since the dissolution process
contemplates a liquidation of the corporate assets and distributions to the shareholders.

Dissolution is not an attractive option, for either the Class A or Class B shareholders, since the price
generated for the Triton assets, even if sold as a unit, would likely be lower than the going-concern price
which would be generated by a non-dissolution sale.

Seperac-J19 Exam-Released MEE Essay Compilation © 2016-2020 846


#194-JUL 1995–MEE Q04: QUESTION FOUR (CORPS-LLCS)

ZuderCo is a corporation that has owned and operated the historic Palms Hotel and two smaller hotels
since 1914. The Palms Hotel constitutes approximately 50% of the fair market value of the total assets of
ZuderCo.

ZuderCo has 15 shareholders. Thirteen of them are descendants of the founder, and the remaining two are
Able and Baker. No shareholder owns more than 10% of the outstanding shares. ZuderCo has a three-
person board of directors, consisting of Able, Baker, and Chase (a Zuder family member).

Able and Baker believe that the property on which the Palms Hotel is located has great potential for
development as an office park. They value it at $18 million based on their own close study of public
documents relating to the development pattern in that area. The other ZuderCo shareholders disagree
because they believe that the Palms property has greater economic potential as a hotel. They arrive at a
more modest valuation of about $13 million for the property's use as an office park. Able and Baker have
obtained three independent appraisals placing the value of the Palms property as an office park at between
$14 million and $18 million.

Able and Baker decide to offer ZuderCo $14.5 million to buy the Palms property, and then to vote as
directors at the next board meeting to accept the offer. They do not plan to approach the third director,
Chase, before the meeting. They expect that Chase will go along with their plan without asking any
questions or causing delay because Able and Baker will have their two votes in favor of the sale and
because the $14 million appraisal will support the amount of their offer. They do intend to disclose the
$14 million appraisal to Chase, but they do not intend to disclose the other two higher appraisals.

The relevant corporate documents for ZuderCo contain no special or extraordinary provisions directly on
point as to the following questions:

(a) Can the board of directors of ZuderCo authorize the sale by its unilateral action? Explain.

(b) What disclosures, if any, must Able and Baker make at the board meeting? Explain.

(c) Will the votes of Able and Baker be sufficient to approve the transaction? Explain.

(d) What duty, if any, does Chase have as a director, in light of the sale proposal presented by Able
and Baker? Explain.

Seperac-J19 Exam-Released MEE Essay Compilation © 2016-2020 847


#194: J95-4 MEE: ANSWER: NCBE (CORPS-LLCS)

POINT (1) [14%] ISSUE: Can the board of directors authorize the sale of approximately 50% of
the corporation's assets without shareholder approval? ANSWER: No. Shareholder approval is not
required for the contemplated sale because the Palms does not constitute "substantially all" of
ZuderCo's property.

POINT (2) [22%] ISSUE: Must self-interested directors disclose all third- party appraisals or other
independent information to the board of directors? ANSWER: Yes. Because of their "conflicting
interests," Able and Baker must disclose the basis of their higher valuation to the board of
directors.

POINT (3) [32%] ISSUE: Can self-interested directors vote on an asset sale? ANSWER: No. As
"nonqualified directors," Able and Baker cannot participate in the board of directors' vote on the
sale proposal.

POINT (4) [32%] ISSUE: What duty does a non-self-interested director have to the corporation
when faced with a proposed sale of corporate assets to insiders? ANSWER: The qualified director,
Chase, has a duty of care to become informed and to act in the best interests of the corporation.

ANSWER EXPLANATION:

Explanation to Point-One (10-15%):

Shareholder approval is not required for the contemplated sale because the Palms does not constitute
"substantially all" of ZuderCo's property.

The organizational documents of ZuderCo do not modify the statutory provisions on shareholder
approvals of asset sales. Under the Revised Model Business Corporation Act (hereinafter “RMBCA”),
shareholder approval is required only for the corporation to “sell, lease, exchange, or otherwise dispose of
all, or substantially all, of its property otherwise than in the usual and regular course of business.”

The contemplated sale does not appear to be in the usual and regular course of ZuderCo's business
because it has owned and operated the Palms property as a hotel since 1914. However, because the Palms
is valued at only approximately 50% of the assets of ZuderCo, it does not constitute “all or substantially
all” of the property of the corporation. Under ordinary circumstances, therefore, shareholder approval is
not required, and the board of directors acting alone may authorize the sale.

However, as is developed in Point Three, the existing size and composition of ZuderCo's board makes
board approval in this case impossible. In such a case, one of the available options to approve the
transaction is to seek full shareholder approval.

Explanation to Point-Two (15-25%):

Because of their "conflicting interests," Able and Baker must disclose the basis of their higher valuation
to the board of directors.

Seperac-J19 Exam-Released MEE Essay Compilation © 2016-2020 848


Under the RMBCA, a director has a “conflicting interest” in a transaction if the director either is a “party
to the transaction,” or “has a beneficial financial interest in the transaction of such financial significance
to the director that the interest would reasonably be expected to exert an influence on the director's
judgment if called upon to vote on the transaction.” Both Able and Baker have a “conflicting interest” in
the contemplated transaction under both of these tests and, therefore, the transaction is deemed to be a
“director's conflicting interest transaction.”

Since the contemplated transaction is a “director's conflicting interest transaction,” the board cannot vote
to authorize the transaction unless the interested directors have made certain required disclosures to the
entire board of directors. The required disclosures include all “facts known to him respecting the subject
matter of the transaction that an ordinarily prudent person would reasonably believe to be material to a
judgment about whether or not to proceed with the transaction.” It is clear that the three appraisals –
especially the appraisals that exceed the amount offered by Able and Baker – are material within the
meaning of the provision. It is also arguable that the public record information regarding the development
pattern of the community would be material within this standard.

Explanation to Point-Three (25-35%):

As "nonqualified directors," Able and Baker cannot participate in the board of directors' vote on the sale
proposal.

Able and Baker's participation in the proposed sale calls into question the duty of loyalty they owe to
ZuderCo. Under common law, a director is charged generally to act in a way “he reasonably believes to
be in the best interests of the corporation.” With respect to business dealings with the corporation, the
common law imposes a specific duty of “fair dealing.” The fair dealing standard requires that the
transactions be “fair to the corporation when entered into,” unless it is approved by disinterested
authorities within the corporation. The statutory obligation, as articulated in the RMBCA, requires the
director to discharge the duties of a director “in a manner he reasonably believes to be in the best interests
of the corporation.” Since Able and Baker truly believe the Palms property to be worth substantially more
than the offer price, they cannot vote as directors to accept the offer.

If the interested directors make the required disclosure, the qualified members of a board of directors can
ordinarily act to authorize a director's conflicting interest transaction. A “qualified director” is a director
who does not have a conflicting interest with respect to the transaction and does not have a familial or
other relationship with a director that has a conflicting interest. The problem for Able and Baker is that
such approval requires “the affirmative vote of a majority (but no fewer than two) of those qualified
directors on the board of directors.” Because Able and Baker are not “qualified directors,” and Chase is
the sole qualified director, the board of directors cannot properly authorize the transaction. Note that some
states (including Delaware) require the vote of a majority of disinterested directors, but such majority may
be only one director.

Explanation to Point-Four (25-35%):

The qualified director, Chase, has a duty of care to become informed and to act in the best interests of the
corporation.

Chase, as a director, has obligations shaped by the common law and by statute. The common-law
obligation is articulated in the business judgment rule. As a disinterested director, Chase is obligated to
become “informed with respect to the subject of his business judgment to the extent he reasonably
believes to be appropriate under the circumstances.” . The director's decision meets the business judgment

Seperac-J19 Exam-Released MEE Essay Compilation © 2016-2020 849


rule standard if “he rationally believes that his business judgment is in the best interests of the
corporation.” The statutory obligation requires the director to discharge the duties of a director “in good
faith,” “with the care an ordinarily prudent person in a like position would exercise under similar
circumstances,” and “in a manner he reasonably believes to be in the best interests of the corporation.”

The combined standard is context-sensitive. In this case, Chase faces a proposed sale of an asset worth
approximately 50% of the fair market value of the total assets of the corporation. The sale is to individuals
with access to ZuderCo's most confidential and proprietary information. Compliance with the common-
law and statutory standards would clearly require additional inquiry into the offer, regardless of Able and
Baker's indication that they will vote in favor of the proposal.

There is no indication that Chase is financially interested in the transaction beyond the interest of a
shareholder in ZuderCo, as would be the case if Chase had been promised employment by Able and Baker
following the transaction. Thus, there is no indication that the duty of loyalty owed by Chase to the
corporation is implicated.

Seperac-J19 Exam-Released MEE Essay Compilation © 2016-2020 850


FAMILY LAW: 39 OF 49 MEE EXAMS: (80%)
#195-JUL 2019–MEE Q05: QUESTION FIVE (FAMILY LAW)

Twelve years ago, Wendy and Frank were married in State A. One year later, their daughter, Danielle,
was born in State A. The couple and their daughter have continued to live in State A.

One year ago, Frank lost his job as a steelworker after suffering a serious back injury. Frank’s doctor has
said that he will not be able to return to work.

One month ago, Frank filed an action against Wendy seeking spousal support. Frank filed the action after
Wendy, a commercial airline pilot whose work frequently necessitates her absence from home, stopped
depositing her wages into the couple’s joint bank account and refused to pay household bills. Frank’s
unemployment insurance is inadequate to pay all the household bills.

Danielle’s school recently sent her parents a note indicating that Danielle will not be allowed to enroll in
school next year unless the parents provide proof of her vaccination. Frank, based on his personal,
nonreligious beliefs, has consistently refused to allow Danielle to receive any vaccinations. Danielle does
not satisfy the requirements for a medical exemption. State A has amended its mandatory vaccination law
by eliminating all nonmedical exemptions based on “personal beliefs.” As amended, the law requires, as a
precondition to a child’s enrollment in any public school, that “the child’s parent or guardian must
provide proof that the child has received all vaccinations mandated by the State Department of Health.”
Frank has brought an action challenging the State A vaccination law under the U.S. Constitution as a
violation of his parental rights.

Two weeks ago, Danielle, age 11, with her parents’ permission, went to visit her aunt in State B. One
week into the visit, the aunt called Frank and Wendy and told them that Danielle did not want to return to
her parents’ home because “Mom is always traveling, Dad is really depressed since his back injury, and I
just can’t stand living there anymore.” The aunt told Frank and Wendy that “I can’t in good conscience
send her home, so I’m immediately going to court to seek legal custody.”

1. May Frank obtain spousal support from Wendy? Explain.

2. Will Frank’s constitutional challenge prevail? Explain.

3. In what state must the aunt file a custody petition? Explain.

4. Is the court likely to grant legal custody of Danielle to her aunt? Explain.

Seperac-J19 Exam-Released MEE Essay Compilation © 2016-2020 851


#195: J19-5 MEE: ANSWER: NCBE (FAMILY LAW)

POINT (1) [20%] ISSUE: May Frank obtain spousal support from Wendy? ANSWER: No.
Because Frank and Wendy are not separated, Frank may not obtain a support order against
Wendy.

POINT (2) [30%] ISSUE: Will Frank’s constitutional challenge prevail? ANSWER: No. Although a
parent’s right to the care, custody, and control of his or her child is constitutionally protected, that
right does not provide a basis for overturning a state vaccination mandate.

POINT (3) [20%] ISSUE: In what state must the aunt file a custody petition? ANSWER: Under
both the UCCJEA and PKPA, only a child’s “home state” may exercise jurisdiction over an initial
child-custody petition. Here, because State A is Danielle’s home state, only a State A court may hear
the aunt’s custody petition.

POINT (4) [30%] ISSUE: Is the court likely to grant legal custody of the daughter, Danielle, to her
aunt? ANSWER: No. It is highly unlikely that the court will grant the aunt’s petition for custody,
given the constitutionally based parental preference.

ANSWER DISCUSSION:

Frank may not obtain a spousal support order against his wife, Wendy. In the absence of a contrary
statute, courts typically do not order support in the context of an intact, co-residential marriage. Although
parents have a constitutional right to the care, custody, and control of their children, including the right to
make most health-care decisions, the state may, pursuant to its police-power authority, mandate
vaccination in order to protect public health. Thus, Frank will not succeed in his action to overturn the
state vaccination requirement. Under the UCCJEA and PKPA, applicable in all states, an initial child-
custody action must be brought in a child’s “home state,” i.e., the state where she has lived for the
preceding six months. Thus, the aunt’s custody petition must be brought in State A. It is highly unlikely
that a court will grant the aunt’s custody petition. In determining child custody, all states today grant a fit
legal parent preference over a nonparent, an approach that is almost certainly mandated under Troxel v.
Granville. Although a child’s preference is relevant in a custody contest, Danielle’s desire to live with her
aunt should not provide a sufficient basis for an award of custody to the aunt.

ANSWER EXPLANATION:

Explanation to Point One (20%):

Because Frank and Wendy are not separated, Frank may not obtain a support order against Wendy.

In all states, marriage establishes a mutual support obligation between spouses. However, the spousal
support obligation is limited by the common law doctrine of nonintervention, which disallows judicial
intervention in an intact family. Although the case law is sparse, courts have relied on the nonintervention
principle to deny a support petition when the couple is living together. Thus, until and unless Frank and
Wendy separate, a court will not intervene in their private affairs. The fact that Wendy is frequently
absent for work-related reasons does not constitute a separation. There is nothing in the facts to suggest
that Wendy has moved or even that she has a second residence.

Seperac-J19 Exam-Released MEE Essay Compilation © 2016-2020 852


[NOTE: Were Frank to purchase necessary household items on credit, the merchant who extended credit
to him might be able to recover from Wendy the value of the goods furnished under the common law
necessaries doctrine. The necessaries doctrine is generally applicable to food, clothing, shelter, and
health care. Here, however, there appears to be no merchant who has furnished goods to Frank on credit.
Therefore, any discussion of the doctrine of necessaries is inapplicable.]

Explanation to Point Two (30%):

Although a parent’s right to the care, custody, and control of his or her child is constitutionally protected,
that right does not provide a basis for overturning a state vaccination mandate.

The Supreme Court of the United States has held that the parental right to the care, custody, and control of
a child is constitutionally protected under the 14th Amendment. Courts have recognized the fundamental
right of parents to make decisions concerning the care, custody, and control of their children.

However, parental rights are not absolute. “The power of the parent, even when linked to a free exercise
claim, may be subject to limitation if it appears that parental decisions will jeopardize the health or safety
of the child, or have a potential for significant social burdens.” The spread of preventable, contagious
illnesses is a “significant social burden.”

Moreover, the Supreme Court has specifically held both that a vaccination mandate is within the state’s
police power to protect the public health and that a state may refuse school admission to a student who
fails to receive a vaccination as mandated. Recent challenges to state rules that disallow school attendance
by unvaccinated students have thus been summarily dismissed.

Explanation to Point Three (20%):

Under both the UCCJEA and PKPA, only a child’s “home state” may exercise jurisdiction over an initial
child-custody petition. Here, because State A is Danielle’s home state, only a State A court may hear the
aunt’s custody petition.

Under the Uniform Child Custody Jurisdiction and Enforcement Act (UCCJEA), which has been adopted
in all but one state (Massachusetts), a court may exercise jurisdiction over a petition for child custody
only if “this State is the home State of the child on the date of the commencement of the proceeding, or
was the home State of the child within six months before the commencement of the proceeding and the
child is absent from this State but a parent or person acting as a parent continues to live in this State” and
no other state’s courts would have jurisdiction under the above standard or other courts having
jurisdiction have declined to exercise it. A “home state” is the state in which the child “lived with a parent
or a person acting as a parent for at least six consecutive months immediately before the commencement
of a child-custody proceeding.” The “physical presence of, or personal jurisdiction over, a party or a child
is not necessary to make a child-custody determination.”

The federal Parental Kidnapping Prevention Act (PKPA) similarly grants exclusive jurisdiction over a
child-custody petition to a child’s “home state.” Under the supremacy clause, PKPA takes precedence
over any conflicting state law.

Here, Danielle has lived in State A continuously since her birth. She has been present at her aunt’s home
in State B for only two weeks. The child’s parents both continue to live in State A. Thus, only a State A
court may exercise jurisdiction over the aunt’s custody petition.

Seperac-J19 Exam-Released MEE Essay Compilation © 2016-2020 853


[NOTE: The UCCJEA “temporary emergency jurisdiction” provision is not relevant here, as it is
applicable only when the child has been “abandoned . . . or is threatened with mistreatment or abuse.”
The mother’s traveling does not constitute abandonment, as she has invariably left the child with the
child’s father. There is nothing in the facts to support a claim that the father has not continued to act as
his child’s guardian.]

Explanation to Point Four (30%):

It is highly unlikely that the court will grant the aunt’s petition for custody, given the constitutionally
based parental preference.

In all states, when a nonparent seeks to obtain a child’s custody from a fit legal parent, the parent is
accorded a preference. Although the strength of the preference varies from one state to the next, in Troxel
v. Granville, the Supreme Court of the United States implied that such a preference is constitutionally
mandated. Troxel involved a state statute under which “any person” could petition for visitation rights “at
any time” and authorized a court to grant such visitation whenever it concluded that “visitation may serve
the best interest of the child.” Because the statute was “breathtakingly broad” and required the court to
give “no special weight at all to a parent’s determination of her daughter’s best interests,” a plurality of
the Court found that it “contravened the traditional presumption that a fit parent will act in the best
interest of his or her child.”

Here, the aunt might argue that Danielle’s desire to live with her is sufficient to overcome the preference
in favor of continued parental custody. And in all states, the views of a child who is mature enough to
form and express a preference are certainly relevant to a custody determination. Indeed, the wishes of an
older child are typically given substantial weight.

However, the child’s wishes “are not treated equally in every case”:

Sometimes the child’s wishes are given controlling effect, while at other times the wishes are disregarded
altogether. The circumstances determining what effect, between these two extremes, should be given to a
child’s wishes in a particular case are, in addition to the comparative effect of objective factors affecting
its welfare generally, and in addition to the natural right of the child’s parent to have its custody
(frequently invoked successfully, at least in the absence of the parent’s long-term abandonment of the
child to another’s custody): the child’s age and judgment capacity; the basis for and strength of its
preference, generally; the treatment extended to the child by the contestants for its custody; and the
wrongful inducement of the child’s wishes.

Although there are a handful of states in which, by statute, the court must defer to the wishes of an older
child when choosing between fit parents, even if State A has such a statute it would not be applicable in
this case because the custody contest is between a fit parent and a nonparent.

Here, Wendy’s frequent absences from home are work-related, and Frank has been at home and able to
provide care at all times. And even if the aunt were to substantiate Danielle’s claim that her father is
“depressed,” the father’s mental state does not represent a showing that his care is inadequate or that he is
unfit. In sum, because of the preference accorded parents in a custody contest with a nonparent, it is
highly unlikely that a court will grant the aunt’s custody petition despite Danielle’s stated preference.

Seperac-J19 Exam-Released MEE Essay Compilation © 2016-2020 854


#196-FEB 2018–MEE Q01: QUESTION ONE (FAMILY LAW)

In 2012, David and Meg had a baby girl, Anna. At the time of Anna's birth, David and Meg were both 21
years old. For the next four years, they lived separately. David and Anna lived with David's mother
(Anna's grandmother). The grandmother cared for Anna while David worked. David cared for Anna most
evenings and weekends. During this period, Meg attended college in a distant city; she called weekly but
visited Anna only during school breaks and for one month each summer.

In 2013, David bought an auto repair business with money he had saved. The grandmother continued to
care for Anna while David was working in his auto repair business.

In 2016, David and Meg were married in a small wedding held at the grandmother's house. One week
before their wedding, David surprised Meg by asking her to sign a premarital agreement prepared by his
attorney. The agreement provided that, in the event of a divorce,

1. all assets owned by each spouse at the time of the marriage would remain the sole property of
that spouse;

2. neither spouse would be entitled to alimony; and

3. the spouses would have joint physical custody of Anna.

Attached to the proposed agreement was an accurate list of David's net assets (his personal possessions,
the auto repair business, a used car, and a small bank account), a list of his liabilities, and his tax returns
for the past three years.

David told Meg that he would not proceed with the marriage unless she signed the agreement. Meg
believed that the marriage would be successful, and she did not want to cancel or postpone the wedding.
She therefore signed the agreement and appended a list of her own debts (student loans); she correctly
indicated that she had no assets other than her personal possessions.

Since the wedding, David, Meg, and Anna have lived together and the grandmother has continued to
provide child care while David and Meg are at work. Meg has worked full-time as a computer engineer,
and David has continued to work full-time in his auto repair business. Their incomes are relatively equal.

They have the following assets: (a) the auto repair business (owned by David); (b) stocks (owned by Meg,
which she inherited last year); and (c) the marital home (purchased by David in his name alone shortly
after the wedding). The down payment and all mortgage payments for the marital home have come from
the couple's employment income.

Last month, David discovered that Meg had been having an affair with a coworker for the past year.

David wants a divorce. He also wants to obtain sole physical custody of Anna; he believes that Meg's
adultery should disqualify her as a custodial parent. His plan is to live with the grandmother, who would
provide child care when he is unavailable.

This jurisdiction has adopted a statute modeled after the Uniform Premarital Agreement Act.

1. May either spouse successfully enforce the premarital agreement in whole or in part? Explain.

Seperac-J19 Exam-Released MEE Essay Compilation © 2016-2020 855


2. Assuming that the premarital agreement is not enforceable, what assets are divisible at divorce?
Explain.

3. Assuming that the premarital agreement is not enforceable, may David obtain sole physical
custody of Anna based on (a) Meg's adultery or (b) other factors? Explain.

Seperac-J19 Exam-Released MEE Essay Compilation © 2016-2020 856


#196: F18-1 MEE: ANSWER: NCBE (FAMILY LAW)

POINT (1)(a) [30%] ISSUE: Under the state statute, when is a premarital agreement regarding
property division and alimony enforceable? ANSWER: Under the state statute, an agreement
regarding property division and alimony that is made voluntarily and with fair and reasonable
disclosure of assets and obligations is binding. Here, it is highly unlikely that Meg can avoid
enforcement of the premarital agreement; David fully disclosed his assets, and the evidence does not
support a claim of involuntariness.

POINT (1)(b) [10%] ISSUE: Under the state statute, when is a premarital agreement regarding
child custody enforceable? ANSWER: A premarital agreement regarding child custody is
unenforceable.

POINT (2) [35%] ISSUE: What assets are divisible at divorce? ANSWER: In most states, assets
acquired before marriage or by inheritance are “separate” property; neither a separate asset nor
appreciation in its value is subject to division at divorce. However, substantial postmarital effort of
an owner spouse or the use of marital funds to add value or obtain equity in separate property
creates marital property. Thus, Meg’s inheritance is separate property. David’s auto repair
business may be partly marital property. The home is marital property.

POINT (3)(a) [10%] ISSUE: May a court deny a parent physical custody of a child based on
adultery? ANSWER: No. Child custody decisions are based on the best interests of the child, and a
parent’s misconduct may not be taken into account unless it causes significant harm to the child.
Thus, David cannot obtain sole physical custody of Anna based on Meg’s adultery.

POINT (3)(b) [15%] ISSUE: What factors may be taken into account in determining child custody?
ANSWER: It is possible that a court would award David sole physical custody based, at least in
part, on his greater role in providing child care and the grandmother’s role as a primary caregiver
during Anna’s formative years, which an award of sole physical custody to David would preserve.
But evidence on other relevant factors is lacking, making it impossible to predict what a court
would do.

ANSWER DISCUSSION:

Under this state statute (modeled on the Uniform Premarital Agreement Act (UPAA)), a premarital
agreement regarding property division and alimony is binding if entered into voluntarily and with
reasonable disclosure of assets and obligations. Here, David accurately disclosed his assets, and it is
unlikely that a court will find that Meg entered into the agreement involuntarily because David presented
the agreement to her a week before the wedding. The wedding was small and thus would not likely have
been difficult or expensive to cancel. Meg also had time to confer with independent counsel. However,
whether or not the property-division and alimony provisions of the agreement are upheld, the provisions
regarding child custody are unenforceable. In most states, assets acquired before marriage or by
inheritance are “separate” property; neither a separate asset nor appreciation in its value is subject to
division at divorce. However, substantial postmarital effort by the owner of a separate asset that enhances
the value of that asset creates marital property; so does the use of marital funds to add value or obtain
equity in separate property. Thus, Meg’s inheritance is a separate asset. David’s auto repair business is
partly separate because it was acquired before the marriage, but may be partly marital if its value
increased because he worked full-time in the business after the marriage. The home is marital, despite the

Seperac-J19 Exam-Released MEE Essay Compilation © 2016-2020 857


fact that David took title in his name only, because marital funds were used to pay for it. Custody
decisions are based on the “best interests of the child.” David cannot obtain sole physical custody based
on Meg’s adultery; this behavior is irrelevant unless he can show that it has caused or will cause
significant harm to Anna. In evaluating a child’s best interests, courts look at a range of factors, including
the child’s wishes, the parents’ physical and mental health, and the interaction and interrelationship of the
child with both parents and any other person who may significantly affect the child’s best interests. Here,
it is possible that David might obtain sole physical custody based on one or more of these factors, but case
outcome cannot be predicted with the information at hand.

ANSWER EXPLANATION:

Explanation to Point-One(a) (30%):

Under the state statute, an agreement regarding property division and alimony that is made voluntarily and
with fair and reasonable disclosure of assets and obligations is binding. Here, it is highly unlikely that
Meg can avoid enforcement of the premarital agreement; David fully disclosed his assets, and the
evidence does not support a claim of involuntariness.

Although courts were once hostile to premarital agreements, today all states permit spouses to contract
premaritally with respect to rights and obligations in property. In all states, the enforceability of such an
agreement turns on three factors: voluntariness, fairness, and disclosure. How courts apply these factors
varies significantly from one state to the next. In many states, an agreement is unenforceable if the party
against whom enforcement is sought succeeds in showing involuntariness, unfairness, or lack of adequate
disclosure. However, under the UPAA, which has been adopted in 26 states and this jurisdiction, the party
against whom enforcement is sought must prove (1) involuntariness or (2) that “the agreement was
unconscionable when it was executed” and that he or she did not receive or waive “fair and reasonable”
disclosure and “did not have, or reasonably could not have had, an adequate knowledge” of the other’s
assets and obligations. Thus, under the state statute, a court may not refuse to enforce a premarital
agreement based on unconscionability unless it also finds lack of adequate disclosure or knowledge.

Here, Meg cannot establish inadequate disclosure and knowledge. David gave her copies of his tax returns
for three years and an accurate list of his assets. Under the UPAA, Meg would thus be required to show
that the execution of the agreement was involuntary in order to avoid its enforcement.

In considering whether a premarital agreement was voluntarily executed, courts look to whether there was
fraud, duress, or coercion. They agree that one party’s insistence on signing the agreement as a condition
of the marriage does not, of itself, render the agreement involuntary, but there is no consensus on what
additional facts are sufficient to establish involuntariness.

Many of the reported cases, like this one, involve a claim of involuntariness based on presentation of an
agreement very close to the wedding. In analyzing whether an agreement signed under these
circumstances is voluntary, courts have looked at a wide range of factors, including the difficulty of
conferring with independent counsel, other reasons for proceeding with the marriage (for example, a
preexisting pregnancy), and financial losses and embarrassment arising from cancellation of the wedding.
A number of courts have held that an agreement signed without the opportunity to consult with
independent counsel will be scrutinized more closely.

Here, David presented the proposed agreement one week before the wedding. There is no evidence that
Meg did not have time to confer with counsel. Nor is there evidence that canceling the wedding would
have entailed any significant expense; the wedding was small and took place at the grandmother’s house.

Seperac-J19 Exam-Released MEE Essay Compilation © 2016-2020 858


Cancellation of the wedding at this late date might have been embarrassing, but it probably would not
have been costly. Thus, it is unlikely that Meg can establish involuntariness and void the premarital
agreement.

[NOTE: Although only about half the states have adopted the UPAA, voluntariness, fairness, and asset
disclosure are relevant to enforceability in all jurisdictions. In non-UPAA jurisdictions, however, a court
may refuse to enforce a premarital agreement on any of these grounds. Thus, in a non-UPAA state, a
court’s analysis of voluntariness will likely track the analysis here. But in a non-UPAA jurisdiction, a
court could also refuse to enforce on fairness grounds even though disclosure was adequate.]

Explanation to Point-One(b) (10%):

A premarital agreement regarding child custody is unenforceable.

Because of the strong public policy in favor of protecting the best interests of children, courts have
invariably found that provisions in a premarital contract relating to children, including provisions relating
to child custody and visitation, are unenforceable. Although the UPAA does not explicitly bar an
agreement respecting child custody, long tradition would seem to ensure that courts would not consider
themselves bound by custody provisions they believe injurious to the child’s interest. The law of
separation agreements in every state is explicit on that point, and there is no reason why premarital
agreements would be treated differently.

Thus, the provisions of the premarital agreement requiring joint custody will not be enforced even if the
alimony and support provisions of the agreement are upheld.

Explanation to Point-Two (35%):

In most states, assets acquired before marriage or by inheritance are “separate” property; neither a
separate asset nor appreciation in its value is subject to division at divorce. However, substantial
postmarital effort of an owner spouse or the use of marital funds to add value or obtain equity in separate
property creates marital property. Thus, Meg’s inheritance is separate property. David’s auto repair
business may be partly marital property. The home is marital property.

In all states, a divorce court may divide assets without regard to title. However, in most states, only
marital property—assets acquired during the marriage except by gift, devise, or inheritance—is subject to
division at divorce. In a minority of “hotchpot” jurisdictions, the court may divide all assets, whenever or
however acquired. A few states permit the division of separate property in special circumstances, such as
hardship.

Although the mere fact that a separate asset appreciates in value during the marriage does not create
marital property, an asset that is initially separate property may be partially transformed into marital
property if marital funds or significant postmarital effort by the owner spouse enhance its value or build
equity. Thus, if a spouse spends a substantial amount of time working in a separate business, that effort
typically creates marital property. And if marital funds are used to reduce mortgage indebtedness, such
equity-building payments typically create marital property.

Here, Meg’s stocks are separate property because she inherited them, and there is no indication that she
has expended substantial effort that has contributed to their current value. David’s auto repair business
was initially separate property; he acquired the asset before the marriage. But he has worked full-time in
the business during the marriage; this would represent substantial effort, and any postmarital increase in

Seperac-J19 Exam-Released MEE Essay Compilation © 2016-2020 859


the value of the business is thus likely to be characterized as at least partly marital. The home is marital
property despite the fact that it is titled solely in David’s name, as marital funds (employment income)
were used for the down payment and all mortgage payments.

In a hotchpot state, all assets owned by David and Meg would be subject to division at divorce.

[NOTE: The above analysis applies in both common law and community-property states. Although the
rules governing asset management and division at death vary depending on whether the jurisdiction is a
common law or community-property state, today, all states disregard title in defining the pool of assets
available for division at divorce. Indeed, the marital property rules applicable in common law states are
sometimes referred to as “deferred community property.”]

Explanation to Point-Three(a) (10%):

Child custody decisions are based on the best interests of the child, and a parent’s misconduct may not be
taken into account unless it causes significant harm to the child. Thus, David cannot obtain sole physical
custody of Anna based on Meg’s adultery.

Child custody decision making is invariably governed by the “best interests of the child” standard. Today,
courts agree that a court may not deprive a parent of custody based on a parent’s values or lifestyle unless
the evidence shows that the parental conduct adversely affects the child. To deprive a parent of custody,
the evidence must support a logical inference that some specific, identifiable behavior or conduct of the
parent will probably cause significant physical or emotional harm to the child. This link between parent’s
conduct and harm to the child, moreover, may not be based on evidence which raises a mere surmise or
speculation of possible harm. Based on this child-centered approach, most courts have ruled that a
parent’s sexual behavior is not by itself sufficient to deny a parent custody.

Thus, a court may not award David sole physical custody of Anna based on Meg’s adultery because there
is no evidence that Meg’s adultery has caused Anna harm.

Explanation to Point-Three(b) (15%):

It is possible that a court would award David sole physical custody based, at least in part, on his greater
role in providing child care and the grandmother’s role as a primary caregiver during Anna’s formative
years, which an award of sole physical custody to David would preserve. But evidence on other relevant
factors is lacking, making it impossible to predict what a court would do.

A custody decision is based on the best interests of the child. That determination is based on a range of
factors, including “(1) the wishes of the child’s parent or parents as to his custody; (2) the wishes of the
child as to his custodian; (3) the interaction and interrelationship of the child with his parent or parents,
his siblings, and any other person who may significantly affect the child’s best interest; (4) the child’s
adjustment to his home, school, and community; and (5) the mental and physical health of all individuals
involved.”

Here, there are some facts that support an award of sole physical (i.e. residential) custody to David. Meg
lived with Anna only for brief periods during the child’s first four, formative years; David lived with
Anna throughout this period and provided care for her when he was not working. During these four years,
the grandmother acted as a primary caregiver to Anna, and she has continued to provide care; an award of
sole physical custody to David would thus enable Anna to maintain an active relationship with a
caregiver, her grandmother, who may have played an important parental role.

Seperac-J19 Exam-Released MEE Essay Compilation © 2016-2020 860


However, evidence is lacking on other relevant factors, including Anna’s preference, each parent’s mental
and physical health, Anna’s current relationships with both parents and with the grandmother, and Anna’s
current adjustment to her home, school, and community.

It is thus possible, but far from certain, that David could obtain sole physical custody of Anna, but without
information on the other relevant factors, it is impossible to predict the outcome here.

Seperac-J19 Exam-Released MEE Essay Compilation © 2016-2020 861


#197-FEB 2017–MEE Q03: QUESTION THREE (FAMILY LAW/CONFLICTS)

In 2005, Andrew and Brenda began living together in State A while both were attending college there.
Andrew proposed marriage to Brenda, but she refused. However, after learning that she was pregnant,
Brenda told Andrew that she wanted to marry him before the baby was born. Andrew was thrilled and
told her that they were already married "in the eyes of God." Brenda agreed.

Andrew and Brenda did not obtain a marriage license or have a formal wedding. Nonetheless, Brenda
started using Andrew's last name even before their daughter, Chloe, was born. After Andrew graduated
from college and started a new job, he listed Brenda as his spouse so that she could qualify for benefits
through Andrew's employer. They also filed joint income tax returns.

In March 2007, just after Chloe's first birthday, Andrew and Brenda decided to separate. They had little
property to divide and readily agreed to its disposition. Andrew agreed that Brenda should have sole
custody of Chloe, and Brenda, desiring the cleanest break possible, agreed that Andrew would not be
responsible for any child support. Andrew told Brenda that no formal divorce was necessary because they
had never formally married.

In June 2007, Brenda and Chloe moved to start a new life in State B. Andrew sent Chloe an occasional
card or birthday gift, but otherwise maintained no contact with Chloe or Brenda. Not long after settling in
State B, Brenda met and fell in love with Daniel.

In 2008, Brenda and Daniel obtained a State B marriage license and wed. Thereafter, Daniel formed a
close and loving bond with Chloe. Indeed, with only very infrequent contact from Andrew, Chloe
regarded Daniel as her father and called him "Dad."

In January 2017, Brenda purchased a lottery ticket. The ticket won a jackpot of $5 million, which was
paid that month. Shortly thereafter, Brenda informed Daniel that she wanted a divorce and that she
intended to use her lottery winnings to launch a new life with Chloe in a distant state and break off all
contact with Daniel. When Chloe learned about this, she became very upset because she continues to
regard Daniel as her father.

State A recognizes common law marriage. State B formerly allowed common law marriage until a statute,
enacted in 2001, prospectively barred the creation of new common law marriages within the state. Neither
State A nor State B is a community-property state.

1. On what basis, if any, would Andrew have a claim to a share of Brenda's lottery winnings?
Explain.

2. Assuming that Andrew and Brenda have a valid marriage, on what basis, if any, would Daniel
have a claim to a share of Brenda's lottery winnings? Explain.

3. If Brenda cuts off all contact between Chloe and Daniel, can Daniel obtain court-ordered
visitation with Chloe? Explain.

Seperac-J19 Exam-Released MEE Essay Compilation © 2016-2020 862


#197: F17-3 MEE: ANSWER: NCBE (FAMILY LAW/CONFLICTS)

POINT (1)(a) [15%] ISSUE: Did Andrew and Brenda’s conduct in State A establish a valid
common law marriage? ANSWER: Yes. Andrew and Brenda formed a valid common law
marriage in State A and remain married.

POINT (1)(b) [10%] ISSUE: Will State B recognize Andrew and Brenda’s common law marriage
as valid, notwithstanding State B’s policy forbidding common law marriages? ANSWER: Yes.
State B will recognize a common law marriage entered into in another state.

POINT (1)(c) [15%] ISSUE: Does Andrew’s common law marriage to Brenda give him a basis for
claiming a share of her lottery winnings? ANSWER: Yes. In most states, Andrew could recover a
share of the lottery proceeds in a divorce action against Brenda, but a court is unlikely to award
him a significant share of Brenda’s winnings.

POINT (2)(a) [15%] ISSUE: Can Daniel claim a share of Brenda’s lottery winnings on the ground
that he and Brenda are married and the winnings are marital property? ANSWER: No. Because
Brenda was already married when she purported to marry Daniel, her marriage to Daniel is void.
Because he is not her legal spouse, he is not entitled to a share of her lottery winnings as marital
property.

POINT (2)(b) [25%] ISSUE: If Daniel and Brenda are not married, is there any other basis on
which Daniel can claim a share of Brenda’s lottery winnings? ANSWER: In some jurisdictions,
Daniel may be entitled to a share of the lottery proceeds under the putative-spouse doctrine, or
under a statute permitting a party to a void marriage to obtain a share of “marital” assets, or under
a cohabitation-contract theory.

POINT (3) [20%] ISSUE: Can Daniel, a nonparent, obtain court-ordered visitation with Chloe
despite Brenda’s objection and her assertion of parental rights? ANSWER: Yes. It is probable that
Daniel will succeed in obtaining court-ordered visitation with Chloe, notwithstanding Brenda’s
objection.

ANSWER DISCUSSION:

Andrew and Brenda established a valid common law marriage in State A, which they did not dissolve by
divorce. Under generally accepted conflict-of-laws principles, State B will recognize the validity of
Andrew and Brenda’s marriage in State A, notwithstanding its own policy against common law marriage.
Because Brenda remains married to Andrew in most states, her lottery winnings qualify as “marital
property” subject to equitable distribution, although a court would be unlikely to award Andrew much, if
any, of the prize. Because Brenda is married to Andrew, her attempted marriage to Daniel is void, and
Daniel will not be able to claim a share of the lottery winnings as “marital” property. However, although
not a spouse, in many states Daniel could assert a claim to some share of the lottery winnings under the
putative-spouse doctrine. In states that do not recognize the putative-spouse doctrine, Daniel might still be
able to claim a share of the lottery winnings under a statute allowing a party to a void marriage to obtain a
share of “marital” assets or under a cohabitation-contract theory. Because Daniel has acted as a parent to
Chloe, it is probable, but not certain, that he will obtain visitation with Chloe, notwithstanding Brenda’s
assertion of parental rights.

Seperac-J19 Exam-Released MEE Essay Compilation © 2016-2020 863


ANSWER EXPLANATION:

Explanation to Point-One(a) (15%):

Andrew and Brenda formed a valid common law marriage in State A and remain married.

Formation of a valid common law marriage requires that the partners (1) cohabited, (2) agreed to be
married, and (3) held themselves out to others as a married couple. Andrew and Brenda satisfied all of
these requirements. While living together, they agreed to marry and appeared to regard themselves as
married; they consistently held themselves out as married to friends, to Andrew’s employer, and to
governmental authorities. In a state recognizing common law marriage, this conduct would establish a
valid marriage.

When Andrew and Brenda separated, they did not divorce and thus continued to be married to each other.
A common law marriage, just like a ceremonial marriage, must be dissolved through a divorce.

Explanation to Point-One(b) (10%):

State B will recognize a common law marriage entered into in another state.

Under generally accepted conflict-of-laws principles, a marriage valid under the law of the state where it
was contracted is valid elsewhere unless it violates the strong public policy of another state which has the
most significant relationship to the spouses and the marriage. Perhaps because the vast majority of states
once permitted common law marriage, courts in states that do not recognize common law marriage have
consistently held that it does not violate any strong public policy to recognize an out-of-state common law
marriage. Under the Restatement of Conflicts, a marriage without any formal ceremony is valid
everywhere if the acts alleged to have created it took place in a state in which such a marriage is valid.
Thus, a State B court will recognize Brenda’s common law marriage to Andrew.

Explanation to Point-One(c) (15%):

In most states, Andrew could recover a share of the lottery proceeds in a divorce action against Brenda,
but a court is unlikely to award him a significant share of Brenda’s winnings.

Property acquired by either spouse during an ongoing marriage, other than by gift or inheritance, is
marital property subject to division on divorce. Although some states provide that marital property ceases
to accrue after the parties’ separation, most hold that marital property continues to accrue until the
marriage is dissolved. Accordingly, because Brenda remains married to Andrew, her lottery winnings are
likely to be considered marital property. Thus, Andrew could seek to obtain a share of the lottery
winnings in a divorce action.

Some states divide marital property equally, but most grant judges authority to divide the property
equitably. Although Andrew, in a divorce action, will likely be able to seek an equitable distribution of
Brenda’s lottery winnings as part of the marital property, it is unlikely that a judge would award him a
significant (or perhaps any) share of the lottery money given the relatively short duration of their
cohabitation, the absence of any contributions by Andrew to the marital estate, and the parties’ long and
near-total estrangement.

Explanation to Point-Two(a) (15%):

Seperac-J19 Exam-Released MEE Essay Compilation © 2016-2020 864


Because Brenda was already married when she purported to marry Daniel, her marriage to Daniel is void.
Because he is not her legal spouse, he is not entitled to a share of her lottery winnings as marital property.

Bigamy is illegal in all states; no individual may have more than one legal spouse at a time. Because of
the ban on bigamy, when a first marriage has not been legally terminated, a second marriage has no legal
effect. Because Brenda and Andrew did not divorce, application of this rule suggests that Brenda and
Daniel are not married, and Daniel would have no claim to a share of marital property because they have
no “marital” property.

One potential impediment to that conclusion is the presumption that the latest in a series of marriages is
valid. The presumption is “one of the strongest known to the law” and can be rebutted only by “strong
evidence that the prior marriage still subsists or by cogent and conclusive evidence.” The presumption is
designed to protect the parties’ expectations; thus, some cases hold that the presumption’s strength
increases with the lapse of time and the birth of children in the later marriage.

Once Daniel establishes his marriage ceremony with Brenda, to overcome the presumption of validity,
Brenda must show that she previously entered into a valid common law marriage with Andrew and that
this marriage was not terminated by divorce or annulment. Most courts have held that the presumption
may be rebutted with a showing that court records in all jurisdictions where the first spouse has lived do
not evidence a divorce. Thus, if Brenda can show that she had a common law marriage with Andrew, and
if there is no evidence that a State A court terminated that marriage, the State B court should find that she
is still Andrew’s wife. Because the facts specify that Andrew and Brenda were not divorced, it seems
likely that Brenda can meet her burden of proof, in which case Daniel will not obtain an equitable
distribution of the lottery proceeds or other property through a divorce.

Explanation to Point-Two(b) (25%):

In some jurisdictions, Daniel may be entitled to a share of the lottery proceeds under the putative-spouse
doctrine, or under a statute permitting a party to a void marriage to obtain a share of “marital” assets, or
under a cohabitation-contract theory.

The putative-spouse doctrine permits a would-be spouse, who participated in a marriage ceremony with a
good-faith but mistaken belief in its validity, to be treated like a spouse for purposes of equitable
distribution of the property acquired by the couple during their invalid marriage. In this case, it appears
that Daniel believed in good faith that he was married to Brenda and was wholly unaware of the defect in
their marriage. (Indeed, even Brenda appears to have been unaware of the defect, as she appears to have
accepted Andrew’s mistaken assurance that they needed no formal divorce to dissolve their common law
marriage.) Accordingly, Daniel would qualify as a putative spouse and could seek an equitable
distribution of any property that would have been classified as marital property had the parties been
validly married, including Brenda’s lottery winnings. Given the duration of their relationship, Daniel’s
equitable claim is likely to be substantial.

In states that do not recognize the putative-spouse doctrine, Daniel might still be able to recover some
share of the lottery winnings. Some states that have not recognized the putative- spouse doctrine
nonetheless permit distribution of “marital” assets in a proceeding to declare a marriage void. “The trend
of modern statutes is increasingly to blur the line not only between void and voidable marriages but even
between them and valid marriages in an attempt to equalize the consequences of termination of marital
relationships on whatever ground.” In such a state, Daniel could obtain a share of “marital” assets even
though his marriage to Brenda is void.

Seperac-J19 Exam-Released MEE Essay Compilation © 2016-2020 865


It is also conceivable that Daniel could claim a share of the lottery proceeds under the law governing
unmarried cohabitants. In states following the California Supreme Court’s ruling in Marvin v. Marvin,
property or support rights between cohabitants can be founded upon an express or implied contract to
share assets. Although the Marvin theory is almost invariably applied to cohabiting couples who have not
entered into a ceremonial marriage, Brenda and Daniel’s attempted marriage does imply an agreement to
share their property as would a married couple. However, some states refuse, on public policy grounds, to
enforce cohabitation agreements.

Explanation to Point-Three (20%):

It is probable that Daniel will succeed in obtaining court-ordered visitation with Chloe, notwithstanding
Brenda’s objection.

Although Chloe regards Daniel as a father, he is not her legal parent. The Supreme Court of the United
States has recognized that parents have a fundamental constitutional right to control the upbringing of
their children, including decisions about with whom their children will visit. A fit parent is presumed to
act in the best interests of her children, and courts are constitutionally required to give “special weight” to
a parent’s reasons for objecting to visitation with a third party before overriding the parent’s objection and
granting visitation.

Accordingly, a court would be required to carefully consider Brenda’s reasons for objecting to Daniel’s
further contact with Chloe, but would be permitted to grant visitation if the balance of considerations
strongly favored ongoing contact. Here, Daniel has lived with Chloe for 9 of her 11 years, and he has
played an active parental role. Chloe calls Daniel “Dad” and regards him as her father. Daniel could thus
be considered a “de facto parent.”

There are facts to support a finding that Chloe would be harmed by termination of contact with Daniel.
Chloe became “very upset” when she learned that Brenda planned to cut off contact. Moreover, Brenda
plans to cut off all contact with Daniel. In Troxel, in explaining why deference to the parent’s objection
was appropriate, the plurality emphasized that the mother sought only to limit the access of the paternal
grandparents and did not seek to cut them off entirely. Brenda’s plan to cut off all contact with the man
Chloe regards as her father thus will probably weigh against her. For example, in one case, the court held
that grandparents who served as caregivers of the child, in loco parentis, had standing to seek visitation,
notwithstanding the mother’s objection, under the state’s Grandparents Visitation Act. Chloe also does
not have an active relationship with her biological father and regards Daniel as her dad. In some states,
this could tip the balance in Daniel’s favor.

State law on nonparent visitation rights varies substantially, however. Thus, it is probable, but not certain,
that Daniel will obtain visitation with Chloe.

Seperac-J19 Exam-Released MEE Essay Compilation © 2016-2020 866


#198-FEB 2016–MEE Q06: QUESTION SIX (FAMILY LAW)

Eight years ago, a woman and a man began living together. The woman worked as an investment banker,
and the man worked part-time as a bartender while he struggled to write his first novel. The couple lived
in a condominium that the woman had purchased shortly before the man moved in. The woman had
purchased the condominium for $300,000 using her own money and had taken title in her own name.

Four years ago, the woman and the man were married at City Hall. One week before the wedding, the
woman presented the man with a proposed premarital agreement and an asset list. The asset list correctly
stated that the woman owned the condominium, then worth $350,000, and a brokerage account, then
worth $500,000. The agreement specified that, in the event of divorce, each spouse would be entitled to
retain “all assets which he or she then owns, whether or not those assets are acquired during the
marriage.” The man was surprised when the woman gave him the agreement to sign, and he contacted a
lawyer friend for advice. The lawyer urged the man not to sign the agreement. Nonetheless, the man
signed the agreement, telling the woman, “I’m a little hurt, but I guess I understand that you want to keep
what you earn.” The woman signed the agreement as well.

After their wedding, the woman and the man continued to live in the woman’s condominium and to work
at the jobs each held before the marriage. The man also continued to work on his novel.

Six months ago, the man’s novel was accepted by a publisher. The novel will be released next spring. The
publisher has estimated that the royalties may total as much as $200,000 over the next five years.

Two months ago, the woman and the man separated. The woman remained in the condominium, now
worth $400,000 as a result of market appreciation. The woman’s brokerage account, worth $500,000
when she and the man married, is now worth $1,000,000 as a result of market appreciation and additional
investments that the woman made with employment bonuses she received during the marriage. The
woman has made no withdrawals from this account.

One month ago, the woman won, but has not yet received, a $5 million lottery jackpot.

One week ago, the man filed for divorce. In the man’s divorce petition, he asks the court to invalidate the
premarital agreement and seeks half of all assets owned by the woman, i.e., the woman’s brokerage
account, her condominium, and her right to the lottery payment. The man owns no assets except for
personal effects and the book contract under which he will receive future royalties based on sales of his
novel.

This jurisdiction has adopted the Uniform Premarital Agreement Act, which in relevant part provides that
“the party against whom enforcement [of the premarital agreement] is sought must prove (1)
involuntariness or (2) both that ‘the agreement was unconscionable when it was executed’ and that he or
she did not receive or waive a ‘fair and reasonable’ disclosure and ‘did not have or reasonably could not
have had . . . an adequate knowledge’ of the other’s assets and obligations.”

The jurisdiction’s divorce law requires “equitable distribution” of all marital (community) assets and
prohibits the division of separate assets.

1. Is the premarital agreement enforceable? Explain.

Seperac-J19 Exam-Released MEE Essay Compilation © 2016-2020 867


2. Assuming that the agreement is unenforceable, what assets are subject to division in the divorce
action, and what factors should a court consider in distributing those assets? Explain.

Seperac-J19 Exam-Released MEE Essay Compilation © 2016-2020 868


#198: F16-6 MEE: ANSWER: NCBE (FAMILY LAW)

POINT (1) [35%] ISSUE: Is the premarital agreement enforceable under the Uniform Premarital
Agreement Act? ANSWER: Yes. Under the UPAA, a premarital agreement entered into after full
disclosure of assets and obligations is binding unless it was entered into involuntarily. Here, the
woman fully disclosed her assets, and the facts do not support a finding that the man signed the
agreement involuntarily. Thus, the agreement is likely enforceable.

POINT (2)(a) [50%] ISSUE: What assets owned by the man and the woman are subject to division
at divorce? ANSWER: Assets acquired before marriage or by inheritance are separate, not marital,
property. In most jurisdictions, neither separate property nor appreciation in its value is subject to
division at divorce. However, significant spousal effort during the marriage that adds value to a
separate asset does create divisible marital property. Thus, the woman’s condominium is separate
and non-divisible; the woman’s lottery winnings are marital and divisible; and some portion of the
woman’s brokerage account and at least some of the man’s royalties are also marital and divisible.

POINT (2)(b) [15%] ISSUE: Under an equitable distribution rule, what factors will a court
consider in distributing the couple’s divisible assets? ANSWER: In dividing marital assets, divorce
courts look at contribution (monetary and nonmonetary), need, and marital duration. Here, the
marriage was short, the man is the needier spouse, and the woman made greater financial
contributions to the marriage, contributions which may have enabled the man to complete his
novel.

ANSWER DISCUSSION:

Under the UPAA, adopted in this state, a premarital agreement will be enforced unless the party against
whom enforcement is sought (1) did not execute the agreement voluntarily or (2) both (a) did not receive
or waive fair and reasonable disclosure of assets of the other party and (b) can show that the agreement
was unconscionable when executed. Here, the agreement is likely enforceable because the woman fully
disclosed her assets and the facts do not support a finding that the man entered into the agreement
involuntarily. At divorce, most states permit the division of only marital or community property (i.e.,
property acquired during the marriage except by gift, descent, or devise). Marital property typically
continues to accrue until the date a divorce is final, although some states use an earlier date, typically the
date of separation or divorce filing. An increase in the value of a premarital asset resulting from
significant spousal effort during marriage is divisible; an increase in the value of a premarital asset
resulting solely from market appreciation remains separate. Thus, the woman’s lottery winnings are
probably divisible because the woman purchased the winning ticket before a divorce action was filed. The
fact that the woman may receive the lottery winnings after divorce does not alter this result. The woman’s
condominium is not divisible marital property because it was acquired before marriage and its increased
value is due to market forces. Some portion of the woman’s brokerage account is divisible, as the woman
made contributions with marital earnings. At least some of the man’s future royalties are divisible because
the man expended significant effort during the marriage on the novel from which those royalties will
derive. Again, the fact that the man will receive the royalties after divorce does not alter their divisibility
at divorce. When equitably dividing marital property, courts consider need, contribution (both monetary
and nonmonetary), and marital duration. Here, the marriage was short, the man is the needier spouse, and
the woman has made larger financial contributions to the marriage. It is difficult to ascertain who made
more nonmonetary contributions to the marriage.

Seperac-J19 Exam-Released MEE Essay Compilation © 2016-2020 869


ANSWER EXPLANATION:

Explanation to Point-One (35%):

Under the UPAA, a premarital agreement entered into after full disclosure of assets and obligations is
binding unless it was entered into involuntarily. Here, the woman fully disclosed her assets, and the facts
do not support a finding that the man signed the agreement involuntarily. Thus, the agreement is likely
enforceable.

Although courts were once hostile to premarital agreements, today all states permit spouses to contract
premaritally with respect to rights and obligations in property whether acquired before or during the
marriage. In all states, the enforceability of such an agreement turns on three factors: voluntariness,
fairness, and disclosure. How courts apply these factors varies significantly from one state to the next. In
many states, an agreement is unenforceable if the party against whom enforcement is sought succeeds in
showing either involuntariness, unfairness, or lack of adequate disclosure.

However, under the UPAA, which has been adopted in 25 states and the state in which this question is set,
the party against whom enforcement is sought must prove: (1) involuntariness; or (2) both that the
agreement was unconscionable when it was executed, and that he or she did not receive or waive a fair
and reasonable disclosure and did not have or reasonably could not have had an adequate knowledge of
the other’s assets and obligations. Thus, a court may not refuse to enforce a premarital agreement based
on substantive unfairness unless it also finds lack of adequate disclosure or knowledge.

Here, the man cannot show inadequate disclosure because the woman gave him an accurate asset list with
the agreement. Under the UPAA, the man thus must show that the execution of the agreement was
involuntary in order to avoid its enforcement.

In considering whether a premarital agreement was voluntarily executed, courts look to whether there was
fraud, duress, or coercion. They agree that one party’s insistence on signing the agreement as a condition
of the marriage does not, of itself, render the agreement involuntary, but there is no consensus on what
additional facts are sufficient to establish involuntariness. Many of the reported cases, like this one,
involve a claim of involuntariness based on presentation of an agreement very close to the wedding. In
analyzing whether an agreement signed under these circumstances is voluntary, courts have looked at a
wide range of factors, including the difficulty of conferring with independent counsel, other reasons for
proceeding with the marriage (for example, a preexisting pregnancy), and financial losses and
embarrassment arising from cancellation of the wedding. The older cases tend to be more sympathetic to a
spouse who receives a last-minute demand – in recent years, more courts have accepted last- minute
agreements.

Here, the woman presented the agreement to the man only a week before the wedding. But the man had
time to confer with independent counsel, and he did so. The man ultimately rejected the advice of the
lawyer he consulted. The man and the woman were married at City Hall, and there are no facts suggesting
any hardship that the man would have suffered from canceling or postponing the ceremony. On these
facts, it is unlikely that the man can establish involuntariness. Under the UPAA, the premarital agreement
thus is likely enforceable.

Explanation to Point-Two(a) (50%):

Assets acquired before marriage or by inheritance are separate, not marital, property. In most
jurisdictions, neither separate property nor appreciation in its value is subject to division at divorce.

Seperac-J19 Exam-Released MEE Essay Compilation © 2016-2020 870


However, significant spousal effort during the marriage that adds value to a separate asset does create
divisible marital property. Thus, the woman’s condominium is separate and non-divisible; the woman’s
lottery winnings are marital and divisible; and some portion of the woman’s brokerage account and at
least some of the man’s royalties are also marital and divisible.

In all states (whether community property or common law jurisdictions), a divorce court may divide
marital (i.e. community) property without regard to title. In this and the vast majority of jurisdictions, a
divorce court may not divide separate property. In a minority of jurisdictions – so-called “hotchpot”
jurisdictions – the court may divide all assets, whenever or however acquired; a few states permit the
division of separate property in special circumstances, such as hardship.

An asset is marital if it was acquired during the marriage by any means other than gift, descent, or devise.
In the majority of states, marital property continues to accrue until a final divorce decree is entered. In
some states, however, marital property ceases to accrue after the date of permanent separation or the date
of divorce filing. However, expectancies created during the marriage are still marital even if payment will
not be received until after the marriage ends. Even contingent expectancies such as unvested pension
rights are subject to division at divorce if they were acquired through spousal effort during the marriage.

An asset that is initially separate property may be partially transformed into divisible marital property if
marital funds or significant effort by the owner-spouse enhance its value or build equity. In the vast
majority of states, the appreciation of separate assets during the marriage does not create marital property.

Here, unless the woman and the man live in a state in which marital property ceases to accrue after
separation, the woman’s lottery award is marital property because the woman purchased the winning
ticket before the man filed for divorce. It does not matter that the woman may not receive any winnings
before the divorce filing.

The entire value of the woman’s condominium is separate property because she purchased the
condominium outright before marriage, and the facts indicate that its increased value is due entirely to
market forces, not spousal efforts or marital funds.

Some portion of the woman’s brokerage account is divisible marital property because the woman
contributed her employment bonuses during the marriage to that account; only the portion attributable to
investments the woman made before marriage remains separate.

Some, if not all, of the royalties the man will receive in the future are divisible marital property despite
the fact that the man began the novel before marriage. Significant spousal effort during marriage that
enhances the value of a separate asset creates marital property, and the man worked at paid employment
only part-time throughout the marriage in order to work on the novel. The facts do not specify how much
the man worked on the novel during the marriage. If the man completed a significant portion of the novel
during the marriage, the value of the novel would be apportioned into separate and marital components. It
is irrelevant that the royalties will not be received until after the marriage ends.

[NOTE: The above analysis applies in both common law and community property jurisdictions. Although
the rules governing asset management and distribution at death vary depending on whether the
jurisdiction is a common law or community property state, today, all states disregard title in defining the
pool of assets available for divorce. Indeed, the marital property rules applicable in common law states
are sometimes referred to as “deferred” community property.]

Explanation to Point-Two(b) (15%):

Seperac-J19 Exam-Released MEE Essay Compilation © 2016-2020 871


In dividing marital assets, divorce courts look at contribution (monetary and nonmonetary), need, and
marital duration. Here, the marriage was short, the man is the needier spouse, and the woman made
greater financial contributions to the marriage, contributions which may have enabled the man to
complete his novel.

In an equitable distribution state, courts have broad authority to divide property fairly. Typically, either by
statute or by case law, courts are given a list of relevant, non-exclusive factors to consider. Relevant
factors usually include the duration of the marriage; indicators of each spouse’s future needs (including
age, health, resources, and occupational opportunities); and contributions of the parties to the marriage
and the acquisition of assets.

Here, the woman appears to have made much larger financial contributions to the marriage than the man,
and she appears to be better off now than the man because of her occupation as an investment banker and
her lottery winnings. The couple has no children, so the man has made no significant nonfinancial
contributions as a parent; there are also no facts to suggest that he has made significant nonfinancial
contributions as a homemaker. Instead, the woman’s support freed the man to work on his novel.

The broad discretion given courts to distribute property equitably often makes it impossible to predict
with certainty how a court will divide marital property.

[NOTE: An examinee may appropriately note that the uncertain value of the man’s expected royalties
may play a role in determining how a property award should be fashioned. With a pension of uncertain
value to be paid in the future, courts often order payment of a specified percentage of pension payments
when those payments are actually made instead of requiring the title-holding spouse to compensate the
other spouse at the time of divorce.]

Seperac-J19 Exam-Released MEE Essay Compilation © 2016-2020 872


#199-JUL 2014–MEE Q03: QUESTION THREE (FAMILY LAW)

In 1994, a man and a woman were married in State A.

In 1998, their daughter was born in State A.

In 2010, the family moved to State B.

In 2012, the husband and wife divorced in State B. Under the terms of the divorce decree:

(a) the husband and wife share legal and physical custody of their daughter;

(b) the husband must pay the wife $1,000 per month in child support until their daughter reaches age 18;

(c) the marital residence was awarded to the wife, with the proviso that if it is sold before the daughter
reaches age 18, the husband will receive 25% of the net sale proceeds remaining after satisfaction of the
mortgage on the residence; and

(d) the remaining marital assets were divided between the husband and the wife equally.

Six months ago, the husband was offered a job in State A that pays significantly less than his job in State
B but provides him with more responsibilities and much better promotion opportunities. The husband
accepted the job in State A and moved from State B back to State A.

Since returning to State A, the husband has not paid child support because, due to his lower salary, he has
had insufficient funds to meet all his obligations.

One month ago, the wife sold the marital home, netting $10,000 after paying off the mortgage. She then
moved to a smaller residence. The husband believes that he should receive more than 25% of the net sale
proceeds given his financial difficulties.

Last week, when the wife brought the daughter to the husband’s State A home for a weekend visit, the
husband served the wife with a summons in a State A action to modify the support and marital-residence-
sale-proceeds provisions of the State B divorce decree. The husband brought the action in the State A
court that adjudicates all domestic relations issues.

1(a). Does the State A court have jurisdiction to modify the child support provision of the State B
divorce decree? Explain.

1(b). Does the State A court have jurisdiction to modify the marital-residence-sale-proceeds
provision of the State B divorce decree? Explain.

2. On the merits, could the husband obtain retroactive modification of his child support obligation
to the daughter? Explain.

2(b). On the merits, could the husband obtain prospective modification of his child support
obligation to the daughter? Explain.

Seperac-J19 Exam-Released MEE Essay Compilation © 2016-2020 873


2(c). On the merits, could the husband obtain modification of the marital-residence-sale-proceeds
provision of the State B divorce decree? Explain.

Seperac-J19 Exam-Released MEE Essay Compilation © 2016-2020 874


#199: J14-3 MEE: ANSWER: NCBE (FAMILY LAW)

POINT (1)(a) [25%] ISSUE: Does the State A court have jurisdiction to modify the State B child
support order? ANSWER: No. Personal jurisdiction over a nonresident respondent does not confer
subject-matter jurisdiction over child support modification. Under UIFSA, a State A court may not
modify a child support order issued by a State B court when, as here, the child or either parent
continues to reside in State B, the jurisdiction that issued the child support order.

POINT (1)(b) [15%] ISSUE: Does the State A court have jurisdiction to modify the marital-
residence-sale-proceeds provision of the State B property-division decree? ANSWER: No. UIFSA
does not apply to disputes over property division. Thus, the State A court may exercise jurisdiction
over the husband’s petition to modify the marital-residence-sale-proceeds provision of the State B
divorce decree because it has personal jurisdiction over the wife.

POINT (2)(a) [20%] ISSUE: May a child support order be modified retroactively? ANSWER: No.
A child support order may not be modified retroactively.

POINT (2)(b) [25%] ISSUE: May a child support order be modified prospectively based on a
change of employment with a lower salary? ANSWER: It is unclear whether the husband could
obtain prospective downward modification of his child support based on his voluntary acceptance
of a job with a lower salary.

POINT (2)(c) [15%] ISSUE: May a property-division order be modified after entry of a divorce
decree? ANSWER: No. A divorce property-division award is not subject to modification.

ANSWER DISCUSSION:

The State A court may exercise personal jurisdiction over the wife because she was personally served in
State A. However, subject-matter jurisdiction over the interstate modification of child support is governed
by the Uniform Interstate Family Support Act (UIFSA). Under UIFSA, State A does not have jurisdiction
to modify the order for the daughter’s support because the wife is still a resident of State B. UIFSA, on
the other hand, does not govern property distributions, and thus a State A court is not precluded from
hearing the husband’s petition to modify the marital-residence-sale-proceeds provision of the divorce
decree. A child support order may not be modified retroactively. A child support order may be modified
prospectively based on a substantial change in circumstances. Courts agree that a significant decrease in
income is a substantial change in circumstances. All states treat voluntary income reductions differently
than involuntary reductions but employ different approaches for evaluating the impact of a voluntary
reduction. Whether the husband could obtain prospective modification of the child support order depends
on which approach is applied. A property-division order is not subject to post-divorce modification based
on a change in circumstances. Thus, the husband may, in some states, obtain prospective modification of
the order for the daughter’s support, but he may not obtain modification of the marital-residence-sale-
proceeds provision.

ANSWER EXPLANATION:

Explanation to Point-One(a) (25%):

Seperac-J19 Exam-Released MEE Essay Compilation © 2016-2020 875


Personal jurisdiction over a nonresident respondent does not confer subject-matter jurisdiction over child
support modification. Under UIFSA, a State A court may not modify a child support order issued by a
State B court when, as here, the child or either parent continues to reside in State B, the jurisdiction that
issued the child support order.

The State A court may exercise personal jurisdiction over the wife. The wife was personally served in
State A, and a state may exercise jurisdiction based on in-state personal service. But personal jurisdiction
over the wife is not enough to give a State A court jurisdiction to modify the State B support order.

The interstate enforcement and modification of child support is governed by the Uniform Interstate
Family Support Act (UIFSA), which has been adopted by all states. Under UIFSA, the state that
originally issued a child support order (here, State B) has continuing, exclusive jurisdiction to modify the
order if that state remains the residence of the obligee, the child, or the obligor and all parties do not
consent to the jurisdiction of another forum. Under the UIFSA, a tribunal of the State shall recognize and
enforce, but may not modify, a registered order if the issuing tribunal had jurisdiction. The wife and
daughter continue to reside in State B, and the wife has not consented to the jurisdiction of another forum.
Thus, a State A court does not have jurisdiction to modify the State B child support order.

[NOTE: Examinees who do not discuss personal jurisdiction but fully discuss UIFSA may receive full
credit.]

Explanation to Point-One(b) (15%):

UIFSA does not apply to disputes over property division. Thus, the State A court may exercise
jurisdiction over the husband’s petition to modify the marital-residence-sale-proceeds provision of the
State B divorce decree because it has personal jurisdiction over the wife.

The State A court in which the husband brought his action has jurisdiction to adjudicate domestic
relations issues. The husband’s petition to modify the property settlement is a domestic relations issue.
The courts of State A may exercise personal jurisdiction over the wife because she was personally served
in State A.

UIFSA does not apply to divorce property-division disputes. Thus, although a State A court may not
adjudicate the husband’s petition to modify his child support obligations, it may adjudicate his property-
division claims. (Even though the court has jurisdiction, it may not modify the property-division award on
the merits.

Explanation to Point-Two(a) (20%):

A child support order may not be modified retroactively.

State courts have long held that obligations to pay child support ordinarily may not be modified
retroactively. “If the hardship is particularly severe, the courts sometimes devised a way to protect the
obligor, but in most instances the courts hold that retroactive modification of this kind is beyond their
power and indeed the governing statute may so provide.”

Federal law now goes further and requires the states, as a condition of federal child-support funding, to
adopt rules that absolutely forbid retroactive modification of the support obligation. The states have
adopted rules consistent with the federal requirements.

Seperac-J19 Exam-Released MEE Essay Compilation © 2016-2020 876


Explanation to Point-Two(b) (25%):

It is unclear whether the husband could obtain prospective downward modification of his child support
based on his voluntary acceptance of a job with a lower salary.

Prospective modification of a child support order is typically available only when the petitioner can show
a substantial change in circumstances. A significant decrease in income is typically viewed as a
substantial change.

However, when a parent seeks to modify a child support obligation because he has voluntarily reduced his
income, a court will not modify the obligation based solely on the income loss. Some courts refuse to
modify whenever the income shift was voluntary. Others look primarily to the petitioner’s intentions and
permit downward modification if he has acted in good faith. Many courts use a multifactor approach.

Here, there is no question that the husband’s loss of income was voluntary. In a jurisdiction in which
voluntary income reduction bars support modification, the husband’s petition would be denied.

In a jurisdiction employing a good-faith or multifactor approach, it is possible, but not certain, that the
husband could obtain downward modification. The evidence supports the husband’s good faith: his
change in employment appears to be based on his new job’s greater responsibilities and better promotion
possibilities. In a jurisdiction using a multifactor approach, the court would likely also consider the impact
of such a shift on the daughter, the likely duration of the husband’s income loss, and the likelihood of a
promotion that would ultimately inure to the daughter’s benefit. Thus, on these facts, it is possible, but by
no means certain, that the husband could prospectively obtain downward modification of his child support
obligation to his daughter.

Explanation to Point-Two(c) (15%):

A divorce property-division award is not subject to modification.

A support order is aimed at meeting the post-divorce needs of the supported individual. Because the
future is unpredictable, courts are empowered to modify a support award to take account of changed
circumstances that may occur during the period in which support is paid.

By contrast, a property-distribution award divides assets of the marriage based on the equities at the time
of divorce. Because the past can be ascertained, a property-division award is not subject to post-divorce
modification.

Here, the husband is seeking modification of a property-division award with respect to an asset owned by
the parties at the time of divorce. Thus, the husband may not obtain a modification of the marital-
residence-sale-proceeds provision of the divorce decree based on his reduced income.

Seperac-J19 Exam-Released MEE Essay Compilation © 2016-2020 877


#200-JUL 2013–MEE Q03: QUESTION THREE (FAMILY LAW)

Seven years ago, a married couple had a daughter.

Recently, the mother joined a small religious group. The group’s members are required to contribute at
least half their earnings to the group, to forgo all conventional medical treatments, and to refrain from all
“frivolous” activities, including athletic competitions and sports. The mother has decided to adhere to all
of the group’s rules.

Accordingly, the mother has told the father that she has given half of her last two paychecks to the group
and that she plans to continue this practice. The father objects to this plan and has accurately told the
mother that “we can’t pay all the bills without your salary.”

The mother has also said that she wants to stop giving their daughter her prescribed asthma medications.
The father opposes this because the daughter has severe asthma, and the daughter’s physician has said that
regular medication use is the only way to prevent asthma attacks, which can be life-threatening. The
mother also wants to stop the daughter’s figure-skating lessons. The father opposes this plan, too, because
their daughter loves skating. Because the father works about 60 hours per week outside the home and the
mother works only 20 hours, the father is afraid that the mother will do what she wants despite his
opposition.

The mother, father, and daughter continue to live together. They do not live in a community property
jurisdiction.

1(a). Can the father or the state child welfare agency obtain an order enjoining the mother from
making contributions from her future paychecks to the religious group? Explain.

1(b). Can the father or the state child welfare agency obtain an order requiring the mother to take
the daughter to skating lessons? Explain.

1(c). Can the father or the state child welfare agency obtain an order requiring the mother to
cooperate in giving the daughter her prescribed asthma medications? Explain.

2. If the father were to file a divorce action against the mother, could a court award custody of the
daughter to him based on the mother’s decision to follow the religious group’s rules? Explain.

Seperac-J19 Exam-Released MEE Essay Compilation © 2016-2020 878


#200: J13-3 MEE: ANSWER: NCBE (FAMILY LAW)

POINT (1)(a) [30%] ISSUE: On what, if any, basis could a court enjoin the mother from making
contributions from her future paychecks to the religious group? ANSWER: No. There is no basis
for a court to order the mother to stop making contributions from her paycheck to the religious
group. Because the mother and father live in an intact household in a non-community-property
state, the father has no management powers with respect to the mother’s earnings. Although the
mother has a support obligation toward the father that would enable a creditor to recover from the
mother the value of “necessaries” furnished to the father, the father could not personally obtain a
court order requiring the mother to spend her earnings in a particular way, nor could a state child
welfare agency.

POINT (1)(b) [20%] ISSUE: On what, if any, basis could a court order one parent to follow the
child-rearing preferences of the other parent when both parents and their child are living together
in an intact family? ANSWER: Because the mother and father are living with their daughter in an
intact family and failure to take skating lessons does not endanger the daughter, a court may not
require the mother to follow the father’s preference with respect to continuing the daughter’s
skating lessons.

POINT (1)(c) [30%] ISSUE: On what, if any, basis could a court order a parent to cooperate in
ensuring that a child obtains medical treatments necessary to protect the child’s health and life?
ANSWER: Because failing to take asthma medications poses a serious risk to the daughter’s health
and safety, if the mother’s actions prevent the daughter from receiving her prescribed medications,
a court may find that the daughter is neglected and order the mother to give the daughter her
medications.

POINT (2) [20%] ISSUE: In a divorce action, could a court grant custody to one parent based on
the other parent’s religious practices when those practices pose risks to the child? ANSWER: Yes.
Although a court may not deny a parent custody based on the parent’s religious faith, it may deny
custody based on a threat to the child’s health or safety. Because failing to take asthma medications
is life-threatening, a court could, in a divorce action filed by the father, award custody to the father
if the court finds that the mother will persist in her refusal to provide the daughter with her
medications.

ANSWER DISCUSSION:

There is no basis for a court to order the mother to stop making contributions from her paychecks to the
religious group. Because the mother and father live in a common-law jurisdiction in an intact household,
the father has no management powers with respect to the mother’s earnings. Although, in most states, the
mother has a support obligation toward the father that would enable a creditor to recover from the mother
the value of “necessaries” furnished to the father, the father may not personally obtain a court order
requiring the mother to spend her earnings in a particular way, nor may a state child welfare agency.
Based on the family privacy doctrine, a court may not order one parent to follow the child-rearing
preferences of the other parent when the parents live together with their child in an intact family. Thus, a
court will not order the mother to take the daughter to skating lessons. However, based on its neglect
jurisdiction, a court could order a parent to take steps to preserve a child’s health or safety, even when the
parent’s actions are religiously motivated. The proper party to initiate a child neglect action is the state,

Seperac-J19 Exam-Released MEE Essay Compilation © 2016-2020 879


not a parent. Thus, a court could order the mother to follow the physician’s medical recommendations and
order appropriate services for the daughter if the mother refused to do so and the state’s child welfare
agency filed a neglect petition on the daughter’s behalf. Although a court may not deny a parent custody
based on a parent’s religious faith, it may deny custody based on a threat to the child’s health or safety.
Because failing to take asthma medications appears to be life-threatening, if the father were to file a
divorce action against the mother, a court could award the father custody of his daughter if the mother
persists in her refusal to provide the daughter with her asthma medication.

ANSWER EXPLANATION:

Explanation to Point-One(a) (30%):

There is no basis for a court to order the mother to stop making contributions from her paycheck to the
religious group. Because the mother and father live in an intact household in a non-community-property
state, the father has no management powers with respect to the mother’s earnings. Although the mother
has a support obligation toward the father that would enable a creditor to recover from the mother the
value of “necessaries” furnished to the father, the father could not personally obtain a court order
requiring the mother to spend her earnings in a particular way, nor could a state child welfare agency.

Under the common law, when a woman married, her identity was swallowed up in her husband’s. As a
result of this marital-unity doctrine, a married woman could not own property. Beginning in the mid-19th
century, legislatures began to enact so-called “Married Women’s Property Acts” that restored to the
married woman the rights she had when unmarried, including the right to acquire, own, or transfer
property. By the end of the 19th century, every state had passed such a statute.

Under the Married Women’s Property Acts, title determines asset ownership and management rights; “the
wife is given the power to contract and full rights to her own earnings.” Because each spouse has full
management rights with respect to his or her earnings, a court may not overrule the spending decisions of
a spouse based on his or her partner’s conclusion that those decisions are ill-advised.

Marriage does, however, create support obligations and, since the Supreme Court’s decision in Orr v. Orr,
those obligations have been gender neutral. Based on this mutual support obligation, a creditor who has
furnished “necessaries” to a husband or wife may, in most states, sue the spouse of the purchaser and
recover on the debt.

However, the necessaries doctrine is available only to a creditor who has already provided goods or
services. Such a creditor has no power to obtain an order altering a spouse’s future spending. The doctrine
does not go so far as to allow one spouse to enjoin expenditures by the other because, as here, he or she is
concerned that the bills of the family won’t get paid. Nor, because of the family privacy doctrine, may a
spouse who disagrees with his partner’s spending decisions obtain an order requiring a different spending
pattern or obtain a support award.

A state child welfare agency may intervene in family decision making only when the decision at issue
endangers the well-being of a child or another family member incapable of protecting his own interests.
Here, there is no evidence that the mother’s decision to donate a portion of her earnings to the religious
group endangers her daughter. Thus, the state agency cannot obtain an order requiring the mother to alter
her spending patterns any more than can the father.

Explanation to Point-One(b) (20%):

Seperac-J19 Exam-Released MEE Essay Compilation © 2016-2020 880


Because the mother and father are living with their daughter in an intact family and failure to take skating
lessons does not endanger the daughter, a court may not require the mother to follow the father’s
preference with respect to continuing the daughter’s skating lessons.

American courts have consistently treated the disputes of intact families as private matters that should be
resolved at home. Thus, they have refused to decide such disputes when a spouse has brought one to
court:

The inherent jurisdiction of courts of equity over infants is a matter of necessity, assumed by the courts
only when it is forfeited by a natural custodian incident to a broken home. The judicial mind and
conscience is repelled by the thought of disruption of the sacred marital relationship, and usually voices
the hope that the breach may somehow be healed by mutual understanding between the parents
themselves.

Even when the spouse who comes to court wishes to enforce a premarital agreement, courts have refused
to intervene in the disputes of couples who are living together. For example, one court refused to enforce
a provision of a premarital agreement that required the child’s education at a religious school.

Here, the mother and father are still living together with their daughter. Until and unless they separate, a
court will not intervene in their dispute regarding the daughter’s skating lessons and issue an order
supporting the views of either parent.

Again, the state child welfare agency may intervene in family decision making only when the decision at
issue endangers the well-being of a child or another family member incapable of protecting his own
interests. Here, because there is no evidence that the mother’s failure to continue skating lessons would
endanger the daughter, the state agency cannot obtain an order requiring the mother to continue the
lessons any more than can the father.

Explanation to Point-One(c) (30%):

Because failing to take asthma medications poses a serious risk to the daughter’s health and safety, if the
mother’s actions prevent the daughter from receiving her prescribed medications, a court may find that the
daughter is neglected and order the mother to give the daughter her medications.

Although courts will not intervene in disputes between parents in an intact household, the state, pursuant
to its jurisdiction over child abuse and neglect, may obtain an order overruling a parental decision and
ordering appropriate services, including medical care, whenever the parental child-rearing decisions
endanger the child.

The fact that parental rights are constitutionally protected does not alter this result, even when the parental
choice is religiously motivated: “the power of the parent, even when linked to a free exercise claim, may
be subject to limitation if it appears that parental decisions will jeopardize the health or safety of the child,
or have a potential for significant social burdens.” Although the court may not inquire into parent’s
general religious beliefs, it may inquire into religious practices, if such practices may adversely affect the
physical or mental health or safety of the child.

[NOTE: Here, a child-neglect action would most likely be triggered by the daughter’s physician, who is
almost certainly a mandated neglect reporter under state law. However, the father could also file a
neglect report. A neglect action is typically commenced after someone with knowledge of a situation that
endangers the child files a report with the appropriate state agency. State laws typically require health

Seperac-J19 Exam-Released MEE Essay Compilation © 2016-2020 881


care workers, among others, to report suspected abuse and neglect. After receiving a report, the agency
investigates and files a neglect petition on the child’s behalf if its investigation confirms the allegations of
neglect. If the court finds that a child is neglected, it may order parents to undertake appropriate actions,
order a state agency to provide the child with services in the home, or even remove the child from
parental custody and place her in foster care.]

Explanation to Point-Two (20%):

Although a court may not deny a parent custody based on the parent’s religious faith, it may deny custody
based on a threat to the child’s health or safety. Because failing to take asthma medications is life-
threatening, a court could, in a divorce action filed by the father, award custody to the father if the court
finds that the mother will persist in her refusal to provide the daughter with her medications.

A child custody contest between parents is decided on the basis of the child’s best interests. Under the
best interests test, the court is free to consider a wide range of factors.

With respect to parental religion, the Establishment Clause forbids a court to favor one religion over
another. However, although “the court cannot evaluate the religion, it can instead evaluate the parent’s
ability to provide for the physical and mental health needs of the child.” Because of the sensitivity of the
constitutional issues at stake, courts have typically refused to consider religion unless the evidence shows
that the parent’s religious practice would imperil the child’s well-being. Some courts have additionally
required that a custody order “make the least possible infringement upon the parent’s liberty interests
consistent with the child’s well-being.”

Here, the daughter’s physician has said that failure to take asthma medications is life-threatening. Given
that the daughter is only seven and needs parental supervision in order to ensure medication compliance,
should the father file a divorce action, a court may thus deny the mother custody based on her refusal to
administer the medications, and may award custody to the father.

Seperac-J19 Exam-Released MEE Essay Compilation © 2016-2020 882


#201-JUL 2012–MEE Q04: QUESTION FOUR (FAMILY LAW)

Fifteen years ago, Mom and Dad were married in State A, where both were domiciled.

Fourteen years ago, Mom gave birth to Daughter in State A. Dad is Daughter’s biological father.

Four years ago, Dad died in State A. After Dad’s death, Mom relied heavily on Dad’s parents,
Grandparents. Mom and Daughter moved to an apartment near Grandparents in State A. Thereafter,
Grandparents visited Mom’s home at least once a week. Daughter was also a frequent visitor at
Grandparents’ home. Grandparents also helped Mom to support Daughter financially.

Four months ago, Mom married Stepdad and moved with Daughter to Stepdad’s home in State B, 500
miles from Mom’s former residence in State A. Stepdad believes that Grandparents discouraged Mom’s
marriage to him, and he asked Mom not to invite Grandparents to visit. Mom agreed to Stepdad’s request.
However, she allowed Daughter to visit Grandparents in State A during a school vacation.

One week ago, Grandparents sent Daughter a bus ticket. Without revealing her plans to Mom, Daughter
used the ticket to go to Grandparents’ home in State A. When she arrived at Grandparents’ home,
Daughter telephoned Mom and said, “I hate State B, I dislike Stepdad, and I want to live with
Grandparents in State A until you leave Stepdad and return to State A, too.”

On the same day that Mom received this telephone call, she was served with a summons to appear in a
State A court proceeding, brought by Grandparents, in which Grandparents seek custody of Daughter.
Grandparents’ petition was brought pursuant to a State A statute that authorizes the award of child
custody to a grandparent when the court finds that (1) the “child has been abandoned or one of the child’s
parents has died” and (2) an award of custody to the petitioner grandparent “serves the child’s best
interests.”

Both State A and State B have enacted the Uniform Child Custody Jurisdiction and Enforcement Act
(UCCJEA).

Mom has sought advice from your law firm. She asks the following questions:

1. Does State A have jurisdiction to award custody of Daughter to Grandparents? Explain.

2. On the merits, may a court deny Grandparents’ custody petition if Daughter testifies that she
wants to live with Grandparents? Explain.

3. Is the State A statute authorizing the award of custody to grandparents constitutional? Explain.

Seperac-J19 Exam-Released MEE Essay Compilation © 2016-2020 883


#201: J12-4 MEE: ANSWER: NCBE (FAMILY LAW)

POINT (1) [40%] ISSUE: When a child has no “home state,” what state has jurisdiction to issue a
valid child-custody decree? ANSWER: State A has jurisdiction because no state is Daughter’s
“home state,” Daughter and Mom both have significant connections with State A, and substantial
evidence is available in State A regarding Daughter’s care.

POINT (2) [25%] ISSUE: Must a court defer to the preference of an older child when awarding
custody? ANSWER: No. Although the wishes of an older child are relevant to a custody
determination and are typically given great weight, they are not determinative; a court may thus
deny Grandparents’ custody petition over the express opposition of Daughter.

POINT (3) [35%] ISSUE: Under the U.S. Constitution, may a state statute authorize a court to
grant a grandparent custody of a child without giving any special weight to the best-interests
determination of a fit and available parent? ANSWER: No. Because the State A statute gives “no
special weight” to a parent’s determination of her child’s best interests, under the Supreme Court’s
plurality opinion in Troxel v. Granville, the statute is unconstitutional.

ANSWER DISCUSSION:

Under the Uniform Child Custody Jurisdiction and Enforcement Act (UCCJEA), when a child has no
“home state,” i.e., a state in which the child has resided for the prior six months, a state may exercise
jurisdiction based on (1) “significant connections” with the child and at least one parent and (2) the
existence of “substantial evidence” relating to child custody in the forum jurisdiction. Here, State A has
significant connections with Daughter and Mom; substantial evidence related to child custody exists in
State A, where Daughter and Grandparents are currently located and where Daughter was domiciled until
very recently. Thus, State A has jurisdiction to make a custody award. On the merits, a court may deny
Grandparents’ custody petition even though Daughter strongly prefers to live with them. In all states, the
preferences of a child are relevant to a determination of the child’s best interests in a custody proceeding,
but they are not determinative. However, the State A statute under which Grandparents have brought a
custody petition is probably unconstitutional. In Troxel v. Granville, a plurality of the Supreme Court
ruled that a state statute which authorized a court to grant visitation to “any person” based on a finding
that such visitation was in “the best interest of the child” unconstitutionally interfered with the
fundamental right of parents to rear their children. Although State A’s statute authorizing the award of
custody to a nonparent based on the best interest of the child is limited to grandparents, it similarly grants
“no special weight” to a parent’s determination of her child’s best interest and thus does not comport with
the standards set forth in Troxel.

ANSWER EXPLANATION:

Explanation to Point-One (40%):

State A has jurisdiction because no state is Daughter’s “home state,” Daughter and Mom both have
significant connections with State A, and substantial evidence is available in State A regarding Daughter’s
care.

Under the Uniform Child Custody Jurisdiction and Enforcement Act, which has been widely adopted, a
state has jurisdiction to make an initial child-custody determination if “this State is the home state of the

Seperac-J19 Exam-Released MEE Essay Compilation © 2016-2020 884


child on the date of the commencement of the proceeding, or was the home State of the child within six
months before the commencement of the proceeding and the child is absent from this State but a parent or
person acting as a parent continues to live in this State” and no other state’s courts would have
jurisdiction under the above standard or other courts having jurisdiction have declined to exercise it.

Neither State A nor State B can claim jurisdiction under the UCCJEA. A “home state” is the state in
which the child “lived with a parent or a person acting as a parent for at least six consecutive months
immediately before the commencement of a child-custody proceeding.” Here, Daughter and Mom moved
to State B only four months before the State A custody petition was filed in State A, so State B is not
Daughter’s home state. State A was the home state of Daughter within six months of the proceeding
commenced in State A. However, State A does not have home-state jurisdiction because neither statutory
condition has been met: a parent (Mom) does not continue to live in State A, and the child is not absent
from State A.

Under the UCCJEA, when – and only when – “a court of another state does not have jurisdiction under
paragraph 1” (i.e., the home-state rule), a court may exercise jurisdiction over a child-custody
determination if

(2)(A) the child and at least one parent have a significant connection with this State other than mere
physical presence; and

(B) substantial evidence is available in this State concerning the child’s care, protection, training, and
personal relationships.

Here, a State A court would have jurisdiction under § 201 because Daughter and Mom both have a
significant connection with State A and substantial evidence relating to Daughter’s care is available in
State A: Mom was married in State A, gave birth to Daughter there, and lived there with Daughter from
the time of her birth until four months ago. Daughter is currently in State A, and so are Grandparents.
Other evidence related to Daughter’s schooling, friendships, and other personal relationships is also
available in State A.

[NOTE: State B would also have jurisdiction under § 201: Daughter and Mom both have a significant
connection with State B as it is where they both lived until Daughter’s recent return to State A and
substantial evidence from Mom, Stepdad, Daughter’s school, etc., is available in State B. However, under
UCCJEA § 206, a court “may not exercise its jurisdiction if a proceeding concerning the custody of the
child has been commenced in a court of another State having jurisdiction substantially in conformity with
this Act.” Thus, State B could not exercise jurisdiction under § 201 because a petition has been filed in
State A.]

The federal Parental Kidnapping Prevention Act (PKPA) sets out child-custody jurisdictional standards
much like those contained in the UCCJEA. Under the PKPA, a State A court would have jurisdiction
because no state is Daughter’s home state, “the child and at least one contestant have a significant
connection with State A other than mere physical presence and there is available in such State substantial
evidence concerning the child’s present or future care. Like the UCCJEA, the PKPA provides that where
a court of another state is exercising jurisdiction that complies with the federal standard, another court
shall not exercise jurisdiction.

Thus, State A has jurisdiction over the custody action initiated by Grandparents.

[NOTE: The UCCJEA had been adopted in almost every state.]

Seperac-J19 Exam-Released MEE Essay Compilation © 2016-2020 885


Explanation to Point-Two (25%):

Although the wishes of an older child are relevant to a custody determination and are typically given great
weight, they are not determinative; a court may thus deny Grandparents’ custody petition over the express
opposition of Daughter.

In all states, the views of a child who is mature enough to form and express a preference are relevant to a
custody determination. The wishes of older children like Daughter are typically given substantial weight.
However, the child’s wishes “are not treated equally in every case”:

Sometimes the child’s wishes are given controlling effect, while at other times the wishes are disregarded
altogether. The circumstances determining what effect, between these two extremes, should be given to a
child’s wishes in a particular case are, in addition to the comparative effect of objective factors affecting
its welfare generally, and in addition to the natural right of the child’s parent to have its custody
(frequently invoked successfully, at least in the absence of the parent’s long-term abandonment of the
child to another’s custody): the child’s age and judgment capacity; the basis for and strength of its
preference, generally; the treatment extended to the child by the contestants for its custody; and the
wrongful inducement of the child’s wishes.

Although there are a handful of states in which, by statute, the court must defer to the wishes of an older
child when choosing between fit parents, even if State A has such a statute it would not be applicable
here, as the custody contest is between a fit parent and a nonparent.

Thus, although the wishes of Daughter are relevant in an action regarding her custody, a court may
disregard those wishes and deny Grandparents’ custody petition.

Explanation to Point-Three (35%):

Because the State A statute gives “no special weight” to a parent’s determination of her child’s best
interests, under the Supreme Court’s plurality opinion in Troxel v. Granville, the statute is
unconstitutional.

In Troxel v. Granville, the Supreme Court addressed the constitutionality of a state statute under which
“any person” could petition for visitation rights “at any time” and authorized a court to grant such
visitation whenever it concluded that “visitation may serve the best interest of the child.” A plurality of
the Court found that the statute provided inadequate protection for a parent’s constitutionally protected
liberty interest in the care, custody, and control of her child. The statute at issue was, in the Court’s view,
“breathtakingly broad,” and it required the court to give “no special weight at all to a parent’s
determination of her daughter’s best interests.” Moreover, the court that awarded visitation pursuant to the
statute used a decision-making framework that “gave no weight” to the parent’s having “assented to
visitation” on a schedule different from that preferred by the grandparent petitioners and that
“contravened the traditional presumption that a fit parent will act in the best interest of his or her child.”

Under Troxel, the State A grandparent-custody statute is almost certainly unconstitutional. Like the
statute struck down in Troxel, the State A statute does not require the court to give any special weight to a
parent’s determination of her child’s best interest; in fact, it does not appear to require the court to give
any weight to a parent’s determination of a child’s best interest. Moreover, the State A statute permits a
much larger intrusion upon parental prerogatives than did the statute considered in Troxel; that statute
permitted a court to grant a nonparent visitation, while the State A statute authorizes a court to award a
nonparent custody.

Seperac-J19 Exam-Released MEE Essay Compilation © 2016-2020 886


Although the State A statute does restrict nonparent custody petitions to grandparents and additionally
requires that the child have been abandoned or have a parent who has died, this should not alter the
constitutional analysis. These facts are not relevant to the Supreme Court’s reasoning in Troxel.
Moreover, Troxel itself involved a visitation petition brought by grandparents whose son had died.

Seperac-J19 Exam-Released MEE Essay Compilation © 2016-2020 887


#202-JUL 2011–MEE Q06: QUESTION SIX (FAMILY LAW/CONFLICTS)

Dave and Meg lived in State A. Three years ago, they began dating. Two years ago, Meg became
pregnant with their child. Shortly thereafter, Dave and Meg discussed marriage. Dave told Meg, “Perhaps
we should get married if we’re going to have a child.” Meg told Dave, “I am committed to marrying you,
but I want a real wedding, and we can’t afford that now.” Meg proposed that Dave move in with her so
that “we can save money to get married.” Dave agreed and began living in Meg’s rented apartment. Meg
did not tell her landlord about Dave. She did tell her family and friends that “Dave, my fiancé, has moved
in.”

Fifteen months ago, Meg gave birth to Child. Meg and Dave agreed that Child’s birth certificate would
identify Dave as Child’s father. Meg and Dave sent birth announcements to friends and relatives noting
the birth of “our son, Child.” After Child’s birth, Meg quit work. Dave took on a second job in order to
support Meg and Child.

Five months ago, Meg took Child and abruptly left Dave. Dave hired a private investigator to find Meg
and Child. The investigator recently discovered that they are living in State B with Husband, whom Meg
married three months ago. The investigator also discovered that Meg and Husband have filed a petition to
terminate Dave’s parental rights and authorize Husband’s adoption of Child.

State B permits a mother or a married father to veto the adoption of his or her child unless he or she “has
willfully refused to support said child for a period of one or more years.”

State B permits an unmarried father to veto the adoption of his child only if he (a) “has consistently
supported such child” and (b) “has maintained a residential relationship with such child for at least 9 of
the 12 months immediately preceding the filing of an adoption petition.”

State B does not have a putative father registry.

State A recognizes common law marriage. State B does not. Both State A and State B have adopted the
Uniform Child Custody Jurisdiction and Enforcement Act (UCCJEA).

1. Will State B recognize a common law marriage contracted in State A? Explain.

2. Did Dave and Meg enter into a common law marriage in State A? Explain.

3. Assuming that Dave is an unmarried father, can State B constitutionally grant Meg and
Husband’s adoption petition over Dave’s opposition? Explain.

4. Does the UCCJEA permit State B to terminate Dave’s parental rights or issue an order awarding
custody of Child to Meg? Explain.

Seperac-J19 Exam-Released MEE Essay Compilation © 2016-2020 888


#202: J11-6 MEE: ANSWER: NCBE (FAMILY LAW/CONFLICTS)

POINT (1) [20%] ISSUE: Is a common law marriage that is valid where contracted also valid in a
state that does not recognize common law marriage? ANSWER: Yes. A common law marriage that
is valid where contracted is also valid in a state that does not permit common law marriage.

POINT (2) [20%] ISSUE: What are the requirements for a valid common law marriage?
ANSWER: Dave and Meg did not enter into a common law marriage.

POINT (3) [40%] ISSUE: Does a state statute that permits the adoption of an unmarried father’s
child when he has not “maintained a residential relationship” with the child for “at least 9 of the 12
months immediately preceding the filing of an adoption petition” violate the U.S. Constitution when
applied to a committed residential parent? ANSWER: Dave is a committed father who consistently
held himself out as a parent, resided with his child, and provided support for his child and the
child’s mother. Thus, Dave has a strong argument that State B is constitutionally precluded from
permitting his child’s adoption without his consent.

POINT (4) [20%] ISSUE: Under the UCCJEA, when may a state issue an order (a) terminating
parental rights and (b) making an initial custody determination? ANSWER: Because State A has
been Child’s home state within the past six months and Dave, a parent, continues to live in State A,
State B does not have jurisdiction to terminate Dave’s parental rights or to make an initial child-
custody determination respecting Child.

ANSWER DISCUSSION:

A common law marriage valid where contracted is valid in all states. To enter into a valid common law
marriage, a couple must cohabit, hold themselves out as a married couple, and enter into a present
agreement that they are married. Although Meg and Dave may have agreed to be married in the future and
Meg has held Dave out as a future husband (her fiancé), this conduct is inadequate to establish a common
law marriage. Thus, Dave would be treated as an unmarried father under State B’s adoption-consent
statute. Dave cannot satisfy the statutory requirements to veto his child’s adoption because he has not
lived with Child for at least 9 of the 12 months prior to filing of the adoption petition. However, the State
B statute is almost certainly unconstitutional as applied to Dave because Dave is a committed father who
consistently held himself out as Child’s father and both lived with and supported Child until Meg’s
unannounced departure to State B. The UCCJEA permits a state to make an “initial child-custody
determination” only if it is the child’s “home state” when the child-custody proceeding was commenced
or was the home state of the child within six months before the commencement of the proceeding and a
parent continues to live in the state. A child-custody proceeding includes a proceeding to terminate
parental rights. Thus, because Meg took Child from State A only five months ago, State B does not have
jurisdiction to terminate Dave’s parental rights or to issue a child custody order.

ANSWER EXPLANATION:

Explanation to Point-One (20%):

A common law marriage that is valid where contracted is also valid in a state that does not permit
common law marriage.

Seperac-J19 Exam-Released MEE Essay Compilation © 2016-2020 889


Under accepted conflict-of-laws principles, a marriage valid under the law of the place in which it was
contracted is valid elsewhere unless it violates a strong public policy of the state which has the most
significant relationship with the spouses and the marriage. Perhaps because the vast majority of states
once permitted common law marriage, courts in states that do not recognize common law marriage have
consistently held that, if a man and woman were domiciled in a state that permits common law marriage
and their conduct met the requirements of that state’s law for establishing such a marriage, recognition of
the marriage does not violate a strong public policy. Thus, if Dave and Meg entered into a valid common
law marriage in State A, it would be recognized by a State B court.

A common law marriage, once established, is the equivalent of a ceremonial marriage. Thus, if Dave can
show that he and Meg entered into a common law marriage, he would be treated as a married father under
State B’s adoption law.

Explanation to Point-Two (20%):

Dave and Meg did not enter into a common law marriage.

To establish a common law marriage, the proponent must show: (1) capacity to enter a marital contract,
(2) a present agreement that the two parties are married, (3) cohabitation, and (4) “holding out” a marital
relationship to the community. The evidence here clearly establishes capacity and cohabitation. It also
suggests an agreement to become married. However, a common law marriage requires a present marriage
agreement. If Dave and Meg made any marriage agreement, it was an agreement to be married in the
future: Meg told Dave that she was “committed to marrying him,” but that she “wanted a real wedding,
and we can’t afford that now;” Meg also proposed that Dave move in with her so that “we can save
money to get married.” There is also no evidence that Dave and Meg held themselves out as a married
couple. Although Meg said to friends and family that Dave was her fiancé, a fiancé is an intended spouse,
not a current marriage partner.

Thus, Dave and Meg did not enter into a valid common law marriage, and Dave would be categorized as
an unmarried father under State B’s adoption-consent statute.

Explanation to Point-Three (40%):

Dave is a committed father who consistently held himself out as a parent, resided with his child, and
provided support for his child and the child’s mother. Thus, Dave has a strong argument that State B is
constitutionally precluded from permitting his child’s adoption without his consent.

“When an unwed father demonstrates a full commitment to the responsibilities of parenthood by ‘coming
forward to participate in the rearing of his child,’ his interest in personal contact with his child acquires
substantial protection under the Fourteenth Amendment’s Due Process Clause.” Interpreting this standard,
the U.S. Supreme Court has held that a father who lived with his children and their mother and held the
children out as his own is entitled to a hearing to demonstrate his parental fitness before his parental rights
may be terminated. The Court has also held that the equal protection clause requires the state to give a
father the opportunity to veto his child’s adoption when state law grants a similarly situated unmarried
mother a veto.

On the other hand, the Court has held that a state adoption law that failed to grant an unmarried father a
right to veto his child’s adoption was constitutional when applied to a nonresidential father who provided
minimal support and did nothing to establish his legal paternity until after the adoption petition was filed.
The Court has also held that an adoption order entered without notice to the unmarried father was

Seperac-J19 Exam-Released MEE Essay Compilation © 2016-2020 890


constitutional when the state provided him with an opportunity to obtain notice and a hearing through
registration with a “putative father registry.”

Dave has demonstrated the kind of commitment that triggers protection under the Fourteenth
Amendment. Like the fathers whose constitutional rights were recognized in Stanley and Caban, Dave
held himself out as Child’s father; he consented to be identified as Child’s father on Child’s birth
certificate and sent birth announcements identifying himself as Child’s father. Like the fathers in Stanley
and Caban, Dave lived with Child and Child’s mother and provided support to them. After Child’s birth,
Dave was the only family breadwinner and held down two jobs in order to provide for Meg and Child. He
also had volunteered to marry Meg and thus would have been a marital father had Meg agreed. Dave is
also blameless for his failure to satisfy the statutory veto requirements; Dave would have lived with Child
and Meg for the relevant nine-month period but for Meg’s abrupt and unannounced departure from their
home. State courts have typically held adoption-consent statutes that would deny a veto right to a father
who has done everything in his power to establish a full relationship with his child are unconstitutional as
applied in such a case.

Although Dave does in some respects resemble the unsuccessful unmarried father in Lehr, it is likely that
Dave would still prevail. The Lehr father, like Dave, was forced to hire an investigator to find his child
and her mother. By the time he succeeded in doing so, an adoption petition had been filed. However, the
Lehr father did not receive notice of this petition because he had not established his paternity and had
failed to enter his name in the state’s putative father registry. Given that he could have obtained notice
and a hearing, the Supreme Court held that the state did not violate his constitutional rights when a judge
refused to delay signing the adoption petition in order to give the father time to file the appropriate papers
and participate in a hearing at which his objection could be heard.

Here, by contrast, Dave established his paternity by voluntarily acknowledging Child and causing his
name to be placed on Child’s birth certificate. Under federal law, a voluntary acknowledgment of
paternity is binding if unchallenged by the named father within 60 days. Additionally, State B does not
provide unmarried fathers with a putative father registry, and Dave thus had no way of obtaining notice of
an adoption proceeding.

Thus, State B’s adoption consent statute appears to be unconstitutional as applied to Dave, a committed
parent who has fully participated in the rearing of his child.

Explanation to Point-Four (20%):

Because State A has been Child’s home state within the past six months and Dave, a parent, continues to
live in State A, State B does not have jurisdiction to terminate Dave’s parental rights or to make an initial
child-custody determination respecting Child.

Under the Uniform Child Custody Jurisdiction and Enforcement Act, which has been adopted in 48 states,
a child’s “home state” has exclusive jurisdiction to issue an initial custody decree. A “home state” is a
state where the child has lived with “a parent, or a person acting as a parent, for at least six consecutive
months immediately before the commencement of a child-custody proceeding.” A “child-custody
proceeding” includes a “proceeding for termination of parental rights.” A home state continues to have
exclusive jurisdiction to issue an initial custody order even if the child is absent from the state when a
custody petition is brought, so long as no more than six months has elapsed since the child’s departure
from the jurisdiction and “a parent or person acting as a parent continues to live in” the home state. The
“physical presence of, or personal jurisdiction over, a party or a child is not necessary to make a child-
custody determination.”

Seperac-J19 Exam-Released MEE Essay Compilation © 2016-2020 891


Here, Meg took Child from the family home in State A only five months ago, and Dave still lives in State
A. Thus, State A has exclusive jurisdiction to issue an initial custody decree and also to terminate parental
rights. State B lacks jurisdiction to do either.

[NOTE: The UCCJEA does not apply to adoption proceedings, but termination of a parent’s rights is
typically a precondition for an adoption. Thus, as a practicality, the UCCJEA should preclude a State B
court from granting a petition for Husband’s adoption of Child.]

Seperac-J19 Exam-Released MEE Essay Compilation © 2016-2020 892


#203-FEB 2011–MEE Q05: QUESTION FIVE (FAMILY LAW)

Husband and Wife married 12 years ago. Two years later, Wife gave birth to Child. Both Husband and
Wife are employed, and each earns approximately $80,000 per year.

Four months ago, Husband and Wife decided to divorce and entered into a written separation agreement
drafted by their respective attorneys. Under this agreement, Wife obtained sole title to assets worth
$175,000 and Husband obtained sole title to assets worth $125,000. All assets were acquired during the
marriage with employment income; there were no other assets. The separation agreement provided that
Wife would have sole custody of Child. It required Husband to pay to Wife $500 per month in spousal
support until her death or remarriage and $400 per month in child support until Child reaches the age of
18.

At the time he signed the separation agreement, Husband was living with Fiancée, a woman with two
teenage children. Indeed, his planned marriage to Fiancée was the primary reason for Husband’s
willingness to sign the separation agreement.

Three months ago, Child was injured in an automobile accident. As a result of blood tests performed
following the accident, Husband discovered that he is not Child’s biological parent.

Two months ago, at a hearing in the Husband-Wife divorce action, Husband petitioned the trial court to
invalidate the separation agreement based on unconscionability and fraud. The trial court refused and
entered a divorce decree incorporating the terms of the separation agreement.

After entry of the divorce judgment, Husband and Fiancée got married. Husband then filed a motion to
modify the divorce decree to

(a) grant him an equal share of the marital assets,

(b) award Wife no more than $200 per month in spousal support so that Husband could “meet the needs
of his new family,” and

(c) eliminate his child-support obligation based on Husband’s “nonpaternity of Child.”

The trial court denied Husband’s motion to modify the divorce decree.

1. Did the trial court err in denying Husband’s petition to invalidate the separation agreement on
the basis of unconscionability and fraud? Explain.

2. Did the trial court err in denying Husband’s motion to modify the divorce decree according to
each of the terms set forth in his motion? Explain.

Seperac-J19 Exam-Released MEE Essay Compilation © 2016-2020 893


#203: F11-5 MEE: ANSWER: NCBE (FAMILY LAW)

POINT (1) [40%] ISSUE: On what basis may the property-division and support provisions of a
separation agreement be invalidated, and is there evidence to support invalidation in this case?
ANSWER: A separation agreement may be invalidated, in whole or in part, based on
unconscionability or fraud. Here, there are insufficient facts to support a finding of
unconscionability with respect to the property-division or spousal-support portions of the
agreement. Given the variety of approaches currently applied by the states to paternity
misrepresentation and disestablishment, it is unclear whether the trial court erred in denying
Husband’s petition to invalidate the child-support provisions of the separation agreement based on
fraud.

POINT (2) [10%] ISSUE: When, if ever, may the property-division provisions of a divorce decree
be modified? ANSWER: A divorce property-division award is not subject to modification.

POINT (3) [25%] ISSUE: When, if ever, may the spousal- and child-support provisions of a divorce
decree be modified? ANSWER: A spousal-support award may be modified when the court finds
that there has been a substantial change in a party’s circumstances. Because Husband knew of his
future obligations to his new spouse and stepchildren at the time he signed the agreement, there is
no change of circumstances in this case.

POINT (4) [25%] ISSUE: May the nonpaternity of a mother’s husband be established after entry of
a divorce judgment when the child is 10 years of age and the husband is the only father the child
has ever known? ANSWER: It is unclear whether the trial court erred in refusing to modify the
child-support award based on Husband’s nonpaternity of Child.

ANSWER DISCUSSION:

Both the support and property-division provisions of a separation agreement may be invalidated based on
a finding of unconscionability or fraud. Here, there are insufficient facts to support a finding of
unconscionability with respect to the property-division or spousal-support portions of the agreement.
Given the variety of approaches currently applied by the states to paternity misrepresentation and
disestablishment, it is unclear whether the court erred in denying Husband’s petition to invalidate the
child-support provisions of the separation agreement. A spousal- or child-support order may be modified
based on a change of circumstances after the order was entered. However, there has been no change of
circumstances in this case because Husband knew or should have known of the economic needs of his
intended wife and stepchildren before signing the agreement, and there is no evidence that those needs
have changed. It is unclear whether Husband can obtain modification of his child-support obligation
based on his nonpaternity of Child.

ANSWER EXPLANATION:

Explanation to Point-One (40%):

A separation agreement may be invalidated, in whole or in part, based on unconscionability or fraud.


Here, there are insufficient facts to support a finding of unconscionability with respect to the property-
division or spousal-support portions of the agreement. Given the variety of approaches currently applied
by the states to paternity misrepresentation and disestablishment, it is unclear whether the trial court erred

Seperac-J19 Exam-Released MEE Essay Compilation © 2016-2020 894


in denying Husband’s petition to invalidate the child-support provisions of the separation agreement
based on fraud.

All states authorize invalidation of a separation agreement, or a specific portion of such an agreement,
based on a finding of fraud or unconscionability. Here, there is no evidence of asset or income
misrepresentation. And, although judicial interpretations of the unconscionability test vary , the facts do
not appear to support an unconscionability finding with respect to the property-division or spousal-
support portions of the agreement. The parties were represented by separate counsel, and there is no
evidence of asset nondisclosure. Nor will the unconscionability doctrine serve to relieve a spouse whose
complaint is simply that he believes he made a bad bargain. It is not even clear that Husband made a bad
bargain. Although he did receive fewer assets than Wife, most states require equitable, not equal, division
of assets, and equity is determined through a complex assessment of contribution and need-based factors.
Moreover, the agreement requires Husband to pay only $10,800 per year (13.5 percent of his total
income) in combined child and spousal support. Many state child-support guidelines would require
Husband to pay this much or more in child support alone. Thus, it is highly unlikely that an appellate
court would find reversible error in the trial court’s refusal to invalidate these portions of the agreement
based on unconscionability.

It is unclear whether an appellate court would find reversible error in the trial court’s refusal to invalidate
the child-support portion of the agreement, however. It is now apparent that Husband is not Child’s
biological parent. Arguably, Wife’s failure to alert Husband to this possibility was a form of
misrepresentation which might support a finding of fraud. A few courts have permitted tort actions
against a wife who misrepresented her husband’s paternity. In a state with such case law, it is possible that
an appellate court might conclude that Wife’s conduct provided a basis for invalidating the child-support
provisions of the agreement.

Explanation to Point-Two (10%):

A divorce property-division award is not subject to modification.

Although a support award may be modified after a divorce decree has been entered, a property-division
award, whether it results from a judicial decision or a divorce settlement agreement, may not be modified
after a divorce decree has been entered. This distinction rests on a fundamental difference between
support and property division. A support order requires payment, over what may be a long period of time,
of post-divorce income, while a property-division award divides assets accrued during the marriage based
on the equities at the time of divorce. Property division thus reflects an evaluation of the past, and support
determination reflects an evaluation of the future. Because the future is unpredictable, courts are
empowered to modify a support award to take account of changed circumstances. Because the past can be
ascertained, courts are not empowered to reconsider that evaluation and modify a property-division award
once it has been embodied in a final judgment of divorce. Thus, the trial court did not err in refusing to
modify the property-division provisions of the divorce decree.

Explanation to Point-Three (25%):

A spousal-support award may be modified when the court finds that there has been a substantial change in
a party’s circumstances. Because Husband knew of his future obligations to his new spouse and
stepchildren at the time he signed the agreement, there is no change of circumstances in this case.

In all states, a spousal-support award may be modified based on a change in a party’s circumstances. Most
courts require that such a change be substantial.

Seperac-J19 Exam-Released MEE Essay Compilation © 2016-2020 895


A court may take into account legal obligations to a new spouse and/or children in determining whether
there has been a change in circumstances that warrants modification of a support award. However, in this
case, Husband does not have new adopted or biological children to whom he owes support; he has
stepchildren. In most states, a stepparent relationship creates no legal support obligations.

More importantly, a change in circumstances that is anticipated may not serve as a basis for modification
of a support order. Here, Husband knew about the support needs of Fiancée and her children at the time
he signed the separation agreement; Husband was living with Fiancée and the children at the time the
agreement was signed, and his planned marriage to Fiancée was the reason Husband was willing to sign
the agreement as presented. Thus, because Husband’s decision to sign the agreement and accept these
support obligations was made with full knowledge of his impending marriage, he cannot now claim that
marriage as a change of circumstances that warrants modification of his obligations under the divorce
decree. Therefore, the trial court should not modify the spousal-support award based on the needs of
Husband’s new wife and her children.

Explanation to Point-Four (25%):

It is unclear whether the trial court erred in refusing to modify the child-support award based on
Husband’s nonpaternity of Child.

Traditionally, courts have been extremely reluctant to terminate an established parent-child relationship
and have relied on a wide range of equitable and procedural principles to avoid doing so. Some courts
have found that a prior legal judgment, such as the divorce decree terminating Husband’s and Wife’s
marriage, bars the introduction of nonpaternity evidence based on application of res judicata or collateral
estoppel principles. Other courts have relied on the equitable doctrines of estoppel and laches to achieve
the same result. Moreover, some courts have simply cited the child’s best interest, without relying on any
specific legal principle, as justification either for denying permission to introduce evidence of
nonpaternity or for denying a petition to disestablish paternity. Under this latter approach, the trial court
correctly denied Husband’s petition to modify his support obligation based on his nonpaternity because
Child is 10 years old, Husband is the only father Child has known, and there is no evidence that another
father-child relationship could be established.

However, in recent years, a number of courts have placed greater emphasis on the interests of erroneously
identified fathers. For example, in M.A.S. v. Mississippi Department of Human Services, the Mississippi
Supreme Court held that a paternity judgment could be vacated more than nine years after its entry based
on DNA tests performed in an unrelated matter which revealed that the petitioner was not the child’s
biological father. Although M.A.S. involved a nonmarital child, the Mississippi courts have since applied
its reasoning in a case brought by a divorced petitioner. Although the petitioner had lived with the child
only during its first month of life and had limited contact thereafter, the court did not rely on the tenuous
parent-child relationship in granting the disestablishment petition. Instead, the court stated its belief that
“the best interest of the child is to know the identity of the natural father” and “refused to sanction the
manifest injustice of forcing a man to support a child which science has proven not to be his.”

Some states have also adopted new paternity disestablishment legislation that places greater emphasis on
the interests of erroneously identified fathers. For example, Georgia mandates paternity disestablishment
in certain circumstances when genetic testing demonstrates a zero percent probability of paternity. Twelve
states allow paternity disestablishment by a married father, and there is a strong trend toward enactment
of such legislation. Under a legislative or judicial standard which permits paternity disestablishment

Seperac-J19 Exam-Released MEE Essay Compilation © 2016-2020 896


without regard to the child’s relational and financial interests, a court could appropriately accept the blood
tests offered by Husband, enter a finding of nonpaternity, and terminate Husband’s support obligation.

In a state where the courts or the legislature has adopted rules favoring the ability of erroneously
identified fathers to prove nonpaternity, Husband’s motion to disestablish his paternity and eliminate his
child-support obligation would be more likely to succeed. Even in such states, if Husband was aware of
his nonpaternity and had an opportunity to litigate the issue at the time of entry of the divorce decree,
courts are likely to treat the divorce decree as res judicata on the paternity issue and conclude that there
are no changed circumstances warranting a modification of the child-support order. However, some courts
have allowed motions to modify a child-support order in a divorce decree on the basis of nonpaternity,
even when that issue was litigated in the original action, reasoning that a motion to modify is a direct
attack on the judgment, not a collateral attack, and therefore is not barred by res judicata principles.

Seperac-J19 Exam-Released MEE Essay Compilation © 2016-2020 897


#204-JUL 2010–MEE Q06: QUESTION SIX (FAMILY LAW)

Fifteen years ago, Husband and Wife married in State A. At the time of the marriage, Husband and Wife
were both 35 years old, and each had a high school education. Wife worked as an administrative assistant
for a manufacturing company, where she had been employed for six years. Husband was unemployed.

Husband was a spendthrift with a history of selling his possessions to fund unsuccessful business
ventures. Aware of his personal failings, Husband asked Wife to enter into a premarital agreement under
which (a) Wife would become the sole owner of all assets owned by Husband before his marriage to
Wife; (b) Wife would pay all of Husband’s premarital debts; (c) if Husband and Wife divorced, Wife
would have exclusive rights to all assets acquired by either Husband or Wife during their marriage; and
(d) if Husband and Wife divorced, both would waive all claims to alimony. Wife agreed to Husband’s
proposal. Husband and Wife both disclosed their assets to each other.

Attorney thereafter prepared a premarital agreement based on Husband’s and Wife’s understanding.
Husband and Wife signed the agreement one week after a meeting at which Attorney explained the
consequences of signing the agreement. The agreement contained a provision stating that both Husband
and Wife had chosen to forgo individual representation by a lawyer. After signing the agreement,
Husband transferred title to his assets to Wife.

Throughout the marriage, Wife has performed virtually all household chores. Wife has also worked full
time at the manufacturing company; at times, she has also held a part-time job to pay for household
expenses. Wife paid Husband’s premarital debts with her earnings and with some of the assets Husband
had transferred to her. She sold the balance of the assets Husband had transferred to her to pay for flying
lessons for Husband. Husband got a commercial pilot’s license after taking these lessons but never tried to
find work as a pilot. Instead, Husband worked at part-time odd jobs until five years ago, when he was
injured in a car accident. Thereafter, Husband claimed he was disabled and ceased working altogether.
Wife currently earns $35,000 per year.

Wife has sued Husband for divorce in State A, which terms spousal support “alimony.” Husband claims
that he is entitled to alimony and a share of the couple’s assets. Wife claims that she is entitled to all
assets titled in her name and in Husband’s name. These assets are (a) the marital home, purchased by
Wife during the marriage and titled in Wife’s name; (b) Wife’s employment pension; and (c) real estate
inherited by Husband during the marriage and titled in Husband’s name.

Wife has asked your law firm these questions:

1. Is the premarital agreement enforceable? Explain.

2. If the premarital agreement is unenforceable, what assets are divisible at divorce? Explain.

3. If the premarital agreement is unenforceable, is Husband entitled to alimony? Explain.

Seperac-J19 Exam-Released MEE Essay Compilation © 2016-2020 898


#204: J10-6 MEE: ANSWER: NCBE (FAMILY LAW)

POINT (1) [50%] ISSUE: Under what circumstances is a premarital agreement enforceable?
ANSWER: In determining whether a premarital agreement governing property distribution at
divorce is enforceable, courts consider whether the agreement was voluntarily made, whether it is
unconscionable, and whether disclosure of assets and obligations was made. The weight assigned to
these factors varies from state to state. Here, Husband may have a basis for challenging the validity
of the premarital agreement in a state that allows a court to strike down an agreement based on
unconscionability at the time enforcement is sought.

POINT (2) [30%] ISSUE: What property is subject to division at divorce? ANSWER: Unless a
valid premarital agreement specifies otherwise, in most states, property acquired during the
marriage through spousal effort is divisible at divorce, but property acquired through gift, descent,
or devise is not. Thus Husband could be awarded part of Wife’s pension and a share in the marital
home.

POINT (3) [20%] ISSUE: When is an alimony award appropriate? ANSWER: Even if the
premarital agreement is unenforceable, it is unclear whether Husband can obtain alimony.
Husband’s failure to contribute to the marriage weighs against his alimony claim, but, if Husband
can establish that his injury precludes employment, that factor coupled with his age and the
duration of the marriage may support an alimony award.

ANSWER DISCUSSION:

Although state rules governing the enforceability of a premarital agreement vary considerably, a
premarital agreement will generally be enforced unless the party against whom enforcement is sought can
show involuntariness, inadequate disclosure by the other spouse, or unconscionability. Although Husband
cannot show involuntariness or inadequate disclosure, it is possible that he can show unconscionability if
this jurisdiction evaluates unconscionability at the time enforcement of the agreement is sought. If the
agreement is unenforceable, in most states Husband’s inherited real estate would not be divisible at
divorce; the marital home titled in Wife’s name would be, and Wife’s pension would be partly marital and
partly separate. In considering whether to award alimony, courts consider a variety of factors, including
the parties’ ages, length of the marriage, contributions by both parties, and the parties’ physical and
mental health. Here, it is unclear whether a court would award Husband alimony.

ANSWER EXPLANATION:

Explanation to Point-One (50%):

In determining whether a premarital agreement governing property distribution at divorce is enforceable,


courts consider whether the agreement was voluntarily made, whether it is unconscionable, and whether
disclosure of assets and obligations was made. The weight assigned to these factors varies from state to
state. Here, Husband may have a basis for challenging the validity of the premarital agreement in a state
that allows a court to strike down an agreement based on unconscionability at the time enforcement is
sought.

Although courts were once hostile to premarital agreements, today all states permit spouses to contract
premaritally with respect to rights and obligations in property. In all states, the enforceability of such

Seperac-J19 Exam-Released MEE Essay Compilation © 2016-2020 899


agreements turns on three factors: voluntariness, unconscionability, and disclosure. How courts apply
these factors varies significantly from one state to the next. In many states, an agreement is unenforceable
if the party against whom enforcement is sought succeeds in showing any one of the three factors:
involuntariness, unconscionability, or inadequate disclosure.

Under the Uniform Premarital Agreement Act (UPAA), which has been adopted in 27 jurisdictions, the
party against whom enforcement is sought must prove (1) involuntariness or (2) that “the agreement was
unconscionable when it was executed” and that he or she did not receive or waive “fair and reasonable”
disclosure and “did not have, or reasonably could not have had, an adequate knowledge” of the other’s
assets and obligations. Thus, under the UPAA, a court may not refuse to enforce a premarital agreement
based on unconscionability unless the court also finds lack of adequate disclosure or knowledge.

Under the UPAA, Husband and Wife’s premarital agreement would be enforceable. The facts show that
Husband and Wife both disclosed their assets, and there is no evidence that Husband signed the
agreement involuntarily. Although lack of independent legal advice is a factor in determining whether a
party entered into an agreement voluntarily, Husband had ample opportunity to obtain legal counsel and
decided against it. Husband also had ample time to review the agreement before signing it, and, perhaps
most importantly, he proposed the agreement. For example, one court upheld the agreement when the
wife received the proposed agreement two weeks before the wedding and had it reviewed by legal
counsel.

In states that have not adopted the UPAA, courts employ a range of standards. In most of these states, a
court may invalidate a premarital agreement on grounds of unconscionability even if both parties have
fully disclosed their assets. In evaluating unconscionability, some states require courts to determine
whether the agreement was unconscionable when signed; others require a determination of
unconscionability at the time of divorce; yet other states permit courts to invalidate an agreement based on
unconscionability at either point in time or to invalidate an agreement simply because it is unfair. And
some states specifically disallow spousal support waivers or apply special standards to such waivers.

Here, the agreement initially benefited Husband by ensuring payment of his debts and protecting him
against his own imprudent speculation; if unconscionability is analyzed at the time the agreement was
signed, the court will likely find it enforceable. At the time of divorce, however, the agreement is
arguably unconscionable in that it would grant Wife not only all marital assets, but also Husband’s
inherited real estate, an asset that, in most states, would be Husband’s separate property. Moreover,
Husband has suffered a serious injury and might succeed in establishing that he is incapable of self-
support. Thus, in a state that permits invalidation of an agreement based on unconscionability at the time
of divorce, a court could find that the premarital agreement between Husband and Wife is unconscionable
and unenforceable. In a state that disallows spousal support waivers, this provision might also be struck
down even if the property division provision were upheld.

[NOTE: A number of states that have adopted the UPAA have varied its enforcement provisions. For
example, although the UPAA provides that “parties to a premarital agreement may contract with respect
to the modification or elimination of spousal support,” some UPAA states have disallowed spousal
support waivers, or allowed such waivers only in limited circumstances. In Illinois, a spousal-support
waiver is unenforceable if it causes the waiving spouse undue hardship in light of circumstances not
reasonably foreseeable at the time of signing. Some states permit a court to invalidate a premarital
agreement on grounds of unconscionability even when there has been full asset disclosure.]

Explanation to Point-Two (30%):

Seperac-J19 Exam-Released MEE Essay Compilation © 2016-2020 900


Unless a valid premarital agreement specifies otherwise, in most states, property acquired during the
marriage through spousal effort is divisible at divorce, but property acquired through gift, descent, or
devise is not. Thus Husband could be awarded part of Wife’s pension and a share in the marital home.

In all states, a divorce court may divide marital property without regard to title. In the majority of states, a
divorce court may not divide separate property. In a minority of “hotchpot” jurisdictions, the court may
divide all assets, whenever or however acquired; a few states permit the division of separate property in
special circumstances, such as hardship.

An asset is marital if it was acquired during the marriage by any means other than gift, descent, or devise.
An asset that is initially separate property may be transformed into marital property if marital funds or
significant efforts by the owner-spouse enhance its value or build equity during the marriage.

Here, if State A is a majority jurisdiction, Husband’s real estate would be classified as separate property,
the marital home as marital, and Wife’s pension as partly marital and partly separate. The real estate was
inherited, and there is no evidence that its value has been enhanced by Husband’s efforts or marital funds.
The marital home was purchased after marriage with Wife’s employment income. The value of Wife’s
pension reflects both marital employment income and premarital employment income; a court would thus
apportion its value.

Thus, absent an enforceable premarital agreement, a divorce court could award Husband a share of Wife’s
pension and the marital home. In most states, it could not award Wife a share of Husband’s real estate.

Explanation to Point-Three (20%):

Even if the premarital agreement is unenforceable, it is unclear whether Husband can obtain alimony.
Husband’s failure to contribute to the marriage weighs against his alimony claim, but, if Husband can
establish that his injury precludes employment, that factor coupled with his age and the duration of the
marriage may support an alimony award.

State A terms spousal support “alimony.”

Rules governing the award of alimony vary from one state to the next, but they almost invariably require
the trial court to consider the parties’ financial resources and needs, their marital contributions, and the
marital duration. Some state statutes also require consideration of spousal misconduct. Although alimony
statutes typically accord substantial discretion to the trial court, they are invariably gender neutral.

In evaluating need, a court typically considers the standard of living enjoyed by the parties during the
marriage and whether the recipient spouse will be able to achieve that level of economic self-sufficiency
within a reasonable period of time following the divorce; health, the existence of separate assets, and the
share of marital assets that the recipient spouse will receive may also be taken into account.

In evaluating a spouse’s contributions to the marriage, a court may consider negative as well as positive
actions, including a spouse’s failure to make economic or noneconomic contributions and misuse or
dissipation of marital funds.

In this case, both Husband and Wife are minimally educated. The 15-year marriage was relatively
lengthy. The marital standard of living has not been high, and throughout the marriage, Wife has made
greatly disproportionate contributions. For most of the marriage, Husband’s failure to contribute more
was unexcused. Although Husband suffered a major injury five years ago, a court may be skeptical of his

Seperac-J19 Exam-Released MEE Essay Compilation © 2016-2020 901


claim that he cannot work given Husband’s employment history, his apparent lack of noneconomic
contributions, and his failure to make use of the pilot’s license he acquired with Wife’s help. Wife does
not earn a high salary that would permit her to pay alimony easily. Finally, because the question assumes
that the agreement is invalid, Husband would, in most states, retain his inherited real estate, which would
provide him with an asset that he could use to defray his expenses.

On the other hand, if Husband succeeds in establishing his disability, a court might find that his current
need outweighs his lack of economic contributions, particularly given the length of the marriage,
Husband’s age, Wife’s earning ability, and the fact that Wife will receive a share of marital assets.

Thus, on these facts, it is possible, but by no means certain, that Husband will obtain an alimony award if
the premarital agreement is found to be invalid.

Seperac-J19 Exam-Released MEE Essay Compilation © 2016-2020 902


#205-FEB 2010–MEE Q03: QUESTION THREE (FAMILY LAW)

Harry and Wendy married when they were both 23, shortly after Harry began his first year of law school.
They decided that Wendy, who had completed two years of college, would temporarily quit college and
get a full-time job until Harry started practicing law.

Wendy obtained a full-time job as a supermarket cashier. All of her earnings were used for Harry’s and
Wendy’s support. Harry did not work during the school year but worked full-time during summers. All of
Harry’s earnings were used to pay his law school tuition.

Harry recently earned his law degree, passed the bar examination, and began working as a public
defender.

Last month, after three years of marriage, Harry told Wendy that he felt they had drifted apart and that he
wanted a divorce. Although deeply hurt, Wendy decided to separate amicably. Harry and Wendy had no
children.

Harry suggested that he draw up a settlement agreement in order to avoid the expense of hiring a lawyer.
Harry proposed that each spouse keep his or her own personal property and that he bear full responsibility
for his educational loans. Harry also told Wendy that a divorce court would “definitely not impose any
further obligations” on him because there were no children of the marriage. Wendy agreed to the terms of
the settlement agreement proposed and prepared by Harry, and both parties signed it. In the agreement,
each spouse waived “any and all rights to the past or future income of the other party.”

Thereafter, following Harry’s instructions, Wendy filed a petition for a no-fault divorce, requesting that
the court incorporate their settlement agreement into the final divorce decree.

Prior to any hearing on the petition, Wendy learned that Harry had been involved in an adulterous
relationship with another woman. Angry and mistrustful, Wendy contacted an attorney, who urged her to
amend her divorce petition. The attorney suggested that Wendy petition the court to invalidate the
settlement agreement, seek a divorce on grounds of adultery, and request alimony and a cash award
representing her share of the value of Harry’s law license, which he acquired during the marriage.

State law does not authorize reimbursement alimony.

1. Should the court invalidate the settlement agreement? Explain.

2. What, if any, are the advantages to Wendy of obtaining a divorce based on Harry’s adultery
instead of on no-fault grounds? Explain.

3(a). Assuming the court invalidates the settlement agreement, can Wendy obtain a cash award
representing her share of the value of Harry’s law license? Explain.

3(b). Assuming the court invalidates the settlement agreement, can Wendy obtain alimony?
Explain.

Seperac-J19 Exam-Released MEE Essay Compilation © 2016-2020 903


#205: F10-3 MEE: ANSWER: NCBE (FAMILY LAW)

POINT (1) [30%] ISSUE: Did Harry’s behavior constitute fraud or overreaching sufficient to
invalidate the settlement agreement? ANSWER: No. The settlement agreement will be invalidated
if the court finds that (1) it was procured by fraud, duress, or overreaching, and (2) it is
substantively unfair. A court will likely find that Harry’s conduct in drafting and negotiating the
agreement satisfies the first part of this test. It is unclear whether the court will conclude that the
agreement is substantively unfair to Wendy.

POINT (2) [20%] ISSUE: Would a divorce obtained on grounds of adultery significantly enhance
Wendy’s prospects of obtaining a monetary award? ANSWER: No. Obtaining a divorce on grounds
of adultery is unlikely to significantly improve Wendy’s prospects of obtaining alimony or a cash
award representing her share of marital property.

POINT (3)(a) [10%] ISSUE: Is Harry’s law degree marital property subject to division at divorce?
ANSWER: Harry’s law degree is not marital property subject to division at divorce, and the facts
do not show that there are any other significant assets.

POINT (3)(b) [40%] ISSUE: Can Wendy obtain alimony (spousal support)? ANSWER: It is not
clear whether Wendy will receive alimony.

ANSWER DISCUSSION:

Courts generally will invalidate a settlement agreement resulting from fraud, overreaching, or duress if it
results in a settlement that is substantively unfair. Harry’s preparation of the agreement, his failure to
advise Wendy to seek independent counsel, and his biased account of Wendy’s legal rights will likely
lead the court to conclude that the agreement should be invalidated if the court also concludes that Wendy
should have obtained an award. A divorce obtained on grounds of adultery will not likely give Wendy a
significant advantage in obtaining an award. In only about half of the states is fault a factor in alimony
determination or property division, and even in those jurisdictions in which fault is relevant, it rarely
plays a major role in judicial decision making. Wendy is not entitled to a cash award based on the value of
Harry’s law license, and the facts do not reveal any other assets that might provide the basis for such an
award. Given that state law does not authorize claims for reimbursement alimony, it is unclear whether
Wendy can obtain an alimony award.

ANSWER EXPLANATION:

Explanation to Point-One (30%):

The settlement agreement will be invalidated if the court finds that (1) it was procured by fraud, duress, or
overreaching, and (2) it is substantively unfair. A court will likely find that Harry’s conduct in drafting
and negotiating the agreement satisfies the first part of this test. It is unclear whether the court will
conclude that the agreement is substantively unfair to Wendy.

In most states, a settlement agreement resulting from fraud, overreaching, or duress may be set aside if it
is substantively unfair. Under the UMDA, a written settlement agreement is binding on the court unless
unconscionable. A few courts have also held that spouses have fiduciary obligations toward each other

Seperac-J19 Exam-Released MEE Essay Compilation © 2016-2020 904


that continue, when one spouse is unrepresented by counsel, during the negotiation of a settlement
agreement.

Here, Wendy was unrepresented and Harry, an attorney, failed to advise her to seek independent legal
counsel. Although a lack of independent legal counsel will not by itself invalidate a settlement agreement,
courts will generally examine the circumstances that led a spouse to forgo counsel or waive marital rights.
Harry also advised Wendy that a court would “definitely not” impose any post-marital obligations on him,
which is probably untrue. Taken together, these facts suggest fraud and overreaching.

However, if Harry’s conduct did not result in an agreement substantively unfair to Wendy, the agreement
should not be invalidated. The agreement is not unfair as to Harry’s law degree. It might be unfair as to
alimony.

[NOTE: Some applicants may state that the court can ignore the settlement agreement because the
divorce is not yet final. This is incorrect, and an applicant who resolves the issue in this way should not
receive credit.]

Explanation to Point-Two (20%):

Obtaining a divorce on grounds of adultery is unlikely to significantly improve Wendy’s prospects of


obtaining alimony or a cash award representing her share of marital property.

Even if Wendy is able to obtain a divorce on grounds of adultery, it is unlikely to give her a financial
advantage. About a third of the states have eliminated fault-based divorce grounds. In these states, Wendy
could not obtain a divorce on grounds of adultery; she could obtain only a no-fault divorce.

Although most states permit consideration of financial misconduct, only about half permit consideration
of marital misconduct, such as adultery, in either property division or alimony determination. Of the
group of states that permit consideration of marital misconduct, about half restrict its use to alimony
determination. And while a claimant’s marital fault was once, in many states, a bar to an alimony award,
in virtually all states that today permit consideration of fault in alimony decision making, it is simply one
factor among many. Some appellate courts in states that permit consideration of marital misconduct have
required trial courts to consider fault only when egregious. For example, in New York, fault is excluded
except in “egregious cases that shock the conscience”. In Virginia, a party’s fault is relevant to the award
determination when the fault had an economic impact on the marriage. Some states have explicitly ruled
that adultery is not sufficiently egregious to have an impact on alimony. In other states, there is older case
law permitting consideration of adultery or similar misconduct, but virtually no modern cases do so.
Commentators reviewing the case law have thus concluded that “the current judicial trend in many states
today is to ignore or severely limit the ultimate effect of fault-based statutory divorce factors except in
serious or egregious circumstances.”

Thus, in most states, proving Harry’s adultery is unlikely to significantly enhance Wendy’s prospects of
obtaining either a property-distribution or an alimony award. In some states, however, this factor might
play a role in the court’s decision.

Explanation to Point-Three(a) (10%):

Harry’s law degree is not marital property subject to division at divorce, and the facts do not show that
there are any other significant assets.

Seperac-J19 Exam-Released MEE Essay Compilation © 2016-2020 905


The states, by an overwhelming margin, have rejected the claim that a professional degree or license is
property subject to division at divorce. Wendy thus has no property interest in Harry’s degree, and a court
could not grant her a cash award in compensation for such an interest. The facts also fail to reveal any
other assets that could be classified as marital property. Thus, a cash award to Wendy representing a share
of marital property would be inappropriate.

Explanation to Point-Three(b) (40%):

It is not clear whether Wendy will receive alimony.

If the court invalidates the settlement agreement, Wendy could seek an alimony award. But permanent or
long-term alimony is seldom awarded except in the case of a long marriage and a significant, long-term
gap between the husband’s and wife’s economic prospects. It is virtually certain that Wendy could not
obtain such an award, given the short duration of the marriage and the possibility that Wendy could
pursue a career that would generate an income equal to or higher than Harry’s since Wendy is still young
and without children.

In a state, like this one, that does not permit reimbursement alimony, Wendy could seek a short-term,
“rehabilitative” alimony award to complete her college education. In some states a spouse must establish
that she lacks the capacity for self-support as a precondition to obtaining a rehabilitative award. Under the
UMDA, a court may not award alimony unless the spouse: (1) lacks sufficient property to provide for his
or her reasonable needs; and (2) is unable to support themselves through appropriate employment.”).
Wendy has no property, but she is capable of self-support. Under a standard like that of the UMDA, she
would be ineligible for alimony unless her employment was not “appropriate.” Some courts have thus
refused to award alimony to a wife capable of self-support even when she supported her husband’s
professional education. For example, in one case, the court held that a schoolteacher wife who supported
her husband through graduate school was not eligible for alimony because she could support herself.
Another court overturned an alimony award to a wife who had supported her husband’s legal education
and planned to finish college and go to medical school because she was capable of self-support.

Most of these decisions are old; courts today typically view capacity for self-support as only one factor
among many in awarding rehabilitative alimony. Here, Wendy’s contributions to the couple’s support and
to Harry’s education and Harry’s ability to contribute to Wendy’s future support would weigh in her
favor. But the process of alimony determination is highly discretionary, and several factors – Wendy’s
youth, the short duration of the marriage, the fact that the marriage produced no children, Harry’s
probably modest salary as a public defender – would all weigh against Wendy’s claim for rehabilitative
alimony.

In sum, Wendy’s prospects of obtaining an alimony award are quite uncertain.

[NOTE: Spousal support and spousal maintenance are alternative terms for alimony. An applicant should
not be penalized for using one of these terms instead of alimony. The applicant’s conclusion should be
given less weight than his or her command of the relevant legal principles and use of the facts. A number
of jurisdictions now authorize “reimbursement” or “restitutional” alimony to compensate one spouse for
significant contributions to the other’s education or career. Given Wendy’s support of Harry during law
school, Wendy would have a strong claim to this form of alimony, but it is not authorized in the state
where her claim was brought.]

Seperac-J19 Exam-Released MEE Essay Compilation © 2016-2020 906


#206-JUL 2009–MEE Q03: QUESTION THREE (FAMILY LAW)

Two years ago, Husband and Wife divorced in State A. The divorce decree specified that

(a) Husband and Wife shall have joint custody of their 10-year-old child, Son, who shall reside with Wife
Sunday through Wednesday and with Husband Thursday through Saturday of each week; and

(b) Husband shall pay Wife $1,000 per month in child support until Son turns 18.

Three months ago, Wife accepted a job in State B, about 600 miles from State A. Wife’s new job raises
her net income by $1,000 per month and offers excellent opportunities for promotion. Wife’s parents also
live near Wife’s new job in State B, and Wife believes that Son will benefit from spending more time
with his grandparents.

Two months ago, Wife moved to State B with Son. Given the distance between Wife’s new home in State
B and Husband’s home in State A, Wife can no longer transport Son to Husband’s home in compliance
with the State A custody decree. Wife therefore telephoned Husband and said, “Son can visit you for six
weeks during the summer,” to make up the time that Husband will lose with Son each week.

Husband has never visited State B and owns no property there; he opposed Wife’s move to State B.
Husband also told Wife that, given her higher earnings from her new job, he would immediately stop
paying child support. Husband has not paid child support since making this statement.

Ten days ago, Wife filed a petition in a State B court seeking modification of the State A custody decree
so as to give her sole custody of Son and to substitute a six-week summer visitation period for Husband’s
weekly time with Son. Wife also registered and sought to enforce the State A child support order. Wife
personally served this petition on Husband in State A.

Five days ago, Husband filed a petition in State A court seeking modification of the custody decree so as
to give him sole custody of Son and to substitute an eight-week summer visitation period for Wife’s
weekly time with Son. Husband also requested elimination of his support obligation retroactive to the date
of Wife’s move. Husband personally served this petition on Wife in State B.

1. Should the State B court enforce the State A child support order? Explain.

2. Should the State B court modify the State A custody order by awarding sole custody of Son to
Wife and substituting a six-week summer visit for Husband’s weekly time with Son? Explain.

3. Should the State A court modify the State A custody order by awarding sole custody of Son to
Husband and substituting an eight-week summer visit for Wife’s weekly time with Son? Explain.

4. Should the State A court modify the State A child support order either retroactively or
prospectively? Explain.

Seperac-J19 Exam-Released MEE Essay Compilation © 2016-2020 907


#206: J09-3 MEE: ANSWER: NCBE (FAMILY LAW)

POINT (1) [17%] ISSUE: May a court in a non-issuing state enforce a child support order against
an obligor over whom it does not have personal jurisdiction? ANSWER: Yes. State B should
enforce the State A child support order because a registered child support order is enforceable in a
non-issuing state even if the non-issuing state does not have personal jurisdiction over the
respondent.

POINT (2) [28%] ISSUE: May a court in a non-issuing state modify a custody decree when one
party continues to reside in the issuing state? ANSWER: No. A State B court may not modify the
State A custody order because, under the PKPA, only courts of the issuing state may modify a
custody order so long as the child or any contestant continues to reside in the issuing state and the
issuing state does not decline to exercise jurisdiction.

POINT (3) [33%] ISSUE: When should a court modify a custody decree based on the relocation of
a parent in a situation where the parents share joint physical custody? ANSWER: Custody
modification is based on a substantial change in circumstances. One parent’s relocation that
significantly impairs the other parent’s ability to exercise his or her custodial rights represents a
substantial change in circumstances. It is unclear whether the court will grant Husband’s
modification petition.

POINT (4) [22%] ISSUE: When should a court grant a support obligor downward modification of
a child support order? ANSWER: Under UIFSA, State A has exclusive jurisdiction to modify the
State A child support order. Although federal law forbids retroactive modification of the support
order, a State A court may modify the order prospectively if it finds a substantial change in
circumstances. It is possible that Husband could obtain elimination of his child support obligation if
his petition for custody modification is granted.

ANSWER DISCUSSION:

Interstate enforcement and modification of child support is governed by the Uniform Interstate Family
Support Act (UIFSA), which has been adopted by all states. Here, UIFSA would permit State B, the non-
issuing state, to enforce a registered support order against Husband through a “two-state” procedure. The
interstate modification of the child custody decree is governed by the federal Parental Kidnapping
Prevention Act (PKPA) and the Uniform Child Custody Jurisdiction and Enforcement Act (UCCJEA).
Neither the PKPA nor the UCCJEA give the State B court authority to modify the State A custody decree.
Custody modification is based on a substantial change in circumstances. Wife’s relocation to State B has
significantly impaired Husband’s ability to exercise his custodial rights and represents a substantial
change in circumstances. On these facts, it is unclear whether Husband will obtain modification of the
custody decree in State A. Under UIFSA, State A has exclusive jurisdiction to modify the State A child
support order prospectively if it finds a substantial change in circumstances. If Husband obtains
modification of the custody decree, he can probably succeed in eliminating his child support obligation. If
he does not obtain modification, he will not be able to eliminate his child support obligation, although it is
possible that he will be able to obtain prospective downward modification.

ANSWER EXPLANATION:

Explanation to Point-One (10-20%):

Seperac-J19 Exam-Released MEE Essay Compilation © 2016-2020 908


State B should enforce the State A child support order because a registered child support order is
enforceable in a non-issuing state even if the non-issuing state does not have personal jurisdiction over the
respondent.

The interstate enforcement and modification of child support is governed by the Uniform Interstate
Family Support Act (UIFSA), which has been adopted by all states. Under UIFSA § 603, a registered
child support order issued in another state is “enforceable in the same manner and is subject to the same
procedures as an order issued by a tribunal of the adopting state.”

State B does not have personal jurisdiction over Husband because he has no State B contacts. However,
UIFSA provides a two-state procedure that avoids the need to obtain personal jurisdiction over Husband.
Using this approach, Wife can obtain an enforcement order in State A (the issuing state) by filing an
enforcement petition in a court of State B (the initiating state) that will be forwarded to the relevant State
A court. Thus, a State B court may enforce the State A child support order.

Because Husband has not paid child support required under the State A order, support enforcement is
appropriate, and State B should enforce the State A child support order.

Explanation to Point-Two (20-30%):

A State B court may not modify the State A custody order because, under the PKPA, only courts of the
issuing state may modify a custody order so long as the child or any contestant continues to reside in the
issuing state and the issuing state does not decline to exercise jurisdiction.

The federal Parental Kidnapping Prevention Act (PKPA) provides that a state may not modify a custody
decree issued by another state if either the child or any party continues to reside in the issuing state and
the issuing state’s courts do not decline to exercise jurisdiction. Under the Supremacy Clause, the PKPA
takes precedence over any conflicting state law.

Virtually all states have now enacted the Uniform Child Custody Jurisdiction and Enforcement Act
(UCCJEA), which contains custody-modification standards virtually identical to those of the PKPA.
Under the UCCJEA, a state that properly issued a custody decree (here, State A) retains continuing,
exclusive jurisdiction until all parties and the child have left the state, or until an issuing-state court has
determined that there is no longer any significant connection between the child and the person remaining
in the state and that substantial evidence is no longer available in that state.

It is highly unlikely that a State A court would decline to exercise jurisdiction under UCCJEA § 202
because Husband remains in State A, Wife moved only two months ago, and Son and Husband had
ongoing weekly contact until that time. Son thus maintains a significant connection with Husband and
substantial evidence is still available in State A. Thus, if Wife wishes to modify the custody arrangement,
she must do so in State A; a State B court may not modify the custody decree.

Explanation to Point-Three (25-35%):

Custody modification is based on a substantial change in circumstances. One parent’s relocation that
significantly impairs the other parent’s ability to exercise his or her custodial rights represents a
substantial change in circumstances. It is unclear whether the court will grant Husband’s modification
petition.

Seperac-J19 Exam-Released MEE Essay Compilation © 2016-2020 909


Because of the need to protect the best interests of a child, custody orders are subject to modification
throughout the child’s minority. But because stability and finality are also important values in child
custody litigation, most states permit modification only when there has been a substantial change in
circumstances since the original custody decree. One parent’s physical relocation that significantly
impairs the other parent’s opportunity to exercise custody and visitation rights provided in the divorce
decree is almost invariably considered a substantial change in circumstances.

Parental relocation is a common triggering factor in custody-modification cases, and “this area of law has
been unusually unstable. However, the clear trend has been that of increasing leniency toward the
relocating parent with whom the child has been primarily living.” One group of states places the burden of
proof on the relocating custodial parent and requires that parent to show that the move serves the child’s
best interests. In some of these states, the relocating parent must additionally show that the move is for a
legitimate purpose and reasonable in light of that purpose. Other states place the burden of proof on the
objecting parent to show that the move does not serve the child’s best interests; some additionally require
the objecting parent to show that the move would be harmful to the child. For example, in Kentucky, the
best interests test is applicable when a modification petition is brought more than two years after initial
decree or when an objecting parent merely seeks to preserve the initial visitation schedule, but the harm
standard is applicable when a custody modification petition is brought within two years of the initial
decree. In Tennessee, the objecting parent must show that the move is not for a reasonable purpose, that
relocation would pose a threat of specific and serious harm to the child that outweighs the threat of harm
to the child of a change of custody, or that parent’s motive for moving is vindictive.

Some states, in joint-custody cases, apply standards that are more protective of the parent who is not
relocating. In Oklahoma, the best interests test is applicable to modifications of joint-custody orders. In
Tennessee, the best interests test is applicable to modifications of joint-custody orders. Even if State A
does not have a special rule for joint-custody cases, relocation, of necessity, has a much larger impact
when, as here, the parent who is not relocating has had more than 40 percent of total residential time with
the child.

In this case, a number of facts support Husband’s petition. Under the custody decree, Husband not only
had joint custody, but Son resided with Husband more than 40 percent of each week. Wife’s move 600
miles away precludes weekly visits by Husband, let alone substantial residential time with Son. Wife’s
move was not necessary; she could have stayed at her former employment. Ten-year-old Son undoubtedly
had established school and social relationships that were disrupted by the move. Perhaps most important,
Wife moved without notifying Husband or seeking court approval; Wife should have sought modification
of the custody decree so as to accommodate her proposed move before physically removing Son from
State A. Her failure to do so will certainly place her at a disadvantage in contesting Husband’s
modification petition.

On the other hand, some facts support Wife’s relocation. Wife’s relocation will provide Son with a higher
family income and better access to his maternal relatives. There are no facts to suggest that Wife’s move
was motivated by vindictiveness.

It is thus possible, but not certain, that Husband will obtain modification of the custody decree.

[NOTE: An applicant’s conclusion about case outcome is less important than his or her analysis of the
relevant issues.]

Explanation to Point-Four (15-25%):

Seperac-J19 Exam-Released MEE Essay Compilation © 2016-2020 910


Under UIFSA, State A has exclusive jurisdiction to modify the State A child support order. Although
federal law forbids retroactive modification of the support order, a State A court may modify the order
prospectively if it finds a substantial change in circumstances. It is possible that Husband could obtain
elimination of his child support obligation if his petition for custody modification is granted.

Under UIFSA, an issuing state has continuing, exclusive jurisdiction to modify a support order if the child
or any party continues to reside in that state and all parties do not consent to the jurisdiction of another
forum. Under UIFSA § 603, a tribunal of the State shall recognize and enforce, but may not modify, a
registered order if the issuing tribunal had jurisdiction. Because Husband continues to reside in State A,
State A courts have exclusive jurisdiction to modify the State A child support order.

A State A court may not retroactively modify the child support order because federal law forbids
retroactive modification in all circumstances.

If Husband succeeds in obtaining modification of the custody decree, a State A court might grant
Husband prospective elimination of his child support obligation. In all states, modification of a support
order is based on a finding that there has been a substantial change in circumstances that significantly
reduces the child’s need for support or the obligor’s capacity to pay. If Husband succeeds in obtaining
modification of the custody decree, Wife would have residential time with Son for only eight weeks per
year. This would certainly represent a substantial change in circumstances, as Wife would no longer be a
custodial parent. Moreover, Wife’s net monthly income has increased by $1,000 per month. On these
facts, a court could prospectively eliminate Husband’s child-support obligation.

If Husband does not obtain modification of the custody decree, the court should not eliminate Husband’s
child support obligation. There is a strong public policy favoring the establishment and maintenance of
legal support obligations, and even poor obligors whose income falls below a value that the state deems
essential for self-support are typically required to make token child support payments. A court might
reduce Husband’s support obligation based on Wife’s increased income if that income is available to
support Son. However, Wife’s move to State B may have increased her expenses and, assuming that
Husband does not obtain sole custody of Son, Son’s increased residential time with Wife would almost
certainly increase her child-related expenses. It is thus unclear whether a State A court would find that
Wife’s increased income warrants a prospective reduction in Husband’s support obligation.

Seperac-J19 Exam-Released MEE Essay Compilation © 2016-2020 911


#207-FEB 2009–MEE Q08: QUESTION EIGHT (FAMILY LAW)

Fourteen years ago, Mom and Dad had a brief romance while Dad was on vacation in State A, where
Mom lived. Nine months after Dad returned to his home in State B, Mom telephoned Dad. Mom told Dad
that she had just given birth to Child and that Dad was Child’s father. Mom also told Dad that, if he would
agree to waive his right to establish his paternity of Child, she would sign a release waiving all rights to
seek child support from him. Dad agreed to Mom’s proposal, and they signed a written contract
containing the terms outlined by Mom.

Mom is a college graduate and had an excellent job when Child was born. However, she has developed a
chronic disease and is no longer able to fully provide for herself and Child, now age 14.

Mom brought an action against Dad in State A seeking to establish his paternity of Child and obtain child
support, claiming that the contract waiving her right to child support is unenforceable. She served Dad by
registered mail in State B, where he has continued to live since Child’s birth. Dad has not visited State A
since his vacation there more than 14 years ago and has never met Child.

State A’s long-arm statute provides that a court has personal jurisdiction over an alleged parent for
purposes of determining paternity and support obligations if the alleged parent “engaged in sexual
intercourse in this state and the child may have been conceived by that act of intercourse.”

Dad moved to dismiss Mom’s petition on the ground that the State A court’s assertion of personal
jurisdiction over him to determine paternity and child support would violate due process requirements.
The trial court denied the motion, and Dad entered a special appearance, preserving his right to appeal on
the jurisdictional claim.

On the merits, Dad argues that the contract he and Mom made shortly after Child’s birth should be
enforced. In the event that the court declines to enforce the contract, Dad argues that the equities of the
case require that the value of any child support awarded to Mom be calculated based on state public-
assistance benefit levels or, in the alternative, that he be awarded custody of Child. Dad also seeks liberal
visitation with Child in the event that the court awards custody to Mom.

1. Was the court’s assertion of personal jurisdiction over Dad to determine Dad’s paternity and
support obligations consistent with due process requirements? Explain.

2. Assuming the court has personal jurisdiction over Dad, how should the court rule on the child
support, custody, and visitation issues? Explain.

Seperac-J19 Exam-Released MEE Essay Compilation © 2016-2020 912


#207: F09-8 MEE: ANSWER: NCBE (FAMILY LAW)

POINT (1) [30%] ISSUE: Did State A’s assertion of personal jurisdiction over Dad to determine
paternity and child support based on his sexual activity in State A more than 14 years earlier meet
due process requirements? ANSWER: Yes. State A’s assertion of personal jurisdiction over Dad
probably met due process requirements because the nature of the claim – paternity and child
support – is closely related to Dad’s State A contacts, and Dad has a continuing obligation to
support Child, a State A domiciliary.

POINT (2) [20%] ISSUE: Will a court enforce an agreement between parents that relieves a parent
of support duties in exchange for that parent relinquishing parental rights? ANSWER: No. The
agreement between Mom and Dad eliminating Dad’s child-support duty is inconsistent with the
best interests of the child and is unenforceable.

POINT (3) [25%] ISSUE: Will a court base the support obligation of an unmarried father who has
had no relationship with his child on public-assistance benefit levels? ANSWER: No. The State A
court will determine the value of Dad’s child-support obligation to Child on the same basis that it
would use to determine support for a child born to married parents. Because the court must
consider, at a minimum, all of Dad’s income and earnings, it will not base the value of a child-
support award on state public-assistance benefit levels.

POINT (4) [25%] ISSUE: Will a court award custody or visitation of a 14-year-old child to a parent
whom the child has never met and who lives in a different state from the child? ANSWER: The
State A court will determine custody and visitation based on Child’s best interests. It will almost
certainly award primary custody to Mom because Dad has had no prior relationship with Child,
but it will likely award Dad visitation.

ANSWER DISCUSSION:

Because Dad’s State A contacts that provide a statutory basis for long-arm jurisdiction are closely related
to the nature of the claim against Dad and Dad has a continuing support obligation, State A’s assertion of
personal jurisdiction over Dad probably met the requirements of the Due Process Clause. The contract
between Mom and Dad waiving Dad’s child-support obligation is unenforceable. Dad’s child-support
obligation will be based on his earnings and income, not state public-assistance benefit levels. Dad is
highly unlikely to succeed in obtaining custody of Child, but may obtain visitation.

ANSWER EXPLANATION:

Explanation to Point-One (25-35%):

State A’s assertion of personal jurisdiction over Dad probably met due process requirements because the
nature of the claim – paternity and child support – is closely related to Dad’s State A contacts, and Dad
has a continuing obligation to support Child, a State A domiciliary.

The long-arm statute quoted in the fact pattern is § 201(6) of the Uniform Interstate Family Support Act
(UIFSA), which has been adopted by all states. The Supreme Court of the United States has not ruled on
the constitutionality of this provision.

Seperac-J19 Exam-Released MEE Essay Compilation © 2016-2020 913


The leading case on the due process requirements governing jurisdiction over a nonresident parent in a
child-support action is Kulko v. Superior Court. In Kulko, the Supreme Court employed the “minimum
contacts” test laid out in International Shoe Co. v. Washington, and reversed a state-court ruling extending
long-arm jurisdiction over a nonresident parent based on his “consent to the permanent residence of the
child” in the forum state. The Kulko court noted that due process demands “some act by which the
defendant purposefully avails himself of the privilege of conducting activities within the forum state,” ,
before the forum state may legitimately exercise long-arm jurisdiction. The Court also held that a father
“who agrees, in the interests of family harmony and his children’s preferences, to allow them to spend
more time in the forum state than was required under a separation agreement can hardly be said to have
‘purposefully availed himself’ of the ‘benefits and protection’ of the forum state’s laws.”

The long-arm statute at issue in this case does rely on purposeful activity, i.e., sexual intercourse.
Although the Supreme Court has not ruled on the constitutionality of such a provision, state courts have
found that statutes like this one meet due process requirements. These courts have relied on the voluntary
nature of the conduct, the fact that the child’s birth could easily be anticipated, and the state’s strong
interest in ensuring that the child is supported.

Dad might argue that the state’s assertion of jurisdiction in this case would violate due process standards
given that more than 14 years have passed since the State A acts that serve as a jurisdictional basis. In
making this argument, Dad might rely on Kulko, in which the Supreme Court stated that the forum state
could not have relied on the defendant’s marriage during a three-day visit to the forum state 13 years
before the support action was brought as a basis for jurisdiction. But, in contrast to the marriage in Kulko,
the sexual activity that serves as the jurisdictional basis in this case is directly related to the basis for
asserting jurisdiction and Dad continues to have a support obligation to Child. It is thus likely that a court
would find that the assertion of jurisdiction is constitutional. (“No time frame is stated for filing a
proceeding; this is based on the fact that the absent parent has a support obligation that extends for at least
the minority of the child (and often longer in many states).”). There are also state-court decisions
exercising long-arm jurisdiction on similar facts.

Explanation to Point-Two (15-25%):

The agreement between Mom and Dad eliminating Dad’s child-support duty is inconsistent with the best
interests of the child and is unenforceable.

Although the agreement between Mom and Dad appears to satisfy other basic contract law requirements,
it is unenforceable. Courts have long held that parents may not enter into enforceable agreements that
adversely affect the rights of their children; courts always retain authority to enter support and custody
orders that are in the children’s best interests. The agreement at issue in this case would eliminate the
relationship between Dad and Child for the convenience of the parents; all courts would find that this
conflicts with the child’s interests in having two parents who can provide support and care.

Explanation to Point-Three (20-30%):

The State A court will determine the value of Dad’s child-support obligation to Child on the same basis
that it would use to determine support for a child born to married parents. Because the court must
consider, at a minimum, all of Dad’s income and earnings, it will not base the value of a child-support
award on state public-assistance benefit levels.

Seperac-J19 Exam-Released MEE Essay Compilation © 2016-2020 914


Federal legislation enacted during the 1980s requires all states, as a condition of federal funding, to
employ numerical child-support guidelines and establish a rebuttable presumption that the award which
would result from application of the guidelines is correct, the guidelines must be applied in all cases,
regardless of the parents’ marital status.

Although state guidelines vary considerably in the way support is calculated, none sets presumptive
awards at a public-assistance level. Federal regulations require state support guidelines to “take into
consideration all earnings and income of the noncustodial parent, be based on specific descriptive and
numeric criteria and result in a computation of the support obligation.”

Thus the State A court should determine the value of Dad’s support obligation based on his income, not
the value of state public-assistance benefits.

[NOTE: Support laws that disregard marital status may be constitutionally mandated. In Gomez v. Perez,
the Supreme Court held that a state law which based the existence of a support obligation on the parents’
marital status violated the Equal Protection Clause. Gomez has often been interpreted as forbidding
marital-status variation in the value of support, although there are some state court opinions that have
taken a contrary view.]

Explanation to Point-Four (20-30%):

The State A court will determine custody and visitation based on Child’s best interests. It will almost
certainly award primary custody to Mom because Dad has had no prior relationship with Child, but it will
likely award Dad visitation.

Custody and visitation disputes are governed by the court’s determination of the child’s best interests.
Residential and relational continuity are both widely recognized as important sources of child well-being.
It is thus highly unlikely that a court would award custody to Dad, who lives at some distance from
Child’s current home and with whom Child has had no relationship whatsoever.

However, meaningful contact with both parents also is recognized as an important source of child well-
being. It is thus extremely rare for a court to deny an adjudicated parent visitation, even when the parent
and child have had no significant relationship. The presumption in favor of visitation and the lack of any
evidence in the fact pattern that visitation would be harmful to Child suggest that Dad’s petition for
visitation would be granted.

The court might nonetheless deny Dad’s visitation petition if it finds that visitation is not in Child’s best
interests. This outcome is more likely if the court finds that the petition was brought in bad faith as a
means of deterring Mom from proceeding with the paternity/support action and if Child strongly objects
to visitation; because Child is 14, most states would require the court to consider Child’s wishes in
fashioning a visitation order.

Seperac-J19 Exam-Released MEE Essay Compilation © 2016-2020 915


#208-JUL 2008–MEE Q07: QUESTION SEVEN (FAMILY LAW)

Six years ago, Hal and Wendy were married in State A. Both of them had been previously divorced. Hal,
age 40, was a successful businessman earning $200,000 per year. Wendy, age 30, was a struggling
songwriter earning $20,000 per year.

Two weeks before their wedding, Hal told Wendy that his lawyer (Lawyer) had advised him not to marry
Wendy unless she signed a premarital agreement. Hal gave Wendy a copy of the agreement Lawyer had
proposed and suggested that she review it with Lawyer or another attorney of her own choosing. The
agreement specified that in the event of divorce:

1. Each spouse waives all claims to property acquired by the other during the first five years of the
marriage;

2. The spouses will share joint physical and legal custody of any children born to them during the
marriage.

When Hal gave Wendy the proposed agreement, she burst into tears. Wendy was very angry and hurt, but
she did not want to call off the wedding at such a late date. Reluctantly, she agreed to discuss the matter
with Lawyer.

Lawyer gave Wendy an accurate list of Hal’s assets and a copy of Hal’s tax returns for the past three
years. Lawyer urged her to consult another attorney. After conferring with her family, but not an attorney,
Wendy decided to sign the proposed agreement. The day before their wedding, she and Hal signed the
agreement in Lawyer’s office in State A.

Four years ago, Hal and Wendy had a child (Child).

Six months ago, Hal and Wendy moved to State B. Hal’s business has continued to prosper. He currently
earns $300,000 per year. Since the marriage, Hal has used his business income to acquire assets worth
about $500,000. Wendy has continued to write songs. Her current income is $30,000 per year.

Three months ago, after Wendy discovered that Hal was having an affair, she took Child and moved back
to State A, where she plans to remain. Since leaving Hal, Wendy has written and recorded several songs
about her marriage. Wendy’s agent believes that these songs “will hit the top of the charts.”

State A has adopted the Uniform Premarital Agreement Act. State B has not. The premarital agreement
contains no choice-of-law provision.

1. Which state’s law governs the enforceability of the premarital agreement? Explain.

2. Is the waiver-of-property-rights provision in the premarital agreement enforceable? Explain.

3. Is the child-custody provision in the premarital agreement enforceable? Explain.

4. Are the profits to be derived from Wendy’s songs written after she left Hal subject to division at
divorce? Explain.

Seperac-J19 Exam-Released MEE Essay Compilation © 2016-2020 916


#208: J08-7 MEE: ANSWER: NCBE (FAMILY LAW)

POINT (1) [22%] ISSUE: Which state’s law governs enforceability of the premarital agreement?
ANSWER: In determining the enforceability of a premarital agreement, states apply either the law
of the state in which the contract was executed (here State A) or the state with the most significant
relationship to the parties and the transaction. Under either test the court will likely apply the law
of State A.

POINT (2) [43%] ISSUE: Is the waiver-of-property-rights provision in the premarital agreement
enforceable? ANSWER: Yes. In determining whether a premarital agreement governing property
distribution at divorce is enforceable, courts consider whether the agreement was voluntarily made,
whether it is substantively fair, and whether full disclosure of assets and obligations was made. The
weight assigned to these factors varies from state to state, but under the UPAA, an agreement
entered voluntarily and with reasonable disclosure of assets and obligations is enforceable.

POINT (3) [8%] ISSUE: Is the child-custody provision in the premarital agreement enforceable?
ANSWER: No. In all states, a premarital agreement regarding child support or custody is
unenforceable if it is not in the best interest of the child.

POINT (4) [27%] ISSUE: Are profits from the songs Wendy wrote after separation but before the
divorce filing marital property subject to division at divorce? ANSWER: Yes. Profits from a song
written after separation but before a final divorce decree is entered are likely to be classified as
marital property subject to division at divorce.

ANSWER DISCUSSION:

In determining the enforceability of a premarital agreement, some states apply the law of the state in
which the contract was signed and others apply the law of the state with which the parties have the most
significant relationship. Under either test, the law of State A will probably govern here. State A has
adopted the Uniform Premarital Agreement Act (UPAA), under which an agreement will be enforced
unless (1) the party against whom enforcement is sought did not execute the agreement voluntarily, or (2)
that party did not receive or waive fair and reasonable disclosure of the assets and obligations of the other
party and can show that the agreement was unconscionable when executed. Thus the property-distribution
provision of the agreement would be enforceable under State A law. In all states, a premarital agreement
respecting child support or custody is voidable based on the child’s best interest. In most states, marital
property continues to accrue at least until a divorce petition has been filed. Profits from property, such as
a song, created by a spouse during the marriage are subject to distribution at divorce.

ANSWER EXPLANATION:

Explanation to Point-One (15-25%):

In determining the enforceability of a premarital agreement, states apply either the law of the state in
which the contract was executed (here State A) or the state with the most significant relationship to the
parties and the transaction. Under either test the court will likely apply the law of State A.

Seperac-J19 Exam-Released MEE Essay Compilation © 2016-2020 917


Some states apply the law of the state in which a premarital contract was executed in determining its
enforceability. Under this approach, the court should apply the law of State A.

Others, probably more numerous, apply the law of the state with the most significant relationship to the
matter at hand. Hal and Wendy entered into a premarital agreement in State A, married there, had a child
there, and lived there for all but a few months of their six-year marriage. Wendy and Child are in State A
and Wendy plans to remain there. Wendy has a strong argument that State A’s law should govern because
of its interest in assuring that contracts executed and marriages consummated within its borders comply
with its policies. On the other hand, Hal is in State B, State B was the marital home when Hal and Wendy
separated, and some marital assets may be located there. Thus, State B may also have an interest in
applying its law.

On these facts, it is likely, but not certain, that State A’s interests are stronger, and State A’s law will
likely apply under either test.

Explanation to Point-Two (35-45%):

In determining whether a premarital agreement governing property distribution at divorce is enforceable,


courts consider whether the agreement was voluntarily made, whether it is substantively fair, and whether
full disclosure of assets and obligations was made. The weight assigned to these factors varies from state
to state, but under the UPAA, an agreement entered voluntarily and with reasonable disclosure of assets
and obligations is enforceable.

Although courts were once hostile to premarital agreements, today all states permit spouses to contract
premaritally with respect to rights and obligations in property. In all states, the enforceability of a
premarital agreement turns on three factors: voluntariness, fairness, and disclosure. How courts apply
these factors varies significantly from one state to the next.

In many states, an agreement is unenforceable if the party against whom enforcement is sought succeeds
in showing any one of involuntariness, unfairness, or lack of adequate disclosure. However, under the
UPAA, which has been adopted in 25 states and State A, the party against whom enforcement is sought
must prove (1) involuntariness or (2) that “the agreement was unconscionable when it was executed” and
that he or she did not receive or waive “fair and reasonable” disclosure and “did not have, or reasonably
could not have had, an adequate knowledge” of the other’s assets and obligations. Thus, under the UPAA,
a court may not refuse to enforce a premarital agreement based on substantive unfairness unless it also
finds inadequate disclosure or lack of knowledge.

Wendy is highly unlikely to succeed in establishing inadequate disclosure or lack of knowledge. Lawyer
gave Wendy copies of Hal’s tax returns for the past three years and an accurate list of his assets. Under
the UPAA, Wendy would thus be required to show that her execution of the agreement was involuntary in
order to avoid its enforcement.

In considering whether a premarital agreement was voluntarily executed, courts look to whether there was
fraud, duress, or coercion. They agree that one party’s insistence on signing the agreement as a condition
of the marriage does not, by itself, render the agreement involuntary, but there is no consensus on what
additional facts are sufficient to establish involuntariness.

Many of the reported cases, like this one, involve a claim of involuntariness based on presentation of an
agreement very close to the wedding date. In analyzing whether an agreement signed under these
circumstances is voluntary, courts have looked at a wide range of factors, including the difficulty of

Seperac-J19 Exam-Released MEE Essay Compilation © 2016-2020 918


conferring with independent counsel, other reasons for proceeding with the marriage (for example, a
preexisting pregnancy), and financial losses and embarrassment arising from cancellation of the wedding.
In this case, Hal presented the proposed agreement a full two weeks before the wedding. Wendy was
mature, had been previously married, and made her decision to sign after conferring with her family.
There are no facts indicating lack of opportunity to confer with counsel or any particular hardship that
would result from canceling the wedding. On these facts, it is unlikely, although not impossible, that
Wendy can establish involuntariness. Under the UPAA, the agreement is likely enforceable.

Explanation to Point-Three: (05-10%):

In all states, a premarital agreement regarding child support or custody is unenforceable if it is not in the
best interest of the child.

The traditional rule is that a premarital agreement cannot bind a court deciding child support or custody.
Although the UPAA does not explicitly bar an agreement respecting child custody, “long tradition would
seem to ensure that courts would not consider themselves bound by custody provisions they believe
injurious to the child’s interest. The law of separation agreements in every state is explicit on that point,
and there is no reason why premarital agreements would be treated differently.”

Explanation to Point-Four (20-30%):

Profits from a song written after separation but before a final divorce decree is entered are likely to be
classified as marital property subject to division at divorce.

In the majority of states, marital property continues to accrue until a final divorce decree is entered. In
some states, however, marital property ceases to accrue after the date of permanent separation or the date
of filing for a divorce. Wendy’s songs were written after the five-year property-distribution waiver period
provided for in the premarital agreement. Thus, because no divorce petition has been filed, Wendy’s
songs are probably marital property.

The fact that Wendy has not yet received profits from the songs does not change this result. Today, even
contingent expectancies such as nonvested pension rights are subject to division at divorce if they were
acquired through spousal effort during the marriage. Thus profits from Wendy’s songs, even if payment
has not yet been made, would be divisible at divorce if the songs were created during the marriage. Of
course, a court might find that the songs were worthless or that their value was too speculative for
distribution.

Seperac-J19 Exam-Released MEE Essay Compilation © 2016-2020 919


#209-FEB 2008–MEE Q03: QUESTION THREE (FAMILY LAW)

Husband and Wife married 10 years ago. Shortly thereafter, Husband adopted Wife’s two children, Amy,
age 6, and Bert, age 9. Neither Amy nor Bert has ever had a relationship with their biological father.

One year ago, Husband and Wife were divorced. The divorce decree provided that:

(1) Husband shall pay Wife $1,000 per month in child support for Amy until Amy is 18 years old;

(2) Husband shall pay child support in the form of college tuition up to $20,000 per year for both Amy
and Bert.

Three months ago, Husband stopped making support payments for Amy and college tuition payments for
Bert. Husband stopped paying for two reasons:

First, Husband was disinclined to continue supporting Amy and Bert, now ages 16 and 19, respectively.
Since the divorce, Husband has quarreled frequently with both children. Bert also disobeyed Husband and
joined a rock band that plays at a local bar four nights per week. Since joining the band, Bert’s college
grades slipped from A’s to C’s, and he was arrested for driving while intoxicated. Bert has refused
Husband’s requests that he leave the band and devote more time to study.

Second, Husband, who formerly worked 40 to 60 hours per week, is now working only 10 to 20 hours per
week so that he can finish writing a novel. Husband has worked on the novel sporadically over the past
few years, but has not had time to complete it. Husband’s current income is only 25% of what it was
when he was employed full time.

Wife, who works full time at the job she has held since her marriage to Husband, has neither reduced nor
increased her income since the divorce. Since Husband stopped paying support, she has been borrowing
money to meet the family’s expenses.

Wife recently filed a petition to obtain a judgment against Husband for the child-support arrears and
Bert’s tuition. Husband responded with a petition seeking:

(1) a dissolution of his adoption of Amy and Bert on the basis of irreconcilable differences;

(2) downward modification of all of his support obligations, on account of his reduced income, retroactive
to the date on which Husband stopped making support and tuition payments; and

(3) a declaration that Husband need not pay Bert’s college tuition so long as Bert continues to perform in
a rock band.

The age of majority in the state is 18. The trial court entered judgment in favor of Wife and denied
Husband’s petition in all respects.

Did the trial court err? Explain.

Seperac-J19 Exam-Released MEE Essay Compilation © 2016-2020 920


#209: F08-3 MEE: ANSWER: NCBE (FAMILY LAW)

POINT (1) [29%] ISSUE: May an adoptive parent dissolve the adoption of a stepchild based on a
quarrelsome relationship with the child? ANSWER: No. The court did not err in refusing to
dissolve Husband’s adoption of Ann and Bert.

POINT (2) [15%] ISSUE: May a support obligor obtain retroactive modification of a support
obligation? ANSWER: No. The court did not err in refusing to retroactively modify Husband’s
support obligation.

POINT (3) [35%] ISSUE: May a support obligor obtain downward modification of his future
support obligation based on a voluntary reduction of his income? ANSWER: No. The trial court
probably did not err in refusing to reduce Husband’s future support obligation to Ann. A support
obligor’s voluntary income reduction cannot be used to obtain downward modification of his future
support obligation unless the reduction was made in good faith. Even a good-faith income reduction
may be inadequate to obtain downward modification if the court finds that the children would
suffer hardship without an offsetting benefit.

POINT (4) [21%] ISSUE: May a divorced parent of a 19-year-old college student refuse to support
that student when the student has refused to obey reasonable parental commands? ANSWER:
Because the support rights of an employable child are contingent on compliance with reasonable
parental demands, the trial court probably erred in denying Husband’s request for future
modification of his support obligation to Bert.

ANSWER DISCUSSION:

A quarrelsome relationship with an adoptee is not a valid basis for dissolving an adoption. Husband is
thus obligated to support Ann and Bert. Moreover, because retroactive modification of a child support
obligation is forbidden by federal law, Husband may not obtain termination or reduction of his support
obligation for the period before he sought modification. Modification of a future support obligation may
be obtained based on a substantial change in circumstances, but a voluntary reduction in income does not
warrant downward modification unless it was made in good faith. Many courts also take the needs and
interests of the supported child into account. Thus the court probably did not err in denying Husband’s
petition to modify his obligation to support Ann. But the court probably did err in denying Husband’s
petition to modify his obligation to Bert, as Bert, an employable college student, had refused to obey
reasonable parental commands.

ANSWER EXPLANATION:

Explanation to Point-One (20-30%):

The court did not err in refusing to dissolve Husband’s adoption of Ann and Bert.

An adoption order works as a complete – and final – transfer of parental rights and responsibilities.
Traditionally, an adoption order could not be dissolved for any reason. In recent years, some states have
enacted statutes authorizing dissolution of adoption in special circumstances, such as the discovery of an
undisclosed mental or physical illness. Stepparent adoptions are no easier to undo than stranger adoptions.

Seperac-J19 Exam-Released MEE Essay Compilation © 2016-2020 921


In evaluating dissolution claims, courts typically look to the length of the relationship, the child’s needs,
and the parent’s motives. Here, Husband has been Ann’s and Bert’s parent for more than a decade and the
only father they have known. Ann needs Husband’s continuing support. The fact that Husband has
reduced his income strongly suggests that he seeks dissolution, at least in part, for financial reasons.
Moreover, the only reason that Husband can offer for dissolution, the fact that he had disagreements with
teenagers Ann and Bert, is clearly inadequate. The trial court thus did not err in denying Husband’s
petition to dissolve the adoption.

Explanation to Point-Two (10-15%):

The court did not err in refusing to retroactively modify Husband’s support obligation.

Federal law absolutely forbids retroactive modification of child support obligations. The trial court thus
did not err in refusing to retroactively modify Husband’s child support obligation.

Additionally, state courts have long held that the obligation to pay accrued and unpaid installments of
child support may not ordinarily be modified. “If the hardship is particularly severe, the courts have
sometimes devised a way to protect the obligor, but in most instances the courts hold that retroactive
modification of this kind is beyond their power and indeed the governing statute may so provide.” Here,
because Wife commenced an enforcement action within a few months of when Husband ceased making
payments and thereby limited the accrual of arrears, Husband has no basis for establishing a particularly
severe hardship.

Explanation to Point-Three (25-35%):

The trial court probably did not err in refusing to reduce Husband’s future support obligation to Ann. A
support obligor’s voluntary income reduction cannot be used to obtain downward modification of his
future support obligation unless the reduction was made in good faith. Even a good-faith income
reduction may be inadequate to obtain downward modification if the court finds that the children would
suffer hardship without an offsetting benefit.

In order to obtain modification of a future support obligation, the petitioner must show a substantial
change in circumstances. Courts will generally find that a significant involuntary income reduction
represents a substantial change warranting relief. But when the income loss is voluntary, they invariably
require a showing that the reduction was made in good faith and not for the purpose of depriving the child
or punishing the custodial parent.

Some courts will grant downward modification based solely on a showing of good faith on the theory
that, if a married parent can reduce his income even if it negatively affects his children’s living standard, a
divorced parent should have the same right. Others will not grant downward modification if the support
reduction would impose a significant hardship on the children. Many take a multifactor approach that
balances the short- and long-term interests of both parent and child.

If the court were to consider hardship or engage in an interest-balancing analysis, the facts support denial
of Husband’s petition. A temporary income reduction for training that will produce a future income gain
is sometimes a valid basis for modification. However, Husband’s income reduction does not have any
obvious end point nor is it calculated to give the children future benefits. Even if Husband succeeds in
becoming a successful novelist, the children’s ages (16 and 19) would likely preclude a higher future
support award to offset the current income loss. Wife, who already works full time, cannot easily make up

Seperac-J19 Exam-Released MEE Essay Compilation © 2016-2020 922


the lost income nor can she continue to borrow indefinitely. But Husband could easily defer his plan to
write a novel or reduce his income much less drastically.

Even if the court looked only at whether the income reduction was made in good faith, there is evidence
to support denying Husband’s petition. Although Husband’s prior work on the novel provides evidence of
good faith, the timing of the decision to reduce his income – the very same time that he wanted to stop
supporting Ann and Bert – is suspect and suggests bad faith. Thus, the court probably did not err in
denying Husband’s petition to modify his support obligation to Ann.

Explanation to Point-Four (15-20%):

Because the support rights of an employable child are contingent on compliance with reasonable parental
demands, the trial court probably erred in denying Husband’s request for future modification of his
support obligation to Bert.

In a number of states, a divorced parent may be required to provide college or other educational support
for a child over the age of majority. But both married and divorced parents may terminate support to
employable children who disobey reasonable parental commands.

Husband’s commands to Bert seem reasonable and calculated to serve Bert’s best interests. A regular,
late-night job at a bar would either prevent Bert from getting enough sleep or prevent him from attending
morning classes; the fact that Bert’s grades have fallen supports Husband’s view that Bert cannot keep up
with his studying while performing with the band. Bert’s arrest for driving while intoxicated also suggests
that he should not be working in a bar. The evidence thus suggests that the trial court erred in denying
Husband’s request for modification of his college-tuition obligation to Bert.

Seperac-J19 Exam-Released MEE Essay Compilation © 2016-2020 923


#210-JUL 2007–MEE Q03: QUESTION THREE (FAMILY LAW)

Husband and Wife married eleven years ago when both were age 19 and college students. Husband
planned to go to medical school and Wife planned to become an accountant. They decided that Wife
would defer her educational plans in order to provide support while Husband completed his medical
studies. Accordingly, Wife dropped out of college, took a job as a filing clerk, and did all of the
household chores in order to allow Husband more time to study. This arrangement continued while
Husband completed his remaining three years of college, four years of medical school, and a three-year
medical residency. Wife expended all of her earnings to support Husband and herself. Husband made
minimal financial contributions to the marriage.

Throughout the marriage, Husband was verbally abusive to Wife and occasionally hit her. Husband was
always contrite after these incidents and attributed his behavior to stress resulting from his studies.

Three months before Husband completed his medical residency, he and Wife had an argument. During the
argument, Husband assaulted Wife and broke her arm. Wife left Husband and filed a petition for divorce.

Husband suggested to Wife that they meet with a divorce mediator, Mediator. Wife agreed to this
proposal. Husband and Wife both gave Mediator information about their assets and incomes. Husband
indicated that his post-residency salary would be $150,000 per year, listed no assets, and listed school
debts totaling $50,000. Wife indicated that her salary was $30,000 per year and listed no assets. Wife also
indicated that she intended to return to college in the fall and that her income would then decline.

Last month, Husband and Wife met with Mediator. Mediator did not explain anything about the mediation
process or divorce law, nor did she inform Husband and Wife that they could obtain independent legal
advice about any agreement reached through mediation. Instead, Mediator asked Husband for a settlement
proposal. Husband proposed that each spouse keep his or her personal property and that Husband bear
responsibility for his educational loans. Mediator responded, “That sounds like a fair settlement. I’ll
prepare the paperwork unless Wife objects.” Within the view of Mediator and Wife, Husband tightened
his fist and gave Wife a menacing look. Wife said, “I have no objection.”

After signing the agreement prepared by Mediator, Wife learned that Mediator was an attorney who had
represented Husband’s family for many years and that Husband had talked to Mediator about the
settlement shortly before their meeting.

No divorce judgment has yet been entered.

Wife has petitioned the trial court to set aside the settlement agreement she and Husband had signed and
to award her spousal maintenance of $25,000 per year for three years.

1. On what grounds, if any, could the court set aside the settlement agreement? Explain.

2. If the settlement agreement is set aside, is Wife entitled to spousal maintenance? Explain.

Seperac-J19 Exam-Released MEE Essay Compilation © 2016-2020 924


#210: J07-3 MEE: ANSWER: NCBE (FAMILY LAW)

POINT (1)(a) [24%] ISSUE: Does Husband’s conduct provide a basis for setting aside the divorce
settlement agreement? ANSWER: Yes. A divorce settlement agreement may be set aside when a
spouse’s fraud, duress, or coercive behavior results in an unfair agreement. Husband’s collusion
with Mediator, his implicit threats against Wife, and his failure to disclose his relationship with
Mediator provide a basis for setting aside the settlement agreement.

POINT (1)(b) [24%] ISSUE: Does Mediator’s conduct provide a basis for setting aside the divorce
settlement agreement? ANSWER: Yes. A divorce settlement agreement may be set aside based on
serious mediator misconduct. Because Mediator was guilty of such misconduct, the trial court
would not err in setting the agreement aside.

POINT (2) [53%] ISSUE: Is Wife entitled to spousal maintenance? ANSWER: Yes. Wife has a
strong claim to spousal maintenance given her disproportionate contributions to the marriage and
to Husband’s education, the substantial disparity in Husband’s and Wife’s economic circumstances,
and the duration of the marriage.

ANSWER DISCUSSION:

A spouse’s fraud, duress, or coercive behavior provides a basis for setting aside a settlement agreement
that is unfair. In this case, there is evidence to support findings of unfairness and impermissible
misconduct by Husband. Additionally, a mediator’s substantial misconduct is a basis for setting aside a
settlement agreement. Such substantial misconduct is clearly evident in this case. Wife has a strong
spousal maintenance claim given her disproportionate contributions to the marriage and to Husband’s
education, the substantial disparity in Husband’s and Wife’s economic circumstances, and the duration of
the marriage.

ANSWER EXPLANATION:

Explanation to Point-One(a) (15-25%):

A divorce settlement agreement may be set aside when a spouse’s fraud, duress, or coercive behavior
results in an unfair agreement. Husband’s collusion with Mediator, his implicit threats against Wife, and
his failure to disclose his relationship with Mediator provide a basis for setting aside the settlement
agreement.

When a divorce settlement agreement is attacked before a final judgment of divorce is entered, the court
generally “has great freedom to reject the agreement, to accept it, or to accept it with modifications.”
Although the case law is far from uniform and often relies heavily on the facts of the case at hand, courts
in most states have held that an agreement may be set aside based on fraud, duress, or coercive behavior
that results in an agreement substantively unfair to one of the parties. A written separation agreement is
binding on the court unless unconscionable.

Courts have been particularly solicitous of divorce litigants who are unrepresented by legal counsel. Even
when there is no general duty of disclosure, a number of courts have held that when negotiating a
separation agreement, a represented spouse has a fiduciary obligation toward an unrepresented spouse.

Seperac-J19 Exam-Released MEE Essay Compilation © 2016-2020 925


Although lack of independent legal counsel will not by itself invalidate an agreement, courts will
generally examine the circumstances that led to the waiver of counsel. Moreover, an attorney may not
ethically represent both spouses in drafting an agreement.

Here, Wife had no attorney before the mediation session, while Husband met with Mediator, an attorney
who had represented his family for many years. Husband also failed to tell Wife about that meeting and
his family’s relationship with Mediator. Taken together, these facts may establish that the agreement was
procured through fraud. Husband’s collusion with Mediator to pressure Wife into signing an agreement
favoring his interests also appears to be coercive; so does Husband’s fist-clenching and “menacing” look.

Husband’s behavior thus offered the trial court a basis for invalidating the settlement agreement if that
agreement was substantively unfair to Wife. Because, in most states, there are strong arguments in favor
of a spousal maintenance award to Wife, the trial court would be justified in finding that the agreement
was substantively unfair.

[NOTE: Cases refusing to invalidate fraudulently obtained settlement agreements that already have been
incorporated into a divorce judgment are inapplicable to this fact pattern.]

Explanation to Point-One(b) (15-25%):

A divorce settlement agreement may be set aside based on serious mediator misconduct. Because
Mediator was guilty of such misconduct, the trial court would not err in setting the agreement aside.

Substantial mediator misconduct is a basis for setting aside a divorce settlement agreement. In this case,
Mediator was guilty of serious misconduct. First, a mediator must be impartial and disclose potential
conflicts of interest. A mediator must “conduct the mediation process in an impartial manner” and
“disclose all actual and potential grounds of bias and conflicts of interest reasonably known to the
mediator. In this case, a number of facts suggest bias: Mediator was the attorney for Husband’s family
and met with Husband prior to the mediation session; Mediator failed to reveal these sources of bias and
showed partiality toward Husband by requesting only his proposal. Second, a mediator is required to
explain the mediation process and make sure that the parties have enough information to ensure informed
decision making. A mediator must facilitate the participants’ understanding of what mediation is. In this
case, Mediator did not explain the mediation process, the law, or the parties’ entitlement to independent
counsel. Third, a mediator should recognize and control a situation, such as domestic violence, that would
result in unfairness or prejudice to one of the parties. A mediator must recognize a family situation
involving domestic abuse and take appropriate steps to shape the mediation process accordingly. Here,
Mediator did not screen for domestic violence and did nothing to adjust the power imbalance between
Husband and Wife. Fourth, a mediator may not coerce or improperly influence any party to make a
decision. A mediator must recognize that mediation is based on the principle of self-determination by the
participants. Here, Mediator characterized Husband’s proposal as “fair” and failed to ask Wife what she
would propose as a settlement.

Mediator’s misconduct offered the trial court another, independent basis for setting aside the settlement
agreement.

Explanation to Point-Two (40-50%):

Wife has a strong claim to spousal maintenance given her disproportionate contributions to the marriage
and to Husband’s education, the substantial disparity in Husband’s and Wife’s economic circumstances,
and the duration of the marriage.

Seperac-J19 Exam-Released MEE Essay Compilation © 2016-2020 926


Rules governing the award of spousal maintenance vary from one state to the next, but they almost
invariably require the trial court to consider the parties’ financial resources and needs, the marital
contributions, and the marital duration. Some state statutes require consideration of spousal misconduct.
Some require the court to consider one spouse’s support for the other’s education or training or mandate
reimbursement for such contributions. Spousal maintenance statutes also accord substantial discretion to
the trial court.

In this case, Husband’s salary will soon be five times that of Wife’s, and his economic prospects are
extremely good. Wife is minimally educated and relatively unskilled; her economic prospects are
extremely poor. The marriage lasted eleven years. During that time, Wife made greatly disproportionate
economic and non-economic contributions to the marriage, Wife greatly facilitated Husband’s medical
training, and Husband was consistently abusive to Wife.

In the vast majority of states, this combination of facts would amply support a spousal maintenance award
to Wife during the three-year period needed to complete her college education. Given Husband’s
anticipated $150,000 annual salary, Wife’s earning capacity (apparently no greater than $30,000 per
year), and her likely inability to work full time while attending college, a $25,000 spousal maintenance
award also seems to be fully justified.

In some states, however, a spouse must establish that she lacks the capacity for self-support as a
precondition to obtaining a spousal maintenance award. Under the UMDA, a court may not award spousal
maintenance unless the spouse: (1) lacks sufficient property to provide for his reasonable needs; and (2) is
unable to support himself through appropriate employment. Wife has no property, but she is capable of
self-support. Under a standard like that of the UMDA, she would be ineligible for spousal maintenance
unless her employment was not “appropriate.” Some courts have thus refused to award spousal
maintenance to a wife capable of self-support even when she supported her husband’s professional
education.

For example, in McDermott v. McDermott, a schoolteacher wife who supported her husband through
graduate school was not eligible for spousal maintenance because she could support herself. Another
court overturned an alimony award to a wife who had supported her husband’s legal education and
planned to finish college and go to medical school because she was capable of self-support.

Decisions denying spousal maintenance on facts like those in the problem are typically quite old,
however. Today, courts and commentators tend to view spousal maintenance as compensation for loss
rather than relief of need. Courts treat spousal maintenance as a remedy for unfair loss allocation. Some
jurisdictions also employ a quasi-contract or unjust enrichment approach. The problem contains facts that
would support this type of argument given that Husband and Wife both expected to benefit from
Husband’s enhanced earning capacity.

Even in a jurisdiction with case law strongly emphasizing self-support, Wife could rely on the fact that
she and Husband had an understanding that she would return to school after his education was completed.
Spousal maintenance can be denied if a spouse's current educational plan was never in the contemplation
of the parties during marriage. That agreement would also provide Wife with an argument that her current
employment is inappropriate.

[NOTE: The applicant’s conclusion should be given less weight than his or her command of the relevant
legal principles and use of the facts.]

Seperac-J19 Exam-Released MEE Essay Compilation © 2016-2020 927


#211-FEB 2007–MEE Q04: QUESTION FOUR (FAMILY LAW)

Twelve years ago, Husband and Wife married in State A. Their marriage appeared to be happy and stable.
However, one year ago, without warning, Husband left Wife and moved to State B, 500 miles away.
Husband obtained a new job in State B and rented an apartment there. He has told Wife that he never
intends to return to State A.

Last week, Wife was personally served in State A with a copy of Husband’s State B divorce petition. The
petition requests the State B court to grant a divorce on grounds of a six-month separation and
irreconcilable differences. The petition also requests the State B court to award Husband the following
assets, all of which are titled solely in Husband’s name:

1. The marital home in State A, which Husband purchased five years before the marriage. During the
marriage, Husband made mortgage payments on the home with his employment income.

2. Stock, which Husband inherited from his grandmother during the marriage.

3. Bonds, which Husband purchased with his employment income during the marriage.

All of these assets appreciated significantly in value during the marriage. Wife has no assets titled in her
name alone or held jointly with Husband.

Wife is determined to fight this divorce. She has never visited State B, knows no one there, and does not
believe she should have to defend a divorce action there. At the same time, she wishes to save her
marriage. She has called Husband several times urging him to return to State A and enter marriage
counseling, but he has refused. Wife is convinced that Husband is going through a “mid-life crisis” and
would “return to his senses” with proper counseling and support. She believes she can get him that
support and counseling if he returns to State A. Finally, if there must be a divorce, Wife believes that she
should receive a share of the assets owned by Husband.

1. Does the State B court have jurisdiction to grant Husband a divorce and award Husband property
acquired during the marriage? Explain.

2. Can Wife prevent Husband from obtaining a divorce on the grounds that she does not consent to
the divorce and that the marriage might yet be saved? Explain.

3. Without regard to any jurisdictional issues, would Wife, in the event of a divorce, be entitled to a
share of any of Husband’s assets? Explain.

Seperac-J19 Exam-Released MEE Essay Compilation © 2016-2020 928


#211: F07-4 MEE: ANSWER: NCBE (FAMILY LAW)

POINT (1)(a) [20%] ISSUE: Does a court have jurisdiction to grant a divorce when one spouse has
no contacts with the forum state? ANSWER: Yes. A court that lacks jurisdiction over one spouse
may grant a divorce.

POINT (1)(b) [20%] ISSUE: Does a court have jurisdiction to divide marital property when one
spouse has no contacts with the forum state? ANSWER: No. A court that does not have jurisdiction
over both spouses may not issue a binding property division order.

POINT (2) [25%] ISSUE: May a spouse who did not consent to a separation and believes that
reconciliation is possible successfully oppose a divorce petition brought on grounds of separation
and irreconcilable differences? ANSWER: No. A court may order a divorce based on separation
and irreconcilable differences even if the separation was nonconsensual and one spouse believes
that reconciliation is possible.

POINT (3) [35%] ISSUE: When are assets owned by one spouse subject to division at divorce?
ANSWER: Assets acquired before marriage or by inheritance are separate, not marital, property.
In most states, neither separate property nor appreciation in its value is subject to division at
divorce. However, the use of marital funds to add value or obtain increased equity in separate
property creates marital property. Property acquired during the marriage with marital funds is
marital property.

ANSWER DISCUSSION:

[NOTE: Applicants answering this question in a community property state should use community
property principles. For this purpose, the phrase “marital property” means community property.]

A court that lacks personal jurisdiction over one spouse may grant a divorce, but may not issue a binding
property division or support order. On the facts of this case, Husband could secure a divorce without
Wife’s consent and despite Wife’s claim that the marriage could be saved if Husband entered counseling.
In all states, a court may divide marital property regardless of which spouse holds title. Property acquired
during the marriage with employment income is marital property subject to division; inherited property
and property acquired before marriage are separate property that is typically not subject to division. If the
value of a separate asset is increased through marital funds or significant spousal labor, it may be
apportioned into separate and marital components.

ANSWER EXPLANATION:

Explanation to Point-One(a) (15-25%):

A court that lacks jurisdiction over one spouse may grant a divorce.

Jurisdiction over both spouses is not necessary to dissolve a marriage. If the plaintiff spouse is a
domiciliary of the forum state, courts of that state have jurisdiction to dissolve the plaintiff’s marriage. In
Williams v. North Carolina (“Williams I”), the United States Supreme Court likened a divorce action to
an in rem proceeding in which the res is marital status. The court found that each spouse “carried” the
marital status to the forum state when he or she established a new domicile in that state; thus a state court

Seperac-J19 Exam-Released MEE Essay Compilation © 2016-2020 929


may grant its domiciliary’s divorce petition as long as he or she has satisfied the state’s jurisdictional
requirements.

Domicile – residence with the intent to remain indefinitely – is a question of fact. On these facts, Husband
is almost certainly a domiciliary of State B. Husband moved to State B one year ago; he rented an
apartment and obtained employment there; he has indicated that he never intends to return to State A.
State B could thus grant Husband a divorce.

Explanation to Point-One(b) (15-25%):

A court that does not have jurisdiction over both spouses may not issue a binding property division order.

In an ex parte divorce, the court’s jurisdiction extends only to the marriage itself. Unless the court also has
personal jurisdiction over the defendant spouse, it may not issue a binding order affecting personal rights
such as property division. The divorce decree is thus “divisible.”

Unless the defendant is personally served within the forum state, a court does not have personal
jurisdiction over a defendant who lacks “minimum contacts” with the state. Wife was served with
Husband’s divorce petition in State A. She has never been to State B; indeed, she knows no residents of
State B except Husband. A State B court thus may not issue a binding support or property division order.

[NOTE: An applicant could receive extra credit for noting that a State B court could issue a binding
property division order if Wife decided to participate in the State B divorce action. When both spouses
participate in the divorce proceeding, principles of preclusion apply; because the participating spouse
could have raised the jurisdictional issue in the forum state, he or she is bound by the forum state’s
determination on this issue.]

Explanation to Point-Two (20-30%):

A court may order a divorce based on separation and irreconcilable differences even if the separation was
nonconsensual and one spouse believes that reconciliation is possible.

In virtually all states, a spouse may obtain a divorce without a showing of fault or consent of the other
spouse. The typical basis for a no-fault divorce is irreconcilable differences, a minimum period of
separation, or both.

Today, a no-fault divorce may be granted without any attempt at reconciliation. Even if one spouse
establishes that marital discord arose from a curable condition (for example, addiction or profligate
spending), the fact that one spouse believes that the marriage can be saved is not an adequate basis to
deny the other spouse a divorce.

In states that require a minimum period of separation as the basis for a no-fault divorce, separation need
not be consensual. The mandated period of separation is usually one year or less.

Thus, any effort by Wife to prevent the divorce would be unsuccessful. The parties have been separated
for a year, Husband appears determined to secure the divorce, and Wife’s belief that the marriage can be
saved is legally irrelevant.

Explanation to Point-Three (30-40%):

Seperac-J19 Exam-Released MEE Essay Compilation © 2016-2020 930


Assets acquired before marriage or by inheritance are separate, not marital, property. In most states,
neither separate property nor appreciation in its value is subject to division at divorce. However, the use
of marital funds to add value or obtain increased equity in separate property creates marital property.
Property acquired during the marriage with marital funds is marital property.

In all states, marital property is divided at divorce without regard to how that property is legally titled.
Thus, the fact that the marital home, the stock, and the bonds are titled in Husband’s name alone is not
determinative of how the assets should be divided.

In a majority of states, “marital property” may be divided between the spouses, but “separate property”
remains the property of the owning spouse. In a minority of “hotchpot” jurisdictions, the court may divide
all assets, whenever or however acquired; a few states permit the division of separate property in special
circumstances, such as hardship.

An asset is marital property if it was acquired during the marriage by any means other than gift, descent,
or devise. An asset that is initially separate property may be transformed into marital property if marital
funds or significant effort by the owner-spouse during the marriage enhances its value or build equity.
The fact that a separate asset appreciates in value during the marriage does not in itself transform that
asset into marital property.

If State A is a majority jurisdiction, Husband’s stock would be classified as separate property, the bonds
as marital property, and the marital home as partly separate and partly marital. The stock was inherited,
and there is no evidence that its value has been enhanced by Husband’s efforts or with marital funds. The
bonds, on the other hand, were purchased with marital property (money Husband earned during the
marriage) and are therefore marital property. The marital home was purchased before the marriage, but
with a mortgage; employment income was used to reduce the mortgage indebtedness and obtain
additional equity in the house. In most states, such equity-building payments create marital property. In a
few states, installment payments create only a marital lien against the value of the separate asset.

Thus, Wife is entitled to have a court make an equitable division of the bonds and the post-marriage
increase in Husband’s home equity. In most states, however, Husband’s stock would be separate property,
not subject to division upon divorce.

Seperac-J19 Exam-Released MEE Essay Compilation © 2016-2020 931


#212-JUL 2006–MEE Q04: QUESTION FOUR (FAMILY LAW)

Ten years ago, Matt Smith, a musician, and Wendy Jones, a business executive, began living together in
State A. One year later, they invited fifty friends and relatives to a “ceremony of commitment” at which
they publicly vowed to “treat each other as an equal owner of all worldly goods acquired during our life
together” and to “forsake all others” until “death do us part.” Matt and Wendy did not obtain a marriage
license. After the ceremony, Matt and Wendy consistently referred to each other as “my companion.”
They also opened a joint bank account and rented a house as “Wendy Jones and Matt Smith.”

Five years ago, Matt and Wendy moved to State B, where Wendy gave birth to Child. Matt and Wendy
thereafter shared child-care responsibilities and spent relatively equal amounts of time with Child.

One year ago, Wendy left Matt and married Steve in State B. After the wedding, Wendy and Child moved
to Steve’s home, and Wendy began to work part time.

Since her marriage to Steve, Wendy has allowed Matt to visit Child infrequently and only at her home.
Matt has tried to give Wendy money for Child’s support, but Wendy has refused to take it.

Matt has filed a petition in State B seeking to establish his paternity and obtain joint custody of Child. In a
separate State B action, Matt seeks a share of Wendy’s property acquired during the time they lived
together. Wendy and Steve have filed an adoption petition seeking Child’s adoption by Steve. Matt
opposes this petition. All of these actions have been consolidated for trial.

Pretrial discovery has established that Matt is Child’s biological parent, that Wendy has assets worth
$300,000, all obtained from income earned while she lived with Matt, and that Matt has assets worth
$1,500. Matt and Wendy each currently earn about $30,000 per year.

A common law marriage can be contracted in State A but not in State B.

1. May the State B court award Matt a share of Wendy’s property? Explain.

2. May the State B court grant Steve’s adoption petition over Matt’s opposition? Explain.

Seperac-J19 Exam-Released MEE Essay Compilation © 2016-2020 932


#212: J06-4 MEE: ANSWER: NCBE (FAMILY LAW)

POINT (1)(a) [10%] ISSUE: Must a state that does not permit common law marriage recognize a
common law marriage that was valid in the state where contracted? ANSWER: Yes. A common
law marriage that is valid where contracted is also valid in a state that does not permit common law
marriage.

POINT (1)(b) [30%] ISSUE: Did Wendy and Matt enter into a valid common law marriage?
ANSWER: No. The court may not find that Wendy and Matt entered into a common law marriage
because they did not hold themselves out as a married couple.

POINT (2) [20%] ISSUE: Did Wendy and Matt enter into an enforceable agreement to share
property? ANSWER: Yes. The court may find that Matt’s and Wendy’s mutual vows to share “our
worldly goods” constituted an enforceable agreement to share property acquired during their
relationship if state law permits a cohabitant to recover based on a verbal cohabitation agreement.

POINT (3) [40%] ISSUE: Should the court grant Steve’s adoption petition over Matt’s objection?
ANSWER: No. The court may not grant Steve’s adoption petition over Matt’s opposition because
Matt has been an involved father who lived with his child for a substantial portion of the child’s life
and who wishes to maintain an active, custodial relationship with the child.

ANSWER DISCUSSION:

If Matt and Wendy entered into a valid common law marriage in State A, it would be recognized in State
B even though State B law does not permit common law marriage. However, Matt and Wendy did not
enter into a valid common law marriage in State A. Although the “ceremony of commitment” might
arguably represent an agreement to be married, Matt and Wendy did not publicly hold themselves out as a
married couple. Thus, Matt may not claim a spousal entitlement to a share of Wendy’s property acquired
during their relationship. However, if State B recognizes oral cohabitation agreements, the court could
find that Matt’s and Wendy’s agreement to “treat each other as an equal owner of all worldly goods
acquired during our life together” was enforceable and award Matt a share of Wendy’s property acquired
during the period they cohabited. Many states recognize such oral agreements. The court may not grant
Steve’s adoption petition over Matt’s opposition. Although the U.S. Supreme Court has upheld
restrictions on the right of an unmarried father to veto his child’s adoption, it has also required the states
to give fathers who have been committed parents the opportunity to veto a stepparent adoption.

ANSWER EXPLANATION:

Explanation to Point-One(a): (05-15%):

A common law marriage that is valid where contracted is also valid in a state that does not permit
common law marriage.

Under universally accepted conflict-of-laws principles, a marriage valid under the law of the place in
which it was contracted will be valid elsewhere unless it violates a strong public policy of the state that
has the most significant relationship to the spouses and the marriage. Perhaps because the vast majority of
states once permitted common law marriage, courts in states that do not recognize common law marriage
have consistently held that, if a man and woman are domiciled in a state that permits common law

Seperac-J19 Exam-Released MEE Essay Compilation © 2016-2020 933


marriage and their conduct meets the requirements of that state’s law for establishing such a marriage,
recognition of the marriage does not violate a strong public policy. Thus, if Matt and Wendy entered into
a valid common law marriage in State A, it would be recognized by a State B court.

Explanation to Point-One(b) (25-35%):

The court may not find that Wendy and Matt entered into a common law marriage because they did not
hold themselves out as a married couple.

A common law marriage creates marital obligations and rights identical to those flowing from a
ceremonial marriage, including the equitable distribution of marital property. To establish a common law
marriage, the proponent must show: (1) capacity to enter a marital contract, (2) a present agreement to be
married, (3) cohabitation, and (4) “holding out” a marital relationship to the community. The evidence
here clearly establishes capacity and cohabitation. Arguably, the commitment ceremony shows an
agreement to be married as it involved an exchange of “vows” much like those made in a traditional
marriage ceremony. (A court might also find that there was no agreement to marry because the ceremony
did not include any references to marriage or use the terms “husband” and “wife.”) But Matt and Wendy
did not hold themselves out to the public as a married couple; they consistently referred to each other as
“companion” rather than husband and wife, and Wendy retained her own name.

Explanation to Point-Two (15-25%):

The court may find that Matt’s and Wendy’s mutual vows to share “our worldly goods” constituted an
enforceable agreement to share property acquired during their relationship if state law permits a
cohabitant to recover based on a verbal cohabitation agreement.

Agreements between cohabitants creating property or support rights were once unenforceable because
they were made in the context of a “meretricious relationship.” But since the California Supreme Court’s
landmark decision in Marvin v. Marvin, many states have abandoned this approach and declared
cohabitation agreements enforceable as long as they do not involve the explicit exchange of money for
sex. Again following Marvin – and in contrast to the approach applied to premarital agreements – most
courts in states that permit enforcement of cohabitation agreements do not require such agreements to be
in writing or even to be clear and specific; oral agreements that vaguely require the cohabitants to share or
“do right by” each other have been held to create enforceable claims to post-relationship property division
and/or support. Many states, yet again following Marvin, will even imply an agreement from the parties’
behavior.

Because Matt and Wendy vowed at their “ceremony of commitment” to “treat each other as an equal
owner” of goods acquired during the relationship, Matt would not face the evidentiary problems that
many cohabitants confront; the “agreement” was made before fifty guests, all of whom are now potential
witnesses. The fact that Matt and Wendy thereafter had a child together, jointly participated in the child’s
care, pooled their earnings in a joint bank account, and remained together in a stable relationship for
nearly a decade provides further behavioral evidence of an agreement to share property.

If applicants find that a valid common law marriage existed, then their discussion should address
equitable principles of property division and/or community property principles if applicable in their
jurisdictions.

[NOTE: Applicants who recognize the alternate theories of constructive trust, quantum meruit,
partnership, or partition may receive additional credit. For example, one court disallowed a cohabitant’s

Seperac-J19 Exam-Released MEE Essay Compilation © 2016-2020 934


quantum meruit claim for improvements to house because of the benefit he received from living in the
house during the relationship.]

Explanation to Point-Three (35-45%):

The court may not grant Steve’s adoption petition over Matt’s opposition because Matt has been an
involved father who lived with his child for a substantial portion of the child’s life and who wishes to
maintain an active, custodial relationship with the child.

Although the U.S. Supreme Court has upheld state statutes that grant the unmarried father only a limited
opportunity to object to his child’s adoption , it has also stated that, “when an unwed father demonstrates
a full commitment to the responsibilities of parenthood by ‘coming forward to participate in the rearing of
his child,’ his interest in personal contact with his child acquires substantial protection under the Due
Process Clause.” Applying this standard, the Supreme Court upheld a statute authorizing a court to
dispense with the requirement of the unmarried father’s consent to the adoption of his child, at least when
the father had “never shouldered any significant responsibility with respect to the daily supervision,
education, protection, or care of the child.” By contrast, the Court struck down a statute dispensing with
the requirement of the unmarried father’s consent as applied to a man who had maintained joint custody
of his children from the time of their birth until they were two and four years old. Both Quilloin and
Caban involved stepparent adoptions like that sought by Steve and Wendy.

The evidence unequivocally shows that Matt has demonstrated “a full commitment to the responsibilities
of parenthood.” Not only did he live with Child for most of Child’s life, but the evidence suggests that he
was an equal participant in Child’s care. Moreover, since Wendy’s marriage, Matt has consistently visited
Child despite Wendy’s rigid visitation restrictions. Matt has volunteered to pay child support, and he took
the initiative to establish his paternity and obtain joint custody.

Because Matt is a committed father entitled to protection under the due process clause, his parental rights
cannot be severed without his consent or a showing of parental unfitness. As there is no evidence of
abuse, neglect, or other grounds for termination, the adoption petition must be denied.

[NOTE: Examinees could receive extra credit for noting that the Supreme Court’s decision in Michael H.
v. Gerald D., upholding a statute that denied an unmarried father the opportunity to establish his
paternity over the combined opposition of the mother and her husband, does not alter this result. The
statute at issue in Michael H. applied only when the mother was married to the husband at the time she
gave birth; the Court relied heavily on the state’s interest in preventing disruption of intact marital
families in its analysis. Those applicants who find that a common law marriage existed could discuss the
presumption of paternity because Child was born in wedlock. However, because the common law
marriage was not registered, they should still go on to analyze Matt’s rights under the unmarried father
principles as discussed above in concluding that Steve’s adoption petition must be denied.]

Seperac-J19 Exam-Released MEE Essay Compilation © 2016-2020 935


#213-FEB 2006–MEE Q04: QUESTION FOUR (FAMILY LAW)

Herb petitioned for divorce from Ann after a 20-year marriage. One year later, the court entered a decree
of divorce based on irreconcilable differences. The court also awarded Ann a share of the marital property
and $1,000 per month in spousal maintenance until her death or remarriage.

After Herb’s petition was filed but before the court issued a final decree, Herb married Betty. Betty
believed that Herb’s divorce was final. After the court issued a final decree, Ann married Charles. Herb
then stopped paying spousal maintenance to Ann.

After her marriage to Charles, Ann learned that Charles had misrepresented the value of his stock. Charles
said that he owned stock worth “millions.” In fact, Charles’s stock was worth only $300,000. Ann has
filed for annulment of her marriage to Charles. In a separate action, she seeks reinstatement of the spousal
maintenance she was awarded in her divorce from Herb.

Betty has petitioned for divorce from Herb. Herb has filed a motion to dismiss the divorce petition on the
ground that he and Betty were not lawfully married.

1. If Ann is granted an annulment of her marriage to Charles, can she obtain reinstatement of the
spousal maintenance she was awarded in her divorce from Herb? Explain.

2. Can Ann obtain an annulment of her marriage to Charles based on Charles’s misrepresentation
of his assets? Explain.

3. Can Herb obtain dismissal of Betty’s divorce petition based on the claim that he and Betty were
not lawfully married? Explain.

Seperac-J19 Exam-Released MEE Essay Compilation © 2016-2020 936


#213: F06-4 MEE: ANSWER: NCBE (FAMILY LAW)

POINT (1) [37%] ISSUE: Will a husband’s alimony obligation that terminated upon his former
wife’s remarriage be reinstated if the former wife’s new marriage is annulled? ANSWER: No. The
court is not likely to reinstate a spousal maintenance award that terminated upon remarriage even
if the remarriage is annulled.

POINT (2) [26%] ISSUE: Can an annulment be obtained based on a spouse’s misrepresentation of
that spouse’s assets? ANSWER: No. Fraud is a basis for annulment only if it goes to the “essentials”
of the marriage, and asset value is not generally considered an essential.

POINT (3) [37%] ISSUE: Is a marriage void if contracted after the filing of a divorce petition but
before the final decree is entered? ANSWER: No. For purposes of ascertaining the validity of a
subsequent marriage, curative doctrines are likely to prevail over the argument that Herb’s
marriage to Betty is void for bigamy because it was entered prior to a final decree of divorce.

ANSWER DISCUSSION:

The court is unlikely to reinstate a spousal maintenance award based on the annulment of the recipient
spouse’s remarriage. Although an annulment legally “erases” a marriage, the weight of modern authority
does not permit reinstatement of alimony based on annulment of the marriage that terminated the alimony
obligation. The court is also unlikely to grant an annulment based on misrepresentation of the value of a
stock portfolio. Fraud is a basis for annulment only if it goes to the “essentials” of the marriage, and
courts typically have not found asset misrepresentation to meet this test. Finally, the court is likely to
declare Herb’s bigamous marriage valid based on one or another curative doctrine and thus deny Herb’s
motion to dismiss.

ANSWER EXPLANATION:

Explanation to Point-One (30-40%):

The court is not likely to reinstate a spousal maintenance award that terminated upon remarriage even if
the remarriage is annulled.

When a marriage has been annulled, it is typically declared invalid from the date of its inception. By
contrast, divorce terminates a marriage upon the date of the divorce. Because an annulment legally
“erases” a marriage, most courts have held that alimony may not be granted in annulment cases without
statutory authority; it seems illogical to say that a marriage never existed and then impose a duty that
arises only from a valid marriage. The same logic would support reinstatement of a prior spousal
maintenance award. If the award terminates on remarriage and no remarriage occurred, then it would
seem that the alimony entitlement still exists.

Some of the older cases apply the “logic of annulment” doctrine and order reinstatement of prior alimony
awards. But, recognizing the payor’s interest in certainty as to the termination of his obligations, the vast
majority of the modern cases deny petitions to revive alimony entitlements that were terminated by an
annulled marriage. Modern statutory standards typically follow this “no-revival” approach. The Uniform
Marriage & Divorce Act provides that the court granting an annulment may, in the interests of justice,
make the decree prospective only and thus equivalent to a divorce decree.

Seperac-J19 Exam-Released MEE Essay Compilation © 2016-2020 937


Some modern courts have reinstated alimony awards when the equities strongly favor the former alimony
recipient. For example, in In re Marriage of Weintraub, the wife alleged that her second marriage took
place only because she was abducted and physically coerced into marriage; the court held that proof of an
involuntary marriage was enough to support reinstatement. Here, there are no facts that justify
reinstatement: Ann chose to marry Charles; although Charles misrepresented his assets, he could have
suffered financial reverses even had he told the truth. Ann is neither disabled nor infirm. Moreover, Ann
could have divorced Charles and thus preserved her entitlement to obtain alimony from him. In many
jurisdictions, statutory law today permits the spouse who obtains an annulment to also seek alimony.

Explanation to Point-Two (20-30%):

Fraud is a basis for annulment only if it goes to the “essentials” of the marriage, and asset value is not
generally considered an essential.

In most states, a marriage is voidable and may be annulled based on fraud. Even in states without explicit
statutory authority, courts have often found that their inherent equity power permits the grant of a fraud-
based annulment. Although some case law permits annulment based on a “material” misrepresentation,
most courts have “required fraud going to the ‘essentials of the marriage.’”

The line between fraud going to the “essentials” and lesser misrepresentation is not always easy to
discern. The cases are fact-based, and some jurisdictions have interpreted the materiality requirements
more liberally than others. But even in “liberal” annulment states, “false representations regarding one’s
character, social standing, or fortune do not constitute fraud sufficient to annul a marriage. While such
factors constitute a species of fraud, they do not afford a basis for destruction of the marriage.”

Annulment is particularly unlikely here as Charles seems to have misrepresented only the value of his
stock. The case is simply one in which a spouse thought her intended was wealthier than he actually was.
Such a situation could arise as a result of market forces as well as misrepresentation. Therefore, Ann is
highly unlikely to obtain an annulment of her marriage to Charles based on his misrepresentation of his
stock portfolio’s value.

Explanation to Point-Three (30-40%):

For purposes of ascertaining the validity of a subsequent marriage, curative doctrines are likely to prevail
over the argument that Herb’s marriage to Betty is void for bigamy because it was entered prior to a final
decree of divorce.

If Herb’s divorce from Ann was not final at the time he married Betty, his marriage to Betty was
bigamous. A bigamous marriage is void ab initio; it has no legal effect.

However, public policy favors finding a marriage to be valid: “the law presumes morality and not
immorality.” Betty might rely on either of two “marriage-saving” doctrines to avoid the conclusion that
her marriage to Herb is void:

1. There is a presumption that the latest of a series of marriages is valid. The presumption is “one of the
strongest presumptions known to the law” and can be rebutted only by “strong evidence that the prior
marriage still subsists or by cogent and conclusive evidence.” The presumption is designed to protect the
parties’ expectations; some cases thus hold that its strength increases with the lapse of time and the birth
of children.

Seperac-J19 Exam-Released MEE Essay Compilation © 2016-2020 938


In some jurisdictions the presumption of validity could be rebutted by evidence that Herb was still
married to Ann when he married Betty, even though his divorce from Ann was later granted. The fact that
Herb’s marriage to Betty was short might also lead a court to find the presumption rebutted. But because
the presumption is essentially an equitable doctrine, many courts would estop Herb from rebutting the
presumption of validity based on his “unclean hands,” Betty’s innocence, and the fact that Ann will not be
harmed by a finding that the marriage to Betty is valid because she has been divorced from Herb for a
period of time.

[NOTE: This issue can be argued either way. Applicants should receive credit for correctly stating the
legal principles and making a cogent argument.]

2. Removal of the impediment (the divorce) renders the subsequent marriage valid. Under the Uniform
Marriage & Divorce Act § 207, parties to “a marriage prohibited under this section who cohabit after
removal of the impediment are lawfully married as of the date of the removal of the impediment.”

Using one or the other or some combination of these marriage-saving doctrines, it is probable that the
court would find that Herb was married to Betty, and therefore deny Herb’s motion to dismiss the divorce
petition.

Seperac-J19 Exam-Released MEE Essay Compilation © 2016-2020 939


#214-JUL 2005–MEE Q04: QUESTION FOUR (FAMILY LAW)

Husband and Wife married and lived together in State X. While married to Husband, Wife had an affair
with Fred and gave birth to Fred’s child, Child. Both Husband and Fred knew that Fred was Child’s
biological father.

After Child’s birth, Husband and Wife separated. Wife and Child immediately moved in with Fred and
stayed with him for three months. During those three months, Fred supported Wife and Child and told
friends and neighbors that Child was his son.

At the end of this three-month period, Wife reconciled with Husband. She and Child left Fred’s home and
resumed living with Husband. Husband held Child out as his son and supported Child. Although Fred no
longer supported Child, he communicated with Child approximately twice a year.

When Child was four years old, Fred filed a petition claiming paternity of Child under the law of State X.
State X law provides that “a proceeding brought by an individual, other than the child, to adjudicate the
parentage of a child having a presumed father must be commenced not later than two years after the birth
of the child.” The court dismissed Fred’s petition, holding that the two-year statute of limitations barred
his action.

Shortly after Fred’s paternity claim was dismissed, Husband, Wife, and Child moved to another city in
State X, and Fred could not locate them.

Two years later, Wife sued Husband for divorce. In her divorce action, she sought custody of Child and
child support. Husband objected to Wife’s child support claim on the ground that Fred, Child’s biological
father, should be required to pay support instead of Husband. A State X court awarded Wife custody of
Child and ordered Husband to pay child support.

Shortly after the divorce was final, Fred located Wife and Child. Fred filed a visitation petition under a
State X statute permitting “any person to petition for visitation at any time” and authorizing a court to
grant visitation whenever “visitation serves the best interests of the child.” The trial court granted Fred’s
petition over Wife’s objection, holding, as a matter of law, that “it is always in a child’s best interests to
know his or her biological parents, regardless of the custodial parent’s views about the child’s needs.”

1. Did the court violate Fred’s substantive due process rights under the U.S. Constitution by
dismissing his paternity petition? Explain.

2. Did the court properly order Husband to pay child support? Explain.

3. Did the court violate Wife’s substantive due process rights under the U.S. Constitution by
granting Fred visitation? Explain.

Seperac-J19 Exam-Released MEE Essay Compilation © 2016-2020 940


#214: J05-4 MEE: ANSWER: NCBE (FAMILY LAW)

POINT (1) [47%] ISSUE: Are a biological father’s parental rights under the U.S. Constitution
violated if he has only two years after the child’s birth to establish that he is the father of a married
woman’s child? ANSWER: No. The statute of limitations in State X’s paternity statute does not
violate the substantive due process rights of Fred, the biological father, under the U.S. Constitution.

POINT (2) [16%] ISSUE: Is a husband who was married to a child’s mother at its birth obligated
to support a child who is not biologically related to him if he has failed to challenge his paternity in
a timely manner and has induced reliance on his continued parental support? ANSWER: Yes.
Husband is conclusively presumed to be Child’s father and is estopped from denying his child
support obligation.

POINT (3) [37%] ISSUE: Are a mother’s parental rights violated by granting visitation to a
biological father when the mother’s ex-husband is the child’s legal father and the court grants no
deference to the mother’s views on visitation? ANSWER: Yes. Wife’s due process rights were
probably violated when the court granted Fred visitation without any deference to her
determination of what is in Child’s best interest.

ANSWER DISCUSSION:

Fred’s parental rights were not violated when the State X court dismissed his untimely petition. Although
unwed fathers who have established a significant relationship with their biological children have a
substantive due process right entitled to protection, it is unlikely that Fred could show such a relationship
given his limited contact with Child and his failure to file a paternity action when he was entitled to do so.
Moreover, even if Fred were found to possess the requisite relationship because his attempts to establish
contact with Child were thwarted by Wife, Fred’s rights would be limited because Husband is presumed
to be Child’s father. The statute of limitations that restricts Fred’s ability to challenge the paternity of
Wife’s husband is a more lenient provision than the conclusive presumption of legitimacy that was upheld
against a substantive due process challenge in Michael H. v. Gerald D.Husband is obligated to pay child
support for Child. He will be estopped from denying his paternity because he induced reliance on his
continued support and failed to challenge his paternity within the two-year statute of limitations. Wife’s
constitutionally protected parental rights were violated by granting visitation to Fred when the court gave
no deference to Wife’s assessment of Child’s best interest.

ANSWER EXPLANATION:

Explanation to Point-One (40-50%):

The statute of limitations in State X’s paternity statute does not violate the substantive due process rights
of Fred, the biological father, under the U.S. Constitution.

Under the U.S. Constitution, an unwed biological father who has a significant relationship with his
biological child has a substantive due process right in that relationship. But the Supreme Court has also
held that it is only “where an unwed father demonstrates a full commitment to the responsibilities of
parenthood by ‘coming forward to participate in the rearing of his child,’ that his interest in personal
contact with his child acquires substantial protection under the Due Process Clause.” Fred lived with
Child for only three months and communicated with him only twice a year thereafter. He paid no support

Seperac-J19 Exam-Released MEE Essay Compilation © 2016-2020 941


and did not offer to do so. He could have established his paternity when Wife moved out, but chose to
wait until Child was four years old. Consequently, a court would almost certainly hold that Fred lacked
the type of relationship warranting substantive due process protection.

Even if Fred could establish a significant parental relationship with Child, the fact that Husband is Child’s
presumed father would limit Fred’s constitutional rights. Under the common law, if a mother was married
at the time she gave birth, her husband was presumed to be her child’s father. Like State X, most U.S.
jurisdictions generally retain, in statutory form, some version of the common law presumption of
legitimacy. In Michael H. v. Gerald D., the U.S. Supreme Court upheld a California statute that precluded
a biological father from bringing a paternity action when the mother of his child was married and she did
not join in his paternity petition. In a plurality decision, the Court held that, because of the state’s interest
in protecting the marital family and the lack of any established tradition recognizing an extramarital
father-child relationship, there was no substantive due process violation even though the father wishing to
establish his paternity had had a significant residential relationship with his child. If an unmarried father
whose child has a presumed father can be completely foreclosed from unilaterally establishing his
paternity, a two-year statute of limitations is certainly constitutional.

In the wake of the Michael H. opinion, many states have enacted statutes which give an unmarried father
no more than two years to establish his paternity when his alleged child has a presumed father. Pursuant
to the Uniform Parentage Act, a two-year period allows an adequate period to resolve the status of a child
within the context of an intact family unit; a longer period may have severe consequences for the child in
that circumstance.

[NOTE: An applicant may erroneously discuss procedural due process or equal protection. If the above
points are made within the context of a procedural due process or equal protection analysis, an applicant
should get credit. However, an answer concluding that a two-year statute of limitations is per se
unconstitutional is incorrect. While the U.S. Supreme Court has struck down one- and two-year paternity
statutes of limitation , these cases involved the right of a child to establish a parental relationship and
obtain a support order. Here, by contrast, Husband is liable for Child’s support.]

Explanation to Point-Two (10-20%):

Husband is conclusively presumed to be Child’s father and is estopped from denying his child support
obligation.

Husband could have challenged the presumption of legitimacy within two years of Child’s birth, but
chose not to do so even though he knew he was not Child’s biological father. Instead, he held Child out as
his own and supported him. He encouraged or, at the very least, acquiesced in Wife’s decision not to
establish Fred’s paternity within the statutory period and in her termination of Fred’s visits with Child.

Even if the State X paternity statute permitted Husband to challenge his paternity, here he would be
obliged to pay child support. When a nonparent consents to act as a parent and the child’s interests would
be harmed by termination of the parental relationship, the nonparent is estopped from disclaiming parental
responsibilities. For example, a same-sex partner who encouraged the mother to utilize artificial
insemination to bear a child would be estopped. Courts have often applied the estoppel principle when, as
here, a mother’s husband who knew her child was not his induced detrimental reliance on his continued
willingness to act as a parent. Wife is now statutorily barred from seeking support from Fred, and Child
has had no meaningful paternal relationship except with Husband. By assuming the rights of a parent,
Husband also assumed the responsibilities of parenthood: he is now Child’s father for all purposes.

Seperac-J19 Exam-Released MEE Essay Compilation © 2016-2020 942


Explanation to Point-Three (30-40%):

Wife’s due process rights were probably violated when the court granted Fred visitation without any
deference to her determination of what is in Child’s best interest.

The State X statute is similar to the Washington statute that was held unconstitutional, as applied, in
Troxel v. Granville. In its plurality decision, the Troxel Court noted that the statute “effectively permits
any third party seeking visitation to subject any decision by a parent concerning visitation of the parent’s
children to state-court review,” and that the state court had given no deference to the parent’s decision that
visitation was not in her child’s best interest. The Court held that, before “a fit parent’s visitation decision
becomes subject to judicial review, the court must accord at least some special weight to the parent’s own
determination.”

Here, the State X court accorded Wife’s decision no weight at all. Instead, it ruled, as a matter of law, that
“it is always in a child’s best interests to know his or her biological parents, regardless of the custodial
parent’s views about the child’s needs.”

Although some state courts have held that Troxel is inapplicable to visitation petitions from a nonparent
who lived with the child and acted as a “de facto” parent, it is highly unlikely that Fred could establish
such a relationship with Child. Fred lived with Child for only three months after Child’s birth and
thereafter saw Child on only a handful of occasions. Child may not even remember Fred.

Nor can Troxel be distinguished because Fred is a biological parent. A child has only one legal father.
Fred’s opportunity to establish parental rights was foreclosed with the dismissal of his paternity suit.
Therefore Fred is a “third party” and has no more rights to visitation than any other “third party.”

[NOTE: An exceptional answer might try to distinguish Troxel from the case at hand based on the fact
that, given Husband’s attempt to disclaim paternity of Child, Child may not have a meaningful
opportunity to enjoy a paternal relationship unless Fred’s visitation petition is granted. It could be
argued that this fact makes Fred different from the typical third-party visitation petitioner and more like
the de facto parents to which some state high courts have refused to apply the Troxel standard.

It is difficult to determine how a court would rule on this argument. Fred is Child’s biological parent and
appears to be more interested at this point in maintaining a relationship with Child than does Husband,
Child’s legal father. But to award Fred visitation would perhaps undercut the policies that underlie the
paternity statute of limitations. In addition, Fred’s earlier relationship with Child was insubstantial.]>

Seperac-J19 Exam-Released MEE Essay Compilation © 2016-2020 943


#215-FEB 2005–MEE Q04: QUESTION FOUR (FAMILY LAW)

Harold and Wendy were divorced in State A and awarded joint custody of their two children, John, age 5,
and Amanda, age 3. Under the terms of the divorce decree, both parents were to share in decisions
concerning the care, education, religion, medical treatment, and general welfare of the children, but
Wendy was awarded primary physical custody, which, in this case, meant that the children would live
with her except during alternate weekends and the month of July. Harold was also ordered to pay monthly
child support in the amount of $500 per child until each child reached the age of 18.

Two years after the divorce, Wendy was offered a job in State B that would double her salary but
necessitate a move to an area of State B located about 100 miles from Harold’s home in State A. When
Wendy told Harold of her desire to accept the job offer and move to State B, he objected that such a move
would prevent him from exercising his visitation rights.

Harold petitioned the State A court to issue an order prohibiting Wendy from relocating with the children.
The court found that Wendy’s decision to move was motivated by a desire to improve her family’s
standard of living. It denied Harold’s petition. Wendy then moved with the children to State B, where she
began her new job, set up a new residence, and registered the child support order.

After Wendy and the children moved to State B, Harold stopped making child support payments. In
response, Wendy sought enforcement of the State A support order in a State B court. Wendy also
petitioned the State B court to extend Harold’s obligation to pay child support until the children reach age
21, as authorized under the law of State B. Under the law of State A, there is no obligation to pay child
support for children over the age of 18. She further petitioned the court to modify the State A custody
order by eliminating the joint decision-making provision. Harold was personally served when he came to
State B to take the children on vacation.

In response to Wendy’s petition, Harold claimed that the State B court did not have jurisdiction to enforce
or modify any of the State A orders.

1. Was the State A court’s decision to allow Wendy to relocate with the children to State B correct?
Explain.

2. May the State B court enforce the State A child support order? Explain.

3. May the State B court modify the State A child support order by extending the support obligation
to age 21? Explain.

4. May the State B court modify the State A custody decree by eliminating the joint decision-
making provision? Explain.

Seperac-J19 Exam-Released MEE Essay Compilation © 2016-2020 944


#215: F05-4 MEE: ANSWER: NCBE (FAMILY LAW)

POINT (1) [30%] ISSUE: May a custodial parent relocate to another state when the move infringes
on the noncustodial parent’s right of visitation, and was it proper to allow relocation of the children
in this case? ANSWER: Yes. If a custodial parent’s relocation petition is made in good faith and
not for the of defeating the other parent’s visitation rights and if the move is in the best interests of
the children, permission to move with the children will be granted.

POINT (2) [23%] ISSUE: May a court in a state other than that of the original child support order
enforce the collection of child support in a case where the obligor still resides in the original state
but has been personally served in the state where modification is sought? ANSWER: Yes. A
registered child support order is enforceable in a nonissuing state. The interstate enforcement of
child support is governed by UIFSA, which has been adopted by all states. Although the courts of
State B have personal jurisdiction over Harold based on in-state personal service , personal
jurisdiction is not enough to give a State B court jurisdiction to enforce or modify the State A
support and custody decrees.

POINT (3) [23%] ISSUE: May a court in a state other than that of the original child support order
modify the order based on its own law in a case where the obligor still resides in the original state
but has been personally served in the state where modification is sought? ANSWER: No. The state
that issued a child support order retains exclusive jurisdiction to modify that order as long as the
support obligor continues to reside in that state.

POINT (4) [23%] ISSUE: May a court in a state other than that in which the original child custody
order was entered modify a child custody decree when the parent-contestant still resides in the
original state but has been personally served in the state where modification is sought? ANSWER:
No. Only the courts of the state that originally issued a child custody order may modify that order
while the child or any contestant continues to reside in the issuing state.

ANSWER DISCUSSION:

Although the various states utilize a variety of decision-making standards in judging parental relocation
requests that inhibit visitation with the noncustodial parent, a move sought in good faith that will serve the
children’s best interests ordinarily will be granted. Although State B obtained personal jurisdiction over
Harold when he was personally served in State B, jurisdiction over the interstate enforcement and
modification of child support orders is governed by the Uniform Interstate Family Support Act (UIFSA).
Under that Act, State B would have jurisdiction to enforce the State A order as long as Wendy registers it,
but because Harold is still a resident of State A, State B would not have jurisdiction to modify the order.
Jurisdiction over the interstate modification of a child custody order is governed by the federal Parental
Kidnapping Prevention Act (PKPA). Under the PKPA, as well as the Uniform Child Custody Jurisdiction
and Enforcement Act (UCCJEA), the State B court would not have jurisdiction to modify the State A
custody order decree because Harold is still a resident of State A.

ANSWER EXPLANATION:

Explanation to Point-One (30-35%):

Seperac-J19 Exam-Released MEE Essay Compilation © 2016-2020 945


If a custodial parent’s relocation petition is made in good faith and not for the of defeating the other
parent’s visitation rights and if the move is in the best interests of the children, permission to move with
the children will be granted.

In enunciating standards to govern relocation disputes, courts have balanced the impact on visitation by
the noncustodial parent against the benefits of the move to both the children and the custodial parent.
Although the law in this area “has been unusually unstable, with some states having undergone rather
significant shifts in their standards over recent years , the clear trend has been that of increasing leniency
toward the parent with whom the child has been primarily living.” Some states now permit the custodial
parent and child to move unless the parent’s motives for moving are vindictive. Other states require the
parent who wishes to relocate to show that the proposed move serves the child’s best interests. Yet others
will permit the custodial parent to relocate unless the evidence shows that the move will be detrimental to
the child. Some courts have also emphasized the availability of an alternative reasonable visitation
schedule; if such an alternative exists, the court will almost always grant the relocation request.

Under any of these approaches, Wendy should be permitted to relocate to State B. There is no evidence
that Wendy’s proposed move was aimed at defeating Harold’s visitation rights, and the court found that
her request was made in good faith. Wendy was offered a job in State B at a salary twice that of her job in
State A, an improvement in her economic circumstances that would clearly benefit the children. There is
no evidence that the move would adversely affect the children socially or educationally. Both are very
young (ages 5 and 7) and thus highly unlikely to suffer from the loss of long-term relationships or the
disruption of an academic program. Because Wendy’s new home in State B is only 100 miles from
Harold’s, it is likely that a reasonable alternative visitation schedule that will fully protect the children’s
relationship with Harold can be worked out: the original visitation order called for visitation on alternate
weekends and during the month of July; certainly the July visitation schedule could be preserved and
some of Harold’s alternate weekends could be replaced with longer visitation periods during school
holidays; if Harold and Wendy were to share the job of transporting the children to Harold’s home,
weekend visits also seem quite plausible. Accordingly, the court was correct in allowing Wendy to
relocate with the children.

[NOTE: Applicants who analyze this issue on the basis of the constitutional right to travel should not
receive credit. However, applicants who note that some courts use that right as a rationale for a
consensual relocation standard might receive a modicum of extra credit.]

Explanation to Point-Two (20-30%):

A registered child support order is enforceable in a nonissuing state. The interstate enforcement of child
support is governed by UIFSA, which has been adopted by all states. Although the courts of State B have
personal jurisdiction over Harold based on in-state personal service , personal jurisdiction is not enough to
give a State B court jurisdiction to enforce or modify the State A support and custody decrees.

Under UIFSA § 603, a registered child support order issued in another state is “enforceable in the same
manner and is subject to the same procedures as an order issued by a tribunal of this State.” Thus, the
State B court may exercise personal jurisdiction over Harold and enforce the State A child support order
because Wendy registered that order.

[NOTE: Applicants could receive extra credit for noting that Wendy could still enforce the State A order
through a State B court even if State B did not have personal jurisdiction over Harold. To do so, she
would utilize UIFSA’s two-state procedures.]

Seperac-J19 Exam-Released MEE Essay Compilation © 2016-2020 946


Explanation to Point-Three (20-30%):

The state that issued a child support order retains exclusive jurisdiction to modify that order as long as the
support obligor continues to reside in that state.

The interstate modification of a child support order is also governed by UIFSA. Under UIFSA, the state
that originally issued a child support order has continuing, exclusive jurisdiction to modify the order if
that state remains the residence of the obligee, the child, or the obligor, and at least one of the parties does
not consent to the jurisdiction of another forum. Under the UIFSA, a tribunal of the State shall recognize
and enforce, but may not modify, a registered order if the issuing tribunal had jurisdiction. Harold
continues to reside in State A and has not consented to the jurisdiction of another forum. Thus, a State B
court does not have jurisdiction to modify the State A order. An extension of the child support obligation
from 18 to 21 years would represent an impermissible modification because, under UIFSA, the law of the
issuing state, State A, governs the support obligation, not the law of the enforcing state, State B. The fact
that Harold was personally served in State B does not change this result.

Explanation to Point-Four (20-30%):

Only the courts of the state that originally issued a child custody order may modify that order while the
child or any contestant continues to reside in the issuing state.

The interstate modification of a child custody order is governed by the federal Parental Kidnapping
Prevention Act (PKPA). The joint decision-making provision that Wendy sought to modify is part of the
State A custody order, and the PKPA provides that a state may not modify a custody order issued by
another state if either the child or any contestant continues to reside in the issuing state and the issuing
state’s courts do not decline to exercise jurisdiction. State B’s personal jurisdiction over Harold does not
alter this result. Under the Supremacy Clause, the PKPA takes precedence over any conflicting state law.

It is also unlikely that State B law would conflict with the PKPA, as the Uniform Child Custody
Jurisdiction and Enforcement Act (UCCJEA), which in 2003 had been adopted in 31 states, contains
custody-modification standards virtually identical to those of the PKPA. Under the UCCJEA, a state that
properly issued a custody decree retains continuing, exclusive jurisdiction until all parties and the child
have left the state, or an issuing state court has determined that there is no longer any significant
connection between the child and the person remaining in the state and that substantial evidence is no
longer available in that state.

Since there are no facts to suggest that State A would decline to exercise jurisdiction, a State B court may
not modify the State A custody decree. Thus, if Wendy wishes to modify the custodial arrangement, she
will be required to institute proceedings in State A.

Seperac-J19 Exam-Released MEE Essay Compilation © 2016-2020 947


#216-JUL 2004–MEE Q03: QUESTION THREE (FAMILY LAW)

Husband and Wife were married for 12 years. The couple had one child, Boy, age 7. Husband and Wife
were both devoted parents.

Wife filed for divorce shortly after she learned that Husband and Secretary were having an affair. In the
divorce action, Husband and Wife each sought sole custody of Boy.

The court appointed a child custody evaluator. Both parents told the evaluator they were not willing or
able to share custody. Boy told the evaluator that he was very upset because his parents were getting a
divorce and that he wanted to live with his father. Based upon the child custody evaluator’s
recommendation, the court awarded Husband sole custody of Boy and gave Wife liberal visitation rights.
Neither party appealed this decree. Husband’s and Wife’s relationship has remained bitter and hostile.

Three months after the custody decree was filed, Secretary moved into Husband’s house. Immediately
thereafter, Wife filed a petition seeking to modify the custody decree and obtain sole custody of Boy.

At a hearing on Wife’s petition, Boy testified, “I miss my mom and I am sad that my parents are
divorced.” Husband testified that there had been no change in Boy’s behavior since Secretary moved into
his home and that Boy got along well with Secretary. Wife testified that Boy should not be exposed to his
father’s nonmarital cohabitation. There was no other testimony. Neither Husband nor Wife sought joint
custody.

The court modified the custody decree and awarded Husband and Wife joint custody. Under the modified
decree, Boy will reside with each parent for alternating two-week periods and the parents will share
decision-making responsibilities. The court held that “this arrangement will give Boy the best of both
parents and allow each parent to counteract any negative influence on Boy by the other parent.”

Husband appeals the court’s determination.

1. Did the court err in modifying the custody decree? Explain.

2. Did the court err in awarding joint custody? Explain.

Seperac-J19 Exam-Released MEE Essay Compilation © 2016-2020 948


#216: J04-3 MEE: ANSWER: NCBE (FAMILY LAW)

POINT (1) [70%] ISSUE: Did Secretary’s move into Husband’s home and Boy’s testimony about
missing his mother constitute a substantial change of circumstances justifying reevaluation of the
custody decree? ANSWER: No. Because the motion to modify was made within three months of the
original custody order and the evidence did not show that either Secretary’s move into Husband’s
home or Boy’s feelings of sadness threatened his well-being, an appellate court would likely find
that Wife had failed to establish a substantial change of circumstances.

POINT (2) [30%] ISSUE: Was joint custody in Boy’s best interests given that his parents did not
agree to joint custody and had a hostile relationship? ANSWER: No. Because the parents did not
agree to joint custody and had a high-conflict relationship, an appellate court would likely find that
joint custody was not in Boy’s best interests.

ANSWER DISCUSSION:

The trial court erred in modifying the custody order. A substantial change of circumstances that would
justify custody reevaluation probably did not exist given the short time that had elapsed since entry of the
original custody order. Although a custodial parent’s nonmarital cohabitation might, in some states,
constitute an unforeseen and substantial change of circumstances justifying custody reevaluation, Wife
did not show that the cohabitation negatively affected Boy or his relationship with his father. Nor did
Wife show that Boy’s feelings of sadness were attributable to the cohabitation or even that they were new.
Assuming that a substantial change of circumstances existed, joint custody would not be in Boy’s best
interests: the parents did not agree to joint custody, they would not cooperate, and the mother already had
liberal visitation.

ANSWER EXPLANATION:

Explanation to Point-One (65-75%):

Because the motion to modify was made within three months of the original custody order and the
evidence did not show that either Secretary’s move into Husband’s home or Boy’s feelings of sadness
threatened his well-being, an appellate court would likely find that Wife had failed to establish a
substantial change of circumstances.

Custody orders are invariably modifiable, but modification is impermissible unless there has been a
change in circumstances since the order was entered. Most states require that the change in circumstances
be substantial and unforeseen. For example, the Uniform Marriage and Divorce Act (UMDA) requires
that there be “facts that have arisen since the prior decree or that were unknown to the court at the time of
entry of the prior decree.”

Most states also disfavor modification when sought shortly after a custody decree has been entered; the
principle of res judicata and the belief that children’s interests are served by stable custody arrangements
both contribute to this view. Thus, if a modification petition is filed within two years of the original
decree, UMDA § 409 authorizes a modification hearing only if the evidence suggests “there is reason to
believe that the child’s present environment may endanger seriously his physical, mental, moral or
emotional heath.” In states that do not have statutory standards restricting early modification, case law
typically disfavors it unless the evidence shows that the child is at risk. Because the modification petition

Seperac-J19 Exam-Released MEE Essay Compilation © 2016-2020 949


was brought within three months of the original order and the evidence did not show any risk of harm to
Boy, appellate courts in most states would likely find that there was an insufficient basis to modify
custody.

In some states, appellate courts have held that a custodial parent’s post-decree nonmarital cohabitation
represents a change of circumstances sufficient to warrant a modification hearing. In these states, an
appellate court would likely find that the trial court did not err in hearing evidence on modification. But,
even in states that authorize a hearing in these circumstances, modification is typically disallowed unless
the petitioner shows that the nonmarital cohabitation has an adverse impact on the children. Under the
UMDA, the court cannot consider the conduct of a proposed custodian that does not affect his relationship
to the child. Wife presented no evidence at the hearing to show such an impact nor did she even show that
Boy’s feelings of sadness were attributable to Secretary’s presence in the home. In other states, however,
the mere fact of cohabitation may disqualify a parent to have custody.

A child’s custody preference is relevant to a custody determination, and alteration of a child’s custody
preference thus can constitute a substantial change of circumstance. Here, Boy’s testimony does not
clearly indicate any alteration of his custody preference; Boy testified that he missed his mother, not that
he wanted to live with her. Nor did the evidence show that Boy’s feelings of sadness are attributable to
living with his father instead of facing his parents’ divorce. But even if Boy’s testimony were interpreted
to mean that he wants to live with his mother, the feelings of young children are typically given less
weight than feelings of those who are mature. Consequently, it is highly unlikely that Boy’s statement
would be sufficient to qualify as a substantial change of circumstance. Under case law, whether a child’s
views qualify as a substantial change of circumstance depends upon the depth, sincerity, and the extent
they reflect changed circumstances within the parent-child relationship or relationship between the
parties.

Explanation to Point-Two (25-35%):

Because the parents did not agree to joint custody and had a high-conflict relationship, an appellate court
would likely find that joint custody was not in Boy’s best interests.

Even if there is a substantial change in circumstances, a court may not modify a custody order unless the
change will serve the child’s best interests. It is generally understood that requiring hostile parents to
share custody can be harmful to children. Thus, “with few exceptions, courts and commentators agree that
joint custody is a viable option only for parents who are able and willing to cooperate with one another in
making decisions for their child.”

Most courts will not impose joint custody on unwilling parents. Even in states that do permit the
imposition of joint custody over a parent’s objection, a court may not order joint custody unless it finds
that this arrangement is in the child’s best interests. In making such an assessment, it would be reversible
error if the court did not consider the extent of parental conflict and the likelihood of achieving both
cooperation between the parents and a stable living situation for the child.

The joint custody order in this case also fails to address the basis for Wife’s modification petition. Boy
will still be living in the same home with Husband’s nonmarital cohabitant for substantial periods of time.

An appellate court is thus likely to find that the trial court erred both in modifying the custody decree and
in ordering joint custody.

Seperac-J19 Exam-Released MEE Essay Compilation © 2016-2020 950


#217-FEB 2004–MEE Q03: QUESTION THREE (FAMILY LAW)

Harold is the CEO and sole owner of a real estate business that he began about 25 years ago, shortly after
he married Wendy. Wendy quit her job when they married and has ever since been a full-time
homemaker. Last year, Harold became romantically involved with Carol, an employee of his real estate
business. Wendy recently discovered this affair and also learned that Harold had bought expensive gifts
for Carol, including a house purchased in Carol’s name. Wendy sued Harold for divorce, based on his
adultery with Carol.

Wendy would agree that she has never been a good housekeeper. The house was often dirty until the
weekly visit by a cleaning company. Meals were often brought in from local restaurants. Wendy spent
much of her time tending to the couple’s two children, who are now adults, although a live-in nanny
helped her until both children entered school. At Harold’s request, Wendy occasionally entertained
Harold’s clients.

Harold has sole title to the real estate business and a private vested pension to which he has contributed
since beginning his real estate business. Harold and Wendy have joint title to their family home and a
bank account. These assets were acquired with funds Harold earned during the marriage. Wendy has sole
title to 1,000 shares of stock she inherited shortly after she married Harold.

1. Are the real estate business, Harold’s pension, the jointly titled family home, the joint bank
account, and the stock subject to division at divorce? Explain.

2. What effect, if any, would Harold’s affair with Carol and his gifts to her have on any property
division? Explain.

3. What arguments regarding property division are available to Harold and Wendy based on
Wendy’s role as a mother and homemaker? Explain.

Seperac-J19 Exam-Released MEE Essay Compilation © 2016-2020 951


#217: F04-3 MEE: ANSWER: NCBE (FAMILY LAW)

POINT (1) [40%] ISSUE: What property is subject to division on divorce? ANSWER: All of
Harold and Wendy’s assets that were acquired with funds earned by Harold during the marriage
are marital property subject to division on divorce. Wendy’s stock, which was acquired by
inheritance, would be subject to division only in those states that authorize the division of all
property.

POINT (2) [30%] ISSUE: Will a court consider Harold’s adultery or his gifts to Carol in dividing
property upon divorce? ANSWER: No. In the majority of jurisdictions, adultery is not relevant to
division of property at divorce, but dissipation of assets is relevant. Harold’s large gifts to Carol
would qualify as dissipation.

POINT (3) [30%] ISSUE: Will a long-term homemaker’s role be relevant to a determination of that
spouse’s share of the property upon divorce? ANSWER: Yes. In most jurisdictions, Wendy’s role
as a homemaker and parent would be relevant to equitable property distribution, both as a
contribution to the marriage and as an indicator of need.

ANSWER DISCUSSION:

The real estate business, the joint bank account, the jointly titled family home, and the pension are all
subject to division as “marital property” because they were acquired with funds earned by a spouse during
the marriage. In most states, Wendy’s stock would not be subject to division because it was acquired by
inheritance and is Wendy’s separate property. However, a minority of states allow courts to divide even
this separate property. Most states would not consider Harold’s affair with Carol relevant to the division
of property but would consider Harold’s large gifts to Carol as relevant. All equitable distribution statutes
require consideration of both need and contribution. Wendy’s homemaking services would be relevant
because they provide a basis for arguments about both her need and contribution. Wendy and Harold will
make very different arguments about how Wendy’s role as a homemaker and mother should affect
property distribution. Ultimately, it is impossible to predict the weight the court would place on Wendy’s
contribution or her need in making the overall property award.

ANSWER EXPLANATION:

Explanation to Point-One (25-35%):

All of Harold and Wendy’s assets that were acquired with funds earned by Harold during the marriage are
marital property subject to division on divorce. Wendy’s stock, which was acquired by inheritance, would
be subject to division only in those states that authorize the division of all property.

A majority of states require the court to first classify the property held at the time of the divorce as marital
or separate and authorize the court then to divide the marital property. In the eight community property
states, community property is treated much the same as marital property. Definitions of marital and
community property are nearly identical. As a result, all states will allow the court to distribute marital or
community property, which includes all property derived from earnings during the marriage.

All of the property, except Wendy’s stock, will be considered marital property in a common-law state or
community property in a community property state. The real estate business, the pension, the jointly titled

Seperac-J19 Exam-Released MEE Essay Compilation © 2016-2020 952


family home, and the joint bank account were acquired with spousal earnings. How the assets are titled is
irrelevant.

Wendy’s stock will not be subject to division in the majority of states. It is Wendy’s separate property
because Wendy inherited it. An inheritance is the separate property of the spouse who inherits it, even if
the spouse acquires it during the marriage.

A minority of states (about fourteen) authorize the courts to divide all the property held by the spouses at
the time of divorce without regard to when, how, or by whom the property was acquired or how the title
was held the so-called “hotchpot” approach. In those states, the stock will be subject to division.

Explanation to Point-Two (20-25%):

In the majority of jurisdictions, adultery is not relevant to division of property at divorce, but dissipation
of assets is relevant. Harold’s large gifts to Carol would qualify as dissipation.

The majority of states do not allow for consideration of adultery in the division of property. A minority of
states could consider Harold’s adultery when making a division of property. Therefore, in most
jurisdictions the fact that Harold had an affair would not be relevant to the property distribution.

In virtually all states, however, “economic” fault, including dissipation of marital assets, is relevant to
property distribution. Under the Uniform Marriage and Divorce Act, courts are directed to divide property
without regard to marital misconduct, yet to consider, inter alia, the contribution or dissipation of each
party in the acquisition, preservation, depreciation, or appreciation in the value of the respective estates.

Dissipation is generally thought to be “the use of marital property for the sole benefit of one of the
spouses for a purpose unrelated to the marriage at a time that the marriage is undergoing an irreconcilable
breakdown.” Harold’s expenditures on Carol were large, were for a nonmarital purpose, inured solely to
the benefit of Harold, and were made during the breakdown of the marriage. Therefore, these
expenditures would be relevant in dividing the property. The remedy would be either to include the value
of the gifts to Carol in the marital estate or to consider the dissipation when making the ultimate
distribution of the remaining property.

Explanation to Point-Three (20-25%):

In most jurisdictions, Wendy’s role as a homemaker and parent would be relevant to equitable property
distribution, both as a contribution to the marriage and as an indicator of need.

In all equitable distribution states, spousal contribution and need are the primary factors in property
division. Homemaking and parenting are recognized forms of spousal contribution. These same services,
when they substitute for paid employment over a lengthy period, will also evidence a spouse’s need.
Therefore, Wendy’s services would provide the basis for two different types of property distribution
arguments.

Most states do not assign any particular value to homemaking services and would permit Harold to argue
that Wendy’s contribution was minimal, given her modest homemaking skills and heavy reliance on paid
help. But some equitable distribution states have a presumption that a homemaker’s contribution is equal
to a breadwinner’s and that an equal division is most just. Whether such a presumption can be rebutted by
evidence that the contribution of the homemaker was not equal to that of the breadwinner varies by state.

Seperac-J19 Exam-Released MEE Essay Compilation © 2016-2020 953


Wendy will likely argue that her homemaking contributions played a significant role in the acquisition of
the marital property. She would emphasize the fact that her role as a full-time homemaker and parent
freed Harold to devote time to building his business. She would also emphasize her services in
entertaining Harold’s clients.

Wendy will also argue that her role as a long-term homemaker has greatly increased her need for marital
property by reducing her capacity for self-support and asset accumulation. This claim would be difficult
for Harold to rebut.

Given that equitable property distribution is highly discretionary, it is impossible to estimate how much
weight the court would place on Wendy’s need as compared to her contribution. It is also impossible to
determine how the court would evaluate Wendy’s and Harold’s contribution-based arguments.

In the small minority of states that require equal division of marital property, arguments about Wendy’s
contribution and need would, of course, be unavailable to both parties.

Seperac-J19 Exam-Released MEE Essay Compilation © 2016-2020 954


#218-JUL 2003–MEE Q06: QUESTION SIX (FAMILY LAW)

NOTE: Applicants answering this question in a community property state should use community
property principles. For this purpose, the phrase “marital property” means community property.

On July 1, 1990, Ann and Burt got married in State X, where they have lived all their lives.

At the time of their marriage, Ann and Burt were each 22 years old. Burt had graduated from high school
and had been working for one year as a data entry technician, earning $30,000 annually. Ann had
graduated from high school and had worked since graduation as a grocery store cashier, earning $27,000
annually. Ann had a trust fund that she acquired when she was 18 years old. The trust fund was worth
$200,000, and Burt knew all about it. Neither had any other property or debts. Ann knew that Burt
planned to become a lawyer, and Burt knew that Ann intended to be a homemaker.

Three months prior to the marriage, Burt told Ann that he would not marry her unless they signed a
premarital agreement. Ann was surprised because Burt had never told her this was a precondition to their
marriage. She reluctantly agreed, and they immediately went to the office of Burt's lawyer, Lawyer.
Lawyer showed Ann and Burt a draft agreement under which both Ann and Burt would waive all rights to
separate and marital property titled solely in the other’s name if they divorced. They would also waive
any right to claim child support if they divorced.

Lawyer told Ann that by signing the agreement she would be waiving her rights to marital property and
child support. Lawyer also told Ann that he represented only Burt, and not her, and that she should retain
her own lawyer. Ann decided not to retain her own lawyer because she trusted Burt. Ann read the
agreement the next day and expressed no reservations about signing it. Ann and Burt then both signed the
agreement.

After the marriage, Burt became a very successful lawyer. Ann became a homemaker and had no out-of-
home employment. The value of the property acquired from Burt’s earnings during the marriage is
$900,000. The value of Ann’s trust fund, managed at all times by her father, is now over $800,000. Ann
and Burt have two children, currently ages seven and ten.

Ann now sues Burt for divorce in State X. Ann and Burt agree that Ann will have custody of the children.
However, the parties cannot agree on the division of property.

Ann seeks property division and child support under the marital dissolution statute. Burt argues that (a)
the court should enforce the premarital agreement, and (b) if the court invalidates the premarital
agreement, that Ann’s trust fund is marital property subject to distribution.

Ann concedes that the premarital agreement was and is substantively fair. Nonetheless, she argues that it
is unenforceable (a) because she lacked legal counsel, and (b) because it addresses property distribution
and child support upon divorce. She also argues that (c) regardless of the premarital agreement’s validity,
her trust fund is not subject to division.

1. How should the court rule on Ann’s and Burt’s arguments regarding the premarital agreement?
Explain.

2. How should the court rule on Ann’s and Burt’s arguments regarding the trust fund? Explain.

Seperac-J19 Exam-Released MEE Essay Compilation © 2016-2020 955


#218: J03-6 MEE: ANSWER: NCBE (FAMILY LAW)

POINT (1)(a) [53%] ISSUE: Should the State X court enforce the premarital agreement even
though the party challenging the agreement lacked legal counsel? ANSWER: Yes. The court in
State X will probably enforce the premarital agreement, except for the provision on child support,
because the agreement was voluntarily entered into.

POINT (1)(b) [24%] ISSUE: Should the State X court uphold the provisions that waive the right to
marital property and child support upon divorce? ANSWER: A premarital agreement can address
property division upon divorce but cannot bind a court on matters of child support.

POINT (2) [24%] ISSUE: Is Ann’s trust fund subject to distribution as marital property?
ANSWER: No. If the premarital agreement is valid, then it would independently shield the trust
fund from Burt’s claim. If the premarital agreement is valid, Burt will probably not be entitled to
any of Ann’s trust fund because it was separate property.

ANSWER DISCUSSION:

[NOTE: Applicants answering this question in a community property state should use community
property principles. For this purpose, the phrase “marital property” means community property.]

The court will probably enforce the premarital agreement’s provisions regarding property division. Ann
appears to have voluntarily entered the agreement despite the fact that she lacked legal counsel. Although
the absence of legal counsel would be a factor a court would consider in assessing voluntariness, the other
facts suggest that the agreement was voluntarily entered into. However, the court will not enforce the
premarital agreement’s provision waiving child support because parents cannot waive a child’s right to
support. If the premarital agreement is upheld, then consistent with the terms, Burt cannot reach Ann’s
trust fund. If the premarital agreement is invalid, in most states Burt will not be able to obtain Ann’s trust
fund because it is separate property. The entire amount is separate property because the base amount was
acquired before marriage and the appreciation in the fund is not the result of spousal labor.

ANSWER EXPLANATION:

Explanation to Point-One(a) (40-50%):

The court in State X will probably enforce the premarital agreement, except for the provision on child
support, because the agreement was voluntarily entered into.

Today in all states premarital agreements are enforceable and not void as “contemplating divorce.”
However, the premarital agreement will not be enforceable against Ann if she can prove that she
involuntarily entered the agreement. Both case law and statutes generally require that consent to a
premarital agreement be voluntary. Here no facts support duress, oppression or unfair surprise.

The fact that Ann did not have a lawyer will probably not affect the court’s determination of whether Ann
entered into the agreement voluntarily. Neither any state nor the UPAA mandates the assistance of
independent counsel. Courts generally have not required independent counsel as a precondition to
enforcing an agreement against an objecting party. Moreover, the lack of independent counsel is probably

Seperac-J19 Exam-Released MEE Essay Compilation © 2016-2020 956


not enough to invalidate the agreement here. Ann was advised to obtain a lawyer, she had plenty of time
before the marriage to do so, and her trust fund and job gave her the resources to do so.

Factors suggesting that the agreement was entered into voluntarily include the length of time between the
date the premarital agreement was presented and the wedding (three months), and the fact that Ann’s
background permitted her to read and understand the agreement (she had a high school education); the
factual history also indicates that Ann read the agreement, that she did not express any reservations about
signing it, that Burt’s lawyer told her of the agreement’s exact effect, and that she had more assets than
Burt when entering the marriage. The facts, on the whole, suggest that Ann voluntarily entered into the
agreement.

Explanation to Point-One(b) (15-25%):

A premarital agreement can address property division upon divorce but cannot bind a court on matters of
child support.

In virtually all states, premarital agreements can include provisions about the disposition of property upon
marital dissolution. Therefore, the provisions on property division are not objectionable because of their
subject matter.

However, almost all states agree that a child’s right to support cannot be adversely affected by a
premarital agreement. The traditional rule is that a contract between prospective spouses cannot bind a
court in deciding child support.

The fact that the child support provision is probably invalid does not preclude enforcement of the rest of
the agreement. Even if the court declares one or more provisions of the premarital agreement to be
unenforceable, the remaining provisions may be valid and enforceable. Typically, if an agreement
contains some unenforceable terms (such as an impermissible waiver of child support), the remaining
terms may be enforced if the parties intended them to be enforceable even without the unenforceable
terms. The standard rules of contract law apply to antenuptial agreements. Here there is no provision
specifying whether the contract is entire or severable, and in all likelihood, the court would find it is
severable and uphold the rest of the agreement.

Explanation to Point-Two (15-25%):

If the premarital agreement is valid, then it would independently shield the trust fund from Burt’s claim. If
the premarital agreement is valid, Burt will probably not be entitled to any of Ann’s trust fund because it
was separate property.

If the premarital agreement is valid, Burt waived his rights in Ann’s separate property and would not be
entitled to any part of the trust fund.

Even if the agreement were not enforced, property acquired before marriage is generally considered
separate property. Therefore, Ann’s trust fund of $200,000 is her separate property. Typically, separate
property is not subject to division in an equitable distribution or a community property regime.

Regarding the additional $600,000 that represents the appreciation in the value of the trust fund, the
general rule is that appreciation of separate property remains separate property if the appreciation is not
attributable to spousal labor. Since the facts state that Ann’s father solely managed the trust, the property
would be considered separate. About fourteen states, however, make all property subject to division upon

Seperac-J19 Exam-Released MEE Essay Compilation © 2016-2020 957


divorce. But even in these states, there is little to support Burt’s contribution to the appreciation of the
asset or his claim of need, the two factors primarily considered by courts in distributing property.

Seperac-J19 Exam-Released MEE Essay Compilation © 2016-2020 958


#219-JUL 2002–MEE Q05: QUESTION FIVE (FAMILY LAW)

Ann and Bert, a married couple, were unable to have biological children because Bert was infertile. They
decided to try artificial insemination by an anonymous donor. Their doctor performed the procedure after
obtaining written consent from both Ann and Bert. As a result of the artificial insemination, Ann became
pregnant.

During the last months of Ann's pregnancy, she and Bert argued constantly, and Ann moved out of the
family home into her own apartment. The baby, Daughter, was born while Ann and Bert were living
apart. Bert visited Ann and Daughter in the hospital and paid their medical expenses. He tried to convince
Ann to reconcile with him, but Ann refused, leaving the hospital with Daughter and returning to her
apartment. Bert continued to visit them and contributed to Daughter's support.

Shortly after Daughter's birth, Ann began an affair with Walt. Walt also spent some time with Daughter
and grew fond of her. When Daughter was one year old, Ann discovered she was pregnant by Walt. When
Walt learned Ann was pregnant, he became very upset and began to abuse Ann verbally and physically.
Ann immediately broke off the relationship with Walt. Shortly thereafter, she reconciled with Bert.

Ann, Bert, and Daughter were living together when Ann's baby, Sonny, was born. Walt, Sonny's
biological father, contacted Ann, apologized for his past abusive behavior, and requested to see Sonny. He
also offered to pay the expenses of Sonny's birth and to contribute to Sonny's support. Ann rejected both
his request and his offer.

When Sonny was six months old and Daughter was two years old, Ann was killed in an automobile
accident. She left a valid will stating that if she died while her children were minors, she wanted Bert to
be named custodian and guardian of both of them.

Walt has sued Bert, seeking to establish himself as Sonny's legal father and requesting custody of both
children. Bert claims that he is the legal father of both Daughter and Sonny and wants to maintain
physical custody of both children. Under the law of the jurisdiction, both parties have standing to raise
these issues.

How should the court rule on Walt's and Bert's claims? Explain.

Seperac-J19 Exam-Released MEE Essay Compilation © 2016-2020 959


#219: J02-5 MEE: ANSWER: NCBE (FAMILY LAW)

POINT (1) [15%] ISSUE: Is the husband of a married woman who conceives a child by artificial
insemination regarded as the legal father of the child if the artificial insemination was performed by
a medical doctor with the husband’s written consent? ANSWER: Yes. Bert is the legal father of
Daughter because he consented in writing to Ann’s artificial insemination, and the procedure was
performed by a doctor.

POINT (2) [35%] ISSUE: If a married woman conceives a child by a man other than her husband,
who is the child’s legal father? ANSWER: Bert, as Ann’s husband, is presumed to be Sonny’s
father. In most jurisdictions, the presumption is rebuttable. Here, Walt, the biological father of
Sonny, can rebut the presumption.

POINT (3) [50%] ISSUE: Under what circumstances can the presumption favoring custody in the
legal parent be overcome? ANSWER: Bert will argue that, as the legal father of Daughter, he is
presumptively entitled to custody of her. The man who is determined to be the legal father of Sonny
will argue that he is presumptively entitled to Sonny’s custody. The presumption of custody in the
legal father can be rebutted, although what is required to rebut the presumption varies from state
to state. Bert would be likely to rebut the presumption, but Walt would not be likely to rebut the
presumption.

ANSWER DISCUSSION:

A child’s legal parent is presumptively entitled to custody both because parental rights are protected by
common-law and constitutional principles and because parental custody is typically consistent with a
child’s best interest. Under the presumption, Bert is Daughter’s legal father because he consented to his
wife’s artificial insemination and the procedure was performed by a doctor. Bert, as Ann’s husband, is
also presumptively Sonny’s father but this presumption is rebutted by the fact that Walt is Sonny’s
biological parent. Presumptively the legal parent is entitled to custody. On the facts, however, Bert may
be able to rebut the presumption that Walt should have custody of Sonny. The third point considers
whether, on the facts given, the presumption favoring custody in a legal parent could be rebutted.

ANSWER EXPLANATION:

Explanation to Point-One (10-20%):

Bert is the legal father of Daughter because he consented in writing to Ann’s artificial insemination, and
the procedure was performed by a doctor.

Even though Bert is not the biological father of Daughter, he is regarded as her legal father in virtually all
American jurisdictions because he consented to his wife’s artificial insemination. The leading case is
People v. Sorensen. The 1973 Uniform Parentage Act provides that a child conceived by artificial
insemination by donor is the legal child of the mother’s husband if he consented in writing and if the
insemination was performed by a medical doctor. Statutes in many states contain similar provisions. Both
these conditions are satisfied here, and Bert is, therefore, the legal father of Daughter.

Explanation to Point-Two (30-40%):

Seperac-J19 Exam-Released MEE Essay Compilation © 2016-2020 960


Bert, as Ann’s husband, is presumed to be Sonny’s father. In most jurisdictions, the presumption is
rebuttable. Here, Walt, the biological father of Sonny, can rebut the presumption.

At common law and under the statutes or case law of most states, a child born to a married woman is
presumed to be her husband’s child. The presumption could generally be rebutted by proof of the
husband’s infertility or his lack of access to his wife.

In those states that would allow the presumption of paternity in the mother’s husband to be rebutted, Walt
could rebut the presumption because Bert is infertile, and because he and Ann were living apart at the
time of Sonny’s conception.

Because Walt can show that he is Sonny’s biological father, Walt will have all the legal rights of a
married father in many states. However, in some states, Walt may still be denied the status of legal father
if Walt has not attempted to assume his parental responsibilities. For example, a New York court found
that the father’s consent to the adoption of his child was unnecessary because the father’s conduct did not
demonstrate an intent to pursue a meaningful relationship with the child. Walt has offered to pay the
expenses of Sonny’s birth and to support Sonny, even though his offers were rejected. He also informally
acknowledged paternity upon Sonny’s birth, although he did not file suit or assert his paternity in a
registry before Ann’s death. On balance, Walt has attempted to exercise his parental responsibilities, and
it would arguably violate Walt’s due process rights to accord him fewer legal rights than other biological
fathers.

In some jurisdictions, however, courts have authority to exclude evidence that would rebut the
presumption favoring paternity in the wife’s husband if rebutting the presumption would be contrary to
the child’s best interest. Bert might argue that rebutting the presumption that he is Sonny’s father is
inconsistent with Sonny’s best interest for essentially the same reasons discussed in Point Three below.

Explanation to Point-Three (45-55%):

Bert will argue that, as the legal father of Daughter, he is presumptively entitled to custody of her. The
man who is determined to be the legal father of Sonny will argue that he is presumptively entitled to
Sonny’s custody. The presumption of custody in the legal father can be rebutted, although what is
required to rebut the presumption varies from state to state. Bert would be likely to rebut the presumption,
but Walt would not be likely to rebut the presumption.

A child’s fit legal parent is presumptively entitled to custody as against a nonparent. In most jurisdictions,
to rebut the presumption favoring custody in the legal parent, a third party must show that awarding
custody to the legal parent will be detrimental to the child. This standard is clearly intended to be more
favorable to the legal parent than is the best interest of the child test, which is used in custody disputes
between parents and does not imply a preference in favor of either claimant. Some states, however, apply
a best interest test to custody disputes between third parties and parents, although this test typically
applies when the nonparent is living with the child and functioning as a parent. An application of the best
interest standard in other situations arguably raises constitutional concerns. For example, the Supreme
Court invalidated the application of a visitation statute where a lower court gave no weight to the
presumption that fit parents act in their children’s best interest.

The presumption of parental fitness applies to the man deemed to be the legal father. Bert, Daughter’s
legal father, benefits from this presumption, and he will undoubtedly be awarded custody of Daughter.
There is no evidence that he is not the fit and proper person to have custody of her. The fact that Walt has
spent “some time” with Daughter and is fond of her is not enough to rebut the presumption. Walt’s best

Seperac-J19 Exam-Released MEE Essay Compilation © 2016-2020 961


argument is that if he is awarded custody of Sonny, he should also be awarded custody of Daughter
because separating Daughter from Sonny would be detrimental to Daughter. Yet, on balance, this
argument would not succeed as Bert can make the same argument (and more persuasively) in the context
of Sonny’s custody.

It is less clear who will be awarded custody of Sonny. The presumption of parental fitness applies to the
man deemed to be the legal father. Bert has a realistic chance of rebutting the presumption if Walt is
Sonny’s legal father. Walt has virtually no chance of rebutting the presumption if Bert is Sonny’s legal
father.

Assuming Walt is Sonny’s legal father, Bert will argue that it would be detrimental to award Sonny to
Walt. Bert is a fit and proper parent; Ann named him as guardian in her will; Sonny is currently living
with him and Daughter; and separating siblings is generally disfavored. Moreover, Bert will argue that
Walt’s history of physical and mental abuse present a question of his fitness for custody.

Assuming Bert is Sonny’s legal father, Walt will have difficulty arguing that an award of custody to Bert
would be detrimental to Sonny. There are no facts given that would support this argument.

Therefore, Walt’s best hope of obtaining custody of Sonny is if he is Sonny’s legal father and the court
rejects Bert’s arguments that an award of custody to Walt would be detrimental to Sonny.

Seperac-J19 Exam-Released MEE Essay Compilation © 2016-2020 962


#220-FEB 2002–MEE Q05: QUESTION FIVE (FAMILY LAW)

Seven months ago, Husband and his wife, Wife, were driving on a highway. Their daughter, Daughter,
age 10, was in the backseat. While driving, Husband negligently turned the wheel of the car to the right
and hit Frank’s car. Frank had parked his car on the shoulder, turned on the emergency flashing lights,
and was changing a flat tire. The accident caused Frank’s car to hit and kill Frank. Frank’s fiancée, Emily,
was with Frank, but was not physically injured. However, she witnessed the accident and held Frank in
her arms as he died.

Wife was seriously injured in the accident. She was hospitalized for a month with broken bones and
internal injuries and is still undergoing physical therapy six months later. Wife and Husband have
remained married.

After the accident, Husband took Daughter to the doctor to be examined. Although Daughter was not
injured in the accident, the doctor found that she was suffering from an incipient bone disease brought on
by calcium deficiency from an imbalanced diet.

Frank and Emily had lived together during their two-year engagement, and Frank died one week before
their planned wedding date. Since Frank’s death, Emily has suffered from depression because she misses
his companionship and intimacy. Consequently, she has been under psychiatric care.

Wife has brought a lawsuit against Husband for his negligence. Emily has also sued Husband for his
negligence. Daughter has sued both Husband and Wife for their failure to provide her with a diet richer in
calcium.

There is no guest statute in this jurisdiction.

1. Assuming Wife can prove a prima facie case of negligence, is Husband liable to Wife? Explain.

2. Are Husband and Wife liable to Daughter for their failure to provide Daughter with a diet richer
in calcium? Explain.

3. On what theory or theories arising from her relationship with Frank might Emily reasonably base
claims for damages against Husband, and what would be the likely outcome of each claim?
Explain.

Seperac-J19 Exam-Released MEE Essay Compilation © 2016-2020 963


#220: F02-5 MEE: ANSWER: NCBE (FAMILY LAW)

POINT (1) [30%] ISSUE: If a wife sues her husband for injuries resulting from her husband’s
negligence, is such action barred by spousal immunity? ANSWER: No. In the great majority of
jurisdictions, negligence suits between husbands and wives are no longer barred by spousal
immunity.

POINT (2) [35%] ISSUE: If a child sues her parents for negligence in an area of parental
discretion, is the action barred? ANSWER: Yes. The majority of courts have abolished absolute
immunity for most parent-child lawsuits. However, in areas dealing with the exercise of ordinary
parental discretion, no suit would be allowed.

POINT (3) [35%] ISSUE: May a bystander who is an unmarried cohabitant and who witnesses her
fiancé’s death recover for negligent infliction of emotional distress or for loss of consortium, or is
such recovery limited only to close family members? ANSWER: No. Both a claim for negligent
infliction of emotional distress and a claim for loss of consortium are likely to fail because Emily
was not married to Frank at the time of the accident.

ANSWER EXPLANATION:

Explanation to Point-One (25-35%):

In the great majority of jurisdictions, negligence suits between husbands and wives are no longer barred
by spousal immunity.

For many years, husbands and wives could not sue each other for negligence. The usual reasons for such
immunity were that such suits would be destructive of marital harmony and would encourage fraud and
collusion against insurance companies.

In the vast majority of jurisdictions, however, interspousal immunity has been abolished as courts have
dismissed fears of disrupting familial harmony and of collusion. In these jurisdictions, if Wife can prove
that Husband’s negligence caused her injuries (as it appears from the facts that it did), then she may
receive compensation for her injuries.

Explanation to Point-Two (30-40%):

The majority of courts have abolished absolute immunity for most parent-child lawsuits. However, in
areas dealing with the exercise of ordinary parental discretion, no suit would be allowed.

Historically, just as husbands and wives could not sue each other, and for comparable concerns about
family harmony, minor children could not sue their parents for personal injury torts. Many jurisdictions,
probably a majority, have abolished this absolute immunity.

Even though parents are not absolutely immune from suits by their children, they have substantial
discretion in making decisions concerning their children’s upbringing. Because of this discretion, children
cannot recover for acts that might otherwise result in liability. Courts are reluctant to substitute their
judgment for that of parents (to act in loco parentis) and to interfere in the arena of family privacy.
Matters falling within the exercise of parental discretion include providing food. (This is clearly not a case

Seperac-J19 Exam-Released MEE Essay Compilation © 2016-2020 964


of true child neglect, where Daughter’s parents have failed to provide sufficient food at all.) Therefore, it
is unlikely that Daughter could successfully maintain a lawsuit against her parents for not giving her more
calcium in her diet.

Explanation to Point-Three (30-40%):

Both a claim for negligent infliction of emotional distress and a claim for loss of consortium are likely to
fail because Emily was not married to Frank at the time of the accident.

In many jurisdictions, when a bystander witnesses an accident which kills or seriously injures a close
family member, and the bystander suffers from severe emotional distress, there may be a cause of action
against the tortfeasor for negligent infliction of emotional distress. In this situation, Emily witnessed the
accident, which was due to Husband’s negligence. After Frank’s death (which occurred while she was
holding him), she suffered the severe distress required to pursue a tort action.

The problem for Emily’s recovery, however, is that she was not Frank’s wife. At the time of his death,
they were engaged and were cohabiting.

A majority of jurisdictions would be unlikely to expand liability, even to a cohabiting fiancée. This
reluctance is based on the difficulties of determining which cohabitants should be allowed to recover and
problems of proving the importance of the relationship. Some courts might also take the position that
recognition of cohabitants’ rights in this cause of action would undermine the strong public policy in
support of marriage.

On the other hand, some jurisdictions have allowed engaged cohabitants to recover in a tort suit of this
type. Emily would argue that her claim should be recognized. Her relationship with Frank was like that of
a husband and wife. They lived together and were to be married in a week. Witnessing his death was
emotionally devastating to her. She had strong emotional ties to Frank, and she should receive
compensation for her severe loss.

Emily might also try to bring a loss of consortium claim. This tort, which is recognized in almost all U.S.
jurisdictions, is intended to compensate a spouse for loss of such things as the other spouse’s
companionship, sexual relations, and affection. While originally only the husband could recover for loss
of consortium, the right was extended to wives during the mid-twentieth century. Compensation for loss
of consortium typically is available only to the legally recognized spouse of the injured party, not to a
fiancée or cohabitant. Thus, Emily would not have a claim for loss of consortium because she and Frank
were not yet married.

[NOTE: Emily might also try to bring a suit for wrongful death, a statutory tort unrecognized at common
law. It is intended to compensate family members for the death of a relative, typically a spouse, child,
parent, etc. Again, however, since Emily and Frank were not married, Emily would not have a claim
worth pursuing.]

Seperac-J19 Exam-Released MEE Essay Compilation © 2016-2020 965


#221-JUL 2001–MEE Q05: QUESTION FIVE (FAMILY LAW)

Father and Mother divorced one year ago after a 14-year marriage. At the time of the divorce, Mother and
Father lived in State A. They were both 39 years old, each had a college education, and they had two
children, aged 10 and 12.

As part of the divorce decree, the court awarded custody of the two children to Mother. The court also
ordered Father to pay Mother $2,000 per month in child support. In addition, the court ordered Father to
pay Mother $500 per month in spousal support for five years. After their property was divided, they each
ended up with $50,000 and a car.

Mother continued living in State A with the children. Mother had been working full-time for $28,000 per
year at a daycare center prior to the divorce. Five months after the divorce, she had a heart attack. This
forced her to cut back to three-quarter-time work, resulting in a pay reduction to $21,000 per year. Her
doctor recommends that she not resume full-time work, because full-time work and caring for the children
and the home would be too stressful.

For the first six months after the divorce, Father paid Mother the full amount he owed; but, for the past six
months, Father has paid Mother nothing. Three months ago, Father was terminated from his $100,000-
per-year job because of company downsizing. He received a lump sum severance payment of $50,000.
Father decided to move to State B, in part because he hoped he could avoid paying Mother and in part
because the job prospects were better there. He transferred all his bank accounts to State B. Although he
has had several interviews and his prospects are good for finding a job comparable to the one he had, he
does not yet have another job.

Mother has brought an action in State B court to collect child support and spousal support from Father.
She claims that the spousal support obligation should be increased to $1,000 per month because she is in
poor health. She also asks that the spousal support be extended for an additional five years.

Father claims that the State A child support order is no longer effective and cannot be enforced because he
has moved to State B. In the alternative, he claims that his child support obligation should be reduced
from $2,000 to $1,000 per month because of his unemployment. In addition, he asks that this modification
be made six months retroactive. Father also opposes any increase in his spousal support obligation.

Both State A and State B are in compliance with federal law concerning the enforcement of child support
orders.

1. Is State B required to recognize the State A child support order? Explain.

2. Does the State B court have jurisdiction to modify Father’s child support obligation? Explain.

3. Without regard to jurisdictional issues, how should a court rule on Father’s requests to modify
his child support obligation? Explain.

4. Without regard to jurisdictional issues, how should the State B court rule on Mother’s request for
an increase in and extension of the spousal support obligations? Explain.

Seperac-J19 Exam-Released MEE Essay Compilation © 2016-2020 966


#221: J01-5 MEE: ANSWER: NCBE (FAMILY LAW)

POINT (1) [23%] ISSUE: Is State B required to recognize the child support order of State A?
ANSWER: Yes. State B is required to recognize the child support order of State A.

POINT (2) [14%] ISSUE: Does State B have jurisdiction to modify the child support order?
ANSWER: No. State B does not have jurisdiction to modify the child support order.

POINT (3) [36%] ISSUE: When will a court reduce a child support award due to changed
circumstances, and can the reduction be made retroactive? ANSWER: A court will reduce a child
support award when there has been a material and substantial change in circumstances; however,
such a change in circumstances has not been demonstrated here. Further, a court cannot order a
retroactive modification of a child support order.

POINT (4) [27%] ISSUE: When will a court increase or extend a spousal support award due to
changed circumstances? ANSWER: An award of spousal support is modifiable only when there has
been a substantial change in circumstances.

ANSWER EXPLANATION:

Explanation to Point-One (20-30%):

State B is required to recognize the child support order of State A.

Under federal law, states are required to give full faith and credit to child support awards from other
states. Under 28 U.S.C. § 1738B, each state: “(1) shall enforce according to its terms a child support order
made consistently with this section by a court of another State.” Section 1738B is known as the Full Faith
and Credit for Child Support Orders Act.

Section IV-D of the Social Security Act also requires a state, as a condition of participation in the
federally funded child support programs, to have procedures that require that “any payment or installment
of support under any child support order be entitled as a judgment to full faith and credit in such State and
in any other State.” This amendment to the Social Security Act may also be called The Child Support
Enforcement Act or the IV-D Program.

Hence, Father cannot evade the State A child support order by moving to State B. State B must recognize
and enforce the State A order. It is no longer the case that child support orders are not entitled to full faith
and credit.

The same result is mandated by the Uniform Interstate Family Support Act (UIFSA), legislation which
states are required to adopt under federal law. Section IV-D of the Social Security Act requires that a
state, as a condition of participation in the federally funded child support programs, have UIFSA in effect.
UIFSA provides a simple procedure for the registration of the child support order of another state. The
order is then enforced in the same manner as an order issued by the registering state. Mother can use
UIFSA to enforce the State A order in State B.

Explanation to Point-Two (10-20%):

Seperac-J19 Exam-Released MEE Essay Compilation © 2016-2020 967


State B does not have jurisdiction to modify the child support order.

State B does not have jurisdiction to modify the State A child support order. Under federal law, each state
“shall not seek or make a modification of a child support order except in accordance with subsection (e).”
(Full Faith and Credit for Child Support Orders Act). Subsection (e) of the Full Faith and Credit for Child
Support Orders Act prohibits the modification of child support orders issued by a court with continuing
exclusive jurisdiction, unless no contestant or child resides there, or unless each contestant has agreed in
writing to allow another state to assert jurisdiction. Those requirements are not met here because Mother
and children still reside in State A, and Mother has not consented to State B’s assuming jurisdiction to
modify.

This same result is mandated by UIFSA. Section 205 of UIFSA confers continuing, exclusive jurisdiction
on the state issuing the child support order unless no litigant or child resides there, or unless each party
has consented to another state’s modification jurisdiction. State A continues to have jurisdiction, and State
B cannot modify. Section 603 of UIFSA requires that states enforce without modification the child
support orders of other states.

Explanation to Point-Three (35-45%):

A court will reduce a child support award when there has been a material and substantial change in
circumstances; however, such a change in circumstances has not been demonstrated here. Further, a court
cannot order a retroactive modification of a child support order.

In most jurisdictions, modifications of child support orders may be made only upon a showing of a
substantial and continuing change in circumstances making the prior order unreasonable. Under the
Uniform Marriage and Divorce Act (UMDA), modification of a child support order is allowed “only upon
a showing of changed circumstances so substantial and continuing as to make the terms unconscionable.”
However, modifications based on changed circumstances are allowed in most jurisdictions upon the less
stringent showing of a material or substantial change of circumstances, although the burden on the party
requesting the modification is still a heavy one. Under any standard, however, the changes must be more
or less permanent, rather than temporary. Although Father has been unemployed for three months, he did
receive 50 percent of his annual salary in severance pay and other benefits at the time of his termination.
Since his job prospects are good, it is likely that his unemployment will only be temporary. In addition,
the stated reasons for Father’s move to State B provide some evidence of bad faith by Father. Therefore, it
is unlikely that his child support payment will be reduced at this time. If, however, the court were to
reduce Father’s child support obligation, the reduction would be calculated under the state’s child support
guidelines and would not likely be the $1,000 that Father requests. If Father turns out to be unable to find
a job after a lengthy period, the court would be more likely to reduce the amount of the payments.

The court could also impute income to Father, based upon his prior employment history, education, and
efforts (or lack thereof) to find employment.

Moreover, under UMDA, a modification of support can be made retroactive only from the date of service
of the motion to modify on the other party. Here, if the court were able to modify the child support
obligation, the modification could only be made retroactive to the date of service on Mother and cannot be
made six months retroactive. Therefore, Father will owe the full amount of the child support arrearage,
which is $12,000.

Federal law requires the same result with respect to retroactive modification of child support orders.
Section IV-D of the Social Security Act requires a state, as a condition of participation in the federally

Seperac-J19 Exam-Released MEE Essay Compilation © 2016-2020 968


funded child support programs, to have procedures that require that “any payment or installment of
support under any child support order be not subject to retroactive modification by such State or by any
other State.”

Explanation to Point-Four (25-35%):

An award of spousal support is modifiable only when there has been a substantial change in
circumstances.

Modification of spousal support is allowed only upon a showing of a substantial and continuing change in
circumstances making the prior order unreasonable. Under UMDA, a modification of spousal support is
allowed “only upon a showing of changed circumstances so substantial and continuing as to make the
terms unconscionable.” Most jurisdictions are not as stringent as UMDA, but most place a heavy burden
on the party requesting the modification (e.g., requiring a “substantial change in circumstances that
rendered the original award unreasonable and unfair.” Courts consider whether the change in
circumstances was anticipated at the time the original award was made and the good faith of the party
asking for the modification.

A change in the payor’s ability to pay or in the recipient’s needs would be the type of change a court
would consider. Mother now has increased need because her income has been reduced by one-fourth.
Since her workload includes being the custodial parent of the parties’ two children, it seems very
reasonable that she would follow her doctor’s orders not to resume full-time work. Her loss of income
was unanticipated and may be permanent. There is no indication of bad faith on her part. Mother presents
a sympathetic case for an extension and an upward modification of her maintenance award. However,
Father’s circumstances have also changed. His circumstances are discussed under Point Three above.

Overall, it is debatable whether Mother would succeed in having her award increased, particularly in a
jurisdiction that took as strong a stand against modification as reflected in UMDA. Although her income
has been reduced by one-fourth, there is no indication of other, unexpected expenses. Since her original
award was for five years, she will be receiving maintenance for four more years under the original award.
Her age, work experience, and duration of the marriage were factors that would have been considered in
making the original award and would have tended not to support an award of long-term maintenance since
she has been employed, she is not near retirement age, and the marriage was not of extremely long
duration. Therefore, it seems unlikely that the court would order an increase in the duration of the
maintenance award at this time.

On the other hand, it is possible that the spousal support amount should be increased at this time because
of the permanent decrease in her earnings. She seems unable to provide adequate self-support based on
the change in her physical condition. It is probably premature, however, to ask for a change in duration.

Seperac-J19 Exam-Released MEE Essay Compilation © 2016-2020 969


#222-FEB 2001–MEE Q04: QUESTION FOUR (FAMILY LAW)

Soon after Daughter was born to Father and Mother, Father was killed in an automobile accident. Mother
found the economic pressures and time restraints of single parenthood overwhelming, and she began to
drink heavily. Mother’s friend, Caretaker, a childless widow making a comfortable living, began to help
Mother financially and with frequent babysitting.

After a few months, Mother asked Caretaker if she would keep Daughter full time until Mother got her
life back together. Caretaker agreed, and for the next four years Daughter lived with Caretaker. Mother,
who had a serious alcohol problem and could not keep a job, visited Daughter only twice and contributed
no money towards Daughter’s support.

Six months ago, Mother joined Alcoholics Anonymous. Since that time, she has been sober, has found
steady work, and has begun visiting Daughter more often. She wants Daughter, now five years old, to
come back to live with her and insists that she never intended to relinquish Daughter to Caretaker
permanently.

Caretaker, to whom Daughter is closely attached, has refused to return her. She believes that it is in
Daughter’s best interest to remain with her, because she and Daughter have a stable relationship and
Caretaker is the only mother figure Daughter knows. A qualified child psychologist has evaluated
Daughter and her relationships with Caretaker and Mother. The psychologist would testify that: (1)
Daughter is bonded to Caretaker, who has become Daughter’s psychological mother; (2) Daughter
recognizes Mother and is not afraid of her but does not have a child-parent relationship with her; and (3)
if Daughter were separated from Caretaker, she would certainly suffer short-term emotional harm and
might suffer permanent emotional damage.

This jurisdiction’s adoption statutes provide that a child cannot be adopted without the mother’s consent
unless the mother has abandoned the child. Caretaker has filed a petition to adopt Daughter, alleging that
Mother had abandoned Daughter. In the alternative, Caretaker’s petition seeks custody of Daughter.
Finally, the petition asks for visitation rights in the event the requests for adoption and custody are denied.
Mother opposes Caretaker’s petition in all respects.

How should the court rule on Caretaker’s petition? Explain.

Seperac-J19 Exam-Released MEE Essay Compilation © 2016-2020 970


#222: F01-4 MEE: ANSWER: NCBE (FAMILY LAW)

POINT (1) [40%] ISSUE: Under what circumstances will a parent be deemed to have abandoned
her child so the child could be adopted by a third-party custodian without the parent’s consent?
ANSWER: No. In some states, a parent cannot be found to have abandoned her child so long as she
subjectively intends to maintain a relationship with the child; in other states, if a parent has failed
to express substantial interest in her child, she may be found to have abandoned the child,
regardless of her subjective intent.

POINT (2) [40%] ISSUE: In a custody dispute between a parent and a third party with whom the
child has lived for a number of years, to whom is the court likely to award custody? ANSWER: In a
custody contest between a parent and a third party, custody in the parent is presumed to be in the
best interest of the child. To rebut this presumption, the third party must generally prove either
that the parent is unfit or that granting custody to the parent would be highly detrimental to the
child. In some states, however, the court may award custody to the third party upon finding that
doing so would be in the child’s best interest.

POINT (3) [20%] ISSUE: Would a third party who has developed a substantial relationship with a
child be awarded visitation rights over the objections of the child’s parent? ANSWER: No. In most
states, a court cannot award visitation rights to a third party without statutory authority, but in a
minority of states, courts may make such an award if it is in the child’s best interest, at least to
parties who have stood in loco parentis to the child.

ANSWER EXPLANATION:

Explanation to Point-One (35-45%):

In some states, a parent cannot be found to have abandoned her child so long as she subjectively intends
to maintain a relationship with the child; in other states, if a parent has failed to express substantial
interest in her child, she may be found to have abandoned the child, regardless of her subjective intent.

Under the traditional law of adoption, to establish “abandonment” courts required proof that the parent
subjectively intended to abandon the relationship; proof of behavior that objectively suggests a fixed loss
of interest in the child was not sufficient. In a state that uses the subjective test, Mother has not abandoned
Daughter because (1) Mother insists that she never intended to terminate her relationship with Daughter,
and (2) she asked Caretaker to keep Daughter only until Mother “got her life back together.” Accordingly,
Caretaker could not adopt Daughter because Mother has not abandoned Daughter and will not consent.

On the other hand, some states use an objective test for abandonment under which a court might find that
Mother had abandoned Daughter. The inquiry under this test is whether the parent has acted in ways that
indicate a commitment to maintaining the parent-child relationship, and includes factors such as whether
the parent paid support or visited the child. Here, a court might well find that Mother’s failure to pay even
a minimal amount of support and her very infrequent visits constituted abandonment, though her recent
efforts to reestablish her relationship with Daughter cut against this finding. If Mother is found to have
abandoned Daughter, then Daughter would be available for adoption, and Caretaker would probably be
able to adopt her.

Explanation to Point-Two (35-45%):

Seperac-J19 Exam-Released MEE Essay Compilation © 2016-2020 971


In a custody contest between a parent and a third party, custody in the parent is presumed to be in the best
interest of the child. To rebut this presumption, the third party must generally prove either that the parent
is unfit or that granting custody to the parent would be highly detrimental to the child. In some states,
however, the court may award custody to the third party upon finding that doing so would be in the
child’s best interest.

In custody cases between two parents, the standard is the best interest of the child. However, in most
states this standard is not used in parent vs. third party disputes. Instead, there is a strong presumption that
a child should be in the custody of the parents; therefore, Caretaker has the burden of proving that she
should have custody of Daughter. In many jurisdictions, as long as the parent is considered fit, the court
will not even consider whether third-party custody would be better for the child. Therefore, in such a
jurisdiction, Mother would almost certainly receive custody of Daughter. She is currently fit, has
employment, is attending Alcoholics Anonymous (and has been for six months), and has not lost contact
with her child (i.e., no abandonment). In those jurisdictions, the court would not consider Daughter’s
strong attachment to Caretaker to mitigate Mother’s claim.

In other jurisdictions, however, the presumption in favor of parental custody can be overcome by showing
that awarding custody to the parent would be very detrimental to the child. Under this standard it is not
unusual for courts to find that breaking a long-term, stable, parent-child bond between the child and the
third-party caretaker will be harmful to the child, at least when expert testimony to this effect is presented.
In our case, the expert would testify that the strength of Daughter’s attachment to Caretaker means that a
disruption would be harmful to her psychologically. This might be sufficient to overcome the preference
for Mother. Daughter has lived four of her five years with Caretaker (almost her entire life), and has seen
her mother only occasionally. For courts less focused on parental rights, these would be critical facts.

A minority of states use the best interest test in all custody contests including those between parents and
nonparents. Under the best interest test, a court would probably award custody to Caretaker because she
has become Daughter’s psychological mother.

An examinee might receive extra credit for noting that, under Troxel, awarding custody to Caretaker
might raise a question of unconstitutional interference with Mother’s parental rights. Although Troxel
concerned visitation and not custody, the plurality in Troxel held that there is a presumption that fit
parents act in their children’s best interest, and that courts must award “special weight” to a parent’s
decisions. Prior to Troxel, the court rejected the argument that the best interest standard violates parents’
constitutional rights. The court reasoned that the Supreme Court cases on parental rights, all concerned
permanent termination of parental status. A custody ruling, the court said, is less drastic and therefore
constitutionally permissible when supported by good reasons (the child’s best interest). Other courts,
however, might find that the best interest standard unconstitutionally infringes on parental rights if it
denies parental custody in favor of a nonparent in cases where the parent is legally “fit.”

Explanation to Point-Three (15-25%):

In most states, a court cannot award visitation rights to a third party without statutory authority, but in a
minority of states, courts may make such an award if it is in the child’s best interest, at least to parties
who have stood in loco parentis to the child.

In some states, statutes authorize courts to order visitation for a nonparent who has a substantial
relationship with a child if visitation is in the child’s best interest. In the absence of such a statute, courts
generally lack jurisdiction to enter such orders because they are inconsistent with the parents’ custodial
rights, which include determining with whom a child will associate. Some courts, however, have held that

Seperac-J19 Exam-Released MEE Essay Compilation © 2016-2020 972


they have inherent authority to make visitation orders in a child’s best interest. For example, one court
awarded visitation to a former stepparent. Another court adopted the in loco parentis approach in holding
that visitation with a surrogate parent may be in the child’s best interest. Since trial courts are given
exclusive jurisdiction of all matters relating to the guardianship, care, custody, maintenance and education
of the children, this includes the granting of visitation rights to a person or persons who the trial court
determines are significant and important to the welfare of the children.

If a court had the authority to grant nonparental visitation, the facts of this case might support such an
order. Arguably, it would be in Daughter’s best interest to continue visiting Caretaker, rather than having
their relationship abruptly terminated. A court must, however, give “special weight” to Mother’s
opposition. Moreover, the court might conclude that it would be better not to allow visitation if it found
that Daughter would be adversely affected by the animosity between Caretaker and Mother.

[NOTE: Troxel does not affect the outcome of this question, as it is distinguishable from the facts here. In
Troxel, grandparents who never had custody of the grandchildren, sought visitation contrary to the
wishes of the children’s custodial parent. Here, Caretaker had custody of Daughter for most of
Daughter’s life prior to bringing her petition. In these circumstances, the court would most likely find that
granting Caretaker visitation rights would not violate the mother’s constitutional rights.]

Seperac-J19 Exam-Released MEE Essay Compilation © 2016-2020 973


#223-JUL 2000–MEE Q05: QUESTION FIVE (FAMILY LAW)

Fred, age 12, has a heart ailment that severely limits his physical activity. His parents do not allow him to
engage in vigorous sports at school. His doctor predicts that Fred’s condition will slowly worsen and that,
if he is left untreated, his life expectancy will be reduced by 20 years. Medical experts say that a routine
operation can correct his heart defect. However, they have also indicated that it is major surgery, which,
with the required anesthetic, is always risky. The hospital stay will be about five days, and recovery from
the operation will take six weeks. Although the medical experts believe that postponing the operation
until Fred reaches adulthood will result in some permanent loss of function, they also believe that the
operation involves slightly less risk if performed on an adult rather than a child.

Fred wants to have the operation performed. He feels left out and different because most of his friends are
active in sports. Some kids call him “sicky.” The school psychologist has indicated that Fred is withdrawn
and unhappy at school and that he is at risk of failing in his school work because of his emotional state.

Fred’s parents are opposed to the surgery. While they are concerned about the risks of surgery, an even
more compelling reason for their opposition is that their religion does not allow such surgery. They
believe that their prayers for Fred are heard and that his illness must be part of a divine plan. Fred’s doctor
disagrees and wishes to obtain a court order that will allow the operation.

Fred’s parents require him to attend weekly religious services with them. Fred, however, has decided that
he does not wish to follow their religion but instead wants to convert to the faith of his best friend. When
Fred’s parents learn that Fred’s best friend has been teaching Fred about his religion, they forbid him from
attending any religious services other than those of their religion.

1. Can the surgery proceed with Fred’s consent only? Explain.

2. Is a court likely to order the surgery over the objection of Fred’s parents? Explain.

3. Do Fred’s parents have the right to direct which religious services Fred will attend? Explain.

Seperac-J19 Exam-Released MEE Essay Compilation © 2016-2020 974


#223: J00-5 MEE: ANSWER: NCBE (FAMILY LAW)

POINT (1) [32%] ISSUE: Can the child consent to surgery or is parental consent required?
ANSWER: No. Parental consent is required for Fred’s surgery because his situation does not fit
any of the usual exceptions to the general rule that the parent, not the child decides what medical
care the child should receive.

POINT (2) [42%] ISSUE: Can a court order an operation over the objection of the parents, even
when the parents’ objections are based on their religious beliefs? ANSWER: Yes. Although a court
can order medical treatment for the child over the objections of the parents when the medical
treatment is urgently needed to prevent serious harm to the child’s health, even when the parents’
objections are based on their religious beliefs, the court would probably not do so here.

POINT (3) [26%] ISSUE: Must the child obey the parents’ commands about the child’s religious
education? ANSWER: Yes. The right of parents to direct the upbringing of their child includes
control over the child’s religious education, and children have a duty to obey their parents.

ANSWER EXPLANATION:

Explanation to Point-One (25-35%):

Parental consent is required for Fred’s surgery because his situation does not fit any of the usual
exceptions to the general rule that the parent, not the child decides what medical care the child should
receive.

A doctor who performs surgery on a minor child without the parents’ consent is liable in tort. There are
exceptions to this general rule, but none applies here. For example, consent is not necessary in emergency
cases when there is no time to obtain parental consent. And, by statute, most states do not require parental
consent for particular types of medical care, usually related to public health concerns, such as treatment
for venereal disease (although sometimes even these statutes exempt children whose parents object to
treatment on religious grounds). Some states, by statute or case law, make an exception to the parental
consent requirement for older children, either specifying an age or indicating that the minor must be
mature. Typically, however, the “mature minor” must be near the age of majority, and the medical
procedure must not be major. Since none of these exceptions fits these facts, Fred’s consent is not
sufficient and parental consent is necessary.

Explanation to Point-Two (35-45%):

Although a court can order medical treatment for the child over the objections of the parents when the
medical treatment is urgently needed to prevent serious harm to the child’s health, even when the parents’
objections are based on their religious beliefs, the court would probably not do so here.

Under the parens patriae authority of the state, the state can intervene to protect children when parents
deny them needed medical care. The child labor case, Prince v. Massachusetts, is often cited to support
the state’s right to protect children. “Parents may be free to become martyrs themselves. But it does not
follow they are free to make martyrs of their children.” Generally, if a parent fails to provide needed
medical care, a child can be adjudicated neglected and the state can order the medical treatment. In
addition, many states have exemptions to the abuse and neglect statutes that allow the state to order the

Seperac-J19 Exam-Released MEE Essay Compilation © 2016-2020 975


medical care without a finding of parental fault if the parents object to the child’s treatment on religious
grounds.

When parents object to the medical care, however, the concept of “need” is interpreted narrowly. Here, no
strong facts are presented that the care is “needed,” because Fred does not face imminent death or serious
injury. The medical condition is not life threatening and can be postponed until Fred is an adult. In these
medical treatment cases, the courts weigh the risks and benefits of treatment. The strongest facts for
ordering the operation are that Fred will suffer some physical damage and is suffering psychological harm
that is having a negative impact on his school performance. In addition, Fred wants the surgery. On the
other hand, the state has a strong respect for parents’ autonomy, the parents are concerned about the risks
inherent in the surgery, and the parents’ religious beliefs are a major basis for their objection. On balance,
a court would probably not order that Fred’s surgery proceed. Nonetheless, even where the child is not in
imminent danger, some courts will order medical treatment, notwithstanding the parents’ religious
objections.

Explanation to Point-Three (20-30%):

The right of parents to direct the upbringing of their child includes control over the child’s religious
education, and children have a duty to obey their parents.

Parents generally have a right to raise their children as they see fit. A child who fails to follow parental
commands is disobedient, and the state seeks to reinforce the parents’ control over their child, not
undermine it. The state would have authority to intervene only if the parents’ commands were so extreme
as to constitute abuse or neglect. Here, the requirement that Fred attend religious services once per week
and follow his parents’ religion is not likely to rise to that level. Thus, Fred must obey his parents’
instructions with respect to religious services.

Seperac-J19 Exam-Released MEE Essay Compilation © 2016-2020 976


#224-JUL 1999–MEE Q04: QUESTION FOUR (FAMILY LAW)

Alice, a financial consultant, and Ben, an engineer, began living together six years ago in a state that
allows common-law marriage. Alice and Ben were generally known as a couple, and they adopted Smith
as their last name. They never formally married. They do not believe in marriage, a fact that is generally
known in their community.

Alice and Ben earned approximately equal salaries during their relationship. They deposited their
paychecks into a joint bank account from which they paid all their expenses. They never spent all their
salaries, and Alice offered to invest the excess “for our retirement.” Ben, who much preferred to tinker in
his workshop than to read the newspaper’s financial pages, gladly agreed. During the years they lived
together, Alice withdrew $50,000 from their joint bank account and invested it in securities titled solely in
her name. The investments are now worth $200,000.

Ben recently received a patent in his name on a device that he invented and developed with his own labor
while living with Alice. Ben is negotiating with a company interested in manufacturing the device and
believes that rights to the device may eventually be worth several million dollars.

Three months ago, Alice gave birth to a child, Carol. Ben does not deny that he is Carol’s biological
father. Alice admits that she conceived Carol after she stopped using birth control pills without telling
Ben, even though Ben had always made it clear that he did not want children. Ben is so angry at Alice that
he has moved out. Ben believes that he should not have to contribute to Carol’s support because Alice
tricked him into fatherhood.

Ben demands that Alice give him half of the securities that she purchased with their savings. Alice
believes that she is entitled to a share of any money Ben makes from his invention.

1. Is Ben obligated to contribute to Carol’s support? Explain.

2. On what theory or theories might Ben assert a right to a share of the securities and might Alice
assert a right to a share in the profits from the invention, and what is the likely result on each
theory? Explain.

Seperac-J19 Exam-Released MEE Essay Compilation © 2016-2020 977


#224: J99-4 MEE: ANSWER: NCBE (FAMILY LAW)

POINT (1) [9%] ISSUE: Is Ben released from his obligation to support Carol where Alice conceived
Carol after she stopped using birth control without Ben’s agreement? ANSWER: No. A biological
father, whether married or not, is obligated to support his child, regardless of whether he intended
or wanted to father a child.

POINT (2) [9%] ISSUE: In the absence of an explicit agreement to share their assets, under what
circumstances is a cohabitant entitled to a share of assets titled in the other cohabitant’s name?
ANSWER: Alice and Ben do not have a common-law marriage and their property rights depend on
legal theories applicable to unmarried cohabitants. Ben is probably entitled to a share of the
securities, but Alice is probably not entitled to share in Ben’s invention.

POINT (2)(a) [31%] ISSUE: Have Alice and Ben entered into a common-law marriage such that
marital property laws apply to them in light of the facts that they have cohabited and share a
common surname but do not subjectively intend to be married? ANSWER: No. If cohabitants do
not subjectively intend to be married, they have not entered into common-law marriage, even
though they share a common name and are otherwise known as a couple.

POINT (2)(b) [52%] ISSUE: In the absence of a common-law marriage, are Alice and Ben entitled
to share in each other’s assets accumulated during the cohabitation? ANSWER: In some states, a
cohabitant has no right to assets titled in the other’s name absent an express agreement to share
assets. In other states, a cohabitant may have a claim based on implied contract, resulting trust,
constructive trust, or quantum meruit. Ben is probably entitled to a share of the securities, but
Alice is probably not entitled to share in Ben’s invention.

ANSWER EXPLANATION:

Explanation to Point-One (7-10%):

A biological father, whether married or not, is obligated to support his child, regardless of whether he
intended or wanted to father a child.

Parents are legally obligated to support their children, regardless of whether they are marital or nonmarital
children. The facts state that Ben does not deny paternity.

Ben will argue that he cannot be held responsible for child support because he did not agree to Alice’s
becoming pregnant and expressly said that he did not want to have children. Courts have, however,
uniformly rejected this argument. The reason is the overriding public policy in favor of children being
supported, combined with the implicit or explicit conclusion that a man who has sexual relations with a
woman necessarily takes the risk that she may become pregnant.

Explanation to Point-Two (7-10%):

Alice and Ben do not have a common-law marriage and their property rights depend on legal theories
applicable to unmarried cohabitants. Ben is probably entitled to a share of the securities, but Alice is
probably not entitled to share in Ben’s invention.

Seperac-J19 Exam-Released MEE Essay Compilation © 2016-2020 978


The economic rights of Alice and Ben depend in the first instance on whether they are married. If so, the
securities and at least some of the earnings from Ben’s patent would be marital assets subject to equitable
distribution if they divorced. As indicated below, however, Alice and Ben do not have a common-law
marriage, and their rights in the assets are, therefore, determined by other legal principles.

Explanation to Point-Two(a) (25-35%):

If cohabitants do not subjectively intend to be married, they have not entered into common-law marriage,
even though they share a common name and are otherwise known as a couple.

In most states that allow people to enter into common-law marriage, the party claiming that such a
marriage exists must prove at a minimum (1) that the parties cohabited, (2) that they held themselves out
as married, and (3) that they intend to be married. Alice and Ben did cohabit, and it could be argued that
they have represented themselves as married, or at least as a couple, by their use of a common surname
and their having commingled their finances. However, because they expressly do not intend to be married,
they have not entered into a common-law marriage. Often, the emphasis in common-law marriage cases is
placed on the couple’s “holding themselves out” as married, and intent to be married is inferred from that
fact. However, in this case, we are told that the parties did not believe in marriage and that fact was
generally known. Under these circumstances, an inference of intent from “holding out” would be
inappropriate. Thus, principles of divorce property distribution do not apply, and any property rights of
Alice and Ben arise outside of marriage.

Explanation to Point-Two(b) (45-55%):

In some states, a cohabitant has no right to assets titled in the other’s name absent an express agreement to
share assets. In other states, a cohabitant may have a claim based on implied contract, resulting trust,
constructive trust, or quantum meruit. Ben is probably entitled to a share of the securities, but Alice is
probably not entitled to share in Ben’s invention.

In the majority of jurisdictions, if unmarried cohabitants agree to share property or otherwise to engage in
forms of economic sharing, their agreement is enforceable, so long as the economic sharing is not
intended as payment for sexual services. Some jurisdictions continue to refuse to recognize such claims.
For example, one court rejected a claim by an unmarried cohabitant to property because the relationship
was contrary to the state's public policy in support of marriage.

Assuming that this jurisdiction recognizes such claims, it appears that Ben has a claim to the securities,
although Alice may not have a claim for a share of Ben’s invention. The facts do not indicate that the
parties had an express agreement to share the benefits of any of Ben’s inventions. Although Ben might
argue that Alice’s statement that she would invest their savings “for retirement” constituted an express
contract to share the securities purchased with those savings, this argument seems thin. However, in
divorce cases from the era before equitable distribution law, on similar facts courts typically found that
such words were not definite enough to create an explicit contract.

Even without an express contract, however, a court might still consider dividing the property. Some
courts, following the lead of the California Supreme Court in Marvin v. Marvin, also allow unmarried
cohabitants to seek remedies based on implied-in-fact contract, resulting trust, constructive trust, or
quantum meruit theories. Courts’ application of these theories varies substantially. Unless a court applied
these remedial theories very narrowly, Ben could probably make a successful claim to a share of the
securities titled in Alice’s name because they are traceable to his earnings, but Alice is less likely to
prevail in her claim for a share of the profits from Ben’s invention.

Seperac-J19 Exam-Released MEE Essay Compilation © 2016-2020 979


First, on these facts, some courts would say that Ben and Alice’s having commingled their finances was
sufficient to show an implied-in-fact contract to share all of the economic fruits of the relationship, while
others would not.

Second, Ben might claim a resulting trust. To make a claim to property titled in the name of another on a
theory of resulting trust, traditionally a person must have contributed money to the purchase of the asset
under circumstances indicating that the contributor intended to retain equitable ownership of an interest in
the asset, rather than making a gift. Under this theory, Ben might successfully claim that he intended to
retain a beneficial ownership in the securities purchased with his savings. Alice, however, did not
contribute money to the development of Ben’s invention, and so would not have a claim based on this
theory.

Third, Ben might argue constructive trust. A constructive trust is imposed to prevent a party from being
unjustly enriched by profiting from wrongful conduct. On this theory Ben might have a claim for a share
of the securities on the theory that Alice had defrauded him into allowing her to take title to his money.
However, some of the divorce cases cited above rejected such arguments on the basis that the party in
whose name the property was titled never misrepresented what he or she was doing, thus taking a very
narrow view of fraud. Alice, however, would have a difficult time making a claim to the invention under
this theory because the facts do not indicate that Ben made any representations to Alice about the
invention upon which she could rely.

Fourth, there is the possibility that either party might claim recovery based on a quantum meruit claim,
which requires that a person pay the fair market value of services rendered to avoid unjust enrichment.
While the facts are not clear as to whether either performed services for the other that resulted in unjust
enrichment, Alice did take care of the investing, while Ben was able to tinker in his workshop. Alice,
then, performed financial services that gave Ben the leisure time to work on his invention. The claim is,
however, unlikely to succeed.

Finally, if either the investment or invention activities could be proved to be joint economic ventures, then
Alice or Ben might have a claim based on a partnership theory. The facts do not, however, support a
partnership claim.

Seperac-J19 Exam-Released MEE Essay Compilation © 2016-2020 980


#225-FEB 1999–MEE Q05: QUESTION FIVE (FAMILY LAW/CONFLICTS)

Father and Mother, an unmarried couple, lived together in State X. Three years ago, Mother gave birth to
Son. Shortly before Son was born, Father moved out of the house and has never supported or visited Son.
Mother and Father have continued to live in State X. When Son was one year old, Mother decided that
raising him alone was more than she could handle emotionally and financially. Mother placed him with
the Adopters, a married couple from State Z. At that time, she signed a document relinquishing her
parental rights and consenting to Son’s adoption by the Adopters.

The Adopters, who were lifelong residents of State Z, picked up Son in State X and immediately returned
to State Z with him. They promptly filed a petition to adopt Son in a State Z court. They did not give
notice to Father, relying on statutes in State X and State Z that provide that notice of an action to adopt a
child born out of wedlock does not have to be given to a putative father if the putative father (1) had
actual knowledge of the birth or impending birth of the child and (2) failed to acknowledge paternity of
the child, or to take any legal action to establish his claim to paternity of the child, or to exercise parental
rights or duties with regard to the child within the first six months of the child’s life. Eighteen months
after the adoption petition was filed, the State Z court granted it and entered the final adoption decree.

Seven months after the final adoption decree was entered, Father learned that Mother had placed Son for
adoption. In a State Z court, Father moved to set aside the adoption on the grounds that State Z lacked
jurisdiction to grant the adoption or, alternatively, that the statute that allowed Son to be adopted without
notice to Father violates Father’s due process rights.

The adoption statute of limitations in State Z, where the Adopters reside and which granted the adoption,
requires that challenges to adoptions be brought within six months after a decree is entered. The adoption
statute of limitations in State X, where Son was born and where Mother and Father have always lived,
provides that challenges to adoptions may be brought within one year of the time the decree is entered.
Relying on the State Z statute of limitations, the Adopters moved to dismiss Father’s motion as untimely.
Father claims that the State X statute of limitations, which has not run, applies.

1. Which statute of limitations applies to Father’s cause of action? Explain.

2. Did the court in State Z have jurisdiction to enter the adoption decree? Explain.

3. Did failure to give Father notice of the adoption deny him due process? Explain.

Seperac-J19 Exam-Released MEE Essay Compilation © 2016-2020 981


#225: F99-5 MEE: ANSWER: NCBE (FAMILY LAW/CONFLICTS)

POINT (1) [20%] ISSUE: In an interstate adoption action, which state’s statute of limitations
applies? ANSWER: Yes. The statute of limitations in the forum state State Z will apply.

POINT (2) [40%] ISSUE: Does the state which was the domicile of the adoptive parents but not the
biological mother or the child have jurisdiction to enter the adoption decree? ANSWER: Under
traditional principles, State Z has jurisdiction to enter the adoption decree because the adoptive
parents are domiciled there. Under modern statutes, whether State Z has jurisdiction depends on
whether the court finds that the connection between the child and State Z is significant and whether
substantial evidence concerning the case is present in State Z.

POINT (3) [40%] ISSUE: Does it violate due process to allow a child born to unmarried parents to
be adopted without notice to a putative father who has knowledge of the child’s birth but has not
visited the child, paid child support or otherwise manifested willingness to take on the rights and
duties of parenthood? ANSWER: No. The due process clause does not require that notice of an
adoption be given to an unwed father who knew of the pending birth of his child but who neither
acknowledged paternity nor took steps to establish legal paternity or an actual parent-child
relationship with the child.

ANSWER EXPLANATION:

Explanation to Point-One (15-25%):

The statute of limitations in the forum state State Z will apply.

Most states treat statutes of limitations as procedural for choice of law purposes. If State Z uses this
approach, it would ordinarily apply its own statute of limitations. The Second Restatement of Conflict of
Laws continues this approach. It provides that the forum’s shorter statute of limitations applies absent
“exceptional circumstances” that make its application “unreasonable.” No such circumstances appear
here.

The Uniform Conflict of Laws Limitations Act, adopted in six states, characterizes statutes of limitations
as substantive. Under the Uniform Act, when a claim “is substantively based” on the law of a particular
state, that state’s statute of limitations applies. In this case, however, Father’s claim raises primarily
federal constitutional issues and is not substantively based on a particular state’s law. In these
circumstances, the forum applies its own statute of limitations.

Accordingly, whether State Z uses traditional law, the Restatement approach, or the Uniform Act, it will
likely apply its six-month limitation to the collateral attack on the adoption decree.

Explanation to Point-Two (35-45%):

Under traditional principles, State Z has jurisdiction to enter the adoption decree because the adoptive
parents are domiciled there. Under modern statutes, whether State Z has jurisdiction depends on whether
the court finds that the connection between the child and State Z is significant and whether substantial
evidence concerning the case is present in State Z.

Seperac-J19 Exam-Released MEE Essay Compilation © 2016-2020 982


Under traditional common-law principles and the Uniform Adoption Act as it existed before 1994, State Z
would be able to assert jurisdiction because it is the domicile of the adoptive parents.

Under more current jurisdictional principles, expressed in the Uniform Child Custody Jurisdiction Act
and the 1994 Uniform Adoption Act, the essential inquiry is whether State Z has a sufficiently strong
connection with the child and whether substantial evidence concerning the case is present there. Section
4(2) of the 1994 Uniform Adoption Act would almost certainly allow State Z to take jurisdiction because
“immediately before commencement of the proceeding, the prospective adoptive parent lived in this State
for at least six consecutive months, excluding periods of temporary absence, and there is available in this
State substantial evidence concerning the minor’s present or future care.”

However, many states have not adopted the Uniform Adoption Act and use the more general principles of
the Uniform Child Custody Jurisdiction Act to determine jurisdiction for adoption.

The provision of the UCCJA which might allow State Z to take jurisdiction in this case is § 3, which
allows assertion of jurisdiction if the child and at least one contestant (here, the Adoptors) have a
significant connection with the state and substantial evidence concerning the case is present in the state.
Whether this test is satisfied is less clear. It can be argued that State Z does have a substantial connection
because Son and the Adoptors intend to live there permanently and the evidence about Son’s future life
with the Adoptors is available there. However, on facts similar to these some courts have held that the
state in which a child has lived with prospective adoptive parents for a short time does not have
jurisdiction under this provision. The rationale is that the child has spent all his life in the state of his birth
and all the evidence about the validity of his relinquishment for adoption is available there. For example,
one court held that a one-month stay in state with prospective adoptive parents was not long enough to
give “significant connection” jurisdiction. It can also be argued that State Z is the child’s “home state” at
the time the decree of adoption is entered, as the child and person acting as a parent (adoptors) have
resided in State Z for a period in excess of 6 months.

Explanation to Point-Three (35-45%):

The due process clause does not require that notice of an adoption be given to an unwed father who knew
of the pending birth of his child but who neither acknowledged paternity nor took steps to establish legal
paternity or an actual parent-child relationship with the child.

Under the State Z adoption statute, Father was not entitled to notice of the adoption because he moved out
shortly before the child was born, had no contact with the child, and took no legal action to establish
paternity until after he learned of the adoption. As applied in this case, the statute allowing the child to be
adopted without notice to the biological father is probably constitutional.

The United States Supreme Court has held that parental rights are fundamental rights protected by the
Constitution. While the Supreme Court has never decided a case about the substantive showing that must
be made to justify terminating parental rights, it can reasonably be inferred that heightened scrutiny would
be used, since the Court has subjected actions limiting parental care and custody to heightened scrutiny.

The critical question raised here is whether Father, the biological unwed father, has a constitutionally
protected interest. Father will rely on Stanley v. Illinois, and Caban v. Mohammed, to argue that he does.
In Stanley, the Supreme Court held that an unwed father who was denied notice of juvenile court
proceedings to determine the custody of his child had a constitutionally protected interest and that due
process entitled him to notice. In Caban, the Court also found that an unwed father had such an interest
and held that a statute which protected his relationship to his children less than it protected their

Seperac-J19 Exam-Released MEE Essay Compilation © 2016-2020 983


relationship to their mother violated equal protection. However, in Lehr v. Robertson, the Supreme Court
held that a state may allow the adoption of a child born outside of marriage without actual notice to a
known biological father who did not satisfy a limited set of criteria for receiving notice. In Lehr, the father
had not established a relationship with his child and had not registered his claim to paternity with the state
putative father registry. He was not, therefore, entitled to notice under the statute. In upholding the statute
the Court said, “The possibility that he may have failed to file with a putative father registry, which would
have entitled him to notice under the statute because of his ignorance of the law cannot be a sufficient
reason for criticizing the law itself. The New York legislature concluded that a more open-ended notice
requirement would merely complicate the adoption process, threaten the privacy interests of unwed
mothers, create the risk of unnecessary controversy, and impair the desired finality of adoption decrees.”
The Court characterized the issue as whether the statute “adequately protected his opportunity to form
such a relationship.” The court held that it had. Based on this reading of Lehr, the statute here would be
constitutional as applied to Father.

Seperac-J19 Exam-Released MEE Essay Compilation © 2016-2020 984


#226-JUL 1998–MEE Q05: QUESTION FIVE (FAMILY LAW)

Anna and Ben were contemplating getting married. Ben had been married before and had gone through an
unpleasant divorce in which he felt that he had received unfair financial treatment. Ben told Anna he
would not marry her unless she signed a premarital agreement with the following provisions: (1) all
property acquired prior to or during the marriage will be held separately; (2) Anna waives all rights to
alimony; (3) if they divorce or separate, custody of any children will be joint and shared; and (4) in the
event Anna obtains sole custody, Ben will not be responsible for child support.

Ben’s attorney prepared an agreement incorporating these provisions. Ben attached a complete statement
of his financial worth to the agreement and gave it to Anna. He recommended to Anna that she show it to
her attorney before signing it. Anna’s attorney had serious reservations about the document and suggested
she refuse to sign it as drafted. However, Anna, who loved Ben and wanted to marry him, signed it
anyway without making any changes. Ben and Anna married each other three months after she signed the
agreement.

At the time of their marriage, Ben was an investment advisor, with a stock portfolio worth approximately
$ 100,000. Anna was a buyer for a specialty clothing store, making a sizeable salary.

Ben and Anna have now been married for ten years. Anna worked for the first seven years of the
marriage. When the second of their two children was born, she left her job permanently to stay home and
care for the children, who are now six and three years of age.

The marriage has broken down, and both Ben and Anna wish to divorce. Ben’s annual salary is $ 150,000
and his portfolio is worth $500,000, in part due to his annually investing part of his salary during the
marriage. Anna has no income. Her only asset is a $10,000 Treasury note that she inherited from her aunt.

In the divorce proceedings, Anna requests a property settlement, including a share of Ben’s portfolio, as
well as alimony, sole custody of the children, and child support. Ben disagrees and insists that they are
bound by the premarital agreement.

How is the court likely to rule on each of Anna’s requests? Explain.

Seperac-J19 Exam-Released MEE Essay Compilation © 2016-2020 985


#226: J98-5 MEE: ANSWER: NCBE (FAMILY LAW)

POINT (1) [33%] ISSUE: For a premarital contract to be valid, what are the general requirements
and were they met in this case? ANSWER: The premarital contract is valid because it is in writing,
was voluntarily executed, and there was full disclosure.

POINT (2) [33%] ISSUE: Is a court likely to hold the provisions involving property and/or support
unfair? ANSWER: The terms of the contract regarding property and spousal support may be
evaluated by a court in terms of fairness. However, courts are more likely to set aside provisions
affecting support obligations than property settlements. A court may intervene to award Anna
alimony.

POINT (3) [33%] ISSUE: Is the court likely to uphold the provision relating to child custody and
support? ANSWER: No. Provisions in premarital contracts concerning child custody and child
support are probably unenforceable and, therefore, these sections of the contract would not be
binding on a court.

ANSWER EXPLANATION:

Explanation to Point-One (30-40%):

The premarital contract is valid because it is in writing, was voluntarily executed, and there was full
disclosure.

Premarital contracts are generally valid if they meet three conditions: they are in writing, they are
voluntary, and they are executed after full disclosure. The contract between Anna and Ben is in writing.
On the issue of voluntariness, Anna will argue that her consent was not voluntary, as Ben drafted the
agreement and refused to marry her unless she signed it. However, Anna did consult with her own
attorney and refused to follow his advice not to sign it. The wedding was three months later, not the next
day, so she had time to reflect. Under these circumstances, it is unlikely a court would hold that there was
duress or coercion sufficient to invalidate the contract. Ben’s attachment of his complete financial
statement to the draft of the contract before Anna signed it would satisfy any requirement of full
disclosure. Therefore, the premarital agreement is valid. The question does not state that Ben signed the
agreement, nor does it address Anna’s disclosure. However, since it is Anna challenging the agreement,
this is not of particular importance although it may be recognized in some answers.

Explanation to Point-Two (30-40%):

The terms of the contract regarding property and spousal support may be evaluated by a court in terms of
fairness. However, courts are more likely to set aside provisions affecting support obligations than
property settlements. A court may intervene to award Anna alimony.

Anna will argue that the provisions of the contract stating that all property should be held separately and
that she should receive no alimony are fundamentally unfair. Almost all of the property is in Ben’s name,
although, absent the premarital agreement, under the marital property rules of all jurisdictions she would
be entitled to make a claim with respect to the property that was earned after marriage, regardless of title.

Seperac-J19 Exam-Released MEE Essay Compilation © 2016-2020 986


Regarding spousal support, at the present time Anna has no independent source of income, has not
worked for several years, and has two small children. She has very little property as well.

Although Ben will argue that Anna is bound by the terms of the contract, some courts would at least set
aside the provision on spousal support under these circumstances. The support obligation is a traditional
and essential part of marriage and divorce, and Anna apparently has the need (and Ben the ability) since
she has no other means of support.

On the other hand, if the jurisdiction follows the Uniform Premarital Agreement Act (UPAA), then the
property distribution and alimony provisions are probably enforceable. Under the UP AA, a premarital
agreement will be enforced unless (a) it was not executed voluntarily or (b) it was unconscionable when
executed and there was a procedural defect with respect to disclosure of financial information. The
agreement was executed voluntarily (as discussed above), and there is little, if any, evidence of procedural
unfairness here. Therefore, at the time of enforcement, even though the provisions are obviously
disadvantageous to Anna, and may even be unconscionable, they would not be set aside in these
jurisdictions. The UPAA does allow (but does not require) a court to order alimony, notwithstanding a
premarital agreement, if the spouse would otherwise become eligible for public support.

A sub-issue of fairness is one of timing: should the agreement be reviewed for fairness as of the time of
the execution of the contract or as of the time of enforcement? Some courts look to fairness at the time of
execution of the contract; others to fairness at the time of enforcement. Ben would argue for fairness at
the time of execution, when each had some property and Anna was making a good salary. The UPAA
provides that unconscionability should be determined as of the time the agreement was executed.

Anna of course would argue that fairness at the time of enforcement is the only realistic measure. It is ten
years after the contract was signed. Circumstances have changed and Anna has a need for spousal support,
which was not present at the time of execution. She left a well-paying job in order to stay home with the
children. If a court is willing to look at fairness (or unconscionability) as of the time of enforcement, Anna
is in a good position to have the provision set aside. However, in a jurisdiction which has adopted the
UPAA, Anna will be unsuccessful.

Explanation to Point-Three (30-40%):

Provisions in premarital contracts concerning child custody and child support are probably unenforceable
and, therefore, these sections of the contract would not be binding on a court.

It is highly unlikely that provisions in a premarital agreement regarding child custody and child support
would be enforceable. Although the ability to contract with respect to child custody is not entirely clear in
the UPAA, most courts would hold that child custody should be determined by the court according to the
best interests of the child at the time of the divorce or custody hearing, rather than by agreement of the
parties prior to marriage. Therefore, a court could and probably would hold that it is not bound by a
premarital custody provision. The court hearing Anna and Ben’s divorce might decide that they should
have joint custody, but this conclusion would be based on a consideration of the children’s best interest,
not because the contract so provided.

Finally, parents have an absolute obligation to support their children. The UPAA § 3 states that a child’s
right to support cannot be adversely affected by a premarital agreement. Moreover, federal law requires
that each state have child support guidelines which set out a rebuttable presumption with respect to the

Seperac-J19 Exam-Released MEE Essay Compilation © 2016-2020 987


amount of child support that will be ordered. Deviations from the guideline must be justified by the court
through specific findings of fact. The conclusion regarding child support is clearly not enforceable.

Seperac-J19 Exam-Released MEE Essay Compilation © 2016-2020 988


#227-FEB 1998–MEE Q05: QUESTION FIVE (FAMILY LAW)

NOTE: Applicants answering this question in a community property state should use community
property principles. For this purpose, the phrase “marital property” means community property
and the phrase “non marital property” means separate property.

Harry and Wanda, who have been married for 13 years, have two children. Harry and Wanda are now
divorcing. Before marrying, they each earned $25,000 per year as teachers. A year after their wedding,
Harry quit teaching to open a television and stereo repair business called Audio-Video Repair (AVR). He
invested in the business $5,000 that the couple had saved since the wedding and $10,000 that Wanda’s
father had given the couple as a wedding gift. Wanda continued to work as a teacher. The couple
postponed having children and cut back on all expenses. They invested every extra penny in the business.

AVR grew rapidly. Harry decided to incorporate AVR, contributing 52% of the shares to an employee
profit-sharing plan and retaining 48% in his name alone. The contribution of shares to the profit-sharing
plan was for legitimate business reasons.

At a time when Harry was earning over $ 150,000 a year, he and Wanda agreed that she would quit her
teaching job and have children. Sonny was born the next year, and Debbie was born two years later.

Wanda took care of the house and children and entertained Harry’s many business guests. AVR continued
to flourish and began to set up franchises in other cities. Harry increasingly devoted himself to the
business, so that he had very little time for the children and even less time for Wanda. When Debbie was
two years old, Wanda began an affair with a neighbor.

Harry and Wanda have agreed that they will divorce and that the children will live with Wanda. Wanda
plans to return to teaching in another two years, when Debbie reaches school age, but she will need to
update her teaching credentials and can expect to earn only $35,000 per year. Through AVR, including
the franchises, Harry now earns $750,000 a year. AVR is worth $20 million. The parties’ only substantial
asset besides the AVR stock is the family house, worth $300,000. Harry agrees that it should be awarded
to Wanda so that she and the children will have a home.

In the divorce proceeding, Wanda demands a portion of the AVR stock and temporary and permanent
spousal support. Harry resists these demands. Wanda also demands child support measured as a
percentage of Harry’s income. Harry acknowledges his obligation to pay child support but is unwilling to
pay the amount Wanda requests. In this jurisdiction only marital property is subject to division at divorce.
Nonmarital property must be awarded to the owner.

1. Are Harry’s shares in AVR subject to division? Explain.

2. Will Wanda most likely be awarded permanent spousal support, temporary spousal support, or
neither? Explain.

3. What will the court consider in determining the amount of Harry s child support obligation?
Explain.

Seperac-J19 Exam-Released MEE Essay Compilation © 2016-2020 989


#227: F98-5 MEE: ANSWER: NCBE (FAMILY LAW)

POINT (1) [36%] ISSUE: Is a spouse’s interest in a closely held corporation, which the spouse
founded and worked in throughout the marriage and in which marital funds were invested, marital
property subject to division at divorce? ANSWER: Yes. Harry’s closely held corporation stock is
marital property because the stock was acquired with marital funds and through the industry of
both spouses during the marriage.

POINT (2) [36%] ISSUE: Is a spouse who divorces after a medium-length marriage and who can be
self-supporting, but not at the marital standard of living, entitled to spousal support, and, if so, for
how long and at what level? ANSWER: Following a medium-length marriage a court would
probably award spousal support only if the dependent spouse’s share of marital property is
insufficient to generate enough income to meet his or her needs. If the income from the property is
insufficient, a court would probably award spousal support for at least long enough to allow the
dependent spouse to become self-supporting. However, some courts might award a spouse in
Wanda’s position permanent spousal support if necessary to enable her to maintain the marital
standard of living.

POINT (3) [27%] ISSUE: How is the amount of child support determined when the noncustodial
parent’s income is very high? ANSWER: Most courts would not award child support based solely
on a percentage of the parents’ income where that income is very high. Instead, they would cap the
award based on some evaluation of the child’s needs.

ANSWER EXPLANATION:

Explanation to Point-One (35-45%):

Harry’s closely held corporation stock is marital property because the stock was acquired with marital
funds and through the industry of both spouses during the marriage.

Property which spouses acquire during the marriage is presumed to be marital property. The fact that title
to the AVR stock is held in Harry’s name alone does not rebut this presumption, since he determined in
whose name the stock would be held. The remaining facts in this problem support the conclusion that the
stock is marital property. The company was initially financed with a wedding gift, which is marital
property, and with money the spouses earned after the wedding, which is also marital property. The
business grew because of Harry’s efforts and skills, and the product of a person’s efforts during marriage
is ordinarily marital property. Further, the business is the family’s major source of livelihood. Moreover,
Wanda worked during the early years, giving Harry the flexibility to invest so much time and energy in
the business. Wanda’s career sacrifice, contribution as homemaker, and entertainer of business guests
enabled Harry to expend his time building the business.

Even though Harry reorganized the business into a corporation, his shares of stock are traceable to his
original interest in the company, which was marital property, and, therefore, they are marital property as
well. Indeed, Wanda might try to argue that the whole corporation (including the stock held by the
employee profit-sharing plan) is Harry’s, on the theory that it is his alter ego. However, the fact that a
third party owns shares in the corporation cuts against this argument, particularly because AVR was
incorporated and the stocks were contributed to the employee profit-sharing plan for valid business
purposes, and there is no evidence of a fraudulent conveyance.

Seperac-J19 Exam-Released MEE Essay Compilation © 2016-2020 990


Explanation to Point-Two (35-45%):

Following a medium-length marriage a court would probably award spousal support only if the dependent
spouse’s share of marital property is insufficient to generate enough income to meet his or her needs. If
the income from the property is insufficient, a court would probably award spousal support for at least
long enough to allow the dependent spouse to become self-supporting. However, some courts might
award a spouse in Wanda’s position permanent spousal support if necessary to enable her to maintain the
marital standard of living.

Spousal support is awarded on the basis of a number of factors, including the payor’s ability to pay, the
recipient’s need and ability to be self-supporting, and the marital standard of living. The first factor is not
a limit here.

The problem is determining what Wanda’s “needs” are. In some states, a person is not considered to be in
need if he or she has sufficient income to support a modest standard of living, regardless of the marital
standard of living. A court following this approach would probably still award Wanda “rehabilitative
alimony,” that is, temporary support at a sufficient level and for sufficient time to enable her to update her
teaching credentials and re-establish her career. Some courts would also extend the period of temporary
support to enable her to remain at home to care for the children while they are young.

However, in many states spouses are considered to be in need if their income is insufficient to maintain
the marital standard of living, at least in some circumstances. Several factors here would support an award
to enable Wanda to maintain the marital standard of living. She stopped working to care for the family
and to entertain Harry’s business friends. Wanda could argue that she sacrificed her own career to further
Harry’s and the children’s interests and that the court should reward these reliance expenditures. In some
states this marriage would be treated as a long-term marriage, which would strengthen Wanda’s claim to
maintain the marital standard of living.

If Wanda is entitled to spousal support, most courts would first determine whether she was awarded
sufficient property to generate the income she needs. If her share of the property is not sufficient to
generate enough income, she will be awarded spousal support, although in this case a property settlement
of one-half of Harry’s stock would be about $4 million, probably enough to vitiate the need for spousal
support.

In a few states Wanda’s commission of adultery would preclude her from being awarded spousal support;
and in others it would diminish the amount she received. However, fault may not be considered under
UMDA § 308, and under the laws of many states.

Explanation to Point-Three (25-35%):

Most courts would not award child support based solely on a percentage of the parents’ income where that
income is very high. Instead, they would cap the award based on some evaluation of the child’s needs.

Child support is based on the parents’ capacity to pay, the parents’ income, and the needs of the children.
All states now have child support guidelines, as required by federal law. The guidelines must consider the
income of the noncustodial parent and must also provide for the child’s health care needs. In all states the
amount produced by the formula is presumed to be the proper amount of the award, although this
presumption is rebuttable. Deviations from the guideline can be made only upon express findings of fact.

Seperac-J19 Exam-Released MEE Essay Compilation © 2016-2020 991


Where the parents’ combined income is very high, as here, some state guidelines would establish a
presumptive amount that would be far more than any measure of the children’s needs, while other
guidelines would not address this situation at all. In either case, most courts probably would not apply the
guidelines or extrapolate from them in a way that resulted in an extremely high award.

Instead, the primary factor that a court would most likely consider is the children’s needs, generously
defined. Another factor might be the degree of luxury established during the marriage.

Seperac-J19 Exam-Released MEE Essay Compilation © 2016-2020 992


#228-JUL 1997–MEE Q04: QUESTION FOUR (FAMILY LAW)

Al and Bonnie were graduate students at State University. They rented a house together, and each paid
half the rent and utilities. They had separate bedrooms and shared the kitchen, bath, and living room.
They usually prepared and ate their evening meals together. They were good friends but not lovers.

Six months ago, Al, who was a foreign student in the United States on a student visa, lost financial
support for his research and contemplated quitting school temporarily in order to work. However, he
knew that if he quit school he would lose his student visa. He explained his plight to Bonnie and asked if
she would go through a marriage ceremony with him but not otherwise change the way they had been
living. He believed this would entitle him to remain in the country as the spouse of a U.S. citizen even
though he was not in school. Bonnie agreed on condition that they sign an agreement providing that they
would (1) divorce whenever either wanted, (2) be free to come and go and associate with whom they
pleased, and (3) not be responsible for each other’s school and other debts. Al agreed, and both signed the
agreement.

Four months ago, Al and Bonnie obtained a marriage license and participated in a marriage ceremony
before a judge at city hall. Thereafter they continued to live as housemates as they had before.

Two weeks ago, Al was seriously injured in an automobile accident. He was brought, unconscious, to
Hospital’s emergency room, where doctors immediately performed surgery in an attempt to save his life.
However, Al died without ever regaining consciousness. The course of treatment the doctors chose was
appropriate under the circumstances. At some point, Hospital personnel found a card in Al’s wallet that
identified Bonnie as his wife and named her as the person to notify in an emergency. They called her, but
she arrived at Hospital after he died. At Hospital, she openly grieved for him.

Some time later, Hospital’s business office called to ask Bonnie about paying for Al’s care, but Bonnie
replied that she was “not really Al’s wife” and was “not responsible for his debts.” Al did not have
medical insurance. Hospital has billed Bonnie for the costs of Al’s care.

Is Bonnie liable for the payment of Hospital’s bill for Al’s care? Explain.

Seperac-J19 Exam-Released MEE Essay Compilation © 2016-2020 993


#228: J97-4 MEE: ANSWER: NCBE (FAMILY LAW)

POINT (1) [40%] ISSUE: Are parties who agree to participate in a wedding ceremony validly
married for purposes of spousal support duties if they do not intend to assume all the incidents of
marriage? ANSWER: Whether Al and Bonnie will be regarded as married for purposes of spousal
support duties when they entered into the marriage only to give Al the immigration status of a
spouse depends on how the court interprets “consent to be married.”

POINT (2) [40%] ISSUE: Under the common law necessaries doctrine or a family expense statute,
is a wife obligated to pay for her husband’s emergency medical care before the provider has
attempted to secure payment from the husband or his estate? ANSWER: Yes. If Al and Bonnie
were married, Bonnie would probably be liable for Al’s medical expenses under the necessaries
doctrine or a family expense statute.

POINT (3) [20%] ISSUE: Is a premarital agreement between spouses that they will not be
financially responsible for the support of each other enforceable and binding on third parties?
ANSWER: No. Under the majority view, spouses may not contract to eliminate fundamental
marital duties, including support duties during marriage. Further, an agreement to allocate
financial responsibilities between spouses is not binding on third parties.

ANSWER DISCUSSION:

The facts give no basis for finding that Bonnie entered into a contract with Hospital to pay for Al’s care.
She may, however, be liable because of her status as Al’s wife and because of her spousal support duties.
Thus, the first question is whether she and Al will be treated as married for these purposes. If not, Bonnie
is not liable. If they were married, Bonnie may be liable under the common law necessaries doctrine or a
statutory alternative, such as a family expense statute. If Al and Bonnie were married, their agreement
eliminating their duties to support each other may be unenforceable as contrary to public policy. Even if
the agreement would be enforceable as between them, it is not binding on Hospital, which was not a party
to the agreement.

ANSWER EXPLANATION:

Explanation to Point-One (35-45%):

Whether Al and Bonnie will be regarded as married for purposes of spousal support duties when they
entered into the marriage only to give Al the immigration status of a spouse depends on how the court
interprets “consent to be married.”

Consent of the parties is a prerequisite to a valid marriage. Bonnie and Al consented to some aspects of
marriage – participation in the wedding ceremony and representing themselves as spouses to immigration
authorities. As evidenced by their written agreement, however, they did not agree to the other
conventional aspects of marriage, such as emotional and sexual fidelity, economic interdependence, and
commitment to the relationship. The problem, therefore, is the legal effect of a “limited purpose”
marriage.

Seperac-J19 Exam-Released MEE Essay Compilation © 2016-2020 994


While it would be possible to interpret the consent requirement as applying only to participation in the
wedding ceremony, courts do not take this approach. Instead, they consider what policies would be
advanced or impeded by treating parties as married in a particular case.

A court taking the traditional approach would treat Al and Bonnie as married because they participated in
a valid ceremony and sought to obtain at least some of the benefits of marriage. The argument supporting
this approach is that parties should not be able to pick and choose which aspects of marriage they want
because this would undermine the state’s interests in determining the legal consequences of marriage.
Under this approach, Al and Bonnie were married for all purposes.

The other major approach is to conclude that the parties are married only if they intended to take on the
“usual obligations” of marriage. The theory underlying this approach is that no good purpose is served by
treating people as married if they have not taken on the commitments of marriage. Under this approach,
Bonnie would argue that she did not intend to be married in the conventional sense.

Hospital would make several arguments against using the “intent to take on usual marital obligations”
analysis, which would result in a conclusion that Al and Bonnie were not married. First, since intent is
subjective, it may be hard to determine after the fact what the parties’ intent was, though here Al and
Bonnie’s written agreement provides evidence of their intent. Second, determining what the “usual
obligations” of marriage are is a problem. People who consider themselves married do not all live their
lives in the same way, and some courts have suggested that a rule purporting to prescribe how married
people must act might well violate the right to family privacy protected by the due process clause. Third,
even if the court settled on a paradigm of how married people usually behave, the evidence here is
conflicting. Al and Bonnie shared a household, shared expenses to a limited extent, were fond of each
other, and their written agreement indicates they thought they had to divorce to be unmarried. On the
other hand, their economics were not fully integrated, and they did not have sexual relations.

Explanation to Point-Two (35-45%):

If Al and Bonnie were married, Bonnie would probably be liable for Al’s medical expenses under the
necessaries doctrine or a family expense statute.

At common law, husbands were legally responsible for supporting their wives. This duty could be
enforced under the necessaries doctrine. This doctrine allows a wife to purchase goods or services on
credit and charge them to her husband. The creditor who furnishes “necessaries” to a wife may recover
the fair market value of the goods or services from the husband even if he did not enter into a contract to
pay for them or otherwise authorize her to pledge his credit. A “necessary” item is something suitable to
the parties’ station in life. Medically necessary care generally is considered to be within the support duty.
As such, a parent is liable for a child’s medical bills under the necessaries doctrine.

At common law, only husbands had spousal support duties; wives did not. Modern cases and statutes,
however, generally recognize that spouses have mutual support duties.

Those courts which have held that the traditional necessaries doctrine cannot survive in the modern era of
formal gender equality differ about whether a creditor may seek payment for necessaries from either
spouse. In a few states, courts have abolished the necessaries doctrine in response to the equal protection
challenge. In those states, Hospital would not have a common law remedy against Bonnie.

Most states, however, have retained the necessaries doctrine but modified it. In some states a creditor may
sue either spouse or both. In such a state Bonnie would be liable, regardless of the solvency of Al’s estate.

Seperac-J19 Exam-Released MEE Essay Compilation © 2016-2020 995


In other states wives are only secondarily liable for necessaries supplied the other spouse, e.g., Marshfield
Clinic v. Discher, and in some states a creditor must first seek payment from the spouse who incurred the
obligation. In either of the latter two kinds of states, Hospital could not sue Bonnie until its efforts to
collect from Al’s estate proved unsuccessful.

About 20 states have family expense statutes making both spouses liable for the expenses of the family.
Medical care would be such an expense. Under this kind of statute Bonnie would be liable.

Explanation to Point-Three (15-25%):

Under the majority view, spouses may not contract to eliminate fundamental marital duties, including
support duties during marriage. Further, an agreement to allocate financial responsibilities between
spouses is not binding on third parties.

Premarital contracts are covered by the Statute of Frauds and so must be in writing to be enforceable. The
contract between Al and Bonnie was in writing. Modern courts and statutes also require either that the
terms of the agreement not be unconscionable or that the spouses have made adequate disclosures to each
other before entering into the agreement. On this basis a court might find the agreement to be
unenforceable, since there is no indication that Al and Bonnie disclosed their finances to each other. On
the other hand, under the circumstances a court might find the terms of the agreement to be not
unconscionable because the bargaining process was fair, and the relationship of Al and Bonnie was
unconventional.

However, there are limits on spouses’ ability to enter into legally binding agreements to alter their marital
obligations. Under the modern view, the spouses’ ability to define the terms of their relationship is much
greater than in earlier times, but courts frequently still do not enforce agreements to limit the spouses’
support duties during marriage. Therefore, a court might refuse to enforce Al and Bonnie’s agreement that
they would not be liable for each other’s bills.

Even if the agreement were enforceable as between Al and Bonnie, it probably is not binding on Hospital.
The obligation to pay creditors imposed by the necessaries doctrine or family expense statutes is not
contractually based but arises by operation of law, and it cannot be altered by a contract to which the
creditor is not a party.

Seperac-J19 Exam-Released MEE Essay Compilation © 2016-2020 996


#229-FEB 1997–MEE Q02: QUESTION TWO (FAMILY LAW)

Four years ago, Father and Mother began living together in Mother’s house in a state that does not allow
common-law marriage. They never married and they never discussed marriage. Father soon took over
appointment-making and bookkeeping for Mother’s veterinary practice. Mother asked Father to quit
touring with his musical group and to stay home to help her with the practice and care for the home and
their pets. “It would make our lives less hectic, and I’ll share what I’m making with you,” she said. He
agreed. Mother did not pay Father a salary for helping her with her veterinary practice or for his
housekeeping services.

Twenty months ago, Mother gave birth to Daughter, who is Father’s biological child. Mother stayed at
home with Daughter for six weeks and then went back to work, leaving Daughter in Father’s full-time
care.

Ten months ago, Mother told Father that she no longer loved him, and she demanded that he move out.
He did so. Within a few weeks Mother married Stepfather, with whom she and Daughter now live. Father
made no attempt to see Daughter after he moved out of Mother’s house, and he has not paid or offered to
pay child support.

Father has learned that a trial court recently entered a decree allowing Stepfather to adopt Daughter. The
governing statute provides that notice of adoption proceedings must be given to unmarried fathers only if
they have filed a declaration of paternity with the Bureau of Vital Statistics or if, within the six months
preceding filing of the motion, they have lived with the child or offered to support the child. Since Father
satisfied none of these criteria, he did not receive notice of the adoption proceeding.

1. If Father moves to set aside the adoption of Daughter on the ground that failure to give him
notice of the proceeding violates due process, should the court grant the motion? Explain.

2. If the adoption decree is not set aside, may a court order that Father be allowed to visit Daughter
over the objections of Mother and Stepfather? Explain.

3. Upon what theories, if any, may Father be entitled to a share of the property that Mother
acquired between the time she asked him to quit traveling and stay home and the time he moved
out of the house? Explain.

Seperac-J19 Exam-Released MEE Essay Compilation © 2016-2020 997


#229: F97-2 MEE: ANSWER: NCBE (FAMILY LAW)

POINT (1) [40%] ISSUE: Is a statute consistent with the due process clause if it allows a child born
out of marriage to be adopted without the consent of the biological father who at one time lived
with and provided full-time care for the child but who has not filed with a putative father registry
or lived with or supported the child within the six months immediately preceding filing of the
adoption petition? ANSWER: No. An adoption statute is probably unconstitutional if it does not
require actual notice to an unwed father who has lived with and formed a substantial relationship
with his child, even though he did not file with a putative father registry or live with or support the
child during the six months before the adoption petition was filed.

POINT (2) [20%] ISSUE: Does a court have authority to order visitation for a biological parent
after that parent’s rights have been terminated by the child’s adoption by a stepparent? ANSWER:
No. In most states, adoption of a child completely severs the legal relationship between the child and
the biological parent whose status as legal parent is terminated by the adoption. Courts are not
authorized to order visitation for such a biological parent. In a few states, courts have discretion to
order visitation when in the child’s best interests.

POINT (3) [40%] ISSUE: Upon what theories may an unmarried cohabitant be entitled to a share
of property acquired during the period of cohabitation after the relationship has ended? ANSWER:
In most states, an express contract between unmarried cohabitants to share property is enforceable,
though often difficult to prove. Many states also order property sharing between cohabitants upon
proof of an implied-in-fact contract or by means of an equitable remedy to prevent unjust
enrichment.

ANSWER EXPLANATION:

Explanation to Point-One (35-45%):

An adoption statute is probably unconstitutional if it does not require actual notice to an unwed father
who has lived with and formed a substantial relationship with his child, even though he did not file with a
putative father registry or live with or support the child during the six months before the adoption petition
was filed.

The United States Supreme Court has held that parental rights are fundamental rights protected by the
Constitution. While the Supreme Court has never decided a case about the substantive showing that must
be made to justify terminating parental rights, it can reasonably be inferred that heightened scrutiny would
be used, since the Court has subjected actions limiting parental care and custody to heightened scrutiny.

If the court finds that Father is entitled to the constitutional rights of a parent, the statute will almost
certainly be held unconstitutional because it does not require that he receive notice of the adoption
proceedings. The state’s interests in facilitating the adoption of children and in expeditious hearings are
valid, but the means chosen here completely foreclose a parent from being heard on the merits. For
example, in one case, an unwed father who was accorded full parental status was denied procedural due
process when he was not given notice of legal proceedings to determine the custody of his children.

The critical question raised here is whether Father, the biological unwed father, has a constitutionally
protected interest. Father will rely on Stanley and Caban v. Mohammed, to argue that he does. In Caban,

Seperac-J19 Exam-Released MEE Essay Compilation © 2016-2020 998


the Court found that an unwed father had such an interest. It also held that a statute which protected his
relationship to his children less than it protected their relationship to their mother violated the equal
protection clause.

Mother, on the other hand, will rely on Lehr v. Robertson, which upheld, against due process and equal
protection challenges, a statute which allowed a child born outside of marriage to be adopted without
actual notice to a known biological father who did not satisfy a limited set of criteria for receiving notice.
The Court said, “The possibility that he may have failed to file with a putative father registry, which
would have entitled him to notice under the statute because of his ignorance of the law cannot be a
sufficient reason for criticizing the law itself. The New York legislature concluded that a more open-
ended notice requirement would merely complicate the adoption process, threaten the privacy interests of
unwed mothers, create the risk of unnecessary controversy, and impair the desired finality of adoption
decrees.” Based on this reading of Lehr, Mother would argue that the statute here is constitutional as
applied to Father because he could have filed a declaration of paternity and assured himself of receiving
notice of petitions to adopt Daughter.

Father can distinguish Lehr on the basis that the father and child in that case had never lived together and
had never developed any kind of a relationship. The Lehr court specifically said, ”we are not assessing the
procedures for terminating a developed relationship.” In this case, Father had fully assumed the
responsibilities of fatherhood and had developed a substantial relationship with Daughter before Mother
interfered with it. Therefore, he might successfully argue under Stanley and Caban that at a minimum he
was entitled to notice of the adoption proceeding so that he had an opportunity to protect his legal
relationship with Daughter.

Explanation to Point-Two (15-25%):

In most states, adoption of a child completely severs the legal relationship between the child and the
biological parent whose status as legal parent is terminated by the adoption. Courts are not authorized to
order visitation for such a biological parent. In a few states, courts have discretion to order visitation when
in the child’s best interests.

In most jurisdictions, the theory of adoption is that the adoptive parent completely replaces the biological
parent, who becomes legally unrelated to the child. In such states, courts may not order visitation for a
biological parent following adoption because such an order would be inconsistent with the fundamental
nature of adoption. In most states, a court does not have discretion to determine whether visitation would
be in Daughter’s best interests and would refuse to issue a visitation order.

In a few states, statutes authorize courts to order visitation for a nonparent who has a substantial
relationship with a child when visitation is in the child’s best interests. However, these statutes may be
applied narrowly in the factual setting of a contested stepparent adoption. In such states, Father would
have a sufficient relationship with Daughter to support a visitation order, but a court might determine on
the facts that it would be contrary to Daughter’ s best interests to order visitation over Mother’s objection.

Explanation to Point-Three (35-45%):

In most states, an express contract between unmarried cohabitants to share property is enforceable, though
often difficult to prove. Many states also order property sharing between cohabitants upon proof of an
implied-in-fact contract or by means of an equitable remedy to prevent unjust enrichment.

Seperac-J19 Exam-Released MEE Essay Compilation © 2016-2020 999


When unmarried cohabitants break up, their property rights are different from the rights of married people
getting a divorce. Most fundamentally, courts do not have authority to apply the law of equitable
distribution to unmarried couples. Instead, it is presumed that the cohabitant in whose name property was
titled during the relationship will own it after the break-up. If property is jointly titled or untitled, division
by a suit in partition is available for relief. The leading American case on property rights of unmarried
cohabitants, Marvin v. Marvin, holds that the cohabitant who does not have title may be able to make a
claim to ownership of some portion of the property under several theories: express contract, implied-in-
fact contract, and constructive trust, resulting trust or quantum meruit to prevent unjust enrichment.

Today, many courts hold that if cohabitants make an express contract to share property or for support, the
contract is enforceable and not contrary to public policy, provided that the sexual aspects of the
relationship can be severed from the economic aspects. In most states the statute of frauds does not
require that an express contract between cohabitants be written. On the facts here, severing the sexual and
economic aspects of any contract is quite plausible, since Father helped in Mother’s business as well as
running the household. Father would have difficulty, however, proving that he and Mother entered into an
express contract that was sufficiently definite for a court to enforce, since her words were very vague.

Father might have more luck arguing that an implied contract to share property could be inferred from his
and Mother’s conduct. They became economically interdependent at her suggestion. The birth of
Daughter and the attendant division of labor, with Mother working in the marketplace and Father caring
for the child, strengthens this argument.

Father might also argue that a court should impose a constructive or resulting trust on property that
Mother acquired during the cohabitation to prevent her unjust enrichment. He would argue that she
induced him to forego his chance to develop his career in reliance on her promises or led him to believe
that the property she acquired would benefit both of them. Alternatively, he might argue that he is entitled
to the fair market value of the services he provided her as business manager, housekeeper, and nanny on a
theory of quantum meruit. Not all courts hold that these remedies are available to cohabitants because
they think their use comes too close to re-creating common-law marriage. If these theories were available,
Mother might argue that Father has been adequately compensated for his services and work by her
supporting him while they lived together.

Seperac-J19 Exam-Released MEE Essay Compilation © 2016-2020 1000


#230-JUL 1996–MEE Q05: QUESTION FIVE (FAMILY LAW/TRUSTS)

When Husband and Wife divorced in 1990, their final divorce decree provided: (1) Husband would pay
Wife $500 per month for ten years in exchange for Wife's release of any interest she had in Husband's
business; (2) Husband would pay Wife an additional $1,000 per month as spousal support; (3) the couple
would have joint legal custody of their child, Daughter, age 10, who would reside primarily with Wife;
and (4) Husband would pay Wife child support of $400 a month.

Shortly after the divorce, Wife's mother validly transferred title of her apartment building to an
irrevocable trust. The trust instrument names Wife as trustee and directs the trustee to use the trust income
for the benefit of Daughter until she attains age 21. At that time, the trust is to terminate, and the trustee is
to distribute the trust property to Wife. Wife used the trust income to pay for sporting equipment, music
lessons, and recreational activities for Daughter.

In 1991, a fire destroyed the apartment building, which Wife, as trustee, had not insured. Wife heard that
a developer might be interested in purchasing the vacant land in the future, and she decided to buy the
vacant land from the trust for herself at its current fair market value, hoping to make a large profit in
several years. Wife executed and delivered a deed to the land from herself as trustee to herself as an
individual.

As trustee, Wife reinvested all of the proceeds from the sale of the land in XYZ stock. XYZ is not a
publicly traded stock. Although XYZ stock had historically paid very small cash dividends, the
underlying value of the stock seemed likely to increase significantly over time. Wife used the cash
dividends for Daughter's sporting equipment, music lessons, and recreational activities.

Manny, Wife's lover, recently moved into the home of Wife and Daughter. Wife and Manny have a
marriage-like relationship, but they do not intend to marry. Manny pays Wife rent and contributes half of
the cost of groceries and utilities. In all other respects, Manny and Wife keep their finances separate.

1. If Husband properly files a motion to terminate his payments of $500 and $1,000 per month to
Wife and to reduce his child support payment of $400 per month on the basis of the fact that Wife
is living with Manny, should the court grant his motion? Explain.

2. Has Wife breached any duties as trustee of the trust created by her mother? Explain.

Seperac-J19 Exam-Released MEE Essay Compilation © 2016-2020 1001


#230: J96-5 MEE: ANSWER: NCBE (FAMILY LAW/TRUSTS)

POINT (1) [50%] ISSUE: Are the property division, spousal support, or child support payments
subject to termination or modification because of Wife's relationship with Manny? ANSWER: No.
The court may not modify or terminate the property division payments, but it may modify or
terminate Wife's spousal support and the child support if it finds that Wife's cohabitation with
Manny is a sufficient change of circumstances.

POINT (2) [50%] ISSUE: Has Wife breached any fiduciary duties as trustee of her mother's trust?
ANSWER: Yes. Wife, as trustee of her mother's trust, has violated the prudent person rule and has
breached her fiduciary duties of care, loyalty, and impartiality.

ANSWER EXPLANATION:

Explanation to Point-One (45-55%):

The court may not modify or terminate the property division payments, but it may modify or terminate
Wife's spousal support and the child support if it finds that Wife's cohabitation with Manny is a sufficient
change of circumstances.

The term providing that Husband will pay Wife $500 per month for ten years is probably an order for
property division because it is a fixed sum and was awarded to compensate Wife for her interest in
Husband's business. Property division terms are final and are not revocable or modifiable, absent
conditions that would justify reopening a judgment, which are not present in this case.

Spousal support, which is also called spousal maintenance or alimony, is intended to protect a former
spouse who was dependent on the other's earnings or assets to assure that the spouse is able to provide for
his or her reasonable needs after the marriage dissolves. Spousal support is modifiable upon showing of a
substantial change of circumstances. Uniform Marriage and Divorce Act § 316 requires that the changed
circumstances be “so substantial and continuing as to make the terms unconscionable,” although most
states do not require such an extreme change.

Is Wife's unmarried cohabitation a sufficiently substantial change of circumstances to warrant


modification? Absent express terms to the contrary, in most states spousal support terminates when the
recipient remarries because the recipient then has an alternative source of support, the new spouse. The
same reasoning does not apply when the recipient merely cohabits with a new partner, because the
unmarried cohabitant does not have a legal duty to support the partner. Nonetheless, some states have
enacted statutes allowing or requiring the modification of spousal support if the recipient cohabits with a
person of the opposite sex.

Absent a statute, most courts refuse to modify spousal support solely because of the recipient's
cohabitation. Instead they determine whether cohabitation has affected the recipient's economic status. If
the cohabitant contributes to the recipient's financial support, a court may find that the recipient's need for
spousal support is less and should be modified. If the cohabitant does not contribute, but is supported by
the recipient, a court may find that the spousal support payment has become excessive and should be
reduced. Although sometimes the morality of the new relationship seems to affect a modification
decision, the former spouse's objections to the situation should not be relevant. Under this analysis, the
court probably should not terminate the payments because Manny will not be responsible for Wife's

Seperac-J19 Exam-Released MEE Essay Compilation © 2016-2020 1002


continuing support if Wife and Manny's relationship ends, and Husband's spousal support duty could not
be reinstated if it were terminated. However, a court might reduce Husband's spousal support payments to
Wife because Manny is sharing household expenses, reducing Wife's need for spousal support.

Child support is generally modifiable upon a showing of substantial change in circumstances. The focus is
on the needs of the child, balanced against the ability of each parent to pay. Permanent changes in the
financial circumstances of either parent may be grounds for a modification. Actual remarriage of the
custodial parent seldom convinces a court to modify child support, for although the parent may have more
available resources, the stepparent generally has no legal duty to support the child. Thus, Wife's
nonmarital cohabitation alone is unlikely to justify modification of Husband's child support obligation,
since Manny has no duty to support Daughter. In addition, Manny's payment of rent may be considered as
income to Wife.

Manny's financial contributions to the household indirectly provide some support for Daughter, but they
do not appear to be substantial enough to support a finding that Daughter's need for support from Husband
is reduced.

Child support guidelines generally do not require that the income of a parent's unmarried cohabitant be
factored into the formula for setting child support. Some guidelines allow courts to deviate from the
formula amount upon a finding that the cohabitant's income has increased the parent's ability to provide
for the child's support. As previously indicated, though, the facts do not indicate that Manny's financial
contributions are significant enough to warrant a deviation from the formula amount.

Explanation to Point-Two (45-55%):

Wife, as trustee of her mother's trust, has violated the prudent person rule and has breached her fiduciary
duties of care, loyalty, and impartiality.

Wife as trustee is a fiduciary holding legal title to the trust assets, and must manage those assets
exclusively for the benefit of all the trust's beneficiaries. If, as trustee, Wife violates her fiduciary duties,
she may be personally liable to the beneficiaries, and she may be enjoined or removed and replaced with a
court-appointed trustee.

The trustee has a continuing duty to administer the trust in a prudent manner. The trustee has an overall
duty to exercise reasonable care and skill in managing trust property to make it profitable. Consistent with
that duty, a trustee can sell trust assets and purchase other assets to make the trust profitable. The trustee
also has a duty to be impartial with respect to all beneficiaries, whose interests in maximizing income and
preserving or increasing principal are often in conflict. A trustee must act with proper motives and
without partiality or vindictiveness.

Trust law imposes an objective standard of skill and care on the trustee, which the “prudent person”
standard defines as the skill and care of a hypothetical person of ordinary prudence. While the Second
Restatement of Trusts regards the standard as that of a prudent person dealing with his own property,
Uniform Probate Code § 7-302 defines the standard as that which would be observed by a “prudent man
dealing with the property of another.” Emphasis added. Under either version, however, a trustee must not
invest in speculative or risky assets, even though an ordinary person of prudence might otherwise do so.

The trustee has a duty of loyalty and may not obtain personal gain from administering the trust, except for
fees. The duty of loyalty forbids the trustee from placing herself in a position of potential personal profit

Seperac-J19 Exam-Released MEE Essay Compilation © 2016-2020 1003


from the trusteeship, and the trustee must avoid any situation or transaction which could create a conflict
between her personal and fiduciary interests. Self-dealing is a breach per se:

It is a breach of trust for the trustee to sell trust property to himself individually, or as trustee to buy
property from himself individually. It is immaterial that the price is the fair market price and that the
purchase or sale is in all other respects proper and in the best interest of the trust

Lastly, while the trustee is not an insurer of the trust assets, the trustee is liable to the beneficiaries if loss
is related to a breach of fiduciary duties, such as the duty to preserve and protect trust assets. The trustee's
duty of care may be violated by failure to carry casualty insurance on buildings held by the trust if a
prudent person would have obtained insurance. A trustee may also incur liability by profiting personally
at the expense of the trust or by making an unauthorized purchase from the trust even if the trustee did not
fail to exercise care.

Wife's use of the trust income for Daughter's sporting equipment, music lessons, and recreational
activities arguably is not a breach of trust but is in furtherance of the trust because the child alone benefits,
and the trust does not limit the use of the income for her support, education, or the like, nor require that it
be accumulated for her later use.

However, Wife has breached her fiduciary duties as trustee in at least three specific respects: failure to
insure trust property, purchasing vacant lot from trust, and investing all trust assets in XYZ stock. Wife's
failure to insure the apartment building against fire breached her duty of care and her duty to protect trust
property, since a reasonably prudent person would have insured the property.

Wife has also breached her duty as trustee to act with care and prudence to preserve and protect the trust
property. Because the vacant lot would not produce income, Wife probably would have complied with the
prudent person standard had she sold the vacant lot to a third party rather than continuing to hold it as a
trust asset on speculation that it would increase in value. However, Wife's sale of the lot to herself is per
se a breach of the duty of loyalty, as self-dealing is strictly forbidden, even though she paid the fair
market value.

Wife's purchase of the XYZ stock seems to be an impermissibly risky investment and would probably
violate the prudent person standard. The investment also violates the trustee's duty to diversify because
Wife invested all of the trust corpus in XYZ stock. In addition, Wife's investment in XYZ stock, which
favors herself as the remainderman because it minimizes dividends to the income beneficiaries and
maximizes capital appreciation, also violates the trustee's duty to act impartially between the income
beneficiaries and remainderman and may also be a breach of the duty of loyalty.

Seperac-J19 Exam-Released MEE Essay Compilation © 2016-2020 1004


#231-FEB 1996–MEE Q03: QUESTION THREE (FAMILY LAW/CONFLICTS)

Fred and Mary were married for 20 years. Until recently they both lived in the home they owned in State
X. They have two children, a 15-year-old boy and a 2-year-old girl. Eight months ago Fred decided that
he no longer wanted to live with Mary. He moved himself and his business permanently to neighboring
State Y and took with him $100,000 in cash that he had earned during the course of the marriage. Mary
has never been in State Y.

Fred visited the children in State X regularly after he left. He decided, however, that he wanted them to
live with him in State Y. Four months ago, over Mary's objections, he took the children from the marital
home in State X and moved them to State Y. Their son, a mature and thoughtful boy, wishes to return to
State X to live with his mother because he misses his mother, his friends, and his school. Their daughter
seems comfortable with Fred, but she also shows signs of distress that is caused by being separated from
her mother.

In State Y Fred filed for divorce, division of property (i.e., the home in State X and the cash and business
in State Y), and child custody. Mary did not want a divorce and wanted the children returned to her. Both
State X and State Y have adopted the Uniform Child Custody Jurisdiction Act. State Y has a long-arm
statute allowing it to assert personal jurisdiction to the maximum extent permitted under the federal due
process clause. Neither state is a community property state.

1. If State Y grants Fred a divorce, must State X recognize and enforce it? Explain.

2. May State Y constitutionally assert jurisdiction to divide the property of Fred and Mary between
them? Explain.

3. Should State Y assert jurisdiction to decide the custody dispute, and, if it does, must State X
recognize and enforce the decree? Explain.

4. What factors should the court consider in determining to whom custody should be awarded, and
how important is each factor? Explain.

Seperac-J19 Exam-Released MEE Essay Compilation © 2016-2020 1005


#231: F96-3 MEE: ANSWER: NCBE (FAMILY LAW/CONFLICTS)

POINT (1) [10%] ISSUE: If State Y grants Fred a divorce, must State X recognize it? ANSWER:
Yes. Under the full faith and credit clause, states must recognize and enforce a divorce granted by a
court of State Y if State Y was the domicile of Fred.

POINT (2) [30%] ISSUE: May State Y constitutionally assert jurisdiction to divide the property of
Fred and Mary incident to divorce? ANSWER: No. State Y may not assert jurisdiction to divide
Fred and Mary's property that remains in State X because it lacks personal jurisdiction over her.
However, it may assert jurisdiction to divide their personal property located in State Y.

POINT (3) [30%] ISSUE: Does State Y have jurisdiction under the UCCJA to decide custody? If so,
should it exercise it, and, if it exercises it, will State X be obliged to recognized the decree under the
PKPA? ANSWER: Under the UCCJA, State Y might find that it had jurisdiction to decide custody
as a "significant connection state." If it did, it might decline to exercise jurisdiction on "unclean
hands" or "inconvenient forum" grounds. If State Y asserted jurisdiction, its custody decree would
not be entitled to full faith and credit.

POINT (4) [30%] ISSUE: How should the court decide the merits of the custody dispute?
ANSWER: The court should consider multiple factors to determine what custody order will serve
the children's best interests.

ANSWER EXPLANATION:

Explanation to Point-One (05-15%):

Under the full faith and credit clause, states must recognize and enforce a divorce granted by a court of
State Y if State Y was the domicile of Fred.

A person is domiciled in the state in which he or she resides with the intent to remain indefinitely. The
facts say that Fred moved himself and his business permanently to State Y eight months ago, indicating
that he had changed his domicile to State Y. If the divorce is collaterally attacked in another state, the
only issue in that state will be whether Fred was in fact domiciled in State Y. If he was, other states must
give the divorce full faith and credit, provided that Fred gave Mary constitutionally adequate notice of the
divorce proceedings.

Explanation to Point-Two (25-35%):

State Y may not assert jurisdiction to divide Fred and Mary's property that remains in State X because it
lacks personal jurisdiction over her. However, it may assert jurisdiction to divide their personal property
located in State Y.

State Y's long-arm statute permits it to assert personal jurisdiction over Mary to the extent permissible
under the Due Process clause. State Y may constitutionally assert jurisdiction over Mary to terminate her
marriage even though she has never been in or had other contacts with State Y, but it may not necessarily
assert personal jurisdiction over her to determine her property rights as well. This is the concept of
divisible divorce. In Kulko v. Superior Court, of California, the Supreme Court held that due process
requires that a state have "minimum contacts" with a defendant in another state to impose a child support

Seperac-J19 Exam-Released MEE Essay Compilation © 2016-2020 1006


obligation on that person. It can reasonably be inferred that the same test must be satisfied before a state
may assert jurisdiction to divide property not located in that state. Mary has had no contacts with State Y,
and if she does not voluntarily submit herself to State Y's jurisdiction, State Y cannot assert jurisdiction to
determine her rights in property not located in State Y.

It would probably be constitutional for State Y to assert jurisdiction over Mary to determine her interest in
the business and the $100,000 in cash which Fred took to State Y. While Shaffer v. Heitner, held that the
presence of property in a state alone is an insufficient basis for the exercise of general personal
jurisdiction, the Court suggested that a state may constitutionally assert jurisdiction to determine the
ownership of property located in that state on that basis alone. Since the business and the $100,000 in
cash are physically located in State Y, State Y may assert jurisdiction over Mary to determine her interest
in that property.

If Mary goes to State Y to litigate division of the $100,000 and the business, she may subject herself to
the general jurisdiction of State Y to determine her interests in property not located in State Y.

Explanation to Point-Three (25-35%):

Under the UCCJA, State Y might find that it had jurisdiction to decide custody as a "significant
connection state." If it did, it might decline to exercise jurisdiction on "unclean hands" or "inconvenient
forum" grounds. If State Y asserted jurisdiction, its custody decree would not be entitled to full faith and
credit.

Under UCCJA § 3, State Y may have jurisdiction to determine custody on the basis that it has a
“significant connection” with the children and one claimant, Fred, that “substantial evidence concerning
the children's present or future care, protection, training and personal relationships” is available in State
Y, and that it is in the children's “best interests” for State Y to assert jurisdiction.

However, even though the children and their father are present in State Y, it is questionable whether State
Y would find that it has jurisdiction on this basis. Under UCCJA § 3, State X has jurisdiction because it
“had been the children's home state within 6 months before commencement of the proceeding and the
children [are] absent from State X because of the children's removal or retention by a person claiming
their custody and a parent continues to live in State X.”

The UCCJA allows a court to assert jurisdiction in two additional situations: When emergency
circumstances require assertion of jurisdiction to protect the children, and when no other state has
jurisdiction. On the facts here, a State Y court could not assert jurisdiction under either criterion.

If State Y nevertheless found that it had jurisdiction under the significant contacts test, the court might
decline to exercise jurisdiction for one of two reasons, both of which are within the discretion of the trial
court.

First, under UCCJA § 7 a court may decline to exercise jurisdiction if it is an inconvenient forum and the
court in another state is a more appropriate forum. This section lists the factors that go into a
determination of whether State Y is an inconvenient forum. The relevant ones which favor a finding that
State Y is an inconvenient forum are: 1) State X is or recently was the child's home state, 2) State X
arguably has a closer connection with child and family, 3) arguably substantial evidence concerning the
child's present or future care, protection, training, and personal relationships is more readily available in
State X, and 4) State Y's assertion of jurisdiction would contravene the purposes of the UCCJA. State Y's
assertion of jurisdiction is inconsistent with at least two important purposes of the UCCJA: to “assure that

Seperac-J19 Exam-Released MEE Essay Compilation © 2016-2020 1007


litigation concerning the custody of a child take place ordinarily in the state with which the child and his
family have the closest connection “ and to “deter abductions and other unilateral removals of children
undertaken to obtain custody awards.”

Second, State Y might decline to exercise jurisdiction under UCCJA § 8 if it finds that Fred “has
wrongfully taken the child from another state or has engaged in similar reprehensible conduct.”

If State Y asserted jurisdiction, its custody order would not be entitled to full faith and credit under the
Parental Kidnapping Prevention Act, 28 USC § 1738A. The PKPA requires states to enforce custody
orders from other states based on “significant connection” jurisdiction only if no other state would have
jurisdiction as a child's home state at the time the order was issued. For purposes of the PKPA, home state
is defined the same way that it is defined under the UCCJA. Therefore, since State X has home state
jurisdiction, State Y's decree would not be entitled to full faith and credit.

Explanation to Point-Four (25-35%):

The court should consider multiple factors to determine what custody order will serve the children's best
interests.

In all American jurisdictions custody of the children would be determined by their “best interests.” On
these facts one of the most important considerations is the wishes of the children. In some states if a child
has reached sufficient age, the court must award custody consistently with the child's wishes. More
commonly courts consider a child's wishes, and as the age and maturity of the child increases, the wishes
are given greater weight. The court would probably grant custody of the son to his mother, consistent with
his wishes, since he is a mature and thoughtful 15-year-old, there is no indication in the facts given that
his mother has subjected him to inappropriate pressure, and no other factors strongly weigh against giving
custody to his mother.

For the two-year-old daughter other considerations will dominate. Even if the daughter had expressed a
preference, which she apparently has not, courts would probably give little weight to her wishes because
she is only two years old. In some states courts still use the “tender years presumption,” that a young child
should be in the custody of the mother unless she is unfit. Because this expressly gender-based rule is of
doubtful constitutionality, many states have abandoned it. A number of states have a statutory or case law
preference or presumption in favor of custody in a child's primary caretaker, an approach which may have
the same effect as the tender years presumption without being expressly gender-based. On these facts it is
unclear who was the child's primary caretaker before Fred took her from State X to State Y.

Some states traditionally expressed preference for granting custody to the parent of the same gender as the
child, which here would favor awarding the daughter to Mary and the son to Fred. This rule is also of
doubtful constitutionality because it is expressly gender-based. Some empirical evidence supports the
claim that on average, children do better when placed with the parent of the same gender. Therefore, a
court could reach the same result without relying on the express gender-based rule. However, as discussed
above, the son's wishes would probably prevail even if a court adopted this approach.

In some states preserving the stability of a child's living situation is an important factor, which could favor
awarding custody to Fred, who has had custody of the daughter for four months, a relatively longer period
of time from her perspective than from that of an adult. On the other hand, the evidence shows that the
daughter shows signs of distress caused by being separated from her mother, indicating that the daughter's
stability and emotional health might be better served by awarding custody to Mary.

Seperac-J19 Exam-Released MEE Essay Compilation © 2016-2020 1008


A final factor is the preference in some states for not splitting up siblings. Children find stability and
support from being with their siblings, and they should not lose this as well because of divorce. How
important this consideration is here, however, is unclear, since the facts do not discuss the relationship
between the son and the daughter. The age difference between the children may weaken the importance of
this factor. If the daughter is very close to her older brother, though, it would be important.

Seperac-J19 Exam-Released MEE Essay Compilation © 2016-2020 1009


#232-JUL 1995–MEE Q05: QUESTION FIVE (FAMILY LAW)

Susan Smith and George Gordon were validly married in State A, where both had been lifelong residents.
Three years later, they separated but did not divorce.

A few months later, Susan participated in a marriage ceremony with William Wilkins, another longtime
resident of State A. William honestly believed Susan had never been married before. He did not ask any
questions on the subject, and Susan did not volunteer any information about her existing marriage to
George. The wedding announcement stated that both were taking the surname "Smith-Wilkins." Under
that name they maintained joint bank and credit card accounts and rented an apartment together in State
A.

Susan and William rented a vacation cabin in State B each summer for the next four years. They generally
stayed at the cabin for four months at a time.

Four years after Susan and William's wedding ceremony, while they were still living in State A, Susan
received a letter from a friend saying, "Your husband, George, has died from a heart attack." William saw
the letter and learned of George's existence. Susan told William about her marriage to George and said,
"As far as I'm concerned, nothing has changed between us. Let's just put this behind us and go on with
life." William said, "This is pretty hard to take. I'll have to think about it." Although Susan and William
continued to live together and to have sexual relations, their relationship became tense and strained. They
did not discuss Susan's marriage to George again.

During the next year, Susan and William again rented a cabin in State B for four months. During this stay
in State B, Susan and William visited neighbors and bought supplies, as usual. Hiding their marital
difficulties, they appeared to be a happily married couple. However, soon after they returned to State A,
they separated but did not file for divorce.

Seven months after Susan and William separated, she was killed in a traffic accident. Susan was insured
under a group life insurance policy furnished by her employer. The policy provided that if she failed to
name a beneficiary, the proceeds would be paid "First to the insured's surviving spouse; or if none, to the
insured's surviving children, in equal shares; or if none, to the insured's surviving parent(s); or if none, to
the insured's estate." Susan had no children and had not named a beneficiary of the insurance policy.

Susan's parents and William claim the insurance proceeds. Their competing claims are being litigated in
State A. Assume that William has no community property interest in the proceeds of the insurance policy
and that, if he is not entitled to the insurance money, it will be awarded to Susan's parents.

Common law marriages can be validly formed in State B but not in State A.

To whom should the court award the insurance proceeds? Explain.

Seperac-J19 Exam-Released MEE Essay Compilation © 2016-2020 1010


#232: J95-5 MEE: ANSWER: NCBE (FAMILY LAW)

POINT (1) [16%] ISSUE: Did Susan and William have a valid ceremonial marriage in State A?
ANSWER: No. Susan and William did not have a valid ceremonial marriage in State A because
Susan was still married to George at the time of the ceremony, though statutes in some states might
validate their marriage after George died.

POINT (2) [42%] ISSUE: Did Susan and William enter into a common law marriage in State B?
ANSWER: Whether Susan and William entered into a common law marriage in State B depends on
whether the law of State B allows an inference that they agreed to be married after Susan's first
husband died.

POINT (3) [42%] ISSUE: If they did enter into a common law marriage in State B, will State A
recognize that marriage so that William is Susan's surviving spouse and is entitled to the insurance
proceeds? ANSWER: Yes. If a common law marriage was formed in State B, whether it will be
recognized in State A so as to entitle Williams to the insurance proceeds depends on the strength of
State A's policy against common law marriage and how courts in State A will treat the limited
contact between Susan and William and State B.

ANSWER EXPLANATION:

Explanation to Point-One (10-20%):

Susan and William did not have a valid ceremonial marriage in State A because Susan was still married to
George at the time of the ceremony, though statutes in some states might validate their marriage after
George died.

William is entitled to the insurance proceeds on Susan's life if he was her surviving spouse. If not, Susan's
parents should receive them. Thus, the essential question is whether Susan and William were married at
her death.

Although Susan and William participated in a marriage ceremony in State A, Susan was already married
to George at the time. No state recognizes the validity of such a bigamous marriage. Since a bigamous
marriage is void, the ceremony was ineffective to create a valid marriage between Susan and William.

A number of states, however, have statutes providing that if a marriage is invalid because there is an
impediment to it when contracted (for example, one party is still married to another spouse), it becomes a
valid marriage when the impediment is removed. Some statutes require, in addition, that one of the parties
have entered the marriage in the good-faith belief that the marriage was valid. In this case, William
contracted marriage with Susan in the good- faith belief that she was unmarried. Therefore, if State A has
a statute of either of these two types, William and Susan's marriage became valid when George died.

Other statutes, however, require that the parties to the initially invalid marriage continue to cohabit after
the impediment is removed and that one party continue to believe in good faith that they were always
validly married. If that is the type of statute, because William learned of the invalidity of their marriage
when George died, the statute would not validate his marriage to Susan.

Explanation to Point-Two (35-45%):

Seperac-J19 Exam-Released MEE Essay Compilation © 2016-2020 1011


Whether Susan and William entered into a common law marriage in State B depends on whether the law
of State B allows an inference that they agreed to be married after Susan's first husband died.

Even if State A has no law validating the ceremonial marriage after the impediment is removed, it is
possible that Susan and William entered into a common law marriage after George's death. Although
State A does not allow common law marriages to be contracted within its borders, State B does. Because
states allow people who are not domiciliaries to marry within their borders, under State B law Susan and
William may have entered into a common law marriage when they stayed in the State B cabin after
George's death.

The requirements for a common law marriage are: (1) that the parties have a present intent to be married
(as opposed to an intent to become married at some future time); and (2) that they mutually and openly
assume the marriage relationship. The second requirement frequently is stated as two related points: that
the parties (a) cohabit continuously and (b) openly hold themselves out as husband and wife.

The common law marriage doctrine is invoked in many fact settings, including, as here, cases in which
the parties participate in a marriage ceremony and one of them is unaware of an impediment to the
marriage. If they continue to cohabit and hold themselves out as spouses after the impediment is removed,
the courts often will infer that a present agreement to marry was made thereafter. Specific proof of a new
or subsequent agreement will not be required.

At first blush, the facts appear to show that Susan and William entered into a common law marriage in
State B because they cohabited in the cabin after George's death and appeared to hold themselves out as
married. However, evidence of a lack of intent to marry after George's death may negate that inference.
William was troubled by knowledge of Susan's un-dissolved marriage with George, and it contributed to
his decision to separate from her. That fact supports an argument that William did not intend to be
married to Susan after George's death. A conclusion going either way is supportable on the basis of the
facts. (It is also possible that State B has a law that would validate the common law marriage of Susan and
William immediately upon the death of George.

Explanation to Point-Three (35-45%):

If a common law marriage was formed in State B, whether it will be recognized in State A so as to entitle
Williams to the insurance proceeds depends on the strength of State A's policy against common law
marriage and how courts in State A will treat the limited contact between Susan and William and State B.

Even if Susan and William satisfied State B's requirements for entering into a common law marriage,
State A may not recognize it because of the parties' limited contacts with State B. The general rule is that
a marriage which is valid where formed is valid everywhere. Under this simple rule, if Susan and William
would have a valid common law marriage in State B, State A would recognize it.

However, an exception to the general rule is that states will not recognize marriages entered into in other
states if to do so would violate a strong public policy of the forum state. State A's refusal to permit
common law marriages to be formed within its borders expresses a policy disfavoring such marriages. It
is possible, though not likely, that State A would find its policy so strong that it would refuse to recognize
common law marriages in all circumstances.

It is more likely that State A would refuse to recognize that Susan and William had a common law
marriage because of the combined effect of State A's policy and the limited contacts between State B and
Susan and William.

Seperac-J19 Exam-Released MEE Essay Compilation © 2016-2020 1012


As a condition of recognizing common law marriages formed elsewhere, states require differing degrees
of contact with the states permitting such unions:

(1) Some states are satisfied with as little as a brief, transitory visit to the permitting state. Under this
position, State A clearly would recognize a common law marriage between Susan and William because
they frequently stayed in the cabin in State B.

(2) Other states require that the parties have been domiciled in the common law marriage state. Under this
view, State A would not recognize the common law marriage because the parties were at all times
domiciled in State A.

(3) In the middle are states that consider the cases individually, weighing the harm to the claimant that
would result from a refusal to recognize the marriage against the state's policy of discouraging it, and
those to be protected by that policy. The facts do not give enough information for a thorough discussion
under the third position, but a successful analysis would recognize and discuss the problem in terms of a
conflict between upsetting the bona fide expectations of William and respecting State A's interest in
promoting ceremonial marriages and discouraging common law marriages.

Seperac-J19 Exam-Released MEE Essay Compilation © 2016-2020 1013


#233-FEB 1995–MEE Q02: QUESTION TWO (FAMILY LAW)

Soon after Herb and Winnie married, Winnie began working as a cook in a local restaurant to support
Herb, who had started chiropractic school. Winnie and Herb decided that she would become a surrogate
mother to supplement the family income. Herb and Winnie entered into a written agreement with Don, a
married man, whose wife Sally had been unable to bear children. The agreement provided that Winnie
would 1) be artificially inseminated with Don's semen, 2) bear the child of the resulting pregnancy, 3)
relinquish all of her rights to the child, and 4) cooperate in proceedings to terminate her parental rights
immediately after the birth of the child. Don agreed to pay Winnie $10,000 and to be responsible for all
medical expenses related to the pregnancy and childbirth. Herb consented to Winnie's artificial
insemination, agreed not to claim parental rights, and agreed to do whatever was necessary to establish
Don as the child's legal father.

Winnie became pregnant as a result of artificial insemination and delivered a baby, Carol, about the time
Herb graduated from chiropractic school. By the time the baby arrived, however, Winnie had decided that
she would keep Carol. Sally and Don were happy to terminate the contract because Sally had
unexpectedly become pregnant. Sally and Don had already paid Winnie's fee and her medical expenses
and did not want a refund. However, Herb disagreed with Winnie's desire to keep Carol and reminded her
that before they married they had agreed in writing not to have children. Despite Herb's objections,
Winnie decided to keep Carol.

Because of their irreconcilable conflict over Carol, Herb and Winnie have agreed to divorce. Winnie, who
will have custody of Carol, seeks child support from Herb. She also wants to be compensated for helping
put Herb through chiropractic school. Herb and Winnie have virtually no property, and Winnie, who quit
school after high school, lacks the skills to get a good job. Herb has a thriving practice as a chiropractor
ahead of him.

1. Is Herb the legal father of Carol? Explain.

2. Assuming that Herb is the legal father of Carol, is he liable for supporting her? Explain.

3. Should the court order Herb to compensate Winnie for helping him through school, and, if so,
what form should the compensation take? Explain.

Seperac-J19 Exam-Released MEE Essay Compilation © 2016-2020 1014


#233: F95-2 MEE: ANSWER: NCBE (FAMILY LAW)

POINT (1) [40%] ISSUE: Is Herb the legal father of Carol? ANSWER: Yes. Herb may be
regarded as Carol's legal father because a child born to a married woman living with her husband
is presumed to be the child of the marriage, and because a child born to a married woman who is
artificially inseminated with her husband's consent is treated as the legal child of the husband.

POINT (2) [25%] ISSUE: If Herb is Carol's legal father, can he escape liability to support her
because of the surrogacy agreement or because of the premarital agreement that he and Winnie
signed? ANSWER: No. If Herb is Carol's legal father, he is probably liable for child support,
notwithstanding the surrogacy contract and the premarital agreement not to have children.

POINT (3) [35%] ISSUE: Is Winnie entitled to compensation for supporting Herb through
professional school? ANSWER: In most states, Winnie could not get a share of Herb's professional
license because it is not regarded as "property," which can be divided, but her contributions
toward helping Herb through school may be considered in awarding marital property or alimony.

ANSWER EXPLANATION:

Explanation to Point-One (35-45%):

Herb may be regarded as Carol's legal father because a child born to a married woman living with her
husband is presumed to be the child of the marriage, and because a child born to a married woman who is
artificially inseminated with her husband's consent is treated as the legal child of the husband.

Ordinarily a child's biological father is the legal father, but even though Herb is not Carol's biological
father, he might be her legal father under either of two principles.

First, in most states a married woman's husband is at least rebuttably presumed to be the father of his
wife's children. If the presumption is rebuttable, blood test evidence and evidence about the surrogacy
agreement would be sufficient to establish that he is not the biological father and, therefore, not the legal
father unless other principles govern. In a few states a husband is conclusively presumed to be the father
of his wife's children, at least if he is not impotent or sterile and was living with her at the time the child
was conceived. In Michael H. v. Gerald D., the Supreme Court upheld such a presumption against a
constitutional attack by a biological father. If Herb attacked the constitutionality of a conclusive
presumption, he might distinguish Michael H on the basis that in that case the child's mother and her
husband wanted to live as a family with a child, so that the presumption served its historic purpose of
creating a legal relationship between the child and the child's mother's husband. Since that is not the effect
of the presumption here, a court might find that Michael H is not controlling and hold the presumption
unconstitutional.

The second basis upon which Herb might be found to be Carol's legal father is his consent to Winnie's
artificial insemination. By statute or case law in most states, when a married woman is artificially
inseminated with the consent of her husband, he is the child's legal father. For example, the Uniform
Parentage Act provides that: “(a) If, with the consent of her husband in writing and signed, a wife is
inseminated artificially with semen donated by a man not her husband, the husband is treated in law as if
he were the natural father of a child thereby conceived.” The facts say that Herb consented to Winnie's
artificial insemination, and a court might, therefore, apply this principle. However, some courts have held

Seperac-J19 Exam-Released MEE Essay Compilation © 2016-2020 1015


that these provisions about the consequences of artificial insemination do not apply in the surrogacy
situation because the legislature drafted the statute for the circumstance in which the husband cannot
conceive, and would not have intended the statute to be applied to this very different situation.

In response to either of these arguments, Herb might argue that Winnie is estopped to claim that he is
Carol's legal father because he never would have agreed to the plan had he contemplated that he might
become the father. Some courts have invoked estoppel in circumstances analogous to these, while others
have refused. Some courts hold that a party may be estopped to deny the husband's paternity. Other courts
hold that estoppel cannot be invoked.

In a state which has adopted the Uniform Status of Children of Assisted Conception Act, Herb would be
Carol's father because the Act provides that if the mother terminates the surrogacy contract, her husband
is the child's father if he was a party to the agreement. The facts say that Herb was a party to the
agreement with Don."

Explanation to Point-Two (20-30%):

If Herb is Carol's legal father, he is probably liable for child support, notwithstanding the surrogacy
contract and the premarital agreement not to have children.

Ordinarily a child's legal father is obligated to support his child. Herb might argue that he has no duty to
support Carol, relying on either the surrogacy contract or the premarital agreement. However, the general
rule is that parties cannot enter into an enforceable contract to excuse a person from a child support duty.

Herb might argue that the surrogacy agreement contemplated that he would not have any of the legal
rights or duties of fatherhood, since Herb agreed not to claim parental rights and to do whatever was
necessary to establish Don's legal fatherhood. In some states this argument would fail because surrogacy
contracts generally are regarded as illegal or contrary to public policy and, therefore, unenforceable. In
contrast, the California Supreme Court held in Johnson v. Calvert, that surrogacy contracts do not violate
public policy, and it held that the child's legal parents were the “intended” parents, that is, in that case the
biological father and his wife. However, Johnson, like Baby M, involved a dispute between two sets of
parents, both of whom wanted the child. The problem presented here is very different, since neither the
sperm donor father and his wife nor the biological/surrogate mother wants to enforce the surrogacy
agreement. The question does not indicate that the surrogacy agreement dealt explicitly with the situation
which occurred here. Whether a court which followed Johnson would interpret this contract so as to
excuse the surrogate mother's husband's support duty is at best speculative.

Herb might also argue that Winnie had breached their premarital agreement not to have children by
deciding to keep Carol, but a court would probably find that this agreement violates public policy as well
and would, therefore, refuse to enforce it. For example, under the Uniform Premarital Agreement Act, the
right of a child to support “may not be adversely affected by a premarital agreement”)."

Explanation to Point-Three (30-40%):

In most states, Winnie could not get a share of Herb's professional license because it is not regarded as
"property," which can be divided, but her contributions toward helping Herb through school may be
considered in awarding marital property or alimony.

Some courts take a supporting spouse's contributions into account in dividing other property, but that
solution is not available here because there is very little other property. Winnie could argue that Herb's

Seperac-J19 Exam-Released MEE Essay Compilation © 2016-2020 1016


chiropractic degree or license, earned during the marriage, is itself property subject to division. A leading
New York case, O'Brien v. O'Brien, holds that a husband's license to practice medicine earned during the
marriage was property subject to division. For example, under Oregon statute, the present value of, and
income resulting from, the future enhanced earning capacity of either party shall be considered as
property.”) If the court treated the degree as property, it would value the degree, award the degree to
Herb, and order Herb to pay Winnie her equitable share in a lump sum or in a series of payments.

However, most courts have held that a degree is not property because a degree is not transferable and has
none of the other attributes of conventional property. The underlying policy reasons are that property
division awards are not modifiable or terminable, regard less of changed circumstances. If a degree were
assigned a value based on speculations about the future career and income of the degree-holder, and if that
value were then divided, substantial injustice might result if the predictions upon which valuation were
based did not come true.

Still, most courts, using an unjust enrichment theory, hold that a supporting spouse is entitled to
reimbursement for his or her contributions toward supporting the other spouse while he or she earned a
degree. If the supporting spouse is otherwise entitled to permanent or temporary (rehabilitative) alimony,
the alimony award might be partly justified by the spouse's contributions to the acquisition of the payor's
degree. Some courts would award the supporting spouse “reimbursement alimony,” an award for a fixed
sum which is not modifiable or terminable, even if the supporting spouse was not otherwise eligible for
spousal support. "

Seperac-J19 Exam-Released MEE Essay Compilation © 2016-2020 1017


SECURED TRANSACTIONS: 33 OF 49 MEE EXAMS: (67%)
#234-FEB 2019–MEE Q02: QUESTION TWO (UCC ART. 9)

A company is in the business of manufacturing and selling stereo equipment. Several months ago, the
company borrowed money from a bank, to be repaid by the company in monthly installments. The loan
agreement, which was signed by the company's owner, provided that, to secure the company's obligation
to repay the loan, the company granted the bank a security interest in "all personal property" owned by the
company. Also that day, under an oral agreement with the company's owner (who had full authority to
speak on behalf of the company), the bank took possession of one of the most valuable items of the
company's property—an original Edison gramophone that the company had acquired because it was the
earliest precursor of the company's digital music players—as part of the collateral for the loan. The bank
properly filed a financing statement in the appropriate filing office, listing the company as debtor and, in
the space for the indication of collateral, listing only "all personal property."

Since borrowing the money, the company has run into various financial troubles. It has missed some loan
payments to the bank and recently lost a lawsuit, resulting in a large judgment against the company. Last
month, the judgment creditor obtained a judicial lien on the gramophone.

Last week, the bank notified the company that it was in default under the loan agreement. Without giving
advance notice to the company, the bank sold the gramophone to an antiques collector in a commercially
reasonable manner. The judgment creditor has learned about the sale of the gramophone and asserts that
he had a superior claim to it.

The sale of the gramophone did not generate enough money to satisfy the company's obligation to the
bank. The bank would like to seize some of the company's other property in which the bank has an
enforceable security interest.

1. Does the company have any claim against the bank with respect to the sale of the gramophone?
Explain.

2. As between the bank and the judgment creditor, who had a superior claim to the gramophone?
Explain.

3. Does the bank have an enforceable security interest in any personal property of the company
other than the gramophone? Explain.

Seperac-J19 Exam-Released MEE Essay Compilation © 2016-2020 1018


#234: F19-2 MEE: ANSWER: NCBE (UCC ART. 9)

POINT (1) [20%] ISSUE: May a secured party dispose of collateral after the debtor's default
without first notifying the debtor? ANSWER: No. The company has a claim against the bank with
respect to the sale of the gramophone because the bank did not send the company a notification of
disposition before the sale.

POINT (2) [40%] ISSUE: Whose rights are superior as between the rights of a secured party
having possession of an item of collateral and a person who has a judicial lien on the same item?
ANSWER: The bank's security interest in the gramophone is superior to the judgment creditor's
lien because the bank's security interest was perfected before the judgment creditor obtained his
lien.

POINT (3) [40%] ISSUE: Does a security agreement describing collateral as "all personal
property" create an enforceable security interest in a debtor's property? ANSWER: No. The bank
does not have an enforceable security interest in the company's other assets because the description
of the collateral in the loan agreement is insufficient to create an enforceable security interest in
those assets.

ANSWER DISCUSSION:

The company has a claim against the bank with respect to the sale of the gramophone because the bank
did not provide the company with advance notification of the bank's intent to dispose of the gramophone.
The bank's security interest in the gramophone is superior to the judicial lien of the judgment creditor
because the bank's security interest was perfected before the lien was created. The bank does not have an
enforceable security interest in other property of the company because the language in the security
agreement is insufficient and thus the bank's security interest is not enforceable or attached.

ANSWER EXPLANATION:

Explanation to Point One (20%):

The company has a claim against the bank with respect to the sale of the gramophone because the bank
did not send the company a notification of disposition before the sale.

After default by the debtor, a secured party may dispose of the collateral. If it does so, the proceeds of that
disposition will be applied first to the expenses of that process and then to the satisfaction of the debtor's
obligation to the secured party. Before disposing of the collateral, however, the secured party must send
the debtor a reasonable authenticated notification of disposition. The only exception to this notification
requirement is if the collateral is perishable or threatens to decline speedily in value or is of a type
customarily sold on a recognized market. There is no indication that the gramophone in this problem fits
into any of those categories. Therefore, because the bank sent no notification to the company before the
disposition, the disposition was improper. This breach of the bank's obligation may expose the bank to
liability to the company for damages or lessen the amount of any deficiency recoverable by the bank after
application of the proceeds of sale to the company's obligation.

Seperac-J19 Exam-Released MEE Essay Compilation © 2016-2020 1019


[NOTE: An examinee might mention that a private sale of collateral, such as occurred here, is
permissible if commercially reasonable. This is true. But this does not obviate the requirements with
respect to pre-disposition notification.]

Explanation to Point Two (40%):

The bank's security interest in the gramophone is superior to the judgment creditor's lien because the
bank's security interest was perfected before the judgment creditor obtained his lien.

Except as otherwise provided in the UCC, a security agreement is effective against creditors. UCC § 9-
317 provides such an exception, however. That section indicates that a security interest is subordinate to
the rights of a person who became a lien creditor before the security interest was perfected. Here, the
judgment creditor is a lien creditor under § 9-102 because of his judicial lien. Thus, the bank's security
interest in the gramophone will be subordinate to the rights of the judgment creditor only if the bank's
security interest was not perfected when the judgment creditor became a lien creditor.

A security interest is not perfected unless it has "attached." A security interest attaches when it becomes
enforceable unless the time of attachment has been postponed by agreement. Thus, the bank's security
interest in the gramophone cannot be perfected unless it is enforceable. A security interest is enforceable
if the three criteria in UCC § 9-203(b) have been satisfied. The first two criteria are clearly satisfied:
"value" has been given (because the loan was made) and the debtor (the company) had rights in the
gramophone (the company owned it). The third criterion can be satisfied in several ways. Here, it is
satisfied because the collateral is in the possession of the secured party pursuant to the debtor's security
agreement. The gramophone was in the possession of the bank, and this was pursuant to the company's
"security agreement." A "security agreement" is an agreement that creates or provides for a security
interest. Thus, the oral agreement between the company's owner (speaking for the company) and the bank
is a security agreement pursuant to which the bank took possession of the gramophone. Accordingly,
when the bank took possession of the gramophone, its security interest was enforceable and attached.

An attached security interest can be perfected in many ways. In the case of security interests in goods, one
method of perfection is for the secured party to take possession of the goods. Thus, the bank's possession
of the gramophone not only was an element of enforceability and attachment but also resulted in
perfection of the security interest.

Because the bank's security interest in the gramophone was perfected before the judgment creditor
became a lien creditor, the bank's security interest is superior to the judgment creditor's judicial lien.

[NOTE: An examinee may mention that the language in the loan agreement, standing alone, would be
insufficient to create an enforceable and attached security interest in the gramophone. This is accurate,
but the bank's possession of the gramophone pursuant to the oral agreement is sufficient. Thus, the
security interest is enforceable and attached whether or not the language in the written loan agreement
would suffice in the absence of the bank's possession. Some examinees may discuss the elements of
enforceability of a possessory security interest in the answer to question 1 and refer back to that
discussion in their answer to this question. Such examinees should get credit for that analysis as part of
their answer to question 2. Examinees may organize their answers either way and receive full credit.]

Explanation to Point Three (40%):

Seperac-J19 Exam-Released MEE Essay Compilation © 2016-2020 1020


The bank does not have an enforceable security interest in the company's other assets because the
description of the collateral in the loan agreement is insufficient to create an enforceable security interest
in those assets.

A security interest attaches when it becomes enforceable unless the time of attachment has been
postponed by agreement. The three criteria in UCC § 9-203(b) have not been satisfied. The first two
criteria are clearly satisfied: "value" has been given (because the loan was made) and the debtor (the
company) has rights in its personal property (because it owns it). The third criterion has not been satisfied.

Here, because the bank does not have possession of the remaining personal property of the company (and
other specialized criteria are inapplicable), the bank's security interest in that personal property is
enforceable only if the requirements of UCC § 9-203(b) are met. Under that provision, the debtor must
have authenticated (i.e., signed or its electronic equivalent) a security agreement that contains a
description of the collateral. The loan agreement is a security agreement inasmuch as it creates or
provides for a security interest. Moreover, it is authenticated inasmuch as we are told in the question that
the loan agreement was signed by the company's owner. However, the description of the collateral in the
security agreement is insufficient. Subject to some exceptions, a description of personal property is
generally sufficient if it reasonably identifies what is described. However, the UCC provides that
descriptions of a debtor's property such as "all of the debtor's personal property" or words of a similar
import do not reasonably identify the collateral. Here, the description of the collateral in the loan
agreement is "all personal property" owned by the company. Therefore, the description is insufficient, and
accordingly the security agreement does not satisfy the requirements of UCC § 9-203(b).

Thus, the bank's security interest in the company's assets (other than the gramophone in the possession of
the bank discussed in Point One) is not enforceable. Since the security interest is not enforceable, it did
not attach; since the security interest did not attach, the security interest is not perfected.

[NOTE: An examinee may mention that "all personal property" is a sufficient indication of collateral in a
financing statement. That is true. But the filing of a financing statement cannot perfect a security interest
that does not attach. Therefore, the security interest is not perfected here even if the financing statement
would be sufficient to perfect a security interest that, unlike here, attached.]

Seperac-J19 Exam-Released MEE Essay Compilation © 2016-2020 1021


#235-JUL 2017–MEE Q03: QUESTION THREE (UCC ART. 9)

A garment manufacturer sells clothing to retail stores on credit terms pursuant to which the retail stores
have 180 days after delivery of the clothing to pay the purchase price. Not surprisingly, the manufacturer
often has cash-flow problems.

On February 1, the manufacturer entered into a transaction with a finance company pursuant to which the
manufacturer sold to the finance company all of the manufacturer’s outstanding rights to be paid by retail
stores for clothing. The transaction was memorialized in a signed writing that described in detail the
payment rights that were being sold. The finance company paid the manufacturer the agreed price for
these rights that day but did not file a financing statement.

On March 15, the manufacturer borrowed money from a bank. Pursuant to the terms of the loan
agreement, which was signed by both parties, the manufacturer granted the bank a security interest in all
of the manufacturer’s “present and future accounts” to secure the manufacturer’s obligation to repay the
loan. On the same day, the bank filed a properly completed financing statement in the appropriate filing
office. The financing statement listed the manufacturer as debtor and the bank as secured party. The
collateral was indicated as “all of [the manufacturer’s] present and future accounts.”

There are no other filed financing statements that list the manufacturer as debtor.

On May 25, the manufacturer defaulted on its repayment obligation to the bank. Shortly thereafter, the
bank sent signed letters to each of the retail stores to which the manufacturer sold clothing on credit. The
letters instructed each retail store to pay to the bank any amounts that the store owed to the manufacturer
for clothing purchased on credit. The letter explained that the manufacturer had defaulted on its obligation
to the bank and that the bank was exercising its rights as a secured party.

The finance company recently learned about the bank’s actions. The finance company informed the bank
that the finance company had purchased some of the rights to payment being claimed by the bank. The
finance company demanded that the bank cease its efforts to collect on those rights to payment.

Meanwhile, some of the retail stores responded to the bank’s letters by refusing to pay the bank. These
stores contend that they have no obligations to the bank and that payment to the manufacturer will
discharge their payment obligations.

1. As between the bank and the finance company, which (if either) has a superior right to the claims
against the retail stores for the money the retail stores owe the manufacturer for clothing they
bought on credit before February 1? Explain.

2. Are the retail stores correct that they have no obligations to the bank and that paying the
manufacturer will discharge their payment obligations? Explain.

Seperac-J19 Exam-Released MEE Essay Compilation © 2016-2020 1022


#235: J17-3 MEE: ANSWER: NCBE (UCC ART. 9)

POINT (1)(a) [30%] ISSUE: What are the rights of a buyer of accounts (i.e., the finance company)
that has not filed a financing statement? ANSWER: The sale by the manufacturer to the finance
company of the right to be paid by the retail stores for items of clothing is a sale of accounts
governed by Article 9 of the UCC, and the finance company is said to have a “security interest.”
Because the finance company did not file a financing statement, this security interest is unperfected.

POINT (1)(b) [30%] ISSUE: What are the rights of a party (i.e., the bank) to whom a security
interest in accounts has been granted if that secured party has filed a financing statement?
ANSWER: The manufacturer granted to the bank an enforceable and attached security interest in
the right to be paid by the retail stores. Because the bank filed a financing statement with respect to
this security interest, the bank’s security interest is perfected.

POINT (1)(c) [20%] ISSUE: If a buyer of accounts has not filed a financing statement, what are
that buyer’s rights against a party to whom a security interest in those accounts has been
subsequently granted and who has filed a financing statement? ANSWER: The finance company’s
unperfected security interest in the rights to be paid by the retail stores is subordinate to the bank’s
perfected security interest in those rights.

POINT (2) [20%] ISSUE: May an account debtor discharge its obligation on an account that has
been assigned by paying the assignor even after the account debtor learns that the account has been
assigned? ANSWER: Yes. Because the retail stores received authenticated (signed) notifications of
the assignment of their accounts to the bank and directions to pay the bank, they can discharge
their obligations on the accounts only by paying the bank, the assignee.

ANSWER DISCUSSION:

The sale of payment rights from the manufacturer to the finance company is a sale of accounts governed
by Article 9 of the Uniform Commercial Code. Because the finance company did not file a financing
statement, it has an unperfected security interest in the accounts. By contrast, the bank subsequently was
granted a security interest in the same accounts, and that security interest was perfected. Accordingly, the
perfected security interest of the bank has priority over the finance company’s unperfected interest in the
accounts. The retail stores are incorrect that they have no obligations to the bank. Because the retail stores
received an authenticated (signed) notification of the assignment of their accounts to the bank, they can
discharge their obligations only by paying the bank.

ANSWER EXPLANATION:

Explanation to Point-One(a) (30%):

The sale by the manufacturer to the finance company of the right to be paid by the retail stores for items
of clothing is a sale of accounts governed by Article 9 of the UCC, and the finance company is said to
have a “security interest.” Because the finance company did not file a financing statement, this security
interest is unperfected.

The sale by the manufacturer to the finance company of the right to be paid by the retail stores is a sale of
“accounts” as that term is defined in Article 9 of the Uniform Commercial Code. A sale of accounts is

Seperac-J19 Exam-Released MEE Essay Compilation © 2016-2020 1023


governed by UCC Article 9 even though the transaction is not one in which property secures an
obligation. Because Article 9 governs a sale of accounts, Article 9 vocabulary is applied to the sale, so
that the rights of the buyer of the accounts are referred to as a “security interest,” the sold accounts are
“collateral,” the seller (the garment manufacturer in this problem) is the “debtor,” and the buyer (the
finance company in this problem) is the “secured party.” Moreover, because the agreement of sale is “an
agreement that creates or provides for a security interest,” it is a security agreement.

The finance company’s security interest is enforceable and has attached because the three criteria in UCC
§ 9-203 are satisfied: (i) “value” has been given inasmuch as the finance company has paid the purchase
price, (ii) the debtor (the manufacturer) had rights in the collateral (the accounts sold), and (iii) the debtor
“authenticated” (signed or its electronic equivalent, a security agreement containing a description of the
collateral. The finance company’s security interest, however, is not “perfected” because no financing
statement has been filed with respect to the transaction.

Accordingly, the finance company has an unperfected security interest in the rights to be paid by the retail
stores.

Explanation to Point-One(b) (30%):

The manufacturer granted to the bank an enforceable and attached security interest in the right to be paid
by the retail stores. Because the bank filed a financing statement with respect to this security interest, the
bank’s security interest is perfected.

The bank also has an enforceable and attached security interest in the right to be paid by the retail stores
that bought clothing from the manufacturer on credit. First, value has been given because the bank loaned
money to the manufacturer. Second, the debtor (the manufacturer) still had rights in the collateral. This is
the case because even though the manufacturer had already sold the accounts to the finance company, the
finance company’s interest was unperfected. Third, the manufacturer authenticated (signed, in this case) a
security agreement containing a description of the collateral (the accounts).

Unlike the finance company’s security interest, the bank’s security interest is perfected because the bank
filed a properly completed financing statement.

[NOTE: An examinee might mistakenly assert that the bank does not have an enforceable security interest
in the pre-February 1 accounts because one of the three elements of an enforceable and attached security
interest (that the debtor had rights in the collateral) is not satisfied inasmuch as the garment
manufacturer had previously sold those accounts to the finance company. This assertion would be
accurate if the finance company had perfected its interest, but, as indicated in UCC § 9-318, if a buyer of
accounts does not perfect its interest, the seller is treated as continuing to have rights in those accounts.
Thus, in this case, the manufacturer continues to have sufficient rights in the pre-February 1 accounts for
the bank’s security interest to attach.

Nonetheless, an examinee who makes that mistaken assertion should receive some credit for recognizing
that a debtor must have rights in the collateral for a creditor to have an enforceable security interest in
that collateral. Such an examinee should also conclude that, because the bank has obtained no rights in
the pre-February 1 accounts from the garment manufacturer, the retail stores would have no obligation
to pay the bank amounts due on those accounts even after receiving the notifications of assignment from
the bank. An examinee who reaches that conclusion based on the mistaken assertion should receive full
credit if the examinee also correctly notes that the bank has a right to collect from the retailers on the
post-February 1 accounts (which were not sold to the finance company). Conversely, an examinee who

Seperac-J19 Exam-Released MEE Essay Compilation © 2016-2020 1024


asserts, in answer to the first question, that the bank does not have an enforceable security interest but
nonetheless concludes, in answer to the second question, that the retail stores must pay the bank to
discharge their payment obligation on the pre-February 1 accounts, should not receive credit for that
conclusion inasmuch as, under UCC § 9-406, the bank would not be an assignee and its notice to the
retail stores would have no effect.]>

Explanation to Point-One(c) (20%):

The finance company’s unperfected security interest in the rights to be paid by the retail stores is
subordinate to the bank’s perfected security interest in those rights.

An unperfected security interest is subordinate to a perfected security interest in the same property.
Therefore, because the finance company’s security interest in the rights to be paid by the retail stores is
unperfected while the bank’s security interest in those rights is perfected, the finance company’s interest
in those rights is subordinate to the bank’s security interest in them. This is the case even though the sale
of the rights to the finance company occurred before the bank was granted a security interest in them.

Explanation to Point-Two (20%):

Because the retail stores received authenticated (signed) notifications of the assignment of their accounts
to the bank and directions to pay the bank, they can discharge their obligations on the accounts only by
paying the bank, the assignee.

The retail stores are “account debtors” on the accounts. Under UCC § 9-406, an account debtor with
respect to assigned accounts is entitled to discharge its obligation by paying the assignor only until the
account debtor receives notice of the assignment to the assignee, authenticated by either the assignor or
the assignee, directing the account debtor to make payment to the assignee. Once such a notice has been
received, the account debtor is entitled to discharge only by paying the assignee. The retail stores have
received a notice of the assignment authenticated (signed) by the assignee (the bank) directing them to
pay the assignee. Accordingly, under UCC § 9-406, the retail stores can discharge their obligations on the
accounts only by paying the bank (the assignee).

[NOTE: When an account debtor receives notification of an assignment, the account debtor may request
reasonable proof that the assignment has been made and, until such proof is furnished, may still
discharge its obligation by paying the assignor. Under these facts, there is no indication that any of the
retail stores (the account debtors) requested such proof.]

Seperac-J19 Exam-Released MEE Essay Compilation © 2016-2020 1025


#236-JUL 2016–MEE Q04: QUESTION FOUR (UCC ART. 9/REAL PROPERTY)

Two years ago, PT Treatment Inc. (PTT), incorporated in State A, decided to build a new $90 million
proton-therapy cancer treatment center in State A. The total cost to PTT for purchasing the land and
constructing the building to house the treatment facility was $30 million. PTT financed the purchase and
construction with $10 million of its own money and $20 million that it borrowed from Bank. To secure its
obligation to Bank, PTT granted Bank a mortgage on the land and all structures erected on the land. The
mortgage was properly recorded in the county real estate records office, but it was not identified as a
construction mortgage.

Two months after the mortgage was recorded, PTT finalized an agreement for the purchase of proton-
therapy equipment from Ion Medical Systems (Ion) for $60 million. PTT made a down payment of $14
million and signed a purchase agreement promising to pay the remaining $46 million in semi-annual
payments over a 10-year period. The purchase agreement provided that Ion has a security interest in the
proton-therapy equipment to secure PTT’s obligation to pay the remaining purchase price. On the same
day, Ion filed a properly completed financing statement with the office of the Secretary of State of State A
(the central statewide filing office designated by statute), listing “PT Treatment Inc.” as debtor and
indicating the proton-therapy equipment as collateral.

Shortly thereafter, Ion delivered the equipment to PTT and PTT’s employees installed it. The equipment
was attached to the building in such a manner that, under State A law, it is considered a fixture and an
interest in the equipment exists in favor of anyone with an interest in the building.

The new PTT Cancer Treatment Center opened for business last year. Unfortunately, it has not been an
economic success. For a short period, PTT contracted with State A Oncology Associates (Oncology) for
the latter’s use of the proton-therapy equipment pursuant to a lease agreement, but Oncology failed to pay
the agreed fee for the use of the equipment, so PTT terminated that arrangement. To date, PTT has been
unsuccessful in its efforts to collect the amounts that Oncology still owes it. PTT’s own doctors and
technicians have not attracted enough business to fully utilize the cancer treatment center or generate
sufficient billings to meet PTT’s financial obligations. PTT currently owes Ion more than $30 million and
is in default under the security agreement. Ion is concerned that PTT will soon declare bankruptcy.

In a few days, Ion will be sending a technician to the PTT facility to perform regular maintenance on the
equipment. Ion is considering instructing the technician to complete the maintenance and then disable the
equipment so that it cannot be used by PTT until PTT pays what it owes.

1. In view of PTT’s default, if Ion disables the proton-therapy equipment, will it incur any liability
to PTT? Explain.

2. If PTT does not pay its debts to either Bank or Ion, which of them has a superior claim to the
proton-therapy equipment? Explain.

3. Does Ion have an enforceable and perfected security interest in any of PTT’s assets other than
the proton-therapy equipment? Explain.

Seperac-J19 Exam-Released MEE Essay Compilation © 2016-2020 1026


#236: J16-4 MEE: ANSWER: NCBE (UCC ART. 9/REAL PROPERTY)

POINT (1) [40%] ISSUE: When the debtor is in default under a security agreement, does the
secured creditor have the right to disable equipment that is part of the collateral? ANSWER: Yes.
Because PTT defaulted under its security agreement with Ion, Ion is entitled to take possession of
the equipment that is collateral for that obligation or leave the equipment at PTT’s premises and
render it unusable.

POINT (2) [40%] ISSUE: When collateral becomes a fixture, what are the relative rights of a
person who has a security interest in the collateral and a mortgagee of the real property to which
the collateral is affixed? ANSWER: Because the equipment has become a fixture and Ion did not
make an effective “fixture filing,” Bank’s interest in the equipment is superior to that of Ion.

POINT (3) [20%] ISSUE: Does a person who has a security interest in equipment also have a
security interest in the debtor’s right to be paid by a lessee who leased the equipment? If so, is the
security interest perfected? ANSWER: Ion has a security interest in PTT’s rights under the lease
with Oncology because those rights are proceeds of its collateral. Because the security interest of
Ion in the equipment was perfected by filing, its security interest in the rights to payment is
perfected.

ANSWER DISCUSSION:

Ion Medical Systems (Ion) has a security interest in the proton-therapy equipment and may disable that
equipment after PTT’s default, so long as that can be done without a breach of the peace. While Ion has a
security interest in the equipment, Bank also has a claim to that equipment under the mortgage because
the equipment has become a fixture. Bank’s interest is superior to that of Ion because Ion did not make a
fixture filing in the real estate filing office. PTT’s rights against Oncology under the lease of the proton-
therapy equipment constitute proceeds of the equipment and thus are also collateral of Ion. Because Ion’s
security interest in the equipment is perfected, its security interest in the lease proceeds is perfected.

ANSWER EXPLANATION:

Explanation to Point-One (40%):

Because PTT defaulted under its security agreement with Ion, Ion is entitled to take possession of the
equipment that is collateral for that obligation or leave the equipment at PTT’s premises and render it
unusable.

Ion has an enforceable security interest in the proton-therapy equipment under the criteria set forth in
UCC § 9-203(b). The security interest is enforceable because (i) Ion gave value to PTT by providing
credit to PTT, which PTT used to purchase the proton-therapy equipment, (ii) PTT acquired rights in the
proton-therapy equipment when it purchased the equipment from Ion, and (iii) PTT authenticated (i.e.,
signed or its electronic equivalent) a security agreement that described the proton-therapy equipment and
granted Ion a security interest in it.

Following a debtor’s default, if the collateral is “equipment,” a secured party may leave the equipment in
place and render it unusable. The secured party may pursue this option “without judicial process, if it

Seperac-J19 Exam-Released MEE Essay Compilation © 2016-2020 1027


proceeds without breach of the peace.” Although “breach of the peace” is not defined, courts may look to,
inter alia, public harm or loss of public order.

“Equipment” is a defined term under Article 9 of the UCC, referring to “goods other than inventory, farm
products, or consumer goods.” The term “goods” means “all things that are movable when a security
interest attaches” and also includes fixtures. Here, the proton-therapy equipment clearly constitutes goods,
and it is not inventory, farm products, or consumer goods, so it is “equipment.”

Accordingly, because PTT has defaulted, Ion is entitled to disable the equipment if it can do so without
breach of the peace. Here, its technician will be in the facility performing routine maintenance on the
equipment and can disable it at that time. The technician will be lawfully in the facility, and there is
nothing to indicate that the technician would have to engage in any violent, disruptive, or illegal behavior
to prevent the equipment from being used. Without some reason to believe that there will be a breach of
the peace, Ion can lawfully instruct its technician to disable the equipment.

[NOTE: Examinees may note that Ion’s security interest is perfected. While analysis of perfection is
necessary in order to answer Point Three, it is not necessary in order to answer this question.]

Explanation to Point-Two (40%):

Because the equipment has become a fixture and Ion did not make an effective “fixture filing,” Bank’s
interest in the equipment is superior to that of Ion.

Bank has a mortgage on the land and the building in which the proton-therapy equipment is housed. The
facts also state that, as a result of the installation, the equipment became a fixture. Accordingly, Bank’s
mortgage extends to the equipment.

Ion has a perfected security interest in the proton-therapy equipment. The security interest is enforceable
under UCC § 9-203(b), and thus (in the absence of an agreement to the contrary under UCC § 9-203(a))
the security interest is attached. This is because all three elements of UCC § 9-203(b) have been satisfied:
“value” has been given (the proton-therapy equipment has been delivered to PTT), PTT has rights in the
proton-therapy equipment, and the signed purchase agreement qualifies as a security agreement that
contains a description of the collateral. Ion’s security interest is perfected under UCC §§ 9-308 and 9-310
because a financing statement properly listing the debtor and indicating the collateral was filed in the
office of the State A Secretary of State. (State A is the correct state in which to file the financing
statement because perfection of this security interest is governed by the law of the state in which PTT is
located; in this case, PTT is located in State A because, as a corporation, PTT is a “registered
organization” and, thus, is located in the state under whose laws it is organized.)

[NOTE: Inasmuch as State A is the only state mentioned in this problem, examinees who do not do this
conflict-of-laws analysis should nonetheless be able to receive full credit.]

A security interest in fixtures (such as that of Ion), even if perfected, is ordinarily subordinate to a
conflicting interest of an “encumbrancer of the related real property,” such as Bank. There are, however, a
number of exceptions to this rule. One exception relates to the priority of a “purchase-money security
interest” in fixtures as against an encumbrancer of the related real property. (Ion has a purchase-money
security interest in the equipment because the equipment secures credit given by Ion to allow PTT to buy
the equipment.) Because the security interest of Ion is a purchase-money security interest, had Ion made a
“fixture filing” before the equipment became a fixture or within 20 days thereafter, the security interest of
Ion would have had priority over Bank’s mortgage. Ion did not make a fixture filing, though. While Ion

Seperac-J19 Exam-Released MEE Essay Compilation © 2016-2020 1028


did perfect its security interest by filing the financing statement with the Secretary of State, that financing
statement did not qualify as a “fixture filing” because a fixture filing must provide a description of the real
property to which the collateral is related and must be filed in the office in which a mortgage on the
related real estate would be filed, not in the state’s central filing office. Therefore, no exception to the
general rule is applicable and the security interest of Ion in the equipment is subordinate to the interest of
Bank.

Explanation to Point-Three (20%):

Ion has a security interest in PTT’s rights under the lease with Oncology because those rights are proceeds
of its collateral. Because the security interest of Ion in the equipment was perfected by filing, its security
interest in the rights to payment is perfected.

A secured party that has a security interest in collateral also has a security interest in any identifiable
“proceeds” of the collateral. Proceeds include “whatever is acquired upon the lease of collateral.” Thus,
PTT’s rights under the lease with Oncology are proceeds of the proton-therapy equipment in which it has
a security interest. As a result, Ion has a security interest in PTT’s rights against Oncology under the
lease.

Whether Ion’s security interest in the rights under the lease is perfected is important because this will
determine Ion’s rights compared to those of other parties who have a claim to the rights under the lease. A
security interest in proceeds of collateral is perfected for at least 20 days if the security interest in the
original collateral was perfected. Ion’s security interest in the proton-therapy equipment was perfected by
the filing of the financing statement with the Secretary of State. As a result, Ion’s security interest in the
proceeds of that equipment (the rights against Oncology under the lease) is perfected for at least 20 days.
The security interest will remain perfected after the 20-day period if, inter alia, the security interest in the
original collateral was perfected by a filing in the same office in which a security interest in the proceeds
could be perfected by filing. Here, PTT’s rights under the lease with Oncology constitute chattel paper
because the lease evidences both a monetary obligation and a lease of specific goods. A security interest
in chattel paper may be perfected by filing a financing statement in the Secretary of State’s office in State
A, the same office in which PTT filed the financing statement with respect to the equipment. Thus, the
security interest in the chattel paper (the proceeds) is perfected not only for the first 20 days but thereafter
as well.

Seperac-J19 Exam-Released MEE Essay Compilation © 2016-2020 1029


#237-FEB 2016–MEE Q01: QUESTION ONE (UCC ART. 9)

Two years ago, a retailer of home electronic equipment borrowed $5 million from a finance company.
The loan agreement, signed by both parties, provided that the retailer granted the finance company a
security interest in all of the retailer’s present and future inventory to secure the retailer’s obligation to
repay the loan. On the same day that it made the loan, the finance company filed in the appropriate state
filing office a properly completed financing statement reflecting this transaction.

Six months ago, a buyer purchased a home entertainment system from the retailer for a total price of
$7,000. The buyer paid $1,000 as a down payment on the system and agreed to make 12 additional
monthly payments of $500 each. The buyer signed a “credit purchase agreement” memorializing the
financial arrangement with the retailer and providing that the retailer would “retain title” to the
entertainment system until the buyer’s obligation to the retailer was paid in full. The buyer then returned
home with her new home entertainment system. The buyer had no knowledge of the retailer’s agreement
with the finance company and acted in good faith in acquiring the home entertainment system. The
retailer did not file a financing statement with respect to this transaction.

Two months ago, the buyer decided that she could no longer afford her monthly $500 payments for the
home entertainment system. She contacted her friend, who had often expressed interest in acquiring a
home entertainment system. After a brief discussion, the friend agreed to buy the home entertainment
system from the buyer for $4,000 if the friend could pay the price 90 days later, when he anticipated
receiving a bonus at work. The buyer accepted the friend’s proposal, and the friend gave the buyer a
check for $4,000. The buyer promised to hold the $4,000 check for 90 days before depositing it. The
friend took the entertainment system and began using it at his own home. The friend had no knowledge of
the buyer’s agreement with the retailer or of the retailer’s agreement with the finance company.

The retailer is in financial distress and has missed a payment owed to the finance company. Meanwhile,
since the friend bought the home entertainment system from the buyer, the buyer has not made any of her
monthly payments to the retailer.

1. Does the finance company have an interest in the home entertainment system? Explain.

2. Does the retailer have an interest in the home entertainment system? Explain.

3. Does the retailer have an interest in the $4,000 check? Explain.

Seperac-J19 Exam-Released MEE Essay Compilation © 2016-2020 1030


#237: F16-1 MEE: ANSWER: NCBE (UCC ART. 9)

POINT (1) [40%] ISSUE: Does a security interest in a store’s inventory survive sale of an item of
that inventory to a buyer who acquires the item in good faith in an ordinary-course transaction?
ANSWER: No. The buyer obtained the home entertainment system free of the finance company’s
perfected security interest because the buyer was a buyer in ordinary course of business. Because
the buyer owned the home entertainment system free of the security interest, the buyer’s sale to the
friend was free of that interest.

POINT (2) [40%] ISSUE: What are the rights of a seller of goods who “sells” those goods on credit
to a consumer but purports to retain title to the goods until the full purchase price is paid? Do the
seller’s rights survive a resale of those goods by the buyer to another consumer? ANSWER: The
retailer’s retention of title to the home entertainment system in the credit sale to the buyer was a
security interest in the system. While the security interest was automatically perfected, the friend
acquired the system free of the retailer’s security interest.

POINT (3) [20%] ISSUE: Does a secured party obtain an interest in the proceeds of a debtor’s sale
of the secured party’s collateral? ANSWER: Yes. Because the retailer had a perfected security
interest in the home entertainment system at the time of its sale to the friend, the retailer obtained a
security interest in the $4,000 check as proceeds from the sale of the system; the security interest in
the check was perfected for 20 days and remained perfected thereafter because the check
constituted “cash proceeds.”

ANSWER DISCUSSION:

The finance company had a perfected security interest in the home entertainment system when it was
owned by the retailer. The buyer acquired the home entertainment system from the retailer free of that
security interest because the buyer was a buyer in ordinary course of business. Because the buyer owned
the home entertainment system free of the security interest, the buyer sold it to her friend free of the
finance company’s interest. The retailer had a perfected security interest in the home entertainment
system while it was owned by the buyer. The friend acquired the home entertainment system from the
buyer free of that security interest because both the buyer and the friend used the home entertainment
system for personal purposes, the friend acquired it without knowledge of the retailer’s security interest,
and the retailer had not filed a financing statement with respect to its security interest. Because the retailer
had a security interest in the home entertainment system when it was owned by the buyer, the retailer
obtained a security interest in the $4,000 check as proceeds of the home entertainment system. The
retailer’s security interest in the check was perfected because its security interest in the home
entertainment system had been perfected and the check constituted “cash proceeds.”

ANSWER EXPLANATION:

Explanation to Point-One (40%):

The buyer obtained the home entertainment system free of the finance company’s perfected security
interest because the buyer was a buyer in ordinary course of business. Because the buyer owned the home
entertainment system free of the security interest, the buyer’s sale to the friend was free of that interest.

Seperac-J19 Exam-Released MEE Essay Compilation © 2016-2020 1031


The finance company had an enforceable, attached security interest in all of the retailer’s inventory,
including the home entertainment system later sold to the buyer. All three elements of enforceability and
attachment under UCC § 9-203 were satisfied: value was given (the loan), the debtor (the retailer) had
rights in the inventory, and the debtor (the retailer) authenticated a security agreement containing a
description of the collateral. Moreover, the finance company’s security interest was perfected by the filing
of the financing statement.

As a general rule, a security interest continues notwithstanding the sale of collateral. But this general rule
is subject to several exceptions. In particular, a “buyer in ordinary course of business” (BIOCOB) takes
free of a security interest created by its seller even if the security interest was perfected. Under these facts,
the buyer was a BIOCOB. She appears to have bought the home entertainment system in good faith and in
the ordinary course from a person in the business of selling goods of the kind (the retailer) without
knowledge that the sale to her violated the rights of a third party. Because the buyer was a BIOCOB, she
acquired the home entertainment system free of the finance company’s security interest.

Under the “shelter principle,” once the buyer acquired the home entertainment system free of the finance
company’s security interest, any subsequent transfer of the system by the buyer to someone else was also
free of the finance company’s security interest. Accordingly, when the friend acquired the home
entertainment system from the buyer, it was free of the finance company’s security interest.

Explanation to Point-Two (40%):

The retailer’s retention of title to the home entertainment system in the credit sale to the buyer was a
security interest in the system. While the security interest was automatically perfected, the friend acquired
the system free of the retailer’s security interest.

While the agreement between the retailer and the buyer indicated that the retailer was retaining title to the
home entertainment system, the substance-over-form rules of secured transactions law indicate that the
retailer’s interest was not ownership but, rather, a security interest. Article 9 governs any transaction
regardless of form that creates a security interest in personal property by contract. All the elements of
attachment and enforceability of this security interest were satisfied.

The retailer’s security interest was perfected even though the retailer did not file a financing statement.
This is because the retailer’s security interest was a “purchase-money security interest” in “consumer
goods.” The security interest was a purchase-money security interest because it was retained by the seller
to secure payment of the remainder of the sales price. The home entertainment system constituted
“consumer goods” because it was used primarily for personal, family, or household purposes. A purchase-
money security interest in consumer goods perfects upon attachment without the filing of a financing
statement. Therefore, the retailer’s security interest in the home entertainment system was a perfected
security interest while the system was owned by the buyer.

As a general rule, a security interest continues notwithstanding the sale of collateral. Thus, under this
general rule, the retailer’s security interest in the home entertainment system would continue after the
buyer’s sale of the system to the friend. But this general rule is subject to several exceptions. In this case,
the friend acquired the system free of the retailer’s security interest under the consumer-to-consumer
exception in UCC § 9-320 because the home entertainment system was used for personal, family, or
household purposes of the buyer before the sale and of the friend after the sale, and the friend acquired the
system for value, without knowledge of the retailer’s security interest, and before a financing statement
was filed with respect to that security interest. (As noted above, the retailer was not required to file a

Seperac-J19 Exam-Released MEE Essay Compilation © 2016-2020 1032


financing statement to perfect its security interest because it was a purchase-money security interest in
consumer goods.)

Explanation to Point-Three (20%):

Because the retailer had a perfected security interest in the home entertainment system at the time of its
sale to the friend, the retailer obtained a security interest in the $4,000 check as proceeds from the sale of
the system; the security interest in the check was perfected for 20 days and remained perfected thereafter
because the check constituted “cash proceeds.”

The check constituted “proceeds” of the home entertainment system because that term includes, inter alia,
whatever is obtained on the sale of collateral. When collateral is disposed of, a secured party
automatically obtains a security interest in identifiable proceeds of the collateral. Accordingly, when the
home entertainment system was sold by the buyer to the friend, and the buyer received the check in
return, the check constituted proceeds of the home entertainment system and the retailer (which had a
security interest in the system) thereby obtained a security interest in the check. Because the retailer’s
security interest in the system had been perfected, its security interest in the check as proceeds was
perfected as well. That perfection ceases after 20 days, however, unless one of three criteria in UCC § 9-
315(d) is satisfied. In this case, UCC § 9-315 was satisfied because the check was “cash proceeds.” Thus,
the retailer’s security interest in the check was continuously perfected after the sale of the system to the
friend.

Seperac-J19 Exam-Released MEE Essay Compilation © 2016-2020 1033


#238-FEB 2015–MEE Q03: QUESTION THREE (UCC ART. 9)

Acme Violins LLC (Acme) is in the business of buying, restoring, and selling rare violins. Acme
frequently sells violins for prices well in excess of $100,000. In addition to restoring violins for resale,
Acme also repairs and restores violins for their owners. In most repair transactions, Acme requires
payment in cash when the violin is picked up by the customer. It does, however, allow some of its repeat
customers to obtain repairs on credit, with full payment due 30 days after completion of the repair. In
those cases, the payment obligation is not secured by any collateral and the payment terms are
handwritten on the receipt.

Acme maintains a stock of rare and valuable wood that it uses in violin restoration. Acme also owns a
variety of tools used in restoration work, including a machine called a “Gambretti plane,” which is used to
shape the body of a violin precisely.

Six months ago, Acme borrowed $1 million from Bank. The loan agreement, which was signed by Acme,
grants Bank a security interest in all of Acme’s “inventory and accounts, as those terms are defined in the
Uniform Commercial Code.” On the same day, Bank filed a properly completed financing statement in
the appropriate state filing office. The financing statement indicated the collateral as “inventory” and
“accounts.”

Last week, Acme sold the most valuable violin in its inventory, the famed “Red Rosa,” to a violinist for
$200,000 (the appraised value of the instrument), which the violinist paid in cash. The sale was made by
Acme in accordance with its usual practices. The violinist, who has done business with Acme for many
years, was aware that Acme regularly borrows money from Bank and that Bank had a security interest in
Acme’s entire inventory. The violinist did not, however, know anything about the terms of Acme’s
agreement with Bank.

Acme is 15 days late in making the payment currently due on its loan from Bank. Bank’s loan officer,
who is worried about Bank’s possible inability to collect the debt owed by Acme, has asked whether the
following items of property are collateral that can be reached by Bank as possible sources of payment:

(1) Acme’s rights to payment from customers for repair services obtained on credit

(2) Used violins for sale in Acme’s store

(3) Violins in Acme’s possession that Acme is repairing for their owners

(4) Wood in Acme’s repair room that Acme uses in repairing violins

(5) The Gambretti plane, used by Acme in violin restoration

(6) The Red Rosa violin that was sold to the violinist

Yesterday, a creditor of Acme obtained a judicial lien on all of Acme’s personal property.

1. In which, if any, of the items listed above does Bank have an enforceable security interest?
Explain.

Seperac-J19 Exam-Released MEE Essay Compilation © 2016-2020 1034


2. For the items in which Bank has an enforceable security interest, is Bank’s claim superior to that
of the judicial lien creditor? Explain.

Seperac-J19 Exam-Released MEE Essay Compilation © 2016-2020 1035


#238: F15-3 MEE: ANSWER: NCBE (UCC ART. 9)

POINT (1) [15%] ISSUE: Does Bank have an enforceable and attached security interest in the
collateral described in the loan agreement? ANSWER: Yes. Bank has an enforceable and attached
security interest in the property described in the loan and security agreement.

POINT (2)(a) [10%] ISSUE: Does Bank’s security interest cover Acme’s rights to payment from
customers who obtained repair services from Acme on credit? ANSWER: Yes. Acme’s rights to
payment from customers who obtained repair services on credit are “accounts” subject to Bank’s
security interest.

POINT (2)(b) [5%] ISSUE: Does Bank’s security interest cover the used violins that are for sale in
Acme’s store? ANSWER: Yes. The used violins that are for sale in Acme’s store are “inventory”
subject to Bank’s security interest.

POINT (2)(c) [10%] ISSUE: Does Bank’s security interest cover the violins in Acme’s possession
that it is repairing for others? ANSWER: No. The violins in Acme’s possession that it is repairing
for others are not covered by Bank’s security interest because they are not “inventory” and are not
owned by Acme.

POINT (2)(d) [5%] ISSUE: Does Bank’s security interest cover the wood used by Acme in repairing
violins? ANSWER: Yes. The wood used by Acme in repairing violins is “inventory” and subject to
Bank’s security interest.

POINT (2)(e) [5%] ISSUE: Does Bank’s security interest cover the Gambretti plane? ANSWER:
No. The Gambretti plane is not included in Bank’s collateral.

POINT (3) [25%] ISSUE: Does Bank’s security interest cover the Red Rosa violin after its sale to
the violinist? ANSWER: No. The Red Rosa violin was subject to Bank’s security interest prior to its
sale, but the violinist was probably a buyer in ordinary course of business and therefore took the
violin free of Bank’s security interest under UCC § 9-320.

POINT (4) [25%] ISSUE: Is Bank’s security interest superior to the interest of a judicial lien
creditor in the same property? ANSWER: Yes. Bank’s security interest in its collateral was
perfected; therefore, the security interest is superior to the rights of a judicial lien creditor that
arose subsequently.

ANSWER DISCUSSION:

Bank has an enforceable and attached security interest in the property described in the loan and security
agreement – Acme’s “inventory and accounts.” The amounts owed to Acme by customers for violins
repaired on credit are “accounts” covered by Bank’s security interest. The used violins for sale in Acme’s
store are “inventory” covered by Bank’s security interest. The violins that Acme is repairing for others are
not subject to the security interest because they are not held by Acme for sale or lease and thus are not
“inventory.” The wood used in repairing violins is “inventory” subject to Bank’s security interest. The
Gambretti plane is equipment, rather than inventory or accounts, and not covered by Bank’s security
interest. Although the Red Rosa violin was inventory subject to Bank’s security interest, Bank’s interest
was cut off by the sale of Red Rosa to a buyer in ordinary course of business. Because Bank’s enforceable

Seperac-J19 Exam-Released MEE Essay Compilation © 2016-2020 1036


and attached security interest was perfected, it is superior to the rights of a judicial lien creditor that arose
subsequent to perfection.

ANSWER EXPLANATION:

Explanation to Point-One (15%):

Bank has an enforceable and attached security interest in the property described in the loan and security
agreement.

A security agreement is enforceable and attached if the three criteria described in UCC § 9-203 are
satisfied. The first criterion is that “value has been given.” This is satisfied by the loan from Bank to
Acme. The second criterion is that the debtor “has rights in the collateral or the power to transfer rights in
the collateral to a secured party.” This is satisfied with respect to Acme’s inventory and accounts. The
third criterion is satisfied because Acme has authenticated (signed, in this case) a security agreement that
provides a description of the collateral. Thus, since all three criteria have been satisfied, Bank’s security
interest in the property described in the loan and security agreement is enforceable and attached.

Explanation to Point-Two(a) (10%):

Acme’s rights to payment from customers who obtained repair services on credit are “accounts” subject to
Bank’s security interest.

Bank’s security interest covers “accounts,” as the term is defined in the Uniform Commercial Code.
Acme’s rights to payment for repair services provided on credit are accounts because they are “rights to
payment for services rendered or to be rendered.” Thus, these rights to payment are subject to Bank’s
security interest.

Explanation to Point-Two(b) (5%):

The used violins that are for sale in Acme’s store are “inventory” subject to Bank’s security interest.

Bank’s security interest covers “inventory.” The used violins that are for sale in Acme’s store are
“inventory” under UCC § 9-102 because they are “goods which are held by a person for sale.” Thus, those
violins are subject to Bank’s security interest.

Explanation to Point-Two(c) (10%):

The violins in Acme’s possession that it is repairing for others are not covered by Bank’s security interest
because they are not “inventory” and are not owned by Acme.

Bank’s security interest does not cover the violins that Acme is repairing for their owners. This
conclusion can be reached in either of two ways. First, those violins are not “inventory” because they are
not held by Acme for sale or lease. Second, because the violins are not owned by Acme, but rather are
owned by the individuals who brought them to Acme to be repaired, the security interest would not attach
because Acme has neither rights in those violins nor the power to transfer rights in them to a secured
party.

[NOTE: An examinee might argue that Acme has some rights in the violins it holds for repair because it
has lawful possession of them, the right to repair them, the right to collect payment for repairs, and (in

Seperac-J19 Exam-Released MEE Essay Compilation © 2016-2020 1037


some jurisdictions) the right to assert an artisan’s lien if a repair bill is unpaid. An examinee who makes
an argument of this sort should also note that these limited rights will not help Bank for two reasons.
First, as already noted, the violins don’t fall within the scope of the security interest in any event, because
they are not “inventory” or “accounts.” Second, even if the violins held for repair were within the scope
of the security agreement, Bank’s rights against the owners of the violins would be no greater than
Acme’s and it could, at best, arrange for repairs to be performed and collect amounts due. Moreover,
while it is possible that, because Acme deals in violins and the owners of the violins brought to Acme for
repair entrusted them to Acme, buyers in ordinary course of business of those violins would get good title
to them under UCC § 2-403, that provision does not give rights to secured parties.]

Explanation to Point-Two(d) (5%):

The wood used by Acme in repairing violins is “inventory” and subject to Bank’s security interest.

Bank’s security interest in inventory covers the wood that Acme uses to repair violins. “Inventory”
includes “raw materials” that are consumed in the business of the debtor. Thus, Bank has a security
interest in the wood as inventory.

Explanation to Point-Two(e) (5%):

The Gambretti plane is not included in Bank’s collateral.

The Gambretti plane that Acme uses to shape the violins it repairs is not “inventory” because it is not held
for sale or lease by Acme and does not otherwise fit into the other categories of “inventory” under UCC §
9-102. Rather, the Gambretti plane is “equipment” and not covered by Bank’s security interest.

Explanation to Point-Three (25%):

The Red Rosa violin was subject to Bank’s security interest prior to its sale, but the violinist was probably
a buyer in ordinary course of business and therefore took the violin free of Bank’s security interest under
UCC § 9-320.

Bank’s security interest in inventory covered the violin “Red Rosa” when it was held by Acme for sale.
Furthermore, under UCC § 9-315, except as otherwise provided in Article 9 “a security interest continues
in collateral notwithstanding sale unless the secured party authorized the disposition free of the security
interest.”

Here, there are no facts to indicate that Bank authorized the disposition of Red Rosa free of its security
interest. Nonetheless, the violinist’s interest in Red Rosa is likely to prevail over Bank’s security interest
because the violinist probably qualifies as a “buyer in ordinary course of business.”

Under UCC § 9-320, “a buyer in ordinary course of business takes free of a security interest created by
the buyer’s seller, even if the security interest is perfected and the buyer knows of its existence.” Under
UCC § 1-201, a buyer in ordinary course of business is “a person that buys goods in good faith, without
knowledge that the sale violates the rights of another person in the goods, and in the ordinary course from
a person in the business of selling goods of that kind.”

Acme is in the business of selling rare violins like Red Rosa, and the violinist apparently bought Red
Rosa in the ordinary course of Acme’s business. There are no facts to indicate that the purchase was in
bad faith or irregular in any way. Although the violinist was aware of Bank’s security interest in Acme’s

Seperac-J19 Exam-Released MEE Essay Compilation © 2016-2020 1038


inventory, there was no indication that the sale to the violinist violated any of Bank’s rights. Thus, while
the violinist was aware of the existence of the security interest, that knowledge does not disqualify the
violinist as a buyer in ordinary course who takes free of a security interest. As a buyer in ordinary course
of business, the violinist would take Red Rosa free of Bank’s security interest.

Explanation to Point-Four (25%):

Bank’s security interest in its collateral was perfected; therefore, the security interest is superior to the
rights of a judicial lien creditor that arose subsequently.

A security interest is subordinate to the rights of a person who becomes a lien creditor before the security
interest is perfected. Thus, if Bank’s security interest was not perfected at the time that creditor obtained
its judicial lien, the security interest would be subordinate to that lien. Bank’s security interest was
perfected, however. A security interest is perfected when it has attached and the other elements of
perfection have been satisfied. As described in Point One, Bank’s security interest in its collateral was
attached. The filing of the financing statement was sufficient to perfect this attached security interest.
Thus, both elements of perfection were satisfied and Bank’s security interest was perfected before the lien
creditor obtained its lien. Therefore, the security interest is superior to the rights of the lien creditor.

Seperac-J19 Exam-Released MEE Essay Compilation © 2016-2020 1039


#239-FEB 2014–MEE Q03: QUESTION THREE (UCC ART. 9)

On March 1, the owner of a manufacturing business entered into negotiations with a bank to obtain a loan
of $100,000 for the business. The bank loan officer informed the business owner that the interest rate for a
loan would be lower if the repayment obligation were secured by all the business’s present and future
equipment. The loan officer also informed the business owner that the bank could not commit to making
the loan until its credit investigation was completed, but that funds could be advanced faster following
loan approval if a financing statement with respect to the transaction were filed in advance. Accordingly,
the business owner signed a form on behalf of the business authorizing the bank to file a financing
statement with respect to the proposed transaction. The bank properly filed a financing statement the next
day, correctly providing the name of the business as the debtor and indicating “equipment” as the
collateral.

On March 15, the business owner had heard nothing from the bank about whether the loan had been
approved, so the business owner approached a finance company for a loan. The finance company quickly
agreed to lend $100,000 to the business, secured by all the business’s present and future equipment. That
same day, the finance company loaned to the business $100,000, and the business owner signed an
agreement obligating the business to repay the loan and granting the finance company a security interest
in all the business’s “present and future equipment” to secure the repayment obligation. Also on that day,
the finance company properly filed a financing statement correctly providing the business’s name as the
debtor and indicating “equipment” as the collateral.

On March 21, the bank loan officer contacted the business owner and indicated that the loan application
had been approved. On the next day, March 22, the bank loaned the business $100,000. The loan
agreement, signed by the owner on behalf of the business, granted the bank a security interest in all the
business’s “present and future equipment.”

On April 10, the business sold an item of manufacturing equipment to a competitor for $20,000. This was
the first time the business had ever sold any of its equipment. The competitor paid the purchase price in
cash and took possession of the equipment that day. The competitor acted in good faith at all times and
had no knowledge of the business’s prior transactions with the bank and the finance company.

The business has defaulted on its obligations with respect to the loans from the bank and the finance
company. Each of them has asserted a claim to all the business’s equipment as well as to the item of
equipment sold to the business’s competitor.

Assume that the business owner had the authority to enter into all these transactions on behalf of the
business.

1. As between the bank and the finance company, which has a superior claim to the business’s
equipment? Explain.

2. Do the claims of the bank and the finance company to the business’s equipment continue in the
item of equipment sold to the competitor? Explain.

Seperac-J19 Exam-Released MEE Essay Compilation © 2016-2020 1040


#239: F14-3 MEE: ANSWER: NCBE (UCC ART. 9)

POINT (1)(a) [25%] ISSUE: What is the nature of the bank’s claim to the business’s equipment?
ANSWER: The bank has a perfected security interest in the business’s equipment.

POINT (1)(b) [10%] ISSUE: What is the nature of the finance company’s claim to the business’s
equipment? ANSWER: The finance company also has a perfected security interest in the business’s
equipment.

POINT (1)(c) [30%] ISSUE: As between the bank and the finance company, whose claim to the
business’s equipment has priority? ANSWER: The bank’s security interest has priority over the
finance company’s security interest because the bank’s financing statement was filed first.

POINT (2) [35%] ISSUE: Do the claims of the bank and the finance company continue in the item
of equipment sold by the business to the competitor? ANSWER: Yes. A security interest in
collateral continues notwithstanding its sale unless an exception applies. Because the security
interests of the bank and the finance company were perfected and the competitor was not a buyer
in ordinary course of business, no exception applies and the security interests of both creditors
continue in the equipment sold to the competitor.

ANSWER DISCUSSION:

The bank and the finance company both have perfected security interests in the business’s equipment.
Even though the finance company’s perfected security interest was created first, the bank’s perfected
security interest has priority because the bank’s financing statement was filed before the finance
company’s financing statement. The security interests of the bank and the finance company continue in
the item of equipment sold by the business to the competitor because their security interests were
perfected and the competitor was not a buyer in ordinary course of business.

ANSWER EXPLANATION:

Explanation to Point-One(a) (25%):

The bank has a perfected security interest in the business’s equipment.

The bank has met all criteria necessary for it to have an attached and enforceable security interest in the
business’s equipment. First, value must be given. This criterion is fulfilled by the loan made by the bank
to the business. Second, the debtor must have rights in the collateral. Clearly, the business has rights in its
equipment. Third, either the secured party must take possession of the collateral or the debtor must
authenticate a security agreement containing a description of the collateral. The agreement that the
business owner signed is a “security agreement” because it is an agreement that creates or provides for a
security interest. By signing the security agreement, the business owner authenticated it. Therefore, all
three criteria are fulfilled, and the bank has an enforceable and attached security interest.

A security interest is perfected when it has attached and when any additional steps required for perfection
have occurred. Generally speaking, the additional steps will either be possession of the collateral by the
secured party or the filing of a financing statement with respect to the collateral. In this case, the bank
filed a financing statement naming the debtor and sufficiently indicating the collateral. The collateral

Seperac-J19 Exam-Released MEE Essay Compilation © 2016-2020 1041


indication is sufficient because it identifies the collateral by type of property. The fact that the financing
statement was filed before the security interest was created is not a problem. Even though the security
agreement had not yet been signed, the business had authorized the filing of the financing statement in an
authenticated record. Moreover, the financing statement may be filed before the security agreement is
created.

Explanation to Point-One(b) (10%):

The finance company also has a perfected security interest in the business’s equipment.

The finance company’s security interest is enforceable and attached for the same reasons as the bank’s
security interest. The loan from the finance company to the business constitutes value, the business has
rights in the collateral, and the business owner has authenticated a security agreement containing a
description of the collateral. The finance company’s security interest is perfected because the finance
company filed a financing statement with respect to it that provides that the business is the debtor and
indicates that the collateral is equipment.

Explanation to Point-One(c) (30%):

The bank’s security interest has priority over the finance company’s security interest because the bank’s
financing statement was filed first.

As between two perfected security interests, the general rule is that the security interest that was the
earlier to be either perfected or the subject of a filed financing statement has priority. While the finance
company’s security interest was perfected before the bank’s (March 15 vs. March 22), the bank’s
financing statement was filed even earlier, on March 2. Thus, under the first-to-file-or-perfect rule of
UCC § 9-322, the bank’s security interest has priority. No exceptions to the general rule apply here.

Explanation to Point-Two (35%):

A security interest in collateral continues notwithstanding its sale unless an exception applies. Because the
security interests of the bank and the finance company were perfected and the competitor was not a buyer
in ordinary course of business, no exception applies and the security interests of both creditors continue in
the equipment sold to the competitor.

As a general rule, a security interest in collateral continues notwithstanding the fact that the debtor has
sold the collateral to another person. Thus, unless an exception applies, the security interests of the bank
and the finance company will continue in the item of equipment sold to the competitor.

A buyer of goods will take free of an unperfected security interest in those goods. However, when the
competitor bought the business’s equipment, both the bank and the finance company had perfected
security interests in the equipment.

A buyer can take free even of a perfected security interest in goods if the buyer is a “buyer in ordinary
course of business.” However, the competitor was not a buyer in ordinary course of business. To be a
“buyer in ordinary course of business,” a buyer must buy goods from a seller that is in the business of
selling goods of that kind. The competitor bought this equipment from a seller that is not in the business
of selling goods of this kind, so the competitor was not a buyer in ordinary course of business with respect
to these goods.

Seperac-J19 Exam-Released MEE Essay Compilation © 2016-2020 1042


Because no exception applies, the security interests of the bank and the finance company continue even
after the item of equipment was sold to the competitor.

Seperac-J19 Exam-Released MEE Essay Compilation © 2016-2020 1043


#240-FEB 2013–MEE Q04: QUESTION FOUR (UCC ART. 9)

On June 1, a bicycle retailer sold two bicycles to a man for a total purchase price of $1,500. The man
made a $200 down payment and agreed to pay the balance in one year. The man also signed a security
agreement that identified the bicycles as collateral for the unpaid purchase price and provided that the
man “shall not sell or dispose of the collateral until the balance owed is paid in full.” The retailer never
filed a financing statement reflecting this security interest.

The man had bought the bicycles for him and his girlfriend to use on vacation. However, shortly after he
bought the bicycles, the man and his girlfriend broke up. The man has never used the bicycles.

On August 1, the man sold one of the bicycles at a garage sale to a buyer who paid the man $400 for the
bicycle. The buyer bought the bicycle to ride for weekend recreation.

On October 1, the man gave the other bicycle to his friend as a birthday present. The friend began using
the bicycle for morning exercise.

Neither the buyer nor the friend had any knowledge of the man’s dealings with the retailer.

1. Does the buyer own the bicycle free of the retailer’s security interest? Explain.

2. Does the friend own the bicycle free of the retailer’s security interest? Explain.

Seperac-J19 Exam-Released MEE Essay Compilation © 2016-2020 1044


#240: F13-4 MEE: ANSWER: NCBE (UCC ART. 9)

POINT (1) [35%] ISSUE: Is a purchase-money security interest in consumer goods perfected even
though there has been no filing of a financing statement? ANSWER: Yes. The retailer’s security
interest in the bicycles attached on June 1. Because this interest was a purchase-money security
interest in consumer goods, it was automatically perfected when it attached.

POINT (2) [35%] ISSUE: Does a person who buys consumer goods for personal use take those
goods free of a prior perfected purchase-money security interest in the goods? ANSWER: Yes. The
buyer took the bicycle free of the retailer’s security interest because (i) the retailer did not file a
financing statement covering the bicycle, (ii) the bicycle was “consumer goods,” and (iii) the buyer
bought the bicycle for value, without knowledge of the retailer’s security interest, and for personal
use.

POINT (3) [30%] ISSUE: Does a person who receives consumer goods as a gift take those goods
subject to a prior perfected security interest in them? ANSWER: Yes. The retailer’s security
interest continues in the bicycle that the man gave to the friend. Thus, the retailer can recover the
bicycle from the friend because the friend did not give value for the bicycle or buy it in the ordinary
course of business.

ANSWER DISCUSSION:

The retailer’s security interest in the bicycles was perfected, even though no financing statement was
filed, because it was a purchase-money security interest in consumer goods. A purchase-money security
interest in consumer goods is automatically perfected upon attachment. The buyer is not subject to the
retailer’s security interest in the bicycle that the buyer bought from the man. Because the bicycle was
consumer goods in the hands of the man, and the retailer never filed a financing statement covering the
bicycle, the retailer’s security interest is not effective against someone, like the buyer, who bought the
bicycle for value, without knowledge of the retailer’s security interest, and for personal use. On the other
hand, the retailer’s security interest continues in the bicycle given to the friend, because the friend did not
give value for the bicycle or buy it in the ordinary course of business.

ANSWER EXPLANATION:

Explanation to Point-One (35%):

The retailer’s security interest in the bicycles attached on June 1. Because this interest was a purchase-
money security interest in consumer goods, it was automatically perfected when it attached.

The retailer’s security interest in the bicycles attached on June 1 when the man bought the bicycles
(acquiring rights in the collateral), signed a security agreement containing a description of the collateral,
and received value from the retailer (by being given credit with which to purchase the bicycles).

Despite the retailer’s failure to file a financing statement, its security interest was perfected. Pursuant to
UCC § 9-309, a security interest is automatically perfected upon attachment if the goods are “consumer
goods” and the security interest is a “purchase-money security interest.”

Seperac-J19 Exam-Released MEE Essay Compilation © 2016-2020 1045


In this case, the bicycles sold by the retailer to the man were consumer goods at the time of sale. The
bicycles were “goods” because they were “movable when a security interest attaches.” They were also
consumer goods because they were “bought for use primarily for personal, family, or household
purposes.”

The retailer’s security interest in these consumer goods was also a “purchase-money security interest.” A
purchase-money security interest is an interest that secures a debt that was incurred in order to “enable the
debtor to acquire rights in or the use of the collateral.” Here, the man incurred an obligation to the retailer
to purchase the bicycles, so the security interest he gave the retailer to secure that obligation was a
purchase-money security interest.

Because the retailer’s security interest was a purchase-money security interest in consumer goods, it was
automatically perfected on June 1, when the interest attached to the bicycles.

Explanation to Point-Two (35%):

The buyer took the bicycle free of the retailer’s security interest because (i) the retailer did not file a
financing statement covering the bicycle, (ii) the bicycle was “consumer goods,” and (iii) the buyer
bought the bicycle for value, without knowledge of the retailer’s security interest, and for personal use.

A security interest continues in collateral, even after a sale or other disposition of that collateral, unless
the creditor authorized the disposition “free of the security interest” or another Article 9 exception applies.

However, a buyer of goods, like the buyer here, can take free of a prior security interest in those goods
under certain circumstances. For example, under UCC § 9-317(b), buyers who give value and receive
delivery of goods without knowledge of an unperfected security interest in the goods can take free of the
prior security interest. Under UCC § 9-320, a buyer in ordinary course of business can take free of the
prior security interest. In this case, the retailer’s security interest was perfected when the buyer purchased
the bicycle, so UCC § 9-317 does not protect the buyer. The buyer also is not a protected “buyer in
ordinary course of business” because he did not purchase from a person who is in the business of selling
bicycles.

The buyer can, however, qualify for the protection of UCC § 9-320. That section provides that a buyer of
goods from a person who used them for personal, family, or household purposes takes free of a perfected
security interest in the goods if (1) the buyer had no knowledge of the security interest, (2) the buyer gave
value for the goods, (3) the buyer purchased the goods primarily for personal, family, or household
purposes, and (4) the purchase occurred before the filing of a financing statement covering the goods.

The buyer met all of these criteria. The man used the bicycle for personal purposes. The buyer purchased
the bicycle from the man, and the buyer had no knowledge of the retailer’s security interest. The buyer
gave value ($400) for the bicycle, and he bought it “primarily for personal, family, or household
purposes,” as he planned to use it for recreation, which is a personal rather than a business use. Finally, no
financing statement had been filed. Therefore, under UCC § 9-320, the buyer took free of the retailer’s
security interest.

Explanation to Point-Three (30%):

The retailer’s security interest continues in the bicycle that the man gave to the friend. Thus, the retailer
can recover the bicycle from the friend because the friend did not give value for the bicycle or buy it in

Seperac-J19 Exam-Released MEE Essay Compilation © 2016-2020 1046


the ordinary course of business.

As noted in Point Two, the retailer did not authorize the man to dispose of the bicycle. Consequently, the
retailer’s security interest continued in the bicycle even after the man transferred ownership of the bicycle
to the friend. The retailer’s security interest in the bicycle will be effective against the friend unless some
other provision of Article 9 allows the friend to take the bicycle free of that security interest.

Unfortunately for the friend, there is no Article 9 provision that allows him to take free of the retailer’s
interest. The friend’s basic problem is that he is not a buyer of the bicycle – he received the bicycle as a
gift and did not give value for it. Thus, the friend is not protected by any of the applicable exceptions,
such as UCC § 9-317 which protects buyers who give value for goods subject to an unperfected security
interest, or UCC § 9-320(a) which protects buyers in the ordinary course of business, or UCC § 9-320
which protects buyers of consumer goods who give value.

In short, the retailer’s security interest continues in the bicycle that the man gave to the friend. The friend
took the bicycle subject to that security interest.

Seperac-J19 Exam-Released MEE Essay Compilation © 2016-2020 1047


#241-JUL 2012–MEE Q05: QUESTION FIVE (UCC ART. 9)

On March 1, Recycled, a business that sells new and used bicycles and bicycle equipment, borrowed
$100,000 from Bank. To secure its obligation to repay the loan, Recycled signed an agreement granting
Bank a security interest in “all the inventory of Recycled, whether now owned or hereafter acquired.”

On March 5, Bank filed a financing statement in the appropriate state office. The financing statement
listed Recycled as debtor and “inventory” as collateral.

Over the next month, Recycled entered into the following transactions:

(a) On March 10, Recycled sold a new bicycle to Consumer for $1,500. The sale was made in accordance
with the usual business practices of Recycled. Both parties acted honestly and in accordance with
reasonable commercial standards of fair dealing, and Consumer was unaware of the financial relationship
between Recycled and Bank.

(b) On March 15, Recycled traded a used bicycle to Student for a used computer that Student no longer
needed. Recycled immediately began using the computer in its business.

(c) On March 31, Recycled bought 100 new bicycle helmets from Manufacturer. The sale was on credit,
with payment due in 15 days. The written sales agreement, signed by Recycled, states that Manufacturer
retains title to the helmets until Recycled pays their purchase price to Manufacturer. No financing
statement was filed. None of the helmets has been sold by Recycled.

Recycled has not paid its utility bills for several months. On April 29, Utility obtained a judgment in the
amount of $2,500 against Recycled and, pursuant to state law, obtained a judgment lien against all the
personal property of Recycled.

Recycled is in default on its repayment obligation to Bank, and it has not paid the amount it owes to
Manufacturer.

Bank claims a security interest in all the bicycles and bicycle helmets owned by Recycled, the bicycle
bought by Consumer, and the computer obtained by Recycled in the transaction with Student.
Manufacturer claims an interest in the bicycle helmets, and Utility seeks to enforce its lien against all the
personal property of Recycled.

1. As between Bank and Consumer, which has a superior claim to the bicycle sold to Consumer?
Explain.

2. As between Bank and Utility, which has a superior claim to the used computer? Explain.

3. As among Bank, Manufacturer, and Utility, which has a superior claim to the 100 bicycle
helmets? Explain.

Seperac-J19 Exam-Released MEE Essay Compilation © 2016-2020 1048


#241: J12-5 MEE: ANSWER: NCBE (UCC ART. 9)

POINT (1) [34%] ISSUE: What are the relative rights of a creditor with a perfected security
interest in a debtor’s inventory and a buyer who bought an item of the inventory in ordinary course
of business? ANSWER: Because Consumer was a buyer in ordinary course of business with respect
to the bicycle, Consumer took the bicycle free of Bank’s security interest.

POINT (2) [33%] ISSUE: What are the relative rights of a creditor with a perfected security
interest in a debtor’s inventory and a lien creditor of the debtor with respect to an item of
equipment that the debtor received in exchange for an item of inventory? ANSWER: Although
Bank’s security agreement does not cover the equipment of Recycled, the used computer, which is
equipment, constitutes proceeds of the inventory of Recycled, and Bank’s security interest in the
inventory extends to proceeds of the inventory. Because Bank’s interest was perfected before Utility
became a lien creditor, Bank has a prior claim to the used computer.

POINT (3) [33%] ISSUE: What are the relative rights of a creditor with a perfected security
interest in a debtor’s inventory, a lien creditor of the debtor, and a supplier who has sold items of
inventory to the debtor on credit, retaining title to the items, but has not filed a financing
statement? ANSWER: Bank’s perfected security interest in the bicycle helmets is superior to
Manufacturer’s unperfected security interest and to Utility’s later judgment lien. Manufacturer’s
security interest is subordinate to Utility’s judgment lien.

ANSWER DISCUSSION:

Bank has a perfected security interest in the present and future inventory of Recycled. That security
interest covers both the bicycles in the inventory of Recycled and the bicycle helmets bought from
Manufacturer. Although, as a general rule, a security interest continues in collateral notwithstanding its
sale by the debtor, Bank does not have a claim to Consumer’s bicycle. As a buyer in ordinary course of
business from Recycled, Consumer took the bicycle free of the security interest in it that Recycled gave to
Bank. Bank also has a perfected security interest in the used computer. Although the computer is not
inventory, it was received in exchange for an item of inventory and constitutes proceeds of that inventory
to which Bank’s security interest extends. Bank’s security interest in the computer is superior to Utility’s
judgment lien, which was created after Bank had obtained its perfected security interest in the computer.
Manufacturer has a security interest in the helmets it sold to Recycled, but its unperfected security interest
is subordinate to the perfected security interest of Bank and to Utility’s judgment lien.

ANSWER EXPLANATION:

Explanation to Point-One (34%):

Because Consumer was a buyer in ordinary course of business with respect to the bicycle, Consumer took
the bicycle free of Bank’s security interest.

While a security interest in collateral generally continues in that collateral even after the collateral has
been sold, that principle is subject to important exceptions. In particular, a buyer in ordinary course of
business “takes free of a security interest” created by its seller “even if the security interest is perfected
and the buyer knows of its existence.”

Seperac-J19 Exam-Released MEE Essay Compilation © 2016-2020 1049


Here, Recycled sold the bicycle to Consumer. Consumer took free of Bank’s security interest – which was
created by Recycled, its seller – if Consumer was a “buyer in ordinary course of business.” A buyer in
ordinary course of business is “a person that buys goods in good faith, without knowledge that the sale
violates the rights of another person in the goods, and in the ordinary course from a person in the business
of selling goods of that kind.”

On these facts, Consumer is a buyer in ordinary course of business. First, Consumer bought the bicycle
from Recycled, which is in the business of selling goods of the kind that Consumer bought – bicycles.
Second, the sale occurred “in the ordinary course,” i.e., in the seller’s usual way. Third, Consumer acted
honestly and in a manner consistent with reasonable commercial standards of fair dealing. Thus,
Consumer acted in “good faith.” Finally, there are no facts to suggest that Consumer knew that the sale
was in violation of Bank’s rights.

As a buyer in ordinary course of business, Consumer took the bicycle free of Bank’s security interest, and
Bank’s security interest is extinguished.

Explanation to Point-Two (33%):

Although Bank’s security agreement does not cover the equipment of Recycled, the used computer, which
is equipment, constitutes proceeds of the inventory of Recycled, and Bank’s security interest in the
inventory extends to proceeds of the inventory. Because Bank’s interest was perfected before Utility
became a lien creditor, Bank has a prior claim to the used computer.

The security agreement pursuant to which Bank obtained its perfected security interest covers only
inventory. The used computer was not held by Recycled for sale or lease, so it is not inventory. Rather, it
is equipment. Thus, it is not included in the description of collateral in the security agreement.

However, Recycled obtained the computer in exchange for a bicycle that was inventory (and, thus,
collateral). Because the computer was “acquired upon the exchange of collateral,” the computer is
“proceeds” of that collateral. A security interest in collateral extends to identifiable proceeds of that
collateral. Thus, Bank has a security interest in the computer. In addition, the security interest in the
computer as proceeds is perfected because the security interest in the original collateral (the bicycle) was
perfected by the filing of a financing statement in the same office in which a financing statement would be
filed in order to perfect a security interest in the computer.

Because Bank has a perfected security interest in the used computer as proceeds, it has priority over
Utility’s later judgment lien.

Explanation to Point-Three (33%):

Bank’s perfected security interest in the bicycle helmets is superior to Manufacturer’s unperfected
security interest and to Utility’s later judgment lien. Manufacturer’s security interest is subordinate to
Utility’s judgment lien.

While the contract with Manufacturer provides that Manufacturer retains title to the helmets until
Recycled pays for them, the purported retention of “title” is limited in effect to retention of a “security
interest.” Manufacturer’s security interest is not perfected because Manufacturer has not filed a financing
statement, and there is no other basis for it to claim perfection of its security interest.

Seperac-J19 Exam-Released MEE Essay Compilation © 2016-2020 1050


Bank has a security interest in the inventory of Recycled. Bank perfected that interest by filing a financing
statement. Because the bicycle helmets are held for sale by Recycled, they are inventory and are subject to
Bank’s security interest.

Bank’s security interest has priority over Manufacturer’s security interest because a perfected security
interest has priority over an unperfected security interest.

Because Bank has a perfected security interest in the bicycle helmets, it has priority over Utility’s later
judgment lien. However, Utility’s judgment lien has priority over Manufacturer’s security interest in the
bicycle helmets because Manufacturer’s interest is unperfected.

[NOTE: An examinee might correctly note that Manufacturer’s security interest is a “purchase-money
security interest.” That point is irrelevant to the answer, however. A perfected purchase-money security
interest can, under proper circumstances, have priority over other security interests, but Manufacturer’s
security interest was not perfected because no financing statement was filed and no other criterion for
perfection has been satisfied. Thus, the general rule of UCC § 9-322 – a perfected security interest has
priority over an unperfected security interest – determines the priority of the competing security interests
in this case. An examinee might erroneously state that Manufacturer’s purchase-money security interest
is automatically perfected without the need for filing pursuant to UCC § 9-309. This is incorrect, because
only a purchase-money security interest in consumer goods qualifies for this special rule, and the helmets
are “inventory” rather than “consumer goods.”]

Seperac-J19 Exam-Released MEE Essay Compilation © 2016-2020 1051


#242-JUL 2011–MEE Q01: QUESTION ONE (UCC ART. 9)

Decorator operates a business that sells decorative items for the office. Eight months ago, Decorator
borrowed $10,000 from Lender and, pursuant to a properly completed and signed security agreement,
granted Lender a security interest in all of Decorator’s present and future inventory and equipment to
secure that indebtedness. Lender filed a properly completed financing statement on the same day that the
loan was made and the security agreement was signed.

Seven months ago, Clockwork and Decorator entered into a signed agreement pursuant to which
Decorator bought and received delivery of 25 decorative clocks from Clockwork for resale to Decorator’s
customers. Under the terms of the agreement, Decorator agreed to pay the $2,500 purchase price in six
months. The agreement also provides that, until the payment of the purchase price to Clockwork by
Decorator, title to the clocks will be retained by Clockwork. No financing statement was filed in
conjunction with this transaction.

Three months ago, Decorator leased an industrial vacuum cleaner from Vac for use in Decorator’s
business. The lease, which was signed by both parties, provides that, at the end of the four-year lease term
(which cannot be terminated early), Decorator will automatically become the owner of the vacuum
cleaner so long as all monthly payments have been made. No financing statement was filed in conjunction
with this transaction.

Decorator has defaulted on all obligations to Lender, Clockwork, and Vac. Your law firm represents
Lender, who has asked the following questions:

1. Who has a superior interest in the clocks? Explain.

2. Who has a superior interest in the vacuum cleaner? Explain.

Seperac-J19 Exam-Released MEE Essay Compilation © 2016-2020 1052


#242: J11-1 MEE: ANSWER: NCBE (UCC ART. 9)

POINT (1) [25%] ISSUE: What interest does Clockwork have in the clocks? ANSWER: Clockwork
has an unperfected security interest in the clocks.

POINT (2) [10%] ISSUE: Does Lender have an interest in the clocks? ANSWER: Yes. Lender has
a perfected security interest in the clocks.

POINT (3) [15%] ISSUE: Whose interest in the clocks – Clockwork’s or Lender’s – has priority?
ANSWER: Lender’s security interest in the clocks is superior to that of Clockwork.

POINT (4) [25%] ISSUE: What interest does Vac have in the vacuum cleaner? ANSWER: Vac has
an unperfected security interest in the vacuum cleaner.

POINT (5) [10%] ISSUE: Does Lender have an interest in the vacuum cleaner? ANSWER: Yes.
Lender has a perfected security interest in the vacuum cleaner.

POINT (6) [15%] ISSUE: Whose interest in the vacuum cleaner – Vac’s or Lender’s – has priority?
ANSWER: Lender’s security interest in the vacuum cleaner is superior to that of Vac.

ANSWER DISCUSSION:

Clockwork’s interest in the clocks is a security interest governed by Article 9 of the UCC. Because
Clockwork did not file a financing statement with respect to its security interest in the clocks, its security
interest is unperfected. The clocks are inventory of Decorator, so Lender also has a security interest in
them. That interest was perfected when Lender filed a financing statement covering it. Lender’s perfected
security interest has priority over Clockwork’s unperfected interest. Vac’s interest in the vacuum cleaner,
although presented as a lease, is a security interest governed by Article 9 of the UCC. Because Vac did
not file a financing statement with respect to its interest in the vacuum cleaner, its security interest is
unperfected. The vacuum cleaner is equipment of Decorator, so Lender has a security interest in it.
Because Lender filed a financing statement, Lender’s security interest in the vacuum cleaner is perfected.
Unperfected security interests are subordinate to perfected security interests, so Lender’s security interest
has priority over Vac’s.

ANSWER EXPLANATION:

Explanation to Point-One (25%):

Clockwork has an unperfected security interest in the clocks.

The agreement between Clockwork and Decorator provides that Clockwork retains title to the clocks until
their purchase price is paid. Under the Uniform Commercial Code, the substance of a transaction controls,
rather than its form or the label given to it by the parties. Because Clockwork’s interest exists to secure
performance of Decorator’s payment obligation, it is a security interest even though the parties did not
label it as such.

Because the Clockwork-Decorator transaction created a security interest by contract, it is governed by


Article 9 of the Uniform Commercial Code. The signed agreement between Clockwork and Decorator is

Seperac-J19 Exam-Released MEE Essay Compilation © 2016-2020 1053


sufficient to create an enforceable and attached security interest because value has been given to
Decorator (the extension of credit), Decorator has rights in the clocks, and the agreement between them,
providing for Clockwork’s retained interest in the clocks, was signed by Decorator. However, in order for
Clockwork’s security interest to be perfected, Clockwork would need to either have possession of the
clocks or file a financing statement with respect to the security interest. Clockwork’s security interest in
the clocks is not perfected because Clockwork neither filed a financing statement with respect to its
security interest nor retained or took possession of the clocks.

[NOTE: An examinee might state that Clockwork’s security interest is a purchase-money security interest
and, therefore, perfected upon attachment without the necessity of filing a financing statement. This is
incorrect. Only purchase-money security interests in consumer goods are perfected upon attachment
without the filing of a financing statement. While Clockwork’s security interest is a purchase-money
security interest, the clocks do not constitute “consumer goods” in the hands of Decorator because the
clocks are being held for sale to Decorator’s customers and were not used or bought for Decorator’s
personal, family, or household purposes.]

Explanation to Point-Two (10%):

Lender has a perfected security interest in the clocks.

Pursuant to its agreement with Decorator, Lender was granted a security interest in all Decorator’s present
and future inventory. Because the clocks are being held by Decorator for resale to its customers, they are
inventory. The security interest with respect to each clock that Decorator has or subsequently acquires is
enforceable and attached because value has been given, Decorator has rights in the clocks, and Decorator
has signed the security agreement. The after-acquired property clause is made enforceable by UCC § 9-
204. The security interest is perfected because Lender filed a properly completed financing statement with
respect to the security interest.

Explanation to Point-Three (15%):

Lender’s security interest in the clocks is superior to that of Clockwork.

While both Clockwork and Lender have security interests in the clocks, only Lender’s security interest is
perfected. Lender’s security interest has priority over that of Clockwork because a perfected security
interest is superior to an unperfected security interest in the same property.

[NOTE: An examinee might state that Clockwork’s security interest has priority under UCC § 9-324
because it is a purchase-money security interest. This is incorrect. While Clockwork’s security interest is
a purchase-money security interest, § 9-324 gives priority only to certain perfected purchase-money
security interests and Clockwork’s security interest is unperfected.]

Explanation to Point-Four (25%):

Vac has an unperfected security interest in the vacuum cleaner.

The agreement between Vac and Decorator is presented as a lease. However, the UCC recognizes that the
transaction is economically equivalent to a secured sale because Decorator will automatically become the
owner of the vacuum cleaner after making all of the lease payments. Accordingly, the interest of Vac is a
“security interest.” Thus, the Vac-Decorator transaction is governed by Article 9 of the Uniform
Commercial Code. The signed agreement between Vac and Decorator is sufficient to create an

Seperac-J19 Exam-Released MEE Essay Compilation © 2016-2020 1054


enforceable and attached security interest because value has been given to Decorator, Decorator has rights
in the vacuum cleaner, and the agreement was signed by Decorator. However, in order for Vac’s security
interest to be perfected, Vac would need to retain or take possession of the vacuum cleaner or file a
financing statement with respect to the security interest. Because Vac neither filed a financing statement
with respect to its security interest nor retained or took possession of the vacuum cleaner, Vac’s security
interest in the vacuum cleaner is not perfected.

Explanation to Point-Five (10%):

Lender has a perfected security interest in the vacuum cleaner.

Pursuant to its agreement with Decorator, Lender was granted a security interest in all of Decorator’s
present and future equipment. Because the vacuum cleaner is used by Decorator in its business and is not
inventory, farm products, or consumer goods, it constitutes equipment. The security interest with respect
to the vacuum cleaner is enforceable and attached because value has been given, Decorator has rights in
the vacuum cleaner, and Decorator has signed the security agreement. Therefore, Lender’s security
interest attaches to the vacuum cleaner. The security interest is perfected because Lender filed a properly
completed financing statement with respect to the security interest in inventory and equipment.

Explanation to Point-Six (15%):

Lender’s security interest in the vacuum cleaner is superior to that of Vac.

While both Vac and Lender have security interests in the vacuum cleaner, only Lender’s security interest
is perfected. Lender’s security interest has priority over that of Vac because a perfected security interest is
superior to an unperfected security interest in the same property.

Seperac-J19 Exam-Released MEE Essay Compilation © 2016-2020 1055


#243-FEB 2011–MEE Q03: QUESTION THREE (UCC ART. 9)

Astronomy Corporation (Astronomy) sells expensive telescopes to home stargazers. Astronomy has a
long-term financing arrangement pursuant to which it borrows money from Bank. In a signed writing,
Astronomy granted Bank a security interest in all its present and future inventory to secure its obligations
to Bank under the financing arrangement. Bank filed a properly completed financing statement reflecting
this transaction. The financing statement lists Astronomy as the debtor and Bank as the secured party. The
financing statement indicates that the collateral is inventory.

Astronomy sells telescopes to some of its customers on credit. For a credit sale, Astronomy requires the
customer to sign an agreement granting Astronomy a security interest in the purchased item to secure the
customer’s obligation to pay the balance of the purchase price.

Six months ago, Johnson, an amateur stargazer, went to Astronomy’s showroom, saw a $3,000 telescope
that he liked, and bought it on credit from Astronomy. Johnson paid $500 in cash and agreed to pay the
$2,500 balance in installment payments of $100 per month for the next 25 months, interest free.
Consistent with Astronomy’s policy for credit sales, Johnson signed an agreement granting Astronomy a
security interest in the telescope to secure Johnson’s obligation to pay the balance of the purchase price.
Astronomy did not file a financing statement with respect to this transaction. At the time of the sale of the
telescope to Johnson, Johnson was unaware of the financial arrangement between Astronomy and Bank.

One month ago, Johnson sold the telescope for $2,700 in cash to his neighbor, Smith, another amateur
stargazer. Smith had no knowledge of any interest of Bank or Astronomy in the telescope. Johnson then
left the country without paying the remaining $2,000 owed to Astronomy and cannot be located.

One week ago, Astronomy defaulted on its obligations to Bank.

Both Bank and Astronomy have discovered that Johnson sold the telescope to Smith. Bank and
Astronomy each have demanded that Smith surrender the telescope on the grounds that it is collateral for
obligations owed to them.

1. Does Bank have a security interest in the telescope that is enforceable against Smith? Explain.

2. Does Astronomy have a security interest in the telescope that is enforceable against Smith?
Explain.

Seperac-J19 Exam-Released MEE Essay Compilation © 2016-2020 1056


#243: F11-3 MEE: ANSWER: NCBE (UCC ART. 9)

POINT (1)(a) [20%] ISSUE: Did Bank have a security interest in the telescope before it was sold by
Astronomy? Was the security interest perfected? ANSWER: Yes. Bank had a perfected security
interest in the telescope before it was sold.

POINT (1)(b) [30%] ISSUE: Did Bank’s security interest in the telescope continue after the
successive sales of the telescope to Johnson and Smith? ANSWER: No. Bank’s security interest in
the telescope did not continue after the successive sales of the telescope to Johnson and Smith.

POINT (2)(a) [25%] ISSUE: Did Astronomy have a security interest in the telescope before
Johnson sold it to Smith? Was the security interest perfected? ANSWER: Yes. Before Johnson sold
the telescope, Astronomy had a perfected security interest in it.

POINT (2)(b) [25%] ISSUE: Did Astronomy’s security interest in the telescope continue to be
enforceable after Johnson sold the telescope to Smith? ANSWER: No. Astronomy’s security
interest in the telescope is not enforceable against Smith.

ANSWER DISCUSSION:

Bank had a perfected security interest in Astronomy’s inventory. When one item of that inventory – the
telescope – was sold to Johnson, Johnson took the telescope free of Bank’s security interest in it because
Johnson was a “buyer in ordinary course of business.” Because Johnson owned the telescope free of
Bank’s security interest, Johnson transferred it to Smith free of that security interest. Astronomy retained
a security interest in the telescope when Johnson bought it from Astronomy. The security interest was
perfected even though Astronomy did not file a financing statement because the security interest was a
purchase-money security interest in consumer goods. When Smith later bought the telescope from
Johnson, he took free of Astronomy’s security interest because of the protection afforded to buyers in
consumer-to-consumer transactions by Article 9 of the Uniform Commercial Code.

ANSWER EXPLANATION:

Explanation to Point-One(a) (20%):

Bank had a perfected security interest in the telescope before it was sold.

Because Astronomy held the telescope for sale, the telescope was part of Astronomy’s inventory. Thus, it
was covered by the security interest granted by Astronomy to Bank. The security interest was enforceable
and attached because the three criteria in UCC § 9-203 were fulfilled: value had been given (Bank loaned
money to Astronomy), Astronomy had rights in the telescope, and Astronomy had authenticated (signed
or its electronic equivalent) a security agreement containing a description of the collateral. Bank’s
security interest in the telescope was also perfected. Filing the statement satisfied the requirement in UCC
§ 9-310 that a financing statement must be filed to perfect a nonpossessory security interest in goods.

Explanation to Point-One(b) (30%):

Bank’s security interest in the telescope did not continue after the successive sales of the telescope to
Johnson and Smith.

Seperac-J19 Exam-Released MEE Essay Compilation © 2016-2020 1057


As a general rule, a security interest continues after sale of the collateral. However, that general rule is
subject to many exceptions, one of which applies here: a buyer in ordinary course of business (BIOCOB)
takes free of a security interest created by that buyer’s seller. A BIOCOB is a buyer who buys goods in
good faith, without knowledge that the sale violates the rights of another, from a person in the business of
selling goods of the kind, in the ordinary course of the seller’s business. Johnson qualifies as a BIOCOB.
Johnson bought the telescope from a person (Astronomy) in the business of selling goods of the kind in
the ordinary course of Astronomy’s business, and there is no indication in these facts that Johnson failed
to act in good faith or had knowledge that the sale violated the rights of another. Because Johnson was a
BIOCOB, he took the telescope free of Bank’s security interest, which was created by his seller,
Astronomy.

Since Johnson owned the telescope free of that security interest, under general property principles all of
Johnson’s property rights in the telescope were transferred to Smith when Johnson sold the telescope to
Smith. Thus, Smith acquired the telescope free of Bank’s security interest. This is known in secured
transactions law as the “shelter principle.”

Explanation to Point-Two(a) (25%):

Before Johnson sold the telescope, Astronomy had a perfected security interest in it.

The security agreement between Johnson and Astronomy provided that Astronomy retained a security
interest in the telescope to secure Johnson’s obligation to pay the remainder of the purchase price. The
security interest was enforceable and attached because the three criteria in UCC § 9-203 were fulfilled:
value had been given (the telescope), Johnson had rights in the telescope, and Johnson had authenticated
(signed or its electronic equivalent) a security agreement containing a description of the collateral.

Astronomy’s security interest in the telescope was also perfected, even though Astronomy did not file a
financing statement. This is because Astronomy’s security interest was a purchase-money security interest
(PMSI) in consumer goods. It was a PMSI because it was retained by the seller of the telescope to secure
the buyer’s obligation to pay the remainder of the purchase price. The telescope constituted consumer
goods because it was bought by Johnson for personal, family, or household purposes. UCC § 9-309
provides that a PMSI in consumer goods perfects automatically upon attachment; thus, it was not
necessary for Astronomy to file a financing statement to perfect its security interest.

Explanation to Point-Two(b) (25%):

Astronomy’s security interest in the telescope is not enforceable against Smith.

Prior to the sale from Johnson to Smith, Astronomy had a perfected security interest in the telescope.
Smith does not qualify as a BIOCOB because, inter alia, Smith did not buy the telescope from someone in
the business of selling goods of that kind. (Johnson was not in the business of selling telescopes.) Thus,
Smith did not take free of Astronomy’s security interest pursuant to UCC § 9-320. However, a buyer of
consumer goods takes free of a security interest in those goods if the buyer buys without knowledge of the
security interest; gives value; buys for personal, family, or household use; and receives the goods before
the filing of any financing statement covering them. Smith fulfilled these criteria. (Recall that Astronomy
was not required to file a financing statement to perfect its security interest because it was a purchase-
money security interest.) Thus, Smith took the telescope free of Astronomy’s security interest in it.

Seperac-J19 Exam-Released MEE Essay Compilation © 2016-2020 1058


[NOTE: Some examinees may note that Astronomy’s rights against Johnson constitute chattel paper in
which Bank has a security interest as proceeds of its original collateral. This is correct but does not affect
the answer to the question posed because Smith is free of both security interests.]

Seperac-J19 Exam-Released MEE Essay Compilation © 2016-2020 1059


#244-FEB 2010–MEE Q01: QUESTION ONE (UCC ART. 9)

Six months ago, Kitchenware, a manufacturer of copper cookware, borrowed $200,000 from Bank and
signed a security agreement granting Bank a security interest in “all inventory that Kitchenware now
owns or that it manufactures or acquires in the future.” Bank filed a properly completed financing
statement reflecting this security interest in the appropriate state office.

Copperco is a company that produces high-quality copper sheet that is suitable for fabrication into
cookware. Two months ago, Kitchenware entered into a contract with Copperco to buy two tons of copper
sheet to be used by Kitchenware to produce cookware. The contract, which was signed by both parties,
required Copperco to deliver the copper sheet to Kitchenware’s factory in two installments, one ton in the
first installment and the second ton 30 days later. Kitchenware was to pay for each of the installments
separately, with one-half of the contract price due 25 days after the first delivery and the balance due 25
days after the second delivery. Copperco’s obligation to ship the second installment was expressly made
conditional on full payment for the first installment. The parties further agreed that Kitchenware would
have no rights in an installment of the copper sheet until it received delivery of that installment and that
Copperco would retain title to all the copper sheet until Kitchenware paid the full contract price.

Copperco promptly delivered the first ton of copper sheet to Kitchenware’s factory. Twenty-three days
after the delivery, Copperco loaded its truck with a second ton of copper sheet for delivery to
Kitchenware’s factory and planned to send the truck to Kitchenware in time to meet its delivery deadline.
However, by 25 days after the first delivery, Kitchenware had not paid for the first installment of copper
sheet. As a result, Copperco exercised its right under the contract to withhold shipment of the second
installment and, accordingly, the truck with the second ton of copper sheet never left Copperco’s plant.

Kitchenware has defaulted on its loan from Bank, and Bank would like to exercise its rights with respect
to its collateral. The first ton of copper sheet delivered to Kitchenware is still at Kitchenware’s factory,
and the second ton of copper sheet that was not delivered to Kitchenware is still at Copperco’s plant.
Bank believes it has a security interest in both tons of copper sheet, while Copperco asserts that it has title
to both tons of copper sheet and that its rights are superior to any rights of Bank.

1. As between Copperco and Bank, which has the superior claim to the first ton of copper sheet that
was delivered to Kitchenware? Explain.

2. As between Copperco and Bank, which has the superior claim to the second ton of copper sheet,
which is still at Copperco’s plant? Explain.

Seperac-J19 Exam-Released MEE Essay Compilation © 2016-2020 1060


#244: F10-1 MEE: ANSWER: NCBE (UCC ART. 9)

POINT (1)(a) [30%] ISSUE: When a contract provides that title to sold goods will not pass until
payment has been made, what rights in the goods do seller and buyer have? ANSWER: Copperco’s
reservation of title to the delivered goods is ineffective except as retention of a security interest;
Copperco’s security interest is unperfected.

POINT (1)(b) [10%] ISSUE: Does a secured party who has a security interest in the debtor’s
inventory have a security interest in raw materials that the debtor will use to manufacture its
products? ANSWER: Yes. The copper sheet constitutes inventory, so it is covered by Bank’s
security interest.

POINT (1)(c) [20%] ISSUE: What interest does a secured party with a security interest in goods
have when the debtor receives goods pursuant to a contract providing that title to the goods does
not pass to the debtor until they are paid for? ANSWER: Bank has a perfected security interest in
the delivered copper sheet.

POINT (1)(d) [20%] ISSUE: As between a lender with a security interest in inventory and an
unpaid seller of the goods constituting inventory who has delivered the goods to the buyer but
retained title to them until they are paid for, who has priority? ANSWER: Bank’s perfected
security interest in the delivered copper sheet is superior to Copperco’s unperfected security
interest.

POINT (2) [20%] ISSUE: Does a secured party with a security interest in a debtor’s inventory have
a security interest in goods that the debtor has contracted to buy but in which the debtor has no
rights? ANSWER: No. Because Kitchenware did not have rights in the undelivered copper sheet,
Bank’s security interest in it did not attach.

ANSWER DISCUSSION:

Copperco’s interest in the copper sheet that has been delivered to Kitchenware is “limited in effect to a
reservation of a security interest.” Thus, Kitchenware is the owner of that copper sheet and Copperco has
a security interest in it. The delivered copper sheet became part of Kitchenware’s inventory and thus,
Bank’s security interest attached to it and was perfected. Copperco’s security interest in the copper sheet
is unperfected. As a result, Bank’s security interest in the delivered copper sheet has priority over
Copperco’s security interest. With respect to the second installment of copper sheet, the security
agreement with Bank will create an enforceable security interest in that sheet only if Kitchenware “has
rights in” the sheet. Because, under the Copperco-Kitchenware contract, Kitchenware has no rights in the
second installment of copper sheet, Bank’s security interest did not attach to that installment and only
Copperco has an interest in it.

ANSWER EXPLANATION:

Explanation to Point-One(a) (30%):

Copperco’s reservation of title to the delivered goods is ineffective except as retention of a security
interest; Copperco’s security interest is unperfected.

Seperac-J19 Exam-Released MEE Essay Compilation © 2016-2020 1061


The sales contract between Copperco and Kitchenware provided that Copperco would retain title to the
copper sheet until it received full payment for it. Under the Uniform Commercial Code, such “retention or
reservation of title by a seller of goods notwithstanding shipment or delivery to the buyer under Section 2-
401 is limited in effect to a reservation of a ‘security interest.’” Thus, Kitchenware owns the copper sheet,
and Copperco has a security interest in it. Copperco has neither retained possession of the copper sheet
nor filed a financing statement with respect to it, so Copperco’s security interest is unperfected.

[NOTE: An applicant might note that Copperco’s security interest is a purchase-money security interest.
This statement is correct, but it does not change the analysis because only a purchase-money security
interest in consumer goods is perfected without filing or possession by the secured party.]

Explanation to Point-One(b) (10%):

The copper sheet constitutes inventory, so it is covered by Bank’s security interest.

Bank has a security interest in Kitchenware’s “inventory.” The copper sheet is inventory of Kitchenware
because it constitutes “raw materials, work in process, or materials used or consumed” in Kitchenware’s
business.

Explanation to Point-One(c) (20%):

Bank has a perfected security interest in the delivered copper sheet.

As noted in Point One(a) above, once the copper sheet was delivered to Kitchenware, Kitchenware
became the owner of the sheet even though the contract provided that Copperco retained title. At that
point, Kitchenware had rights in the delivered copper sheet, and Bank’s security interest in inventory
attached to it. Under UCC § 9-203, a security interest attaches when value is given, a security agreement
has been signed, and the debtor has rights in the collateral. A security interest may attach to after-acquired
collateral. Bank filed a financing statement with respect to its security interest, so its security interest in
the copper sheet is perfected.

Explanation to Point-One(d) (20%):

Bank’s perfected security interest in the delivered copper sheet is superior to Copperco’s unperfected
security interest.

As noted above, Bank and Copperco both have security interests in the delivered copper sheet. Bank’s
security interest in the copper sheet was perfected by filing. Copperco’s interest, on the other hand, was
not perfected at all. As between the perfected and an unperfected security interests, Bank’s perfected
interest has priority.

Explanation to Point-Two (20%):

Because Kitchenware did not have rights in the undelivered copper sheet, Bank’s security interest in it did
not attach.

One of the elements of an enforceable and attached security interest is that the debtor have “rights in the
collateral.” Thus, Bank has a security interest in the undelivered copper only if Kitchenware has rights in
it. Under the terms of the contract between Copperco and Kitchenware, Kitchenware obtained no rights
(even rights that are short of title) in the undelivered copper (and UCC § 2-401 does not limit Copperco’s

Seperac-J19 Exam-Released MEE Essay Compilation © 2016-2020 1062


rights to a security interest because the goods were neither shipped nor delivered to Kitchenware). Thus,
Kitchenware had no interest in the copper sheet. Accordingly, Bank’s security interest did not attach to
the undelivered copper sheet. Thus, as between Copperco and Bank, only Copperco has an interest in the
undelivered copper sheet.

[NOTE: An applicant might argue that Kitchenware had some rights in the undelivered copper sheet
because, when the copper sheet was identified to the contract by Copperco (when Copperco designated it
for delivery to Kitchenware), this gave Kitchenware a “special property” in the copper sheet, and Bank’s
security interest could attach to that set of rights in the sheet. Limited rights in collateral are sufficient for
attachment even if the rights are less than full ownership. However, that “special property” would not
give Kitchenware (or Bank) any right to claim the goods from Copperco in the absence of payment of the
contract price. Moreover, Article 9 states expressly that a seller with a security interest created by
retention of title under § 2-401 “has priority over a conflicting security interest created by the debtor” in
any case where the debtor has not yet obtained possession of the goods. Here, Copperco has at least a
security interest in the goods under § 2-401. Because the goods were never in Kitchenware’s possession,
Copperco’s security interest will be superior to Bank’s interest, which was created by the debtor,
Kitchenware. An applicant who makes these points should receive some credit.]

Seperac-J19 Exam-Released MEE Essay Compilation © 2016-2020 1063


#245-JUL 2009–MEE Q04: QUESTION FOUR (UCC ART. 9)

Three years ago, Printco, a printing company, borrowed $250,000 from Bank and entered into an
enforceable agreement giving Bank a security interest in “all Printco’s equipment, whether now owned or
hereafter acquired.” Bank promptly filed a proper financing statement in the appropriate state office,
listing itself as the secured party, listing Printco as the debtor, and indicating “all debtor’s equipment,
whether now owned or hereafter acquired” as collateral.

Two years after signing the security agreement with Bank, Printco entered into a signed agreement with
Leaseco, a leasing company, pursuant to which Leaseco agreed to purchase a $100,000 printing press and
to immediately lease the press to Printco. The agreement provided that Leaseco retained title to the
printing press and required Printco to pay Leaseco $2,500 per month for five years for the use of the
press. The agreement also provided that it could not be terminated by Printco for any reason. At the
conclusion of the five-year lease, Printco was required to return the press to Leaseco or to purchase the
press for $10.

Printco’s full compliance with the agreement would allow Leaseco to recover rental payments equal to the
full cost of the press plus an annual return of about 10%.

Leaseco delivered the printing press to Printco.

Bank was unaware of Printco’s agreement with Leaseco, and Leaseco never filed a financing statement
covering the printing press.

Recently, Printco defaulted on its obligations to Bank. At the time of the default, Printco owed $150,000
to Bank. Bank promptly and peacefully took possession of all of the equipment on Printco’s premises,
including the printing press. After giving proper notice of the sale to Printco, Bank sold all of Printco’s
equipment at a public auction for a total of $75,000. Purchaser, acting in good faith, bought the printing
press for $50,000. All aspects of the auction and sale were commercially reasonable.

One week after the sale, Leaseco contacted Bank and informed Bank that the printing press was Leaseco’s
property, not Printco’s, and that Bank had no right to sell it. Leaseco demanded that Bank pay Leaseco
$50,000, the amount that Bank received for the press. Leaseco also contacted Purchaser and demanded
that Purchaser return the press to Leaseco.

1. What is the nature of Leaseco’s interest in the printing press? Explain.

2. Did Bank have a right to repossess and sell the printing press? Explain.

3. As between Bank and Leaseco, which has a superior interest in the proceeds of the sale of the
printing press? Explain.

4. Does Leaseco have the right to recover the printing press from Purchaser? Explain.

Seperac-J19 Exam-Released MEE Essay Compilation © 2016-2020 1064


#245: J09-4 MEE: ANSWER: NCBE (UCC ART. 9)

POINT (1) [30%] ISSUE: What is the nature of Leaseco’s interest in the printing press? ANSWER:
Although Leaseco’s arrangement with Printco was denominated a “lease,” Leaseco actually
transferred ownership of the press to Printco and retained an unperfected security interest.

POINT (2) [25%] ISSUE: Did Bank have a right to repossess and sell the printing press?
ANSWER: Yes. Bank had a perfected security interest in the press and was entitled to repossess
and sell the printing press.

POINT (3) [25%] ISSUE: As between Bank and Leaseco, which has a superior interest in the
proceeds of the sale of the printing press? ANSWER: Because Bank’s security interest was superior
to Leaseco’s, Bank has a superior claim to the proceeds of the sale of the printing press, and it need
not share those proceeds with Leaseco unless the proceeds exceed the amount due to Bank.

POINT (4) [20%] ISSUE: Does Leaseco have the right to recover the printing press from
Purchaser? ANSWER: No. Leaseco cannot recover the printing press from Purchaser because
Bank’s sale of the press to Purchaser discharged Leaseco’s security interest and therefore
eliminated its claim to the press.

ANSWER DISCUSSION:

The “lease” transaction between Leaseco and Printco created a security interest, not a lease. Therefore,
Leaseco’s interest in the press is governed by Article 9. Leaseco’s security interest in the press is
unperfected because it did not file a financing statement or take any other steps to perfect its interest.
Bank also has a security interest in the press by virtue of its security agreement with Printco. Bank’s
security interest is perfected. Bank’s perfected security interest has priority over Leaseco’s unperfected
interest. As the $50,000 recovered by the sale of the press did not fully cover Printco’s debt to Bank,
Bank is entitled to retain the $50,000 proceeds of the sale and Leaseco is not entitled to any recovery from
Bank. Leaseco cannot recover the press from Purchaser. Bank’s proper foreclosure sale of the press to
Purchaser transferred all of Printco’s rights in the press to Purchaser and also discharged both Bank’s and
Leaseco’s security interests.

ANSWER EXPLANATION:

Explanation to Point-One (25-35%):

Although Leaseco’s arrangement with Printco was denominated a “lease,” Leaseco actually transferred
ownership of the press to Printco and retained an unperfected security interest.

Whether a transaction in the form of a lease actually creates a “true lease” or a security interest depends
on the “economic realities” of the transaction, not on the form of the transaction or the supposed intent of
the parties.

In this case, the lease transaction was structured so that Printco was obligated to pay an amount that
would fully cover the cost of the printing press and also ensure Leaseco a 10% return. Moreover, Printco
was not entitled to terminate the lease at any point prior to full performance. If Printco failed to perform,
Leaseco could recover the printing press. If Printco performed, Printco could keep the printing press for

Seperac-J19 Exam-Released MEE Essay Compilation © 2016-2020 1065


the trivial sum of $10. In economic reality, this was not a lease of the printing press at all. It was the
economic equivalent of an installment sale of the press, with Leaseco’s retained title constituting only a
security interest (the right to recover the press if payments were not made, thereby securing Printco’s
obligation to pay the purchase price).

UCC § 1-203 identifies certain situations in which a transaction creates a security interest, not a lease. In
particular, UCC § 1-203(b) explicitly states that “a transaction in the form of a lease creates a security
interest if” lease payments must be made for the full term of the lease and are not subject to termination,
and the lessee has an option to “become the owner of the goods for nominal additional consideration” at
the conclusion of the lease agreement. That is precisely the structure of the Printco-Leaseco contract, and
therefore this transaction should be treated as a sale of the press with Leaseco retaining a security interest.

The fact that the lease agreement provided for title to remain in Leaseco’s name is immaterial. Leaseco’s
interest in the press is a security interest and is governed by Article 9. Because Leaseco did not file a
financing statement, its security interest is unperfected.

[NOTE: Examinees need not know the details of the UCC rules. However, they should realize that the
form of the transaction is not controlling, and that a so-called “lease” will be treated as a security
interest if the “economic realities” so dictate. They should also be able to identify some of the facts that
are relevant to a resolution of this issue.]

Explanation to Point-Two (20-30%):

Bank had a perfected security interest in the press and was entitled to repossess and sell the printing press.

In a signed agreement, Printco gave Bank a security interest in “all Printco’s equipment, whether now
owned or hereafter acquired.” Bank advanced value to Printco by making a loan to Printco. Finally,
Printco obtained rights in the printing press so that Bank’s security interest attached to the press. A
security interest attaches to collateral and is enforceable only if the debtor has rights in the collateral, but
“limited rights in collateral, short of full ownership, are sufficient for a security interest to attach.” Here
Printco had at least the right to possession of the press for the lease term.

In addition, because Bank filed its financing statement covering Printco’s equipment, Bank’s security
interest was perfected.

Following a debtor’s default, a secured party may repossess and sell any collateral that secures its debt.
UCC § 9-609 gives a secured party the right to repossess and UCC § 9-610 gives a secured party the right
to dispose of collateral by sale. As a secured party, Bank had the right to repossess and sell the printing
press.

[NOTE: Moreover, the facts suggest that Bank fully complied with applicable laws: the repossession was
peaceful, and the sale was made at a public auction, in a commercially reasonable manner, and after
notice to the debtor.]

Explanation to Point-Three (20-30%):

Because Bank’s security interest was superior to Leaseco’s, Bank has a superior claim to the proceeds of
the sale of the printing press, and it need not share those proceeds with Leaseco unless the proceeds
exceed the amount due to Bank.

Seperac-J19 Exam-Released MEE Essay Compilation © 2016-2020 1066


Bank had a perfected security interest. Leaseco’s interest was unperfected. A perfected security interest
has priority over an unperfected security interest.

[NOTE: If Leaseco had filed a financing statement within 20 days of delivery of the printing press to
Printco, it would have had a perfected purchase-money security interest with priority over Bank’s general
security interest. The facts state that Leaseco did not make such a filing.]

When a secured party with priority disposes of collateral, the proceeds of that disposition are applied in
the following order: (1) the expenses of the disposition, (2) satisfaction of the obligation owed to the
disposing secured party, and (3) satisfaction of any obligation secured by a subordinate interest.

Because Bank had priority over Leaseco, Bank was entitled to use the proceeds from the sale of the
collateral to cover its expenses and any amounts owed to it by Printco before making any payment to
Leaseco. The amount recovered from the sale of all of Printco’s collateral, including the printing press,
was only $75,000. The amount owed to Bank by Printco, on the other hand, was $150,000 (plus the
expenses of the disposition). Thus, the proceeds of the collateral did not fully satisfy Printco’s obligations
to Bank, and there was no surplus that Leaseco could claim. Leaseco therefore has no valid claim to any
of the proceeds of the sale of the printing press.

Explanation to Point-Four (15-25%):

Leaseco cannot recover the printing press from Purchaser because Bank’s sale of the press to Purchaser
discharged Leaseco’s security interest and therefore eliminated its claim to the press.

A secured party’s disposition of collateral after a debtor’s default transfers the debtor’s rights in the
collateral to any transferee for value, and also discharges the secured party’s interest in the collateral and
“any subordinate security interest.”

Bank sold the printing press to Purchaser for $50,000. As a good-faith buyer of collateral at a foreclosure
sale, Purchaser is a “transferee for value,” and has the rights set out in UCC § 9-617. Thus, Purchaser
takes free of Leaseco’s interest, because Leaseco’s interest is nothing more than a “security interest” that
is subordinate to Bank’s interest.

Seperac-J19 Exam-Released MEE Essay Compilation © 2016-2020 1067


#246-JUL 2008–MEE Q01: QUESTION ONE (UCC ART. 9)

Debtor sells and delivers medical supplies to hospitals. It owns a fleet of 40 trucks that it uses to make
deliveries. Debtor has a state-issued certificate of title for each truck. The state has a statute that provides:
“All security interests in a motor vehicle must be noted on the vehicle’s certificate of title as a condition
for perfection.”

On June 1, Bank made a $100,000 loan to Debtor. In order to secure the loan, Debtor signed a valid
security agreement granting Bank a security interest in “all Debtor’s inventory and equipment, whether
now owned or hereafter acquired.” Bank did not note its security interest on the certificates of title issued
for the trucks. However, Bank immediately filed an appropriate financing statement in the proper state
office that listed its collateral as “all Debtor’s inventory and equipment, whether now owned or hereafter
acquired.”

On July 1, Finance Company loaned $75,000 to Debtor. On that same day, Debtor signed a valid security
agreement granting Finance Company a security interest in 25 of Debtor’s “delivery trucks” and “any
accessories now or hereafter installed.” Each truck was identified individually and by its vehicle
identification number. Although Finance Company never filed a financing statement, its security interest
was noted on the certificate of title for each truck.

On August 1, Debtor entered into a contract with Global Inc. to buy 40 global positioning system units
(GPS units) to be installed on all 40 delivery trucks (including the 25 trucks covered by Finance
Company’s security agreement). GPS units are bolted on the dashboard of a vehicle and hooked up
directly to the battery for power, but they otherwise operate independently of the vehicle.

The GPS units cost $50,000. Debtor made a down payment of $10,000 and signed an agreement to pay
the remaining $40,000, plus interest, in equal monthly installments over a two-year period. To secure the
amount owed on the contract, Debtor signed a security agreement giving Global a security interest in “the
GPS units to be installed on Debtor’s delivery trucks.”

On August 2, Global properly filed a financing statement covering the GPS units. The GPS units were
delivered to Debtor on August 10, and Debtor installed them on its delivery trucks shortly thereafter.

On October 1, Debtor defaulted on its obligations to Bank, Finance Company, and Global. Bank has
repossessed all 40 of Debtor’s trucks, and it is now negotiating with Finance Company and Global about
their respective claims to the trucks and the 40 GPS units installed in them.

1. Which of Bank or Finance Company has the superior claim to the 25 delivery trucks claimed by
Finance Company? Explain.

2. Which of Global or Finance Company has the superior claim to the GPS units installed in the 25
delivery trucks claimed by Finance Company? Explain.

3. Which of Bank or Global has the superior claim to the remaining 15 GPS units? Explain.

Seperac-J19 Exam-Released MEE Essay Compilation © 2016-2020 1068


#246: J08-1 MEE: ANSWER: NCBE (UCC ART. 9)

POINT (1) [35%] ISSUE: Does a prior security interest in equipment that is perfected by filing
prevail over a later security interest that is perfected by notation on a certificate of title when the
equipment in question is vehicles subject to a certificate-of-title statute? ANSWER: No. Finance
Company has a superior claim to the 25 vehicles in which it has a security interest because its
interest is perfected and Bank’s is not. Where a motor vehicle certificate-of-title statute calls for
perfection by notation on the certificate of title, the filing of a financing statement will not perfect
an interest in the vehicles.

POINT (2)(a) [15%] ISSUE: Does a security interest in goods continue after the goods are attached
to other goods in which someone else has a security interest? ANSWER: Yes. Once the GPS units
were attached to Debtor’s delivery trucks, they became accessions to those motor vehicles.
Nonetheless, Global’s interest in the GPS units continues after their attachment to the delivery
trucks.

POINT (2)(b) [15%] ISSUE: Does a properly perfected purchase-money security interest in goods
that are accessions to motor vehicles prevail over a prior security interest in the whole vehicle that
is perfected by notation on the certificate of title? ANSWER: Finance Company’s interest in the
delivery trucks and installed GPS units is superior to Global’s interest in the GPS units.

POINT (3) [35%] ISSUE: Does a properly perfected purchase-money security interest in goods that
are equipment prevail over a prior perfected security interest in the same equipment? ANSWER:
Yes. Global’s purchase-money security interest prevails over Bank’s conflicting security interest in
the remaining GPS units.

ANSWER DISCUSSION:

Bank’s filing of a financing statement was ineffective to perfect its interest in Debtor’s delivery trucks
because the trucks were covered by a certificate-of-title statute. Finance Company’s interest is therefore
superior to Bank’s because Finance Company properly perfected its interest by notation on the trucks’
certificates of title. Global’s security interest in the GPS units continued after the units were attached to
the trucks, but that interest will be subordinate to Finance Company’s interest if Finance Company’s
security interest in “delivery trucks” is interpreted to include the GPS units installed in the trucks. Bank’s
security interest in the GPS units installed in the remaining 15 trucks is a perfected security interest in
“equipment.” However, its interest is subordinate to Global’s perfected purchase-money security interest
in the same equipment.

ANSWER EXPLANATION:

Explanation to Point-One (30-40%):

Finance Company has a superior claim to the 25 vehicles in which it has a security interest because its
interest is perfected and Bank’s is not. Where a motor vehicle certificate-of-title statute calls for
perfection by notation on the certificate of title, the filing of a financing statement will not perfect an
interest in the vehicles.

Seperac-J19 Exam-Released MEE Essay Compilation © 2016-2020 1069


Debtor’s fleet of delivery trucks is “equipment” for purposes of Article 9. Bank has a security interest in
that equipment. However, the facts tell us that the delivery trucks are subject to a certificate-of-title statute
that provides for perfection by notation of a security interest on the certificate of title. Filing a financing
statement is “not effective to perfect a security interest in property subject to” such a statute. Thus, Bank
did not perfect its interest in Debtor’s delivery trucks by filing a financing statement.

Finance Company, on the other hand, did perfect its interest in the 25 trucks by complying with the
applicable certificate-of-title statute and noting its interest on the certificates of title. Thus, Finance
Company’s perfected security interest in the 25 trucks is superior to Bank’s unperfected interest.

Explanation to Point-Two(a) (10-20%):

Once the GPS units were attached to Debtor’s delivery trucks, they became accessions to those motor
vehicles. Nonetheless, Global’s interest in the GPS units continues after their attachment to the delivery
trucks.

Where, as here, “goods are physically united with other goods in such a manner that the identity of the
original goods is not lost,” the goods become accessions. A security interest in goods that is created and
perfected before the goods become accessions continues after the goods become accessions. Thus,
Global’s perfected security interest in the GPS units continued despite the fact that the units were attached
to Debtor’s trucks.

When the collateral of one creditor becomes united with the collateral of another, each creditor’s
collateral is an “accession” to the other creditor’s collateral, and the two items of collateral together are
regarded as “the whole.” Whether either creditor’s security interest applies to “the whole,” or applies only
to its original collateral, “turns on the description of the collateral in that creditor’s security agreement.”

In this case, Global’s security interest in the GPS units certainly does not cover “the whole” (i.e., delivery
trucks with GPS units installed).

On the other hand, Finance Company’s described security interest in the specifically identified delivery
trucks expressly includes all installed accessories and therefore covers the GPS units.

Explanation to Point-Two(b) (10-20%):

Finance Company’s interest in the delivery trucks and installed GPS units is superior to Global’s interest
in the GPS units.

The priority rules governing accessions are normally the same as the rules for other collateral. However,
as it does in other places, Article 9 makes an exception when, as here, there is an applicable certificate-of-
title statute. UCC § 9-335 provides that “a security interest in an accession is subordinate to a security
interest in the whole which is perfected by compliance with the requirements of a certificate-of-title
statute.”

Thus, Global’s security interest in the GPS units installed in the trucks claimed by Finance Company is
subordinate to Finance Company’s claim, which was perfected by notation on the trucks’ certificates of
title. The policy rationale for preferring Finance Company in these circumstances is to “enable a secured
party to rely upon a certificate of title without having to check the UCC files to determine whether any
components of the collateral may be encumbered.”

Seperac-J19 Exam-Released MEE Essay Compilation © 2016-2020 1070


Explanation to Point-Three (30-40%):

Global’s purchase-money security interest prevails over Bank’s conflicting security interest in the
remaining GPS units.

Bank’s perfected security interest covers inventory and equipment, including after-acquired equipment
and inventory. The GPS units are “goods” that are properly classified as equipment because they are not
inventory, farm products, or consumer goods. The after-acquired collateral clause in Bank’s security
agreement is valid, and Bank’s priority dates to the time it filed its financing statement covering
equipment, even though its interest in the GPS units did not attach or perfect until Debtor acquired rights
in the units. Thus, Bank has a perfected security interest in the GPS units.

Global, however, has a purchase-money security interest in the GPS units. A security interest in goods is a
purchase-money security interest if the collateral “secures a purchase-money obligation incurred with
respect to that collateral.” Global’s interest in the GPS units secures a purchase-money obligation – the
$40,000 debt to Global that Debtor incurred “as all or part of the price of the collateral.” Hence, the
security interest is a purchase-money security interest. Global promptly filed a financing statement and its
interest was perfected as soon as Debtor obtained possession of the GPS units.

A purchase-money security interest in equipment that is perfected when the debtor takes possession of the
collateral prevails over a conflicting perfected interest in the same equipment, even if the conflicting
interest is earlier in time. Thus, although Bank’s priority dates to June 1, its interest is nonetheless
subordinate to Global’s later-in-time purchase-money security interest.

Seperac-J19 Exam-Released MEE Essay Compilation © 2016-2020 1071


#247-FEB 2008–MEE Q07: QUESTION SEVEN (UCC ART. 9)

On October 1, Dart Corporation (DC), a State A corporation, borrowed $300,000 from State Bank, a State
A bank. On behalf of DC, DC’s president signed a written security agreement giving State Bank a
security interest in DC’s State Bank checking account (a demand account) to secure its obligation to repay
the loan. State Bank did not file a financing statement reflecting that interest.

On December 1, First Bank, another State A bank, agreed to loan $60,000 to DC. DC’s president, on
behalf of DC, signed a written security agreement granting First Bank a security interest in all of DC’s
“office equipment and deposit accounts.” First Bank immediately advanced $60,000 to DC and filed a
financing statement reflecting its interest.

First Bank’s financing statement was filed in the correct filing office in State A and listed the collateral as
“office equipment and deposit accounts.” It correctly identified First Bank as the secured party, and it
correctly gave the address of the debtor as “123 Smith Street, City, State A.” However, First Bank’s
financing statement incorrectly listed the name of the debtor as “Dart Incorporated,” rather than “Dart
Corporation,” the correct name of the company as reflected on its certificate of incorporation and other
public records. Despite this error, a search under the name “Dart Corporation” using the State A filing
office’s standard search logic would turn up the financing statement listing “Dart Incorporated” as the
debtor.

Shortly after receiving the loan from First Bank, DC defaulted on the obligations it owed to State Bank
and First Bank.

On December 23, State Bank obtained a judgment against DC in connection with its unpaid $300,000
loan.

On January 3 of the following year, a State A sheriff levied on DC’s office equipment on behalf of State
Bank.

1. As between First Bank and State Bank, which has a superior claim to DC’s checking account at
State Bank? Explain.

2. As between First Bank and State Bank, which has a superior claim to DC’s office equipment?
Explain.

Seperac-J19 Exam-Released MEE Essay Compilation © 2016-2020 1072


#247: F08-7 MEE: ANSWER: NCBE (UCC ART. 9)

POINT (1) [50%] ISSUE: As between the bank in which a deposit account is maintained and a
bank that has filed a financing statement covering the account, whose security interest in the
account has priority? ANSWER: First Bank did not have control of DC’s deposit account at State
Bank. As a result, First Bank’s interest is unperfected and is subordinate to State Bank’s perfected
interest, even though First Bank filed a financing statement and State Bank did not.

POINT (2)(a) [25%] ISSUE: Is a financing statement effective to perfect a security interest when
the financing statement does not give the debtor’s correct name, but a search under the correct
name would reveal the financing statement? ANSWER: Yes. First Bank’s security interest in DC’s
office equipment is perfected despite the error in DC’s name on the financing statement.

POINT (2)(b) [25%] ISSUE: Does a perfected security interest have priority over a judicial lien that
has attached to collateral? ANSWER: Yes. First Bank’s perfected security interest in the office
equipment is superior to State Bank’s judgment lien.

ANSWER DISCUSSION:

A security interest in a deposit account is perfected only if the secured party has control of the account.
First Bank did not have control of the deposit account; State Bank did. Thus, State Bank’s interest is the
only perfected interest, and State Bank’s claim is superior to First Bank’s claim. First Bank’s interest in
Dart Corporation’s office equipment was perfected even though its financing statement gave an incorrect
name for Dart Corporation (DC). Because a search of the filing office records using DC’s correct name
would disclose the financing statement, it is effective to perfect First Bank’s interest. Because First Bank
is a perfected secured party, its interest prevails over the interest of State Bank, a judgment creditor whose
lien attached after First Bank had perfected its interest.

ANSWER EXPLANATION:

Explanation to Point-One (45-55%):

First Bank did not have control of DC’s deposit account at State Bank. As a result, First Bank’s interest is
unperfected and is subordinate to State Bank’s perfected interest, even though First Bank filed a financing
statement and State Bank did not.

DC’s checking account at State Bank is a “deposit account” for Article 9 purposes. The facts state that the
account is a demand account, and demand accounts are Article 9 “deposit accounts.” Both State Bank and
First Bank have enforceable security interests in the account because the following criteria are fulfilled:
(1) DC’s president signed a security agreement granting each bank an interest in the account, (2) DC has
rights in the account, and (3) each bank extended value to DC by giving DC a loan.

Although First Bank filed a financing statement covering the deposit account, First Bank’s interest is not
perfected. Under UCC § 9-312, “a security interest in a deposit account may be perfected only by
control.” Under UCC § 9-104, a creditor only has control of a deposit account if: (1) “the secured party is
the bank with which the deposit account is maintained” (the bank account is at State Bank, not First
Bank), (2) the bank where the account is held has agreed in writing to follow the instructions of the
secured party (no such agreement exists), or (3) the secured party becomes the bank’s customer with

Seperac-J19 Exam-Released MEE Essay Compilation © 2016-2020 1073


respect to the account (which was not done). Because none of the UCC § 9-104 requirements for control
has been met by First Bank, First Bank’s security interest in the bank account is unperfected.

State Bank, on the other hand, does have a perfected security interest in the deposit account, even though
it did not file a financing statement, because State Bank is “the bank with which the deposit account is
maintained,” and therefore has control of the account. As between an unperfected secured creditor (First
Bank) and a perfected secured creditor (State Bank), the perfected secured creditor has the superior claim
to the collateral.

Explanation to Point-Two(a) (20-30%):

First Bank’s security interest in DC’s office equipment is perfected despite the error in DC’s name on the
financing statement.

First Bank filed a financing statement covering DC’s office equipment. For a financing statement to be
effective under UCC § 9-502, it must contain three pieces of information: (1) the name of the debtor, (2)
the name of the secured party, and (3) an indication of the collateral. In the case of First Bank’s financing
statement, the name of the secured party is correct and the collateral is sufficiently indicated as “office
equipment.” (DC’s office equipment would be categorized as “equipment” under Article 9. ) The name of
the debtor, however, contains an error. The issue is whether this error renders the financing statement
ineffective.

Under UCC § 9-506, minor errors do not render a financing statement ineffective, unless those errors
make the financing statement “seriously misleading.” However, a corporation is a registered organization,
and under § 9-503, a financing statement sufficiently states the name of a “registered organization” only if
the financing statement provides the name of the organization that is “indicated on the public record” that
“shows the debtor to have been organized.” Here, the financing statement should have stated the debtor’s
name as “Dart Corporation.” Moreover, under § 9-506, “a financing statement that fails sufficiently to
provide the name of the debtor in accordance with Section 9-503 is seriously misleading.” Thus, First
Bank’s failure to include DC’s correct name on the financing statement raises the possibility that the
financing statement will be deemed “seriously misleading” and therefore ineffective.

Luckily for First Bank, however, UCC § 9-506 provides a safe harbor that deals with this situation. Even
where the debtor’s name is incorrect on the financing statement,

“If a search of the records of the filing office under the debtor’s correct name, using the filing office’s
standard search logic, if any, would disclose a financing statement that otherwise fails to provide the name
of the debtor the name provided does not make the financing statement seriously misleading.”

The facts state that a search in the State A filing office using DC’s correct name, “Dart Corporation,” will
turn up the filed financing statement, even though it incorrectly states the name of the debtor as “Dart
Incorporated.” As a result, First Bank’s financing statement is not “seriously misleading” under the
relevant Article 9 provisions, and First Bank’s interest is perfected.

Explanation to Point-Two(b) (20-30%):

First Bank’s perfected security interest in the office equipment is superior to State Bank’s judgment lien.

First Bank’s claim to the office equipment is superior to State Bank’s claim. First Bank is a perfected
secured creditor, and State Bank is a judgment lien creditor. Under UCC § 9-317, a judgment lien creditor

Seperac-J19 Exam-Released MEE Essay Compilation © 2016-2020 1074


takes priority over a security interest only if the creditor becomes a “lien creditor” before the conflicting
security interest is perfected. State Bank became a “lien creditor” when the sheriff levied on January 3.
First Bank, however, perfected its security interest in the office equipment by filing on December 1, a
month earlier.

Seperac-J19 Exam-Released MEE Essay Compilation © 2016-2020 1075


#248-FEB 2007–MEE Q07: QUESTION SEVEN (UCC ART. 9)

Nine months ago, Feagle Construction Company, Inc. (Feagle), which does business under the trade name
of On Top Roofing, obtained a $150,000 loan from National Bank. To secure the loan, Feagle assigned to
National Bank “all rights to payment owed to Feagle Construction Company, Inc. by Hotel Corporation
for the roofing construction project on its Broadway Street Hotel.” The Broadway Street Hotel roofing
project was by far the largest of Feagle’s fifteen roofing projects and represented its largest account
receivable.

National Bank promptly filed a financing statement with this same collateral description, using the trade
name of On Top Roofing for the debtor. The financing statement was filed in the appropriate location, and
indexed only in the name of On Top Roofing.

Three months after these events, Feagle needed additional monies. It approached State Bank for a loan of
$100,000. State Bank conducted a search of the filing office’s records for financing statements relating to
“Feagle Construction Company, Inc.,” but its search did not retrieve National Bank’s financing statement.
Accordingly, State Bank granted the $100,000 loan and obtained a written agreement from Feagle
granting State Bank a security interest in “all Feagle Construction Company, Inc.’s accounts, whether
now owned or hereafter acquired.” State Bank filed a financing statement in the appropriate location in
the name of “Feagle Construction Company, Inc.” as debtor.

Another six months later, mounting financial stress forced Feagle to default on its loans to both National
Bank and State Bank. Feagle has about $75,000 worth of outstanding accounts receivable. In particular,
Hotel Corporation still owes Feagle $50,000 for the hotel roofing project. In addition, Feagle is owed
about $25,000 on a total of ten other roofing projects it has recently completed. Both National Bank and
State Bank are seeking to recover some of what Feagle owes them by collecting from Hotel Corporation
the amount it still owes Feagle.

Does National Bank or State Bank have first priority in the money owed to Feagle by Hotel Corporation?
Explain.

Seperac-J19 Exam-Released MEE Essay Compilation © 2016-2020 1076


#248: F07-7 MEE: ANSWER: NCBE (UCC ART. 9)

POINT (1) [42%] ISSUE: Was the financing statement filed by National Bank sufficient to perfect
its security interest in the monies owed to Feagle by Hotel Corporation? ANSWER: Yes. Security
interests in accounts may be perfected by a financing statement that complies with Code
requirements; a filing in the name of a debtor’s trade name instead of its official corporate name is
not effective unless the trade name is so similar that it is not seriously misleading.

POINT (2) [42%] ISSUE: Was National Bank’s security interest in the monies owed to Feagle by
Hotel Corporation perfected automatically because National Bank did not receive a “significant
part” of Feagle’s accounts? ANSWER: No. A security agreement in accounts may be perfected
automatically if the assignment of accounts did not transfer a significant part of the assignor’s
outstanding accounts to the creditor. Given that the Hotel Corporation account appears to
constitute a significant portion of all Feagle’s outstanding accounts, and considering that National
Bank is a professional lender, not a casual or isolated lender, National Bank’s security interest will
probably not be considered to have been automatically perfected.

POINT (3) [16%] ISSUE: As between National Bank’s unperfected interest and State Bank’s
perfected interest, whose interest prevails? ANSWER: State Bank has first priority because its
interest is perfected while National Bank’s is not.

ANSWER DISCUSSION:

National Bank’s financing statement was insufficient to perfect its interest in the Hotel Corporation
account because the financing statement was filed under Feagle’s trade name rather than under its true
corporate name. Moreover, the facts suggest that the Hotel Corporation account constituted a significant
part of Feagle’s accounts, thus precluding National Bank’s interest from being perfected automatically.
By contrast, State Bank filed a proper financing statement and had a perfected security interest in Feagle’s
accounts, including the Hotel Corporation account. State Bank’s perfected security interest has priority
over National Bank’s unperfected interest, even though State Bank acquired its interest three months after
National Bank.

ANSWER EXPLANATION:

Explanation to Point-One (35-45%):

Security interests in accounts may be perfected by a financing statement that complies with Code
requirements; a filing in the name of a debtor’s trade name instead of its official corporate name is not
effective unless the trade name is so similar that it is not seriously misleading.

National Bank obtained a security interest in the monies owed by Hotel Corporation to Feagle by having
Feagle sign an agreement granting National Bank an interest in the specified collateral. That collateral –
the monies owed to Feagle by Hotel Corporation – is an “account” for purposes of Article 9 of the UCC.(
a “right to payment for services rendered or to be rendered” is an “account”). A security interest in
accounts must ordinarily be perfected by filing a proper financing statement in the appropriate
government office.

Seperac-J19 Exam-Released MEE Essay Compilation © 2016-2020 1077


In this problem, National Bank filed a financing statement, but the financing statement was ineffective. In
order to be effective, a financing statement must sufficiently provide the name of the debtor. Where the
debtor is a registered organization such as a corporation, the financing statement must use the official
registered corporate name of the debtor. A financing statement that provides only the debtor’s trade name
does not sufficiently provide the name of the debtor.

Although the UCC forgives minor errors that do not render the financing statement seriously misleading,
an error in the debtor’s name is seriously misleading unless a search of the records under the debtor’s
correct name (Feagle Construction Company, Inc.) using the filing office’s standard search logic would
disclose the financing statement. Because of the complete dissimilarity between the trade name (On Top
Roofing) under which the financing statement was filed and the debtor’s correct name (Feagle), this
financing statement would be found ineffective and National Bank’s security interest will not be perfected
by filing.

Explanation to Point-Two (35-45%):

A security agreement in accounts may be perfected automatically if the assignment of accounts did not
transfer a significant part of the assignor’s outstanding accounts to the creditor. Given that the Hotel
Corporation account appears to constitute a significant portion of all Feagle’s outstanding accounts, and
considering that National Bank is a professional lender, not a casual or isolated lender, National Bank’s
security interest will probably not be considered to have been automatically perfected.

In addition to providing for perfection of a security interest in accounts by filing, the UCC provides for
perfection upon attachment (known as automatic perfection) for assignments of accounts that do not
“transfer a significant part of the assignor’s outstanding accounts” to the secured party. Under the facts
given, National Bank received an assignment of only one of Feagle’s several outstanding accounts.
However, the single Hotel Corporation account was, at the time of default, worth more than all Feagle’s
other accounts combined. The question, then, is whether this assignment of a single account was a transfer
of “a significant part” of Feagle’s outstanding accounts.

The UCC gives no guidance on the meaning of “significant part,” but National Bank will have a difficult
time arguing that an account amounting to more than 50 percent of the total value of Feagle’s accounts is
not significant. Moreover, the comments to UCC § 9-309 suggest that its purpose is to protect “casual or
isolated assignments which no one would think of filing,” and that persons who “regularly” take
assignments should file. National Bank took the assignment from Feagle as a part of a normal secured
financing arrangement; it was a standard business transaction of the sort that almost always includes the
filing of a financing statement. Indeed, National Bank did file (albeit ineffectively).

On these facts it is highly likely that a court would conclude that National Bank’s interest was not
automatically perfected.

Explanation to Point-Three (10-20%):

State Bank has first priority because its interest is perfected while National Bank’s is not.

A perfected security interest has priority over a conflicting unperfected interest. Because National Bank’s
interest was unperfected and State Bank’s interest was perfected, State Bank has priority under the UCC.

[NOTE: If an applicant mistakenly concludes that National Bank’s interest was perfected either by filing
or automatically, then the applicant should conclude that National Bank would have priority over State

Seperac-J19 Exam-Released MEE Essay Compilation © 2016-2020 1078


Bank under the “first to file or perfect” rule. Consider giving full credit for Point Three in this
circumstance, at least if the applicant’s answer shows a full understanding of the priority rules and is
wrong only because the applicant erroneously concluded that National Bank’s interest was perfected.]

Seperac-J19 Exam-Released MEE Essay Compilation © 2016-2020 1079


#249-JUL 2006–MEE Q07: QUESTION SEVEN (UCC ART. 9)

Joe purchased a boat for use by his family from Dealer. Dealer extended credit for the purchase and
retained a security interest in the boat as collateral for payment of the balance due. Joe failed to make a
number of the payments, and, on May 1, Dealer peacefully repossessed the boat as Dealer was entitled to
do under the UCC.

On May 3, Dealer mailed Joe a letter that stated the following: the balance due on the boat was $10,000;
Dealer had credited Joe with $7,500, Dealer’s good faith estimate of the boat’s value, leaving a deficiency
balance due of $2,500; Dealer had disposed of the boat by purchasing it for use as a rental boat in
Dealer’s marina; and Dealer had saved Joe money by disposing of the boat in this way because, if Dealer
had publicized the availability of the boat for sale to third parties, Joe would have had to pay the
additional cost of the publicity. The letter also demanded that Joe pay the $2,500 deficiency.

This letter was the only communication between Dealer and Joe after the peaceful repossession of the
boat.

Used boats are typically sold through privately negotiated sales or in occasional dealer auctions. There is
no recognized market on which they are customarily sold. Moreover, used boats are not subject to
standardized price quotations. Quoted pricing is widely variable, with used boats of the same age and
model being subject to different prices depending on their condition and features. Sale prices are also
heavily influenced by the negotiating skills of the parties to the transaction. Nonetheless, the $7,500 value
that Dealer placed on Joe’s boat was in the middle of the range of prices for which used boats of that age
and model have sold in the area over the past year.

1. Did Dealer properly dispose of the collateral under the UCC? Explain.

2. On these facts, what are Joe’s rights, remedies, and obligations under the UCC? Explain.

Seperac-J19 Exam-Released MEE Essay Compilation © 2016-2020 1080


#249: J06-7 MEE: ANSWER: NCBE (UCC ART. 9)

POINT (1) [37%] ISSUE: Did Dealer properly dispose of the collateral? ANSWER: No. Dealer’s
disposition of the collateral failed to comply with UCC requirements in two respects. First, Dealer
purchased the boat in a private sale, which was inappropriate under the circumstances. Second,
Dealer failed to give Joe advance notice of the sale.

POINT (2)(a) [11%] ISSUE: Is the remedy of actual damages available to Joe as a result of Dealer’s
failure to properly enforce Dealer’s security interest? ANSWER: Yes. Joe has the remedy of actual
damages due to Dealer’s failure to properly enforce Dealer’s security interest.

POINT (2)(b) [11%] ISSUE: Is the remedy of statutory damages available to Joe as a result of
Dealer’s failure to properly enforce Dealer’s security interest? ANSWER: Yes. Because this is a
consumer transaction, Joe may recover “statutory damages” even if he suffered no actual damages.

POINT (2)(c) [11%] ISSUE: Is the remedy of a court-ordered alternative disposition of the
collateral available to Joe as a result of Dealer’s failure to properly enforce Dealer’s security
interest? ANSWER: Yes. Given that Dealer is still in possession of the collateral, a court could
order Dealer to dispose of the collateral by a proper sale or even allow Joe to exercise his right of
redemption.

POINT (3) [32%] ISSUE: Is Joe responsible for the $2,500 deficiency? ANSWER: Joe’s liability for
a deficiency depends on whether the jurisdiction adopts the absolute bar rule or the rebuttable
presumption rule.

ANSWER DISCUSSION:

Dealer was required to dispose of the collateral in a “commercially reasonable” manner, which involves
giving Joe reasonable advance notice of a public sale after adequate publicity preceding the sale. Here,
Dealer’s purchase of the boat for Dealer’s own use without such advance notice and in a private sale did
not satisfy the requirements for a commercially reasonable sale. Where there has been a noncomplying
disposition of consumer goods (the boat was purchased for Joe’s family use, which makes it a consumer
good), the consumer is entitled to recover actual damages and, in any event, a minimum statutory damage
recovery. Because Dealer is still in possession of the boat, a court might also order Dealer to dispose of it
through a proper public sale or even to permit Joe to redeem it. Joe may or may not have the obligation to
pay the deficiency. In consumer transactions, there is no “Code” rule. Some courts follow the absolute bar
rule, under which a noncomplying sale completely bars recovery of a deficiency. Other courts adopt a
rebuttable presumption that a complying sale would have produced proceeds sufficient to pay the existing
debt, but the creditor is entitled to a deficiency judgment if it can rebut the presumption and prove that a
complying sale would have produced less than the amount due.

ANSWER EXPLANATION:

Explanation to Point-One (30-40%):

Dealer’s disposition of the collateral failed to comply with UCC requirements in two respects. First,
Dealer purchased the boat in a private sale, which was inappropriate under the circumstances. Second,
Dealer failed to give Joe advance notice of the sale.

Seperac-J19 Exam-Released MEE Essay Compilation © 2016-2020 1081


Under the UCC, every aspect of a disposition must be commercially reasonable. In addition, the UCC sets
out some specific standards governing the disposition of collateral. One of these standards is the rule that
a secured party may purchase the collateral only if the secured party buys it

(1) at a public disposition; or

(2) at a private disposition only if the collateral is of a kind that is customarily sold on a recognized
market or the subject of widely distributed standard price quotations.

Neither requirement was met here.

First, Dealer purchased the collateral in a private, not a public, disposition. Public dispositions are those
with some form of advertisement or public notice preceding the sale and where the public has access to
the sale. There is no suggestion that Dealer sought any third-party bids for the boat or made any public
offer to sell it. Second, used boats are not sold under circumstances that would make a private sale
appropriate. The facts make clear that used boats are not sold on a recognized market or pursuant to
standard price quotations. To the contrary, boat prices fluctuate based on many factors and are subject to
individual bids and negotiations. Thus, Dealer’s private purchase of the boat was improper.

Dealer’s disposition was also improper because Dealer did not inform Joe of the disposition until after it
had occurred. The UCC requires a secured party to provide notice to the debtor before disposing of
collateral. A proper notice must describe the intended disposition and must be sent within a reasonable
time before the disposition. Under the facts, Joe was never informed of the impending disposition. The
only notice he received was after the fact. This failure to notify was also a violation of the UCC.

[NOTE: The facts state that Dealer “disposed of the boat by purchasing it,” which is one manner of
disposition of collateral under UCC § 9-610. The problem is designed to force applicants to discuss the
standards for disposition of collateral and whether Dealer met them. However, some applicants may
incorrectly treat Dealer’s actions as an “acceptance of collateral in full or partial satisfaction of the
obligation,” rather than as a disposition. Those applicants could receive some credit if their critique of
Dealer’s actions addresses the central problems of lack of notice and the commercial unreasonableness
of Dealer’s purchasing this type of collateral from itself in a private sale.]

Explanation to Point-Two(a): (05-15%):

Joe has the remedy of actual damages due to Dealer’s failure to properly enforce Dealer’s security
interest.

Joe may recover any actual damages.

Dealer is liable for actual damages in the amount of any loss caused by failing to comply with the UCC
rules for notice of disposition and a commercially reasonable disposition. Damages are “those reasonably
calculated to put an eligible claimant in the position that it would have occupied had no violation
occurred.” The most likely measure of Joe’s actual loss is the difference between the amount credited to
Joe by Dealer and the amount that would have been obtained in a sale that complied with UCC
requirements. Under these facts, however, Joe would probably have trouble proving that a proper sale
would have yielded a significantly better price for the boat.

Explanation to Point-Two(b): (05-15%):

Seperac-J19 Exam-Released MEE Essay Compilation © 2016-2020 1082


Because this is a consumer transaction, Joe may recover “statutory damages” even if he suffered no actual
damages.

Where the collateral is consumer goods, as here, the debtor is guaranteed a minimum recovery of
“statutory damages” in an amount not less than the credit service charge (the interest payable on the loan)
plus 10 percent ($1,000) of the principal amount of the loan. The purpose of this remedy is to “ensure that
every noncompliance (with UCC requirements relating to the disposition of collateral) in a consumer
goods transaction results in liability, regardless of any injury that may have resulted.”

Explanation to Point-Two(c): (05-15%):

Given that Dealer is still in possession of the collateral, a court could order Dealer to dispose of the
collateral by a proper sale or even allow Joe to exercise his right of redemption.

Where a secured party is disposing of collateral improperly, a court “may order or restrain disposition on
appropriate terms and conditions.” Given the absence of any third-party purchaser of the collateral, a
court might void Dealer’s improper sale to itself and order Dealer to sell the collateral by a proper public
disposition.

In addition, Joe has a right to redeem repossessed collateral by paying all amounts owed along with
Dealer’s repossession expenses. This redemption right is usually extinguished by a proper disposition of
collateral. But a court might allow Joe to redeem under these circumstances (if he wishes to do so and can
pay the required amounts), given the impropriety of the private sale by Dealer to itself and the fact that the
boat is still in Dealer’s possession.

Explanation to Point-Three (25-35%):

Joe’s liability for a deficiency depends on whether the jurisdiction adopts the absolute bar rule or the
rebuttable presumption rule.

Although ordinarily the secured party may recover a deficiency remaining after disposition, Joe has
several arguments to reduce or eliminate the claimed deficiency of $2,500.

Under UCC § 9-626(b), the impact of noncompliance with Article 9 on recovery of a deficiency in a
consumer goods transaction is left to the court to determine. Courts tend to follow two approaches in
consumer transactions. Under the so-called “absolute bar” rule, the creditor’s noncompliance bars
recovery of any deficiency. Under the “rebuttable presumption” rule (adopted in UCC § 9-626 for
commercial transactions), the creditor’s noncompliance results in a presumption that a complying
disposition would have realized an amount of proceeds sufficient to cover the entire debt and expenses
(and hence that there is no deficiency), but the creditor is given the opportunity to rebut this presumption
by proving that even a complying sale would have realized less than the full amount due.

If the absolute bar rule is followed, Joe owes nothing to Dealer. If the rebuttable presumption rule is
followed, Joe may be liable for a deficiency if Dealer can prove that a proper disposition of the collateral
would have yielded less than what Joe owed (plus expenses). The facts state that the price the Dealer
credited to Joe was within a range of likely prices, so it is possible that Dealer could show that even a
proper sale would have yielded less than the amount owed on the boat.

[NOTE: It is possible that Joe would be permitted both to recover his statutory damages and to have any
deficiency eliminated through the application of one of these rules. Although double recovery or over-

Seperac-J19 Exam-Released MEE Essay Compilation © 2016-2020 1083


compensation of this sort is forbidden in a non-consumer context, the Code “is silent as to whether a
double recovery or other over-compensation is possible in a consumer transaction.” Some courts have
therefore concluded that denial of deficiency does not preclude a consumer from also recovering
statutory damages.]

Seperac-J19 Exam-Released MEE Essay Compilation © 2016-2020 1084


#250-FEB 2006–MEE Q06: QUESTION SIX (UCC ART. 9)

Specialty Audio, Inc. (Specialty) manufactures high-quality stereo speakers worth about $2,000 each.
Because its business is fairly small, Specialty has no showroom or sales staff of its own.

Specialty entered into a consignment arrangement with Giant Electronics Store (Giant). Pursuant to the
written consignment agreement, Specialty delivers speakers to Giant, and Giant displays those speakers
on its showroom floor and sells them on behalf of Specialty. When a customer buys a pair of speakers,
Giant keeps a commission as compensation for Giant’s effort and pays the rest of the customer’s purchase
price to Specialty. At any given time, Giant has about two dozen pairs of Specialty’s speakers on hand.

Specialty holds title to the speakers until they are sold. However, as far as any third party can discern, the
speakers are part of Giant’s own inventory. There is no indication that Specialty is the owner of the
speakers. Moreover, Giant does not generally engage in the sale of goods for others and is not known by
its creditors to do so.

After establishing the consignment relationship with Specialty, Giant borrowed $1 million from Bank to
open new stores. As security for its obligation to repay the money, Giant granted Bank a security interest
in “all inventory, whether now owned or hereafter acquired.” Bank properly perfected its security interest
by filing a financing statement in the appropriate state office.

Giant defaulted on its loan from Bank. Bank peaceably repossessed all of Giant’s goods, including all the
Specialty speakers Giant had on display and, after providing proper notice, sold everything at a
commercially reasonable sale.

1. Did Bank have the right to repossess and sell the Specialty speakers even though, under the
consignment agreement, they belonged to Specialty and not to Giant? Explain.

2. What rights, if any, does Specialty have against Bank to recover the proceeds from the sale of
the speakers? Explain.

3. What action might Specialty have taken to protect its interest in the speakers more completely?
Explain.

Seperac-J19 Exam-Released MEE Essay Compilation © 2016-2020 1085


#250: F06-6 MEE: ANSWER: NCBE (UCC ART. 9)

POINT (1) [40%] ISSUE: Does a secured party’s collateral include property that has been
consigned to the debtor? ANSWER: Yes. Bank’s collateral includes the Specialty speakers, even
though they are not owned by Giant, because Giant is deemed to have all the rights and title in the
speakers that Specialty has, and Giant gave Bank a security interest that covered the speakers.

POINT (2) [30%] ISSUE: What is the priority between the unperfected security interest of a
consignor and the perfected security interest of an inventory lender, and how does that priority
affect the consignor after foreclosure? ANSWER: Specialty is treated as having a security interest
in the speakers, and this interest is unperfected. Accordingly, Specialty’s interest is junior to Bank’s
perfected security interest, and it is therefore extinguished upon Bank’s foreclosure.

POINT (3) [30%] ISSUE: How can a consignor achieve priority over an inventory lender?
ANSWER: Specialty could have achieved priority over Bank by promptly filing a financing
statement against Giant.

ANSWER DISCUSSION:

The effect of Specialty’s consignment of the speakers to Giant and Giant’s grant of a security interest in
“inventory” is that Bank acquired a security interest in the speakers. It does not matter that Specialty
retained title to the speakers. Specialty’s interest in the speakers is a purchase money security interest
under Article 9. Because Specialty did not file a financing statement to perfect its interest, the interest is
junior to Bank’s. Specialty’s interest could have been made senior to Bank’s security interest if Specialty
had filed a financing statement when it delivered the speakers to Giant.

ANSWER EXPLANATION:

Explanation to Point-One (35-45%):

Bank’s collateral includes the Specialty speakers, even though they are not owned by Giant, because
Giant is deemed to have all the rights and title in the speakers that Specialty has, and Giant gave Bank a
security interest that covered the speakers.

Bank had a security interest in Giant’s inventory. Ordinarily, Bank’s security interest would only attach to
property in which the debtor (Giant) had rights, and it would only be effective to the extent of Giant’s
rights. Under the terms of the consignment arrangement, the Specialty speakers are not owned by Giant
(Specialty retains title), and accordingly, Giant would not ordinarily have ownership rights in them for
purposes of attachment. However, UCC § 9-319 provides that, for purposes of determining the rights of a
creditor of a consignee (e.g., the rights of Bank in this case), the consignee of goods under a consignment
governed by Article 9 is “deemed to have rights and title to the goods identical to those of the consignor.”
Thus, if this consignment is governed by Article 9, Giant will be treated as having had the full ownership
interest that Specialty had in the speakers, and Bank’s security interest will attach to the speakers.

On these facts, the Giant-Specialty transaction created an Article 9 consignment. Under UCC § 9-102, an
Article 9 consignment exists where goods are delivered to a merchant (other than an auctioneer) for the
purpose of sale, the merchant deals in those goods under a name other than the name of the consignor, the
goods are not consumer goods immediately before their delivery to the merchant, the value of the goods

Seperac-J19 Exam-Released MEE Essay Compilation © 2016-2020 1086


exceeds $1,000, and the transaction does not otherwise create a security interest. Here, goods valued over
$1,000 were delivered to Giant (a merchant dealing in goods of that kind under its own name). Although
destined to be sold to consumers, the goods were not consumer goods before delivery; they were part of
Specialty’s inventory. There is no indication that the Specialty-Giant transaction otherwise created a
security interest. In short, the transaction is a consignment governed by Article 9, and Giant will be
treated as having right and title to the speakers for Article 9 purposes.

As a result, Bank’s security interest attaches to the Specialty speakers. Now that Giant has defaulted on its
debt, Bank has the right to repossess the speakers, sell them within the guidelines of Article 9, and be
repaid on its debt from the proceeds. Specialty would have no cause of action against Bank for doing so
unless Bank conducted its sale in a commercially unreasonable manner or otherwise violated the dictates
of Article 9’s foreclosure provisions.

[NOTE: The facts don’t indicate any Article 9 violations by Bank, and the question does not invite
applicants to hypothesize such violations. Points can be given for short mentions of violations that would
actually result in monetary harm to Specialty, but the main thrust of answers should be that there is
nothing inherently wrongful in Bank’s proceeding against Specialty’s property, due to the consignment
and attachment rules discussed above.]

Explanation to Point-Two (25-35%):

Specialty is treated as having a security interest in the speakers, and this interest is unperfected.
Accordingly, Specialty’s interest is junior to Bank’s perfected security interest, and it is therefore
extinguished upon Bank’s foreclosure.

Although Specialty is the outright owner of the speakers, Article 9 treats Specialty as if its interest in the
speakers were a purchase-money security interest in inventory. So the question is whether Specialty’s
purchase-money security interest in inventory has priority over Bank’s ordinary security interest in the
speakers as inventory of Giant such that Specialty can recover the proceeds of the sale of the speakers.

On these facts, Bank’s interest clearly prevails. Bank perfected its interest by filing a financing statement,
whereas Specialty’s interest is unperfected. Perfection of a purchase-money security interest in inventory
would normally require Specialty to file a financing statement. There is no indication in the facts that
Specialty took this step. Indeed, there is no hint that Specialty did anything to alert third parties to its
interest, and the facts say explicitly that “as far as any third party can discern, the speakers are part of
Giant’s own inventory.”

A perfected security interest in inventory has priority over an unperfected security interest in the same
inventory, even if the unperfected interest is a purchase-money interest. This priority over the inventory
extends to the proceeds of that inventory. Thus, Bank has priority over Specialty as to the proceeds of the
sale of the speakers.

Explanation to Point-Three (25-35%):

Specialty could have achieved priority over Bank by promptly filing a financing statement against Giant.

As noted above, Article 9 treats Specialty’s ownership of the speakers as if it were a purchase-money
security interest. Specialty could have protected its interest in the speakers had it filed a financing
statement to perfect that interest. Had Specialty perfected its interest before Bank, Specialty would have
achieved priority. Moreover, because Specialty’s interest is treated as a “purchase-money security interest

Seperac-J19 Exam-Released MEE Essay Compilation © 2016-2020 1087


in inventory,” it could have prevailed over Bank even if it perfected after Bank perfected, so long as it (1)
filed a financing statement against Giant by the time Giant received the speakers, and (2) sent a
notification to all holders of conflicting security interests in Giant’s inventory explaining that Specialty
was keeping a purchase money security interest in the consigned speakers.

[NOTE: Had Specialty achieved priority over Bank, Bank would technically still have had the right to
foreclose on Specialty’s speakers (because any junior party is free to foreclose, but Specialty’s senior
security interest would not be discharged by such a foreclosure. In other words, Specialty’s interest would
attach to the proceeds of the sale. Moreover, Specialty’s interest would follow the speakers into the hands
of anyone who bought them at Bank’s foreclosure sale, and Specialty would be able to take the speakers
back from the buyer since they are, after all, Specialty’s property.]

Seperac-J19 Exam-Released MEE Essay Compilation © 2016-2020 1088


#251-JUL 2005–MEE Q03: QUESTION THREE (UCC ART. 9)

On August 1, 2003, Builder, a construction company, borrowed $2 million from Lender to finance the
purchase of a heavy-duty construction crane. Shareholder, a very wealthy shareholder of Builder,
personally guaranteed the loan. Builder signed an agreement granting Lender a security interest in the new
crane to secure the loan. However, Lender neglected to file a financing statement reflecting its security
interest.

Due to severe cash flow problems, Builder stopped making scheduled loan payments four months ago,
leaving a balance of $1.5 million on the loan.

In response to Builder’s failure to make the scheduled loan payments, Lender hired a repossession service
to recover the crane. Employees of the repossession service visited Builder’s unprotected job site and,
without notice to Builder, took the crane away. This occurred on Sunday, June 5, 2005, when no one from
Builder was present and thus no dispute erupted.

On June 7, 2005, Lender mailed a notice to Builder announcing that Lender was going to sell the crane at
an auction on June 21, 2005. Lender did not send separate notice of the auction to Shareholder. On the
announced auction date, Lender sold the crane to Dealer, the highest bidder at the auction, for $1 million.
Shareholder knew about the auction but did not bid at it. It can be proved that if Shareholder had bid, the
bid would have been $1.2 million.

Builder asserts that: (a) Lender’s actions amounted to a conversion of the crane because Lender’s security
interest was not perfected, and (b) Lender handled the repossession and sale improperly. Builder sues
Lender for $1.7 million, which Builder says is the fair market value of the crane.

Lender counterclaims for the $500,000 deficiency, which it says is still owed by Builder on the loan after
taking into account the proceeds received by Lender from the sale of the crane.

1. Did Lender have the right to repossess and sell the crane? Explain.

2. Did Lender’s handling of the repossession, foreclosure, and sale comply with all legal
requirements, and is Lender entitled to recover the $500,000 deficiency from Builder? Explain.

Seperac-J19 Exam-Released MEE Essay Compilation © 2016-2020 1089


#251: J05-3 MEE: ANSWER: NCBE (UCC ART. 9)

POINT (1)(a) [42%] ISSUE: What are the necessary requirements for attachment of an enforceable
security interest and were they met in this case; and (b) may a secured party exercise self-help
repossession and foreclosure remedies even if its security interest is unperfected? ANSWER:
Lender’s security interest is unperfected, but Lender nonetheless has the right to exercise self-help
remedies after the debtor’s default.

POINT (2)(a) [42%] ISSUE: Were the repossession, foreclosure, and sale of the crane handled
properly when Lender failed to give notice of any of these actions to a guarantor of the loan?
ANSWER: Yes. A secured party need not give notice or follow any established process in
repossessing the collateral, so long as the repossession occurs without a breach of the peace.
However, it must give advance notice of a foreclosure sale to the debtor, including any guarantor,
and its failure to do so renders the foreclosure sale improper.

POINT (2)(b) [16%] ISSUE: What effect does a violation of UCC Article 9’s notice rules have on
the debtor’s liability for a deficiency? ANSWER: When the secured party fails to comply with
Article 9’s foreclosure rules and the case does not involve a consumer transaction, the debtor’s
liability for a deficiency is wholly eliminated, except to the extent that the secured party can prove
that a deficiency would have existed even in a complying foreclosure.

ANSWER DISCUSSION:

Lender has a security interest in the crane and is entitled to enforce that interest against Builder, even
though the interest is unperfected. It can enforce through self-help repossession, as it did here, so long as
there is no breach of the peace. However, Lender should have given notice to both Builder and
Shareholder before it sold the crane. Because of Lender’s failure to give proper notice of the sale, Article
9 of the Uniform Commercial Code (UCC) creates a rebuttable presumption that a proper sale would have
generated enough money to entirely pay off Builder’s debt. Thus, Lender is not entitled to recover any
deficiency unless it can prove that even in a proper sale the crane would have sold for less than the
amount outstanding on the loan. Moreover, Lender is liable for any damages that can be shown to have
resulted from its failure to give notice (though the facts of this problem do not support a damages claim).

ANSWER EXPLANATION:

Explanation to Point-One(a)(b) (35-45%):

Lender’s security interest is unperfected, but Lender nonetheless has the right to exercise self-help
remedies after the debtor’s default.

Lender can assert rights in the crane as collateral if lender has a security interest that has attached to the
crane. On the facts of this problem, the requirements for attachment are satisfied. First, Builder signed an
agreement granting Lender a security interest in the crane. Second, Lender gave value to Builder. Finally,
Builder apparently had rights in the crane, as the facts indicate that the money was borrowed to purchase
the crane and the crane was in Builder’s possession. The security interest accordingly attached to the
crane, and Lender has the right to enforce that interest against Builder.

Seperac-J19 Exam-Released MEE Essay Compilation © 2016-2020 1090


Because Lender failed to file a financing statement, Lender’s security interest is unperfected. However, a
secured party may use Article 9’s self-help remedies after default, regardless of the secured party’s
perfected or unperfected status. These self-help remedies include repossession and sale of the collateral
and are inherent in a security interest. Here, Builder was clearly in default, having missed three payments
in a row. Thus, Lender was entitled to repossess and sell the collateral.

Explanation to Point-Two(a) (35-45%):

A secured party need not give notice or follow any established process in repossessing the collateral, so
long as the repossession occurs without a breach of the peace. However, it must give advance notice of a
foreclosure sale to the debtor, including any guarantor, and its failure to do so renders the foreclosure sale
improper.

Article 9’s self-help remedies include repossessing the collateral. No notice is required before
repossession. The only substantial restriction on self-help repossession is that no “breach of the peace”
can result. In this case, the facts indicate that there was no breach of the peace. The crane was repossessed
from an unsecured job site and under circumstances where there was no opposition to the repossession
and no dispute of any sort. Consequently, the repossession was conducted in a manner that complied with
Article 9.

However, Lender’s sale of the collateral did not fully comply with Article 9. Before conducting a
foreclosure sale, a secured party must send reasonable notification to certain persons. These persons
include not only the debtor, whom Lender did notify here, but also “any secondary obligor.” Shareholder,
as a guarantor of the secured debt, was a secondary obligor to whom notice should have been given.
Accordingly, Lender’s failure to notify Shareholder of the foreclosure auction was a violation of the
notice provisions of Article 9.

Shareholder’s actual knowledge of the auction probably does not alleviate the sending-of-notice violation.
Even though Shareholder’s actual knowledge provided Shareholder with the opportunity to make a bid,
thus serving the purpose of generating a more competitive auction, the statute plainly requires the sending
of a notice, and the statute does not provide for actual knowledge to be a substitute for the sending of
notice. In part, this is to avoid disputes over what information the debtor or secondary obligor did or did
not have. Indeed, the statute requires that the notification be “authenticated,” precisely to overturn earlier
cases holding that oral notifications were sufficient.

In all other ways, Lender properly complied with the notice requirement. First, regarding the timing of the
notice to Builder, the notice was sent more than 10 days before the auction, and it therefore falls within
the timing safe-harbor provided by UCC § 9-612. Second, regarding the contents of the notice sent to
Builder, the notice specifies that the sale will be by auction on a certain date, and it therefore satisfies the
requirements of UCC § 9-613 that the notice state the method and date of disposition.

Explanation to Point-Two(b) (10-20%):

When the secured party fails to comply with Article 9’s foreclosure rules and the case does not involve a
consumer transaction, the debtor’s liability for a deficiency is wholly eliminated, except to the extent that
the secured party can prove that a deficiency would have existed even in a complying foreclosure.

Article 9 clarifies the rules on the availability of deficiency judgments in cases other than consumer
transactions (that is, cases other than those in which the debt and collateral have personal rather than
business purposes, Resolving a split of authority, Article 9 adopts the “rebuttable presumption rule” as its

Seperac-J19 Exam-Released MEE Essay Compilation © 2016-2020 1091


primary approach to evaluating deficiency claims in these situations. Under the rebuttable presumption
rule, the court must rebuttably presume that, had the secured party complied with the foreclosure rules,
the foreclosure sale would have generated enough money to fully discharge the secured debt.

In this case, the court will presume that a complying foreclosure sale would have generated $1.5 million
(the amount of the debt), thus eliminating Builder’s liability for the remaining $500,000 owed on the loan.
The presumption that a compliant disposition would have generated $1.5 million is rebuttable. If Lender
can prove that even a fully complying disposition (that is, one in which proper advance notice had been
sent to Shareholder) still would have resulted in a deficiency of some amount, then Builder will be liable
for that amount. The burden of proof is on Lender.

[NOTE: Some applicants will continue the analysis by examining whether Lender is likely to be able to
rebut the presumption noted above. The two sides of this debate are fairly subtle and difficult, so
applicants should get credit regardless of their conclusions on this point, so long as their conclusions are
well-supported.]

On the one hand, Shareholder’s actual advance knowledge of the auction, though irrelevant in Point Two,
might be important here. Since Shareholder declined to bid despite actual knowledge, Lender can make a
strong argument that Shareholder would also have declined to bid even if Lender had sent Shareholder the
statutorily required notice. In other words, there is apparently no reason to think that Lender’s full
compliance with the notice rules would have led to a smaller deficiency, and so the full $500,000 would
be recoverable.

On the other hand, if Lender had sent notice to Shareholder and that notice had arrived earlier than the
point at which Shareholder in fact gained actual knowledge, then Shareholder may have been able to react
earlier and have arranged to make a substantial bid. If Builder can make a good enough case on this point
to defeat Lender’s burden of proof, then the deficiency to which Lender is entitled should be reduced by
the amount by which Shareholder’s participation would have increased the sale price of the cranes.
Because Shareholder’s bid would have been for up to $1.2 million ($200,000 more than the sale price),
Lender’s deficiency entitlement would be reduced by up to $200,000, leaving Builder liable for only the
remaining $300,000.

Seperac-J19 Exam-Released MEE Essay Compilation © 2016-2020 1092


#252-FEB 2005–MEE Q06: QUESTION SIX (UCC ART. 9)

Bill operated a restaurant in a building he owned. On March 1, he purchased a large oven on credit for
$8,000 from Sal’s Appliance Emporium. On that same day, the oven was set aside in Sal’s warehouse and
marked with a tag reading, “Sold to Bill.” Also on March 1, Bill signed a security agreement that gave
Sal’s a security interest in the oven to secure the unpaid purchase price of the oven.

On March 14, the oven was delivered and bolted permanently into a specially built niche in the kitchen in
Bill’s restaurant, thus becoming a fixture under local real estate law.

On March 26, the local sheriff came to the restaurant and announced that he was there to execute a levy to
satisfy a $3,000 judgment against Bill in favor of Local Bank. The sheriff saw the new oven in the
restaurant’s kitchen and announced he was levying on the oven. He then physically disabled the oven by
handcuffing it shut and placed a notice-of-levy sticker on it, which qualified as a valid levy under state
law.

On March 28, after learning from Bill about what happened with the sheriff and the oven, Sal’s filed its
financing statement in the Secretary of State’s office but not in the local real estate records office. Sal’s
financing statement properly described the oven.

On March 29, Bill obtained a $10,000 loan from Finance Company and gave Finance Company a
mortgage on his building to secure repayment of the loan. On the same day, Finance Company properly
perfected a valid lien against the restaurant building and accompanying fixtures by filing in the local real
estate records office designated by state law.

1. As between Sal’s and Local Bank, who has priority as to the oven? Explain.

2. As between Sal’s and Finance Company, who has priority as to the oven? Explain.

Seperac-J19 Exam-Released MEE Essay Compilation © 2016-2020 1093


#252: F05-6 MEE: ANSWER: NCBE (UCC ART. 9)

POINT (1) [60%] ISSUE: Did Sal’s purchase-money security interest in the oven, which involved a
delayed filing, defeat Local Bank where Local Bank is an intervening judicial lien creditor?
ANSWER: Yes. Because Sal’s interest in the oven was a purchase-money security interest, Sal’s
had priority over Local Bank, an intervening judicial lien creditor, even though Sal’s failed to file
its financing statement until after the judicial lien arose.

POINT (2) [40%] ISSUE: Did Sal’s purchase-money security interest in the oven, filed only with
the Secretary of State’s Office, defeat Finance Company, a subsequent mortgage lender that has
filed a lien on the building in which the oven is located in the appropriate local real estate records
office? ANSWER: No. Because Sal’s failed to file in the local real estate records, Finance Company
has priority over Sal’s; Finance Company is a subsequent mortgagee that perfected its interest in
the restaurant building where the oven is located by filing in the local real estate records.

ANSWER DISCUSSION:

Sal’s extension of credit to Bill provided the purchase money for Bill to buy the oven. When Bill signed
the security agreement on March 1 granting Sal’s a security interest in the oven, which was “goods” at the
time of the purchase, a purchase-money security interest attached in favor of Sal’s. It was not perfected as
against other creditors at that time because Sal’s had not filed a financing statement. However, a
purchase-money secured creditor has 20 days after delivery of the collateral to perfect by filing. If the
creditor does so, the security interest is perfected retroactively to the date of attachment. Under these
facts, Sal’s perfected by filing on March 28, which was less than 20 days after the March 14 delivery to
Bill. Ordinarily, a lien creditor, such as Local Bank, would have priority over an earlier unperfected
secured creditor. However, because of the retroactive effect of Sal’s filing its purchase-money interest,
which related back to March 1 (the day Sal’s security interest attached), Sal’s prevails over Local Bank as
to the oven. However, Sal’s failure to effect a valid “fixture filing” produces the opposite result vis-à-vis
Finance Company. The oven is clearly a fixture. In order to perfect its security interest in the oven as a
fixture against persons with interest in the real estate, Sal’s was required to file a record of its interest in
the local real estate records office designated by state law. Because Sal’s did not do this, Sal’s interest was
not perfected as against Finance Company. Finance Company, as an “encumbrancer” by virtue of the
mortgage granted to it by Bill, perfected its security interest in the building (and therefore in the fixtures
as well) by filing in the appropriate real estate records office. Thus, Finance Company’s interest in the
oven prevails over Sal’s.

ANSWER EXPLANATION:

Explanation to Point-One (55-65%):

Because Sal’s interest in the oven was a purchase-money security interest, Sal’s had priority over Local
Bank, an intervening judicial lien creditor, even though Sal’s failed to file its financing statement until
after the judicial lien arose.

Because Sal’s extension of credit enabled Bill to purchase the oven, Sal’s had a purchase-money security
interest in “goods.” The oven qualified as “goods” under UCC § 9-102 because it was “moveable” when
the security interest attached on March 1. Although a judicial lien creditor would normally have priority
over an unperfected secured creditor, in this case Sal’s lien had priority over the interest of Local Bank

Seperac-J19 Exam-Released MEE Essay Compilation © 2016-2020 1094


because Sal’s perfected a purchase-money security interest within the grace period provided for in UCC §
9-317.

UCC § 9-317 provides that, if a purchase-money secured creditor files its financing statement within 20
days after the debtor receives delivery of the collateral, then the secured creditor will take priority over the
rights of a judicial lien creditor whose lien arose between the attachment of the purchase-money security
interest and the filing of the financing statement. In this case, Sal’s perfected its interest on March 28
when it filed its financing statement in the Secretary of State’s Office. Under Article 9 of the UCC,
attachment plus filing a financing statement will qualify as perfection. That date was more than 20 days
after Sal’s security interest in the oven attached on March 1. Under Article 9 of the UCC, attachment
occurs where the creditor has given value, the debtor has rights in the collateral, and the debtor has signed
a security agreement. However, Sal’s perfected within 20 days after Bill received delivery of the collateral
on March 14. Therefore, Sal’s perfected security interest in the oven relates back to the date of
attachment, March 1, and takes priority over Local Bank’s judicial lien, which became perfected on
March 26.

Explanation to Point-Two (35-45%):

Because Sal’s failed to file in the local real estate records, Finance Company has priority over Sal’s;
Finance Company is a subsequent mortgagee that perfected its interest in the restaurant building where
the oven is located by filing in the local real estate records.

The oven in which Sal’s has a purchase-money security interest is a “fixture” under Article 9 and
applicable state law because it is “so related to particular real property that an interest in it arises under
real property law.”

The general rule under Article 9 for priority in fixtures is that a security interest in the fixture will be
subordinate to “a conflicting interest of an encumbrancer or owner of the related real property other than
the debtor.” Here the related real property is the restaurant building, and Finance Company’s mortgage in
that building qualifies as an encumbrance on it. Thus, Finance Company is as an “encumbrancer of the
related real property.”

UCC § 9-334 includes a number of exceptions to the general rule that security interests in fixtures are
subordinate to a conflicting interest of an owner or encumbrancer of the related real property. The only
exceptions that might have been available to Sal’s are a purchase-money priority exception and where the
debtor had a record interest in the realty. However, these exceptions would require, among other things,
that Sal’s had effected a proper “fixture filing.” A fixture filing must be recorded in the local real estate
records office. In this case, Sal’s failed to file in the local real estate records office.

Because Finance Company is an encumbrancer of the related real property and because Sal’s fails to
qualify for any of the exceptions listed in UCC § 9-334, Finance Company has priority as to the oven
even though Finance Company’s security interest was created after Sal’s perfected its interest in the oven
by filing in the Secretary of State’s office.

Seperac-J19 Exam-Released MEE Essay Compilation © 2016-2020 1095


#253-JUL 2004–MEE Q07: QUESTION SEVEN (UCC ART. 9)

Seller is in the business of selling new and used road construction equipment. On January 15, Buyer, a
road builder, entered into a written purchase agreement with Seller for the sale of a used excavator for
$100,000. The purchase agreement provided that Seller would add certain attachments to the excavator.
The cost of the attachments was included in the purchase price. Buyer and Seller agreed that the specially
equipped excavator would be ready for Buyer by April 5, in time for the start of the road construction
season.

When Buyer signed the contract, Buyer gave Seller a $25,000 down payment. The remaining $75,000 was
to be paid as follows: $25,000 on March 1; $25,000 on April 1; and $25,000 on May 1.

On January 20, Seller borrowed $1 million from Finance Co. to finance Seller’s business operations.
Finance Co. obtained a properly perfected security interest specifically assigning to Finance Co. all
payments from Seller’s accounts receivable and chattel paper.

On February 20, Seller failed to make a payment due on its loan from Finance Co. Finance Co. declared
Seller in default. However, it informed Seller that it would not require immediate repayment of the full
amount of the loan, but would instead begin collecting payments on all of Seller’s outstanding accounts
and chattel paper directly from the account debtors.

Finance Co. properly notified Buyer that Seller had assigned to Finance Co. the right to receive payment
under the purchase agreement and that Buyer should make all future payments on the purchase agreement
directly to Finance Co. However, Seller told Buyer to disregard Finance Co.’s notification and to continue
to make payments directly to Seller. Buyer then sent its March and April payments directly to Seller.

On April 5, Buyer went to pick up the excavator from Seller. Buyer discovered that Seller had not
equipped the excavator with any of the attachments required by their purchase agreement. Buyer informed
Seller that it would take the excavator, despite its non-conformity with the contract, but that it would find
and add the necessary attachments itself and would withhold from its final payment the cost of making the
goods conform to the contract. Seller stated that it expected full payment but allowed Buyer to take the
excavator.

On May 5, Finance Co. demanded that Buyer immediately pay Finance Co. $75,000, the full amount of
the March, April, and May payments under the purchase agreement. Buyer refused, asserting that it made
the March and April payments to Seller and that no remaining payment was due because the cost of
bringing the excavator up to contract specifications exceeded $30,000, substantially more than the amount
of the May payment.

How much, if anything, does Buyer owe Finance Co.? Explain.

Seperac-J19 Exam-Released MEE Essay Compilation © 2016-2020 1096


#253: J04-7 MEE: ANSWER: NCBE (UCC ART. 9)

POINT (1) [50%] ISSUE: Did Finance Co.’s letter to Buyer informing it of the assignment of the
purchase agreement to Finance Co. obligate Buyer to make future payments under the contract
directly to Finance Co.? ANSWER: Yes. Once Finance Co. properly notified Buyer of the
assignment of the purchase agreement and instructed Buyer to make payments directly to Finance
Co., Buyer was obligated to make all payments under the contract to Finance Co.; Buyer’s
payments to Seller did not discharge that obligation.

POINT (2) [50%] ISSUE: Can Buyer reduce the amount it owes Finance Co. by subtracting the
expenses it incurred to add the attachments to the excavator? ANSWER: Yes. Buyer can assert
Seller’s breach of contract as a defense to payment and offset its damages against the amount it
owes Finance Co.

ANSWER DISCUSSION:

Once Finance Co. notified Buyer of the assignment of the right to receive payment and demanded
payment of the balance, Buyer was obligated to pay Finance Co., notwithstanding Seller’s instruction to
the contrary. However, Buyer had the right to offset against Finance Co. its damages for Seller’s breach of
contract, even though Buyer had accepted delivery of the excavator.

ANSWER EXPLANATION:

Explanation to Point-One (45-55%):

Once Finance Co. properly notified Buyer of the assignment of the purchase agreement and instructed
Buyer to make payments directly to Finance Co., Buyer was obligated to make all payments under the
contract to Finance Co.; Buyer’s payments to Seller did not discharge that obligation.

As a result of the security agreement signed by Seller, Finance Co. had a security interest in all of Seller’s
accounts and chattel paper. Moreover, the security agreement expressly provided that payments arising
from Seller’s “accounts and chattel paper” had been assigned to Finance Co. Under Article 9 of the
Uniform Commercial Code, a security interest in “accounts” covers any “right to payment of a monetary
obligation, whether or not earned by performance, for property that has been or is to be sold.”
Consequently, Finance Co.’s security interest covered the amount owed by Buyer to Seller pursuant to
Buyer’s agreement to purchase the excavator.

Article 9 gives a secured party or assignee of an account (i.e., Finance Co.) the right to collect directly
from the account debtor (i.e., Buyer) in the event of a default by the debtor (i.e., Seller). To exercise that
right effectively, Finance Co. must send an authenticated notification to the account debtor informing the
account debtor that the amount due has been assigned and that payment is to be made to the assignee. The
facts state that Finance Co. properly notified Buyer of the assignment of Seller’s right to receive payment
and directed Buyer to make future payments to Finance Co.

Once an account debtor receives proper notification to make future payments directly to an assignee, the
account debtor may discharge its payment obligation only by payment to the assignee. Payments made to
the assignor do not result in discharge.

Seperac-J19 Exam-Released MEE Essay Compilation © 2016-2020 1097


The fact that Seller told Buyer to disregard the notice from Finance Co. is no defense. Proper notice from
Finance Co. (assignee) is all that is required to obligate Buyer to make payment directly to Finance Co.,
and nothing in the UCC allows an assignor like Seller to interfere with the rights of the assignee in this
way. Rather than ignoring the assignment and relying on Seller for instructions, Buyer should have
demanded proof of the assignment from Finance Co. Had it done so, Buyer could have safely paid Seller
until reasonable proof was offered. What Buyer cannot do is what it did here – ignore the notification of
the assignment and continue to pay Seller.

Consequently, Buyer was obligated to make the March and April payments to Finance Co., and it remains
liable for those payments. Buyer was not discharged by reason of its payment to Seller. Buyer is also
obliged to make the May payment to Finance Co., subject to an offset, as discussed below.

Explanation to Point-Two (45-55%):

Buyer can assert Seller’s breach of contract as a defense to payment and offset its damages against the
amount it owes Finance Co.

The rights of an assignee of an account (Finance Co.) are subject to “any defense or claim in recoupment
arising from the transaction that gave rise to the contract.” The account debtor may, of course, waive the
right to assert defenses against an assignee, but there is nothing in the facts to suggest that Buyer waived
this right.

Here, the facts establish that Buyer has a defense against full payment of the contract price. Seller agreed
to supply Buyer with an excavator that was specially equipped. Moreover, Seller agreed to supply the
excavator by a particular date, when Seller knew that Buyer needed it to begin work. When that date
arrived, the excavator had not been equipped as required by the contract. Buyer nevertheless took delivery
of the excavator, despite its failure to conform to the contract.

When a buyer takes goods in spite of a known nonconformity with the contract, the buyer has accepted
the goods for the purposes of sales law. Acceptance ordinarily obligates a buyer to pay the contract price.
However, acceptance of nonconforming goods does not bar a remedy for breach, so long as the buyer
notifies the seller of the breach within a reasonable time of its discovery. Here, Buyer notified Seller of
the nonconforming nature of the excavator at the time it took possession of the equipment and told Seller
it intended to offset its costs for the attachments. As a result, Buyer is entitled to assert its remedies for
breach, despite its acceptance of the goods.

In general, Buyer’s remedy is the recovery of damages for the nonconformity, defined as “the loss
resulting in the ordinary course of events from the seller’s breach as determined in any manner which is
reasonable.” Moreover, Buyer “may deduct all or any part of the damages from any part of the price still
due,” so long as Buyer notifies Seller of the intention to do so. Here, Buyer notified Seller of its intention
in this regard.

Buyer’s contract claims against Seller may be asserted against Finance Co., as assignee of the contract, to
reduce the amount owed by Buyer.

Under the contract, Buyer owes $75,000 to Finance Co. (the amount of the March and April payments,
which Buyer improperly sent to Seller, and the amount of the upcoming May payment). Buyer claims
$30,000 in damages from Seller’s breach, and it may deduct that amount from the amount owed under the
contract. Consequently, Buyer owes Finance Co. $45,000.

Seperac-J19 Exam-Released MEE Essay Compilation © 2016-2020 1098


#254-FEB 2004–MEE Q05: QUESTION FIVE (UCC ART. 9)

PC is a professional law corporation formed by Ted and Teresa. In 2002, PC obtained a $100,000 line of
credit from First Bank, and PC authenticated a security agreement that granted First Bank a security
interest in “all of PC’s equipment, now owned or hereafter acquired.”

Although PC did not specifically authorize it, First Bank promptly filed a financing statement in the
appropriate state offices. The financing statement, which PC had not signed, recited erroneously that First
Bank had a security interest in “all of PC’s equipment, inventory, and accounts receivable.” PC knew that
First Bank had filed a financing statement but was unaware that the financing statement covered collateral
not mentioned in the security agreement.

In 2003, PC entered into contracts to provide legal services to a number of municipalities, one of which
was the City of Eden. To perform these contracts, PC decided to expand its operation by hiring an
additional attorney, adding office staff, and increasing its office space. When First Bank refused to
increase PC’s line of credit to finance the expansion, PC approached Second Bank about obtaining a
separate business expansion loan.

Second Bank agreed to loan PC $100,000, provided the loan was secured by a first security interest in
PC’s accounts receivable. However, the arrangement fell through when Second Bank discovered that First
Bank had already filed a financing statement covering PC’s accounts receivable. First Bank ignored
repeated requests from PC to terminate or amend the financing statement, and Second Bank was unwilling
to make a loan until First Bank’s filing was changed. As a result, PC was not able to secure financing
from Second Bank.

Because of its inability to secure this financing, PC was unable to hire additional staff and could not
adequately perform the contracts. After PC’s attorneys failed to attend two important city council
meetings because they were occupied with other duties, the City of Eden terminated its contract with PC,
a contract that PC had expected to generate $250,000 in revenue over two years.

Also in 2003, Luke, a former employee of PC’s, sued PC and won a judgment for $50,000. Luke
immediately obtained a judicial lien against all of PC’s assets, including its equipment and accounts
receivable.

1. As between Luke and First Bank, who has the superior claim to PC’s equipment?

Explain.

2. As between Luke and First Bank, who has the superior claim to PC’s accounts receivable?
Explain.

3. What claims, if any, does PC have against First Bank under the Uniform Commercial Code?
Explain.

Seperac-J19 Exam-Released MEE Essay Compilation © 2016-2020 1099


#254: F04-5 MEE: ANSWER: NCBE (UCC ART. 9)

POINT (1) [33%] ISSUE: Did First Bank properly perfect its security interest in PC’s equipment so
that it has priority over a subsequent lien creditor? ANSWER: Yes. Because PC had authenticated
a security agreement granting First Bank a security interest in PC’s equipment, First Bank
perfected its security interest by filing, even though PC did not specifically authorize First Bank to
file a financing statement. To that extent, First Bank had priority over Luke as to PC’s equipment.

POINT (2) [33%] ISSUE: Does First Bank’s financing statement claiming a security interest in
PC’s accounts receivable give it a claim to those accounts when the security agreement does not
mention those accounts and the financing statement was filed without PC’s authorization?
ANSWER: No. Luke has a superior claim to PC’s accounts because First Bank had no security
interest in the accounts and the inclusion of the accounts in its financing statement was
unauthorized and ineffective.

POINT (3) [33%] ISSUE: Is First Bank liable for any damages PC sustained as a result of First
Bank’s filing of an overbroad financing statement and its subsequent refusal to terminate or amend
the statement? ANSWER: Yes. First Bank violated Article 9 by filing an overbroad financing
statement and by refusing to correct that filing. As a result, it is liable to PC for any loss caused by
those violations, including losses caused by PC’s inability to secure financing from Second Bank.

ANSWER DISCUSSION:

First Bank will prevail over Luke, the subsequent lien creditor, if First Bank had a prior, perfected
security interest in the disputed equipment. First Bank had a prior, perfected interest in PC’s equipment,
but not in PC’s accounts receivable. Thus, First Bank prevails as to the equipment, but not as to the
accounts receivable. Luke prevails as to the accounts receivable. Moreover, because First Bank
improperly filed a financing statement covering PC’s accounts, First Bank is liable to PC for any damages
caused by that improper filing.

ANSWER EXPLANATION:

Explanation to Point-One (25-35%):

Because PC had authenticated a security agreement granting First Bank a security interest in PC’s
equipment, First Bank perfected its security interest by filing, even though PC did not specifically
authorize First Bank to file a financing statement. To that extent, First Bank had priority over Luke as to
PC’s equipment.

As between a lien creditor and a secured party, the secured party has the superior claim to a debtor’s
assets if the secured party has a perfected security interest that predates the judicial lien. In this case, First
Bank obtained a security interest in PC’s equipment, filed a financing statement covering the equipment,
and advanced funds to PC on the strength of that arrangement. As a result, First Bank had a perfected
security interest in PC’s equipment. Moreover, these steps were taken before Luke acquired a lien against
PC’s assets. Consequently, First Bank has the superior claim to PC’s equipment. Under UCC 9-201,
except as otherwise provided in the UCC, a security agreement is effective against other creditors of the
debtor, and under UCC 9-317, a lien creditor prevails over a secured party only if the lien creditor obtains
an interest before security interest is perfected.

Seperac-J19 Exam-Released MEE Essay Compilation © 2016-2020 1100


It makes no difference that PC did not sign or expressly authorize the filing of the financing statement.
Under the old version of Article 9, financing statements were required to be signed by the debtor. Under
new Article 9, the debtor’s signature is not necessary. Although filings must still be authorized by the
debtor, the debtor’s authentication of a security agreement covering particular collateral is ipso facto an
authorization for the secured party to file a financing statement covering that collateral. Because PC
authenticated a security agreement granting First Bank a security interest in equipment, First Bank was
entitled to file a financing statement covering equipment, which was effective to perfect First Bank’s
interest.

Explanation to Point-Two (25-35%):

Luke has a superior claim to PC’s accounts because First Bank had no security interest in the accounts
and the inclusion of the accounts in its financing statement was unauthorized and ineffective.

First Bank has no security interest in PC’s accounts. To obtain a security interest in a debtor’s assets, a
creditor must receive the debtor’s agreement to grant such an interest. In general, no security interest will
attach to collateral unless “the debtor has authenticated a security agreement that provides a description of
the collateral.” Here, First Bank has absolutely no claim to a security interest in PC’s accounts because
the PC-First Bank security agreement did not include accounts receivable in the description of collateral,
but mentioned only equipment. There is nothing in the facts to suggest that there was any intention to
include accounts. To the contrary, the facts suggest that the security interest was intentionally limited to
equipment.

If no security interest has attached, it cannot be perfected no matter what the creditor does. Here, no
security interest ever attached to the accounts. Because First Bank has no security interest in the accounts
and no other basis for claiming a property interest in the accounts, Luke’s interest prevails.

The financing statement does not help First Bank in this regard because it is not a security agreement and,
also, because it was overbroad. First Bank did not have the right to file a financing statement covering
additional collateral unless PC authorized such a filing, which PC did not. A financing statement is
effective “only to the extent that it was filed by a person that may file it.” Because First Bank was not
entitled to file a financing statement covering PC’s accounts receivable, the financing statement is not
effective to cover those accounts. Thus, Luke has a superior claim to the accounts receivable.

Explanation to Point-Three (25-35%):

First Bank violated Article 9 by filing an overbroad financing statement and by refusing to correct that
filing. As a result, it is liable to PC for any loss caused by those violations, including losses caused by
PC’s inability to secure financing from Second Bank.

Revised Article 9 provides generally that “a person is liable for damages in the amount of any loss caused
by a failure to comply with this article.” Damages may be recovered by a person who is a debtor at the
time of the failure of compliance with Article 9.

On our facts, First Bank filed an overbroad financing statement. The problem, as noted earlier, is that First
Bank had no express authorization from the debtor to file the financing statement, and the authorization
provided ipso facto by debtor’s authentication of a security agreement extended only to the collateral
mentioned in the security agreement, equipment. Hence, by filing an overbroad financing statement, First
Bank violated its “duty to refrain from filing an unauthorized financing statement.” Under Article 9 of the
UCC, a secured party may file a financing statement only if authorized. Moreover, a secured party has a

Seperac-J19 Exam-Released MEE Essay Compilation © 2016-2020 1101


duty to terminate an unauthorized filing when termination is requested by the debtor. First Bank also
failed to comply with this duty.

First Bank is liable for any loss caused to PC by its breach of these duties. Recoverable losses include
losses “resulting from the debtor’s inability to obtain alternative financing.” In this case, those losses may
be substantial. First Bank’s overbroad financing statement made it impossible for PC to secure a loan
from Second Bank. PC’s inability to secure financing for its business expansion prevented it from
adequately servicing its contracts. As a result, the City of Eden canceled its contract, which had been
expected to yield significant revenue for PC. PC must, of course, establish both the amount of the loss and
that the loss was caused by First Bank’s failure to comply with the Code. Moreover, damages should be
limited to an amount “reasonably calculated to put PC in the position that it would have occupied had no
violation occurred.”

Seperac-J19 Exam-Released MEE Essay Compilation © 2016-2020 1102


#255-JUL 2003–MEE Q07: QUESTION SEVEN (UCC ART. 9)

Debtor, the sole proprietor of a small restaurant, borrowed $20,000 from Bank. The loan was used to
purchase kitchen equipment for Debtor’s business, including a stove unit, a large refrigerator, two
freezers, and a commercial microwave oven. Debtor signed a promissory note and a security agreement
granting Bank a security interest in the items purchased. The security agreement contained clauses (1)
waiving Debtor’s right of redemption should default occur, and (2) providing that, if Debtor failed to
make any regular installment payment, “the entire unpaid obligation due from Debtor to Bank shall,
without further notice, immediately become due and payable.” Bank properly perfected its security
interest in the collateral.

Debtor missed four monthly installment payments. When Debtor was approximately $1,600 in arrears,
Bank’s representative drove a large truck to the alley behind the restaurant. Bank’s representative was
accompanied by two movers. The time was 11:10 p.m., shortly after the restaurant’s closing time. Bank’s
representative pounded loudly on the locked back door, which was opened by one of the restaurant’s
employees. “We’re here to repossess the kitchen equipment,” stated Bank’s representative. “Well, my
boss isn’t here, and I don’t think I should let you in,” replied the employee. Without further discussion,
Bank’s representative and the two movers walked right past the employee and began moving the kitchen
equipment out to the truck. At first, the employee objected loudly, but soon he shrugged and watched
quietly as they removed the equipment.

The next day, Debtor went to Bank and offered to pay the $1,600 arrearage in monthly payments. Bank
refused the tender, reminding Debtor the entire balance was now due. When Debtor then offered to pay
the entire balance due including Bank’s repossession expenses, Bank told Debtor that it would not allow
him to redeem because he had waived his right of redemption. Thereafter, Bank sent Debtor a notice of
public sale and then duly held the sale, the terms of which were commercially reasonable. Bank now has
threatened to sue Debtor for a deficiency judgment of $4,000, the difference between Debtor’s unpaid
loan obligation and what Bank recovered in the sale.

Debtor has had to close the restaurant because the kitchen equipment was repossessed.

What are Debtor’s rights, remedies, and liabilities under the Uniform Commercial Code? Explain.

Seperac-J19 Exam-Released MEE Essay Compilation © 2016-2020 1103


#255: J03-7 MEE: ANSWER: NCBE (UCC ART. 9)

POINT (1) [33%] ISSUE: When will a repossession be invalidated as having been conducted in
breach of the peace? ANSWER: Debtor can argue that Bank’s repossession of the collateral was
conducted in breach of the peace. Bank equally can assert that its repossession was peaceable.

POINT (2) [33%] ISSUE: What are the requirements for a valid waiver of the right of redemption,
and what must the debtor tender if the security agreement contains an acceleration clause?
ANSWER: The right of redemption cannot be waived in the security agreement, and Debtor was
entitled to redeem the collateral by tendering fulfillment of all obligations before Bank had disposed
of the collateral. Debtor had to tender the entire loan balance due because the security agreement
contains an acceleration clause. Bank’s refusal to accept a tender of the full balance due violated
Debtor’s rights.

POINT (3) [33%] ISSUE: What are a debtor’s remedies if a secured party fails to comply with the
debtor’s rights of redemption and seeks to recover a deficiency judgment? ANSWER: Debtor is
entitled to the remedies provided in UCC Sections 9-625 and 9-627. Debtor may be able to recover
damages if the repossession is found to be unlawful. It may also be able to challenge the imposition
of a deficiency judgment.

ANSWER DISCUSSION:

It is debatable whether the repossession, which Bank had the right to carry out, was or was not conducted
without a breach of the peace. In any case, Bank’s refusal of Debtor’s tender of the entire balance violated
Debtor’s rights because the right of redemption cannot be waived in the security agreement. Given that
the repossession was unlawfully carried out because of Bank’s refusal to accept Debtor’s tender of
redemption, Debtor is entitled to recover any damages caused by Bank’s failure to comply with the rules
relating to the repossession. This might include damages for loss of business. Debtor can also challenge
Bank’s claim for a deficiency, but, since the sale was commercially reasonable, it is unlikely that Debtor
will prevail on his challenge.

ANSWER EXPLANATION:

Explanation to Point-One (30-35%):

Debtor can argue that Bank’s repossession of the collateral was conducted in breach of the peace. Bank
equally can assert that its repossession was peaceable.

Section 9-609 of the Uniform Commercial Code provides that “after default, a secured party may take
possession of the collateral,” and may proceed without judicial process if this can be done “without
breach of the peace.” This “self-help repossession” is a significant right accruing to the secured party
under Article 9’s default provisions, and it may be exercised without giving prior notice to Debtor.
Because Debtor was in default in its monthly payments in the amount of $1,600, Bank was within its
rights in repossessing the kitchen equipment, so long as Bank did so without a breach of the peace.

“Breach of the peace” is not defined in Article 9. White and Summers suggest that claims of breach of the
peace be analyzed with a “crude two-factor formula of creditor entry and debtor response refined by a

Seperac-J19 Exam-Released MEE Essay Compilation © 2016-2020 1104


consideration of third-party response, the type of premises entered and possible creditor deceit in
procuring consent.”

Applicants should discuss a number of facts in considering whether the repossession was carried out
without a breach of the peace. First, the repossession took place on business property, rather than in a
private home. The courts are most sensitive to unauthorized entries into a debtor’s residence. Here,
because the repossession took place at a business rather than at a home, the public policy against intrusion
upon a debtor’s solitude and safety is less compelling.

Second, although Debtor was not present, Debtor’s employee was and Debtor’s employee objected to the
repossession. White and Summers state: “An opposition to the entry or seizure, however slight and even
though merely verbal, normally results in a breach of the peace. The law should not make a debtor
physically confront a repossessor in order to sustain a claim of breach of the peace.” On the other hand,
some courts have held that oral protest alone does not unequivocally preclude self-help repossession. For
example, one court found that there was no breach of the peace where the repossession took place in the
parking lot of the debtor’s place of work, notwithstanding the debtor’s initial oral protest. The Eighth
Circuit in Clarin acknowledged that the “courts are divided on the issue of whether an unequivocal oral
protest amounts to a breach of the peace.” Moreover, the employee’s protest was less significant than the
debtor’s own protest would have been.

Apart from the oral protest, the facts do not suggest that entry was otherwise obtained by a breach of the
peace. The repossessors did not attempt repossession during regular business hours; they did not pick the
lock or break a door down or otherwise damage Debtor’s business; the repossessors walked past the
employee through an open door; the employee at first objected loudly, but later was quiet as the
repossession proceeded; the employee did not physically attempt to halt the repossession or call law
enforcement authorities; there was no threat of violence; Bank’s representative did not use a ruse or
stealth to gain entry, but honestly stated the purpose of the visit.

Explanation to Point-Two (30-35%):

The right of redemption cannot be waived in the security agreement, and Debtor was entitled to redeem
the collateral by tendering fulfillment of all obligations before Bank had disposed of the collateral. Debtor
had to tender the entire loan balance due because the security agreement contains an acceleration clause.
Bank’s refusal to accept a tender of the full balance due violated Debtor’s rights.

Section 9-623 of the UCC provides that a debtor may redeem collateral “at any time before a secured
party has disposed of collateral or entered into a contract for its disposition under Section 9-610.” To
redeem the collateral, debtor must tender “fulfillment of all obligations secured by the collateral” as well
as the expenses reasonably incurred by the secured party in retaking, holding and preparing the collateral
for disposition.

The right to redeem collateral is a mandatory right that may not be waived in the security agreement. As a
result, the clause in the security agreement purporting to waive Debtor’s right of redemption is
ineffective. A waiver of Debtor’s right to redeem is effective only if it is done “by an agreement to that
effect entered into and authenticated after default.” There was no such agreement here. Thus, Debtor was
entitled to redeem the collateral at the time he attempted to do so, because it was “before the secured party
had disposed of the collateral under Section 9-610.”

To redeem the collateral, a debtor must “tender fulfillment of all obligations secured by the collateral.”
Where, as here, the security agreement contains an acceleration clause, this would include the entire

Seperac-J19 Exam-Released MEE Essay Compilation © 2016-2020 1105


balance due on the loan. In addition, the redemption amount must include “the expenses reasonably
incurred by the secured party in retaking, holding and preparing the collateral for disposition, in arranging
for the sale, and to the extent provided in the agreement and not prohibited by law, the secured party’s
reasonable attorneys’ fees and legal expenses.”

In this case, Debtor’s initial tender of the amount of his arrearage in monthly payments was inadequate to
redeem the collateral. When Debtor later offered to tender the full amount due, Bank made it clear that it
would not accept any redemption. Bank based its refusal on the redemption waiver clause in the security
agreement. But Part 6 of Article 9 deems such clauses unenforceable. In short, Bank’s refusal to allow
Debtor to redeem was a violation of Debtor’s rights.

Explanation to Point-Three (30-35%):

Debtor is entitled to the remedies provided in UCC Sections 9-625 and 9-627. Debtor may be able to
recover damages if the repossession is found to be unlawful. It may also be able to challenge the
imposition of a deficiency judgment.

Section 9-625 of the UCC provides: “If it is established that a secured party is not proceeding in
accordance with this article, a court may order or restrain disposition of collateral. A person is liable for
damages in the amount of any loss caused by a failure to comply with this article.”

A commercially reasonable sale under § 9-610 has already taken place. Thus, Debtor now must seek to
recover from Bank for “any loss caused by a failure to comply” with Article 9. Presumably, this would
include any loss attributable to the failure of Bank to allow Debtor to redeem and may include any loss
attributable to improper repossession (assuming that Debtor can prove a breach of the peace was
committed).

The collateral in this case – stove, refrigerator, freezers, and microwave oven for use in Debtor’s
restaurant business – would be classified as “equipment” (“used or bought for use primarily in business”)
under § 9-102. In the case of business equipment, Article 9 provides no clear guidance on the question of
measuring damages for “any loss caused by a failure to comply with this article.” White and Summers
state that a debtor’s “loss” for these purposes “is probably the difference between the net amount actually
realized on resale and the amount that would have been realized had the creditor complied with the
Code’s requirements.” The new Code states expressly, however, that damages should also include, in
appropriate cases, such consequential losses as those caused by the “debtor’s inability to obtain alternative
financing.” Some commentators endorse the possibility of punitive damages.

In addition, UCC § 9-626 authorizes denial of a deficiency judgment when the creditor has failed to
comply with Article 9’s rules concerning the “collection, enforcement, disposition or acceptance” of
collateral. Article 9 adopts the so-called “rebuttable presumption” rule: Where the creditor has violated a
relevant rule, the creditor will not be allowed to obtain a deficiency judgment unless the creditor proves
that the amount that “would have been realized” if the creditor had acted properly “is less than” the
amount to which the creditor was entitled. However, reduction of a deficiency appears to preclude a
debtor from also recovering actual damages.

In this case, Debtor might argue that consequential damages should include damages for loss of business
opportunity, because Bank’s wrongful repossession and/or refusal to permit redemption resulted in the
closure of Debtor’s business. Consequential damages must be reasonably foreseeable under the traditional
rule of Hadley v. Baxendale. It is arguably foreseeable to the creditor that an improper refusal to allow
redemption of essential kitchen equipment may result in substantial business losses to a restaurant

Seperac-J19 Exam-Released MEE Essay Compilation © 2016-2020 1106


operation. In addition, Debtor might argue that Bank’s refusal to allow redemption interfered with
Debtor’s ability to secure alternative financing because it deprived Debtor of the collateral necessary for
an alternative loan.

On the other hand, apart from these consequential damages, it will be difficult for Debtor to establish any
loss. The sale of the kitchen equipment was commercially reasonable, and there is no evidence that
Bank’s wrongful conduct affected the price it received for the equipment. Consequently, Bank may well
be able to establish that it is entitled to a deficiency judgment, despite its violations of Article 9.

Debtor had the absolute obligation to mitigate his damages. If he had enough money to pay off the
balance and Bank’s expenses, then he naturally had enough money to appear at Bank’s foreclosure sale
and buy the equipment back at the sale. Debtor would still have damages for interim loss of business, his
own expenses, attorneys’ fees, etc., but Debtor may be unable to recover for permanent loss of business.

Seperac-J19 Exam-Released MEE Essay Compilation © 2016-2020 1107


#256-JUL 2002–MEE Q01: QUESTION ONE (UCC ART. 9)

Debtor bought a motor home from Uptown RV Sales on credit. Debtor signed a security agreement
granting Uptown a security interest in the motor home. Two years later, Debtor lost her job and then
defaulted on her loan by failing to make several monthly payments. Without sending any notice of default
to Debtor, Uptown dispatched Ernest, one of its employees, to take possession of the motor home.

Ernest located the motor home parked on a public street. When he opened the door with a duplicate key,
he found Debtor inside and told her that he was there to repossess the motor home. Debtor began yelling
at him, “Get out of my home or I’ll throw you out! This is the only place I have to live, and, anyway, you
don’t have any right to take my clothes and other stuff.” Ernest departed without the motor home.

Two weeks later, the owner of Uptown sent Ernest back, this time to post a coupon on the wind-shield of
the motor home fictitiously advertising a free steak dinner at the grand opening of a local restaurant on
Friday evening. Debtor fell for the ploy. She went to the restaurant on Friday evening, parked the motor
home in the lot at the rear of the restaurant, and went inside to see about her free dinner. She had left the
door to the motor home unlocked.

Ernest, who had followed Debtor, waited for Debtor to get inside the restaurant, entered the motor home
through the unlocked door, “hot-wired” the engine, and drove the motor home back to Uptown’s garage.
Debtor came out of the restaurant to find her motor home was gone.

Uptown’s owner had arranged with a uniformed deputy sheriff to stand by in case it became necessary to
keep the peace. The deputy sheriff observed the events from her patrol car parked some distance away but
did not otherwise assist in the repossession.

1. Did Uptown have the right to repossess the motor home without sending notice of default to
Debtor and without judicial process? Explain.

2. What arguments might Debtor reasonably make, based on the facts, that Uptown failed to carry
out the repossession in a lawful manner? Explain.

Seperac-J19 Exam-Released MEE Essay Compilation © 2016-2020 1108


#256: J02-1 MEE: ANSWER: NCBE (UCC ART. 9)

POINT (1) [25%] ISSUE: Does a secured party have the right to a self-help repossession without
prior notice of default to the debtor and without judicial process? ANSWER: Yes. Following
default, a secured party has the right to take possession of the collateral without prior notice of
default and without judicial process.

POINT (2) [75%] ISSUE: Under the circumstances of this case can it be reasonably argued that the
repossession was carried out in breach of the peace? ANSWER: Yes. Debtor could argue that
Uptown’s repossession was in breach of the peace because it involved an unconsented entry into
Debtor’s place of residence, it was accomplished by “hot-wiring” the vehicle, it enlisted the aid of a
law enforcement officer, and it was carried out over the verbal protests of Debtor.

ANSWER DISCUSSION:

After a debtor’s default, a secured party may take possession of the collateral without notice and without
judicial process. However, this right to self-help repossession exists only if the repossession can be
accomplished without a “breach of the peace,” a standard that the UCC leaves to the courts to define.
Because the “breach of the peace” standard is open-ended and subject to judicial interpretation, the
outcome of this case cannot be definitively stated. But the facts provide a basis for Debtor to make a
reasonable argument that Uptown should have abandoned its attempt at self-help repossession and relied
on judicial process to enforce its claim. First, Debtor had made it clear to Uptown that she would resist
non-judicial repossession, and Uptown relied on trickery to avoid that resistance. A number of courts
disfavor such “fraudulent” conduct by creditors. Second, the repossession was arguably done by an
unauthorized entry into Debtor’s “home,” required “hot-wiring” of the motor home (ordinarily the act of a
thief), and involved the incidental (but intentional) seizure of possessions of Debtor to which Uptown had
no lawful claim. Such “trespassory” conduct may also be disfavored. In addition, the creditor’s use of a
police officer (without securing judicial approval) is also treated by courts as improper, though the police
officer did not actually effect this repossession.

ANSWER EXPLANATION:

Explanation to Point-One (20-30%):

Following default, a secured party has the right to take possession of the collateral without prior notice of
default and without judicial process.

The secured party’s right to repossess the collateral arises on the debtor’s default. There is nothing in the
Code that requires the secured party to give either notice of default or notice of intent to repossess.
Repossession can be through self-help and without the need to commence judicial proceedings. The only
limitation is that the repossession must be accomplished without a breach of the peace.

Under the facts, Debtor, having missed “several monthly payments,” was clearly in default. Accordingly,
Uptown was free to proceed with the self-help remedy of repossession.

Explanation to Point-Two (70-80%):

Seperac-J19 Exam-Released MEE Essay Compilation © 2016-2020 1109


Debtor could argue that Uptown’s repossession was in breach of the peace because it involved an
unconsented entry into Debtor’s place of residence, it was accomplished by “hot-wiring” the vehicle, it
enlisted the aid of a law enforcement officer, and it was carried out over the verbal protests of Debtor.

UCC § 9-609 authorizes the secured party to engage in self-help “without judicial process if this can be
done without breach of the peace.” The term “breach of the peace” is not defined in the Code. Rather, the
drafters relied on pre-Code case law to prescribe the parameters of permissible conduct. Particularly
important facts include (1) whether the creditor has entered upon the premises of the debtor and (2)
whether the debtor has consented to the entry and repossession. Moreover, it is the potential for violence
and not necessarily the occurrence or imminence of violence that defines a breach of the peace.
Ordinarily, repossession of a vehicle from a driveway or public street or parking lot would not be found,
without more, to be in breach of the peace. The matter is complicated in this case, however, because the
motor home could be viewed as Debtor’s residence.

Most courts would find that an unauthorized entry into the residence of a debtor would, in and of itself,
constitute a breach of the peace. In this situation, if the motor home is viewed as Debtor’s home, it is
likely that unauthorized entry would be held to be trespassory and a breach of the peace. The fact that
Debtor lived in the motor home and it contained Debtor’s “clothes and other stuff” supports Debtor’s
claim that the motor home was her residence, so that the unauthorized entry would be a breach of the
peace. Any effort to take the motor home in that manner probably would be considered a breach of the
peace.

Debtor might also argue that her initial objection was enough to preclude any further attempts at self-help
repossession without her consent. Under this view, the fact that the repossession occurred a couple of
weeks later and under circumstances where there was little potential for violence probably would not
change the outcome. It was clear that Debtor opposed the repossession and would have resisted had she
known that the motor home was about to be taken. On the other hand, it can be argued that the passage of
time served to cool down the threat of violence. If so, Debtor’s initial objection to repossession is
irrelevant to an evaluation of the later action.

Even though Uptown’s entry into the motor home was accomplished without force, the fact that Ernest,
the employee, “hot-wired” the engine might arguably be found to be a breach of the peace. For example,
one court found that breaking the lock to a debtor’s mobile home in order to gain entry constituted a
breach of the peace. Another court found that cutting a locked chain on a ranch gate constituted a breach
of the peace.

The fact that the repossession was accompanied by trickery (i.e., the fictitious coupon) may matter. The
courts go both ways. Some courts find that trickery is not a breach of the peace, while other courts find
that trickery is a breach of the peace. The fact that the deputy sheriff was standing by does not contribute
to a finding of breach of the peace because the deputy did not participate or serve to intimidate Debtor.
There is no evidence that Debtor saw the deputy or was at all affected by the deputy’s presence.

The argument could go either way. On balance, however, a court would probably find a breach of the
peace under the circumstances and that the repossession was unlawfully carried out.

Seperac-J19 Exam-Released MEE Essay Compilation © 2016-2020 1110


#257-FEB 2002–MEE Q02: QUESTION TWO (UCC ART. 9)

On January 2, Bank loaned Debtor $5,000 and took a security interest in Debtor’s “equipment now owned
or hereafter acquired.” On that date, Debtor signed a valid security agreement and financing statement
that properly described the collateral. The security agreement contained a proper future advance clause.
On January 3, Bank filed the financing statement in the correct locations.

On February 1, Seller sold a $10,000 computer to Debtor on credit for use in Debtor’s business. To secure
payment of the purchase price, Seller took a security interest in the computer. Also on February 1, Debtor
signed a valid security agreement and financing statement for Seller that properly described the collateral.
Seller, however, did not file the financing statement. Debtor received possession of the computer on
February 12 and began using it in her business on that same day.

On February 14, the sheriff lawfully seized the computer pursuant to a writ of execution obtained by
Larry, a judgment creditor of Debtor, and sent notice of the levy to Bank. Bank received the notice on
February 16. On February 17, Bank advanced Debtor $12,000.

On February 20, Seller learned of Larry’s levy. That same day, Seller filed in the correct locations the
financing statement it had received from Debtor on February 1.

On March 1, Debtor defaulted on her obligation to Bank and on her obligation to Seller.

1. As between Bank and Seller, which has the superior security interest in the computer? Explain.

2. As between Seller and Larry, which has a superior interest in the computer? Explain.

3. Is Bank’s security interest in the computer as collateral for the $12,000 advance superior to
Larry’s claim? Explain.

Seperac-J19 Exam-Released MEE Essay Compilation © 2016-2020 1111


#257: F02-2 MEE: ANSWER: NCBE (UCC ART. 9)

POINT (1) [37%] ISSUE: Does a purchase money security interest perfected later than a general
security interest prevail over the earlier perfected general security interest? ANSWER: Yes.
Because Seller perfected a purchase money security interest (PMSI) within 20 days after Debtor
received possession of the computer, Seller prevails over Bank by virtue of the PMSI exception in
UCC § 9-324.

POINT (2) [37%] ISSUE: Does an initially unperfected purchase money secured creditor prevail
over a later lien creditor? ANSWER: Yes. Because Seller filed within 20 days after Debtor received
possession of the computer, Seller prevails over Larry by virtue of the PMSI exception in UCC § 9-
317.

POINT (3) [26%] ISSUE: Does a lien creditor prevail over a secured creditor with respect to a post-
lien future advance? ANSWER: No. Because Bank’s future advance was made within 45 days of
Larry’s becoming a lien creditor, Bank has priority in the future advance.

ANSWER EXPLANATION:

Explanation to Point-One (30-40%):

Because Seller perfected a purchase money security interest (PMSI) within 20 days after Debtor received
possession of the computer, Seller prevails over Bank by virtue of the PMSI exception in UCC § 9-324.

As between two secured creditors, the general priority rule is found in UCC § 9-322 – first to file or
perfect has priority. Bank filed on January 3 and it perfected on that date as well, since its security interest
had previously attached (i.e., on January 2, Debtor had rights in its existing equipment, Bank gave value
through its loan, and Debtor signed a security agreement). Seller, however, did not file and perfect until
February 20. Thus, under the general “first to file or perfect” rule, Bank would have priority over Seller.

There is, however, an exception to the general rule. Under UCC § 9-324, a PMSI in collateral other than
inventory has priority over a conflicting security interest in the same collateral if the PMSI is perfected at
the time the debtor receives possession of the collateral or within 20 days thereafter. Seller did take a
PMSI because Seller took a security interest to secure all or part of the price of the computer that it sold.
In addition, it is a PMSI in “collateral other than inventory” because the problem states that the computer
is “for use in Debtor’s business.” Thus, the computer is properly characterized as “equipment” of Debtor.
Finally, as mentioned, the PMSI was perfected on February 20 – within 20 days after Debtor received
possession of the computer, because Debtor received possession on February 12. Thus, pursuant to the
PMSI exception, Seller has priority over Bank. (Note that Bank does hold a conflicting security interest
because its after-acquired property clause attached to the computer as soon as Debtor had rights in it,
which was no later than February 12, when Debtor took possession.

Explanation to Point-Two (30-40%):

Because Seller filed within 20 days after Debtor received possession of the computer, Seller prevails over
Larry by virtue of the PMSI exception in UCC § 9-317.

Seperac-J19 Exam-Released MEE Essay Compilation © 2016-2020 1112


Larry qualifies as a lien creditor because he acquired his lien on Debtor’s computer by way of the sheriff’s
levy on February 14. As between a secured creditor and a lien creditor, the general priority rule is found
in UCC § 9-317 and § 9-323 – a perfected security interest prevails over a lien creditor whose lien arises
after the time of perfection. Under the majority rule, Larry became a lien creditor on February 14 – the
date of the sheriff’s levy. Another way of viewing it is to say that “The unperfected security interest is
subordinate to the rights of a lien creditor.” Seller did not perfect until February 20. Thus, under the
general rule, Larry would prevail over Seller.

There is, however, an exception to this general rule as well. UCC § 9-317 notes that a secured party who
files with respect to a PMSI within 20 days after the debtor receives possession of the collateral takes
priority over the rights of a lien creditor that arise between the time the security interest attaches and the
time of filing. As previously explained, Seller’s interest is a PMSI, and Seller filed within the 20-day
period after Debtor received possession (filing on February 20; Debtor’s possession on February 12).
Moreover, Larry’s lien arose on February 14, which is between the time of Seller’s attachment (February
12) and the time of filing (February 20). Thus, under the UCC § 9-317 exception, Seller prevails over
Larry.

Explanation to Point-Three (20-30%):

Because Bank’s future advance was made within 45 days of Larry’s becoming a lien creditor, Bank has
priority in the future advance.

The general rule for contests over future advances between lien creditors and secured creditors is found in
UCC § 9-323. That section provides that lien creditors are subordinate to perfected security interests only
to the extent that the interest secures advances made (1) before the lien arose; (2) within 45 days of the
lien; (3) without knowledge of the lien; or (4) pursuant to a commitment entered into without knowledge
of the lien.

The problem is somewhat complicated because the facts indicate that Bank had knowledge of the lien on
February 16 – one day before the future advance was made. Some applicants may believe, therefore, that
Bank’s knowledge of the lien precludes it from gaining priority in the future advance. Pursuant to UCC §
9-323, however, future advances made within 45 days of the lien, even if the secured creditor is aware of
the lien’s existence, take precedence over the lien. As previously explained, the lien arose on February 14.
The advance was made on February 17 – well within the 45-day window of UCC § 9-323. Thus, Bank’s
security interest prevails over Larry’s lien as to the future advance.

Seperac-J19 Exam-Released MEE Essay Compilation © 2016-2020 1113


#258-JUL 2001–MEE Q03: QUESTION THREE (UCC ART. 9)

Arcade, Inc. is in the business of selling pinball machines. Arcade needed funds to buy furniture for its
showroom. On February 1, Arcade borrowed $10,000 from Bank and signed a valid financing statement,
which stated that it covered all of Arcade’s “inventory now owned or hereafter acquired.” Bank neglected
to obtain a signed security agreement from Arcade but properly filed the financing statement in the correct
locations on February 12.

On June 1, Arcade borrowed $40,000 for working capital from Finance and signed a valid security
agreement granting to Finance a security interest in its “inventory now owned or hereafter acquired.” Also
on June 1, Arcade signed financing statements, which properly described the collateral. On June 5,
Finance filed proper financing statements in the correct locations.

On July 1, Bank realized that it had failed to obtain a security agreement from Arcade. On that date and at
Bank’s request, Arcade signed a valid security agreement, properly describing the collateral as “inventory
now owned or hereafter acquired.”

On August 20, Arcade purchased three pinball machines from Supplier on credit. The three machines
were set aside in Supplier’s warehouse and tagged “Sold to Arcade.” Supplier took a security interest in
the three pinball machines to secure payment of the purchase price. Also on August 20, Arcade signed
both a valid security agreement covering the three pinball machines and valid financing statements.
Supplier filed the financing statements in the correct locations on August 21 and sent proper notice of its
security interest to Bank and Finance, both of whom received and read the notice by August 25. Arcade
took possession of the machines on August 31 and placed them in its showroom for sale to customers.

On October 1, Oscar purchased a pinball machine from Arcade. This machine was one of the three
machines that Arcade had purchased from Supplier. Oscar had no actual knowledge of any of the security
interests held by Arcade’s creditors.

Arcade defaulted on its loans to Bank and Finance and failed to pay Supplier. At the time of the default,
Arcade’s inventory consisted only of the two remaining pinball machines that had been previously
purchased from Supplier. Bank, Finance, and Supplier are all claiming security interests in the two
remaining pinball machines, as well as in the pinball machine purchased by Oscar.

1. What is the order of priority among Bank, Finance, and Supplier in their respective claims to a
security interest in the two remaining pinball machines left in Arcade’s inventory? Explain.

2. Can Bank, Finance, or Supplier successfully enforce a security interest in the pinball machine
purchased by Oscar? Explain.

Seperac-J19 Exam-Released MEE Essay Compilation © 2016-2020 1114


#258: J01-3 MEE: ANSWER: NCBE (UCC ART. 9)

POINT (1) [65%] ISSUE: What is the relative priority of the security interests of Arcade’s three
creditors? ANSWER: Under the first-to-file-or-perfect rule, Bank would have priority over Finance
and Supplier. However, Supplier’s security interest qualifies for priority because it is a purchase
money security interest (PMSI) and Supplier complied with the requirements of UCC § 9-312.
Accordingly, Supplier has priority, followed by Bank, then Finance.

POINT (2) [35%] ISSUE: Does Oscar, as a buyer in the ordinary course of business, take free of
security interests created by Arcade? ANSWER: Yes. Because Oscar is a buyer in the ordinary
course, he takes free of all security interests created by his seller (Arcade). Thus, Bank, Finance,
and Supplier have no security interest to enforce in the pinball machine purchased by Oscar.

ANSWER EXPLANATION:

Explanation to Point-One (60-70%):

Under the first-to-file-or-perfect rule, Bank would have priority over Finance and Supplier. However,
Supplier’s security interest qualifies for priority because it is a purchase money security interest (PMSI)
and Supplier complied with the requirements of UCC § 9-312. Accordingly, Supplier has priority,
followed by Bank, then Finance.

As between secured creditors, the general priority rule is UCC § 9-312(5) – the first to file or perfect has
priority. Bank filed on February 12 although it did not perfect until July 1 when Arcade signed a security
agreement on Bank’s behalf. Under Article 9 of the UCC, perfection requires attachment and that
attachment typically requires a signed security agreement. Pursuant to the general rule, however, February
12 is Bank’s priority date.

Finance filed and perfected on June 5 since its security interest had previously attached (i.e., on June 1,
Arcade had rights in its existing inventory, Finance gave value through its loan, and Arcade signed a
security agreement). Thus, June 5 is Finance’s priority date. Bank, therefore, beats Finance in the
remaining inventory. (Both have an after-acquired property clause in their security agreements that gives
rise to a security interest in any new inventory that Arcade purchases.)

Supplier filed and perfected on August 21, assuming that Arcade had some rights in the collateral as of
the purchase date of August 20. For example, the actual pinball machines that were the subject of the sale
were identified on that date. In any event, perfection would occur no later than possession by Arcade on
August 31. Pursuant to the general rule, Supplier’s priority is behind both Bank and Finance. Supplier,
however, may take advantage of an exception to the general rule for purchase money security interests
(PMSIs) in inventory. Section 9-312(3) of the UCC states that a perfected PMSI in inventory has priority
over a conflicting security interest in the same inventory if the PMSI is perfected at the time the debtor
receives possession of the inventory, and if the conflicting security interest holders receive proper notice
within five years before the debtor receives possession of the inventory.

Supplier did take a PMSI because Supplier took a security interest to secure all or part of the price of the
pinball machines that it sold to Arcade. In addition, it is a PMSI in inventory because Arcade is a seller of
pinball arcade games; thus, the machines purchased by Arcade are presumably “held by Arcade who
holds them for sale.” Indeed, the facts of the problem state that the machines were placed in Arcade’s

Seperac-J19 Exam-Released MEE Essay Compilation © 2016-2020 1115


showroom “for sale to customers.” Thus, the pinball machines are properly characterized as “inventory”
of Arcade.

As mentioned, Supplier filed and probably perfected on August 21, which indicates that the PMSI was
perfected “at the time Arcade received possession of the inventory” on August 31. Finally, the facts of the
problem indicate that “proper notice of Supplier’s security interest was sent to all relevant creditors,
including Bank and Finance,” and that such notice was received before Arcade got possession of the
machines on August 31. Thus, the notice requirements of UCC § 9-312(3) are met. (The facts of the
problem state that “proper” notice was sent so there is no need to discuss whether the content of the notice
was correct.) Because the requirements of UCC § 9-312(3) are met, Supplier has priority in the pinball
machines over both Bank and Finance, as they hold “conflicting security interests in the same inventory.”
(Note that Bank and Finance hold conflicting security interests because their after-acquired property
clauses attached to the pinball machines as soon as Arcade had rights in the collateral, which would be no
later than possession on August 31).

In summary, therefore, the order of priority in the two remaining pinball machines is (1) Supplier, (2)
Bank, and (3) Finance.

Explanation to Point-Two (30-40%):

Because Oscar is a buyer in the ordinary course, he takes free of all security interests created by his seller
(Arcade). Thus, Bank, Finance, and Supplier have no security interest to enforce in the pinball machine
purchased by Oscar.

The general rule is that security interests are enforceable against purchasers of the collateral. Indeed, in
general, security interests continue in collateral notwithstanding its sale, exchange, or other disposition.
Under UCC § 9-307, however, a buyer in the ordinary course of business takes free of a security interest
created by his seller even if the security interest is perfected and even if the buyer knows of the security
interest. A buyer in the ordinary course means a person who, in good faith and without knowledge that the
sale to him is in violation of the security interest of a third party, buys in the ordinary course from a
person in the business of selling goods of that kind. Arcade is in the business of selling pinball arcade
games. There are no facts suggesting that Oscar acted in bad faith. In addition, the problem states that
Oscar “had no actual knowledge” of any security interests in the pinball machine he purchased.

Because Oscar’s purchase satisfies the requirements of UCC § 9-307, Oscar takes the pinball machine
free of all three security interests. Bank, Finance, and Supplier have no security interest to enforce in the
pinball machine purchased by Oscar.

Seperac-J19 Exam-Released MEE Essay Compilation © 2016-2020 1116


#259-JUL 2000–MEE Q06: QUESTION SIX (UCC ART. 9/CONTRACTS)

Dealer owns and operates an antique store. On January 4, Bank loaned $250,000 to Dealer to finance
Dealer’s business operations. In connection with the loan, Dealer signed a security agreement that granted
Bank a security interest in Dealer’s “inventory” and assigned Dealer’s “accounts and chattel paper” to
Bank. The agreement covered “any and all obligations owed by Dealer to Bank including future
advances.” Later that day, Bank properly filed a financing statement reflecting its security interest.

On May 1, Thief stole a 17th-century cedar chest from Owner. Thief then sold it to Dealer. Dealer did not
know the chest had been stolen. On June 1, Dealer sold the cedar chest to Purchaser, an interior designer,
for $50,000. Purchaser bought the cedar chest for resale to a client.

Purchaser made a $10,000 down payment and signed an installment sales contract for the remainder of the
purchase price, plus interest, to be paid monthly over 12 months. The installment sales contract granted to
Dealer a security interest in the cedar chest and provided that all payments were to be made directly to
Dealer. Purchaser subsequently made the payments due on July 1, August 1, and September 1 by sending
checks to Dealer.

On September 1, Dealer failed to make a payment due on its loan from Bank. Bank immediately declared
Dealer in default, as it was permitted to do under the loan and security agreements between the parties.
Bank also sent a letter to Purchaser informing Purchaser of Dealer’s default, advising Purchaser of Bank’s
security interest, and directing Purchaser to make all further payments on the installment sales contract
directly to Bank. Purchaser contacted Dealer, who admitted that he was in default and that Bank had a
security interest. However, Dealer told Purchaser, “Don’t worry. 1 will pay Bank my September payment
soon. Things will be straightened out. You should continue to send your monthly payments directly to me.
Bank has no right to collect from you in any event. Our arrangement is between us.”

On September 20, Purchaser was served with papers in a replevin action brought by Owner to recover the
cedar chest. According to the complaint in the replevin action, the cedar chest had been stolen from
Owner, and the bill of sale and other documents presented to Purchaser by Dealer as proof of Dealer’s
ownership were forgeries. Purchaser immediately answered the replevin action with a pleading alleging
(a) that as a good faith purchaser for value, Purchaser’s claim was superior to Owner’s and (b) that any
right of Owner to recover the cedar chest should be conditioned on Owner’s reimbursing Purchaser for
amounts paid to Dealer.

1. Who should prevail in Owner’s replevin action to recover the cedar chest? Explain.

2. Assuming Purchaser prevails in the replevin action, should future payments on the installment
sales contract be made to Dealer or to Bank? Explain.

3. Assuming Purchaser loses in the replevin action, does Purchaser continue to be liable for the
payments on the installment sales contract either to Dealer or Bank? Explain.

Seperac-J19 Exam-Released MEE Essay Compilation © 2016-2020 1117


#259: J00-6 MEE: ANSWER: NCBE (UCC ART. 9/CONTRACTS)

POINT (1) [33%] ISSUE: Who has the superior claim to personal property, a previous owner from
whom the property was stolen or a good faith purchaser for value? ANSWER: An owner of stolen
property may recover that property from a subsequent purchaser without reimbursing the
purchaser and regardless of the purchaser’s good faith in the transaction in which the purchaser
acquired the property.

POINT (2) [33%] ISSUE: May a secured party with a security interest in accounts and/or chattel
paper require the obligors on the accounts to pay it directly upon the default of the debtor on the
secured obligation? ANSWER: Yes. After default by the principal debtor, a secured party with a
security interest in accounts or chattel paper is entitled to collect payments directly from the
account debtors.

POINT (3) [33%] ISSUE: May an account debtor raise defenses arising out of the underlying sales
contract against the obligee of the account or assignees? ANSWER: Yes. Purchaser has a breach of
warranty claim against Dealer. This claim may be asserted against either Dealer or Bank as a
defense to payment on the installment sales contract. Thus, Purchaser is not liable for payments to
either Dealer or Bank.

ANSWER EXPLANATION:

Explanation to Point-One (30-40%):

An owner of stolen property may recover that property from a subsequent purchaser without reimbursing
the purchaser and regardless of the purchaser’s good faith in the transaction in which the purchaser
acquired the property.

Section 2-403 of the Uniform Commercial Code adopts the common-law principle of nemo dat: no one
can give or sell what he does not have. Thus, under Article 2, a purchaser of goods acquires only that title
“which his transferor had or had power to transfer.”

On the facts of this problem, the cedar chest was stolen from Owner. The thief who stole it had no title
and no power to transfer title. Any purchaser from the thief would therefore acquire no title. Subsequent
purchasers would likewise acquire only such title as their transferors had, i.e., no title.

There are exceptions to the nemo dat rule. In particular, good faith purchasers for value are sometimes
able to acquire good title even if their transferors had no title or defective title. However, the facts of this
problem do not raise even a possibility of applying any of the recognized protections for good faith
purchasers. In particular, the protection granted to buyers in the ordinary course from merchants who deal
in goods of the kind purchased (e.g., the antique dealer in this problem) applies only if the goods were
entrusted to the merchant by the true owner. Nothing of the sort happened in this problem.

Consequently, Owner will prevail in the replevin action, and Purchaser will be forced to give up the cedar
chest with no reimbursement from Owner for payments already made to Dealer. Purchaser’s remedy is
against Dealer.

Explanation to Point-Two (30-40%):

Seperac-J19 Exam-Released MEE Essay Compilation © 2016-2020 1118


After default by the principal debtor, a secured party with a security interest in accounts or chattel paper is
entitled to collect payments directly from the account debtors.

The obligation owed by Purchaser to Dealer under the installment sales contract is either an account or
chattel paper (if the installment sales contract includes a provision creating a security interest in favor of
Dealer). In either case, Uniform Commercial Code § 9-502 permits a secured party to “notify an account
debtor to make payment” when the account creditor (Dealer) is in default on the secured obligation. The
purpose is to permit a party whose financing is secured by accounts or chattel paper to “liquidate by
collecting whatever may become due on the collateral.” The account debtor must, after notification from
the secured party, make payments directly to the secured party rather than to the defaulting debtor, at least
insofar as there is no question about the legitimacy of the secured party’s claim. Under Article 9 of the
UCC, after notice of an assignment of an account and that payment is to be made to the assignee, the
account debtor must pay the assignee, subject to the account debtor’s right to demand proof of the
assignment.

Dealer’s statements to Purchaser did not deny that Bank had a security interest or that Dealer was in
default. Dealer simply denied that Bank had collection rights. Article 9, however, clearly grants Bank
such rights in the event of Dealer’s default, even if the security agreement does not give Bank collection
rights. Consequently, Purchaser should make payments to Bank and not to Dealer.

Explanation to Point-Three (30-40%):

Purchaser has a breach of warranty claim against Dealer. This claim may be asserted against either Dealer
or Bank as a defense to payment on the installment sales contract. Thus, Purchaser is not liable for
payments to either Dealer or Bank.

As the facts make clear, Dealer had no title to the cedar chest that was sold to Purchaser. Under § 2-312 of
the Uniform Commercial Code, every seller of goods makes a warranty of good title to the buyer, unless
that warranty is “excluded or modified only by specific language or by circumstances” that put the buyer
on notice that the seller claims no title. Nothing in these facts indicates any exclusion or modification of
the warranty of good title.

Purchaser’s remedies in the case of breach of warranty of good title for accepted goods include revocation
of acceptance and include recovery for “the loss resulting in the ordinary course of events from the
seller’s breach.” In this case, of course, Purchaser has lost the cedar chest and its full value. This loss is an
ample defense to any attempt on the part of Dealer to recover further payments on the installment sales
contract. This argument can also be effectively couched in terms of failure of consideration, with the same
result.

Bank’s efforts to collect on the installment sales contract are also subject to Purchaser’s claim against
Dealer for losses due to Dealer’s breach of the warranty of title. Section 9-318 of the Code provides that
any assignee of accounts or chattel paper is subject to “any defense or claim arising” out of the contract
between the account debtor and the assignor unless the account debtor has agreed not to assert defenses.
On our facts, no such agreement is indicated. An assignee-secured creditor exercising collection rights
under UCC § 9-502 is subject to the account debtor’s defenses under § 9-318, just as any other assignee
would be subject to those defenses. Accordingly, Purchaser has no obligation to pay Bank.

Seperac-J19 Exam-Released MEE Essay Compilation © 2016-2020 1119


#260-FEB 2000–MEE Q01: QUESTION ONE (UCC ART. 9)

Jane wanted to purchase a computer for home use and for her young children to use for school work and
entertainment. On January 15, Jane went to Dealer, a retail store specializing in computers, and told a
salesperson of her family’s needs. She was shown a computer that she was told would suffice. She agreed
to buy the computer and told Dealer that she would have to borrow the money to pay for it. Dealer agreed
to finance the sale and gave Jane $3,000 in credit to buy the computer. Jane took delivery and signed a
security agreement that described the computer and granted Dealer a security interest in it. Dealer did not
file a financing statement.

Immediately after purchasing the computer from Dealer, Jane changed her mind about how she would use
it. Jane had just begun work as an independent consultant, so she took the computer to her office and
began using it for business purposes. She did not inform Dealer of this change of use.

On May 1, Jane borrowed $25,000 from Bank as operating capital for her business. She executed a
security agreement granting Bank a security interest in her office machinery, including the computer. Also
on May 1, Bank filed a valid financing statement in all proper public offices.

Jane defaulted on the loans to both Dealer and Bank. On December 1, Dealer peacefully repossessed the
computer, Both Dealer and Bank claim a security interest in the computer.

As between Dealer and Bank, whose security takes priority? Explain.

Seperac-J19 Exam-Released MEE Essay Compilation © 2016-2020 1120


#260: F00-1 MEE: ANSWER: NCBE (UCC ART. 9)

POINT (1) [23%] ISSUE: Should the computer be classified as a consumer good or as equipment?
ANSWER: When Jane bought the computer, she expressed to Dealer an initial intent to use it for
“personal, family, or household” use. Therefore, the computer would most likely be classified as a
consumer good.

POINT (2) [33%] ISSUE: Does Dealer have a perfected security interest? ANSWER: Yes. Dealer
acquired a purchase money security interest, which attached and was automatically perfected at the
moment of sale without the need for the filing of a financing statement.

POINT (3) [23%] ISSUE: Does Bank have a perfected security interest? ANSWER: Yes. Bank’s
security interest attached and was perfected by filing on May 1.

POINT (4) [23%] ISSUE: Does Dealer’s security interest take priority over Bank’s security
interest? ANSWER: Assuming that the collateral is properly classified as a consumer good.
Dealer’s security interest takes priority over Bank’s because Dealer was the first to perfect its
security interest. Bank’s security interest would take priority over Dealer’s if the collateral were
classified as equipment.

ANSWER EXPLANATION:

Explanation to Point-One (20-25%):

When Jane bought the computer, she expressed to Dealer an initial intent to use it for “personal, family, or
household” use. Therefore, the computer would most likely be classified as a consumer good.

The relevant rules for resolving the priority dispute between Dealer and Bank will be determined by how
one classifies the collateral. To do that, one must look at the expressed intent at the time of purchase and
the use of the collateral in the hands of the debtor. The collateral’s intended use at the time of the sale was
as consumer goods under § 9-109 of the Uniform Commercial Code. Jane originally intended to use the
computer for “personal, family, or household” purposes. However, Jane’s actual use turned out to be
“primarily in business.” If the latter use governs, the collateral will be “equipment” as defined in UCC §
9-109.

Which use governs? One approach is that the debtor’s intended use at the time of the sale should govern
the classification. This approach is justified on the policy ground that the secured party should be able to
rely on what the debtor represents as the intended use and should not be encouraged to police the
collateral and keep the debtor under surveillance. Moreover, UCC § 9-109 states that goods are consumer
goods if they are “used or bought” for personal use. Accordingly, the computer should be classified as a
consumer good.

There is, however, authority for the proposition that the ultimate use controls. If the latter view is adopted,
the computer will be deemed to be “equipment.”

Explanation to Point-Two (30-35%):

Seperac-J19 Exam-Released MEE Essay Compilation © 2016-2020 1121


Dealer acquired a purchase money security interest, which attached and was automatically perfected at the
moment of sale without the need for the filing of a financing statement.

Assuming that the collateral is correctly classified as a consumer good, Dealer enjoyed a purchase money
security interest in it. Under UCC § 9-107, a purchase money secured creditor includes a seller who
allows the debtor to purchase the goods on credit and takes a security interest in the goods sold. Under
UCC § 9-302, filing is not required to perfect a purchase money security interest in consumer goods. The
security interest is perfected automatically upon its attachment because the burden of filing in numerous
small transactions would outweigh the benefits achieved from notice to the public. Accordingly, Dealer’s
failure to file a financing statement was of no consequence.

There is no doubt here that a security interest attached in Dealer’s favor at the point of sale. Under UCC §
9-203 there are three requirements for attachment of a security interest: (1) The secured party must give
value, which includes “the extension of immediately available credit”; (2) The debtor must have sufficient
property rights in the collateral, which Jane had when she took delivery of the computer (3) Finally, for
non-possessory collateral (i.e., collateral not in possession of the creditor), the debtor must have signed a
security agreement, which Jane did.

If, on the other hand, the actual use of the computer governs the classification of the collateral, then the
computer will be viewed as equipment, in such a case, Dealer’s security interest would have attached by
virtue of Jane’s having signed the security assignment, but would have remained unperfected initially as a
result of Dealer’s failure to file. However, under UCC § 9-305, a security interest in goods may be
perfected by the secured party’s taking possession of the collateral. In this case, Dealer’s perfection would
have been delayed until December 1, when Dealer peacefully repossessed the computer following Jane’s
default.

Explanation to Point-Three (20-25%):

Bank’s security interest attached and was perfected by filing on May 1.

Bank’s security interest attached because the three elements of UCC § 9-203 were met: (1) Bank gave
value by extending credit to Jane (2) Jane already owned the computer collateral and had the necessary
property rights; and (3) Jane signed a security agreement. Because Bank properly filed a financing
statement, Bank had a perfected security interest in Jane’s equipment on May 1.

Explanation to Point-Four (20-25%):

Assuming that the collateral is properly classified as a consumer good. Dealer’s security interest takes
priority over Bank’s because Dealer was the first to perfect its security interest. Bank’s security interest
would take priority over Dealer’s if the collateral were classified as equipment.

Priority between two perfected security interests is governed by the first-to-file-or-perfect rule. Assuming
that Dealer’s security interest was automatically perfected, it would take priority over Bank’s later filing.
Dealer’s security interest was automatically perfected on January 15 when the security interest attached.
Bank’s security interest was perfected on May 1 when it made a conforming central filing.

On the other hand, if a court were to hold that actual use governed the classification of the computer for
Dealer, it would be classified as equipment, and Dealer would not enjoy automatic perfection. Dealer’s
perfection would be delayed until December 1 when it perfected by taking possession of the computer.

Seperac-J19 Exam-Released MEE Essay Compilation © 2016-2020 1122


The priority result would then be reversed, because Bank would be the first secured party to file or
perfect. Bank filed its financing statement on May 1, long before Dealer repossessed.

Seperac-J19 Exam-Released MEE Essay Compilation © 2016-2020 1123


#261-FEB 1999–MEE Q07: QUESTION SEVEN (UCC ART. 9)

On April 1, Debtor bought a used car from Smith Motors for his personal use. Debtor financed the sale
with Smith Motors. The cash price was $16,500, and the credit service charge (total interest payments for
the life of the loan) was $4,000. Debtor made a $500 down payment and agreed to pay the remaining
$20,000 in 24 monthly payments. At the time of the purchase, Debtor executed a sales contract and
granted a security interest in the car to Smith Motors. Smith Motors properly perfected that security
interest.

After making only one monthly payment, Debtor defaulted on the contract. Following a peaceful
repossession, Smith Motors decided to sell the car at auction on August 15. At that time, the unpaid
balance of principal and accrued interest was $ 17,000. Smith Motors contacted a few dealers but did not
advertise the auction.

On August 12, Smith Motors mailed notice of the August 15 auction by regular mail to Debtor at his
home address. The notice arrived on August 14, one day before the auction. On August 15, the car was
sold at auction for $ 10,000, which was well below the car’s market value of $ 14,000. The only bidder at
the auction was a dealer who specialized in repossessed vehicles.

Smith Motors has sued Debtor for the deficiency of $7,000 between the unpaid contract price and the
amount for which the car was sold.

1. On what theories and in what amounts may Debtor recover damages from Smith Motors under
the Uniform Commercial Code? Explain.

2. Is Debtor liable to Smith Motors for any part of the $7,000 deficiency? Explain.

Seperac-J19 Exam-Released MEE Essay Compilation © 2016-2020 1124


#261: F99-7 MEE: ANSWER: NCBE (UCC ART. 9)

POINT (1)(a) [33%] ISSUE: Was the resale commercially reasonable? ANSWER: No. The resale
was not conducted in a commercially reasonable manner, because Smith Motors did not adequately
publicize the sale, failed to give adequate notice of the sale to Debtor, and sold the car for
substantially less than its market value.

POINT (1)(b) [33%] ISSUE: To what damages is Debtor entitled? ANSWER: Under UCC § 9-507,
Debtor is entitled to recover either the loss caused by Smith Motors’ failure to conduct a
commercially reasonable sale or a minimum recovery in the form of a civil penalty.

POINT (2) [33%] ISSUE: Is Smith Motors entitled to a deficiency judgment against Debtor?
ANSWER: Smith Motors may or may not be entitled to a deficiency judgment against Debtor
depending upon the position of the jurisdiction as to the effect of creditor misbehavior.

ANSWER EXPLANATION:

Explanation to Point-One(a) (30-40%):

The resale was not conducted in a commercially reasonable manner, because Smith Motors did not
adequately publicize the sale, failed to give adequate notice of the sale to Debtor, and sold the car for
substantially less than its market value.

Smith Motors had a purchase money security interest under Uniform Commercial Code § 9-107. Because
the car was used for personal purposes, it would be classified as consumer goods under UCC § 9-109.
Thus any Article 9 default provisions applicable to purchase or non-purchase money security interests in
consumer goods would govern the problem. Smith Motors purportedly held a public sale of the car
pursuant to UCC § 9-504. A public sale is attended by the public pursuant to notice and often
contemplates competitive bidding. Under UCC § 9-504, reasonable notice of a proposed resale must be
sent to Debtor, and any resale must be commercially reasonable as to “method, manner, time, place, and
terms.”

These standards were not met for three reasons:

(1) Smith Motors did not give adequate notice of resale to Debtor. UCC § 9-504 requires that “reasonable
notification of the time and place of any public sale” shall be “sent by the secured party to the debtor.”
Comment 5 to UCC § 9-504 suggests that reasonable notification “must be sent in such time that persons
entitled to receive it will have sufficient time to take appropriate steps to protect their interests by taking
part in the sale or other disposition if they so desire.” The notice was sent by mail and was delivered one
day before the sale was to take place. It was too late to be of any use to Debtor. The purpose of notice – to
give Debtor a chance to redeem the car under UCC § 9-506 or to maximize the number of bidders at the
sale and thereby reduce any deficiency – was thereby thwarted, and the accelerated sale was commercially
unreasonable. None of the UCC § 9-504 exceptions to debtor notice apply: the car was not perishable, it
was not sold on a recognized market, and Debtor did not waive his rights to notice in writing after default.
This was not sufficient for a commercially reasonable sale.

Seperac-J19 Exam-Released MEE Essay Compilation © 2016-2020 1125


(2) Smith Motors did not give sufficient advance publicity to insure the success of a public sale. Advance
publicity is important to increase the likelihood that a public sale will realize the market value of the
collateral. Smith Motors did not do any advertising, and its contacts were with only a few dealers.

(3) The resale price of $10,000 to the lone bidder was below the fair market value of the car. While UCC
§ 9-507 specifies that a low price received at the sale does not always establish a lack of commercial
reasonableness, most courts assume that a low price is a factor in deciding on commercial reasonableness.
When accompanied with procedural irregularities, a low resale price is the “paradigm commercially
unreasonable resale.”

In addition, the resale was at wholesale to a dealer intending to resell. It could be argued that Smith
Motors had sufficient expertise to resell at retail, thereby enhancing the sale price and reducing Debtor’s
deficiency. However, a persuasive counterargument finds that requirement burdensome to retailers and
wholesale disposition to be commercially reasonable.

Explanation to Point-One(b) (30-40%):

Under UCC § 9-507, Debtor is entitled to recover either the loss caused by Smith Motors’ failure to
conduct a commercially reasonable sale or a minimum recovery in the form of a civil penalty.

As it is too late for any injunction against resale, the Uniform Commercial Code gives Debtor two options
regarding damages. Under UCC § 9-507, Debtor is first entitled to recover “any loss caused by a failure to
comply with the provisions” of Article 9 regarding default. Debtor may recover the difference between
the resale price and the fair market value on August 15 ($14,000 - 10,000 = $4,000). In any event,
because the collateral was consumer goods, Debtor may recover, as a civil penalty, not less than the
greater of the credit service charge ($4,000) plus 10% of the principal of the debt ($1,600) or the time
price differential ($4,000) plus 10% of the cash price ($1,650). (Note: Some jurisdictions would not
include the down payment as part of the “cash price.” In such a jurisdiction, 10% of the cash price would
be $ 1,600.) Thus, Debtor could recover a civil penalty of $5,650 ($4,000 + $1,650). Either of these
amounts could be applied to reduce the deficiency owed to Smith Motors, and Debtor may recover the
larger of the two.

Explanation to Point-Two (30-40%):

Smith Motors may or may not be entitled to a deficiency judgment against Debtor depending upon the
position of the jurisdiction as to the effect of creditor misbehavior.

The consequences for failure to publicize the sale and provide notice are provided in UCC § 9-507, which
provides, in part: If it is established that the secured party is not proceeding in accordance with the
provisions of this Part, disposition may be ordered or restrained on appropriate terms and conditions. If
the disposition has occurred, the debtor or any person entitled to notification has a right to recover from
the secured party any loss caused by a failure to comply with the provisions of this Part.

The Uniform Commercial Code nowhere addresses the right of a creditor to recover a deficiency
judgment after violating the rules contained in Part 5 (including the notice requirement of UCC § 9-504).
However, many courts have addressed the effect of the secured party’s violation of such debtor protection
provisions. Courts have taken three approaches, none of which can be clearly identified as the majority.
White and Summers provide an extensive grouping of cases under each approach.

Seperac-J19 Exam-Released MEE Essay Compilation © 2016-2020 1126


The first approach is to deny the secured creditor any deficiency if it is shown that Part 5 of Article 9 has
been violated. In Skeels v. Universal CIT Credit Corp., the court held that “a security holder who sells
without notice may not look to the debtor for any loss.” Thus, under this approach, Debtor has an absolute
defense to Smith Motors’ suit, in addition to his right to recover damages.

A second line of cases applies what is known as the “rebuttable presumption” rule, under which the debtor
who has not been notified has the benefit of a presumption that the value of the collateral was equal to the
debt. Under this rule, Smith Motors would carry the burden of showing that the collateral was worth less
than the amount owed. The amount of Smith Motors’ recovery would be limited to the difference between
the value of the collateral as proved by it and the amount of the claim. In this question, there are not
enough facts with which to calculate Smith Motors’ recovery under this theory.

While a majority of courts would follow one of the two approaches above, a minority of courts allow only
a set-off for the loss proven by the debtor to have been caused by the secured party’s failure to provide
notice, or, alternatively, a set-off for Debtor’s statutory recovery. Again using the evidence provided by
the facts, Debtor may be able to prove damages of $5,650 ($4,000 + $1,650 – see Point One) to offset
against the $7,000 deficiency claim, for a net recovery by Smith Motors of $1,350.

Seperac-J19 Exam-Released MEE Essay Compilation © 2016-2020 1127


#262-JUL 1998–MEE Q04: QUESTION FOUR (UCC ART. 9)

Builder constructs and manages large condominium developments. In January, Builder began work on a
240-unit complex of luxury condominiums known as the “Eden Project.” Builder borrowed $6 million
from Bank to finance this project and granted Bank a construction mortgage on the “land and any
improvements thereon” constituting the Eden Project. Bank recorded the mortgage in the proper real
estate recording office. The mortgage document recited that the mortgage secured funds loaned “for
acquisition of the land and for construction of improvements on the land.”

In March, Builder contacted Seller, an appliance manufacturer, about supplying built-in dishwashers and
ranges, and freestanding clothes washers and dryers for the Eden Project. After preliminary negotiations,
Builder sent Seller a Purchase Order for the necessary numbers and types of appliances at a total price of
$300,000. The Purchase Order also provided that: “Goods are sold to Builder on 120-day credit without
interest. Builder hereby grants Seller an unconditional security interest in any and all goods delivered
under this contract to secure payment of all amounts owing pursuant to this contract.” Builder signed the
Purchase Order.

Seller responded to the Purchase Order by sending Builder an “Order Acknowledgment.” The Order
Acknowledgment recited the quantity and price information that appeared on the Purchase Order and
indicated that payment should be made “on credit, per Builder’s Purchase Order, subject to the terms and
conditions on the reverse of this Acknowledgment.” The back of the Order Acknowledgment contained
several preprinted terms, including one term which read as follows: “In addition to the purchase price, the
buyer shall pay the seller the amount of all shipping and handling charges.” Builder’s Purchase Order did
not mention shipping and handling.

In April, Seller shipped the appliances. Builder installed the appliances in the condominium units in May.
On August 1, Seller sent Builder a bill for $315,000, of which $300,000 was for the appliances and
$15,000 was for shipping and handling.

It is November, and Builder is unable to pay its bills. It has not paid Seller and is far behind on its
payments to Bank.

Seller wishes to repossess the appliances sold to Builder and remove them from the condominium project.

1. Is Builder liable to Seller for the shipping and handling charges? Explain.

2. Does Seller have the right to repossess any or all of the appliances? Explain.

Seperac-J19 Exam-Released MEE Essay Compilation © 2016-2020 1128


#262: J98-4 MEE: ANSWER: NCBE (UCC ART. 9)

POINT (1) [47%] ISSUE: Is the provision for Builder’s payment of “all shipping and handling
charges” a part of the contract between Builder and Seller? ANSWER: No. Because the provision
for payment of shipping and handling charges was an additional term that materially altered the
offer made by Builder in its Purchase Order, the provision did not become part of the contract.

POINT (2)(a) [16%] ISSUE: Does Seller have a security interest in the appliances? ANSWER: Yes.
The sales contract included a written security agreement signed by the debtor. The security interest
attached when Seller delivered the appliances to Builder.

POINT (2)(b) [37%] ISSUE: May Seller repossess the appliances when they have been installed in a
building covered by a prior construction mortgage held by Bank? ANSWER: Yes. Seller may
repossess the appliances only if they are not fixtures under local real estate law.

ANSWER EXPLANATION:

Explanation to Point-One (40-50%):

Because the provision for payment of shipping and handling charges was an additional term that
materially altered the offer made by Builder in its Purchase Order, the provision did not become part of
the contract.

This is a classic “battle of the forms” problem. Builder’s Purchase Order is an offer to buy appliances
from Seller. Seller’s Order Acknowledgment purports to accept the Purchase Order, but the printed terms
on the back of the Order Acknowledgment add a provision that "the buyer (Builder in this case) shall pay
the seller the amount of all shipping and handling charges.” The question is whether this new term is part
of the contract.

Under the traditional common law “mirror image” rule, Seller’s Order Acknowledgment would be a
rejection of Builder’s offer and a counteroffer because the Order Acknowledgment differs significantly
from the offer. Builder’s subsequent acceptance of the appliances would in turn be an acceptance by
conduct of the counteroffer, and Seller would be entitled to recover its shipping and handling charges.

Under the Uniform Commercial Code, the result changes. The Order Acknowledgment, which on its face
appears to accept the Purchase Order/Offer, would operate as an acceptance of the contract under UCC §
2-207, “even though it states terms additional to or different from those offered.” In this case, the
provision for Builder’s reimbursement of Seller’s shipping and handling charges should be considered an
additional term rather than a different term, as there is no indication that the Purchase Order itself
addressed shipping and handling charges in any way.

UCC § 2-207 provides that “additional terms are to be construed as proposals for addition to the
contract.” Additional terms become part of any contract between merchants unless (a) “the offer expressly
limits acceptance to the terms of the offer”; (b) they materially alter it; or (c) notification of objection to
them “is properly given.” In this problem, the question is whether the new provision materially alters the
offer. Neither of the other bases for dropping it applies.

Seperac-J19 Exam-Released MEE Essay Compilation © 2016-2020 1129


The Code does not define “materially alter,” but the comments to § 2-207 indicate that a clause will
materially alter a contract if it changes the bargain in such a way as to “result in surprise or hardship if
incorporated without express awareness by the other party.” A clause of this sort, which has the potential
to significantly increase a buyer’s financial obligation under the contract, is probably a material change in
the parties’ bargain.

On the other hand, payment of shipping and handling charges in addition to the stated price is a normal
occurrence in many sales transactions. Thus, an argument might fairly be made that the disputed provision
on the Order Acknowledgment should not be considered a surprising or hardship-inducing addition to the
contract. One would also need to examine the usage in the trade. If it were shown that it is standard for the
purchaser to pay shipping and handling, this would lead one to conclude that inclusion of the clause
would not materially alter the contract.

Explanation to Point-Two(a) (10-20%):

The sales contract included a written security agreement signed by the debtor. The security interest
attached when Seller delivered the appliances to Builder.

The Purchase Order stated that “Builder hereby grants Seller an unconditional security interest in any and
all goods delivered under this contract.” Nothing in Seller’s Order Acknowledgment, which was an
acceptance of the terms of the Purchase Order, altered that provision. Thus, the provision for a security
interest was part of the contract between the parties.

As a result, the three requirements of Article 9 for attachment of security interest were satisfied by this
transaction. First, the debtor (Builder) must have “signed a security agreement which contains a
description of the collateral.” No particular formalities are imposed in connection with this requirement. A
security agreement is simply “an agreement which creates or provides for a security interest.” The sales
contract would seem to fit this definition, and the Purchase Order, which continues the agreement’s key
terms, was signed by Builder.

The second requirement for a security interest to be created is that value be given. Clearly, Seller gave
value when it provided appliances to Builder. Finally, Builder must have rights in the collateral. Again,
this requirement was satisfied when Seller transferred the appliances to Builder pursuant to the sales
contract.

Explanation to Point-Two(b) (30-40%):

Seller may repossess the appliances only if they are not fixtures under local real estate law.

A secured party ordinarily has the right to repossess collateral that secures an obligation when the debtor
is in default on the obligation, as is the case here.

However, Bank’s presence as holder of a construction mortgage on the Eden Project may defeat Seller’s
repossession rights. The issue turns on whether the appliances became “fixtures” when they were installed
in the Eden Project’s condominiums.

The Code provides that “goods are ‘fixtures’ when they become so related to particular real estate that an
interest in them arises under real estate law.” This provision leaves the determination of the status of
goods as fixtures to the local real property law of the jurisdiction concerned. In most jurisdictions this is a
fairly complicated property law issue, but the facts here give room for argument that the built-in

Seperac-J19 Exam-Released MEE Essay Compilation © 2016-2020 1130


dishwashers and ranges, which have probably been affixed to the realty, are fixtures and that the
freestanding washers and dryers, which probably have not been affixed, are not fixtures.

a. Assuming the appliances are not fixtures.

If the appliances are not fixtures, Seller would have an absolute right to repossess them under UCC §§ 9-
503 and 9-504. There are no competing creditors, secured or otherwise. Bank’s interest is triggered only if
the appliances are fixtures.

b. Assuming the appliances are fixtures.

If the appliances became fixtures upon their installation in the condominium project, then they are subject
to Bank’s construction mortgage, which covers “land and any improvements thereon” constituting the
Eden Project. The question is whether the Bank’s interest will preclude Seller from repossessing the
collateral.

The relevant code provision, has a complicated set of priority rules and restrictions on remedies for
situations like these. Examinees should not be expected to know these rules in detail, but should be
expected to discuss at least some of the key issues raised by these facts.

First, the Code generally makes security interests in fixtures subordinate to the interests created by prior
construction mortgages. This priority for prior construction mortgage interests applies even as against
purchase money lenders such as Seller, and it extends to mortgages securing “an obligation incurred for
construction of an improvement on land including the acquisition cost of the land, if the reduced writing
so indicates.” On our facts, therefore, Bank has priority over Seller as to the appliances (if they are
fixtures) because Bank holds a prior construction mortgage.

Second, it should be noted that Seller has never perfected its security interest. Quite apart from the
construction mortgage rule, Seller’s unperfected security interest in fixtures would be subordinate to any
“conflicting interest of an encumbrancer of the related real estate.” except in limited circumstances not
applicable here.

Finally, the protection given by the Code to encumbrancers of real estate over holders of security interests
in fixtures operates to limit the secured party’s repossession rights. Under UCC § 9-313, a secured party
who “has priority over all owners and encumbrancers of the real estate” is entitled, upon default, to
“remove his collateral from the real estate.” This section does not describe explicitly the rights of the
holder of a subordinate security interest in fixtures, but the negative implication of this section is that a
secured party does not have a right to remove the collateral from real estate unless the secured party has
priority over all real property interests. Moreover, even if the secured party has priority over all real estate
interests, the secured party must reimburse the real estate interests for any physical injury caused by
removal of the fixtures.

The bottom line is that Seller will not be able to repossess the appliances if they are fixtures under real
estate law. First, they are subject to a prior construction mortgage. Second, Seller’s interest is unperfected
and thus subject to all real estate interests, except in limited circumstances. Finally, repossession is not an
option under UCC § 9-313 where there are real estate interests in the fixtures that are superior to the
security interest.

Seperac-J19 Exam-Released MEE Essay Compilation © 2016-2020 1131


#263-FEB 1998–MEE Q07: QUESTION SEVEN (UCC ART. 9)

Debtor publishes a newspaper. On May 3, Lender loaned Debtor $100,000 for general operating expenses.
On May 4, Debtor signed a security agreement, which stated in full:

Debtor hereby grants Lender a security interest in Debtor’s equipment to secure repayment of the $
100,000 loan made by Lender to Debtor on May 3.

Also on May 4, Lender filed a valid financing statement, signed by Debtor, in the proper UCC filing
offices.

On May 13, a second lender, Bank, also loaned Debtor $ 100,000 for general operating expenses. In
conjunction with that loan, Debtor signed a standard form security agreement provided by Bank giving
Bank a security interest in:

All Debtor’s equipment now owned or hereafter acquired to secure repayment of all debts of whatever
nature owed by Debtor to Bank, including all loans and future advances.

Bank failed to file a financing statement covering this transaction. At the time of both of these loans,
Debtor owned a single printing press, Press No. 1.

On June 1, Debtor borrowed $50,000 from a third lender, CreditCo, for the specific purpose of purchasing
a second printing press, Press No. 2. In conjunction with that loan, CreditCo had Debtor sign on that same
day a valid security agreement that granted CreditCo a security interest in Press No. 2. Debtor used the
proceeds of CreditCo’s loan to purchase Press No. 2 for $50,000. Debtor took possession of Press No. 2
on June 2. No financing statement was filed in connection with this transaction.

Debtor made regular payments on its obligations to Lender and Bank. By July 15, Debtor had completely
paid off its original debts to both Lender and Bank.

On August 1, however, Debtor borrowed an additional $50,000 from Lender and an additional $100,000
from Bank. No new security agreements or financing statements were generated in connection with these
new loans.

It is now September 15. Debtor owes $50,000 each to Lender and CreditCo and $100,000 to Bank. Debtor
is in default on all three loans, and Debtor’s creditors are seeking to foreclose on Press No. 1 and Press
No. 2. The presses are each worth $50,000.

1. As between Lender and Bank, which has the superior claim to Press No. 1 ? Explain.

2. As among Lender, Bank, and CreditCo, which has the superior claim to Press No. 2? Explain

Seperac-J19 Exam-Released MEE Essay Compilation © 2016-2020 1132


#263: F98-7 MEE: ANSWER: NCBE (UCC ART. 9)

POINT (1) [50%] ISSUE: Does Lender’s security interest cover future advances when the security
agreement is silent on the point? ANSWER: No. Because Lender’s security agreement explicitly
applied only to the May 3 loan, it does not cover future advances made by Lender. Thus, Lender’s
August 1 loan is unsecured and Lender no longer has a security interest in Press No. 1. Bank
prevails.

POINT (2) [50%] ISSUE: Which of two unperfected security interests prevails? ANSWER: With
respect to Press No. 2. Bank and CreditCo have conflicting unperfected security interests. In such a
situation, the first security interest to attach prevails, (b) CreditCo’s interest was first to attach
because Bank’s unperfected interest secures only the August 1 loan, which was made after
CreditCo’s interest attached.

ANSWER EXPLANATION:

Explanation to Point-One (40-60%):

Because Lender’s security agreement explicitly applied only to the May 3 loan, it does not cover future
advances made by Lender. Thus, Lender’s August 1 loan is unsecured and Lender no longer has a security
interest in Press No. 1. Bank prevails.

The security agreement between Lender and Debtor provided that Lender had a security interest in
Debtor’s equipment to secure the $ 100,000 loan granted “on May 3.” Thus, on its face the agreement
does not cover any obligation owed to Lender by Debtor other than the May 3 loan.

Article 9 of the Uniform Commercial Code permits security agreements to cover future advances.
However, a security agreement covers fixture advances only if the security agreement explicitly includes
a future advances clause.

Here, Lender’s May 3 loan has been paid in full. It has made a further loan, but there is no agreement
giving it a security interest in Press No. 1 or Press No. 2 to secure that further loan. Thus, Lender is
completely unsecured. By contrast, Bank’s security agreement with Debtor expressly covers “all loans
and future advances.” Thus, Bank’s August 1 loan is secured.

Some examinees will be tempted to conclude that Lender has priority over Bank because of the fact that
Lender was the only party to file a financing statement. However, although Lender has filed a financing
statement, it is unsecured as to its second loan (i.e., future advance). Bank, on the other hand, is secured
although its interest is unperfected. Because Lender is only a general creditor of Debtor with no security
interest or lien to secure the August 1 loan, Lender has no enforceable interest in Debtor’s property. Thus,
Bank is the only party with a security interest in Press No. 1 even though it failed to file a financing
statement. Although that interest is unperfected, its interest still has priority over Lender.

Explanation to Point-Two(a) (40-60%):

With respect to Press No. 2. Bank and CreditCo have conflicting unperfected security interests. In such a
situation, the first security interest to attach prevails, (b) CreditCo’s interest was first to attach because

Seperac-J19 Exam-Released MEE Essay Compilation © 2016-2020 1133


Bank’s unperfected interest secures only the August 1 loan, which was made after CreditCo’s interest
attached.

As noted above, Lender has no interest in Press No. 2 because Debtor’s current obligation to Lender is
unsecured.

(a) First-to-attach has priority.

Bank does have an interest in Press No. 2. Its interest secures the $100,000 advance it made on August 1
because Bank’s security agreement with Debtor had an explicit future advance clause and an after-
acquired property clause. However, Bank’s interest is unperfected because it never filed a financing
statement.

CreditCo also has a security interest in Press No. 2 to secure the purchase money loan it made to Debtor
to purchase the machine, but CreditCo also neglected to file a financing statement.

Neither Bank nor CreditCo has a perfected interest, because neither filed a financing statement. In the
event of a conflict between unperfected security interests, “the first to attach has priority.”

(b) CreditCo was first to attach.

The first-to-attach rule favors CreditCo in this case. A security interest based on a written security
agreement attaches when the last of three things occurs: (i) debtor has signed the security agreement, (ii)
value has been given, and (iii) debtor has rights in the collateral. In this case, Debtor signed the agreement
with Bank in May and Bank advanced value to Debtor at that time. However, Debtor did not acquire
rights in Press No. 2 until June 2, so Bank’s security interest in Press No. 2 attached at that time.
However, CreditCo’s security interest attached at the same instant. CreditCo and Debtor signed a security
agreement on June 1 and CreditCo gave value at that time. Debtor acquired rights in the collateral on June
2, so CreditCo’s interest attached the instant Debtor acquired rights in the collateral, just as Bank’s did.
But Debtor paid off Bank’s May loan, extinguishing its security interest in Press No. 2, and the security
interest did not re-attach until the August 1 loan was made. (The priority of future advances dates back to
the time of the first advance under § 9-312, but only if the security interest is perfected.)

Seperac-J19 Exam-Released MEE Essay Compilation © 2016-2020 1134


#264-FEB 1997–MEE Q03: QUESTION THREE (UCC ART. 9)

Debtor manufactures widgets. On May 3, 1996, Lender loaned Debtor $ 100,000 for general operating
expenses. That same day, Lender filed a valid financing statement, signed by Debtor, in the proper UCC
filing offices. The financing statement covered all widget machines owned by Debtor, “including after-
acquired widget machines.” On that date, May 3, Debtor owned just one widget machine, Machine No. 1,
which was worth $50,000. No security agreement was signed at that time.

On May 13, 1996, a second lender, Bank, loaned Debtor an additional $100,000 for general operating
expenses. On the same day, Debtor signed a valid security agreement that granted Bank a security interest
in “all widget machines, now owned or hereafter acquired by Debtor.” Debtor also signed a valid
financing statement which covered all widget machines owned by Debtor, “including after-acquired
widget machines.” That day, Bank filed the financing statement in the proper UCC filing offices. At that
point, Debtor still owned just one widget machine, Machine No. 1.

On May 16, 1996, a third lender, CreditCo, loaned Debtor an additional $50,000 to purchase a second
widget machine, Machine No. 2. On that day, Debtor signed a valid security agreement that granted
CreditCo a security interest in Machine No. 2. Debtor used the proceeds of CreditCo’s loan to purchase
Machine No. 2 for $50,000 on the next day, May 17. Debtor, however, did not receive possession of
Machine No. 2 until May 28. On May 31, CreditCo filed a valid financing statement that covered
Machine No. 2, signed by Debtor, in the proper UCC filing offices.

Meanwhile, on May 29, 1996, Lender discovered that it had forgotten to have Debtor sign a security
agreement for the May 3 loan. Later that day, May 29, Lender got Debtor to sign a security agreement in
which Debtor granted to Lender a security interest in all of Debtor’s widget machines, “including after-
acquired widget machines.” On that date, May 29, Debtor owned both widget machines, Machine No. 1
and Machine No. 2, each of which was worth $50,000.

Debtor subsequently defaulted on all three loans. Shortly after the default, but before any of Debtor’s
creditors took steps to repossess the widget machines, Debtor sold Machine No. 2 to WidgetCo, another
widget manufacturer, for $25,000. WidgetCo purchased the machine in good faith and had no actual
knowledge that Machine No. 2 was covered by a security interest.

1. As between Lender and Bank, whose security interest has first priority as to Machine No. 1?
Explain.

2. As among Lender, Bank, and CreditCo, whose security interest has first priority as to Machine
No. 2? Explain.

3. Can the secured party with priority recover Machine No. 2 from the purchaser, WidgetCo?
Explain

Seperac-J19 Exam-Released MEE Essay Compilation © 2016-2020 1135


#264: F97-3 MEE: ANSWER: NCBE (UCC ART. 9)

POINT (1) [33%] ISSUE: Did Lender’s delay in getting a signed security agreement from Debtor
subordinate Lender’s position in Machine No. 1 to Bank, which filed after Lender but perfected
before Lender? ANSWER: No. Because Lender was the first creditor to file a financing statement
covering Machine No. 1, Lender’s interest in the machine has priority over Bank’s even though
Bank was the first creditor to perfect its interest in the machine.

POINT (2) [33%] ISSUE: Did CreditCo’s delay in filing its financing statement prevent CreditCo
from receiving the special priority benefits of its status as a purchase-money secured lender?
ANSWER: No. Because CreditCo had a purchase-money security interest and filed its financing
statement within 10 days after Debtor received possession of the collateral, CreditCo takes priority
over both Lender and Bank as to Machine No. 2 even though both Lender and Bank filed before
CreditCo.

POINT (3) [33%] ISSUE: Can WidgetCo, a good-faith purchaser for value, retain Machine No. 2
when a secured party seeks to recover it after Debtor’s default? ANSWER: No. WidgetCo loses to
any of the perfected secured parties because the sale was not authorized and was not in the
ordinary course of business.

ANSWER EXPLANATION:

Explanation to Point-One (30-40%):

Because Lender was the first creditor to file a financing statement covering Machine No. 1, Lender’s
interest in the machine has priority over Bank’s even though Bank was the first creditor to perfect its
interest in the machine.

The general priority rule among competing secured creditors under UCC Article 9 is that the first creditor
to file a financing statement or perfect its interest will prevail. In this case, Lender filed its financing
statement against Machine No. 1 on May 3, the same day it lent Debtor $ 100,000. Because Lender failed
to get a signed security agreement from Debtor, however, Lender did not perfect its security interest in the
machine until May 29, when it belatedly obtained such a signed agreement.

According to the facts of this case, Bank both filed a financing statement and perfected its interest in
Machine No. 1 on May 13. Notwithstanding that Bank’s perfection came 16 days prior to Lender’s, UCC
§ 9-312 gives Lender priority in the machine over Bank because Lender filed its financing statement 10
days before Bank either perfected or filed its financing statement. Because Lender ultimately perfected, its
priority dated back to the time it filed the financing statement.

Explanation to Point-Two (30-40%):

Because CreditCo had a purchase-money security interest and filed its financing statement within 10 days
after Debtor received possession of the collateral, CreditCo takes priority over both Lender and Bank as to
Machine No. 2 even though both Lender and Bank filed before CreditCo.

Creditors who have purchase-money security interests qualify for a special priority under UCC Article 9.
If the purchase-money interest is in collateral other than inventory, the secured creditor with that special

Seperac-J19 Exam-Released MEE Essay Compilation © 2016-2020 1136


interest takes priority over competing interests in the same collateral as long as the purchase-money
interest was perfected at the time the debtor received possession of the collateral or within 10 days
thereafter.

Under the facts of this case, CreditCo qualified as a purchase-money lender under Article 9. CreditCo
gave value to enable Debtor to purchase Machine No. 2, and Debtor in fact used CreditCo’s loan proceeds
to purchase Machine No. 2. Although CreditCo failed to file a financing statement until 15 days after it
made the enabling loan to Debtor and 14 days after Debtor in fact purchased the machine, Debtor did not
have possession of the machine until May 28, just three days before CreditCo filed its financing
statement. The 10-day grace period given to purchase-money lenders under UCC § 9-312 measures the
period between when the debtor receives possession of the collateral and when the creditor files a
financing statement. Under the facts, CreditCo fell within that grace period in this case. Therefore, as to
Machine No. 2, CreditCo’s interest prevails over both Lender’s and Bank’s, even though Lender filed 28
days before CreditCo did, and Bank perfected 18 days before CreditCo did.

Explanation to Point-Three (30-40%):

WidgetCo loses to any of the perfected secured parties because the sale was not authorized and was not in
the ordinary course of business.

A security interest continues in collateral notwithstanding its sale unless the secured party authorized the
sale. Moreover, the security agreement (and hence the security interest) is effective against purchasers of
the collateral. Buyers in the ordinary course of business and some purchasers of consumer goods can
sometimes prevail over a protected secured party. However, Debtor does not appear to be a merchant in
the business of selling widget machines, so WidgetCo is not a buyer in the ordinary course. If widget
manufacturers regularly sell their used equipment to other manufacturers, a case might be made that the
sale was “in the ordinary course.” However, the purpose of the BIOC priority is to protect buyers out of
inventory, which WidgetCo clearly was not.

Thus, WidgetCo would lose to any of the three secured parties, each of whom has a perfected security
interest which continues in the collateral after its sale and is effective against the purchaser (UCC § 9-
201).

Seperac-J19 Exam-Released MEE Essay Compilation © 2016-2020 1137


#265-JUL 1996–MEE Q06: QUESTION SIX (UCC ART. 9)

On September 10,1995, Bank loaned $400,000 to PartsCo, a wholesale supplier of automobile parts to
retail auto parts stores. The loan was evidenced by a duly executed promissory note and a security
agreement that granted Bank a security interest in all of PartsCo's "now existing and hereafter acquired
inventory." Also on September 10, 1995, Bank filed a UCC financing statement in the proper places in the
state.

Bank had previously searched the official UCC financing statement records on September 6, 1995, and
found a financing statement filed on November 1, 1992, naming PartsCo as debtor, identifying Finance as
the secured party, and describing the collateral as PartsCo's "existing and after-acquired inventory."
Neither the underlying security agreement nor the financing statement mentioned future advances. When
Bank asked PartsCo about this financing statement, PartsCo's president truthfully replied, "We paid that
off in 1994."

On November 15, 1995, Finance offered to reestablish its lending relationship with PartsCo, which
needed additional funds. Finance loaned PartsCo $200,000. PartsCo signed another security agreement
providing that the $200,000 loan was secured by PartsCo's now existing and hereafter acquired inventory.
Also on November 15, 1995, a proper financing statement was executed by PartsCo and filed by Finance
in the proper places in the state.

PartsCo is in default on both loans, owing Bank $300,000 and Finance $175,000. Bank has begun judicial
proceedings to foreclose on PartsCo's inventory (worth approximately $150,000), as well as $50,000 in
uncollected accounts receivable that arose from sales of PartsCo's inventory and $30,000 in a special
"compensating balance" deposit account set up at the request of Bank and in which PartsCo has deposited
only the proceeds of inventory sales. Both the accounts receivable and the $30,000 deposit account were
generated by sales made after January 1, 1996.

1. As between Bank and Finance, which one has superior rights in PartsCo's unsold inventory?
Explain.

2. As between Bank and Finance, which one has superior rights in the proceeds of the sale of
PartsCo's inventory? Explain.

3. In light of the fact that Bank has commenced judicial foreclosure proceedings, what should
Finance do to best protect its interests, and how should the proceeds of a foreclosure sale be
distributed as between Bank and Finance? Explain.

Seperac-J19 Exam-Released MEE Essay Compilation © 2016-2020 1138


#265: J96-6 MEE: ANSWER: NCBE (UCC ART. 9)

POINT (1) [48%] ISSUE: As between Bank and Finance, which has priority to recover PartsCo's
unsold inventory? ANSWER: Finance retained priority over Bank in the $150,000 of PartsCo's
inventory. This is a contest between two perfected secured parties. Bank's security interest in
PartsCo's inventory was properly perfected upon the execution of a security agreement granting it
a security interest in PartsCo's now owned and hereafter acquired inventory and the filing of the
properly executed financing statement in the proper places within the state. Bank's security interest
was perfected on September 10, 1995.

POINT (2) [24%] ISSUE: As between Bank and Finance, which has priority to recover the
proceeds from the sales of PartsCo's inventory? ANSWER: Finance also retained priority over
Bank in the proceeds of the inventory. In general, the date of filing or perfection as to the collateral
is also the date of filing or perfection as to the proceeds of the collateral for the purposes of
resolving priority disputes. Thus, if both Finance and Bank continue to have perfected security
interests in the proceeds, Finance as the first to file or perfect should prevail as to the proceeds as
well.

POINT (3) [29%] ISSUE: Should Finance, in order to protect its interests, intervene in the judicial
proceedings commenced by Bank and can Bank obtain anything from the proceedings? ANSWER:
Yes. Intervention in the judicial proceedings initiated by Bank is the best way for Finance to protect
its interests. Finance is entitled to have its unpaid balance and costs of foreclosure satisfied first,
with any balance going to Bank.

ANSWER EXPLANATION:

Explanation to Point-One (45-55%):

Finance retained priority over Bank in the $150,000 of PartsCo's inventory. This is a contest between two
perfected secured parties. Bank's security interest in PartsCo's inventory was properly perfected upon the
execution of a security agreement granting it a security interest in PartsCo's now owned and hereafter
acquired inventory and the filing of the properly executed financing statement in the proper places within
the state. Bank's security interest was perfected on September 10, 1995.

Finance also has a perfected security interest in PartsCo's existing and hereafter acquired inventory.
Finance's security interest did not attach until November 15, 1995, when PartsCo took the new $200,000
loan and signed a security agreement granting Finance a security interest in the inventory. The 1992
PartsCo/Finance security agreement did not include future advances, so execution of a new 1995
PartsCo/Finance security agreement was necessary for finance to acquire a security interest in PartsCo's
inventory. Because the 1992 Finance/PartsCo financing statement was still on file and had not been
terminated, Finance's new security interest was automatically perfected as soon as it attached. Under UCC
§ 9-403, a financing statement is effective for five years.

Under these circumstances, Finance has priority over Bank under the first-to-file-or-perfect rule because it
filed in 1992, well before Bank either filed or perfected. Finance's 1992 filing is the basis of its perfection
in November 1995 and, accordingly, gives it priority over Bank. Bank should have caused the Finance
financing statement to be terminated when it was discovered during Bank's financing statement search in

Seperac-J19 Exam-Released MEE Essay Compilation © 2016-2020 1139


early September. Having failed to do so, Bank allowed this relation back upon Finance's subsequent
advance to trump Bank's security interest.

Explanation to Point-Two (20-30%):

Finance also retained priority over Bank in the proceeds of the inventory. In general, the date of filing or
perfection as to the collateral is also the date of filing or perfection as to the proceeds of the collateral for
the purposes of resolving priority disputes. Thus, if both Finance and Bank continue to have perfected
security interests in the proceeds, Finance as the first to file or perfect should prevail as to the proceeds as
well.

There is no requirement that the secured parties have mentioned in the security agreement or financing
statements that their security interest extends to proceeds. The security interest in proceeds is created
when the security interest in the underlying collateral is properly created. The secured party's security
interest only continues, however, in “identifiable proceeds.” In this case, both the uncollected accounts
receivable and the money on deposit in the special deposit account that contains only proceeds are clearly
identified as arising from PartsCo's inventory, and thus constitute identifiable proceeds.

Furthermore, the security interests of both Finance and Bank continue to be perfected in the proceeds. In
the case of the accounts receivable, UCC §9-306 provides that the security interest remains perfected if a
filed financing statement covers the original collateral and the proceeds are a type of collateral in which a
security interest may be filed in the same office where the inventory financing statements were filed. In
most states (if not all) a financing statement would be filed in the same office where filings are made to
perfect a security interest in inventory and in accounts.

The security interests of Finance and Bank also continue perfected in the $30,000 on deposit in the special
account containing only proceeds of inventory sales because the proceeds are identifiable cash proceeds
under UCC §9-306. A deposit account is considered cash proceeds, and these proceeds are identifiable
because the entire balance of the account represents money received from inventory sales.

Thus, the security interests of both Bank and Finance remained perfected, but Finance had priority
because its interests related back to the 1992 filing.

Explanation to Point-Three (25-35%):

Intervention in the judicial proceedings initiated by Bank is the best way for Finance to protect its
interests. Finance is entitled to have its unpaid balance and costs of foreclosure satisfied first, with any
balance going to Bank.

Finance should intervene in Bank's judicial action and, as the prior in time secured party, should conduct
the foreclosure sale. If the disposition of the collateral and proceeds produce the $280,000 indicated in the
facts, these proceeds will be applied first to the expenses of conducting the sale, including reasonable
attorney's fees, then to satisfy the debt of Finance. The remaining balance may then be used to satisfy the
claims of subordinate secured parties, including Bank. Depending on the amount of the expenses, it is
possible that upon satisfaction of Finance's $175,000 balance, some additional funds would go to Bank as
the junior secured party.

If Finance did not intervene to conduct the foreclosure sale, the collateral would be sold subject to
Finance's security interest and its recourse would be to repossess the collateral from the buyer at the

Seperac-J19 Exam-Released MEE Essay Compilation © 2016-2020 1140


foreclosure sale and sell it to satisfy its security interest. Obviously, it would be more prudent for Finance
to intervene at the front end.

Seperac-J19 Exam-Released MEE Essay Compilation © 2016-2020 1141


#266-FEB 1996–MEE Q06: QUESTION SIX (UCC ART. 9)

In 1993, Debtor, a retail seller of power boats, established a line of credit with Bank. In order to secure the
loan, Debtor signed a security agreement granting Bank a security interest in its "inventory of boats, now
existing and hereafter acquired." Bank promptly and properly filed a financing statement, which described
the collateral as "inventory." Bank made periodic advances to Debtor in an amount up to 75% of the value
of Debtor's inventory.

Recently Debtor's business slackened, and it needed an additional source of financing. Debtor approached
Finance Company and asked for a loan. Finance loaned money to Debtor on January 2, 1995. As
collateral for the loan, Debtor transferred to Finance all of Debtor's then-existing chattel paper, all of
which had been generated by sales of Debtor's inventory prior to January 2. Debtor has not transferred to
Finance any chattel paper generated from several boat sales that occurred after January 2, 1995.

In mid-January, Debtor sold a large boat to Purchaser for recreational use by Purchaser's family.
Purchaser signed a contract (chattel paper) promising to pay the purchase price in monthly installments
over the next three years and granting Debtor a security interest in the boat.

On March 1, 1995, Debtor defaulted on its loans to Bank and Finance.

1. After Debtor's default, what rights do Bank and Finance have with respect to chattel paper
generated from the sales of Debtor's inventory? Explain.

2. After Debtor's default, what rights do Bank and Finance have with respect to Purchaser's boat?
Explain.

Seperac-J19 Exam-Released MEE Essay Compilation © 2016-2020 1142


#266: F96-6 MEE: ANSWER: NCBE (UCC ART. 9)

POINT (1)(a) [25%] ISSUE: What is the nature of Bank's interest in the chattel paper? ANSWER:
Because Debtor's chattel paper is identifiable proceeds of its inventory, Bank's perfected security
interest in the inventory continues in the chattel paper.

POINT (1)(b) [25%] ISSUE: What is the nature of Finance Company's interest in the chattel
paper? ANSWER: Finance Company has a perfected security interest in any chattel paper in its
possession.

POINT (1)(c) [25%] ISSUE: What is the relative priority of a purchaser of chattel paper and an
inventory financer who claims a security interest in chattel paper as proceeds of inventory?
ANSWER: Finance Company has priority over Bank in Debtor's chattel paper which Finance
Company has in its possession. Bank gets any chattel paper not in the possession of Finance
Company, including the chattel paper generated by the sale to Purchaser.

POINT (2) [25%] ISSUE: Does the security interest of an inventory financer continue in inventory
in the hands of the good faith purchaser from the debtor? ANSWER: No. Bank may not get
possession of the boat from Purchaser because Purchaser is a buyer in the ordinary course of
business. Finance has no interest in Purchaser's boat since it does not have possession of the chattel
paper granting a security interest in the boat to Debtor.

ANSWER EXPLANATION:

Explanation to Point-One(a) (20-30%):

Because Debtor's chattel paper is identifiable proceeds of its inventory, Bank's perfected security interest
in the inventory continues in the chattel paper.

Bank has a perfected security interest in Debtor's inventory and after-acquired collateral. Although the
financing statement that Bank filed does not refer to after-acquired collateral, such a reference is not
necessary so long as it is included in the security agreement. Bank's security interest also extends to
identifiable proceeds of the inventory whether or not proceeds are specifically mentioned in the security
agreement. Chattel paper is proceeds of inventory and Bank's security interest would continue in the
chattel paper, which could easily be identified to the collateral sold. Further, Bank's security interest in the
chattel paper would be automatically perfected because a financing statement to perfect a security interest
in chattel paper would be filed in the same place as the financing statement previously filed to perfect a
security interest in the inventory for which the chattel paper was received as proceeds.

Explanation to Point-One(b) (20-30%):

Finance Company has a perfected security interest in any chattel paper in its possession.

Finance Company also has a perfected security interest in Debtor's chattel paper. This security interest is
evidenced by Finance Company's possession of the chattel paper pursuant to its agreement with Debtor.
In this instance, a written security agreement is not required. Furthermore, possession of the chattel paper
by Finance serves to perfect Finance's security interest in it. Finance's security interest extends only to
chattel paper in its possession and not to any chattel paper generated after January 2.

Seperac-J19 Exam-Released MEE Essay Compilation © 2016-2020 1143


Explanation to Point-One(c) (20-30%):

Finance Company has priority over Bank in Debtor's chattel paper which Finance Company has in its
possession. Bank gets any chattel paper not in the possession of Finance Company, including the chattel
paper generated by the sale to Purchaser.

Upon default by Debtor the priority contest between both perfected secured parties as to chattel paper
generated prior to January 2 will be resolved by UCC § 9-308. Finance is a purchaser of chattel paper.
UCC § 1-201 defines a purchaser to include one who takes a security interest in property. If Finance took
possession of the chattel paper in the ordinary course of its business, it has priority over an earlier security
interest in the chattel paper which is claimed merely as proceeds, even if Finance knew of Bank's earlier-
in-time security interest. Because Bank advanced funds to Debtor pursuant to the value of Debtor's
inventory (rather than the value of its inventory and chattel paper), Bank is claiming a security interest in
the chattel paper merely as proceeds of its inventory collateral; the security agreement between Bank and
Debtor did not create an independent security interest in chattel paper. Even if Bank did advance funds
against the chattel paper, Finance would have priority if it had no actual knowledge of Bank's security
interest. Bank prevails as to the post- January 2 chattel paper, however, because Finance does not have a
security interest in it, having failed to take possession of any chattel paper generated after January 2.

Explanation to Point-Two (20-30%):

Bank may not get possession of the boat from Purchaser because Purchaser is a buyer in the ordinary
course of business. Finance has no interest in Purchaser's boat since it does not have possession of the
chattel paper granting a security interest in the boat to Debtor.

There are two bases upon which to argue that Bank has no rights in the boat. First, if Bank authorized
Debtor to sell its inventory either expressly in the security agreement or impliedly through its course of
conduct, then Bank's security interest would not continue in the boat in the hands of Purchaser, the buyer.
Presumably, Bank would have authorized sales from Debtor's inventory because this is how Debtor would
stay in business and earn the money necessary to pay back Bank.

Alternatively, Purchaser probably qualifies as a buyer in the ordinary course of business who takes free of
a security interest in the boat created by his seller, Debtor. To apply this section it does not matter that
Bank's security interest is perfected or even that Purchaser is aware of the perfected security interest.
Purchaser is probably a buyer in the ordinary course of business because he bought in good faith in the
ordinary course from a person (Debtor) in the business of selling goods of the kind for new value
(Purchaser's promise to pay). Purchaser would not be protected under 9-307 special limited protection for
purchasers of consumer goods, because Bank filed a financing statement. Thus, Purchaser must rely on
the general protection for a buyer in the ordinary course of business.

In either case, Purchaser takes free of Bank's security interest, and, as long as Purchaser continues to
make payments under the contract, he is entitled to retain possession of the boat. However, Bank has an
interest in the chattel paper and a right to receive payments on the boat from Purchaser. If Purchaser fails
to make those payments, Bank (as holder of the chattel paper) would have an interest in the boat.

Finance Company has no interest in Purchaser's boat because it did not take possession of the chattel
paper granting Debtor a security interest in the boat.

Seperac-J19 Exam-Released MEE Essay Compilation © 2016-2020 1144


TRUSTS: 30 OF 49 MEE EXAMS: (61%)
#267-FEB 2019–MEE Q05: QUESTION FIVE (TRUSTS & FUTURE INTERESTS)

Eight years ago, a settlor created a $300,000 irrevocable trust. The settlor's brother is the sole trustee of
the trust. The trust's primary beneficiaries are the settlor's son and daughter. The trust instrument provides,
in relevant part:

During the term of this trust, the trustee shall pay to and between my two children so much, if any,
of trust income and principal as he deems advisable, in his sole discretion, for each child's support.
Upon the death of the survivor of my children, the trustee shall distribute any remaining
undistributed trust principal and income equally among my surviving grandchildren.

The trust contains a spendthrift clause that prohibits the voluntary assignment of a beneficiary's interest
and does not allow a beneficiary's creditors to reach that interest.

Two months after creating the trust, the settlor died. Both the settlor's son, now age 35, and the settlor's
daughter, now age 32, survived the settlor and are still alive. The settlor's son has three living children,
now 9, 11, and 14 years of age. These children currently live with their mother, from whom the settlor's
son was divorced seven years ago. The settlor's daughter is unmarried and has no children. Both the son
(employed as a waiter) and the daughter (employed as a bookkeeper) have earned, on average, less than
$35,000 per year during the past seven years.

Over the past eight years, the son has incurred and has not paid the following debts:

(a) $10,000 to a hospital for the son's emergency-room care

(b) $35,000 to his former wife in unpaid, judicially ordered child support

(c) $5,000 to a friend for repayment of a loan, five years ago, to purchase a high-end computer-
gaming system for recreational use

Repayment of the debt to the friend was due last year, but the son defaulted on the loan.

During the first year of the trust, the trustee distributed $9,000 of trust income to each of the settlor's two
children for their support. Thereafter, relations between the settlor's son and the trustee deteriorated. After
the son and his wife divorced, the trustee frequently told others, behind the son's back and without any
direct basis, that the son was an "adulterer" and a "terrible father." The trustee often referred to the son as
a "bum," and he told the settlor's daughter, without any explanation, "Your brother is rude to me."

Over the last seven years, although the son's and daughter's financial needs were similar, the trustee has
distributed $80,000 from trust income and principal to the settlor's daughter and nothing to the settlor's
son, despite the son's repeated requests for trust distributions to help him pay his hospital bill, child
support, and loan.

1. Given the terms of the trust the settlor created, could the trustee have properly distributed trust
assets to the son to enable him to pay (a) his hospital bill, (b) child support, and (c) the loan to
purchase the computer-gaming system? Explain.

Seperac-J19 Exam-Released MEE Essay Compilation © 2016-2020 1145


2. Did the trustee abuse his discretion in refusing to make any distributions to the son during the
past seven years? Explain.

3. In light of both the discretion granted the trustee and the spendthrift clause in the trust, may the
son's three creditors obtain orders requiring the trustee to pay their claims against the son from trust
assets? Explain.

Seperac-J19 Exam-Released MEE Essay Compilation © 2016-2020 1146


#267: F19-5 MEE: ANSWER: NCBE (TRUSTS & FUTURE INTERESTS)

POINT (1) [40%] ISSUE: Could the trustee have properly distributed trust assets to the son to
enable him to pay his hospital bill, child support obligations, or loan to purchase the computer-
gaming system? ANSWER: The settlor created a discretionary support trust subject to a
spendthrift clause. The hospital bill and child support debt are support expenses for which the
trustee could distribute trust assets to the son. The loan to purchase a computer-gaming system is
not likely to be characterized as support.

POINT (2) [35%] ISSUE: Did the trustee abuse his discretion in refusing to make any distributions
to the son? ANSWER: Yes. The trustee likely abused his discretion in failing to make any
distributions to the son, including distributions for the payment of the hospital bill and child
support.

POINT (3) [25%] ISSUE: In light of both the discretion granted the trustee and the spendthrift
clause in the trust, may the son's three creditors obtain orders requiring the trustee to pay their
claims against the son from trust assets? ANSWER: Despite the trust's spendthrift clause, the son's
former wife and, assuming that the necessaries doctrine applies, the hospital may reach the son's
interest in the trust to satisfy their claims if the trustee abused his discretion. However, the friend
cannot reach the son's interest to satisfy his claim against the son.

ANSWER DISCUSSION:

The settlor created a discretionary support trust with spendthrift protection. A discretionary support trust
permits distributions only for the support of trust beneficiaries, and it provides a trustee with discretion
(which the trustee may not abuse) to withhold a distribution even when a beneficiary needs support. A
hospital bill and court-ordered child support are debts incurred for support, which the trustee could pay in
his discretion, but a loan to purchase a recreational computer-gaming system would not likely be
characterized as a debt incurred for support. Given the trustee's personal animosity toward the son, the
trustee's comparatively liberal distributions to the daughter, and the absence of any reasonable
justification for withholding distributions to the son, the son should be able to show that the trustee
breached his duty by refusing to make distributions to the son. Thus, the son should be able to bring a
successful action against the trustee for the trustee's failure to make distributions that would have enabled
him (in whole or in part) to pay the hospital and child-support debts or receive other distributions for
support. Under the Restatement (Third) of Trusts, a creditor of a support trust beneficiary with spendthrift
protection may reach trust assets if the creditor provided a necessary to the trust beneficiary or seeks to
enforce a child-support claim. The son's former wife and the hospital thus should be able to obtain orders
against the trustee for their respective claims. However, because the purchase of a computer-gaming
system is not a "necessary" for a person of modest means, the friend cannot obtain an order against the
trustee seeking payment for that debt. Under the provisions of the Uniform Trust Code, only the child
support would be recoverable.

ANSWER EXPLANATION:

Explanation to Point One (40%):

Seperac-J19 Exam-Released MEE Essay Compilation © 2016-2020 1147


The settlor created a discretionary support trust subject to a spendthrift clause. The hospital bill and child
support debt are support expenses for which the trustee could distribute trust assets to the son. The loan to
purchase a computer-gaming system is not likely to be characterized as support.

The settlor created a discretionary support trust subject to a spendthrift clause. A support trust permits
distributions from the trust to enable the beneficiary to maintain his or her accustomed standard of living.
When a trust instrument grants the trustee of a support trust discretion whether or not to pay a
beneficiary's support-related expenses, the trustee's judgment controls unless the trustee abuses his
discretion.

This trust is also subject to a spendthrift clause. This clause may prevent a beneficiary's creditors from
reaching trust assets, but it does not prevent the beneficiary himself from reaching trust assets if the
trustee has abused his discretion in failing to make payments to the beneficiary.

The meaning of the term "support" is fact-dependent. Support includes more than necessities or bare
essentials. Most courts measure support in terms of the lifestyle to which the beneficiary has become
accustomed even if the trust instrument does not expressly refer to that lifestyle. A beneficiary's
accustomed lifestyle is determined at the time the beneficiary's trust interest is created, but is subject to
adjustment to accommodate the beneficiary's changing needs.

Necessary medical care is invariably treated as support. Support also invariably includes reasonable
amounts for the support of minor children who reside elsewhere but for whom the beneficiary either
chooses or is required to provide support.

Here, the trustee could properly have distributed trust assets to the son to pay the son's hospital bill and
child support obligation.

On the other hand, without more facts, a distribution to allow repayment of the son's debt incurred for the
purchase of a computer-gaming system would not appear to be a distribution for the son's support. Of
course, if the son could establish that such a system is necessary to allow him to live in accordance with
his accustomed lifestyle, then it might be considered a support expense. However, the facts state that the
son's income is about $35,000, a modest sum in light of the son's child support obligation and expenses
for basic needs such as health care, food, clothing, shelter, and transportation. Given this income, it is
unlikely that the son could establish a lifestyle that would warrant the trustee's conclusion that the
recreational computer-gaming system is a support expense.

Explanation to Point Two (35%):

The trustee likely abused his discretion in failing to make any distributions to the son, including
distributions for the payment of the hospital bill and child support.

The fact that the son's hospital bill and child support obligations should be characterized as support is not
the end of the matter. When the trust instrument grants a trustee discretion to pay a beneficiary's support-
related expenses, the trustee's judgment controls unless the trustee abuses his discretion. A trustee's
discretionary power is subject to judicial control only to prevent abuse of the discretion by the trustee.
Trustees may have a number of legitimate reasons to withhold payments from a beneficiary, including
discharging the trustee's duty to act impartially with respect to all trust beneficiaries and the beneficiary's
ability to pay the expenses from other resources. However, if a court finds that a trustee acted in bad faith
or with an improper motive, it may overrule his decisions.

Seperac-J19 Exam-Released MEE Essay Compilation © 2016-2020 1148


What constitutes an abuse of discretion, according to the Restatement (Third), depends upon the terms of
the trust instrument and the other duties of the trustee, such as the duty to administer the trust in
accordance with its terms, the duty to act impartially, and the duty of care. These duties, read together,
entitle the beneficiary to general information concerning the bases upon which the trustee's discretionary
judgments have been or will be made. Furthermore, a trustee abuses discretion by acting in bad faith or
with an improper motive.

As section 50 of the Restatement (Third) suggests, where, as here, there are multiple trust beneficiaries,
the trustee's duty of impartiality requires that, in making distributions, the trustee act with due regard for
the diverse beneficial interests created by the terms of the trust. The duty of impartiality is an extension of
the duty of loyalty to beneficiaries but involves, in typical trust situations, unavoidably and thus
permissibly conflicting duties to various beneficiaries with their competing economic interests.

It would be overly simplistic, and therefore misleading, to equate impartiality with some concept of
'equality' of treatment or concern—that is, to assume that the interests of all beneficiaries have the same
priority and are entitled to the same weight in the trustee's balancing of those interests. Impartiality does
mean that a trustee's treatment of beneficiaries or conduct in administering a trust is not to be influenced
by the trustee's personal favoritism or animosity toward individual beneficiaries, even if the latter results
from antagonism that sometimes arises in the course of administration.

Here, there is a strong argument that the trustee abused his discretion by withholding support distributions
from the son. He ignored the son's repeated requests for distributions, without offering any reason for his
refusals. He made substantial distributions to the daughter, whose income was not, on average, different
from the son's. The trustee made disparaging comments about the son (that the son was a "bum," a
"terrible father," and an "adulterer," and was "rude" to the trustee). Together, these facts support an
inference that the trustee withheld distributions from the son because of personal animus rather than a
valid reason. All of this supports the conclusion that the trustee abused his discretion in withholding
distributions from the son.

[NOTE: Although the question does not ask whether the son could successfully sue based on the trustee's
abuse of discretion or what he could recover if his suit were successful, in cases like this one, if the court
finds an abuse of discretion, it will typically make an independent judgment of what the trustee would
have distributed when carrying out his fiduciary duties and then direct that such payment be made from
the trust, if trust assets are available, or otherwise surcharge the trustee.]

Explanation to Point Three (25%):

Despite the trust's spendthrift clause, the son's former wife and, assuming that the necessaries doctrine
applies, the hospital may reach the son's interest in the trust to satisfy their claims if the trustee abused his
discretion. However, the friend cannot reach the son's interest to satisfy his claim against the son.

The trust created by the settlor was discretionary. If the terms of a trust provide for a beneficiary to
receive distributions in the trustee's discretion, a creditor of the beneficiary is entitled to receive any
distributions the trustee is required to make in the exercise of that discretion.

This general principle does not apply, however, when a trust includes a spendthrift clause. A spendthrift
clause puts trust assets out of the reach of most creditors of a trust beneficiary until such time as trust
assets are distributed to that beneficiary.

Seperac-J19 Exam-Released MEE Essay Compilation © 2016-2020 1149


However, even when a trust provides spendthrift protection, claims against a beneficiary for unpaid child
support may still be enforced against the trust. A court may order the trustee to pay the child such amount
as is equitable under the circumstances but not more than the amount the trustee would have been
required to distribute for the benefit of the beneficiary had the trustee complied with the support standard
or not abused the discretion. Here there is a strong argument that the trustee breached his discretion, such
that the former wife will be able to obtain payment from the trustee.

In some jurisdictions, creditors who provided the beneficiary with "necessaries" such as health care may
reach the beneficiary's interest in satisfaction of any unpaid debt despite a spendthrift clause. Comments
to the Restatement (Third) of Trusts suggest that the creditor may reach the beneficiary's interest but may
not force a sale of the interest. In such a case, the court could direct the trustee to distribute trust income
to the creditor until the claim is paid. Medical care is invariably treated as a necessary. But if in addition
to a spendthrift clause, distributions are discretionary with the trustee, the creditor who provided the
beneficiary with a necessary cannot compel a distribution if the beneficiary could not do so. In other
words, the creditor cannot compel a distribution in the absence of an abuse of discretion.

The necessaries exception is not recognized in the Uniform Trust Code. Comments to the Restatement
also suggest that the necessaries rule, while consistent with prior trust Restatements, is not followed in
some U.S. jurisdictions. In these jurisdictions, the hospital cannot reach the son's interest in the trust in
satisfaction of its claim.

In jurisdictions that follow the necessaries doctrine as applied to a spendthrift clause, the hospital's claim
is not barred by the spendthrift clause. But because distributions from the trust are discretionary, the
hospital will not be able to reach the son's interest unless the trustee abused his discretion. Here, there is a
strong argument that the trustee abused his discretion, thus allowing the hospital to reach the son's interest
and obtain some payment from the trustee.

The friend should not succeed in his suit against the trustee. The friend's claim is not a claim for support,
and here, the facts cannot support an assertion that a $5,000 computer-gaming system is a "necessary" for
the son.

Seperac-J19 Exam-Released MEE Essay Compilation © 2016-2020 1150


#268-JUL 2018–MEE Q04: QUESTION FOUR (TRUSTS)

By his will, a testator created a trust of a small house and an apartment building containing six three-
bedroom apartments. The will directed the trustee to sell the house within six months of the testator’s
death. The will also provided, in relevant part, that “all trust income will be paid to my cousin, Albert,
during his lifetime” and that “upon Albert’s death, all trust principal will be distributed to my
granddaughter, Betty.” Neither the will nor the trust made any provision for the testator’s son, who was
living at the time the will was executed. Shortly after making this will in 2006, the testator died.

After the trust was created, the trustee sold the house for $100,000 and properly invested the sale
proceeds. All six apartments in the apartment building were rented at market rates ranging from $1,200 to
$1,400 per month.

In 2010, one apartment, which had been rented for $1,300 per month, was vacated. The trustee thereafter
rented this apartment to himself for $1,300 per month. The other five apartments continued to be rented
throughout the term of the trust at market rates of between $1,200 and $1,400 per month.

In 2012, a portion of the apartment building’s roof was destroyed by fire. Because the trustee had not
purchased a fire insurance policy, he spent $50,000 to repair the roof. The trustee charged this expense to
trust income even though the trust had liquid assets of more than $120,000 that could have been used to
pay for the repair. Because the roof repair was charged to trust income, Albert received $50,000 less
income from the trust in 2012 than he had received in prior years.

In 2013, Betty died. Betty was survived by her husband and a daughter. Under Betty’s duly probated will,
she left her entire estate to her husband. If Betty had died intestate, her estate would have been distributed
equally between her husband and her daughter.

There is no applicable statute relevant to the disposition of Betty’s interest in the trust.

In 2018, Albert died. Albert was survived by Betty’s husband and Betty’s daughter. Albert was also
survived by the testator’s son.

1. What fiduciary duties, if any, did the trustee violate in administering the trust? Explain.

2. Upon Albert’s death, how should the trust principal be distributed? Explain.

Seperac-J19 Exam-Released MEE Essay Compilation © 2016-2020 1151


#268: J18-4 MEE: ANSWER: NCBE (TRUSTS)

POINT (1)(a) [25%] ISSUE: Did the trustee breach the duty of loyalty by renting a trust-owned
apartment to himself at a market rate? ANSWER: Yes. The trustee breached his duty of loyalty
when he rented a trust-owned apartment to himself even though he paid a market rate.

POINT (1)(b) [25%] ISSUE: Did the trustee breach the duty of prudent administration (a/k/a duty
of care) by failing to purchase fire/casualty insurance on trust property, resulting in a $50,000
uninsured loss? ANSWER: Yes. The trustee breached his duty of prudent administration (a/k/a
duty of care) when he failed to purchase fire/casualty insurance on the real property.

POINT (1)(c) [30%] ISSUE: Did the trustee breach the duty to administer in accordance with
applicable law by allocating the entire $50,000 expense occasioned by the unexpected roof repair to
trust income? ANSWER: Yes. The trustee breached his duty to administer the trust in accordance
with applicable law by allocating the $50,000 repair expense exclusively to income.

POINT (2) [20%] ISSUE: Upon Albert’s death, should the trust principal be distributed to Betty’s
husband, Betty’s daughter, equally to both of them, or to the testator’s heirs? ANSWER: Under the
common law, upon Albert’s death, the trust principal should be distributed to Betty’s husband
because she devised her vested remainder in the trust to him.

ANSWER DISCUSSION:

The trustee breached the fiduciary duty of loyalty when the trustee rented one of the apartments to himself
even though it appears that he paid a market rate for the apartment. The trustee breached the duty of
prudent administration (a/k/a duty of care) by failing to purchase fire insurance on the trust’s real
property. The trustee breached the duty to administer in accordance with applicable law by charging the
entire uninsured loss to income rather than principal. Under the common law, upon Albert’s death the
trust principal would pass to Betty’s husband because she had a vested remainder in the trust that passed
to him under her will. The testator’s son would not share in trust principal because there is no reversion to
pass to the testator’s heirs.

ANSWER EXPLANATION:

Explanation to Point One(a) (25%):

The trustee breached his duty of loyalty when he rented a trust-owned apartment to himself even though
he paid a market rate.

A trustee owes trust beneficiaries a duty of loyalty. This duty arises under the common law and by statute.
Under the common law, a trustee is prohibited from making transactions that place the interest of the
trustee or another above the interest of trust beneficiaries. The duty of loyalty prohibits two types of
transactions: self-dealing, where the trustee deals with trust property for the trustee’s personal benefit, and
conflict of interest, where the trustee acts on “behalf of others to whom the trustee also owes obligations.”
Here, the trustee’s lease of a trust-owned apartment to himself was self-dealing. The lease thus breached
the trustee’s duty of loyalty.

Seperac-J19 Exam-Released MEE Essay Compilation © 2016-2020 1152


Because of the so-called “no-further-inquiry” rule, the fact that the trustee paid a market rate is irrelevant
to the conclusion that the trustee engaged in self-dealing. The trustee is strictly prohibited from engaging
in transactions that involve self-dealing or that otherwise involve or create a conflict between the trustee’s
fiduciary duties and personal interests. As noted in the Restatement (Third), under the no-further-inquiry
rule, “it is immaterial that the trustee may be able to show that the action in question was taken in good
faith, that the terms of the transaction were fair, and that no profit resulted to the trustee.” This strict
approach is justified on the grounds that “it may be difficult for a trustee to resist temptation when
personal interests conflict with fiduciary duty. In such situations, for reasons peculiar to typical trust
relationships, the policy of the trust law is to prefer (as a matter of default law) to remove altogether the
occasions of temptation rather than to monitor fiduciary behavior and attempt to uncover and punish
abuses when a trustee has actually succumbed to temptation. This policy of strict prohibition also
provides a reasonable circumstantial assurance (except as waived by the settlor or an affected beneficiary)
that beneficiaries will not be deprived of a trustee’s disinterested and objective judgment.”

The Uniform Trust Code takes a similar approach. It adopts the no-further-inquiry rule and provides that a
self-dealing transaction is voidable by trust beneficiaries. As noted in the comments, “transactions
involving trust property entered into by a trustee for the trustee’s own personal account are voidable
without further proof. It is immaterial whether the trustee acts in good faith or pays a fair consideration.”

Explanation to Point One(b) (25%):

The trustee breached his duty of prudent administration (a/k/a duty of care) when he failed to purchase
fire/casualty insurance on the real property.

Both the Restatement of Trusts and the Uniform Trust Code impose on trustees a “duty of prudent
administration.” Under the Restatement, the trustee has a duty to administer the trust as a prudent person
would, in light of the purposes, terms, and other circumstances of the trust. Under the UTC, a trustee shall
administer the trust as a prudent person would, by considering the purposes, terms, distributional
requirements and other circumstances of the trust. Both authorities agree that, to satisfy the duty of
prudent administration, a trustee must “exercise reasonable care, skill, and caution.”

The Uniform Trust Code further provides that a “trustee shall take reasonable steps to take control of and
protect the trust property.” This section is based upon the Second Restatement of Trusts § 176. A
comment to § 176 of the Restatement specifies that a trustee’s failure to purchase fire/casualty insurance
for trust property when such “insurance is customarily taken by a prudent” person is a breach of the duty
to protect (a subset of the duty of care/prudent administration).

Here, the trustee failed to purchase a fire/casualty policy on the trust’s rental property, resulting in a loss
to the trust. This is a breach of trust because a prudent person would have purchased such a policy on the
property.

[NOTE: An accurate analysis of the legal standards is more important than the examinee’s conclusion
regarding whether the purchase of insurance would have been prudent.]

Explanation to Point One(c) (30%):

The trustee breached his duty to administer the trust in accordance with applicable law by allocating the
$50,000 repair expense exclusively to income.

Seperac-J19 Exam-Released MEE Essay Compilation © 2016-2020 1153


A trustee has a duty to administer the trust “diligently and in good faith, in accordance with the terms of
the trust and applicable law.” Because of the trustee’s obligation to comply with applicable law, the
trustee must comply with the Uniform Principal and Income Act, or other state statutes of like effect, in
allocating receipts and disbursements between trust income and principal.

Under the Uniform Principal and Income Act, “all ordinary expenses incurred in connection with the
preservation of trust property including ordinary repairs” are allocated to income. Extraordinary repairs
are allocated to principal. Ordinary repairs are repairs required by day-to-day wear and tear. Extraordinary
repairs are repairs required by “an unusual or unforeseen occurrence that does not destroy the asset but
merely renders it less suited to its intended use, a repair that is beyond the usual, customary, or regular
kind.”

Here, the $50,000 roof repair was extraordinary because it was required by an unforeseen occurrence, a
fire. It should therefore have been allocated to principal, not income, and paid from the trust principal’s
$120,000 in liquid assets. This conclusion is bolstered by the fact that, had the trustee obtained insurance
on the property, the insurance proceeds, representing the value of needed repairs, would have been
allocated to principal.

[NOTE: The duties discussed in this section are sometimes referred to as an aspect of the duty of
impartiality.]

Explanation to Point Two (20%):

Under the common law, upon Albert’s death, the trust principal should be distributed to Betty’s husband
because she devised her vested remainder in the trust to him.

Under the terms of the trust, the testator created a life estate in Albert and a remainder interest in Betty.
Under the common law, Betty’s interest was vested because it was subject to no contingencies (such as
“to Betty if she survives Albert”) that would cause her to lose her interest should she predecease Albert.
Vested remainders are devisable. Thus, if the remainderman is not living when the remainder becomes
possessory, it passes to the devisee of the interest under the deceased remainderman’s will.

Here, trust principal became possessory at Albert’s death, and Betty was not living at that time. The trust
principal thus should go to the devisee named in Betty’s will, her husband. Under the common law, the
testator’s son would not take a share of trust principal as there is no reversionary interest to pass to the
testator’s heirs.

[NOTE: If Betty’s trust interest were subject to a survivorship contingency, that would have caused
Betty’s interest to fail because she predeceased Albert. In that case, her interest would have reverted to
the testator’s estate. In such case, it would have passed to his heirs if he died intestate or to his
beneficiaries if he died with a will.]

Seperac-J19 Exam-Released MEE Essay Compilation © 2016-2020 1154


#269-FEB 2017–MEE Q02: QUESTION TWO (TRUSTS)

Forty years ago, Settlor, a successful businesswoman, married a less-than-successful writer. Settlor and
her husband had two children, a son and a daughter.

Two years ago, Settlor transferred most of her wealth into a revocable trust. Under the terms of the trust
instrument, a local bank was designated as trustee, and the trustee was directed to distribute all trust
income to Settlor during her lifetime. The trust instrument further provided that "upon Settlor's death, the
trustee will distribute trust principal to one or more of Settlor's children as Settlor shall appoint by her
duly probated last will or, in the absence of such appointment, to Charity." The trust instrument also
stated that Settlor's power of revocation was exercisable only "during Settlor's lifetime and by a written
instrument."

Following the creation of the trust, Settlor gave written direction to the trustee to accumulate trust income
instead of distributing the income to Settlor as specified in the trust instrument. The trustee did so.

Six months ago, Settlor executed a valid will. The will, exercising the power of appointment created
under Settlor's revocable trust, directed the trustee of Settlor's trust, upon Settlor's death,

(1) to distribute half of the trust assets to Settlor's daughter,

(2) to hold the other half of the trust assets in continuing trust and pay income to Settlor's son
during the son's lifetime, and

(3) upon the son's death, to distribute the trust principal in equal shares to the son's surviving
children (grandchildren of Settlor).

Settlor also bequeathed $50,000 "to my descendants, other than my children, in equal shares," and she left
the residue of her estate to her husband, whom she also named as the executor of her estate.

Two months ago, Settlor died. At Settlor's death, the trust assets were worth $500,000 and Settlor's
probate assets were worth $100,000. Settlor was survived by her husband, her daughter, her son, and her
son's child (Settlor's grandchild, age 18).

A statute in this jurisdiction provides that a decedent's surviving spouse is entitled to a "one-third elective
share of the decedent's probate estate." There are no other relevant statutes.

1. Was it proper for the trustee to accumulate trust income during Settlor's lifetime? Explain.

2. Under Settlor's will and the trust instrument, what, if any, is Charity's interest in the trust assets?
Explain.

3. Does Settlor's husband have a valid claim to any trust or probate assets? Explain.

Seperac-J19 Exam-Released MEE Essay Compilation © 2016-2020 1155


#269: F17-2 MEE: ANSWER: NCBE (TRUSTS)

POINT (1) [25%] ISSUE: Can the trustee of a revocable trust, who is required by the terms of the
trust instrument to pay all trust income to the settlor, accumulate income upon the settlor’s written
direction? ANSWER: Yes. It was proper for the trustee to accumulate trust income because Settlor
retained a power to revoke the trust during her lifetime in a written instrument, and the trustee
accumulated income based on written directions from Settlor during her lifetime.

POINT (2)(a) [25%] ISSUE: Can the holder of a testamentary power of appointment exercisable in
favor of the holder’s children exercise the power by appointing trust assets to the holder’s
grandchildren? ANSWER: No. Settlor could not appoint an interest in the trust to her son’s
children (Settlor’s grandchildren) because they were not permissible objects of her power of
appointment.

POINT (2)(b) [20%] ISSUE: Who takes trust assets that have been impermissibly appointed?
ANSWER: As the named taker in default of appointment, Charity is entitled to the portion of the
trust ineffectively appointed to Settlor’s grandchildren. It is not entitled to any interest appointed to
the son.

POINT (3) [30%] ISSUE: Can the surviving spouse of the decedent settlor of a revocable trust
claim an elective share of assets in that trust even though the spouse is not a named beneficiary of
the trust? ANSWER: Yes. Settlor’s husband may be entitled to an elective (i.e., forced) share of
Settlor’s revocable trust assets under the illusory-transfer doctrine, the fraudulent-transfer
doctrine, or a like doctrine. He is also entitled to one-third of the probate assets.

ANSWER DISCUSSION:

A trustee is obligated to administer a trust in accordance with the trust terms. However, when the settlor
of a trust has the power to revoke, she also has the power to amend the terms of the trust. Here, the trust
was revocable, Settlor had the power to amend, and the accumulation of trust income at Settlor’s written
direction was the equivalent of an amendment to the trust. Thus, the trustee properly accumulated trust
income. A power of appointment can only be exercised in favor of a permissible object of the power; any
property impermissibly appointed passes to the taker in default of appointment. Here, the trust instrument
gave Settlor a special testamentary power of appointment; the only permissible objects of the power were
Settlor’s son and daughter. Thus, Settlor’s appointment of trust assets to her grandchildren was
impermissible. Settlor’s testamentary appointment to the son’s children cannot be treated as a valid trust
amendment because Settlor’s retained power of revocation was exercisable only during her lifetime and,
here, the appointment was by her will. Because the testamentary appointment of trust assets to the son’s
children was impermissible, the share that would have passed to them upon the son’s death passes to
Charity, the taker in default of appointment. The power to appoint trust assets includes the power to
appoint in further trust. Thus, Settlor’s appointment of her son’s share of trust assets in trust was proper.
Accordingly, Charity cannot reach the son’s interest in the trust by claiming that the appointment to the
son in trust was impermissible. Settlor’s husband is entitled to a one-third elective share of the probate
estate. If he could not reach the trust assets, he would not make the election but would merely claim the
$50,000 left to him under the will, as that amount exceeds one-third of the probate estate. If, as is more
likely, he could reach the trust assets, the probate and trust estates would have an aggregated value of
$600,000 and he would be entitled to a $200,000 elective share. Presumably, in satisfying that share he

Seperac-J19 Exam-Released MEE Essay Compilation © 2016-2020 1156


would take $50,000 from the probate estate, which was left to him by Settlor’s will, and the rest from the
trust.

ANSWER EXPLANATION:

Explanation to Point-One (25%):

It was proper for the trustee to accumulate trust income because Settlor retained a power to revoke the
trust during her lifetime in a written instrument, and the trustee accumulated income based on written
directions from Settlor during her lifetime.

A retained power to revoke a trust includes the power to modify or amend the trust instrument. Courts
have taken this position to avoid the triumph of form over substance; the contrary position would require
a settlor who wants to amend a trust and lacks clear authority to do so to first revoke, and then to
completely restate, the terms of the trust with the intended amendment. Such cumbersome formalities
should not be encouraged; thus, the power to revoke includes the power to amend.

The Uniform Trust Code (UTC) follows this approach; under the UTC, a trust is both revocable and
amendable unless the trust instrument expressly provides otherwise. Under the UTC, the power to revoke
or amend is exercisable by will unless, as here, the trust instrument provides otherwise.

Here, the trust instrument required the trustee to distribute all trust income to Settlor during her lifetime;
Settlor retained the power to revoke the trust “during Settlor’s lifetime”; and the trustee accumulated trust
income at Settlor’s written direction, drafted during her lifetime. Settlor’s written direction effectively
amended the terms of the trust as initially created. Thus, no improper accumulation of income occurred.

Explanation to Point-Two(a) (25%):

Settlor could not appoint an interest in the trust to her son’s children (Settlor’s grandchildren) because
they were not permissible objects of her power of appointment.

The donee of a special (nongeneral) power can appoint the property over which the power is exercisable
only to “permissible appointees” or “objects” of the power. Permissible appointees are “the persons to
whom an appointment is authorized.” Appointments to impermissible appointees are invalid. However,
objects of a power include only those who receive a “beneficial interest.” Thus, when, as here, property is
appointed in further trust, the trustee is not an impermissible appointee.

Although the trustee is not an impermissible appointee, Settlor’s grandchildren (her son’s children) are
impermissible appointees. Under the trust instrument, Settlor’s “children” were the only permissible
objects of her testamentary power of appointment. It might be argued that Settlor intended to include
more remote descendants, such as grandchildren, when she used the word “children” when creating the
power of appointment. However, Settlor’s will – executed contemporaneously with the trust instrument –
includes a $50,000 bequest to her “descendants other than my children,” clearly evidencing Settlor’s
ability to distinguish between children and more remote descendants. The argument that Settlor intended
to include more remote descendants in the term “children” is thus highly likely to fail, and Settlor’s
appointment of trust assets to the children of Settlor’s son thus should be ineffective.

The trust instrument allowed for amendments during Settlor’s lifetime by a written instrument. However,
the appointment in Settlor’s will cannot be treated as an amendment made during Settlor’s lifetime by a
written instrument.

Seperac-J19 Exam-Released MEE Essay Compilation © 2016-2020 1157


Explanation to Point-Two(b) (20%):

As the named taker in default of appointment, Charity is entitled to the portion of the trust ineffectively
appointed to Settlor’s grandchildren. It is not entitled to any interest appointed to the son.

“If part of an appointment is ineffective and another part, if standing alone, would be effective, the
effective part is given effect, except to the extent that the donee’s scheme of disposition is more closely
approximated by concluding that some or all of the otherwise effective part should be treated as
ineffective.” To the extent that a power is ineffectively appointed, the ineffectively appointed property
passes to the so-called “taker in default of appointment” (here, Charity) designated by the donor of the
power (here, Settlor).

Thus, assuming that the appointment to Settlor’s son’s children is ineffective, the share they would have
received upon the son’s death passes to Charity. The appointment in trust to the son is permissible. Thus,
Charity has no claim to the trust income interest appointed to the son.

Explanation to Point-Three (30%):

Settlor’s husband may be entitled to an elective (i.e., forced) share of Settlor’s revocable trust assets under
the illusory-transfer doctrine, the fraudulent-transfer doctrine, or a like doctrine. He is also entitled to one-
third of the probate assets.

Although many states have statutes under which a surviving spouse’s elective (forced) share of the
decedent spouse’s estate extends to assets placed into a revocable trust by the decedent spouse, here, the
jurisdiction’s elective-share statute provides that the spousal right of election extends only to probate
assets. Because the disposition of assets in a revocable trust is determined by the terms of the trust
instrument, trust assets are not probate assets. Thus, the statute does not give Settlor’s husband any claim
to assets in her revocable trust. Assuming that the husband has no claim to the trust assets, he will not
claim an elective share because that share ($33,333) would be less than the amount he is entitled to under
the will ($50,000).

However, in many states, case law permits a surviving spouse to claim an elective share of assets
contained in a revocable trust under either the illusory-transfer doctrine, the fraudulent-transfer doctrine,
or a like doctrine.

Under the illusory-transfer doctrine, a surviving spouse can reach assets transferred during the marriage
by the deceased spouse into a revocable trust on the theory that the transfer is economically “illusory”
because, by the simple expedient of exercising the power of revocation – typically with nothing more than
a signature on a piece of paper – the deceased spouse could have recaptured the assets she had placed in
the trust.

Under the fraudulent-transfer doctrine, a surviving spouse can reach assets transferred into a revocable
trust on the theory that, as to the surviving spouse, the transfer was “fraudulent.” The assumption behind
this doctrine is that a state statute providing surviving spouses with an elective-share entitlement gives
spouses a legitimate expectancy in assets that would have been included in the decedent spouse’s probate
estate but for their transfer into a revocable trust; such a transfer is treated as defrauding the surviving
spouse of his or her expectancy.

Thus, if this jurisdiction recognizes one of these doctrines or another similar doctrine, Settlor’s husband
will be entitled to receive one-third of the combined probate ($100,000) and trust estate ($500,000) or

Seperac-J19 Exam-Released MEE Essay Compilation © 2016-2020 1158


$200,000 (1/3 of $600,000). However, this $200,000 would be reduced to $150,000 to take account of the
fact that he received $50,000 under the will.

Not all jurisdictions recognize such a doctrine, however. In those jurisdictions that do not, the husband
would receive nothing beyond the $50,000 he would receive under the will because that amount exceeds
the elective share (1/3 of $100,000).

[NOTE: If an examinee concludes that the husband can elect against the trust assets, the examinee may
also discuss who would bear the burden of paying the husband’s share. No credit should be given for this
discussion, as the payment issue is beyond the scope of the question.]

Seperac-J19 Exam-Released MEE Essay Compilation © 2016-2020 1159


#270-JUL 2015–MEE Q06: QUESTION SIX (TRUSTS)

In 1995, a man and his friend created a corporation. The man owned 55% of the stock, and the friend
owned 45% of the stock. When the man died in 2005, he left all of his stock in the corporation to his wife.

In 2009, the wife died. Under her duly probated will, the wife bequeathed the stock her husband had left
her to a testamentary trust and named her husband’s friend as trustee. Under the wife’s will, the trustee
was required to distribute all trust income to the wife’s son “for so long as he shall live or until such time
as he shall marry” and, upon the son’s death or marriage, to distribute the trust principal to a designated
charity. The stock, valued at $500,000 at the wife’s death, comprised the only asset of this trust.

In 2013, after the stock’s value had risen to $1.5 million, the trustee’s lawyer properly advised the trustee
to sell the stock in order to comply with the state’s prudent investor act. Because of this advice, the trustee
decided to sell the stock. However, instead of testing the market for potential buyers, the trustee
purchased the stock himself for $1.2 million. Thereafter, on behalf of the trust, the trustee invested the
$1.2 million sales proceeds in a balanced portfolio of five mutual funds (including both stocks and bonds)
with strong growth and current income potential.

Recently, both the son and the charity discovered the trustee’s sale of the stock to himself and his
reinvestment of the proceeds from the stock’s sale. They learned that, due to general economic conditions,
the stock in the corporation that had been purchased by the trustee for $1.2 million had declined in value
to $450,000 and the value of the trust’s mutual-fund portfolio had declined from $1.2 million to $1
million. Both the son and the charity have threatened to sue the trustee.

The son has also decided that he wants to get married and has notified the trustee that he believes the trust
provision terminating his income interest upon marriage is invalid.

1. Would the son’s interest in the trust terminate upon the son’s marriage? Explain.

2. Did the trustee breach any duties by buying the trust’s stock and, if yes, what remedies are
available to the trust beneficiaries if they sue the trustee? Explain.

3. Did the trustee breach any duties in acquiring and retaining the portfolio of mutual funds and, if
yes, what remedies are available to the trust beneficiaries if they sue the trustee? Explain.

Seperac-J19 Exam-Released MEE Essay Compilation © 2016-2020 1160


#270: J15-6 MEE: ANSWER: NCBE (TRUSTS)

POINT (1) [30%] ISSUE: Is the trust condition providing for the termination of an income interest
upon marriage invalid as a matter of public policy? ANSWER: No. The son’s income interest in the
trust would not terminate upon his marriage because the condition against marriage is void as a
matter of public policy.

POINT (2) [35%] ISSUE: Did the trustee violate his duty of loyalty by purchasing the stock of the
closely held corporation from the trust and, if so, what damages or other remedies are the trust
beneficiaries likely to obtain? ANSWER: Yes. The trustee breached his duty of loyalty by
purchasing stock from the trust, an act of self-dealing. In such a case, trust beneficiaries may obtain
an order setting aside the transaction or seek damages based on the difference between the
purchased assets’ fair market value at the time of purchase and the sales price paid by the trustee.

POINT (3) [35%] ISSUE: Did the trustee violate the prudent investor rule by investing trust
principal in a balanced portfolio of mutual funds with potential for growth and current income and
retaining those investments after they declined in value? ANSWER: No. A trustee has a duty to
invest trust assets in a prudent manner. Here, there are no facts suggesting that the trustee
breached any prudent investment duty with respect to the selection and management of the
investments he made.

ANSWER DISCUSSION:

A trust condition that provides for the termination of a beneficiary’s interest upon marriage is void as
against public policy, except when the beneficiary is the trust creator’s spouse. Thus, the son’s interest
would not terminate upon his marriage. A trustee’s purchase of trust assets is an act of self-dealing and a
breach of the duty of loyalty. The trust beneficiaries can either rescind the transaction or seek damages.
Where, as here, the value of the asset has substantially declined, the beneficiaries are more likely to seek
money damages (here, $300,000) for the breach. Neither the investment of trust assets in a balanced
portfolio of mutual funds with potential for growth and current income nor the retention of those assets
during a period of general economic decline violates the prudent investor rule. Thus, neither the son nor
the charity is entitled to damages here.

ANSWER EXPLANATION:

Explanation to Point-One (30%):

The son’s income interest in the trust would not terminate upon his marriage because the condition
against marriage is void as a matter of public policy.

Provisions of trusts that violate public policy are void. Trust provisions that restrain a first marriage have
generally been held to violate public policy. Under the Second Restatement of Property, a provision
restraining any first marriage is invalid and the transfer takes effect “as though the restriction had not been
imposed. “The common law has a well-defined interest in preserving freedom of marriage and in the
preservation of the family relation. Hence provisions in a trust instrument which provide that a beneficiary
shall have no interest under the trust unless he obeys the instructions of the settlor regarding marriage may
be held illegal as against public policy.” Although courts have upheld some restrictions on marriage – for
example, remarriage – none have upheld a complete restraint on a first marriage. Thus, because the son’s

Seperac-J19 Exam-Released MEE Essay Compilation © 2016-2020 1161


income interest would terminate upon his marriage, no matter what the circumstances of that marriage,
the provision is void and the son’s income interest continues.

A restraint on marriage might be upheld if the trustee’s motive was merely to provide support for a
beneficiary while the beneficiary is single. Here, there are no facts to support that motive. In fact, because
this is a mandatory income payout trust, the trust income is payable to the son without regard to his
support needs.

Explanation to Point-Two (35%):

The trustee breached his duty of loyalty by purchasing stock from the trust, an act of self-dealing. In such
a case, trust beneficiaries may obtain an order setting aside the transaction or seek damages based on the
difference between the purchased assets’ fair market value at the time of purchase and the sales price paid
by the trustee.

A trustee owes a fiduciary duty of loyalty to a trust; self-dealing, such as a purchase of trust assets by the
trustee in his individual capacity, violates this obligation. Under the “no further inquiry” rule, there is no
need to inquire into the motivation for the self-dealing transaction or even its fairness.

Any trust beneficiary can cause a self-dealing purchase by a trustee to be set aside or obtain a damages
award. If a beneficiary elects to set aside the transaction, the trust property purchased by the trustee is
returned to the trust and the amount the trustee paid for the property is refunded by the trust. If a
beneficiary seeks damages, those damages are based on the difference in the fair market value of the trust
assets at the time of the self-dealing transaction and the amount paid by the trustee. Where, as here, the
assets purchased by the trustee have declined in value since the self-dealing transaction, trust beneficiaries
are likely to seek damages instead of setting aside the transaction.

Here, the beneficiaries should elect to seek damages from the trustee, not rescission. First, with rescission,
the trustee would return the shares to the trust, and the trust would have to refund the purchase price to the
trustee. This would leave the trust with assets worth only $450,000. Furthermore, because of the market
declines, the trust assets are currently worth only $1,000,000; thus the trust doesn’t have sufficient assets
to refund the purchase price. On the other hand, by seeking damages, the trust would collect $300,000,
representing the difference between the value of the shares when purchased by the trustee ($1,500,000)
and the purchase price ($1,200,000), leaving the trust with $1,300,000 in assets.

The trustee also breached his duty of care when selling the stock because of his failure to test the market.

Explanation to Point-Three (35%):

A trustee has a duty to invest trust assets in a prudent manner. Here, there are no facts suggesting that the
trustee breached any prudent investment duty with respect to the selection and management of the
investments he made.

Under the Uniform Trust Code, a trustee must administer “the trust as a prudent person would, using
reasonable care, skill, and caution.” One of the hallmarks of prudent investing is diversification. A
balanced portfolio reduces aggregate risk by investing in different investment categories. Diversification
thus is strong evidence of prudent investing. Indeed, failure to diversify is likely the reason why the
trustee in this case was advised to sell the closely held corporate stock.

Seperac-J19 Exam-Released MEE Essay Compilation © 2016-2020 1162


“A trustee’s investment and management decisions respecting individual assets must be evaluated not in
isolation but in the context of the trust portfolio as a whole and as a part of an overall investment strategy
having risk and return objectives reasonably suited to the trust.” The trustee should consider the purposes,
terms, distribution requirements, and other circumstances affecting the trust. The prudent investor rule
applies to both investment and management decisions. Management includes monitoring; thus, the trustee
has a duty to monitor investments prudently made to assure that retention of those investments remains
prudent. If retention is not prudent, the trustee should sell the imprudent investments and reinvest the
proceeds in prudent investments. A trustee, however, is not liable for declines in value due to a downturn
resulting from general economic conditions.

Here, the trustee’s investment of the sale proceeds seems to satisfy the diversification requirement. The
trustee selected a balanced group of mutual funds; the portfolio included both stock and bond funds; it
contained both growth and income funds. The trustee also appears to have considered the needs of both
the income beneficiary and remainderman; growth funds are aimed at achieving principal appreciation
and income funds at producing current income. The funds’ decline in value during a period of general
economic decline – when most types of investments may well have fallen in value – does not evidence
lack of prudence, and there are no facts to show any failure to monitor the portfolio.

Seperac-J19 Exam-Released MEE Essay Compilation © 2016-2020 1163


#271-FEB 2014–MEE Q02: QUESTION TWO (TRUSTS)

Ten years ago, a testator died, survived by his only children: a son, age 26, and a daughter, age 18.

A testamentary trust was created under the testator’s duly probated will. The will specified that all trust
income would be paid to the son during the son’s lifetime and that upon the son’s death, the trust would
terminate and trust principal would be distributed to the testator’s “grandchildren who shall survive” the
son. The testator provided for his daughter in other sections of the will.

Five years ago, the trustee of the testamentary trust purchased an office building with $500,000 from the
trust principal. Other than this building, the trust assets consist of publicly traded securities.

Last year, the trustee received $30,000 in rents from the office building. The trustee also received, with
respect to the securities owned by the trust, cash dividends of $20,000 and a stock dividend of 400 shares
of Acme Corp. common stock distributed to the trust by Acme Corp.

Eight months ago, the trustee sold the office building for $700,000.

Six months ago, the son delivered a letter to the trustee stating: “I hereby disclaim any interest I may have
in the income interest of the trust.” On the date the son delivered this letter to the trustee, the son had no
living children; the daughter had one living minor child.

A statute in this jurisdiction provides that “a disclaimer of any interest created by will is valid only if
made within nine months after the testator’s death, and if an interest is validly disclaimed, the disclaiming
party is deemed to have predeceased the testator.”

1. How should the rents, sales proceeds, cash dividends, and stock dividends received prior to the
trustee’s receipt of the son’s letter have been allocated between trust principal and income?
Explain.

2. How, if at all, does the son’s letter to the trustee affect the future distribution of trust income and
principal? Explain.

Seperac-J19 Exam-Released MEE Essay Compilation © 2016-2020 1164


#271: F14-2 MEE: ANSWER: NCBE (TRUSTS)

POINT (1) [45%] ISSUE: How should rents, dividends, and sales proceeds received by the trustee
prior to receipt of the son’s letter have been allocated between trust income and principal?
ANSWER: Cash dividends and rents are allocable to income; sales proceeds and stock dividends
are allocable to principal. Items allocable to income for the period prior to the son’s attempted
disclaimer were distributable to the son.

POINT (2)(a) [45%] ISSUE: Did the remainder interest in the trust accelerate and become
immediately payable to the daughter’s minor child upon the trustee’s receipt of the son’s letter,
and, if not, how should the trustee handle the distribution of the principal in the future? ANSWER:
No. Because the son did not disclaim within nine months of the testator’s death, there is no valid
disclaimer under state law. Therefore, the son is not deemed to have predeceased the testator.
Furthermore, because of the express survivorship contingency in the will, the remainder in the trust
does not accelerate and become distributable until the son in fact dies. When the son dies, the trust
principal will be distributable to the testator’s then-living grandchildren, or if none, then to the
testator’s then-living heirs.

POINT (2)(b) [10%] ISSUE: Following the trustee’s receipt of the son’s letter, how should the
trustee distribute future receipts of income prior to the distribution of the principal? ANSWER:
Until the trust terminates, the trustee must continue to hold the trust assets. The distribution of
income in the meantime is unclear. There are at least three possibilities. Income earned on the
undistributed assets could be distributed to the son and daughter as the testator’s heirs,
accumulated and added to principal for distribution to the ultimate remaindermen, or distributed
from time to time to those persons who are presumptively remaindermen.

ANSWER DISCUSSION:

Prior to the trustee’s receipt of the son’s letter, cash dividends and rents should have been allocated to
trust income and were distributable to the son, the income beneficiary of the trust; sales proceeds and
stock dividends should have been allocated to principal. Because the son’s letter to the trustee did not
result in a valid disclaimer under state law (having been made more than nine months after the testator’s
death), the son is not deemed to have predeceased the testator. Because the son is still living, the class gift
to the testator’s grandchildren who survive the son has not closed and is not possessory; it will not
become possessory until the son dies. The daughter’s minor child, being the testator’s only living
grandchild, is not currently entitled to a distribution of trust principal. Trust principal will instead be
distributable upon the son’s death to the testator’s then-living grandchildren or, if there are none, to the
testator’s then-living heirs. As for future income, the trustee should either distribute the trust income to
the son and the daughter as the testator’s heirs, accumulate the income for future distribution to those
individuals ultimately entitled to the trust principal, or distribute it to those presumptively entitled to the
principal upon the son’s death, i.e., the daughter’s minor child.

ANSWER EXPLANATION:

Explanation to Point-One (45%):

Seperac-J19 Exam-Released MEE Essay Compilation © 2016-2020 1165


Cash dividends and rents are allocable to income; sales proceeds and stock dividends are allocable to
principal. Items allocable to income for the period prior to the son’s attempted disclaimer were
distributable to the son.

Receipts earned during the administration of a trust are allocable either to income or to principal. Almost
all states have adopted the most recent or an earlier version of the Uniform Principal and Income Act (the
Act), which specifies how such receipts should be allocated.

Under the Act, rents and cash dividends received from a corporation are allocable to income and are
distributable to the income beneficiary of the trust.

Sales proceeds and dividends paid in the stock of the distributing corporation are allocable to principal
and added to the principal of the trust.

Here, the cash dividends and office building rents should have been allocated to income and, until the
trustee received the son’s letter, should have been distributed to him as the sole income beneficiary of the
trust. The stock dividend and proceeds from the sale of the office building should have been allocated to
principal and held by the trustee for future distribution to the ultimate remaindermen of the trust.

[NOTE: The 2000 Uniform Principal and Income Act has been adopted in 19 states.]

Explanation to Point-Two(a) (45%):

Because the son did not disclaim within nine months of the testator’s death, there is no valid disclaimer
under state law. Therefore, the son is not deemed to have predeceased the testator. Furthermore, because
of the express survivorship contingency in the will, the remainder in the trust does not accelerate and
become distributable until the son in fact dies. When the son dies, the trust principal will be distributable
to the testator’s then-living grandchildren, or if none, then to the testator’s then-living heirs.

When a trust remainder is given to a class, the class closes (i.e., no new persons can join the class) when
there is no outstanding income interest, and at least one member of the class is then entitled to demand
possession of his or her share of the remainder. This principle is called the rule of convenience. A class
member may demand possession of his or her share of the remainder upon termination of the income
interest only when the class member’s interest is not otherwise subject to a condition precedent.

When a beneficiary timely disclaims an interest in a trust, that beneficiary is treated as if he had
predeceased the testator. Here, had the son disclaimed within nine months of the testator’s death as
required by the state statute, he would have been deemed to have predeceased the testator. This would
have closed the class of remaindermen, and the testator’s then-living grandchildren (i.e., the daughter’s
child) would have been entitled to the trust principal. However, under the state statute, the son’s
disclaimer was not timely because he did not disclaim within nine months of the testator’s death. Thus,
because the statute is inapplicable and the son is still alive, the class of grandchildren entitled to share in
trust principal did not close.

Because, here, the statute is inapplicable due to the son’s failure to comply with the statutory time
requirements, then presumably the common-law rule allowing disclaimers (a/k/a renunciations) at any
time should apply. Under the common law, if a life estate is renounced, the remainder interest accelerates
and becomes immediately distributable to the remaindermen of the trust if the remainder is vested but not
if the remainder is contingent. Here, because the remainder is contingent upon there being grandchildren
who survive the son, the remainder will not accelerate. It will remain open until the son dies, leaving open

Seperac-J19 Exam-Released MEE Essay Compilation © 2016-2020 1166


the possibility that additional grandchildren will be included in the class, or the daughter’s child could fall
out of the class because that child fails to survive the son.

And if none of the testator’s grandchildren survive the son, the trust principal will be distributed to the
testator’s heirs living at the son’s death.

Explanation to Point-Two(b) (10%):

Until the trust terminates, the trustee must continue to hold the trust assets. The distribution of income in
the meantime is unclear. There are at least three possibilities. Income earned on the undistributed assets
could be distributed to the son and daughter as the testator’s heirs, accumulated and added to principal for
distribution to the ultimate remaindermen, or distributed from time to time to those persons who are
presumptively remaindermen.

When trust principal is not immediately distributable, the trustee must continue to hold trust assets until
the ultimate remaindermen are ascertained. During this period, trust income will be distributed or retained
according to any instructions contained in the trust instrument.

Here, the testator did not specify what the trustee should do with trust income in the event the son’s
disclaimer did not comply with the state statute. There are at least three approaches. One approach would
have the trustee distribute the trust income to the testator’s heirs on the theory that the income represents
property that was not disposed of by the testator’s will and which thus passes by partial intestacy to the
testator’s heirs. A second approach would have the trustee accumulate trust income for distribution to the
ultimate remaindermen. Under this approach, only those individuals ultimately entitled to the principal
would share in the income. A third approach would have the trustee distribute trust income to those
individuals who would be the remaindermen if the trust were to terminate when the income is received by
the trustee; under this approach, trust income would be distributed to the daughter’s minor child until
another presumptive remainderman is born. This approach could result in individuals not ultimately
entitled to principal, say because they do not survive the son, receiving income. It could also result in a
disproportionate distribution of income among the individuals ultimately entitled to income.

[NOTE: Examinees should demonstrate a recognition and understanding of the income-allocation


problem and the alternatives available to address that issue. There is no widely accepted solution to the
problem. Examinees who cite any of these possible problem-solving approaches may receive credit.]

Seperac-J19 Exam-Released MEE Essay Compilation © 2016-2020 1167


#272-FEB 2013–MEE Q08: QUESTION EIGHT (TRUSTS)

Ten years ago, Settlor validly created an inter vivos trust and named Bank as trustee. The trust instrument
provided that Settlor would receive all of the trust income during her lifetime. The trust instrument further
provided that

Upon Settlor’s death, the trust income shall be paid, in equal shares, to Settlor’s surviving children for
their lives. Upon the death of the last surviving child, the trust income shall be paid, in equal shares, to
Settlor’s then-living grandchildren for their lives. Upon the death of the survivor of Settlor’s children and
grandchildren, the trust corpus shall be distributed, in equal shares, to Settlor’s then-living great-
grandchildren.

The trust instrument expressly specified that the trust was revocable, but it was silent regarding whether
Settlor could amend the trust instrument.

Immediately after creating the trust, Settlor validly executed a will leaving her entire estate to Bank, as
trustee of her inter vivos trust, to “hold in accordance with the terms of the trust.”

Five years ago, Settlor signed an amendment to the inter vivos trust. The amendment changed the
disposition of the remainder interest, specifying that all trust assets “shall be paid upon Settlor’s death to
University.” Settlor’s signature on this amendment was not witnessed.

A state statute provides that any trust interest that violates the common law Rule Against Perpetuities “is
nonetheless valid if the nonvested interest in the trust actually vests or fails to vest either (a) within 21
years of lives in being at the creation of the nonvested interest or (b) within 90 years of its creation.”

Recently, Settlor died, leaving a probate estate of $200,000. She was survived by no children, one
granddaughter (who would be Settlor’s only heir), and no great-grandchildren. The granddaughter has
consulted your law firm and has raised four questions regarding this trust:

1. Was Settlor’s amendment of the inter vivos trust valid? Explain.

2. Assuming that the trust amendment was valid, do its provisions apply to Settlor’s probate assets?
Explain.

3. Assuming that the trust amendment was valid, how should trust assets be distributed? Explain.

4. Assuming that the trust amendment was invalid, how should trust assets be distributed? Explain.

Seperac-J19 Exam-Released MEE Essay Compilation © 2016-2020 1168


#272: F13-8 MEE: ANSWER: NCBE (TRUSTS)

POINT (1)(a) [30%] ISSUE: Was the revocable trust amendable? ANSWER: Yes. Settlor retained
the right to amend the inter vivos trust despite her failure to expressly reserve this power.

POINT (1)(b) [10%] ISSUE: If the trust was amendable, must the amendment have been executed
in accordance with the state Statute of Wills in order to be valid? ANSWER: No. Settlor’s
amendment of the trust was valid despite her failure to have her signature to the trust amendment
witnessed.

POINT (2) [20%] ISSUE: If the trust amendment was valid, does the amendment apply to the
probate estate assets passing to the trust pursuant to Settlor’s will? ANSWER: Yes. Under the
Uniform Testamentary Additions to Trusts Act, a revocable trust may be amended at any time
prior to the settlor’s death, and the amendment applies to probate assets poured into the trust at
the settlor’s death pursuant to the settlor’s will even when the will was executed prior to the date of
the amendment.

POINT (3) [10%] ISSUE: If the trust amendment was valid, should the trust property be
distributed to University? ANSWER: Yes. If the trust amendment was valid, then the trust assets,
including assets passing to the trust under Settlor’s will, should go to University.

POINT (4) [30%] ISSUE: If the trust amendment was not valid, should the trust property be
distributed to Settlor’s grandchild (her only heir) or held in further trust in accordance with the
terms of the original trust instrument? ANSWER: If the trust amendment was invalid, trust assets,
including assets received pursuant to Settlor’s will, should be held in accordance with the terms of
the original trust instrument because those terms do not violate the Rule Against Perpetuities.

ANSWER DISCUSSION:

A revocable trust is amendable even if the trust instrument does not expressly grant to the trust settlor a
power to amend. Both inter vivos trusts and amendments thereto are valid even though not executed in
accordance with the requirements applicable to wills. Under the Uniform Testamentary Additions to
Trusts Act, a revocable trust may be amended at any time prior to the settlor’s death, and the amendment
applies to the disposition of assets conveyed to the trust pursuant to a will even if the will was executed
prior to the date of the amendment. At Settlor’s death, trust assets, including probate assets passing to the
trust under Settlor’s will, would go to University if, as is the case here, the trust amendment was valid. If
the amendment was invalid, the trust assets would continue to be held in further trust because there is no
violation of the common law Rule Against Perpetuities.

ANSWER EXPLANATION:

Explanation to Point-One(a) (30%):

Settlor retained the right to amend the inter vivos trust despite her failure to expressly reserve this power.

At issue here is whether a retained power of revocation includes the power to amend, sometimes referred
to as the power to modify. The Second Restatement of Trusts § 331 provides that if a settlor has a power
to revoke, that retained power ordinarily includes a power to modify (amend) as well. Comment g also

Seperac-J19 Exam-Released MEE Essay Compilation © 2016-2020 1169


notes that the power to amend includes both a power to withdraw trust assets and a power to “modify the
terms of the trust.” The Uniform Trust Code, which provides that a power to revoke includes the power to
amend, is consistent with this view. The theory is that even though a power to amend was not expressly
retained by a settlor, the goal of amendment, assuming the power was not included in the power to
revoke, could easily be achieved by first revoking the trust and then creating a new trust with the same
terms contemplated by the amendment. To require this would put form over substance.

Thus, by expressly retaining the power to revoke the trust, Settlor retained a power to amend the inter
vivos trust despite her failure to expressly reserve this power.

[NOTE: Under the common law, a trust is irrevocable unless the settlor expressly retains a power to
revoke the trust. Conversely, under the Uniform Trust Code, a trust is revocable unless the terms of the
trust expressly provide otherwise. The Trust Code’s position on revocation follows the minority view in
the United States and is inconsistent with prior Restatements of Trusts. Here, the trust is revocable
because Settlor expressly retained a power of revocation.

The Uniform Trust Code has been adopted in 24 states.]>

Explanation to Point-One(b) (10%):

Settlor’s amendment of the trust was valid despite her failure to have her signature to the trust amendment
witnessed.

Neither the common law nor state statutes require a trust instrument or an amendment to a trust
instrument to be executed in accordance with the formalities prescribed for execution of a will. Indeed, an
inter vivos trust that does not involve real estate can be created orally. Under the Uniform Trust Code, the
only requirements for creating a valid inter vivos trust are intent, the specification of beneficiaries, and the
designation of a trustee.

Here, the amendment meets the requirements of both the Uniform Trust Code and the common law. Thus,
the fact that Settlor’s signature was not witnessed when she signed the amendment to the trust does not
make the amendment invalid.

Explanation to Point-Two (20%):

Under the Uniform Testamentary Additions to Trusts Act, a revocable trust may be amended at any time
prior to the settlor’s death, and the amendment applies to probate assets poured into the trust at the
settlor’s death pursuant to the settlor’s will even when the will was executed prior to the date of the
amendment.

Historically, property owned by an individual at her death passed to the individual’s heirs or to
beneficiaries designated in a will executed with the formalities (writing, signing, witnessing) prescribed
by state law. However, when a will devises property to the trustee of an inter vivos trust, then the
provisions of the trust – which may not have been executed in accordance with the formalities required
for wills – effectively determine who will receive the property. Because of this possibility, some early
cases held that if an inter vivos trust was not executed with the same formalities required for a valid will,
then the trust was ineffective to dispose of probate assets poured into the trust at the settlor’s death
pursuant to the settlor’s will.

Seperac-J19 Exam-Released MEE Essay Compilation © 2016-2020 1170


This line of cases has been overturned by the Uniform Testamentary Additions to Trusts Act (the Act),
now Uniform Probate Code § 2-511. Under the Act, adopted in almost all jurisdictions, a testamentary
bequest to the trustee of an inter vivos trust established by the testator during his or her lifetime is valid if
the trust is in writing, it is identified in the testator’s will, and the trust instrument was executed before,
concurrently with, or after the execution of the will. The Act further specifies that such a bequest is valid
even if the trust is amendable or revocable and that a later amendment applies to assets passing to the trust
by a previously executed will.

Thus, because the trust amendment is valid, its terms apply to assets received by Bank from Settlor’s
estate.

Explanation to Point-Three (10%):

If the trust amendment was valid, then the trust assets, including assets passing to the trust under Settlor’s
will, should go to University.

Under the trust amendment, all trust assets (including the assets of Settlor’s probate estate poured into the
trust) pass to University. The facts provide no basis for failing to comply with Settlor’s stated intentions.

Explanation to Point-Four (30%):

If the trust amendment was invalid, trust assets, including assets received pursuant to Settlor’s will,
should be held in accordance with the terms of the original trust instrument because those terms do not
violate the Rule Against Perpetuities.

Under the dispositive terms of the original trust instrument, Settlor created successive income interests in
her surviving children and grandchildren with a remainder interest in her great-grandchildren. Because the
trust was revocable, the period during which the common law Rule Against Perpetuities requires that
interests vest (i.e., 21 years plus lives in being) began to run from the date Settlor no longer had a power
of revocation (here, her death), not the date on which the trust was created.

Under the common law Rule Against Perpetuities, Settlor’s trust is thus valid. At the time of Settlor’s
death, she was survived by no children, one granddaughter, and no great-grandchildren. Because Settlor
cannot have more children after her death, the only income beneficiary of the trust is Settlor’s surviving
granddaughter. This granddaughter is the only person who can produce great-grandchildren of Settlor;
thus, all great-grandchildren must, of necessity, be born during the lifetime of Settlor’s only surviving
granddaughter, who is a life in being. The granddaughter’s interest vested at Settlor’s death, and the great-
grandchildren’s interest will vest at the death of the granddaughter. There is no need to wait the additional
21 years permitted under the Rule. Thus, under the common law and the statute given in the facts, the
nonvested interest in the great-grandchildren is valid.

[NOTE: Both modern wait-and-see statutes and the Uniform Statutory Rule Against Perpetuities upon
which the statute in the facts is modeled provide that before using either reform to validate an otherwise
invalid nonvested interest, one should first determine if the nonvested interest violates the common law
Rule. If it does not, then there is no need to reform. This proposition, which is applicable in all
jurisdictions that have not simply abrogated the rule, is tested by this problem.]

Seperac-J19 Exam-Released MEE Essay Compilation © 2016-2020 1171


#273-JUL 2012–MEE Q01: QUESTION ONE (TRUSTS)

Thirty years ago, Settlor entered into an irrevocable trust agreement with Trustee. Pursuant to the terms of
this trust, all trust income was payable to Settlor’s Husband, and upon Husband’s death, all trust assets
were to be distributed to “Settlor’s children.” The trust also provided that Husband’s income interest
would terminate if Husband remarried after Settlor’s death.

When the trust was created, Settlor and Husband had three children. Five years later, Settlor and Husband
had a fourth child.

Ten years later, Settlor died.

This year, when the trust principal was worth $750,000, Husband wrote to his four children. Husband
noted that he was about to retire and wanted cash to buy a retirement home. He asked the children to
agree to terminate the trust and to direct Trustee to distribute $250,000 of trust principal to Husband and
the remaining $500,000, in equal shares, to the four children. All four children agreed to Husband’s
proposal. Husband and the four children then wrote Trustee the following letter:

We, the only beneficiaries of the trust, direct you to terminate the trust and distribute $250,000 of trust
assets to Husband and the remainder, in equal shares, to Settlor’s four children.

Trustee’s response stated:

I cannot make the requested distribution to you for the following reasons:

(1) The trust is irrevocable and cannot be terminated.

(2) Even if the trust were terminable, termination would require the consent of all beneficiaries. This is
not obtainable because, if a child of Settlor predeceases Husband, one or more of Settlor’s future
grandchildren might be entitled to trust assets at Husband’s death.

(3) Even if the trust were terminable, only the three children living when the trust was created have a
beneficial interest in the trust; therefore no distribution of trust principal can be made to Settlor’s youngest
child.

(4) The actuarial value of Husband’s interest is only $150,000. Therefore, even if the trust were
terminable, any distribution of trust principal to Husband in excess of that amount would be a breach of
trust.

Is Trustee correct? Explain.

Seperac-J19 Exam-Released MEE Essay Compilation © 2016-2020 1172


#273: J12-1 MEE: ANSWER: NCBE (TRUSTS)

POINT (1) [35%] ISSUE: Is there a material purpose of the trust yet to be performed? ANSWER:
Yes. Husband and Settlor’s four children may terminate the irrevocable trust if a court finds that
there is no material purpose of the trust yet to be performed. However, if a court were to find that
the limitation on remarriage was a material purpose, the trust could not be terminated merely with
the consent of Husband and the four children.

POINT (2) [30%] ISSUE: Do Settlor’s children have the only remainder interest in the trust?
ANSWER: Yes. Under the common law, Settlor’s children are the only trust remaindermen.
Because each child’s interest is vested and transmissible, it would pass to the child’s estate, not the
child’s issue, should the child predecease Husband. Thus, the four children and Husband are the
only beneficiaries of the trust and could consent to terminate the trust. However, under UPC § 2-
707 alternative remainder interests are created in the issue of any of Settlor’s children who
predecease Husband.

POINT (3) [25%] ISSUE: Was the class gift in favor of Settlor’s children open to admit Settlor’s
fourth child? ANSWER: Yes. Settlor’s youngest child is entitled to a share of the trust because that
child was born and became a class member before the class closed.

POINT (4) [10%] ISSUE: Would Trustee breach any fiduciary duties by terminating the trust as
requested? ANSWER: No. While Trustee has a duty to carry out the terms of the trust, when the
trust beneficiaries properly terminate the trust, they can direct that the trust property be
distributed in a manner of their own choosing, and a distribution consistent with the beneficiaries’
direction is not a breach of trust.

ANSWER DISCUSSION:

Husband and Settlor’s four children may terminate the irrevocable trust if a court finds that they are the
only trust beneficiaries and there is no material purpose of the trust yet to be performed. It is unclear
whether Settlor’s desire to terminate Husband’s interest if Husband remarries would be viewed as a
material purpose of the trust yet to be performed. Under the common law, only Husband and Settlor’s
children have an interest in the trust, as the children’s interest is not subject to any conditions. The
children’s interest is both vested and transmissible. However, if the jurisdiction has adopted a
survivorship statute like Uniform Probate Code (UPC) § 2-707, then the children and Husband would not
be the only trust beneficiaries. Under that statute, if a child dies before Husband leaving surviving issue,
the issue would also have an interest in the trust. Because Settlor’s class gift to “Settlor’s children”
remained open until Settlor’s death, all four children are entitled to share in the trust remainder. The
proposed distribution of principal, which would give Husband more than the actuarial value of Husband’s
interest, would not breach any fiduciary duty because it would be made at the request of all trust
beneficiaries. Thus, unless UPC § 2-707 or a like statute applies or there is a material purpose of the trust
yet to be performed, Trustee’s assertions are incorrect.

ANSWER EXPLANATION:

Explanation to Point-One (35%):

Seperac-J19 Exam-Released MEE Essay Compilation © 2016-2020 1173


Husband and Settlor’s four children may terminate the irrevocable trust if a court finds that there is no
material purpose of the trust yet to be performed. However, if a court were to find that the limitation on
remarriage was a material purpose, the trust could not be terminated merely with the consent of Husband
and the four children.

Generally, even an irrevocable trust can be terminated prior to the death of all income beneficiaries if both
the income beneficiaries and the remaindermen unanimously consent. However, if there is a material
purpose of the trust yet to be performed, the beneficiaries alone may not terminate the trust.

Here, it is unclear whether there is a material purpose yet to be performed. According to the Restatement,
a material purpose should not be inferred from “the mere fact that the settlor created a trust for successive
beneficiaries. In the absence of additional circumstances indicating a further purpose, the inference is that
the trust was intended merely to allow one or more persons to enjoy the benefits of the property during the
period of the trust and to allow the other beneficiaries to receive the property thereafter.” However, the
trust provision specifying that Husband’s interest terminates upon his remarriage arguably demonstrates
that a material purpose of Settlor was ensuring that trust assets did not benefit Husband’s second wife. If
Settlor did have such a material purpose, the proposed distribution of $250,000 to Husband could defeat
that purpose should Husband remarry.

If the court were to find that Settlor had a material purpose that would be defeated by trust termination, it
is unclear whether a court would permit termination of the trust. Under the First and Second Restatements
of Trusts, “if the continuance of the trust is necessary to carry out a material purpose of the trust, the
beneficiaries cannot compel its termination.” Similarly, the Uniform Trust Code § 411 provides that “a
noncharitable irrevocable trust may be terminated upon consent of all of the beneficiaries only if the court
concludes that continuance of the trust is not necessary to achieve any material purpose of the trust.”

However, under the Third Restatement of Trusts, which no court to date has followed, even if the court
finds that the settlor had a material purpose that trust termination might defeat, it may nonetheless
approve trust termination if “the reason for termination or modification outweighs the material purpose.”

If the court were to employ the balancing approach approved by the Third Restatement, it is unclear what
decision it would reach on the available facts. The reason for trust termination is to provide Husband with
cash for his impending retirement. Without knowing more about Husband’s finances and the income
available from the trust, it is impossible to assess how Husband’s goal compares to Settlor’s goal of
ensuring that no future spouse of Husband receives benefits traceable to trust assets.

[NOTE: While conditioning the continuation of a child’s income interest on the child’s marriage or
divorce is void on public policy grounds, a similar marriage restriction tied to the surviving spouse’s
interest is not.]

[NOTE: The Uniform Trust Code has been adopted in 24 states.]

Explanation to Point-Two (30%):

Under the common law, Settlor’s children are the only trust remaindermen. Because each child’s interest
is vested and transmissible, it would pass to the child’s estate, not the child’s issue, should the child
predecease Husband. Thus, the four children and Husband are the only beneficiaries of the trust and could
consent to terminate the trust. However, under UPC § 2-707 alternative remainder interests are created in
the issue of any of Settlor’s children who predecease Husband.

Seperac-J19 Exam-Released MEE Essay Compilation © 2016-2020 1174


A class gift is a gift to a group of persons described collectively, typically by their relationship to a
common ancestor. Here, the gift to “Settlor’s children” is a class gift.

Under the common law, when a class gift is made to a group who are equally related to a common
ancestor and the gift is not expressly subject to a condition of survivorship, the gift is not impliedly
subject to such a condition. If a member of such a class fails to survive until the time of distribution (here,
Husband’s death), that member’s share passes to his or her estate, not to his or her issue.

Here, the remainder interest in the trust is limited to a class of “Settlor’s children.” All class members are
equally related to a common ancestor, Settlor, and the trust imposes no express condition of survivorship.
Therefore, each child has a vested and transmissible interest that would pass to his or her estate should the
child die before the distribution date. If the common law applies, then only Husband and the four children
are the beneficiaries of the trust and, if there is no material purpose to be performed, they could consent to
a termination of the trust.

Critics of the common law approach have argued that it fails to carry out the intentions of the typical trust
settlor, permits a deceased beneficiary to transmit her share of trust assets to persons who are “strangers”
to the settlor, and increases administrative costs by requiring the reopening of deceased beneficiaries’
estates and possibly successive estates. In response to these criticisms, some modern statutes do not
follow the common law approach. For example, UPC § 2-707 provides that when a beneficiary does not
survive to the distribution date, the beneficiary’s interest passes to his or her issue unless the trust
instrument specifies an alternate disposition. Here, under the Uniform Probate Code approach, issue of a
deceased child would have beneficial interest in the trust and would have to be part of any termination
effort unless their interest could be represented by another.

Section 304 of the Uniform Trust Code provides: “Unless otherwise represented, a minor, incapacitated,
or unborn individual, may be represented by and bound by another having a substantially identical interest
with respect to the particular question or dispute, but only to the extent there is no conflict of interest
between the representative and the person represented.” This statute permits one group of remaindermen
to represent another group of remaindermen in the absence of a conflict of interest. Here, the four children
and their issue would have a conflict in light of the proposed distribution that would exclude the issue
from potentially sharing in any trust assets. Thus, under UPC § 2-707 or a like statute, the trust could not
be terminated merely with the consent of Husband and the four children.

[NOTE: Section 2-707 of the Uniform Probate Code has been adopted in 9 states.]

Explanation to Point-Three (25%):

Settlor’s youngest child is entitled to a share of the trust because that child was born and became a class
member before the class closed.

As noted under Point Two, the gift to “Settlor’s children” was a class gift. When a class gift is made to a
group of individuals, such as the children of a named person, the class does not “close” (that is, additional
individuals may join the class by satisfying the class eligibility requirements) until the named person dies
or the gift becomes possessory.

Here, Settlor’s fourth child was born prior to Settlor’s death and therefore joined the class before it closed.
Thus, this child is entitled to a share of the trust property.

Explanation to Point-Four (10%):

Seperac-J19 Exam-Released MEE Essay Compilation © 2016-2020 1175


While Trustee has a duty to carry out the terms of the trust, when the trust beneficiaries properly terminate
the trust, they can direct that the trust property be distributed in a manner of their own choosing, and a
distribution consistent with the beneficiaries’ direction is not a breach of trust.

If trust beneficiaries properly terminate a trust, trust assets vest in them. After termination, the
beneficiaries may themselves distribute trust assets in any manner they choose. They may also direct a
trustee to distribute trust assets as their agent. A trustee who complies with such directions does not
violate any fiduciary duty.

Here, assuming that the trust beneficiaries may terminate the trust, Trustee may comply with their
distribution instructions. The fact that Husband would receive more than the actuarial value of his life
estate does not, under these circumstances, constitute a breach of trust. Moreover, even if Trustee were
breaching a fiduciary obligation in complying with the children’s directions, they would be deemed to
waive any claims against Trustee because they ordered the distribution.

Seperac-J19 Exam-Released MEE Essay Compilation © 2016-2020 1176


#274-JUL 2011–MEE Q03: QUESTION THREE (TRUSTS)

Forty years ago, Testator executed a valid will under which he devised his home to the trustee of a trust.
Testator’s will directed the trustee to retain the home to ensure that my Daughter has a comfortable
residence throughout her life. The home shall not be sold until Daughter dies. After Daughter’s death, I
direct the trustee to sell the home and to distribute the sale proceeds to Charity, a charitable corporation
organized to end homelessness in Capital City.

Thirty years ago, Testator died. At the time of his death, Testator still owned the home, which was located
in Capital City in a quiet residential neighborhood of single-family homes near both a local college and
the city’s business district. The business district was commercially successful, but it had attracted a large
number of homeless people.

During the last 30 years, the character of the neighborhood where the home is located has changed
dramatically. Many apartment buildings have been built, greatly increasing population density and noise.
Several bars and restaurants catering to college students have also opened in the formerly residential
blocks near the home. Stores in the city’s business district have moved to suburban shopping malls, and
the vacated buildings have been converted to bars and dance clubs. This shift has increased public
rowdiness, but it has also been associated with a marked decline in the number of homeless individuals in
the business district.

Daughter recently decided that she wants to move from the home to a rental apartment in a quieter and
less congested neighborhood. Daughter consulted a real estate agent, who correctly told her that the home
is worth about $300,000 and will easily sell for that amount.

Daughter asked the trustee to sell the home, to hold the expected sale proceeds of $300,000 in trust, and to
use the proceeds and the income to pay Daughter’s rent, which will be about $2,000 per month. The
monthly income from the $300,000 trust, however, is expected to be only $1,000.

When the trustee attempted to contact Charity to discuss Daughter’s request, he discovered that Charity
no longer exists.

The trustee has consulted the law firm where you work for advice on these questions:

1. Can the terms of Testator’s testamentary trust be reformed to permit the sale of the home?
Explain.

2. Assuming that the trust can be reformed to permit the sale of the home, can the trustee also
obtain authorization to use the sale proceeds and the earnings thereon to pay Daughter’s rent?
Explain.

3. After Daughter’s death, will the trust assets pass to Testator’s estate? Explain.

Seperac-J19 Exam-Released MEE Essay Compilation © 2016-2020 1177


#274: J11-3 MEE: ANSWER: NCBE (TRUSTS)

POINT (1) [30%] ISSUE: May a court reform the terms of Testator’s testamentary trust to permit
the sale of the home? ANSWER: Yes. Under both the common law equitable-deviation doctrine
and the Uniform Trust Code, a court could reform the provisions of the trust to authorize the
trustee to sell the home instead of retaining it.

POINT (2) [40%] ISSUE: May a court reform the terms of Testator’s testamentary trust to
authorize the use of both the home’s sale proceeds and the earnings on those proceeds to pay
Daughter $2,000 monthly for her expected rent? ANSWER: Yes. Under modern statutes, but not
the common law, a court may alter a dispositive provision of a trust to further trust purposes. It is
unclear whether a court would find that using the proceeds of the home’s sale to fund Daughter’s
rent would further trust purposes.

POINT (3) [30%] ISSUE: Will the trust property pass to Testator’s estate upon Daughter’s death?
ANSWER: No. Upon Daughter’s death, the trust property likely will not pass to Testator’s estate
because of the cy pres doctrine.

ANSWER DISCUSSION:

Under the “equitable deviation” doctrine, a court may modify an administrative provision of a trust based
on unanticipated changed circumstances. Here, the trust’s “no-sale” provision is administrative, and there
is a strong argument that the change in the neighborhood around the home represents a circumstance not
anticipated by Testator. Under the common law, courts may not change the dispositive provisions of a
trust. The trust provisions regarding disposition of the home’s sale proceeds are dispositive. Thus, if the
home is sold, a court may not alter the relative interests of Daughter and Charity in the sale proceeds.
However, under Uniform Trust Code § 412 and like statutes, a court may modify the “dispositive terms of
a trust if, because of circumstances not anticipated by the settlor, modification will further the purposes of
the trust.” It is unclear whether a court would reform the trust in the manner Daughter seeks under this
provision. If a charitable beneficiary no longer exists when a trust terminates, trust property reverts to the
trust settlor or his or her estate unless the court, using its cy pres power, substitutes another charity for the
nonexistent charity. Here, Testator’s intentions are unclear. However, because there is a presumption of
general charitable intent, the court would likely exercise its cy pres power. Under the Uniform Trust
Code, which establishes a conclusive presumption of general charitable intent, the court would certainly
exercise its cy pres power and substitute a different charity. Thus, the trust property would not revert to
Testator’s estate.

ANSWER EXPLANATION:

Explanation to Point-One (30%):

Under both the common law equitable-deviation doctrine and the Uniform Trust Code, a court could
reform the provisions of the trust to authorize the trustee to sell the home instead of retaining it.

Under the common law, a court may order “equitable deviation” from the terms of a trust when an
unanticipated change in circumstances would otherwise “defeat or substantially impair the
accomplishment of the purposes of the trust.”

Seperac-J19 Exam-Released MEE Essay Compilation © 2016-2020 1178


The Uniform Trust Code recognizes and expands the equitable-deviation doctrine. Under the Code, an
administrative provision of a trust may be modified “if, because of circumstances not anticipated by the
settlor, modification or termination will further the purposes of the trust.” An administrative provision of
a trust is one relating to the management of trust property instead of the allocation of benefits among trust
beneficiaries. An unanticipated change in the character of the community where realty held by a trust is
located represents the sort of change to which the equitable-deviation doctrine is applicable.

Here, the facts support application of the equitable-deviation doctrine. The trust provision in question is
administrative: it directs the trustee to retain the home and not to sell it. The facts also show that the
neighborhood in which the home is located has changed substantially and in a way that Testator did not
anticipate. The trust instrument’s directive that the trustee retain the home until Daughter dies seems to
have been based on the view that the home would be in a desirable residential neighborhood. Instead, the
neighborhood is now dominated by apartment houses and noisy commercial establishments. Thus, the
equitable-deviation doctrine should permit reformation of the trust.

The Uniform Trust Code also provides that, even if circumstances have not changed in an unanticipated
manner, an administrative provision may be modified if “continuation of the trust on its existing terms
would be impracticable or wasteful or impair the trust’s administration.” This expansion rests on the view
that “a policy of rigid adherence to the letter of the donative instrument is likely to frustrate both the
donor’s purposes and the efficient use of resources.”

Here, there is a strong argument that retention of the home would impair the trust goal of providing
Daughter with adequate and desirable housing. Thus, reformation of the trust should also be permitted
under Uniform Trust Code § 412.

[NOTE: Because the request to sell the home affects an administrative provision of the trust, the
permission of the trust beneficiaries and/or trustee is not required to reform the trust. “Lack of consent by
a beneficiary is no bar if a modification will have no effect on the non-consenting beneficiary.” ]

Explanation to Point-Two (40%):

Under modern statutes, but not the common law, a court may alter a dispositive provision of a trust to
further trust purposes. It is unclear whether a court would find that using the proceeds of the home’s sale
to fund Daughter’s rent would further trust purposes.

Under the common law, the equitable-deviation doctrine applies only to administrative provisions of a
trust. Courts are not empowered to alter dispositive provisions that determine the allocation of trust assets
and income among trust beneficiaries.

Here, although the trust provision directing that the home be retained until Daughter’s death is
administrative, the provisions directing that Daughter has the right to live in the home until her death and
that the home sale proceeds go to Charity when Daughter dies are dispositive; effectively, these
provisions grant Daughter an entitlement to trust income (but not trust principal) and grant Charity the
proceeds from the home’s sale. Thus, under the common law, a court may not alter the beneficial rights of
Daughter and Charity in the proceeds obtained from selling the home.

However, under Uniform Trust Code § 412 and like statutes, a court may modify the “dispositive terms of
a trust if, because of circumstances not anticipated by the settlor, modification will further the purposes of
the trust. To the extent practicable, the modification must be made in accordance with the settlor’s
probable intention.”

Seperac-J19 Exam-Released MEE Essay Compilation © 2016-2020 1179


Here, it is unclear whether a court would find that trust modification would further the purposes of the
trust. There is an argument that Testator intended to provide Daughter with a desirable residence during
her lifetime and, although the home was such a residence during Testator’s lifetime, an unanticipated
change of circumstances has altered its desirability. Thus, a court could find that modification would
further Testator’s purposes, even if use of the proceeds would ultimately reduce the value of Charity’s
remainder interest.

There is an opposing argument, perhaps even stronger. Daughter has only a life interest in the home.
However, her proposed modification would likely result in the complete loss of the remainder interest.
Daughter proposes that the trust expend $24,000 annually from an estimated $300,000 trust. A payout of
this size would likely cause a depletion of the entire trust corpus in due course and deprive the
remainderman of its interest. In fact, if Daughter lives long enough, that payout would exhaust the trust
principal, leaving no money to pay her either. On the other hand, the trustee could be authorized to
purchase a new residence with the proceeds from the home’s sale. This approach would both provide
Daughter with a desirable place to live for the rest of her life, as Testator intended, and still preserve the
remainder interest.

[NOTE: Extra points could be awarded to the examinee who notes that to the extent the remainder
interest fails and the property reverts to Testator’s estate, the argument in favor of Daughter is
strengthened if she is the sole beneficiary of Testator’s estate.]

Explanation to Point-Three (30%):

Upon Daughter’s death, the trust property likely will not pass to Testator’s estate because of the cy pres
doctrine.

Under the common law, when trust property to be used for a charitable purpose could not be distributed as
directed in the trust instrument, the trust did not necessarily fail. Instead, the court determined whether to
exercise its cy pres power and redirect the charitable gift to another like charity.

The common law cy pres doctrine requires an initial inquiry into the settlor’s intent: If the court
determines that the settlor had a specific charitable intention limited to the charitable purpose stated in the
trust instrument, the property reverts to the settlor or the settlor’s estate. If the court determines that the
settlor had a general charitable intention, it substitutes for the named charity another one with activities
consistent with the settlor’s intentions. The common law and the Third Restatement of Trusts both
presume that a settlor has a general charitable intent.

Here, there is little evidence regarding Testator’s intentions. Testator may have wanted to help the
homeless generally; Testator may also have wanted to address homelessness in Capital City’s business
district specifically. There is no evidence that Testator viewed Charity as the sole means to either of these
goals; nor is there evidence to the contrary. Therefore, the presumption of general charitable intent would
likely prevail.

The Uniform Trust Code appears to establish a conclusive presumption of general charitable intention.
Section 413 of the Code provides that “if a particular charitable purpose becomes unlawful, impracticable,
impossible to achieve, or wasteful: (1) the trust does not fail, in whole or in part; (2) the trust property
does not revert to the settlor or the settlor’s successors in interest; and (3) the court may apply cy pres to
modify the trust by directing that the trust property be applied or distributed, in whole or in part, in a
manner consistent with the settlor’s charitable purposes.” Thus, under § 413, the court should apply the cy

Seperac-J19 Exam-Released MEE Essay Compilation © 2016-2020 1180


pres doctrine and substitute a different charity for Charity. The trust assets would not revert to Testator’s
estate.

[NOTE: Twenty-five states have adopted the Uniform Trust Code.]

Seperac-J19 Exam-Released MEE Essay Compilation © 2016-2020 1181


#275-FEB 2011–MEE Q01: QUESTION ONE (TRUSTS)

In 1994, Testator died, survived by her sons, Ron and Sam, and Ron’s son, Peter. Under Testator’s duly
probated will, Testator created a trust and provided that Trustee shall distribute to Ron, for the duration of
his life, as much of the income and principal from the trust as Trustee, in her uncontrolled discretion,
deems advisable. At Ron’s death, any remaining trust funds shall be distributed to my grandchildren, with
the children of any deceased grandchild taking the deceased parent’s share.

In another section of her will, Testator bequeathed property to her son Sam.

In 2006, Ron adopted Carol, who was then 14 years old.

On January 2, 2008, Ron borrowed $50,000 from a friend and gave the friend a note for that amount
payable in five annual installments of $10,000 plus interest.

Between 1994 and 2009, Trustee distributed all the trust income (which averaged $15,000 per year) to
Ron.

In 2009, Peter, Ron’s son, died in an automobile accident survived by his wife, Ginny, but by no
descendants. Under Peter’s valid will, his entire estate passed to Ginny.

On January 2, 2010, Ron failed to make the required payment to his friend on the $50,000 note. Ron’s
friend then demanded that Trustee distribute that year’s trust income to him instead of to Ron until the
$10,000 plus interest due that year was fully paid. At that point, Trustee stopped paying trust income to
Ron and refused to make payments to Ron’s friend. Instead, Trustee began to accumulate trust income.

In late 2010, Ron died. He was survived by his adopted daughter, Carol; his daughter-in-law, Ginny; his
brother, Sam; and Sam’s wife. Sam has no children, but he and his wife plan to have a child in the near
future.

1. Was Ron’s friend entitled to any of the trust income earned during 2010? Explain.

2. At Ron’s death, to whom should the trust principal be distributed? Explain.

Seperac-J19 Exam-Released MEE Essay Compilation © 2016-2020 1182


#275: F11-1 MEE: ANSWER: NCBE (TRUSTS)

POINT (1) [30%] ISSUE: Does a creditor of a beneficiary of a discretionary trust have the right to
compel the trustee to make payments to the creditor? ANSWER: No. Because the trust instrument
conferred discretion on Trustee, Trustee was not obligated to make payments to Ron or to Ron’s
creditor.

POINT (2) [25%] ISSUE: When an income beneficiary of a testamentary trust adopts a child after
the testator’s death, does the adopted child qualify as a “grandchild” for purposes of sharing in a
trust remainder? ANSWER: Yes. Carol, the adopted daughter of Ron, is entitled to a share of the
remainder allocated to “grandchildren.”

POINT (3) [20%] ISSUE: In a class gift to grandchildren following the death of a life tenant, does
the class close upon the life tenant’s death when there is a grandchild eligible to take, even though
more grandchildren could be born after that date? ANSWER: Yes. Because at least one grandchild
was entitled to take at the time of Ron’s death, the class of grandchildren closed at that time,
requiring distribution of the remainder at that time and excluding future grandchildren from
sharing in the remainder.

POINT (4) [25%] ISSUE: When a trust instrument creates a remainder in “grandchildren, with the
children of any deceased grandchild taking the deceased parent’s share,” does any interest vest in
the estate of a grandchild who dies without issue before the remainder becomes possessory?
ANSWER: Ginny, the beneficiary of Peter’s will, is entitled to share in the remainder to
grandchildren unless the jurisdiction has adopted a statute akin to Uniform Probate Code § 2-707.

ANSWER DISCUSSION:

A creditor of the beneficiary of a discretionary trust may not compel the trustee to make payments to the
creditor when the beneficiary cannot compel the trustee to make payments. Thus, Ron’s friend had no
claim against Trustee. Adoption statutes generally treat adopted children equally with biological children
for purposes of inheritance and taking as class gift members under a will. Thus Carol, Ron’s adopted
daughter, will be entitled to share in the remainder interest in the trust even though she was not adopted
until after Testator’s death, because the class of grandchildren was still open at that time. When a trust
instrument creates a future interest in grandchildren, the class of grandchildren closes when a grandchild
becomes entitled to take, even if that means excluding other potential (but yet unborn) grandchildren.
Here, grandchildren became entitled to take at the death of Testator’s son Ron, and the class closed at that
time, thus excluding any future-born grandchildren. When a trust creates a future interest in
grandchildren, with the children of any deceased grandchild taking the deceased parent’s share, the estate
of a grandchild who dies without issue before the time for distribution succeeds to the grandchild’s
interest unless the jurisdiction has adopted a statute akin to Uniform Probate Code § 2-707, which creates
a presumption that future interests under the terms of a trust are contingent on survivorship until the
distribution date. [NOTE: There is no perpetuity issue in this question because Testator had a living
grandchild when she died (Peter) and thus the remainder interest is guaranteed to vest no later than the
death of Ron, who is a life in being.]

ANSWER EXPLANATION:

Explanation to Point-One (30%):

Seperac-J19 Exam-Released MEE Essay Compilation © 2016-2020 1183


Because the trust instrument conferred discretion on Trustee, Trustee was not obligated to make payments
to Ron or to Ron’s creditor.

The language of the trust instrument conferred uncontrolled discretion on Trustee to make payments to, or
withhold payments from, Ron. When, as here, a trustee’s discretion is uncontrolled, there is no abuse of
discretion if the trustee acts “honestly and in a state of mind contemplated by” the trust creator. As a
result, Ron would be able to compel payments only if Ron could demonstrate that Trustee had acted
dishonestly or in a state of mind not contemplated by the trust’s creator. There are no facts here to support
such a finding.

Because Ron’s creditor could have no greater rights in the trust than Ron had, the creditor, Ron’s friend,
could not compel Trustee to make payments to him. If the beneficiary himself cannot compel the trustee
to pay over any part of the trust fund, his creditors are in no better position. Moreover, even in the
unlikely event that the friend could establish that Trustee’s failure to make payments constituted an abuse
of discretion, some modern authorities hold that a creditor cannot assert the beneficiary’s right to
complain of abuse of discretion. The Uniform Trust Code provides that a creditor may not compel the
trustee of a discretionary trust to make a distribution, even when failure to make the distribution would
constitute an abuse of discretion.

[NOTE: This problem does not involve the law relating to the ability of creditors to come after a
beneficiary’s interest when the instrument includes a spendthrift clause, as no such clause appears in the
instrument here.]

[NOTE: Twenty-five states have adopted the Uniform Trust Code.]

Explanation to Point-Two (25%):

Carol, the adopted daughter of Ron, is entitled to a share of the remainder allocated to “grandchildren.”

Section 2-705 of the Uniform Probate Code provides that “adopted individuals are included in class gifts
in accordance with the rules for intestate succession.” Section 2-114 of the Uniform Probate Code, which
deals with intestate succession, provides that “an adopted individual is the child of his or her adopting
parent or parents.” Thus, for purposes of the trust, Carol is treated equally with a child born to Ron after
Testator’s death. In construing a class gift created by someone other than the adoptive parent, the adopted
child is treated as a child of the adoptive parent, but only if the adoption took place before the child
reached the age of majority.

Because it is clear that Carol should be treated as though she were Ron’s biological child, the question is
whether she qualifies to take the remainder even though she was not yet adopted at Testator’s death. The
answer is yes. The class of grandchildren closes when one person becomes entitled to possession of the
remainder interest. Because no person became entitled to a share of the trust principal/remainder interest
until Ron’s death, Carol is entitled to share because she was adopted before that time.

Explanation to Point-Three (20%):

Because at least one grandchild was entitled to take at the time of Ron’s death, the class of grandchildren
closed at that time, requiring distribution of the remainder at that time and excluding future grandchildren
from sharing in the remainder.

Seperac-J19 Exam-Released MEE Essay Compilation © 2016-2020 1184


A class gift closes to future entrants when at least one class member is entitled to distribution. In this case,
because grandchildren were entitled to take at Ron’s death, the class closed, and any children
subsequently born to Sam were excluded from sharing in the remainder.

Explanation to Point-Four (25%):

Ginny, the beneficiary of Peter’s will, is entitled to share in the remainder to grandchildren unless the
jurisdiction has adopted a statute akin to Uniform Probate Code § 2-707.

When a trust creates a vested remainder in a person or member of a class and then provides that the
remainder should pass to that person’s child if the remainder person predeceases the life tenant, the
remainder is divested only if the deceased remainder person has a child. If, on the other hand, the
remainder person dies childless, the remainder is not divested and passes to the remainder person’s estate.
Here, Peter’s remainder would have been divested in favor of his child if he had been survived by a child,
but because he was not survived by a child, the remainder vested in his estate. And, because Peter left his
entire estate to his wife, Ginny, she succeeds to Peter’s interest in the remainder.

The result would be different if the jurisdiction has adopted § 2-707 of the Uniform Probate Code or a
similar statute. Section 2-707(b), a controversial provision, makes future interests under the terms of a
trust contingent on the beneficiary’s surviving to the distribution date. Under UPC § 2-707(b), then,
Peter’s remainder was contingent on surviving Ron. Because Peter did not survive Ron, his interest did
not vest. As a result, he had no interest to pass to Ginny.

Hence, at common law, Carol and Ginny would share the remainder equally; if UPC § 2-707 or a similar
statute is in force, only Carol would succeed to the entire remainder.

[NOTE: Nine states have enacted a form of Uniform Probate Code § 2-707.]

Seperac-J19 Exam-Released MEE Essay Compilation © 2016-2020 1185


#276-FEB 2010–MEE Q08: QUESTION EIGHT (TRUSTS)

Settlor created a revocable trust naming Bank as trustee. The trust instrument directed Bank, as trustee, to
pay all trust income to Settlor and, upon Settlor’s death, to distribute all trust assets to “Settlor’s surviving
children.” When Settlor created the trust, he had three living children, Alan, Ben, and Claire.

Settlor died last year. Alan predeceased him. Settlor was survived by three children, Ben, Claire, and
Doris (born after Settlor created the trust), and two grandchildren. One of the surviving grandchildren was
Claire’s child and one was Alan’s child. Alan’s child was his only heir.

When Settlor created the trust, he funded it with cash. Bank promptly invested the cash in a broad range
of stocks and bonds and held this broadly diversified portfolio for just over twenty years. Although the
portfolio had by then significantly increased in value, Settlor was dissatisfied with the rate of
appreciation. Settlor therefore directed Bank to sell 90% of the trust portfolio and to reinvest the proceeds
in the stock of XYZ, a closely held corporation that Settlor believed would substantially appreciate in
value.

The investment in XYZ appreciated more than 50% during the first two years after Bank purchased the
stock. However, during the five years preceding Settlor’s death, the XYZ investment depreciated to about
70% of its initial value. This depreciation was largely due to mismanagement by XYZ’s board of
directors. Although Settlor was neither a director nor an officer of XYZ, he was fully aware of the
management problems. He discussed these problems with Bank and told Bank, “I expect things will turn
around soon.”

Immediately upon Settlor’s death, Bank liquidated the trust’s interest in XYZ, thus avoiding further losses
from this investment.

One month after Settlor died, Claire wrote to Bank disclaiming all of her interest in the trust.

1. To whom should the trust assets be distributed? Explain.

2. Is Bank liable for losses on the investment in XYZ stock? Explain.

Seperac-J19 Exam-Released MEE Essay Compilation © 2016-2020 1186


#276: F10-8 MEE: ANSWER: NCBE (TRUSTS)

POINT (1) [20%] ISSUE: Did the class of “Settlor’s surviving children” remain open following the
creation of the trust so that Doris shares in the gift? ANSWER: Yes. Doris is entitled to share in the
trust remainder because the class of “Settlor’s surviving children” did not close until Settlor’s
death.

POINT (2) [20%] ISSUE: Does the share of Alan, who predeceased Settlor, pass to Alan’s child or
Settlor’s surviving children? ANSWER: The share that would have passed to Alan had he survived
Settlor passes to Ben and Doris, the surviving members of the class, unless the state has adopted a
survivorship rule like Uniform Probate Code § 2-707. If the Uniform Probate Code applies, Alan’s
share passes to his child.

POINT (3) [30%] ISSUE: Did Claire effectively disclaim her remainder interest in the trust, and, if
so, to whom does her disclaimed interest pass? ANSWER: Claire effectively disclaimed her interest
in the trust. Under most state laws, her interest goes to those persons to whom it would have passed
had she predeceased Settlor. In this case, the interest would pass to Ben and Doris, the remaining
members of the class of “Settlor’s surviving children.” However, in states that have adopted a
survivorship rule like Uniform Probate Code § 2-707, Claire’s interest would pass to her child.

POINT (4) [30%] ISSUE: Did Bank breach its obligation to invest trust assets prudently by
investing, at Settlor’s direction and acquiescence, 90% of the portfolio in the stock of a closely held
corporation and by maintaining this investment despite the stock’s depreciation? ANSWER: Bank
may have breached its duties to invest prudently and diversify trust investments. However, whether
Bank is liable for these breaches depends upon whether acting pursuant to the directions of the
settlor of a revocable trust excuses Bank from its duties to diversify and act prudently.

ANSWER DISCUSSION:

Doris is eligible to take a share of trust assets because the class of “Settlor’s surviving children” did not
close until Settlor’s death. In all states, Ben and Doris would receive a share of the assets as surviving
children of Settlor. Alan, who predeceased Settlor, would lose his share as would Claire, because her
disclaimer is effective and precludes her from sharing in the estate. In most states Ben and Doris would
each take one-half of the trust assets because Claire is deemed to have predeceased Settlor because of the
disclaimer. However, in states that have adopted Uniform Probate Code § 2-707 or a like statute, trust
assets would be divided into four equal shares, and the children of Alan and Claire would take, by
representation, the shares their respective parents would have received. Because Settlor directed the
investment in XYZ and acquiesced in its retention by Bank, Bank is probably not liable for losses on that
investment.

ANSWER EXPLANATION:

Explanation to Point-One (20%):

Doris is entitled to share in the trust remainder because the class of “Settlor’s surviving children” did not
close until Settlor’s death.

Seperac-J19 Exam-Released MEE Essay Compilation © 2016-2020 1187


If a gift is made to a class of persons, such as a named person’s children, the class closes (i.e., additional
persons may no longer join the class) when the named person dies or the gift becomes possessory.

Here, Settlor’s death closed the class of Settlor’s “surviving children” under either test. Thus, all of
Settlor’s children, including Doris, who was born before the class closed, are eligible to share in the gift.

Explanation to Point-Two (20%):

The share that would have passed to Alan had he survived Settlor passes to Ben and Doris, the surviving
members of the class, unless the state has adopted a survivorship rule like Uniform Probate Code § 2-707.
If the Uniform Probate Code applies, Alan’s share passes to his child.

When a remainderman (such as Alan) predeceases the life tenant, the trust assets are distributed based on
the directives contained in the trust instrument. Here the assets pass to Settlor’s other “surviving children”
(i.e., Ben, Claire, and Doris) as the trust instrument specifically provided that, upon Settlor’s death, the
trust property should be distributed to Settlor’s surviving children. (But see Point Three with respect to
Claire’s disclaimer of her interest.)

In states that have adopted Uniform Probate Code § 2-707 or a like statute, the outcome is different.
Under § 2-707, if a gift is made in trust to a class of persons described as “children,” a deceased child’s
descendants take the deceased child’s share by representation. The use of the word “surviving,” as here,
does not change that result. Thus, if the Uniform Probate Code or a like statute applies, Alan’s share
would pass to Alan’s child.

[NOTE: Nine states have enacted a form of Uniform Probate Code § 2-707.]

Explanation to Point-Three (30%):

Claire effectively disclaimed her interest in the trust. Under most state laws, her interest goes to those
persons to whom it would have passed had she predeceased Settlor. In this case, the interest would pass to
Ben and Doris, the remaining members of the class of “Settlor’s surviving children.” However, in states
that have adopted a survivorship rule like Uniform Probate Code § 2-707, Claire’s interest would pass to
her child.

Almost all states have enacted disclaimer statutes that permit beneficiaries of wills and trusts to disclaim
their interests in the estate or trust property. In most states, a disclaimer is not effective unless it is in
writing and is made, for a testamentary transfer, within nine months of the decedent’s death or, for a
future interest in a nontestamentary transfer, within nine months after the future interest would become
“indefeasibly vested.” Both requirements were met in this case, with the result that Claire’s disclaimer is
effective. Claire’s disclaimer would also be effective under the newest version of the Uniform Probate
Code, which permits a disclaimer at any time prior to acceptance of the interest.

When the holder of a future interest effectively disclaims that interest, the disclaimant is deemed to have
predeceased the life tenant. Thus, Claire is deemed to have predeceased Settlor. Under the common law,
Claire's disclaimed share would pass to Ben and Doris, the surviving class members. However, under § 2-
707 or a like statute, Claire’s child would take Claire’s share notwithstanding the express survivorship
contingency in the trust instrument.

[NOTE: Nine states have enacted a form of Uniform Probate Code § 2-707.]

Seperac-J19 Exam-Released MEE Essay Compilation © 2016-2020 1188


Explanation to Point-Four (30%):

Bank may have breached its duties to invest prudently and diversify trust investments. However, whether
Bank is liable for these breaches depends upon whether acting pursuant to the directions of the settlor of a
revocable trust excuses Bank from its duties to diversify and act prudently.

A trustee is under a duty to “invest and manage trust assets as a prudent investor would.” The obligation
to invest and manage prudently normally requires the trustee to diversify trust investments. A trustee is
also expected to consider the trust’s “needs for liquidity and preservation or appreciation of capital.”

If the trust here had been irrevocable, there is almost no doubt that Bank would be found to have breached
its duties. First, by investing 90% of the trust assets in one stock, Bank failed to diversify. Second, Bank
failed to preserve trust capital. Bank was aware that XYZ was declining in value and was suspicious of
mismanagement by its directors, but Bank nonetheless retained the investment. XYZ was also a closely
held stock. Although such an investment is not explicitly proscribed by the Uniform Act, it was
disfavored in some earlier versions of the prudent investor rule. The commitment of 90% of trust assets to
such an investment thus may have conflicted with the trustee’s obligation to preserve liquidity and capital.

While a strong case can be made that Bank acted imprudently, Bank may not be liable because it acted in
accordance with Settlor’s express direction and continued acquiescence. Acting in accordance with a
settlor’s directives is inadequate to absolve a trustee from liability when a trust is irrevocable because the
trustee’s obligations are owed to trust beneficiaries. But when, as here, there were no income beneficiaries
other than Settlor and Settlor held the power to revoke the trust, Settlor could be treated as the effective
owner, thus absolving Bank of liability to Settlor and all other trust beneficiaries.

Uniform Trust Code § 603 specifically provides that, with respect to a revocable trust, a trustee’s duties
are “owed exclusively to” the settlor. Moreover, Uniform Prudent Investor Act § 3 provides that a trustee
shall diversify trust investments unless the trustee “reasonably determines that, because of special
circumstances, the purposes of the trust are better served without diversifying,” and Uniform Prudent
Investor Act § 2 directs the trustee to “consider the purposes, terms, and other circumstances of the trust”
in managing its assets. Under the Restatement of Trusts, the terms of a trust include the “intentions of the
settlor manifested in any way that admits of proof.” This approach reflects the view that the settlor of a
revocable trust has an interest in trust assets that is the practical equivalent of ownership. In addition,
trustees are not generally liable for “consented to” acts and can defend against charges of imprudence by
proving a waiver.

Given these principles and the fact that Settlor directed Bank to invest in XYZ and was aware of the
situation at XYZ, but told Bank that he expected things would turn around, an argument can be made that
Bank is not liable for the imprudent investment in XYZ.

Seperac-J19 Exam-Released MEE Essay Compilation © 2016-2020 1189


#277-JUL 2009–MEE Q01: QUESTION ONE (TRUSTS)

Prior to his death, Settlor created a valid irrevocable trust and named Trustee as trustee. The trust
instrument specified that:

1) Trustee shall distribute trust income to Trustee’s adult children, David and Edna, in such amounts as
Trustee, in Trustee’s absolute and uncontrolled discretion, shall determine.

2) Trustee may distribute all trust income to either David or Edna.

3) Trustee shall have the power to sell any trust property.

4) This trust shall terminate in 10 years and, upon its termination, Trustee shall distribute 60% of trust
property to David and Edna, in equal shares.

5) In view of my long-standing interest in the area of education, Trustee shall distribute all remaining trust
property to my alma mater, Business College.

During the last year of the trust, Trustee paid all trust income to David. Trustee’s decision was motivated
solely by her personal disagreement with Edna’s political opinions.

During the same year, Trustee decided to sell certain trust assets. Solely to avoid paying a 6% sales
commission that would have reduced the trust’s sales proceeds, Trustee, in her personal capacity,
purchased these assets for their undisputed fair market value. After the sale, the assets unexpectedly
increased in value. Trustee recently sold these assets to a bona fide purchaser for a significant profit and
retained the profit for herself.

The trust has terminated, and Trustee has discovered that Business College no longer exists. Trustee seeks
judicial approval to distribute Business College’s share of the trust to Settlor’s estate.

1. Did Trustee breach any fiduciary duty by paying all trust income to David during the last year of
the trust? Explain.

2. Must Trustee return to the trust the profit she retained from assets she purchased from the trust?
Explain.

3. Should a court authorize Trustee to distribute trust property that was payable to Business College
to Settlor’s estate? Explain.

Seperac-J19 Exam-Released MEE Essay Compilation © 2016-2020 1190


#277: J09-1 MEE: ANSWER: NCBE (TRUSTS)

POINT (1) [33%] ISSUE: May a trustee with absolute and uncontrolled discretion be held liable for
abuse of discretion? ANSWER: Yes. Even when a trustee has absolute and uncontrolled discretion,
she may be held liable for actions representing an abuse of discretion. A failure to distribute trust
income based entirely on a personal motive constitutes an abuse of discretion.

POINT (2) [33%] ISSUE: Does a trustee breach her duty of loyalty by purchasing trust property at
its fair market value, and if so, must the trustee return the profits to the trust? ANSWER: Yes.
Because Trustee breached her duty of loyalty by purchasing trust property in her personal
capacity, Trustee must return the profits to the trust.

POINT (3) [33%] ISSUE: When a charitable gift of trust assets cannot be carried out because the
charitable beneficiary no longer exists, should these assets be returned to the settlor’s estate?
ANSWER: The court should find that Settlor had a general charitable intent. Therefore, the trust
property should not be returned to Settlor’s estate. Instead, the court should exercise its cy pres
power and order that the property be distributed to a charity that falls within the general
charitable purposes expressed in the trust.

ANSWER DISCUSSION:

Even when a trustee has absolute and uncontrolled discretion, she may be held liable for actions
representing an abuse of discretion. Failure to distribute trust income to Edna based on a personal motive
constitutes an abuse of discretion, and it is likely that Trustee would be liable for withholding income
from Edna based on her disagreement with Edna’s political opinions. A trustee has a duty of loyalty to
trust beneficiaries. A trustee breaches the duty of loyalty by purchasing trust assets from the trust in her
personal capacity, even if she pays the trust the fair market value of those assets. Because Trustee
breached her duty of loyalty by purchasing property from the trust, Trustee must return the profit to the
trust. When a charitable disposition of trust assets cannot be carried out and the trust has terminated, the
court should determine whether, in making the gift, the settlor had a general or specific charitable
purpose. If the court finds that the settlor had a general charitable purpose, as is likely in this case, it
should exercise its cy pres power and authorize distribution of the trust property to a charity that falls
within the settlor’s general charitable purpose.

ANSWER EXPLANATION:

Explanation to Point-One (30-40%):

Even when a trustee has absolute and uncontrolled discretion, she may be held liable for actions
representing an abuse of discretion. A failure to distribute trust income based entirely on a personal
motive constitutes an abuse of discretion.

A trustee has a fiduciary duty to carry out the terms of the trust established by the settlor; failure to do so
constitutes a breach of trust. In this case, the trust instrument conferred upon Trustee absolute and
uncontrolled discretion to make income distributions, including distribution of all trust income to only one
beneficiary. Trustee’s distribution thus appears to be consistent with the terms of the trust, and a court will
not ordinarily question a trustee’s exercise of a discretionary power, particularly when the trustee is
granted absolute and uncontrolled discretion.

Seperac-J19 Exam-Released MEE Essay Compilation © 2016-2020 1191


However, even when a trustee is granted absolute and uncontrolled discretion, her actions may be
reviewed for abuse of discretion. Application of this standard will insulate the trustee with respect to
decisions about which reasonable individuals might disagree. But when the trustee’s decision is based
exclusively on personal reasons unrelated to the settlor’s goals, the trustee’s decision may be overturned.

Here, Trustee withheld all income from Edna solely because of her personal disagreement with Edna’s
political opinions. Such vindictiveness probably represents an abuse of discretion. It is likely that a court
would hold that Trustee’s decision not to pay Edna any trust income during the last year of the trust was
an abuse of discretion and therefore a breach of the duty to carry out the terms of the trust.

[NOTE: The call is structured in such a way that the applicant need not discuss what damages Edna
might be entitled to for this breach. The amount of her damages may not be easily calculable.]

Explanation to Point-Two (30-40%):

Because Trustee breached her duty of loyalty by purchasing trust property in her personal capacity,
Trustee must return the profits to the trust.

It is a fundamental trust concept that trustees have a duty of loyalty to trust beneficiaries. The duty of
loyalty includes the duty not to engage in self-dealing, such as purchasing property from the trust without
court approval. The prohibition on self-dealing applies even if the purchase price is fair and reasonable.

[NOTE: The trust standard differs from the corporate standard. Under the corporate standard, fairness,
under certain circumstances, may shield directors from liability for a self-dealing transaction.]

Trustee clearly breached her duty of loyalty to the trust beneficiaries by buying trust assets in her personal
capacity. Under the no-further-inquiry rule, the reasonableness of Trustee’s action and her goal to save the
trust the sales commission are irrelevant. Therefore, Trustee is liable to the trust beneficiaries for all
profits resulting from the breach.

Explanation to Point-Three (30-40%):

The court should find that Settlor had a general charitable intent. Therefore, the trust property should not
be returned to Settlor’s estate. Instead, the court should exercise its cy pres power and order that the
property be distributed to a charity that falls within the general charitable purposes expressed in the trust.

Although a settlor or the settlor’s estate is generally entitled to the return of any trust property that cannot
be distributed in accordance with the terms of the trust, an exception applies to charitable trust
dispositions that reflect a general charitable intention. In such a case, a court should exercise its cy pres
power and direct distribution of the failed disposition to another charity.

Until recently, it was necessary to demonstrate that the settlor had a general charitable intention before the
cy pres doctrine could be applied. However, the latest Restatement of Trusts adopts the position that there
should be a presumption that the settlor had a general charitable intention.

Under either approach, a court should find that Settlor had a general charitable intention given that the
trust instrument itself specified that the gift to Business College derived from Settlor’s “long-standing
interest in the area of education.” Because Settlor had a general charitable intention to benefit education,
Settlor’s estate is not entitled to a return of any portion of the trust property. Instead, a court should
exercise its cy pres power and direct distribution to another educational institution.

Seperac-J19 Exam-Released MEE Essay Compilation © 2016-2020 1192


#278-JUL 2008–MEE Q06: QUESTION SIX (TRUSTS)

Ten years ago, Wife created a valid trust (Wife’s Trust) and named Bank as trustee. The trust instrument
directed Bank to: (1) pay all trust income to Wife during her lifetime; (2) upon Wife’s death, pay all trust
income to Niece during Niece’s lifetime; and (3) upon Niece’s death, distribute the trust principal to
Niece’s “then living issue.”

Article Five of Wife’s Trust provided as follows:

Wife may revoke or amend this trust at any time prior to her death by a written instrument delivered to
Bank.

Contemporaneously with the creation of Wife’s Trust, Wife and her husband, Husband, executed separate
wills. Each will devised the respective testator’s entire probate estate to Bank “to hold as part of the
principal of Wife’s Trust, which was created simultaneously with the execution of my will.”

Five years ago, Wife delivered to Bank a written instrument titled “Amendment to Wife’s Trust.” In this
instrument, Wife directed Bank to distribute the trust principal to Niece’s “children age 21 years or older
who are living when Wife’s Trust terminates.”

Four months ago, Wife, Husband, and Niece were involved in an automobile accident. Wife was
pronounced dead in the ambulance. Husband died three days later, and Niece died seven days later. Wife,
who had previously transferred all of her assets to Wife’s Trust, had no probate estate. Husband’s probate
estate was worth $300,000.

Niece had two children: Son, now age 20, and Daughter, who died one year ago at the age of 28. Daughter
had one child (Grandchild), now age 4.

1. Does the Amendment to Wife’s Trust apply to the assets distributable to that trust from
Husband’s probate estate? Explain.

2. Is Son entitled to a share of the assets of Wife’s Trust given the language of the Amendment to
Wife’s Trust? Explain.

3. Is Grandchild entitled to a share of the assets of Wife’s Trust given the language of the
Amendment to Wife’s Trust? Explain.

Seperac-J19 Exam-Released MEE Essay Compilation © 2016-2020 1193


#278: J08-6 MEE: ANSWER: NCBE (TRUSTS)

POINT (1) [31%] ISSUE: Can Husband pour over the assets of his probate estate into a revocable
trust he did not create and, if yes, does an amendment to that trust made after Husband executed
his will apply to the disposition of Husband’s probate estate? ANSWER: Yes. Husband can
bequeath his probate assets to the trustees of a revocable trust created by Wife. The disposition of
those assets will be governed by the terms of the trust, including amendments made after Husband
executed his will.

POINT (2) [38%] ISSUE: Is Son entitled to a share of the assets contributed to Wife’s Trust given
that he had not reached age 21 when Wife and Niece died? ANSWER: Whether Son takes depends
upon the construction of the amendment to Wife’s Trust. If the provision is construed to mean that
Niece’s children must be 21 or older at the time they survive Niece, Son will not take. If the
provision is construed to require only that Niece’s children who survive her eventually reach the
age of 21, Son will take if and when he reaches age 21.

POINT (3) [31%] ISSUE: Is Grandchild entitled to a share of the assets contributed to Wife’s Trust
under either the amendment to or the original terms of Wife’s Trust? ANSWER: Grandchild can
take a share of the trust only if Grandchild is a substituted taker for Daughter under state law. In
most states, Grandchild would not be and thus would not take.

ANSWER DISCUSSION:

Wife retained the power to amend Wife’s Trust and therefore her amendment to that trust was valid.
Under the Uniform Testamentary Additions to Trusts Act and like statutes, if a testator (Husband)
bequeaths assets to the trustees of a revocable trust created by another (Wife), the terms of such trust,
including amendments made after the testator signed his will, govern the disposition of the bequeathed
assets. The terms of Wife’s Trust, as amended, thus govern the disposition of Husband’s probate estate.
Son will take a share unless the trust instrument is construed to mean that the age contingency had to have
been met when Niece died. Grandchild will take if he is treated as a substitute taker of Daughter’s
remainder interest. Under traditional anti-lapse statutes, Grandchild would not be a substitute taker; under
some modern survivorship statutes, Grandchild would be.

ANSWER EXPLANATION:

Explanation to Point-One (20-30%):

Husband can bequeath his probate assets to the trustees of a revocable trust created by Wife. The
disposition of those assets will be governed by the terms of the trust, including amendments made after
Husband executed his will.

Under the Uniform Testamentary Additions to Trusts Act and Uniform Probate Code (UPC), a person
may bequeath assets to a trust created during the testator’s lifetime, by the testator or another, so long as
the trust is identified in the testator’s will and its terms are incorporated in a writing executed before or
concurrently with the execution of the testator’s will. Under these acts, a devise is valid even if the trust is
revocable or amendable. And, even if the trust is amended after the testator’s will was executed, the terms
of the amendment will govern the distribution of assets bequeathed to the trust. It makes no difference that
the amendment was made by someone other than the testator. One of these uniform acts or a like statute

Seperac-J19 Exam-Released MEE Essay Compilation © 2016-2020 1194


has been enacted in every state. Thus Husband could pour his estate over to Wife’s Trust, and the
amendments to that trust would govern the disposition of his estate.

Explanation to Point-Two (25-35%):

Whether Son takes depends upon the construction of the amendment to Wife’s Trust. If the provision is
construed to mean that Niece’s children must be 21 or older at the time they survive Niece, Son will not
take. If the provision is construed to require only that Niece’s children who survive her eventually reach
the age of 21, Son will take if and when he reaches age 21.

The amendment to Wife’s Trust is capable of two interpretations. The narrower interpretation is that
Niece’s children must satisfy both the age and survivorship contingencies at Niece’s death in order to
take. The more liberal interpretation is that the age contingency can be satisfied after Niece’s death. In
construing the instrument, the court will attempt to effectuate Wife’s intent. A strong argument can be
made that only Niece’s children satisfying both the age and survivorship contingencies at her death should
take since the amendment specifies distribution to “children age 21 or older who are living” when the
trust terminates. However, it is hard to imagine why Wife would have wanted to exclude a child of Niece
simply because the child had not yet attained the age of 21 at Niece’s death. It can also be argued that
given Son was only 15 when the amendment was signed, the sole purpose of the age 21 requirement was
to assure that a beneficiary was of sufficient maturity when he/she took possession of the property and
that Wife never anticipated that Niece would die only five years later. Bank could easily keep the trust
open until it was clear whether the underage child could meet the age contingency. The fact that the trust
amendment specifies that principal must be paid when the trust terminates, not when income beneficiaries
die, might also support this interpretation of the instrument.

Explanation to Point-Three (20-30%):

Grandchild can take a share of the trust only if Grandchild is a substituted taker for Daughter under state
law. In most states, Grandchild would not be and thus would not take.

Grandchild is not a taker under the trust instrument because the gift was limited to Niece’s children who
survived the trust termination and the word “children” includes the ancestor’s immediate offspring, not
more remote descendants. Most state anti-lapse statutes apply only to wills and thus would not apply to
the trust amendment. In these states, Grandchild would not take any share of the trust funded by gifts
from Wife or from Husband’s estate.

However, some states have enacted Uniform Probate Code § 2-707 or a like statute. Under this statute,
when a remainder class gift is bequeathed to a class composed of children and a child dies before the
event upon which the remainder becomes possessory (here Niece’s death), a substitute gift is created in
the descendants of the deceased child; words of survivorship (such as “who are living”) do not affect the
creation of a substitute gift. Thus, under Uniform Probate Code § 2-707 and like statutes, Grandchild
would take as the representative of the deceased child beneficiary, Daughter. However, in some states
with statutes similar, but not identical, to § 2-707, the survivorship contingency contained in the trust
amendment could eliminate any subsequent gift to Grandchild.

Seperac-J19 Exam-Released MEE Essay Compilation © 2016-2020 1195


#279-FEB 2008–MEE Q09: QUESTION NINE (TRUSTS)

Six years ago, Settlor created a valid inter vivos irrevocable trust. Settlor funded the trust with publicly
traded securities, named Friend as sole trustee, and directed Friend, as trustee, to distribute all trust
income to James, whom Settlor had been supporting for several years. Settlor also gave James the right to
withdraw up to 5% of the trust’s principal annually during the first 10 years of the trust’s existence.

Four years ago, Friend sold all of the trust’s publicly traded securities and reinvested 45% of the proceeds
in the preferred stock of A Corp., 45% of the proceeds in the preferred stock of B Corp., and 10% of the
proceeds in different publicly traded securities. Both A Corp. and B Corp. were closely held companies
that were newly organized and cash poor. Their preferred stocks were not publicly traded and were
subject to valid restraints on alienation that effectively made them nontransferable by the trustee for the
next 10 years. Furthermore, although both preferred stocks guaranteed an annual 5% dividend, these
dividends were not payable until after the 10-year restraint-on-alienation period had expired.

At the time Friend invested in the preferred stocks of A Corp. and B Corp., both companies were
developing competing technologies to make ballpoint pens that also function as cell phones. Neither
company had yet developed working prototypes of the proposed product. Friend honestly believed that
the preferred stocks represented a safe investment that would produce returns in excess of those available
from any other investment. Friend did not individually own any preferred stock of A Corp. or B Corp.
However, Friend did individually own 70% of the common stock of A Corp., but had no interest in B
Corp.

Despite Friend’s confidence, the A Corp. and B Corp. investments have proven disastrous. Since the
preferred stocks were purchased, neither company has succeeded in producing a marketable product or
making a profit. Both are now near bankruptcy. Moreover, because the stocks have not paid dividends and
cannot be sold, James’s income from the trust has declined precipitously. Furthermore, because of the
valid restraints on alienation, Friend has truthfully told James that any withdrawal requests cannot be
honored.

Friend has consulted your law firm, concerned that he may have breached one or more of his fiduciary
duties. Explain to Friend which duties, if any, he may have breached in carrying out his responsibilities.

Seperac-J19 Exam-Released MEE Essay Compilation © 2016-2020 1196


#279: F08-9 MEE: ANSWER: NCBE (TRUSTS)

POINT (1) [22%] ISSUE: Did Friend breach the duty of loyalty by investing trust assets in a closely
held corporation in which he had a substantial financial interest? ANSWER: Yes. Friend breached
the duty of loyalty by investing the trust assets in a corporation in which Friend, in his individual
capacity, had a substantial investment.

POINT (2) [33%] ISSUE: Did Friend breach the duty to invest prudently by investing 90% of trust
assets in the illiquid stock of two closely held corporations? ANSWER: Yes. Friend breached the
duty to invest prudently by investing the trust assets in the illiquid stocks of cash-poor, closely held
corporations without proven, marketable products.

POINT (3) [22%] ISSUE: Did Friend breach the duty to diversify by investing 90% of trust assets
in two corporations with virtually identical businesses? ANSWER: Yes. Friend breached the duty
to diversify by investing 90% of the trust’s assets in two corporations that were involved in the
same type of business and subject to the same market risks.

POINT (4) [22%] ISSUE: Did Friend breach the duty of care by making investments that produced
no current income and made it impossible for James to exercise his 5% withdrawal power?
ANSWER: Yes. Friend breached the duty of care by investing the trust assets in a manner that
precluded administration of the trust in accordance with its terms and purposes.

ANSWER DISCUSSION:

Friend breached the duty of loyalty, the duty to invest prudently, the duty to diversify the trust
investments, and the duty of care when he invested 90% of the trust’s assets in the preferred stock of two
closely held corporations. Friend breached the duty of loyalty because he had a substantial financial
interest in one of the closely held corporations. He breached the duty to invest prudently and the duty to
diversify because the investments significantly diminished the liquidity of the trust principal and because
A Corp. and B Corp. were involved in similar high-risk businesses. He breached the duty of care by
investing in a manner that precluded carrying out the trust’s obligations to James with respect to the
payment of income and James’s annual 5% withdrawal power.

ANSWER EXPLANATION:

Explanation to Point-One (15-25%):

Friend breached the duty of loyalty by investing the trust assets in a corporation in which Friend, in his
individual capacity, had a substantial investment.

A trustee owes trust beneficiaries a duty of loyalty and must administer the trust exclusively in the
beneficiaries’ interests. Because the trustee must act on behalf of the beneficiaries, and not on behalf of
himself, the duty of loyalty is breached when a trustee enters into transactions, on behalf of the trust, that
involve a conflict of interest or self-dealing. Friend entered into a transaction involving a conflict of
interest.

Uniform Trust Code § 802(c) provides that an investment in “a corporation in which the trustee has an
interest that might affect the trustee’s best judgment” is presumptively a breach of the duty of loyalty.

Seperac-J19 Exam-Released MEE Essay Compilation © 2016-2020 1197


Here, Friend invested in A Corp., in which he held 70% of the common stock. Although the comments to
§ 802 note that the presumption of a breach can be rebutted by showing that the terms of the transaction
were fair or that the transaction would have been made by an independent party, there is no evidence to
support such a showing in this case. It is also hard to imagine that an independent trustee would have
made such a risky and illiquid investment.

Explanation to Point-Two (25-35%):

Friend breached the duty to invest prudently by investing the trust assets in the illiquid stocks of cash-
poor, closely held corporations without proven, marketable products.

Almost all states have adopted, in one form or another, the Uniform Prudent Investor Act. Under this act,
a trustee owes trust beneficiaries a duty to invest trust assets prudently. In assessing whether a trustee has
breached this duty, the Act requires consideration of a number of factors, including (1) the distribution
requirements of the trust, (2) general economic conditions, (3) the role the investment plays in
relationship to the trust’s overall investment portfolio, and (4) the trust’s need for liquidity, regularity of
income, and preservation or appreciation of capital. All of these factors support a finding that Friend’s
investment was imprudent.

The investments in A Corp. and B Corp. represented a very large share (90%) of the trust assets and
dominated the portfolio. The investments drastically reduced the liquidity of the trust, making it
impossible to meet the trust’s distribution requirements. The investments also reduced the trust’s annual
income, resulting in a reduction in mandatory income distributions. The effects of the investments on the
trust income and liquidity were not unexpected, but were known at the outset. Moreover, the technologies
that both companies planned to develop were unproven; working prototypes had not been developed, and
there was no evidence that such inventions would succeed in the highly competitive cell-phone market.
Lastly, since the companies were developing competing technologies, there is a good chance that
ultimately one of them would fail (e.g., Blu-ray vs. HD DVD).

Explanation to Point-Three (15-25%):

Friend breached the duty to diversify by investing 90% of the trust’s assets in two corporations that were
involved in the same type of business and subject to the same market risks.

A trustee owes a duty to diversify trust investments unless he “reasonably determines that, because of
special circumstances, the purposes of the trust are better served without diversifying.”

The diversification duty rests on the assumption that it is risky to place all or substantially all of one’s
assets in the same investment basket. Although there is “no automatic rule for identifying how much
diversification is enough,” it seems highly unlikely that any fact finder would find the investment of 90%
of trust assets in one narrow and unproven market sector sufficiently diverse.

Additionally, there are no facts in this case to support a finding that a lack of diversification was
warranted because of special circumstances. Settlor funded the trust with publicly traded securities. The
terms of the trust contemplated investments that would produce income and distributable principal.

Explanation to Point-Four (15-25%):

Friend breached the duty of care by investing the trust assets in a manner that precluded administration of
the trust in accordance with its terms and purposes.

Seperac-J19 Exam-Released MEE Essay Compilation © 2016-2020 1198


Under the common law, a trustee owes beneficiaries the duty to act with care, skill, and prudence.
Although the Uniform Trust Code does not impose on trustees a duty of care, it does impose a duty to
“administer the trust in good faith, in accordance with its terms and purposes and the interests of the
beneficiaries.”

Whichever formulation is used, Friend breached his duty by making investments that made it impossible
to carry out the terms of the trust. The facts clearly evidence Settlor’s intention to provide James with an
annual income and imply that this income may have been intended to supplant the support Settlor
provided James before creating the trust. The facts also clearly show that Settlor gave James the power to
annually withdraw up to 5% of trust principal over the first 10 years of the trust. The investments in A
Corp. and B Corp. effectively prevented James from exercising his withdrawal power and dramatically
curtailed his income from the trust.

Seperac-J19 Exam-Released MEE Essay Compilation © 2016-2020 1199


#280-JUL 2007–MEE Q08: QUESTION EIGHT (TRUSTS)

Settlor, age 60, consulted an attorney, Attorney, about the creation of a trust. Settlor gave Attorney a
memorandum containing the following information about his family:

1. Family Member: Wife; Relationship: Spouse; Age: 48

2. Family Member: Son; Relationship: Child; Age: 21

3. Family Member: Daughter; Relationship: Child; Age: 16

4. Family Member: Daughter; Relationship: Child; Age: 10

5. Family Member: Grandchild; Relationship: Grandchild (Son’s child); Age: 1

Settlor also outlined his goals in creating the trust:

1. I want to fully control trust assets and enjoy all trust income until I die.

2. After I die, I want trust assets used to ensure that Wife is comfortably provided for. I also want Wife to
be able to use trust assets to reward, in her will, whichever children have been most helpful to her. I don’t
want Wife to be a trustee; she doesn’t have the financial background.

3. After Wife dies, I want my children to get the remaining trust assets. But, of course, if Son dies before
Wife, I’d want his share to go to Grandchild.

4. I’m planning to fund the trust with cash and stocks. I may want to add some other assets later, but I’m
not sure.

Based on the information provided by Settlor, Attorney drafted the following trust instrument:

SETTLOR TRUST AGREEMENT

1. I appoint Bank as trustee of the Settlor Trust.

2. I direct Bank to hold all assets listed on Schedule A in trust, and I direct Bank to dispose of these assets
as follows:

a. Bank shall pay all trust income to Settlor during Settlor’s lifetime.

b. After Settlor’s death, Bank shall pay trust income and principal to Wife in such amounts as Bank, in its
sole discretion, deems appropriate.

c. After Wife’s death, Bank shall distribute all remaining trust assets equally among Settlor’s surviving
children, share and share alike.

3. Bank accepts and agrees to faithfully carry out the terms of this trust.

[Signatures, dates, and acknowledgments are omitted]

Seperac-J19 Exam-Released MEE Essay Compilation © 2016-2020 1200


SCHEDULE A

12,000 Shares of XYZ Corporation common stock

$150,000 (cash)

How would you revise the Settlor Trust Agreement to more fully meet Settlor’s stated goals? Explain.

Seperac-J19 Exam-Released MEE Essay Compilation © 2016-2020 1201


#280: J07-8 MEE: ANSWER: NCBE (TRUSTS)

POINT (1) [26%] ISSUE: How can Settlor retain full control of trust assets and income?
ANSWER: Because Settlor wants to retain full control of trust assets, he should retain a power of
revocation. To give Settlor more control, he might be also given a right of withdrawal and be named
sole trustee.

POINT (2) [13%] ISSUE: How can Settlor retain the right to make future additions to the trust?
ANSWER: Because Settlor contemplated the possibility of adding assets to the trust, the trust
should have an additions clause.

POINT (3) [26%] ISSUE: How can Settlor ensure that trust assets are used so that Wife is
comfortably provided for after his death? ANSWER: Because Settlor wanted to ensure that Wife
would be comfortably provided for the trust instrument should require distribution in accordance
with an ascertainable standard relating to Wife’s maintenance and support, and Wife should be
granted a withdrawal power.

POINT (4) [21%] ISSUE: How can Settlor ensure that assets remaining in the trust after Wife’s
death go to their children and give Wife the power to reward the children in her will? ANSWER:
Because Settlor wanted to leave trust principal to his children and to enable Wife “to use trust
assets as a way of rewarding, in her will, whichever children have been most helpful,” and also
expressed a desire that the descendants of a deceased child should take by substitution, Wife should
have been given a special testamentary power of appointment. This power should extend to issue,
not just children, to enable Wife to appoint to more remote descendants.

POINT (5) [15%] ISSUE: How can Settlor ensure that issue of a child who predeceases Wife will
take the deceased child’s share of trust principal? ANSWER: Because Settlor wanted the issue of
any deceased child to take his parent’s share of trust assets, the trust instrument should have so
specified.

ANSWER DISCUSSION:

A redlined draft of the trust instrument showing alterations that would better meet Settlor’s stated goals
follows (removed language in parenthesis): SETTLOR TRUST AGREEMENT 1. I appoint (Bank) Settlor
as trustee of the Settlor Trust and, upon Settlor’s death, I appoint Bank as trustee. 2. I reserve the right to
revoke this trust at any time, by deed or will. 3. I reserve the right to withdraw and add assets to this trust
at any time, by deed or will. 4. I direct (Bank) the trustee to hold all assets listed on Schedule A in trust
and to dispose of these assets as follows: a. (Bank) Settlor, as trustee, shall pay all trust income to Settlor
during Settlor’s lifetime. b. After Settlor’s death, Bank, as trustee, shall pay such trust income and
principal to Wife (in such amounts as Bank, in its sole discretion, deems appropriate) as is necessary to
comfortably provide for her support and maintenance. c. Wife shall have the right to withdraw trust
principal as is necessary to comfortably provide for her support and maintenance. d. After Wife’s death,
Bank shall distribute all remaining trust assets to such one or more of Settlor’s issue as Wife shall appoint
by will. If Wife does not exercise her power of appointment, Bank shall distribute all remaining trust
assets equally among Settlor’s surviving children, share and share alike, provided, however, that if any
child of Settlor predeceases Wife leaving issue surviving, the share of the deceased child shall be paid to
that child’s issue. 5. (Bank) Settlor accepts and agrees to faithfully carry out the terms of this trust.
[Signatures, dates, and acknowledgments are omitted] SCHEDULE A 12,000 Shares of XYZ Corporation

Seperac-J19 Exam-Released MEE Essay Compilation © 2016-2020 1202


common stock $150,000 (cash) [NOTE: This new draft does not necessarily represent the best method of
meeting Settlor’s stated goals, but it does respond to the most obvious deficiencies in Attorney’s draft.]

ANSWER EXPLANATION:

Explanation to Point-One (20-30%):

Because Settlor wants to retain full control of trust assets, he should retain a power of revocation. To give
Settlor more control, he might be also given a right of withdrawal and be named sole trustee.

In some states an inter vivos trust is revocable unless the instrument expressly provides otherwise, but in
most jurisdictions it is irrevocable unless the instrument expressly provides otherwise. Thus a well-drafted
trust instrument should always expressly state whether it is revocable or irrevocable.

Here Settlor told Attorney: “I want to fully control trust assets until I die.” An irrevocable trust is
incompatible with this aim. To ensure that Settlor has the power to revoke as part of a testamentary plan
as well as during his lifetime, the revocation clause should expressly provide that it can be exercised by
deed or will.

Additional control could be conferred upon Settlor through a retained right of withdrawal. This power
would enable Settlor to withdraw assets from the trust without revoking it.

Finally, Settlor could be named sole trustee with Bank the successor trustee. This step would give Settlor
complete control of the management and administration of the trust.

Explanation to Point-Two (10-15%):

Because Settlor contemplated the possibility of adding assets to the trust, the trust should have an
additions clause.

As a general proposition, a trustee’s duties cannot be unilaterally enlarged by the settlor after the trustee
has accepted the office. The Uniform Trust Code provides that upon acceptance of a trusteeship, the
trustee shall administer the trust in good faith, in accordance with its terms and purposes.”). Thus a well-
drafted trust instrument should contain an additions clause if the settlor contemplates enlarging the
trustee’s responsibilities by augmenting the trust with additional assets. (Even then, however, a trustee
may be able to reject additions.

Explanation to Point-Three (20-30%):

Because Settlor wanted to ensure that Wife would be comfortably provided for the trust instrument should
require distribution in accordance with an ascertainable standard relating to Wife’s maintenance and
support, and Wife should be granted a withdrawal power.

Settlor specified that he wanted Wife “to be comfortably provided for.” Granting the trustee discretionary
power to distribute income and principal to Wife cannot ensure this result because a beneficiary cannot
compel a trustee to distribute trust property not otherwise subject to a standard unless he or she can show
an abuse of discretion. An ascertainable standard, such as “comfortable maintenance and support,” sets
bounds on the trustee’s discretion and would empower Wife to compel distributions from the trust if
necessary.

Seperac-J19 Exam-Released MEE Essay Compilation © 2016-2020 1203


To further ensure that Wife is comfortably provided for, she could be given a right to withdraw trust
principal. Such a power could be unconditional or, as in the revised instrument, might be limited to
withdrawals necessary to ensure her comfortable support.

[NOTE: Unconditional withdrawal powers are often limited to $5,000 or 5 percent of trust principal in
order to obtain tax advantages.]

Explanation to Point-Four (15-25%):

Because Settlor wanted to leave trust principal to his children and to enable Wife “to use trust assets as a
way of rewarding, in her will, whichever children have been most helpful,” and also expressed a desire
that the descendants of a deceased child should take by substitution, Wife should have been given a
special testamentary power of appointment. This power should extend to issue, not just children, to enable
Wife to appoint to more remote descendants.

Settlor expressed the desire that his children take remaining trust principal and that Wife have the right to
reward their children in her will. He also expressed a desire to have a deceased child’s descendants take
the deceased child’s share. To accomplish these goals, Wife should have been given a special
testamentary power of appointment. Such a power would enable Wife to favor some children over others,
as Settlor wants. But because the exercise of a special power is limited to the group selected by the donor
of the power (here, Settlor), Wife could not appoint trust assets to individuals other than Settlor’s issue.
The special power would both ensure that trust assets ultimately go to Settlor’s children or more remote
descendants and give Wife control. Exercise of a power can also be restricted, as Settlor has specified, to
testamentary bequests. No other drafting tool can fully meet all of Settlor’s goals. As drafted, Wife could
appoint to a grandchild even if the grandchild’s parent was alive. This could be avoided, if Settlor desired,
by providing: “After Wife’s death, Bank shall distribute all remaining trust assets to such one or more of
the Settlor’s children and issue of deceased children as Wife.”

[NOTE: The word “issue” gives Wife the power to appoint to children as well as more remote
descendants. This allows Wife the power, consistent with Settlor’s intent, to give the property to the
descendants of the deceased child.]

Explanation to Point-Five (10-20%):

Because Settlor wanted the issue of any deceased child to take his parent’s share of trust assets, the trust
instrument should have so specified.

Settlor stated that, “if Son dies before Wife, I’d want his share to go to Grandchild.” Although Settlor did
not specify that he would want surviving issue to take the shares of his other children, it’s likely that he
focused on Son because Son was the only child to have a child.

In many jurisdictions, the instrument drafted by Attorney will not accomplish the aim of ensuring that
surviving issue take their deceased parents’ shares. Many state anti-lapse statutes apply only to
testamentary bequests, and the Settlor Trust was established during Settlor’s lifetime. Thus the trust
instrument should expressly state that issue take a deceased parent’s share.

Seperac-J19 Exam-Released MEE Essay Compilation © 2016-2020 1204


#281-FEB 2007–MEE Q02: QUESTION TWO (TRUSTS)

Testator died three years ago. His duly probated will provided that:

1. I give $100,000 to Trustee to hold in trust and to distribute the trust income equally among those
persons who are my friends at my death. After 10 years, the trust shall terminate and the trust property
shall be distributed equally between my son, Sam, and the Fine Arts Program at State University. In no
event shall this trust terminate earlier than 10 years after my death.

2. I give the rest of my estate to my daughter, Donna.

Both Sam and Donna survived Testator.

Walter and Janice, two neighbors of Testator, correctly claim they were good friends of Testator at the
time of his death and demand that Trustee pay the income from the $100,000 trust to them. Claimant, who
has a tort judgment against Sam, demands that Trustee immediately pay Claimant $25,000 from the trust
to satisfy the judgment.

Two years after Testator died, State University closed as a result of a state budget crunch and the
legislature’s determination that the programs at State University, including its fine arts program, were
largely duplicative of the programs at State Polytech, the other public university in the state.

1. To whom should the income from the $100,000 trust be distributed? Explain.

2. Should Trustee immediately pay $25,000 from the trust to Claimant? Explain.

3. To whom should Trustee pay the trust principal at the end of the 10-year period? Explain.

Seperac-J19 Exam-Released MEE Essay Compilation © 2016-2020 1205


#281: F07-2 MEE: ANSWER: NCBE (TRUSTS)

POINT (1) [38%] ISSUE: Does Testator’s gift of trust income to friends fail for lack of definite
beneficiaries? ANSWER: Yes. Trustee should pay the trust income to Donna, the residuary taker,
because the gift of income to Testator’s friends fails for want of definite beneficiaries.

POINT (2) [33%] ISSUE: Does Claimant, as a judgment creditor of Sam, have a right to immediate
payment of $25,000 from the trust? ANSWER: No. Trustee should not immediately pay over
$25,000 to Claimant because, as a creditor of a trust beneficiary, Claimant’s rights cannot exceed
those of Sam, the beneficiary, and Sam has only a right to half of the principal in 10 years.

POINT (3) [29%] ISSUE: Should the gift to State University fail because State University ceased to
exist shortly after Testator’s death, or should it be paid to another charity? ANSWER: At the end
of the 10-year period, one-half of the trust principal should be distributed to Sam. The other half
should be distributed, in all likelihood, to State Polytech in lieu of State University by application of
the court’s cy pres power.

ANSWER DISCUSSION:

The trust’s income should be paid to Donna, the residuary legatee under Testator’s will, because the gift
of the income fails for want of definite beneficiaries. As to the principal, nothing should be paid to
Claimant before the trust terminates because Claimant’s rights cannot exceed those of Sam, the trust
beneficiary, and Sam only has a right to receive trust principal after 10 years. When the trust terminates,
one half of the corpus should pass to Sam and the other half, in all likelihood, to State Polytech, as a result
of the probate court’s exercise of its cy pres power.

ANSWER EXPLANATION:

Explanation to Point-One (35-45%):

Trustee should pay the trust income to Donna, the residuary taker, because the gift of income to Testator’s
friends fails for want of definite beneficiaries.

It is a well-established trust law principle that a private express trust must have definite beneficiaries. The
term “friends” is indefinite because it is impossible to determine the precise number of persons who fit
this description. In a leading case, a trust was invalidated where the trustee had the discretion to select
among friends. Although the modern trend would allow a trustee to select a beneficiary from an indefinite
class, this liberalized position does not apply if the trustee must distribute equally to all members of an
indefinite class. Here the trust instrument requires Trustee to distribute income equally among all of
Testator’s friends. Thus, the income portion of the trust fails for want of definite beneficiaries. The fact
that Walter and Janice may have been friends of Testator’s does not entitle them to any distribution
because the income must be distributed equally among all of Testator’s friends.

Where an attempt to create a trust fails, the trustee holds a resulting trust for the settlor or the settlor’s
successors in interest. Where a testamentary trust, as here, fails, the residuary legatee succeeds to the
property interest. Donna, the residuary legatee under Testator’s will, would be Testator’s successor-in-
interest of the resulting trust. Accordingly, Trustee should pay over trust income to Donna during the trust
term.

Seperac-J19 Exam-Released MEE Essay Compilation © 2016-2020 1206


It could be argued that the income for the first ten years either should be accumulated for ultimate
distribution to the remainder beneficiaries or, alternatively, should be distributed to the presumptive
remainder beneficiaries currently. An applicant could reasonably argue in favor of accumulating trust
income for ultimate distribution to the remainder beneficiaries when the trust terminates. However,
because the trust prohibited a distribution to Sam and State University earlier than 10 years after
Testator’s death, the trust should not terminate under the acceleration of remainder doctrine. This fact
makes the alternative of current income distribution to the remainder beneficiaries less likely.

Explanation to Point-Two (30-40%):

Trustee should not immediately pay over $25,000 to Claimant because, as a creditor of a trust beneficiary,
Claimant’s rights cannot exceed those of Sam, the beneficiary, and Sam has only a right to half of the
principal in 10 years.

A creditor may reach a beneficiary’s interest in a trust if the trust does not contain a spendthrift provision.
But a beneficiary’s creditor can have no greater rights in trust assets than the beneficiary, and a
beneficiary of trust principal is not entitled to trust principal until the termination of all preceding estates.
Thus, Claimant would not be entitled to any trust principal until the trust terminates. Indeed, if the trust
principal were reachable during the 10-year period, then the rights of any income beneficiaries would be
adversely affected because their rights are based on income derived from the entire principal.

Therefore, Claimant is not entitled to receive $25,000 of trust principal. Claimant can only reach what
Sam had – a right to receive trust principal after 10 years. Of course, Claimant would be entitled to a
$25,000 distribution from Trustee when the trust terminates, assuming that the judgment against Sam is
still outstanding and that his interest in the trust at that time is at least $25,000.

Explanation to Point-Three (25-35%):

At the end of the 10-year period, one-half of the trust principal should be distributed to Sam. The other
half should be distributed, in all likelihood, to State Polytech in lieu of State University by application of
the court’s cy pres power.

The trust provides that, at the end of ten years, the trust principal shall be distributed equally to Sam and
State University. Sam is entitled to half the principal at that time, subject to a possible $25,000 reduction
to pay Claimant.

The other half of the principal is payable to State University, but this gift cannot be paid because State
University no longer exists. However, State University is a charity. It is a well-established principle of
trust law that, when a charitable purpose cannot be carried out, the court should determine whether to
exercise its cy pres power and redirect the charitable gift to another like charity.

The cy pres doctrine requires an initial inquiry into the settlor’s intent: if the court determines that the
settlor had a specific charitable intention limited to the stated charitable purpose, the gift goes to the
residuary legatee, in this case Donna. However, if the court determines that the settlor had a general
charitable intention, it should substitute for the named charity another charity that is consistent with the
settlor’s intentions. Both the UTC and Restatement (Third) of Trusts presume that a settlor has a general
charitable intent.

Given the presumption of general charitable intent, there is a strong argument that the court should
exercise its cy pres power and substitute the fine arts program at State Polytech for that at State

Seperac-J19 Exam-Released MEE Essay Compilation © 2016-2020 1207


University. The will evidences a desire to benefit a fine arts program. It evidences no intention of limiting
that benefit to the program at State University. Testator could not have known that State University would
cease to exist after his death. The state legislature has already determined that State University’s fine arts
program largely duplicates that at State Polytech. Substituting State Polytech for State University would
maintain Testator’s commitment to the fine arts and public education in his state of residence. For
example, if the testator left money to a particular university to teach courses in dianetics, and the
university refused to accept the funds on that basis, the court may permit the legacy to be applied to the
establishment of such a course in another university, assuming the settlor has not provided otherwise.

If the court does not apply the cy pres doctrine, the gift to charity fails and the share State University was
intended to receive either passes wholly to Sam as the only remainder beneficiary or wholly to Donna, the
residuary legatee under Testator’s will.

Seperac-J19 Exam-Released MEE Essay Compilation © 2016-2020 1208


#282-FEB 2006–MEE Q01: QUESTION ONE (TRUSTS)

Seven years ago, Settlor announced at a dinner party:

“I am hereby creating the Settlor’s Family Trust and naming myself as trustee. In about three years, I
expect to sell some stock and to fund this trust with the sale proceeds. I reserve the power to pay to myself
trust income and principal for my support at any time. When I die, all remaining trust assets are to be
distributed to my daughter, Dawn. Neither my creditors nor Dawn’s creditors can reach the trust assets to
satisfy their claims, and neither of us is free to sell or otherwise transfer our respective trust interests.
Please join me in a toast as I sign this cocktail napkin on which I have written the terms of the trust.”

Five years ago, Settlor’s sister-in-law, In-Law, executed a will leaving her entire $300,000 estate to the
Settlor’s Family Trust.

Four years ago, Settlor sold some stock and deposited the $100,000 sale proceeds into an account at First
Bank in the name of “Settlor, as Trustee of the Settlor’s Family Trust.”

Two months ago, In-Law died. Had In-Law died intestate, her estate would have passed to her brother,
Bill. Both Settlor, as trustee of the Settlor’s Family Trust, and Bill claim In-Law’s estate.

One month ago, Victim obtained a $75,000 tort judgment against Settlor because of injuries sustained as a
result of Settlor’s negligence. Victim immediately sought to collect that judgment from the First Bank
account, notwithstanding Settlor’s claims that this account was beyond the reach of his judgment
creditors. Before this dispute could be resolved, Settlor died.

1. Was the Settlor’s Family Trust validly created? Explain.

2. Who is entitled to In-Law’s estate? Explain.

3. Can Victim reach the assets of the First Bank account to satisfy Victim’s claim against Settlor?
Explain.

Seperac-J19 Exam-Released MEE Essay Compilation © 2016-2020 1209


#282: F06-1 MEE: ANSWER: NCBE (TRUSTS)

POINT (1) [37%] ISSUE: Did Settlor create a valid trust? ANSWER: Yes. Settlor’s Family Trust
became a valid trust when the First Bank account was opened in the name of Settlor, as trustee. The
trust had both a trustee and beneficiaries and, once the account was opened, trust assets.

POINT (2) [32%] ISSUE: Was In-Law’s pour-over bequest to the Settlor’s Family Trust valid?
ANSWER: Yes. The pour-over bequest is valid because the trust was clearly identified in In-Law’s
will and the trust’s terms were incorporated in a writing antedating that will.

POINT (3) [32%] ISSUE: Did Settlor effectively shield trust assets from creditors when he created
a support trust with a spendthrift clause and named himself as the sole income beneficiary?
ANSWER: The judgment creditor can reach trust assets to the extent the assets of this self-settled
trust could be reached by Settlor.

ANSWER DISCUSSION:

Settlor did not create a valid trust when he made a declaration of trust because the trust lacked assets. The
trust became valid when Settlor deposited funds in the First Bank account and re-manifested his intention
to create a trust by opening the account in the name of Settlor, as Trustee of the Settlor’s Family Trust. In-
Law’s pour-over bequest was valid because the trust was clearly identified in In-Law’s will and its terms
were incorporated in a writing that predated that will. Victim can reach trust assets in the First Bank
account because the spendthrift provision is ineffective, both because the trust was self-settled and, in
some states, because it would be deemed revocable.

ANSWER EXPLANATION:

Explanation to Point-One (30-40%):

Settlor’s Family Trust became a valid trust when the First Bank account was opened in the name of
Settlor, as trustee. The trust had both a trustee and beneficiaries and, once the account was opened, trust
assets.

A trust of personal property is valid if it has a trustee, a beneficiary, and trust property. Under the Uniform
Trust Code, a writing is not necessary to create an enforceable inter vivos trust. After Settlor made his
announcement and signed the napkin memorializing his intentions, the Settlor Family Trust had a trustee,
Settlor, and two named beneficiaries, Settlor and Dawn. But it lacked assets because the stock sale
proceeds that Settlor declared as trust assets did not yet exist. The announcement and napkin thus
evidenced nothing more than the intent to create a trust in the future, and a promise to create a trust in the
future is typically unenforceable without consideration.

However, the $100,000 deposit into the First Bank account three years later was sufficient to create a
valid trust at that time. If a trust that is invalid for lack of assets is later funded, a trust arises at that time if
the settlor re-manifests the intention to create the trust. By depositing $100,000 into an account at First
Bank in the name of Settlor, as Trustee of the Settlor’s Family Trust, Settlor re-manifested an intention to
create a trust with the terms described in Settlor’s original declaration. Thus, while the trust was not valid
when Settlor made his announcement, it was valid after the deposit into the First Bank account.

Seperac-J19 Exam-Released MEE Essay Compilation © 2016-2020 1210


Explanation to Point-Two (25-35%):

The pour-over bequest is valid because the trust was clearly identified in In-Law’s will and the trust’s
terms were incorporated in a writing antedating that will.

Under the Uniform Testamentary Additions to Trust Act and Uniform Probate Code, a person may
bequeath assets to a trust created during the testator’s lifetime, by the testator or another, so long as the
trust is identified in the testator’s will and its terms are incorporated in a writing executed before or
concurrently with the execution of the testator’s will. Under these acts, such a bequest is valid even if the
trust is unfunded, revocable, and amendable. In-Law clearly identified the “Settlor’s Family Trust” in her
will, and its terms were incorporated in a writing, the cocktail napkin, predating the execution of that will.
As the acts do not require a formal document, the napkin satisfies the writing requirement. One of the
uniform acts or a like statute has been enacted in all states. Therefore, In-law could pour her estate over to
the Settlor’s Family Trust.

The common law incorporation-by-reference doctrine, under which a testator may direct the distribution
of his probate assets in accordance with the terms of another instrument that exists when the testator’s will
was executed and which is specifically referred to in the will, would also permit a court to distribute In-
Law’s estate in accordance with the provisions of the Settlor’s Family Trust. Thus, under either the
common law or modern uniform acts, Settlor, as trustee, would take In-Law’s estate.

Explanation to Point-Three (25-35%):

The judgment creditor can reach trust assets to the extent the assets of this self-settled trust could be
reached by Settlor.

Because the Settlor’s Family Trust is a self-settled trust, its spendthrift provisions are not enforceable on
behalf of the trust settlor. The fact that Settlor’s right of withdrawal is limited to funds “for my support”
makes no difference; when the settlor of a trust is also a trust beneficiary, his creditors are entitled to the
maximum amount that could be distributed from the trust to the settlor, even when withdrawals are
discretionary or limited by a support standard.

Settlor’s creditors might also argue that trust assets can be reached because the Settlor’s Family Trust is
revocable. Many states provide that creditors of the settlor of a revocable trust can reach trust assets in
satisfaction of their claims. Contrary to the common law, some states now presume that a self-settled inter
vivos trust is revocable. In other states, the creditors might argue that Settlor’s retained power to distribute
income and principal to himself is effectively a power of revocation.

[NOTE: Applicants should not receive credit for the conclusion that the spendthrift provision is invalid
because the creditor is a tort victim. A few state courts have refused, on public policy grounds, to enforce
spendthrift provisions against the claims of tort victims who can show gross negligence or intentional
misconduct. But the facts here do not show gross negligence or an intentional tort and only a handful of
courts have adopted this position.]

Seperac-J19 Exam-Released MEE Essay Compilation © 2016-2020 1211


#283-JUL 2005–MEE Q07: QUESTION SEVEN (TRUSTS)

Two years ago, Testator died. By her duly probated will, Testator created two separate trusts naming First
Bank as the trustee of each trust.

1. Testator left $100,000 to Friends’ Trust and directed First Bank “to distribute the income annually
among my friends in equal shares. At the end of ten years, this trust shall terminate and the trust corpus
shall be added to the principal of Residuary Trust created under this will.”

2. Testator left $500,000 to Residuary Trust and directed First Bank “to pay the income to Carrie for life
and to distribute the principal, during Carrie’s life or upon her death, to any or all of Carrie’s issue as she
appoints by deed or will and, in the absence of such appointment, to my alma mater, University, upon
Carrie’s death.”

Last week, First Bank received two instruments in the mail. The first was a letter from George seeking a
distribution of income from Friends’ Trust and correctly claiming that he was a close friend of Testator.

The second was a deed labeled “Appointment” from Carrie directing First Bank to hold the $500,000
Residuary Trust in a new trust and to distribute income to Carrie’s son, John, during his life. This deed
further provided that upon John’s death, First Bank should distribute the trust principal to Charity, a
charitable organization, instead of to University as directed by Testator in her will.

1. Can First Bank distribute income from Friends’ Trust to George? Explain.

2. Did Carrie validly appoint an interest in Residuary Trust to her son John? Explain.

3. In light of Carrie’s deed of appointment, what, if any, are the interests of Charity and University
in the new trust? Explain.

Seperac-J19 Exam-Released MEE Essay Compilation © 2016-2020 1212


#283: J05-7 MEE: ANSWER: NCBE (TRUSTS)

POINT (1) [38%] ISSUE: Is Friends’ Trust invalid for failure to have “definite beneficiaries”?
ANSWER: Yes. A trust for “friends” is probably invalid for want of definite beneficiaries. In order
to create a valid trust, there must be intent, delivery, and acceptance. Additionally, there must be
trust property, a trustee, and one or more beneficiaries. The Friends’ Trust has a trustee, First
Bank, and property, $100,000. The question is whether there is a beneficiary.

POINT (2)(a) [19%] ISSUE: Can the donee of a special power of appointment exercisable in favor
of the donee’s issue exercise the power by appointing only a life estate to a permissible object of the
power? ANSWER: Yes. The donee of a special power of appointment exercisable in favor of the
donee’s issue can exercise the power by appointing only a life estate to a permissible object of the
power. Thus, the appointment to John is valid.

POINT (2)(b) [19%] ISSUE: Can the donee of a special power of appointment exercisable in favor
of the donee’s issue exercise the power by appointing an interest in the property to a charitable
organization? ANSWER: No. Carrie’s purported exercise of her power in favor of Charity is
invalid because Charity is an impermissible object of the power.

POINT (3) [25%] ISSUE: If a donee of a special power of appointment limited in favor of issue
appoints assets subject to the power to an impermissible object of the power, who succeeds to the
interest that was invalidly appointed? ANSWER: Because of Carrie’s partially ineffective exercise
of the special power, the remainder of the new trust at John’s death will pass to University, the
taker in default under Testator’s will.

ANSWER DISCUSSION:

A trust for “friends” is invalid because it does not have “definite beneficiaries.” Thus, First Bank should
make no distribution to George. Carrie’s appointment of income to her son John is valid. Although
Testator did not expressly state that appointments in further trust are permissible, the majority view is
that, if the donee can appoint trust assets outright, the donee can also create lesser interests in permissible
objects of the power. However, Charity is an impermissible object of Carrie’s special power; thus Carrie
made an ineffective appointment of the remainder. That interest will pass at John’s death to University,
the taker in default of appointment designated by Testator.

ANSWER EXPLANATION:

Explanation to Point-One (25-35%):

A trust for “friends” is probably invalid for want of definite beneficiaries. In order to create a valid trust,
there must be intent, delivery, and acceptance. Additionally, there must be trust property, a trustee, and
one or more beneficiaries. The Friends’ Trust has a trustee, First Bank, and property, $100,000. The
question is whether there is a beneficiary.

A trust for indefinite beneficiaries is invalid. This rule derives from the principle that trust beneficiaries
must be able to enforce the terms of the trust. A trust for the benefit of “friends” is a trust with indefinite
beneficiaries.

Seperac-J19 Exam-Released MEE Essay Compilation © 2016-2020 1213


The Restatement of Trusts (Third) takes the position that a trust for friends may be valid if “some
ascertainable group of friends was intended or an implied term of the trust authorizes the trustee to
determine who the friends are.” However, there are no facts to support such a finding in this case. The
disposition cannot be saved by characterizing Friends’ Trust as a power of appointment because a power
that must be exercised (a so-called “imperative power”), as here, also requires definite beneficiaries. A
discretionary power, vesting in the trustee the power both to select beneficiaries and to determine
distributions, is valid and would permit a trustee to distribute to friends. But here the instrument creating
Friends’ Trust did not give First Bank discretion to select beneficiaries or determine distributions.

Explanation to Point-Two(a) (10-20%):

The donee of a special power of appointment exercisable in favor of the donee’s issue can exercise the
power by appointing only a life estate to a permissible object of the power. Thus, the appointment to John
is valid.

The language of Residuary Trust does not specify whether appointments in further trust (rather than
outright) are permissible. In light of the language in Testator’s will, applicants could argue that Carrie can
appoint only principal to John. This argument, however, is rejected by the Restatement of Property and
the weight of the modern case law, which take the position that, if a donee can appoint trust assets
(principal) outright, then the donee can also create more limited interests unless the evidence shows that
the donor intended otherwise. Likewise, the Restatement permits donees to appoint property in trust for
objects of the power. Therefore, the appointment of only a life estate to John is valid.

[NOTE: Local law may contain contrary authority.]

Explanation to Point-Two(b) (10-20%):

Carrie’s purported exercise of her power in favor of Charity is invalid because Charity is an impermissible
object of the power.

The donee of a special power of appointment (a power that excludes the donee, the donee’s creditors, and
the donee’s estate as permissible objects) can only exercise the special power in favor of objects
designated by the donor of the power. Here, the objects of the power were limited by Testator, the donor
of the power, to Carrie’s issue. Thus, Carrie’s attempted appointment to Charity is invalid.

Explanation to Point-Three (15-25%):

Because of Carrie’s partially ineffective exercise of the special power, the remainder of the new trust at
John’s death will pass to University, the taker in default under Testator’s will.

Carrie exercised her special power of appointment by attempting to appoint to both permissible (John)
and impermissible (Charity) objects. As a general rule, “the ineffectiveness of the appointments to the
non-objects does not affect the appointments to the objects.” Thus, the purported appointment to Charity
does not invalidate the otherwise effective appointment to John in trust. Where a power has been
ineffectively exercised, the property subject to the power not effectively exercised passes to the so-called
“taker in default of appointment” designated by the donor of the power (here Testator). Testator’s will
specifies that University is the taker in default of appointment and that University takes to the extent that
its interest has not been divested by an exercise of Carrie’s special power. Here, University has been
divested of the life estate effectively appointed to John but not the remainder interest ineffectively

Seperac-J19 Exam-Released MEE Essay Compilation © 2016-2020 1214


appointed to Charity. Thus, upon creation of the new trust John has a life estate and University has the
remainder. Charity has no interest in this trust.

Seperac-J19 Exam-Released MEE Essay Compilation © 2016-2020 1215


#284-JUL 2004–MEE Q01: QUESTION ONE (TRUSTS)

Five years ago, Settlor created an irrevocable trust (the “Settlor Trust”). The trust provided that Trustee
should pay annually “all of the trust income to my son Zack for life, with Zack to use such income to send
Zack’s children to college.” The trust instrument further provided that, upon Zack’s death, Trustee should
“distribute the trust corpus in equal shares to Zack’s children, issue of any deceased child to take his or
her parent’s share.”

When Settlor Trust was created, Zack was married to Spouse. Zack and Spouse had three living children,
Abel, age 23; Brian, age 19; and Carrie, age 15. Abel had one living child, Grandchild. Neither Brian, a
college student, nor Carrie, a high-school student, was married or had children.

Two years later, Zack and Spouse had another child, Debbie. Following Debbie’s birth, Zack stopped
paying Brian’s college expenses and told Carrie that he would not pay her future college expenses.

Trustee distributed last year’s trust income to Zack. Shortly thereafter, Abel, Carrie, and Zack were all
killed in an automobile accident. None of them had a will. Zack had not spent any of last year’s income
distribution. Zack was survived by Spouse, Brian, Debbie, and Grandchild.

Under state law, Abel and Carrie are deemed to have predeceased Zack. State law also provides that an
intestate’s estate passes to the intestate’s spouse and children in equal shares or, if there is no surviving
spouse or children, to the intestate’s parents in equal shares or, if there is no surviving parent, to the
intestate’s siblings.

1. Can Brian impress a trust upon the income distributed to Zack from Settlor Trust to pay for
Brian’s college education? Explain.

2. To whom, and in what shares, should the principal of Settlor Trust be distributed? Explain.

Seperac-J19 Exam-Released MEE Essay Compilation © 2016-2020 1216


#284: J04-1 MEE: ANSWER: NCBE (TRUSTS)

POINT (1) [32%] ISSUE: Did the language in Settlor Trust specifying that Zack “use . . . income to
send Zack’s children to college” create a trust in favor of Zack’s children? ANSWER: Yes. The
language of Settlor Trust probably creates a trust for the benefit of Zack’s children in the income of
the trust. Thus, Brian can impress a trust upon the income distributed to Zack from Settlor Trust.
It can be argued, however, that the college expenses condition was precatory.

POINT (2) [23%] ISSUE: Is Debbie, a child born to Zack after the creation of Settlor Trust,
included in the class gift in favor of Zack’s children? ANSWER: Yes. Debbie was born before the
class gift to Zack’s children closed and thus is a member of the class entitled to share in the trust
corpus.

POINT (3) [23%] ISSUE: Does the share of Settlor Trust that Abel would have taken had Abel
survived Zack pass to Abel’s children or to the surviving members of the class? ANSWER: Abel’s
share of the trust corpus passes to Grandchild as provided by the terms of Settlor Trust.

POINT (4) [23%] ISSUE: Does the share of Settlor Trust that Carrie would have taken had Carrie
survived Zack pass to Carrie’s heirs or to the surviving members of the class? ANSWER: Under
the common law, Carrie’s interest was vested. Her interest was not divested when she died before
Zack because she had no issue who survived Zack. Since Carrie died intestate, her share passed to
her mother, Spouse, who is her sole heir. Under the 1990 Uniform Probate Code, Carrie’s interest
would fail and go to Brian, Debbie, and Grandchild, the surviving members of the class.

ANSWER DISCUSSION:

If Settlor imposed a trust upon Zack to use trust income to send Zack’s children to college, Brian can
impress a trust upon the funds distributed to Zack from Settlor Trust. There is a strong, but not conclusive,
argument that Settlor did intend to impose a trust upon Zack. Upon Zack’s death, the trust corpus passed
to Brian, Spouse, Debbie, and Grandchild. The trust created a vested remainder in a class of persons
(Zack’s children) and that class remained opened until Zack’s death. Thus Debbie is included in the class.
Class members’ interests were divested only if they failed to survive Zack and had issue who did survive
him. Thus, Abel’s share was divested in favor of his issue (Grandchild). Carrie’s share was not divested
and passes to her heir, Spouse, except in a state that has adopted § 2-707 of the 1990 Uniform Probate
Code (UPC), or a like provision.

ANSWER EXPLANATION:

Explanation to Point-One (30-40%):

The language of Settlor Trust probably creates a trust for the benefit of Zack’s children in the income of
the trust. Thus, Brian can impress a trust upon the income distributed to Zack from Settlor Trust. It can be
argued, however, that the college expenses condition was precatory.

Settlor Trust provided that trust income be distributed to Settlor’s son Zack, with Zack “to use such
income to send Zack’s children to college.” The question is whether this language merely expressed
Settlor’s desire or imposed on Zack a binding obligation in trust. The answer depends on Settlor’s intent:
a trust is created “only if the settlor properly manifests an intention to create a trust relationship.”

Seperac-J19 Exam-Released MEE Essay Compilation © 2016-2020 1217


In order to determine a settlor’s intent, the courts consider:

(1) the specific terms and overall tenor of the words used; (2) the definiteness or indefiniteness of the
property involved; (3) the ease or difficulty of ascertaining possible trust purposes and terms, and the
specificity or vagueness of the possible beneficiaries and their interests; (4) the interests or motives and
the nature and degree of concerns that may reasonably be supposed to have influenced the transferor; (5)
the financial situation, dependencies, and expectations of the parties; (6) the transferor’s prior conduct,
statements, and relationships with respect to possible trust beneficiaries; (7) the personal and any
fiduciary relationships between the transferor and the transferee; (8) other dispositions the transferor is
making or has made of his or her wealth; and (9) whether the result of construing the disposition as
involving a trust or not would be such as a person in the situation of the transferor would be likely to
desire.

In cases where courts find “precatory language” not evidencing a trust, the settlor has typically used
words such as “like,” “request,” “hope,” or “wish” when stating what should be done with the funds.
Settlor Trust did not contain such precatory language; instead, it specified that Zack was to use trust
income for college expenses. The beneficiaries, purpose, and funds to be used can all be easily identified
from the trust instrument. Thus, there is a strong argument that a trust was imposed upon Zack to use trust
income for his children’s college expenses. If a trust was imposed on Zack, then Brian can impose a trust
on the income distributed to Zack before Zack died. It can also be argued that in light of the language in
Settlor Trust, Brian could impose a constructive trust on the income distributed to Zack; otherwise Zack
would be unjustly enriched.

The question could be argued the other way. Settlor was vague as to the amount to be spent on college,
the period for which college expenses were to be paid, etc. Furthermore, Zack was to receive all of the
income for life, including the period after which he had no children in college. This might be evidence
that the direction to use the income for the children’s college education was precatory.

Explanation to Point-Two (20-30%):

Debbie was born before the class gift to Zack’s children closed and thus is a member of the class entitled
to share in the trust corpus.

Settlor created a class gift in the corpus of Settlor Trust with such gift to become possessory upon Zack’s
death. A class remains open and may admit new members until at least one class member is entitled to
obtain possession of the gift. Because Settlor’s class gift did not become possessory until Zack’s death,
Debbie is a member of the class and entitled to take a share of the corpus. Debbie’s share of the corpus is
one-quarter for the reasons described in Points Three and Four.

Explanation to Point-Three (20-30%):

Abel’s share of the trust corpus passes to Grandchild as provided by the terms of Settlor Trust.

Settlor Trust expressly provided that the “issue of any deceased child take his or her parent’s share.” This
language in a trust instrument is typically interpreted to mean that, when the life tenant is predeceased by
a child who otherwise would have shared in the remainder and that child has issue who survive the life
tenant, the deceased child’s issue are to take the share that the deceased child would have taken had he
survived the life tenant. This interpretation seems consistent with Settlor’s intent and would result in
Grandchild taking Abel’s share.

Seperac-J19 Exam-Released MEE Essay Compilation © 2016-2020 1218


UPC § 2-707 and similar statutes achieve the same result by different means. Under the 1990 UPC, all
future interests in trust, whether or not the gift explicitly requires survival to take, are treated as if they
require survival; Abel’s interest is thus contingent on Abel’s surviving Zack. However, if, as here, the
governing instrument creates an alternative gift, then the takers of the alternative gift (the issue of Abel)
succeed to that interest despite the implied survival condition.

Explanation to Point-Four (20-30%):

Under the common law, Carrie’s interest was vested. Her interest was not divested when she died before
Zack because she had no issue who survived Zack. Since Carrie died intestate, her share passed to her
mother, Spouse, who is her sole heir. Under the 1990 Uniform Probate Code, Carrie’s interest would fail
and go to Brian, Debbie, and Grandchild, the surviving members of the class.

Under the common law, a remainder interest to children that was not expressly conditioned on survival
was not impliedly conditioned on survival. Here, the remainder to children was subject to a survival
contingency, but that contingency applied only when the deceased child had issue who survived the life
tenant. Thus in contrast to Abel’s interest, Carrie’s interest was not divested because the divesting
condition was not applicable as to her. As Carrie died intestate, her one-quarter interest would pass to
Spouse.

UPC § 2-707 and like statutes produce a different result. Under the 1990 UPC, Carrie’s interest is
impliedly conditioned on her being alive when Zack’s estate terminated. Because she was not alive, her
interest failed. And since she had no surviving issue, there is no alternative gift. Accordingly, the share to
which Carrie would have been entitled had she survived Zack goes to Brian, Debbie, and Grandchild, in
equal shares.

Seperac-J19 Exam-Released MEE Essay Compilation © 2016-2020 1219


#285-FEB 2004–MEE Q07: QUESTION SEVEN (TRUSTS)

Testator, a widower, died one year ago at the age of 86 survived by his only two children, Angela and
Brian (ages 40 and 45), twelve grandchildren (ranging in age from 17 to 27), and six great-grandchildren
(ranging in age from 2 to 7). Testator’s will bequeathed his entire estate to his “great-grandchildren living
when my will is probated.” Testator’s will was probated two months after Testator died.

Before he died, Testator created a revocable trust with Friend as trustee. Friend was a longtime confidante
of Testator and his family, with intimate knowledge of all of their financial and personal affairs. The trust
provided that upon Testator’s death the trust income could be paid to and among Testator’s issue in such
shares as Friend determined, with any unpaid income to be accumulated. It also provided that upon the
death of Angela and Brian, the trust would terminate, and the corpus would be distributed to Testator’s
then living issue.

The trust contained no provisions relating to its administration. Because Friend was not very experienced
in the administration of trusts and particularly in selecting appropriate trust investments, she interviewed a
number of possible bank trust officers to assist her in administering the trust. Then she contracted with
Bank, an institution wholly unfamiliar with Testator’s family, to make all determinations regarding the
distribution of trust income and how the trust’s assets should be prudently invested.

Bank immediately sold all of the trust assets and reinvested all of the proceeds in XYZ Corporation, a
telecommunications company whose shares are publicly traded. Bank paid $60 per share for this stock.
Today, however, each share is worth only $21. The loss in value is due to the general decline in the
telecommunications industry and not to any inherent weaknesses in XYZ.

Twice, after contracting with Bank, Friend called Bank to see how things were going. On these two
occasions Friend was advised that “all is well.” Friend never made further inquiries.

The common-law Rule Against Perpetuities, as modified by the wait-and-see doctrine, applies in this
jurisdiction.

1. To whom should Testator’s probate estate be distributed? Explain.

2. What fiduciary duties, if any, did Friend breach with respect to the trust? Explain.

Seperac-J19 Exam-Released MEE Essay Compilation © 2016-2020 1220


#285: F04-7 MEE: ANSWER: NCBE (TRUSTS)

POINT (1) [24%] ISSUE: Does the bequest under Testator’s will violate the Rule Against
Perpetuities? ANSWER: The bequest under Testator’s will would be void under the common-law
Rule Against Perpetuities. However, the bequest is valid under the governing wait-and-see doctrine
because it vested two months after Testator’s death. Thus, the estate passes to Testator’s great-
grandchildren who are alive when the will is probated.

POINT (2) [38%] ISSUE: Did Friend improperly delegate her duties to Bank? ANSWER: A trustee
is responsible for administering the trust. Accordingly, at common law it is a breach of trust to
delegate to a third party significant and discretionary duties, such as the duties to make
distributions and investments. Under one or more Uniform Acts, however, at least the trustee’s
investment duty is delegable to a third party, although a breach of trust for that delegation could
nonetheless occur for failing to properly supervise the third party.

POINT (3) [38%] ISSUE: Did Bank, acting as agent of Friend, improperly invest the trust assets by
failing to diversify the trust investments such that Friend, as trustee, is liable for any losses
resulting from such failure? ANSWER: Yes. Friend probably acted imprudently with respect to
the trust investments by failing in her oversight responsibilities to require that the trust investments
be diversified.

ANSWER DISCUSSION:

The bequest to great-grandchildren under Testator’s will is invalid under the common-law Rule Against
Perpetuities but valid under the wait-and-see doctrine because it actually vested before the perpetuity
period had expired. Friend, as trustee of the revocable trust, breached the duty not to delegate and the duty
to diversify trust investments and is liable for the resulting losses.

ANSWER EXPLANATION:

Explanation to Point-One (20-30%):

The bequest under Testator’s will would be void under the common-law Rule Against Perpetuities.
However, the bequest is valid under the governing wait-and-see doctrine because it vested two months
after Testator’s death. Thus, the estate passes to Testator’s great-grandchildren who are alive when the
will is probated.

Under the common-law Rule Against Perpetuities, “no interest is good unless it must vest, if at all, no
later than twenty-one years after some life in being at the creation of the interest.” Furthermore, under the
common-law rule, if there was any possibility, however improbable, that an interest might vest too
remotely, that interest would be invalid even though in fact it actually vested in a timely manner.

For testamentary bequests, as here, the nonvested interest is deemed created at Testator’s death. Thus, the
testamentary gift to great-grandchildren is void if there is any possibility it could become possessory more
than 21 years after the death of any of Testator’s issue who survived him, a possibility that could occur
here. For example, within one year after Testator died all of his issue who survived him except the 27-
year-old grandchild could die. While that is improbable, it is certainly possible. Then the 27-year-old
grandchild who survived Testator could have a child who would be a great-grandchild of Testator. And,

Seperac-J19 Exam-Released MEE Essay Compilation © 2016-2020 1221


then the 27-year-old grandchild could die. At this point, all of Testator’s issue who survived him are dead,
leaving only the later born great-grandchild. Then, 22 years later (more than 21 years after the death of all
relevant lives) Testator’s will could be probated. This would result (but for the Rule) in the gift vesting in
Testator’s great-grandchildren living when his will was probated. However, since this vesting occurs, as
hypothesized, beyond the permissible period, the gift is invalid. It bears repeating that a nonvested interest
is invalid if it might have vested too remotely even if, in fact, as here, that did not occur because
Testator’s will was probated two months after Testator died.

An invalid residuary gift is stricken from the will, and absent an alternative valid gift, the residue passes
to the Testator’s heirs, here Angela and Brian, his only two children.

This jurisdiction, however, has enacted the wait-and-see rule. Under the wait-and-see rule, the gift to the
great-grandchildren is valid if it in fact vests within the perpetuity period. Here the will was probated two
months after Testator died. Thus, the gift to the great-grandchildren vests in a timely manner.

Explanation to Point-Two (35-45%):

A trustee is responsible for administering the trust. Accordingly, at common law it is a breach of trust to
delegate to a third party significant and discretionary duties, such as the duties to make distributions and
investments. Under one or more Uniform Acts, however, at least the trustee’s investment duty is
delegable to a third party, although a breach of trust for that delegation could nonetheless occur for failing
to properly supervise the third party.

The selection by the settlor of a trust of another person to act as a trustee evidences the settlor’s trust and
confidence in the designated trustee to properly administer the trust, including deciding what, if any,
discretionary distributions of trust property should be made to beneficiaries, and how trust assets should
be invested. Flowing from this confidence is the duty to exercise due care and the duty not to delegate
those duties that the trustee can reasonably be expected to perform.

According to the Restatement, a trustee cannot “delegate to another power to select investments.”
According to Scott, “if the trustee entrusts funds to the agent for this purpose and through the negligence
of the agent the funds are lost, the trustee is personally liable.”

Similarly, Friend, as trustee, cannot delegate such an important function as determining how the trust
property shall be distributed among the named beneficiaries, because this is an act that, in light of all the
circumstances, it would appear Testator expected Friend to perform. Thus, Friend’s discretionary power,
as trustee, to distribute trust principal to the income beneficiaries cannot be delegated to Bank.

Under the Uniform Trust Code, however, a trustee can delegate such duties that “a prudent trustee of
comparable skills could properly delegate under the circumstances.” Likewise, under § 9 of the Uniform
Prudent Investor Act and § 3 of the Uniform Trustee’s Powers Act, a trustee is given broad authority to
delegate trust duties, effectively abrogating the common-law and Second Restatement nondelegation rule,
at least where a delegation would be deemed prudent under the circumstances. Nonetheless, it is highly
unlikely that a prudent trustee who was a longtime friend of Testator’s family and who presumably best
knew how to make discretionary distributions among Testator’s issue would delegate that function to a
corporate agent that had no familiarity with Testator’s family.

On the other hand, the Uniform Trust Code and Uniform Prudent Investor Act clearly contemplate the
complete delegation of the trustee’s investment responsibilities so long as the trustee exercises reasonable
care, skill, and caution in selecting the agent and “periodically reviews the agent’s actions in order to

Seperac-J19 Exam-Released MEE Essay Compilation © 2016-2020 1222


monitor the agent’s performance and compliance with the terms of the delegation.” Here, there may have
been a proper delegation but there is also lack of supervision, which brings Friend outside of the
protection of the Third Restatement’s black letter rule.

Explanation to Point-Three (35-45%):

Friend probably acted imprudently with respect to the trust investments by failing in her oversight
responsibilities to require that the trust investments be diversified.

“A trustee shall diversify the investments of the trust unless the trustee reasonably determines that,
because of special circumstances, the purposes of the trust are better served without diversifying.” This
rule reflects the time-honored principle that it is inappropriate to put all of your eggs into one basket.
While in a limited number of instances diversification may not be necessary, none of them appear relevant
here.

The rationale for diversification is clear. “Diversification reduces risk because stock price movements are
not uniform. They are imperfectly correlated. This means that if one holds a well-diversified portfolio, the
gains in one investment will cancel out the losses in another.”

Even though it was Bank that failed to diversify, Friend, as trustee, is liable for this failure because (1)
Bank was Friend’s agent and (2) Friend, as trustee, had a duty to oversee the acts of the agent, including
the duty to assure that investments were made in a prudent manner. Under the Uniform Prudent Investor
Act, a trustee must exercise care, skill and caution in periodically reviewing the agent’s actions in order to
monitor the agent’s performance and compliance with the terms of the delegation. Friend can be liable for
the failure to diversify even though Friend committed no breach of trust by initially delegating her
investment duties to Bank.

Seperac-J19 Exam-Released MEE Essay Compilation © 2016-2020 1223


#286-FEB 2003–MEE Q06: QUESTION SIX (TRUSTS)

Decedent died one year ago. Decedent’s duly probated will created a $1 million trust. Trustee is the
trustee of this trust. The trust provides that:

Trust income shall be payable annually to my son Adam for 10 years. Adam’s interest shall be free from
control, debts, liabilities, and assignments by Adam and shall not be subject to execution or process for
the enforcement of judgments or claims of any sort against Adam. After 10 years, I direct that the income
be paid for 5 years to Charity, a charitable organization. After this 5-year period, the trust will terminate
and the principal shall be paid to my daughter Beth.

Decedent was survived by Adam and Beth. Adam had a history of lavish spending, which Decedent
deplored. Beth was very careful in her financial dealings.

Three months ago, Susan, Adam’s former spouse, gave Trustee a copy of a judgment for alimony she had
obtained against Adam and proof that Adam had failed to pay her the required $5,000. Susan demanded
that Trustee pay her $5,000, which was less than the trust’s annual income.

Two months ago, Beth gave Trustee a copy of a tort judgment that John had obtained against Beth. She
then requested that Trustee pay John $10,000, the amount of that judgment, from the trust principal.

Last month, Adam, Beth, and Charity commenced an appropriate judicial action to terminate the trust and
have the trust assets distributed to them. Trustee has filed appropriate objections in this action.

1. Is Susan entitled to be paid $5,000 from the trust income? Explain.

2. Can Trustee properly pay John $10,000 from the trust principal? Explain.

3. Should the court terminate the trust and distribute the trust assets to Adam, Beth, and Charity?
Explain.

Seperac-J19 Exam-Released MEE Essay Compilation © 2016-2020 1224


#286: F03-6 MEE: ANSWER: NCBE (TRUSTS)

POINT (1) [33%] ISSUE: Should Trustee distribute $5,000 from the trust income to Susan in
payment of her unpaid alimony claim? ANSWER: Yes. Trustee should distribute $5,000 from trust
income to Susan in payment of her unpaid alimony claim notwithstanding that Adam’s interest is
subject to a spendthrift clause.

POINT (2) [33%] ISSUE: Should Trustee distribute $10,000 from the trust principal to John in
payment of his tort judgment against Beth? ANSWER: No. Trustee cannot properly pay $10,000 to
John from the trust principal in payment of his tort judgment against Beth because Beth’s interest
is a remainder interest.

POINT (3) [33%] ISSUE: Should the court terminate the trust and distribute the trust assets to
Adam, Beth, and Charity? ANSWER: No. The court should refuse to terminate the trust and
should not distribute the trust assets to the beneficiaries.

ANSWER DISCUSSION:

Generally, an income interest subject to a spendthrift clause is not available for payment of claims against
the income beneficiary. However, many states, for public policy reasons, do not apply this rule to unpaid
alimony claims. In such states, Trustee should pay Susan’s $5,000 claim. On the other hand, Trustee
cannot pay John’s claim from the trust because Beth is not entitled to any principal until the trust
terminates and a payment to John would harm both Adam and Charity. Lastly, the court should refuse to
terminate the trust because termination would be inconsistent with Decedent’s intent as evidenced by the
presence of a spendthrift clause in the trust instrument.

ANSWER EXPLANATION:

Explanation to Point-One (30-40%):

Trustee should distribute $5,000 from trust income to Susan in payment of her unpaid alimony claim
notwithstanding that Adam’s interest is subject to a spendthrift clause.

The testamentary trust created by Decedent contains language that clearly indicates Decedent’s intention
to subject Adam’s 10-year term interest to a spendthrift restriction. Spendthrift clauses are widely
recognized. The effect of a spendthrift clause is to bar a creditor from reaching the beneficiary’s interest
in satisfaction of the creditor’s claim.

However, there are exceptions to the general rule for public policy purposes. A well-established exception
enables a former spouse with an unpaid alimony judgment to reach a spendthrift trust interest of his or her
former spouse. This exception recognizes a strong public policy against allowing a trust beneficiary to
enjoy a trust interest while neglecting to pay the court-ordered support of the beneficiary’s former spouse.
Accordingly, Trustee should pay Susan $5,000 to satisfy her unpaid alimony judgment against Adam.

[NOTE: Even though Susan is entitled to receive $5,000 from the trust income, Trustee, as a practical
matter, should not make that distribution to her without an authorizing court order. This protects Trustee

Seperac-J19 Exam-Released MEE Essay Compilation © 2016-2020 1225


from any possible liability for having made an inappropriate distribution to Susan. Also, if the jurisdiction
does not adhere to the alimony exception, Trustee should distribute nothing from the trust to Susan.]

Explanation to Point-Two (30-40%):

Trustee cannot properly pay $10,000 to John from the trust principal in payment of his tort judgment
against Beth because Beth’s interest is a remainder interest.

The testamentary trust created by Decedent contains no language evidencing Decedent’s intent to subject
Beth’s remainder interest to a spendthrift restriction. Absent such a restriction, Beth’s interest would
generally be alienable and reachable by her creditors.

However, because Beth is a remainder beneficiary and not an income beneficiary, she has no immediate
right to the possession and enjoyment of any trust property. Rather, she must await the termination of the
trust to receive any trust property. John, as her creditor, can have no greater rights in the trust property
than she had. As her creditor he simply steps into her shoes. Thus, he cannot obtain possession of her
share any earlier than she could have obtained possession of it. Payment of trust principal to him at this
time, therefore, would be premature.

Furthermore, if Trustee gave John any trust principal at this time, the rights of both Adam and Charity
would be adversely affected since the income to which they are and will be entitled is generated from the
principal. Any payment to John, therefore, would reduce the future income flow from the trust.

Accordingly, Trustee should not pay John’s $10,000 claim from the trust principal.

Explanation to Point-Three (30-40%):

The court should refuse to terminate the trust and should not distribute the trust assets to the beneficiaries.

It is a well-established rule that a testamentary trust can be terminated by a court upon the request and
consent of all trust beneficiaries unless a material purpose remains to be accomplished. The bar against
termination when a material purpose remains is referred to as the Claflin doctrine and is based on the
seminal case, Claflin v. Claflin. The Claflin doctrine assures that a trust will not be terminated when
termination would be inconsistent with the grantor’s intent.

Accordingly, the question is whether there is a material purpose of the trust that would be defeated if the
trust were to terminate. The traditional view is that a material purpose remains if any trust interest is
subject to a spendthrift restriction that bars alienation of the spendthrift interest. Termination is
inappropriate because, if trust assets are distributed to a beneficiary of a spendthrift interest, the
beneficiary could later alienate the property. Under this view, the court should refuse to terminate the trust
and should not order Trustee to distribute the trust assets to the beneficiaries.

Even under the third Restatement, the court should refuse to terminate the trust. This Restatement rejects
the per se prohibition on trust termination where there is a spendthrift clause. It requires a finding that the
trust grantor really intended the spendthrift provision to bar premature trust termination. Under this test, a
spendthrift clause inserted in a trust as mere boilerplate might not bar a requested termination when all
trust beneficiaries consent to the termination. Under the Uniform Trust Code, a spendthrift clause is not
presumed to constitute a material purpose of the trust. Here, however, the facts clearly indicate that
Decedent would not have wanted Adam to prematurely reach his interest due to Decedent’s unhappiness
with Adam’s lavish spending, and therefore the court should not terminate the trust.

Seperac-J19 Exam-Released MEE Essay Compilation © 2016-2020 1226


#287-JUL 2002–MEE Q07: QUESTION SEVEN (TRUSTS)

On February 5, 1999, Testator created a revocable inter vivos trust. This trust was validly executed. Bank
was designated trustee of the revocable trust.

Under the terms of the trust, Testator retained all income for life. The trust then provided in Article II that
"upon my death the principal shall be held in further trust with the income payable to my wife, Wanda, for
life, remainder to my children."

Contemporaneous with the creation of the revocable trust, Testator validly executed a will devising
Testator's entire probate estate to the trustee of the revocable trust to be disposed of as part of that trust.
The will further provided that, if the trust was revoked prior to Testator's death, Testator's entire estate
should pass to Wanda.

On March 1, 2000, Testator sent a validly executed trust amendment to Bank revoking Article II of the
revocable trust and substituting for it a new Article II. This new article stated that "upon my death the
principal shall be held in further trust with the income payable to my wife, Wanda, for the immediate two
years after my death, and, at the end of the two-year period, the trust principal shall be distributed to my
surviving children."

In March 2001, Testator and Wanda were divorced.

In March 2002, Testator died, leaving a probate estate of $100,000. The revocable trust had not been
funded prior to Testator's death. Testator was survived by Wanda and by their three children, Adam, Ben,
and Carrie.

Two months later, both Wanda and Adam died in an automobile accident. Under Wanda's probated will,
her entire estate passed to a charitable institution, Hope, and under Adam's probated will, his entire estate
passed to University. Ben and Carrie are alive. They each have one child who also survived Testator.
Adam never had any children.

1. Which instruments control the disposition of the property included in Testator's probate estate?
Explain.

2. Upon Testator's death, what interest, if any, did Wanda have under the revocable trust? Explain.

3. Assuming that Wanda had an interest under the trust, what rights, if any, do Hope and University
have under the revocable trust upon the deaths of Wanda and Adam? Explain.

Seperac-J19 Exam-Released MEE Essay Compilation © 2016-2020 1227


#287: J02-7 MEE: ANSWER: NCBE (TRUSTS)

POINT (1)(a) [18%] ISSUE: Can Testator’s probate estate be disposed of according to the terms of
a revocable inter vivos trust that was not executed in accordance with the Statute of Wills?
ANSWER: Yes. Under the Uniform Testamentary Additions to Trust Act, a will may pour over the
probate estate assets to the trustee of an unfunded inter vivos trust even if the trust instrument was
not executed in accordance with the Statute of Wills.

POINT (1)(b) [18%] ISSUE: Assuming the probate estate assets can be disposed of as a part of the
revocable trust, do the terms of the trust in existence at the time the will was executed, or rather
those as reflected in the subsequent amendment, apply? ANSWER: Under the Uniform
Testamentary Additions to Trust Act, the disposition of the probate estate assets are governed by
the terms of the revocable trust, including all amendments to the trust.

POINT (2) [26%] ISSUE: Are the terms of the trust in favor of Wanda, Testator’s former wife,
revoked by operation of law because of their divorce? ANSWER: No. Under the laws of most states
today, Wanda’s two-year term interest is not revoked by the divorce. Therefore, the trustee should
hold the trust property and pay her the income for the next two years in accordance with the
amended trust terms.

POINT (3) [38%] ISSUE: Assuming the terms of the trust amendment apply, what is Hope’s
interest under the revocable trust? ANSWER: Assuming Wanda had an interest in the trust, it
would pass under her will to Hope since an interest for a two-year term does not terminate at death.
Whether University has an interest in the trust depends on whether the phrase “surviving children”
means surviving Testator or surviving the two-year period set aside to pay the income to Wanda.

ANSWER DISCUSSION:

The disposition of Testator’s estate is governed by the will and the terms of the revocable trust as
amended. Absent a governing statute or case law to the contrary, divorce does not revoke the provisions
of a revocable trust in favor of a former spouse. Furthermore, if that interest is alienable and devisable (as
is here the term interest in income), upon Wanda’s death it would pass to Hope, the legatee under
Wanda’s will. As for Adam’s interest in the remainder, which is limited to “surviving children,” if
governing law requires him to be alive when Wanda’s two-year term ends, his interest fails since he died
before the term ended. The remainder, thus, passes wholly to Testator’s surviving children, Ben and
Carrie.

ANSWER EXPLANATION:

Explanation to Point-One(a) (15-20%):

Under the Uniform Testamentary Additions to Trust Act, a will may pour over the probate estate assets to
the trustee of an unfunded inter vivos trust even if the trust instrument was not executed in accordance
with the Statute of Wills.

At common law, the terms of the revocable trust could control the disposition of the testator’s probate
estate under the doctrine of incorporation by reference. Under this doctrine, if the will referred to an

Seperac-J19 Exam-Released MEE Essay Compilation © 2016-2020 1228


unattested document that was in existence at the time the will was signed, the terms of that document
could be given effect in the same manner as if it had been properly executed.

This doctrine is no longer necessary to validate the so-called “pour over” will because of the almost
universal enactment of the Uniform Testamentary Additions to Trust Act (UTATA). Under UTATA, a
trust, even an unfunded trust, as here, can be the beneficiary of the testator’s probate estate so long as the
trust is identified in the testator’s will and its terms are set forth in a written instrument. The validity of
the pour over arrangement is unaffected by the fact that the trust was amended after the execution of the
will.

Explanation to Point-One(b): (15-20%):

Under the Uniform Testamentary Additions to Trust Act, the disposition of the probate estate assets are
governed by the terms of the revocable trust, including all amendments to the trust.

Under the incorporation by reference doctrine, the terms of the amended revocable trust would not apply
to the disposition of the probate estate assets as that amendment was not in existence when the will was
executed. Under the Uniform Testamentary Additions to Trust Act, however, the amendment applies.
Thus, as of Testator’s death, the probate estate assets are to be held by Bank in trust to pay the income to
Wanda for two years, and then to distribute the trust principal to Testator’s surviving children.

Thus, the will and amended trust govern the disposition of Testator’s estate.

Explanation to Point-Two (20-30%):

Under the laws of most states today, Wanda’s two-year term interest is not revoked by the divorce.
Therefore, the trustee should hold the trust property and pay her the income for the next two years in
accordance with the amended trust terms.

Many probate codes provide that if, subsequent to the execution of a will, the testator and the spouse are
divorced, provisions in a will in favor of the former spouse are automatically revoked by operation of law.
In such case, the property that would have passed to the spouse is disposed of as if the spouse had
predeceased the decedent. If Testator had created a testamentary trust, then under such a statute, Wanda’s
two-year income interest would have been revoked by operation of law.

Here, however, Wanda’s interest is created under the terms of the revocable trust. Typically, state statutes
affecting a divorced spouse’s interest under a will are inapplicable to interests created under a revocable
trust and, therefore, Wanda’s interest is not revoked.

However, the law in this regard is slowly changing. For example, § 2-804 of the Uniform Probate Code
would cause Wanda’s two-year interest in the revocable trust to be revoked upon her divorce from
Testator. Some states have judicially reached that result by viewing the will and revocable trust as
integrated estate planning documents and treating the probate statute’s revocation of the spouse interest
upon divorce as reaching the trust as well. Of course, if her interest is revoked, then nothing passes to
Wanda under the trust after Testator’s death.

Explanation to Point-Three (35-40%):

Assuming Wanda had an interest in the trust, it would pass under her will to Hope since an interest for a
two-year term does not terminate at death. Whether University has an interest in the trust depends on

Seperac-J19 Exam-Released MEE Essay Compilation © 2016-2020 1229


whether the phrase “surviving children” means surviving Testator or surviving the two-year period set
aside to pay the income to Wanda.

Trust interests are alienable, devisable, and descendible unless the terms of a trust expressly or impliedly
provide otherwise.

Here, Wanda has a two-year term, which is devisable in the event she were to die within that period. Since
Wanda has died prior to the expiration of the two-year term, her interest passes to Hope, the sole
beneficiary of her estate.

University has an interest only if Adam had an interest. Whether Adam had an interest depends on
whether the word “surviving” in the gift to Testator’s “surviving children” means “surviving Testator” or
“surviving the two-year period.”

The preferred rule is to construe the word to mean surviving to the time of distribution. Under this rule,
only those children of Testator alive two years after his death are entitled to the trust principal. This
would, of course, exclude Adam; if Adam has no interest to devise, then University, the beneficiary under
Adam’s will, would have no interest in the trust either.

On the other hand, in some states, surviving refers to surviving Testator. Under this interpretation, each of
Testator’s three children – Adam, Ben, and Carrie – had vested interests in the trust principal as of
Testator’s death. Vested interests are not forfeited even if the beneficiary dies within two years of
Testator. Given that trust interests are devisable, the vested interest of Adam would pass under his will
solely to University, at least under the common law. Construing “surviving” to mean surviving Testator
results in the trust remainder vesting at the earliest time.

Under the Uniform Probate Code, the analysis would be somewhat different. Section 2-707(b)(3) provides
that the word “surviving” does not evidence any intent that § 2-707 not apply, as that word often does
under anti-lapse statutes on which § 2-707 is modeled. Under § 2-707(b), if a class gift is limited in favor
of a class of children, only those children alive at the time of distribution are entitled to possession of the
property (Ben and Carrie). However, if a deceased child left surviving issue, such issue would take the
deceased child’s share. Here, because Adam had no such descendants, the general rule applies and only
those children of Testator alive at the time of distribution (Ben and Carrie) are entitled to possession.
Under the Code, therefore, University would have no interest.

Seperac-J19 Exam-Released MEE Essay Compilation © 2016-2020 1230


#288-JUL 2001–MEE Q02: QUESTION TWO (TRUSTS)

Harry, a widower, and Wanda, residents of State A, decided to marry. Prior to the wedding ceremony,
they signed a prenuptial agreement in which Wanda waived her right to receive alimony in the event of a
divorce. The agreement further stated that “in consideration of this waiver, Harry shall establish an inter
vivos trust of the first $1 million he inherits from his mother, with himself as trustee to pay the income to
Wanda for life.” They agreed that Harry was free to designate any person to take the trust property at
Wanda’s death.

Three years later, Harry’s mother died, leaving her substantial estate to him. In addition to Harry, her only
surviving relatives were Charles, who is Harry’s child from his first marriage, and Mary and Pat, Harry’s
two nieces.

Two months after his mother died, Harry orally declared himself trustee of $1 million. In making this oral
declaration Harry stated: “I will pay all trust income to my wife, Wanda, for her life and, when she dies, if
I don’t have any issue who survive me, then I or my successor should distribute the trust principal to my
surviving nieces and nephews.” Harry did not state who would succeed him as the trustee if he died prior
to the termination of trust.

Harry died two months ago at his home in State A. Until his death, he faithfully paid the trust income to
Wanda. Harry was survived by Wanda and by Charles, Mary, and Pat. One month after Harry died,
Wanda, who had recently inherited substantial property from her father, disclaimed her income interest in
the trust property.

Under the laws of State A, Wanda’s disclaimer of her interest in the trust property is valid. In addition,
under the intestacy laws of State A, any property distributable as part of Harry’s probate estate passes
one-third to Wanda and two-thirds to Charles.

1. Was Harry’s promise in the prenuptial agreement to create a trust in the future for Wanda legally
enforceable? Explain.

2. Was the trust Harry later created for Wanda validly created? Explain.

3. Assuming the trust for Wanda was validly created, did the trust terminate at Harry’s death
because there was then no trustee? Explain.

4. Assuming the trust for Wanda was validly created, what is the effect of her valid disclaimer, and
should the trust principal be immediately distributed and, if so, to whom? Explain.

Seperac-J19 Exam-Released MEE Essay Compilation © 2016-2020 1231


#288: J01-2 MEE: ANSWER: NCBE (TRUSTS)

POINT (1) [31%] ISSUE: Is a promise to create a trust in the future legally enforceable if the
promise is supported by consideration? ANSWER: Yes. Harry’s promise to create a trust of
property he might inherit from his mother was legally enforceable.

POINT (2) [31%] ISSUE: Is an oral trust of intangible personal property valid? ANSWER: Yes. A
trust of intangible personal property may be declared orally. No writing is required. Thus, the trust
for Wanda was validly created.

POINT (3) [7%] ISSUE: Did the trust terminate at Harry’s death because there was then no
trustee? ANSWER: No. A trust will not fail for want of a trustee. Thus, even though no trustee is
acting at Harry’s death, the trust is valid and a court of equity can appoint a successor trustee.

POINT (4) [31%] ISSUE: What is the effect of Wanda’s disclaimer of the income interest and
should the principal be immediately distributed (a) to Wanda and Charles, as Harry’s heirs, (b) to
Charles, as the implied remainder beneficiary of the trust, or (c) to Mary and Pat, as remainder
beneficiaries of the trust? ANSWER: Because Wanda has validly disclaimed her income interest in
the trust, it is appropriate to accelerate the distribution of the trust principal under these facts and
make an immediate distribution to the persons entitled to the trust principal without awaiting
Wanda’s death. However, it is unclear whether there is an implied gift to Harry’s issue (Charles in
this case). If so, Charles is entitled to the trust principal. If not, the trust principal is distributable to
Harry’s heirs (i.e., one-third to Wanda and two-thirds to Charles).

ANSWER EXPLANATION:

Explanation to Point-One (30-40%):

Harry’s promise to create a trust of property he might inherit from his mother was legally enforceable.

A promise to create a trust in the future is unenforceable unless the promise is supported by consideration
sufficient for the formation of a contract. Thus, if Harry had merely promised to create a trust of property
he might later inherit from his mother, that promise would be unenforceable. Here, however, Harry’s
promise was incorporated into a prenuptial agreement in which the promise to create the trust was made
in consideration of Wanda’s waiver of alimony rights. This waiver is sufficient consideration to support
Harry’s promise to create the trust in the future, just as it is sufficient to support the validity of the
prenuptial agreement.

Explanation to Point-Two (30-40%):

A trust of intangible personal property may be declared orally. No writing is required. Thus, the trust for
Wanda was validly created.

In order to create a trust, the grantor must have intended to create the trust and there must be a trust res
(property), one or more trust beneficiaries, and a trustee. The intent to create a trust can be found in both
the grantor’s words and deeds. Here, the evidence is clear that Harry intended to create a trust.

Seperac-J19 Exam-Released MEE Essay Compilation © 2016-2020 1232


At the time of the declaration of trust there was also trust property ($1 million), one or more beneficiaries,
and a trustee (Harry). Thus, the only possible remaining issue relating to whether the trust was validly
created is whether an oral declaration of trust is valid. Most states require a writing to create an inter vivos
trust of land. At common law, an inter vivos oral trust of personalty was valid. Generally, state statutes of
frauds do not change this common-law rule. Thus, the trust was validly created even though created
orally.

Explanation to Point-Three (05-10%):

A trust will not fail for want of a trustee. Thus, even though no trustee is acting at Harry’s death, the trust
is valid and a court of equity can appoint a successor trustee.

A trust will not fail for want of a trustee. Thus, even if the sole individual acting as trustee dies, the trust
does not terminate if the grantor intended the trust to continue beyond the death of the trustee. Rather, the
court of equity having jurisdiction over the trust can appoint another to act as a successor trustee. Here,
Harry’s intent is clear – that the trust continue for Wanda’s lifetime, not that the trust terminate at his
death. This intent is evidenced not only by the provision that Wanda’s income interest terminates at her
death but also by the language of the orally declared trust specifically contemplating the appointment of a
successor trustee. Thus, it is entirely appropriate for the court to appoint a successor trustee.

If the trust terminates as a result of Wanda’s disclaimer, it would still be necessary to appoint a successor
trustee to prepare the necessary accounting and distribute the trust principal to the correct beneficiaries.
However, a court may simply direct the personal representative of Harry’s estate to assume these duties.

Explanation to Point-Four (30-40%):

Because Wanda has validly disclaimed her income interest in the trust, it is appropriate to accelerate the
distribution of the trust principal under these facts and make an immediate distribution to the persons
entitled to the trust principal without awaiting Wanda’s death. However, it is unclear whether there is an
implied gift to Harry’s issue (Charles in this case). If so, Charles is entitled to the trust principal. If not,
the trust principal is distributable to Harry’s heirs (i.e., one-third to Wanda and two-thirds to Charles).

Under the doctrine of acceleration, if the income beneficiary of a trust disclaims his or her interest, the
trust principal becomes immediately distributable to the presumptive remainder beneficiaries of the trust
provided no one would be harmed by making a distribution to them earlier than it would have been made
had the income beneficiary not disclaimed. For example, a remainder might not accelerate if to do so
would result in the class gift to the remainder beneficiaries closing earlier than it otherwise would.

In this case, however, no one would be harmed by an acceleration because the identity of the persons
entitled to the trust principal is not dependent on whether the distribution of the remainder accelerates or
must await the death of Wanda. Rather, the ultimate identity of the takers of the trust principal depends on
whether there is an implied gift to Harry’s issue who survived Harry. At the time the trust was created,
Harry clearly specified what was to happen if he died without issue. He stated that in such case the
principal should be distributed to his nieces and nephews – Mary and Pat. Thus, their interest could vest
only if Harry died without issue. Because Harry died with issue, the gift to the nieces and nephews clearly
fails.

Harry, however, failed to state who would take the trust property under the circumstances that actually
occurred – if he died with issue. In such a case, the courts have split between inferring a gift to the issue
(i.e., construing the trust to mean “upon Wanda’s death, to Harry’s issue but if Harry dies without issue,

Seperac-J19 Exam-Released MEE Essay Compilation © 2016-2020 1233


to Harry’s surviving nieces and nephews”) or finding that the gift of the remainder interest failed,
resulting in the trust property reverting to the grantor’s estate as undisposed of property. Theoretically, the
controlling principle in construing the ambiguous instrument is to give effect to Harry’s intent. Often,
however, the grantor’s intent is not readily determinable.

Some courts in these cases infer a gift to issue because “it is difficult to see why the grantor has made no
provision for his issue although he has expressly provided for his death without issue.” Thus, the court
assumes that the intended gift to issue was inadvertently omitted and, therefore, constructs the gift to
accomplish the grantor’s presumed intent.

Other courts hold that the trust property is distributed to the successors of the grantor’s estate on the
theory that the grantor retained a reversion and that to construct a gift to issue is tantamount to writing a
trust for the grantor and making a gift that is wholly speculative. If the trust property is distributable
through Harry’s estate by way of a reversion, it passes to his heirs via intestate succession, and it is
distributable one-third to Wanda and two-thirds to Charles. Because Wanda’s disclaimer was limited to
her interest in the trust property, the disclaimer did not apply to any property distributable to her as part of
Harry’s estate.

Seperac-J19 Exam-Released MEE Essay Compilation © 2016-2020 1234


#289-FEB 2001–MEE Q06: QUESTION SIX (TRUSTS)

In 1998, Grantor created an irrevocable inter vivos trust naming Bank as trustee. The trust directed the
trustee to pay the income to Grantor’s child, Son, for life. The instrument further provided that upon
Son’s death, the trust principal should be distributed in equal shares to Son’s surviving children. The trust
instrument did not authorize the trustee to invade the trust principal for the benefit of Son.

At the time the trust was created, Son was divorced and was in some financial difficulties. Accordingly,
Grantor included the following clause in the trust instrument:

The interest of any trust beneficiary, whether in the income or principal of the trust, shall not be capable
of assignment, anticipation, or seizure by legal process. In particular, none of the trust income shall be
used to satisfy any claim against any beneficiary for unpaid alimony or child support.

The trust principal consisted of 500 shares of XYZ Corporation common stock.

Prior to 1996, Son’s former spouse, Wife, had obtained a judgment against him for $10,000 for unpaid
alimony. In 1997, Creditor had obtained a judgment against Son for $20,000 for money it lent to Son to
finance Son’s failed business venture. Wife and Creditor seek to enforce their respective judgments
against Son’s interest in the trust.

In 2000, XYZ Corporation, which typically distributed dividends in cash, paid the trustee a stock dividend
of one-tenth of a share of stock for each share the trust owned.

1. Can Creditor compel Bank to pay its claim against Son from either the trust’s income or
principal? Explain.

2. Can Wife compel Bank to pay her claim for unpaid alimony from either the trust’s income or
principal? Explain.

3. Can Son be compelled to pay the claims of Wife or Creditor from the trust income that is
actually distributed to him from the trust? Explain.

4. Should Bank distribute the stock dividend to Son as income or retain the dividend as part of the
trust principal? Explain.

Seperac-J19 Exam-Released MEE Essay Compilation © 2016-2020 1235


#289: F01-6 MEE: ANSWER: NCBE (TRUSTS)

POINT (1) [35%] ISSUE: Can judgment claims against the income beneficiary for his failure to pay
debts be paid by a trustee to the judgment creditor from either trust income or principal under
these facts? ANSWER: No. Most likely, Creditor will not be able to compel Bank to distribute trust
income to it in satisfaction of its claim against Son because the trust instrument includes a
spendthrift clause. Creditor also will not be able to compel distributions to it from the principal of
the trust because Son is not entitled to any share of trust principal.

POINT (2) [35%] ISSUE: Can claims against the income beneficiary of a trust for unpaid alimony
be paid by a trustee to the former spouse of the income beneficiary from either trust income or
principal under these facts? ANSWER: Wife will not be able to compel distributions to her from
the trust principal. Most likely, however, Wife will be able to compel Bank to distribute trust
income to her in satisfaction of her unpaid alimony claims against Son notwithstanding the
spendthrift clause in the trust.

POINT (3) [15%] ISSUE: Can the income beneficiary of a trust be compelled to use trust income
actually distributed to the beneficiary to pay judgment claims where the trust contains a
spendthrift clause? ANSWER: Yes. Son can be compelled to use trust income distributed to him
from the trust to pay judgment claims against him even though the trust contains a spendthrift
clause.

POINT (4) [15%] ISSUE: Are dividends paid to a trustee in stock, rather than cash, allocable to the
income or principal accounts for trust accounting purposes? ANSWER: Bank should hold the
dividend paid in stock as part of the trust principal for ultimate distribution to Son’s children when
Son dies.

ANSWER EXPLANATION:

Explanation to Point-One (30-40%):

Most likely, Creditor will not be able to compel Bank to distribute trust income to it in satisfaction of its
claim against Son because the trust instrument includes a spendthrift clause. Creditor also will not be able
to compel distributions to it from the principal of the trust because Son is not entitled to any share of trust
principal.

The creditors of the beneficiary of a trust have no greater rights in the trust property than the beneficiary
has; however, they can have fewer rights. The trust instrument makes no provision for the distribution of
trust principal to Son. Accordingly, because he has no right to receive trust principal, his creditors have no
right to reach trust principal. If Son’s creditors could reach trust principal, they would infringe on the
property rights of the remainder beneficiaries of the trust.

Son, on the other hand, is entitled to trust income and, thus, absent a spendthrift clause in the instrument,
his creditors would be able to attach that interest. However, if the governing instrument, as here, contains
a spendthrift clause prohibiting a beneficiary’s creditors from attaching the beneficiary’s interest, the
beneficiary’s creditors usually cannot reach the beneficiary’s trust interest in satisfaction of their claims.
Most courts uphold spendthrift clauses, at least when applied against the claims of most creditors of a
beneficiary, although a minority of states hold that spendthrift clauses are unenforceable.

Seperac-J19 Exam-Released MEE Essay Compilation © 2016-2020 1236


In the minority of states that bar the enforcement of spendthrift clauses, Creditor could compel Bank to
pay its claim against Son from trust income. However, in the large majority of states that uphold
spendthrift clauses, Creditor could not compel payment of the claim from trust income while in the hands
of Bank. Creditor stands on a different footing than Wife, and to date no state that generally upholds
spendthrift clauses has found a countervailing public policy exception for the kind of claim Creditor has
against Son.

Explanation to Point-Two (30-40%):

Wife will not be able to compel distributions to her from the trust principal. Most likely, however, Wife
will be able to compel Bank to distribute trust income to her in satisfaction of her unpaid alimony claims
against Son notwithstanding the spendthrift clause in the trust.

Wife cannot reach the trust principal because Son has no interest in trust principal.

However, she may be entitled to distributions from trust income. Even in the large majority of states that
uphold spendthrift clauses, if the particular creditor’s claim is for unpaid alimony, the policy of
effectuating the settlor’s intent by generally upholding spendthrift clauses runs afoul of a stronger public
policy favoring the payment of alimony and child support. Today, most state courts hold that the policy
favoring the payment of alimony and child support trumps the policy of generally supporting spendthrift
clauses. In those states a trust beneficiary is not permitted to enjoy trust income while failing to support
his children and former spouse. Hence, Wife should be able to compel Bank to satisfy her claim for
alimony from the trust’s income.

In a small minority of states, spendthrift clauses continue to be valid even as against unpaid alimony. In
such states, Wife could not compel a distribution of trust income in payment of her claim.

Explanation to Point-Three (10-20%):

Son can be compelled to use trust income distributed to him from the trust to pay judgment claims against
him even though the trust contains a spendthrift clause.

Spendthrift clauses in a trust do not operate to bar a trust beneficiary’s creditors from reaching the
distributed income or principal once it has actually been paid to the beneficiary. If Grantor had tried to
restrain Son’s ability to alienate the trust income once distributed to Son, that would have been an invalid
restraint on alienation. Thus, once trust income is actually paid to Son, Wife and Creditor are entitled to
reach that income in satisfaction of their claims by resorting to the customary legal processes for the
enforcement of judgments.

Explanation to Point-Four (10-20%):

Bank should hold the dividend paid in stock as part of the trust principal for ultimate distribution to Son’s
children when Son dies.

The Uniform Principal and Income Act treats distributions of stock, whether characterized as a stock
dividend or as a stock split, as principal. The Revised Act adopts the same position.

The Revised Act gives Bank a limited power to allocate the stock dividend between income and principal
in cases where the distributing corporation had made no distributions to shareholders except in the form
of dividends paid in stock. In such cases, Bank might be justified in allocating some of the stock dividend

Seperac-J19 Exam-Released MEE Essay Compilation © 2016-2020 1237


to income to effectuate Grantor’s intent to provide Son with trust income. However, the facts do not
support such an allocation in this case.

In the absence of a statute, the law varies. In some states, dividends paid in the stock of the distributing
corporation are income; in others, principal. Some states allocate the dividend between income and
principal under the so-called “intact value” rule.

Seperac-J19 Exam-Released MEE Essay Compilation © 2016-2020 1238


#290-JUL 2000–MEE Q07: QUESTION SEVEN (TRUSTS)

Grantor was wealthy, childless, and unmarried. Grantor created a revocable living trust and transferred
assets to Trustee. The trust instrument directed Trustee “to pay the trust income to Grantor for life and,
upon her death, to pay the trust income to Grantor’s brother, Brother, for his life and, upon his death, to
distribute the trust principal to Cousin’s children who attain age 21.”

Grantor died several years later without having modified or revoked the trust. Grantor was survived by
Brother and Cousin.

When Grantor created the trust, Cousin was married to Wife and had a young child, Adam, from that
marriage. Shortly after Grantor’s death, Cousin and Wife had another child, Ellen. Later, Cousin and Wife
divorced, and Cousin married Second Wife. Cousin adopted Second Wife’s child, Doris, a minor. Doris
presently lives with Cousin and Second Wife.

Brother has recently died survived by Cousin; Cousin’s child, Adam, age 22; and Ellen and Doris, both of
whom are under the age of 21.

1. Does the trust violate the common-law Rule Against Perpetuities? Explain.

2. Assuming that the trust does not violate the common-law Rule Against Perpetuities, are Adam,
Ellen, and Doris entitled to a share of the trust principal and when, if at all, would each be entitled
to possession of that share? Explain.

Seperac-J19 Exam-Released MEE Essay Compilation © 2016-2020 1239


#290: J00-7 MEE: ANSWER: NCBE (TRUSTS)

POINT (1) [13%] ISSUE: Does the trust violate the common-law Rule Against Perpetuities?
ANSWER: No. The trust does not violate the common-law Rule Against Perpetuities.

POINT (2)(a) [46%] ISSUE: Are Cousin’s children Adam (who was alive when Grantor created the
trust), Ellen (born after Grantor’s death), and Doris (a minor adopted child) included in the class
gift of a remainder interest in the trust? ANSWER: Yes. Cousin’s children Adam (who was alive at
the time the trust was created) and Ellen (who was born after Grantor’s death) are members of the
class of “Cousin’s children” and have remainder interests in the trust. Cousin’s adopted child.
Doris, also likely qualifies as a member of the class.

POINT (2)(b) [41%] ISSUE: When, if at all, will Cousin’s children be entitled to receive their
shares? ANSWER: Under the “rule of convenience,” Adam may receive distribution of a pro rata
portion of the principal of the trust at Brother’s death since Adam has then fulfilled the condition
precedent of attaining age 21 and there is then no outstanding present possessory estate. Ellen and
Doris will receive their shares only if they reach age 21.

ANSWER EXPLANATION:

Explanation to Point-One (10-15%):

The trust does not violate the common-law Rule Against Perpetuities.

The trust does not violate the Rule Against Perpetuities. Since the trust was revocable, the period of the
rule begins to run from Grantor’s death, not the date the inter vivos trust was created. At Grantor’s death,
Cousin was a life in being, and the non-vested interest created in Cousin’s children vests or fails to vest
no later than 21 years after Cousin dies. Thus, the non-vested interest vests within the permissible
perpetuity period of lives in being plus 21 years. Brother’s interest does not violate the Rule because the
interest vests immediately at Grantor’s death.

In some states, the common-law Rule Against Perpetuities has been repealed by statute. In such states, all
interests in the trust would be valid.

Explanation to Point-Two(a) (40-50%):

Cousin’s children Adam (who was alive at the time the trust was created) and Ellen (who was born after
Grantor’s death) are members of the class of “Cousin’s children” and have remainder interests in the trust.
Cousin’s adopted child. Doris, also likely qualifies as a member of the class.

Grantor created a class gift in a contingent remainder when she created an interest in “Cousin’s children
who attain age 21.” A class gift is a gift to a group of people collectively described, typically by reference
to their relationship to a common ancestor, rather than a gift to named individuals. A class gift by its
nature is capable of passing to multiple takers. The nature of a class gift is that the class membership, i.e.,
persons who may share in the gift, may fluctuate over time. At issue is the identity of the members of the
class. This class consists of all of Cousin’s children who join the class before it closes. However, only
those class members who reach age 21 are entitled to a share of the property.

Seperac-J19 Exam-Released MEE Essay Compilation © 2016-2020 1240


Cousin’s child, Adam, who was alive at the time Grantor created the trust, is included in the gift of the
remainder interest in the trust as of the time the gift was created. However, because the gift is a future
interest, not an immediate gift, the class gift remains open until Brother’s death, when the class closes. As
long as the class remains open, new members may join the class. Therefore, since Cousin’s other
biological child, Ellen, was born prior to Brother’s death and while the class was open, she is also a class
member.

The more difficult question is whether Doris, who is Cousin’s adopted child, also is included in the class.
While adopted children generally inherit from their adoptive parents, under the “stranger to the adoption”
rule they were, until very recently, excluded from class gifts created by someone who did not know of the
adoption and was not the adopting parent.

Today, however, in many states, adopted children are more likely than they were in the past to be
included in class gifts created by persons other than their parents. However, states differ in the criteria
used for their inclusion in a class gift. For example, in states that follow the Uniform Probate Code,
adopted children qualify as part of a class of “children” only if they “lived while a minor as a regular
member of the household” of the named parent, unless the governing instrument indicates a contrary
intent. Under the UPC, Doris is likely to be included because she is under 21 and lives in Cousin’s
household.

In non-UPC states that have rejected the “stranger to the adoption” rule, an adopted child is included in a
class gift created by someone other than the adopting parent on the theory that such child has been
brought into the family of the adopting parent for all purposes and that society no longer distinguishes
between adopted and biological children. In such states, adopted children are included in a class gift
unless the governing instrument provides otherwise. In light of these modern reforms, it is most likely that
Doris, Cousin’s adopted child, will be included in the class gift since she was adopted before the class
closed.

Depending on which of Cousin’s children, as a group, qualify for an interest in the trust, those children, as
earlier noted, acquired a contingent remainder in the trust. Two contingencies existed: (1) being born or
adopted (becoming a child of Cousin) before the class closes and (2) attaining the age of 21. Once a child
of Cousin reaches 21, that child’s interest becomes vested, subject to open, i.e., subject to partial
divestment by other children of Cousin. Once vested, the child is assured of obtaining a share of the trust
principal because no other contingencies are stated, such as survival to any particular time, including the
time for distribution. As more children attain the age of 21, the share of each becomes smaller because all
the children in the class when it finally closes will ultimately share equally. Until the class closes, more
potential class members may enter the class as they are born, but they may drop out of the class by not
attaining the age of 21. Thus, the size of the class membership may fluctuate.

Explanation to Point-Two(b) (35-45%):

Under the “rule of convenience,” Adam may receive distribution of a pro rata portion of the principal of
the trust at Brother’s death since Adam has then fulfilled the condition precedent of attaining age 21 and
there is then no outstanding present possessory estate. Ellen and Doris will receive their shares only if
they reach age 21.

Although Brother has died, Cousin is still living and may have more children. Thus, the class of
“Cousin’s children” will not close physiologically until Cousin’s death. However, absent any contrary
intent of Grantor, the rule of convenience, which is a rule of construction, will close the class when any
class member has a right to demand possession. Once a class closes, no new members can join the class.

Seperac-J19 Exam-Released MEE Essay Compilation © 2016-2020 1241


A class member can demand possession of his share if there is no outstanding present possessory estate
and the class member’s interest is not subject to the happening of any unfulfilled condition precedent.
Once the class closes, no new class members will be permitted to enter and take a share of the gift,
although some may drop out of the class by failing to attain 21. “The rule strikes a balance between a
donor’s desire to include all possible members of the class and the desire to allow distribution to those
who have qualified to take. It also avoids the administrative problems that would develop if living
beneficiaries took property, subject to giving some back as new class members appeared.”

Under the rule of convenience, Adam can demand a share at Brother’s death because both life tenants,
Grantor and Brother, are dead, and Adam is a child of Cousin who has attained the age of 21. Adam will
be entitled to a share of the trust principal in proportion to the total number of Cousin’s qualifying
children who are currently living. At the time the class closes, there are two (or three) members of the
class since they were born (or adopted) before the class closed. If either Ellen or Doris attains age 21, she
will receive her proportionate share. If either of them fails to attain age 21, she falls out of the class and
Adam (and the other member who has attained 21) will also receive a pro rata portion of that deceased
class member’s share. If Cousin has additional children after the class closes, those children will not be
permitted to share in the trust because the class will have closed prematurely under the rule of
convenience.

Seperac-J19 Exam-Released MEE Essay Compilation © 2016-2020 1242


#291-JUL 1999–MEE Q01: QUESTION ONE (TRUSTS)

Testator bequeathed her residuary estate “to Trustee, as trustee,” on the following terms:

I direct Trustee to distribute the income from the trust to my son, Adam, until he reaches the age of 30. In
addition, I authorize Trustee to invade the principal of the trust if, in its absolute discretion, it deems it
necessary to provide for Adam’s education. At Adam’s 30th birthday, I direct Trustee to terminate the
trust and to distribute the trust principal equally between my two children, Adam and Betsy.

When Testator executed her will, her estate was large enough so that her residuary estate would have been
valued at $800,000. However, business reverses during the two years between execution of the will and
Testator’s death reduced the value of the residuary estate to $300,000.

Testator died last year survived by her husband, Husband, by their son, Adam, and by Betsy, Testator’s
daughter from a prior marriage.

Adam is currently 23 years old, has no assets other than the interest in the trust, and has just entered
dental school.

Husband has an annual income of $80,000 and assets of $400,000. Last month, Husband loaned Adam
$25,000 to pay tuition at dental school.

The trust income is currently $ 18,000 per year, which Adam uses to pay for rent and food. Adam now
asks Trustee to invade the trust principal to permit Adam to purchase a $50,000 automobile and to repay
the $25,000 tuition loan from Husband.

1. Should Trustee invade the trust principal to enable Adam to purchase the automobile? Explain.

2. Can Trustee refuse to invade the trust principal to enable Adam to repay the loan he received
from Husband? Explain.

3. If Trustee distributes trust principal to Adam to enable him to purchase the automobile, what are
the rights and liabilities of Trustee, Adam, and Betsy? Explain.

Seperac-J19 Exam-Released MEE Essay Compilation © 2016-2020 1243


#291: J99-1 MEE: ANSWER: NCBE (TRUSTS)

POINT (1) [25%] ISSUE: Should Trustee invade trust principal for the purpose of enabling Adam
to purchase an automobile? ANSWER: No. Trustee should not invade trust principal for the
purpose of permitting Adam to purchase a $50.000 automobile.

POINT (2) [40%] ISSUE: Can Trustee refuse to invade trust principal to enable Adam to repay a
loan to Husband incurred to permit Adam to continue his education? ANSWER: Yes. Because
Trustee was granted absolute discretion to determine whether to invade trust principal to provide
for Adam’s education, Trustee need not invade the principal to permit Adam to repay the loan to
Husband, and Trustee’s failure to do so would not be an abuse of discretion. On the other hand,
Trustee would not be violating the terms of the trust should Trustee decide to invade the corpus to
permit Adam to repay the education loan to Husband.

POINT (3) [35%] ISSUE: If Trustee invades principal improperly on Adam’s behalf, does Trustee
or Adam bear personal liability for the improper invasion? ANSWER: Yes. If Trustee wrongfully
invades the trust principal, without court approval, to pay Adam money to purchase the
automobile, Trustee is personally liable to Betsy, but it will be entitled to indemnity from Adam
unless Adam has detrimentally relied on the wrongful payment. Trustee might also be removed as
trustee for this breach of the trust. Adam, a participant in the breach, would also be liable to Betsy.

ANSWER EXPLANATION:

Explanation to Point-One (20-30%):

Trustee should not invade trust principal for the purpose of permitting Adam to purchase a $50.000
automobile.

In general, unless the trust instrument authorizes the trustee to invade trust principal, an income
beneficiary is entitled only to trust income. While in some cases a power of invasion can be implied, here
Testator demonstrated that she knew how to create a power of invasion, since she expressly created one to
provide for Adam’s education. The inference is strong that she did not intend to confer on Trustee any
other power of invasion for Adam’s benefit. Trustee should refuse to make the distribution from principal
without a court order because the trust does not expressly permit the requested $50,000 distribution and,
therefore, the distribution without a court order would be a breach of trust.

However, when the income beneficiary will ultimately receive the trust principal, as here, a court may
permit the trustee to invade principal to assist the income beneficiary, unless an invasion of trust principal
would be inconsistent with the testator’s express directions in creating the trust. If Adam were the only
trust beneficiary, a court might authorize Trustee to invade principal to permit Adam to purchase the
automobile. Of course, that is not the case here.

A court might also permit an invasion due to a significant change in circumstances since the execution of
Testator’s will. For example, when Testator executed the will, she undoubtedly believed that the trust
income (possibly $48,000 annually assuming a 6% interest rate) would have been adequate to meet
Adam’s needs until Adam reached the age of 30. Testator’s business misfortunes significantly reduced her
expectation that Adam would receive such amount.

Seperac-J19 Exam-Released MEE Essay Compilation © 2016-2020 1244


The problem, however, is that an invasion of principal to pay for the automobile when not specifically
authorized would adversely affect the interests of the other beneficiary, Betsy. Accordingly, a court
should not authorize invasion absent express language in the trust instrument or Betsy’s consent.

Although Adam might argue that any depletion in principal could be subtracted from Adam’s own share
of the remainder, that argument is not plausible in light of Trustee’s power to invade principal for Adam’s
education. That is, since Trustee has the express power to invade principal for Adam’s education, Adam’s
share of the principal when the trust terminates may be too small to cover the cost of the car. As a result,
invasion would threaten the interest of Betsy, and hence would be unjustified.

Adam might argue that buying a car helps to further his education, particularly if a car is necessary to
permit Adam to go to and from school. While there may be some merit to that argument, it would not
warrant a $50,000 invasion.

Explanation to Point-Two (35-45%):

Because Trustee was granted absolute discretion to determine whether to invade trust principal to provide
for Adam’s education, Trustee need not invade the principal to permit Adam to repay the loan to
Husband, and Trustee’s failure to do so would not be an abuse of discretion. On the other hand, Trustee
would not be violating the terms of the trust should Trustee decide to invade the corpus to permit Adam to
repay the education loan to Husband.

When a testator confers discretion (or absolute discretion) on a trustee to make or withhold distributions,
courts do not compel trustees to exercise that discretion unless the trustee’s failure constitutes an abuse of
discretion. Likewise, courts do not enjoin trustees from exercising such power unless an exercise of the
power would be an abuse of discretion. The question here is whether Trustee would be abusing its
discretion by paying, or by refusing to pay, for Adam’s tuition. The facts do not warrant that conclusion.
Therefore, Adam cannot compel Trustee to distribute principal to him.

Because the trust instrument provides a standard by which Trustee’s conduct may be judged – are funds
necessary for Adam’s education – a court “will control the trustee in the exercise of a power where he acts
beyond the bounds of a reasonable judgment.” If, contrary to the actual facts, Adam had been without
other funds to pursue an education, Trustee would likely have been obligated to invade the trust principal
to provide Adam with assistance, absent some strong argument that even then an invasion would be
inadvisable. Here, however, Adam’s father, Husband, had assets and actually advanced Adam a loan to
pay for Adam’s tuition. These facts should permit Trustee to argue, if it chose not to invade principal, that
it was exercising its discretion reasonably because Adam’s educational expenses had been met by the loan
from his father and thus Adam’s need was fulfilled, and the fact that ultimately a beneficiary, in addition
to Adam, would be entitled to some share of the trust principal.

Courts generally presume that a testator intended the beneficiary to receive support or education from the
trust, regardless of other sources of support, unless the trust instrument otherwise provides. Moreover,
since Adam has reached the age of majority, Husband was under no legal obligation to provide for
Adam’s education. Therefore, Adam could argue that Trustee would be abusing its discretion if it did not
invade principal to repay Husband. However, this argument does not seem compelling.

Explanation to Point-Three (30-40%):

If Trustee wrongfully invades the trust principal, without court approval, to pay Adam money to purchase
the automobile, Trustee is personally liable to Betsy, but it will be entitled to indemnity from Adam

Seperac-J19 Exam-Released MEE Essay Compilation © 2016-2020 1245


unless Adam has detrimentally relied on the wrongful payment. Trustee might also be removed as trustee
for this breach of the trust. Adam, a participant in the breach, would also be liable to Betsy.

When a trustee abuses discretion and makes excessive payments to a beneficiary or distributes the trust
property to a beneficiary in breach of the terms of the trust, the trustee is liable for the resulting loss to the
trust and other beneficiaries. Here, if Trustee paid Adam money to purchase a car, such payment would be
inconsistent with the terms limiting principal invasions only for education, and Trustee would be liable to
reimburse the trust for resulting losses.

At the same time, however, when a trustee overpays one of the beneficiaries, the trustee is entitled to
recover the overpayment, unless the beneficiary has so changed his position that it would be inequitable to
require repayment. If Adam had already spent the money advanced, and if it were impractical to recover
the money from Adam, Trustee would be entitled to subject Adam’s own remainder interest to a charge in
the amount of the overpayment, thus effectively reimbursing Betsy for any losses she would otherwise
have suffered.

If Trustee’s unjustified payments to Adam reduced Adam’s share of the principal to such a degree that
reimbursement from Adam’s share would not make Betsy whole, she would be entitled to recover from
Trustee personally. Moreover, if a trustee breaches the terms of a trust, another remedy is the trustee’s
removal from office. That remedy is more likely to occur where the trustee personally benefits from the
breach, which did not occur in this case. Thus, it is unlikely that Betsy could successfully seek Trustee’s
removal.

When a beneficiary is unjustly enriched by a trustee’s breach of trust, the beneficiary can be liable to the
other beneficiary for any loss he or she sustains. Thus, Adam would also be liable to Betsy if neither
Trustee nor the trust assets satisfied her claim for breach of trust.

Seperac-J19 Exam-Released MEE Essay Compilation © 2016-2020 1246


#292-JUL 1998–MEE Q07: QUESTION SEVEN (TRUSTS)

Testator owned 50% of the stock of the ABC Corporation. Testator’s sister, Sister, owned the other 50%
of the stock.

Testator died in 1994. Under the terms of his testamentary trust, funded solely with his ABC stock, all
income was payable to Testator’s wife, Wife, for her life. Upon Wife’s death the trust principal was
payable to Child, who was the child of Testator from a prior marriage. Sister was named trustee of this
trust.

In 1995, Sister, as trustee, attempted to sell the trust’s ABC stock. The best offer she received was from
Buyer, who offered her $80,000 for the stock. Buyer also offered Sister $400,000 if Sister would Sell
Buyer both the trust’s ABC stock and Sister’s own 50% share of the outstanding ABC stock. Sister
declined this offer, preferring to retain her stock in ABC. After three more months, during which time no
other offer materialized, Sister bought the ABC stock from the trust for $100,000. This represented a 25%
premium over Buyer’s offer.

Sister then invested the entire $100,000 held in the trust in 1,000 shares of Lo-Tech, Inc., a publicly
traded company, which had a long record of paying steady and high dividends. At a time when other
suitable trust investments were yielding a return of 5% per year, Lo-Tech paid a cash dividend of $8 per
share on shares selling for $ 100 each. Just before Sister bought Lo-Tech stock for the trust, Lo-Tech hit
hard times. Nonetheless, it announced that it would continue to pay its usual high dividends, even if share
prices would decline as a result. Lo-Tech did so until 1997, and Sister paid all cash dividends to Wife.

In 1997, as Lo-Tech’s business continued to slide, Lo-Tech substituted a stock dividend (one-tenth of a
share for every share owned) for its previous cash dividend. Thereafter, Sister paid all stock dividends to
Wife.

In 1998, Sister sold the trust’s 1,000 shares of Lo-Tech because of her concerns about Lo-Tech’s financial
health. She realized only $50,000 as a result of the company’s financial slide. Sister invested the $50,000
in a low-risk mutual fund.

Wife has just died.

What fiduciary duties owed to Child did Sister violate and, as to each:

1. what amount of damages may Child recover; and

2. what other remedies, if any, are available to Child? Explain.

Seperac-J19 Exam-Released MEE Essay Compilation © 2016-2020 1247


#292: J98-7 MEE: ANSWER: NCBE (TRUSTS)

POINT (1) [40%] ISSUE: Did Sister breach her duty of loyalty to the trust when she bought the
trust’s ABC stock, and, if so, what remedies were available to Child for Sister’s breach? ANSWER:
Yes. Sister breached the duty of loyalty when she purchased the ABC stock from the trust for her
own account. As a result. Child is entitled to a share of the gain accruing to Sister as a result of that
breach.

POINT (2) [20%] ISSUE: Did Sister breach her duty to act prudently by putting all of the trust’s
assets into Lo-Tech stock, and, if so, what remedy may Child pursue for any breach? ANSWER:
Sister breached her duty to act prudently by putting all of the trust’s assets into Lo-Tech stocks and
Child would be entitled to recover any loss resulting from the failure to diversify the trust’s
investments.

POINT (3) [20%] ISSUE: Did Sister breach her duty to act impartially by purchasing stock that
paid high dividends with very little promise of appreciation, and if so, what remedy may Child
pursue for the breach? ANSWER: Yes. Sister breached her duty to act impartially by purchasing
stock that paid high dividends with very little promise of appreciation.

POINT (4) [20%] ISSUE: Did Sister misallocate dividends when she distributed the stock dividend
to Wife? ANSWER: Yes. Sister misallocated dividends when she distributed the stock dividend to
Wife and therefore. Child is entitled to recover the value of stock dividends paid to Wife that should
have been added to trust principal for ultimate distribution to Child.

ANSWER EXPLANATION:

Explanation to Point-One (35-45%):

Sister breached the duty of loyalty when she purchased the ABC stock from the trust for her own account.
As a result. Child is entitled to a share of the gain accruing to Sister as a result of that breach.

A trustee’s duty of loyalty prevents the trustee from purchasing trust property for the trustee’s own
account. This rule is based on the “well-known quality of human nature that it is extremely difficult, or
perhaps impossible, for an individual to act fairly in the interests of others whom he represents and at the
same time to consider his own financial advantage.” Before entering into a “self-dealing” transaction with
the trust, the trustee is obligated to seek court approval. In this case, Sister obtained no prior judicial
approval of her purchase of the stock from the trust. Therefore, she breached her duty of loyalty.

Sister might argue that Child is entitled to no damages for this breach, asserting that Sister’s self-dealing
transaction benefitted the trust: Sister paid the trust $ 100,000 for the ABC stock, while the best outside
offer was only $80,000. This argument fails, however, because Child is entitled to recover any profit
made by Sister as a result of Sister’s breach. In this case, when Sister purchased the trust’s ABC stock,
she cemented her personal control over ABC Corporation. Buyer had offered $400,000 for the combined
shares of Sister and the trust; Sister refused this offer but bought the trust’s share for herself. When Sister
made this purchase, she actually realized an economic gain of $220,000: the value of the combined shares
($400,000) as reflected by Buyer’s offer, minus the price she paid to the trust ($100,000) and the value of
Sister’s own shares ($80,000 on the market). Even if the court declines to award Child all of Sister’s gain
(the full $220,000 premium) as a result of the self-dealing transaction, Child should at least be entitled to

Seperac-J19 Exam-Released MEE Essay Compilation © 2016-2020 1248


one-half of the combined value of the shares of $400,000, since Sister and the trust each started with half
of the total ABC shares. On this theory, Child should be entitled to $100,000: one half of the total value of
the shares (1/2 x $400,000) minus the money the trust already received for its interest ($100,000).

Child might also seek Sister’s removal as trustee, but that remedy is not worth much because the trust is
about to terminate.

Sister could also be required to return the ABC stock to the trust if the beneficiaries would prefer this
remedy.

Explanation to Point-Two (15-25%):

Sister breached her duty to act prudently by putting all of the trust’s assets into Lo-Tech stocks and Child
would be entitled to recover any loss resulting from the failure to diversify the trust’s investments.

A trustee is required to make only prudent investments for a trust. While Lo-Tech stock might generally
be a prudent investment (although under the facts this may be debatable), a trustee is also under a specific
duty to diversify the trust’s investments, unless such duty is waived by the grantors. By placing all of the
trust’s assets into a single security, Sister acted improperly.

Child may not be able to recover the entire $50,000 loss incurred as a result of the Lo-Tech purchase,
because Sister might have been entitled to invest some trust assets in Lo-Tech. For example, if Sister was
entitled, consistent with the prudent person investor rule, to invest $10,000 rather than $100,000 in Lo-
Tech, Child would be entitled to recover 90% of the loss on Lo-Tech stock, or $45,000.

Explanation to Point-Three (15-25%):

Sister breached her duty to act impartially by purchasing stock that paid high dividends with very little
promise of appreciation.

A trustee is under a duty to deal impartially with the beneficiaries. Purchase of Lo-Tech, which paid high
dividends at the expense of principal, when there were also no offsetting investments in the trust to
provide benefits for Child, who ultimately was entitled to trust principal, was a violation of Sister’s duty
to Child as the remainder beneficiary. Sister’s duty to Child would have permitted her to make such an
investment only if she had adopted “accounting, investment, or other administrative practices reasonably
designed to protect” the trust property’s purchasing power, assuming the Lo-Tech investments were
otherwise proper.

Child should be entitled to recover the amounts Child would have received had Sister made appropriate
provision for protecting the remainderman.

Explanation to Point-Four (15-25%):

Sister misallocated dividends when she distributed the stock dividend to Wife and therefore. Child is
entitled to recover the value of stock dividends paid to Wife that should have been added to trust principal
for ultimate distribution to Child.

The Uniform Principal and Income Act requires that all dividends paid in stock be treated as principal.
Therefore, by distributing the stock dividend to Wife, Sister breached her fiduciary duty to Child. As a
result, Child should be entitled to recover the value of the shares distributed to Wife.

Seperac-J19 Exam-Released MEE Essay Compilation © 2016-2020 1249


#293-JUL 1997–MEE Q02: QUESTION TWO (TRUSTS)

In 1980, Smith, a graduate of Education College, created the Smith Trust, naming himself as Trustee.
Under the terms of the Smith Trust, all income was to be used to “provide four-year full scholarships for
deserving children of Smith Corporation employees to attend Education College.” The Smith Trust also
provided that “if, for any reason, Education College should cease operations as a four-year college, the
trust shall terminate and the trust property shall revert to Smith.” Smith reserved the power to revoke and
amend this trust. Smith Corporation employs 15,000 people.

In 1994, Testator, also a graduate of Education College, and a resident of Smalltown, executed a will
leaving $300,000 to Testator’s child, Able. Testator’s will also provided:

My child, Able, has been well provided for, and I do not want to discourage him from becoming a useful
and productive citizen by leaving him more. For that reason, and because I have seen firsthand the
tremendous benefits that Education College provides to people of diverse backgrounds, I give my
residuary estate to the Trustee of the Smith Trust, as the same is amended from time to time, to be held
and disposed of as part of the principal of the Smith Trust.

In 1995, Smith amended the terms of the Smith Trust to limit scholarships to “Presbyterian children of
Smith Corporation employees,” rather than “deserving children of Smith Corporation employees.”

Three months later, Smith died intestate survived by only one child, Harry. Bank became successor
trustee of the Smith trust.

On September 1, 1996, Education College closed permanently because of a lack of resources.

One month later, Testator died leaving an estate of $2 million. Able was Testator’s only heir.

Harry claims that the Smith Trust terminated upon the closing of Education College and that Harry is
entitled to the property Smith transferred to this trust.

Able claims that Testator’s residuary bequest to the Trustee of the Smith Trust fails and that Able is
entitled to this bequest. As successor trustee, Bank claims that the Smith Trust is entitled to Testator’s
residuary estate in accordance with the terms of Testator’s will. The Smalltown Civic Corporation, a local
nonprofit corporation that annually awards $5,000 scholarships to high-ranking Smalltown High School
graduates, also claims that it is entitled to Testator’s residuary estate. It proposes to use the income from
the bequest to award additional scholarships to high school graduates of diverse backgrounds.

1. Is Harry entitled to the property Smith transferred to the Smith Trust? Explain.

2. To whom should Testator’s estate be distributed? Explain.

Seperac-J19 Exam-Released MEE Essay Compilation © 2016-2020 1250


#293: J97-2 MEE: ANSWER: NCBE (TRUSTS)

POINT (1) [22%] ISSUE: Is the Smith Trust a valid charitable trust? ANSWER: Yes. The Smith
Trust was a valid charitable trust.

POINT (2) [22%] ISSUE: Did the Smith Trust fail as a result of the closing of Education College,
and if so, how should the assets of the Smith Trust be distributed? ANSWER: Yes. The closing of
Education College caused the Smith Trust to fail because Smith did not have a general charitable
intent. This results in a reversion of the trust principal to Smith’s estate as expressly provided by
the terms of the trust.

POINT (3) [26%] ISSUE: Was the bequest in Testator’s will sufficient to pour over Testator’s
estate assets into the Smith Trust? ANSWER: No. The bequest in Testator’s will to the Smith Trust
fails.

POINT (4) [30%] ISSUE: If the Smith Trust failed, should the cy pres doctrine be applied to save
Testator’s bequest, and if so, how should the money be distributed? ANSWER: The cy pres
doctrine should be applied to save Testator’s charitable bequest for the benefit of the Smalltown
Civic Corporation’s scholarship program.

ANSWER EXPLANATION:

Explanation to Point-One (20-30%):

The Smith Trust was a valid charitable trust.

A trust for the advancement of education is a charitable trust. A trust to establish scholarships is a trust for
the advancement of education, even if the scholarship is open only to a limited class of potential
beneficiaries. If, on the other hand, the persons who are to benefit from a trust are “not of a sufficiently
large or indefinite class so that the community is interested in the enforcement of the trust,” the trust is not
charitable. In this case, the gift, as amended, to “Presbyterian children” is probably a sufficient description
of a class to qualify the Smith Trust as a charitable trust as such class is not so narrow as to violate the
rule of Restatement § 375. Generally, a trust whose potential beneficiaries are employees of a corporation
is sufficiently broad to qualify as a charitable trust if its purposes are otherwise charitable. Moreover,
Smith Corporation has thousands of employees. There is no reason to assume that the religious restriction
unduly narrows the potential beneficiaries. Therefore, the trust is charitable and governed by the law of
charitable trusts.

Explanation to Point-Two (20-30%):

The closing of Education College caused the Smith Trust to fail because Smith did not have a general
charitable intent. This results in a reversion of the trust principal to Smith’s estate as expressly provided
by the terms of the trust.

Section 401(1) of the Second Restatement of Trusts, provides that when, by the terms of a charitable trust,
the trust is to terminate upon the happening of a specified event, then the trust will terminate on the
occurrence of that event unless the termination provision is invalid. In this case, Smith expressly provided
that the trust was to terminate if, for any reason, Education College should cease operations and that upon

Seperac-J19 Exam-Released MEE Essay Compilation © 2016-2020 1251


such termination the trust property should revert to Smith. Since Education College ceased operating as a
four-year college, the trust terminated, and the trust property should be distributed to Harry, the sole
successor to Smith’s estate. The cy pres doctrine is inapplicable because Smith did not have a general
charitable intent as evidenced by the express and rather specific terms of the trust. Upon termination, the
trust principal reverts to Harry.

Explanation to Point-Three (25-35%):

The bequest in Testator’s will to the Smith Trust fails.

Section 2-511 of the Uniform Probate Code provides that a will may validly devise property to a trust
created either by testator or by another person, even if the trust is amendable or revocable. Moreover,
Section 2-511 provides that unless the will provides otherwise, the property devised should be
administered in accordance with the provisions of the trust instrument, including any amendments made
before or after testator’s death. In jurisdictions that have not adopted the Uniform Probate Code, the same
conclusions are reflected in the 1991 version of the Uniform Testamentary Additions to Trusts Act. (A
similar result could follow under the common law incorporation by reference doctrine, at least with
respect to the terms of the trust in effect when testator signed his will.) However, if the trust terminates
prior to the testator’s death, the bequest lapses. Since the Smith Trust terminated before Testator died, the
bequest lapses.

Explanation to Point-Four (30-40%):

The cy pres doctrine should be applied to save Testator’s charitable bequest for the benefit of the
Smalltown Civic Corporation’s scholarship program.

The Smith Trust failed when Education College closed. Likewise, the bequest to the trust under Testator’s
will failed. Nevertheless, with respect to Testator’s bequest to the trust, the cy pres doctrine is applicable.
The cy pres doctrine is applicable when property is placed in trust for a charitable purpose that has
become impossible to effectuate, and where testator has manifested a general intention to devote the
property to charitable purposes. Testator exhibited a general charitable intention by giving reasons for
limiting the share of the estate to go to Able, and by omitting any reverter clause to take effect if the
charitable bequest were to fail. Although Able might argue that Testator’s charitable intent was limited to
schemes involving only Education College, Testator’s expressed desire to assist people of “diverse
backgrounds” should be sufficient, in the absence of any reverter clause, to establish a general charitable
intent. Thus, the bequest should be saved for the benefit of another charity; Able should not be entitled to
the bequest.

In determining which charity should receive the bequest, the court will seek to effectuate Testator’s
wishes as closely as possible. Where Testator expressed an interest in benefiting a diverse group of
students, the court would be unlikely to award the property to the Smith Trustee, who would be required
by the terms of the amended trust instrument to limit beneficiaries to Presbyterian children of Smith
Corporation employees. Instead, the court would more likely award the property to the Smalltown Civic
Corporation, whose beneficiaries would be more likely to exhibit the diversity sought by Testator and,
thus, would more likely carry out Testator’s intent.

Seperac-J19 Exam-Released MEE Essay Compilation © 2016-2020 1252


#294-FEB 1997–MEE Q07: QUESTION SEVEN (TRUSTS/WILLS-ESTATES/FAMILY LAW)

Husband, a freelance writer, and Wife, a successful attorney, divorced amicably after ten years of
marriage. Their separation agreement, which was merged into their divorce decree, gave Husband custody
of their child, Son, who is now eight years old. The decree required Wife to (1) pay $800 monthly to
Husband for child support until Son reached age 18, (2) set aside $5,000 annually for ten years to finance
Son’s college education, and (3) transfer $20,000 annually to Husband during each of the following five
years to compensate Husband for his financial contributions toward supporting the family during the
marriage while Wife earned her law degree.

One year after the divorce, Wife married New Husband. Two days after they married, Wife executed a
valid will leaving her entire estate to New Husband. One year later, she and New Husband had a child,
Daughter. Wife died unexpectedly a week after Daughter was born.

In each of the two years immediately following her divorce from Husband, Wife had deposited $5,000
into a savings account in her name alone at Bank. Wife made only one withdrawal from this account,
taking out $ 1,200 just one month before she died.

Although Wife had paid child support to Husband for the two years immediately before her death, she had
made only one $20,000 payment to Husband before she died.

Wife owned $500,000 at her death, including the savings account at Bank. New Husband claims Wife’s
entire estate under her will.

1. Does Wife’s obligation under the divorce decree to support Son survive Wife’s death? Explain.

2. On what theory could Son make a specific claim to the Bank savings account, and would such a
claim be likely to succeed? Explain.

3. Is Wife’s estate obligated to pay Husband the unpaid amounts Wife owed him for his financial
contributions toward supporting the family while she earned her law degree? Explain.

4. Do Son and Daughter have any other claims to any share of Wife’s estate, and, if so, will they
succeed? Explain.

Seperac-J19 Exam-Released MEE Essay Compilation © 2016-2020 1253


#294: F97-7 MEE: ANSWER: NCBE (TRUSTS/WILLS-ESTATES/FAMILY LAW)

POINT (1) [27%] ISSUE: Does Wife’s obligation to pay child support for Son survive her death?
ANSWER: No. Obligations to pay child support usually terminate upon the death of either the
child or the obligor. Wife’s estate probably has no obligation to continue child support for Son.

POINT (2) [36%] ISSUE: Can a trust be imposed on a savings account that does not indicate that
the depositor was acting as a fiduciary? ANSWER: Yes. The evidence is insufficient to establish
Wife’s intent to create a trust for Son. While no specific words are necessary to create a trust, there
must be some evidence of such an intent and no language in either the separation agreement, the
decree into which it merged, or the name on the savings account indicates that Wife intended to
create a trust.

POINT (3) [18%] ISSUE: Does a decedent’s obligation to transfer property to a former spouse
pursuant to a divorce decree survive the obligor’s death? ANSWER: Yes. Property division
obligations incident to a divorce remain enforceable against a deceased obligor’s estate. If Wife’s
obligation to transfer $20,000 annually to Husband for five years is considered a division of
property, Wife’s estate is liable for both the $20,000 past-due amount and the $60.000 in future
payments that Wife was ordered to pay Husband.

POINT (4) [18%] ISSUE: Do children living when a will is executed or born after the will is
executed have a valid claim against their deceased parent’s estate when they are not otherwise
provided for in the will? ANSWER: Son is not entitled to a share of Wife’s estate because he was
alive when Wife’s will was signed and, therefore, was not unintentionally omitted from Wife’s will:
Daughter, an after-born child, is deemed to have been unintentionally omitted from the will and
may be entitled to a share of Wife’s estate. Son may also have claims arising from Wife’s obligation
to set aside funds for Son’s college education.

ANSWER EXPLANATION:

Explanation to Point-One (25-35%):

Obligations to pay child support usually terminate upon the death of either the child or the obligor. Wife’s
estate probably has no obligation to continue child support for Son.

Wife’s obligation to support Son is based upon the divorce decree into which the separation agreement
between Wife and Husband merged. As a general rule, parental support obligations do not survive a
parent’s death even when those obligations have been reduced to a specific sum pursuant to a divorce
decree.

The children of divorced parents, however, have fared better than the children of ongoing marriages in
some cases holding that the support obligation established by a divorce decree is not discharged by death.

The weight of authority, however, still holds that court-ordered child support payments do not survive the
obligor parent’s death.

Explanation to Point-Two (35-45%):

Seperac-J19 Exam-Released MEE Essay Compilation © 2016-2020 1254


The evidence is insufficient to establish Wife’s intent to create a trust for Son. While no specific words
are necessary to create a trust, there must be some evidence of such an intent and no language in either the
separation agreement, the decree into which it merged, or the name on the savings account indicates that
Wife intended to create a trust.

In order to create a trust, the settlor must intend to create the trust. The language of the separation
agreement and the decree into which it was merged is unclear regarding Wife’s intent to create a trust of
the amounts she agreed to “set aside” for Son. Furthermore, there are no words on the savings account
registration evidencing that intent. Although no specific words are necessary to create a trust, there must
be some evidence of an intent to create a trust at the time the funds were set aside to hold Wife to the
obligations of a trustee.

An argument can be made, however, that the two $5,000 deposits into the savings account were actually
held by Wife in trust for Son, particularly since they were made each year and no other deposits were
made into the account. Cutting against this argument, however, is the fact that Wife withdrew $ 1,200
from the savings account one month before she died. On the theory that Wife would not intentionally
withdraw monies that were not hers, this may evidence an intent on her part not to have created a trust. On
the other hand, if the money was intended to be held in trust for Son’s college education, Wife’s
withdrawal would be a breach of trust for which her estate would be liable to Son. In addition, the entire
balance in the savings account remaining at Wife’s death would belong to Son.

It might also be argued that Wife held the funds in a constructive trust for Son on the theory that Wife
promised to formally create a trust and she should be held to that promise. The facts do not support this
interpretation because the separation agreement made no reference to the creation of a trust.

Explanation to Point-Three (15-25%):

Property division obligations incident to a divorce remain enforceable against a deceased obligor’s estate.
If Wife’s obligation to transfer $20,000 annually to Husband for five years is considered a division of
property, Wife’s estate is liable for both the $20,000 past-due amount and the $60.000 in future payments
that Wife was ordered to pay Husband.

In some jurisdictions, money awarded to a spouse who supported a former spouse while he or she earned
a graduate degree is considered, in essence, a property division. Unpaid property division obligations are
treated like other creditors’ claims against a decedent’s estate. If Husband’s right to the five $20,000
payments are unpaid property division claims, then Husband is entitled to receive $80,000 from Wife’s
estate.

In other jurisdictions, the award is called “reimbursement alimony.” Depending upon the terms of the
court decree, an award of reimbursement alimony may terminate on the obligor’s death.

Explanation to Point-Four (15-25%):

Son is not entitled to a share of Wife’s estate because he was alive when Wife’s will was signed and,
therefore, was not unintentionally omitted from Wife’s will: Daughter, an after-born child, is deemed to
have been unintentionally omitted from the will and may be entitled to a share of Wife’s estate. Son may
also have claims arising from Wife’s obligation to set aside funds for Son’s college education.

Under the Uniform Probate Code, only an after-born child unintentionally omitted from a parent’s will
may claim his or her intestate share of the decedent’s estate, but not if the deceased parent’s will gives

Seperac-J19 Exam-Released MEE Essay Compilation © 2016-2020 1255


substantially all of the estate to that child’s other parent. Therefore, under the Uniform Probate Code, Son
can claim nothing from Wife’s estate because he was alive when Wife signed the will; Daughter can
claim nothing from Wife’s estate because Wife willed her entire estate to Daughter’s other parent, New
Husband.

In many states, however, an after-born child is entitled to a share of the deceased parent’s estate
(generally equal to what that child’s intestate share would have been if the parent had actually died
intestate) even though the parent willed all or substantially all of her estate to the surviving parent of the
child absent evidence of an intent to disinherit the after-born child. In such states, Daughter would be
entitled to a share of Wife’s estate since Wife’s will was silent on whether Daughter was intentionally
disinherited.

In a few states, even a child alive when the will was signed may be entitled to a share of the deceased
parent’s estate on the theory that the child was unintentionally omitted from the will.

If, pursuant to Point Two, a trust was created for Son such that Wife’s $1,200 withdrawal was a breach of
trust, Son would also have a claim for $1,200 against Wife’s estate. Furthermore, Wife’s estate may be
liable to Son for Wife’s unfulfilled promise to set aside $5,000 annually for ten years to finance Son’s
education unless the promise is in the nature of child support which terminates at Wife’s death.

Seperac-J19 Exam-Released MEE Essay Compilation © 2016-2020 1256


#295-FEB 1996–MEE Q07: QUESTION SEVEN (TRUSTS)

Settlor executed a trust instrument entitled "Settlor's Trust" naming Trustee-1 as trustee. Settlor's Trust
includes the following terms:

1. All income is payable to Beneficiary for life.

2. Upon Beneficiary's death, the trust principal is distributable, outright or in further trust, among such of
Beneficiary's issue as Beneficiary appoints by Beneficiary's last will but, if Beneficiary fails effectively to
exercise the power, the trust property is distributable to Default Taker upon Beneficiary's death.

3. In order to exercise the power, Beneficiary must make a specific reference to the power of appointment
and to the instrument creating the power.

4. No interest of any individual is assignable or reachable by the individual's creditors.

When Settlor's Trust was created, Beneficiary had no issue. However, three years later, Beneficiary had a
son, Child. During Beneficiary's lifetime, Default Taker signed a deed conveying all of her right, title, and
interest in the trust to Assignee.

Beneficiary died last year, leaving a valid will drafted by Beneficiary's attorney. In their discussions
leading to the preparation of this will, Beneficiary told the attorney to merge Beneficiary's personal assets
and the assets of Settlor's Trust. The last paragraph of Beneficiary's will provides:

All the rest, residue and remainder of my estate, including any property over which I have a power of
appointment, I give, devise, bequeath and appoint to Trustee- 2 to hold in further trust to pay the income
to Child for life, remainder to such of Child's issue as attain the age of twenty-five.

Six months ago, the executor of Beneficiary's estate funded this testamentary trust. Trustee- 2 then
contacted Trustee-1 requesting that the assets of Settlor's Trust also be distributed to Trustee-2 to hold in
trust under the terms of Beneficiary's will. Trustee-1 then brought a declaratory judgment action seeking
judicial instructions regarding the respective rights of Child, Child's issue, Default Taker, and Assignee in
the assets of Settlor's Trust.

1. Was the language of the last paragraph of Beneficiary's will sufficient to exercise Beneficiary's
power of appointment in light of the requirements set forth in Settlor's Trust? Explain.

2. If the language of Beneficiary's will was sufficient to exercise Beneficiary's power of


appointment, what interests in the principal, if any, were created as a result of that exercise in
Child, Child's issue, Default Taker, and Assignee? Explain.

3. If the language of Beneficiary's will was not sufficient to exercise Beneficiary's power of
appointment, what interests, if any, would Child, Child's issue, Default Taker, and Assignee have in
the principal? Explain.

Seperac-J19 Exam-Released MEE Essay Compilation © 2016-2020 1257


#295: F96-7 MEE: ANSWER: NCBE (TRUSTS)

POINT (1) [43%] ISSUE: Was Beneficiary's power of appointment effectively exercised by the
residuary clause in Beneficiary's will in light of the specific reference requirement in Settlor's Trust
and the fact that Beneficiary's will used only a general residuary clause coupled with a blanket
exercise clause? ANSWER: Yes. Beneficiary's power was effectively exercised, as evidenced by the
fact that Beneficiary specifically intended to exercise the power and used a blanket exercise clause
in the general residuary clause to set forth that intent. The blanket exercise clause satisfies the
specific reference requirement imposed by the donor of the power. Accordingly, the assets of
Settlor's Trust should be distributed to Trustee-2 to hold in trust in accordance with the provisions
of Beneficiary's will.

POINT (2) [38%] ISSUE: If Beneficiary's general residuary clause was effective to exercise the
power, did the exercise violate the Rule Against Perpetuities, in whole or in part? ANSWER:
Although Beneficiary purported to exercise the power by Beneficiary's will, under the common-law
Rule Against Perpetuities, the power was, in part, invalidly exercised because the remainder
interest limited in favor of Child's issue might vest more than 21 years after the death of lives in
being when the power was created in Beneficiary. However, Child's issue's interest might be upheld
if the jurisdiction applies either the wait-and-see, cy pres, or Uniform Statutory Rule Against
Perpetuities reform rules.

POINT (3) [19%] ISSUE: If the Beneficiary's general residuary clause was ineffective to exercise
the power because of the Rule Against Perpetuities, does the spendthrift clause in Settlor's Trust
effectively prevent the assignment of Default Taker's interest to Assignee? ANSWER: Yes. If some
interest passes to Default Taker either because the power was not effectively exercised by
Beneficiary's residuary clause or because the purported exercise violated the Rule Against
Perpetuities, the spendthrift clause in Settlor's Trust which is generally valid effectively prevents
the assignment of Default Taker's interest to Assignee. Thus, the property not effectively appointed
passes to Default Taker, not Assignee.

ANSWER EXPLANATION:

Explanation to Point-One (40-50%):

Beneficiary's power was effectively exercised, as evidenced by the fact that Beneficiary specifically
intended to exercise the power and used a blanket exercise clause in the general residuary clause to set
forth that intent. The blanket exercise clause satisfies the specific reference requirement imposed by the
donor of the power. Accordingly, the assets of Settlor's Trust should be distributed to Trustee-2 to hold in
trust in accordance with the provisions of Beneficiary's will.

A general residuary clause (“I give all of my property…”) in a will, coupled with a blanket exercise
clause (“I give all of my property, including any property over which I have a power of appointment”),
generally is sufficient to exercise powers of appointment, since it evidences the donee's intent to exercise
the power. However, prevailing case law as well as Section 2-704 of the Uniform Probate Code adopts the
position that a blanket exercise clause generally is not sufficient to exercise a power if the donor of the
power mandated a specific reference to the power. The question in this case is whether Beneficiary's
clearly indicated intent to exercise the power created by Settlor's Trust will make the blanket exercise
clause effective.

Seperac-J19 Exam-Released MEE Essay Compilation © 2016-2020 1258


The Uniform Probate Code assumes that the purpose of a specific reference requirement is to avoid an
inadvertent exercise of the power by the donee. It, therefore, gives effect even to blanket exercise clauses
if it is clear that the donee knew of the specific power and intended to exercise it.

Mere use by the donee of a blanket-exercise clause would be ineffective to exercise the power because
such a clause would not make a sufficient reference to the particular power. If, however, it could be
shown that the donee had knowledge of and intended to exercise the power, the blanket-exercise clause
would be sufficient to exercise the power, unless the presumption that the donor's intent was merely to
prevent an inadvertent exercise is overcome, that presumption would be overcome if it could be shown
that the donor's intention was not merely to prevent an inadvertent exercise of the power but was to
prevent any exercise of the power, intentional or inadvertent, that failed to identify in explicit terms the
specific power or the creating instrument.

Under Section 2-704 and its rationale, it appears likely that the presumption against exercise could be
overcome because the facts state that Beneficiary instructed the attorney who prepared Beneficiary's will
to merge Beneficiary's personal assets with the assets of Settlor's Trust. Thus, under the UPC the blanket
exercise clause is effective to exercise the power unless other facts could be established showing that the
donor of the power intended to do more with the specific reference clause than merely prevent an
inadvertent exercise of the power. Accordingly, Child will be entitled to the income from these assets for
life and, as discussed in Point 2, if the appointment did not violate the Rule Against Perpetuities, upon
Child's death the trust property will be distributed to Child's issue who attain age 25.

In other jurisdictions, however, a blanket exercise clause may be insufficient to exercise the power where,
as in this case, Settlor imposed a specific reference requirement. In that case, upon Beneficiary's death,
the principal of Settlor's Trust will be distributed to Default Taker, assuming, as developed in Point 3, the
deed from Default Taker to Assignee was invalid. Otherwise, the property would be distributed to
Assignee.

Explanation to Point-Two (35-45%):

Although Beneficiary purported to exercise the power by Beneficiary's will, under the common-law Rule
Against Perpetuities, the power was, in part, invalidly exercised because the remainder interest limited in
favor of Child's issue might vest more than 21 years after the death of lives in being when the power was
created in Beneficiary. However, Child's issue's interest might be upheld if the jurisdiction applies either
the wait-and-see, cy pres, or Uniform Statutory Rule Against Perpetuities reform rules.

Beneficiary was granted a special testamentary power of appointment exercisable in favor of a limited
group of persons – Beneficiary's issue. Under the common-law Rule Against Perpetuities (“Rule”) an
interest created by the exercise of a special power of appointment is valid only if the interest will in all
events vest, or fail to vest, no later than 21 years after the death of lives in being living at the time the
power was created. Under the Rule the appointment to Child for life is valid because it vests no later than
the death of Beneficiary, and Beneficiary was alive when the power was created – when Settlor's Trust
was created. However, Beneficiary's exercise of the power in favor of Child's issue who survive Child is
void under the Rule. It is void because it might vest in interest more than 21 years after the death of lives
in being when the power was created in Beneficiary. For example, the remainder interest in the trust
created under Beneficiary's will might vest too remotely if Child, who was not a life in being, had a child
more than 21 years after the death of the Settlor and Beneficiary who survived Child, or if Child died
survived by a child under the age of 4 who could not reach the age of25 within 21 years of the death of
the survivor of the lives in being. The interest of Child's issue cannot be saved under the so-called “second

Seperac-J19 Exam-Released MEE Essay Compilation © 2016-2020 1259


look” doctrine because Child was not alive when the power was created. To the extent the power was not
effectively exercised, the trust principal would pass to Default Taker under Settlor's Trust.

The interest of Child's issue, however, might be valid if the jurisdiction applies a reform doctrine such as
wait-and-see, cy pres, or the Statutory Rule Against Perpetuities. Under wait-and-see, if the interest of
Child's issue actually vests within 21 years of Beneficiary's (or Settlor's) death, for example, it is valid
because it would, in fact, have vested no later than 21 years after the death of lives in being. Under cy pres
it is valid if a court reforms Beneficiary's will to require the interest of Child's issue to vest no later than
21 years after the death of lives in being when the power was created. Lastly, under the Statutory Rule
Against Perpetuities, the interest is valid if it actually vests no later than 90 years after the power was
created.

Explanation to Point-Three (15-25%):

If some interest passes to Default Taker either because the power was not effectively exercised by
Beneficiary's residuary clause or because the purported exercise violated the Rule Against Perpetuities,
the spendthrift clause in Settlor's Trust which is generally valid effectively prevents the assignment of
Default Taker's interest to Assignee. Thus, the property not effectively appointed passes to Default Taker,
not Assignee.

If Beneficiary's special power was not exercised because Beneficiary's will did not satisfy the specific
reference requirement, then upon Beneficiary's death, no interest passes to Beneficiary's issue. Rather, the
principal of Settlor's Trust passes to the taker in default (Default Taker) or, if the assignment is valid,
Assignee. (Likewise, if the power of appointment is exercised by Beneficiary's will but ineffectively
because of the Rule, the taker in default clause in Settlor's Trust would come into play.) While there has
been a long debate over the validity of spendthrift clauses which prevent the voluntary assignment of an
interest of a trust beneficiary or prevent creditors of the beneficiary reaching the beneficiary's interest, the
prevailing rule is that spendthrift clauses are valid. While spendthrift clauses are often held to be invalid
with respect to alimony, child support, and perhaps, tax claims, in this problem no such creditor claim is
involved. Accordingly, under prevailing law, Default Taker's assignment to Assignee is invalid.
Therefore, in the event it is determined that the interest of Child's issue is invalid (or that the power of
appointment was not effectively exercised), that interest will ultimately be distributable to Default Taker.

Seperac-J19 Exam-Released MEE Essay Compilation © 2016-2020 1260


#296-JUL 1995–MEE Q07: QUESTION SEVEN (TRUSTS)

Testator transferred property to Trustee to hold in a testamentary spendthrift trust according to the
following terms and provisions:

To pay to, or apply toward the benefit of, Daughter (1) whatever income is necessary to provide for her
support and (2) so much of the principal of the trust as Trustee deems advisable in its absolute and
unreviewable discretion to provide for Daughter's comfort and happiness. Upon Daughter's death, the trust
principal is distributable to Testator's brothers and sisters.

Because of her profound physical and mental disabilities, Daughter resides at Comfort Acres, a long-term
care facility, which provides her with all of her support. Daughter must reside at Comfort Acres or a
similar facility for the rest of her life.

For the last 15 years, Trustee has paid Comfort Acres $25,000 annually from the trust income to support
Daughter. Recently, however, even though the investments were prudent, the trust income has declined to
$40,000 annually. In addition, Comfort Acres has advised Trustee that its annual charges will increase to
$45,000. Trustee has advised Comfort Acres and Guardian, Daughter's legal guardian, that it will not pay
more than $35,000 from the trust income toward Comfort Acres's annual charge. Trustee takes this
position for three reasons. First, because Daughter has a life expectancy of approximately 30 years, it is
concerned that the trust property will be exhausted by periodic invasions of the principal before she dies.
Second, Trustee is concerned over the substantial increase in Comfort Acres's charges which appear out
of line with charges Trustee will pay next year from other trusts on behalf of similarly situated
beneficiaries who are confined to nursing homes. Third, it is also concerned about its potential liability to
Testator's brothers and sisters who are the remaindermen of the trust.

Guardian, on the other hand, believes that Trustee must pay the entire Comfort Acres bill from the income
and principal of the trust. Guardian also has told Trustee that if it does not make the full payment to
Comfort Acres, Guardian will commence a judicial proceeding to terminate the trust.

(a) Can Guardian compel Trustee to distribute trust income in payment of Comfort Acres' annual
charge? Explain.

(b) Can Guardian compel Trustee to distribute any of the trust principal in payment of Comfort
Acres' annual charge? Explain.

(c) Can a court revoke the trust upon the unilateral application of Guardian? Explain.

Seperac-J19 Exam-Released MEE Essay Compilation © 2016-2020 1261


#296: J95-7 MEE: ANSWER: NCBE (TRUSTS)

POINT (1) [35%] ISSUE: Can the guardian of the income beneficiary of a support trust compel
Trustee to expend trust income in payment of Comfort Acres' charge? ANSWER: Yes. The
guardian of the income beneficiary of a support trust should be able to compel Trustee to distribute
all of the trust income toward the payment of Daughter's support unless Trustee determines the
proposed charge is unnecessary.

POINT (2) [35%] ISSUE: Can the guardian of the beneficiary of a discretionary trust compel
Trustee to expend trust principal for the benefit of the beneficiary? ANSWER: No. Guardian
probably will not be able to compel Trustee to distribute principal in payment of Daughter's
support need because the trust instrument grants Trustee broad discretion to determine whether or
not to make support payments for Daughter from the principal.

POINT (3) [29%] ISSUE: Can the court revoke the trust on the unilateral application of the
guardian of the beneficiary? ANSWER: No. A court will not revoke the trust upon the unilateral
application of Guardian because not all beneficiaries who have an interest in the trust have agreed
to the termination and a material purpose of the trust remains to be performed.

ANSWER EXPLANATION:

Explanation to Point-One (25-35%):

The guardian of the income beneficiary of a support trust should be able to compel Trustee to distribute
all of the trust income toward the payment of Daughter's support unless Trustee determines the proposed
charge is unnecessary.

By way of preliminary background, trusts for support can be divided into two broad categories: a pure
support trust and a discretionary support trust. Classification of support trusts can be made only after a
careful review of the governing instrument in order to deter mine the grantor's intent.

A pure support trust is a trust under which payments are limited for a beneficiary's support. If the
beneficiary has a support need, Trustee must pay the beneficiary trust property (income or principal,
depending upon the terms of the trust) to the extent of available funds. (In some jurisdictions, this rule
may be tempered by another rule which would require Trustee to consider the beneficiary's non-trust
financial resources before using trust property for the beneficiary's support.

A discretionary support trust is a trust under which a trustee has discretion to withhold payments of trust
property from a beneficiary who has a support need.

In addition to discretionary support trusts, discretionary trusts may also be created to provide for more
than a beneficiary's support needs. Under this type of trust, a trustee might have discretion to pay
expenses incurred by a beneficiary for more than mere support items, such as luxuries. Similar to the
discretionary support trust, the trustee still has discretion to withhold payment even though the expense
incurred would otherwise be a proper item for the trustee to pay.

Seperac-J19 Exam-Released MEE Essay Compilation © 2016-2020 1262


A review of the trust in this problem suggests that a pure support trust was created with respect to the
beneficiary's income interest, and a discretionary trust for “comfort and happiness” was created with
respect to the beneficiary's rights in the principal of the trust.

Under the express terms of the trust, whatever income is necessary for Daughter's support is to be paid to
or applied toward Daughter's support. Trustee is granted no discretion to withhold payments of income
necessary for Daughter's support, although under the “pay or apply” language Trustee has the authority
either to pay income to Daughter directly or pay it to Daughter's creditor directly.

Trustee, of course, has the duty to determine whether the requested payment of $45,000 to Comfort Acres
is in discharge of the beneficiary's support need and whether it is necessary. Therefore, Trustee has an
important role to play in assessing whether income payments should be made. This judgment should be
made by Trustee based upon all the facts and circumstances and, if made in good faith, should not be open
to challenge. Three facts suggest that the proposed charge should not be paid: (1) the significant one-year
increase (from $25,000 to $45,000) in the charge, (2) the comparative charges at other nursing homes, and
(3) the fact that use of all the income would not be sufficient to provide the beneficiary with necessary
housing throughout the year.

If it was concluded that Trustee had no discretion or duty to assess “necessity,” one would also have to
conclude that Trustee should pay the Comfort Acres charge from income since, under the terms of the
instrument, it had no discretion to withhold support payments. A beneficiary of a support trust has an
enforceable legal right to trust property that, under the terms of the trust, must be distributed in
satisfaction of the beneficiary's support need when that need arises. Conversely, Trustee's refusal to
distribute amounts that must be paid to Daughter as beneficiary for Daughter's support is a breach of trust
for which Trustee can be held account able.

Since the beneficiary's right to enforce payment from the trust may be asserted by the beneficiary's
guardian, Guardian would be entitled to bring an action seeking payment.

Explanation to Point-Two (25-35%):

Guardian probably will not be able to compel Trustee to distribute principal in payment of Daughter's
support need because the trust instrument grants Trustee broad discretion to determine whether or not to
make support payments for Daughter from the principal.

Under the terms of the trust, Trustee is directed to pay to Daughter, or apply toward Daughter's benefit,
“so much of the principal as the trustee deems advisable in its absolute and unreviewable discretion to
provide for Daughter's comfort and happiness.” The emphasized words evidence the grantor's intent to
vest Trustee with broad discretion to make or withhold payments of principal that otherwise would be
proper in providing for the beneficiary's “comfort and happiness.” Since Trustee has the discretion to
withhold payments from a beneficiary, the beneficiary has no right to compel payment of trust property so
long as Trustee is not abusing its discretion by withholding payments.

In this case, Trustee indicates that it is withholding payment out of a concern for the financial interest of
the trust remaindermen and out of a concern that if it invades principal there will be fewer funds to
provide for Daughter's long-term financial security. Trustee might also argue that Daughter, of all the
named beneficiaries, would be uppermost in Testator's mind as deserving of financial protection and that
Testator's intent in that regard is best met by Trustee's judiciously withholding the principal from
Daughter. These reasons belie any notion that Trustee is acting out of a conflict of interest. Under
prevailing law, a trustee's discretion to distribute or withhold trust property from a beneficiary will not be

Seperac-J19 Exam-Released MEE Essay Compilation © 2016-2020 1263


overturned by a court absent an abuse of discretion. Therefore, it is unlikely that Guardian will be able to
compel Trustee to distribute principal to Daughter.

Explanation to Point-Three (20-30%):

A court will not revoke the trust upon the unilateral application of Guardian because not all beneficiaries
who have an interest in the trust have agreed to the termination and a material purpose of the trust remains
to be performed.

A trust may be revoked with the unanimous agreement of the grantor and all of the trust's beneficiaries. If
there are unborn beneficiaries or incompetent beneficiaries, the trust cannot be revoked since their consent
to revoke cannot be obtained, although in some jurisdictions courts may permit a guardian ad litem to
appear and consent on their behalf.

If the grantor is dead (as in this case) court approval can substitute for the testator's consent. However, for
a court to approve a revocation of a trust, the court must find that all material purposes of the trust have
been performed.

Under these rules it is clear that Guardian cannot compel the termination of the trust by a unilateral
application to the court. Even if all of Testator's siblings were agreeable to a termination of the trust, there
is the possibility that if Testator's parents are living, other siblings could be born whose consent could not
be obtained at this time. (In some jurisdictions it might be possible to overcome this concern by applying
a doctrine known as the doctrine of “virtual representation,” under which living members of a class can
approve an action that binds later born members of the class.)

Furthermore, even if Testator had no surviving parents so that the remainder interest class gift is closed in
favor of living siblings, the fact that the trust is a spendthrift trust may make the trust indestructible in
some jurisdictions on the theory that the spendthrift clause evidences that a material purpose of the trust
would be defeated if the trust were terminated. Perhaps, even more important, if the trust were terminated
and Daughter received only the present value of her income interest, she would be deprived of a lifetime
source of continuing income from the entire trust, a protection Testator may have preferred Daughter to
have instead of receiving a large outright sum. For example, in Townsend v. Rainier National Bank, the
court ruled that the power to distribute principal to an income beneficiary for his or her care, support, and
maintenance evidences the testator's intent that the income beneficiary not receive trust interest outright).

Seperac-J19 Exam-Released MEE Essay Compilation © 2016-2020 1264


WILLS & ESTATES: 37 OF 49 MEE EXAMS: (76%)
#297-JUL 2019–MEE Q01: QUESTION ONE (WILLS-ESTATES/CONFLICTS)

Testator’s handwritten and signed will provided, in its entirety,

I am extremely afraid of flying, but I have to fly to City for an urgent engagement. Given that I
might die on the trip to City, I write to convey my wish that my entire estate be distributed, in equal
shares, to my son John and his delightful wife of many years if anything should happen to me.

Dated: January 4, 2010

Signed: Testator

When Testator wrote the will, he was domiciled in State A, and his son John was married to Martha,
whom he had married in 2003. Testator had known Martha and her parents for many years, and Testator
had introduced Martha to John. At the time John and Martha married, Martha was a widow with two
children, ages five and six. Following their wedding, John and Martha raised Martha’s children together,
although John never adopted them.

Two years ago, Martha was killed in an automobile accident.

Six months ago, John married Nancy.

Four months ago, Testator died while domiciled in State B. All of his assets were in State B. The
handwritten will of January 4, 2010, was found in Testator’s bedside table. Testator was survived by his
sons, John and Robert, and John’s wife Nancy. Testator was also survived by Martha’s two children, who
have continued to live in John’s home since Martha’s death.

State A does not recognize holographic wills. State B, on the other hand, recognizes “wills in a testator’s
handwriting so long as the will is dated and subscribed by the testator.”

Statutes in both State A and State B provide that “if a beneficiary under a will predeceases the testator, the
deceased beneficiary’s surviving issue take the share the deceased beneficiary would have taken unless
the will expressly provides otherwise.”

How should Testator’s estate be distributed? Explain.

Seperac-J19 Exam-Released MEE Essay Compilation © 2016-2020 1265


#297: J19-1 MEE: ANSWER: NCBE (WILLS-ESTATES/CONFLICTS)

POINT (1) [10%] ISSUE: Does the law of State A or State B determine whether Testator’s
holographic will is valid? ANSWER: The law of a testator’s domicile at the time of the testator’s
death determines the validity of the testator’s will. Thus, State B’s law controls the validity of
Testator’s will.

POINT (2) [10%] ISSUE: Is Testator’s holographic will valid under the law of State B? ANSWER:
Yes. Testator’s will was validly executed because it was in Testator’s handwriting, it was dated, and
it was subscribed by Testator.

POINT (3) [15%] ISSUE: Was Testator’s will contingent on his dying during his trip to City?
ANSWER: No. Most likely, Testator’s will was not contingent on his death during travel to City
because the facts support the conclusion that Testator’s statement described his motive for writing
the will rather than a condition to its operation.

POINT (4) [35%] ISSUE: Was Martha or Nancy the intended beneficiary of Testator’s bequest to
John’s “delightful wife of many years”? ANSWER: Because the phrase “delightful wife of many
years” is ambiguous, a court will admit extrinsic evidence to determine its meaning, and all relevant
evidence suggests that Martha, not Nancy, was the intended beneficiary under Testator’s will.

POINT (5) [20%] ISSUE: Assuming that Martha is the intended beneficiary of the bequest to
John’s wife, are her children entitled to take Martha’s share under the State B anti-lapse statute?
ANSWER: Yes. Martha’s children will take Martha’s share of Testator’s estate under the State B
anti-lapse statute.

POINT (6) [10%] ISSUE: Assuming the will is valid, is Robert entitled to any share of the
Testator’s estate? ANSWER: Robert could be entitled to an omitted child’ share only if he had been
born after Testator’s will was signed in 2010. There are no facts that state when Robert was born.

ANSWER DISCUSSION:

The law of Testator’s domicile at death (here, State B) determines the validity of his will. Thus, Testator’s
will is valid because it is in Testator’s handwriting, it is dated, and it is “subscribed” (i.e., signed by
Testator at the end). Testator’s will was not contingent on his dying while flying to City because the
evidence shows that the text of the will reflects Testator’s motive for writing a will rather than stating a
condition precedent to its operation. A will is typically construed as of a testator’s death; here, Nancy was
John’s wife at Testator’s death. But Testator’s use of the phrase “delightful wife of many years,” plus the
other facts, strongly suggests that his intention was to benefit a specific individual, namely Martha. Thus,
a court is likely to find that Martha was the intended beneficiary of Testator’s bequest to “[John’s]
delightful wife of many years.” Under State B’s broad anti-lapse statute, Martha’s children would take
Martha’s share. Thus, Testator’s estate will be distributed according to the will as follows: one half to his
son John and the other half in equal shares to Martha’s two children. Neither Testator’s son Robert nor
John’s wife Nancy will take anything. Nevertheless, if the will is valid, Robert could be entitled to a share
of the estate if he was born after the will was signed in 2010. No facts tell us when he was born, but from
the surrounding facts it appears most likely he was living in 2010.

ANSWER EXPLANATION:

Seperac-J19 Exam-Released MEE Essay Compilation © 2016-2020 1266


Explanation to Point One (10%):

The law of a testator’s domicile at the time of the testator’s death determines the validity of the testator’s
will. Thus, State B’s law controls the validity of Testator’s will.

At common law, the validity of a testator’s will was determined under the law of the state where the
testator was domiciled at the time of his death. Thus, under the common law, State B law controls the
validity of Testator’s will.

Under the Uniform Probate Code (UPC), a will is valid if it complies either with the law of the state in
which it was executed or with the law of the place where the testator was domiciled when he signed his
will or when he died. Many non-UPC states have similar statutes. Testator’s will is valid under this
statutory approach because it was valid under the law of State B, the state of Testator’s domicile at death.

[NOTE: The following MEE jurisdictions have enacted the Uniform Probate Code: Arizona, Colorado,
Hawaii, Idaho, Montana, Nebraska, New Mexico, North Dakota, South Dakota, Utah, and Wisconsin.]

Explanation to Point Two (10%):

Testator’s will was validly executed because it was in Testator’s handwriting, it was dated, and it was
subscribed by Testator.

State B law provides that a will is valid if it is “in a testator’s handwriting so long as the will is dated and
subscribed by the testator.” Courts have held that when a statute requires a will to be subscribed, the
testator’s signature must appear at the end of the will.

Here, Testator’s will met the statutory requirements of State B: the will was in Testator’s handwriting, it
was dated, and it was signed by Testator at the end.

Explanation to Point Three (15%):

Most likely, Testator’s will was not contingent on his death during travel to City because the facts support
the conclusion that Testator’s statement described his motive for writing the will rather than a condition to
its operation.

In determining whether a will is conditional or contingent, a court must first determine whether the
happening of the possibility referred to is a condition to the operation of the will, or whether the
possibility of the happening was only a statement of the motive or inducement which led to the
preparation and execution of the instrument. A number of factors can be considered in resolving this
issue, including where the will was kept after the purported condition lapsed, whether there were any
other testamentary documents, whether setting aside the will would result in intestacy, and whether
effectuating the terms of the document would result in an inequitable distribution.

Here, two facts support the conclusion that Testator’s words—“I am extremely afraid of flying, but I have
to fly to City for an urgent engagement. Given that I might die on the trip to City, I write to convey my
wish that my entire estate be distributed, in equal shares, to my son John and his delightful wife of many
years if anything should happen to me.”— merely stated Testator’s motive for executing a will and not a
condition precedent that had to occur for the will to be given effect.

Seperac-J19 Exam-Released MEE Essay Compilation © 2016-2020 1267


First, Testator did not destroy the will after returning from City but kept it in his bedside table until the
time of his death many years later.

Second, there are no other known testamentary documents, and if the holographic will were construed as
conditional, Testator would die intestate; intestacy would result in a portion of Testator’s estate being
distributed to his other son, Robert, a person not mentioned in Testator’s will. This seems inconsistent
with Testator’s intentions.

Additionally, it does not seem inequitable to permit Martha’s children to inherit her share of Testator’s
estate given that they were being raised by Testator’s son John up to the time of Testator’s death. Testator,
who introduced Martha to John and had a long-standing relationship with Martha and her parents,
presumably knew the children and knew that his son John was raising them.

Thus, a court will likely find that Testator’s holographic will was not contingent on his dying during his
trip to City.

[NOTE: If an examinee were to conclude that the will was conditional, then because the condition did not
occur, the will would no longer be valid and Testator would have died intestate. In this case, the estate
would pass to Testator’s heirs, John and Robert.]

Explanation to Point Four (35%):

Because the phrase “delightful wife of many years” is ambiguous, a court will admit extrinsic evidence to
determine its meaning, and all relevant evidence suggests that Martha, not Nancy, was the intended
beneficiary under Testator’s will.

Testator left his estate in equal shares to his son John and John’s “delightful wife of many years.” Testator
did not mention John’s wife by name in the will. Because Martha died and John later married Nancy, it is
ambiguous as to which wife Testator referred to in his will.

Wills are generally construed as of the time of the testator’s death. Under this approach, the phrase
“delightful wife of many years,” if it were not ambiguous, could be construed in favor of John’s wife at
the time of Testator’s death, i.e., Nancy. However, in light of the phrase “of many years,” there is clearly
an ambiguity in the will in that John and Nancy, in fact, have not been married “for many years,” whereas
John and Martha had been.

When a will is ambiguous, courts allow extrinsic evidence to resolve the ambiguity. To resolve
ambiguities, the facts and circumstances surrounding the execution of the will may be taken into account.
As noted above, a court would likely find that the phrase “delightful wife of many years” is ambiguous
because Nancy and John had been married for only a few months when Testator died, whereas John and
Martha had been married for seven years when the will was executed in 2010. Furthermore, the fact that
Testator introduced Martha to John and had a long-standing prior connection to her and her parents
suggests that Testator was referring to Martha when he used the phrase “delightful wife of many years”
and was not using the word “wife” in a generic sense. Because the evidence suggests that Testator meant
Martha specifically when he left a portion of his estate to John’s “delightful wife of many years,” a court
is likely to rule that Martha, not Nancy, is the intended beneficiary.

Explanation to Point Five (20%):

Martha’s children will take Martha’s share of Testator’s estate under the State B anti-lapse statute.

Seperac-J19 Exam-Released MEE Essay Compilation © 2016-2020 1268


Under the common law, a bequest to a beneficiary who predeceased the testator failed. All states have
abandoned the common law rule by enacting so-called anti-lapse statutes. Anti-lapse statutes, when
applicable, typically substitute a deceased beneficiary’s issue as the beneficiaries of the bequest to the
deceased beneficiary. The typical anti-lapse statute extends anti-lapse protection only to blood relatives of
the testator. State B, however, has a very broad anti-lapse statute, which provides that “if a beneficiary
under a will predeceases the testator, the deceased beneficiary’s surviving issue take the share the
deceased beneficiary would have taken unless the will expressly provides otherwise.” Under this statute,
Martha’s children take the share to which Martha would have been entitled had she survived Testator,
assuming that the court finds that Martha is the intended beneficiary.

[NOTE: One of the most important skills a minimally competent lawyer should have is the ability to read
and apply a statute. A purpose of this point is to test examinees’ ability to read a statute, and for this
purpose, we have given them an uncommon anti-lapse statute.]

Explanation to Point Six (10%):

Robert could be entitled to an omitted child’ share only if he had been born after Testator’s will was
signed in 2010. There are no facts that state when Robert was born.

All states have a statute providing a forced share of an estate for a child who was born after the will was
signed but was not provided for in the will. These statutes vary, however, on the size of that omitted
child’s share.

Robert would qualify for this share if he was born after Testator’s will was signed in 2010. The facts do
not provide us with information as to when he was born, so whether he is entitled to that share is unclear.
However, it is more likely than not that Robert was living when Testator signed his will given the other
facts in the problem.

[NOTE: An important lawyering skill is knowing when the lawyer needs more facts to answer a legal
problem. Here the question asks how Testator’s estate should be distributed, and an important fact to
know in order to properly answer that question is when Robert was born.]

Seperac-J19 Exam-Released MEE Essay Compilation © 2016-2020 1269


#298-JUL 2017–MEE Q04: QUESTION FOUR (WILLS-ESTATES/TRUSTS)

In 2012, Testator wrote by hand a document labeled “My Will.” The dispositive provisions in that
document read:

A. I give $50,000 to my cousin, Bob;

B. I give my household goods to those persons mentioned in a memorandum I will write addressed
to my executor; and

C. I leave the balance of my estate to Bank, as trustee, to hold in trust to pay the income to my
child, Sam, for life and, when Sam dies, to distribute the trust principal in equal shares to his
children who attain age 21.

After Testator finished writing the will, he walked into his kitchen where his cousin (Bob) and his
neighbor were sitting. After showing them the will and telling them what it was but not what it said,
Testator signed it at the end in their presence. Testator then asked Bob and his neighbor to be witnesses.
They agreed and then signed, as witnesses, immediately below Testator’s signature. The will did not
contain an attestation clause or a self-proving will affidavit.

When the will was signed, Sam and his only child, Amy, age 19, were living. Testator also had an adult
daughter.

In 2015, Testator saw an attorney about a new will because he wanted to change the age at which Sam’s
children would take the trust principal from 21 to 25. The attorney told Testator that he could avoid the
expense of a new will by executing a codicil that would republish the earlier will and provide that, when
Sam died, the trust principal would pass to Sam’s children who attain age 25. The attorney then prepared
a codicil to that effect, which was properly executed and witnessed by two individuals unrelated to
Testator.

Two months ago, Testator died. The documents prepared by Testator and his attorney were found among
Testator’s possessions, together with a memorandum addressed to his executor in which Testator stated
that he wanted his furniture to go to his aunt. This memorandum was dated three days after Testator’s
codicil was duly executed. The memorandum was signed by Testator, but it was not witnessed.

Testator is survived by his aunt, his cousin Bob, and Sam’s two children, Amy, age 24, and Dan, age 3.
(Sam predeceased Testator.) Testator is also survived by his adult daughter, who was not mentioned in
any of the documents found among Testator’s possessions.

This jurisdiction does not recognize holographic wills. Under its laws, Testator’s daughter is not a
pretermitted heir. The jurisdiction has enacted the following statute:

Any nonvested interest that is invalid under the common law Rule Against Perpetuities is
nonetheless valid if it actually vests, or fails to vest, within 21 years after some life in being at the
creation of the interest.

To whom should Testator’s estate be distributed? Explain.

Seperac-J19 Exam-Released MEE Essay Compilation © 2016-2020 1270


#298: J17-4 MEE: ANSWER: NCBE (WILLS-ESTATES/TRUSTS)

POINT (1) [30%] ISSUE: Was Testator’s will validly executed? ANSWER: Yes. Testator’s will
was validly executed.

POINT (2) [30%] ISSUE: Is Bob, an interested witness, entitled to take the $50,000 bequest because
Testator’s codicil stated that he republished his will? ANSWER: Yes. The bequest to Bob, an
interested witness, was not forfeited. In some states, this is because Testator’s will was republished
by a codicil that was witnessed by two disinterested witnesses. In other states, this is because
bequests to interested witnesses are never forfeited.

POINT (3) [30%] ISSUE: Is the handwritten memorandum signed by Testator disposing of his
household goods valid to distribute that property to his aunt? ANSWER: Yes. In some states, the
aunt is not entitled to the furniture because a will may not incorporate an unattested document
written after the execution of the will. In many states, however, a testator may dispose of tangible
personal property in an unattested memorandum signed after his will was executed if the will
includes language evidencing an intent to give effect to the writing.

POINT (4) [10%] ISSUE: Under this jurisdiction’s statute, is Testator’s bequest of the balance of
his estate to Sam’s children who attain age 25 valid, or does it pass to Testator’s heirs? ANSWER:
Testator’s bequest to Sam’s children who attain age 25 is valid and does not violate the common law
Rule Against Perpetuities (RAP) or the statute because the gift vests or fails to vest within their own
lifetimes.

ANSWER DISCUSSION:

Testator’s will was validly executed because it was executed in a manner that satisfies the statute of wills
in every state: Testator declared the document to be his will to the witnesses, the witnesses acted at his
request, Testator signed it at the end, in the presence of the witnesses, and they signed in his presence and
in the presence of each other. Although, at common law, a will was invalid if witnessed by an interested
witness (a witness taking a bequest under the will), that is not the law in any state today. Instead, if a will
is witnessed by an interested witness, it is valid, but the bequest to the interested witness is either
unaffected or, in many states, is forfeited unless the will was also witnessed by two disinterested
witnesses. However, even in the latter states, the bequest to Bob would be saved because the codicil,
which was witnessed by two disinterested witnesses, republished the will in its entirety. Thus, the original
will is deemed to have been witnessed by the witnesses to the codicil and not by Bob and the neighbor.
Many states statutorily validate the bequest of tangibles even though contained in an unwitnessed
holographic writing signed after the will was executed. However, without such a statute, the
memorandum would not be valid under the incorporation-by-reference doctrine to dispose of Testator’s
household goods because it was not in existence when the will or the codicil was signed. The bequest to
the grandchildren is valid under the common law Rule Against Perpetuities. Because Sam predeceased
Testator, the grandchildren are the validating measuring lives under the Rule Against Perpetuities. Thus,
the residue of Testator’s estate passes to Sam’s children who reach 25, or if none, to Testator’s heirs.

ANSWER EXPLANATION:

Explanation to Point-One (30%):

Seperac-J19 Exam-Released MEE Essay Compilation © 2016-2020 1271


Testator’s will was validly executed.

Every statute of wills has formalities that must be met for a will to be validly executed. These invariably
include a writing, a signature of the testator, and at least two witnesses. All of these requirements were
met here. Additional requirements may apply, including a requirement that the will be declared as such to
the witnesses, that the testator request the witnesses to sign, and that the testator sign the will at the end.
These requirements were met here, too. Thus, the will complies with every statute of wills.

Under the common law, Testator’s will would be invalid because one of the witnesses was an interested
witness (i.e., a witness to whom a bequest was made under the will). Today, however, no state applies that
harsh rule. The signing of a will by an interested witness does not invalidate the will or any provision of
it.

[NOTE: The fact that the will was handwritten does not alter this analysis because the will was signed
and witnessed. Therefore, it is not governed by the laws relating to holographic wills.]

Explanation to Point-Two (30%):

The bequest to Bob, an interested witness, was not forfeited. In some states, this is because Testator’s will
was republished by a codicil that was witnessed by two disinterested witnesses. In other states, this is
because bequests to interested witnesses are never forfeited.

Some states provide by statute that a bequest to an interested witness is void unless the will is witnessed
by two disinterested witnesses. The policy underlying these interested-witness statutes is to discourage
fraud or undue influence.

However, under the republication-by-codicil doctrine, defects in a previously validly executed will can be
cured if the will is “republished” in the properly executed codicil. The doctrine effectively treats the
portions of the original valid will that are not inconsistent with the terms of the codicil as recited in the
codicil and witnessed by the witnesses to the codicil. Curing an interested witness problem by codicil is a
classic example of the republication-by-codicil doctrine. Thus, in states that require disinterested
witnesses, the bequest to Bob would nonetheless be valid because the will which he witnessed was later
republished by a codicil, which itself was witnessed by two disinterested witnesses.

Some states follow the Uniform Probate Code approach and simply permit interested witnesses to take
their bequests. In those states, Bob’s bequest was valid even though he was a witness to the will and
republication was not necessary to save that.

Explanation to Point-Three (30%):

In some states, the aunt is not entitled to the furniture because a will may not incorporate an unattested
document written after the execution of the will. In many states, however, a testator may dispose of
tangible personal property in an unattested memorandum signed after his will was executed if the will
includes language evidencing an intent to give effect to the writing.

At common law, the aunt would not take the furniture under the unattested memorandum because it was
executed after both the will and the codicil were signed, and thus it was not incorporated by reference into
either of those documents. Under the incorporation-by-reference doctrine, only documents in existence
when the will was signed can be incorporated by reference.

Seperac-J19 Exam-Released MEE Essay Compilation © 2016-2020 1272


Many states, however, have enacted statutes allowing a testator to dispose of tangible personal property
by a memorandum signed after the execution of the will if the will evidences an intent to dispose of the
tangibles in that manner. The writing here meets the requirements of such a statute in that (1) it was
signed, and it described the gifted items with sufficient particularity so that the gifted items are readily
identifiable. In such states, the aunt is entitled to the furniture.

Explanation to Point-Four (10%):

Testator’s bequest to Sam’s children who attain age 25 is valid and does not violate the common law Rule
Against Perpetuities (RAP) or the statute because the gift vests or fails to vest within their own lifetimes.

Under the common law RAP, “no interest is good unless it must vest, if at all, within 21 years of some life
in being at its creation.” With respect to wills, the beneficiary’s interests are created at the testator’s
death—not at the time the will is executed—because wills are “ambulatory” and can be revoked or
changed at any time up to the testator’s death.

In his will (as amended by the codicil), Testator bequeathed the balance of his estate to Sam for life, with
a remainder to Sam’s children who attain age 25. Because Sam predeceased Testator, the intended life
estate never became effective, leaving the class gift to Sam’s children who attain age 25. Because neither
child has reached 25 at Testator’s death, the children’s interest will remain in trust until they reach the
applicable age.

A class gift will vest for purposes of the RAP when the class is closed and all members of the class have
met any conditions precedent. Here, the class of “Sam’s children” closed when Sam died, leaving his two
surviving children as the only potential beneficiaries of the gift. As a result, the two children are “lives in
being” for purposes of the RAP, and the only question left is whether they will satisfy the age
contingency. This event is certain to be resolved within their lifetimes because either they will satisfy the
contingency and take the gift or they will die before reaching age 25 and the gift will fail. Accordingly,
the children are able to serve as their own validating lives, making their residuary gift valid under the
common law rule. Because the gift is valid under the common law rule, the jurisdiction’s wait-and-see
statute is inapplicable.

Thus, the balance of Testator’s estate should be distributed to the trustee to hold in trust and to distribute
the assets to Sam’s two children, when and if they attain age 25. However, their shares will be lost if they
fail to attain age 25. If both Amy and Dan meet the age contingency, each takes one half. If only one does,
that one takes the whole. And if neither does, the residue passes to Testator’s heirs.

[NOTE: An examinee who wrongly concludes that the gift to Sam’s children violates the rule should
conclude that the balance of Testator’s estate passes to his daughter and to Sam’s two children as partial
intestate property.]

Seperac-J19 Exam-Released MEE Essay Compilation © 2016-2020 1273


#299-FEB 2016–MEE Q05: QUESTION FIVE (WILLS-ESTATES)

Last year, a patient, age 80, was diagnosed with cancer. Shortly after receiving the cancer diagnosis, the
patient signed a durable health-care power of attorney (POA) designating her son as her “agent to make
all health-care decisions on my behalf when I lack capacity to make them myself.” The POA contained no
other provisions relevant to the commencement or duration of the agent’s authority. The patient thereafter
underwent several cancer therapies which were so successful that, two months ago, the patient’s doctor
said that, in his opinion, the patient’s cancer was in “complete remission.”

Last week, the patient was struck by an automobile, suffered serious injuries to her head and neck, and
underwent emergency surgery for those injuries. Following surgery, the patient’s doctor explained to her
son that there was a more than 50% risk that the patient would not regain consciousness and would need
to be maintained on life-support systems to provide her with food, hydration, and respiration. The doctor
also noted that, during the next few days, there was a large risk of a stroke or cardiac arrest, which would
substantially increase the risk that the patient would never regain consciousness, and which could be fatal.

The patient’s son was confident that his mother would not want to be kept on life support if she were
permanently unconscious but believed that she would want to be maintained on life support until her
status was clear. He thus instructed the doctor to put the patient on life support but not to resuscitate her if
she were to experience a stroke or cardiac arrest. The son issued these instructions after conferring with
the doctor and with his two sisters. The sisters disagreed with their brother’s decision and told the doctor
to ignore the instructions “because we have as much right to say what happens to Mom as he does, and we
want her resuscitated in all events.” Nonetheless, the doctor thereafter placed a “do not resuscitate”
(DNR) order in the patient’s chart.

Four days ago, the patient, who had not regained consciousness, suffered a cardiac arrest. Following the
DNR order, the nursing staff did not attempt to resuscitate the patient, and she died.

The patient’s valid will devised her estate to her three children in equal shares. All three children survived
the patient.

This jurisdiction has a typical statute authorizing durable health-care powers of attorney. This jurisdiction
also has a statute providing that “[n]o person shall share in the estate of a decedent when he or she
intentionally caused the decedent’s death.”

The patient’s two daughters have consulted an attorney, who has advised them that (1) the patient’s son
had no authority to instruct the doctor to write the DNR order; (2) in a wrongful death action, the son
would be liable for the patient’s death; and (3) the son is barred from taking under the patient’s will
because his actions intentionally caused her death.

Is the attorney correct? Explain.

Seperac-J19 Exam-Released MEE Essay Compilation © 2016-2020 1274


#299: F16-5 MEE: ANSWER: NCBE (WILLS-ESTATES)

POINT (1) [40%] ISSUE: Did the patient’s son, her health-care agent designated under a durable
power of attorney, have the power to instruct the doctor to enter a DNR order? ANSWER: Yes. An
agent acting under a durable health-care POA is empowered to make health-care decisions
whenever the principal lacks capacity to make a health-care decision. Thus, the patient’s son could
authorize the doctor to enter the DNR order here.

POINT (2) [30%] ISSUE: Could the patient’s son be found liable for the patient’s death in a
wrongful death action? ANSWER: No. An agent who acts in good faith is not liable for damages in
a wrongful death action. Here, because there is no evidence that the son did not act in good faith, he
would not be liable for damages in a wrongful death action.

POINT (3) [30%] ISSUE: Is the patient’s son barred from taking a share of the patient’s estate
under the state statute? ANSWER: No. The son is not barred from claiming a share of the patient’s
estate. The statute bars claimants who “intentionally” cause the decedent’s death, which the son did
not do. Moreover, the son is not the type of claimant the statute is intended to bar.

ANSWER DISCUSSION:

An agent acting under a durable health-care power of attorney (POA) is authorized to make health-care
decisions on behalf of the principal whenever the principal lacks the capacity to make a decision herself.
A duly appointed agent may make decisions on behalf of the principal without the concurrence of the
principal’s relatives. A durable power is not dependent upon a particular diagnosis and remains in effect
until and unless rescinded by the principal. Here, because the patient was unconscious and unable to make
health-care decisions, the son, her duly appointed agent, was authorized to direct the doctor to enter a
DNR order, even though the patient signed the power when faced with a different medical condition. The
opposition of the patient’s other children is irrelevant. Typical health-care durable powers statutes provide
that agents acting in “good faith” are immune from civil suit (such as a wrongful death action). Here,
because there is no evidence of bad faith, the son is likely immune from liability for the patient’s death.
The statute which precludes distribution of a share of a decedent’s estate to an individual who caused the
decedent’s death should be deemed inapplicable to the son. On the facts, there is a strong argument that
the son did not intentionally kill his mother. Moreover, most durable powers statutes specify that a death
resulting from the withholding of treatment at the agent’s directive is not to be treated as a homicide or an
intentional taking of the patient’s life. The statute also appears to be a form of slayer statute, which
generally applies only to intentional and felonious killing.

ANSWER EXPLANATION:

Explanation to Point-One (40%):

An agent acting under a durable health-care POA is empowered to make health-care decisions whenever
the principal lacks capacity to make a health-care decision. Thus, the patient’s son could authorize the
doctor to enter the DNR order here.

All states have adopted statutes authorizing advance directives and durable health-care powers. An
advance directive (sometimes called a living will) specifies the patient’s treatment and nontreatement

Seperac-J19 Exam-Released MEE Essay Compilation © 2016-2020 1275


preferences should he or she become incapacitated. A durable health-care POA empowers a designated
agent to make health-care decisions for the principal in the event of the principal’s incapacity.

Unless a durable POA specifies otherwise, a designated agent is empowered to make health-care
decisions for the principal whenever the principal lacks capacity – the power is not limited to a particular
illness or for a particular time period. Here, there is no language in the power conditioning the agent’s
authority to act for the principal except the patient’s incapacity. Thus, the fact that the patient was
motivated to execute the power because of a cancer diagnosis does not limit her designated agent’s power
to cancer-related decisions.

In the absence of specific instructions, a designated agent shall make decisions for the principal in
accordance with the agent’s determination of the principal’s best interest, considering the principal’s
personal values to the extent known to the agent. Here, the facts specify that the patient’s son was
confident that his mother would not want to be kept on life support if permanently unconscious. Given
that a cardiac arrest would substantially increase the already high risk of permanent unconsciousness, the
son’s instruction not to resuscitate appears to be consistent with the patient’s best interest. This conclusion
is arguably supported by the fact that the doctor does not seem to have disagreed with entry of a DNR
order. Indeed, if the doctor did not believe that entry of a DNR order was consistent with the patient’s
interests and the applicable standard of care, he could – and probably should – have declined to issue a
DNR order.

Had the patient not executed a durable health-care power, a majority of the siblings would have had to
agree upon health-care decisions for their mother. However, when, as here, an agent has been designated
by the patient, the agent may act on behalf of the patient without consulting family members who are not
designated agents of the patient.

[NOTE: In some cases of family conflict about treatment for an incapacitated principal, courts have
required a surrogate decision maker to provide clear and convincing evidence of the patient’s wishes as a
precondition to withdrawing life-sustaining treatment. However, in none of the reported cases did the
surrogate hold a durable power, and at least one state high court has noted that the clear and convincing
evidence requirement is not applicable to an agent with a durable power.]

Explanation to Point-Two (30%):

An agent who acts in good faith is not liable for damages in a wrongful death action. Here, because there
is no evidence that the son did not act in good faith, he would not be liable for damages in a wrongful
death action.

State laws governing durable health-care powers of attorney typically insulate an agent who has acted in
good faith from civil and criminal liability. Under the Uniform Health-Care Decisions Act, an individual
acting as an agent is not subject to civil or criminal liability for health-care decisions made in good faith.
A health-care decision includes directions to provide, withhold, or withdraw all other forms of health care.
Health care means any care, treatment, service or procedure to maintain, diagnose, or otherwise affect an
individual’s physical condition.

Here, the son appears to have acted in good faith. The facts specify that the son was confident that the
patient, his mother, would not want to be maintained in a permanently unconscious condition, there was
more than a 50 percent chance of permanent unconsciousness, the patient was of advanced age, both
cardiac arrest and stroke presented the risk of additional brain damage, and the doctor’s acquiescence
suggests that a DNR order was consistent with the standard of care applicable to the patient. There are no

Seperac-J19 Exam-Released MEE Essay Compilation © 2016-2020 1276


facts suggesting that the son acted in bad faith. The fact that the patient named her son as agent may also
indicate that she trusted his judgment and understanding of her values.

Explanation to Point-Three (30%):

The son is not barred from claiming a share of the patient’s estate. The statute bars claimants who
“intentionally” cause the decedent’s death, which the son did not do. Moreover, the son is not the type of
claimant the statute is intended to bar.

The state statute provides: “No person shall share in the estate of a decedent when he or she intentionally
caused the decedent’s death.” The statute appears to be a so-called “slayer statute,” which typically bars
those who have feloniously and intentionally killed a decedent from taking a portion of the decedent’s
estate. For example, under the UPC, a person who feloniously and intentionally kills the decedent forfeits
all benefits.

Here, the statute applies to slayers who intentionally cause the decedent’s death, creating an argument that
it applies to the patient’s son. Although the Uniform Health-Care Decisions Act provides that death
resulting from the withholding of health care in accordance with the Act does not for any purpose
constitute suicide or homicide, this does not resolve the question whether an actor “caused” a death.
However, there is a strong argument that the patient’s son did not “intentionally cause” his mother’s death
as that phrase is used in the statute. Moreover, factually the son did not cause his mother’s death. The son
specified only that medical personnel should not resuscitate the patient in given circumstances, and the
doctor agreed and entered a DNR order on the patient’s chart. The immediate cause of the patient’s death
was cardiac arrest, and it is not even clear that a resuscitation attempt would have been successful.
Moreover, medical ethicists do not typically regard withholding of treatment as the cause of death, and
this perspective has found judicial support.

Applying the state slayer statute to the son would also be inconsistent with the typical immunity provision
in power of attorney statutes. For example, the Uniform Health-Care Decisions Act immunizes agents
acting in good faith from civil liability, and disinheritance is akin to civil liability, as both result in the
loss of assets to which the penalized individual is otherwise entitled. Applying the statute to the son
would also undermine an important purpose of durable power of attorney statutes, ensuring that
individuals can choose who makes decisions on their behalf in the event of incapacity. Individuals
typically choose as agents close relatives who are potential heirs or distributees; many of these individuals
would be reluctant to serve if they stood to lose a share of the estate based on making a good-faith
decision which led to the principal’s death.

Thus, the son should not be barred from taking a share of his mother’s estate.

[NOTE: An examinee who concludes that the patient’s son caused, or contributed to, his mother’s death
but finds that the slayer statute does not apply should receive credit.]

Seperac-J19 Exam-Released MEE Essay Compilation © 2016-2020 1277


#300-FEB 2015–MEE Q06: QUESTION SIX (WILLS-ESTATES)

A husband and wife were married in 2005.

In 2009, the husband transferred $600,000 of his money to a revocable trust. Under the terms of the
properly executed trust instrument, upon the husband’s death all trust assets would pass to his alma mater,
University.

In 2012, the husband properly executed a will, prepared by his attorney based on the husband’s oral
instructions. Under the will, the husband bequeathed $5,000 to his best friend and the balance of his estate
“to my wife, regardless of whether we have children.” The husband failed to mention the revocable trust
to his attorney during the preparation of this will, and the attorney did not ask the husband whether he had
made any significant transfers in prior years.

In 2013, the husband and wife had a daughter.

In 2014, the husband was killed in an automobile accident. After his death, the wife found the husband’s
will and the revocable trust instrument on his desk. On the first page of the will, beginning in the left-hand
margin and extending over the words setting forth the bequests to the husband’s best friend and his wife,
were the following words: “This will makes no sense, as most of my assets are in the trust for University
and neither my wife nor my daughter seems adequately provided for. Estate plan should be changed. Call
lawyer to fix.” The statement was indisputably in the husband’s handwriting. The wife also found a voice
message on the phone from the husband’s lawyer, which said, “Calling back. I understand you have
concerns about your will.”

The husband is survived by his wife, their daughter, and the husband’s best friend. The assets in the
revocable trust are now worth $900,000. The husband’s probate estate is worth $300,000. He owed no
debts at his death.

All the foregoing events occurred in State A, which is not a community property state. State A has
enacted all of the customary probate statutes, but of particular relevance to the wife are the following:

(i) If a decedent dies intestate survived by a spouse and issue, the decedent’s surviving spouse takes one-
half of the estate and the decedent’s surviving issue take the other half.

(ii) A revocable trust created by a decedent during the decedent’s marriage is deemed illusory and the
decedent’s surviving spouse is entitled to receive one-half of the trust’s assets.

1. How should the assets of the husband’s probate estate be distributed? Explain.

2. How should the assets of the revocable trust be distributed? Explain.

Seperac-J19 Exam-Released MEE Essay Compilation © 2016-2020 1278


#300: F15-6 MEE: ANSWER: NCBE (WILLS-ESTATES)

POINT (1) [55%] ISSUE: Was the husband’s will revoked by physical act because of the
handwritten statement across the bequests to his wife and his best friend? ANSWER: No. The
husband’s will was not revoked because there is no evidence that he intended to revoke the will
when he wrote on it.

POINT (2) [15%] ISSUE: Is the husband’s daughter, born after the execution of his will, entitled to
a share of his probate estate? ANSWER: No. Most likely the husband’s daughter is not entitled to a
pretermitted child’s share. The phrase in the will “regardless of whether we have children” suggests
that the husband wanted the wife to take even if they had children, thus evidencing his intent that
the pretermitted child statute not apply.

POINT (3) [30%] ISSUE: How should the assets of the revocable trust deemed illusory under state
law be distributed upon the husband’s death after the wife claims her one-half share in those
assets? ANSWER: The wife is entitled to take one-half of the revocable trust under the state statute.
Either the wife, as residuary legatee under the husband’s will, or University, as remainderman of
the trust, is entitled to the other half.

ANSWER DISCUSSION:

A will can be revoked by a physical act, such as a cancellation, when the marks of cancellation are placed
on the text of the will with the intent to revoke the will. Here, the words the husband wrote on the will
evidence only an intent to have the will re-evaluated. Thus, the will was not revoked and its terms, subject
to a possible claim by his daughter, will be given effect. Under some state pretermitted (a/k/a “afterborn”
or “omitted”) child statutes, the husband’s daughter, born after the execution of his will, would be entitled
to that share of his probate estate she would have taken by intestacy unless the will otherwise provides. In
other states, however, she would not be entitled to a share of the husband’s probate estate because his will
left the bulk of his probate estate to his wife, the daughter’s other parent. Here, there are two reasons the
daughter may not take a pretermitted child’s share: (1) the phrase “regardless of whether we have
children” evidences the husband’s intent that the daughter not take that share, or (2) the assumed state law
bars the share if the will left the whole or substantially all of the estate (as here) to the child’s surviving
parent. Under the state statute, the wife is entitled to a one-half share of the revocable trust created by the
husband. It is unclear whether the remaining half would pass to the wife as residuary legatee under his
will or to University, the named remainderman of the revocable trust. Case law is divided on this point.
[NOTE: If the will was revoked, the husband’s probate estate of $300,000 would pass $150,000 to each of
the wife and the daughter and nothing would pass to the friend; if not revoked, the friend takes $5,000 and
the wife $295,000 from the probate estate, subject to any claim the daughter might have. Since the
pretermitted child’s claim is weak here, the conservator for the daughter has an incentive to challenge the
will.]

ANSWER EXPLANATION:

Explanation to Point-One (55%):

The husband’s will was not revoked because there is no evidence that he intended to revoke the will when
he wrote on it.

Seperac-J19 Exam-Released MEE Essay Compilation © 2016-2020 1279


A will may be revoked by the execution of a new will or by some physical act, such as cancellation or
other writings on the will, if the testator (or someone acting at the testator’s direction) performs the
physical act with the intent to revoke the will. The burden of proof to establish that a validly executed will
has been revoked is upon the party seeking to revoke the will.

Here, that burden cannot be met. The husband’s handwritten statement on the will does not show intent to
revoke the will. Instead, it suggests that he wanted to re-evaluate his overall estate plan in light of the fact
that a large portion of his assets were held in the revocable trust and that he did not believe that either his
wife or his daughter was adequately provided for. The phrases “estate plan should be changed” and “call
lawyer to fix” show intent to do something in the future after consultation with his attorney, not to revoke
the will now. In addition, the discovery of the documents found on the husband’s desk, his sudden death,
and the voice message on his phone from his lawyer suggest that the husband had only recently
discovered the problem and called his lawyer to work it out, not that he had revoked his will.

Thus, because the will was not revoked, the friend should take $5,000 under the husband’s will and his
wife should take $295,000, the balance of the probate estate (possibly increased by assets from the trust
but subject to the daughter’s claim, if any).

[NOTE: Alternatively, the markings on the will might be construed as evidencing an intent to revoke the
will. The husband appears to have recognized that the will made no sense in light of his family
circumstances. The markings state that the plan should be changed and that the husband will take steps to
make that change by calling his lawyer which, in fact, he did. Thus, so the argument goes, the will was
revoked by cancellation. If the will was revoked, then the husband died intestate and half the estate would
pass to the wife and half to the daughter under the intestacy law of the state.]

[NOTE: The argument claiming that there was an intent to revoke is not as strong as the argument in
favor of no revocation because of the language of futurity in those markings.]

Explanation to Point-Two (15%):

Most likely the husband’s daughter is not entitled to a pretermitted child’s share. The phrase in the will
“regardless of whether we have children” suggests that the husband wanted the wife to take even if they
had children, thus evidencing his intent that the pretermitted child statute not apply.

Most states have “pretermitted child” statutes aimed at ensuring that children born after the execution of a
will are not inadvertently disinherited. Many of these statutes provide that, under certain circumstances, a
child born to a testator after the testator’s will is executed is entitled to whatever share of the testator’s
estate the child would have received if the testator had died intestate. In states with such statutes, bequests
in favor of an afterborn child’s other parent are irrelevant. For example, In Iowa, a surviving spouse who
is the parent of the decedent’s surviving child is entitled to the entire estate, and the daughter would take
nothing. Meanwhile in Illinois, a surviving spouse is only entitled to one-half of an intestate decedent’s
estate, and the daughter’s share would be one-half of the estate. However, a testator can avoid the
consequences of such a statute if the will evidences intent to do so. Here, that seems to be the husband’s
intent as evidenced by the phrase “regardless of whether we have children.”

In other states, an afterborn child is denied a share of the decedent parent’s estate if the decedent parent
bequeathed all or substantially all of his estate to the child’s other parent. In states that follow the UPC
approach, the daughter would not be entitled to any share of the husband’s estate because her mother (the
wife) is entitled to substantially all of the husband’s estate.

Seperac-J19 Exam-Released MEE Essay Compilation © 2016-2020 1280


[NOTE: Regardless of how examinees conclude Point One, there were enough signals in the question to
have prompted discussion of this issue even if they concluded that the will had been revoked.]

Explanation to Point-Three (30%):

The wife is entitled to take one-half of the revocable trust under the state statute. Either the wife, as
residuary legatee under the husband’s will, or University, as remainderman of the trust, is entitled to the
other half.

Under the law of the state, the wife is entitled to a one-half share of the revocable trust created by the
husband because the trust was in existence during the marriage. This leaves open the question of who is
entitled to the balance of the trust’s assets. Under the statute, the trust is characterized as “illusory.” This
characterization is ambiguous regarding whether it is just illusory as to the wife or illusory for all
purposes. If the former, then the wife is entitled to her half share, as that share only is illusory, and the
balance of the trust (not deemed illusory) should pass to University, as the designated remainderman. On
the other hand, if a court deems the trust illusory for all purposes, then the trust is void and the trust assets
are distributed to the wife as residuary legatee of the husband’s estate, assuming that the will is valid. If,
however, the will is invalid, the trust assets pass equally to the wife and her daughter as intestate property.

Seperac-J19 Exam-Released MEE Essay Compilation © 2016-2020 1281


#301-JUL 2013–MEE Q09: QUESTION NINE (WILLS-ESTATES)

Twenty years ago, John and Mary were married. One month before their wedding, John and Mary signed
a valid prenuptial agreement in which each of them waived “any property rights in the estate or property
of the other to which he or she might otherwise be legally entitled upon the termination of their marriage
by death or divorce.”

Seventeen years ago, John executed a valid will, which provided as follows:

I, John, leave my entire estate to my wife, Mary. However, if I should hereafter have children, then I leave
three-fourths of my estate to my wife, Mary, and one-fourth of my estate to my children who survive me,
in equal shares.

Fifteen years ago, John had an extramarital affair with Beth, who gave birth to their child, Son. Both Beth
and John consented to Son’s adoption by Aunt. At the time of the adoption, Beth, John, and Aunt agreed
that Son would not be told that he was the biological child of Beth and John.

Three years ago, Aunt died, and Son moved into John and Mary’s home. At that time, John admitted to
Mary that he had had an extramarital affair with Beth which had resulted in Son’s birth.

Three months ago, Mary filed for divorce. Nonetheless, she and John continued to live together.

One month ago, before John and Mary’s divorce decree was entered, John was killed in a car accident.
John’s will, executed 17 years ago, has been offered for probate. John’s will did not designate anyone to
act as the personal representative of his estate.

John was survived by Mary, Son, and John’s mother.

1. To whom should John’s estate be distributed? Explain.

2. Who should be appointed as the personal representative of John’s estate? Explain.

Seperac-J19 Exam-Released MEE Essay Compilation © 2016-2020 1282


#301: J13-9 MEE: ANSWER: NCBE (WILLS-ESTATES)

POINT (1)(a) [25%] ISSUE: Is Mary barred from taking under John’s will because she and John
signed a prenuptial agreement? ANSWER: No. A prenuptial agreement waiving rights that a
spouse might otherwise assert against the estate of the other does not preclude a surviving spouse
from taking a bequest voluntarily devised to the surviving spouse by a deceased spouse. Thus, Mary
is not precluded from taking a share of John’s estate by the prenuptial agreement that she signed.

POINT (1)(b) [25%] ISSUE: Is Mary barred from taking under John’s will because, at the time of
John’s death, there was a divorce action pending between them? ANSWER: No. Although divorce
bars a former spouse from taking a bequest under a will executed prior to the divorce, the filing of
a divorce action does not. Thus, Mary is entitled to take under John’s will notwithstanding the
pendency of a divorce action at the time of his death.

POINT (1)(c) [35%] ISSUE: Under John’s will, is John’s biological, but adopted-out, child entitled
to take a share of John’s estate? ANSWER: In most states, Son will be entitled to a share of John’s
estate only if the court concludes that John intended to include an adopted-out child in the bequest
to “my children who survive me.” However, John’s adopted-out child, Son, is entitled to a share of
John’s estate under UPC § 2-119 or a like statute.

POINT (2) [15%] ISSUE: Who should be appointed as the personal representative of John’s estate?
ANSWER: Mary will be appointed personal representative of John’s estate.

ANSWER DISCUSSION:

At least three-fourths of John’s estate should be distributed to Mary. A prenuptial agreement waiving a
share of a spouse’s estate does not bar a surviving spouse from taking a bequest that was voluntarily made
after the agreement was signed, and a pending divorce does not revoke a testamentary disposition in favor
of a spouse. In states that have adopted Uniform Probate Code § 2-119 or a like statute, Son is entitled to
one-quarter of John’s estate. In other states, it is unclear whether one-quarter of John’s estate should be
distributed to Son; in these states, the court will construe John’s intent, and there are arguments for and
against the claim that John intended Son to take under his will. The court will appoint Mary as the
personal representative of John’s estate because a surviving spouse has first priority to serve in this
capacity. Son may not be appointed personal representative of the estate because Son is a minor.

ANSWER EXPLANATION:

Explanation to Point-One(a) (25%):

A prenuptial agreement waiving rights that a spouse might otherwise assert against the estate of the other
does not preclude a surviving spouse from taking a bequest voluntarily devised to the surviving spouse by
a deceased spouse. Thus, Mary is not precluded from taking a share of John’s estate by the prenuptial
agreement that she signed.

A prenuptial agreement in which spouses waive rights to a share of each other’s assets upon death or
divorce does not bar either party from making subsequent gifts or bequests to the other spouse. Such an
agreement bars only claims that do not arise from a voluntary gift or bequest. Thus, the prenuptial
agreement signed by John and Mary does not bar Mary from taking a share of John’s estate under his will.

Seperac-J19 Exam-Released MEE Essay Compilation © 2016-2020 1283


Explanation to Point-One(b) (25%):

Although divorce bars a former spouse from taking a bequest under a will executed prior to the divorce,
the filing of a divorce action does not. Thus, Mary is entitled to take under John’s will notwithstanding
the pendency of a divorce action at the time of his death.

A bequest may be revoked by operation of law when a change in circumstance occurs that makes it
unlikely that the testator would have wanted a beneficiary named in the will to take under the will. All
states provide that if, after the execution of a decedent’s will, the decedent is divorced, a bequest in favor
of the decedent’s former spouse is revoked by operation of law. However, the typical statute does not
apply if a divorce proceeding is pending when one of the spouses dies. This approach derives from the
fact that, until the divorce is finalized, no property division order will be entered; thus the moral and legal
claims of each spouse in the property of the other have not yet been satisfied.

Explanation to Point-One(c) (35%):

In most states, Son will be entitled to a share of John’s estate only if the court concludes that John
intended to include an adopted-out child in the bequest to “my children who survive me.” However,
John’s adopted-out child, Son, is entitled to a share of John’s estate under UPC § 2-119 or a like statute.

Under the terms of John’s will, one-fourth of John’s estate is distributable to John’s “children who survive
me.” Son is a biological child of John who survived John. However, John consented to Son’s adoption by
Aunt, and an adoption typically severs the parent-child relationship between the child and his biological
parents. A few states do not sever the parent-child relationship when the child is adopted by a relative of a
biological parent. In these states, Son would take under John’s will.

In most states, Son would take only if the court concluded that John, when using the words “my children”
in his will, intended to include an adopted-out child. The evidence on this question is far from
determinative. On the one hand, John gave Son up for adoption to a relative, Aunt, and he took Son into
his own home after Aunt’s death. John also told Mary that he was Son’s father, and he did so after the will
was executed. On the other hand, there is no evidence that John attempted to formalize his relationship
with Son. It is thus unclear whether Son would take a share of John’s estate.

Explanation to Point-Two (15%):

Mary will be appointed personal representative of John’s estate.

An individual named as personal representative in a decedent’s will has priority to receive letters
testamentary from the court overseeing the administration of the estate. Where, however, the will is silent
regarding the appointment of the personal representative, the court will appoint a person granted priority
under the governing statute if that person is otherwise qualified. Typically, the decedent’s surviving
spouse is the individual with the first priority.

Under the Uniform Probate Code, a surviving spouse has first priority only if the spouse is a devisee of
the decedent. Mary satisfies that condition. Therefore, Mary will be appointed personal representative of
the estate. Even if Mary were not a devisee, she would still be entitled to be appointed under the Code
because John’s devisees (i.e., “my children”) are not qualified to be appointed personal representatives
because Son is a minor.

Seperac-J19 Exam-Released MEE Essay Compilation © 2016-2020 1284


#302-JUL 2012–MEE Q09: QUESTION NINE (WILLS-ESTATES/CONFLICTS)

Zachary died a domiciliary of State A. At Zachary’s death, he owned a house located in State A. Zachary
also owned a farm located in State B and had a savings account at a bank in State B.

Zachary left a handwritten document containing instructions for the disposition of his assets. The only
words on this document were the following:

I, Zachary, being of sound and disposing mind, leave my entire estate to my alma mater, University. I
appoint Bank as executor of my estate.

Zachary’s wife predeceased him. Zachary was survived by three children, Alex, Brian, and Carrie. Alex
was the biological child of Zachary and his deceased wife. Brian was the biological child of Zachary’s
deceased wife and her first husband, but Zachary adopted Brian when Brian was 12. Carrie was the
biological child of Zachary and a woman whom Zachary never married. Zachary’s paternity of Carrie was
adjudicated during Zachary’s lifetime.

State A law provides that a holographic will “entirely handwritten and signed at the end by the testator” is
valid. State A law also provides that if a decedent dies intestate and leaves no surviving spouse, the
decedent’s estate passes in equal shares to the decedent’s “surviving children.” The phrase “surviving
children” is defined to exclude “nonmarital children.” There are no other relevant statutes in State A.

State B law provides that (1) the will of a nonresident that bequeaths real property located in State B must
comply with the law of State B; (2) a will is invalid unless it was signed by the testator and two witnesses;
and (3) the estate of an intestate decedent who leaves no surviving spouse passes to the decedent’s
“biological and adopted children, in equal shares.” There are no other relevant statutes in State B.

How should Zachary’s three assets be distributed? Explain.

Seperac-J19 Exam-Released MEE Essay Compilation © 2016-2020 1285


#302: J12-9 MEE: ANSWER: NCBE (WILLS-ESTATES/CONFLICTS)

POINT (1) [30%] ISSUE: Which state’s laws govern the disposition of Zachary’s bank account,
Zachary’s house, and Zachary’s farm? ANSWER: Because the disposition of real property is
governed by the law of the situs, State A law will govern the disposition of Zachary’s house and
State B law will govern the disposition of Zachary’s farm. State A law also governs the distribution
of the personal property, the bank account, because State A was the decedent’s domicile at the time
of his death.

POINT (2) [30%] ISSUE: Is Zachary’s will valid in (a) State A and (b) State B? ANSWER: No.
Zachary’s will is invalid under the law of State A because Zachary did not sign it at the end. The
will also is invalid under the law of State B because it was not signed by two witnesses. Therefore,
Zachary died intestate under the laws of both states.

POINT (3)(a) [5%] ISSUE: If Zachary’s will is invalid, is Zachary’s child Alex (who was the
biological child of Zachary and his deceased wife) entitled to a share of his assets in State A and
State B? ANSWER: Yes. Alex is entitled to share in the house, the farm, and the bank account
under the applicable laws of each state, as Alex is a biological child of Zachary and was born in
wedlock.

POINT (3)(b) [15%] ISSUE: If Zachary’s will is invalid, is Zachary’s child Brian (who was the
biological child of Zachary’s deceased wife and her first husband, but Zachary adopted Brian when
Brian was 12) entitled to a share of his assets in State A and State B? ANSWER: Brian takes a
share of the house and the bank account under State A law if “children” includes adopted children;
Brian takes a share of the farm under State B law because adopted children are treated the same as
biological children.

POINT (3)(c) [20%] ISSUE: If Zachary’s will is invalid, is Zachary’s child Carrie (who was the
biological child of Zachary and a woman whom Zachary never married) entitled to a share of his
assets in State A and State B? ANSWER: Yes. Because State A’s law disallowing inheritance by all
nonmarital children does not meet constitutional standards, Carrie is entitled to take as a child of
Zachary.

ANSWER DISCUSSION:

The postmortem distribution of personal property is governed by the law of the state in which the
decedent was domiciled at the time of his death. The postmortem distribution of real property is governed
by the law of the situs. Thus, State A law controls the disposition of Zachary’s bank account and
Zachary’s house, and State B law controls the disposition of Zachary’s farm. In State A, Zachary’s will is
invalid because Zachary did not sign it at the end. In State B, Zachary’s will is invalid because it was not
signed by two witnesses. Thus, Zachary died intestate in both states. Under the laws of both State A and
State B, Alex is entitled to a share of Zachary’s estate. In State A, Brian is entitled to a share if the word
“children” in the State A statute is construed to include adopted children. Although the State A intestate-
distribution statute excludes nonmarital children, Carrie is nonetheless entitled to a share because
intestate-distribution laws that exclude nonmarital children whose paternity has been established are
unconstitutional. Under State B law, Alex and Carrie are clearly entitled to shares of Zachary’s assets.
Brian also is entitled to a share because the State B statute specifically includes adopted children.

Seperac-J19 Exam-Released MEE Essay Compilation © 2016-2020 1286


ANSWER EXPLANATION:

Explanation to Point-One (30%):

Because the disposition of real property is governed by the law of the situs, State A law will govern the
disposition of Zachary’s house and State B law will govern the disposition of Zachary’s farm. State A law
also governs the distribution of the personal property, the bank account, because State A was the
decedent’s domicile at the time of his death.

The law of the state where the real property is located governs the disposition of real property. This
approach reflects the situs state’s interest in the regularity of titles and the interests of third parties who
rely on local land records. Therefore the law of State A controls the disposition of the house and the law
of State B controls the disposition of the farm. Each state’s statutes must be consulted to determine the
validity of Zachary’s will and, if the will is invalid, the rules governing intestate succession.

The law of the state in which the decedent was domiciled at his death governs the disposition of personal
property. Because Zachary was a domiciliary of State A, State A law governs the disposition of his bank
account even though the account was maintained at a bank in State B.

[NOTE: Comments to the Second Restatement suggest that when the situs state (State B) has a statute
recognizing the validity of a will properly executed in the state where the testator was domiciled, it may
determine the validity of a landowner’s will under the law in his state of domicile (State A). Most states
have such will-validation statutes, but the facts provide that State B has no such law and instead expressly
requires nonresidents to comply with its will-execution requirements. The question is designed, in part, to
test the examinee’s ability to read and apply a specific statute. Zachary’s will is invalid under State A law
in any event.]

Explanation to Point-Two (30%):

Zachary’s will is invalid under the law of State A because Zachary did not sign it at the end. The will also
is invalid under the law of State B because it was not signed by two witnesses. Therefore, Zachary died
intestate under the laws of both states.

Although State A permits holographic (i.e., handwritten and unwitnessed) wills, State A law expressly
requires the testator to have “signed” the holograph at the end. Zachary’s will is shown in its entirety in
the facts; Zachary did not sign it at the end. Therefore, the will is invalid under the laws of State A and
Zachary’s bank account and his house in State A pass in accordance with State A’s laws of intestate
succession.

State B validates signed holographic wills only if they have been signed by two witnesses, and Zachary’s
will was not. Therefore Zachary’s farm passes in accordance with State B’s laws of intestate succession.

Therefore, in both State A and State B, University, the beneficiary under the invalid will, takes nothing.

Explanation to Point-Three(a) (5%):

Alex is entitled to share in the house, the farm, and the bank account under the applicable laws of each
state, as Alex is a biological child of Zachary and was born in wedlock.

Seperac-J19 Exam-Released MEE Essay Compilation © 2016-2020 1287


The intestacy laws of both State A and State B provide that the property of an intestate decedent passes to
his surviving children. There is no possible argument, under either state’s law, that a marital child like
Alex would be excluded from the class of surviving children who take.

Explanation to Point-Three(b) (15%):

Brian takes a share of the house and the bank account under State A law if “children” includes adopted
children; Brian takes a share of the farm under State B law because adopted children are treated the same
as biological children.

Under State A law, an intestate’s property passes to his “surviving children,” excluding nonmarital
children. The statute is silent on the status of adopted children.

At common law, only blood relations could inherit from an intestate decedent. Although all states today
grant adopted children inheritance rights in at least some circumstances, there is typically an explicit
statutory command that achieves this result. State A has no statutory provision expressly altering the
common law. In the absence of such a statute, a court might conclude that had the legislature intended to
give adopted children the same rights as biological children, it would have said so; alternatively, it might
conclude, based on general nondiscrimination goals, that the legislature must have intended to give
adopted children the same rights as biological children. There are cases going both ways on this issue.

In State B, Brian clearly inherits because the statute says so.

[NOTE: The examinee’s conclusion on this point is less important than his or her demonstrated ability to
recognize the statutory ambiguity and formulate arguments in support of a position.]

Explanation to Point-Three(c) (20%):

Because State A’s law disallowing inheritance by all nonmarital children does not meet constitutional
standards, Carrie is entitled to take as a child of Zachary.

Under the common law, a nonmarital child could not inherit from either parent. Today, all states grant
nonmarital children the right to inherit from their mothers and to inherit from their fathers when at least
one statutorily defined method of establishing paternity has been satisfied. The Supreme Court has held
that a statute disallowing inheritance by a nonmarital child from her father when the father’s paternity has
been adjudicated during his lifetime is unconstitutional. Because Zachary’s paternity of Carrie was
adjudicated during his lifetime, Carrie would be entitled to a share of Zachary’s estate despite the
language of the State A statute. Carrie also takes under the laws of State B because she is Zachary’s
biological child.

Seperac-J19 Exam-Released MEE Essay Compilation © 2016-2020 1288


#303-FEB 2012–MEE Q05: QUESTION FIVE (WILLS-ESTATES)

Five years ago, Testator asked her attorney to draft a will that would leave Testator’s entire estate to
Nephew. One week later, the attorney mailed to Testator a document captioned “Last Will and
Testament.” Although the document complied with Testator’s instructions, Testator did not sign it or have
it witnessed.

Three years ago, Testator called her attorney and said, “I want my 400 shares of XYZ Corporation
common stock to go to Aunt instead of Nephew.” Testator added, “I also want my home to go to Cousin.
The house has five bedrooms, and Cousin has such a large family.” Testator told the attorney that her
home was located at 340 Green Avenue, Springfield, State A.

Subsequently, the attorney sent Testator a document stating in its entirety:

I, Testator, being of sound and disposing mind, give my home, located at 340 Green Avenue, Springfield,
State A, to Cousin and my 400 shares of XYZ Corporation common stock to Aunt. In all other respects, I
republish my will.

Upon receipt of this document, Testator properly executed it.

Two years ago, Testator sold her five-bedroom house at 340 Green Avenue and used the proceeds to
purchase a two-bedroom house located at 12 Elm Street in Springfield. The same year, Testator received
200 shares of XYZ common stock from XYZ Corporation in the form of a “dividend paid in stock.”

Three weeks ago, Testator died. Her probate estate consists of $200,000, her house at 12 Elm Street, and
600 shares of XYZ Corporation common stock, consisting of Testator’s original 400 shares and the 200-
share stock dividend.

Testator is survived by Daughter, Daughter’s child (Grandson), Nephew, Cousin, and Aunt.

Fifteen years ago, Daughter was convicted of murdering her father, Testator’s husband. Testator and
Daughter have had little contact since Daughter’s conviction, and Daughter remains in prison.

Testator is a resident of State A, and all of Testator’s assets are located in State A.

How should Testator’s probate assets be distributed? Explain.

Seperac-J19 Exam-Released MEE Essay Compilation © 2016-2020 1289


#303: F12-5 MEE: ANSWER: NCBE (WILLS-ESTATES)

POINT (1) [20%] ISSUE: Was the document captioned “Last Will and Testament” republished by
codicil? ANSWER: No. Testator’s valid partial will did not republish the unexecuted, unwitnessed
document captioned “Last Will and Testament.”

POINT (2) [15%] ISSUE: Was the document captioned “Last Will and Testament” incorporated by
reference into Testator’s valid partial will? ANSWER: No. The document captioned “Last Will and
Testament” was not incorporated by reference into Testator’s valid partial will because the partial
will did not specifically refer to the earlier document or reflect Testator’s intent to incorporate that
will.

POINT (3) [15%] ISSUE: Assuming that the document captioned “Last Will and Testament”
drafted by Testator’s attorney is not a valid will, will Daughter be barred from inheriting from
Testator by the state “slayer statute”? ANSWER: No. Daughter is Testator’s sole heir, and she is
not barred from inheriting from Testator by a slayer statute because these statutes apply only when
the heir has murdered the decedent whose estate is at issue.

POINT (4) [35%] ISSUE: Did Testator’s bequest of her house located at 340 Green Avenue adeem?
ANSWER: Under the common law ademption doctrine, the bequest of Testator’s house to Cousin
adeemed because Testator’s house at 340 Green Avenue, Springfield, State A, was not in Testator’s
probate estate. Possibly under the intent theory of ademption, and certainly under Uniform Probate
Code § 2-606 or a like statute, the house located at 12 Elm Street could pass to Cousin as a
substitute gift.

POINT (5) [15%] ISSUE: Did the dividend of 200 shares of XYZ Corporation common stock pass
to the devisee, Aunt? ANSWER: Yes. Aunt is entitled to both the 400 shares of XYZ Corporation
common stock bequeathed to her by Testator and the 200 additional shares of XYZ Corporation
common stock distributed to Testator as a stock dividend. Thus, Aunt takes all 600 shares of that
stock.

ANSWER DISCUSSION:

Because the document captioned “Last Will and Testament” was never executed by Testator, it cannot be
republished by codicil. Because the partial will (an intended codicil) did not specifically refer to the
document captioned “Last Will and Testament,” that document cannot be incorporated by reference.
Thus, although the partial will is valid and determines the disposition of the assets named therein, the rest
of Testator’s estate passes to Daughter, Testator’s sole heir, through partial intestacy. Nephew gets
nothing. Daughter is not barred from inheriting from Testator, because slayer statutes bar from inheriting
only a beneficiary who killed the decedent whose estate is at issue. Here, Daughter was convicted of
murdering Testator’s husband, not Testator. Thus, Daughter takes the $200,000. Under the common law
ademption doctrine, the bequest of Testator’s house at 340 Green Avenue was adeemed by extinction, and
Cousin would take nothing. Under the Uniform Probate Code or a like statute, however, Cousin would
take Testator’s home at 12 Elm Street as replacement property. Under the law of most states, Aunt would
be entitled to both the 400 shares of XYZ Corporation common stock devised to her and the 200 shares
Testator received as a stock dividend.

ANSWER EXPLANATION:

Seperac-J19 Exam-Released MEE Essay Compilation © 2016-2020 1290


Explanation to Point-One (20%):

Testator’s valid partial will did not republish the unexecuted, unwitnessed document captioned “Last Will
and Testament.”

The document captioned “Last Will and Testament” is not a valid will because it was never executed.
However, the document the attorney next sent was properly executed by Testator, and it is a valid partial
will. It thus determines the disposition of the assets named therein (Testator’s home and 400 shares of
XYZ stock).

A valid partial will, or codicil, that refers to an earlier will is said to republish that will. When
republication takes place, the republished will is deemed to be executed on the same day as the codicil.
Republication can cure defects that might otherwise affect the validity of bequests made under a will. For
example, in a jurisdiction that voids bequests to interested witnesses, a bequest to such a witness can be
saved if the will is republished by a later-executed codicil witnessed by two disinterested witnesses.

However, by most accounts, a document that is not a valid will cannot be republished by codicil. There is
some contrary authority, but those cases involve codicils written on the unexecuted dispositive document
itself. Here, the codicil was a separate document.

Thus, the “Last Will and Testament” unexecuted by Testator (and which left Testator’s entire estate to
Nephew) was not republished when Testator properly executed a partial will, and it will have no effect on
the distribution of Testator’s assets. Nephew gets nothing.

[NOTE: Four states have adopted the dispensing power in the Uniform Probate Code § 2-503. That
power allows a court to validate a will not executed in accordance with the statute of wills if the evidence
is clear and convincing that the testator intended the document to be a will. It is not clear, however,
whether that statute can apply where the will was neither signed by the testator nor witnessed by anyone.
It is also unclear that Testator intended the first, unsigned document to be her will. Examinees who
believe that the document captioned “Last Will and Testament” can be saved under the dispensing power
would have to conclude that the later document was a codicil only and that assets not disposed of by the
later codicil pass to Nephew under the earlier will.]

Explanation to Point-Two (15%):

The document captioned “Last Will and Testament” was not incorporated by reference into Testator’s
valid partial will because the partial will did not specifically refer to the earlier document or reflect
Testator’s intent to incorporate that will.

“A writing that is not valid as a will but is in existence when a will is executed may be incorporated by
reference into the will if the will manifests an intent to incorporate the writing and the writing to be
incorporated is identified with reasonable certainty.” The Uniform Probate Code allows for incorporation
by reference when the will describes “the writing sufficiently to permit its identification”.

Here, the document captioned “Last Will and Testament” drafted by Testator’s attorney was in existence
at the time Testator executed the second, partial will disposing of Testator’s real estate and stock.
However, it is unclear whether the partial will identifies the “Last Will and Testament” with “reasonable
certainty.”

Seperac-J19 Exam-Released MEE Essay Compilation © 2016-2020 1291


More importantly, the partial will does not evidence an intention to incorporate the prior “Last Will and
Testament” by reference. Indeed, quite to the contrary, the only possible “reference” to the first document
is in the statement “I republish my will.” “I republish” is not the type of reference to a prior document that
an experienced attorney would use to incorporate a document by reference; “republish” is a term of art
that has legal consequences.

Thus, the “Last Will and Testament” first drafted by the attorney was not incorporated by reference into
the second document, Testator’s valid partial will. Therefore, assets not passing under the valid partial
will should pass to Testator’s heir(s) by intestacy.

Explanation to Point-Three (15%):

Daughter is Testator’s sole heir, and she is not barred from inheriting from Testator by a slayer statute
because these statutes apply only when the heir has murdered the decedent whose estate is at issue.

Because Testator’s valid will did not dispose of Testator’s entire estate, the remaining assets will pass to
Testator’s heirs. Under all intestate succession statutes, Daughter would be Testator’s only heir.

Although murderers are often barred from inheriting under a “slayer statute,” such a statute would not bar
Daughter from inheriting from Testator because these statutes apply only when the heir killed the
decedent whose estate is at issue. The Uniform Probate Code bars a legatee or heir from taking a share of
the estate when he or she killed the decedent. No slayer statute bars an heir from inheriting from a
decedent’s estate because the heir killed another person, even if the heir’s victim left property to the
decedent that is included in the decedent’s estate.

[NOTE: If an examinee erroneously concludes that Daughter is barred from taking a share of Testator’s
estate, the examinee should conclude that whatever Daughter would have taken will pass to her child,
Grandson, absent a statute that bars both the slayer and the slayer’s descendants from taking.]

Explanation to Point-Four (35%):

Under the common law ademption doctrine, the bequest of Testator’s house to Cousin adeemed because
Testator’s house at 340 Green Avenue, Springfield, State A, was not in Testator’s probate estate. Possibly
under the intent theory of ademption, and certainly under Uniform Probate Code § 2-606 or a like statute,
the house located at 12 Elm Street could pass to Cousin as a substitute gift.

Under the common law ademption doctrine, if specifically devised property (i.e., property that is
specifically described in the will) is not in the testator’s estate when the testator dies, the bequest adeems
(i.e., fails). Under this common law doctrine, the testator’s intentions are irrelevant; all that matters is
whether the testator owned the specifically devised asset at his or her death.

Some modern courts have rejected the common law “identity test” for ademption in favor of an intent test.
Under the intent test, a beneficiary of specifically devised property is entitled to substitute property that
was owned by the testator at his or her death if the beneficiary proves that the testator intended the
beneficiary to take the substitute property.

Testator’s bequest of the property at 340 Green Avenue was a specific bequest and thus subject to the
ademption doctrine. Under the common law identity test, the bequest would adeem because Testator did
not own this property at her death. The home would then pass to Daughter as Testator’s heir.

Seperac-J19 Exam-Released MEE Essay Compilation © 2016-2020 1292


Under the intent test, Cousin would be entitled to Testator’s home at 12 Elm Street if he or she could
establish that Testator wanted Cousin to take this home as a substitute for the property at 340 Green
Avenue. It is unclear whether Cousin could make such a showing. Indeed, the fact that Testator told her
attorney that she wanted Cousin to have the five-bedroom house at 340 Green Avenue because “Cousin
has such a large family” arguably suggests that Testator did not want Cousin to take the two-bedroom
house at 12 Elm Street, which would likely be too small for Cousin’s family.

[NOTE: An examinee who demonstrates an understanding of the intent theory of ademption should not be
penalized if he or she reaches a different conclusion.]

Under the Uniform Probate Code, a specific devisee has the right to any “real property owned by the
testator at death which the testator acquired as a replacement for specifically devised real property.”
Under this or a like statute, Cousin would be entitled to the property at 12 Elm Street because Testator
acquired that property as a replacement for 340 Green Avenue.

Explanation to Point-Five (15%):

Aunt is entitled to both the 400 shares of XYZ Corporation common stock bequeathed to her by Testator
and the 200 additional shares of XYZ Corporation common stock distributed to Testator as a stock
dividend. Thus, Aunt takes all 600 shares of that stock.

Under the common law, a devisee of common stock was entitled to additional shares of that stock
obtained by the testator through a stock split, but not to additional shares acquired as a stock dividend.
Under this approach, Aunt would be entitled to only 400 shares of XYZ common stock.

Today, virtually all jurisdictions treat stock splits and dividends the same way; in each case, additional
shares obtained by the testator go to the specific devisee. Under this approach, Aunt would be entitled to
all 600 shares of XYZ common stock.

Seperac-J19 Exam-Released MEE Essay Compilation © 2016-2020 1293


#304-FEB 2011–MEE Q09: QUESTION NINE (WILLS-ESTATES)

Two years ago, Testator purchased a $50,000 life insurance policy and named Niece as beneficiary.

One year ago, Testator invited three friends to dinner. After dessert had been served, Testator brought a
handwritten document to the table and stated, “This is my will. I would like each of you to witness it.”
Testator then signed and dated the document. The three friends watched Testator sign her name, and
immediately thereafter, they signed their names below Testator’s name.

One month ago, Testator died. Testator was survived by Niece, Cousin, and Son. Son is Testator’s child
from her first marriage. Testator’s second husband, Husband, died six months before Testator. Husband’s
daughter from a prior marriage also survived Testator.

The handwritten document that Testator signed and that the three friends witnessed was found in
Testator’s desk. Its dispositive provisions provide in their entirety:

I, Testator, hereby make my Last Will and Testament.

I give my life insurance proceeds to Cousin.

I give the items listed in a memorandum to be found in my safe-deposit box to Niece.

I give $25,000 each to Church, Library, and School.

I give $40,000 to Husband.

I give the remainder of my assets to Son. >

At Testator’s death, she owned the following assets:

1. The $50,000 life insurance policy, payable on Testator’s death “to Niece”

2. Jewelry worth $15,000

3. A bank account with a balance of $60,000

The jewelry was found in Testator’s safe-deposit box with a handwritten memorandum signed and dated
by Testator the day before she signed her will. The memorandum lists each piece of jewelry and states, “I
want Niece to have all the jewelry here.”

The terms of Testator’s life insurance contract provide that the beneficiary may be changed only by
submitting the change on the insurer’s change-of-beneficiary form to the insurance company.

State law explicitly disallows “all holographic wills and codicils.” To be valid, a will must be
“acknowledged by the testator to the witnesses and signed by the testator in the presence of at least two
attesting witnesses, who shall sign their names below that of the testator within 30 days.”

1. Is Testator’s will valid? Explain.

Seperac-J19 Exam-Released MEE Essay Compilation © 2016-2020 1294


2(a). Assuming that Testator’s will is valid, who is entitled to Testator’s life insurance policy?
Explain.

2(b). Assuming that Testator’s will is valid, who is entitled to Testator’s jewelry? Explain.

2(c). Assuming that Testator’s will is valid, who is entitled to Testator’s bank account? Explain.

Seperac-J19 Exam-Released MEE Essay Compilation © 2016-2020 1295


#304: F11-9 MEE: ANSWER: NCBE (WILLS-ESTATES)

POINT (1) [20%] ISSUE: Was Testator’s will validly executed? ANSWER: Yes. Because Testator
complied with the statutory execution requirements, her will is valid.

POINT (2) [20%] ISSUE: Should the life insurance proceeds be distributed to Niece or Cousin?
ANSWER: Niece will probably take Testator’s life insurance because, in most states, Testator could
not change her life insurance beneficiary except as specified in her life insurance contract.

POINT (3) [20%] ISSUE: Does the handwritten memorandum found in Testator’s safe-deposit box
determine the disposition of Testator’s jewelry? ANSWER: Yes. Niece will probably take
Testator’s jewelry because, in most states, the memorandum describing the bequest was
incorporated by reference into Testator’s will.

POINT (4) [20%] ISSUE: Did the bequest to Husband lapse? ANSWER: Yes. The bequest to
Husband lapsed because he predeceased Testator. Under the typical antilapse statute, the bequest
to Husband would not be saved from lapse even though he left issue who survived Testator.

POINT (5) [20%] ISSUE: How should the general bequests in Testator’s will abate? ANSWER:
Testator’s general legacies must be abated because there are insufficient assets to pay all named
beneficiaries. Under standard abatement principles, Son will receive nothing and Library, Church,
and School will receive equal, pro rata general bequests. Niece will take the jewelry because specific
devises abate last. She also will take the life insurance because it passes outside the will.

ANSWER DISCUSSION:

Testator’s will is valid and should be admitted to probate because Testator complied with the statutory
requirements in executing the will. Testator’s life insurance proceeds will probably go to Niece because
life insurance contracts almost always disallow alteration of a beneficiary designation in a life insurance
contract by will, and most states have upheld such contractual requirements. Testator’s jewelry will
probably go to Niece because the memorandum describing the gift to Niece preexisted Testator’s will and
thus was incorporated by reference into Testator’s will. The gift of $40,000 to Husband lapsed because he
predeceased Testator, and bequests to spouses are not typically saved from lapse by an antilapse statute. It
is irrelevant that Husband had a child who survived Testator. The general bequests of $25,000 each to
Church, Library, and School must be abated because there are insufficient funds ($60,000) to pay these
bequests. Because there are no indications in the will specifying a different abatement scheme, Church,
Library, and School will take equal shares (i.e., $20,000) of Testator’s bank account. As a result of the
abatement, nothing remains in the estate to pass to Son. Niece need not contribute to those general
bequests because life insurance proceeds are nonprobate assets not governed by the will. Furthermore, the
devise of items listed in a memorandum was specific, and specific devises are the last to abate.

ANSWER EXPLANATION:

Explanation to Point-One (20%):

Because Testator complied with the statutory execution requirements, her will is valid.

Seperac-J19 Exam-Released MEE Essay Compilation © 2016-2020 1296


Here, state law disallows “all holographic wills and codicils.” A holographic will is an unwitnessed will
in a testator’s handwriting. Testator’s will was not holographic even though written in the testator’s hand
because it was signed and witnessed.

State law does provide that a will is not valid unless it is “acknowledged by the testator to the witnesses
and signed by the testator in the presence of at least two attesting witnesses, who shall sign their names
below that of the testator within 30 days.” Testator signed her will in the presence of three witnesses who,
on the same day, signed their names below hers. Testator acknowledged the instrument to be her will
when she stated, “This is my will. I would like each of you to witness it.” Therefore, Testator’s will is
valid because its execution complied with the law of the state.

Explanation to Point-Two (20%):

Niece will probably take Testator’s life insurance because, in most states, Testator could not change her
life insurance beneficiary except as specified in her life insurance contract.

If an insured dies while a life insurance policy is in effect, the policy proceeds are payable to the named
beneficiary. Although the owner of a life insurance policy typically retains the right to change the named
beneficiary without obtaining that beneficiary’s consent, the owner must do so in accordance with
procedures specified in the life insurance contract.

Life insurance contracts almost never permit a change of beneficiary by will, and courts have almost
invariably upheld such restrictions. Courts have sometimes found that a beneficiary change that does not
comply with the terms of the insurance contract is valid if the policyholder has “substantially complied”
with the contract by taking all reasonable steps within his or her power to make the change in accordance
with the contract terms. Courts in a few states have also upheld a change of beneficiary by will. In such
instances, the courts rely on the insurance company’s failure to object.

Here, Testator attempted to change the beneficiary of her policy by naming Cousin as the beneficiary in
her will. Assuming that Testator had a standard life insurance contract, such a change would be
disallowed. There is nothing in the facts to show that Testator made any attempt to comply with the
formalities required by the insurance contract. Thus, unless this state follows the minority approach,
Niece, the named beneficiary in the policy, will take the policy proceeds.

Explanation to Point-Three (20%):

Niece will probably take Testator’s jewelry because, in most states, the memorandum describing the
bequest was incorporated by reference into Testator’s will.

In most jurisdictions, a writing that exists at the time a will is executed may be incorporated by reference
into the will. The will must describe the writing with sufficient particularity that it can be identified, but
the writing need not be witnessed or executed with testamentary formalities.

Here, the memorandum describing the bequest of jewelry to Niece is clearly identified in Testator’s will,
and it was found in the location described by Testator. The memorandum is dated on the day before
Testator’s will was executed, so it qualifies as a document that can be incorporated by reference. Thus,
assuming that this state follows the approach of the overwhelming majority of states and allows
incorporation by reference, Niece will take the jewelry.

Seperac-J19 Exam-Released MEE Essay Compilation © 2016-2020 1297


In addition, some states recognize the right of a testator to dispose of tangible personal property by a
signed memorandum, whether prepared before or after the execution of the will, even though it is not
executed with the formalities required of wills.

[NOTE: The difference between UPC § 2-513 and the incorporation-by-reference doctrine is that the
former allows a testator to dispose of tangible personal property by a signed memorandum executed both
before and after the will was signed. Under the incorporation-by-reference doctrine, the memorandum
must be in existence at the time the will is executed.]

Explanation to Point-Four (20%):

The bequest to Husband lapsed because he predeceased Testator. Under the typical antilapse statute, the
bequest to Husband would not be saved from lapse even though he left issue who survived Testator.

If a beneficiary does not survive the testator, the bequest lapses and falls into the residue of the estate.
Under the common law, lapse occurred whenever a beneficiary predeceased the testator and the testator
specified no alternate disposition of the assets in question. A lapsed bequest passes into the residue of the
testator’s estate.

Today, all states have antilapse statutes. Under a typical antilapse statute, if a beneficiary dies before the
testator and the beneficiary was both related by blood to the testator within a certain degree of relationship
and had issue who survived the testator, the bequest to the deceased beneficiary is saved from lapse and
the deceased beneficiary’s issue takes in lieu of the deceased beneficiary.

The typical antilapse statute, therefore, would not save a bequest to a spouse even if the spouse
predeceased the testator leaving issue who survived the testator.

Thus, the $40,000 bequest to Husband lapses and passes into the residuary estate.

[NOTE: In New Hampshire and Kansas the result differs, because their antilapse statutes would apply to
Husband. In those states, his share would pass to his daughter.]

Explanation to Point-Five (20%):

Testator’s general legacies must be abated because there are insufficient assets to pay all named
beneficiaries. Under standard abatement principles, Son will receive nothing and Library, Church, and
School will receive equal, pro rata general bequests. Niece will take the jewelry because specific devises
abate last. She also will take the life insurance because it passes outside the will.

When the assets of a testator’s estate are insufficient to pay all of the bequests payable under the testator’s
will, these bequests are reduced, or abated. Under the common law and in most states, unless the testator
specifies a different abatement scheme, testamentary bequests abate in the following order: (1) residuary
bequests, (2) general bequests (i.e., bequests of a fixed dollar amount), and (3) specific bequests (i.e.,
identifiable property such as “my jewelry”). Abatement within each category is pro rata.

Here, there are insufficient assets in Testator’s estate to pay the bequests made under her will, and
Testator specified no abatement scheme. Thus, the residuary bequest to Son abates first; Son will receive
nothing. The general bequests to Church, Library, and School, totaling $75,000, abate next and pro rata as
each of them was bequeathed $25,000. Thus, Church, Library, and School will each receive $20,000 from

Seperac-J19 Exam-Released MEE Essay Compilation © 2016-2020 1298


the available $60,000. Niece need not contribute because the bequest of jewelry is a specific devise and,
assuming this state follows the majority view, the life insurance is a nonprobate asset that will not be
taken into account when abating the testamentary bequests.

[NOTE: If Husband’s daughter is entitled to the $40,000 bequest to Husband, there will be $115,000 of
general legacies to abate proportionally when distributing the $60,000.]

Seperac-J19 Exam-Released MEE Essay Compilation © 2016-2020 1299


#305-JUL 2010–MEE Q03: QUESTION THREE (WILLS-ESTATES)

Three years ago, Testator told his attorney to draft a will leaving $20,000 to his Sister and the balance of
his estate to his children. Testator told his attorney that he was divorced and that he had two children,
Abby and Bruce, both biological children born during Testator’s first marriage. Testator did not tell the
attorney that he had adopted his stepchild, Carl, when Carl was two years old.

The attorney prepared a typed will based on Testator’s instructions. When Testator came to the attorney’s
office to execute the will, the attorney placed the three unstapled pages of the will on her desk and said to
Testator, “Please sign your will.”

Page 1 of the will included introductory clauses and two bequests reading: “I give $20,000 to Sister. I
give the balance of my estate to my children, in equal shares.” Pages 2 and 3 of the will contained clauses
relating to the responsibilities of the executor. At the bottom of page 3, there were lines for Testator’s and
his witnesses’ signatures and an attestation clause.

Testator, in the presence of two witnesses, read the three pages, declared the document on the attorney’s
desk to be his will, and signed it on the line provided on page 3. Both witnesses, who were able to see all
three pages, signed their names underneath Testator’s signature and again under the attestation clause.
The attorney thereafter folded the three pages together and gave them to Testator.

Two years ago, Testator decided that he wanted to leave more money to Sister. To accomplish this,
Testator crossed out the bequest reading “I give $20,000 to Sister” on page 1 of the will and wrote above
the crossed-out phrase, “I give $40,000 to Sister.”

Last month, Testator died. Testator’s will, all three pages folded together, was found in a night table in
Testator’s bedroom. Abby, Bruce, Carl, and Sister survived Testator. In addition, Don claims to be
Testator’s nonmarital child.

The only relevant state statutes provide:

I. “No will or codicil thereto is valid unless signed by the testator and two attesting witnesses.”

II. “A will may be revoked, in whole or in part, by destruction or cancellation.”

III. “If a decedent dies intestate survived by children and no spouse, the decedent’s entire estate passes to
his children, in equal shares.”

1. Is Testator’s will valid? Explain.

2. To whom should Testator’s estate be distributed? Explain.

Seperac-J19 Exam-Released MEE Essay Compilation © 2016-2020 1300


#305: J10-3 MEE: ANSWER: NCBE (WILLS-ESTATES)

POINT (1) [25%] ISSUE: Did Testator execute a valid will when he did not sign the only page
containing bequests, but all three pages of the will were physically present at the time he signed the
last page and Testator intended them to be his will? ANSWER: Yes. Because all pages of Testator’s
will were present at the same time that Testator signed the last page of his will and the pages were
intended by Testator to be his will, the will is valid.

POINT (2) [15%] ISSUE: Was Testator’s handwritten codicil valid? ANSWER: No. Testator’s
handwritten codicil to the will is invalid because state law requires all testamentary provisions to be
signed by the testator and witnessed by two attesting witnesses.

POINT (3) [30%] ISSUE: Did Testator revoke the bequest to Sister by striking out the clause in his
will giving her $20,000? ANSWER: No. Notwithstanding the fact that Testator crossed out the
bequest to Sister, the bequest is not revoked. Under the doctrine of dependent relative revocation, a
revocation based on a mistaken assumption of law or fact (here, the validity of the handwritten
bequest) is not given effect if it appears that the testator would not have revoked his will or some
portion thereof had he known the truth.

POINT (4) [25%] ISSUE: Does Testator’s gift to “children” include adopted children and
nonmarital children? ANSWER: The bequest to “my children” would include Testator’s two
biological children born during marriage and his adopted stepchild. It is unclear whether it would
include Testator’s nonmarital child.

POINT (5) [5%] ISSUE: May a court reform Testator’s will when Testator’s likely intention
conflicts with the will as drafted? ANSWER: Yes. Most jurisdictions disallow reformation of a will
to correct a mistake. In those states, the bequest to “my children” would stand despite Testator’s
apparent intention to benefit only his biological children, Abby and Bruce.

ANSWER DISCUSSION:

Under the doctrine of integration, all pages of a will that are together when the last page is signed and
witnessed are deemed to have been validly executed. Thus, Testator’s will is valid. Testator’s handwritten
codicil is invalid because it was neither signed by Testator nor witnessed. However, although crossing out
a portion of a will (here, the bequest to Sister) would normally revoke that bequest under this state’s law,
because of the doctrine of dependent relative revocation, Testator’s bequest to Sister was not revoked
because it appears that Testator struck out this bequest on the mistaken assumption that the handwritten
codicil was valid. In all states, a gift to children includes adopted and marital children. In most states, a
gift to children would include a nonmarital child if clear and convincing evidence of paternity was
available. Although Testator appears to have intended to benefit only some of his children, Abby and
Bruce, the will gives the residuary estate to all of Testator’s “children,” and most courts do not allow the
admission of extrinsic evidence to alter the plain meaning of a will. Thus, Sister would receive $20,000
from Testator’s estate and the balance would likely be distributed, in equal shares, to Abby, Bruce, Carl,
and Don, if Don can establish Testator’s paternity. If the will was not validly executed Testator would
have died intestate and his estate would be distributed only to his heirs. His heirs would be Abby, Bruce,
and Carl in all states and, depending upon the state rules relating to proof of paternity, possibly Don.

ANSWER EXPLANATION:

Seperac-J19 Exam-Released MEE Essay Compilation © 2016-2020 1301


Explanation to Point-One (25%):

Because all pages of Testator’s will were present at the same time that Testator signed the last page of his
will and the pages were intended by Testator to be his will, the will is valid.

Under the relevant state statute, a will, to be valid, must be signed by the testator and witnessed by two
witnesses. Like all wills statutes, the statute applicable here does not require that the pages of the will be
stapled or that the testator and witnesses sign each page.

Under the doctrine of integration, a multi-page will is valid even if only the last page is executed as long
as the proponent of the will can establish that all pages were physically present and together when the
testator and witnesses signed the last page of the will and that each page was intended by the testator to be
part of his will. Here, all three pages of the will were present and together when Testator signed the will,
and Testator intended the document to be his will. The proponent of the will may also rely upon a
presumption that all pages of the will were present and together when the last page was signed if the
pages, when read together, set out an orderly dispositional plan. Here, page 1 of the will contained a
bequest consistent with Testator’s instructions to the attorney, and this page was followed by other pages
containing matters commonly included in wills.

In sum, the will was executed properly and is valid.

Explanation to Point-Two (15%):

Testator’s handwritten codicil to the will is invalid because state law requires all testamentary provisions
to be signed by the testator and witnessed by two attesting witnesses.

Under state law, all testamentary provisions, to be valid, must be signed by the testator and witnessed by
two attesting witnesses. Here, Testator attempted to change the bequest to Sister with a handwritten,
unsigned, and unwitnessed writing. This provision is ineffective under state law.

Explanation to Point-Three (30%):

Notwithstanding the fact that Testator crossed out the bequest to Sister, the bequest is not revoked. Under
the doctrine of dependent relative revocation, a revocation based on a mistaken assumption of law or fact
(here, the validity of the handwritten bequest) is not given effect if it appears that the testator would not
have revoked his will or some portion thereof had he known the truth.

Testator crossed out the bequest to Sister with the intention of substituting for it a larger bequest. This
alternative bequest was invalid because it was neither witnessed nor signed by Testator.

Under the doctrine of dependent relative revocation, if a testator revokes a will or bequest based on a
mistaken assumption of law or fact, the revocation is ineffective if it appears that the testator would not
have revoked the bequest had he had accurate information. Here, Testator’s cancellation of the bequest to
Sister was motivated by a desire to give Sister more money. Had Testator known that the alternative
larger bequest would be invalid, he would not have wanted to cancel the lesser bequest to Sister, leaving
her with nothing at all. Thus, under the doctrine of dependent relative revocation, the revocation of the
bequest to Sister is ignored and she is entitled to the $20,000.

Explanation to Point-Four (25%):

Seperac-J19 Exam-Released MEE Essay Compilation © 2016-2020 1302


The bequest to “my children” would include Testator’s two biological children born during marriage and
his adopted stepchild. It is unclear whether it would include Testator’s nonmarital child.

State laws generally provide that the word “children” in a will should be interpreted consistently with the
definition of the word “children” used in determining rights to intestate succession.

Biological children born to a married couple and adopted children are included within the category of
children entitled to take through intestate succession. Thus, Abby, Bruce, and Carl are all included in
Testator’s bequest to his “children.”

In all states, a nonmarital child (here, Don) is included within a bequest to children only if paternity is
established under the relevant statute. In most states, paternity for purposes of intestate succession may be
established by evidence of subsequent marriage of the parents, by acknowledgment by the father, by an
adjudication of paternity during the life of the father, or by other clear and convincing evidence. It is
unclear whether Testator’s paternity of Don was adjudicated or acknowledged during Testator’s lifetime;
it is also unclear whether there is clear and convincing evidence of Testator’s paternity.

In sum, it is uncertain whether Testator’s nonmarital child, Don, will be included within the bequest to
Testator’s “children.”

[NOTE: If evidence of Testator’s paternity is not currently available, Don might seek an order requiring
decedent’s marital children to submit to blood testing in order to establish Testator’s paternity; some
courts have granted such petitions. In recent years, some courts have also permitted exhumation of a
decedent’s remains for purposes of establishing paternity. Other courts have refused to grant such
requests. In one case, a request for an exhumation was denied when postmortem DNA analysis of
decedent’s blood samples established that the petitioner was not decedent’s son.]

Explanation to Point-Five (5%):

Most jurisdictions disallow reformation of a will to correct a mistake. In those states, the bequest to “my
children” would stand despite Testator’s apparent intention to benefit only his biological children, Abby
and Bruce.

The fact that Testator identified only Abby and Bruce as his children when talking with his lawyer is
unlikely to result in an interpretation of the will limiting the word “children” to Abby and Bruce.
Although the “controlling consideration in determining the meaning of a donative document is the
donor’s intention”, a majority of jurisdictions bar the admission of extrinsic evidence to vary the literal
meaning of the words used in a will and refuse to allow reformation of a will to correct a mistake.
“Hypothetical or imaginary mistakes of testators cannot be corrected. The only means for ascertaining the
intent of the testator are the words written and the acts done by him.” Under this formalist approach,
Testator’s attorney could not testify that Testator intended to leave his estate to only Abby and Bruce,
based on their conversations.

In recent years, a few states have begun to move away from the formalist approach and toward the more
liberal approach adopted in the Third Restatement of Property. Under the Restatement, a court may
reform even an unambiguous donative document based on clear and convincing evidence of (a) a mistake
of fact or law or (b) the donor’s intention. For example, one court reformed a will based on the drafting
attorney’s testimony that the omission of particular beneficiary was due to a computer error. Using this
approach, a court would admit the attorney’s testimony to show that Testator’s bequest to “children” was
based on a mistake of law (the meaning of “children” for purposes of intestate succession) and that

Seperac-J19 Exam-Released MEE Essay Compilation © 2016-2020 1303


Testator intended to benefit only Abby and Bruce. However, it is unclear whether such testimony would
meet a clear and convincing evidentiary standard given that Testator did not discuss his intentions toward
Carl and Don with the attorney.

In sum, Abby and Bruce will certainly take a share of Testator’s estate. It is highly probable, but not
certain, that Carl will. It is uncertain whether Don will as well, depending on whether Testator’s
nonmarital children are included in the class gift to “children” under state law.

Seperac-J19 Exam-Released MEE Essay Compilation © 2016-2020 1304


#306-JUL 2009–MEE Q09: QUESTION NINE (WILLS-ESTATES)

Two years ago, Testator, age 70, met Friend, age 50, through a dating service. Testator was a successful
businessman and a widower. Friend worked for low-income wages. Friend showered Testator with
affection and appeared to enjoy sharing his interests.

Three months ago, Testator proposed marriage to Friend, and Friend accepted. Thereafter, Testator
decided to consult Friend’s Brother, an attorney, about executing a will that would provide for Friend
after Testator’s death.

Without Testator’s knowledge, Friend promised Brother that she would “be very generous” to him if
Testator left her everything.

Testator consulted Brother and told him that he would like to leave his entire estate to a testamentary trust
that would give Friend all trust income during her lifetime and give Charity the remaining trust assets
after Friend’s death. Brother told Testator that he would draft a will in accordance with these instructions.
Brother instead drafted a will in which Testator bequeathed all of his assets to a trust, named Friend the
beneficiary of “all trust income during her lifetime,” gave Friend a “general power of appointment
exercisable by deed or will” over trust assets, and named Charity the taker in default of appointment.
After Brother advised Testator that the will reflected Testator’s instructions, Testator properly executed
the will drafted by Brother.

Shortly thereafter, Friend properly executed a will leaving her entire residuary estate to her Sister. Neither
the residuary clause nor any other clause in Friend’s will made reference to the power of appointment in
Testator’s will.

On the way to their wedding, Testator and Friend were in an automobile accident. Testator died
immediately, and Friend died one week later.

Testator left a substantial estate. He was survived by his elderly Uncle and his Niece, both of whom he
had not seen in several years. Friend is survived by Sister and Brother.

1. Is Testator’s will invalid on the basis of undue influence? Explain.

2. Is Testator’s will invalid on the basis of fraud? Explain.

3. If Testator’s will is valid, to whom should Testator’s estate be distributed? Explain.

4. If Testator’s will is invalid, to whom should Testator’s estate be distributed? Explain.

Seperac-J19 Exam-Released MEE Essay Compilation © 2016-2020 1305


#306: J09-9 MEE: ANSWER: NCBE (WILLS-ESTATES)

POINT (1) [26%] ISSUE: Is Testator’s will invalid because it was the product of undue influence?
ANSWER: No. The facts do not establish that Testator’s will is invalid as the product of undue
influence.

POINT (2) [26%] ISSUE: Is Testator’s will invalid because it was induced by fraud? ANSWER:
Yes. Because the facts establish that Testator’s will was induced by fraud in the execution, it is
invalid in whole or in part.

POINT (3) [26%] ISSUE: If Testator’s will is valid, did Friend effectively exercise her general
power of appointment over trust assets with the general residuary clause in her will? ANSWER: If
Testator’s entire will is valid, whether Charity or Sister takes Testator’s estate depends on whether
Friend effectively exercised her general power of appointment. Under the majority view, a
residuary clause in a will that makes no reference to the power is an ineffective exercise.

POINT (4) [21%] ISSUE: If Testator’s will is invalid, who are Testator’s heirs? ANSWER: If
Testator’s entire will is invalid, Testator’s entire estate would pass to Niece under the parentelic
method of determining heirship, but it would pass equally to Niece and Uncle under intestacy
schemes governed by the civil law consanguinity method.

ANSWER DISCUSSION:

If a will was procured through undue influence or induced by fraud, it is invalid. In this case, the facts, if
proven at trial, would not establish undue influence but would establish that Testator’s will was induced
by fraud in the execution. If the will is valid, under the majority view, Testator’s estate will probably pass
to Charity following Friend’s death because Friend did not effectively exercise her power of appointment
by the general residuary clause in her will. If Testator’s will is invalid, his estate passes to Niece if the
intestacy statute utilizes the parentelic method of determining heirship and to both Niece and Uncle if the
statute utilizes the civil law consanguinity method. If only the general power of Friend is set aside as a
product of either undue influence or fraud but the court rules that the remaining provisions of Testator’s
will are valid, then Charity takes.

ANSWER EXPLANATION:

Explanation to Point-One (20-30%):

The facts do not establish that Testator’s will is invalid as the product of undue influence.

A will is invalid if it was executed as the result of undue influence. Undue influence occurs when the
“wrongdoer exerted such influence over the testator that it overcame the [testator’s] free will and caused
the testator to make a donative transfer that the testator would not otherwise have made.”

The burden of establishing undue influence is on the will contestant, who must show that (1) the testator
was susceptible to undue influence, (2) the alleged influencer had the opportunity to exert undue influence
upon the testator, (3) the alleged influencer had a disposition to exert undue influence, and (4) the will
appears to be the product of undue influence.

Seperac-J19 Exam-Released MEE Essay Compilation © 2016-2020 1306


In this case, there are facts showing that Friend had both the opportunity and the disposition to exert
undue influence. Friend and Testator were intimates over a two-year period. Friend showered Testator
with affection. Friend told Brother that she would “be generous” to him if Testator left her everything,
suggesting her desire to influence Testator’s will.

It is less clear that Testator was susceptible to undue influence. Susceptibility is typically based on “age,
personality, physical and mental health, and ability to handle business affairs.” Testator was 70 years old
and a successful business executive. There is no evidence that he was in poor physical or mental health.
On the other hand, Testator does seem to have been influenced by Friend’s demonstrations of affection,
interest, and commitment.

The more important weakness in a will contest based on undue influence is the fact that neither Friend nor
Brother attempted to persuade Testator to give Friend a general power of appointment or bequeath other
assets to her, nor did they attempt to dissuade Testator from making bequests to others or interfere with
his relationships. Instead, Brother used deception to achieve Friend’s goal of obtaining control of
Testator’s assets. While this deception is important to a claim of fraud, it does not fit the usual pattern of
undue influence claims.

Explanation to Point-Two (20-30%):

Because the facts establish that Testator’s will was induced by fraud in the execution, it is invalid in
whole or in part.

A will or provision in a will procured by fraud is invalid. Fraud occurs when a testator is deceived by a
misrepresentation and is thereby led to execute a will that the testator would not otherwise have made.
Most courts additionally require a finding that the misrepresentation was made with the intent to deceive
the testator and for the purpose of influencing the testamentary disposition. Fraud in the inducement
occurs when a person misrepresents facts – for example, whether a proposed beneficiary is alive. Fraud in
the execution occurs when a person misrepresents the character or contents of the instrument signed by
the testator.

The facts of this case support a finding of fraud in the execution. Testator told Brother that he wanted to
leave the trust remainder to Charity. Brother told Testator that the will he had drafted was in accordance
with Testator’s instructions. It was not, because the will gave Friend a general power of appointment over
trust assets; she could appoint those assets to herself and deprive Charity of the remainder. Because
Friend told Brother that she wanted the will to leave her “everything,” the evidence supports a finding that
Brother intended to deceive Testator. If the will contestant succeeds in establishing these facts at trial, the
court should invalidate the will.

If the court finds that the will was tainted by fraud or undue influence, it may invalidate the entire will or
only those portions “infected” by the fraud or undue influence. As there are no facts to suggest that Friend
or Brother played a role in the selection of Charity as taker in default, a court might invalidate the general
power alone, leaving Charity as the residuary taker.

Explanation to Point-Three (20-30%):

If Testator’s entire will is valid, whether Charity or Sister takes Testator’s estate depends on whether
Friend effectively exercised her general power of appointment. Under the majority view, a residuary
clause in a will that makes no reference to the power is an ineffective exercise.

Seperac-J19 Exam-Released MEE Essay Compilation © 2016-2020 1307


If fraud cannot be proven and Testator’s entire will is valid, Friend’s lifetime interest in the trust and
general power of appointment would be valid also. Whether Charity or Sister takes the trust principal
depends on whether Friend, who survived Testator, effectively exercised her general power.

Whether a power is effectively exercised depends on the donee’s intent and any formalities mandated by
the donor. In this case, the donor required no formalities. However, since the donee’s (i.e., Friend’s) will
does not specifically refer to the power, it is unclear whether she intended to exercise her general power.
Under the majority approach, also followed by the Restatement of Property, a residuary article in a
donee’s will that makes no reference to a power of appointment is not an effective exercise of a general
power. Under this approach, Charity, the taker in default of appointment, would take Testator’s estate
because Friend did not effectively exercise the power of appointment. Under the minority view, a general
residuary clause does effectively exercise a power of appointment. If this approach is followed, Sister
would take Testator’s estate.

[NOTE: Friend survived Testator by a week; thus, under the Uniform Probate Code and the Simultaneous
Death Act, she survived Testator because she survived him by 120 hours. If the state has the old version of
the Uniform Simultaneous Death Act, Friend might be deemed to have predeceased Testator if they are
viewed as having died in a common accident or disaster. In such a case, Charity takes as well.]

Explanation to Point-Four (15-25%):

If Testator’s entire will is invalid, Testator’s entire estate would pass to Niece under the parentelic method
of determining heirship, but it would pass equally to Niece and Uncle under intestacy schemes governed
by the civil law consanguinity method.

If the court finds that Testator’s entire will is the product of fraud, his estate will be distributed based on
state rules of intestate succession. Had the accident taken place after Testator and Friend were married,
Friend would have taken Testator’s entire estate. But given that the wedding had not yet taken place, no
state grants Friend a share of Testator’s assets.

Under the so-called parentelic method of determining heirship employed in the Uniform Probate Code
and most state intestacy rules, the issue of an intestate’s parents take to the exclusion of any issue of the
intestate’s grandparents. Niece is a grandchild of Testator’s parents; Uncle is a child of Testator’s
grandparents. Thus, Niece takes to the exclusion of Uncle. Under the civil law consanguinity method
employed in a minority of jurisdictions, Niece and Uncle would share equally because both of them are in
the third degree of consanguinity to Testator.

[NOTE: An applicant might argue that only a portion of the will, the power of appointment, is invalid due
to fraud. In such a case, the power would be excised from the will and upon Friend’s death the trust
assets would pass to Charity.]

Seperac-J19 Exam-Released MEE Essay Compilation © 2016-2020 1308


#307-FEB 2009–MEE Q03: QUESTION THREE (WILLS-ESTATES)

In 2004, Testator duly executed a will providing as follows:

1. I give my 100 shares of XYZ common stock to my cousin Andy.

2. I give my home at 4 Cypress Garden to my cousin Ben.

3. I give my automobile to my friend Carrie.

4. I give $10,000 to my friend Donna.

5. I give the residue of my estate to my friend Ed.

In 2006, Testator sold her home at 4 Cypress Garden and, with the entire sales proceeds, purchased a
condominium as her new home.

In 2007, Testator traded the white automobile that she owned when her will was executed for a blue
automobile.

In 2008, Testator died. At the time of her death, Testator owned 200 shares of XYZ common stock,
having acquired an additional 100 shares as the result of a dividend paid by XYZ to its shareholders in its
own stock. Testator also owned the condominium, the blue automobile, and a $50,000 bank account.

Testator was survived by Andy, Ben, Carrie, Donna, and Ed. She was also survived by Donna’s daughter.
Three months after Testator died, Donna made a valid disclaimer of any rights to the $10,000 bequest to
which she might otherwise be entitled. Testator’s will was admitted to probate.

To whom should Testator’s probate estate be distributed? Explain.

Seperac-J19 Exam-Released MEE Essay Compilation © 2016-2020 1309


#307: F09-3 MEE: ANSWER: NCBE (WILLS-ESTATES)

POINT (1) [25%] ISSUE: Does a bequest of a specific number of shares of stock owned by Testator
when the will was executed include other shares of the same stock acquired after the will was
executed as a result of a stock dividend? ANSWER: Yes. Andy is entitled to the 200 shares of XYZ
stock Testator owned at the time of death unless the jurisdiction does not follow the modern
approach.

POINT (2) [25%] ISSUE: Does a bequest of a specifically described home include another home
acquired with the proceeds from the sale of the specifically described home? ANSWER: In most
states, the bequest of the home to Ben adeems. However, in some states Ben would be entitled to the
substitute home.

POINT (3) [25%] ISSUE: Does a bequest of a generically described automobile adeem when the
automobile owned when the will was executed is traded for another automobile? ANSWER: No.
Carrie is entitled to the blue automobile.

POINT (4) [25%] ISSUE: Does Donna’s disclaimer of the $10,000 bequest result in that bequest
passing to her daughter or to the residuary legatee? ANSWER: The $10,000 bequest to Donna fails
because of Donna’s disclaimer and because Donna is not the type of legatee typically described in
an anti-lapse statute. The bequest passes to Ed, the residuary legatee.

ANSWER DISCUSSION:

In most jurisdictions today, Andy would be entitled to all 200 shares of XYZ stock. Ben might be entitled
to the condo as replacement property. Carrie would be entitled to the blue automobile. The $10,000
bequest to Donna would fail as a result of her disclaimer and would pass to Ed, as the residuary legatee.
However, in jurisdictions that do not follow the modern approach, Andy would only be entitled to 100
shares of the XYZ stock and Ben would not be entitled to the condo because that request adeemed. Any
property not passing to Andy, Ben, or Carrie passes to Ed, the residuary legatee.

ANSWER EXPLANATION:

Explanation to Point-One (20-30%):

Andy is entitled to the 200 shares of XYZ stock Testator owned at the time of death unless the jurisdiction
does not follow the modern approach.

Historically, a bequest of stock owned by a testator when the testator’s will was signed excluded
subsequently acquired shares of the same stock acquired by the testator as the result of a stock dividend.
The rule was justified on the ground that, if the testator had wanted the legatee to take the later-acquired
shares, the testator could have changed the will to take those additionally acquired shares into account. A
contrary rule applied to shares acquired as a result of a stock split.

Today, stock dividends are typically treated like stock splits because, in each case, nothing of value has
been distributed by the corporation to the shareholder. Additional shares of the same company, when
acquired by stock split or stock dividend, are, from the shareholder’s economic perspective, merely a
change in form, not substance. Under the Uniform Probate Code, a devise of stock owned by the testator

Seperac-J19 Exam-Released MEE Essay Compilation © 2016-2020 1310


when the will is executed includes such additional stock owned by the testator at death, whether acquired
by a stock split or a stock dividend. Under the UPC or a like statute, Andy would be entitled to all 200
shares of XYZ stock owned by Testator at her death.

Explanation to Point-Two (20-30%):

In most states, the bequest of the home to Ben adeems. However, in some states Ben would be entitled to
the substitute home.

Under common law, an ademption occurs when the subject matter of specific devise is not found in the
probate estate at the time of the testator’s death. Here, Testator specifically bequeathed to Ben Testator’s
home located at 4 Cypress Garden. Since Testator did not own that home when she died, the bequest
adeems. As a result, the condo passes to Ed, the residuary legatee.

Under UPC § 2-606 or a like statute, however, Ben would be entitled to Testator’s condominium. This
section provides that the devisee of specifically devised real estate is entitled to any “real property owned
by the testator at death which the testator acquired as a replacement for specifically devised real
property.” The facts support the conclusion that Testator purchased the condominium as a replacement
home for the home that was specifically devised to Ben.

The “replacement property” concept appears to have little support in case law. However, the provision of
the UPC should be applied by courts in those cases “in which the result appears to be consistent with the
testator’s intent.”

Explanation to Point-Three (20-30%):

Carrie is entitled to the blue automobile.

Because of the time-honored rule of construction that a will “speaks” at the time of death, a bequest of
generically described property (e.g., my automobile) applies to property that meets the generic description
at the testator’s death. Thus, the bequest of the automobile is unlike the bequest of the home in that the
latter, but not the former, is described in a non-generic manner and thus is subject to the rules of
ademption.

[NOTE: At first blush an examinee might think that the bequest of the automobile raises an ademption
problem because the automobile owned when the will was signed (the white automobile) was not the same
as the automobile owned when Testator died (the blue automobile). However, this is not an ademption
problem because Testator did not refer to a specific automobile when executing her will. Also, an
examinee might analyze this under the doctrine of independent significance and get to the same result.]

Explanation to Point-Four (20-30%):

The $10,000 bequest to Donna fails because of Donna’s disclaimer and because Donna is not the type of
legatee typically described in an anti-lapse statute. The bequest passes to Ed, the residuary legatee.

If Donna had not disclaimed the $10,000 legacy, it would have passed to her under the provisions of
Testator’s will. However, she did disclaim the bequest. Under the typical disclaimer statute, if a legatee
disclaims a general bequest, the bequest passes as if the disclaimant had predeceased the testator.

Seperac-J19 Exam-Released MEE Essay Compilation © 2016-2020 1311


When a disclaimant is deemed to have predeceased the testator, the question arises whether the bequest to
that disclaimant passes to her issue. The answer here depends upon the application of the anti-lapse
statute. The typical anti-lapse statute would not apply here because Donna was a friend, not a relative, of
Testator. Under the UPC, the anti-lapse statute is applicable to a legatee who is a grandparent or issue of a
grandparent of the testator. If the anti-lapse statute is inapplicable, then the $10,000 passes to Ed as the
residuary legatee.

[NOTE: If the anti-lapse statute did apply, as it might in a distinct minority of states, then the bequest to
Donna would pass to Donna’s daughter. However, given how few states have adopted this view, an
examinee who merely said that Donna’s disclaimed bequest passed to her issue should receive little or no
credit unless that is the rule in the applicant’s state.]

Seperac-J19 Exam-Released MEE Essay Compilation © 2016-2020 1312


#308-FEB 2008–MEE Q01: QUESTION ONE (WILLS-ESTATES)

Six years ago, Testator retired from his work as a business executive. Testator continued to serve as a
trustee of several nonprofit organizations and manage all of his own financial affairs. He maintained these
activities until his death.

Five years ago, Testator hired a housekeeper, Harriet.

Four years ago, Harriet began to ask Testator to provide for her in his will. She also began to interfere
with Testator’s relationship with his daughter, Doris. When Doris called, Harriet sometimes falsely told
her that Testator was sleeping and could not talk on the phone. When Doris came to visit Testator, Harriet
often stayed in the room to overhear their conversations. Harriet also made critical remarks about Doris to
Testator and told him that Doris should visit him more regularly.

On a number of occasions, Harriet threatened to quit if Testator did not provide for her in his will. These
threats made Testator fearful, particularly during the last year of his life when his declining health made
him increasingly dependent on Harriet.

Six months ago, Harriet again threatened to quit if Testator did not provide for her in his will and told
Testator that he should see her attorney. Testator told Harriet: “Stop bugging me. I’ll see my own
attorney.”

Three months ago, Testator executed a will in accordance with the applicable statute of wills. The will
was drafted by Testator’s attorney pursuant to Testator’s handwritten instructions.

The will specified as follows:

“I leave my estate in equal shares to my housekeeper, Harriet, and my daughter, Doris.”

This is the only will Testator ever executed.

Testator recently died at age 78. Testator left a substantial estate.

Both Harriet and Doris survived Testator. Testator was also survived by a son (Sam), a grandchild (Ella),
who was the child of Doris, and a grandchild (Fred), who was the child of Testator’s son, Bob. Both
Testator’s spouse and Bob predeceased Testator. Testator and Sam had been estranged for several years
prior to the time of Testator’s death.

1. Is the will invalid in whole or in part? Explain.

2. Assuming the will is invalid in whole, to whom and in what shares should Testator’s estate be
distributed? Explain.

3. Assuming the will is invalid in part, to whom and in what shares should Testator’s estate be
distributed? Explain.

Seperac-J19 Exam-Released MEE Essay Compilation © 2016-2020 1313


#308: F08-1 MEE: ANSWER: NCBE (WILLS-ESTATES)

POINT (1) [44%] ISSUE: Did a housekeeper unduly influence a Testator to make her a beneficiary
under his will? ANSWER: Yes. The court may find that Testator’s will, in whole or in part, was
the product of Harriet’s undue influence.

POINT (2) [17%] ISSUE: If a person unduly influenced the Testator, is his will partially or entirely
invalid? ANSWER: Yes. If the court finds that the bequest to Harriet was the product of undue
influence, it may invalidate the entire will or only the bequest to Harriet.

POINT (3)(a) [17%] ISSUE: To whom should Testator’s estate be distributed if the will is entirely
invalid? ANSWER: If the entire will is invalidated, Testator’s estate would pass to his heirs, Doris,
Sam, and Fred.

POINT (3)(b) [22%] ISSUE: To whom should Testator’s estate be distributed if the will is partially
invalid? ANSWER: If the bequest to Harriet alone is invalidated, Harriet’s one-half share of the
estate would either pass to Doris in its entirety or be distributed equally to Doris, Sam, and Fred,
depending on whether the jurisdiction applies the common law “no residue of a residue” rule.

ANSWER DISCUSSION:

Testator’s will is invalid, in whole or in part, if it was the product of undue influence. Here, the facts are
not conclusive, but may support an undue influence finding. If the will is valid, the estate will pass in
equal shares to Harriet and Doris. If the bequest to Harriet was the product of undue influence, the court
may invalidate the entire will or only Harriet’s bequest. If the entire will is invalidated, the estate would
pass under the laws of intestate succession: Doris, Sam, and Fred, who would take as the representative of
his deceased parent, Bob, would take equal 1/3 shares. If only the bequest to Harriet is invalidated, one-
half of the estate would pass to Doris under the will and the remaining half would pass either to Doris as
the remaining residuary legatee or, if the common law “no residue of a residue” rule is applied, to Doris,
Sam, and Fred under the laws of intestate succession. In that case, Doris would take 2/3 of the estate, and
Sam and Fred would each take 1/6.

ANSWER EXPLANATION:

Explanation to Point-One (35-45%):

The court may find that Testator’s will, in whole or in part, was the product of Harriet’s undue influence.

If a will or portion of a will is the product of undue influence, it may be set aside. Undue influence occurs
when the “wrongdoer exerted such influence over the testator that it overcame the [testator’s] free will
and caused the testator to make a donative transfer that the testator would not otherwise have made.” The
burden of establishing undue influence is on the will contestant, who must show that: (1) the testator was
susceptible to undue influence, (2) the alleged wrongdoer had the opportunity to exert undue influence
upon the testator, (3) the alleged wrongdoer had a disposition to exert undue influence, and (4) the will
appears to be the product of undue influence. Undue influence may be established by circumstantial
evidence.

Seperac-J19 Exam-Released MEE Essay Compilation © 2016-2020 1314


The facts of this case, if proven at trial, would certainly establish that Harriet had opportunity and a
disposition to influence Testator. Harriet lived in Testator’s home, criticized and limited his contacts with
Doris, a natural object of Testator’s bounty, and urged him repeatedly to make a will in her favor. The
will contestant should also be able to show that the bequest to Harriet was linked to her attempts to
influence the will, as Testator executed the will only after Harriet threatened to quit and at a time when he
had become increasingly dependent upon her; Testator also bequeathed to Harriet, an employee who was
not the natural object of his bounty, one-half of his probate estate.

The facts do not so clearly establish Testator’s susceptibility to undue influence. On the one hand,
Testator was increasingly ill and dependent on Harriet. He also agreed to see an attorney after Harriet
threatened him. On the other hand, Testator maintained control over his financial affairs until his death,
continued to play an active role as trustee of various nonprofits, refused to see the attorney Harriet
suggested, and communicated his dispositional plan to his own attorney through handwritten instructions.

Because the evidence is inconclusive, the outcome might depend on who bears the burden of proof.
Normally, this rests on the party challenging the will. However, in many jurisdictions, a presumption of
undue influence arises when there is a confidential relationship between the testator and the alleged
influencer and suspicious circumstances surround the drafting of the will. Most of the cases involving
confidential relationships involve attorneys and close relatives; it is not clear that a confidential
relationship between Testator and Harriet could be established in this case. But if it were, then the burden
of proof on the issue of undue influence would shift to Harriet.

Explanation to Point-Two (10-20%):

If the court finds that the bequest to Harriet was the product of undue influence, it may invalidate the
entire will or only the bequest to Harriet.

A finding of undue influence may lead a court to invalidate the entire will or only a portion of it. Most
courts will invalidate only those portions that are “infected” by undue influence. Clearly, if there was
undue influence, it affected the bequest to Harriet and that bequest should be invalidated.

It is less clear whether the entire will was infected by Harriet’s undue influence. Given his estrangement
from Sam, it is possible that Testator would have left his entire estate to Doris had he not left a one-half
share to Harriet. However, it is also possible that Testator might have left a share of the estate to his
grandchild, Fred; the facts do not show that Testator was estranged from either Fred or Fred’s deceased
parent, Bob. Testator might also have left a share to Sam’s child, Ella, or even to Sam himself. Indeed,
Testator might not have executed a will at all. Given this uncertainty, the court might invalidate the entire
will or only the bequest to Harriet.

Explanation to Point-Three(a) (10-20%):

If the entire will is invalidated, Testator’s estate would pass to his heirs, Doris, Sam, and Fred.

Assuming that the court invalidates the entire will as the product of undue influence, Testator’s estate will
be distributed to his heirs. In every jurisdiction, Testator’s heirs would be his two surviving children,
Doris and Sam, and his grandchild, Fred, who takes as the representative of Testator’s deceased child,
Bob. Doris’s child, Ella, would not take as an heir because Doris survived Testator and there is no basis
for Ella to represent her mother.

Explanation to Point-Three(b) (15-25%):

Seperac-J19 Exam-Released MEE Essay Compilation © 2016-2020 1315


If the bequest to Harriet alone is invalidated, Harriet’s one-half share of the estate would either pass to
Doris in its entirety or be distributed equally to Doris, Sam, and Fred, depending on whether the
jurisdiction applies the common law “no residue of a residue” rule.

When a residuary bequest fails, the question arises as to whether the invalidated share passes to the
testator’s heirs or to the remaining residuary legatee(s). Under the common law “no residue of a residue”
rule, the invalid share passes to the testator’s heirs. The theory behind this approach is that, if the invalid
share were to pass to the remaining residuary legatee, it would increase her share without an executed
bequest and thus violate the policies that underlie typical statutory execution requirements. Under the
common law approach, Doris would take one-half of the estate under the will, and the remaining half
would be distributed in equal shares to Doris, Sam, and Fred under the law of intestate succession.

Many courts today reject the common law rule, theorizing that the testator’s execution of a will evidences
an intention to benefit the legatees named in the will to the exclusion of heirs, particularly would-be heirs
not mentioned in the will. This “residue of a residue” approach has been adopted in Uniform Probate
Code § 2-604. Under this approach, Harriet’s invalid share would go to Doris, the only other residuary
legatee. Neither Sam nor Fred would take a share of Testator’s estate.

Seperac-J19 Exam-Released MEE Essay Compilation © 2016-2020 1316


#309-FEB 2007–MEE Q05: QUESTION FIVE (WILLS-ESTATES)

In 2000, Testator executed a valid will. The will provided:

1. I give my 100 shares of stock in XYZ Company to Brother.

2. I give $3,000 to Sister.

3. I give $5,000 to Uncle.

4. I give $10,000 to Cousin.

5. I give the residue of my estate to my alma mater, Polytech.

In 2001, XYZ Company issued its annual dividend in stock. For each 100 shares held, the dividend was 6
shares of XYZ stock.

In 2002, Testator gave $5,000 to Uncle on Uncle’s birthday.

In 2005, Testator died, survived by Brother, Sister, Uncle, and Cousin. Testator’s estate consists of the
following assets: 106 shares of XYZ Company stock (worth $1 per share) and $9,000. Sister made a valid
disclaimer of her interest in Testator’s estate.

How should Testator’s estate be distributed? Explain.

Seperac-J19 Exam-Released MEE Essay Compilation © 2016-2020 1317


#309: F07-5 MEE: ANSWER: NCBE (WILLS-ESTATES)

POINT (1) [25%] ISSUE: How many shares of XYZ Company stock is Brother entitled to claim?
ANSWER: Whether Brother is entitled to 100 or 106 shares of XYZ stock depends on whether state
law allocates stock dividends to a specific legatee.

POINT (2) [25%] ISSUE: What is the effect of Sister’s disclaimer? ANSWER: The effect of Sister’s
disclaimer is that the property passes as if Sister had failed to survive Testator.

POINT (3) [25%] ISSUE: Does the inter vivos gift of $5,000 satisfy the bequest to Uncle?
ANSWER: No. The inter vivos gift of $5,000 does not satisfy the bequest to Uncle.

POINT (4) [25%] ISSUE: Given that there are insufficient assets to carry out all of Testator’s
directions, how should the bequests abate? ANSWER: The bequests abate according to their
classifications, hence in the following order: first the residuary gift to Polytech, then the general
gifts to Uncle and Cousin, pro rata, and last, the specific gift to Brother.

ANSWER DISCUSSION:

Sister’s disclaimer effectively deletes her bequest from the will. The treatment of the remaining bequests
turns on their classification. The specific bequest to Brother entitles him to 100 shares under the common
law view and 106 shares under the Uniform Probate Code and like statutes. The general bequest to Uncle
is not satisfied by the inter vivos gift. The general bequest to Uncle and the general bequest to Cousin will
be abated pro rata. There are insufficient assets to satisfy the residuary bequest to Polytech. If Brother is
not entitled to the six additional shares, two of them go to Uncle and four of them to Cousin in payment of
their general legacies.

ANSWER EXPLANATION:

Explanation to Point-One (20-30%):

Whether Brother is entitled to 100 or 106 shares of XYZ stock depends on whether state law allocates
stock dividends to a specific legatee.

The bequest to Brother of “my 100 shares” is a specific bequest. Many states hold that a stock dividend,
like a cash dividend, is a property interest distinct from stock given by specific bequest. In these states,
Brother is entitled only to 100 shares of XYZ stock.

In states that have adopted Uniform Probate Code § 2-605 or a like statute, a stock dividend is treated like
a stock split instead of a cash dividend. Under this approach, which aims to maintain the legatee’s
percentage of ownership in the corporation, Brother is entitled to all 106 shares.

Explanation to Point-Two (20-30%):

The effect of Sister’s disclaimer is that the property passes as if Sister had failed to survive Testator.

At common law and under the Uniform Probate Code, Sister’s disclaimer causes Testator’s property to
pass as if Sister had failed to survive Testator. Because Sister has no descendants who survive Testator,

Seperac-J19 Exam-Released MEE Essay Compilation © 2016-2020 1318


the state’s anti-lapse statute will not be applicable to the bequest to Sister. The $3,000 bequest to Sister
lapses.

Explanation to Point-Three (20-30%):

The inter vivos gift of $5,000 does not satisfy the bequest to Uncle.

The bequest of $5,000 to Uncle is a general pecuniary bequest. Under the common law, an inter vivos gift
made after a will’s execution can satisfy a general bequest, but only if the testator so intends. Here, there
is no evidence of such intention. Moreover, no presumption of such intention exists where, as here, the
testator does not stand in loco parentis to the legatee. Uniform Probate Code § 2-609 produces the same
result because none of the statutory bases for satisfaction of a bequest by inter vivos gift have been met:
the will does not provide for deduction of the gift, and neither the testator nor the legatee has declared in a
contemporaneous writing that the gift is in satisfaction of the bequest.

Explanation to Point-Four (20-30%):

The bequests abate according to their classifications, hence in the following order: first the residuary gift
to Polytech, then the general gifts to Uncle and Cousin, pro rata, and last, the specific gift to Brother.

Under both the common law and the Uniform Probate Code, the shares of distributees abate in the
following order: (1) property not disposed of in the will, (2) residuary gifts, (3) general gifts, and (4)
specific gifts. Abatement within each category is pro rata. Thus, the residuary gift to Polytech abates first;
Polytech receives nothing. The general gifts to Uncle and Cousin abate next and pro rata, meaning that the
1-to-2 ratio of Uncle-to-Cousin is maintained. Thus, Uncle and Cousin receive $3,000 and $6,000,
respectively, plus (under the majority view but not under the Uniform Probate Code) two and four shares,
respectively, of XYZ Company stock. Finally, Brother receives the XYZ Company stock to which he is
entitled: 100 shares under the common law view and 106 shares under the Uniform Probate Code and like
statutes.

Seperac-J19 Exam-Released MEE Essay Compilation © 2016-2020 1319


#310-JUL 2006–MEE Q03: QUESTION THREE (WILLS-ESTATES)

In 1995, Husband and Wife duly executed a joint will which provided, in relevant part:

Each of us agrees that, when one of us dies, all of our property shall be distributed to the survivor.
Furthermore, upon the death of the survivor we agree that: (1) $1,000 shall be distributed to the person
who is then the pastor of the First Avenue Church; (2) $1,000 shall be distributed to the person named in a
memorandum that the survivor shall leave in our safe deposit box at the Main Street Bank; and (3) at the
survivor’s death, the remainder of the survivor’s property, however acquired, shall be distributed to our
child, Child.

When this joint will was executed, George was the pastor of the First Avenue Church.

In 2000, Husband died. His estate of $150,000 was distributed to Wife pursuant to the joint will.

In 2001, Wife inherited $200,000 from her sister.

In 2002, Wife duly executed a new will providing in relevant part:

Upon my death I give: (1) $1,000 to the person who is then the pastor of the First Avenue Church; (2)
$1,000 to the person named in a memorandum to be left in my safe deposit box; (3) $100,000 to my child,
Child; and (4) the balance of my estate to my boyfriend, John, who has provided me with loving
companionship since my late husband died.

In late 2003, Wife died leaving an estate valued at $400,000. A memorandum, dated February 2, 2003,
and signed by Wife, was found in Wife’s safe deposit box directing that $1,000 be distributed to her
friend, Robin. Wife was survived by John, Robin, Child, George, and Ted, who had been appointed pastor
of the First Avenue Church one week before Wife died. Wife’s 2002 will was duly admitted to probate.

To whom should Wife’s estate be distributed? Explain.

Seperac-J19 Exam-Released MEE Essay Compilation © 2016-2020 1320


#310: J06-3 MEE: ANSWER: NCBE (WILLS-ESTATES)

POINT (1) [41%] ISSUE: Did Husband and Wife have a contractual will such that Child is entitled
to receive the portion of Wife’s estate bequeathed to Child under that joint will? ANSWER: No.
The joint will executed by Husband and Wife constituted their contract that the survivor would not
change their joint plan for distribution of their assets. As a result, Ted and Child can enforce the
provisions of the 1995 will without contesting the 2002 will.

POINT (2) [29%] ISSUE: Does either the doctrine of facts of independent significance, or the rule
that wills “speak at the time of death,” apply to save the bequest in the 1995 will to the pastor?
ANSWER: Yes. Under either the doctrine of facts of independent significance, or the general rule
of construction that wills “speak at the time of death,” Ted, the current pastor of First Avenue
Church, takes $1,000.

POINT (3) [29%] ISSUE: Is Robin entitled to the $1,000 bequest notwithstanding that her status as
a beneficiary was evidenced by an unattested writing? ANSWER: No. The $1,000 bequest to Robin
is invalid because it is not evidenced by a testamentary instrument.

ANSWER DISCUSSION:

Wife’s estate should be distributed as follows: $1,000 to Ted, pastor of First Avenue Church, and the
residue to Child. Child is a contract creditor of the Wife’s estate because Wife and Husband signed a
contractual will in 1995. Robin is entitled to nothing because the attempted bequest to her was evidenced
by an unattested writing. John, who would have been entitled to the bulk of Wife’s estate under the 2002
will, takes nothing because Wife’s entire estate is exhausted by the claims of Ted and Child. John would
take from Wife’s estate only if the 1995 will was not contractual and the 2002 will could not be
successfully contested.

ANSWER EXPLANATION:

Explanation to Point-One (30-40%):

The joint will executed by Husband and Wife constituted their contract that the survivor would not change
their joint plan for distribution of their assets. As a result, Ted and Child can enforce the provisions of the
1995 will without contesting the 2002 will.

Although the existence of a joint will does not by itself establish the existence of a contract between the
two testators to dispose of their property in a certain way, a will contract is created when the joint will
includes material provisions of the contract or has language in it that refers to a contract between the
testators. Here, the language of the joint will “each of us agrees” should be more than sufficient to
establish the existence of a contract between Husband and Wife to dispose of the property in a certain
way.

The contract became irrevocable upon the death of Husband. On the death of one party, leaving in effect a
will which contains the provisions prescribed by the contract, the transaction is said to become an
irrevocable contract as to the survivor. Because the terms of the contract mandated that Wife, the
survivor, distribute all of “her property” in accordance with the joint plan of distribution, Wife was

Seperac-J19 Exam-Released MEE Essay Compilation © 2016-2020 1321


obligated to distribute not only the couple’s joint property, but also the property she acquired after
Husband’s death, in accordance with the will contract as reflected in the 1995 joint will.

The will contract does not by itself invalidate Wife’s 2002 will. Instead, the will contract makes the
beneficiaries under the joint will contract creditors of Wife’s estate. Here, the contract required
distribution of the entire estate to the beneficiaries of the joint will. As a result, although the 2002 will
would properly be admitted to probate, the beneficiaries under that will would take nothing; rather, the
entire estate would be paid to the creditors, Child and Ted, under the prior will.

If applicants conclude that the phrase “each of us agrees” does not sufficiently state contractual intent,
then the 1995 will would not be contractual and would be revoked by the 2002 will to the extent they
were inconsistent. Therefore, Ted would take $1,000. Child would take only $100,000 under the 2002
will and John would take the residue.

Explanation to Point-Two (20-30%):

Under either the doctrine of facts of independent significance, or the general rule of construction that wills
“speak at the time of death,” Ted, the current pastor of First Avenue Church, takes $1,000.

In general, a testator must identify beneficiaries in the will itself; a testator may not change will
beneficiaries without testamentary formalities. The doctrine of facts of independent significance,
however, gives effect to a will provision that disposes of property “by reference to acts and events that
have significance apart from their effect upon the dispositions made by the will.” The principle behind the
doctrine is that the possibility of undue influence or fraud is reduced when the change in beneficiary has
significance apart from the change in the testator’s will. Hence, since the church was unlikely to choose
its pastor in order to assure that the pastor chosen would inherit $1,000 from the estate of Husband and
Wife, that fact the pastor’s identity has significance apart from the bequest. Hence, the doctrine would
give effect to the $1,000 bequest to the then-current pastor, which was Ted, not George.

By contrast, the memorandum in the safe deposit box directing that $1,000 be distributed to Robin cannot
be given effect under this doctrine because that memorandum had no purpose independent of an attempt
to make a bequest.

The bequest to the pastor also is effective under the general rule of construction that a will “speaks” as of
the time of death and at that time Ted is the only person qualifying for the bequest to the pastor of the
First Avenue Church.

[NOTE: Applicants could reach the same result using the so-called “plain meaning” rule. Under this
rule, Ted takes because the words in the will are unambiguous that the bequest passes to the church
pastor who survives Wife.]

Explanation to Point-Three (20-30%):

The $1,000 bequest to Robin is invalid because it is not evidenced by a testamentary instrument.

While a will may incorporate an attempted bequest in a document not executed with the formalities
required of a will, here the attempted bequest to Robin in such a document is invalid because the
incorporation by reference doctrine requires that the document to be incorporated be in existence at the
time the will was signed. Here the document to be incorporated was executed after both the 1995 and the
2002 wills were executed.

Seperac-J19 Exam-Released MEE Essay Compilation © 2016-2020 1322


Furthermore, even if the jurisdiction had adopted a statute like Uniform Probate Code § 2-513, the
bequest would still be invalid. That statute permits tangible personal property to be disposed of in
accordance with the terms of an unattested memorandum without regard to when it was executed.
However, § 2-513 applies only to tangibles; here, Wife attempted to bequeath Robin money, which is not
a tangible.

Lastly, this bequest cannot be sustained under the doctrine of independent significance because the
memorandum’s only significance was to make a testamentary gift.

[NOTE: Applicants who assume that the memorandum is in Wife’s handwriting and that holographic
works are valid under state law could conclude that the $1,000 gift to Robin was valid. However, no facts
state that the memorandum was in Wife’s handwriting.]

Seperac-J19 Exam-Released MEE Essay Compilation © 2016-2020 1323


#311-FEB 2006–MEE Q07: QUESTION SEVEN (WILLS-ESTATES)

Dorothy had three children, Abel, Brandon, and Carrie. Abel had two children, Grandchild 1 and
Grandchild 2; Brandon had three children, Grandchild 3, Grandchild 4, and Grandchild 5; and Carrie had
one child, Grandchild 6.

Following the deaths of all three of her children, Dorothy was judicially appointed the guardian of all six
grandchildren. Dorothy raised all of the grandchildren in her home and loved them all equally.

Five years ago, Dorothy gave $60,000 to Grandchild 6 to help Grandchild 6 buy a new home. The only
statement Dorothy ever made regarding this payment was a contemporaneous statement to Grandchild 6:
“This is for you because I love you.” Dorothy made no other transfers to her grandchildren.

One year ago, Dorothy executed a valid will providing: “I give my entire estate to my heirs, said heirs to
take the same shares thereof that they would have taken had I died intestate.”

Three months ago, Dorothy was visiting Grandchild 1’s home. While Dorothy was working in the front
yard, Grandchild 1 backed a car out of the garage and, inadvertently, albeit negligently, struck Dorothy.
Dorothy later died from the injuries.

Dorothy’s only survivors are all six grandchildren and one great-grandchild, who is a child of Grandchild
1. Dorothy left a probate estate of $120,000.

To whom should Dorothy’s $120,000 probate estate be distributed, and what is the amount of each
person’s share? Explain.

Seperac-J19 Exam-Released MEE Essay Compilation © 2016-2020 1324


#311: F06-7 MEE: ANSWER: NCBE (WILLS-ESTATES)

POINT (1) [35%] ISSUE: Is the lifetime gift of $60,000 to Grandchild 6 an advancement?
ANSWER: No. There are potentially two reasons the gift to Grandchild 6 is not an advancement.
First, the advancement doctrine is inapplicable when the decedent dies with a will. Second, under a
typical advancement statute, lifetime gifts are not an advancement absent evidence that the
transferor intended them as such.

POINT (2) [20%] ISSUE: Is Grandchild 1 barred from sharing in Dorothy’s estate by a “slayer
statute”? ANSWER: No. Grandchild 1 should not be barred from sharing in Dorothy’s estate
under the typical slayer statute because Grandchild 1 did not act with felonious intent.

POINT (3) [45%] ISSUE: To whom should the probate estate be distributed and what is each
person’s share? ANSWER: Assuming both Grandchild 1 and Grandchild 6 share in Dorothy’s
estate, all six grandchildren take equal shares if the estate is distributed to them per capita. If the
estate is distributed per stirpes, the same six grandchildren receive the entire estate but in different
amounts. Under either system, Great-Grandchild takes nothing.

ANSWER DISCUSSION:

The transfer to Grandchild 6 is presumptively a gift, not an advancement. Grandchild 1 is not barred from
sharing in the estate because Grandchild 1 did not feloniously take Dorothy’s life. Thus, Dorothy’s estate
is distributable to her surviving grandchildren. The shares they take will depend on whether they take per
stirpes or per capita under the applicable intestacy statute.

ANSWER EXPLANATION:

Explanation to Point-One (30-40%):

There are potentially two reasons the gift to Grandchild 6 is not an advancement. First, the advancement
doctrine is inapplicable when the decedent dies with a will. Second, under a typical advancement statute,
lifetime gifts are not an advancement absent evidence that the transferor intended them as such.

The doctrine of advancements usually applies only to intestate succession. In states that follow this rule,
the $60,000 gift to Grandchild 6 would be ignored when computing the “heirs” share of Dorothy’s estate.
However, there is some authority for the proposition that the doctrine of advancements would apply if, as
here, a will leaves property to the testator’s heirs. Furthermore, here Dorothy has expressly directed an
intestate-style distribution. Whether the $60,000 would be taken into account in determining Grandchild
6’s share of Dorothy’s estate would depend on the applicable advancement statute.

Under the common law, a lifetime transfer to an heir was presumptively treated as a down payment on the
heir’s intestate share. In most, if not all, states today, a lifetime transfer is presumed to be a gift and is
ignored in computing the heir’s intestate share unless there is evidence to show a contrary intent. Here,
there is no evidence that Dorothy intended the $60,000 to be an advancement.

Explanation to Point-Two (15-25%):

Seperac-J19 Exam-Released MEE Essay Compilation © 2016-2020 1325


Grandchild 1 should not be barred from sharing in Dorothy’s estate under the typical slayer statute
because Grandchild 1 did not act with felonious intent.

Under the typical slayer statute, an heir or beneficiary is barred from taking from the decedent if the heir
or beneficiary feloniously and intentionally killed the decedent. The typical slayer statute excludes
accidental killings even if they amount to manslaughter. Here, Grandchild 1 acted both inadvertently and
negligently and thus is not barred by the slayer statute.

[NOTE: If an applicant concluded that Grandchild 1 would be barred by the slayer statute, Grandchild
1’s forfeited share of Dorothy’s estate would be disposed of as if Grandchild 1 predeceased Dorothy.
Here, that would mean that Grandchild 1’s share would pass to Great-Grandchild, Grandchild 1’s child.]

Explanation to Point-Three (40-50%):

Assuming both Grandchild 1 and Grandchild 6 share in Dorothy’s estate, all six grandchildren take equal
shares if the estate is distributed to them per capita. If the estate is distributed per stirpes, the same six
grandchildren receive the entire estate but in different amounts. Under either system, Great-Grandchild
takes nothing.

Dorothy’s will incorporates by reference the state intestacy scheme. Thus, the distribution of the estate is
determined as if Dorothy died intestate. State laws differ regarding the distribution among issue where the
decedent died intestate. Many states make the initial division of shares by computing the number of the
decedent’s children who had either survived the decedent or who left issue who survived. In such states,
the initial division would be into thirds (per stirpes system). In many other states, including Uniform
Probate Code states, the initial division is made by counting the number of heirs at the generational level
at which there is at least one living member (per capita per generation system). Here, that would be six,
since the first level at which there are surviving descendants of Dorothy is the grandchild level.

Taking the latter (Uniform Probate Code approach) first, since no advancement was made to Grandchild 6
that grandchild shares in the estate. In addition, Grandchild 1 is not barred by the typical “slayer statute”
because he did not intentionally take Dorothy’s life. Thus, under the latter approach, all six grandchildren
take equal $20,000 shares of Dorothy’s estate.

In those states that initially divide the estate into thirds, Dorothy’s estate would be divided into three
$40,000 shares. Abel’s two children would be entitled to split 1/3 of the estate, taking $20,000 each. Each
of Brandon’s three children would be entitled to 1/3 of a $40,000 share and Carrie’s $40,000 share would
pass entirely to Grandchild 6. Great-grandchild (the child of Grandchild 1) takes nothing because
Grandchild 1 is entitled to take a share of Dorothy’s estate and in all states a remote descendant is
excluded from inheriting when that descendant’s ancestor survives.

[NOTE: The foregoing discussion assumes that no advancement was made to Grandchild 6 and that
Grandchild 1 was not barred by a slayer statute. If an applicant made other assumptions, the applicant
should be expected to have a different analysis to Point Three.]

Seperac-J19 Exam-Released MEE Essay Compilation © 2016-2020 1326


#312-JUL 2005–MEE Q01: QUESTION ONE (WILLS-ESTATES)

Ten years ago, Testator purchased an insurance policy on his life from Insurer. The policy provided that
Insurer would pay the proceeds only to the person named on a beneficiary form filed with Insurer.
Testator filed such a form with Insurer, naming his son Sam as the sole beneficiary.

A year later, Testator, concerned about his failing physical health, opened Account #1 at Bank in the
name of “Testator and Sam” as joint tenants with right of survivorship, not as tenants in common.
Testator thereafter gave Sam checks that would enable him to withdraw funds from Account #1. Testator
was the only person who deposited funds into Account #1, and he received all statements relating to it.

Five years ago, Testator duly executed a will containing the following dispositive clauses:

1. I give the proceeds of my Insurer life insurance policy to my daughter, Doris.

2. I give Account #1 at Bank to my daughter, Doris.

3. I give the balance of my estate to the children of my son Sam, to be divided equally among them.

Three months ago, Testator died and his will was duly probated. Testator was survived by Sam, Doris,
and one of Sam’s three children. Two of Sam’s children predeceased Testator. One of the predeceased
children, Ann, died seven years ago, and the other, Bill, died two years ago. Ann had a child who survived
Testator, and Bill had a child who survived Testator.

Testator was a domiciliary of State A. State A law provides that, “if a beneficiary who is a descendant of
the testator predeceases the testator, the beneficiary’s surviving issue take the share the deceased
beneficiary would have taken had the beneficiary survived.”

To whom should the life insurance proceeds, Account #1, and the balance of Testator’s estate be
distributed? Explain.

Seperac-J19 Exam-Released MEE Essay Compilation © 2016-2020 1327


#312: J05-1 MEE: ANSWER: NCBE (WILLS-ESTATES)

POINT (1) [18%] ISSUE: Can an insured change a life insurance beneficiary designation by will?
ANSWER: No. In most states, a life insurance beneficiary designation cannot be changed by will.
Thus the insurance proceeds are probably payable to Sam.

POINT (2) [41%] ISSUE: Does a joint tenant who deposited all funds into a joint tenancy bank
account retain the right to bequeath this asset to an individual who is not the surviving joint tenant?
ANSWER: The surviving tenant of a joint bank account is ordinarily entitled to the account
balance on the death of the other tenant. Some states would distribute the account balance as part
of the deceased tenant’s probate estate if the evidence shows that the deceased tenant created the
joint tenancy for his own convenience and had no intent to give the surviving tenant an ownership
interest.

POINT (3) [41%] ISSUE: With respect to the state lapse statute: (a) does it apply to class gifts? If
yes, (b) does it apply to class members who died before the testator executed his will; and (c) does it
apply to class members who predeceased the testator but were alive when his will was executed?
ANSWER: The children of Ann and Bill can share in the residuary estate only if the shares they
would have taken had they survived Testator pass to their issue under the governing lapse statute.
This determination will depend on statutory interpretation.

ANSWER DISCUSSION:

The life insurance proceeds should probably be distributed to Sam because a beneficiary designation
cannot, in most states, be changed by will. The joint tenancy bank account should be distributed to Sam
unless it is established that, in creating the joint tenancy, Testator intended to give Sam check-writing
privileges instead of an ownership interest. In some states, evidence of such intention would be sufficient
to void the joint tenancy. Sam’s surviving child is clearly entitled to share in Testator’s estate; whether
Sam’s deceased children also share depends on whether the state lapse statute applies to class gifts and, if
so, whether it extends to class members who died before Testator’s will was executed.

ANSWER EXPLANATION:

Explanation to Point-One (10-20%):

In most states, a life insurance beneficiary designation cannot be changed by will. Thus the insurance
proceeds are probably payable to Sam.

In most jurisdictions, life insurance proceeds are payable to the beneficiary named in the beneficiary-
designation form filed with the insurance company even if the insured names a different beneficiary in a
later-executed will. This rule is typically justified as a matter of contract: as in this case, life insurance
policies generally provide that policy proceeds will only be paid to a beneficiary named on an appropriate
form filed with the insurance company; other possible methods of changing a beneficiary are thus viewed
as being excluded by the insurance contract.

However, some courts have rejected the majority rule on the grounds that the requirement that a
beneficiary change be evidenced by a form filed with the insurance company is for the exclusive benefit
of the company. These courts permit an insured to change a beneficiary designation by will if his

Seperac-J19 Exam-Released MEE Essay Compilation © 2016-2020 1328


insurance company does not object. If State A has adopted this minority approach, the insurance proceeds
would pass to Doris under the terms of Testator’s will.

[NOTE: An applicant need not describe both the majority and minority approaches to receive credit. The
revocability of a life insurance beneficiary designation by will does not appear to be addressed by any
state statute.]

Explanation to Point-Two (30-40%):

The surviving tenant of a joint bank account is ordinarily entitled to the account balance on the death of
the other tenant. Some states would distribute the account balance as part of the deceased tenant’s probate
estate if the evidence shows that the deceased tenant created the joint tenancy for his own convenience
and had no intent to give the surviving tenant an ownership interest.

Ordinarily, a bank account in the name of “A and B” as joint tenants creates a right of survivorship. Upon
the death of one tenant, the survivor takes the entire account balance.

Where a joint tenancy was created merely for a depositor’s convenience, for example, to give the other
co-tenant check-writing privileges, some courts will set aside the joint tenancy if the evidence shows that
the depositor intended to convey only a power of attorney to write checks. The UPC has adopted this
approach. Other courts invariably affirm a joint tenancy, relying on the parol evidence rule to exclude
evidence of the depositor tenant’s intentions. And even among courts that permit a joint tenancy to be set
aside, some require clear and convincing evidence of the depositor tenant’s intentions.

In this case, there is no direct evidence that Testator intended to convey only check-writing privileges in
Account #1. That Testator made all deposits and received all statements demonstrates nothing about his
intentions, nor does the fact that Testator was concerned about his physical health. However, one might
infer an intention to create a convenience account from Testator’s belief that he could bequeath this asset
by will. Thus, Sam would prevail in a state that disallows evidence of depositor intentions and in a state
that requires clear and convincing evidence to overturn a joint tenancy designation; it is possible that
Doris would prevail in a state that permits a joint tenancy designation to be overturned by a
preponderance of the evidence.

A constructive trust theory has sometimes been utilized to award a joint tenancy bank account to the
beneficiary named in the depositor tenant’s will. Under a constructive trust approach, Sam would take the
account but would hold the balance in the account in trust for Doris. However, the imposition of a
constructive trust ordinarily requires evidence of misconduct, and there is no evidence of wrongdoing by
Sam.

Explanation to Point-Three (30-40%):

The children of Ann and Bill can share in the residuary estate only if the shares they would have taken
had they survived Testator pass to their issue under the governing lapse statute. This determination will
depend on statutory interpretation.

The State A lapse statute provides that, “if a beneficiary who is a descendant of the testator predeceases
the testator, the beneficiary’s surviving issue take the share the deceased beneficiary would have taken
had the beneficiary survived.” The statute is silent regarding its application to class gifts and, if it applies
to class gifts, whether it also applies to persons who died before the execution of the will creating a class
gift.

Seperac-J19 Exam-Released MEE Essay Compilation © 2016-2020 1329


At common law, a gift to a class (such as a group of persons related to each other through a common
ancestor) implied a survivorship condition, with the result that only those class members who survived the
testator shared in the gift. Under the common law approach, only Sam’s surviving child would take the
balance of Testator’s estate. Some states with lapse statutes that do not specify whether they apply to class
gifts utilize the common law approach.

Most state lapse statutes, either expressly or by judicial construction, do apply to class gifts. In these
states, Bill’s child would take the share that Bill would have taken had Bill survived Testator because Bill
was alive when Testator’s will was executed. However, even when a lapse statute applies to class gifts,
some states, by statute or case law, refuse to extend its reach to persons, like Ann, who predeceased both
the testator and the execution of the testator’s will. The theory behind such a limitation is that, because the
testator would have known that such person was dead when the testator executed his or her will, the
testator would not have intended to include either the deceased person or that person’s issue in the class
gift, unless the testator specifically made such a bequest in his will.

The 1990 UPC takes a different approach; anti-lapse protection is extended even to potential class
members who died before the class gift was created. The UPC approach is based on the view that testators
would not typically want to exclude the family line of a deceased class member. Under this approach, the
children of both Ann and Bill would share in Testator’s residuary estate.

Seperac-J19 Exam-Released MEE Essay Compilation © 2016-2020 1330


#313-FEB 2005–MEE Q05: QUESTION FIVE (WILLS-ESTATES)

In 1991, Testator validly executed a typewritten will. Its dispositive provision provided that:

1. I give $10,000 to Cousin.

2. I give Blackacre, my family home, to Sister.

3. I give the residue of my estate to University, my alma mater.

Three months after executing this will, Testator, desiring to increase the bequest to Cousin, scratched out
Item 1 in its entirety and immediately above it wrote in by hand: “I give $100,000 to Cousin.” This
handwritten $100,000 bequest was not witnessed.

In 1994, Testator sold Blackacre, the family home, and reinvested the entire sales proceeds in Whiteacre,
which became Testator’s new family home.

In 1994, one month after buying Whiteacre and following a heated argument with Cousin, Testator
validly executed two copies of a new typewritten will that left his entire estate to University. Testator then
put both executed copies of the 1994 will in his safe deposit box, where the 1991 will was also located.

In 1999, Testator and Cousin reconciled. Immediately thereafter, Testator went to the safe deposit box and
removed one of the executed copies of the 1994 will. In the course of reviewing it, Testator had second
thoughts about leaving nothing to Cousin. However, rather than executing a new will, he tore up that copy
of the 1994 will in the presence of his neighbor and stated: “I feel better now. Cousin is taken care of.”

Last year, Testator, a domiciliary of State A, died leaving a substantial estate, including Whiteacre. Both
the 1991 will with the handwritten changes and the remaining executed copy of the 1994 will were found
in Testator’s safe deposit box.

Both Cousin and Sister survived Testator. Under State A intestacy law, Sister would be Testator’s only
heir. State A also has a statute providing: “The revocation of a will that revoked an earlier will revives the
earlier will in the absence of a contrary intention.” State A does not permit holographic wills.

What, if anything, are Cousin and Sister entitled to receive from Testator’s estate? Explain.

Seperac-J19 Exam-Released MEE Essay Compilation © 2016-2020 1331


#313: F05-5 MEE: ANSWER: NCBE (WILLS-ESTATES)

POINT (1) [30%] ISSUE: Does the physical destruction of one executed copy of the 1994 will, done
with the intent to revoke, effectively revoke the 1994 will if the other executed copy has not been
physically destroyed? ANSWER: Yes. Testator’s 1994 will was revoked by the physical destruction
of one of two executed copies of that will.

POINT (2) [10%] ISSUE: Is the 1991 will revived by the revocation of the 1994 will? ANSWER:
Yes. Under the State A statute, the 1991 will was revived by the revocation of the 1994 ill because
there is no evidence that Testator did not intend a revival of the earlier will.

POINT (3) [40%] ISSUE: Assuming the 1991 will is revived, is Cousin entitled to $100,000, $10,000,
or nothing? ANSWER: Under the doctrine of dependent relative revocation, Cousin is entitled to
$10,000. Cousin is not entitled to $100,000 because that intended bequest was made by an
unattested act.

POINT (4) [20%] ISSUE: Assuming the 1991 will is revived, did the bequest to Sister adeem?
ANSWER: Under the common law, the bequest to Sister adeemed, although some states have
statutes substituting the new home for the home that was sold.

ANSWER DISCUSSION:

Testator’s 1994 will was revoked by physical destruction even though only one of the two executed
copies was physically destroyed because in doing so Testator intended to revoke the 1994 will. Under the
State A statute, revocation of the 1994 will revived the 1991 will. Under the 1991 will, $10,000 passed to
Cousin because the handwritten cancellation of that bequest was conditioned on the effectiveness of the
$100,000 bequest. However, the $100,000 bequest cannot be given effect because it was not witnessed.
Furthermore, while under the common law the bequest to Sister adeems, in some states Sister is entitled to
Whiteacre, the replacement property for Blackacre. If an applicant concludes that the 1994 will was not
revoked because there was one remaining executed copy, Cousin and Sister take nothing.

ANSWER EXPLANATION:

Explanation to Point-One (25-35%):

Testator’s 1994 will was revoked by the physical destruction of one of two executed copies of that will.

All states permit the revocation of a will by physical destruction if that act is accompanied by an intent to
revoke. If Testator had not executed the 1994 will in duplicate, there would be no question that it was
revoked because the facts evidence both a physical destruction and an intent to revoke.

However, the 1994 will was executed in duplicate, and the physical destruction of one copy may have
been motivated either by a desire either to revoke the will or to revoke only the copy. While the matter
has not been considered in all states, the prevailing view is that the revocation of one copy presumptively
revokes the will. The proponent of the copy of the will that was not destroyed must then prove that
Testator did not intend to revoke the will. Here, Testator’s statement accompanying the destruction of one
copy as well as the other surrounding facts suggest that Testator intended to revoke the 1994 will for the

Seperac-J19 Exam-Released MEE Essay Compilation © 2016-2020 1332


purpose of reviving the 1991 will. Thus, Testator would not have intended the other copy of the 1994 will
to remain in effect.

Explanation to Point-Two: (05-15%):

Under the State A statute, the 1991 will was revived by the revocation of the 1994 ill because there is no
evidence that Testator did not intend a revival of the earlier will.

The State A statute provides that “the revocation of a will that revoked an earlier will revives the earlier
will in the absence of a contrary intention.” Here, the facts support the conclusion that Testator, by
revoking the 1994 will, intended to revive the 1991 will. In particular, when revoking the 1994 will,
Testator made reference to the fact that by revoking the 1994 will Cousin would be taken care of. That
result would only be true if the 1991 will were revived. Given that intent and the statutory directive, the
1991 will is revived.

Explanation to Point-Three (35-45%):

Under the doctrine of dependent relative revocation, Cousin is entitled to $10,000. Cousin is not entitled
to $100,000 because that intended bequest was made by an unattested act.

On the assumption that the 1991 will was revived, the question arises whether Cousin takes $100,000 as
Testator likely intended, $10,000 under the doctrine of dependent relative revocation, or nothing because
the typewritten bequest to Cousin was revoked by cancellation.

While Testator may well have intended Cousin to take $100,000 as evidenced by the handwriting on the
will and the statement accompanying the revocation of the 1994 will, since State A does not permit
holographic wills, that writing cannot be given effect as a valid will or codicil because it was not
witnessed. While the facts do not state what the State A statute requires for execution formalities, all
states minimally require two witnesses to create a valid will unless they permit holographic wills.

Given that Cousin cannot take the $100,000, the issue arises whether Testator’s scratching out of the
$10,000 bequest to Cousin results in the revocation of that bequest under the typical revocation statute.

Under the doctrine of dependent relative revocation, the physical revocation of a bequest can be ignored if
evidence exists to suggest that the testator revoked that bequest on the mistaken assumption that some
other bequest would be effective. Such evidence appears here not only from the fact that immediately
above the scratching out of the bequest appeared the words of the greater, albeit ineffective, bequest, but
also from the statements Testator made to his neighbor regarding Cousin when the 1994 will was revoked.
If the doctrine does not apply, however, then Cousin takes nothing because the $100,000 is ineffective as
a bequest and the $10,000 bequest was revoked by cancellation.

Explanation to Point-Four (15-25%):

Under the common law, the bequest to Sister adeemed, although some states have statutes substituting the
new home for the home that was sold.

An ademption of a bequest occurs when the subject of a specific devise has been sold by the testator
between the time the will was executed and the time the testator dies. The theory behind ademption is that
a testator, knowing of the disposition of the specifically devised property, could execute a new will to take
account of that disposition if the testator wanted some substitute bequest to pass to the beneficiary. Absent

Seperac-J19 Exam-Released MEE Essay Compilation © 2016-2020 1333


the execution of a new will, Testator presumably does not intend the beneficiary to take any substituted
property.

Some states, however, have adopted statutes that substitute so-called replacement property for the specific
property disposed of by the testator. For example, § 2-606 of the Uniform Probate Code (UPC) provides
that a specific devisee (here, Sister) is entitled to any property acquired by a testator as “replacement
property” for the property that the testator sold. The facts support the conclusion that Whiteacre was
acquired by Testator as a substitute for Blackacre. Thus, under the UPC, or a similar statute, Sister would
be entitled to Whiteacre. Absent such a statute, Sister would be entitled to nothing under the common law.

In the absence of a statute, a court might also reach the same result by noting that the will reflects the
intent to devise Sister the family home, arguably without regard to whether the family home was
Blackacre or Whiteacre.

Seperac-J19 Exam-Released MEE Essay Compilation © 2016-2020 1334


#314-JUL 2004–MEE Q04: QUESTION FOUR (WILLS-ESTATES)

Decedent and his only child, Clara, died as the result of an accident when Clara’s car was struck from the
rear by a truck. Clara was driving and Decedent was riding in the back seat directly behind her. The
emergency medical team that arrived at the accident scene found no evidence that either of them was
alive. The emergency room physician examined their bodies as they were being removed from the
ambulance. She first pronounced Decedent dead and then pronounced Clara dead.

Clara was survived by her spouse, Son-in-Law, who was named as the sole beneficiary of her estate under
her duly probated will. Clara had no descendants.

Decedent died intestate leaving an estate of approximately $300,000. Decedent left no surviving spouse.
Decedent’s parents had predeceased him by many years. Decedent’s closest surviving relatives are:

1. a brother, Brother;

2. a half-sister, Half-Sister, who is related to Decedent through a common mother;

3. an adopted sister, Adopted-Sister, who was adopted by Decedent’s parents; and

4. his paternal grandfather, Gramps.

Three years before Decedent died, he gave Brother a check for $90,000 to enable Brother to buy a new
home.

Among Son-in-Law, Brother, Half-Sister, Adopted-Sister, and Gramps, who will share in Decedent’s
estate, and what is the value of the share each will receive? Explain.

Seperac-J19 Exam-Released MEE Essay Compilation © 2016-2020 1335


#314: J04-4 MEE: ANSWER: NCBE (WILLS-ESTATES)

POINT (1) [32%] ISSUE: Did Clara survive Decedent such that Decedent’s estate passes under
Clara’s will to Son-in-Law? ANSWER: No. Under the Uniform Simultaneous Death Act, there is a
presumption that Clara predeceased Decedent and there is no evidence here to rebut that
presumption. Likewise, under the UPC’s 120-hour survivorship rule, Clara predeceased Decedent
because she did not survive him by 120 hours. Thus, Son-in-Law is not entitled to any share of
Decedent’s estate.

POINT (2) [14%] ISSUE: Is Half-Sister an heir of Decedent? ANSWER: Yes. Half-Sister is an heir
of Decedent. Depending on state law, she is entitled either to the same share as a whole sibling or a
lesser share.

POINT (3) [23%] ISSUE: Is Adopted-Sister an heir of Decedent? ANSWER: Yes. Adopted-Sister
is an heir of Decedent.

POINT (4) [23%] ISSUE: Does the $90,000 gift to Brother reduce Brother’s share of Decedent’s
estate? ANSWER: No. Decedent’s lifetime gift to Brother is not an advancement and thus does not
reduce Brother’s share of Decedent’s estate.

POINT (5) [9%] ISSUE: Is Gramps entitled to any share of Decedent’s estate? ANSWER: Gramps
is not entitled to receive any share of Decedent’s estate unless heirship in the state was determined
under the civil law method of consanguinity.

ANSWER DISCUSSION:

Under the Uniform Simultaneous Death Act, Clara is deemed to have predeceased Decedent. Likewise,
Clara predeceased Decedent under the 120-hour survivorship rule of the Uniform Probate Code (UPC).
Therefore, Son-in-Law cannot claim any share of Decedent’s estate. Decedent’s heirs are his siblings:
Brother, Half-Sister, and Adopted-Sister. However, their respective shares of Decedent’s estate may differ
depending upon underlying state law. The gift to Brother is not an advancement and thus does not reduce
Brother’s share of Decedent’s estate. If Half-Sister takes an equal share, then Brother, Half-Sister, and
Adopted-Sister are each entitled to $100,000. Gramps would take only in a state determining heirship
under the civil law method of consanguinity, which very few states apply.

ANSWER EXPLANATION:

Explanation to Point-One (30-40%):

Under the Uniform Simultaneous Death Act, there is a presumption that Clara predeceased Decedent and
there is no evidence here to rebut that presumption. Likewise, under the UPC’s 120-hour survivorship
rule, Clara predeceased Decedent because she did not survive him by 120 hours. Thus, Son-in-Law is not
entitled to any share of Decedent’s estate.

Son-in-Law, as the sole beneficiary under Clara’s will, would be entitled to Decedent’s estate only if
Clara had been entitled to inherit Decedent’s estate. Clara would be an heir of Decedent only if she
survived Decedent, because an heir must be alive at the time of the intestate’s death. However, in a
situation such as this one, where the two decedents were killed in the same traffic accident, Clara is

Seperac-J19 Exam-Released MEE Essay Compilation © 2016-2020 1336


deemed to have predeceased Decedent if there is insufficient evidence that she and Decedent died other
than simultaneously. Since the evidence here is insufficient to establish that they did not die
simultaneously, under the Uniform Act, Clara is deemed to have predeceased Decedent.

While it might be argued that Clara survived because the truck hit her car from the rear and presumably
hit Decedent first, it is also possible that the mere impact at the rear of Clara’s car was sufficient to cause
both their deaths. Furthermore, the only direct evidence of their deaths was that neither appeared to be
alive at the scene of the accident. Thus, there is no direct evidence to establish that Clara survived
Decedent. The fact that Decedent was declared dead first should not be controlling because the
declarations of death were made simply in the order in which they were removed from the ambulance, and
the factual evidence supports the conclusion that neither was alive when they were placed into the
ambulance.

Under § 2-104 of the UPC, Clara is also not an heir of Decedent because she failed to survive him by 120
hours.

Explanation to Point-Two (10-20%):

Half-Sister is an heir of Decedent. Depending on state law, she is entitled either to the same share as a
whole sibling or a lesser share.

Decedent’s heirs will be the descendants of his deceased parents and perhaps Gramps. The issue therefore
arises whether siblings of the half-blood and adopted siblings inherit along with siblings of the whole
blood.

Half-Sister and Decedent are half-bloods because they share only one common parent. Under § 2-107 of
the UPC, relatives of the half-blood inherit the same share as relatives of the whole blood. Thus, Half-
Sister would take $100,000.

However, in some jurisdictions they inherit a smaller share.

Explanation to Point-Three (20-30%):

Adopted-Sister is an heir of Decedent.

At common law, an adopted child could not take from or through an adopting parent. Thus, at common
law, Adopted-Sister would not be Decedent’s heir. By the end of the twentieth century, motivated by a
policy to treat the adopted child as part of the adopting parents’ nuclear family, most state laws were
changed to provide that an adopted child is treated as a biological child for purposes of inheritance. Thus,
under the UPC, Decedent’s estate is distributed equally to the descendants of his deceased parents. The
descendants of Decedent’s parents include both their biological and adopted children. Thus, Adopted-
Sister would take AP139$100,000. AP173

Explanation to Point-Four (20-30%):

Decedent’s lifetime gift to Brother is not an advancement and thus does not reduce Brother’s share of
Decedent’s estate.

Seperac-J19 Exam-Released MEE Essay Compilation © 2016-2020 1337


At common law, a lifetime transfer to a child who would be the transferor’s heir was treated as a down
payment on the child’s intestate share. Under the common-law rule, the gift to Brother would not be an
advancement as Brother is not Decedent’s child.

Many states have broadened the common-law rule to include transfers to any person who is an heir.
However, most states also provide that a lifetime transfer to a person who would be the transferor’s heir is
presumptively a gift and not an advancement.

States differ in what it takes to rebut the gift presumption. Some states permit any competent evidence to
be used to rebut the presumption. Others have a heightened evidentiary requirement. For example, under
the UPC, the presumption that a lifetime transfer was a gift can be rebutted by a contemporaneous writing
of either the decedent or the donee stating either that the gift was an advancement or that it was to be
taken into account in computing the distribution of the decedent’s estate. Here, however, there is neither a
writing nor any other competent evidence that Decedent intended an advancement when he made the
transfer to Brother. Consequently, the transfer to Brother would be treated as a gift, not an advancement,
and Brother would take $100,000 of Decedent’s estate.

Lastly, in some states an advancement only occurs when the donee would have been an heir of the
decedent at the time the transfer was made. If that rule applies here, the transfer could not be an
advancement because at the time Decedent transferred the money to Brother, Clara was living and thus
would have been Decedent’s only heir.

Explanation to Point-Five (05-15%):

Gramps is not entitled to receive any share of Decedent’s estate unless heirship in the state was
determined under the civil law method of consanguinity.

Under the civil law method of consanguinity, Gramps as well as Decedent’s siblings would be related to
Decedent within the second degree of consanguinity and Gramps would share equally with the siblings in
the estate. Most states have rejected this method of determining heirship. In those states, including all
those that have adopted the UPC and those that follow the parentelic method of descent, Gramps would
not be entitled to share in Decedent’s estate.

Seperac-J19 Exam-Released MEE Essay Compilation © 2016-2020 1338


#315-FEB 2004–MEE Q01: QUESTION ONE (WILLS-ESTATES)

In 1995, Testator, age 85, executed a will in the presence of two witnesses. Immediately before signing
the document, Testator’s attorney asked Testator if she declared the instrument to be her will. Testator
responded: “You bet it is. I want Charity to have everything. My family has enough.” Then the attorney
had Testator sign the document on the line provided for her signature. The two witnesses signed
immediately below Testator’s signature without any further direction or comment from Testator.

When Testator executed this will, she was suffering from cancer and her medications made it very
difficult for her to remember facts. For example, when she executed her will she knew, correctly, that her
estate was worth $500,000 and that she had previously made large gifts to her child and some of her
grandchildren. However, she could neither remember the name of her stockbroker nor recount the names
of her stocks under her stockbroker’s management. Also, she had no difficulty correctly naming her child
and all of her grandchildren, but she could not recall that she had a great-grandchild. She also knew she
owned both a home and a condominium but could not recall the precise street address for either residence.

Testator died in 2002 survived by her only child, Mary, and by three grandchildren and one great-
grandchild, all of whom are descendants of Mary. Testator’s will, which devised her entire estate to
Charity, was timely offered for probate by Bank, the executor named in the will. Mary and one of her
children, Grandchild, have initiated a timely contest of the will.

Governing state law provides that a will is properly executed if the testator signs the will in the presence
of two witnesses after having (a) declared the instrument to be her will, and (b) requested the witnesses to
act in such capacity.

1. Do Mary and Grandchild each have standing to contest Testator’s will? Explain.

2. On what theory or theories, other than undue influence, might a person with standing contest
Testator’s will, what defenses might Bank, as executor, assert, and what is the likely outcome?
Explain.

Seperac-J19 Exam-Released MEE Essay Compilation © 2016-2020 1339


#315: F04-1 MEE: ANSWER: NCBE (WILLS-ESTATES)

POINT (1) [26%] ISSUE: Do Mary and Grandchild each have standing to contest Testator’s will?
ANSWER: Mary has standing to contest the will. However, Grandchild does not have standing to
contest the will.

POINT (2)(a) [37%] ISSUE: Can Testator’s will be successfully contested on the grounds of lack of
mental capacity? ANSWER: No. Testator probably had the mental capacity to execute a will at the
time she executed it.

POINT (2)(b) [37%] ISSUE: Can Testator’s will be successfully contested on the grounds of lack of
due execution? ANSWER: Testator’s will was not duly executed under the strict compliance
doctrine but could be upheld as valid if the court either adopts the substantial compliance doctrine
or infers that Testator requested the witnesses to sign.

ANSWER DISCUSSION:

Mary has standing to contest the will because, if the will were denied probate, she would be Testator’s
sole heir. On the other hand, Grandchild lacks standing to contest the will because if the will were denied
probate, Grandchild would not be entitled to any share of Testator’s estate. Mary, who is entitled to
contest the will, is not likely to be successful on the ground of lack of mental capacity but might prevail
on the ground of lack of due execution in states applying the strict compliance doctrine.

ANSWER EXPLANATION:

Explanation to Point-One (20-30%):

Mary has standing to contest the will. However, Grandchild does not have standing to contest the will.

Wills can be contested only by persons who would be better off financially if the will were denied probate
than they would be if the will were admitted to probate. If the rule were otherwise, wills would be subject
to suits by persons seeking to extort money from legitimate beneficiaries who want to avoid litigation.

Under the laws of all states, an intestate’s children take to the exclusion of their own descendants. Thus, if
Testator’s will were denied probate, Testator’s entire estate would pass to Mary; nothing would pass to
the grandchildren or great-grandchild. Mary, therefore, would have standing to contest the will because as
Testator’s only child she would be her only heir if the will were denied probate. On the other hand,
Grandchild would not be financially better off if the will were denied probate and, thus, would lack
standing to contest the will.

Explanation to Point-Two(a) (30-40%):

Testator probably had the mental capacity to execute a will at the time she executed it.

In order to execute a will, a testator must have mental capacity. A testator meets this requirement if the
testator knows (1) the nature and extent of her property, (2) the persons who are the natural objects of the
testator’s bounty and have the highest moral claims to the testator’s property, (3) the disposition the

Seperac-J19 Exam-Released MEE Essay Compilation © 2016-2020 1340


testator is attempting to make, and (4) the interrelationship of these items in connection with the
testamentary plan formulated in the will.

All persons are afforded the presumption that they have mental capacity. As a result, the burden of
proving that the testator lacks mental capacity rests on the contestant to the will. Here, there are two
arguments to support a will contest on mental incapacity grounds, although Bank has good responses to
both arguments.

First, Testator arguably did not know the nature and extent of her property. Testator could not identify the
stocks that she owned or name the stockbroker who managed them. Also, she could not recall the
addresses of her two residences. On the other hand, she did know what she was worth and she did know
that she owned two residences. Thus, whether she lacked mental capacity on this ground may ultimately
depend on how precise a testator must be regarding the nature and extent of her property. Courts often are
very lenient and uphold the wills of elderly testators who at least grasp the big picture about their financial
affairs. Furthermore, because of the burden of proof, in a very close case the contestant will lose.

Second, Testator could not identify one of the persons with a high moral claim to her property, namely her
great-grandchild. On the other hand, Testator could identify her more closely related relatives, and in
particular Mary, whom she was intending to disinherit. Her failure to recall whether she had a great-
grandchild probably would not be dispositive of her mental capacity. Courts have not required elderly
testators to know the number of their remote descendants, particularly in a mobile society where people
may have infrequent contacts with grandchildren and great-grandchildren.

Explanation to Point-Two(b) (30-40%):

Testator’s will was not duly executed under the strict compliance doctrine but could be upheld as valid if
the court either adopts the substantial compliance doctrine or infers that Testator requested the witnesses
to sign.

At common law, if the execution of a will did not strictly adhere to the required formalities, the will
would be invalid and the testator would have died intestate. The facts state that under the governing law, a
properly executed will must be signed by the testator in the presence of two witnesses after the testator
has declared the instrument to be her will and has requested that the witnesses act in such capacity. Here,
the signing, witnessing, and declaration requirements have been satisfied, but the facts state that Testator
did not specifically request the witnesses to act as such. Thus, courts adhering to the strict common-law
approach (known as the strict compliance doctrine) would invalidate the will.

On the other hand, statutes in some states and a judicial decision in at least one state have tempered the
harsh consequences of the common-law rule by adopting a “substantial compliance” approach to
determining whether a will has been validly executed. Under this approach, if the execution of a will
substantially complies with most of the formalities, or at least the most important of them, the will is
valid. In such cases, a will is valid even if the witnesses only witnessed the self-proving will affidavit and
did not witness the actual will.

Similarly, § 2-503 of the Uniform Probate Code (UPC) grants a court a so-called “dispensing power”
under which a court can probate a will when its execution failed to comply with all of the execution
formalities so long as the evidence is clear and convincing that the decedent intended the document to be
her will. In this case, the failure to request that the witnesses act as such when they signed in the physical
presence of Testator can easily be ignored under this statute if, as the facts suggest, Testator clearly

Seperac-J19 Exam-Released MEE Essay Compilation © 2016-2020 1341


intended Charity to take under her will. The statement, “My family has enough” is evidence of Testator’s
intent.

It might also be argued from the context and surrounding circumstances that Testator implicitly asked the
witnesses to sign even if she did not do so expressly. She saw them sign, and she declared the instrument
to be her will. At least one court has adopted that approach and thus avoided an all-out adoption of the
substantial compliance theory.

Seperac-J19 Exam-Released MEE Essay Compilation © 2016-2020 1342


#316-JUL 2003–MEE Q02: QUESTION TWO (WILLS-ESTATES/TRUSTS)

In 1988, Testator duly executed a will devising Blackacre to Adam, $100,000 to Carrie, and the residue of
her estate to Doris. However, in 1992, Testator telephoned her lawyer, Lawyer, who had possession of the
1988 will, and asked her to destroy it because Testator had changed her mind. Lawyer agreed.
Immediately after hanging up the phone, Lawyer found the will, shredded it, and threw it away.

In 1996, Testator signed and dated a wholly handwritten document that stated: “I devise Blackacre to Earl
and $2,500,000 to my good friend, Fred.”

Testator died in 2002, a domiciliary of State A. She was survived by Greg, age 30, who was her child and
only heir. Adam, Carrie, Doris, Earl, and Fred also survived Testator. There was no surviving spouse.
Testator’s net probate estate (after taxes, debts, and expenses) consisted of $5,000,000 plus Blackacre.

The 1996 document, together with an unexecuted copy of the 1988 will, the original of which Lawyer had
shredded, were found among Testator's valuable papers. Both documents were offered for probate.

Under State A law, holographic wills are valid.

1. Which documents, if any, govern the distribution of Testator’s estate? Explain.

2. What are the respective shares, if any, in Testator’s estate of each of the following: Adam,
Carrie, Doris, Earl, Fred, and Greg? Explain.

Seperac-J19 Exam-Released MEE Essay Compilation © 2016-2020 1343


#316: J03-2 MEE: ANSWER: NCBE (WILLS-ESTATES/TRUSTS)

POINT (1) [31%] ISSUE: Was the 1988 will that was destroyed by Testator’s lawyer outside of
Testator’s presence legally revoked by physical destruction? ANSWER: No. The 1988 will was not
revoked because it was physically destroyed by someone other than the testator and not in the
testator’s presence.

POINT (2) [44%] ISSUE: Is Testator’s 1996 handwritten will entitled to be probated, and if so, as a
subsequent testamentary instrument, did it revoke Testator’s 1988 will? ANSWER: Because
holographic wills are valid in State A, the 1996 will can be probated. The 1988 will was revoked by
the 1996 holographic will only to the extent that it was inconsistent with the later holographic will.

POINT (3) [25%] ISSUE: Is Greg entitled to any share of the estate as a “pretermitted heir”?
ANSWER: No. Greg, Testator’s child and only heir, is not entitled to any share of Testator’s estate
because he is not a pretermitted heir.

ANSWER DISCUSSION:

Blackacre is distributable to Earl under the terms of the valid holographic will and the balance of the
estate is distributed as follows: $100,000 to Carrie under the 1988 will, $2,500,000 to Fred under the 1996
will, and the residue to Doris under the 1988 will. Adam and Greg take nothing. This distribution results
from the fact that the 1988 will was not wholly revoked by destruction but only partially revoked by the
inconsistent provisions of the codicil. Furthermore, Greg is not entitled to a forced share because he was
alive when the wills were signed.

ANSWER EXPLANATION:

Explanation to Point-One (25-30%):

The 1988 will was not revoked because it was physically destroyed by someone other than the testator
and not in the testator’s presence.

Most states require that a revocatory act (such as physical destruction of a will) by someone other than the
testator be done in the testator’s presence, or at least in the testator’s “conscious presence,” and at the
testator’s request. Therefore, in most states, the 1988 will would not have been legally revoked. Testator
asked Lawyer to destroy the will, but the destruction did not occur in Testator’s presence.

Although the 1988 will was physically destroyed, it was not revoked and the unexecuted copy is available
to prove its content.

Explanation to Point-Two (35-45%):

Because holographic wills are valid in State A, the 1996 will can be probated. The 1988 will was revoked
by the 1996 holographic will only to the extent that it was inconsistent with the later holographic will.

The 1996 document is a holographic will because it is entirely in Testator’s handwriting; under the laws
of State A holographic wills are valid and therefore the 1996 will is entitled to be probated. The fact that
the 1996 will is entitled to be probated, however, does not preclude the possibility that the 1988 will can

Seperac-J19 Exam-Released MEE Essay Compilation © 2016-2020 1344


also be probated so long as it was not revoked. Wills may be revoked either by physical destruction,
accompanied by an intent to revoke, or by the execution of a subsequent will. Typically, if a will has been
executed and then another will is executed, the latter will revokes the former will only to the extent that
they are inconsistent, unless, of course, the latter will has an express revocation clause. Here, the 1996
will did not have an express revocation clause. Because it did not have a residuary clause (it contained
only a specific bequest and a general bequest), its terms are not wholly inconsistent with the terms of the
1988 will. In fact, the only inconsistency between them is the disposition of Blackacre. Under the 1988
will, Blackacre was bequeathed to Adam; under the 1996 will, it was bequeathed to Earl. Since the later
will trumps the earlier will, Blackacre is distributable to Earl.

Assuming that both documents can be probated for the reasons stated, the probate estate should be
distributed as follows: Blackacre to Earl, $100,000 to Carrie, and $2,500,000 to Fred; Doris, the residuary
legatee under the 1988 will, takes the remaining $2,400,000. Adam takes nothing.

Explanation to Point-Three (20-25%):

Greg, Testator’s child and only heir, is not entitled to any share of Testator’s estate because he is not a
pretermitted heir.

In most jurisdictions, pretermitted heir statutes have been enacted permitting children of a testator under
certain circumstances to claim a share of the estate even though they were omitted from the deceased
testator’s will. Typically, these statutes apply only if the child was born or adopted after the execution of
the will. Here, Greg was alive when both the 1988 will and the 1996 will were executed. Thus, Greg
would not be entitled to a “forced share” under the typical pretermitted heir statute. However, if the
statute protects children born before the will was executed who were not expressly disinherited, then Greg
would take the entire estate.

Seperac-J19 Exam-Released MEE Essay Compilation © 2016-2020 1345


#317-FEB 2003–MEE Q01: QUESTION ONE (WILLS-ESTATES)

Testator was an 80-year-old mentally alert widow. Testator retained Lawyer to prepare her will naming
Charity, a charitable organization, as the sole beneficiary of her estate. One week later, Testator received a
photocopy of a proposed will that Lawyer had prepared for her.

A few days later, on October 1, 1998, Lawyer called Testator to inquire whether the proposed will
conformed with her wishes. When Testator responded that it did, Lawyer suggested that Testator make an
appointment to come to his office so that she could execute the original, which was in his possession.
Testator responded that, because of her arthritic condition, it would not be convenient for her to do so, and
she told him, “Just go ahead and sign the will for me.” Lawyer said, “OK.”

Later that day, Lawyer inserted “October 1, 1998” as the date of execution on the original will and signed
Testator’s name on the will in front of three secretaries who acted as witnesses. The secretaries then
signed their names in the spaces provided on the will. All of them saw Lawyer sign Testator’s name, and
each of them saw the others sign their own names. Lawyer then called Testator and told her the will had
been signed and witnessed. Testator replied, “Good, now it’s done. Please keep the will for me.”

A year later, Testator decided that she wanted to change the will to give her diamond ring to her niece,
Nora. Deciding to make the change herself, she asked a friend to type up a document, which was
identified as “a codicil to my existing last will.” This document was then validly executed and stated, “I
leave my diamond ring to my niece, Nora. In all other respects I hereby affirm my existing last will,
executed on October 1, 1998.”

A year later, Testator had a falling out with Nora. Remembering that she had devised her diamond ring to
Nora, she gave the ring to another niece, Betty, as a gift. Testator died a few months later. Her closest
surviving relatives were her two nieces, Nora and Betty. At the time of her death, her only asset was a
parcel of real estate known as “Blackacre.”

Who is entitled to Blackacre and to the diamond ring? Explain.

Seperac-J19 Exam-Released MEE Essay Compilation © 2016-2020 1346


#317: F03-1 MEE: ANSWER: NCBE (WILLS-ESTATES)

POINT (1) [40%] ISSUE: Was Testator’s October 1, 1998, will properly executed? ANSWER: No.
Testator’s October 1, 1998, will was not effectively executed because Lawyer, while signing it at
Testator’s direction, did not sign it in her presence.

POINT (2) [30%] ISSUE: Did Testator’s codicil effectively incorporate her earlier will by
reference? ANSWER: Yes. The validly executed codicil incorporated the October 1, 1998, will by
reference, and therefore the October 1, 1998, will can be given legal effect even though it was
defectively executed. Therefore, Charity is entitled to Blackacre.

POINT (3) [30%] ISSUE: Did Testator’s gift of the diamond ring to Betty adeem the codicil’s
bequest of the ring to Nora? ANSWER: Yes. The bequest of the diamond ring to Nora was
adeemed by Testator’s gift of the ring to Betty. Therefore, Nora is not entitled to the ring and Betty
keeps it.

ANSWER DISCUSSION:

Although Testator’s will was not validly executed because it was signed in her name but outside of her
presence, it was validly incorporated by reference into Testator’s duly executed codicil, which expressly
referred to it by date. Thus, the will, as incorporated by reference into the codicil, is valid to pass title to
Blackacre to Charity, the beneficiary under the will. Because the bequest of the ring in the codicil to Nora
adeemed by virtue of Testator’s gift of the ring to Betty, Nora is not entitled to the ring.

ANSWER EXPLANATION:

Explanation to Point-One (35-45%):

Testator’s October 1, 1998, will was not effectively executed because Lawyer, while signing it at
Testator’s direction, did not sign it in her presence.

Will execution statutes commonly provide that another person may sign a testator’s will if it is done at the
testator’s direction and in the testator’s conscious presence. A few statutes additionally require that
persons who sign for the testator must also sign their own names and sometimes also give their addresses.
Many wills statutes require one or both of the following additional requirements: (1) that the testator
declare the instrument to be his or her will to the witnesses, or (2) that the testator ask the individuals
selected as witnesses to witness his or her execution of the instrument.

In this case, Lawyer signed Testator’s name at her direction, but the question is whether he did so in her
presence. The likely answer is that he did not. Courts have used two tests to determine presence: the
majority “line-of-sight” test and the minority “conscious presence” test. Under the “conscious presence”
test, the will execution is sufficient “if it was done in the testator’s conscious presence, i.e., within the
range of the testator’s senses such as hearing.”

Lawyer clearly did not satisfy the “line-of-sight” test because Testator did not see Lawyer signing the will
on her behalf. Under the “line-of-sight” test, the testator should be capable of seeing the witnesses and the
person who is signing the will on the testator’s behalf actually sign the will. In those cases where the

Seperac-J19 Exam-Released MEE Essay Compilation © 2016-2020 1347


testator must specifically ask the witnesses to act as such, the witnesses should be capable of hearing that
request. Even under the more liberal “conscious presence” test (which the UPC expressly adopts), it is
very unlikely that the court will conclude that the presence requirement was satisfied. In the analogous
situation where a testator acknowledges her signature to witnesses over the telephone, courts have held
that this does not satisfy the “conscious presence” requirement. There is too much room for fraudulent or
mistaken substitution of one document for another under these circumstances to permit the will to be
admitted to probate.

The defect with the signature requirement is itself sufficient reason to conclude that the will was
defectively executed. Additionally, since Testator did not sign the will, acknowledge her signature,
acknowledge the will to the witnesses in their presence (under either test), or ask them to act as witnesses,
there was defective execution with respect to the witnessing requirements of the will statutes in most
states.

The Uniform Probate Code’s harmless error provision, UPC § 2-503, would not likely allow the October
1, 1998, writing to be admitted as a will. There are two defects here, signature and attestation, and the
Comment to § 2-503 provides that the greater the departure from the required formalities of execution, the
less likely that the provision will cure the defects. This is especially true with respect to defects in the
signature requirement.

Explanation to Point-Two (25-35%):

The validly executed codicil incorporated the October 1, 1998, will by reference, and therefore the
October 1, 1998, will can be given legal effect even though it was defectively executed. Therefore,
Charity is entitled to Blackacre.

The codicil is Testator’s only duly executed will. A properly executed codicil can incorporate by
reference an earlier will that was defectively executed, as here. Therefore, if the jurisdiction recognizes
the doctrine of incorporation by reference, the court should give effect to the October 1, 1998, will as an
incorporated extrinsic writing.

The doctrine of incorporation by reference is recognized in most states. Either by statute (e.g., UPC § 2-
510) or by judicial decision, several requirements must be met in order for a properly executed will to
incorporate an extrinsic writing by reference. The usual requirements are: (1) the extrinsic writing must in
fact have been in existence at the time the incorporating will was executed; (2) the incorporating will must
refer to the extrinsic writing as having been in existence at the time of execution; (3) the incorporating
will must refer to the extrinsic writing in such a way as to reasonably identify it; and (4) the incorporating
will must manifest the testator’s intent to incorporate the extrinsic writing. All four of the requirements
were met here. The codicil’s reference to the existing October 1, 1998, will is sufficiently clear, as is the
intention to incorporate the earlier writing. Therefore, Charity is entitled to Blackacre.

[NOTE: If the jurisdiction does not recognize the incorporation doctrine, the probate court should not
give any effect to the October 1, 1998, writing. Specifically, the doctrine of republication by codicil
cannot be used to republish an earlier will that was defectively executed. The republication requirement
can be used to give effect to an earlier will that was invalid for reasons other than defective execution.
The October 1, 1998, will was invalid solely because it was defectively executed, so republication is not
available here. If the will was not incorporated by reference, Blackacre would pass to Nora and Betty
equally as Testator’s heirs.]

Explanation to Point-Three (25-35%):

Seperac-J19 Exam-Released MEE Essay Compilation © 2016-2020 1348


The bequest of the diamond ring to Nora was adeemed by Testator’s gift of the ring to Betty. Therefore,
Nora is not entitled to the ring and Betty keeps it.

The doctrine of ademption by extinction provides that if the subject matter of a specific bequest is not part
of the estate at the testator’s death, the bequest is adeemed. The doctrine applies only to specific bequests.
The bequest here was specific because the will referred to a specific asset. Under the traditional approach
to ademption – the so-called “identity theory” – the testator’s intent as to ademption is irrelevant. All that
matters is whether or not the specifically bequeathed asset is part of the estate at death. Accordingly,
Testator’s gift of the ring to Betty caused the bequest to Nora to adeem.

Even under the Uniform Probate Code’s non-ademption provision, UPC § 2-606, which adopts the so-
called “intent theory” of ademption, the result would be the same. In the Comment to § 2-606, the drafters
provide a hypothetical problem with facts similar to those of this case. The Comment states, “G’s
deliberate act of giving away the specifically devised property is a fact and circumstance indicating that
ademption of the devise was intended.” Therefore, § 2-606 would yield the same result as the traditional
ademption doctrine, particularly because the facts state that Testator gifted the ring to Betty in order to
ensure that Nora did not take under the codicil.

Seperac-J19 Exam-Released MEE Essay Compilation © 2016-2020 1349


#318-JUL 2002–MEE Q02: QUESTION TWO (WILLS-ESTATES)

On February 10, 2000, Testator signed her last will, which was witnessed by two witnesses – Testator’s
nephew, Nephew, and Testator’s next-door neighbor. Testator died on May 10, 2000, after a brief
hospitalization.

During the six months before she died, Testator experienced frequent episodes of forgetfulness. For
example, Testator often missed appointments with her physicians and her bank trust officer. Testator had
also become increasingly forgetful about matters of personal hygiene. On the other hand, throughout that
six-month period, Testator maintained all of her financial records and visited in person and by telephone
with each of her 20 living relatives, all of whom she easily recognized and identified. On April 3, she
contacted her broker to advise him to sell her shares in Able Corporation because she had lost complete
faith in the corporation’s management following the release of its poor quarterly earnings report.

Testator’s will bequeathed $100,000 to Nephew and the residue of her estate to Charity, a charitable
organization with which Testator had been associated for more than 35 years. Nephew had no knowledge
of the $100,000 bequest until after Testator died. She left no bequest to her three nieces, who are
Nephew’s three sisters. None of Testator’s other living relatives was as closely related to her as Nephew
and her three nieces.

Contemporaneous with the execution of her last will, Testator signed a durable health care power of
attorney designating Nephew as her agent to make all health care decisions for her in the event she could
no longer make them for herself. Nephew and Testator’s next-door neighbor also witnessed this
document.

One week before she died, Testator was admitted into a local hospital following a massive stroke causing
severe brain damage. The following day, she lapsed into a coma and was connected to a life-support
system. Four days later, Testator’s physician advised Nephew that there was nothing medical science
could do for Testator. After considering this advice, Nephew directed the physician to remove Testator
from all life-support systems. The following day, Testator was removed from the life-support system and
she died. She left an estate in excess of $1 million.

Testator’s three nieces argue that Testator’s durable health care power was not valid and that as a result
Nephew should be liable in wrongful death for causing Testator’s death because Nephew directed
Testator’s physician to withdraw Testator’s life-support systems. Furthermore, they claim that either
Testator’s will is invalid or that, at minimum, the bequest to Nephew should be forfeited.

1. Is Nephew liable in wrongful death for causing Testator’s death? Explain.

2. Is Testator’s will invalid because of incapacity? Explain.

3. Assuming Testator’s will is valid, is the bequest to Nephew valid? Explain

Seperac-J19 Exam-Released MEE Essay Compilation © 2016-2020 1350


#318: J02-2 MEE: ANSWER: NCBE (WILLS-ESTATES)

POINT (1) [38%] ISSUE: Is Nephew liable in wrongful death for causing Testator’s death: (a) Was
Testator’s durable health care power of attorney validly executed; and (b) was Nephew, a
beneficiary under Testator’s will and a witness to Testator’s health care power of attorney,
prohibited from acting as an agent? ANSWER: No. Under the typical durable health care power of
attorney statute, if Nephew acted under a valid durable health care power and in good faith, he
would be shielded from civil liability that might otherwise arise as a result of his directing the
withdrawal of the life-support systems.

POINT (2) [38%] ISSUE: Can the nieces successfully challenge the will because Testator lacked the
mental capacity to execute her will? ANSWER: No. Testator’s will is valid notwithstanding that
Testator was sometimes forgetful, as the facts support the conclusion that Testator had the mental
capacity to execute a will.

POINT (3) [25%] ISSUE: Does Nephew forfeit the bequest under the interested witness statute?
ANSWER: No. Nephew would not forfeit any of his bequest under the will.

ANSWER DISCUSSION:

Nephew would not be liable in wrongful death if, as it appears, he acted under a valid durable health care
power of attorney and in good faith. His decision to direct the withdrawal of life support was entirely
consistent with the doctor’s medical advice. Testator’s will appears to have been validly executed.
Arguments that Testator lacked mental capacity should fail because the facts show that she knew both the
objects of her bounty and the nature and extent of her property. Lastly, Nephew would not forfeit his
share under typical state interested witness statutes because the will bequeathed him a share less than what
would have been his intestate share. Likewise, under the UPC, he would take his share as that statute has
no interested witness forfeiture statute.

ANSWER EXPLANATION:

Explanation to Point-One (35-40%):

Under the typical durable health care power of attorney statute, if Nephew acted under a valid durable
health care power and in good faith, he would be shielded from civil liability that might otherwise arise as
a result of his directing the withdrawal of the life-support systems.

The typical durable health care power of attorney statute immunizes the agent of the principal from civil
liability for health care decisions made in good faith. Health care decisions include the decision to
withhold or withdraw life-sustaining treatment, including food and hydration.

Agents act within the scope of the statute when they act pursuant to a properly executed durable health
care power of attorney. State laws vary, however, on whether the person designated as the agent can be a
witness to the durable health care power of attorney. For example, under the Uniform Health Care
Decisions Act, the designated agent is not prohibited from being a witness. In fact, under the Uniform
Act, no witnessing of the power is required. On the other hand, in many states, the person designated as
the agent cannot be a witness to the power.

Seperac-J19 Exam-Released MEE Essay Compilation © 2016-2020 1351


Even if Nephew was not legally constituted as Testator’s agent, he may nonetheless have acted
appropriately under so-called “family consent” laws. These laws permit close family members (typically
in the order listed in the statute) to act as a surrogate decision maker for a patient where there is no
properly authorized agent acting under a durable power. For example, under § 5 of the Uniform Health
Care Decisions Act, Nephew could act as a surrogate decision maker since there are no more closely
related relatives and Nephew is an adult “who exhibited special care and concern for the patient.”

Whether Nephew was acting as a properly constituted agent or as a surrogate under the family consent
law, ultimately the issue of civil liability most likely depends on whether Nephew acted in “good faith.”
Here the facts suggest that he did. Testator had suffered a massive stroke and had lapsed into a coma, and
Nephew’s direction to withdraw Testator’s life support appears to be consistent with sound medical
advice to the effect that more care would be futile. While the fact that Nephew is named in Testator’s will
as a beneficiary may create the appearance of impropriety, this fact alone is not sufficient evidence of bad
faith; empirically most agents and surrogates are persons who are both close to the principal and named as
beneficiaries under the principal’s will.

Explanation to Point-Two (35-40%):

Testator’s will is valid notwithstanding that Testator was sometimes forgetful, as the facts support the
conclusion that Testator had the mental capacity to execute a will.

In order to validly execute a will, Testator must have “mental capacity.” A testator has mental capacity if
the testator knows (1) the nature and extent of the testator’s property, (2) those persons who are the
natural objects of the testator’s bounty, (3) the nature of the instrument that the testator is signing, and (4)
the disposition that is being made in the will.

Here Testator was forgetful, which at first blush suggests the absence of mental capacity. But simply
relying on that fact is insufficient to determine whether Testator had mental capacity since the criteria that
determine mental capacity for purposes of validating a will look specifically to Testator’s understanding
of her property interests, her beneficiaries, and her disposition. On these issues, the facts suggest that
Testator did have mental capacity.

Testator appears to have been actively involved in the management and administration of her property
affairs, suggesting that she knew the nature and extent of her property. She knew those persons who might
be the objects of her bounty as evidenced by the fact that she frequently visited with her 20 relatives,
including those not mentioned in her will. Furthermore, the bulk of her estate is bequeathed to Charity, an
organization with which she had had a long association. Thus, the will does not reflect a disposition that
would likely be inconsistent with Testator’s testamentary intent. Given that the burden of proof to
establish the lack of testamentary capacity rests on the contestants, it is most unlikely that they could
sustain that burden on these facts.

Explanation to Point-Three (20-30%):

Nephew would not forfeit any of his bequest under the will.

Under the Uniform Probate Code, Nephew would not forfeit his bequest even though Nephew was a
witness to the will because the Code does not have an interested witness statute barring interested
witnesses from taking under a will.

Seperac-J19 Exam-Released MEE Essay Compilation © 2016-2020 1352


At common law, if a will was not witnessed by two disinterested witnesses the will was invalid. In the
United States, this harsh rule has been replaced by statutes in most states barring the interested witness
from taking the bequest but not invalidating the will.

The typical interested witness statute, however, contains one or two important exceptions. The first is that
if the will is witnessed by three or more persons such that the will would have been valid without the
witnessing by the interested witness, the interested witness does not forfeit the bequest. That exception
would not apply here as Testator’s will was witnessed only by two persons.

The second exception is that if the interested witness also would have been an heir of the testator, the
witness forfeits only so much of the bequest as exceeds what would have been that witness’s intestate
share. Under that exception, Nephew forfeits nothing, as the bequest to Nephew of $100,000 is less than
what his intestate share would have been. Nephew’s intestate share would have been in excess of
$250,000, as the estate is valued at more than $1 million and there are four heirs.

In light of the discussion under Point One, there is no reasonable basis on which to argue that Nephew’s
bequest is invalid under the slayer statutes.

Likewise, the facts state that Nephew was wholly unaware of the will provision in his favor. Thus, there is
no basis to invalidate the will on undue influence grounds.

[NOTE: If the will is valid, the nieces would probably have no standing to raise the interested witness
issue. If the interested witness statute applied and the bequest, or some portion of it, were forfeited, the
forfeited portion would pass to Charity as the residuary legatee under Testator’s will.]

Seperac-J19 Exam-Released MEE Essay Compilation © 2016-2020 1353


#319-FEB 2002–MEE Q04: QUESTION FOUR (WILLS-ESTATES)

Testator duly executed a will dated March 1, 1998. Among other things, the will stated:

1. I direct that all of my just debts and expenses be paid by my executor.

2. I give my family home to my daughter, Daughter.

3. I give my 24-carat gold watch to my son, Son.

4. I give the rest of my estate, including any property over which I may have a power of appointment, to
Trustee to hold in trust for the primary benefit of Son and Daughter, with the remainder to their children.

Testator was the income beneficiary of two separate testamentary trusts, one created by Testator’s mother,
Mary, the other by Testator’s father, Frank.

Testator had a special testamentary power of appointment exercisable in favor of Testator’s issue over the
testamentary trust created by Mary. Mary’s will provided that Testator could exercise the power only by a
specific reference to the power of appointment created by Mary’s will.

Testator had a general testamentary power of appointment over the testamentary trust created by Frank.

Both Mary and Frank died in 1990. They were survived by Testator, Son, and Daughter.

At Testator’s death in October 2001, Testator owned (a) the family home on which Testator was
personally liable for a $50,000 mortgage, (b) an insurance policy that specifically insured Testator’s 24-
carat gold watch, which had been stolen from Testator five days before Testator died, and (c) a portfolio
of stocks and bonds. Testator owned no other assets.

Bank was appointed executor of the estate. Trustee was named trustee of the trust created in paragraph 4
of Testator’s will. Bank collected the value of the stolen watch from the insurance company.

1. Did Testator effectively exercise the two powers of appointment he had at the time of his death?
Explain.

2. Should the $50,000 mortgage on Testator’s home be paid out of the assets of Testator’s residuary
estate? Explain

3. Should the insurance proceeds for the stolen watch be distributed to Son, as legatee of the watch,
or to Trustee? Explain.

Seperac-J19 Exam-Released MEE Essay Compilation © 2016-2020 1354


#319: F02-4 MEE: ANSWER: NCBE (WILLS-ESTATES)

POINT (1)(a) [23%] ISSUE: Did Testator’s will, which included a general residuary clause
purporting to exercise all of the donee’s powers of appointment (a so-called “blanket exercise”
clause), effectively exercise the special testamentary power of appointment that was created in
Mary’s will? ANSWER: No. The “blanket exercise” clause in Testator’s will did not effectively
exercise the power of appointment given to Testator by Mary’s will because Mary’s will required
Testator to refer specifically to the power when exercising it.

POINT (1)(b) [14%] ISSUE: Did Testator’s will, which included a blanket exercise clause,
effectively exercise the general testamentary power of appointment created in Frank’s will?
ANSWER: Yes. Testator’s general testamentary power created by Frank’s will was effectively
exercised by a blanket exercise clause.

POINT (2) [29%] ISSUE: Does Daughter, the devisee of specifically devised encumbered property,
take that property subject to the mortgage where the will contains a clause directing that the
decedent’s debts be paid by the executor? ANSWER: At common law, Daughter takes the family
home free of the mortgage. Under the law of most states today, Daughter takes the home subject to
the mortgage on which Testator was liable at the time of his death. The mortgage is not payable
from the assets of Testator’s estate.

POINT (3) [34%] ISSUE: Is Son, the specific legatee of the watch, entitled to the insurance
proceeds on the watch that were payable as a result of the theft of the watch shortly before
Testator’s death? ANSWER: Yes. The insurance proceeds payable as a result of the theft of the
watch shortly before Testator’s death should pass to Son as the specific devisee of the watch. There
is, however, contrary authority.

ANSWER EXPLANATION:

Explanation to Point-One(a) (15-25%):

The “blanket exercise” clause in Testator’s will did not effectively exercise the power of appointment
given to Testator by Mary’s will because Mary’s will required Testator to refer specifically to the power
when exercising it.

In most states, a general residuary clause in a will (e.g., “I give all of my estate.”) does not exercise
powers of appointment. The situation is different if the general residuary clause is coupled with what is
called a “blanket exercise” clause (e.g., “including all property over which I have a power of
appointment”), as is the case here. Under these circumstances, any power of appointment held by the
donee is exercised, unless the donor of the power of appointment specifically required the donee to refer
to the instrument creating the power when exercising the power.

Here, Mary, the donor of Testator’s special power, expressly required that Testator specifically refer to
her will in which the power was created if Testator, as donee of the power, sought to exercise the power.
Testator’s will contained only a blanket exercise clause, which failed to satisfy a condition on the exercise
of the power imposed by the donor. Thus, the power was not effectively exercised.

Explanation to Point-One(b) (10-15%):

Seperac-J19 Exam-Released MEE Essay Compilation © 2016-2020 1355


Testator’s general testamentary power created by Frank’s will was effectively exercised by a blanket
exercise clause.

No facts indicate that Frank imposed a specific reference requirement with respect to the general
testamentary power that Frank, as donor, granted Testator, as donee. Therefore, under the rule that a
blanket exercise clause is effective to exercise powers absent a specific reference requirement, Testator’s
will effectively exercised the general testamentary power of appointment.

Explanation to Point-Two (20-30%):

At common law, Daughter takes the family home free of the mortgage. Under the law of most states
today, Daughter takes the home subject to the mortgage on which Testator was liable at the time of his
death. The mortgage is not payable from the assets of Testator’s estate.

Under the common-law doctrine of exoneration, the specific devisee of encumbered real property was
entitled to have the mortgage on the property paid from the estate as a debt of the decedent unless there
was evidence of a contrary intent on the part of the testator.

On the other hand, many states have adopted statutes contrary to this common-law rule of exoneration.
For example, the Uniform Probate Code provides that “a specific devise passes subject to any mortgage
interest existing at the date of death, without right of exoneration, regardless of a general directive in the
will to pay debts.” In states with statutes of this type, the specific devisee of encumbered property takes
subject to the mortgage notwithstanding the fact that the will contained a clause directing the executor to
pay the decedent’s debts.

Courts have also held that a general directive to pay debts is insufficient to evidence an intent to exonerate
the devisee of specifically devised property. Depending on the underlying state law, Daughter takes either
subject to the mortgage or free of the mortgage.

Explanation to Point-Three (25-35%):

The insurance proceeds payable as a result of the theft of the watch shortly before Testator’s death should
pass to Son as the specific devisee of the watch. There is, however, contrary authority.

Under the doctrine of ademption, if the subject matter of a specific devise is not in the probate estate at
the time of the testator’s death, the bequest to the devisee adeems (or fails). However, if the property was
destroyed by fire or lost by theft and proceeds of insurance are paid to the executor of the estate in
settlement of claims against the insurance company, some courts hold that the insurance proceeds are
payable to the specific devisee as a substitute for the specifically devised property.

However, some state courts have held to the contrary, finding that the insurance proceeds pass as part of
the testator’s residuary estate. They find that the insurance policy is a separate and distinct asset from the
property subject to the specific devise and that the proceeds, as specifically undisposed property, pass to
the residuary legatees under the will.

It seems that the majority rule, as well as in the Uniform Probate Code and some other state codes is the
better rule because it is more likely to accord with the reasonable expectations of a testator.

Seperac-J19 Exam-Released MEE Essay Compilation © 2016-2020 1356


#320-JUL 2001–MEE Q04: QUESTION FOUR (WILLS-ESTATES)

On May 18, 1997, Testator duly executed a typewritten will in the presence of three witnesses. The will
contained only the following three paragraphs:

1. I give my watch to my brother, Ben.

2. I give my dining room table to my sister, Sarah.

3. I give the balance of my tangible personal property to the person named in a letter I signed and dated
May 17, 1997, which I have placed in the desk in my home.

Testator died on January 2, 2000, a domiciliary of State A. The foregoing will was found in the desk in
Testator’s home. However, in paragraph 2 of the will, the phrase “dining room table” had been scratched
out and immediately above it the word “automobile” was typed. And, on the back of the will, the
following language appeared in Testator’s handwriting: “I don’t want Ben to have my watch. I want it to
go to my first cousin, Chris.” No signatures appeared on the back of the will beneath this writing.

The letter referred to in paragraph 3 of the will was found in the desk, and named Nicole, the daughter of
Sarah, as the beneficiary.

Testator’s only surviving blood relatives are Ben, Sarah, Chris, and Nicole. In addition to the watch,
dining room table, and automobile, Testator left a $10,000 bank account.

State A permits wills to be completely or partially revoked by the execution of a subsequent will or
codicil, by physical act or by cancellation, when accompanied by an intent to revoke. State A law also
provides that “unsigned holographic wills or codicils are valid.”

To whom should Testator’s estate be distributed? Explain.

Seperac-J19 Exam-Released MEE Essay Compilation © 2016-2020 1357


#320: J01-4 MEE: ANSWER: NCBE (WILLS-ESTATES)

POINT (1) [26%] ISSUE: Was the bequest of the watch to Ben properly revoked, such that the
watch passes to Chris? ANSWER: No. The gift of the watch to Ben was not revoked by either
physical act or cancellation. It was revoked, however, by a subsequent will since State A law
permits testators to execute holographic wills. Thus, the watch passes to Chris.

POINT (2) [40%] ISSUE: Is Sarah entitled to receive the dining room table, the automobile, or
neither? ANSWER: The bequest of the dining room table to Sarah was revoked by physical act.
Testator revoked the original bequest with the apparent intent of substituting for it a bequest of the
automobile. However, the substituted bequest cannot be sustained as a holographic will because it
was typewritten. Furthermore, the doctrine of dependent relative revocation is not likely to apply to
prevent the bequest of the table from being revoked.

POINT (3) [17%] ISSUE: Is Nicole entitled to the balance of Testator’s tangible personal property
under the doctrine of incorporation by reference? ANSWER: Yes. Under the doctrine of
incorporation by reference, Nicole should be entitled to the balance of Testator’s tangible personal
property.

POINT (4) [17%] ISSUE: To whom should the undisposed of property (the $10,000 bank account)
be distributed? ANSWER: The bank account should be distributed to Sarah and Ben as the sole
heirs of Testator.

ANSWER EXPLANATION:

Explanation to Point-One (25-30%):

The gift of the watch to Ben was not revoked by either physical act or cancellation. It was revoked,
however, by a subsequent will since State A law permits testators to execute holographic wills. Thus, the
watch passes to Chris.

State A law permits wills to be completely or partially revoked by the execution of a subsequent will or
by physical act or cancellation. In this case, no physical act was done to the face of the will to evidence
that either the entire will or any of its provisions were revoked. On the other hand, on the back of the will,
Testator, in her own handwriting, stated: “I don’t want Ben to have my watch. I want it to go to my first
cousin, Chris.” Because these words do not come into physical contact with the words of the will, they are
not words of revocation. In contrast, the Uniform Probate Code permits revocation by cancellation even
though words of cancellation do not touch words of the will. However, under the laws of State A, these
words can be construed as a holographic codicil.

In some states, the handwriting on the back of the will would not be a valid holographic will because of
underlying state statutes requiring holographic wills to be signed. For example the Uniform Probate Code
requires holographic wills to be signed. State A, however, has no signature requirement.

A holographic will can revoke an earlier typewritten will. If the subsequently executed holographic will
only disposes of a part of the estate disposed of in the typewritten will, then the typewritten will is
revoked only to the extent that it is inconsistent with the provisions of the holographic will. Here, the

Seperac-J19 Exam-Released MEE Essay Compilation © 2016-2020 1358


holographic will is inconsistent with the typewritten will only to the extent that it leaves the watch to
Chris, rather than Ben. Thus, no other disposition in the typewritten will is revoked.

Explanation to Point-Two (35-50%):

The bequest of the dining room table to Sarah was revoked by physical act. Testator revoked the original
bequest with the apparent intent of substituting for it a bequest of the automobile. However, the
substituted bequest cannot be sustained as a holographic will because it was typewritten. Furthermore, the
doctrine of dependent relative revocation is not likely to apply to prevent the bequest of the table from
being revoked.

Under State A law, a bequest may be revoked by cancellation if the cancellation is accompanied by an
intent to revoke. When a will is found with marks of cancellation upon it (here, the scratches through the
phrase “dining room table”), a presumption arises that such marks were made by the testator with the
intent to revoke. Here, no facts suggest that such marks of cancellation could have been done by anyone
but Testator and in light of the presumption, such marks were done by the Testator with the intent to
revoke. However, typing the word “automobile” above “dining room table” suggests that Testator
revoked the bequest of the dining room table in order to substitute for it a bequest of the automobile.
Unfortunately, that typed bequest was neither signed nor witnessed and cannot be sustained as a valid
holographic codicil because it was not in Testator’s handwriting. Thus, Sarah takes nothing unless the
revocation of the bequest of the dining room table can be avoided under the doctrine of dependent relative
revocation.

Under the doctrine of dependent relative revocation, a revocation can be deemed conditional on the
validity of a subsequently executed will or codicil if that would accomplish the testator’s intent. Under
this doctrine, if the subsequently executed will is invalid, then the revocation which was dependent upon
it is ignored. Typically, however, courts apply that doctrine only when there is a sufficiently close identity
between the bequest that was revoked and the bequest that was expressed in the invalid subsequent will.
Here, there is no such close identity between a dining room table and an automobile, and accordingly a
court is unlikely to apply the doctrine of dependent relative revocation to prevent the bequest of the dining
room table from being revoked. If, on the other hand, Sarah could find evidence that Testator’s revocation
was conditional, then she would be entitled to the dining room table. See Point Three for disposition of
the automobile and the dining room table.

Explanation to Point-Three (15-20%):

Under the doctrine of incorporation by reference, Nicole should be entitled to the balance of Testator’s
tangible personal property.

Under the doctrine of incorporation by reference, a will may refer to an unattested written document and
incorporate its terms into the will so long as that written document was in existence at the time the will
was signed and the evidence is clear that testator intended to incorporate its terms into her will. Where
such a document is incorporated the dispositive provisions in it are given the same effect as if they had
been set forth in the duly attested will. Here, the letter referred to in Testator’s will was dated May 17,
1997, the day before Testator signed her will. Thus, it was in existence on the date Testator signed the
will. Testator’s will specifically refers to the letter by date and reflects a clear intent that the disposition in
the letter be given effect. Thus, Nicole should be entitled to all of Testator’s remaining tangibles, and
probably the specific tangibles ineffectively bequeathed to Sarah. (This presumes that item three of the
will operates like a residuary clause but with respect to the tangibles only.) Some states have statutes that

Seperac-J19 Exam-Released MEE Essay Compilation © 2016-2020 1359


specifically authorize the bequest of tangibles in the manner used here and validate such bequests even if
the memo was executed after the will.

Explanation to Point-Four (15-20%):

The bank account should be distributed to Sarah and Ben as the sole heirs of Testator.

Where a testator’s will fails to dispose of all of the testator’s property, the undisposed of property passes
to the testator’s heirs. Typically, the intestacy laws of the states provide that, absent descendants or a
spouse, an intestate’s property is distributed to (1) parents, (2) the descendants of the parents, (3) more
remote ancestors, and (4) their descendants, in that order. Here, Testator’s closest living relatives are
Sarah and Ben and as such they are equally entitled to all of Testator’s property not disposed of by
Testator’s will.

Seperac-J19 Exam-Released MEE Essay Compilation © 2016-2020 1360


#321-FEB 2001–MEE Q01: QUESTION ONE (WILLS-ESTATES)

Testator’s last will included the following four dispositive provisions:

1. I leave my family portrait painted by Painter to my sister, Susan.

2. I leave $100,000 each to my child Leslie, my child Doris, and my wife, Wendy.

3. I leave $40,000 to my brother, Ben, and direct that my stamp collection be sold to satisfy this bequest.

4. I leave the balance of my estate to the trustees of my alma mater, University.

Testator’s will was signed by Testator and witnessed by Testator’s brother, Ben, and by Testator’s
accountant, Aaron.

Two years after the will was executed, Testator and Wendy divorced. As a result of their property
settlement agreement incident to that divorce, Testator transferred $200,000 to Wendy. That agreement
imposed no further obligations on Testator.

Testator never remarried. Three years after the divorce, Testator died a domiciliary of State A. He was
survived by Leslie and Doris, his only heirs, and the other individual devisees named in his will.
Testator’s estate consisted of the family portrait valued at $2,000, the stamp collection valued at $20,000,
and $110,000 on deposit at a local bank.

State A law provides that a will must be signed by the testator and witnessed by two witnesses.

How should Testator’s estate of $132,000 be distributed? Explain.

Seperac-J19 Exam-Released MEE Essay Compilation © 2016-2020 1361


#321: F01-1 MEE: ANSWER: NCBE (WILLS-ESTATES)

POINT (1) [29%] ISSUE: Was either the will or the bequest to Ben invalidated because Ben was
one of the two witnesses to Testator’s will? ANSWER: The validity of the will is unaffected by the
fact that Ben was a witness to the will. However, in most states Ben forfeits the $40,000 bequest
because two disinterested witnesses did not witness the will. On the other hand, in states that have
adopted the Uniform Probate Code, Ben does not forfeit that bequest.

POINT (2) [29%] ISSUE: Was the bequest to Wendy revoked as a result of her divorce from
Testator? ANSWER: Yes. The bequest of $100,000 to Wendy is revoked as a result of her divorce
from Testator after the execution of the will.

POINT (3) [42%] ISSUE: Since the assets of Testator’s estate of $132,000 are insufficient to satisfy
all of the bequests under the will, how should the bequests abate? ANSWER: Legacies abate in the
following order: residuary, general, and specific. Testator’s $132,000 estate is insufficient to satisfy
all of the general and specific legacies under the will. Thus, the residuary estate fully abates, the
specific bequest of the portrait can be fully funded, and the remaining bequests abate to a limited
extent.

ANSWER EXPLANATION:

Explanation to Point-One (30-40%):

The validity of the will is unaffected by the fact that Ben was a witness to the will. However, in most
states Ben forfeits the $40,000 bequest because two disinterested witnesses did not witness the will. On
the other hand, in states that have adopted the Uniform Probate Code, Ben does not forfeit that bequest.

At common law, if a witness who received a benefit under a will witnessed the will, the will was invalid
unless two disinterested witnesses also witnessed the will. The will was invalid because the interested
witness was not competent to testify about the validity of the will in court; thus, its validity could not be
judicially established. In virtually every state, the common-law rule has been abolished and the witnessing
of a will by an interested witness does not affect the validity of the will. On the other hand, most states
have statutes that provide that unless two disinterested witnesses witness the will, the interested witness
forfeits the bequest to that witness. Some states temper that result if the interested witness would have
been an heir of the testator by limiting the amount forfeited to the amount, if any, by which the bequest to
the interested witness exceeds the beneficiary-witness’s intestate share. If such statutes applied here, Ben
would forfeit the entire $40,000 bequest, as he is not an heir of Testator.

Under the Uniform Probate Code, on the other hand, an interested witness does not forfeit any portion of
the bequest to the witness. The Uniform Probate Code justifies that approach by noting that the interested
witness statute rarely prevents undue influence (an argument usually made to justify such statutes) and
typically adversely affects bequests to innocent persons who act as witnesses to wills at a testator’s
express request. Under the Uniform Probate Code, Ben would not forfeit any portion of the $40,000
bequest.

Explanation to Point-Two (30-40%):

Seperac-J19 Exam-Released MEE Essay Compilation © 2016-2020 1362


The bequest of $100,000 to Wendy is revoked as a result of her divorce from Testator after the execution
of the will.

At common law, divorce did not revoke a bequest to a former spouse by operation of law. In most states
today, a divorce revokes a provision in a will for the testator’s former spouse unless the will, a contract
between the former spouses, or a court order expressly provides otherwise. In other states, divorce
revokes a will provision for the spouse but only if the divorce is accompanied by a property settlement
agreement. Here, Testator’s divorce from Wendy was accompanied by a property settlement agreement.
Thus, the provision for Wendy was revoked and she is not entitled to any share of Testator’s estate.

Where a provision in favor of the former spouse is revoked by operation of law, as in this case, the
bequest passes as if the former spouse predeceased the testator. While Testator’s will did not expressly
require Wendy to survive to take, by virtue of the statutory fiction that Wendy predeceased Testator, the
bequest lapses. It lapses because lapse statutes do not apply to bequests to former spouses. Typically,
lapsed general legacies are distributed as part of the residuary estate.

Explanation to Point-Three (40-60%):

Legacies abate in the following order: residuary, general, and specific. Testator’s $132,000 estate is
insufficient to satisfy all of the general and specific legacies under the will. Thus, the residuary estate
fully abates, the specific bequest of the portrait can be fully funded, and the remaining bequests abate to a
limited extent.

Both at common law and under the typical state abatement statute, legacies abate in the following order:
(1) residuary bequests, (2) general bequests, and (3) specific bequests. For purposes of abatement, a
demonstrative bequest is treated as specific to the extent of the designated fund from which it is payable,
and treated as general to the excess, if any.

Under Testator’s will, Testator specifically bequeathed the family portrait to Susan. Testator created two
effective general legacies of $100,000 each in Leslie and Doris. (The bequest to Wendy was revoked by
operation of law.) The legacy to Ben is forfeited under the interested witness statute in most jurisdictions.
In those states where it is not forfeited, the bequest of $40,000 to Ben is treated as a demonstrative legacy
that is specific to the extent of the value of the stamp collection ($20,000) and general as to the remaining
$20,000 of value.

If the bequest to Ben is forfeited because of an interested witness statute, there are $130,000 of assets (not
counting the specifically devised portrait) available for distribution to Leslie and Doris, who would each
take $65,000. Susan would take the portrait valued at $2,000. The residuary bequest to University is
totally abated.

If the bequest of $40,000 to Ben is not forfeited, then $20,000 of that bequest (the portion that cannot be
satisfied by the specific bequest of the stamp collection) as well as the $100,000 bequests to Leslie and
Doris are treated as general legacies for a total of $220,000. Thus, the $110,000 of the estate remaining
after $22,000 of assets is distributed to Ben and Susan, the two specific legatees, would have to be
allocated among the three general legatees to whom $220,000 had been bequeathed. Of that, $10,000
(1/11th) would be payable to Ben, $50,000 (5/11th) would be payable to Leslie, and $50,000 (5/11th)
would be payable to Doris. The residuary estate is totally abated.

Seperac-J19 Exam-Released MEE Essay Compilation © 2016-2020 1363


#322-JUL 2000–MEE Q02: QUESTION TWO (WILLS-ESTATES)

Testator, a domiciliary of State A, died on February I, 1998, leaving a net estate valued at $100,000.
Testator was survived by three children, Andy, Billy, and Cory, and one grandchild, Gary, who is Cory’s
only child. The following documents were found in Testator’s safe deposit box:

1. At the top, there was an envelope containing Testator’s canceled check dated February 1, 1990, made
payable to Charity Y in the amount of $15,000. Attached to this check were copies of other canceled
checks to Charity Y paid over the previous 15 years and a copy of a note from Testator to Charity Y dated
February 1, 1990, stating: “Enclosed is my check in the amount of $15,000.1 know you will use this as
you have all of my prior gifts to carry out the important work of your organization. I hope I’ll be in a
position to do more for you in the future. Your work is very important to me.”

2. Underneath the first envelope was a second envelope containing a one-page typewritten will dated
September 10, 1989, leaving all of Testator’s estate to Andy and Billy. Testator had duly executed this
will. However, the phrase “VOID – May 18, 1993” was written in large print across the entire face of the
will, and on the back of the will the following words appeared: “Revoked, because I really prefer my
March 2, 1983 will.”

3. Underneath the second envelope was Testator’s duly executed one-page typewritten will dated March
2, 1983. In this will, Testator bequeathed “$50,000 to Cory, $10,000 to Charity Y, and the residue of the
estate to Andy and Billy.”

On October 30, 1998, Cory filed a properly acknowledged document with the appropriate persons,
renouncing all rights Cory had under Testator’s two wills.

State A does not permit holographic wills. Under State A law, (1) an intestate’s heirs are the intestate’s
surviving children, and (2) the issue of any child who predeceases the intestate take the deceased child’s
share.

1. Under what theory could the March 2, 1983 will be probated, and to whom would the estate be
distributed under that will? Explain.

2. Under what theory could the September 10, 1989 will be probated, and to whom would the estate
be distributed under that will? Explain.

Seperac-J19 Exam-Released MEE Essay Compilation © 2016-2020 1364


#322: J00-2 MEE: ANSWER: NCBE (WILLS-ESTATES)

POINT (1)(a) [33%] ISSUE: Is the March 2 will entitled to be probated because it was revived
when Testator’s September 10 will was revoked? ANSWER: Yes. The March 2 will can be
probated if State A permits a will that has been revoked by the execution of a later will to be
revived if the later will is revoked. If the March 2, will, is revived. Andy and Billy take the residue
but how the general bequests are distributed depends on the resolution of two separate issues.

POINT (1)(b) [14%] ISSUE: Assuming the March 2 will is entitled to be probated, should $10,000
be distributed to Charity Y or was the bequest to Charity Y satisfied before Testator died by
Testator’s gift to the charity of $15,000 on February 1, 1990? ANSWER: If the March 2 will is
probated, then $10,000 should be distributed to Charity Y. Testator’s lifetime gift to Charity Y is
not a gift in satisfaction of this general bequest.

POINT (1)(c) [14%] ISSUE: Assuming the March 2 will is entitled to be probated, did Cory timely
disclaim his interest in the $50,000 bequest and, if so, does the bequest pass to Gary or to the
residuary legatees under Testator’s will? ANSWER: If the March 2 will is entitled to be probated
Cory timely disclaimed his interest in the $50,000 bequest. The bequest should pass to his child,
Gary.

POINT (2) [38%] ISSUE: If the March 2 will was not revived, can the September 10 will be
probated under the doctrine of dependent relative revocation and, if so, how will the estate be
distributed? ANSWER: If the March 2 will was not revived, then the September 10 will is entitled
to be probated under the doctrine of dependent relative revocation (DRR) if it is established that
Testator would have preferred the disposition in the September 10 will to intestacy. Otherwise,
Testator died intestate.

ANSWER EXPLANATION:

Explanation to Point-One(a) (30-40%):

The March 2 will can be probated if State A permits a will that has been revoked by the execution of a
later will to be revived if the later will is revoked. If the March 2, will, is revived. Andy and Billy take the
residue but how the general bequests are distributed depends on the resolution of two separate issues.

The execution of the September 10 will revoked the March 2 will under the laws of all states. Under the
common law, however, the later revocation of the second will revives the first will. This rule followed
from the principle that wills only speak at the time of death and, since the second will was revoked and
therefore is a nullity, only the prior will had legal effect. Because of the belief that the common-law rule
frustrated the intent of testators who revoked the later will with the intent of dying intestate, many states
adopted statutes rejecting the common-law revival rule. These statutes can take at least two forms. One is
that the revocation of the second will does not revive the earlier will. The other form, as exemplified by
the Uniform Probate Code § 2-509, is that the revocation of the second will does not revive the earlier will
unless “it is evident from the circumstances of the revocation of the subsequent will or from the testator’s
contemporary or subsequent declarations that the testator intended the previous will to take effect as
executed.”

Seperac-J19 Exam-Released MEE Essay Compilation © 2016-2020 1365


A will may be revoked by a cancellation. When revoking the September 10 will by cancellation, the result
of writing “VOID” across the face of the will, Testator explicitly indicated that he preferred his March 2
will. This statement is evidence of his intent to revive the earlier will. Thus, either under the common-law
revival rule or a statutory substitute incorporating an intent standard, the March 2 will is entitled to be
probated. Only if State A has an absolute no-revival rule would the March 2 will not be entitled to be
probated.

Explanation to Point-One(b) (10-20%):

If the March 2 will is probated, then $10,000 should be distributed to Charity Y. Testator’s lifetime gift to
Charity Y is not a gift in satisfaction of this general bequest.

Under the common law, if, subsequent to the execution of a will, a testator gives property to a person who
is named as a general legatee in a will executed by the testator prior to the making of the gift, a rebuttable
presumption arises that the testator intended the gift to be in satisfaction of the general legacy. This is
known as the doctrine of “satisfaction” or “ademption by satisfaction” The common-law rule is often
abrogated or modified by statute. For example, under the Uniform Probate Code § 2-609, a gift to a
legatee is not in satisfaction of the legacy unless the testator declared in a contemporaneous writing that
he/she intended the gift to be in satisfaction of the bequest. In other states, the gift is not in satisfaction
absent some writing evidencing testator’s intent that the gift be in satisfaction of the bequest.

However, the ultimate result in this case is arguably the same whichever version of the doctrine applies
because it appears Testator did not intend the $15,000 gift to be taken into account in determining the
amount distributable to Charity Y under the will. This is evident from the language in the document found
with the check that the $15,000 was one of many gifts and that Testator hoped to be in a position to make
future gifts to Charity Y because the charity’s work was so important to him. While it might be argued
that the mere fact that Testator kept the check and note with his will evidences an intent that the gift of
$15,000 be in satisfaction of the bequest, that argument seems weak in light of the more specific language
on the note evidencing Testator’s intent to continue making gifts to the charity.

Explanation to Point-One(c) (10-20%):

If the March 2 will is entitled to be probated Cory timely disclaimed his interest in the $50,000 bequest.
The bequest should pass to his child, Gary.

The typical disclaimer statute provides that a disclaimer must be made within nine months of the
testator’s death. The nine-month period is typical because state disclaimer statutes are designed to
comport with the federal disclaimer rules. Since Cory disclaimed on October 30, 1998, Cory’s disclaimer
was made within nine months of Testator’s death and therefore was timely.

The typical disclaimer statute further provides that if a timely disclaimer is made, the disclaimed property
interest passes as if the disclaimant died before the testator. In this case, the disclaimed property interest
would pass in accordance with any applicable lapse statute to Cory’s child, Gary. The lapse statute applies
because (1) Cory is a descendant of Testator (a typical requirement of a lapse statute), (2) Cory has issue
who survive Testator (another typical requirement of a lapse statute), and (3) Testator’s will did not
specially condition Cory’s bequest on Cory surviving Testator.

Explanation to Point-Two (35-45%):

Seperac-J19 Exam-Released MEE Essay Compilation © 2016-2020 1366


If the March 2 will was not revived, then the September 10 will is entitled to be probated under the
doctrine of dependent relative revocation (DRR) if it is established that Testator would have preferred the
disposition in the September 10 will to intestacy. Otherwise, Testator died intestate.

The March 2 will was revoked by the execution of the September 10 will regardless of whether the latter
will was revoked. If the March 2 will is not revived, either the revocation of the September 10 will is
ignored under DRR or Testator died intestate. Most likely Testator died intestate.

If a valid will is revoked by a physical act (such as cancellation) under Testator’s mistaken belief that in
doing so Testator could revive an earlier will, the revocation of the will can be ignored under DRR.
Because DRR is an intent-effectuating doctrine, it applies only when the court determines that the testator
would prefer the disposition in the revoked will to that resulting from a determination that the testator
died intestate. Courts are more likely to apply DRR if the distribution of the estate using DRR
approximates more closely the distribution testator actually intended than the distribution that would
occur if DRR did not apply. When that is not the case, courts are more likely to reject using DRR.

For example, if the intended revocation of the September 10 will is ignored using DRR and, thus, that will
is entitled to probate, Testator’s entire $100,000 estate would be distributed equally to Andy and Billy.
Cory, on the other hand, would take nothing. Alternatively, if the September 10 will is revoked and DRR
is not applied, Testator would be deemed to have died intestate and Testator’s entire estate would be
distributed in equal thirds to Andy, Billy, and to Cory, since Cory’s disclaimer did not apply to any
intestate share.

It would appear that a distribution of $33,000, $33,000, and $33,000 to Andy, Billy, and Cory (ignoring
DRR and using intestacy) approximates more closely what Testator actually intended – $50,000 to Cory,
$10,000 to Charity Y, $20,000 to Andy and $20,000 to Billy, which was the distribution under the March
2 will – than a distribution of $50,000 to Andy and $50,000 to Billy (applying DRR and distributing the
estate under the September 10 will). Here, Cory gets less than intended but at least something, while
Andy and Billy get somewhat more than Testator intended. Thus, not applying DRR arguably more
closely approximates Testator’s intent. Of course, this is a close question, and examinees could argue
either way.

Seperac-J19 Exam-Released MEE Essay Compilation © 2016-2020 1367


#323-FEB 2000–MEE Q04: QUESTION FOUR (WILLS-ESTATES)

Banks was a 70-year-old bachelor. His closest living relatives were his two nieces. Banks had long been a
patron of the arts and of the Opera Society. On January 15, 1997, Banks executed a will bequeathing
$100,000 to each of his nieces and leaving his residuary estate to the Banks Foundation for Opera, a
private foundation that he planned to establish when he retired. He hoped that in gratitude for his
generosity the Opera Society would name the new opera house after him.

A year later, Banks learned that he had a terminal disease. He told his nieces he would leave them his
entire estate if they would take care of him. They agreed to do so, and, in February of 1998, Banks moved
in with his two nieces and asked his attorney to draft a new will bequeathing them all his property and
expressly revoking his January 15, 1997 will.

Shortly before Banks was to execute the new will, the president of the Opera Society told Banks that the
Board had agreed to name the building in his honor if they received a substantial bequest from him. Banks
then decided to execute the new will using his two nieces as the only witnesses, believing this would
render the will invalid. A day later, Banks signed the new will and, at his request, the nieces witnessed it.
It was dated February 10, 1998. Banks placed this will inside an envelope, wrote in ink on the outside of
the envelope, “Not to be opened until my death,” and initialed the writing.

Banks died without having created the Banks Foundation for Opera. The February 10, 1998 will was
found in the envelope with the following unsigned note in Banks’s handwriting on the back of the last
page of the will:

Ignore this will. I never intended to give my whole estate to my nieces. I signed it only to trick them into
caring for me. Long live the Banks Opera House!

1. For what reasons, if any, could the February 10, 1998 will be denied probate as Banks’s last
will? Explain.

2. If the will of January 15, 1997 is probated, on what theory or theories, other than fraud, may the
nieces assert a claim against the estate of their uncle for their care of him? Explain.

3. If the will of January 15, 1997 is probated, on what theory, if any, might the Opera Society assert
a claim to the residuary estate? Explain.

Seperac-J19 Exam-Released MEE Essay Compilation © 2016-2020 1368


#323: F00-4 MEE: ANSWER: NCBE (WILLS-ESTATES)

POINT (1) [50%] ISSUE: Can the 1998 will witnessed only by interested witnesses be challenged on
the grounds that it was not executed with testamentary intent, it was witnessed by interested
persons, or it was revoked? ANSWER: Testator’s 1998 purported will can be denied probate
because it was not executed with the requisite testamentary intent. If evidence of a lack of intent is
inadmissible such that on its face the will is valid, the 1998 document might arguably have been
revoked by cancellation or by codicil, and therefore is not entitled to be probated. If not so revoked,
it is entitled to be probated.

POINT (2) [15%] ISSUE: Assuming that only the 1997 will is entitled to be probated, can the
testator’s nieces claim any share of the testator’s estate on the theory that they have a claim against
his estate? ANSWER: No. Banks’s nieces probably cannot establish a claim against his estate on the
basis of breach of contract to make a will, but they may be entitled to recover the reasonable value
of their services to him.

POINT (3) [35%] ISSUE: Assuming that only the 1997 will is entitled to be probated, can the
testator’s residuary estate be distributed to the Opera Society in lieu of the Banks Foundation for
Opera under the cy pres doctrine, or does the bequest to the foundation fail, so that the residue of
the testator’s estate is distributed only to his heirs? ANSWER: Yes. Assuming that the 1997 will is
probated and the nieces are not entitled to the whole estate, the residue of Banks’s estate should be
distributed to the Opera Society in lieu of the Banks Foundation by application of the cy pres
doctrine.

ANSWER EXPLANATION:

Explanation to Point-One (40-60%):

Testator’s 1998 purported will can be denied probate because it was not executed with the requisite
testamentary intent. If evidence of a lack of intent is inadmissible such that on its face the will is valid, the
1998 document might arguably have been revoked by cancellation or by codicil, and therefore is not
entitled to be probated. If not so revoked, it is entitled to be probated.

The facts indicate that Banks intended to have the 1998 document executed in such a way that it would be
invalid. He did this because he wanted his property to pass to the Banks Foundation for Opera under the
terms of his 1997 will without his nieces knowing this, so they would continue to take care of him. What
Banks failed to appreciate was that the mere fact that a will is witnessed by beneficiaries under the will
does not mean the will is invalid. In fact, in almost all jurisdictions, the validity of the will is unaffected
by whether a witness is interested, although interestedness on the part of a witness may have an adverse
impact on the witness’s right to receive the bequest. Jurisdictions which have adopted the Uniform
Probate Code (UPC) are even less restrictive since in those jurisdictions an interested witness does not
forfeit the testamentary gift.

Although Banks’s will was properly executed, it is invalid as a will if it was not executed with any
testamentary intent. In the absence of testamentary intent, a document, even though executed with the
statutory formalities, is invalid. Testamentary intent can be established by extrinsic evidence, and while
cases such as this – where it is the absence of intent that is at issue – are rare, presumably lack of intent
can also be shown by extrinsic evidence.

Seperac-J19 Exam-Released MEE Essay Compilation © 2016-2020 1369


The evidence in this case, that Banks executed the 1998 will without testamentary intent and to
accomplish his goals used his nieces as witnesses mistakenly believing this would invalidate his will, is
also reaffirmed by Banks’s handwritten note on the back of the last page of the will. This note would
likely constitute sufficient evidence to prove that the will was not executed with testamentary intent. On
the other hand, if the note were not admissible for that purpose (e.g., in jurisdictions with a very restrictive
view regarding the admissibility of extrinsic evidence), the handwritten note might be a sufficient
revocation of the will by cancellation, at least in some jurisdictions.

Under § 2-507 of the Uniform Probate Code, a will can be revoked by a number of acts, including a
cancellation. Under the UPC, words of cancellation need not touch any of the words of the will, but they
must be somewhere on the will. Under UPC § 2-507, the note revokes the will because it was written on
the will. A different result follows in jurisdictions that follow the common-law rule, requiring words of
cancellation to “touch” the words of the will. In such jurisdictions, words of cancellation are ineffective if
they did not come in contact with the will.

Could the 1998 will be viewed as having been revoked by a codicil? That would depend upon whether
local law recognized holographic wills, whether holographic wills had to be signed, and whether Banks’s
initials on the envelope would meet any signing requirement, information that is not provided here.

Explanation to Point-Two (10-20%):

Banks’s nieces probably cannot establish a claim against his estate on the basis of breach of contract to
make a will, but they may be entitled to recover the reasonable value of their services to him.

Ordinarily, contracts to make a will must be in writing. In the absence of a writing, such contracts are
unenforceable. Jurisdictions that do not expressly address this matter in their probate codes may reach the
same result by applying a statute of frauds. On the facts given, Banks’s agreement with his nieces was
made orally. No facts suggest that it was reduced to writing. Accordingly, any claim by the nieces for
breach of a contract to make a will would face the hurdle that they never put their agreement with their
uncle in writing, as required by most probate codes.

On the other hand, it could be argued that there is enough written evidence of the agreement to satisfy any
writing requirement even though there is no formal written contract. The 1998 will itself combined with
the note on the back of that will provide some written evidence of Banks’s promise to his nieces. In
addition, if a local statute of frauds (rather than the probate code) creates the requirement of a writing,
arguably the doctrine of part performance or estoppel may avoid the operation of that statute. On that
basis the nieces may be able to assert against Bank’s estate a claim of breach of contract to make a will.

If a jurisdiction does not require a writing to establish a contract to make a will, the nieces should be able
to recover the amount of their damages, assuming the 1997 will is probated.

If the nieces’ claim cannot be sustained as a contract to make a will, they may be able to recover damages
in an amount equal to the reasonable value of their services or impose a constructive trust for such amount
on the successor of Banks’s estate.

Explanation to Point-Three (30-40%):

Assuming that the 1997 will is probated and the nieces are not entitled to the whole estate, the residue of
Banks’s estate should be distributed to the Opera Society in lieu of the Banks Foundation by application
of the cy pres doctrine.

Seperac-J19 Exam-Released MEE Essay Compilation © 2016-2020 1370


The residuary bequest in favor of the Banks Foundation for Opera cannot be given effect, even if this will
is probated, because the Foundation was never organized prior to Banks’s death. Where a charitable
bequest becomes impossible or impracticable to fulfill, a court may use the doctrine of cy pres to reform
that gift to accomplish the testator’s general charitable intent. Cy pres is appropriate unless a court
determined that Banks’s intent was limited only to benefiting the foundation that was never formed. This
seems unlikely under the facts of the case since the purpose of the foundation was itself to benefit the
Opera Society, which can just as easily be accomplished by substituting it as the residuary legatee under
the will.

Seperac-J19 Exam-Released MEE Essay Compilation © 2016-2020 1371


#324-JUL 1999–MEE Q06: QUESTION SIX (WILLS-ESTATES)

Tom and Wanda were married and had a child, Anna. After Wanda died, Tom had a nonmarital
relationship with Nancy, who had a child, Bill, from a prior marriage. At the time the relationship
between Tom and Nancy began, Anna was 12 and Bill was 17. Three years after the relationship between
Tom and Nancy began, they had a child, Chris. Shortly thereafter Nancy was killed in an automobile
accident. Tom continued to raise all three children as his own but took no steps to adopt Bill. Although he
acknowledged he was Chris’s father, he took no steps to establish his paternity judicially.

During the 20 years following Nancy’s death, the relationship between Tom and Anna soured, but Tom’s
relationships with Bill and Chris blossomed. Eventually, Tom came to have little to do with Anna. Last
year, things came to a head. At Thanksgiving dinner, Tom handed each of the children a check in the
amount of $50,000. Each check was accompanied by a personal handwritten note signed by Tom. The
note to Anna read: “You have been a constant disappointment to me over these years. Take this but expect
no more from me.” The notes to Bill and Chris, which were identical, read: “You have been a wonderful
child to me. Use this gift well. I love you very much.”

Four months ago, Anna died survived by her son, Danny. One month later, Tom died intestate leaving a
probate estate of $250,000. Tom was survived by Bill, Chris, and Danny.

A state statute provides that property descends to issue if the intestate has no surviving spouse. It defines
“children” as children born during marriage, adopted children, and children born out of wedlock, provided
paternity is established judicially or is otherwise recognized by the father. State law prohibits holographic
wills.

How should Tom’s estate be distributed? Explain.

Seperac-J19 Exam-Released MEE Essay Compilation © 2016-2020 1372


#324: J99-6 MEE: ANSWER: NCBE (WILLS-ESTATES)

POINT (1) [36%] ISSUE: Is Bill, who was never adopted by Tom but who was raised by Tom,
entitled to a share of Tom’s estate? ANSWER: No. Bill would not be entitled to any share of Tom’s
estate unless Bill could establish that he was entitled to a share of the estate under the doctrine of
“equitable adoption” or “adoption by estoppel.” However, it is unlikely Bill could successfully claim
to have been equitably adopted.

POINT (2) [31%] ISSUE: Is Chris, born to Tom out of wedlock, entitled to a share of Tom’s estate?
ANSWER: Yes. Chris, born to Tom out of wedlock, is entitled to a share of Tom’s estate because
Tom recognized his paternity of Chris.

POINT (3) [25%] ISSUE: Is the lifetime gift to Anna an advancement, and, if so, is it taken into
account in computing Danny’s share of Tom’s estate? ANSWER: The lifetime gift to Anna was
probably intended minimally as an advancement and, if so intended, reduces the share of Tom’s
estate passing by intestacy to Anna’s child, if the governing statute were to so provide.

POINT (4) [8%] ISSUE: Would any of the lifetime gifts to Bill or Chris be taken into account in
computing any share of Tom’s estate to which they might be entitled? ANSWER: No. The lifetime
gifts to Bill or Chris were not intended as advancements and, therefore, to the extent either of them
is entitled to a share of Tom’s estate, they would not reduce the size of the share either of them
would otherwise be entitled to receive.

ANSWER DISCUSSION:

In all states, an intestate’s estate is distributable solely to the intestate’s surviving issue when the intestate
dies without a spouse, as Tom did. In this problem, the question arises who among Anna, Bill, Chris, and
Danny are issue of the intestate and whether any of the lifetime transfers to them are advancements for
purposes of computing their shares in Tom’s estate. Under the laws of all states and here, there is no
doubt that issue born-in-wedlock are an intestate’s heir. Thus Danny’s rights as an heir are unquestioned.
However the amount of his share could be affected by the doctrine of advancements.

ANSWER EXPLANATION:

Explanation to Point-One (30-35%):

Bill would not be entitled to any share of Tom’s estate unless Bill could establish that he was entitled to a
share of the estate under the doctrine of “equitable adoption” or “adoption by estoppel.” However, it is
unlikely Bill could successfully claim to have been equitably adopted.

Bill was never legally adopted by Tom and, in fact, has no biological or legally created relationship to
Tom. Thus, without more, Bill is entitled to no share of Tom’s estate. However, if Bill could establish an
adoption by estoppel, Bill could inherit from Tom as if Bill were Tom’s legally adopted child.

Courts apply the doctrine of adoption by estoppel or equitable adoption where there is a finding of either
an express or implied promise to adopt and the child “stood from an age of tender years in a position
exactly equivalent to a formally adopted child.” The facts in this case do not appear to support Bill’s
claim to an equitable adoption because, among other things, there is no express promise to adopt and Bill

Seperac-J19 Exam-Released MEE Essay Compilation © 2016-2020 1373


was well beyond “tender years” before he even came into Tom’s home. Further, a promise to adopt is not
likely to be implied. Courts have rarely implied a contract to adopt even when a stepparent raises a
stepchild, which is not even the fact in this case.

Even if Tom had married Nancy, Bill, as a stepchild, would not have been entitled to any share of Tom’s
estate since stepchildren are not “children” for purposes of inheritance under the state law.

Explanation to Point-Two (25-30%):

Chris, born to Tom out of wedlock, is entitled to a share of Tom’s estate because Tom recognized his
paternity of Chris.

Chris is a biological child of Tom. However, Chris was born out of wedlock. While the common law
prohibited a child born out of wedlock from inheriting, most states have enacted statutes allowing
children born out of wedlock to inherit under certain circumstances. Typically, an illegitimate child can
inherit from a deceased father provided paternity is proved. Laws that preclude an illegitimate child from
inheriting from the father are unconstitutional. States differ, however, regarding the burden of proof that a
child born out of wedlock must meet to establish paternity, ranging from “preponderance of evidence” in
some states to evidence that is “clear and convincing” in other states. Some states may also impose
additional requirements such as recognition of paternity in writing or by some other act; some may require
a judicial order of paternity. Under the Uniform Probate Code, a person is a child of his or her biological
parents without regard to the parents’ marital status.

In this problem, a specific statute deals with the right of a child born out of wedlock to inherit from the
father. Although Tom’s paternity was never judicially established, Chris can prevail if paternity was
otherwise recognized by Tom. This should be easy for Chris to prove or demonstrate. First, the facts state
that Tom had acknowledged that he was Chris’ father. Second, Tom’s letter to Chris at the time he gave
Chris the $50,000 referred to Chris as his child.

Explanation to Point-Three (20-25%):

The lifetime gift to Anna was probably intended minimally as an advancement and, if so intended,
reduces the share of Tom’s estate passing by intestacy to Anna’s child, if the governing statute were to so
provide.

Historically, a lifetime transfer to a person who was the transferor’s heir was treated as a down payment
(also known as an advancement) of the transferee’s share of the transferor’s estate. Most states now treat
such transfers as gifts, rather than advancements, but preserve the concept of advancements under certain
circumstances. State laws vary greatly on their laws of advancement and particularly on what acts are
sufficient to rebut the presumption that a lifetime transfer is a gift and not an advancement. For example,
under the Uniform Probate Code a transfer can be an advancement “only if (i) the decedent declared his or
her intent to that effect in a contemporaneous writing,” or the heir acknowledged the transfer to be an
advancement, or “the decedent’s contemporaneous writing otherwise indicates that the gift is to be taken
into account in computing the division and distribution of the decedent’s intestate estate.” On the other
hand, in many states no writing is required and any competent proof is admissible to establish the
transferor’s intent that the transfer was an advancement.

In this problem, it could be argued that Tom would have intended any share Anna could claim in Tom’s
estate to be reduced by $50,000. This is evidenced by the contemporaneously written note Tom gave
Anna stating: “You have been a constant disappointment to me over these years. Take this but expect no

Seperac-J19 Exam-Released MEE Essay Compilation © 2016-2020 1374


more from me.” While Tom did not expressly state that the $50,000 was an advancement, it can be argued
that this note evidences an intent to limit the amount Anna would receive from Tom in the future.
Furthermore, when this note is read alongside the other notes, it seems clear Tom did not intend Anna to
receive any more from his estate.

It could also be argued that the note expresses nothing more than Tom’s statement that Anna would be
receiving no more gifts from Tom, and that Tom did not intend the note to evidence any intent that Anna
not take an intestate share. This argument seems less plausible in light of all the facts.

If the transfer to Anna is an advancement, since Anna predeceased Tom, it remains to be determined what
effect the transfer to Anna has on the share of Anna’s son Danny (Tom’s grandchild) who takes whatever
share of Tom’s estate Anna would have been entitled to take. State laws differ on the question of whether
the share of an heir of an advancee is diminished by advancements to his or her ancestors. In some states,
the share of the successor to the advancee is affected, while in other states, the share of the successor of
the advancee is unaffected unless the transferor (by contemporaneous writing) indicates it should be.
Thus, whether the grandchild’s share is affected depends on underlying state law, which the facts do not
specify.

Explanation to Point-Four (05-10%):

The lifetime gifts to Bill or Chris were not intended as advancements and, therefore, to the extent either of
them is entitled to a share of Tom’s estate, they would not reduce the size of the share either of them
would otherwise be entitled to receive.

In his notes to Bill and Chris, Tom referred to the $50,000 as gifts. Further, no language in the note
suggests that Tom intended the gift to reduce the size of the share either Bill or Chris would receive in
Tom’s estate. Of course, Bill may not be entitled to any share of Tom’s estate, in which case the question
of whether the $50,000 transferred to Bill is an advancement is moot.

In summary, Chris is treated as an heir, the gift to him is not an advancement, Bill is probably not an heir,
and whether Danny’s share is reduced by the advancement to his mother is unclear.

Seperac-J19 Exam-Released MEE Essay Compilation © 2016-2020 1375


#325-FEB 1999–MEE Q03: QUESTION THREE (WILLS-ESTATES)

Testator’s validly executed will dated September 1, 1996, reads as follows:

I, Testator, being of sound and disposing mind, give:

1. All of my tangible personal property to Sister.

2. $1,000 to Uncle, to be paid from my bank account at Bank.

3. $5,000 to Friend.

4. 100 shares of ABC common stock to Brother.

5. The residue of my estate to Charity, a charitable organization.

At Testator’s death, Testator’s estate was valued at $50,000. This included tangibles valued at $1,000,
Testator’s bank account at Bank with a balance of $750, and 100 shares of ABC common stock valued at
$20,000. ABC is a closely held corporation. Testator bought the 100 shares in December 1995. The
balance of Testator’s estate was held in publicly traded securities.

Testator died in an automobile accident resulting from Testator’s negligence. The other party in that
accident filed a claim against Testator’s estate in the amount of $28,000. This claim was duly allowed by
the court having jurisdiction over the estate, and $28,000 of the publicly traded securities were liquidated
to pay the claim. There are no other debts, expenses, or taxes for Testator’s estate.

Sister, Uncle, Friend, and Brother survived Testator. Uncle timely disclaimed any interest in the $1,000
bequest. Uncle’s child, Son, also survived Testator.

How should Testator’s estate remaining after the payment of the $28,000 claim be distributed among
Sister, Uncle, Friend, Brother, Charity, and Son? Explain.

Seperac-J19 Exam-Released MEE Essay Compilation © 2016-2020 1376


#325: F99-3 MEE: ANSWER: NCBE (WILLS-ESTATES)

POINT (1) [23%] ISSUE: Generally, what is the order of abatement among general, specific,
demonstrative, and residuary bequests? ANSWER: Typically, when assets of an estate are
insufficient to pay all claims against the estate, the bequests abate in the following order to pay
claims: residuary bequests, general bequests, and specific bequests. Bequests charged to a specific
fund for their source of payment are specific to the extent of the fund: to the extent they exceed the
fund, they are general.

POINT (2) [33%] ISSUE: Is the bequest to Uncle general, specific, or demonstrative for purposes of
abatement? ANSWER: The bequest to Uncle of $1,000 to be paid from Testator’s bank account at
Bank is a demonstrative legacy. Therefore, it is specific to the extent of $750 and general as to $250.
Accordingly, only $250 is subject to possible abatement as a general legacy, and then only in
proportion to the other general legacies available for the payment of debts.

POINT (3) [21%] ISSUE: Should the bequest to Uncle that does not abate, if any, pass to Uncle’s
son who survives Testator or as a part of the residuary bequest? ANSWER: The interest of a
disclaimant in either a specific, general, or demonstrative bequest passes as if the disclaimant
predeceased the testator. Unless the will otherwise provides, if the disclaimant is a person described
in the applicable anti-lapse statute, the disclaimed interest passes to the disclaimant’s surviving
issue.

POINT (4) [23%] ISSUE: Is the bequest of the ABC common stock general or specific for purposes
of abatement? ANSWER: The bequest of stock is most likely specific.

ANSWER EXPLANATION:

Explanation to Point-One (20-25%):

Typically, when assets of an estate are insufficient to pay all claims against the estate, the bequests abate
in the following order to pay claims: residuary bequests, general bequests, and specific bequests. Bequests
charged to a specific fund for their source of payment are specific to the extent of the fund: to the extent
they exceed the fund, they are general.

Beneficiaries under wills are entitled only to the net value of estate assets. The net value represents what
remains of the decedent’s assets owned at death after the payment of debts, expenses, and taxes. All states
have statutes (known as abatement statutes) that determine how debts, expenses, and taxes should be
allocated among the beneficiaries of residuary, specific, and general legacies. Invariably state statutes
provide that, unless the will otherwise provides, legacies abate in the following order to pay debts,
expenses, and taxes: (1) residuary devises, (2) general devises, and then (3) specific devises. Many
abatement statutes also provide that “a general devise charged on any specific property or fund is a
specific devise to the extent of the value of the property on which it is charged, and upon the failure or
insufficiency of the property on which it is charged, a general devise to the extent of the failure or
insufficiency.”

The general and specific bequests under the will add up to $27,000. The value of the estate after payment
of the $28,000 claim is only $22,000. Because the remaining value of the estate is $5,000 less than the
general and specific bequests, the residuary bequest abates completely.

Seperac-J19 Exam-Released MEE Essay Compilation © 2016-2020 1377


Moreover, the $5,000 shortfall must be allocated among the non-residuary legatees.

A bequest is specific if the subject matter of the bequest is specific property. A bequest is general if it can
be satisfied with any estate assets. Typically, a bequest of a specific dollar amount is general since it can
be funded with cash or other property having a value equal to the amount of the bequest. A bequest of a
specific dollar amount that is payable from a designated fund is a demonstrative bequest. To the extent of
the designated fund, a demonstrative bequest is specific since it is the designated fund from which the
bequest is to be paid. A demonstrative bequest is general, however, to the extent the fund is insufficient to
pay the dollar amount of the bequest in full.

Explanation to Point-Two (30-35%):

The bequest to Uncle of $1,000 to be paid from Testator’s bank account at Bank is a demonstrative
legacy. Therefore, it is specific to the extent of $750 and general as to $250. Accordingly, only $250 is
subject to possible abatement as a general legacy, and then only in proportion to the other general legacies
available for the payment of debts.

In light of the definition of a demonstrative legacy, the bequest to Uncle is part specific (the $750 in the
bank account), part general ($250), for purposes of the abatement statute. The portion that is specific will
not abate if there are sufficient general legacies under the will to pay debts, expenses, and taxes that
exceed the value of the residuary estate. Therefore, the amount of the abatement depends on what the
aggregate value of all the general legacies is.

There is no doubt that the bequest to Friend of $5,000 is a general legacy.

If Friend’s bequest is the only other general legacy in addition to the $250 to Uncle, then $250/$5,250
times $5,000 (or $238) of Uncle’s bequest abates to pay the $5,000 of claims for which the residuary
estate is insufficient and $4,762 of Friend’s bequest abates as well. This would leave Uncle with $762 and
Friend with $238. Of course, Sister gets the tangibles of $ 1,000 and Brother gets $20,000 of specifically
bequeathed closely held corporate stock. This fully accounts for the $22,000 available for distribution.

If, on the other hand, the bequest of the stock (valued at $20,000) is also a general legacy, which is
unlikely, then $250/$25,250 (or $50) of Uncle’s bequest abates; $991 of Friend’s bequest abates; and
$3,959 of Brother’s bequest abates. As noted in Point Four below, this is unlikely.

Explanation to Point-Three (15-25%):

The interest of a disclaimant in either a specific, general, or demonstrative bequest passes as if the
disclaimant predeceased the testator. Unless the will otherwise provides, if the disclaimant is a person
described in the applicable anti-lapse statute, the disclaimed interest passes to the disclaimant’s surviving
issue.

Under the typical disclaimer statute, if a disclaimant files a timely disclaimer (generally within nine
months of a testator’s death), the disclaimed interest passes as if the disclaimant had predeceased the
testator. In other words, the typical disclaimer statute creates the fiction that the disclaimant predeceased
the testator.

Under the typical lapse statute, if Uncle had predeceased Testator, the bequest to Uncle does not lapse; it
passes to Uncle’s issue. For example, under UPC § 2-603, a devise to a person who is a descendant of the
testator’s grandparents does not lapse if the deceased person’s descendants survive the testator. Rather, a

Seperac-J19 Exam-Released MEE Essay Compilation © 2016-2020 1378


substituted gift is created in the deceased person’s descendants. Under this and other typical lapse statutes,
a substituted gift is created in Uncle’s child, Son.

If an examinee assumes there is no lapse statute (which would be the common law rule) the amount to
abate reduces from $5,000 to $4,000 and the entire amount would come from Friend’s general bequest.
This results from the fact that the bequest to Uncle is available to pay part of the $28,000 claim against
the estate, leaving only $4,000 to come from the non-residuary bequests.

Explanation to Point-Four (15-30%):

The bequest of stock is most likely specific.

The classification of a bequest as either general or specific may depend on the intent of the testator. The
classification of bequests of stock has proved most troublesome. If the language in the will evidences that
the testator owned the securities at the time the will was executed (such as a will reading: “I give my stock
in ABC Corporation”), courts generally conclude the gift is specific. This construction is based on the fact
that the testator was referring to securities that he or she actually owned. Absent possessive words, courts
use other factors to assist in the classification of the bequest. For example, if the stock is of a closely held
business, the gift is usually specific on the theory that an executor could not readily acquire the stock for
the named legatee on the open market, which would be the expectation if the bequest were classified as
general. Also, even if the testator did not use the possessive word “my” to describe the gift, if, when the
will was signed, the testator owned shares equal to the amount described in the will, courts generally
construe the gift as specific. On the other hand, if the testator did not then own shares at least equal to the
amount of the gift, the tendency is to construe the gift as general.

In this case, ABC stock is closely held, and, at the time the will was executed, Testator owned 100 shares.
Thus, more likely than not, the gift will be construed as specific and, therefore, will abate only if the
residuary bequest and the general bequests are insufficient to pay all claims against the estate. Here, the
general legacies are sufficient to pay the amount of debt that could not be paid from the residuary
estate/Thus, the bequest of the ABC stock to Brother should not abate at all.

Seperac-J19 Exam-Released MEE Essay Compilation © 2016-2020 1379


#326-JUL 1998–MEE Q01: QUESTION ONE (WILLS-ESTATES)

On January 2, 1995, Testator died testate in a nursing care facility called Nursing Home, Inc. Testator’s
will, signed on December 28, 1994, when she was mentally competent, makes two bequests: (1) a bequest
of $ 100,000 to “my heirs at law” and (2) a bequest of “my residuary estate to Nursing Home, where I
shall be resident at my death.” President, who is both a director and the president of Nursing Home, was
named executor. Testator’s will was drafted by Lawyer, a shareholder of Nursing Home, who also serves
as Nursing Home’s attorney. Testator’s only heir is her cousin, Charles, who last saw Testator nearly a
decade ago. T

Throughout the 1980s, Testator’s health declined. During the 1980s, while still living on her own,
Testator became close friends with President. He visited her apartment many times and arranged for
employees of Nursing Home to moonlight as nurses in her apartment. In 1992, Testator’s physical
condition deteriorated so significantly that she needed full-time care. President helped Testator make
arrangements for admission to Nursing Home. While Testator was living at Nursing Home, a number of
“estate planning seminars” were presented there by Lawyer, and a pamphlet titled “Will Information
Guide” was distributed to the residents. The pamphlet, written by President, had extensive provisions
regarding Nursing Home’s need for charitable gifts and the proper method of including bequests to
Nursing Home in a will.

On the evening of December 28,1994, after attending an estate planning seminar given by Lawyer,
Testator said she wanted to write her will. Testator, Lawyer, and President immediately went into
President’s office, where Lawyer drafted the will in full accord with Testator’s instructions and with
President present. Two visiting friends of Testator were also present in the room when the will was
signed. They will testify that after Testator declared the document to be her will and asked them to sign as
witnesses, she attempted to sign the will. However, because of a chronic tremor, Testator’s hand shook so
badly that she was unable to hold the pen despite repeated attempts. In frustration, she finally asked
President if he would write her name for her on the will, which President did. Immediately thereafter, the
two friends signed the will as witnesses.

After the will was executed, Lawyer gave the will to Testator. She later placed it in her night table drawer
for safekeeping.

After Testator’s death, a nurse found the will in the night table. She gave it to President. When he read the
will he saw that the bequest of $ 100,000 to Testator’s heirs at law had been crossed out with a straight
black line made by a felt-tip marking pen.

President petitions for the probate of Testator’s will. In the petition, President alleges that the entire estate
should pass to Nursing Home because the will was validly executed and the bequest to Charles had been
revoked.

1. On what grounds might Charles contest the will, and how likely is he to succeed on each
ground? Explain.

2. If the will is probated, is Charles entitled to the $ 100,000 bequest? Explain.

Seperac-J19 Exam-Released MEE Essay Compilation © 2016-2020 1380


#326: J98-1 MEE: ANSWER: NCBE (WILLS-ESTATES)

POINT (1) [26%] ISSUE: Did Testator execute the will with the formalities required by the statute
of wills? ANSWER: Yes. Under the statutes of wills imposing minimum formalities of execution,
the will appears to have been validly executed in that it was signed for Testator by a third party in
Testator’s presence and at Testator’s direction, and witnessed by at least two witnesses.

POINT (2) [47%] ISSUE: In light of the facts, could Testator’s will be denied probate (or have its
probate set aside) on the grounds that the Testator executed the will while under the undue
influence of others? ANSWER: No. It is unlikely under the facts presented that the court would
find that the will was procured by undue influence.

POINT (3) [26%] ISSUE: Was the $100,000 bequest revoked by cancellation as a result of the fact
that, when the will was found, the language of the bequest had been crossed out with a black felt-tip
pen? ANSWER: Unless the presumption about marks of cancellation found on a will is overcome
which may be possible under the facts), the bequest to Charles will be revoked if the jurisdiction
permits partial revocations.

ANSWER EXPLANATION:

Explanation to Point-One (20-30%):

Under the statutes of wills imposing minimum formalities of execution, the will appears to have been
validly executed in that it was signed for Testator by a third party in Testator’s presence and at Testator’s
direction, and witnessed by at least two witnesses.

The Uniform Probate Code § 2-502 requires either that the testator sign her will or that another sign the
testator’s name “in the testator’s conscious presence and by the testator’s direction.” This statute, in
common with most statutes of wills, contemplates the possibility that under certain circumstances the
testator may be physically unable to sign his or her name. While “third-party signing” statutory provisions
were historically motivated by a concern that a testator would not know how to write, the language is
sufficiently broad to validate third-party executed wills whenever the testator’s signature has been affixed
to the document by another acting at the testator’s direction and in the testator’s presence. While the UPC
has no declaration or verbal attestation request requirement, factually both occurred here and the will
would not fail for want of due execution on those grounds, if required by the governing statute.

Under the facts, Testator first attempted to sign the will on her own. When that failed because of her
shaking hand, she requested President to sign for her, which he did, in her presence. Thus the statutory
requirements appear to be fully satisfied and the will was validly executed.

Explanation to Point-Two (40-50%):

It is unlikely under the facts presented that the court would find that the will was procured by undue
influence.

Wills may be contested on a number of grounds, including undue influence. The burden to prove that a
will should be denied probate on this ground is upon the contestant.

Seperac-J19 Exam-Released MEE Essay Compilation © 2016-2020 1381


Courts scrutinize cases for undue influence when the will leaves property to persons who would not
otherwise be deemed to be natural objects of a testator’s bounty. In this case, it is possible that a court
might find Nursing Home a natural object of Testator’s bounty since she had spent much time there, was
apparently well-cared for, and was not close to her blood relatives.

To set aside a will on the grounds of undue influence, the contestant must prove that, at the time the will
was signed, the testator was susceptible to undue influence, that the procurer had motive and opportunity
to influence, and that the will was a result of the substitution of wills. As to the first requirement,
susceptibility, the contestant alleges weakened physical state; on the other hand, the facts state Testator
was mentally competent and include no facts suggesting a weakened mental state.

Motive (to secure the bequest) and opportunity (confinement in nursing home; participation in execution)
are easily demonstrated.

The most difficult element to prove is substitution of wills. If the burden of proof remains on the
contestant, he cannot merely imply it from the existence of other elements, and substitution by direct
evidence is difficult to prove. Weakened physical state is not evidence of weakened mental state; in fact,
here the Testator was mentally competent when she signed the will; furthermore, there is no evidence that
Testator’s free agency was overcome by President, so that the will can be regarded as a product of his
volition rather than hers. Here, for example, it appears that Testator initiated the draft of her new will.
Furthermore, Testator had no close relatives who had attended her that she might have preferred to
Nursing Home; she hadn’t seen Cousin in a decade.

There is the possibility of arguing that a confidential relationship existed between Testator and President,
raising a higher degree of suspicion and perhaps even a presumption of undue influence. While President
received nothing under the will, jurisdictions usually do not require personal benefit. However, courts
have held that the mere showing of a confidential relationship is insufficient to set aside a will on the
grounds of undue influence.

Explanation to Point-Three (20-30%):

Unless the presumption about marks of cancellation found on a will is overcome which may be possible
under the facts), the bequest to Charles will be revoked if the jurisdiction permits partial revocations.

Most jurisdictions recognize “partial revocation by physical act.” The straight black line through the
bequest is a “cancellation” within the meaning of the statute since it came in contact with the words of the
will. When marks of cancellation are found on a will known to last have been in the testator’s possession,
a presumption arises that such marks were made by the testator with the intent to revoke. Under this
presumption, it would appear that the bequest had been revoked by the testator. If the bequest is revoked,
then it passes as part of the residuary estate to Nursing Home, as President claims. In jurisdictions that do
not permit partial revocations, the bequest to Charles is valid because partial revocations are
impermissible.

The presumption of revocation is rebuttable. The burden to overcome this presumption is on the party
claiming that the bequest has not been revoked. Here, two facts may aid in rebutting the presumption. The
first is that while the will was in the custody of the decedent until her death (and the law, therefore,
presumes that the cancellation was made by the decedent with intent to revoke), the will was found by an
individual who, as an employee of the beneficiary, might have an interest in seeing the bequest revoked.
While this fact alone is probably not sufficient to overcome the presumption, there is also the fact that the
marks of cancellation were made by a straight black line. Testator, at the time she attempted to sign the

Seperac-J19 Exam-Released MEE Essay Compilation © 2016-2020 1382


will, had a chronic tremor and was in such weakened physical condition that she was incapable of holding
a pen. Given that condition, it is unlikely that her condition had improved between December 28 when the
will was signed and the following January 2 when she died. There is, accordingly, good reason to believe
that she did not make the marks on the will. In light of this, a jury could conclude that the presumption
had been overcome and that the bequest to Charles was not revoked.

If the state does not permit partial revocations, the bequest to Charles is not revoked.

Seperac-J19 Exam-Released MEE Essay Compilation © 2016-2020 1383


#327-FEB 1998–MEE Q03: QUESTION THREE (WILLS-ESTATES/CONFLICTS)

Child was the biological daughter of Dad and Mom. When Child was an infant, Husband and Wife legally
adopted her in State A with the consent of Dad and Mom. All the parties then lived in State A.

Three years after Child was adopted, Husband died. Thereafter, Wife married New Husband. At the time
of this marriage, Child was four years old. Five years later, New Husband died intestate, domiciled in
State A. He was survived by Wife, Child, and his closest blood relative, Uncle. Although New Husband
never adopted Child, Child resided with Wife and New Husband until New Husband’s death. New
Husband raised Child and treated Child in all respects as if she were his biological child.

After New Husband’s death, Wife moved to State B and established her domicile there. She died intestate
in State B, survived by Child and by Grandmother, Wife’s closest living blood relative. At the time of
Wife’s death, she owned property in State B and a summer vacation home in State A.

Grandmother, who was a domiciliary of State B, had a close relationship with Wife and Child. One week
after Wife died, Grandmother died testate. Her valid will, executed two years earlier, bequeathed her
entire estate to Wife, or “if Wife predeceases me, to such of Wife’s children who survive me.”

The laws of intestate succession in States A and B provide that one-half of an intestate decedent’s
property is distributable to the decedent’s spouse and the other half to the decedent’s issue per stirpes, or
absent such issue, to the decedent’s ancestors or, if none, to the decedent’s collateral blood relatives.

The law of State A provides that an adopted child cannot inherit from the biological parents but can
inherit from the adopting parents as if the adopted child were a biological child of the adopting parents.

The law of State B provides that an adopted child cannot inherit from an adopting parent who died
intestate. However, State B law concerning the construction of wills provides, “If an adopting parent
leaves a testamentary class gift to his or her children, adopted children are included in the class gift.”
There are no other governing state statutes.

1. Is Child entitled to inherit from New Husband’s estate? Explain.

2. Is Child entitled to inherit from Wife’s estate? Explain.

3. Is Child entitled to inherit from Grandmother’s estate? Explain.

Seperac-J19 Exam-Released MEE Essay Compilation © 2016-2020 1384


#327: F98-3 MEE: ANSWER: NCBE (WILLS-ESTATES/CONFLICTS)

POINT (1) [26%] ISSUE: Is Child entitled to take one-half of New Husband’s estate, as New
Husband’s issue, on the ground that Child had been equitably adopted by New Husband?
ANSWER: Yes. Child is entitled to one-half of New Husband’s estate only if State A recognizes the
doctrine of equitable adoption and facts can be established to prove there had been an equitable
adoption. Among other things, to find an equitable adoption, a court might require that New
Husband had promised to adopt Child and, if so. Child cannot take from New Husband’s estate. If
State does not recognize doctrine or Child cannot prove the required elements, then Child cannot
inherit from New Husband’s estate.

POINT (2) [37%] ISSUE: Is Child entitled to Wife’s estate, located in State A or State B?
ANSWER: Child is not entitled to inherit Wife’s property in State B but is entitled to inherit Wife’s
real property in State A. State B law controls inheritance rights for all of Wife’s property other
than the real property located in State A because State B was the state of Wife’s domicile at the
time of her death. State A law, however, controls the disposition of the State A real property.

POINT (3) [37%] ISSUE: Is Child entitled to Grandmother’s entire estate as the alternate
beneficiary named in Grandmother’s will? ANSWER: Child is not entitled to take Grandmother’s
estate as the alternate beneficiary under her will under the State B class gifts statute. However,
there are good arguments that under generally applicable rules for construing wills, Child should
take under Grandmother’s will because that is what she intended.

ANSWER EXPLANATION:

Explanation to Point-One (20-30%):

Child is entitled to one-half of New Husband’s estate only if State A recognizes the doctrine of equitable
adoption and facts can be established to prove there had been an equitable adoption. Among other things,
to find an equitable adoption, a court might require that New Husband had promised to adopt Child and, if
so. Child cannot take from New Husband’s estate. If State does not recognize doctrine or Child cannot
prove the required elements, then Child cannot inherit from New Husband’s estate.

Under the law of State A, an adopted child is entitled to inherit from the estate of the deceased adopting
parent. Thus, if Child had been legally adopted by New Husband, Child would have been entitled to
receive one-half of New Husband’s estate. (The other half, according to the governing statutes, would
have passed to Wife as New Husband’s surviving spouse.)

In fact, Child was not adopted by New Husband. Thus, whether Child can inherit from New Husband
depends on whether State A recognizes the doctrine of “adoption by estoppel” or “equitable adoption.”

If State A does not recognize the doctrine of adoption by estoppel, Child cannot inherit. Many states,
however, recognize the doctrine of adoption by estoppel. Where recognized, an equitably adopted child is
entitled to inherit from the “adopting parent” in the same manner as a child who had been legally adopted
through the appropriate statutory procedures. While there are variations among the states, most states
require the child claiming to have been equitably adopted to prove by clear and convincing evidence that
“he has stood from an age of tender years in a position exactly equivalent to a formally adopted child.”
The requirement is that the “adopting parent” have treated the “adopted child” in the same manner as a

Seperac-J19 Exam-Released MEE Essay Compilation © 2016-2020 1385


parent generally would treat a child. Relevant factors establishing the relationship might include: (1) the
“adopting parent’s” bestowal of love and affection on the child, (2) the “adopting parent’s” performance
of parental duties toward the child, (3) the child’s obedience to and companionship toward the “adopting
parent,” (4) the child’s reliance on an adoptive relationship, and (5) the “adoptive parent’s” holding out
the child as his child. Many courts also require the child to prove that the “adopting parent” had promised,
agreed, or attempted to adopt the child formally.

The facts state that New Husband raised Child and in all respects treated Child as if Child were his
biological child. This statement might be construed to suggest that the five elements mentioned above
were satisfied. However, even if that were the case, in many states there must also have been a promise or
agreement to adopt the child and no facts suggest such a promise was made here. If a promise or
agreement were required, Child could not claim an equitable adoption. If a promise or agreement is not
required an equitable adoption might be formed. Absent an equitable adoption, New Husband’s estate
would pass to Wife and Uncle (as New Husband’s closest living blood relative).

In a jurisdiction recognizing the doctrine of equitable adoption, if Child could prove sufficient facts to
establish a basis for invoking the doctrine, New Husband’s estate would be distributed one-half to Wife
and one-half to Child.

Explanation to Point-Two (30-40%):

Child is not entitled to inherit Wife’s property in State B but is entitled to inherit Wife’s real property in
State A. State B law controls inheritance rights for all of Wife’s property other than the real property
located in State A because State B was the state of Wife’s domicile at the time of her death. State A law,
however, controls the disposition of the State A real property.

Whether Child is entitled to inherit as an heir of Wife depends upon whether the law of State A
(permitting inheritance), where Child was adopted and certain real property is located, or the law of State
B (not permitting inheritance), where Wife was domiciled at the time of death, apply. According to the
Second Restatement of Conflicts § 262, whether an adopted child is entitled to inherit the property of an
intestate depends upon the law of the intestate’s domicile at the time of death. Thus, the comments
provide that “if the law that would be applied by the courts of the state where the decedent was domiciled
at the time of his death does not allow an adopted child to take a distributive share, he will not do so
although the law governing the adoption would allow him to take.” This rule might be justified on the
ground that the state of the intestate’s domicile at the time of intestate’s death has the most significant
contacts with respect to this issue as to how the intestate’s estate should be administered as well as the
most significant interest in determining which children in the state should have claims against a
decedent’s property. Furthermore, estate administration is eased if only the laws of one jurisdiction
control for purposes of determining the distribution of the decedent’s personal property.

However, this rule is inapplicable to real property located outside of the state of the intestate’s domicile.
Under the Second Restatement of Conflicts § 263, the law governing the disposition of real property from
an intestate is the law of the situs.

Thus, Child is entitled to inherit the State A real property because State A permits an adopted child to
inherit from his or her adopting parent. On the other hand, Child is not entitled to inherit Wife’s State B
property.

Explanation to Point-Three (30-40%):

Seperac-J19 Exam-Released MEE Essay Compilation © 2016-2020 1386


Child is not entitled to take Grandmother’s estate as the alternate beneficiary under her will under the
State B class gifts statute. However, there are good arguments that under generally applicable rules for
construing wills, Child should take under Grandmother’s will because that is what she intended.

Whether Child is entitled to take under Grandmother’s will also depends on the law of State B because
she died a domiciliary of State B. State B law provides that an adopted child cannot inherit from an
adopting parent who dies intestate. It also provides that “if an adopting parent leaves a testamentary class
gift to his or her children, adopted children are included in the class gift. Under this statute, if Wife had
bequeathed property to her “children,” Child would have been entitled to share in that class gift because
the bequest would have been made by the adopting parent. The statute, however, does not provide similar
treatment for class gifts to grandchildren. Thus, Child could not rely on this statute to argue that she is
included in the class gift to children.

On the other hand, Child may be included in the class gift under Grandmother’s will under general
common-law principles. The governing principle is that in construing a testator’s will, courts should,
whenever possible, give effect to a testator’s intent. Under this “intent principle,” there is a strong
argument that Grandmother intended Child to be included in the testamentary class gift. This conclusion
is based upon the fact that Grandmother’s will was signed two years before she died and at a time when
she knew that Wife had only one child and was then a widow with little or no expectation of having more
children. Furthermore, Grandmother was close to the child. Although Grandmother was a so-called
“stranger to the adoption,” most courts today would include an adopted child within a class gift under the
facts of this problem even though no state statute specifically so provides.

Seperac-J19 Exam-Released MEE Essay Compilation © 2016-2020 1387


#328-JUL 1997–MEE Q07: QUESTION SEVEN (WILLS-ESTATES)

Decedent recently died, owning a bank account at First Bank registered in the name of “Decedent in trust
for Grandchild.” At her death, the balance in this account was $15,000. Decedent had made all the
deposits into this account.

Decedent was also a co-owner of an account at Second Bank, registered in the name of “Decedent and
Son, as joint tenants with right of survivorship.” Only Decedent had made deposits into this account, It
had is balance of 520,000 at her death. Decedent had opened this account three year; before she died, near
the time that she sold the family home and moved into Son’s home rather than a nursing home. At the
time Decedent opened this account, the Second Bank bank officer told her that “a joint account is an ideal
way to let Son draw on the account in case you need to reach the money you have put into it.”

Three weeks before she-died, Decedent typed a Letter to Friend, who lived in another state. The letter
said: “I haven’t been feeling well lately. I give you my diamond ring to remember me by. It’s yours, but
Son would be upset if he learned I had given it to you, and so I’ll hold onto it for you. When I die, bring
this letter to him and get it.” Decedent signed and immediately mailed the letter to Friend, who received it
the next day. The ring, which was found among Decedent’s personal effects, was valued at $5,000.

Decedent died indebted and with only the foregoing assets. Her creditors claim that the two bank accounts
and the ring must be used to satisfy their claims against her estate.

Are the two bank accounts and diamond ring available for payment of the creditors’ claims? Explain.

Seperac-J19 Exam-Released MEE Essay Compilation © 2016-2020 1388


#328: J97-7 MEE: ANSWER: NCBE (WILLS-ESTATES)

POINT (1) [23%] ISSUE: Do the funds on deposit at First Bank in the name of “Decedent in trust
for Grandchild” pass outside of the probate estate to Grandchild, or are they assets of Decedent’s
estate subject to the claims of Decedent’s creditors? ANSWER: Decedent’s creditors can reach
First Bank “trust account” assets if the assets in her probate estate are insufficient to pay the claims
against her estate.

POINT (2) [32%] ISSUE: Can Decedent’s creditors reach the money in the joint bank account on
the theory that the joint bank account was opened merely as a convenience to Decedent? ANSWER:
Yes. Under the Uniform Probate Code the creditors of Decedent’s estate can reach the money in the
joint account if the other assets of the estate are insufficient to satisfy their claims. At common law
the creditors cannot reach the joint account assets unless the joint account is opened for the
depositor’s convenience and no survivorship feature was intended.

POINT (3) [45%] ISSUE: Can Decedent’s creditors reach the diamond ring on the theory that
Decedent did not effectively give it to Friend while Decedent was alive? ANSWER: No. The gift of
the ring to Friend is valid because Decedent intended to make the gift, there was an appropriate
delivery under the circumstances, and Friend is presumed to have accepted the gift. Therefore, the
creditors of Decedent’s estate cannot reach the ring.

ANSWER EXPLANATION:

Explanation to Point-One (20-30%):

Decedent’s creditors can reach First Bank “trust account” assets if the assets in her probate estate are
insufficient to pay the claims against her estate.

A person may deposit funds in a bank in the form “A in trust for B” and create a so-called tentative trust
account, bank account trust, or “Totten” trust (so named for the famous New York case that upheld the
account against a challenge that it violated the policies underlying the statute of wills). These accounts are
also recognized by statutes in many states.

The depositor is the sole owner of the account throughout his lifetime, absent clear and convincing
evidence of a different intent. Upon the owner’s death, the remaining sums on deposit belong to the
designated beneficiary. However, courts and statutes provide that if the depositor’s probate estate is
insufficient to pay claims against the estate, then the money on deposit in the trust account is available for
the payment of claims. The right of creditors to reach the trust assets is justified on the ground that up
until the moment of Decedent’s death, Decedent had the unrestricted right to the use and enjoyment of the
funds in the same manner as Decedent had with respect to other assets that would have been included in
the Decedent’s probate estate.

Explanation to Point-Two (30-40%):

Under the Uniform Probate Code the creditors of Decedent’s estate can reach the money in the joint
account if the other assets of the estate are insufficient to satisfy their claims. At common law the
creditors cannot reach the joint account assets unless the joint account is opened for the depositor’s
convenience and no survivorship feature was intended.

Seperac-J19 Exam-Released MEE Essay Compilation © 2016-2020 1389


Under the Uniform Probate Code if a decedent’s probate estate is insufficient to pay the claims of
creditors, the creditors’ claim to funds in a joint account is superior to the rights of the surviving joint
tenant to the extent that the decedent deposited money into the joint bank account.

Absent such a statute, the surviving joint tenant of a joint bank account (e.g. “A and B as joint tenants
with right of survivorship”) ordinarily is entitled to the money in the account at the death of the first joint
tenant, and the deceased depositor’s creditors have no claim to the money. In some states the survivorship
feature is conclusive. In others the survivorship feature can be avoided if fraud, duress, undue influence or
mental incapacity is shown. In these states and under the facts, Son is entitled to the account balance to
the exclusion of Decedent’s creditors.

Some courts have held that no right of survivorship is created if the evidence shows that the person who
created the joint bank account did not intend the other person(s) named on the account to have a right of
survivorship. The intent of the person creating the joint bank account is not self-evident from the form of
registration “A and B as joint tenants with right of survivorship.” The depositor may have intended that
during their joint lives either was empowered to withdraw funds from the account with the survivor being
entitled to the balance. This might be called a “true joint tenancy account.” Alternatively, the depositor
may have intended only that the survivor be entitled to the funds in the account at the depositor’s death,
but that the non-depositor have no right of withdrawal during the depositor’s lifetime. This would be a
pay-on-death account. Then again, the depositor may have intended that the non-depositor have the right
to withdraw funds during the depositor’s lifetime, but not be entitled to the funds on deposit at the
depositor’s death. This intent suggests the account was intended to be a mere agency account. If an
agency account was intended, then a court could hold that the survivor is not entitled to the funds in the
account at the depositor’s death. Only on this construction of the depositor’s intent would the funds in the
account be an asset of the decedent’s estate, subject to the claims of creditors.

Under the facts of the problem, it appears that at minimum the account was opened for Decedent’s
convenience and as a device to enable Son to reach the deposited funds. What is unclear, however, is
whether Decedent intended Son to take the funds remaining in the account at Decedent’s death as the
“survivor.” It might be inferred that Decedent had no such intent, based on the fact that the bank officer
suggested the account as a vehicle to enable Decedent’s child to obtain access to Decedent’s money in
case of Decedent’s need. On the other hand, it is likely that Decedent would want Son to have these funds
as a “reward” for taking care of Decedent in old age, perhaps even as a form of compensation for Son’s
time and expense in taking care of Decedent and assuring that Decedent did not have to go to a nursing
home. Since there are no direct statements from Decedent that the survivorship feature was not intended,
it is unlikely under either a preponderance of the evidence standard or a clear and convincing standard
that Decedent’s creditors will be able to defeat the survivorship feature.

Explanation to Point-Three (45-55%):

The gift of the ring to Friend is valid because Decedent intended to make the gift, there was an
appropriate delivery under the circumstances, and Friend is presumed to have accepted the gift. Therefore,
the creditors of Decedent’s estate cannot reach the ring.

Whether the ring is subject to the claims against the estate depends on whether it was in Decedent’s estate
when she died or had been given away by Decedent before her death. An inter vivos gift requires intent,
delivery and acceptance. Acceptance is generally presumed, particularly when the subject matter of the
gift is something of value. Thus in most cases, the issue is whether the donor intended to vest the donee
with an interest in property immediately or only at some time in the future. If the latter is true, the donor

Seperac-J19 Exam-Released MEE Essay Compilation © 2016-2020 1390


has not made a gift. If the former is true, the gift was completed only if the delivery was appropriate under
the facts and circumstances.

Cases in which the transfer of physical possession of the subject matter of the gift is deferred to some time
in the future present a difficult case. The deferral of the transfer until the future might suggest the absence
of an intent to make an immediate gift, or it might suggest an intent to make an immediate gift of a “future
interest” in the property because the donor wants to retain possession of the item for life. The common
law recognizes both present and future interests in property as distinct property interests. Thus, there is no
reason why one cannot make an immediate gift of a future interest. In Gruen v. Gruen, the court held that
a donor had made a gift to his son of a future interest in a painting where the donor wished to defer
transferring possession of the painting to his son so that the donor could retain possession of the painting
for the rest of his life.

The facts, particularly the language in the signed typed letter which Decedent sent to Friend, indicate that
Decedent intended to make a present gift of either a present or a future interest in the ring. If the court
finds that Decedent intended to make a present gift to Friend, the next question is whether delivery was
appropriate under the circumstances.

If a court views the ring itself as the subject of the gift, delivery of the letter would likely be considered
appropriate under the circumstances, since the donee was not physically present for the donor to hand the
ring to at the time the donor intended to make the gift. Courts have generally recognized that constructive
or symbolic delivery of a gift is appropriate if the subject matter of the gift cannot be physically delivered
(to the donee) at the time the donor wishes to complete the gift.

If the subject matter of the gift is a future interest in a ring rather than the ring itself, then the gift can only
be evidenced by a constructive or symbolic delivery, since the subject matter of the gift has no physical
shape. In such case the best delivery would be a writing that is signed and delivered to the donee, as was
done here.

Under either interpretation, Decedent made a present gift to Friend of a future interest in the ring, and
Decedent’s creditors have no claim to it. On the other hand, if the court finds that delivery was
insufficient, the ring would be an asset of Decedent’s estate subject to the creditors’ claims.

Decedent’s creditors might argue that this was a not an inter vivos gift but a gift causa mortis. The latter is
a gift made under an immediate apprehension of death, usually from a known peril, which is revocable if
the donor does not die. Like an inter vivos gift, a gift causa mortis must be evidenced by intent, delivery
and acceptance. Some courts appear to require a greater degree of proof of intent, delivery and acceptance
in the case of a causa mortis gift because of a concern that such gifts are inconsistent with policies
underlying statutes of will in that they are often unwitnessed transfers. On the facts, it is not clear whether
the decedent wrote the letter under the immediate apprehension of death. Thus, it is unclear whether the
gift would be characterized as a gift causa mortis. If the court treated this gift as a gift causa mortis but
regarded it with suspicion, it might find that delivery was insufficient. In this case, the ring would be an
asset of Decedent’s estate, subject to the creditors’ claims.

Seperac-J19 Exam-Released MEE Essay Compilation © 2016-2020 1391


#329-FEB 1997–MEE Q01: QUESTION ONE (WILLS-ESTATES)

Husband’s valid will provides:

I, Husband, leave my entire estate to Wife, or, if she predeceases me, I leave my entire estate in equal
shares to my son, Son, and my daughter, Daughter.

Wife’s valid will bequeaths her entire estate to Husband. It does not provide for an alternate gift if
Husband predeceases her.

Husband and Wife recently died in a fire in their home. The fire was set by Son in his second-floor
bedroom at 2:00 a.m. Wife’s body was found in her nightclothes in the second-floor bedroom she shared
with Husband. Husband’s body was found in his nightclothes at the base of the stairs on the first floor of
their home with his head facing the front door, his feet toward the upstairs landing, and red marks on his
stomach indicating that he had been crawling down the stairs when he had been overcome by smoke.

Husband and Wife were survived by Son and Daughter and by Son’s daughter, Granddaughter.

Son claims the fire was set inadvertently when a cigarette he was smoking in bed dropped to the mattress,
causing it to ignite. The fire marshal has found traces of a combustible substance on the premises,
suggesting the possibility that the fire may have been set intentionally. The local prosecutors, however,
have declined to seek an indictment because, although the available evidence indicates that it is more
likely than not that Son intentionally set the fire, the evidence is insufficient to support a criminal
conviction.

The intestate succession statute of this jurisdiction provides that a decedent’s surviving spouse is his or
her sole heir. If there is no surviving spouse, the decedent’s issue take the estate per stirpes. This
jurisdiction also has enacted the Uniform Simultaneous Death Act.

1. To whom should Husband’s estate be distributed? Explain.

2. To whom should Wife’s estate be distributed? Explain.

Seperac-J19 Exam-Released MEE Essay Compilation © 2016-2020 1392


#329: F97-1 MEE: ANSWER: NCBE (WILLS-ESTATES)

POINT (1) [40%] ISSUE: Did Husband and Wife die simultaneously, or is the evidence sufficient to
establish that Husband survived Wife? ANSWER: No. Under the original Uniform Simultaneous
Death Act Husband and Wife would probably not be treated as having died simultaneously.
Instead, Husband would probably be treated as having survived Wife. However, under the revised
version of the Act and the Uniform Probate Code, Husband would be treated as having predeceased
Wife because he did not survive her by 120 hours.

POINT (2) [40%] ISSUE: Is Son barred from inheriting from his parents because he caused the fire
that resulted in their death? ANSWER: Whether Son is barred from inheriting from his parents
because he caused the fire that resulted in their deaths depends upon whether the probate court
would conclude that he feloniously and intentionally took the lives of his parents. If so, he would be
barred from inheriting. If not, he would be entitled to claim as heir or devisee from his parents.

POINT (3) [20%] ISSUE: If Son is barred from inheriting from his parents because he feloniously
and intentionally took the lives of his parents, to whom should the shares that he would have
received from their estates pass? ANSWER: If Son is barred from inheriting from Wife, her estate
passes to her heirs – Granddaughter and Daughter. If Son is barred from inheriting from Husband,
his one-half share passes to Granddaughter.

ANSWER DISCUSSION:

The distribution of the estates of Husband and Wife depends on whether each is treated as having
survived the other, on whether Son is barred from inheriting from his parents because he set the fire that
killed them, and, depending on the resolution of these issues, on whether the share of a residuary legatee
who is disqualified from taking passes to the other residuary legatees.

ANSWER EXPLANATION:

Explanation to Point-One (35-45%):

Under the original Uniform Simultaneous Death Act Husband and Wife would probably not be treated as
having died simultaneously. Instead, Husband would probably be treated as having survived Wife.
However, under the revised version of the Act and the Uniform Probate Code, Husband would be treated
as having predeceased Wife because he did not survive her by 120 hours.

The original version of the Uniform Simultaneous Death Act provides that:

Where the title to property or the devolution thereof depends upon priority of death and there is no
sufficient evidence that the persons have died otherwise than simultaneously, the property of each person
shall be disposed of as if he had survived, (emphasis added)

The Act was revised in 1991 to provide that, even if there is sufficient evidence to establish that a
beneficiary named in the will in fact survived the testator, a beneficiary is only treated as having survived
the testator if there is clear and convincing evidence that he or she survived the testator by 120 hours. The
Uniform Probate Code also treats a beneficiary who does not survive the testator by at least 120 hours as
having predeceased the testator.

Seperac-J19 Exam-Released MEE Essay Compilation © 2016-2020 1393


Under the original Act, a jury might conclude that there was sufficient evidence to establish that Husband
in fact survived Wife, even though both died in the fire. Evidence supporting that conclusion includes: (1)
the fire was started on the second floor of the home; (2) the spouses were apparently in bed when the fire
occurred, based upon the time of the fire and the fact that both spouses were in their bedclothes; and (3)
Wife was found in the second-floor bedroom while Husband was found on the first floor at the bottom of
the stairs, suggesting that both of them attempted to exit the room but that Wife was overcome by smoke
before she could reach the stairs. This implies that she died before she could get out of the bedroom
whereas he was able to get out of the bedroom and crawl down to the first floor, as evidenced by the
position of his body and the marks on his body.

Of course, if the revised Act or the Uniform Probate Code applied, Husband would be treated as having
predeceased Wife. Although he may have survived her for some brief period of time, he did not survive
her by 120 hours (i.e., 5 days).

Because Wife did not in fact survive Husband, for purposes of distributing Husband’s estate she would
not be treated as having survived him under either version of the simultaneous death act. Therefore,
Husband’s estate passes in equal shares to Son and Daughter, as provided in Husband’s will, unless Son is
prevented from inheriting.

If Husband is treated as having survived Wife for purposes of distributing Wife’s estate, the estate passes
to Husband under her will, and it then passes to the successors of Husband’s estate. On the other hand, if
Husband is treated as not having survived Wife, Wife’s estate passes by intestate succession to her heirs
because her will did not provide who would take her estate if Husband predeceased her, and antilapse
statutes generally do not save gifts to the spouse who dies before the testator. Wife’s heirs would be her
children, Daughter and Son, unless, as discussed in Point Two, Son is barred from inheriting.

Explanation to Point-Two (35-45%):

Whether Son is barred from inheriting from his parents because he caused the fire that resulted in their
deaths depends upon whether the probate court would conclude that he feloniously and intentionally took
the lives of his parents. If so, he would be barred from inheriting. If not, he would be entitled to claim as
heir or devisee from his parents.

Under the Uniform Probate Code and most modern cases, a person who feloniously and intentionally kills
a decedent is barred from claiming a share of the decedent’ s estate as either heir or beneficiary under the
decedent’s will. In a few states only murderers are disqualified, and in a few states all felonious killers are
disqualified. Generally, the decedent’s estate is then disposed of as if the killer had predeceased the
decedent.

A conviction of felonious and intentional homicide conclusively establishes that the killer feloniously and
intentionally killed the decedent, for purposes of the probate proceedings. In some states inheritance is
barred only if there has been a conviction. In most states, if there has been no conviction, the court having
jurisdiction over the decedent’s estate must determine whether a preponderance of evidence supports the
conclusion that the alleged killer feloniously and intentionally took the life of the decedent. Some states,
however, use a clear and convincing standard, rather than a preponderance of the evidence standard.

In some states where there is no slayer statute, courts apply the equitable principle that a wrongdoer
cannot profit from his own wrong to prevent the killer from inheriting. In such states, the court is likely to
hold that the killer succeeds to the property of the deceased victim but holds it as constructive trustee for
others. The facts say that there is sufficient evidence to establish by a preponderance of the evidence that

Seperac-J19 Exam-Released MEE Essay Compilation © 2016-2020 1394


Son intentionally set the fire. If the jurisdiction uses this standard of proof, Son would be barred from
taking his parents’ estates, and they would be distributed as discussed in Point Three.

If Son is not barred from inheriting from his parents, their estates pass to him and Daughter as described
at the end of Point One.

Explanation to Point-Three (15-25%):

If Son is barred from inheriting from Wife, her estate passes to her heirs – Granddaughter and Daughter.
If Son is barred from inheriting from Husband, his one-half share passes to Granddaughter.

Wife’s estate: As noted above, if Husband is treated as having survived Wife, he takes her entire estate
under the terms of her last will, and it will be distributed to those entitled to take his estate. If he is treated
as having predeceased her, her estate passes to her heirs, Son and Daughter, unless Son is barred from
inheriting from Wife. The bequest to Husband is not saved by a lapse statute because those statutes do not
apply to bequests to spouses. If Son is barred, he is treated as having predeceased Wife. Under intestate
succession statutes, Son’s child Granddaughter takes by right of representation the one-half share Son
would have taken.

Husband’s estate: As noted above, if Husband is treated as having survived Wife, his estate passes to the
beneficiaries named in his will, Son and Daughter, unless Son is barred from inheriting Husband’s estate.
If Son is barred, the question arises whether Son’s one-half share passes to Granddaughter, Son’s child, or
to Daughter, the other beneficiary named in Husband’s will. As noted above, when a person is barred
from inheriting because he feloniously and intentionally killed another, the victim’s estate passes as if the
killer predeceased the victim.

The argument for Granddaughter is that Husband’s will did not expressly condition the gift to Son and
Daughter on survivorship and that his will did not provide that the share of either who predeceased him
should pass to the survivor. Furthermore, under the typical state antilapse statute, the share of any
deceased beneficiary, such as Son, passes to that beneficiary’s surviving issue unless the will otherwise
provides. Under this analysis, Son’s one-half share passes to Granddaughter.

Daughter, however, might argue that Son’s one-half share passes to her under the “residue of a residue”
rule. Under this rule, a residuary bequest that fails to take effect ordinarily passes to the surviving
residuary legatee. If the analysis ends there, Daughter takes all of Husband’s estate. However, under the
Uniform Probate Code, Granddaughter would take the share that Son would have taken, because Uniform
Probate Code § 2-604 provides that the antilapse statute prevails over the rule regarding multiple
residuary legatees.

Seperac-J19 Exam-Released MEE Essay Compilation © 2016-2020 1395


#330-JUL 1996–MEE Q03: QUESTION THREE (WILLS-ESTATES)

Following extensive conversations and negotiations, Husband and Wife agreed to the terms of reciprocal
wills under which each left his or her entire estate to the survivor and the survivor left the survivor's entire
estate one-half to Husband's family and one-half to Wife's family. Each of their wills, dated September
10, 1994, was properly executed. Each will contained the following clause: "This will and the will of my
spouse each were executed after extensive conversations and negotiations."

On May 10, 1995, Husband wrote a valid holographic will that reads:

Notwithstanding the provisions of my will dated September 10, 1994, I leave $50,000 to Wife; my stamp
collection, which is located in my safe deposit box, to those friends of mine listed on a sheet of paper that
is attached to the cover of the collection; and the balance of my estate to Sister.

Husband placed the May instrument in a desk drawer in his home office. Wife inadvertently discovered
the instrument and confronted Husband about it. In the course of a heated argument, Husband told Wife
he had changed his will because he did not like her family. One month later, Husband had a fatal heart
attack.

Husband's estate consists only of $400,000 in cash and the stamp collection. Wife, Sister, and Friend,
whose name is the only name that appears on a sheet of paper attached to the cover of Husband's stamp
collection in his safe deposit box, dispute how the estate should be distributed.

Under state law, holographic wills are valid.

How should Husband's estate be distributed? Explain.

Seperac-J19 Exam-Released MEE Essay Compilation © 2016-2020 1396


#330: J96-3 MEE: ANSWER: NCBE (WILLS-ESTATES)

POINT (1) [42%] ISSUE: Can the separate September wills of Husband and Wife be construed as
contractual wills? ANSWER: No. The September wills probably cannot be construed as contractual
whether or not the jurisdiction permits proof of the contractual agreement by extrinsic evidence or
requires that evidence of the contractual intent appear on the face of the will.

POINT (2) [16%] ISSUE: If the September wills can be construed as contractual, were they
revoked by Husband's execution of the May holographic will which was discovered by Wife?
ANSWER: Yes. If two testators have executed contractual wills, either may revoke the will during
their joint lifetimes if notice is given to the other of the intent to revoke.

POINT (3) [32%] ISSUE: If the May holographic will revokes the September will, is the bequest to
Friend of the stamp collection valid under the doctrine of incorporation by reference? ANSWER: If
the holographic will revokes the September will, the bequest to Friend of the stamp collection is
likely invalid and cannot be saved by the doctrine of incorporation by reference since the evidence
is weak that the referred to sheet was in existence when the will as signed. On the other hand, if the
jurisdiction has enacted a "list" statute, the gift to Friend may be valid.

POINT (4) [11%] ISSUE: If the May holographic will revokes the September will, is Wife entitled
to claim a forced share of Husband's estate? ANSWER: Yes. If the holographic will revokes the
September will, Wife likely is entitled to a forced share of Husband's estate.

ANSWER EXPLANATION:

Explanation to Point-One (35-45%):

The September wills probably cannot be construed as contractual whether or not the jurisdiction permits
proof of the contractual agreement by extrinsic evidence or requires that evidence of the contractual intent
appear on the face of the will.

Contractual (sometimes called “mutual”) wills have been upheld if the evidence is clear and convincing
that a contractual will was intended and the terms of the contract are readily ascertainable. The burden of
proof is on the party alleging that a contractual will exists.

Concern that a will might be falsely alleged to be contractual based upon extrinsic evidence introduced
after a testator's death when testator is no longer in a position to respond has led many legislatures to
adopt rules such as Uniform Probate Code § 2-514:

A contract to make a will or devise, or not to revoke a will or devise may be established only by (i)
provisions of a will stating material provisions of the contract, (ii) an express reference in a will to a
contract and extrinsic evidence proving the terms of the contract, or (iii) a writing signed by the decedent
evidencing the contract. The execution of mutual wills does not create a presumption of a contract not to
revoke the will or wills.

The facts state that Husband and Wife both negotiated and agreed to the terms of their wills and that their
wills contained the following clause: “This will and the will of my spouse each were executed after
extensive conversations and negotiations.” This language does not expressly state that the parties entered

Seperac-J19 Exam-Released MEE Essay Compilation © 2016-2020 1397


into a contract. Rather, the language merely states that the terms of their separate (albeit reciprocal) wills
were executed after “extensive conversations and negotiations.” Arguably, this language is insufficient to
establish that such conversations and negotiations led to the formation of a contract. Under UPC § 2-514,
for example, the words in the will appear to fail to establish the necessary intent since they do not refer to
a “contract.” It can be argued that there is sufficient extrinsic evidence to support the conclusion that the
parties intended to execute contractual wills. However, no extrinsic evidence appears with respect to the
parties' understanding regarding the revocability of their contract during the joint lives of the testators,
and the wills are silent on that point. Thus, the requirements of the statute requiring proof of a contract not
to revoke the will are not met.

Since the burden of proof is upon the party alleging the existence of a contract (Wife), the preceding
arguments suggest that Wife cannot meet that burden.

If a court were to conclude that the parties had entered into a contractual will, the question would arise
whether the contract was revocable.

Explanation to Point-Two (10-20%):

If two testators have executed contractual wills, either may revoke the will during their joint lifetimes if
notice is given to the other of the intent to revoke.

A party to a contractual will may revoke it if notice of an intent to revoke is given to the other party.
Revocation is permitted with notice on the theory that once notice is given there can be no detrimental
reliance since the party to whom notice has been given can change his or her will. When the testator who
desires to change the contractual will survives, the contract is deemed revoked only if the first to die had
been notified prior to death of the other's intent to revoke the contract. On the other hand, when the
testator who desires to change the contractual will is the first to die, notice to the other testator is provided
by the terms of the will of the first testator to die which presumably reflects a disposition inconsistent
with the terms of the contract. The UPC has no provision dealing with the revocation of a contractual will.

Where the first testator to die is the one who revoked the will, the other testator's remedy only includes
the right to change his or her will. He or she is no longer bound by the contract. The surviving testator is
not entitled to damages, nor to impose a constructive trust on the successors to the estate of the first
testator to die.

If Husband and Wife executed contractual wills, the issue arises whether the notice of revocation was
sufficient. Wife received notice by a variety of means. It appears that Wife initially received notice when
she inadvertently found the will in her husband's home office rather than by a direct communication from
Husband. It is also clear from the conversation she had with him prior to his death that he did intend to
revoke his 1994 will and that his intent to do so was orally communicated to her. Furthermore, she had at
least one month before he died to change her will. Lastly, of course, Wife received notice of Husband's
intent to revoke from the terms of the holographic will that was probated. Thus, even if the 1994 wills
were construed as initially contractual, the contract was revoked by Husband's revocation of his 1994 will,
and his communication of an intent to revoke that will to Wife.

If the execution of the holographic will and Wife's knowledge thereof were not deemed sufficient to
revoke the contractual will (if the September will were so construed), Wife would be entitled to Husband's
entire estate.

Explanation to Point-Three (25-35%):

Seperac-J19 Exam-Released MEE Essay Compilation © 2016-2020 1398


If the holographic will revokes the September will, the bequest to Friend of the stamp collection is likely
invalid and cannot be saved by the doctrine of incorporation by reference since the evidence is weak that
the referred to sheet was in existence when the will as signed. On the other hand, if the jurisdiction has
enacted a "list" statute, the gift to Friend may be valid.

If Husband's 1994 will was not contractual (or if it was but under state law contractual wills are revocable
and that will was properly revoked) the issue arises concerning the dispositive effect of the valid
holographic will. This document is a later executed will and not a codicil because it purports to dispose of
Husband's entire estate. The facts state this was a valid holographic will and that holographs are valid in
the state. (Thus, discussion of validity is non-germane). As a later valid executed will, it revokes prior
wills under all revocation statutes.

Under the doctrine of incorporation by reference, a will may incorporate into the will another document
that is extrinsic to the will. The dispositive terms in that extrinsic document may be given testamentary
effect even if the extrinsic document was not executed in accordance with the formalities required by the
statute of wills. In order for the doctrine to apply, the extrinsic document must have been clearly
identified in the will and must have been in existence when the will was signed. Under the UPC, a writing
in existence when a will is executed may be incorporated by reference if the language of the will
manifests this intent and describes the writing sufficiently to permit its identification.

Here, it is unclear whether the sheet of paper attached to the stamp collection was in existence when the
will was signed. It seems likely that it was, as only one month passed between the time the holographic
will was signed and the time testator died, and the paper was in existence when testator died. However, it
is likely that the burden of proof on this issue will be on Friend because Friend is seeking to give effect to
a writing that was not executed with the formality of a will. Because the facts are sufficiently unclear,
Friend may be unable to meet that burden of proof.

Most damaging to Friend's case is the fact that the holographic will refers to “those friends listed on a
sheet of paper” whereas the actual sheet of paper only has Friend's name on it. The conflict between the
reference in the holographic will and the actual sheet of paper suggests that the actual sheet was placed on
the stamp collection subsequent to the making of the holographic will.

Many states have enacted statutes permitting the disposition of tangibles by the use of a list, whether or
not in existence when the will is signed. In such jurisdictions, the gift to Friend, if the holographic will is
effective, would be valid.

Explanation to Point-Four (05-15%):

If the holographic will revokes the September will, Wife likely is entitled to a forced share of Husband's
estate.

Most states create a statutory forced share for the benefit of a surviving spouse. The forced share is a
reflection of a public policy that it is to some extent inappropriate to disinherit a spouse who contributed
toward the accumulation of the deceased spouse's wealth during the marriage. The size of the share may
vary among the states, but a typical share is 1/3. The precise calculation of the elective share can vary
depending on the elective share laws of the jurisdiction. It is typical, however, that the spouse's elective
share will be charged first against the residuary estate and then against the specific bequest passing to
Friend.

Seperac-J19 Exam-Released MEE Essay Compilation © 2016-2020 1399


#331-FEB 1996–MEE Q02: QUESTION TWO (WILLS-ESTATES)

Wife and Husband were married in 1988. They had two children, Anna and Barbara, born in 1989 and
1991. In 1992, Wife validly executed a will which provided:

I leave Husband $100,000 and hope and expect that he will continue to provide for my mother Mattie, as I
have done for many years. I leave the rest of my estate to Husband. I name Husband executor of my
estate. I intentionally disinherit any person not named in this will.

In February 1995, Wife filed for divorce from Husband. In August 1995, the trial court entered a decree
that granted Wife a divorce, divided the property of Wife and Husband, and ordered Husband to pay child
support. The economic terms of the decree became effective immediately, but the portion of the decree
that actually ended the marriage between Wife and Husband was not to become final until November
1995.

Wife was killed in an automobile accident in September 1995, two months before the divorce decree was
to become final. She was survived by Husband, her children Anna and Barbara, and her mother Mattie.
For ten years before her death, Wife had sent Mattie, who was disabled, $400 per month.

Under the law of intestate succession in this jurisdiction, a decedent's surviving spouse is entitled to one-
half of the estate if the decedent is survived by children; decedent's children are entitled to one-half of the
estate if there is a surviving spouse and to the entire estate if there is no surviving spouse.

1. Is Husband barred from taking under Wife's will? Explain.

2. Assuming Husband is not barred from taking under Wife's will, to whom should Wife's estate be
distributed? Explain.

Seperac-J19 Exam-Released MEE Essay Compilation © 2016-2020 1400


#331: F96-2 MEE: ANSWER: NCBE (WILLS-ESTATES)

POINT (1) [35%] ISSUE: If a court enters a divorce decree dividing the property of spouses and
one spouse dies before the divorce becomes final, is the surviving spouse entitled to take under the
decedent's will? ANSWER: No. In a growing number of states as well as under the UPC if a court
has entered a divorce decree dividing the property of spouses and one spouse dies before the
divorce becomes final, the surviving spouse has no right to take under the decedent's will, and the
bequest to the divorced spouse passes as if the spouse predeceased the testator.

POINT (2) [40%] ISSUE: Under what circumstances will an indefinite request to a devisee to take
care of another person suffice to create a trust for the benefit of that person? ANSWER: Whether a
bequest to one person accompanied by language asking that the property be used to care for
another creates a trust depends on the language used, to whom the bequest is made, and the prior
relationship of the testator and the other parties.

POINT (3) [25%] ISSUE: Are children entitled to the rights of pretermitted heirs if they are born
before a will is executed and/or the will expresses an intent to disinherit anyone not named in the
will? ANSWER: In most jurisdictions children who are born before a will is executed are not
entitled to take a pretermitted child's share of the testator's estate. In states in which such children
are covered by the pretermitted heir statute, general language disinheriting anyone not named in
the will probably does not express an intent to disinherit that will preclude the children's claim.

ANSWER EXPLANATION:

Explanation to Point-One (30-40%):

In a growing number of states as well as under the UPC if a court has entered a divorce decree dividing
the property of spouses and one spouse dies before the divorce becomes final, the surviving spouse has no
right to take under the decedent's will, and the bequest to the divorced spouse passes as if the spouse
predeceased the testator.

At common law, divorce alone was not sufficient to revoke a will or even the provisions in a will for a
divorced spouse. In states following the common-law rule, Husband is entitled to all of the benefits
provided him by the will, regardless of whether his divorce from Wife was final. A growing number of
states, if not most, as well as Uniform Probate Code § 2-804 now provide that divorce revokes any
provision in a preexisting will for a divorced spouse. Typically, these statutes only apply if the divorce is
final. Additionally, under UPC § 2-804, if one of the spouses dies before the divorce becomes final but
the divorce decree divides the property of the spouses, provisions in the will for the surviving spouse are
also revoked. However, the UPC section on economic provisions has not been widely adopted and most
states have not passed on the issue. In states which do not have such a statute, the same result might be
reached if the court were to conclude that the economic provisions in the divorce decree, if satisfied, were
intended to be in lieu of the benefits passing under the will to the former spouse. Under UPC § 2-804,
divorce also revokes the designation of a former spouse as a fiduciary.

Although not specifically called for by the first question, if the provisions for Husband are revoked, all
testamentary benefits for Husband pass to Wife's heirs as partial intestate property. No residuary legatee
other than Husband is named in the will. Wife's heirs are her children, because Husband had no right to
his intestate share because of the divorce and property settlement. If the $100,000 bequest to Husband for

Seperac-J19 Exam-Released MEE Essay Compilation © 2016-2020 1401


the benefit of Mattie is a trust for Mattie, the children would not take this bequest. If, on the other hand,
the $100,000 bequest is a bequest to Husband with only precatory language that he use the funds for
Mattie, it also passes to Wife's heirs, assuming that Husband is not entitled to take under Wife's will.

Explanation to Point-Two (35-45%):

Whether a bequest to one person accompanied by language asking that the property be used to care for
another creates a trust depends on the language used, to whom the bequest is made, and the prior
relationship of the testator and the other parties.

A settlor must manifest the intention to create a trust to establish a valid express trust. In her will, Wife
said that she “hopes and requests” that Husband will use some of her estate to provide for her mother
Mattie. Language such as “desire,” “request,” and “hope” has given rise to considerable litigation. The
use of such language raises doubt as to whether the testator intends to create a trust (with a legal duty
upon the trustee) or merely a moral obligation unenforceable at law. Mattie's success in arguing for a trust
depends on which of two views regarding the legal effect of such language the court follows. If the court
follows the early English view, it will treat Wife's words as creating enforceable trust duties and hold that
she established a valid trust. However, most states, following the approach of later English courts, would
hold that Wife's words are ambiguous and create a trust only if her intent to impose a legal duty on
Husband is shown by extrinsic evidence.

Factors that influence a court in construing such language include: clear and specific directions to one
already a fiduciary, a natural result, and the relationship and circumstances of the parties. Mattie can rely
on several factors to support her claim that Wife intended to create a trust for her benefit, including the
fact that Wife's testamentary suggestion is made to a fiduciary, her husband whom she appointed
executor; the nature of the relationship between Wife and Mattie (daughter and mother); Wife's history of
supporting her mother; and the fact that there are two separate provisions providing benefits to Husband
rather than only one.

Explanation to Point-Three (20-30%):

In most jurisdictions children who are born before a will is executed are not entitled to take a pretermitted
child's share of the testator's estate. In states in which such children are covered by the pretermitted heir
statute, general language disinheriting anyone not named in the will probably does not express an intent to
disinherit that will preclude the children's claim.

Under a pretermitted or omitted heir statute, Anna and Barbara may have the right to take their intestate
shares from Wife's estate. The first issue is whether children born before a will is executed are covered by
the jurisdiction's statute. UPC § 2-302 does not protect children born before the will is executed. Under
such a statute Anna and Barbara have no claim as pretermitted heirs, although if Husband is barred from
taking under Wife's will, they would take all property that otherwise would have passed to him because
they would be her sole heirs under the statute set forth in the facts.

In states where children who are alive when the will is executed are entitled to a share under the
pretermitted heir statute, the next issue is whether general language of disinheritance, such as that used in
Wife's will, is sufficient to indicate that the testator intentionally omitted a child. This issue has been
much litigated, and the case law is divided. General language of disinheritance is more likely to be given
effect where the bulk of the testator's estate is bequeathed to the other parent of the pretermitted children
since in all likelihood the surviving parent will use the property for the benefit of the children or bequeath

Seperac-J19 Exam-Released MEE Essay Compilation © 2016-2020 1402


it to them when that parent dies. Otherwise such language is insufficient to bar the children from claiming
as pretermitted heirs."

Seperac-J19 Exam-Released MEE Essay Compilation © 2016-2020 1403


#332-JUL 1995–MEE Q01: QUESTION ONE (WILLS-ESTATES)

Testator was a successful businesswoman and had acquired a considerable personal for-tune. When
Testator executed her will, she was married to Husband. Husband had a child from a prior marriage,
Child. Testator had great affection for Child.

Testator's properly executed will made the following dispositions: "First, I give my diamond ring to
Friend. Second, I give $10,000 to Child. Third, I give the rest of my estate to Husband."

Ten years after Testator executed this will, she and Husband divorced and entered into a property
settlement agreement. Thereafter, Testator lost all contact with Child.

Three years after the divorce, Testator suffered a severe stroke. A conservator was appointed to manage
Testator's property. The conservator sold Testator's diamond ring for $50,000 and put the proceeds in a
savings account to pay Testator's living expenses.

Testator died three months after her stroke. She was survived by Husband (from whom she was divorced),
Child, Friend, and three grandnieces, Alice, Barbara, and Carol. The three grandnieces were the
grandchildren of Testator's brother, Barry, who had predeceased Testator. Barry had two children, Donna
and Ed, both of whom also predeceased Testator. Alice was Donna's child, and Barbara and Carol were
Ed's children. Testator's relatives are shown on the following family tree:

Parents

-----------------------------

| |

Testator Barry

-----------------------

| |

Donna Ed

| |

Alice -----------------

| |

Barbara Carol

Seperac-J19 Exam-Released MEE Essay Compilation © 2016-2020 1404


Testator had no other surviving relatives. Testator never revised her will after her divorce. At the time of
her death, her $600,000 estate included the $50,000 savings account consisting entirely of the proceeds
from the sale of her diamond ring.

How should Testator's estate be distributed? Explain.

Seperac-J19 Exam-Released MEE Essay Compilation © 2016-2020 1405


#332: J95-1 MEE: ANSWER: NCBE (WILLS-ESTATES)

POINT (1) [21%] ISSUE: Was the bequest of the diamond ring to Friend adeemed by the
conservator's sale? ANSWER: No. The sale of the diamond ring by Testator's conservator did not
adeem the specific bequest to Friend, and Friend is entitled to the full $50,000 in proceeds from the
sale.

POINT (2) [21%] ISSUE: What was the effect, if any, of the divorce on: (a) the residuary bequest to
Husband? ANSWER: Testator's divorce from Husband revoked her bequest to him by operation of
law.

POINT (2) [32%] ISSUE: What was the effect, if any, of the divorce on: (b) the bequest to Child?
ANSWER: Testator's divorce from Husband did not cause her bequest to Child to be revoked by
operation of law.

POINT (3) [26%] ISSUE: How should the intestate portion of Testator's estate be distributed?
ANSWER: Under the Uniform Probate Code, Testator's intestate estate would be distributed one-
third to each of her grandnieces, Alice, Barbara, and Carol.

ANSWER EXPLANATION:

Explanation to Point-One (15-25%):

The sale of the diamond ring by Testator's conservator did not adeem the specific bequest to Friend, and
Friend is entitled to the full $50,000 in proceeds from the sale.

If a conservator, acting on behalf of a legally incompetent person, sells an asset that is the subject matter
of a specific bequest in the ward's will, the sale does not cause an ademption by extinction of the bequest.
This is a well-recognized exception to the general rule that a specific bequest is adeemed if the subject
matter of the bequest is not part of the testator's estate at death, regardless of the testator's intent.
Testator's bequest of her diamond ring to Friend is a specific bequest because it can be satisfied only by a
specifically identified asset. The legatee is entitled to a general pecuniary bequest equal to the net sales
price, or, according to some courts, the proceeds remaining in the testator's estate. Under either view,
Friend is entitled to $50,000, since the proceeds of the sale remain in Testator's estate.

Explanation to Point-Two(a) (15-25%):

Testator's divorce from Husband revoked her bequest to him by operation of law.

A majority of states now have statutes providing that a divorce automatically revokes provisions of a will
in favor of the former spouse, including all bequests to the former spouse. The will is read as though the
legatee/former spouse predeceased the testator. Under these statutes, the residuary bequest to Husband
was revoked by Testator's divorce from Husband, and the will would be read as though Husband
predeceased Testator. In this case, the residuary estate would pass by intestacy because, although
Husband is treated as having predeceased Testator leaving a surviving child, the anti-lapse statutes
typically apply only if the predeceased legatee was a blood relative of the testator.

Seperac-J19 Exam-Released MEE Essay Compilation © 2016-2020 1406


In jurisdictions that do not have statutes expressly revoking provisions of a will in favor of a spouse upon
divorce, most courts have adopted the rule that divorce accompanied by a property settlement revokes all
dispositive provisions in favor of the former spouse. The theory is that the testator is presumed not to
want to benefit her/his former spouse who has already been provided for by the property settlement. This
presumption is irrebuttable. If there had been no property settlement, then the residuary bequest to
Husband might still be valid.

Explanation to Point-Two(b) (25-35%):

Testator's divorce from Husband did not cause her bequest to Child to be revoked by operation of law.

Although divorce revokes provisions in a will in favor of the testator's former spouse, ordinarily it does
not revoke provisions in favor of relatives of the former spouse. The effect of the divorce on testator's
relationship with legatees who were relatives of the former spouse is irrelevant. On the facts here, and
under the general rule, the $10,000 bequest to Child, Husband's child from a prior marriage, is still
effective, and Child is entitled to $10,000 from general estate assets.

Under the Uniform Probate Code, however, a divorce revokes a bequest to a relative of the divorced
spouse. If that Code applies, then the bequest to Child is revoked and it passes as part of the residuary
estate.

Explanation to Point-Three (20-30%):

Under the Uniform Probate Code, Testator's intestate estate would be distributed one-third to each of her
grandnieces, Alice, Barbara, and Carol.

Since the residuary bequest to Husband is revoked by operation of law, Testator's residuary estate passes
to her heirs. Under the UPC, Testator's heirs are her three surviving grand-nieces, who are the issue of her
parents, and they take “by representation.” The UPC defines taking by representation according to a
method under which the intestate estate is divided at the first generation that includes one or more living
members. On the facts here, that generation is the grand-nieces' generation. Each living member takes an
equal share, so each of the three grandnieces takes a one-third share.

Some intestacy statutes call for equal division among all of the heirs if they are all of the same degree of
kinship to the decedent. Statutes of this type would provide the same result as the UPC.

In some states a different method is used to determine the shares of those who take by representation.
Under this “pure per stirpes” method, the intestate estate is first divided at the generation of the decedent's
siblings even though there are no surviving members of that generation. It is divided into as many shares
as there are living members of that generation and deceased members who left surviving issue. Here, that
is only one (Barry). That share is then divided in the same way. Barry's share would be divided at the
generation of nieces and nephews into two shares (for Donna and Ed), even though there are no surviving
members of that generation. Donna's one-half share (i.e., the share she would get if she had survived
Testator) goes to her daughter Alice, and Ed's one-half (i.e., the share he would get if he had survived
Testator) is equally divided between his two children, Barbara and Carol. Barbara and Carol, therefore,
get smaller shares than Alice, even though they are in the same degree of kinship to Testator.

Seperac-J19 Exam-Released MEE Essay Compilation © 2016-2020 1407


#333-FEB 1995–MEE Q05: QUESTION FIVE (WILLS-ESTATES)

Five years ago Harry married Wilma. Wilma had a child from her prior marriage, Dorothy, who was 14.
Although Harry did not adopt Dorothy, he raised her as if she were his biological child. Harry had a son
from a prior marriage, Steve. Wilma and Steve never got along.

Three months ago Harry properly executed a will leaving 1) $10,000 to Steve, 2) $50,000 to Dorothy, and
3) the rest of his estate (valued at $700,000) to Wilma. This will, prepared under the supervision of
Harry's longtime attorney, also provided that "if any person shall contest any provision of this will, such
person shall forfeit any right to claim any share of my estate." During the 35 years prior to his death at age
62, Harry had been a successful commercial banker with a reputation within the community for being
"fair but very hard-nosed."

Harry died unexpectedly in an automobile accident while on his way to a meeting with a client to close a
$5 million deal. Five minutes before the accident, he had dropped Wilma off at their stockbroker's office
to "sign off' on changes in their jointly held stock portfolio. They had argued strenuously about these
changes, but Harry had prevailed.

In many ways, Steve was the exact opposite of Harry. During Harry and Wilma's marriage, Steve had
been unsuccessful in holding any job for longer than four months. He often asked Harry for money, which
Harry thought Steve spent too lavishly. Harry often complained that Steve "refused to settle down."

Steve, who is now 24 years old, seeks to invalidate the will and claim an intestate share of his father's
estate. Steve says that for the three years immediately preceding Harry's death Harry appeared mildly
distracted and occasionally forgetful. Steve also claims that Wilma was "constantly carping" to Harry
about Steve. He also says that Wilma 1) refused to be in the family home when Steve came to visit, 2) on
at least one occasion hid a letter Steve had written to Harry while Steve was on vacation in Europe, and 3)
many times refused to let Steve talk to Harry on the telephone, falsely claiming Harry was not home.
Steve also claims that Wilma always insisted that Harry attend every school function in which Dorothy
participated, praised Dorothy's school work to Harry, and denigrated Steve to Harry whenever possible.

(a) On what grounds might Steve seek to invalidate Harry's will, and is Steve likely to be
successful? Explain.

(b) If Steve is not successful in contesting Harry's will, what amount, if any, will Steve receive
from Harry's estate? Explain.

Seperac-J19 Exam-Released MEE Essay Compilation © 2016-2020 1408


#333: F95-5 MEE: ANSWER: NCBE (WILLS-ESTATES)

POINT (1) [27%] ISSUE: Is Harry's will invalid on the ground of mental incapacity? ANSWER:
No. Harry's will cannot be invalidated on the grounds that he lacked the mental capacity to execute
the will.

POINT (2) [52%] ISSUE: Is Harry's will invalid on the ground of undue influence? ANSWER: No.
Harry's will is not likely to be invalidated on the grounds of undue influence.

POINT (3) [21%] ISSUE: Is the no-contest clause in Harry's will valid to the effect that Steve is
barred from claiming any share of Harry's estate? ANSWER: Yes. Assuming the will is not
successfully contested, Steve will be barred from claiming the $10,000 bequest unless he can
establish that he had probable cause to contest the will.

ANSWER EXPLANATION:

Explanation to Point-One (20-25%):

Harry's will cannot be invalidated on the grounds that he lacked the mental capacity to execute the will.

A will is invalid if the testator lacked mental capacity to execute the will. In order to prevail in a will
contest on this ground, the contestant must prove that the testator did not know either (1) the nature and
extent of his property, (2) the persons who are the natural objects of his bounty, or (3) the disposition he
was making of his property. There is a presumption of mental capacity.

The only facts in the problem at all suggesting that Harry lacked mental capacity are that he “appeared
mildly distracted and occasion ally forgetful.” Counter balancing any implication from those facts that he
might not know the nature and extent of his property, the natural objects of his bounty, and the disposition
he was making of his property are the facts that (1) he was a successful businessman in the world of
finance and apparently was soup until the time of his death, given the fact that he was on his way to
closing a large deal, and (2) he had sufficient acumen to consider changes in his stock portfolio.

Steve has the burden of proof in this issue.

Explanation to Point-Two (35-50%):

Harry's will is not likely to be invalidated on the grounds of undue influence.

A will is invalid if the testator executed the will while under undue influence. In order to prevail in a will
contest on this ground, the contestant must prove that “such control was exercised over the mind of the
testator as to overcome his free agency and free will and to substitute the will of another so as to cause the
testator to do what he would not otherwise have done but for such control.” As other courts have noted, to
be successful the contestant must prove the following: (1) testator was susceptible to undue influence, (2)
the alleged influencer had the opportunity to exercise undue influence over the testator, (3) the alleged
influencer was disposed to influence the testator to obtain an improper favor, and (4) the will evidenced a
result which appears to be the effect of undue influence. (In some cases the burden of proof shifts to the
proponent if the proponent stood in a confidential relationship to the testator. That relationship, however,
is not present here.)

Seperac-J19 Exam-Released MEE Essay Compilation © 2016-2020 1409


The question then arises as to whether the facts support a claim for undue influence. Steve will argue that
Harry's distractibility and forgetfulness made him susceptible to undue influence, and that Wilma's
actions, such as the carping, belittling, and the withholding of letters, were designed to create a gulf
between Steve and Harry.

On the other hand, Wilma will claim that her comments to Harry about Steve were nothing more than the
kinds of inter-spousal criticisms one spouse makes to another about children. She further argues that even
if her conduct may have caused Harry to be negative to Steve, the evidence that Harry's will was the
product of undue influence is thin. She notes that Harry was a successful, hard- - nosed business man. She
notes that just before he died they had a disagreement over changes in his investment portfolio in which
he prevailed. She further argues that it is not at all unusual for the will to have left her the bulk of Harry's
estate, for that is a disposition reflective of what the overwhelming number of married (and remarried)
testators do. Furthermore, the bequest to Dorothy was not the product of her influence as much as it was
of the relationship that Harry and Dorothy had. Thus, Harry's will reflects the normal dispositive patterns
of a husband and stepfather close to a stepchild and not the outcome of undue influence. She also argues
that Harry's will was prepared under the supervision of Harry's longtime attorney and that if undue
influence had been present it is likely counsel would have intervened. She further argues that Harry was
dismayed by Steve's behavior, particularly his inability to hold a job for more than four months and his
refusal to settle down.

On balance, it would appear that Steve has a weak case to invalidate the will on the basis of undue
influence.

Explanation to Point-Three (10-25%):

Assuming the will is not successfully contested, Steve will be barred from claiming the $10,000 bequest
unless he can establish that he had probable cause to contest the will.

Harry's will contains what is commonly called a forfeiture (or no-contest) clause. The purpose of this
clause is to discourage potential will contestants by forcing them to choose between the bequest in the
will if no contest is filed and nothing should the contest fail. Some jurisdictions enforce no-contest
clauses, others find them invalid as a matter of public policy, and most follow the rule set forth in the
Uniform Probate Code and the Restatement of Property, Second, to the effect that the clause is
unenforceable “if probable cause exists for” instituting a will contest. Probable cause is generally defined
objectively. “A contestant's good-faith belief is not enough if there was no reasonable basis for it, but
reliance on the advice of counsel suffices if it was “sought in good faith after a full disclosure of facts.'“

Whether Steve acted with probable cause in this case is a close question, and one's conclusion on that
question is obviously affected by how strong or weak one believes Steve's case to be. There are no facts
suggesting Steve has acted on the advice of counsel, and that may adversely affect his ability to claim that
he acted with probable cause.

If probable cause exists and the majority rule is followed, Steve will receive $10,000 from Harry's estate
even though he unsuccessfully contests Harry's will. If probable cause does not exist, Steve will receive
nothing and the $10,000 will go to Wilma as part of the residuary estate.

Seperac-J19 Exam-Released MEE Essay Compilation © 2016-2020 1410

You might also like